[ { "id": "train_0", "context": "In rheumatoid arthritis, the body' s immune system misfunctions by attacking healthy cells in the joints causing the release of a hormone that in turn causes pain and swelling. This hormone is normally activated only in reaction to injury or infection. A new arthritis medication will contain a protein that inhibits the functioning of the hormone that causes pain and swelling in the joints.", "question": "The statements above, if true, most strongly support which one of the following conclusions?", "answers": "['Unlike aspirin and other medications that reduce pain and swelling and that are currently available, the new medication would repair existing cell damage that had been caused by rheumatoid arthritis.', 'A patient treated with the new medication for rheumatoid arthritis could sustain a joint injury without becoming aware of it.', 'Joint diseases other than rheumatoid arthritis would not be affected by the new medication.', \"The benefits to rheumatoid arthritis sufferers of the new medication would outweigh the medication's possible harmful side effects.\"]", "label": 1 }, { "id": "train_1", "context": "Patient: Pharmacists maintain that doctors should not be permitted to sell the medicine that they prescribe because doctors would then be tempted to prescribe unnecessary medicines in order to earn extra income. But pharmacists have a financial interest in having a monopoly on the sale of prescription medicines, so their objection to the sale of medicines by doctors cannot be taken seriously.", "question": "The patient's argument proceeds by", "answers": "['attempting to discredit a position by questioning the motives of the proponents of that position', 'rejecting a questionable position on the grounds that the general public does not support that position', \"pointing out an unstated assumption on which the pharmacists' argument relies and then refuting it\", 'asserting that pharmacists lack the appropriate knowledge to have informed opinions on the subject under discussion']", "label": 0 }, { "id": "train_2", "context": "Paula will visit the dentist tomorrow morning only if Bill goes golfing in the morning. Bill will not go golfing unless Damien agrees to go golfing too. However, Damien has decided not to go golfing. Ttherefore, Paula will not be visiting the dentist tomorrow morning.", "question": "The pattern of reasoning displayed above most closely parallels which of the following?", "answers": "['If Marge goes to the bank today, Lauren will not cash her check tomorrow. Marge will not wash her car unless it is sunny. However, it is sunny, so Marge will wash her car and go shopping with Lauren.', 'Kevin will wash his car tomorrow only if Brittany has to go visit her grandmother. Unless Aunt Susan has to run errands, Brittany will not have to go visit her grandmother. Since Aunt Susan does not have to run errands, Kevin will not wash his car tomorrow.', 'Renee will do her homework tonight if there is nothing good on television and if her neighbors do not have a party. Although, there is something good on television; her neighbors are also having a party. Ttherefore, Renee will attend the party.', 'Maddie will plan a picnic only if one of her friends, Lisa or Kenny, will come. Kenny will not come to the picnic, but Lisa will. Ttherefore, Maddie will plan a picnic.']", "label": 1 }, { "id": "train_3", "context": "Some theorists argue that literary critics should strive to be value-neutral in their literary criticism. These theorists maintain that by exposing the meaning of literary works without evaluating them, critics will enable readers to make their own judgments about the works' merits. But literary criticism cannot be completely value-neutral. Thus, some theorists are mistaken about what is an appropriate goal for literary criticism.", "question": "The argument's conclusion follows logically if which one of the following is assumed?", "answers": "[\"Any critic who is able to help readers make their own judgments about literary works' merits should strive to produce value-neutral criticism.\", 'If it is impossible to produce completely value- neutral literary criticism, then critics should not even try to be value-neutral.', \"The less readers understand the meaning of a literary work, the less capable they will be of evaluating that work's merits.\", 'Critics are more likely to provide criticisms of the works they like than to provide criticisms of the works they dislike.']", "label": 1 }, { "id": "train_4", "context": "Shipping Clerk: The five specially ordered shipments sent out last week were sent out on Thursday. Last week, all of the shipments that were sent out on Friday consisted entirely of building supplies, and the shipping department then closed for the weekend. Four shipments were sent to Truax Construction last week, only three of which consisted of building supplies.", "question": "If the shipping clerk's statements are true, which of the following must also be true?", "answers": "['At least one of the shipments sent to Truax Construction last week was sent out before Friday.', \"At least one of last week's specially ordered shipments did not consist of building supplies.\", 'At least one of the shipments sent to Truax Construction last week was specially ordered.', 'At least one of the shipments sent to Truax Construction was not sent out on Thursday of last week.']", "label": 0 }, { "id": "train_5", "context": "Firms adopting \"profit-related-pay\" (PRP) contracts pay wages at levels that vary with the firm' s profits. In the metalworking industry last year, firms with PRP contracts in place showed productivity per worker on average 13 percent higher than that of their competitors who used more traditional contracts.", "question": "If, on the basis of the evidence above, it is argued that PRP contracts increase worker productivity, which of the following, if true, would most seriously weaken that argument?", "answers": "['Many firms in the metalworking industry have modernized production equipment in the last five years, and most of these introduced PRP contracts at the same time.', 'Under PRP contracts costs other than labor costs, such as plant, machinery, and energy, make up an increased proportion of the total cost of each unit of output.', \"Because introducing PRP contracts greatly changes individual workers' relationships to the firm, negotiating the introduction of PRP contracts in complex and time consuming.\", 'In firms in the metalworking industry where PRP contracts are in place, the average take-home pay is 15 percent higher than it is in those firms where workers have more traditional contracts.']", "label": 0 }, { "id": "train_6", "context": "Astronomer: Mount Shalko is the perfect site for the proposed astronomical observatory. The summit would accommodate the complex as currently designed, with some room left for expansion. There are no large cities near the mountain, so neither smog nor artificial light interferes with atmospheric transparency. Critics claim that Mount Shalko is a unique ecological site, but the observatory need not be a threat to endemic life-forms. In fact, since it would preclude recreational use of the mountain, it should be their salvation. It is estimated that 20, 000 recreational users visit the mountain every year, posing a threat to the wildlife.", "question": "Which one of the following, if true, most weakens the astronomer's argument?", "answers": "['More than a dozen insect and plant species endemic to Mount Shalko are found nowhere else on earth.', 'The building of the observatory would not cause the small towns near Mount Shalko eventually to develop into a large city, complete with smog, bright lights, and an influx of recreation seekers.', 'A survey conducted by a team of park rangers concluded that two other mountains in the same general area have more potential for recreational use than Mount Shalko.', 'Having a complex that covers most of the summit, as well as having the necessary security fences and access road on the mountain, could involve just as much ecological disruption as does the current level of recreational use.']", "label": 3 }, { "id": "train_7", "context": "Opponents of peat harvesting in this country argue that it would alter the ecological balance of our peatrich wetlands and that, as a direct consequence of this, much of the country' s water supply would be threatened with contamination. But this cannot be true, for in Ireland, where peat has been harvested for centuries, the water supply is not contaminated. We can safely proceed with the harvesting of peat.", "question": "Which one of the following, if true, most strengthens the argument?", "answers": "[\"The peat-harvesting industry of Ireland has been able to supply most of that country's fuel for generations.\", 'The original ecology of the peat-harvesting areas of Ireland was virtually identical to that of the undisturbed wetlands of this country.', 'Over hundreds of years, the ecological balance of all areas changes slowly but significantly, sometimes to the advantage of certain flora and fauna.', 'The activities of other industries in coming years are likely to have adverse effects on the water supply of this country.']", "label": 1 }, { "id": "train_8", "context": "Shortly after the power plant opened and began discharging its wastewater into the river, there was a decrease in the overall number of bass caught by anglers downstream from the plant.", "question": "Each of the following, if true, could explain the decrease described above EXCEPT:", "answers": "['The discharged water was warmer than the normal river temperature, leading the bass to seek cooler water elsewhere.', 'Because of the new plant, access to the part of the river downstream from the plant was improved, leading to an increase in the number of anglers fishing for bass.', 'Because of the new plant, the level of noise downstream increased, making that section of the river a less popular place for anglers to fish.', 'The discharged wastewater made the river more attractive to fish that are the natural predators of bass.']", "label": 1 }, { "id": "train_9", "context": "Any sale item that is purchased can be returned for store credit but not for a refund of the purchase price. Every home appliance and every piece of gardening equipment is on sale along with selected construction tools.", "question": "If the statements above are true, which one of the following must also be true?", "answers": "['Any item that is not on sale cannot be returned for store credit.', 'Some construction tools are not returnable for store credit.', 'No piece of gardening equipment is returnable for a refund.', 'None of the things that are returnable for a refund are construction tools.']", "label": 2 }, { "id": "train_10", "context": "Detective: Laser-printer drums are easily damaged, and any nick in a drum will produce a blemish of similar dimensions on each page produced by that printer. So in matching a blemish on a page with a nick on a drum, we can reliably trace a suspicious laser-printed document to the precise printer on which it was produced.", "question": "Which one of the following, if true, most weakens the detective's argument?", "answers": "['Most suspicious documents are not produced on laser printers.', 'Criminals are unlikely to use their own laser printers to produce suspicious documents.', 'Drum nicks are usually so small that it requires skill to accurately determine their size and shape.', 'The manufacturing process often produces the same nick on several drums.']", "label": 3 }, { "id": "train_11", "context": "Biologist: A careful study of the behavior of six individual chameleons concluded that lizards such as chameleons bask in the sun not only for warmth but also to regulate their production of vitamin D. Critics of the study -- although correct in observing that its sample size was very small -- are wrong to doubt its results. After all, the study' s author is well regarded professionally and has been doing excellent work for years.", "question": "The reasoning in the biologist's argument is most vulnerable to criticism on the grounds that the argument", "answers": "[\"focuses its attention on the study's author rather than on the study itself\", 'takes the behavior of chameleons to be generalizable to lizards as a whole', \"fails to demonstrate that the study's critics have relevant expertise\", \"holds the study's author to a higher standard than it holds the study's critics\"]", "label": 0 }, { "id": "train_12", "context": "The use of space-based satellites to study environmental conditions on Earth is an important development in the conservation movement' s history. Environmental problems may now be observed long before they otherwise would be noticed, allowing for intervention before they reach the crisis stage. It is no wonder that environmentalists fail to consider both that spacecraft may damage the ozone layer and that this damage could be serious enough to warrant discontinuing spaceflight.", "question": "The reasoning above most closely conforms to which one of the following principles?", "answers": "['People tend to ignore possible objectionable consequences of actions that support their activities.', 'Attempts to employ technology often have unforeseen consequences that may be negative.', 'Technology usually has at least some negative impact on the environment, even if it is largely beneficial.', 'A negative consequence of an activity may be outweighed by its great positive consequences.']", "label": 0 }, { "id": "train_13", "context": "Department store manager: There is absolutely no reason to offer our customers free gift wrapping again this holiday season. If most customers take the offer, it will be expensive and time-consuming for us. On the other hand, if only a few customers want it, there is no advantage in offering it.", "question": "Which one of the following is an assumption required by the department store manager's argument?", "answers": "['It would be expensive to inform customers about the free gift wrapping service.', 'Gift wrapping would cost the store more during this holiday season than in previous holiday seasons.', 'Either few customers would want free gift wrapping or most customers would want it.', 'Anything that slows down shoppers during the holiday season costs the store money.']", "label": 2 }, { "id": "train_14", "context": "A study conducted over a 6-month period analyzed daily attendance and average length of visit at the local art museum. The results showed that when the museum was not featuring a special exhibition, attendance tended to be lower but patrons spent an average of 45 minutes longer in the museum than when it was featuring a special exhibition.", "question": "Each of the following, if true, could help to explain the differing average lengths of visits to the museum EXCEPT:", "answers": "['Visitors to the museum during special exhibitions tend to have narrower artistic interests, and do not view as many different exhibits during their visit.', 'A plan to extend normal museum hours during special exhibitions was considered but not enacted during the period studied.', 'Admission tickets to the special exhibitions at the museum are issued for a specific 1-hour period on a specific day.', 'Many people who go to special exhibitions are on organized tours and do not have the opportunity to browse.']", "label": 1 }, { "id": "train_15", "context": "Traveler: Southern Airways has a far worse safety record than Air Dacentaria over the past few years, in terms of both the frequency of accidents and the severity of accidents. Ttherefore, although Air Dacentaria is slightly more expensive, I will choose it over Southern Airways for my flight from Pederton to Dacenta, since it is worth paying extra to have a safer flight.", "question": "Which of the following, if true, casts the most serious doubt on the traveler's argument?", "answers": "['Although the frequency of accidents is greater on Southern Airways, on both airlines the proportion of flights that have accidents is very small.', \"Most Southern Airways flights, but not its flight from Pederton to Dacenta, use Sarmouth airport, which because of its mountainous location is one of the world's most dangerous.\", 'Only in the last few years has the cost of flying from Pederton to Dacenta been more expensive on Air Dacentaria than on Southern Airways.', \"Air Dacentaria's flight from Pederton to Dacenta is nonstop, whereas Southern Airways' flight touches down in Gorinda on the way.\"]", "label": 1 }, { "id": "train_16", "context": "In a study, pairs of trained dogs were placed side by side and given a command such as \"sit. \" After both obeyed the command, one dog was given a treat while its partner was given no reward at all. Over time, the dogs who went unrewarded began to disobey the command. This shows that dogs have an aversion to being treated unfairly.", "question": "Which one of the following would be most useful to know in order to evaluate the argument?", "answers": "['Were dogs who were accustomed to receiving regular rewards prior to the study more inclined to obey the command?', 'How many repetitions were required before the unrewarded dogs began to disobey the command?', 'Is there a decline in obedience if rewards are withheld from both dogs in the pair?', 'Were dogs who received treats in one trial ever used as dogs that did not receive treats in other trials?']", "label": 2 }, { "id": "train_17", "context": "The advanced technology of ski boots and bindings has brought a dramatic drop in the incidence of injuries that occur on the slopes of ski resorts: from 9 injuries per 1, 000 skiers in 1950 to 3 in 1980. As a result, the remainder of ski-related injuries, which includes all injuries occurring on the premises of a ski resort but not on the slopes, rose from 10 percent of all ski-related injuries in 1950 to 25 percent in 1980. The incidence of these injuries, including accidents such as falling down steps, increases with the amount of alcohol consumed per skier.", "question": "Which one of the following can be properly inferred from the passage?", "answers": "['Injuries that occurred on the slopes of ski resorts made up a smaller percentage of ski-related injuries in 1980 than in 1950.', 'As the number of ski injuries that occur on the slopes decreases, the number of injuries that occur on the premises of ski resorts increases.', 'If the technology of ski boots and bindings continues to advance, the incidence of ski-related injuries will continue to decline.', 'The technology of ski boots and bindings affects the incidence of each type of ski-related injury.']", "label": 0 }, { "id": "train_18", "context": "Environmentalist: An increased number of oil spills and the consequent damage to the environment indicate the need for stricter safety standards for the oil industry. Since the industry refuses to take action, it is the national government that must regulate industry safety standards. In particular, the government has to at least require oil companies to put double hulls on their tankers and to assume financial responsibility for accidents. Industry representative: The industry alone should be responsible for devising safety standards because of its expertise in handling oil and its understanding of the cost entailed. Implementing the double-hull proposal is not currently feasible because it creates new safety issues. Furthermore, the cost would be burdensome to the industry and consumers.", "question": "Which one of the following is an assumption on which the argument of the environmentalist depends?", "answers": "['Government safety regulations are developed in a process of negotiation with industry leaders and independent experts.', 'Environmental concerns outweigh all financial considerations when developing safety standards.', 'The requirement of two hulls on oil tankers, although initially costly, will save money over time by reducing cleanup costs.', 'The only effective sources of increased stringency in safety standards for oil tankers are action by the industry itself or national government regulation.']", "label": 3 }, { "id": "train_19", "context": "Fossilized teeth of an extinct species of herbivorous great ape have on them phytoliths, which are microscopic petrified remains of plants. Since only phytoliths from certain species of plants are found on the teeth, the apes' diet must have consisted only of those plants.", "question": "The argument assumes which one of the following?", "answers": "['Plants of every type eaten by the apes left phytoliths on their teeth.', 'Phytoliths have also been found on the fossilized teeth of apes of other extinct species.', \"None of the plant species that left phytoliths on the apes' teeth has since become extinct.\", 'Each of the teeth examined had phytoliths of the same plant species on it as all the other teeth.']", "label": 0 }, { "id": "train_20", "context": "Life on Earth depends on water. Unfortunately, water is the world' s most threatened natural resource as a result of unrelenting pollution. In the near future, wars will be fought over clean drinking water, as climate change reduces the amount of potable water and pollution taints the drinking water. However, this trend can be broken. Citizens around the globe can aggressively lobby their governments to invest in sustainable energy sources, cleanup polluted waterways, and implement regulations to prevent commercial dumping. Citizens can also have a lasting impact by recycling and taking other steps to reduce their household' s biological footprint. As long as citizens actively respond, then the crisis can be mitigated.", "question": "What is the argument's primary purpose?", "answers": "['To criticize the status quo in government.', 'To spur activism amongst citizens.', 'To inform citizens about the consequences of continued pollution.', 'To promote the benefits of recycling.']", "label": 1 }, { "id": "train_21", "context": "Tony: Few anarchists have ever performed violent actions. These few are vastly outnumbered by the violent adherents of other political ideologies. Ttherefore, the special association in the public mind between anarchism and political violence is unwarranted. Keisha: Anarchists have always been few in number, whereas other ideologies have often spawned mass movements. Ttherefore, the proportion of anarchists who are violent is possibly greater than the proportion of adherents of other ideologies who are violent.", "question": "Keisha responds to Tony's argument in which one of the following ways?", "answers": "[\"She attempts to undermine Tony's conclusion by introducing plausible evidence that is incompatible with the evidence Tony offers in support of that conclusion.\", 'She presents evidence that the two groups Tony has compared have no significant qualities in common.', \"She shows that Tony's conclusion is questionable because Tony bases it on a comparison that inappropriately involves absolute numbers rather than proportions.\", 'She questions the accuracy of the claims on which Tony bases his conclusion.']", "label": 2 }, { "id": "train_22", "context": "Journalist: Every election year at this time the state government releases the financial disclosures that potential candidates must make in order to be eligible to run for office. Among those making the required financial disclosure this year is a prominent local businessman, Arnold Bergeron. There has often been talk in the past of Mr. Bergeron' s running for governor, not least from Mr. Bergeron himself. This year it is likely he finally will , since those who have discounted the possibility of a Bergeron candidacy have always pointed to the necessity of making financial disclosure as the main obstacle to such a candidacy.", "question": "In the journalist's argument, the two boldfaced portions play which of the following roles?", "answers": "['Each provides evidence in support of an intermediate conclusion that supports a further conclusion stated in the argument.', 'The first provides information without which the argument lacks force; the second states an intermediate conclusion that is used to support a further conclusion.', 'The first provides information without which the argument lacks force; the second states the main conclusion of the argument.', 'The first states evidence bearing against the main conclusion of the argument; the second states that conclusion.']", "label": 2 }, { "id": "train_23", "context": "Leona: If the average consumption of eggs in the United States were cut in half, an estimated 5, 000 lives might be saved each year. Thomas: How can that be? That would mean that if people adopt this single change in diet for ten years, the population ten years from now will be greater by 50, 000 people than it otherwise would have been.", "question": "Which one of the following is a statement that Leona could offer Thomas to clarify her own claim and to address the point he has made?", "answers": "['It is accurate to say that 5, 000 lives have been saved as long as 5, 000 people who would have died in a given year as a result of not changing their diet, did not do so -- even if they died for some other reason.', 'If egg consumption were reduced by more than half, the estimated number of lives saved each year could be even more than 5, 000.', 'For the average consumption of eggs to be cut by half, many individual consumers would have to cut their own consumption by much more than half.', 'The actual rate of population growth depends not only on the birth rate, but also on changes in life expectancy.']", "label": 0 }, { "id": "train_24", "context": "The conventional process for tanning leather uses large amounts of calcium oxide and sodium sulfide. Tanning leather using biological catalysts costs about the same as using these conventional chemicals if the cost of waste disposal is left out of the comparison. However, nearly 20 percent less waste is produced with biological catalysts, and waste disposal is a substantial part of the overall cost of tanning. It is ttherefore less costly to tan leather if biological catalysts are used instead.", "question": "Which one of the following is an assumption required by the argument?", "answers": "['Leather tanned using the conventional process is not lower in quality than is leather tanned using biological catalysts.', 'Disposal of tanning waste produced with biological catalysts does not cost significantly more than disposal of the same amount of waste produced with the conventional process.', 'The biological catalysts that can be used in the tanning process are less costly by weight than are calcium oxide and sodium sulfide.', 'The labor costs associated with tanning leather using biological catalysts are not any greater than the labor costs associated with the conventional tanning process.']", "label": 1 }, { "id": "train_25", "context": "A summer day is \"pleasant\" if there are intermittent periods of wind and the temperature stays below 84 degree F (29 degree C) all afternoon. A summer day with high humidity levels is \"oppressive\" either if the temperature stays above 84 degree F (29 degree C) all afternoon or if there is no wind.", "question": "Which one of the following summer weather reports most closely conforms to the principles stated above?", "answers": "['On Tuesday, the temperature neither rose above nor fell below 84 degree F (29 degree C) throughout late morning and all afternoon. It was a pleasant day because there were occasional periods of wind.', 'On Thursday morning, the air was very still, and it remained windless for the whole day. Humidity levels for the day were high, and even though the temperature fell below 84 degree F (29 degree C) between early and late afternoon, it was an oppressive day.', \"On Wednesday, a refreshing breeze in the early morning became intermittent by late morning, and the day's humidity levels were constantly high. It was an oppressive day, even though the temperature did not rise above 84 degree F (29 degree C) all day.\", 'On Monday, the temperature ranged from 85 degree F to 90 degree F (30 degree C to 32 degree C) from early morning until night. It was an oppressive day even though the humidity levels were low.']", "label": 1 }, { "id": "train_26", "context": "A certain cultivated herb is one of a group of closely related plants that thrive in soil with high concentrations of metals that are toxic to most other plants. Agronomists studying the growth of this herb have discovered that it produces large amounts of histidine, an amino acid that, in test-tube solutions, renders these metals chemically inert. Hence, the herb's high histidine production must be the key feature that allows it to grow in metal-rich soils.", "question": "In evaluating the argument, it would be most important to determine which of the following?", "answers": "['Whether the herb can thrive in soil that does not have high concentrations of the toxic metals', 'Whether growing the herb in soil with high concentrations of the metals will, over time, reduce their concentrations in the soil', \"Whether the herb's high level of histidine production is associated with an unusually low level of production of some other amino acid\", 'Whether others of the closely related group of plants also produce histidine in large quantities']", "label": 3 }, { "id": "train_27", "context": "Maria won this year' s local sailboat race by beating Sue, the winner in each of the four previous years. We can conclude from this that Maria trained hard.", "question": "The conclusion follows logically if which one of the following is assumed?", "answers": "['Sue did not train as hard as Maria trained.', 'If Maria trained hard, she would win the sailboat race.', 'Maria could beat a four-time winner only if she trained hard.', 'If Sue trained hard, she would win the sailboat race.']", "label": 2 }, { "id": "train_28", "context": "Two lakes in the Pawpaw mountains, Quapaw and Highwater, were suffering from serious declines in their fish populations ten years ago. Since that time, there has been a moratorium on fishing at Quapaw Lake, and the fish population there has recovered. At Highwater Lake, no such moratorium has been imposed, and the fish population has continued to decline. Thus, the ban on fishing is probably responsible for the rebound in the fish population at Quapaw Lake.", "question": "Which one of the following, if true, most seriously weakens the argument above?", "answers": "['There used to be a greater variety of fish species in Highwater Lake than in Quapaw Lake, but there no longer is.', 'Highwater Lake is much larger than Quapaw Lake.', 'Prior to the ban, there was practically no fishing at Quapaw Lake.', 'Highwater Lake is in an area of the mountains that is highly susceptible to acid rain.']", "label": 2 }, { "id": "train_29", "context": "Buying elaborate screensavers -- programs that put moving images on a computer monitor to prevent damage -- can cost a company far more in employee time than it saves in electricity and monitor protection. Employees cannot resist spending time playing with screensavers that flash interesting graphics across their screens.", "question": "Which one of the following most closely conforms to the principle illustrated above?", "answers": "['An electronic keyboard may be cheaper to buy than a piano but more expensive to repair.', 'An energy-efficient insulation system may cost more up front but will ultimately save money over the life of the house.', 'The time that it takes to have a pizza delivered may be longer than it takes to cook a complete dinner.', 'A complicated hotel security system may cost more in customer goodwill than it saves in losses by theft.']", "label": 3 }, { "id": "train_30", "context": "The simultaneous and apparently independent development in several ancient cultures of a myth of creatures who were half human and half horse parallels the increased use of horses in these cultures. But despite the nobility and gentleness traditionally ascribed to the horse, the mythical half-horse, half-humans were frequently portrayed as violent and savage. Many human cultures use myth to express unconscious thoughts, so these mythical creatures obviously reflect people' s unconscious fear of the horse.", "question": "The reasoning in the argument is flawed because the argument", "answers": "[\"fails to show that the mythical creature mentioned represents the horse in people's minds\", 'fails to explain why people use myth for the expression of unconscious thoughts', 'confuses the expression of unconscious thoughts with the suppression of them', 'fails to demonstrate that the myth was not borrowed from one of the cultures by the others']", "label": 0 }, { "id": "train_31", "context": "The growing popularity of computer-based activities was widely predicted to result in a corresponding decline in television viewing. Recent studies have found that, in the United States, people who own computers watch, on average, significantly less television than people who do not own computers. In itself, however, this finding does very little to show that computer use tends to reduce television viewing time, since __.", "question": "Which of the following most logically completes the argument?", "answers": "['many people use their computers primarily for tasks such as correspondence that can be done more rapidly on the computer, and doing so leaves more leisure time for watching television', 'many people who watch little or no television do not own a computer', 'computer owners in the United States predominantly belong to demographic groups that have long been known to spend less time watching television than the population as a whole does', 'even though most computer owners in the United States watch significantly less television than the national average, some computer owners watch far more television than the national average']", "label": 2 }, { "id": "train_32", "context": "Adjusted for inflation, the income earned from wool sales by a certain family of Australian sheep farmers grew substantially during the period from 1840 to I860. This is because the price for wool sold on the international market was higher than the price paid on domestic markets and the percentage and amount of its wool that this family sold internationally increased dramatically during that period. But even though the family generated more income from selling their wool, they failed to enjoy a commensurate increase in prosperity.", "question": "Which one of the following would, if true, help most to resolve the apparent paradox described above?", "answers": "['The prices of wool sold to domestic markets by Australian sheep farmers decreased dramatically during the period in question.', 'At the end of the 1800s, prices in general in Australia rose more rapidly than did the wholesale price of wool sold domestically.', 'Competition in wool sales increased during the period in question, leaving Australian wool producers in a less favorable position than previously.', 'The international and domestic prices for mutton, sheepskins, and certain other products produced by all Australian sheep farmers fell sharply during the period in question.']", "label": 3 }, { "id": "train_33", "context": "Even though she thought the informant was untrustworthy, the journalist promised not to reveal his identity so long as the information he provided did not tum out to be false. However, she will publicly reveal the informant' s identity if she is ordered to do so by a judge or her editor. After all, the information concerns safety violations at the power plant. Thus, the journalist will surely reveal the informant' s identity even if the information is accurate.", "question": "The conclusion of the argument follows logically if which one of the following is assumed?", "answers": "['The informant understood, at the time the journalist promised him confidentiality, that she would break this promise if ordered to do so by a judge.', \"The journalist's editor will not order her to reveal the informant's identity unless the information is accurate and concerns public safety.\", 'The information that the informant provided is known to be false.', \"If the information concerns safety at the power plant, a judge will order the journalist to reveal her informant's identity.\"]", "label": 3 }, { "id": "train_34", "context": "A group of 1, 000 students was randomly selected from three high schools in a medium-sized city and asked the question, \"Do you plan to finish your high school education? \"More than 89 percent answered \"Yes. \" This shows that the overwhelming majority of students want to finish high school, and that if the national dropout rate among high school students is high, it cannot be due to a lack of desire on the part of the students.", "question": "The reasoning of the argument above is questionable because the argument", "answers": "['treats high school students from a particular medium-sized city as if they are representative of high school students nationwide', 'attempts to draw two conflicting conclusions from the results of one survey', 'contradicts itself by admitting that there may be a high dropout rate among students while claiming that most students want to finish high school', 'overlooks the possibility that there may in fact not be a high dropout rate among high school students']", "label": 0 }, { "id": "train_35", "context": "H. G Wells' s great dramatic novel The Time Machine is classified as science fiction simply because it takes place in the future. But this classification is inappropriate because Wells' s book possesses something that great dramatic novels have and science fiction generally lacks -- compelling characters that enable the reader to become absorbed in their plight and not just in the author' s representation of the future of technology.", "question": "The argument's conclusion follows logically if which one of the following is assumed?", "answers": "['All novels that contain compelling characters are great dramatic novels.', 'Novels can always be clearly classified into distinct genres.', 'A dramatic novel cannot both be great and belong to the genre of science fiction.', 'A work of science fiction cannot achieve greatness unless it contains compelling characters.']", "label": 2 }, { "id": "train_36", "context": "The interview is an essential part of a successful hiring program because, with it, job applicants who have personalities that are unsuited to the requirements of the job will be eliminated from consideration.", "question": "The argument above logically depends on which of the following assumptions?", "answers": "['A hiring program will be successful if it includes interviews.', \"The fit of job applicants' personalities to the requirements of the job was once the most important factor in making hiring decisions.\", \"The only purpose of an interview is to evaluate whether job applicants' personalities are suited to the requirements of the job.\", 'Interviewers can accurately identify applicants whose personalities are unsuited to the requirements of the job.']", "label": 3 }, { "id": "train_37", "context": "Dario: The government should continue to grant patents for all new drug compounds. Patents promote innovation by rewarding pharmaceutical companies for undertaking the costly research needed to develop new drugs. Cynthia: Promoting innovation is certainly important. For this reason, patents should be granted only for truly innovative drugs, not for minor variants of previously existing drugs. Since it is much less expensive to tweak an existing drug than to develop a wholly new one, pharmaceutical companies tend to focus primarily on the cheaper alternative.", "question": "Dario and Cynthia disagree over whether", "answers": "['patents should be granted for all drug compounds', 'developing truly innovative drugs is costly', 'drug patents can promote innovation', 'pharmaceutical companies have an incentive to create minor variants of existing drugs']", "label": 0 }, { "id": "train_38", "context": "Bardis: Extensive research shows that television advertisements affect the buying habits of consumers. Some people conclude from this that violent television imagery sometimes causes violent behavior. But the effectiveness of television advertisements could be a result of those televised images being specifically designed to alter buying habits, whereas television violence is not designed to cause violent behavior. Hence we can safely conclude that violent television imagery does not cause violence.", "question": "The reasoning in Bardis's argument is flawed because that argument", "answers": "['undermines its own position by questioning the persuasive power of television advertising', 'fails to distinguish a type of behavior from a type of stimulus that may or may not affect behavior', 'relies on an illegitimate inference from the fact that advertisements can change behavior to the claim that advertisements can cause violent behavior', 'concludes that a claim is false on the basis of one purported fault in an argument in favor of that claim']", "label": 3 }, { "id": "train_39", "context": "A luxury apartment condo recently opened up along the Biltmore's waterfront. Within the first two months, 80% of the standard units in the first ten of the condo's twelve stories were sold. Nonetheless, only two of the eight penthouses, all of which are located in the top two stories of the building, have sold. In order to sell the remaining six penthouses, the corporation that owns the property, should drop the rate of the penthouses by 20%.", "question": "Which of the following, if true, would argue against the proposal above?", "answers": "['Biltmore has recently suffered a recession and economists do not predict an imminent recovery.', 'After the proposed discount each penthouse will only be 15% more costly than a standard unit.', 'Typically, the very first unit to sell in a luxury condo is a standard unit.', 'There have been so many buyers ready to purchase the penthouses that apartment management has been carefully selecting those applicants with the best credit histories.']", "label": 3 }, { "id": "train_40", "context": "The first publicly available fantasy football league was launched in 1997, and within three years, every major football media website had launched their own sites. From 2000 until 2015, viewership for the National Football League rose by 27 percent, and it is currently the most popular televised sport in the United States. Fantasy football heavily contributed to the increased viewership since fantasy players had a vested interest in nearly every game.", "question": "Upon which one of the following assumptions does the author's argument rely?", "answers": "['Some fantasy players watch National Football League games.', \"Fantasy football increased the play ers' knowledge of the National Football League.\", 'Football was the least popular sport in the United States before 2000.', 'The National Football League earns a large portion of its revenue from high television ratings.']", "label": 0 }, { "id": "train_41", "context": "A newspaper article on Britain' s unions argued that their strength was declining. The article' s evidence was the decreasing number and size of strikes, as if the reason for the unions' existence was to organize strikes. Surely, in a modern industrial society, the calling of a strike is evidence that the negotiating position of the union was too weak. Strong unions do not need to call strikes. They can concentrate their efforts on working with others in the labor market to achieve common goals, such as profitable and humane working conditions.", "question": "The argument criticizing the newspaper article is directed toward establishing which one of the following as its main conclusion?", "answers": "[\"The reason for unions' existence is to work for goals such as profitable and humane working conditions by organizing strikes.\", 'Although unions represent the interests of their members, that does not preclude them from having interests in common with other participants in the labor market.', 'There is no reason to believe, on the basis of what the newspaper article said, that union strength in Britain is declining.', 'The negotiating position of a union is weak if the only means it has of achieving its end is a strike or the threat of a strike.']", "label": 2 }, { "id": "train_42", "context": "On the first day of the course, a philosophy professor told the class that no student has ever earned an A without reading all of the mandatory books. Jorge read all of the mandatory books and suggested course materials for his philosophy course. Ttherefore, Jorge will earn an A in his philosophy course.", "question": "What mistake does the argument commit in its reasoning?", "answers": "['It confuses probability and certainty.', 'It confuses relative and absolute solutions.', 'It confuses a necessary and sufficient condition.', 'It confuses correlation and causation.']", "label": 2 }, { "id": "train_43", "context": "Politician: We should impose a tariff on imported fruit to make it cost consumers more than domestic fruit. Otherwise, growers from other countries who can grow better fruit more cheaply will put domestic fruit growers out of business. This will result in farmland' s being converted to more lucrative industrial uses and the consequent vanishing of a unique way of life.", "question": "The politician's recommendation most closely conforms to which one of the following principles?", "answers": "['A country should put the interests of its own citizens ahead of those of citizens of other countries.', 'Government intervention sometimes creates more economic efficiency than free markets.', 'A country should put its own economic interest over that of other countries.', 'Social concerns should sometimes take precedence over economic efficiency.']", "label": 3 }, { "id": "train_44", "context": "Only an expert in some branch of psychology could understand why Patrick is behaving irrationally. But no expert is certain of being able to solve someone else' s problem. Patrick wants to devise a solution to his own behavioral problem.", "question": "Which one of the following conclusions can be validly drawn from the passage?", "answers": "['Patrick is not certain of being able to devise a solution to his own behavioral problem.', \"Unless Charles is an expert in some branch of psychology, Charles should not offer a solution to Patrick's behavioral problem.\", \"If Charles is certain of being able to solve Patrick's behavioral problem, then Charles does not understand why Patrick is behaving in this way.\", 'Patrick is not an expert in psychology.']", "label": 2 }, { "id": "train_45", "context": "Sponges attach to the ocean floor, continually filtering seawater for food and ejecting water they have just filtered to avoid reingesting it. Tubular and vase-shaped sponges can eject filtered water without assistance from surrounding ocean currents and thus are adapted to slow-moving, quiet waters. Because of their shape, however, these sponges cannot live in strong currents, since strong currents would dislodge them. Both of these varieties of sponge were widespread during the late Jurassic period.", "question": "The statements above, if true, most strongly support which one of the following claims?", "answers": "['No types of sponge live in large colonies, since sponges do not flourish in areas where much of the water has been filtered by other sponges.', 'Few tubular or vase-shaped sponges lived before the late Jurassic period.', 'During the late Jurassic period there were many areas of the ocean floor where currents were weak.', 'All sponges that are neither tubular nor vase-shaped inhabit areas of the ocean floor where there are extremely strong currents.']", "label": 2 }, { "id": "train_46", "context": "Government department head: We already have a code of ethics that companies doing business with the department are urged to abide by. The fact that virtually all of the companies have agreed to abide by it indicates that it is successful. Ttherefore, neither stronger ethics regulations nor enforcement mechanisms are necessary to ensure ethical behavior by companies doing business with the department.", "question": "Which of the following, if true, casts most doubt on the department head's conclusion?", "answers": "['A study of codes of ethics for companies found that the codes are most effective when the top executives of companies that agree to abide by them are fully committed to following them.', 'The code of ethics was instituted only after it was discovered that several companies had committed serious violations of ethics in their business with the department.', 'The code of ethics applies only to companies that do business with the department.', \"A government investigation found that most of the companies that agreed to abide by the department's code of ethics are not complying with it,\"]", "label": 3 }, { "id": "train_47", "context": "After examining the options, the budget committee discovered that QI' s office-phone system would be inexpensive enough to be within the cost limit that had been set for the committee. However, Corelink' s system must also be inexpensive enough to be within the limit, since it is even less expensive than QI' s system.", "question": "The reasoning in the argument above is most closely paralleled by that in which one of the following?", "answers": "[\"John's blood-alcohol level was far above the legal limit for driving, so even if it turns out that Paul's blood-alcohol level was lower than John's, it too must have been above the legal limit.\", 'Marissa is just tall enough that she can touch the ceiling when she jumps as high as she can, and since Jeff is taller than Marissa, he too must be able to touch the ceiling when he jumps.', 'By reducing the number of cigarettes she smoked per day, Kate was able to run five miles, and since Lana smokes fewer cigarettes per day than Kate now does, she too must be able to run five miles.', 'Health Dairy\\'s sharp cheddar cheese is low enough in fat to meet the labeling standard for \"low fat\" cheddar cheese, and since its mild cheddar cheese is even lower in fat, it too must meet the labeling standard.']", "label": 3 }, { "id": "train_48", "context": "Doctors in Britain have long suspected that patients who wear tinted eyeglasses are abnormally prone to depression and hypochondria. Psychological tests given there to hospital patients admitted for physical complaints like heart pain and digestive distress confirmed such a relationship. Perhaps people whose relationship to the world is psychologically painful choose such glasses to reduce visual stimulation, which is perceived as irritating. At any rate, it can be concluded that when such glasses are worn, it is because the wearer has a tendency to be depressed or hypochondriacal.", "question": "The argument assumes which one of the following?", "answers": "[\"For hypochondriacs wearing tinted glasses, the glasses serve as a visual signal to others that the wearer's health is delicate.\", 'Depression is not caused in some cases by an organic condition of the body.', \"The tinting does not dim light to the eye enough to depress the wearer's mood substantially.\", 'Wearers do not think of the tinted glasses as a means of distancing themselves from other people.']", "label": 2 }, { "id": "train_49", "context": "In 1988 the government of Country X began using a new computer program to determine the number of people employed in that country. The program simply tallied the number of paychecks per pay period issued by employers in X, and used that figure as its estimate of employment. The government reported that, compared with the immediately preceding quarter, there was a large increase in the number of people employed for the first quarter for which the program was used.", "question": "Which one of the following can be properly concluded from the information above, together with the fact that in the first quarter for which the program was used, some employers in X issued separate paychecks for the regular hours an employee worked, for the employee's overtime, and for bonuses and commissions?", "answers": "[\"The government's estimate of growth in the number of people employed was less accurate after the government began using the new program than it had been before.\", 'The government overestimated the total number of people employed in X for the first quarter for which the program was used.', 'Contrary to the claims of the government of Country X, there was no growth in the number of people employed in X in the first quarter for which the program was used.', 'The government had not reported strong growth in the number of people employed for the quarters just before the new program was used.']", "label": 1 }, { "id": "train_50", "context": "In 1992 outlaw fishing boats began illegally harvesting lobsters from the territorial waters of the country of Belukia. Soon after, the annual tonnage of lobster legally harvested in Belukian waters began declining; in 1996, despite there being no reduction in the level of legal lobster fishing activity, the local catch was 9, 000 tons below pre-1992 levels. It is ttherefore highly likely that the outlaw fishing boats harvested about 9, 000 tons of lobster illegally that year.", "question": "Which of the following is an assumption on which the argument depends?", "answers": "['The annual legal lobster harvest in Belukia in 1996 was not significantly less than 9, 000 tons.', 'Outlaw fishing boats do not, as a group, harvest more lobsters than do licensed lobster-fishing boats.', \"The illegal lobster harvesting was not so extensive that the population of catchable lobsters in Belukia's territorial waters had sharply declined by 1996.\", 'The average annual lobster catch, in tons, of an outlaw fishing boat has increased steadily since 1992.']", "label": 2 }, { "id": "train_51", "context": "In a recent study, one group of participants watched video recordings of themselves running on treadmills, and a second group watched recordings of other people running on treadmills. When contacted later, participants in the first group reported exercising, on average, 1 hour longer each day than did the other participants. This shows that watching a recording of yourself exercising can motivate you to exercise more.", "question": "Which one of the following, if true, most weakens the argument?", "answers": "['In studies of identical twins, participants who observed their twin reading overreported by a significant amount how much time they themselves spent reading in the days that followed.', \"Another study's members exhibited an increased willingness to give to charity after hearing stories in which people with whom they identified did so.\", 'Participants who were already highly motivated to exercise did not report exercising for any longer each day than they had before the study.', 'In another study, people who watched recordings of themselves lifting weights exercised for more time each day than did people who watched recordings of themselves running.']", "label": 0 }, { "id": "train_52", "context": "Archaeologist: After the last ice age, groups of paleohumans left Siberia and crossed the Bering land bridge, which no longer exists, into North America. Archaeologists have discovered in Siberia a cache of Clovis points -- the distinctive stone spear points made by paleohumans. This shows that, contrary to previous belief, the Clovis point was not invented in North America.", "question": "Which one of the following, if true, would most strengthen the archaeologist's argument?", "answers": "['Archaeologists have discovered in Siberia artifacts that date from after the time paleohumans left Siberia.', 'The Clovis points found in Siberia are older than any of those that have been found in North America.', 'Clovis points were more effective hunting weapons than earlier spear points had been.', 'Some paleohuman groups that migrated from Siberia to North America via the Bering land bridge eventually returned to Siberia.']", "label": 1 }, { "id": "train_53", "context": "A recent study involved feeding a high-salt diet to a rat colony. A few months after the experiment began, standard tests of the rats' blood pressure revealed that about 25 percent of the colony had normal, healthy blood pressure, about 70 percent of the colony had high blood pressure, and 5 percent of the colony had extremely high blood pressure. The conclusion from these results is that high-salt diets are linked to high blood pressure in rats.", "question": "The answer to which one of the following questions is most relevant to evaluating the conclusion drawn above?", "answers": "['How many rats in the colony studied had abnormally high blood pressure before the study began?', \"How much more salt than is contained in a rat's normal diet was there in the high-salt diet?\", 'What percentage of naturally occurring rat colonies feed on high-salt diets?', 'Have other species of rodents been used in experiments of the same kind?']", "label": 0 }, { "id": "train_54", "context": "Carl: Researchers who perform operations on animals for experimental purposes are legally required to complete detailed pain protocols indicating whether the animals will be at risk of pain and, if so, what steps will be taken to minimize or alleviate it. Yet when human beings undergo operations, such protocols are never required. If lawmakers were as concerned about human beings as they seem to be about animals, there would be pain protocols for human beings too. Debbie: But consider this: a person for whom a doctor wants to schedule surgery can simply be told what pain to expect and can then decide whether or not to undergo the operation. So you see, pain protocols are unnecessary for human beings.", "question": "Debbie attempts to counter Carl' s argument by", "answers": "[\"drawing an analogy that illustrates a major flaw in Carl's argument\", \"claiming that Carl's argument should be rejected because it is based on an appeal to sentimentality rather than on reasoned principles\", 'pointing out a relevant difference to undermine an analogy on which Carl bases his conclusion', \"offering a specific example to demonstrate that Carl's argument is based on a claim that can be neither confirmed nor disproved\"]", "label": 2 }, { "id": "train_55", "context": "Literary historian: William Shakespeare, a humble actor, could have written the love poetry attributed to him. But the dramas attributed to him evince such insight into the minds of powerful rulers that they could only have been written by one who had spent much time among them; Francis Bacon associated with rulers, but Shakespeare did not.", "question": "Which one of the following logically follows from the literary historian's claims?", "answers": "['Shakespeare wrote neither the love poetry nor the dramas attributed to him.', 'Bacon wrote both the love poetry and the dramas attributed to Shakespeare.', 'Shakespeare may have written the love poetry but did not write the dramas attributed to him.', 'One person could not have written both the love poetry and the dramas attributed to Shakespeare.']", "label": 2 }, { "id": "train_56", "context": "Engine noise from boats travelling through killer whales' habitats ranges in frequency from 100 hertz to 3, 000 hertz, an acoustical range that overlaps that in which the whales communicate through screams and squeals. Though killer whales do not seem to behave differently around running boat engines, engine noise from boats can be loud enough to damage their hearing over time. Ttherefore, __.", "question": "Which one of the following most logically completes the argument?", "answers": "['younger killer whales are better able to tolerate engine noise from boats than older whales are', 'killer whales are most likely to prefer areas where boat traffic is present, but light', \"noise from boat engines may impair killer whales' ability to communicate\", 'killer whales are less likely to attempt to communicate with one another when boat engines are operating nearby']", "label": 2 }, { "id": "train_57", "context": "Almost all of the books published in the past 150 years were printed on acidic paper. Unfortunately, every kind of acidic paper gradually destroys itself due to its very acidity. This process of deterioration can be slowed if the books are stored in a cool, dry environment. Techniques, which are now being developed, to deacidify books will probably be applied only to books with historical significance.", "question": "If all of the statements in the passage above are true, which one of the following must also be true?", "answers": "['If a book was published in the past 150 years and is historically insignificant, it will probably deteriorate completely.', 'Almost all of the books published in the past 150 years will gradually destroy themselves.', 'Books published on acidic paper in 1900 should now all be at about the same state of deterioration.', 'If a book is of historical significance and was printed before 150 years ago, it will be deacidified.']", "label": 0 }, { "id": "train_58", "context": "When politicians describe their opponents' positions, they typically make those positions seem implausible and unattractive. In contrast, scholars try to make opposing positions seem as plausible and attractive as possible. Doing so makes their arguments against those positions more persuasive to their professional colleagues. Politicians should take note: they could persuade more voters with their arguments if they simply followed the scholars in charitably formulating their opponents' positions.", "question": "The reasoning in the argument is most vulnerable to criticism on the grounds that it", "answers": "['fails to address the possibility that an approach that works with one kind of audience will not work with another', 'takes for granted that both scholars and politicians have persuasion as their aim', 'fails to account for the difficulty of coming up with charitable formulations of positions to which one is opposed', 'focuses on the differences between two styles of argumentation even though those styles might be suited to similar audiences']", "label": 0 }, { "id": "train_59", "context": "Crops can be traded on the futures market before they are harvested. If a poor corn harvest is predicted, prices of corn futures rise; if a bountiful corn harvest is predicted, prices of corn futures fall. This morning meteorologists are predicting much-needed rain for the corn-growing region starting tomorrow. Ttherefore, since adequate moisture is essential for the current crop' s survival, prices of corn futures will fall sharply today.", "question": "Which of the following, if true, most weakens the argument above?", "answers": "['Most people who trade in corn futures rarely take physical possession of the corn they trade.', 'Futures prices for corn have been fluctuating more dramatically this season than last season.', 'Agriculture experts announced today that a disease that has devastated some of the corn crop will spread widely before the end of the growing season.', 'Corn that does not receive adequate moisture during its critical pollination stage will not produce a bountiful harvest.']", "label": 2 }, { "id": "train_60", "context": "Numismatist: In medieval Spain, most gold coins were minted from gold mined in West Africa, in the area that is now Senegal. The gold mined in this region was the purest known. Its gold content of 92 percent allowed coins to be minted without refining the gold, and indeed coins minted from this source of gold can be recognized because they have that gold content. The mints could refine gold and produced other kinds of coins that had much purer gold content, but the Senegalese gold was never refined. As a preliminary to negotiating prices, merchants selling goods often specified that payment should be in the coins minted from Senegalese gold.", "question": "Which one of the following, if true, most helps to explain this preference?", "answers": "['During this period most day-to-day trading was conducted using silver coins, though gold coins were used for costly transactions and long-distance commerce.', \"Since gold coins' monetary value rested on the gold they contained, payments were frequently made using coins minted in several different countries.\", 'The mints were able to determine the purity, and hence the value, of gold coins by measuring their density.', 'Because refined gold varied considerably in purity, specifying a price as a number of refined-gold coins did not fix the quantity of gold received in payment.']", "label": 3 }, { "id": "train_61", "context": "Advertisement: Attention homeowners! All the pests in your yard will die within minutes of being treated by JTE pesticide. What' s better, JTE pesticide is not harmful to the birds or small animals that make your yard their home. By using JTE, you can eliminate the pests in your yard and keep the rabbits and birds.", "question": "Which one of the following is an assumption on which the argument depends?", "answers": "['JTE pesticide will not harm the underground water supply.', 'JTE has met all state and federal environmental standards.', 'The rabbits, birds, or other small animals are not in your yard because of the pests.', 'The effectiveness of JTE pesticide does not depend on the strict adherence to specific application procedures.']", "label": 2 }, { "id": "train_62", "context": "It is pointless to debate the truth of the law of noncontradiction, a fundamental logical principle according to which two statements that contradict each other cannot both be true. For a debate to be productive, participants must hold some basic principles in common. But the principles held in common in a debate over the law of noncontradiction would be much less certain than that law, so it matters little whether the law of noncontradiction can be defended on the basis of those principles.", "question": "Which one of the following most accurately expresses the overall conclusion drawn in the argument?", "answers": "['The law of noncontradiction is a principle that the participants in a productive debate must hold in common.', 'Statements that contradict each other cannot both be true.', 'Any principles that could be used to defend the law of noncontradiction are less certain than it is.', 'It is pointless to debate the truth of the law of noncontradiction.']", "label": 3 }, { "id": "train_63", "context": "Film historians have made two major criticisms of Depression-era filmmakers: first, that they were too uncritical of the economic status quo; and second, that they self-indulgently created films reflecting their own dreams and desires. However, these filmmakers made their movies with an eye to profit, and so they provided what their audiences most wanted in a film: a chance to imagine being wealthy enough not to have a care in the world. Thus, the second criticism cannot be accurate.", "question": "The conclusion of the argument follows logically if which one of the following is assumed?", "answers": "['To avoid self-indulgence, filmmakers should take a critical stance toward the existing economic system and should allow audiences to form their own personal aspirations.', 'It cannot be self-indulgent for a filmmaker to give an audience what it most wants.', 'The people who regularly went to movies during the Depression were those likely to have been most satisfied with the economic status quo.', 'Depression-era filmmakers who did not make films for profit could not take radical critical stances toward then-current economic and political issues.']", "label": 1 }, { "id": "train_64", "context": "The Interstate Bridge over the Apache River, built in the 1950s, shows a substantial amount of rust: as much as 45% of its surface is coated in rust. Community activists have argued that the bridge presents a hazard: it is likely to collapse in places where it has rusted through. Professors of mechanical engineering at the local university did an extensive analysis of the bridge. These professors and their graduate students determined that 98% of the rust on the bridge exists on the surface only, and actually seals the underlying steel from the corrosive influence of the elements. The results of this academic study suggest that the bridge is safe for continued use. ", "question": "In the argument given, the two portions in boldface play which of the following roles?", "answers": "['The first is a position that the argument opposes; the second is the conclusion of the argument.', 'The first is evidence taken to support one conclusion; the second provides evidence that calls this conclusion into question.', 'The first is the main conclusion of the argument; the second provides evidence that calls this conclusion into question.', 'The first is an interpretation of evidence; the second calls that evidence into question.']", "label": 0 }, { "id": "train_65", "context": "Kim: Some people claim that the battery-powered electric car represents a potential solution to the problem of air pollution. But they forget that it takes electricity to recharge batteries and that most of our electricity is generated by burning polluting fossil fuels. Increasing the number of electric cars on the road would require building more generating facilities since current facilities are operating at maximum capacity. So even if all of the gasoline-powered cars on the roads today were replaced by electric cars, it would at best be an exchange of one source of fossil-fuel pollution for another.", "question": "The main point made in Kim's argument is that", "answers": "['gasoline-powered cars will probably remain a common means of transportation for the foreseeable future', 'a significant reduction in air pollution cannot be achieved unless people drive less', 'replacing gasoline-powered cars with battery- powered electric cars will require building more generating facilities', 'battery-powered electric cars are not a viable solution to the air-pollution problem']", "label": 3 }, { "id": "train_66", "context": "The television show Henry was not widely watched until it was scheduled for Tuesday evenings immediately after That' s Life, the most popular show on television. During the year after the move, Henry was consistently one of the ten most-watched shows on television. Since Henry' s recent move to Wednesday evenings, however, it has been watched by far fewer people. We must conclude that Henry was widely watched before the move to Wednesday evenings because it followed That' s Life and not because people especially liked it.", "question": "Which one of the following, if true, most strengthens the argument?", "answers": "[\"The show that now follows That's Life on Tuesdays has double the number of viewers it had before being moved.\", \"Henry has been on the air for three years, but That's Life has been on the air for only two years.\", 'After its recent move to Wednesday, Henry was aired at the same time as the second most popular show on television.', \"That's Life was not widely watched during the first year it was aired.\"]", "label": 0 }, { "id": "train_67", "context": "The Bluetail Moonbeam, endemic to the streams and rivers of Asia, is regularly imported into the United States by companies wishing to breed it for food. An infectious virus has been threatening Bluetail Moonbeam stocks in the United States. Even before a Bluetail Moonbeam develops any symptoms, biologists can establish the presence of the infection by an inexpensive test for antibodies in the fish' s blood. Yet, after the instance of infection, antibodies take about four weeks to appear in the fish' s blood, and the test will find no results before then. Catching a live Bluetail Moonbeam and shipping it to the United States takes a little more than a week. Once the Bluetail Moonbeam is in captivity, shippers can eliminate all new sources of infection, but unfortunately, some Bluetail Moonbeams might be infected days or even hours before being caught. Thus, even running the antibody test on each Bluetail Moonbeam entering the country would still allow infected fish into the American stocks.", "question": "Which of the following, if performed by the government, could logically be expected to overcome the problem with the plan to limit the spread of this virus in the Bluetail Moonbeam?", "answers": "['Funding research into either an inoculation or a cure for this viral infection.', 'Tracking the imported Bluetail Moonbeam into industry held stock, and for each imported fish, four weeks after capture, removing it from stock to be tested; only uninfected fish would be returned to stock.', 'Quarantining each imported Bluetail Moonbeam for at least three weeks, then performing the antibody test on each, and allowing only the uninfected to move into industrial stocks.', 'Launching an educational campaign to make those industries that breed the Bluetail Moonbeam aware of the limitations of the antibody test for the virus.']", "label": 2 }, { "id": "train_68", "context": "Several companies will soon offer personalized electronic news services, delivered via cable or telephone lines and displayed on a television. People using these services can view continually updated stories on those topics for which they subscribe. Since these services will provide people with the information they are looking for more quickly and efficiently than printed newspapers can, newspaper sales will decline drastically if these services become widely available.", "question": "Which one of the following, if true, most seriously weakens the argument?", "answers": "['Approximately 30 percent of people have never relied on newspapers for information but instead have always relied on news programs broadcast on television and radio.', 'In reading newspapers, most people not only look for stories on specific topics but also like to idly browse through headlines or pictures for amusing stories on unfamiliar or unusual topics.', 'Companies offering personalized electronic news services will differ greatly in what they charge for access to their services, depending on how wide a range of topics they cover.', \"The average monthly cost of subscribing to several channels on a personalized electronic news service will approximately equal the cost of a month's subscription to a newspaper.\"]", "label": 1 }, { "id": "train_69", "context": "Some environmentalists question the prudence of exploiting features of the environment, arguing that there are no economic benefits to be gained from forests, mountains, or wetlands that no longer exist. Many environmentalists claim that because nature has intrinsic value it would be wrong to destroy such features of the environment, even if the economic costs of doing so were outweighed by the economic costs of not doing so.", "question": "Which one of the following can be logically inferred from the passage?", "answers": "['Many environmentalists provide only a noneconomic justification in questioning the defensibility of exploiting features of the environment.', 'Even if there is no economic reason for protecting the environment, there is a sound noneconomic justification for doing so.', 'Most environmentalists appeal to economic reasons in questioning the defensibility of exploiting features of the environment.', 'Some environmentalists appeal to a noneconomic justification in questioning the defensibility of exploiting features of the environment.']", "label": 3 }, { "id": "train_70", "context": "Unprecedented industrial growth in the country of Remo has created serious environmental problems because factories there lack adequate pollution-control systems. Remo is developing a clean growth plan that includes environmental regulations that will require the installation of such systems. Since no companies in Remo currently produce pollution-control systems, the plan, if implemented, will create significant opportunities for foreign exporters to market pollution-control systems.", "question": "Which of the following, if true, most seriously weakens the argument?", "answers": "['Foreign exporters would provide factory-trained technicians to maintain the pollution-control systems sold to Remo.', 'The clean growth plan will provide tax incentives for local businesses to develop and manufacture pollution-control devices.', 'The regulations that Remo plans to implement are much less strict than those in neighboring nations.', 'Industrial lobbyists sponsored by local businesses in Remo are trying to prevent the implementation of the government regulations.']", "label": 1 }, { "id": "train_71", "context": "Rhizobium bacteria living in the roots of bean plants or other legumes produce fixed nitrogen, which is one of the essential plant nutrients and which for nonlegume crops, such as wheat, normally must be supplied by applications of nitrogen-based fertilizer. So if biotechnology succeeds in producing wheat strains whose roots will play host to Rhizobium bacteria, the need for artificial fertilizers will be reduced.", "question": "The argument above makes which one of the following assumptions?", "answers": "['Fixed nitrogen is currently the only soil nutrient that must be supplied by artificial fertilizer for growing wheat crops.', 'Rhizobium bacteria living in the roots of wheat would produce fixed nitrogen.', 'There are no naturally occurring strains of wheat or other grasses that have Rhizobium bacteria living in their roots.', 'Biotechnology should be directed toward producing plants that do not require artificial fertilizer.']", "label": 1 }, { "id": "train_72", "context": "Wirth: All efforts to identify a gene responsible for predisposing people to manic-depression have failed. In fact, nearly all researchers now agree that there is no \"manic-depression gene. \" Ttherefore, if these researchers are right, any claim that some people are genetically predisposed to manic-depression is simply false. Chang: I do not dispute your evidence, but I take issue with your conclusion. Many of the researchers you refer to have found evidence that a set of several genes is involved and that complex interactions among these genes produce a predisposition to manic-depression.", "question": "The point at issue between Wirth and Chang is whether", "answers": "['nearly all researchers now agree that there is no manic-depression gene', 'it is likely that researchers will ever be able to find a single gene that predisposes people to manic-depression', 'efforts to identify a gene or set of several genes responsible for predisposing people to manic-depression have all failed', 'current research supports the claim that no one is genetically predisposed to manic-depression']", "label": 3 }, { "id": "train_73", "context": "Some visitors to the park engage in practices that seriously harm the animals. Surely, no one who knew that these practices seriously harm the animals would engage in them. So it must be concluded that some of the visitors do not know that these practices seriously harm the animals.", "question": "The pattern of reasoning exhibited by which one of the following arguments is most similar to that exhibited by the argument above?", "answers": "[\"Some of the people who signed the petition were among the mayor's supporters. Yet the mayor denounced everyone who signed the petition. Hence the mayor denounced some of her own supporters.\", 'Some of the people who worked on the failed project will be fired. Everyone in this department played an important part in that project. Ttherefore some people in this department will be fired.', 'All of the five original planners are responsible for this problem. Yet none of the original planners will admit responsibility for the problem. Thus some of the people responsible for the problem will not admit responsibility.', 'Some of the people polled live outside the city limits. However, no one who can vote in city elections lives outside the city. Ttherefore some of the people polled cannot vote in the upcoming city election.']", "label": 3 }, { "id": "train_74", "context": "Beck: Our computer program estimates municipal automotive use based on weekly data. Some staff question the accuracy of the program' s estimates. But because the figures it provides are remarkably consistent from week to week, we can be confident of its accuracy.", "question": "The reasoning in Beck's argument is flawed in that it", "answers": "['fails to establish that consistency is a more important consideration than accuracy', \"regards accuracy as the sole criterion for judging the program's value\", 'fails to consider that the program could produce consistent but inaccurate output', \"takes for granted that the program's output would be consistent even if its estimates were inaccurate\"]", "label": 2 }, { "id": "train_75", "context": "Researcher: Every year approximately the same number of people die of iatrogenic \"disease\" -- that is, as a direct result of medical treatments or hospitalization -- as die of all other causes combined. Ttherefore, if medicine could find ways of preventing all iatrogenic disease, the number of deaths per year would decrease by half.", "question": "The reasoning in the researcher's argument is flawed because the argument fails to consider that", "answers": "['people who do not die of one cause may soon die of another cause', 'prevention of noniatrogenic disease will have an effect on the occurrence of iatrogenic disease', 'some medical treatments can be replaced by less invasive or damaging alternatives', 'there is no one way to prevent all cases of death from iatrogenic disease']", "label": 0 }, { "id": "train_76", "context": "Before the printing press, books could be purchased only in expensive manuscript copies. The printing press produced books that were significantly less expensive than the manuscript editions. The public' s demand for printed books in the first years after the invention of the printing press was many times greater than demand had been for manuscript copies. This increase demonstrates that there was a dramatic jump in the number of people who learned how to read in the years after publishers first started producing books on the printing press.", "question": "Which one of the following statements, if true, casts doubt on the argument?", "answers": "['Books that were printed on the printing press in the first years after its invention often circulated among friends in informal reading clubs or libraries.', 'During the first years after the invention of the printing press, letter writing by people who wrote without the assistance of scribes or clerks exhibited a dramatic increase.', 'The first printed books published after the invention of the printing press would have been useless to illiterate people, since the books had virtually no illustrations.', 'In the first years after the printing press was invented, printed books were purchased primarily by people who had always bought and read expensive manuscripts but could afford a greater number of printed books for the same money.']", "label": 3 }, { "id": "train_77", "context": "A recent study monitored the blood pressure of people petting domestic animals in the laboratory. The blood pressure of some of these people lowered while petting the animals. Ttherefore, for any one of the people so affected, owning a pet would result in that person having a lower average blood pressure.", "question": "The flawed pattern of reasoning in the argument above is most similar to that in which one of the following?", "answers": "['Since riding in a boat for a few minutes is relaxing for some people, those people would be more relaxed generally if those people owned boats.', 'Since pruning houseplants is enjoyable for some people, those people should get rid of houseplants that do not require frequent pruning.', 'Because buying an automobile is very expensive, people should hold on to an automobile, once bought, for as long as it can be maintained in running condition.', 'Since giving a fence one coat of white paint makes the fence white, giving it two coats of white paint would make it even whiter.']", "label": 0 }, { "id": "train_78", "context": "Among the many temptations of the digital age, manipulation of photographs has proved particularly troublesome for science. Recently, a journal of cellular biology began using a software tool to examine the digital images submitted along with articles for publication. It discovered that dozens of authors had submitted digital images that had been manipulated in ways that violated the journal' s guidelines. Clearly, scientific fraud is a widespread problem among the authors submitting to that journal.", "question": "Which one of the following is an assumption required by the argument?", "answers": "['Scientific fraud is possible in the field of cellular biology only if the research is documented with digital images.', 'Scientific fraud is a widespread problem only among scientists who submit articles to journals of cellular biology.', 'The scientists who submitted manipulated images were aware that the journal used software to examine digital images for evidence of manipulation.', 'Many of the scientists who submitted articles with manipulated images did so in order to misrepresent the information conveyed by those images.']", "label": 3 }, { "id": "train_79", "context": "Teachers should not do anything to cause their students to lose respect for them. And students can sense when someone is trying to hide his or her ignorance. Ttherefore, a teacher who does not know the answer to a question a student has asked should not pretend to know the answer.", "question": "The conclusion is properly drawn if which one of the following is assumed?", "answers": "['Students respect honesty above all else.', 'Students lose respect for teachers whenever they sense that the teachers are trying to hide their ignorance.', \"Students' respect for a teacher is independent of the amount of knowledge they attribute to that teacher.\", 'A teacher cannot be effective unless he or she retains the respect of students.']", "label": 1 }, { "id": "train_80", "context": "Insurance Company X is considering issuing a new policy to cover services required by elderly people who suffer from diseases that afflict the elderly. Premiums for the policy must be low enough to attract customers. Ttherefore, Company X is concerned that the income from the policies would not be sufficient to pay for the claims that would be made.", "question": "Which of the following strategies would be most likely to minimize Company X's losses on the policies?", "answers": "['Insuring only those individuals who did not suffer any serious diseases as children', 'Insuring only those individuals who are wealthy enough to pay for the medical services', 'Insuring only those individuals who were rejected by other companies for similar policies', 'Attracting middle-aged customers unlikely to submit claims for benefits for many years']", "label": 3 }, { "id": "train_81", "context": "Restaurant critic: Most people agree that the food at Marva' s Diner is exceptional, while the food at the more popular Traintrack Inn is fairly ordinary. This discrepancy should come as no surprise, since the Traintrack Inn' s more convenient location is by itself almost enough to guarantee a steady flow of customers .", "question": "Which one of the following is a principle that, if valid, most helps to justify the restaurant critic's reasoning?", "answers": "['The best way for a business to attract a steady flow of customers is to improve its products .', 'The quality of the food at a restaurant is the most important factor in its popularity.', \"There is no relationship between the quality of a restaurant's food and the popularity of that restaurant.\", 'A business will improve its products only when it is necessary to do so in order to attract customers .']", "label": 3 }, { "id": "train_82", "context": "Hospitals, universities, labor unions, and other institutions may well have public purposes and be quite successful at achieving them even though each of their individual staff members does what he or she does only for selfish reasons.", "question": "Which one of the following generalizations is most clearly illustrated by the passage?", "answers": "['What is true of some social organizations is not necessarily true of all such organizations.', 'Often an instrument created for one purpose will be found to serve another purpose just as effectively.', 'Many social institutions have social consequences unintended by those who founded them.', 'An organization can have a property that not all of its members possess.']", "label": 3 }, { "id": "train_83", "context": "MetroBank made loans to ten small companies, in amounts ranging from $1, 000 to $100, 000. These ten loans all had graduated payment plans, i. e. , the scheduled monthly loan payment increased slightly each month over the five-year term of the loan. Nonetheless, the average payment received by MetroBank for these ten loans had decreased by the end of the five-year term.", "question": "Which one of the following, if true, most helps to resolve the apparent discrepancy in the statements above?", "answers": "['Of the ten small companies, the three that had borrowed the largest amounts paid off their loans within three years.', 'For some loans made by MetroBank, the monthly payment decreases slightly over the term of the loan.', 'The number of small companies receiving new loans from MetroBank increased over the five-year term.', 'Most banks offer a greater number of loans for under $100, 000 than for over $100, 000.']", "label": 0 }, { "id": "train_84", "context": "A recent survey showed that 50 percent of people polled believe that elected officials should resign if indicted for a crime, whereas 35 percent believe that elected officials should resign only if they are convicted of a crime. Ttherefore, more people believe that elected officials should resign if indicted than believe that they should resign if convicted.", "question": "The reasoning above is flawed because it", "answers": "['draws a conclusion about a specific belief based on responses to queries about two different specific beliefs', 'contains premises that cannot all be true', 'confuses a sufficient condition with a required condition', 'is based on an ambiguity of one of its terms']", "label": 2 }, { "id": "train_85", "context": "On May first, in order to reduce the number of overdue books, a children's library instituted a policy of forgiving fines and giving bookmarks to children returning all of their overdue books. On July first there were twice as many overdue books as there had been on May first, although a record number of books had been returned during the interim.", "question": "Which of the following, if true, most helps to explain the apparent inconsistency in the results of the library's policy?", "answers": "['The bookmarks became popular among the children, so in order to collect the bookmarks, many children borrowed many more books than they usually did and kept them past their due date.', 'The children were allowed to borrow a maximum of five books for a two-week period, and hence each child could keep a maximum of fifteen books beyond their due date within a two-month period.', 'Although the grace period enticed some children to return all of their overdue books, it did not convince all of the children with overdue books to return all of their books.', 'Although the library forgave overdue fines during the grace period, the amount previously charged the children was minimal; hence, the forgiveness of the fines did not provide enough incentive for them to return their overdue books.']", "label": 0 }, { "id": "train_86", "context": "When storing Renaissance oil paintings, museums conform to standards that call for careful control of the surrounding temperature and humidity, with variations confined within narrow margins. Maintaining this environment is very costly, and recent research shows that even old oil paint is unaffected by wide fluctuations in temperature and humidity. Ttherefore, museums could relax their standards and save money without endangering their Renaissance oil paintings.", "question": "Which of the following is an assumption on which the argument depends?", "answers": "['Renaissance paintings were created in conditions involving far greater fluctuations in temperature and humidity than those permitted by current standards.', 'None of the materials in Renaissance oil paintings other than the paint are vulnerable enough to relatively wide fluctuations in temperature and humidity to cause damage to the paintings.', 'Most Renaissance oil paintings are stored in museums located in regions near the regions where the paintings were created.', 'Under the current standards that museums use when storing Renaissance oil paintings, those paintings do not deteriorate at all.']", "label": 1 }, { "id": "train_87", "context": "As advances in medical research and technology have improved the ability of the medical profession to diagnose and treat a wide variety of illnesses and injuries, life spans have increased and overall health has improved. Yet, over the past few decades there has been a steady and significant increase in the rate of serious infections.", "question": "Which one of the following, if true, most helps to resolve the apparent discrepancy in the information above?", "answers": "[\"Modern treatments for many otherwise fatal illnesses increase the patient's susceptibility to infection.\", 'It remains true that doctors sometimes prescribe ineffective medications due to misdiagnosis.', 'Life spans have increased precisely because overall health has improved.', 'As a population increases in size, there is a directly proportional increase in the number of serious infections.']", "label": 0 }, { "id": "train_88", "context": "In Yasukawa' s month-long study of blackbirds, the percentage of smaller birds that survived the duration of the study exceeded the percentage of larger birds that survived. However, Yasukawa' s conclusion that size is a determinant of a blackbird' s chances of survival over a month-long period is probably mistaken, since smaller blackbirds are generally younger than larger ones.", "question": "The statements above, if true, support which one of the following inferences?", "answers": "['Among the blackbirds that survived the month-long study, there was no relation between size and age.', 'Among blackbirds of the same age, a difference in size may not indicate a difference in chances of survival over a month-long period.', 'With a larger sample of blackbirds, the percentage of smaller birds that survive a one-month period would be the same as the percentage of larger birds that survive.', 'Larger blackbirds of a given age are actually more likely to survive over a one-month period than are smaller blackbirds of the same age.']", "label": 1 }, { "id": "train_89", "context": "Editorial: Contrary to popular belief, teaching preschoolers is not especially difficult, for they develop strict systems (e. g. , for sorting toys by shape), which help them to learn, and they are always intensely curious about something new in their world.", "question": "Which one of the following, if true, most seriously weakens the editorial's argument?", "answers": "['Children intensely curious about new things have very short attention spans.', 'Preschool teachers generally report lower levels of stress than do other teachers.', 'Preschoolers have a tendency to imitate adults, and most adults follow strict routines.', 'Some older children also develop strict systems that help them learn.']", "label": 0 }, { "id": "train_90", "context": "Miguel has four family members who plan to come to his graduation on Sunday afternoon, but it is likely that only three of them will be allowed to attend. Normally graduation is held in the football stadium, where there is no limit on the number of family members who can attend. However, the ceremony is relocated to the gymnasium if it rains, and each graduate receives just three admission tickets for use by family members.", "question": "The conclusion of the argument is most strongly supported if which one of the following is assumed?", "answers": "['Miguel has several friends who have fewer than three family members coming to graduation.', \"Miguel's graduating class is much larger than usual.\", 'Miguel has a fifth family member who is unable to come to his graduation.', 'The weather service has indicated that there is a very high likelihood of rain on Sunday afternoon.']", "label": 3 }, { "id": "train_91", "context": "We can learn about the living conditions of a vanished culture by examining its language. Thus, it is likely that the people who spoke Proto-Indo-European, the language from which all IndoEuropean languages descended, lived in a cold climate, isolated from ocean or sea, because ProtoIndo-European lacks a word for \"sea, \"yet contains words for \"winter, \"\"snow, \" and \"wolf. ", "question": "Which one of the following, if true, most seriously weakens the argument?", "answers": "['Some languages lack words for prominent elements of the environments of their speakers.', 'A word meaning \"fish\"was used by the people who spoke Proto-Indo-European.', 'The people who spoke Proto-Indo-European were nomadic.', 'There are no known languages today that lack a word for \"sea. \"']", "label": 0 }, { "id": "train_92", "context": "Guitar strings often go \"dead\"-become less responsive and bright in tone-after a few weeks of intense use. A researcher whose son is a classical guitarist hypothesized that dirt and oil, rather than changes in the material properties of the string, were responsible.", "question": "Which of the following investigations is most likely to yield significant information that would help evaluate the researcher's hypothesis?", "answers": "['Determining whether a metal alloy is used to make the strings used by classical guitarists', 'Determining whether classical guitarists make their strings go dead faster than do folk guitarists', 'Determining whether smearing various substances on new guitar strings causes them to go dead', 'Determining whether a dead string and a new string produce different qualities of sound']", "label": 2 }, { "id": "train_93", "context": "In some countries, national planners have attempted to address the problems resulting from increasing urbanization by reducing migration from rural areas. But some economists have suggested an alternative approach. These economists assert that planners could solve these problems effectively by trading goods or services produced by a predominantly urban population in order to obtain the agricultural products that were previously produced domestically.", "question": "Which one of the following, if true, would provide the most support for the economists' assertion?", "answers": "['Government subsidies to urban manufacturers can ease the problems caused by the migration of people from rural to urban areas.', 'Problems associated with migration to cities from rural areas are primarily due to trade imbalances between countries.', 'A scarcity of agricultural products is a central element of many problems created by urbanization.', 'All problems that have economic causes must have economic solutions.']", "label": 2 }, { "id": "train_94", "context": "Nutrition education in schools once promoted daily consumption of food from each of the \"four food groups\": milk, meat, fruit and vegetables, and breads and cereals. This recommendation was, however, dangerous to health.", "question": "Each of the following, if true, provides support for the critique above EXCEPT:", "answers": "['A healthy diet should include the consumption of several different fruits and vegetables daily, but the recommendation was often interpreted as satisfied by the consumption of a single serving of a fruit or vegetable.', 'The recommendation that some food from the fruit and vegetable group be consumed daily constituted a reminder not to neglect this group, which provides needed vitamins, minerals, and fiber.', 'The omission of fish, which contains beneficial oils, from the names of groups in the list gave erroneous impression that it is less healthy as a food than is red meat.', 'The division into four groups gave the impression that an equal amount of each should be consumed, but milk and meat tend to contain fats that promote heart disease and cancer and should be eaten in lesser amounts.']", "label": 1 }, { "id": "train_95", "context": "Ethicist: Utilitarianism argues that it is good and right to act in the best interests of all entities involved, and says the best action is that which serves the maximum number of those entities. For example, making vaccination of children a requirement is considered utilitarianism because __.", "question": "Which of the following most logically completes the argument?", "answers": "['it is seen as an act of serving the majority of public good.', 'it does more harm than good for the receivers of the vaccinations.', 'is considered utilitarianism because there are more children than adults in the world.', 'it is seen as giving the parents complete control over the decisions they make for their children.']", "label": 0 }, { "id": "train_96", "context": "One should apologize only to a person one has wronged, and only for having wronged that person. To apologize sincerely is to acknowledge that one has acted wrongfully. One cannot apologize sincerely unless one intends not to repeat that wrongful act. To accept an apology sincerely is to acknowledge a wrong, but also to vow not to hold a grudge against the wrongdoer.", "question": "The statements above, if true, most strongly support which one of the following?", "answers": "['An apology cannot be both sincerely offered and sincerely accepted unless each person acknowledges that a wrongful act has occurred.', 'One cannot sincerely accept an apology that was not sincerely offered.', 'If one commits a wrongful act, then one should sincerely apologize for that act.', 'If one apologizes and subsequently repeats the wrongful act for which one has apologized, then one has not apologized sincerely.']", "label": 0 }, { "id": "train_97", "context": "Art critic: Nowadays, museum visitors seldom pause to look at a work of art for even a full minute. They look, perhaps take a snapshot, and move on. This tells us something about how our relationship to art has changed over time. People have become less willing to engage with works of art than they once were.", "question": "The art critic's argument depends on the assumption that", "answers": "['visitors would enjoy their museum experiences more if they took more time with individual works of art', 'the amount of time spent looking at a work of art is a reliable measure of engagement with that work', 'museum visitors who take snapshots of works of art rarely look at the pictures afterward', 'museum visitors today generally look at more pieces of art during each museum visit than museum visitors looked at in the past']", "label": 1 }, { "id": "train_98", "context": "If we are to expand the exploration of our solar system, our next manned flight should be to Phobos, one of Mars' s moons, rather than to Mars itself. The flight times to each are the same, but the Phobos expedition would require less than half the fuel load of a Mars expedition and would, ttherefore, be much less costly. So, it is clear that Phobos should be our next step in space exploration.", "question": "Which one of the following, if true, would most help to explain the difference in fuel requirements?", "answers": "['Smaller spaceships require less fuel than larger spaceships.', 'The shortest distance between Phobos and Mars is less than half the shortest distance between Earth and Mars.', 'More equipment would be required to explore Phobos than to explore Mars.', \"Lift-off for the return trip from Phobos requires much less fuel than that from Mars because of Phobos' weaker gravitational pull.\"]", "label": 3 }, { "id": "train_99", "context": "The position that punishment should be proportional to how serious the offense is but that repeat offenders should receive harsher punishments than first-time offenders is unsustainable. It implies that considerations as remote as what an offender did years ago are relevant to the seriousness of an offense. If such remote considerations were relevant, almost every other consideration would be too. But this would make determining the seriousness of an offense so difficult that it would be impossible to apply the proportionality principle.", "question": "The statement that considerations as remote as what an offender did years ago are relevant to the seriousness of an offense plays which one of the following roles in the argument?", "answers": "[\"It is an allegedly untenable consequence of a view rejected in the argument's overall conclusion.\", 'It is a statement the argument provides grounds to accept and from which the overall conclusion is inferred.', 'It is the overall conclusion in favor of which the argument offers evidence.', 'It is a premise offered in support of an intermediate conclusion of the argument.']", "label": 0 }, { "id": "train_100", "context": "A new commercial radio station in Greenfield plans to play songs that were popular hits fifteen to twenty-five years ago. It hopes in this way to attract an audience made up mainly of people between thirty-five and forty-five years old and thereby to have a strong market appeal to advertisers.", "question": "Each of the following, if true, strengthens the prospects that the radio station's plan will succeed EXCEPT:", "answers": "['The thirty-five- to forty-five-year-old age group is one in which people tend to have comparatively high levels of income and are involved in making household purchases.', 'People in the thirty-five- to forty-five-year-old age group are more likely to listen to the radio for news than for music.', 'In a number of cities demographically similar to Greenfield, radio stations that play recordings of popular music from fifteen to twenty-five years ago have succeeded commercially.', 'Among the few radio stations in the Greenfield area, there is none that plays music from this particular period for more than a few hours per week.']", "label": 1 }, { "id": "train_101", "context": "It is often said that beauty is subjective. But this judgment has to be false. If one tries to glean the standard of beauty of earlier cultures from the artistic works they considered most beautiful, one cannot but be impressed by its similarity to our own standard. In many fundamental ways, what was considered beautiful in those cultures is still considered beautiful in our own time.", "question": "Which one of the following statements, if true, most weakens the argument?", "answers": "['In most cultures art is owned by a small social elite.', 'Our own standard of beauty was strongly influenced by our exposure to works that were considered beautiful in earlier cultures.', 'Few contemporary artists have been significantly exposed to the art of earlier cultures.', 'The arts held a much more important place in earlier cultures than they do in our culture.']", "label": 1 }, { "id": "train_102", "context": "Altogether, the students in Ms. Tarnowski' s Milton Elementary School class collected more aluminum cans than did the students in any of the school' s other classes. Ttherefore, the Milton student who collected the most aluminum cans was in Ms. Tarnowski' s class.", "question": "Which one of the following arguments contains flawed reasoning that is most parallel to that in the argument above?", "answers": [ "Mr. Rowe's Milton Elementary School class raised more money by selling candy bars than Ms. Hunt's class raised by holding a raffle. Ttherefore, the number of candy bars sold by Mr. Rowe's class was greater than the number of raffle tickets sold by Ms. Hunt's class.", "More than half of Milton Elementary School's students play in the band and more than half of the school's students sing in the choir. Ttherefore, every student at Milton Elementary School either plays in the band or sings in the choir.", "The total number of tickets to the school fair sold by the students in Ms. Ramirez's Milton Elementary School class was greater than the number sold by Milton students from any other class. Ttherefore, the Milton student who sold the most tickets to the school fair was a student in Ms. Ramirez's class.", "Ms. Ventura's Milton Elementary School class assembled more birdhouses than did any of the school's other classes. Since Ms. Ventura's class had fewer students than any other Milton class, her students assembled more birdhouses, on average, than did the students in any other Milton class." ], "label": 2 }, { "id": "train_103", "context": "Our tomato soup provides good nutrition: for instance, a warm bowl of it contains more units of vitamin C than does a serving of apricots or fresh carrots!", "question": "The advertisement is misleading if which one of the following is true?", "answers": "['Few people depend exclusively on apricots and carrots to supply vitamin C to their diets.', 'A liquid can lose vitamins if it stands in contact with the air for a protracted period of time.', 'The amount of vitamin C provided by a serving of the advertised soup is less than the amount furnished by a serving of fresh strawberries.', 'Apricots and fresh carrots are widely known to be nutritious, but their contribution consists primarily in providing a large amount of vitamin A, not a large amount of vitamin C.']", "label": 3 }, { "id": "train_104", "context": "Harry: Airlines have made it possible for anyone to travel around the world in much less time than was formerly possible. Judith: That is not true. Many nights are too expensive for all but the rich.", "question": "Judith's response shows that she interprets Harry's statement to imply that", "answers": "['world travel is only possible via routes serviced by airlines', 'most forms of world travel are not affordable for most people', 'anyone can afford to travel long distances by air', 'the majority of people are rich']", "label": 2 }, { "id": "train_105", "context": "Food company engineer: I stand by my decision to order the dumping of small amounts of chemicals into the local river even though there is some evidence that this material may pose health problems. 1 fish in the river myself and will continue to do so. Furthermore, I will have no problem if other food manufacturers do what our company does.", "question": "The engineer's reasoning most closely conforms to which one of the following principles?", "answers": "['One is justified in performing an act if one is willing to submit oneself to the consequences of that action performed by oneself or others.', 'One should always choose to act in a way that will benefit the greatest number of people.', 'One is justified in performing an act if other people are also planning to perform that kind of act.', 'One should never perform an act until one has fully analyzed all the ways in which that act could impact others.']", "label": 0 }, { "id": "train_106", "context": "Although nondairy coffee lighteners made with coconut oil contain 2 grams of saturated fat per tablespoon, or 7 times more than does whole milk, those lighteners usually contain no cholesterol. Yet one tablespoon of such lighteners causes the consumer' s blood cholesterol to rise to a higher level than does an identical amount of whole milk, which contains 2 milligrams of cholesterol per tablespoon.", "question": "Which one of the following, if true, contributes most to an explanation of the apparent discrepancy noted above?", "answers": "['One gram of saturated fat in food has roughly the same effect on blood cholesterol as 25 milligrams of cholesterol in food.', 'Light cream, a dairy product that contains 5 times more cholesterol than does whole milk, is often chosen as a lightener by consumers who normally prefer whole milk.', 'Certain nondairy coffee lighteners made without coconut oil contain less saturated fat and less cholesterol than does whole milk.', 'Nutritionists recommend that adults consume as little saturated fat as possible and no more than 250 milligrams of cholesterol a day.']", "label": 0 }, { "id": "train_107", "context": "The authors of a recent article examined warnings of an impending wave of extinctions of animal species within the next 100 years. These authors say that no evidence exists to support the idea that the rate of extinction of animal species is now accelerating. They are wrong, however. Consider only the data on fishes: 40 species and subspecies of North American fishes have vanished in the twentieth century, 13 between 1900 and 1950, and 27 since 1950.", "question": "Which one of the following is the main point of the argument?", "answers": "['A substantial number of fish species are in danger of imminent extinction.', 'The rate of extinction of North American fishes is parallel to the rate of extinction of all animal species taken together.', 'There is evidence that the rate of extinction of animal species is accelerating.', 'Forty species and subspecies of North American fishes have vanished in the twentieth century.']", "label": 2 }, { "id": "train_108", "context": "Ethanol is a derivative of corn and other grains. When burned as fuel, it emits significantly lower levels of carbon monoxide, a major atmospheric pollutant, than does gasoline. For that reason, environmentalists claim that ethanol is a better source of energy than gasoline.", "question": "Which of the following, if true, most strongly supports the environmentalists' claim?", "answers": "['Ethanol is comparable in price to gasoline.', 'When burned as fuel, ethanol does not release any pollutants at higher levels than does gasoline.', \"Available supplies of corn are sufficient to supply several years' worth of ethanol.\", 'Ethanol can be used as heating fuel.']", "label": 1 }, { "id": "train_109", "context": "Anselm of Canterbury (1033 -- 1109) was a medieval theologian. According to Anselm' s ontological argument for the existence of God, \"accidental beings\" are all those things -- essentially all sense objects -- whose non-existence could be imagined without inherent contradiction, and \"necessary beings\" are those things whose existence is guaranteed precisely by what they are. Because accidental beings could not have guaranteed that they ever would come into existence, there must be a necessary being upon whom all the accidental beings depends to bring them into existence; and this necessary being Anselm identifies with God, who ttherefore clearly must exist.", "question": "In our modern analysis, this eleventh century argument is most vulnerable to what criticism?", "answers": "['It presents as evidence in support of a claim information that is inconsistent with other evidence presented in support of the same claim.', 'It establishes an effect that must exist well before its cause.', 'It makes a distinction that presupposes the truth of the conclusions that is to be established.', 'The conclusion supports facts that directly contradict the evidence given to support it.']", "label": 2 }, { "id": "train_110", "context": "The indigenous people of Tasmania are clearly related to the indigenous people of Australia, but were separated from them when the land bridge between Australia and Tasmania disappeared approximately 10, 000 years ago. Two thousand years after the disappearance of the land bridge, however, there were major differences between the culture and technology of the indigenous Tasmanians and those of the indigenous Australians. The indigenous Tasmanians, unlike their Australian relatives, had no domesticated dogs, fishing nets, polished stone tools, or hunting implements like the boomerang and the spear-thrower.", "question": "Each of the following, if true, would contribute to an explanation of differences described above EXCEPT:", "answers": "['Indigenous people of Australia developed hunting implements like the boomerang and the spear-thrower after the disappearance of the land bridge.', 'After the disappearance of the land bridge the indigenous Tasmanians simply abandoned certain practices and technologies that they had originally shared with their Australian relatives.', 'Although the technological and cultural innovations were developed in Australia more than 10, 000 years ago, they were developed by groups in northern Australia with whom the indigenous Tasmanians had no contact prior to the disappearance of the land bridge.', 'Devices such as the spear-thrower and the boomerang were developed by the indigenous Tasmanians more than 10, 000 years ago.']", "label": 3 }, { "id": "train_111", "context": "While studies indicate that university professors spend much of their time engaged in personal research not clearly related to teaching, it is unlikely that additional financial compensation for, say, a larger course load or longer office hours would be the most effective of various possible means of shifting the focus of their activities; this would be the best means only if their motivation to educate were no more than a mechanical response to the prospect of more money, which surely is not the case.", "question": "Which one of the following most accurately expresses the main conclusion of the argument?", "answers": "['The research conducted by most professors should be clearly related to their teaching activities.', 'Monetary incentives would probably be less effective than some other ways that professors might be induced to focus more on teaching activities.', 'Increased financial compensation would be a somewhat effective way to motivate professors to focus their activities on teaching.', 'Effective incentives to encourage larger course loads or longer office hours would shift the focus of professors toward teaching.']", "label": 1 }, { "id": "train_112", "context": "The difference between manners and morals is that the former are necessarily social in nature whereas the latter are not necessarily social in nature. So the rules of etiquette do not apply when one is alone.", "question": "The statements above, if true, most strongly support which one of the following inferences?", "answers": "['One could be immoral without ever having caused any other person any harm', 'What is social in nature could not be a matter of morality.', 'It is more important to be moral than to have good manners.', 'The rules of morality apply only when one is alone.']", "label": 0 }, { "id": "train_113", "context": "The problem that environmental economics aims to remedy is the following: people making economic decisions cannot readily compare environmental factors, such as clean air and the survival of endangered species, with other costs and benefits. As environmental economists recognize, solving this problem requires assigning monetary values to environmental factors. But monetary values result from people comparing costs and benefits in order to arrive at economic decisions. Thus, environmental economics is stymied by what motivates it.", "question": "If the considerations advanced in its support are true, the passage's conclusion is supported", "answers": "['at best weakly, because the passage fails to establish that economic decision-makers do not by and large take adequate account of environmental factors', 'not at all, since the argument is circular, taking that conclusion as one of its premises', 'strongly, on the assumption that monetary values for environmental factors cannot be assigned unless people make economic decisions about these factors', 'strongly, unless economic decision-making has not yet had any effect on the things categorized as environmental factors']", "label": 2 }, { "id": "train_114", "context": "Despite the fact that antilock brakes are designed to make driving safer, research suggests that people who drive cars equipped with antilock brakes have more accidents than those who drive cars not equipped with antilock brakes.", "question": "Each of the following, if true, would help resolve the apparent discrepancy described above EXCEPT:", "answers": "['Antilock brakes require expensive specialized maintenance to be even as effective as unmaintained regular brakes.', 'Antilock brakes were designed for safety in congested urban driving, but accidents of the most serious nature take place on highways.', 'Most people who drive cars equipped with antilock brakes do not know how to use those brakes properly.', 'Antilock brakes malfunction more often than regular brakes.']", "label": 1 }, { "id": "train_115", "context": "Eighteenth-century European aesthetics was reasonably successful in providing an understanding of all art, including early abstract art, until the 1960s, when artists self-consciously rebelled against earlier notions of art. Since the work of these rebellious artists is quite beautiful but outside the bounds of the aesthetic theory then current, there can be no complete theory of aesthetics.", "question": "The reasoning above is most vulnerable to criticism in that it", "answers": "['takes for granted that it is more important for a complete aesthetic theory to account for the beauty of traditional art than for it to account for the beauty of self-consciously rebellious art', 'presumes, without providing justification, that eighteenth-century European aesthetics is as encompassing as an aesthetic theory can be', 'presumes, without providing justification, that an aesthetic theory developed in one part of the world cannot be applied in another', \"presumes, without providing justification, that artists' rebellion in the 1960s against earlier notions of art was not guided by their knowledge of eighteenth-century European aesthetic theory\"]", "label": 1 }, { "id": "train_116", "context": "Several ancient Greek texts provide accounts of people being poisoned by honey that texts suggest was made from the nectar of rhododendron or oleander plants. Honey made from such nectar can cause the effects the texts describe, but only if eaten fresh, since the honey loses its toxicity within a few weeks of being made. In Greece, rhododendrons and oleander bloom only in springtime, when they are the predominant sources of nectar.", "question": "Which of the following, if true, most strongly support the accounts of Greek texts?", "answers": "[\"A beehive's honeycomb cannot have some sections that contain toxic honey and other sections that contain nontoxic honey\", 'The incidents of honey poisoning that are related in the ancient texts occurred in the springtime or in the early summer', 'Whether the honey in a beehive is toxic depends solely on which plants were the source of that was used to make the honey.', 'There are no plants that Greece in ancient times that produce more nectar than rhododendrons or oleanders does.']", "label": 1 }, { "id": "train_117", "context": "When a married couple has frequent emotionally satisfying conversations, they tend in overwhelming percentages to remain married throughout their lives. Queen Melinda and Prince Jonathan, Duke of Westphalia, have been married for over sixty years, so clearly they must have emotionally satisfying conversations all the time.", "question": "This argument is most vulnerable to what criticism?", "answers": "['It takes a condition to be the effect of something that has happened only after the condition already existed.', 'It takes one possible cause of a condition to be the actual cause of that condition without considering any other possible causes.', 'It makes a distinction that presupposes the truth of the conclusions that is to be established.', 'It presents as evidence in support of a claim information that is inconsistent with other evidence presented in support of the same claim.']", "label": 1 }, { "id": "train_118", "context": "If you study history, then you will appreciate the vast differences among past civilizations, and you will appreciate these differences provided that you reflect on your own civilization. Hence, if you study history you will reflect on your own civilization.", "question": "Which one of the following is most closely parallel in its flawed reasoning to the flawed reasoning in the argument above?", "answers": "['One can become informed about the world provided that one reads the newspaper daily. If one is informed about the world, then one has an appreciation of other cultures. So if one reads the newspaper daily, then one can come to appreciate other cultures.', 'If you learn Latin, you can improve your vocabulary, and you can improve your vocabulary if you study great works of literature. So you will study great works of literature if you learn Latin.', \"Traveling to other countries deepens one's appreciation for their cultures, and this appreciation often encourages one to study the history of those lands. So the study of history increases one's desire to travel.\", 'By studying ancient art you begin to appreciate how much was accomplished with limited materials. Appreciation of ancient art leads to a deeper understanding of modem art. Ttherefore, studying ancient art can engender a profound new appreciation for modem art.']", "label": 1 }, { "id": "train_119", "context": "Herpetologist: Some psychologists attribute complex reasoning to reptiles, claiming that simple stimulus-response explanations of some reptiles' behaviors, such as food gathering, cannot account for the complexity of such behavior. But since experiments show that reptiles are incapable of making major alterations in their behavior, for example, when faced with significant changes in their environment, these animals must be incapable of complex reasoning.", "question": "Which one of the following is an assumption required by the herpetologist's argument?", "answers": "['Reptile behavior appears more complex in the field than laboratory experiments reveal it to be.', 'If reptiles were capable of complex reasoning, they would sometimes be able to make major changes in their behavior.', 'Simple stimulus-response explanations can in principle account for all reptile behaviors.', 'Animals could make major changes in their behavior only if they were capable of complex reasoning.']", "label": 1 }, { "id": "train_120", "context": "Clearly, fitness consultants who smoke cigarettes cannot help their clients become healthier. If they do not care about their own health, they cannot really care for their clients' health, and if they do not care for their clients' health, they cannot help them to become healthier.", "question": "The conclusion follows logically if which one of the following is assumed?", "answers": "['Anyone who does not care for his or her own health cannot help others become healthier.', 'Anyone who cares about his or her own health does not smoke.', 'Anyone who cares about the health of others can help others become healthier.', 'Anyone who does not care for the health of others cannot help them become healthier.']", "label": 1 }, { "id": "train_121", "context": "Consumer advocate: A recent study concluded that top-loading washing machines are superior overall to front-loaders. But front-loaders have the controls and access in front. This is more convenient for wheelchair users, some of whom find it highly inconvenient to remove laundry from top-loaders. So for some consumers front-loaders are superior.", "question": "Which one of the following is an assumption upon which the consumer advocate's argument depends?", "answers": "['For some consumers the convenience of front- loaders outweighs the advantages of top-loaders in assessing which is superior.', 'Convenience is the only important factor in determining which type of washing machine is superior.', 'Retrieving clothes from front-loaders is inconvenient for people who are not wheelchair users.', 'Retrieving clothes from a top-loader is convenient for people who do not use wheelchairs.']", "label": 0 }, { "id": "train_122", "context": "Counselor: Those who believe that criticism should be gentle rather than harsh should consider the following: change requires a motive, and criticism that is unpleasant provides a motive. Since harsh criticism is unpleasant, harsh criticism provides a motive. Ttherefore, only harsh criticism will cause the person criticized to change.", "question": "The reasoning in the counselor's argument is most vulnerable to criticism on the grounds that the argument", "answers": "['fails to address the possibility that in some cases the primary goal of criticism is something other than bringing about change in the person being criticized', 'takes for granted that everyone who is motivated to change will change', 'infers that something that is sufficient to provide a motive is necessary to provide a motive', 'confuses a motive for doing something with a motive for avoiding something']", "label": 2 }, { "id": "train_123", "context": "Many scientific studies have suggested that taking melatonin tablets can induce sleep. But this does not mean that melatonin is helpful in treating insomnia. Most of the studies examined only people without insomnia, and in many of the studies, only a few of the subjects given melatonin appeared to be significantly affected by it.", "question": "Which one of the following, if true, most strengthens the argument?", "answers": "['None of the studies that suggested that taking melatonin tablets can induce sleep examined a fully representative sample of the human population.', 'Several people who were in control groups and only given placebos claimed that the tablets induced sleep.', 'If melatonin were helpful in treating insomnia, then every person with insomnia who took doses of melatonin would appear to be significantly affected by it.', 'In the studies that included subjects with insomnia, only subjects without insomnia were significantly affected by doses of melatonin.']", "label": 3 }, { "id": "train_124", "context": "Doctors urge people to reduce their cholesterol levels through dietary changes. But moderate dietary changes often do not work to lower cholesterol levels. One may need, ttherefore, to make more dramatic changes, such as switching to a vegetarian diet.", "question": "The statement that moderate dietary changes often do not work to lower cholesterol levels plays which one of the following roles in the argument?", "answers": "['It is offered as an explanation of the success of vegetarian diets in reducing cholesterol levels.', \"It is a conclusion for which the claim that dramatic changes in one's diet are sometimes required to reduce cholesterol levels is offered as support.\", 'It is a premise offered in support of the claim that reducing cholesterol levels may require greater than moderate dietary changes.', 'It is a premise offered in support of the claim that vegetarian diets are more healthful than any diets containing meat.']", "label": 2 }, { "id": "train_125", "context": "Ph. D. programs are valuable only if they inculcate good scholarship and expedite the student' s full participation in the field. Hence, doctoral dissertations should not be required in the humanities. Undertaking a quality book-length dissertation demands an accumulation of knowledge virtually impossible for those relatively new to their disciplines. The student consequently either seeks to compensate for poor quality with quantity or ends up spending years producing a work of quality. Either way, the dissertation is counterproductive and frustrates the appropriate goals of the doctoral program.", "question": "The claim that doctoral dissertations should not be required in the humanities plays which one of the following roles in the argument?", "answers": "['It provides evidence for the assumption that requirements for degrees in the humanities differ from requirements for degrees in other disciplines.', 'It confirms the observation that the requirement for a dissertation can frustrate the goals of a doctoral program.', 'It is what the argument is attempting to establish.', 'It provides essential support for the conclusion.']", "label": 2 }, { "id": "train_126", "context": "A fourteen-year study of finches on the Galapagos islands concluded that there is a definite relationship between climate and the population size of finch species that thrive at various times. During droughts, more members of large finch species survive because their bills are large enough to crack large, hard seeds, giving them a food supply unavailable to smaller birds. In rainy years, fewer members of the large finch species survive because the additional moisture fosters the growth of plants that produce small seeds. The larger finch varieties have to consume enormous numbers of small seeds to meet their energy demands, and some just cannot eat them fast enough.", "question": "Which one of the following must be assumed in order to justify the conclusion that climatic variations cause a major difference in survival rates of small and large finches?", "answers": "['During drought conditions, the weather promotes the growth of plants that produce small, hard seeds.', 'A lengthy period of rainy weather results in fewer large, hard seeds being produced.', 'In rainy periods, the small finches gather enough food to grow much larger and heavier, but their ultimate size is limited by their inability to eat small seeds fast.', 'The Galapagos climate during this fourteen- year period had about as much dry weather as it had wet weather.']", "label": 1 }, { "id": "train_127", "context": "Certain politicians in the country of Birangi argue that a 50 percent tax on new automobiles would halt the rapid increase of automobiles on Birangi' s roads and thereby slow the deterioration of Birangi' s air quality. Although most experts agree that such a tax would result in fewer Birangians buying new vehicles and gradually reduce the number of automobiles on Birangi' s roads, they contend that it would have little impact on Birangi' s air-quality problem.", "question": "Which of the following, if true in Birangi, would most strongly support the experts' contention about the effect of the proposed automobile tax on Birangi's air-quality problem?", "answers": "['Automobiles become less fuel efficient and ttherefore contribute more to air pollution as they age.', 'Currently, the sales tax on new tax on new automobiles is considerably lower than 50 percent.', 'Some of the proceeds from the new tax would go toward expanding the nonpolluting commuter rail system.', 'The scrapping of automobiles causes insignificant amounts of air pollution.']", "label": 0 }, { "id": "train_128", "context": "Multiple sclerosis is an autoimmune disease: white blood cells attack the myelin sheath that protects nerve fibers in the spinal cord and brain. Medical science now has a drug that can be used to successfully treat multiple sclerosis, but the path that led medical researchers to this drug was hardly straightforward. Initially, some scientists believed attacks characteristic of multiple sclerosis might be triggered by chronic viral infections. So in 1984 they began testing gamma interferon, one of the body' s own antiviral weapons. To their horror, all the multiple sclerosis patients tested became dramatically worse. The false step proved to be instructive however.", "question": "Which one of the following is LEAST compatible with the results of the gamma interferon experiment?", "answers": "['Gamma interferon stops white blood cells from producing myelin-destroying compounds.', 'Administering gamma interferon to those without multiple sclerosis causes an increase in the number of white blood cells.', 'The drug now used to treat multiple sclerosis is known to inhibit the activity of gamma interferon.', 'Medical researchers have discovered that the gamma interferon level in the cerebrospinal fluid skyrockets just before and during multiple sclerosis attacks.']", "label": 0 }, { "id": "train_129", "context": " Delta products, Inc. , has recently switched at least partly from older technologies using fossil fuels to new technologies powered by electricity . The question has been raised whether it can be concluded that for a given level of output, Delta' s operation now causes less fossil fuel to be consumed than it did formerly. The answer, clearly, is yes, since the amount of fossil fuel used to generate the electricity needed to power the new technologies is less than the amount needed to power the older technologies, provided that the level of output is held constant. ", "question": "In the argument given, the two boldface portions play which of the following roles?", "answers": "['The first identifies the content of the conclusion of the argument; the second provides support for that conclusion', 'Each provides evidence that calls the conclusion of the argument into question.', 'Each provides support for the conclusion of the argument', 'The first states the position that the argument opposes; the second states the conclusion of the argument']", "label": 2 }, { "id": "train_130", "context": "Acquiring complete detailed information about all the pros and cons of a product one might purchase would clearly be difficult and expensive. It is rational not to acquire such information unless one expects that the benefits of doing so will outweigh the cost and difficulty of doing so. Ttherefore, consumers who do not bother to acquire such information are thereby behaving rationally.", "question": "The conclusion of the argument is properly drawn if which one of the following is assumed?", "answers": "['Whenever it is rational not to acquire detailed information about a product, it would be irrational to bother to acquire such information.', 'The benefits of acquiring detailed information about a product one might purchase usually do not outweigh the cost and difficulty of doing so.', 'Rational consumers who do not expect that the benefits outweigh the cost and difficulty of acquiring detailed information about a product they might purchase usually do not bother to acquire such information.', 'Consumers who do not bother to acquire complete detailed information about a product they might purchase do not expect that the benefits of acquiring such information will outweigh the cost and difficulty of doing so.']", "label": 3 }, { "id": "train_131", "context": "Archaeologist: The allegation that members of the excavation team recovered artifacts outside the authorized site is unfounded. Archaeologists, unlike most treasure hunters, excavate artifacts to publish the results of their findings. But material recovered illegally could not be used in a publication without the illegal act being discovered. So it would be of no use to an archaeologist.", "question": "The archaeologist's reasoning is most vulnerable to criticism because it", "answers": "['presumes without providing adequate justification that most treasure hunters excavate artifacts to sell them', 'assumes without providing warrant that any use of illegally recovered material is itself illegal', 'ignores the possibility that not all members of the excavation team were archaeologists', 'fails to consider that not all treasure hunters act illegally']", "label": 2 }, { "id": "train_132", "context": "Unless negotiations begin soon, the cease-fire will be violated by one of the two sides to the dispute. Negotiations will be held only if other countries have pressured the two sides to negotiate; an agreement will emerge only if other countries continue such pressure throughout the negotiations. But no negotiations will be held until international troops enforcing the cease-fire have demonstrated their ability to counter any aggression from either side, thus suppressing a major incentive for the two sides to resume fighting.", "question": "If the statements above are true, and if negotiations between the two sides do begin soon, at the time those negotiations begin each of the following must also be true EXCEPT:", "answers": "['International troops enforcing the cease-fire have demonstrated that they can counter aggression from either of the two sides.', 'A major incentive for the two sides to resume hostilities has been suppressed.', 'The cease-fire has not been violated by either of the two sides.', \"The negotiations' reaching an agreement depends in part on the actions of other countries.\"]", "label": 2 }, { "id": "train_133", "context": "Of every 100 burglar alarms police answer, 99 are false alarms. This situation causes an enormous and dangerous drain on increasingly scarce public resources. Each false alarm wastes an average of 45 minutes of police time. As a result police are consistently taken away from responding to other legitimate calls for service, and a disproportionate share of police service goes to alarm system users, who are mostly businesses and affluent homeowners. However, burglar alarm systems, unlike car alarm systems, are effective in deterring burglaries, so the only acceptable solution is to fine burglar alarm system owners the cost of 45 minutes of police time for each false alarm their systems generate.", "question": "The statement that burglar alarm systems, unlike car alarm systems, are effective in deterring burglaries plays which one of the following roles in the argument?", "answers": "['It provides a basis for excluding as unacceptable one obvious alternative to the proposal of fining owners of burglar alarm systems for false alarms.', 'It explains why a disproportionate number of the burglar alarms responded to by police come from alarm systems owned by businesses.', 'It gives a reason why police might be more inclined to respond to burglar alarms than to car alarms.', 'It provides background information needed to make plausible the claim that the number of burglar alarms police are called on to answer is great enough to be a drain on public resources.']", "label": 0 }, { "id": "train_134", "context": "The population of songbirds throughout England has decreased in recent years. Many people explain this decrease as the result of an increase during the same period in the population of magpies, which eat the eggs and chicks of songbirds.", "question": "Which one of the following, if true, argues most strongly against the explanation reported in the passage?", "answers": "[\"The number of eggs laid yearly by a female songbird varies widely according to the songbird's species.\", \"Although magpies eat the eggs and chicks of songbirds, magpies' diets consist of a wide variety of other foods as well.\", 'The population of magpies has increased because farmers no longer shoot or trap magpies to any great extent, though farmers still consider magpies to be pests.', 'Although the overall population of magpies has increased, in most areas of England in which the songbird population has decreased, the number of magpies has remained stable.']", "label": 3 }, { "id": "train_135", "context": "All known living things are made of the same basic kinds of matter, are carbon based, and are equipped with genetic codes. So human life has the same origin as all other known life.", "question": "The conclusion follows logically if which one of the following is assumed?", "answers": "['There can never be any living thing that does not have a genetic code.', 'Any two living things made of the same basic kinds of matter have the same origin.', 'Without the existence of other life forms, human life would never have come into existence.', 'There are not any living beings that have genetic codes but are not carbon based.']", "label": 1 }, { "id": "train_136", "context": "It is wrong to waste our natural resources, and it is an incredible waste of resources to burn huge amounts of trash in incinerators. When trash is recycled, fewer resources are wasted. Because less trash will be recycled if an incinerator is built, the city should not build an incinerator.", "question": "Which one of the following can be properly inferred from the statements above?", "answers": "['If the city does not burn trash, it will not waste resources.', 'By recycling more trash, the city can stop wasting resources entirely.', 'If the city is to avoid wasting resources, huge amounts of trash cannot be burned in any city incinerator.', 'The most effective way to conserve resources is to recycle trash.']", "label": 2 }, { "id": "train_137", "context": "Scientists are discussing ways to remove excess carbon dioxide from the atmosphere by increasing the amount that is absorbed by plant life. One plan to accomplish this is to establish giant floating seaweed farms in the oceans. When the seaweed plants die, they will be disposed of by being burned for fuel.", "question": "Which of the following, if true, would indicate the most serious weakness in the plan above?", "answers": "['Even if seaweed farms prove effective, some people will be reluctant to switch to this new fuel.', 'Some areas of ocean in the Southern Hemisphere do not contain sufficient nutrients to support large seaweed farms.', 'Each year about seven billion tons of carbon dioxide are released into the atmosphere but only about five billion tons are absorbed by plant life.', 'When a seaweed plant is burned, it releases an amount of carbon dioxide comparable to the amount it has absorbed in its lifetime.']", "label": 3 }, { "id": "train_138", "context": "Research has shown that shoppers who purchase garments in a \"luxury\" material, such as cashmere or angora, are likely to purchase garments in more than one other luxury materials, as well. A recent livestock disease has significantly reduced the amount of cashmere available on the market. This, in turn, has driven up the prices of the cashmere products still available for purchase. We can ttherefore conclude that more people will be buying angora items of clothing this year than they did in previous years.", "question": "Which of the following, if true, best supports the claim that more people will be buying angora clothing this year?", "answers": "['The number of customers purchasing angora has increased every year for the past decade in steady increments.', 'In a recent study, shoppers who had recently purchased a garment in a luxury material cited similar reasons for buying angora as buying cashmere.', 'Most consumers cannot tell the difference between cashmere and angora, which are both expensive and similar in texture.', 'The same disease that affected the livestock producing cashmere has also affected the farms that produce angora.']", "label": 1 }, { "id": "train_139", "context": "Like airplane accidents, most accidents in medical care result from human error, particularly failures in communication, leadership, and decision making. Following the introduction of standard procedures governing these three areas, the airline industry succeeded in significantly reducing the number and severity of accidents. Since airplane flights and health care delivery both require the cooperation of groups of people with diverse knowledge and skills, the medical care community should adopt a similar set of procedures in order to reduce errors.", "question": "Which one of the following is a technique of reasoning used in the argument?", "answers": "['providing examples in order to show that two seemingly dissimilar situations are in fact the same', 'defending a general principle by presenting two separate successful applications of that principle', 'arguing for taking a course of action based on results of taking such action in an analogous situation', 'justifying the introduction of a set of procedures by outlining undesirable results in an analogous situation in which those procedures were not followed']", "label": 2 }, { "id": "train_140", "context": "The United States government generally tries to protect valuable natural resources. But one resource has been ignored for too long. In the United States, each bushel of corn produced might result in the loss of as much as two bushels of topsoil. Moreover, in the last 100 years, the topsoil in many states, which once was about fourteen inches thick, has been eroded to only six or eight inches. Nonetheless, federal expenditures for nationwide soil conservation programs have remained at ridiculously low levels. Total federal expenditures for nationwide soil conservation programs have been less than the allocations of some individual states.", "question": "Which one of the following best expresses the main point of the argument?", "answers": "[\"The federal government's expenditures for soil conservation in the various states have been inequitable.\", 'Corn is not a cost-effective product and substitutes should be found where possible.', 'Soil conservation is a responsibility of the federal government, not the states.', 'The federal government should spend much more on soil conservation than it has been spending.']", "label": 3 }, { "id": "train_141", "context": "Critic: Photographers, by deciding which subjects to depict and how to depict them, express their own worldviews in their photographs, however realistically those photographs may represent reality. Thus, photographs are interpretations of reality.", "question": "The argument's conclusion is properly drawn if which one of the following is assumed?", "answers": "['To express a worldview is to interpret reality.', \"All visual art expresses the artist's worldview.\", 'Any interpretation of reality involves the expression of a worldview.', 'Nonrealistic photographs, like realistic photographs, express the worldviews of the photographers who take them.']", "label": 0 }, { "id": "train_142", "context": "Columnist: Over the last 20 years the demand in North America for Japanese-made automobiles has increased, whereas the Japanese demand for North American-made automobiles has been stagnant. Until recently, this imbalance could plausibly be attributed to Japanese models' superior fuel efficiency and reliability, but now cars made in North America are by these standards the equal of any Japanese model. What North American exporters continue to ignore is that the Japanese drive on the left side of the road. Ttherefore, one obstacle to reducing the automotive trade imbalance will be removed if North American manufacturers simply produce more cars with right-side steering wheels.", "question": "Which one of the following is an assumption required by the columnist's argument?", "answers": "['If the Japanese drive on the left side of the road, then they are not inclined to buy cars with left-side steering wheels.', 'Given a choice between similar Japanese and North American models, all with right-side steering wheels, most Japanese would choose the North American model.', 'Japanese automotive safety standards require that all new domestic automobiles have rightside steering wheels.', 'The automotive trade imbalance can be lessened only if North American manufacturers produce automobiles that more effectively meet the needs of Japanese buyers.']", "label": 0 }, { "id": "train_143", "context": "The tendency toward overspecialization in the study of artifacts is unfortunate. Scholars can enhance their understanding of a certain artistic period by studying art from earlier periods that had a significant influence on it. For instance, because of its influence on Spanish artisans, a proper understanding of Arabic porcelain is indispensable for a proper understanding of Spanish porcelain.", "question": "Of the following, which one most closely conforms to the principle that the passage as a whole illustrates?", "answers": "['To appreciate fully the French language, one must understand the other languages that share its linguistic ancestry.', 'To understand properly any academic discipline, one must have at least a superficial acquaintance with the practices of the wider academic community.', 'To understand fully the historical events of this century, a historian must have an understanding of similar events in earlier centuries.', \"To understand completely Aristotle's philosophy, one must be well acquainted with the philosophy of his intellectual mentor, Plato.\"]", "label": 3 }, { "id": "train_144", "context": "Scientist: My research indicates that children who engage in impulsive behavior similar to adult thrill-seeking behavior are twice as likely as other children to have a gene variant that increases sensitivity to dopamine. From this, I conclude that there is a causal relationship between this gene variant and an inclination toward thrill-seeking behavior.", "question": "Which one of the following, if true, most calls into question the scientist's argument?", "answers": "['Children are often described by adults as engaging in thrill-seeking behavior simply because they act impulsively.', 'Many impulsive adults are not unusually sensitive to dopamine.', 'Many people exhibit behavioral tendencies as adults that they did not exhibit as children.', 'It is not possible to reliably distinguish impulsive behavior from other behavior.']", "label": 3 }, { "id": "train_145", "context": "Editorialist: In a large corporation, one of the functions of the corporation' s president is to promote the key interests of the shareholders. Ttherefore, the president has a duty to keep the corporation' s profits high.", "question": "Which one of the following, if true, would most strengthen the editorialist's argument?", "answers": "['In considering where to invest, most potential shareholders are interested in more than just the profitability of a corporation.', 'Shareholders sometimes will be satisfied even if dividends paid to them from company profits are not high.', 'The president and the board of directors of a corporation are jointly responsible for advancing the key interests of the shareholders.', \"Keeping a corporation's profits high is likely to advance the important interests of the corporation's shareholders.\"]", "label": 3 }, { "id": "train_146", "context": "Light is registered in the retina when photons hit molecules of the pigment rhodopsin and change the molecules' shape. Even when they have not been struck by photons of light, rhodopsin molecules sometimes change shape because of normal molecular motion, thereby introducing error into the visual system. The amount of this molecular motion is directly proportional to the temperature of the retina.", "question": "Which one of the following conclusions is most strongly supported by the information above?", "answers": "['Molecules of rhodopsin are the only pigment molecules that occur naturally in the retina.', 'The visual systems of animals whose body temperature matches that of their surroundings are more error-prone in hot surroundings than in cold ones.', 'As the temperature of the retina rises, rhodopsin molecules react more slowly to being struck by photons.', \"The temperature of an animal's retina depends on the amount of light the retina is absorbing.\"]", "label": 1 }, { "id": "train_147", "context": "Phoebe: There have been many reported sightings of strange glowing lights, but a number of these sightings have a straightforward, natural explanation. They occurred clustered in time and location around the epicenters of three earthquakes, and so were almost certainly earthquake lights, a form of ball lightning caused by stresses in the ground. Quincy: I am skeptical that the association between the lights and the earthquakes is anything more than a coincidence. The theory that ground stresses related to earthquakes can cause any kind of lightning is extremely speculative.", "question": "In responding to Phoebe, Quincy", "answers": "[\"offers an explanation of the glowing lights different from Phoebe's\", \"criticizes Phoebe's explanation as unsubstantiated\", 'accuses Phoebe of introducing irrelevant information', 'challenges the accuracy of the data about sightings that Phoebe takes for granted']", "label": 1 }, { "id": "train_148", "context": "Many workers who handled substance T in factories became seriously ill years later. We now know T caused at least some of their illnesses. Earlier ignorance of this connection does not absolve T' s manufacturer of all responsibility. For had it investigated the safety of T before allowing workers to be exposed to it, many of their illnesses would have been prevented.", "question": "Which one of the following principles most helps to justify the conclusion above?", "answers": "['Manufacturers should be held responsible for the consequences of any of their actions that harm innocent people if those consequences were preventable.', 'Manufacturers have an obligation to inform workers of health risks of which they are aware.', 'Employees who are harmed by substances they handle on the job should be compensated for medical costs they incur as a result.', 'Manufacturers should be held responsible only for the preventable consequences of their actions.']", "label": 0 }, { "id": "train_149", "context": "The quantity and type of pollution that entered the river last Thursday night suggest that the local auto repair shop is responsible. But the penalty for this type of pollution is so severe that, unless stronger evidence is discovered or the perpetrator admits responsibility, we cannot be sufficiently certain of the identity of the polluter to justify imposing the penalty.", "question": "Which one of the following principles, if valid, most helps to justify the reasoning in the argument?", "answers": "['The more severe the penalty for an infraction is, the more certain one must be of the guilt of a party before being justified in imposing the penalty on that party.', 'The more severe the penalty for an offense is, the less likely it is that someone will come forward and admit responsibility for the offense.', 'The severity of the penalty imposed for an infraction should be proportional to the harm caused by that infraction.', \"Penalties for crimes should be severe enough to deter people from committing them, but not so severe as to undermine one's willingness to impose them.\"]", "label": 0 }, { "id": "train_150", "context": "The striking similarities between Greek temples and subsequent Roman villas testify to the fact that great architects typically draw inspiration from the work of other architects. Such drawing of inspiration does not mean, however, a slavish lack of originality; building according to formulas does not make good buildings. When formulas are used, the results are either bland or disastrous. By contrast, a great architect creates fresh architectural compositions out of familiar forms and synthesizes past and present into a new, richer whole.", "question": "Which one of the following can be inferred from the passage?", "answers": "['Some Greek temples were not themselves inspired by older models.', 'Some Roman architecture exemplifies the creation of fresh architectural compositions out of familiar forms.', 'Buildings with unfamiliar forms are likely to be either bland or architectural disasters.', 'Roman architects designed many buildings with little or no originality of thought.']", "label": 1 }, { "id": "train_151", "context": "Letter to the editor: Your newspaper' s advertisement claims that you provide coverage of the high school' s most popular sports. Clearly this is false advertising. Of the school' s students, 15 percent compete on the track team, while only 5 percent of the students play basketball. Hence, track is far more popular than basketball, yet track gets no coverage and basketball gets full-page coverage.", "question": "The reasoning in the letter to the editor is most vulnerable to the criticism that it", "answers": "['bases its conclusion on a sample that is too small', 'employs as a premise the contention it purports to show', \"misinterprets a key word in the newspaper's advertisement\", 'criticizes the source of a claim rather than the claim itself']", "label": 2 }, { "id": "train_152", "context": "Stylistic evidence and laboratory evidence strongly support the claim that the magnificent painting Garden of Eden is a work of the Flemish master van Eyck. Nevertheless, the painting must have been the work of someone else, as any one with a little historical and zoological knowledge can tell merely by looking at the painting. The animals in the painting are all vivid representations of actual animals, including armadillos. Yet armadillos are native only to the Americas, and van Eyck died decades before Europeans reached the Americas.", "question": "In the argument given, the two highlighted portions play which of the following roles?", "answers": "['The first is a position that the argument seeks to reject; the second is evidence that the argument uses against that position.', 'The first is a judgment that serves as the basis for the main conclusion of the argument; the second states that main conclusion.', 'The first is an intermediate conclusion drawn in order to support a further conclusion stated in the argument; the second provides evidence in support of that intermediate conclusion.', 'The first presents the main conclusion of the argument; the second provides evidence in support of that conclusion.']", "label": 3 }, { "id": "train_153", "context": "Microbiologist: A lethal strain of salmonella recently showed up in a European country, causing an outbreak of illness that killed two people and infected twenty-seven others. Investigators blame the severity of the outbreak on the overuse of antibiotics, since the salmonella bacteria tested were shown to be drug-resistant. But this is unlikely because patients in the country where the outbreak occurred cannot obtain antibiotics to treat illness without a prescription, and the country's doctors prescribe antibiotics less readily than do doctors in any other European country.", "question": "Which of the following, if true, would most weaken the microbiologist's reasoning?", "answers": "['Salmonella has been shown to spread easily along the distribution chains of certain vegetables, such as raw tomatoes.', 'Use of antibiotics in two countries that neighbor the country where the outbreak occurred has risen over the past decade.', 'Physicians in the country where the outbreak occurred have become hesitant to prescribe antibiotics since they are frequently in short supply.', 'People in the country where the outbreak occurred often consume foods produced from animals that eat antibiotics-laden livestock feed.']", "label": 3 }, { "id": "train_154", "context": "Consumer advocate: Economists reason that price gouging -- increasing the price of goods when no alternative seller is available -- is efficient because it allocates goods to people whose willingness to pay more shows that they really need those goods. But willingness to pay is not proportional to need. In the real world, some people simply cannot pay as much as others. As a result, a price increase will allocate goods to the people with the most money, not to those with the most need.", "question": "Which one of the following most accurately describes the role played in the consumer advocate's argument by the claim that willingness to pay is not proportional to need?", "answers": "['It is the overall conclusion of the argument.', 'It denies a claim that the argument takes to be assumed in the reasoning that it rejects.', 'It is a general principle whose validity the argument questions.', 'It is a component of reasoning disputed in the argument.']", "label": 1 }, { "id": "train_155", "context": "Ten years ago, the Salisbury City Council passed the Culinary Bill, new legislation to protect the interests of local non-franchise restaurants. Of the 120 local non-franchise restaurants in Salisbury today, 85 opened during the last ten years. Clearly the Culinary Bill has caused a surge in the number of local non-franchise restaurants operating in Salisbury over the past ten years.", "question": "Which of the following is an assumption on which the argument depends?", "answers": "['All economic indicators suggest that household incomes in Salisbury have risen substantially over the past ten years.', 'The Culinary Bill provides no benefit to restaurants that are members of national franchise chains', 'Most of the consumers in Salisbury who patronize these local restaurants are aware of the provisions of the Culinary Bill and approve of them.', 'Of the local non-franchise restaurants in Salisbury ten years ago, fewer than 85 have closed.']", "label": 3 }, { "id": "train_156", "context": "Studies suggest that, for the vast majority of people who have normal blood pressure, any amount of sodium greater than that required by the body is simply excreted and does not significantly raise blood pressure. So only persons who have high blood pressure and whose bodies are incapable of safely processing excess sodium need to restrict their sodium intake.", "question": "Which one of the following, if true, would most seriously weaken the argument?", "answers": "['Every human being has a physiological need for at least some sodium.', 'Any sodium not used by the body will increase blood pressure unless it is excreted.', \"Excess sodium intake over time often destroys the body's ability to process excess sodium.\", 'High blood pressure is more harmful than was previously believed.']", "label": 2 }, { "id": "train_157", "context": "The solidity of bridge piers built on pilings depends largely on how deep the pilings are driven. Prior to 1700, pilings were driven to \"refusal, \" that is, to the point at which they refused to go any deeper. In a 1588 inquiry into the solidity of piers for Venice' s Rialto Bridge, it was determined that the bridge' s builder, Antonio Da Ponte, had met the contemporary standard for refusal: he had caused the pilings to be driven until additional penetration into the ground was no greater than two inches after twenty-four hammer blows.", "question": "Which one of the following can properly be inferred from the passage?", "answers": "['After 1588, no bridges were built on pilings that were driven to the point of refusal.', \"Da Ponte's standard of refusal was less strict than that of other bridge builders of his day.\", 'The Rialto Bridge was built on unsafe pilings.', 'It is possible that the pilings of the Rialto Bridge could have been driven deeper even after the standard of refusal had been met.']", "label": 3 }, { "id": "train_158", "context": "In view of the considerable length of the police chief' s tenure as head of the department, the chief should be held accountable for the widespread corruption in the department. That no evidence was discovered that the chief was involved in any way in the corruption does not allow us to escape this conclusion.", "question": "Which one of the following principles, if valid, most helps to justify the argument?", "answers": "['If corruption is rampant among a certain group, then the person in charge cannot be expected to take corrective action if that person has not been in charge for very long.', 'A supervisor should not be held accountable for widespread corruption among his or her subordinates unless the supervisor could reasonably be expected to know about that corruption.', 'Any supervisor who knowingly tolerates widespread corruption among his or her subordinates should be held accountable .', 'If a person has been in a position of authority for a long time, then there is no excuse that can absolve the person of responsibility for widespread corruption among subordinates.']", "label": 3 }, { "id": "train_159", "context": "In the past, combining children of different ages in one classroom was usually a failure; it resulted in confused younger children, who were given inadequate attention and instruction, and bored older ones, who had to sit through previously learned lessons. Recently, however, the practice has been revived with excellent results. Mixed-age classrooms today are stimulating to older children and enable younger children to learn much more efficiently than in standard classrooms.", "question": "Which one of the following, if true, most helps to resolve the apparent discrepancy in the passage?", "answers": "[\"Today's mixed-age classrooms, unlike those of the past, emphasize group projects that are engaging to students of different ages.\", 'On average, mixed-age classrooms today are somewhat larger in enrollment than were the ones of the past.', 'Few of the teachers who are reviving mixed- age classrooms today were students in mixed-age classrooms when they were young.', 'Mixed-age classrooms of the past were better equipped than are those of today.']", "label": 0 }, { "id": "train_160", "context": "A recent study showed that people who address problems quickly and directly are significantly less likely to have gum disease than are people who react to problems by refusing to think about them. Since stress can have a negative effect on the immune system, the study' s results clearly indicate that some forms of gum disease are caused or aggravated by suppression of the immune system.", "question": "The argument requires the assumption that", "answers": "['people who tend to address problems quickly and directly will invariably seek dental care at the first sign of problems', \"painful conditions will interfere with a person's ability to address problems quickly and directly\", 'people who have highly stressful lives tend to address problems quickly and directly', \"refusing to think about something troubling contributes to a person's level of stress\"]", "label": 3 }, { "id": "train_161", "context": "Blogger: Traditionally, newspapers have taken objectivity to be an essential of good journalism . However, today' s newer media are more inclined to try to create a stir with openly partisan reporting. This contrast in journalistic standards is best understood in terms of differing business strategies. The newer media outlets need to differentiate themselves in a crowded marketplace. The standard of objectivity developed primarily among newspapers with no serious rivals, so the most important objective was to avoid offending potential readers.", "question": "Which one of the following is an assumption required by the blogger's argument?", "answers": "['People prefer objective reporting to partisan reporting that merely reinforces their own partisan leanings.', 'Journalists at traditional newspapers are just as partisan as journalists who work for newer media outlets.', 'Newspapers have regarded objective reporting as less likely to offend people than openly partisan reporting.', 'The newer media outlets are increasing in popularity at the expense of traditional newspapers.']", "label": 2 }, { "id": "train_162", "context": "Pundit: Grenier will almost certainly not be elected as mayor. Although she says she believes in raising city employees' wages, it was only a few years ago that she was arguing that their wages should not be increased. Her claim that she has learned more about the issue since then -- though sincere -- will not matter to most voters, who will see her as insincere.", "question": "Which one of the following principles, if valid, most helps to justify the pundit's reasoning?", "answers": "[\"Voters are unlikely to be influenced by what a politician's views were in the past if the voters agree with the politician's current positions.\", 'Voters are likely to elect a politician who they believe understands their financial concerns.', 'Voters are likely to question the sincerity of a politician who does not hold the same beliefs they do.', 'Voters are unlikely to vote for a politician whom they perceive to be insincere.']", "label": 3 }, { "id": "train_163", "context": "One of the most important events for modern astronomy was the series of collisions, during a single week in 1994, of more than a dozen large objects with Jupiter. The collision of these objects, which once formed most of the comet Shoemaker-Levy 9, resulted in formations that showed no signs of water. There was thus no water involved in the collisions, so we know that none of the comet' s fragments penetrated to Jupiter' s lower atmosphere and that the comet was composed of rock rather than ice.", "question": "Which one of the following is an assumption on which the argument depends?", "answers": "['Comets tend to be composed largely of ice while asteroids are composed mainly of rock.', 'The post-collision analysis of Jupiter showed that the formations all had exactly the same composition.', \"The deeper the explosion occurred in Jupiter's atmosphere, the more difficult it would be to detect from Earth.\", \"If Jupiter's lower atmosphere had been penetrated by the comet's fragments, the resulting formations would show signs of water.\"]", "label": 3 }, { "id": "train_164", "context": "To perform an act that is morally wrong is to offend against humanity, and all offenses against humanity are equally bad. Because murder is morally wrong, it is just as bad to have murdered one person by setting off a bomb as it would have been to have murdered a hundred people by setting off that bond.", "question": "Which one of the following judgments conforms to the principles invoked above?", "answers": "['If lying is morally wrong, telling a lie is as bad as murdering someone.', \"Risking one's life to save the lives of a hundred people is morally no better than risking one's life to save one person.\", 'If stealing is morally wrong, it is equally important to society to prevent people from stealing as it is to prevent them from committing murder.', 'Accidentally causing the death of a person is just as bad as murdering that person.']", "label": 0 }, { "id": "train_165", "context": "Many important types of medicine have been developed from substances discovered in plants that grow only in tropical rain forests. There are thousands of plant species in these rain forests that have not yet been studied by scientists, and it is very likely that many such plants also contain substances of medicinal value. Thus, if the tropical rain forests are not preserved, important types of medicine will never be developed.", "question": "Which one of the following is an assumption required by the argument?", "answers": "['Most of the tropical rain forest plants that contain substances of medicinal value can also be found growing in other types of environment.', 'The tropical rain forests should be preserved to make it possible for important medicines to be developed from plant species that have not yet been studied by scientists.', 'Any substance of medicinal value contained in plant species indigenous to tropical rain forests will eventually be discovered if those species are studied by scientists.', 'There are substances of medicinal value contained in tropical rain forest plants not yet studied by scientists that differ from those substances already discovered in tropical rain forest plants.']", "label": 3 }, { "id": "train_166", "context": "Journalist: The trade union members at AutoFaber Inc. are planning to go on strike. Independent arbitration would avert a strike, but only if both sides agree to accept the arbitrator' s recommendations as binding. However, based on past experience, the union is quite unlikely to agree to this, so a strike is likely.", "question": "Which one of the following arguments exhibits a pattern of reasoning most similar to that exhibited by the journalist's argument?", "answers": "[\"Lopez will run in tomorrow's marathon. Lopez will win the marathon only if his sponsors do a good job of keeping him hydrated. But his sponsors are known to be poor at keeping their athletes hydrated. So it is probable that Lopez will not win the marathon.\", \"Rodriguez will donate her paintings to the museum only if the new wing is named after her. The only other person the new wing could be named after is the museum's founder, Wu. But it was decided yesterday that the gardens, not the new wing, would be named after Wu. So Rodriguez will donate her paintings to the museum.\", 'The company will downsize unless more stock is issued. Furthermore, if the company downsizes, the shareholders will demand a change. Since no more stock is being issued, we can be sure that the shareholders will demand a change.', 'The new course in microeconomics is offered either in the fall or in the spring. The new course will be offered in the spring if there is a qualified instructor available. Since the economics department currently lacks a qualified instructor for such courses, however, the course will not be offered in the spring.']", "label": 0 }, { "id": "train_167", "context": "Up until about 2 billion years ago, the sun was 30 percent dimmer than it is now. If the sun were that dim now, our oceans would be completely frozen. According to fossil evidence, however, life and liquid water were both present as early as 3. 8 billion years ago.", "question": "Which one of the following, if true, most helps to resolve the apparent discrepancy described above?", "answers": "['Evidence suggests that certain regions of ocean remained frozen until much more recently than 2 billion years ago.', 'Our atmosphere currently holds in significantly less heat than it did 3. 8 billion years ago.', \"When large portions of the globe are ice-covered, more of the sun's heat is reflected and not absorbed by the earth than when only the poles are ice-covered.\", 'The liquid water present 3. 8 billion years ago later froze, only to melt again about 2 billion years ago.']", "label": 1 }, { "id": "train_168", "context": "If one wants to succeed, then one should act as though one were genuinely confident about one' s abilities, even if one actually distrusts one' s skills. Success is much more easily obtained by those who genuinely believe themselves capable of succeeding than by those filled with self-doubts.", "question": "Which one of the following statements, if true, most strengthens the argument?", "answers": "['Success is usually more a matter of luck or determination than of skill.', 'Self-doubt can hamper as well as aid the development of the skills necessary for success.', 'Genuine confidence is often a by-product of pretended self-confidence.', 'Those who convince others that they are capable of succeeding usually have few self-doubts.']", "label": 2 }, { "id": "train_169", "context": "In polluted industrial English cities during the Industrial Revolution, two plant diseases -- black spot, which infects roses, and tar spot, which infects sycamore trees -- disappeared. It is likely that air pollution eradicated these diseases.", "question": "Which one of the following, if true, most strengthens the reasoning above?", "answers": "['Scientists theorize that some plants can develop a resistance to air pollution.', 'Black spot and tar spot returned when the air in the cities became less polluted.', 'For many plant species, scientists have not determined the effects of air pollution.', 'Black spot and tar spot were the only plant diseases that disappeared in any English cities during the Industrial Revolution.']", "label": 1 }, { "id": "train_170", "context": "After 1950, in response to record growth in worldwide food demand, farmers worldwide sharply increased fertilizer use. As a result, the productivity of farmland more than doubled by 1985. Since 1985, farmers have sought to increase farmland productivity even further. Nevertheless, worldwide fertilizer use has declined by 6 percent between 1985 and the present.", "question": "Which one of the following, if true, most helps to resolve the apparent discrepancy in the information above?", "answers": "['After fertilizer has been added to soil for several years, adding fertilizer to the soil in subsequent years does not significantly improve crop production.', 'Several varieties of crop plants that have become popular recently, such as soybeans, are as responsive to fertilizer as are traditional grain crops.', 'Between 1950 and 1985 farmers were able to increase the yield of many varieties of crop plants.', 'Between 1975 and 1980 fertilizer prices temporarily increased because of labor disputes in several fertilizer-exporting nations, and these disputes disrupted worldwide fertilizer production.']", "label": 0 }, { "id": "train_171", "context": "Chronic fatigue syndrome is characterized by prolonged fatigue, muscular pain, and neurological problems. It is not known whether these symptoms are all caused by a single virus or whether each symptom is the result of a separate viral infection. A newly synthesized drug has been tested on those who suffer from chronic fatigue syndrome. Although the specific antiviral effects of this drug are unknown, it has lessened the severity of all of the symptoms of chronic fatigue syndrome. Thus there is evidence that chronic fatigue syndrome is, in fact, caused by one virus.", "question": "The argument assumes which one of the following?", "answers": "['An antiviral medication that eliminates the most severe symptoms of chronic fatigue syndrome thereby cures chronic fatigue syndrome.', 'It is more likely that the new drug counteracts one virus than that it counteracts several viruses.', 'Most syndromes that are characterized by related symptoms are each caused by a single viral infection.', 'All those who suffer from prolonged fatigue also suffer from neurological problems.']", "label": 1 }, { "id": "train_172", "context": "Ecologist: The Scottish Highlands were once the site of extensive forests, but these forests have mostly disappeared and been replaced by peat bogs. The common view is that the Highlands' deforestation was caused by human activity, especially agriculture. However, agriculture began in the Highlands less than 2, 000 years ago . Peat bogs, which consist of compressed decayed vegetable matter, build up by only about one foot per 1, 000 years and, throughout the Highlands, remains of trees in peat bogs are almost all at depths great than four feet . Since climate changes that occurred between 7, 000 and 4, 000 years ago favored the development of peat bogs rather than the survival of forests, the deforestation was more likely the result of natural processes than of human activity.", "question": "In the ecologist's argument the two portions in boldface play which of the following roles?", "answers": "['The first is a position that the ecologist rejects; the second provides evidence in support of that rejection.', \"The first is evidence that, in light of the evidence provided in the second, serves as grounds for the ecologist's rejection of a certain position.\", 'The first is a position for which the ecologist argues; the second provides evidence to support that position.', 'The first is evidence that has been used in support of a position that the ecologist rejects; the second is a finding that the ecologist uses to counter that evidence.']", "label": 1 }, { "id": "train_173", "context": "Rita: No matter how you look at them, your survey results are misleading. Since people generally lie on such surveys, the numbers you collected are serious underestimates. Hiro: I have no doubt that people lie on surveys of this type. The question is whether some people lie more than others. While the raw numbers surely underestimate what I' m trying to measure, the relative rates those numbers represent are probably close to being accurate.", "question": "Rita and Hiro disagree over whether", "answers": "['the survey results are misleading regardless of how they are interpreted', 'people tend to lie on certain kinds of surveys', 'the raw numbers collected are serious underestimates', 'a different type of measure than a survey would produce results that are less misleading']", "label": 0 }, { "id": "train_174", "context": "A number of Grandville' s wealthiest citizens have been criminals. So, since it is of utmost importance that the Grandville Planning Committee be composed solely of individuals whose personal standards of ethics are beyond reproach, no wealthy person should be appointed to that committee.", "question": "The argument is most vulnerable to the criticism that it", "answers": "['mistakes a temporal relationship for a causal relationship', 'judges only by subjective standards something that can be readily evaluated according to objective standards', 'confuses a result with something that is sufficient for bringing about that result', 'generalizes on the basis of what could be exceptional cases']", "label": 3 }, { "id": "train_175", "context": "Scientists conjecture that certain microbes consume organic molecules in exposed shale and similar sediments. In so doing, the microbes remove oxygen from the atmosphere and generate carbon dioxide, a gas that, evidence indicates, promotes global warming. They also conjecture that these microbes reproduce more quickly at higher temperatures.", "question": "The scientists' conjectures, if true, provide the most support for which one of the following statements?", "answers": "['Every organism that generates carbon dioxide reproduces more quickly at high temperatures.', \"The microbes' activity will soon diminish as the organic molecules in exposed sediments are depleted.\", 'If global warming occurs, it will be exacerbated by the activity of the microbes.', 'The microbes do not remove any element other than oxygen from the atmosphere.']", "label": 2 }, { "id": "train_176", "context": "Whittaker: There can be no such thing as the number of medical school students who drop out before their second year, because if they drop out, they never have a second year. Hudson: By your reasoning I cannot help but become rich, because there is similarly no such thing as my dying before my first million dollars is in the bank.", "question": "Hudson responds to Whittaker by", "answers": "[\"showing that Whittaker's argument relies on analyzing an extreme and unrepresentative case\", 'showing that a relevantly analogous argument leads to an untenable conclusion', \"citing a specific example to counter Whittaker's general claim\", 'claiming that what Whittaker says cannot be true because Whittaker acts as if it were false']", "label": 1 }, { "id": "train_177", "context": "Some people believe there is intelligent life somewhere in the universe besides Earth. But no one who believes that could also believe all planets other than Earth are devoid of life. Thus, some people do not believe all planets other than Earth are devoid of life.", "question": "The reasoning in which one of the following is most similar to the reasoning in the argument above?", "answers": "['Some dog owners believe their dogs are emotionally attached to them. Since no one could believe that and also believe dogs are incapable of thought, it follows that some dog owners do not believe dogs are incapable of thought.', 'Some people believe there is life on planets other than Earth. Since anyone who believes that must also believe other planets have atmospheres that can support life, it follows that other planets do have atmospheres that can support life.', 'Some cat owners believe cats are incapable of feeling guilty. Since one could think that an animal is capable of feeling guilty only if one thought the animal had a conception of morality, it follows that some cat owners do not believe cats have a conception of morality.', 'Some people believe computers are capable of thought. Since no one who does not believe computer chess programs sometimes beat human chess masters believes computers can think, it follows that computer chess programs do sometimes beat human chess masters.']", "label": 0 }, { "id": "train_178", "context": "If a civilization as technologically advanced as human civilization existed on another planet and that planet were within 50 light years of Earth, that civilization would have found evidence of intelligent life on Earth and could have easily contacted us. Scientists can thus rule out the possibility of finding a civilization as technologically advanced as our own within 50 light years of Earth.", "question": "Which one of the following is an assumption required by the argument?", "answers": "['There is no reason to doubt the possibility that there are technologically advanced civilizations on planets more than 50 light years from Earth.', 'A technologically advanced civilization on another planet would want to communicate with intelligent life that it detected on Earth.', 'Intelligent life forms on other planets would be able to recognize all signs of intelligent life on Earth.', 'Scientists who are searching for evidence of extraterrestrial life forms generally focus their search on evidence of technologically advanced life forms.']", "label": 1 }, { "id": "train_179", "context": "Trainer: I recently developed an exercise routine that can get anybody to meet his or her goals. The routine combines cardio and bodybuilding during each session for the purpose of losing weight. Every person I' ve trained has lost weight on the program.", "question": "The strength of the argument depends on which one of the following?", "answers": "['Every client the trainer has worked with has weight loss as a goal.', 'Every client the trainer has worked with has prior experience lifting weights.', 'Every client the trainer has worked with has also adopted a healthy diet.', 'Losing weight is always a healthy outcome.']", "label": 0 }, { "id": "train_180", "context": "Faden: Most of our exercise machines are still in use after one year. A recent survey of our customers shows this. Greenwall: But many of those customers could easily be lying because they are too embarrassed to admit that they don' t exercise anymore. Faden: You have no way of showing that customers were lying. Your objection is absurd.", "question": "Which one of the following most accurately describes a flaw in the reasoning above?", "answers": "['Faden presumes, without providing justification, that the more conclusive the evidence is for a claim, the less believable the claim becomes.', 'Faden presumes, without providing justification, that the evidence for a claim has not been undermined unless that evidence has been proven false.', 'Greenwall ignores the possibility that some people stopped using the equipment but were not embarrassed about it.', 'Greenwall presumes, without giving justification, that most people are dishonest about their personal habits.']", "label": 1 }, { "id": "train_181", "context": "Ancient humans in eastern North America hunted mammoths until the mammoth disappeared from the area around 13, 000 years ago. Recently, a fossil bone with an engraving that depicts a mammoth was found in an ancient settlement in eastern North America. This shows that the settlement was occupied at a time when mammoths lived in this area.", "question": "The argument requires the assumption that", "answers": "['the fossil on which the engraving was made was not a mammoth bone', 'when mammoths disappeared from eastern North America, there were no mammoths left anywhere in North America', 'there is no scientific way of dating when the engraving of the mammoth was made', 'the engraving was made during the time when the settlement was occupied']", "label": 3 }, { "id": "train_182", "context": "The difference in average annual income in favor of employees who have college degrees, compared with those who do not have such degrees, doubled between 1980 and 1990. Some analysts have hypothesized that increased competition between employers for employees with college degrees drove up income for such employees.", "question": "Which of the following, if true, most seriously undermines the explanation described above?", "answers": "['The unemployment rate changed very little throughout the 1980s', 'During the 1980s there were some employees with no college degree who earned incomes comparable to the top incomes earned by employees with a college degree.', 'During the 1980s a growing percentage of college graduates, unable to find jobs requiring a college degree, took unskilled jobs.', 'The average age of all employees increased slightly during the 1980s.']", "label": 2 }, { "id": "train_183", "context": "Editorial: It is clear that if this country' s universities were living up to both their moral and their intellectual responsibilities, the best-selling publications in most university bookstores would not be frivolous ones like TV Today and Gossip Review. However, in most university bookstores the only publication that sells better than Gossip Review is TV Today.", "question": "If the statements in the editorial are true, which one of the following must also be true on the basis of them?", "answers": "[\"At least some of this country's universities are not meeting their moral responsibilities or their intellectual responsibilities or both.\", 'People who purchase publications that are devoted primarily to gossip or to television programming are intellectually irresponsible.', \"Many people who attend this country's universities fail to live up to both their moral and their intellectual responsibilities.\", 'It is irresponsible for university bookstores to carry publications such as Gossip Review and TV Today.']", "label": 0 }, { "id": "train_184", "context": "Medical research has established that the Beta Diet is healthier than a more conventional diet. But on average, people who have followed the Beta Diet for several decades are much more likely to be in poor health than are people whose diet is more conventional.", "question": "Which one of the following, if true, most helps to resolve the apparent conflict between the two statements above?", "answers": "['Recent research has shown that a diet high in fruits, vegetables, and skim milk is even healthier than the Beta Diet.', 'The Beta Diet is used primarily as a treatment for a condition that adversely affects overall health.', 'The Beta Diet provides dramatic health benefits for some people but only minor benefits for others.', 'On average, people who have followed the Beta Diet for their entire lives are much more likely to have a variety of healthful habits than are people whose diet is more conventional.']", "label": 1 }, { "id": "train_185", "context": "The ancient Romans understood the principles of water power very well, and in some outlying parts of their empire they made extensive and excellent use of water as an energy source. This makes it all the more striking that the Romans made do without water power in regions dominated by large cities.", "question": "Which one of the following, if true, contributes most to an explanation of the difference described above in the Romans' use of water power?", "answers": "['In heavily populated areas the introduction of water power would have been certain to cause social unrest by depriving large numbers of people of their livelihood.', 'In the areas in which water power was not used, water flow in rivers and streams was substantial throughout the year but nevertheless exhibited some seasonal variation.', 'In most areas to which the use of water power was not extended, other, more traditional sources of energy continued to be used.', 'The ancient Romans were adept at constructing and maintaining aqueducts that could carry quantities of water sufficient to supply large cities over considerable distances.']", "label": 0 }, { "id": "train_186", "context": "One good clue as to which geographical regions an ancient relic was moved through in the past involves the analysis of pollen that clings to the surface of the relic. A relic is linked to a geographical area by the identification of pollen from plants that are known to have been unique to that area.", "question": "Which one of the following, if true, casts the most doubt on the reliability of the method described above?", "answers": "['Data are scarce as to the geographical distribution of the pollens of many ancient plants.', 'Pollens are often transported from one region to another by wind or human movement.', 'Many types of pollen were common to several geographical regions in the ancient world.', 'Pollen analysis is a painstaking process that is also expensive to conduct.']", "label": 1 }, { "id": "train_187", "context": "If cold fusion worked, it would provide almost limitless power from very inexpensive raw materials, materials far cheaper than coal or oil. But replacing all the traditional electric generators that use these fuels with cold-fusion power plants would result in a reduction of no more than 25 percent in the average residential electric bill.", "question": "Each of the following, if true, would help to resolve the apparent discrepancy between the predictions above EXCEPT:", "answers": "['Personnel costs for the distribution of power to consumers are unrelated to the type of raw materials an electric company uses.', 'Most electric companies would be willing to incorporate cold-fusion technology into their power plants.', 'Cold-fusion power plants would be more expensive to build and maintain than traditional electric generators are.', \"Only a relatively small portion of any residential electric bill is determined by the electric company's expenses for fuel.\"]", "label": 1 }, { "id": "train_188", "context": "In a few recent cases, some teenagers with advanced programming abilities used a new programming language, FANTOD, to hack into ETS and change their own SAT scores. All of the teenagers convicted of this crime were highly skilled in programming FANTOD. In light of these cases, some colleges have discounted the official SAT scores of applicants with a knowledge of FANTOD, and have required them to take special admission tests in supervised conditions on their own campuses.", "question": "Which of following conclusions can most properly be drawn from the information above?", "answers": "['Students who learn FANTOD tend to have much lower SAT scores than do their peers.', 'Most people who learn to program in FANTOD do so to commit some kind of hacking.', 'The college admissions process possibly places some students with a knowledge of FANTOD at a disadvantage.', 'Colleges should rely on their own admissions tests instead of the SATs']", "label": 2 }, { "id": "train_189", "context": "Orthodox medicine is ineffective at both ends of the spectrum of ailments. At the more trivial end, orthodox medicine is largely ineffective in treating aches, pains, and allergies, and, at the other extreme, it has yet to produce a cure for serious, life-threatening diseases such as advanced cancer and lupus. People turn to alternative medicine when orthodox medicine fails to help them and when it produces side effects that are unacceptable to them. One of the reasons alternative medicine is free of such side effects is that it does not have any effects at all.", "question": "If the statements above are true, which one of the following can be properly inferred from them?", "answers": "['Orthodox medicine will eventually produce a solution for the diseases that are currently incurable.', 'There are some medical conditions for which no orthodox or alternative treatment is effective.', 'There are some trivial illnesses that can be treated effectively by the methods of alternative medicine.', 'Practitioners of alternative medicine are acting in bad faith.']", "label": 1 }, { "id": "train_190", "context": "Direct democracy is the best system of government for every society. No other system of government maximizes individual freedom more than democracy. In direct democracies, the people' s will is manifested in the state' s policies, as they can directly vote on every political issue. All of the politicians who killed the most people, like Adolf Hitler and Joseph Stalin, led dictatorships. Direct democracy is obviously better than a dictatorship, so it is the best form of government.", "question": "Which one of the following most accurately describes how the argument proceeds?", "answers": "['The argument leads with a generalization, offers some examples, and then finishes with a conclusion.', 'The argument offers several premises and then creates a false dichotomy to support its conclusion.', 'The argument leads with a conclusion, offers several premises, and then sets up a false dichotomy to support its conclusion.', 'The argument leads with a conclusion then provides several illustrative examples.']", "label": 2 }, { "id": "train_191", "context": "A medical journal used a questionnaire survey to determine whether a particular change in its format would increase its readership. Sixty-two percent of those who returned the questionnaire supported that change. On the basis of this outcome, the decision was made to introduce the new format.", "question": "Which one of the following, if it were determined to be true, would provide the best evidence that the journal's decision will have the desired effect?", "answers": "['Of the readers who received questionnaires, 90 percent returned them.', 'The percentage of surveyed readers who like the format change was almost the same as the percentage of the entire potential readership who would like the format change.', 'Ninety percent of the readers who were dissatisfied with the old format and only 50 percent of the readers who liked the old format returned their questionnaires.', 'It was determined that the new format would be less costly than the old format.']", "label": 1 }, { "id": "train_192", "context": "Dr. Theresa Pagano, a biologist, has found that the checkerspot butterfly is becoming more prevalent in regions farther north than before and less prevalent in regions farther south. The northward shift of the butterflies is almost perfectly correlated with the northward shift of the warm zones in the global climate, and Dr. Pagano has ttherefore concluded that the changing climate is responsible for the northward movement of the butterflies.", "question": "Each of the following, if true, supports Dr. Pagano's reasoning EXCEPT:", "answers": "['Experimental evidence suggests that the checkerspot butterfly can adapt easily to a wide range of temperatures and geographic conditions.', 'Climate does not affect checkerspot butterflies themselves directly, but the plants they depend on thrive best in warm climates.', 'Several studies have shown that several other species of butterfly closely related to the checkerspot butterfly survive only in warm climates.', 'Checkerspot butterfly colonies observed under laboratory conditions are critically affected by small temperature changes.']", "label": 0 }, { "id": "train_193", "context": "Criminologist: The main purpose of most criminal organizations is to generate profits. The ongoing revolutions in biotechnology and information technology promise to generate enormous profits. Ttherefore, criminal organizations will undoubtedly try to become increasingly involved in these areas.", "question": "The conclusion of the criminologist's argument is properly inferred if which one of the following is assumed?", "answers": "['Criminal organizations are already heavily involved in every activity that promises to generate enormous profits.', 'At least some criminal organizations are or will at some point become aware that the ongoing revolutions in biotechnology and information technology promise to generate enormous profits.', 'Any organization whose main purpose is to generate profits will try to become increasingly involved in any technological revolution that promises to generate enormous profits.', 'Most criminal organizations are willing to become involved in legal activities if those activities are sufficiently profitable.']", "label": 2 }, { "id": "train_194", "context": "Why would anyone want to make nonprofits as inefficient as Byworks Corporation, which has posted huge losses for years? The reasoning of the argument in the letter is most vulnerable to criticism on the grounds that the argument", "question": "Letter to the editor: You have asserted that philanthropists want to make the nonprofit sector as efficient as private business in this country. Philanthropists want no such thing, of course.", "answers": "[\"concludes that a phenomenon will have a certain property merely because the phenomenon's cause has that property\", 'rejects a claim by attacking the proponent of the claim rather than addressing the claim itself', 'draws a conclusion about what ought to be the case from premises that are entirely about what is the case', 'takes the condition of one member of a category to be representative of the category in general']", "label": 3 }, { "id": "train_195", "context": "In a scene in an ancient Greek play, Knights, the character Demosthenes opens a writing tablet on which an oracle had written a prophecy, and while looking at the tablet, he continuously expresses his amazement at its contents. His companion presses him for information, whereupon Demosthenes explains what the oracle had written.", "question": "Of the following claims, which one can most justifiably be rejected on the basis of the statements above?", "answers": "['The character of Demosthenes in Knights is not based on a historical figure.', 'Only rarely in ancient Greece were prophecies written down on writing tablets.', 'In ancient Greece, people did not read silently to themselves.', 'In ancient Greek plays, the reading aloud of written texts commonly occurred as part of the on-stage action.']", "label": 2 }, { "id": "train_196", "context": "Teacher to a student: You agree that it is bad to break promises. But when we speak to each other we all make an implicit promise to tell the truth, and lying is the breaking of that promise. So even if you promised Jeanne that you would tell me she is home sick, you should not tell me that, if you know that she is well.", "question": "Which one of the following is an assumption on which the teacher's argument depends?", "answers": "['One should never break a promise.', \"It is sometimes better to act in a friend's best interests than to keep a promise to that friend.\", 'Some implicit promises are worse to break than some explicit ones.', 'Breaking a promise leads to worse consequences than does telling a lie.']", "label": 2 }, { "id": "train_197", "context": "People often admonish us to learn the lessons of history, but, even if it were easy to discover what the past was really like, it is nearly impossible to discover its lessons. We are supposed, for example, to learn the lessons of World War I. But what are they? And were we ever to discover what they are, it is not clear that we could ever apply them, for we shall never again have a situation just like World War I.", "question": "That we should learn the lessons of history figures in the argument in which one of the following ways?", "answers": "[\"It is compatible with accepting the argument's conclusion and with denying it.\", 'It is a position that the argument simply takes for granted is false.', \"It is an assumption that is required in order to establish the argument's conclusion.\", 'It expresses the position the argument as a whole is directed toward discrediting.']", "label": 3 }, { "id": "train_198", "context": "In a study of heart patients awaiting treatment for reduced blood flow to the heart, those still waiting to find out whether they would need surgery were less likely to experience pain from the condition than were those who knew what type of treatment they would receive. Assuming that this uncertainty is more stressful than knowing what one' s future holds, then it is reasonable to conclude that__.", "question": "Which one of the following most logically completes the argument?", "answers": "['stress sometimes reduces the amount of pain a heart patient experiences', 'heart patients suffering from reduced blood flow to the heart who are experiencing pain from the condition are more likely to require surgery than are such patients who are not experiencing pain', \"the severity of a heart patient's condition is usually worsened by withholding information from the patient about the treatment that that patient will receive\", 'stress is probably an effect rather than a cause of reduced blood flow to the heart']", "label": 0 }, { "id": "train_199", "context": "Various mid-fourteenth-century European writers show an interest in games, but no writer of this period mentions the playing of cards. Nor do any of the mid-fourteenth-century statutes that proscribe or limit the play of games mention cards, though they do mention dice, chess, and other games. It is ttherefore likely that, contrary to what is sometimes claimed, at that time playing cards was not yet common in Europe.", "question": "The pattern of reasoning in which one of the following is most similar to that in the argument above?", "answers": [ "This evening's television news reported that the cruise ship was only damaged in the fire last night, whereas the newspaper reported that it was destroyed. The television news is based on more recent information, so probably the ship was not destroyed.", "The newspaper is generally more reliable than the television news, and the newspaper reported that the damage from last night's fire in the port was not severe. Ttherefore, the damage probably was not severe.", "Among the buildings that are near the port is the newspaper's printing plant. Early editions of this morning's paper were very late. Ttherefore, the fire at the port probably affected areas beyond the port itself.", "Neither today's newspapers nor this evening's television news mentioned a huge fire that was rumored to have happened in the port last night. Ttherefore, there probably was no such fire." ], "label": 3 }, { "id": "train_200", "context": "Sociologist: Marriage is one of the most important societal institutions. Marital relationship provides numerous structural benefits for married couples and their offspring. Studies consistently show that children born out of wedlock are less likely to attend college and more likely to work low-paying jobs. Additionally, married people are more likely to be homeowners and save for retirement. Ttherefore, if marriage rates decline, __.", "question": "Which one of the following most logically completes the sociologist's argument?", "answers": "['Nobody would own homes.', 'College attendance would probably decline.', 'People would be happier.', 'Society will collapse.']", "label": 1 }, { "id": "train_201", "context": "Books that present a utopian future in which the inequities and sufferings of the present are replaced by more harmonious and rational social arrangements will always find enthusiastic buyers. Since gloomy books predicting that even more terrifying times await us are clearly not of this genre, they are unlikely to be very popular.", "question": "The questionable pattern of reasoning in which one of the following arguments is most similar to that in the argument above?", "answers": "['Action movies that involve complicated and dangerous special-effects scenes are enormously expensive to produce. Hence, since traditional dramatic or comedic films contain no such scenes, it is probable that they are relatively inexpensive to produce.', 'Art that portrays people as happy and contented has a tranquilizing effect on the viewer, an effect that is appealing to those who are tense or anxious. Thus, people who dislike such art are neither tense nor anxious.', \"People who enjoy participating in activities such as fishing or hiking may nevertheless enjoy watching such spectator sports as boxing or football. Thus, one cannot infer from someone's participating in vigorous contact sports that he or she is not also fond of less violent forms of recreation.\", 'All self-employed businesspeople have salaries that fluctuate with the fortunes of the general economy, but government bureaucrats are not self-employed. Ttherefore, not everyone with an income that fluctuates with the fortunes of the general economy is a government bureaucrat.']", "label": 0 }, { "id": "train_202", "context": "Science writer: The deterioration of cognitive faculties associated with Alzheimer' s disease is evidently caused by the activities of microglia -- the brain' s own immune cells. For one thing, this deterioration can be slowed by some anti-inflammatory drugs, such as acetylsalicylic acid. Furthermore, patients with Alzheimer' s are unable to eliminate the protein BA from the brain, where it accumulates and forms deposits. The microglia attack these protein deposits by releasing poisons that destroy surrounding healthy brain cells, thereby impairing the brain' s cognitive functions.", "question": "Which one of the following, if true, most helps to support the science writer's argument?", "answers": "['Acetylsalicylic acid reduces the production of immune cells in the brain.', \"Immune reactions by microglia occur in certain diseases of the brain other than Alzheimer's.\", \"The inability of Alzheimer's patients to eliminate the protein BA from the brain is due to a deficiency in the brain's immune system.\", 'The protein BA directly interferes with the cognitive functions of the brain.']", "label": 0 }, { "id": "train_203", "context": "Sarah, who is an excellent mechanic, said that in her opinion the used car John is considering is in good mechanical condition. However, it is clear that Sarah cannot be trusted to give an honest opinion, since when Emmett asked her opinion of his new haircut she lied and said she thought it looked good. Ttherefore, it is very likely that Sarah also lied in giving her opinion of the mechanical condition of that car.", "question": "The argument is flawed by virtue of having committed which one of the following errors of reasoning?", "answers": "['It bases a sweeping claim on the evidence provided by an instance that is not clearly relevant.', 'It presents evidence in value-laden terms that presuppose the conclusion for which that evidence is offered.', 'It fails to offer any grounds for the attack it makes on the character of the person.', 'It wrongly assumes that because someone is a competent judge of one kind of thing, that person will be a competent judge of a very different kind of thing.']", "label": 0 }, { "id": "train_204", "context": "Businesses are suffering because of a lack of money available for development loans. To help businesses, the government plans to modify the income-tax structure in order to induce individual taxpayers to put a larger portion of their incomes into retirement savings accounts, because as more money is deposited in such accounts, more money becomes available to borrowers.", "question": "Which of the following, if true, raises the most serious doubt regarding the effectiveness of the government's plan to increase the amount of money available for development loans for businesses?", "answers": "['When levels of personal retirement savings increase, consumer borrowing always increases correspondingly.', 'Bankers generally will not continue to lend money to businesses whose prospective earnings are insufficient to meet their loan repayment schedules.', 'Even with tax incentives, some people will choose not to increase their levels of retirement savings.', 'The modified tax structure would give all taxpayers, regardless of their incomes, the same tax savings for a given increase in their retirement savings.']", "label": 0 }, { "id": "train_205", "context": "Tallulah: The columnist attributes the decline of interest in novels to consumerism, technology, and the laziness of people who prefer watching television to reading a novel. However, in reaching this conclusion, the columnist has overlooked important evidence. It is surely relevant that contemporary fiction is frequently of poor quality -- indeed, much of it is meaningless and depressing -- whereas many good newspapers, magazines, professional journals, and books of other types are currently available.", "question": "Which one of the following most accurately expresses the main conclusion of Tallulah's argument?", "answers": "['People read as much as they used to, but most of the works they now read are not novels.', 'Contemporary fiction is unpopular because it is meaningless, depressing, and of poor overall quality.', 'A large number of high-quality newspapers, magazines, professional journals, and nonfiction books are currently published.', 'The view expressed by the columnist was formed without considering all of the pertinent evidence.']", "label": 3 }, { "id": "train_206", "context": "This spring, Bill, who has worked for the City of Hortonville for just over three years, plans to spend the entire three weeks of paid vacation to which he is entitled this year with his family. Anyone who has worked for the City of Hortonville from between one and four years is entitled to two weeks paid vacation each year but can apply up to half of any vacation time that remains unused at the end of the year to the next year' s vacation, as long as the unused vacation time is not more than eight weeks.", "question": "If the statements above are all true, which one of the following must also be true on the basis of them?", "answers": "['Bill had exactly two weeks of vacation left unused at the end of last year.', 'Bill did not have any vacation time left and the end of his first year.', 'Bill will never have eight weeks of vacation time.', 'Bill has used one week of vacation time each year for the past two year.']", "label": 0 }, { "id": "train_207", "context": "Public health will improve more quickly in the wake of new medical discoveries if medical researchers abandon their practice of waiting until their findings are published in peer-reviewed journals before informing the press of important research results. This is because the public release of new medical information allows people to use that information in order to improve their health, but the peer-review process is unavoidably very slow.", "question": "Which one of the following is an assumption on which the argument depends?", "answers": "['People would use new medical information even if it were not first published in peer-reviewed journals.', 'New medical information that is first published in peer-reviewed journals does not usually receive public attention.', 'The peer-review process could be speeded up enough to produce a significant improvement in public health.', 'Reviewers for many medical journals are not themselves medical researchers.']", "label": 0 }, { "id": "train_208", "context": "After a nuclear power plant accident, researchers found radioactive isotopes of iodine, tellurium, and cesium-but no heavy isotopes-in the atmosphere downwind. This material came either from spent fuel rods or from the plant' s core. Spent fuel rods never contain significant quantities of tellurium isotopes. Radioactive material ejected into the atmosphere directly from the core would include heavy isotopes. After the accident, steam, which may have been in contact with the core, was released from the plant. The core contains iodine, tellurium, and cesium isotopes, which are easily dissolved by steam.", "question": "Of the following statements, which one is most strongly supported by the information above?", "answers": "[\"The nuclear power plant's spent fuel rods were not damaged.\", 'Spent fuel rods do not contain heavy isotopes in significant quantities.', \"The researchers found some radioactive material from spent fuel rods as well as some material that was ejected into the atmosphere directly from the plant's core.\", 'The radioactive material detected by the researchers was carried into the atmosphere by the steam that was released from the plant.']", "label": 3 }, { "id": "train_209", "context": "The number of automobile thefts has declined steadily during the past five years, and it is more likely now than it was five years ago that someone who steals a car will be convicted of the crime.", "question": "Which one of the following, if true, most helps to explain the facts cited above?", "answers": "['There are more adolescent car thieves now than there were five years ago, and the sentences given to young criminals tend to be far more lenient than those given to adult criminals.', 'An upsurge in home burglaries over the last five years has required police departments to divert limited resources to investigation of these cases.', 'Although there are fewer car thieves now than there were five years ago, the proportion of thieves who tend to abandon cars before their owners notice that they have been stolen has also decreased.', 'Car alarms are more common than they were five years ago, but their propensity to be triggered in the absence of any criminal activity has resulted in people generally ignoring them when they are triggered.']", "label": 2 }, { "id": "train_210", "context": "The widespread staff reductions in a certain region' s economy are said to be causing people who still have their jobs to cut back on new purchases as though they, too, had become economically distressed. Clearly, however, actual spending by such people is undiminished, because there has been no unusual increase in the amount of money held by those people in savings account.", "question": "The argument in the passage proceeds by doing which one of the following?", "answers": "['concluding that since an expected consequence of a supposed development did not occur, that development itself did not take place', 'concluding that since the evidence concerning a supposed change is ambiguous, it is most likely that no change is actually taking place', 'arguing that since two alternative developments exhaust all the plausible possibilities, one of those developments occurred and the other did not', \"arguing that since people's economic behavior is guided by economic self-interest, only misinformation or error will cause people to engage in economic behavior that harms them economically\"]", "label": 0 }, { "id": "train_211", "context": "Lack of exercise produces the same or similar bodily effects as aging. In fact, the physical changes that accompany aging can often be slowed down by appropriate exercise. No drug, however, holds any promise for slowing down the changes associated with aging. Ttherefore, __.", "question": "Which one of the following provides a logical completion to the passage above?", "answers": "['people who do not exercise when they are young will gain few benefits from beginning to exercise at a later age', 'appropriate exercise can prevent the physical changes associated with aging', 'if the physical changes of aging are to be slowed, it is more practical to rely on exercise than on drugs', 'people who do not exercise are likely to need drugs to sustain their health']", "label": 2 }, { "id": "train_212", "context": "The violent crime rate (number of violent crimes per 1, 000 residents) in Meadowbrook is 60 percent higher now than it was four years ago. The corresponding increase for Parkdale is only 10 percent. These figures support the conclusion that residents of Meadowbrook are more likely to become victims of violent crime than are residents of Parkdale.", "question": "The argument above is flawed because it fails to take into account", "answers": "['the ratio of violent to nonviolent crimes committed during the past four years in Meadowbrook and Parkdale', 'changes in the population density of both Parkdale and Meadowbrook over the past four years', \"how Meadowbrook's expenditures for crime prevention over the past four years compare to Parkdale's expenditures\", 'the violent crime rates in Meadowbrook and Parkdale four years ago']", "label": 3 }, { "id": "train_213", "context": "Economist: Many of my colleagues are arguing that interest rates should be further lowered in order to stimulate economic growth. However, no such stimulation is needed: the economy is already growing at a sustainable rate. So, currently there is no reason to lower interest rates further.", "question": "The reasoning in the economist's argument is questionable in that the argument", "answers": "['relies solely on the testimony of experts', 'presumes that a need to stimulate economic growth is the only possible reason to lower interest rates now', 'takes what is merely one way of stimulating economic growth to be the only way of stimulating economic growth', 'confuses economic growth with what stimulates it']", "label": 1 }, { "id": "train_214", "context": "Every week, the programming office at an FM radio station reviewed unsolicited letters from listeners who were expressing comments on the station' s programs. One week, the station received 50 letters with favorable comments about the station' s news reporting and music selection and 10 letters with unfavorable comments on the station' s new movie review segment of the evening program. Faced with this information, the programming director assumed that if some listeners did not like the movie review segment, then there must be other listeners who did like it. Ttherefore, he decided to continue the movie review segment of the evening program.", "question": "Which one of the following identifies a problem with the programming director's decision process?", "answers": "['He could not properly infer from the fact that some listeners did not like the movie review segment that some others did.', 'He did not wait until he received at least 50 letters with unfavorable comments about the movie review segment before making his decision.', 'He failed to take into account the relation existing between the movie review segment and the news.', 'He failed to recognize that people are more likely to write letters of criticism than of praise.']", "label": 0 }, { "id": "train_215", "context": "A reason Larson cannot do the assignment is that she has an unavoidable scheduling conflict. On the other hand, a reason Franks cannot do the assignment is that he does not quite have the assertiveness the task requires. So, the task must be assigned to Parker, the only supervisor in the shipping department other than Larson and Franks.", "question": "The argument depends on assuming which one of the following?", "answers": "['Franks would be assigned the task if Franks had the assertiveness the task requires.', 'Larson has the assertiveness the task requires.', 'The task cannot be assigned to anyone who has any kind of scheduling conflict.', 'The task cannot be assigned to anyone other than a supervisor in the shipping department.']", "label": 3 }, { "id": "train_216", "context": "Ninety percent of recent car buyers say safety was an important factor in their purchase. Yet of these car buyers, only half consulted objective sources of vehicle safety information before making their purchase; the others relied on advertisements and promotional materials. Thus, these other buyers were mistaken in saying that safety was important to them.", "question": "The argument's conclusion follows logically if which one of the following is assumed?", "answers": "['Someone who claims that safety was an important factor in a buying decision does not necessarily mean that safety was the most important factor.', 'Advertisements and promotional materials sometimes provide incomplete vehicle safety information.', 'Most consumers are aware that advertisements and promotional materials are not objective sources of vehicle safety information.', 'Anyone to whom safety is an important factor in purchasing a car will consult an objective source of vehicle safety information before buying.']", "label": 3 }, { "id": "train_217", "context": "Economist: A country' s trade deficit may indicate weakness in its economy, but it does not in itself weaken that economy. So restricting imports to reduce a trade deficit would be like sticking a thermometer into a glass of cold water in the hope of bringing down a patient' s feverish temperature.", "question": "The economist's argument employs which one of the following techniques?", "answers": "['showing that a recommended course of action would have disastrous consequences', 'demonstrating that an analogy explicitly used to establish a certain conclusion is faulty', 'calling into question the authority on the basis of which a claim is made', 'appealing to an analogy in order to indicate the futility of a course of action']", "label": 3 }, { "id": "train_218", "context": "The similarity between ichthyosaurs and fish is an example of convergence, a process by which different classes of organisms adapt to the same environment by independently developing one or more similar external body features. Ichthyosaurs were marine reptiles and thus do not belong to the same class of organisms as fish. However, ichthyosaurs adapted to their marine environment by converging on external body features similar to those of fish. Most strikingly, ichthyosaurs, like fish, had fins.", "question": "If the statements above are true, which one of the following is an inference that can be properly drawn on the basis of them?", "answers": "['Whenever two classes of organisms share the same environment, members of one class will differ from members of the other class in several external body features.', 'An organism does not necessarily belong to a class simply because the organism has one or more external body features similar to those of members of that class.', 'The members of a single class of organisms that inhabit the same environment must be identical in all their external body features.', 'The members of a single class of organisms must exhibit one or more similar external body features that differentiate that class from all other classes of organisms.']", "label": 1 }, { "id": "train_219", "context": "Citizen: The primary factor determining a dog' s disposition is not its breed, but its home environment. A bad owner can undo generations of careful breeding. Legislation focusing on specific breeds of dogs would not address the effects of human behavior in raising and training animals. As a result, such breed-specific legislation could never effectively protect the public from vicious dogs. Moreover, in my view, the current laws are perfectly adequate.", "question": "Which one of the following most accurately expresses the conclusion drawn by the citizen?", "answers": "['The home environment of dogs would not be regulated by breed-specific legislation.', 'Irresponsible dog owners are capable of producing dogs with bad dispositions regardless of generations of careful breeding.', 'The vicious-dog laws that are currently in effect do not address the effects of human behavior in raising and training dogs.', 'The public would not be effectively protected from violent dogs by breed-specific legislation.']", "label": 3 }, { "id": "train_220", "context": "Nuclear fusion is a process whereby the nuclei of atoms are joined, or \"fused, \" and in which energy is released. One of the by-products of fusion is helium-4 gas. A recent fusion experiment was conducted using \"heavy\" water contained in a sealed flask. The flask was, in turn, contained in an air-filled chamber designed to eliminate extraneous vibration. After the experiment, a measurable amount of helium-4 gas was found in the air of the chamber. The experimenters cited this evidence in support of their conclusion that fusion had been achieved.", "question": "Which one of the following, if true, would cast doubt on the experimenters' conclusion?", "answers": "['Helium-4 gas rapidly breaks down, forming ordinary helium gas after a few hours.', 'When fusion is achieved, it normally produces several by-products, including tritium and gamma rays.', 'Nuclear fusion reactions are characterized by the release of large amounts of heat.', \"The amount of helium-4 found in the chamber's air did not exceed the amount of helium-4 that is found in ordinary air.\"]", "label": 3 }, { "id": "train_221", "context": "The TJ-700 and the Beta Turbomax are two models of cars of different manufacturers. These two models have almost identical specifications and features, and they sell for almost the same price. Both were introduced on the market five years ago, and each year, the respective manufacturers spend about the same amount in advertising. In the past five year, the total number of TJ-700' s sold is about 80% greater than the total number of Beta Turbomax' s sold. Clearly, the advertising firm that created TJ-700 ads created much more effective ads than the firm that created Beta Turbomax ads.", "question": "In evaluating the argument, it would be most useful to know which of the following?", "answers": "['What percentage the first two years maintenance costs of each model is of their respective advertising budgets.', 'How the advertising budgets over the last five years for the TJ-700 and the Beta Turbomax compare to the budgets over the same period for comparable models of other manufacturers.', 'Whether buyers of the TJ-700 and the Beta Turbomax reported being familiar with the ads of these cars.', 'Whether buyer of cars of other manufacturers in the last five years are familiar with the ads for the TJ-700 and the Beta Turbomax.']", "label": 2 }, { "id": "train_222", "context": "Huang: Most people who commit violent crimes do not carefully consider whether or how they will be punished for these crimes. And those who don' t commit violent crimes have no inclination to do so. Rather than impose harsh mandatory sentences, we should attend to the root causes of violence to reduce the rate of violent crime. Suarez: Would you say the same about nonviolent crimes, such as tax evasion? Surely mandatory penalties are a useful deterrent in these cases. At any rate, I am confident that mandatory sentences prevent most people who would otherwise physically harm others from doing so.", "question": "The dialogue between Huang and Suarez most strongly supports the claim that they disagree about whether", "answers": "['mandatory sentences will deter most people who might otherwise commit violent crimes', 'the best way to reduce violent crime is to address the root causes of violence', 'people who commit violent crimes carefully consider how they will be punished for their crimes', 'severe penalties reduce the incidence of tax evasion']", "label": 0 }, { "id": "train_223", "context": "An analysis of the XO Candy Corporation divided its products into two categories: chocolate-based candies and non-chocolate-based candies. The analysis shows that, unlike the non-chocolate candies, few chocolate candies sold enough units to be profitable. Nevertheless, stopping production of the unprofitable chocolate candies will not necessarily make the entire company more profitable, because__ .", "question": "Which of the following most logically completes the passage?", "answers": "['the recipes of the chocolate candies date from the 19th century, while the non-chocolate candies were all developed more recently.', \"a large proportion of XO's customers are those who initially bought XO's chocolate candies and eventually went on to buy other candies made by XO.\", 'XO has recently removed its two lowest-selling chocolate candies from their product line and replaced them with different types of chocolate candies.', 'the chocolate candies are distributed differently than the non-chocolate candies, as chocolates are more popular on the coasts.']", "label": 1 }, { "id": "train_224", "context": "In 1955, legislation in a certain country gave the government increased control over industrial workplace safety conditions. Among the high-risk industries in that country, the likelihood that a worker will suffer a serious injury has decreased since 1955. The legislation, ttherefore, has increased overall worker safety within high-risk industries.", "question": "Which one of the following, if true, most weakens the argument above?", "answers": "['Most of the work-related injuries that occurred before 1955 were the result of worker carelessness.', 'The number of work-related injuries occurring within industries not considered high-risk has increased annually since 1955.', 'Because of technological innovation, most workplaces in the high-risk industries do not require as much unprotected interaction between workers and heavy machinery as they did in 1955.', 'Workplace safety conditions in all industries have improved steadily since 1955.']", "label": 2 }, { "id": "train_225", "context": "The caterpillar of the monarch butterfly feeds on milkweed plants, whose toxins make the adult monarch poisonous to many predators. The viceroy butterfly, whose caterpillars do not feed on milkweed plants, is very similar in appearance to the monarch. Ttherefore, it can be concluded that the viceroy is so seldom preyed on because of its visual resemblance to the monarch.", "question": "Which one of the following, if it were discovered to be true, would most seriously undermine the argument?", "answers": "['Some of the predators of the monarch butterfly also prey on viceroys.', 'The viceroy butterfly is toxic to most predators.', 'Some predators do not have a toxic reaction to insects that feed on milkweed plants.', 'Toxicity to predators is the principal means of protection for only a few butterfly species.']", "label": 1 }, { "id": "train_226", "context": "In his book, published in 1892, Grey used the same metaphor that Jordan used in her book, which was published in 1885. The metaphor is so unusual that there is little chance that two different people independently created it. Ttherefore, it is highly likely that Grey read Jordan' s book.", "question": "Which one of the following, if true, most weakens the argument?", "answers": "[\"Both Grey's book and Jordan's book were written for the same audience.\", 'According to most scholars, Grey was generally a more inventive writer than Jordan and developed many original metaphors.', 'Jordan used the same metaphor in a work that she wrote in 1894 and published in 1895.', 'A text that was probably known to both Jordan and Grey was published in 1860 and also contained the same unusual metaphor.']", "label": 3 }, { "id": "train_227", "context": "If a piece of legislation is the result of negotiation and compromise between competing interest groups, it will not satisfy any of those groups. So, we can see that the recently enacted trade agreement represents a series of compromises among the various interest groups that are concerned with it, because all of those groups are clearly unhappy with it.", "question": "Which one of the following most accurately describes a logical flaw in the argument?", "answers": "['It takes for granted that no piece of legislation can ever satisfy all competing interest groups.', 'It concludes that a condition is necessary for a certain result merely from the claim that the condition leads to that result.', 'It draws a conclusion that is merely a disguised restatement of one of its premises.', 'It bases a conclusion about a particular case on a general principle that concerns a different kind of case.']", "label": 1 }, { "id": "train_228", "context": "Traditionally, decision making by managers that is reasoned step-by-step has been considered preferable to intuitive decision making. However, a recent study found that top managers used intuition significantly more than did most middle-or lower-level managers. This confirms the alternative view that intuition is actually more effective than careful, methodical reasoning.", "question": "The conclusion above is based on which of the following assumptions?", "answers": "['The decisions made by middle-and lower-level managers can be made as easily by using methodical reasoning as by using intuitive reasoning.', 'Top managers are more effective at decision making than middle-or lower-level managers.', 'Methodical, step-by-step reasoning is inappropriate for making many real-life management decisions.', 'Top managers have the ability to use either Intuitive reasoning or methodical, step-by-step reasoning in making decisions.']", "label": 1 }, { "id": "train_229", "context": "The most prestigious attorneys in law firm X are the ones who have shown the greatest ability to win cases. However, every year for the past five years, the average number of cases won by the most prestigious attorneys in law firm X has not been greater than the average cases won by the lower ranking attorneys.", "question": "Which one of the following, if true, most helps to resolve the apparent discrepancy?", "answers": "[\"None of the lower-ranking attorneys work on their cases with law firm X's prestigious attorneys.\", 'Law firm X has a policy of giving the cases that will be most difficult to win to the prestigious attorneys.', 'All of the cases taken by law firm X deal with patent infringement or intellectual property.', 'Normally, it takes at least seven years for a new attorney at law firm X to be considered a prestigious attorney.']", "label": 1 }, { "id": "train_230", "context": "Boreal owls range over a much larger area than do other owls of similar size. The reason for this behavior is probably that the small mammals on which owls feed are especially scarce in the forests where boreal owls live, and the relative scarcity of prey requires the owls to range more extensively to find sufficient food.", "question": "Which of the following, if true, most helps to confirm the explanation above?", "answers": "['Boreal owls range over larger areas in regions where food of the sort eaten by small mammals is sparse than they do in regions where such food is abundant.', 'The boreal owl requires less food, relative to its weight, than is required by members of other owl species.', 'Some boreal owls range over an area eight times larger than the area over which any other owl of similar size ranges.', 'After their young hatch, boreal owls must hunt more often than before in order to feed both themselves and their newly hatched young.']", "label": 0 }, { "id": "train_231", "context": "City council member: Despite the city' s desperate need to exploit any available source of revenue, the mayor has repeatedly blocked council members' attempts to pass legislation imposing real estate development fees. It is clear that in doing so the mayor is sacrificing the city' s interests to personal interests. The mayor cites figures to show that, in the current market, fees of the size proposed would significantly reduce the number of building starts and thus, on balance, result in a revenue loss to the city. But the important point is that the mayor' s family is heavily involved in real estate development and thus has a strong financial interest in the matter.", "question": "Which one of the following most accurately and completely expresses the main conclusion of the city council member's argument?", "answers": "['Significantly reducing the number of building starts would not, on balance, result in revenue loss to the city.', 'Imposing real estate development fees is the best way for the city to exploit the available sources of revenue.', \"The mayor's family has a strong financial interest in preventing the passage of legislation that would impose real estate development fees.\", \"In blocking council members' attempts to impose real estate development fees, the mayor is sacrificing the city's interests to personal interests.\"]", "label": 3 }, { "id": "train_232", "context": "Sabina: The words used in expressing facts affect neither the facts nor the conclusions those facts will support. Moreover, if the words are clearly defined and consistently used, the actual words chosen make no difference to an argument' s soundness. Thus, how an argument is expressed can have no bearing on whether it is a good argument. Emile: Badly chosen words can make even the soundest argument a poor one. After all, many words have social and political connotations that influence people' s response to claims expressed in those words, regardless of how carefully and explicitly those words are defined. Since whether people will acknowledge a fact is affected by how the fact is expressed, the conclusions they actually draw are also affected.", "question": "The point at issue between Emile and Sabina is whether", "answers": "['a sound argument in support of a given conclusion is a better argument than any unsound argument for that same conclusion', 'a word can be defined without taking into account its social and political connotations', 'it would be a good policy to avoid using words that are likely to lead people either to misunderstand the claims being made or to reason badly about those claims', \"a factor that affects neither the truth of an argument's premises nor the logical relation between its premises and its conclusion can cause an argument to be a bad one\"]", "label": 3 }, { "id": "train_233", "context": "A university study reported that between 1975 and 1983 the length of the average workweek in a certain country increased significantly. A governmental study, on the other hand, shows a significant decline in the length of the average workweek for the same period. Examination of the studies shows, however, that they used different methods of investigation; thus there is no need to look further for an explanation of the difference in the studies' results.", "question": "The argument's reasoning is flawed because the argument fails to", "answers": "['recognize that varying economic conditions result in the average workweek changing in length', 'recognize that two different methods of investigation can yield identical results', 'distinguish between a study produced for the purposes of the operation of government and a study produced as part of university research', 'distinguish between a method of investigation and the purpose of an investigation']", "label": 1 }, { "id": "train_234", "context": "Not surprisingly, there are no professors under the age of eighteen. And, as well known, no one under eighteen can vote legally. Finally, some brilliant people are professors, some are legal voters, and some are under eighteen.", "question": "If the statements above are true, then on the basis of them which one of the following must also be true?", "answers": "['No professors are eighteen-year-olds.', 'All brilliant people are either professors, legal voters, or under eighteen.', 'Some brilliant people are neither professors nor legal voters.', 'Some legal voters are not professors.']", "label": 2 }, { "id": "train_235", "context": "Sociologists study folktales because they provide a means of understanding the distinctive values of a culture. However, the folktales in almost all cultures are adaptations of the same ancient narratives to the local milieu.", "question": "Which one of the following, if true, most helps to resolve the apparent discrepancy in the information above?", "answers": "['The ancient narratives persist largely because they speak to basic themes and features of the human condition.', 'Because no single person is the author of a folktale, folktales must reflect the values of a culture rather than those of an individual.', 'Folktales are often oral traditions that persist from times when few people left written materials.', 'The manner in which a culture adapts its narratives reveals information about the values of that culture.']", "label": 3 }, { "id": "train_236", "context": "Studies in restaurants show that the tips left by customers who pay their bill in cash tend to be larger when the bill is presented on a tray that bears a credit card logo. Consumer psychologists hypothesize that simply seeing a credit-card logo makes many creditcard holders willing to spend more because it reminds them that their spending power exceeds the cash they have immediately available.", "question": "Which of the following, if true, most strongly supports the psychologists' interpretation of the studies?", "answers": "['Patrons who are under financial pressure from their credit-card obligations tend to tip less when presented with a restaurant bill on a tray with a credit-card logo than when the tray has no logo.', 'The effect noted in the studies is not limited to patrons who have credit cards.', 'In virtually all of the cases in the studies, the patrons who paid bills in cash did not possess credit cards.', 'In general, restaurant patrons who pay their bills in cash leave larger tips than do those who pay by credit card.']", "label": 0 }, { "id": "train_237", "context": "Magazine article: Punishment for crimes is justified if it actually deters people from committing them. But a great deal of carefully assembled and analyzed empirical data show clearly that punishment is not a deterrent. So punishment is never justified.", "question": "The reasoning in the magazine article's argument is flawed because the argument", "answers": "['attempts to be more precise than its subject matter properly allows', 'ignores the problem of mistakenly punishing the innocent', 'mistakenly allows the key term \"punishment\" to shift in meaning', 'mistakes being sufficient to justify punishment for being required to justify it']", "label": 3 }, { "id": "train_238", "context": "Frobisher, a sixteenth-century English explorer, had soil samples from Canada' s Kodlunarn Island examined for gold content. Because high gold content was reported, Elizabeth I funded two mining expeditions. Neither expedition found any gold there. Modern analysis of the island' s soil indicates a very low gold content. Thus the methods used to determine the gold content of Frobisher' s samples must have been inaccurate.", "question": "Which of the following is an assumption on which the argument depends?", "answers": "['The two mining expeditions funded by Elizabeth I did not mine the same part of Kodlunarn Island.', 'Frobisher did not have soil samples from any other Canadian island examined for gold content.', 'Gold was not added to the soil samples collected by Frobisher before the samples were examined.', 'The gold content of the soil on Kodlunarn Island is much lower today than it was in the sixteenth century.']", "label": 2 }, { "id": "train_239", "context": "Psychologist: Identical twins are virtually the same genetically. Moreover, according to some studies, identical twins separated at birth and brought up in vastly different environments show a strong tendency to report similar ethical beliefs, dress in the same way, and have similar careers. Thus, many of our inclinations must be genetic in origin, and not subject to environmental influences.", "question": "Which one of the following, if true, would most weaken the psychologist's argument?", "answers": "['Scientists are far from being able to link any specific genes to specific inclinations.', 'Many people, including identical twins, undergo radical changes in their lifestyles at some point in their lives.', 'While some studies of identical twins separated at birth reveal a high percentage of similar personality traits, they also show a few differences.', 'Identical twins who grow up together tend to develop different beliefs, tastes, and careers in order to differentiate themselves from each other.']", "label": 3 }, { "id": "train_240", "context": "John: There have been complaints about the lack of parks in our city. Some people favor turning the old airport land into biking and hiking trails, but there may be more productive ways of using that land. Pablo: But the old airport land is ideal for biking and hiking trails. Our citizens have gone too long with an inadequate number of parks; we should not dismiss this proposal for biking and hiking trails without further consideration.", "question": "Pablo's criticism suggests that he interpreted John to be", "answers": "['maintaining that converting the old airport land into biking and hiking trails would be entirely unproductive use of that land', 'asserting that it may not be possible to convert the old airport land into adequate biking and hiking results', 'recommending that the proposal for converting the old airport land into biking and hiking trails should be promptly dismissed', 'favoring the development of parks other than biking and hiking trails']", "label": 2 }, { "id": "train_241", "context": "The top prize in architecture, the Pritzker Prize, is awarded for individual achievement, like Nobel Prizes for science. But architects are judged by their buildings, and buildings are the result of teamwork. As achievements, buildings are not like scientific discoveries, but like movies, which compete for awards for best picture. Thus, it would be better if the top prize in architecture were awarded to the best building rather than the best architect.", "question": "The argument proceeds by", "answers": "['making a distinction between two different types of objects in order to conclude that one has more inherent value than the other', 'contending that an action is inappropriate by presenting an argument that a corresponding action in an analogous case is inappropriate', 'reaching a conclusion about the way something should be done in one field on the basis of comparisons with corresponding practices in other fields', 'pointing to similarities between two practices as a basis for concluding that criticisms of one practice can rightly be applied to the other']", "label": 2 }, { "id": "train_242", "context": "Many people change their wills on their own every few years, in response to significant changes in their personal or financial circumstances. This practice can create a problem for the executor when these people are careless and do not date their wills: the executor will then often know neither which one of several undated wills is the most recent, nor whether the will drawn up last has ever been found. Ttherefore, people should not only date their wills but also state in any new will which will it supersedes, for then there would not be a problem to begin with.", "question": "The reasoning in the argument is flawed because the argument", "answers": "['fails to distinguish between prevention of a problem and successful containment of the adverse effects that the problem might cause', 'treats a partial solution to the stated problem as though it were a complete solution', 'proposes a solution to the stated problem that does not actually solve the problem but merely makes someone else responsible for solving the problem', 'claims that a certain action would be a change for the better without explicitly considering what negative consequences the action might have']", "label": 1 }, { "id": "train_243", "context": "Announcement for a television program: Are female physicians more sensitive than male physicians to the needs of women patients? To get the answer, we' ll ask physicians of both sexes this question. Tune in tomorrow.", "question": "Which one of the following, if true, identifies a flaw in the plan for the program?", "answers": "['There still are fewer women than men who are physicians, so a patient might not have the opportunity to choose a woman as a physician.', 'Women as patients are now beginning to take a more active role in managing their care and making sure that they understand the medical alternatives.', 'Those who are best able to provide answers to the question are patients, rather than physicians.', 'Physicians are in general unwilling to describe the treatment style of other physicians.']", "label": 2 }, { "id": "train_244", "context": "Enforcement of local speed limits through police monitoring has proven unsuccessful in the town of Ardane. In many nearby towns, speed humps (raised areas of pavement placed across residential streets, about 300 feet apart) have reduced traffic speeds on residential streets by 20 to 25 percent. In order to reduce traffic speed and thereby enhance safety residential neighborhoods, Ardane' s transportation commission plans to install multiple speed humps in those neighborhoods.", "question": "Which of the following, if true, identifies a potentially serious drawback to the plan for installing speed humps in Ardane?", "answers": "['On residential streets without speed humps, many vehicles travel at speeds more than 25 percent above the posted speed limit.', 'Bicyclists generally prefer that speed humps be constructed so as to leave a space on the side of the road where bicycles can travel without going over the humps', 'Because of their high weight, emergency vehicles such as fire trucks and ambulances must slow almost to a stop at speed humps.', 'The residential speed limit in Ardane is higher than that of the nearby towns where speed humps were installed.']", "label": 2 }, { "id": "train_245", "context": "A certain type of insect trap uses a scented lure to attract rose beetles into a plastic bag from which it is difficult for them to escape. If several of these traps are installed in a backyard garden, the number of rose beetles in the garden will be greatly reduced. If only one trap is installed, however, the number of rose beetles in the garden will actually increase.", "question": "Which one of the following, if true, most helps to resolve the apparent discrepancy?", "answers": "['When there are several traps in a garden, they each capture fewer rose beetles than any single trap would if it were the only trap in the garden.', \"The scent of a single trap's lure usually cannot be detected throughout a backyard garden by rose beetles.\", 'The presence of any traps in a backyard garden will attract more rose beetles than one trap can catch, but several traps will not attract significantly more rose beetles to a garden than one trap will.', 'When there is only one trap in the garden, the plastic bag quickly becomes filled to capacity, allowing some rose beetles to escape.']", "label": 2 }, { "id": "train_246", "context": "Even if many more people in the world excluded meat from their diet, world hunger would not thereby be significantly reduced.", "question": "Which one of the following, if true, most calls into question the claim above?", "answers": "['Often people go hungry because they live in remote barren areas where there is no efficient distribution for emergency food relief.', 'Hunger often results from natural disasters like typhoons or hurricanes, which sweep away everything in their path.', 'Both herds and crops are susceptible to devastating viral and other diseases.', 'The amount of land needed to produce enough meat to feed one person for a week can grow enough grain to feed more than ten people for a week.']", "label": 3 }, { "id": "train_247", "context": "Advertisement: The Country Classic is the only kind of car in its class that offers an antilock braking system that includes TrackAid. An antilock braking system keeps your wheels from locking up during hard braking, and TrackAid keeps your rear wheels from spinning on slippery surfaces. So if you are a safety-conscious person in the market for a car in this class, the Country Classic is the only car for you.", "question": "The advertisement is misleading if which one of the following is true?", "answers": "['Other cars in the same class as the Country Classic offer an antilock braking system that uses a method other than TrackAid to prevent rear wheels from spinning on slippery surfaces.', 'All of the cars that are in the same class as the Country Classic offer some kind of antilock braking system.', 'Without an antilock braking system, the wheels of the Country Classic and other cars in its class are more likely to lock up during hard braking than they are to spin on slippery surfaces.', 'The Country Classic is more expensive than any other car in its class.']", "label": 0 }, { "id": "train_248", "context": "Cox: The consumer council did not provide sufficient justification for its action when it required that Derma-35 be recalled from the market. Crockett: I disagree. Derma-35 in fact causes inflammation, but in citing only the side effect of blemishes as the justification for its decision, the council rightly acknowledged that blemishes are a legitimate health concern.", "question": "Cox and Crockett disagree over whether", "answers": "['inflammation is a serious health threat', 'the council based its decision on the threat of inflammation or on the threat of blemishes', 'the council gave an adequate reason for its decision to recall Derma-35', 'Derma-35 should remain on the market']", "label": 2 }, { "id": "train_249", "context": "A certain credit-card company awards its customers bonus points for using its credit card. Customers can use accumulated points in the purchase of brand name merchandise by mail at prices lower than the manufacturers' suggested retail prices. At any given time, ttherefore, customers who purchase merchandise using the bonus points spend less than they would spend if they purchased the same merchandise in retail stores.", "question": "Which one of the following is an assumption on which the argument depends?", "answers": "['The bonus points cannot be used by the creditcard customers in the purchase of brand name merchandise that is not available for purchase in retail stones.', \"The credit-card company does not require its customers to accumulate a large number of bonus points before becoming eligible to order merchandise at prices lower than the manufacturers' suggested retail price.\", 'The amount credit-card customers pay for shipping the merchandise ordered by mail does not increase the amount customers spend to an amount greater than they would spend if they purchased the same merchandise in retail stores.', \"The merchandise available to the company's credit-card customers using the bonus points is frequently sold in retail stores at prices that are higher than the manufacturers' suggested retail prices.\"]", "label": 2 }, { "id": "train_250", "context": "Correctly measuring the productivity of service workers is complex. Consider, for example, postal workers: they are often said to be more productive if more letters are delivered per postal worker. But is this really true? What if more letters are lost or delayed per worker at the same time that more are delivered?", "question": "The objection implied above to the productivity measure described is based on doubts about the truth of which of the following statements?", "answers": "['The delivery of letters is the primary activity of the postal service.', 'Postal workers are representative of service workers in general.', 'The quality of services rendered can appropriately be ignored in computing productivity.', 'The number of letters delivered is relevant to measuring the productivity of postal workers.']", "label": 2 }, { "id": "train_251", "context": "Soft Drinks Manufacturer: Despite attractive offers, our company has rejected any plans to expand our market into the Czech Republic at this time. We simply have no data on how well received our products would be in the Czech Republic. Industry Analyst: Your position is inconsistent. Last year, you expanded into Bolivia, even after taste tests had definitely found that approximately 40% of the Bolivian population did not care for any of your products. Ttherefore, concerns about how well received your product will be cannot be motivating this position.", "question": "The industry analyst's argument is flawed because it fails to consider that:", "answers": "[\"the company's most popular soft drink in Bolivia might be different from the company's most popular soft drink in the Czech Republic.\", 'known risks can be assessed, but unknown risks cannot.', 'coffee is more popular than most soft drinks in both Bolivia and in the Czech Republic', 'the Czech currency is much stronger than the Boliviano, the national currency of Bolivia.']", "label": 1 }, { "id": "train_252", "context": "Library preservationist: Due to the continual physical deterioration of the medieval manuscripts in our library' s collection, we have decided to restore most of our medieval manuscripts that are of widely acknowledged cultural significance, though this means that some medieval manuscripts whose authenticity is suspect will be restored. However, only manuscripts whose safety can be ensured during the restoration process will be restored, and manuscripts that are not frequently consulted by researchers will not be restored.", "question": "If all of the library preservationist's statements are true, which one of the following must be true of the medieval manuscripts in the library's collection?", "answers": "['The medieval manuscripts most susceptible to deterioration are those most frequently consulted by researchers.', 'All of the medieval manuscripts widely acknowledged to be of cultural significance are manuscripts whose safety can be ensured during the restoration process.', 'All of the medieval manuscripts whose safety can be ensured during the restoration process are frequently consulted by researchers.', 'Some of the medieval manuscripts whose authenticity is suspect are frequently consulted by researchers.']", "label": 3 }, { "id": "train_253", "context": "Excavations of the Roman city of Sepphoris have uncovered numerous detailed mosaics depicting several readily identifiable animal species: a hare, a partridge, and various Mediterranean fish. Oddly, most of the species represented did not live in the Sepphoris region when these mosaics were created. Since identical motifs appear in mosaics found in other Roman cities, however, the mosaics of Sepphoris were very likely created by traveling artisans from some other part of the Roman Empire.", "question": "Which of the following is an assumption on which the argument depends?", "answers": "['No motifs appear in the Sepphoris mosaics that do not also appear in the mosaics of some other Roman city.', 'There was not a common repertory of mosaic designs with which artisans who lived in various parts of the Roman Empire were familiar.', 'All of the animal figures in the Sepphoris mosaics are readily identifiable as representations of known species.', 'There is no single region to which all the species depicted in the Sepphoris mosaics are native.']", "label": 1 }, { "id": "train_254", "context": "Concert promoter: Some critics claim that our concert series lacks popular appeal. But our income from the sales of t-shirts and other memorabilia at the concerts is equal to or greater than that for similar sales at comparable series. So those critics are mistaken.", "question": "The concert promoter's argument is flawed in that it", "answers": "['takes for granted that the comparable series possess popular appeal', 'draws a conclusion about the popularity of a series based on a comparison with other, dissimilar events', 'takes for granted that income from sales of memorabilia is the sole indicator of popular appeal', 'attacks the critics on the basis of emotional considerations rather than factual ones']", "label": 0 }, { "id": "train_255", "context": "When a polygraph test is judged inconclusive, this is no reflection on the examinee. Rather, such a judgment means that the test has failed to show whether the examinee was truthful or untruthful. Nevertheless, employers will sometimes refuse to hire a job applicant because of an inconclusive polygraph test result.", "question": "Which of the following conclusions can most properly be drawn from the information above?", "answers": "['Most examinees with inconclusive polygraph test results are in fact untruthful.', 'An inconclusive polygraph test result is sometimes unfairly held against the examinee.', 'A polygraph test indicating that an examinee is untruthful can sometimes be mistaken.', 'Some employers have refused to consider the results of polygraph tests when evaluating job applicants.']", "label": 1 }, { "id": "train_256", "context": "Having an efficient, appealing store layout makes good economic sense. So, the grocery store needs to purchase new shelving systems, since the grocery store should always do what makes the best economic sense.", "question": "The conclusion drawn above follows logically if which one of the following is assumed?", "answers": "['There is not some other improvement that makes more economic sense for the grocery store than does investing in a new shelving system.', 'New shelving is required in order for the grocery store to have a floor plan that is efficient and appealing.', 'The grocery store can afford to purchase the new shelving that is being proposed.', 'Cost-effective shelving is an integral part of an efficient grocery store.']", "label": 0 }, { "id": "train_257", "context": "Physician: In itself, exercise does not cause heart attacks; rather, a sudden increase in an exercise regimen can be a cause. When people of any physical condition suddenly increase their amount of exercise, they also increase their risk of heart attack. As a result, there will be an increased risk of heart attack among employees of this company due to the new health program.", "question": "The conclusion drawn by the physician follows logically if which one of the following is assumed?", "answers": "['Employees will abruptly increase their amount of exercise as a result of the new health program.', 'The new health program will force employees of all levels of health to exercise regularly.', \"The new health program constitutes a sudden change in the company's policy.\", 'All employees, no matter what their physical condition, will participate in the new health program.']", "label": 0 }, { "id": "train_258", "context": "The chances that tropical storms will develop in a given area increase whenever the temperature of a large body of water in that area exceeds 26 degrees Celsius to a depth of about 60 meters. If the amount of carbon dioxide in the Earth' s atmosphere continues to increase, the temperatures of all of the Earth' s waters will rise, with the result that the number of large bodies of water whose temperatures exceed 26 degrees Celsius to a depth of about 60 meters will eventually be greater than it is today.", "question": "The statements above, if true, most strongly support which one of the following conclusions?", "answers": "['The number of large bodies of water whose temperatures exceed 26 degrees Celsius to a depth of about 60 meters is greater today than it ever was.', \"The ferocity of tropical storms does not depend on the amount of carbon dioxide in the Earth's atmosphere.\", \"There are likely to be more tropical storms if the amount of carbon dioxide in the Earth's atmosphere continues to increase.\", \"Any increase in the temperatures of the Earth's oceans would cause the amount of carbon dioxide in the atmosphere to increase as well.\"]", "label": 2 }, { "id": "train_259", "context": "A new gardening rake with an S-shaped handle reduces compression stress on the spine during the pull stroke to about one-fifth of what it is with a straight-handled rake. During the push stroke, however, compression stress is five times more with the new rake than with a straight-handled rake. Neither the push stroke nor the pull stroke with a straight-handled rake produces enough compression stress to cause injury, but compression stress during the push stroke with the new rake is above the danger level. Ttherefore, straight-handled rakes are better than the new rakes for minimizing risk of spinal injury.", "question": "The conclusion above is properly drawn from the premises given if which one of the following is true?", "answers": "['Compression stress resulting from pushing is the only cause of injuries to the spine that occur as a result of raking.', 'A garden rake can never be used in such a way that all the strokes with that rake are push strokes.', 'It is not possible to design a garden rake with a handle that is other than straight or S-shaped.', 'Raking is a frequent cause of spinal injury among gardeners.']", "label": 0 }, { "id": "train_260", "context": "In the paintings by seventeenth-century Dutch artist Vermeer, we find several recurrent items: a satin jacket, a certain Turkish carpet, and wooden chairs with lion' s head finials. These reappearing objects might seem to evince a dearth of props. Yet we know that many of the props Vermeer used were expensive. Thus, while we might speculate about exactly why Vermeer worked with a small number of familiar objects, it was clearly not for lack of props that the recurrent items were used.", "question": "The conclusion follows logically if which one of the following is assumed?", "answers": [ "If a dearth of props accounted for the recurrent objects in Vermeer's paintings, we would not see expensive props in any of them.", "The props that recur in Vermeer's paintings were always available to him.", "The satin jacket and wooden chairs that recur in the paintings were owned by Vermeer's sister.", "The several recurrent items that appeared in Vermeer's paintings had special sentimental importance for him." ], "label": 0 }, { "id": "train_261", "context": "Some plants have extremely sensitive biological thermometers. For example, the leaves of rhododendrons curl when the temperature of the air around them is below 0 degree C (Celsius). Similarly, mature crocus blossoms open in temperatures above 2 degree C. So someone who simultaneously observed rhododendrons with uncurled leaves, crocuses with mature but unopened blossoms, and a thermometer showing 1 degree C could determine that the thermometer' s reading was accurate to within plus or minus 1 degree C.", "question": "Which one of the following, if true, most seriously undermines the reasoning above?", "answers": "['Certain types of thermometers that are commonly used to measure outdoor temperatures can be extremely accurate in moderate temperature ranges but much less accurate in warmer or colder temperature ranges.', 'The climate and soil conditions that favor the growth of rhododendrons are also favorable to the growth of crocuses.', 'Neither rhododendrons nor crocuses bloom for more than a few weeks each year, and the blossoms of rhododendrons growing in any area do not appear until at least several weeks after crocuses growing in that area have ceased to bloom.', 'Air temperature surrounding rhododendrons, which can grow 12 feet tall, is likely to differ from air temperature surrounding crocuses, which are normally only a few inches high, by more than 2 degree C, even if the two plants are growing side by side.']", "label": 3 }, { "id": "train_262", "context": "Escalating worldwide demand for corn is expected to cause the market price of corn to rise sharply and remain high. Although corn is extensively used as feed for livestock, livestock feed accounts for only a small fraction of the retail price of meat. Ttherefore, the increase in corn prices is very unlikely to produce any comparable long-term increase in the retail price of meat.", "question": "Which of the following, if true, most seriously weakens the argument?", "answers": "['Livestock producers who currently use corn to feed their livestock have the option of switching to other kinds of feed.', 'The price of corn affects the retail price of a variety of other food products more than it affects the price of meat.', 'The rising cost of feed is leading some livestock producers to leave the business, thereby reducing the future supply of meat.', 'Worldwide demand for grains other than corn has also increased.']", "label": 2 }, { "id": "train_263", "context": "A public research facility should not patent its inventions. Such facilities, because they are supported by public funds, should encourage the free flow of ideas. A public research facility that retains the right to patent an invention has a motive to suppress the free flow of ideas in order to profit from the patent. Retaining the right to patent inventions is contrary to the public research facility' s obligation to do work intended for the benefit of the public.", "question": "The claim that a public research facility should not patent its inventions plays what role in the argument?", "answers": "['It is a claim that is later proven in the argument.', 'It is the conclusion of the argument.', 'It provides a roadmap for the passage.', 'It is an assumption upon which the argument is built.']", "label": 1 }, { "id": "train_264", "context": "People who have specialized knowledge about a scientific or technical issue are systematically excluded from juries for trials where that issue is relevant. Thus, trial by jury is not a fair means of settling disputes involving such issues.", "question": "Which one of the following, if true, most seriously weakens the argument?", "answers": "['The more a juror knows about a particular scientific or technical issue involved in a trial, the more likely it is that the juror will be prejudiced in favor of one of the litigating parties before the trial begins.', 'Expert witnesses in specialized fields often command fees that are so high that many people involved in litigation cannot afford their services.', 'The more complicated the issue being litigated, the less likely it is that a juror without specialized knowledge of the field involved will be able to comprehend the testimony being given.', 'Appointing an impartial arbitrator is not a fair means of settling disputes involving scientific or technical issues, because arbitrators tend to favor settlements in which both parties compromise on the issues.']", "label": 0 }, { "id": "train_265", "context": "Bunty: The primary job of police officers is keeping the peace. Since their subsidiary jobs, such as controlling traffic, hinder their performance of the primary one, people other than police officers should be hired to perform the subsidiary jobs. Naina: To perform their primary job well, police officers must have the trust of citizens. They gain that trust by performing their subsidiary jobs, which are the only functions most citizens see them fulfill.", "question": "Bunty and Naina disagree with each other about", "answers": "['whether the subsidiary jobs police officers perform enable them to perform their primary job effectively', 'whether the primary job of police officers is keeping the peace', 'whether police officers can effectively gain the trust of citizens', 'whether police officers need to win the trust of law-abiding citizens in order to keep the peace effectively']", "label": 0 }, { "id": "train_266", "context": "Jay: Of course there are many good reasons to support the expansion of preventive medical care, but arguments claiming that it will lead to greater societal economic gains are misguided. Some of the greatest societal expenses arise from frequent urgent-care needs for people who have attained a long life due to preventive care. Sunil: Your argument fails because you neglect economic gains outside the health care system: society suffers an economic loss when any of its productive members suffer preventable illnesses.", "question": "Sunil's response to Jay makes which of the following assumptions?", "answers": "['Jay is incorrect in stating that patients who receive preventive medical care are long-lived.', 'Productive members of society are more likely than others to suffer preventable illnesses.', 'Those who receive preventive care are not more likely to need urgent care than are those who do not receive preventive care.', 'The economic contributions of those who receive preventive medical care may outweigh the economic losses caused by preventive care.']", "label": 3 }, { "id": "train_267", "context": "Infants younger than six months who have normal hearing can readily distinguish between acoustically similar sounds that are used as part of any language -- not only those used in the language spoken by the people who raise them. Young adults can readily distinguish between such sounds only in languages that they regularly use. It is known that the physiological capacity to hear begins to deteriorate after infancy. So the observed difference in the abilities of infants and young adults to distinguish between acoustically similar speech sounds must be the result of the physiological deterioration of hearing.", "question": "The reasoning in the argument is flawed because the argument", "answers": "['does not explain the procedures used to measure the abilities of two very different populations', 'takes a factor that might contribute to an explanation of the observed difference as a sufficient explanation for that difference', 'sets an arbitrary cutoff point of six months for the age below which infants are able to distinguish acoustically similar speech sounds', 'assumes that what is true of a group of people taken collectively is also true of any individual within that group']", "label": 1 }, { "id": "train_268", "context": "Gamba: Munoz claims that the Southwest Hopeville Neighbors Association overwhelmingly opposes the new water system, citing this as evidence of citywide opposition. The association did pass a resolution opposing the new water system, but only 25 of 350 members voted, with 10 in favor of the system. Furthermore, the 15 opposing votes represent far less than 1 percent of Hopeville' s population. One should not assume that so few votes represent the view of the majority of Hopeville' s residents.", "question": "Of the following, which one most accurately describes Gamba's strategy of argumentation?", "answers": "['criticizing a view on the grounds that the view is based on evidence that is in principle impossible to disconfirm', 'attempting to cast doubt on a conclusion by claiming that the statistical sample on which the conclusion is based is too small to be dependable', 'attempting to refute an argument by showing that, contrary to what has been claimed, the truth of the premises does not guarantee the truth of the conclusion', 'questioning a conclusion based on the results of a vote, on the grounds that people with certain views are more likely to vote']", "label": 1 }, { "id": "train_269", "context": "If one is to participate in the regional band, one must practice very hard or be very talented. Ttherefore, Lily, who is first trombonist in the regional band and is very talented, does not practice hard.", "question": "The flawed reasoning in which one of the following arguments most closely resembles the flawed reasoning in the argument above?", "answers": "['If Johnson is to win the local election, then neither Horan nor Jacobs can enter the race. Since neither of them plans to run, Johnson will win the race.', 'In order to have a chance to meet its objectives, the army needs good weather as a precondition for retaining its mobility. The weather is good today, so the army will meet its objectives.', 'If Wayne is to get a ride home from the library, either Yvette or Marty must be there. Yvette is not at the library, so Marty must be there.', 'To stay informed about current events, one must read a major newspaper or watch national TV news every day. So Julie, who is informed about current events and reads a major newspaper every day, does not watch TV news.']", "label": 3 }, { "id": "train_270", "context": "Ethicist: Studies have documented the capacity of placebos to reduce pain in patients who believe that they are receiving beneficial drugs. Some doctors say that they administer placebos because medically effective treatment reinforced by the placebo effect sometimes helps patients recover faster than good treatment alone. But administering placebos is nonetheless ethically questionable, for even if a placebo benefits a patient, a doctor might, for example, have prescribed it just to give the patient satisfaction that something was being done.", "question": "The ethicist's argument depends on which one of the following assumptions?", "answers": "['The pain relief produced by the placebo effect justifies the deception involved in administering a placebo.', 'Medical treatment that relies on the placebo effect alone is ethically indefensible.', 'The motivation for administering a placebo can be relevant to the ethical justification for doing so.', \"A patient's psychological satisfaction is not a consideration in administering medical treatment.\"]", "label": 2 }, { "id": "train_271", "context": "The formation of hurricanes that threaten the United States mainland is triggered by high atmospheric winds off the western coast of Africa. When abundant rain falls in sub-Saharan Africa, hurricanes afterward hit the United States mainland with particular frequency. Ttherefore, the abundant rains must somehow promote the ability of the winds to form hurricanes.", "question": "Which one of the following arguments contains a flaw that is most similar to one in the argument above?", "answers": "['People who exercise vigorously tend to sleep well. Ttherefore, people who exercise vigorously tend to be healthy.', \"Many people who later become successful entrepreneurs played competitive sports in college. Ttherefore, playing competitive sports must enhance a person's entrepreneurial ability.\", 'Cars drive faster on long city blocks than on short city blocks. Long blocks are thus more dangerous for pedestrians than short blocks.', \"The blossoms of the chicory plant close up in full sun. Ttherefore, the chicory plant's blossoms must open up in the dark.\"]", "label": 1 }, { "id": "train_272", "context": "Many people limit the intake of calories and cholesterol in their diet in order to lose weight and reduce the level of cholesterol in their blood. When a person loses weight, the fat cells in that person' s body decrease in size but not in number. As they decrease in size, fat cells spill the cholesterol they contain into the bloodstream. Ttherefore, a person who goes on a low-calorie, low cholesterol diet __ .", "question": "Which one of the following most logically completes the argument?", "answers": "['will both decrease the level of cholesterol in his or her blood and gain weight', 'might at first have an increased level of cholesterol in his or her blood', 'will not lose weight any faster than will a person whose diet is high in calories', 'will not decrease the size of his or her fat cells']", "label": 1 }, { "id": "train_273", "context": "In a town containing a tourist attraction, hotel and restaurant revenues each increased more rapidly over the past year than did revenue from the sale of passes to the attraction, which are valid for a full year. This led those in charge of the attraction to hypothesize that visitors were illicitly selling or sharing the passes.", "question": "Each of the following, if true, helps to undermine the hypothesis of those in charge of the tourist attraction EXCEPT:", "answers": "['During the past year other tourist attractions have opened up in the area.', 'While the cost of passes is unchanged since last year, hotel and meal prices have risen.', 'The local board of tourism reports that the average length of stay for tourists remained unchanged over the past year.', 'Those possessing passes made more frequent trips to the attraction last year than in previous years.']", "label": 2 }, { "id": "train_274", "context": "According to a prediction of the not-so-distant future published in 1940, electricity would revolutionize agriculture. Electrodes would be inserted into the soil, and the current between them would kill bugs and weeds and make crop plants stronger.", "question": "Which of the following, if true, most strongly indicates that the logic of the prediction above is flawed?", "answers": "['In order for farmers to avoid electric shock while working in the fields, the current could be turned off at such times without diminishing the intended effects.', 'Because a planting machine would need to avoid coming into contact with the electrodes, new parts for planting machines would need to be designed.', 'It cannot be taken for granted that the use of electricity is always beneficial.', 'Since weeds are plants, electricity would affect weeds in the same way as it would affect crop plants.']", "label": 3 }, { "id": "train_275", "context": "Lactose, a sugar found in milk, aids in the absorption of calcium, which in turn is required for bone repair. In addition to there being shortages of milk in tropical areas, inhabitants of these areas lose the ability to absorb lactose, unlike people from nontropical areas. Yet inhabitants of tropical areas have no more problems with bone repair than do people who inhabit nontropical areas.", "question": "Which one of the following, if true, most helps to resolve the apparent paradox described above?", "answers": "['People living in tropical areas periodically take tablets containing lactose when there are shortages of milk.', 'Milk consumption has fallen in both tropical and nontropical areas.', 'The extent to which people living in tropical areas lose the ability to absorb lactose depends on a genetic predisposition.', 'The abundant sunlight in tropical areas causes the human body to produce vitamin D naturally, which aids in the absorption of calcium.']", "label": 3 }, { "id": "train_276", "context": "Doctor: Medication to reduce blood pressure often has unhealthy side effects. However, lifestyle changes such as exercising more and avoiding fatty foods reduce blood pressure just as effectively as taking medication does. Ttherefore, it is healthier to rely on these lifestyle changes than on medication to reduce blood pressure.", "question": "Which one of the following is an assumption that the doctor's argument requires?", "answers": "['Other than medication, the only way to reduce blood pressure is by making lifestyle changes such as exercising more and avoiding fatty foods.', 'The side effects, if any, of exercising more and avoiding fatty foods in order to reduce blood pressure are less unhealthy than those of taking medication to reduce blood pressure.', 'If it is healthier to rely on a lifestyle change than on medication to reduce blood pressure, then that lifestyle change reduces blood pressure at least as effectively as medication does.', 'If an alternative to medication relieves a medical condition just as effectively as medication does, then it is always healthier to rely on that alternative than on medication to relieve that medical condition.']", "label": 1 }, { "id": "train_277", "context": "In a recent poll of chief executive officers (CEOs) of 125 large corporations, the overwhelming majority claimed that employee training and welfare is of the same high priority as customer satisfaction. So the popular belief that the top management of large corporations behaves indifferently to the needs and aspirations of employees is unfounded.", "question": "The argument is most vulnerable to criticism on the grounds that it", "answers": "['fails to define adequately the term \"top management\"', \"presumes, without giving justification, that the CEOs' priorities tend to be misplaced\", \"presumes, without giving justification, that the CEOs' claims are reflected in actual practice\", 'makes a generalization based on an unrepresentative sample']", "label": 2 }, { "id": "train_278", "context": "Most road repairs require more time and money than is budgeted, but last summer' s nighttime repairs of Highway 93 and similar roads required no more time or money than had been budgeted. Ttherefore, making summer repairs to major roads at night would save both time and money.", "question": "Which of the following, if true, most strongly supports the conclusion drawn above?", "answers": "['Asphalt used in road repair tends to expand in warmer temperatures and contract in cooler temperatures.', 'The budget for the repairs to Highway 93 was generous enough to make it unlikely that it would be exceeded.', 'The smaller number of cars on the roads at night and more comfortable nighttime temperatures allow road workers to work more quickly.', 'Road repair crews that work at night mark their work sites with bright hashing lights in addition to the orange cones they use during the daytime.']", "label": 2 }, { "id": "train_279", "context": "A very popular ice-cream shop recently raised the price of an ice-cream soda by $0. 20, from $1. 80 to $2. 00. Within a week of the price increase, all of the waiters-and waitresses had quit.", "question": "Which of the following, if true, would most help to explain the resignations of the waiters and waitresses?", "answers": "['The price increase allowed the shop to retain its popular generous portions of ice cream.', 'The waiters and waitresses received fewer tips than before because the increased price deprived customers of change that many had left as tips.', 'The increased price of ice-cream sodas did not affect the wage rate at which the waiters and waitresses were paid.', 'Regular customers continued to frequent the shop, despite the increase in the price of ice cream sodas.']", "label": 1 }, { "id": "train_280", "context": "Certain messenger molecules fight damage to the lungs from noxious air by telling the muscle cells encircling the lungs' airways to contract. This partially seals off the lungs. An asthma attack occurs when the messenger molecules are activated unnecessarily, in response to harmless things like pollen or household dust.", "question": "Which of the following, if true, points to the most serious flaw of a plan to develop a medication that would prevent asthma attacks by blocking receipt of any messages sent by the messenger molecules referred to above?", "answers": "['Researchers do not yet know how the body produces the messenger molecules that trigger asthma attacks.', 'Such a medication would be a preventative only and would be unable to alleviate an asthma attack once it had started.', 'Such a medication would be unable to distinguish between messages triggered by pollen and household dust and messages triggered by noxious air.', 'Such a medication would not become available for several years, because of long lead times in both development and manufacture.']", "label": 2 }, { "id": "train_281", "context": "Certain bacteria that produce hydrogen sulfide as a waste product would die if directly exposed to oxygen. The hydrogen sulfide reacts with oxygen, removing it and so preventing it from harming the bacteria. Furthermore, the hydrogen sulfide tends to kill other organisms in the area, thereby providing the bacteria with a source of food. As a result, a dense colony of these bacteria produces for itself an environment in which it can continue to thrive indefinitely.", "question": "Which one of the following is most strongly supported by the information above?", "answers": "['The bacteria can continue to thrive indefinitely only in an environment in which the hydrogen sulfide they produce has removed all oxygen and killed other organisms in the area.', 'Most organisms, if killed by the hydrogen sulfide produced by the bacteria, can provide a source of food for the bacteria.', 'The hydrogen sulfide produced by the bacteria kills other organisms in the area by reacting with and removing oxygen.', 'A dense colony of the bacteria can indefinitely continue to produce enough hydrogen sulfide to kill other organisms in the area and to prevent oxygen from harming the bacteria.']", "label": 3 }, { "id": "train_282", "context": "Columnist: Until very recently, Presorbin and Veltrex, two medications used to block excess stomach acid, were both available only with a prescription written by a doctor. In an advertisement for Presorbin, its makers argue that Presorbin is superior on the grounds that doctors have written 200 million prescriptions for Presorbin, as compared to 100 million for Veltrex. It can be argued that the number of prescriptions written is never a worthwhile criterion for comparing the merits of medicines, but that the advertisement' s argument is absurd is quite adequately revealed by observing that Presorbin was available as a prescription medicine years before Veltrex was.", "question": "In the columnist's argument, the two highlighted portions play which of the following roles?", "answers": [ "The first states the main conclusion of the columnist's argument; the second states a conclusion that the columnist draws in defending that conclusion against an objection.", "The first identifies the conclusion of an argument that the columnist's argument is directed against; the second states the main conclusion of the columnist's argument.", "The first is a claim that the columnist's argument seeks to clarify; the second states a conclusion drawn about one possible interpretation of that claim.", "The first identifies an assumption made in an argument that the columnist's argument is directed against; the second states the main conclusion of the columnist's argument." ], "label": 1 }, { "id": "train_283", "context": "Politician: Homelessness is a serious social problem, but further government spending to provide low-income housing is not the cure for homelessness. The most cursory glance at the real-estate section of any major newspaper is enough to show that there is no lack of housing units available to rent. So the frequent claim that people are homeless because of a lack of available housing is wrong.", "question": "That homelessness is a serious social problem figures in the argument in which one of the following ways?", "answers": "['It summarizes a position the argument as a whole is directed toward discrediting.', 'It is compatible either with accepting the conclusion or with denying it.', 'It is required in order to establish the conclusion.', 'It sets out a problem the argument is designed to resolve.']", "label": 1 }, { "id": "train_284", "context": "Trent is a member of the SWAT Team, the most elite tactical unit at the city police department. SWAT apprehends more suspected criminals than all other police units combined. Taken as a whole, the police department solves a higher percentage of crime than ever before in its history. Within the SWAT team, Trent' s four-man unit is the most successful. However, the number of unsolved crime increases every year.", "question": "Which of the following statements, if true, most logically resolves the apparent paradox?", "answers": "[\"Trent's SWAT team is the city's best police unit.\", 'The total number of crimes increases every year.', 'Violent crime has decreased dramatically, while petty drug offenses have increased substantially.', 'The police department focuses more on crimes involving serious injury or significant property damage.']", "label": 1 }, { "id": "train_285", "context": "Footwear designer John de los Santos last year released several limited editions of his best-known model of sneaker in exotic colors and prints. Although the new releases were priced substantially higher than their counterparts in more traditional colors, they sold out within a week of their release, and have since been selling on the resale market for up to four times the original price. The cost of producing the sneakers in exotic prints is no greater than that of producing them in more traditional colors, so de los Santos could earn a higher profit per unit by producing a greater percentage of his sneakers in such prints.", "question": "Which of the following is an assumption made in drawing the conclusion above?", "answers": "[\"Consumers' willingness to pay higher prices for the exotic sneakers was not influenced by the relative scarcity of those sneakers.\", \"The workmanship of de los Santos's sneakers is of such high quality that it is impossible for lower-budget shoemakers to produce counterfeit versions of them.\", \"De los Santos's sneakers are not priced substantially higher than those of the designers who compete most directly with him.\", 'The designers who compete most directly with de los Santos will not produce similar lines of limited-edition sneakers in the near future.']", "label": 0 }, { "id": "train_286", "context": "Lawrence County has seven county run hospitals. Because three privately run hospitals are now in existence in the county, Lawrence has decided to close one of its seven county hospitals, to reduce costs to the county. The county executives want to choose a hospital to close so that its absence has the fewest negative health consequences for citizens of the county. An independent consulting firm, advising the county executives, has recommended closing the hospital with the highest fatality rate.", "question": "Which of the following, if true, most seriously calls into question the consulting firm's recommendation for the choosing the hospital to close?", "answers": "['The county hospital that is the most expensive for the county to operate is the one with the highest fatality rate.', 'Most patients who would have gone to the closed county hospital will go to one of the privately run hospitals, instead of one of the six remaining county hospitals, driving up costs at these places.', 'The most at-risk patients throughout the county are referred to those public hospitals that have a high concentration of trained specialists on staff.', 'Each one of the county hospitals has lower fatality rate than at least one of the privately run hospitals.']", "label": 2 }, { "id": "train_287", "context": "For a trade embargo against a particular country to succeed, a high degree of both international accord and ability to prevent goods from entering or leaving that country must be sustained. A total blockade of Patria's ports is necessary to an embargo, but such an action would be likely to cause international discord over the embargo.", "question": "The claims above, if true, most strongly support which of the following conclusions?", "answers": "['The balance of opinion is likely to favor Patria in the event of a blockade.', 'Any trade embargo against Patria would be likely to fail at some time.', \"A naval blockade of Patria's ports would ensure that no goods enter or leave Patria.\", \"For a blockade of Patria's ports to be successful, international opinion must be unanimous.\"]", "label": 1 }, { "id": "train_288", "context": "Most people are indignant at the suggestion that they are not reliable authorities about their real wants. Such self-knowledge, however, is not the easiest kind of knowledge to acquire. Indeed, acquiring it often requires hard and even potentially risky work. To avoid such effort, people unconsciously convince themselves that they want what society says they should want.", "question": "The main point of the argument is that", "answers": "['people cannot really want what they should want', 'knowledge of what one really wants is not as desirable as it is usually thought to be', 'people are not necessarily reliable authorities about what they really want', 'people usually avoid making difficult decisions']", "label": 2 }, { "id": "train_289", "context": "The primary task of a university is to educate. But to teach well, professors must be informed about new developments in their disciplines, and that requires research. Yet many universities cannot afford to support faculty research adequately. So a lack of funds for research adversely affects the degree to which a university can fulfill its central mission.", "question": "Which one of the following most accurately expresses the conclusion of the argument?", "answers": "['Lack of financial support for faculty research is the root of ineffective teaching at universities.', 'In order to be able to teach well, university professors must conduct research.', 'Effective teaching is the primary mission of a university.', 'Lack of funds for research reduces the quality of education a university provides.']", "label": 3 }, { "id": "train_290", "context": "Twenty years ago, Balzania put in place regulations requiring operators of surface mines to pay for the reclamation of mined-out land. Since then, reclamation technology has not improved. Yet, the average reclamation cost for a surface coal mine being reclaimed today is only four dollars per ton of coal that the mine produced, less than half what it cost to reclaim surface mines in the years immediately after the regulations took effect.", "question": "Which of the following, if true, most helps to account for the drop in reclamation costs described?", "answers": "['Even after Balzania began requiring surface mine operators to pay reclamation costs, coal mines in Balzania continued to be less expensive to operate than coal mines in almost any other country.', 'In the twenty years since the regulations took effect, the use of coal as a fuel has declined from the level it was at in the previous twenty years.', 'Even after Balzania began requiring surface mine operators to pay reclamation costs, surface mines continued to produce coal at a lower total cost than underground mines.', 'Mine operators have generally ceased surface mining in the mountainous areas of Balzania because reclamation costs per ton of coal produced are particularly high for mines in such areas.']", "label": 3 }, { "id": "train_291", "context": "DataCom, a company that filed many patents last year, was financially more successful last year than were its competitors, none of which filed many patents. It is ttherefore likely that DataCom owed its greater financial success to the fact that it filed many patents last year.", "question": "The argument is most vulnerable to criticism on the grounds that it", "answers": "['gives no reason to exclude the possibility that other differences between DataCom and its competitors accounted for its comparative financial success', 'applies a generalization to an exceptional case', \"presupposes what it sets out to demonstrate about the relationship between the financial success of DataCom's competitors and the number of patents they filed\", \"confuses a company's financial success with its technological innovativeness\"]", "label": 0 }, { "id": "train_292", "context": "Political analyst: Several years ago, McFarlane, the military dictator, had Brooks, the former prime minister, arrested on charges of corruption. After years of negotiation, McFarlane has pardoned Brooks, and she has agreed to join his government. Almost all of McFarlane' s supporters believe that Brooks is guilty of corruption. Moreover, almost all of McFarlane' s opponents will oppose anyone who agrees to join his government. So Brooks will have few supporters in this country.", "question": "The political analyst's argument depends on the assumption that", "answers": "[\"there is less corruption in the country's government now than when Brooks was prime minister\", 'most people in the country are either supporters or opponents of McFarlane', 'the charges on which Brooks was arrested were unfounded', \"Brooks's joining McFarlane's government inappropriately gives that government a semblance of legitimacy\"]", "label": 1 }, { "id": "train_293", "context": "Complaints that milk bottlers take enormous markups on the bottled milk sold to consumers are most likely to arise when least warranted by the actual spread between the price that bottlers pay for raw milk and the price at which they sell bottled milk. The complaints occur when the bottled-milk price rises, yet these price increases most often merely reflect the rising price of the raw milk that bottlers buy from dairy farmers. When the raw-milk price is rising, the bottlers' markups are actually smallest proportionate to the retail price. When the raw-milk price is falling, however, the markups are greatest.", "question": "If all of the statements above are true, which one of the following must also be true on the basis of them?", "answers": "['Consumers pay more for bottled milk when raw-milk prices are falling than when these prices are rising.', 'Consumers tend to complain more about the price they pay for bottled milk when dairy farmers are earning their smallest profits.', \"Increases in dairy farmers' cost of producing milk are generally not passed on to consumers.\", 'Milk bottlers generally do not respond to a decrease in raw-milk prices by straightaway proportionately lowering the price of the bottled milk they sell.']", "label": 3 }, { "id": "train_294", "context": "Which one of the following exhibits a pattern of reasoning most similar to that in the argument above?", "question": "If that insect is a bee, it can only sting once. It only did sting once. So it is a bee.", "answers": "['Only one more thunderstorm was needed to ruin that roof. But the roof was still fine a month later. There must not have been any thunderstorms over that month.', 'Old and brittle paintings are always moved with extreme care. That particular painting is never moved with extreme care. So it must not be old and brittle.', \"To survive in the wild requires physical stamina like Mark's. All the same, Mark's fear of spiders would prevent his survival.\", 'Spring is here. It has to be, because when it is spring, I cannot stop sneezing; and I just sneezed.']", "label": 3 }, { "id": "train_295", "context": "Chinh: Television producers should not pay attention to the preferences of the viewing public when making creative decisions. Great painters do not consider what the museum-going public wants to see. Lana: But television is expressly for the viewing public. So a producer is more like a CEO than like an artist. Just as a company would be foolhardy not to consider consumers' tastes when developing products, the TV producer must consider viewers' preferences.", "question": "According to Lana, Chinh's argument is flawed in that it", "answers": "['offers a faulty analogy', 'relies on a sample of consumers that is unrepresentative of consumers in general', 'is circular', 'fails to consider the possibility that painters may in fact try to please the museum-going public']", "label": 0 }, { "id": "train_296", "context": "A positive correlation has been found between the amount of soot in the atmosphere of cities and the frequency of a certain ailment among those cities' populations. However, the soot itself probably does not cause this ailment, since in cities where there are large amounts of soot in the air, there are usually also high concentrations of many other air pollutants.", "question": "Which one of the following statements, if true, most weakens the argument?", "answers": "[\"If high concentrations of many different pollutants in a city's air are correlated with a high frequency of the ailment among that city's population, then it is possible that two or more of those pollutants each causally contributes to the ailment.\", 'In cities in which there are high concentrations of many air pollutants, there are generally also high concentrations of other forms of pollution that are very likely to contribute causally to the ailment.', 'If the ailment rarely occurs except in cities in which there are large amounts of soot in the air, then the soot is probably the cause of the ailment.', 'In each of the cities where there are large amounts of soot in the air but little other air pollution, the frequency of the ailment is at least as high as it is anywhere else.']", "label": 3 }, { "id": "train_297", "context": "Public health expert: Increasing the urgency of a public health message may be counterproductive. In addition to irritating the majority who already behave responsibly, it may undermine all government pronouncements on health by convincing people that such messages are overly cautious. And there is no reason to believe that those who ignore measured voices will listen to shouting.", "question": "The two sections in boldface play which of the following roles in the public health expert's argument?", "answers": "[\"The first is the argument's main conclusion; the second supports that conclusion and is itself a conclusion for which support is provided.\", \"The first is the argument's only explicit conclusion; the second is a premise supporting that conclusion.\", 'The first is a premise supporting the only explicit conclusion; so is the second.', \"The first is a premise supporting the argument's only conclusion; the second is that conclusion.\"]", "label": 1 }, { "id": "train_298", "context": "In the last election, 89 percent of reporters voted for the incumbent. The content of news programs reveals that reporters allowed the personal biases reflected in this voting pattern to affect their news coverage: 54 percent of coverage concerning the challenger was negative, compared with only 30 percent of that concerning the incumbent.", "question": "The argument is logically most vulnerable to criticism on the grounds that it", "answers": "['ignores the possibility that there was more negative news worthy of reporting concerning the challenger than there was concerning the incumbent', 'ignores the possibility that reporters generally fear losing access to incumbents more than they fear losing access to challengers', \"ignores the possibility that the electorate's voting behavior is not significantly affected by the content of coverage of candidates\", 'presumes, without providing justification, that both candidates received equal amounts of coverage overall']", "label": 0 }, { "id": "train_299", "context": "Child-rearing Expert: Parents should never discipline their children in public. Some children who are disciplined in public suffer from attention deficit disorder. This shows that these children do not need discipline, but rather need assistance from their parents or firm instructions to help the children pay attention to their surroundings.", "question": "Each of the following describes a flaw in the child-rearing expert's reasoning EXCEPT:", "answers": "['It assumes that children with attention deficit disorder can pay attention to their surroundings without discipline.', 'It presumes, with justification, that children who are disciplined in private also suffer from attention deficit disorder.', 'It overlooks the possibility that the children who are disciplined in public, but do not suffer from attention deficit disorder, might benefit from being disciplined in public.', 'It neglects the possibility that providing firm instructions could be a form of disciplining children in public.']", "label": 1 }, { "id": "train_300", "context": "Every adult male woolly monkey is larger than even the largest female woolly monkey. In colonies of woolly monkeys, any adult male will dominate any female.", "question": "If the statements above are true, which one of the following must on the basis of them be true of woolly monkeys in colonies?", "answers": "['If a female woolly monkey dominates a male of the species, the dominated male monkey is not an adult.', 'Some large adolescent male woolly monkeys dominate some smaller females of the species.', 'Size is the primary determinant of relations of dominance among woolly monkeys.', 'If a male woolly monkey is larger than a female of the species, that male will dominate that female.']", "label": 0 }, { "id": "train_301", "context": "The corpus callosum -- the thick band of nerve fibers connecting the brain' s two hemispheres -- of a musician is on average larger than that of a nonmusician. The differences in the size of corpora callosa are particularly striking when adult musicians who began training around the age of seven are compared to adult nonmusicians. Ttherefore, musical training, particularly when it begins at a young age, causes certain anatomic brain changes.", "question": "Which one of the following is an assumption on which the argument depends?", "answers": "['Musical training late in life does not cause anatomic changes to the brain.', 'Adult nonmusicians did not participate in activities when they were children that would have stimulated any growth of the corpus callosum.', 'For any two musicians whose training began around the age of seven, their corpora callosa are approximately the same size.', 'The corpora callosa of musicians, before they started training, do not tend to be larger than those of nonmusicians of the same age.']", "label": 3 }, { "id": "train_302", "context": "It is now common for people to identify as gluten intolerant. In ancient societies, it was common for people to identify as poultry intolerant. This eventually ended when people realized they were misdiagnosing poultry intolerance with food poisoning caused by mistakes in the birds' preparation. Eventually, people will realize that they are not actually gluten sensitive.", "question": "The reference to the ancient civilization's poultry intolerance plays which of the following roles in the argument?", "answers": "['Serves as a historical example of food intolerance.', \"Ties the argument's reasoning together.\", 'Provides an example of how societies can misdiagnose food intolerance.', \"Distracts the reader from the argument's primary purpose.\"]", "label": 2 }, { "id": "train_303", "context": "The average cable television company offers its customers 50 channels, but new fiber-optic lines will enable telephone companies to provide 100 to 150 television channels to their customers for the same price as cable companies charge for 50. Ttherefore, cable companies will be displaced by the new television services offered by telephone companies within a few years.", "question": "Which one of the following, if true, most helps to strengthen the argument?", "answers": "['Some telephone companies own cable companies in areas other than those in which they provide telephone services.', 'The most popular movies and programs on channels carried by cable companies will also be offered on channels carried by the fiberoptic lines owned by the telephone companies.', 'Cable television companies will respond to competition from the telephone companies by increasing the number of channels they offer.', 'The initial cost per household of installing new fiber-optic television service will exceed the current cost of installing cable television service.']", "label": 1 }, { "id": "train_304", "context": "A new tax law aimed at encouraging the reforestation of cleared land in order to increase the amount of forested land in a particular region offers lumber companies tax incentives for each unit of cleared land they reforest. One lumber company has accordingly reduced its tax liability by purchasing a large tract of cleared land in the region and reforesting it. The company paid for the purchase by clearing a larger tract of land in the region, a tract that it had planned to hold in long-term reserve.", "question": "If the statements above are true, which one of the following must be true about the new tax law?", "answers": "['It can provide a motivation for companies to act in a manner contrary to the purpose of the law while taking advantage of the tax incentives.', 'It will have no immediate effect on the amount of forested land in the region.', 'It will ultimately cause lumber companies to plant trees on approximately as much land as they harvest in the region.', 'It will provide lumber companies with a tax incentive that will ultimately be responsible for a massive decrease in the number of mature forests in the region.']", "label": 0 }, { "id": "train_305", "context": "Political scientist: Some analysts point to the government' s acceptance of the recent protest rally as proof that the government supports freedom of popular expression. But the government supports no such thing. Supporting freedom of popular expression means accepting the expression of ideas that the government opposes as well as the expression of ideas that the government supports. The message of the protest rally was one that the government entirely supports.", "question": "Which one of the following is an assumption that is required by the political scientist's argument?", "answers": "['The government would not have accepted a protest rally whose message it opposed.', 'The message of the recent protest rally did not concern any function of the government.', 'The government feared a backlash if it did not show acceptance of the recent protest rally.', 'There are groups that are inhibited from staging a protest rally out of a fear of government response.']", "label": 0 }, { "id": "train_306", "context": "When a group of people starts a company, the founders usually serve as sources both of funding and of skills in marketing, management, and technical matters. It is unlikely that a single individual can both provide adequate funding and be skilled in marketing, management, and technical matters. Ttherefore, companies founded by groups are more likely to succeed than companies founded by individuals.", "question": "Which one of the following is an assumption required by the argument?", "answers": "['Some founding members of successful companies can provide both funding and skills in marketing, management, or technical matters.', 'Founders of a new company can more easily acquire marketing and management abilities than technical abilities.', 'New companies are more likely to succeed when their founders can provide adequate funding and skills in marketing, management, and technical abilities than if they must secure funding or skills from nonfounders.', \"A new company is more likely to succeed if every founding member contributes equally to the company's initial funding than if some members contribute more funds than others.\"]", "label": 2 }, { "id": "train_307", "context": "Someone' s benefiting from having done harm to another person is morally justifiable only if the person who was harmed knew that what was done could cause that harm but consented to its being done anyway.", "question": "Which one of the following judgments most closely conforms to the principle above?", "answers": "[\"Attempting to avoid being kept after school as punishment for breaking a window, Sonia falsely claimed that her brother had broken it; Sonia's action was morally unjustifiable since it resulted in both children being kept after school for something only Sonia had done.\", 'For James, who was convicted of having defrauded a large number of people out of their savings and wrote a book about his scheme while in prison, to be denied the profits from his book would be morally unjustifiable since he has already been punished for his crime.', \"Because Roger's mother suffered severe complications as a result of donating a kidney to him for a lifesaving kidney transplant, it was morally unjustifiable for Roger to receive the transplant, even though his mother, herself a doctor, had been eager for the transplant to be performed.\", 'Wesley, a doctor, persuaded Max to take part in a medical experiment in which a new drug was being tested; since Wesley failed to warn Max about the serious side effects of the drug and the drug proved to have no other effects, Wesley was morally unjustified in using the results obtained from Max in his report.']", "label": 3 }, { "id": "train_308", "context": "For centuries, mathematics seemed to exist in service to the natural sciences. Many of the great mathematical breakthroughs, like Newton' s Calculus, were spearheaded in the effort to solve scientific questions. In the nineteenth century, though, mathematicians made it clear that the primary object of mathematical study were the abstract creations of mathematics-numbers and shapes and functions and their more abstruse extensions-irrespective of whether this study has any bearing on scientific investigation. Ttherefore, mathematics is not properly one of the natural sciences.", "question": "Which of the following is an assumption that supports drawing the conclusion above from the reasons given for that conclusion?", "answers": "['The object of a natural science must be something tangible that can be perceived by the senses.', 'Some great scientists, like Einstein, by their own admission, were not very good at math.', 'Mathematics can also be used to serve social sciences, such as economics.', 'If one discipline serves another discipline, it can never rise to the same rank as the discipline it serves.']", "label": 0 }, { "id": "train_309", "context": "The Minoan civilization flourished on the island of Crete around 2000 B. (C) The discovery on Crete of large numbers of bronze implements and the furnaces used in their manufacture shows that the Minoans had a thriving bronze industry. Moreover, many bronze artifacts from this period that are similar in style to those produced on Crete have been found in southern Greece. Hence it is probable that, besides making bronzeware for domestic use, the Minoans exported bronzeware to southern Greece.", "question": "Which of the following, if true, most strengthens the argument given?", "answers": "['No furnaces dating from the Minoan era and suitable for the manufacture of bronze implements have been found in southern Greece.', 'Copper and tin, the main ingredients of bronze, were present in Southern Greece.', 'The Minoans exported a great deal of pottery, which was highly valued by other ancient civilizations.', 'Some of the pieces of bronzeware found on Crete were non-Minoan in origin.']", "label": 0 }, { "id": "train_310", "context": "Political scientist: The economies of a number of European countries are currently in severe difficulty. Germany is the only neighboring country that has the resources to resuscitate these economies. Ttherefore, Germany should begin aiding these economically troubled countries.", "question": "Which one of the following principles most helps to justify the political scientist's reasoning?", "answers": "['Any nation that alone has the capacity to economically resuscitate neighboring countries should exercise that capacity.', 'Any nation that can afford to give economic aid to just a few other nations ought to aid just those few.', 'Any nation that alone has an obligation to economically resuscitate neighboring countries ought to be the only nation to provide any economic aid.', 'Only nations that alone have the capacity to economically resuscitate neighboring countries should exercise that capacity.']", "label": 0 }, { "id": "train_311", "context": "Some students attending a small university with a well-known choir live off campus. From the fact that all music majors are members of the choir, a professor in the music department concluded that none of the students who live off campus is a music major.", "question": "The professor's conclusion is properly drawn if which one of the following is assumed?", "answers": "['None of the students who live off campus is a member of the choir.', 'All students who are members of the choir are music majors.', 'Some of the students who do not live off campus are not music majors.', 'All students who live on campus are music majors.']", "label": 0 }, { "id": "train_312", "context": "Consumers planning to buy recreational equipment tend to buy higher quality, more expensive equipment when the economy is strong than when it is weak. Hill and Dale is a business that sells high-quality, expensive camping and hiking equipment in Boravia. Although all the signs are that Boravia' s economy is now entering a period of sustained strength, the managers of the business do not expect a substantial increase in sales.", "question": "Which of the following, if true, would provide the strongest justification for the managers' judgment?", "answers": "[\"The economic upturn is likely to allow Boravia's national parks, where most of the camping and hiking is done, to receive extra funding to improve their visitor facilities.\", 'Advances in materials technology have led to the development of hiking and camping equipment that is more comfortable and lightweight than before.', 'In Boravia when the economy is strong, those who might otherwise go camping tend to take vacations overseas.', \"Many people in Boravia not only are committed to preserving the country's wilderness areas but also are interested in spending some time in them.\"]", "label": 2 }, { "id": "train_313", "context": "Most scholars agree that King Alfred (A. D. 849-899) personally translated a number of Latin texts into Old English. One historian contends that Alfred also personally penned his own law code, arguing that the numerous differences between the language of the law code and Alfred' s translations of Latin texts are outweighed by the even more numerous similarities. Linguistic similarities, however, are what one expects in texts from the same language, the same time, and the same region. Apart from Alfred' s surviving translations and law code, there are only two other extant works from the same dialect and milieu, so it is risky to assume here that linguistic similarities point to common authorship.", "question": "The passage above proceeds by", "answers": "['showing that the premises of another argument are mutually inconsistent', 'questioning the plausibility of an assumption on which another argument depends', \"providing examples that underscore another argument's conclusion\", 'showing that a principle if generally applied would have anomalous consequences']", "label": 1 }, { "id": "train_314", "context": "Journalists agree universally that lying is absolutely taboo. Yet, while many reporters claim that spoken words ought to be quoted verbatim, many others believe that tightening a quote from a person who is interviewed is legitimate on grounds that the speaker' s remarks would have been more concise if the speaker had written them instead. Also, many reporters believe that, to expose wrongdoing, failing to identify oneself as a reporter is permissible, while others condemn such behavior as a type of lying.", "question": "Which one of the following is most supported by the information above?", "answers": "['Since lying is permissible in some situations, reporters are mistaken to think that it is absolutely taboo.', 'Reporters disagree on what sort of behavior qualifies as lying.', 'There is no correct answer to the question of whether lying in a given situation is right or wrong.', 'Omission of the truth is the same thing as lying.']", "label": 1 }, { "id": "train_315", "context": "News item: The result of a recent public survey has been called into question because one of the pollsters admitted to falsifying data. The survey originally concluded that most people in the country favor investing more money in information technologies. Because falsified data were included in the survey, its conclusion is not true; a majority does not favor more investment in information technologies.", "question": "The news item's argument is vulnerable to criticism because it fails to consider the possibility that", "answers": "[\"people's opinions about investing in information technologies can change as new technologies are developed\", 'some people who responded to the survey were lying', 'the conclusion of the survey will be accepted by the public even though falsified data were used', 'the conclusion of the survey would be verified if the falsified data were excluded']", "label": 3 }, { "id": "train_316", "context": "To suit the needs of corporate clients, advertising agencies have successfully modified a strategy originally developed for political campaigns. This strategy aims to provide clients with free publicity and air time by designing an advertising campaign that is controversial, thus drawing prime-time media coverage and evoking public comment by officials.", "question": "The statements above, if true, most seriously undermine which one of the following assertions?", "answers": "['Campaign managers have transformed political campaigns by making increasing use of strategies borrowed from corporate advertising campaigns.', 'Only a small percentage of eligible voters admit to being influenced by advertising campaigns in deciding how to vote.', 'Advertising agencies that specialize in campaigns for corporate clients are not usually chosen for political campaigns.', \"The usefulness of an advertising campaign is based solely on the degree to which the campaign's advertisements persuade their audiences.\"]", "label": 3 }, { "id": "train_317", "context": "Pacifist: It is immoral to do anything that causes harm to another person. But, since using force causes harm to another person, it is also immoral to threaten to use force, even when such a threat is made in self-defense.", "question": "Which one of the following principles, if valid, would most help to justify the pacifist's reasoning?", "answers": "['Given the potential harm caused by the use of force, the line between use of force in self-defense and the aggressive use of force is always vague.', 'Whether a threat made in self-defense is immoral depends on the circumstances.', 'It is immoral to threaten to do what it is immoral to do.', 'It is immoral to carry out a threat if making the threat is itself immoral.']", "label": 2 }, { "id": "train_318", "context": "Trustee: The recent exhibit at the art museum was extensively covered by the local media, and this coverage seems to have contributed to the record-breaking attendance it drew. If the attendance at the exhibit had been low, the museum would have gone bankrupt and closed permanently, so the museum could not have remained open had it not been for the coverage from the local media.", "question": "The reasoning in the trustee's argument is most vulnerable to criticism on the grounds that the argument", "answers": "['presupposes the very conclusion that it is trying to prove', 'takes for granted that most people who read articles about the exhibit also attended the exhibit', 'fails to address the possibility that the exhibit would have drawn enough visitors to prevent bankruptcy even without media coverage', 'takes for granted that no previous exhibit at the museum had received such extensive media coverage']", "label": 2 }, { "id": "train_319", "context": "Five thousand of the 50, 000 books published in country Z in 1991 were novels. Exactly 25 of the films released in country Z in 1992 were based on those novels. Since 100 films were released in country Z in 1992, no more than one-quarter of them were based on books published in country Z in 1991.", "question": "Which one of the following, if assumed, allows the conclusion above to be properly drawn?", "answers": "['Some of the films released in 1991 in country Z were based on novels that were published in 1991.', 'None of the films released in country Z in 1992 were based on books other than novels.', 'None of the books that were published in country Z in 1992 were based on plots of films released in 1991.', 'Some of the films released in country Z in 1992 were based on older films that had been released for the first time many years earlier.']", "label": 1 }, { "id": "train_320", "context": "The following exchange occurred after the Baseball Coach' s team suffered a heartbreaking loss in the final inning. Reporter: The team clearly did not rise to the challenge. I' m sure that getting zero hits in twenty at-bats with runners in scoring position hurt the team' s chances at winning the game. What are your thoughts on this devastating loss? Baseball Coach: Hitting with runners in scoring position was not the reason we lost this game. We made numerous errors in the field, and our pitchers gave out too many free passes. Also, we did not even need a hit with runners in scoring position. Many of those at-bats could have driven in the run by simply making contact. Our team did not deserve to win the game.", "question": "Which of the following best describes the main point of dispute between the reporter and baseball coach?", "answers": "['Whether the loss was heartbreaking.', 'Whether the team rose to the challenge.', 'Numerous errors in the field and pitchers giving too many free passes caused the loss.', 'Whether getting zero hits in twenty at-bats with runners in scoring position caused the loss.']", "label": 3 }, { "id": "train_321", "context": "On the basis of incontestable proof that car safety seats will greatly reduce the number of serious injuries sustained by children in car accidents, laws have been passed mandating the use of these seats. Unexpectedly, it has since been found that a large number of children who are riding in safety seats continue to receive serious injuries that safety seats were specifically designed to avoid, and in the prevention of which they in fact have proven to be effective.", "question": "Which one of the following, if true, could by itself adequately explain the unexpected finding reported in the passage?", "answers": "['The protection afforded by child safety seats depends on their being used properly, which many parents fail to do.', 'The high cost of child safety seats has caused many parents to delay purchasing them.', 'Many parents are defying the law by not using safety seats for their children.', 'Children are more likely to make automobile trips now than they were before the introduction of the safety seat.']", "label": 0 }, { "id": "train_322", "context": "Newscaster: In order for the public to participate in a meaningful way in the current public policy debate, one requirement is that the issues be stated in terms the public can understand. The mayor' s speech has just stated these issues in such terms, so now the public at least might be able to participate in a meaningful way in the current public policy debate.", "question": "Which one of the following most closely parallels the newscaster's argument in its reasoning?", "answers": "['In order to discover the meaning of certain seldom-used words, one must use a good dictionary. Paul has used a good dictionary, so Paul must have discovered the meaning of those words.', 'One must at least have warm clothing if one is to survive in a very cold climate. Jerome has obtained warm clothing; ttherefore, he might be able to survive in a very cold climate.', \"One cannot confuse the majority of one's students if one wants to be a good teacher. Hugo wants to be a good teacher; ttherefore, he might be able to avoid confusing the majority of his students.\", \"In order to reach one's goals, one must be able to consider these goals carefully. Laura has reached her goals, so she must have been able to consider those goals carefully.\"]", "label": 1 }, { "id": "train_323", "context": "Paleontologists have discovered fossils of centipedes that are 414 million years old. These fossils are at least 20 million years older than the earliest land-dwelling animals previously identified. The paleontologists are confident that these centipedes lived on land, even though the fossilized centipedes were discovered in rock that also contained fossilized remains of animals known to be water-dwelling.", "question": "The paleontologists' view would be LEAST supported by the truth of which one of the following?", "answers": "['Fossils of spiders with respiratory systems adapted only to breathing air were found in the same rock as the centipede fossils.', 'The legs of the fossilized centipedes were particularly suited to being a means of locomotion on land.', 'Fossils of the earliest land-dwelling animals that had previously been identified were found in rock that did not contain fossilized remains of water-dwelling animals.', 'All of the centipedes that had previously been discovered were land dwellers.']", "label": 2 }, { "id": "train_324", "context": "Montgomery, a biologist who is also well read in archaeology, has recently written a book on the origin and purpose of ancient monumental architecture. This book has received much positive attention in the popular press but has been severely criticized by many professional archaeologists for being too extreme. Montgomery' s views do not deserve a negative appraisal, however, since those views are no more extreme than the views of some professional archaeologists.", "question": "The argument is most vulnerable to which one of the following criticisms?", "answers": "[\"It fails to establish that professional archaeologists' views that are at least as extreme as Montgomery's views do not deserve negative appraisal for that reason.\", \"It provides no independent evidence to show that the majority of professional archaeologists do not support Montgomery's views.\", 'It attempts to support its position by calling into question the motives of anyone who supports an opposing position.', 'It assumes without warrant that many professional archaeologists consider biologists unqualified to discuss ancient architecture.']", "label": 0 }, { "id": "train_325", "context": "All actors are exuberant people and all exuberant people are extroverts, but nevertheless it is true that some shy people are actors.", "question": "If the statements above are true, each of the following must also be true EXCEPT:", "answers": "['Some exuberant people who are actors are shy.', 'All people who are not extroverts are not actors.', 'Some shy extroverts are not actors.', 'Some shy people are extroverts.']", "label": 2 }, { "id": "train_326", "context": "Until he was dismissed amid great controversy, Hastings was considered one of the greatest intelligence agents of all time. It is clear that if his dismissal was justified, then Hastings was either incompetent or else disloyal. Soon after the dismissal, however, it was shown that he had never been incompetent. Thus, one is forced to conclude that Hastings must have been disloyal.", "question": "Which one of the following states an assumption upon which the argument depends?", "answers": "['If someone was disloyal or incompetent, then his dismissal was justified.', \"Hastings's dismissal was justified.\", 'The dismissal of anyone who was disloyal would be justified.', 'Anyone whose dismissal was justified was disloyal.']", "label": 1 }, { "id": "train_327", "context": "Concerned citizen: The mayor, an outspoken critic of the proposed restoration of city hall, is right when he notes that the building is outdated, but that the restoration would be expensive at a time when the budget is already tight. We cannot afford such a luxury item in this time of financial restraint, he says. However, I respectfully disagree. The building provides the last remaining link to the days of the city' s founding, and preserving a sense of municipal history is crucial to maintaining respect for our city government and its authority.", "question": "So to the question, \"Can we really afford to? \"I can only respond, \"Can we afford not to? \" Which one of the following most accurately characterizes a flaw in the concerned citizen's argument?", "answers": "['The argument ambiguously uses the word \"afford. \"', 'The argument is solely an emotional appeal to history.', 'The argument inappropriately appeals to the authority of the mayor.', 'The argument inappropriately relies on the emotional connotations of words such as \"outdated\" and \"luxury. \"']", "label": 0 }, { "id": "train_328", "context": "In a study of the relationship between aggression and television viewing in nursery school children, many interesting interactions among family styles, aggression, and television viewing were found. High aggression occurred in both high-viewing and low-viewing children and this seemed to be related to parental lifestyle. High-achieving, competitive, middle-class parents, whose children did not watch much television, had more aggressive children than parents who planned their lives in an organized, child-centered way, which included larger amounts of television viewing.", "question": "Which one of the following conclusions is best supported by the passage?", "answers": "['If high-achieving, competitive, parents were more child-centered, their children would be less aggressive.', 'Low levels of television viewing often lead to high levels of aggression among children.', 'The level of aggression of a child cannot be predicted from levels of television viewing alone.', 'Parental lifestyle is less important than the amount of television viewing in determining the aggressiveness of children.']", "label": 2 }, { "id": "train_329", "context": "Humans' emotional tendencies are essentially unchanged from those of the earliest members of our species. Accordingly, although technology makes possible a wider range of individual and societal choices than in centuries past, humans are generally unable to choose more wisely.", "question": "The argument depends on assuming which one of the following?", "answers": "['Regardless of the range of choices available to humans, they choose on the basis of their emotions alone.', 'Humans have undergone no significant changes since the origin of the species.', \"Humans would now be able to make wiser choices than in centuries past only if an essential change had taken place in humans' emotional dispositions.\", 'Humans who make wise choices are generally in control of their emotions.']", "label": 2 }, { "id": "train_330", "context": "Economics professor: Marty' s Pizza and Checkers Pizza are the two major pizza parlors in our town. Marty' s sold coupon books including coupons good for one large plain pizza at any local pizza parlor, at Marty' s expense. But Checkers refused to accept these coupons, even though they were redeemed by all other local pizza parlors. Accepting them would have cost Checkers nothing and would have satisfied those of its potential customers who had purchased the coupon books. This shows that Checkers' s motive in refusing to accept the coupons was simply to hurt Marty' s Pizza.", "question": "Which one of the following, if assumed, enables the economics professor's conclusion to be properly drawn?", "answers": "['At least one company has refused to accept coupons issued by its major local competitor simply in order to hurt that competitor, even though those coupons were accepted by all other local competitors.', \"Any company that accepts its major competitor's coupons helps its competitor by doing so, even if it also satisfies its own actual or potential customers.\", 'Any company that wishes to hurt a competitor by refusing to accept coupons issued by that competitor will refuse to accept them even when accepting them would cost nothing and would satisfy its potential customers.', \"Any company that refuses to accept coupons issued by a competitor when doing so would satisfy some of the company's potential customers is motivated solely by the desire to hurt that competitor.\"]", "label": 3 }, { "id": "train_331", "context": "In recent years, many cabinetmakers have been winning acclaim as artists. But since furniture must be useful, cabinetmakers must exercise their craft with an eye to the practical utility of their product. For this reason, cabinetmaking is not art.", "question": "Which of the following is an assumption that supports drawing the conclusion above from the reason given for that conclusion?", "answers": "['Some furniture is made to be placed in museums, where it will not be used by anyone.', 'Artists are not concerned with the monetary value of their products.', 'Cabinetmakers should be more concerned with the practical utility of their products than they currently are.', \"An object is not an art object if its maker pays attention to the object's practical utility.\"]", "label": 3 }, { "id": "train_332", "context": "Undoubtedly, one' s freedom is always worth the risk of losing one' s life. Consider a person who is locked in a bare cement room with no hope of escape. This person is not really living and has nothing to lose.", "question": "A flaw in the argument's reasoning is that the argument", "answers": "['generalizes inappropriately from a single extreme case to a universal claim', \"presumes, without providing justification, that nothing can have greater value than one's own freedom\", \"fails to consider that it is not always possible to rebel physically against an encroachment on one's freedom\", 'fails to establish that the freedom of others is worth taking risks for']", "label": 0 }, { "id": "train_333", "context": "If one does not have enough information to make a well-informed decision, one should not make a decision solely on the basis of the information one does possess. Instead, one should continue to seek information until a well-informed decision can be made.", "question": "Of the following, which one most closely conforms to the principle stated above?", "answers": "['When there is not enough information available to determine the meaning of a line of poetry, one should not form an opinion based on the insufficient information. Instead, one should simply acknowledge that it is impossible to determine what the line means.', 'Though a researcher may know a great deal about a topic, she or he should not assume that all information relevant to the research is already in her or his possession. A good researcher always looks for further relevant information.', 'When deciding which career to pursue, one needs to consider carefully all of the information one has. One should not choose a career solely on the basis of financial compensation; instead, one should consider other factors such as how likely one is to succeed at the career and how much one would enjoy it.', \"When one wants to buy a reliable car, one should not choose which car to buy just on the inadequate basis of one's personal experience with cars. Rather, one should study various models' reliability histories that summarize many owners' experiences.\"]", "label": 3 }, { "id": "train_334", "context": "After its customers complained about being pressured to buy unneeded insurance, an insurance agency stopped rewarding its agents for high sales volume and instead gave them bonuses for high levels of customer satisfaction. Under this new plan, both customer satisfaction and the insurance agency' s sales increased.", "question": "Each of the following, if true, helps to explain how the change in incentives for agents could have resulted in increased sales EXCEPT:", "answers": "['Agents listened more closely to customers of long standing and were able to sell them additional insurance policies that met new needs.', 'Dissatisfied customers of other agencies, attracted by the reports of the change in agency policy, became customers of the agency.', \"Having come to trust the increased judiciousness of the agents' recommendations, customers approached the agency to discuss and ultimately to buy more supplementary insurance than they previously had bought under pressure.\", 'Agents more frequently postponed completing the attendant paperwork even after the terms for an insurance policy were settled to the satisfaction of the client.']", "label": 3 }, { "id": "train_335", "context": "Chemical fertilizers not only create potential health hazards, they also destroy earthworms, which are highly beneficial to soil. For this reason alone the use of chemical fertilizers should be avoided. The castings earthworms leave behind are much richer than the soil they ingest, thus making a garden rich in earthworms much more fertile than a garden without them.", "question": "Which one of the following most accurately expresses the main conclusion of the argument?", "answers": "['The castings that earthworms leave behind are much richer than the soil they ingest.', 'Chemical fertilizers destroy earthworms.', 'The use of chemical fertilizers should be avoided.', 'Earthworms are highly beneficial to soil.']", "label": 2 }, { "id": "train_336", "context": "Business ethicist: Managers of corporations have an obligation to serve shareholders as the shareholders would want to be served. Ttherefore, corporate managers have an obligation to act in the shareholders' best interest.", "question": "The business ethicist's conclusion follows logically if which one of the following is assumed?", "answers": "['Shareholders would want to be served only in ways that are in their own best interest.', 'Corporate managers are always able to discern what is in the best interest of shareholders.', 'All shareholders want to be served in identical ways.', 'The shareholders have interests that can best be served by corporate managers.']", "label": 0 }, { "id": "train_337", "context": "Appendicitis (inflammation of the appendix) is potentially fatal; consequently, patients with symptoms strongly suggesting appendicitis almost always have their appendix removed. The appropriate surgery is low-risk but performed unnecessarily in about 20 percent of all cases. A newly developed internal scan for appendicitis is highly accurate, producing two misdiagnoses for every 98 correct diagnoses. Clearly, using this test, doctors can largely avoid unnecessary removals of the appendix without, however, performing any fewer necessary ones than before, since __.", "question": "Which of the following most logically completes the passage?", "answers": "['all of the patients who are diagnosed with this test as having appendicitis do, in fact, have appendicitis', 'the misdiagnoses produced by this test are always instances of attributing appendicitis to someone who does not, in fact, have it', 'every patient who is diagnosed with this test as having appendicitis has more than one of the symptoms generally associated with appendicitis', 'the only patients who are misdiagnosed using this test are patients who lack one or more of the symptoms that are generally associated with appendicitis']", "label": 1 }, { "id": "train_338", "context": "Professor Riley characterized the university president' s speech as inflammatory and argued that it was ttherefore inappropriate. However, Riley has had a long-standing feud with the president, and so we should not conclude that her speech was inflammatory solely on the basis of Riley' s testimony. Ttherefore, unless there are independent reasons to deem the president' s speech inflammatory, it is not true that her speech was inappropriate.", "question": "The argument is flawed in that it", "answers": "['fails to adequately address the possibility that inflammatory speeches may be appropriate for some audiences', \"fails to adequately address the possibility that Riley's animosity toward the university president is well founded\", \"concludes that Riley's claim is false merely on the grounds that Riley has something to gain if the claim is accepted as true\", 'takes for granted that the speech could not be inappropriate if it was not inflammatory']", "label": 3 }, { "id": "train_339", "context": "Historian: During the Industrial Revolution, for the first time in history, the productivity of the economy grew at a faster rate than the population and thus dramatically improved living standards. An economist theorizes that this growth was made possible by the spread of values such as hard work and thrift. But successful explanations need to be based on facts, so no one should accept this explanation until historical evidence demonstrates that a change in values occurred prior to the Industrial Revolution.", "question": "The overall conclusion of the historian's argument is that", "answers": "[\"no one should accept the economist's explanation until historical evidence demonstrates that a change in values occurred prior to the Industrial Revolution\", 'during the Industrial Revolution the productivity of the economy grew at a faster rate than the population', 'the improvement in living standards that occurred during the Industrial Revolution was not due to the spread of a change in values', 'values such as hard work and thrift did not become widespread prior to the Industrial Revolution']", "label": 0 }, { "id": "train_340", "context": "No one wants this job as much as Joshua does, but he is not applying for it. It follows that there will not be any applicants, no matter how high the salary that is being offered.", "question": "The flawed reasoning in the argument above most closely parallels that in which one of the following?", "answers": "[\"Although he has the strongest motive of anyone for buying Anna's plot of land, Manfred is not pursuing the matter. Ttherefore, regardless of how low a price Anna is prepared to accept, she will be looking for a buyer in vain.\", 'Beth knows better than anyone else how to spot errors in a computer program, yet even she has not found any in this program so far. So it is clear that the errors must all be in the rest of the program.', 'Three times Paul would have liked to take advantage of a special vacation package for himself and his family, but each time he was indispensable at the factory just then. So the more seniority Paul acquires, the greater are the constraints on his personal life.', 'If anyone can decipher this inscription, it is Professor Alvarez, but she is so involved with her new research that it will be impossible to interest her in this sort of task. Ttherefore, all we can do now is hope to find someone else.']", "label": 0 }, { "id": "train_341", "context": "Over the last thousand years, plant species native to islands have gone extinct at a much faster rate than have those native to mainland regions. Biologists believe that this is because island plants have not adapted the defenses against being eaten by large land mammals that mainland plants have. Ordinarily, populations of large land mammals are not established on islands until after the island is colonized by humans. Which one of the following.", "question": "if true, most strongly supports the biologist's explanation cited above?", "answers": "['Large land mammals tend to prefer plants from species native to mainland regions over plants from species native to islands.', 'The rate of extinction of native plant species on an island tends to increase dramatically after human colonization.', 'Many plant species that are not native to islands have become very well established on islands throughout the world.', 'Most of the plant species in the world that have not yet gone extinct are native to mainland regions.']", "label": 1 }, { "id": "train_342", "context": "Scientists have recently discovered that, in doses massive enough to kill cells, almost any chemical is carcinogenic because cell death causes rapid division among surviving cells, promoting cancer-causing mutations. A few chemicals are also carcinogenic without causing cell death. Until now studies of the carcinogenicity of food additives have always involved administering to test animals doses of additives larger than the largest possible human exposure levels and massive enough to kill large numbers of cells in the animals, and then observing the animals' cancer rates.", "question": "If the statements above are true, which one of the following conclusions is most strongly supported by them?", "answers": "['Carcinogenic chemicals are usually absorbed in small doses in the natural environment.', 'Most of the food additives that are now banned because of carcinogenicity could safely be used in human foods.', 'In the doses heretofore used in carcinogenicity studies of food additives, cell death often does not occur in test animals.', 'Until now results of carcinogenicity studies encouraged overestimation of the degree to which some food additives are carcinogenic.']", "label": 3 }, { "id": "train_343", "context": "Commentator: Because of teacher hiring freezes, the quality of education in that country will not improve. Thus, it will surely deteriorate.", "question": "The flawed reasoning in which one of the following is most similar to that in the commentator's argument?", "answers": "['Since funding levels for social programs are being frozen, our society will not become more harmonious. Thus, it may become more discordant.', 'Given that over 250 years of attempts to prove the Goldbach conjecture have failed, it will probably never be proved. Hence, it is more likely to be disproved than proved.', 'Because Raoul is a vegetarian, he will not have the pepperoni pizza for lunch. It follows that he will have the cheese pizza.', 'Since there is a storm moving in, the outside temperature cannot rise this afternoon. Ttherefore, it must fall.']", "label": 3 }, { "id": "train_344", "context": "Kernland imposes a high tariff on the export of unprocessed cashew nuts in order to ensure that the nuts are sold to domestic processing plants. If the tariff were lifted and unprocessed cashews were sold at world market prices, more farmers could profit by growing cashews. However, since all the processing plants are in urban areas, removing the tariff would seriously hamper the government' s effort to reduce urban unemployment over the next five years.", "question": "Which of the following, if true, most seriously weakens the argument?", "answers": "['Buying unprocessed cashews at lower than world market prices enables cashew processors in Kernland to sell processed nuts at competitive prices.', 'A lack of profitable crops is driving an increasing number of small farmers in Kernland off their land and into the cities.', 'Other countries in which cashews are processed subsidize their processing plants.', 'Some of the by-products of processing cashews are used for manufacturing paints and plastics.']", "label": 1 }, { "id": "train_345", "context": "The children in the second grade at a local elementary school scored lower on tests of math skills this year than the second graders did last year. The school board blames the disappointing results on the budget cuts that forced the school to consolidate several classes of second-grade students, thus depriving them of the individualized attention that they had previously enjoyed.", "question": "Which of the following, if true, best supports the explanation offered by the school board for the students' lower test scores?", "answers": "['The test that the second graders took this year was not more difficult than the test that the second graders took last year.', 'Students at other elementary schools in the area also scored poorly on the same test of math skills this year.', 'No new textbooks were purchased by the school board this year.', 'Other schools in the area also experienced severe budget cuts.']", "label": 0 }, { "id": "train_346", "context": "The true scientific significance of a group of unusual fossils discovered by the paleontologist Charles Walcott is more likely to be reflected in a recent classification than it was in Walcott' s own classification. Walcott was, after all, a prominent member of the scientific establishment. His classifications are thus unlikely to have done anything but confirm what established science had already taken to be true.", "question": "Which one of the following most accurately describes a questionable technique used in the argument?", "answers": "['It analyzes the past on the basis of social and political categories that properly apply only to the present and uses the results of this analysis to support its conclusion.', 'It cites two pieces of evidence, each of which is both questionable and unverifiable, and uses this evidence to support its conclusions.', \"It draws conclusions about the merit of a position and about the content of that position from evidence about the position's source.\", 'It bases a conclusion on two premises that contradict each other and minimizes this contradiction by the vagueness of the terms employed.']", "label": 2 }, { "id": "train_347", "context": "As many as 98, 000 people die each year due to medical error. In a campaign to reduce lethal errors, thousands of hospitals introduced six key changes, including rapid-response teams, re-checks of patient medication, and new guidelines for preventing infection. The campaign estimated that, over an 18-month period, more than 100, 000 lives were saved as a direct result of the program.", "question": "Which of the following can be most properly inferred from the above statements?", "answers": "['If the campaign had not been implemented, more than 100, 000 people might have died during the 18-month period due to medical error.', 'The key changes initiated by the campaign will continue to be implemented in the future.', 'In the future, no one will die because of medical error.', 'The campaign saved all of the people who otherwise would have died due to medical error in that time period.']", "label": 0 }, { "id": "train_348", "context": "The elementary schools in district X began a new athletic program where students participated in school-sponsored sports at least three times per week. During this last school year, the number of obese students attending elementary schools in district X decreased substantially. Based on these figures, the district' s superintendent decided that the new athletic program was more effectual in keeping children, at least while in elementary school, from being obese.", "question": "All of the following, if true, weaken the superintendent's conclusion EXCEPT:", "answers": "['The government, during this last school year, began requiring monthly weigh-ins for obese children and parental counseling about proper nutrition.', \"A large number of the obese children's conditions were caused by depression, which was greatly alleviated by installing psychiatrists in each elementary school in district X.\", 'Obesity, like other unhealthy conditions, can lead to decreased cardiovascular health, increased joint pain, and self-esteem problems.', 'Over the last year, parents with obese children in district X began sending their children to special weekend camps designed to help children lose weight.']", "label": 2 }, { "id": "train_349", "context": "The traditional view of the Roman emperor Caligula as a cruel and insane tyrant has been challenged by some modern historians. They point out that little documentation of Caligula' s alleged cruelty or outrageous behavior survives from the time of his reign and that the histories that have come down to us were written by his enemies.", "question": "Which one of the following, if true, adds the most support for the challenge from the modem historians?", "answers": "['People who have lived under someone regarded as a cruel tyrant are more likely to view that person unfavorably than favorably.', 'The specific outrageous acts attributed to Caligula in Roman documentation are very similar to acts attributed in earlier writings to other rulers alleged to be cruel tyrants.', \"There is less documentation of any sort from Caligula's reign than from the reigns of most other Roman emperors of Caligula's era.\", 'There is ample documentation of modern tyrants being responsible for outrageous acts worse than those attributed to Caligula.']", "label": 1 }, { "id": "train_350", "context": "In the past the country of Siduria has relied heavily on imported oil. Siduria recently implemented a program to convert heating systems from oil to natural gas. Siduria already produces more natural gas each year than it burns, and oil production in Sidurian oil fields is increasing at a steady pace. If these trends in fuel production and usage continue, ttherefore, Sidurian reliance on foreign sources for fuel should decline soon.", "question": "Which of the following is an assumption on which the argument depends?", "answers": "['Buildings cannot be heated by solar energy rather than by oil or natural gas.', 'No fuel other than natural gas is expected to be used as a replacement for oil in Siduria.', 'In Siduria the rate of fuel consumption is rising no more quickly than the rate of fuel production.', 'All new homes that are being built will have natural-gas-burning heating systems.']", "label": 2 }, { "id": "train_351", "context": "Biologists agree that human beings evolved from a fish, but they disagree about which species of fish. Since biologists agree that frogs are definitely related to the species of fish from which human beings evolved, on the basis of a close match between the mitochondrial DNA of lungfish and that of frogs Dr. Stevens-Hoyt claims that this ancestor must be lungfish. Dr. Grover, on the other hand, contends that mitochondrial DNA evolves too rapidly to be a reliable indicator of relationships between species over long periods of time, and citing the close chemical match between the hemoglobin of coelacanths (a saltwater fish) and that of tadpoles, claims that human beings must be descended from coelacanths.", "question": "Which one of the following most accurately describes the role played in the dispute above by the proposition that frogs are definitely related to the species of fish from which human beings evolved?", "answers": "['It is offered as evidence for the contention that human beings must be descended from either lungfish or coelacanths.', 'Since it implies that human beings are not descended from lungfish, it is cited as evidence against the claim that humans are descended from lungfish.', 'It is an assumption that both parties to the dispute use as a starting point for their arguments about human evolution.', 'It implies that either a match of mitochondrial DNA or a match of hemoglobin between lungfish and coelacanths would show that human beings evolved from one of these two species.']", "label": 2 }, { "id": "train_352", "context": "It is well documented that people have positive responses to some words, such as \"kind\" and \"wonderful, \" and negative responses to others, such as \"evil\" and \"nausea. \" Recently, psychological experiments have revealed that people also have positive or negative responses to many nonsense words. This shows that people' s responses to words are conditioned not only by what the words mean, but also by how they sound.", "question": "The claim that people have positive or negative responses to many nonsense words plays which one of the following roles in the argument?", "answers": "[\"It is a conclusion for which the only support provided is the claim that people's responses to words are conditioned both by what the words mean and by how they sound.\", 'It is a generalization partially supported by the claim that meaningful words can trigger positive or negative responses in people.', \"It is a premise offered in support of the conclusion that people's responses to words are engendered not only by what the words mean, but also by how they sound.\", 'It is a premise offered in support of the conclusion that people have either a positive or a negative response to any word.']", "label": 2 }, { "id": "train_353", "context": "Because of strong overall stylistic similarities, prehistoric cave paintings in El Castillo, Spain and others in Altamira, Spain, were thought to date from the same period. Recent pigment analyses, however, established the Altamira paintings as by far the older and as about the same age as a third group of paintings done in a different overall style. Nevertheless, this case does not prove that stylistic similarities cannot be a reliable indicator of closeness in age, since __.", "question": "Which of the following most logically completes the passage?", "answers": "['Determining the age of a prehistoric cave painting simply by reference to the age of the pigments used in that painting substitutes technology for artistic judgment', 'Experts who knew the results of the pigment analyses reexamined these paintings and found no previously unnoticed stylistic similarities between the Altamira paintings and the paintings in the third group', 'There are experts who maintain that the results of the recent pigment analyses leave the striking stylistic similarities between the El Castillo paintings and the Altamira paintings unexplained', 'There are certain stylistic features that the Altamira paintings share with all known paintings of their period and do not share with the El Castillo paintings']", "label": 3 }, { "id": "train_354", "context": "Researchers have found that when very overweight people, who tend to have relatively low metabolic rates, lose weight primarily through dieting, their metabolisms generally remain unchanged. They will thus burn significantly fewer calories at the new weight than do people whose weight is normally at that level. Such newly thin persons will, ttherefore, ultimately regain weight until their body size again matches their metabolic rate.", "question": "The conclusion of the argument above depends on which of the following assumptions?", "answers": "['The amount of calories that a person usually burns in a day is determined more by the amount that is consumed that day than by the current weight of the individual.', 'The metabolisms of people who are usually not overweight are much more able to vary than the metabolisms of people who have been very overweight.', 'Researchers have not yet determined whether the metabolic rates of formerly very overweight individuals can be accelerated by means of chemical agents.', 'Relatively few very overweight people who have dieted down to a new weight tend to continue to consume substantially fewer calories than do people whose normal weight is at that level.']", "label": 3 }, { "id": "train_355", "context": "Advertisement: In today' s world, you make a statement about the person you are by the car you own. The message of the SKX Mach-5 is unambiguous: Its owner is Dynamic, Aggressive, and Successful.", "question": "Shouldn't you own an SKX Mach-5? If the claims made in the advertisement are true, which one of the following must also be true on the basis of them?", "answers": "['Anyone who is not both dynamic and successful would misrepresent himself or herself by being the owner of an SKX Mach-5.', 'People who buy the SKX Mach-5 are usually more aggressive than people who buy other cars.', 'Almost no one would fail to recognize the kind of person who would choose to own an SKX Mach-5.', 'Anyone who is dynamic and aggressive is also successful.']", "label": 0 }, { "id": "train_356", "context": "The book To Save the Earth is so persuasive that no one who reads it can fail to heed its environmentalist message. Members of the Earth Association have given away 2, 000 copies in the last month. Thus the Earth Association can justly claim credit for at least 2, 000 people in one month converted to the environmentalist cause.", "question": "Which one of the following is an assumption on which the argument depends?", "answers": "['No other environmental organization gave away copies of To Save the Earth during the month in which the Earth Association gave away its 2, 000 copies.', 'The copies of To Save the Earth given away by members of the Earth Association were printed on recycled paper.', 'Every recipient of To Save the Earth will embrace the environmental program advocated by the Earth Association.', 'None of those who received To Save the Earth from a member of the Earth Association were already committed to the environmentalist cause when they received this book.']", "label": 3 }, { "id": "train_357", "context": "Anyone who believes in extraterrestrials believes in UFOs. But the existence of UFOs has been conclusively refuted. Ttherefore a belief in extraterrestrials is false as well.", "question": "Which one of the following arguments contains flawed reasoning most similar to that in the argument above?", "answers": "['Anyone who believes in unicorns believes in centaurs. But it has been conclusively proven that there is no such thing as a unicorn, so a belief in centaurs is mistaken as well.', 'Anyone who believes in unicorns believes in centaurs. But it has been demonstrated that there are no centaurs, so there are no unicorns either.', 'Anyone who believes in unicorns believes in centaurs. But you do not believe in centaurs, so you do not believe in unicorns either.', 'Anyone who believes in unicorns believes in centaurs. But you do not believe in unicorns, so you do not believe in centaurs either.']", "label": 1 }, { "id": "train_358", "context": "Modern physicians often employ laboratory tests, in addition to physical examinations, in order to diagnose diseases accurately. Insurance company regulations that deny coverage for certain laboratory tests ttherefore decrease the quality of medical care provided to patients.", "question": "Which one of the following is an assumption that would serve to justify the conclusion above?", "answers": "['Physical examinations and the uncovered laboratory tests together provide a more accurate diagnosis of many diseases than do physical examinations alone.', 'Many physicians generally oppose insurance company regulations that, in order to reduce costs, limit the use of laboratory tests.', 'There are some illnesses that experienced physicians can diagnose accurately from physical examination alone.', 'Many patients who might benefit from the uncovered laboratory tests do not have any form of health insurance.']", "label": 0 }, { "id": "train_359", "context": "Soil scientists studying the role of compost in horticulture have found that, while compost is useful for building soil structure, it does not supply large enough quantities of the nutrients essential for plant growth to make it a replacement for fertilizer. Many home gardeners, however, have found they can grow healthy and highly productive plants in soil that lacked essential nutrients by enriching the soil with nothing but compost.", "question": "Which one of the following, if true, most helps to explain the discrepant findings of the soil scientists and the home gardeners?", "answers": "[\"Most plants grown in home gardens and in scientists' test plots need a favorable soil structure, as well as essential nutrients, in order to thrive.\", 'The findings of soil scientists who are employed by fertilizer manufacturers do not differ widely from those of scientists employed by the government or by universities.', 'Some of the varieties of plants grown by home gardeners require greater quantities of nutrients in order to be healthy than do the varieties of plants generally grown by the soil scientists in test plots.', 'Compost used in research projects is usually made from leaves and grass clippings only, whereas compost used in home gardens is generally made from a wide variety of ingredients.']", "label": 3 }, { "id": "train_360", "context": "Magazine editor: I know that some of our regular advertisers have been pressuring us to give favorable mention to their products in our articles, but they should realize that for us to yield to their wishes would actually be against their interests. To remain an effective advertising vehicle we must have loyal readership, and we would soon lose that readership if our readers suspect that our editorial integrity has been compromised by pandering to advertisers. Advertising-sales director: You underestimate the sophistication of our readers. They recognize that the advertisements we carry are not articles, so their response to the advertisements has never depended on their opinion of the editorial integrity of the magazine as a whole.", "question": "Which one of the following is the most accurate assessment of the advertising-sales director's argument as a response to the magazine editor's argument?", "answers": [ "It succeeds because it undermines the editor's claim about how the magazine's editorial integrity would be affected by allowing advertisers to influence articles.", "It fails because the editor's argument does not depend on any assumption about readers' response to the advertisements they see in the magazine.", "It succeeds because it shows that the editor's argument depends on an unwarranted assumption about factors affecting an advertisement's effectiveness.", "It fails because it is based on a misunderstanding of the editor's view about how readers respond to advertisements they see in the magazine." ], "label": 1 }, { "id": "train_361", "context": "Plumb-Ace advertises that its plumbers are more qualified than plumbers at any other major plumbing firm in the region because Plumb-Ace plumbers must complete a very difficult certification process. Plumb-Ace plumbers may or may not be more qualified, but clearly the certification process is not very difficult, because nearly everyone who takes the written portion of the certification exam passes it very easily.", "question": "The reasoning in the argument is flawed in that it", "answers": "['takes for granted that plumbers are not qualified unless they complete some certification process', 'presumes that since one part of a whole lacks a certain characteristic, the whole must lack that characteristic as well', \"overlooks the possibility that plumbers at other firms in the region complete certification processes that are even easier than that completed by Plumb-Ace's plumbers\", 'infers that a claim is false on the grounds that an inadequate argument has been given for that claim']", "label": 1 }, { "id": "train_362", "context": "A technology company plans to develop a prototype laptop that uses iris scanners that can distinguish one user from another. The inventor of this laptop claims that it will reduce insurance and security costs at offices, as only authorized users will be able to turn on the laptop, making it a less appealing target for thieves.", "question": "Which of the following, if true, most supports the company's implementing its plan to develop the prototype?", "answers": "['Colored contacts allow people to change their eye color, without altering any of the other characteristics of their iris.', 'Insurance and security companies provide other services to companies in addition to protecting laptops.', \"There is considerable variation in the color and patterns present in most people's irises.\", 'In-person security services will not suffer from technological glitches when allowing access to laptops.']", "label": 2 }, { "id": "train_363", "context": "Any popular television series that is groundbreaking is critically acclaimed. But not all popular television series are critically acclaimed. Thus, not all popular television series are groundbreaking.", "question": "The pattern of reasoning in the argument above is most similar to that in which one of the following arguments?", "answers": "['Mr. Schwartz is polite to anyone who is polite to him. So, since all of his colleagues are polite to him, it must be that he is polite to all his colleagues.', 'If articles use specialized technical terminology, they are not widely read. So, since all academic works use specialized technical terminology, articles are not widely read if they are academic works.', \"Professor Attah gives students high grades if she thinks their work is greatly improved. So, since she gives some of her students high grades, she thinks those students' work is greatly improved.\", 'If a biography is unbiased, it contains embarrassing facts about its subject. So, since not all biographies contain embarrassing facts about their subjects, not all biographies are unbiased.']", "label": 3 }, { "id": "train_364", "context": "More pedestrian injuries occur at crosswalks marked by both striping on the roadway and flashing lights than occur at crosswalks not so marked. Obviously these so-called safety features are a waste of taxpayer money.", "question": "The reasoning in the argument is most vulnerable to criticism because the argument", "answers": "['takes for granted that crosswalks with both striping and flashing lights have no other safety features', 'fails to consider that crosswalks marked by both striping and flashing lights are marked in this way precisely because they are the most dangerous ones', 'presumes that there are less expensive features that will reduce the number of pedestrian injuries just as effectively as striping and flashing lights', 'takes for granted that safety features that fail to reduce the number of injuries are a waste of taxpayer money']", "label": 1 }, { "id": "train_365", "context": "In response to high mortality in area hospitals, surgery was restricted to emergency procedures during a five-week period. Mortality in these hospitals was found to have fallen by nearly one-third during the period. The number of deaths rose again when elective surgery (surgery that can be postponed) was resumed. It can be concluded that, before the five-week period, the risks of elective surgery had been incurred unnecessarily often in the area.", "question": "Which one of the following, if true, most seriously undermines the conclusion above?", "answers": "['The physicians planning elective surgery performed before the five-week period had fully informed the patients who would undergo it of the possible risks of the procedures.', 'Even if a surgical procedure is successful, the patient can die of a hospital-contracted infection with a bacterium that is resistant to antibiotic treatment.', 'The conditions for which elective surgery was performed would in the long run have been life-threatening, and surgery for them would have become riskier with time.', 'Elective surgery is, in general, less risky than is emergency surgery because the conditions requiring or indicating surgery are often less severe.']", "label": 2 }, { "id": "train_366", "context": "An editorial in the Grandburg Daily Herald claims that Grandburg' s voters would generally welcome the defeat of the political party now in control of the Grandburg City Council. The editorial bases its claim on a recent survey that found that 59 percent of Grandburg' s registered voters think that the party will definitely be out of power after next year' s city council elections.", "question": "Which one of the following is a principle that, if established, would provide the strongest justification for the editorial's conclusion?", "answers": "['It can reasonably be assumed that registered voters who respond to a survey regarding the outcome of a future election will exercise their right to vote in that election.', 'The way voters feel about a political party at a given time can reasonably be considered a reliable indicator of the way they will continue to feel about that party, barring unforeseeable political developments.', 'An increase in ill-feeling toward a political party that is in power can reasonably be expected to result in a corresponding increase in support for rival political parties.', 'The proportion of voters who expect a given political possibility to be realized can legitimately be assumed to approximate the proportion of voters who are in favor of that possibility being realized.']", "label": 3 }, { "id": "train_367", "context": "In the past decade, a decreasing percentage of money spent on treating disease X went to pay for standard methods of treatment, which are known to be effective though they are expensive and painful. An increasing percentage is being spent on nonstandard treatments, which cause little discomfort. Unfortunately, the nonstandard treatments have proved to be ineffective. Obviously, less money is being spent now on effective treatments of disease X than was spent ten years ago.", "question": "Which one of the following, if assumed, allows the conclusion above to be properly drawn?", "answers": "['Varieties of disease X requiring expensive special treatment have become less common during the past decade.', 'Of total medical expenditures, the percentage that is due to treatment of disease X increased during the past decade.', 'The total amount of money spent on treating disease X slowly declined during the past decade.', 'Most of the money spent on treating disease X during the last decade went to pay for nonstandard treatments.']", "label": 2 }, { "id": "train_368", "context": "Recently, a team of scientists digging through a tar pit unearthed a jawbone fossil. Initially, the team hypothesized that the jawbone came from a young gomphothere, a now extinct distant relative of the elephant, since the teeth were those of a juvenile. The gomphothere, however, is known for its large molars, and the teeth on the jawbone would not allow enough room for the molars of an adult gomphothere to fit. Based on this evidence, the scientists conclude that the jawbone fossil provides evidence of a distinct species closely related to the gomphothere.", "question": "Which of the following, if true, would best provide evidence showing that the conclusion above is possibly flawed?", "answers": "['In order for the molars of an adult gomphothereto emerge, several juvenile teeth are first forced out of the gums to accommodate the molars.', 'The juvenile jawbone of a species related to a gomphothere is longer than the juvenile jawbone of a gomphothere.', 'The manner in which teeth grow provide sufficient evidence for the accurate classification of a bygone species.', 'The molars of an adult mastodon, a close relative of the gomphothere, are similar in size to those of an adult gomphothere.']", "label": 0 }, { "id": "train_369", "context": "Situation: For five years the Souper restaurant chain has maintained rapid sales growth in Danport, primarily by opening new restaurants that draw patrons away from other restaurants in their vicinity. Goal: Souper wishes to achieve continued rapid sales growth in Danport over the next two years. Proposal for consideration: Continue to open new restaurants in Danport during the next two years at the same rate as in the last two years.", "question": "In light of the situation, which of the following, if true, most strongly argues that adopting the proposal would be an ineffective way of achieving the goal?", "answers": "['The Souper chain has generally opened new restaurants in locations that are in the vicinity of a large number of other restaurants.', 'Souper restaurants generally offer a much smaller variety of foods than many of the other restaurants in their vicinity.', 'Virtually all potential sites for new Souper restaurants in Danport are located in the vicinity of existing Souper restaurants.', 'Souper restaurants have always offered meals that are somewhat less expensive than meals at restaurants that compete with Souper for patrons.']", "label": 2 }, { "id": "train_370", "context": "In the Centerville Botanical Gardens, all tulip trees are older than any maples. A majority, but not all, of the garden' s sycamores are older than any of its maples. All the garden' s maples are older than any of its dogwoods.", "question": "If the statements above are true, which one of the following must also be true of trees in the Centerville Botanical Gardens?", "answers": "['Some tulip trees are not as old as the oldest sycamores.', 'Some dogwoods are as old as the youngest sycamores.', 'Some sycamores are not as old as the oldest dogwoods.', 'Some sycamores are not as old as the youngest tulip trees.']", "label": 3 }, { "id": "train_371", "context": "Technologically, it is already possible to produce nonpolluting cars that burn hydrogen rather than gasoline. But the national system of fuel stations that would be needed to provide the hydrogen fuel for such cars does not yet exist. However, this infrastructure is likely to appear and grow rapidly. A century ago no fuel-distribution infrastructure existed for gasoline-powered vehicles, yet it quickly developed in response to consumer demand.", "question": "Which one of the following most accurately expresses the conclusion drawn in the argument?", "answers": "['Hydrogen-powered vehicles will be similar to gasoline-powered vehicles with regard to the amount of consumer demand for their fuel-distribution infrastructure.', 'The fuel-distribution infrastructure for hydrogen-powered cars is likely to appear and grow rapidly.', 'It is already technologically possible to produce nonpolluting cars that burn hydrogen rather than gasoline.', 'If a new kind of technology is developed, the infrastructure needed to support that technology is likely to quickly develop in response to consumer demands.']", "label": 1 }, { "id": "train_372", "context": "Migraines are caused by changes in certain blood vessels of the brain that result in a local disturbance in the action of a specific nerve-activating chemical. Two migraine remedies, drug K and drug L, have exactly the same restorative effects on the local action of this chemical, but L also disrupts the action of several other chemicals in the brain that governmental activity and the body' s metabolism.", "question": "The information above most strongly supports which one of the following hypotheses?", "answers": "['K, unlike L, should be administered immediately after the onset of a migraine.', 'People whose migraines are treated with L are likely to experience a wider range of effects beyond the cessation of migraines than are people whose migraines are treated with K.', 'Treatment with L is likely to be slower-acting in its effect on a migraine than is treatment with K.', 'Neither treatment with L nor treatment with K is likely to have a significant effect on the symptoms of a migraine.']", "label": 1 }, { "id": "train_373", "context": "Critic: It has been argued that the real value of a product is not equal to the price people are willing to pay for it, on the grounds that this price often exceeds the cost of the labor required to create it, and the excess goes to the investor as an unjustifiably acquired profit. But since the existence of the product is as dependent on the investor' s initial capital outlay as on the laborer' s toil, this argument is fallacious.", "question": "The critic's argument requires assuming which one of the following?", "answers": "['People who make the initial investment required to create a product have a right to a greater percentage of the receipts from its sale than do those whose contributions occur later.', 'Everyone whose activity contributes to the creation of a product deserves an equal share of the proceeds from its sale.', \"An investor's profit is not justified in those cases in which the existence of the product does not depend on the investor's initial capital outlay.\", 'The real value of a product is not completely determined by the cost of the labor needed to bring the product into existence.']", "label": 3 }, { "id": "train_374", "context": "In a national park located on an island, a herd of moose was increasing in number and threatening to destroy species of native plants. Wolves were introduced to the island to reduce the herd and thereby prevent destruction of the vegetation. Although the wolves prospered, the moose herd continued to grow.", "question": "Which one of the following, if true, most helps to explain the failure of the strategy involving wolves?", "answers": "['The presence of wolves in an area tends to discourage other predators from moving into the area.', 'Healthy moose generally consume more vegetation than do those that are diseased or injured.', 'Attempts to control moose populations in other national parks by introducing predators have also been unsuccessful.', 'Wolves often kill moose weakened by diseases that probably would have spread to other moose.']", "label": 3 }, { "id": "train_375", "context": "In Country X' s last election, the Reform Party beat its main opponent, the Conservative Party, although pollsters, employing in-person interviews shortly before the vote, had projected a Conservative Party victory. Afterwards, the pollsters determined that, unlike Conservative Party supporters, Reform Party supporters were less likely to express their party preference during in-person interviews than they were during telephone interviews. Ttherefore, using only telephone interviews instead would likely result in more accurate projections for the next election.", "question": "Which of the following statements, if true, would most support the argument in the passage?", "answers": "['For each person interviewed in telephone polls before the next election, pollsters will be able to reasonably determine the likelihood of that person voting.', 'In the next election, pollsters will be able to conduct more in-person interviews than telephone interviews.', \"The number of voters in Country X's next election will be significantly larger than the number of voters in the last election.\", 'The Conservative Party will win the next election.']", "label": 0 }, { "id": "train_376", "context": "Paulsville and Longtown cannot both be included in the candidate' s itinerary of campaign stops. The candidate will make a stop in Paulsville unless Salisbury is made part of the itinerary. Unfortunately, a stop in Salisbury is out of the question. Clearly, then, a stop in Longtown can be ruled out.", "question": "The reasoning in the argument above most closely parallels that in which one of the following arguments?", "answers": "[\"The chef never has both fresh radishes and fresh green peppers available for the chef's salad at the same time. If she uses fresh radishes, she also uses spinach. But currently there is no spinach to be had. It can be inferred, then, that she will not be using fresh green peppers.\", \"Tom will definitely support Parker if Mendoza does not apply; and Tom will not support both Parker and Chung. Since, as it turns out, Mendoza will not apply, it follows that Chung will not get Tom's support.\", 'In agricultural pest control, either pesticides or the introduction of natural enemies of the pest, but not both, will work. Of course, neither will be needed if pest-resistant crops are planted. So if pesticides are in fact needed, it must be that there are no natural enemies of the pest.', 'The city cannot afford to build both a new stadium and the new road that would be needed to get there. But neither of the two projects is worth doing without the other. Since the city will not undertake any but worthwhile projects, the new stadium will not be constructed at this time.']", "label": 1 }, { "id": "train_377", "context": "Essayist: One of the claims of laissez-faire economics is that increasing the minimum wage reduces the total number of minimum-wage jobs available. In a recent study, however, it was found that after an increase in the minimum wage, fast-food restaurants kept on roughly the same number of minimum-wage employees as before the increase. Ttherefore, laissez-faire economics is not entirely accurate.", "question": "The essayist's argument depends on assuming which one of the following?", "answers": "['If laissez-faire economics makes an incorrect prediction about the minimum wage, then all the doctrines of laissez-faire economics are inaccurate.', 'The national unemployment rate did not increase following the increase in the minimum wage.', 'Minimum-wage job availability at fast-food restaurants included in the study was representative of minimum-wage job availability in general.', 'No study has ever found that a business has decreased the number of its minimum-wage employees after an increase in the minimum wage.']", "label": 2 }, { "id": "train_378", "context": "Carol Morris wants to own a majority of the shares of the city' s largest newspaper, The Daily. The only obstacle to Morris' s amassing a majority of these shares is that Azedcorp, which currently owns a majority, has steadfastly refused to sell. Industry analysts nevertheless predict that Morris will soon be the majority owner of The Daily.", "question": "Which one of the following, if true, provides the most support for the industry analysts' prediction?", "answers": "['Morris already owns more shares of The Daily than anyone except Azedcorp.', \"No one other than Morris has expressed any interest in purchasing a majority of The Daily's shares.\", 'Azedcorp is financially so weak that bankruptcy will probably soon force the sale of its newspaper holdings.', 'Azedcorp does not own shares of any newspaper other than The Daily.']", "label": 2 }, { "id": "train_379", "context": "A 1955 analysis of paint samples from an Italian painting found evidence of cobalt, suggesting the use of cobalt blue, a pigment not used in Europe before 1804. The painting was thus deemed to have been produced sometime after 1804. A 2009 analysis also found cobalt, but that analysis suggested that the painting might have been produced before 1804.", "question": "Which one of the following, if true, most helps to resolve the apparent discrepancy in the information above?", "answers": "['The 2009 analysis used sophisticated scientific equipment that can detect much smaller amounts of cobalt than could the equipment used for the 1955 analysis.', 'New information that came to light in the 1990s suggested that cobalt blue was used only rarely in Italy in the years immediately following 1804.', 'The 2009 analysis took more samples from the painting than the 1955 analysis did, though those samples were smaller.', 'The 2009 analysis revealed that cobalt was located only in the topmost paint layer, which was possibly applied to conceal damage to original paint layers.']", "label": 3 }, { "id": "train_380", "context": "Recent studies indicate that people who spend significant time browsing the World Wide Web do so at the expense of time spent in face-to-face interaction with other people. This has led some to fear that social isolation will greatly increase because of the popularity of browsing the Web. But, since browsing the Web is just one of many popular solitary activities, and any time spent on a solitary activity is time spent not interacting face-to-face with other people, such fears are unwarranted.", "question": "Which one of the following most accurately describes the role played in the argument by the view that social isolation will greatly increase because of the popularity of browsing the Web?", "answers": "['It is a premise that, together with another premise, is meant to support the conclusion of the argument.', 'It is the overall conclusion of the argument.', 'It is the claim that the argument attempts to undermine.', 'It is presented as evidence that independently supports the conclusion of the argument.']", "label": 2 }, { "id": "train_381", "context": "The graphical illustrations mathematics teachers use enable students to learn geometry more easily by providing them with an intuitive understanding of geometric concepts, which makes it easier to acquire the ability to manipulate symbols for the purpose of calculation. Illustrating algebraic concepts graphically would be equally effective pedagogically, even though the deepest mathematical understanding is abstract, not imagistic.", "question": "The statements above provide some support for each of the following EXCEPT:", "answers": "['There are strategies that can be effectively employed in the teaching both of algebra and of geometry.', 'People who are very good at manipulating symbols do not necessarily have any mathematical understanding.', 'Illustrating geometric concepts graphically is an effective teaching method.', 'Pictorial understanding is not the final stage of mathematical understanding.']", "label": 1 }, { "id": "train_382", "context": "Background information: This year, each film submitted to the Barbizon Film Festival was submitted in one of ten categories. For each category, there was a panel that decided which submitted films to accept. Fact 1: Within each category, the rate of acceptance for domestic films was the same as that for foreign films. Fact 2: The overall rate of acceptance of domestic films was significantly higher than that of foreign films.", "question": "In light of the background information, which of the following, if true, can account for fact 1 and fact 2 both being true of the submissions to this year's Barbizon Film Festival?", "answers": "['Most foreign films, unlike most domestic films, were submitted in categories with high prestige, but with correspondingly low rates of acceptance.', 'Significantly more domestic films than foreign films were submitted to the festival.', 'In each of the past three years, the overall acceptance rate was higher for foreign than for domestic films, an outcome that had upset some domestic filmmakers.', 'The number of films to be selected in each category was predetermined, but in no category was it required that the acceptance rate of foreign films should equal that of domestic films.']", "label": 0 }, { "id": "train_383", "context": "Recent research shows that hesitation, shifting posture, and failure to maintain eye contact are not reliable indicators in discriminating between those who are lying and those who are telling the truth. The research indicates that behavior that cannot be controlled is a much better clue, at least when the lie is important to the liar. Such behavior includes the dilation of eye pupils, which indicates emotional arousal, and small movements of facial muscles, which indicate distress, fear, or anger.", "question": "Which one of the following provides the strongest reason for exercising caution when relying on the \"better\" clues mentioned above in order to discover whether someone is lying?", "answers": "['A person telling the truth might be affected emotionally by being suspected of lying or by some other aspect of the situation.', 'A person who is lying might be aware that he or she is being closely observed for indications of lying.', 'Someone who is telling the truth might nevertheless have a past history of lying.', 'A practiced liar might have achieved great control over body posture and eye contact.']", "label": 0 }, { "id": "train_384", "context": "The government recently released a study of drinking water, in which it was reported that consumers who bought bottled water were in many cases getting water that was less safe than what they could obtain much more cheaply from the public water supply. In spite of the enormous publicity that the study received, sales of bottled water have continued to rise.", "question": "Which one of the following, if true, is most helpful in resolving the apparent paradox?", "answers": "['The rate of increase in the sales of bottled water has slowed since the publication of the government study.', 'Most consumers who habitually drink the bottled water discussed in the study cannot differentiate between the taste of their usual brand of bottled water and that of water from public sources.', 'Government health warnings concerning food have become so frequent that consumers have begun to doubt the safety of many everyday foods.', 'Increased consumption of the five best-selling brands of bottled water, which the report said were safer than both public water and most other brands of bottled water, accounted for the increase in sales.']", "label": 3 }, { "id": "train_385", "context": "Editorial: Despite the fact that recent military success has made the prime minister personally popular, her party will lose the next election unless the economy is no longer in recession; the electorate will blame the state of the economy on her, even though she is hardly responsible for it and her policies will be beneficial in the long run.", "question": "Which one of the following is an assumption on which the editorial's argument depends?", "answers": "['In the next election, voters will place as much weight on economic performance as on military success.', \"The prime minister's party will win the next election if the economy shows some signs of recovery from the recession at the time of the election.\", \"In the next election, the majority of voters will base their votes on which party's leader they admire most.\", \"The prime minister's party will not win the next election if the electorate believes the prime minister is responsible for a continuing recession.\"]", "label": 3 }, { "id": "train_386", "context": "For Megan to get to the grocery store, she must take either the bus or the subway. The bus is not running this week, so although Megan generally does not like to take the subway, she must have used it today, since she was seen at the grocery store today.", "question": "Which one of the following most accurately describes the method of reasoning used to reach the conclusion above?", "answers": "['showing that there are additional alternatives that are unknown', 'showing that certain possibilities are not exclusive', 'showing that no exceptions could apply', 'showing that something must have occurred by excluding the only alternative']", "label": 3 }, { "id": "train_387", "context": "Astronomer: I have asserted that our solar system does not contain enough meteoroids and other cosmic debris to have caused the extensive cratering on the far side of the moon. My opponents have repeatedly failed to demonstrate the falsity of this thesis. Their evidence is simply inconclusive; thus they should admit that my thesis is correct.", "question": "The reasoning in the astronomer's argument is flawed because this argument", "answers": "[\"criticizes the astronomer's opponents rather than their arguments\", \"presumes that the astronomer's thesis should not be subject to rational discussion and criticism\", 'fails to precisely define the key word \"meteoroids\"', \"infers the truth of the astronomer's thesis from the mere claim that it has not been proven false\"]", "label": 3 }, { "id": "train_388", "context": "Philosopher: The most moral actions provide the most benefits to the most people at the lowest costs without any regard for intentions.", "question": "Which one of the following exhibits the most moral action, as described by the philosopher?", "answers": "[\"Tyree quits smoking cigarettes, and as a result, he will live longer and save taxpayers' money.\", \"Isabella founds a non-profit organization that teaches sustainable farming on a small island in the Pacific Ocean, but the island's villages adopt a different farming practice\", 'Leroy develops a vaccine for malaria, but the cure decimates the mosquito population, destroying the local ecosystem, and causing famine across an entire continent.', \"Trevor starts a website that provides people with the best solutions for reforming his nation's corrupt government, but nobody ever reads it.\"]", "label": 0 }, { "id": "train_389", "context": "Astronomer: Does a recent meteorite from Mars contain fossilized bacteria? Professor Tagar, a biologist, argues that the bacteria-like structures found in the meteorite cannot be fossilized bacteria, on the grounds that they are one-tenth of 1 percent the volume of the smallest earthly bacteria. However, Tagar' s view cannot be right. Tagar does not accept the views of biologists Swiderski and Terrada, who maintain that Martian bacteria would shrink to one-tenth of 1 percent of their normal volume when water or other nutrients were in short supply.", "question": "Which one of the following most accurately describes a flaw in the reasoning in the astronomer's argument?", "answers": "['The argument presumes, without providing justification, that the authorities cited have always held the views attributed to them.', 'The argument presumes, without providing justification, that the opinions of all experts are equally justified.', 'The argument appeals to views that contradict rather than support one another.', 'The argument provides no justification for giving preference to the views of one rather than the other of two competing sets of authorities.']", "label": 3 }, { "id": "train_390", "context": "Tax reformer: The proposed tax reform legislation is being criticized by political groups on the right for being too specific and by political groups on the left for being too vague. Since one and the same statement cannot be both too specific and too vague, the criticisms just go to show that the legislation is framed just as it should be.", "question": "Which one of the following is an assumption on which the argument depends?", "answers": "['The proposed legislation has not been criticized by any group that does not identify itself with the political right or the political left.', 'It is rare for political groups both on the right and on the left to criticize a particular tax reform proposal.', 'The proposed legislation is not made up of a set of statements some of which are overly specific and some of which are overly vague.', 'The proposed legislation as it is framed was not meant to satisfy either political groups on the right or political groups on the left.']", "label": 2 }, { "id": "train_391", "context": "In the past, when there was no highway speed limit, the highway accident rate increased yearly, peaking a decade ago. At that time, the speed limit on highways was set at 90 kilometers per hour (kph) (55 miles per hour). Every year since the introduction of the highway speed limit, the highway accident rate has been at least 15 percent lower than that of its peak rate. Thus, setting the highway speed limit at 90 kph (55 mph) has reduced the highway accident rate by at least 15 percent.", "question": "Which one of the following, if true, most seriously weakens the argument?", "answers": "['Thanks to changes in automobile design in the past ten years, drivers are better able to maintain control of their cars in dangerous situations.', 'It was not until shortly after the introduction of the highway speed limit that most cars were equipped with features such as seat belts and airbags designed to prevent harm to passengers.', 'In the years prior to the introduction of the highway speed limit, many cars could go faster than 90 kph (55 mph).', 'Ten years ago, at least 95 percent of all automobile accidents in the area occurred on roads with a speed limit of under 80 kph (50 mph).']", "label": 0 }, { "id": "train_392", "context": "When a threat to life is common, as are automobile and industrial accidents, only unusual instances tend to be prominently reported by the news media. Instances of rare threats, such as product tampering, however, are seen as news by reporters and are universally reported in featured stories. People in general tend to estimate the risk of various threats by how frequently those threats come to their attention.", "question": "If the statements above are true, which one of the following is most strongly supported on the basis of them?", "answers": "['The resources that are spent on avoiding product tampering are greater than the resources that are spent on avoiding threats that stem from the weather.', 'Those who get their information primarily from the news media tend to overestimate the risk of uncommon threats relative to the risk of common threats.', 'Reporters tend not to seek out information about long-range future threats but to concentrate their attention on the immediate past and future.', 'People tend to magnify the risk of a threat if the threat seems particularly dreadful or if those who would be affected have no control over it.']", "label": 1 }, { "id": "train_393", "context": "In 1860 Bavarian quarry workers discovered the impression of a feather in a limestone slab dating to the Mesozoic era. It had previously been assumed that birds developed only after the close of the Mesozoic era and after the disappearance of pterosaurs, a species characteristic of that era. But there in limestone lay the imprint of a fully aerodynamic, three-inch-long feather. This, ttherefore, must have been the earliest bird -- certainly, the earliest found to that date.", "question": "The argument assumes which one of the following?", "answers": "['In the Mesozoic era, no creatures other than birds had such feathers.', 'The creature to which the feather belonged was a descendant of the pterosaurs.', 'The earliest bird flew in an awkward manner.', 'The feather belonged to a Mesozoic creature that was neither a pterosaur nor a bird, but an intermediate between them.']", "label": 0 }, { "id": "train_394", "context": "Political theorist: The chief foundations of all governments are the legal system and the police force; and as there cannot be a good legal system where the police are not well paid, it follows that where the police are well paid there will be a good legal system.", "question": "The reasoning in the argument is not sound because it fails to establish that", "answers": "['many governments with bad legal systems have poorly paid police forces', 'a well-paid police force is sufficient to guarantee a good legal system', 'some bad governments have good legal systems', 'a well-paid police force cannot be effective without a good legal system']", "label": 1 }, { "id": "train_395", "context": "Some paleontologists have suggested that Apatosaurus, a huge dinosaur, was able to gallop. This, however, is unlikely, because galloping would probably have broken Apatosaurus' s legs. Experiments with modern bones show how much strain they can withstand before breaking. By taking into account the diameter and density of Apatosaurus leg bones, it is possible to calculate that those bones could not have withstood the strains of galloping.", "question": "Which one of the following most accurately expresses the conclusion drawn by the argument as a whole?", "answers": "['Galloping would probably have broken the legs of Apatosaurus.', 'It is possible to calculate that Apatosaurus leg bones could not have withstood the strain of galloping.', 'If galloping would have broken the legs of Apatosaurus, then Apatosaurus was probably unable to gallop.', 'The claim of paleontologists that Apatosaurus was able to gallop is likely to be incorrect.']", "label": 3 }, { "id": "train_396", "context": "Throughout the Popoya Islands community pressure is exerted on people who win the national lottery to share their good fortune with their neighbors. When people living in rural areas win the lottery they invariably throw elaborate neighborhood feasts, often wiping out all of their lottery winnings. However, in the cities, lottery winners frequently use their winnings for their own personal investment rather than sharing their good fortune with their neighbors.", "question": "Which one of the following, if true, contributes most to an explanation of the difference between the behavior of lottery winners in rural areas and those in cities?", "answers": "['Lottery winners in rural areas are notified of winning by public posting of lists of winners, but notification in the city is by private mail.', 'Families in rural areas in the Popoyas many contain twelve or fourteen people, but city families average six or seven.', 'Popoyan city dwellers tend to buy several lottery tickets at a time, but they buy tickets less frequently than do rural dwellers.', 'Twice as many lottery tickets are sold in rural areas as are sold in the city.']", "label": 0 }, { "id": "train_397", "context": "Kimmy is a world famous actress. Millions of people downloaded her leaked movie co-starring her previous boyfriend. Kimmy earns millions through her television show and marketing appearances. There' s little wonder that paparazzi track her every move.", "question": "What is the argument's primary purpose?", "answers": "['Kimmy is a highly compensated and extremely popular television and movie actress.', 'Kimmy starred in an extremely popular movie.', \"The paparazzi track Kimmy's every move for good reason.\", 'Kimmy does not deserve her fame.']", "label": 0 }, { "id": "train_398", "context": "Philosopher: Wolves do not tolerate an attack by one wolf on another if the latter wolf demonstrates submission by baring its throat. The same is true of foxes and domesticated dogs. So it would be erroneous to deny that animals have rights on the grounds that only human beings are capable of obeying moral rules.", "question": "The philosopher's argument proceeds by attempting to", "answers": "['cast doubt on the principle that being capable of obeying moral rules is a necessary condition for having rights', 'provide evidence suggesting that the concept of morality is often applied too broadly', 'provide counterexamples to refute a premise on which a particular conclusion is based', 'establish a claim by showing that the denial of that claim entails a logical contradiction']", "label": 2 }, { "id": "train_399", "context": "Much of the best scientific research of today shows that many of the results of earlier scientific work that was regarded in its time as good are in fact mistaken. Yet despite the fact that scientists are above all concerned to discover the truth, it is valuable for today' s scientists to study firsthand accounts of earlier scientific work.", "question": "Which one of the following, if true, would best reconcile the two statements above?", "answers": "['Scientists can make valuable contributions to the scientific work of their time even if the results of their work will later be shown to be mistaken.', 'Lessons in scientific methodology can be learned by seeing how earlier scientific work was carried out, sometimes especially when the results of that work are known to be incorrect.', 'There are many scientists today who are not thoroughly familiar with earlier scientific research.', 'Many firsthand accounts of earlier, flawed scientific work are not generally known to be mistaken.']", "label": 1 }, { "id": "train_400", "context": "Many scholars believe that official medieval persecutions of various minority groups were undertaken very reluctantly by medieval authorities and only in order to soothe popular unrest caused by underlying popular hostility to the groups in question. This belief is highly questionable. For one thing, there are few indications of any profound underlying popular hostility toward persecuted groups in cases where persecutions were particularly violent and sustained. For another, the most serious and widespread persecutions carried out by medieval authorities seem to have had as targets exactly those groups that most effectively disputed these authorities' right to govern.", "question": "The argument proceeds by", "answers": "['referring to the large numbers of scholarly adherents of a certain view to support the claim that the view is correct', 'establishing a general principle and using the principle to justify a conclusion about a particular category of cases', 'citing both a lack of evidence supporting a particular explanation and further evidence that suggests an alternative explanation', 'providing reasons to suspect the reliability of any conclusions based on evidence concerning the distant past']", "label": 2 }, { "id": "train_401", "context": "Doctor: While a few alternative medicines have dangerous side effects, some, such as many herbs, have been proven safe to consume. Thus, though there is little firm evidence of medicinal effect, advocates of these herbs as remedies for serious illnesses should always be allowed to prescribe them, since their patients will not be harmed, and might be helped, by the use of these products.", "question": "Which one of the following, if true, most seriously weakens the doctor's argument?", "answers": "['Some patients may have allergic reactions to certain medicines that have been tolerated by other patients.', \"Any pain relief or other benefits of many herbs have been proven to derive entirely from patients' belief in the remedy, rather than from its biochemical properties.\", \"The vast majority of purveyors of alternative medicines are driven as much by the profit motive as by a regard for their patients' health.\", 'Many practitioners and patients neglect more effective conventional medicines in favor of herbal remedies.']", "label": 3 }, { "id": "train_402", "context": "Philosopher: The eighteenth-century thesis that motion is absolute asserts that the change in an object' s position over time could be measured without reference to the position of any other object. A well-respected physicist, however, claims that this thesis is incoherent. Since a thesis that is incoherent cannot be accepted as a description of reality, motion cannot be absolute.", "question": "The argument uses which one of the following argumentative techniques?", "answers": "['attempting to persuade by the mere use of technical terminology', 'inferring from what has been observed to be the case under experimental conditions to what is in principle true', 'generalizing from what is true in one region of space to what must be true in all regions of space', 'relying on the authority of an expert to support a premise']", "label": 3 }, { "id": "train_403", "context": "In Kravonia, the average salary for jobs requiring a college degree has always been higher than the average salary for jobs that do not require a degree. Over the last few years, the number of Kravonians enrolled in college has been growing steadily. Ttherefore, the number of Kravonians entering the job market who have at least the qualification of a college degree will eventually be significantly higher than it has been over the last few years.", "question": "Which of the following, if true, most seriously weakens the argument?", "answers": "['In recent years, employers have been requiring college degrees for workers in jobs that were previously performed successfully by Kravonians who did not have college degrees.', 'Kravonians with more than one college degree earn little more, on average, than do Kravonians with only one college degree.', 'The average number of years Kravonian college students remain enrolled before completing a degree has been increasing over the past several years.', 'For many years, employers in Kravonia have had difficulty finding enough college graduates to fill the high-paying jobs that were available.']", "label": 2 }, { "id": "train_404", "context": "Between 1971 and 1975, the government office that monitors drug companies issued an average of 60 citations a year for serious violations of drug-promotion laws. Between 1976 and 1980, the annual average for issuance of suchs citations was only 5. This decrease indicates that the government office was, on average, considerably more lax in enforcing drug-promotion laws between 1976 and 1980 than it was between 1971 and 1975.", "question": "The argument assumes which one of the following?", "answers": "['The government office should not issue more than an average of 5 citations a year to drug companies for serious violations of drug-promotion laws.', 'Before 1971 the government office issued more than 60 citations a year to drug companies for serious violations of drug-promotion laws.', 'A change in enforcement of drug-promotion laws did not apply to minor violations.', \"The decrease in the number of citations was not caused by a decrease in drug companies' violations of drug-promotion laws.\"]", "label": 3 }, { "id": "train_405", "context": "Lydia: Each year, thousands of seabirds are injured when they become entangled in equipment owned by fishing companies. Ttherefore, the fishing companies should assume responsibility for funding veterinary treatment for the injured birds. Jonathan: Your feelings for the birds are admirable. Your proposal, however, should not be adopted because treatment of the most seriously injured birds would inhumanely prolong the lives of animals no longer able to live in the wild, as all wildlife should.", "question": "Jonathan uses which one of the following techniques in his response to Lydia?", "answers": "['He questions the appropriateness of interfering with wildlife in any way, even if the goal of the interference is to help.', 'He attempts to discredit her proposal by discussing its implications for only those birds that it serves least well.', 'He evades discussion of her proposal by raising the issue of whether her feelings about the birds are justified.', 'He directs a personal attack against her rather than addressing the argument she advances.']", "label": 1 }, { "id": "train_406", "context": "If there is an election, you can either vote or not. If you vote, you have the satisfaction of knowing you influenced the results of the election; if you do not vote, you have no right to complain about the results. So, following an election, either you will have the satisfaction of knowing you influenced its results or you will have no right to complain.", "question": "The reasoning in which one of the following most closely resembles that in the argument above?", "answers": "['When manufacturers use a natural resource, they are either efficient or inefficient. If they are inefficient, the resource will be depleted quickly. If they are efficient, the resource will last much longer. So either manufacturers are efficient or they should be fined.', 'If you go for a walk, when you are finished either you will feel relaxed or you will not. If you feel relaxed, then your muscles will likely not be sore the next day, though your muscles will more likely become conditioned faster if they do feel sore. Ttherefore, either your muscles will feel sore, or they will become better conditioned.', 'If you use a computer, its messages are either easily readable or not. If the messages are easily readable, they are merely password protected. If they are not easily readable, they are electronically encrypted. So any message on the computer you use is either password protected or electronically encrypted.', 'If you attend school, you will find the courses stimulating or you will not. If your teachers are motivated, you will find the courses stimulating. If your teachers are not motivated, you will not. So either your teachers are motivated, or their courses are not stimulating.']", "label": 2 }, { "id": "train_407", "context": "A smoker trying to quit is more likely to succeed if his or her doctor greatly exaggerates the dangers of smoking. Similar strategies can be used to break other habits. But since such strategies involve deception, individuals cannot easily adopt them unless a doctor or some other third party provides the warning.", "question": "Which one of the following is an assumption on which the argument depends?", "answers": "[\"The more the relevant danger is exaggerated, the more likely one is to break one's habit.\", 'People generally do not find it easy to deceive themselves.', 'A doctor is justified in deceiving a patient whenever doing so is likely to make the patient healthier.', 'Most of the techniques that help people quit smoking can also help people break other habits.']", "label": 1 }, { "id": "train_408", "context": "An undergraduate degree is necessary for appointment to the executive board. Further, no one with a felony conviction can be appointed to the board. Thus, Manu, an accountant with both a bachelor' s and a master' s degree, cannot be accepted for the position of Executive Administrator, since he has a felony conviction.", "question": "The argument's conclusion follows logically if which one of the following is assumed?", "answers": "['If Manu did not have a felony conviction, he would be accepted for the position of Executive Administrator.', 'Only candidates eligible for appointment to the executive board can be accepted for the position of Executive Administrator.', 'The felony charge on which Manu was convicted is relevant to the duties of the position of Executive Administrator.', 'An undergraduate degree is not necessary for acceptance for the position of Executive Administrator.']", "label": 1 }, { "id": "train_409", "context": "No widget factory is located in the Western Time Zone. Some widget factories are employee owned and operated, and nearly all widget factories are publicly owned. Widget factories are the only businesses with more than one thousand employees. A privately-owned factory employs two thousand people.", "question": "If the statements above are correct, which one of the following must be true?", "answers": "['The factory is not a widget factory.', 'The factory is employee-owned and -operated.', 'The factory is located in the Central Time Zone.', 'The factory is a widget factory.']", "label": 3 }, { "id": "train_410", "context": "Given that employees of the XYZ Company could, in theory, do their work at home, the company developed a radical plan to increase efficiency: eliminate office-space expenditures by having employees work at home. To evaluate this plan, XYZ' s managers asked volunteers from the company' s staff to try the arrangement for six months. There were several volunteers; significantly, their productivity during this period was as high as or higher than before.", "question": "Which of the following, if true, would argue most strongly against deciding, on the basis of the trial results, to implement the company's plan?", "answers": "[\"The employees who agreed to participate in the test of the plan were among the company's most self-motivated and independent workers.\", 'The volunteers who worked at home were able to communicate with other employees as necessary for performing the work.', 'The savings that would accrue from reduced office-space expenditures alone would be sufficient to justify implementation of the plan apart from any productivity increases.', 'Minor changes in the way office work is organized at XYZ would yield increases in employee productivity similar to those achieved in the trial.']", "label": 0 }, { "id": "train_411", "context": "Mathematics teacher: Teaching students calculus before they attend university may significantly benefit them. Yet if students are taught calculus before they are ready for the level of abstraction involved, they may abandon the study of mathematics altogether. So if we are going to teach pre-university students calculus, we must make sure they can handle the level of abstraction involved.", "question": "Which one of the following principles most helps to justify the mathematics teacher's argument?", "answers": "['Teachers who teach university-level mathematics to pre-university students should be aware that students are likely to learn effectively only when the application of mathematics to concrete problems is shown.', 'Cognitive tasks that require exceptional effort tend to undermine the motivation of those who attempt them.', 'Only those who, without losing motivation, can meet the cognitive challenges that new intellectual work involves should be introduced to it.', 'Only those parts of university-level mathematics that are the most concrete should be taught to pre-university students.']", "label": 2 }, { "id": "train_412", "context": "First-time computer buyers buying PXC home computers typically buy models that cost much less and have a smaller profit margin per computer than do PXC computers bought by people replacing their computers with more powerful models. Last year PXC' s profits from computer sales were substantially higher than the previous year, although about the same number of PXC computers were sold and the prices and profit margins for each computer model that PXC sells remained unchanged.", "question": "If the statements above are true, which of the following is most strongly supported by them?", "answers": "['The number of people buying PXC computers who also bought PXC computer-related products, such as printers, was larger last year than the previous year.', \"PXC's production costs for its computers were lower last year than they had been the previous year.\", 'The proportion of PXC computers bought by first-time computer buyers was smaller last year than the previous year.', 'Among computer buyers who bought a PXC computer to replace their existing computer, the proportion who were replacing a computer made by a competitor of PXC was greater last year than the previous year.']", "label": 2 }, { "id": "train_413", "context": "Inertia affects the flow of water pumped through a closed system of pipes. When the pump is first switched on, the water, which has mass, takes time to reach full speed. When the pump is switched off, inertia causes the decrease in the water flow to be gradual. The effects of inductance in electrical circuits are similar to the effects of inertia in water pipes.", "question": "The information above provides the most support for which one of the following?", "answers": "[\"Inertia in the flow of water pumped by an electrically powered pump is caused by inductance in the pump's circuits.\", 'The rate at which electrical current flows is affected by inductance.', 'The flow of electrical current in a circuit requires inertia.', 'Electrical engineers try to minimize the effects of inductance in electrical circuits.']", "label": 1 }, { "id": "train_414", "context": "Pettengill: Bebop jazz musicians showed their distaste for jazz classics by taking great liberties with them, as though the songs could be made interesting only through radical reshaping. Romney: Only compelling, versatile songs can stand such radical reshaping. Bebop musicians recognized this, and their revolutionary approach to the jazz classics enabled them to discover previously unknown depths in the music.", "question": "Pettengill and Romney disagree over whether", "answers": "['bebop musicians showed appreciation for jazz classics in radically reshaping them', 'jazz music requires musicians to adhere closely to the original version in order to be widely popular', 'bebop jazz was an improvement on the jazz classics that preceded it', 'bebop jazz was radically different from the jazz music that preceded it']", "label": 0 }, { "id": "train_415", "context": "Pulford: Scientists who study the remains of ancient historical figures to learn about their health history need to first ask themselves if their investigation is a legitimate scientific inquiry or is motivated by mere curiosity. An investigation into a private matter such as health history is justified only if it is done for the advancement of scientific knowledge. Varela: You forget that curiosity is the root of scientific inquiry. Many great scientific discoveries were motivated by curiosity alone.", "question": "Varela responds to Pulford's argument by", "answers": "['attempting to draw a distinction between two views that Pulford treats as a single view', 'disputing the validity of a principle that Pulford explicitly states', \"maintaining that Pulford's argument is based on inconsistent premises\", \"contending that Pulford's argument rests on an untenable distinction\"]", "label": 3 }, { "id": "train_416", "context": "Political strategist: Clearly, attacking an opposing candidate on philosophical grounds is generally more effective than attacking the details of the opponent' s policy proposals. A philosophical attack links an opponent' s policy proposals to an overarching ideological scheme, thereby telling a story and providing context. This makes the attack emotionally compelling.", "question": "Which one of the following is an assumption required by the political strategist' s argument?", "answers": "['The stories that people are most likely to remember are those that are emotionally compelling.', 'Political attacks that tell a story are able to provide more context than those that do not.', \"Voters are typically uninterested in the details of candidates' policy proposals.\", 'Political attacks that are emotionally compelling are generally more effective than those that are not.']", "label": 3 }, { "id": "train_417", "context": "There is a widespread belief that people can predict impending earthquakes from unusual animal behavior. Skeptics claim that this belief is based on selective coincidence: people whose dogs behaved oddly just before an earthquake will be especially likely to remember that fact. At any given time, the skeptics say, some of the world' s dogs will be behaving oddly.", "question": "Clarification of which one of the following issues would be most important to an evaluation of the skeptics' position?", "answers": "['Are the sorts of behavior supposedly predictive of earthquakes as pronounced in dogs as they are in other animals?', 'Are there animals about whose behavior people know too little to be able to distinguish unusual from everyday behavior?', 'Which is larger, the number of skeptics or the number of people who believe that animal behavior can foreshadow earthquakes?', 'Is the animal behavior supposedly predictive of earthquakes specific to impending earthquakes or can it be any kind of unusual behavior?']", "label": 3 }, { "id": "train_418", "context": "Vandenburg: This art museum is not adhering to its purpose. Its founders intended it to devote as much attention to contemporary art as to the art of earlier periods, but its collection of contemporary art is far smaller than its other collections. Simpson: The relatively small size of the museum' s contemporary art collection is appropriate. It' s an art museum, not an ethnographic museum designed to collect every style of every period. Its contemporary art collection is small because its curators believe that there is little high-quality contemporary art.", "question": "Which one of the following principles, if valid, most helps to justify the reasoning in Simpson's response to Vandenburg?", "answers": "[\"The intentions of an art museum's curators should not determine what is collected by that museum.\", \"An art museum's purpose need not be to collect every style of every period.\", \"An ethnographic museum's purpose should be defined according to its curators' beliefs.\", 'An art museum should collect only works that its curators consider to be of high artistic quality.']", "label": 3 }, { "id": "train_419", "context": "Herbalist: Many herbal medicines work best when they have a chance to influence the body gently over several months. However, many of these herbal medicines have toxic side effects when taken daily for such long periods. Ttherefore, at least some people who use herbal medicines daily should occasionally skip their usual dose for a day or two, to give the body a chance to recuperate.", "question": "Which one of the following is an assumption required by the herbalist's argument?", "answers": "['At least some people who use herbal medicines daily use them for periods long enough for the medicines to have side effects.', \"At least some herbal medicines work less well in achieving their desired effects if one occasionally skips one's usual dose than if one does not.\", 'Some herbal medicines have toxic side effects when taken for several months, even if the usual dose is occasionally skipped for a day or two to give the body a chance to recuperate.', 'Anyone who uses herbal medicines should give those medicines a chance to influence the body gently over several months at least.']", "label": 0 }, { "id": "train_420", "context": "The Canadian elk has traditionally been limited in its geographic range by the presence of elderberries, its primary source of food, which only grow up until a certain point in the tundra. A recent rise in temperatures, however, has seen a spread in the growth of elderberries northwards into the tundra. Ttherefore, the overall range of the Canadian elk can be expected to increase.", "question": "Which of the following, if true, best casts doubt on the argument?", "answers": "[\"The grizzly bear, the Canadian elk's primary predator, has also started moving north into the tundra.\", 'During the summer months, many Canadian elk are hunted both for sport and for their meat.', 'The permafrost, the region above the tundra, in which the temperatures never reach above freezing, will unlikely see elderberry growth.', \"Increasing temperatures have created conditions too warm for elderberry growth in the southern half of the Canadian elk's traditional territory.\"]", "label": 3 }, { "id": "train_421", "context": "Literature professor: Critics charge that the work of C. F. Providence' s best-known follower, S. N. Sauk, lacks aesthetic merit because it employs Providence' s own uniquely potent system of symbolic motifs in the service of a political ideal that Providence -- and, significantly, some of these critics as well -- would reject. Granting that Sauk is more imitator than innovator, and that he maintained political views very different from those Providence maintained, it has yet to be shown that these facts make his writings any less subtly or powerfully crafted than those of his more esteemed mentor. So the critics' argument should be rejected.", "question": "The literature professor argues that the conclusion drawn by the critics has not really been established, on the grounds that", "answers": "[\"these critics are motivated by antipathy toward Sauk's political ideas\", 'the claims made in support of this conclusion have not been shown to be relevant to it', \"Sauk's work has aesthetic merit\", 'the claims made in support of this conclusion have not been shown to be correct']", "label": 1 }, { "id": "train_422", "context": "Many movies starring top actors will do well at the box office because the actors are already well known and have a loyal following. Movies starring unknown actors are ttherefore unlikely to do well.", "question": "The flawed reasoning in the argument above is most similar to that in which one of the following?", "answers": "['Many animals must devote most of their energy to locating food, or they will not get enough food to maintain optimal energy levels. Thus, if immediate survival requires such an animal to devote most of its energy to some other purpose, optimal energy levels generally will not be maintained.', 'Often the presence of the flower bee balm in a garden will attract bumblebees that pollinate the plants and enable the garden to produce an abundant crop. So, gardens that lack bee balm usually do not produce abundant crops.', 'Visual aids can be very useful in effectively teaching math skills, because they generally allow vivid conceptualization of math principles. If such visual aids were never employed, ttherefore, teaching math skills might sometimes be more difficult.', 'An understanding of the rules of perspective is necessary for achieving success as a painter, since it is the understanding of these most basic rules that allows the painter to paint realistically. Thus, painters with an understanding of the rules of perspective will achieve success.']", "label": 1 }, { "id": "train_423", "context": "A famous artist once claimed that all great art imitates nature. If this claim is correct, then any music that is great art would imitate nature. But while some music may imitate ocean waves or the galloping of horses, for example, most great music imitates nothing at all.", "question": "Which one of the following most accurately expresses the main point of the argument?", "answers": "['Music is inferior to the other arts.', 'Like some great music, some great painting and sculpture may fail to imitate nature.', 'Sounds that do not imitate nature are not great music.', \"Either the artist's claim is incorrect, or most great music is not great art.\"]", "label": 3 }, { "id": "train_424", "context": "According to the theory of continental drift, in prehistoric times, many of today' s separate continents were part of a single huge landmass. As the plates on which this landmass rested began to move, the mass broke apart, and ocean water filled the newly created chasms. It is hypothesized, for example, that South America was once joined on its east coast with what is now the west coast of Africa.", "question": "Which one of the following discoveries, if it were made, would most support the above hypothesis about South America and Africa?", "answers": "['Some of the oldest tribes of people living in eastern South America speak languages linguistically similar to various languages spoken by certain western African peoples.', 'A large band of ancient rock of a rare type along the east coast of South America is of the same type as a band on the west coast of Africa.', 'The climates of western Africa and of the east coast of South America resemble each other.', 'Several species of plants found in western Africa closely resemble plants growing in South America.']", "label": 1 }, { "id": "train_425", "context": "Scientist: In our study, chemical R did not cause cancer in laboratory rats. But we cannot conclude from this that chemical R is safe for humans. After all, many substances known to be carcinogenic to humans cause no cancer in rats; this is probably because some carcinogens cause cancer only via long-term exposure and rats are short lived.", "question": "Which one of the following most precisely describes the role played in the scientist's argument by the statement that chemical R did not cause cancer in laboratory rats?", "answers": "['It is cited as being insufficient to support the conclusion that chemical R is safe for humans.', 'It is advanced to support the contention that test results obtained from laboratory rats cannot be extrapolated to humans.', 'It is used as evidence to support the hypothesis that chemical R causes cancer in humans via long-term exposure.', 'It illustrates the claim that rats are too short lived to be suitable as test subjects for the carcinogenic properties of substances to which humans are chronically exposed.']", "label": 0 }, { "id": "train_426", "context": "Evan: I am a vegetarian because I believe it is immoral to inflict pain on animals to obtain food. Some vegetarians who share this moral reason nonetheless consume some seafood, on the grounds that it is not known whether certain sea creatures can experience pleasure or pain. But if it is truly wrong to inflict needless suffering, we should extend the benefit of the doubt to sea animals and refrain from eating seafood.", "question": "Which one of the following most closely conforms to the principle illustrated by Evan's criticism of vegetarians who eat seafood?", "answers": "['It is uncertain whether all owners of the defective vehicles know that their vehicles are being recalled by the manufacturer. Thus, we should expect that some vehicles that have been recalled have not been returned.', \"I am opposed to using incentives such as reduced taxes to attract businesses to our region. These incentives would attract businesses interested only in short-term profits. Such businesses would make our region's economy less stable, because they have no long-term commitment to the community.\", 'I do not know if I have repaid Farah the money she lent me for a movie ticket. She says that she does not remember whether or not I repaid her. In order to be sure that I have repaid her, I will give her the money now.', \"Isabel Allende lived through the tragic events of her country's recent history; no doubt her novels have been inspired by her memories of those events. Yet Allende's characters are hopeful and full of joy, indicating that Allende's own view of life has not been negatively marked by her experiences.\"]", "label": 2 }, { "id": "train_427", "context": "A recently passed law requires all places of public accommodation to eliminate discrimination against persons with disabilities by removing all physical barriers to accessibility. Private schools, ttherefore, are legally obligated to make their campuses physically accessible to persons with disabilities.", "question": "The conclusion above follows logically if which one of the following is assumed?", "answers": "['Private schools, like public schools are places of public accommodation.', 'Private schools have enough funds to make their campuses barrier-free.', 'No private school can legally deny admission to a person with a disability.', 'Private schools have historically been resistant to changes in government policy on discrimination.']", "label": 0 }, { "id": "train_428", "context": "There is a difference between beauty and truth. After all, if there were no difference, then the most realistic pieces of art would be the best as well, since the most realistic pieces are the most truthful. But many of the most realistic artworks are not among the best.", "question": "Which one of the following is an assumption required by the argument?", "answers": "[\"An artwork's beauty is inherently subjective and depends on who is viewing it.\", 'If an artwork contains nonrealistic elements, then it is not at all truthful.', 'None of the best artworks are realistic.', 'The most beautiful artworks are the best artworks.']", "label": 3 }, { "id": "train_429", "context": "To decrease the number of crimes in city Y, the city' s Police Commissioner proposed taking some police officers from low-crime districts of the city and moving them to high-crime districts of the city. His proposal is based on city Y crime data that show that the number of crimes in any district of the city decreases when additional police officers are moved into that district.", "question": "The Police Commissioner's proposal depends on which of the following assumptions?", "answers": "['There are more low-crime districts than high-crime districts in city Y.', 'The number of crimes committed in all high-crime districts of city Y is more than triple the number of crimes committed in all low-crime districts of city Y.', 'City X experienced a drastic reduction in crime after implementing a proposal similar to that proposed by the Police Commissioner of city Y.', 'Districts of the city from which police officers are removed do not experience significant crime increases shortly after the removal of those officers.']", "label": 3 }, { "id": "train_430", "context": "Doctor: The practice of using this therapy to treat the illness cannot be adequately supported by the claim that any therapy for treating the illness is more effective than no therapy at all. What must also be taken into account is that this therapy is expensive and complicated.", "question": "Which one of the following most accurately expresses the main point of the doctor's argument?", "answers": "['The therapy is more effective than other forms of treatment for the illness.', \"The therapy's possible effectiveness in treating the illness is not sufficient justification for using it.\", 'The therapy is more expensive and complicated than other forms of treatment for the illness.', 'The therapy should not be used to treat the illness unless it is either effective or inexpensive.']", "label": 1 }, { "id": "train_431", "context": "Editorial: It is the popular assumption that millennials are less altruistic than the generation that came before them. Company reports of top-down volunteer programs and suggested donations through work are less likely to be performed by millennials and more likely to be performed by the generation that preceded them. Millennials who are on board with company donation programs are rare.", "question": "Which of the following most accurately describes a flaw in the argument?", "answers": "['The argument assumes that corporate greed is the byproduct of the millennial attitude with no sufficient evidence backing this claim. The evidence presented has more to do with millennial behavior related to volunteering and less to do with direct corporate greed.', 'The argument fails to acknowledge corporate altruism, which is a responsibility that a corporation has to the environment and the impact it has on society. Corporate responsibility is just as important as individual responsibility, especially when it has so large an effect on the world.', 'The argument fails to consider altruistic millennial behavior outside of the corporate environment. It only mentions \"top-down\" volunteer programs and donations through a company. The argument does not take note of what causes millennials are contributing to in their own free time.', 'The argument assumes that for millenials to perform well they must be in competition with each other. This is not necessarily true, as there is evidence in the article that contradicts this notion.']", "label": 2 }, { "id": "train_432", "context": "Over 90 percent of the human brain currently serves no purpose, as is evident from the fact that many people with significant brain damage show no discernible adverse effects. So once humans begin to tap into this tremendous source of creativity and innovation, many problems that today seem insurmountable will be within our ability to solve.", "question": "Which one of the following most accurately describes a flaw in the argument?", "answers": "['The argument infers that certain parts of the brain do nothing merely on the basis of the assertion that we do not know what they do.', 'The argument presumes, without providing justification, that the only reason that any problem remains unsolved is a lack of creativity and innovation.', 'The argument infers that problems will be solved merely on the basis of the claim that they will be within our ability to solve.', 'The argument presumes, without providing justification, that the currently unused parts of the brain are a potential source of tremendous creativity and innovation.']", "label": 3 }, { "id": "train_433", "context": "All potatoes naturally contain solanine, which is poisonous in large quantities. Domesticated potatoes contain only very small amounts of solanine, but many wild potatoes contain poisonous levels of solanine. Since most of the solanine in potatoes is concentrated in the skin, however, peeling wild potatoes makes them at least as safe to eat as unpeeled domesticated potatoes of the same size.", "question": "Which one of the following, if assumed, allows the conclusion above to be properly drawn?", "answers": "['Wild potatoes are generally much smaller than domesticated potatoes.', 'The amount of solanine concentrated in the skin of a wild potato is large enough by itself to be poisonous.', 'There are no poisonous substances in domesticated potatoes other than solanine.', 'There is no more solanine in a peeled wild potato than in an unpeeled domesticated potato of the same size.']", "label": 3 }, { "id": "train_434", "context": "Director of personnel: Ms. Tours has formally requested a salary adjustment on the grounds that she was denied merit raises to which she was entitled. Since such grounds provide a possible basis for adjustments, an official response is required. Ms. Tours presents compelling evidence that her job performance has been both excellent in itself and markedly superior to that of others in her department who were awarded merit raises. Her complaint that she was treated unfairly thus appears justified. Nevertheless, her request should be denied. To raise Ms. Tours' s salary because of her complaint would jeopardize the integrity of the firm' s merit-based reward system by sending the message that employees can get their salaries raised if they just complain enough.", "question": "The personnel director's reasoning is most vulnerable to criticism on the grounds that it", "answers": "[\"fails to consider the possibility that Ms. Tours's complaint could be handled on an unofficial basis\", 'ignores the possibility that some of the people who did receive merit increases were not entitled to them', \"overlooks the implications for the integrity of the firm's merit-based reward system of denying Ms. Tours's request\", 'sidesteps the issue of whether superior job performance is a suitable basis for awarding salary increases']", "label": 2 }, { "id": "train_435", "context": "Lightbox, Inc. , owns almost all of the movie theaters in Washington County and has announced plans to double the number of movie screens it has in the county within five years. Yet attendance at Lightbox' s theaters is only just large enough for profitability now and the county' s population is not expected to increase over the next ten years. Clearly, ttherefore, if there is indeed no increase in population, Lightbox' s new screens are unlikely to prove profitable.", "question": "Which of the following, if true about Washington County, most seriously weakens the argument?", "answers": "[\"In selecting the mix of movies shown at its theaters, Lightbox's policy is to avoid those that appeal to only a small segment of the moviegoing population.\", \"The sales of snacks and drinks in its movie theaters account for more of Lightbox's profits than ticket sales do.\", 'Spending on video purchases, as well as spending on video rentals, is currently no longer increasing.', 'Though little change in the size of the population is expected, a pronounced shift toward a younger, more affluent, and more entertainment-oriented population is expected to occur.']", "label": 3 }, { "id": "train_436", "context": "Taxi driver: My passengers complained when, on a hot day, I turned off my cab' s air conditioner while driving up a steep hill. While the engine is powerful enough to both run the air conditioner and climb the hill without slowing, this would have decreased my fuel economy considerably. So turning off the air conditioner was the right decision.", "question": "Which one of the following principles, if valid, most supports the taxi driver's reasoning?", "answers": "['A taxi driver should always act in a way that is most likely to ensure customer satisfaction.', \"A taxi driver should run a cab's air conditioner only if doing so does not cause fuel economy to drop below normal levels.\", 'A taxi driver should try to balance concern for fuel economy with concern for passenger comfort.', \"A taxi driver's turning off air-conditioning for a short period of time is acceptable only if passengers do not complain.\"]", "label": 1 }, { "id": "train_437", "context": "Journalists sometimes use historical photographs to illustrate articles about current events. But this recycling of old photographs overstates the similarities between past and present, and thereby denies the individual significance of those current events. Hence, the use of historical photographs in this manner by journalists distorts public understanding of the present by presenting current events as mere repetitions of historical incidents.", "question": "Which one of the following, if assumed, enables the conclusion of the argument to be properly inferred?", "answers": "['If journalists believe current events to be mere repetitions of historical incidents, then public understanding of the present will be distorted.', 'If a journalistic practice distorts public understanding of the present by overstating the similarities between past and present, then it denies the individual significance of any articles about current events.', 'Any practice by which journalists present current events as mere repetitions of historical incidents overstates the similarities between past and present.', 'If the work of a journalist overstates the similarities between past and present, then it distorts public understanding of the present by presenting current events as mere repetitions of historical incidents.']", "label": 3 }, { "id": "train_438", "context": "Conventional wisdom suggests vaccinating elderly people first in flu season, because they are at greatest risk of dying if they contract the virus. This year's flu virus poses particular risk to elderly people and almost none at all to younger people, particularly children. Nevertheless, health professionals are recommending vaccinating children first against the virus rather than elderly people.", "question": "Which of the following, if true, provides the strongest reason for the health professionals' recommendation?", "answers": "['Children who catch one strain of the flu virus and then recover are likely to develop immunity to at least some strains with which they have not yet come in contact.', 'Children are particularly unconcerned with hygiene and ttherefore are the group most responsible for spreading the flu virus to others.', \"The vaccinations received last year will confer no immunity to this year's flu virus.\", 'Children are no more likely than adults to have immunity to a particular flu virus if they have never lived through a previous epidemic of the same virus.']", "label": 1 }, { "id": "train_439", "context": "Electronic cigarettes should not be subject to the same regulation as other products that contain nicotine. Recent studies indicate that electronic cigarettes help people quit smoking by providing nicotine without the harmful tar and additive chemicals. Although electronic cigarettes also contain their own additives, they are much less harmful in the short-term than traditional cigarettes. People who smoke electronic cigarettes are ten times less likely to die from cancer than smokers of traditional cigarettes.", "question": "Which one of the following most weakens the argument?", "answers": "['The current regulations are designed to prevent children from using nicotine.', 'The recent studies are not conclusive.', 'The additives in electronic cigarettes have not been tested as thoroughly as those in traditional cigarettes.', 'More smokers die from heart disease than cancer.']", "label": 0 }, { "id": "train_440", "context": "Art history professor: Costa criticizes my theories about the distinction between baroque and neoclassical Austrian painting. He argues that since there are no features possessed by all and only the works from a given historical period, assigning works of art to period styles is intellectually bankrupt . His reasoning can be discounted, however, since his own current theories on the transition from classical to romantic French opera also presuppose such an assignment.", "question": "Which one of the following most accurately describes a flaw in the art history professor 's argument?", "answers": "['The argument presumes, without providing justification, that theories about one type of art cannot be compared to theories about another.', \"The argument confuses a necessary condition for discounting a person's reasoning with a sufficient condition for discounting a person's reasoning .\", 'The argument presumes, without providing justification, that what is true of art in general must also be true of every particular type of art.', 'The argument rejects the reasoning on which a criticism is based merely on the grounds that that very criticism could be applied to theories of the person who offered it.']", "label": 3 }, { "id": "train_441", "context": "Farm animals have certain behavioral tendencies that result from the evolutionary history of these species. By imposing on these animals a type of organization that conflicts with their behavioral tendencies, current farm-management practices cause the animals more pain and distress than do practices that more closely conform to the animals' behavioral tendencies. Because the animals tend to resist this type of organization, current practices can also be less efficient than those other farm-management practices.", "question": "If the statements above are true, which one of the following can be properly inferred from them?", "answers": "['In order to implement efficient farm-management practices, it is necessary to be familiar with the evolutionary history of farm animals.', 'In order to create farm-management practices that cause less pain and distress to farm animals, a significant loss of efficiency will be required.', 'Farm-management practices that cause the least amount of pain and distress to farm animals are also the most efficient management practices.', 'Some changes in farm-management practices that lessen the pain and distress experienced by farm animals can result in gains in efficiency.']", "label": 3 }, { "id": "train_442", "context": "The chairperson should not have released the Election Commission' s report to the public, for the chairperson did not consult any other members of the commission about releasing the report before having it released.", "question": "The argument's conclusion can be properly inferred if which one of the following is assumed?", "answers": "[\"It would have been permissible for the chairperson to release the commission's report to the public only if most other members of the commission had first given their consent.\", \"The chairperson would have been justified in releasing the report only if each of the commission's members would have agreed to its being released had they been consulted.\", 'Some members of the commission would have preferred that the report not be released to the public.', 'All of the members of the commission had signed the report prior to its release.']", "label": 0 }, { "id": "train_443", "context": "Which one of the following exhibits both of the logical flaws exhibited in the argument above?", "question": "All intelligent people are nearsighted. I am very nearsighted. So I must be a genius.", "answers": "['All chickens have beaks. This bird has a beak. So this bird must be a chicken.', 'All pigs have four legs, but this spider has eight legs. So this spider must be twice as big as any pig.', 'John is extremely happy, so he must be extremely tall because all tall people are happy.', 'I must be stupid because all intelligent people are nearsighted and I have perfect eyesight.']", "label": 2 }, { "id": "train_444", "context": "The following is an excerpt from a campaign speech. Senator Baker: My opponent, Candidate Rothmore, has called for increased taxes to fund programs that help the long-term unemployed of the state. Such action would address an immediate symptom for a select group, without doing much to address the overall problems we all face. Only through lowering taxes can we stimulate the growth of small businesses, which will revitalize the state' s whole economy. That, in turn, will result in greater prosperity and in more jobs, including jobs for those who have been unemployed for a while. Through lowering taxes, everyone wins, and those currently unemployed get the best help the economy can provide --- a real job.", "question": "In the argument, the two portions in boldface play which of the following roles?", "answers": "['The first is evidence supporting the main argument; the second is the main conclusion.', 'The first is a prediction about a recommendation the main argument opposes; the second is a conclusion drawn in order to support the main conclusion.', 'The first is a prediction that, if accurate, would provide support for the main conclusion of the argument; the second is the main conclusion.', \"The first is a conclusion drawn by Senator Baker's opponent; the second is Senator Baker's main conclusion.\"]", "label": 1 }, { "id": "train_445", "context": "Professor Edwards must have revealed information that was embarrassing to the university. After all, to have been publicly censured by the head of the university, as Edwards was, a professor must either have revealed information that embarrassed the university or have been guilty of gross professional negligence, and Edwards' professional behavior is impeccable.", "question": "Which one of the following arguments exhibits a pattern of reasoning most similar to that in the argument above?", "answers": "['Anyone who is either awarded a letter of commendation or who receives a bonus must be recommended by a company officer. Simon has been recommended by a company officer and will receive a bonus, so he must not have been awarded a letter of commendation.', 'To earn a merit salary increase, an employee of TGX must either bring in new clients or develop innovative products. No innovative products were developed at TGX this year, however, so TGX employees must have brought in many new clients.', 'Any employee of Wilkins, Waddel, and Sloan who discusses a client with a member of the press will be either fired or demoted. But since Wilkins employees never discuss their clients at all, no Wilkins employee will ever be demoted.', 'Anyone promoted to supervisor must either have worked on the shop floor for three years or have an influential sponsor. Daniels, ttherefore, clearly has an influential sponsor, since he was promoted to supervisor after only one year on the shop floor.']", "label": 3 }, { "id": "train_446", "context": "People who have doctorates in the liberal arts are interested in improving their intellects. Companies, however, rarely hire people who are not concerned with the financial gain that can be obtained by hard work in the business world. As a result, companies rarely hire people who have doctorates in the liberal arts.", "question": "The conclusion of the argument follows logically if which one of the following is assumed?", "answers": "['Some people who are interested in the liberal arts do not care about money.', 'Only people not concerned with making money in the business world are interested in improving their intellects.', 'The only people not interested in making money in the business world are people who are interested in improving their intellects.', 'People with doctorates in the liberal arts are interested in employment in the business world.']", "label": 1 }, { "id": "train_447", "context": "Forest fragmentation occurs when development severs a continuous area of forest, breaking it down into small patches. Some animals, such as white-footed mice, thrive in conditions of forest fragmentation, reaching their highest population densities in small forest patches. These mice are the main carrier of the bacteria that cause Lyme disease, a debilitating illness that is often transmitted from white-footed mice to humans by deer ticks.", "question": "Which one of the following is most strongly supported by the information above?", "answers": "['The population density for most species of small animals increases when a continuous area of forest becomes fragmented.', 'Deer ticks reach their highest population densities in small forest patches.', 'White-footed mice are very rarely found in unfragmented forests.', 'Efforts to stop the fragmentation of forests can have a beneficial effect on human health.']", "label": 3 }, { "id": "train_448", "context": "Consumer: I would like to have the features contained in the latest upgrade to your computer software package, but I am leery of installing the upgrade because a friend has told me he had a problem with it. Company representative: We have distributed nearly 3, 000 copies of the upgrade and we have received fewer than 100 calls saying that it has caused problems. So it is very unlikely that you will experience any problems with the upgrade.", "question": "The reasoning in the company representative's argument is most vulnerable to criticism because it fails to consider the possibility that", "answers": "['a significant number of people have experienced problems with the upgrade but have not reported them', 'some of the reported problems were a result of users failing to follow instructions', 'the consumer will experience software problems if the upgrade is not installed', 'some of the problems people have experienced with the upgrade have been quite serious']", "label": 0 }, { "id": "train_449", "context": "Near-Earth objects (NEOs) such as asteroids threaten the Earth because they have the potential to collide with it. The goal of NEO research is to develop measures to counteract a possible hit by a sizable NEO. Government funding of this research is not a waste of money. Buying home insurance makes good fiscal sense, and governments fund NEO research for the same reason that people insure their homes.", "question": "The statement that buying home insurance makes good fiscal sense plays which one of the following roles in the argument?", "answers": "['It is a general principle for which the argument attempts to provide support.', \"It connects an analogy made in the argument to the argument's conclusion.\", 'It is the overall conclusion of the argument.', 'It provides a contrast to the situation that is the main focus of the argument.']", "label": 1 }, { "id": "train_450", "context": "At many electronics retail stores, the consumer has the option of purchasing product warranties that extend beyond the manufacturer' s warranty. However, consumers are generally better off not buying extended warranties. Most problems with electronic goods occur within the period covered by the manufacturer' s warranty.", "question": "Which one of the following, if true, most strengthens the argument?", "answers": "['Most of those who buy extended warranties on electronic goods do so because special circumstances make their item more likely to break than is usually the case.', 'Retail stores sell extended warranties in part because consumers who purchase them are likely to purchase other products from the same store.', \"Problems with electronic goods that occur after the manufacturer's warranty expires are generally inexpensive to fix in comparison with the cost of an extended warranty.\", \"Some extended warranties on electronic goods cover the product for the period covered by the manufacturer's warranty as well as subsequent years.\"]", "label": 2 }, { "id": "train_451", "context": "One' s palate is to a great extent socially determined: that is, if we notice that a lot of people enjoy consuming a certain type of food, we will eventually come to like the food as well, once we have become accustomed to the food.", "question": "Which one of the following most closely conforms to the principle above?", "answers": "['Sally found jalapeno peppers to be too hot when she first tried them, but now she can eat them without discomfort, because her family members use them frequently in their cooking.', 'Yolanda dislikes pickles because she has observed that many of her relatives wince when eating pickles.', 'Maxine spoke to her neighbor about the many different ways he prepared pasta, and after trying some of his recipes found out that she loves to eat pasta.', \"All of George's Ukrainian relatives love to eat pierogis, and by staying with them for several summers, George has become very fond of pierogis as well.\"]", "label": 3 }, { "id": "train_452", "context": "All social systems are based upon a division of economic roles. The values of a social system are embodied in the prestige accorded persons who fill various economic roles. It is ttherefore unsurprising that, for any social system, the introduction of labor-saving technology that makes certain economic roles obsolete will tend to undermine the values in that social system.", "question": "Which one of the following can most reasonably be concluded on the basis of the information above?", "answers": "['A technologically advanced society will place little value on the prestige associated with an economic role.', 'No type of technology will fail to undermine the values in a social system.', 'Social systems will have unchanging values if they are shielded from technological advancement.', 'A social system whose values are not susceptible to change would not be one in which technology can eliminate economic roles.']", "label": 3 }, { "id": "train_453", "context": "Although many political candidates object to being made the target of advertising designed to cast them in an adverse light, such advertising actually benefits its targets because most elections have been won by candidates who were the targets of that kind of advertising.", "question": "The pattern of flawed reasoning in the argument most closely parallels that in which one of the following?", "answers": "['Although many people dislike feeling sleepy as a result of staying up late the previous night, such sleepiness must be acceptable to those who experience it because most people who stay up late enjoy doing so.', 'Although many actors dislike harsh reviews of their work, such reviews actually help their careers because most of the really prestigious acting awards have gone to actors who have had performances of theirs reviewed harshly.', 'Although many film critics dislike horror films, such films are bound to be successful because a large number of people are eager to attend them.', 'Although many students dislike studying, it must be a good way to achieve academic success because most students who study pass their courses.']", "label": 1 }, { "id": "train_454", "context": "Paperback books wear out more quickly than hardcover books do, but paperback books cost much less. Ttherefore, users of public libraries would be better served if public libraries bought only paperback books, since by so doing these libraries could increase the number of new book titles added to their collections without increasing their budgets.", "question": "Which one of the following, if true, most seriously weakens the argument?", "answers": "['People are more likely to buy for themselves a copy of a book they had previously borrowed from the public library if that book is available in paperback.', 'Library users as a group depend on their public library for access to a wide variety of up-to-date reference books that are published in hardcover only.', 'Paperback books can very inexpensively have their covers reinforced in order to make them last longer.', \"If a public library's overall budget is cut, the budget for new acquisitions is usually cut back more than is that for day-to-day operations.\"]", "label": 1 }, { "id": "train_455", "context": "A certain auto repair shop services a particular make of car in the downtown region of a particular city. Three years ago, 4% of all repairs at this shop were not successful --- the mechanics were not able to restore the car to a drivable state. This year, only 2% of all repairs were not successful. Clearly, the mechanics training course, given over a year ago, has considerably increased the skill of mechanics at this particular shop, so that fewer cars are beyond their ability to repair.", "question": "All of the following statements, if true, weaken the argument, EXCEPT:", "answers": "['The manufacturer of this particular make of car recalled a few models due to defects that potentially jeopardized the engine.', 'The total number of cars served per year by this shop has increased by 35% over the past three years.', 'Three years ago, statewide smog regulations had the effect of removing the oldest and most unreliable vehicles from the road.', 'A new shop uptown specializes in repairing cars of this make more than six years old, and most of these older cars are taken to this shop.']", "label": 1 }, { "id": "train_456", "context": "There are about 75 brands of microwave popcorn on the market; altogether, they account for a little over half of the money from sales of microwave food products. It takes three minutes to pop corn in the microwave, compared to seven minutes to pop corn conventionally. Yet by weight, microwave popcorn typically costs over five times as much as conventional popcorn. Judging by the popularity of microwave popcorn, many people are willing to pay a high price for just a little additional convenience.", "question": "If the statements in the passage are true, which one of the following must also be true?", "answers": "['More money is spent on microwave food products that take three minutes or less to cook than on microwave food products that take longer to cook.', 'No single brand of microwave popcorn accounts for a large share of microwave food product sales.', 'There are more brands of microwave popcorn on the market than there are of any other microwave food product.', 'Of the total number of microwave food products on the market, most are microwave popcorn products.']", "label": 0 }, { "id": "train_457", "context": "Sociologist: Many rural residents claim to only approve of modest lifestyles; yet, many of these same rural residents often buy magazines focused on the lives of celebrities. However, it is proven that humans are often fascinated by the very things they condemn. Ttherefore, it is predictable that these celebrity magazines would be popular with these rural residents.", "question": "The sociologist's argument depends on which one of the following assumptions?", "answers": "['More rural residents than before regard modest living highly.', 'All people who buy magazines focused on the lives of celebrities condemn such lifestyles.', 'Most rural residents who buy the magazines focused on celebrities do not regard the celebrity lifestyles as modest.', 'All rural residents who buy the magazines focused on the lives of celebrities are fascinated by the things that they condemn.']", "label": 2 }, { "id": "train_458", "context": "Activist: As electronic monitoring of employees grows more commonplace and invasive, we hear more and more attempted justifications of this practice by employers. Surveillance, they explain, keeps employees honest, efficient, and polite to customers. Such explanations are obviously self-serving, and so should not be taken to justify these unwarranted invasions of privacy.", "question": "A questionable technique used in the activist's argument is to", "answers": "[\"attack employers' motives instead of addressing their arguments\", 'attack an argument different from that actually offered by the employers', 'insist that modern business practices meet moral standards far higher than those accepted in the past', 'make a generalization based on a sample that there is reason to believe is biased']", "label": 0 }, { "id": "train_459", "context": "Mayor Pollister always repays his contributors by passing the laws they desire. Since Mayor Pollister recently passed the city' s first clean air act, he must have had a contributor to repay.", "question": "Which one of the following is an assumption on which the argument depends?", "answers": "['Mayor Pollister recently gained a great deal of support from a contributor.', 'Mayor Pollister would not have passed the law unless he was doing so for a contributor.', 'Mayor Pollister needed contributions to succeed in the election.', 'The city will be benefited by the passage of the clean air act.']", "label": 1 }, { "id": "train_460", "context": "The interstitial nucleus, a subregion of the brain' s hypothalamus, is typically smaller for male cats than for female cats. A neurobiologist performed autopsies on male cats who died from disease X, a disease affecting no more than . 05 percent of male cats, and found that these male cats had interstitial nuclei that were as large as those generally found in female cats. Thus, the size of the interstitial nucleus determines whether or not male cats can contract disease X.", "question": "Which of the following statements, if true, most seriously weakens the argument?", "answers": "['The hypothalamus is known not to be causally linked to disease Y, and disease X is a subtype of disease Y.', 'The interstitial nuclei of female cats who contract disease X are larger than those of female cats who do not contract disease X.', 'Of 1, 000 autopsies on male cats who did not contract disease X, 5 revealed interstitial nuclei larger than those of the average male cat.', 'No female cats have been known to contract disease X, which is a subtype of disease Y.']", "label": 0 }, { "id": "train_461", "context": "The attribution of the choral work Lacrimae to the composer Pescard (1400-1474) has been regarded as tentative, since it was based on a single treatise from the early 1500' s that named Pescard as the composer. Recently, several musical treatises from the late 1500' s have come to light, all of which name Pescard as the composer of Lacrimae. Unfortunately, these newly discovered treatises lend no support to the attribution of Lacrimae to Pescard, since __.", "question": "Which of the following most logically completes the argument?", "answers": "['the later treatises probably had no source for their attribution other than the earlier treatise', \"the author of the treatise from the early 1500's had no very strong evidence on which to base the identification of Pescard as the composer of Lacrimae\", \"the treatise from the early 1500's misidentifies the composers of some of the musical works it considers\", \"there are works that can conclusively be attributed to Pescard that are not even mentioned in the treatise from the early 1500's\"]", "label": 0 }, { "id": "train_462", "context": "Leaf beetles damage willow trees by stripping away their leaves, but a combination of parasites and predators generally keeps populations of these beetles in check. Researchers have found that severe air pollution results in reduced predator populations. The parasites, by contrast, are not adversely affected by pollution; nevertheless, the researchers' discovery probably does explain why leaf beetles cause particularly severe damage to willows in areas with severe air pollution, since __.", "question": "Which of the following most logically completes the passage?", "answers": "['where air pollution is not especially severe, predators have much more impact on leaf-beetle populations than parasites do', 'the damage caused by leaf beetles is usually not enough to kill a willow tree outright', 'the parasites that attack leaf beetles actually tend to be more prevalent in areas with severe air pollution than they are elsewhere', 'willows often grow in areas where air pollution is especially severe']", "label": 0 }, { "id": "train_463", "context": "Dogs learn best when they are trained using both voice commands and hand signals. After all, a recent study shows that dogs who were trained using both voice commands and hand signals were twice as likely to obey as were dogs who were trained using only voice commands.", "question": "The claim that dogs learn best when they are trained using both voice commands and hand signals figures in the argument in which one of the following ways?", "answers": "['It is a statement of background information offered to help facilitate understanding the issue in the argument.', 'It is a statement that the argument claims is supported by the study.', 'It is an implicit assumption of the argument.', \"It is an intermediate conclusion that is offered as direct support for the argument's main conclusion.\"]", "label": 1 }, { "id": "train_464", "context": "Formal performance evaluations in the professional world are conducted using realistic situations. Physicians are allowed to consult medical texts freely, attorneys may refer to law books and case records, and physicists and engineers have their manuals at hand for ready reference. Students, then, should likewise have access to their textbooks whenever they take examinations.", "question": "The reasoning in the argument is questionable because the argument", "answers": "['neglects to take into account the fact that professionals were once students who also did not have access to textbooks during examinations', 'neglects to take into account the fact that, unlike students, professionals have devoted many years of study to one subject', 'cites examples that are insufficient to support the generalization that performance evaluations in the professional world are conducted in realistic situations', 'fails to consider the possibility that the purposes of evaluation in the professional world and in school situations are quite dissimilar']", "label": 3 }, { "id": "train_465", "context": "Geographers and historians have traditionally held the view that Antarctica was first sighted around 1820, but some sixteenth-century European maps show a body that resembles the polar landmass, even though explorers of the period never saw it. Some scholars, ttherefore, argue that the continent must have been discovered and mapped by the ancients, whose maps are known to have served as models for the European cartographers.", "question": "Which of the following, if true, is most damaging to the inference drawn by the scholars?", "answers": "['Ancient philosophers believed that there had to be a large landmass at the South Pole to balance the northern continents and make the world symmetrical.', 'Between 3, 000 and 9, 000 years ago, the world was warmer than it is now, and the polar landmass was presumably smaller.', 'There are only a few sixteenth-century global maps that show a continental landmass as the South Pole.', 'The question of who first sighted Antarctica in modern times is still much debated, and no one has been able to present conclusive evidence.']", "label": 0 }, { "id": "train_466", "context": "Louis: People' s intentions cannot be, on the whole, more bad than good. Were we to believe otherwise, we would inevitably cease to trust each other, and no society can survive without mutual trust among its members.", "question": "The argument is most vulnerable to which one of the following criticisms?", "answers": "['It mistakenly assumes that if two claims cannot at the same time both be true, then they cannot at the same time both be false.', 'It assumes without warrant that in any situation with two possible outcomes, the most negative one will inevitably occur.', 'It challenges the truth of a claim merely by calling into question the motives of those who profess that they believe it to be true.', 'It fails to rule out the possibility that a true belief can have deleterious consequences.']", "label": 3 }, { "id": "train_467", "context": "On average, city bus drivers who are using the new computerized fare-collection system have a much better on-time record than do drivers using the old fare-collection system. Millicent Smith has the best on-time record of any bus driver in the city. Ttherefore, she must be using the computerized fare-collection system.", "question": "Which one of the following contains flawed reasoning most similar to that contained in the argument above?", "answers": "[\"All the city's solid-waste collection vehicles acquired after 1988 have a larger capacity than any of those acquired before 1988. This vehicle has the largest capacity of any the city owns, so it must have been acquired after 1988.\", \"This tomato is the largest of this year's crop. Since the tomatoes in the experimental plot are on average larger than those grown in the regular plots, this tomato must have been grown in the experimental plot.\", \"Last week's snowstorm in Toronto was probably an average storm for the area. It was certainly heavier than any snowstorm known to have occurred in Miami, but any average snowstorm in Toronto leaves more snow than ever falls in Miami.\", 'Lawn mowers powered by electricity generally require less maintenance than do lawn mowers powered by gasoline. This lawn mower is powered by gasoline, so it will probably require a lot of maintenance.']", "label": 1 }, { "id": "train_468", "context": "In yesterday' s council election a majority of voters supported conservative candidates, and a majority of voters supported candidates who voted in favor of the antipollution act. Ttherefore, it must be that a majority of voters in yesterday' s council election supported conservative candidates who voted in favor of the antipollution act.", "question": "Which one of the following is an argument that contains flawed reasoning most similar to the flawed reasoning in the argument above?", "answers": "['Mark will go on a picnic today only if it does not rain. Susan will go on a picnic today only if Mark goes too. Since it is not going to rain today, both Mark and Susan will go on a picnic.', 'According to Sara, most children like pies. According to Robert, most children like blueberries. So if Sara and Robert are both right, it must be that most children like pies that contain blueberries.', \"The majority of customers who regularly eat at this restaurant always order both fish and stuffed mushrooms. Thus, fish and stuffed mushrooms must be the restaurant's most frequently ordered dishes.\", 'Bill claims that soil can be damaged if it is tilled when it is too wet, and Sue claims that seeds planted in wet soil can rot. Ttherefore, if both claims are true, gardeners who till and plant their gardens when the soil is wet damage both their soil and their seeds.']", "label": 1 }, { "id": "train_469", "context": "The press reports on political campaigns these days as if they were chess games. One candidate' s campaign advisor makes a move; the other candidate' s advisor makes a countermove. The press then reports on the campaign advisors and not on the candidates. The losers in this chess game are the voters. They are deprived of the information they need to make informed decisions because the press is ignoring substantive policy issues and reporting only on the process of the campaign. It is clear that the campaign advisors should stay out of the limelight and let the press report on the most revealing positions on substantive issues the candidates have taken.", "question": "Which one of the following is an assumption upon which the argument in the passage depends?", "answers": "['There is no difference between reporting on the political process and reporting on substantive issues.', 'The voters are not paying enough attention to the election to be able to make informed decisions.', 'The candidates in the election are taking positions on substantive policy issues.', 'How the press reports politics determines the substantive issues in the campaign.']", "label": 2 }, { "id": "train_470", "context": "Sherrie: Scientists now agree that nicotine in tobacco is addictive inasmuch as smokers who try to stop smoking suffer withdrawal symptoms. For this reason alone, tobacco should be treated the same way as other dangerous drugs. Governments worldwide have a duty to restrict the manufacture and sale of tobacco. Fran: By your own admission, \"addictive\" is broad enough to include other commonly consumed products, such as coffee and soft drinks containing caffeine. But of course the manufacture and sale of these products should not be restricted.", "question": "The dialogue above lends the most support to the claim that Sherrie and Fran disagree with each other about which one of the following statements?", "answers": "['The manufacture and sale of all drugs should be regulated by governments.', 'Coffee and soft drinks that contain caffeine should not be regulated by governments.', 'Scientists and governments have a duty to cooperate in regulating drugs to protect the public health.', 'Agreement by scientists that a substance is addictive justifies government restrictions on products containing that substance.']", "label": 3 }, { "id": "train_471", "context": "Researcher: Dinosaurs lack turbinates -- nasal cavity bone structures in warm-blooded species that minimize water loss during breathing. According to some paleobiologists, this implies that all dinosaurs were cold-blooded. These paleobiologists must be mistaken, however, for fossil records show that some dinosaur species lived in Australia and Alaska, where temperatures drop below freezing. Only warm-blooded animals could survive such temperatures.", "question": "Which one of the following most accurately describes the role played in the researcher's argument by the claim that only warm-blooded animals could survive temperatures below freezing?", "answers": [ "It is presented as counterevidence to the paleobiologists' assertion that dinosaurs lack turbinates.", "It is presented as a potential counterexample to the argument's main conclusion.", "It is a premise offered in support of the argument's main conclusion.", "It is the argument's main conclusion." ], "label": 2 }, { "id": "train_472", "context": "In jurisdictions where use of headlights is optional when visibility is good, drivers who use headlights at all times are less likely to be involved in a collision than are drivers who use headlights only when visibility is poor. Yet Highway Safety Department records show that making use of headlights mandatory at all times does nothing to reduce the overall number of collisions.", "question": "Which one of the following, if true, most helps to resolve the apparent discrepancy in the information above?", "answers": "['In jurisdictions where use of headlights is optional when visibility is good, one driver in four uses headlights for daytime driving in good weather.', 'Only very careful drivers use headlights when their use is not legally required.', 'The jurisdictions where use of headlights is mandatory at all times are those where daytime visibility is frequently poor.', 'A law making use of headlights mandatory at all times is not especially difficult to enforce.']", "label": 1 }, { "id": "train_473", "context": "To protect certain fledgling industries, the government of country Z banned imports of the types of products those industries were starting to make. As a direct result, the cost of those products to the buyers, several export-dependent industries in Z, went up, sharply limiting the ability of those industries to compete effectively in their export markets.", "question": "Which of the following can be most properly inferred from the passage about the products whose importation was banned?", "answers": "['Those products had become more and more expensive to import, which resulted in a foreign trade deficit just before the ban.', 'Those products were the ones that country Z was hoping to export in its turn, once the fledgling industries matured.', \"Those products had been cheaper to import than they were to make within country Z's fledgling industries.\", \"Those products used to be imported from just those countries to which country Z's exports went.\"]", "label": 2 }, { "id": "train_474", "context": "Economist: Our economy' s weakness is the direct result of consumers' continued reluctance to spend, which in turn is caused by factors such as high-priced goods and services. This reluctance is exacerbated by the fact that the average income is significantly lower than it was five years ago. Thus, even though it is not a perfect solution, if the government were to lower income taxes, the economy would improve.", "question": "Which one of the following is an assumption required by the economist's argument?", "answers": "['Lowering income taxes will have no effect on government spending.', 'Increasing consumer spending will cause prices for goods and services to decrease.', \"If income taxes are not lowered, consumers' wages will decline even further.\", 'Consumers will be less reluctant to spend money if income taxes are lowered.']", "label": 3 }, { "id": "train_475", "context": "Physician: In an experiment, 50 patients with chronic back pain were divided into two groups. Small magnets were applied to the backs of one group; the other group received no treatment. Most of the patients in the first group, but very few in the second group, reported a significant reduction in pain. This shows that magnetic fields are probably effective at relieving some back pain.", "question": "Which one of the following, if true, constitutes the logically strongest counter to the physician's argument?", "answers": "['There was wide variation in the specific causes of the chronic back pain suffered by the patients in the experiment.', \"A patient's merely knowing that a treatment has been applied can lead to improvement in his or her condition.\", 'Most physicians believe that medication relieves chronic back pain more effectively than magnets do.', 'No other experiments have been done showing that magnetic fields reduce pain in any area other than the back.']", "label": 1 }, { "id": "train_476", "context": "In the nation of Partoria, large trucks currently have a much higher rate of traffic accidents per mile driven than other vehicles do. However, the very largest trucks-those with three trailers-had less than a third of the accident rate of single- and double-trailer trucks. Clearly, ttherefore, one way for Partoria to reduce the number of traffic accidents would be to require shippers to increase their use of triple-trailer trucks.", "question": "Which of the following, if true, most seriously weakens the argument?", "answers": "['In Partoria, the safety record of the trucking industry as a whole has improved slightly over the past ten years.', 'No matter what changes Partoria makes in the regulation of trucking, it will have to keep some smaller roads off-limits to all large trucks.', \"Increased use of triple-trailer trucks would mean that large trucks would account for a smaller proportion of all miles driven on Partoria's roads than they currently do.\", 'Partorian trucking companies currently use triple-trailer trucks only for long trips using major highways, which is the safest kind of trip for large trucks.']", "label": 3 }, { "id": "train_477", "context": "Advertisement: Clark brand-name parts are made for cars manufactured in this country. They satisfy all of our government automotive tests -- the toughest such tests in the world. With foreign-made parts, you never know which might be reliable and which are cheap look-alikes that are poorly constructed and liable to cost you hundreds of dollars in repairs. Ttherefore, be smart and insist on brand-name parts by Clark for your car.", "question": "The argument requires the assumption that", "answers": "['foreign-made parts are not suitable for cars manufactured in this country', 'no foreign-made parts satisfy our government standards', 'if parts are made for cars manufactured in our country, they are not poorly constructed', 'parts that satisfy our government standards are not as poorly constructed as cheap foreign-made parts']", "label": 3 }, { "id": "train_478", "context": "In 1995, ecologists started to reintroduce the wolf into Yellowstone Park, where the wolf had been eliminated intentionally half a century earlier. Rangers expected a rise in the number of wolf would place limits on the elk population and on the populations of many mid-size mammals. One unexpected consequence was a dramatic rise in the population of foxes, which in turn helped to control many rodent populations.", "question": "Which of the following, if true, most helps explain the dramatic rise in the fox population?", "answers": "['Foxes choose very different breeding area than do wolves', 'Elk hunting is more popular, and more widely marketable, than is fox hunting', 'The main predators of the fox were coyotes, on which the wolf preys.', 'In the agricultural counties surrounding Yellowstone, there are significantly lower wolf and fox populations.']", "label": 2 }, { "id": "train_479", "context": "To prevent a newly built dam on the Chiff River from blocking the route of fish migrating to breeding grounds upstream, the dam includes a fish pass, a mechanism designed to allow fish through the dam. Before the construction of the dam and fish pass, several thousand fish a day swam upriver during spawning season. But in the first season after the project' s completion, only 300 per day made the journey. Clearly, the fish pass is defective.", "question": "Which of the following, if true, most seriously weakens the argument?", "answers": "['On other rivers in the region, the construction of dams with fish passes has led to only small decreases in the number of fish migrating upstream.', 'During spawning season, the dam releases sufficient water for migratory fish below the dam to swim upstream.', 'Populations of migratory fish in the Chiff River have been declining slightly over the last 20 years.', 'The construction of the dam stirred up potentially toxic river sediments that were carried downstream.']", "label": 3 }, { "id": "train_480", "context": "Historian: The revolutionary party has been accused of having many overambitious goals and of having caused great suffering. However, most of the party' s goals were quickly achieved and the party did not have enough power to cause the suffering the critics claim it caused. So it is clear that the party was not overambitious and caused no suffering.", "question": "The reasoning in the historian's argument is flawed because the argument", "answers": "['fails to establish that the revolutionary party caused no suffering', \"provides no evidence that the revolutionary party's goals were not overambitious\", \"fails to establish that any of the revolutionary party's critics underestimated the party's power\", 'gives mutually inconsistent responses to the two criticisms']", "label": 0 }, { "id": "train_481", "context": "Researcher: It is commonly believed that species belonging to the same biological order, such as rodents, descended from a single common ancestor. However, I compared the genetic pattern in 3 rodent species -- guinea pigs, rats, and mice -- as well as in 13 nonrodent mammals, and found that while rats and mice are genetically quite similar, the genetic differences between guinea pigs and mice are as great as those between mice and some nonrodent species. Thus, despite their similar physical form, guinea pigs stem from a separate ancestor.", "question": "Which one of the following, if true, most seriously undermines the researcher's reasoning?", "answers": "['Some pairs of species not having a common ancestor are genetically more similar to each other than are some pairs that do have a common ancestor.', 'For some genuine biological orders, the most recent common ancestor dates from later epochs than does the most recent common ancestor of other biological orders.', 'Peculiarities of body structure, such as distinctive teeth and olfactory structures, are shared by all rodents, including guinea pigs.', 'The researcher selected nonrodent species that have the specific cell structures she wanted to analyze genetically, though many nonrodent mammals lack these cell structures.']", "label": 0 }, { "id": "train_482", "context": "Principle: A government should reduce taxes on imports if doing so would financially benefit many consumers in its domestic economy. There is a notable exception, however: it should never reduce import taxes if one or more of its domestic industries would be significantly harmed by the added competition. Conclusion: The government should not reduce taxes on textile imports.", "question": "Which one of the following is a statement from which the conclusion can be properly drawn using the principle?", "answers": "['The domestic textile industry faces significant competition in many of its export markets.', 'Reducing taxes on textile imports would financially benefit some consumers in the domestic economy but would not benefit the domestic textile industry.', 'The added competition produced by any reduction of taxes on imports would significantly harm the domestic textile industry.', 'The domestic textile industry and consumers in the domestic economy would benefit less from reductions in taxes on textile imports than they would from other measures.']", "label": 2 }, { "id": "train_483", "context": "Guideline: It is improper for public officials to influence the award of contracts or to perform other acts related to their office in a way that benefits themselves. Even the appearance of such impropriety should be avoided. Application: Greenville' s mayor acted improperly in urging the award of the city' s street maintenance contract to a company owned and operated by one of the mayor' s relatives, whose business would have been in serious financial trouble had it not been awarded the contract.", "question": "Which one of the following principles most helps in justifying the application of the guideline?", "answers": "['Creating the appearance of impropriety is as blameworthy as acting improperly.', 'Awarding a contract to a financially troubled business should be regarded as taking excessive risk.', 'Publicly funded contracts should be awarded based primarily on cost and the reliability of the contractor.', \"Benefiting one's family or friends should be regarded as benefiting oneself.\"]", "label": 3 }, { "id": "train_484", "context": "Police statistics have shown that automobile antitheft devices reduce the risk of car theft, but a statistical study of automobile theft by the automobile insurance industry claims that cars equipped with antitheft devices are, paradoxically, more likely to be stolen than cars that are not so equipped.", "question": "Which one of the following, if true, does the most to resolve the apparent paradox?", "answers": "['Most automobile thefts are the work of professional thieves against whose efforts antitheft devices offer scant protection.', 'The most common automobile antitheft devices are audible alarms, which typically produce ten false alarms for every actual attempted theft.', 'Automobile owners who have particularly theft-prone cars and live in areas of greatest incidence of car theft are those who are most likely to have antitheft devices installed.', 'Owners of stolen cars almost invariably report the theft immediately to the police but tend to delay notifying their insurance company, in the hope that the vehicle will be recovered.']", "label": 2 }, { "id": "train_485", "context": "Editorial: The town would not need to spend as much as it does on removing trash if all town residents sorted their household garbage. However, while telling residents that they must sort their garbage would get some of them to do so, many would resent the order and refuse to comply. The current voluntary system, then, is to be preferred, because it costs about as much as a nonvoluntary system would and it does not engender nearly as much resentment.", "question": "The contention that the town would not have to spend as much as it does on removing trash if all town residents sorted their garbage plays which one of the following roles in the editorial's argument?", "answers": "['It is a claim that the editorial is trying to show is false.', 'It is a fact granted by the editorial that lends some support to an alternative to the practice that the editorial defends as preferable.', 'It is an example of a difficulty facing the claim that the editorial is attempting to refute.', \"It is a premise that the editorial's argument relies on in reaching its conclusion.\"]", "label": 1 }, { "id": "train_486", "context": "In every case of political unrest in a certain country, the police have discovered that some unknown person or persons organized and fomented that unrest. Clearly, ttherefore, behind all the cases of political unrest in that country there has been a single mastermind who organized and fomented them all.", "question": "The flawed reasoning in the argument above most closely parallels that in which one of the following?", "answers": "['Every telephone number in North America has an area code, so there must be at least as many area codes as telephone numbers in North America.', 'Every loss of a single hair is insignificant, so no one who has a full head of hair at twenty ever becomes bald.', 'Every citizen of Edmonton has a social insurance number, so there must be one number that is the social insurance number for all citizens of Edmonton.', \"Every Chicago driver has a number on his or her license, so the number on some Chicago driver's license is the exact average of the numbers on all Chicago drivers' licenses.\"]", "label": 2 }, { "id": "train_487", "context": "Julia joined Michael Scott Paperless Company, a small New York based tech startup company, last month. Michael Scott Paperless recently received a valuation of ten million dollars. Julia is clearly the reason for the valuation.", "question": "Which of the following statements, if true, most weakens the argument?", "answers": "['Julia only started working two weeks before the valuation.', 'Julia is an expert in her field.', 'Michael Scott Paperless Company released an extremely popular mobile application shortly before hiring Julia.', 'Michael Scott Paperless Company is wildly overvalued.']", "label": 2 }, { "id": "train_488", "context": "When old-growth forests are cleared of tall trees, more sunlight reaches the forest floor. This results in a sharp increase in the population of leafy shrubs on which the mule deer depend for food. Yet mule deer herds that inhabit cleared forests are less well-nourished than are herds living in old-growth forests.", "question": "Which one of the following, if true, most helps to resolve the apparent paradox?", "answers": "['Insect parasites, such as certain species of ticks, that feed primarily on mule deer often dwell in trees, from which they drop onto passing deer.', 'Mule deer have enzyme-rich saliva and specialized digestive organs that enable the deer to digest tough plants inedible to other deer species.', 'Mule deer herds that inhabit cleared forests tend to have more females with young offspring and fewer adult males than do other mule deer populations.', \"As plants receive more sunlight, they produce higher amounts of tannins, compounds that inhibit digestion of the plants' proteins.\"]", "label": 3 }, { "id": "train_489", "context": "Letter to the editor: Recently, the city council passed an ordinance that prohibits loitering at the local shopping mall. The council' s declared goal was to eliminate overcrowding and alleviate pedestrian congestion, thereby improving the mall' s business and restoring its family-oriented image. But despite these claims, reducing overcrowding and congestion cannot be the actual goals of this measure, because even when fully implemented, the ordinance would not accomplish them.", "question": "Which one of the following most accurately describes a flaw in the argument's reasoning?", "answers": "['The argument takes for granted that something cannot be the goal of an action performed unless the action will in fact achieve that goal.', 'The argument takes for granted that an action that does not accomplish its stated goals will not have any beneficial effects.', \"The argument treats a condition that is necessary for achieving an action's stated goals as if this condition were sufficient for achieving these goals.\", 'The argument dismisses a claim because of its source rather than because of its content.']", "label": 0 }, { "id": "train_490", "context": "It is an absurd idea that whatever artistic endeavor the government refuses to support it does not allow, as one can see by rephrasing the statement to read: No one is allowed to create art without a government subsidy.", "question": "The pattern of reasoning in which one of the following is most similar to that in the argument above?", "answers": "['The notion that every scientist who has been supported by a government grant will be successful is absurd, as one can see by rewording it:No scientist is allowed to do research without a government grant.', 'The notion that every scientist who is supported by a government grant will be successful is absurd, as one can see by rewording it:No scientist lacking governmental support will be successful.', 'The claim that any driver who is not arrested does not break the law is absurd, as one can see by rewording it: Every driver who gets arrested has broken the law.', 'The claim that any driver who is not arrested does not break the law is absurd, as one can see by rewording it: Every driver who breaks the law gets arrested.']", "label": 3 }, { "id": "train_491", "context": "Though ice cream is an excellent source of calcium, dairy farmers report that during the past ten years there has been a sharp decline in ice cream sales. And during the same period, sales of cheddar cheese have nearly doubled. Ttherefore, more and more people must be choosing to increase their intake of calcium by eating cheddar cheese rather than ice cream.", "question": "The reasoning above is most vulnerable to criticism on the grounds that it", "answers": "[\"fails to produce statistical evidence supporting the dairy farmers' claims\", 'presumes, without providing justification, that ice cream is a better source of calcium than is cheddar cheese', 'fails to consider alternative explanations of the decline in sales of ice cream', 'relies solely on the testimony of individuals who are likely to be biased']", "label": 2 }, { "id": "train_492", "context": "Reducing stress lessens a person' s sensitivity to pain. This is the conclusion reached by researchers who played extended audiotapes to patients before they underwent surgery and afterward while they were recovering. One tape consisted of conversation; the other consisted of music. Those who listened only to the latter tape required less anesthesia during surgery and fewer painkillers afterward than those who listened only to the former tape.", "question": "Which one of the following is an assumption on which the researchers' reasoning depends?", "answers": "['All of the patients in the study listened to the same tape before surgery as they listened to after surgery.', 'Both anesthesia and painkillers tend to reduce stress.', 'The psychological effects of music are not changed by anesthesia or painkillers.', 'Listening to music reduces stress.']", "label": 3 }, { "id": "train_493", "context": "In virtually any industry, technological improvements increase labor productivity, which is the output of goods and services per person-hour worked. In Parland' s industries, labor productivity is significantly higher than it is in Vergia' s industries. Clearly, ttherefore, Parland' s industries must, on the whole, be further advanced technologically than Vergia' s are.", "question": "The argument is most vulnerable to which of the following criticisms?", "answers": "['It takes a condition to be the effect of something that happened only after the condition already existed.', 'It takes one possible cause of a condition to be the actual cause of that condition without considering any other possible causes.', 'It makes a distinction that presupposes the truth of the conclusion that is to be established', 'It presents as evidence in support of claim information that is inconsistent with other evidence presented in support of the same claim']", "label": 1 }, { "id": "train_494", "context": "In an experiment, Dr. Farouk studied houseplants that had flourished for years. Over a three-day period he spoke to the plants encouragingly. He then transplanted them outside into a garden and stopped talking to them. Although he continued watering and fertilizing the plants, they soon wilted. When they had not recovered after two days, Dr. Farouk became so concerned that he started talking to them every hour; the next day they began to recover.", "question": "Which one of the following is most strongly supported by the information above?", "answers": "['Watering plants too much is as hazardous to their well-being as not watering them enough.', 'Talking to plants enhances their health.', 'Changes in growing conditions can affect the flourishing of plants.', 'Plants always need at least two days to adjust to transplantation.']", "label": 2 }, { "id": "train_495", "context": "Biologists studying plant life in the prehistoric rainforest by utilizing samples of ancient ice and fossilized evidence of biological processes found that most of the plants removed an astonishingly small amount of carbon dioxide from the atmosphere. One biologist found that a certain tree currently found in the rainforest absorbs carbon dioxide from the atmosphere at a greater rate than any tree on the planet. The biologist hypothesized that the prevalence of this tree in the prehistoric rainforest accounts for the other plants' failure to remove larger amounts of carbon dioxide from the atmosphere.", "question": "Which of the following, if true, would most seriously undermine the biologist's hypothesis?", "answers": "['The amount of carbon dioxide in the atmosphere has remained steady since prehistoric times.', 'The trees currently found in the rainforest are largely hybrids and a product of crossbreeding that occurred in the late twentieth century.', 'The plant life in the prehistoric rainforest also contained larger amounts of chlorophyll than modern day plants.', 'The prehistoric humans did not seem to be adversely affected by the amount of carbon dioxide in the atmosphere.']", "label": 1 }, { "id": "train_496", "context": "TrueSave is a mail-order company that ships electronic products from its warehouses to customers worldwide. The company' s shipping manager is proposing that customer orders be packed with newer, more expensive packing materials that virtually eliminate damage during shipping. The manager argues that overall costs would essentially remain unaffected, since the extra cost of the new packing materials roughly equals the current cost of replacing products returned by customers because they arrived in damaged condition.", "question": "Which of the following would it be most important to ascertain in determining whether implementing the shipping manager's proposal would have the argued-for effect on costs?", "answers": "['Whether the products shipped by TrueSave are more vulnerable to incurring damage during shipping than are typical electronic products', 'Whether a sizable proportion of returned items are returned because of damage already present when those items were packed for shipping', 'Whether electronic products are damaged more frequently in transit than are most other products shipped by mail-order companies', 'Whether TrueSave continually monitors the performance of the shipping companies it uses to ship products to its customers']", "label": 1 }, { "id": "train_497", "context": "That the application of new technology can increase the productivity of existing coal mines is demonstrated by the case of Tribnia' s coal industry. Coal output per miner in Tribnia is double what it was five years ago, even though no new mines have opened.", "question": "Which of the following can be properly concluded from the statement about coal output per miner in the passage?", "answers": "['Any individual Tribnian coal mine that achieved an increase in overall output in the past five years has also experienced an increase in output per miner.', 'In Tribnia the cost of producing a given quantity of coal has declined over the past five years.', \"If the number of miners working in Tribnian coal mines has remained constant in the past five years, Tribnia's total coal production has doubled in that period of time.\", 'If any new coal mines had opened in Tribnia in the past five years, then the increase in output per miner would have been even greater than it actually was.']", "label": 2 }, { "id": "train_498", "context": "Atrens: An early entomologist observed ants carrying particles to neighboring ant colonies and inferred that the ants were bringing food to their neighbors. Further research, however, revealed that the ants were emptying their own colony' s dumping site. Thus, the early entomologist was wrong.", "question": "Atrens's conclusion follows logically if which one of the following is assumed?", "answers": "['There is only weak evidence for the view that ants have the capacity to make use of objects as gifts.', 'Ant dumping sites do not contain particles that could be used as food.', 'The entomologist cited retracted his conclusion when it was determined that the particles the ants carried came from their dumping site.', 'Ant societies do not interact in all the same ways that human societies interact.']", "label": 1 }, { "id": "train_499", "context": "Fraenger' s assertion that the artist Hieronymus Bosch belonged to the Brethren of the Free Spirit, a nonmainstream religious group, is unlikely to be correct. Fraenger' s hypothesis explains much of Bosch' s unusual subject matter. However, there is evidence that Bosch was a member of a mainstream church, and no evidence that he was a member of the Brethren.", "question": "The statement that there is no evidence that Bosch was a member of the Brethren figures in the argument in which one of the following ways?", "answers": [ "It is used to dispute Fraenger's hypothesis by questioning Fraenger's credibility.", "It is intended to help show that Bosch's choice of subject matter remains unexplained.", "It is intended to cast doubt on Fraenger's hypothesis by questioning the sufficiency of Fraenger's evidence.", "It is a premise that, when combined with the other premises, guarantees the falsity of Fraenger's assertion." ], "label": 2 }, { "id": "train_500", "context": "A year ago the government reduced the highway speed limit, and in the year since, there have been significantly fewer highway fatalities than there were in the previous year. Ttherefore, speed limit reduction can reduce traffic fatalities.", "question": "The argument is most vulnerable to the criticism that it takes for granted that", "answers": "['highway traffic has not increased over the past year', 'the number of traffic fatalities the year before the new speed limit was introduced was not abnormally high', 'the new speed limit was more strictly enforced than the old', 'the majority of drivers obeyed the new speed limit']", "label": 1 }, { "id": "train_501", "context": "Engineer: Semiplaning monohulls are a new kind of ship that can attain twice the speed of conventional ships. Due to increased fuel needs, transportation will be much more expensive on semiplaning monohulls than on conventional ships. Similarly, travel on jet airplanes was more expensive than travel on other planes at first, but jet airplanes still attracted enough passengers to be profitable, because they offered greater speed and reliability. Semiplaning monohulls offer the same advantages over traditional ships. Thus they will probably be profitable as well.", "question": "Which one of the following most accurately describes the role played in the engineer's argument by the statement that transportation will be much more expensive on semiplaning monohulls than on traditional ships?", "answers": [ "It draws an analogy between semiplaning monohulls and conventional ships that constitutes an objection to the argument's main conclusion, one that is subsequently rejected by appeal to another analogy.", "It constitutes a potential objection to the argument's main conclusion, but is subsequently countered by an analogy drawn between ships and airplanes.", "It draws a distinction between characteristics of semiplaning monohulls and characteristics of conventional ships that the argument's main conclusion compares to a distinction between types of airplanes.", "It serves as one of two analogies drawn between semiplaning monohulls and jet airplanes, which function together to support the argument's main conclusion." ], "label": 1 }, { "id": "train_502", "context": "Fish currently costs about the same at seafood stores throughout Eastville and its surrounding suburbs. Seafood stores buy fish from the same wholesalers and at the same prices, and other business expenses have also been about the same. But new tax breaks will substantially lower the cost of doing business within the city. Ttherefore, in the future, profit margins will be higher at seafood stores within the city than at suburban seafood stores.", "question": "For the purposes of evaluating the argument, it would be most useful to know whether", "answers": "['Seafood stores within the city will in the future set prices that are lower than those at suburban seafood stores', 'Any people who currently own seafood stores in the suburbs surrounding Eastville will relocate their businesses nearer to the city', 'The wholesale price of fish is likely to fall in the future', 'More fish wholesalers are located within the city than in the surrounding suburbs']", "label": 0 }, { "id": "train_503", "context": "A professional baseball team manager, in order to have the funds to sign a new second-baseman, discreetly arranged to trade one of the most popular outfielders on the team for a lesser-known player and an undisclosed amount of money. The manager secretly considered the outfielder to be overrated and overpaid. Reporters forcefully criticized the trade, arguing that the team had lost its best player and that the manager had violated his fiduciary duties to the team and the fans. A few weeks after being traded, the outfielder was retraded, for twice the value received by the original team manager. Plainly, the outfielder' s price shows that the reporters' criticism of the manager' s action was accurate.", "question": "The reasoning in the argument is vulnerable to the criticism that the argument does which one of the following?", "answers": "['The argument bases its conclusion on what the best decision is for the present on uncertain projections about what the best decision will be for the future.', 'The argument rejects a well-established way of achieving an end without explaining why an unconventional way is better suited for achieving the end.', 'The argument ignores the opinions of expert reporters in the field of baseball when there is no superior source of information.', \"The argument bases its conclusion on facts that could, considering the circumstances, have been the product of circumstances other than those presumed by the argument's proponents.\"]", "label": 3 }, { "id": "train_504", "context": "Quality control investigator: Upon testing samples of products from our supplier that were sent by our field inspectors from various manufacturing locations, our laboratory discovered that over 20 percent of the samples were defective. Since our supplier is contractually required to limit the rate of defects among items it manufactures for us to below 5 percent, it has violated its contract with us.", "question": "The reasoning in the quality control investigator's argument is flawed in that the argument", "answers": "['presumes, without providing justification, that the field inspectors were just as likely to choose a defective item for testing as they were to choose a nondefective item', 'presumes, without providing justification, that the field inspectors made an equal number of visits to each of the various manufacturing sites of the supplier', 'overlooks the possibility that the field inspectors tend to choose items for testing that they suspect are defective', 'bases its conclusion on too small a sample of items tested by the laboratory']", "label": 2 }, { "id": "train_505", "context": "Pediatrician: Swollen tonsils give rise to breathing problems during sleep, and the surgical removal of children' s swollen tonsils has been shown to alleviate sleep disturbances. So removing children' s tonsils before swelling even occurs will ensure that the children do not experience any breathing problems during sleep.", "question": "The pediatrician's argument is most vulnerable to the criticism that it", "answers": "[\"fails to consider the possibility that there may be other medical reasons for surgically removing a child's tonsils\", 'infers from the fact that an action has a certain effect that the action is intended to produce that effect', 'fails to consider the possibility that some breathing problems during sleep may be caused by something other than swollen tonsils', 'relies on an inappropriate appeal to authority']", "label": 2 }, { "id": "train_506", "context": "The Levant -- the area that borders the eastern Mediterranean-was heavily populated in prehistoric times. The southern Levant was abandoned about 6, 000 years ago, although the northern Levant, which shared the same climate, remained heavily populated. Recently archaeologists have hypothesized that the sudden depopulation in the southern Levant was due to an economic collapse resulting from deforestation.", "question": "If the statements above are true and the archaeologists' hypothesis is correct, which one of the following CANNOT be true?", "answers": "['The sheep and goats herded by the peoples of the southern Levant until 6, 000 years ago grazed extensively on the seedlings and saplings of indigenous tree species.', 'Carbon dating of organic remains from the southern Levant reliably demonstrates that there were no forests present in that area prior to 6, 000 years ago.', 'Organic remains from the northern Levant reliably indicate that tree species flourished there without interruption during the period when the southern Levant was being abandoned.', 'Since there are few traces of either quarried stone or of mud brick in buildings excavated in the southern Levant, it is likely that the buildings built there prior to 6, 000 years ago were made almost entirely of timber.']", "label": 1 }, { "id": "train_507", "context": "If you know a lot about history, it will be easy for you to impress people who are intellectuals. But unfortunately, you will not know much about history if you have not, for example, read a large number of history books. Ttherefore, if you are not well versed in history due to a lack of reading, it will not be easy for you to impress people who are intellectuals.", "question": "The argument's reasoning is flawed because the argument overlooks the possibility that", "answers": "['it is more important to impress people who are not intellectuals than people who are intellectuals', 'many intellectuals are not widely read in history', 'there are other easy ways to impress intellectuals that do not involve knowing history', 'there are people who learn about history who do not impress intellectuals']", "label": 2 }, { "id": "train_508", "context": "The most successful economies have been, and will continue to be, those that train as many people as possible in the human skills required to research, to develop, and to apply new technology. Japan is a model for this sort of training effort. Europe as a whole is in a weaker position: there is a shortage of skilled labor trained to use the new technologies, and there are not enough scientists able to develop and apply the technology. However, even in Japan there is a shortage of technically qualified people, and, like most European countries, Japan has far too many workers qualified to perform only menial tasks.", "question": "Which one of the following can be properly inferred from the passage?", "answers": "['To be economically more successful, Europe needs to train more people in the new technologies.', \"Japan's successful economy depends upon an uncommonly narrow base of highly skilled labor.\", \"Japan is not the best country against which to measure a country's economic success.\", 'European countries have economies that are more successful than those of most other countries.']", "label": 0 }, { "id": "train_509", "context": "Not all tenured faculty are full professors. Ttherefore, although every faculty member in the linguistics department has tenure, it must be the case that not all of the faculty members in the linguistics department are full professors.", "question": "The flawed pattern of reasoning exhibited by the argument above is most similar to that exhibited by which one of the following?", "answers": "['Although some cathedrals are not built of stone, every cathedral is impressive. Ttherefore, buildings can be impressive even though they are not built of stone.', 'Not all public buildings are well designed, but some poorly designed public buildings were originally intended for private use. Ttherefore, the poorly designed public buildings were all originally designed for private use.', 'All municipal hospital buildings are massive, but not all municipal hospital buildings are forbidding in appearance. Ttherefore, massive buildings need not present a forbidding appearance.', 'Although some buildings designed by famous architects are not well proportioned, all government buildings are designed by famous architects. Ttherefore, some government buildings are not well proportioned.']", "label": 3 }, { "id": "train_510", "context": "Councilperson X: We have an obligation to help ensure that electricity rates are the lowest possible. Since the proposed design for a new generating station would clearly allow for the lowest rates, it must be the design we endorse if we agree that we have no choice but to approve construction of a new plant. Councilperson Y: Helping to ensure the lowest electricity rates is not the council' s only job; we also have an obligation not to lower the quality of life of our community. A plant of the type specified by the design would damage our community' s air quality to such an extent that the benefit of lower rates would be outweighed.", "question": "Which one of the following is an issue about which the two councilpersons disagree?", "answers": "[\"It is the council's responsibility to improve the community's quality of life.\", 'If a new generating station is to be built, the council should endorse a plant of the type specified by the design in question.', 'A plant of the type specified by the design in question would allow for the lowest electricity rates.', 'The council should recommend the building of a new generating station.']", "label": 1 }, { "id": "train_511", "context": "Chemist: The molecules of a certain weed-killer are always present in two forms, one the mirror image of the other. One form of the molecule kills weeds, while the other has no effect on them. As a result, the effectiveness of the weed-killer in a given situation is heavily influenced by which of the two forms is more concentrated in the soil, which in turn varies widely because local soil conditions will usually favor the breakdown of one form or the other. Thus, much of the data on the effects of this weed-killer are probably misleading.", "question": "Which one of the following, if true, most strengthens the chemist's argument?", "answers": "[\"Almost all of the data on the effects of the weed-killer are drawn from laboratory studies in which both forms of the weed-killer's molecules are equally concentrated in the soil and equally likely to break down in that soil.\", 'In general, if the molecules of a weed-killer are always present in two forms, then it is likely that weeds are killed by one of those two forms but unaffected by the other.', \"Data on the weed-killer's effects that rely solely on the examination of the effects of only one of the two forms of the weed-killer's molecules will almost certainly be misleading.\", \"Of the two forms of the weed-killer's molecules, the one that kills weeds is found in most local soil conditions to be the more concentrated form.\"]", "label": 0 }, { "id": "train_512", "context": "Preschool children who spend the day in daycare nurseries are ill more often than those who do not. They catch many common illnesses, to which they are exposed by other children in the nurseries. However, when these children reach school age they tend to be ill less often than their classmates who did not spend the day in daycare nurseries during their preschool years.", "question": "Which one of the following, if true, best explains the discrepancy in the information above?", "answers": "['The number of infectious illnesses that children in a daycare nursery or school develop is roughly proportional to the number of children in the facility, and daycare nurseries are smaller than most schools.', 'Although in general the illnesses that children contract through contact with other children at daycare nurseries are not serious, some of those illnesses if untreated have serious complications.', 'Those children who have older siblings are likely to catch any common infectious illnesses that their older siblings have.', 'By school age, children who have been in daycare nurseries have developed the immunities to common childhood illnesses that children who have not been in such nurseries have yet to develop.']", "label": 3 }, { "id": "train_513", "context": "Diomedes Motors has just decided to start using Rapilux Tires on most models of its new cars. The tires cost the same as the previous tires, and the change in tires will not change the sticker price of any car, nor will it change the profit on the sale of any particular car. Nevertheless, the CEO of Diomedes Motors expects this change in tires to increase Diomedes' profits in the coming year.", "question": "Which of the following, if true, provides the best reason for the expectation?", "answers": "['The change to Rapilux Tires in most Diomedes models moves those models to the highest safety rating category; several national consumer advocacy groups give focused advertising to cars in this category.', 'Some of the tires previously used on these particular Diomedes models are being phased out by those tire manufacturers, and would have to be replaced anyway, and no tire that would fit these Diomedes Models is any cheaper than the Rapilux Tires chosen.', 'The new Rapilux Tires facilitate loading the new cars onto the trucks that deliver them to the Diomedes showroom.', 'Few car buyers are likely to decide against buying a Diomedes Motors model simply because it now comes with Rapilux Tires.']", "label": 0 }, { "id": "train_514", "context": "Safety inspector: The number of laboratory samples of rabies virus sent through the university delivery service has recently grown dangerously high. We need to limit this use of the service. Biologist: There is no need for a limit. The university delivery service has been handling rabies virus samples for 20 years with no accidents.", "question": "As a rebuttal of the safety inspector's claim, the biologist's reasoning is flawed in that it", "answers": "['fails to explain why the number of samples sent through the service has increased', \"offers no defense of the university's need for continued delivery of rabies virus samples\", 'overlooks the possibility that there has been a change in the hazardousness of the rabies virus samples themselves', 'does not address the potential for harm that is posed by the recent increase in the number of samples sent through the service']", "label": 3 }, { "id": "train_515", "context": "A new machine for harvesting corn will allow rows to be planted only fifteen inches apart, instead of the usual thirty inches. Corn planted this closely will produce lower yields per plant. Nevertheless, the new machine will allow corn growers to double their profits per acre because __.", "question": "Which of the following most logically completes the argument?", "answers": "['with the larger number of plants growing per acre, more fertilizer will be required', 'with the closer spacing of the rows, the acreage on which corn is planted will be utilized much more intensively than it was before, requiring more frequent fallow years in which corn fields are left unplanted', 'with the closer spacing of the rows, the growing corn plants will quickly form a dense canopy of leaves, which will, by shading the ground, minimize the need for costly weed control and irrigation', 'with the closer spacing of the rows, corn plants will be forced to grow taller because of increased competition for sunlight from neighboring corn plants']", "label": 2 }, { "id": "train_516", "context": "Mammals cannot digest cellulose and ttherefore cannot directly obtain glucose from wood. Mushrooms can, however; and some mushrooms use cellulose to make highly branched polymers, the branches of which are a form of glucose called beta-glucans. Beta-glucan extracts from various types of mushrooms slow, reverse, or prevent the growth of cancerous tumors in mammals, and the antitumor activity of beta-glucans increases as the degree of branching increases. These extracts prevent tumor growth not by killing cancer cells directly but by increasing immune-cell activity.", "question": "Which one of the following is most strongly supported by the information above?", "answers": "['If extracts from a type of mushroom slow, reverse, or prevent the growth of cancerous tumors in mammals, then the mushroom is capable of using cellulose to make beta-glucans.', 'Any organism capable of obtaining glucose from wood can use cellulose to make beta-glucans.', 'The greater the degree of branching of beta-glucans, the greater the degree of immune-cell activity it triggers in mammals.', 'Mammals obtain no beneficial health effects from eating cellulose.']", "label": 2 }, { "id": "train_517", "context": "Between 1951 and 1963, it was illegal in the country 25. of Geronia to manufacture, sell, or transport any alcoholic beverages. Despite this prohibition, however, the death rate from diseases related to excessive alcohol consumption was higher during the first five years of the period than it was during the five years prior to 1951. Ttherefore, the attempt to prevent alcohol use merely made people want and use alcohol more than they would have if it had not been forbidden.", "question": "Each of the following, if true, weakens the argument EXCEPT:", "answers": "['Death from an alcohol-related disease generally does not occur until five to ten years after the onset of excessive alcohol consumption.', 'The diseases that can be caused by excessive alcohol consumption can also be caused by other kinds of behavior that increased between 1951 and 1963.', 'Many who died of alcohol-related diseases between 1951 and 1963 consumed illegally imported alcoholic beverages produced by the same methods as those used within Geronia.', 'The death rate resulting from alcohol-related diseases increased just as sharply during the ten years before and the ten years after the prohibition of alcohol as it did during the years of prohibition.']", "label": 2 }, { "id": "train_518", "context": "Forty miles to the west of Tucson, a developer attempted to develop an \"oasis\" glade in the Sonoran Desert. He created a glade that, the developer claims, is as lush and plant-filled as the ambient 35 degree C temperatures allow. In particular, the developer planted several native fruit-bearing plants, such as prickly-pear cactus. Once the glade was established, a number of desert birds and mammals would regularly inhabit the glade, often eating most of the fruit that falls from the plants.", "question": "The considerations given best serve as part of an argument that:", "answers": "['the presence of the desert animals in the glade do not serve as evidence against the claim of the developer.', 'most of these desert animals in the glade would have perished if the glade had not been created in that location.', 'the shade from all the plants in the glade measurably reduces the ambient temperature within the glade.', \"enclosing the fruit -- bearing plants in wire cages to prevent consumption by desert animals would allow for fruit sales to cover the cost of the glade's development.\"]", "label": 0 }, { "id": "train_519", "context": "Some scientists believe that 65 million years ago an asteroid struck what is now the Yucatan Peninsula, thereby causing extinction of the dinosaurs. These scientists have established that such a strike could have hurled enough debris into the atmosphere to block sunlight and cool the atmosphere. Without adequate sunlight, food sources for herbivorous dinosaurs would have disappeared, and no dinosaurs could have survived a prolonged period of low temperatures. These same scientists, however, have also established that most debris launched by the asteroid would have settled to the ground within six months, too soon for the plants to disappear or the dinosaurs to freeze.", "question": "Which one of the following, if true, most helps to resolve the apparent discrepancy between the scientists' beliefs and the scientists' results, as described above?", "answers": "['Dinosaurs were susceptible to fatal respiratory problems caused by contamination of the air by asteroid debris.', \"A cloud of debris capable of diminishing sunlight by 20 percent would have cooled the earth's surface by 7 to 10 degrees Celsius.\", 'Loss of the herbivorous dinosaurs would have deprived the carnivorous dinosaurs of their food source.', 'Dinosaurs inhabited most landmasses on the planet but were not especially abundant in the area of the asteroid strike.']", "label": 0 }, { "id": "train_520", "context": "The five senses have traditionally been viewed as distinct yet complementary. Each sense is thought to have its own range of stimuli that are incapable of stimulating the other senses. However, recent research has discovered that some people taste a banana and claim that they are tasting blue, or see a color and say that it has a specific smell. This shows that such people, called synesthesiacs, have senses that do not respect the usual boundaries between the five recognized senses.", "question": "Which one of the following statements, if true, most seriously weakens the argument?", "answers": "['Recent evidence strongly suggests that there are other senses besides sight, touch, smell, hearing, and taste.', 'Synesthesiacs can be temporarily rid of their synesthetic experiences by the use of drugs.', 'The particular ways in which sensory experiences overlap in synesthesiacs follow a definite pattern.', 'Synesthesiacs demonstrate a general, systematic impairment in their ability to use and understand words.']", "label": 3 }, { "id": "train_521", "context": "Multiple use\" refers to the utilization of natural resources in combinations that will best meet the present and future needs of the public. Designating land as a wilderness area does not necessarily violate the multiple-use philosophy, for even when such use does not provide the greatest dollar return, it can provide the greatest overall benefit from that site.", "question": "Which one of the following is an assumption required by the argument?", "answers": "['The multiple-use philosophy takes into account some nonfinancial needs of the public.', 'Natural resources should be used in combinations that will most greatly benefit present and future generations.', 'The present and future needs of the public would best be met by designating greater numbers of wilderness areas.', 'Designating a wilderness area prevents any exploitation of natural resources in that area.']", "label": 0 }, { "id": "train_522", "context": "The Ergonomic Society conducted a study that indicated that many people develop severe back problems during adulthood, and that virtually all such people who received chiropractic treatment showed great improvement. Ttherefore, in order to minimize the proportion of the population that suffers from back pain, the Ergonomic Society recommended that chiropractic treatment be directed toward those adults who suffer from severe back problems.", "question": "Which of the following is an assumption on which the argument depends?", "answers": "['Severe back pain and other problems often cause individuals to miss workdays.', 'Chiropractic treatment is more effective in treating severe back problems when utilized over a long period of time, as opposed to sporadically.', 'Individuals who receive chiropractic or other treatment prior to developing severe back problems are not less likely to develop back pain than those who do not.', 'Any person who receives chiropractic treatment for back pain may also benefit from other forms of treatment.']", "label": 2 }, { "id": "train_523", "context": "Essayist: It is much less difficult to live an enjoyable life if one is able to make lifestyle choices that accord with one' s personal beliefs and then see those choices accepted by others. It is possible for people to find this kind of acceptance by choosing friends and associates who share many of their personal beliefs. Thus, no one should be denied the freedom to choose the people with whom he or she will associate.", "question": "Which one of the following principles, if valid, most helps to justify the essayist's argument?", "answers": "['One may choose for oneself the people with whom one will associate, if doing so could make it easier to live an enjoyable life.', 'No one should be denied the freedom to make lifestyle choices that accord with his or her personal beliefs.', 'If having a given freedom could make it less difficult for someone to live an enjoyable life, then no one should be denied that freedom.', 'No one whose enjoyment of life depends, at least in part, on friends and associates who share many of the same personal beliefs should be deliberately prevented from having such friends and associates.']", "label": 2 }, { "id": "train_524", "context": "Politician: The mandatory jail sentences that became law two years ago for certain crimes have enhanced the integrity of our system of justice, for no longer are there two kinds of justice, the kind dispensed by lenient judges and the kind dispensed by severe ones. Public advocate: But with judges stripped of discretionary powers, there can be no leniency even where it would be appropriate. So juries now sometimes acquit a given defendant solely because the jurors feel that the mandatory sentence would be too harsh. Those juries, then, do not return an accurate verdict on the defendant' s guilt. This is why it is imperative that the legislation instituting mandatory jail sentences be repealed.", "question": "The public advocate responds to the politician's argument by doing which one of the following?", "answers": "['arguing that leniency is not a trait of individuals but that, rather, it is a property of certain kinds of decisions', 'charging that the politician exaggerated the severity of a problem in order to justify a sweeping solution', \"claiming that the politician's evidence, properly analyzed, has no bearing on the conclusion the politician derives from it\", \"arguing that an analysis of the consequences of certain legislation undermines the politician's conclusion\"]", "label": 3 }, { "id": "train_525", "context": "Policy analyst: Increasing the size of a police force is only a stopgap method of crime prevention; it does not get at the root causes of crime. Ttherefore, city officials should not respond to rising crime rates by increasing the size of their city' s police force.", "question": "The flawed reasoning in which one of the following arguments most closely resembles the flawed reasoning in the policy analyst's argument?", "answers": "[\"Because taking this drug does not cure the disease for which it is prescribed, but only reduces the disease's most harmful effects, doctors should not continue to prescribe this drug.\", \"We will never fully understand what causes people to engage in criminal activity. Ttherefore, we should investigate other ways to improve society's ability to combat crime.\", 'Although less effective in preventing theft than security guards, burglar alarm systems are more affordable to maintain. Because the greater loss from theft when alarms are used is outweighed by their lower cost, companies are advised always to use burglar alarm systems.', 'Swamps play an important role in allaying the harsh effects of floods because they absorb a great deal of water. Although dams prevent many floods, they worsen the effects of the greatest floods by drying up swamps. Thus dams should not be built.']", "label": 0 }, { "id": "train_526", "context": "In preparation for a large-scale tree-planting project, Thomasville has recorded the location, species, and condition of every tree in its downtown area. The results will be compared with results of the last such tree census, taken 20 years ago. Planting trees primarily from the species that turns out to have the best record of survival will assure downtown Thomasville of an abundant tree population 20 years from now.", "question": "Which of the following, if true, most seriously undermines the claim that the tree-planting proposal will assure an abundant tree population?", "answers": "['Because of strict laws governing industrial development, environmental conditions in Thomasville are unlikely to become harsher for trees than they have been during the last 20 years.', 'A number of trees from one of the species widespread in Thomasville 20 years ago reached the end of their natural life span in the intervening period.', 'Thomasville has reliable records to show which trees in existence 20 years ago were cut down to permit new construction downtown.', 'A city with a high proportion of any one species of trees can have its tree population devastated by one outbreak of a disease.']", "label": 3 }, { "id": "train_527", "context": "When butterfat was considered nutritious and healthful, a law was enacted requiring that manufacturers use the term \"imitation butter\" to indicate butter whose butterfat content had been diminished through the addition of water. Today, it is known that the high cholesterol content of butterfat makes it harmful to human health. Since the public should be encouraged to eat foods with lower rather than higher butterfat content and since the term \"imitation\" with its connotations of falsity deters many people from purchasing products so designated, manufacturers who wish to give reduced-butterfat butter the more appealing name of \"lite butter\" should be allowed to do so.", "question": "Which one of the following, if true, most seriously undermines the argument?", "answers": "['Most people deterred from eating \"imitation butter\" because of its name choose alternatives with a lower butterfat content than this product has.', 'Some individuals who need to reduce their intake of cholesterol are not deterred from using the reduced-butterfat product by the negative connotations of the term \"imitation. \"', 'The manufacturers who wish to call their product \"lite butter\" plan to change the composition of the product so that it contains more water than it now does.', 'The manufacturers who prefer to use the word \"lite\" instead of \"imitation\" are motivated principally by the financial interest of their stockholders.']", "label": 0 }, { "id": "train_528", "context": "Although tales of wonder and the fantastic are integral to all world literatures, only recently has the fantasy genre had a commercial resurgence in North America. During the last 20 years, sales of fantasy-fiction books written for adults have gone from 1 to 10 percent of total adult-fiction sales. At the same time, the number of favorable reviews of fantasy books has increased markedly. Some booksellers say that the increased sales of fantasy books written for adults can be traced to the increased favorable attention given the genre by book reviewers.", "question": "Which one of the following, if true, undermines the booksellers' explanation of the growth in sales of fantasy-fiction books for adults?", "answers": "['Although the increase in the percentage of fantasy books sold has been substantial, publishers estimate that sales could increase even further.', 'Publishers often select a manuscript on the basis of whether they think that the published book will receive favorable reviews by book reviewers.', 'Few readers of fantasy fiction read book reviews, and even fewer select books to purchase on the basis of those reviews.', 'Many of the book reviews of new fantasy- fiction novels also mention great fantasy novels of the past.']", "label": 2 }, { "id": "train_529", "context": "Air pollution from a rural factory may have caused increased levels of carbon dioxide in the air, but the argument that the increased level of carbon dioxide is harming the local livestock is clearly without merit; the amount of carbon dioxide near the factory is, in fact, less than that found in cities like Los Angeles every day.", "question": "The argument is vulnerable to criticism because it", "answers": "['uses generalizations about a class to draw conclusions about an individual entity in that class', \"uses a fact to support a conclusion when that fact's relevance has yet to be determined\", \"in fact, grants a concession that is fatal to the argument's conclusion\", 'supports its conclusion by using two conflicting claims']", "label": 1 }, { "id": "train_530", "context": "Columnist: Contrary to what many people believe, the number of species on Earth is probably not dwindling. Extinction is a natural process, and about as many species are likely to go extinct this year as went extinct in 1970. But the emergence of new species is also a natural process; there is no reason to doubt that new species are emerging at about the same rate as they have been for the last several centuries.", "question": "Which one of the following, if true, most weakens the columnist's argument?", "answers": "['In 1970 fewer new species emerged than went extinct.', 'The regions of the world where new species tend to emerge at the highest rate are also where species tend to go extinct at the highest rate.', 'Scientists are now better able to identify species facing serious risk of extinction than they were in 1970.', 'The vast majority of the species that have ever existed are now extinct.']", "label": 0 }, { "id": "train_531", "context": "The Rienzi, a passenger ship, sank as a result of a hole in its hull, possibly caused by sabotage. Normally, when a holed ship sinks as rapidly as the Rienzi did, water does not enter the ship quickly enough for the ship to be fully flooded when it reaches the ocean floor. Full flooding can be achieved, however, by sabotage. Any ship that sinks deep into the ocean when not fully flooded will implode. Deep-sea photographs, taken of the sunken Rienzi where it rests on the ocean floor, reveal that the Rienzi did not implode.", "question": "Which one of the following must be true on the basis of the information above?", "answers": "['If the Rienzi was not sunk by sabotage, water flooded into it unusually fast.', 'The Rienzi was so strongly constructed as to resist imploding under deep-sea pressure.', 'If the Rienzi had sunk more slowly, it would have imploded.', 'If the Rienzi became fully flooded, it did so only after it reached the ocean floor.']", "label": 0 }, { "id": "train_532", "context": "The workers at Bell Manufacturing will shortly go on strike unless the management increases their wages. As Bell' s president is well aware, however, in order to increase the workers' wages, Bell would have to sell off some of its subsidiaries. So, some of Bell' s subsidiaries will be sold.", "question": "The conclusion above is properly drawn if which one of the following is assumed?", "answers": "[\"Bell's workers will not accept a package of improved benefits in place of their desired wage increase.\", 'The workers at Bell Manufacturing will not be going on strike.', \"Bell's management will refuse to increase its workers' wages.\", 'Bell Manufacturing will begin to suffer increased losses.']", "label": 1 }, { "id": "train_533", "context": "Congress has passed a law that scales back the tax credits offered to purchasers of hybrid cars. Because of this, the number of hybrid cars sold each year, which has approximately doubled in each of the last five years, is likely to flatten in the next few years.", "question": "Which of the following statements, if true, most weakens the conclusion that the number of hybrid cars sold annually is likely to flatten in the next few years?", "answers": "['Manufacturers have significantly increased their budgets for advertisements touting the fuel efficiency of hybrid cars.', 'Hybrid vehicles have a reputation for having less power than comparable non-hybrid vehicles.', 'Most people buy hybrid cars for reasons other than to save money in the long run.', 'Gasoline prices are projected to fall over the next few years.']", "label": 2 }, { "id": "train_534", "context": "Psychologist: Phonemic awareness, or the knowledge that spoken language can be broken into component sounds, is essential for learning to read an alphabetic language. But one also needs to learn how sounds are symbolically represented by means of letters; otherwise, phonemic awareness will not translate into the ability to read an alphabetic language. Yet many children who are taught by the whole-language method, which emphasizes the ways words sound, learn to read alphabetic languages.", "question": "Which one of the following can be properly inferred from the psychologist's statements?", "answers": "['The whole-language method invariably succeeds in teaching awareness of how spoken language can be broken into component sounds.', 'Some children who are taught by the whole-language method are not prevented from learning how sounds are represented by means of letters.', 'The whole-language method succeeds in teaching many children how to represent sounds symbolically by means of letters.', 'When the whole-language method succeeds in teaching someone how to represent sounds by means of letters, that person acquires the ability to read an alphabetic language.']", "label": 1 }, { "id": "train_535", "context": "Five years ago, as part of a plan to encourage citizens of Levaska to increase the amount of money they put into savings, Levaska' s government introduced special savings accounts in which up to $3, 000 a year can be saved with no tax due on the interest unless money is withdrawn before the account holder reaches the age of sixty-five. Millions of dollars have accumulated in the special accounts, so the government' s plan is obviously working.", "question": "Which of the following, if true, most seriously weakens the argument?", "answers": "['The rate at which interest earned on money deposited in regular savings accounts is taxed depends on the income bracket of the account holder.', 'Many Levaskans who already had long-term savings have steadily been transferring those savings into the special accounts.', \"Many of the economists who now claim that the government's plan has been successful criticized it when it was introduced.\", 'A substantial number of Levaskans have withdrawn at least some of the money they had invested in the special accounts.']", "label": 1 }, { "id": "train_536", "context": "A company plans to develop a prototype weeding machine that uses cutting blades with optical sensors and microprocessors that distinguish weeds from crop plants by differences in shade of color. The inventor of the machine claims that it will reduce labor costs by virtually eliminating the need for manual weeding.", "question": "Which of the following is a consideration in favor of the company's implementing its plan to develop the prototype?", "answers": "['Selection and genetic manipulation allow plants of virtually any species to be economically bred to have a distinctive shade of color without altering their other characteristics.', 'There is a considerable degree of variation in shade of color between weeds of different species.', 'Farm laborers who are responsible for the manual weeding of crops carry out other agricultural duties at times in the growing season when extensive weeding is not necessary.', 'When crops are weeded manually, overall size and leaf shape are taken into account in distinguishing crop plants from weeds.']", "label": 0 }, { "id": "train_537", "context": "Numerous books describe the rules of etiquette. Usually the authors of such books merely codify standards of behavior by classifying various behaviors as polite or rude. However, this suggests that there is a single, objective standard of politeness. Clearly, standards of politeness vary from culture to culture, so it is absurd to label any one set of behaviors as correct and others as incorrect.", "question": "The reasoning in the argument is most vulnerable to criticism on the grounds that the argument", "answers": "['overlooks the possibility that authors of etiquette books are purporting to state what is correct behavior for one particular culture only', 'reaches a conclusion about how people actually behave on the basis of assertions regarding how they ought to behave', 'fails to justify its presumption regarding the influence of rules of etiquette on individual behavior', 'bases a generalization about all books of etiquette on the actions of a few authors']", "label": 0 }, { "id": "train_538", "context": "I. Room air conditioners produced by Japanese manufacturers tend to be more reliable than those produced by United States manufacturers. II. The average lifetime of room air conditioners produced by United States manufacturers is about fifteen years, the same as that of room air conditioners produced by Japanese manufacturers.", "question": "Which one of the following, if true, would best reconcile the two statements above?", "answers": "['Reliability is a measure of how long a product functions without needing repair.', 'Damage to room air conditioners during shipping and installation does not occur with great frequency in the United States or in Japan.', 'Production facilities of firms designated as United States manufacturers are not all located in the United States.', 'Japanese manufacturers often use more reliable components in their room air conditioners than do United States manufacturers.']", "label": 0 }, { "id": "train_539", "context": "The proposed change to the patent system is bound to have a chilling effect on scientific research. Under current rules, researchers have one full year after the initial publication of a new discovery to patent the discovery. This allows research results to be shared widely prior to the patent application. The proposed change would have the application precede initial publication, which would delay the communication of discoveries.", "question": "The conclusion drawn above follows logically if which one of the following is assumed?", "answers": "['The proposed change will encourage more patent applications to be filed.', 'The current rules for patent applications facilitate progress in scientific research by rewarding the communication of discoveries.', 'Most researchers oppose the proposed change to the patent system.', 'Delays in the communication of discoveries will have a chilling effect on scientific research.']", "label": 3 }, { "id": "train_540", "context": "The Maxilux car company' s design for its new luxury model, the Max 100, included a special design for the tires that was intended to complement the model' s image. The winning bid for supplying these tires was submitted by Rubco. Analysts concluded that the bid would only just cover Rubco' s costs on the tires, but Rubco executives claim that winning the bid will actually make a profit for the company.", "question": "Which of the following, if true, most strongly justifies the claim made by Rubco's executives?", "answers": "[\"When Maxilux awarded the tire contract to Rubco, the only criterion on which Rubco's bid was clearly ahead of its competitors' bids was price.\", 'In any Maxilux model, the spare tire is exactly the same make and model as the tires that are mounted on the wheels.', 'The production facilities for the Max 100 and those for the tires to be supplied by Rubco are located very near each other.', 'When people who have purchased a carefully designed luxury automobile need to replace a worn part of it, they almost invariably replace it with a part of exactly the same make and type.']", "label": 3 }, { "id": "train_541", "context": "Antiwhaling activist: Not all species of whales are threatened with extinction. Nevertheless, given the highly mechanized technology used in whaling, a ban on the whaling of endangered species cannot be enforced without banning the whaling of all other species. Hence, since hunting endangered whale species should be banned, all whale-hunting should be banned.", "question": "Which one of the following principles, if established, would do the most to justify the conclusion drawn by the antiwhaling activist?", "answers": "['If a certain activity ought to be prohibited, so should any other activity that interferes with the enforcement of that prohibition.', 'If a commercial form of a certain activity ought to be prohibited, then so should all noncommercial forms of that activity.', 'The fishing industry has a right to hunt members of species that are not endangered.', 'All actions that are instrumental in achieving ends that are permissible are themselves permissible.']", "label": 0 }, { "id": "train_542", "context": "According to promotional material published by the city of Springfield, more tourists stay in hotels in Springfield than stay in the neighboring city of Harristown. A brochure from the largest hotel in Harristown claims that more tourists stay in that hotel than stay in the Royal Arms Hotel in Springfield. If both of these sources are accurate, however, the \"Report on Tourism\" for the region must be in error in stating that __.", "question": "which of the following most logically completes the argument below?", "answers": "['The royal arms hotel is the only hotel in Springfield.', 'The Royal Arms hotel is the largest hotel in Springfield', 'There is only one hotel in Harristown that is larger than the Royal Arms Hotel.', 'More tourists stay in hotels in Harristown than stay in the Royal Arms Hotel.']", "label": 0 }, { "id": "train_543", "context": "Data froma recent biodiversity study of the Ukuluku river estuary in Tritonia showed that last year the pollution rate of this environmentally sensitive region was significantly lower than in previous years. The Tritonian government, which spent millions of dollars last year to enforce laws against dumping waste into the river, is claiming that the study data indicate that its increased efforts to halt the pollution are proving effective.", "question": "Which one of the following, if true, most seriously undermines the government's claim?", "answers": "['Government agents had to issue fines totaling over $27 million to 650 violators of the river dumping regulations.', 'Much of the money that was designated last year for estuary preservation has been spent on research and not enforcement.', \"Factory owner opposition to the government's anti-pollution efforts grew more violent last year in response to the increased enforcement.\", 'The largest known polluter on the Ukuluku river relocated to another part of Tritonia, closer to the airport, so as to decrease its shipping costs.']", "label": 3 }, { "id": "train_544", "context": "Meerkat \"sentinels, \" so-called because they watch for predators while other meerkat group members forage, almost never fall victim to those predators, yet the foragers often do. This advantage accruing to the sentinel does not mean that its watchful behavior is entirely self-interested. On the contrary, the sentinel' s behavior is an example of animal behavior motivated at least in part by altruism. The loud bark emitted by the sentinel as it dashes for the cover of the nearest hole alerts other group members to the presence of danger.", "question": "Which one of the following is a questionable reasoning technique employed in the argument?", "answers": "['concluding that a claim is false on the grounds that insufficient evidence has been offered to support it', \"appealing to evidence that tends to undermine rather than support the argument's conclusion\", \"appealing to evidence that presupposes the truth of the argument's conclusion\", 'inferring solely from an effect produced by an action that a purpose of the action is to produce that effect']", "label": 3 }, { "id": "train_545", "context": "Does watching someone else eat fill one' s own stomach? In which one of the following does the reasoning most closely parallel that employed in the passage?", "question": "Some people believe that witnessing violence in movies will discharge aggressive energy.", "answers": "[\"Some people think nationalism is defensible. Hasn't nationalism been the excuse for committing abominable crimes?\", \"Some people think workaholics are compensating for a lack of interpersonal skills. However, aren't most doctors workaholics?\", 'Some people think that economists can control inflation. Can meteorologists make the sun shine?', \"Some people think appropriating supplies at work for their own personal use is morally wrong. Isn't shoplifting morally wrong?\"]", "label": 2 }, { "id": "train_546", "context": "In constructing a self-driving robotic car, engineers face the challenge of designing a car that avoids common traffic problems like crashes and congestion. These problems can also affect fish traveling together in schools. However, the principles fish use to navigate in schools ensure that these problems are much less common within schools of fish than among cars on the road. Hence, __.", "question": "Which one of the following most logically completes the argument?", "answers": "['the best drivers use the same navigational principles that fish use in schools', 'a self-driving robotic car using the navigational principles that fish use in schools would be better than a human-driven car at avoiding crashes and congestion', 'studying the principles fish use to navigate in schools could help engineers to design a self-driving robotic car that avoids common traffic problems', 'constructing a self-driving robotic car requires expertise in fish biology']", "label": 2 }, { "id": "train_547", "context": "Branson: Most of the air pollution in this country comes from our largest cities. These cities would pollute less if they were less populated. So if many people in these cities were to move to rural areas, air pollution in the country as a whole would be reduced.", "question": "Which one of the following demonstrates most effectively by parallel reasoning that Branson's argument is flawed?", "answers": "[\"Similarly, we could conclude that Karen's family would have more living space if they moved from an apartment to a single-family home. After all, single-family homes are typically larger than apartments.\", \"Similarly, we could conclude that most of Ward's farm is planted with corn. After all, in Ward's county most of the fields that used to be planted with other crops are now planted with corn.\", \"Similarly, we could conclude that most of this city's air pollution would be eliminated if this city built a public transportation system. After all, public transportation produces much less pollution per passenger, and all automobile trips could be replaced by trips on public transportation.\", 'Similarly, we could conclude that Javier could consume fewer calories by eating for breakfast, lunch, and dinner only a portion of what he now eats, and eating the remainder as snacks. After all, breakfast, lunch, and dinner together account for most of the calories Javier consumes.']", "label": 3 }, { "id": "train_548", "context": "Acme' s bank loan must be immediately repaid in full if Acme' s earnings fall below $1 million per year. If Acme has to repay the entire loan immediately, it will have to declare bankruptcy. Acme had seemed safe from bankruptcy, having reported annual earnings of well over $1 million in each year it has had the bank loan. However, Acme has now admitted overstating its earnings for last year, so it will have to declare bankruptcy.", "question": "The argument requires the assumption that", "answers": "['Acme would be able to avoid bankruptcy if it did not have to repay the bank loan', 'Acme has other debts besides the bank loan', 'last year is not the only year for which Acme overstated earnings', \"Acme's earnings for last year, when accurately stated, are below $1 million\"]", "label": 3 }, { "id": "train_549", "context": "Film Director: It is true that certain characters and plot twists in my newly released film The Big Heist are similar to characters and plot twists in Thieves, a movie that came out last year. Pointing to these similarities, the film studio that produced Thieves is now accusing me of taking ideas from that film. The accusation is clearly without merit. All production work on The Big Heist was actually completed months before Thieves was released.", "question": "Which of the following, if true, provides the strongest support for the directors position?", "answers": "['Production work on Thieves began before production work on The B/g Heist was started', 'The director of Thieves worked with the director of The Big Heist on several earlier projects', 'Before Thieves began production, its script had been circulating for several years among various film studios, including the studio that produced The Big Heist.', 'The characters and plot twists that are most similar in the two films have close parallels in many earlier films of the same genre.']", "label": 3 }, { "id": "train_550", "context": "At Tromen University this semester, some students taking French Literature 205 are also taking Biology 218. Every student taking Biology 218 at Tromen is a biology major. Ttherefore, some of the students taking French Literature 205 are not French-literature majors.", "question": "The conclusion drawn above follows logically if which one of the following is assumed to be true at Tromen University?", "answers": "['It is not possible to major in both biology and French literature.', 'There are more biology majors than there are French-literature majors.', 'French Literature 205 is a required course for French-literature majors.', 'Only biology majors are allowed to take Biology 218.']", "label": 0 }, { "id": "train_551", "context": "Measurements of the extent of amino-acid decomposition in fragments of eggshell found at archaeological sites in such places as southern Africa can be used to obtain accurate dates for sites up to 200, 000 years old. Because the decomposition is slower in cool climates, the technique can be used to obtain accurate dates for sites almost a million years old in cooler regions.", "question": "The information above provides the most support for which one of the following conclusions?", "answers": "['The amino-acid decomposition that enables eggshells to be used in dating does not take place in other organic matter found at ancient archaeological sites.', 'The oldest archaeological sites are not in southern Africa, but rather in cooler regions of the world.', 'If the site being dated has been subject to large unsuspected climatic fluctuations during the time the eggshell has been at the site, application of the technique is less likely to yield accurate results.', 'After 200, 000 years in a cool climate, less than one-fifth of the amino acids in a fragment of eggshell that would provide material for dating with the technique will have decomposed and will thus no longer be suitable for examination by the technique.']", "label": 2 }, { "id": "train_552", "context": "A cup of raw milk, after being heated in a microwave oven to 50 degrees Celsius, contains half its initial concentration of a particular enzyme, lysozyme. If, however, the milk reaches that temperature through exposure to a conventional heat source of 50 degrees Celsius, it will contain nearly all of its initial concentration of the enzyme. Ttherefore, what destroys the enzyme is not heat but microwaves, which generate heat.", "question": "Which one of the following, if true, most seriously weakens the argument?", "answers": "['Heating raw milk in a microwave oven to a temperature of 100 degrees Celsius destroys nearly all of the lysozyme initially present in that milk.', 'A liquid exposed to a conventional heat source of exactly 50 degrees Celsius will reach that temperature more slowly than it would if it were exposed to a conventional heat source hotter than 50 degrees Celsius.', 'Heating any liquid by microwave creates small zones within it that are much hotter than the overall temperature that the liquid will ultimately reach.', 'Enzymes in raw milk that are destroyed through excessive heating can be replaced by adding enzymes that have been extracted from other sources.']", "label": 2 }, { "id": "train_553", "context": "Arbitrator: The shipping manager admits that he decided to close the old facility on October 14 and to schedule the new facility' s opening for October 17, the following Monday. But he also claims that he is not responsible for the business that was lost due to the new facility' s failing to open as scheduled. He blames the contractor for not finishing on time, but he too, is to blame, for he was aware of the contractor' s typical delays and should have planned for this contingency.", "question": "Which one of the following principles underlies the arbitrator's argument?", "answers": "['A manager should take foreseeable problems into account when making decisions.', 'A manager should be held responsible for mistakes made by those whom the manager directly supervises.', \"A manager, and only a manager, should be held responsible for a project's failure.\", 'A manager should be able to depend on contractors to do their jobs promptly.']", "label": 0 }, { "id": "train_554", "context": "Beginning in the 1950s, popular music was revolutionized by the electrification of musical instruments, which has enabled musicians to play with increased volume. Because individual musicians can play with increased volume, the average number of musicians per band has decreased. Nevertheless, electrification has increased rather than decreased the overall number of musicians who play popular music professionally.", "question": "Which one of the following is most strongly supported by the statements above, if those statements are true?", "answers": "['The number of amateur musicians who play popular music has decreased.', 'Many professional musicians play in more than one band.', 'The number of professional musicians in some bands has increased.', 'The total number of professional bands has increased as a result of electrification.']", "label": 3 }, { "id": "train_555", "context": "Economist: Although average hourly wages vary considerably between different regions of this country, in each region, the average hourly wage for full-time jobs increased last year. Paradoxically, however, in the country as a whole, the average hourly wage for full-time jobs decreased last year.", "question": "Which one of the following, if true of the economist's country, most helps to resolve the apparent paradox in the economist's statements?", "answers": "['In the country as a whole, the average hourly wage for full-time jobs has decreased slightly for each of the last three years.', 'The year before last, the unemployment rate reached a ten-year low; last year, however, the unemployment rate increased slightly.', 'Last year, to reduce costs, employers moved many full-time jobs from regions with relatively high hourly wages to regions where those jobs typically pay much less.', 'Last year, hourly wages for most full-time jobs in the manufacturing sector declined while those for most full-time jobs in the service sector increased.']", "label": 2 }, { "id": "train_556", "context": "Activist: Accidents at the Three Mile Island and Chernobyl nuclear plants have shown the dangers of nuclear power. It was earlier argued that nuclear power was necessary because fossil fuels will eventually run out. Recently, however, a technology has been developed for deriving from sewage sludge an oil that can be used to generate power. This new technology, ttherefore, together with the possibility of using alternative sources of energy like solar, wind, and hydroelectric power, raises the hope that we can dispense altogether with nuclear power and that we can meet our energy needs in a way that better protects the environment from harm than we do at present.", "question": "Which one of the following considerations is LEAST relevant in evaluating the degree of practicability of the hope expressed by the activist above?", "answers": "['whether the current methods of disposing of sewage sludge by dumping do environmental damage', 'whether the burning of oil from sewage sludge would, in contrast to nuclear power production, produce gases that would have a harmful warming effect on climate worldwide', 'whether the processes that are used to turn sewage into clean water and sewage sludge have been improved in recent decades', 'whether the cost of producing and using oil from sewage sludge would be economically sustainable']", "label": 2 }, { "id": "train_557", "context": "Coherent solutions for the problem of reducing health-care costs cannot be found within the current piecemeal system of paying these costs. The reason is that this system gives health-care providers and insurers every incentive to shift, wherever possible, the costs of treating illness onto each other or any other party, including the patient. That clearly is the lesson of the various reforms of the 1980s: push in on one part of this pliable spending balloon and an equally expensive bulge pops up elsewhere. For example, when the government health-care insurance program for the poor cut costs by disallowing payments for some visits to physicians, patients with advanced illness later presented themselves at hospital emergency rooms in increased numbers.", "question": "The argument proceeds by", "answers": "['employing an analogy to characterize interrelationships', 'demonstrating that cooperation is feasible by citing an instance', 'attributing without justification fraudulent intent to people', 'showing that shifting costs onto the patient contradicts the premise of health-care reimbursement']", "label": 0 }, { "id": "train_558", "context": "Fine short story writers are unlikely to become great novelists. Short story writers must master the ability to interweave the many small details that together allow mundane incidents to illuminate important truths. Because the novel drowns in such detail, novelists must focus on larger matters. Only a few writers possess both the ability to weave together many small details and the ability to focus on larger matters.", "question": "The reasoning in which one of the following is most similar to the reasoning above?", "answers": "['Because of the vast amount of medical knowledge one needs in order to become a successful specialist and because few people have the motivation required to obtain such knowledge, most people are unlikely to become successful specialists.', 'Those who excel at one sport are unlikely to excel at another, because it is rare for someone who has the specialized talents necessary for success in one sport to also have the different specialized talents necessary for success in a different sport.', 'Engineers can never design an automobile that both meets high standards for comfort and safety and uses fuel efficiently, because high levels of comfort and safety generally require that a car be heavy, while efficient use of fuel usually requires that a car be light.', 'Good painters cannot become good scholars of painting. Painters are inevitably biased toward their own style of painting and, accordingly, cannot be objective scholars.']", "label": 1 }, { "id": "train_559", "context": "If the flowers Drew received today had been sent by someone who knows Drew well, that person would have known that Drew prefers violets to roses. Yet Drew received roses. On the other hand, if the flowers had been sent by someone who does not know Drew well, then that person would have sent a signed card with the flowers. Yet Drew received no card. Ttherefore, the florist must have made some sort of mistake: either Drew was supposed to receive violets, or a card, or these flowers were intended for someone else.", "question": "Which one of the following statements, if true, most weakens the argument?", "answers": "['Some people who know Drew well have sent Drew cards along with flowers.', 'Some people send flowers for a reason other than the desire to please.', 'Most people send roses when they send flowers.', 'Someone who does not know Drew well would be unlikely to send Drew flowers.']", "label": 1 }, { "id": "train_560", "context": "Even those who believe that the art of each age and culture has its own standards of beauty must admit that some painters are simply superior to others in the execution of their artistic visions. But this superiority must be measured in light of the artist' s purposes, since the high merits, for example, of Jose Rey Toledo' s work and his extraordinary artistic skills are not in doubt, despite the fact that his paintings do not literally resemble what they represent.", "question": "The claim that some painters are superior to others in the execution of their artistic visions plays which one of the following roles in the argument?", "answers": "['It is a claim that the argument derives from another claim and that it uses to support its conclusion.', 'It is a claim that, according to the argument, is to be understood in a manner specified by the conclusion.', 'It is a generalization, one sort of objection to which the argument illustrates by giving an example.', 'It is a generalization that the argument uses to justify the relevance of the specific example it cites.']", "label": 1 }, { "id": "train_561", "context": "A promise is defined as any agreement that yields an obligation. It is acceptable not to keep one' s promise if either the person to whom the promise is made tells the promisor not to keep it, or the promisor is unable to fulfill the obligation due to circumstances beyond the promisor' s control.", "question": "Which one of the following is an application of the principle above pertaining to promising?", "answers": "['Paul agreed to lend Tom $10. But Paul also owes Maria $10. So it is unacceptable for Paul to fulfill his promise to lend Tom $10 while he still owes Maria money.', 'Felicia agreed to tutor Alan in mathematics on Tuesday. However, on his way to the tutorial session, Alan was injured in a car accident and was unable to attend that day. Thus, it was acceptable for Felicia not to tutor Alan in mathematics on Tuesday.', \"Quinn agreed to water Laura's plants while Laura was out of town. But Laura had forgotten to give Quinn the keys to her place. However, it was unacceptable for Quinn to fail to fulfill his promise to Laura, since Laura did not excuse Quinn from his obligation to water her plants.\", 'Ian agreed to pick up Ali at the train station. But Ian decided to stop and return his overdue library books first. As a result, Ian was unable to pick up Ali at the train station. Thus, it was acceptable for Ian to fail to pick up Ali at the train station.']", "label": 1 }, { "id": "train_562", "context": "Early in this century Alfred Wegener developed the concept of continental drift. His ideas were rejected vehemently because he postulated no identifiable force strong enough to make the continents move. We have come to accept Wegener' s theory, not because we have pinpointed such a force, but because new instruments have finally allowed continental movement to be confirmed by observation.", "question": "The passage best illustrates which one of the following statements about science?", "answers": "['Science, employing statistics and the laws of probability, is concerned not with the single event but with mass behavior.', 'In accepting a mathematical description of nature, science has become far more accurate at identifying underlying forces.', 'The aim of science is to define the manifold of nature within the terms of a single harmonious theory.', 'When the events a theory postulates are detected, the theory is accepted even without an explanation of how those events are brought about.']", "label": 3 }, { "id": "train_563", "context": "Dwight works at a mid-sized regional tech company. He approaches all tasks with unmatched enthusiasm and leads the company in annual sales. The top salesman is always the best employee. Ttherefore, Dwight is the best employee.", "question": "Which of the following most accurately describes how the argument proceeds?", "answers": "['The argument proceeds by stating a general fact, offering specific anecdotes, and then drawing a conclusion.', 'The argument proceeds by stating several facts that serve as the basis for the conclusion at the end of the argument.', 'The argument proceeds by first stating a conclusion and then offering several premises to justify that conclusion.', 'The argument proceeds by stating several facts, offering a universal rule, and then drawing a conclusion by applying the facts to the rule.']", "label": 3 }, { "id": "train_564", "context": "Unplugging a peripheral component such as a \"mouse\" from a personal computer renders all of the software programs that require that component unusable on that computer. On Fred' s personal computer, a software program that requires a mouse has become unusable. So it must be that the mouse for Fred' s computer became unplugged.", "question": "The argument is most vulnerable to which one of the following criticisms?", "answers": "['It attempts to support its conclusion by citing a generalization that is too broad.', 'It overlooks the possibility that some programs do not require a peripheral component such as a mouse.', 'It introduces information unrelated to its conclusion as evidence in support of that conclusion.', 'It treats an event that can cause a certain result as though that event is necessary to bring about that result.']", "label": 3 }, { "id": "train_565", "context": "In the years following an eight-cent increase in the federal tax on a pack of cigarettes, sales of cigarettes fell ten percent. In contrast, in the year prior to the tax increase, sales had fallen one percent. The volume of cigarette sales is ttherefore strongly related to the after-tax price of a pack of cigarettes.", "question": "The argument above requires which of the following assumptions?", "answers": "['As the after-tax price of a pack of cigarettes rises, the pretax price also rises.', 'The pretax price of a pack of cigarettes gradually decreased throughout the year before and the year after the tax increase.', 'During the year following the tax increase, the pretax price of a pack of cigarettes did not increase by as much as it had during the year prior to the tax increase.', 'For the year following the tax increase, the pretax price of a pack of cigarettes was not eight or more cents lower than it had been the previous year.']", "label": 3 }, { "id": "train_566", "context": "Alice will volunteer to work on the hospital fundraising drive only if her brother Bruce also volunteers and a majority of the others working on the drive promise to select Bruce to manage the drive. However, although Bruce is willing to volunteer, none of the others working on the drive will promise to select Bruce to manage the drive. Thus it is certain that Alice will not volunteer.", "question": "In which one of the following arguments is the pattern of reasoning most similar to the pattern of reasoning in the argument above?", "answers": "['Leon will go to the picnic only if Vera also goes. However, according to his friends Leon does not plan to go to the picnic: so it is certain that Vera is not planning to go.', 'Therese will work in the yard tomorrow only if Maria helps her and it is not raining. Although Maria does not like working in the yard, she will help Therese. So unless it rains tomorrow, Therese will certainly work in the yard.', 'Jim will go to the party only if both Sam and Elaine also go. Sam is going to the party, but Elaine is not going. So it is certain that Jim will not go to the party.', 'If Paula works with Elise, then Jane will work with Arthur only. However, if Paula does not work, Elise will also work with Jane. So if Paula does not work, Arthur will work with both Jane and Elise.']", "label": 2 }, { "id": "train_567", "context": "Legislator: We should not waste any more of the taxpayers' money on the government' s job-creation program. The unemployment rate in this country has actually risen since the program was begun, so the program has clearly been a failure.", "question": "Which of the following is an assumption on which the legislator's argument depends?", "answers": "['Other government programs are no more effective in reducing unemployment than is the job-creation program.', 'The unemployment rate is higher now than at any time before the inception of the job-creation program.', 'If the job-creation program had been run more efficiently, it could have better served its purpose.', 'The unemployment rate would not have risen even more than it has if the job- creation program had not been in existence.']", "label": 3 }, { "id": "train_568", "context": "When individual students are all treated equally in that they have identical exposure to curriculum material, the rate, quality, and quantity of learning will vary from student to student. If all students are to master a given curriculum, some of them need different types of help than others, as any experienced teacher knows.", "question": "If the statements above are both true, which one of the following conclusions can be drawn on the basis of them?", "answers": "['The rate and quality of learning, with learning understood as the acquiring of the ability to solve problems within a given curriculum area, depend on the quantity of teaching an individual student receives in any given curriculum.', 'All students should have identical exposure to learn the material being taught in any given curriculum.', 'Unequal treatment, in a sense, of individual students is required in order to ensure equality with respect to the educational tasks they master.', 'The more experienced the teacher is, the more the students will learn.']", "label": 2 }, { "id": "train_569", "context": "The French novelist Colette (1873-1954) has been widely praised for the vividness of her language. But many critics complain that her novels are indifferent to important moral questions. This charge is unfair. Each of her novels is a poetic condensation of a major emotional crisis in the life of an ordinary person of her time. Such emotional crises almost invariably raise important moral questions.", "question": "Which one of the following is an assumption on which the argument depends?", "answers": "['A novel that poetically condenses a major emotional crisis does not have to be indifferent to the important moral questions raised by that crisis.', \"To deserve the level of praise that Colette has received, a novelist's work must concern itself with important moral questions.\", \"The vividness of Colette's language was not itself the result of poetic condensation.\", \"Critics who suggest that Colette's novels are indifferent to great moral questions of her time greatly underestimate her literary achievements.\"]", "label": 0 }, { "id": "train_570", "context": "Humans get Lyme disease from infected ticks. Ticks get infected by feeding on animals with Lyme disease, but the ease of transmission from host animal to tick varies. With most species of host animal, transmission of Lyme disease to ticks is extremely rare, but white-footed mice are an exception, readily passing Lyme disease to ticks. And white-footed mouse populations greatly expand, becoming the main food source for ticks, in areas where biodiversity is in decline.", "question": "The information in the passage most strongly supports which of the following?", "answers": "['Humans are less at risk of contracting Lyme disease in areas where biodiversity is high.', 'The greater the biodiversity of an area, the more likely any given host animal in that area is to pass Lyme disease to ticks.', 'Ticks feed on white-footed mice only when other host species are not available to them.', 'Very few animals that live in areas where there are no white-footed mice are infected with Lyme disease.']", "label": 0 }, { "id": "train_571", "context": "Some video game makers have sold the movie rights for popular games. However, this move is rarely good from a business perspective. After all, StarQuanta sold the movie rights to its popular game Nostroma, but the poorly made film adaptation of the game was hated by critics and the public alike. Subsequent versions of the Nostroma video game, although better than the original, sold poorly.", "question": "The reasoning in the argument is most vulnerable to criticism in that the argument", "answers": "['infers that a product will be disliked by the public merely from the claim that the product was disliked by critics', 'takes for granted that products with similar content that are in different media will be of roughly equal popularity', 'restates as a conclusion a claim earlier presented as evidence for that conclusion', 'draws a general conclusion on the basis of just one individual case']", "label": 3 }, { "id": "train_572", "context": "Industry Analyst: This summer is expected to be particularly hot and dry in the major orange-growing regions of Florida, the state accounting for the largest share of orange production in the United States. While hot and dry weather usually leads to a substantial drop in the yield of oranges, my projections indicate that the revenues of Florida orange producers will not be substantially affected by these unfavorable conditions.", "question": "Which of the following statements, if true, would support the claim of the Industry Analyst?", "answers": "['This summer, Florida orange producers will employ a large number of migrant workers, a strategy that will substantially decrease their labor costs.', 'The hot weather is likely to increase the revenues of companies engaged in tourism and recreation that comprise the largest portion of the state economy.', 'Because of the hot and dry weather, the costs of irrigation for Florida orange growers will be nearly 30% higher this summer.', 'The decline in the supply of oranges caused by the unfavorable weather will result in a substantial increase in their market price.']", "label": 3 }, { "id": "train_573", "context": "Comets do not give off their own light but reflect light from other sources, such as the Sun. Scientists estimate the mass of comets by their brightness: the greater a comet' s mass, the more light that comet will reflect. A satellite probe, however, has revealed that the material of which Halley' s comet is composed reflects 60 times less light per unit of mass than had been previously thought.", "question": "The statements above, if true, give the most support to which one of the following?", "answers": "['The reflective properties of the material of which comets are composed vary considerably from comet to comet.', \"Previous estimates of the mass of Halley's comet which were based on its brightness were too low.\", \"The total amount of light reflected from Halley's comet is less than scientists had previously thought.\", \"Scientists need more information before they can make a good estimate of the mass of Halley's comet.\"]", "label": 1 }, { "id": "train_574", "context": "Criminologist: Some legislators advocate mandating a sentence of life in prison for anyone who, having twice served sentences for serious crimes, is subsequently convicted of a third serious crime. These legislators argue that such a policy would reduce crime dramatically , since it would take people with a proven tendency to commit crimes off the streets permanently. What this reasoning overlooks, however, is that people old enough to have served two prison sentences for serious crimes rarely commit more than one subsequent crime. Filling our prisons with such individuals would have exactly the opposite of the desired effect, since it would limit our ability to incarcerate younger criminals, who commit a far greater proportion of serious crimes.", "question": "In the argument as a whole, the two boldfaced portions play which of the following roles?", "answers": "['The first is a conclusion that the argument as a whole seeks to refute; the second is a claim that has been advanced in support of that conclusion.', 'The first is the main conclusion of the argument; the second is an objection that has been raised against that conclusion.', 'The first is the main conclusion of the argument; the second is a prediction made on the basis of that conclusion.', 'The first is a conclusion that the argument as a whole seeks to refute; the second is the main conclusion of the argument.']", "label": 3 }, { "id": "train_575", "context": "Researchers working in Western Australia have discovered the oldest fragments of the Earth' s early crust that have yet been identified: microdiamonds. These microscopic crystals measure only 50 microns across and were formed 4. 2 billion years ago. This discovery sheds light on how long it took for the Earth' s crust to form, since this date is only 300 million years after the formation of the Earth itself.", "question": "If the statements above are true, which one of the following must also be true?", "answers": [ "The Earth's crust first formed in the area that is now Western Australia.", "The Earth's crust took no longer than 300 million years to start to form.", "All naturally occurring microdiamonds were formed at the time the Earth's crust was being formed.", "The Earth's crust took billions of years to form." ], "label": 1 }, { "id": "train_576", "context": "Although water in deep aquifers does not contain disease-causing bacteria, when public water supplies are drawn from deep aquifers, chlorine is often added to the water as a disinfectant because contamination can occur as a result of flaws in pipes or storage tanks. Of 50 municipalities that all pumped water from the same deep aquifer, 30 chlorinated their water and 20 did not. The water in all of the municipalities met the regional government' s standards for cleanliness, yet the water supplied by the 20 municipalities that did not chlorinate had less bacterial contamination than the water supplied by the municipalities that added chlorine.", "question": "Which one of the following can properly be concluded from the information given above?", "answers": "[\"The regional government's standards allow some bacteria in municipal water supplies.\", 'Chlorine is the least effective disinfecting agent.', 'Where accessible, deep aquifers are the best choice as a source for a municipal water supply.', \"A municipality's initial decision whether or not to use chlorine is based on the amount of bacterial contamination in the water source.\"]", "label": 0 }, { "id": "train_577", "context": "Last year a record number of new manufacturing jobs were created. Will this year bring another record? Well, a new manufacturing job is created either within an existing company or by the start-up of a new company. Within existing firms, new jobs have been created this year at well below last year' s record pace. At the same time, there is considerable evidence that the number of new companies starting up will be no higher this year than it was last year, and surely the new companies starting up this year will create no more jobs per company than did last year' s start-ups . Clearly, it can be concluded that the number of new jobs created this year will fall short of last year' s record .", "question": "In the argument given, the two portions in boldface play which of the following roles?", "answers": "['The first is a prediction that, if accurate, would provide support for the main conclusion of the argument; the second is that main conclusion.', 'The first is a prediction that, if accurate, would provide support for the main conclusion of the argument; the second is a conclusion drawn in order to support that main conclusion.', 'The first is an objection that the argument rejects; the second presents a conclusion that could be drawn if that objection were allowed to stand.', 'The first is an objection that the argument rejects; the second is the main conclusion of the argument.']", "label": 0 }, { "id": "train_578", "context": "Paper& Print is a chain of British stores selling magazines, books, and stationery products. In Britain, magazines' retail prices are set by publishers, and the retailer' s share of a magazine' s retail price is 25 percent. Since Paper& Print' s margin on books and stationery products is much higher, the chain' s management plans to devote more of its stores' shelf space to books and stationery products and reduce the number of magazine titles that its stores carry.", "question": "Which of the following, if true, most strongly argues that the plan, if put into effect, will not increase Paper& Print's profits?", "answers": "[\"The publisher's share of a magazine's retail price is 50 percent, and the publisher also retains all of the magazine's advertising revenue.\", 'In market research surveys, few consumers identify Paper&Print as a book or stationery store but many recognize and value the broad range of magazines it carries.', 'Recently magazine publishers, seeking to increase share in competitive sectors of the market, have been competitively cutting the retail prices of some of the largest circulation magazines.', 'Consumers who subscribe to a magazine generally pay less per issue than they would if they bought the magazine through a retail outlet such as Paper& Print.']", "label": 1 }, { "id": "train_579", "context": "Novel X and Novel Y are both semiautobiographical novels and contain many very similar themes and situations, which might lead one to suspect plagiarism on the part of one of the authors. However, it is more likely that the similarity of themes and situations in the two novels is merely coincidental, since both authors are from very similar backgrounds and have led similar lives.", "question": "Which one of the following most accurately expresses the conclusion drawn in the argument?", "answers": "['Novel X and Novel Y are both semiautobiographical novels, and the two novels contain many very similar themes and situations.', 'It is less likely that one of the authors of Novel X or Novel Y is guilty of plagiarism than that the similarity of themes and situations in the two novels is merely coincidental.', 'The fact that Novel X and Novel Y are both semiautobiographical novels and contain many very similar themes and situations might lead one to suspect plagiarism on the part of one of the authors.', 'If the authors of Novel X and Novel Y are from very similar backgrounds and have led similar lives, suspicions that either of the authors plagiarized are very likely to be unwarranted.']", "label": 1 }, { "id": "train_580", "context": "If there are any inspired performances in the concert, the audience will be treated to a good show. But there will not be a good show unless there are sophisticated listeners in the audience, and to be a sophisticated listener one must understand one' s musical roots.", "question": "If all of the statements above are true, which one of the following must also be true?", "answers": "['If there are no sophisticated listeners in the audience, then there will be no inspired musical performances in the concert.', 'If there will be people in the audience who understand their musical roots, then at least one musical performance in the concert will be inspired.', 'If there are sophisticated listeners in the audience, then there will be inspired musical performances in the concert.', 'No people who understand their musical roots will be in the audience if the audience will not be treated to a good show.']", "label": 0 }, { "id": "train_581", "context": "The computer industry's estimate that it loses millions of dollars when users illegally copy programs without paying for them is greatly exaggerated. Most of the illegal copying is done by people with no serious interest in the programs. Thus, the loss to the industry is quite small, because__.", "question": "Which of the following best completes the passage below?", "answers": "['the number of programs that are frequently copied illegally is low in comparison to the number of programs available for sale', 'the total market value of all illegal copies is low in comparison to the total revenue of the computer industry', 'most people who illegally copy programs would not purchase them even if purchasing them were the only way to obtain them', 'many users who illegally copy programs never find any use for them']", "label": 2 }, { "id": "train_582", "context": "The format of network television news programs generally allows advocates of a point of view only 30 seconds to convey their message. Consequently, regular watchers become accustomed to thinking of issues in terms only of slogans and catch phrases, and so the expectation of careful discussion of public issues gradually disappears from their awareness. The format of newspaper stories, on the other hand, leads readers to pursue details of stories headed by the most important facts and so has the opposite effect on regular readers -- that of maintaining the expectation of careful discussion of public issues. Ttherefore, in contrast to regular newspaper reading, regular watching of network television news programs increases the tendency to think of public issues in oversimplified terms.", "question": "The argument assumes which one of the following?", "answers": "['In network television news reports, it is not usual for a reporter to offer additional factual evidence and background information to develop a story in which opposing views are presented briefly by their advocates.', 'It is not possible for television to present public issues in a way that allows for the nuanced presentation of diverse views and a good-faith interchange between advocates of opposing views.', 'Television news reporters introduce more of their own biases into news stories than do newspaper reporters.', 'Since it is not possible to present striking images that would symbolize events for viewers, and since images hold sway over words in television, television must oversimplify.']", "label": 0 }, { "id": "train_583", "context": "An overwhelming proportion of the most productive employees at SaleCo' s regional offices work not eight hours a day, five days a week, as do other SaleCo employees, but rather ten hours a day, four days a week, with Friday off. Noting this phenomenon, SaleCo' s president plans to increase overall productivity by keeping the offices closed on Fridays and having all employees work the same schedule-ten hours a day, four days a week.", "question": "Which of the following, if true, provides the most reason to doubt that the president's plan, if implemented, will achieve its stated purpose?", "answers": "[\"Typically, a SaleCo employee's least productive hours in the workplace are the early afternoon hours.\", 'Those SaleCo employees who have a four-day workweek do not take any of their office work to do at home on Fridays.', 'Employees at SaleCo are compensated not on the basis of how many hours a week they work but on the basis of how productive they are during the hours they are at work.', 'Working ten hours a day has allowed the most productive employees to work two hours alone each day in their respective offices relatively undisturbed by fellow employees.']", "label": 3 }, { "id": "train_584", "context": "Wanda: It is common sense that one cannot create visual art without visual stimuli in one' s work area, just as a writer needs written stimuli. A stark, empty work area would hinder my creativity. This is why there are so many things in my studio. Vernon: But a writer needs to read good writing, not supermarket tabloids.", "question": "Are you inspired by the piles of laundry and empty soda bottles in your studio? Which one of the following most accurately expresses the principle underlying Vernon's response to Wanda?", "answers": "['Supermarket tabloids should not be considered stimulating.', 'One should be able to be creative even in a stark, empty work area.', 'It is unhealthy to work in a cluttered work area.', \"The quality of the stimuli in an artist's environment matters.\"]", "label": 3 }, { "id": "train_585", "context": "Philosopher: Some of the most ardent philosophical opponents of democracy have rightly noted that both the inherently best and the inherently worst possible forms of government are those that concentrate political power in the hands of a few. Thus, since democracy is a consistently mediocre form of government, it is a better choice than rule by the few.", "question": "Which one of the following principles, if valid, most helps to justify the philosopher's argument?", "answers": "['The best form of government is the one that is most likely to produce an outcome that is on the whole good.', 'A society should adopt a democratic form of government if and only if most members of the society prefer a democratic form of government.', 'In choosing a form of government, it is better for a society to avoid the inherently worst than to seek to attain the best.', 'Democratic governments are not truly equitable unless they are designed to prevent interest groups from exerting undue influence on the political process.']", "label": 2 }, { "id": "train_586", "context": "Peter: Because the leaves of mildly drought-stressed plants are tougher in texture than the leaves of abundantly watered plants, insects prefer to feed on the leaves of abundantly watered plants. Ttherefore, to minimize crop damage, farmers should water crops only just enough to ensure that there is no substantial threat, from a lack of water, to either the growth or the yield of the crops. Jennifer: Indeed. In fact, a mildly drought-stressed plant will divert a small amount of its resources from normal growth to the development of pesticidal toxins, but abundantly watered plants will not.", "question": "Jennifer's comment is related to Peter's argument in which one of the following ways?", "answers": "[\"It supports Peter's argument by offering an explanation of all of Peter's premises.\", 'It offers information that supports each of the claims that Peter makes in his argument.', \"It supports Peter's argument by supplying a premise without which Peter's conclusion cannot properly be drawn.\", \"It supports the conclusion of Peter's argument by offering independent grounds for that conclusion.\"]", "label": 3 }, { "id": "train_587", "context": "Traverton's city council wants to minimize the city's average yearly expenditures on its traffic signal lights and so is considering replacing the incandescent bulbs currently in use with arrays of light-emitting diodes (LEDs) as the incandescent bulbs burn out. Compared to incandescent bulbs, LED arrays consume significantly less energy and cost no more to purchase. Moreover, the costs associated with the conversion of existing fixtures so as to accept LED arrays would be minimal.", "question": "Which of the following would it be most useful to know in determining whether switching to LED arrays would be likely to help minimize Traverton's yearly maintenance costs?", "answers": "[\"Whether the crews that currently replace incandescent bulbs in Traverton's traffic signals know how to convert the existing fixtures so as to accept LED arrays\", 'Whether the expected service life of LED arrays is at least as long as that of the currently used incandescent bulbs', 'Whether any cities have switched from incandescent lights in their traffic signals to lighting elements other than LED arrays', \"Whether Traverton's city council plans to increase the number of traffic signal lights in Traverton\"]", "label": 1 }, { "id": "train_588", "context": "Employer: In the current economic climate, the best way to run a business is to pay employees the least amount possible to do the job. The supply of labor is far outpacing demand since the number of college graduates increases every year and the average age of retirement is also increasing. Applicants will typically take the first job offer on the table, and any employee who demands a raise can be easily replaced from the labor pool. Even if the employee is unhappy, he or she will often remain on the job due to the competition in the job market. Keeping payroll costs low allows more resources to be devoted to innovation, delivering a higher quality product to customers.", "question": "Each of the following, if true, weakens the employer's argument EXCEPT:", "answers": "['Training new employees costs more than giving existing employees a raise.', \"Paying employees the minimum will hurt the company's image amongst customers.\", 'Dissatisfied employees lead to labor unrest, and the resulting protests disrupt business.', 'Automation is the leading cause for unemployment.']", "label": 3 }, { "id": "train_589", "context": "A newly discovered painting seems to be the work of one of two seventeenth-century artists, either the northern German Johannes Drechen or the Frenchman Louis Birelle, who sometimes painted in the same style as Drechen. Analysis of the carved picture frame, which has been identified as the painting's original seventeenth-century frame, showed that it is made of wood found widely in northern Germany at the time, but rare in the part of France where Birelle lived. This shows that the painting is most likely the work of Drechen.", "question": "Which of the following is an assumption that the argument requires?", "answers": "['Sometimes a painting so resembles others of its era that no expert is able to confidently decide who painted it.', 'Drechen is unlikely to have ever visited the home region of Birelle in France.', 'The carving style of the picture frame is not typical of any specific region of Europe.', 'The frame was made from wood local to the region where the picture was painted.']", "label": 3 }, { "id": "train_590", "context": "Government statistics show that the real (adjusted for inflation) average income for families has risen over the last five years. Ttherefore, since this year the Andersen family' s income is average for families, the family' s real income must have increased over the last five years.", "question": "The reasoning in the argument is most vulnerable to criticism on the grounds that the argument", "answers": "[\"overlooks the possibility that most families' incomes are below average\", \"fails to take into account inflation with respect to the Andersen family's income\", 'presumes, without providing justification, that the government makes no errors in gathering accurate estimates of family income', \"fails to consider the possibility that the Andersen family's real income was above average in the recent past\"]", "label": 3 }, { "id": "train_591", "context": "Members of the Amazonian Akabe people commonly take an early-morning drink of a tea made from the leaves of a forest plant. Although they greatly enjoy this drink, at dawn they drink it only in small amounts. Anthropologists hypothesize that since this tea is extraordinarily high in caffeine, the explanation for the Akabe' s not drinking more of it at dawn is that high caffeine intake would destroy the surefootedness that their daily tasks require.", "question": "Which one of the following, if true, most seriously calls the anthropologists' explanation into question?", "answers": "['The leaves used for the tea contain a soluble narcotic.', \"The Akabe also drink the tea in the evening, after their day's work is done.\", 'The drink is full of nutrients otherwise absent from the Akabe diet.', 'When celebrating, the Akabe drink the tea in large quantities.']", "label": 0 }, { "id": "train_592", "context": "Air traffic controllers and nuclear power plant operators are not allowed to work exceptionally long hours, because to do so would jeopardize lives. Yet physicians in residency training are typically required to work 80-hour weeks. The aforementioned restrictions on working exceptionally long hours should also be applied to resident physicians, since they too are engaged in work of a life-or-death nature.", "question": "Which one of the following is an assumption the argument depends on?", "answers": "['Resident physicians have a more direct effect on the lives of others than do air traffic controllers and nuclear power plant operators.', 'Those who are not engaged in work that has life- or-death consequences should only sometimes be allowed to work exceptionally long hours.', 'There is no indispensable aspect of residency training that requires resident physicians to work exceptionally long hours.', 'Some resident physicians would like to complete their residency training without working exceptionally long hours.']", "label": 2 }, { "id": "train_593", "context": "Statistical studies show that last year there was the greatest drop in the violent crime rate over the course of a year since such statistics were first gathered. But they also reveal that at the same time public anxiety about violent crime substantially increased.", "question": "Which one of the following, if true, most helps to resolve the apparent discrepancy described above?", "answers": "['Longer prison sentences were the primary cause of the decrease in the violent crime rate over the course of last year.', \"As in the past, last year's increase in public anxiety about violent crime has been consistently underreported in the news media.\", 'People who feel the most anxiety about violent crime usually live in areas with relatively high violent crime rates.', 'The proportion of violent crimes covered in the news media nearly doubled over the course of last year.']", "label": 3 }, { "id": "train_594", "context": "Often patients with ankle fractures that are stable, and thus do not require surgery, are given follow-up x-rays because their orthopedists are concerned about possibly having misjudged the stability of the fracture. When a number of follow-up x-rays were reviewed, however, all the fractures that had initially been judged stable were found to have healed correctly. Ttherefore, it is a waste of money to order follow-up x-rays of ankle fracture initially judged stable.", "question": "Which of the following, if true, most strengthens the argument?", "answers": "['The healing of ankle fractures that have been surgically repaired is always checked by means of a follow-up x-ray.', 'Doctors who are general practitioners rather than orthopedists are less likely than orthopedists to judge the stability of an ankle fracture correctly.', 'X-rays of patients of many different orthopedists working in several hospitals were reviewed.', 'Many ankle injuries for which an initial x-ray is ordered are revealed by the x-ray not to involve any fracture of the ankle.']", "label": 2 }, { "id": "train_595", "context": "A significant number of Qualitex Corporation' s department heads are due to retire this year. The number of employees other than current department heads who could take on the position of department head is equal to only about half of the expected vacancies. Oualitex is not going to hire department heads from outside the company or have current department heads take over more than one department, so some departments will be without department heads next year unless Qualitex __.", "question": "Which of the following most logically completes the argument?", "answers": "['reduces the average number of employees per department', 'reduces the number of its departments', 'raises the salary for department heads', 'promotes some current department heads to higher-level managerial positions']", "label": 1 }, { "id": "train_596", "context": "It is illegal to advertise prescription medications in Hedland except directly to physicians, either by mail or in medical journals. A proposed law would allow general advertising of prescription medications. Opponents object that the general population lacks the specialized knowledge to evaluate such advertisements and might ask their physicians for inappropriate medications. But since physicians have the final say as to whether to prescribe a medication for a patient, inappropriate prescriptions would not become more common.", "question": "Which of the following would it be most useful to establish in order to evaluate the argument?", "answers": "['Whether advertisements for prescription medications are currently an important source of information about newly available medications for physicians', 'Whether some people might go to a physician for no reason other than to ask for a particular medication they have seen advertised', \"Whether physicians would give in to a patient's demand for a prescription medication chosen by the patient when the one originally prescribed by the physician fails to perform as desired\", 'Whether advertising for prescription medications might alert patients to the existence of effective treatments for minor ailments that they had previously thought to be untreatable']", "label": 2 }, { "id": "train_597", "context": "The energy an animal must expend to move uphill is proportional to its body weight, whereas the animal' s energy output available to perform this task is proportional to its surface area. This is the reason that small animals, like squirrels, can run up a tree trunk almost as fast as they can move on level ground, whereas large animals tend to slow down when they are moving uphill.", "question": "Which one of the following is an assumption on which the explanation above depends?", "answers": "['The amount of energy needed to run at a given speed is proportional to the surface area of the running animal.', 'The ratio of surface area to body weight is smaller in large animals than it is in small animals.', 'Small animals can move more rapidly than large animals can.', 'There is little variation in the ratio of energy output to body weight among animals.']", "label": 1 }, { "id": "train_598", "context": "Because the native salmon in Lake Clearwater had nearly disappeared, sockeye salmon were introduced in 1940. After being introduced, this genetically uniform group of sockeyes split into two distinct populations that do not interbreed, one inhabiting deep areas of the lake and the other inhabiting shallow areas. Since the two populations now differ genetically, some researchers hypothesize that each has adapted genetically to its distinct habitat.", "question": "Which of the following, if true, most strongly supports the researchers' hypothesis?", "answers": "['Most types of salmon that inhabit lakes spend part of the time in shallow water and part in deeper water.', 'Neither of the two populations of sockeyes has interbred with the native salmon.', 'When the native salmon in Lake Clearwater were numerous, they comprised two distinct populations that did not interbreed.', 'The total number of sockeye salmon in the lake is not as large as the number of native salmon had been many years ago.']", "label": 1 }, { "id": "train_599", "context": "Consumer advocate: In some countries, certain produce is routinely irradiated with gamma rays in order to extend shelf life. There are, however, good reasons to avoid irradiated foods. First, they are exposed to the radioactive substances that produce the gamma rays. Second, irradiation can reduce the vitamin content of fresh foods, leaving behind harmful chemical residues. Third, irradiation spawns unique radiolytic products that cause serious health problems, including cancer.", "question": "Each of the following, if true, weakens the consumer advocate's argument EXCEPT:", "answers": "['Unique radiolytic products have seldom been found in any irradiated food.', 'Cancer and other serious health problems have many causes that are unrelated to radioactive substances and gamma rays.', 'A study showed that irradiation leaves the vitamin content of virtually all fruits and vegetables unchanged.', 'A study showed that the cancer rate is no higher among people who eat irradiated food than among those who do not.']", "label": 1 }, { "id": "train_600", "context": "Lewis: Those who do not learn from past mistakes -- their own and those of others -- are condemned to repeat them. In order to benefit from the lessons of history, however, we first have to know history. That is why the acquisition of broad historical knowledge is so important. Morris: The trouble is that the past is infinitely various. From its inexhaustible storehouse of events it is possible to prove anything or its contrary.", "question": "The issue that Morris raises in objecting to Lewis' view is whether", "answers": "['historical knowledge can be too narrow to be useful', 'there are conventional criteria for calling a past action a mistake', 'history teaches any unequivocal lessons', 'there are any uncontested historical facts']", "label": 2 }, { "id": "train_601", "context": "Of patients over 65 years old who survived coronary bypass surgery-a procedure widely prescribed for people with heart disease-only 75 percent benefited from the surgery. Thus it appears that for one in four such patients, the doctors who advised them to undergo this surgery, with its attendant risks and expense, were more interested in an opportunity to practice their skills and in their fee than in helping the patient.", "question": "Which of the following, if true, most seriously undermines the argument?", "answers": "['Many of the patients who receive coronary bypass surgery are less than 55 years old.', 'The patients who underwent coronary bypass surgery but who did not benefit from it were medically indistinguishable, prior to their surgery, from the patients who did benefit.', 'The patients over 65 years old who did not benefit from the coronary bypass surgery were as fully informed as those who did benefit from the surgery as to the risks of the surgery prior to undergoing it.', 'Most of the patients in the survey decided to undergo coronary bypass surgery because they were advised that the surgery would reduce their risk of future heart attacks.']", "label": 1 }, { "id": "train_602", "context": "A sudden increase in the production of elephant ivory artifacts on the Mediterranean coast of North Africa occurred in the tenth century. Historians explain this increase as the result of an area opening up as a new source of ivory and argue on this basis that the important medieval trade between North Africa and East Africa began at this period.", "question": "Each of the following, if true, provides some support for the historians' account described above EXCEPT:", "answers": "['In East Africa gold coins from Mediterranean North Africa have been found at a tenth-century site but at no earlier sites.', 'Documents from Mediterranean Europe and North Africa that date back earlier than the tenth century show knowledge of East African animals.', 'East African carvings in a style characteristic of the tenth century depict seagoing vessels very different from those used by local sailors but of a type common in the Mediterranean.', 'The many surviving letters of pre-tenth-century North African merchants include no mention of business transactions involving East Africa.']", "label": 1 }, { "id": "train_603", "context": "Eva: A \"smart highway\" system should be installed, one that would monitor areawide traffic patterns and communicate with computers in vehicles or with programmable highway signs to give drivers information about traffic congestion and alternate routes. Such a system, we can infer, would result in improved traffic flow in and around cities that would do more than improve drivers' tempers; it would decrease the considerable loss of money and productivity that now results from traffic congestion. Louis: There are already traffic reports on the radio.", "question": "Why would a \"smart highway\" system be any better? If Eva responded to Luis by saying that the current one-minute radio reports are too short to give a sufficient description of overall patterns of traffic congestion, which one of the following, if true, would most strengthen Luis's challenge?", "answers": "['It would be less costly to have radio stations that give continual, lengthier traffic reports than to install a \"smart highway\" system.', 'Bad weather, which radio stations report, would cause traffic to slow down whether or not a \"smart highway\" system was in operation.', 'The proposed traffic monitoring would not reduce the privacy of drivers.', 'Radio reports can take note of congestion once it occurs, but a \"smart highway\" system could anticipate and forestall it in many instances.']", "label": 0 }, { "id": "train_604", "context": "Paleontologist: Plesiosauromorphs were gigantic, long-necked marine reptiles that ruled the oceans during the age of the dinosaurs. Most experts believe that plesiosauromorphs lurked and quickly ambushed their prey. However, plesiosauromorphs probably hunted by chasing their prey over long distances. Plesiosauromorph fins were quite long and thin, like the wings of birds specialized for long-distance flight.", "question": "Which one of the following is an assumption on which the paleontologist's argument depends?", "answers": "['Birds and reptiles share many physical features because they descend from common evolutionary ancestors.', 'During the age of dinosaurs, plesiosauromorphs were the only marine reptiles that had long, thin fins.', \"The shape of a marine animal's fin affects the way the animal swims in the same way as the shape of a bird's wing affects the way the bird flies.\", 'A gigantic marine animal would not be able to find enough food to meet the caloric requirements dictated by its body size if it did not hunt by chasing prey over long distances.']", "label": 2 }, { "id": "train_605", "context": "Anthropologist: Many people think that if human language evolved, then something like it must be present in those species most closely related to humans, such as chimpanzees. They reason that since new traits evolve gradually, something like human language, albeit cruder, must exist in some species from which humans evolved. This general line of argument may be reasonable, but it simply does not follow that chimpanzees have anything like human language, because humans did not evolve from chimpanzees. While chimpanzees are indeed closely related to humans, this is because both evolved from a common ancestor. The evolution of human language might easily have begun after the extinction of that common ancestor.", "question": "Which one of the following most accurately expresses the main conclusion of the anthropologist's argument?", "answers": "['The evolution of human language began after the disappearance of an extinct species from which both humans and chimpanzees evolved.', 'The assumption that something like human language must exist in some species from which humans evolved has no clearcut linguistic implications for chimpanzees.', 'The communicative systems of chimpanzees are cruder than human language.', 'Human language is a by-product of human intelligence, which chimpanzees lack.']", "label": 1 }, { "id": "train_606", "context": "A philosophical paradox is a particularly baffling sort of argument. Your intuitions tell you that the conclusion of a philosophical paradox is false, but they also tell you that its conclusion follows logically from true premises. Solving a philosophical paradox requires accepting any one of three things: that its conclusion is true, that at least one of its premises is not true, or that its conclusion does not really follow logically from its premises.", "question": "If the statements above are true, which one of the following must also be true?", "answers": "['Philosophical paradoxes with one or two premises are more baffling than those with several premises.', 'Solving a philosophical paradox requires accepting something that intuitively seems to be incorrect.', \"The conclusion of a philosophical paradox cannot be false if all the paradox's premises are true.\", 'If it is not possible to accept that the conclusion of a particular philosophical paradox is true, then it is not possible to solve that paradox.']", "label": 1 }, { "id": "train_607", "context": "That the policy of nuclear deterrence has worked thus far is unquestionable. Since the end of the Second World War, the very fact that there were nuclear armaments in existence has kept major powers from using nuclear weapons, for fear of starting a worldwide nuclear exchange that would make the land of the power initiating it uninhabitable. The proof is that a third world war between superpowers has not happened.", "question": "Which one of the following, if true, indicates a flaw in the argument?", "answers": "['Continuing to produce nuclear weapons beyond the minimum needed for deterrence increases the likelihood of a nuclear accident.', 'The major powers have engaged in many smaller-scale military operations since the end of the Second World War, while refraining from a nuclear confrontation.', \"Maintaining a high level of nuclear armaments represents a significant drain on a country's economy.\", 'It cannot be known whether it was nuclear deterrence that worked, or some other factor, such as a recognition of the economic value of remaining at peace.']", "label": 3 }, { "id": "train_608", "context": "A recent poll showed that almost half of the city' s residents believe that Mayor Walker is guilty of ethics violations. Surprisingly, however, 52 percent of those surveyed judged Walker' s performance as mayor to be good or excellent, which is no lower than it was before anyone accused him of ethics violations.", "question": "Which one of the following, if true, most helps to explain the surprising fact stated above?", "answers": "['About a fifth of those polled did not know that Walker had been accused of ethics violations.', 'Walker has defended himself against the accusations by arguing that the alleged ethics violations were the result of honest mistakes by his staff members.', \"In the time since Walker was accused of ethics violations, there has been an increase in the percentage of city residents who judge the performance of Walker's political opponents to be good or excellent.\", 'Almost all of the people who believe that Walker is guilty of ethics violations had thought, even before he was accused of those violations, that his performance as mayor was poor.']", "label": 3 }, { "id": "train_609", "context": "Adults who work outside the home spend, on average, 100 minutes less time each week in preparing dinner than adults who do not work outside the home. But, contrary to expectation, comparisons show that the dinners eaten at home by the two groups of adults do not differ significantly with respect to nutritional value, variety of menus, or number of courses.", "question": "Which one of the following, if true, most helps to resolve the apparent discrepancy in the information above?", "answers": "['The fat content of the dinners eaten at home by adults who do not work outside the home is 25 percent higher than national guidelines recommend.', 'Adults who work outside the home eat dinner at home 20 percent less often than do adults who do not work outside the home.', 'Adults who do not work outside the home tend to prepare breakfast more often than adults who work outside the home.', 'Adults who work outside the home spend 2 hours less time per day on all household responsibilities, including dinner preparation, than do adults who do not work outside the home.']", "label": 1 }, { "id": "train_610", "context": "The writing styles in works of high literary quality are not well suited to the avoidance of misinterpretation. For this reason, the writing in judicial decisions, which are primarily intended as determinations of law, is rarely of high literary quality. However, it is not uncommon to find writing of high literary quality in dissenting opinions, which are sometimes included in written decisions in cases heard by a panel of judges.", "question": "Which one of the following, if true, most helps to resolve the apparent discrepancy in the statements above?", "answers": "['It is not uncommon for more than one judge to have an influence on the way a dissenting opinion is written.', 'The law is not to any great extent determined by dissenting opinions.', 'Judicial decisions issued by panels of judges are likely to be more widely read than are judicial decisions issued by a single judge who hears a case alone.', 'Unlike literary works, legal opinions rely heavily on the use of technical terminology.']", "label": 1 }, { "id": "train_611", "context": "The proposal to extend clinical trials, which are routinely used as systematic tests of pharmaceutical innovations, to new surgical procedures should not be implemented. The point is that surgical procedures differ in one important respect from medicinal drugs: a correctly prescribed drug depends for its effectiveness only on the drug' s composition, whereas the effectiveness of even the most appropriate surgical procedure is transparently related to the skills of the surgeon who uses it.", "question": "The reasoning in the argument is flawed because the argument", "answers": "['describes a dissimilarity without citing any scientific evidence for the existence of that dissimilarity', 'ignores the possibility that the challenged proposal is deliberately crude in a way designed to elicit criticism to be used in refining the proposal', 'does not consider that new surgical procedures might be found to be intrinsically more harmful than the best treatment previously available', \"assumes that a surgeon's skills remain unchanged throughout the surgeon's professional life\"]", "label": 2 }, { "id": "train_612", "context": "In seventeenth-century France, many remunerative government positions in the provinces were sold by the king and then passed from father to son. Historians have concluded that this system was more effective than a purely meritocratic system in ensuring not only that most of the officials were competent, but that they were more sympathetic to the local people than to the king.", "question": "Each of the following, if true of seventeenth-century France, helps to support the historians' conclusion EXCEPT:", "answers": "['To raise revenues, new offices were often created and sold by the king.', 'Most government officials had close ties to the people in the provinces in which they held their positions.', 'It was difficult for the king to oust a disloyal government official from an inherited office.', 'Government officials often received financial gratuities from local merchants.']", "label": 0 }, { "id": "train_613", "context": "Children should be discouraged from reading Jones' s books. Reading them is like eating candy, which provides intense, short-term sensory stimulation but leaves one poorly nourished and dulls one' s taste for better fare. In other words, the problem with letting children read Jones' s books is that__.", "question": "Which one of the following most logically completes the argument above?", "answers": "['it will lead them to develop a taste for candy and sweets', 'their doing so interferes with the development of appreciation for more challenging literature', 'children may become so enthralled with books that they will want to spend all their time reading', 'their message may undermine the positive teaching done by parents']", "label": 1 }, { "id": "train_614", "context": "Editorial: Our city' s public transportation agency is facing a budget shortfall. The fastest growing part of the budget has been employee retirement benefits, which are exceptionally generous. Unless the budget shortfall is resolved, transportation service will be cut, and many transportation employees will lose their jobs. Thus, it would be in the employees' best interest for their union to accept cuts in retirement benefits.", "question": "Which of the following is an assumption the editorial's argument requires?", "answers": "['The only feasible way for the agency to resolve the budget shortfall would involve cutting transportation service and eliminating jobs.', \"The transportation employees' union should not accept cuts in retirement benefits if doing so would not be in the employees' best interest.\", \"Cutting the retirement benefits would help resolve the agency's budget shortfall.\", \"The transportation employees' union will not accept cuts in retirement benefits if doing so will not allow more transportation employees to keep their jobs.\"]", "label": 2 }, { "id": "train_615", "context": "A society' s infant mortality rate is an accepted indicator of that society' s general health status. Even though in some localities in the United States the rate is higher than in many developing countries, in the United States overall the rate has been steadily declining. This decline does not necessarily indicate, however, that babies in the United States are now, on the average, healthier at birth than they were in the past.", "question": "Which one of the following reasons, if true, most strongly supports the claim made above about the implications of the decline?", "answers": "['The United States has been developing and has achieved extremely sophisticated technology for saving premature and low-birth-weight babies, most of whom require extended hospital stays.', 'Low birth weight is a contributing factor in more than half of the infant deaths in the United States.', 'Babies who do not receive adequate attention from a caregiver fail to thrive and so they gain weight slowly.', 'In eleven states of the United States, the infant mortality rate declined last year.']", "label": 0 }, { "id": "train_616", "context": "Editorial: Almost every year the Smithfield River floods the coastal fishing community of Redhook, which annually spends $3 million on the cleanup. Some residents have proposed damming the river, which would cost $5 million but would prevent the flooding. However, their position is misguided. A dam would prevent nutrients in the river from flowing into the ocean. Fish that now feed on those nutrients would start feeding elsewhere. The loss of these fish would cost Redhook $10 million annually.", "question": "Which one of the following most accurately expresses the main conclusion of the editorial's argument?", "answers": "['The Smithfield River should be dammed to prevent flooding.', 'Nutrients from the Smithfield River are essential to the local fish population.', 'For Redhook to build a dam on the Smithfield River would be a mistake.', 'The Smithfield River floods cost Redhook $3 million every year.']", "label": 2 }, { "id": "train_617", "context": "Unquestionably, inventors of useful devices deserve credit for their ingenuity, but the engineers who help develop an invention get too little recognition. Although inventors sometimes serve as their own engineers, more often, engineers must translate an inventor' s insight into something workable and useful. Ttherefore, engineers also deserve credit for their contribution.", "question": "The claim that inventors sometimes serve as their own engineers plays which one of the following roles in the argument?", "answers": "['It supports an earlier statement regarding what is at issue in the argument.', 'It introduces an alternative solution to the problem the argument is addressing.', 'It indicates that the problem identified in the argument does not arise in every instance.', 'It concedes that a distinction on which the argument relies is unclear.']", "label": 2 }, { "id": "train_618", "context": "The Fenwicks returned home from a trip to find two broken bottles on their kitchen floor. There was no sign of forced entry and nothing in the house appeared to have been taken. Although the Fenwicks have a pet cat that had free run of the house while they were away, the Fenwicks hypothesized that they had left a back door unlocked and that neighborhood children had entered through it, attempted to raid the kitchen, and left after breaking the bottles.", "question": "Each of the following, if true, helps to support the Fenwicks' hypothesis EXCEPT:", "answers": "[\"There have been several recent burglaries in the Fenwicks' neighborhood in which neighborhood children were suspected.\", 'The Fenwicks returned home from their trip later than they had planned.', \"When the Fenwicks returned home, they found children's footprints on the back porch that had not been there before their trip.\", \"A neighbor thought he had seen the Fenwicks' back door closing while the Fenwicks were away.\"]", "label": 1 }, { "id": "train_619", "context": "A long-term health study that followed a group of people who were age 35 in 1950 found that those whose weight increased by approximately half a kilogram or one pound per year after the age of 35 tended, on the whole, to live longer than those who maintained the weight they had at age 35. This finding seems at variance with other studies that have associated weight gain with a host of health problems that tend to lower life expectancy.", "question": "Which one of the following, if true, most helps to resolve the apparently conflicting findings?", "answers": "['Diets that tend to lead to weight gain often contain not only excess fat but also unhealthful concentrations of sugar and sodium.', 'Smokers, who tend to be leaner than nonsmokers, tend to have shorter life spans than nonsmokers.', 'Individuals who reduce their cholesterol levels by losing weight can thereby also reduce their risk of dying from heart attacks or strokes.', 'The normal deterioration of the human immune system with age can be slowed down by a reduction in the number of calories consumed.']", "label": 1 }, { "id": "train_620", "context": "Medical columnist: Some doctors recommend taking vitamin C to help maintain overall health because vitamin C is an antioxidant, a substance that protects the body from certain types of oxygen particles that can trigger disease. People suffering from various ailments are encouraged to take vitamin C to guard against developing other health problems. However, doctors are now discouraging some cancer patients from taking vitamin C, even when they are undergoing therapies with side effects that are detrimental to their overall health.", "question": "Which one of the following, if true, most helps to explain why the doctors' recommendation to some cancer patients differs from the general recommendation regarding vitamin C?", "answers": "[\"Certain side effects of cancer therapies that are detrimental to patients' overall health are not affected by vitamin C.\", 'Cancer cells that are susceptible to certain types of cancer therapies are not likely to be affected by the presence of vitamin C.', \"Vitamin C has not been shown to reduce people's risk of developing cancer, even at the very high dosage levels recommended by some doctors.\", 'Some kinds of cancer cells absorb large amounts of vitamin C, which interferes with the oxidation mechanism by which many cancer therapies kill cancer cells.']", "label": 3 }, { "id": "train_621", "context": "Medical reporter: Studies have consistently found that taking an aspirin a day thins the blood slightly, thereby helping to prevent or reduce the severity of heart disease. Since heart disease is one of the most common types of ill health in industrialized nations, most people in such nations would ttherefore be in better health if they took an aspirin a day.", "question": "The reasoning in the doctor's argument is most vulnerable to criticism on which one of the following grounds?", "answers": "['It fails to address the possibility that the studies on the beneficial effects of aspirin were conducted only in industrialized nations.', 'It overlooks the possibility that preventing or reducing the severity of heart disease has little or no effect on any of the other most common diseases in industrialized nations.', 'It overlooks the possibility that even if a disease is one of the most common in a nation, most people in that nation are not in significant danger of developing that disease.', 'It takes for granted that if medication can reduce the severity of heart disease, it can also prevent some cases of heart disease.']", "label": 2 }, { "id": "train_622", "context": "A report of a government survey concluded that Center City was among the ten cities in the nation with the highest dropout rate from its schools. The survey data were obtained by asking all city residents over the age of 19 whether they were high school graduates and computing the proportion who were not. A city school of official objected that the result did not seem accurate according to the schools' figures.", "question": "The school official can most properly criticize the reasoning by which the survey report reached its result for failure to do which one of me following?", "answers": "['predict the effect of the information contained in the report on future high school dropout rates for the city', \"take into account instances of respondents' dropping out that occurred before the respondents reached high school\", \"consider whether a diploma from the city's high schools signaled the same level of achievement over time\", \"distinguish between residents who had attended the city's schools and those who had received their schooling elsewhere\"]", "label": 3 }, { "id": "train_623", "context": "Researchers studying athletes found that those who played mainly for the love of their sport actually had sharper vision during athletic competitions than those whose main goal was winning a trophy or championship. The vision of the first group of athletes was sharper because the concentration necessary for acute vision during an activity is typically possessed to a greater degree by those whose attention is focused on the activity itself.", "question": "Which one of the following is most strongly supported by the statements above?", "answers": "['Winning a trophy or championship is not important to athletes who play mainly for the love of their sport.', 'Athletes who play mainly for the love of their sport concentrate more on the sport itself during athletic competitions than do athletes whose main goal is winning a trophy or championship.', \"If an athlete's main goal during an athletic competition is winning a trophy or championship, that athlete will lack the concentration necessary for adequate vision during that competition .\", 'It is impossible for an athlete to concentrate on more than one thing at a time during an athletic competition.']", "label": 1 }, { "id": "train_624", "context": "The more sunlight our planet reflects back into space, the cooler the global atmosphere tends to become. Snow and ice reflect much more sunlight back into space than do ocean water or land without snow cover. Ttherefore, the greater the area of Earth' s surface that is covered with snow and ice, the cooler, on average, the global atmosphere is likely to become.", "question": "Which one of the following, if true, would most strengthen the argument?", "answers": "[\"Other factors besides the reflectivity of ice and snow affect the cooling of Earth's atmosphere.\", 'The atmosphere derives most of its heat from the passage of sunlight through it.', 'Lighter-colored soil reflects more sunlight back into space than does darker-colored soil.', \"Ocean water and land heated by sunlight in turn warm Earth's atmosphere.\"]", "label": 3 }, { "id": "train_625", "context": "Columnist: Video games are not works of art. No matter how rich the aesthetic experience produced by a video game might be, it is interactive: players make choices that affect the outcome of the game. For something to be a work of art, it must produce an aesthetic experience that is controlled by the artist or artists who created the work.", "question": "The conclusion of the columnist's argument can be properly drawn if which one of the following is assumed?", "answers": "[\"Players' choices that have no effect on the outcome of a video game are irrelevant to the aesthetic experience produced by that game.\", 'An aesthetic experience cannot be both interactive and controlled by the artist or artists who created the work.', 'Most video game creators do not intend their video games to be works of art.', 'Typically, video game players do not themselves create video games.']", "label": 1 }, { "id": "train_626", "context": "The asteroid that hit the Yucatan Peninsula 65 million years ago caused both long-term climatic change and a tremendous firestorm that swept across North America. We cannot show that it was this fire that caused the extinction of the triceratops, a North American dinosaur in existence at the time of the impact of the asteroid. Nor can we show that the triceratops became extinct due to the climatic changes resulting from the asteroid' s impact. Hence, we cannot attribute the triceratops' s extinction to the asteroid' s impact.", "question": "Which one of the following has flawed reasoning most similar to the flawed reasoning in the argument above?", "answers": "['The flooding in the basement caused damage to the furnace and also caused a short in the electrical system. Fire investigators could not show that the damage to the furnace caused the fire that resulted shortly after the flooding, nor could they show that the fire was caused by the short in the electrical system. Ttherefore, we cannot claim that the flooding in the basement caused the fire.', 'We have good reason to believe that the cause of the flooding along the coast was the unusually high tides. We also have good reason to believe that the cause of the unusually high tides was either the sun or the moon. So it is reasonable to maintain that the cause of the flooding was either the sun or the moon.', \"It has not been conclusively proven that the accident was caused by John's driving at excessive speeds. Nor has it been conclusively proven that the accident was the result of John's weaving out of his lane. Hence, it has been conclusively proven that the cause of the accident was neither John's driving at excessive speeds nor John's weaving out of his lane.\", 'I know that one cannot move this piano unless one can lift at least 150 kilograms. I doubt that either Leon or Pam can lift 150 kilograms alone. So I doubt that either Leon or Pam can move this piano alone. Thus, I doubt that Leon and Pam can move this piano together.']", "label": 0 }, { "id": "train_627", "context": "Commentator: In the new century, only nations with all the latest electronic technology will experience great economic prosperity. The people in these nations will be constantly bombarded with images of how people in other countries live. This will increase their tendency to question their own customs and traditions, leading to a dissolution of those customs and traditions. Hence, in the new century, the stability of a nation' s cultural identity will likely __ .", "question": "Which one of the following most logically completes the commentator's argument?", "answers": "['be best maintained by ensuring gradual assimilation of new technical knowledge and skills', 'be ensured by laws that protect the customs and traditions of that culture', 'be threatened only if the people of that culture fail to acquire the latest technical skills', 'decrease if that nation comes to have a high level of economic wealth']", "label": 3 }, { "id": "train_628", "context": "Technological improvements will enable food production to increase as populations increase. However, increases in food production will be negligible unless societies become more centralized so that all factors contributing to the production of food can be better coordinated. But, historically, the more centralized a society was, the greater the percentage of its people who perished if and when it collapsed. Thus, increasing the centralization of societies in an effort to increase food production via better technology will merely exacerbate the disasters associated with societal collapse.", "question": "The statements above, if true, most strongly support which one of the following?", "answers": "['The more centralized a society is, the greater its need for increased food production.', 'Societies have become more centralized as technology has improved.', 'Not every problem associated with the collapse of a centralized society would be prevented by technological improvements.', 'The production of food can be increased only by improved technology.']", "label": 2 }, { "id": "train_629", "context": "Last summer one out of every five teenagers who sought employment failed to find it. This is hard to understand, since during this same time employers were searching for teenagers to fill an abundance of summer jobs. Many employers had been offering good pay and, in some cases, medical benefits.", "question": "Which of the following, if true, most helps to explain why teen-age unemployment was high when there were many job openings?", "answers": "['Many summer jobs offer training that will help the teen-ager secure a full-time position after graduation.', 'During the summer, some teenagers continue to work at jobs that they have held during the school year.', 'Many state programs that have been developed to help teen-agers find summer employment have recently had their budgets increased.', 'Most summer jobs are located in suburban areas that are inaccessible to the high proportion of teen-agers who live in cities.']", "label": 3 }, { "id": "train_630", "context": "Beginning in 1966 all new cars sold in Morodia were required to have safety belts and power steering. Previously, most cars in Morodia were without these features. Safety belts help to prevent injuries in collisions, and power steering helps to avoid collisions in the first place. But even though in 1966 one-seventh of the cars in Morodia were replaced with new cars, the number of car collisions and collision-related injuries did not decline.", "question": "Which of the following, if true about Morodia, most helps to explain why the number of collisions and collision-related injuries in Morodia failed to decline in 1966?", "answers": "['In 1965, subtantially more than one- seventh of the cars in Morodia were replaced with new cars.', 'Because of a driver-education campaign, most drivers and passengers in cars that did have safety belts used them in 1966.', 'An excessive reliance on the new safety features led many owners of new cars to drive less cautiously in 1966 than before.', 'Most of the new cars bought in 1966 were bought in the months of January and February.']", "label": 2 }, { "id": "train_631", "context": "In the nation of Partoria, large trucks currently account for 6 percent of miles driven on Partoria' s roads but are involved in 12 percent of all highway fatalities. The very largest trucks-those with three trailers-had less than a third of the accident rate of single-and double-trailer trucks. Clearly, ttherefore, one way for Partoria to reduce highway deaths would be to require shippers to increase their use of triple-trailer trucks.", "question": "Which of the following, if true, most seriously weakens the argument?", "answers": "['Very few fatal collisions involving trucks in Partoria are collisions between two trucks.', 'No matter what changes Partoria makes in the regulation of trucking, it will have to keep some smaller roads off-limits to all large trucks.', 'Partorian trucking companies have so far used triple-trailer trucks on lightly traveled sections of major highways only.', 'In Partoria, the safety record of the trucking industry as a whole has improved slightly over the past ten years.']", "label": 2 }, { "id": "train_632", "context": "Archaeologists use technology to analyze ancient sites. It is likely that this technology will advance considerably in the near future, allowing archaeologists to gather more information than is currently possible. If they study certain sites now, they risk contaminating or compromising them for future studies. Ttherefore, in order to maximize the potential for gathering knowledge in the long run, a team of archaeologists plans to delay the examination of a newly excavated site.", "question": "Which of the following would be most useful to investigate for the purpose of evaluating the plan's prospects for achieving its goal?", "answers": "['Whether any of the contents of the site will significantly deteriorate before the anticipated technology is available', 'Whether the site was inhabited by a very ancient culture', 'Whether the team can study a site other than the newly excavated site for the time being', 'Whether the anticipated technology will damage objects under study']", "label": 0 }, { "id": "train_633", "context": "Trade negotiator: Increasing economic prosperity in a country tends to bring political freedom to its inhabitants. Ttherefore, it is wrong for any country to adopt trade policies that are likely to seriously hinder growth in the prosperity of any other country.", "question": "Which one of the following principles, if valid, would most help to justify the trade negotiator's reasoning?", "answers": "['The primary reason that any country seeks economic prosperity is to foster political freedom in that country.', 'Every country should adopt at least some policies that encourage the development of political freedom in other countries.', 'A country should not do anything that might hinder the growth of political freedom in any other country.', \"Both economic prosperity and political freedom can contribute to the overall well-being of any country's inhabitants.\"]", "label": 2 }, { "id": "train_634", "context": "Astronauts who experience weightlessness frequently get motion sickness. The astronauts see their own motion relative to passing objects, but while the astronauts are weightless their inner ears indicate that their bodies are not moving. The astronauts' experience is best explained by the hypothesis that conflicting information received by the brain about the body' s motion causes motion sickness.", "question": "Which one of the following, if true, provides the strongest additional support for the hypotheses above?", "answers": "['During rough voyages ship passengers in cabins providing a view of the water are less likely to get motion sickness than are passengers in cabins providing no view.', 'Some automobile passengers whose inner ears indicate that they are moving and who have a clear view of the objects they are passing get motion sickness.', 'Many people who are experienced airplane passengers occasionally get motion sickness.', 'People who have aisle seats in trains or airplanes are as likely to get motion sickness as are people who have window seats.']", "label": 0 }, { "id": "train_635", "context": "Productivity is average output per worker per unit of time. High productivity cannot be achieved without adequate training of workers. So high productivity does not depend on having high-tech equipment.", "question": "The reasoning in the argument is most vulnerable to criticism on the grounds that the argument", "answers": "['ignores the possibility that having high-tech equipment is required for adequate training of workers', 'overlooks the fact that increases in productivity may not be desirable in some circumstances', 'confuses a stated requirement for having high- tech equipment with a sufficient condition for having high-tech equipment', 'presumes without giving justification that educating workers always results in an increase in their productivity']", "label": 0 }, { "id": "train_636", "context": "When expert witnesses give testimony, jurors often do not understand the technical information and thereby are in no position to evaluate such testimony. Although expert witnesses on opposite sides often make conflicting claims, the expert witnesses on both sides frequently seem competent, leaving the jury unable to assess the reliability of their testimonies.", "question": "The statements above, if true, most strongly support which one of the following?", "answers": "['Jurors who understand the technical information presented in a case can usually assess its legal implications accurately.', 'There should be limits placed on how much technical information can be considered by both sides in preparing a legal case.', 'Jury decisions in cases involving expert witness testimonies are not always determined by the reliability of those testimonies.', 'Expert witnesses who testify on opposite sides in legal cases are likely to agree in their evaluations of technical claims.']", "label": 2 }, { "id": "train_637", "context": "In Washington County, attendance at the movies is just large enough for the cinema operators to make modest profits. The size of the county's population is stable and is not expected to increase much. Yet there are investors ready to double the number of movie screens in the county within five years, and they are predicting solid profits both for themselves and for the established cinema operators.", "question": "Which of the following, if true about Washington County, most helps to provide a justification for the investors' prediction?", "answers": "[\"The sale of snacks and drinks in cinemas accounts for a steadily growing share of most cinema operators' profits.\", 'As distinct from the existing cinemas, most of the cinemas being planned would be located in downtown areas, in hopes of stimulating an economic revitalization of those areas.', \"Over the next ten years, people in their teenage years, the prime moviegoing age, will be a rapidly growing proportion of the county's population.\", 'Spending on video purchases, as well as spending on video rentals, has been increasing modestly each year for the past ten years.']", "label": 2 }, { "id": "train_638", "context": "Although the pesticide TDX has been widely used by fruit growers since the early 1960' s, a regulation in force since 1960 has prohibited sale of fruit on which any TDX residue can be detected. That regulation is about to be replaced by one that allows sale of fruit on which trace amounts of TDX residue are detected. In fact, however, the change will not allow more TDX on fruit than was allowed in the 1960' s, because __.", "question": "Which of the following most logically completes the argument?", "answers": "['pre-1970 techniques for detecting TDX residue could detect it only when it was present on fruit in more than the trace amounts allowed by the new regulations', \"at least a small fraction of the fruit sold each year since the early 1960's has had on it greater levels of TDX than the regulation allows\", \"many more people today than in the 1960's habitually purchase and eat fruit without making an effort to clean residues off the fruit\", 'the presence of TDX on fruit in greater than trace amounts has not been shown to cause any harm even to children who eat large amounts of fruit']", "label": 0 }, { "id": "train_639", "context": "Hunter: Many people blame hunters alone for the decline in Greenrock National Forest' s deer population over the past ten years. Yet clearly, black bears have also played an important role in this decline. In the past ten years, the forest's protected black bear population has risen sharply, and examination of black bears found dead in the forest during the deer hunting season showed that a number of them had recently fed on deer.", "question": "In the hunter's argument, the portion in boldface plays which of the following roles?", "answers": "['It introduces a judgment that the argument opposes.', 'It is an explanation that the argument concludes is correct.', 'It provides evidence in support of the main conclusion of the argument.', 'It is the main conclusion of the argument.']", "label": 0 }, { "id": "train_640", "context": "On Wednesdays, Kate usually buys some guava juice. But the only place she can buy guava juice is the local health food store. It follows that she must sometimes shop at the local health food store on Wednesdays.", "question": "The argument above is most similar in its pattern of reasoning to which one of the following arguments?", "answers": "[\"Only teachers at the Culinary Institute are allowed to use the institute's main kitchen. Most dinners at Cafe Delice are prepared in that kitchen. So at least some dinners at Cafe Delice must be prepared by Culinary Institute teachers.\", 'Only teachers at the Culinary Institute are allowed to use the main kitchen of the institute. Dinners at Cafe Delice are usually prepared by Culinary Institute teachers. So dinners at Cafe Delice must sometimes be prepared in the main kitchen of the Culinary Institute.', 'Most dinners at Cafe Delice are prepared in the main kitchen of the Culinary Institute. All the teachers at the institute are allowed to use that kitchen. So at least some dinners at Cafe Delice must be prepared by Culinary Institute teachers.', 'All dinners at Cafe Delice are prepared in the main kitchen of the Culinary Institute. But only teachers at the institute are allowed to use that kitchen. So the dinners at Cafe Delice must be prepared by Culinary Institute teachers.']", "label": 0 }, { "id": "train_641", "context": "Garnet and RenCo each provide health care for their employees. Garnet pays for both testing of its employees' cholesterol levels and treatment of high cholesterol. This policy saves Garnet money, since high cholesterol left untreated for many years leads to conditions that require very expensive treatment. However, RenCo does not have the same financial incentive to adopt such a policy, because __.", "question": "Which of the following most logically completes the passage?", "answers": "['the mass media regularly feature stories encouraging people to maintain diets that are low in cholesterol', 'the average length of time an employee stays with RenCo is less than it is with Garnet', 'RenCo has significantly more employees than Garnet has', \"RenCo's employees are unlikely to have higher cholesterol levels than Garnet's employees\"]", "label": 1 }, { "id": "train_642", "context": "Nature constantly adjusts the atmospheric carbon level. An increase in the level causes the atmosphere to hold more heat, which causes more water to evaporate from the oceans, which causes increased rain. Rain washes some carbon from the air into the oceans, where it eventually becomes part of the seabed. A decrease in atmospheric carbon causes the atmosphere to hold less heat, which causes decreased evaporation from the oceans, which causes less rain, and thus less carbon is washed into the oceans. Yet some environmentalists worry that burning fossil fuels may raise atmospheric carbon to a dangerous level. It is true that a sustained increase would threaten human life. But the environmentalists should relax -- nature will continually adjust the carbon level.", "question": "Each of the following can be inferred from the information in the passage EXCEPT:", "answers": "['A decrease in the level of carbon in the atmosphere causes decreased evaporation of ocean water.', 'A decrease in the level of atmospheric heat causes a decrease in the level of carbon in the atmosphere.', 'An increase in the level of atmospheric heat causes increased rainfall.', 'A decrease in the level of atmospheric heat causes a decrease in the amount of carbon that rain washes into the oceans from the air.']", "label": 1 }, { "id": "train_643", "context": "Lobbyist: Those who claim that automobile exhaust emissions are a risk to public health are mistaken. During the last century, as automobile exhaust emissions increased, every relevant indicator of public health improved dramatically rather than deteriorated.", "question": "The flaw in the lobbyist's reasoning can most effectively be demonstrated by noting that, by parallel reasoning, we could conclude that", "answers": "['people with insurance do not need to lock their doors because if anything is stolen the insurance company will pay to replace it', 'using a cell phone while driving is not dangerous because the number of traffic accidents has decreased since the invention of the cell phone', \"smoking cigarettes is not bad for one's health because not all cigarette smokers get smoking-related illnesses\", 'inspecting commercial airplanes for safety is unnecessary because the number of commercial airplane crashes has decreased over the last decade']", "label": 1 }, { "id": "train_644", "context": "Mainstream economic theory holds that manufacturers, in deciding what kinds of products to manufacture and what form those products should have, simply respond to the needs and desires of consumers. However, most major manufacturers manipulate and even create consumer demand, as anyone who watches television knows. Since even mainstream economic theorists watch television, their motive in advancing this theory must be something other than disinterested concern for scientific truth.", "question": "The claim that manufacturers manipulate and create consumer demand plays which one of the following roles in the argument?", "answers": "['It provides supplementary background information.', \"It states a possible objection to the argument's conclusion.\", 'It is the conclusion of the argument.', 'It is one of the claims on which the conclusion is based.']", "label": 3 }, { "id": "train_645", "context": "Therapist: The ability to trust other people is essential to happiness, for without trust there can be no meaningful emotional connection to another human being, and without meaningful emotional connections to others we feel isolated.", "question": "Which one of the following, if assumed, allows the conclusion of the therapist's argument to be properly inferred?", "answers": "['Anyone who is able to trust other people has a meaningful emotional connection to at least one other human being.', 'To avoid feeling isolated, it is essential to trust other people.', 'No one who is feeling isolated can feel happy.', 'Anyone who has a meaningful emotional connection to another human being can be happy.']", "label": 2 }, { "id": "train_646", "context": "Environmentalist: When bacteria degrade household cleaning products, vapors that are toxic to humans are produced. Unfortunately, household cleaning products are often found in landfills. Thus, the common practice of converting landfills into public parks is damaging human health.", "question": "Which one of the following is an assumption the environmentalist's argument requires?", "answers": "['Converting a landfill into a public park will cause no damage to human health unless toxic vapors are produced in that landfill and humans are exposed to them.', 'If vapors toxic to humans are produced by the degradation of household cleaning products by bacteria in any landfill, then the health of at least some humans will suffer.', 'In at least some landfills that have been converted into public parks there are bacteria that degrade household cleaning products.', 'If a practice involves the exposure of humans to vapors from household cleaning products, then it causes at least some damage to human health.']", "label": 2 }, { "id": "train_647", "context": "Several carefully conducted studies showed that 75 percent of strict vegetarians reached age 50 without developing serious heart disease. We can conclude from this that avoiding meat increases one' s chances of avoiding serious heart disease. Ttherefore, people who want to reduce the risk of serious heart disease should not eat meat.", "question": "The flawed pattern of reasoning exhibited by which one of the following is most similar to that exhibited by the argument above?", "answers": "['The majority of people who regularly drink coffee experience dental problems in the latter part of their lives. Since there is this correlation between drinking coffee and incurring dental problems, the government should make coffee less accessible to the general public.', \"Studies show that people who do not exercise regularly have a shorter life expectancy than those who exercise regularly. To help increase their patients' life expectancy, doctors should recommend regular exercise to their patients.\", \"Studies have shown that cigarette smokers have a greater chance of incurring heart disease than people who do not smoke. Since cigarette smoking increases one's chances of incurring heart disease, people who want to try to avoid heart disease should give up cigarette smoking.\", \"Most people who exercise regularly are able to handle stress. This shows that exercising regularly decreases one's chances of being overwhelmed by stress. So people who want to be able to handle stress should regularly engage in exercise.\"]", "label": 3 }, { "id": "train_648", "context": "When glass products are made from recycled glass, the resulting products can be equal in quality to glass products made from quartz sand, the usual raw material. When plastics are recycled, however, the result is inevitably a plastic of a lower grade than the plastic from which it is derived. Moreover, no applications have been found for grades of plastic that are lower than the currently lowest commercial grade.", "question": "Which one of the following is a conclusion that can be properly drawn from the statements above?", "answers": "['Glass products made from recycled glass are less expensive than comparable products made from quartz sand.', 'Products cannot presently be made out of plastic recycled entirely from the currently lowest commercial grade.', 'It is impossible to make glass products from recycled glass that are equal in quality to the best glass products made from the usual raw material.', 'The difference in quality between different grades of glass is not as great as that between different grades of plastic.']", "label": 1 }, { "id": "train_649", "context": "If the price it pays for coffee beans continues to increase, the Coffee Shoppe will have to increase its prices. In that case, either the Coffee Shoppe will begin selling noncoffee products or its coffee sales will decrease. But selling noncoffee products will decrease the Coffee Shoppe' s overall profitability. Moreover, the Coffee Shoppe can avoid a decrease in overall profitability only if its coffee sales do not decrease.", "question": "Which one of the following statements follows logically from the statements above?", "answers": [ "If the Coffee Shoppe's overall profitability decreases, the price it pays for coffee beans will have continued to increase.", "The price it pays for coffee beans cannot decrease without the Coffee Shoppe's overall profitability also decreasing.", "The Coffee Shoppe's overall profitability will decrease if the price it pays for coffee beans continues to increase.", "Either the price it pays for coffee beans will continue to increase or the Coffee Shoppe's coffee sales will increase." ], "label": 2 }, { "id": "train_650", "context": "Mountain County and Sunrise County are two comparable counties, with similar numbers of registered voters. Both are in the same region of the state. In both counties, about 60% of the registered voters are registered Republicans, and almost all others are registered Democrats. In 2005, Republican candidate Alf Landon ran for supervisor of Mountain County, and Republican candidate Thomas Dewey ran for supervisor of Sunrise County. Both took similar stands on issues, and both ran against conservative Democratic candidates in the general county-wide election, and none of these four candidates was an incumbent at the time of this election. The Republican Congressman representing Sunrise County endorsed Mr. Dewey, but the Republican Congressman representing Mountain County refrained from endorsing Mr. Landon. Mr. Dewey won his election, but Mr. Landon lost his election. Clearly, the endorsement positions of the respective Congressmen were the deciding factors in these elections.", "question": "In evaluating the argument, it would be most useful to know which of the following?", "answers": "['What stands Mr. Landon and Mr. Dewey took on issues considered most important by registered Democrat voters', \"How many voters in each of these two counties were aware of their respective Congressman's endorsement positions.\", 'How these two counties have voted in US Congressional and US Presidential elections over the past decade.', 'How typical it has been throughout the state, in the past decade, for a Congressman to endorse a county supervisor candidate running in his Congressional district.']", "label": 1 }, { "id": "train_651", "context": "The city of Workney, in raising bus fares from $1. 00 to $1. 25, proposed that 18 fare tokens be sold for $20. 00 to alleviate the extra burden of the fare increase on the city' s low-income residents. Critics suggested alternatively that 9 fare tokens be sold for $10. 00, because a $20. 00 outlay would be prohibitive for low-income riders.", "question": "The alternative proposal depends on which of the following assumptions?", "answers": "['Low-income residents of Workney will continue to ride the buses in the same numbers despite the fare increase.', 'Buses provide the only means of public transportation in the city of Workney.', \"The proposed fare increase is needed for the purchase of new buses for the city's bus system.\", 'The outlay of $10. 00 for the purchase of 9 fare tokens would not be prohibitive for low-income bus riders.']", "label": 3 }, { "id": "train_652", "context": "The term \"sex\" and \"gender\" are often used interchangeably. But \"sex\" more properly refers to biological differences of male and female, while \"gender\" refers to society' s construction of a system that identifies what is masculine and feminine. Unlike the set of characteristics defining biological sex, the set of traits that are associated with gender does not sort people into two nonoverlapping groups. The traits characterize people in a complex way, so that a person may have both \"masculine\" and \"feminine\" traits.", "question": "Which one of the following statements best expresses a main point of the argument?", "answers": "['Distinctions based on gender are frequently arbitrary.', 'Society rather than the individual decides what is considered proper behavior.', 'The terms \"sex\" and \"gender\" are not properly interchangeable.', 'Gender traits are not determined at birth.']", "label": 2 }, { "id": "train_653", "context": "A controversial program rewards prison inmates who behave particularly well in prison by giving them the chance to receive free cosmetic plastic surgery performed by medical students. The program is obviously morally questionable, both in its assumptions about what inmates might want and in its use of the prison population to train future surgeons. Putting these moral issues aside, however, the surgery clearly has a powerful rehabilitative effect, as is shown by the fact that, among recipients of the surgery, the proportion who are convicted of new crimes committed after release is only half that for the prison population as a whole.", "question": "A flaw in the reasoning of the passage is that it", "answers": "['dismisses moral considerations on the grounds that only matters of fact are relevant', 'relies on evidence drawn from a sample that there is reason to believe is unrepresentative', 'asserts that the rehabilitation of criminals is not a moral issue', 'allows moral issues to be a consideration in presenting evidence about matters of fact']", "label": 1 }, { "id": "train_654", "context": "Daniel: There are certain actions that moral duty obliges us to perform regardless of their consequences. However, an action is not morally good simply because it fulfills a moral obligation. No action can be morally good unless it is performed with the right motivations. Carrie: Our motivations for our actions are not subject to our conscious control. Ttherefore, the only thing that can be required for an action to be morally good is that it fulfill a moral obligation.", "question": "The dialogue most supports the claim that Daniel and Carrie are committed to disagreeing with each other about the truth of which one of the following statements?", "answers": "['No one can be morally required to do something that is impossible to do.', 'An action performed with the wrong motivations cannot be morally good.', \"If a person's motivations for acting are based on a sense of duty, then that person's action is morally good.\", 'Some actions that are performed with the right motivations are not morally good.']", "label": 1 }, { "id": "train_655", "context": "In a recent study, each member of two groups of people, Group A (composed of persons sixty-five to seventy-five years old) and Group B (composed of college students), was required to make a telephone call to a certain number at a specified time. The time when each call was initiated was recorded electronically. Group A proved far better at remembering to make a telephone call precisely at a specified time than did Group B. There were fourteen lapses in Group B but only one lapse in Group A. Clearly, at least one type of memory does not suffer as a person ages.", "question": "Which one of the following, if all of them are true, is LEAST helpful in establishing that the conclusion above is properly drawn?", "answers": "['The members of the two groups received their instructions approximately the same amount of time before they were to make their telephone calls.', 'The same group of researchers answered the calls made by the callers in both study groups.', 'Both groups had unrestricted access to telephones for making the required calls.', 'There was the same number of people in each group.']", "label": 1 }, { "id": "train_656", "context": "Personnel officer: The exorbitant cost of our health-insurance benefits reflects the high dollar amount of medical expenses incurred by our employees. Employees who are out of shape, as a group, have higher doctor bills and longer hospital stays than do their colleagues who are fit. Ttherefore, since we must reduce our health-insurance costs, we should offer a rigorous fitness program of jogging and weight lifting to all employees, and require employees who are out of shape to participate.", "question": "Which of the following, if true, provides the most support for the personnel officer's proposal?", "answers": "['More otherwise fit people are injured by participating in rigorous jogging and weight-lifting programs than are injured by participating in moderate jogging and weight-lifting programs.', \"Moderate fitness programs increase the average person's fitness to the same extent that rigorous fitness programs do.\", 'The likelihood of incurring medical expenses is slightly greater for people who participate in fitness programs offered by their employers than it is for people who participate in programs offered commercially.', 'The medical expenses incurred by fit people who participate in a program of jogging and weight lifting are less than those incurred by fit people who do not participate in such a program.']", "label": 3 }, { "id": "train_657", "context": "Though they soon will, patients should not have a legal right to see their medical records. As a doctor, I see two reasons for this. First, giving them access will be time-wasting because it will significantly reduce the amount of time that medical staff can spend on more important duties, by forcing them to retrieve and return files. Second, if my experience is anything to go by, no patients are going to ask for access to their records anyway. ", "question": "Which one of the following, if true, establishes that the doctor's second reason does not cancel out the first?", "answers": "['The new law does not rule out that doctors may charge patients for extra expenses incurred specifically in order to comply with the new law.', \"The new law will require that doctors, when seeing a patient in their office, must be ready to produce the patient's records immediately, not just ready to retrieve them.\", \"Some doctors have all along had a policy of allowing their patients access to their medical records, but those doctors' patients took no advantage of this policy.\", \"The task of retrieving and returning files would fall to the lowest-paid member of a doctor's office staff.\"]", "label": 1 }, { "id": "train_658", "context": "Biologist: Marine animals known as box jellyfish have eyes with well-formed lenses capable of producing sharp images that reveal fine detail. But the box jellyfish' s retinas are too far forward to receive a clear image, so these jellyfish can receive only a blurry image that reveals prominent features of objects but not fine detail. This example shows that eyes are adapted only to an animal' s needs rather than to some abstract sense of how a good eye would be designed.", "question": "The argument requires assuming which one of the following?", "answers": "['Box jellyfish developed from jellyfish whose retinas received clear images.', 'Box jellyfish have a need to detect prominent features of objects but not fine details.', 'Box jellyfish would benefit from having retinas that allowed their eyes to focus more sharply.', 'Box jellyfish use vision as their main means of detecting prey.']", "label": 1 }, { "id": "train_659", "context": "Unlike the lungs and the kidneys, the liver is an organ well suited for living-donor transplants to children because the portion of the liver which must be removed from a healthy adult to replace the diseased liver of a child will be regenerated in the healthy adult within a few months.", "question": "Which of the following, if true, provides the most additional support for the conclusion drawn above?", "answers": "['There is less danger that a transplanted organ will be rejected when the organ donor is a, close relative of the recipient.', 'In removing the portion of liver for transplant there is danger of causing damage to the spleen of the donor, and that could necessitate the removal of the spleen.', 'Each lobe of the liver functions separately, and it is possible to extract a portion of one lobe without disrupting critical liver functions.', 'There are so few suitable donors that about one-half of the babies with liver disease born in the United States each year die before they can receive a liver transplant.']", "label": 2 }, { "id": "train_660", "context": "Medical research findings are customarily not made public prior to their publication in a medical journal that has had them reviewed by a panel of experts in a process called peer review. It is claimed that this practice delays public access to potentially beneficial information that, in extreme instances, could save lives. Yet prepublication peer review is the only way to prevent erroneous and ttherefore potentially harmful information from reaching a public that is ill equipped to evaluate medical claims on its own. Ttherefore, waiting until a medical journal has published the research findings that have passed peer review is the price that must be paid to protect the public from making decisions based on possibly substandard research.", "question": "The argument assumes that", "answers": "['unless medical research findings are brought to peer review by a medical journal, peer review will not occur', 'the general public does not have access to the medical journals in which research findings are published', 'all medical research findings are subjected to prepublication peer review', 'anyone who does not serve on a medical review panel does not have the necessary knowledge and expertise to evaluate medical research findings']", "label": 0 }, { "id": "train_661", "context": "People widely believed that the controlled use of fire originated 200, 000 years ago with the human species Homo sapiens. However, a site containing a deposit, estimated to be nearly 500, 000 years old, consisting of charcoal, burned animal bones, and charred rocks, has recently been found. Although homo sapiens did not exist at that time, this discovery provides no basis to the claim that the controlled use of fire preceded the Homo sapiens, since __.", "question": "Which of the following most logically completes the passage?", "answers": "['It is possible that fire was used much earlier than 500, 000 years ago', 'The use of fire might have originated independently in several different parts of the world', 'It is impossible to determine whether a charred deposit dating from so long ago resulted from a fire caused by lightening or from the purposeful use of fire', 'Animal bones could have been discarded in a fire used for purposes such as protection or heat rather than for cooking']", "label": 2 }, { "id": "train_662", "context": "Sportfishers introduced the Zander, a type of perch, to Britain' s rivers and canals in the 1970s. Because zander eat large numbers of smaller fish, they have had a devastating effect on native fish populations. To protect the native fish, a government program removed a significant proportion of the zander from Britain' s waterways last year. Surprisingly, this year the loss of native fish to zander has been greater than before.", "question": "Which of the following, if true, would most help to explain the greater effect of zander on the native fish population?", "answers": "['Most of the zander removed were fully grown, and fully grown zander eat large numbers of smaller zander.', \"Every year a large number of zander are caught by sportfisher in Britain's waterway.\", \"Previous government program designed to remove nonnative species from Britain's waterways have failed.\", 'The climate in Britain is very similar to the climate in regions to which zander are native.']", "label": 0 }, { "id": "train_663", "context": "There is no mystery as to why figurative painting revived in the late 1970s. People want to look at recognizable images. Sorting out art theories reflected in abstract paintings is no substitute for the sense of empathy that comes from looking at a realistic painting of a figure in a landscape. Perhaps members of the art-viewing public resented abstract art because they felt that its lack of realistic subject matter was a rejection of the viewers and their world.", "question": "Which one of the following most accurately expresses the main point of the passage?", "answers": "['Abstract paintings often include shapes or forms that are suggestive of real objects or emotions.', 'The general public is unable to understand the theories on which abstract painting is based.', 'The artistic preferences of the art-viewing public stimulated the revival.', 'The art-viewing public wished to see traditional subjects treated in a nontraditional manner.']", "label": 2 }, { "id": "train_664", "context": "Parents should not necessarily raise their children in the ways experts recommend, even if some of those experts are themselves parents. After all, parents are the ones who directly experience which methods are successful in raising their own children.", "question": "Which one of the following most closely conforms to the principle that the passage above illustrates?", "answers": "['In deciding the best way to proceed, a climber familiar with a mountain might do well to ignore the advice of mountain climbing experts unfamiliar with that mountain.', 'Although music theory is intrinsically interesting and may be helpful to certain musicians, it does not distinguish good music from bad:that is a matter of taste and not of theory.', 'A typical farmer is less likely to know what types of soil are most productive than is someone with an advanced degree in agricultural science.', \"Unlike society, one's own conscience speaks with a single voice; it is better to follow the advice of one's own conscience than the advice of society.\"]", "label": 0 }, { "id": "train_665", "context": "People may praise the talent of a painter capable of realistically portraying a scene and dismiss as artistically worthless the efforts of abstract expressionists, but obviously an exact replica of the scene depicted is not the only thing people appreciate in a painting, for otherwise photography would have entirely displaced painting as an art form.", "question": "The argument proceeds by", "answers": "['appealing to an aesthetic principle to defend the tastes that people have', \"appealing to a historical fact to support a claim about people's artistic preferences\", 'explaining a historical fact in terms of the artistic preferences of people', 'considering historical context to defend the artistic preferences of people']", "label": 1 }, { "id": "train_666", "context": "There is evidence to suggest that our cave-dwelling ancestors polished many of their flints to a degree far surpassing what was necessary for hunting purposes. It seems, ttherefore, that early humans possessed an aesthetic sense.", "question": "Which one of the following statements, if true, most seriously weakens the argument?", "answers": "['Most flints used by our cave-dwelling ancestors were not highly polished.', 'The caves in which the highly polished flints were found are unadorned by cave paintings.', 'Flints were often used by early humans for everyday chores other than hunting.', 'Any benefits that an aesthetic sense would have given to cave-dwelling humans are poorly understood.']", "label": 2 }, { "id": "train_667", "context": "On the nights immediately following the mysterious Tunguska event, which destroyed a tract of Siberian wilderness in 1908, eyewitnesses reported seeing noctilucent clouds -- brilliant night-visible clouds made up of ice particles that form rarely and only at very high altitudes. Recently, noctilucent clouds have been observed on the nights following launches of rockets that release large amounts of water vapor into the upper atmosphere. This shows that it was a comet impact and not the impact of an asteroid that caused the destruction in Siberia.", "question": "The argument depends on assuming which one of the following?", "answers": "['Comets but not asteroids release large amounts of water vapor into the upper atmosphere when they collide with Earth.', 'The fact that noctilucent clouds are made of ice particles in the upper atmosphere was only recently discovered.', 'Comets collide with Earth more frequently than asteroids do.', 'Noctilucent clouds are visible for many consecutive nights following the release of water vapor into the upper atmosphere.']", "label": 0 }, { "id": "train_668", "context": "Despite the anti-virus protection program installed on its computers, SmartTech has received many complaints from consumers that their computers had become infected with viruses. In order to reduce the number of its computers vulnerable to viruses, the CEO of SmartTech Corp. has had the company' s engineers install three different virus-blocking programs on each computer before shipping them to consumers. Nevertheless, customer reports of computer viruses have risen since the institution of this initiative.", "question": "Which of the following, if true, most helps to explain why acting on the CEO's plan failed to achieve her goal?", "answers": "['When more than one anti-virus program is installed on a machine, the programs can become less effective.', \"SmartTech has lost approximately 25 percent of its clients as a result of its computers' vulnerability to viruses, though it regained a third of these with its recovery efforts.\", 'Many consumers have found that it is more difficult to use a computer with three anti-virus programs than with one.', \"Corporations that are already vulnerable to hackers often use SmartTech's computers but have stopped doing so since the increased viruses.\"]", "label": 0 }, { "id": "train_669", "context": "A development company has proposed building an airport near the city of Dalton. If the majority of Dalton' s residents favor the proposal, the airport will be built. However, it is unlikely that a majority of Dalton' s residents would favor the proposal, for most of them believe that the airport would create noise problems. Thus, it is unlikely that the airport will be built.", "question": "The reasoning in the argument is flawed in that the argument", "answers": "[\"treats a sufficient condition for the airport's being built as a necessary condition\", 'overlooks the possibility that a new airport could benefit the local economy', 'fails to consider whether people living near Dalton would favor building the airport', 'concludes, on the basis that a certain event is unlikely to occur, that the event will not occur']", "label": 0 }, { "id": "train_670", "context": "Very powerful volcanic eruptions send large amounts of ash high into the atmosphere, blocking sunlight and causing abnormally cold temperatures for a year or more after the eruption. In 44 B. C. there was a powerful eruption of Mount Etna in Sicily. In the following year, Chinese historians recorded summer frosts and dimmed sunlight in China, thousands of miles east of Sicily. If these phenomena were caused by volcanic ash in the atmosphere, then the ash sent into the atmosphere by Mount Etna' s eruption must have spread over great distances.", "question": "In evaluating the support given for the conclusion advanced in the last sentence of the passage, it would be most important to determine whether", "answers": "[\"there were any volcanic eruptions near China around the time of Mount Etna's eruption\", 'modern monitoring equipment can detect the precise path of volcanic ash in the atmosphere', 'the abnormal weather in China lasted for a full year or longer', 'subsequent eruptions of Mount Etna were as powerful as the one in 44 B. C.']", "label": 0 }, { "id": "train_671", "context": "Psychologists have claimed that many people are more susceptible to psychological problems in the winter than in the summer; the psychologists call this condition seasonal affective disorder. Their claim is based on the results of surveys in which people were asked to recall how they felt at various times in the past. However, it is not clear that people are able to report accurately on their past psychological states. Ttherefore, these survey results do not justify the psychologists' claim that there is any such condition as seasonal affective disorder.", "question": "The author criticizes the psychologists' claim by", "answers": "['offering an alternative explanation of the variation in the occurrence of psychological problems across seasons', 'demonstrating that fewer people actually suffer from seasonal affective disorder than psychologists had previously thought', 'questioning the representativeness of the population sample surveyed by the psychologists', 'questioning an assumption that the author attributes to the psychologists']", "label": 3 }, { "id": "train_672", "context": "One should always capitalize the main words and the first and last words of a title. But one should never capitalize articles, or prepositions and conjunctions with fewer than five letters, when they occur in the middle of a title.", "question": "Which one of the following can be properly inferred from the statements above?", "answers": "['If a word that is a preposition or conjunction should be capitalized, then it is the first or last word of the title.', 'Prepositions and conjunctions with five or more letters should be capitalized in any text.', 'If a word in the middle of a title should be capitalized, then that word is neither an article nor a conjunction shorter than five letters.', 'If a word is neither a main word nor a first or last word of a title, then it should not be capitalized.']", "label": 2 }, { "id": "train_673", "context": "There is speculation that the mayor will hire a new staff member this week. But before every prior hiring of a new staff member, the mayor has met with the city' s budget director and finance advisor. No meetings between the mayor and the city' s budget director and finance advisor have occurred or are planned. Ttherefore, the speculation is more than likely baseless.", "question": "Which one of the following most accurately expresses a principle of reasoning employed by the argument in the paragraph above?", "answers": "['A hypothesis can be true even when it is not supported by available data.', 'If two assertions are known to be incompatible with each other and if one assertion is known to be false, it cannot be deduced from known facts that the other assertion is true.', 'Even if history suggests that an event was caused by a certain set of circumstances, it would be a mistake to assume that the event will only reoccur under the same set of circumstance as it occurred under before.', 'A conjecture is undermined when certain events do not occur that would be expected to occur if the conjecture were true.']", "label": 3 }, { "id": "train_674", "context": "The Magno-Blanket is probably able to relieve arthritic pain in older dogs. A hospital study of people suffering from severe joint pain found that 76 percent of those who were treated with magnets reported reduced pain after just 3 weeks. Dogs and humans have similar physiologies and the Magno-Blanket brings magnets into the same proximity to the dog' s joints as they were to patients' joints in the hospital study.", "question": "Which one of the following, if true, most strengthens the argument?", "answers": "['The Magno-Blanket is likely to be effective on cats and other pets as well if it is effective at reducing joint pain in arthritic dogs.', 'The patients in the hospital study suffering from severe joint pain who, after being treated with magnets, did not report reduced pain tended not to be those suffering from the most severe pain.', 'Most of the patients in the hospital study suffering from severe joint pain who received a placebo rather than treatment with magnets did not report reduced pain.', \"Magnets have been shown to be capable of intensifying the transmission of messages from people's nerve cells to their brains.\"]", "label": 2 }, { "id": "train_675", "context": "Epidemiologist: Malaria passes into the human population when a mosquito carrying the malaria protozoa bites a human who has no immunity. The malaria parasite can remain for up to forty days in the blood of an infected person. The disease cannot be passed from person to person, unless a non-infected person is exposed to the blood of an infected person. Theoretically, malaria could be eradicated in any given area, if all the mosquitoes carrying malaria in that area are exterminated. If such a course of action is carried out at a worldwide level, then the global eradication of malaria is possible.", "question": "Which of the following, if true, suggests that the epidemiologist's plan for eliminating malaria is not viable?", "answers": "['A person who is infected with malaria can infect a mosquito that is not carrying malaria, if that mosquito bites such a person.', 'Malaria is still endemic in many parts of the world, and many health workers believe that the global eradication of malaria is not possible.', 'Some people in areas where malaria is rife have developed an immunity to mosquitos, yet they also show a higher incidence of genetic disorders such as sickle-cell anemia.', 'Unless a mosquito bites an infected person, and then bites a non-infected person, malaria cannot be passed directly from human to human.']", "label": 0 }, { "id": "train_676", "context": "John' s literature professor believes that the ability to judge the greatness of literary works accurately can be acquired only after years of specialized training. Such training is, in fact, what is required to become a literature professor. She is also well aware that the vast majority of the reading public does not have access to this specialized training.", "question": "Which one of the following statements must be true if what John's literature professor believes is true?", "answers": "['Literature professors do not belong to the reading public.', \"John's literature professor can judge the greatness of works of literature accurately.\", 'The vast majority of the reading public is unable to judge the greatness of works of literature accurately.', 'Anyone who is not a literature professor cannot judge the greatness of works of literature accurately.']", "label": 2 }, { "id": "train_677", "context": "Given the shape of the hip and foot bones of the Kodiak bear, it has been determined that standing and walking upright is completely natural behavior for these bears. Thus, walking on hind legs is instinctive and not a learned behavior of the Kodiak.", "question": "To which one of the following criticisms is the argument most vulnerable?", "answers": "['The argument presumes, without giving justification, that all behavior can be explained in one or both of only two ways.', 'The argument incorrectly appeals to the authority of science in order to support its conclusion.', 'The argument fails to consider the possibility that walking on hind legs is the result of both learning and an innate capacity.', 'The word \"behavior\" illicitly changes meaning during the course of the argument.']", "label": 2 }, { "id": "train_678", "context": "When a nation is on the brink of financial crisis, its government does not violate free-market principles if, in order to prevent economic collapse, it limits the extent to which foreign investors and lenders can withdraw their money. After all, the right to free speech does not include the right to shout \"Fire! \" in a crowded theatre, and the harm done as investors and lenders rush madly to get their money out before everyone else does can be just as real as the harm resulting from a stampede in a theatre.", "question": "The argument does which one of the following?", "answers": "['tries to show that a set of principles is limited in a specific way by using an analogy to a similar principle that is limited in a similar way', 'presents numerous experimental results as evidence for a general principle', 'applies an empirical generalization to reach a conclusion about a particular case', 'attempts to demonstrate that an explanation of a phenomenon is flawed by showing that it fails to explain a particular instance of that phenomenon']", "label": 0 }, { "id": "train_679", "context": "A plausible explanation of the disappearance of the dinosaurs is what is known as the comet theory. A large enough comet colliding with Earth could have caused a cloud of dust that enshrouded the planet and cooled the climate long enough to result in the dinosaurs' demise.", "question": "Which one of the following statements, if true, most seriously weakens the argument?", "answers": "['Many other animal species from the era of the dinosaurs did not become extinct at the same time the dinosaurs did.', 'It cannot be determined from a study of dinosaur skeletons whether the animals died from the effects of a dust cloud.', 'The consequences for vegetation and animals of a comet colliding with Earth are not fully understood.', 'Various species of animals from the same era as the dinosaurs and similar to them in physiology and habitat did not become extinct when the dinosaurs did.']", "label": 3 }, { "id": "train_680", "context": "If the magazine' s circulation continues to rise as it has over the last ten years, in another ten years it will be the largest-selling martial arts magazine in the world. Unfortunately, it has now become clear that the magazine' s publisher will not allow the managing editor to make the changes she has proposed, and without these changes, the magazine' s circulation will not rise as quickly over the next ten years as it has over the last ten. So the magazine will not be the largest-selling martial arts magazine ten years from now.", "question": "The argument's reasoning is flawed because the argument", "answers": "[\"equates a reduction in the rate at which the magazine's circulation is increasing with a decline in the magazine's circulation\", 'treats an occurrence that will ensure a certain outcome as something that is required for that outcome', \"identifies some changes required for the magazine's circulation to continue its rapid increase and concludes from this that no other changes are needed\", 'draws a conclusion that simply restates a claim that is presented in support of that conclusion']", "label": 1 }, { "id": "train_681", "context": "During a press conference, Council Member Johns stated that the advisory council appointed by Mayor Smith has been one of the least effective in recent history. When asked to support that claim, Johns pointed out that most citizens could not name any member of the advisory council.", "question": "The rationale given by Council Member Johns depends on the assumption that", "answers": "['A member of the advisory council who is doing a bad job is just as likely to be familiar to the public as a member of the advisory council who is doing a good job.', \"Only council members are qualified to judge the quality of the advisory council's work.\", \"The public's familiarity with the composition of the advisory council is an indication of the effectiveness of the advisory council's work.\", 'The members of the advisory council were selected because each of them was already well known to the citizenry.']", "label": 2 }, { "id": "train_682", "context": "Automobile emissions are a significant source of air pollutants, and cars over five years old typically generate significantly greater amounts of pollutants than newer cars. In Torinia, which has recently built its first automobile manufacturing plant, most cars are over five years old. Aiming to boost Torinia' s economy and reduce air pollution, the government plans to introduce incentives for Torinians to scrap their old cars every five years and replace them with new ones.", "question": "Which of the following, if true, most seriously undermines the likelihood that the planned incentives, if implemented, will achieve both of the cited aims?", "answers": "['Without the implementation of the planned incentives, most Torinians who own an old car would be unlikely to buy a new car.', 'The manufacture and the scrapping of cars each generate significant amounts of air pollutants.', 'The largest source of atmospheric pollutants in Torinia is not automobile emissions, but emissions from power plants.', \"Torinia's automobile plant manufactures car models that typically generate smaller amounts of air pollutants than most similarly sized car models manufactured elsewhere.\"]", "label": 1 }, { "id": "train_683", "context": "Conservative Politician: Social welfare programs are destroying our country. These programs are not only adding to the annual deficit, which increases the national debt, but they also discourage hard work. Our country must continue producing leaders who bootstrap their way to the top. None of our country' s citizens truly need assistance from the government; rather, the assistance just makes things easier. Liberal Politician: Our great country is founded on the principle of hope. The country is built on the backs of immigrants who came here with nothing, except for the hope of a better life. Our country is too wealthy not to provide basic necessities for the less fortunate. Recent immigrants, single mothers, historically disenfranchised, disabled persons, and the elderly all require an ample safety net.", "question": "What is the main point of dispute between the politicians?", "answers": "[\"All of the country 's leaders have bootstrapped their way to the top.\", 'Certain classes of people rely on social welfare programs to meet their basic needs.', 'Certain classes of people would be irreparably harmed if the country failed to provide a social welfare program.', 'Spending on social welfare programs increases the national debt.']", "label": 2 }, { "id": "train_684", "context": "Henry: Some scientists explain the dance of honeybees as the means by which honeybees communicate the location of whatever food source they have just visited to other members of the hive. But honeybees do not need so complicated a mechanism to communicate that information. Forager honeybees returning to their hive simply leave a scent trail from the food source they have just visited. There must ttherefore be some other explanation for the honeybees' dance. Winifred: Not necessarily. Most animals have several ways of accomplishing critical tasks. Bees of some species can navigate using either the position of the Sun or the memory of landmarks. Similarly, for honeybees, scent trails are a supplementary not an exclusive means of communicating.", "question": "The point at issue between Henry and Winifred is whether", "answers": "['there is more than one valid explanation for the dance of the honeybees', 'the honeybee is the only species of bee that is capable of communicating navigational information to other hive members', 'honeybees communicate the location of food sources through their dance', 'theories of animal behavior can be established on the basis of evidence about only one species of animal']", "label": 2 }, { "id": "train_685", "context": "In some places, iceberg lilies are the mainstay of grizzly bears' summer diets. The bears forage meadows for the lilies, uprooting them and eating their bulbs. Although the bears annually destroy a large percentage of the lilies, scientists have determined that the bears' feeding habits actually promote the survival of iceberg lilies.", "question": "Which one of the following, if true, most helps to resolve the apparent discrepancy in the statements above?", "answers": "['When grizzly bears forage for iceberg lilies, they generally kill many more lilies than they eat.', 'The geographic regions in which iceberg lilies are most prevalent are those regions populated by grizzly bears.', \"Iceberg lilies contain plentiful amounts of some nutrients that are necessary for grizzly bears' survival.\", 'Iceberg lilies produce so many offspring that, when undisturbed, they quickly deplete the resources necessary for their own survival.']", "label": 3 }, { "id": "train_686", "context": "The use of growth-promoting antibiotics in hog farming can weaken their effectiveness in treating humans because such use can spread resistance to those antibiotics among microorganisms. But now the Smee Company, one of the largest pork marketers, may stop buying pork raised on feed containing these antibiotics. Smee has 60 percent of the pork market, and farmers who sell to Smee would certainly stop using antibiotics in order to avoid jeopardizing their sales. So if Smee makes this change, it will probably significantly slow the decline in antibiotics' effectiveness for humans.", "question": "Which of the following, if true, would most strengthen the argument above?", "answers": "['A phaseout of use of antibiotics for hogs in one country reduced usage by over 50 percent over five years.', 'Other major pork marketers will probably stop buying pork raised on feed containing growth-promoting antibiotics if Smee no longer buys such pork.', 'Authorities are promoting the use of antibiotics to which microorganisms have not yet developed resistance.', 'The decline in hog growth due to discontinuation of antibiotics can be offset by improved hygiene.']", "label": 1 }, { "id": "train_687", "context": "Carl is clearly an incompetent detective. He has solved a smaller percentage of the cases assigned to him in the last 3 years -- only 1 out of 25 -- than any other detective on the police force.", "question": "Which one of the following, if true, most seriously weakens the argument above?", "answers": "['Before he became a detective, Carl was a neighborhood police officer and was highly respected by the residents of the neighborhood he patrolled.', 'Because the police chief regards Carl as the most capable detective, she assigns him only the most difficult cases, ones that others have failed to solve.', 'Carl was previously a detective in a police department in another city, and in the 4 years he spent there, he solved only 1 out of 30 crimes.', 'Many of the officers in the police department in which Carl serves were hired or promoted within the last 5 years.']", "label": 1 }, { "id": "train_688", "context": "There is little point in looking to artists for insights into political issues. Most of them hold political views that are less insightful than those of any reasonably well-educated person who is not an artist. Indeed, when taken as a whole, the statements made by artists, including those considered to be great, indicate that artistic talent and political insight are rarely found together.", "question": "Which one of the following can be inferred from the passage?", "answers": "['Every reasonably well-educated person who is not an artist has more insight into political issues than any artist.', 'Politicians rarely have any artistic talent.', 'Some artists are no less politically insightful than some reasonably well-educated persons who are not artists.', 'A thorough education in art makes a person reasonably well educated.']", "label": 2 }, { "id": "train_689", "context": "The stated goal of the government' s funding program for the arts is to encourage the creation of works of artistic excellence. Senator Beton claims, however, that a government-funded artwork can never reflect the independent artistic conscience of the artist because artists, like anyone else who accepts financial support, will inevitably try to please those who control the distribution of that support. Senator Beton concludes that government funding of the arts not only is a burden on taxpayers but also cannot lead to the creation of works of true artistic excellence.", "question": "Which one of the following is an assumption on which Senator Beton's argument is based?", "answers": "['Once an artist has produced works of true artistic excellence, he or she will never accept government funding.', 'Most taxpayers have little or no interest in the creation of works of true artistic excellence.', 'A contemporary work of art that does not reflect the independent artistic conscience of the artist cannot be a work of true artistic excellence.', 'Distribution of government funds for the arts is based on a broad agreement as to what constitutes artistic excellence.']", "label": 2 }, { "id": "train_690", "context": "Tissue biopsies taken on patients who have undergone throat surgery show that those who snored frequently were significantly more likely to have serious abnormalities in their throat muscles than those who snored rarely or not at all. This shows that snoring can damage the throat of the snorer.", "question": "Which one of the following, if true, most strengthens the argument?", "answers": "['People who have undergone throat surgery are no more likely to snore than people who have not undergone throat surgery.', 'The abnormalities in the throat muscles discovered in the study do not cause snoring.', \"The patients' throat surgery was not undertaken to treat abnormalities in their throat muscles.\", \"The study relied on the subjects' self-reporting to determine whether or not they snored frequently.\"]", "label": 1 }, { "id": "train_691", "context": "A two-year study beginning in 1977 found that, among 85-year-old people, those whose immune systems were weakest were twice as likely to die within two years as others in the study. The cause of their deaths, however, was more often heart disease, against which the immune system does not protect, than cancer or infections, which are attacked by the immune system.", "question": "Which of the following, if true, would offer the best prospects for explaining deaths in which weakness of the immune system, though present, played no causal role?", "answers": "['Some of the drugs that had been used to treat the symptoms of heart disease had a side effect of weakening the immune system.', 'Those in the study who survived into the 1980s had, in 1976, strengthened their immune systems through drug therapy.', 'Most of those in the study who survived beyond the two-year period had recovered from a serious infection sometime prior to 1978.', 'There were twice as many infections among those in the study with the weakest immune systems as among those with the strongest immune systems.']", "label": 0 }, { "id": "train_692", "context": "The human brain and its associated mental capacities evolved to assist self-preservation. Thus, the capacity of make aesthetic judgments is an adaptation to past environments in which humans lived. So an individual' s aesthetic judgments must be evaluated in terms of the extent to which they promote the survival of that individual.", "question": "Which one of the following is a principle that would, if valid, provide the strongest justification for the reasoning above?", "answers": "['All human adaptations to past environments were based on the human brain and its associated mental capacities.', 'Judgments that depend on individual preference or taste cannot be evaluated as true or false.', 'If something develops to serve a given function, the standard by which it must be judged is how well it serves that function.', 'Human capacities that do not contribute to the biological success of the human species cannot be evaluated.']", "label": 2 }, { "id": "train_693", "context": "When trying to identify new technologies that promise to transform the marketplace, market researchers survey the managers of those companies that are developing new technologies. Such managers have an enormous stake in succeeding, so they invariably overstate the potential of their new technologies. Surprisingly, however, market researchers typically do not survey a new technology' s potential buyers, even though it is the buyers-not the producers-who will ultimately determine a technology' s commercial success.", "question": "Which of the following, if true, best accounts for the typical survey practices among market researchers?", "answers": "['The developers of a new technology are generally no better positioned than its potential buyers to gauge how rapidly the new technology can be efficiently mass-produced.', 'The potential buyers for not-yet-available technologies can seldom be reliably identified.', 'People who promote the virtues of a new technology typically fail to consider that the old technology that is currently in use continues to be improved, often substantially.', 'Investors are unlikely to invest substantial amounts of capital in a company whose own managers are skeptical about the commercial prospects of a new technology they are developing.']", "label": 1 }, { "id": "train_694", "context": "James: Community colleges, by their very nature, work to meet the educational needs of the communities they are in. The same is not true of universities, whose primary goals differ from those of community colleges. Margaret: A primary goal of any university is to serve the needs of the community where it is located. The main reason people have for attending a university is the same as that for attending a community college: preparing oneself for a career.", "question": "James's and Margaret's statements provide the most support for the claim that they disagree over the truth of which one of the following?", "answers": "['The main reason people have for attending a university is to prepare themselves for a career.', 'A primary goal of any university is to serve the educational needs of its community.', 'In a typical community, the primary educational need is to prepare community residents for careers.', 'Most universities adequately serve the educational needs of the communities in which they are located.']", "label": 1 }, { "id": "train_695", "context": "Cultural anthropological theory tends to fall into two camps. One focuses on everyday social behavior as a system that has developed in response to human needs in a given environment. The other rejects this approach, focusing on the systems of meanings by which thoughts, rituals, and mythology in a society are structured. Cultural anthropologists, however, should employ both approaches, and also attend to a third, often neglected dimension: the view of a community as a set of individuals whose actions constitute the actual stuff of everyday life.", "question": "Which one of the following statements is most strongly supported by the information above?", "answers": "['Disagreement among cultural anthropological theorists does not necessarily imply that their approaches are incompatible.', 'Cultural anthropological theorists who focus on issues of meaning overlook the humanity of their individual subjects.', 'Cultural anthropologists too often rely on a conception of human needs that excludes the notion of community.', 'Systems of behavior can be understood only by experiencing the environments to which they respond.']", "label": 0 }, { "id": "train_696", "context": "On a short trip a driver is more likely to have an accident if there is a passenger in the car, presumably because passengers distract drivers. However, on a long trip a driver is more likely to have an accident if the driver is alone.", "question": "Which one of the following, if true, most helps to explain the facts described above?", "answers": "['Good drivers tend to take more long trips than bad drivers.', 'The longer a car trip is, the more likely a passenger is to help the driver maintain alertness.', 'Most drivers take far more short trips than long trips.', 'On a long trip the likelihood of an accident does not increase with each additional passenger.']", "label": 1 }, { "id": "train_697", "context": "Energy analyst: During this record-breaking heat wave, air conditioner use has overloaded the region' s electrical power grid, resulting in frequent power blackouts throughout the region. For this reason, residents have been asked to cut back voluntarily on air conditioner use in their homes. But even if this request is heeded, blackouts will probably occur unless the heat wave abates.", "question": "Which one of the following, if true, most helps to resolve the apparent discrepancy in the information above?", "answers": "['Most air-conditioning in the region is used to cool businesses and factories.', 'Most air-conditioning systems could be made more energy efficient by implementing simple design modifications.', 'The heat wave is expected to abate in the near future.', 'Air-conditioning is not the only significant drain on the electrical system in the area.']", "label": 0 }, { "id": "train_698", "context": "Roland: The alarming fact is that 90 percent of the people in this country now report that they know someone who is unemployed. Sharon: But a normal, moderate level of unemployment is 5 percent, with 1 out of 20 workers unemployed. So at any given time if a person knows approximately 50 workers, one or more will very likely be unemployed.", "question": "Sharon's argument is structured to lead to which of the following as a conclusion?", "answers": "['The current level of unemployment is not moderate.', 'The fact that 90% of the people know someone who is unemployed is not an indication that unemployment is abnormally high.', 'It is unlikely that the people whose statements Roland cites are giving accurate reports.', 'If an unemployment figure is given as a certain percent, the actual percentage of those without jobs is even higher.']", "label": 1 }, { "id": "train_699", "context": "Yorco and Zortech are two corporations that employ large numbers of full-time workers who are paid by the hour. Publicly available records indicate that Yorco employs roughly the same number of such hourly wage workers as Zortech does but spends a far higher total sum per year on wages for such workers. Ttherefore, hourly wages must be higher, on average, at Yorco than at Zortech, since __.", "question": "Which of the following most logically completes the argument?", "answers": "['overtime work, which is paid at a substantially higher rate than work done during the regular work week, is rare at both Yorco and Zortech', 'the proportion of all company employees who are hourly wage workers is significantly greater at Yorco than it is at Zortech', 'the work performed by hourly wage workers at Zortech does not require a significantly higher level of skill than the work performed by hourly wage workers at Yorco does', 'Zortech spends a higher total sum per year than Yorco does to provide its hourly wage workers with benefits other than wages']", "label": 0 }, { "id": "train_700", "context": "Economist: Some critics of the media have contended that negative news reports on the state of the economy can actually harm the economy because such reports damage people' s confidence in it, and this lack of confidence in turn adversely affects people' s willingness to spend money. But studies show that spending trends correlate very closely with people' s confidence in their own immediate economic situations. Thus these media critics are mistaken.", "question": "The economist's argument is flawed in that it fails to consider the possibility that", "answers": "['news reports about the state of the economy are not always accurate', 'people who have little confidence in the overall economy generally take a pessimistic view concerning their own immediate economic situations', 'an economic slowdown usually has a greater impact on the economic situations of individuals if it takes people by surprise than if people are forewarned', 'people who pay no attention to economic reports in the media always judge accurately whether their own economic situation is likely to deteriorate or improve']", "label": 1 }, { "id": "train_701", "context": "Attorneys for a criminal defendant charged that the government, in a cover-up, had destroyed evidence that would have supported the defendant in a case. The government replied that there is no evidence that would even tend to support the defendant in the case.", "question": "Which one of the following is the most accurate evaluation of the government' s reply?", "answers": "['If true, it effectively disproves the charge made on behalf of the defendant.', 'It leaves open the question of whether the government had destroyed such evidence.', 'It shows that the attorneys did not know whether their charge was true.', \"It demonstrates the government's failure to search for evidence in its files.\"]", "label": 1 }, { "id": "train_702", "context": "Administrators of educational institutions are enthusiastic about the educational use of computers because they believe that it will enable schools to teach far more courses with far fewer teachers than traditional methods allow. Many teachers fear computers for the same reason. But this reason is mistaken. Computerized instruction requires more, not less, time of instructors, which indicates that any reduction in the number of teachers would require an accompanying reduction in courses offered.", "question": "The statement that the educational use of computers enables schools to teach far more courses with far fewer teachers figures in the argument in which one of the following ways?", "answers": "[\"It is a statement offered in support of the argument's main conclusion.\", \"It is the argument's main conclusion.\", 'It is presented as a possible explanation for an observation that follows it.', 'It is a statement that the argument is designed to refute.']", "label": 3 }, { "id": "train_703", "context": "Before 1986 physicists believed they could describe the universe in terms of four universal forces. Experiments then suggested, however, a fifth universal force of mutual repulsion between particles of matter. This fifth force would explain the occurrence in the experiments of a smaller measurement of the gravitational attraction between bodies than the established theory predicted.", "question": "Which one of the following, if true, most strengthens the argument that there is a fifth universal force?", "answers": "['No previously established scientific results are incompatible with the notion of a fifth universal force.', 'The extremely sophisticated equipment used for the experiments was not available to physicists before the 1970s.', 'The experiments were conducted by physicists in remote geological settings in which factors affecting the force of gravity could not be measured with any degree of precision.', 'Some scientists have suggested that the alleged fifth universal force is an aspect of gravity rather than being fundamental in itself.']", "label": 0 }, { "id": "train_704", "context": "It has been argued that the immense size of Tyrannosaurus rex would have made it so slow that it could only have been a scavenger, not a hunter, since it would not have been able to chase down its prey. This, however, is an overly hasty inference. T. rex' s prey, if it was even larger than T rex, would probably have been slower than T. rex. The claim that T.", "question": "rex could only have been a scavenger, not a hunter, plays which one of the following roles in the argument?", "answers": "['It is offered as evidence that is necessary for drawing the conclusion advanced by the argument', 'It is a hypothesis that is claimed in the argument to be logically inconsistent with the conclusion advanced by the argument.', 'It is offered as evidence in support of a hypothesis that the argument concludes to be false.', 'It is a hypothesis that the argument attempts to undermine by calling into question the sufficiency of the evidence.']", "label": 3 }, { "id": "train_705", "context": "Modern science is built on the process of posing hypotheses and testing them against observations -- in essence, attempting to show that the hypotheses are incorrect. Nothing brings more recognition than overthrowing conventional wisdom. It is accordingly unsurprising that some scientists are skeptical of the widely accepted predictions of global warming. What is instead remarkable is that with hundreds of researchers striving to make breakthroughs in climatology, very few find evidence that global warming is unlikely.", "question": "The information above provides the most support for which one of the following statements?", "answers": "['Research in global warming is primarily driven by a desire for recognition in the scientific community.', 'Most researchers in climatology have substantial motive to find evidence that would discredit the global warming hypothesis.', 'There is evidence that conclusively shows that the global warming hypothesis is true.', 'Scientists who are skeptical about global warming have not offered any alternative hypotheses to explain climatological data.']", "label": 1 }, { "id": "train_706", "context": "Sasha: Handwriting analysis should be banned in court as evidence of a person' s character: handwriting analysts called as witnesses habitually exaggerate the reliability of their analyses. Gregory: You are right that the current use of handwriting analysis as evidence is problematic. But this problem exists only because there is no licensing board to set professional standards and thus deter irresponsible analysts from making exaggerated claims. When such a board is established, however, handwriting analysis by licensed practitioners will be a legitimate courtroom tool for character assessment.", "question": "Gregory does which one of the following in responding to Sasha's argument?", "answers": "['He defends a principle by restricting the class to which it is to be applied.', \"He shows that Sasha's argument itself manifests the undesirable characteristic that it condemns.\", \"He identifies a self-contradictory statement in Sasha's argument.\", 'He abstracts a general principle from specific evidence.']", "label": 0 }, { "id": "train_707", "context": "Historian: Anyone who thinks that the terrors of the ancient regime of Q were exclusively the work of fanatics is overlooking a basic truth: the regime was made up primarily of ordinary people enthusiastically seeking paradise. The regime executed many people in pursuit of its goal; but it later became clear that paradise, as they defined it, is unrealizable. So at least some of the ordinary people of Q were in fact murderers.", "question": "Which one of the following principles, if valid, provides the most support for the historian's argumentation?", "answers": "['The pursuit of paradise does not justify murder.', 'The pursuit of paradise justifies fanaticism.', 'Execution in pursuit of what is later found to be unattainable constitutes murder.', 'Enthusiasm in pursuit of what is eventually found to be unattainable constitutes fanaticism.']", "label": 2 }, { "id": "train_708", "context": "The average hourly wage of television assemblers in Vernland has long been significantly lower than that in neighboring Borodia. Since Borodia dropped all tariffs on Vernlandian televisions three years ago, the number of televisions sold annually in Borodia has not changed. However, recent statistics show a drop in the number of television assemblers in Borodia. Ttherefore, updated trade statistics will probably indicate that the number of televisions Borodia imports annually from Vernland has increased.", "question": "Which of the following is an assumption on which the argument depends?", "answers": "['Televisions assembled in Vernland have features that televisions assembled in Borodia do not have', 'The average number of hours it takes a Borodian television assembler to assemble a television has not decreased significantly during the past three years', 'The number of television assemblers in Vernland has increased by at least as much as the number of television assemblers in Borodia has decreased', 'The difference between the hourly wage of television assemblers in Vernland and the hourly wage of television assemblers in Borodia is likely to decrease in the next few years']", "label": 1 }, { "id": "train_709", "context": "Claim: Country X' s government lowered tariff barriers because doing so served the interests of powerful foreign companies. Principle: In order for a change to be explained by the advantage some person or group gained from it, it must be shown how the interests of the person or group played a role in bringing about the change.", "question": "Which one of the following, if true, can most logically serve as a premise for an argument that uses the principle to counter the claim?", "answers": "['Although foreign companies benefited when Country X lowered tariff barriers, there is no other evidence that these foreign companies induced the change.', \"In the period since tariff barriers were lowered, price competition among importers has severely limited importers' profits from selling foreign companies' products in Country X.\", \"It was impossible to predict how Country X's economic reforms, which included lowering tariff barriers, would affect the economy in the short term.\", \"Many of the foreign companies that benefited from Country X's lowering tariff barriers compete fiercely among themselves both in Country X and in other markets.\"]", "label": 0 }, { "id": "train_710", "context": "The use of radar detectors in commercial vehicles was banned because commercial truck and bus drivers were using these devices to drive faster than the posted speed limit without fear of arrest. Since drivers of noncommercial vehicles also use radar detectors and since speeding decreases safety for any vehicle, use of radar detectors should also be banned in noncommercial vehicles to increase safety.", "question": "Which of the following, if true, most strongly supports the argument above?", "answers": "['Traffic accidents involving a truck or bus generally pose a greater threat of injury or death than do other accidents.', 'Radar detectors are sometimes set off by devices other than the law enforcement devices that the radar detectors were built to detect.', 'In many places airplanes or helicopters are used rather than radar to locate vehicles traveling faster than the posted speed limit.', 'The ban on radar detectors in commercial vehicles has been effective in deterring them from speeding.']", "label": 3 }, { "id": "train_711", "context": "Environmental scientist: It is true that over the past ten years, there has been a sixfold increase in government funding for the preservation of wetlands, while the total area of wetlands needing such preservation has increased only twofold (although this area was already large ten years ago). Even when inflation is taken into account, the amount of funding now is at least three times what it was ten years ago. Nevertheless, the current amount of government funding for the preservation of wetlands is inadequate and should be augmented.", "question": "Which one of the following, if true, most helps to reconcile the environmental scientist's conclusion with the evidence cited above?", "answers": "['Over the past ten years, the salaries of scientists employed by the government to work on the preservation of wetlands have increased at a rate higher than the inflation rate.', 'The governmental agency responsible for administering wetland-preservation funds has been consistently mismanaged and run inefficiently over the past ten years.', 'Unlike today, funding for the preservation of wetlands was almost nonexistent ten years ago.', 'Research over the past ten years has enabled scientists today to identify wetlands in need of preservation well before the areas are at serious risk of destruction.']", "label": 2 }, { "id": "train_712", "context": "Social media websites rely on user engagement. Increasing the number of users and those users' activity means more advertising revenue. Most social media websites offer the service at no cost in order to attract more users, relying exclusively on advertising revenue to make a profit. The most popular articles shared on social media websites involve sensationalized stories of dubious value, including misleading titles and incorrect factual information. However, many users will stop using a social media website when the sensational stories become too overwhelming. As a result, social media companies would be best served by. . .", "question": "Which one of the following best completes the argument?", "answers": "['lying to advertisers about their user engagement.', 'surveying their users to determine what type of content they most prefer.', 'monitoring the ratio of sensationalized and factual articles.', 'prohibiting sensationalized articles.']", "label": 2 }, { "id": "train_713", "context": "Old rose varieties have less vibrant colors than modern rose varieties, but are superior in fragrance. This situation has arisen because the modern rose varieties were developed to meet customer demand and marketed mainly through catalogs: colors, but not fragrances, can be photographed.", "question": "Which one of the following, if true, most strengthens the support for the causal claim made above?", "answers": "['People in earlier times were interested in the fragrances of roses as well as their colors.', 'State-of-the-art photographic equipment makes possible unprecedentedly accurate color reproductions of roses.', 'Breeding roses to enhance one trait often has unanticipated consequences for other traits.', 'People in earlier times would have found the vibrant colors of modern roses too harsh.']", "label": 2 }, { "id": "train_714", "context": "Last year the rate of inflation was 1. 2 percent, but during the current year it has been 4 percent. We can conclude that inflation is on an upward trend and the rate will be still higher next year.", "question": "Which of the following, if true, most seriously weakens the conclusion above?", "answers": "['The 1. 2 percent rate of inflation last year represented a 10-year low.', 'The inflation figures were computed on the basis of a representative sample of economic data rather than all the available data.', 'Increases in the pay of some workers are tied to the level of inflation, and at an inflation rate of 4 percent or above, these pay raises constitute a force causing further inflation.', 'Last year a dip in oil prices brought inflation temporarily below its recent stable annual level of 4 percent.']", "label": 3 }, { "id": "train_715", "context": "Nuts are high in calories. All else being equal, the more calories one consumes, the more likely one is to become overweight. However, studies have found that people who eat nuts regularly are actually less likely to be overweight than people who never eat nuts.", "question": "Which one of the following, if true, would most help to resolve the apparent discrepancy described above?", "answers": "['If people who avoid a certain kind of food are more likely to be overweight than people who consume that food regularly, then that food should not be avoided for weight loss purposes .', 'The likelihood of becoming overweight depends not only on how many calories one consumes but also on how rapidly one burns those calories through physical activity.', 'On average, people who never eat nuts consume the same total calories per day as people who regularly eat nuts.', 'Most people who regularly eat nuts eat correspondingly less of foods whose taste, unlike that of nuts, stimulates a hunger response.']", "label": 3 }, { "id": "train_716", "context": "Ethicist: Only when we know a lot about the events that led to an action are we justified in praising or blaming a person for that action -- as we sometimes are. We must ttherefore reject Tolstoy' s rash claim that if we knew a lot about the events leading up to any action, we would cease to regard that action as freely performed.", "question": "Which one of the following, if assumed, enables the conclusion of the ethicist's argument to be properly drawn?", "answers": "['People should not be regarded as subject to praise or blame for actions that were caused by conditions beyond their control.', 'The responsibility a person bears for an action is not a matter of degree; however, our inclination to blame or praise whoever performed the action varies with the amount of information available.', 'If we do not know much about the events leading up to any given action, we will regard that action as freely performed.', 'We can be justified in praising or blaming a person for an action only when we regard that action as freely performed.']", "label": 3 }, { "id": "train_717", "context": "Conservationist: The risk to airplane passengers from collisions between airplanes using the airport and birds from the wildlife refuge is negligible. In the 10 years since the refuge was established, only 20 planes have been damaged in collisions with birds, and no passenger has been injured as a result of such a collision. The wildlife refuge ttherefore poses no safety risk. Pilot: You neglect to mention that 17 of those 20 collisions occurred within the past 2 years, and that the number of birds in the refuge is rapidly increasing. As the number of collisions between birds and airplanes increases, so does the likelihood that at least one such collision will result in passenger injuries.", "question": "The pilot counters the conservationist by", "answers": "['disputing the accuracy of the figures cited by the conservationist', \"discrediting the moral principle on which the conservationist's argument is based\", \"attempting to show that the conservationist's description of the facts is misleading\", 'asserting that dangerous situations inevitably become more dangerous with the passage of time']", "label": 2 }, { "id": "train_718", "context": "Sydonie: Parents differ in their beliefs about the rules to which their children should be subject. So any disciplinary structure in schools is bound to create resentment because it will contradict some parental approaches to raising children. Stephanie: Your conclusion is incorrect; educational research shows that when parents list the things that they most want their children' s schools to provide, good discipline is always high on the list.", "question": "Stephanie's argument is most vulnerable to criticism on the grounds that", "answers": "[\"it addresses a more general issue than that addressed in Sydonie's argument\", \"it does not counter Sydonie's suggestion that parents have diverse ideas of what constitutes good discipline\", 'the phrase \"high on the list\" is not specific enough to give useful information about what parents desire from a school', \"it fails to discuss educators' attitudes toward discipline in schools\"]", "label": 1 }, { "id": "train_719", "context": "Aristophanes' play The Clouds, which was written when the philosopher Socrates was in his mid-forties, portrays Socrates as an atheistic philosopher primarily concerned with issues in natural science. The only other surviving portrayals of Socrates were written after Socrates' death at age 70. They portrayed Socrates as having a religious dimension and a strong focus on ethical issues.", "question": "Which one of the following, if true, would most help to resolve the apparent discrepancy between Aristophanes' portrayal of Socrates and the other surviving portrayals?", "answers": "['Socrates had an influence on many subsequent philosophers who were primarily concerned with natural science.', 'Socrates was a much more controversial figure in the years before his death than he was in his mid-forties.', \"Socrates' philosophical views and interests changed sometime after his mid-forties.\", \"Aristophanes' portrayal of Socrates in The Clouds was unflattering, whereas the other portrayals were very flattering.\"]", "label": 2 }, { "id": "train_720", "context": "Roger Bacon, the thirteenth-century scientist, is said to have made important discoveries in optics. He was an early advocate of hands-on experimentation, and as a teacher warned his students against relying uncritically on the opinions of authorities. Nevertheless, this did not stop Bacon himself from appealing to authority when it was expedient for his own argumentation. Thus, Bacon' s work on optics should be generally disregarded, in view of the contradiction between his statements and his own behavior.", "question": "The reasoning in the argument is flawed because the argument", "answers": "[\"criticizes Bacon's character in order to question his scientific findings\", \"uses Bacon's remarks to his students as evidence of his opinions\", 'ignores the fact that thirteenth-century science may not hold up well today', \"attacks Bacon's uncritical reliance on authority opinion\"]", "label": 0 }, { "id": "train_721", "context": "Despite increasing international efforts to protect the natural habitats of endangered species of animals, the rate at which these species are becoming extinct continues to rise. It is clear that these efforts are wasted.", "question": "Which one of the following, if true, most weakens the argument?", "answers": "['Many countries do not recognize the increased economic benefit of tourism associated with preserved natural habitats.', 'Species that would have become extinct have been saved due to the establishment of animal refuges.', 'Scientists are better able to preserve the habitats of endangered species now than ever before.', 'Scientists estimate that at least 2000 species become extinct every year.']", "label": 1 }, { "id": "train_722", "context": "The United States Food and Drug Administration (FDA) regulates the introduction of new therapeutic agents into the marketplace. Consequently, it plays a critical role in improving health care in the United States. While it is those in the academic and government research communities who engage in the long process of initial discovery and clinical testing of new therapeutic agents, it is the FDA' s role and responsibility to facilitate the transfer of new discoveries from the laboratory to the marketplace. Only after the transfer can important new therapies help patients.", "question": "Which one of the following statements can be inferred from the passage?", "answers": "['Before new therapeutic agents reach the marketplace they do not help patients.', 'The FDA should work more closely with researchers to ensure that the quality of therapeutic agents is maintained.', 'If a new medical discovery has been transferred from the laboratory to the marketplace, it will help patients.', 'The research community is responsible for the excessively long testing period for new drugs, not the FDA.']", "label": 0 }, { "id": "train_723", "context": "A group of 5, 000 undergraduate students was randomly selected from three universities in a large state and asked the question, \"Do you plan to attain your master' s degree eventually? \" More than 82 percent answered \"Yes. \" This shows that the overwhelming majority of students want to attain their master' s degree, and that if the national number of students earning a master' s degree is low, it cannot be due to a lack of desire on the part of the students.", "question": "The reasoning of the argument above is questionable because it", "answers": "[\"contradicts itself by admitting that there might be a low number of students earning master's degrees, while claiming that most students want to acquire such a degree\", \"overlooks the possibility that there might, in fact, be a majority of students who will eventually earn a master's degree\", 'treats university students from a particular state as if they are representative of university students nationwide', 'fails to justify its presumption that 82 percent is an overwhelming majority']", "label": 2 }, { "id": "train_724", "context": "Nutritionist: Your company's fruit-processing technologies add unacceptable amounts of copper to the orange juice you sell. Because copper blocks the absorption of Vitamin C, your advertising campaign claiming that your juice is a good source of Vitamin C is faulty and should be removed. Juice Manufacturer: Some amount of copper is necessary for optimal health. Recent studies have shown that as many as 25 percent of Americans do not get enough copper in their diets; ttherefore, the benefits of the copper that our process adds to the juice outweigh the costs of any Vitamin C that it may block.", "question": "The juice manufacturer's response is flawed as a refutation of the nutritionist's argument because it", "answers": "['does not address the issue of whether sufficient amounts of copper are present to invalidate its advertising claims.', \"shows that the nutritionist's evidence about copper is irrelevant but fails to demonstrate any flaws in the nutritionist's assumptions.\", \"addresses the nutritionist's argument in general terms, rather than in terms of the health of individuals.\", 'relies on the unfounded assumption that copper may be as good for health as Vitamin C.']", "label": 0 }, { "id": "train_725", "context": "Principle: When none of the fully qualified candidates for a new position at Arvue Corporation currently works for that company, it should hire the candidate who would be most productive in that position. Application: Arvue should not hire Krall for the new position, because Delacruz is a candidate and is fully qualified.", "question": "Which one of the following, if true, justifies the above application of the principle?", "answers": "['None of the candidates already works for Arvue, and Delacruz is the candidate who would be most productive in the new position.', 'Of all the candidates who do not already work for Arvue, Delacruz would be the most productive in the new position.', 'Krall works for Arvue, but Delacruz is the candidate who would be most productive in the new position.', 'Several candidates currently work for Arvue, but Krall and Delacruz do not.']", "label": 0 }, { "id": "train_726", "context": "Start-up companies financed by venture capitalists have a much lower failure rate than companies financed by other means. Source of financing, ttherefore, must be a more important causative factor in the success of a start-up company than are such factors as the personal characteristics of the entrepreneur, the quality of strategic planning, or the management structure of the company.", "question": "Which of the following, if true, most seriously weakens the argument above?", "answers": "['The management structures of start-up companies are generally less formal than the management structures of ongoing businesses.', 'More than half of all new companies fail within five years.', 'Venture capitalists base their decisions to fund start. up companies on such factors as the characteristics of the entrepreneur and quality of strategic planning of the company.', 'The strategic planning of a start-up company is a less important factor in the long-term success of the company than are the personal characteristics of the entrepreneur.']", "label": 2 }, { "id": "train_727", "context": "According to a survey of consumers conducted one week before the end of a national call-in campaign to decide the newest flavor of Freak Cola, more of those surveyed responded that they enjoyed the Citrusea Swirl flavor than responded that they enjoyed any other flavor choice. Regardless of the survey results, a different flavor, Vanilla Bonanza, was the national favorite by a considerable percentage.", "question": "Each of the following, if true, contributes to a resolution of the discrepancy described above EXCEPT:", "answers": "['More than 60% of those responding to the survey in favor of Vanilla Bonanza stated they were likely to call in and vote, whereas only 25% of those supporting Citrusea Swirl claimed they would be calling in their votes.', 'Freak Cola made an announcement four days before the end of the call-in campaign stating that the third option, VeriBlueBeri, was being removed from consideration as the new flavor choice.', 'A smaller percentage of those favoring the Vanilla Bonanza flavor knew the call-in number than did those favoring Citrusea Swirl.', 'The entire survey was developed and conducted by members of the Freak Cola design team that created Citrusea Swirl.']", "label": 2 }, { "id": "train_728", "context": "Due to the profitable but unlawful practice of selling elephant tusks, elephants have been hunted to near extinction by unlicensed poachers. Ttherefore, an efficient way to prevent the hunting of elephants would be to file down the tusks from time to time, thus removing any incentive for hunting elephants.", "question": "Which of the following is an assumption required by the argument?", "answers": "['Elephants are only valued by poachers for their tusks.', 'Poachers often steal baby elephants to raise in captivity and later sell them for a profit.', 'Elephants whose tusks have been trimmed will be unable to defend themselves against poachers.', 'The demand for elephant tusks will decrease as supply dwindles.']", "label": 0 }, { "id": "train_729", "context": "The mathematics of the scientific theory known as \"complexity\" describes those phenomena that are not quite stable and not quite chaotic. For example, the mathematics of complexity can be used to describe sand dunes: although sand dunes generally retain their shape, the addition of a tiny amount of sand can cause a sandslide. Certain scientists have concluded that there is good evidence that complexity is correct. After entering complexity-based mathematical models of real-world phenomena in computers, they found that the computerized mathematical models evolve much like the real-world phenomena actually evolve.", "question": "Which one of the following principles, if valid, most justifies the scientists' conclusion?", "answers": "['If actual phenomena can be pictured as computerized models, computers themselves will eventually discover the mathematics of the correct theory explaining those phenomena.', 'If computers verify that there are mathematical errors in the calculations of scientists, then the theories of those scientists are probably incorrect.', 'If computerized models based on a theory behave like their real-world counterparts behave, then that theory is probably correct.', 'If they evolve exactly like real-world phenomena, computer models are neither purely stable nor purely chaotic, which is what complexity predicts.']", "label": 2 }, { "id": "train_730", "context": "Even in a democracy, it is necessary to restrict the dissemination of advanced technological knowledge that is of commercial or national-security value. Dissemination to certain countries, those that are or will be competitors or enemies, should be selectively prohibited. There must, however, be free exchange of scientific information.", "question": "In order to act in accordance with the position above, it would be necessary to be able to rely on each of the following EXCEPT:", "answers": "[\"In a democracy, it is not necessary that the public have detailed knowledge of the country's advanced technology in order, for example, to make informed decisions about the direction public policy should take.\", 'It is possible to distinguish with confidence, despite any changes in the international environment, friendly or noncompetitive from hostile or competitive nations.', 'In cases where a company that uses advanced technology is a multinational organization, it is possible to keep information about the technology from being passed across designated national boundaries.', 'In most fields of science, basic scientific research is further advanced in countries that are democracies than in countries that are not democracies.']", "label": 3 }, { "id": "train_731", "context": "A museum has been offered an undocumented statue, supposedly Greek and from the sixth century B. C. Possibly the statue is genuine but undocumented because it was recently unearthed or because it has been privately owned. However, an ancient surface usually has uneven weathering, whereas the surface of this statue has the uniform quality characteristically produced by a chemical bath used by forgers to imitate a weathered surface. Ttherefore, the statue is probably a forgery.", "question": "Which of the following, if true, most seriously weakens the argument?", "answers": "['The chemical bath that forgers use was at one time used by dealers and collectors to remove the splotchy surface appearance of genuinely ancient sculptures.', 'An allegedly Roman sculpture with a uniform surface Similar to that of the statue being offered to the museum was recently shown to be a forgery.', 'Museum officials believe that forgers have no technique that can convincingly simulate the patchy weathering characteristic of the surfaces of ancient sculptures.', \"The subject's pose and other aspects of the subject's treatment exhibit all the most common features of Greek statues of the sixth century B. C.\"]", "label": 0 }, { "id": "train_732", "context": "Programmer: We computer programmers at Mytheco are demanding raises to make our average salary comparable with that of the technical writers here who receive, on average, 20 percent more in salary and benefits than we do. This pay difference is unfair and intolerable. Mytheco executive: But many of the technical writers have worked for Mytheco longer than have many of the programmers. Since salary and benefits at Mytheco are directly tied to seniority, the 20 percent pay difference you mention is perfectly acceptable.", "question": "Evaluating the adequacy of the Mytheco executive's response requires a clarification of which one of the following?", "answers": "[\"how the Mytheco executive's salary compares with that of the programmers\", 'how the average seniority of programmers compares with the average seniority of technical writers', 'whether any of the technical writers at Mytheco once worked as programmers at the company', 'whether the sorts of benefits an employee of Mytheco receives are tied to the salary of that employee']", "label": 1 }, { "id": "train_733", "context": "Large inequalities in wealth always threaten the viability of true democracy, since wealth is the basis of political power, and true democracy depends on the equal distribution of political power among all citizens.", "question": "The reasoning in which one of the following arguments most closely parallels the reasoning in the argument above?", "answers": "['Honesty is important in maintaining friendships. But sometimes honesty can lead to arguments, so it is difficult to predict the effect a particular honest act will have on a friendship.', \"Repeated encroachments on one's leisure time by a demanding job interfere with the requirements of good health. The reason is that good health depends on regular moderate exercise, but adequate leisure time is essential to regular exercise.\", 'If Sara went to the bookstore every time her pocket was full, Sara would never have enough money to cover her living expenses, since books are her love and they are getting very expensive.', 'Consumer culture and an emphasis on technological innovation are a dangerous combination, since together they are uncontrollable and lead to irrational excess.']", "label": 1 }, { "id": "train_734", "context": "The makers of Activite, a natural dietary supplement, claim that it promotes energy and mental alertness. To back up their claim, they offer a month' s supply of Activite free to new customers. Clearly, Activite must be effective, since otherwise it would not be in the company' s interest to make such an offer.", "question": "Which one of the following, if true, most weakens the argument?", "answers": "['The makers of Activite charge a handling fee that is considerably more than what it costs them to pack and ship their product.', 'The mere fact that a dietary supplement contains only natural ingredients does not insure that it has no harmful side effects.', 'The nutrients in Activite can all be obtained from a sufficiently varied and well-balanced diet.', 'There are less expensive dietary supplements on the market that are just as effective as Activite.']", "label": 0 }, { "id": "train_735", "context": "The gu, the hu, and the jue are types of bronze libation vessels that were produced in China during the Shang dynasty, almost 4, 000 years ago. Close examination of authentic gu, hu, and jue vessels reveals that they all bear incised patterns symbolizing the taotie, a mythological beast of greed. It must be true then that any bronze libation vessel that does not bear incised patterns symbolizing the taotie is not an authentic vessel produced in China during the Shang dynasty.", "question": "The argument makes which one of the following errors of reasoning?", "answers": "['treating the fact that some members of a category possess a certain characteristic as sufficient evidence that possession of the characteristic is necessary for membership in that category', 'basing a generalization on examples that the argument itself admits are atypical', 'treating the fact that some members of a given category possess a certain characteristic as sufficient evidence that other objects that possess the characteristic are also members of that category', 'treating the facts that certain specific objects belong to a given category and that some other objects belonging to that category possess a certain characteristic as sufficient evidence that the former objects also possess that characteristic']", "label": 0 }, { "id": "train_736", "context": "Classicist: In the ancient Athenian democracy, unlike in any other political system in world history, the legislature consisted of all eligible voters. Athenian voters, not elected representatives, made all political decisions by direct vote after public debate in the Assembly. Thus, if the world has ever known true democracy, it was only in ancient Athens.", "question": "The conclusion of the classicist's argument follows logically if which one of the following is assumed?", "answers": "['Athens was the only city in ancient Greece that had a democratic political system.', 'Most political systems in world history have not been democracies.', 'Public debate is an essential part of any democratic decision-making process.', 'A political system is not a true democracy unless the eligible voters themselves vote directly on all political decisions.']", "label": 3 }, { "id": "train_737", "context": "Famous personalities found guilty of many types of crimes in well-publicized trials are increasingly sentenced to the performance of community service, though unknown defendants convicted of similar crimes almost always serve prison sentences. However, the principle of equality before the law rules out using fame and publicity as relevant considerations in the sentencing of convicted criminals.", "question": "The statements above, if true, most strongly support which one of the following conclusions?", "answers": "['The principle of equality before the law can properly be overridden by other principles in some cases.', 'The sentencing of celebrities to community service instead of prison constitutes a violation of the principle of equality before the law in many cases.', 'The principle of equality before the law is rigorously applied in only a few types of criminal trials.', 'The number of convicted celebrities sentenced to community service should equal the number of convicted unknown defendants sentenced to community service.']", "label": 1 }, { "id": "train_738", "context": "Like a genetic profile, a functional magnetic-resonance image (fMRI) of the brain can contain information that a patient wishes to keep private. An fMRI of a brain also contains enough information about a patient' s skull to create a recognizable image of that patient' s face. A genetic profile can be linked to a patient only by referring to labels or records.", "question": "The statements above, if true, most strongly support which one of the following?", "answers": "['Patients are more concerned about threats to privacy posed by fMRIs than they are about those posed by genetic profiles.', 'In most cases patients cannot be reasonably sure that the information in a genetic profile will be kept private.', \"It is not important that medical providers apply labels to fMRIs of patients' brains.\", 'An fMRI has the potential to compromise patient privacy in circumstances in which a genetic profile would not.']", "label": 3 }, { "id": "train_739", "context": "Anyone who knows Ellsworth knows that he is bursting with self-righteousness, touting the idealism of his generation over the greed of the previous generation. So no one who knows him will be surprised that Ellsworth is offended by the suggestions in the media that he has engaged in unethical business practices.", "question": "The conclusion drawn above follows logically if which one of the following is assumed?", "answers": "['Hypocrites often hide behind righteous indignation.', 'Everyone suspects self-righteous people of being, in actuality, unethical.', 'Ellsworth is in fact innocent of all wrongdoing.', 'Everyone expects self-righteous people to be easily offended.']", "label": 3 }, { "id": "train_740", "context": "Columnist: Taking a strong position on an issue makes one likely to misinterpret or ignore additional evidence that conflicts with one' s stand. But in order to understand an issue fully, it is essential to consider such evidence impartially. Thus, it is best not to take a strong position on an issue unless one has already considered all important evidence conflicting with that position.", "question": "The columnist's reasoning most closely conforms to which one of the following principles?", "answers": "['It is reasonable to take a strong position on an issue if one fully understands the issue and has considered the evidence regarding that issue impartially.', 'One should try to understand an issue fully if doing so will help one to avoid misinterpreting or ignoring evidence regarding that issue.', 'To ensure that one has impartially considered the evidence regarding an issue on which one has taken a strong position, one should avoid misinterpreting or ignoring evidence regarding that issue.', 'Anyone who does not understand an issue fully should avoid taking a strong position on it.']", "label": 3 }, { "id": "train_741", "context": "The result of flipping an evenly weighted, or \"fair, \" coin, a process commonly thought to be random, is, in fact, well determined by the impulse given the coin and by the height above the floor from which the coin starts. Yet it is difficult to predict the result of a fair coin flip.", "question": "Which of the following, if true, contributes most to an explanation of why the outcome of a coin flip is difficult to predict even though it is well determined?", "answers": "['Coin flipping has been used as a prime example of a random process for decades.', 'That the results of coin flipping are well determined runs counter to the randomness that physicists have been finding in more and more processes once thought to be determined.', 'An accurate prediction of the result of a coin flip requires extraordinarily precise estimation of height and impulse.', 'If the impulses of coin flippings remain perfectly constant, the results are determined only by the height from which the coin falls.']", "label": 2 }, { "id": "train_742", "context": "Citizen: Each year since 1970, a new record has been set for the number of murders committed in this city. This fact points to the decreasing ability of our law enforcement system to prevent violent crime. City official: You overlook the fact that the city' s population has risen steadily since 1970. In fact, the number of murder victims per 100 people has actually fallen slightly in the city since 1970.", "question": "Which one of the following, if true, would most strongly counter the city official's response?", "answers": "['If the health care received by assault victims last year had been of the same quality as it was in 1970, the murder rate in the city last year would have turned out to be several times what it actually was.', 'The incidence of fraud has greatly increased in the city since 1970.', 'Murders and other violent crimes are more likely to be reported now than they were in 1970.', 'The rate of murders in the city since 1970 decreased according to the age group of the victim, decreasing more for younger victims.']", "label": 0 }, { "id": "train_743", "context": "Some statisticians believe that the method called extreme value theory (EVT) is a powerful analytical tool. The curves generated by traditional statistical methods to analyze empirical data on human longevity predict that some humans would live beyond 130 years. According to the curves EVT generates, however, the limit on human life spans is probably between 113 and 124 years. To date, no one has lived beyond the upper limits indicated by EVT analysis.", "question": "Which one of the following can be properly inferred from the statements above?", "answers": "['EVT is, in general, a more reliable method for projecting future trends based on past observations than are traditional statistical methods.', 'Traditional statistical methods of empirical data analysis should eventually be replaced by some version of EVT.', 'EVT fits the data about the highest observed human life spans more closely than do traditional statistical methods.', 'According to the findings derived through the use of EVT, it is physically impossible for any human being to live longer than 124 years.']", "label": 2 }, { "id": "train_744", "context": "Although Damon had ample time earlier in the month to complete the paper he is scheduled to present at a professional conference tomorrow morning, he repeatedly put off doing it. Damon could still get the paper ready in time, but only if he works on it all evening without interruption. However, his seven-year-old daughter' s tap-dance recital takes place this evening, and Damon had promised both to attend and to take his daughter and her friends out for ice cream afterward. Thus, because of his procrastination, Damon will be forced to choose between his professional and his family responsibilities.", "question": "The argument proceeds by", "answers": "['showing that two situations are similar in order to justify the claim that someone with certain responsibilities in the first situation has similar responsibilities in the second situation', \"invoking sympathy for someone who finds himself in a dilemma in order to excuse that person's failure to meet all of his responsibilities\", \"making clear the extent to which someone's actions resulted in harm to others in order to support the claim that those actions were irresponsible\", 'demonstrating that two situations cannot both occur by showing that something necessary for one of those situations is incompatible with something necessary for the other situation']", "label": 3 }, { "id": "train_745", "context": "Mary, a veterinary student, has been assigned an experiment in mammalian physiology that would require her to take a healthy, anesthetized dog and subject it to a drastic blood loss in order to observe the physiological consequences of shock. The dog would neither regain consciousness nor survive the experiment. Mary decides not to do this assignment.", "question": "Mary's decision most closely accords with which one of the following principles?", "answers": "['The only sufficient justification for experimenting on animals is that future animal suffering is thereby prevented.', 'Practicing veterinarians have a professional obligation to strive to prevent the unnecessary death of an animal except in cases of severely ill or injured animals whose prospects for recovery are dim.', 'Taking the life of an animal is not justifiable unless doing so would immediately assist in saving several animal lives or in protecting the health of a person.', 'No one is ever justified in acting with the sole intention of causing the death of a living thing, be it animal or human.']", "label": 2 }, { "id": "train_746", "context": "Jackson City has outgrown its current fire station. The existing station cannot be expanded to fit the city' s needs because of building code restrictions. Real estate in the area of the existing station is far more expensive than building space on the west side of the city. Since the price of a building is a considerable portion of the total cost, it would be more economical to move the fire station to the west side of the city.", "question": "Which of the following, if true, most seriously undermines the argument's conclusion?", "answers": "[\"City residents have expressed approval of the plan to move the city's fire station to the west side of the city.\", 'If the fire station is moved to the west side of the city, insurance rates will skyrocket because the west side of the city has higher arson and vandalism rates.', 'If the fire station is moved to the west side of the city, residents there will have to contend with increased noise and traffic.', 'There is already an ample water supply to support the fire station on the west side of the city.']", "label": 1 }, { "id": "train_747", "context": "Perkins: According to an article I read, the woolly mammoth's extinction in North America coincided with a migration of humans onto the continent 12, 000 years ago, and stone spearheads from this period indicate that these people were hunters. But the author' s contention that being hunted by humans contributed to the woolly mammoth's extinction is surely wrong since, as paleontologists know, no spearheads have ever been found among the many mammoth bones that have been unearthed.", "question": "Which of the following, if true, provides the strongest reason for discounting the evidence Perkins cites in arguing against the contention that being hunted by humans contributed to the North American extinction of woolly mammoths?", "answers": "['Cave paintings in North America that date from 12, 000 years ago depict woolly mammoths as well as a variety of other animals, including deer and buffalo.', 'Because of the great effort that would have been required to produce each stone spearhead, hunters would have been unlikely to leave them behind.', 'At sites where mammoth bones dating from 12, 000 years ago have been unearthed, bones of other mammals have rarely been found.', 'The stone from which stone spearheads were made is unlikely to have disintegrated over the course of 12, 000 years.']", "label": 1 }, { "id": "train_748", "context": "Pieces of music consist of sounds and silences presented to the listener in a temporal order. A painting, in contrast, is not presented one part at a time to the viewer; there is thus no particular path that the viewer' s eye must follow in order to \"read\" the painting. Ttherefore, an essential distinction between the experiences of hearing music and of viewing paintings is that hearing music has a temporal dimension but viewing a painting has none.", "question": "The reasoning in the argument is flawed because", "answers": "['relying on the metaphor of \"reading\" to characterize how a painting is viewed presupposes the correctness of the conclusion to be drawn on the basis of that characterization', 'the argument does not allow for the possibility of being immersed in experiencing a painting without being conscious of the passage of time', 'the absence of a particular path that the eye must follow does not entail that the eye follows no path', 'the argument fails to bring out the aspects of music and painting that are common to both as forms of artistic expression']", "label": 2 }, { "id": "train_749", "context": "When volcanic lava solidifies, it becomes uniformly magnetized in the direction in which the Earth' s magnetic field points. There are significant differences in the direction of magnetization among solidified lava flows from different volcanoes that erupted at different times over the past several million years. Ttherefore, it must be that the direction of the Earth' s magnetic field has changed over time. Since lava flows differing by thousands of years in age often have very similar directions of magnetization, the change in the direction of the Earth' s magnetic field must take place very gradually over hundreds of thousands of years.", "question": "The argument that the direction of the Earth's magnetic field has changed over time requires the assumption that", "answers": "['not all solidified lava has changed the direction of its magnetization unpredictably', \"only lava can be used to measure the direction of the Earth's magnetic field as it existed in the distant past\", 'as lava flows down the side of a volcano, it picks up magnetized rocks', 'a single volcano can produce lava of differing consistencies during different eruptions']", "label": 0 }, { "id": "train_750", "context": "Electrical engineers have repeatedly demonstrated that the best solid-state amplifiers are indistinguishable from the best vacuum-tube amplifiers with respect to the characteristics commonly measured in evaluating the quality of an amplifier' s musical reproduction. Ttherefore, those music lovers who insist that recorded music sounds better when played with the best vacuum-tube amplifier than when played with the best solid-state amplifier must be imagining the difference in quality that they claim to hear.", "question": "Which one of the following, if true, most seriously weakens the argument?", "answers": "['Many people cannot tell from listening to it whether a recording is being played with a very good solid-state amplifier or a very good vacuum-tube amplifier.', 'Some of the characteristics that are important in determining how music sounds to a listener cannot be measured.', 'The range of variation with respect to the quality of musical reproduction is greater for vacuum-tube amplifiers than for solid-state amplifiers.', \"Some vacuum-tube amplifiers are clearly superior to some sold-state amplifiers with respect to the characteristics commonly measured in the laboratory to evaluate the quality of an amplifier's musical reproduction.\"]", "label": 1 }, { "id": "train_751", "context": "Some educators have had remarkable success with an unusual approach to science education. For example, a physics lesson on the properties of waves might begin by plucking guitar strings. After observing the strings vibrate, students would begin to apply names to the phenomena they had witnessed, such as frequency and amplitude. Only after fully understanding these real-life concepts would they begin to apply abstract terms and equations to the phenomena.", "question": "Which one of the following principles is best illustrated by the example above?", "answers": "['Familiarity with phenomena facilitates the learning of more formal treatments of these phenomena.', 'Students are unable to form abstract concepts concerning phenomena until they become familiar with these phenomena.', 'Students will understand abstract terms only if they are applicable to phenomena they have observed.', 'Students learn best when teachers appeal to the interests of their students.']", "label": 0 }, { "id": "train_752", "context": "Dobson: Some historians claim that the people who built a ring of stones thousands of years ago in Britain were knowledgeable about celestial events. The ground for this claim is that two of the stones determine a line pointing directly to the position of the sun at sunrise at the spring equinox. There are many stones in the ring, however, so the chance that one pair will point in a celestially significant direction is large. Ttherefore, the people who built the ring were not knowledgeable about celestial events .", "question": "Which one of the following is an error of reasoning in Dobson's argument?", "answers": "[\"Statements that absolutely establish Dobson's conclusion are treated as if they merely give some support to that conclusion.\", 'The failure of cited evidence to establish a statement is taken as evidence that statement is false.', 'Something that is merely a matter of opinion is treated as if it were subject to verification as a matter of fact.', \"Dobson's conclusion logically contradicts some of the evidence presented in support of it.\"]", "label": 1 }, { "id": "train_753", "context": "For a television program about astrology, investigators went into the street and found twenty volunteers born under the sign of Gemini who were willing to be interviewed on the program and to take a personality test. The test confirmed the investigators' personal impressions that each of the volunteers was more sociable and extroverted than people are on average. This modest investigation thus supports the claim that one' s astrological birth sign influences one' s personality.", "question": "Which one of the following, if true, indicates the most serious flaw in the method used by the investigators?", "answers": "['People born under astrological signs other than Gemini have been judged by astrologers to be much less sociable than those born under Gemini.', 'There is not likely to be a greater proportion of people born under the sign of Gemini on the street than in the population as a whole.', 'People who are not sociable and extroverted are not likely to agree to participate in such an investigation.', \"The personal impressions the investigators first formed of other people have tended to be confirmed by the investigators' later experience of those people.\"]", "label": 2 }, { "id": "train_754", "context": "Quillium is the most popular blood pressure regulating prescription drug on the market. Giant Pharma, Inc. , the largest prescription drug manufacturer in the country, owns the patent on Quillium. Giant Pharma stock is hitting unprecedented high valuations. As a result, Quillium is by far the most effective drug available in treating irregular blood pressure.", "question": "Which of the following, if true, most weakens the argument?", "answers": "[\"Giant Pharma gouges its customers on Quillium's price.\", 'Quillium has numerous alternate applications.', 'The most lucrative and popular pharmaceuticals are not always the most effective.', \"Giant Pharma's high stock prices are attributable to recent patent acquisitions other than Quillium.\"]", "label": 2 }, { "id": "train_755", "context": "A recent study confirmed that salt intake tends to increase blood pressure and found that, as a result, people with high blood pressure who significantly cut their salt intake during the study had lower blood pressure by the end of the study. However, it was also found that some people who had very high salt intake both before and throughout the study maintained very low blood pressure.", "question": "Which one of the following, if true, contributes the most to an explanation of the results of the study?", "answers": "['At the beginning of the study, some people who had very low salt intake also had very high blood pressure.', 'For most people who have high blood pressure, reducing salt intake is not the most effective dietary change they can make to reduce their blood pressure.', 'Persons suffering from abnormally low blood pressure have heightened salt cravings, which ensure that their blood pressure does not drop too low.', 'Study participants with high blood pressure who cut their salt intake only slightly during the study did not have significantly lower blood pressure by the end of the study.']", "label": 2 }, { "id": "train_756", "context": "Opposition leader: Our country has the least fair court system of any country on the continent and ought not to be the model for others. Thus, our highest court is the least fair of any on the continent and ought not to be emulated by other countries.", "question": "The flawed reasoning in which one of the following arguments is most similar to that in the opposition leader's argument?", "answers": "['Domestic cats are closely related to tigers. Ttherefore, even though they are far smaller than tigers, their eating habits are almost the same as those of tigers.', 'The residents of medium-sized towns are, on average, more highly educated than people who do not live in such towns. Ttherefore, Maureen, who was born in a medium-sized town, is more highly educated than Monica, who has just moved to such a town.', 'At a certain college, either philosophy or engineering is the most demanding major. Ttherefore, either the introductory course in philosophy or the introductory course in engineering is the most demanding introductory-level course at that college.', 'If a suit of questionable merit is brought in the first district rather than the second district, its chances of being immediately thrown out are greater. Ttherefore, to have the best chance of winning the case, the lawyers will bring the suit in the second district.']", "label": 2 }, { "id": "train_757", "context": "Between 1980 and 2000 the sea otter population of the Aleutian Islands declined precipitously. There were no signs of disease or malnutrition, so there was probably an increase in the number of otters being eaten by predators. Orcas will eat otters when seals, their normal prey, are unavailable, and the Aleutian Islands seal population declined dramatically in the 1980s. Ttherefore, orcas were most likely the immediate cause of the otter population decline.", "question": "Which of the following, if true, most strengthens the argument?", "answers": "['The population of orcas in the Aleutian Islands has declined since the 1980s.', 'Seals do not eat sea otters, nor do they compete with sea otters for food.', 'Most of the surviving sea otters live in a bay that is inaccessible to orcas.', 'The population of sea urchins, the main food of sea otters, has increased since the sea otter population declined.']", "label": 2 }, { "id": "train_758", "context": "In a certain wildlife park, park rangers are able to track the movements of many rhinoceroses because those animals wear radio collars. When, as often happens, a collar slips off, it is put back on. Putting a collar on a rhinoceros involves immobilizing the animal by shooting it with a tranquilizer dart. Female rhinoceroses that have been frequently recollared have significantly lower fertility rates than uncollared females. Probably, ttherefore, some substance in the tranquilizer inhibits fertility.", "question": "In evaluating the argument, it would be most useful to determine which of the following?", "answers": "[\"Whether male rhinoceroses in the wildlife park lose their collars any more often than the park's female rhinoceroses do\", 'Whether radio collars are the only practical means that park rangers have for tracking the movements of rhinoceroses in the park', 'How often park rangers need to use tranquilizer darts to immobilize rhinoceroses for reasons other than attaching radio collars', 'Whether there are more collared female rhinoceroses than uncollared female rhinoceroses in the park']", "label": 2 }, { "id": "train_759", "context": "Jamal: It' s incorrect to maintain that current data suggest a weakness in the economy. Stocks are paying higher dividends than ever and corporations' profits are very high. Yet you criticize corporations on the basis of wage data, which are the only data showing a decline. Jennifer: You' re right that only wage data show a decline, but those data are important because they show a shift of income from companies' workers to their shareholders. The statistics you cite mean little to the expanding numbers of workers finding it increasingly difficult to make a living.", "question": "Jamal's and Jennifer's statements provide the most support for holding that they agree about the truth of which one of the following?", "answers": "['Corporations are currently making considerable profits.', 'The current priorities of many corporations are misplaced.', 'Statistical information tends to reflect the biases of its compilers.', 'The decline in wages is a worrisome economic trend.']", "label": 0 }, { "id": "train_760", "context": "In social science research, \"highest education level attained\" would refer to the most advanced grade or degree achieved by an individual-for some individuals, it may be a grade in grade school, and for other individuals, it may be a Bachelor' s Degree, a Master' s Degree, or Ph. D. (which is considered the highest education level). A recent study has shown a strong correlation between highest education level attained and proficiency in chess. Another result, studied at many points throughout the 20th century, shows a marked positive correlation between highest education level attained and income level.", "question": "Assuming the statements above are true, what conclusion can be drawn?", "answers": "[\"An individual's proficiency at chess rises consistently during that individual's years of school, and levels off once that individual has finished her years of formal education.\", 'If Jane has a Ph. D. , and Chris has not finished his undergraduate degree, then Jane will usually beat Chris in chess.', 'It is possible that a person who has attained only a sixth grade level of education could earn more than a person who has a Ph. D.', \"If one practices chess enough to raise one's proficiency, one has a good chance of raising one's income level.\"]", "label": 2 }, { "id": "train_761", "context": "Astronomer: Astronomical observations in many areas have become useless because light from nearby cities obscures the stars Many people argue that since streetlights are needed for safety, such interference from lights is inevitable. Here in Sandsville, however, the local observatory' s view remains relatively clear, since the city has restricted unnecessary lighting and installed special street lamps that direct all their light downward. It is ttherefore possible to have both well-lighted streets and relatively dark skies.", "question": "The astronomer's argument proceeds by", "answers": "['offering a counter example to a general claim', 'proposing an alternative scientific explanation for a natural phenomenon', 'appealing to a scientific authority to challenge a widely held belief', 'making a distinction between terms']", "label": 0 }, { "id": "train_762", "context": "From a book review: The authors blithely claim that there are \"three basic ways to store energy: as heat, as electricity, or as kinetic energy. \" However, I cannot call to mind any effective ways to store energy as electricity, whereas any capable student of physics could readily suggest a few more ways to store energy: chemical, gravitational, nuclear.", "question": "The reviewer makes which one of the following criticisms of a claim that appears in the book under review?", "answers": "['Except possibly for electricity, all ways to store energy are equally effective and ttherefore equally basic.', 'There is no reason to consider any particular way to store energy any more basic than any other.', 'What needs to be considered is not whether various ways to store energy are basic but whether they are effective.', 'The list given of ways to store energy is possibly inaccurate and certainly not exhaustive.']", "label": 3 }, { "id": "train_763", "context": "Some people have maintained that private ownership of the means of production ultimately destroys any society that sanctions it. This may be true of a less technologically advanced society that must share its economic resources to survive. But since only private ownership of the means of production permits individuals to test new technologies without the majority' s consent, a technologically advanced society will actually endanger its survival if the means of production become public property.", "question": "The proposition that private ownership of the means of production ultimately destroys any society that sanctions it plays which one of the following roles in the argument above?", "answers": "['It is a generalization that the argument suggests is no more applicable to less technologically advanced societies than to more technologically advanced societies.', 'It is a general hypothesis that the argument suggests is inapplicable to societies more dependent for survival upon the introduction of new technologies than upon the sharing of resources.', 'It is a contention about the consequences of an economic arrangement that the argument claims is incompatible with the needs of any society.', 'It is a generalization about societies that according to the argument is true for any society in which the majority of its citizens does not impede the introduction of new technologies.']", "label": 1 }, { "id": "train_764", "context": "Of the two proposals for solving the traffic problems on Main Street, Chen' s plan is better for the city as a whole, as is clear from the fact that the principal supporter of Ripley' s plan is Smith Stores. Smith Stores, with its highly paid consultants, knows where its own interest lies and, moreover, has supported its own interests in the past, even to the detriment of the city as a whole.", "question": "The faulty reasoning in which one of the following is most parallel to that in the argument above?", "answers": "['The planned light-rail system will clearly serve suburban areas well, since its main opponent is the city government, which has always ignored the needs of the suburbs and sought only to protect the interests of the city.', \"Nomura is clearly a better candidate for college president than Miller, since Nomura has the support of the three deans who best understand the president's job and with whom the president will have to work most closely.\", 'The school board should support the plan for the new high school since this plan was recommended by the well-qualified consultants whom the school board hired at great expense.', \"Surely Centreville should oppose adoption of the regional planning commission's new plan since it is not in Centreville's interest, even though it might be in the interest of some towns in the region.\"]", "label": 0 }, { "id": "train_765", "context": "Normally, businesses operate at a profit. In other words, they sell their products to customers for more money than the products initially cost. This pricing structure permits businesses to sustain themselves over time. The Stop and Go Supermarket, however, sells its milk for less money than it costs.", "question": "Which of the following, if true, would best explain why the pricing structure for milk makes economic sense for Stop and Go?", "answers": "['Stop and Go directly competes with several lower-priced markets and feels it must match their prices in order to keep its customers.', 'Because Stop and Go does not carry premier brands of milk, it feels an obligation to sell its lesser brands at a loss.', 'Milk is a commodity good, and Stop and Go believes all of its customers deserve the right to purchase commodity goods at reasonable prices.', 'Low prices for milk attract large numbers of customers to the store, and these customers purchase many additional profitable items.']", "label": 3 }, { "id": "train_766", "context": "A study was designed to establish what effect, if any, the long-term operation of offshore oil rigs had on animal life on the bottom of the sea. The study compared the sea-bottom communities near rigs with those located in control sites several miles from any rig and found no significant differences. The researchers concluded that oil rigs had no adverse effect on sea-bottom animals.", "question": "Which one of the following, if true, most seriously weakens the researchers' conclusion?", "answers": "['Commercially important fish depend on sea- bottom animals for much of their food, so a drop in catches of those fish would be evidence of damage to sea-bottom communities.', 'Only part of any oil discharged into the ocean reaches the ocean floor: some oil evaporates, and some remains in the water as suspended drops.', 'Contamination of the ocean floor from sewage and industrial effluent does not result in the destruction of all sea-bottom animals but instead reduces species diversity as well as density of animal life.', 'The discharge of oil from offshore oil rigs typically occurs at the surface of the water, and currents often carry the oil considerable distances before it settles on the ocean floor.']", "label": 3 }, { "id": "train_767", "context": "Industrialist: Environmentalists contend that emissions from our factory pose a health risk to those living downwind. The only testimony presented in support of this contention comes from residents of the communities surrounding the factory. But only a trained scientist can determine whether or not these emissions are dangerous, and none of the residents are scientists. Hence our factory' s emissions present no health risk.", "question": "The reasoning in the industrialist's argument is flawed because the argument", "answers": "['equivocates between two different notions of the term \"health risk\"', 'presents no testimony from scientists that the emissions are safe', 'fails to discuss the benefits of the factory to the surrounding community', 'does not consider the safety of emissions from other sources in the area']", "label": 1 }, { "id": "train_768", "context": "Parents who consistently laud their children for every attempt to accomplish something, whether successful or not, actually erode the youngsters' sense of self-esteem. Children require commendation for their achievements, but if uniformly praised for both what they have accomplished and what they have merely attempted, they will eventually discount all words of commendation. In effect, such children never hear any praise at all.", "question": "Which one of the following most accurately expresses the overall conclusion of the argument?", "answers": "['Children will develop low self-esteem if their parents do not praise them when they succeed.', \"Children's self-esteem will suffer if their parents uniformly praise their attempts to accomplish things regardless of their success or failure.\", 'Parents should praise their children for their achievements.', 'Children whose actions are praised undeservedly eventually learn to discount all words of praise.']", "label": 1 }, { "id": "train_769", "context": "Benford's Law states that most phenomena in nature begin with '1's and '2' s. The law has been shown to apply very accurately to the amount of money listed on people's income statements. Investigators have used this fact when investigating financial fraud, and can often determine when people have fabricated numbers, since the perpetrators will choose varying amounts so that the first number in each of those amounts is distributed evenly from '1' to '9'. Proponents of Benford's Law, ttherefore, argue that as the technique of applying Benford's law becomes even more refined, an expert's ability to determine whether a plaintiff has engaged in financial fraud will become as accurate in determining guilt as a fingerprint is today.", "question": "Which of the following, if true, casts the most doubt on the conclusion drawn by proponents of Benford's Law?", "answers": "[\"In one financial fraud case, the numbers in which the amount was stated was not distributed evenly across '1's and '9's.\", \"In the last year, several instances of financial fraud were discovered using other means besides Benford's Law.\", \"Benford's Law, after being used in several notable cases, has become highly publicized and intimately known to many likely to commit such crimes.\", 'The use of fingerprinting is highly sophisticated since it has had over a century to evolve as a technology.']", "label": 2 }, { "id": "train_770", "context": "If something would have been justifiably regretted if it had occurred, then it is something that one should not have desired in the first place. It follows that many forgone pleasures should not have been desired in the first place.", "question": "The conclusion above follows logically if which one of the following is assumed?", "answers": "['Everything that one desires and then regrets not having is a forgone pleasure.', \"One should never regret one's pleasures.\", 'Many forgone pleasures would have been justifiably regretted.', 'Forgone pleasures that were not desired would not have been justifiably regretted.']", "label": 2 }, { "id": "train_771", "context": "Court analyst: Courts should not allow the use of DNA tests in criminal cases. There exists considerable controversy among scientific experts about how reliable these tests are. Unless there is widespread agreement in the scientific community about how reliable a certain test is, it is unreasonable for the courts to allow evidence based on that test.", "question": "The court analyst's reasoning is flawed because it fails to take into account that", "answers": "['experts may agree that the tests are highly reliable while disagreeing about exactly how reliable they are', 'there are also controversies about reliability of evidence in noncriminal cases', 'the standard against which evidence in a criminal case is measured should not be absolute certainty', 'data should not be admitted as evidence in a court of law without scientific witnesses having agreed about how reliable they are']", "label": 0 }, { "id": "train_772", "context": "According to the \"bottom-up\" theory of how ecosystems are structured, the availability of edible plants is what primarily determines an ecosystem' s characteristics since it determines how many herbivores the ecosystem can support, which in turn determines how many predators it can support. This theory also holds that a reduction in the number of predators will have little impact on the rest of the ecosystem.", "question": "Which one of the following, if true, would provide evidence against the bottom-up theory?", "answers": "['In an effort to build up the population of a rare species of monkey on Vahique Island, monkeys were bred in zoos and released into the wild. However, the effort failed because the trees on which the monkeys fed were also nearly extinct.', \"After many of the trees on Jaevix Island were cleared, the island's leaf-cutter ants, which require a forested ecosystem, experienced a substantial decrease in population, as did the island's anteaters.\", 'Plants that are a dietary staple of wild pigs on Sedif Island have flourished over the last three decades, and the population of the pigs has not changed much in spite of extensive hunting.', 'After hunting virtually eliminated predators on Rigu Island, the population of many herbivore species increased more than tenfold, causing the density of plants to be dramatically reduced.']", "label": 3 }, { "id": "train_773", "context": "Builder: Within ten years, most of the new homes constructed in North America will have steel frameworks rather than wood ones. After all, two-by-fours and two-by-sixes -- the sizes of lumber most commonly used in home construction -- are deteriorating in quality and increasing in cost, while environment-friendly steel is decreasing in cost. In addition, unlike wood, steel will not warp, rot, or split.", "question": "Which one of the following, if true, most seriously weakens the builder's reasoning?", "answers": "['Training home construction workers to work with steel is very costly.', 'Steel-framed homes do not have to be treated with pesticides or other chemicals that can contribute to indoor air pollution.', 'Because lumber prices have increased over the last decade, currently most new homes are built with steel frameworks.', 'The number of houses built each year is expected to decrease over the next decade.']", "label": 0 }, { "id": "train_774", "context": "Essayist: One of the drawbacks of extreme personal and political freedom is that free choices are often made for the worst. To expect people to thrive when they are given the freedom to make unwise decisions is frequently unrealistic. Once people see the destructive consequences of extreme freedom, they may prefer to establish totalitarian political regimes that allow virtually no freedom. Thus, one should not support political systems that allow extreme freedom.", "question": "Which one of the following principles, if valid, most helps to justify the essayist's reasoning?", "answers": "['One should not support any political system whose destructive consequences could lead people to prefer totalitarian political regimes.', \"One should not support any political system that is based on unrealistic expectations about people's behavior under that system.\", 'One should not support any political system that will inevitably lead to the establishment of a totalitarian political regime.', \"One should not expect everyone to thrive even in a political system that maximizes people's freedom in the long run.\"]", "label": 0 }, { "id": "train_775", "context": "A recent study of perfect pitch -- the ability to identify the pitch of an isolated musical note -- found that a high percentage of people who have perfect pitch are related to someone else who has it. Among those without perfect pitch, the percentage was much lower. This shows that having perfect pitch is a consequence of genetic factors.", "question": "Which one of the following, if true, most strengthens the argument?", "answers": "['People who have relatives with perfect pitch generally receive no more musical training than do others.', 'All of the researchers conducting the study had perfect pitch.', 'People with perfect pitch are more likely than others to choose music as a career.', 'People with perfect pitch are more likely than others to make sure that their children receive musical training.']", "label": 0 }, { "id": "train_776", "context": "Salesperson: When a salesperson is successful, it is certain that that person has been in sales for at least three years. This is because to succeed as a salesperson, one must first establish a strong client base, and studies have shown that anyone who spends at least three years developing a client base can eventually make a comfortable living in sales.", "question": "The reasoning in the salesperson's argument is vulnerable to criticism on the grounds that it fails to consider the possibility that", "answers": "['it takes longer than three years for a salesperson to develop a strong client base', 'a salesperson who has not spent three years developing a client base may not succeed in sales', 'some salespeople require fewer than three years in which to develop a strong client base', 'salespeople who have spent three years developing a client base might not yet be successful in sales']", "label": 2 }, { "id": "train_777", "context": "The success of a private university depends on the relative long-term success of its graduates. This is evident from the fact that unless the majority of a private university' s graduates are successful, it will be impossible for the graduates to donate large sums of money to their alma mater. It should be obvious that a private university is successful only if its graduates donate large amounts of money to it.", "question": "The claim that a private university is successful only if its graduates donate large amounts of money to it plays which of the following roles in the argument?", "answers": [ "It is used to illustrate a consequence of the argument's conclusion.", "It is a claim that must be rebutted for the argument's conclusion to stand.", "It is the argument's conclusion.", "It is designed to support the argument's conclusion." ], "label": 3 }, { "id": "train_778", "context": "Theoretically, analog systems are superior to digital systems. A signal in a pure analog system can be infinitely detailed, while digital systems cannot produce signals that are more precise than their digital units. With this theoretical advantage there is a practical disadvantage, however. Since there is no limit on the potential detail of the signal, the duplication of an analog representation allows tiny variations from the original, which are errors. These errors tend to accumulate as signals are duplicated, until this \"noise\" obliterates the information embodied in the original signal.", "question": "The statements above, if true, most strongly support which one of the following?", "answers": "['Digital systems are the best information systems because error cannot occur in the emission of digital signals.', 'Analog representation of information is impractical because we do not need infinitely detailed information.', 'Many ideas that work well in theory do not work well in practice.', 'Digital systems are preferable to analog systems when the signal must be reproduced many times.']", "label": 3 }, { "id": "train_779", "context": "Advertisement: Anyone who thinks moisturizers are not important for beautiful skin should consider what happens to the earth, the skin of the world, in time of drought. Without regular infusions of moisture the ground becomes lined and cracked and its lush liveliness fades away. Thus your skin, too, should be protected from the ravages caused by lack of moisture; give it the protection provided by regular infusions of Dewyfresh, the drought-defying moisturizer.", "question": "The Dewyfresh advertisement exhibits which one of the following errors of reasoning?", "answers": "['It relies on an analogy between two things that are insufficiently alike in the respects in which they would have to be alike for the conclusion to be supported.', 'It treats something that is necessary for bringing about a state of affairs as something that is sufficient to bring about that state of affairs.', 'It treats the fact that two things regularly occur together as proof that there is a single thing that its the cause of them both.', 'It relies on the ambiguity of the term \"infusion, \"which can designate either a process or the product of that process.']", "label": 0 }, { "id": "train_780", "context": "Drama critic: Audiences will enjoy Warner' s latest play, about the disintegration of a family, because of the superb acting. The chemistry between the actors could not be more intense, and Ziegler, who plays the child, captures convincingly the guilt and despair experienced as the family members grow more estranged from each other.", "question": "Each of the following, if true, weakens the argument EXCEPT:", "answers": "[\"The plot of the play is similar in some respects to plots of Warner's other works.\", \"Audiences usually find drama critics' reviews unreliable.\", 'Generally, audiences enjoy romantic comedies but find tragedies upsetting.', 'The company staging the play has an unbroken history of dull performances.']", "label": 0 }, { "id": "train_781", "context": "Researcher: Results indicate that the higher their educational level, the better are students' mathematical skills. These results do not prove that education improves mathematical skills, however, since it is possible that students who have better mathematical skills to start with are the students who reach higher educational levels.", "question": "The reasoning of the researcher's argument is most similar to that of which one of the following arguments?", "answers": "['Results indicate that the quality of papers submitted for publication varies significantly from university to university. This may say nothing about the quality of education offered at different schools, but may only reflect a defect in the review process.', \"Results from competition indicate that professional athletes outperform amateur athletes. These results do not prove that becoming a professional athlete improves one's athletic performance, since it is possible that the athletes who become professionals are those whose performance is better to begin with.\", 'Studies suggest that some people who are gifted in higher mathematics are inept at performing simple arithmetical calculations. These studies do not show that being good at mathematics precludes being good at arithmetic, since there are also many people who are good at both.', 'Surveys indicate that politicians with law degrees are better at what they do than politicians without law degrees. These surveys do not prove that having a law degree makes one a better politician, since it is possible that many politicians without law degrees were left out of the survey.']", "label": 1 }, { "id": "train_782", "context": "Client: The owners of the catering company we use decided to raise their rates. They argued that the increase was necessary to allow them to hire and train new staff to accommodate their expanding client base. They should reconsider that decision and not raise their rates. After all, the mission of the company is to provide low-cost gourmet catering, and this mission will be jeopardized if they raise rates.", "question": "Which one of the following most accurately expresses the main conclusion of the client's argument?", "answers": "[\"The catering company's mission is to provide low-cost gourmet catering.\", \"The catering company's mission will be jeopardized if its rates are increased.\", \"The catering company's rates should not be raised.\", 'The owners of the catering company decided to raise their rates.']", "label": 2 }, { "id": "train_783", "context": "Keeler wanted the institute to receive bad publicity. He and Greene were the only ones in a position to tell the press about the institute' s potentially embarrassing affiliations, but Greene had no reason to do so. Ttherefore, it must have been Keeler who notified the press.", "question": "Which one of the following arguments is most closely parallel in its reasoning to the argument above?", "answers": "['Cutter and Rengo are the only serious candidates for designing the new monument. Rengo has designed several beautiful monuments and has connections to the selection committee. Ttherefore, it will probably be Rengo who is awarded the job of designing the monument.', 'Other than Helms and Lapinski, no one had access to the equipment on Thursday, the day it was tampered with. Thus, since Helms had reason to tamper with the equipment and Lapinski did not, it must have been Helms who did it.', 'The only people who had any reason to write the anonymous letter were Johnson and Ringwold. Johnson and Ringwold both deny doing so. Ringwold, however, admits that she has written anonymous letters in the past. Thus, it must have been Ringwold who wrote the letter.', 'Carter and Whitequill were the only ones who had any motive to bribe the public official. But Whitequill would have been too fearful that the bribery might somehow be made public. Carter, ttherefore, must be the person who bribed the public official.']", "label": 1 }, { "id": "train_784", "context": "Journalist: Can you give me a summary of the 25. novel you are working on? Novelist: Well, I assume that by \"summary\" you mean something brief and not a version of the novel itself. The reason I write novels is that what I want to communicate can be communicated only in the form of a novel. So I am afraid I cannot summarize my novel for you in a way that would tell you what I am trying to communicate with this novel.", "question": "Which one of the following exhibits a pattern of reasoning that is most parallel to that used by the novelist?", "answers": "['A banquet menu foretells the content of a meal, but some people collect menus in order to remind themselves of great meals they have eaten. Thus a banquet menu has a function not only before, but also after, a meal has been served.', 'Watching a travelog is not the same as traveling. But a travelog confers some of the benefits of travel without the hardships of travel. So many people just watch travelogs and do not undergo the hardships of travel.', 'Only a statement that does not divulge company secrets can be used as a press release. This statement does not divulge company secrets, but it is uninformative and ttherefore cannot be used as a press release.', 'Only a three-dimensional representation of a landscape can convey the experience of being in that landscape. A photograph taken with a traditional camera is not three-dimensional. Ttherefore a photograph taken with a traditional camera can never convey the experience of being in a landscape.']", "label": 3 }, { "id": "train_785", "context": "In Malsenia sales of classical records are soaring. The buyers responsible for this boom are quite new to classical music and were drawn to it either by classical scores from television commercials or by theme tunes introducing major sports events on television. Audiences at classical concerts, however, are continually shrinking in Malsenia. It can be concluded from this that the new Malsenian converts to classical music, having initially experienced this music as recorded music, are most comfortable with classical music as recorded music and really have no desire to hear live performances.", "question": "The argument assumes which one of the following?", "answers": "['Classical concerts in Malsenia are not limited to music that is readily available on recordings.', 'The classical records available in Malsenia are, for the most part, not recordings of actual public concerts.', 'To sell well in Malsenia, a classical record must include at least one piece familiar from television.', 'The number of classical concerts performed in Malsenia has not decreased in response to smaller audiences.']", "label": 3 }, { "id": "train_786", "context": "Professor Robinson: A large meteorite impact crater in a certain region was thought to be the clue to explaining the mass extinction of plant and animal species that occurred at the end of the Mesozoic era. However, the crystalline structure of rocks recovered at the site indicates that the impact that formed this crater was not the culprit. When molten rocks crystallize, they display the polarity of Earth' s magnetic field at that time. But the recrystallized rocks recovered at the site display normal magnetic polarity, even though Earth' s magnetic field was reversed at the time of the mass extinction.", "question": "Each of the following is an assumption on which Professor Robinson' s argument depends EXCEPT:", "answers": "['The crater indicates an impact of more than sufficient size to have caused the mass extinction.', 'No other event caused the rocks to melt after the impact formed the crater.', 'The recovered rocks melted as a result of the impact that formed the crater.', 'The recovered rocks recrystallized shortly after they melted.']", "label": 0 }, { "id": "train_787", "context": "Because dried peat moss, which is derived from sphagnum moss, contains no chemical additives and is a renewable resource, many gardeners use large amounts of it as a soil conditioner in the belief that the practice is environmentally sound. They are mistaken. The millions of acres of sphagnum moss in the world contribute more oxygen to the atmosphere than do all of the world' s rain forests combined, and the garden soil industry is depleting these areas much faster than they can renew themselves.", "question": "Which one of the following principles, if valid, most helps to justify the argument's reasoning?", "answers": "['A practice is environmentally sound if it helps to protect rain forests that contribute large amounts of oxygen to the atmosphere.', 'A practice is not environmentally sound if it significantly reduces the amount of oxygen entering the atmosphere.', 'Using a product may be environmentally unsound even if the product is a renewable resource and contains no chemical additive.', 'If the environmental benefits of a practice outweigh the environmental costs, that practice can be legitimately considered environmentally sound.']", "label": 1 }, { "id": "train_788", "context": "Jane: According to an article in this newsmagazine, children' s hand-eye coordination suffers when they spend a great amount of time watching television. Ttherefore, we must restrict the amount of time Jacqueline and Mildred are allowed to watch television. Alan: Rubbish! The article says that only children under three are affected in that way. Jacqueline is ten and Mildred is eight. Ttherefore, we need not restrict their television viewing.", "question": "Alan's argument against Jane's conclusion makes which one of the following errors in reasoning?", "answers": "['It relies on the same source that Jane cited in support of her conclusion.', 'It confuses undermining an argument in support of a given conclusion with showing that the conclusion itself is false.', 'It fails to distinguish the consequences of a certain practice from the causes of the practice.', 'It makes an irrelevant appeal to an authority.']", "label": 1 }, { "id": "train_789", "context": "After an area has been hit by a natural disaster, there is often a great demand for plywood for repairing damaged homes. Retailers in the area often raise prices on new shipments of plywood to well above their predisaster prices, and some people denounce these retailers for taking advantage of a disaster to make more money on each sheet of plywood they sell. In fact, however, these retailers do not make more money on each sheet of plywood than before the disaster, because transporting the plywood into devastated areas is difficult and expensive, and ttherefore the plywood' s cost to retailers is higher than it was before the disaster.", "question": "Which one of the following is an assumption on which the argument depends?", "answers": "['The increase in the prices charged by retailers for plywood following a natural disaster does not exceed the increase in cost to those retailers.', 'Suppliers of plywood do not transport as much plywood to an area after it has been affected by a natural disaster as they did before it was so affected.', 'Retailers must pay the full amount of any increase in shipping costs.', 'No retailer makes enough money on each sheet of plywood sold to absorb for long an increase in shipping costs without raising prices.']", "label": 0 }, { "id": "train_790", "context": "Because of the recent transformation of the market, Quore, Inc. , must increase productivity 10 percent over the course of the next two years, or it will certainly go bankrupt. In fact, however, Quore' s production structure is such that if a 10 percent productivity increase is possible, then a 20 percent increase is attainable.", "question": "If the statements above are true, which one of the following must on the basis of them also be true?", "answers": "['If a 20 percent productivity increase is unattainable for Quore, then it must go bankrupt.', 'Quore will not go bankrupt if it achieves a productivity increase of 20 percent over the next two years.', \"It is only Quore's production structure that makes it possible for Quore to survive the transformation of the market.\", 'Because of the transformation of the market, Quore will achieve a productivity increase of 10 percent over the next two years.']", "label": 0 }, { "id": "train_791", "context": "A scientific team compared gold samples from several ancient artifacts with gold samples from an ancient mine in western Asia . The ratios of the trace elements in these samples were all very similar, and they were unlike the trace-element ratios from any other known mine. It is ttherefore likely that the gold in the artifacts was dug from the ancient mine.", "question": "Which one of the following, if true, most weakens the argument?", "answers": "['Much of the gold dug from the ancient mine in western Asia was transported to faraway destinations.', 'Ancient gold artifacts were often constructed from gold taken from earlier artifacts.', 'The ancient mine tapped into a large underground deposit that also supplied nearby riverbeds with significant quantities of gold. GO ON TO THE NEXT PAGE', 'The ancient mine was first operated many centuries before the artifacts were constructed.']", "label": 2 }, { "id": "train_792", "context": "Detective: People who repeatedly commit crimes like embezzlement or bribery without being caught tend to become more confident. With each success, they believe that getting caught is less likely. However, the more crimes a person commits, the greater the chance that one of those crimes will be solved. It is ttherefore likely that most people who commit embezzlement or bribery will eventually be caught.", "question": "Which one of the following is an assumption required by the detective's argument?", "answers": "['The majority of people who commit embezzlement or bribery do so repeatedly.', 'People who commit embezzlement or bribery tend to be people who feel confident.', 'People who repeatedly commit embezzlement or bribery become more and more careless the longer they avoid detection.', 'Embezzlement and bribery are more likely to be solved than are many other types of crimes.']", "label": 0 }, { "id": "train_793", "context": "One variety of partially biodegradable plastic beverage container is manufactured from small bits of plastic bound together by a degradable bonding agent such as cornstarch. Since only the bonding agent degrades, leaving the small bits of plastic, no less plastic refuse per container is produced when such containers are discarded than when comparable nonbiodegradable containers are discarded.", "question": "Which of the following, if true, most strengthens the argument above?", "answers": "['Technological problems with recycling currently prevent the reuse as food or beverage containers of the plastic from either type of plastic beverage container.', 'Many consumers are ecology-minded and prefer to buy a product sold in the partially biodegradable plastic beverage containers rather than in nonbiodegradable containers, even if the price is higher.', 'The partially biodegradable plastic beverage containers are made with more plastic than comparable nonbiodegradable ones in order to compensate for the weakening effect of the bonding agents.', 'Both partially biodegradable and nonbiodegradable plastic beverage containers can be crushed completely flat by refuse compactors.']", "label": 2 }, { "id": "train_794", "context": "Economist: Some people argue that when large countries split into several small countries, the world economy is harmed by increased barriers to free trade in the form of an increased number of national tariffs. But small countries do not think of themselves as economically self-sufficient. Ttherefore, such division of large countries does not increase barriers to free trade.", "question": "Which one of the following, if assumed, enables the economist's conclusion to be properly drawn?", "answers": "['Large countries tend to be more economically self-sufficient than small countries.', 'A country has the right to split into smaller countries even if some of the economic consequences of division would harm the world economy.', 'All countries that impose national tariffs or other barriers to free trade think of themselves as economically self-sufficient.', 'There is strong evidence that national tariffs and other barriers to free trade harm the world economy.']", "label": 2 }, { "id": "train_795", "context": "Principle: Even if an art auction house identifies the descriptions in its catalog as opinions, it is guilty of misrepresentation if such a description is a deliberate attempt to mislead bidders. Application: Although Healy' s, an art auction house, states that all descriptions in its catalog are opinions, Healy' s was guilty of misrepresentation when its catalog described a vase as dating from the mid-eighteenth century when it was actually a modern reproduction.", "question": "Which one of the following, if true, most justifies the above application of the principle?", "answers": "[\"Without consulting anyone with expertise in authenticating vases, Healy's described the vase as dating from the mid-eighteenth century merely in order to increase its auction price.\", 'Although pottery that is similar to the vase is currently extremely popular among art collectors, none of the collectors who are knowledgeable about such pottery were willing to bid on the vase.', \"Some Healy's staff members believe that the auction house's catalog should not contain any descriptions that have not been certified to be true by independent experts.\", 'An authentic work of art from the mid-eighteenth century will usually sell for at least ten times more than a modern reproduction of a similar work from that period.']", "label": 0 }, { "id": "train_796", "context": "Grant says that, as an engineer, he knows that no engineer enjoys fishing. And, since most engineers are linear thinkers, at least some of the people who enjoy fishing are not linear thinkers.", "question": "Which one of the following is most parallel in its reasoning to the flawed reasoning above?", "answers": "['Sulema says that, as a marketing director, she knows that no marketing directors like to waste time. And since most marketing directors are ethical, at least some people who enjoy wasting time are ethical people.', 'Charlotte says that, as a florist, she knows that no white roses are as fragrant as the most fragrant red roses. And, since most Kentucky roses are white, Kentucky roses are inferior to at least the best California red roses.', 'Rebecca says that, as a mother of three children, she knows that no mother wants her children to watch horror films. And, since most mothers are adults, at least some of the people who want children to watch horror films are children.', 'Cooper says that, as an experienced tailor, he knows that no cotton is as soft as the best silk. And since, most T-shirts are made of cotton, at least some T-shirts are not as soft as the best silk garments.']", "label": 2 }, { "id": "train_797", "context": "Sparrow Airlines is planning to reduce its costs by cleaning its planes' engines once a month, rather than the industry standard of every six months. With cleaner engines, Sparrow can postpone engine overhauls, which take planes out of service for up to 18 months. Furthermore, cleaning an engine reduces its fuel consumption by roughly 1. 2 percent.", "question": "The airline's plan assumes that", "answers": "[\"the cost of monthly cleaning of an airplane's engines is not significantly greater in the long run than is the cost of an engine overhaul\", 'Sparrow Airlines has had greater problems with engine overhauls and fuel consumption than other airlines have', 'engine cleaning does not remove an airplane from service', 'cleaning engines once a month will give Sparrow Airlines a competitive advantage over other airlines']", "label": 0 }, { "id": "train_798", "context": "Historian: In rebuttal of my claim that West influenced Stuart, some people point out that West' s work is mentioned only once in Stuart' s diaries. But Stuart' s diaries mention several meetings with West, and Stuart' s close friend, Abella, studied under West. Furthermore, Stuart' s work often uses West' s terminology which, though now commonplace, none of Stuart' s contemporaries used.", "question": "Which one of the following propositions is most supported by the historian's statements, if those statements are true?", "answers": "[\"Stuart's work was not entirely free from West's influence\", \"Stuart's contemporaries were not influenced by West.\", 'It is more likely that Stuart influenced West than that West influenced Stuart.', \"Stuart's discussions with Abella were one of the means by which West influenced Stuart.\"]", "label": 0 }, { "id": "train_799", "context": "Researcher: This fall I returned to a research site to recover the armadillos I had tagged there the previous spring. Since a large majority of the armadillos I recaptured were found within a few hundred yards of the location of their tagging last spring, I concluded that armadillos do not move rapidly into new territories.", "question": "Which one of the following is an assumption required by the researcher's argument?", "answers": "['A large majority of the recaptured armadillos did not move to a new territory in the intervening summer and then move back to the old territory by the fall.', 'Predators did not kill any of the armadillos that had been tagged the previous spring.', 'The tags identifying the armadillos cannot be removed by the armadillos, either by accident or deliberately.', 'Of the armadillos living in the area of the tagging site last spring, few were able to avoid being tagged by the researcher.']", "label": 0 }, { "id": "train_800", "context": "A diet high in saturated fats increases a person' s risk of developing heart disease. Regular consumption of red wine reduces that risk. Per-capita consumption of saturated fats is currently about the same in France as in the United States, but there is less heart disease there than in the United States because consumption of red wine is higher in France. The difference in regular red-wine consumption has been narrowing, but no similar convergence in heart-disease rates has occurred.", "question": "Which of the following, if true, most helps to account for the lack of convergence noted above?", "answers": "['Regular consumption of red wine is declining dramatically among young adults in France, and heart disease typically does not manifest itself until middle age.', 'Consumption of saturated fats is related more strongly to the growth of fatty deposits on artery walls, which reduce blood flow to the heart, than it is to heart disease directly.', 'Cigarette smoking, which can also contribute to heart disease, is only slightly more common in France than in the United States.', 'Over the past 30 years, per-capita consumption of saturated fats has remained essentially unchanged in the United States but has increased somewhat in France.']", "label": 0 }, { "id": "train_801", "context": "Many uses have been claimed for hypnosis, from combating drug addiction to overcoming common phobias. A recent experimental study helps illuminate the supposed connection between hypnosis and increased power of recall. A number of subjects listened to a long, unfamiliar piece of instrumental music. Under subsequent hypnosis, half the subjects were asked to recall salient passages from the musical piece and half were asked to describe scenes from \"the film they had just viewed, \" despite their not having just seen a film. The study found that the subjects in the second group were equally confident and detailed in their movie recollections as the subjects in the first group were in their music recollections.", "question": "Which one of the following statements is most supported by the information above?", "answers": "['Recalling events under hypnosis inevitably results in false memories.', 'What people recall under hypnosis depends to at least some extent on suggestion.', 'Visual memory is enhanced more by hypnosis than is auditory memory.', \"Hypnosis cannot significantly increase a person's power of recall.\"]", "label": 1 }, { "id": "train_802", "context": "One of the advantages of Bacillus thuringiensis (B. t. ) toxins over chemical insecticides results from their specificity for pest insects. The toxins have no known detrimental effects on mammals or birds. In addition, the limited range of activity of the toxins toward insects means that often a particular toxin will kill pest species but not affect insects that prey upon the species. This advantage makes B. t. toxins preferable to chemical insecticides for use as components of insect pest management programs.", "question": "Which one of the following statements, if true, most weakens the argument?", "answers": "['Chemical insecticides cause harm to a greater number of insect species than do B. t. toxins.', 'No particular B. t. toxin is effective against all insects.', 'B. t. toxins do not harm weeds that do damage to farm crops.', 'Insects build up resistance more readily to B. t. toxins than to chemical insecticides.']", "label": 3 }, { "id": "train_803", "context": "Editorial: Our political discussions tend to focus largely on the flaws of our nation' s leaders, but we need to remind ourselves that these leaders were chosen democratically. The real question that needs answering is how our nation' s institutions and procedures enable such people to attain positions of power. Thus, to focus our attention on the flaws of our leaders is to indulge in a pointless distraction.", "question": "Which one of the following is an assumption that the argument requires?", "answers": [ "As yet, no one in the nation has made the effort to critically examine the details of the nation's institutions and procedures.", "Concentrating on the flaws of the nation's leaders creates greater dissatisfaction with those leaders.", "Examining an individual leader's personal flaws does not reveal anything about how the nation's institutions and procedures influence the selection of leaders.", "Political discussions that focus on the flaws of the nation's leaders will become even more common if the nation's institutions and procedures are not examined." ], "label": 2 }, { "id": "train_804", "context": "An analysis of the language in social media messages posted via the Internet determined that, on average, the use of words associated with positive moods is common in the morning, decreases gradually to a low point midaftemoon, and then increases sharply throughout the evening. This shows that a person' s mood typically starts out happy in the morning, declines during the day, and improves in the evening.", "question": "The reasoning in the argument is most vulnerable to criticism on the grounds that the argument overlooks the possibility that", "answers": "['most of the social media messages posted in the evening are posted by people who rarely post such messages in the morning', 'the number of social media messages posted in the morning is not significantly different from the number posted in the evening', 'the frequency in the use of words in social media is not necessarily indicative of the frequency of the use of those words in other forms of communication', \"people's overall moods are lowest at the beginning of the workweek and rise later, peaking on the weekend\"]", "label": 0 }, { "id": "train_805", "context": "According to a review of 61 studies of patients suffering from severely debilitating depression, a large majority of the patients reported that missing a night' s sleep immediately lifted their depression. Yet sleep-deprivation is not used to treat depression even though the conventional treatments, which use drugs and electric shocks, often have serious side effects.", "question": "Which of the following, if true, best explains the fact that sleep-deprivation is not used as a treatment for depression?", "answers": "['The dramatic shifts in mood connected with sleep and wakefulness have not been traced to particular changes in brain chemistry.', 'Keeping depressed patients awake is more difficult than keeping awake people who are not depressed.', 'Depression returns in full force as soon as the patient sleeps for even a few minutes.', \"For a small percentage of depressed patients, missing a night's sleep induces a temporary sense of euphoria.\"]", "label": 2 }, { "id": "train_806", "context": "In mountainous regions, the timberline is the highest altitude at which trees grow. In the Rocky Mountains, the current timberline is at the altitude above which growing season temperatures remain cooler than 10 degrees centigrade. Fossilized remains of trees that grew 10, 000 years ago have been found 100 meters above the current Rocky Mountain timberline. Clearly, ttherefore, the climate of the Rocky Mountains is cooler now than it was 10, 000 years ago.", "question": "Which of the following is an assumption on which the argument relies?", "answers": "[\"In the past 10, 000 years, the only trees to have grown above today's timberline are the trees whose fossilized remains been found.\", 'The climate of the Rocky Mountains has never been significantly warmer than during the lifetime of the trees whose fossilized remains have been found.', 'The fossils are not of species of trees that were able to tolerate cooler growing temperatures than are the species that currently grow near the timberline.', 'No trees grew 10, 000 years ago at altitudes higher than the ones at which fossilized tree remains have been found.']", "label": 2 }, { "id": "train_807", "context": "Proposal: Carbon dioxide and methane in the atmosphere block the escape of heat into space. So emission of these \"greenhouse\" gases contributes to global warming. In order to reduce global warming, emission of greenhouse gases needs to be reduced. Ttherefore, the methane now emitted from open landfills should instead be burned to produce electricity. Objection: The burning of methane generates carbon dioxide that is released into the atmosphere.", "question": "Which of the following, if true, most adequately counters the objection made to the proposal?", "answers": "['The conversion of methane to electricity would occur at a considerable distance from the landfills.', 'The methane that is used to generate electricity would generally be used as a substitute for a fuel that does not produce any greenhouse gases when burned.', 'Every time a human being or other mammal exhales, there is some carbon dioxide released into the air.', 'Methane in the atmosphere is more effective in blocking the escape of heat from the Earth than is carbon dioxide.']", "label": 3 }, { "id": "train_808", "context": "Loss of the Gocha mangrove forests has caused coastal erosion, reducing fish populations and requiring the Gocha Fishing Cooperative (GFC) to partially fund dredging and new shore facilities. However, as part of its subsidiary businesses, the GFC has now invested in a program to replant significant parts of the coast with mangrove trees. Given income from a controlled harvest of wood with continuing replanting, the mangrove regeneration effort makes it more likely that the cooperative will increase its net income.", "question": "Which of the following, if true, would most strengthen the argument that mangrove replanting will increase the Gocha cooperative's net income?", "answers": "['A controlled harvesting of mangrove wood by the GFC would have little effect on coastal erosion.', 'The GFC derives 10 percent of its revenue from salt-production facilities in an area previously cleared of mangroves.', 'Mangrove forests tend to increase the commercial fish populations in coastal fishing grounds.', 'The GFC will be able to hire local workers to assist with the mangrove replanting.']", "label": 2 }, { "id": "train_809", "context": "Paleontologist: It is widely, but falsely, held that life began in the ocean and did not exist on land until half a billion years ago. Traces of carbon 14 have been found throughout certain 1. 2-billion-year-old rock in North America. Carbon 14 is extracted by plants and microbes from the atmosphere to fuel their chemical reactions, and is then released when the plants and microbes die.", "question": "Each of the following, if true, provides additional support for the paleontologist's conclusion EXCEPT:", "answers": "['The 1. 2-billion-year-old rocks were formed from soil containing very small amounts of carbon 14 that entered the soil directly from the atmosphere.', 'Research suggests that some portions of the 1. 2-billion-year-old rocks were at one time submerged in water, though portions clearly never were.', 'The severity of conditions in the primordial oceans would have made it difficult for life to begin there.', 'Uranium testing has been used to confirm that the age of the rocks is 1. 2 billion years.']", "label": 0 }, { "id": "train_810", "context": "A tax preparation company automatically adds the following disclaimer to every e-mail message sent to its clients: \"Any tax advice in this e-mail should not be construed as advocating any violation of the provisions of the tax code. \" The only purpose this disclaimer could serve is to provide legal protection for the company. But if the e-mail elsewhere suggests that the client do something illegal, then the disclaimer offers no legal protection. So the disclaimer serves no purpose.", "question": "The argument's conclusion can be properly drawn if which one of the following is assumed?", "answers": "['If the e-mail does not elsewhere suggest that the client do anything illegal, then the company does not need legal protection.', \"Some of the tax preparation company's clients would try to illegally evade penalties if they knew how to do so.\", \"At least some of the recipients of the company's e-mails will follow the advice contained in the body of at least some of the e-mails they receive.\", 'A disclaimer that is included in every e-mail message sent by a company will tend to be ignored by recipients who have already received many e-mails from that company.']", "label": 0 }, { "id": "train_811", "context": "One can be at home and be in the backyard, that is, not in one' s house at all. One can also be in one' s house but not at home, if one owns the house but rents it out to others, for example. So one' s being at home is not required for one' s being in one' s own house.", "question": "Which one of the following most accurately describes the relationship between the argument's conclusion and its claim that one can be at home without being in one's own house?", "answers": "['The claim inadvertently contradicts the conclusion.', 'The claim points out an ambiguity in the phrase \"at home. \"', 'The claim is compatible with the truth or falsity of the conclusion.', 'The claim is required to establish the conclusion.']", "label": 2 }, { "id": "train_812", "context": "When feeding, aquatic birds known as phalaropes often spin rapidly on the water' s surface, pecking for food during each revolution. To execute these spins, phalaropes kick one leg harder than the other. This action creates upwelling currents. Because plankton on which phalaropes feed typically occurs in greater quantities well below the surface, it is hypothesized that by spinning phalaropes gain access to food that would otherwise be beyond their reach.", "question": "Which of the following, if true, most strongly supports the hypothesis?", "answers": "['Phalaropes rarely feed while on land.', 'Phalaropes do not usually spin when food is abundant at the surface.', 'Phalaropes sometimes spin when they are not feeding.', 'A given phalarope spins exclusively either to the right or to the left.']", "label": 1 }, { "id": "train_813", "context": "This company will not be training any more pilots in the foreseeable future, since we have 400 trained pilots on our waiting list who are seeking employment. The other five major companies each have roughly the same number of trained pilots on their waiting lists, and since the projected requirement of each company is for not many more than 100 additional pilots, there will be no shortage of personnel despite the current upswing in the aviation industry. ", "question": "Which one of the following, if true, casts the most doubt on the accuracy of the above conclusion?", "answers": "['Most of the trained pilots who are on a waiting list for a job are on the waiting lists of all the major companies.', 'Some of the other major companies are still training pilots but with no presumption of subsequent employment.', 'The quoted personnel projections take account of the current upswing in the aviation industry.', 'In the long run, pilot training will become necessary to compensate for ordinary attention.']", "label": 0 }, { "id": "train_814", "context": "Book collector: The demand for out-of-print books is increasing. It has been spurred by the rise of the Internet, the search capabilities of which make it much easier to locate the out-of-print books one seeks.", "question": "The book collector's statements, if true, most strongly support which one of the following?", "answers": "['Book collectors are now using the Internet to find book titles that they previously did not know existed.', 'Only people who have access to the Internet can locate out-of-print books.', 'The amount of demand for out-of-print books is affected by the ease of finding such books.', 'Fewer people try to find books that are in print than try to find books that are out of print.']", "label": 2 }, { "id": "train_815", "context": "Government official: Residents who are foreign citizens can serve as public servants at most levels, but not as cabinet secretaries. This is wise, since cabinet secretaries perform some duties that should be performed only by citizens, and no one should be appointed to a position if it involves duties that person should not perform. Moreover, a cabinet undersecretary is expected to serve as cabinet secretary when the actual secretary is unavailable. So, __.", "question": "Which one of the following most logically completes the government official's statement?", "answers": "['cabinet undersecretaries should not be expected to stand in for cabinet secretaries', 'only former cabinet undersecretaries should be appointed as cabinet secretaries', 'foreign citizens should not be appointed as cabinet undersecretaries', 'foreign citizens who serve as public servants should be granted citizenship in the country they serve']", "label": 2 }, { "id": "train_816", "context": "Traditional hatcheries raise fish in featureless environments and subject them to dull routines, whereas new, experimental hatcheries raise fish in visually stimulating environments with varied routines. When released into the wild, fish from the experimental hatcheries are bolder than those from traditional hatcheries in exploring new environments and trying new types of food. Fish raised in the experimental hatcheries, ttherefore, are more likely to survive after their release.", "question": "Which one of the following is an assumption required by the argument?", "answers": "[\"The quality of the environments into which hatchery-raised fish are released has little effect on the fish's survival rate.\", 'Some fish raised in traditional hatcheries die because they are too timid in their foraging for food.', 'It is economically feasible for hatchery operators to expose fish to greater visual stimulation and to more varied routines.', 'Fish in the wild always live in visually stimulating environments.']", "label": 1 }, { "id": "train_817", "context": "Since multinational grain companies operate so as to maximize profits, they cannot be relied on to initiate economic changes that would reform the world' s food-distribution system. Although it is true that the actions of multinational companies sometimes do result in such economic change, this result is incidental, arising not from the desire for reform but from the desire to maximize profits. The maximization of profits normally depends on a stable economic environment, one that discourages change.", "question": "The main point of the argument is that", "answers": "['multinational grain companies operate so as to maximize profits', \"multinational grain companies cannot be relied on to initiate reform of the world's food-distribution system\", 'the maximization of profits depends on a stable economic environment', 'when economic change accompanies business activity, that change is initiated by concern for the profit motive']", "label": 1 }, { "id": "train_818", "context": "Tanya is a lawyer. Nearly all lawyers dutifully represent their clients' best interests, but a few unethical ones charge exorbitant and fraudulent fees for services. Some lawyers become millionaires, while others work in the best interest of the public. However, all lawyers are bound by extensive ethical codes, which vary slightly by jurisdiction.", "question": "If the statements above are true, which one of the following must also be true?", "answers": "['Tanya works for the public sector.', 'Tanya charges exorbitant fees for her services.', \"Tanya dutifully represents her clients' best interests.\", 'Tanya is bound by extensive ethical codes.']", "label": 3 }, { "id": "train_819", "context": "Critic: Works of literature often present protagonists who scorn allegiance to their society and who advocate detachment rather than civic-mindedness. However, modern literature is distinguished from the literature of earlier eras in part because it more frequently treats such protagonists sympathetically. Sympathetic treatment of such characters suggests to readers that one should be unconcerned about contributing to societal good. Thus, modern literature can damage individuals who appropriate this attitude, as well as damage society at large.", "question": "Which one of the following is an assumption on which the critic's argument relies?", "answers": "['Some individuals must believe that their society is better than most before they can become concerned with benefiting it.', 'Modern literature is generally not as conducive to societal good as was the literature of earlier eras.', 'It is to the advantage of some individuals that they be concerned with contributing to societal good.', 'Some individuals in earlier eras were more concerned about contributing to societal good than is any modern individual.']", "label": 2 }, { "id": "train_820", "context": "A study of adults who suffer from migraine headaches revealed that a significant proportion of the study participants suffer from a complex syndrome characterized by a set of three symptoms. Those who suffer from the syndrome experienced excessive anxiety during early childhood. As adolescents, these people began experiencing migraine headaches. As these people approached the age of 20, they also began to experience recurring bouts of depression. Since this pattern is invariant, always with excessive anxiety at its beginning, it follows that excessive anxiety in childhood is one of the causes of migraine headaches and depression in later life.", "question": "The reasoning in the argument is vulnerable to criticism on which one of the following grounds?", "answers": "['It fails to rule out the possibility that all of the characteristic symptoms of the syndrome have a common cause.', 'It does not specify the proportion of those in the general population who suffer from the syndrome.', 'It does not establish why the study of migraine sufferers was restricted to adult participants.', 'It makes a generalization that is inconsistent with the evidence.']", "label": 0 }, { "id": "train_821", "context": "Dietitian: Eating fish can lower one' s cholesterol level. In a study of cholesterol levels and diet, two groups were studied. The first group ate a balanced diet including two servings of fish per week. The second group ate a very similar diet, but ate no fish. The first group showed lower cholesterol levels, on average, than the second group. The two groups had displayed similar average cholesterol levels prior to the study.", "question": "Which one of the following most accurately describes the role played in the dietitian's argument by the claim that the two groups had displayed similar average cholesterol levels prior to the study?", "answers": "['It expresses the main conclusion of the argument.', 'It provides background information on the purpose of the study.', 'It introduces an alternative explanation of the phenomenon described in the main conclusion.', 'It rules out an alternative explanation of the data collected in the study.']", "label": 3 }, { "id": "train_822", "context": "A study of kindergarten students has shown that children from households that own many books, an average of 54 books, show average or high reading achievement levels, while the households of children with lower reading achievement levels tend to own fewer books, 5 books on average.", "question": "What additional information would strengthen the assertion that parents who don't buy children's books to keep in the home cause their children to fall behind their peers in reading skills?", "answers": "['Children whose parents buy them books feel happier about their lives.', 'The non-school hours of every child in the study were entirely filled with sports activities, television watching, and playing with friends.', 'When children have books of their own, they develop more advanced reading skills because adults are more likely to read to them regularly.', 'Many of the books in the homes of the high achieving readers from the study were advanced science textbooks that were stored out of reach of the children.']", "label": 2 }, { "id": "train_823", "context": "People in isolated rain-forest communities tend to live on a largely vegetarian diet, and they eat little salt. Few of them suffer from high blood pressure, and their blood pressure does not tend to increase with age, as is common in industrialized countries. Such people often do develop high blood pressure when they move to cities and adopt high-salt diets. Though suggestive, these facts do not establish salt as the culprit in high blood pressure, however, because__.", "question": "Which of the following most logically completes the argument given below?", "answers": "['it is possible to have a low-salt diet while living in a city in an industrialized country', 'genetic factors could account for the lack of increase of blood pressure with age among such people', 'there are changes in other aspects of diet when such people move to the city', 'salt is a necessity for human life, and death can occur when the body loses too much salt']", "label": 2 }, { "id": "train_824", "context": "Novice bird-watcher: I don' t know much about animal tracks, but I do know that birds typically have four toes, and most birds have three toes pointing forward and one toe pointing backward. Since this track was made by an animal with four toes, of which three point forward and one points backward, we can conclude it was made by some kind of bird.", "question": "The argument is flawed because it", "answers": "['relies on the vagueness of the term \"track\"', 'depends on evidence about an individual bird rather than about birds in general', 'does not define birds as animals with four toes', 'does not establish that only a bird could have made the track']", "label": 3 }, { "id": "train_825", "context": "The higher the average cholesterol intake among the residents of a nation, the higher the incidence of heart disease in that nation; the lower the average cholesterol intake, the lower the incidence of heart disease. So people who want to decrease their risk of heart disease should reduce their cholesterol intake.", "question": "Which one of the following, if true, most weakens the argument?", "answers": "['Any individual resident of a nation whose population has a high average cholesterol intake may have a diet low in cholesterol.', 'The nations with a high average cholesterol intake tend to be among the most impoverished in the world.', 'The nations with high average cholesterol intake are also the countries with the highest levels of unemployment.', 'Individual residents of high cholesterol nations who consume low cholesterol diets have the same rate of heart disease as the national average.']", "label": 3 }, { "id": "train_826", "context": "Does the position of a car driver' s seat have a significant impact on driving safety? It probably does. Driving position affects both comfort and the ability to see the road clearly. A driver who is uncomfortable eventually becomes fatigued, which makes it difficult to concentrate on the road. Likewise, the better the visibility from the driver' s seat, the more aware the driver can be of road conditions and other vehicles.", "question": "Which one of the following most accurately describes the role played in the argument by the claim that driving position affects both comfort and the ability to see the road clearly?", "answers": "['It is a claim that the argument shows to be inconsistent with available evidence.', 'It is a premise offered in support of the conclusion drawn in the argument.', 'It is the conclusion drawn in the argument.', 'It is used to provide a causal explanation for an observed phenomenon.']", "label": 1 }, { "id": "train_827", "context": "Naturalist: For several years, coyote have had a robust population throughout the Susquehanna National Forest. Over the past seven years, the Canic Chemical Corporation in nearby Harrisville has been releasing low levels of bromide compounds into the air and groundwater. Some of these compounds, in sufficient concentrations, are suspected of compromising the immune systems of young coyote. A recent study over the entire forest established that 30% of young coyote did not survive their first year of life. Clearly, the bromide compounds released by Canic pose a direct threat to the well-being of coyote in the forest.", "question": "The answer to which of the following would be most useful for evaluating the naturalist's reasoning?", "answers": "['How many of the natural springs in the forest derive from groundwater that flows in the vicinity of Harrisville?', 'What naturally available nutrients would strengthen the immune system of young coyote against any effects of these bromide compounds?', 'What percentage of healthy coyote survive to adulthood in the wild?', 'Have significant levels of these bromide compounds been measured in the blood of other mammals in the forest?']", "label": 2 }, { "id": "train_828", "context": "The Iliad and the Odyssey were both attributed to Homer in ancient times. But these two poems differ greatly in tone and vocabulary and in certain details of the fictional world they depict. So they are almost certainly not the work of the same poet.", "question": "Which one of the following statements, if true, most weakens the reasoning above?", "answers": "['Both the Iliad and the Odyssey were the result of an extended process of oral composition in which many poets were involved.', 'Works known to have been written by the same modern writer are as different from each other in the respects mentioned as are the Iliad and the Odyssey.', 'Several hymns that were also attributed to Homer in ancient times differ more from the Iliad in the respects mentioned than does the Odyssey.', 'Both the Iliad and the Odyssey have come down to us in manuscripts that have suffered from minor copying errors and other textual corruptions.']", "label": 1 }, { "id": "train_829", "context": "Linguist: One group of art critics claims that postimpressionist paintings are not really art and so should be neither studied nor displayed. Another group of critics disagrees, insisting that these paintings are works of art. But since the second group grants that there are paintings that are not works of art and should ttherefore be ignored in the manner suggested by the first group, their disagreement is not over the meaning of the word \"art. ", "question": "The claim that there are paintings that are not works of art plays which one of the following roles in the linguist's argument?", "answers": "['It is a contention that the argument purports to show is the main point of disagreement between the two groups of critics mentioned.', 'It is a claim whose acceptance by critics who differ on other issues is cited by the argument as evidence of its truth.', 'It is a claim whose acceptance by both of the two disputing parties is cited as evidence for a conclusion the argument draws about the disagreement.', 'It is cited as a commonly accepted reason for accepting a hypothesis for which the argument offers independent evidence.']", "label": 2 }, { "id": "train_830", "context": "A work of architecture, if it is to be both inviting and functional for public use, must be unobtrusive, taking second place to the total environment. Modern architects, plagued by egoism, have violated this precept. They have let their strong personalities take over their work, producing buildings that are not functional for public use.", "question": "Which one of the statements below follows logically from the statements in the passage?", "answers": "['An architect with a strong personality cannot produce buildings that function well for the public.', 'Unobtrusive architecture is both inviting and functional.', \"A work of architecture cannot simultaneously express its architect's personality and be functional for public use.\", 'Modern architects who let their strong personalities take over their work produce buildings that are not unobtrusive.']", "label": 3 }, { "id": "train_831", "context": "Images from ground-based telescopes are invariably distorted by the Earth's atmosphere. Orbiting space telescopes, however, operating above Earth's atmosphere, should provide superbly detailed images. Ttherefore, ground-based telescopes will soon become obsolete for advanced astronomical research purposes.", "question": "Which of the following statements, if true, would cast the most doubt on the conclusion drawn above?", "answers": "['By careful choice of observatory location, it is possible for large-aperture telescopes to avoid most of the kind of wind turbulence that can distort image quality.', 'Detailed spectral analyses, upon which astronomers rely for determining the chemical composition and evolutionary history of stars, require telescopes with more light-gathering capacity than space telescopes can provide.', 'An orbiting space telescope due to be launched this year is far behind schedule and over budget, whereas the largest ground-based telescope was both within budget and on schedule.', 'Ground-based telescopes located on mountain summits are not subject to the kinds of atmospheric distortion which, at low altitudes, make stars appear to twinkle.']", "label": 1 }, { "id": "train_832", "context": "All the evidence so far gathered fits both Dr. Grippen' s theory and Professor Heissmann' s. However, the predictions that these theories make about the result of the planned experiment cannot both be true. Ttherefore, the result of this experiment will confirm one of these theories at the expense of the other.", "question": "The argument above exhibits an erroneous pattern of reasoning most similar to that exhibited by which one of the following?", "answers": "[\"David thinks this tree is a beech. Jane thinks it is an elm. Maria, unlike David or Jane, is expert at tree identification, so when Maria gives her opinion it will verify either David's or Jane's opinion.\", 'David and Jane both think they know how to distinguish beech trees from elms, but when they look at trees together they often disagree. Ttherefore, at least one of them must have an erroneous method.', \"David and Jane have been equally good at identifying trees so far. But David says this one is an elm, whereas Jane is unsure. Ttherefore, if this tree turns out to be an elm, we'll know David is better.\", \"David thinks that there are more beeches than elms in this forest. Jane thinks he is wrong. The section of forest we examined was small, but examination of the whole forest would either confirm David's view or disprove it.\"]", "label": 0 }, { "id": "train_833", "context": "Advertisement for a lactase supplement: Lactase, an enzyme produced by the body, aids in the digestion of lactose, a natural sugar found in dairy foods. Many subjects in an experiment who drank a liter of milk on an empty stomach showed signs of lactose intolerance -- difficulty in digesting dairy products because of insufficient lactase. Thus, extrapolating from the number of subjects adversely affected, at least 50 million people in North America alone should take lactase supplements.", "question": "Which one of the following statements, if true, most seriously weakens the argument?", "answers": "['Lactose intolerance can interfere with proper nutrition.', 'Some dairy foods, such as cheese, contain a form of lactose more difficult to digest than that found in milk.', 'Most people who consume dairy products consume less lactose at each meal than the amount found in a liter of milk.', 'Eating solid food when drinking milk can decrease the amount of lactase produced by the body.']", "label": 2 }, { "id": "train_834", "context": "Restaurant manager: In response to requests from our patrons for vegetarian main dishes, we recently introduced three: an eggplant and zucchini casserole with tomatoes, brown rice with mushrooms, and potatoes baked with cheese. The first two are frequently ordered, but no one orders the potato dish, although it costs less than the other two. Clearly, then, our patrons prefer not to eat potatoes.", "question": "Which one of the following is an error of reasoning in the restaurant manager's argument?", "answers": "['ignoring possible differences between what people say they want and what they actually choose', 'treating one of several plausible explanations of a phenomenon as the only possible explanation', 'concluding that two things that occur at the same time have a common cause', 'drawing a conclusion that is inconsistent with one premise of the argument']", "label": 1 }, { "id": "train_835", "context": "The director of a secondary school where many students were having severe academic problems impaneled a committee to study the matter. The committee reported that these students were having academic problems because they spent large amounts of time on school sports and too little time studying. The director then prohibited all students who were having academic problems from taking part in sports in which they were active. He stated that this would ensure that such students would do well academically.", "question": "The reasoning on which the director bases his statement is not sound because he fails to establish that", "answers": "['some students who spend time on sports do not have academic problems', 'no students who do well academically spend time on sports', 'at least some of the time the students will save by not participating in sports will be spent on solving their academic problems', 'all students who do well academically do so because of time saved by not participating in sports']", "label": 2 }, { "id": "train_836", "context": "Midlevel managers at large corporations are unlikely to suggest reductions in staff in their own departments even when these departments are obviously overstaffed.", "question": "Each of the following, if true, supports the claim above EXCEPT:", "answers": "['Staff morale and productivity often suffer when workers are laid off.', 'Midlevel managers have less work to do when their departments are overstaffed.', 'Many large corporations allow managers to offer early retirement as a means of reducing staff.', 'Departmental workloads at most large corporations increase and decrease significantly and unpredictably.']", "label": 2 }, { "id": "train_837", "context": "In Australia the population that is of driving age has grown larger over the last five years, but the annual number of traffic fatalities has declined. This leads to the conclusion that, overall, the driving-age population of Australia consists of more skillful drivers now than five years ago.", "question": "Each of the statements below, if true, weakens the argument EXCEPT:", "answers": "['Five years ago, Australia began a major road repair project.', 'The number of hospital emergency facilities in Australia has doubled in the last five years.', 'Three years ago, a mandatory seat-belt law went into effect throughout Australia.', 'In response to an increase in traffic fatalities, Australia instituted a program of mandatory driver education five years ago.']", "label": 3 }, { "id": "train_838", "context": "Margaret came home after a long day of work to find her front door open and her waste basket knocked over with all of the contents strewn about the kitchen floor. She believes that her dog must have knocked it over in search for some food scraps to eat, but this belief cannot be correct. Her husband took the dog with him to work all day.", "question": "The stated argument requires which of the following assumptions?", "answers": "['Margaret has rats in her house.', \"Margaret's husband accidently knocked it over on his way out the door this morning.\", \"Margaret's husband brought the dog home on his lunch break.\", \"Margaret's husband left the door open that morning and a raccoon got in.\"]", "label": 2 }, { "id": "train_839", "context": "Even a slight change in the timing of this traffic light would cause a traffic backup. On the one hand, if the timing were slower, traffic would back up waiting for the light to turn green. On the other hand, if the timing were faster, traffic would back up because too few cars would get through each green light. Ttherefore, this traffic light was timed to keep traffic from backing up.", "question": "The pattern of reasoning in which one of the following arguments is most similar to that in the argument above?", "answers": "[\"According to the law of supply and demand, even a slight change in the price of a product will affect profits. For instance, if the price goes up, fewer products will be sold; if the price goes down, a company will make less money on each item. Ttherefore, a company's profits will be affected by the law of supply and demand.\", 'Objects in nature show the same complexity as many objects created by humans. If a natural object shows the same complexity as an object created by humans, then it was also the result of design. Ttherefore, objects in nature are the result of design rather than chance.', 'Even a slight change in this diet will result in unhealthy eating. On the one hand, eating larger portions will cause weight gain; on the other hand, eating smaller portions will not provide sufficient nutrients. Ttherefore, the patient ought to follow this diet exactly.', 'Even a slight change in this spaghetti sauce recipe will result in an unhealthful sauce. On the one hand, if too many tomatoes are used, the sauce will be too acidic; on the other hand, if too few tomatoes are used, the sauce will not have enough vitamin C. Ttherefore, this recipe is designed to produce healthful spaghetti sauce.']", "label": 3 }, { "id": "train_840", "context": "Columnist: Although most people favor the bill and the bill does not violate anyone' s basic human rights, it will not be passed for many years, if at all; nor will any similar bill. Those people who would be adversely affected were it to become law are very influential. This shows that, if this country is a democracy at all, it is not a well-functioning one.", "question": "Which one of the following principles, if valid, most helps to justify the columnist's reasoning?", "answers": "[\"In a well-functioning democracy, any bill that would benefit most people will be passed into law within a few years if it does not violate anyone's basic human rights.\", 'In a well-functioning democracy, a bill that is favored by most people will become law within a few years only if those who oppose it are not very influential.', \"A bill that most people favor will be passed promptly into law in a well-functioning democracy if the bill does not violate anyone's basic human rights.\", \"Any bill passed into law in a well-functioning democracy will be favored by most people and be consistent with individuals' basic human rights.\"]", "label": 2 }, { "id": "train_841", "context": "If shoplifting at ToySMart continues at the current rate, the shop' s owner will be forced to close the store. Raising the prices of the goods sold at ToySMart by 10% will help make up the shortfall in the store' s income; this change will ttherefore protect ToySMart from having to close.", "question": "Which of the following is an assumption made in drawing the conclusion above?", "answers": "[\"Visitors to the store will be more likely to consider purchasing more of the shop's less-expensive products.\", 'The rise in prices will not induce visitors who otherwise would not have shoplifted to shoplift items they can now not afford.', 'The rise in prices will have no effect on the number of goods sold, even if the rate of shoplifting remains the same.', 'Other stores may experience an increase in shoplifting as the practice is reduced at ToySMart.']", "label": 1 }, { "id": "train_842", "context": "To increase the productivity of its attorneys, X law firm has decided to implement a part-time program, which would allow attorneys to have more flexibility in the number of days per week they work. Research has shown that giving employees the option of working part time is associated with increased company loyalty.", "question": "The argument depends on the assumption that", "answers": "['an increase in productivity is always signaled by a change in the number of days per week worked by employees', 'an increase in company loyalty may lead to an increase in productivity in the attorneys', 'all law firms will experience increased company loyalty by implementing a parttime program', 'attorneys are unable to fulfill their duties and maintain company loyalty']", "label": 1 }, { "id": "train_843", "context": "Factory manager: One reason the automobile parts this factory produces are expensive is that our manufacturing equipment is outdated and inefficient. Our products would be more competitively priced if we were to refurbish the factory completely with new, more efficient equipment. Ttherefore, since to survive in today' s market we have to make our products more competitively priced, we must completely refurbish the factory in order to survive.", "question": "The reasoning in the factory manager's argument is flawed because this argument", "answers": "['shifts without justification from treating something as one way of achieving a goal to treating it as the only way of achieving that goal', 'fails to recognize that the price of a particular commodity can change over time', 'recommends a solution to a problem without first considering any possible causes of that problem', 'fails to make a definite recommendation and instead merely suggests that some possible course of action might be effective']", "label": 0 }, { "id": "train_844", "context": "Guidebook writer: I have visited hotels throughout the country and have noticed that in those built before 1930 the quality of the original carpentry work is generally superior to that in hotels built afterward. Clearly carpenters working on hotels before 1930 typically worked with more skill, care, and effort than carpenters who have worked on hotels built subsequently.", "question": "Which of the following, if true, most seriously weakens the guidebook writer's argument?", "answers": "['The better the quality of original carpentry in a building, the less likely that building is to fall into disuse and be demolished.', 'The quality of original carpentry in hotels is generally far superior to the quality of original carpentry in other structures, such as houses and stores.', 'The materials available to carpenters working before 1930 were not significantly different in quality from the materials available to carpenters working after 1930.', 'The average length of apprenticeship for carpenters has declined significantly since 1930.']", "label": 0 }, { "id": "train_845", "context": "A proposed amendment would allow the city council to decide that certain city elections be conducted solely by mail. But voting is a sacred right in democracies, one that has always been exercised by voting in person and not by mail. Ttherefore, voting by mail should not be allowed, and the proposed amendment should be rejected.", "question": "The reasoning in the argument is most vulnerable to criticism on the grounds that the argument", "answers": "['presumes, without providing justification, that if citizens have always had a certain legal right, they will continue to have that right in the future', 'presents an appeal to tradition as the only reason for rejecting the proposal', 'presumes, without providing justification, that the right to vote is not violated unless elections are conducted solely by mail', 'fails to consider the possibility that, even if it gains the power to do so, the city council might never require voting by mail']", "label": 1 }, { "id": "train_846", "context": "Wife: The work of the artist who painted the portrait of my grandparents 50 years ago has become quite popular lately, so the portrait has recently become valuable. But since these sorts of artistic fads fade rapidly, the practical thing to do would be to sell the portrait while it is still worth something, and thereby enable our daughter to attend the college she has chosen. Husband: How could you make such a suggestion? That painting is the only thing you own that belonged to your grandparents. I don' t think it' s a very good painting, but it has great sentimental value. Besides, you owe it to our daughter to keep it in the family as a link to her family' s past.", "question": "Which one of the following principles, if established, does most to justify the husband's reply?", "answers": "['Gifts offered as sentimental tokens of affection should not be accepted if the recipient intends to sell them later for profit.', \"Providing one's children with an education is more important than providing them with material goods, even if the goods have sentimental value.\", \"It is more important for parents to provide their children with tangible links to the family's past than it is to enable them to attend the college of their choice.\", 'Children and grandchildren have a duty to preserve family heirlooms only if they have promised their parents or grandparents that they would do so.']", "label": 2 }, { "id": "train_847", "context": "Last year a record number of new manufacturing jobs were created. Will this year bring another record? Well, any new manufacturing job is created either within an existing company or by the start-up of a new company . Within existing firms, new jobs have been created this year at well below last year' s record pace. At the same time, there is considerable evidence that the number of new companies starting up this year will be no higher than it was last year and there is no reason to think that the new companies starting up this year will create more jobs per company than did last year' s start-ups . So clearly, the number of new jobs created this year will fall short of last year' s record.", "question": "In the argument given, the two portions in boldface play which of the following roles?", "answers": "['The first is a generalization that the argument seeks to establish; the second is a judgment that has been advanced in order to challenge that generalization.', 'The first is presented as obvious truth on which the argument is based; the second is a judgment advanced in support of the main conclusion of the argument.', 'The first is presented as obvious truth on which the argument is based; the second is a claim that has been advanced in support of a position that the argument opposes.', 'The first provides evidence in support of the main conclusion of the argument; the second is a claim that argument challenges.']", "label": 1 }, { "id": "train_848", "context": "Baking for winter holidays is a tradition that may have a sound medical basis. In midwinter, when days are short, many people suffer from a specific type of seasonal depression caused by lack of sunlight. Carbohydrates, both sugars and starches, boost the brain' s levels of serotonin, a neurotransmitter that improves the mood. In this respect, carbohydrates act on the brain in the same way as some antidepressants. Thus, eating holiday cookies may provide an effective form of self-prescribed medication.", "question": "Which one of the following can be properly inferred from the passage?", "answers": "['Lack of sunlight lowers the level of serotonin in the brain.', 'Seasonal depression is one of the most easily treated forms of depression.', \"Some antidepressants act by changing the brain's level of serotonin.\", 'People are more likely to be depressed in midwinter than at other times of the year.']", "label": 2 }, { "id": "train_849", "context": "Companies that offer \"employer sponsored insurance\" (ESI) pay a portion of employees' health care costs. In the manufacturing sector last year, companies that offered ESI had worker absentee rates 22% lower, on average, than those at companies that did not offer ESI.", "question": "If, on the basis of the evidence above, it is argued that ESI decreases worker absenteeism, which of the following, if true, would most seriously weaken that argument?", "answers": "['In manufacturing firms where ESI is offered, the average productivity is 2% higher than it is in those firms where workers are not covered by an ESI plan.', 'At companies that offer ESI, employees have access to preventative health care such as regular check-ups, routine laboratory tests, and nutrition counseling.', 'Results similar to those cited for the manufacturing sector have been found in other sectors of the economy where ESI is offered.', 'Many firms in the manufacturing sector have improved workplace safety and decreased the occurrence of on-the-job injuries in the last five years, and most of these companies introduced ESI at the same time.']", "label": 3 }, { "id": "train_850", "context": "Professor: Each government should do all that it can to improve the well-being of all the children in the society it governs. Ttherefore, governments should help finance high-quality day care since such day care will become available to families of all income levels if and only if it is subsidized.", "question": "Which one of the following is an assumption on which the professor's argument depends?", "answers": "['Only governments that subsidize high-quality day care take an interest in the well-being of all the children in the societies they govern.', 'Government is a more efficient provider of certain services than is private enterprise.', 'Government subsidy of high-quality day care would not be so expensive that it would cause a government to eliminate benefits for adults.', 'At least some children would benefit from high-quality day care.']", "label": 3 }, { "id": "train_851", "context": "If squirrels eat from a bird feeder, it will not attract many birds. However, squirrels eat from a bird feeder only if it lacks a protective cover. So a bird feeder will not attract many birds if it does not have a protective cover.", "question": "The flawed pattern of reasoning in the argument above is most similar to that in which one of the following arguments?", "answers": "[\"If a tire's pressure is too low, the tire will wear out prematurely. But tire pressure will become too low only if the car owner neglects to check the pressure regularly. So a tire will wear out prematurely if the car owner neglects to check the pressure regularly.\", \"If a tire's pressure is too low, the tire will wear out prematurely. But tire pressure will become too low if the car owner neglects to check the pressure regularly. Ttherefore, if the car owner neglects to check the pressure regularly, a tire will wear out prematurely.\", \"If a tire's pressure is too low, the tire will wear out prematurely, and if a tire wears out prematurely, a likely cause is that the pressure was too low. So if a car owner checks the tire pressure regularly, the tires will not wear out prematurely.\", 'Tires wear out prematurely if car owners neglect to check the tire pressure regularly. Unless car owners are unaware of this fact, they check the tire pressure regularly. So car owners need to be made aware of the consequences of neglecting to check the tire pressure.']", "label": 0 }, { "id": "train_852", "context": "Researchers have noticed that people whose blood shows abnormally low levels of calcium usually have laryngeal polyps, which can permanently damage vocal cords and result in partial or even total loss of voice. In order to reduce the risk of polyps, the researchers recommend a diet high in calcium-rich foods such as dairy and green, leafy vegetables.", "question": "Which of the following, if true, most strongly suggests that following the researchers' recommendation would NOT be effective?", "answers": "['Fresh vegetables are not always available in all seasons.', 'Low levels of calcium can sometimes be remedied with vitamin pills.', 'Laryngeal polyps sometimes disappear without treatment.', 'Laryngeal polyps cause a change in body chemistry that blocks the absorption of calcium.']", "label": 3 }, { "id": "train_853", "context": "Democratic governments are unlikely to take a morally principled stand on controversial issues, because there are almost always groups of people with differing principles who strongly resist any policy that consistently adheres to any particular moral principle. Thus, the compromises that opposition groups force on democratic governments make policies about controversial issues morally arbitrary.", "question": "The reasoning in the argument is most vulnerable to criticism on the grounds that it fails to consider the possibility that", "answers": "['any form of government is more efficient when it is not overly restricted by moral principles', 'other forms of government are no more likely to take a stand on controversial moral issues than are democratic governments', 'democratic governments benefit citizens more by failing to take a stand on controversial issues than by adhering strictly to moral principles', 'democratic governments appeal to moral principles in effecting compromises between those with opposing moral principles']", "label": 3 }, { "id": "train_854", "context": "Small experimental vacuum tubes can operate in heat that makes semiconductor components fail. Any component whose resistance to heat is greater than that of semiconductors would be preferable for use in digital circuits, but only if that component were also comparable to semiconductors in all other significant respects, such as maximum current capacity. However, vacuum tubes' maximum current capacity is presently not comparable to that of semiconductors.", "question": "If the statements above are true, which one of the following must also be true?", "answers": "['Vacuum tubes are not now preferable to semiconductors for use in digital circuits.', 'Resistance to heat is the only advantage that vacuum tubes have over semiconductors.', 'Semiconductors will always be preferable to vacuum tubes for use in many applications other than digital circuits.', 'Once vacuum tubes and semiconductors have comparable maximum current capacity, vacuum tubes will be used in some digital circuits.']", "label": 0 }, { "id": "train_855", "context": "Global ecological problems reduce to the problem of balancing supply and demand. Supply is strictly confined by the earth' s limitations. Demand, however, is essentially unlimited, as there are no limits on the potential demands made by humans. The natural tendency for there to be an imbalance between demand and sustainable supply is the source of these global problems. Ttherefore, any solutions require reducing current human demand.", "question": "Which one of the following is an assumption on which the argument depends?", "answers": "['Human consumption does not decrease the environmental supply.', \"Actual human demand exceeds the earth's sustainable supply.\", \"It is possible to determine the limitations of the earth's sustainable supply.\", 'Supply and demand tend to balance themselves in the long run.']", "label": 1 }, { "id": "train_856", "context": "A study of rabbits in the 1940s convinced many biologists that parthenogenesis -- reproduction without fertilization of an egg -- sometimes occurs in mammals. However, the study' s methods have since been shown to be flawed, and no other studies have succeeded in demonstrating mammalian parthenogenesis. Thus, since parthenogenesis is known to occur in a wide variety of nonmammalian vertebrates, there must be something about mammalian chromosomes that precludes the possibility of parthenogenesis.", "question": "A flaw in the reasoning of the argument is that the argument", "answers": "['infers that a characteristic is shared by all nonmammalian vertebrate species merely because it is shared by some nonmammalian vertebrate species', 'confuses a necessary condition for parthenogenesis with a sufficient condition for it', 'takes for granted that something that has not been proven to be true is for that reason shown to be false', \"assumes that the methods used in a study of one mammalian species were flawed merely because the study's findings cannot be generalized to all other mammalian species\"]", "label": 2 }, { "id": "train_857", "context": "Sonya: The government of Copeland is raising the cigarette tax. Copeland' s cigarette prices will still be reasonably low, so cigarette consumption will probably not be affected much. Consequently, government revenue from the tax will increase. Raoul: True, smoking is unlikely to decrease, because Copeland' s cigarette prices will still not be high. They will, however, no longer be the lowest in the region, so we might begin to see substantial illegal sales of smuggled cigarettes in Copeland.", "question": "Raoul responds to Sonya's argument by doing which of the following?", "answers": "[\"Arguing that Sonya's conclusion would be better supported if Sonya could cite a precedent for what she predicts will happen\", 'Pointing out that a certain initiative is not bold enough to have the effect that Sonya Predicts it will have', \"Calling Sonya's conclusion into question by pointing to a possible effect of a certain change\", \"Questioning the support for Sonya's conclusion by distinguishing carefully between No change and no decrease\"]", "label": 2 }, { "id": "train_858", "context": "Doctor: Angiotensinogen is a protein in human blood. Typically, the higher a person' s angiotensinogen levels are, the higher that person' s blood pressure is. Disease X usually causes an increase in angiotensinogen levels. Ttherefore, disease X must be a cause of high blood pressure.", "question": "The doctor 's argument is most vulnerable to criticism on which one of the following grounds?", "answers": "['It overlooks the possibility that even if a condition causally contributes to a given effect, other factors may fully counteract that effect in the presence of that condition.', 'It takes for granted that if one phenomenon often causes a second phenomenon and that second phenomenon often causes a third phenomenon, then the first phenomenon cannot ever be the immediate cause of the third.', 'It illicitly infers, solely on the basis of two phenomena being correlated, that one causally contributes to the other.', 'It confuses a necessary condition for a sufficient condition.']", "label": 2 }, { "id": "train_859", "context": "In an experiment, researchers played a series of musical intervals -- two-note sequences -- to a large, diverse group of six-month-old babies. They found that the babies paid significantly more attention when the intervals were perfect octaves, fifths, or fourths than otherwise. These intervals are prevalent in the musical systems of most cultures around the world. Thus, humans probably have a biological predisposition to pay more attention to those intervals than to others.", "question": "Which one of the following, if true, most strengthens the argument?", "answers": "['Octaves, fifths, and fourths were played more frequently during the experiment than other musical intervals were.', 'Several similar experiments using older children and adults found that these subjects, too, had a general tendency to pay more attention to octaves, fifths, and fourths than to other musical intervals.', 'All of the babies in the experiment had been exposed to music drawn equally from a wide variety of cultures around the world.', 'None of the babies in the experiment had previous exposure to music from any culture.']", "label": 3 }, { "id": "train_860", "context": "Two things are true of all immoral actions. First, if they are performed in public, they offend public sensibilities. Second, they are accompanied by feelings of guilt.", "question": "If all of the statements above are true, then which one of the following must be false?", "answers": "['Immoral actions are wrong solely by virtue of being accompanied by feelings of guilt.', 'Every action performed in public that is accompanied by feelings of guilt is immoral.', 'Some actions that offend public sensibilities if they are performed in public are not accompanied by feelings of guilt.', 'Some immoral actions that are not performed in public are not accompanied by feelings of guilt.']", "label": 3 }, { "id": "train_861", "context": "Psychologist: Satisfaction and happiness with one' s family cannot exist unless families have established strong values of mutual trust, love, and respect. Such bonds are formed and strengthened only through making these key values the basis for all actions within the family. It is obvious then that widespread reliance on movies and electronic media for entertainment has an inherently corrosive effect on one' s family life.", "question": "Which one of the following is an assumption on which the psychiatrist's argument depends?", "answers": "['Families cannot usefully increase their happiness by using electronic media.', 'People who rely on movies and electronic media for entertainment are generally not as happy with their families as those who do not.', 'People who rely on movies and electronic information for entertainment are unable to form strong values of trust, love, and respect with their families.', 'Newspapers and other forms of print media strengthen, rather than weaken, family values.']", "label": 2 }, { "id": "train_862", "context": "The introduction of symbols for numbers is an event lost in prehistory, but the earliest known number symbols, in the form of simple grooves and scratches on bones and stones, date back 20, 000 years or more. Nevertheless, since it was not until 5, 500 years ago that systematic methods for writing numerals were invented, it was only then that any sort of computation became possible.", "question": "Which one of the following is an assumption on which the argument relies?", "answers": "['Grooves and scratches found on bones and stones were all made by people, and none resulted from natural processes.', 'Some kinds of surfaces upon which numeric symbols could have been made in the period before 5, 500 years ago were not used for that purpose.', 'Computation of any sort required a systematic method for writing numerals.', 'Grooves and scratches inscribed on bones and stones do not date back to the time of the earliest people.']", "label": 2 }, { "id": "train_863", "context": "According to ancient records, the first tax that the government of Selea imposed on a basic commodity was a tax of two centima coins on every jar of cooking oil sold in Selea. Tax records show that despite a stable population and strict enforcement of tax laws, revenues from the oil tax declined steeply over the first two years that the tax was in effect.", "question": "Which of the following, if true, most helps to explain the decline in Selean oil-tax revenues?", "answers": "['Two years after implementing the tax on cooking oil, the Selean government began to implement taxes on numerous other basic commodities.', 'After the tax was imposed, Selean merchants began selling cooking oil in larger jars than before.', 'Jars of cooking oil were traditionally bought as wedding gifts in Selea at the time the tax went into effect, and gifts of cooking oil increased after the implementation of the tax.', 'Few Selean households began to produce their own cooking oil after the tax was imposed.']", "label": 1 }, { "id": "train_864", "context": "Astronomer: Most stars are born in groups of thousands, each star in a group forming from the same parent cloud of gas. Each cloud has a unique, homogeneous chemical composition. Ttherefore, whenever two stars have the same chemical composition as each other, they must have originated from the same cloud of gas.", "question": "Which of the following, if true, would most strengthen the astronomer's argument?", "answers": "['Many stars in vastly different parts of the universe are quite similar in their chemical compositions.', 'Clouds of gas of similar or identical chemical composition may be remote from each other.', 'In some groups of stars, not every star originated from the same parent cloud of gas.', 'Whenever a star forms, it inherits the chemical composition of its parent cloud of gas.']", "label": 3 }, { "id": "train_865", "context": "The OLEX Petroleum Company has recently determined that it could cut its refining costs by closing its Grenville refinery and consolidating all refining at its Tasberg refinery. Closing the Grenville refinery, however, would mean the immediate loss of about 1, 200 jobs in the Grenville area. Eventually the lives of more than 10, 000 people would be seriously disrupted. Ttherefore, OLEX' s decision, announced yesterday, to keep Grenville open shows that at OLEX social concerns sometimes outweigh the desire for higher profits.", "question": "Which of the following, if true, most seriously undermines the argument given?", "answers": "['Even though OLEX could consolidate all its refining at the Tasberg plant, doing so at the Grenville plant would not be feasible.', 'Closure of the Grenville refinery would mean compliance, at enormous cost, with demanding local codes regulating the cleanup of abandoned industrial sites.', 'If the Grenville refinery were ever closed and operations at the Tasberg refinery expanded, job openings at Tasberg would to the extent possible be filled with people formerly employed at Grenville.', 'The Tasberg refinery is more favorably situated than the Grenville refinery with respect to the major supply routes for raw petroleum.']", "label": 1 }, { "id": "train_866", "context": "The diet of Heliothis subflexa caterpillars consists entirely of fruit from plants of the genus Physalis. These fruit do not contain linolenic acid, which is necessary to the growth and maturation of many insects other than H. subflexa. Linolenic acid in an insect' s diet is also necessary for the production of a chemical called volicitin. While most caterpillar species have volicitin in their saliva, H. subflexa does not.", "question": "Which one of the following can be properly inferred from the statements above?", "answers": "['No caterpillars other than H. subflexa eat fruit from plants of the genus Physalis.', 'Most species of caterpillar have sources of linolenic acid in their diets.', 'H. subflexa caterpillars synthesize linolenic acid within their bodies.', 'Any caterpillar that has linolenic acid in its diet has volicitin in its saliva.']", "label": 1 }, { "id": "train_867", "context": "Within the earth' s core, which is iron, pressure increases with depth. Because the temperature at which iron melts increases with pressure, the inner core is solid and the outer core is molten. Physicists can determine the melting temperature of iron at any given pressure and the pressure for any given depth in the earth. Ttherefore, the actual temperature at the boundary of the earth' s outer and inner cores-the melting temperature of iron there-can be determined, since __.", "question": "Which of the following most logically completes the argument below?", "answers": [ "nowhere in the earth's core can the temperature be measured directly", "pressures within the earth's outer core are much greater than pressures above the outer core", "the depth beneath the earth's surface of the boundary between the outer and inner cores is known", "the temperatures within the earth's inner core are higher than in the outer core" ], "label": 2 }, { "id": "train_868", "context": "Spokesperson: The claim by the minister of education that teachers' attitudes are causing a decline in student performance is belied by events at any meeting of our Teachers Association. The workshops at these meetings are filled with teachers struggling to find new ways to teach their students. No one, not even a very unsympathetic observer, could detect any negative attitudes toward students.", "question": "Which one of the following most accurately describes a flaw in the spokesperson's argument?", "answers": "[\"The argument focuses on the opponent rather than on the opponent's argument.\", \"The argument takes for granted that only when teachers' attitudes toward students are negative will those attitudes cause a decline in student performance.\", \"The argument presumes, without providing justification, that teachers' attitudes toward students cannot have any effect on student performance.\", 'The argument draws a conclusion that simply restates a claim presented in support of that conclusion.']", "label": 1 }, { "id": "train_869", "context": "An independent audit found no indication of tax avoidance on the part of the firm in the firm' s accounts; ttherefore, no such problem exists.", "question": "The questionable reasoning in the argument above is most closely paralleled by that in which one of the following?", "answers": "['The overall budget for the projects has been exceeded by a large amount; ttherefore, at least one of the projects has exceeded its budget by a large amount.', 'An examination of the index of the book found no listing for the most prominent critic of the theory the book advocates; ttherefore, the book fails to refer to that critic.', 'A survey of schools in the district found no school without a need for building repair; ttherefore, the education provided to students in the district is substandard.', 'A compilation of the best student essays of the year includes no essays on current events; ttherefore, students have become apathetic toward current events.']", "label": 1 }, { "id": "train_870", "context": "Diamond Enterprises is a store in Apisville that sells specialty luxury items. For several years, Diamond reaped substantial profits and was considering building branch stores in nearby counties. Stibium Industries, for several years the single largest employer in Apisville and the surrounding region, abruptly closed its plant last year, causing widespread unemployment. Only a fraction of the former Stibium workers have found new jobs, and many of these at much lower wages. Early this year, Diamond Enterprises has filed for Chapter 11 bankruptcy, citing the closure of Stibium as one of the primary causes.", "question": "Which of the following inferences is best supported by the passage?", "answers": "['Diamond Enterprises would have avoided bankruptcy if it had followed through with the plan to build branch stores during its more prosperous years.', 'After Stibium Industry closed, Diamond Enterprises was the single largest employer in Apisville.', \"Stibium's management team had a corporate account with Diamond Enterprises, and ordered several luxury items used in business meetings and to entertain prospective clients.\", 'The closure of Stibium resulted in a loss of revenue for Diamond Enterprises.']", "label": 3 }, { "id": "train_871", "context": "Societies in which value is measured primarily in financial terms invariably fragment into isolated social units. But since money is not the main measure of value in nonindustrial societies, they must tend in contrast to be socially unified.", "question": "The flawed reasoning in which one of the following is most similar to that in the argument above?", "answers": "['Animals of different genera cannot interbreed. But that does not prove that jackals and wolves cannot interbreed, for they belong to the same genus.', 'Poets frequently convey their thoughts via nonliteral uses of language such as metaphors and analogies. But journalists are not poets, so surely journalists always use language literally.', 'Ecosystems close to the equator usually have more species than those closer to the poles. Thus, the Sahara Desert must contain more species than Siberia does, since the latter is farther from the equator.', 'Insects pass through several stages of maturation: egg, larva, pupa, and adult. Since insects are arthropods, all arthropods probably undergo similar maturation processes.']", "label": 1 }, { "id": "train_872", "context": "Sociologist: Romantics who claim that people are not born evil but may be made evil by the imperfect institutions that they form cannot be right, for they misunderstand the causal relationship between people and their institutions. After all, institutions are merely collections of people.", "question": "Which one of the following principles, if valid, would most help to justify the sociologist's argument?", "answers": "['People should not be overly optimistic in their view of individual human beings.', 'The whole does not determine the properties of the things that compose it.', 'People acting together in institutions can do more good or evil than can people acting individually.', \"A society's institutions are the surest gauge of that society's values.\"]", "label": 1 }, { "id": "train_873", "context": "Escalating worldwide demand for corn has led to a sharp increase in the market price of corn, and corn prices are likely to remain high. Corn is extensive used as feed for livestock, and because profit margins are tight in the livestock business, many farmers are expected to leave the business. With fewer suppliers, meat prices will surely rise. Nonetheless, observers expect an immediate short-term decrease in meat prices.", "question": "Which of the following, if true, most helps to justify the observers' expectation?", "answers": "['Generally, farmers who are squeezed out of the livestock business send their livestock to market much earlier than they otherwise would.', 'The increase in corn prices is due more to a decline in the supply of corn than to a growth in demand for it.', 'Some people who ate meat regularly in the past are converting to diets that include little or no meat.', 'As meat prices rise, the number of livestock producers is likely to rise again.']", "label": 0 }, { "id": "train_874", "context": "People who switch to a low-calorie diet generally do not lose any weight. This is surprising since it is understood that decreasing the calories in one' s diet, while maintaining the same level of activity, causes one to lose weight.", "question": "Which one of the following, if true, most helps to resolve the apparent discrepancy?", "answers": "['Food that are low in calories tend to cost more money.', 'People who switch to low-calorie diets also tend to decrease their level of activity.', 'People who eat a low-calorie diet generally weigh less than people eating a higher calorie diet.', 'Foods that are low in calories generally taste worse than high-calorie foods.']", "label": 1 }, { "id": "train_875", "context": "Plant species differ in that renewed growth in spring can be triggered by day length or by temperature or else by a combination of both. Day length is the same, year after year, for any given date. Ttherefore, any plant species that starts to grow again on widely different dates in different years resumes growth at least in part in response to temperature.", "question": "Which one of the following arguments is most similar in its pattern of reasoning to the argument above?", "answers": "['In Xandia, medical assistant trainees must either complete a formal training course or work for one year under the close supervision of a physician. Since few physicians are willing to act as supervisors, it must be true that most medical assistant trainees in Xandia take the training course.', 'Depending on volume of business, either one or two or three store detectives are needed for adequate protection against shoplifting. Ttherefore, if on any particular day store management has decided that three detectives will be needed, it must be because business that day is expected to be heavy.', 'Some landfills charge garbage companies by volume only, some charge by weight only, and all others use a formula sensitive to both volume and weight. So if at a particular landfill the charges for two particular loads of equal volume dumped on the same day are different, weight must determine, or help determine, charges at that landfill.', 'In the Crawford area, easterly winds mean rain will come and westerly winds mean dry weather will come; winds from other directions do not occur. Ttherefore, since it is currently raining in Crawford, there must be an easterly wind blowing there now.']", "label": 2 }, { "id": "train_876", "context": "University president: Research institutions have an obligation to promote research in any field of theoretical investigation if that research shows some promise of yielding insights into the causes of practical problems that affect people' s quality of life.", "question": "The principle stated by the university president, if valid, most helps to justify which one of the following actions?", "answers": "['A research institute funds an investigation into the mathematical properties of folded structures that is likely to aid in understanding the structure of proteins that cause disease.', 'A university funds a research position in the physics department that involves no teaching but has the responsibility for managing all the grant applications by members of the physics faculty.', 'A foundation decides not to fund a research proposal in applied mathematics that sought to model certain poorly understood aspects of economic behavior.', 'A government agency funds a research project in astrophysics designed to determine whether there are theoretical limits on the magnitude of planets in distant solar systems.']", "label": 0 }, { "id": "train_877", "context": "Scientists are sometimes said to assume that something is not the case until there is proof that it is the case. Now suppose the question arises whether a given food additive is safe. At that point, it would be neither known to be safe nor known not to be safe. By the characterization above, scientists would assume the additive not to be safe because it has not been proven safe. But they would also assume it to be safe because it has not been proven otherwise. But no scientist could assume without contradiction that a given substance is both safe and not safe; so this characterization of scientists is clearly wrong.", "question": "Which one of the following describes the technique of reasoning used above?", "answers": "['A general statement is argued to be false by showing that it has deliberately been formulated to mislead.', 'A statement is shown to be false by showing that it directly contradicts a second statement that is taken to be true.', 'A statement is argued to be false by showing that taking it to be true leads to implausible consequences.', 'A general statement is shown to be uninformative by showing that there are as many specific instances in which it is false as there are instances in which it is true.']", "label": 2 }, { "id": "train_878", "context": "Advertisers are often criticized for their unscrupulous manipulation of people' s tastes and wants. There is evidence, however, that some advertisers are motivated by moral as well as financial considerations. A particular publication decided to change its image from being a family newspaper to concentrating on sex and violence, thus appealing to a different readership. Some advertisers withdrew their advertisements from the publication, and this must have been because they morally disapproved of publishing salacious material.", "question": "Which one of the following, if true, would most strengthen the argument?", "answers": "['Some advertisers switched from family newspapers to advertise in the changed publication.', 'People who generally read family newspapers are not likely to buy newspapers that concentrate on sex and violence.', 'It was expected that the changed publication would appeal principally to those in a different income group.', 'The advertisers expected their product sales to increase if they stayed with the changed publication, but to decrease if they withdrew.']", "label": 3 }, { "id": "train_879", "context": "People cannot be morally responsible for things over which they have no control. Ttherefore, they should not be held morally responsible for any inevitable consequences of such things, either. Determining whether adults have any control over the treatment they are receiving can be difficult. Hence in some cases it can be difficult to know whether adults bear any moral responsibility for the way they are treated. Everyone, however, sometimes acts in ways that are an inevitable consequence of treatment received as an infant, and infants clearly cannot control, and so are not morally responsible for, the treatment they receive.", "question": "Anyone making the claims above would be logically committed to which one of the following further claims?", "answers": "[\"If a given action is within a certain person's control that person should be held morally responsible for the consequences of that action.\", 'An infant should never be held morally responsible for an action that infant has performed.', 'There are certain commonly performed actions for which no one performing those actions should ever be held morally responsible.', 'No adult should be held morally responsible for every action he or she performs.']", "label": 3 }, { "id": "train_880", "context": "Sixty adults were asked to keep a diary of their meals, including what they consumed, when, and in the company of how many people. It was found that at meals with which they drank alcoholic beverages, they consumed about 175 calories more from nonalcoholic sources than they did at meals with which they did not drink alcoholic beverages.", "question": "Each of the following, if true, contributes to an explanation of the difference in caloric intake EXCEPT:", "answers": "['At meals that included alcohol, relatively more of the total calories consumed came from carbohydrates and relatively fewer of them came from fats and proteins.', 'Diners spent a much longer time at meals served with alcohol than they did at those served without alcohol.', 'People eat more when there are more people present at the meal, and more people tended to be present at meals served with alcohol than at meals served without alcohol.', 'The meals that were most carefully prepared and most attractively served tended to be those at which alcoholic beverages were consumed.']", "label": 0 }, { "id": "train_881", "context": "One sure way you can tell how quickly a new idea -- for example, \"globalization\" -- is taking hold among the population is to monitor how fast the word or words expressing that particular idea are passing into common usage. Professional opinions of whether or not words can indeed be said to have passed into common usage are available from dictionary authors, who are acutely aware of these changes.", "question": "The method described above for determining how quickly a new idea is taking hold relies on which one of the following assumptions?", "answers": "['Dictionary authors are not professionally interested in words that are only rarely used.', 'Dictionary authors have an exact numerical calculation for deciding when a word has passed into common usage.', 'For a new idea to take hold, dictionary authors have to include the relevant word or word in their dictionaries.', \"The word's meaning does not undergo any severe distortions in the process of passing into common usage.\"]", "label": 3 }, { "id": "train_882", "context": "Analyst: Any new natural-gas-powered electrical generation station needs to be located close to a natural-gas pipeline, a large body of water for cooling, and transmission lines. It also must be situated in a region where residents will not oppose construction. Our country has an extensive system of transmission lines, but our natural-gas pipelines run in the vicinity of only three of our large bodies of water, and residents would oppose any significant construction projects near these bodies of water.", "question": "The analyst's statements, if true, most strongly support which one of the following statements about the analyst's country?", "answers": "['There currently is no natural-gas-powered electrical generation station near any of the three largest bodies of water.', 'Future electrical needs will have to be met by alternatives to natural-gas-powered generation.', 'If a new natural-gas -powered electrical station is built in a region, many residents will move away from that region.', 'No site would be suitable for constructing a natural-gas-powered electrical station unless the existing system of natural-gas pipelines is expanded.']", "label": 3 }, { "id": "train_883", "context": "Senior Class President: Our school, in suspending a student who verbally challenged his lab partner' s results, has erred by penalizing the student for doing what he surely has a right to do: speak his mind! Superintendent: But what you' re saying is that our school district should endorse verbal harassment. Yet surely if we did that, we would threaten the free flow of ideas that is the core of this school' s academic tradition.", "question": "Which one of the following is a questionable technique that the superintendent uses to refute the class president?", "answers": "['Questioning the motives of the class president as opposed to offering reasons for his own conclusion.', \"Incorrectly describing the class president's position, thereby making it easily to challenge.\", \"Challenging the class president's knowledge of the process by which the student was suspended.\", 'Relying on a position of power to silence the opposing viewpoint through threats.']", "label": 1 }, { "id": "train_884", "context": "A theoretical framework facilitates conceptual organization of material and fruitful expansions of research. Many historians argue that historical analysis is ttherefore done best within a theoretical framework. But the past is too complex for all of its main trends to be captured within a theoretical framework. Ttherefore, __.", "question": "Which one of the following most logically completes the argument?", "answers": "['there is no benefit ever to be gained in recommending to historians that they place their work within a theoretical framework', 'the value of theoretical work in extending research has been emphasized by historians who recommend doing historical analysis within a theoretical framework', 'there is no difference between historical analysis that is placed within a theoretical framework and historical analysis that is not', \"even the best historical analysis done within a theoretical framework fails to capture all of history's main trends\"]", "label": 3 }, { "id": "train_885", "context": "The teeth of some mammals show \"growth rings\" that result from the constant depositing of layers of cementum as opaque bands in summer and translucent bands in winter. Cross sections of pigs' teeth found in an excavated Stone Age trash pit revealed bands of remarkably constant width except that the band deposited last, which was invariably translucent, was only about half the normal width.", "question": "The statements above most strongly support the conclusion that the animals died", "answers": "['in an unusually early winter', 'from starvation', 'roughly in midwinter', 'at roughly the same age']", "label": 2 }, { "id": "train_886", "context": "Last year a record number of new manufacturing jobs were created. Will this year bring another record? Well, any new manufacturing job is created either within an existing company or by the start-up of a new company. Within existing firms, new jobs have been created this year at well below last years record pace. At the same time, there is considerable evidence that the number of new companies starting up will be no higher this year than it was last year and there is no reason to think that the new companies starting up this year will create more jobs per company than did last year' s start-ups. So clearly, the number of new jobs created this year will fall short of last years record.", "question": "In the argument given, the two portions in boldface play which of the following ?", "answers": "['The first provides evidence in support of the main conclusion of the argument; the second is a claim that the argument challenges.', 'The first is presented as an obvious truth on which the argument is based; the second is a judgment advanced in support of the main conclusion of the argument.', 'The first is a generalization that the argument seeks to establish; the second is a conclusion that the argument draws in order to support that generalization.', 'The first is presented as an obvious truth on which the argument is based; the second is a claim that has been advanced In support of a position that the argument opposes.']", "label": 1 }, { "id": "train_887", "context": "Until recently it was thought that ink used before the sixteenth century did not contain titanium. However, a new type of analysis detected titanium in the ink of the famous Bible printed by Johannes Gutenberg and in that of another fifteenth-century Bible known as B-36, though not in the ink of any of numerous other fifteenth-century books analyzed. This finding is of great significance, since it not only strongly supports the hypothesis that B-36 was printed by Gutenberg but also shows that the presence of titanium in the ink of the purportedly fifteenth-century Vinland Map can no longer be regarded as a reason for doubting the map' s authenticity.", "question": "The reasoning in the passage is vulnerable to criticism on the ground that", "answers": "[\"it is unreasonable to suppose that determination of the date and location of a document's printing or drawing can be made solely on the basis of the presence or absence of a single element in the ink used in the document\", 'if the technology that makes it possible to detect titanium in printing ink has only recently become available, it is unlikely that printers or artists in the fifteenth century would know whether their ink contained titanium or not', 'the discovery of titanium in the ink of the Vinland Map must have occurred before titanium was discovered in the ink of the Gutenberg Bible and the B-36 Bible', 'the results of the analysis are interpreted as indicating that the use of titanium as an ingredient in fifteenth-century ink both was, and was not, extremely restricted']", "label": 3 }, { "id": "train_888", "context": "Humans began to spread across North America around 12, 000 years ago, as the climate became warmer. During the same period the large mammals that were once abundant in North America, such as the mastodon, the woolly mammoth, and the saber-toothed tiger, became extinct. Thus, contrary to the myth that humans formerly lived in harmony with the rest of nature, it is clear that even 12, 000 years ago human activity was causing the extinction of animal species.", "question": "The argument is most vulnerable to the criticism that", "answers": "['for early inhabitants of North America the destruction of mastodons, woolly mammoths, and saber-toothed tigers might have had very different significance than the extinction of mammal species does for modern humans', 'in calling the idea that humans once lived in harmony with nature a myth the argument presupposes what it attempts to prove', \"the evidence it cites is consistent with the alternative hypothesis that the large mammals' extinction was a direct result of the same change in climate that allowed humans to spread across North America\", 'there might have been many other species of animals, besides mastodons, woolly mammoths, and saber-toothed tigers, that became extinct as the result of the spread of humans across North America']", "label": 2 }, { "id": "train_889", "context": "Citizen of Mooresville: Mooresville' s current city council is having a ruinous effect on municipal finances. Since a majority of the incumbents are running for reelection, I am going to campaign against all these incumbents in the upcoming city council election. The only incumbent I will support and vote for is the one who represents my own neighborhood, because she has the experience necessary to ensure that our neighborhood' s interests are served. If everyone in Mooresville would follow my example, we could substantially change the council' s membership.", "question": "Assuming that each citizen of Mooresville is allowed to vote only for a city council representative from his or her own neighborhood, for the council's membership to be changed substantially, it must be true that", "answers": "['few of the incumbents on the Mooresville city council have run for reelection in previous elections', \"none of the challengers in the upcoming election for seats on Mooresville's city council are better able to serve the interests of their neighborhoods than were the incumbents\", 'most of the eligible voters in Mooresville vote in the upcoming election', 'at least some other voters in Mooresville do not make the same exception for their own incumbent in the upcoming election']", "label": 3 }, { "id": "train_890", "context": "If a mother' s first child is born before its due date, it is likely that her second child will be also. Jackie' s second child was not born before its due date, so it is likely that Jackie' s first child was not born before its due date either.", "question": "The questionable reasoning in the argument above is most similar in its reasoning to which one of the following?", "answers": "['If a business is likely to fail, people will not invest in it. Pallid Starr is likely to fail, ttherefore no one is likely to invest in it.', 'Tai will go sailing only if the weather is nice. The weather will be nice, thus Tai will probably go sailing.', 'Artisans who finish their projects before the craft fair will probably go to the craft fair. Ben will not finish his project before the fair. So he probably will not go to the craft fair.', 'If a movie is a box-office hit, it is likely that its sequel will be also. Hawkman II, the sequel to Hawkman I, was not a box-office hit, so Hawkman I was probably not a box-office hit.']", "label": 3 }, { "id": "train_891", "context": "Alexia: Our ignorance about the lives of so many novelists is unfortunate. How much deeper and more complete our appreciation of their works would be if we knew more about the personal wellsprings of their thought and art! Malik: I disagree. Because we know virtually nothing of their personal lives, we can resist the temptation to reduce consideration of their works to biography and psychoanalysis, and instead engage each work on its own artistic terms.", "question": "The dialogue provides the most support for the claim that Alexia and Malik disagree over whether", "answers": "['understanding the personal wellsprings of the work of certain novelists is impossible without knowledge of their lives', \"knowledge about a novelist's personal life can interfere with the appropriate understanding of that novelist's work\", \"a psychoanalytic approach provides an inadequate method of understanding a novelist's life\", \"a novelist's work is not influenced by the details of his or her personal life\"]", "label": 1 }, { "id": "train_892", "context": "Principle: Meetings should be kept short, addressing only those issues relevant to a majority of those attending. A person should not be required to attend a meeting if none of the issues to be addressed at the meeting are relevant to that person. Application: Terry should not be required to attend today' s two o' clock meeting.", "question": "Which one of the following, if true, most justifies the stated application of the principle?", "answers": "['No issue relevant to Terry could be relevant to a majority of those attending the meeting.', 'If Terry attends the meeting a different set of issues will be relevant to a majority of those attending than if Terry does not attend.', 'If Terry makes a presentation at the meeting, the meeting will not be kept short.', 'The majority of the issues to be addressed at the meeting are not relevant to Terry.']", "label": 0 }, { "id": "train_893", "context": "Physicalists\" expect that ultimately all mental functions will be explainable in neurobiological terms. Achieving this goal requires knowledge of neurons and their basic functions, a knowledge of how neurons interact, and a delineation of the psychological faculties to be explained. At present, there is a substantial amount of fundamental knowledge about the basic functions of neurons, and the scope and character of such psychological capacities as visual perception and memory are well understood. Thus, as the physicalists claim, mental functions are bound to receive explanations in neurobiological terms in the near future.", "question": "Which one of the following indicates an error in the reasoning in the passage?", "answers": "['The passage does not indicate that any knowledge has been achieved about how neurons interact.', 'The passage fails to describe exactly what is currently known about the basic functions of neurons.', 'The word \"neurobiological\" is used as though it had the same meaning as the word \"mental. \"', 'The conclusion contradicts the claim of the physicalists.']", "label": 0 }, { "id": "train_894", "context": "Bus driver: Had the garbage truck not been exceeding the speed limit, it would not have collided with the bus I was driving. I, on the other hand, was abiding by all traffic regulations -- as the police report confirms. Ttherefore, although I might have been able to avoid the collision had I reacted more quickly, the bus company should not reprimand me for the accident.", "question": "Which one of the following principles, if valid, most helps to justify the reasoning in the bus driver's argument?", "answers": "['A company that employs bus drivers should reprimand those drivers only when they become involved in collisions that they reasonably could have been expected to avoid.', 'A bus company should not reprimand one of its drivers whose bus is involved in a collision if a police report confirms that the collision was completely the fault of the driver of another vehicle.', 'If a vehicle whose driver is violating a traffic regulation collides with a vehicle whose driver is not, the driver of the first vehicle is solely responsible for the accident.', \"When a bus is involved in a collision, the bus driver should not be reprimanded by the bus company if the collision did not result from the bus driver's violating a traffic regulation.\"]", "label": 3 }, { "id": "train_895", "context": "People without cognitive impairments tend to read with ease. People with cognitive impairments tend to have great difficulty reading. Ttherefore, if people with cognitive impairments are taught to read with ease, the proportion of people with cognitive impairments will decrease.", "question": "A reasoning error in the argument is that the argument", "answers": "['confuses the relationship between the cause of something and the resulting effect', 'takes the lack of evidence for the existence of a causative relationship to prove that a causative relationship does not exist', 'presupposes that which is to be proved', 'refutes a generalization by means of an exceptional case']", "label": 0 }, { "id": "train_896", "context": "Editorial: Many observers note with dismay the decline in the number of nongovernmental, voluntary community organizations. They argue that this decline is caused by the corresponding growth of government services once provided by these voluntary community groups. But this may not be true. The increase in government services may coincide with a decrease in volunteerism, but the former does not necessarily cause the latter; the latter may indeed cause the former.", "question": "The editorial undermines the conclusion of the causal argument by", "answers": "['offering a counterexample to the alleged correlation', 'offering an alternate explanation of the correlation cited', 'proving that governments must do what community organizations fail to do', 'showing that there is no causality involved']", "label": 1 }, { "id": "train_897", "context": "Mary to Jamal: You acknowledge that as the legitimate owner of this business I have the legal right to sell it whenever I wish. But also you claim that because loyal employees will suffer if I sell it, I ttherefore have no right to do so. Obviously, your statements taken together are absurd.", "question": "Mary's reasoning is most vulnerable to the criticism that she", "answers": "['overlooks the possibility that her employees also have rights related to the sale of the business', 'overlooks the possibility that Jamal is referring to two different kinds of right', \"attacks Jamal's character rather than his argument\", 'overlooks the possibility that when Jamal claims that she has no right to sell the business, he simply means she has no right to do so at this time']", "label": 1 }, { "id": "train_898", "context": "Politician: The cohesion of a society depends on its members' accepting a set of basic principles. When these principles are routinely called into question, the society begins to erode. Any society, ttherefore, that allows schools to present the society' s set of basic principles as simply one of several alternatives, with the suggestion that students may choose which to accept on the basis of the principles' worth, is inviting its own demise.", "question": "Which one of the following would, if true, most strengthen the politician's argument?", "answers": "['Children are more likely than adults to question the wisdom of those who founded and shaped a society.', 'Unless people believe that they have freely chosen to adopt the principles that govern their societies, they will tend to repudiate these principles.', 'One cannot evaluate a set of political principles without having first established criteria of evaluation.', 'Given the chance to assess the merits of the principles governing their societies, individuals will rarely find those principles acceptable.']", "label": 3 }, { "id": "train_899", "context": "The top priority of the school administration should be student attendance. No matter how good the teachers, texts, and facilities are, none of these does any good if few students come to school.", "question": "The pattern of reasoning in the argument above is LEAST similar to that in which one of the following?", "answers": "[\"The top priority of a person lost in the wilderness should be food-gathering. Knowing how to find one's way back or how to build a comfortable shelter does one no good if one does not have enough food to survive.\", 'The top priority of a detective should be to gather physical evidence. High-tech crime lab equipment and the most sophisticated criminological analysis are of no use if crucial clues are not gathered.', 'The top priority of a criminal defense lawyer should be to ensure that innocent clients are found not guilty. Such clients can justly be released from jail and resume their normal lives if they are found not guilty.', 'The top priority of a library should be to maintain its collection of books. A knowledgeable staff and beautiful facilities are of no value if there is an inadequate supply of books to lend.']", "label": 2 }, { "id": "train_900", "context": "If John is not attending medical school, then he is attending business school. John is not attending business school, so he must be attending medical school.", "question": "Which one of the following most closely parallels the argument's reasoning?", "answers": "['All scientists hold a graduate degree, and Dan is a scientist, so he must hold a graduate degree.', 'If Eric is eating soup, then he is not eating a sandwich. Eric is eating a sandwich, so he is not eating soup.', 'Hank is the best college basketball player, and ttherefore, he will play professionally next year.', 'If Maria attends the ballet, then she will wear a dress. Maria is wearing a dress, so she is at the ballet.']", "label": 1 }, { "id": "train_901", "context": "Advertisement: Last year, Factorial Mutual Fund continued its strong record of investment performance. Investors who included Factorial Fund in their portfolios realized an average capital gain of 15% across all of their investment assets, nearly double the market return. Ttherefore, investors interested in high returns should consider adding Factorial Mutual Fund to their portfolios.", "question": "The claim in the advertisement above is based on which of the following assumptions?", "answers": "['Last year, Factorial Fund outperformed all other funds with similar style and investment objectives.', 'The fees charged by Factorial Fund are among the lowest in the investment industry.', \"A substantial part of the 15% portfolio appreciation was attributable to the returns of Factorial Fund and this level of fund's performance is likely to persist in the future.\", 'Factorial Fund is one of the oldest and largest mutual funds in the investment industry.']", "label": 2 }, { "id": "train_902", "context": "Anyone who argues that graffiti is legitimate artwork should also agree that street gangs gave rise to legitimate artwork, since street gangs gave rise to graffiti.", "question": "The pattern of reasoning displayed in the argument above most closely parallels that displayed in which one of the following?", "answers": "['Anyone who claims that exercise is beneficial should also acknowledge that some exercise is harmful if done in excess.', 'A person who consumes large quantities of dairy products is probably thin since most people who consume dairy products generally consume fewer calories than people who do not consume dairy products.', 'Anyone who argues that tomatoes are a fruit should also hold that candy X is higher in vitamin C than some fruit, since candy X is higher in vitamin C than tomatoes.', \"Anyone who dislikes rock music should also agree that some people like rock music, if that person agrees people's taste in music differ.\"]", "label": 2 }, { "id": "train_903", "context": "Scientists have discovered a new species of butterfly that lives only in a small region of Central America and is active only at night. During the day, it rests in treetops, where its green color matches the foliage perfectly. Ttherefore, the scientists must have discovered the butterfly at night.", "question": "The argument depends on which of the following assumptions?", "answers": "['The butterfly cannot survive in areas outside of Central America.', 'There is no way for the scientists to detect the butterfly during the day.', 'No other butterfly species lives in this region of Central America.', \"The foliage in the butterfly's habitat is completely green.\"]", "label": 1 }, { "id": "train_904", "context": "Some legislators refuse to commit public funds for new scientific research if they cannot be assured that the research will contribute to the public welfare. Such a position ignores the lessons of experience. Many important contributions to the public welfare that resulted from scientific research were never predicted as potential outcomes of that research.", "question": "Suppose that a scientist in the early twentieth century had applied for public funds to study molds: who would have predicted that such research would lead to the discovery of antibiotics -- one of the greatest contributions ever made to the public welfare? Which one of the following most accurately expresses the main point of the argument?", "answers": "['Lack of guarantees that new scientific research will contribute to the public welfare is not sufficient reason for legislators to refuse to commit public funds to new scientific research.', 'The committal of public funds for new scientific research will ensure that the public welfare will be enhanced.', 'Scientific discoveries that have contributed to the public welfare would have occurred sooner if public funds had been committed to the research that generated those discoveries.', 'In order to ensure that scientific research is directed toward contributing to the public welfare, legislators must commit public funds to new scientific research.']", "label": 0 }, { "id": "train_905", "context": "Paper and plastic grocery bags are a continuing problem for the city, both as litter and in landfills. To discourage their use, the city has proposed a tax on each bag, to be paid by the supermarkets that supply them. Several environmental groups, however, oppose the tax, despite having brought the issue to the council' s attention in the first place.", "question": "Which of the following, if true, would best explain the environmental groups' opposition to the proposed tax?", "answers": "['Only supermarkets distributing more than a certain number of grocery bags each month would be subject to the proposed tax.', \"Plastic bags account for 90 percent of the grocery bags that litter the city's streets and take up space in landfills.\", 'The cost to the city of collecting and disposing of a grocery bag is greater than the proposed tax per bag.', 'Supermarkets plan to pass the tax on to consumers, who are unlikely to change their behavior based on the small increase in their grocery bills.']", "label": 3 }, { "id": "train_906", "context": "Editor: Many candidates say that if elected they will reduce governmental intrusion into voters' lives. But voters actually elect politicians who instead promise that the government will provide assistance to solve their most pressing problems. Governmental assistance, however, costs money, and money can come only from taxes, which can be considered a form of governmental intrusion. Thus, governmental intrusion into the lives of voters will rarely be substantially reduced over time in a democracy.", "question": "Which one of the following, if true, would most strengthen the editor's argument?", "answers": "['Politicians who promise to do what they actually believe ought to be done are rarely elected.', 'Politicians never promise what they really intend to do once in office.', 'Politicians who win their elections usually keep their campaign promises.', 'The most common problems people have are financial problems.']", "label": 2 }, { "id": "train_907", "context": "A television commercial argued as follows: both the product advertised and its competitor contain the same active ingredients, but the product advertised contains them in higher concentrations; tests show that the higher concentrations are completely safe; since a product that provides faster relief is always preferable, one should use the product advertised.", "question": "The television commercial's argument is most vulnerable to criticism on the grounds that it", "answers": "['takes for granted that the product with the higher concentration of active ingredients provides faster relief than its competitor', 'attempts to manipulate the emotions of potential customers of the advertised product rather than presenting logically sound reasons for preferring the product advertised', 'attempts to establish its conclusion on the basis of evidence that is in principle impossible to disprove', \"dismisses its competitor's claims because of their source rather than their content\"]", "label": 0 }, { "id": "train_908", "context": "Radioactive waste from nuclear power plants has been temporarily stored on-site, but this is not a satisfactory kind of place for long-range storage. Since no suitable plan of safe permanent storage of such waste from the nation' s existing and planned nuclear plants has been devised, some people propose that we should stop trying to develop such a plan and instead should shut down all present nuclear plants and build no new nuclear plants.", "question": "The proposal mentioned above falls short of offering a complete solution to the problem it addresses because", "answers": "['it would prevent the development of safe technologies for producing electric power', 'it does not distinguish between nuclear plants that have, and plants that do not have, a reputation for operating safely', 'it does not provide for the permanent storage of already-existing waste', 'the risks of unsafe disposal of waste from nuclear power plants lie in the future, but the benefits from such plants are in the present']", "label": 2 }, { "id": "train_909", "context": "Columnist on the arts: My elected government representatives were within their rights to vote to support the arts with tax dollars. While funded by the government, however, some artists have produced works of art that are morally or aesthetically offensive to many taxpayers. Nonetheless, my conclusion is that no taxpayers have been treated unjustly whose tax dollars are used to fund some particular work of art that they may find abominable.", "question": "Which one of the following principles, if valid, most supports the columnist's argument?", "answers": "['The funding of a particular activity is warranted if it is funded by elected representatives who legitimately fund that activity in general.', 'Those who resent taxation to subsidize offensive art should vote against their incumbent government representatives.', 'Elected representatives are within their rights to fund any activity that is supported by a majority of their constituents.', 'Taxpayers should be allowed to decide whether a portion of their tax dollars is to be used to fund the arts.']", "label": 0 }, { "id": "train_910", "context": "Politician: Some proponents of unilateral nuclear arms reduction argue that it would encourage other countries to reduce their own nuclear arsenals, eventually leading to an international agreement on nuclear arms reduction. Our acting on the basis of this argument would be dangerous, because the argument ignores the countries presently on the verge of civil wars. These countries, many of which have nuclear capability, cannot be relied upon to conform to any international military policy.", "question": "Which one of the following most accurately expresses the conclusion of the politician's argument?", "answers": "[\"It is risky for the politician's country to unilaterally reduce nuclear arms in hopes of achieving an international agreement on arms reduction.\", 'Countries that are on the verge of civil wars are unlikely to agree to reduce either their nuclear arms or their conventional weapons.', 'It is unlikely that an international agreement on nuclear disarmament will ever be achieved.', \"Unilateral nuclear arms reduction by the politician's country would encourage all countries to reduce their nuclear arsenals.\"]", "label": 0 }, { "id": "train_911", "context": "Last year, a software company held a contest to generate ideas for their new logo. According to the rules, everyone who entered the contest would receive several prizes, including a T-shirt with the company' s new logo. Juan has a T-shirt with the company' s new logo, so he must have entered the contest.", "question": "The reasoning in the argument is flawed in that the argument", "answers": "['infers that every member of a group has a feature in common on the grounds that the group as a whole has that feature', 'takes a condition that is sufficient for a particular outcome as one that is necessary for that outcome', 'infers a causal relationship when the evidence only supports a correlation', 'has a premise that presupposes the truth of the conclusion']", "label": 1 }, { "id": "train_912", "context": "FastMart, a convenience store chain, is planning to add pancake syrup to the items it sells. FastMart stores do not have shelf space to stock more than one variety of syrup. Surveys of FastMart customers indicate that one-fourth of them prefer low-calorie syrup, while three-fourths prefer regular syrup. Since FastMart' s highest priority is to maximize sales, the obvious strategy for it is to stock regular syrup.", "question": "Which of the following, if true, most seriously weakens the argument?", "answers": "['People buying pancake syrup at convenience stores, unlike those buying it at supermarkets, generally buy it only a few times.', 'Regular syrup does not sell for a higher price per unit than low-calorie syrup.', 'Sales of syrup are not expected to account for a large proportion of total dollar sales at the average FastMart store.', 'In general, customers who prefer regular syrup will buy low-calorie syrup if regular is unavailable, but those who prefer low-calorie will not buy regular syrup.']", "label": 3 }, { "id": "train_913", "context": "A severe blow to the head can cause one to lose consciousness; from this some people infer that consciousness is a product of the brain and cannot survive bodily death. But a radio that becomes damaged may suddenly cease to broadcast the program it had been receiving, and we do not conclude from this that the program itself has ceased to exist. Similarly, more substantial evidence would be needed to conclude that consciousness does not survive bodily death.", "question": "Which one of the following most accurately describes the role played in the argument by the example of the damaged radio?", "answers": "['It is cited as evidence that consciousness does in fact survive bodily death.', 'It is cited as the primary piece of evidence for the conclusion that the relationship of consciousness to the brain is analogous to that of a radio program to the radio that receives it.', 'It is cited as a counterexample to a widely accepted belief about the nature of consciousness.', 'It is cited as a case analogous to loss of consciousness in which people do not draw the same sort of conclusion that some people draw about consciousness.']", "label": 3 }, { "id": "train_914", "context": "The same task triggers different levels of awareness of one' s surroundings, called environmental awareness, in different individuals. Mathematical puzzles, for example, cause most people to increase such an awareness. Some people -- those who formulate the answer visually, imagining the numbers in their mind' s eye -- will, in an attempt to freeze the picture, experience a decrease in environmental awareness while solving the puzzle. Other people' s environmental awareness may rise during the exercise, because their brains are signaling a rest at the end of every stage of problem solving.", "question": "Which one of the following is most strongly supported by the information above?", "answers": "['People who visually formulate answers differ from other problem solvers in that the former are aware of their surroundings.', 'People tend to be more aware of their surroundings when solving mathematical problems than when solving nonmathematical problems.', 'There are some people for whom mathematical puzzles do not cause an increase in their level of environmental awareness.', \"Mathematical problem solving requires frequent periods of rest in the form of increased awareness of the problem solver's surroundings.\"]", "label": 2 }, { "id": "train_915", "context": "A recent study has found that, surprisingly, the risk of serious injuries to workers is higher in industries that are monitored by government safety inspectors than in industries that are not so monitored.", "question": "Which one of the following, if true, most helps to explain the surprising finding described above?", "answers": "['Government safety inspectors do not have the authority to enforce safety regulations.', 'Workers behave especially cautiously when they believe their performance is being monitored by government safety inspectors.', 'Only those industries with an inherently high risk of on-the-job injury are monitored by government safety inspectors.', 'Government safety inspectors not only monitor but also train employees of the inspected firms to follow safe practices.']", "label": 2 }, { "id": "train_916", "context": "On the Discount Phoneline, any domestic long-distance call starting between 9 A. M. and 5 P. M. costs 15 cents a minute, and any other domestic long-distance call costs 10 cents a minute. So any domestic long-distance call on the Discount Phoneline that does not cost 10 cents a minute costs 15 cents a minute.", "question": "The pattern of reasoning in which one of the following arguments is most similar to that in the argument above?", "answers": "['If a university class involves extensive lab work, the class will be conducted in a laboratory; otherwise, it will be conducted in a normal classroom. Thus, if a university class does not involve extensive lab work, it will not be conducted in a laboratory.', 'If a university class involves extensive lab work, the class will be conducted in a laboratory; otherwise, it will be conducted in a normal classroom. Thus, if a university class is not conducted in a normal classroom, it will involve extensive lab work.', 'If a university class involves extensive lab work, the class will be conducted in a laboratory; otherwise, it will be conducted in a normal classroom. Thus, if a university class is not conducted in a normal classroom, it will be conducted in a laboratory.', 'If a university class involves extensive lab work, the class will be conducted in a laboratory; otherwise, it will be conducted in a normal classroom. Thus, if a university class involves extensive lab work, it will not be conducted in a normal classroom.']", "label": 2 }, { "id": "train_917", "context": "Hillsburgh and Nansensville are two major metropolitan areas only 50 miles apart. Until recently, the only road connecting the two cities was a traffic-clogged, six-lane Interstate Highway. Six years in the making, the Metro Expressway, a four-lane road connecting the two cities, was hoped to relieve this congestion. After the first month, however, the traffic along the Interstate Highway has not improved.", "question": "Which of the following, if true, could account for the continued traffic congestion along the Interstate Highway?", "answers": "['The Hillsburgh Gazette reports that police activity along the Interstate Highway has decreased since many police are now patrolling the Metro Expressway.', 'During the month that the Metro Expressway opened, the railway workers who service the train connecting the two cities went on strike.', 'The Metro Expressway opened up in July, a month when many of the citizens leave for vacation.', 'In the next five years, both cities plan on building another expressway connecting both cities.']", "label": 1 }, { "id": "train_918", "context": "A recent study found that snoring, though not common in either group, is more common among smokers than among nonsmokers. On the basis of this evidence, the author hypothesized that smoking by itself can induce snoring.", "question": "Which one of the following, if true, casts the most doubt on the author's hypothesis?", "answers": "['Most snorers do not smoke.', 'Stress induces both snoring and smoking in certain individuals.', 'Most smokers do not snore.', 'Both smoking and snoring cause throat problems.']", "label": 1 }, { "id": "train_919", "context": "A research study revealed that, in most cases, once existing highways near urban areas are widened and extended in an attempt to reduce traffic congestion and resulting delays for motorists, these problems actually increase rather than decrease.", "question": "Which one of the following, if true, most helps to explain the discrepancy between the intended results of the highway improvements and the results revealed in the study?", "answers": "['Urban traffic generally moves at a slower pace and involves more congestion and delays than rural and suburban traffic.', 'Typically, road widening or extension projects are undertaken only after the population near the road in question has increased and then leveled off, leaving a higher average population level.', 'Widened and extended roads tend to attract many more motorists than used them before their improvement.', 'As a general rule, the greater the number of lanes on a given length of highway, the lower the rate of accidents per 100, 000 vehicles traveling on it.']", "label": 2 }, { "id": "train_920", "context": "Production manager: The building materials that we produce meet industry safety codes but pose some safety risk. Since we have recently developed the technology to make a safer version of our product, we should stop producing our current product and sell only the safer version in order to protect public safety. Sales manager: If we stop selling our current product, we will have no money to develop and promote the safer product. We need to continue to sell the less-safe product in order to be in a position to market the safer product successfully.", "question": "Which one of the following principles, if established, most helps to justify the production manager's conclusion?", "answers": "['In order to make building materials safer, companies should continually research new technologies whether or not they are required to do so in order to comply with safety codes.', 'Companies should not sell a product that poses safety risks if they are technologically capable of producing a safer version of that product.', 'Product safety codes should be reviewed whenever an industry replaces one version of a product with a technologically more advanced version of that product.', 'That a product does not meet industry safety codes should be taken as sufficient indication that the product poses some safety risks.']", "label": 1 }, { "id": "train_921", "context": "Marife: That was a bad movie because, by not providing viewers with all the information necessary for solving the murder, it violated a requirement of murder mysteries. Nguyen: But the filmmaker wanted viewers to focus on the complex relationship between the chief detective and her assistant. The murder just provided the context in which the relationship developed, and should not be taken as a defining characteristic of the film.", "question": "Marife's and Nguyen's comments indicate that they disagree about", "answers": "['whether the filmmaker wanted viewers to be able to solve the murder', 'whether the movie was a bad one', 'whether the relationship between the chief detective and her assistant was an important part of the movie', 'whether the movie should be classified as a murder mystery']", "label": 3 }, { "id": "train_922", "context": "Politician: The current crisis in mathematics education must be overcome if we are to remain competitive in the global economy. Alleviating this crisis requires the employment of successful teaching methods. No method of teaching a subject can succeed that does not get students to spend a significant amount of time outside of class studying that subject.", "question": "Which one of the following statements follows logically from the statements above?", "answers": "['The current crisis in mathematics education will not be overcome unless students spend a significant amount of time outside of class studying mathematics.', \"Students' spending a significant amount of time outside of class studying mathematics would help us to remain competitive in the global economy.\", 'Few subjects are as important as mathematics to the effort to remain competitive in the global economy.', 'If students spend a significant amount of time outside of class studying mathematics, the current crisis in mathematics education will be overcome.']", "label": 0 }, { "id": "train_923", "context": "When investigators discovered that the director of a local charity had repeatedly overstated the number of people his charity had helped, the director accepted responsibility for the deception. However, the investigators claimed that journalists were as much to blame as the director was for inflating the charity' s reputation, since they had naively accepted what the director told them, and simply reported as fact the numbers he gave them.", "question": "Which one of the following principles, if valid, most helps to justify the investigators' claim?", "answers": "['Anyone who presents as factual a story that turns out to be untrue without first attempting to verify that story is no less responsible for the consequences of that story than anyone else is.', 'Anyone who knowingly aids a liar by trying to conceal the truth from others is also a liar.', 'Anyone who lies in order to advance his or her own career is more deserving of blame than someone who lies in order to promote a good cause.', 'Anyone who works for a charitable organization is obliged to be completely honest about the activities of that organization.']", "label": 0 }, { "id": "train_924", "context": "Rabbits were introduced to Numa Island in the nineteenth century. Overgrazing by the enormous population of rabbits now menaces the island' s agriculture. The government proposes to reduce the population by using a virus that has caused devastating epidemics in rabbit populations elsewhere. There is, however, a chance that the virus will infect the bilby, an endangered native marsupial. The government' s plan, ttherefore, may serve the interests of agriculture but will clearly increase the threat to native wildlife.", "question": "Which of the following, if true, most seriously weakens the argument?", "answers": "['There are no species of animals on the island that prey on the rabbits.', 'The virus that the government proposes to use has been successfully used elsewhere to control populations of rabbits.', 'Overgrazing by rabbits endangers many of the plants on which bilbies feed.', 'There is no alternative means of reducing the rabbit population that would involve no threat to the bilby.']", "label": 2 }, { "id": "train_925", "context": "To be great, an artwork must express a deep emotion, such as sorrow or love. But an artwork cannot express an emotion that the artwork' s creator is incapable of experiencing.", "question": "Which one of the following can be properly inferred from the statements above?", "answers": "['The greatest art is produced by those who have experienced the deepest emotions.', 'An artwork that expresses a deep emotion of its creator is a great artwork.', 'Only artworks that succeed in expressing deep emotions are the products of great artists.', 'As long as computers are constructed so as to be incapable of experiencing emotions they will not create great artworks.']", "label": 3 }, { "id": "train_926", "context": "The presence of bees is necessary for excellent pollination, which, in turn, usually results in abundant fruits and vegetables. Establishing a beehive or two near one' s garden ensures the presence of bees. Keeping bees is economical, however, only if the gardener has a use for homegrown honey. Thus, gardeners who have no use for homegrown honey will tend not to have beehives, so their gardens will fail to have excellent pollination.", "question": "Which one of the following most accurately describes a flaw in the reasoning of the argument?", "answers": "['The argument fails to consider that bees might be present even in the absence of a particular condition that would ensure their presence.', 'The argument confuses what is necessary for an abundance of fruits and vegetables with what is usually conducive to it.', 'The argument confuses what is necessary for pollination to take place with what would guarantee that it takes place.', 'The argument fails to consider the possibility that obtaining homegrown honey is only one of several advantages of beehives.']", "label": 0 }, { "id": "train_927", "context": "It is proposed to allow the sale, without prescription, of a medication that physicians currently prescribe to treat the common ear inflammation called \"swimmer' s ear. \" The principal objection is that most people lack the expertise for proper self-diagnosis and might not seek medical help for more serious conditions in the mistaken belief that they have swimmer' s ear. Yet in a recent study, of 1, 000 people who suspected that they had swimmer' s ear, 84 percent had made a correct diagnosis -- a slightly better accuracy rate than physicians have in diagnosing swimmer' s ear. Thus, clearly, most people can diagnose swimmer' s ear in themselves without ever having to consult a physician.", "question": "Which one of the following, if true, most undermines the conclusion?", "answers": "[\"Cases in which swimmer's ear progresses to more serious infections are very rare.\", \"For many people who develop swimmer's ear, the condition disappears without medical or pharmaceutical intervention.\", 'Physicians who specialize in ear diseases are generally able to provide more accurate diagnoses than those provided by general practitioners.', \"Most of the people who diagnosed themselves correctly had been treated by a physician for prior occurrence of swimmer's ear.\"]", "label": 3 }, { "id": "train_928", "context": "In a recent study of arthritis, researchers tried but failed to find any correlation between pain intensity and any of those features of the weather -- humidity, temperature swings, barometric pressure -- usually cited by arthritis sufferers as the cause of their increased pain. Those arthritis sufferers in the study who were convinced of the existence of such a correlation gave widely varying accounts of the time delay between the occurrence of what they believed to be the relevant feature of the weather and the increased intensity of the pain. Thus, this study __.", "question": "Of the following, which one most logically completes the argument?", "answers": "[\"indicates that arthritis sufferers' beliefs about the causes of the pain they feel may affect their assessment of the intensity of that pain\", 'indicates that the weather affects some arthritis sufferers more quickly than it does other arthritis sufferers', 'suggests that arthritis sufferers are imagining the correlation they assert to exist', 'suggests that some people are more susceptible to weather-induced arthritis pain than are others']", "label": 2 }, { "id": "train_929", "context": "Writer: I collaborated with another writer on my last book, instead of writing alone as I usually do. Because the book sold so well as a result of this joint effort, I should collaborate with a writer on my next book so that book will sell well too.", "question": "Which one of the following principles, if valid, most helps to justify the reasoning above?", "answers": "['Writers who collaborate on books, if they are good writers, usually produce books that sell well.', 'Writers who do not collaborate on books have a smaller chance of writing a book that will sell well.', \"If a person's book sells well because of a collaboration, that person's next book will sell well, if he or she collaborates with the same writer.\", \"If a person's book sells well because of a collaboration, future collaborations on the part of that person will produce other books that sell well.\"]", "label": 3 }, { "id": "train_930", "context": "Music critic: How well an underground rock group' s recordings sell is no mark of that group' s success as an underground group. After all, if a recording sells well, it may be because some of the music on the recording is too trendy to be authentically underground; accordingly, many underground musicians consider it desirable for a recording not to sell well. But weak sales may simply be the result of the group' s incompetence.", "question": "Which one of the following principles, if valid, most helps to justify the music critic's argument?", "answers": "['An underground rock group is successful as an underground group if the group is competent but its recordings nonetheless do not sell well.', 'An underground rock group is unsuccessful as an underground group if it is incompetent or if any of its music is too trendy to be authentically underground, or both.', 'For an underground rock group, competence and the creation of authentically underground music are not in themselves marks of success.', \"Whether an underground group's recordings meet criteria that many underground musicians consider desirable is not a mark of that group's success.\"]", "label": 1 }, { "id": "train_931", "context": "That long-term cigarette smoking can lead to health problems including cancer and lung disease is a scientifically well-established fact. Contrary to what many people seem to believe, however, it is not necessary to deny this fact in order to reject the view that tobacco companies should be held either morally or legally responsible for the poor health of smokers. After all, excessive consumption of candy undeniably leads to such health problems as tooth decay, but no one seriously believes that candy eaters who get cavities should be able to sue candy manufacturers.", "question": "The main point of the argument is that", "answers": "['no one should feel it necessary to deny the scientifically well-established fact that longterm cigarette smoking can lead to health problems', 'excessive consumption of candy will lead to health problems just as surely as long-term cigarette smoking will', \"the fact that smokers' health problems can be caused by their smoking is not enough to justify holding tobacco companies either legally or morally responsible for those problems\", 'if candy manufacturers were held responsible for tooth decay among candy eaters then tobacco companies should also be held responsible for health problems suffered by smokers']", "label": 2 }, { "id": "train_932", "context": "One sure way you can tell how quickly a new idea -- for example, the idea of \"privatization\" -- is taking hold among the population is to monitor how fast the word or words expressing that particular idea are passing into common usage. Professional opinions of whether or not words can indeed be said to have passed into common usage are available from dictionary editors, who are vitally concerned with this question.", "question": "The method described above for determining how quickly a new idea is taking hold relies on which one of the following assumptions?", "answers": "['Dictionary editors have exact numerical criteria for telling when a word has passed into common usage.', 'For a new idea to take hold, dictionary editors have to include the relevant word or words in their dictionaries.', 'As a word passes into common usage, its meaning does not undergo any severe distortions in the process.', 'Words denoting new ideas tend to be used before the ideas denoted are understood.']", "label": 2 }, { "id": "train_933", "context": "Recently, many medical students have been enrolling in joint M. D. /J. D. programs, in which they receive both medical and law degrees at the conclusion of their studies. But since medical students primarily earn their law degree in order to protect themselves from malpractice lawsuits as doctors, we must still conclude that they are primarily doctors, and not lawyers.", "question": "Which of the following is an assumption that supports drawing the conclusion above from the reason given?", "answers": "['Some students who complete such joint programs choose to become lawyers in fields in which they do not use their medical training, while those who choose to become doctors do use legal training.', 'If such joint degrees required that students focus more on their legal studies than their medical studies, students could be considered lawyers after graduation.', 'Certain recipients of these joint degrees are not any less diligent than others in the legal courses that they take, with notable exceptions.', 'A person cannot be considered a lawyer if he or she received a law degree for purposes other than practicing the law.']", "label": 3 }, { "id": "train_934", "context": "Students asked by a psychologist to tell a lie before discussion groups vastly overestimated how many people in the discussion groups could tell they were lying. Other research has found that when volleyball players perform unusually poorly on the court, teammates notice this far less often than the players expect. Finally, in one research experiment a student wearing a funny T-shirt entered a room full of people. Questioning revealed that only a small fraction of the people in the room noticed the shirt, contrary to the student' s expectations.", "question": "Which one of the following is best illustrated by the statements above?", "answers": "['People tend to believe their appearance and behavior are noticed by others more often than is actually the case.', 'People tend not to notice the appearance or behavior of others.', 'We are actually less observant of the appearance and behavior of others than we think ourselves to be.', 'People tend to be far less aware of their own appearance and behavior than are other people.']", "label": 0 }, { "id": "train_935", "context": "The rate of health complications of patients on intravenous (IV) therapy at a particular hospital was higher than usual. Government inspectors found that the typical IV solutions used in this hospital had somewhat high concentrations of sodium and potassium, which were raising patients' blood pressure and taxing their kidneys. The government inspectors mandated lowering the sodium and potassium in these IV preparations, and threatened with a possible government fine. In compliance, the hospital lowered the sodium and potassium levels in the IV solutions to the correct levels. Nevertheless, patients on IV therapy at that hospital continued to have a high rate of health complications.", "question": "Which of the following, if true, most helps to explain why acting on the government inspectors' recommendations failed to achieve its goal?", "answers": "['When sodium and potassium levels in the blood fall below their baseline level, it can damage cells throughout the body by reverse osmosis.', 'A high proportion of patients at this hospital are older, and older patients are more vulnerable to infections that can accompany IVs.', \"It is typical for a patient's appetite to increase to healthy levels once they have completed a course of IV therapy.\", 'The change in IV solution procedure meant a number of related legal documents had to be renegotiated and rewritten, at great cost.']", "label": 1 }, { "id": "train_936", "context": "The amount of electricity consumed in Millville on any day in August is directly proportional to peak humidity on that day. Since the average peak humidity this August was three points higher than the average peak humidity last August, it follows that more energy was consumed in Millville this August than last August.", "question": "Which one of the following arguments has a pattern of reasoning most similar to the one in the argument above?", "answers": "[\"The amount of art supplies used in any of the Aesthetic Institute's 25 classes is directly proportional to the number of students in that class. Since in these classes the institute enrolled 20 percent more students overall last year than in the previous year, more art supplies were used in the institute's classes last year than in the previous year.\", 'The fees paid by a student at the Aesthetic Institute are directly proportional to the number of classes in which that student enrolls. Since the number of students at the Aesthetic Institute is increasing, it follows that the institute is collecting a greater amount if fees paid by students that it used to.', 'The number of instructors employed by the Aesthetic Institute in any term is directly proportional to the number of classes offered in that term and also directly proportional to the number of students enrolled at the institute. Thus, the number of classes offered by the institute in any term is directly proportional to the number of students enrolled in that term.', 'The number of new students enrolled at the Aesthetic Institute in any given year is directly proportional to the amount of advertising the institute has done in the previous year. Hence, if the institute seeks to increase its student body it must increase the amount it spends on advertising.']", "label": 0 }, { "id": "train_937", "context": "Fast-food restaurants make up 45 percent of all restaurants in Cantaria. Customers at these restaurants tend to be young; in fact, studies have shown that the older people get, the less likely they are to eat in fast-food restaurants. Since the average age of the Canatrian population is gradually rising and will continue to do so, the number of fast-food restaurants is likely to decrease.", "question": "Which of the following, if true, most seriously weakens the argument?", "answers": "['As the population of Canatria gets older, more people are eating at home.', 'The overall population of Canatria is growing steadily.', 'Fast-food restaurants in Canatria are getting bigger, so each one can serve more customers.', 'Many people who rarely eat in fast-food restaurants nevertheless eat regularly in restaurants.']", "label": 1 }, { "id": "train_938", "context": "If Suarez is not the most qualified of the candidates for sheriff, then Anderson is. Thus, if the most qualified candidate is elected and Suarez is not elected, then Anderson will be.", "question": "The reasoning in which one of the following is most similar to the reasoning in the argument above?", "answers": "['If Perez is not the lowest bidder on the catering contract, then Sullivan is. So if Sullivan does not get the contract and Perez does not get it either, then it will not be awarded to the lowest bidder.', 'If the excavation contract does not go to the lowest bidder, then it will go to Caldwell. So if Qiu gets the contract and Caldwell does not, then the contract will have been awarded to the lowest bidder.', 'If the lowest bidder on the sanitation contract is not Dillon, then it is Ramsey. So if the contract goes to the lowest bidder and it does not go to Dillon, then it will go to Ramsey.', 'If Kapshaw is not awarded the landscaping contract, then Johnson will be. So if the contract goes to the lowest bidder and it does not go to Johnson, then it will go to Kapshaw.']", "label": 2 }, { "id": "train_939", "context": "Twelve years ago and again five years ago, there were extended periods when the Darfir Republic' s currency, the pundra, was weak: its value was unusually low relative to the world' s most stable currencies. Both times a weak pundra made Darfir' s manufactured products a bargain on world markets, and Darfir' s exports were up substantially. Now some politicians are saying that, in order to cause another similarly sized increase in exports, the government should allow the pundra to become weak again.", "question": "Which of the following, if true, provides the government with the strongest grounds to doubt that the politicians' recommendation, if followed, will achieve its aim?", "answers": "['Several of the politicians now recommending that the pundra be allowed to become weak made that same recommendation before each of the last two periods of currency weakness.', \"A sharp improvement in the efficiency of Darfir's manufacturing plants would make Darfir's products a bargain on world markets even without any weakening of the pundra relative to other currencies.\", \"The economy of a country experiencing a rise in exports will become healthier only if the country's currency is strong or the rise in exports is significant.\", \"After several decades of operating well below peak capacity, Darfir's manufacturing sector is now operating at near-peak levels.\"]", "label": 3 }, { "id": "train_940", "context": "When Copernicus changed the way we think about the solar system, he did so not by discovering new information, but by looking differently at information already available. Edward Jenner' s discovery of a smallpox vaccine occurred when he shifted his focus to disease prevention from the then more common emphasis on cure. History is replete with breakthroughs of this sort.", "question": "The examples provided above illustrate which one of the following?", "answers": "['The ability to look at information from a different point of view is rare.', 'Shifting from earlier modes of thought can result in important advances.', 'Many valuable intellectual accomplishments occur by chance.', 'Understanding is advanced less often by better organization of available information than it is by the accumulation of new information.']", "label": 1 }, { "id": "train_941", "context": "Because it permits a slower and more natural rhythm of life, living in the country is supposed to be more healthy and relaxed than living in the city. But surveys show that people living in the country become ill as often and as seriously as people living in the city, and that they experience an equal amount of stress.", "question": "The statements above, if true, provide the most support for which one of the following?", "answers": "['Living in the country is neither healthier nor more relaxing than living in the city.', \"The amount of stress a person experiences depends on that person's rhythm of life.\", 'People whose rhythm of life is slow and natural recover quickly from illness.', 'Despite what people believe, a natural rhythm of life is unhealthy.']", "label": 0 }, { "id": "train_942", "context": "Shy adolescents often devote themselves totally to a hobby to help distract them from the loneliness brought on by their shyness. Sometimes they are able to become friends with others who share their hobby. But if they lose interest in that hobby, their loneliness may be exacerbated. So developing an all-consuming hobby is not a successful strategy for overcoming adolescent loneliness.", "question": "Which one of the following assumptions does the argument depend on?", "answers": "['Shy adolescents devote themselves to hobbies mainly because they want to make friends.', 'Shy adolescents will lose interest in their hobbies if they do not make friends through their engagement in those hobbies.', 'No successful strategy for overcoming adolescent loneliness ever intensifies that loneliness.', 'Some other strategy for overcoming adolescent loneliness is generally more successful than is developing an all-consuming hobby.']", "label": 2 }, { "id": "train_943", "context": "Anita: Since 1960 the spotted owl population has declined alarmingly. Timber companies that have been clearing the old-growth forests where the spotted owl lives are responsible for this. Jean: No, the spotted owl' s decline is due not to the timber companies but to a rival species. For the past three decades, the more prolific barred owl has been moving steadily into the spotted owl' s habitat and replacing the spotted owl.", "question": "Jean does which one of the following in her response to Anita?", "answers": "[\"challenges Anita's assumption that the decline in the population of the spotted owl poses a threat to the species'continued survival\", 'proposes an alternative explanation for the decline in the spotted owl population', \"denies the truth of Anita's premise that timber companies have been clearing old-growth forests\", 'suggests that Anita overlooked the possibility that spotted owls are able to live in forests that are not old-growth forests']", "label": 1 }, { "id": "train_944", "context": "If an external force intervenes to give members of a community political self-determination, then that political community will almost surely fail to be truly free, since it is during the people' s struggle to become free by their own efforts that the political virtues necessary for maintaining freedom have the best chance of arising.", "question": "The reasoning above conforms most closely to which one of the following principles?", "answers": "['Self-determination is not the first political virtue that the members of a community achieve in their struggle to become free.', 'A community cannot remain free without first having developed certain political virtues.', 'Political freedom is a virtue that a community can attain through an external force.', 'Real freedom should not be imposed on a community by external forces.']", "label": 1 }, { "id": "train_945", "context": "Economist: During a recession, a company can cut personnel costs either by laying off some employees without reducing the wages of remaining employees or by reducing the wages of all employees without laying off anyone. Both damage morale, but layoffs damage it less, since the aggrieved have, after all, left. Thus, when companies must reduce personnel costs during recessions, they are likely to lay off employees.", "question": "Which one of the following, if true, most strengthens the economist's reasoning?", "answers": "['Some companies will be unable to make a profit during recessions no matter how much they reduce personnel costs.', 'Some companies that have laid off employees during recessions have had difficulty finding enough qualified employees once economic growth resumed.', \"Employee morale is usually the primary concern driving companies' decisions about whether to lay off employees or to reduce their wages.\", 'In general, companies increase wages only when they are unable to find enough qualified employees.']", "label": 2 }, { "id": "train_946", "context": "Ethicist: In a recent judicial decision, a woman was ordered to pay restitution to her husband because of their recent divorce, even though the couple had signed a written prenuptial agreement prior to getting married which stated that each party would be financially independent and not seek alimony should the couple decide to divorce. Thus, it was morally wrong for the man to change his mind and seek restitution.", "question": "Which one of the following principles, if valid, most helps to justify the ethicist's reasoning?", "answers": "['It is morally wrong for one party not to abide by its part of an agreement only if the other party abides by its part of the agreement.', 'It is morally wrong for one person to seek to penalize another person for an action that the first person induced the other person to perform.', 'It is morally wrong to seek compensation for an action performed in the context of a promise to forego such compensation.', 'It is morally wrong to seek a penalty for an action for which the agent is unable to make restitution.']", "label": 2 }, { "id": "train_947", "context": "Forty to 60 percent of students report, in anonymous surveys, that they plagiarized at least once as undergraduates, and evidence indicates that plagiarism also occurs in our medical and business schools. Researchers have found that students who plagiarize are more likely to engage in subsequent professional misconduct such as falsified research results and fraudulent business practices. Thus, a reduction of academic plagiarism will lead to a reduction of professional misconduct.", "question": "Which one of the following most accurately describes a flaw in the reasoning in the argument?", "answers": "['The argument presumes, without providing justification, that a certain phenomenon is the only factor contributing to the incidence of a certain other phenomenon.', 'The argument infers the existence of a causal connection merely on the basis of an association.', 'The argument takes for granted that a certain behavior is more prevalent among members of one population than it is among members of another.', 'The argument introduces one subject for debate, but proceeds to give premises relevant to a different subject.']", "label": 1 }, { "id": "train_948", "context": "Last June, Endsville installed highway traffic cameras, in the hope that drivers along the highways would reduce their speeds. This June, the number of motorists caught speeding by the traffic cameras is nearly twice that caught last June.", "question": "All of the following, if true, can help account for the increase in motorists caught speeding by traffic cameras EXCEPT?", "answers": "['Since the cameras are virtually hidden, many motorists remain oblivious to the fact that the city has installed traffic cameras.', 'Police, throughout the year, have been increasingly less likely to pull over speeding motorists, since each month more highways in Endsville are equipped with traffic cameras.', 'In the last year, there has been an influx of residents to Endsville and thus there are more vehicles on the road at any one time.', 'This June, compared to the last, saw more than a twofold increase in the number of traffic cameras installed along the highways of Endsville.']", "label": 0 }, { "id": "train_949", "context": "The principle of equality before the law rules out using media access and renown as valid considerations in the sentencing of convicted criminals. Nevertheless, movie stars found guilty of drug abuse in well-publicized trials are often sentenced to perform community service, while unknown defendants convicted of the same crimes usually serve prison sentences.", "question": "The statements above, if true, most strongly support which one of the following conclusions?", "answers": "['The principle of equality before the law can properly be overridden by other principles in some cases.', 'The sentencing of movie stars to community service instead of prison constitutes a violation of the principle of equality before the law in many cases.', 'The principle of equality before the law is rigorously applied in only a few types of criminal trials.', 'The principle of equality before the law does not allow for leniency in sentencing.']", "label": 1 }, { "id": "train_950", "context": "A recent government study links the high rates of respiratory ailments in Groverston to airborne pollutants released by the Woodco plywood manufacturing plant there. To address the problem the government imposed strict regulations on emissions which will go into effect in four years. Although Woodco plans to cut its emissions in half two years ahead of schedule, it is unlikely that the rate of respiratory ailments will decline before the regulations go into effect, since __.", "question": "Which of the following most logically completes the passage?", "answers": "['reducing emissions even further than planned would necessitate decreasing production at Woodco', 'three new plywood manufacturing plants are about to go into production in Groverston', 'the number of facilities capable of treating respiratory ailments is not likely to increase', 'it is difficult to make accurate, long-term predictions about emissions']", "label": 1 }, { "id": "train_951", "context": "Manager: When Sullivan was passed over for promotion, people said that the deciding factor was his being much older than the competition. But this is clearly not the case. Several recent promotions have been given to people older than Sullivan.", "question": "The manager's argument is most vulnerable to criticism because it fails to consider the possibility that", "answers": "['the people older than Sullivan who were promoted had no younger competitors', 'age is only one of a number of factors that kept Sullivan from being promoted', 'people often associate age with experience and good judgment', \"Sullivan's employer tries to keep deliberations involving promotion decisions confidential\"]", "label": 0 }, { "id": "train_952", "context": "Environment minister: Many countries have signed an international agreement that is intended to reduce pollution in the world' s oceans. While conformity to this agreement probably would significantly reduce pollution in the world' s oceans, it would also probably reduce economic growth in our country and others. Ttherefore, our country should not sign the agreement.", "question": "Which one of the following principles, if valid, would most help to justify the environment minister's argument?", "answers": "['A country should not sign an agreement that is unlikely to achieve its stated goal.', 'If a policy is likely to protect the environment and is unlikely to reduce economic growth, then governments should implement that policy.', \"When deciding whether to sign an agreement, a country should consider the agreement's effects on other countries' economies as well as on its own economy.\", \"It is more important to maintain economic growth in one's own country than it is to reduce pollution in the world's oceans.\"]", "label": 3 }, { "id": "train_953", "context": "A gram of the artificial sweetener aspartame is much sweeter than a gram of sugar. Soft drinks that are sweetened with sugar are, of course, sweet, so those sweetened with aspartame must be even sweeter. Thus people who regularly drink soft drinks sweetened with aspartame will develop a preference for extremely sweet products.", "question": "Which one of the following arguments exhibits flawed reasoning that is most similar to flawed reasoning in the argument above?", "answers": "['Most people own more books than televisions. Moreover, it takes longer to read a book than to watch an episode of a television show. So most people must spend more time reading than they do watching television.', \"Guillermo has a much shorter drive to work than Abdul does. So Guillermo's estimate of the average commute for workers in the country as a whole is likely to be lower than Abdul's estimate.\", 'Stephanie likes hot summer weather much more than Katherine does. So the place where Stephanie grew up must have had more days of hot summer weather than the place where Katherine grew up.', \"Joe's piggy bank has only pennies in it, and Maria's has only nickels. Nickels are worth much more than pennies. It ttherefore follows that there is more money in Maria's piggy bank than in Joe's.\"]", "label": 3 }, { "id": "train_954", "context": "The Sumpton town council recently voted to pay a prominent artist to create an abstract sculpture for the town square. Critics of this decision protested that town residents tend to dislike most abstract art, and any art in the town square should reflect their tastes. But a town council spokesperson dismissed this criticism, pointing out that other public abstract sculptures that the same sculptor has installed in other cities have been extremely popular with those cities' local residents.", "question": "The statements above most strongly suggest that the main point of disagreement between the critics and the spokesperson is whether", "answers": "['a more traditional sculpture in the town square would be popular among local residents', 'most Sumpton residents will find the new sculpture to their taste', 'abstract sculptures by the same sculptor have truly been popular in other cities', 'public art that the residents of Sumpton would find desirable would probably be found desirable by the residents of other cities']", "label": 1 }, { "id": "train_955", "context": "A hardware store generally sells roughly equal numbers of Maxlast brand hammers and Styron brand hammers. Last week, all of the Maxlast hammers were put on sale and placed in a display case just inside the store entrance while the Styron hammers retained their usual price and location. Surprisingly, the Styron hammers slightly outsold the Maxlast hammers.", "question": "Which one of the following, if true, does most to explain the surprising result?", "answers": "['Customers who bought the Maxlast hammers last week commonly mentioned the sale as their reason for buying a hammer at that time.', 'The hardware store circulated flyers that publicized the sale prices on Maxlast hammers.', \"For the first several seconds after shoppers enter a store, they do not take detailed notice of the store's merchandise.\", 'In general, a single item that is on sale will not motivate shoppers to make a special trip to a store.']", "label": 2 }, { "id": "train_956", "context": "Addiction\"has been defined as \"dependence on and abuse of a psychoactive substance. \" Dependence and abuse do not always go hand in hand, however. For example, cancer patients can become dependent on morphine to relieve their pain, but this is not abusing the drug. Correspondingly, a person can abuse a drug without being dependent on it. Ttherefore, the definition of \"addiction\" is incorrect.", "question": "The relevance of the example of cancer patients to the argument depends on the assumption that", "answers": "['cancer patients often become dependent on morphine', 'cancer patients who are dependent on morphine are addicted to it', 'cancer patients never abuse morphine', 'cancer patients who abuse a drug are dependent on it']", "label": 1 }, { "id": "train_957", "context": "Police chief: This department' s officers are, of course, prohibited from drinking on the job. However, there is one exception: it is extremely valuable for officers to work undercover to investigate nightclubs that have chronic crime problems, and officers may drink in moderation during such work.", "question": "Which one of the following, if true, most helps to justify the exception to the police department's rule stated above?", "answers": "['Only very experienced police officers are allowed to work undercover investigating nightclubs.', 'Over the last several years, the police department has significantly increased its undercover operations in nightclubs.', 'Many nightclub patrons would suspect that people in a nightclub who refrained from drinking were police officers.', 'For the most part, the public is aware that police officers are allowed to drink during undercover operations in nightclubs.']", "label": 2 }, { "id": "train_958", "context": "Until recently it was widely believed that only a limited number of species could reproduce through parthenogenesis, reproduction by a female alone. But lately, as interest in the topic has increased, parthenogenesis has been found in a variety of unexpected cases, including sharks and Komodo dragons. So the number of species that can reproduce through parthenogenesis must be increasing.", "question": "The reasoning in the argument is most vulnerable to criticism on the grounds that the argument", "answers": "['takes ignorance of the occurrence of something as conclusive evidence that it did not occur', 'takes for granted that because one thing follows another, the one must have been caused by the other', 'overlooks a crucial difference between two situations that the argument presents as being similar', 'equates mere interest in a subject with real understanding of that subject']", "label": 0 }, { "id": "train_959", "context": "Astronaut: Any moon, by definition, orbits some planet in a solar system. So, the moons in solar system S4 all orbit the planet Alpha.", "question": "The astronaut's conclusion follows logically if which one of the following is assumed?", "answers": "['There is at least one moon that orbits Alpha.', 'Every moon in S4 orbits the same planet.', 'Every planet in S4 is orbited by more than one moon', 'Alpha is the only planet in S4.']", "label": 3 }, { "id": "train_960", "context": "A local marsh would need to be drained before the proposed office complex could be built. Such marshes often play crucial roles in purifying groundwater and there has been no scientific assessment of the marsh' s role in maintaining the quality of the city' s well water. The city should ttherefore block the proposed office complex pending such an assessment.", "question": "The principle underlying the argument above is most similar to the principle underlying which one of the following arguments?", "answers": "['Building a light rail line to serve the downtown core could involve cost overruns and expensive delays, but traffic congestion will become intolerable within ten years without a light rail line. A light rail line should ttherefore be built.', \"Defective products can cost an appliance manufacturer millions of dollars because of product recalls and lawsuits. Yova Corporation's new line of appliances has not yet been thoroughly tested for defects. Thus, Yova should not bring its new line to market at this time.\", 'Building the new highway along the proposed northern route would inevitably damage an adjoining wilderness area. The highway would also cause environmental damage if it follows the proposed southern route, but it would not harm any wilderness areas. Thus, if the highway is built, it should be built on the southern route.', 'A laboratory safety check of a portable grill requires a week of uninterrupted testing. The new portable grill has been at the testing lab for a week, but the testing could not begin until the paperwork arrived three days later. Ttherefore, no report on the results of the safety check should be released at this time.']", "label": 1 }, { "id": "train_961", "context": "It is inaccurate to say that a diet high in refined sugar cannot cause adult-onset diabetes, since a diet high in refined sugar can make a person overweight, and being overweight can predispose a person to adult-onset diabetes.", "question": "The argument is most parallel, in its logical structure, to which one of the following?", "answers": "['It is inaccurate to say that being in cold air can cause a person to catch a cold, since colds are caused by viruses, and viruses flourish in warm, crowded places.', 'It is inaccurate to say that Alexander the Great was a student of Plato; Alexander was a student of Aristotle and Aristotle was a student of Plato.', 'It is correct to say that overfertilization is the primary cause of lawn disease, since fertilizer causes lawn grass to grow rapidly and rapidly growing grass has little resistance to disease.', 'It is incorrect to say that inferior motor oil cannot cause a car to get poorer gasoline mileage, since inferior motor oil can cause engine valve deterioration, and engine valve deterioration can lead to poorer gasoline mileage.']", "label": 3 }, { "id": "train_962", "context": "The editor of a magazine has pointed out several errors of spelling and grammar committed on a recent TV program. But she can hardly be trusted to pass judgment on such matters: similar errors have been found in her own magazine.", "question": "The flawed reasoning in the argument above is most similar to that in which one of the following?", "answers": "['Your regulatory agency cannot condemn our product as unsafe: selling it is allowed under an existing-product clause.', \"Your newspaper cannot be trusted with the prerogative to criticize the ethics of our company: you misspelled our president's name.\", 'Your teen magazine should not run this feature on problems afflicting modern high schools: your revenue depends on not alienating the high school audience.', 'Your news program cannot be trusted to judge our hiring practices as unfair: you yourselves unfairly discriminate in hiring and promotion decisions.']", "label": 3 }, { "id": "train_963", "context": "Genetic analyses show that all varieties of domestic dogs are descendants of the wolf, and studies show that wolves have brains nearly twice the size of those of dogs of comparable size and that wolves are correspondingly more intelligent. But given that there are about 38 thousand wolves in North America, while there are over 50 million domestic dogs, it is obvious that dogs have been amply compensated, from an evolutionary standpoint, for the losses produced by their association with humanity.", "question": "Which one of the following principles most helps to justify the reasoning above?", "answers": "['Evolutionary success of a species should be measured by the number of individuals in that species rather than by the abilities of those individuals.', \"It need not be the case that an organism's chances of survival will be enhanced by its having a larger brain and a higher intelligence.\", 'Evolutionary success is not always attained by the group or species whose members are most fit.', 'Evolutionary success of a species is determined by genetic rather than environmental factors.']", "label": 0 }, { "id": "train_964", "context": "A company is considering changing its policy concerning daily working hours. Currently, this company requires all employees to arrive at work at 8 a. m. The proposed policy would permit each employee to decide when to arrive-from as early as 6 a. m. to as late as 11 a. m.", "question": "The adoption of this policy would be most likely to decrease employees' productivity if the employees' job functions required them to", "answers": "['consult at least once a day with employees from other companies', 'work without interruption from other employees', \"submit their work for a supervisor's eventual approval\", 'interact frequently with each other throughout the entire workday']", "label": 3 }, { "id": "train_965", "context": "Many elementary schools have recently offered computer-assisted educational programs. Students' reactions after several years have been decidedly mixed. Whereas students have found computers very useful in studying arithmetic, they have found them of little help in studying science, and of no help at all with their reading and writing skills.", "question": "Which one of the following, if true, most helps to explain the students' mixed reactions?", "answers": "['Of the disciplines and skills mentioned, the exactness of arithmetic makes it most suitable to computer-assisted education.', 'Young students are more likely to maintain interest in training programs that use the newest computers and video graphics than in those that do not.', 'Students in these schools began reading and doing arithmetic before learning to use computers.', 'Many elementary school teachers are reluctant to use computer technology in their classrooms.']", "label": 0 }, { "id": "train_966", "context": "A large apartment building in a city was assessed by a city environmental bureau for the comparatively high consumption of electricity in the building, and the landlord was threatened with a possible fine if the consumption did not drop significantly in a specified period. While all built-in appliances and lighting in common areas are energy efficient, the landlord found, upon doing an inspection, many residents were using older portable heaters that are not very energy efficient. The landlord decided to send each resident a detailed letter about the electric energy consumption of these older portable heaters, recommending newer more energy efficient models that could be purchased at local stores. He hopes the letter will motivate enough residents to buy more energy efficient heaters.", "question": "Which of the following, if true, would provide most support for the prediction that the landlord's letter will have its intended effect?", "answers": "['Throughout the city, this same environmental agency threatened other landlords and business owners with fines for various reasons, and in almost every case, the concerned parties were able to make sufficient changes to avoid the fines.', 'Those apartments in the building with southern exposure get abundant sunlight, which has a substantial warming effect, and many of those residents do not use portable heaters.', 'This landlord owns six apartment buildings of varying sizes in the city, and at each of the other five, the average electrical consumption is at or below the citywide average for buildings of that size.', 'Residents pay for their own monthly electrical use, and the rates in this city are high, so consuming less electricity would result in a sizable monthly savings.']", "label": 3 }, { "id": "train_967", "context": "During 2003, the number of people in the city of M who received unemployment compensation from the government doubled, even though the number of people in M who did not have jobs and qualified for unemployment remained unchanged.", "question": "Which one of the following, if true, most helps to resolve the apparent discrepancy in the information above?", "answers": "['During 2003, the number of applicants for unemployment assistance in M who were rejected on the basis that they had not been unemployed for a long enough period of time was approximately the same as it had been in 2002.', 'During 2003, many residents of a nearby city lost their jobs and moved to M in search of work.', \"During 2003, M's program of rent assistance for low-income tenants advertised widely, and then informed all applicants about other assistance programs for which they would be qualified.\", 'In 2002, the M social service department estimated the number of people in M who might be eligible for the unemployment program, and then informed the M City Council of the total amount of assistance likely to be needed.']", "label": 2 }, { "id": "train_968", "context": "During the period in which there are no competitive races, two runners--Runners A and Runners B--take part in an experiment measuring their VO2 max, the volume of oxygen an athlete can use. During these sessions, the runners engaged in moderate aerobic activity, or a sustained heart rate between 146-154 beats per minute. At the end of the sessions, Runner A had a greater VO2 max than did Runner B. Ttherefore, once the two runners begin identical intensive training--sessions involving over 168 beats per minute-- for the race season, Runner A will continue to have the greater VO2 max, assuming that neither become injured and that both train with similar intensity.", "question": "Which of the following is an assumption upon which the argument rests?", "answers": "['Runner A and Runner B had similar VO2 maxes upon entering the study.', \"The amount one trains does not influence one's VO2 max.\", 'Regarding their VO2 maxes, runners respond equally to intensive training.', 'Intensive training involves sessions in which athletes maintain a heartbeat over 168 beats per minute.']", "label": 2 }, { "id": "train_969", "context": "Political scientist: All governments worthy of respect allow their citizens to dissent from governmental policies. No government worthy of respect leaves minorities unprotected. Thus any government that protects minorities permits criticism of its policies.", "question": "The flawed pattern of reasoning in which one of the following most closely parallels that in the political scientist's argument?", "answers": "['All jazz musicians are capable of improvising and no jazz musician is incapable of reading music. Ttherefore all musicians who can read music can improvise.', 'Politicians are admirable if they put the interests of those they serve above their own interests. So politicians who sometimes ignore the interests of their own constituents in favor of the nation as a whole deserve admiration, for they are putting the interests of those they serve above their own.', 'Some intellectuals are not socially active, and no intellectual is a professional athlete. Ttherefore any professional athlete is socially active.', \"First-person narratives reveal the thoughts of the narrator but conceal those of the other characters. Some third-person narratives reveal the motives of every character. Thus books that rely on making all characters' motives apparent should be written in the third person.\"]", "label": 0 }, { "id": "train_970", "context": "Pro-Teet Insurance Company has recently been paying out more on car-theft claims than it expected. Cars with special antitheft devices or alarm systems are much less likely to be stolen than are other cars. Consequently Pro-Teet, as part of an effort to reduce its annual payouts, will offer a discount to holders of car-theft policies if their cars have antitheft devices or alarm systems.", "question": "Which of the following, if true, provides the strongest indication that the plan is likely to achieve its goal?", "answers": "['The decrease in the risk of car theft conferred by having a car alarm is greatest when only a few cars hav. e such alarms.', 'Currently, Pro-Teet cannot legally raise the premiums it charges for a given amount of insurance against car theft.', 'In one or two years, the discount that Pro-Teet is offering will amount to more than the cost of buying certain highly effective antitheft devices.', 'The amount Pro-Teet has been paying out on car theft claims has been greater for some models of car than for others.']", "label": 2 }, { "id": "train_971", "context": "Commentator: The theory of trade retaliation states that countries closed out of any of another country's markets should close some of their own markets to the other country in order to pressure the other country to reopen its markets. If every country acted according to this theory, no country would trade with any other.", "question": "The commentator's argument relies on which of the following assumptions?", "answers": "['For any two countries, at least one has some market closed to the other.', 'Countries close their markets to foreigners to protect domestic producers.', 'Trade disputes should be settled by international tribunal.', 'No country actually acts according to the theory of trade retaliation.']", "label": 0 }, { "id": "train_972", "context": "Editorial: Our society has a vested interest in maintaining a political system in which candidates are free to adhere to their principles. Yet campaigning for elected office is extremely costly, and because only the wealthiest individuals are able to finance their own political campaigns, most candidates must seek funding from private sources. In so doing, the candidates are almost invariably obliged to compromise their principles. Thus, government itself should assume the cost of candidates' campaigns.", "question": "Which one of the following principles, if valid, most helps to justify the conclusion as it is drawn in the argument?", "answers": "['Candidates should not run for elected office if doing so would compel the candidates to compromise their principles.', 'Voters should not support a candidate if that candidate is known to have accepted funding from private sources.', 'Candidates wealthy enough to finance their own political campaigns should not be permitted to raise additional funds from private sources.', 'The government should finance a given activity if doing so will further a vested interest of society.']", "label": 3 }, { "id": "train_973", "context": "Criticism that the press ignores positive stories and focuses solely on salacious stories neglects to consider that the press must make a profit to survive. Because of the need to make a profit, the press must focus on stories that will draw an audience. The only alternative to a profit-driven press is a government-supported press. A government-supported press would have pressure to curtail any critique of the government, thereby limiting the power of the press.", "question": "It can properly be inferred from the passage that the power of the press", "answers": "['comes from its ability to tell a good story', 'comes from its ability to critique the government', 'comes from its desire to cover salacious stories', 'comes from its desire to seek the truth']", "label": 1 }, { "id": "train_974", "context": "Although it is unwise to take a developmental view of an art like music -- as if Beethoven were an advance over Josquin, or Miles Davis an advance over Louis Armstrong -- there are ways in which it makes sense to talk about musical knowledge growing over time. We certainly know more about certain sounds than was known five centuries ago; that is, we understand how sounds that earlier composers avoided can be used effectively in musical compositions. For example, we now know how the interval of the third, which is considered dissonant, can be used in compositions to create consonant musical phrases.", "question": "Which one of the following most accurately expresses the main conclusion of the argument?", "answers": "['Sounds that were once considered dissonant are more pleasing to modern listeners.', 'Our understanding of music can improve over the course of time.', 'It is unwise to say that one composer is better than another.', 'Sounds that were never used in past musical compositions are used today.']", "label": 1 }, { "id": "train_975", "context": "Editorial: A recently passed law limits freedom of speech in order to silence dissenters. It has been said that those who are ignorant of history will repeat its patterns. If this is true, then those responsible for passing the law must be ignorant of a great deal of history. Historically, silencing dissenters has tended to promote undemocratic policies and the establishment of authoritarian regimes.", "question": "The editorialist's reasoning is flawed in that it fails to take into account that", "answers": "['certain freedoms might sometimes need to be limited in order to ensure the protection of certain other freedoms', 'even those who are not ignorant of history may repeat its patterns', 'the law may have other purposes in addition to silencing dissenters', 'some historical accounts report that legal restrictions on freedom of speech have occasionally undermined the establishment of authoritarian regimes']", "label": 1 }, { "id": "train_976", "context": "A gift is not generous unless it is intended to benefit the recipient and is worth more than what is expected or customary in the situation; a gift is selfish if it is given to benefit the giver or is less valuable than is customary.", "question": "Which one of the following judgments most closely conforms to the principle above?", "answers": [ "Olga gives her daughter a computer as a graduation gift. Since this is the gift that all children in Olga's family receive for graduation, it is not generous.", "Emily gives her brother a year's membership in a health club. She thinks that this will allow her brother to get the exercise he needs. However, the gift is selfish because Emily's brother is hurt and offended by it.", "Amanda gives each of her clients an expensive bottle of wine every year. Amanda's gifts are generous, since they cause the clients to continue giving Amanda business.", "Charles, who hates opera, was given two expensive tickets to the opera. He in turn gave them to his cousin, who loves opera, as a birthday gift. Charles's gift was selfish because he paid nothing for the tickets." ], "label": 0 }, { "id": "train_977", "context": "Birds and mammals can be infected with West Nile virus only through mosquito bites. Mosquitoes, in turn, become infected with the virus when they bite certain infected birds or mammals. The virus was originally detected in northern Africa and spread to North America in the 1990s. Humans sometimes catch West Nile virus, but the virus never becomes abundant enough in human blood to infect a mosquito.", "question": "The statements above, if true, most strongly support which one of the following?", "answers": "['Some people who become infected with West Nile virus never show symptoms of illness.', 'West Nile virus is most common in those parts of North America with the highest density of mosquitoes.', 'West Nile virus will never be a common disease among humans.', 'West Nile virus was not carried to North America via an infected person.']", "label": 3 }, { "id": "train_978", "context": "Archaeologists excavating a Neanderthal campsite found discarded gazelle teeth there whose coloration indicated that gazelles had been hunted throughout the year. The archaeologists concluded that the Neanderthals had inhabited the campsite year-round and thus were not nomadic. In contrast, the archaeologists cite a neighboring campsite of nomadic Cro-Magnons that contained teeth from gazelles all killed during the same season.", "question": "Which one of the following, if true, most seriously weakens the archaeologists' reasoning?", "answers": "['Cro-Magnons usually followed the migrations of the animals they hunted.', 'Cro-Magnons and Neanderthals sometimes exchanged tools.', 'Neanderthals saved gazelle teeth for use in religious rituals and later discarded them.', 'Gazelles inhabited the area around the campsites year-round.']", "label": 2 }, { "id": "train_979", "context": "Consumer advocate: Even if one can of fruit or vegetables weighs more than another, the heavier can does not necessarily contain more food. Canned fruits and vegetables are typically packed in water, which can make up more than half the total weight of the can' s contents. And nothing stops unscrupulous canning companies from including more water per can than others include.", "question": "Which one of the following most accurately expresses the conclusion drawn in the consumer advocate's argument?", "answers": "['The heavier of two cans of fruit or vegetables does not necessarily contain more food than the lighter of the two cans contains.', 'Nothing stops unscrupulous canning companies from including more water per can than others include.', 'The heavier of two cans of fruits or vegetables may include more water than the lighter of the two cans contains.', \"The weight of the water in a can of fruit or vegetables can be more than half the total weight of the can's contents.\"]", "label": 0 }, { "id": "train_980", "context": "A physically active lifestyle has been shown to help increase longevity. In the Wistar region of Bellaria, the average age at death is considerably higher than in any other part of the country. Wistar is the only mountainous part of Bellaria. A mountainous terrain makes even such basic activities as walking relatively strenuous; it essentially imposes a physically active lifestyle on people. Clearly, this circumstance explains the long lives of people in Wistar.", "question": "Which of the following, if true, most seriously weakens the argument?", "answers": "['Many people who live in the Wistar region have moved there in middle age or upon retirement.', 'Per capita spending on recreational activities is no higher in Wistar than it is in other regions of Bellaria.', 'In Bellaria all medical expenses are paid by the government, so that personal income does not affect the quality of health care a person receives.', 'The Wistar region is one of Bellaria, s least populated regions.']", "label": 0 }, { "id": "train_981", "context": "Style manual: Archaic spellings and styles of punctuation in direct quotations from older works are to be preserved if they occur infrequently and do not interfere with a reader' s comprehension. However, if they occur frequently, the editor may modernize them, inserting a note with an explanation to this effect in the text, or if similar modernizing has been done in more than one quotation, inserting a general statement in the preface. On the other hand, obvious typographical errors in quotations from modern works may be corrected without explanation.", "question": "Which one of the following follows logically from the statements above?", "answers": "['An editor may modernize punctuation directly quoted from an older work if that punctuation occurs frequently and interferes with reader comprehension.', 'If an editor corrects the spelling of a quoted word and the word occurs only once in the text, then an explanation should appear in a note or in the text.', 'If an editor modernizes only one of several similar instances of quoted archaic punctuation, an explanation should appear in the preface of the work.', 'An editor may modernize an archaic spelling of a word found in a modern work without providing an explanation.']", "label": 0 }, { "id": "train_982", "context": "Humans are supposedly rational: in other words, they have a capacity for well-considered thinking and behavior. This is supposedly the difference that makes them superior to other animals. But humans knowingly pollute the world' s precious air and water and, through bad farming practices, deplete the soil that feeds them. Thus, humans are not rational after all, so it is absurd to regard them as superior to other animals.", "question": "The reasoning above is flawed in that it", "answers": "['neglects to show that the irrational acts perpetrated by humans are not also perpetrated by other animals', 'presumes, without offering justification, that humans are no worse than other animals', 'fails to recognize that humans may possess a capacity without displaying it in a given activity', 'takes for granted that humans are aware that their acts are irrational']", "label": 2 }, { "id": "train_983", "context": "The number of applications for teaching positions in Newtown' s public schools was 5. 7 percent lower in 1993 than in 1985 and 5. 9 percent lower in 1994 than in 1985. Despite a steadily growing student population and an increasing number of teacher resignations, however, Newtown does not face a teacher shortage in the late 1990' s.", "question": "Which of the following, if true, would contribute most to an explanation of the apparent discrepancy above?", "answers": [ "New housing developments planned for Newtown are slated for occupancy in 1997 and are expected to increase the number of elementary school students in Newtown's public schools by 12 percent.", "In 1993 Newtown's public schools received 40 percent more applications for teaching positions than there were positions available.", "The Newtown school board does not contemplate increasing the ratio of students to teachers in the 1990's.", "Teachers' colleges in and near Newtown produced fewer graduates in 1994 than in 1993." ], "label": 1 }, { "id": "train_984", "context": "The museum' s night security guard maintains that the thieves who stole the portrait did not enter the museum at any point at or above ground level. Ttherefore, the thieves must have gained access to the museum from below ground level.", "question": "The flawed pattern of reasoning in the argument above is most similar to that in which one of the following?", "answers": "['As had generally been expected, not all questionnaires were sent in by the official deadline. It follows that plans must have been made for the processing of questionnaires received late.', \"The store's competitors claim that the store, in selling off the shirts at those prices, neither made any profit nor broke even. Consequently, the store's customers must have been able to buy shirts there at less than the store's cost.\", 'The product label establishes that this insecticide is safe for both humans and pets. Ttherefore, the insecticide must also be safe for such wild mammals as deer and rabbits.', 'If the census is to be believed, the percentage of men who are married is higher than the percentage of women who are married. Thus, the census must show a higher number of men than of women overall.']", "label": 1 }, { "id": "train_985", "context": "If the jury did not return a verdict, there would still be media trucks outside the courthouse. There are no media trucks outside the courthouse, so the jury must have returned a verdict.", "question": "The pattern of reasoning in the argument above is most similar to that in which one of the following arguments?", "answers": "['If Ralph had told Manuela about the problem, Manuela would have solved it. But Ralph did not tell Manuela about the problem, so someone else must have solved it.', \"If Kay's television was not working last night, she would have gone to a movie. Her television has not been working for the past week, so she must have gone to a movie last night.\", 'If Peter did not buy a house, he would have rented an apartment. Peter did not rent an apartment, so he must have bought a house.', 'If a hurricane arises off the coast this summer, our town will see less tourism than usual. But since there will be no hurricane this summer, there will be no less tourism than usual.']", "label": 2 }, { "id": "train_986", "context": "All any reporter knows about the accident is what the press agent has said. Ttherefore, if the press agent told every reporter everything about the accident, then no reporter knows any more about it than any other reporter. If no reporter knows any more about the accident than any other reporter, then no reporter can scoop all of the other reporters. However, the press agent did not tell every reporter everything about the accident. It follows that some reporter can scoop all of the other reporters.", "question": "The argument's reasoning is flawed because the argument fails to recognize that which one of the following is consistent with the facts the argument presents?", "answers": "['The press agent may not know any more about the accident than the most knowledgeable reporter.', 'No reporter knows any more about the accident than any other reporter.', 'Even if some reporter knows more about the accident than all of the other reporters, that reporter need not scoop any other reporter.', 'Some reporter may have been told something about the accident that the reporter tells all of the other reporters.']", "label": 1 }, { "id": "train_987", "context": "For most people, the left half of the brain controls linguistic capabilities, but some people have their language centers in the right half. When a language center of the brain is damaged, for example by a stroke, linguistic capabilities are impaired in some way. Ttherefore, people who have suffered a serious stroke on the left side of the brain without suffering any such impairment must have their language centers in the right half.", "question": "Which of the following is an assumption on which the reasoning in the argument above depends?", "answers": "[\"It is impossible to determine which side of the brain contains a person's language centers if the person has not suffered damage to either side of the brain.\", \"No part of a person's brain that is damaged by a stroke ever recovers.\", 'Strokes tend to impair linguistic capabilities more severely than does any other cause of damage to language centers in the brain.', 'If there are language centers on the left side of the brain, any serious stroke affecting that side of the brain damages at least one of them.']", "label": 3 }, { "id": "train_988", "context": "Politicians often advocate increased overall economic productivity while ignoring its drawbacks. For example, attempting to increase the productivity of a corporation means attempting to increase its profitability, which typically leads to a reduction in the number of workers employed by that corporation. Thus, attempting to increase productivity in the economy as a whole may benefit business owners, but will increase the number of unemployed workers.", "question": "The reasoning in the argument is most vulnerable to criticism on the grounds that the argument", "answers": "['unfairly criticizes politicians in general on the basis of the actions of a few who are unwilling to consider the drawbacks of attempting to increase productivity', 'fails to address all potential drawbacks and benefits of attempting to increase productivity at a single corporation', 'presumes, without providing justification, that increased unemployment is sufficient reason to abandon increased productivity as an economic goal', 'fails to justify its presumption that attempting to increase productivity in the economy as a whole would produce results similar to those produced by attempting to increase productivity in a single corporation']", "label": 3 }, { "id": "train_989", "context": "On the basis of the available evidence, Antarctica has generally been thought to have been covered by ice for at least the past 14 million years. Recently, however, three-million-year-old fossils of a kind previously found only in ocean-floor sediments were discovered under the ice sheet covering central Antarctica. About three million years ago, ttherefore, the Antarctic ice sheet must temporarily have melted. After all, either severe climatic warming or volcanic activity in Antarctica' s mountains could have melted the ice sheet, thus raising sea levels and submerging the continent.", "question": "Which one of the following is the main conclusion of the argument?", "answers": "['What caused Antarctica to be submerged under the sea was the melting of the ice sheet that had previously covered the continent.', 'It is not the case that ancient fossils of the kind recently found in Antarctica are found only in ocean-floor sediments.', 'The ice sheet covering Antarctica has not been continuously present throughout the past 14 million years.', 'Antarctica is no longer generally thought to have been covered by ice for the past 14 million years.']", "label": 2 }, { "id": "train_990", "context": "While 65 percent of the eligible voters who were recently polled favor Perkins over Samuels in the coming election, the results of that poll are dubious because it was not based on a representative sample. Given that Perkins predominantly advocates the interests of the upper-middle class and that the survey was conducted at high-priced shopping malls, it is quite probable that Perkins' s supporters were overrepresented.", "question": "Which one of the following statements most accurately expresses the main conclusion of the argument?", "answers": "[\"The poll's results probably do not accurately represent the opinions of the voters in the coming election.\", 'Samuels is quite likely to have a good chance of winning the coming election.', 'The poll was intentionally designed to favor Perkins over Samuels.', \"Samuels's supporters believe that they were probably not adequately represented in the poll.\"]", "label": 0 }, { "id": "train_991", "context": "Artists have different ways of producing contours and hatching, and analysis of these stylistic features can help to distinguish works by a famous artist both from forgeries and from works genuinely by other artists. Indeed, this analysis has shown that many of the drawings formerly attributed to Michelangelo are actually by the artist Giulio Clovio, Michelangelo' s contemporary.", "question": "If the statements above are true, then which one of the following must also be true?", "answers": "[\"The contour and hatching styles used to identify the drawings of Clovio cited can be shown to be features of all Clovio's works.\", \"There is an analyzable difference between Clovio's contour and hatching styles and those of Michelangelo.\", 'Many of the drawings formerly attributed to Michelangelo are actually forgeries.', 'Contours and hatching are the main features that distinguish the drawing styles of different artists.']", "label": 1 }, { "id": "train_992", "context": "Some scientists believe that the relationship between mice and humans has, over time, diminished the ability of mice to survive in nature, so that now they must depend upon human civilization for their continued existence. This opinion, however, ignores significant facts. Despite numerous predators and humanity' s enmity, mice have distributed themselves more widely across the planet than any other mammal except humans. Mice reproduce rapidly and, more important to their survival, they have the ability to adapt to an extraordinary range of habitats. Should the environment ever become too extreme to support human life, naturalists predict that mice would be able to adapt and survive.", "question": "Which one of the following, if true, would most support the naturalists' prediction?", "answers": "['Fossil remains prove that mice inhabited North America prior to the arrival of humans.', 'Mice have colonized an island near Antarctica which is too bleak and harsh to support human life.', 'Under optimum conditions, mice reproduce every four weeks, with five to seven pups per litter.', 'The size of the mouse population is limited by the availability of food.']", "label": 1 }, { "id": "train_993", "context": "Operations at the Green Hills Gold Mine are continuously releasing mercury into the headwaters of the Apache River, and most fish in the Apache River now have mercury levels at or above the legal limit for human consumption. With the price of gold rising, forcing more and more mining at Green Hills, there seems no hope for food fishing along the Apache River.", "question": "Which of the following plans, if feasible, would allow the state to assist the food fishing industries along the Apache?", "answers": "['The state bank will start to buy up large amounts of gold, preferentially buying directly from the Green Hills mines.', 'The state will mount a public awareness campaign, educating private citizen and restaurateurs about the dangers of high levels of mercury in food fish.', 'State workers will test the algae and underwater plants for mercury, removing those above a certain mercury threshold. These algae and plants are the principal food source of the food fish.', 'Immediately downstream from Green Hills, the state will install a sophisticated ionic filtration plant, which will substantially reduce the level of heavy metals (including mercury) in the water.']", "label": 3 }, { "id": "train_994", "context": " Delta Products Inc. has recently switched at least partly from older technologies using fossil fuels to new technologies powered by electricity. The question has been raised whether it can be concluded that for a given level of output Delta' s operation now causes less fossil fuel to be consumed than it did formerly . The answer, clearly, is yes, since the amount of fossil fuel used to generate the electricity needed to power the new technologies is less than the amount needed to power the older technologies, provided level of output is held constant.", "question": "In the argument given, the two boldfaced portions play which of the following roles?", "answers": "['The first provides support for the conclusion of the argument; the second identifies the content of that conclusion.', 'Each provides support for the conclusion of the argument.', 'The first states the conclusion of the argument; the second calls that conclusion into question.', 'The first identifies the content of the conclusion of the argument; the second provides support for that conclusion.']", "label": 0 }, { "id": "train_995", "context": "The ruins of the prehistoric Bolivian city of Tiwanaku feature green andacite stones weighing up to 40 tons. These stones were quarried at Copacabana, which is across a lake and about 90 kilometers away. Archaeologists hypothesize that the stones were brought to Tiwanaku on reed boats. To show this was possible, experimenters transported a 9-ton stone from Copacabana to Tiwanaku using a reed boat built with locally available materials and techniques traditional to the area.", "question": "Which one of the following would be most useful to know in order to evaluate the support for the archaeologists' hypothesis?", "answers": "['whether the traditional techniques for building reed boats were in use at the time Tiwanaku was inhabited', 'whether the reed boat built for the experimenters is durable enough to remain usable for several years', 'whether green andacite stones quarried at the time Tiwanaku was inhabited were used at any sites near Copacabana', 'whether the green andacite stones at Tiwanaku are the largest stones at the site']", "label": 0 }, { "id": "train_996", "context": "Lathyrism, a debilitating neurological disorder caused by the consumption of the legume Lathyrus sativus, is widespread among the domestic animals of some countries. Attempts to use rats to study lathyrism have generally failed. Rats that ingested Lathyrus sativus did not produce the symptoms associated with the disorder.", "question": "Which one of the following is most strongly supported by the information above?", "answers": "['Laboratory conditions are not conducive to the development of lathyrism.', 'The physiology of rats is radically different from that of domestic animals.', 'The rats did not consume as much Lathyrus sativus as did the domestic animals that contracted lathyrism.', 'Not all animal species are equally susceptible to lathyrism.']", "label": 3 }, { "id": "train_997", "context": "A large company has been convicted of engaging in monopolistic practices. The penalty imposed on the company will probably have little if any effect on its behavior. Still, the trial was worthwhile, since it provided useful information about the company' s practices. After all, this information has emboldened the company' s direct competitors, alerted potential rivals, and forced the company to restrain its unfair behavior toward customers and competitors.", "question": "Which one of the following most accurately expresses the overall conclusion drawn in the argument?", "answers": "[\"The light shed on the company's practices by the trial has emboldened its competitors, alerted potential rivals, and forced the company to restrain its unfair behavior.\", 'The penalty imposed on the company will likely have little or no effect on its behavior.', 'The penalty imposed on the company in the trial should have been larger.', \"The company's trial on charges of engaging in monopolistic practices was worthwhile.\"]", "label": 3 }, { "id": "train_998", "context": "In her recent book a psychologist described several cases that exhibit the following pattern: A child, denied something by its parent, initiates problematic behavior such as screaming; the behavior escalates until finally the exasperated parent acquiesces to the child' s demand. At this point the child, having obtained the desired goal, stops the problematic behavior, to the parent' s relief. This self-reinforcing pattern of misbehavior and accommodation is repeated with steadily increasing levels of misbehavior by the child.", "question": "The cases described by the psychologist illustrate each of the following generalizations EXCEPT:", "answers": "[\"Parents, by their choices, can inadvertently increase their child's level of misbehavior.\", 'A child can develop problematic behavior patterns as a result of getting what it wants.', \"A child can unintentionally influence a parent's behavior in ways contrary to the child's intended goals.\", \"A child can get what it wants by doing what its parent doesn't want it to do.\"]", "label": 2 }, { "id": "train_999", "context": "Psychiatrist: In treating first-year students at this university, I have noticed that those reporting the highest levels of spending on recreation score at about the same level on standard screening instruments for anxiety and depression as those reporting the lowest levels of spending on recreation. This suggests that the first-year students with high levels of spending on recreation could reduce that spending without increasing their anxiety or depression.", "question": "Each of the following, if true, strengthens the psychiatrist's argument EXCEPT:", "answers": "['The screening instruments used by the psychiatrist are extremely accurate in revealing levels of anxiety and depression among university students.', 'At other universities, first-year students reporting the highest levels of spending on recreation also show the same degree of anxiety and depression as do those reporting the lowest levels of such spending.', 'Among adults between the ages of 40 and 60, increased levels of spending on recreation are strongly correlated with decreased levels of anxiety and depression.', \"Several of the psychiatrist's patients who are first-year students at the university have reduced their spending on recreation from very high levels to very low levels without increasing their anxiety or depression.\"]", "label": 2 }, { "id": "train_1000", "context": "A graduate degree in policymaking is necessary to serve in the presidential cabinet. In addition, every member of the cabinet must pass a security clearance. No person with a felony can pass a security clearance. Rick holds a graduate degree in policymaking, but he has a conviction for driving under the influence. Ttherefore, Rick cannot serve in the cabinet.", "question": "The argument's conclusion follows logically if which one of the following is assumed?", "answers": "['Holding a graduate degree is less important than having a felony conviction.', 'Driving under the influence is a felony.', 'Anyone without a felony conviction can pass a security clearance.', \"Rick's conviction for drunk driving calls his character in question.\"]", "label": 1 }, { "id": "train_1001", "context": "An undergraduate degree is necessary for appointment to the executive board. Further, no one with a felony conviction can be appointed to the board. Thus, Murray, an accountant with both a bachelor' s and a master' s degree, cannot be accepted for the position of Executive Administrator, since he has a felony conviction.", "question": "The argument's conclusion follows logically if which one of the following is assumed?", "answers": "['An undergraduate degree is not necessary for acceptance for the position of Executive Administrator.', \"Anyone with a master's degree and without a felony conviction is eligible for appointment to the executive board.\", 'Only candidates eligible for appointment to the executive board can be accepted for the position of Executive Administrator.', 'The felony charge on which Murray was convicted is relevant to the duties of the position of Executive Administrator.']", "label": 2 }, { "id": "train_1002", "context": "A company that imports and sells collectibles sought to have some of its collectible figurines classified as toys, which are subject to lower import tariffs than collectibles. The company argued that the figurines amuse customers, just as toys do. However, the government agency responsible for tariffs rejected the company' s request on the grounds that the figurines are marketed as collector' s items rather than toys.", "question": "Which one of the following principles, if valid, most helps to justify the government agency's decision?", "answers": "['The tariff classification of an item should depend primarily on how the item is marketed.', 'Objects that are developed primarily to provide amusement should be subject to lower tariffs than other objects.', 'When importing products, a company should seek the tariff classification that results in the lowest tariffs.', 'A company should market its products as collectibles rather than toys if doing so enables it to sell them for higher prices.']", "label": 0 }, { "id": "train_1003", "context": "Increasing Mary' s allowance would cause her to become spoiled and harm her chances of making new friends. Since Mary' s parents would not do anything to cause their daughter to be unable to make new friends, they will not increase Mary' s allowance.", "question": "Which of the following patterns of reasoning is most similar to that in the paragraph above?", "answers": "['In school districts that use standardized tests, student achievement is lower than in other districts. Ttherefore, eliminating standardized tests will increase student achievements in these districts.', 'Children who attend schools with diverse populations are more likely to participate in community service projects. Since diverse populations in schools lead to increased community service participation, then all schools should have diverse populations.', 'If the new budget does not pass this election, a new school cannot be built to alleviate the overcrowding at the existing school. Voters will not continue to tolerate overcrowding at the existing school so the new budget will pass.', 'Older children function less well at earlier hours than younger children. Ttherefore, it makes more sense to have the elementary school students start earlier than high school students because this may lead to increased productivity.']", "label": 2 }, { "id": "train_1004", "context": "Scientist: There is little doubt that the ice ages were caused by the unusually rich growth of vegetation worldwide. Since vegetation converts carbon dioxide into oxygen, excessive vegetation would have depleted the carbon dioxide in the atmosphere. Carbon dioxide helps the atmosphere retain the sun' s heat. Thus, if this carbon dioxide is depleted, the earth cools significantly, thereby causing an ice age.", "question": "Which one of the following most accurately expresses the main conclusion of the scientist's argument?", "answers": "['If unusually rich growth of vegetation caused the ice ages, it undoubtedly did so by depleting the carbon dioxide in the atmosphere.', 'An excessive growth of vegetation causes the carbon dioxide in the atmosphere to be depleted.', 'If the carbon dioxide in the atmosphere is depleted, the earth cools significantly, thereby causing an ice age.', 'Unusually rich growth of vegetation worldwide was almost certainly the cause of the ice ages.']", "label": 3 }, { "id": "train_1005", "context": "Politician: Those economists who claim that consumer price increases have averaged less than 3 percent over the last year are mistaken. They clearly have not shopped anywhere recently. Gasoline is up 10 percent over the last year; my auto insurance, 12 percent; newspapers, 15 percent; propane, 13 percent; bread, 50 percent.", "question": "The reasoning in the politician's argument is most vulnerable to criticism on the grounds that the argument", "answers": "['attempts to persuade by making an emotional appeal', 'fails to show that the economists mentioned are not experts in the area of consumer prices', 'uses evidence drawn from a small sample that may well be unrepresentative', 'impugns the character of the economists rather than addressing their arguments']", "label": 2 }, { "id": "train_1006", "context": "Veterinarians generally derive some of their income from selling several manufacturers' lines of pet-care products. Knowing that pet owners rarely throw away mail from their pet' s veterinarian unread, one manufacturer of pet-care products offered free promotional materials on its products to veterinarians for mailing to their clients. Very few veterinarians accepted the offer, however, even though the manufacturer's products are of high quality.", "question": "Which of the following, if true, most helps to explain the veterinarian's reaction to the manufacturer's promotional scheme?", "answers": "['Veterinarians sometimes recommend that pet owners use products formulated for people when no suitable product specially formulated for animals is available.', 'Many pet owners have begun demanding quality in products they buy for their pets that is as high as that in products they buy for themselves.', \"The special promotional materials were intended as a supplement to the manufacturer's usual promotional activities rather than as a replacement for them.\", \"The manufacturer's products, unlike most equally good competing products sold by veterinarians, are also available in pet stores and in supermarkets.\"]", "label": 3 }, { "id": "train_1007", "context": "Amateur gardeners who plant based on the phases of the moon tend to get better results than those who do not. This seems surprising since the phases of the moon do not affect how plants grow. An alternative practice often found among amateur gardeners is to plant during the first warm spell of spring, which leads to problems when a frost follows. So, amateur gardeners who use the phases of the moon are less likely to lose plants to a frost.", "question": "The argument requires assuming which one of the following?", "answers": "['Amateur gardeners who use the phases of the moon tend to plant different types of plants than do other amateur gardeners.', 'Amateur gardeners cannot improve their results unless they understand why their methods work as they do.', 'The phases of the moon affect whether a frost follows the first warm spell of spring.', 'Using the phases of the moon usually leads amateur gardeners to plant later in the spring than those planting at the first warm spell.']", "label": 3 }, { "id": "train_1008", "context": "Cognitive scientist: Using the pioneering work of comparative psychologist Gordon Gallup as a model, several studies have investigated animals' capacity for mirror self-recognition (MSR). Most animals exposed to a mirror respond only with social behavior, such as aggression. However, in the case of the great apes, repeated exposure to mirrors leads to self-directed behaviors, such as exploring the inside of the mouth, suggesting that these animals recognize the reflection as an image of self. The implication of these studies is that the great apes have a capacity for self-awareness unique among nonhuman species.", "question": "The cognitive scientist makes which of the following assumptions in the argument above?", "answers": "['If an animal does not have the capacity for MSR, it does not have the capacity for self-awareness.', 'When exposed to a mirror, all animals display either social behavior or self-directed behavior.', 'Animals that do not exhibit MSR may demonstrate a capacity for self-awareness in other ways.', 'If a researcher exposes an animal to a mirror and that animal exhibits social behavior, that animal is incapable of being self-aware.']", "label": 0 }, { "id": "train_1009", "context": "The cost of producing radios in Country Q is 10 percent less than the cost of producing radios in Country Y. Even after transportation fees and tariff charges are added, it is still cheaper for a company to import radios from Country Q to Country Y than to produce radios in Country Y.", "question": "The statements above, if true, best support which of the following assertions?", "answers": "['Importing radios from Country Q to Country Y will eliminate 10 percent of the manufacturing jobs in Country Y.', 'The tariff on a radio imported from Country Q to Country Y is less than 10 percent of the cost of manufacturing the radio in Country Y.', 'Labor costs in Country Q are 10 percent below those in Country Y.', 'It takes 10 percent less time to manufacture a radio in Country Q than it does in Country Y.']", "label": 1 }, { "id": "train_1010", "context": "Recent investigations of earthquakes have turned up a previously unknown type of seismic shock, known as a displacement pulse, which is believed to be present in all earthquakes. Alarmingly, high-rise buildings are especially vulnerable to displacement pulses, according to computer models. Yet examination of high-rises within cities damaged by recent powerful earthquakes indicates little significant damage to these structures.", "question": "Which one of the following, if true, contributes to a resolution of the apparent paradox?", "answers": "['Computer models have been very successful in predicting the effects of other types of seismic shock.', 'Scientific predictions based on computer models often fail when tested in the field.', 'The displacement pulses made by low- and medium-intensity earthquakes are much less powerful than those made by the strongest earthquakes.', 'While displacement pulses have only recently been discovered, they have accompanied all earthquakes that have ever occurred.']", "label": 1 }, { "id": "train_1011", "context": "In Sheldon most bicyclists aged 18 and over have lights on their bicycles, whereas most bicyclists under the age of 18 do not. It follows that in Sheldon most bicyclists who have lights on their bicycles are at least 18 years old.", "question": "Which one of the following exhibits a pattern of flawed reasoning most similar to that in the argument above?", "answers": "['Most of the people in Sheldon buy gasoline on Mondays only. But almost everyone in Sheldon buys groceries on Tuesdays only. It follows that fewer than half of the people in Sheldon buy gasoline on the same day on which they buy groceries.', 'The Sheldon Library lent more books during the week after it began lending videos than it had in the entire preceding month. It follows that the availability of videos was responsible for the increase in the number of books lent.', \"Most of the residents of Sheldon who voted in the last election are on the Conservative party's mailing list, whereas most of Sheldon's residents who did not vote are not on the list. It follows that most of the residents of Sheldon on the Conservative party's mailing list voted in the last election.\", 'In the county where Sheldon is located, every town that has two or more fire trucks has a town pool, whereas most towns that have fewer than two fire trucks do not have a town pool. It follows that Sheldon, which has a town pool, must have at least two fire trucks.']", "label": 2 }, { "id": "train_1012", "context": "In order to reduce the number of items damaged while in transit to customers, packaging consultants recommended that the TrueSave mail-order company increase the amount of packing material so as to fill any empty spaces in its cartons. Accordingly, TrueSave officials instructed the company's packers to use more packing material than before, and the packers zealously acted on these instructions and used as much as they could. Nevertheless, customer reports of damaged items rose somewhat.", "question": "Which of the following, if true, most helps to explain why acting on the consultants' recommendation failed to achieve its goal?", "answers": "['TrueSave has lost some of its regular customers as a result of the high number of damaged items they received.', 'The amount of packing material used in a carton does not significantly influence the ease with which a customer can unpack the package.', 'The change in packing policy led to an increase in expenditure on packing material and labor.', 'When packing material is compressed too densely, it loses some of its capacity to absorb shock.']", "label": 3 }, { "id": "train_1013", "context": "A survey has shown that public perceptions of comparative risks differ radically from the assessments of risk-management experts. For example, living near a nuclear power plant was judged a much greater risk by the survey respondents than it was by the experts. On the other hand, exposure to X-rays was judged a significantly lower risk by the survey respondents than it was by the experts. Psychologists have found that people tend to be more worried about risks they consider involuntary than about risks they consider voluntary. Thus the survey results were probably caused by the respondents' tendency to consider involuntary risks to be greater than they actually are.", "question": "Which one of the following is an assumption required by the argument?", "answers": "[\"The survey's respondents did not include people living near nuclear power plants.\", 'Not all of the survey respondents considered living near a nuclear power plant to be a voluntary risk.', 'It is not the case that involuntary risks are usually greater than voluntary risks.', 'People never consider risks that they are greatly worried about to be lower than they actually are.']", "label": 1 }, { "id": "train_1014", "context": "Charles: During recessions unemployment typically rises. Thus, during a recession air pollution due to automobile exhaust decreases, since fewer people commute in cars to jobs and so cars emitting pollutants into the air are used less. Darla: Why think that air pollution would decrease? During a recession fewer people can afford to buy new cars, and cars tend to emit more pollutants as they get older.", "question": "Which one of the following most accurately describes how Darla's response is related to Charles's argument?", "answers": "['It calls into question the truth of the premises that Charles uses to support his conclusion.', \"It presents an argument showing that the premises in Charles' s argument support an absurd conclusion that Charles has overlooked.\", \"It presents an additional consideration that weakens the support given to Charles's conclusion by his evidence.\", \"It argues that Charles's conclusion is true, although not for the reasons Charles gives to support that conclusion.\"]", "label": 2 }, { "id": "train_1015", "context": "Studies show that the moderate consumption of alcohol, particularly red wine, offers some health benefits. In addition, even if deemed appropriate, prohibition would be impossible since the demand is so high. However, the heavy consumption of alcohol can be addictive and deadly.", "question": "Which one of the following conclusions most logically follows from the argument?", "answers": "['Regulation of alcohol would work better than prohibition.', 'Prohibition of alcohol is ill-conceived.', 'The benefits of alcohol outweigh the costs.', 'Excessive consumption of alcohol is harmful.']", "label": 0 }, { "id": "train_1016", "context": "Publishers, like all private businesses, have always been concerned about profitability. But, while it used to be at least possible to get a book published that was unlikely to be popular, and hence unlikely to be a moneymaker, profitability is now the sole criterion. Thus, no matter how much literary merit a book possesses, it will not be published in today' s market if it looks like it will not be popular.", "question": "Which one of the following sentences most accurately expresses the main conclusion of the argument?", "answers": "['Good books are published now only if they are also likely to be popular.', 'Unpopular books are not likely to make money for the publisher.', \"Books of literary merit cannot be published in today's market.\", 'It is harder to get a book published today than it once was.']", "label": 0 }, { "id": "train_1017", "context": "People should avoid taking the antacid calcium carbonate in doses larger than half a gram, for despite its capacity to neutralize stomach acids, calcium carbonate can increase the calcium level in the blood and thus impair kidney function. Moreover, just half a gram of it can stimulate the production of gastrin, a stomach hormone that triggers acid secretion.", "question": "Which one of the following is most strongly supported by the information above?", "answers": "['Half a gram of calcium carbonate can causally contribute to both the secretion and the neutralization of stomach acids.', 'Impaired kidney function may increase the level of calcium in the blood.', 'People who avoid taking more than half a gram of calcium carbonate are less likely than average to suffer from impaired kidney function.', 'Doses of calcium carbonate smaller than half a gram can reduce stomach acid more effectively than much larger doses do.']", "label": 0 }, { "id": "train_1018", "context": "Social critic: The operas composed by Bizet and Verdi are nineteenth-century European creations, reflecting the attitudes and values in France and Italy at the end of that century. Several recent studies impugn these operas on the grounds that they reinforce in our society many stereotypes about women. But only a small minority of contemporary North Americans, namely opera lovers, have had any significant exposure to these works.", "question": "Which one of the following most accurately expresses the conclusion that the social critic's argument, as it is stated above, is structured to establish?", "answers": "['The operas of Bizet and Verdi have not significantly contributed to stereotypical images of women in contemporary North America.', 'Bizet and Verdi constructed images of women that have significantly influenced contemporary stereotypes.', 'Nineteenth-century French and Italian images of women are quite different from contemporary North American images of women.', 'People cannot be influenced by things they are not directly exposed to.']", "label": 0 }, { "id": "train_1019", "context": "It now seems clear that the significant role initially predicted for personal computers in the classroom has not become fact. One need only look to the dramatic decline in sales of computers for classroom use in the past year for proof that the fad has passed.", "question": "Which one of the following arguments contains flawed reasoning parallel to that in the argument above?", "answers": "['Mechanical translation from one language into another, not merely in narrow contexts such as airline reservations but generally, is clearly an idea whose time has come. Since experts have been working on the problem for 40 years, it is now time for the accumulated expertise to achieve a breakthrough.', 'Clearly, government legislation mandating the reduction of automobile emissions has been at least partially successful, as is demonstrated by the fact that the air of the 20 largest cities now contains smaller amounts of the major pollutants mentioned in the legislation than it did before the legislation was passed.', 'It is apparent that consumers have tired of microwave ovens as quickly as they initially came to accept this recent invention. In contrast to several years of increasing sales following the introduction of microwave ovens, sales of microwave ovens flattened last year, indicating that consumers have found relatively little use for these devices.', 'Sales of computers for home use will never reach the levels optimistically projected by manufacturers. The reason is that home use was envisioned as encompassing tasks, such as menu planning and checkbook reconciliation, that most homemakers perform in much simpler ways than using a computer would require.']", "label": 2 }, { "id": "train_1020", "context": "In a medical study of all of the residents of Groverhill, 35 people reported consulting their physician last year seeking relief from severe headaches. Those same physicians' records, however, indicate that 105 consultations occurred last year with Groverhill patients seeking relief from severe headaches. Obviously, then, many residents who consulted physicians for this condition did not remember doing so.", "question": "The reasoning in the argument is most vulnerable to criticism on the grounds that the argument", "answers": "['overlooks the possibility that residents of Groverhill visited their physicians more than once during the year for the same condition', 'fails to provide any evidence to support the claim that the residents of Groverhill have an unusually high occurrence of severe headaches', 'takes for granted that every resident of Groverhill who suffers from severe headaches would consult a physician about this condition', 'fails to consider whether any residents of Groverhill visit physicians who are not located in Groverhill']", "label": 0 }, { "id": "train_1021", "context": "The faculty of Grey County Public Schools will shortly go on strike unless the school board increases their salary. As the school board president is well aware, however, in order to increase the faculty' s salary, Grey County Public Schools would have to cut some of its after-school programs. So, some of Grey County Public Schools' after-school programs will be cut.", "question": "The conclusion above is properly drawn if which one of the following is assumed?", "answers": "[\"The school board will refuse to increase the faculty's salary.\", 'The school board president has the authority to offer the faculty their desired salary increase.', 'The faculty of Grey County Public Schools will not be going on strike.', 'The faculty of Grey County Public Schools will not accept a package of improved benefits in place of their desired wage increase.']", "label": 2 }, { "id": "train_1022", "context": "Ideally, scientific laws should display the virtues of precision and generality, as do the laws of physics. However, because of the nature of their subject matter, laws of social science often have to use terms that are imprecise: for example, one knows only vaguely what is meant by \"republicanism\" or \"class. \" As for generality, laws that apply only in certain social systems are typically the only ones possible for the social sciences.", "question": "Which one of the following statements is most strongly supported by the information above?", "answers": "['Social scientists should make an effort to construct more laws that apply to all societies.', 'All else being equal, a precise, general scientific law is to be preferred over one that is not general.', 'Terms such as \"class\" should be more precisely formulated by social scientists.', 'The laws of social science are invariably not truly scientific.']", "label": 1 }, { "id": "train_1023", "context": "Statistics from the National Booksellers Association indicate that during the last five years most bookstores have started to experience declining revenues from the sale of fiction, despite national campaigns to encourage people to read more fiction. Ttherefore, these reading campaigns have been largely unsuccessful.", "question": "Which one of the following statements, if true, most seriously weakens the argument?", "answers": "['The National Booksellers Association statistics do not include profits from selling novels by mail to overseas customers.', 'During the last five years the most profitable items in bookstores have been newspapers and periodicals rather than novels.', 'Due to the poor economic conditions that have prevailed during the last five years, most libraries report substantial increases in the number of patrons seeking books on changing careers and starting new businesses.', 'Mail order book clubs have enjoyed substantial growth in fiction sales throughout the last five years.']", "label": 3 }, { "id": "train_1024", "context": "A history book written hundreds of years ago contains several inconsistencies. Some scholars argue that because the book contains inconsistencies, the author must have been getting information from more than one source.", "question": "The conclusion cited does not follow unless", "answers": "[\"the history book's author used no source that contained inconsistencies repeated in the history book\", 'the author of the history book was familiar with all of the available source material that was relevant to the history book', 'the inconsistencies would be apparent to the average reader of the history book at the present time', 'the author of the history book was aware of the kinds of inconsistencies that can arise when multiple sources are consulted']", "label": 0 }, { "id": "train_1025", "context": "Very little is known about prehistoric hominid cave dwellers. However, a recent study of skeletons of these hominids has revealed an important clue about their daily activities: skeletal fractures present are most like the type and distribution of fractures sustained by rodeo riders. Ttherefore, it is likely that these cave dwellers engaged in activities similar to rodeo riders -- chasing and tackling animals.", "question": "Which one of the following principles, if valid, most helps to justify the argumentation above?", "answers": "['The most important aspect of prehistoric life to be studied is how food was obtained.', 'The primary source of clues about the lives of prehistoric hominids is their skeletal remains.', 'If direct evidence as to the cause of a phenomenon is available, then indirect evidence should not be sought.', 'If there is a similarity between two effects, then there is probably a similarity between their causes.']", "label": 3 }, { "id": "train_1026", "context": "Reasonable people adapt themselves to the world; unreasonable people persist in trying to adapt the world to themselves. Ttherefore, all progress depends on unreasonable people.", "question": "If all of the statements in the passage above are true, which one of the following statements must also be true?", "answers": "['Reasonable people and unreasonable people are incompatible.', 'If there are only reasonable people, there cannot be progress.', 'Some unreasonable people are unable to bring about progress.', 'If there are unreasonable people, there will be progress.']", "label": 1 }, { "id": "train_1027", "context": "Interpreting the lines in the desert as landing strips for spaceship-traveling aliens, an investigator argues that they could hardly have been Inca roads, asking, \"What use to the Inca would have been closely spaced roads that ran parallel? That intersected in a sunburst pattern? That came abruptly to an end in the middle of an uninhabited plain? \" The argumentative strategy of the investigator quoted is to", "question": "In Peru, ancient disturbances in the dark surface material of a desert show up as light-colored lines that are the width of a footpath and stretch for long distances. One group of lines branching out like rays from a single point crosses over curved lines that form a very large bird figure.", "answers": "['challenge the investigative methods used by those who developed the alternative interpretation', 'introduce evidence newly discovered by the investigator which discredits the alternative interpretation', 'support one interpretation by calling into question the plausibility of the alternative interpretation', \"reject out of hand direct counterevidence to the investigator's own interpretation\"]", "label": 2 }, { "id": "train_1028", "context": "Congressman Jones experienced a drop in his approval numbers after announcing that taxes in his district would increase the following year. His office also received a significant number of phone calls decrying this tax raise. Nevertheless, he claims that reactions to the tax increase were unrelated to his drop in voters' approval of him.", "question": "Which of the following, if true, most strongly supports Congressman Jones's position?", "answers": "['Congressman Jones previously raised taxes in his district and received a similar number of phone calls.', 'Elected officials are only likely to attribute drops in their approval numbers to policies only when citizens call their offices about these policies.', \"The citizens who called in to Congressman Jones's office had all voted for him in the previous election.\", 'All elected officials in his area experienced similar drops in approval numbers at the same time.']", "label": 3 }, { "id": "train_1029", "context": "Fred argued that, since Kathleen is a successful film director, she has probably worked with famous actors. But, while Fred is right in supposing that most successful film directors work with famous actors, his conclusion is not warranted. For, as he knows, Kathleen works only on documentary films, and directors of documentaries rarely work with famous actors.", "question": "Which one of the following strategies is used above to criticize Fred's reasoning?", "answers": "['claiming that Fred has failed to take relevant information into account', 'showing that Kathleen must not have worked with famous actors', 'demonstrating that Fred has failed to show that most successful film directors work with famous actors', 'maintaining that too little is known about Kathleen to justify any conclusion']", "label": 0 }, { "id": "train_1030", "context": "Politician: Most of those at the meeting were not persuaded by Kuyler' s argument, nor should they have been, for Kuyler' s argument implied that it would be improper to enter into a contract with the government; and yet -- as many people know -- Kuyler' s company has had numerous lucrative contracts with the government.", "question": "Which one of the following describes a flaw in the politician's argument?", "answers": "['It rejects an argument merely on the grounds that the arguer has not behaved in a way that is consistent with the argument.', 'It concludes that an argument is defective merely on the grounds that the argument has failed to persuade anyone of the truth of its conclusion.', 'It rejects an argument on the basis of an appeal to popular opinion.', 'It rejects a position merely on the grounds that an inadequate argument has been given for it.']", "label": 0 }, { "id": "train_1031", "context": "A deadly virus that has claimed the lives of local villagers has been traced to the spotted fruit bat, in which the virus resides between periodic outbreaks. Biologists speculate that the bat might also be one of the reservoirs for a number of other viruses that have led to village fatalities. The local government has proposed to eliminate the spotted fruit bat by cutting off passageways that allow the bat to emerge from its caves. Once the bat is unable to emerge from the caves, the government will have achieved its objective of reducing the number of village deaths attributable to viruses.", "question": "Which of the following, if true, would best indicate that the government's plan will not lead to its objective?", "answers": "[\"Though there are several other species of fruit bats in the area, the spotted fruit bat is the only one that serves as a reservoir for deadly viruses, which result in a majority of the village's fatalities.\", 'Another species of bat has been observed feeding on small animals, and has also been reported to have bitten human beings, who report feeling severely weakened after awakening from the attack.', 'The spotted fruit bat dwells deep inside the caves where government workers, despite using sophisticated technology, have been unable to penetrate completely.', 'Many villagers who would have otherwise succumbed to a viral disease have been cured after ingesting a serum derived from a local flower whose only means of germinating is via the fur of the spotted fruit bat']", "label": 3 }, { "id": "train_1032", "context": "In early 1990, Queenston instituted a tax increase that gave its school system a larger operating budget. The school system used the larger budget to increase the total number of teachers in the system by 30 percent between 1990 and 1993. Nevertheless, there was no change in the average number of students per teacher between 1990 and 1993.", "question": "If the statements above are true, then on the basis of them which one of the following must also be true?", "answers": [ "The total number of students enrolled in Queenston's school system increased between 1990 and 1993.", "No classes in Queenston's school system experienced an increase in enrollment between 1990 and 1993.", "The quality of education in Queenston's school system improved between 1990 and 1993.", "Most teachers who worked for Queenston's school system in 1990 were still working for the system in 1993." ], "label": 0 }, { "id": "train_1033", "context": "A major art theft from a museum was remarkable in that the pieces stolen clearly had been carefully selected. The criterion for selection, however, clearly had not been greatest estimated market value. It follows that the theft was specifically carried out to suit the taste of some individual collector for whose private collection the pieces were destined.", "question": "The argument tacitly appeals to which one of the following principles?", "answers": "['Any art theft committed at the direction of a single individual results in a pattern of works taken and works left alone that defies rational analysis.', 'The pattern of works taken and works left alone in an art theft can be particularly damaging to the integrity of the remaining collection.', 'The pattern of works taken and works left alone can sometimes distinguish one type of art theft from another.', 'Art thefts committed with no preexisting plan for the disposition of the stolen works do not always involve theft of the most valuable pieces only.']", "label": 2 }, { "id": "train_1034", "context": "Although most people know what their bad habits are and want to rid themselves of them, a majority of these people find it very difficult to do so. This is because cessation of habitual behavior is immediately and vividly painful, while whatever benefit is to be gained by the absence of the habit is perceived only dimly because it is remote.", "question": "The information above most strongly supports the statement that the people who are most successful at ending their bad habits are those who", "answers": "['have succeeded in the past at modifying their behavior', 'can vividly imagine remote but attainable benefit', 'can vividly imagine their present pain being felt in the future', 'are relatively unaware of their own behavioral characteristics']", "label": 1 }, { "id": "train_1035", "context": "Despite the best efforts of astronomers, no one has yet succeeded in exchanging messages with intelligent life on other planets or in other solar systems. In fact, no one has even managed to prove that any kind of extraterrestrial life exists. Thus, there is clearly no intelligent life anywhere but on Earth.", "question": "The argument's reasoning is flawed because the argument", "answers": "['relies on a weak analogy rather than on evidence to draw a conclusion', 'confuses an absence of evidence for a hypothesis with the existence of evidence against the hypothesis', 'makes an inference that relies on the vagueness of the term \"life\"', 'fails to consider that there might be extraterrestrial forms of intelligence that are not living beings']", "label": 1 }, { "id": "train_1036", "context": "Movie Director: Movies come out the best, in terms of both critical and box office success, when a single person has full creative control over the final product. Studios providing \"notes\" to the director, writer, producer, or whomever controls that vision, stifle the creative process. Studio Head: At the end of the day, the film industry is a business. Movies will only be made if they are profitable. The most successful movies focus on subjects that the audience cares about, and the studios spend millions of dollars on test screenings to determine what audiences prefer. Whether the studio transmits its research by \"notes\" or informal discussion, the studio' s input must be considered.", "question": "Which one of the following best describes the main point in dispute?", "answers": "['The best way for the studio to communicate its feedback is always through \"notes. \"', 'Critical acclaim is more important than box office success.', 'More than one person should be involved in the creative process.', 'Test screenings are a waste of money.']", "label": 2 }, { "id": "train_1037", "context": "Trisha: Today' s family is declining in its ability to carry out its functions of child-rearing and providing stability for adult life. There must be a return to the traditional values of commitment and responsibility. Jerod: We ought to leave what is good enough alone. Contemporary families may be less stable than traditionally, but most people do not find that to be bad. Contemporary criticisms of the family are overblown and destructive.", "question": "Trisha and Jerod disagree over whether the institution of the family is", "answers": "['valued by most people', 'changing over time', 'adequate as it is', 'no longer traditional']", "label": 2 }, { "id": "train_1038", "context": "One adaptation that enables an animal species to survive despite predation by other species is effective camouflage. Yet some prey species with few or no other adaptations to counteract predation have endured for a long time with black-and-white coloration that seems unlikely to provide effective camouflage.", "question": "Which one of the following, if true, most contributes to a resolution of the apparent discrepancy mentioned above?", "answers": "['Animals of many predatory species do not perceive color or pattern in the same manner as humans do.', 'Conspicuous black-and-white areas help animals of the same species avoid encounters with one another.', 'Most species with black-and-white coloration are more populous than the species that prey upon them.', 'Black-and-white coloration is not as great a liability against predators at night as it is during the day.']", "label": 0 }, { "id": "train_1039", "context": "Heavy rains during Centralia' s corn planting season prevented some farmers there from planting corn. It is now the planting season for soybeans, another of Centralia' s principal crops, and those fields originally intended for corn are dry enough for planting. Nonetheless, even though soybean prices are unusually high at present, the farmers will leave most of these fields empty rather than plant them with soybeans, since__.", "question": "Which of the following most logically completes the passage below?", "answers": "['some Centralian farmers anticipate serious financial losses due to the extremely wet spring planting season', 'many Centralian farmers grow both corn and soybeans', 'the extensive rains have led to an increase in the price of corn', 'chemicals that were used to prepare the fields for corn planting would stunt the growth of soybeans']", "label": 3 }, { "id": "train_1040", "context": "In a vast ocean region, phosphorus levels have doubled in the past few decades due to agricultural runoff pouring out of a large river nearby. The phosphorus stimulates the growth of plankton near the ocean surface. Decaying plankton fall to the ocean floor, where bacteria devour them, consuming oxygen in the process. Due to the resulting oxygen depletion, few fish can survive in this region.", "question": "Which one of the following can be properly inferred from the information above?", "answers": "['The quantity of agricultural runoff pouring out of the river has doubled in the past few decades.', 'Before phosphorus levels doubled in the ocean region, most fish were able to survive in that region.', 'If agricultural runoff ceased pouring out of the river, there would be no bacteria on the ocean floor devouring decaying plankton.', 'The agricultural runoff pouring out of the river contributes to the growth of plankton near the ocean surface.']", "label": 3 }, { "id": "train_1041", "context": "Pauline and Lorraine are paid the same amount of wages per hour for working at the same company, but Pauline has a more difficult job than Lorraine. Pauline argues that because her job is more difficult, the company should pay her more than it pays Lorraine.", "question": "Which of the following statements best supports Pauline's argument?", "answers": "['Pauline has fewer years of experience in her current job than Lorraine has in hers.', 'Pauline expects to be promoted to a higher-paying position.', \"The company's pay scale is intended to compensate more difficult work with higher wages.\", 'Lorraine is not opposed to being paid less than Pauline.']", "label": 2 }, { "id": "train_1042", "context": "Sonya: Anyone who lives without constant awareness of the fragility and precariousness of human life has a mind clouded by illusion. Yet those people who are perpetually cognizant of the fragility and precariousness of human life surely taint their emotional outlook on existence.", "question": "Sonya's statements, if true, most strongly support which one of the following?", "answers": "['It is better to be aware of the fragility and precariousness of human life than to have an untainted emotional outlook on existence.', 'It is impossible for anyone to live without some degree of self-deception.', 'Everyone whose emotional outlook on existence is untainted has a mind clouded by illusion.', 'Either no one has a tainted emotional outlook on existence, or no one has a mind clouded by illusion.']", "label": 2 }, { "id": "train_1043", "context": "Anger in response to insults is unreasonable, for insults are merely assertions that someone has undesirable characteristics. If such an assertion is false, the insulted party ought to pity the ignorance prompting the insult. If it is true, the insulted party should be grateful for such useful information.", "question": "Which one of the following, if assumed, enables the argument's conclusion to be properly drawn?", "answers": "['Anger is an unreasonable response to any action that should prompt pity or gratitude.', 'Gratitude and pity are reasonable responses to some forms of hostile or insensitive behavior.', 'Actions prompted by ignorance do not warrant hostile reactions.', 'Pity is the only reasonable reaction to people with undesirable characteristics.']", "label": 0 }, { "id": "train_1044", "context": "Some educators claim that it is best that school courses cover only basic subject matter, but cover it in depth. These educators argue that if students achieve a solid grasp of the basic concepts and investigatory techniques in a subject, they will be able to explore the breadth of that subject on their own after the course is over. But if they simply learn a lot of factual information, without truly understanding its significance, they will not be well equipped for further study on their own.", "question": "The educators' reasoning provides grounds for accepting which one of the following statements?", "answers": "['It is easier to understand any Greek tragedy after one has analyzed a few of them in detail.', 'It is easier to understand how plants and animals are classified after learning how plants and animals can be useful.', 'It is easier to learn many simple ideas well than to learn a few complicated ideas well.', 'It is easier to remember new ideas explained personally by a teacher than ideas that one explores independently.']", "label": 0 }, { "id": "train_1045", "context": "The city of Workney, in raising bus fares from $1. 00 to $1. 25, proposed that 18 fare tokens be sold for $20. 00 to alleviate the extra burden of the fare increase on the city' s poor people. Critics suggested alternatively that 9 fare tokens be sold for $10. 00, because a $20. 00 outlay would be prohibitive for poor riders", "question": "The alternative proposal depends on which of the following assumptions?", "answers": "['Fewer riders would regularly purchase 18 fare tokens at once than would purchase only 9 fare tokens at once', 'The outlay of $10. 00 for the purchase of 9 fare tokens would not be prohibitive for bus riders who are poor.', 'Poor residents of Orkney will continue to ride the buses in the same numbers despite the fare increase', 'Riders who are poor would be more likely to take advantage of the savings afforded by the 9-token offer than would riders who are not poor.']", "label": 1 }, { "id": "train_1046", "context": "Mayor: To keep our neighborhoods clean, every street in town will be swept at least once a month. If a neighborhood needs more frequent sweepings, due to excessive dirt from major construction for example, that neighborhood will be qualified for interim sweepings. All requests for interim sweepings from qualified neighborhoods will be satisfied immediately.", "question": "If all of the mayor's statements are true, then which one of the following must also be true?", "answers": "['A qualified neighborhood that requests an interim sweeping will have its streets swept more than once a month.', 'All qualified neighborhoods will get their streets swept more than once a month.', 'No street will be swept more than once a month unless it is located in a qualified neighborhood.', 'All neighborhoods in which construction is under way are qualified neighborhoods.']", "label": 0 }, { "id": "train_1047", "context": "Taste buds were the primary tool early humans used for testing foods. Sour taste warns of possible spoilage of food while bitterness is a warning of many poisons. Early humans also recognized sweet foods and salty foods as meeting nutritional needs. So the fact that people can now clearly distinguish these four tastes-sour, bitter, sweet, and salty-is completely explained by people' s use of taste to test for the healthfulness of foods.", "question": "The reasoning in the argument is most vulnerable to criticism on the grounds that the argument", "answers": "['takes a necessary condition for the truth of its conclusion to be sufficient to justify that conclusion', 'takes what might be only a partial explanation of a phenomenon to be the complete explanation', 'fails to consider that many people associate foods more with their smells than with their tastes', 'fails to consider that some nutritious foods are bitter when raw but not after being cooked']", "label": 1 }, { "id": "train_1048", "context": "Advertisement: Bonner cabinets, made in the traditional German way, are the finest quality cabinets in the world. The surveys prove this claim. In a recent survey, 9 out of 10 shoppers surveyed expressed a preference for Bonner cabinets, stating that they would buy no other cabinets.", "question": "Of the following, which one provides the strongest reason that the advertisement is potentially misleading?", "answers": "['The people who stated a preference for Bonner cabinets might not have Bonner cabinets in their homes.', 'People who own cabinets might prefer wood cabinets.', 'The preference for Bonner cabinets might be based on a factor other than quality.', 'The American way of making cabinets usually requires the use of oak or mahogany.']", "label": 2 }, { "id": "train_1049", "context": "Interviewer: A certain company released a model of computer whose microprocessor design was flawed, making that computer liable to process information incorrectly. How did this happen? Industry spokesperson: Given the huge number of circuits in the microprocessor of any modern computer, not every circuit can be manually checked before a computer model that contains the microprocessor is released. Interviewer: Then what guarantee do we have that new microprocessors will not be similarly flawed? Industry spokesperson: There is no chance of further microprocessor design flaws, since all microprocessors are now entirely computer-designed.", "question": "The industry spokesperson's argument is most vulnerable to criticism on the grounds that it", "answers": "['ignores the possibility that a microprocessor can have a flaw other than a design flaw', 'overlooks the possibility that a new computer model is liable to malfunction for reasons other than a microprocessor flaw', 'takes for granted, despite evidence to the contrary, that some computers are not liable to error', 'presumes, without providing justification, that the microprocessor quality-control procedures of the company mentioned are not representative of those followed throughout the industry']", "label": 2 }, { "id": "train_1050", "context": "Rhett: Otto gives me a ride to work every morning and expects me to help pay for fuel. But I shouldn' t have to pay. Giving me a ride doesn' t increase the amount he spends on fuel, since he has to go right by my house anyway. Barbara: By that flawed logic, you would be entitled to the warm air that heats the unused spare room of my house, if you could divert it to your house without increasing my expenses.", "question": "The dialogue provides the most support for the claim that Rhett and Barbara disagree over whether", "answers": "['Otto requires Rhett to help pay for fuel for the ride to work', \"giving Rhett a ride to work increases Otto's fuel expenses\", \"Rhett is entitled to the warm air that heats Barbara's unused spare room\", \"Rhett should have to help pay Otto's fuel expenses\"]", "label": 3 }, { "id": "train_1051", "context": "We ought to pay attention only to the intrinsic properties of a work of art. Its other, extrinsic properties are irrelevant to our aesthetic interactions with it. For example, when we look at a painting we should consider only what is directly presented in our experience of it. What is really aesthetically relevant, ttherefore, is not what a painting symbolizes, but what it directly presents to experience.", "question": "The conclusion follows logically if which one of the following is added to the premises?", "answers": "['There are certain properties of our experiences of artworks that can be distinguished as symbolic properties.', \"Only an artwork's intrinsic properties are relevant to our aesthetic interactions with it.\", 'It is possible in theory for an artwork to symbolize nothing.', 'What an artwork symbolizes involves only extrinsic properties of that work.']", "label": 3 }, { "id": "train_1052", "context": "The Dvorak keyboard requires less finger movement than the ubiquitous QWERTY keyboard. As a result, Dvorak keyboard users are not only able to type more words per minute, but are also less vulnerable to both repetitive stress disorder and carpal tunnel syndrome. Nonetheless, businesses, as well as consumers, have not adopted the Dvorak keyboard. Clearly, if the Dvorak keyboard is to become more widely used, its benefits must be more widely touted.", "question": "Which of the following, if true, casts the most doubt on the author's conclusion?", "answers": "['Those suffering from repetitive stress injuries often attribute the injuries to multiple factors', 'Businesses have found that many employees who believe the QWERTY keyboard is responsible for stress-induced injuries are unaware of the Dvorak keyboard.', 'Many who have attempted using a Dvorak keyboard claim that learning the configuration of keys takes weeks', 'Businesses that have educated employees on the benefits of the Dvorak keyboard, have found that employees continue to use the QWERTY keyboard']", "label": 3 }, { "id": "train_1053", "context": "Having lived through extraordinary childhood circumstances, Robin has no conception of the moral difference between right and wrong, only between what is legally permitted and what is not. When Robin committed an offense, Robin did not recognize the fact that it was a morally wrong act, despite knowing that it was illegal.", "question": "From the statements above, which one of the following can be properly inferred?", "answers": "['Robin did something that was morally wrong.', 'Robin could now be brought to see the moral difference between right and wrong.', \"Robin's childhood could have provided more adequate moral training even in the circumstances.\", 'Moral ignorance is never excusable in the eyes of the law.']", "label": 0 }, { "id": "train_1054", "context": "The amount of time it takes for most of a worker's occupational knowledge and skills to become obsolete has been declining because of the introduction of advanced manufacturing technology (AMT). Given the rate at which AMT is currently being introduced in manufacturing, the average worker's old skills become obsolete and new skills are required within as little as five years.", "question": "Which of the following plans, if feasible, would allow a company to prepare most effectively for the rapid obsolescence of skills described above?", "answers": "['The company will ensure that it can offer its employees any training necessary for meeting their job requirements.', 'The company will develop a program to offer selected employees the opportunity to receive training six years after they were originally hired.', 'The company will increase its investment in AMT every year for a period of at least five years.', 'The company will periodically survey its employees to determine how the introduction of AMT has affected them.']", "label": 0 }, { "id": "train_1055", "context": "If there are sentient beings on planets outside our solar system, we will not be able to determine this anytime in the near future unless some of these beings are at least as intelligent as humans. We will not be able to send spacecraft to planets outside our solar system anytime in the near future, and any sentient being on another planet capable of communicating with us anytime in the near future would have to be at least as intelligent as we are.", "question": "The argument's conclusion can be properly inferred if which one of the following is assumed?", "answers": "['Any sentient beings on planets outside our solar system that are at least as intelligent as humans would be capable of communicating with us.', 'If a sentient being on another planet cannot communicate with us, then the only way to detect its existence is by sending a spacecraft to its planet.', 'There are no sentient beings on planets in our solar system other than those on Earth.', \"If there is a sentient being on another planet that is as intelligent as humans are, we will not be able to send spacecraft to the being's planet anytime in the near future.\"]", "label": 1 }, { "id": "train_1056", "context": "Raphaela: Forcing people to help others is morally wrong. Ttherefore, no government has the right to redistribute resources via taxation. Anyone who wants can help others voluntarily. Edward: Governments do have that right, insofar as they give people the freedom to leave and hence not to live under their authority.", "question": "Raphaela and Edward disagree about the truth of which one of the following?", "answers": "['Any government that forces people to help others should permit emigration.', 'Any government that permits emigration has the right to redistribute resources via taxation.', 'Any government that redistributes resources via taxation forces people to help others.', 'Every government should allow people to help others voluntarily.']", "label": 1 }, { "id": "train_1057", "context": "Some doctors believe that a certain drug reduces the duration of episodes of vertigo, claiming that the average duration of vertigo for people who suffer from it has decreased since the drug was introduced. However, during a recent three-month shortage of the drug, there was no significant change in the average duration of vertigo. Thus, we can conclude that the drug has no effect on the duration of vertigo.", "question": "Which one of the following is an assumption required by the argument?", "answers": "['Changes in diet and smoking habits are not responsible for any change in the average duration of vertigo since the introduction of the drug.', 'If there were any change in the average duration of vertigo since the introduction of the drug, it would have demonstrated that the drug has an effect on the duration of vertigo.', 'A period of time greater than three months would not have been better to use in judging whether the drug has an effect on the duration of vertigo.', 'If a drug made a difference in the duration of vertigo, a three-month shortage of that drug would have caused a significant change in the average duration of vertigo.']", "label": 3 }, { "id": "train_1058", "context": "Only computer scientists understand the architecture of personal computers, and only those who understand the architecture of personal computers appreciate the advances in technology made in the last decade. It follows that only those who appreciate these advances are computer scientists.", "question": "Which one of the following most accurately describes a flaw in the reasoning in the argument?", "answers": "['The premises of the argument are stated in such a way that they exclude the possibility of drawing any logical conclusion.', 'The argument ignores the fact that some computer scientists may not appreciate the advances in technology made in the last decade.', 'The argument contains no stated or implied relationship between computer scientists and those who appreciate the advances in technology in the last decade.', 'The premises of the argument presuppose that everyone understands the architecture of personal computers.']", "label": 1 }, { "id": "train_1059", "context": "In the country of Marut, the Foreign Trade Agency' s records were reviewed in 1994 in light of information then newly available about neighboring Goro. The review revealed that in every year since 1963, the agency' s projection of what Goro' s gross national product(GNP) would be five years later was a serious underestimate. The review also revealed that in every year since 1963, the agency estimated Goro' s GNP for the previous year--a Goro state secret--very accurately.", "question": "Of the following claims, which is most strongly supported by the statement given?", "answers": "['Even before the new information came to light, the agency had reason to think that at least some of the five-year projections it had made were inaccurate.', 'The amount by which the agency underestimated the GNP it projected for Goro tended to increase over time.', 'Prior to 1995, Goro had not released data intended to mislead the agency in making its five-year projections.', \"The agency's five-year projections of Goro's GNP had no impact on economic planning in Marut.\"]", "label": 0 }, { "id": "train_1060", "context": "Legal commentator: The goal of a recently enacted law that bans smoking in workplaces is to protect employees from secondhand smoke. But the law is written in such a way that it cannot be interpreted as ever prohibiting people from smoking in their own homes.", "question": "The statements above, if true, provide a basis for rejecting which one of the following claims?", "answers": "['Most people believe that smokers have a fundamental right to smoke in their own homes.', 'The law offers no protection from secondhand smoke for people outside of their workplaces.', 'The law will protect domestic workers such as housecleaners from secondhand smoke in their workplaces.', 'Supporters of the law believe that it will have a significant impact on the health of many workers.']", "label": 2 }, { "id": "train_1061", "context": "Marketing agent: A survey of my business clients reveals that, of those who made a profit last year, 90 percent made at least $100, 000 in profit for the year. In prior years, not one of these businesses made an annual profit of more than $10, 000. So, 90 percent of my business clients increased their profits at least tenfold last year.", "question": "The reasoning in the marketing agent's argument is most vulnerable to criticism on the grounds that the argument", "answers": "['overlooks the possibility that the business clients who made more than $100, 000 last year made only slightly more than $100, 000', 'fails to explain why some of the business clients who made a profit did not increase their profits at least tenfold last year', 'overlooks the possibility that not all of the business clients made an annual profit of more than $10, 000 last year', 'draws a conclusion about all of the business clients from premises about the business clients who made a profit last year']", "label": 3 }, { "id": "train_1062", "context": "A major network news organization experienced a drop in viewership in the week following the airing of a controversial report on the economy. The network also received a very large number of complaints regarding the report. The network, however, maintains that negative reactions to the report had nothing to do with its loss of viewers.", "question": "Which of the following, if true, most strongly supports the network's position?", "answers": "['Major network news organizations publicly attribute drops in viewership to their own reports only when they receive complaints about those reports.', 'Most network news viewers rely on network news broadcasts as their primary source of information regarding the economy.', 'The other major network news organizations reported similar reductions in viewership during the same week.', \"The viewers who registered complaints with the network were regular viewers of the news organization's programs.\"]", "label": 2 }, { "id": "train_1063", "context": "Professor: A guest speaker recently delivered a talk entitled \"The Functions of Democratic Governments\" to a Political Ideologies class at this university. The talk was carefully researched and theoretical in nature. But two students who disagreed with the theory hurled vicious taunts at the speaker. Several others applauded their attempt to humiliate the speaker. This incident shows that universities these days do not foster fair-minded and tolerant intellectual debate.", "question": "The professor's reasoning is flawed in that it", "answers": "['draws a general conclusion based on too small a sample', \"is inconsistent in advocating tolerance while showing intolerance of the dissenting students' views\", \"draws a conclusion based on the professor's own opinion rather than on that of the majority of the students present at the talk\", 'relies primarily on an emotional appeal']", "label": 0 }, { "id": "train_1064", "context": "Whenever a major political scandal erupts before an election and voters blame the scandal on all parties about equally, virtually all incumbents, from whatever party, seeking reelection are returned to office. However, when voters blame such a scandal on only one party, incumbents from that party are likely to be defeated by challengers from other parties. The proportion of incumbents who seek reelection is high and remarkably constant from election to election.", "question": "If the voters' reactions are guided by a principle, which one of the following principles would best account for the contrast in reactions described above?", "answers": "['Major political scandals can practically always be blamed on incumbents, but whether those incumbents should be voted out of office depends on who their challengers are.', 'When a major political scandal is blamed on incumbents from all parties, that judgment is more accurate than any judgment that incumbents from only one party are to blame.', 'When major political scandals are less the responsibility of individual incumbents than of the parties to which they belong, whatever party was responsible must be penalized when possible.', 'Incumbents who are rightly blamed for a major political scandal should not seek reelection, but if they do, they should not be returned to office.']", "label": 2 }, { "id": "train_1065", "context": "During its caterpillar stage, the leopard magpie moth feeds on a plant called the Natal grass cycad and by so doing laces its body with macrozamin, a toxin that makes the moth highly unpalatable to would-be predators. Since the Natal grass cycad is now endangered and facing extinction, the leopard magpie moth is also in danger of extinction.", "question": "Which one of the following is an assumption required by the argument?", "answers": "[\"Potential predators of the leopard magpie moth cannot determine from appearance alone whether a moth's body is laced with macrozamin.\", 'The leopard magpie moth does not have the speed or the agility to escape from any of its potential predators.', 'None of the potential predators of the leopard magpie moth have developed a tolerance to macrozamin.', 'Feeding on the Natal grass cycad is the only means by which the leopard magpie moth can make itself highly unpalatable to predators.']", "label": 3 }, { "id": "train_1066", "context": "Evolution does not always optimize survival of an organism. Male moose evolved giant antlers as a way of fighting other males for mates, giving those with the largest antlers an evolutionary advantage. But those antlers also make it harder to escape predators, since they can easily get tangled in trees. All male moose would be better off with antlers half the current size: they would all be less vulnerable to predators, and those with the largest antlers would maintain their relative advantage.", "question": "Which one of the following is a technique of reasoning used in the argument?", "answers": "['disputing the relevance of an example thought to support an opposing view', 'undermining a claim by showing that it is self-contradictory', 'employing an analogy in order to dispute a generalization', 'challenging a general claim by presenting a counterexample']", "label": 3 }, { "id": "train_1067", "context": "According to experts on shopping behavior, more shoppers would shop at Jerrod' s department store if they were offered the convenience of shopping carts. In fact, even if the amount spent by these additional customers only just covered the cost of providing the carts, providing carts would still probably increase Jerrod' s profits, since __.", "question": "Which of the following most logically completes the argument?", "answers": "[\"the layout of Jerrod's is open enough to accommodate shopping carts comfortably\", 'a customer with a shopping cart buys more, on average, than a customer without a cart', 'stores that make shopping carts available to customers usually have to hire people to retrieve them from parking areas', \"there are some potential customers who would not be enticed to shop at Jerrod's by the availability of shopping carts\"]", "label": 1 }, { "id": "train_1068", "context": "Taylor: From observing close friends and relatives, it is clear to me that telepathy is indeed possible between people with close psychic ties. The amazing frequency with which a good friend or family member knows what one is thinking or feeling cannot be dismissed as mere coincidence.", "question": "Taylor's reasoning is most vulnerable to criticism on the grounds that it", "answers": "['appeals to a premise one would accept only if one already accepted the truth of the conclusion', 'is based on too small a sample to yield a reliable conclusion', 'presumes, without providing justification, that one can never know what a stranger is thinking or feeling', 'fails to address a highly plausible alternative explanation for all instances of the observed phenomenon']", "label": 3 }, { "id": "train_1069", "context": "During the recent economic downturn, banks contributed to the decline by loaning less money. Prior to the downturn, regulatory standards for loanmaking by banks were tightened. Clearly, ttherefore, banks will lend more money if those standards are relaxed.", "question": "The argument assumes that", "answers": "['the reason for tightening the regulatory standards was not arbitrary', 'no relaxation of standards for loanmaking by banks would compensate for the effects of the downturn', 'no economic downturn is accompanied by a significant decrease in the amount of money loaned out by banks to individual borrowers and to businesses', 'the downturn did not cause a significant decrease in the total amount of money on deposit with banks which is the source of funds for banks to lend']", "label": 3 }, { "id": "train_1070", "context": "Nutritionist: The food pyramid was amended to provide the public with a healthier nutrition plan to combat the obesity epidemic. The base of the previous food pyramid -- the grains group -- advised people to eat six-to-eleven servings of pasta and bread per day. In contrast, the updated pictorial nutrition guide shows a plate, and it reduces the percentage of dietary intake of grains and replaces the difference with lean proteins and vegetables. The updated guide has been wildly successful, as citizens have lost a significant amount of weight.", "question": "Which one of the following, if true, most strengthens the nutritionist's conclusion?", "answers": "['Since the publication of the updated food guide, citizens have started to exercise more.', 'Eating more lean proteins and vegetables contributes to weight loss.', \"Eating six-to-eleven servings of pasta and bread increased citizens' risk of heart disease.\", 'The old food pyramid used horizontal layers, while the updated guide uses vertical layers.']", "label": 1 }, { "id": "train_1071", "context": "Because mining and refining nickel is costly, researchers have developed an alternative method for extracting nickel using Streptanthus polygaloides, a plant that absorbs and stores nickel from the soil as it grows. The researchers incinerated a crop of Streptanthus they grew in nickel-rich soil. By chemically extracting nickel from the ash, they produced 100 pounds of nickel per acre of land at a total cost per pound slightly above that of current mining.", "question": "Which of the following, if true, most strongly supports the conclusion that the use of Streptanthus to extract nickel will be commercially adopted?", "answers": "['The season in which the researchers grew Streptanthus was an unusually favorable one, with the right amount of precipitation to maximize the growth rate of Streptanthus.', 'The land on which the researchers planted Streptanthus was unusually free of the various weeds that can compete with Streptanthus for water, nutrients, and sunlight.', \"It is extremely rare for soil to contain higher concentrations of nickel than the concentrations present in the researchers' experimental plot.\", 'Because lowering the concentration of nickel in the soil can make land much better for agriculture in general, a plot on which Streptanthus has been grown and harvested can be sold for substantially more than it cost.']", "label": 3 }, { "id": "train_1072", "context": "The tidal range at a particular location is the difference in height between high tide and low tide. Tidal studies have shown that one of the greatest tidal ranges in the world is found in the Bay of Fundy and reaches more than seventeen meters. Since the only forces involved in inducing the tides are the sun' s and moon' s gravity, the magnitudes of tidal ranges also must be explained entirely by gravitational forces.", "question": "Which one of the following most accurately describes a flaw in the reasoning above?", "answers": "['It does not differentiate between the tidal effect of the sun and the tidal effect of the moon.', 'It fails to consider that the size of a tidal range could be affected by the conditions in which gravitational forces act.', \"It presumes, without providing warrant, that most activity within the world's oceans is a result of an interplay of gravitational forces.\", 'It gives only one example of a tidal range.']", "label": 1 }, { "id": "train_1073", "context": "Economist: Technology now changes so rapidly that workers need periodic retraining. Such retraining can be efficient only if it allows individual companies to meet their own shortterm needs. Hence, large governmental job retraining programs are no longer a viable option in the effort to retrain workers efficiently.", "question": "Which one of the following is an assumption required by the economist's argument?", "answers": "['No single type of retraining program is most efficient at retraining technological workers.', 'Large governmental job retraining programs do not meet the short-term needs of individual companies.', 'Technological workers are more likely now than in the past to move in order to find work for which they are already trained.', 'Large job retraining programs will be less efficient than smaller programs if the pace of technological change slows.']", "label": 1 }, { "id": "train_1074", "context": "Ten thousand years ago many communities in western Asia stopped procuring food by hunting and gathering and began instead to cultivate food. Archaeological evidence reveals that compared to their hunter-gatherer forebears, the early agricultural peoples ate a poorly balanced diet and had diet-related health problems, yet these peoples never returned to hunting and gathering.", "question": "Which one of the following, if true, most helps to explain why the agricultural peoples of western Asia never returned to hunting and gathering?", "answers": "['Both hunter-gatherers and agriculturalists sometimes depended on stored and preserved foods instead of fresh foods.', 'An increase in population density at the time required a higher food production rate than hunting and gathering could provide.', 'Thousands of years ago similar shifts from hunting and gathering to agriculture occurred in many other parts of the world.', 'The plants and animals that the agricultural peoples began to cultivate continued to exist in the wild.']", "label": 1 }, { "id": "train_1075", "context": "To get the free dessert, one must order an entree and a salad. But anyone who orders either an entree or a salad can receive a free soft drink. Thus, anyone who is not eligible for a free soft drink is not eligible for a free dessert.", "question": "The reasoning in the argument above is most similar to the reasoning in which one of the following arguments?", "answers": "['To be elected class president, one must be well liked and well known. Anyone who is well liked or well known has something better to do than run for class president. Ttherefore, no one who has something better to do will be elected class president.', 'To get an executive position at Teltech, one needs a university diploma and sales experience. But anyone who has worked at Teltech for more than six months who does not have sales experience has a university diploma. Thus, one cannot get an executive position at Teltech unless one has worked there for six months.', 'To get a discount on ice cream, one must buy frozen raspberries and ice cream together. Anyone who buys ice cream or raspberries will get a coupon for a later purchase. So, anyone who does not get the discount on ice cream will not get a coupon for a later purchase.', 'To grow good azaleas, one needs soil that is both rich in humus and low in acidity. Anyone who has soil that is rich in humus or low in acidity can grow blueberries. So, anyone who cannot grow blueberries cannot grow good azaleas.']", "label": 3 }, { "id": "train_1076", "context": "A successful chess-playing computer would prove either that a machine can think or that chess does not involve thinking. In either case the conception of human intelligence would certainly change.", "question": "The reasoning above is most vulnerable to criticism on the grounds that it does not consider the possibility that", "answers": "['the conception of intelligence is inextricably linked to that of thought', 'the inability to play chess has more to do with lack of opportunity than with lack of intelligence', 'computer programs have been successfully applied to games other than chess', 'a successful chess-playing computer would not model a human approach to chess playing']", "label": 3 }, { "id": "train_1077", "context": "Of 2, 500 people who survived a first heart attack, those who did not smoke had their first heart attack at a median age of 62. However, of those 2, 500, people who smoked two packs of cigarettes a day had their first heart attack at a median age of 51. On the basis of this information, it can be concluded that nonsmokers tend to have a first heart attack eleven years later than do people who smoke two packs of cigarettes a day.", "question": "The conclusion is incorrectly drawn from the information given because this information does not include", "answers": "['the earliest age at which a person who smoked two packs a day had his or her first heart attack', 'the relative severity of heart attacks suffered by smokers and nonsmokers', 'data on people who did not survive a first heart attack', 'how many of the 2, 500 people studied suffered a second heart attack']", "label": 2 }, { "id": "train_1078", "context": "The basic ingredients from which cement is made are both cheap and plentiful. Materials as common as limestone and clay will do. Nevertheless, the price of cement is influenced by the price of oil, because turning the basic ingredients into cement in high-temperature kilns uses large amounts of energy.", "question": "Which one of the following can be logically inferred from the passage?", "answers": "['Whenever oil prices rise, cement prices drop.', 'Oil is a source of energy for some of the kilns used in the making of cement.', 'The higher the price of cement rises, the higher the price of clay rises.', 'Oil is one of the basic ingredients that make up cement.']", "label": 1 }, { "id": "train_1079", "context": "People always seem to associate high prices of products with high quality. But price is not necessarily an indicator of quality. The best teas are often no more expensive than the lower-quality teas.", "question": "Which one of the following, if true, does most to explain the apparent counterexample described above?", "answers": "['Packing and advertising triple the price of all teas.', 'Low-quality teas are generally easier to obtain than high quality teas.', 'Most people buy low-quality tea, thus keeping its price up.', 'The price of tea generally does not vary from region to region.']", "label": 2 }, { "id": "train_1080", "context": "For the condor to survive in the wild, its breeding population must be greatly increased. But because only a few eggs can be produced by a breeding pair over their lifetime, any significant increase in the number of birds depends upon most of these eggs hatching, which is extremely unlikely in the wild due to environmental dangers. One possible way to eliminate the effects of these factors is to breed the birds in captivity and subsequently return them to the wild.", "question": "Which one of the following is most strongly supported by the information above?", "answers": "['The condor as a species will eventually become extinct in the wild.', 'It is almost impossible to eliminate all the environmental threats to the eggs of condors.', 'The most feasible way to save the condor from extinction is to increase egg production.', 'If more condor eggs do not hatch, the condor as a species will not survive in the wild.']", "label": 3 }, { "id": "train_1081", "context": "Only if a family manages its finances wisely will it obtain fiscal security. But without realistic projections of financial obligations, a functional budget cannot be devised. So, if either fiscal security is to be obtained or a workable budget devised, finances must be managed wisely or financial obligations must be projected realistically.", "question": "Which one of the following arguments is most similar in its pattern of reasoning to that in the argument above?", "answers": "['Only employees who work diligently will be retained in this company until they are eligible for retirement. Also, we can retain only those employees who fit the new organizational structure of our proposed redesign process. So, if this redesign process is carried out, any employee who seeks continuing employment here must work diligently and fit the new organizational structure.', 'Reasonably healthy people who wish to lose weight must alter their balance of caloric intake and caloric burn off. But without benefit of medical supervision, drastic changes in diet or exercise patterns would be harmful. So, one who wishes to lose weight but does not want to risk health problems should seek medical supervision before beginning a diet or exercise program.', \"Without continued use of pesticides it is not possible to grow enough food for the world's people. But only when researchers develop pesticides harmless to humans will persistent health problems be reduced. Thus, pesticide use must continue or pesticides harmless to humans must be developed if enough food is to be produced or persistent health problems are to be reduced.\", 'A successful charity drive requires detailed planning. Volunteers must be recruited and trained, and equipment and facilities must be prepared months before the drive is to begin. Thus, if a group is organizing a charity drive, the group must formulate a detailed plan well ahead of time or it can expect failure.']", "label": 2 }, { "id": "train_1082", "context": "The crux of creativity resides in the ability to manufacture variation on a theme. If we look at the history of science, for instance, we see that every idea is built upon a thousand related ideas. Careful analysis leads us to understand that what we choose to call a new theme or a new discovery is itself always and without exception some sort of variation, on a deep level, of previous themes.", "question": "If all of the statements in the passage are true, each of the following must also be true EXCEPT:", "answers": "['All great scientific discoverers have been able to manufacture a variation on a theme.', 'Careful analysis of a specific variation can reveal previous themes of which it is a variation.', 'Some new scientific discoveries do not represent, on a deep level, a variation on previous themes.', 'No scientific idea is entirely independent of all other ideas.']", "label": 2 }, { "id": "train_1083", "context": "Ronan rarely attended class during his first semester at law school. Some of his professors didn' t know he was in the class until the final exam. Ronan finished the year in the top 10 percent of his class and earned a spot on the school' s prestigious law review.", "question": "Which of the following, if true, would explain the apparent paradox?", "answers": "['Ronan learns better through reading than listening, and he read the most relevant treatise on every class.', 'Class attendance is optional in law school.', 'Ronan is lazy.', \"Ronan's professors were unqualified to teach the law.\"]", "label": 0 }, { "id": "train_1084", "context": "Pollster: When opinion researchers need a population sample that reflects the demographic characteristics of the national population, they choose their sample on the basis of national census data. Not everyone participates in the national census, despite its being mandatory. If, however, census participation became voluntary, as some have proposed, the participation rate would be much lower. So if census participation became voluntary, polls designed to discover the opinions of the national population would have less accurate results.", "question": "Which one of the following is an assumption on which the pollster's argument depends?", "answers": "['Among people who do not currently participate in the national census, few, if any, would agree to participate if participation were voluntary.', 'Using data from the national census is not the only way for opinion researchers to get a population sample that reflects the demographic characteristics of the national population.', 'The group of people who would participate in a voluntary national census would differ in its demographic characteristics from the group of people who would participate in a mandatory national census.', 'The people who refuse to participate in opinion polls comprise a group with approximately the same demographic characteristics as the group of people who do not currently participate in the national census.']", "label": 2 }, { "id": "train_1085", "context": "Human rights activists have urged consumers to be conscious of the labor practices employed by manufacturers, especially in developing nations, saying that if we accept moral responsibility for our impact on third world workers, we must buy products from manufacturers that do not take advantage of their workers. It is nearly impossible, however, to determine which manufacturers are employing fair labor practices. Because of this impossibility there can be no moral obligation to choose products in the way these activists urge, because__.", "question": "Which one of the following principles provides the most appropriate completion for the argument?", "answers": "['A moral choice is more important than a moral action.', \"The morally correct option is the one that maximizes a person's effort to do good.\", 'It is more important to identify the moral choice than to act.', 'A person cannot have a moral duty to do what is impossible.']", "label": 3 }, { "id": "train_1086", "context": "Letter to the editor: According to your last edition' s anonymous article, we should all be required to carry identification cards and show them on demand. This opinion is wrong. After all, the writers of the article should not have asserted that the right to remain anonymous was trivial, unless they were prepared to put their names to that assertion.", "question": "The reasoning above is most vulnerable to criticism on the grounds that it", "answers": "['commits the same error in reasoning that it accuses the writers of committing', 'confuses two meanings of the term \"anonymous\"', 'attacks the integrity of the writers without knowing anything about them', \"diverts attention from the content of the article by focusing on the writers' actions\"]", "label": 3 }, { "id": "train_1087", "context": "A recent study suggests that consuming three glasses of wine daily substantially decreases the risk of stroke. Critics of the study, defending earlier research recommending one glass of wine daily, claim that binge drinkers (who drink once a week or less, but drink three or more drinks when they do drink) are the people most likely to drink three glasses of wine in one day and are more likely to die from sudden heart attacks than are other drinkers. According to these critics, drinking three glasses of wine daily would not benefit health overall, since the decrease in the risk of stroke associated with that level of consumption is negated by its associated increased risk of sudden heart attack.", "question": "The critics' argument is most vulnerable to criticism on the grounds that it", "answers": "['inappropriately attributes the consequences of binge drinking to persons whose regular consumption of wine is three glasses a day', 'presumes, without providing justification, that there is no significant difference between wine and other alcoholic beverages in terms of health benefits and risks', 'confuses the risk of sudden alcohol-induced heart attacks with other health risks', 'fails to address specifically the reduction in risk of stroke conferred by the level of consumption in the recent study']", "label": 0 }, { "id": "train_1088", "context": "Though sucking zinc lozenges has been promoted as a treatment for the common cold, research has revealed no consistent effect. Recently, however, a zinc gel applied nasally has been shown to greatly reduce the duration of colds. Since the gel contains zinc in the same form and concentration as the lozenges, the greater effectiveness of the gel must be due to the fact that cold viruses tend to concentrate in the nose, not the mouth.", "question": "Which of the following, if true, most seriously weakens the argument?", "answers": "['Experimental subjects who used the zinc gel not only had colds of shorter duration but also had less severe symptoms than did those who used a gel that did not contain zinc.', 'Drug-company researchers experimenting with a nasal spray based on zinc have found that it has much the same effect on colds as the gel does.', 'To make them palatable, zinc lozenges generally contain other ingredients, such as citric acid, that can interfere with the chemical activity of zinc.', 'The mechanism by which zinc affects the viruses that cause the common cold has not been conclusively established.']", "label": 2 }, { "id": "train_1089", "context": "The rise in the prosperity of England subsequent to 1840 can be attributed to the adoption of the policy of free trade, since economic conditions improved only when that policy had been implemented.", "question": "The reasoning in the above argument most closely parallels that in which one of the following?", "answers": "['An exhaustive search of the marshes last year revealed no sign of marsh hawks, so it can be assumed that a similar search this year would reveal equally little sign of that kind of bird.', 'Before the banks raised their interest rates, people on average incomes could almost afford a mortgage for an amount twice their salary, hence the rate increase has now put mortgages beyond their reach.', \"Since the improvement in the company's profitability began to occur after the vice president's new morale-building program was put in place, that program can be credited with the improved result.\", 'The extinction of the dinosaurs was brought about by an asteroid colliding with Earth, so their extinction could not have come before the collision.']", "label": 2 }, { "id": "train_1090", "context": "Scientific research that involves international collaboration has produced papers of greater influence, as measured by the number of times a paper is cited in subsequent papers, than has research without any collaboration. Papers that result from international collaboration are cited an average of seven times, whereas papers with single authors are cited only three times on average. This difference shows that research projects conducted by international research teams are of greater importance than those conducted by single researchers.", "question": "Which one of the following is an assumption on which the argument depends?", "answers": "['International research teams tend to be more generously funded than are single researchers.', 'It is possible to ascertain whether or not a paper is the product of international collaboration by determining the number of citations it has received.', 'The number of citations a paper receives is a measure of the importance of the research it reports.', 'The collaborative efforts of scientists who are citizens of the same country do not produce papers that are as important as papers that are produced by international collaboration.']", "label": 2 }, { "id": "train_1091", "context": "Editorialist: Despite the importance it seems to have in our lives, money does not really exist. This is evident from the fact that all that would be needed to make money disappear would be a universal loss of belief in it. We witness this phenomenon on a small scale daily in the rises and falls of financial markets, whose fluctuations are often entirely independent of concrete causes and are the results of mere beliefs of investors.", "question": "The conclusion of the editorialist's argument can be properly drawn if which one of the following is assumed?", "answers": "['Whatever is true of money is true of financial markets generally.', 'If everyone believes in something, then that thing exists.', 'Only if one can have mistaken beliefs about a thing does that thing exist, strictly speaking.', 'Anything that exists would continue to exist even if everyone were to stop believing in it.']", "label": 3 }, { "id": "train_1092", "context": "Although all birds have feathers and all birds have wings, some birds do not fly. For example, penguins and ostriches use their wings to move in a different way from other birds. Penguins use their wings only to swim under water at high speeds. Ostriches use their wings only to run with the wind by lifting them as if they were sails.", "question": "Which one of the following is most parallel in its reasoning to the argument above?", "answers": "['Ancient philosophers tried to explain not how the world functions but why it functions. In contrast, most contemporary biologists seek comprehensive theories of how organisms function, but many refuse to speculate about purpose.', 'Some chairs are used only as decorations, and other chairs are used only to tame lions. Ttherefore, not all chairs are used for sitting in despite the fact that all chairs have a seat and some support such as legs.', 'Sailing ships move in a different way from steamships. Both sailing ships and steamships navigate over water, but only sailing ships use sails to move over the surface.', 'Some musicians in a symphony orchestra play the violin, and others play the viola, but these are both in the same category of musical instruments, namely string instruments.']", "label": 1 }, { "id": "train_1093", "context": "Durth: Increasingly, businesses use direct mail advertising instead of paying for advertising space in newspapers, in magazines, or on billboards. This practice is annoying and also immoral. Most direct mail advertisements are thrown out without ever being read, and the paper on which they are printed is wasted. If anyone else wasted this much paper, it would be considered unconscionable.", "question": "Which one of the following most accurately describes Durth's method of reasoning?", "answers": "['claiming that direct mail advertising is immoral because one of its results would be deemed immoral in other contexts', 'asserting that other advertising methods do not have the negative effects of direct mail advertising', 'presenting a specific counterexample to the contention that direct mail advertising is not immoral', 'basing a conclusion on the claim that direct mail advertising is annoying to those who receive it']", "label": 0 }, { "id": "train_1094", "context": "Heern: I object to the mayor' s proposal that taxicabs pick up and drop off passengers only at designated stops. This proposal aims to reduce traffic jams and accidents, but if the mayor herself were affected by such a law, she would oppose it. Thus the proposal is without merit.", "question": "The reasoning in Heern's argument is most vulnerable to criticism on the grounds that this argument", "answers": "['takes for granted that the frequency and location of designated stops would inconvenience most taxicab users', \"takes for granted that other people would share the mayor's dissatisfaction with the proposed law\", 'presumes, without providing justification, that the proposed law would not be the most effective way to reduce traffic jams and accidents', \"focuses on the mayor's preferences instead of addressing the merits of the proposal\"]", "label": 3 }, { "id": "train_1095", "context": "Consumers will be hurt by the new lower ceilings on halibut catches. Given the law of supply and demand these restrictions are likely to result in an increase in the price of the fish.", "question": "Which one of the following, if assumed, would do most to justify the claim that the price of halibut will increase?", "answers": "['The demand for halibut will not decrease substantially after the new restrictions are imposed.', 'The demand for other fish will be affected by the new restrictions.', 'There is a connection between the supply of halibut and the demand for it.', 'The amount of halibut consumed represents a very small proportion of all fish consumed.']", "label": 0 }, { "id": "train_1096", "context": "Debater: As a pedagogical practice, lecturing embodies hierarchy, since the lecturer is superior to the student in mastery of the subject. But people learn best from peer interaction. Thus, the hierarchy in lecturing is a great weakness. Respondent: By definition, all teaching and learning are hierarchical, for all teaching and learning must proceed from simple to complex. In teaching mathematics, for example, arithmetic must precede calculus. Thus, the hierarchy in lecturing is a strength.", "question": "The respondent's reply to the debater's argument is most vulnerable to criticism on the grounds that the respondent", "answers": "['takes for granted that the conceptual structure of mathematics is sufficiently representative of the conceptual structure of at least some other academic disciplines', 'fails to consider the possibility that some characteristics of lecturing other than hierarchy are weaknesses', \"concedes one of the major assumptions on which the debater's argument depends\", 'applies a key concept to a different aspect of education than the aspect to which the debater applied it']", "label": 3 }, { "id": "train_1097", "context": "The report released by the interior ministry states that within the past 5 years the national land-reclamation program has resulted in a 19 percent increase in the amount of arable land within the country. If these figures are accurate, the program has been a resounding success. Senator Armand, a distinguished mathematician and a woman of indisputable brilliance, maintains, however, that the reclamation program could not possibly have been successful. Clearly, ttherefore, the figures cited in the report cannot be accurate.", "question": "The argument above exhibits an erroneous pattern of reasoning most similar to that exhibited by which one of the following?", "answers": "[\"Moira, after observing the finish of the 60-kilometer bicycle race, reports that Lee won with Adams a distant third. Lomas, a bicycle engineering expert, insists, however, that Lee could not have won a race in which Adams competed; so Moira's report cannot be true.\", \"Amos claims that he can hold his breath under water for a full hour. Dr. Treviso, a cardiopulmonary specialist, has stated that humans are physiologically incapable of holding their breath for even half that long; so Amos' claim cannot be true.\", 'Evelyn reports that she got home before midnight. Robert, who always knows the time, insists that she did not. If Robert is right, Evelyn could not possibly have listened to the late news; since she admits not having listened to the late news, her report cannot be true.', \"Albert's father claims that Albert does not know where the spare car keys are hidden. Yesterday, however, Albert reported that he had discovered the spare car keys in the garage toolbox, so his father's claim cannot be true.\"]", "label": 0 }, { "id": "train_1098", "context": "Editorialist: The national media is composed of private companies that depend on ratings to increase advertising revenue, which depends on how many people watch. People are only going to watch topics they find interesting. In much the same way that the local news focuses on violent crimes, the national media focuses on political scandals. Topics such as election reform are rarely covered.", "question": "The argument most strongly supports which one of the following assertions?", "answers": "['The national media covers violent crimes.', 'Election reform is not interesting to people.', 'The national media only covers political scandals.', 'The local news focuses on political scandals as well as violent crimes.']", "label": 1 }, { "id": "train_1099", "context": "One of the most vexing problems in historiography is dating an event when the usual sources offer conflicting chronologies of the event. Historians should attempt to minimize the number of competing sources, perhaps by eliminating the less credible ones. Once this is achieved and several sources are left, as often happens, historians may try, though on occasion unsuccessfully, to determine independently of the usual sources which date is more likely to be right.", "question": "Which one of the following inferences is most strongly supported by the information above?", "answers": "['Attaching a reliable date to any event requires determining which of several conflicting chronologies is most likely to be true.', 'The soundest approach to dating an event for which the usual sources give conflicting chronologies is to undermine the credibility of as many of these sources as possible.', 'Some of the events for which there are conflicting chronologies and for which attempts have been made by historians to determine the right date cannot be dated reliably by historians.', 'We have no plausible chronology of most of the events for which attempts have been made by historians to determine the right date.']", "label": 2 }, { "id": "train_1100", "context": "Taking advanced mathematics courses should increase a student' s grade point average, for, as numerous studies have shown, students who have taken one or more advanced mathematics courses are far more likely to have high grade point averages than students who have not taken such courses.", "question": "The flawed pattern of reasoning in the argument above is most similar to that in which one of the following?", "answers": "['Eating a diet consisting primarily of fats and carbohydrates may cause weight gain in some people. Studies have shown that many overweight people eat such diets.', 'Fur color is in large measure hereditary, for, as many studies have shown, black cats are more likely than others to have black kittens, and orange cats are more likely to have orange kittens.', 'Buying running shoes should increase the frequency with which a person exercises, since those who buy two or more pairs of running shoes each year tend to exercise more often than those who buy at most one pair.', 'Reading to children at an early age should inspire them to read on their own later, since studies have shown that children who have not been read to are less likely to develop an interest in reading than children who have been read to.']", "label": 2 }, { "id": "train_1101", "context": "Reviewer: Almost all books that offer management advice are written from the perspective of the CEO. But most managers aren' t CEOs and don' t have the same perspective as CEOs. So the advice in management books is of limited use for most managers.", "question": "The conclusion of the reviewer 's argument can be properly drawn if which one of the following is assumed?", "answers": "['Most managers prefer to read books that they think will be useful to them in their work.', 'Most people who read management advice books aspire to be CEOs.', 'Advice books rarely take the perspective of their intended audience.', 'Advice is of limited use unless it is offered from the perspective of the recipient.']", "label": 3 }, { "id": "train_1102", "context": "Brand X, which currently markets luxury watches via billboards and advertisements in popular magazines, is considering saving money by advertising its newest line of luxury watches only in niche publications read by the wealthy. A recent study shows that most purchasers of Brand X's luxury watches have only recently become wealthy, and purchase the watches because Brand X is highly coveted by those aspiring to wealth, who learn about Brand X watches via advertising. Ttherefore, Brand X should not implement its new advertising strategy because __.", "question": "Which of the following best completes the passage below?", "answers": "['the money saved by implementing the strategy will be outweighed by the reduction in revenue from the less expensive line of watches', 'consumers who are newly wealthy prefer to purchase products that are purchased regularly by the non-wealthy', 'newly rich consumers will not buy the watches if the watches are not advertised to people who are not yet wealthy who will then hold the watches in high regard', \"most purchasers of Brand X's luxury watches do not read magazines regularly\"]", "label": 2 }, { "id": "train_1103", "context": "I agree that Hogan' s actions resulted in grievous injury to Winters. And I do not deny that Hogan fully realized the nature of his actions and the effects that they would have. Indeed, I would not disagree if you pointed out that intentionally causing such effects is reprehensible, other things being equal. But in asking you to concur with me that Hogan' s actions not be wholly condemned I emphasize again that Hogan mistakenly believed Winters to be the robber who had been terrorizing west-side apartment buildings for the past several months.", "question": "Which one of the following most accurately expresses the conclusion of the argument?", "answers": "['Hogan thought that Winters was the person who had been terrorizing west-side apartment buildings for the last few months.', \"The robber who had been terrorizing west-side apartment buildings should be considered to be as responsible for Winters's injuries as Hogan.\", 'The actions of Hogan that seriously injured Winters were reprehensible, other things being equal.', 'The actions of Hogan that seriously injured Winters are not completely blameworthy.']", "label": 3 }, { "id": "train_1104", "context": "Everyone who is a gourmet cook enjoys a wide variety of foods and spices. Since no one who enjoys a wide variety of foods and spices prefers bland foods to all other foods, it follows that anyone who prefers bland foods to all other foods is not a gourmet cook.", "question": "The pattern of reasoning displayed in the argument above is most similar to that displayed in which one of the following?", "answers": "[\"All of the paintings in the Huang Collection are abstract. Since no abstract painting will be included in next week's art auction, nothing to be included in next week's art auction is a painting in the Huang Collection.\", 'All of the paintings in the Huang Collection are superb works of art. Since none of the paintings in the Huang Collection is by Roue, it stands to reason that no painting by Roue is a superb work of art.', \"Every painting from the Huang Collection that is to be auctioned off next week is a major work of art. No price can adequately reflect the true value of a major work of art. Hence the prices that will be paid at next week's auction will not adequately reflect the true value of the paintings sold.\", 'All of the paintings in the Huang Collection will be put up for auction next week. Since the paintings to be auctioned next week are by a wide variety of artists, it follows that the paintings in the Huang Collection are by a wide variety of artists.']", "label": 0 }, { "id": "train_1105", "context": "Herbicides allow cereal crops to be grown very efficiently, with virtually no competition from weeds. In Britain, partridge populations have been steadily decreasing since herbicide use became widespread. Some environmentalists claim that these birds, which live in and around cereal crop fields, are being poisoned by the herbicides. However, tests show no more than trace quantities of herbicides in partridges on herbicide-treated land. Ttherefore, something other than herbicide use must be responsible for the population decrease.", "question": "Which of the following, if true about Britain, most seriously weakens the argument?", "answers": "['Birds other than partridges that live in or around cereal crop fields have also been suffering population declines.', 'Some of the weeds that are eliminated from cereal crop fields by herbicides are much smaller than the crop plants themselves and would have no negative effect on crop yield if they were allowed to grow.', \"The elimination of certain weeds from cereal crop fields has reduced the population of the small insects that live on those weeds and that form a major part of partridge chicks' diet.\", 'The toxins contained in herbicides typically used on cereal crops can be readily identified in the tissues of animals that have ingested them.']", "label": 2 }, { "id": "train_1106", "context": "Sociologist: Research shows, contrary to popular opinion, that, all other things being equal, most people who have pets are less happy than most people who do not. Ttherefore, any person who wants to be as happy as possible would do well to consider not having a pet.", "question": "Which one of the following, if true, most seriously weakens the sociologist's argument?", "answers": "['Most people who have no pets occasionally wish that they had pets.', 'All people who have no pets admit to feeling unhappy sometimes.', 'Most people who have pets feel happier because they have pets.', 'Some people who have pets are happier than most people who do not.']", "label": 2 }, { "id": "train_1107", "context": "Winston: The rules for awarding Nobel Prizes stipulate that no more than three people can share the same prize. Nobel Prizes in scientific disciplines are generally given in recognition of particular scientific results, however, and many important results are the work of four or more scientists. Sanjay: Those rules also stipulate that prize winners must be living, but some highly influential scientists died before their work was fully appreciated.", "question": "The dialogue most strongly supports the claim that Winston and Sanjay agree that", "answers": "['the rules that govern the awarding of Nobel Prizes should be changed so that prizes can be awarded to deceased persons', \"Nobel Prizes are inaccurate indicators of scientists' contributions to their disciplines\", 'the rules that govern the awarding of Nobel Prizes in scientific disciplines should be different from the rules for other Nobel Prizes', 'Nobel Prizes in scientific disciplines should not be given in recognition of particular scientific results']", "label": 1 }, { "id": "train_1108", "context": "The average 40-year-old North American will have watched more than one million TV commercials in his or her lifetime. We may safely conclude, ttherefore, that the TV commercial has influenced North American habits of thought.", "question": "The conclusion above follows logically if which one of the following is assumed?", "answers": "['Anything people are exposed to in great quantity will influence their habits of thought.', 'It is impossible to avoid or ignore television commercials.', 'Some people find television commercials more interesting to watch than the programs themselves.', 'The habits of thought that people develop are largely determined by external influences.']", "label": 0 }, { "id": "train_1109", "context": "Economist: In general, several apprentices working together produce about the same amount in an hour as a single more highly trained worker. Hence the more highly trained worker can usually command several times the hourly wage of an apprentice. Thus if the apprentice wage is increased, the hourly wages of more highly trained workers will generally rise by a proportionate amount. Ttherefore the reason that more highly trained workers favor an increased apprentice wage is that it would increase their own wages.", "question": "The economist's reasoning is flawed because the economist takes for granted that", "answers": "['an increase in the apprentice wage would benefit only highly trained workers', 'if a policy change that people support would work in their favor, that is why they support it', 'the wages of highly trained workers will usually not increase unless the apprentice wage increases', 'if one event causes another event, then the first event occurs whenever the second event occurs']", "label": 1 }, { "id": "train_1110", "context": "Edwina: True appreciation of Mozart' s music demands that you hear it exactly as he intended it to be heard; that is, exactly as he heard it. Since he heard it on eighteenth-century instruments, it follows that so should we. Alberto: But what makes you think that Mozart ever heard his music played as he had intended it to be played? After all, Mozart was writing at a time when the performer was expected, as a matter of course, not just to interpret but to modify the written score.", "question": "Alberto adopts which one of the following strategies in criticizing Edwina's position?", "answers": "[\"He attacks her judgment by suggesting that she does not recognize the importance of the performer's creativity to the audience's appreciation of a musical composition.\", 'He defends a competing view of musical authenticity.', 'He appeals to an academic authority in order to challenge the factual basis of her conclusion.', 'He offers a reason to believe that one of the premises of her argument is false.']", "label": 3 }, { "id": "train_1111", "context": "People often pay more taxes than necessary, due to their failure to take advantage of the numerous deductibles offered by the government. If more people filed their taxes online, they would save more money.", "question": "The strength of the argument depends on which one of the following?", "answers": "['It is easier to take advantage of deductibles by filing taxes online.', \"Taking advantage of deductibles will not hurt the government's revenue.\", 'Saving money on taxes is beneficial.', 'The government makes it difficult to take advantage of deductibles.']", "label": 0 }, { "id": "train_1112", "context": "Petrochemical industry officials have said that the extreme pressure exerted on plant managers during the last five years to improve profits by cutting costs has done nothing to impair the industry's ability to operate safely. However, environmentalists contend that the recent rash of serious oil spills and accidents at petrochemical plants is traceable to cost-cutting measures.", "question": "Which of the following, if true, would provide the strongest support for the position held by industry officials?", "answers": "['There is evidence that the most damaging of the recent oil spills would have been prevented had cost-cutting measures not been instituted.', 'The petrochemical industry benefits if accidents do not occur, since accidents involve risk of employee injury as well as loss of equipment and product.', 'Despite major cutbacks in most other areas of operation, the petrochemical industry has devoted more of its resources to environmental and safety measures in the last five years than in the preceding five years.', 'Both the large fines and adverse publicity generated by the most recent oil spills have prompted the petrochemical industry to increase the resources devoted to oil-spill prevention.']", "label": 2 }, { "id": "train_1113", "context": "The goal of reforesting degraded land is to create an area with a multitude of thriving tree species. But some experienced land managers use a reforesting strategy that involves planting a single fast-growing tree species.", "question": "Which one of the following, if true, most helps to resolve the apparent discrepancy in the information above?", "answers": "['The process of reforesting degraded sites is time consuming and labor intensive.', 'The growth of trees attracts wildlife whose activities contribute to the dispersal of a large variety of tree seeds from surrounding areas.', 'Tree species that require abundant sunlight tend to grow quickly on degraded land.', 'The reforestation of degraded land is generally unsuccessful unless the land is planted with tree species that are native to the area designated for reforestation.']", "label": 1 }, { "id": "train_1114", "context": "Winston: The Public Transportation Authority (PTA) cannot fulfill its mandate to operate without a budget deficit unless it eliminates service during late-night periods of low ridership. Since the fares collected during these periods are less than the cost of providing the service, these cuts would reduce the deficit and should be made. Transit law prohibits unauthorized fare increases, and fare-increase authorization would take two years. Ping: Such service cuts might cost the PTA more in lost fares than they would save in costs, for the PTA would lose those riders who leave home during the day but must return late at night. Thus the PTA would lose two fares, while realizing cost savings for only one leg of such trips.", "question": "The relationship of Ping's response to Winston's argument is that Ping's response", "answers": "[\"carefully redefines a term used in Winston's argument\", \"questions Winston's proposal by raising considerations not addressed by Winston\", 'introduces detailed statistical evidence that is more persuasive than that offered by Winston', \"supplies a premise that could have been used as part of the support for Winston's argument\"]", "label": 1 }, { "id": "train_1115", "context": "The word \"loophole\" is a loaded, partisan word, one that implies wrongdoing and scandal. When \"loophole\" creeps into news stories, they start to read like editorials. So news reporters should not use the term \"loophole\" in their stories unless they provide evidence of wrongdoing.", "question": "Which one of the following principles, if valid, most helps to justify the reasoning in the argument?", "answers": "['Making use of a loophole never constitutes wrongdoing or scandal.', 'News stories need to give evidence to back up any suggestions of misconduct.', 'Editorials should meet the same journalistic standards as news stories.', 'Editorial writers should be free to use loaded, partisan words.']", "label": 1 }, { "id": "train_1116", "context": "Gecko lizards are found in any environment where there is an abundant population of gnats. Gnats can survive only in wet climates. Because there are no gecko lizards living here, there must not be an abundant population of gnats here. Consequently, the climate here must not be wet.", "question": "The reasoning in the argument is flawed in that it", "answers": "['presumes, without providing justification, that all wet environments contain abundant populations of gnats', 'does not mention whether gecko lizards eat anything besides gnats', 'does not consider whether small populations of gnats can survive in climates that are not wet', 'fails to establish that some gecko lizards could not survive in a dry climate containing only a small population of gnats']", "label": 0 }, { "id": "train_1117", "context": "According to some astronomers, Earth is struck by a meteorite large enough to cause an ice age on an average of once every 100 million years. The last such incident occurred nearly 100 million years ago, so we can expect that Earth will be struck by such a meteorite in the near future. This clearly warrants funding to determine whether there is a means to protect our planet from such meteorite strikes.", "question": "The reasoning in the argument is most subject to criticism on the grounds that the argument", "answers": "['presumes, without providing justification, that some feasible means can be found to deter large meteorite strikes', 'makes a bold prescription on the basis of evidence that establishes only a high probability for a disastrous event', 'moves from evidence about the average frequency of an event to a specific prediction about when the next such event will occur', \"presumes, without providing justification, that the probability of a chance event's occurring is not affected by whether the event has occurred during a period in which it would be expected to occur\"]", "label": 2 }, { "id": "train_1118", "context": "Columnist: The amount of acidic pollutants released into the air has decreased throughout the world over the last several decades. We can expect, then, an overall decrease in the negative environmental effects of acid rain, which is caused by these acidic pollutants.", "question": "Each of the following, if true, would weaken the columnist's argument EXCEPT:", "answers": "['Some ecosystems have developed sophisticated mechanisms that reduce the negative effects of increased levels of acids in the environment.', 'The current decrease in acidic pollutants is expected to end soon, as more countries turn to coal for the generation of electricity.', \"The soils of many ecosystems exposed to acid rain have been exhausted of minerals that help protect them from acid rain's harmful effects.\", 'The effects of acid rain are cumulative and largely independent of current acid rain levels.']", "label": 0 }, { "id": "train_1119", "context": "Professor: Unfortunately, pharmaceutical companies and other profit-driven institutions provide nearly all of the funding for the chemistry department' s research. Moreover, unless we can secure more funding for basic science research, it is highly unlikely that any significant advances in basic research will come out of the department. Thus, without increased funding from sources other than profit-driven institutions, the chemistry department is unlikely to gain the prestige that only achievements in basic science research confer.", "question": "Which one of the following is an assumption on which the professor's argument relies?", "answers": "[\"If the chemistry department's prestige increases substantially, then it is highly likely that the department's funding from sources other than profit-driven institutions will subsequently increase.\", \"The chemistry department's funding for basic science research is not likely to increase if its funding from sources other than profit-driven institutions does not increase.\", 'Members of the chemistry department are unlikely to make significant advances in basic science research if the department does not forego the funding it currently receives from profit-driven institutions.', \"The profit-driven institutions that currently provide almost all of the chemistry department's funding are not likely to benefit from basic scientific research.\"]", "label": 1 }, { "id": "train_1120", "context": "Travel agent: Studies show that people who travel frequently normally earn substantially more money, enjoy more time with their family, and have better physical health than people who do not travel often. Traveling is often described as a pleasurable experience. It is wonderful to see that traveling also increases a person' s quality of life.", "question": "The conclusion above is unwarranted because", "answers": "[\"the travel agent erroneously assumes that earning more money, spending time with family, and having better physical health are the only things that increase a person's pleasure while not traveling\", 'people who have more money, time to spend with their families, and better physical health are more likely to travel frequently', 'traveling might not itself be a pleasurable experience; instead, it might be done solely to earn more money, enjoy time with family, and have better physical health', 'people who do not travel frequently often do not find traveling a pleasurable experience']", "label": 1 }, { "id": "train_1121", "context": "When the products of several competing suppliers are perceived by consumers to be essentially the same, classical economics predicts that price competition will reduce prices to the same minimal levels and all suppliers' profits to the same minimal levels. Ttherefore, if classical economics is true, and given suppliers' desire to make as much profit as possible, it should be expected that__.", "question": "Which of the following best completes the passage below?", "answers": "['in a crowded market widely differing prices will be charged for products that are essentially the same as each other', 'when consumers are unable to distinguish the products in a crowded market, consumers will judge that the higher-priced products are of higher quality', \"each supplier in a crowded market will try to convince consumers that its product differs significantly from its competitors' products.\", \"suppliers in crowded markets will have more incentive to reduce prices and thus increase sales than to introduce innovations that would distinguish their product from their competitors' products\"]", "label": 2 }, { "id": "train_1122", "context": "Near many cities, contamination of lakes and rivers from pollutants in rainwater runoff exceeds that from industrial discharge. As the runoff washes over buildings and pavements, it picks up oil and other pollutants. Thus, water itself is among the biggest water polluters.", "question": "The statement that contamination of lakes and rivers from pollutants in rainwater runoff exceeds that from industrial discharge plays which one of the following roles in the argument?", "answers": "['It is a conclusion for which the claim that water itself should be considered a polluter is offered as support.', 'It is stated to provide an example of a typical kind of city pollution.', 'It is a premise offered in support of the conclusion that water itself is among the biggest water polluters.', 'It is a generalization based on the observation that rainwater runoff picks up oil and other pollutants as it washes over buildings and pavements.']", "label": 2 }, { "id": "train_1123", "context": "Scott: The Hippocratic oath demands, specifically, that doctors \"never divulge\" information about patients. Hence the psychiatrist who released tapes of a poet' s therapy sessions after the poet' s death violated his oath by his actions, even though the tapes were released after the poet' s death and to the poet' s official biographer. It makes no difference that the poet' s published works were written in a confessional manner or that she had assured the psychiatrist that he could do with the tapes as he saw fit. Bonara: 1 agree that doctors are bound not to divulge patient information and would not myself release such tapes without written permission from the patient. Nevertheless, I disagree that the circumstances were irrelevant in this case. I do not think the poet' s psychiatrist violated the Hippocratic oath.", "question": "Which one of the following principles, if established, helps most to justify Scott's evaluation of the psychiatrist's actions?", "answers": [ "If a patient has expressed an intention to make public information about himself or herself that is in a psychiatrist's possession, the psychiatrist is released from the obligation to keep that information confidential.", "Since any psychiatrist's divulging to the public information about a patient could undermine the trust of other patients in their psychiatrists, no patient can release a psychiatrist from the obligation to keep patient information confidential.", "Since a psychiatrist could influence a patient's opinions during therapy, any directives to the psychiatrist by the patient must be interpreted in the light of the patient's actions outside the therapeutic setting.", "Once a patient has granted a psychiatrist permission to release confidential information, that information can be released to anyone at the psychiatrist's discretion." ], "label": 1 }, { "id": "train_1124", "context": "While many people think of genetic manipulation of food crops as being aimed at developing larger and larger plant varieties, some plant breeders have in fact concentrated on discovering or producing dwarf varieties, which are roughly half as tall as normal varieties.", "question": "Which of the following would, if true, most help to explain the strategy of the plant breeders referred to above?", "answers": "['Plant varieties used as food by some are used as ornamentals by others.', 'Nations with large industrial sectors tend to consume more processed grains.', 'The wholesale prices of a given crop decrease as the supply of it increases.', 'Short plants are less vulnerable to strong wind and heavy rains.']", "label": 3 }, { "id": "train_1125", "context": "Archaeologist: How did the Parthenon' s stonemasons manage to carve columns that all bulged outward in the center in precisely the same way? One hypothesis is suggested by the discovery of a scale drawing of a column etched into the stone of a Greek temple at Didyma. The drawing is a profile view of a column surrounded by a grid, which makes it possible to determine the correct width at every height of the column. The stonemasons who carved the Parthenon' s columns may have relied on a drawing like the one at Didyma.", "question": "Which one of the following, if true, adds the most support for the archaeologist's hypothesis?", "answers": "['The construction of the temple at Didyma was begun over a century after the Parthenon was constructed.', 'The surviving columns at Didyma are almost twice as tall as the columns at the Parthenon.', 'Scale drawings were commonly used in many types of construction in ancient Greece.', 'Modern attempts to recreate columns like those at the Parthenon have only been partially successful.']", "label": 2 }, { "id": "train_1126", "context": "The level of triglycerides in the blood rises when triglycerides are inadequately metabolized. Research shows that patients with blood triglyceride levels above 1 milligram per milliliter are twice as prone to heart attacks as others. Thus, it is likely that consuming large amounts of fat, processed sugar, or alcohol, each known to increase triglyceride levels in the blood, is a factor causing heart disease.", "question": "Which one of the following, if true, most weakens the argument?", "answers": "[\"Heart disease interferes with the body's ability to metabolize triglycerides.\", 'People with a high-fat diet who engage in regular, vigorous physical activity are much less likely to develop heart disease than are sedentary people with a low-fat diet.', 'People who maintain strict regimens for their health tend to adopt low-fat diets and to avoid alcohol and processed sugar.', 'Triglyceride levels above 2 milligrams per milliliter increase the risk of some serious illnesses not related to heart disease.']", "label": 0 }, { "id": "train_1127", "context": "Journalist: Many people working on difficult problems in mathematics report going to sleep without a solution, but upon awaking discover they have a solution in mind. This phenomenon occurs among all age groups past infancy.", "question": "Which one of the following is most strongly supported by the journalist's statements?", "answers": "['Certain types of mathematical problems cannot be solved while one is consciously seeking a solution.', 'The ability to carry out mental processing gradually develops during infancy.', \"Everyone's mind works unconsciously on difficult problems while sleeping.\", 'Consciously seeking a solution is not the only mental process by which one can solve a mathematical problem.']", "label": 3 }, { "id": "train_1128", "context": "Ecologist: The incidence of alligator attacks on humans in the vicinity of the Blue Lagoon has increased in recent years. Relocating at least half of the Blue Lagoon's alligator population would help decrease the incidence of alligator attacks on humans.", "question": "The ecologist's claim relies on which of the following assumptions?", "answers": "[\"Relocating half of the lagoon's alligator population would not be prohibitively expensive.\", 'Not all alligator attacks on humans are reported to authorities.', 'In recent years, there has been no significant change in the size of the alligator population in the Blue Lagoon.', 'There is a correlation between the size of the alligator population and the incidence of alligator attacks on humans.']", "label": 3 }, { "id": "train_1129", "context": "A new government policy has been developed to avoid many serious cases of influenza. This goal will be accomplished by the annual vaccination of high-risk individuals: everyone 65 and older as well as anyone with a chronic disease that might cause them to experience complications from the influenza virus. Each year' s vaccination will protect only against the strain of the influenza virus deemed most likely to be prevalent that year, so every year it will be necessary for all high-risk individuals to receive a vaccine for a different strain of the virus.", "question": "Which one of the following is an assumption that would allow the conclusion above to be properly drawn?", "answers": "['The number of individuals in the high-risk group for influenza will not significantly change from year to year.', 'The likelihood that a serious influenza epidemic will occur varies from year to year.', 'Each year the strain of influenza virus deemed most likely to be prevalent will be one that had not previously been deemed most likely to be prevalent.', 'No vaccine for the influenza virus protects against more than one strain of that virus.']", "label": 2 }, { "id": "train_1130", "context": "Proposed new safety rules for the Beach City airport would lengthen considerably the minimum time between takeoffs from the airport. In consequence, the airport would be able to accommodate 10 percent fewer flights than currently use the airport daily. The city's operating budget depends heavily on taxes generated by tourist spending, and most of the tourists come by plane. Ttherefore, the proposed new safety rules, if adopted, will reduce the revenue available for the operating budget.", "question": "The argument depends on assuming which of the following?", "answers": "['Few, if any, of the tourists who use the Beach City airport do so when their main destination is a neighboring community and not Beach City itself.', 'If the proposed safety rules are adopted, the reduction in tourist numbers will not result mainly from a reduction in the number of tourists who spend relatively little in Beach City.', 'Increasing the minimum time between takeoffs is the only way to achieve necessary safety improvements without a large expenditure by the city government on airport enhancements.', 'The response to the adoption of the new safety rules would not include an increase in the number of passengers per flight.']", "label": 3 }, { "id": "train_1131", "context": "In Swartkans territory, archaeologists discovered charred bone fragments dating back one million years. Analysis of the fragments, which came from a variety of animals, showed that they had been heated to temperatures no higher than those produced in experimental campfires made from branches of white stinkwood, the most common tree around Swartkans.", "question": "Which of the following, if true, would, together with the information above, provide the best basis for the claim that the charred bone fragments are evidence of the use of fire by early hominids?", "answers": "['The white stinkwood tree is used for building material by the present-day inhabitants of Swartkans.', 'The bone fragments were fitted together by the archaeologists to form the complete skeletons of several animals.', 'Forest fires can heat wood to a range of temperatures that occur in campfires.', 'The bone fragments were found in several distinct layers of limestone that contained primitive cutting tools known to have been used by early hominids.']", "label": 3 }, { "id": "train_1132", "context": "No oral surgeons are orthodontists, and all members of the Patterson Group are orthodontists. Ttherefore, no members of the Patterson Group are employees of the Davidson Consortium.", "question": "The conclusion above is properly drawn if which one of the following is assumed?", "answers": "['All employees of the Davidson Consortium are oral surgeons.', 'All orthodontists are members of the Patterson Group.', 'No oral surgeons are members of the Patterson Group.', 'No employees of the Davidson Consortium are oral surgeons.']", "label": 0 }, { "id": "train_1133", "context": "Heart attacks are most likely to occur on Mondays. The accepted explanation is that because Monday is the first day of the workweek, people feel more stress on Mondays than on other days. However, research shows that even unemployed retired people are more likely to have heart attacks on Mondays than on other days.", "question": "Which one of the following, if true, most helps to explain the increased likelihood that an unemployed retiree will have a heart attack on a Monday?", "answers": "['Because they associate Monday with work, retired people are more likely to begin large projects on Mondays.', 'Many retired people take up part-time jobs after they retire from their careers.', 'People seldom change their dietary and other health habits after retirement.', 'Unemployed retired people are even more likely to have heart attacks than are people who have jobs.']", "label": 0 }, { "id": "train_1134", "context": "It has been suggested that a television set should be thought of as nothing more than \"a toaster with pictures\" and that since we let market forces determine the design of kitchen appliances we can let them determine what is seen on television. But that approach is too simple. Some governmental control is needed, since television is so important politically and culturally. It is a major source of commercial entertainment. It plays an important political role because it is the primary medium through which many voters obtain information about current affairs. It is a significant cultural force in that in the average home it is on for more than five hours a day.", "question": "Which one of the following most accurately expresses the role played in the argument by the claim that television is so important politically and culturally?", "answers": "['It is an intermediate conclusion that is offered in support of the claim that some governmental control of television is needed and for which the claim that the television is on for more than five hours a day in the average home is offered as partial support.', 'It is an intermediate conclusion that is offered in support of the claim that a television set should be thought of as nothing more than \"a toaster with pictures\" and for which the claim that we can let market forces determine what is seen on television is offered as support.', 'It is a premise that is offered in support of the claim that television is the primary medium through which many voters obtain information about current affairs.', 'It is a premise that is offered in support of the claim that we let market forces determine the design of kitchen appliances.']", "label": 0 }, { "id": "train_1135", "context": "A certain species of desert lizard digs tunnels in which to lay its eggs. The eggs must incubate inside the tunnel for several weeks before hatching, and they fail to hatch if they are disturbed at any time during this incubation period. Yet these lizards guard their tunnels for only a few days after laying their eggs.", "question": "Which of the following, if true, most helps explain why there is no need for lizards to guard their tunnels for more than a few days?", "answers": "['The eggs are at risk of being disturbed only during the brief egg-laying season when many lizards are digging in a relatively small area.', 'The temperature and humidity within the tunnels will not be suitable for the incubating eggs unless the tunnels are plugged with sand immediately after the eggs are laid.', 'The only way to disturb the eggs of this lizard species is by opening up one of the tunnels in which they are laid.', 'Each female lizard lays from 15 to 20 eggs, only about 10 of which hatch even if the eggs are not disturbed at any time during the incubation period.']", "label": 0 }, { "id": "train_1136", "context": "Concerns for the environment have led chemists to develop plastics that are degradable. All degradable plastics, which are potentially useful packaging materials, need just the right conditions to break down. Some need exposure to sunlight, some need to be buried in soil and some need to be submerged in water. It should be cautioned that some degradable plastics leave residues of unknown toxicity.", "question": "If all of the statements above are true, which one of the following must also be true?", "answers": "['Some materials that need to be buried in soil to break down leave residues of unknown toxicity.', 'Some degradable plastics need both sunlight and submersion in order to break down.', 'Some materials that are potentially useful for packaging leave residues of unknown toxicity.', 'Some materials that leave residues of unknown toxicity are not degradable plastics.']", "label": 2 }, { "id": "train_1137", "context": "Someone who gets sick from eating a meal will often develop a strong distaste for the one food in the meal that had the most distinctive flavor, whether or not that food caused the sickness. This phenomenon explains why children are especially likely to develop strong aversions to some foods.", "question": "Which one of the following, if true, provides the strongest support for the explanation?", "answers": "['Children tend to have more acute taste and to become sick more often than adults do.', 'Children typically recover more slowly than adults do from sickness caused by food.', 'Children are more likely than are adults to refuse to eat unfamiliar foods.', 'Children are less likely than adults to see a connection between their health and the foods they eat.']", "label": 0 }, { "id": "train_1138", "context": "During construction of the Quebec Bridge in 1907, the bridge' s designer, Theodore Cooper, received word that the suspended span being built out from the bridge' s cantilever was deflecting downward by a fraction of an inch. Before he could telegraph to freeze the project, the whole cantilever arm broke off and plunged, along with seven dozen workers, into the St. Lawrence River. It was the worst bridge construction disaster in history. As a direct result of the inquiry that followed, the engineering \"rules of thumb\" by which thousands of bridges had been built went down with the Quebec Bridge. Twentieth-century bridge engineers would thereafter depend on far more rigorous applications of mathematical analysis.", "question": "Which one of the following statements can be properly inferred from the passage?", "answers": "[\"Cooper's absence from the Quebec Bridge construction site resulted in the breaking off of the cantilever.\", 'Bridges built before about 1907 were built without thorough mathematical analysis and, ttherefore, were unsafe for the public to use.', 'Nineteenth-century bridge engineers relied on their rules of thumb because analytical methods were inadequate to solve their design problems.', 'Prior to 1907 the mathematical analysis incorporated in engineering rules of thumb was insufficient to completely assure the safety of bridges under construction.']", "label": 3 }, { "id": "train_1139", "context": "A dog who is emotionally indifferent and not securely attached to its human companion neither whimpers when the human leaves the room nor looks up to acknowledge the human' s return. Some dogs do not whimper when their human companions leave them at kennels. These dogs, ttherefore, are emotionally indifferent and not securely attached to their human companions.", "question": "The flawed pattern of reasoning in the argument above is most similar to that in which one of the following?", "answers": "['If new jobs are created, the unemployment rate will decrease. New jobs are being created by both new industries and rejuvenated older industries. Ttherefore, the unemployment rate will decrease.', \"At the bookstore's anniversary sale all novels are being sold at a discount. This travel guide is being sold at the bookstore but not at a discount. Ttherefore, the bookstore's anniversary sale has not begun yet.\", 'Creative people do not fear attempting to solve new problems and do not confine their thinking to solutions other people propose. Neena is not afraid of trying to solve new problems. Ttherefore, Neena is a creative person.', 'A happy person is neither bitter nor depressed. Some successful people are bitter. These people, ttherefore, are not happy.']", "label": 2 }, { "id": "train_1140", "context": "Ethicist: People who avoid alcoholic beverages simply because they regard them as a luxury beyond their financial means should not be praised for their abstinence. Similarly, those who avoid alcohol simply because they lack the desire to partake should not be praised, unless this disinclination has somehow resulted from an arduous process of disciplining oneself to refrain from acting indiscriminately on one' s desires.", "question": "Which one of the following principles, if valid, most helps to justify the ethicist's claims?", "answers": "['A person is praiseworthy for a particular behavior only if, in order to adopt that behavior, the person at some point had to overcome a desire to do something that she or he felt able to afford to do.', 'The apportionment of praise and blame should be commensurate with the arduousness or ease of the lives of those who receive praise or blame.', 'Whether behavior should be regarded as praiseworthy is a function of both its consequences and the social context in which the agent acts.', 'A person should be blamed for an action only if that action was not motivated by a desire to be virtuous or if the person did not have to overcome any obstacles in order to perform that action.']", "label": 0 }, { "id": "train_1141", "context": "The economy of Colonia has been in recession for the past eight years. Most companies that have not been forced into bankruptcy have survived thanks to the high efficiency of the employees they retained, which helped the companies control costs. In recent months, however, the Colonian economy has begun to recover, and companies are beginning to expand their workforces. Colonia, ttherefore, will soon experience a drop in average worker efficiency, since __.", "question": "Which of the following, if true, most logically complete the argument?", "answers": "['People who have been employed throughout the recession will, no doubt, continue to be employed.', 'Most companies will find that few of the workers available for hiring are as efficient as those they retained during the recession.', 'Many companies that were forced into bankruptcy in the past eight years had a fair number of efficient workers.', 'Colonia is expected to begin importing more goods from other countries.']", "label": 1 }, { "id": "train_1142", "context": "Credit card companies justify charging cardholders additional fees for late payments by asserting the principle that those who expose other individuals, companies, or institutions to financial risk should pay for that risk, and by pointing out that late-paying cardholders present a greater risk of default than other cardholders. Without late fees, the companies argue, they would have to spread the cost of the risk over all cardholders.", "question": "The principle invoked by the credit card companies would, if established, be most usefully invoked in which one of the following arguments?", "answers": "['School authorities should use student activity funds to pay for student-caused damages to school property since, even though only a few students cause any significant damage, authorities cannot in most instances determine which students caused the damage.', 'Municipalities should use tax money to pay for the maintenance of municipal roads, since if individuals paid for only those roads they used, some important roads in remote areas would be inadequately maintained.', 'Cities should impose high fines for littering. The risk of being caught littering is quite low, so the fine for those who are caught must be correspondingly high in order to deter people from littering.', 'Insurance companies should demand higher insurance rates of drivers of sports cars than of other drivers, since sports car divers are more likely to cause accidents and thus are more likely to require the companies to pay out money in claims.']", "label": 3 }, { "id": "train_1143", "context": "Computer store manager: Last year we made an average of 13 percent profit on the high-end computer models -- those priced over $1, 000 -- that we sold, while low-end models -- those priced below $1, 000 -- typically returned at least 25 percent profit. Since there is a limit to how many models we can display and sell, we should sell only low-end models. This would maximize our profits, since we would probably sell as many low-end models if that is all we sold as we would sell both kinds combined if we continued to sell both.", "question": "The reasoning in the manager's argument is vulnerable to criticism on which one of the following grounds?", "answers": "['The argument ignores the possibility that some customers who come into a computer store expecting to purchase a low-end model end up purchasing a high-end model.', 'The argument fails to address the possibility that, despite the price differential, the store sold as many high-end models as low-end models last year.', 'The argument fails to recognize that future sales of low-end computers may not be the same as past sales.', 'The argument fails to consider the possibility that the money earned on each high-end computer is significantly higher than the money earned on each low-end computer.']", "label": 3 }, { "id": "train_1144", "context": "Adult frogs are vulnerable to dehydration because of their highly permeable skins. Unlike large adult frogs, small adult frogs have such a low ratio of body weight to skin surface area that they cannot survive in arid climates. The animals' moisture requirements constitute the most important factor determining where frogs can live in the Yucatan peninsula, which has an arid climate in the north and a wet climate in the south.", "question": "The information above most strongly supports which one of the following conclusions about frogs in the Yucatan peninsula?", "answers": "['Fewer small adult frogs live in the south than do large adult frogs.', 'Large adult frogs can live in more of the area than small adult frogs can.', 'Small adult frogs in the south have less permeable skins than small adult frogs in the north.', 'Large adult frogs cannot coexist with small adult frogs in the wet areas.']", "label": 1 }, { "id": "train_1145", "context": "A doctor investigated whether an insufficient amount of dopamine in the brain causes Parkinson' s disease. The doctor injected dopamine into the bloodstreams of patients with Parkinson' s disease. The progress of the disease was not halted, nor did any of the patients improve. The doctor concluded that Parkinson' s disease must be caused by something other than a lack of dopamine in the brain.", "question": "Which of the following, if true, casts the most doubt on the doctor's conclusion?", "answers": "['Recent research has shown that dopamine, when injected into the bloods, cannot enter the brain.', \"Research has shown that dopamine often causes diseases other than Parkinson's disease.\", \"Parkinson's disease causes the brain to produce less dopamine than it normally would.\", 'Research has shown that many different chemicals interact to influence the workings of the brain.']", "label": 0 }, { "id": "train_1146", "context": "A favored theory to explain the extinction of dinosaurs, together with many other species, has been the globally catastrophic collision of a large asteroid with the Earth. Supporting evidence is an extraterrestrial chemical element in a layer of dust found worldwide at a geological level laid down contemporaneously with the supposed event. A new competing theory contends that any asteroid impact was irrelevant, because it was massive volcanic activity that caused the extinctions by putting enough dust into the atmosphere to cool the planet. The Deccan region of India contains extensive volcanic flows that occurred within the same time period as the supposed asteroid impact and the extinctions.", "question": "The new theory assumes that", "answers": "['the massive volcanic activity was not caused by the impact of an asteroid', 'the extinctions took place over a longer time period than they would have if caused by the impact of an asteroid', 'no individual dinosaurs survived the impact of the asteroid, if it occurred', 'it is not possible to determine which would have occurred first, the volcanic flows in the Deccan region or the supposed impact of an asteroid']", "label": 0 }, { "id": "train_1147", "context": "Chronic back pain is usually caused by a herniated or degenerated spinal disk. In most cases the disk will have been damaged years before chronic pain develops, and in fact an estimated one in five people over the age of 30 has a herniated or degenerated disk that shows no chronic symptoms. If chronic pain later develops in such a case, it is generally brought about by a deterioration of the abdominal and spinal muscles caused by insufficient exercise.", "question": "The statements above, if true, most strongly support which one of the following?", "answers": "['Four out of five people over the age of 30 can be sure they will never develop chronic back pain.', 'Doctors can accurately predict which people who do not have chronic back pain will develop it in the future.', 'People who exercise their abdominal and spinal muscles regularly are sure to be free from chronic back pain.', 'There is a strategy that can be effective in delaying or preventing the onset of pain from a currently asymptomatic herniated or degenerated spinal disk.']", "label": 3 }, { "id": "train_1148", "context": "A neighborhood group plans to protest the closing of the neighborhood' s only recreation center on the grounds that to do so would leave the neighborhood without local access to a recreation center. \"Our neighborhood already has the most residents per center of any neighborhood in the city, \" complained one resident, \"and closing this center would make the situation unacceptable since access to recreational facilities is a necessity for this neighborhood. ", "question": "Each of the following, if true, weakens the resident's argument EXCEPT:", "answers": "[\"Programs that are routinely filled at other recreation centers must be canceled at the neighborhood's recreation center due to lack of interest.\", 'As people become more involved in computers and computer games, recreation centers are becoming increasingly less important.', \"A large number of the neighborhood's residents are unable to travel outside their locality to gain access to recreational facilities.\", 'Often the recreation center in the neighborhood is open but not being used.']", "label": 2 }, { "id": "train_1149", "context": "There is little plausibility to the claim that it is absurd to criticize anyone for being critical. Obviously, people must assess one another and not all assessments will be positive. However, there is wisdom behind the injunction against being judgmental. To be judgmental is not merely to assess someone negatively, but to do so prior to a serious effort at understanding.", "question": "Which one of the following most accurately expresses the main conclusion drawn in the argument?", "answers": "['It is absurd to criticize anyone for being critical.', 'There is some plausibility to the claim that it is absurd to criticize anyone for being critical.', 'Not all assessments people make of one another will be positive.', 'There is wisdom behind the injunction against being judgmental.']", "label": 3 }, { "id": "train_1150", "context": "As regards memory, the brain responds best to repeated patterns, such as the melodic and rhythmic patterns of music. This is why we can remember long strings of information or text, which would normally be impossible to memorize, when they are put to music. Given that music aids memory, it might seem that funny jokes would be easy to remember, since, like music, they normally elicit an emotional response in us. However, jokes are usually very difficult to remember, since . __.", "question": "Which one of the following most logically completes the passage?", "answers": "['for most people, certain memories elicit a strong emotional response', 'people can hold in short-term memory only a few chunks of unpattemed information at a time', 'jokes work not by conforming to repeated patterns but by breaking them', 'jokes, unlike music, always have content that is verbal or at least clearly symbolic']", "label": 2 }, { "id": "train_1151", "context": "Statistical records of crime rates probably often reflect as much about the motives and methods of those who compile or cite them as they do about the actual incidence of crime. The police may underreport crime in order to convey the impression of their own success or overreport crime to make the case for a budget increase. Politicians may magnify crime rates to get elected or minimize them to remain in office. Newspapers, of course, often sensationalize crime statistics to increase readership.", "question": "The argument proceeds by doing which one of the following?", "answers": "['deriving implications of a generalization that it assumes to be true', 'citing examples in support of its conclusion', 'enumerating problems for which it proposes a general solution', 'showing how evidence that apparently contradicts its conclusion actually supports that conclusion']", "label": 1 }, { "id": "train_1152", "context": "The genuine creative genius is someone who is dissatisfied with merely habitual assent to widely held beliefs; thus these rare innovators tend to anger the majority. Those who are dissatisfied with merely habitual assent to widely held beliefs tend to seek out controversy, and controversy seekers enjoy demonstrating the falsehood of popular viewpoints.", "question": "The conclusion of the argument follows logically if which one of the following is assumed?", "answers": "['People who enjoy demonstrating the falsehood of popular viewpoints anger the majority.', 'People become angry when they are dissatisfied with merely habitual assent to widely held beliefs.', 'People tend to get angry with individuals who hold beliefs not held by a majority of people.', 'People who anger the majority enjoy demonstrating the falsehood of popular viewpoints.']", "label": 0 }, { "id": "train_1153", "context": "Columnist: Several recent studies show, and insurance statistics confirm, that more pedestrians are killed every year in North American cities when crossing with the light than when crossing against it. Crossing against the light in North American cities is ttherefore less dangerous than crossing with the light.", "question": "The columnist's reasoning is most vulnerable to criticism on the grounds that it", "answers": "['ignores possible differences in the frequency of the two actions whose risk is being assessed', 'presumes, without providing justification, that because two things are correlated there must be a causal relationship between them', 'does not adequately consider the possibility that a correlation between two events may be explained by a common cause', 'relies on sources that are likely to be biased in their reporting']", "label": 0 }, { "id": "train_1154", "context": "The shooting at the convenience store occurred just before dawn this morning, and the last police officer did not leave the scene until late this afternoon. With all the commotion, no one could have been anywhere within the vicinity of the scene of the shooting and fail to notice it. Jeffrey must have noticed the scene, no matter what he says. He admits that he went to work as usual that morning, and the only way for him to get from his home to his worksite is to go past the convenience store.", "question": "The main conclusion of the argument is that", "answers": "['Jeffrey did not go to work this morning', 'Jeffrey was in the vicinity of the shooting when it happened', 'Jeffrey claimed he did not notice the scene this morning', 'Jeffrey noticed the scene of the shooting this morning']", "label": 3 }, { "id": "train_1155", "context": "Ethicist: Marital vows often contain the promise to love \"until death do us part. \" If \"love\" here refers to a feeling, then this promise makes no sense, for feelings are not within one' s control, and a promise to do something not within one' s control makes no sense. Thus, no one -- including those making marital vows -- should take \"love\" in this context to be referring to feelings.", "question": "The ethicist's conclusion follows logically if which one of the following is assumed?", "answers": "['People should not make promises to do something that is not within their control.', 'Promises should not be interpreted in such a way that they make no sense.', 'Promises that cannot be kept do not make any sense.', '\"Love\" can legitimately be taken to refer to something other than feelings.']", "label": 1 }, { "id": "train_1156", "context": "Capuchin monkeys in Venezuela often rub a certain type of millipede into their fur. Secretions of these millipedes have been shown to contain two chemicals that are potent mosquito repellents, and mosquitoes carry parasites that debilitate the capuchins. The rubbing behavior is rare except during the rainy season, when mosquito populations are at their peak. Ttherefore, the monkeys probably rub the millipedes into their fur because doing so helps protect them against mosquitoes.", "question": "Which of the following would it be most useful to determine in order to evaluate the argument?", "answers": "['Whether the two chemicals provide any protection for the millipedes against their own predators', 'Whether animals other than capuchins rub insects of any kind into their fur', 'Whether the type of millipede used by the capuchin monkeys in Venezuela is found in other parts of the world', 'Whether the only time the millipedes are readily available to the capuchins is during the rainy season']", "label": 3 }, { "id": "train_1157", "context": "Many people think that the only way to remedy the problem of crime is by increasing the number of police officers, but recent statistics show that many major cities had similar ratios of police officers to citizens, yet diverged widely in their crime rates.", "question": "The statistics cited function in the argument to", "answers": "['illustrate the need for increasing the number of police officers in major cities', 'suggest that the number of police officers is not the only influence on the crime rate', 'demonstrate that there is no relation between the number of police officers and the crime rate', 'prove that there are factors other than the number of police officers that are more important in reducing the crime rate']", "label": 1 }, { "id": "train_1158", "context": "The translator of poetry must realize that word-for-word equivalents do not exist across languages, any more than piano sounds exist in the violin. The violin can, however, play recognizably the same music as the piano, but only if the violinist is guided by the nature and possibilities of the violin as well as by the original composition.", "question": "As applied to the act of translating poetry from one language into another, the analogy above can best be understood as saying that", "answers": "['some languages are inherently more musical and more suitable to poetic composition than others', 'the translator must observe the spirit of the original and also the qualities of expression that characterize the language into which the original is translated', 'the translator should be primarily concerned with reproducing the rhythms and sound patterns of the original, not with transcribing its meaning exactly', 'poetry is easier to translate if it focuses on philosophical insights or natural descriptions rather than on subjective impressions']", "label": 1 }, { "id": "train_1159", "context": "The Hyksos invaded the Nile Delta of Egypt and ruled it from 1650 to 1550 B. C. Their origin is uncertain, but archaeologists hypothesize that they were Canaatites. In support of this hypothesis, the archaeologists point out that excavations of Avaris, the Hyksos capital in Egypt, have uncovered large numbers of artifacts virtually identical to artifacts produced in Ashkelon, a major city of Canaan at the time of the Hyksos' invasion.", "question": "In order to evaluate the force of the archaeologists' evidence, it would be most useful to determine which of the following ?", "answers": "['Whether Avaris was the nearest Hyksos city in Egypt to Canaan', 'Whether Ashkelon after 1550 B. C. continued to produce artifacts similar to those found at Avaris', 'Whether any artifacts produced by the Hyksos after 1550 B. C. have been found in Egypt', 'Whether artifacts from Ashkelon were widely traded to non-Canaanite cities']", "label": 3 }, { "id": "train_1160", "context": "Muscular strength is a limited resource, and athletic techniques help to use this resource efficiently. Since top athletes do not differ greatly from each other in muscular strength, it follows that a requirement for an athlete to become a champion is a superior mastery of athletic techniques.", "question": "Which one of the following most accurately expresses the conclusion of the argument?", "answers": "['The differences in muscular strength between top athletes are not great.', 'No athlete can become a champion without a superior mastery of athletic techniques.', 'Only champion athletes have a superior mastery of athletic techniques.', 'Superior muscular strength is a requirement for an athlete to become a champion.']", "label": 1 }, { "id": "train_1161", "context": "Watching music videos from the 1970s would give the viewer the impression that the music of the time was dominated by synthesizer pop and punk rock. But this would be a misleading impression. Because music videos were a new art form at the time, they attracted primarily cutting-edge musicians.", "question": "Which one of the following arguments is most similar in its reasoning to that of the argument above?", "answers": "[\"Our notion of fashion trends will probably be accurate if we rely on TV fashion programs, despite the fact that these programs deliberately select the most outrageous outfits in order to get the viewers' attention.\", 'Our view of pre-printing-press literature can never be accurate, because the surviving works of ancient authors are those that were deemed by copyists most likely to be of interest to future readers.', 'Our memory of 1960s TV shows could hardly be improved, because so many of the television programs of the era are still rerun today.', \"Future generations' understanding of today's publishing trends will be distorted if they judge by works published in CD-ROM format, since it is primarily publishers interested in computer games that are using CD-ROM.\"]", "label": 3 }, { "id": "train_1162", "context": "Company policy: An employee of our company must be impartial, particularly when dealing with family members. This obligation extends to all aspects of the job, including hiring and firing practices and the quality of service the employee provides customers.", "question": "Which one of the following employee behaviors most clearly violates the company policy cited above?", "answers": "[\"never firing a family member, even though three of one's siblings work under one's supervision and authority\", \"refusing to hire any of one's five siblings, even though they are each more qualified than any other applicant\", 'repeatedly refusing to advance an employee, claiming that he has sometimes skipped work and that his work has been sloppy, even though no such instances have occurred for over two years', \"receiving over a hundred complaints about the service one's office provides and sending a complimentary product to all those who complain, including one's mother\"]", "label": 1 }, { "id": "train_1163", "context": "Bookstore owner: Consumers should buy books only from an independent bookstore, not from a bookstore that belongs to a bookstore chain. An independent bookstore tends to carry a much wider variety of books than does a chain bookstore, so because chains often threaten the existence of independent bookstores, they tend to reduce the variety of books available to consumers.", "question": "Which one of the following principles, if valid, most helps to justify the bookstore owner's argumentation?", "answers": "['Consumers should not make purchases from any bookstore that deliberately forces competing bookstores out of business.', 'If consumers have no access to any independent bookstore, they should buy books from the chain bookstore with the greatest diversity of books.', 'Consumers should buy books from only those bookstores whose existence does not tend to reduce the variety of books available to consumers.', 'The best interest of the bookselling business is not served when consumers purchase books from businesses whose existence tends to reduce the variety of books available to consumers.']", "label": 2 }, { "id": "train_1164", "context": "Frequently, people who diet to lose weight become trapped in a vicious cycle. When those people diet, they not only lose weight, but their bodies become used to fewer calories and become accustomed to functioning at that lower rate of caloric intake. As a result, when they stop dieting and go back to eating amounts of food that would have just maintained their weight in the days before the diet, they take in far more calories than they need. Those excess calories produce excess weight.", "question": "The passage above best supports which one of the following conclusions about people who diet to lose weight?", "answers": "['They are bound to end up weighing more than when they started dieting.', 'They must not go back to eating pre-diet amounts of food if they are to maintain their weight at the reduced level resulting from dieting.', 'They never can go back to their pre-diet caloric intake without regaining all of the weight lost by dieting.', 'They should not diet if they desire to maintain their reduced weight.']", "label": 1 }, { "id": "train_1165", "context": "Annie: Our university libraries have been sadly neglected. Few new books have been purchased during the last decade, and most of the older books are damaged. The university' s administrators should admit that their library policies have been in error and should remedy this situation in the fastest way possible, which is to charge students a library fee and use the funds for library improvements. Matilda: The current poor condition of the university libraries is the fault of the library officials, not the students. Students should not have to pay for the mistakes of careless library administrators.", "question": "Annie and Matilda disagree about whether", "answers": "['students will ultimately benefit from the library improvements that could be funded by additional student fees', 'funds for library improvements could be raised without additional student fees', 'those not responsible for the current condition of the libraries should bear the cost for remedying it', 'library improvements could be most quickly effected through charging students additional fees']", "label": 2 }, { "id": "train_1166", "context": "Some people prefer to avoid facing unpleasant truths and resent those whose unwanted honesty forces them into such a confrontation. Others dislike having any information, however painful, knowingly withheld from them. It is obvious then that if those in the former group are guided by the directive to treat others as they themselves want to be treated, __.", "question": "Which one of the following most reasonably completes the argument above?", "answers": "['they will sometimes withhold comment in situations in which they would otherwise be willing to speak', 'those in the latter group will respond by concealing unpleasant truths', 'they will sometimes treat those in the latter group in a manner the members of this latter group do not like', 'those in the latter group must be guided by an entirely different principle of behavior']", "label": 2 }, { "id": "train_1167", "context": "Aerobics instructor: Compared to many forms of exercise, kickboxing aerobics is highly risky. Overextending when kicking often leads to hip, knee, or lower-back injuries. Such overextension is very likely to occur when beginners try to match the high kicks of more skilled practitioners.", "question": "Which one of the following is most strongly supported by the aerobics instructor's statements?", "answers": "['Kickboxing aerobics is more risky than forms of aerobic exercise that do not involve high kicks.', 'To reduce the risk of injuries, beginners at kickboxing aerobics should avoid trying to match the high kicks of more skilled practitioners.', 'Beginners at kickboxing aerobics will not experience injuries if they avoid trying to match the high kicks of more skilled practitioners.', 'Skilled practitioners of kickboxing aerobics are unlikely to experience injuries from overextending while kicking.']", "label": 1 }, { "id": "train_1168", "context": "Legislator: The recently passed highway bill is clearly very unpopular with voters. After all, polls predict that the majority party, which supported the bill' s passage, will lose more than a dozen seats in the upcoming election.", "question": "The reasoning in the legislator's argument is most vulnerable to criticism on the grounds that the argument", "answers": "['takes for granted that the bill is unpopular just because the legislator wishes it to be unpopular', 'bases its conclusion on the views of voters without establishing their relevant expertise on the issues involved', 'gives no reason to think that the predicted election outcome would be different if the majority party had not supported the bill', 'focuses on the popularity of the bill to the exclusion of its merit']", "label": 2 }, { "id": "train_1169", "context": "S: Our nation is becoming too averse to risk. We boycott any food reported to contain a toxic chemical, even though the risk, as a mathematical ratio, might be minimal. With this mentality, Columbus would never have sailed west. T: A risk-taker in one context can be risk-averse in another: the same person can drive recklessly, but refuse to eat food not grown organically.", "question": "T responds to S by showing that", "answers": "['risk cannot be defined in relation to perceived probable benefit', 'a distinction should be made between avoidable and unavoidable risks', 'to risk cannot be reliably assessed without reference to context', 'mathematical odds concerning risk give an unwarranted impression of precision']", "label": 2 }, { "id": "train_1170", "context": "Companies wishing to boost sales of merchandise should use in-store displays to catch customers' attention. According to a marketing study, today' s busy shoppers have less time for coupon-clipping and pay little attention to direct-mail advertising; instead, they make two-thirds of their buying decisions on the spot at the store.", "question": "Which one of the following is an assumption that the argument requires?", "answers": "[\"Many of today's shoppers are too busy to pay careful attention to in-store displays.\", 'Coupons and direct-mail advertising were at one time more effective means of boosting sales of merchandise than they are now.', \"Companies are increasingly using in-store displays to catch customers' attention.\", \"In-store displays that catch customers' attention increase the likelihood that customers will decide on the spot to buy the company's merchandise.\"]", "label": 3 }, { "id": "train_1171", "context": "Biologists with a predilection for theory have tried-and largely failed-to define what it is that makes something a living thing. Organisms take in energy- providing materials and excrete waste products, but so do automobiles. Living things replicate and take part in evolution, but so do some computer programs. We must be open to the possibility that there are living things on other planets. Ttherefore, we will not be successful in defining what it is that makes something a living thing merely by examining living things on Earth--the only ones we know. Trying to do so is analogous to trying to specify__.", "question": "Which of the following most logically completes the passage?", "answers": "['the laws of physics by using pure mathematics', 'what a fish is by listing its chemical components', 'what an animal is by examining a plant', 'what a mammal is by examining a zebra']", "label": 3 }, { "id": "train_1172", "context": "This year, the number of applications at College X increased by nearly 5%, while the size of the entering class and the structure of the admissions process remained the same. However, the admissions director of College X claims that this year a greater proportion of its applicants received offers of admission from the college than in years past.", "question": "Which of the following statements best reconciles the claim of the admissions director with other evidence presented in the argument?", "answers": "['This year College X moved up in all major rankings of undergraduate programs.', 'This year, the number of high school graduates who applied to colleges increased by 5%.', 'This year, nearly twice as many accepted applicants at College X decided to attend other colleges.', 'The admissions process at College X is substantially more competitive, as compared to other colleges of similar academic profile.']", "label": 2 }, { "id": "train_1173", "context": "Editorial: To qualify as an effective law, as opposed to merely an impressive declaration, a command must be backed up by an effective enforcement mechanism. That is why societies have police. The power of the police to enforce a society' s laws makes those laws effective. But there is currently no international police force. Hence, what is called \"international law\" is not effective law.", "question": "Which one of the following is an assumption required by the editorial's argument?", "answers": "[\"The only difference between international law and the law of an individual society is the former's lack of an effective enforcement mechanism.\", 'If an international police force were established, then so-called international law would become effective law.', 'Only an international police force could effectively enforce international law.', 'The primary purpose of a police force is to enforce the laws of the society.']", "label": 2 }, { "id": "train_1174", "context": "In a study of the effect of radiation from nuclear weapons plants on people living in areas near them, researchers compared death rates in the areas near the plants with death rates in areas that had no such plants. Finding no difference in these rates, the researchers concluded that radiation from the nuclear weapons plants poses no health hazards to people living near them.", "question": "Which one of the following, if true, most seriously weakens the researchers' argument?", "answers": "['Only a small number of areas have nuclear weapons plants.', 'Nuclear power plants were not included in the study.', 'Exposure to nuclear radiation can cause many serious diseases that do not necessarily result in death.', 'The researchers did not study the possible health hazards of radiation on people who were employed at the nuclear weapons plants if those employees did not live in the study areas.']", "label": 2 }, { "id": "train_1175", "context": "Lawyer: If you take something that you have good reason to think is someone else' s property, that is stealing, and stealing is wrong. However, Meyers had no good reason to think that the compost in the public garden was anyone else' s property, so it was not wrong for Meyers to take it.", "question": "The reasoning in the lawyer's argument is flawed in that the argument", "answers": "['takes a condition that by itself is enough to make an action wrong to also be necessary in order for the action to be wrong', \"fails to consider the possibility that the compost was Meyers' property\", \"concludes that something is certainly someone else's property when there is merely good, but not conclusive, reason to think that it is someone else's property\", \"takes for granted that Meyers would not have taken the compost if he had good reason to believe that it was someone else's property\"]", "label": 0 }, { "id": "train_1176", "context": "John: For 40 years, fluoride has been added to public drinking water. According to a study, fluoridated public drinking water when given to laboratory rats causes bone cancer. Ninety percent of all the male rats in the test sample were affected, but the female rats were unaffected. Even though I am healthy now, I should nevertheless stop drinking fluoridated water; only then will I be sure not to develop bone cancer.", "question": "Which one of the following is the strongest criticism of John's reasoning?", "answers": "['John does not consider whether fluoridated water causes other diseases.', 'John does not consider whether there were any brief periods during the 40 years in which fluoride was not added to the water.', 'John does not consider the possibility of other causes of bone cancer.', 'John does not focus on the positive effects that fluoridated water has on people']", "label": 2 }, { "id": "train_1177", "context": "Oscar: Emerging information technologies will soon make speed of information processing the single most important factor in the creation of individual, corporate, and national wealth. Consequently, the division of the world into northern countries -- in general rich -- and southern countries -- in general poor -- will soon be obsolete. Instead, there simply will be fast countries and slow countries, and thus a country' s economic well-being will not be a function of its geographical position but just a matter of its relative success in incorporating those new technologies. Sylvia: But the poor countries of the south lack the economic resources to acquire those technologies and will ttherefore remain poor. The technologies will thus only widen the existing economic gap between north and south.", "question": "Sylvia's reasoning depends on the assumption that", "answers": "['the speed at which information processing takes place will continue to increase indefinitely', 'there are technologies other than information technologies whose development could help narrow the existing economic gap between north and south', 'at least some of the rich countries of the north will be effective in incorporating new information technologies into their economies', 'the prosperity of the rich countries of the north depends, at least in part, on the natural resources of the poor countries of the south']", "label": 2 }, { "id": "train_1178", "context": "If a wig has any handmade components, it is more expensive than one with none. Similarly, a made-to-measure wig ranges from medium-priced to expensive. Handmade foundations are never found on wigs that do not use human hair. Furthermore, any wig that contains human hair should be dry-cleaned. So all made-to-measure wigs should be dry-cleaned.", "question": "The conclusion of the argument follows logically if which one of the following is assumed?", "answers": "['Any wig whose price falls in the medium-priced to expensive range has a handmade foundation.', \"If a wig's foundation is handmade, then its price is at least in the medium range.\", 'Any wig that should be dry-cleaned has a foundation that is handmade.', \"If a wig's foundation is handmade, then it is more expensive than one whose foundation is not handmade.\"]", "label": 0 }, { "id": "train_1179", "context": "Kennel club members who frequently discipline their dogs report a higher incidence of misbehavior than do members who rarely or never discipline their dogs. We can conclude from this that discipline does not improve dogs' behavior; on the contrary, it encourages misbehavior.", "question": "The argument is flawed in that it fails to consider the possibility that", "answers": "['kennel club members are more likely to use discipline than are other dog owners', \"dogs' misbehavior is the cause of, rather than the result of, frequent discipline\", 'dogs learn from past experience how their owners are likely to react to misbehavior', 'kennel club members tend to be more skilled at raising dogs than are other dog owners']", "label": 1 }, { "id": "train_1180", "context": "Commentator: In academic scholarship, sources are always cited, and methodology and theoretical assumptions are set out, so as to allow critical study, replication, and expansion of scholarship. In open-source software, the code in which the program is written can be viewed and modified by individual users for their purposes without getting permission from the producer or paying a fee. In contrast, the code of proprietary software is kept secret, and modifications can be made only by the producer, for a fee. This shows that open-source software better matches the values embodied in academic scholarship, and since scholarship is central to the mission of universities, universities should use only open-source software.", "question": "The commentator's reasoning most closely conforms to which one of the following principles?", "answers": "['Universities should use the type of software technology that is least expensive, as long as that type of software technology is adequate for the purposes of academic scholarship.', 'A university should not pursue any activity that would block the achievement of the goals of academic scholarship at that university.', 'The form of software technology that best matches the values embodied in the activities that are central to the mission of universities is the form of software technology that is most efficient for universities to use.', 'Universities should choose the type of software technology that best matches the values embodied in the activities that are central to the mission of universities.']", "label": 3 }, { "id": "train_1181", "context": "Psychologist: Some psychologists mistakenly argue that because dreams result from electrical discharges in the brain, they must be understood purely in terms of their physiological function. They conclude, against Freud, that dreams reveal nothing about the character of the dreamer. But since dream content varies enormously, then even if electrical discharges provide the terms of the physiological explanation of dreams, they cannot completely explain the phenomenon of dreaming.", "question": "The claim that dream content varies enormously plays which one of the following roles in the argument?", "answers": "['It is used to illustrate the difficulty of providing a complete explanation of the phenomenon of dreaming.', 'It is used to support the explicitly stated conclusion that a fully satisfactory account of dreams must allow for the possibility of their revealing significant information about the dreamer.', 'It is used to support the anti-Freudian conclusion that some psychologists draw concerning dreams.', \"It is used to undermine a claim that some psychologists use to argue against a view of Freud's.\"]", "label": 3 }, { "id": "train_1182", "context": "Critic: Rock music is musically bankrupt and socially destructive, but at least the album covers of rock LPs from the 1960s and 1970s often featured innovative visual art. But now, since the success of digital music has almost ended the production of LPs, rock music has nothing going for it.", "question": "Which one of the following is an assumption on which the critic's argument relies?", "answers": "['In the 1960s and 1970s, only rock LPs featured innovative album cover art.', 'Digital music is not distributed with accompanying innovative visual art.', 'Although very few LPs are produced today, most of these are rock LPs.', 'The LPs being produced today have innovative album cover art.']", "label": 1 }, { "id": "train_1183", "context": "There have been no new cases of naturally occurring polio in North America in recent years. Yet there are approximately 12 new cases of polio each year in North America, all caused by the commonly administered live oral polio vaccine (OPV). Substituting inactivated polio vaccine (IPV) for most childhood polio immunizations would cut the number of cases of vaccination-caused polio about in half. Clearly it is time to switch from OPV to IPV as the most commonly used polio vaccine for North American children.", "question": "Which one of the following, if true, most weakens the argument?", "answers": "['Although IPV is preferred in some European nations, most countries with comprehensive child immunization programs use OPV.', 'The vast majority of cases of polio caused by OPV have occurred in children with preexisting but unsuspected immunodeficiency disorders.', 'If IPV replaces OPV as the most commonly used polio vaccine, at least a few new cases of naturally occurring polio in North America will result each year.', \"A child's risk of contracting polio from OPV has been estimated at 1 in 8. 7 million, which is significantly less than the risk of being struck by lightning.\"]", "label": 2 }, { "id": "train_1184", "context": "Books updating the classification systems used by many libraries are not free -- in fact they are very expensive. The only way to sell copies of them is to make the potential buyers believe they need to adopt the most recent system. Thus, these frequent changes in the classification systems are just a ploy by the publishers to make libraries buy their products.", "question": "The reasoning above is most vulnerable to criticism because it", "answers": "['concludes that there is no need ever to change classification systems', 'concludes that a possible ulterior motive must be the only motive', 'fails to consider that there may be potential buyers of these books other than libraries', 'fails to consider that the libraries cannot afford to buy every book they want']", "label": 1 }, { "id": "train_1185", "context": "A certain tropical island received food donations in the form of powdered milk for distribution to its poorest residents, who were thought to be malnourished. Subsequently, the rate of liver cancers among those islanders increased sharply. The donated milk was probably to blame: recent laboratory research on rats has shown that rats briefly exposed to the substances aflatoxin tend to develop liver cancer when fed casein, a milk protein. This result is relevant because __.", "question": "Which of the following most logically completes the passage?", "answers": "['powdered milk is the most appropriate form in which to send milk to a tropical destination', 'the people who were given the donated milk had been screened for their ability to digest milk', 'casein is not the only protein contained in milk', 'in the tropics, peanuts, a staple of these island residents, support a mold growth that produces aflatoxin']", "label": 3 }, { "id": "train_1186", "context": "Manufacturing plants in Arundia have recently been acquired in substantial numbers by investors from abroad. Arundian politicians are proposing legislative action to stop such investment, justifying the proposal by arguing that foreign investors, opportunistically exploiting a recent fall in the value of the Arundian currency, were able to buy Arundian assets at less than their true value.", "question": "Which of the following, if true, casts the most serious doubt on the adequacy of the Arundian politicians' justification for the proposed legislation?", "answers": "['Foreign investors who acquired Arundian manufacturing plants generally did so with no intention of keeping and running those plants over the long term.', 'In Concordia, a country broadly similar to Arundia, the share of manufacturing assets that is foreign- controlled is 60 percent higher than it is in Arundia.', 'The true value of an investment is determined by the value of the profits from it, and the low value of the Arundian currency has depressed the value of any profits earned by foreign investors from Arundian assets.', 'The Arundian government originally welcomed the fall in the value of the Arundian currency because the fall made Arundian exports more competitive on international markets.']", "label": 2 }, { "id": "train_1187", "context": "Veterinarian: A disease of purebred racehorses that is caused by a genetic defect prevents afflicted horses from racing and can cause paralysis and death. Some horse breeders conclude that because the disease can have such serious consequences, horses with this defect should not be bred. But they are wrong because, in most cases, the severity of the disease can be controlled by diet and medication, and the defect also produces horses of extreme beauty that are in great demand in the horse show industry.", "question": "The point of the veterinarian's response to the horse breeders is most accurately expressed by which one of the following?", "answers": "['Racehorses that have the genetic defect need not be prevented from racing.', 'Racehorses that are severely afflicted with the disease have not been provided with the proper diet.', 'There should not be an absolute ban on breeding racehorses that have the genetic defect.', 'There should be no prohibition against breeding racehorses that have any disease that can be controlled by diet and exercise.']", "label": 2 }, { "id": "train_1188", "context": "Most water companies in the United States add fluoride to tap water to help prevent cavities. Some dentists argue, however, that this practice actually causes more harm than good because people overestimate the protection afforded by the fluoride and do not take the proper steps to care for their teeth, such as brushing and flossing after every meal. If water companies did not add fluoride, the dentists claim, people would be forced to be more active in their dental hygiene and tooth decay would decline as a result.", "question": "Which of the following, if true, most weakens the dentists' claims?", "answers": "['Most Americans already brush their teeth daily.', 'Annual visits to the dentist are the most effective means of controlling tooth decay.', 'Most Americans are not aware that fluoride is added to tap water.', 'Fluoride is widely used in commercially available dental care products.']", "label": 2 }, { "id": "train_1189", "context": "No nonfiction book published by Carriage Books has ever earned a profit. Since Carriage Books earned a profit on every book it published last year, it clearly did not publish a nonfiction book last year.", "question": "The pattern of reasoning in the argument above is most similar to that in which one of the following arguments?", "answers": "['James Benson has never done business with the city of Waldville. Since Waldville only maintains business files on individuals that it does business with, it clearly does not have a business file on James Benson.', 'Pranwich Corporation has never given a bonus to an employee in its marketing division. Since Pranwich gave bonuses to every one of its systems analysts last year, it is clear that the company employed no systems analysts in its marketing division at that time.', 'No actor represented by the talent agent Mira Roberts has ever won an important role in a major movie. Since every actor represented by Ms. Roberts had at least one important acting role last year, it is clear that none of those actors worked in a movie last year.', 'Conway Flooring has never installed hardwood flooring for any customer in Woodridge. Since Conway Flooring has had a lot of customers in Woodridge, the company clearly does not install hardwood flooring.']", "label": 1 }, { "id": "train_1190", "context": "Cognitive psychologist: The argument against IQ as a hereditary trait that is fixed throughout one's lifetime ignores the evidence of eye orientation. That is, there is a high correlation between the speed in which a person's eyes orient towards a stimulus and that person's IQ. Specifically, an experiment measured the number of milliseconds subjects required to orient their eyes to the where on a large screen a word was flashed. The study found that the more rapid the response the higher that person's IQ.", "question": "Which of the following is an assumption the cognitive psychologist makes?", "answers": "[\"The speed at which one can orient one's eye to a stimulus has also been highly correlated with overall health.\", 'The speed in which a person orients his or her eyes towards a stimulus is a skill that cannot be modified by experience.', 'The screen used in the experiment was so large that subjects had to shift their bodies in order to read the words presented on the screen.', \"The ability of scientists to measure a person's IQ depends on whether that person is literate.\"]", "label": 1 }, { "id": "train_1191", "context": "New legislation would require a seven-day waiting period in the sale of handguns to private individuals, in order that records of prisons could be checked and the sale of handguns to people likely to hurt other people thereby prevented. People opposed to this legislation claim that prison records are so full of errors that the proposed law would prevent as many law-abiding citizens as criminals from having access to handguns.", "question": "If the claim made by people opposed to the new legislation is true, which one of the following is a principle that, if established, would do the most to justify opposition to the new legislation on the basis of that claim?", "answers": "['Legislation should not be enacted if no benefit could accrue to society as a result of that legislation.', 'Even citizens who are neither fugitives nor felons should not be permitted to own a handgun unless they have received adequate training.', 'Nothing should be done to restrict potential criminals at the cost of placing restrictions on law-abiding citizens.', 'No restrictions should be placed on the sale of merchandise unless sale of that merchandise could endanger innocent people.']", "label": 2 }, { "id": "train_1192", "context": "Manager: One reason productivity in our office is not as high as it could be is that office workers spend too much time taking unauthorized breaks. Since the number of office workers assigned to each manager will soon be reduced, managers will be able to supervise workers more closely in the future to make sure that they are not taking unauthorized breaks. Ttherefore, productivity in our office will soon increase.", "question": "Which one of the following is an assumption on which the manager's argument depends?", "answers": "['Supervising employees more closely to reduce unauthorized breaks is the most efficient way of increasing overall office productivity.', 'The gain in productivity that will result from reducing unauthorized breaks will exceed any loss in productivity caused by closer supervision.', 'Giving financial incentives to workers whose divisions increase their productivity would not have any significant effect on overall office productivity.', 'The best way to improve productivity is to reduce the number of employees and give more work to employees who remain on the job.']", "label": 1 }, { "id": "train_1193", "context": "No one who works at Leila' s Electronics has received both a poor performance evaluation and a raise. Lester has not received a raise, so it must be that he has received a poor performance evaluation.", "question": "The flawed reasoning in the argument above is most similar to the reasoning in which one of the following arguments?", "answers": "['Anyone who lives in a house but does not own it pays rent on it. My next-door neighbors do not own their house. Ttherefore, it must be that they pay rent on it.', 'No one who lives in a house both owns it and pays rent on it. So, since my next-door neighbors pay rent on their house, it must be that they do not own it.', 'My neighbors have not paid any rent on their house. Since anyone who lives in a house but does not rent it owns it, it must be that they own it.', 'My next-door neighbors do not own their house. Since no one who lives in a house both owns it and pays rent on it, it must be that my next-door neighbors pay rent on their house.']", "label": 3 }, { "id": "train_1194", "context": "Secondary school students achieve broad mastery of the curriculum if they are taught with methods appropriate to their learning styles and they devote significant effort to their studies. Thus, if such broad mastery is not achieved by the students in a particular secondary school, those students are not being taught with methods appropriate to their learning styles.", "question": "The conclusion can be properly drawn if which one of the following is assumed?", "answers": "['Secondary school students who devote significant effort to their studies do not always achieve broad mastery of the curriculum.', 'Even if secondary school students are taught with methods appropriate to their learning styles, they will not achieve broad mastery of the curriculum if they do not devote significant effort to their studies.', 'Secondary school students do not achieve broad mastery of the curriculum if they are not taught with methods appropriate to their learning styles.', 'As long as secondary school students are taught with methods appropriate to their learning styles, they will devote significant effort to their studies.']", "label": 3 }, { "id": "train_1195", "context": "Compared to us, people who lived a century ago had very few diversions to amuse them. Ttherefore, they likely read much more than we do today.", "question": "Which one of the following statements, if true, most weakens the argument?", "answers": "['Many of the books published a century ago were of low literary quality.', 'The number of books sold today is larger than it was a century ago.', 'One of the popular diversions of a century ago was horse racing.', 'On average, people who lived a century ago had considerably less leisure time than we do today.']", "label": 3 }, { "id": "train_1196", "context": "Art historian: This painting, purportedly by Mary Cassatt, is a forgery. Although the canvas and other materials are consistent with most of Cassatt' s work, and the subject matter is similar to that of Cassatt' s finest paintings, the brush style of this painting is not found in any work known to be Cassatt' s. Hence this painting is definitely not a genuine Cassatt.", "question": "The art historian's argument depends on assuming which one of the following?", "answers": [ "No painter other than Cassatt would be able to match Cassatt's brush style perfectly.", "None of Cassatt's works is painted using a brush style that is not exhibited in any of her known works.", "The most characteristic feature of Cassatt's work is her brush style.", "Cassatt's work generally had a characteristic subject matter that distinguished it from the work of other painters of her era." ], "label": 1 }, { "id": "train_1197", "context": "In fact, the state' s water systems are directly related to the health of the aquifers. Aquifers are underground, porous rocks that store groundwater. This water can be brought up to the surface through natural springs or through pumping. In this state, many communities use the aquifers for drinking water. This is why it' s so important that aquifers remain uncontaminated. Not only are they very difficult to fix after contamination, but many people rely on the purity of the water for their health and wellbeing.", "question": "Which of the following scenarios would the above statement most likely support?", "answers": "[\"Kyle is concerned with the economy of his community. He has a family of six and has decided that he'll support any legislation expanding job growth in his city.\", 'Gabriella is the CEO of a company that pumps groundwater out of the aquifers to provide to the public. She wants to educate the community on water conservation because she believes many people are unaware of the water scarcity perils we could face in the next century.', \"Abram loves free diving and wants to convince his friends that the nearby springs are a perfect place to learn. Although the springs are not as vibrant as in his mother's day, Abram still believes that he and his friends can enjoy the beauty of the aquifers.\", 'Nida is protesting a pipeline in North Dakota due to the high risk of contamination of the surrounding aquifers. The pipeline is meant to carry 570, 000 barrels of oil a day.']", "label": 3 }, { "id": "train_1198", "context": "Adults have the right to vote; so should adolescents. Admittedly, adolescents and adults are not the same. But to the extent that adolescents and adults are different, adults cannot be expected to represent the interests of adolescents. If adults cannot represent the interests of adolescents, then only by giving adolescents the vote will these interests be represented.", "question": "The argument relies on which one of the following assumptions?", "answers": "['The right to vote is a right that all human beings should have.', 'Adolescents and adults differ in most respects that are important.', 'Adolescents should have their interests represented.', 'Anyone who has the right to vote has all the rights an adult has.']", "label": 2 }, { "id": "train_1199", "context": "University president: When a faculty member' s falsification of research was uncovered, the media treated it as evidence of the university' s low standards, even though in truth it was a mere case of dishonesty. But since vigilance with respect to academic standards is always necessary, it' s good that standards have become a topic of discussion.", "question": "Which one of the following conforms most closely to the principle illustrated above?", "answers": "['The latest government scandal has been analyzed as a case of simple corruption, although corruption had little to do with it. Because the true cause of the scandal was lack of oversight, attributing the cause of the scandal to simple corruption is harmful.', 'The latest government scandal has been analyzed as a case of simple corruption, with no mention of the role played by lack of oversight. Nonetheless, the focus on corruption is welcome, because corruption played the largest role in the scandal.', 'The latest government scandal has been attributed to lack of oversight, although the true cause of the scandal was simple corruption. Nonetheless, this discussion of oversight is welcome, because oversight is important in its own right.', 'The latest government scandal has been attributed to both lack of oversight and corruption. As a result, these important concerns are now being discussed. So, despite the harm that it caused, it is good that the scandal occurred.']", "label": 2 }, { "id": "train_1200", "context": "The fact that people who run marathons are ill less often than other people does not prove that running marathons prevents illness, for whether one runs marathons or not depends in part on one' s preexisting state of health.", "question": "The reasoning in which one of the following arguments is most similar to that in the argument above?", "answers": "['The fact that two diseases have similar symptoms does not establish that they have the same underlying cause, for dissimilar causes can have similar effects.', 'Since what music people tend to listen to often depends on what music their friends listen to, the fact that a person listens to a style of music does not prove that he or she truly prefers that style to any other.', 'Having strong verbal skills encourages people to read more, so the fact that habitual readers tend to be verbally skilled does not prove that reading produces verbal skill.', 'Musical and mathematical skills are often produced by the same talent for perceiving abstract patterns, so the fact that some mathematicians are not skilled musicians does not prove that they lack the talent that can produce musical skill.']", "label": 2 }, { "id": "train_1201", "context": "Retailers that excel in neither convenience nor variety of merchandise tend not to be very successful. Yet many successful retailers excel in just one of the areas and meet competitors' standards for the other. Hence, a retailer' s success need not depend on excellence in both areas.", "question": "The structure of the reasoning in the argument above is most parallel to that in which one of the following?", "answers": "['Runners who have only average speed and endurance are unlikely to win long-distance races. Some long-distance champions, however, win by being above average in speed or endurance only; ttherefore, being above average in both speed and endurance is not necessary.', 'Most plants that are neither ornamental nor edible are useless and are thus classified as weeds; yet many such plants are useful for purposes other than food or ornamentation, and are thus not classified as weeds. Hence, not all inedible and non-ornamental plants are weeds.', 'Excellence in a particular swimming stroke is not always necessary in order for a swimmer to win a race that requires each swimmer to use several different strokes in sequence, and many swimmers win these races without being the best at any of the strokes. Ttherefore, anyone who does excel at all the strokes is almost certain to win.', 'Apples that are neither especially firm nor especially flavorful are unsuitable for baking; yet while flavor is essential for both baking and eating, many flavorful apples that are soft are suitable for eating. Hence, the apples that are best for eating need not be both firm and flavorful.']", "label": 0 }, { "id": "train_1202", "context": "In a poll of a representative sample of a province' s residents, the provincial capital was the city most often selected as the best place to live in that province. Since the capital is also the largest of that province' s many cities, the poll shows that most residents of that province generally prefer life in large cities to life in small cities.", "question": "The argument is most vulnerable to the criticism that it", "answers": "['overlooks the possibility that what is true of the residents of the province may not be true of other people', 'overlooks the possibility that most people may have voted for small cities even though a large city received more votes than any other single city', 'takes for granted that when people are polled for their preferences among cities, they tend to vote for the city that they think is the best place to live', 'overlooks the possibility that the people who preferred small cities over the provincial capital did so not because of their general feelings about the sizes of cities, but because of their general feelings about capital cities']", "label": 1 }, { "id": "train_1203", "context": "For the United States meat industry, there is good news and bad news. The bad news is that over the last three years people in the United States have been eating less meat. The good news, is that during this same time period meat producers' profits have risen steadily.", "question": "Which of the following, if true during the last three years, contributes most to a resolution of the apparent discrepancy above?", "answers": "['In the United States many new restaurants that specialize in vegetarian meals have opened.', 'Meat producers have invested large amounts of money in advertising in order to win back consumers who have decreased their meet consumption.', 'Countries that are unable to produce enough meat for their populations have become a new and lucrative market for United States meat producers.', 'Competition from the growing fish industry has forced meat producers to sell meat at bargain prices in an attempt to attract more consumers.']", "label": 2 }, { "id": "train_1204", "context": "Some accountants calculate with simple adding machines, and some use complex computers. One can perform more calculations in less time with a computer than with an adding machine. Ttherefore, assuming the costs of using the two types of machines are equal, an accountant who uses a computer generally can earn more per hour than an accountant who uses an adding machine.", "question": "Which one of the following is an assumption that would make the conclusion in the passage a logical one?", "answers": "['The more hours an accountant spends on the job, the more money he or she will earn.', 'The more calculations an accountant performs, the more money he or she will earn.', 'In general, accountants vary in terms of the number of calculations they make and the amount of money they earn.', 'More accountants use computers than use adding machines.']", "label": 1 }, { "id": "train_1205", "context": "The highest mountain ranges are formed by geological forces that raise the earth' s crust: two continent-bearing tectonic plates of comparable density collide and crumple upward, causing a thickening of the crust. The erosive forces of wind and precipitation inexorably wear these mountains down. Yet the highest mountain ranges tend to be found in places where these erosive forces are most prevalent.", "question": "Which one of the following, if true, most helps to reconcile the apparent conflict described above?", "answers": "['Patterns of extreme wind and precipitation often result from the dramatic differences in elevation commonly found in the highest mountain ranges.', 'Some lower mountain ranges are formed by a different collision process, whereby one tectonic plate simply slides beneath another of lesser density.', 'The amount of precipitation that a given region of the earth receives may vary considerably over the lifetime of an average mountain range.', 'The highest mountain ranges have less erosion-reducing vegetation near their peaks than do other mountain ranges.']", "label": 0 }, { "id": "train_1206", "context": "A child learning to play the piano will not succeed unless the child has an instrument at home on which to practice. However, good-quality pianos, whether new or secondhand, are costly. Buying one is justified only if the child has the necessary talent and perseverance, which is precisely what one cannot know in advance. Consequently, parents should buy an inexpensive secondhand instrument at first and upgrade if and when the child' s ability and inclination are proven.", "question": "Which of the following, if true, casts the most serious doubt on the course of action recommended for parents?", "answers": "['Learners, particularly those with genuine musical talent, are apt to lose interest in the instrument if they have to play on a piano that fails to produce a pleasing sound.', 'Very young beginners often make remarkable progress at playing the piano at first, but then appear to stand still for a considerable period of time.', 'Reputable piano teachers do not accept children as pupils unless they know that the children can practice on a piano at home.', 'In some parents, spending increasing amounts of money on having their children learn to play the piano produces increasing anxiety to hear immediate results.']", "label": 0 }, { "id": "train_1207", "context": "Researchers examined 100 people suffering from herniated disks in their backs. Five of them were found to have a defect in a particular gene. The researchers also examined 100 people who had no problems with the disks in their backs; none had the genetic defect. They concluded that the genetic defect increases the likelihood of herniated disks.", "question": "Which one of the following, if true, most strengthens the researchers' reasoning?", "answers": "['Another team of researchers examined a different group of 100 people who suffered from herniated disks, and they found that none of them had the defective gene.', 'The researchers also examined a group of 100 people who did not have the defective gene; 80 were found to have herniated disks in their backs.', 'When the researchers examined the family of one of the subjects who did not suffer from herniated disks, they found 30 family members who did not have the defective gene, and 20 of them suffered from herniated disks.', 'When the researchers examined the families of the 5 subjects who had the defective gene, they found that 30 family members also had the defective gene, and each of them suffered from herniated disks.']", "label": 3 }, { "id": "train_1208", "context": "Parents who wish to provide a strong foundation for the musical ability of their children should provide them with a good musical education. Since formal instruction is often a part of a good musical education, parents who wish to provide this strong foundation need to ensure that their children receive formal instruction.", "question": "The reasoning is most vulnerable to criticism on the grounds that it fails to consider that", "answers": "['some children might not be interested in receiving a strong foundation for their musical ability', 'there are many examples of people with formal instruction whose musical ability is poor', \"parents might not be the only source of a child's musical education\", 'formal instruction might not always be a part of a good musical education']", "label": 3 }, { "id": "train_1209", "context": "Generations of European-history students have been taught that a political assassination caused the First World War. Without some qualification, however, this teaching is bound to mislead, since the war would not have happened without the treaties and alliances that were already in effect and the military force that was already amassed. These were the deeper causes of the war, whereas the assassination was a cause only in a trivial sense. It was like the individual spark that happens to ignite a conflagration that was, in the prevailing conditions, inevitable.", "question": "Which one of the following most accurately restates the main point of the passage?", "answers": "['Though the statement that the assassination caused the war is true, the term \"cause\"more fundamentally applies to the conditions that made it possible for that event to start the war.', 'Although the conditions prevailing at the time the war started made war inevitable, if the war had not been triggered by the assassination it would not have taken the course with which students of history are familiar.', 'The assassination did not cause the war, since the assassination was only the last in a chain of events leading up to the war, each of which had equal claim to being called its \"cause. \"', 'The war was destined to happen, since the course of history up to that point could not have been altered.']", "label": 0 }, { "id": "train_1210", "context": "Joel: A myth is a narrative told to convey a community' s traditional wisdom. Myths are not generally told in the modern world because there are no longer bodies of generally accepted truths that can be conveyed in this way. Giselle: Of course there are myths in the modern world. For example, there is the myth of the machine: we see the human body as a machine, to be fixed by mending defective parts.", "question": "This may not be a narrative, but what medically trained specialist can deny the existence of that myth? Which one of the following most accurately characterizes Giselle's response to Joel's statement?", "answers": "['It assumes that Joel is a medically trained specialist.', 'It is based on an unsupported distinction between traditional societies and the modern world.', 'It offers a scientific explanation to a problem of literary theory.', \"It offers a counterexample that calls into question part of Joel's definition of myth.\"]", "label": 3 }, { "id": "train_1211", "context": "The more worried investors are about losing their money, the more they will demand a high potential return on their investment: great risks must be offset by the chance of great rewards. This principle is the fundamental one in determining interest rates, and it is illustrated by the fact that __ .", "question": "Which of the following best completes the passage below?", "answers": "['in times of high inflation, the interest paid to depositors by banks can actually be below the rate of inflation', 'successful investors are distinguished by an ability to make very risky investments without worrying about their money', 'lenders receive higher Interest rates on unsecured loans than on loans backed by collateral', 'at any one time, a commercial bank will have a single rate of interest that it will expect all of its individual borrowers to pay']", "label": 2 }, { "id": "train_1212", "context": "The government will purchase and install new severe weather sirens for this area next year if replacement parts for the old sirens are difficult to obtain. The newspaper claims that public safety in the event of severe weather would be enhanced if new sirens were to be installed. The local company from which replacement parts were purchased last year has since gone out of business. So, if the newspaper is correct, the public will be safer during severe weather in the future.", "question": "The argument's conclusion follows logically from its premises if which one of the following is assumed?", "answers": "['Replacement parts for the old sirens will be difficult to obtain if the government cannot obtain them from the company it purchased them from last year.', 'The newspaper was correct in claiming that public safety in the event of severe weather would be enhanced if new sirens were purchased.', 'Because the local company from which replacement parts had been purchased went out of business, the only available parts are of such inferior quality that use of them would make the sirens less reliable.', 'The local company from which replacement parts for the old sirens were purchased last year was the only company in the area that sold them.']", "label": 0 }, { "id": "train_1213", "context": "Field studies, which have long been a staple of anthropological research, involve the researcher living within the community being studied. However, the usefulness of field studies tends to be overrated by anthropologists. Although most anthropologists do realize that living within the community one is studying affects that community, they generally underestimate the extent of such effects.", "question": "Which one of the following most accurately expresses the conclusion drawn in the argument?", "answers": "['Field studies have been a central feature of anthropological research for a long time.', 'Anthropologists tend to overestimate the value of field studies.', \"Most anthropologists underestimate how much of an effect the researcher's presence has on a community being studied.\", 'Most anthropologists know that when they live within a community being studied, the community is affected at least somewhat.']", "label": 1 }, { "id": "train_1214", "context": "For years the beautiful Renaissance buildings in Palitito have been damaged by exhaust from the many tour buses that come to the city. There has been little parking space, so most buses have idled at the curb during each stop on their tour, and idling produces as much exhaust as driving. The city has now provided parking that accommodates a third of the tour buses, so damage to Palitito' s buildings from the buses' exhaust will diminish significantly.", "question": "Which of the following, if true, most strongly supports the argument?", "answers": "['Tour buses typically spend less than one-quarter of the time they are in Palitito transporting passengers from one site to another.', \"Palitito's Renaissance buildings are not threatened by pollution other than engine exhaust.\", \"The exhaust from Palitito's few automobiles is not a significant threat to Palitito's buildings.\", 'Some of the tour buses that are unable to find parking drive around Palitito while their passengers are visiting a site.']", "label": 0 }, { "id": "train_1215", "context": "Thirty years ago, the percentage of the British people who vacationed in foreign countries was very small compared with the large percentage of the British population who travel abroad for vacations now. Foreign travel is, and always has been, expensive from Britain. Ttherefore, British people must have, on average, more money to spend on vacations now than they did 30 years ago.", "question": "The argument requires assuming which one of the following?", "answers": "['If more of the British people of 30 years ago had had enough money to vacation abroad, more would have done so.', 'If foreign travel had been less expensive 30 years ago, British people would still not have had enough money to take vacations abroad.', 'If the percentage of British people vacationing abroad was lower 30 years ago, then the British people of 30 years ago must have spent more money on domestic vacations.', 'If British people are now wealthier than they were 30 years ago, then they must have more money to spend on vacations now than they did 30 years ago.']", "label": 0 }, { "id": "train_1216", "context": "The economy around Lake Paqua depends on fishing of the lake' s landlocked salmon population. In recent years, scarcity of food for salmon there has caused a decline in both the number and the size of the adult salmon in the lake. As a result, the region' s revenues from salmon fishing have declined significantly. To remedy this situation, officials plan to introduce shrimp, which can serve as a food source for adult salmon, into Lake Paqua.", "question": "Which of the following, if true, most seriously calls into question the plan's chances for success?", "answers": "['Salmon is not a popular food among residents of the Lake Paqua region.', 'The shrimp to be introduced into Lake Paqua are of a variety that is too small to be harvested for human consumption.', 'The primary food for both shrimp and juvenile salmon is plankton, which is not abundant in Lake Paqua.', 'Fishing regulations prohibit people from keeping any salmon they have caught in Lake Paqua that are smaller than a certain minimum size.']", "label": 2 }, { "id": "train_1217", "context": "G: The group of works exhibited in this year' s Metropolitan Art Show reveals a bias in favor of photographers. Equal numbers of photographers, sculptors, and painters submitted works that met the traditional criteria for the show, yet more photographs were exhibited than either sculptures or paintings. As you know, each artist was allowed to submit work in one medium only. H: How could there have been bias? All submitted works that met the traditional criteria -- and only those works -- were exhibited in the show.", "question": "If both G's assertions and H's assertion are true, which one of the following must also be true?", "answers": "['Some of the photographs submitted for the Metropolitan Art Show did not meet the traditional criteria for the show.', 'More photographers than sculptors or painters submitted works to be considered for exhibition in the Metropolitan Art Show.', 'More works that met the traditional criteria for the Metropolitan Art Show were submitted by photographers than by sculptors or painters.', 'All the works submitted for the Metropolitan Art Show met the traditional criteria for the show.']", "label": 2 }, { "id": "train_1218", "context": "Thyrian lawmaker: Thyria's Cheese Importation Board inspects all cheese shipments to Thyria and rejects shipments not meeting specified standards. Yet only 1 percent is ever rejected. Ttherefore, since the health consequences and associated economic costs of not rejecting that 1 percent are negligible, whereas the board's operating costs are considerable, for economic reasons alone the board should be disbanded. Consultant: I disagree. The threat of having their shipments rejected deters many cheese exporters from shipping substandard product.", "question": "The consultant responds to the lawmaker's argument by __.", "answers": "[\"pointing out a benefi t of maintaining the board, which the lawmaker's argument has failed to consider\", \"objecting to the lawmaker's introducing into the discussion factors that are not strictly economic\", 'shifting the discussion from the argument at hand to an attack on the integrity of the cheese inspectors', \"rejecting the lawmaker's argument while proposing that the standards according to which the board inspects imported cheese should be raised\"]", "label": 0 }, { "id": "train_1219", "context": "Many people believe that in every group of attorneys, there are only a small number of contemptible ones. However, if being antagonistic is a contemptible quality, then all successful attorneys are contemptible. Attorneys must be antagonistic in order to serve their clients well. People who are overly friendly and always seek to be gracious and affable will never be successful attorneys.", "question": "Assuming that the author's statements are accurate, which one of the following statements CANNOT be true?", "answers": "['Most people believe that all attorneys are contemptible.', 'Some people believe that all attorneys should be overly friendly and always seek to be gracious and affable.', 'Some overly friendly attorneys, who always seek to be gracious and affable, are successful.', 'Most attorneys are overly friendly and always seek to be gracious and affable.']", "label": 2 }, { "id": "train_1220", "context": "Two years ago, the human resources department at Harrison' s Technologies had 175 sexual harassment cases open, a number that at that time led all companies in the sector. At that time, a typical claim took six months to process and clear. In response to this high number, the CEO' s Advisory Team published a comprehensive set of guidelines for training employees in the identification and avoidance of behaviors that could constitute sexual harassment. At this writing, the human resources department has only sixty sexual harassment cases open. Clearly, the guidelines published by the CEO' s Advisory Team were effective in reducing the occurrence of new sexual harassment claims at Harrison' s Technologies.", "question": "Which of the following, if true, most seriously weakens the conclusion above?", "answers": "[\"The human resources department at Anthony Information Industries, a principal rival of Harrison's with a similar number of employees, opened no new sexual harassment claims in the past year.\", \"New paperwork and procedure, also introduced two years ago, has allowed the human resources department at Harrison's Technologies to process and close almost all new claims of sexual harassment in less than one month.\", 'The majority of sexual harassment claims lead to the termination of employees found guilty; new employees hired to fill these positions need to be trained in the guidelines.', \"Independent consultants using scientifically designed surveys have found that the majority of current employees at Harrison's Technologies are not properly educated on exactly what behaviors constitute sexual harassment.\"]", "label": 1 }, { "id": "train_1221", "context": "A senator, near the end of his first six-year term and running for reelection, made the claim: \"Citizens of our state are thriving. While national unemployment levels have remained high, our state unemployment rate has been at astonishingly low levels for eleven years running. Clearly, everyone in our state has benefitted from the economical packages I have introduced during my time in the Senate. Ttherefore, grateful citizens of our state ought to vote for my second term. ", "question": "This argument is most vulnerable to what criticism?", "answers": "['The economical packages introduced by the senator may not have been as beneficial to citizens of other states.', 'It takes a condition to be the effect of something that has happened only after the condition already existed.', 'It introduces several different types of evidence, not all of which are compatible with one another.', 'Even if what the senator is saying is true, it may not be in his self-interest to argue in favor of it.']", "label": 1 }, { "id": "train_1222", "context": "Philosopher: Both the consequences and the motives of human actions have bearing on the moral worth of those actions. Nonetheless, to be a moral agent one must have free will, because one cannot be a moral agent without desiring to conform to a principle.", "question": "The philosopher's argument requires the assumption that", "answers": "['nobody who acts without taking the consequences of the action into consideration is free', 'desiring to conform to a principle requires free will', 'it is impossible to have desires without also being a moral agent', 'one cannot be a moral agent if one lacks a concern for the consequences of actions']", "label": 1 }, { "id": "train_1223", "context": "Tanco, a leather manufacturer, uses large quantities of common salt to preserve animal hides. New environmental regulations have significantly increased the cost of disposing of salt water that results from this use, and, in consequence, Tanco is considering a plan to use potassium chloride in place of common salt. Research has shown that Tanco could reprocess the by-product of potassium chloride use to yield a crop fertilizer, leaving a relatively small volume of waste for disposal.", "question": "In determining the impact on company profits of using potassium chloride in place of common salt, it would be important for Tanco to research all of the following EXCEPT:", "answers": "['What environmental regulations, if any, constrain the disposal of the waste generated in reprocessing the by-product of potassium chloride?', 'Are the chemical properties that make potassium chloride an effective means for preserving animal hides the same as those that make common salt an effective means for doing so?', 'What difference, if any, is there between the cost of the common salt needed to preserve a given quantity of animal hides and the cost of the potassium chloride needed to preserve the same quantity of hides?', 'To what extent is the equipment involved in preserving animal hides using common salt suitable for preserving animal hides using potassium chloride?']", "label": 1 }, { "id": "train_1224", "context": "Country X contains many rivers that flow down from its high mountains. These rivers have been dammed to harness the hydroelectric power that can be derived from this resource. Enough power is generated to create a reserve that is more than enough to meet the country' s year-round energy needs. Yet, citizens of Country X often experience power shortages or even outages.", "question": "Which of the following, if true, best explains the situation described above?", "answers": "['The flow of the rivers is heavier in the spring than at other times of year.', 'Most of the electricity generated in Country X is sold to other countries.', 'Citizens of Country X rely heavily on electronic appliances in their homes.', 'Some of the most powerful rivers in Country X have yet to be dammed.']", "label": 1 }, { "id": "train_1225", "context": "Eight large craters run in a long straight line across a geographical region. Although some of the craters contain rocks that have undergone high-pressure shocks characteristic of meteorites slamming into Earth, these shocks could also have been caused by extreme volcanic events. Because of the linearity of the craters, it is very unlikely that some of them were caused by volcanoes and others were caused by meteorites. Thus, since the craters are all different ages, they were probably caused by volcanic events rather than meteorites.", "question": "Which one of the following statements, if true, would most strengthen the argument?", "answers": "['No known natural cause would likely account for eight meteorite craters of different ages forming a straight line.', 'A similar but shorter line of craters that are all the same age is known to have been caused by volcanic activity.', 'No known single meteor shower has created exactly eight impact craters that form a straight line.', 'There is no independent evidence of a volcanic event strong enough to have created the high-pressure shocks that are characteristic of meteorites slamming into Earth.']", "label": 0 }, { "id": "train_1226", "context": "Although this bottle is labeled \"vinegar, \" no fizzing occurred when some of the liquid in it was added to powder from this box labeled \"baking soda. \" But when an acidic liquid such as vinegar is added to baking soda the resulting mixture fizzes, so this bottle clearly has been mislabeled.", "question": "A flaw in the reasoning in the argument above is that this argument", "answers": "['depends on the use of the imprecise term \"fizz\"', 'assumes that the fact of a labeling error is proof of an intention to deceive', 'fails to exclude an alternative explanation for the observed effect', 'does not take into account the fact that scientific principles can be definitively tested only under controlled laboratory conditions']", "label": 2 }, { "id": "train_1227", "context": "By examining fossilized beetles, a research team has produced the most detailed description yet of temperatures in Britain over the past 22, 000 years. Fossils of species that still exist were selected and dated. When individuals of several species found in the same place were found to date to the same period, the known temperature tolerances of the existing beetle species were used to determine the maximum summer temperature that could have existed at that place and period.", "question": "The procedure of the researchers assumes which one of the following?", "answers": "['The temperature tolerances of the beetle species did not change significantly during the 22, 000-year period.', 'Beetles can tolerate warm weather better than cold weather.', 'The process of dating is more accurate for beetles than for other organisms.', 'Fossils of different species found in the same place belonged to different period.']", "label": 0 }, { "id": "train_1228", "context": "Loggerhead turtles live and breed in distinct groups, of which some are in the Pacific Ocean and some are in the Atlantic. New evidence suggests that juvenile Pacific loggerheads that feed near the Baja peninsula hatch in Japanese waters 10, 000 kilometers away. Ninety-five percent of the DNA samples taken from the Baja turtles match those taken from turtles at the Japanese nesting sites.", "question": "Which one of the following, if true, most seriously weakens the reasoning above?", "answers": "['The distance between nesting sites and feeding sites of Atlantic loggerhead turtles is less than 5, 000 kilometers.', 'Commercial aquariums have been successfully breeding Atlantic loggerheads with Pacific loggerheads for the last five years.', 'Ninety-five percent of the DNA samples taken from the Baja turtles match those taken from Atlantic loggerhead turtles.', 'Loggerhead hatchlings in Japanese waters have been declining in number for the last decade while the number of nesting sites near the Baja peninsula has remained constant.']", "label": 2 }, { "id": "train_1229", "context": "Along the mudflats of Garantia exist two types of annelids: earthworms and lugworms. Earthworms tend to emerge from the mud early in the morning when there is more moisture and nutrients. As a result, earthworms have access to more nutrients and thus reproduce far faster than the lugworm, which spends much of the day submerged in the mud. Nonetheless, the lugworm population far outnumbers that of the earthworm.", "question": "Which of the following, if true, helps LEAST to resolve the apparent discrepancy noted above?", "answers": "['Lugworms have specialized taste receivers, which allow them to access nutrients below ground far more easily than earthworms can.', \"The whooping crane, the earthworm's chief predator, hunts along the mudflats during the time the earthworms are feeding at the surface.\", 'Compared to the lugworm, earthworms are able to consume fewer nutrients at one time.', 'Lugworms resemble the color of the mudflat, and are thus more difficult to spot by predators than earthworms are.']", "label": 2 }, { "id": "train_1230", "context": "On average, corporations that encourage frequent social events in the workplace show higher profits than those that rarely do. This suggests that the EZ Corporation could boost its profits by having more staff parties during business hours.", "question": "Which one of the following, if true, most weakens the argument above?", "answers": "['Frequent social events in a corporate workplace leave employees with less time to perform their assigned duties than they would otherwise have.', 'The great majority of corporations that encourage frequent social events in the workplace do so at least in part because they are already earning above-average profits.', 'The EZ Corporation already earns above average profits, and it almost never brings play into the workplace.', 'Corporations that have frequent staff parties after business hours sometimes have higher profits than do corporations that have frequent staff parties during business hours.']", "label": 1 }, { "id": "train_1231", "context": "Commentator: The quality of health care is declining. Medical schools have been graduating fewer people than are needed to replace retiring physicians. Furthermore, on average, a physician now spends only 15 minutes with a patient on the patient' s first visit.", "question": "Which one of the following, if true, most weakens the commentator's argument?", "answers": "['Most patients visiting a physician are suffering from ailments that are not life threatening. Questions 16-17', 'The average length of all patient-physician visits is 20 minutes.', \"Most patients do not like to spend an inordinate amount of time in physicians' offices.\", 'Five years ago, the average first patient-physician visit lasted 10 minutes.']", "label": 3 }, { "id": "train_1232", "context": "Lawyer: One is justified in accessing information in computer files without securing authorization from the computer' s owner only if the computer is typically used in the operation of a business. If, in addition, there exist reasonable grounds for believing that such a computer contains data usable as evidence in a legal proceeding against the computer' s owner, then accessing the data in those computer files without the owner' s authorization is justified.", "question": "The principles stated by the lawyer most strongly support which one of the following judgments?", "answers": [ "Against the company owner's wishes, a police officer accessed some of the files on one of the company's computers. Although the computer was typically used in the operation of the company's business, the particular files accessed by the police officer were personal letters written by one of the company's employees. Thus, the police officer's unauthorized use of the computer was not justified.", "Police department investigators accessed the electronic accounting files of the central computer owned by a consulting firm that was on trial for fraudulent business practices without seeking permission from the firm's owners. Contrary to the investigators' reasonable beliefs, however, the files ultimately provided no evidence of wrongdoing. Nevertheless, the investigators' action was justified.", "Customs officials examined all of the files stored on a laptop computer confiscated from an importer whom they suspected of smuggling. Because there were reasonable grounds for believing that the computer had typically been used in the operation of the importer's legitimate business, the customs officials' action was justified.", "A police officer accessed, without Natalie's permission, files on the computer that Natalie owned and used exclusively in the operation of her small business. Since the police officer's search of the files on Natalie's computer produced no evidence usable in any legal proceeding against Natalie, the police officer's action was clearly not justified." ], "label": 1 }, { "id": "train_1233", "context": "Half a dozen Q40 airliners have crashed in the past two years. The manufacturer has countered claims that the Q40' s design is faulty by pointing out that, in each of the accidents, investigations indicated pilot error as the cause.", "question": "Which of the following, if true, provides the strongest support for the manufacturer's position?", "answers": "['One of the crashes occurred in very poor visibility when the pilot was attempting to land.', \"In four of the crashes, the pilot's error involved disregarding instructions or information supplied by air-traffic control\", 'Each of the investigations identified several factors other than pilot error as subsidiary causes.', 'Like most modern airliners, the Q40 was designed with the goal that it should be highly tolerant of pilot error.']", "label": 1 }, { "id": "train_1234", "context": "A clothing manufacturer reports that unsalable garments, those found to be defective by inspectors plus those returned by retailers, total 7 percent of the garments produced. Further, it reports that all of its unsalable garments are recycled as scrap, but the percentage of garments reported as recycled scrap is 9 percent.", "question": "Which one of the following, if true, could contribute most to explaining the discrepancy between the reported percentages?", "answers": "['The total number of garments produced by the manufacturer has increased slightly over the past year.', 'Some inspectors, in order to appear more efficient, tend to overreport defects.', 'Garments with minor blemishes are sent to wholesale outlets for sale at discounted prices and are not returned for recycling.', 'Insalable garments are recorded by count, but recycled garments are recorded by weight.']", "label": 3 }, { "id": "train_1235", "context": "People who object to the proposed hazardous waste storage site by appealing to extremely implausible scenarios in which the site fails to contain the waste safely are overlooking the significant risks associated with delays in moving the waste from its present unsafe location. If we wait to remove the waste until we find a site certain to contain it safely, the waste will remain in its current location for many years, since it is currently impossible to guarantee that any site can meet that criterion. Yet keeping the waste at the current location for that long clearly poses unacceptable risks.", "question": "The statements above, if true, most strongly support which one of the following?", "answers": "['The waste should be placed in the most secure location that can ever be found.', 'Moving the waste to the proposed site would reduce the threat posed by the waste.', 'Any site to which the waste could be moved will be safer than its present site.', 'Whenever waste must be moved, one should limit the amount of time allotted to locating alternative waste storage sites.']", "label": 1 }, { "id": "train_1236", "context": "Charlene: Environmental cleanup increasingly relies on microorganisms to metabolize pollutants. This has its limitations, though, since microbes become less active when a region' s temperature drops below normal. Olaf: I don' t think that' s right. Researchers studying oil spills in the Arctic and subtropical regions measured the carbon dioxide output of organisms that digest petroleum pollutants and found that Arctic and subtropical microbes ate oil at nearly the same rate.", "question": "Olaf's reply suggests that he misinterprets Charlene's use of the term", "answers": "['cleanup', 'active', 'limitations', 'normal']", "label": 3 }, { "id": "train_1237", "context": "Critics: Not so. The farms the academy selected to study were the ones that seemed most likely to be successful in using natural methods. What about the farmers who have tried such methods and failed? Which one of the following is the most adequate evaluation of the logical force of the critics' response?", "question": "Science Academy study: It has been demonstrated that with natural methods, some well-managed farms are able to reduce the amounts of synthetic fertilizer and pesticide and also of antibiotics they use without necessarily decreasing yields; in some cases yields can be increased.", "answers": "['The critics demonstrate that natural methods are not suitable for the majority of farmers.', 'The critics show that the result of the study would have been different if twice as many farms had been studied.', 'The issue is only to show that something is possible, so it is not relevant whether the instances studied were representative.', 'Success and failure in farming are rarely due only to luck, because farming is the management of chance occurrences.']", "label": 2 }, { "id": "train_1238", "context": "The faster a car is traveling, the less time the driver has to avoid a potential accident, and if a car does crash, higher speeds increase the risk of a fatality. Between 1995 and 2000, average highway speeds increased significantly in the United States, yet, over that time, there was a drop in the number of car-crash fatalities per highway mile driven by cars.", "question": "Which of the following, if true about the United States between 1995 and 2000, most helps to explain why the fatality rate decreased in spite of the increase in average highway speeds?", "answers": "['The increase in average highway speeds occurred as legal speed limits were raised on one highway after another.', 'The average number of passengers per car on highways increased.', 'There were increases in both the proportion of people who wore seat belts and the proportion of cars that were equipped with airbags as safety devices.', 'In most locations on the highways, the density of vehicles on the highway did not decrease, although individual vehicles, on average, made their trips more quickly.']", "label": 2 }, { "id": "train_1239", "context": "In the past, most children who went sledding in the winter snow in Verland used wooden sleds with runners and steering bars. Ten years ago, smooth plastic sleds became popular; they go faster than wooden sleds but are harder to steer and slow. The concern that plastic sleds are more dangerous is clearly borne out by the fact that the number of children injured while sledding was much higher last winter than it was ten years ago.", "question": "Which of the following, if true in Verland, most seriously undermines the force of the evidence cited?", "answers": "['A few children still use traditional wooden sleds.', 'Plastic sleds can be used in a much wider variety of snow conditions than wooden sleds can.', 'Most sledding injuries occur when a sled collides with a tree, a rock, or, another sled.', 'Very few children wear any kind of protective gear, such as helmets, while sledding.']", "label": 1 }, { "id": "train_1240", "context": "Besides laying eggs in her own nest, any female wood duck will lay an egg in the nest of another female wood duck if she sees the other duck leaving her nest. Under natural nesting conditions, this parasitic behavior is relatively rare because the ducks' nests are well hidden. However, when people put up nesting boxes to help the ducks breed, they actually undercut the ducks' reproductive efforts. These nesting boxes become so crowded with extra eggs that few, if any, of the eggs in those boxes hatch.", "question": "The statements above, if true, most strongly support which one of the following?", "answers": "['The nesting boxes for wood ducks have less space for eggs than do natural nesting sites.', 'Female wood ducks will establish nests in nest boxes only when natural nesting sites are not available.', 'Nesting female wood ducks who often see other female wood ducks are the most successful in their breeding efforts.', 'The nesting boxes would be more effective in helping wood ducks breed if they were less visible to other wood ducks than they currently are.']", "label": 3 }, { "id": "train_1241", "context": "The habit many students on today' s campuses have of scribbling in their textbooks is inexcusable. It is harmful to books, aesthetically displeasing, and distracting to readers who buy the textbooks used.", "question": "Which one of the following, if true, most seriously weakens the argument?", "answers": "['Many students do not scribble in their textbooks.', 'In most areas, scribbling in textbooks is not illegal.', 'Many recent surveys show that books in public libraries are abused more than textbooks.', 'Most students who buy used textbooks say they are aesthetically indifferent to the scribbling and sometimes find it helpful.']", "label": 3 }, { "id": "train_1242", "context": "Leptin, a protein occurring naturally in the blood, appears to regulate how much fat the body carries by speeding up the metabolism and decreasing the appetite when the body has too much fat. Mice that do not naturally produce leptin have more fat than other mice, but lose fat rapidly when they are given leptin injections. Unfortunately, however, leptin cannot be used as a dietary supplement to control fat, since__.", "question": "Which of the following most logically completes the passage?", "answers": "['mice whose bodies did produce leptin also lost some of their body fat when given leptin injections', 'the mice that do not naturally produce leptin were from a specially bred strain of mice', 'the digestive system breaks down proteins before they can enter the bloodstream', 'people with unusually low levels of leptin in their blood tend to have a high percentage of body fat']", "label": 2 }, { "id": "train_1243", "context": "Principle: If you sell an item that you know to be defective, telling the buyer that the item is sound, you thereby commit fraud. Application: Wilton sold a used bicycle to Harris, knowing very little about its condition. Wilton told Harris that the bicycle was in good working condition, but Harris soon learned that the brakes were defective. Wilton was ttherefore guilty of fraud.", "question": "The application of the principle is most vulnerable to criticism on the grounds that", "answers": "['the application fails to establish whether Wilton was given the opportunity to repair the brakes', 'asserting something without justification is not the same as asserting something one knows to be false', \"Harris might not have believed Wilton's statement about the bicycle's condition\", 'the application uses the word \"defective\" in a sense that is crucially different from how it is used in the statement of the principle']", "label": 1 }, { "id": "train_1244", "context": " Two investigative journalists have raised the question whether the new blood pressure drug, Transerythropaxil, has sensitized patients to develop fish allergies. The study was published last month in a major newspaper, and has been picked up by media throughout the country. Lab data has long shown that Transerythropaxil reacts strongly with Eicosapentaenoic acid, found in abundance in fish oil, and this reaction can produce biological irritants. Despite strenuous objections from Transerythropaxil' s manufacturer, scientists working in federal labs have just published papers that provide conclusive evidence to the journalists' claim.", "question": "In the argument given, the two portions in boldface play which of the following roles?", "answers": "['The first provides support for conclusion of the argument; the second calls that conclusion into question.', 'The first states the conclusion of the argument; the second calls that conclusion into question.', 'Each provides support for the conclusion of the argument', 'The first identifies the content of the conclusion of the argument; the second provides support for that conclusion.']", "label": 3 }, { "id": "train_1245", "context": "X-ray examination of a recently discovered painting -- judged by some authorities to be a self-portrait by Vincent van Gogh -- revealed an underimage of a woman' s face. Either van Gogh or another painter covered the first painting with the portrait now seen on the surface of the canvas. Because the face of the woman in the underimage also appears on canvases van Gogh is known to have painted, the surface painting must be an authentic self-portrait by van Gogh.", "question": "The conclusion above is properly drawn if which of the following is assumed?", "answers": "['A painted canvas cannot be reliably attributed to an artist unless the authenticity of any underimage that painting might contain can be reliably attributed to the artist.', 'If a canvas already bears a painted image produced by an artist. a second artist who uses the canvas to produce a new painting tends to be influenced by the style of the first artist.', 'Many painted canvases that can be reliably attributed to van Gogh contain underimages of subjects that appear on at least one other canvas that van Gogh is known to have painted.', 'Any painted canvas incorrectly attributed to van Gogh would not contain an underimage of a subject that appears in authentic paintings by that artist.']", "label": 3 }, { "id": "train_1246", "context": "It is highly likely that Claudette is a classical pianist. Like most classical pianists, Claudette recognizes many of Clara Schumann' s works. The vast majority of people who are not classical pianists do not. In fact, many people who are not classical pianists have not even heard of Clara Schumann.", "question": "The reasoning in the argument above is flawed in that it", "answers": "['presumes, without providing justification, that people who have not heard of Clara Schumann do not recognize her works', 'presumes, without providing justification, that classical pianists cannot also play other musical instruments', 'relies for its plausibility on the vagueness of the term \"classical\"', \"ignores the possibility that the majority of people who recognize many of Clara Schumann's works are not classical pianists\"]", "label": 3 }, { "id": "train_1247", "context": "FilmPro sells millions of videocassettes directly to consumers at $25 apiece for a $10 profit on each. However, FilmPro is losing money because people are buying illegally copied versions of its $25 videocassettes at far cheaper prices. So far, one million illegally copied cassettes have been sold for $5 each. Illegal copying, ttherefore, has cost the company at least $10 million in potential profits.", "question": "Which of the following is an assumption that, if true, would allow the conclusion above to be properly drawn?", "answers": "['The illegally copied tapes are of such high quality that it is virtually impossible to differentiate between them and the originals.', 'FilmPro never sells discontinued videocassettes at less than $25 apiece.', 'At least one million more cassettes would have been purchased from FilmPro for $25 apiece if the illegally copied cassettes had not been available.', 'The price of the illegally copied videocassettes never goes below $5 apiece.']", "label": 2 }, { "id": "train_1248", "context": "Educator: Some experimental educational programs, based on the principle that children' s first education should take place at home, instruct parents in how to be their child' s \"first teacher. \" The school performance of the children in these programs is better than average. This shows that these programs are successful and should be expanded.", "question": "Which one of the following, if true, most weakens the educator's argument?", "answers": "['The cost of expanding the programs has not been precisely determined.', 'Some children who did not participate in the programs performed exceptionally well in school.', 'Not all small children enjoy being taught by their parents.', 'Most of the parents participating in the programs have prior experience as educators.']", "label": 3 }, { "id": "train_1249", "context": "This boulder is volcanic in origin and yet the rest of the rock in this area is sedimentary. Since this area was covered by southward-moving glaciers during the last ice age, this boulder was probably deposited here, hundreds of miles from its geological birthplace, by a glacier.", "question": "Which one of the following, if true, most seriously undermines the conclusion drawn in the argument above?", "answers": "['Most boulders that have been moved by glaciers have not been moved more than 100 miles.', 'The closest geological source of volcanic rock is 50 miles north of this boulder.', 'The closest geological source of volcanic rock is 50 miles south of this boulder.', 'There are no geological sources of volcanic rock north of this boulder.']", "label": 3 }, { "id": "train_1250", "context": "Therapist: In a recent study, researchers measured how quickly 60 different psychological problems waned as a large, diverse sample of people underwent weekly behavioral therapy sessions. About 75 percent of the 60 problems consistently cleared up within 50 weeks of therapy. This shows that 50 weekly behavioral therapy sessions are all that most people need.", "question": "The therapist's argument is logically most vulnerable to criticism on the grounds that it", "answers": "['takes for granted that no one suffers from more than one of the 60 psychological problems studied', 'takes for granted that there are no psychological problems that usually take significantly longer to clear up than the 60 psychological problems studied', 'fails to address the possibility that some forms of therapy have never been proven to be effective as treatments for psychological problems', 'fails to address the possibility that any given one of the 60 psychological problems studied might afflict most people']", "label": 3 }, { "id": "train_1251", "context": "The level of sulfur dioxide in the atmosphere is slightly higher than it was ten years ago. This increase is troubling because ten years ago the Interior Ministry imposed new, stricter regulations on emissions from coal-burning power plants. If these regulations had been followed, then the level of sulfur dioxide in the atmosphere would have decreased.", "question": "Which one of the following can be properly inferred from the statements above?", "answers": "['If current regulations on emissions from coal-burning power plants are not followed from now on, then the level of sulfur dioxide in the atmosphere will continue to increase.', 'There have been violations of the regulations on emissions from coal-burning power plants that were imposed ten years ago.', 'Government regulations will never reduce the level of sulfur dioxide in the atmosphere.', 'Emissions from coal-burning power plants are one of the main sources of air pollution.']", "label": 1 }, { "id": "train_1252", "context": "Philosopher: We should not disapprove of the unearthing of truths that we would rather not acknowledge or that, by their dissemination, might influence society in pernicious ways.", "question": "Which one of the following conforms most closely to the principle stated by the philosopher?", "answers": "['A poem need not adhere too strictly to the truth. Art is exempt from such requirements -- it matters only that the poem provoke a response in the reader.', \"A physician should never withhold the truth from a patient, except in cases where depression induced by bad news might significantly affect the patient's recuperation.\", 'A law enforcement officer should not act upon illegally obtained information, even though such action might, in some cases, result in a benefit to society.', 'Scientific research should not be restricted even if it could lead to harmful applications, such as the manufacture of sophisticated weapons.']", "label": 3 }, { "id": "train_1253", "context": "Sport utility vehicles (SUVs) are, because of their weight, extremely expensive to operate but, for the same reason, in an accident they are safer for their occupants than smaller vehicles are. Nonetheless, an analysis of recent traffic fatality statistics has led auto safety experts to conclude that the increasing popularity of SUVs is an alarming trend.", "question": "Which one of the following, if true, most helps to account for the response of auto safety experts to the popularity of SUVs?", "answers": "['Recent statistics suggest that SUVs are as likely to be involved in collisions as smaller vehicles are.', 'Vehicles with a reputation for being safer than others tend to be driven more carefully than other vehicles.', 'Recent statistics suggest that large vehicles such as SUVs tend to carry more passengers than smaller vehicles do.', 'Recent statistics suggest that the average number of fatalities in collisions between SUVs and smaller vehicles is higher than for other collisions.']", "label": 3 }, { "id": "train_1254", "context": "Environmentalist: The commissioner of the Fish and Game Authority would have the public believe that increases in the number of marine fish caught demonstrate that this resource is no longer endangered. This is a specious argument, as unsound as it would be to assert that the ever-increasing rate at which rain forests are being cut down demonstrates a lack of danger to that resource. The real cause of the increased fish-catch is a greater efficiency in using technologies that deplete resources.", "question": "The environmentalist's statements, if true, best support which of the following as a conclusion?", "answers": "['It is possible to determine how many fish are in the sea in some way other than by catching fish.', 'Marine fish continue to be an endangered resource.', 'Modern technologies waste resources by catching inedible fish.', 'The proportion of marine fish that are caught is as high as the proportion of rain forest trees that are cut down each year.']", "label": 1 }, { "id": "train_1255", "context": "Zoologist: In the Lake Champlain area, as the North American snowshoe hare population grows, so do the populations of its predators. As predator numbers increase, the hares seek food in more heavily forested areas, which contain less food, and so the hare population declines. Predator populations thus decline, the hare population starts to increase, and the cycle begins again. Yet these facts alone cannot explain why populations of snowshoe hares everywhere behave simultaneously in this cyclical way. Since the hare population cycle is well correlated with the regular cycle of sunspot activity, that activity is probably a causal factor as well.", "question": "Each of the following, if true, supports the zoologist's reasoning EXCEPT:", "answers": "['Reproduction in predator populations increases when sunspot activity indirectly affects hormonal processes associated with reproduction.', 'The variation from cycle to cycle in the magnitude of the highs and lows in snowshoe hare populations is highly correlated with variations from cycle to cycle in the intensity of highs and lows in sunspot activity.', 'Brighter light during sunspot activity subtly but significantly improves the ability of predators to detect and capture hares.', \"Local weather patterns that can affect species' population changes can occur both in the presence of sunspot activity and in its absence.\"]", "label": 3 }, { "id": "train_1256", "context": "Community activist: If Morganville wants to keep its central shopping district healthy, it should prevent the opening of a huge SaveAll discount department store on the outskirts of Morganville. Records from other small towns show that whenever SaveAll has opened a store outside the central shopping district of a small town, within five years the town has experienced the bankruptcies of more than a quarter of the stores in the shopping district.", "question": "The answer to which of the following would be most useful for evaluating the community activist's reasoning?", "answers": "['In towns with healthy central shopping districts, what proportion of the stores in those districts suffer bankruptcy during a typical five-year period?', 'Have community activists in other towns successfully campaigned against the opening of a SaveAll store on the outskirts of their towns?', 'Do newly opened SaveAll stores ever lose money during their first five years of operation?', 'Do a large percentage of the residents of Morganville currently do almost all of their shopping at stores in Morganville?']", "label": 0 }, { "id": "train_1257", "context": "It is clear that humans during the Upper Paleolithic period used lamps for light in caves. Though lamps can be dated to the entire Upper Paleolithic, the distribution of known lamps from the period is skewed, with the greatest number being associated with the late Upper Paleolithic period, when the Magdalenian culture was dominant.", "question": "Each of the following, if true, contributes to an explanation of the skewed distribution of lamps EXCEPT:", "answers": "['Artifacts from early in the Upper Paleolithic period are harder to identify than those that originated later in the period.', 'More efficient lamp-making techniques were developed by the Magdalenian culture than by earlier cultures.', 'More kinds of lamps were produced by the Magdalenian culture than by earlier cultures.', 'Fire pits were much more common in caves early in the Upper Paleolithic period than they were later in that period.']", "label": 2 }, { "id": "train_1258", "context": "Editorial: A proposed new law would limit elementary school class sizes to a maximum of 20 students. Most parents support this measure and argue that making classes smaller allows teachers to devote more time to each student, with the result that students become more engaged in the learning process. However, researchers who conducted a recent study conclude from their results that this reasoning is questionable. The researchers studied schools that had undergone recent reductions in class size, and found that despite an increase in the amount of time teachers spent individually with students, the students' average grades were unchanged.", "question": "Which one of the following is an assumption required by the researchers' argument?", "answers": "['The only schools appropriate for study are large elementary schools.', 'Teachers generally devote the same amount of individualized attention to each student in a class.', 'Parental support for the proposed law rests solely on expectations of increased student engagement in the learning process.', \"Degree of student engagement in the learning process correlates well with students' average grades.\"]", "label": 3 }, { "id": "train_1259", "context": "Carol: The best choice for investment is real estate because the payout is huge. Even though the market can be a little unpredictable, it' s the quickest way to turn a sizable profit. Kenny: Real estate is way too risky and people get huge sums of money tied up into one house that sometimes they can' t sell or the value depreciates precipitously. It' s much wiser to invest in dependable things like utilities or bonds.", "question": "Based on the text above, which of the following can be assumed about Carol and Kenny's investment strategies?", "answers": "['Kenny is willing to take risks only if the potential payout is huge.', 'Kenny would invest in stocks before he would invest in real estate.', 'Carol is very risk averse.', 'Carol is more willing to take risks than Kenny.']", "label": 3 }, { "id": "train_1260", "context": "Since empathy is essential for people to be willing to follow moral codes that sometimes require them to ignore their own welfare to help others, civilized society could not exist without empathy.", "question": "Which one of the following is an assumption required by the argument?", "answers": "['If everyone in a society is sometimes willing to ignore his or her own welfare to help others, that society will be civilized.', 'People who feel empathy tend to ignore their own welfare for the sake of others.', 'Civilized society can exist only if there are people who are willing to at least sometimes ignore their own welfare to help others.', 'Moral codes that include the requirement that people disregard their own welfare in order to help others have arisen within some civilized societies.']", "label": 2 }, { "id": "train_1261", "context": "The obsession of economists with consumption as a measure of economic well-being has prevented us from understanding the true nature of economic well-being. We get very little satisfaction out of the fact that our clothing wears out, our automobiles depreciate, and the gasoline in our tanks burns up and must be replaced.", "question": "The author is arguing that", "answers": "['modern products are designed for early obsolescence', 'economic well-being cannot be defined solely in terms of consumption', 'valid measures of consumption cannot be devised', 'satisfaction is possible without consumption']", "label": 1 }, { "id": "train_1262", "context": "Terry: Months ago, I submitted a claim for my stolen bicycle to my insurance company. After hearing nothing for several weeks, I contacted the firm and found they had no record of my claim. Since then, I have resubmitted the claim twice and called the firm repeatedly, but I have yet to receive a settlement. Anyone can make mistakes, of course, but the persistence of the error makes me conclude that the company is deliberately avoiding paying up.", "question": "Which one of the following principles is violated by Terry's reasoning?", "answers": "[\"In judging the quality of service of a corporation, a consumer should rely primarily on the consumer's own experience with the corporation.\", 'Consumers should attempt to keep themselves informed of corporate behavior that directly affects their interests.', \"In judging the morality of a corporation's behavior, as opposed to that of an individual, mitigating circumstances are irrelevant.\", 'Consumers should avoid attributing dishonesty to a corporation when the actions of the corporation might instead be explained by incompetence.']", "label": 3 }, { "id": "train_1263", "context": "PhishCo runs a number of farms in the arid province of Nufa, depending largely on irrigation. Now, as part of a plan to efficiently increase the farms' total production, it plans to drill down to an aquifer containing warm, slightly salty water that will be used to raise fish in ponds. The water from the ponds will later be used to supplement piped-in irrigation water for PhishCo's vegetable fields, and the ponds and accompanying vegetation should help reduce the heat in the area of the farms.", "question": "Which of the following would, if true, most strongly suggest that the plan, if implemented, would increase the overall efficiency of PhishCo's farms?", "answers": "['Organic waste from fish in the pond water will help to fertilize fields where it is used for irrigation.', \"Fish raised on PhishCo's farms are likely to be saleable in the nearest urban areas.\", 'Ponds will be located on low-lying land now partially occupied by grain crops.', 'The government of Nufa will help to arrange loan financing to partially cover the costs of drilling.']", "label": 0 }, { "id": "train_1264", "context": "Advertising tends to have a greater influence on consumer preferences regarding brands of yogurt than it does on consumer preferences regarding brands of milk. Yet, since the LargeCo supermarket chain began advertising its store-brand products, sales of its storebrand milk increased more than sales of its store-brand yogurt.", "question": "Which one of the following, if true, most helps to resolve the apparent discrepancy described above?", "answers": "[\"Shoppers at LargeCo tend to purchase the chain's own brand of dairy products more frequently than other brands of dairy products.\", 'Consumers tend to purchase store brands of yogurt, but purchase whichever brand of milk is least expensive.', 'Supermarkets throughout the entire nation have experienced a sharp decrease in sales of yogurt recently.', 'The typical shopper going to LargeCo for the purpose of buying milk does not go with the intention of also buying yogurt.']", "label": 2 }, { "id": "train_1265", "context": "Salesperson: If your vacuuming needs are limited to cleaning small areas of uncarpeted floors, an inexpensive handheld vacuum cleaner is likely to be sufficient. After all, most are easy to use and will likely satisfy all your vacuuming needs on wood and tile floors.", "question": "The conclusion of the salesperson's argument is most strongly supported if which one of the following is assumed?", "answers": "['If your household cleaning needs include cleaning small areas of uncarpeted floors, it is likely that you will need a vacuum cleaner.', 'Inexpensive handheld vacuum cleaners are sufficient for cleaning small areas of carpeted floors.', 'The only types of floor surfaces that most consumers encounter arc carpet, wood, and tile.', 'The more versatile a vacuum cleaner is, the more likely it is to be expensive.']", "label": 2 }, { "id": "train_1266", "context": "Pharmacist: A large study of people aged 65 -- 81 and suffering from insomnia showed that most of insomnia' s symptoms are substantially alleviated by ingesting melatonin, a hormone produced by the pineal gland, which plays a role in the regulation of the body' s biological clock. Thus, the recent claims made by manufacturers of melatonin supplements that the pineal gland produces less melatonin as it ages are evidently correct.", "question": "The pharmacist's argument is flawed in that it", "answers": "['infers from the effect of an action that the action is intended to produce that effect', 'depends on using two different meanings for the same term to draw its conclusion', 'confuses an effect of a phenomenon with its cause', 'relies on a sample that is unrepresentative']", "label": 3 }, { "id": "train_1267", "context": "Aaron: A prominent judge, criticizing \"famous lawyers who come before courts ill-prepared to argue their cases, \" recently said, \"This sort of cavalier attitude offends the court and can do nothing but harm to the client' s cause. \" I find the judge' s remarks irresponsible. Belinda: I find it natural and an admirable display of candor. Letting people know of the damage their negligence causes is responsible behavior.", "question": "The point at issue between Aaron and Belinda is whether", "answers": "['it is to be expected that ill-prepared lawyers would offend the court in which they appear', 'famous lawyers have a greater responsibility to be well prepared than do lawyers who are not famous', \"the judge's criticism of lawyers is irresponsible\", \"ill-prepared lawyers damage their clients' causes\"]", "label": 2 }, { "id": "train_1268", "context": "Recent trends in the clothing industry have suggested that more clothing is being produced more quickly and cheaply than in the past; styles can go from the runway to stores in as little as three weeks. While most experts have focused on minimizing the time this process takes, True Clothing has in fact made efforts to take longer to produce its clothing.", "question": "Which of the following would, if true, most help to explain the strategy of True Clothing referred to above?", "answers": "['Some items of clothing can be worn in different ways to create different styles, depending on the individual consumers.', 'The sales of well-made clothing, which takes longer to produce, are less likely to fluctuate in different market conditions.', 'Companies with large market shares in the clothing industry tend to be less concerned with industrial trends.', 'Clothing once produced only for adult consumers is increasingly being produced for teenagers.']", "label": 1 }, { "id": "train_1269", "context": "Journalist: Well-known businessman Arnold Bergeron has long been popular in the state, and he has often talked about running for governor, but he has never run. However, we have just learned that Bergeron has fulfilled the financial disclosure requirement for candidacy by submitting a detailed list of his current financial holdings to the election commission. So, it is very likely that Bergeron will be a candidate for governor this year.", "question": "The answer to which of the following questions would be most useful in evaluating the journalist's argument?", "answers": "[\"Is submitting a list of holdings the only way to fulfill the election commission's financial disclosure requirements?\", 'Did the information recently obtained by the journalists come directly from the election commission?', 'Had Bergeron also fulfilled the financial disclosure requirements for candidacy before any previous gubernatorial elections?', \"Have Bergeron's financial holdings increased in value in recent years?\"]", "label": 2 }, { "id": "train_1270", "context": "The Yorgorian people, many of whom still live in the jungle in traditional conditions, eat many meats of jungle mammals, resulting in a cuisine that is high in the protein RX7G, much higher than the cuisine of the industrial nation of Transpraxia. Medical research has shown a high correlation between the protein RX7G and coronary heart disease, and yet, the Yorgorian people have a much lower incidence of such disease than found in Transpraxia. Medical researchers have dubbed this the Yorgorian Paradox.", "question": "Which of the following, if true, helps to explain the Yorgorian Paradox?", "answers": "['Citizens of Transpraxia, per capita, consume almost the same amount of the protein RX7G as do the Yorgorian people.', 'The protein RX7G is in higher concentration in the meat of some jungle mammals than in the meat of some domesticated farm mammals.', 'Some upscale restaurants in Transpraxia serve the meat of jungle mammals, some of which are imported from the jungles the Yorgorian people occupy.', 'The diet of the Yorgorian people is very high in the berries, and the nutrients of these berries neutralize the most harmful effects of protein RX7G.']", "label": 3 }, { "id": "train_1271", "context": "Last winter was mild enough to allow most bird species to forage naturally, which explains why the proportion of birds visiting feeders was much lower than usual. The mild winter also allowed many species to stay in their summer range all winter without migrating south, thereby limiting the usual attrition accompanying migration. Hence, last year' s mild winter is responsible for this year' s larger-than-usual bird population.", "question": "Which one of the following, if true, would most strengthen the reasoning in the argument?", "answers": "['Birds sometimes visit feeders even when they are able to find sufficient food for survival by foraging naturally.', 'Increases in bird populations sometimes occur following unusual weather patterns.', \"Birds that remain in their summer range all winter often exhaust that range's food supply before spring.\", 'Birds eating at feeders are more vulnerable to predators than are birds foraging naturally.']", "label": 3 }, { "id": "train_1272", "context": "Hollywood restaurant is replacing some of its standard tables with tall tables and stools. The restaurant already fills every available seat during its operating hours, and the change in seating arrangements will not result in an increase in the restaurant' s seating capacity. Nonetheless, the restaurant' s management expects revenue to increase as a result of the seating change without any concurrent change in menu, prices or operating hours.", "question": "Which of the following, if true, provides the best reason for the expectation?", "answers": "['The standard tables being replaced by tall tables would otherwise have to be replaced with new standard tables at a greater expense.', 'Few diners are likely to avoid the restaurant because of the new seating arrangement.', 'Diners seated on stools typically do not linger over dinner as long as diners seated at standard tables.', 'One of the taller tables takes up less floor space than one of the standard tables.']", "label": 2 }, { "id": "train_1273", "context": "The Ixazyiquo tribe are believed to be one of the few remaining tribes in the country of Malugiato to have never had any contact with the Lamandians, settlers who arrived in Malugiain during the last hundred years. Using satellite imagery, a team of anthropologists determined that the Ixazyiquo land is covered with thriving banana plantations. The anthropologists conclude that the Ixazyiquo tribe has in fact had direct contact with the Lamandians.", "question": "Which of the following, if true, would best support the anthropologists' conclusion?", "answers": "['There is no standard shape to the Ixazyiquobanana plantations, which have taken on a form that best fits the soil conditions.', 'The Ixazyiquo are a hunter-gather society, which travel over vast areas of land to hunt.', 'Maintaining banana plantations requires sophisticated knowledge that only the Lamadians possess.', 'The Ixazyiquo have had extensive contact with the Wamalami, who have had direct contact with the Lamandians.']", "label": 2 }, { "id": "train_1274", "context": "Unlike other primroses, self-pollinating primroses do not need to rely on insects for pollination. In many years insect pollinators are scarce, and in those years a typical non-self-pollinating primrose produces fewer seeds than does a typical self-pollinating primrose. In other years, seed production is approximately equal. Thus, self-pollinating primroses have the advantage of higher average seed production. Aside from seed production, these self-pollinating primroses are indistinguishable from non-self-pollinating primroses. Nevertheless, self-pollinating primrose plants remain rare among primroses.", "question": "Which one of the following, if true, most helps to resolve the apparent discrepancy in the information above?", "answers": "['Self-pollinating primroses that are located in areas with few insects produce no fewer seeds than do self-pollinating primroses that are located in areas with many insects.', 'When insect pollinators are scarce, non-self- pollinating primroses produce larger seeds that are more likely to germinate than are seeds from self-pollinating primroses.', 'Self-pollinating primroses can be assisted by insects during pollination but do not require the assistance of insects to be pollinated.', 'Many primroses are located in areas in which the soil conditions that are optimal for seed germination are not present.']", "label": 1 }, { "id": "train_1275", "context": "There are circumstances in which it is not immoral to make certain threats, and there are circumstances in which it is not immoral to ask for money or some other favor. Ttherefore, there are circumstances in which it is not immoral to ask for money or a favor while making a threat.", "question": "Which one of the following exhibits a flawed pattern of reasoning most similar to that in the argument above?", "answers": "['The mountain roads are treacherous after it rains, and the mountain streams are full after a rain. So, if the roads in the mountains are treacherous, and the mountain streams are full, it surely has rained recently.', 'Heavy trucks are generally operated in a safe manner, but the ability to drive a truck safely can be impaired by certain prescription drugs. Ttherefore, heavy trucks cannot be operated safely while the driver is under the effect of a prescription drug.', 'It is healthful to take drug A for a headache, and it is healthful to take drug B for a headache; ttherefore, it is healthful to take drug A together with drug B for a headache.', 'It is usually easy to move a piano after you have convinced five people to help you, provided that you do not need to take it up or down stairs. Ttherefore, it is usually easy to move a piano.']", "label": 2 }, { "id": "train_1276", "context": "Leatherbacks, the largest of the sea turtles, when subjected to the conditions of captivity, are susceptible to a wide variety of fatal diseases with which they would never come in contact if they lived in the wild. It is surprising, ttherefore, that the likelihood that a leatherback will reach its theoretical maximum life expectancy is about the same whether that animal is living in captivity or in the wild.", "question": "Which one of the following, if true, most helps to resolve the apparent discrepancy?", "answers": "['The size of a leatherback is an untrustworthy indicator of its age.', 'Most leatherbacks that perish in the wild are killed by predators.', 'Fewer diseases attach leatherbacks than attack other large aquatic reptiles.', 'The average life expectancy of sea turtles in general is longer than that of almost all other marine animals.']", "label": 1 }, { "id": "train_1277", "context": "Rolanda: The house on Oak Avenue has a larger yard than any other house we' ve looked at in Prairieview, so that' s the best one to rent. Tom: No, it isn' t. Its yard isn' t really as big as it looks. Property lines in Prairieview actually start 20 feet from the street. So what looks like part of the yard is really city property. Rolanda: But that' s true of all the other properties we' ve looked at too!", "question": "Rolanda's response to Tom suggests that Tom commits which one of the following reasoning errors?", "answers": "['He presumes, without providing justification, that property that belongs to the city is available for private use.', 'He improperly applies a generalization to an instance that it was not intended to cover.', 'He presumes, without providing justification, that whatever is true of a part of a thing is also true of the whole.', 'He fails to apply a general rule to all relevant instances.']", "label": 3 }, { "id": "train_1278", "context": "Some planning committee members -- those representing the construction industry -- have significant financial interests in the committee' s decisions. No one who is on the planning committee lives in the suburbs, although many of them work there.", "question": "If the statements above are true, which one of the following must also be true?", "answers": [ "No person who has significant financial interest in the planning committee's decisions lives in the suburbs.", "No persons with significant financial interests in the planning committee's decisions are not in the construction industry.", "Some persons with significant financial interests in the planning committee's decisions do not live in the suburbs.", "Some persons with significant financial interests in the planning committee's decisions work in the suburbs." ], "label": 2 }, { "id": "train_1279", "context": "Patel: It is often thought that direct experience, unlike language and culture, can always serve as common ground for communication among human beings. But people from different cultures inhabit different sensory worlds. Because selective screening of sensory data admits some perceptions while filtering out others, one person' s experience, perceived through one set of culturally patterned sensory screens, is quite different from what anyone from another culture would experience when encountering the same sensory data.", "question": "Which one of the following is an assumption required by Patel's argument?", "answers": "['The experience of one person is quite different from the experience of any other person.', 'No two people from different cultures ever encounter the same sensory data.', 'Two people from the same culture sometimes have different sensory screens.', 'No two people from different cultures have the same sensory screens.']", "label": 3 }, { "id": "train_1280", "context": "A certain medication that is frequently prescribed to lower a patient' s cholesterol level is generally effective. A recent study of 1, 000 subjects ranging widely in age indicates, however, that the cholesterol level of someone taking the medication is typically 12 to 15 percent higher than the average for that person' s age group.", "question": "Which one of the following, if true, most helps to explain how both of the claims made above could be true?", "answers": "['The medication described above is usually prescribed only for people whose cholesterol level is at least 30 percent above the average for their age group.', 'Another medication is prescribed to treat high cholesterol just as often as the medication described above is.', 'Within the population as a whole, approximately the same number of people have relatively high cholesterol levels as have relatively low cholesterol levels.', 'A recently developed cholesterol-lowering medication is more effective than the medication described above.']", "label": 0 }, { "id": "train_1281", "context": "Ms. Smith: I am upset that my son' s entire class lost two days of recess because some of the children were throwing raisins in the cafeteria. He was not throwing raisins, and it was clear to everyone just who the culprits were. Principal: I' m sorry you' re upset, Ms. Smith, but your son' s situation is like being caught in a traffic jam caused by an accident. People who aren' t involved in the accident nevertheless have to suffer by sitting there in the middle of it.", "question": "The principal's response to Ms. Smith's complaint is most vulnerable to criticism on which one of the following grounds?", "answers": "['It does not acknowledge the fact that a traffic jam following an accident is unavoidable while the mass punishment was avoidable.', 'It makes a generalization about all the children in the class which is not justified by the facts.', \"It assumes that Ms. Smith's son is guilty when there is evidence to the contrary which the principal has disregarded.\", 'It suggests that throwing raisins in the cafeteria produces as much inconvenience as does being caught in a traffic jam.']", "label": 0 }, { "id": "train_1282", "context": "Statistical analysis is a common tool for explanation in the physical sciences. It can only be used, however, to explain events that can be replicated to the last detail. Since human mental events never precisely recur, statistical analysis cannot be employed to explain these events. Ttherefore, they cannot be explained by the physical sciences.", "question": "Which one of the following arguments is most similar in its flawed reasoning to the argument above?", "answers": "['Computer modeling is used to try to explain the way in which wind resistance affects the movement of bicycles. To use computer modeling, the phenomenon being modeled must be predictable. But wind resistance is not predictable. Ttherefore, the way in which wind resistance affects the movement of bicycles cannot be explained by computer modeling.', 'A good way to explain historical events is to construct a coherent narrative about those events. In order to construct such a narrative, a great many details about the events must be known. Virtually no details can be known of certain very ancient historical events. Ttherefore, no historical explanation can be given for these events.', 'The best way to explain why an object has a particular color is in terms of the interaction of light and matter. It is sometimes impossible to find out what kind of matter constitutes an object. Ttherefore, the color of such objects has nothing to do with the interaction of light and matter.', 'To determine which explanation of the origin of the universe is correct, we would need to know about the first moments of the existence of the universe. Due to the immense time that has passed since the universe began, it is impossible to get such information. Ttherefore, none of the explanations of the origin of the universe is likely to be correct.']", "label": 1 }, { "id": "train_1283", "context": "Environmentalist: The complex ecosystem of the North American prairie has largely been destroyed to produce cattle feed. But the prairie ecosystem once supported 30 to 70 million bison, whereas North American agriculture now supports about 50 million cattle. Since bison yield as much meat as cattle, and the natural prairie required neither pesticides, machinery, nor government subsidies, returning as much land as possible to an uncultivated state could restore biodiversity without a major decrease in meat production.", "question": "Which one of the following most accurately expresses the environmentalist's main conclusion?", "answers": "['If earlier North American agricultural techniques were reintroduced, meat production would decrease only slightly.', 'Protecting the habitat of wild animals so that we can utilize these animals as a food source is more cost effective than raising domesticated animals.', 'The devastation of the North American prairie ecosystem could be largely reversed without significantly decreasing meat production.', 'Preservation of the remaining North American bison would be a sensible policy.']", "label": 2 }, { "id": "train_1284", "context": "Commentator: Many social critics claim that contemporary journalists' cynical tendency to look for selfish motives behind the seemingly altruistic actions of powerful people undermines our society' s well-being by convincing people that success is invariably associated with greed and mendacity. But the critics' claim is absurd. The cynicism of contemporary journalists cannot be a contributing factor to the undermining of our society' s well-being, for journalists have always been cynics. Today' s journalists are, if anything, more restrained than their predecessors.", "question": "The reasoning in the commentator's argument is most vulnerable to criticism on the grounds that it overlooks the possibility that", "answers": "['the work of contemporary journalists reflects a cynicism that is not really genuine', 'widespread cynicism is beneficial to the well-being of society', 'cynicism of this type on the part of journalists has always had a negative effect on the well-being of society', 'cynicism about the motives of powerful people increases with the amount of information one has about them']", "label": 2 }, { "id": "train_1285", "context": "Birds need so much food energy to maintain their body temperatures that some of them spend most of their time eating. But a comparison of a bird of a seed-eating species to a bird of a nectar-eating species that has the same overall energy requirement would surely show that the seed-eating bird spends more time eating than does the nectar-eating bird, since a given amount of nectar provides more energy than does the same amount of seeds.", "question": "The argument relies on which one of the following questionable assumptions?", "answers": "['The time it takes for the nectar-eating bird to eat a given amount of nectar is not longer than the time it takes the seed-eating bird to eat the same amount of seeds.', 'Birds of different species do not generally have the same overall energy requirements as each other.', 'The nectar-eating bird does not sometimes also eat seeds.', 'The overall energy requirements of a given bird do not depend on factors such as the size of the bird, its nest-building habits, and the climate of the region in which it lives.']", "label": 0 }, { "id": "train_1286", "context": "In Berinia, the age at which people could begin to drink alcohol legally used to be 18. In 1990, in an attempt to reduce alcohol consumption and thereby to reduce alcohol-related traffic deaths among Berinians under 21, the legal drinking age was raised to 21. Alcohol-related traffic deaths among people under 21 have decreased significantly since 1990. Nevertheless, surveys show that people in that age-group drink just as much alcohol as they did before 1990.", "question": "Which of the following, if true of Berinia, most helps to resolve the apparent discrepancy?", "answers": "['Alcohol-related traffic deaths among people over 21 have increased slightly since 1990.', 'For the population as a whole, annual alcohol consumption is no lower now than it was in 1990.', 'The proportion of people under 21 who own a car is higher now than it was in 1990.', 'Alcohol consumption away from home, for example in bars and restaurants, is much lower among people under 21 than it was in 1990.']", "label": 3 }, { "id": "train_1287", "context": "The proposal to hire ten new police officers in Middletown is quite foolish. There is sufficient funding to pay the salaries of the new officers, but not the salaries of additional court and prison employees to process the increased caseload of arrests and convictions that new officers usually generate.", "question": "> Which of the following, if true, will most seriously weaken the conclusion drawn above?", "answers": "[\"If funding for the new police officers' salaries is approved, support for other city services will have to be reduced during the next fiscal year\", \"Middletown's ratio of police officers to citizens has reached a level at which an increase in the number of officers will have a deterrent effect on crime\", 'In most US cities, not all arrests result in convictions, and not all convictions result in prison terms', 'When one major city increased its police force by 19% last year, there were 40% more arrests and 13% more convictions']", "label": 1 }, { "id": "train_1288", "context": "A person is more likely to become disabled as that person ages. Among adults in the country of East Wendell, however, the proportion receiving disability benefit payments shrinks from 4 percent among 55 to 64 year olds to 2 percent for those aged 65 to 74 and 1 percent for those aged 75 and older. The explanation of this discrepancy is that the proportion of jobs offering such a disability benefit has greatly increased in recent years.", "question": "Which one of the following, if true about East Wendell, shows that the explanation above is at best incomplete?", "answers": "['The treatment of newly incurred disabilities is more successful now than in the past in restoring partial function in the affected area within six months.', \"Under most employers' plans, disability benefit payments stop when an employee with a disability reaches the usual retirement age of 65.\", 'Medical advances have prolonged the average lifespan beyond what it was 20 years ago.', 'For persons receiving disability benefit payments, those payments on average represent a smaller share of their predisability income now than was the case 20 years ago.']", "label": 1 }, { "id": "train_1289", "context": "Several companies that make herbal teas containing ginseng assert in their marketing that ginseng counteracts the effects of stress. As a result, many people buy these products hoping to improve their health. Yet no definitive scientific study links ginseng with the relief of stress. Thus, these marketing campaigns make false claims.", "question": "The reasoning in the argument is flawed in that the argument", "answers": "['concludes that a claim is false merely on the grounds that it has not been shown to be true', 'fails to address the possibility that some ingredients other than ginseng in the herbal teas containing ginseng counteract the effects of stress', \"rejects an argument because of its source without evaluating the argument's logical strength\", 'fails to address the possibility that many people buy herbal teas containing ginseng because they enjoy drinking the tea']", "label": 0 }, { "id": "train_1290", "context": "Most people invest in the stock market without doing any research of their own. Some of these people rely solely on their broker' s advice, whereas some others make decisions based merely on hunches. Other people do some research of their own, but just as often rely only on their broker or on hunches. Only a few always do their own research before investing. Nonetheless, a majority of investors in the stock market make a profit.", "question": "If the statements in the passage are true, which one of the following must also be true?", "answers": "['Most people who rely solely on their broker rather than on hunches make a profit in the stock market.', 'Most people who invest in the stock market without doing any research of their own make a profit.', 'Some people who do investment research on their own, while just as often relying on their broker or on hunches, make a profit in the stock market.', 'Some people who make a profit on their investments in the stock market do so without doing any research of their own.']", "label": 3 }, { "id": "train_1291", "context": "The introduction of the new drugs into the market is frequently prevented by a shortage of human subjects for the clinical trials needed to show that the drugs are safe and effective. Since the lives and health of people in future generations may depend on treatments that are currently experimental, practicing physicians are morally in the wrong when, in the absence of any treatment proven to be effective, they fail to encourage suitable patients to volunteer for clinical trials.", "question": "Which of the following, if true, casts most doubt on the conclusion of the argument?", "answers": "['Physicians have an overriding moral and legal duty to care for the health and safety of their current patients.', 'Many drugs undergoing clinical trials are intended for the treatment of conditions for which there is currently no effective treatment.', 'An experimental drug cannot legally be made available to patients unless those patients are subjects in clinical trials of the drug.', \"Patients do not share the physician's professional concern for public health, but everyone has a moral obligation to alleviate suffering when able to do so.\"]", "label": 0 }, { "id": "train_1292", "context": "Quasars -- celestial objects so far away that their light takes at least 500 million years to reach Earth -- have been seen since 1963. For anything that far away to appear from Earth the way quasars do, it would have to burn steadily at a rate that produces more light than 90 billion suns would produce. But nothing that burns at a rate that produces that much light could exist for more than about 100 million years.", "question": "If the statements above are true, which one of the following must also be true on the basis of them?", "answers": "['Instruments in use before 1963 were not sensitive enough to permit quasars to be seen.', 'Light from quasars first began reaching Earth in 1963.', 'Nothing that is as far from Earth as quasars are can continue to exist for more than about 100 million years.', 'No quasar that has ever been seen from Earth exists any longer.']", "label": 3 }, { "id": "train_1293", "context": "From an analysis of broken pottery and statuary, archaeologists have estimated that an ancient settlement in southwestern Arabia was established around 1000 B. C. However, new evidence suggests that the settlement is considerably older: tests show that a piece of building timber recently uncovered at the site is substantially older than the pottery and statuary.", "question": "Which one of the following, if true, most seriously undermines the conclusion drawn from the new evidence?", "answers": "['The building timber bore marks suggesting that it had been salvaged from an earlier settlement.', 'The pieces of pottery and fragments of statues that were analyzed come from several parts of the site.', 'The type of pottery found at the site is similar to a type of pottery associated with civilizations that existed before 1000 B. C.', 'The tests used to determine the age of the pottery and statuary had been devised more recently than those used to determine the age of the building timber.']", "label": 0 }, { "id": "train_1294", "context": "Theorist: To be capable of planned locomotion, an organism must be able both to form an internal representation of its environment and to send messages to its muscles to control movements. Such an organism must ttherefore have a central nervous system. Thus, an organism incapable of planned locomotion does not have a central nervous system.", "question": "The theorist's argument is flawed in that it", "answers": "['takes for granted that adaptations that serve a biologically useful purpose originally came about for that purpose', 'presumes, without providing justification, that an internal representation of its environment can be formed by an organism with even a rudimentary nervous system', \"presumes, without providing justification, that planned locomotion is the only biologically useful purpose for an organism's forming an internal representation of its environment\", \"confuses a necessary condition for an organism's possessing a capacity with a sufficient one\"]", "label": 3 }, { "id": "train_1295", "context": "Alia: Hawthorne admits that he has influence with high government officials. He further admits that he sold that influence to an environmental interest group. There can be no justification for this kind of unethical behavior. Martha: I disagree that he was unethical. The group that retained Hawthorne' s services is dedicated to the cause of preventing water pollution. So, in using his influence to benefit this group, Hawthorne also benefited the public.", "question": "Alia and Martha disagree on whether", "answers": "['the meaning of ethical behavior is the same in a public situation as in a private one', \"the consequences of Hawthorne's behavior can ethically justify that behavior\", 'the standards for judging ethical behavior can be imposed on Hawthorne by another', 'the meaning of ethical behavior has changed over time']", "label": 1 }, { "id": "train_1296", "context": "Shanna: Owners of any work of art, simply by virtue of ownership, ethically have the right to destroy that artwork if they find it morally or aesthetically distasteful, or if caring for it becomes inconvenient. Jorge: Ownership of unique artworks, unlike ownership of other kinds of objects, carries the moral right to possess but not to destroy. A unique work of art with aesthetic or historical value belongs to posterity and so must be preserved, whatever the personal wishes of its legal owner.", "question": "Which one of the following principles, if accepted, would contribute most to Shanna's defense of her position against that of Jorge?", "answers": "['There are certain entities over which no one would be ethically justified in claiming absolute rights to ownership.', \"The autonomy of individuals to do what they wish with what is theirs must not be compromised in the absence of a threat to anyone's health or safety.\", 'Truly great works of art are never morally or aesthetically distasteful to any serious student of the history of art.', 'It would be imprudent to allow the present stock of artworks to be destroyed without some guarantee that the artists of the future will produce works as great as those produced in the past.']", "label": 1 }, { "id": "train_1297", "context": "Science columnist: It is clear why humans have so many diseases in common with cats. Many human diseases are genetically based, and cats are genetically closer to humans than are any other mammals except nonhuman primates. Each of the genes identified so far in cats has an exact counterpart in humans.", "question": "Which one of the following, if true, most weakens the science columnist's explanation for the claim that humans have so many diseases in common with cats?", "answers": "['Cats have built up resistance to many of the diseases they have in common with humans.', 'Humans have more genes in common with nonhuman primates than with cats.', 'Cats have more diseases in common with nonhuman primates than with humans.', 'Most diseases that humans have in common with cats have no genetic basis.']", "label": 3 }, { "id": "train_1298", "context": "Numerous paintings and engravings representing warfare can be found in remains of all civilizations going back to and including the Neolithic period, when agriculture was first developed. However, no paintings or engravings of warfare are found dating from before the Neolithic period. Ttherefore, warfare must have first developed as a result of the transition to an agricultural society.", "question": "Which one of the following is an assumption required by the argument?", "answers": "['Warfare in the Neolithic period was always motivated by territorial disputes over agricultural land.', 'Paintings and engravings of agricultural life began to be made at the same time as paintings and engravings of warfare.', 'Warfare is the inevitable result of the development of a civilization.', 'There was no warfare prior to the period in which paintings and engravings of warfare were first created.']", "label": 3 }, { "id": "train_1299", "context": "A scientist made three observations: (1) in the world' s temperate zones, food is more plentiful in the ocean than it is in fresh water; (2) migratory fish in temperate zones generally mature in the ocean and spawn in fresh water; and (3) migratory fish need much nourishment as they mature but little or none during the part of their lives when they spawn. On the basis of those observations, the scientist formulated the hypothesis that food availability is a determining factor in the migration of migratory fish. Subsequently the scientist learned that in the tropics migratory fish generally mature in fresh water and spawn in the ocean.", "question": "Which one of the following would it be most helpful to know in order to judge whether what the scientist subsequently learned calls into question the hypothesis?", "answers": "['whether in the tropics food is less plentiful in the ocean than in fresh water', 'whether any species of fish with populations in temperate zones also have populations that live in the tropics', \"whether in the world's temperate zones, the temperatures of bodies of fresh water tend to be lower than those of the regions of the oceans into which they flow\", 'whether there are more species of migratory fish in the tropics than there are in temperate zones']", "label": 0 }, { "id": "train_1300", "context": "Doctor: Recent pharmaceutical advances will lead the way in weight loss. Prior to these advancements, obesity-related deaths outnumbered all other causes of death by a wide margin. The new drugs will curb appetite and increase metabolism. Thanks to these advancements, obesity will dramatically decline in the near future.", "question": "Each of the following, if true, strengthens the doctor's argument EXCEPT:", "answers": "['Participants in several studies reported significant appetite reductions.', 'Nutritious food is not readily available to many people suffering from obesity.', 'Increasing metabolism would significantly reduce obesity across the country.', 'Most people will not be able to afford these prescriptions since the majority of health care plans will not cover the new drugs.']", "label": 3 }, { "id": "train_1301", "context": "Many successful graphic designers began their careers after years of formal training, although a significant number learned their trade more informally on the job. But no designer ever became successful who ignored the wishes of a client.", "question": "If all of the statements above are true, which one of the following must also be true?", "answers": "['All graphic designers who are unsuccessful have ignored the wishes of a client.', \"Not all formally trained graphic designers ignore clients' wishes.\", \"No graphic designers who learn their trade on the job will ignore clients' wishes.\", \"The more attentive a graphic designer is to a client's wishes, the more likely the designer is to be successful.\"]", "label": 1 }, { "id": "train_1302", "context": "Some airlines allegedly reduce fares on certain routes to a level at which they lose money, in order to drive competitors off those routes. However, this method of eliminating competition cannot be profitable in the long run. Once an airline successfully implements this method, any attempt to recoup the earlier losses by charging high fares on that route for an extended period would only provide competitors with a better opportunity to undercut the airline' s fares.", "question": "Which of the following, if true, most seriously weakens the argument?", "answers": "['Airline executives generally believe that a company that once underpriced its fares to drive away competitors is very likely to do so again if new competitors emerge.', 'On deciding to stop serving particular routes, most airlines shift resources to other routes rather than reduce the size of their operations.', 'In some countries it is not illegal for a company to drive away competitors by selling a product below cost.', 'When airlines dramatically reduce their fares on a particular route, the total number of air passengers on that route increases greatly.']", "label": 0 }, { "id": "train_1303", "context": "Codex Berinensis, a Florentine copy of an ancient Roman medical treatise, is undated but contains clues to when it was produced. Its first 80 pages are by a single copyist, but the remaining 20 pages are by three different copyists, which indicates some significant disruption. Since a letter in handwriting identified as that of the fourth copyist mentions a plague that killed many people in Florence in 1148, Codex Berinensis was probably produced in that year.", "question": "Which of the following, if true, most strongly supports the hypothesis that Codex Berinensis was produced in 1148?", "answers": "['According to the account by the fourth copyist, the plague went on for 10 months.', 'A scribe would be able to copy a page of text the size and style of Codex Berinensis in a day.', 'There was only one outbreak of plague in Florence in the 1100s.', 'Other than Codex Berinensis, there are no known samples of the handwriting of the first three copyists.']", "label": 2 }, { "id": "train_1304", "context": "Company president: Grievance procedures should allow the grievant and the respondent to select a mediator who will attempt to work out a resolution. Grievances are costly and mediation could help to resolve many of them. However, beginning mediation fairly late in the process, as our human resources department proposes, would be relatively ineffective.", "question": "Which one of the following, if true, most helps to justify the company president's criticism of the human resources department's proposal?", "answers": "[\"Adversaries' positions tend to harden as a dispute wears on, making compromise less likely.\", 'The mediation process itself is likely to cost as much in time and money as the present grievance procedures.', 'Many disagreements are already being solved without the intervention of a mediator.', 'Respondents tend to be supervisors who cannot give in to employees without losing authority.']", "label": 0 }, { "id": "train_1305", "context": "Business Consultant: Some corporations shun the use of executive titles because they fear that the use of titles indicating position in the corporation tends to inhibit communication up and down the corporate hierarchy. Since an executive who uses a title is treated with more respect by outsiders, however, use of a title can facilitate an executive' s dealings with external businesses . Clearly, corporations should adopt the compromise of encouraging their executives to use their corporate titles externally but not internally, since even if it is widely known that the corporation' s executives use titles outside their organization, this knowledge does not by itself inhibit communication within the corporation.", "question": "In the consultant's reasoning, the two portions in boldface play which of the following roles?", "answers": "['The first describes a policy for which the consultant seeks to provide a justification; the second is a consideration the consultant raises as part of that justification.', 'The first describes a strategy that has been adopted to avoid a certain problem; the second presents a drawback to that strategy.', 'The first is part of an explanation that the consultant offers for a certain phenomenon; the second is that phenomenon.', 'The first describes a strategy that has been adopted to avoid a certain problem; the second is a consideration the consultant raises in questioning the significance of that problem.']", "label": 1 }, { "id": "train_1306", "context": "Consumer advocate: There is ample evidence that the model of car one drives greatly affects the chances that one' s car will be stolen. The model of car stolen most often in our country last year, for example, was also the model stolen most often in the preceding year.", "question": "The consumer advocate's reasoning is most vulnerable to criticism on the grounds that it", "answers": "['presumes, without providing justification, that someone considering whether or not to steal a particular car considers only what model the car is', 'fails to address adequately the possibility that the car model that was stolen most often last year was stolen as often as it was because it has a very high resale value', \"fails to address adequately the possibility that the model of car that was stolen most often last year was the most common model of car in the consumer advocate's country\", 'fails to address adequately the possibility that the age of a car also greatly affects its chances of being stolen']", "label": 2 }, { "id": "train_1307", "context": "Until about 400 million years ago, fishes -- the first true swimmers -- were jawless. Their feeding methods were limited to either sucking in surface plankton or sucking in food particles from bottom mud. With the development of biting jaws, however, the life of fishes changed dramatically, since jaws allowed them actively to pursue prey, to seize it in their jaws, and to manipulate it between their teeth. The jawed fishes then developed along two main lines: one retained cartilage for its skeletons, for example, sharks and rays; the other adopted bone as its principal skeletal material. From the latter group evolved the most abundant and diverse of all of today' s vertebrate groups, the \"teleosts, \"some 21, 000 species, which vary from barracudas to sea horses.", "question": "If all of the statements in the passage are true, which one of the following must also be true?", "answers": "['The jawless fishes did not prey upon other fish.', 'Fish are the primary prey of all jawed fishes.', 'Jawless fishes did not have cartilage as their skeletal material.', 'Teleosts do not feed upon particles found in bottom mud.']", "label": 0 }, { "id": "train_1308", "context": "In any field, experience is required for a proficient person to become an expert. Through experience, a proficient person gradually develops a repertory of model situations that allows an immediate, intuitive response to each new situation. This is the hallmark of expertise, and for this reason computerized \"expert systems\" cannot be as good as human experts. Although computers have the ability to store millions of bits of information, the knowledge of human experts, who benefit from the experience of thousands of situations, is not stored within their brains in the form of rules and facts.", "question": "The argument requires the assumption of which one of the following?", "answers": "['The knowledge of human experts cannot be adequately rendered into the type of information that a computer can store.', 'Computers can show no more originality in responding to a situation than that built into them by their designers.', 'Future advances in computer technology will not render computers capable of sorting through greater amounts of information.', 'Human experts rely heavily on intuition while they are developing a repertory of model situations.']", "label": 0 }, { "id": "train_1309", "context": "A person' s dietary consumption of cholesterol and fat is one of the most important factors determining the level of cholesterol in the person' s blood (serum cholesterol). Serum cholesterol levels rise proportionally to increased cholesterol and fat consumption until that consumption reaches a threshold, but once consumption of these substances exceeds that threshold, serum cholesterol levels rise only gradually, even with dramatic increases in consumption. The threshold is one fourth the consumption level of cholesterol and fat in today' s average North American diet.", "question": "The statements above, if true, most strongly support which one of the following?", "answers": "['People who consume half as much cholesterol and fat as in the average North American diet will not necessarily have half the average serum cholesterol level.', 'Serum cholesterol levels cannot be affected by nondietary modifications in behavior, such as exercising more or smoking less.', 'People who consume less cholesterol and fat than the threshold cannot reduce their serum cholesterol levels.', 'People who consume an average North American diet cannot increase their consumption of cholesterol and fat without dramatically increasing their serum cholesterol levels.']", "label": 0 }, { "id": "train_1310", "context": "Goswami: I support the striking workers at Ergon Foods. They are underpaid. The majority of them make less that $20, 000 per year. Nordecki: If pay is the issue, I must disagree. The average annual salary of the striking workers at Ergon Foods is over $29, 000.", "question": "Goswmi and Nordecki disagree over the truth of which one of the following statements?", "answers": "['It was unreasonable for the workers at Ergon Foods to go on strike.', 'The average annual salary at Ergon Foods is over $29, 000.', 'The striking workers at Ergon Foods are underpaid.', 'It is reasonable to support striking workers who are underpaid.']", "label": 2 }, { "id": "train_1311", "context": "The Asian elephant walks with at least two, and sometimes three, feet on the ground at all times. Even though it can accelerate, it does so merely by taking quicker and longer steps. So the Asian elephant does not actually run.", "question": "The conclusion drawn above follows logically if which one of the following is assumed?", "answers": "['To run, an animal must have all of its feet off the ground at once.', 'If an animal cannot accelerate, then it cannot run.', 'It is unusual for a four-legged animal to keep three feet on the ground while walking.', 'The Asian elephant can walk as quickly as some animals run.']", "label": 0 }, { "id": "train_1312", "context": "Archaeologist: Researchers excavating a burial site in Cyprus found a feline skeleton lying near a human skeleton. Both skeletons were in the same sediment at the same depth and equally well-preserved, suggesting that the feline and human were buried together about 9, 500 years ago. This shows that felines were domesticated around the time farming began, when they would have been useful in protecting stores of grain from mice.", "question": "Which of the following, if true, would most seriously weaken the archaeologist's argument?", "answers": "['In Cyprus, there are many burial sites dating from around 9, 500 years ago in which the remains of wild animals appear to have been buried alongside human remains.', 'Before felines were domesticated, early farmers had no effective way to protect stores of grain from mice.', 'Paintings found near the burial site seem to show people keeping felines as domestic companions, but do not show felines hunting mice.', 'Archaeologists have not found any remains of stores of grain in the immediate vicinity of the burial site.']", "label": 0 }, { "id": "train_1313", "context": "Lines can be parallel in a Euclidean system of geometry. But the non-Euclidean system of geometry that has the most empirical verification is regarded by several prominent physicists as correctly describing the universe we inhabit. If these physicists are right, in our universe there are no parallel lines.", "question": "Which one of the following is an assumption that is required by the argument?", "answers": "['Only physicists who are not prominent doubt the view that the universe is correctly described by the non-Euclidean system of geometry that has the most empirical verification.', 'Most physicists have not doubted the view that the universe is correctly described by the non-Euclidean system of geometry that has the most empirical verification.', 'There are no parallel lines in the non- Euclidean system of geometry that has the most empirical verification.', 'The universe is correctly described by the non- Euclidean system of geometry that has the most empirical verification if prominent physicists maintain that it is.']", "label": 2 }, { "id": "train_1314", "context": "A successful coffee stand that serves the workers in a large office building plans to increase its profits by adding oatmeal-a hot cereal popular among the workers-to its menu. The manager says that oatmeal is \"the ultimate convenience food\" because it can be served by the cup, as coffee is; further, offering oatmeal will enable the stand to get into the breakfast business without making a large investment, because it does not require elaborate cooking equipment.", "question": "Which of the following would, if true, most strengthen the likelihood that the manager's plan will achieve its goal?", "answers": "['Oatmeal manufacturers have recently reduced their oatmeal offerings, which had included oatmeal in microwaveable cups and convenient cereal bars made with rolled oats.', 'In order to offer more flexible schedules to their workers, several businesses in the office building will soon be expanding their hours.', 'The ingredients needed to make oatmeal are inexpensive and do not spoil easily the way many fresh breakfast foods typically sold at coffee stands do.', 'Supermarket sales of oatmeal and other cereals slowed this year following large gains last year.']", "label": 2 }, { "id": "train_1315", "context": "The effects of technology on language and the effects of language on culture as a whole are profound and complex. The telegraph, the telephone, and the television have all changed the way people speak to one another. The best current example of such a change is the advent of electronic mail, which has effected a widespread loosening of language usage rules. This loosening has, in turn, made relationships between people more casual than ever before.", "question": "Which one of the following propositions is best illustrated by the statements above?", "answers": "['A widespread loosening of overly rigid language-usage rules can improve communication.', 'Changes in communication media can cause interpersonal relationships to change.', 'Changes in interpersonal relationships can cause changes in the way people speak to one another.', 'A decrease in linguistic sophistication can lead to an increase in technological sophistication.']", "label": 1 }, { "id": "train_1316", "context": "Critic: The recent biography of Shakespeare does not explain what is of most interest about him. It is by an expert on the history of Elizabethan England, and so does a good job of showing what life would have been like for Shakespeare as a man of that time. But it does not explain what made Shakespeare different from his contemporaries.", "question": "The conclusion of the argument can be properly drawn if which one of the following is assumed?", "answers": "['The life of the average man in Elizabethan England is uninteresting.', 'There is no way to know what made Shakespeare different from his contemporaries.', 'What is most interesting about Shakespeare is what made him different from his contemporaries.', 'Shakespeare was very different from the other men of his time.']", "label": 2 }, { "id": "train_1317", "context": "Game show host: Humans are no better than apes at investing, that is, they do not attain a better return on their investments than apes do. We gave five stock analysts and one chimpanzee $1, 350 each to invest. After one month, the chimp won, having increased its net worth by $210. The net worth of the analyst who came in second increased by only $140.", "question": "Each of the following describes a flaw in the game show host's reasoning EXCEPT:", "answers": "['No evidence is offered that chimpanzees are capable of understanding stock reports and making reasoned investment decisions.', \"No evidence is considered about the long-term performance of the chimpanzee's portfolio versus that of the analysts' portfolios.\", 'A conclusion is drawn about apes in general on the basis of an experiment involving one chimpanzee.', 'Too general a conclusion is made about investing on the basis of a single experiment involving short-term investing but not longterm investing.']", "label": 0 }, { "id": "train_1318", "context": "This year a flood devastated a small river town. Hollyville, also a river town, responded with an outpouring of aid in which a majority of its residents participated, a proportion that far surpassed that of a few years ago when Hollyville sent aid to victims of a highly publicized earthquake. This year' s circumstances were a reversal of last year' s, when Hollyville itself was the scene of a deadly tornado and so the recipient rather than the supplier of emergency aid.", "question": "The situation described above most closely conforms to which one of the following generalizations?", "answers": "['People are more likely to aid people they know than they are to aid strangers.', 'Once a disaster has struck them, people are more likely to aid others in need than they were before the disaster.', 'People are more likely to aid those who have experienced a hardship similar to one they themselves have experienced than to aid those who have experienced a dissimilar hardship.', 'Those who have received aid are more likely to be in favor of government relief programs than are those who have not.']", "label": 1 }, { "id": "train_1319", "context": "A researcher studying corporate executives found that they tend to have \"take charge\" personalities, with the predominant traits of assertiveness, decisiveness, and self-confidence. The researcher concluded that people who are more \"take charge\" than the average person are more likely to become corporate executives.", "question": "Which of the following, if true, most seriously weakens the researcher's conclusion?", "answers": "['Many people who aspire to become executives exhibit different management styles in their current jobs.', 'The executives that the researcher studied were often unsuccessful when they tried to manage their family activities as they do their business activities.', 'Holding the job of a corporate executive causes people to develop \"take charge\" personality traits.', 'Some people who are not executives have stronger \"take charge\" personalities than some people who currently serve as corporate executives.']", "label": 2 }, { "id": "train_1320", "context": "Although withholding information from someone who would find that information painful is sometimes justified, there is no such justification if the person would benefit from having the information. Ttherefore, even though it would be painful for Jason to learn that his supervisor is displeased with his work, his colleague Jane should nonetheless inform Jason of this fact, for knowing that his supervisor was displeased would enable Jason to improve his supervisor' s opinion of his work.", "question": "Which one of the following is an assumption on which the argument relies?", "answers": "[\"Jason might eventually improve his supervisor's opinion of his work even if he never learns that his supervisor is displeased with his work.\", 'If Jane tells Jason that his supervisor is displeased with his work, Jason will be grateful for the information even though it will be painful for him to learn it.', \"If Jane does not tell Jason that his supervisor is displeased with his work, then Jason's situation will worsen.\", \"Jason would benefit if he were able to improve his supervisor's opinion of his work.\"]", "label": 3 }, { "id": "train_1321", "context": "Any organization must consider changing its basic structure if there is a dramatic change in its size. Doubling or tripling in size can lead to inefficiency and mismanagement, which restructuring often alleviates. Experience further shows that the more dramatic the change in size, the more fundamental the restructuring needs to be. Ttherefore, any organization must consider changing its basic structure once it is 20 years old.", "question": "The argument's conclusion follows logically if which one of the following is assumed?", "answers": "['No organization that has not been restructured is as efficient as any organization that has been restructured.', 'Any organization that has existed for 20 years has undergone a dramatic change in size.', 'No organization that has remained the same size for 20 years is efficient.', 'No organization that has not changed in size needs restructuring.']", "label": 1 }, { "id": "train_1322", "context": "When the rate of inflation exceeds the rate of return on the most profitable investment available, the difference between those two rates will be the percentage by which, at a minimum, the value of any investment will decline. If in such a circumstance the value of a particular investment declines by more than that percentage, it must be true that __ .", "question": "Which one of the following logically completes the argument?", "answers": "['the rate of inflation has risen', 'the investment in question is becoming less profitable', 'the rate of return on the most profitable investment available has declined', 'the investment in question is less profitable than the most profitable investment available']", "label": 3 }, { "id": "train_1323", "context": "Letter to the editor: After Baerton' s factory closed, there was a sharp increase in the number of claims filed for job-related injury compensation by the factory' s former employees. Hence there is reason to believe that most of those who filed for compensation after the factory closed were just out to gain benefits they did not deserve, and filed only to help them weather their job loss.", "question": "Each of the following, if true, weakens the argument above EXCEPT:", "answers": "['Most workers who receive an injury on the job file for compensation on the day they suffer the injury.', \"In the years before the factory closed, the factory's managers dismissed several employees who had filed injury claims.\", 'Workers who incur partial disabilities due to injuries on the job often do not file for compensation because they would have to stop working to receive compensation but cannot afford to live on that compensation alone.', 'Workers who are aware that they will soon be laid off from a job often become depressed, making them more prone to job-related injuries.']", "label": 0 }, { "id": "train_1324", "context": "Raj: Chiropractics should be outlawed in this state as treatment for neck pain; physicians have presented evidence that there is a link between neck manipulations and strokes. Nancy: You are correct in pointing out that neck manipulations have been linked to strokes. However, this risk only exists when the practitioner gives the neck a high-velocity twist. If there were better established requirements of using less forceful techniques for neck manipulations, then chiropractics would be a safe and effective treatment for neck pain.", "question": "Nancy does which one of the following in responding to Raj's argument?", "answers": "['She defends an opinion by limiting the set of circumstances to which it is to be applied.', \"She points out that Raj's argument itself creates the objectionable quality that it denounces.\", 'She refuses to acknowledge the facts that Raj uses to support his argument to be true.', \"She establishes that Raj's premises are self- contradictory.\"]", "label": 0 }, { "id": "train_1325", "context": "A recent study confirms that nutritious breakfasts make workers more productive. For one month, workers at Plant A received free nutritious breakfasts every day before work, while workers in Plant B did not. The productivity of Plant A' s workers increased, while that of Plant B' s workers did not.", "question": "Which one of the following, if true, most strengthens the argument?", "answers": "['Workers from Plant A took fewer vacation days per capita during the month than did workers from Plant B.', 'Few workers in Plant B consumed nutritious breakfasts during the month of the study.', 'Workers in Plant B were more productive during the month of the study than were workers from Plant A.', 'During the month before the study, workers at Plant A and Plant B were equally productive.']", "label": 1 }, { "id": "train_1326", "context": "Readers give more credibility and respond more positively to negative restaurant reviews if the critics begin their reviews by discussing the things they enjoyed about the restaurant. Studies show that readers enjoy hearing positive comments before being told unpleasant details. Ttherefore, any restaurant critic should make sure to start every review by giving ample praise about the positive aspects of the restaurant before launching into any negative commentary.", "question": "Which one of the following, if true, most seriously casts doubt on the wisdom of this recommendation?", "answers": "['People usually decide which restaurants to patronize based on the negative comments made by restaurant critics.', 'Most restaurant critics do not have any official culinary training or formal training in the food industry.', 'Restaurant critics often receive bribes from restaurants to write positive reviews.', 'Newspapers and magazines will often only publish the beginning of a restaurant review to save space.']", "label": 3 }, { "id": "train_1327", "context": "Planetary bodies differ from one another in their composition, but most of those in the Solar System have solid surfaces. Unless the core of such a planetary body generates enough heat to cause volcanic action, the surface of the body will not be renewed for millions of years. Any planetary body with a solid surface whose surface is not renewed for millions of years becomes heavily pockmarked by meteorite craters, just like the Earth' s Moon. Some old planetary bodies in the Solar System, such as Europa, a very cold moon belonging to Jupiter, have solid icy surfaces with very few meteorite craters.", "question": "If the claims above are true, which one of the following must, on the basis of them, be true?", "answers": "['Some very cold planetary bodies have cores that generate enough heat to cause volcanic action.', \"Some of Jupiter's moons are heavily pockmarked by meteorite craters.\", 'Some planetary bodies whose cores generate enough heat to cause volcanic action do not have solid icy surfaces.', 'If a planetary body does not have a heavily pockmarked surface, its core does not generate enough heat to cause volcanic action.']", "label": 0 }, { "id": "train_1328", "context": "A person' s failure to keep a promise is wrong only if, first, doing so harms the one to whom the promise is made and, second, all of those who discover the failure to keep the promise lose confidence in the person' s ability to keep promises.", "question": "Which one of the following judgments most closely conforms to the principle above?", "answers": [ "Jonathan took an oath of secrecy concerning the corporation's technical secrets, but he sold them to a competitor. His action was wrong even though the corporation intended that he leak these secrets to its competitors.", "Ann kept her promise to repay Felicia the money she owed her. Further, this convinced everyone who knew Ann that she is trustworthy. Thus, Ann's keeping her promise was not wrong.", "Elizabeth promised to return the book she borrowed from Steven within a week, but she was unable to do so because she became acutely ill. Not knowing this, Steven lost confidence in her ability to keep a promise. So, Elizabeth's failure to return the book to Steven was wrong.", "Because he lost his job, Carlo was unable to repay the money he promised to Miriam. However, Miriam did not need this money nor did she lose confidence in Carlo's ability to keep promises. So, Carlo's failure to keep his promise to Miriam was not wrong." ], "label": 3 }, { "id": "train_1329", "context": "Journalism' s purpose is to inform people about matters relevant to the choices they must make. Yet, clearly, people often buy newspapers or watch television news programs precisely because they contain sensationalistic gossip about people whom they will never meet and whose business is of little relevance to their lives. Obviously, then, the sensationalistic gossip contained in newspapers and television news programs__.", "question": "Which one of the following most logically completes the argument?", "answers": "['is of no value to people who are interested in journalism', \"is more relevant to people's lives now than it used to be\", 'is at least sometimes included for nonjournalistic reasons', 'prevents those news media from achieving their purpose']", "label": 2 }, { "id": "train_1330", "context": "A mass of \"black water\" containing noxious organic material swept through Laurel Bay last year. Some scientists believe that this event was a naturally occurring but infrequent phenomenon. The black water completely wiped out five species of coral in the bay, including mounds of coral that were more than two centuries old. Ttherefore, even if this black water phenomenon has struck the bay before, it did not reach last year' s intensity at any time in the past two centuries.", "question": "Which one of the following is an assumption required by the argument?", "answers": "['Older specimens of coral in the bay were more vulnerable to damage from the influx of black water than were young specimens.', 'Every species of coral in the bay was seriously harmed by the mass of black water that swept in last year.', 'The mounds of centuries-old coral that were destroyed were not in especially fragile condition just before the black water swept in last year.', 'Masses of black water such as that observed last summer come into the bay more frequently than just once every two centuries.']", "label": 2 }, { "id": "train_1331", "context": "Commentator: In many countries the influence of fringe movements is increasing. The great centrifugal engine of modern culture turns faster and faster, spinning off fashions, ideologies, religions, artistic movements, economic theories, cults, and dogmas in fabulous profusion. Hence, modern culture threatens the national identities that now exist in the world.", "question": "Which one of the following statements, if true, most seriously weakens the commentator's argument?", "answers": "['New national identities are often forged out of conflicts among diverse groups.', 'It is preferable to have a pluralistic rather than a monolithic national culture.', 'The rate of cultural change in most countries will soon change drastically.', 'A stable national identity is typically a composite of a staggering number of subcultures.']", "label": 3 }, { "id": "train_1332", "context": "Finnish author Jaakko Mikkeli was accused by Norwegian author Kirsten Halden of plagiarizing a book that she had written and that had been published 20 years before Mikkeli' s. The two books, although set in different periods and regions, contain enough plot similarities to make coincidental resemblance unlikely. Mikkeli' s defense rests on his argument that plagiarism was impossible in this case because Halden' s book has been published only in Norwegian, a language Mikkeli does not understand, and because no reviews of Halden' s book have ever been published.", "question": "The argument in Mikkeli's defense depends on the assumption that", "answers": "[\"nobody related the plot of Halden's book in detail to Mikkeli before Mikkeli wrote his book\", 'there is a common European myth to which both authors referred subconsciously in the books in question', 'Mikkeli is not familiar with Old Icelandic, an extinct language related to an earlier form of Norwegian', 'Mikkeli has never met Halden']", "label": 0 }, { "id": "train_1333", "context": "CEO: We have been falsely criticized for not being an environmentally responsible corporation. Environmentally responsible corporations are corporations that do all they can to pollute less. Our current production methods pollute significantly less than our old methods did, and there currently are no methods that do not produce any pollution.", "question": "The reasoning in the CEO's argument is flawed in that it", "answers": "['ignores the possibility that there are currently production methods that would allow the corporation to produce less pollution than it does now', 'generalizes too hastily from the inapplicability of a specific criticism to the inapplicability of a class of criticisms', 'takes for granted that production methods that do not produce pollution cannot be developed', 'takes for granted that because the company has attempted to reduce the amount of pollution produced, they must have succeeded']", "label": 0 }, { "id": "train_1334", "context": "Any driver involved in an accident leading to personal injury or property damage exceeding $500 is legally required to report the accident to the department of motor vehicles, unless the driver is incapable of doing so. Ted is not required to report the accident in which he was involved as a driver.", "question": "Which one of the following can be properly inferred from the statements above?", "answers": "['Someone other than Ted is legally required to report the accident to the department of motor vehicles.', 'If Ted is incapable of reporting the accident, then the accident did not lead to property damage exceeding $500.', \"If Ted's car was damaged in excess of $500 in the accident, then he is incapable of reporting the accident to the department of motor vehicles.\", 'Either no one was injured in the accident or the accident did not lead to property damage exceeding $500.']", "label": 2 }, { "id": "train_1335", "context": "Studies indicate that the rate at which water pollution is increasing is leveling off: the amount of water pollution caused this year is almost identical to the amount caused last year. If this trend continues, the water pollution problem will no longer be getting more serious.", "question": "The reasoning is questionable because it ignores the possibility that", "answers": "['the effects of water pollution are cumulative', 'air and soil pollution are becoming more serious', 'the leveling-off trend of water pollution will not continue', 'some types of water pollution have no noticeable effect on organisms that use the water']", "label": 0 }, { "id": "train_1336", "context": "On the whole, scientists do their most creative work before age forty, a tendency that has been taken to show that aging carries with it a loss of creative capacity. An alternative explanation is that by age forty most scientists have worked in their field for fifteen or more years and that by then they have exhausted the opportunity for creative work in that field. Supporting this explanation is the finding that __.", "question": "Which of the following most logically completes the passage?", "answers": "['a disproportionately large number of the scientists who produce highly creative work beyond age forty entered their field at an older age than is common', 'the average age of recipients of scientific research grants is significantly greater than forty', 'many scientists temper their own expectations of what they can achieve in their research work by their belief that their creativity will decline as they age', 'scientists who are older than forty tend to find more satisfaction in other activities, such as teaching and mentoring, than they do in pursuing their own research']", "label": 0 }, { "id": "train_1337", "context": "In 1988, a significant percentage of seals in the Baltic Sea died from viral diseases; off the coast of Scotland, however, the death rate due to viral diseases was approximately half what it was for the Baltic seals. The Baltic seals had significantly higher levels of pollutants in their blood than did the Scottish seals. Since pollutants are known to impair marine mammals' ability to fight off viral infection, it is likely that the higher death rate among the Baltic seals was due to the higher levels of pollutants in their blood.", "question": "Which one of the following, if true, provides the most additional support for the argument?", "answers": "['The kinds of pollutants found in the Baltic Sea are significantly different from those that have been detected in the waters off the coast of Scotland.', 'The strain of virus that killed Scottish seals overwhelms impaired immune systems much more quickly than it does healthy immune systems.', 'The large majority of Scottish seals that died were either old or unhealthy animals.', 'Among marine mammals other than seals, the death rate due to viral diseases in 1988 was higher in the Baltic Sea than it was off the Scottish coast.']", "label": 3 }, { "id": "train_1338", "context": "Last year Comfort Airlines had twice as many delayed flights as the year before, but the number of complaints from passengers about delayed flights went up three times. It is unlikely that this disproportionate increase in complaints was rooted in an increase in overall dissatisfaction with the service Comfort Airlines provides, since the airline made a special effort to improve other aspects of its service last year.", "question": "Which of the following, if true, most helps to explain the disproportionate increase in customer complaints?", "answers": "['The average length of a flight delay was greater last year than it was the year before.', 'Some of the improvements that Comfort Airlines made in its service were required by new government regulations.', 'The average number of passengers per flight was no higher last year than the year before.', 'Comfort Airlines had more flights last year than the year before.']", "label": 0 }, { "id": "train_1339", "context": "Goronian lawmaker: Goronia' s Cheese Importation Board, the agency responsible for inspecting all wholesale shipments of cheese entering Goronia from abroad and rejecting shipments that fail to meet specified standards, rejects about one percent of the cheese that it inspects. Since the health consequences and associated costs of not rejecting that one percent would be negligible, whereas the cost of maintaining the agency is not, the agency' s cost clearly outweighs the benefits it provides.", "question": "Knowing the answer to which of the following would be most useful in evaluating the lawmaker's argument?", "answers": "['How is the cheese rejected by the Cheese Importation Board disposed of?', 'Has the Cheese Importation Board, over the last several years, reduced its operating costs by eliminating inefficiencies within the agency itself?', 'Are any of the types of cheeses that are imported into Goronia also produced in Goronia?', 'Does the possibility of having merchandise rejected by the Cheese Importation Board deter many cheese exporters from shipping substandard cheese to Goronia?']", "label": 3 }, { "id": "train_1340", "context": "A survey was recently conducted among ferry passengers on the North Sea. Among the results was this: more of those who had taken anti-seasickness medication before their trip reported symptoms of seasickness than those who had not taken such medication. It is clear, then, that despite claims by drug companies that clinical tests show the contrary, people would be better off not taking anti-seasickness medications.", "question": "Which one of the following, if true, would most weaken the conclusion above?", "answers": "['Given rough enough weather, most ferry passengers will have some symptoms of seasickness.', 'The seasickness symptoms of the people who took anti-seasickness medication would have been more severe had they not taken the medication.', \"The clinical tests reported by the drug companies were conducted by the drug companies' staffs.\", 'People who do not take anti-seasickness medication are just as likely to respond to a survey on seasickness as people who do.']", "label": 1 }, { "id": "train_1341", "context": "Some government economists view their home countries as immune to outside influence. But economies are always open systems; international trade significantly affects prices and wages. Just as physicists learned the shortcomings of a mechanics based on idealizations such as the postulation of perfectly frictionless bodies, government economists must look beyond national borders if their nations' economies are to prosper.", "question": "The argument's conclusion follows logically if which one of the following is assumed?", "answers": "['Some government economists have been ignoring the effects of international trade on prices and wages.', 'Economic theories relying on idealizations are generally less accurate than economic theories that do not rely on idealizations.', \"A national economy cannot prosper unless every significant influence on it has been examined by that nation's government economists.\", 'Economics is weakly analogous to the physical sciences.']", "label": 2 }, { "id": "train_1342", "context": "Activist: Medical conditions such as cancer and birth defects have been linked to pollutants in water. Organic pollutants such as dioxins, and inorganic pollutants such as mercury, are ingested by fish and move up the food chain to people, where they accumulate in tissue. Since most cancers and birth defects are incurable, we need to aim at their prevention. Clearly, the only effective way to reduce significantly their overall incidence is to halt industries known to produce these pollutants, given that such industries are unlikely to comply adequately with strict environmental regulations.", "question": "A flaw in the activist's reasoning is that it", "answers": "['fails to consider whether industries may voluntarily decrease their output of pollutants', 'takes for granted that certain effects can be produced independently by several different causes', 'fails to consider the possibility that chemicals now classified as pollutants have some beneficial effects not yet discovered', 'fails to consider the possibility that a significant number of occurrences of cancer and birth defects may be caused by preventable factors other than industrial pollutants']", "label": 3 }, { "id": "train_1343", "context": "Double-blind techniques should be used whenever possible in scientific experiments. They help prevent the misinterpretations that often arise due to expectations and opinions that scientists already hold, and clearly scientists should be extremely diligent in trying to avoid such misinterpretations.", "question": "Which one of the following most accurately expresses the main conclusion of the argument?", "answers": "['It is advisable for scientists to use double-blind techniques in as high a proportion of their experiments as they can.', 'Scientists sometimes neglect to adequately consider the risk of misinterpreting evidence on the basis of prior expectations and opinions.', \"Scientists' objectivity may be impeded by interpreting experimental evidence on the basis of expectations and opinions that they already hold.\", 'Double-blind experimental techniques are often an effective way of ensuring scientific objectivity.']", "label": 0 }, { "id": "train_1344", "context": "Mystery stories often feature a brilliant detective and the detective' s dull companion. Clues are presented in the story, and the companion wrongly infers an inaccurate solution to the mystery using the same clues that the detective uses to deduce the correct solution. Thus, the author' s strategy of including the dull companion gives readers a chance to solve the mystery while also diverting them from the correct solution.", "question": "Which one of the following is most strongly supported by the information above?", "answers": "['Most mystery stories feature a brilliant detective who solves the mystery presented in the story.', \"Mystery readers often solve the mystery in a story simply by spotting the mistakes in the reasoning of the detective's dull companion in that story.\", \"The detective's dull companion in a mystery story generally uncovers the misleading clues that divert readers from the mystery's correct solution.\", 'Some mystery stories give readers enough clues to infer the correct solution to the mystery.']", "label": 3 }, { "id": "train_1345", "context": "Teacher: Students don' t need parental involvement to succeed. In my class of twenty kids, the two highest achieving students come from foster homes. There are too many children in the foster homes for their parents to monitor homework and enforce study habits. It' s always the case that students can overcome their parents' indifference.", "question": "What mistake does the teacher commit in his reasoning?", "answers": "['The teacher fails to consider competing theories.', \"The teacher's conclusion is totally unjustified.\", 'The teacher is biased.', 'The teacher relies on an unreasonably small sample size in drawing his conclusion.']", "label": 3 }, { "id": "train_1346", "context": "More and more companies have begun to consume less energy by making themselves more efficient. Over time, these efforts could place the United States at the forefront of an emerging global market for cleaner technologies. Such efforts are also essential to tackling the two big energy-related issues of the age: global warming and the dependence on precarious supplies of oil. The federal government should encourage these efforts by providing the necessary incentives, whether as loans, direct grants or targeted tax breaks.", "question": "Which of the following, if true, provides the most effective support for the argument?", "answers": "['In the past, government incentives have made advances in energy conservation feasible, especially in the auto industry.', 'The market for cleaner technologies is currently relatively small because of the infrastructure requirements.', 'The dependence on foreign oil is a greater problem in the present than global warming.', 'On the average, Canadian companies are more energy efficient than those in the United States.']", "label": 0 }, { "id": "train_1347", "context": "Nutritionist: Vitamins synthesized by chemists are exactly the same as vitamins that occur naturally in foods. Ttherefore, it is a waste of money to pay extra for brands of vitamin pills that are advertised as made of higher-quality ingredients or more natural ingredients than other brands are.", "question": "The nutritionist's advice is based on which one of the following assumptions?", "answers": "['All brands of vitamin pills contain some synthesized vitamins.', 'Some producers of vitamin pills are guilty of false advertising.', 'It is a waste of money for people to supplement their diets with vitamin pills.', 'There is no nonvitamin ingredient in vitamin pills whose quality makes one brand worth more money than another brand.']", "label": 3 }, { "id": "train_1348", "context": "Clark: Our local community theater often produces plays by critically acclaimed playwrights. In fact, the production director says that critical acclaim is one of the main factors considered in the selection of plays to perform. So, since my neighbor Michaela' s new play will be performed by the theater this season, she must be a critically acclaimed playwright.", "question": "The reasoning in Clark's argument is most vulnerable to criticism on the grounds that the argument", "answers": "['uses as evidence a source that there is reason to believe is unreliable', 'fails to consider that several different effects may be produced by a single cause', 'treats one main factor considered in the selection of plays to perform as though it were a condition that must be met in order for a play to be selected', \"takes a condition necessary for a playwright's being critically acclaimed to be a condition sufficient for a playwright's being critically acclaimed\"]", "label": 2 }, { "id": "train_1349", "context": "A computer equipped with signature-recognition software, which restricts access to a computer to those people whose signatures are on file, identifies a person' s signature by analyzing not only the form of the signature but also such characteristics as pen pressure and signing speed. Even the most adept forgers cannot duplicate all of the characteristics the program analyzes.", "question": "Which of the following can be logically concluded from the passage above?", "answers": "['Signature-recognition software has taken many years to develop and perfect.', 'Computers equipped with the software will soon be installed In most banks.', 'Nobody can gain access to a computer equipped with the software solely by virtue of skill at forging signatures.', 'The time it takes to record and analyze a signature makes the software impractical for everyday use.']", "label": 2 }, { "id": "train_1350", "context": "Every new play that runs for more than three months is either a commercial or a critical success. Last year, all new plays that were critical successes were also commercial successes. Ttherefore, every new play that ran for more than three months last year was a commercial success.", "question": "The pattern of reasoning in which one of the following arguments is most similar to that in the argument above?", "answers": "['Most new restaurants require either good publicity or a good location in order to succeed. But most restaurants with a good location also receive good publicity. Hence, a restaurant that has a good location is guaranteed to succeed.', 'All students at the Freeman School of Cooking study either desserts or soups in their second year. This year, all Freeman students studying soups are also studying desserts. Ttherefore, every second-year student at Freeman is studying desserts this year.', \"Every catering service in Woodside Township will accept both residential and business catering assignments. Peggy's Fine Foods is a catering service that will not accept business catering assignments. Hence, Peggy's Fine Foods is not in Woodside Township.\", \"Every best-selling cookbook published last year is both well written and contains beautiful photographs. The cookbook Cynthia Cleveland published last year is well written and contains beautiful photographs. Ttherefore, Cleveland's cookbook is a best seller.\"]", "label": 1 }, { "id": "train_1351", "context": "An independent computer service company tallied the service requests it receives for individual brands of personal computers. It found that, after factoring in each brand' s market share, KRV brand computers had the largest proportion of service requests, whereas ProBit brand computers had the smallest proportion of service requests. Obviously, ProBit is the more reliable personal computer brand.", "question": "Which one of the following, if true, most seriously weakens the argument?", "answers": "['The company that did the tally receives more service requests for ProBit brand computers than does any other independent computer service company.', \"The computer brands covered in the computer service company's tally differ greatly with respect to their market share.\", \"For some computer brands, but not for others, most service requests are made to the manufacturer's service department rather than to an independent service company.\", 'ProBit has been selling personal computers for many more years than has KRV.']", "label": 2 }, { "id": "train_1352", "context": "Pundit: Our city made a mistake when it sold a private company the rights to assess and collect parking fees. The private company raised parking fees and so has been able to reap profits far greater than what it paid for the rights to assess and collect the fees. If the city had not sold the rights, then that money would have gone to the city.", "question": "The pundit's argument requires the assumption that", "answers": "['the city could have raised parking fees had it not sold the rights', 'municipal functions like assessing and collecting parking fees should always be handled directly by the municipality in question', 'other private companies would have been willing to pay for the rights to assess and collect parking fees', 'private companies assess and collect parking fees more efficiently than cities do']", "label": 0 }, { "id": "train_1353", "context": "Hospital auditor: The Rodriguez family stipulated that the funds they donated to the neurological clinic all be used to minimize patients' suffering. The clinic administration is clearly violating those terms, since it has allocated nearly one fifth of those funds for research into new diagnostic technologies, instead of letting that money flow directly to its patients. Clinic administrator: But the successful development of new technologies will allow early diagnosis of many neurological disorders. In most cases, patients who are treated in the early stages of neurological disorders suffer far less than do patients who are not treated until their neurological disorders reach advanced stages.", "question": "Which one of the following is the main point at issue between the hospital auditor and the clinic administrator?", "answers": "[\"whether the Rodriguez family clearly stipulated that the funds they donated to the neurological clinic be used to minimize patients' suffering\", 'whether the patients being treated at the neurological clinic are currently receiving adequate treatment for the neurological disorders from which they suffer', 'whether early treatment of many neurological disorders lessens the suffering associated with those disorders rather than completely eliminating such suffering', 'whether the neurological clinic is adhering strictly to the conditions the Rodriguez family placed on the allocation of the funds they donated to the clinic']", "label": 3 }, { "id": "train_1354", "context": "Essayist: Computers have the capacity to represent and to perform logical transformations on pieces of information. Since exactly the same applies to the human mind, the human mind is a type of computer.", "question": "The flawed pattern of reasoning in which one of the following most closely resembles the flawed pattern of reasoning in the essayist's argument?", "answers": "['In any organism, the proper functioning of each component depends upon the proper functioning of every other component. Thus, communities belong to the category of organisms, since communities are invariably characterized by this same interdependence of components.', 'Often individual animals sacrifice their lives when the survival of their offspring or close relatives is threatened. It is probable, ttherefore, that there is a biological basis for the fact that human beings are similarly often willing to sacrifice their own well-being for the good of their community.', 'In the plastic arts, such as sculpture or painting, no work can depend for its effectiveness upon a verbal narrative that explains it. Since the same can be said of poetry, we cannot consider this characteristic as a reasonable criterion for distinguishing the plastic arts from other arts.', 'Friendship often involves obligations whose fulfillment can be painful or burdensome. The same can be said of various forms of cooperation that cannot strictly be called friendship. Thus cooperation, like friendship, can require that priority be given to goals other than mere self-interest.']", "label": 0 }, { "id": "train_1355", "context": "A species in which mutations frequently occur will develop new evolutionary adaptations in each generation. Since species survive dramatic environmental changes only if they develop new evolutionary adaptations in each generation, a species in which mutations frequently occur will survive dramatic environmental changes.", "question": "The flawed pattern of reasoning in which one of the following is most closely parallel to that in the argument above?", "answers": "[\"In a stone wall that is properly built, every stone supports another stone. Since a wall's being sturdy depends upon its being properly built, only walls that are composed entirely of stones supporting other stones are sturdy.\", 'A person who is perfectly honest will tell the truth in every situation. Since in order to be a morally upright person one must tell the truth at all times, a perfectly honest person will also be a morally upright person.', 'A play that is performed before a different audience every time will never get the same reaction from any two audiences. Since no plays are performed before the same audience every time, no play ever gets the same reaction from any two audiences.', 'An herb garden is productive only if the soil that it is planted in is well drained. Since soil that is well drained is good soil, an herb garden is not productive unless it is planted in good soil.']", "label": 1 }, { "id": "train_1356", "context": "Parent 1: Ten years ago, children in communities like ours did not date until they were thirteen to fifteen years old. Now our nine to eleven year olds are dating. Obviously, children in communities like ours are becoming romantically interested in members of the opposite sex at an earlier age today than they did ten years ago. Parent 2: I disagree. Our nine to eleven year olds do not want to date, but they feel intense peer pressure to act grown up by dating.", "question": "Parent 2, in responding to Parent 1, does which one of the following?", "answers": "['criticizes Parent 1 as a proponent of a claim rather than criticizing the claim itself', 'refutes a generalization about nine to eleven- year-old children by means of an exceptional case overlooked by Parent 1', \"provides an alternative explanation for the changes in children's dating described by Parent 1\", 'assumes that nine to eleven-year-old children are as interested in dating as thirteen to fifteen-year-old children']", "label": 2 }, { "id": "train_1357", "context": "Creating a database of all the plant species in the scientific record has proved to be no easy task. For centuries, botanists have been collecting and naming plants without realizing that many were in fact already named. And by using DNA analysis, botanists have shown that varieties of plants long thought to belong to the same species actually belong to different species.", "question": "Of the following claims, which one can most justifiably be rejected on the basis of the statements above?", "answers": "['Duplicates and omissions in the scientific record also occur in fields other than botany.', 'Botanists have no techniques for determining whether distinct plant species have been given distinct names.', \"A person who consults the scientific record looking under only one of a plant's names may miss available information about that plant.\", 'Most of the duplicates and omissions among plant names in the scientific record have yet to be cleared up.']", "label": 1 }, { "id": "train_1358", "context": "In a certain factory, the union membership was higher in 2010 than in 2008. For the period from 2007 to 2011, the percentage of factory workers that were members of the union remained constant.", "question": "The statements above, if true, most strongly support which conclusion?", "answers": "['Throughout this entire manufacturing sector, factory worker unions saw growth from 2008 to 2010', 'The union membership in 2009 was higher than it was in 2008', 'The factory had more employees in 2010 than in 2008', 'In 2010, there were fewer non-union employees in this factory than there were in 2008']", "label": 2 }, { "id": "train_1359", "context": "Arguing that there was no trade between Europe and East Asia in the early Middle Ages because there are no written records of such trade is like arguing that the yeti, an apelike creature supposedly existing in the Himalayas, does not exist because there have been no scientifically confirmed sightings. A verifiable sighting of the yeti would prove that the creature does exist, but the absence of sightings cannot prove that it does not.", "question": "Which one of the following best expresses the point of the argument?", "answers": "['The view that there was trade between Europe and East Asia in the early Middle Ages can only be disproved by showing that no references to this trade exist in surviving records.', 'In order to prove that in the early Middle Ages there was trade between Europe and East Asia it is necessary to find both Asian and European evidence that such trade existed.', 'There is no more evidence that trade between Europe and East Asia existed in the early Middle Ages than there is that the yeti exists.', 'That trade between Europe and East Asia did not exist in the early Middle Ages cannot be established simply by the absence of a certain sort of evidence that this trade existed.']", "label": 3 }, { "id": "train_1360", "context": "People who need to reduce their intake of fat and to consume fewer calories often turn to fat substitutes, especially those with zero calories such as N5. But studies indicate that N5 is of no use to such people. Subjects who ate foods prepared with N5 almost invariably reported feeling hungrier afterwards than after eating foods prepared with real fat and consequently they ate more, quickly making up for the calories initially saved by using N5.", "question": "The reasoning in the argument is most vulnerable to criticism on the grounds that the argument fails to consider the possibility that", "answers": "['the total fat intake of people who eat foods prepared with N5 tends to decrease even if their caloric intake does not', 'N5 has mild but unpleasant side effects', 'many foods cannot be prepared with N5', 'people who know N5 contains zero calories tend to eat more foods prepared with N5 than do people who are unaware that N5 is calorie-free']", "label": 0 }, { "id": "train_1361", "context": "It takes 365. 25 days for the Earth to make one complete revolution around the Sun. Long-standing convention makes a year 365 days long, with an extra day added every fourth year, and the year is divided into 52 seven-day weeks. But since 52 times 7 is only 364, anniversaries do not fall on the same day of the week each year. Many scheduling problems could be avoided if the last day of each year and an additional day every fourth year belonged to no week, so that January 1 would be a Sunday every year.", "question": "The proposal above, once put into effect, would be most likely to result in continued scheduling conflicts for which one of the following groups?", "answers": "['employed people whose strict religious observances require that they refrain from working every seventh day', 'people who have birthdays or other anniversaries on December 30 or 31', 'people who have to plan events several years before those events occur', 'employed people who have three-day breaks from work when holidays are celebrated on Mondays or Fridays']", "label": 0 }, { "id": "train_1362", "context": "The process by which nylon is manufactured releases large amounts of the gas nitrous oxide, which is harmful to the environment. Since the processing of cotton fiber does not release environmentally harmful gases, there would be less environmental damage done if cotton fiber rather than nylon were used to make products such as thread and rope.", "question": "Which one of the following, if true, would weaken the argument?", "answers": "['Even if the quantity of nitrous oxide released into the environment decreased, many environmental problems would remain unsolved.', 'If cotton fiber replaced nylon in the production of thread and rope, there would be a resulting increase in the amount of nylon used in other manufactured products.', 'If the quantity of nylon manufactured annually decreased substantially, the volume of several pollutants that are released into the environment during its manufacture would be reduced.', 'Even if only some of the thread and rope that is currently being made from nylon were instead made from cotton fiber, some environmental damage would be avoided.']", "label": 1 }, { "id": "train_1363", "context": "In comparison to the standard typewriter keyboard, the EFCO keyboard, which places the most-used keys nearest the typist's strongest fingers, allows faster typing and results in less fatigue. Ttherefore, replacement of standard keyboards with the EFCO keyboard will result in an immediate reduction of typing costs.", "question": "Which of the following, if true, would most weaken the conclusion drawn above?", "answers": "['The number of businesses and government agencies that use EFCO keyboards is increasing each year.', 'People who use both standard and EFCO keyboards report greater difficulty in the transition from the EFCO keyboard to the standard keyboard than in the transition from the standard keyboard to the EFCO keyboard.', 'Novice typists can learn to use the EFCO keyboard in about the same amount of time that it takes them to learn to use the standard keyboard.', 'The more training and experience an employee has had with the standard keyboard, the more costly it is to train that employee to use the EFCO keyboard.']", "label": 3 }, { "id": "train_1364", "context": "Professor Smith published a paper arguing that a chemical found in minute quantities in most drinking water had an adverse effect on the human nervous system. Existing scientific theory held that no such effect was possible because there was no neural mechanism for bringing it about. Several papers by well-known scientists in the field followed, unanimously purporting to prove Professor Smith wrong. This clearly shows that the scientific establishment was threatened by Professor Smith' s work and conspired to discredit it.", "question": "Which one of the following is the central flaw in the argument given by the author of the passage?", "answers": "['The author fails to mention whether or not Professor Smith knew that the existence of the alleged new effect was incompatible with established scientific theory.', \"The author passes over the possibility that Professor Smith had much to gain should Professor Smith's discovery have found general acceptance.\", 'The author fails to show why the other scientists could not have been presenting evidence in order to establish the truth of the matter.', \"The author fails to indicate what, if any, effect the publication of Professor Smith's paper had on the public's confidence in the safety of most drinking water.\"]", "label": 2 }, { "id": "train_1365", "context": "City Official: At City Hospital, uninsured patients tend to have shorter stays and fewer procedures performed than do insured patients, even though insured patients, on average, have slightly less serious medical problems at the time of admission to the hospital than uninsured patients have. Critics of the hospital have concluded that the uninsured patients are not receiving proper medical care . However, this conclusion is almost certainly false . Careful investigation has recently shown two things: insured patients have much longer stays in the hospital than necessary, and they tend to have more procedures performed than are medically necessary.", "question": "In the city official's argument, the two boldface portions play which of the following roles?", "answers": [ "The first states the position that the city official's argument opposes; the second states the conclusion of the city official's argument.", "The first was used to support the conclusion drawn by hospital critics; the second provides support for the conclusion of the city official's argument.", "The first was used to support the conclusion drawn by hospital critics; the second states the position that the city official's argument opposes.", "The first is used to support the conclusion of the city official's argument; the second states that conclusion." ], "label": 0 }, { "id": "train_1366", "context": "Businessman: Sales are the most important part of any business. Marketing and communications are never as effective as a strong sales team. Persuasive salesmen can sell as many units of an inferior product as average salesmen with better quality products. Our company should eliminate every department except the sales team.", "question": "Which one of the following is an assumption on which the businessman's argument depends?", "answers": "[\"The sales team's effectiveness depends on contributions from the other departments.\", 'Companies often separate their departments into distinct teams.', 'Businesses often have other departments besides marketing, communications, and sales.', 'The company would be better off with only a sales team.']", "label": 0 }, { "id": "train_1367", "context": "It is said that people should accept themselves as they are instead of being dissatisfied with their own abilities. But this is clearly a bad principle if the goal is a society whose citizens are genuinely happy, for no one can be genuinely happy if he or she is not pursuing personal excellence and is unwilling to undergo personal change of any kind.", "question": "Which one of the following is an assumption required by the argument?", "answers": "['People are justified in feeling content with themselves when they have achieved some degree of personal excellence.', 'People who are not dissatisfied with themselves are less likely than others to pursue personal excellence.', 'Happiness is not genuine unless it is based on something that is painful to obtain.', 'Those who are willing to change will probably find genuine happiness.']", "label": 1 }, { "id": "train_1368", "context": "Commentator: Although the present freshwater supply is adequate for today' s patterns of water use, the human population will increase substantially over the next few decades, drastically increasing the need for freshwater. Hence, restrictions on water use will be necessary to meet the freshwater needs of humankind in the not-too-distant future.", "question": "Which one of the following is an assumption required by the argument?", "answers": "['The freshwater supply will not increase sufficiently to meet the increased needs of humankind.', 'Humans will adapt to restrictions on the use of water without resorting to wasteful use of other natural resources.', 'The total supply of freshwater has not diminished in recent years.', 'No water conservation measure previously attempted yielded an increase in the supply of freshwater available for human use.']", "label": 0 }, { "id": "train_1369", "context": "Lenore: It is naive to think that historical explanations can be objective. In evaluating evidence, historians are always influenced by their national, political, and class loyalties. Victor: Still, the very fact that cases of biased thinking have been detected and sources of bias identified shows that there are people who can maintain objectivity.", "question": "Victor's response does not succeed as a rebuttal of Lenore's argument because his response", "answers": "['overlooks the possibility that those who detect and identify bias are themselves biased in some way', \"displays the same kind of biased thinking as that against which Lenore's argument is directed\", 'does not consider sources of bias in historical explanation other than those that are due to national, political, and class loyalties.', 'fails to provide examples of cases in which biased thinking has been detected and the source of that bias identified']", "label": 0 }, { "id": "train_1370", "context": "Psychology researchers observed that parents feel emotion while singing to their infants. The researchers hypothesized that this emotion noticeably affects the sound of the singing. To test this hypothesis the parents were recorded while singing to their infants and while singing with no infant present. They were instructed to make the two renditions as similar as possible. These recordings were then played for psychologists who had not observed the recordings being made. For 80 percent of the recordings, these psychologists were able to correctly identify, by listening alone, which recordings were of parents singing to their children. The researchers concluded that their hypothesis was correct.", "question": "Which one of the following, if true, would most strengthen the researchers' reasoning?", "answers": "['Most of the parents who participated in the study believed that the emotion they felt while singing to their infants affected their singing.', 'Parents displayed little emotion when singing with no child or adult present.', 'When a person feels emotion, that emotion provokes involuntary physiological responses that affect the vocal cords and lungs.', 'A separate study by the same researchers found that parents feel more emotion when singing to their own children than when singing to other children.']", "label": 2 }, { "id": "train_1371", "context": "Conscientiousness is high on most firms' list of traits they want in employees. Yet a recent study found that laid-off conscientious individuals are less likely to find jobs within five months than are their peers who shirked their workplace responsibilities.", "question": "Each of the following, if true, helps to resolve the apparent paradox above EXCEPT:", "answers": "['People who shirk their workplace responsibilities are less likely to keep the jobs they have, so there are more of them looking for jobs.', 'Finding a job is less urgent for the conscientious, because they tend to have larger savings.', 'Conscientious people tend to have a greater than average concern with finding the job most suited to their interests and abilities.', 'Resentment about having been laid off in spite of their conscientiousness leads some people to perform poorly in interviews.']", "label": 0 }, { "id": "train_1372", "context": "Letter to the editor: Your article was unjustified in criticizing environmentalists for claiming that more wolves on Vancouver Island are killed by hunters than are born each year. You stated that this claim was disproven by recent studies that indicate that the total number of wolves on Vancouver Island has remained roughly constant for 20 years. But you failed to account for the fact that, fearing the extinction of this wolf population, environmentalists have been introducing new wolves into the Vancouver Island wolf population for 20 years.", "question": "Which one of the following most accurately expresses the conclusion of the argument in the letter to the editor?", "answers": "['Environmentalists have been successfully maintaining the wolf population on Vancouver Island for 20 years.', 'As many wolves on Vancouver Island are killed by hunters as are born each year.', 'The recent studies indicating that the total number of wolves on Vancouver Island has remained roughly constant for 20 years were flawed.', \"The stability in the size of the Vancouver Island wolf population does not warrant the article's criticism of the environmentalists' claim.\"]", "label": 3 }, { "id": "train_1373", "context": "Motorcoach driver: Professional drivers spend much more time driving, on average, than do other people and hence are more competent drivers than are other, less experienced drivers. Ttherefore, the speed limit on major highways should not be reduced, because that action would have the undesirable effect of forcing some people who are now both law-abiding and competent drivers to break the law. Police officer: All drivers can drive within the legal speed limit if they wish, so it is not true to say that reducing the speed limit would be the cause of such illegal behavior.", "question": "The point at issue between the motorcoach driver and the police officer is whether", "answers": "['professional drivers are more competent drivers than are other, less experienced drivers', 'reducing the speed limit on major highways would cause some professional drivers to break the law', 'all drivers wish to drive within the speed limit', 'professional drivers will drive within the legal speed limit if that limit is reduced']", "label": 1 }, { "id": "train_1374", "context": "In response to requests made by the dairy industry, the government is considering whether to approve the synthetic hormone BST for use in dairy cows. BST increases milk production but also leads to recurring udder inflammation, decreased fertility, and symptoms of stress in cows who receive the hormone. All of these problems can be kept under control with constant veterinary care, but such levels of veterinary help would cost big farms far less per cow than they would small farms.", "question": "If the statements above are true, which one of the following claims is most strongly supported by them?", "answers": "['At the present time milk from cows raised on small farms is safer to drink than milk from cows raised on big farms.', 'The proportion of cows that suffer from udder inflammation, decreased fertility, and symptoms of stress is currently greater on big dairy farms than on small ones.', 'Owners of big farms stand to gain more from government approval of BST than do owners of small farms.', 'The government is unlikely to approve the synthetic hormone BST for use in cows.']", "label": 2 }, { "id": "train_1375", "context": "Emissions from automobiles that burn gasoline and automobiles that burn diesel fuel are threatening the quality of life on our planet, contaminating both urban air and global atmosphere. Ttherefore, the only effective way to reduce such emissions is to replace the conventional diesel fuel and gasoline used in automobiles with cleaner-burning fuels, such as methanol, that create fewer emissions.", "question": "Which one of the following is an assumption on which the argument depends?", "answers": "['Reducing the use of automobiles would not be a more effective means to reduce automobile emissions than the use of methanol.', 'Automobile emissions constitute the most serious threat to the global environment.', 'At any given time there is a direct correlation between the level of urban air pollution and the level of contamination present in the global atmosphere.', 'If given a choice of automobile fuels, automobile owners would not select gasoline over methanol.']", "label": 0 }, { "id": "train_1376", "context": "It would not be surprising to discover that the trade routes between China and the West were opened many centuries, even millennia, earlier than 200 B. C. , contrary to what is currently believed. After all, what made the Great Silk Road so attractive as a trade route linking China and the West -- level terrain, easily traversable mountain passes, and desert oases -- would also have made it an attractive route for the original emigrants to China from Africa and the Middle East, and this early migration began at least one million years ago.", "question": "That a migration from Africa and the Middle East to China occurred at least one million years ago figures in the above reasoning in which one of the following ways?", "answers": "['It is cited as conclusive evidence for the claim that trade links between China and the Middle East were established long before 200 B. C.', 'It is offered as evidence against the claim that trade routes between China and Africa preceded those eventually established between China and the Middle East.', 'It is offered as evidence in support of the claim that trade routes between China and the West could easily have been established much earlier than is currently believed.', 'It is the main conclusion that the argument attempts to establish about intercourse between China and the West.']", "label": 2 }, { "id": "train_1377", "context": "Although early jazz music featured a great deal of improvisation, this improvisation did not stray too far from the rather simple, catchy melodies upon which it was based. So, however interesting it may be, later music featuring improvisation that strays far from the melody ought not to be classified as jazz.", "question": "Which one of the following, if true, most weakens the argument?", "answers": "['Some of the later music featuring improvisation was performed by artists who had been jazz musicians earlier in their careers.', 'For a piece of music to be classified as jazz, it must feature some amount of improvisation .', 'The later music featuring improvisation has much more in common with early jazz than with any other type of music.', 'Many types of music other than jazz feature a great deal of improvisation.']", "label": 2 }, { "id": "train_1378", "context": "Bacteria that benefit human beings when they are present in the body are called commensals. The bacterium Helicobacter pylori plays a primary role in the development of stomach ulcers. But since stomach ulcers occur in less than 10 percent of those harboring H. pylori, and since it allegedly strengthens immune response, many scientists now consider it a commensal. But this is surely misguided. Only about 10 percent of the people who harbor Mycobacter tuberculosis -- a bacterium that can cause tuberculosis -- get sick from it, yet no one would call M. tuberculosis a commensal.", "question": "Which one of the following, if true, most seriously undermines the argument's reasoning?", "answers": "['There are more people who harbor M. tuberculosis than people who harbor H. pylori.', 'People who harbor M. tuberculosis derive no benefit from its presence.', 'Stomach ulcers caused by H. pylori and tuberculosis can both be effectively treated with antibiotics.', 'Cases of tuberculosis usually last longer than ulcers caused by H. pylori.']", "label": 1 }, { "id": "train_1379", "context": "Working residents of Springfield live, on average, farther from their workplaces than do working residents of Rorchester. Thus, one would expect that the demand for public transportation would be greater in Springfield than in Rorchester. However, Springfield has only half as many bus routes as Rorchester.", "question": "Each of the following, if true, contributes to a resolution of the apparent discrepancy described above EXCEPT:", "answers": "['Three-fourths of the Springfield workforce is employed at the same factory outside the city limits.', 'Springfield has a larger population than Rorchester does.', 'The average number of cars per household is higher in Springfield than in Rorchester.', 'Buses in Springfield run more frequently and on longer routes than in Rorchester.']", "label": 1 }, { "id": "train_1380", "context": "Historian: The standard \"QWERTY\" configuration of the keys on typewriters and computer keyboards was originally designed to be awkward and limit typing speed. This was because early typewriters would jam frequently if adjacent keys were struck in quick succession. Experiments have shown that keyboard configurations more efficient than QWERTY can double typing speed while tremendously reducing typing effort. However, the expense and inconvenience of switching to a new keyboard configuration prevent any configuration other than QWERTY from attaining widespread use.", "question": "Which one of the following is most strongly supported by the historian's statements?", "answers": "['If the keyboard had been designed for computers, then it would not have been designed to limit typing speed.', 'The benefit to society that would result from switching to a keyboard configuration other than QWERTY is significantly greater than the overall cost of such a switch.', 'Most people who have tried typing with non- QWERTY keyboards have typed significantly more quickly using those keyboards than they usually have done using QWERTY keyboards.', 'Early QWERTY typewriters were less likely to jam than were at least some more recent typewriters if adjacent keys were struck in quick succession.']", "label": 0 }, { "id": "train_1381", "context": "Many environmentalists have urged environmental awareness on consumers, saying that if we accept moral responsibility for our effects on the environment, then products that directly or indirectly harm the environment ought to be avoided. Unfortunately it is usually impossible for consumers to assess the environmental impact of a product, and thus impossible for them to consciously restrict their purchases to environmentally benign products. Because of this impossibility there can be no moral duty to choose products in the way these environmentalists urge, since __.", "question": "Which one of the following principles provides the most appropriate completion for the argument?", "answers": "['the morally right action is always the one whose effects produce the least total harm', 'a moral duty to perform an action is never based solely on the effects the action will have on other people', 'a person cannot possibly have a moral duty to do what he or she is unable to do', 'where a moral duty exists, it supersedes any legal duty and any other kind of duty']", "label": 2 }, { "id": "train_1382", "context": "Principle: If an insurance policy is written in such a way that a reasonable person seeking insurance would not read it thoroughly before signing it, then the reasonable expectations of the policyholder concerning the policy' s coverage should take legal precedence over specific language in the written policy itself. Application: The insurance company should be required to cover the hail damage to Celia' s car, even though specific language in the written policy Celia signed excluded coverage for hail damage.", "question": "Which one of the following, if true, most justifies the above application of the principle?", "answers": "['Celia did not read the written insurance policy thoroughly before signing it, and a reasonable person in her position would not have done so either.', 'Celia is a reasonable person, and she expected the insurance policy to cover hail damage to her car.', \"Given the way it was written, a reasonable person would not have read Celia's insurance policy thoroughly before signing it, and Celia reasonably expected the policy to cover hail damage.\", 'The insurance policy that Celia signed was written in such a way that a reasonable person would not read it thoroughly before signing it, but Celia did read the policy thoroughly before signing it.']", "label": 2 }, { "id": "train_1383", "context": "If Thompson appeals to moderates, his most ardent supporters will desert him and he will not have enough votes to win the election. However, if he does not appeal to moderates, these moderates will vote for his opponent and he will not have enough votes to win the election. Either way, Thompson is bound to lose the election.", "question": "Which one of the following arguments is most similar in its reasoning to the argument above?", "answers": "['If the company decides to relocate, it will lose a number of employees, since many employees are unwilling to move. But if the company decides to stay where it is now, it will lose a number of employees, since many employees are unhappy in this location. So the company will lose employees no matter what it decides.', 'If the city council approves a small increase in property taxes, there will be enough money to restore the city swimming pools in Rice Park. If the city council approves a large increase in property taxes, there will be enough money to restore the city swimming pools and build a recreational center in Rice Park. Either way, the city will make progress in improving the facilities in Rice Park.', \"If Chen supports building a new art center, the center will be built. If Chen opposes building a new art center, the center will not be built. So whether or not a new art center will be built depends on Chen's decision.\", \"If Madsen decides to support the proposed shopping mall, she will make an effective spokesperson for the project. But if Madsen decides to oppose the shopping mall, there will be no difficulty in finding another effective spokesperson. So the project's success does not depend on Madsen's decision.\"]", "label": 0 }, { "id": "train_1384", "context": "Whenever an artist endowed with both a high level of artistic skill and a high degree of creativity combines these two abilities in the process of creating an artwork, the resulting product is a great work of art. Moreover, no work of art can be great unless both of these elements are combined in its execution. Thus, great works of art are necessarily rare.", "question": "Which one of the following is an assumption required by the argument?", "answers": "['Not every artist possesses a high level of artistic skill.', 'An artist endowed with a high degree of creativity and a high level of artistic skill will necessarily produce great works of art.', 'A high degree of creativity and a high level of artistic skill are seldom combined in the creation of a work of art.', 'Anyone endowed with both a high level of artistic skill and a high degree of creativity will produce only a few great works of art.']", "label": 2 }, { "id": "train_1385", "context": "Only a minority of those who engage in political action do so out of a sense of social justice. Ttherefore, some people who have a sense of social justice do not engage in political action.", "question": "Which one of the following uses flawed reasoning most similar to that used in the argument above?", "answers": "['Some corporations only feign a concern for the environment when they advertise a product as environmentally friendly. Thus, no corporation has a genuine concern for the environment.', 'Most scholars are not motivated by a desire to win prestigious academic prizes. Thus, some of those who want to win prestigious academic prizes are not scholars.', 'Only a small percentage of the profits that companies make are directly attributable to good management decisions. Thus, even companies that are managed badly will usually turn a profit.', \"Some parents show no interest in the curricula used in the schools that their children attend. Thus, some of the decisions regarding school curricula should be made without regard for the wishes of the children's parents.\"]", "label": 1 }, { "id": "train_1386", "context": "Roses always provide a stunning display of color, but only those flowers that smell sweet are worth growing in a garden. Some roses have no scent.", "question": "Which one the following conclusions can be properly drawn from the passage?", "answers": "['Some roses which smell sweet are not worth growing in a garden.', 'No sweet-smelling flower is worth growing in a garden unless it provides a stunning display of color.', 'All flowers with no scent provide a stunning display of color.', 'Some flowers which provide a stunning display of color are not worth growing in a garden.']", "label": 3 }, { "id": "train_1387", "context": "Leslie lost her job as a cashier at Locally Sourced Food Market because the store went out of business. Two days later, Randy' s Ammunition Warehouse closed down for good in the same shopping center. Ttherefore, the Locally Sourced Food Market' s closing clearly caused Randy' s to close.", "question": "The flawed reasoning in which of the following arguments most mirrors the flawed reasoning presented in the argument above:", "answers": "['Tammy overslept and missed her early class. That same day, she experienced car trouble and missed her night class. Ttherefore, Tammy did not go to school today.', 'Theresa lost her job on Monday, but she received an unsolicited offer to consult for a hedge fund that same day. Ttherefore, losing one job led to another one.', 'Angela recently ran into some rotten luck. Last week she fell off her skateboard, and two days later, she crashed her car. Ttherefore, Angela needs to recover from her injuries.', \"Tito's Taco Shop recently closed down due to lack of foot traffic. Nearby Bubba's Burrito Bowls also closed down later that month for the same reason. Ttherefore, a lack of foot traffic caused both businesses to close.\"]", "label": 1 }, { "id": "train_1388", "context": "Organization president: The stationery and envelopes used in all of the mailings from our national headquarters are made from recycled paper, and we never put anything but letters in the envelopes. When the envelopes have windows, these windows are also made from recycled material. Ttherefore the envelopes, and thus these mailings, are completely recyclable.", "question": "Which one of the following is an assumption on which the organization president's argument depends?", "answers": "[\"The envelopes and stationery used in the organization's are always recycled.\", \"The mailings from the organization's national headquarters always use envelopes that have windows.\", 'The organization sends mailings only from its national headquarters.', 'The envelope windows made from recycled material are recyclable.']", "label": 3 }, { "id": "train_1389", "context": "In modern deep-diving marine mammals, such as whales, the outer shell of the bones is porous. This has the effect of making the bones light enough so that it is easy for the animals to swim back to the surface after a deep dive. The outer shell of the bones was also porous in the ichthyosaur, an extinct prehistoric marine reptile. We can conclude from this that ichthyosaurs were deep divers.", "question": "Which one of the following, if true, most weakens the argument?", "answers": "['Some deep-diving marine species must surface after dives but do not have bones with porous outer shells.', 'In most modern and prehistoric marine reptile species that are not deep divers, the outer shell of the bones is porous.', 'In addition to the porous outer shells of their bones, whales have at least some characteristics suited to deep diving for which there is no clear evidence whether these were shared by ichthyosaurs.', 'There is evidence that the bones of ichthyosaurs would have been light enough to allow surfacing even if the outer shells were not porous.']", "label": 1 }, { "id": "train_1390", "context": "Studies of the political orientations of 1, 055 college students revealed that the plurality of students in an eastern, big-city, private university was liberal, whereas in a state-supported, southern college, the plurality was conservative. Orientations were independent of the student' s region of origin, and the trends were much more pronounced in seniors than in beginning students.", "question": "Which of the following hypotheses is best supported by the observations stated above?", "answers": "['The political orientations of college seniors depend significantly on experiences they have had while in college.', 'College students tend to become more conservative politically as they become older and are confronted with pressures for financial success.', 'The political orientations of college students are more similar to the political orientations of their parents when the students start college than when the students are seniors.', \"Whether their college is state-supported or private is the determining factor in college students' political orientations.\"]", "label": 0 }, { "id": "train_1391", "context": "In the United States proven oil reserves -- the amount of oil considered extractable from known fields -- are at the same level as they were ten years ago. Yet over this same period no new oil fields of any consequence have been discovered, and the annual consumption of domestically produced oil has increased.", "question": "Which one of the following, if true, best reconciles the discrepancy described above?", "answers": "['Oil exploration in the United States has slowed due to increased concern over the environmental impact of such exploration.', 'Conservation measures have lowered the rate of growth of domestic oil consumption from what it was a decade ago.', 'Over the past decade the annual consumption of imported oil has increased more rapidly than that of domestic oil in the United States.', 'Due to technological advances over the last decade, much oil previously considered unextractable is now considered extractable.']", "label": 3 }, { "id": "train_1392", "context": "Although Samantha likes both oolong and green tea, none of her friends likes both. However, all of her friends like black tea.", "question": "If the statements above are true, each of the following could be true EXCEPT:", "answers": "[\"None of Samantha's friends likes green tea.\", 'Samantha likes black tea.', \"One of Samantha's friends likes all the kinds of teas that Samantha likes.\", \"One of Samantha's friends likes neither oolong nor green tea.\"]", "label": 2 }, { "id": "train_1393", "context": "Attorney: I ask you to find Mr. Smith guilty of assaulting Mr. Jackson. Regrettably, there were no eyewitnesses to the crime, but Mr. Smith has a violent character: Ms. Lopez testified earlier that Mr. Smith, shouting loudly, had threatened her. Smith never refuted this testimony.", "question": "The attorney's argument is fallacious because it reasons that", "answers": "['having a violent character is not necessarily associated with the commission of violent crimes', 'aggressive behavior is not a sure indicator of a violent character', \"Smith's testimony is unreliable since he is loud and aggressive\", 'since Smith never disproved the claim that he threatened Lopez, he did in fact threaten her']", "label": 3 }, { "id": "train_1394", "context": "Extract from lease: The tenant should record all preexisting damage on the preexisting damage list, because the tenant need not pay for preexisting damage recorded there. The tenant must pay for damage that was not recorded on the preexisting damage list, except for any damage caused by a circumstance beyond the tenant' s control.", "question": "In which one of the following instances does the extract from the lease most strongly support the view that the tenant is not required to pay for the damage?", "answers": "[\"a tear in the linoleum that was not preexisting but that was caused by one of the tenant's children\", \"a crack in a window caused by a factor beyond the tenant's control and not recorded on the preexisting damage list\", \"a hole in the wall that was not recorded on the preexisting damage list and that was the result of an event within the tenant's control\", 'a missing light fixture that was present when the tenant moved in but was later removed by the tenant']", "label": 1 }, { "id": "train_1395", "context": "Sharon heard her favorite novelist speak out against a political candidate that Sharon has supported for years. As a result, Sharon' s estimation of the novelist declined but her estimation of the candidate did not change.", "question": "The situation described above conforms most closely to which one of the following principles?", "answers": "['Artists who speak out on political matters will have influence only among their most dedicated fans.', 'Someone who hears testimony that contradicts a long-standing opinion will generally entertain doubts about the source of the testimony rather than the correctness of the opinion.', 'Artists should limit their public political statements to issues that are somehow related to the arts.', 'A political statement from an artist should be considered only if the artist has established a reputation for being an honest and knowledgeable observer of politics.']", "label": 1 }, { "id": "train_1396", "context": "Gene splicing can give rise to new varieties of farm animals that have only a partially understood genetic makeup. In addition to introducing the genes for whichever trait is desired, the technique can introduce genes governing the production of toxins or carcinogens, and these latter undesirable traits might not be easily discoverable.", "question": "The statements above, if true, most strongly support which one of the following?", "answers": "['Gene splicing to produce new varieties of farm animals should be used cautiously.', 'Most new varieties of farm animals produced by gene splicing will develop cancer.', 'Gene splicing is not effective as a way of producing new varieties of farm animals.', 'All toxin production is genetically controlled.']", "label": 0 }, { "id": "train_1397", "context": "Critic: In her presentation of important works of art in her art history textbook, Waverly claims to have presented only objective accounts: \"I have sought neither to advocate nor to denigrate what I included. \" In writing about art, a pretense of objectivity never succeeds: clearly, Waverly writes much better about art she likes than about art to which she is indifferent.", "question": "The critic's statements, if true, most strongly support which one of the following?", "answers": "['Waverly has not succeeded in her intended objectivity about works of art discussed in her textbook.', 'Waverly does not really believe that objectivity is a desirable trait in an art history textbook.', 'Waverly has only included works of art that she has strong opinions about in her textbook.', 'Waverly believes that a historian of art should not prefer certain works of art to other works of art.']", "label": 0 }, { "id": "train_1398", "context": "Some people believe that good health is due to luck. However, studies from many countries indicate a strong correlation between good health and high educational levels. Thus research supports the view that good health is largely the result of making informed lifestyle choices.", "question": "The reasoning in the argument is most vulnerable to criticism on the grounds that the argument", "answers": "['overlooks the possibility that the same thing may causally contribute both to education and to good health', 'presumes, without providing justification, that only highly educated people make informed lifestyle choices', 'overlooks the possibility that people who make informed lifestyle choices may nonetheless suffer from inherited diseases', 'does not acknowledge that some people who fail to make informed lifestyle choices are in good health']", "label": 0 }, { "id": "train_1399", "context": "Insects can see ultraviolet light and are known to identify important food sources and mating sites by sensing the characteristic patterns of ultraviolet light that these things reflect. Insects are also attracted to Glomosus spiderwebs, which reflect ultraviolet light. Thus, insects are probably attracted to these webs because of the specific patterns of ultraviolet light that these webs reflect.", "question": "Which one of the following, if true, most strongly supports the argument?", "answers": "['When Drosophila fruit flies were placed before a Glomosus web and a synthetic web of similar pattern that also reflected ultraviolet light and both webs were illuminated with white light containing an ultraviolet component, many of the fruit flies flew to the Glomosus web.', 'When Drosophila fruit flies were placed before two Glomosus webs, one illuminated with white light containing an ultraviolet component and one illuminated with white light without an ultraviolet component, the majority flew to the ultraviolet reflecting web.', 'When webs of many different species of spider were illuminated with a uniform source of white light containing an ultraviolet component, many of these webs did not reflect the ultraviolet light.', 'When the silks of spiders that spin silk only for lining burrows and covering eggs were illuminated with white light containing an ultraviolet component, the silks of these spiders reflected ultraviolet light.']", "label": 1 }, { "id": "train_1400", "context": "A decade can be characterized just like an individual. Decades have specific character and unique quirks. They all start with a departure from the past decade and develop their personality throughout their timespans. Just as people in their twilight years start to look back on the events of their lives, people at decade' s end__.", "question": "Which one of the following most logically completes the argument?", "answers": "['Focus on what the next decade will bring.', 'Throw a big party.', 'Become very interested in evaluating the events of the last decade.', 'Reminisce about their lives.']", "label": 2 }, { "id": "train_1401", "context": "When interviewing job candidates, personnel managers not only evaluate a candidate' s work experience and educational background but also inquire about hobbies. Personnel managers try to justify these inquiries by noting that the enthusiasm someone shows for a hobby may well carry over to enthusiasm for a job. But such enthusiasm may also indicate that the candidate is less concerned with work than with play. Ttherefore personnel managers should not inquire about a candidate' s hobbies.", "question": "The argument is flawed because it overlooks each of the following possibilities EXCEPT:", "answers": "['Inquiries about a hobby may put candidates at ease, eliciting more honest responses about important questions.', 'Candidates who have no hobbies may pretend that they have one when asked in an interview.', \"A candidate's involvement in particular hobbies may indicate a capacity to make longterm commitments.\", 'Having certain kinds of hobbies may indicate that a candidate has good organizational skills.']", "label": 1 }, { "id": "train_1402", "context": "Ferber's syndrome, a viral disease that frequently affects cattle, is transmitted to these animals through infected feed. Even though chickens commercially raised for meat are often fed the type of feed identified as the source of infection in cattle, Ferber's syndrome is only rarely observed in chickens. This fact, however, does not indicate that most chickens are immune to the virus that causes Ferber's syndrome, since__.", "question": "Which of the following most logically completes the argument?", "answers": [ "resistance to some infectious organisms such as the virus that causes Ferber's syndrome can be acquired by exposure to a closely related infectious organism", "chickens and cattle take more than a year to show symptoms of Ferber's syndrome, and chickens commercially raised for meat, unlike cattle, are generally brought to market during the first year of life", "the type of feed liable to be infected with the virus that causes Ferber's syndrome generally constitutes a larger proportion of the diet of commercially raised chickens than of commercially raised cattle", "Ferber's syndrome has been found in animals that have not been fed the type of feed liable to be contaminated with the virus that can cause the disease" ], "label": 1 }, { "id": "train_1403", "context": "Demand levels for some categories of products are closely interdependent. When one type of a product can serve as a complement for another product, the levels of demand for these goods are positively related. An increase in the demand for one product usually increases the demand for its complementary product. Similarly, a decrease in the demand for one good causes a decrease in the demand for its complementary product.", "question": "Which of the following statements provides an example of the relationship between product demands described above?", "answers": "[\"A decrease in the demand for CD players results in a decrease in the demand for CD's.\", 'An increase in the demand for butter results in an increase in the demand for margarine.', 'A decrease in the demand for cable television results in an increase in the demand for satellite antennas.', 'An increase in the demand for toothpaste results in an increase in the demand for cellular phones.']", "label": 0 }, { "id": "train_1404", "context": "Since it has become known that several of a bank' s top executives have been buying shares in their own bank , the bank' s depositors, who had been worried by rumors that the bank faced impending financial collapse, have been greatly relieved. They reason that, since top executives evidently have faith in the bank's financial soundness, those worrisome rumors must be false. Such reasoning might well be overoptimistic, however, since corporate executives have been known to buy shares in their own company in a calculated attempt to dispel negative rumors about the company' s health .", "question": "In the argument given, the two boldfaced portions play which of the following roles?", "answers": "['The first provides evidence in support of the main conclusion of the argument; the second states that conclusion.', 'The first describes the circumstance that the argument as a whole seeks to explain; the second provides evidence in support of the explanation that the argument seeks to establish.', 'The first describes the circumstance that the argument as a whole seeks to explain; the second gives the explanation that the argument seeks to establish.', 'The first describes evidence that has been taken as supporting a conclusion; the second gives a reason for questioning that support.']", "label": 3 }, { "id": "train_1405", "context": "Rail Executive: Five years ago we discontinued train service between Lamberton and its suburbs because low ridership caused total fares collected to be substantially lower than the cost of operating the service. It is true that recent population growth in the suburban communities suggests increased potential ridership. Nevertheless, since most of the newer residents own automobiles, restoring the train service this year would still result in serious deficits", "question": "Which of the following , if true casts the most serious doubt on the rail executive's argument?", "answers": "['Recently there have been sharp increases in parking fees in Lamberton and in highway tolls on major commuter routes', 'A survey conducted last year showed that few people living in Lamberton or its suburbs support increased public funding for mass transportation', \"Most of the new residents of Lamberton's suburbs do not need to travel into Lamberton regularly for shopping or recreation\", 'If train service were restored this year with increased hours of service, there would not be an increase in total ridership']", "label": 0 }, { "id": "train_1406", "context": "Most of Earth' s surface is ocean. The ocean floor is inaccessible for extensive research without equipment of greater technological sophistication than is currently available. It must ttherefore be true that scientists know less about the ocean floor environment than about almost any other environment on Earth.", "question": "Which of the following, if true, provides the most support for the conclusion?", "answers": "['Strong water currents circulate on the ocean floor, but the general pattern of their movement is not so well understood as is the pattern of air currents that circulate over land.', 'Many mountain ranges lie entirely beneath the ocean surface, yet new underwater surveying equipment has produced three-dimensional charts of them that are as accurate as those available for mountain ranges on land.', 'Very few people have seen detailed maps of extended regions of the ocean floor, even though such maps are available in almost all large libraries.', 'Animals living on the ocean floor must be able to withstand water pressure that is far greater than the atmospheric pressure with which land animals live.']", "label": 0 }, { "id": "train_1407", "context": "J. J. Thomson, the discoverer of the electron and a recipient of the Nobel Prize in physics, trained many physicists, among them seven Nobel Prize winners, 32 fellows of the Royal Society of London, and 83 professors of physics. This shows that the skills needed for creative research can be taught and learned.", "question": "Which one of the following is an assumption on which the argument depends?", "answers": "['At least one of the eminent scientists trained by J. J. Thomson was not a creative researcher before coming to study with him.', 'Scientists who go on to be the most successful researchers often receive their scientific education in classes taught by renowned research scientists.', 'J. J. Thomson was an internationally known physicist, and scientists came from all over the world to work with him.', 'All the scientists trained by J. J. Thomson were renowned for their creative scientific research.']", "label": 0 }, { "id": "train_1408", "context": "A naturally occurring deposit of radioactive material similar in composition to the dangerous waste produced by applications of nuclear technology has been found deep within the Earth' s crust. No ill effects have been traced to this naturally occurring deposit. Since engineers have now developed a way to store manufactured nuclear waste as deeply within the Earth' s crust as the recently found natural radioactive deposit, the safe disposal of nuclear waste can now be initiated.", "question": "Which one of the following, if true, most seriously weakens the argument?", "answers": "['Continued production of nuclear waste at the current rate will eventually outstrip available capacity for safe storage.', 'The total amount of dangerous waste produced annually has been declining in recent years.', 'It cannot currently be determined what the geological conditions are under which the naturally occurring radioactive deposit has apparently been safely contained.', 'Not one of the current methods used to dispose of nuclear waste has proved to be completely safe.']", "label": 2 }, { "id": "train_1409", "context": "Certain companies require their managers to rank workers in the groups they supervise from best to worst, giving each worker a unique ranking based on job performance. The top 10 percent of the workers in each group are rewarded and the bottom 10 percent are penalized or fired. But this system is unfair to workers. Good workers could receive low rankings merely because they belong to groups of exceptionally good workers. Furthermore, managers often give the highest rankings to workers who share the manager' s interests outside of work.", "question": "Which one of the following most accurately expresses the conclusion drawn in the argument?", "answers": "['Under the ranking system, the top 10 percent of the workers in each group are rewarded and the bottom 10 percent are penalized or fired.', 'The ranking system is not a fair way to determine penalties or rewards for workers.', 'Some companies require their managers to give unique rankings to the workers they supervise.', \"Managers often give the highest rankings to workers who share the manager's outside interests.\"]", "label": 1 }, { "id": "train_1410", "context": "Over the past twenty-five years the introduction of laborsaving technologies has greatly reduced the average amount of time a worker needs to produce a given output, potentially both reducing the number of hours each worker works each week and increasing workers' leisure time correspondingly. The average amount of leisure time per worker, however, has increased at only half the rate at which the average hourly output per worker has grown.", "question": "If the statements above are true, which one of the following is most strongly supported by them?", "answers": "['The average hourly output per worker has not risen as much as had been anticipated when modern labor-saving technologies were first introduced.', 'The percentage of the population that is in the work force has grown over the past twenty-five years.', 'Twenty-five years ago the average weekly output per worker was less than it is today.', 'Workers, on average, spend more money on leisure activities today than they did twenty-five years ago.']", "label": 2 }, { "id": "train_1411", "context": "Good hunter\" and \"bad hunter\" are standard terms in the study of cats. Good hunters can kill prey that weigh up to half their body weight. All good hunters have a high muscle-to-fat ratio. Most wild cats are good hunters, but some domestic cats are good hunters as well.", "question": "If the statements above are true, which one of the following must also be true?", "answers": "['All cats that have a high muscle-to-fat ratio can kill prey that weigh up to half their body weight.', 'Some cats that have a high muscle-to-fat ratio are domestic.', 'A smaller number of domestic cats than wild cats have a high muscle-to-fat ratio.', 'All cats that are bad hunters have a low muscle-to-fat ratio.']", "label": 1 }, { "id": "train_1412", "context": "English and the Austronesian language Mbarbaram both use the word \"dog\" for canines. These two languages are unrelated, and since speakers of the two languages only came in contact with one another long after the word \"dog\" was first used in this way in either language, neither language could have borrowed the word from the other. Thus this case shows that sometimes when languages share words that are similar in sound and meaning the similarity is due neither to language relatedness nor to borrowing.", "question": "The argument requires that which one of the following be assumed?", "answers": "['Usually when two languages share a word, those languages are related to each other.', 'English and Mbarbaram share no words other than \"dog. \"', 'There is no third language from which both English and Mbarbaram borrowed the word \"dog. \"', 'If two unrelated languages share a word, speakers of those two languages must have come in contact with one another at some time.']", "label": 2 }, { "id": "train_1413", "context": "The Frauenkirche in Dresden, a historic church destroyed by bombing in World War II, has been reconstructed to serve as a place for church services and cultural events. The foundation doing the reconstruction took extraordinary care to return the church to its original form. It is a puzzle, then, why the foundation chose not to rebuild the eighteenth-century baroque organ originally designed for the church and instead built a modern organ, even though a donor had offered to pay the full cost of rebuilding the original.", "question": "Which one of the following, if true, would most help to resolve the puzzle described above?", "answers": "['An eighteenth-century baroque organ cannot adequately produce much of the organ music now played in church services and concerts.', 'By the time the church was destroyed in World War II, the eighteenth-century baroque organ had been modified several times.', 'The organ originally designed for the church had some features that modern organs lack.', 'The donation for rebuilding the original eighteenth-century baroque organ was designated for that purpose alone.']", "label": 0 }, { "id": "train_1414", "context": "Psychotherapy has been described as a form of moral coercion. However, when people are coerced, their ability to make choices is restricted, and the goal of psychotherapy is to enhance people' s ability to make choices. Hence, psychotherapy cannot possibly be a form of coercion.", "question": "Which one of the following describes a flaw in the argument?", "answers": "['The goals of psychotherapy are taken to justify any means that are used to achieve those goals.', 'Psychotherapy is unfairly criticized for having a single goal, rather than having many complex goals.', 'It offers no argument to show that moral coercion is always undesirable.', 'No allowance is made for the fact that the practice or results of psychotherapy might run counter to its goals.']", "label": 3 }, { "id": "train_1415", "context": "The new perfume Aurora smells worse to Joan than any comparably priced perfume, and none of her friends likes the smell of Aurora as much as the smell of other perfumes. However, she and her friends must have a defect in their sense of smell, since Professor Jameson prefers the smell of Aurora to that of any other perfume and she is one of the world' s foremost experts on the physiology of smell.", "question": "The reasoning is flawed because it", "answers": "['misrepresents the position against which it is directed', 'calls into question the truthfulness of the opponent rather than addressing the point at issue', 'ignores the well-known fact that someone can prefer one thing to another without liking either very much', 'makes an illegitimate appeal to the authority of an expert']", "label": 3 }, { "id": "train_1416", "context": "Networks of blood vessels in bats' wings serve only to disperse heat generated in flight. This heat is generated only because bats flap their wings. Thus paleontologists' recent discovery that the winged dinosaur Sandactylus had similar networks of blood vessels in the skin of its wings provides evidence for the hypothesis that Sandactylus flew by flapping its wings, not just by gliding.", "question": "In the passage, the author develops the argument by", "answers": "['speculating about how structures observed in present-day creatures might have developed from similar structures in creatures now extinct', 'using an analogy with a known phenomenon to draw a conclusion about an unknown phenomenon', 'reinterpreting evidence that had been used to support an earlier theory', 'forming the hypothesis that best explains several apparently conflicting pieces of evidence']", "label": 1 }, { "id": "train_1417", "context": "Raising the tax rate on essential goods -- a traditional means of increasing government revenues -- invariably turns low- and middle-income taxpayers against the government. Hence government officials have proposed adding a new tax on purchases of luxury items such as yachts, private planes, jewels, and furs. The officials claim that this tax will result in a substantial increase in government revenues while affecting only the wealthy individuals and corporations who can afford to purchase such items.", "question": "The answer to which one of the following questions would be the most relevant in evaluating the accuracy of the government officials' prediction?", "answers": "['Will luxury goods be taxed at a higher rate than that at which essential goods are currently taxed?', 'Will the revenues generated by the proposed tax be comparable to those that are currently being generated by taxes on essential goods?', 'Will purchases of luxury items by corporations account for more of the revenue generated by the proposed tax than will purchases of luxury items by wealthy individuals?', 'Will sales of the luxury items subject to the proposed tax occur at current rates once the proposed tax on luxury items has been passed?']", "label": 3 }, { "id": "train_1418", "context": "Seemingly inconsequential changes in sea temperature due to global warming eventually result in declines in fish and seabird populations. A rise of just two degrees prevents the vertical mixing of seawater from different strata. This restricts the availability of upwelling nutrients to phytoplankton. Since zooplankton, which feed upon phytoplankton, feed the rest of the food chain, the declines are inevitable.", "question": "Which one of the following most accurately describes the role played in the argument by the statement that zooplankton feed upon phytoplankton?", "answers": "['It helps show how global temperature changes affect larger sea animals indirectly.', 'It is intended to provide an example of the ways in which the vertical mixing of seawater affects feeding habits.', 'It is offered in support of the idea that global warming poses a threat to all organisms.', 'It is offered as one reason that global warming must be curtailed.']", "label": 0 }, { "id": "train_1419", "context": "To date, most of the proposals that have been endorsed by the Citizens League have been passed by the city council. Thus, any future proposal that is endorsed by the Citizens League will probably be passed as well.", "question": "The pattern of reasoning in which one of the following arguments is most similar to that in the argument above?", "answers": "['Most of the individual trees growing on the coastal islands in this area are deciduous. Ttherefore, most of the tree species on these islands are probably deciduous varieties.', 'Most of the stone artifacts that have been found at the archaeological site have been domestic tools. Thus, if the next artifact found at the site is made of stone, it will probably be a domestic tool.', \"Most of the entries that were received after the deadline for last year's photography contest were rejected by the judges' committee. Thus, the people whose entries were received after the deadline last year will probably send them in well before the deadline this year.\", 'Most of the editors who have worked for the local newspaper have not been sympathetic to local farmers. Thus, if the newspaper hires someone who is sympathetic to local farmers, they will probably not be hired as an editor.']", "label": 1 }, { "id": "train_1420", "context": "The official listing of an animal species as endangered triggers the enforcement of legal safeguards designed to protect endangered species, such as tighter animal export and trade restrictions and stronger antipoaching laws. Nevertheless, there have been many cases in which the decline in the wild population of a species was more rapid after that species was listed as endangered than before it was so listed.", "question": "Which one of the following, if true, does most to account for the increase in the rate of population decline described above?", "answers": "[\"Public campaigns to save endangered animal species often focus only on those species that garner the public's affection.\", \"Poachers find it progressively more difficult to locate animals of a particular species as that species' population declines.\", 'Animals are more desirable to collectors when they are perceived to be rare.', 'The process of officially listing a species as endangered can take many years.']", "label": 2 }, { "id": "train_1421", "context": "Mel: The official salary for judges has always been too low to attract the best candidates to the job. The legislature' s move to raise the salary has done nothing to improve the situation, because it was coupled with a ban on receiving money for lectures and teaching engagements. Pat: No, the raise in salary really does improve the situation. Since very few judges teach or give lectures, the ban will have little or no negative effect.", "question": "Pat's response to Mel is inadequate in that it", "answers": "[\"simply denies Mel's claim without putting forward any evidence in support of that denial\", 'assumes that changes that benefit the most able members of a group necessarily benefit all members of that group.', 'mistakenly takes the cause of a certain change to be an effect of that change', 'attempts to assess how a certain change will affect potential members of a group by providing evidence about its effect on the current members.']", "label": 3 }, { "id": "train_1422", "context": "The milk of many mammals contains cannabinoids, substances that are known to stimulate certain receptors in the brain. To investigate the function of cannabinoids, researchers injected newborn mice with a chemical that is known to block cannabinoids from reaching their receptors in the brain. The injected mice showed far less interest in feeding than normal newborn mice do. Ttherefore, cannabinoids probably function to stimulate the appetite.", "question": "Which of the following is an assumption on which the argument depends?", "answers": "[\"Newborn mice do not normally ingest any substance other than their mothers' milk.\", 'The chemical that blocks cannabinoids from stimulating their brain receptors does not independently inhibit the appetite.', 'The mothers of newborn mice do not normally make any effort to encourage their babies to feed.', \"Cannabinoids are the only substances in mammals' milk that stimulate the appetite.\"]", "label": 1 }, { "id": "train_1423", "context": "Consumer advocate: Ephedra is a naturally occurring compound that some people use as a weight-loss aid. Recently, the government prohibited the sale of dietary supplements containing ephedra on the grounds that ephedra has been shown to have grave side effects. This prohibition is unreasonable. Echinacea is another natural compound that has been shown to have side effects, yet echinacea is widely available at health food stores.", "question": "The consumer advocate's argument depends on which of the following assumptions?", "answers": "['The side effects of echinacea and ephedra are comparably serious.', 'All natural compounds are safe for human consumption.', 'The government should not hav the authority to prohibit natural compounds.', 'Before the prohibition, ephedra had been available in health food stores.']", "label": 0 }, { "id": "train_1424", "context": "Astronomers have found new evidence that the number of galaxies in the universe is not 10 billion, as previously believed, but 50 billion. This discovery will have an important effect on theories about how galaxies are formed. But even though astronomers now believe 40 billion more galaxies exist, many astronomers' estimates of the universe' s total mass remain virtually unchanged.", "question": "Which one of the following, if true, does most to explain why the estimates remain virtually unchanged?", "answers": "['The overwhelming majority of galaxies are so far from Earth that their mass can be only roughly estimated.', 'The number of galaxies that astronomers believe exist tends to grow as the instruments used to detect galaxies become more sophisticated.', \"The mass of galaxies is thought to make up only a tiny percentage of the universe's total mass.\", \"There is no consensus among astronomers on the proper procedures for estimating the universe's total mass.\"]", "label": 2 }, { "id": "train_1425", "context": "Heavy salting of Albritten' s roads to melt winter ice and snow began about 20 years ago. The area' s groundwater now contains approximately 100 milligrams of dissolved salt per liter. Groundwater in a nearby, less highly urbanized area, where little salt is used and where traffic patterns resemble those of Albritten 20 years ago, contains only about 10 milligrams of dissolved salt per liter. Since water that contains 250 or more milligrams of dissolved salt per liter tastes unacceptably salty, continuing the salting of Albritten' s roads at its present rate will render Albritten' s groundwater unpalatable within the next few decades.", "question": "Which one of the following, if true, most seriously weakens the argument?", "answers": "[\"Salting of Albritten's roads is likely to decrease over the next few decades.\", 'Even water that contains up to 5, 000 milligrams of dissolved salt per liter is safe to drink.', \"Albritten's groundwater contained roughly 90 milligrams of dissolved salt per liter 20 years ago.\", 'Salting icy roads is the simplest way to prevent accidents on those roads.']", "label": 2 }, { "id": "train_1426", "context": "Consumer advocate: Businesses are typically motivated primarily by the desire to make as great a profit as possible, and advertising helps businesses to achieve this goal. But it is clear that the motive of maximizing profits does not impel businesses to present accurate information in their advertisements. It follows that consumers should be skeptical of the claims made in advertisements.", "question": "Each of the following, if true, would strengthen the consumer advocate's argument EXCEPT:", "answers": "['Many consumers have a cynical attitude toward advertising.', 'Those who create advertisements are less concerned with the accuracy than with the creativity of advertisements.', 'The laws regulating truth in advertising are not applicable to many of the most common forms of inaccurate advertising.', 'Businesses know that they can usually maximize their profits by using inaccurate information in their advertisements.']", "label": 0 }, { "id": "train_1427", "context": "Candidate: The children in our nation need a better education. My opponent maintains that our outdated school system is the major impediment to achieving this goal. In fact our school system does need reform. Nonetheless, my opponent' s position places far too much blame on our schools, for it seems to equate education with schooling, yet other parts of society are at least as responsible for educating our youth as our schools are.", "question": "The statement that the school system needs reform figures in the candidate's argument in which one of the following ways?", "answers": "['It is offered as an example of one of the social problems for which the argument proposes a solution.', \"It is used to indicate how the failings of the school system are partially responsible for society's problems.\", \"It is the main conclusion that the argument is attempting to establish about the position of the candidate's opponent.\", \"It is a limited concession made to the candidate's opponent in the context of a broader challenge to the opponent's position.\"]", "label": 3 }, { "id": "train_1428", "context": "Lawyer: In a risky surgical procedure that is performed only with the patient' s informed consent, doctors intentionally cause the patient' s heart and brain functions to stop by drastically reducing the patient' s body temperature. When the procedure is completed, body temperature is quickly restored. Because the doctors deliberately stop the patient' s life functions, if these functions do not resume following the procedure, the medical team is technically guilty of manslaughter.", "question": "Which one of the following principles, if valid, most helps to justify the lawyer's analysis?", "answers": [ "Deliberately bringing about the cessation of a person's life functions is manslaughter if and only if the cessation is permanent.", "One is guilty of manslaughter only when one intends to cause irreversible loss of a person's life functions.", "Any time a medical procedure could result in the patient's death, the medical team could be charged with manslaughter.", "If a medical procedure is known to carry a very high risk of causing the patient's death, then only if the patient does die can the doctors be guilty of manslaughter." ], "label": 0 }, { "id": "train_1429", "context": "Health officials claim that because the foods and beverages mentioned or consumed on many television programs are extremely low in nutritional value, watching television has a bad influence on the dietary habits of television viewers.", "question": "The claim by health officials depends on the presupposition that", "answers": "['television viewers are only interested in the people on television programs who have the same eating and drinking habits as they do', 'seeing some foods and beverages being consumed on, or hearing them mentioned on, television programs increases the likelihood that viewers will consume similar kinds of foods and beverages', 'the eating and drinking habits of people on television programs provide health officials with accurate predictions about the food and beverages that will become popular among television viewers', 'the eating and drinking habits of people on television programs are designed to mirror the eating and drinking habits of television viewers']", "label": 1 }, { "id": "train_1430", "context": "Though Earth' s human population is increasing, it currently uses only a relatively small fraction of the supply of fresh water. Thus, claims that water shortages will plague humankind in the near future unless population growth trends change are simply mistaken.", "question": "Which one of the following, if true, most seriously weakens the argument above?", "answers": "['The percentage of fresh water used for agriculture is likely to grow more quickly than is the percentage used for industry.', \"Not all of Earth's population will adopt water conservation methods in the near future.\", 'Population growth trends are notoriously hard to predict with reasonable accuracy.', \"The amount of fresh water available to meet the needs of Earth's population varies significantly from region to region.\"]", "label": 3 }, { "id": "train_1431", "context": "Maude is incessantly engaging in diatribes against people who are materialistic. But her hypocrisy is evinced by the sentimental treatment of the watch her grandmother gave her. She certainly is very fond of the watch -- she worries about damaging it; in fact she always sets it carefully in a special box before going to bed.", "question": "Which one of the following is an assumption on which the argument depends?", "answers": "['People who are not materialistic tend to have merely sentimental attachments to things.', \"Maude's watch is not the only material thing she especially cares for.\", 'Possessions that come from relatives are treated with better care than those that do not.', 'Sentimental attachment to a single possession indicates being materialistic.']", "label": 3 }, { "id": "train_1432", "context": "Fluoride enters a region' s groundwater when rain dissolves fluoride-bearing minerals in the soil. In a recent study, researchers found that when rainfall, concentrations of fluoride-bearing minerals, and other relevant variables are held constant, fluoride concentrations in groundwater are significantly higher in areas where the groundwater also contains a high concentration of sodium.", "question": "Which one of the following can most reasonably be concluded on the basis of the researchers' findings?", "answers": "['Sodium in groundwater increases the rate at which fluoride-bearing minerals dissolve.', 'Fluoride-bearing minerals are not the primary source of fluoride found in groundwater.', 'Rainfall does not affect fluoride concentrations in groundwater.', 'Sodium-bearing minerals dissolve at a faster rate than fluoride-bearing minerals.']", "label": 0 }, { "id": "train_1433", "context": "During the past year, Pro-Tect Insurance Company' s total payout on car-theft claims has been larger than the company can afford to sustain. Pro-Tect cannot reduce the number of car-theft policies it carries, so it cannot protect itself against continued large payouts that way. Ttherefore, Pro-Tect has decided to offer a discount to holders of car-theft policies whose cars have antitheft devices. Many policyholders will respond to the discount by installing such devices, since the amount of the discount will within two years typically more than cover the cost of installation. Thus, because cars with antitheft devices are rarely stolen, Pro-Tect' s plan is likely to reduce its annual payouts. ", "question": "In the argument above, the two portions in boldface play which of the following roles?", "answers": "['The first is a development that the argument seeks to explain; the second is a prediction the argument makes in support of the explanation it offers.', 'The first presents a problem a response to which the argument assesses; the second is the judgment reached by that assessment.', 'The first and the second are both evidence offered by the argument as support for its main conclusion.', 'The first presents a development whose likely outcome is at issue in the argument; the second is a judgment the argument uses in support of its conclusion about that outcome.']", "label": 1 }, { "id": "train_1434", "context": "Electric washing machines, first introduced in the United States in 1925, significantly reduced the amount of time spent washing a given amount of clothes, yet the average amount of time households spent washing clothes increased after 1925. This increase is partially accounted for by the fact that many urban households had previously sent their clothes to professional laundries. But the average amount of time spent washing clothes also increased for rural households with no access to professional laundries.", "question": "Which of the following, if true, most helps to explain why the time spent washing clothes increased in rural areas?", "answers": "['People with access to an electric washing machine typically wore their clothes many fewer times before washing them than did people without access to electric washing machines.', 'The earliest electric washing machines required the user to spend much more time beside the machine than do modern electric washing machines.', 'In the 1920s and 1930s the proportion of rural households with electricity was smaller than the proportion of urban households with electricity.', 'People living in urban households that had previously sent their clothes to professional laundries typically owned more clothes than did people living in rural households.']", "label": 0 }, { "id": "train_1435", "context": "Although most smoking-related illnesses are caused by inhaling the tar in tobacco smoke, it is addiction to nicotine that prevents most smokers from quitting. In an effort to decrease the incidence of smoking-related illnesses, lawmakers in Sandonia plan to reduce the average quantity of nicotine per cigarette by half over the next five years. Unfortunately, smokers who are already addicted to nicotine tend to react to such reductions by smoking correspondingly more cigarettes.", "question": "The information above most strongly supports which of the following predictions about the effects of implementing the Sandonian government's plan?", "answers": "['The annual number of Sandonian smokers developing smoking-related illnesses will probably decrease during the next five years.', 'The number of Sandonians who quit smoking during the next five years will probably exceed the number who quit during the last five years.', 'The average quantity of tar inhaled by Sandonian smokers who are currently addicted to nicotine will probably not decrease during the next five years.', 'The proportion of Sandonians attempting to quit smoking who succeed in that attempt will probably decrease during the next five years.']", "label": 2 }, { "id": "train_1436", "context": "Cable TV stations have advantages that enable them to attract many more advertisers than broadcast networks attract. For example, cable stations are able to target particular audiences with 24-hour news, sports, or movies, whereas broadcast networks must offer a variety of programming. Cable can also offer lower advertising rates than any broadcast network can, because it is subsidized by viewers through subscriber fees. Additionally, many cable stations have expanded worldwide with multinational programming.", "question": "The statements above, if true, provide support for each of the following EXCEPT:", "answers": "['The audiences that some advertisers prefer to target watch 24-hour news stations.', 'Low costs are often an important factor for advertisers in selecting a station or network on which to run a TV ad.', 'Some advertisers prefer to have the opportunity to address a worldwide audience.', 'Some broadcast networks can be viewed in several countries.']", "label": 3 }, { "id": "train_1437", "context": "Mouth guards are increasingly becoming required equipment for contact sports. Besides the obvious benefit of protecting an athlete' s teeth, mouth guards also prevent concussions. Youth league referees should penalize teams with players participating without a sanctioned mouth guard.", "question": "Which of the following most accurately expresses the argument's main conclusion?", "answers": "['It is generally preferable to wear mouth guards while playing contact sports.', 'Mouth guards protect teeth and prevent concussions.', 'Mouth guards should always be worn during contact sports.', 'Mouth guards save lives.']", "label": 2 }, { "id": "train_1438", "context": "The pharmaceutical industry argues that because new drugs will not be developed unless heavy development costs can be recouped in later sales, the current 20 years of protection provided by patents should be extended in the case of newly developed drugs. However, in other industries new-product development continues despite high development costs, a fact that indicates that the extension is unnecessary.", "question": "Which of the following, if true, most strongly supports the pharmaceutical industry's argument against the challenge made above?", "answers": "['Clinical trials of new drugs, which occur after the patent is granted and before the new drug can be marketed, often now take as long as 10 years to complete.', 'Much recent industrial innovation has occurred in products-for example, in the computer and electronics industries-for which patent protection is often very ineffective.', 'An existing patent for a drug does not legally prevent pharmaceutical companies from bringing to market alternative drugs, provided they are sufficiently dissimilar to the patented drug.', 'No industries other than the pharmaceutical industry have asked for an extension of the 20-year limit on patent protection.']", "label": 0 }, { "id": "train_1439", "context": "Jenkins maintains that the movie Firepower was not intended to provoke antisocial behavior, arguing that, on the contrary, it is in the interest of Firepower' s director to prevent such behavior. Yet Jenkins' s conclusion must be rejected, because the movie has clearly produced antisocial behavior among many of those who have seen it.", "question": "The reasoning in the argument is flawed in that it", "answers": "['concludes from the mere fact that an action had a certain effect that the effect was intended by the person who performed the action', 'rejects an argument on the grounds that it was offered by a person who was biased', 'infers that something is true of a whole solely on the grounds that it is true of a part of the whole', 'overlooks the possibility that people can act in a way that is contrary to their expressed interest']", "label": 0 }, { "id": "train_1440", "context": "Geologist: A geological fault in the mountain under which the proposed nuclear waste storage facility would be buried could, after a few thousand years, cause the contents to seep out or water to seep in. Since nuclear waste remains dangerous for up to 25, 000 years, such seepage would be disastrous. So we should not place a nuclear waste storage facility under this mountain until scientists investigate whether this mountain has any geological faults.", "question": "Which one of the following arguments, if true, most strengthens the geologist's argumentation?", "answers": "[\"The scientists' investigation would conclusively show whether or not the mountain has any geological faults.\", 'Nuclear waste could be stored in the proposed facility on a temporary basis.', 'The proposed facility was not initially intended to be used for the storage of nuclear waste.', \"The scientists' investigation would increase dramatically the cost of storing nuclear waste under the mountain.\"]", "label": 0 }, { "id": "train_1441", "context": "In order to withstand tidal currents, juvenile horseshoe crabs frequently burrow in the sand. Such burrowing discourages barnacles from clinging to their shells. When fully grown, however, the crabs can readily withstand tidal currents without burrowing, and thus they acquire substantial populations of barnacles. Surprisingly, in areas where tidal currents are very weak, juvenile horseshoe crabs are found not to have significant barnacle populations, even though they seldom burrow.", "question": "Which of the following, if true, most helps to explain the surprising finding?", "answers": "['Until they are fully grown, horseshoe crabs shed their shells and grow new ones several times a year.', 'The strength of the tidal currents in a given location varies widely over the course of a day.', 'A very large barnacle population can significantly decrease the ability of a horseshoe crab to find food.', 'Tidal currents do not themselves dislodge barnacles from the shells of horseshoe crabs.']", "label": 0 }, { "id": "train_1442", "context": "High-technology medicine is driving up the nation' s health care costs. Recent advances in cataract surgery illustrate why this is occurring. Cataracts are a major cause of blindness, especially in elderly people. Ten years ago, cataract surgery was painful and not always effective. Thanks to the new technology used in cataract surgery, the operation now restores vision dramatically and is less expensive. These two factors have caused the number of cataract operations performed to increase greatly, which has, in turn, driven up the total amount spent on cataract surgery.", "question": "Which one of the following can be inferred from the passage?", "answers": "['Ten years ago, cataract surgery was affordable for more people than it was last year.', 'The total amount spent on cataract surgery has increased because the increased number of people electing to have the surgery more than offsets the decrease in cost per operation.', 'Ten years ago, few people had successful cataract surgery.', 'In the long run, the advantages of advanced medical technology are likely to be outweighed by the disadvantages.']", "label": 1 }, { "id": "train_1443", "context": "When uncontrollable factors such as lack of rain cause farmers' wheat crops to fail, fertilizer and seed dealers, as well as truckers and mechanics, lose business, and fuel suppliers are unable to sell enough diesel fuel to make a profit.", "question": "Which one of the following claims follows logically from the information above?", "answers": "[\"Farmers are not responsible for the consequences of a wheat crop's failing if wheat growth has been affected by lack of rain.\", \"The consequences of a drought are not restricted to the drought's impact on farm productivity.\", \"A country's dependence on agriculture can lead to major economic crises.\", 'If several of the businesses that sell to farmers do not prosper, it is because farming itself is not prospering.']", "label": 1 }, { "id": "train_1444", "context": "Superconductors are substances that conduct electricity without resistance at low temperatures. Their use, however, will never be economically feasible, unless there is a substance that superconducts at a temperature above minus 148 degrees Celsius. If there is such a substance, that substance must be an alloy of niobium and germanium. Unfortunately, such alloys superconduct at temperatures no higher than minus 160 degrees Celsius.", "question": "If the statements above are true, which one of the following must also be true?", "answers": "['The use of superconductors will never be economically feasible.', 'The use of superconductors could be economically feasible if there is a substance that superconducts at temperatures below minus 148 degrees Celsius.', 'No use of alloys of niobium and germanium will ever be economically feasible.', 'Alloys of niobium and germanium do not superconduct at temperatures below minus 160 degrees Celsius.']", "label": 0 }, { "id": "train_1445", "context": "Frankie: If jelly makers were given incentives to make a certain percentage of their jellies from cloudberries, income for cloudberry gatherers would increase. Anna: That plan would fail. Cacao, like cloudberries, was once harvested from wild plants. When chocolate became popular in Europe, the cacao gathers could not supply enough to meet the increased demand, and farmers began to grow large quantities of it at low cost. Now all cacao used in commercial chocolate production is grown on farms. Likewise, if the demand for cloudberries increases, domesticated berries grown on farms will completely supplant berries gathered in the wild.", "question": "Anna's argument proceeds by", "answers": "['proposing a general theory as a way of explaining a specific market situation', 'giving a reason why a proposed course of action would be beneficial to all those affected by it', 'projecting the result of following a proposal in a given situation by comparing that situation with a past situation', 'contending that the uses for one product are similar to the uses for another product']", "label": 2 }, { "id": "train_1446", "context": "Depression is a serious problem for residents of nursing homes. However, a recent study has found that residents who developed personal bonds with pets had significantly lower rates of depression than did residents who did not develop personal bonds with pets.", "question": "Which one of the following statements is most strongly supported by the information above?", "answers": "['The best method for helping a nursing-home resident to overcome depression is to provide access to a pet.', 'High rates of depression among nursing-home residents may result at least in part from a lack of companionship.', 'Allowing free access to pets in nursing homes would eliminate problems relating to depression.', 'Animal companionship is essential for psychological well-being.']", "label": 1 }, { "id": "train_1447", "context": "From 1980 to 1989, total consumption of fish in the country of Jurania increased by 4. 5 percent, and total consumption of poultry products there increased by 9. 0 percent. During the same period, the population of Jurania increased by 6 percent, in part due to immigration to Jurania from other countries in the region.", "question": "If the statements above are true, which of the following must also be true on the basis of them?", "answers": "['Per capita consumption of fish in Jurania was lower in 1989 than in 1980', \"During the 1980's in Jurania, profits of wholesale distributors of poultry products increased at a greater rate than did profits of wholesale distributors of fish\", 'In 1989 Juranians consumed twice as much poultry as fish', \"For people who immigrated to Jurania during the 1980's, fish was less likely to be a major part of their diet than was poultry\"]", "label": 0 }, { "id": "train_1448", "context": "Company Spokesperson: This year, more than 70% of the guests who stayed with us over the past summer are planning to return to our resorts for a summer vacation. Ttherefore, I believe that we are well-positioned for another strong and profitable quarter. Industry Analyst: This is not necessarily true. Nearly three quarters of the company's profits come from the attendance of its theme parks located at its major resorts, and it is not certain how this segment will perform this summer.", "question": "Which of the following statements, if true, would support the criticism of the Industry Analyst?", "answers": "[\"The company's management possesses more accurate financial information than is accessible to industry analysts.\", \"The vast majority of guests who visit the company's resorts attend its theme parks only once, usually during their first week of stay.\", 'The industry analyst represents an independent investment firm, which is not affiliated with the company.', \"Because of the seasonality in the vacation travel and theme park attendance, summer is the most important season for the firm's operations, accounting for over half of the firm's annual profits.\"]", "label": 1 }, { "id": "train_1449", "context": "A commonly held belief is that cracking one' s knuckles causes arthritis. But this is, in fact, false, as indicated by the fact that most people with arthritis never cracked their knuckles.", "question": "The reasoning is flawed because the argument", "answers": "['concludes that knuckle-cracking does not cause arthritis by appealing to an applicable principle', 'treats the absence of a condition in some people afflicted with a disorder as evidence that the condition never causes the disorder', 'ignores the possibility that arthritis is caused by genetic factors', 'makes a distinction between people who crack their knuckles and people who do not when there is no actual difference between these two groups of people']", "label": 1 }, { "id": "train_1450", "context": "History student: It is unfair for the History Department to prohibit students from citing certain online encyclopedias in their research papers merely because these sources are not peer reviewed. In their research, students should be allowed to read whatever they wish; otherwise, it is censorship. History professor: Students are allowed to read whatever they like. The rule stipulates only that certain online encyclopedias are not to be cited as references since, given that they are not peer reviewed, they cannot reasonably be treated as reliable support for any claim.", "question": "The dialogue provides most support for the claim that the student and the professor disagree over whether", "answers": "['students should be allowed to read whatever they wish to in preparing to write a research paper for a history class', 'sources that are not peer reviewed often have solid support for the claims that they make', 'prohibiting a certain sort of online source material from being cited as a research reference amounts to prohibiting students from reading that source material', 'research papers written for a history class require some citations to be from sources that have been peer reviewed']", "label": 2 }, { "id": "train_1451", "context": "Bacteria from food can survive for several days on the surface of plastic cutting boards, but bacteria can penetrate wooden cutting boards almost immediately, leaving the surface free of contamination. Ttherefore, wooden cutting boards, unlike plastic cutting boards, need not be washed in order to prevent their contaminating food that is cut on them; wiping them off to remove food debris is sufficient.", "question": "Which one of the following is an assumption on which the argument depends?", "answers": "['Bacteria that penetrate into wooden cutting boards do not reemerge on the surface after the cutting boards have been used.', 'Washing plastic cutting boards does not remove all bacteria from the surface.', 'Prevention of bacteria contamination is the only respect in which wooden cutting boards are superior to plastic cutting boards.', 'Washing wooden cutting boards kills bacteria below the surface of the cutting boards.']", "label": 0 }, { "id": "train_1452", "context": "Auto industry executive: Statistics show that cars that were built smaller after 1977 to make them more fuel-efficient had a higher incidence of accident-related fatalities than did their earlier, larger counterparts. For this reason we oppose recent guidelines that would require us to produce cars with higher fuel efficiency.", "question": "Which one of the following, if true, would constitute the strongest objection to the executive's argument?", "answers": "['New computerized fuel systems can enable large cars to meet fuel efficiency standards established by the recent guidelines.', 'Fuel efficiency in models of large cars rose immediately after 1977 but has been declining ever since.', 'Modern technology can make small cars more fuel-efficient today than at any other time in their production history.', 'Even after 1977, large automobiles were frequently involved in accidents that caused death or serious injury.']", "label": 0 }, { "id": "train_1453", "context": "The consumer price index is a measure that detects monthly changes in the retail prices of goods and services. The payment of some government retirement benefits is based on the consumer price index so that those benefits reflect the change in the cost of living as the index changes. However, the consumer price index does not consider technological innovations that may drastically reduce the cost of producing some goods. Ttherefore, the value of government benefits is sometimes greater than is warranted by the true change in costs.", "question": "The reasoning in the argument is most vulnerable to the criticism that the argument", "answers": "['presumes, without providing warrant, that retirement benefits are not generally used to purchase unusual goods', 'fails to consider the possibility that there are years in which there is no change in the consumer price index', 'makes an irrelevant shift from discussing retail prices to discussing production costs', 'fails to make explicit which goods and services are included in the consumer price index']", "label": 2 }, { "id": "train_1454", "context": "Efraim: The popular press continually advises people to avoid various health risks. Yet by seeking to avoid health risks, people inevitably become anxious, and anxiety, in turn, poses a major health risk. Thus, paradoxical as it seems, simply disregarding journalists' advice about reducing health risks is bound to pose less of a health risk than does trying to follow any such advice. Felicity: But history shows that you are wrong. It was articles in the popular press that made people aware of the health risks posed by smoking cigarettes. Many people stopped smoking in response, and they led longer and healthier lives than they would have otherwise.", "question": "Which one of the following most accurately expresses the point at issue between Efraim and Felicity?", "answers": "['whether people can reduce risks to their health by heeding the advice of articles in the popular press', \"whether there are any people who ignore journalists' advice about avoiding health risks\", 'whether products and practices described by journalists as posing health risks do in fact pose health risks', 'whether the sort of anxiety that results from efforts to avoid health risks can be overcome']", "label": 0 }, { "id": "train_1455", "context": "Tariffs on particular products tend to protect the small percentage of the population that works in industries that make those products while hurting everyone else through higher costs. Polls show that in fact most people oppose such tariffs. So politicians would be more likely to be reelected if they voted against these tariffs.", "question": "Which one of the following is an assumption on which the argument relies?", "answers": "[\"Supporters of tariffs on particular products are not significantly more likely than opponents to base their vote for a politician on the politician's stand on this issue.\", 'Politicians should support only general tariffs, since such tariffs would be more widely popular with voters than tariffs on particular products.', 'Politicians should never support measures that favor only a small percentage of the population.', 'Politicians always vote according to what is most likely to get them reelected.']", "label": 0 }, { "id": "train_1456", "context": "If the government increases its funding for civilian scientific research, private patrons and industries will believe that such research has become primarily the government' s responsibility. When they believe that research is no longer primarily their responsibility, private patrons and industries will decrease their contributions toward research. Ttherefore, in order to keep from depressing the overall level of funding for civilian scientific research, the government should not increase its own funding.", "question": "Which one of the following is an assumption on which the argument relies?", "answers": "['Private donations toward research are no longer welcomed by researchers whose work receives government funding.', 'Funding for civilian scientific research is currently at the highest possible level.', 'Governments should bear the majority of the financial burden of funding for civilian scientific research.', 'Any increase in government funding would displace more private funding for funding for civilian scientific research than it would provide.']", "label": 3 }, { "id": "train_1457", "context": "Letter to the editor: I was shocked to learn that Judge Mosston was convicted of criminal assault, but I disagree with my fellow citizens who believe that he should be forced to resign. I have played cards with Judge Mosston for many years, and he has always shown himself to be fair toward me and our fellow cardplayers. Our judicial system simply cannot afford to lose any just and fair judges.", "question": "Which one of the following most accurately expresses the main conclusion of the letter writer's argument?", "answers": "[\"Individuals who have interacted with a judge outside that judge's courtroom are the most objective assessors of that judge's fairness.\", 'Judge Mosston is a fair judge.', 'Judge Mosston should not be forced to resign.', 'Fairness is the most important quality in a judge.']", "label": 2 }, { "id": "train_1458", "context": "Principle: Any person or business knowingly aiding someone' s infringement on a copyright is also guilty of copyright infringement. Application: Grandview Department Store, which features a self-service photo-printing kiosk, is guilty of copyright infringement since a customer using the kiosk infringed on a wedding photographer' s copyright by printing photographs whose copyright is held by the photographer.", "question": "Which one of the following, if assumed, most helps to justify the application of the principle?", "answers": "['A person or business providing a service that can be expected to be used to infringe on a copyright should be considered to knowingly aid any copyright infringer using the service.', 'Owners of self-service facilities should monitor those facilities in order to ensure that they are not used for illegal or unethical purposes.', 'The operator of a business has the same legal obligations to customers who use self-service facilities as it has to customers who use full-service facilities.', 'The management of a business that is open to the public is obligated to report to the authorities any illegal activity that it witnesses on its property.']", "label": 0 }, { "id": "train_1459", "context": "Almost all microbe species live together in dense, interdependent communities, supporting the environment for each other, and regulating the population balances for their different species through a complex system of chemical signals. For this reason, it is currently impossible to cultivate any one such species in isolation. Thus, microbiologists lack complete knowledge of most microbe species.", "question": "Which one of the following, if assumed, enables the argument's conclusion to be properly drawn?", "answers": "['If it is currently impossible to reproduce the environmental supports and chemical signals in dense, interdependent communities of microbe species, then it is also impossible to cultivate any microbe species from such a community in isolation.', 'At least some microbiologists lack complete knowledge of any microbe species that live together in dense, interdependent communities.', 'No microbe species that normally lives together with other microbe species in dense, interdependent communities can survive outside such a community.', 'No microbiologist can have complete knowledge of any species of organism unless that microbiologist can cultivate that species in isolation.']", "label": 3 }, { "id": "train_1460", "context": "Increases in the level of high-density lipoprotein (HDL)in the human bloodstream lower bloodstream cholesterol levels by increasing the body' s capacity to rid itself of excess cholesterol. Levels of HDL in the bloodstream of some individuals are significantly increased by a program of regular exercise and weight reduction.", "question": "Which of the following can be correctly inferred from the statements above?", "answers": "['A program of regular exercise and weight reduction lowers cholesterol levels in the bloodstream of some individuals.', 'Individuals who are underweight do not run any risk of developing high levels of cholesterol in the bloodstream.', 'Exercise and weight reduction are the most effective methods of lowering bloodstream cholesterol levels in humans.', 'Individuals who do not exercise regularly have a high risk of developing high levels of cholesterol in the bloodstream late in life.']", "label": 0 }, { "id": "train_1461", "context": "When a major record label signs a contract with a band, the label assumes considerable financial risk. It pays for videos, album art, management, and promotions. Hence, the band does not need to assume nearly as much risk as it would if it produced its own records independently. For this reason, it is only fair for a major label to take a large portion of the profits from the record sales of any band signed with it.", "question": "Which one of the following most accurately describes the role played in the argument by the claim that a band signed with a major label does not need to assume nearly as much risk as it would if it produced its own records independently?", "answers": "['Premises are used to support it, and it is used to support the main conclusion.', \"It is a general principle from which the argument's conclusion follows as a specific instance.\", 'It describes a phenomenon for which the rest of the argument offers an explanation.', 'It is one of two unrelated conclusions, each of which the same premises are used to support.']", "label": 0 }, { "id": "train_1462", "context": "Several years ago, the number of wood bison became so low that conservationists began breeding the wood bison in an enclosed refuge. In this time, the number of wood bison has tripled and, according to conservationists, this number is enough for a species to remain viable in the wild. Yet since the wood bison's habitat is still imperiled, conservationists plan to move the wood bison 2, 000 miles away, where the American buffalo, a distinct species, flourishes. The conservationists predict that such a plan will allow for the long-term viability of the wood bison as a species.", "question": "Which of the following, if true, calls into question the conservationist's prediction?", "answers": "['No wood bison has ever been within 1, 000 miles of the area in which the conservationists are planning to move the herd.', \"American buffalo once shared territory with the wood bison, but the American buffalo's territory slowly moved south so that now the American buffalo no longer inhabits the same area as the wood bison.\", 'The American buffalo and the wood bison can thrive off of the same resources.', 'The American buffalo and the wood bison readily mingle, and, in the past, this has lead the two species to mix so that the number of actual wood bison declines steadily.']", "label": 3 }, { "id": "train_1463", "context": "Striking newspaper carrier: The fact that this subscriber canceled her newspaper subscription is a triumph for the striking workers. It shows that the subscriber supports the striking worker' s claim that they are not getting paid enough. Newspaper company X' s president: That is ridiculous! The canceled subscription does not mean that the subscriber supports the striking workers. The subscription was canceled because the subscriber has said she does not read the newspaper every day and the subscriber prefers another newspaper. Ttherefore, the canceled subscription had nothing to do with supporting your strike.", "question": "The reasoning in newspaper company X's president's argument is flawed because the argument", "answers": "['gives a detailed analogy to show how his opponent has mischaracterized the situation', \"overlooks the possibility that the subscriber's proffered reasons for canceling her subscription may themselves show support for the striking workers\", 'fails to consider that any subscriber may wish to cancel his subscription in support of the striking workers', 'concedes a point made by an opponent that weakens the overall conclusion made by the company president']", "label": 1 }, { "id": "train_1464", "context": "When students receive negative criticism generated by computer programs, they are less likely to respond positively than when the critic is a human. Since the acceptance of criticism requires that one respond positively to it, students are more likely to learn from criticism by humans than from criticism by computers.", "question": "Which one of the following is an assumption on which the argument depends?", "answers": "['Unlike human critics, computers are incapable of showing compassion.', 'Students always know whether their critics are computers or humans.', 'Criticism generated by computers is likely to be no more or less favorable than that produced by human critics in response to the same work.', 'Students are more likely to learn from criticism that they accept than from criticism they do not accept.']", "label": 3 }, { "id": "train_1465", "context": "Irrigation runoff from neighboring farms may well have increased the concentration of phosphorus in the local swamp above previous levels, but the claim that the increase in phosphorus is harming the swamp' s native aquatic wildlife is false; the phosphorus concentration in the swamp is actually less than that found in certain kinds of bottled water that some people drink every day.", "question": "The argument is vulnerable to criticism on the ground that it", "answers": "['makes a generalization that is unwarranted because the sources of the data on which it is based have not been specified', 'bases its conclusion on two contradictory claims', 'relies on evidence the relevance of which has not been established', 'concedes the very point that it argues against']", "label": 2 }, { "id": "train_1466", "context": "Engineer: Thermophotovoltaic generators are devices that convert heat into electricity. The process of manufacturing steel produces huge amounts of heat that currently go to waste. So if steel-manufacturing plants could feed the heat they produce into thermophotovoltaic generators, they would greatly reduce their electric bills, thereby saving money.", "question": "Which one of the following is an assumption on which the engineer's argument depends?", "answers": "['There is no other means of utilizing the heat produced by the steel-manufacturing process that would be more cost effective than installing thermophotovoltaic generators.', 'Using current technology, it would be possible for steel-manufacturing plants to feed the heat they produce into thermophotovoltaic generators in such a way that those generators could convert at least some of that heat into electricity.', 'The amount steel-manufacturing plants would save on their electric bills by feeding heat into thermophotovoltaic generators would be sufficient to cover the cost of purchasing and installing those generators.', 'At least some steel-manufacturing plants rely on electricity as their primary source of energy in the steel-manufacturing process.']", "label": 2 }, { "id": "train_1467", "context": "Brianna: It would have been better to buy a tree last summer rather than this summer. The one we bought this summer is struggling to survive this summer' s drought. If we had bought one last summer, it would have been able to survive this summer' s drought, because last summer' s normal rainfall would have enabled it to develop established roots. Trees with established roots can better withstand droughts.", "question": "Which one of the following most accurately expresses the overall conclusion drawn in Brianna's argument?", "answers": "['Trees with established roots can better withstand droughts.', 'It would have been better to buy a tree last summer rather than this summer.', \"If a tree had been purchased last summer, it would be better able to survive this summer's drought.\", \"The tree purchased this summer is struggling to survive this summer's drought.\"]", "label": 1 }, { "id": "train_1468", "context": "The odds of winning any major lottery jackpot are extremely slight. However, the very few people who do win major jackpots receive a great deal of attention from the media. Thus, since most people come to have at least some awareness of events that receive extensive media coverage, it is likely that many people greatly overestimate the odds of their winning a major jackpot.", "question": "Which one of the following is an assumption on which the argument depends?", "answers": "['Very few people other than those who win major jackpots receive a great deal of attention from the media.', 'If it were not for media attention, most people who purchase lottery tickets would not overestimate their chances of winning a jackpot.', 'Most people who overestimate the likelihood of winning a major jackpot do so at least in part because media coverage of other people who have won major jackpots downplays the odds against winning such a jackpot.', 'Becoming aware of individuals who have won a major jackpot leads at least some people to incorrectly estimate their own chances of winning such a jackpot.']", "label": 3 }, { "id": "train_1469", "context": "Researcher: Dinosaur fossils come in various forms, including mineralized bones and tracks in dried mud flats. However, mineralized dinosaur bones and dinosaur tracks in dried mud flats are rarely found together. This isn' t surprising, because creatures that scavenged dinosaur carcasses most likely frequented mud flats to find food.", "question": "Which one of the following, if true, would most strengthen the researcher's argument?", "answers": "['Dinosaur fossils other than mineralized bone or tracks in dried mud flats are quite common.', 'Scavengers commonly drag a carcass away from the site where it was found.', 'Dinosaur tracks are also found in locations other than mud flats.', 'Researchers have found more fossil dinosaur tracks than fossil dinosaur bones.']", "label": 1 }, { "id": "train_1470", "context": "A mosquito bite can transmit to a person the parasite that causes malaria, and the use of mosquito nets over children' s beds can significantly reduce the incidence of malarial infection for children in areas where malaria is common. Yet public health officials are reluctant to recommend the use of mosquito nets over children' s beds in such areas.", "question": "Which of the following, if true, would provide the strongest grounds for the public health officials' reluctance?", "answers": "[\"Early exposure to malaria increases the body's resistance to it and results in a lesser likelihood of severe life-threatening episodes of malaria.\", 'Although there are vaccines available for many childhood diseases, no vaccine has been developed that is effective against malaria.', 'Mosquito nets provide protection from some insect pests other than mosquitoes.', 'The pesticides that are most effective against mosquitoes in regions where malaria is common have significant detrimental effects on human health.']", "label": 0 }, { "id": "train_1471", "context": "Sparva, unlike Treland' s other provinces, requires automobile insurers to pay for any medical treatment sought by someone who has been involved in an accident; in the other provinces, insurers pay for nonemergency treatment only if they preapprove the treatment. Clearly, Sparva' s less restrictive policy must be the explanation for the fact that altogether insurers there pay for far more treatments after accidents than insurers in other provinces, even though Sparva does not have the largest population.", "question": "Which of the following, if true, most strengthens the argument?", "answers": "['Different insurance companies have different standards for determining what constitutes emergency treatment.', 'There are fewer traffic accidents annually in Sparva than in any of the provinces of comparable or greater population.', 'The cost of medical care in Sparva is higher than the national average.', 'Fewer insurance companies operate in Sparva than in any other province.']", "label": 1 }, { "id": "train_1472", "context": "Sales manager: Last year the total number of meals sold in our company' s restaurants was much higher than it was the year before. Obviously consumers find our meals desirable. Accountant: If you look at individual restaurants, however, you find that the number of meals sold actually decreased substantially at every one of our restaurants that was in operation both last year and the year before. The desirability of our meals to consumers has clearly decreased, given that this group of restaurants -- the only ones for which we have sales figures that permit a comparison between last year and the year before -- demonstrates a trend toward fewer sales.", "question": "If the sales figures cited by the accountant and the sales manager are both accurate, which one of the following must be true?", "answers": "[\"The market share captured by the company's restaurants fell last year.\", \"The prices of the company's meals have changed over the past two years.\", 'The company opened at least one new restaurant in the last two years.', \"The quality of the company's meals has not improved over the past two years.\"]", "label": 2 }, { "id": "train_1473", "context": "Maria: Calling any state totalitarian is misleading: it implies total state control of all aspects of life. The real world contains no political entity exercising literally total control over even one such aspect. This is because any system of control is inefficient, and, ttherefore, its degree of control is partial. James: A one-party state that has tried to exercise control over most aspects of a society and that has, broadly speaking, managed to do so is totalitarian. Such a system' s practical inefficiencies do not limit the aptness of the term, which does not describe a state s actual degree of control as much as it describes the nature of a state' s ambitions.", "question": "Which one of the following most accurately expresses Maria's main conclusion?", "answers": "['No existing state currently has even one aspect of society under total control.', 'No state can be called totalitarian without inviting a mistaken belief.', 'The degree of control exercised by a state is necessarily partial.', 'To be totalitarian, a state must totally control society.']", "label": 1 }, { "id": "train_1474", "context": "Economist: The most economically efficient way to reduce emissions of air pollutants is to tax them in proportion to the damage they are likely to cause. But in Country Y, many serious pollutants are untaxed and unregulated, and policy makers strongly oppose new taxes. Ttherefore, the best way to achieve a reduction in air pollutant emissions in Country Y would be to institute fixed upper limits on them.", "question": "Which of the following is an assumption of the economist's argument?", "answers": "['Policy makers in Country Y oppose all new taxes equally strongly, regardless of any benefits they may provide.', 'Policy makers in Country Y strongly favor reductions in air pollutant emissions.', \"Country Y's policy makers believe that air pollutant emissions should be reduced with maximum economic efficiency.\", 'Policy makers in Country Y do not oppose setting fixed upper limits on air pollutant emissions as strongly as they oppose new taxes.']", "label": 3 }, { "id": "train_1475", "context": "Perception cannot be a relationship between a conscious being and a material object that causes that being to have beliefs about that object. For there are many imperceptible material objects about which we have beliefs.", "question": "Which one of the following is most closely parallel in its flawed reasoning to the flawed reasoning in the argument above?", "answers": "['Physics cannot be the science that investigates the ultimate principles of nature. For human beings are finite, and the ultimate principles cannot be understood by finite beings.', 'Art cannot be an artifact created by someone with the express purpose of causing an aesthetic reaction in its audience. For we often have aesthetic reactions to artifacts that are not art.', 'Happiness cannot be the state of mind in which pleasure both qualitatively and quantitatively predominates over pain. For we simply cannot compare pain and pleasure qualitatively.', 'Liberty cannot be the obligation of other people not to prevent one from doing as one wishes. For no matter what one tries to do some people will try to prevent it.']", "label": 1 }, { "id": "train_1476", "context": "Offshore oil-drilling operations entail an unavoidable risk of an oil spill, but importing oil on tankers presently entails an even greater such risk per barrel of oil. Ttherefore, if we are to reduce the risk of an oil spill without curtailing our use of oil, we must invest more in offshore operations and import less oil on tankers.", "question": "Which of the following, if true, most seriously weakens the argument above?", "answers": "['Tankers can easily be redesigned so that their use entails less risk of an oil spill.', 'Offshore operations usually damage the ocean floor, but tankers rarely cause such damage.', 'Importing oil on tankers is currently less expensive than drilling for it offshore.', 'Oil spills caused by tankers have generally been more serious than those caused by offshore operations.']", "label": 0 }, { "id": "train_1477", "context": "Restaurant owner: The newspaper reporter who panned my restaurant acknowledges having no special expertise about food and its preparation. His previous job was as a political reporter. He is a good writer, but he is not a true restaurant critic. A newspaper would never call someone a drama critic who had no special training in theater.", "question": "Which one of the following most accurately expresses the conclusion drawn in the restaurant owner's argument?", "answers": "['The newspaper reporter who panned the restaurant acknowledges having no special expertise about food and its preparation.', 'The newspaper reporter who panned the restaurant is a good writer.', 'The newspaper reporter who panned the restaurant is not a true restaurant critic.', 'A newspaper would never call someone a drama critic who had no special training in theater.']", "label": 2 }, { "id": "train_1478", "context": "United States hospitals have traditionally relied primarily on revenues from paying patients to offset losses from unreimbursed care. Almost all paying patients now rely on governmental or private health insurance to pay hospital bill. Recently, insurers have been strictly limiting what they pay hospitals for the care of insured patients to amounts at or below actual costs.", "question": "Which of the following conclusions is best supported by the information above?", "answers": "['If hospitals do not find ways of raising additional Income for unreimbursed care, they must either deny some of that care or suffer losses if they give it.', 'Some patients have incomes too, high for eligibility for governmental health insurance but are unable to afford private insurance for hospital care.', 'Even though philanthropic donations have traditionally provided some support for the hospitals, such donations are at present declining.', 'If the hospitals reduce their costs in providing care, insurance companies will maintain the current level of reimbursement, thereby providing more funds for unreimbursed care.']", "label": 0 }, { "id": "train_1479", "context": "The only physical factor preventing a human journey to Mars has been weight. Carrying enough fuel to propel a conventional spacecraft to Mars and back would make even the lightest craft too heavy to be launched from Earth. A device has recently been invented, however, that allows an otherwise conventional spacecraft to refill the craft' s fuel tanks with fuel manufactured from the Martian atmosphere for the return trip. Ttherefore, it is possible for people to go to Mars in a spacecraft that carries this device and then return.", "question": "Which one of the following is an assumption on which the argument depends?", "answers": "['A conventional spacecraft equipped with the device would not be appreciably more expensive to construct than current spacecraft typically are.', \"The device for manufacturing fuel from the Martian atmosphere would not take up any of the spaceship crew's living space.\", 'The device for manufacturing fuel for the return to Earth weighs less than the tanks of fuel that a conventional spacecraft would otherwise need to carry from Earth for the return trip.', 'The fuel manufactured from the Martian atmosphere would not differ in composition from the fuel used to travel to Mars.']", "label": 2 }, { "id": "train_1480", "context": "A common genetic mutation that lowers levels of the enzyme cathepsin C severely reduces a person' s ability to ward off periodontitis, or gum disease. The enzyme triggers immunological reactions that destroy diseased cells and eliminate infections in the mouth. But researchers are developing ways to restore the enzyme to normal levels. Once that happens, we will be able to eliminate periodontitis.", "question": "Which one of the following is an assumption on which the argument depends?", "answers": "['A person whose cathepsin C level has been restored to normal will not suffer from periodontitis.', 'Genetic mutation is the only cause of lowered levels of cathepsin C.', 'Researchers will soon succeed in finding means of restoring cathepsin C to normal levels.', 'Persons who do not have the genetic mutation that lowers levels of cathepsin C do not get gum disease.']", "label": 0 }, { "id": "train_1481", "context": "Some ornithologists believe that many species of songbirds are threatened by deforestation. Yet they also claim that, despite recent reforestation, matters continue to worsen, since it is fragmentation of forest rather than reduction of forest size that endangers songbird species. The introduction of open spaces and corridors in forests reduces the distance of songbird nests from unforested areas and thus reduces the songbirds' natural shield from predators.", "question": "The claim that there has recently been reforestation plays which one of the following roles in the ornithologists' argument?", "answers": "[\"It is presented as evidence for the claim that songbirds' predators are threatened by extinction unless they have open spaces and corridors that give them access to their prey.\", \"It is presented as a phenomenon that is compatible with the ornithologists' claim that the threat to songbirds continues to worsen.\", \"It is used as evidence that songbirds' predators will continue to have a habitat and so will continue to pose a threat to songbirds.\", 'It is used as evidence that various species of songbirds will continue to be threatened with extinction.']", "label": 1 }, { "id": "train_1482", "context": "Policy: The factory' s safety inspector should not approve a new manufacturing process unless it has been used safely for more than a year at another factory or it will demonstrably increase safety at the factory. Application: The safety inspector should not approve the proposed new welding process, for it cannot be shown to increase safety at the factory.", "question": "Which one of the following, if true, justifies the above application of the policy?", "answers": "['The factory at which the new welding process was first introduced has had several problems associated with the process.', 'The safety inspector will not approve any new process that has not been used extensively elsewhere.', 'Some of the manufacturing processes currently in use at the factory are not demonstrably safer than the new welding process.', 'The proposed new welding process has not been used in any other factory.']", "label": 3 }, { "id": "train_1483", "context": "From 1973 to 1986, growth in the United States economy was over 33 percent, while the percent growth in United States energy consumption was zero. The number of barrels of oil being saved per day by energy efficiency improvements made since 1973 is now 13 million.", "question": "If the information above is correct, which of the following conclusions can properly be drawn on the basis of it?", "answers": "['A reduction in the consumption of gasoline was the reason overall energy consumption remained steady.', 'Oil imports cannot be reduced unless energy consumption does not grow at all.', 'The development of nontraditional energy sources will make it possible for the United States economy to grow even faster.', 'It is possible for an economy to grow without consuming additional energy.']", "label": 3 }, { "id": "train_1484", "context": "Since the 1970s, environmentalists have largely succeeded in convincing legislators to enact extensive environmental regulations. Yet, as environmentalists themselves not only admit but insist, the condition of the environment is worsening, not improving. Clearly, more environmental regulations are not the solution to the environment' s problems.", "question": "The argument's reasoning is flawed because the argument", "answers": "['fails to justify its presumption that reducing excessive regulations is more important than preserving the environment', 'presumes, without providing warrant, that only an absence of environmental regulations could prevent environmental degradation', 'attacks the environmentalists themselves instead of their positions', 'fails to consider the possibility that the condition of the environment would have worsened even more without environmental regulations']", "label": 3 }, { "id": "train_1485", "context": "Gardener: The design of Japanese gardens should display harmony with nature. Hence, rocks chosen for placement in such gardens should vary widely in appearance, since rocks found in nature also vary widely in appearance.", "question": "The gardener's argument depends on assuming which one of the following?", "answers": "['In the selection of rocks for Japanese gardens, imitation of nature helps to achieve harmony with nature.', 'The selection of rocks for placement in a Japanese garden should reflect every key value embodied in the design of Japanese gardens.', 'The only criterion for selecting rocks for placement in a Japanese garden is the expression of harmony with nature.', 'Each component of a genuine Japanese garden is varied.']", "label": 0 }, { "id": "train_1486", "context": "Twenty professional income-tax advisors were given identical records from which to prepare an income-tax return. The advisors were not aware that they were dealing with fictitious records compiled by a financial magazine. No two of the completed tax returns agreed with each other, and only one was technically correct.", "question": "If the information above is correct, which one of the following conclusions can be properly drawn on the basis of it?", "answers": "['People are more likely to have an incorrectly prepared tax return if they prepare their own tax returns than if they hire a professional income-tax advisor.', 'All professional income-tax advisors make mistakes on at least some of the tax returns they prepare.', 'In order to insure that tax returns are correct, it is necessary to hire professional income-tax advisors to prepare them.', 'The fact that a tax return has been prepared by a professional income-tax advisor provides no guarantee that the tax return has been correctly prepared.']", "label": 3 }, { "id": "train_1487", "context": "Computers perform actions that are closer to thinking than anything nonhuman animals do. But computers do not have volitional powers, although some nonhuman animals do.", "question": "Which one of the following is most strongly supported by the information above?", "answers": "['Having volitional powers need not involve thinking.', 'It is necessary to have volitional powers in order to think.', 'Things that are not animals do not have volitional powers.', 'Computers will never be able to think as human beings do.']", "label": 0 }, { "id": "train_1488", "context": "Critic: Political utility determines the popularity of a metaphor. In authoritarian societies, the metaphor of society as a human body governed by a head is pervasive. Ttherefore, the society-as-body metaphor, with its connection between society' s proper functioning and governance by a head, promotes greater acceptance of authoritarian repression than do other metaphors, such as likening society to a family.", "question": "Which one of the following statements, if true, most weakens the critic's argument?", "answers": "['In authoritarian societies, the metaphor of society as a family is just as pervasive as the society-as-body metaphor.', 'The metaphor of society as a human body is sometimes used in nonauthoritarian societies.', 'Every society tries to justify the legitimacy of its government through the use of metaphor.', 'Authoritarian leaders are always searching for new metaphors for society in their effort to maintain their power.']", "label": 0 }, { "id": "train_1489", "context": "The Biocarb Company wants to build a sterilization plant to treat contaminated medical waste in a city neighborhood where residents and environmental activists fear that such a facility will pollute the area. Biocarb' s president argues that the operation of the plant cannot cause pollution because the waste would be sterile after processing by the plant. He claims that after exposure for an hour to superheated steam in the autoclave, such refuse would be far cleaner than food prepared in the cleanest kitchen.", "question": "The president's argument depends on which one of the following assumptions?", "answers": "['Treatment by superheated steam represents the surest method of sterilization.', 'Fear of pollution is the only argument against construction of an autoclave facility for medical waste.', 'No others besides environmental activists are concerned about pollution hazards that can result from processing medical waste.', 'Handling of the waste before treatment in the proposed facility will not pose a threat of pollution to the area.']", "label": 3 }, { "id": "train_1490", "context": "Forester: The great majority of the forests remaining in the world are only sickly fragments of the fully functioning ecosystems they once were. These fragmented forest ecosystems have typically lost their ability to sustain themselves in the long term, yet they include the last refuges for some of the world' s most endangered species. To maintain its full complement of plant and animal species, a fragmented forest requires regular interventions by resource managers.", "question": "The forester's statements, if true, most strongly support which one of the following?", "answers": "[\"At present, resource managers intervene regularly in only some of the world's fragmented forest ecosystems.\", 'A fragmented forest ecosystem cannot sustain itself in the long term if it loses any of its plant or animal species.', \"Most of the world's forests will lose at least some of their plant or animal species if no one intervenes.\", \"Unless resource managers regularly intervene in most of the world's remaining forests, many of the world's most endangered species will not survive.\"]", "label": 2 }, { "id": "train_1491", "context": "In the two years following the unification of Germany in 1989, the number of cars owned by residents of East Germany and the total distance traveled by cars in East Germany both increased by about 40 percent. In those two years, however, the number of East German residents killed each year as car occupants in traffic accidents increased by about 300 percent.", "question": "Which of the following, if true, most helps to explain the disproportionate increase in traffic fatalities?", "answers": "['After unification, many people who had been living in East Germany relocated to West Germany.', 'Drivers who had driven little or not at all before 1989 accounted for much of the increase in the total distance traveled by cars.', 'After unification, a smaller proportion of the cars being purchased by East German residents were used vehicles.', 'Over the same two-year period in East Germany, other road users, such as motorcyclists, bicyclists, and pedestrians, experienced only small increases in traffic fatalities.']", "label": 1 }, { "id": "train_1492", "context": "Walter: Although cigarette smoking is legal, it should be banned on all airline flights. Cigarette smoking in the confines of an aircraft exposes nonsmokers to harmful secondhand smoke that they cannot avoid.", "question": "Which one of the following principles, if established, would justify the proposal put forth by Walter?", "answers": "['People should be prohibited from engaging in an otherwise legal activity in those situations in which that activity would unavoidably expose others to harm.', 'A legal activity that has the potential for causing harm to others in certain situations should be modified in those situations to render it harmless.', \"If an activity is legal in some situations in which a person's engaging in that activity could harm others, then that activity should be legal in all situations.\", 'An activity should be banned only if most situations in which a person engages in that activity would inevitably expose others to harm.']", "label": 0 }, { "id": "train_1493", "context": "Researcher: Any country can determine which type of public school system will work best for it by investigating the public school systems of other countries. Nationwide tests could be given in each country and other countries could adopt the system of the country that has the best scores on these tests.", "question": "Which one of the following is an assumption required by the researcher's argument?", "answers": "['If two countries performed differently on these nationwide tests, further testing could determine what features of the school systems account for the differences.', 'A type of school system that works well in one country will work well in any other country.', 'Most countries in the research sample already administer nationwide tests to their public school students.', 'A number of children in each country in the research sample are educated in private schools.']", "label": 1 }, { "id": "train_1494", "context": "Legislator: The recently released crime statistics clearly show that the new laws requiring stiffer punishments for violators have reduced the crime rate. In the areas covered by those laws, the incidence of crime has decreased by one-fourth over the four years since the legislation was enacted. Analyst: The statistics are welcome news, but they do not provide strong evidence that the new laws caused the drop in crime. Many comparable areas that lack such legislation have reported a similar drop in the crime rate during the same period.", "question": "Which one of the following most accurately describes the strategy used by the analyst to call into question the legislator's argument?", "answers": "['arguing that the legislator has unreasonably concluded that one event has caused another without ruling out the possibility that both events are effects of a common cause', 'implying that the legislator has drawn a conclusion about cause and effect without considering how often the alleged effect has occurred in the absence of the alleged cause', 'objecting that the statistics on which the legislator is basing his conclusion are drawn from a time period that is too short to yield a meaningful data sample', \"pointing out that the legislator has provided no evidence of the reliability of the statistics on which the legislator's conclusion is based\"]", "label": 1 }, { "id": "train_1495", "context": "After purchasing a pot-bellied pig at the pet store in Springfield, Amy was informed by a Springfield city official that she would not be allowed to keep the pig as a pet, since city codes classify pigs as livestock, and individuals may not keep livestock in Springfield.", "question": "The city official's argument depends on assuming which one of the following?", "answers": "['Pigs are not classified as pets in Springfield.', 'Any animal not classified as livestock may be kept in Springfield.', 'Amy lives in Springfield.', 'Dogs and cats are not classified as livestock in Springfield.']", "label": 2 }, { "id": "train_1496", "context": "Most people who shop for groceries no more than three times a month buy prepared frozen dinners regularly. In Hallstown most people shop for groceries no more than three times a month. Ttherefore, in Hallstown most people buy prepared frozen dinners regularly.", "question": "Which one of the following arguments has a flawed pattern of reasoning most like the flawed reasoning in the argument above?", "answers": "[\"It is clear that Fernando's friends usually drive to school, since all of his friends can drive and all of his friends go to school.\", 'It is clear that John cannot drive, since he does not own a car and no one in his family who does not own a car can drive.', 'It is clear that most people in Highland County drive sedans, since most people who commute to work drive sedans and most people in Highland County commute to work.', 'It is clear that most drivers in West Ansland are safe drivers since there are very few driving accidents in West Ansland and most accidents there are not serious.']", "label": 2 }, { "id": "train_1497", "context": "In the year following an eight-cent increase in the federal tax on a pack of cigarettes, sales of cigarettes fell ten percent. In contrast, in the year prior to the tax increase, sales had fallen one percent. The volume of cigarette sales is ttherefore strongly related to the after-tax price of a pack of cigarettes.", "question": "Which of the following, if true, would most strengthen the argument above?", "answers": "['During the year following the cigarette tax increase, many consumers had less income, in inflation-adjusted dollars, than they had had in the previous year.', 'Most consumers were unaware that the tax on cigarettes was going to increase.', 'The information available to consumers on the health risks of smoking remained largely unchanged in the period before and after the tax increase.', 'During the year after the tax increase, there was a greater variety of cigarettes on the market than there had been during the previous year.']", "label": 2 }, { "id": "train_1498", "context": "A customer returning defective merchandise should be given an immediate refund if the merchandise was defective when purchased and was not on sale; if the customer is returning merchandise that is not defective or was damaged by customer negligence or customer abuse, the customer should be referred to the manager.", "question": "Which one of the following judgments conforms most closely to the principle stated above?", "answers": "['A customer purchased a lawn mower, which became inoperative after the customer used it only five times. Even though the lawn mower was not on sale when purchased, the customer should be referred to the manager.', 'A customer purchased a set of wrought-iron patio furniture at the regular price. The welding on two of the chairs was faulty, causing both to collapse during their first use. The customer should be given an immediate refund.', 'A customer purchased a microwave oven that stopped working after the customer accidentally dropped it. The oven was not on sale and was not damaged by abuse, so the customer should be given an immediate refund.', 'A customer purchased twelve bags of cement at the regular price, to build a walkway. The walkway was completed with only ten bags and the customer wishes to return the two remaining bags. The customer should be given an immediate refund.']", "label": 1 }, { "id": "train_1499", "context": "Medical school professor: Most malpractice suits arise out of patients' perceptions that their doctors are acting negligently or carelessly. Many doctors now regard medicine as a science rather than an art, and are less compassionate as a result. Harried doctors sometimes treat patients rudely, discourage them from asking questions, or patronize them. Lawsuits could be avoided if doctors learned to listen better to patients. Unfortunately, certain economic incentives encourage doctors to treat patients rudely.", "question": "The medical school professor's statements, if true, most strongly support which one of the following?", "answers": "['Doctors foster, by their actions, the perception that they do not really care about their patients.', 'Malpractice suits brought against doctors are, for the most part, unjustified.', 'The scientific outlook in medicine should be replaced by an entirely different approach to medicine.', 'Economic incentives to treat patients rudely are the main cause of doctors being sued for malpractice.']", "label": 0 }, { "id": "train_1500", "context": "Provinces and states with stringent car safety requirements, including required use of seat belts and annual safety inspections, have on average higher rates of accidents per kilometer driven than do provinces and states with less stringent requirements. Nevertheless, most highway safety experts agree that more stringent requirements do reduce accident rates.", "question": "Which one of the following, if true, most helps to reconcile the safety experts' belief with the apparently contrary evidence described above?", "answers": "['The roads in provinces and states with stringent car safety requirements are far more congested and ttherefore dangerous than in other provinces and states.', 'Provinces and states with stringent car safety requirements have, on average, many more kilometers of roads than do other provinces and states.', 'Annual safety inspections ensure that car tires are replaced before they grow old.', 'Drivers often become overconfident after their cars have passed a thorough safety inspection.']", "label": 0 }, { "id": "train_1501", "context": "Although marathons are 26. 2 miles (42. 2 kilometers) long and take even world-class marathoners over 2 hours to run, athletes who train by running 90 minutes a day fare better in marathons than do those who train by running 120 minutes or more a day.", "question": "Each of the following, if true, contributes to an explanation of the difference in marathon performances described above EXCEPT:", "answers": "['The longer the run, the more frequent is the occurrence of joint injuries that significantly interfere with overall training.', 'The longer the period of time that one runs daily, the easier it is to adjust to different race lengths.', 'Runners who train over 90 minutes per day grow bored with running and become less motivated.', 'The longer period of time that one runs daily, the greater the chances of suffering adverse health effects due to air pollution.']", "label": 1 }, { "id": "train_1502", "context": "Contrary to Malthus' s arguments, human foodproducing capacity has increased more rapidly than human population. Yet, agricultural advances often compromise biological diversity. Ttherefore, Malthus' s prediction that insufficient food will doom humanity to war, pestilence, and famine will likely be proven correct in the future, because a lack of biodiversity will eventually erode our capacity to produce food.", "question": "The statement that human food-producing capacity has increased more rapidly than human population plays which one of the following roles in the argument?", "answers": "['It is a part of the evidence used in the argument to support the conclusion that a well-known view is misguided.', 'It is a general fact that the argument offers reason to believe will eventually change.', \"It is an observation that the argument suggests actually supports Malthus's position.\", 'It is a hypothesis the argument provides reasons for believing to be presently false.']", "label": 1 }, { "id": "train_1503", "context": "It would be wrong to conclude that a person has a Streptococcus infection if there is no other evidence than the fact that Streptococcus bacilli are present in the person' s throat; after all, infection does not occur unless the host is physically run down.", "question": "The reasoning in which one of the following is most similar to the reasoning in the argument above?", "answers": "['When a bee colony fails to survive the winter, it would be wrong to conclude that low temperatures were the cause. Bees have very good defense mechanisms against extreme cold which are designed to ensure survival of the colony, though not of individual bees.', 'Even if a healthy lavender plant receives six or more hours of direct sunlight each day, one cannot predict on that basis alone that the plant will bloom, because lavender requires both six or more hours of sunlight per day and slightly alkaline soil to bloom.', \"When a person experiences blurred vision, it does not follow that a physical defect in the person's eyes Is the cause, since blurring of a person's vision also can be induced by certain drugs.\", 'A female holly plant cannot produce berries without a male plant nearby to provide pollen. But it does not follow that two or more male hollies in the vicinity will cause a female plant to bear more berries than it would with only a single male holly nearby.']", "label": 1 }, { "id": "train_1504", "context": "Area resident: Childhood lead poisoning has declined steadily since the 1970s, when leaded gasoline was phased out and lead paint was banned. But recent statistics indicate that 25 percent of this area' s homes still contain lead paint that poses significant health hazards. Ttherefore, if we eliminate the lead paint in those homes, childhood lead poisoning in the area will finally be eradicated.", "question": "The area resident's argument is flawed in that it", "answers": "['takes for granted that children reside in all of the homes in the area that contain lead paint', 'relies on an assumption that is tantamount to assuming that the conclusion is true', \"fails to consider that there may be other significant sources of lead in the area's environment\", 'relies on statistical claims that are likely to be unreliable']", "label": 2 }, { "id": "train_1505", "context": "A just government never restricts the right of its citizens to act upon their desires except when their acting upon their desires is a direct threat to the health or property of other of its citizens.", "question": "Which one of the following judgments most closely conforms to the principle cited above?", "answers": "['A just government would not censor writings of Shakespeare, but it could censor magazines and movies that criticize the government.', 'A just government would not ban the sale of sports cars, but it could prohibit unrestricted racing of them on public highways.', 'An unjust government would incarcerate one of its citizens even though it had been several years since that citizen harmed someone.', 'An unjust government would abolish many public services if these services did not require compulsory labor.']", "label": 1 }, { "id": "train_1506", "context": "In an experiment, each volunteer was allowed to choose between an easy task and a hard task and was told that another volunteer would do the other task. Each volunteer could also choose to have a computer assign the two tasks randomly. Most volunteers chose the easy task for themselves and under questioning later said they had acted fairly. But when the scenario was described to another group of volunteers, almost all said choosing the easy task would be unfair. This shows that most people apply weaker moral standards to themselves than to others.", "question": "Which of the following is an assumption required by this argument?", "answers": "['The most moral choice for the volunteers would have been to have the computer assign the two tasks randomly.', 'At least some volunteers given the choice between assigning the tasks themselves and having the computer assign them felt that they had made the only fair choice available to them.', 'On average, the volunteers to whom the scenario was described were more accurate in their moral judgments than the other volunteers were.', 'At least some volunteers who said they had acted fairly in choosing the easy task would have said that it was unfair for someone else to do so.']", "label": 3 }, { "id": "train_1507", "context": "Hatfield Airport, the closest airport to the city of Hatfield, has recently opened a new terminal for AirJet International, a popular low-cost airline. Since the terminal has opened, Hatfield Airport has welcomed significantly more customers, to the extent that airline staff members have complained that they can no longer serve their customers efficiently. If AirJet International were to move their major terminal to the more remote Chesterfield Airport, the staff argues, the customers of both AirJet and the airlines based at Hatfield Airport would be happier.", "question": "Which of the following is an assumption that supports drawing the conclusion above from the reasons given for that conclusion?", "answers": "[\"AirJet customers would not mind traveling further from Hatfield, since AirJet's low fares save them money.\", \"The increased traffic at the airport is due to the popularity of AirJet's flights.\", 'Customers are unhappy with the slowness of service at Hatfield Airport.', 'Other airlines based at Hatfield Airport are not as widely used as AirJet.']", "label": 1 }, { "id": "train_1508", "context": "Hine' s emerald dragonflies are an endangered species that live in wetlands. The larvae of these dragonflies can survive only in the water, where they are subject to predation by several species including red devil crayfish. Surprisingly, the dragonfly populations are more likely to remain healthy in areas where red devil crayfish are present than in areas without red devil crayfish.", "question": "Which one of the following, if true, most helps to explain the surprising fact?", "answers": "[\"Populations of red devil crayfish in a wetland do not drop significantly if the local population of Hine's emerald dragonflies dies out.\", 'Red devil crayfish dig chambers that remain filled with water even when the surrounding wetlands dry up.', \"Red devil crayfish present no threat to adult Hine's emerald dragonflies.\", 'The varied diet of the red devil crayfish does not include any animal species that prey on dragonfly larvae.']", "label": 1 }, { "id": "train_1509", "context": "Algorpal Industries produces high quality pneumatic filters. For years, they have primarily served manufacturers of domestic agricultural equipment, and over time, this market has been decreasing. A consultant recommended that, with a minor modification, Algorpal could expand into pneumatic filters for biomedical research technology, a rapidly expanding market. The consultant argued that this single change could reverse a ten-year decline in Algorpal' s profits.", "question": "Which of the following would be the most useful to establish in evaluating the consultant's recommendation?", "answers": "['Whether the number of biomedical devices at any single research center is greater than the number of pieces of equipment requiring filters on a single farm.', 'Whether other markets, such as food processing and food service, would offer greater opportunities for potential profits.', 'Whether the competition in the biomedical technology sector would allow for profits similar to what those in the agricultural sector had been', 'How the ambient indoor air quality in a biomedical research center compares to the typical outdoor air quality in an agricultural zone.']", "label": 2 }, { "id": "train_1510", "context": "At the end of the year, Wilson' s Department Store awards free merchandise to its top salespeople. When presented with the fact that the number of salespeople receiving these awards has declined markedly over the past fifteen years, the newly appointed president of the company responded, \"In that case, since our award criterion at present is membership in the top third of our sales force, we can also say that the number of salespeople passed over for these awards has similarly declined. ", "question": "Which one of the following is an assumption that would allow the company president's conclusion to be properly drawn?", "answers": [ "Wilson's calculates its salespeople's sales figures in the same way as it did fifteen years ago.", "Policies at Wilson's with regard to hiring salespeople have not become more lax over the past fifteen years.", "The number of salespeople at Wilson's has increased over the past fifteen years.", "The criterion used by Wilson's for selecting its award recipients has remained the same for the past fifteen years." ], "label": 3 }, { "id": "train_1511", "context": "People cannot devote themselves to the study of natural processes unless they have leisure, and people have leisure when resources are plentiful, not when resources are scarce. Although some anthropologists claim that agriculture, the cultivation of crops, actually began under conditions of drought and hunger, the early societies that domesticated plants must first have discovered how the plants they cultivated reproduced themselves and grew to maturity. These complex discoveries were the result of the active study of natural processes.", "question": "The argument is structured to lead to the conclusion that", "answers": "['early societies could have discovered by accident how the plants they cultivated reproduced and grew', 'agriculture first began in societies that at some time in their history had plentiful resources', 'early agricultural societies knew more about the natural sciences than did early nonagricultural societies', 'whenever a society has plentiful resources, some members of that society devote themselves to the study of natural processes']", "label": 1 }, { "id": "train_1512", "context": "The emission of sulfur dioxide when high-sulfur coal is burned is restricted by law. New coal-burning plants usually comply with the law by installing expensive equipment to filter sulfur dioxide from their emissions. These new plants could save money by installing instead less expensive cleaning equipment that chemically removes most sulfur from coal before combustion.", "question": "Which of the following, if known, would be most relevant to evaluating the claim above about how new coal-burning plants could save money?", "answers": "['Whether the expense of installing the cleaning equipment in a new plant is less than the expense of installing the cleaning equipment in an older plant', 'Whether the process of cleaning the coal is more expensive than the process of filtering the emissions', 'Whether existing oil-burning plants are required to filter sulfur dioxide from their emissions', 'Whether existing plants that use the filtering equipment could replace this equipment with the cleaning equipment and still compete with new plants that install the cleaning equipment']", "label": 1 }, { "id": "train_1513", "context": "A manager cannot extract the best performance from employees by threatening them with termination or offering financial rewards for high productivity. Rather, employees must come to want to do a good job for its own sake. One of the best ways for a manager to achieve this is to delegate responsibility to them, especially for decisions that previously had to be made by the manager.", "question": "Which one of the following propositions is best illustrated by the situation described in the passage?", "answers": "[\"Increased responsibility can improve a person's sense of how power should be used.\", 'People who carry out decisions are in the best position to determine what those decisions should be.', \"In some cases one's effectiveness in a particular role can be enhanced by a partial relinquishing of control.\", 'Business works best by harnessing the self-interest of individuals to benefit the company as a whole.']", "label": 2 }, { "id": "train_1514", "context": "People who say that Dooney County is flat are clearly wrong. On flat land, soil erosion by water is not a problem. Consequently, farmers whose land is flat do not build terraces to prevent erosion. Yet I hear that the farms in Dooney County are dotted with terraces.", "question": "The author's conclusion in the passage depends on the assumption that", "answers": "['on flat land there is no soil erosion', 'the only terraces in Dooney County are on farmland', 'there are terraces on farmland in Dooney County which were built to prevent soil erosion', 'the only cause of soil erosion is water']", "label": 2 }, { "id": "train_1515", "context": "Situation: Sometimes people follow unhealthy diets to lose weight more quickly. Analysis: People sometimes prioritize perceived beauty over health.", "question": "The analysis provided for this situation is most similar to which of the following situations?", "answers": "['An actor starts brushing his teeth with new special whitening toothpaste that contains several known carcinogens.', 'A musician buys a new leather jacket in preparation for an upcoming show.', 'A couple joins a gym to start running more.', 'A model undergoes an elective cosmetic procedure to appear thinner.']", "label": 0 }, { "id": "train_1516", "context": "A study of 8, 000 randomly chosen adults in a European country found that 75 percent of those who consumed alcohol an average of twice or more per week over a six-month period suffered at least one cold during that period. We can tentatively conclude from this that consuming alcohol tends to increase one' s susceptibility to colds.", "question": "Which one of the following, if true, would most weaken the argument?", "answers": "['Sixty percent of those in the study who did not suffer any colds over the six-month period consumed alcohol once per week or more.', 'Seventy-five percent of those in the study who never consumed alcohol suffered at least one cold over the six-month period.', 'Fifty percent of those in the study who consumed alcohol an average of once per week suffered colds over the six-month period.', 'Eighty percent of those in the study who consumed alcohol three times or more per week suffered colds over the six-month period.']", "label": 1 }, { "id": "train_1517", "context": "The body of anyone infected by virus X will, after a week, produce antibodies to fight the virus; the antibodies will increase in number for the next year or so. There is now a test that reliably indicates how many antibodies are present in a person' s body. If positive, this test can be used during the first year of infection to estimate to within a month how long that person has had the virus.", "question": "Which one of the following conclusions is best supported by the statements above?", "answers": "[\"If a person remains infected by virus X indefinitely, there is no limit to the number of antibodies that can be present in the person's body.\", 'Anyone infected by virus X will for a time fail to exhibit infection if tested by the antibody test.', 'Antibodies are produced only for viral infections that cannot be fought by any other body defenses.', 'Antibodies increase in number only until they have defeated the virus.']", "label": 1 }, { "id": "train_1518", "context": "Political Analyst: \"Although citizens of this state normally oppose any new taxes, they are overwhelmingly in favor of taxes that support the medical initiative. Candidate Johnson vowed to cut these taxes, and he was trounced in the primary elections. Furthermore, in a poll that asked citizens, \"Would you pay higher taxes if it meant having the benefit of the new medical initiative? \", an astonishing 82% replied \"Yes. \" This is a pattern of support for taxes we have not seen before in this state. ", "question": "In the political analyst's argument, the portion in boldface plays which of the following roles?", "answers": "['It is an explanation that the argument concludes is correct.', 'It is a finding that calls the main conclusion into question.', 'It provides evidence in support of the main conclusion of the argument.', 'It is the main conclusion of the argument.']", "label": 2 }, { "id": "train_1519", "context": "Environmentalist: The United Kingdom recently instituted a law requiring that foods containing genetically altered ingredients be labeled accordingly. Food producers, fearing that consumers would understand the labels as warnings and thus avoid their products, rushed to rid those products of genetically altered ingredients. Other countries contemplating such labeling should ttherefore refrain, because many crops are genetically altered to be pest resistant; loss of demand for these genetically altered crops would necessitate production alternatives, all of which are dangerous and pesticide intensive.", "question": "Which one of the following is an assumption on which the environmentalist's argument depends?", "answers": "['Countries that institute new food labeling regulations often experience a change in consumer eating habits.', 'The reactions of food producers in other countries to laws requiring labeling of foods containing genetically altered ingredients are likely to be similar to the reactions of food producers in the United Kingdom.', 'Warning labels on food products have proven to be effective in reducing consumption of those products.', 'In general, people interpret labels stating that some food ingredients are genetically altered as warnings.']", "label": 1 }, { "id": "train_1520", "context": "Omega Corporation has hired two management consultant firms, ManageFix! and EfficiencyRun, to help cut unnecessary costs by streamlining its corporate structure. By following ManageFix! ' s plan, Omega Corp. will save more money than it would by following EfficiencyRun' s plan. Ttherefore, by following ManageFix! ' s plan, Omega Corp. will be doing the most that can be done to cut unnecessary costs due to inefficient corporate structure.", "question": "Which of the following is an assumption on which the argument depends?", "answers": "['Omega Corp. has not yet determined with certainty that it wishes to make changes in its corporate structure.', \"Unnecessary costs cannot be reduced more by implementing both plans together than by implementing ManageFix! 's alone.\", \"Omega Corp. can implement ManageFix! 's plan in one of several ways, some of which are more expensive than others.\", 'Other than in its corporate structure, Omega Corp. does not waste money in other areas.']", "label": 1 }, { "id": "train_1521", "context": "Since John Lindy was named vice president of sales, the highest ranked salespeople have been successfully recruited by rival companies, sales in all departments have plummeted, and company morale is down. This information clearly provides proof that John Lindy was named vice president of sales to ruin the company.", "question": "The reasoning in the argument is weak because the argument", "answers": "['presupposes that because conditions resulted from a change, the change was made to create those conditions', 'fails to show proof that the company has been ruined by John Lindy being named vice president of sales', 'fails to show how company morale is related to sales plummeting in all departments', 'bases its conclusion on hearsay rather that hard evidence about the effect John Lindy has had on the company']", "label": 0 }, { "id": "train_1522", "context": "Meteorologist: The number of tornadoes reported annually has more than doubled since the 1950s. But their actual number has probably not increased. Our ability to find tornadoes has improved, so we' re probably just finding a higher percentage of them than we used to.", "question": "Which one of the following, if true, provides the most support for the meteorologist's argument?", "answers": "['The annual number of deaths due to tornadoes has increased steadily since the 1950s.', 'The number of tornadoes hitting major population centers annually has more than doubled since the 1950s.', 'The geographic range in which tornadoes are most prevalent has remained roughly constant since the 1950s.', 'The number of large and medium sized tornadoes reported annually has remained roughly constant since the 1950s.']", "label": 3 }, { "id": "train_1523", "context": "Zachary: One would have to be blind to the reality of moral obligation to deny that people who believe a course of action to be morally obligatory for them have both the right and the duty to pursue that action, and that no one else has any right to stop them from doing so. Cynthia: But imagine an artist who feels morally obliged to do whatever she can to prevent works of art from being destroyed confronting a morally committed antipornography demonstrator engaged in destroying artworks he deems pornographic. According to your principle that artist has, simultaneously, both the right and the duty to stop the destruction and no right whatsoever to stop it.", "question": "Cynthia's response to Zachary's claim is structured to demonstrate that", "answers": "[\"Zachary's principle is untenable on its own terms\", \"Zachary's principle is based on an understanding of moral obligation that is too narrow to encompass the kind of moral obligation artists feel toward works of art\", 'because the term \"moral obligation\" is understood differently by different people, it is impossible to find a principle concerning moral rights and duties that applies to everyone', 'the concept of moral obligation is incoherent']", "label": 0 }, { "id": "train_1524", "context": "Economist: If the belief were to become widespread that losing one' s job is not a sign of personal shortcomings but instead an effect of impersonal social forces (which is surely correct), there would be growth in the societal demand for more government control of the economy to protect individuals from these forces, just as the government now protects them from military invasion. Such extensive government control of the economy would lead to an economic disaster, however.", "question": "The economist's statements, if true, most strongly support which one of the following?", "answers": "['In general, people should feel no more responsible for economic disasters than for military invasions.', 'Increased knowledge of the causes of job loss could lead to economic disaster.', 'Governments should never interfere with economic forces.', 'Societal demand for government control of the economy is growing.']", "label": 1 }, { "id": "train_1525", "context": "Hana said she was not going to invite her brothers to her birthday party. However, among the gifts Hana received at her party was a recording in which she had expressed an interest. Since her brothers had planned to give her that recording, at least some of Hana' s brothers must have been among the guests at Hana' s birthday party after all.", "question": "A reasoning error in the argument is that the argument", "answers": "['uses a term that is intrinsically evaluative as though that term was purely descriptive', 'fails to establish that something true of some people is true of only those people', 'disregards the possibility that a change of mind might be justified by a change in circumstances', \"overlooks the possibility that a person's interest in one kind of thing is compatible with that person's interest in a different kind of thing\"]", "label": 1 }, { "id": "train_1526", "context": "Editor: When asked to name a poet contemporaneous with Shakespeare, 60 percent of high school students picked a twentieth-century poet. Admittedly, it is hard to interpret this result accurately. Does it show that most high school students do not know any poets of Shakespeare' s era, or do they just not know what \"contemporaneous\" means? However, either way, there is clearly something deeply wrong with the educational system.", "question": "The statement that the majority of students picked a twentieth-century poet functions primarily in the argument", "answers": "['as evidence that something is deeply wrong with the educational system', 'as evidence that the educational system is producing students who are ignorant of the history of poetry', 'as evidence of the ambiguity of some questions', 'to illustrate that research results are difficult to interpret']", "label": 0 }, { "id": "train_1527", "context": "Dr. Z: Many of the characterizations of my work offered by Dr. Q are imprecise, and such characterizations do not provide an adequate basis for sound criticism of my work.", "question": "Which one of the following can be properly inferred from Dr. Z's statement?", "answers": [ "Some of Dr. Q's characterizations of Dr. Z's work provide an adequate basis for sound criticism of Dr. Z's work.", "At least one of Dr. Q's characterizations of Dr. Z's work fails to provide an adequate basis for sound criticism of that work.", "All of Dr. Q's characterizations of Dr. Z's work that are not imprecise provide an adequate basis for sound criticism of Dr. Z's work.", "If the characterization of someone's work is precise, then it provides a sound basis for criticizing that work." ], "label": 1 }, { "id": "train_1528", "context": "Market analyst: According to my research, 59 percent of consumers anticipate paying off their credit card balances in full before interest charges start to accrue, intending to use the cards only to avoid carrying cash and writing checks. This research also suggests that in trying to win business from their competitors, credit card companies tend to concentrate on improving the services their customers are the most interested in. Ttherefore, my research would lead us to expect that __.", "question": "Which one of the following most logically completes the market analyst's argument?", "answers": "[\"most consumers would be indifferent about which company's credit card they use\", 'credit card companies would not make the interest rates they charge on cards the main selling point', 'most consumers would prefer paying interest on credit card debts over borrowing money from banks', 'the most intense competition among credit card companies would be over the number of places that they can get to accept their credit card']", "label": 1 }, { "id": "train_1529", "context": "The postmodern view involves the rejection of modern assumptions about order and the universality of truth. The grand theories of the modern era are now seen as limited by the social and historical contexts in which they were elaborated. Also, the belief in order has given way to a belief in the importance of irregularity and chaos. It follows that we inhabit a world full of irregular events, and in which there are no universal truths.", "question": "The argument's reasoning is questionable because the argument", "answers": "['uses the term \"order\" ambiguously', 'relies on the use of emotional terms to bolster its conclusion', 'infers that something is the case because it is believed to be the case', 'uses the term \"universal\" ambiguously']", "label": 2 }, { "id": "train_1530", "context": "The number of serious traffic accidents (accidents resulting in hospitalization or death) that occurred on Park Road from 1986 to 1990 was 35 percent lower than the number of serious accidents from 1981 to 1985. The speed limit on Park Road was lowered in 1986. Hence, the reduction of the speed limit led to the decrease in serious accidents.", "question": "Which one of the following statements, if true, most weakens the argument?", "answers": "['Until 1986 accidents were classified as \"serious\" only if they resulted in an extended hospital stay.', 'The annual number of accidents on Park Road that did not result in hospitalization remained roughly constant from 1981 to 1990.', 'The annual number of vehicles using Park Road decreased significantly and steadily from 1981 to 1990.', 'The number of speeding tickets issued annually on Park Road remained roughly constant from 1981 to 1990.']", "label": 2 }, { "id": "train_1531", "context": "All of John' s friends say they know someone who has smoked 40 cigarettes a day for the past 40 years and yet who is really fit and well. John does not know anyone like that and it is quite certain that he is not unique among his friends in this respect.", "question": "If the statements in the passage are true, then which one of the following must also be true?", "answers": "[\"Some of John's friends are not telling the truth.\", \"Most of John's friends are not telling the truth.\", 'Smokers often lie about how much they smoke.', \"All John's friends know the same lifelong heavy smoker.\"]", "label": 0 }, { "id": "train_1532", "context": "Ethicist: On average, animals raised on grain must be fed sixteen pounds of grain to produce one pound of meat. A pound of meat is more nutritious for humans than a pound of grain, but sixteen pounds of grain could feed many more people than could a pound of meat. With grain yields leveling off, large areas of farmland going out of production each year, and the population rapidly expanding, we must accept the fact that consumption of meat will soon be morally unacceptable.", "question": "Which one of the following, if true, would most weaken the ethicist's argument?", "answers": "['Often, cattle or sheep can be raised to maturity on grass from pastureland that is unsuitable for any other kind of farming.', 'Although prime farmland near metropolitan areas is being lost rapidly to suburban development, we could reverse this trend by choosing to live in areas that are already urban.', 'If a grain diet is supplemented with protein derived from non-animal sources, it can have nutritional value equivalent to that of a diet containing meat.', 'Nutritionists agree that a diet composed solely of grain products is not adequate for human health.']", "label": 0 }, { "id": "train_1533", "context": "A local group had planned a parade for tomorrow, but city hall has not yet acted on its application for a permit. The group had applied for the permit well in advance, had made sure their application satisfied all the requirements, and was clearly entitled to a permit. Although the law prohibits parades without a permit, the group plans to proceed with its parade. The group' s leader defended its decision by appealing to the principle that citizens need not refrain from actions that fail to comply with the law if they have made a good-faith effort to comply but are prevented from doing so by government inaction.", "question": "Which one of the following actions would be justified by the principle to which the leader of the group appealed in defending the decision to proceed?", "answers": "[\"A real-estate developer obtained a permit to demolish a historic apartment building that had not yet been declared a governmentally protected historic landmark. Despite the protests of citizens' groups, the developer then demolished the building.\", 'The law requires that no car be operated without a certain amount of insurance coverage. But since the authorities have been unable to design an effective procedure for prosecuting owners of cars that are driven without insurance, many car owners are allowing their insurance to lapse.', \"A physician who had been trained in one country applied for a license to practice medicine in another country. Although he knew he met all the qualifications for this license, he had not yet received it one year after he applied for it. He began to practice medicine without the license in the second country despite the law's requirement for a license.\", 'A city resident applied for rezoning of her property so that she could build a bowling alley in a residential community. She based her application on the need for recreational facilities in the community. Her application was turned down by the zoning board, so she decided to forego construction.']", "label": 2 }, { "id": "train_1534", "context": "A person' s personality is linked to that person' s genes. And since a person' s genes do not ordinarily change over time, it follows that a person' s personality remains unchanged with the passing of time.", "question": "Which one of the following is most closely parallel in its reasoning to the flawed reasoning in the argument above?", "answers": "[\"The levels of certain hormones control body temperature. Ttherefore, if one has a high fever, the levels of one's hormones must be elevated as well.\", \"Market forces are to some degree influenced by governmental actions. Hence, a change in the government's policies could result in a change in the economy.\", 'It is well known that some diseases have genetic causes. Ttherefore, it should be possible to prevent such diseases by manipulating the genes that cause them.', \"The way historians understand the First World War is related to what happened in that war. But what actually happened in that war cannot change. Ttherefore, historians' understanding of the war cannot change.\"]", "label": 3 }, { "id": "train_1535", "context": "To allay public concern about chemicals that are leaking into a river from a chemical company' s long-established dump, a company representative said, \"Federal law requires that every new chemical be tested for safety before it is put onto the market. This is analogous to the federal law mandating testing of every pharmaceutical substance for safety. ", "question": "Which one of the following, if true, most seriously weakens the representative's implied argument that the public need not be concerned about the leak?", "answers": "['Leakage from the dump has occurred in noticeable amounts only in the last few months.', 'Before the federal law requiring testing of nonpharmaceutical chemicals went into effect recently, there were 40, 000 such chemicals being manufactured, many of them dangerous.', 'When pharmaceutical substances are tested for safety pursuant to federal requirements, a delay is imposed on the entry of potentially lifesaving substances onto the market.', 'The concentration of chemicals leaking into the river is diluted, first by rainwater and then by the water in the river.']", "label": 1 }, { "id": "train_1536", "context": "The companies that are the prime purchasers of computer software will not buy a software package if the costs of training staff to use it are high, and we know that it is expensive to teach people a software package that demands the memorization of unfamiliar commands. As a result, to be successful, commercial computer software cannot require users to memorize unfamiliar commands.", "question": "The conclusion above follows logically if which one of the following is assumed?", "answers": "['The more difficult it is to learn how to use a piece of software, the more expensive it is to teach a person to use that software.', 'If the initial cost of computer software is high, but the cost of training users is low, prime purchasers will still buy that software.', 'Commercial computer software will not be successful unless prime purchasers buy it.', 'If most prime purchasers of computer software buy a software product, that product will be successful.']", "label": 2 }, { "id": "train_1537", "context": "Red admiral butterflies fly in a highly irregular fashion, constantly varying their speed, wing strokes, and flight path. While predators avoid poisonous butterfly species, nonpoisonous butterflies like the red admiral need to elude predators to survive. Scientists ttherefore hypothesize that the red admiral' s flight style, which is clearly not energy efficient, evolved as a means of avoiding predators.", "question": "Which one of the following, if true, most strengthens the support for the scientists' hypothesis?", "answers": "['Attacks from predators are not the most common cause of death for butterflies.', 'All of the predators that prey on the red admiral also prey on other species of nonpoisonous butterflies.', 'It is much more energy efficient for butterflies to fly in an irregular fashion than it is for heavier varieties of insects.', 'No species of poisonous butterfly has an irregular flight style like that of the red admiral.']", "label": 3 }, { "id": "train_1538", "context": "Most people feel that they are being confused by the information from broadcast news. This could be the effect of the information' s being delivered too quickly or of its being poorly organized. Analysis of the information content of a typical broadcast news story shows that news stories are far lower in information density than the maximum information density with which most people can cope at any one time. So the information in typical broadcast news stories is poorly organized.", "question": "Which one of the following is an assumption that the argument requires in order for its conclusion to be properly drawn?", "answers": "['Poor organization of information in a news story makes it impossible to understand the information.', 'Being exposed to more broadcast news stories within a given day would help a person to better understand the news.', 'It is not the number of broadcast news stories to which a person is exposed that is the source of the feeling of confusion.', 'Some people are being overwhelmed by too much information.']", "label": 2 }, { "id": "train_1539", "context": "Magazine article: Sugar consumption may exacerbate attention deficit disorder (ADD) in children. A recent study found that children produce large amounts of adrenaline within hours after consuming large amounts of sugar. This increase in adrenaline is especially noticeable if the source of sugar is candy, in which case the sugar' s effects are not ameliorated by the ingestion of other foodstuffs.", "question": "Which one of the following is an assumption on which the argument in the magazine article depends?", "answers": "['Increased adrenaline production can make ADD more severe in children.', 'Sugar consumed with food substances other than candy does not substantially increase the level of adrenaline in the bloodstream of children with ADD.', 'The adrenaline level of children who do not have ADD is not increased by excessive sugar consumption.', 'The most effective way to treat ADD in children is to restrict their intake of sugars.']", "label": 0 }, { "id": "train_1540", "context": "Those who have the ability to fully concentrate are always of above-average intelligence. Also, being successfully trained in speed-reading will usually be accompanied by an increased ability to concentrate.", "question": "If the statements above are true, then each of the following could be true EXCEPT:", "answers": "['All people who can speed-read are able to concentrate to some extent.', 'Some people can speed-read, and are able to fully concentrate, but are of below-average intelligence.', 'All people who can speed-read are of above average intelligence.', 'Many people of above-average intelligence are unable to fully concentrate.']", "label": 1 }, { "id": "train_1541", "context": "Manager: I recommend that our company reconsider the decision to completely abandon our allegedly difficult-to-use computer software and replace it companywide with a new software package advertised as more flexible and easier to use. Several other companies in our region officially replaced the software we currently use with the new package, and while their employees can all use the new software, unofficially many continue to use their former software as much as possible.", "question": "Which one of the following is most strongly supported by the manager's statements?", "answers": "[\"The employees of the manager's company would find that the new software package lacks some of the capabilities of the present software.\", 'The familiarity that employees have with a computer software package is a more important consideration in selecting software than flexibility or initial ease of use.', 'Adopting the new software package would create two classes of employees, those who can use it and those who cannot.', \"Many of the employees in the manager's company would not prefer the new software package to the software currently in use.\"]", "label": 3 }, { "id": "train_1542", "context": "Scientist: All other things being equal, the intensity of heat increases as the distance from the heat source decreases. Knowing this, most people conclude that the Earth's seasons are caused by the Earth's changing distance from the sun. In other words, winter occurs when the Earth is far from the sun, and summer occurs when the earth is close to the sun. However, we know that as North America experiences summer, South America experiences winter, even though the difference in the continents' distance to the sun is negligible . Ttherefore, the earth's changing distance from the sun does not cause the seasons.", "question": "In the argument, the two portions in boldface play which of the following roles?", "answers": "['The first describes a common belief that the scientist later deems to be ill conceived; the second is evidence against this belief.', \"The first describes a commonly held belief that is contrary to the scientist's final conclusion; the second is evidence in support of this belief.\", \"The first is a statement presented in support of the scientist's final conclusion; the second is the scientist's conclusion.\", 'The first describes a common belief that follows logically from the statement before it; the second is factual information that the scientist deems to be irrelevant to her argument.']", "label": 0 }, { "id": "train_1543", "context": "Psychologist: There are theories that posit completely different causal mechanisms from those posited by Freudian psychological theory and that are more successful at predicting human behavior. Ttherefore, Freudian theories of behavior, no matter how suggestive or complex they are, ought to be abandoned in favor of these other theories.", "question": "Which one of the following is an assumption on which the psychologist's argument depends?", "answers": "['Freudian theories have offered interesting suggestions, which have been shown to be false, about the causes of human behavior.', 'Freudian theory has had little success in predicting how people will behave in various situations.', 'Measuring the predictive success of a psychological theory involves considering other theories that attempt to explain the same phenomena.', 'A psychological theory with greater predictive success than another is scientifically preferable to it.']", "label": 3 }, { "id": "train_1544", "context": "North Americans who travel to Europe for the first time should include significant time in Italy on their itinerary. To develop an appreciation of a continent that goes beyond the mere accumulation of impressions, one needs to acquire a thorough knowledge of at least one country, and North Americans seem to find it easier to get to know Italy than other European countries.", "question": "Which one of the following best illustrates the principle illustrated by the argument above?", "answers": "['To overcome a fear of water that prevents one from swimming, one should paddle about in shallow water with a trusted friend who is a good swimmer.', 'One should travel through North America by train rather than by automobile, because train travel imparts the same sense of open space as does automobile travel, while also affording one the full leisure to attend to the scenery.', 'Edith Wharton is the most accessible of the classical U. S. writers. So in order to provide a superb introduction to U. S. literature, a class should emphasize her work while also studying the works of others.', 'One can appreciate Taiko-drumming only if one understands how physically demanding it is. Thus, one should see Taiko-drumming and not just hear it in order to appreciate it fully.']", "label": 2 }, { "id": "train_1545", "context": "Consumer advocate: Last year' s worldwide alarm about a computer \"virus\" -- a surreptitiously introduced computer program that can destroy other programs and data -- was a fraud. Companies selling programs to protect computers against such viruses raised worldwide concern about the possibility that a destructive virus would be activated on a certain date. There was more smoke than fire, however, only about a thousand cases of damage were reported around the world. Multitudes of antivirus programs were sold, so the companies' warning was clearly only an effort to stimulate sales.", "question": "The reasoning in the consumer advocate's argument is flawed because this argument", "answers": "['overlooks the possibility that the protective steps taken did work and, for many computers, prevented the virus from causing damage', 'restates its conclusion without attempting to offer a reason to accept it', 'asserts that the occurrence of one event after another shows that the earlier event was the cause of the later one', 'uses inflammatory language as a substitute for providing any evidence']", "label": 0 }, { "id": "train_1546", "context": "Proponents of the electric car maintain that when the technical problems associated with its battery design are solved, such cars will be widely used and, because they are emission-free, will result in an abatement of the environmental degradation caused by auto emissions. But unless we dam more rivers, the electricity to charge these batteries will come from nuclear or coal-fired power plants. Each of these three power sources produces considerable environmental damage. Thus, the electric car __.", "question": "Which one of the following most logically completes the argument?", "answers": "['will have worse environmental consequences than its proponents may believe', 'will probably remain less popular than other types of cars', 'will not produce a net reduction in environmental degradation', 'requires that purely technical problems be solved before it can succeed']", "label": 0 }, { "id": "train_1547", "context": "Research indicates that 90 percent of extreme insomniacs consume large amount of coffee. Since Tom drinks a lot of coffee, it is quite likely that he is an extreme insomniac.", "question": "Which one of the following most accurately describes a flaw in the argument's reasoning?", "answers": "['It draws an inference about one specific individual from evidence that describes only the characteristics of a class of individuals', 'It fails to consider the possible contribution to extreme insomnia of other causes of insomnia besides coffee.', 'It fails to acknowledge the possibility that Tom is among the 10 percent of people who drink large amounts of coffee who are not extreme insomniacs.', 'It relies on evidence that does not indicate the frequency of extreme insomnia among people who drink large amounts of coffee.']", "label": 3 }, { "id": "train_1548", "context": "An experiment was done in which human subjects recognize a pattern within a matrix of abstract designs and then select another design that completes that pattern. The results of the experiment were surprising. The lowest expenditure of energy in neurons in the brain was found in those subjects who performed most successfully in the experiments.", "question": "Which of the following hypotheses best accounts for the findings of the experiment?", "answers": "[\"The energy expenditure of the subjects' brains increases when a design that completes the initially recognized pattern is determined.\", 'Those who performed best in the experiment experienced more satisfaction when working with abstract patterns than did those who performed less well.', 'The task of completing a given design is more capably performed by athletes, whose energy expenditure is lower when they are at rest.', 'People who are better at abstract pattern recognition have more energy-efficient neural connections.']", "label": 3 }, { "id": "train_1549", "context": "Dr. Schilling: Those who advocate replacing my country' s private health insurance system with. nationalized health insurance because of the rising costs of medical care fail to consider the high human costs that consumers pay in countries with nationalized insurance: access to high-technology medicine is restricted. Kidney transplants and open-heart surgery-familiar life-saving procedures are rationed. People are denied their right to treatments they want and need. Dr. Laforte: Your country' s reliance on private health insurance denies access even to basic, conventional medicine to the many people who cannot afford adequate health coverage. With nationalized insurance, rich and poor have equal access to life-saving medical procedures, and people' s right to decent medical treatment regardless of income is not violated.", "question": "In responding to Dr. Schilling, Dr. Laforte employs which one of the following argumentative strategies?", "answers": "['demonstrating that Dr. Schilling\\'s reasoning is persuasive only because of his ambiguous use of the key word \"consumer\"', \"producing counterexamples to Dr. Schilling's claims that nationalized health insurance schemes extract high human costs from consumers\", 'showing that the objections raised by Dr. Schilling have no bearing on the question of which of the two systems under consideration is the superior system', \"showing that the force of Dr. Schilling's criticism depends on construing the key notion of access in a particular limited way\"]", "label": 3 }, { "id": "train_1550", "context": "A major chemical spill occurred five years ago at Baker' s Beach, the world' s sole nesting ground for Merrick sea turtles, and prevented nearly all the eggs laid that year from hatching. Yet the number of adult female Merricks returning to lay their eggs at Baker' s Beach has actually increased somewhat since five years ago. Clearly, environmentalists' prediction that the world' s Merrick population would decline as a result of the spill has proven unfounded.", "question": "Which of the following, if true, most seriously undermines the argument offered in refutation of the environmentalists' prediction?", "answers": "[\"Under normal conditions, only a small proportion of hatchling female Merrick sea turtles survive in the ocean until adulthood and return to lay their eggs at Baker's Beach.\", 'Environmental pressures unrelated to the chemical spill have caused a significant decline in the population of one of the several species of sea birds that prey on Merrick sea turtle eggs.', \"The chemical spill five years ago occurred at a time when there were neither Merrick sea turtles nor Merrick sea turtle eggs on Baker's Beach.\", \"Female Merrick sea turtles begin returning to Baker's Beach to lay their eggs when they are ten years old.\"]", "label": 3 }, { "id": "train_1551", "context": "Editorial: The roof of Northtown Council' s equipment-storage building collapsed under the weight of last week' s heavy snowfall. The building was constructed recently and met local building-safety codes in every particular, except that the nails used for attaching roof supports to the building' s columns were of a smaller size than the codes specify for this purpose. Clearly, this collapse exemplifies how even a single, apparently insignificant, departure from safety standards can have severe consequences.", "question": "Which of the following, if true, most strongly weakens the editorial's argument?", "answers": "['Because the equipment-storage building was not intended for human occupation, some safety-code provisions that would have applied to an office building did not apply to it.', 'The amount of snow that accumulated on the roof of the equipment-storage building was greater than the predicted maximum that was used in drawing up the safety codes.', 'The only other buildings whose roofs collapsed from the weight of the snowfall were older buildings constructed according to less exacting standards than those in the safety codes.', 'Because the equipment-storage building was where the council kept snow-removal equipment, the building was almost completely empty when the roof collapsed.']", "label": 1 }, { "id": "train_1552", "context": "Tent caterpillars' routes between their nests and potential food sources are marked with chemical traces called pheromones that the caterpillars leave behind. Moreover, routes from food sources back to the nest are marked more heavily than are merely exploratory routes that have failed to turn up a food source. Thus, tent caterpillars are apparently among the insect species that engage in communal foraging, which consists in the conveying of information concerning the location of food to other members of the colony, nest, or hive.", "question": "Which one of the following, if true, adds the most support to the argument?", "answers": "['Sometimes individual tent caterpillars will not return to the nest until a food source is located.', 'The pheromones that tent caterpillars leave behind are detectable by certain other species of caterpillars.', 'A hungry tent caterpillar is more likely to follow heavily marked routes than lightly marked routes.', 'Tent caterpillars can detect the presence but not the concentration of pheromones.']", "label": 2 }, { "id": "train_1553", "context": "An unknown simian virus recently killed off nearly half the human population of a remote jungle town. Because the disease spread at an alarming rate-victims usually exhibited signs within 8 hours of exposure, and many died within 36 hours-the simian virus, if an outbreak occurs again, is likely to cause more deaths than has any other previous virus.", "question": "Which of the following pieces of information most effectively calls into question the validity of the conclusion?", "answers": "['Viruses tend to be most destructive in densely populated areas, oftentimes spreading amongst thousands of people before any quarantine can be put in place.', 'The Zairean strain of the Ebola virus, known as Zaire ebolavirus, has a mortality rate of nearly 75%.', 'The town in which the outbreak occurred had only fifty people, of whom twenty-four died.', 'Viruses that have very little time to incubate before destroying their hosts tend not to lead to widespread epidemics, because quarantines are put in place before the disease can spread.']", "label": 3 }, { "id": "train_1554", "context": "A person reading a new book for pleasure is like a tourist traveling to a new place. The reader reads, just as the tourist travels, to enlarge understanding rather than simply to acquire information. Thus, it is better to read fewer books and spend more time on each rather than to quickly read as many as one can, just as it is better to travel to fewer places and spend more time in each rather than to spend a small amount of time in many different places.", "question": "Which one of the following, if true, most strengthens the argument?", "answers": "['Tourists gain much more understanding of a place once they have spent several days at that place than they do in their first few days there.', 'Tourists typically learn something about the places they visit even when they are there only to relax.', 'Many people report that they can learn far more about a place by visiting it than they can by reading about it.', 'Tourists who have read about a place beforehand tend to stay longer in that place.']", "label": 0 }, { "id": "train_1555", "context": "Some scientists believe that small humanoid skeletons found on an Indonesian island are the remains of human beings with a growth disorder. It is more likely that they represent a distinct human species that became smaller over time due to environmental pressure. These skeletons do not fit the pattern of known growth disorders. And evidence suggests that certain fox and mouse species on the island have evolved into smaller versions of their common counterparts.", "question": "Which one of the following most accurately expresses the conclusion drawn in the argument?", "answers": "['Certain fox and mouse species on an Indonesian island have evolved into smaller versions of their common counterparts.', 'Environmental pressure can cause species living on islands to become smaller over time.', 'The humanoid skeletons do not fit the pattern of known growth disorders.', 'It is more likely that the humanoid skeletons represent a distinct human species than that they are the remains of human beings with a growth disorder.']", "label": 3 }, { "id": "train_1556", "context": "In countries where automobile insurance includes compensation for whiplash injuries sustained in automobile accidents, reports of having suffered such injuries are twice as frequent as they are in countries where whiplash is not covered. Some commentators have argued, correctly, that since there is presently no objective test for whiplash , spurious reports of whiplash injuries cannot be readily identified. These commentators are, however, wrong to draw the further conclusion that in the countries with the higher rates of reported whiplash injuries, half of the reported cases are spurious : clearly, in countries where automobile insurance does not include compensation for whiplash, people often have little incentive to report whiplash injuries that they actually have suffered.", "question": "In the argument given, the two boldfaced portions play which of the following roles?", "answers": "['The first is evidence that has been used to support a conclusion that the argument criticizes; the second is that conclusion.', 'The first is a claim that has been used to support a conclusion that the argument accepts; the second is the position that the argument defends.', 'The first is an intermediate conclusion that has been used to support a conclusion that the argument defends; the second is the position that the argument opposes.', 'The first is evidence that has been used to support a conclusion that the argument criticizes; the second is the position that the argument defends.']", "label": 0 }, { "id": "train_1557", "context": "Astrologer: Although some scientists have claimed that there is no correlation between people' s astrological signs and their personality types, this claim is scientifically unjustified. Since science does not have precise criteria for distinguishing one personality type from another, scientific studies cannot be used to disprove a correlation between personality type and any other phenomenon.", "question": "Which one of the following most accurately describes the role played in the astrologer's argument by the statement that scientific studies cannot be used to disprove a correlation between personality type and any other phenomenon?", "answers": "['It is a claim offered as support for a conclusion that is in turn offered as support for the overall conclusion drawn in the argument.', 'It summarizes a position that the argument as a whole is directed toward discrediting.', 'It is a conclusion for which support is offered and that in turn is offered as support for the overall conclusion drawn in the argument.', 'It provides a specific instance of the general principle that the argument as a whole is directed toward establishing.']", "label": 2 }, { "id": "train_1558", "context": "Deirdre: Many philosophers have argued that the goal of every individual is to achieve happiness -- that is, the satisfaction derived from fully living up to one' s potential. They have also claimed that happiness is elusive and can be achieved only after years of sustained effort. But these philosophers have been unduly pessimistic, since they have clearly exaggerated the difficulty of being happy. Simply walking along the seashore on a sunny afternoon causes many people to experience feelings of happiness.", "question": "Which one of the following most accurately describes a reasoning flaw in Deirdre's argument?", "answers": "['It allows the key term \"happiness\" to shift in meaning illicitly in the course of the argument.', 'It fails to take into account that what brings someone happiness at one moment may not bring that person happiness at another time.', 'It makes a generalization based on the testimony of a group whose views have not been shown to be representative.', 'It dismisses a claim because of its source rather than because of its content.']", "label": 0 }, { "id": "train_1559", "context": "When workers do not find their assignments challenging, they become bored and so achieve less than their abilities would allow. On the other hand, when workers find their assignments too difficult, they give up and so again achieve less than what they are capable of achieving. It is, ttherefore, clear that no worker' s full potential will ever be realized.", "question": "Which one of the following is an error of reasoning contained in the argument?", "answers": "['assuming without warrant that a situation allows only two possibilities', 'depending on the ambiguous use of a key term', 'mistakenly equating what is actual and what is merely possible', 'relying on subjective rather than objective evidence']", "label": 0 }, { "id": "train_1560", "context": "The cost of a semester' s tuition at a certain university is based on the number of courses in which a student enrolls that semester. Although the cost per course at that university has not risen in four years, many of its students who could afford the tuition when they first enrolled now claim they can no longer afford it.", "question": "Each of the following, if true, helps to resolve the apparent discrepancy above EXCEPT:", "answers": "['The university awards new students a large number of scholarships that are renewed each year for the students who maintain high grade averages.', 'The cost of living in the vicinity of the university has risen over the last two years.', 'Faculty salaries at the university have risen slightly over the past four years.', 'The university has turned many of its part- time office jobs, for which students had generally been hired, into full-time, nonstudent positions.']", "label": 2 }, { "id": "train_1561", "context": "High blood cholesterol levels are bad for the heart. Like meat, eggs, and poultry, shellfish contains cholesterol. But shellfish is not necessarily bad for the heart; it is very low in saturated fat, which affects blood cholesterol levels much more than dietary cholesterol does.", "question": "Which one of the following, if true, most strengthens the argument?", "answers": "['Small quantities of foods high in saturated fat are not bad for the heart', 'A serving of meat or poultry is typically larger than a serving of shellfish.', 'Shellfish has less cholesterol per gram than meat, eggs, and poultry do.', 'Foods low in saturated fat promote low blood cholesterol.']", "label": 3 }, { "id": "train_1562", "context": "Switching to \"low-yield\"cigarettes, those that yield less nicotine, tar, and carbon monoxide than regular cigarettes when tested on a standard machine, does not, in general, reduce the incidence of heart attack. This result is surprising, since nicotine and carbon monoxide have been implicated as contributing to heart disease.", "question": "Which one of the following, if true, most helps to resolve the apparent discrepancy?", "answers": "['Smoking low-yield cigarettes has become fashionable, as relatively healthier styles of life have become more popular than those that have been identified as risky.', 'People who switch from high-yield to low-yield cigarettes often compensate by increasing the number and depth of puffs in order to maintain their accustomed nicotine levels.', 'Carbon monoxide and cigarette tar are not addictive substances.', 'For those who are themselves smokers, inhaling the smoke of others is not generally a significant factor contributing to an increased risk of heart disease.']", "label": 1 }, { "id": "train_1563", "context": "The higher the level of certain vitamins and minerals in the bloodstream, the better a person' s lung function, as measured by the amount of air the person can expel in one second. The lung function of smokers is significantly worse, on average, than that of nonsmokers. Clearly, ttherefore, one way for smokers to improve their lung function is for them to increase their intake of foods that are rich in these helpful vitamins and minerals.", "question": "Which of the following is an assumption on which this argument depends?", "answers": "['Smokers are less likely than nonsmokers to have diets that are rich in vitamins and minerals.', 'Smoking does not introduce into the body chemicals that prevent the helpful vitamins and minerals from entering the bloodstream.', 'People whose diets are deficient in those vitamins and minerals do not typically have other health problems in addition to diminished lung function.', 'Stopping smoking will not typically improve lung function more than any diet changes can.']", "label": 1 }, { "id": "train_1564", "context": "Government official: A satisfactory way of eliminating chronic food shortages in our country is not easily achievable. Direct aid from other countries in the form of food shipments tends to undermine our prospects for long-term agricultural self-sufficiency. If external sources of food are delivered effectively by external institutions, local food producers and suppliers are forced out of business. On the other hand, foreign capital funneled to long-term development projects would inject so much cash into our economy that inflation would drive the price of food beyond the reach of most of our citizens.", "question": "The claim that foreign capital funneled into the economy would cause inflation plays which one of the following roles in the government official's argument?", "answers": [ "It is supported by the claim that food producers and suppliers in the official's country may be forced out of business by donations of food from other countries.", "It is supported by the claim that the official's country must someday be agriculturally self-sufficient.", "It supports the claim that the official's country must someday be agriculturally self-sufficient.", "It supports the claim that there is no easy solution to the problem of chronic food shortages in the official's country." ], "label": 3 }, { "id": "train_1565", "context": "In January of last year the Moviemania chain of movie theaters started popping its popcorn in canola oil, instead of the less healthful coconut oil that it had been using until then. Now Moviemania is planning to switch back, saying that the change has hurt popcorn sales. That claim is false, however, since according to Moviemania' s own sales figures, Moviemania sold 5 percent more popcorn last year than in the previous year.", "question": "That claim is false, however, since according to Moviemania's own sales figures, Moviemania sold 5 percent more popcorn last year than in the previous year.
Which of the following, if true, most strongly supports the argument against Moviemania's claim?", "answers": "[\"Total sales of all refreshments at Moviemania's movie theaters increased by less than 5 percent last year.\", \"Moviemania's customers prefer the taste of popcorn popped in coconut oil to that of popcorn popped in canola oil.\", \"Total attendance at Moviemania's movie theaters was more than 20 percent higher last year than the year before.\", 'The year before last, Moviemania experienced a 10 percent increase in popcorn sales over the previous year.']", "label": 0 }, { "id": "train_1566", "context": "Lobsters and other crustaceans eaten by humans are more likely to contract gill diseases when sewage contaminates their water. Under a recent proposal, millions of gallons of local sewage each day would be rerouted many kilometers offshore. Although this would substantially reduce the amount of sewage in the harbor where lobsters are caught, the proposal is pointless, because hardly any lobsters live long enough to be harmed by those diseases.", "question": "Which one of the following, if true, most seriously weakens the argument?", "answers": "['Lobsters, like other crustaceans, live longer in the open ocean than in industrial harbors.', 'Lobsters breed as readily in sewage-contaminated water as in unpolluted water.', 'Contaminants in the harbor other than sewage are equally harmful to lobsters.', 'Humans often become ill as a result of eating lobsters with gill diseases.']", "label": 3 }, { "id": "train_1567", "context": "Safety consultant: Judged by the number of injuries per licensed vehicle, minivans are the safest vehicles on the road. However, in carefully designed crash tests, minivans show no greater ability to protect their occupants than other vehicles of similar size do. Thus, the reason minivans have such a good safety record is probably not that they are inherently safer than other vehicles, but rather that they are driven primarily by low-risk drivers.", "question": "Which one of the following, if true, most strengthens the safety consultant's argument?", "answers": "['Minivans generally have worse braking and emergency handling capabilities than other vehicles of similar size.', 'When choosing what kind of vehicle to drive, low-risk drivers often select a kind that they know to perform particularly well in crash tests.', 'Judged by the number of accidents per licensed vehicle, minivans are no safer than most other kinds of vehicles are.', 'Minivans tend to carry more passengers at any given time than do most other vehicles.']", "label": 0 }, { "id": "train_1568", "context": "Half of the subjects in an experiment-the experimental group-consumed large quantities of a popular artificial sweetener. Afterward, this group showed lower cognitive abilities than did the other half of the subjects-the control group who did not consume the sweetener. The detrimental effects were attributed to an amino acid that is one of the sweetener' s principal constituents.", "question": "Which of the following, if true, would best support the conclusion that some ingredient of the sweetener was responsible for the experimental results?", "answers": "['The two groups of subjects were evenly matched with regard to cognitive abilities prior to the experiment.', 'A second experiment in which subjects consumed large quantities of the sweetener lacked a control group of subjects who were not given the sweetener.', 'The amino acid referred to in the conclusion is a component of all proteins, some of which must be consumed for adequate nutrition.', 'Most consumers of the sweetener do not consume as much of it as the experimental group members did.']", "label": 0 }, { "id": "train_1569", "context": "Art restorers who have been studying the factors that cause Renaissance oil paintings to deteriorate physically when subject to climatic changes have found that the oil paint used in these paintings actually adjusts to these changes well. The restorers ttherefore hypothesize that it is a layer of material called gesso, which is under the paint, that causes the deterioration.", "question": "Which of the following, if true, most strongly supports the restorers' hypothesis?", "answers": "['Oil paint expands and contracts readily in response to changes in temperature, but it absorbs little water and so is little affected by changes in humidity.', 'Renaissance oil paintings are often painted on wooden panels, which swell when humidity increases and contract when it declines.', 'Renaissance oil paintings with a thin layer of gesso are less likely to show deterioration in response to climatic changes than those with a thicker layer.', 'An especially hard and nonabsorbent type of gesso was the raw material for moldings on the frames of Renaissance oil paintings.']", "label": 2 }, { "id": "train_1570", "context": "How do the airlines expect to prevent commercial plane crashes? Studies have shown that pilot error contributes to two-thirds of all such crashes. To address this problem, the airlines have upgraded their training programs by increasing the hours of classroom instruction and emphasizing communication skills in the cockpit. But it is unrealistic to expect such measures to compensate for pilots' lack of actual flying time. Ttherefore, the airlines should rethink their training approach to reducing commercial crashes.", "question": "Which one of the following is an assumption upon which the argument depends?", "answers": "['The number of airline crashes will decrease if pilot training programs focus on increasing actual flying time.', 'Lack of actual flying time is an important contributor to pilot error in commercial plane crashes.', 'Training programs can eliminate pilot errors.', 'Commercial pilots routinely undergo additional training throughout their careers.']", "label": 1 }, { "id": "train_1571", "context": "Radio stations with radio data system (RDS) technology broadcast special program information that only radios with an RDS feature can receive. Between 1994 and 1996, the number of RDS radio stations in Verdland increased from 250 to 600. However, since the number of RDS-equipped radios in Verdland was about the same in 1996 as in 1994, the number of Verdlanders receiving the special program information probably did not increase significantly.", "question": "Which of the following is an assumption on which the argument depends?", "answers": "['The RDS radio stations in Verdland in 1996 did not all offer the same type of programming.', 'In 1996 Verdlanders who did not own radios equipped to receive RDS could not receive any programming from the RDS radio stations that began broadcasting in Verdland after 1994.', 'Few if any of the RDS radio stations that began broadcasting in Verdland after 1994 broadcast to people with RDS-equipped radios living in areas not previously reached by RDS stations.', 'In 1996 most Verdlanders who lived within the listening area of an RDS station already had a radio equipped to receive RDS.']", "label": 2 }, { "id": "train_1572", "context": "Few politicians will support legislation that conflicts with their own self-interest. A case in point is August Frenson, who throughout his eight terms in office consistently opposed measures limiting the advantage incumbents enjoy over their challengers. Ttherefore, if such measures are to be enacted, they must result from direct popular vote rather than from legislative action.", "question": "The case of August Frenson plays which one of the following roles in the argument?", "answers": "['It demonstrates the extent to which incumbents have the advantage over challengers.', \"It provides evidence, the falsity of which would guarantee the falsity of the author's conclusion.\", 'It gives essential background information concerning a measure being advocated.', 'It is cited as an example illustrating the generalization that is invoked.']", "label": 3 }, { "id": "train_1573", "context": "In a study of tropical forests it was found that while the species of trees that is most common in a particular forest also reproduces the most, trees of the species that is rarest there tend to survive longer. This pattern holds regardless of which species of trees is the most common and which is the rarest.", "question": "Which one of the following, if true, most helps to explain why trees of the rarest species tend to survive longer than trees of the most common species?", "answers": "['The study tracked preexisting tree species but did not introduce any new species to the tropical forests.', 'Older trees tend to reproduce the least.', 'The trees of the common species have more competition for the resources they need than do the trees of the rare species.', 'The survival of the trees of the rarer species enables tropical forests to recover more easily from moderate destruction.']", "label": 2 }, { "id": "train_1574", "context": "To classify a work of art as truly great, it is necessary that the work have both originality and far-reaching influence upon the artistic community.", "question": "The principle above, if valid, most strongly supports which one of the following arguments?", "answers": "['The piece of art in the lobby is clearly not classified as truly great, so it follows that it fails to be original.', 'Some of the most original art being produced today is found in isolated communities, but because of this isolation these works have only minor influence, and hence cannot be considered truly great.', \"Since Bach's music is truly great, it not only has both originality and a major influence on musicians, it has broad popular appeal as well.\", 'By breaking down traditional schemes of representation, Picasso redefined painting. It is this extreme originality that warrants his work being considered truly great.']", "label": 1 }, { "id": "train_1575", "context": "Astronomer: Earlier estimates of the distances of certain stars from Earth would mean that these stars are about 1 billion years older than the universe itself, an impossible scenario. My estimates of the distances indicate that these stars are much farther away than previously thought. And the farther away the stars are, the greater their intrinsic brightness must be, given their appearance to us on Earth. So the new estimates of these stars' distances from Earth help resolve the earlier conflict between the ages of these stars and the age of the universe.", "question": "Which one of the following, if true, most helps to explain why the astronomer's estimates of the stars' distances from Earth help resolve the earlier conflict between the ages of these stars and the age of the universe?", "answers": "['How bright celestial objects appear to be depends on how far away from the observer they are.', 'The brighter a star is, the younger it is.', 'New telescopes allow astronomers to see a greater number of distant stars.', 'The younger the universe is, the more bright stars it is likely to have.']", "label": 1 }, { "id": "train_1576", "context": "In a recent study, a group of subjects had their normal daily caloric intake increased by 25 percent. This increase was entirely in the form of alcohol. Another group of similar subjects had alcohol replace nonalcoholic sources of 25 percent of their normal daily caloric intake. All subjects gained body fat over the course of the study, and the amount of body fat gained was the same for both groups.", "question": "Which one of the following is most strongly supported by the information above?", "answers": "['The proportion of calories from alcohol in a diet is more significant for body fat gain than are the total calories from alcohol.', 'Body fat gain is not dependent solely on the number of calories one consumes.', 'In the general population, alcohol is the primary cause of gains in body fat.', 'An increased amount of body fat does not necessarily imply a weight gain.']", "label": 1 }, { "id": "train_1577", "context": "No mathematical proposition can be proven true by observation. It follows that it is impossible to know any mathematical proposition to be true.", "question": "The conclusion follows logically if which one of the following is assumed?", "answers": "['Knowing a proposition to be true is impossible only if it cannot be proven true by observation.', 'Observation alone cannot be used to prove the truth of any proposition.', 'If a proposition can be proven true by observation, then it can be known to be true.', 'Knowing a proposition to be true requires proving it true by observation.']", "label": 3 }, { "id": "train_1578", "context": "Microscopic plants called algae grow inside the top layer of sea ice in the Antarctic if enough sunlight reaches that layer of ice and enough seawater, which contains nutrients, reaches that layer after splashing onto the surface. Even though moderate snow cover reduces the sunlight that filters into the top layer, sea ice with moderate snow cover typically contains even more algae in the top layer than does sea ice with less snow cover.", "question": "Which of the following, if true, most helps to explain the apparent discrepancy?", "answers": "['The nutrients on which algae rely are common in seawater throughout the Antarctic.', 'Seawater in the Antarctic often contains higher concentrations of nutrients than does seawater in more temperate regions.', 'More sunlight filters through a layer of ice than through an equally thick layer of snow.', 'As the weight of accumulated snow forces ice lower in the water, more seawater splashes onto the surface and seeps through the snow.']", "label": 3 }, { "id": "train_1579", "context": "All the stretch limousines owned by Elegant are more than 20 feet long, and all the town cars owned by Elegant are less than 20 feet long. Most of the vehicles owned by Elegant were manufactured before 2000. All the stretch limousines and town cars owned by Style were built after 2000, and all are less than 20 feet long. The carwash at Newport services only vehicles owned by Elegant or Style and can only wash vehicles less than 20 feet long. Currently, a stretch limousine identified as #217 is parked at the carwash at Newport.", "question": "If all the statements above are true, which of the following must also be true?", "answers": "['All the vehicles owned by Elegant were manufactured before any of the vehicles owned by Style.', 'The carwash at Newport does more business with Style than with Elegant.', 'Stretch limousine #217 was manufactured after 2000.', 'The carwash at Newport only services stretch limousines.']", "label": 2 }, { "id": "train_1580", "context": "Plantings of cotton bioengineered to produce its own insecticide against bollworms, a major cause of crop failure, sustained little bollworm damage until this year. This year the plantings are being seriously damaged by bollworms. Bollworms, however, are not necessarily developing resistance to the cotton' s insecticide. Bollworms breed on corn, and last year more corn than usual was planted throughout cotton-growing regions. So it is likely that the cotton is simply being overwhelmed by corn-bred bollworms.", "question": "In evaluating the argument, which of the following would it be most useful to establish?", "answers": "['Whether plantings of cotton that does not produce the insecticide are suffering unusually extensive damage from bollworms this year', 'Whether corn could be bioengineered to produce the insecticide', 'Whether there are insecticides that can be used against bollworms that have developed resistance to the insecticide produced by the bioengineered cotton', 'Whether plantings of bioengineered cotton are frequently damaged by insect pests other than bollworms']", "label": 0 }, { "id": "train_1581", "context": "Biologist: Some computer scientists imagine that all that is required for making an artificial intelligence is to create a computer program that encapsulates the information contained in the human genome. They are mistaken. The operation of the human brain is governed by the interactions of proteins whose structures are encoded in the human genome.", "question": "Which one of the following is an assumption required by the biologist's argument?", "answers": "['The interactions of the proteins that govern the operation of the human brain are not determined by the information contained in the human genome.', 'The functions of the human brain are governed by processes that cannot be simulated by a computer.', 'The only way to create an artificial intelligence is to model it on the operation of the human brain.', 'The amount of information contained in the human genome is too large to be easily encapsulated by a computer program.']", "label": 0 }, { "id": "train_1582", "context": "As air-breathing mammals, whales must once have lived on land and needed hind limbs capable of supporting the mammals' weight. Whales have the bare remnants of a pelvis. If animals have a pelvis, we expect them to have hind limbs. A newly discovered fossilized whale skeleton has very fragile hind limbs that could not have supported the animal' s weight on land. This skeleton had a partial pelvis.", "question": "If the statements above are true, which one of the following, if also true, would most strongly support the conclusion that the fragile hind limbs are remnants of limbs that land-dwelling whales once had?", "answers": "['Scientists are uncertain whether the apparently nonfunctioning limbs of other early mammals derived from once-functioning limbs of their ancestors.', 'Some smaller sea-dwelling mammals, such as modern dolphins, have no visible indications of hind limbs.', \"No skeletons of ancient whales with intact hind limbs capable of supporting the mammals' weight have ever been found.\", 'Whale bones older than the fossilized hind limbs confirm that ancient whales had full pelvises.']", "label": 3 }, { "id": "train_1583", "context": "If a dietician gives a client only a few options for a weight-loss plan, the client is more likely to adhere to the dietician' s advice than if the dietician gives the client many options.", "question": "Which one of the following most accurately expresses the principle illustrated above?", "answers": "[\"Most people are unlikely to follow their dietician's advice unless they can vividly imagine the consequences of not following the advice.\", 'People dislike calculating the best of a variety of choices unless they can see a clear difference among the benefits that would result from each choice.', 'People are especially likely to ignore the counsel they get from dieticians if they are confused about that advice.', 'The tendency people have to alter their behavior varies inversely with the number of alternatives available to them for weight loss.']", "label": 3 }, { "id": "train_1584", "context": "A mathematical theorem proved by one mathematician should not be accepted until each step in its proof has been independently verified. Computer-assisted proofs generally proceed by conducting a vast number of calculations -- surveying all the possible types of instances in which the theorem could apply and proving that the theorem holds for each type. In most computer-assisted proofs there are astronomically many types of instances to survey, and no human being could review every step in the proof. Hence, computer-assisted proofs involving astronomically many types of instances should not be accepted.", "question": "Which one of the following is an assumption on which the argument relies?", "answers": "['Most attempts to construct proofs of mathematical theorems do not result in demonstrations that the theorems are true.', 'Computers cannot be used to assist in generating proofs of mathematical theorems that involve only a very limited number of steps.', \"The use of the computer to assist in the proof of mathematical theorems has greatly simplified the mathematician's task.\", 'The use of an independent computer program does not satisfy the requirement for independent verification of each step in a proof that is extended enough to be otherwise unverifiable.']", "label": 3 }, { "id": "train_1585", "context": "A person with an antique to sell can entrust a certain online brokerage company to sell it online. Since the Internet company will only give the seller 75% of the antique' s fair market value regardless of what the antique sells for, a person trying to get the most money for his antique should try to sell the antique himself.", "question": "The argument depends on the assumption that", "answers": "['a person selling an antique on his own can typically sell it for more than 75% of its fair market value', 'at least 25% of the buyers in the antique market prefer to shop online for antiques', \"if the brokerage company gave the seller more than 75% of the antique's fair market value, it would go out of business.\", 'the cost to the person selling an antique on his own does not exceed the cost incurred by the brokerage company if it were to sell the antique']", "label": 0 }, { "id": "train_1586", "context": "Researchers put two electrodes in a pool that a dolphin swam in. When the dolphin swam near the electrodes, the researchers would sometimes create a weak electric field by activating the electrodes. The dolphin would swim away if the electrodes were activated; otherwise it acted normally. The researchers then placed a plastic shield over small organs called vibrissal crypts located on the dolphin' s snout. With the crypts covered, the dolphin no longer swam away when the electrodes were activated.", "question": "The statements above, if true, most strongly support which one of the following?", "answers": "['In the wild, dolphins sometimes encounter strong electric fields.', 'Under normal circumstances, dolphins are unable to sense electric fields.', 'Vibrissal crypts enable dolphins to sense electric fields.', \"Electric fields interfere with the normal functioning of dolphins' vibrissal crypts.\"]", "label": 2 }, { "id": "train_1587", "context": "If the government institutes criminal justice reform, then the percentage of citizens in prison will decrease. If the government does not institute criminal justice reform during this legislative session, then the government will go bankrupt. All attempts at criminal justice reform require 60 Purple Party politicians to hold office. There are currently 55 Purple Party politicians in office during this legislative session.", "question": "According to the passage above, what must follow?", "answers": "['The percentage of citizens in prison will decrease during this legislative session.', 'The government will go bankrupt during this legislative session.', 'A bipartisan compromise will prevent the government from going bankrupt during this legislative session.', 'Criminal justice reform will be enacted when five more Purple Party politicians hold office.']", "label": 1 }, { "id": "train_1588", "context": "The frozen, well-preserved body of a man was recently discovered in a glacier as the glacier melted. Artifacts found on the body reveal that the man lived at least 4, 000 years ago. The nature of the glacier indicates that the man died on virtually the same spot where his body was discovered. If the body had not been sealed in the glacier shortly after the man died, or if the body had thawed in the intervening millennia, it would not still be preserved.", "question": "Which one of the following is most strongly supported if all of the statements above are true?", "answers": "['The man whose body was discovered in the glacier did not freeze to death.', 'The artifacts found on the frozen body would not have been preserved if they had not also been frozen.', 'The glacier that melted to reveal the ancient is at least 4, 000 years old.', 'None of the artifacts found on the body were more than 4, 000 years old.']", "label": 2 }, { "id": "train_1589", "context": "The mainstream news media emphasize violence occurring in the Middle East at the expense of covering violence occurring in many other areas of the world. Although the media focus is on violence in the Middle East, the problem of violence is much greater in Central Africa. Yet the public tends to interpret the amount of coverage the media give to these occurrences as indicating the degree of the problem.", "question": "If the statements above are true, which one of the following conclusions is most strongly supported by them?", "answers": "[\"The emphasis given to violence in the Middle East is dictated by the public's interest in that region.\", 'The Internet is more influential than print media in most areas of the world.', 'Where news media are the main source of information, public opinion regarding the severity of a problem is swayed by coverage.', 'The problem in the Middle East is a more complicated problem than is the problem in Central Africa.']", "label": 2 }, { "id": "train_1590", "context": "Since Professor Smythe has been head of the department, the most distinguished member of the faculty has resigned, fewer new courses have been developed, student enrollment has dropped, and the reputation of the department has gone down. These facts provide conclusive evidence that Professor Smythe was appointed to undermine the department.", "question": "The reasoning in the argument is flawed because the argument", "answers": "['bases a general claim on a few exceptional instances', 'overlooks the fact that something can have the reputation for being of poor quality without being of poor quality', 'fails to distinguish between a decline in quantity and a decline in quality', 'assumes that because an action was followed by a change, the action was undertaken to bring about that change']", "label": 3 }, { "id": "train_1591", "context": "Throughout a certain nation, electricity has actually become increasingly available to people in urban areas while energy production has been subsidized to help residents of rural areas gain access to electricity. However, even with the subsidy, many of the most isolated rural populations still have no access to electricity. Thus, the energy subsidy has failed to achieve its intended purpose.", "question": "The reasoning in the argument is most vulnerable to criticism on the grounds that the argument", "answers": "['fails to take into account that the subsidy could have helped many of the rural residents in the nation gain access to electricity even if many other rural residents in the nation were not helped in this way', 'presumes, without providing justification, that the intended purpose of the subsidy was to benefit not only rural populations in the nation who have no electricity, but other people in the nation as well', 'overlooks the possibility that even many of the people in the nation who live in urban areas would have difficulty gaining access to electricity without the subsidy', 'takes for granted that if a subsidy has any benefit for those whom it was not intended to benefit, then that subsidy has failed to achieve its intended purpose']", "label": 0 }, { "id": "train_1592", "context": "Spring Mountain College recently sent out a survey to the graduating class of 2003 asking that the graduates provide their class rank. Surprisingly, the survey results revealed that most of the students ranked in the top third of the class.", "question": "The survey results would most likely be explained by which one of the following?", "answers": "['All the graduates of 2003 responded to the survey.', 'Many of the respondents ranked in the lower half of the class.', 'A majority of the respondents ranked high in the graduating class.', 'All the graduates of 2003 had a positive experience while at Spring Mountain College.']", "label": 2 }, { "id": "train_1593", "context": "Most commentators on Baroque painting consider Caravaggio an early practitioner of that style, believing that his realism and novel use of the interplay of light and shadow broke sharply with current styles of Caravaggio' s time and significantly influenced seventeenth-century Baroque painting. One must ttherefore either abandon the opinion of this majority of commentators or reject Mather' s definition of Baroque painting, which says that for any painting to be considered Baroque, it must display opulence, heroic sweep, and extravagance.", "question": "The conclusion of the argument can be properly drawn if which one of the following is assumed?", "answers": "[\"Opulence, heroic sweep, and extravagance are not present in Caravaggio's paintings.\", 'A painter who makes use of the interplay of light and shadow need not for that reason be considered a nonrealistic painter.', 'A realistic painting usually does not depict the world as opulent, heroic, or extravagant.', 'Realism was not widely used by painters prior to the seventeenth century.']", "label": 0 }, { "id": "train_1594", "context": "Ming: Since trans fat is particularly unhealthy, it' s fortunate for the consumer that so many cookie manufacturers have completely eliminated it from their products. Carol: Why do you say that? Even without trans fat, desserts do not make for healthy eating.", "question": "Carol's response indicates that she interpreted Ming's remarks to mean that", "answers": "['if a food is not healthy, then it is unhealthy', \"food that doesn't contain trans fat is healthy food\", 'the more trans fat a cookie contains, the more unhealthy it is', 'consumers should purchase cookies only if they do not contain trans fat']", "label": 1 }, { "id": "train_1595", "context": "A developing country can substantially increase its economic growth if its businesspeople are willing to invest in modern industries that have not yet been pursued there. But being the first to invest in an industry is very risky. Moreover, businesspeople have little incentive to take this risk since if the business succeeds, many other people will invest in the same industry, and the competition will cut into their profits.", "question": "The statements above, if true, most strongly support which one of the following claims?", "answers": "['A developing country can increase its prospects for economic growth by providing added incentive for investment in modern industries that have not yet been pursued there.', 'In developing countries, there is greater competition within modern industries than within traditional industries.', 'Investments in a modern industry in a developing country carry little risk as long as the country has at least one other business in that industry.', 'A developing country will not experience economic growth unless its businesspeople invest in modern industries.']", "label": 0 }, { "id": "train_1596", "context": "Franklin: It is inconsistent to pay sports celebrities ten times what Nobel laureates are paid. Both have rare talents and work hard. Tomeka: What you' ve neglected to consider is that unlike Nobel laureates, sports celebrities earn millions of dollars for their employers in the form of gate receipts and TV rights.", "question": "Franklin's and Tomeka's statements provide the most support for holding that they disagree about the truth of which one of the following?", "answers": "['Sports celebrities and Nobel laureates work equally hard for their employers.', 'Nobel laureates should be paid more than sports celebrities.', 'Nobel laureates should be taken more seriously.', 'There is no rational basis for the salary difference between sports celebrities and Nobel laureates.']", "label": 3 }, { "id": "train_1597", "context": "Brick and Mortar Bookstore Owner: Bookstores are the backbone of our country. Democracies depend on a literate population, and reading fosters the creativity necessary to drive innovation. Brick and mortar bookstores introduce people to new books and entice people to expand their literary preferences. Without brick and mortar bookstores, the demand for books would collapse, killing the publishing industry. Digital Bookstore Owner: There is no denying the importance of reading for any democracy. However, digital books are the future. People can easily access our enormous catalogue of books, which is far greater than any brick and mortar bookstore. We provide synopses and reviews that allow people to discover new interests. In addition, digital books are cheaper than paper books.", "question": "What is the main point of dispute in the two arguments?", "answers": "['Customers prefer paper books.', 'Digital bookstores depend on the existence of brick and mortar bookstores.', 'Digital books are cheaper than paper books.', 'Digital books will someday replace paper books altogether.']", "label": 1 }, { "id": "train_1598", "context": "Anatomical bilateral symmetry is a common trait. It follows, ttherefore, that it confers survival advantages on organisms. After all, if bilateral symmetry did not confer such advantages, it would not be common.", "question": "The pattern of reasoning in which one of the following arguments is most similar to that in the argument above?", "answers": "['Since it is Sawyer who is negotiating for the city government, it must be true that the city takes the matter seriously. After all, if Sawyer had not been available, the city would have insisted that the negotiations be deferred.', 'Clearly, no candidate is better qualified for the job than Trumbull. In fact, even to suggest that there might be a more highly qualified candidate seems absurd to those who have seen Trumbull at work.', 'Since Varga was away on vacation at the time, it must have been Rivers who conducted the secret negotiations. Any other scenario makes little sense, for Rivers never does the negotiating unless Varga is unavailable.', 'If Powell lacked superior negotiating skills, she would not have been appointed arbitrator in this case. As everyone knows, she is the appointed arbitrator, so her negotiating skills are, detractors notwithstanding, bound to be superior.']", "label": 3 }, { "id": "train_1599", "context": "A well-known Native American story -- depicting Native American literature -- contains several inconsistencies. Some scholars argue that because the story contains inconsistencies, the original storyteller must have gotten information from more than one source.", "question": "The conclusion cited above does not follow unless", "answers": "['storytellers generally try to reconcile discrepancies between sources', 'the original storyteller used no source of information that contained inconsistencies repeated in the original story', 'the storyteller attempted to tell an accurate story', 'the storyteller was aware of the kind of inconsistencies that could ruin a story']", "label": 1 }, { "id": "train_1600", "context": "Researchers wanted to know if secondhand tobacco smoke is a significant factor in contracting lung diseases. They conducted a study comparing the health of a group of nonsmoking spouses of smokers with that of a group of nonsmoking spouses of nonsmokers. It was discovered that the spouses of smokers were significantly more susceptible to lung diseases than were the spouses of nonsmokers. The researchers concluded that secondhand tobacco smoke is indeed a factor in the development of lung diseases.", "question": "Which one of the following, if true, most calls into question the researchers' conclusion?", "answers": "['Significantly more of the spouses of smokers were raised by smokers than were the spouses of nonsmokers.', 'The group of spouses of smokers was selected from a heavily industrialized urban area, while the group of spouses of nonsmokers was chosen from a rural area.', 'The members of the group of spouses of smokers were on average several months younger than the members of the group of spouses of nonsmokers.', 'The spouses of nonsmokers are no more likely to be former smokers than are the spouses of smokers.']", "label": 1 }, { "id": "train_1601", "context": "Brochure: Help conserve our city' s water supply. By converting the landscaping in your yard to a water-conserving landscape, you can greatly reduce your outdoor water use. A water-conserving landscape is natural and attractive, and it also saves you money. Criticism: For most people with yards, the savings from converting to a water-conserving landscape cannot justify the expense of new landscaping, since typically the conversion would save less than twenty dollars on a homeowner' s yearly water bills.", "question": "Which of the following, if true, provides the best basis for a rebuttal of the criticism?", "answers": "['A conventional landscape generally requires a much greater expenditure on fertilizer and herbicide than does a water-conserving landscape.', 'Even homeowners whose yards do not have water-conserving landscapes can conserve water by installing water-saving devices in their homes.', 'A significant proportion of the residents of the city live in buildings that do not have yards.', 'Some homeowners use more water to maintain their yards than they use for all other purposes combined.']", "label": 0 }, { "id": "train_1602", "context": "Anders: The physical structure of the brain plays an important role in thinking. So researchers developing \"thinking machines\" -- computers that can make decisions based on both common sense and factual knowledge -- should closely model those machines on the structure of the brain. Yang: Important does not mean essential. After all, no flying machine closely modeled on birds has worked; workable aircraft are structurally very different from birds. So thinking machines closely modeled on the brain are also likely to fail. In developing a workable thinking machine, researchers would ttherefore increase their chances of success if they focus on the brain' s function and simply ignore its physical structure.", "question": "The statement \"thinking machines closely modeled on the brain are also likely to fail\" serves which one of the following roles in Yang's argument?", "answers": "['a particular example illustrating a general claim', 'the main conclusion of the argument', 'a subsidiary conclusion used in support of the main conclusion', 'a principle of research invoked in support of the conclusion']", "label": 2 }, { "id": "train_1603", "context": "From 1900 until the 1930s, pellagra, a disease later discovered to result from a deficiency of the vitamin niacin, was common among poor cotton farmers in the United States whose diet consisted mostly of corn, the least costly food they could buy. Corn does not contain niacin in usable form. Curiously, during the Depression of the early 1930s, when cotton' s price collapsed and cotton farmers' income declined, the incidence of pellagra among those farmers also declined.", "question": "Which of the following, if true, most helps to explain the decline in the incidence of pellagra?", "answers": "['In parts of Mexico, where people subsisted on corn but pellagra was uncommon, corn was typically processed with alkaline substances, which make the niacin in the corn absorbable.', \"Until the discovery of pellagra's link with niacin, it was widely believed that the disease was an infection that could be transmitted from person to person.\", 'People whose diets consist largely of corn often suffer from protein deficiency as well as niacin deficiency.', 'When growing a cash crop could not generate adequate income, poor farmers in the United States responded by planting their land with vegetables and fruits for their own consumption.']", "label": 3 }, { "id": "train_1604", "context": "Art critic: Criticism focuses on two issues: first, whether the value of an artwork is intrinsic to the work; and second, whether judgments about an artwork' s quality are objective rather than merely matters of taste. These issues are related, for if an artwork' s value is not intrinsic, then it must be extrinsic, and thus judgments about the quality of the work can only be a matter of taste.", "question": "The art critic's reasoning is most vulnerable to the criticism that it takes for granted that", "answers": "['judgments about the quality of an artwork are always a matter of taste', \"an artwork's value is sometimes intrinsic to it\", 'judgments about intrinsic value are always objective', 'judgments about extrinsic value cannot be objective']", "label": 3 }, { "id": "train_1605", "context": "Terry: Some actions considered to be bad by our society have favorable consequences. But an action is good only if it has favorable consequences. So, some actions considered to be bad by our society are actually good. Pat: I agree with your conclusion, but not with the reasons you give for it. Some good actions actually do not have favorable consequences. But no actions considered to be bad by our society have favorable consequences, so your conclusion, that some actions our society considers bad are actually good, still holds.", "question": "Which one of the following correctly describes both an error in Terry's reasoning and an error in Pat's reasoning?", "answers": "['presupposing that if a certain property is shared by two types of action, then that property is the only property distinguishing the two types of action from actions of other types', 'presupposing that if a certain property is shared by actions of a certain type in a given society, then that property is shared by actions of that type in every society', \"presupposing that if an action's having a certain property is necessary for its being a certain type of action, then having that property is sufficient for being that type of action\", 'presupposing that if a certain property distinguishes one type of action from another type of action, then that property is one of many properties distinguishing the two types of action']", "label": 2 }, { "id": "train_1606", "context": "Columnist: Much of North America and western Europe is more heavily forested and has less acid rain and better air quality now than five decades ago. Though this may be due largely to policies advocated by environmentalists, it nonetheless lends credibility to the claims of people who reject predictions of imminent ecological doom and argue that environmental policies that excessively restrict the use of natural resources may diminish the wealth necessary to adopt and sustain the policies that brought about these improvements.", "question": "Which one of the following, if true, most strengthens the columnist's reasoning?", "answers": "[\"The more advanced the technology used in a nation's industries, the greater is that nation's ability to devote a portion of its resources to social programs.\", \"The concern demonstrated by a nation for the health and integrity of its natural ecosystems leads to an increase in that nation's wealth.\", 'Nations sustain their wealth largely through industrial use of the natural resources found within their boundaries.', 'A majority of ecological disasters arise from causes that are beyond human control.']", "label": 2 }, { "id": "train_1607", "context": "Something must be done to ease traffic congestion. In traditional small towns, people used to work and shop in the same town in which they lived; but now that stores and workplaces are located far away from residential areas, people cannot avoid traveling long distances each day. Traffic congestion is so heavy on all roads that, even on major highways where the maximum speed limit is 55 miles per hour, the actual speed averages only 35 miles per hour.", "question": "Which one of the following proposals is most supported by the statements above?", "answers": "['Residents of the remaining traditional small towns should be encouraged to move to the suburbs.', 'People who now travel on major highways should be encouraged to travel on secondary roads instead.', 'The maximum speed limit on major highways should be increased.', 'New businesses should be encouraged to locate closer to where their workers would live.']", "label": 3 }, { "id": "train_1608", "context": "A number of measures indicate the viability of a nation' s economy. The level and rate of growth of aggregate output are the most significant indicators, but unemployment and inflation rates are also important. Further, Switzerland, Austria, Israel, Ireland, Denmark, and Finland all have viable economics, but none has a very large population. Switzerland and Austria each have populations of about seven million; the other populations are at least one-fourth smaller.", "question": "Which one of the following is most strongly supported by the information above?", "answers": "[\"A nation's population is the most significant contributor to the level and rate of growth of aggregate output.\", \"A nation's population affects the level and rate of growth of aggregate output more than it affects unemployment and inflation rates.\", 'Having a population larger than seven million ensures that a nation will be economically viable.', 'Economic viability does not require a population of at least seven million.']", "label": 3 }, { "id": "train_1609", "context": "Communism is the greatest source of evil on the planet. In the twentieth century, communism was the leading cause of death, killing more than 90 million people in the Soviet Union, China, North Korea, Afghanistan, and Eastern Europe. The death toll even surpasses the number of people who died during either World War. The leading cause of death in communist countries was famine, which did not occur in any country that was not communist. Despite this hard evidence to the contrary, more than ten percent of Americans believe communism would be better than our current system.", "question": "The author would be most likely to agree with which one of the following?", "answers": "['Communism caused both World Wars.', 'Some people cannot be trusted to decide what system their government should adopt.', 'Communism failed in the twentieth century only because countries failed to follow it properly.', 'Every country should adopt capitalism.']", "label": 1 }, { "id": "train_1610", "context": "Professor: The best users of a language are its great authors. However, these authors often use language in ways that are innovative and idiosyncratic, and are ttherefore less respectful of the strictures of proper usage than most of us are.", "question": "The Professor's statements, if true, most support which one of the following?", "answers": "[\"People who want to become great writers should not imitate great authors' use of language.\", 'Those most talented at using a language are not as likely as most other people to observe proper language usage.', 'People who use an innovative or idiosyncratic writing style often incur criticism of their language usage.', 'Writers who do not observe proper language usage risk developing a peculiar or idiosyncratic style.']", "label": 1 }, { "id": "train_1611", "context": "A certain airport security scanner designed to detect explosives in luggage will alert the scanner' s operator whenever the piece of luggage passing under the scanner contains an explosive. The scanner will erroneously alert the operator for only one percent of the pieces of luggage that contain no explosives. Thus in ninety-nine out of a hundred alerts explosives will actually be present.", "question": "The reasoning in the argument is flawed because the argument", "answers": "['fails to acknowledge the possibility that the scanner will not be equally sensitive to all kinds of explosives', \"ignores the possibility of human error on the part of the scanner's operator once the scanner has alerted him or her\", 'draws a general conclusion about reliability on the basis of a sample that is likely to be biased', 'substitutes one group for a different group in the statement of a percentage']", "label": 3 }, { "id": "train_1612", "context": "The number of applications for admission reported by North American Ph. D. programs in art history has declined in each of the last four years. We can conclude from this that interest among recent North American college and university graduates in choosing art history as a career has declined in the last four years.", "question": "Each of the following, if true, weakens the argument EXCEPT:", "answers": "['The average age of applicants for admission to North American Ph. D. programs in art history has increased in each of the last four years.', 'The percentage of applications for admission received from outside North America by North American Ph. D. programs in art history has declined substantially in the last four years.', 'The number of North American Ph. D. programs in art history that opted to report data about applications for admission has declined in each of the last four years.', 'The number of errors in data about applications for admission to North American Ph. D. programs in art history has increased substantially during the last four years.']", "label": 0 }, { "id": "train_1613", "context": "Professor Donnelly' s exams are always more difficult than Professor Curtis' s exams. The question about dinosaurs was on Professor Donnelly' s last exam. Ttherefore, the question must be difficult.", "question": "Which one of the following exhibits both of the logical flaws exhibited in the argument above?", "answers": "['Mathematics is more difficult than history. Ttherefore, my calculus test will be more difficult than my history test.', \"Shield's first novel has a more complicated plot than any other that she has written. Hence, that plot must be very complex.\", \"Porter's new book of poetry is better than any of her other books of poetry. This poem is from Porter's new book, so it must be good.\", \"Lewis is a better baker than Stockman. Lewis made this cake. Ttherefore, it must be better than most of Stockman's cakes.\"]", "label": 2 }, { "id": "train_1614", "context": "Among people who live to the age of 100 or more, a large proportion have led \"unhealthy\" lives: smoking, consuming alcohol, eating fatty foods, and getting little exercise. Since such behavior often leads to shortened life spans, it is likely that exceptionally long-lived people are genetically disposed to having long lives.", "question": "Which one of the following, if true, most strengthens the argument?", "answers": "['Nearly all people who live to 100 or more have siblings who are also long-lived.', 'Some of the exceptionally long-lived people who exercise regularly and avoid fatty foods do smoke or consume alcohol.', 'Some people who do not live to the age of 100 also lead unhealthy lives.', 'Some of the exceptionally long-lived people who do not smoke or drink do eat fatty foods and get little exercise.']", "label": 0 }, { "id": "train_1615", "context": "Student: If a person has an immunity to infection by a microorganism, then that microorganism does not cause them to develop harmful symptoms. Since many people are exposed to staphylococcus without developing any harmful symptoms, it follows that they have an immunity to infection by this microorganism.", "question": "The student's argument is most similar in its flawed pattern of reasoning to which one of the following?", "answers": "['Isabel said that she would take the medication. Obviously, though, she did not do so, because medication either cures disease or alleviates its symptoms, and Isabel is still quite ill.', 'Everything morally right is just, but some actions that best serve the interests of everyone are not just. Thus, some morally right actions do not serve the interests of everyone.', 'Studies show that doctors tend to wash their hands less often than any other health care professionals. This shows that the procedure cannot be of much value in preventing disease.', 'When business owners are subjected to excessive taxation, they become less willing to expand their businesses. The recent decline in business expansions thus shows that their taxes are too high.']", "label": 3 }, { "id": "train_1616", "context": "Columnist: George Orwell' s book 1984 has exercised much influence on a great number of this newspaper' s readers. One thousand readers were surveyed and asked to name the one book that had the most influence on their lives. The book chosen most often was the Bible; 1984 was second.", "question": "The answer to which one of the following questions would most help in evaluating the columnist's argument?", "answers": "['How many books had each person surveyed read?', 'How many people chose books other than 1984?', \"How many people read the columnist's newspaper?\", 'How many of those surveyed had actually read the books they chose?']", "label": 1 }, { "id": "train_1617", "context": "Magazine Publisher: Our magazine does not have a liberal bias. It is true that when a book review we had commissioned last year turned out to express distinctly conservative views, we did not publish it until we had also obtained a second review that took a strongly liberal position. Clearly, however, our actions demonstrate not a bias in favor of liberal views but rather a commitment to a balanced presentation of diverse opinions.", "question": "Determining which of the following would be most useful in evaluating the cogency of the magazine publisher's response?", "answers": "['Whether in the event that a first review commissioned by the magazine takes a clearly liberal position the magazine would make any efforts to obtain further reviews', 'Whether the book that was the subject of the two reviews was itself written from a clearly conservative or a clearly liberal point of view', 'Whether most of the readers of the magazine regularly read the book reviews that the magazine publishes', 'Whether any other magazines in which the book was reviewed carried more than one review of the book']", "label": 0 }, { "id": "train_1618", "context": "Critic: Emily Dickinson' s poetry demonstrates that meaning cannot reside entirely within a poem itself, but is always the unique result of an interaction between a reader' s system of beliefs and the poem; and, of course, any two readers from different cultures or eras have radically different systems of beliefs.", "question": "If the critic's statements are true, each of the following could be true EXCEPT:", "answers": [ "A reader's interpretation of a poem by Dickinson is affected by someone else's interpretation of it.", "A reader's enjoyment of a poem is enhanced by knowing the poet's interpretation of it.", "A modern reader and a nineteenth-century reader interpret one of Shakespeare's sonnets in the same way.", "A reader's interpretation of a poem evolves over time." ], "label": 2 }, { "id": "train_1619", "context": "Ostrich farming requires far less acreage than cattle ranching requires, and ostriches reproduce much faster than cattle. Starting out in cattle ranching requires a large herd of cows, one bull, and at least two acres per cow. By contrast, two pairs of yearling ostriches and one acre of similar land are enough to begin ostrich farming. The start-up costs for ostrich farming are greater, but it can eventually bring in as much as five times what cattle ranching does.", "question": "Which one of the following is most strongly supported by the information above?", "answers": "['Ostrich farmers typically lose money during their first year.', 'Two pairs of yearling ostriches are more expensive than a herd of cows and a bull.', 'A cow consumes no more feed than an ostrich does.', 'Cattle ranching is not a good source of income.']", "label": 1 }, { "id": "train_1620", "context": "Journalist: There is no downside to journalists using anonymous sources. National newspapers require journalists to use multiple sources to corroborate anonymous tips, and corroboration removes any possibility of abuse. Any failure to corroborate sources would risk national security through the publication of misleading information.", "question": "The strength of the argument depends upon which one of the following?", "answers": "['Journalists should only use anonymous sources.', 'All journalists work for national newspapers.', 'Journalists should only use anonymous sources under extraordinary circumstances.', 'The government has never infringed on the freedom of press.']", "label": 1 }, { "id": "train_1621", "context": "The ancient Nubians inhabited an area in which typhus occurs, yet surprisingly few of their skeletons show the usual evidence of this disease. The skeletons do show deposits of tetracycline, an antibiotic produced by a bacterium common in Nubian soil. This bacterium can flourish on the dried grain used for making two staples of the Nubian diet, beer and bread. Thus, tetracycline in their food probably explains the low incidence of typhus among ancient Nubians.", "question": "Which of the following is an assumption on which the argument relies?", "answers": "['Typhus is generally fatal', 'Typhus cannot be transmitted by ingesting bread or beer contaminated with the infectious agents of this disease', 'Infectious diseases other than typhus to which the ancient Nubians were exposed are unaffected by tetracycline', 'Tetracycline is not rendered ineffective as an antibiotic by exposure to the processes involved in making bread and beer']", "label": 3 }, { "id": "train_1622", "context": "Scientists removed all viruses from a seawater sample and then measured the growth rate of the plankton population in the water. They expected the rate to increase dramatically, but the population actually got smaller.", "question": "Which one of the following, if true, most helps to explain the unexpected result described above?", "answers": "['Viruses in seawater help to keep the plankton population below the maximum level that the resources in the water will support.', 'Plankton and viruses in seawater compete for some of the same nutrients.', 'Plankton utilize the nutrients released by the death of organisms killed by viruses.', 'The absence of viruses can facilitate the flourishing of bacteria that sometimes damage other organisms.']", "label": 2 }, { "id": "train_1623", "context": "For over two centuries, no one had been able to make Damascus blades-blades with a distinctive serpentine surface pattern-but a contemporary sword maker may just have rediscovered how. Using iron with trace impurities that precisely matched those present in the iron used in historic Damascus blades, this contemporary sword maker seems to have finally hit on an intricate process by which he can produce a blade indistinguishable from a true Damascus blade.", "question": "Which of the following, if true, provides the strongest support for the hypothesis that trace impurities in the iron are essential for the production of Damascus blades?", "answers": "['The iron with which the contemporary sword maker made Damascus blades came from a source of iron that was unknown two centuries ago.', 'Although Damascus blades were renowned for maintaining a sharp edge, the blade made by the contemporary sword maker suggests that they may have maintained their edge less well than blades made using what is now the standard process for making blades.', \"Production of Damascus blades by sword makers of the past ceased abruptly after those sword makers' original source of iron became exhausted.\", 'Almost all the tools used by the contemporary sword maker were updated versions of tools that were used by sword makers over two centuries ago.']", "label": 2 }, { "id": "train_1624", "context": "The more television children watch, the less competent they are in mathematical knowledge. More than a third of children in the United States watch television for more than five hours a day; in South Korea the figure is only 7 percent. But whereas less than 15 percent of children in the United States understand advanced measurement and geometric concepts, 40 percent of South Korean children are competent in these areas. Ttherefore, if United States children are to do well in mathematics, they must watch less television.", "question": "Which one of the following is an assumption upon which the argument depends?", "answers": "[\"A child's ability in advanced measurement and geometry increases if he or she watches less than one hour of television a day.\", 'South Korean children are more disciplined about doing schoolwork than are children in the United States.', 'Children in the United States are less interested in advanced measurement and geometric concepts than are South Korean children.', 'The instruction in advanced measurement and geometric concepts available to children in the United States is not substantially worse than that available to South Korean children.']", "label": 3 }, { "id": "train_1625", "context": "A study at a company found that most meetings showed diminishing returns after 30 minutes, and little could be expected after 60 minutes. Moreover, the most productive meetings were those for which a clear time frame was established. For a meeting at the company to achieve maximum productivity, then, it needs to have a clear time frame and be no more than 30 minutes long.", "question": "Which one of the following most accurately expresses the conclusion drawn in the argument?", "answers": "['Most meetings at the company show diminishing returns after 30 minutes, according to a study.', 'In general, a meeting at the company that is no more than 30 minutes long and has a clear time frame will achieve maximum productivity.', 'A meeting at the company will be maximally productive only if it has a clear time frame and lasts no more than 30 minutes.', 'According to a study, meetings at the company were the most productive when they had clear time frames.']", "label": 2 }, { "id": "train_1626", "context": "The spacing of the four holes on a fragment of a bone flute excavated at a Neanderthal campsite is just what is required to play the third through sixth notes of the diatonic scale--the seven-note musical scale used in much of Western music since the Renaissance. Musicologists ttherefore hypothesize that the diatonic musical scale was developed and used thousands of years before it was adopted by Western musicians.", "question": "Which of the following, if true, most strongly supports the hypothesis?", "answers": "['The cave-bear leg bone used to make the Neanderthal flute would have been long enough to make a flute capable of playing a complete diatonic scale.', 'No musical instrument that is known to have used a diatonic scale is of an earlier date than the flute found at the Neanderthal campsite.', 'Flutes are the simplest wind instrument that can be constructed to allow playing a diatonic scale.', 'The flute was made from a cave-bear bone and the campsite at which the flute fragment was excavated was in a cave that also contained skeletal remains of cave bears.']", "label": 0 }, { "id": "train_1627", "context": "Economist: As should be obvious, raising the minimum wage significantly would make it more expensive for businesses to pay workers for minimum-wage jobs. Ttherefore, businesses could not afford to continue to employ as many workers for such jobs. So raising the minimum wage significantly will cause an increase in unemployment.", "question": "Which one of the following, if true, most weakens the economist's argument?", "answers": "['A modest increase in unemployment is acceptable because the current minimum wage is not a livable wage.', 'The unemployment rate has been declining steadily in recent years.', 'Most workers are earning more than the current minimum wage.', 'Businesses typically pass the cost of increased wages on to consumers without adversely affecting profits.']", "label": 3 }, { "id": "train_1628", "context": "Letter to the editor: Allowing everyone to voice personal views can have the effect of inhibiting some from voicing their concerns. Thus, allowing unrestricted free speech really inhibits free speech.", "question": "Which one of the following, if true, does most to justify the apparently contradictory conclusion above?", "answers": "['When free speech is unrestricted, people offended by the views of others are likely to voice their disagreement, leading toward a resolution of conflict.', 'Since unrestricted free speech can be offensive, free speech should be restricted when the pain that it causes is great.', \"Claiming that unrestricted free speech inhibits free speech is like claiming that increasing someone's salary makes the person poorer.\", 'When free speech is unrestricted, many people will be shocked by the power of the views of others and thereby become afraid to voice their own concerns.']", "label": 3 }, { "id": "train_1629", "context": "Every fall Croton' s jays migrate south. The jays always join flocks of migrating crookbeaks with which they share the same summer and winter territories. If a jay becomes separated from the crookbeaks it is accompanying, it wanders until it comes across another flock of crookbeaks. Clearly, ttherefore, Croton' s jays lack the navigational ability to find their way south on their own.", "question": "Which of the following, if true, most strengthens the argument?", "answers": "[\"Croton's jays lay their eggs in the nests of crookbeaks which breed upon completing their southern migration\", \"Species other than Croton's jays occasionally accompany flocks of migrating crookbeaks\", 'The three species most closely related to crookbeaks do not migrate at all', \"In the spring, Croton's jays migrate north in the company of Tattersall warblers\"]", "label": 3 }, { "id": "train_1630", "context": "In 1712 the government of Country Y appointed a censor to prohibit the publication of any book critical of Country Y' s government; all new books legally published in the country after 1712 were approved by a censor. Under the first censor, one half of the book manuscripts submitted to the censor were not approved for publication. Under the next censor, only one quarter of the book manuscripts submitted were not approved, but the number of book manuscripts that were approved was the same under both censors.", "question": "It the statements in the passage are true, which one of the following can be properly concluded from them?", "answers": "['The first censor and the second censor prohibited the publication of the same number of book manuscripts.', \"More books critical of Country Y's government were published before the appointment of the first censor than after it.\", \"The second censor allowed some book manuscripts to be published that the first censor would have considered critical of Country Y's government.\", 'More book manuscripts were submitted for approval to the first censor than to the second.']", "label": 3 }, { "id": "train_1631", "context": "Building a space station, in which astronauts would live for a considerable time, is essential even if the space station project were to contribute no new knowledge about space or Earth that could not otherwise be obtained. For future missions to explore Mars, we will need the medical knowledge that the space station project will give us about the limits of human capacities to live in spacecraft for an extended time.", "question": "The argument makes the assumption that", "answers": "['the capacities of astronauts are typical of those of ordinary human beings', 'the exploration of Mars will be carried out by people traveling in spacecraft and not by robots alone', 'living in a spaceship for an extended time presents insurmountable medical problems', 'a mission to Mars will be the first of many missions that will explore the solar system']", "label": 1 }, { "id": "train_1632", "context": "In order to reduce traffic congestion and raise revenue for the city, the mayor plans to implement a charge of $10 per day for driving in the downtown area. Payment of this charge will be enforced using a highly sophisticated system that employs digital cameras and computerized automobile registration. This system will not be ready until the end of next year. Without this system, however, mass evasion of the charge will result. Ttherefore, when the mayor' s plan is first implemented, payment of the charge will not be effectively enforced.", "question": "Which one of the following is an assumption on which the argument depends for its conclusion to be properly drawn?", "answers": "[\"The mayor's plan to charge for driving downtown will be implemented before the end of next year.\", 'A daily charge for driving downtown is the most effective way to reduce traffic congestion.', 'The plan to charge for driving downtown should be implemented as soon as payment of the charge can be effectively enforced.', 'Raising revenue is a more important consideration for the city than is reducing traffic congestion.']", "label": 0 }, { "id": "train_1633", "context": "Macrophages are cells that play a role in the response of the immune system of mice and other mammals to invasive organisms such as bacteria. Unlike other mice, mice that are genetically incapable of making these particular cells do not show elevated levels of nitrates when infected with bacteria.", "question": "The statements above, if true, provide the most support for which of the following conclusions?", "answers": "[\"Injections of nitrates into mice that lack macrophages will not enhance the ability of these animals' immune systems to fight off infection.\", 'In mice, macrophages play a role in the production of nitrates or inhibit a process by which nitrates are broken down or otherwise eliminated.', \"When a healthy mouse becomes infected with an invasive organism, the number of macrophages in the mouse's body decreases.\", 'Mice that show elevated levels of nitrates can easily fight off most types of bacterial infections.']", "label": 1 }, { "id": "train_1634", "context": "Unless the building permit is obtained by February 1 of this year or some of the other activities necessary for construction of the new library can be completed in less time than originally planned, the new library will not be completed on schedule. It is now clear that the building permit cannot be obtained by February 1, so the new library will not be completed on schedule.", "question": "The conclusion drawn follows logically from the premises if which one of the following is assumed?", "answers": "['The officials in charge of construction of the new library have admitted that it probably will not be completed on schedule.', 'It is not possible to convince authorities to allow construction of the library to begin before the building permit is obtained.', 'The application for a building permit was rejected the first time it was submitted, and it had to be resubmitted with a revised building plan.', 'All of the other activities necessary for construction of the library will take at least as much time as originally planned.']", "label": 3 }, { "id": "train_1635", "context": "Ambiguity inspires interpretation. The saying, \"We are the measure of all things, \" for instance, has been interpreted by some people to imply that humans are centrally important in the universe, while others have interpreted it to mean simply that, since all knowledge is human knowledge, humans must rely on themselves to find the truth.", "question": "The claim that ambiguity inspires interpretation figures in the argument in which one of the following ways?", "answers": "['It sets out a difficulty the argument is intended to solve.', 'It is a view that other statements in the argument are intended to support.', 'It is an illustration of the claim that we are the measure of all things.', \"It is compatible with either accepting or rejecting the argument's conclusion.\"]", "label": 1 }, { "id": "train_1636", "context": "Senator: Jones is highly qualified for appointment as a judge, as evidenced by Jones' s receiving a unanimous vote of \"qualified\" on the formal rating scale used by the Lawyers' Committee. That committee advises the Senate on judicial appointments.", "question": "Which of the following, if true, is the best reason for dismissing the senator's claim that Jones is highly qualified?", "answers": "['The Lawyers\\' Committee gives a unanimous vote of \"qualified\" only to those candidates for judicial appointments who meet the committee\\'s stringent standards for appropriate prior experience and ethical conduct.', \"Jones, like most lawyers, is a member of the professional organization that originally suggested the establishment of the Lawyers' Committee.\", \"Several members of the Lawyers' Committee are not themselves qualified for judicial appointments.\", 'The Lawyers\\' Committee gives a unanimous vote of either \"highly qualified\" or \"very highly qualified\" to 95 percent of all candidates for judicial appointments.']", "label": 3 }, { "id": "train_1637", "context": "Farmers who use genetically engineered plants on a large scale are at great financial risk because at any time a study could be published that would undermine what little confidence consumers have in genetically engineered foods. It is unwise for farmers to grow such crops. They do not fetch a high enough price to compensate for the risk.", "question": "Which one of the following most accurately expresses the conclusion of the argument as a whole?", "answers": "['A study could come out at any time that would greatly undermine public confidence in genetically engineered foods.', 'A farmer who grows genetically engineered crops on a large scale is taking a financial risk.', 'It is not prudent for a farmer to grow genetically engineered crops.', 'The price paid for genetically engineered crops does not compensate for the financial risk farmers incur by growing them.']", "label": 2 }, { "id": "train_1638", "context": "Dr. Jones: The new technology dubbed \"telemedicine\" will provide sustained improvement in at least rural patient care since it allows rural physicians to televise medical examinations to specialists who live at great distances -- specialists who will thus be able to provide advice the rural patient would otherwise not receive. Dr. Carabella: Not so. Telemedicine might help rural patient care initially. However, small hospitals will soon realize that they can minimize expenses by replacing physicians with technicians who can use telemedicine to transmit examinations to large medical centers, resulting in fewer patients being able to receive traditional, direct medical examination. Eventually, it will be the rare individual who ever gets truly personal attention. Hence, rural as well as urban patient care will suffer.", "question": "Dr. Carabella uses which one of the following strategies in responding to Dr. Jones?", "answers": "['citing evidence that Dr. Jones lacks the professional training to judge the case at issue', 'listing a set of considerations to show that a prescribed treatment that seems to be benefiting a patient in fact harms that patient', 'describing the application of the technology discussed by Dr. Jones as one step that initiates a process that leads to an undesirable end', \"providing grounds for dismissing Dr. Jones's interpretation of a key term in medical technology\"]", "label": 2 }, { "id": "train_1639", "context": "Pure science -- research with no immediate commercial or technological application -- is a public good. Such research requires a great amount of financial support and does not yield profits in the short term. Since private corporations will not undertake to support activities that do not yield short-term profits, a society that wants to reap the benefits of pure science ought to use public funds to support such research.", "question": "The claim about private corporations serves which one of the following functions in the argument?", "answers": "['It distracts attention from the point at issue by introducing a different but related goal.', 'It expresses the conclusion of the argument.', 'It explains what is meant by the expression \"pure research\" in the context of the argument.', 'It supports the conclusion by ruling out an alternative way of achieving the benefits mentioned.']", "label": 3 }, { "id": "train_1640", "context": "Psychologist: Substance abuse, in a sense, is both genetic and learned. Some families may have a predisposition to depression or anxiety, which is known to increase an individual' s use of substances. Children who have parents who abused substances are four times more likely than the average person to abuse substances. The latter, however, is up for debate on whether this statistic is due to genetic predisposition or learned behavior.", "question": "Which of the following is most strongly supported by the statements above?", "answers": "['The proof for the genetic factor in substance abuse lies in the fact that children with parents who abused substances are four times more likely to use substances themselves.', 'The proof for the environmental factor in substance abuse lies in the fact that children with parents who abused substances are four times more likely to use substances themselves.', 'The proof for the genetic factor in substance abuse lies in the fact that there may be indirect genes within families, such as depression or anxiety, that increase the potential use for substance abuse.', 'Substance abuse is a learned behavior only; there is no \"addictive\" gene, and ttherefore we cannot consider the issue of substance abuse genetic in any way.']", "label": 2 }, { "id": "train_1641", "context": "Accountant: Shareholder profits are down 30 percent because of a lack of interest in our services. If our stock continues to decline, the corporation might have to consider merging with a larger organization. We must increase interest in our services. Accordingly, we must revamp our marketing department and hire new advertising executives. Shareholder: Wrong. Instead of revamping the marketing department and hiring new advertising executives, the corporation could consider changing its services to better meet the needs of our twenty-first-century clients.", "question": "The shareholder's suggestion that the corporation could consider changing its services to better meet the needs of the corporation's twenty-first-century clients plays which one of the following roles in the argument?", "answers": [ "It presents supporting arguments that strengthen the accountant's proposal.", "It is a claim that must be corroborated if the accountant's conclusion is to be established.", "It highlights a potential flaw in the accountant's argument.", "It is used to illustrate the general principle that the accountant's argument presupposes." ], "label": 2 }, { "id": "train_1642", "context": "Scientist: A number of errors can plague a data-collection process. Since examining the collected data enables researchers to detect many of these errors, it is standard practice for researchers to correct collected data. However, in my field, there is a striking tendency for such corrections to favor Jones' s theory; that is, the majority of corrections result in the corrected data' s being closer than the uncorrected data to what Jones' s theory predicts.", "question": "Which one of the following, if true, most helps to explain the tendency of corrections in the scientist's field to favor Jones 's theory?", "answers": "['Even if researchers fail to detect errors in a data-collection process when they examine the data that they collected, that does not guarantee that no such errors exist.', \"Researchers in the scientist's field have formulated several other theories that attempt to explain the same range of phenomena that Jones's theory attempts to explain.\", 'Researchers normally give data that is in line with a theory the same weight as data that conflicts with that theory when they are determining whether to accept that theory.', \"Researchers in the scientist's field give data that conflicts with Jones 's theory greater scrutiny than they give data that is in line with Jones's theory.\"]", "label": 3 }, { "id": "train_1643", "context": "When machines are invented and technologies are developed, they alter the range of choices open to us. The clock, for example, made possible the synchronization of human affairs, which resulted in an increase in productivity. At the same time that the clock opened up some avenues, it closed others. It has become harder and harder to live except by the clock, so that now people have no choice in the matter at all.", "question": "Which one of the following propositions is best illustrated by the example presented in the passage?", "answers": "['Most new machines and technologies make our lives more synchronized and productive.', 'People should make a concerted effort to free themselves from the clock.', 'New machines and technologies can enslave as well as liberate us.', 'Some new machines and technologies bring no improvement to our lives.']", "label": 2 }, { "id": "train_1644", "context": "In a recent study, researchers collected current prices for the 300 most common pharmaceutical drugs from the leading wholesalers specializing in bulk sales. It was found that these prices average 60 to 80 percent below the suggested wholesale prices listed for the same drugs in the current annual edition of a widely used, independently published pharmaceutical price guidebook.", "question": "Each of the following, if true, would help to explain the situation described above EXCEPT:", "answers": "['Suggested wholesale prices for the most common pharmaceutical drugs tend to be less than those for less common pharmaceutical drugs.', 'Wholesale prices for pharmaceutical drugs often fluctuate dramatically from one month to the next.', 'The prices suggested by the independently published pharmaceutical price guidebook are for sales of relatively small quantities of pharmaceutical drugs to individual doctors.', \"A price war wherein pharmaceutical drug wholesalers tried to undercut each others' prices began shortly before the study was conducted.\"]", "label": 0 }, { "id": "train_1645", "context": "Everyone sitting in the waiting room of the school' s athletic office this morning at nine o' clock had just registered for a beginners tennis clinic. John, Mary, and Teresa were all sitting in the waiting room this morning at nine o' clock. No accomplished tennis player would register for a beginners tennis clinic.", "question": "It the statements above are true, which one of the following must also be true on the basis of them?", "answers": "['John, Mary, and Teresa were the only people who registered for a beginners tennis clinic this morning.', \"John, Mary, and Teresa were the only people sitting in the waiting room of the school's athletic office this morning at nine o'clock.\", 'Neither John nor Teresa is an accomplished tennis player.', \"None of the people sitting in the school's athletic office this morning at nine o'clock had ever played tennis.\"]", "label": 2 }, { "id": "train_1646", "context": "Coach: Our team has often been criticized for our enthusiasm in response to both our successes and our opponents' failures. But this behavior is hardly unprofessional, as our critics have claimed. On the contrary, if one looks at the professionals in this sport, one will find that they are even more effusive. Our critics should leave the team alone and let the players enjoy the game.", "question": "The coach's argument is most vulnerable to the charge that it", "answers": "['misleadingly equates enthusiasm with unethical play', \"misinterprets the critics' claim that the team is unprofessional\", \"shifts the blame for the team's behavior to professional players\", 'too quickly generalizes from the sport at one level to the sport at a different level']", "label": 1 }, { "id": "train_1647", "context": "When researchers discovered that cuttlefish have the ability to make themselves suddenly appear larger, they presumed that this behavior, called a \"startle display, \" was used to scare off predators. A long-term study, however, reveals that cuttlefish never use startle displays to scare off predators but instead only use such displays to scare off small fish that do not prey on cuttlefish.", "question": "Which one of the following, if true, most helps to explain why cuttlefish use startle displays?", "answers": "['Cuttlefish have acute senses and are able to change colors.', \"Groups of small fish are likely to attract a cuttlefish's predators .\", 'Unlike insects that use startle displays, cuttlefish are usually able to move faster than their predators .', \"Small fish are more easily scared off by a startle display than are a cuttlefish's predators.\"]", "label": 1 }, { "id": "train_1648", "context": "Most plants have developed chemical defenses against parasites. The average plant contains about 40 natural pesticides -- chemical compounds toxic to bacteria, fungi, and other parasites. Humans ingest these natural pesticides without harm every day. Ttherefore, the additional threat posed by synthetic pesticides sprayed on crop plants by humans is minimal.", "question": "Each of the following, if true, weakens the argument EXCEPT:", "answers": "['The synthetic pesticides sprayed on crop plants by humans usually have chemical structures similar to those of the natural pesticides produced by the plants.', 'Natural plant pesticides generally serve only as defenses against specific parasites, whereas synthetic pesticides are often harmful to a wide variety of organisms.', 'The concentrations of natural pesticides in plants are typically much lower than the concentrations of synthetic pesticides in sprayed crop plants.', 'Natural plant pesticides are typically less potent than synthetic pesticides, whose toxicity is highly concentrated.']", "label": 0 }, { "id": "train_1649", "context": "University president: Our pool of applicants has been shrinking over the past few years. One possible explanation of this unwelcome phenomenon is that we charge too little for tuition and fees. Prospective students and their parents conclude that the quality of education they would receive at this institution is not as high as that offered by institutions with higher tuition. So, if we want to increase the size of our applicant pool, we need to raise our tuition and fees.", "question": "The university president's argument requires the assumption that", "answers": "['the proposed explanation for the decline in applications applies in this case', \"there is no additional explanation for the university's shrinking applicant pool\", 'the quality of a university education is dependent on the amount of tuition charged by the university', 'the amount charged by the university for tuition has not increased in recent years']", "label": 0 }, { "id": "train_1650", "context": "Legal activist: The opportunities for presenting flawed science in the courtroom are numerous, and, over time, such flawed testimony in trials can influence procedural requirements for entire professions. For example, so-called expert witnesses in medical malpractice suits have testified that the use of electronic fetal monitors would have prevented certain delivery-room complications, even though their views were not corroborated by medical science. Yet, as a result of the decisions based on such testimony, the use of electronic fetal monitors during delivery has become the professional norm.", "question": "Which one of the following most accurately expresses the main conclusion drawn in the legal activist's argument?", "answers": "['Unreliable scientific testimony given in trials can have marked effects on the procedures used by the experts in certain fields.', 'Scientists who are engaged in good science should not allow so-called experts to give deceptive courtroom testimony.', 'The scientific testimony at many trials is not as reliable as juries are led to believe.', 'Medical personnel should not be allowed to give legal testimony on topics in which they are not experts.']", "label": 0 }, { "id": "train_1651", "context": "All poets, aside from those who write only epigrams, have wit. All lyrical composers are poets. Azriel does not write epigrams, though he is a lyrical composer. So Azriel has wit.", "question": "The pattern of reasoning in which one of the following is most similar to that in the argument above?", "answers": "[\"All visas are assigned by this office, except for those that are issued through diplomatic channels. All visit permits are visas. Thus, the visit permit in Will's passport was assigned through diplomatic channels.\", 'All squeeze toys, except those designed for cats, are safe for infants. All squeeze toys are sold prewrapped. This item is not designed for cats, and it is sold prewrapped. So it must be safe for infants.', 'All residential buildings are subject to the original fire code, except for those built last year. All townhouses are residential buildings. Bloom House was not built last year, and it is a townhouse, so it is subject to the original fire code.', 'Aside from the dogcatcher and the police chief, all of the politicians in town are lawyers. All of the politicians in town have websites. Sal is a politician in town, but is neither the dogcatcher nor the police chief. Since Sal is a politician in town he must have a website.']", "label": 2 }, { "id": "train_1652", "context": "Critic: Most chorale preludes were written for the organ, and most great chorale preludes written for the organ were written by J. S. Bach. One of Bach' s chorale preludes dramatizes one hymn' s perspective on the year' s end. This prelude is agonizing and fixed on the passing of the old year, with its dashed hopes and lost opportunities. It does not necessarily reveal Bach' s own attitude toward the change of the year, but does reflect the tone of the hymn' s text. People often think that artists create in order to express their own feelings. Some artists do. Master artists never do, and Bach was a master artist.", "question": "If the critic's statements are true, then on the basis of them which one of the following CANNOT be true?", "answers": "['In composing music about a particular subject, Bach did not write the music in order to express his own attitude toward the subject.', \"Most of Bach's chorale preludes were written for instruments other than the organ.\", 'Bach believed that the close of the year was not a time for optimism and joyous celebration.', 'In compositions other than chorale preludes, Bach wrote music in order to express his feelings toward various subjects.']", "label": 3 }, { "id": "train_1653", "context": "Historian: One traditional childrearing practice in the nineteenth century was to make a child who misbehaved sit alone outside. Anyone passing by would conclude that the child had misbehaved. Nowadays, many child psychologists would disapprove of this practice because they believe that such practices damage the child' s self-esteem and that damage to children' s self-esteem makes them less confident as adults. However, no one disagrees that adults raised under that traditional practice were, on average, as confident as adults not so raised.", "question": "Which one of the following can be properly inferred from the historian's statements?", "answers": "[\"With the traditional childrearing practice, passersby did not always make correct inferences about children's behavior by observing them outdoors.\", 'The beliefs of many present-day child psychologists about the consequences of loss of self-esteem are incorrect.', 'The most confident adults are those who developed the highest level of self-esteem in childhood.', \"If children's loss of self-esteem makes them less confident as adults, then the traditional childrearing practice in question did not tend to cause significant loss of self-esteem.\"]", "label": 3 }, { "id": "train_1654", "context": "Kernland' s government restricts the export of unprocessed cashew nuts in order to ensure a low-cost supply for domestic processing plants. Though the policy constrains farm income and limits the number of farmers who can profitably grow cashews, the government defends it on the grounds that, since the processing plants are in urban areas, removing the restrictions would hurt efforts to reduce urban unemployment. However, the policy may actually have contributed to urban unemployment, since __.", "question": "Which of the following, if true, most logically completes the reasoning in the passage?", "answers": "[\"some of the by-products of cashew processing are used as raw materials by other industries located in Kernland's cities\", 'a lack of profitable crops is driving many small cashew farmers in Kernland off their land and into the cities', 'the government does not place similar restrictions on the export of any crop other than cashews', 'the income earned by workers in the processing plants is generally greater than that earned by agricultural laborers in rural areas']", "label": 1 }, { "id": "train_1655", "context": "Sociologist: Television, telephones, and other electronic media encourage imprecise, uncritical thinking. Yet critical thinking is the only adequate protection against political demagogues, who seek to exploit people by presenting emotionally loaded language as an objective description of reality.", "question": "If the sociologist's statements are true, then each of the following statements could be true EXCEPT:", "answers": "['The mere presence of an orderly system of government in a society provides adequate protection against political demagogues.', 'There are no political demagogues in some highly technological societies.', 'Political demagogues are not the only ones who seek to exploit people by presenting emotionally loaded language as an objective description of reality.', 'Highly emotional people are more easily exploited than less emotional people.']", "label": 0 }, { "id": "train_1656", "context": "Aisha: Vadim is going to be laid off. Vadim' s work as a programmer has been exemplary since joining the firm. But management has already made the decision to lay off a programmer. And this firm strictly follows a policy of laying off the most recently hired programmer in such cases.", "question": "Aisha's conclusion follows logically if which one of the following is assumed?", "answers": "['Vadim is the most recently hired programmer at the firm.', 'Every other programmer at the firm has done better work than Vadim.', 'When Vadim was hired, the policy of laying off the most recently hired programmer was clearly explained.', 'The firm values experience in its programmers more highly than any other quality.']", "label": 0 }, { "id": "train_1657", "context": "Rainfall in the drought-plagued metropolitan area was heavier than usual for the month of June. Nevertheless, by the first of July the city' s water shortage was more severe than ever, and officials proposed drastic restrictions on the use of water.", "question": "Which one of the following, if true, helps to explain why the city's water shortage was not alleviated by the first of July?", "answers": "['Moderate restrictions on the industrial use of water had gone into effect in the metropolitan area several months earlier.', 'People in the metropolitan area who had voluntarily reduced their use of water in earlier months when officials voiced alarm used greater than normal amounts of water when rainfall seemed plentiful in June.', 'During the drought most residents of the metropolitan area had been informed about water conservation methods that would help them to reduce their water consumption significantly with a minimal reduction in their standard of living.', 'Because of the heavier rainfall, people watered their lawns much less in June than they usually do in the metropolitan area during that month.']", "label": 1 }, { "id": "train_1658", "context": "Herbalist: While standard antibiotics typically have just one active ingredient, herbal antibacterial remedies typically contain several. Thus, such herbal remedies are more likely to retain their effectiveness against new, resistant strains of bacteria than are standard antibiotics. For a strain of bacteria, the difficulty of developing resistance to an herbal antibacterial remedy is like a cook' s difficulty in trying to prepare a single meal that will please all of several dozen guests, a task far more difficult than preparing one meal that will please a single guest.", "question": "In the analogy drawn in the argument above, which one of the following corresponds to a standard antibiotic?", "answers": "['several dozen guests', 'a cook', 'a single guest', 'the ingredients available to a cook']", "label": 2 }, { "id": "train_1659", "context": "Four randomly chosen market research companies each produced population estimates for three middle-sized cities; the estimates of each company were then compared with those of the other companies. Two of the cities had relatively stable populations, and for them estimates of current population and of projected population in five years varied little from company to company. However, for the third city, which was growing rapidly, estimates varied greatly from company to company.", "question": "The passage provides the most support for which one of the following?", "answers": "['It is more difficult to estimate the population of middle-sized cities than of smaller cities.', 'The market research companies are likely to be equally reliable in estimating the population of stable cities.', \"Estimates of a city's future population are likely to be more accurate than are estimates of that city's current population.\", 'The rate of change in population of rapidly growing cities does not fluctuate.']", "label": 1 }, { "id": "train_1660", "context": "Many bird and reptile species use hissing as a threat device against potential predators. The way these species produce hissing sounds is similar enough that it is likely that this behavior developed in an early common ancestor. At the time this common ancestor would have lived, however, none of its potential predators would have yet acquired the anatomy necessary to hear hissing sounds.", "question": "Which one of the following, if true, most helps to resolve the apparent discrepancy in the information above?", "answers": "['The production of a hissing sound would have increased the apparent body size of the common ancestor of bird and reptile species.', 'Like its potential predators, the common ancestor of bird and reptile species would have lacked the anatomy necessary to hear hissing sounds.', 'The common ancestor of bird and reptile species would probably have employed multiple threat devices against potential predators.', 'The use of hissing as a threat device would have been less energetically costly than other threat behaviors available to the common ancestor of bird and reptile species.']", "label": 0 }, { "id": "train_1661", "context": "Dairy farmer: On our farm, we have great concern for our cows' environmental conditions. We have recently made improvements that increase their comfort, such as providing them with special sleeping mattresses. These changes are intended to increase blood flow to the udder. This increased blood flow would boost milk output and thus increase profits.", "question": "Of the following propositions, which one is best illustrated by the dairy farmer's statements?", "answers": "['Farming practices introduced for the sake of maximizing profits can improve the living conditions of farm animals.', 'More than other farm animals, dairy cows respond favorably to improvements in their living environments.', 'Dairy cows cannot have comfortable living conditions unless farmers have some knowledge about the physiology of milk production.', \"The key to maximizing profits on a dairy farm is having a concern for dairy cows' environment.\"]", "label": 0 }, { "id": "train_1662", "context": "Prime minister: Our nation' s government should give priority to satisfying the needs of our nation' s people over satisfying the needs of people of any other nation. This is despite the fact that the people of other nations are equal in worth to the people of our nation, which means that it is objectively no more important to satisfy the needs of our nation' s people than to satisfy those of other nations' people.", "question": "Which one of the following principles, if valid, most helps to reconcile the apparent conflict among the prime minister's claims?", "answers": [ "When the people of two nations are equally worthy, the needs of the people of each of those nations should be satisfied primarily by the people's own governments.", "The priority a nation's government should place on satisfying the needs of a group of people depends mainly on how objectively important it is for the needs of those people to be satisfied.", "A nation's government should give priority to satisfying the needs of its own people over satisfying the needs of another nation's people only if its own people are more worthy than the other nation's people.", "A nation's government should give priority to the satisfaction of the needs of a group of people if, but only if, there is no other way for that group's needs to be satisfied." ], "label": 0 }, { "id": "train_1663", "context": "On May first, in order to reduce the number of overdue books, a children' s library instituted a policy of forgiving fines and giving bookmarks to children returning all of their overdue books. On July first there were twice as many overdue books as there had been on May first, although a record number of books had been returned during the interim.", "question": "Which of the following, if true, most helps to explain the apparent inconsistency in the results of the library's policy?", "answers": "['The children were allowed to borrow a maximum of five books for a two-week period, and hence each child could keep a maximum of fifteen books beyond their due date within a two-month period.', 'Although the library forgave overdue fines during the grace period, the amount previously charged the children was minimal; hence, the forgiveness of the fines did not provide enough incentive for them to return their overdue books.', 'The librarians did not keep accurate records of how many children took advantage of the grace period, and some of the children returning overdue books did not return all of their overdue books.', 'The bookmarks became popular among the children, so in order to collect the bookmarks, many children borrowed many more books than they usually did and kept them past their due date.']", "label": 3 }, { "id": "train_1664", "context": "Human beings can exhibit complex, goal-oriented behavior without conscious awareness of what they are doing. Thus, merely establishing that nonhuman animals are intelligent will not establish that they have consciousness.", "question": "Which one of the following is an assumption on which the argument depends?", "answers": "['Some intelligent human behavior is neither complex nor goal-oriented.', 'All forms of conscious behavior involve the exercise of intelligence.', 'Complex, goal-oriented behavior requires intelligence.', 'The possession of intelligence entails the possession of consciousness.']", "label": 2 }, { "id": "train_1665", "context": "Caffeine can kill or inhibit the growth of the larvae of several species of insects. One recent experiment showed that tobacco hornworm larvae die when they ingest a preparation that consists, in part, of finely powdered tea leaves, which contain caffeine. This result is evidence for the hypothesis that the presence of non-negligible quantities of caffeine in various parts of many diverse species of plants is not accidental but evolved as a defense for those plants.", "question": "The argument assumes that", "answers": "['caffeine-producing plants grow wherever insect larvae pose a major threat to indigenous plants or once posed a major threat to the ancestors of those plants', 'the tobacco plant is among the plant species that produce caffeine for their own defense', 'caffeine-producing plants or their ancestors have at some time been subject to being fed upon by creatures sensitive to caffeine', 'caffeine is stored in leaves and other parts of caffeine-producing plants in concentrations roughly equal to the caffeine concentration of the preparation fed to the tobacco hornworm larvae']", "label": 2 }, { "id": "train_1666", "context": "Voter: Our prime minister is evidently seeking a job at an international organization. Anyone seeking a job at an international organization would surely spend a lot of time traveling abroad, and our prime minister has spent more days abroad than at home so far this year.", "question": "Which one of the following arguments is most similar in its flawed reasoning to the voter's argument?", "answers": "['Kao must be a golfer. Kao is planning to run for office, and most people who run for office play golf.', 'Thompson must be negotiating a personal loan. Thompson was at the bank yesterday, and people who are negotiating a personal loan go to the bank to meet with a loan agent.', 'McKinsey must have committed a crime at some point. After all, despite extensive background checks no one has been able to show that McKinsey has never committed a crime.', 'Franklin will lose the coming election. The opposing candidate has better policy ideas and brings more relevant experience to the job.']", "label": 1 }, { "id": "train_1667", "context": "In a highly publicized kidnapping case in Ontario, the judge barred all media and spectators from the courtroom. Her decision was based on the judgment that the public interest would not be served by allowing spectators. A local citizen argued, \"They pleaded with the public to help find the victim; they pleaded with the public to provide tips; they aroused the public interest, then they claimed that allowing us to attend would not serve the public interest. These actions are inconsistent. ", "question": "The reasoning in the local citizen's argument is flawed because this argument", "answers": "['attempts to support its conclusion by making sensationalistic appeals', 'generalizes from an atypical case', \"presumes that the public's right to know is obviously more important than the defendant's right to a fair trial\", 'trades on an ambiguity with respect to the term \"public interest\"']", "label": 3 }, { "id": "train_1668", "context": "Music critic: Some people argue that, unlike certain works of Handel, which set to music familiar religious texts, the organ symphonies of Louis Vieme are not religious music. Quite the contrary. Sitting in Notre Dame cathedral in Paris and hearing his organ symphonies demonstrates that Vierne' s works are divinely inspired.", "question": "The music critic's reasoning is vulnerable to criticism on the ground that it", "answers": "['takes for granted that all organ symphonies are religious music', 'confuses two different meanings of the term \"religious\"', 'confuses two different meanings of the term \"symphonies\"', 'overlooks the possibility that some organ music is not divinely inspired']", "label": 1 }, { "id": "train_1669", "context": "Which of the following, if true, provides the most logical completion of the passage below? Cars fueled by methanol have a much lower level of emissions of pollutants such as carbon monoxide and environmentally harmful hydrocarbons than gasoline-fueled cars do. Methanol fuel does produce somewhat higher formaldehyde emissions than gasoline does.", "question": "Nevertheless, a methanol-powered car actually produces less atmospheric formaldehyde pollution than a comparable gasoline-powered car, because .", "answers": "['compared to carbon monoxide and some hydrocarbons produced by gasoline-powered cars, formaldehyde pollution is not a serious threat to the environment', \"measuring a car's emissions is generally an accurate method of assessing that car's contribution to atmospheric pollution\", 'most formaldehyde pollution generated by gasoline-powered cars results from the photo-chemical conversion of hydrocarbon emissions into formaldehyde in the atmosphere', 'gasoline-powered cars are required by United States law to be equipped with catalytic converters that reduce emissions of many pollutants']", "label": 2 }, { "id": "train_1670", "context": "Insectivorous plants, which unlike other plants have the ability to trap and digest insects, can thrive in soils that are too poor in minerals to support noninsectivorous plants. Yet the mineral requirements of insectivorous plants are not noticeably different from the mineral requirements of noninsectivorous plants.", "question": "The statements above, if true, most strongly support which one of the following hypotheses?", "answers": "['Insectivorous plants can get some of the minerals they require from the insects they trap and digest.', 'Insectivorous plants thrive only in soils that are too poor in minerals to support noninsectivorous plants.', 'The insects that insectivorous plants trap and digest are especially abundant where the soil is poor in minerals.', 'The types of minerals required by noninsectivorous plants are more likely than are the types of minerals required by insectivorous plants to be found in soils poor in minerals.']", "label": 0 }, { "id": "train_1671", "context": "In a study of patients who enrolled at a sleep clinic because of insomnia, those who inhaled the scent of peppermint before going to bed were more likely to have difficulty falling asleep than were patients who inhaled the scent of bitter orange. Since it is known that inhaling bitter orange does not help people fall asleep more easily, this study shows that inhaling the scent of peppermint makes insomnia worse.", "question": "Which one of the following, if true, most seriously weakens the argument above?", "answers": "['Some of the patients who enrolled in the sleep clinic also had difficulty staying asleep once they fell asleep.', 'Several studies have revealed that in many cases inhaling certain pleasant scents can dramatically affect the degree to which a patient suffers from insomnia.', 'Because the scents of peppermint and bitter orange are each very distinctive, it was not possible to prevent the patients from knowing that they were undergoing some sort of study of the effects of inhaling various scents.', 'The patients who inhaled the scent of bitter orange were, on average, suffering from milder cases of insomnia than were the patients who inhaled the scent of peppermint.']", "label": 3 }, { "id": "train_1672", "context": "Last year a large firm set a goal of decreasing its workforce by 25 percent. Three divisions, totaling 25 percent of its workforce at that time, were to be eliminated and no new people hired. These divisions have since been eliminated and no new people have joined the firm, but its workforce has decreased by only 15 percent.", "question": "Which one of the following, if true, contributes most to an explanation of the difference in the planned versus the actual reduction in the workforce?", "answers": "['Some of the employees in the eliminated divisions were eligible for early retirement and chose that option.', 'The three divisions that were eliminated were well run and had the potential to earn profits.', 'Normal attrition in the retained divisions continued to reduce staff because no new people were added to the firm.', 'As the divisions were being eliminated some of their employees were assigned to other divisions.']", "label": 3 }, { "id": "train_1673", "context": "Advertisement: When your car's engine is running at its normal operating temperature, any major brand of motor oil will protect it about as well as Tuff does. When the engine is cold, it is a different story: Tuff motor oil flows better at lower temperatures than its major competitors do. So, if you want your car's engine to have maximum protection, you should use Tuff.", "question": "which of the following, if true, most strengthens the argument in the advertisement?", "answers": "['Tuff motor oil provides above-average protection for engines that happen to overheat.', \"Tuff motor oil's share of the engine oil market peaked three years ago.\", 'Tuff motor oil, like any motor oil, is thicker and flows less freely at cold temperatures than at hot temperatures.', 'Tuff motor oil is manufactured at only one refinery and shipped from there to all markets.']", "label": 0 }, { "id": "train_1674", "context": "Acme Corporation offers unskilled workers excellent opportunities for advancement. As evidence, consider the fact that the president of the company, Ms. Garon, worked as an assembly line worker, an entry-level position requiring no special skills, when she first started at Acme.", "question": "Which one of the following statements, if true, most weakens the reasoning above?", "answers": "[\"Acme's vice president of operations also worked as an assembly line worker when he first started at Acme.\", 'Acme pays entry-level employees slightly higher wages than most other businesses in the same industry.', 'Acme regularly hires top graduates of business schools and employs them briefly in each of a succession of entry-level positions before promoting them to management.', 'Acme promotes its own employees to senior management positions much more frequently than it hires senior managers from other companies.']", "label": 2 }, { "id": "train_1675", "context": "Bowers: A few theorists hold the extreme view that society could flourish in a condition of anarchy, the absence of government. Some of these theorists have even produced interesting arguments to support that position. One writer, for example, contends that anarchy is laissez-faire capitalism taken to its logical extreme. But these theorists' views ignore the fundamental principle of social philosophy -- that an acceptable social philosophy must promote peace and order. Any social philosophy that countenances chaos, i. e. , anarchy, accordingly deserves no further attention.", "question": "The reasoning in Bowers's argument is most vulnerable to criticism on the grounds that", "answers": "['the argument fails to show that laissez-faire capitalism deserves to be rejected as a social philosophy', 'the truth or falsity of a view is not determined by the number of people who accept it as true', 'the meaning of a key term shifts illicitly during the course of the argument', 'it is unreasonable to reject a view merely because it can be described as extreme']", "label": 2 }, { "id": "train_1676", "context": "Editorialist: The positions advanced by radical environmentalists often contain hypotheses that are false and proposals that are economically infeasible. But there is a positive role to be played even by extremists, for the social and political inertia that attends environmental issues is so stubborn that even small areas of progress can be made only if the populace fears environmental disaster, however untenable the reasons for those fears may be.", "question": "Which one of the following most accurately expresses the main conclusion of the editorialist's argument?", "answers": "['Radical environmentalists advocate positions without regard for factual support or economic feasibility.', 'Radical environmentalists often put forth untenable positions in order to produce the fear that is required to bring about moderate reforms.', 'Radical environmentalists, by promoting their views, stimulate progress on environmental issues.', 'Social and political inertia is most effectively overcome by an extremely fearful populace, regardless of whether its fears are well-founded.']", "label": 2 }, { "id": "train_1677", "context": "Long-term and short-term relaxation training are two common forms of treatment for individuals experiencing problematic levels of anxiety. Yet studies show that on average, regardless of which form of treatment one receives, symptoms of anxiety decrease to a normal level within the short-term-training time period. Thus, for most people the generally more expensive long-term training is unwarranted.", "question": "Which one of the following, if true, most weakens the argument?", "answers": "['Recipients of long-term training are much less likely than recipients of short-term training to have recurrences of problematic levels of anxiety.', 'A decrease in symptoms of anxiety often occurs even with no treatment or intervention by a mental health professional.', 'Short-term relaxation training conducted by a more experienced practitioner can be more expensive than long-term training conducted by a less experienced practitioner.', \"The fact that an individual thinks that a treatment will reduce his or her anxiety tends, in and of itself, to reduce the individual's anxiety.\"]", "label": 0 }, { "id": "train_1678", "context": "A computer equipped with signature-recognition software, which restricts access to a computer to those people whose signatures are on file, identifies a person' s signature by analyzing not only the form of the signature but also such characteristics as pen pressure and signing speed. Even the most adept forgers cannot duplicate all of the characteristics the program analyzes.", "question": "Which of the following can be logically concluded from the passage above?", "answers": "['Signature-recognition software has taken many years to develop and perfect.', 'The time it takes to record and analyze a signature makes the software impractical for everyday use.', 'In many cases even authorized users are denied legitimate access to computers equipped with the software.', 'Nobody can gain access to a computer equipped with the software solely by virtue of skill at forging signatures.']", "label": 3 }, { "id": "train_1679", "context": "The cafeteria at Acme Company can offer only four main dishes at lunchtime, and the same four choices have been offered for years. Recently mushroom casserole was offered in place of one of the other main dishes for two days, during which more people chose mushroom casserole than any other main dish. Clearly, if the cafeteria wants to please its customers, mushroom casserole should replace one of the regular dishes as a permanent part of the menu.", "question": "The argument is most vulnerable to criticism on the grounds that it fails to consider", "answers": "['what foods other than main dishes are regularly offered at lunchtime by the cafeteria', \"a desire for variety as a reason for people's choice of mushroom casserole during the days it was offered\", 'the proportion of Acme Company employees who regularly eat lunch in the company cafeteria', 'whether other meals besides lunch are served in the Acme Company cafeteria']", "label": 1 }, { "id": "train_1680", "context": "Using broad-spectrum weed killers on weeds that are competing with crops for sunlight, water, and nutrients presents a difficulty: how to keep the crop from being killed along with the weeds. For at least some food crops, specially treated seed that produces plants resistant to weed killers is under development. This resistance wears off as the plants mature. Ttherefore, the special seed treatment will be especially useful for plants that __.", "question": "Which of the following most logically completes the argument below?", "answers": "['provide, as they approach maturity, shade dense enough to keep weeds from growing', 'produce large seeds that are easy to treat individually, as corn and beans do', 'are cultivated specifically for the seed they produce rather than for their leaves or roots', 'produce their crop over an extended period of time, as summer squash does']", "label": 0 }, { "id": "train_1681", "context": "Polls have shown that a higher percentage of graduating university students are against proposals to reduce government social services than are students entering their first year at a university. These polls lead us to the conclusion that people with a university education are more likely to favor retaining or increasing the present level of government social services than are members of the overall population.", "question": "Which one of the following, if true, most seriously weakens the argument?", "answers": "['Polls of retired persons who have not graduated from a university show a higher percentage of persons in favor of reducing government social services than do polls of retired persons who have graduated from a university.', 'Polls of those who graduated from a university more than five years before being polled show a higher percentage of people in favor of reducing government social services than do polls of the overall population.', 'In the polls cited, graduating university students were more likely to express strong opinions about the question of reducing government social services than were students entering a university.', 'The polls of graduating university students were designed to avoid overrepresenting any single academic discipline.']", "label": 1 }, { "id": "train_1682", "context": "Legal theorist: Governments should not be allowed to use the personal diaries of an individual who is the subject of a criminal prosecution as evidence against that individual. A diary is a silent conversation with oneself and there is no relevant difference between speaking to oneself, writing one' s thoughts down, and keeping one' s thoughts to oneself.", "question": "Which one of the following principles, if valid, provides the most support for the legal theorist's argument?", "answers": "['Governments should not be allowed to compel corporate officials to surrender interoffice memos to government investigators.', \"Governments should not have the power to confiscate an individual's personal correspondence to use as evidence against the individual in a criminal trial.\", 'When crime is a serious problem, governments should be given increased power to investigate and prosecute suspected wrongdoers, and some restrictions on admissible evidence should be relaxed.', \"Governments should not be allowed to use an individual's remarks to prosecute the individual for criminal activity unless the remarks were intended for other people.\"]", "label": 3 }, { "id": "train_1683", "context": "High levels of fertilizer and pesticides, needed when farmers try to produce high yields of the same crop year after year, pollute water supplies. Experts ttherefore urge farmers to diversify their crops and to rotate their plantings yearly. To receive governmental price-support benefits for a crop, farmers must have produced that same crop for the past several years.", "question": "The statements above, if true, best support which of the following conclusions?", "answers": "['Farmers can continue to make a profit by rotating diverse crops, thus reducing costs for chemicals, but not by planting the same crop each year.', 'The rules for governmental support of farm prices work against efforts to reduce water pollution.', 'The only solution to the problem of water pollution from fertilizers and pesticides is to take farmland out of production.', 'Governmental price supports for farm products are set at levels that are not high enough to allow farmers to get out of debt.']", "label": 1 }, { "id": "train_1684", "context": "Medical doctor: Sleep deprivation is the cause of many social ills, ranging from irritability to potentially dangerous instances of impaired decision making. Most people today suffer from sleep deprivation to some degree. Ttherefore we should restructure the workday to allow people flexibility in scheduling their work hours.", "question": "Which one of the following, if true, would most strengthen the medical doctor's argument?", "answers": "['Employees would get more sleep if they had greater latitude in scheduling their work hours.', 'The primary cause of sleep deprivation is overwork.', 'More people would suffer from sleep deprivation today than did in the past if the average number of hours worked per week had not decreased.', 'Individuals vary widely in the amount of sleep they require.']", "label": 0 }, { "id": "train_1685", "context": "One-year-olds ordinarily prefer the taste of sweet food to that of salty food. Yet if one feeds a one-year-old salty food rather than sweet food, then over a period of about a year he or she will develop a taste for the salty flavor and choose to eat salty food rather than sweet food. Thus, a young child' s taste preferences can be affected by the type of food he or she has been exposed to.", "question": "Which one of the following is an assumption required by the argument?", "answers": "[\"A child's taste preferences usually change between age one and age two.\", 'Sweet food is better for infant development than is salty food.', 'Two-year-olds do not naturally prefer salty food to sweet food.', 'The salty food fed to infants in order to change their taste preferences must taste pleasant.']", "label": 2 }, { "id": "train_1686", "context": "The public in the United States has in the past been conditioned to support a substantial defense budget by the threat of confrontation with the Eastern bloc. Now that that threat is dissolving, along with the Eastern bloc itself, it is doubtful whether the public can be persuaded to support an adequate defense budget.", "question": "Which one of the following indicates a weakness in the position expressed above?", "answers": "['It hinges on the term \"adequate, \" the precise meaning of which requires reevaluation in the new context.', 'It assumes as fact what it seeks to establish by reasoning.', \"It presupposes that public opinion can be manipulated indefinitely, without the public's becoming aware of that manipulation.\", 'It refers to past and present events that do not have a causal connection with public support of the budget.']", "label": 0 }, { "id": "train_1687", "context": "Whenever a company loses a major product-liability lawsuit, the value of the company' s stocks falls significantly within hours after the announcement. Cotoy has long been involved in a major product-liability lawsuit, and its stocks fell significantly in value today. Ttherefore, we can be sure that an unfavorable judgment against Cotoy in that lawsuit was announced earlier today.", "question": "Which one of me following contains flawed reasoning that most closely parallels that in the argument above?", "answers": "['Whenever an entering student at Cashman College wins the Performance Fellowship, he or she receives $10, 000. Ttherefore, Eula, a student who has enrolled at Cashman, must have won the Performance Fellowship, because she just received $10, 000 from the college.', \"Whenever the large airlines decrease fares, the financial stability of smaller competing airlines is adversely affected. Ttherefore, the smaller competing airlines' financial stability must be seriously threatened when the large airlines announce a large price decrease.\", \"Whenever a company advertises its products effectively, the company's sales increase. Oroco's sales have not increased; ttherefore, it is likely that the company did not advertise its products effectively.\", 'Whenever a business treats its customers discourteously, its customers begin to shop elsewhere. Shopwell wants to keep all of its customers; ttherefore, its employees will never treat customers discourteously.']", "label": 0 }, { "id": "train_1688", "context": "In recent years the climate has been generally cool in northern Asia. But during periods when the average daily temperature and humidity in northern Asia were slightly higher than their normal levels the yields of most crops grown there increased significantly. In the next century, the increased average daily temperature and humidity attained during those periods are expected to become the norm. Yet scientists predict that the yearly yields of most of the region' s crops will decrease during the next century.", "question": "Which one of the following, if true, most helps to resolve the apparent paradox in the information above?", "answers": "['Crop yields in southern Asia are expected to remain constant even after the average daily temperature and humidity there increase from recent levels.', 'The climate in northern Asia has generally been too cool and dry in recent years for populations of many crop insect pests to become established.', 'Any increases in temperature and humidity would be accompanied by higher levels of atmospheric carbon dioxide, which is vital to plant respiration.', 'In many parts of Asia, the increased annual precipitation that would result from warmer and wetter climates would cause most edible plant species to flourish.']", "label": 1 }, { "id": "train_1689", "context": "For the average person who needs a transfusion, blood from a relative is more likely to be infected with hepatitis than is blood from a blood bank. Ttherefore, the risk of contracting hepatitis from a transfusion is higher for people receiving blood from relatives than for people receiving blood from blood banks.", "question": "Which one of the following, if true, most seriously weakens the argument?", "answers": "['Blood transfusions only rarely result in the recipient being infected with hepatitis.', 'Donors to blood banks are always asked whether they have ever been infected with hepatitis.', \"Blood taken from a relative is highly likely to match a transfusion recipient's blood type.\", 'Blood that is to be used in a transfusion is always screened for hepatitis.']", "label": 3 }, { "id": "train_1690", "context": "Societies cannot dedicate themselves to the study of fine arts unless they have spare time, and people only have spare time when food, shelter, and water are abundant. Although some archeologists believe that the first paintings were actually made to illustrate the plight of starving prehistoric man during the dry season, these early painters must have first discovered the ability to mix minerals to make paint colors. These complex experimentations were the result of a dedication to the study of fine arts.", "question": "The argument leads to the conclusion that", "answers": "['prehistoric man could have accidentally discovered how to mix minerals to make paint colors', 'the study of fine art began in societies that at some time had abundant food, shelter, and water', 'whenever people have food, shelter, and water, they will dedicate themselves to the study of fine arts', 'the study of fine art cannot be done unless one has the ability to mix minerals to make paint colors']", "label": 1 }, { "id": "train_1691", "context": "The only vehicles that have high resale values are those that are well maintained. Thus any well-maintained vehicle has a high resale value.", "question": "The flawed nature of the argument can most effectively be demonstrated by noting that, by parallel reasoning, we could argue that", "answers": "['since the best mediators have the longest track records, the worst mediators have the shortest track records', \"since one's need for medical care decreases as one's health improves, a person who is in an excellent state of health has no need of medical care\", 'since all city dwellers prefer waterfalls to traffic jams, anyone who prefers waterfalls to traffic jams is a city dweller', 'since none of the plants in this garden have been pruned before, no plant in this garden needs pruning']", "label": 2 }, { "id": "train_1692", "context": "Essayist: If Earth' s population continues to grow geometrically, then in a few centuries there will be ten people for every square meter (approximately one person per square foot) of Earth' s surface. Some people have claimed that this will probably not be a problem, since humans will have learned by then how to colonize other planets. This would, however, be a temporary solution at best: if the population continues to double every 30 years, and if in the year 2500 half of Earth' s population emigrated to Mars, then by the year 2530 Earth would be just as crowded as it had been before the emigration.", "question": "Which one of the following most accurately expresses the conclusion drawn in the essayist's argument?", "answers": "['Learning how to colonize other planets would, at best, be a temporary solution to the overcrowding of Earth.', \"The population of Earth's surface will probably continue to grow geometrically even if temporary solutions to population growth, such as colonizing other planets, are adopted.\", \"If Earth's population continues to grow geometrically, then in a few centuries the population density of Earth's surface will be ten people per square meter.\", \"Due to the continuing geometric growth of Earth's population, the problem of overpopulation of Earth will probably persist.\"]", "label": 0 }, { "id": "train_1693", "context": "All highly successful salespersons are both well organized and self-motivated, characteristics absent from many salespersons who are not highly successful. Further, although only those who are highly successful are well known among their peers, no salespersons who are self-motivated regret their career choices.", "question": "If all of the statements above are true, which one of the following must be true?", "answers": "['All salespersons who are well organized but not highly successful are self-motivated.', 'No self-motivated salespersons who are not highly successful are well organized.', 'All salespersons who do not regret their career choices are highly successful.', 'No salespersons who are well known among their peers regret their career choices.']", "label": 3 }, { "id": "train_1694", "context": "Experts hired to testify in court need to know how to make convincing presentations. Such experts are evaluated by juries in terms of their ability to present the steps by which they arrived at their conclusions clearly and confidently. As a result, some less expert authorities who are skilled at producing convincing testimony are asked to testify rather than highly knowledgeable but less persuasive experts.", "question": "Which one of the following most closely conforms to the principle illustrated by the passage above?", "answers": "['Trial lawyers often use the techniques employed by actors to influence the emotions of jurors. Many lawyers have studied drama expressly for the purpose of improving their courtroom skills.', 'The opera singer with the best voice is the appropriate choice even for minor roles, despite the fact that an audience may be more affected by a singer with greater dramatic ability but a lesser voice.', 'Successful politicians are not always the ones who best understand how to help their country. Some lack insight into important political issues but are highly skilled at conducting an election campaign.', 'Job applicants are usually hired because their skills and training best meet a recognized set of qualifications. Only rarely is a prospective employer convinced to tailor a position to suit the skills of a particular applicant.']", "label": 2 }, { "id": "train_1695", "context": "Robert: Speed limits on residential streets in Crownsbury are routinely ignored by drivers. People crossing those streets are endangered by speeding drivers, yet the city does not have enough police officers to patrol every street. So the city should install speed bumps and signs warning of their presence on residential streets to slow down traffic. Sheila: That is a bad idea. People who are driving too fast can easily lose control of their vehicles when they hit a speed bump.", "question": "Sheila's response depends on the presupposition that", "answers": "['with speed bumps and warning signs in place, there would still be drivers who would not slow down to a safe speed', \"Robert's proposal is intended to address a problem that Robert does not in fact intend it to address\", 'most of the people who are affected by the problem Robert describes would be harmed by the installation of speed bumps and warning signs', 'problems of the kind that Robert describes do not occur on any nonresidential streets in Crownsbury']", "label": 0 }, { "id": "train_1696", "context": "John: You told me once that no United States citizen who supports union labor should buy an imported car. Yet you are buying an Alma. Since Alma is one of the biggest makers of imports, I infer that you no longer support unions. Harry: I still support labor unions. Even though Alma is a foreign car company, the car I am buying, the Alma Deluxe, is designed, engineered, and manufactured in the United States.", "question": "Harry's method of defending his purchase of an Alma is to", "answers": "[\"show that John's argument involves a false unstated assumption\", 'disown the principle he formerly held', 'point out that one of the statements John makes in support of his argument is false', 'claim that his is a special case in which the rule need not apply']", "label": 0 }, { "id": "train_1697", "context": "Scientists typically do their most creative work before the age of forty. It is commonly thought that this happens because aging by itself brings about a loss of creative capacity. However, studies show that of scientists who produce highly creative work beyond the age of forty, a disproportionately large number entered their field at an older age than is usual . Since by the age of forty the large majority of scientists have been working in their field for at least fifteen years, the studies' finding strongly suggests that the real reason why scientists over forty rarely produce highly creative work is not that they have aged but rather that scientists over forty have generally spent too long in their field .", "question": "In the argument given, the two portions in boldface play which of the following roles?", "answers": "['The first is an objection that has been raised against a position defended in the argument; the second is that position.', 'The first is evidence that has been used to support an explanation that the argument challenges; the second is a competing explanation that the argument favors.', 'The first provides evidence to support an explanation that the argument favors; the second is that explanation.', 'The first is a claim, the accuracy of which is at issue in the argument; the second is a conclusion drawn on the basis of that claim.']", "label": 2 }, { "id": "train_1698", "context": "When an ordinary piece of steel is put under pressure, the steel compresses; that is, its volume slightly decreases. Glass, however, is a fluid, so rather than compressing, it flows when put under pressure; its volume remains unchanged. Any portion of a sheet of glass that is under sustained pressure will very slowly flow to areas under less pressure. Ttherefore, if a single, extremely heavy object is placed in the middle of a horizontal sheet of glass of uniform thickness and if the glass is able to support the weight without cracking, then the sheet of glass will eventually __.", "question": "Which one of the following most logically completes the argument?", "answers": "['flow toward the point at which the pressure of the object is greatest', 'be thinner in the portion of the glass that is under the pressure of the object than in those portions of the glass that are not under that pressure', 'compress, although not as much as a piece of steel would', 'become larger in size yet still be of uniform thickness']", "label": 1 }, { "id": "train_1699", "context": "The sense of delayed gratification, of working now for later pleasure, has helped shape the economic behavior of our society. However, that sense is no longer nurtured as consistently in our children as it once was. For example, it used to take a bit of patience to put together the toys that children got in cereal boxes; now the toys come from the boxes whole.", "question": "Which of the following is an assumption of the passage above?", "answers": "['The influence of promotion gimmicks on the economic behavior of our society has increased over the years.', 'The toys in cereal boxes have changed partly because the economic conditions of our society have improved.', 'The toys that used to come in cereal boxes were put together by the same children who played with them.', \"Today's children do not expect a single toy to provide pleasure for a long period of time.\"]", "label": 2 }, { "id": "train_1700", "context": "A recent report on an environmental improvement program was criticized for focusing solely on pragmatic solutions to the large number of significant problems that plague the program instead of seriously trying to produce a coherent vision for the future of the program. In response the report' s authors granted that the critics had raised a valid point but explained that, to do anything at all, the program needed continued government funding, and that to get such funding the program first needed to regain a reputation for competence.", "question": "The basic position taken by the report's authors on the criticism leveled against the report is that", "answers": "[\"addressing the critics' concern now would be premature\", 'the notion of a coherent vision would be inappropriate to a program of the sort at issue', 'giving the report a single focus is less desirable than the critics claim', \"the critics' motives are self-serving\"]", "label": 0 }, { "id": "train_1701", "context": "Suffering from kidney failure and having fallen into a coma, Mr. Smith urgently required a kidney transplant. When Mr. Smith' s cousin volunteered to donate a kidney, Mr. Smith' s daughter had to decide whether to sign the consent form permitting the transplant. She knew that her father strongly objected to transplantation of organs from live donors on the grounds that these operations caused serious disability to the organ donor. Although she knew her father' s objection was based on a mistaken belief about the danger to kidney donors, she decided not to consent to the surgery.", "question": "Mr. Smith's daughter's decision conforms to which one of the following principles?", "answers": "[\"A patient's rights to self-determination impose an obligation on others to accede to the patient's preferences concerning treatment, even when those preferences can only be inferred from a general knowledge of the patient's commitments, beliefs, and desires.\", 'A son or a daughter must do everything possible to protect the life and health of a parent unless the parent has prohibited the son or daughter from doing so.', \"Anyone called on to make a medical decision concerning another person should always treat the interests of that person as paramount and decide according to what would most help promote that person's health, regardless of that person's age, mental condition, or physical disabilities.\", \"Withholding a medically advisable treatment from an unconscious patient is justified if securing that treatment would result in another person's becoming seriously disabled.\"]", "label": 0 }, { "id": "train_1702", "context": " Several of a certain bank's top executives have recently been purchasing shares in their own bank. This activity has occasioned some surprise, since it is widely believed that the bank, carrying a large number of bad loans, is on the brink of collapse. Since the executives are well placed to know the bank's true condition, it might seem that their share purchases show that the danger of collapse is exaggerated. However, the available information about the bank's condition is from reliable and informed sources, and corporate executives do sometimes buy shares in their own company in a calculated attempt to calm worries about their company's condition. On balance, ttherefore, it is likely that the executives of the bank are following this example. ", "question": "In the argument given, the two boldfaced portions play which of the following roles?", "answers": "['Each provides evidence to support the position that the argument seeks to establish.', 'The first describes the circumstance the explanation of which is the issue that the argument addresses; the second states the main conclusion of the argument.', 'The first provides evidence to defend the position that the argument seeks to establish against opposing positions; the second states the main conclusion of the argument.', 'The first describes the circumstance the explanation of which is the issue that the argument addresses; the second states a conclusion that is drawn in order to support the main conclusion of the argument.']", "label": 1 }, { "id": "train_1703", "context": "Accountant: The newspaper industry habitually cites the rising cost of newsprint to explain falling profits. But when corrected for inflation, the cost of newsprint is no more than it was ten years ago. Far from being victims of high costs, newspapers have been benefiting from cheap newsprint for decades. The real threats to their profitability are falling circulation and falling advertising.", "question": "The accountant's argument proceeds by", "answers": "['criticizing a newly developed method by demonstrating that a conventional method shows better results', 'challenging an explanation that has been given for a phenomenon in order to introduce a different explanation', 'reinterpreting a popular analogy in order to use that analogy to support an alternative conclusion', 'using economic data to raise doubts about the current effectiveness of a historically accepted approach']", "label": 1 }, { "id": "train_1704", "context": "Kate: The recent decline in numbers of the Tennessee warbler, a North American songbird that migrates each fall to coffee plantations in South America, is due to the elimination of the dense tree cover that formerly was a feature of most South American coffee plantations. Scott: The population of the spruce budworm, the warbler' s favorite prey in North America, has been dropping. This is a more likely explanation of the warbler' s decline.", "question": "Which of the following, if true, most seriously calls Scott's hypothesis into question?", "answers": "['The spruce-budworm population has dropped because of a disease that can infect budworms but not Tennessee warblers.', 'Although many North American songbirds have declined in numbers, no other species has experienced as great a decline as has the Tennessee warbler.', 'The numbers of the Baltimore oriole, a songbird that does not eat budworms but is as dependent on South American coffee plantations as is the Tennessee warbler, are declining.', 'Many Tennessee warbler have begun migrating in the fall to places other than traditional coffee plantations.']", "label": 2 }, { "id": "train_1705", "context": "In an experiment, volunteers walked individually through a dark, abandoned theater. Half of the volunteers had been told that the theater was haunted and the other half that it was under renovation. The first half reported significantly more unusual experiences than the second did. The researchers concluded that reports of encounters with ghosts and other supernatural entities generally result from prior expectations of such experiences.", "question": "Which of the following, if true, would most seriously weaken the researchers' reasoning?", "answers": "['The researchers did not believe that the theater was haunted.', 'None of the volunteers in the second half believed that the unusual experiences they reported were supernatural.', 'Each unusual experience reported by the volunteers had a cause that did not involve the supernatural.', \"All of the volunteers in the first half believed that the researchers' statement that the theater was haunted was a lie.\"]", "label": 3 }, { "id": "train_1706", "context": "Columnist: Wildlife activists have proposed that the practice of stringing cable TV lines from the same poles that carry electric power lines should be banned because cable TV lines, while electrically neutral themselves, make it easier for animals to climb near electric power lines, risking electrocution. This particular argument for banning the practice fails, however, since some animals are electrocuted by power lines even where cable TV lines are all underground.", "question": "Which one of the following most accurately describes a flaw in the columnist's reasoning?", "answers": "['It rejects an argument for a proposal merely on the grounds that the proposal would not completely eliminate the problem it is intended to address.', 'It rejects a proposal to address a problem merely on the grounds that other proposals to address the problem would also be effective.', 'It fails to consider the additional advantageous effects that a proposal to address a problem might have.', \"It takes a sufficient condition for an argument's being inadequate to be a necessary condition for its being inadequate.\"]", "label": 0 }, { "id": "train_1707", "context": "Many parents rigorously organize their children' s activities during playtime, thinking that doing so will enhance their children' s cognitive development. But this belief is incorrect. To thoroughly structure a child' s playtime and expect this to produce a creative and resourceful child would be like expecting a good novel to be produced by someone who was told exactly what the plot and characters must be.", "question": "The argument is most vulnerable to criticism on which one of the following grounds?", "answers": "[\"It fails to consider the possibility that something could enhance a child's overall cognitive development without enhancing the child's creativity and resourcefulness.\", 'It takes for granted that if something is conducive to a certain goal it cannot also be conducive to some other goal.', 'It overlooks the possibility that many children enjoy rigorously organized playtime.', \"It takes a necessary condition for something's enhancing a child's creativity and resourcefulness to be a sufficient condition for its doing so.\"]", "label": 0 }, { "id": "train_1708", "context": "The studies showing that increased consumption of fruits and vegetables may help decrease the incidence of some types of cancer do not distinguish between organically grown and nonorganically grown produce; they were conducted with produce at least some of which contained pesticide residues. The studies may also be taken as showing, ttherefore, that there is no increased health risk associated with eating fruits and vegetables containing pesticide residues.", "question": "The pattern of flawed reasoning in which one of the following is most similar to the pattern of flawed reasoning in the argument above?", "answers": "['Research shows that there is no greater longterm health benefit associated with taking vitamin supplements than with a moderate increase in the intake of fruits and vegetables. Clearly, then, there is no long-term health risk associated with the failure to take vitamin supplements, so long as enough fruits and vegetables are consumed.', 'Research has shown that there is no long-term health risk associated with a diet consisting largely of foods high in saturated fat and cholesterol if such a diet is consumed by someone with a physically active lifestyle. So, exercise is a more useful strategy for achieving cardiological health than is dietary restriction.', 'Research has shown that young people who drive motorcycles and receive one full year of extensive driving instruction are in fact less likely to become involved in accidents than those who simply pass a driving test and drive cars. This shows that there is not an inherently greater risk associated with driving a motorcycle than with driving a car.', 'Research shows that the incidence of certain major illnesses, including heart disease and cancer, is decreased in communities that have a modern power plant. The fact that this tendency is present whether the power plant is nuclear or not shows that there is no increased health risk associated with living next to a nuclear power plant.']", "label": 3 }, { "id": "train_1709", "context": "In response to office workers' worries about the health risks associated with using video display terminals (VDTs), researchers asked office workers to estimate both the amount of time they had spent using VDTs and how often they had suffered headaches over the previous year. According to the survey, frequent VDT users suffered from headaches more often than other office workers did, leading researchers to conclude that VDTs cause headaches.", "question": "Which one of the following, if true, most undermines the researchers' conclusion?", "answers": "['Previous studies have shown that the glare from VDT screens causes some users to suffer eyestrain.', 'Few of the office workers surveyed participated in regular health programs during the year in question.', 'Office workers who experienced frequent headaches were more likely than other workers to overestimate how much time they spent using VDTs.', 'In their study the researchers failed to ask the workers to distinguish between severe migraine headaches and mild headaches.']", "label": 2 }, { "id": "train_1710", "context": "Most people in the United States view neither big nor small business as particularly efficient or dynamic and regard both as providing consumers with fairly priced goods and services. However, most people consistently perceive small business as a force for good in society, whereas big business is perceived as socially responsible only in times of prosperity.", "question": "The statements above, if true, would provide the strongest support for which one of the following hypotheses?", "answers": "['Most people in the United States give little thought to the value of business to society.', 'Even if people did not regard big business as providing consumers with value for their money, they would still regard it as socially responsible in times of general prosperity.', 'Many people in the United States regard the social responsibility of big business as extending beyond providing consumers with fairly priced goods and services.', 'If small business were regarded as being more dynamic, it, too would receive strongly favorable ratings only in times of general prosperity.']", "label": 2 }, { "id": "train_1711", "context": "The traditional treatment of strep infections has been a seven-day course of antibiotics, either penicillin or erythromycin. However, since many patients stop taking those drugs within three days, reinfection is common in cases where those drugs are prescribed. A new antibiotic requires only a three-day course of treatment. Ttherefore, reinfection will probably be less common in cases where the new antibiotic is prescribed than in cases where either penicillin or erythromycin is prescribed.", "question": "Which of the following, if true, most strengthens the argument?", "answers": "['Regardless of whether they take a traditional antibiotic or the new one, most patients feel fully recovered after taking the drug for three days.', 'Some of the people who are allergic to penicillin are likely to be allergic to the new antibiotic.', 'Some physicians have already begun to prescribe the new antibiotic instead of penicillin or erythromycin for the treatment of some strep infections.', 'A course of treatment with the new antibiotic costs about the same as a course of treatment with either penicillin or erythromycin.']", "label": 0 }, { "id": "train_1712", "context": "In Debbie' s magic act, a volunteer supposedly selects a card in a random fashion, looks at it without showing it to her, and replaces it in the deck. After several shuffles, Debbie cuts the deck and supposedly reveals the same selected card, A skeptic conducted three trials. In the first, Debbie was videotaped, and no sleight of hand was found. In the second, the skeptic instead supplied a standard deck of cards. For the third trial, the skeptic selected the card. Each time, Debbie apparently revealed the selected card. The skeptic concluded that Debbie uses neither sleight of hand, nor a trick deck, nor a planted \"volunteer\" to achieve her effect.", "question": "Which one of the following most accurately describes a flaw in the skeptic's reasoning?", "answers": "['The skeptic failed to consider the possibility that Debbie requires both sleight of hand and a trick deck to achieve her effect.', 'The skeptic failed to consider the possibility that sleight of hand could also be detected by some means other than videotaping.', 'The skeptic failed to consider the possibility that Debbie used something other than sleight of hand, a trick deck, or a planted \"volunteer\" to achieve her effect.', 'The skeptic failed to consider the possibility that Debbie did not always use the same method to achieve her effect.']", "label": 3 }, { "id": "train_1713", "context": "Some twentieth-century art is great art. All great art involves original ideas, and any art that is not influential cannot be great art.", "question": "Each of the following statements follows logically from the set of statements above EXCEPT:", "answers": "['Only art that involves original ideas is influential.', 'Only art that is influential and involves original ideas is great art.', 'Some twentieth-century art involves original ideas.', 'Some influential art involves original ideas.']", "label": 0 }, { "id": "train_1714", "context": "Cool Games Corporation derives all of its revenues through the sales of video games. Last year, it fell just short of making a profit although it had strong revenues of $200 million worldwide, half from children and half from adults. This year, the company's sales to children rose 20% to $120 million, but its sales to adults dropped 40% to a disappointing $60 million. One of the most renowned Wall Street analysts had forecasted that Cool Games Corporation would need to at least match its revenues of last year in order to have a chance to make a profit on video games this year. Upon hearing the news of the Cool Games Corporation's revenue, the analyst concluded that it was not possible for the company to have made a profit this year.", "question": "The analyst's conclusion is based on which of the following assumptions?", "answers": "['Cool Game Corporation increased its marketing efforts to children this year.', 'The average sales of all video game companies decreased significantly this year.', 'Sales to adults are more profitable than sales to children.', 'The total cost and expenses of Cool Games Corporation did not decrease by more than the decrease in revenue for the corporation.']", "label": 3 }, { "id": "train_1715", "context": "Scientific and technological discoveries have considerable effects on the development of any society. It follows that predictions of the future condition of societies in which scientific and technological discovery is particularly frequent are particularly untrustworthy.", "question": "The argument depends on assuming which one of the following?", "answers": "['It is not as difficult to predict scientific and technological discoveries in a technologically more advanced society as it is in a technologically less advanced society.', 'The development of a society requires scientific and technological discoveries.', 'Predictions of scientific and technological discoveries or predictions of their effects have harmful consequences in some societies.', 'Forecasts of scientific and technological discoveries, or forecasts of their effects, are not entirely reliable.']", "label": 3 }, { "id": "train_1716", "context": "To reduce the mosquito population in a resort area, hundreds of trees were planted that bear fruit attractive to birds. Over the years, as the trees matured, they attracted a variety of bird species and greatly increased the summer bird population in the area. As expected, the birds ate many mosquitoes. However, the planting of the fruit trees had the very opposite of its intended effect.", "question": "Which one of the following, if true, most helps to explain the apparently paradoxical result?", "answers": "['Most of the species of birds that were attracted by the trees that were planted did not eat mosquitoes.', 'Increases and decreases in mosquito populations tend to follow a cyclical pattern.', 'The species of birds that were attracted in the greatest number by the fruit of the trees that were planted did not eat mosquitoes.', 'The birds attracted to the area by the trees ate many more insects that prey on mosquitoes than they did mosquitoes.']", "label": 3 }, { "id": "train_1717", "context": "The members of the United States Marine Corps (USMC) are famed for their intense devotion to one another. The Marine motto \"Semper Fidelis\" (Latin, \"Always Faithful\") is taken to connote, among other things, the lifelong devotion a Marine has to his or her comrades in the corps. Psychologists who study cohesion among groups have concluded that the USMC creates this intense bond through its unique Recruit Training program.", "question": "Which of the following, if true, most helps to justify the psychologists' position?", "answers": "['Throughout their time in the Corps, whether during war or peace, Marines frequently engage in activities that reinforce the value of cooperation and camaraderie.', 'New recruits are far more likely to engage in independent behavior during the early stages of the USMC Recruit Training than during later stages.', 'During their time in the USMC Recruit Training, new recruits have minimal interactions with people outside of the corps.', 'All branches of the armed forces have levels of internal cohesion much higher than typical civilian groups of similar aged individuals.']", "label": 1 }, { "id": "train_1718", "context": "Fish with teeth specialized for scraping algae occur in both Flower Lake and Blue Lake. Some biologists argue that because such specialized characteristics are rare, fish species that have them should be expected to be closely related. If they are closely related, then the algae-scraping specialization evolved only once. But genetic tests show that the two algae-scraping species, although possibly related, are not closely related. Thus, the algae-scraping specialization evolved more than once.", "question": "The reasoning in the argument is flawed in that it", "answers": "['takes a sufficient condition as a necessary one', 'appeals to the authority of biologists who may not be representative of all biologists with expertise in the relevant area', 'infers merely because something was likely to occur that it did occur', 'infers a cause merely from a correlation']", "label": 0 }, { "id": "train_1719", "context": "A recent study reveals that television advertising does not significantly affect children' s preferences for breakfast cereals. The study compared two groups of children. One group had watched no television, and the other group had watched average amounts of television and its advertising. Both groups strongly preferred the sugary cereals heavily advertised on television.", "question": "Which one of the following statements, if true, most weakens the argument?", "answers": "['Cereal preferences of adults who watch television are known to be significantly different from the cereal preferences of adults who do not watch television.', 'Both groups rejected cereals low in sugar even when these cereals were heavily advertised on television.', 'The preference for sweets is not a universal trait in humans, and can be influenced by environmental factors such as television advertising.', 'The preferences of children who do not watch television advertising are influenced by the preferences of children who watch the advertising.']", "label": 3 }, { "id": "train_1720", "context": "The druid stones discovered in Ireland are very, very old. But this particular druid stone was discovered in Scotland; hence, it must be of more recent vintage.", "question": "The argument is flawed because it", "answers": "['allows a key term to shift in meaning from one use to the next', 'takes the fact that all members of a group have a certain property to constitute evidence that the members of the group are the only things with that property', 'takes for granted the very claim that it sets out to establish', 'takes the fact that most members of a group have a certain property to constitute evidence that all members of the group have that property']", "label": 1 }, { "id": "train_1721", "context": "Technological innovation rarely serves the interests of society as a whole. This can be seen from the fact that those responsible for technological advances are almost without exception motivated by considerations of personal gain rather than societal benefit in that they strive to develop commercially viable technology.", "question": "The argument is most vulnerable to criticism on the grounds that it", "answers": "['contains a premise that cannot possibly be true', 'takes for granted that an action is unlikely to produce a certain outcome unless it is motivated by a desire to produce that outcome', 'takes for granted that technology beneficial to society as a whole cannot be commercially viable', 'fails to consider the possibility that actions motivated by a desire for personal gain often do not result in personal gain']", "label": 1 }, { "id": "train_1722", "context": "To the editor: For generations, magnificent racehorses have been bred in our area. Our most valuable product, however, has been generations of children raised with the character that makes them winners in the contests of life. Gambling is wrong, and children raised in an atmosphere where the goal is to get something for nothing will not develop good character. Those who favor developing good character in children over gambling on horses should vote against allowing our first racetrack to be built. L. E.", "question": "Which one of the following, if true, most weakens L. E. 's argument?", "answers": "['Where voters have had the opportunity to vote on horse racing, they have consistently approved it.', 'Children whose parents gamble do not necessarily gamble when they become adults.', 'If good character is developed in children early, the children continue to have good character in different environments.', 'In other areas with gambling, parents are able to raise children of good character.']", "label": 3 }, { "id": "train_1723", "context": "Press Secretary: Our critics claim that the President' s recent highway project cancellations demonstrate a vindictive desire to punish legislative districts controlled by opposition parties. They offer as evidence the fact that 90 percent of the projects canceled were in such districts. But all of the canceled projects had been identified as wasteful in a report written by respected nonpartisan auditors. So the President' s choice was clearly motivated by sound budgetary policy, not partisan politics.", "question": "Which of the following is an assumption on which the press secretary's argument depends?", "answers": "['Reports by nonpartisan auditors are not generally regarded by the opposition parties as a source of objective assessments of government projects.', \"The highway projects canceled in districts controlled by the President's party were not generally more expensive than the projects canceled in districts controlled by opposition parties.\", \"The scheduled highway projects identified as wasteful in the report were not mostly projects in districts controlled by the President's party.\", 'Canceling highway projects was not the only way for the President to punish legislative districts controlled by opposition parties.']", "label": 2 }, { "id": "train_1724", "context": "Journalist: Recent studies have demonstrated that a regular smoker who has just smoked a cigarette will typically display significantly better short-term memory skills than a nonsmoker, whether or not the nonsmoker has also just smoked a cigarette for the purposes of the study. Moreover, the majority of those smokers who exhibit this superiority in shortterm memory skills will do so for at least eight hours after having last smoked.", "question": "If the journalist's statements are true, then each of the following could be true EXCEPT:", "answers": "['The short-term memory skills exhibited by a regular smoker who last smoked a cigarette five hours ago are typically superior to those exhibited by a regular smoker who has just smoked a cigarette.', 'The short-term memory skills exhibited by a nonsmoker who has just smoked a cigarette are typically superior to those exhibited by a regular smoker who has not smoked for more than eight hours.', 'The short-term memory skills exhibited by a nonsmoker who has just smoked a cigarette are typically superior to those exhibited by a regular smoker who has just smoked a cigarette.', 'A regular smoker who, immediately after smoking a cigarette, exhibits short-term memory skills no better than those typically exhibited by a nonsmoker is nevertheless likely to exhibit superior short-term memory skills in the hours following a period of heavy smoking.']", "label": 2 }, { "id": "train_1725", "context": "Richard: Because it fails to meet the fundamental requirement of art -- that it represent -- abstract art will eventually be seen as an aberration. Jung-Su: Although artists, like musicians, may reject literal representation, makers of abstract art choose to represent the purely formal features of objects, which are discovered only when everyday perspectives are rejected. Thus, whatever others might come to say, abstract art is part of the artistic mainstream.", "question": "Richard and Jung-Su disagree over whether", "answers": "['musicians may reject literal representation', 'abstract art is representational', 'makers of abstract art reject literal representation', 'abstract art will be seen as an aberration']", "label": 1 }, { "id": "train_1726", "context": "Science writer: All scientists have beliefs and values that might slant their interpretations of the data from which they draw their conclusions. However, serious scientific papers are carefully reviewed by many other scientists before publication. These reviewers are likely to notice and object to biases that they do not share. Thus, any slanted interpretations of scientific data will generally have been removed before publication.", "question": "Which one of the following is an assumption required by the science writer's argument?", "answers": "['Biases that are present in published scientific papers and shared by most scientists, including those who review the papers, are unlikely to impair the scientific value of those papers.', 'The interpretation of data is the only part of a serious scientific paper that is sometimes slanted by the beliefs and values of scientists.', 'The scientists reviewing serious scientific papers for publication do not always have biases likely to slant their interpretations of the data in those papers.', 'In general, biases that slant interpretations of data in serious scientific papers being reviewed for publication are not shared among all scientists.']", "label": 3 }, { "id": "train_1727", "context": "Some residents of Midville claim that Midville is generally more expensive to live in than nearby towns are, but these people are mistaken. They focus on Midville' s relatively high tax rate while ignoring the services paid for by their taxes. Only Midville provides residents with trash removal, rent- and mortgage-assistance programs, and reasonably priced public transportation. In nearby towns individuals pay for their own trash removal, and housing and transportation costs are high in comparison to Midville.", "question": "Which one of the following is the main point of the argument?", "answers": "['Some residents of Midville are unaware of many of the services that Midville provides.', 'Some of the residents of Midville consider their taxes to be too high.', 'Midville is generally no more expensive to live in than nearby towns are.', 'Services funded by a municipality are generally less expensive than those services would be if privately funded.']", "label": 2 }, { "id": "train_1728", "context": "The recently negotiated North American Free Trade Agreement among Canada, Mexico, and the United States is misnamed, because it would not result in truly free trade. Adam Smith, the economist who first articulated the principles of free trade held that any obstacle placed in the way of the free movement of goods, investment, or labor would defeat free trade. So since under the agreement workers would be restricted by national boundaries from seeking the best conditions they could find, the resulting obstruction of the flow of trade would, from a free-trade perspective, be harmful.", "question": "The argument proceeds by", "answers": "['responding to a different issue from the one posed', 'using a term in two different senses', 'appealing to a relevant authority', 'citing a nonrepresentative instance']", "label": 2 }, { "id": "train_1729", "context": "The cumbersome spears that were the principal weapons used by certain tribes in the early Bronze Age precluded widespread casualties during intertribal conflicts. But the comparatively high number of warrior tombs found in recent excavations of the same tribes' late Bronze Age settlements indicates that in the late Bronze Age, wars between these tribes were frequent, and the casualty rate was high. Hence some archaeologists claim that by the late Bronze Age, these tribes had developed new methods of warfare designed to inflict many casualties.", "question": "Which one of the following, if true, most supports the archaeologists' claim?", "answers": "['Some tombs of warriors dating to the late Bronze Age contain armor and weapons that anthropologists believe were trophies taken from enemies in battle.', 'A royal tomb dating to the early Bronze Age contained pottery depicting battle scenes in which warriors use spears.', 'Scenes of violent warfare, painted in bright hues, frequently appear on pottery that has been found in some early Bronze Age tombs of warriors.', 'The marks on the bones of many of the late Bronze Age warriors whose tombs were excavated are consistent with the kind of wounds inflicted by arrowheads also found in many late Bronze Age settlements.']", "label": 3 }, { "id": "train_1730", "context": "Researcher: The rate of psychological problems is higher among children of divorced parents than among other children. But it would be a mistake to conclude that these problems are caused by the difficulty the children have adjusting to divorce. It is just as reasonable to infer that certain behaviors that increase the likelihood of divorce -- hostility, distrust, lack of empathy -- are learned by children from their parents, and that it is these learned behaviors, rather than the difficulty of adjusting to divorce, that cause the children' s psychological problems.", "question": "The assertion that children of divorced parents have a higher rate of psychological problems than other children figures in the argument in which one of the following ways?", "answers": "['It is offered as evidence for the claim that certain behaviors are often responsible for divorce.', 'It is the claim that the argument tries to refute.', 'It is cited as an established finding for which the argument proposes an explanation.', 'It is the conclusion of the argument.']", "label": 2 }, { "id": "train_1731", "context": "Environmental activist groups, which clean up waste left on beaches, in parks, and along the side of the road, report that glass bottles make up an ever-increasing percentage of waste they remove. It is clear that attempts to decrease the amount of glass that people litter are failing.", "question": "Which one of the following, if true, most seriously weakens the argument?", "answers": "['Although most glass bottles are recyclable if whole, many of the glass bottles removed by activist companies are shattered into pieces.', 'An increasing proportion of the paper, plastics, and metal cans that the groups used to remove is now being recycled.', 'Because litter poses a sanitary health risk to the public, activist groups are doubling their removal efforts in parks and on beaches.', 'People are more likely to save and reuse glass containers than containers made of lighter materials such as paper or plastic.']", "label": 1 }, { "id": "train_1732", "context": "During the month of May, crabs arrive on Delaware' s beaches to lay eggs. Certain shorebirds depend on these eggs for food during their annual spring migration to their northern breeding grounds. Delaware' s crab population has declined recently. This decline, however, poses no serious threat to the migrating shorebird populations: by remaining longer in Delaware, the birds will be able to eat enough crab eggs to allow them to complete their migration.", "question": "Which of the following, if true, most seriously weakens the argument?", "answers": "[\"The decline in the number of crabs on Delaware's beaches is due to the harvesting of crabs by humans.\", 'The crabs do not conceal their eggs.', \"No other food available to the birds on Delaware's beaches is as nutritious and accessible as are crab eggs.\", 'The earlier in the season the shorebirds complete their migration, the more likely they are to breed successfully.']", "label": 3 }, { "id": "train_1733", "context": "Shortly after Isaac Newton circulated some of his theories of light in 1672, his colleague Robert Hooke claimed that most of those theories were based on Hooke' s own work. A modern reader might interpret Newton' s famous comment, \"if I have seen further it is by standing on the shoulders of giants, \" as a conciliatory gesture acknowledging indebtedness to Hooke and other contemporary scientists for some of his theories. Conciliatory gestures acknowledging indebtedness were uncharacteristic of Newton, however, and in his day such allusions to \"giants\" typically referred to the ancient Greeks, not to contemporary scientists.", "question": "The statements in the passage, if true, most strongly support which one of the following?", "answers": "[\"Newton did not believe that any of Hooke's theories of light were based on those of the ancient Greeks.\", \"Newton did not intend the quoted comment to be an acknowledgment that his theories of light were largely derived from Hooke's.\", \"Newton was not familiar with Hooke's work on the theory of light.\", 'Newton intended to credit some contemporary scientists other than Hooke for some of the advances that Newton made in the theory of light.']", "label": 1 }, { "id": "train_1734", "context": "Commentator: A political constitution that provides the framework for the laws of a nation must be interpreted to apply to new situations not envisioned by its authors. Although these interpretations express the moral and political beliefs of the interpreters, they are usually portrayed as embodying the intentions of the authors. This fiction is vital because without it the illusion, so necessary for political stability, that laws are the bequest of a long tradition rather than the preferences of contemporary politicians would vanish.", "question": "Which one of the following is most strongly supported by the commentator's statements, if they are true?", "answers": "['The perceived lack of a long legal tradition in a nation makes the political stability of that nation dependent upon the fiction that its present political leaders share the intentions of the authors of the constitution of that nation.', 'Political instability will ensue if people come to believe there is a divergence between the beliefs of the authors of their constitution and those of their present political leaders.', 'A written constitution preserves the illusion that laws are the bequest of a long tradition rather than the creations of modern politicians.', 'Political instability will increase if the people of a nation cease to believe that their constitution is being interpreted consistently with the intentions of its authors.']", "label": 3 }, { "id": "train_1735", "context": "Poppy petals function to attract pollinating insects. The pollination of a poppy flower triggers the release into that flower of a substance that causes its petals to wilt within one or two days. If the flower is not pollinated, the substance will not be released and the petals will remain fresh for a week or longer, as long as the plant can nourish them. Cutting an unpollinated poppy flower from the plant triggers the release into the flower of the same substance whose release is triggered by pollination.", "question": "The statement above, if true, most strongly support which one of the following?", "answers": "['Pollinating insects are not attracted to wilted poppy flowers.', 'Poppy plants are unable to draw nutrients from soil or water after the substance that causes wilting has been released.', 'The pollen on pollinated poppy flowers prevents their petals from absorbing the nutrients carried to them by their stems.', 'Even if cut poppies are given all necessary nutrients, their petals will tend to wilt within a few days.']", "label": 3 }, { "id": "train_1736", "context": "Transnational cooperation among corporations is experiencing a modest renaissance among United States firms. even though projects undertaken by two or more corporations under a collaborative agreement are less profitable than projects undertaken by a single corporation. The advantage of transnational cooperation is that such joint international projects may allow United States firms to win foreign contracts that they would not otherwise be able to win.", "question": "Which of the following is information provided by the passage above?", "answers": "['Transnational cooperation involves projects too big for a single corporation to handle.', 'Transnational cooperation has in the past been both more common and less common than it Is now among United States firms.', 'Joint projects between United States and foreign corporations are not profitable enough to be worth undertaking.', 'Transnational cooperation results in a pooling of resources leading to high-quality performance.']", "label": 1 }, { "id": "train_1737", "context": "Ecologist: Without the intervention of conservationists, squirrel monkeys will become extinct. But they will survive if large tracts of second-growth forest habitat are preserved for them. Squirrel monkeys flourish in second-growth forest because of the plentiful supply of their favorite insects and fruit.", "question": "Which one of the following can be properly inferred from the ecologist's statements?", "answers": "['If conservationists intervene to help squirrel monkeys survive, then the squirrel monkeys will not become extinct.', 'At least some of the conservationists who intervene to help the squirrel monkeys survive will do so by preserving second-growth forest habitat for the monkeys.', 'Without the intervention of conservationists, large tracts of second-growth forest habitat will not be preserved for squirrel monkeys.', \"No habitat other than second-growth forest contains plentiful supplies of squirrel monkeys' favorite insects and fruit.\"]", "label": 2 }, { "id": "train_1738", "context": "In each of the past five years, Barraland' s prison population has increased. Yet, according to official government statistics, for none of those years has there been either an increase in the number of criminal cases brought to trial, or an increase in the rate at which convictions have been obtained. Clearly, ttherefore, the percentage of people convicted of crimes who are being given prison sentences is on the increase.", "question": "Which of the following, if true, most seriously weakens the argument?", "answers": "['The number of people in Barraland who feel that crime is on the increase is significantly greater now than it was five years ago.', 'Barraland has been supervising convicts on parole more closely in recent years, with the result that parole violations have become significantly less frequent.', 'Ten years ago, Barraland reformed its criminal justice system, imposing longer minimum sentences for those crimes for which a prison sentence had long been mandatory.', 'In Barraland the range of punishments that can be imposed instead of a prison sentence is wide.']", "label": 2 }, { "id": "train_1739", "context": "Jewel collectors, fearing that their eyes will be deceived by a counterfeit, will not buy a diamond unless the dealer guarantees that it is genuine. But why should a counterfeit give any less aesthetic pleasure when the naked eye cannot distinguish it from a real diamond? Both jewels should be deemed of equal value.", "question": "Which one of the following principles, if valid, most helps to justify the reasoning in the argument above?", "answers": "['The value of a jewel should depend at least partly on market demand.', 'Jewel collectors should not buy counterfeit jewels unless they are unable to distinguish counterfeit jewels from real ones.', 'The value of a jewel should derive solely from the aesthetic pleasure it provides.', 'It should not be assumed that everyone who likes diamonds receives the same degree of aesthetic pleasure from them.']", "label": 2 }, { "id": "train_1740", "context": "Under the influence of today' s computer-oriented culture, publishing for children has taken on a flashy new look that emphasizes illustrations and graphic design; the resulting lack of substance leads to books that are short-lived items covering mainly trendy subjects. The changes also include more humorous content, simplification of difficult material, and a narrower focus on specific topics.", "question": "Which one of the following is most strongly supported by the information above?", "answers": "[\"The substance of a children's book is important to its longevity.\", \"The lack of substance of a children's book is unlikely to be important to its popularity.\", 'Children judge books primarily on the basis of graphic design.', \"The inclusion of humorous material and a narrower focus detract from the substance of a children's book.\"]", "label": 0 }, { "id": "train_1741", "context": "Economist: Real wages in this country will increase significantly only if productivity increases notably. Thus, it is unlikely that real wages will increase significantly in the near future, since this country' s businesses are currently investing very little in new technology and this pattern is likely to continue for at least several more years.", "question": "Which one of the following, if assumed about the economist's country, allows the economist's conclusion to be properly drawn?", "answers": "['Productivity will not increase if businesses do not make a substantial investment in new technology.', 'The new technology in which businesses are currently investing is not contributing to an increase in productivity.', 'Neither real wages nor productivity have increased in the last several years.', 'Sooner or later real wages will increase significantly.']", "label": 0 }, { "id": "train_1742", "context": "Office manager: I will not order recycled paper for this office. Our letters to clients must make a good impression, so we cannot print them on inferior paper. Stationery supplier: Recycled paper is not necessarily inferior. In fact, from the beginning, the finest paper has been made of recycled material. it was only in the 1850s that paper began to be made from wood fiber, and then only because there were no longer enough rags to meet the demand for paper.", "question": "In which one of the following ways does the stationer's response fail to address the office manager's objection to recycled paper?", "answers": "[\"It ignores the office manager's legitimate concern about quality.\", 'It assumes that the office manager is concerned about environmental issues.', \"It does not recognize that the office manager's prejudice against recycled paper stems from ignorance.\", 'It uses irrelevant facts to justify a claim about the quality of the disputed product.']", "label": 3 }, { "id": "train_1743", "context": "Citizens of Parktown are worried by the increased frequency of serious crimes committed by local teenagers. In response, the city government has instituted a series of measures designed to keep teenagers at home in the late evening. Even if the measures succeed in keeping teenagers at home, however, they are unlikely to affect the problem that concerns citizens, since most crimes committed by local teenagers take place between 3 p. m. and 6 p. m.", "question": "Which of the following, if true, most substantially weakens the argument?", "answers": "['Any decrease in the need for police patrols in the late evening would not mean that there could be more intensive patrolling in the afternoon.', 'The schools in Parktown have introduced a number of after-school programs that will be available to teenagers until 6 p. m. on weekday afternoons.', 'The crimes committed by teenagers in the afternoon are mostly small thefts and inconsequential vandalism.', 'Teenagers are much less likely to commit serious crimes when they are at home than when they are not at home.']", "label": 2 }, { "id": "train_1744", "context": "Police officer: Traffic on major highways has been sluggish due to increased cell phone use while driving. Cell phone use while driving causes drivers to drive under the speed limit and to cause other drivers to pass them on the road. Drivers argue that talking on cell phones is necessary to discuss plans and ask directions and that traffic delays are inevitable. However, in Denton, the local officials have created a special lane for drivers talking on cell phones. This prevents these drivers from slowing down the rest of traffic on the highway. Ttherefore, it is possible to allow people to make plans and find out how to get to their destinations without causing sluggish traffic.", "question": "The police officer's argument proceeds by", "answers": "['proposing an alternative method for drivers to discuss their plans and ask directions', 'producing a counterexample to a general assertion', \"appealing to the police officer's training on highway safety to discredit a commonly held view\", 'pointing out a dissimilarity between drivers who talk on cell phones while driving and drivers in Denton']", "label": 1 }, { "id": "train_1745", "context": "The axis of Earth' s daily rotation is tilted with respect to the plane of its orbit at an angle of roughly 23 degrees. That angle can be kept fairly stable only by the gravitational influence of Earth' s large, nearby Moon. Without such a stable and moderate axis tilt, a planet' s climate is too extreme and unstable to support life. Mars, for example, has only very small moons, tilts at wildly fluctuating angles, and cannot support life.", "question": "If the statements above are true, which one of the following must also be true on the basis of them?", "answers": "[\"Gravitational influences other than moons have little or no effect on the magnitude of the tilt angle of either Earth's or Mars's axis.\", \"If Earth's Moon were to leave Earth's orbit, Earth's climate would be unable to support life.\", 'No planet that has more than one moon can support life.', 'Any planet with a stable, moderate axis tilt can support life.']", "label": 1 }, { "id": "train_1746", "context": "Last year a record number of new manufacturing jobs were created. Will this year bring another record? Well, any new manufacturing job is created either within an existing company or by the start-up of a new company. Within existing firms, new jobs have been created this year at well below last year's record pace. At the same time, there is considerable evidence that the number of new companies starting up will be no higher this year than it was last year and there is no reason to think that the new companies starting up this year will create more jobs per company than did last year's start-ups. So clearly, the number of new jobs created this year will fall short of last year's record. ", "question": "In the argument given, the two portions in boldface play which of the following roles?", "answers": "['The first is a generalization that the argument seeks to establish; the second is the main conclusion of the argument.', 'The first and the second each provide evidence in support of the main conclusion of the argument.', 'The first is presented as an obvious truth on which the argument is based; the second is a conclusion drawn in order to support the main conclusion of the argument.', 'The first is presented as an obvious truth on which the argument is based; the second is the main conclusion of the argument.']", "label": 3 }, { "id": "train_1747", "context": "Last year a record number of new manufacturing jobs were created. Will this year bring another record? Well, any new manufacturing job is created either within an existing company or by the start-up of a new company. Within existing firms, new jobs have been created this year at well below last year's record pace . At the same time, there is considerable evidence that the number of new companies starting up will be no higher this year than it was last year and there is no reason to think that the new companies starting up this year will create more jobs per company than did last year's start-ups . So clearly, the number of new jobs created this year will fall short of last year's record.", "question": "In the argument given, the two portions in boldface play which of the following roles?", "answers": "['The first is a claim that the argument challenges; the second is a judgment advanced in support of the main conclusion of the argument.', 'The first provides evidence in support of the main conclusion of the argument; the second is an objection that has been raised against that main conclusion.', 'The first and the second are each claims that have been advanced in support of a position that the argument opposes.', 'The first provides evidence in support of the main conclusion of the argument; the second is a judgment advanced in support of that main conclusion']", "label": 3 }, { "id": "train_1748", "context": "In a recent study of dust-mite allergy sufferers, one group slept on mite-proof bedding, while a control group slept on bedding that was not mite-proof. The group using mite-proof bedding had a 69 percent reduction in the dust-mite allergen in their mattresses, whereas there was no significant reduction in the control group. However, even though bedding is the main source of exposure to dust mites, no symptom reduction was reported in either group.", "question": "Which one of the following, if true, most helps to resolve the apparent conflict in the statements above?", "answers": "['The medical community does not fully understand how dust-mite allergens cause allergy.', \"Dust-mite allergens in bedding tend to irritate many allergy sufferers' nasal passages more than do the same allergens in other locations, such as carpets.\", 'When people report their own allergy symptoms, they tend to exaggerate the severity of those symptoms.', 'For dust-mite allergy sufferers to get relief from their allergies, dust-mite allergens must be reduced by 90 to 95 percent.']", "label": 3 }, { "id": "train_1749", "context": "Editorialist: Additional restrictions should be placed on driver' s licenses of teenagers because teenagers lack basic driving skills. Even though drivers of age nineteen and younger make up only 7 percent of registered drivers, they are responsible for over 14 percent of traffic fatalities.", "question": "Each of the following, if true, weakens the argument that teenagers lack basic driving skills EXCEPT:", "answers": "['Teenagers cause car accidents that are more serious than those caused by others.', 'Teenagers tend to drive older and less stable cars than other drivers.', 'Teenagers and their passengers are less likely to use seat belts and shoulder straps than others.', 'Teenagers are likely to drive with more passengers than the average driver.']", "label": 0 }, { "id": "train_1750", "context": "Last year, pharmaceutical manufacturers significantly increased the amount of money they spent promoting new drugs, which they do mainly by sending sales representatives to visit physicians in their offices. However, two years ago there was an average of 640 such visits per representative, whereas last year that figure fell to 501. So the additional promotion must have been counterproductive, making physicians less willing to receive visits by pharmaceutical sales representatives.", "question": "Which one of the following, if true, most weakens the argument?", "answers": "['Most pharmaceutical companies did not increase the amount of money they spend promoting drugs through advertising targeted directly at consumers.', 'Most pharmaceutical manufacturers increased the size of their sales forces so that their sales representatives could devote more time to each physician.', \"The more visits a physician receives from a pharmaceutical sales representative, the more likely he or she is to prescribe drugs made by that representative's company.\", 'Most physicians who agree to receive a visit from a pharmaceutical sales representative will see that representative more than once during a given year.']", "label": 1 }, { "id": "train_1751", "context": "Committee member: We should not vote to put at the top of the military' s chain of command an individual whose history of excessive drinking is such that that person would be barred from commanding a missile wing, a bomber squadron, or a contingent of fighter jets. Leadership must be established from the top down.", "question": "The committee member's argument conforms most closely to which one of the following principles?", "answers": "['No one who would be barred from important jobs in an organization should lead that organization.', 'Whoever leads an organization must have served at every level in the organization.', \"No one who drinks excessively should hold a leadership position anywhere along the military's chain of command.\", 'Whoever leads an organization must be qualified to hold each important job in the organization.']", "label": 0 }, { "id": "train_1752", "context": "One might expect that within a particular species, any individuals that managed to slow down the aging process would leave more offspring. Natural selection should ttherefore favor extreme longevity-but this does not seem to be the case. A possible explanation is that aging is a product of the inevitable wear and tear of living, similar to how household appliances generally accumulate faults that lead to their eventual demise. However, most researchers do not find this analogy satisfactory as an explanation.", "question": "Which of the following would, if true, provide the strongest explanation for the researchers' reaction?", "answers": "['Organisms are capable of maintenance and self-repair and can remedy much of the damage that they accumulate.', 'Some organisms are capable of living much longer than other organisms.', 'Some organisms reproduce very quickly despite having short lifespans.', 'Some organisms generate much more wear and tear on their bodies than others.']", "label": 0 }, { "id": "train_1753", "context": "A factory spokesperson argued that the factory should not be required to clean up the water in the nearby wetlands, maintaining that although wastewater from the factory polluted the wetlands over the past several years, the factory is not to blame for this, since the disposal of the factory' s wastewater is handled entirely by an independent contractor.", "question": "Which one of the following arguments most closely conforms to the principle underlying the reasoning in the spokesperson's argument?", "answers": "['I object to the policy of making parents responsible for the offenses of their older adolescent children. After all, these adolescents have minds of their own and freely choose to act as they do, often in ways that do not reflect the wishes of their parents.', 'The students are justified in their objection to the reading assignment. Many of the topics concern material that is not covered in class, and students should not be required to do such reading in order to do well in the course.', \"A recent survey revealed that over two-thirds of the teachers in the district are permitted to teach classes on subjects in which they have received no formal training. Thus parents of students in the district should check the qualifications of their children's teachers.\", \"Despite all the publicity, I am skeptical of the politician's claims of having just returned from the remote village. Just two days ago a reporter spoke with the villagers and said that not a single one reported seeing the politician in the past several months.\"]", "label": 0 }, { "id": "train_1754", "context": "In 1963, a young macaque monkey was observed venturing into a hot spring to retrieve food which had fallen in. Soon, other macaques began to enter the spring, and over a few years this behavior was adopted by the entire troop. Prior to 1963, no macaques had ever been observed in the hot spring; by 1990, the troop was regularly spending time there during the winters. Thus, these macaques are able to adopt and pass on new patterns of social behavior, and are not complete captives of their genetic heritage.", "question": "Which one of the following is an assumption required by the argument above?", "answers": "['The social behaviors of macaques are completely independent of their genetic heritage.', 'New patterns of behavior that emerge in macaque populations over the course of a few years or decades are not necessarily genetically predetermined.', 'Mutations in the genetic heritage of a certain variety of macaques can occur over a time span as short as a few years or decades.', \"The macaques' new pattern of behavior will persist over several generations.\"]", "label": 1 }, { "id": "train_1755", "context": "Over the last five years, every new major alternative-energy initiative that initially was promised government funding has since seen that funding severely curtailed. In no such case has the government come even close to providing the level of funds initially earmarked for these projects. Since large corporations have made it a point to discourage alternative-energy projects, it is likely that the corporations' actions influenced the government' s funding decisions.", "question": "Which one of the following, if true, most strengthens the reasoning above?", "answers": "['The funding initially earmarked for a government project is always subject to change, given the mechanisms by which the political process operates.', 'For the past two decades, most alternative- energy initiatives have received little or no government funding.', 'All large corporations have made it a point to discourage some forms of research.', 'The only research projects whose government funding has been severely curtailed are those that large corporations have made it a point to discourage.']", "label": 3 }, { "id": "train_1756", "context": "Bernard: For which language, and thus which frequency distribution of letters and letter sequences, was the standard typewriter keyboard designed? Cora: To ask this question, you must be making a mistaken assumption: that typing speed was to be maximized. The real danger with early typewriters was that operators would hit successive keys too quickly, thereby crashing typebars into each other, bending connecting wires, and so on. So the idea was to slow the operator down by making the most common letter sequences awkward to type. Bernard: This is surely not right! These technological limitations have long since vanished, yet the keyboard is still as it was then.", "question": "Which one of the following, if true, could be used by Cora to counter Bernard's rejection of her explanation?", "answers": "['Typewriters and word-processing equipment are typically sold to people who have learned to use the standard keyboard and who, ttherefore, demand it in equipment they buy.', 'The standard keyboard allows skilled operators to achieve considerable typing speeds, though it makes acquiring such skills relatively difficult.', 'A person who has learned one keyboard layout can readily learn to use a second one in place of the first, but only with difficulty learn to use a second one alongside the first.', 'It is now possible to construct typewriters and word-processing equipment in which a single keyboard can accommodate two or even more different keyboard layouts, each accessible to the operator at will.']", "label": 0 }, { "id": "train_1757", "context": "Ted, a senior employee, believes he is underpaid and attempts to compensate by routinely keeping short hours, though it is obvious to everyone that he still makes some valuable, unique, and perhaps irreplaceable contributions. Tatiana, Ted' s supervisor, is aware of the deficit in Ted' s performance, and realizes other workers work harder than they should to make up for it. Nevertheless, Tatiana decides that she should not request that Ted be replaced.", "question": "Which one of the following principles, if valid, would most help to justify Tatiana's decision?", "answers": "['Supervisors should request that an employee be replaced only if they know that all the work done by that employee can be performed equally well by another employee.', 'Workers in a work setting should regard themselves as jointly responsible for the work to be performed.', \"Only someone with greater authority than a particular employee's supervisor is entitled to decide whether that employee should be replaced.\", 'Employers should compensate all their employees in a way that is adequate in relation to the value of the contributions they make.']", "label": 0 }, { "id": "train_1758", "context": "An electric utility has determined that a new power plant is needed and has decided to build either a natural gas-fired plant or a waste-to-energy plant that would serve as both a trash incinerator and a power plant. Surprisingly, although the waste-to-energy plant would produce roughly three times as much air pollution as the gas-fired plant, environmentalists have come out in unanimous support of this option.", "question": "Which one of the following, if true, most helps to justify the environmentalists' position?", "answers": "['In the area where the utility operates, both energy use and the volume of trash produced have increased substantially over the last several years.', 'The waste-to-energy plant would replace an existing trash incinerator that produces much more air pollution than the waste-to-energy plant would.', 'The vast majority of the air pollution in the area where the utility operates is produced by trucks and automobiles.', 'Modern gas-fired power plants produce significantly less pollution than gas-fired power plants that were built several decades ago.']", "label": 1 }, { "id": "train_1759", "context": "Patrick usually provides child care for six children. Parents leave their children at Patrick' s house in the morning and pick them up after work. At the end of each workweek, the parents pay Patrick at an hourly rate for the child care provided that week. The weekly income Patrick receives is usually adequate but not always uniform, particularly in the winter, when children are likely to get sick and be unpredictably absent.", "question": "Which of the following plans, if put into effect, has the best prospect of making Patrick's weekly income both uniform and adequate?", "answers": "['Increase the hourly rate to a level that would provide adequate income even in a week when half of the children Patrick usually cares for are absent.', 'Hire a full-time helper and invest in facilities for providing child care to sick children.', \"Increase the number of hours made available for child care each day, so that parents can leave their children in Patrick's care for a longer period each day at the current hourly rate.\", 'Replace payment by actual hours of child care provided with a fixed weekly fee based upon the number of hours of child care that Patrick would typically be expected to provide.']", "label": 3 }, { "id": "train_1760", "context": "A tiny, tree-living thrips is the first species identified that can reproduce both by laying eggs and by bearing live young, although for any one instance of reproduction, a given female will use only one of the two methods. Interestingly, biologists have observed that all of the offspring of this insect that are hatched from eggs are females, and all of the offspring that are live-born are males. It has also been noted that any particular live-born brood will comprise fewer individuals than any particular brood hatched from eggs. However, a much larger proportion of male offspring than of female offspring survives to adulthood, and among thrips reaching adulthood the ratio of males to females is about even.", "question": "Which one of the following can be properly inferred about the species of thrips described in the passage?", "answers": "['Any female of the species that can reproduce by laying eggs can also reproduce by bearing live young but not necessarily vice versa.', 'There are females that use only one of the two methods of reproduction over the course of their lives.', 'On average, across the species, more of the offspring are born by hatching from eggs than are born live.', 'For the species as a whole, there are, over time, about as many instances of live-born broods as there are broods hatched from eggs.']", "label": 2 }, { "id": "train_1761", "context": "A high percentage of injuries to the anterior cruciate ligament (ACL) occurs in athletes. A sophisticated statistical analysis looked at the number of athletes who suffered an ACL injury, and assigned numerical values to both the nature of the stress that produced the injury and to the resultant level of damage to the ACL. Researchers expected that, controlling for level of injury-causing stress, all athletically inclined individuals would show similar tendency toward ACL injury regardless of the specific sport. A surprising outcome is that, equalizing for injuring-causing stress, it appears that gymnasts have a much higher tendency toward ACL injury than do members of the National Football League (NFL), but this is most likely because __", "question": "Which of the following most logically completes the argument?", "answers": "['gymnasts are often given credit for inventing new positions or moves, while football players far more regularly follow the typical moves associated with their position.', 'high school and college football players who have a higher tendency to ACL injury are far less likely to advance to the NFL.', 'gymnasts are in general far more flexible than football players.', 'most gymnasts, as those preparing for the Olympics, are amateur athletes, whereas NFL players are professional athletes.']", "label": 1 }, { "id": "train_1762", "context": "On the first day of trout season a team of biologists went with local trout anglers to the Macawber River. Each angler who caught at least 2 trout chose exactly 2 of these trout for the biologists to weigh. A total of 90 fish were weighed. The measurements show that at the beginning of this season the average trout in the Macawber River weighed approximately 1. 6 kilograms.", "question": "The reasoning above is most vulnerable to criticism on the grounds that it", "answers": "['does not consider whether any fish other than trout were caught', 'fails to take into account measurements from the same time in previous seasons', 'makes a generalization from a sample that is unlikely to be representative', 'relies on evidence that is anecdotal rather than scientific']", "label": 2 }, { "id": "train_1763", "context": "The ancient reptile Thrinaxodon, an ancestor of mammals, had skull features suggesting that it had sensory whiskers. If Thrinaxodon had whiskers, it clearly also had hair on other parts of its body, which would have served as insulation that regulated body temperature. Ttherefore, Thrinaxodon was probably warm-blooded, for such insulation would be of little use to a cold-blooded animal.", "question": "Which one of the following most accurately describes the role played in the argument by the statement that if Thrinaxodon had whiskers, it clearly also had hair on other parts of its body, which would have served as insulation that regulated body temperature?", "answers": "['It is a premise offered in support of the main conclusion drawn in the argument.', \"It is offered as an explanation of the phenomenon described by the argument's main conclusion, but it is not itself used to provide support for that conclusion.\", 'It is a premise offered in support of the conclusion that insulation regulating body temperature would be of little use to a coldblooded animal.', 'It is a conclusion for which the claim that Thrinaxodon had skull features suggesting that it had sensory whiskers is offered as support.']", "label": 0 }, { "id": "train_1764", "context": "A physician who is too thorough in conducting a medical checkup is likely to subject the patient to the discomfort and expense of unnecessary tests. One who is not thorough enough is likely to miss some serious problem and ttherefore give the patient a false sense of security. It is difficult for physicians to judge exactly how thorough they should be. Ttherefore, it is generally unwise for patients to have medical checkups when they do not feel ill.", "question": "Which one of the following, if true, would most seriously weaken the argument in the passage?", "answers": "['Some serious diseases in their early stages have symptoms that physicians can readily detect, although patients are not aware of any problem.', 'Patients not medically trained are unable to judge for themselves what degree of thoroughness is appropriate for physicians in conducting medical checkups.', 'Many people are financially unable to afford regular medical checkups.', 'Under the pressure of reduced reimbursements, physicians have been reducing the average amount of time they spend on each medical checkup.']", "label": 0 }, { "id": "train_1765", "context": "Pundit: People complain about how ineffectual their legislative representatives are, but this apparent ineffectuality is simply the manifestation of compromises these representatives must make when they do what they were elected to do: compete for the government' s scarce funds. So, when people express dissatisfaction with their legislative representatives, we can be assured that these representatives are simply doing what they were elected to do.", "question": "The pundit's argument is flawed because it takes for granted that", "answers": "['legislative compromise inevitably results in popular dissatisfaction with politicians', 'constituents would continue to be dissatisfied with the effectuality of their legislative representatives if constituents were aware of the cause of this apparent ineffectuality', 'only elected public servants tend to elicit dissatisfaction among the public', 'the apparent ineffectuality of legislative representatives is the only source of popular dissatisfaction with those representatives']", "label": 3 }, { "id": "train_1766", "context": "Historian: The Land Party achieved its only national victory in Banestria in 1935. It received most of its support that year in rural and semirural areas, where the bulk of Banestria' s population lived at the time. The economic woes of the years surrounding that election hit agricultural and small business interests the hardest, and the Land Party specifically targeted those groups in 1935. I conclude that the success of the Land Party that year was due to the combination of the Land Party' s specifically addressing the concerns of these groups and the depth of the economic problems people in these groups were facing.", "question": "Each of the following, if true, strengthens the historian's argument EXCEPT:", "answers": "['Voters are more likely to vote for a political party that focuses on their problems.', 'In preceding elections the Land Party made no attempt to address the interests of economically distressed urban groups.', 'The greater the degree of economic distress someone is in, the more likely that person is to vote.', 'No other major party in Banestria specifically addressed the issues of people who lived in semirural areas in 1935.']", "label": 1 }, { "id": "train_1767", "context": "Television host: While it' s true that the defendant presented a strong alibi and considerable exculpatory evidence and was quickly acquitted by the jury, I still believe that there must be good reason to think that the defendant is not completely innocent in the case. Otherwise, the prosecutor would not have brought charges in the first place.", "question": "The reasoning in the television host's argument is flawed in that the argument", "answers": "['presupposes as evidence the conclusion that it is trying to establish', 'confuses legal standards for guilt with moral standards for guilt', 'places undue reliance on the judgments of an authority figure', 'takes lack of evidence for a view as grounds for concluding that the view is false']", "label": 2 }, { "id": "train_1768", "context": "Most opera singers who add demanding roles to their repertoires at a young age lose their voices early. It has been said that this is because their voices have not yet matured and hence lack the power for such roles. But young singers with great vocal power are the most likely to ruin their voices. The real problem is that most young singers lack the technical training necessary to avoid straining their vocal cords -- especially when using their full vocal strength. Such misuse of the cords inevitably leads to a truncated singing career.", "question": "Which one of the following does the information above most strongly support?", "answers": "['Young opera singers without great vocal power are unlikely to ruin their voices by singing demanding roles.', 'Most young opera singers who sing demanding roles strain their vocal cords.', 'Only mature opera singers can sing demanding roles without undue strain on their vocal cords.', 'Some young opera singers ruin their voices while singing demanding roles because their vocal cords have not yet matured.']", "label": 1 }, { "id": "train_1769", "context": "A company that manufactures plastic products from recyclable plastic is, surprisingly, unconcerned that economic conditions may worsen, despite analysts' belief that consumers would then consider ecofriendly plastic products an expensive luxury. But the company reasons that it will be able to lower its prices because, in a weakenedeconomy, other ecofriendly plastic manufacturers are likely to fail. Demand among manufacturers for recyclable plastics as raw materials would then plummet, creating an oversupply of such materials, making them less expensive for the manufacturer to purchase and thus lowering the company' s costs.", "question": "Which of the following, if true, most weakens the company's reasoning?", "answers": "['Retailers, including retailers that cater to consumers seeking certain types of ecofriendly products, may lose some business if economic conditions worsen.', 'Smaller ecofriendly plastic manufacturers are more likely to fail in a weakened economy than larger ecofriendly manufacturers are.', 'The plastics used by the company in its products were, after a recent investigation by a regulatory body, declared to be safe for consumers.', \"Consumers would likely soon learn of the oversupply of recyclable plastics and cease recycling them, significantly raising manufacturers' raw-material costs.\"]", "label": 3 }, { "id": "train_1770", "context": "Surveys consistently show that the best-selling ice cream flavor is vanilla, although those who prefer chocolate rarely order vanilla. Vanilla-flavored candy, then, probably sells better than chocolate-flavored candy.", "question": "Which of the following is an assumption upon which the author of the argument relies?", "answers": "['Preferences for certain ice cream flavors are similar to preferences for candy flavors.', 'Because someone prefers vanilla ice cream does not mean he prefers vanilla-flavored candy.', 'Someone who prefers vanilla ice cream may still order chocolate on occasion.', 'Those who prefer neither vanilla nor chocolate ice cream also prefer other flavors of candy.']", "label": 0 }, { "id": "train_1771", "context": "Producer: It has been argued that, while the government should not censor television shows, the public should boycott the advertisers of shows that promote violence and erode our country' s values. But this would be censorship nonetheless, for if the public boycotted the advertisers, then they would cancel their advertisements, causing some shows to go off the air; the result would be a restriction of the shows that the public can watch.", "question": "The producer's conclusion is properly inferred if which one of the following is assumed?", "answers": "['If there is neither government censorship nor boycotting of advertisers, there will be no restriction of the television shows that the public can watch.', 'Public boycotts could force some shows off the air even though the shows neither promote violence nor erode values.', 'For any television show that promotes violence and erodes values, there will be an audience.', 'Any action that leads to a restriction of what the public can view is censorship.']", "label": 3 }, { "id": "train_1772", "context": "Community organizer: Before last year' s community cleanup, only 77 of the local residents signed up to participate, but then well over 100 actually participated. This year, 85 residents have signed up to participate. Since our community cleanup will be a success if we have at least 100 participants, we can be confident that this year' s cleanup will be a success.", "question": "The reasoning in the community organizer's argument is most vulnerable to criticism on the grounds that the argument", "answers": "['defines a term in such a way as to ensure that whatever the outcome, it will be considered a positive outcome', 'confuses a condition that is required for an outcome with one that is sufficient for that outcome', \"takes for granted that people who participated in last year's cleanup will participate this year\", 'generalizes about the outcome of an event based on a single observation of a similar situation']", "label": 3 }, { "id": "train_1773", "context": "Fossil-fuel producers say that it would be prohibitively expensive to reduce levels of carbon dioxide emitted by the use of fossil fuels enough to halt global warming. This claim is probably false. Several years ago, the chemical industry said that finding an economical alternative to the chlorofluorocarbons (CFCs) destroying the ozone layer would be impossible. Yet once the industry was forced, by international agreements, to find substitutes for CFCs, it managed to phase them out completely well before the mandated deadline, in many cases at a profit.", "question": "Which one of the following, if true, most strengthens the argument?", "answers": "['There are ways of reducing carbon dioxide emissions that could halt global warming without hurting profits of fossil-fuel producers significantly more than phasing out CFCs hurt those of the chemical industry.', 'The use of CFCs never contributed as greatly to the destruction of the ozone layer as the carbon dioxide emitted by the use of fossil fuels currently contributes to global warming.', 'In some countries, the amount of carbon dioxide emitted by the use of fossil fuels has already been reduced without prohibitive expense, but at some cost in convenience to the users of such fuels.', 'If international agreements forced fossil-fuel producers to find ways to reduce carbon dioxide emissions enough to halt global warming, the fossil-fuel producers could find substitutes for fossil fuels.']", "label": 0 }, { "id": "train_1774", "context": "Claude: Because of the relatively high number of middle-aged people in the workforce, there will be fewer opportunities for promotion into upper-management positions. Since this will decrease people' s incentive to work hard, economic productivity and the quality of life will diminish. Thelma: This glut of middle-aged workers will lead many people to form their own companies. They will work hard and thus increase economic productivity, improving the quality of life even if many of the companies ultimately fail.", "question": "On the basis of their statements, Claude and Thelma are committed to agreeing about which one of the following?", "answers": "[\"How hard a company's employees work is a function of what they think their chances for promotion are in that company.\", 'The failure of many companies will not necessarily have a negative effect on overall economic productivity.', \"The quality of life in a society affects that society's economic productivity.\", 'Economic productivity will be affected by the number of middle-aged people in the workforce.']", "label": 3 }, { "id": "train_1775", "context": "Editor: Most of the books of fiction we have published were submitted by literary agents for writers they represented; the rest were received directly from fiction writers from whom we requested submissions. No nonfiction manuscript has been given serious attention, let alone been published, unless it was from a renowned figure or we had requested the manuscript after careful review of the writer' s book proposal.", "question": "Which one of the following can be properly inferred from the editor's statements?", "answers": "['Most of the books that the publishing house publishes that are not by renowned authors are books of fiction.', \"The publishing house is less likely to give careful consideration to a manuscript that was submitted directly by a writer than one that was submitted by a writer's literary agent.\", 'Any unrequested manuscripts not submitted by literary agents that the publishing house has published were written by renowned figures.', 'Most unrequested manuscripts that the publishing house receives are not given serious attention.']", "label": 2 }, { "id": "train_1776", "context": "Bankruptcy is a process that depends on a variety of structural, fiscal, and human variables. These variables are different at every company. Ttherefore, the pattern of companies declaring bankruptcy should be random. Yet tax records from 2010 demonstrate a pattern: a large number of companies throughout the United States declared bankruptcy at the same time.", "question": "Which of the following, if true, forms the best basis for at least a partial explanation for the pattern of bankruptcy shown by the tax records?", "answers": "['Certain financial problems affect only some types of businesses with particular sets of characteristics unique to their industry.', 'Many companies go bankrupt because the economies of the states in which they are located force them to go into gradual but increasing debt.', 'Patterns of bankruptcy emerge when widespread economic issues affect numerous companies.', 'Companies without franchises in more than one country are more likely to declare bankruptcy.']", "label": 2 }, { "id": "train_1777", "context": "Although Ackerburg's subway system is currently operating at a deficit, the transit authority will lower subway fares next year. The authority projects that the lower fares will result in a ten percent increase in the number of subway riders. Since the additional income from the larger ridership will more than offset the decrease due to lower fares, the transit authority actually expects the fare reduction to reduce or eliminate the subway system's operating deficit for next year.", "question": "Which of the following, if true, provides the most support for the transit authority's expectation of reducing the subway system's operating deficit?", "answers": "['The subway system can accommodate a ten percent increase in ridership without increasing the number of trains it runs each day.', 'Next year, the transit authority will have to undertake several large-scale track maintenance projects.', 'The current subway fares in Ackerburg are higher than subway fares in other cities in the region.', 'Throughout the years that the subway system has operated, fares have never before been reduced.']", "label": 0 }, { "id": "train_1778", "context": "The notion that one might be justified in behaving irrationally in the service of a sufficiently worthy end is incoherent. For if such an action is justified, then one would be behaving rationally, not irrationally.", "question": "Which one of the following arguments is most similar in its reasoning to the argument above?", "answers": "[\"One cannot live the good life and be unhappy. If one's own neighbors see that one is unhappy, then they will see that one is not living the good life.\", 'A representative of the law, such as a judge or a police officer, ought not to commit crimes. For if representatives of the law commit crimes, they will be ineffective in preventing crime.', \"One ought not to have both a cat and a goldfish. The goldfish is the natural prey of the cat, so it is unethical to place it at the cat's disposal.\", 'One cannot intend to spill a glass of water accidentally. Spilling it accidentally means that the act will not have been done intentionally.']", "label": 3 }, { "id": "train_1779", "context": "Linguist: Most people can tell whether a sequence of words in their own dialect is grammatical. Yet few people who can do so are able to specify the relevant grammatical rules.", "question": "Which one of the following best illustrates the principle underlying the linguist's statements?", "answers": "['Some people are able to tell whether any given piece of music is a waltz. But the majority of these people cannot state the defining characteristics of a waltz.', 'Quite a few people know the rules of chess, but only a small number of them can play chess very well.', 'Engineers who apply the principles of physics to design buildings and bridges must know a great deal more than do the physicists who discover these principles.', 'Some people are able to write cogent and accurate narrative descriptions of events. But these people are not necessarily also capable of composing emotionally moving and satisfying poems.']", "label": 0 }, { "id": "train_1780", "context": "Columnist: Even if the primary purpose of university education is to make students employable, such education should emphasize the liberal arts rather than the more narrow kind of technical training that prepares one for a particular sort of job. This is because the reasoning skills one acquires from a liberal arts education allow one to adapt to new intellectual challenges and thus to perform jobs for which one has received no specialized training.", "question": "Which one of the following, if true, most strengthens the columnist's argument?", "answers": "['Having a general understanding of life is more important than possessing practical skills.', 'It is better for people to have good educations than good jobs.', 'Technical training does not help students acquire reasoning skills.', 'Having a series of different jobs is more interesting than having only one job.']", "label": 2 }, { "id": "train_1781", "context": "The common procedure for determining whether a food additive should be banned from use is to compare its health-related benefits with its potential risks. Yellow Dye No. 5, an additive used to color lemon soda, might cause allergic reactions in a few consumers. For most consumers of lemon soda, however, the coloring enhances their enjoyment of the beverage. This particular additive should not be banned, ttherefore, because its benefits greatly outweigh its risks.", "question": "A flaw in the argument is that the author", "answers": "['ignores the possibility that some food additives are harmful to most people', 'implies that the dye entails no health-related risks', 'presumes that most consumers heed the warning labels on beverage containers', 'treats enjoyment of a beverage as a health-related benefit']", "label": 3 }, { "id": "train_1782", "context": "Genevieve: Increasing costs have led commercial airlines to cut back on airplane maintenance. Also, reductions in public spending have led to air traffic control centers being underfunded and understaffed. For these and other reasons it is becoming quite unsafe to fly, and so one should avoid doing it. Harold: Your reasoning may be sound, but I can hardly accept your conclusion when you yourself have recently been flying on commercial airlines even more than before.", "question": "Which one of the following relies on a questionable technique most similar to that used in Harold's reply to Genevieve?", "answers": [ "Susan has been trying to persuade me to go rock climbing with her, claiming that it's quite safe, but last week she fell and broke her collarbone, so I don't believe her.", "David says that the new film is not very good, but he has not seen it himself, so I don't accept his opinion.", "Pat has shown me research that proves that eating raw green vegetables is very beneficial and that one should eat them daily, but I don't believe it, since she hardly ever eats raw green vegetables.", "Gabriel has all the qualifications we have specified for the job and has much relevant work experience, but I don't believe we should hire him, because when he worked in a similar position before his performance was mediocre." ], "label": 2 }, { "id": "train_1783", "context": "Transportation expenses accounted for a large portion of the total dollar amount spent on trips for pleasure by residents of the United States in 1997, and about half of the total dollar amount spent on the transportation was airfare. However, the large majority of United States residents who took trips for pleasure in 1997 did not travel by airplane but used other means of transportation.", "question": "If the statements above are true, which of the following must also be true about United States residents who took trips for pleasure in 1997?", "answers": "['Most of those who traveled by airplane did so because the airfare to their destination was lower than the cost of other available means of transportation.', 'Per mile traveled, those who traveled by airplane tended to spend more on transportation to their destination than did those who used other means of transportation.', 'Most of those who traveled by airplane did so because other means of transportation to their destinations were unavailable.', 'Overall, people who did not travel by airplane had lower average transportation expenses than people who did.']", "label": 3 }, { "id": "train_1784", "context": "Geologists recently discovered marks that closely resemble worm tracks in a piece of sandstone. These marks were made more than half a billion years earlier than the earliest known traces of multicellular animal life. Ttherefore, the marks are probably the traces of geological processes rather than of worms.", "question": "Which one of the following, if true, most weakens the argument?", "answers": "['At the place where the sandstone was found, the only geological processes that are likely to mark sandstone in ways that resemble worm tracks could not have occurred at the time the marks were made.', 'There were some early life forms other than worms that are known to have left marks that are hard to distinguish from those found in the piece of sandstone.', 'Most scientists knowledgeable about early animal life believe that worms are likely to have been among the earliest forms of multicellular animal life on Earth, but evidence of their earliest existence is scarce because they are composed solely of soft tissue.', 'It is sometimes difficult to estimate the precise age of a piece of sandstone.']", "label": 0 }, { "id": "train_1785", "context": "Economist: Countries with an uneducated population are destined to be weak economically and politically, whereas those with an educated population have governments that display a serious financial commitment to public education. So any nation with a government that has made such a commitment will avoid economic and political weakness.", "question": "The pattern of flawed reasoning in which one of the following arguments is most similar to that in the economist's argument?", "answers": "['Animal species with a very narrow diet will have more difficulty surviving if the climate suddenly changes, but a species with a broader diet will not; for changes in the climate can remove the traditional food supply.', 'People who dislike exercise are unlikely to lose weight without sharply curtailing their food intake; but since those who dislike activity generally tend to avoid it, people who like to eat but dislike exercise will probably fail to lose weight.', 'People incapable of empathy are not good candidates for public office, but those who do have the capacity for empathy are able to manipulate others easily; hence, people who can manipulate others are good candidates for public office.', 'Poets who create poetry of high quality are those who have studied traditional poetry, because poets who have not studied traditional poetry are the poets most likely to create something shockingly inventive, and poetry that is shockingly inventive is rarely fine poetry.']", "label": 2 }, { "id": "train_1786", "context": "Some flowering plant species, entirely dependent on bees for pollination, lure their pollinators with abundant nectar and pollen, which are the only source of food for bees. Often the pollinating species is so highly adapted that it can feed from -- and thus pollinate -- only a single species of plant. Similarly, some plant species have evolved flowers that only a single species of bee can pollinate -- an arrangement that places the plant species at great risk of extinction. If careless applications of pesticides destroy the pollinating bee species, the plant species itself can no longer reproduce.", "question": "The information above, if true, most strongly supports which one of the following?", "answers": "['Some bees are able to gather pollen and nectar from any species of plant.', 'The blossoms of most species of flowering plants attract some species of bees and do not attract others.', 'The total destruction of the habitat of some plant species could cause some bee species to become extinct.', 'If the sole pollinator of a certain plant species is in no danger of extinction, the plant species it pollinates is also unlikely to become extinct.']", "label": 2 }, { "id": "train_1787", "context": "Naima: The proposed new computer system, once we fully implemented it, would operate more smoothly and efficiently than the current system. So we should devote the resources necessary to accomplish the conversion as soon as possible. Nakai: We should keep the current system for as long as we can. The cost in time and money of converting to the new system would be greater than any predicted benefits.", "question": "Naima and Nakai disagree with each other over whether", "answers": "['the current computer system does not work well enough to do what it is supposed to do', 'the conversion to a new computer system should be delayed', 'it is essential to have the best computer system available', 'the predicted benefits of the new computer system will be realized']", "label": 1 }, { "id": "train_1788", "context": "Robin: When a region' s economy is faltering, many people lose their jobs. As a result, spending on consumer goods declines, leading in turn to more lost jobs and a worsening of the economy. Eventually, the economy becomes so bad that prices collapse; the lower prices encourage people to increase spending on consumer goods, and this higher spending results in economic improvement. Terry: People cannot increase their spending if they have no jobs and no money for anything other than basic necessities, so price collapses cannot lead to economic improvement.", "question": "Which one of the following, if true, most undermines Terry's objection to Robin's analysis?", "answers": "['Even people who do not lose their jobs spend less in bad economic times and thus have savings available to spend when prices collapse.', 'People who have lost their jobs must continue to buy some basic goods such as food, even during bad economic times.', 'The prices of some consumer goods remain stable, even during a general price collapse.', 'Companies hire more workers after the economy starts to improve again, and many newly hired workers then make long-deferred purchases.']", "label": 0 }, { "id": "train_1789", "context": "Eating turmeric, a spice commonly found in curries, probably helps prevent Alzheimer' s disease. More turmeric is consumed per capita in India than in the rest of the world, and the incidence of Alzheimer' s disease is much lower there than it is worldwide. Furthermore, Alzheimer' s disease is characterized by the buildup of amyloid protein plaques in the brain, and studies on animals found that curcumin -- a compound found in turmeric -- reduces the accumulation of amyloid proteins. Which one of the following.", "question": "if true, most strengthens the argument?", "answers": "[\"The parts of India that have the highest per capita rates of cuny consumption have the lowest incidence of Alzheimer's disease.\", 'None of the other compounds found in turmeric have been studied to see whether they affect the accumulation of amyloid proteins.', \"The proportion of people living in India who fall within the age group that is most prone to developing Alzheimer's disease is smaller than the proportion of people worldwide who fall within that age group.\", \"Many scientists believe that the buildup of amyloid protein plaques in the brain is a symptom of Alzheimer's disease rather than a cause.\"]", "label": 0 }, { "id": "train_1790", "context": "Criticism that the press panders to public sentiment neglects to consider that the press is a profit-making institution. Like other private enterprises, it has to make money to survive. If the press were not profit-making, who would support it? The only alternative is subsidy and, with it, outside control. It is easy to get subsidies for propaganda, but no one will subsidize honest journalism.", "question": "It can be properly inferred from the passage that if the press is", "answers": "['not to be subsidized, it cannot be a profit-making institution', 'to produce honest journalism, it must be a profit-making institution', 'not subsidized, it will not produce propaganda', 'to make a profit, it must produce honest journalism']", "label": 1 }, { "id": "train_1791", "context": "Under a new clean air proposal, the government has decided to tighten controls on the release of certain toxic chemicals, including benzene, formaldehyde, and other carcinogens, by chemical plants. The stated purpose of this proposal is to reduce cancers caused by air pollution. Yet, the chemical industry, rather than the government, is responsible for monitoring the implementation of the proposal. If the past actions of certain polluters in the chemical industry are any indication of future behavior, the net result of the new proposal will be an increase, rather than a decrease, in carcinogens released into the air.", "question": "The author is arguing that __.", "answers": "['no chemical companies can be trusted to follow the clean air proposal', 'to ensure effective implementation, the government should always monitor the execution of its proposals', 'allowing self-monitoring for the new clean air proposal will result in the opposite of its intended consequence', 'benzene and formaldehyde are two of the most hazardous cancer-causing chemicals']", "label": 2 }, { "id": "train_1792", "context": "A salesperson who makes a sale does not change the desires of the customer. Rather, the salesperson finds out what these desires are and then convinces the customer that a particular product will satisfy them. Persuading people to vote for a politician to whom they are initially indifferent is not significantly different. After discovering what policies the prospective voter would like to see in place, one tries to__.", "question": "Which one of the following most logically completes the argument?", "answers": "['convince the voter that the policies favored by the politician in question are preferable to those favored by the voter', 'persuade the voter that voting for the politician in question is the best way to get these policies adopted', 'disguise any difference between the policies the politician supports and the policies supported by other candidates', 'show that the opponents of the politician in question do not favor all of those policies']", "label": 1 }, { "id": "train_1793", "context": "As a construction material, bamboo is as strong as steel and sturdier than concrete. Moreover, in tropical areas bamboo is a much less expensive construction material than either steel or concrete and is always readily available. In tropical areas, ttherefore, building with bamboo makes better economic sense than building with steel or concrete, except where land values are high.", "question": "Which of the following, if true, most helps to explain exception noted above?", "answers": "['Bamboo is unsuitable as a building material for multistory buildings.', 'In order to protect it from being damaged by termites and beetles, bamboo must be soaked, at some expense, in a preservative.', 'Bamboo growing in an area where land values are increasing is often cleared to make way for construction.', 'Buildings constructed of bamboo are less likely to suffer earthquake damage than are steel and concrete buildings.']", "label": 0 }, { "id": "train_1794", "context": "Several people came down with an illness caused by a type of bacteria in seafood. Health officials traced the history of each person who became ill to the same restaurant and date. Careful testing showed that most people who ate seafood at the restaurant on that date had not come in contact with the bacteria in question. Despite this finding, health officials remained confident that contaminated seafood from this restaurant caused the cases of illness.", "question": "Which one of the following, if true, most helps to resolve the apparent discrepancy indicated above?", "answers": "['All and only those who ate contaminated seafood at the restaurant on that date were allergic to the monosodium glutamate in a sauce that they used.', 'Those made ill by the bacteria had all been served by a waiter who subsequently became ill.', 'All and only those who ate a particular seafood dish at the restaurant contracted the illness.', 'The restaurant in question had recently been given a warning about violations of health regulations.']", "label": 2 }, { "id": "train_1795", "context": "Technology is radically improving the quality of life in some communities, and not only by direct application of innovations. After all, the design, production, testing, and marketing of new technology has itself become a growing industry that is turning around the fortunes of once-ailing communities. The companies involved create jobs, add to the tax base, and contribute to an upbeat spirit of renewal.", "question": "Which one of the following most accurately expresses the conclusion drawn by the argument as a whole?", "answers": "['The direct application of innovations is not the only way in which technology is radically improving the quality of life in some communities.', 'The only ways in which technology is radically improving the quality of life in some communities are by creating jobs, adding to the tax base, and contributing to an upbeat spirit of renewal.', 'Companies involved in the design, production, testing, and marketing of new technology create jobs, add to the tax base, and contribute to an upbeat spirit of renewal.', 'Either the creation or the direct application of technological innovations is radically improving the quality of life in most communities.']", "label": 0 }, { "id": "train_1796", "context": "Individual pyrrole molecules readily join together into larger molecules called polypyrroles. If polypyrroles form from pyrrole in the presence of zeolites, they do so by attaching to the zeolite either in lumps on the outer surface of the zeolite or in delicate chains within the zeolite' s inner channels. When zeolite changes color from yellow to black, it means that on or in that zeolite polypyrroles have formed from pyrrole. Yellow zeolite free of any pyrrole was submerged in dissolved pyrrole. The zeolite turned black even though no polypyrroles formed on its outer surface.", "question": "If the statements above are true, which one of the following must on the basis of them be true?", "answers": "[\"Little, if any, of the pyrrole in which the zeolite was submerged reached the zeolite's inner channels.\", 'None of the pyrrole in which the zeolite was submerged attached itself to the zeolite.', 'Lumps of polypyrrole attached to the zeolite were responsible for its color change.', 'At least some of the pyrrole in which the zeolite was submerged formed polypyrrole chains.']", "label": 3 }, { "id": "train_1797", "context": "Once a child' s imagination becomes developed, a host of imaginary creatures may torment the child. But this newly developed cognitive capacity may also be used to render these creatures harmless. For instance, a child' s new toy may be imagined as an ally, powerful enough to ward off any imaginary threats.", "question": "The type of situation described above most closely conforms to which one of the following propositions?", "answers": "['Some newly developed capacities only give rise to problems.', 'The most effective way for children to address their fears is to acknowledge them.', 'Most problems associated with child-rearing can be solved with a little imagination.', 'Sometimes the cause of a problem may also provide its solution.']", "label": 3 }, { "id": "train_1798", "context": "Members of large-animal species must consume enormous amounts of food to survive. When climatic conditions in their environment deteriorate, such animals are often unable to find enough food. This fact helps make large-animal species more vulnerable to extinction than small-animal species, which can maintain greater populations on smaller amounts of food.", "question": "The statements above, if true, most support which one of the following?", "answers": "['When conditions deteriorate in a given environment, no small-animal species will become extinct unless some large-animal species also becomes extinct.', 'The vulnerability of an animal species to extinction depends at least in part on how much food individuals of that species must consume to survive.', 'Whenever climatic conditions in a given environment are bad enough to threaten large-animal species with extinction, small-animal species are able to find enough food to survive.', 'The maximum population size that an animal species could maintain on any given amount of food is the main factor determining whether that species will become extinct.']", "label": 1 }, { "id": "train_1799", "context": "In the aftermath of the Cold War, international relations between Cold War allies became more difficult. Leaders of previously allied nations were required to conduct tactful economic negotiations in order not to arouse tensions that had previously been overlooked.", "question": "The situation described above conforms most closely to which one of the following propositions?", "answers": "['International economic competition is a greater cause of tension than is international military competition.', 'Bonds between allies are stronger when they derive from fear of a common enemy than when they derive from common economic goals.', 'Economic matters are considered unimportant during periods of actual or threatened war.', 'A common enemy contributes to a strengthened bond between nations, enabling them to ignore economic tensions that would otherwise be problematic.']", "label": 3 }, { "id": "train_1800", "context": "Those who participate in local politics include people who are genuinely interested in public service and people who are selfish opportunists. Everyone who participates in local politics has an influence on the community' s values.", "question": "If the statements above are true, which one of the following must also be true?", "answers": [ "Some persons who are interested in public service do not have an influence on the community's values.", "All those who have an influence on the community's values are either interested in public service or are selfish opportunists.", "Some selfish opportunists have an influence on the community's values.", "Some of those who influence the community's values neither are interested in public service nor are selfish opportunists." ], "label": 2 }, { "id": "train_1801", "context": "Certainly, pesticides can adversely affect the environment in localities distant from where the pesticide has actually been used. Nevertheless, regulation of pesticide use should not take place at the national level but at the local level. It is in the areas where pesticides are actually applied that they have their most serious effects. Just how serious these effects are depends on local conditions such as climate, soil type, and water supply. And local officials are much more likely than national legislators to be truly knowledgeable about such local conditions.", "question": "In the argument given, the two boldface portions play which of the following roles?", "answers": "['Each provides support for the conclusion of the argument.', 'The first identifies grounds for a potential objection to the conclusion of the argument; the second provides support for that conclusion.', 'The first provides support for the conclusion of the argument; the second states that conclusion.', 'The first identifies grounds for a potential objection to the conclusion of the argument; the second states that conclusion.']", "label": 1 }, { "id": "train_1802", "context": "According to a 1990 law, Native American tribes are entitled to artifacts or remains removed from their lands, provided that the tribes can show a direct archaeological, geological, or historical link to the artifacts. Recently, the Umatilla tribes of Oregon and Washington laid claim to a set of skeletal remains more than 500 years old.", "question": "Each of the following, if true, strengthens the Umatilla's claim to the remains EXCEPT:", "answers": "['The remains were found in close proximity to an excavation of an ancient Umatilla settlement.', 'The Umatilla claim that the remains of all native peoples deserve the respect of a traditional reburial ceremony.', 'The remains were buried with artifacts similar to those found with other Umatilla remains.', 'DNA evidence from the remains shows a direct link to current members of the Umatilla.']", "label": 1 }, { "id": "train_1803", "context": "Since 1990 the percentage of bacterial sinus infections in Aqadestan that are resistant to the antibiotic perxicillin has increased substantially. Bacteria can quickly develop resistance to an antibiotic when it is prescribed indiscriminately or when patients fail to take it as prescribed. Since perxicillin has not been indiscriminately prescribed, health officials hypothesize that the increase in perxicillin-resistant sinus infections is largely due to patients' failure to take this medication as prescribed.", "question": "Which of the following, if true of Aqadestan, provides most support for the health officials' hypothesis?", "answers": "['Aqadestani health clinics provide antibiotics to their patients at cost.', 'A large number of Aqadestanis never seek medical help when they have a sinus infection.', 'When it first became available, perxicillin was much more effective in treating bacterial sinus infections than any other antibiotic used for such infections at the time.', 'Many patients who take perxicillin experience severe side effects within the first few days of their prescribed regimen.']", "label": 3 }, { "id": "train_1804", "context": "Businesses frequently use customer surveys in an attempt to improve sales and increase profits. However, a recent study of the effectiveness of these surveys found that among a group of businesses that sold similar products, profits declined in most of the businesses that used surveys during the course of the study but not in most of the businesses that did not use any surveys during the course of the study.", "question": "Which one of the following, if true, most helps to explain why the profits of businesses that did not use customer surveys did not decline while the profits of those that used surveys did decline?", "answers": "['When one business increases its profits, its competitors often report a decline in profits.', 'Some of the businesses included in the study did not analyze the results of the customer surveys they conducted.', 'Most businesses of the kind included in the study generally administer customer surveys only as a response to complaints by customers.', 'Customers who complete surveys do not always respond accurately to all the questions on the survey.']", "label": 2 }, { "id": "train_1805", "context": "Psychologist: Although studies of young children have revealed important facts about the influence of the environment on language acquisition, it is clear that one cannot attribute such acquisition solely to environmental influences: innate mechanisms also play a role. So, the most reasonable question that ought to be studied is whether__.", "question": "Which one of the following most logically completes the passage?", "answers": "[\"parents and peers are the most important influence on a child's learning of a language\", 'language acquisition can ever be fully explained', \"innate mechanisms play a more important role in language acquisition than a child's immediate environment\", 'innate mechanisms are a contributing factor in language learning']", "label": 2 }, { "id": "train_1806", "context": "Raising the humidity of a room protects furniture, draperies, and computers from damage caused by excessively dry air. Further, it can make people feel warmer, helps the body' s defenses against viruses, and alleviates some skin rashes.", "question": "Each of the following is supported by the information above EXCEPT:", "answers": "['Dry air can feel cooler than humid air of the same temperature.', 'Increased humidity can be beneficial to the skin.', 'Humidity can be bad for computers.', 'The human immune system can benefit from humidity.']", "label": 2 }, { "id": "train_1807", "context": "There is strong evidence that the cause of migraines (severe recurrent headaches) is not psychological but instead is purely physiological. Yet several studies have found that people being professionally treated for migraines rate higher on a standard psychological scale of anxiety than do people not being professionally treated for migraines.", "question": "Which one of the following, if true, most helps to resolve the apparent discrepancy in the information above?", "answers": "['Most people who have migraines and who seek professional treatment remain in treatment until they stop having migraines, whether their doctors consider the cause to be physiological or psychological.', 'People who have migraine headaches tend to have relatives who also have migraine headaches.', 'People who have migraine headaches often suffer these headaches when under emotional stress.', 'People who rate higher on the standard psychological scale of anxiety are more likely to seek professional treatment than are people who rate lower on the scale.']", "label": 3 }, { "id": "train_1808", "context": "A sports arena is considering a new stadium design, one in which each block of seats would be in an enclosed weather-proof room with a view of the playing field and a large screen TV showing the action of play, including the instant replays typical of television broadcasts of sports games. The developer reasons that exposure to elements and lack of access to instant replays significantly limits the typical spectator' s enjoyment of the live game.", "question": "This stadium design would be unlikely to increase the pleasure the spectators take from live games if they enjoyed", "answers": "['watching the game without interruptions from other spectators', 'attending evening games more than midday games.', 'coming to multiple games in a single season', 'feeling the immediate camaraderie with the crowd of other spectators at the game']", "label": 3 }, { "id": "train_1809", "context": "In order to improve the long-term savings rate of its citizens, Levaska' s government has decided to introduce special savings accounts. Citizens can save up to $3, 000 a year in special accounts without having to pay tax on the interest, unless they withdraw money from the account before they reach the age of sixty-five. If they do withdraw any money before that age, they have to pay tax on the accumulated interest and a penalty.", "question": "Which of the following, if true, most seriously threatens the success of the government's plan?", "answers": "['For the past ten years, Levaskans have been depositing an ever smaller percentage of their income in long-term savings.', 'In certain circumstances, such as a serious illness, the government plans to waive the penalty on early withdrawals from the special accounts.', 'The banks and financial institutions where the special accounts will be held lobbied hard for their introduction.', 'Nearly all workers in Levaska can already save money in tax-free accounts through their workplace.']", "label": 3 }, { "id": "train_1810", "context": "Although the charter of Westside School states that the student body must include some students with special educational needs, no students with learning disabilities have yet enrolled in the school. Ttherefore, the school is currently in violation of its charter.", "question": "The conclusion of the argument follows logically if which one of the following is assumed?", "answers": "['All students with learning disabilities have special educational needs.', 'The school should enroll students with special educational needs.', 'The only students with special educational needs are students with learning disabilities.', 'The school currently has no student with learning disabilities.']", "label": 2 }, { "id": "train_1811", "context": "No one knows what purposes, if any, dreams serve, although there are a number of hypotheses. According to one hypothesis, dreams are produced when the brain is erasing \"parasitic connections\" (meaningless, accidental associations between ideas), which accumulate during the day and which would otherwise clog up our memories. Interestingly, the only mammal that does not have rapid eye movement sleep, in which we humans typically have our most vivid dreams, is the spiny anteater, which has been seen as anomalous in that it has a very large brain relative to the animal' s size. This fact provides some confirmation for the parasitic-connection hypothesis, since the hypothesis predicts that for an animal that did not dream to have an effective memory that animal would need extra memory space for the parasitic connections.", "question": "The parasitic-connection hypothesis, if true, most strongly supports which one of the following?", "answers": "[\"Insofar as a person's description of a dream involves meaningful associations between ideas, it is an inaccurate description.\", 'The animals with the smallest brains spend the most time sleeping.', 'When a mammal that would normally dream is prevented from dreaming, the functioning of its memory will be impaired.', 'All animals other than the spiny anteater dream.']", "label": 2 }, { "id": "train_1812", "context": "Jordan: If a business invests the money necessary to implement ecologically sound practices, its market share will decrease. But if it doesn' t implement these practices, it pollutes the environment and wastes resources. Terry: But if consumers demand environmental responsibility of all businesses, no particular business will be especially hurt.", "question": "In which one of the following exchanges is the logical relationship between Jordan's and Terry's statements most similar to the logical relationship between their statements above?", "answers": [ "Jordan: If the dam's spillway is opened, the river might flood the eastern part of town, but if the spillway is not opened, the dam might burst. Terry: There's no real danger of the dam's bursting, but if we get more heavy rain, opening the spillway is the most prudent policy.", "Jordan: If we remodel the kitchen, the house will be more valuable, but even if we do, there's no guarantee that we'll actually get more for the house when we sell it. Terry: But if we don't remodel the kitchen, we might get even less for the house than we paid for it.", "Jordan: Each person can have either an enjoyable life or a long life, for one must eat vegetables and exercise continuously to stay healthy. Terry: That's not true:there are many happy health-conscious people.", "Jordan: Either it will rain and our plans for a picnic will be thwarted or it won't rain and the garden will go yet another day without much-needed watering. Terry: But if it doesn't rain, we can buy a hose and water the garden with the hose." ], "label": 3 }, { "id": "train_1813", "context": "Printwell' s Ink Jet Division manufactures ink-jet printers and the ink cartridges they use. Sales of its ink-jet printers have increased. Monthly revenues from those sales, however, have not increased, because competition has forced Printwell to cut the prices of its printers. Unfortunately, Printwell has been unable to bring down the cost of manufacturing a printer. Thus, despite the increase in printer sales, the Ink Jet Division must be contributing less to the company' s profits than it used to.", "question": "Which of the following, if true, most seriously weakens the argument?", "answers": "['Some printer manufacturers have been forced to reduce the sale price of their ink-jet printers even more than Printwell has.', 'In the past year, no competing manufacturer of ink-jet printers has had as great an increase in unit sales of printers as Printwell has.', \"Unlike some competing companies, Printwell sells all of its printers through retailers, and these retailers' costs account for a sizable proportion of the printers' ultimate retail price.\", \"Ink-jet printers in regular use frequently need new ink cartridges, and Printwell's printers only accept Printwell's ink cartridges.\"]", "label": 3 }, { "id": "train_1814", "context": "To reduce productivity losses from employees calling in sick, Corporation X implemented a new policy requiring employees to come into work unless they were so sick that they had to go to a doctor. But a year after the policy was implemented, a study found that Corporation X's overall productivity losses due to reported employee illnesses had increased.", "question": "Which of the following, if true, would best explain why the policy produced the reverse of its intended effect?", "answers": "['Employees coming into work when sick often infect many of their coworkers.', 'After the policy was implemented, employees more frequently went to the doctor when they felt sick.', 'Before the policy was implemented, employees who were not sick at all often called in sick.', 'There are many other factors besides employee illness that can adversely affect productivity.']", "label": 0 }, { "id": "train_1815", "context": "McKinley: A double-blind study, in which neither the patient nor the primary researcher knows whether the patient is being given the drug being tested or a placebo, is the most effective procedure for testing the efficacy of a drug. But we will not be able to perform such a study on this new drug, since the drug will have various effects on the patients' bodies, which will make us aware of whether the patients are getting the drug or a placebo. Engle: You cannot draw that conclusion at this point, for you are assuming you know what the outcome of the study will be.", "question": "Engle's statement indicates that he is most likely interpreting McKinley's remarks to be", "answers": "[\"referring to the drug's therapeutic effects rather than to any known side effects\", 'presuming that a double-blind study is the only effective way to test new drugs', 'based on a confusion about when a drug is efficacious', \"presuming that the placebo will produce no effects whatever on the patients' bodies\"]", "label": 0 }, { "id": "train_1816", "context": "The government has proposed a plan requiring young people to perform services to correct various current social ills, especially those in education and housing. Government service, however, should be compelled only in response to a direct threat to the nation' s existence. For that reason, the proposed program should not be implemented.", "question": "Which one of the following is an assumption on which the argument depends?", "answers": "[\"Some of the social ills that currently afflict the nation do not pose a direct threat to the nation's existence.\", \"The nation's young people believe that current social ills pose no direct threat to the nation's existence.\", 'Government-required service by young people cannot correct all social ills.', \"Crises in education and housing constitute a threat to the nation's existence.\"]", "label": 0 }, { "id": "train_1817", "context": "High school students who feel that they are not succeeding in school often drop out before graduating and go to work. Last year, however, the city' s high school dropout rate was significantly lower than the previous year' s rate. This is encouraging evidence that the program instituted two years ago to improve the morale of high school students has begun to take effect to reduce dropouts.", "question": "Which one of the following, if true about the last year, most seriously weakens the argument?", "answers": "['The morale of students who dropped out of high school had been low even before they reached high school.', 'There was a recession that caused a high level of unemployment in the city.', \"The antidropout program was primarily aimed at improving students' morale in those high schools with the highest dropout rates.\", 'As in the preceding year, more high school students remained in school than dropped out.']", "label": 1 }, { "id": "train_1818", "context": "If, when the twenty-third century arrives, the history of the Mughal Empire is better known than that of our time, it will be because of our enthusiasm for electronically stored digital files. The contents of most digital media vanish long before words written on high-quality paper would, and they become obsolete and unusable even sooner due to rapid technological innovation. While information written on paper can be read directly, access to digital information is doubly indirect: the sequence of digits representing the information must be retrieved, and then that sequence must be decoded by the appropriate software.", "question": "Which one of the following statements most accurately expresses the main conclusion of the argument?", "answers": "['Information written on paper is more readily accessible than the contents of digital documents.', 'Digitally stored information is particularly vulnerable because of the two-step process required to retrieve it.', 'Historically important records from the present era may be lost because of the medium in which they are stored.', 'The obsolescence brought about by ongoing technological innovation will make historical research increasingly difficult in the future.']", "label": 2 }, { "id": "train_1819", "context": "Moral integrity in business is of the highest concern, for businesses that lose their integrity cannot survive. By definition, a business that has lost its integrity is no longer integrated; hence businesses that lose their integrity literally disintegrate.", "question": "The argument's reasoning is flawed because the argument", "answers": "['takes for granted that the survival of businesses is the only important ethical concern', 'confuses a cause of integrity with an effect of integrity', 'contains a key term that shifts in meaning from one sense to another', 'overlooks the possibility that integrity is not a public-relations or management goal of some businesses']", "label": 2 }, { "id": "train_1820", "context": "Some rich people cheat on their taxes, but no one pays zero taxes, except those who do not own land. Jacob is a rich landowner.", "question": "Assuming all of the statements above are correct, which one of the following must be true?", "answers": "['Jacob cheats on his taxes.', 'Jacob pays taxes.', 'Jacob pays more taxes than the average person.', 'Jacob does not pay taxes.']", "label": 1 }, { "id": "train_1821", "context": "Pit vipers are so called because of the small pits on the sides of their heads which function as infrared sensors, giving the viper a thermal impression of objects in its vicinity. Because all species of pit vipers are predators, scientists have long thought that these sensors were used to find prey. One researcher has hypothesized, however, that the infrared sensor serves primarily to assist the viper in judging the size of approaching predators and how far away they are.", "question": "Which one of the following, if true, would most support the researcher's hypothesis?", "answers": "['Pit vipers have a well-developed sense of smell that they use to supplement the thermal impressions they receive from their infrared sensors.', 'Pit vipers do not differ in their predatory behavior from closely related vipers without pits, but they do differ markedly from these other species in their strategies of defense against predators.', 'The rattle of the diamondback rattlesnake, one of the most common kinds of pit viper, functions as a defense mechanism to intimidate predators.', 'Pit vipers are distinguishable from other viper species not only by their pits but also by the chemical composition of their venom.']", "label": 1 }, { "id": "train_1822", "context": "One should not play a practical joke on someone if it shows contempt for that person or if one believes it might bring significant harm to that person.", "question": "The principle stated above, if valid, most helps to justify the reasoning in which one of the following arguments?", "answers": "['Because of the circumstances, it would be wrong for me to play the practical joke I had intended to play on you. Even though it would not show contempt for anyone, it could easily bring you significant harm.', 'I should not have played that practical joke on you yesterday. Even if it was not contemptuous, I should have realized that it would bring significant harm to someone.', 'It would have been wrong for me to play the practical joke that I had intended to play on you. Even though I did not have reason to think that it would significantly harm anyone, I did think that it would show contempt for someone.', 'Someone was harmed as a result of my practical joke. Thus, even though it did not show contempt for the person I played the joke on, I should not have played it.']", "label": 0 }, { "id": "train_1823", "context": "Mall owner: Our mall' s occupancy rate is so low that we are barely making a profit. We cannot raise rents because of an unacceptably high risk of losing established tenants. On the other hand, a mall that is fully occupied costs about as much to run as one in which a rental space here and a rental space there stands empty. Clearly, ttherefore, to increase profits we must sign up new tenants.", "question": "Which of the following, if true, most seriously weakens the argument?", "answers": "['The mall is located in a geographic area in which costs incurred for air-conditioning in the hot summers exceed those incurred for heating in the mid winters by a wide margin.', \"The mall's occupancy rate, though relatively low, has been relatively stable for several years.\", \"The mall's operating costs could be cut by consolidating currently rented spaces in such a way that an entire wing of the mall could be closed up.\", \"None of the mall's established tenants is likely to need additional floor space there in the foreseeable future.\"]", "label": 2 }, { "id": "train_1824", "context": "Vanwilligan: Some have argued that professional athletes receive unfairly high salaries. But in an unrestricted free market, such as the market these athletes compete in, salaries are determined by what someone else is willing to pay for their services. These athletes make enormous profits for their teams' owners, and that is why owners are willing to pay them extraordinary salaries. Thus the salaries they receive are fair.", "question": "Vanwilligan's conclusion follows logically if which one of the following is assumed?", "answers": "[\"If a professional athlete's salary is fair, then that salary is determined by what an individual is willing to pay for the athlete's services in an unrestricted free market.\", 'The fairest economic system for a society is one in which the values of most goods and services are determined by the unrestricted free market.', 'If professional athletes were paid less for their services, then the teams for which they play would not make as much money.', 'Any salary that a team owner is willing to pay for the services of a professional athlete is a fair salary.']", "label": 3 }, { "id": "train_1825", "context": "Computer manufacturers and retailers tell us that the complexity involved in connecting the various components of personal computers is not a widespread obstacle to their use, but this is wrong. Customers who install accessories to their personal computers have to take full responsibility for the setting of jumpers and switches to satisfy mysterious specifications. Many accessories require extra software that can cause other accessories to stop working; adding a modem, for instance, may disable a printer.", "question": "Which one of the following, if true, most seriously weakens the argument?", "answers": "['Computer manufacturers rarely take into account ease of installation when they are designing programs or accessories.', 'A personal computer is usually sold as part of a package that includes accessories and free installation.', 'Software for accessories can often be obtained for free.', 'Personal computer instruction manuals usually explain the purposes of the jumpers and switches.']", "label": 1 }, { "id": "train_1826", "context": "In the past decade, two Motorlux models, the T-300, a sports car, and the AG-75, a sports utility vehicle, have been manufactured with the same timing belt, which controls the timing of the engines valves: when the engine is running at a higher RPM, there is more strain on the timing belt. Recent evidence indicates that this particular brand of timing belt has a usable life of about 135, 000 miles, somewhat less the 200, 000 miles life typical of standard timing belt. Although the same model of timing belt has been used for both models of cars, the incidence of timing belt failure is considerably higher in the AG-75 than in the T-300. This is most likely because __", "question": "Which of the following, if true, most logically completes the argument?", "answers": "['it is common for sports utility vehicles to tow heavy objects, such as motor boats; whereas sport cars almost never tow anything.', 'the timing belts typically used in most other models of sports cars on the market are of higher quality than the timing belts typically used in most other models of sports utility vehicles on the market.', 'where legal, drivers of the T-300 have been known to drive it at speeds in excessive of 100 mph; whereas the AG-75 is rarely driven that fast.', 'timing belts are automatically replaced when the entire transmission systems is repaired or replaced, and, unlike sports utility vehicles, sport cars such as the T-300 tend to experience transmission systems problems before the 100, 000 mile mark.']", "label": 3 }, { "id": "train_1827", "context": "Columnist: Although much has been learned, we are still largely ignorant of the intricate interrelationships among species of living organisms. We should, ttherefore, try to preserve the maximum number of species if we have an interest in preserving any, since allowing species toward which we are indifferent to perish might undermine the viability of other species.", "question": "Which one of the following principles, if valid, most helps to justify the columnist's argument?", "answers": "['We should always undertake the course of action that is likely to have the best consequences in the immediate future.', 'It is strongly in our interest to preserve certain plant and animal species.', 'We should not allow the number of species to diminish any further than is necessary for the flourishing of present and future human populations.', 'We should not allow a change to occur unless we are assured that that change will not jeopardize anything that is important to us.']", "label": 3 }, { "id": "train_1828", "context": "In an attempt to counter complaints that a certain pesticide is potentially hazardous to humans if absorbed into edible plants, the pesticide manufacturer has advertised that \"ounce for ounce, the active ingredient in this pesticide is less toxic than the active ingredient in mouthwash. ", "question": "Which one of the following, if true, indicates a weakness in the manufacturer's argument?", "answers": "['On average, the toxins present in the pesticide take longer than the toxins present in mouthwash to reach harmful levels in the human body.', 'Since the government began to regulate the pesticide industry over ten years ago, there has been a growing awareness of the dangers of toxins used in pesticides.', 'The quantity of toxins humans ingest by consuming plants treated with the pesticide is, on average, much higher than the quantity of toxins humans ingest by using mouthwash.', 'The ounce-for-ounce toxicity of the active ingredient in mouthwash is less than that of most products meant for external use by humans, such as nail polish or other cosmetics.']", "label": 2 }, { "id": "train_1829", "context": "Confronted with financial indebtedness, the Capital City Soccer Club' s administrators reduced the budget for the youth soccer program from last year' s $500, 000 to $200, 000 for the coming year. However, the youth program cannot run on less than $350, 000. Since the administrators cannot divert funds away from other programs to the youth program, there is no way that the youth soccer program can continue running for the coming year.", "question": "The conclusion for the argument is properly drawn if which one of the following is assumed?", "answers": "['The administrators at the soccer club value other programs at the club more highly than they do the youth soccer program.', 'The youth soccer program did not use all of the $500, 000 that was budgeted to it last year.', \"The youth soccer program has no source of funding other than that budgeted to it for the coming year by the Capital City Soccer Club's administrators.\", 'The budgets of other programs at the soccer club were also reduced.']", "label": 2 }, { "id": "train_1830", "context": "Geothermal power plants produce power using heat from underground reservoirs of hot water or steam heated by the surrounding rock. In the limited areas of the world where such underground hot water and steam can currently be reached by drilling, geothermal power plants produce power more economically than conventional, fossil fuel power plants. However, advocates contend that in the near future economical power from geothermal power plants will be available in most areas.", "question": "Which one of the following, if true, most helps to justify the advocates' contention?", "answers": "['Conventional power plants, unlike geothermal power plants, release large amounts of pollutants into the air.', 'The high start-up costs of geothermal power plants discourages their construction even in locations where they are more economical than conventional power plants in the long run.', 'Advanced drilling technology is being developed that will soon make it both feasible and economical to drill wells many times deeper than it is currently possible to drill.', 'Recent research has led to discoveries that could significantly lower production costs for nearly all types of power plants.']", "label": 2 }, { "id": "train_1831", "context": "To evaluate a plan to save money on office-space expenditures by having its employees work at home, XYZ Company asked volunteers from its staff to try the arrangement for six months. During this period, the productivity of these employees was as high as or higher than before.", "question": "Which of the following, if true, would argue most strongly against deciding, on the basis of the trial results, to implement the company's plan?", "answers": "['The volunteers who worked at home were able to communicate with other employees as necessary for performing the work.', \"The employees who agreed to participate in the test of the plan were among the company's most self-motivated and independent workers.\", 'Other companies that have achieved successful results from work-at-home plans have workforces that are substantially larger than that of XYZ.', 'Minor changes in the way office work is organized at XYZ would yield increases in employee productivity similar to those achieved in the trial.']", "label": 1 }, { "id": "train_1832", "context": "Consumers seek to purchase the highest quality at the lowest prices. Companies that do not offer products that attract consumers eventually go bankrupt. Ttherefore, companies that offer neither the best quality nor the lowest price will eventually go bankrupt.", "question": "The conclusion above follows logically if which one of the following is assumed?", "answers": "['Products that are neither highest in quality nor lowest in price do not attract consumers.', 'Some consumers will not continue to patronize a company purely out of brand loyalty.', 'Any company that offers either the highest quality or the lowest price will avoid bankruptcy.', 'No company is driven from the market for reasons other than failing to meet consumer demands.']", "label": 0 }, { "id": "train_1833", "context": "It is widely believed that lancelets -- small, primitive sea animals -- do not have hearts. Each lancelet has a contracting vessel, but this vessel is considered an artery rather than a heart. However, this vessel is indeed a heart. After all, it strongly resembles the structure of the heart of certain other sea animals. Moreover, the muscular contractions in the lancelet' s vessel closely resemble the muscular contractions of other animals' hearts.", "question": "The argument's conclusion follows logically if which one of the following is assumed?", "answers": "['Some primitive animals other than lancelets have what is widely held to be a heart.', 'A vessel whose structure and actions closely resemble those of other animal hearts is a heart.', 'No animal that has a heart lacks an artery.', 'Only animals that have contracting vessels have hearts.']", "label": 1 }, { "id": "train_1834", "context": "Editorialist: Some people propose that, to raise revenues and encourage conservation, our country' s taxes on oil, gasoline, and coal should be increased. Such a tax increase, however, would do more harm than good. By raising energy costs, the tax increase would decrease our competitiveness with other countries. Many families would be unfairly burdened with higher transportation costs. Finally, by reducing the demand for energy, the tax increase would reduce the number of energy production jobs.", "question": "Each of the following, if true, would weaken the editorialist's argument EXCEPT:", "answers": "['Higher gasoline prices will encourage people to carpool, which will reduce individual transportation costs.', 'The proposed tax increase would be larger for some energy sources than for others.', 'Higher gasoline prices tend to lead to a cleaner environment, because people do less nonessential driving.', \"The editorialist's country's budget deficit will decrease if the energy tax increase is implemented, thus benefiting the economy.\"]", "label": 1 }, { "id": "train_1835", "context": "All material bodies are divisible into parts, and everything divisible is imperfect. It follows that all material bodies are imperfect. It likewise follows that the spirit is not a material body.", "question": "The final conclusion above follows logically if which one of the following is assumed?", "answers": "['The spirit is either indivisible or imperfect.', 'The spirit is divisible.', 'Nothing imperfect is indivisible.', 'The spirit is perfect.']", "label": 3 }, { "id": "train_1836", "context": "Sociologist: Suggestions for improved efficiency that derive from employers are unlikely to elicit positive responses from employees, who tend to resent suggestions they did not generate. An employer should ttherefore engage the employee in a nonthreatening dialogue that emphasizes the positive contributions of the employee to the development of such ideas. Then the ideas employers want to try will be implemented more quickly and effectively.", "question": "Which one of the following principles, if valid, most helps to justify the sociologist's reasoning?", "answers": "['Employees are more likely to carry out ideas for improved efficiency that they believe they have participated in generating.', 'Employees are more likely to implement ideas for improved efficiency that derive from a dialogue in which they have participated than from a dialogue in which they have not participated.', 'Employees are more likely to accept suggestions for improved efficiency when these suggestions are not obviously directed at them.', \"Employees are more likely to resent employers who attempt to implement the employers' rather than the employees' ideas for improved efficiency.\"]", "label": 0 }, { "id": "train_1837", "context": "After replacing his old gas water heater with a new, pilotless, gas water heater that is rated as highly efficient, Jimmy' s gas bills increased.", "question": "Each of the following, if true, contributes to an explanation of the increase mentioned above EXCEPT:", "answers": "['After having done his laundry at a laundromat, Jimmy bought and started using a gas dryer when he replaced his water heater.', \"Shortly after the new water heater was installed, Jimmy's uncle came to live with him, doubling the size of the household.\", \"The new water heater uses a smaller percentage of the gas used by Jimmy's household than did the old one.\", \"Jimmy's utility company raised the rates for gas consumption following installation of the new water heater.\"]", "label": 2 }, { "id": "train_1838", "context": "The waters off the coast of Iceland are filled with pods of killer whales, which migrate there during the summer. Wildlife parks that rely on the killer whales for entertainment hunt the killer whale almost exclusively in the water of Iceland, because strict sanctions forbid them from doing so off the coast of North America, an area also abundant in killer whales. Since Iceland recently gave into pressure from international groups opposed to the hunting of killer whales, it too will forbid the hunting of killer whales off its coast. Ttherefore, all wildlife parks will be forced to end their shows featuring killer whales once their current killer whales are unable to perform.", "question": "All of the following cast doubt on the conclusion of the argument EXCEPT?", "answers": "['The recent ban only extends to within one hundred miles of Iceland, though killer whales are plentiful along the shores of Greenland, which fall outside this range.', 'In-park killer whale births have become increasingly common, especially in those wildlife parks that harbor a large number of killer whales.', 'The incoming prime minister of Canada, who is more conservative, is planning on lifting the ban on hunting killer whales off the coast of Canada.', 'It is nearly impossible to catch killer whales in deep waters, so hunters typically rely on luring killer whales into coves.']", "label": 3 }, { "id": "train_1839", "context": "The best jazz singers use their voices much as horn players use their instruments. The great Billie Holiday thought of her singing voice as a horn, reshaping melody and words to increase their impact. Conversely, jazz horn players achieve their distinctive sounds by emulating the spontaneous twists and turns of an impassioned voice. So jazz consists largely of voicelike horns and hornlike voices.", "question": "Which one of the following most accurately describes the role played in the argument by the claim that the best jazz singers use their voices much as horn players use their instruments?", "answers": [ "It is a statement used to support a conclusion that in turn is used to support the argument's main conclusion.", "It is a statement for which some evidence is provided and which in turn is used to provide support for the argument's main conclusion.", "It is the argument's only conclusion, and each of the other statements in the argument is used to support it.", "It is the argument's main conclusion and is supported by another statement, which is itself supported by a further statement." ], "label": 1 }, { "id": "train_1840", "context": "Pundit: The only airline providing service for our town announced that because the service is unprofitable it will discontinue this service next year. Town officials have urged the community to use the airline' s service more frequently so that the airline will change its decision. There is no reason to comply with their recommendation, however, for just last week these same officials drove to an out-of town conference instead of flying.", "question": "The pundit's reasoning is most vulnerable to criticism on the grounds that it presumes, without providing justification, that", "answers": "['if the town officials did not follow their own advice then that advice is not worth following', \"the town officials paid for their trip with taxpayers' money rather than their own money\", \"suspending service and losing money by continuing service are the airline's only options\", 'ground transportation is usually no less expensive than airplane transportation']", "label": 0 }, { "id": "train_1841", "context": "Tom: Executives in this country make around 85 times what the average worker earns. This is an extraordinarily large disparity, and ttherefore public resentment over the size of executives' salaries is justified. Martha: Such resentment is not justified, since wealth is created by taking risks and making decisions, actions most people prefer to avoid. Generous rewards for those who choose not to avoid these actions are both fair and necessary. Tom: I think you misunderstood me. I' m not saying that people resent that there is a large disparity here between executives' salaries and workers' salaries, but rather they resent that it is atypically large: in other countries executives earn only 20 or 30 times what the average worker earns.", "question": "Which one of the following most accurately expresses the main point at issue between Tom and Martha?", "answers": "['whether public resentment of the size of executive salaries is justified', \"whether executives in this country make 85 times the average worker's salary\", 'whether executives in this country create as much wealth as do those in other countries', 'whether executives deserve higher salaries than workers deserve']", "label": 0 }, { "id": "train_1842", "context": "Detective: Because the embezzler must have had specialized knowledge and access to internal financial records, we can presume that the embezzler worked for XYZ Corporation as either an accountant or an actuary. But an accountant would probably not make the kind of mistakes in ledger entries that led to the discovery of the embezzlement. Thus it is likely that the embezzler is one of the actuaries.", "question": "Each of the following weakens the detective's argument EXCEPT:", "answers": "['An independent report released before the crime took place concluded that XYZ Corporation was vulnerable to embezzlement.', 'There is evidence of breaches in computer security at the time of the embezzlement that could have given persons outside XYZ Corporation access to internal financial records.', \"The actuaries' activities while working for XYZ Corporation were more closely scrutinized by supervisors than were the activities of the accountants.\", 'XYZ Corporation employs eight accountants, whereas it has only two actuaries on its staff.']", "label": 0 }, { "id": "train_1843", "context": "On Saturday Melvin suggested that Jerome take the following week off from work and accompany him on a trip to the mountains. Jerome refused, claiming that he could not afford the cost of the trip added to the wages he would forfeit by taking off without notice. It is clear, however, that cost cannot be the real reason for Jerome' s unwillingness to go with Melvin to the mountains, since he makes the same excuse every time Melvin asks him to take an unscheduled vacation regardless of where Melvin proposes to go.", "question": "The reasoning is most vulnerable to which one of the following criticisms?", "answers": "[\"It does not examine the possibility that Jerome's behavior is adequately explained by the reason he gives for it.\", \"It attempts to forestall an attack on Melvin's behavior by focusing attention on the behavior of Jerome.\", 'It overlooks the possibility that Jerome, unlike Melvin, prefers vacations that have been planned far in advance.', 'It fails to establish that Melvin could no more afford to take an unscheduled vacation trip to the mountains than could Jerome.']", "label": 0 }, { "id": "train_1844", "context": "Last year a chain of fast-food restaurants, whose menu had always centered on hamburger, added its first vegetarian sandwich, much lower in fat than the chain's other offerings. Despite heavy marketing, the new sandwich accounts for a very small proportion of the chain's sales. The sandwich's sales would have to quadruple to cover the costs associated with including it on the menu. Since such an increase is unlikely, the chain would be more profitable if it dropped the sandwich.", "question": "Which of the following, if true, most seriously weakens the argument?", "answers": [ "An attempt by the chain to introduce a lower-fat hamburger failed several years ago, since it attracted few new customers and most of the chain's regular customers greatly preferred the taste of the regular hamburger.", "Although many of the chain's customers have never tried the vegetarian sandwich, in a market research survey most of those who had tried it reported that they were very satisfied with it.", "Many of the people who eat at the chain's restaurants also eat at the restaurants of competing chains and report no strong preference among the competitors.", "When even one member of group of diner's is a vegetarian or has a preference for low-fat food, the group tends to avoid restaurants that lack vegetarian or low-fat menu options." ], "label": 3 }, { "id": "train_1845", "context": "Linguist: Regional dialects, many of which eventually become distinct languages, are responses by local populations to their own particular communicative needs. So even when the unification of the world economy forces the adoption of a universal language for use in international trade, this language itself will inevitably develop many regional dialects.", "question": "Which one of the following is an assumption that the linguist's argument requires?", "answers": "['A universal language for use in international trade will not arise unless the world economy is unified.', 'After the unification of the world economy, there will be variation among many different local populations in their communicative needs in international trade.', 'No two local populations have the same communicative needs as each other.', 'When the unification of the world economy forces the adoption of a universal language for use in international trade, many regional dialects of other languages will be eradicated.']", "label": 1 }, { "id": "train_1846", "context": "The retail price of decaffeinated coffee is considerably higher than that of regular coffee. However, the process by which coffee beans are decaffeinated is fairly simple and not very costly. Ttherefore, the price difference cannot be accounted for by the greater cost of providing decaffeinated coffee to the consumer.", "question": "The argument relies on assuming which one of the following?", "answers": "['Processing regular coffee costs more than processing decaffeinated coffee.', 'Retail coffee-sellers do not expect that consumers are content to pay more for decaffeinated coffee than for regular coffee.', 'Price differences between products can generally be accounted for by such factors as supply and demand, not by differences in production costs.', 'The beans used for producing decaffeinated coffee do not cost much more before processing than the beans used for producing regular coffee.']", "label": 3 }, { "id": "train_1847", "context": "Twenty years ago the Republic of Rosinia produced nearly 100 million tons of potatoes, but last year the harvest barely reached 60 million tons. Agricultural researchers, who have failed to develop new higher-yielding strains of potatoes, are to blame for this decrease, since they have been concerned only with their own research and not with the needs of Rosinia.", "question": "Which one of the following is an assumption on which the argument depends?", "answers": "['Agricultural researchers often find concrete solutions to practical problems when investigating seemingly unrelated questions.', 'Strains of potatoes most commonly grown in Rosinia could not have produced the yields last year that they once did.', 'Agricultural research in Rosinia is funded by government grants.', 'Wide fluctuations in the size of the potato crop over a twenty-year period are not unusual.']", "label": 1 }, { "id": "train_1848", "context": "The older a country is, the more likely it is to be ruled by a monarch. Thus, since most countries are not ruled by monarchs, if a country is particularly new it is probably not ruled by a monarch.", "question": "The pattern of reasoning in the argument above is most similar to that in which one of the following arguments?", "answers": "['Most novels are not made into movies. However, the more popular a movie is, the more likely it is that the movie was based on a novel. Thus, if a novel is particularly popular, it will probably be made into a movie.', 'Most novels are not made into movies. Moreover, if a novel is particularly unpopular, it will probably not be made into a movie. Thus, the more popular a novel is, the more likely it is to be made into a movie.', 'Most novels are not made into movies. However, the more popular a novel is, the more likely it is to be made into a movie. Thus, if a movie is quite unpopular, it was probably not based on a novel.', 'Most novels are not made into movies. However, the more popular a novel is, the more likely it is to be made into a movie. Thus, if a novel is quite unpopular, it will probably not be made into a movie.']", "label": 3 }, { "id": "train_1849", "context": "Mallotech portrays itself to the public as a socially responsible company, but critics charge that employees in many of its factories work in unsanitary conditions. Unless these critics are mistaken, then, Mallotech is not accurately portraying itself to the public.", "question": "The argument's conclusion follows logically if which one of the following is assumed?", "answers": "['A socially responsible company would never lie about whether its employees are working in unsanitary conditions.', 'A socially responsible company would not have employees working in unsanitary conditions.', 'No company that conceals information from the public is socially responsible.', \"Many employees in Mallotech's factories work in unsanitary conditions.\"]", "label": 1 }, { "id": "train_1850", "context": "Senator Strongwood reported that, contrary to a study cited by the administration, a thorough study by his own party concluded that a reduction in the capital gains tax would lead to an increase in the federal deficit. \"Hooray for common sense, \" he said. \"Everyone knows that when you cut taxes you lose revenue. \" He concluded that the administration' s plan for reducing the capital gains tax was now dead, because he could not imagine any senator voting to increase the deficit.", "question": "Which one of the following accurately describes something Senator Strongwood does in advancing his argument?", "answers": "['He resorts to name-calling by expressly stating that his opponents lack common sense.', 'He assumes that senators will rarely vote for unpopular legislation.', 'He assumes that a study commissioned by his party must be more objective than one commissioned by the administration.', \"He assumes senators will believe his party's report instead of the administration's.\"]", "label": 3 }, { "id": "train_1851", "context": "Economist: In today' s post-industrial economy, higher education is essential for achieving prosperity. Without a college degree or higher, citizens do not have access to the highest quartile of salaries, and only individuals earning salaries in this highest quartile have enough disposable income to buy real estate and invest in long term assets.", "question": "The argument assumes that", "answers": "['everyone in the highest quartile of salaries lives in a house that he or she owns', 'understanding what prosperity is, from a college-educated perspective, is essential to achieving it', 'all the jobs in the highest quartile of salaries require skills that are always covered as part of a college education', 'prosperity has to include ownership of real estate or long term assets.']", "label": 3 }, { "id": "train_1852", "context": "The largest volcano on Mars rises 27 kilometers above the surrounding plain and covers an area roughly the size of Romania. Even if the Earth' s gravity were as low as the gravity of Mars is, no volcano of such size could exist on Earth, for the Earth' s crust, although of essentially the same composition as that of Mars, is too thin to support even a small fraction of that mass and would buckle under it, causing the mountain to sink.", "question": "If the statements above are true, which one of the following must also be true on the basis of them?", "answers": "['The crust of Mars, at least at certain points on the planet, is thicker than the crust of the Earth.', 'On average, volcanoes on Mars are higher than those on Earth.', 'The surface of Mars is less subject to forces of erosion than is the surface of the Earth.', 'The highest volcanoes on Mars occur where its crust is thickest.']", "label": 0 }, { "id": "train_1853", "context": "While it might be expected that those neighborhoods most heavily patrolled by police have the least crime, the statistical evidence overwhelmingly supports the claim that such neighborhoods have the most crime. This shows that the presence of police does not decrease crime in a neighborhood.", "question": "The reasoning in the argument is flawed because the argument", "answers": "['fails to consider the possibility that police presence in a particular area is often a response to the relatively high crime rate in that area', 'draws a general conclusion from too small a sample of data', 'attempts to support its conclusion by making an appeal to emotions', 'fails to consider the possibility that criminals may commit crimes in more than one neighborhood']", "label": 0 }, { "id": "train_1854", "context": "Vitacorp, a manufacturer, wishes to make its information booth at an industry convention more productive in terms of boosting sales. The booth offers information introducing the company' s new products and services. To achieve the desired result, Vitacorp' s marketing department will attempt to attract more people to the booth. The marketing director' s first measure was to instruct each salesperson to call his or her five best customers and personally invite them to visit the booth.", "question": "Which of the following, if true, most strongly supports the prediction that the marketing director's first measure will contribute to meeting the goal of boosting sales?", "answers": "[\"Most of Vitacorp's best customers also have business dealings with Vitacorp's competitors.\", \"Vitacorp's salespeople routinely inform each important customer about new products and services as soon as the decision to launch them has been made.\", 'An information booth that is well attended tends to attract visitors who would not otherwise have attended the booth.', 'Vitacorp has fewer new products and services available this year than it had in previous years.']", "label": 2 }, { "id": "train_1855", "context": "Anyone who believes in democracy has a high regard for the wisdom of the masses. Griley, however, is an elitist who believes that any artwork that is popular is unlikely to be good. Thus, Griley does not believe in democracy.", "question": "The conclusion follows logically if which one of the following is assumed?", "answers": "['Anyone who believes that an artwork is unlikely to be good if it is popular is an elitist.', 'Anyone who believes that if an artwork is popular it is unlikely to be good does not have a high regard for the wisdom of the masses.', 'If Griley is not an elitist, then he has a high regard for the wisdom of the masses.', 'Anyone who does not have a high regard for the wisdom of the masses is an elitist who believes that if an artwork is popular it is unlikely to be good.']", "label": 1 }, { "id": "train_1856", "context": "Most auto mechanics have extensive experience. Furthermore, most mechanics with extensive experience understand electronic circuits. Thus, most auto mechanics understand electronic circuits.", "question": "The pattern of flawed reasoning in which one of the following arguments is most similar to that in the argument above?", "answers": "['It is not surprising that most speeding tickets in this region are issued to drivers of sports cars. After all, most drivers who are not interested in driving fast do not buy sports cars.', 'Most snow-removal companies run lawn-care services during the summer. Also, most companies that run lawn-care services during the summer hire additional workers in the summer. Thus, most snow-removal companies hire additional workers in the summer.', 'Most nature photographers find portrait photography boring. Moreover, most portrait photographers especially enjoy photographing dignitaries. Thus, most nature photographers find photographing dignitaries especially boring.', 'The most common species of birds in this region are migratory. Moreover, most migratory birds have left this region by the end of November. Hence, few birds remain in this region during the winter.']", "label": 1 }, { "id": "train_1857", "context": "Over the last few decades, public outcries against pollution have brought about stricter regulations of emissions. The cities that had the most polluted air 30 years ago now have greatly improved air quality. This would not have happened without these stricter regulations.", "question": "Which one of the following can be properly inferred from the statements above?", "answers": "['Public criticism led to an improvement in the air quality of the cities that had the most polluted air 30 years ago.', 'Most of the public outcries against pollution came from people in the cities that had the most polluted air.', 'In the city with the worst air pollution today, the air quality is better than it was 30 years ago.', 'The most polluted cities today are not the cities that were the most polluted 30 years ago.']", "label": 0 }, { "id": "train_1858", "context": "Lecturer: Given our current state of knowledge and technology, we can say that the generalization that the entropy of a closed system cannot decrease for any spontaneous process has not been falsified by any of our tests of that generalization. So we conclude it to be true universally. Yet, it must be admitted that this generalization has not been conclusively verified, in the sense that it has not been tested in every corner of the universe, under every feasible condition. Nevertheless, this generalization is correctly regarded as a scientific law; indeed, it is referred to as the Second Law of Thermodynamics.", "question": "Which one of the following principles, if valid, most justifies the lecturer's classification of the generalization described above?", "answers": "['If a generalization is confirmed to the extent current science allows, then it is considered a scientific law.', 'If a generalization is confirmed only under a few circumstances, it should not be considered a scientific law.', 'Whatever is regarded as a scientific law will eventually be conclusively verified.', 'Whatever is true universally will eventually be confirmed to the extent current science allows.']", "label": 0 }, { "id": "train_1859", "context": "Marmosets are the only primates other than humans known to display a preference for using one hand rather than the other. Significantly more marmosets are left-handed than are right-handed. Since infant marmosets engage in much imitative behavior, researchers hypothesize that it is by imitation that infant marmosets learn which hand to use, so that offspring reared by left-handed parents generally share their parents' handedness.", "question": "Which one of the following, if true, most supports the researchers' hypothesis?", "answers": "['Ninety percent of humans are right-handed, but those who are left-handed are likely to have at least one left-handed parent.', 'Right-handed marmosets virtually all have at least one sibling who is left-handed.', 'Marmosets raised in captivity with right-handed adult marmosets to whom they are not related are more likely to be right-handed than left-handed.', 'A study conducted on adult marmosets revealed that many were right-handed.']", "label": 2 }, { "id": "train_1860", "context": "A recent study examined the daytime and nighttime activity patterns of two populations of tree-dwelling lemurs -- the first living in a rain forest, where tree canopy cover is consistent year-round, and the second living in a deciduous forest, where many trees lose their leaves during the winter months. Both groups of lemurs were found to be more nocturnal during winter months than they were the rest of the year. However, the winter increase in nocturnal activity was significantly more pronounced for the population living in the deciduous forest than it was for the population living in the rain forest.", "question": "Which one of the following, if true, most helps to explain the difference between the two lemur populations with respect to winter activity patterns?", "answers": "['The primary predators for both lemur populations are high-flying birds that rely on their eyesight to hunt prey during daylight.', 'In both habitats, species of predatory snakes active during daylight are most active during winter months.', 'For both lemur populations, the primary competitors for food resources are species active during daylight.', 'The lemur population in the rain forest eats both plants and insects whereas the population in the deciduous forest eats only plants.']", "label": 0 }, { "id": "train_1861", "context": " Plant scientists have used genetic engineering on seeds to produce crop plants that are highly resistant to insect damage . Unfortunately, the seeds themselves are quite expensive, and the plants require more fertilizer and water to grow well than normal ones. Accordingly, for most farmers the savings on pesticides would not compensate for the higher seed costs and the cost of additional fertilizer . However, since consumer demand for grains, fruits and vegetables grown without the use of pesticides continues to rise, the use of genetically engineered seeds of this kind is likely to become widespread", "question": ". In the argument given, the two portions in boldface play which of the following roles?", "answers": "['The first and the second each provide evidence to support the arguments main conclusion', 'The first provides evidence to support a prediction that the argument seeks to defend; the second is that prediction', 'The first presents a development that the argument Predicts will have a certain outcome, the second acknowledges a consideration that tends to weigh against that prediction', 'The first supplies a context for the argument; the second is the arguments main conclusion']", "label": 2 }, { "id": "train_1862", "context": "In the late 1980' s, the population of sea otters in the North Pacific Ocean began to decline. Of the two plausible explanations for the decline-increased predation by killer whales or disease-disease is the more likely. After all, a concurrent sharp decline in the populations of seals and sea lions was almost certainly caused by a pollution-related disease, which could have spread to sea otters, whereas the population of killer whales did not change noticeably.", "question": "Which of the following, if true, most seriously weakens the reasoning?", "answers": "[\"There is no indication that substantial numbers of sea otters migrated to other locations from the North Pacific in the 1980's.\", \"Along the Pacific coast of North America in the 1980's, sea otters were absent from many locations where they had been relatively common in former times.\", 'The North Pacific populations of seals and sea lions cover a wider geographic area than does the population of sea otters.', 'Killer whales in the North Pacific usually prey on seals and sea lions but will, when this food source is scarce, seek out other prey.']", "label": 3 }, { "id": "train_1863", "context": "Each year red-winged blackbirds stop in a certain region of Midland Province on their spring and fall migrations. In the fall, they eat a significant portion of the province' s sunflower crop. This year Midland farmers sought permits to set out small amounts of poisoned rice during the blackbirds' spring stop in order to reduce the fall blackbird population. Some residents voiced concern that the rice could threaten certain species of rare migratory birds. Nevertheless, the wildlife agency approved the permits.", "question": "Which of the following, if true, most helps to justify the wildlife agency's approval of the permits, given the concerns voiced by some residents?", "answers": "['The poison that farmers want to use does not kill birds but rather makes them incapable of producing viable eggs.', 'In the region where the red-winged blackbirds stop, they are the first birds to be present in the spring.', 'The poison that farmers got approval to use has no taste or smell that would make it detectable by birds.', 'Without the permit, any farmers shown to have set out poison for the blackbirds would be heavily fined.']", "label": 1 }, { "id": "train_1864", "context": "Decision makers tend to have distinctive styles. One such style is for the decision maker to seek the widest possible input from advisers and to explore alternatives while making up his or her mind. In fact, decision makers of this sort will often argue vigorously for a particular idea, emphasizing its strong points and downplaying its weaknesses, not because they actually believe in the idea but because they want to see if their real reservations about it are idiosyncratic or are held independently by their advisers.", "question": "Which one of the following is most strongly supported by the statements above?", "answers": "[\"If certain decision makers' statements are quoted accurately and at length, the content of the quote could nonetheless be greatly at variance with the decision eventually made.\", 'Certain decision makers do not know which ideas they do not really believe in until after they have presented a variety of ideas to their advisers.', \"If certain decision makers' advisers know the actual beliefs of those they advise, those advisers will give better advice than they would if they did not know those beliefs.\", 'Certain decision makers proceed in a way that makes it likely that they will frequently decide in favor of ideas in which they do not believe.']", "label": 0 }, { "id": "train_1865", "context": "Municipal legislator: The mayor proposes that the city accept a lighting company' s gift of several high-tech streetlights. Surely there would be no problem in accepting these despite some people' s fear that the company wants to influence the city' s decision regarding park lighting contracts. The only ulterior motive I can find is the company' s desire to have its products seen by mayors who will visit the city for an upcoming convention. In any case, favoritism in city contracts is prevented by our competitive-bidding procedure.", "question": "Which one of the following most accurately expresses the main conclusion of the municipal legislator's argument?", "answers": "[\"The lighting company's desire to display its products to visiting mayors is the real motivation behind the suggested gift of streetlights.\", \"Some people's fear that the company wants to influence the city's decision regarding park lighting contracts is unfounded.\", 'It is not appropriate that any company should have the unique opportunity to display its products to mayors attending the upcoming convention.', \"The mayor's proposal to accept the gift of streetlights should not be considered problematic.\"]", "label": 3 }, { "id": "train_1866", "context": "It is possible to grow agricultural crops that can thrive when irrigated with seawater. Such farming, if undertaken near oceans, would actually be cheaper than most other irrigated agriculture, since the water would not have to be pumped far. The greatest expense in irrigated agriculture is in pumping the water, and the pumping costs increase with the distance the water is pumped.", "question": "Which one of the following most accurately describes the role played in the argument by the claim that the greatest expense in irrigated agriculture is in pumping the water?", "answers": "[\"It is a claim for which the argument provides evidence, but which is not the argument's conclusion.\", 'It is a claim that the argument shows to be false.', \"It is evidence provided to support the argument's conclusion.\", \"It is the argument's conclusion.\"]", "label": 2 }, { "id": "train_1867", "context": "The ability to analyze genomes - sequences of DNA - has grown more and more sophisticated. Scientists are able to examine the biological past in finer detail and with greater accuracy. A new analysis of the genetic links between early humans and chimpanzees has led to a hypothesis that the two species diverged more recently than previous estimates indicated. This gives credence to a very startling new theory: the ancestors of humans and chimpanzees might have hybridized to produce the lineage from which modern humans eventually developed.", "question": "What can be inferred from the statements above?", "answers": "['Political and religious pressures will affect the course of future research.', 'Hybridized species combine the best of both lineages.', 'Earlier genome analysis established genetic links between early humans and chimpanzees.', 'Human DNA is more complex than that of chimpanzees.']", "label": 2 }, { "id": "train_1868", "context": "Resident: Residents of this locale should not consider their loss of farming as a way of life to be a tragedy. When this area was a rural area it was economically depressed, but it is now a growing bastion of high-tech industry with high-wage jobs, and supports over 20 times the number of jobs it did then.", "question": "Which one of the following, if true, does the most to justify the conclusion of the resident's argument?", "answers": "['Residents of this locale do not value a rural way of life more than they value economic prosperity.', 'The development of high-tech industry is more valuable to national security than is farming.', 'The loss of a family farm is often perceived as tragic even when no financial hardship results.', 'Many residents of this locale have annual incomes that are twice what they were when the locale was primarily agricultural.']", "label": 0 }, { "id": "train_1869", "context": "Scientists have modified feed corn genetically, increasing its resistance to insect pests. Farmers who tried out the genetically modified corn last season applied less insecticide to their corn fields and still got yields comparable to those they would have gotten with ordinary corn. Ordinary corn seed, however, costs less, and what these farmers saved on insecticide rarely exceeded their extra costs for seed. Ttherefore, for most feed-corn farmers, switching to genetically modified seed would be unlikely to increase profits.", "question": "Which of the following would it be most useful to know in order to evaluate the argument?", "answers": "['Whether the price that farmers receive for feed corn has remained steady over the past few years', 'Whether most of the farmers who tried the genetically modified corn last season applied more insecticide than was actually necessary', 'Whether, for most farmers who plant feed corn, it is their most profitable crop', 'Whether the insecticides typically used on feed corn tend to be more expensive than insecticides typically used on other crops']", "label": 1 }, { "id": "train_1870", "context": "All societies recognize certain rules to be so crucial that they define those rules as duties, such as rules restricting violence and those requiring the keeping of agreements. Contained in the notion of a duty is the idea that its fulfillment is so fundamental to a properly functioning society that persons obligated by it cannot be excused on the ground that its fulfillment would be harmful to their self-interest. This shows that __.", "question": "Which one of the following most reasonably completes the argument?", "answers": "['all societies overrate the benefits of certain rules, such as those governing the keeping of agreements', 'all societies recognize the possibility of clashes between individual self-interest and the performance of duty', 'societies have no right to expect people always to perform their duties', 'all societies have certain rules that no people are capable of following']", "label": 1 }, { "id": "train_1871", "context": "Pundit: The average salary for teachers in our society is lower than the average salary for athletes. Obviously, our society values sports more than it values education.", "question": "The reasoning in the pundit's argument is questionable because the argument", "answers": "['presumes, without providing justification, that sports have some educational value', 'fails to consider that the total amount of money spent on education may be much greater than the total spent on sports', \"fails to compare salaries for teachers in the pundit's society to salaries for teachers in other societies\", \"compares teachers' salaries only to those of professional athletes rather than also to the salaries of other professionals\"]", "label": 1 }, { "id": "train_1872", "context": "Sam: Mountain lions, a protected species, are preying on bighorn sheep, another protected species. We must let nature take its course and hope the bighorns survive. Meli: Nonsense. We must do what we can to ensure the survival of the bighorn, even if that means limiting the mountain lion population.", "question": "Which one of the following is a point of disagreement between Meli and Sam?", "answers": "['Humans should not intervene to protect bighorn sheep from mountain lions.', 'If the population of mountain lions is not limited, the bighorn sheep species will not survive.', 'The preservation of a predatory species is easier to ensure than the preservation of the species preyed upon.', 'Any measures to limit the mountain lion population would likely push the species to extinction.']", "label": 0 }, { "id": "train_1873", "context": "Journalist: Obviously, though some animals are purely carnivorous, none would survive without plants. But the dependence is mutual. Many plant species would never have come to be had there been no animals to pollinate, fertilize, and broadcast their seeds. Also, plants' photosynthetic activity would deplete the carbon dioxide in Earth' s atmosphere were it not constantly being replenished by the exhalation of animals, engine fumes, and smoke from fires, many set by human beings.", "question": "Which one of the following most accurately expresses the main conclusion of the journalist's argument?", "answers": "['The photosynthetic activity of plants is necessary for animal life, but animal life is also necessary for the occurrence of photosynthesis in plants.', 'Some purely carnivorous animals would not survive without plants.', \"Human activity is part of what prevents plants from depleting the oxygen in Earth's atmosphere on which plants and animals alike depend.\", 'Just as animals are dependent on plants for their survival, plants are dependent on animals for theirs.']", "label": 3 }, { "id": "train_1874", "context": "The more frequently employees take time to exercise during working hours each week, the fewer sick days they take. Even employees who exercise only once a week during working hours take less sick time than those who do not exercise. Ttherefore, if companies started fitness programs, the absentee rate in those companies would decrease significantly.", "question": "Which of the following, if true, most seriously weakens the argument above?", "answers": "[\"Employees who exercise in their company's fitness program use their working time no more productively than those who do not exercise.\", 'Employees who are frequently absent are the least likely to cooperate with or to join a corporate fitness program.', \"Employees who exercise only once a week in their company's fitness program usually also exercise after work.\", 'Employees who exercise during working hours occasionally fall asleep for short periods of time after they exercise.']", "label": 1 }, { "id": "train_1875", "context": "Many athletes inhale pure oxygen after exercise in an attempt to increase muscular reabsorption of oxygen. Measured continuously after exercise, however, the blood lactate levels of athletes who inhale pure oxygen are practically identical, on average, to those of athletes who breathe normal air. The lower the blood lactate level is, the higher the muscular reabsorption of oxygen is.", "question": "If the statements above are all true, they most strongly support which of the following conclusions?", "answers": "[\"The amount of oxygen reabsorbed by an athlete's muscles always remains constant.\", 'The inhaling of pure oxygen has no legitimate role in athletics.', 'High blood lactate levels cannot be reduced.', \"Athletes' muscular reabsorption of oxygen is not increased when they inhale pure oxygen instead of normal air.\"]", "label": 3 }, { "id": "train_1876", "context": "Some of the world' s most beautiful cats are Persian cats. However, it must be acknowledged that all Persian cats are pompous, and pompous cats are invariably irritating.", "question": "If the statements above are true, each of the following must also be true on the basis of them EXCEPT:", "answers": "[\"Some of the world's most beautiful cats are irritating.\", \"Some irritating cats are among the world's most beautiful cats.\", 'Any cat that is not irritating is not a Persian cat.', 'Some irritating and beautiful cats are not Persian cats.']", "label": 3 }, { "id": "train_1877", "context": "Fares on the city-run public buses in Greenville are subsidized by city tax revenues, but among the beneficiaries of the low fares are many people who commute from outside the city to jobs in Greenville. Some city councillors argue that city taxes should be used primarily to benefit the people who pay them, and ttherefore that bus fares should be raised enough to cover the cost of the service.", "question": "Each of the following, if true, would weaken the argument advanced by the city councilors EXCEPT:", "answers": "['Increasing transit fares would disadvantage those residents of the city whose low incomes make them exempt from city taxes, and all city councilors agree that these residents should be able to take advantage of city-run services.', 'People who work in Greenville and earn wages above the nationally mandated minimum all pay the city wage tax of 5 percent.', 'Voters in the city, many of whom benefit from the low transit fares, are strongly opposed to increasing local taxes.', \"Many businesses whose presence in the city is beneficial to the city's taxpayers would relocate outside the city if public-transit fares were more expensive.\"]", "label": 2 }, { "id": "train_1878", "context": "A recent study of 6, 403 people showed that those treated with the drug pravastatin, one of the effects of which is to reduce cholesterol, had about one-third fewer nonfatal heart attacks and one-third fewer deaths from coronary disease than did those not taking the drug. This result is consistent with other studies, which show that those who have heart disease often have higher than average cholesterol levels. This shows that lowering cholesterol levels reduces the risk of heart disease.", "question": "The argument's reasoning is flawed because the argument", "answers": "['fails to consider that pravastatin may reduce the risk of heart disease but not as a consequence of its lowering cholesterol levels', 'relies on past findings, rather than drawing its principal conclusion from the data found in the specific study cited', 'neglects the possibility that pravastatin may have severe side effects', 'draws a conclusion regarding the effects of lowering cholesterol levels on heart disease, when in fact the conclusion should focus on the relation between pravastatin and cholesterol levels']", "label": 0 }, { "id": "train_1879", "context": "On completing both the course in experimental design and the developmental psychology course, Angela will have earned a degree in psychology. Since experimental design, which must be completed before taking developmental psychology, will not be offered until next term, it will be at least two terms before Angela gets her psychology degree.", "question": "If the statements above are all true, which one of the following must also be true?", "answers": "['There are no prerequisites for the course in experimental design.', 'The course in experimental design is an easier course than the course in developmental psychology.', 'Anyone who earns a degree in psychology from the university Angela attends will have completed the course in experimental design.', 'Once Angela completes the developmental psychology course, she will have earned a degree in psychology.']", "label": 3 }, { "id": "train_1880", "context": "If the county continues to collect residential trash at current levels, landfills will soon be overflowing and parkland will need to be used in order to create more space. Charging each household a fee for each pound of trash it puts out for collection will induce residents to reduce the amount of trash they create; this charge will ttherefore protect the remaining county parkland.", "question": "Which of the following is an assumption made in drawing the conclusion above?", "answers": "[\"The beauty of county parkland is an important issue for most of the county's residents.\", 'Residents will reduce the amount of trash they put out for collection by reducing the number of products they buy.', 'The collection fee will not significantly affect the purchasing power of most residents, even if their households do not reduce the amount of trash they put out.', 'The collection fee will not induce residents to dump their trash in the parklands illegally.']", "label": 3 }, { "id": "train_1881", "context": "Images from ground-based telescopes are invariably distorted by the Earth' s atmosphere. Orbiting space telescopes, however, operating above Earth' s atmosphere, should provide superbly detailed images. Ttherefore, ground-based telescopes will soon become obsolete for advanced astronomical research purposes.", "question": "Which of the following statements, if true, would cast the most doubt on the conclusion drawn above?", "answers": "['Ground-based telescopes located on mountain summits are not subject to the kinds of atmospheric distortion which, at low altitudes, make stars appear to twinkle.', 'By careful choice of observatory location, it is possible for large-aperture telescopes to avoid most of the kind of wind turbulence that can distort image quality.', 'Detailed spectral analyses, upon which astronomers rely for determining the chemical composition and evolutionary history of stars, require telescopes with more light-gathering capacity than space telescopes can provide.', 'When large-aperture telescopes are located at high altitudes near the equator, they permit the best Earth-based observations of the center of the Milky Way Galaxy, a prime target of astronomical research.']", "label": 2 }, { "id": "train_1882", "context": "As part of a new trend in the writing of history, an emphasis on the details of historical events and motivations has replaced the previous emphasis on overarching historical trends and movements, with the result that the latter are often overlooked. In consequence, the ominous parallels that may exist between historical trends and current trends are also overlooked, which lessens our ability to learn from history.", "question": "The statements above, if true, most strongly support which one of the following?", "answers": "['History should be interpreted in a way that gives equal emphasis to overarching historical trends and movements and to the details of historical events and motivations.', 'A change in emphasis in the interpretation of history has lessened our ability to learn from history.', 'Overarching historical trends and movements can be discerned only when details of historical events and motivations are not emphasized.', 'Those who attend to overall trends and movements in history and not to details are the best able to learn from history.']", "label": 1 }, { "id": "train_1883", "context": "Executive: In order to add to our profits, I was planning to promote and distribute herbal tinctures. However, some members of my advisory staff questioned the medical efficacy of such products. So I have consulted a variety of reliable medical publications, and these likewise claim that herbal tinctures are ineffective. Ttherefore, I must conclude that marketing such products would not produce the result I intended.", "question": "The executive's reasoning most closely conforms to which one of the following generalizations?", "answers": "['The promotion and distribution of a new line of products will not prove profitable if a number of reliable authorities declare them to be ineffective.', 'If a majority of reliable sources conclude that a particular substance is medically ineffective, then that substance almost certainly is medically ineffective.', 'To be reliable, a medical publication that evaluates consumer products must include at least some independent evidence.', 'Consulting reliable publications is not, by itself, a reliable basis for determining whether or not the promotion of a new line of products will be profitable.']", "label": 0 }, { "id": "train_1884", "context": "Nearly every criminal trial includes eyewitness testimony, and cognitive psychologists have hypothesized that misidentification by eyewitnesses is a common reason for mistaken convictions in criminal trials.", "question": "Each of the following, if true, supports the cognitive psychologists' hypothesis EXCEPT:", "answers": "[\"The shock of witnessing a crime makes it likely that a witness's memory of the perpetrator's face will be distorted.\", \"Eyewitnesses' reports are the most common reason for conviction.\", 'Judges often instruct juries about those circumstances under which testimony of eyewitnesses is fallible.', 'In most crimes, eyewitnesses have seen the perpetrator only briefly, and people are generally poor at remembering the faces of people they have seen only briefly.']", "label": 2 }, { "id": "train_1885", "context": "When the famous art collector Vidmar died, a public auction of her collection, the largest privately owned, was held. \"I can' t possibly afford any of those works because hers is among the most valuable collections ever assembled by a single person, \" declared art lover MacNeil.", "question": "The flawed pattern of reasoning in which one of the following is most closely parallel to that in MacNeil's argument?", "answers": "['This paragraph is long. So the sentences that comprise it are long.', 'The members of the company are old. So the company itself is old.', 'Each word in the book is in French. So the whole book is in French.', 'The city council voted unanimously to adopt the plan. So councilperson Martinez voted to adopt the plan.']", "label": 0 }, { "id": "train_1886", "context": "One of the requirements for admission to the Lunnville Roller Skating Club is a high degree of skill in roller skating. The club president has expressed concern that the club may have discriminated against qualified women in its admissions this year. Yet half of the applicants admitted to the club this year were women. This proves that there was no discrimination against qualified women applicants in the club' s admissions this year.", "question": "Which one of the following is an assumption on which the conclusion of the argument depends?", "answers": "['No more than half of all the roller skaters in Lunnville are women.', 'No more than half of all the roller skaters in Lunnville are men.', 'This year no more than half of the applicants who met all the qualifications for admission to the club were women.', \"This year no more than half of the members of the club's committee that makes decisions about applicants' qualifications were men.\"]", "label": 2 }, { "id": "train_1887", "context": "In its search for new technologies for solar energy that can produce lower-cost electricity, Smith-Diatom is developing a new way to make dye-sensitive solar cells, in which photons strike light-sensitive dyes. The process uses diatoms, which are unicellular algae that have silicon shells with a complex structure. First, the organic material is removed, and then the shells are coated with a titanium dioxide film that acts as a semiconductor. The diatoms' structure results in more photon activity and thus more efficient production of electricity than with current dye-sensitized solar cells, which in turn lowers the cost.", "question": "Which of the following considerations would, if true, most strongly support the hypothesis that the plan, if implemented, will produce low-cost electricity from dye-sensitive solar cells?", "answers": "['Diatoms are an important link in oceanic food chains and help cycle carbon dioxide from the atmosphere.', 'Because diatoms occur naturally, no special engineering processes are needed to produce the basic dye-sensitive solar cell structures.', 'Dye-sensitive solar cells work somewhat more efficiently in lower light than previous solar cell technologies.', 'The production of dye-sensitive solar cells primarily uses materials that do not harm the environment.']", "label": 1 }, { "id": "train_1888", "context": "Companies O and P each have the same number of employees who work the same number of hours per week. According to records maintained by each company, the employees of Company O had fewer job-related accidents last year than did the employees of Company P. Ttherefore, employees of Company O are less likely to have job-related accidents than are employees of Company P.", "question": "Which of the following, if true, would most weaken the conclusion above?", "answers": "['Company P considered more types of accidents to be job-related than did Company O.', 'The employees of Company P lost more time at work due to job-related accidents than did the employees of Company O.', 'Several employees of Company O each had more than one job-related accident.', 'The majority of job-related accidents at Company O involved a single machine.']", "label": 0 }, { "id": "train_1889", "context": "Camera manufacturers typically advertise their products by citing the resolution of their cameras' lenses, the resolution of a lens being the degree of detail the lens is capable of reproducing in the image it projects onto the film. Differences between cameras in this respect are irrelevant for practical photography, however, since all modern lenses are so excellent that they project far more detail onto the film than any photographic film is capable of reproducing in a developed image.", "question": "Which one of the following most accurately states the main point of the argument?", "answers": "[\"Advertised differences among cameras in the resolution of their lenses have no practical bearing on the cameras' relative quality as photographic tools.\", 'In concentrating their advertising on the issue of image quality, manufacturers are making a mistake about the interests of potential purchasers of cameras.', 'Differences among photographic films in the amount of detail they reproduce have a more significant effect on the quality of the developed image than do differences in the resolution of camera lenses.', \"Camera manufacturers ought to concentrate on building other desirable qualities into their cameras' lenses, rather than concentrating only on the lenses' resolution.\"]", "label": 0 }, { "id": "train_1890", "context": "Newspaper editor: Law enforcement experts, as well as most citizens, have finally come to recognize that legal prohibitions against gambling all share a common flaw: no matter how diligent the effort, the laws are impossible to enforce. Ethical qualms notwithstanding, when a law fails to be effective, it should not be a law. That is why there should be no legal prohibition against gambling.", "question": "Which one of the following, if assumed, allows the argument's conclusion to be properly drawn?", "answers": "['Most citizens must agree with a law for the law to be enforceable.', 'No effective law is unenforceable.', 'No legal prohibitions against gambling are enforceable.', 'Most citizens must agree with a law for the law to be effective.']", "label": 1 }, { "id": "train_1891", "context": "Many vaccines create immunity to viral diseases by introducing a certain portion of the disease-causing virus' s outer coating into the body. Exposure to that part of a virus is as effective as exposure to the whole virus in stimulating production of antibodies that will subsequently recognize and kill the whole virus. To create a successful vaccine of this type, doctors must first isolate in the disease-causing virus a portion that stimulates antibody production. Now that a suitable portion of the virus that causes hepatitis E has been isolated, doctors claim they can produce a vaccine that will produce permanent immunity to that disease.", "question": "Which one of the following, if true, most strongly counters the doctors' claim?", "answers": "['Most of the people who contract hepatitis E are young adults who were probably exposed to the virus in childhood also.', 'Some laboratory animals exposed to one strain of the hepatitis virus developed immunity to all strains of the virus.', 'Many children who are exposed to viruses that cause childhood diseases such as chicken pox never develop those diseases.', 'Researchers developed a successful vaccine for another strain of hepatitis, hepatitis B, after first isolating the virus that causes it.']", "label": 0 }, { "id": "train_1892", "context": "A report that many apples contain a cancer-causing preservative called Alar apparently had little effect on consumers. Few consumers planned to change their apple-buying habits as a result of the report. Nonetheless, sales of apples in grocery stores fell sharply in March, a month after the report was issued.", "question": "Which of the following, if true, best explains the reason for the apparent discrepancy described above?", "answers": "['Because of a growing number of food-safety warnings, consumers in March were indifferent to such warnings.', \"In March, many grocers removed apples from their shelves in order to demonstrate concern about their customers' health.\", 'Public health officials did not believe that apples posed a health threat because only minute traces of Alar were present in affected apples.', 'The report was delivered on television and also appeared in newspapers.']", "label": 1 }, { "id": "train_1893", "context": "Situation: Someone living in a cold climate buys a winter coat that is stylish but not warm in order to appear sophisticated. Analysis: People are sometimes willing to sacrifice sensual comfort or pleasure for the sake of appearances.", "question": "The analysis provided for the situation above is most appropriate for which one of the following situations?", "answers": "['A person sets her thermostat at a low temperature during the winter because she is concerned about the environmental damage caused by using fossil fuels to heat her home.', 'A parent buys a car seat for a young child because it is more colorful and more comfortable for the child than the other car seats on the market, though no safer.', 'A couple buys a particular wine even though their favorite wine is less expensive and better tasting because they think it will impress their dinner guests.', 'An acrobat convinces the circus that employs him to purchase an expensive outfit for him so that he can wear it during his act to impress the audience.']", "label": 2 }, { "id": "train_1894", "context": "Plastic has been replacing aluminum in a widening array of military equipment because it offers equal protection at less weight. A plastic-armored vehicle is, however, twice as costly to make as an aluminum-armored vehicle. Ttherefore, the replacement of aluminum-armored vehicles by plastic-armored vehicles that offer equal protection is not yet advisable from a financial point of view.", "question": "Which of the following, if true, casts most doubt on the conclusion above?", "answers": "['Although the cost of plastic is expected to remain stable, the growing practice of recycling metals will probably drive down the cost of metals.', 'The cost of repairing body damage on plastic-armored vehicles would exceed what the military currently spends to repair body damage on aluminum-armored vehicles.', 'A plastic-armored vehicle could be operated at one-third the current cost of operating an aluminum-armored vehicle.', 'The aluminum-armored vehicle is the least costly to maintain of all the metal-armored vehicles currently used by the military.']", "label": 2 }, { "id": "train_1895", "context": "Journalist: A recent study showed that people who drink three cups of decaffeinated coffee per day are twice as likely to develop arthritis -- inflammation of joints resulting from damage to connective tissue -- as those who drink three cups of regular coffee per day. Clearly, decaffeinated coffee must contain something that damages connective tissue and that is not present in regular coffee.", "question": "Which one of the following would be most useful to know in order to evaluate the journalist's argument?", "answers": "['whether most coffee drinkers drink more than three cups of coffee per day', 'whether people who exercise regularly are more likely to drink decaffeinated beverages than those who do not', 'whether people who drink decaffeinated coffee tend to drink coffee less often than those who drink regular coffee', 'whether the degeneration of connective tissue is slowed by consumption of caffeine and other stimulants']", "label": 3 }, { "id": "train_1896", "context": "Developers of neighborhood Y are trying to lure potential homebuyers by offering a large discount on newly built homes. As a result, the homes in neighborhood Y are available for 20% less than comparably sized homes in neighborhood X. So unless potential homebuyers distrust the developers of neighborhood Y, neighborhood X is unlikely to attract many homebuyers because there are plenty of houses available in neighborhood Y.", "question": "Which of the following is an assumption on which the argument depends?", "answers": "['The comparably priced homes in neighborhood X and neighborhood Y are all situated on similarly sized lots.', 'There are fewer houses for sale in neighborhood X than there are in neighborhood Y.', 'Neighborhood X does not have any qualities or features that would induce some homebuyers to pay higher prices for homes in neighborhood X.', 'None of the houses in neighborhood X are selling for less than the houses in neighborhood Y.']", "label": 2 }, { "id": "train_1897", "context": "Medical Investigator: \"Podiatrists initially assumed that Meyer' s Rash, an itchy rash between the toes, was a fungal infection similar to athlete' s foot. At first, only Dr. Pannikar argued that Meyer' s Rash was viral in nature. All anti-fungal medications proved utterly ineffective in addressing Meyer' s Rash. Now, recent antibody studies seem to suggest, quite surprisingly, that Meyer' s Rash is caused by a virus.", "question": " In the medical investigator's argument, the portion in boldface plays which of the following roles?", "answers": "['It provides evidence in support of the main conclusion of the argument.', 'It introduces a judgment that the argument opposes.', 'It is the main conclusion of the argument.', 'It presents an explanation that the argument concludes is correct.']", "label": 3 }, { "id": "train_1898", "context": "If the forest continues to disappear at its present pace, the koala will approach extinction, \" said the biologist. \"So all that is needed to save the koala is to stop deforestation, \" said the politician.", "question": "Which one of the following statements is consistent with the biologist's claim but not with the politician's claim?", "answers": "['Deforestation is stopped and the koala becomes extinct.', 'Reforestation begins and the koala survives.', 'Deforestation is slowed and the koala survives.', 'Deforestation continues and the koala becomes extinct.']", "label": 0 }, { "id": "train_1899", "context": "Essayist: Wisdom and intelligence are desirable qualities. However, being intelligent does not imply that one is wise, nor does being wise imply that one is intelligent. In my own experience, the people I meet have one or the other of these qualities but not both.", "question": "If the essayist's statements are true, then each of the following could be true EXCEPT:", "answers": "['Many people are intelligent and yet lack wisdom.', 'Most people are both intelligent and wise.', 'No one is either wise or intelligent.', 'Most people are neither intelligent or wise.']", "label": 2 }, { "id": "train_1900", "context": "Dietitian: Many diet-conscious consumers are excited about new \"fake fat\" products designed to give food the flavor and consistency of fatty foods, yet without fat' s harmful effects. Consumers who expect the new fat substitute to help them lose weight arc likely to be disappointed, however. Research has shown that when people knowingly or unknowingly eat foods containing \"fake fat, \"they tend to take in at least as many additional calories as are saved by eating \"fake fat. ", "question": "Which one of the following most accurately expresses the conclusion of the dietitian's argument?", "answers": "['People tend to take in a certain number of daily calories, no matter what types of food they eat', '\"Fake fat\"in foods is probably not going to help consumers meet weight loss goals.', 'Most consumers who think that foods with \"fake fat\" are more nutritious than fatty foods are destined to be disappointed.', '\"Fake fat\" products are likely to contribute to obesity more than do other foods.']", "label": 1 }, { "id": "train_1901", "context": "The Testament of William Thorpe was published around 1530 as an appendix to Thorpe's longer Examination. Many scholars, however, doubt the attribution of the Testament to Thorpe because, whereas the Examination is dated 1406, the Testament is dated 1460. One scholar has recently argued that the 1460 date be amended to 1409, based on the observation that when these numbers are expressed as Roman numerals, MCCCCLX and MCCCCIX, it becomes easy to see how the dates might have become confused through scribal error.", "question": "Which of the following, if true, would most support the scholar's hypothesis concerning the date of the Testament?", "answers": "['In the preface to the 1530 publication, the editor attributes both works to William Thorpe.', 'The Testament contains few references to historical events that occurred later than 1406.', 'The Testament alludes to a date, \"Friday, September 20, \" as apparently contemporaneous with the writing of the Testament, and September 20 fell on a Friday in 1409 but not in 1460.', 'Few writers in fifteenth-century England marked dates in their works using only Roman numerals.']", "label": 2 }, { "id": "train_1902", "context": "In 1998, archaeologists found a forty-million-year-old baby bird frozen in amber. Although the sample was not large enough to determine its flying habits at such a young age, scientists guessed that the three-day-old bird would have been able to fly from birth and might have been hunting at the time the tree sap fell onto the bird, preserving it for millions of years. This proves how different this species of birds is from modern birds; modern birds depend on their parents for care for 2 to 3 weeks, while this ancient specimen was able to hunt right out of the nest.", "question": "The reasoning in the argument is most vulnerable to criticism on the grounds that the argument", "answers": "['does not consider the possibility that immediate flight was a risk to the species.', 'draws its conclusion from data that cannot be proven true.', 'confuses the cause of death with the effects of death.', 'takes an inconsistent stance regarding its premise and conclusion.']", "label": 1 }, { "id": "train_1903", "context": "Harry Trevalga: You and your publication have unfairly discriminated against my poems. I have submitted thirty poems in the last two years and you have not published any of them! It is all because I won the Fenner Poetry Award two years ago and your poetry editor thought she deserved it. Publisher: Ridiculous! Our editorial policy and practice is perfectly fair, since our poetry editor judges all submissions for publication without ever seeing the names of the poets, and hence cannot possibly have known who wrote your poems.", "question": "The publisher makes which one of the following assumptions in replying to Trevalga's charges of unfair discrimination?", "answers": "[\"The poetry editor cannot recognize the poems submitted by Harry Trevalga as his unless Trevalga's name is attached to them.\", \"It is not unusual for poets to contribute many poems to the publisher's publication without ever having any accepted for publication.\", \"The poetry editor's decisions on which poems to publish are not based strictly on judgments of intrinsic merit.\", 'The poetry editor does not bear a grudge against Harry Trevalga for his winning the Fenner Poetry Award.']", "label": 0 }, { "id": "train_1904", "context": "No projects that involve historical restorations were granted building permits this month. Since some of the current projects of the firm of Stein and Sapin are historical restorations, at least some of Stein and Sapin' s projects were not granted building permits this month.", "question": "The pattern of reasoning in the argument above is most similar to that in which one of the following?", "answers": "['None of the doctors working at City Hospital were trained abroad. So, although some hospitals require doctors trained abroad to pass an extra qualifying exam, until now, at least, this has not been an issue for City Hospital.', 'None of the new members of the orchestra have completed their paperwork. Since only those people who have completed their paperwork can be paid this week, at least some of the new members of the orchestra are likely to be paid late.', \"Some of the city's most beautiful parks are not larger than a few acres, and some of the parks only a few acres in size are among the city's oldest. Ttherefore, some of the city's most beautiful parks are also its oldest parks.\", \"Several films directed by Hannah Barker were released this season, but none of the films released this season were enthusiastically reviewed. Ttherefore, at least some of Hannah Barker's films have not received enthusiastic reviews.\"]", "label": 3 }, { "id": "train_1905", "context": "The number of fat grams in a cup of butter is the same as in an equal amount of margarine. Ttherefore, any baked good made with a given amount of butter is no higher in fat than a baked good made with an equal amount of margarine.", "question": "The reasoning in the argument is flawed because the argument", "answers": "['fails to realize that butter or margarine might not be the only fat-containing ingredient in a baked good', 'fails to consider that baked goods differ in the amount of protein and carbohydrates they contain', 'mistakenly believes that all baked goods contain the same amount of butter or margarine', 'tries to differentiate between two type of ingredients that are in fact too similar to be distinguishable']", "label": 0 }, { "id": "train_1906", "context": "From 1973 to 1989 total energy use in this country increased less than 10 percent. However, the use of electrical energy in this country during this same period grew by more than 50 percent, as did the gross national product -- the total value of all goods and services produced in the nation.", "question": "If the statements above are true, then which one of the following must also be true?", "answers": "['In 1989 electrical energy constituted a larger proportion of the energy used to produce the gross national product than did any other form of energy.', 'From 1973 to 1989 there was a decline in the use of energy other than electrical energy in this country.', 'Most of the energy used in this country in 1989 was electrical energy.', 'From 1973 to 1989 there was an increase in the proportion of energy use in this country that consisted of electrical energy use.']", "label": 3 }, { "id": "train_1907", "context": "Career consultant: The most popular career advice suggests emphasizing one' s strengths to employers and downplaying one' s weaknesses. Research shows this advice to be incorrect. A study of 314 managers shows that those who use self-deprecating humor in front of their employees are more likely to be seen by them as even-handed, thoughtful, and concerned than are those who do not.", "question": "The career consultant's reasoning is most vulnerable to criticism on the grounds that it", "answers": "['bases a conclusion about certain popular career advice on a critique of only one part of that advice', 'bases a conclusion about how one group will respond to self-deprecation on information about how a different group responds to it', 'infers from the fact that self-deprecating humor was viewed positively that nonhumorous self-deprecation would not be viewed positively', 'ignores the possibility that non-self-deprecating humor might have been viewed even more positively than self-deprecating humor']", "label": 1 }, { "id": "train_1908", "context": "One should not intentionally misrepresent another person' s beliefs unless one' s purpose in doing so is to act in the interest of that other person.", "question": "Which one of the following actions most clearly violates the principle stated?", "answers": "['Ann told someone that Bruce thought the Apollo missions to the moon were elaborate hoaxes, even though she knew he did not think this; she did so merely to make him look ridiculous.', 'Claude told someone that Thelma believed in extraterrestrial beings, even though he knew she believed no such thing; he did so solely to keep this other person from bothering her.', 'Harvey told Josephine that he thought Josephine would someday be famous. Harvey did not really think that Josephine would ever be famous, but he said she would because he thought she would like him as a result.', 'Wanda told people that George thought Egypt is in Asia. Wanda herself knew that Egypt is in Africa, but she told people that George thought it was in Asia because she wanted people to know that George knew little about geography.']", "label": 0 }, { "id": "train_1909", "context": "Psychologist: In a study, researchers gave 100 volunteers a psychological questionnaire designed to measure their self-esteem. The researchers then asked each volunteer to rate the strength of his or her own social skills. The volunteers with the highest levels of self-esteem consistently rated themselves as having much better social skills than did the volunteers with moderate levels. This suggests that attaining an exceptionally high level of self-esteem greatly improves one' s social skills.", "question": "The psychologist's argument is most vulnerable to criticism on which of the following grounds?", "answers": "['It relies on evidence from a group of volunteers that is too small to provide any support for any inferences regarding people in general.', 'It overlooks the possibility that people with very high levels of self-esteem may tend to have a less accurate perception of the strength of their own social skills than do people with moderate levels of self-esteem.', \"It overlooks the possibility that factors other than level of self-esteem may be of much greater importance in determining the strength of one's social skills.\", 'It fails to adequately address the possibility that many of the volunteers may not have understood what the psychological questionnaire was designed to measure.']", "label": 1 }, { "id": "train_1910", "context": "Gabriella: By raising interest rates, the government has induced people to borrow less money and ttherefore to spend less, thereby slowing the country' s economy. Ivan: I disagree with your analysis. The country' s economy is tied to the global economy. Whatever happens to the global economy also happens here, and the global economy has slowed. Ttherefore, the government' s action did not cause the economy' s slowdown.", "question": "Gabriella and Ivan disagree about whether", "answers": "['the global economy has slowed', \"raising interest rates caused the country's economy to slow\", 'the economic slowdown in the country has caused people to spend less', 'raising interest rates caused a significant decrease in borrowing']", "label": 1 }, { "id": "train_1911", "context": "In 1980 there was growing concern that the protective ozone layer over the Antarctic might be decreasing and thereby allowing so much harmful ultraviolet radiation to reach the Earth that polar marine life would be damaged. Some government officials dismissed these concerns, since statistics indicated that global atmospheric ozone levels remained constant.", "question": "The relevance of the evidence cited by the government officials in support of their position would be most seriously undermined if it were true that", "answers": "['quantities of atmospheric ozone shifted away from the polar caps, correspondingly increasing ozone levels in other regions', 'decreases in the amount of atmospheric ozone over the Antarctic ice cap tend to be seasonal rather than constant', 'decreases in the amount of atmospheric ozone were of little concern before 1980', \"even where the amount of atmospheric ozone is normal, some ultraviolet light reaches the Earth's surface\"]", "label": 0 }, { "id": "train_1912", "context": "The stable functioning of a society depends upon the relatively long-term stability of the goals of its citizens. This is clear from the fact that unless the majority of individuals have a predictable and enduring set of aspirations, it will be impossible for a legislature to craft laws that will augment the satisfaction of the citizenry, and it should be obvious that a society is stable only if its laws tend to increase the happiness of its citizens.", "question": "The claim that a society is stable only if its laws tend to increase the happiness of its citizens plays which one of the following roles in the argument?", "answers": "['It is used to illustrate the general principle that the argument presupposes.', 'It is a claim that must be refuted if the conclusion is to be established.', 'It is the conclusion of the argument.', 'It helps to support the conclusion of the argument.']", "label": 3 }, { "id": "train_1913", "context": "Automobile Dealer' s Advertisement: The Highway Traffic Safety Institute reports that the PZ 1000 has the fewest injuries per accident of any car in its class. This shows that the PZ 1000 is one of the safest cars available today.", "question": "Which of the following, if true, most seriously weakens the argument in the advertisement?", "answers": "[\"In recent years many more PZ 1000's have been sold than have any other kind of car in its class.\", 'Cars in the class to which the PZ 1000 belongs are more likely to be involved in accidents than are other types of cars.', 'The Highway Traffic Safety Institute issues reports only once a year', 'The difference between the number of injuries per accident for the PZ 1000 and that for other cars in its class is quite pronounced.']", "label": 1 }, { "id": "train_1914", "context": "A poem is any work of art that exploits some of the musical characteristics of language, such as meter, rhythm, euphony, and rhyme. A novel, though it may be a work of art in language, does not usually exploit the musical characteristics of language. A symphony, though it may be a work of art that exploit the musical characteristics of sounds, rarely involves language. A limerick, though it may exploit some musical characteristics of language, is not, strictly speaking, art.", "question": "The statements above, if true, most strongly support which one of the following?", "answers": "['If a novel exploits meter and rhyme while standing as a work of art, then it is both a novel and a poem.', 'If a symphony does not exploit the musical characteristics of sound, then it is not a work of art.', 'An example of so-called blank verse, which does not rhyme, is not really a poem.', 'Limericks constitute a nonartistic type of poetry.']", "label": 0 }, { "id": "train_1915", "context": "Sheila: Health experts generally agree that smoking a tobacco product for many years is very likely to be harmful to the smoker' s health. Tim: On the contrary, smoking has no effect on health at all: although my grandfather smoked three cigars a day from the age of fourteen, he died at age ninety-six.", "question": "A major weakness of Tim's counterargument is that his counterargument", "answers": "['tacitly assumes that those health experts who are in agreement on this issue arrived at that agreement independently of one another', 'presupposes that longevity and health status are unrelated to each other in the general population', 'attempts to refute a probabilistic conclusion by claiming the existence of a single counterexample', \"describes an individual case that is explicitly discounted as an exception to the experts' conclusion\"]", "label": 2 }, { "id": "train_1916", "context": "Eating garlic reduces the levels of cholesterol and triglycerides in the blood and so helps reduce the risk of cardiovascular disease. Evidence that eating garlic reduces these levels is that a group of patients taking a garlic tablet each day for four months showed a 12 percent reduction in cholesterol and a 17 percent reduction in triglycerides; over the same period, a group of similar patients taking a medically inert tablet showed only a 2 percent reduction in triglycerides and a 3 percent reduction in cholesterol.", "question": "It would be most important to determine which one of the following in evaluating the argument?", "answers": "['whether the garlic tablets are readily available to the public', 'what the diets of the two groups were during the period', 'what effect taking the garlic tablets each day for a period of less than four months had on the levels of cholesterol and triglycerides', 'whether large amounts of garlic are well tolerated by all patients']", "label": 1 }, { "id": "train_1917", "context": "Inventor: All highly successful entrepreneurs have as their main desire the wish to leave a mark on the world. Highly successful entrepreneurs are unique in that whenever they see a solution to a problem, they implement that idea. All other people see solutions to problems but are too interested in leisure time or job security to always have the motivation to implement their ideas.", "question": "If the inventor's statements are true, which one of the following must also be true?", "answers": "['All people who invariably implement their solutions to problems have at least some interest in leisure time or job security.', 'All people whose main desire is to implement their solutions to problems leave a mark on the world.', 'The main desire of all people who implement solutions whenever they detect them is to leave a mark on the world .', 'Most people do not want to leave a mark on the world because trying to do so would reduce their leisure time or job security.']", "label": 2 }, { "id": "train_1918", "context": "On Pacific islands, a newly arrived gecko species, the house gecko, is displacing the previously established mourning gecko in urban areas, but populations of the two species are more stable in rural areas far from human settlement. The house gecko does not attack the mourning gecko, but in areas where insects congregate it prevents the mourning gecko from feeding on them.", "question": "Which of the following contributes most to an explanation of the difference between gecko populations in urban and rural areas?", "answers": "['Mourning geckos are all females and reproduce asexually, but house geckos reproduce sexually.', 'In urban areas, geckos are valued because they eat so many insects.', 'Geckos defend territories against other members of the same species.', 'In places where there are lighted buildings, insects tend to gather around the light.']", "label": 3 }, { "id": "train_1919", "context": "Columnist: A government-owned water utility has received approval to collect an additional charge on water bills and to use that additional revenue to build a dam. A member of the legislature has proposed not building the dam but instead spending the extra money from water bills to build new roads. That proposal is unacceptable.", "question": "Which one of the following principles, if valid, most helps to justify the columnist's judgment that the legislator's proposal is unacceptable?", "answers": "['Customers of a utility have a right to know how the money they pay to the utility will be used.', 'An additional charge on water bills should not be imposed unless it is approved by the legislature.', 'A water utility should not collect an additional charge unless the money collected is used for water-related expenditures.', 'Money designated for projects that benefit an entire community should not be used for projects that benefit only some members of a community.']", "label": 2 }, { "id": "train_1920", "context": "All parrots can learn to speak a few words and phrases. Not all parrots have equally pleasant dispositions, though some of those native to Australia can be counted on for a sweet temper. Almost any parrot, however, will show tremendous affection for an owner who raised the bird from a chick by hand-feeding it.", "question": "If the statements above are true, then which one of the following must be true?", "answers": "['If a parrot is not native to Australia, then it will be sweet tempered only if it is hand-fed as a chick.', 'Some parrots that can learn to speak are sweet tempered.', 'All species of pet birds that are native to Australia can be counted on for a sweet temper.', 'The sweetest-tempered parrots are those native to Australia.']", "label": 1 }, { "id": "train_1921", "context": "Professor: Politicians should only be required to abide by the strict letter of the law -- not the spirit of the law -- since they need flexibility to govern. Trading political favors for a politician' s personal financial gain is illegal under an anti-corruption statute. A state governor awarded a contract to a local construction company outside of the usual bidding process. Only the local construction company made any personal financial gain, so. . .", "question": "Which one of the following most logically completes the professor's argument?", "answers": "['the state governor should have followed the usual bidding process.', 'the state governor is guilty of violating the anti-corruption statute.', 'the state governor is not guilty of violating the anti-corruption statute.', 'the state governor broke the spirit of the law, while abiding by the strict letter of the law.']", "label": 2 }, { "id": "train_1922", "context": "Since it has become known that several of a bank' s top executives have been buying shares in their own bank, the bank' s depositors, who had been worried by rumors that the bank faced impending financial collapse, have been greatly relieved. They reason that since top executives evidently have faith in the bank' s financial soundness , those worrisome rumors must be false. They might well be overoptimistic , however, since corporate executives have sometimes bought shares in their own company in a calculated attempt to dispel negative rumors about the company' s health.", "question": " They might well be overoptimistic , however, since corporate executives have sometimes bought shares in their own company in a calculated attempt to dispel negative rumors about the company's health. In the argument given, the two boldfaced portions play which of the following roles?", "answers": "['The first is an intermediate conclusion that forms part of the reasoning called into question by the argument; the second is evidence that undermines the support for this intermediate conclusion.', 'The first summarizes the evidence used in the reasoning called into question by the argument; the second is an intermediate conclusion supported by that evidence.', 'The first is an intermediate conclusion that forms part of the reasoning called into question by the argument; the second is the main conclusion of the argument.', 'The first summarizes the evidence used in the reasoning called into question by the argument; the second states the counterevidence on which the argument relies.']", "label": 2 }, { "id": "train_1923", "context": "Patricia: During Japan' s Tokugawa period, martial arts experts known as ninjas were trained for the purposes of espionage and assassination. Yet at that time there was actually very little ninja activity in Japan, and most Japanese did not fear ninjas. Tamara: That is not true. Many wealthy Japanese during the Tokugawa period had their houses constructed with intentionally squeaky floors so that they would receive warning if a ninja were in the house.", "question": "Of the following, which one, if true, is the strongest counter Patricia can make to Tamara's objection?", "answers": "['As part of their secret training, ninjas learned to walk on squeaky floors without making a sound.', 'The fighting prowess of ninjas was exaggerated to mythic proportions in the years following the Tokugawa period.', 'Many poor Japanese during the Tokugawa period also had houses constructed with intentionally squeaky floors.', \"The wealthy made up a small portion of Japan's population during the Tokugawa period.\"]", "label": 3 }, { "id": "train_1924", "context": "Special kinds of cotton that grow fibers of green or brown have been around since the 1930s but only recently became commercially feasible when a long-fibered variety that can be spun by machine was finally bred. Since the cotton need not be dyed, processing plants avoid the expense of dyeing and the ecological hazards of getting rid of leftover dye and by-products.", "question": "Which one of the following can be properly inferred from the passage?", "answers": "['Green and brown cottons that can be spun only by hand are not commercially viable.', 'Hand-spun cotton is more ecologically safe than machine-spun cotton.', 'Short-fibered regular cottons are economically competitive with synthetic fabrics.', 'Garments made of green and brown cottons are less expensive than garments made of regular cotton.']", "label": 0 }, { "id": "train_1925", "context": "Kris: Years ago, the chemical industry claimed that technological progress cannot occur without pollution. Today, in the name of technological progress, the cellular phone industry manufactures and promotes a product that causes environmental pollution in the form of ringing phones and loud conversations in public places. Clearly, the cellular industry must be regulated, just as the chemical industry is now regulated. Terry: That' s absurd. Chemical pollution can cause physical harm, but the worst harm that cellular phones can cause is annoyance.", "question": "Terry responds to Kris's argument by doing which one of the following?", "answers": "[\"questioning the strength of the analogy on which Kris's argument is based\", 'rejecting Kris\\'s interpretation of the term \"technological progress\"', 'attacking the accuracy of the evidence about the chemical industry that Kris puts forward', \"questioning the reliability of the source of crucial information in Kris's argument\"]", "label": 0 }, { "id": "train_1926", "context": "Within the earth's core, which is iron, pressure increases with depth. Because the temperature at which iron melts increases with pressure, the inner core is solid and the outer core is molten. Physicists can determine the melting temperature of iron at any given pressure and the pressure for any given depth in the earth. Ttherefore, the actual temperature at the boundary of the earth's outer and inner cores-the melting temperature of iron there -- can be determined, since__.", "question": "Which of the following most logically completes the argument below?", "answers": [ "some of the heat from the earth's core flows to the surface of the earth", "the temperatures within the earth's inner core are higher than in the outer core", "the depth beneath the earth's surface of the boundary between the outer and inner cores is known", "nowhere in the earth's core can the temperature be measured directly" ], "label": 2 }, { "id": "train_1927", "context": "Viruses can have beneficial effects. For example, some kill more-complex microorganisms, some of which are deadly to humans. But viruses have such simple structures that replacing just a few of a beneficial virus' s several million atoms can make it deadly to humans. Clearly, since alterations of greater complexity than this are commonly produced by random mutations, any virus could easily become dangerous to humans.", "question": "If the statements above are true, than each of the following statements could also be true EXCEPT:", "answers": "['No virus that is deadly to organisms of greater complexity than itself is beneficial to humans.', 'Random mutation makes some deadly viruses beneficial to humans.', 'Some microorganisms that are more complex than viruses are beneficial to humans.', 'Some viruses that fail to kill other viruses that are deadly to humans are nevertheless beneficial to humans.']", "label": 0 }, { "id": "train_1928", "context": "Television allows us to transmit images of ourselves that propagate into space. The earliest of these transmissions have by now reached all of our neighboring star systems. None of these transmissions, so far as we know, has been recognized; we have yet to receive any messages of extraterrestrial origin. We must conclude that there is no extraterrestrial intelligence in any of our neighboring star systems.", "question": "The reasoning in the argument is questionable because the argument", "answers": "['assigns too little importance to the possibility that there is extraterrestrial intelligence beyond our neighboring star systems', 'neglects to mention that some governments have sent meticulously prepared messages and recordings on spacecraft', 'infers that there is no extraterrestrial intelligence in neighboring star systems from the lack of proof that there is', 'overlooks the immense probability that most star systems are uninhabited']", "label": 2 }, { "id": "train_1929", "context": "Medical researcher: As expected, records covering the last four years of ten major hospitals indicate that babies born prematurely were more likely to have low birth weights and to suffer from health problems than were babies not born prematurely. These records also indicate that mothers who had received adequate prenatal care were less likely to have low birth weight babies than were mothers who had received inadequate prenatal care. Adequate prenatal care, ttherefore, significantly decreases the risk of low birth weight babies.", "question": "Which one of the following, if true, most weakens the medical researcher's argument?", "answers": "['The hospital records indicate that low birth weight babies were routinely classified as having been born prematurely.', 'Mothers giving birth prematurely are routinely classified by hospitals as having received inadequate prenatal care when the record of that care is not available.', 'The hospital records indicate that many babies that are born with normal birth weights are born to mothers who had inadequate prenatal care.', 'Women who receive adequate prenatal care, are less likely to give birth prematurely than are women who do not receive adequate prenatal care.']", "label": 1 }, { "id": "train_1930", "context": "Sascha: The attempt to ban parliament' s right to pass directed-spending bills-bills that contain provisions specifically funding the favorite projects of some powerful politicians-is antidemocratic. Our nation' s constitution requires that money be drawn from our treasury only when so stipulated by laws passed by parliament, the branch of government most directly representative of the citizens. This requirement is based on the belief that exercising the power to spend public resources involves the ultimate exercise of state authority and that ttherefore __.", "question": "Which of the following most logically completes Sascha's argument?", "answers": "['only officials who are motivated by concerns for reelection should retain that power', 'exercising the power to spend public resources is in most cases-but not all-protected by the constitution', 'designating funding specifically for the favorite projects of some powerful politicians should be considered antidemocratic', 'the right to exercise such a power should belong exclusively to the branch of government most directly representative of the citizens']", "label": 3 }, { "id": "train_1931", "context": "When a group is unable to reach a consensus, group members are often accused of being stubborn, bull-headed, or unyielding. Such epithets often seem abusive, are difficult to prove, and rarely help the group reach a resolution. Those who wish to make such an accusation stick, however, should choose \"unyielding, \" because one can always appeal to the fact that the accused has not yielded; obviously if one acknowledges that a person has not yielded, then one cannot deny that the person is unyielding, at least on this issue.", "question": "Which one of the following most accurately describes the argumentative technique employed above?", "answers": "['conditionally advocating a tactic on the grounds that it results in an accusation that is less offensive than the alternatives', 'conditionally advocating a tactic on the grounds that it results in an argument that would help the group to reach a consensus on the issue in question', 'conditionally advocating a tactic on the grounds that it results in an argument for which one could not consistently accept the premise but deny the conclusion', 'rejecting a tactic on the grounds that it constitutes an attack on the character of a person and has no substance in fact']", "label": 2 }, { "id": "train_1932", "context": " In countries where automobile insurance includes compensation for whiplash injuries sustained in automobile accidents, reports of having suffered such injuries are twice as frequent as they are in countries where whiplash is not covered. Some commentators have argued, correctly, that since there is presently no objective test for whiplash, spurious reports of whiplash injuries cannot be readily identified . These commentators are, however, wrong to draw the further conclusion that in the countries with the higher rates of reported whiplash injuries, half of the reported cases are spurious: clearly, in countries where automobile insurance does not include compensation for whiplash, people often have little incentive to report whiplash injuries that they actually have suffered.", "question": "In the argument given, the two boldfaced portions play which of the following roles?", "answers": "['The first is a claim that has been used to support a conclusion that the argument accepts; the second is that conclusion.', 'The first is a finding whose accuracy is evaluated in the argument; the second is an intermediate conclusion drawn to support the judgment reached by the argument on the accuracy of that finding.', 'The first is a claim that the argument disputes; the second is a narrower claim that the argument accepts.', 'The first is a finding whose implications are at issue in the argument; the second is an intermediate conclusion that has been used to support a conclusion that the argument criticizes.']", "label": 3 }, { "id": "train_1933", "context": "Scientist: Some colonies of bacteria produce antibiotic molecules called phenazines, which they use to fend off other bacteria. We hypothesize that phenazines also serve as molecular pipelines that give interior bacteria access to essential nutrients in the environment surrounding the colony.", "question": "Which one of the following, if true, provides the most support for the scientist's hypothesis?", "answers": "['Within bacteria colonies that produce phenazines, interior bacteria are more likely to die than are bacteria along the edges.', 'When bacteria colonies that do not produce phenazines are buried in nutrient-rich soil, they grow as quickly as colonies that do produce phenazines.', 'Bacteria colonies that do not produce phenazines form wrinkled surfaces, thus increasing the number of bacteria that are in direct contact with the surrounding environment.', 'The rate at which a bacteria colony produces phenazines is determined by the number of foreign bacteria in the environment immediately surrounding the colony.']", "label": 2 }, { "id": "train_1934", "context": "A safety report indicates that, on average, traffic fatalities decline by about 7 percent in those areas in which strict laws requiring drivers and passengers to wear seat belts have been passed. In a certain city, seat belt laws have been in effect for two years, but the city' s public safety records show that the number of traffic deaths per year has remained the same.", "question": "Which one of the following, if true, does NOT help resolve the apparent discrepancy between the safety report and the city's public safety records?", "answers": "[\"Because the city's seat belt laws have been so rarely enforced, few drivers in the city have complied with them.\", 'Two years ago speed limits in the city were increased by as much as 15 kph (9 mph).', 'In the last two years, most of the people killed in car accidents in the city were not wearing seat belts.', 'In the time since the seat belt laws were passed, the city has experienced a higher than average increase in automobile traffic.']", "label": 2 }, { "id": "train_1935", "context": "There are two kinds of horror stories: those that describe a mad scientist' s experiments and those that describe a monstrous beast. In some horror stories about monstrous beasts, the monster symbolizes a psychological disturbance in the protagonist. Horror stories about mad scientists, on the other hand, typically express the author' s feeling that scientific knowledge alone is not enough to guide human endeavor. However, despite these differences, both kinds of horror stories share two features: they describe violations of the laws of nature and they are intended to produce dread in the reader.", "question": "If the statements above are true, which one of the following would also have to be true?", "answers": "[\"Most stories about mad scientists express the author's antiscientific views.\", 'Some stories that employ symbolism describe violations of the laws of nature.', 'Horror stories of any kind usually describe characters who are psychologically disturbed.', 'All descriptions of monstrous beasts describe violations of the laws of nature.']", "label": 1 }, { "id": "train_1936", "context": "In a transportation company, a certain syndrome often attributed to stress by medical experts afflicts a significantly higher percentage of workers in Department F than in any other department. We can conclude, ttherefore, that the work done in Department F subjects workers to higher stress levels than does the work in the other departments in the company.", "question": "Which one of the following, if true, most helps to support the argrument?", "answers": "['Some experts believe that the syndrome can be caused by various factors, only one of which is high stress.', 'Job-related stress has been the most frequently cited cause for dissatisfaction among workers at the company.', 'Many workers who transfer into Department F from elsewhere in the company soon begin to develop the syndrome.', 'Department F has more employees than any other department in the company.']", "label": 2 }, { "id": "train_1937", "context": "Car accidents that happen at excessive speeds are more likely than other car accidents to result in fatalities. However, the majority of accidents that happen at excessive speeds do not result in fatalities. Ttherefore, there is no need to enact laws that address car accidents at excessive speeds as being distinct from other kinds of car accidents.", "question": "The pattern of flawed reasoning displayed in the argument above most closely resembles that in which one of the following?", "answers": "['Foods rich in cholesterol and fat pose a serious health threat to most people. However, many people are reluctant to give up eating foods that they greatly enjoy. Ttherefore, people who refuse to give up rich foods need to spend more time exercising than do other people.', 'Most physicians recommend a balanced diet for those who want to remain in good health. Yet many people find that nontraditional dietary regimens such as \"fad diets\" do their health no serious harm. Ttherefore, there is no need for everyone to avoid nontraditional dietary regimens.', 'Many serious health problems are the result of dietary disorders. Yet these disorders are often brought about by psychological factors. Hence people suffering from serious health problems should undergo psychological evaluation.', 'Overweight people are at a higher risk of developing heart disease than other people. However, more than half of all overweight people never develop heart disease. Hence it is unnecessary for physicians to be more careful to emphasize the danger of heart disease to their overweight patients than to their other patients.']", "label": 3 }, { "id": "train_1938", "context": "Proponents of the recently introduced tax on sales of new luxury boats had argued that a tax of this sort would be an equitable way to increase government revenue because the admittedly heavy tax burden would fall only on wealthy people and neither they nor anyone else would suffer any economic hardship. In fact, however, 20 percent of the workers employed by manufacturers of luxury boats have lost their jobs as a direct result of this tax.", "question": "The information given, if true, most strongly supports which of the following?", "answers": "['Before the tax was instituted, luxury boats were largely bought by people who were not wealthy.', 'The market for luxury boats would have collapsed even if the new tax on luxury boats had been lower.', 'Taxes can be equitable only if their burden is evenly distributed over the entire population.', \"The new tax would produce a net gain in tax revenue for the government only if the yearly total revenue that it generates exceeds the total of any yearly tax-revenue decrease resulting from the workers' loss of jobs.\"]", "label": 3 }, { "id": "train_1939", "context": "Electronic computer chips made of tiny silicon wafers now regularly contain millions of electronic switches. Unfortunately, electronic switches that are this small cannot withstand intense radiation. Micro-Mechanics plans to produce a chip that, because it uses only microscopic mechanical switches, will be invulnerable to radiation damage. The switches will, however, be slower than electronic switches and the chip will contain only 12, 000 switches.", "question": "For there to be a market for Micro-Mechanics' chip as a result of the apparent advantage described above, each of the following would have to be true EXCEPT:", "answers": "['There will be applications for computer chips in environments where the chips may have to survive intense radiation', 'Some devices in which computer chips will be used will have other components that will be able to function during or after exposure to radiation', 'There will be applications in which the speed attainable by an electronic switch is not essential', 'Manufacturers are able to protect electronic computer chips against exposure to intense radiation, where this protection is necessary']", "label": 3 }, { "id": "train_1940", "context": "Mayor: Our city faces a difficult environmental problem caused by the enormous amount of garbage that we must dispose of. Although new recycling projects could greatly reduce this amount, these projects would actually be counterproductive to the goal of minimizing the overall amount of environmental damage.", "question": "Which one of the following, if true, would most help to resolve the apparent inconsistency in the mayor's claims about new recycling projects?", "answers": "[\"More recycling would give industries in the mayor's city a greater incentive to use recycled materials in their manufacturing processes.\", 'The vehicles that pick up materials for recycling create less pollution than would be caused by incinerating those materials.', 'The great costs of new recycling projects would prevent other pollution-reducing projects from being undertaken.', 'People who recycle feel less justified in consuming more than they need than do people who do not recycle.']", "label": 2 }, { "id": "train_1941", "context": "Professor: Economists argue that buying lottery tickets is an unwise use of resources, because the average payoff for the tickets sold in a lottery is much lower than the cost of a ticket. But this reasoning is faulty. The average amount paid out on individual insurance policies is much lower than the average cost of a policy, yet nobody would argue that purchasing insurance is an unwise use of resources.", "question": "Which one of the following, if true, most weakens the professor's argument?", "answers": "[\"The protection against loss that insurance provides is more important to one's well-being than is the possibility of a windfall gain.\", 'Individuals spend, on average, much more on insurance than on lottery tickets.', \"Taking small financial risks can often greatly increase one's chances of obtaining much larger benefits.\", 'In general, the odds of winning the grand prize in a lottery are significantly lower than the odds of collecting a settlement from a typical insurance policy.']", "label": 0 }, { "id": "train_1942", "context": "High school class president: Honor students are more responsible and law-abiding than other students and, ttherefore, cause far fewer problems at night. Ttherefore, the city should not consider changing the curfew from 11: 00 p. m. to 10: 00 p. m. because that will have the undesirable effect of forcing some responsible and law-abiding students to break the law. Mayor: All students can obey the curfew if they wish, so it is not true to say that changing the curfew would be the cause of such illegal behavior.", "question": "The thing at issue between the high school class president and the mayor is whether", "answers": "['non-honor students will obey the proposed curfew change', 'changing the curfew would cause some honor students to break the law', 'honor students are more responsible and law-abiding than other students', 'all students hope the curfew will not be changed from 11:00 p. m. to 10:00 p. m.']", "label": 1 }, { "id": "train_1943", "context": "In Europe, schoolchildren devote time during each school day to calisthenics. North American schools rarely offer a daily calisthenics program. Tests prove that North American children are weaker, slower, and shorter-winded than European children. We must conclude that North American children can be made physically fit only if they participate in school calisthenics on a daily basis.", "question": "Which one of the following is assumed in the passage?", "answers": "[\"School calisthenics are an indispensable factor in European children's superior physical fitness.\", 'All children can be made physically fit by daily calisthenics.', 'Superior physical fitness produces superior health.', 'North American children can learn to eat a more nutritious diet as well as to exercise daily.']", "label": 0 }, { "id": "train_1944", "context": "Some critics argue that an opera' s stage directions are never reflected in its music. Many comic scenes in Mozart' s operas, however, open with violin phrases that sound like the squeaking of changing scenery. Clearly Mozart intended the music to echo the sounds occurring while stage directions are carried out. Hence, a change of scenery -- the most basic and frequent stage direction -- can be reflected in the music, which means that other operatic stage directions can be as well.", "question": "In the argument, the statement that many comic scenes in Mozart's operas open with violin phrases that sound like the squeaking of changing scenery is offered in support of the claim that", "answers": [ "a variety of stage directions can be reflected in an opera's music", "the most frequent relation between an opera's music and its stage directions is one of musical imitation of the sounds that occur when a direction is carried out", "a change of scenery is the stage direction most frequently reflected in an opera's music", "an opera's stage directions are never reflected in its music" ], "label": 0 }, { "id": "train_1945", "context": "Police commissioner: Last year our city experienced a 15 percent decrease in the rate of violent crime. At the beginning of that year a new mandatory sentencing law was enacted, which requires that all violent criminals serve time in prison. Since no other major policy changes were made last year, the drop in the crime rate must have been due to the new mandatory sentencing law.", "question": "Which one of the following, if true, most seriously weakens the police commissioner's argument?", "answers": "[\"Last year, the city's overall crime rate decreased by only 5 percent.\", 'At the beginning of last year, the police department\\'s definition of \"violent crime\" was broadened to include 2 crimes not previously classified as \"violent. \"', 'The city enacted a policy 2 years ago requiring that 100 new police officers be hired in each of the 3 subsequent years.', 'Studies of many other cities have shown a correlation between improving economic conditions and decreased crime rates.']", "label": 2 }, { "id": "train_1946", "context": "Only people who are willing to compromise should undergo mediation to resolve their conflicts. Actual litigation should be pursued only when one is sure that one' s position is correct. People whose conflicts are based on ideology are unwilling to compromise.", "question": "If the statements above are true, then which one of the following must be true?", "answers": "['People whose conflicts are not based on ideology should attempt to resolve their conflicts by means of litigation.', 'People whose conflicts are based on ideology are not people who should undergo mediation to resolve their conflicts.', 'People whose conflicts are based on ideology are not always sure that their positions are correct.', 'People who are sure of the correctness of their positions are not people who should undergo mediation to resolve their conflicts.']", "label": 1 }, { "id": "train_1947", "context": "After all, most doctors do eat meat, and who knows more about health than doctors do? Which one of the following is a flaw in Smith' s reasoning?", "question": "Smith: Meat in the diet is healthy, despite what some people say.", "answers": "[\"attacking the opponents' motives instead of their argument\", 'taking for granted that experts do not act counter to what, according to their expertise, is in their best interest', 'generalizing on the basis of a sample consisting of atypical cases', 'assuming at the outset what the argument claims to establish through reasoning']", "label": 1 }, { "id": "train_1948", "context": "Ticks attach themselves to host animals to feed. Having fed to capacity, and not before then, the ticks drop off their host. Deer ticks feeding off whitefooted mice invariably drop off their hosts between noon and sunset, regardless of time of attachment. White-footed mice are strictly nocturnal animals that spend all daytime hours in their underground nests.", "question": "Which one of the following conclusions can be properly drawn from the statements above?", "answers": "['Deer ticks all attach themselves to whitefooted mice during the same part of the day, regardless of day of attachment.', \"Deer ticks that feed off white-footed mice drop off their hosts in the hosts' nests.\", 'White-footed mice to which deer ticks have attached themselves are not aware of the ticks.', 'Deer ticks sometimes drop off their hosts without having fed at all.']", "label": 1 }, { "id": "train_1949", "context": "Linda: During the summer months, people normally take vacations. Ttherefore, during the summer months, the local economy suffers since local residents spend their money at their vacation destinations. Billy: Why do you think that the local economy would be suffering? During the summer months, tourists who come to our city actually spend lots of money at local businesses, attractions, and restaurants.", "question": "Which of the following most correctly explains how Billy's reply is related to Linda's argument?", "answers": "['The reply offers another reason why the local economy suffers during the summer months that is unrelated to vacations.', 'The reply shows that Linda fails to realize that people often are forced by their employers to take their vacations in the summer.', \"The reply argues that Linda's conclusion is correct, but not because of the reasons that Linda claims.\", \"The reply offers an additional factor to consider that weakens Linda's conclusion.\"]", "label": 3 }, { "id": "train_1950", "context": "Rossi: It is undemocratic for people to live under a government in which their interests are not represented. So children should have the right to vote, since sometimes the interests of children are different from those of their parents. Smith: Granted, children' s interests are not always the same as their parents' ; governmental deficits incurred by their parents' generation will later affect their own generation' s standard of living. But even if children are told about the issues affecting them, which is not generally the case, their conceptions of what can or should be done are too simple, and their time horizons are radically different from those of adults, so we cannot give them the responsibility of voting.", "question": "Which one of the following most accurately describes Rossi's argument?", "answers": "['It makes an appeal to a general principle.', 'It employs a term on two different occasions in different senses.', 'It relies on evaluating the predictable consequences of a proposal.', 'It substitutes description for giving a rationale for a policy.']", "label": 0 }, { "id": "train_1951", "context": "Native speakers perceive sentences of their own language as sequences of separate words. But this perception is an illusion. This is shown by the fact that travelers who do not know a local language hear an unintelligible, uninterrupted stream of sound, not sentences with distinct words.", "question": "Which one of the following is an assumption on which the argument depends?", "answers": "[\"Native speakers' perceptions of their own language are not more accurate than are the perceptions of persons who do not know that language.\", 'Accomplished non-native speakers of a language do not perceive sentences as streams of sound.', 'It is impossible to understand sentences if they are in fact uninterrupted streams of sound.', 'People pay less close attention to the way their own language sounds than they do to the way an unfamiliar language sounds.']", "label": 0 }, { "id": "train_1952", "context": "Health officials now recommend that people reduce their intake of foods that are high in cholesterol, such as red meat. The recent decline in the total consumption of beef indicates that many people are following this recommendation. But restaurants specializing in steak are flourishing despite an overall decline in the restaurant industry. So clearly there still are a lot of people completely ignoring the health recommendation.", "question": "The argument is vulnerable to criticism on which one of the following grounds?", "answers": "['It presupposes that the popularity of restaurants that specialize in steaks is a result of a decrease in the price of beef.', 'It assumes without warrant that people who eat steak at steak restaurants do not need to reduce their intake of foods that are high in cholesterol.', 'It neglects to consider whether restaurants that specialize in steak try to attract customers by offering steak dinners at low prices.', 'It fails to consider whether the people who patronize steak restaurants have heeded the health officials by reducing their cholesterol intake in their at-home diets.']", "label": 3 }, { "id": "train_1953", "context": "Grow-Again ointment is a proven treatment for reversing male hereditary baldness. Five drops daily is the recommended dose, and exceeding this quantity does not increase the product' s effectiveness. Ttherefore, offering a manufacturer' s rebate on the purchase price of Grow-Again will not increase sales and consequently would be unprofitable for the manufacturer.", "question": "Which one of the following, if true, would most strengthen the argument?", "answers": "['When using an ointment, people tend to believe that applying it in greater quantities can make it more effective.', 'Grow-Again is a product whose per-unit manufacturing cost does not fall significantly when the product is produced in large quantities.', 'The rebate, if offered, would not attract purchasers who otherwise might not use Grow-Again.', 'Grow-Again is more effective on some of the men who use it than it is on others.']", "label": 2 }, { "id": "train_1954", "context": "As part of a survey, approximately 10, 000 randomly selected individuals were telephoned and asked a number of questions about their income and savings. Those conducting the survey observed that the older the person being queried, the more likely it was that he or she would refuse to answer any of the questions. This finding clearly demonstrates that, in general, people are more willing when they are younger than when they are older to reveal personal financial information to strangers over the telephone.", "question": "The argument above is vulnerable to criticism on the grounds that the argument", "answers": "['assumes without warrant that age is the main determinant of personal income and savings levels', 'offers no evidence that the individuals queried would have responded differently had they been asked the same questions in years prior to the survey', 'provides no reason to believe that what is true of a given age group in general is also true of all individuals within that age group', 'assumes from the outset what it purports to establish on the basis of a body of statistical evidence']", "label": 1 }, { "id": "train_1955", "context": "As part of major renovations to Flowertown' s Main Street train station, consultants to the train company proposed moving the station' s entrance from its current valuable Main Street location to a low-rent adjoining side street and then leasing the high-rent entrance space to retail businesses. In that way, the train company could easily pay for those and all other proposed renovations without negative impact on its tight budget.", "question": "Which of the following, if true, would most strongly support the consultants' proposal?", "answers": "[\"The high-rent block of Flowertown's Main Street includes several buildings whose owners currently seek to replace long-standing tenants lost in recent months.\", 'Ridership on Flowertown trains declined only slightly from 1970 to 1985 while other train companies lost large numbers of commuters.', \"A reliable survey of Flowertown's commuters showed that virtually none of them would use the train any less frequently if the station's entrance were moved.\", \"If the station's entrance were moved, the train company would need to begin costly renovations to its Main Street entrance space.\"]", "label": 2 }, { "id": "train_1956", "context": "Criminologist: A judicial system that tries and punishes criminals without delay is an effective deterrent to violent crime. Long, drawn-out trials and successful legal maneuvering may add to criminals' feelings of invulnerability. But if potential violent criminals know that being caught means prompt punishment, they will hesitate to break the law.", "question": "Which one of the following, if true, would most seriously weaken the criminologist's argument?", "answers": "['It is in the nature of violent crime that it is not premeditated.', 'Everyone accused of a crime has the right to a trial.', 'About one-fourth of all suspects first arrested for a crime are actually innocent.', 'Many violent crimes are committed by first- time offenders.']", "label": 0 }, { "id": "train_1957", "context": "M: The Greek alphabet must have been invented by some individual who knew the Phoenician writing system and who wanted to have some way of recording Homeric epics and thereby preserving expressions of a highly developed tradition of oral poetry. P: Your hypothesis is laughable! What would have been the point of such a person' s writing Homeric epics down? Surely a person who knew them well enough to write them down would not need to read them; and no one else could read them, according to your hypothesis.", "question": "Which one of the following is an argumentative strategy that P uses in responding to M?", "answers": "['providing an alternative interpretation of evidence put forward by M', \"attempting to undermine M's hypothesis by making it appear absurd\", \"disagreeing with M's thesis without attempting to refute it\", \"challenging M's knowledge of the Phoenician writing system\"]", "label": 1 }, { "id": "train_1958", "context": "Educator: Only those students who are genuinely curious about a topic can successfully learn about that topic. They find the satisfaction of their curiosity intrinsically gratifying, and appreciate the inherent rewards of the learning process itself. However, almost no child enters the classroom with sufficient curiosity to learn successfully all that the teacher must instill. A teacher' s job, ttherefore, __.", "question": "Which one of the following most logically completes the educator's argument?", "answers": "['requires for the fulfillment of its goals the stimulation as well as the satisfaction of curiosity', 'becomes easier if students realize that some learning is not necessarily enjoyable', \"is facilitated by students' taking responsibility for their own learning\", 'necessitates the creative use of rewards that are not inherent in the learning process itself']", "label": 0 }, { "id": "train_1959", "context": "Some argue that laws are instituted at least in part to help establish a particular moral fabric in society. But the primary function of law is surely to help order society so that its institutions, organizations, and citizenry can work together harmoniously, regardless of any further moral aims of the law. Indeed, the highest courts have on occasion treated moral beliefs based on conscience or religious faith as grounds for making exceptions in the application of laws.", "question": "The statements above, if true, most strongly support which one of the following?", "answers": "[\"The best way to promote cooperation among a society's institutions, organizations, and citizenry is to institute order in that society by means of law.\", 'The law has as one of its functions the ordering of society but is devoid of moral aims.', 'The way a society is ordered by law should not reflect any moral convictions about the way society ought to be ordered.', 'The manner in which laws are applied sometimes takes into account the beliefs of the people governed by those laws.']", "label": 3 }, { "id": "train_1960", "context": "Animal feed should not include genetically modified plants. A study found that laboratory rats fed genetically modified potatoes for 30 days tended to develop intestinal deformities and a weakened immune system, whereas rats fed a normal diet of foods that were not genetically modified did not develop these problems.", "question": "Which one of the following, if true, most weakens the argument?", "answers": "['The researchers conducting the study were unable to explain how the genetic modifications of the potatoes would have caused the intestinal deformities or a weakened immune system in the rats.', 'Potatoes are not normally a part of the diet of laboratory rats.', 'The rats tended to eat more of the genetically modified potatoes at the beginning of the 30 days than they did toward the end of the 30 days.', 'Intestinal deformities at birth are not uncommon among rats bred in laboratory conditions.']", "label": 1 }, { "id": "train_1961", "context": "The human emotional response presents an apparent paradox. People believe that they can be genuinely moved only by those things and events that they believe to be actual, yet they have genuine emotional responses to what they know to be fictional.", "question": "Which one of the following situations most closely conforms to the principle cited above?", "answers": "['Fred was watching a horror movie. Although he did not expect to be bothered by make-believe monsters, he nonetheless felt frightened when they appeared on the screen.', 'Tamara was reading Hamlet. Although she knew that it was a work of fiction, she still made statements such as \"Hamlet was born in Denmark\" and \"Hamlet was a prince. \"', 'Raheem thought that his sister was in the hospital. Although he was mistaken, he was nevertheless genuinely worried when he believed she was there.', \"Jeremy was upset by the actions that a writer attributed to a secret organization, although he considered it unlikely that the writer's account was accurate.\"]", "label": 0 }, { "id": "train_1962", "context": "Although 90 percent of the population believes itself to be well informed about health care, only 20 percent knows enough about DNA to understand a news story about DNA. So apparently at least 80 percent of the population does not know enough about medical concepts to make well-informed personal medical choices or to make good public policy decisions about health care.", "question": "The argument's reasoning is questionable because the argument fails to demonstrate that", "answers": "[\"one's being able to make well-informed personal medical choices ensures that one makes good public policy decisions about health care\", 'since 90 percent of the population believes itself to be well informed about health care, at least 70 percent of the population is mistaken in that belief', 'more than 20 percent of the population needs to be well informed about health care for good public policy decisions about health care to be made', 'an understanding of DNA is essential to making well-informed personal medical choices or to making good public policy decisions about health care']", "label": 3 }, { "id": "train_1963", "context": "For the past several years, a certain technology has been widely used to transmit data among networked computers. Recently two data transmission companies, Aptron and Gammatech, have each developed separate systems that allow network data transmission at rates ten times faster than the current technology allows. Although the systems are similarly priced and are equally easy to use, Aptron' s product is likely to dominate the market, because __.", "question": "Which of the following most logically completes the passage?", "answers": "['unlike many data transmission companies, Aptron and Gammatech develop computers in addition to data transmission systems', \"it is easier for users of the current data transmission technology to switch to Aptron's product than to Gammatech's\", 'Gammatech has been in the business of designing data transmission systems for several years more than Aptron has', \"it is much more likely that Gammatech's system will be expandable to meet future needs\"]", "label": 1 }, { "id": "train_1964", "context": "We are taught that pedestrians should cross the street at a corner and that jaywalking, in the sense of crossing other than at a corner, is dangerous and illegal. It also seems true that drivers anticipate people crossing at corners more than drivers anticipate people crossing elsewhere. Thus we might infer that crossing at a corner is safer than jaywalking. Nevertheless, statistics show that more pedestrians die crossing at corners than while jaywalking.", "question": "Which one of the following, if true, most helps to explain the statistical claim cited above?", "answers": "['Drivers are often unfamiliar with the laws concerning jaywalking.', 'Good drivers anticipate jaywalkers as much as they anticipate pedestrians crossing at corners.', 'Traffic laws in most locations state that the pedestrian always has the right of way, whether or not the pedestrian is crossing at a corner.', 'Far more pedestrians cross at corners than jaywalk.']", "label": 3 }, { "id": "train_1965", "context": "To qualify as a medical specialist, one must usually graduate from a university, then complete approximately four years of medical school, followed by a residency of two to six years in one' s specialty. Finally, a physician who desires to become a recognized specialist must complete an evaluation program directed by a medical specialty board. Ttherefore, anyone who has qualified as a recognized medical specialist is competent to practice in his or her specialty.", "question": "Which one of the following is an assumption on which the argument depends?", "answers": "['People who are not highly motivated will not complete the demanding course of study and examination required to become qualified as a recognized medical specialist.', 'Only the most talented people will successfully complete the rigorous course of study necessary for qualification as a recognized medical specialist.', 'Usually, six to ten years of medical training beyond a university degree is necessary to render someone competent to practice in his or her medical specialty.', 'No one incompetent to practice a particular specialty completes the evaluation program for that specialty.']", "label": 3 }, { "id": "train_1966", "context": "The ends of modern centuries have been greeted with both apocalyptic anxieties and utopian fantasies. It is not surprising that both reactions have consistently proven to be misplaced. After all, the precise time when a century happens to end cannot have any special significance, since the Gregorian calendar, though widely used, is only one among many that people have devised.", "question": "Which one of the following, if true, could be substituted for the reason cited above while still preserving the force of the argument?", "answers": "['The firm expectation that something extraordinary is about to happen can make people behave in a manner that makes it less likely that something extraordinary will happen.', 'Since a century far exceeds the normal human life span, people do not live long enough to learn from mistakes that they themselves made one hundred years before.', 'The number system based on the number ten, in the absence of which one hundred years would not have the appearance of being a significant period of time, is by no means the only one that people have created.', 'It is logically impossible for both reactions to be correct at the same time.']", "label": 2 }, { "id": "train_1967", "context": "Reviewer: Many historians claim, in their own treatment of subject matter, to be as little affected as any natural scientist by moral or aesthetic preconceptions. But we clearly cannot accept these proclamations of objectivity, for it is easy to find instances of false historical explanations embodying the ideological and other prejudices of their authors.", "question": "The reviewer's reasoning is most vulnerable to criticism on the grounds that it", "answers": "['takes for granted that the model of objectivity offered by the natural sciences should apply in other fields', 'takes for granted that some historical work that embodies prejudices is written by historians who purport to be objective', 'offers evidence that undermines rather than supports the conclusion it reaches', 'fails to recognize that not all historical explanations embodying ideologies are false']", "label": 1 }, { "id": "train_1968", "context": "In order to reduce dependence on imported oil, the government of Jalica has imposed minimum fuel-efficiency requirements on all new cars, beginning this year. The more fuel-efficient a car, the less pollution it produces per mile driven. As Jalicans replace their old cars with cars that meet the new requirements, annual pollution from car traffic is likely to decrease in Jalica.", "question": "Which of the following, if true, most seriously weakens the argument?", "answers": "['Jalicans who get cars that are more fuel-efficient tend to do more driving than before.', 'In Jalica, domestically produced oil is more expensive than imported oil.', 'The new regulation requires no change in the chemical formulation of fuel for cars in Jalica.', 'The Jalican government did not intend the new fuel-efficiency requirement to be a pollution reduction measure.']", "label": 0 }, { "id": "train_1969", "context": "Benson: In order to maintain the quality of life in our city, we need to restrict growth. That is why I support the new zoning regulations. Willett: I had heard such arguments ten years ago, and again five years ago. Each time the city council was justified in deciding not to restrict growth. Since there is nothing new in this idea of restricting growth, I oppose the regulations.", "question": "Which one of the following most accurately describes a way in which Willett's reasoning is questionable?", "answers": "['It fails to take into account the variety of factors that contribute to the quality of life in a city.', \"It is based on attacking Benson personally rather than responding to Benson's reasoning.\", 'It overlooks the possibility that the city council of ten years ago was poorly qualified to decide on zoning regulations.', 'It ignores the possibility that new reasons for restricting growth have arisen in the past five years.']", "label": 3 }, { "id": "train_1970", "context": "Many banks have a drive-through automated teller machine, built into the wall flanking the driver' s side of a narrow aisle. Even though blind people obviously do not drive, these machines nonetheless invariably feature Braille -- the system of raised dots used by the blind for reading and writing -- in addition to the standard letters and numerals on their control panels.", "question": "Which of the following, if true, does the most to explain the situation described?", "answers": "['Moderately visually impaired people who are still allowed to drive typically do not use Braille to read or write.', 'Blind people can often read Braille just as quickly and accurately as sighted people can read regular writing.', 'The embossing technique used to form the raised dots does not typically add much to the overall cost of the automated teller machine.', 'In order to maximize profits per unit, manufacturers produce only a single model of automated teller machine.']", "label": 3 }, { "id": "train_1971", "context": "The average cost of groceries will rise again next month. Consequently, butter and eggs can be expected to cost more next month.", "question": "The flawed reasoning in the argument above most closely parallels the reasoning in which one of the following?", "answers": "['The average amount of time spent by people younger than 20 in watching television has recently risen rapidly. Ttherefore, the amount of time fourth graders spend watching television must have risen recently.', 'Since sugar is a major ingredient in ice cream, the price of ice cream increases whenever the price of sugar increases. The price of sugar is expected to increase next month. Ttherefore, the price of ice cream can be expected to increase next month.', 'Real estate prices go down when the population of those from 20 to 30 years old declines, and the number in that age group will decrease over the next decade. Ttherefore, real estate prices will go down over that period.', 'The price of gasoline has been rising each month for the past year. Ttherefore, we can expect to pay more for gasoline next month.']", "label": 0 }, { "id": "train_1972", "context": "When a caterpillar emerges from the egg on a tree branch, it immediately climbs upward until it finds a leaf bud to eat. Biologists thought that this behavior displayed an innate tendency to move in the direction opposite to the pull of gravity. In a recent experiment, a strong light source was placed at the bottom of a tree, and caterpillars, after hatching, climbed downward.", "question": "Which of the following hypotheses is best supported by the statements given?", "answers": "['Newly hatched caterpillars move toward the leaf bud nearest to them.', 'Caterpillars have an innate tendency to move in the direction of gravity.', 'Newly hatched caterpillars move towards the strongest light source in the environment.', 'Newly hatched caterpillars are unable to see in the dark.']", "label": 2 }, { "id": "train_1973", "context": "The vomeronasal organ (VNO) is found inside the noses of various animals. While its structural development and function are clearer in other animals, most humans have a VNO that is detectable, though only microscopically. When researchers have been able to stimulate VNO cells in humans, the subjects have reported experiencing subtle smell sensations. It seems, then, that the VNO, though not completely understood, is a functioning sensory organ in most humans.", "question": "Which one of the following, if true, most weakens the argument?", "answers": "['Relative to its occurrence in certain other animals, the human VNO appears to be anatomically rudimentary and underdeveloped.', 'For many animal species, the VNO is thought to subtly enhance the sense of smell.', 'Certain chemicals that play a leading role in the way the VNO functions in animals in which it is highly developed do not appear to play a role in its functioning in humans.', \"It is not known whether the researchers succeeded in stimulating only VNO cells in the human subjects' noses.\"]", "label": 3 }, { "id": "train_1974", "context": "Despite a steady decrease in the average number of hours worked per person per week, the share of the population that reads a daily newspaper has declined greatly in the past 20 years. But the percentage of the population that watches television daily has shown a similarly dramatic increase over the same period. Clearly, increased television viewing has caused a simultaneous decline in newspaper reading.", "question": "Which one of the following, if true, would be most damaging to the explanation given above for the decline in newspaper reading?", "answers": "['Of those members of the population who do not watch television, the percentage who read a newspaper every day has also shown a dramatic decrease.', 'The time people spend with the books and newspapers they read has increased, on average, from 1 to 3 hours per week in the past 20 years.', 'People who spend large amounts of time each day watching television are less able to process and remember printed information than are those who do not watch television.', 'A typical television set is on 6 hours a day, down from an average of 6 1/2 hours a day 5 years ago.']", "label": 0 }, { "id": "train_1975", "context": "Spokesperson: The major school lunch vendors recently agreed to stop selling high-calorie beverages in elementary and middle schools because studies show that children of ages 7 to 8 who substitute one low-calorie beverage for one high-calorie soft drink in their daily diets will, on average, weigh 20 pounds less than they would have by the time they reach high school. Since only low-calorie beverages will be sold in schools, within six to eight years, we can expect to see a reduction in the percentage of overweight high-school children.", "question": "The spokesperson assumes which of the following to be true?", "answers": "['The companies will resume selling high-calorie beverages to schools in eight years.', 'All low-calorie beverages are better for children than any high-calorie beverages.', 'Elementary and middle school students who used to buy high-calorie soft drinks at school will not bring them to school or drink extra high-calorie beverages at home as a substitute.', \"Those children whose parents are concerned about healthy weight maintenance already limit their children's consumption of soft drinks.\"]", "label": 2 }, { "id": "train_1976", "context": "Dried parsley should never be used in cooking, for it is far less tasty and healthful than fresh parsley is.", "question": "Which one of the following principles, if valid, most clearly helps to justify the argument above?", "answers": "['Ingredients that should never be used in cooking are generally neither tasty nor healthful.', 'Parsley that is not both tasty and healthful should never be used in cooking.', 'Fresh ingredients should be used in cooking whenever possible.', 'Only the tastiest ingredients should ever be used in cooking.']", "label": 3 }, { "id": "train_1977", "context": "In recent years, networks of fiber-optic cable have been replacing electrical wire for transmitting telecommunications signals. Signals running through fiber-optic cables deteriorate, and so the signals must be run through a piece of equipment called a regenerator before being transmitted farther. Light-Line is the leading manufacturer of signal regenerators. Ttherefore, Light-Line is one of the companies that will most benefit if new long-distance fiber-optic telecommunications networks are constructed.", "question": "Which of the following, if true, casts the most doubt on the prediction about Light-Line's prospects?", "answers": "['Several manufacturers are developing regenerator technologies that will allow signals to be transmitted many times farther than at present before requiring regeneration.', 'The volume of signals that a fiber-optic cable can carry is several thousand times greater than the volume that can be carried by an electrical wire of similar size.', 'Telecommunications signals sent along electrical wires can travel much farther than signals transmitted through fiber-optic before needing to be regenerated.', 'Expanding its production of regenerators will require Light-Line to spend proportionately more on labor and materials.']", "label": 0 }, { "id": "train_1978", "context": "The number of calories in a gram of refined cane sugar is the same as in an equal amount of fructose, the natural sugar found in fruits and vegetables. Ttherefore, a piece of candy made with a given amount of refined cane sugar is no higher in calories than a piece of fruit that contains an equal amount of fructose.", "question": "The reasoning in the argument is flawed because the argument", "answers": "['presupposes what it sets out to establish, that fruit does not differ from sugar-based candy in the number of calories each contains', 'fails to consider the possibility that fruit might contain noncaloric nutrients that candy does not contain', 'overlooks the possibility that sugar might not be the only calorie-containing ingredient in candy or fruit', 'presupposes that all candy is made with similar amounts of sugar']", "label": 2 }, { "id": "train_1979", "context": "Herons must eat large numbers of fish to survive. Ttherefore, according to naturalists, when many herons nest near each other in a marsh, large numbers of fish must be available to them. But many herons successfully nest in the Pahargaon marsh, and that marsh has been nearly empty of fish for many years.", "question": "Which one of the following, if true, most helps to reconcile the apparent discrepancy described above?", "answers": "['The herons in the Pahargaon marsh have been successfully raising unusually large families, with an average of almost two offspring surviving from each nesting.', 'Herons nesting in the Pahargaon marsh normally feed on fish living in nearby marshes.', 'The Pahargaon marsh has been growing saltier over the years, killing off much of the vegetation that the fish needed to eat.', 'Pesticides used on nearby farms have seeped into the waters of the Pahargaon marsh and killed most of the fish.']", "label": 1 }, { "id": "train_1980", "context": "People who are good at playing the game Drackedary are invariably skilled with their hands. Mary is a very competent watchmaker. Ttherefore, Mary would make a good Drackedary player.", "question": "The flawed pattern of reasoning in the argument above is most similar to that in which one of the following?", "answers": "['The role of Santa Claus in a shopping mall is often played by an experienced actor. Erwin has played Santa Claus in shopping malls for years. Ttherefore, Erwin must be an experienced actor.', 'All race car drivers have good reflexes. Chris is a champion table tennis player. Ttherefore, Chris would make a good race car driver.', \"People with long legs make good runners. Everyone in Daryl's family has long legs. Ttherefore, Daryl would make a good runner.\", 'People who write for a living invariably enjoy reading. Julie has been a published novelist for many years. Ttherefore, Julie enjoys reading.']", "label": 1 }, { "id": "train_1981", "context": "Livestock is a major contributor to climate change, accounting for 18 percent of the greenhouse gas released every year. In addition, livestock accounts for eight percent of global water use, and as much as 70 percent of deforestation is driven by the need for more pastures and feed crops. Dietary changes can dramatically decrease humanity' s environmental footprint, such as adopting a vegan or vegetarian lifestyle.", "question": "Which one of the following most accurately represents the author's conclusion?", "answers": "['The Earth will be destroyed unless everyone stops eating meat.', 'People can reduce their environmental impact by adopting dietary changes.', \"Dietary changes are the only way to decrease humanity's environmental footprint.\", 'Deforestation contributes to climate change.']", "label": 1 }, { "id": "train_1982", "context": "Curator: If our museum lends Venus to the Hart Institute for their show this spring, they will lend us their Rembrandt etchings for our print exhibition next fall. Having those etchings will increase attendance to the exhibition and hence increase revenue from our general admission fee. Museum Administrator: But Venus is our biggest attraction. Moreover the Hart's show will run for twice as long as our exhibition. So on balance the number of patrons may decrease.", "question": "The point of the administrator's response to the curator is to question", "answers": "[\"whether, if Venus is lent, the museum's revenue from general admission fees during the print exhibition will exceed its revenue from general admission fees during the Hart Institute's exhibition\", 'whether getting the Rembrandt etchings from the Hart Institute is likely to increase attendance at the print exhibition', \"whether the Hart Institute or the curator's museum will have the greater financial gain from the proposed exchange of artworks\", \"whether the number of patrons attracted by the Hart Institute's Rembrandt etchings will be larger than the number of patrons who do not come in the spring because Venus is on loan\"]", "label": 3 }, { "id": "train_1983", "context": "Executives at the Fizzles Beverage Company plan to boost profits in Country X on their range of fruit-flavored drinks by introducing new flavors based on tropical fruits that are little known there. The executives reason that since the fruit drinks of other companies have none of these flavors, Fizzles will not have to compete for customers and thus will be able to sell the drinks at a higher price.", "question": "Which of the following, if true, presents the most serious potential weakness of the plan?", "answers": "['The new fruit drinks would be priced significantly higher than other Fizzles fruit drinks with more conventional flavors.', 'Few consumers who are loyal to a specific brand of fruit-flavored drinks would willingly switch to another brand that costs more.', 'To build widespread demand for the new flavors, Fizzles would have to launch an advertising campaign to familiarize consumers with them.', 'Consumers choosing among fruit-flavored drinks of different brands generally buy on the basis of name recognition and price rather than the specific fruit flavor.']", "label": 3 }, { "id": "train_1984", "context": "Proposals for extending the United States school year to bring it more in line with its European and Japanese counterparts are often met with the objection that curtailing the schools' three-month summer vacation would violate an established United States tradition dating from the nineteenth century. However, this objection misses its mark. True, in the nineteenth century the majority of schools closed for three months every summer, but only because they were in rural areas where successful harvests depended on children' s labor. If any policy could be justified by those appeals to tradition, it would be the policy of determining the length of the school year according to the needs of the economy.", "question": "Which one of the following principles, if accepted, would provide the strongest justification for the conclusion?", "answers": "['The actual tradition embodied in a given practice can be accurately identified only by reference to the reasons that originally prompted that practice.', 'Traditional principles should be discarded when they no longer serve the needs of the economy.', 'Because appeals to tradition often serve to mask the real interests at issue, such appeals should be disregarded.', 'That a given social policy has traditionally been in force justifies maintaining that policy only if doing so does not conflict with more pressing social needs.']", "label": 0 }, { "id": "train_1985", "context": "Nick: The Pincus family and their construction company have supported our university financially for decades. The university should not give the contract for building its new library to the family' s main competitor. Doing so would be disloyal to a friend of the university. Pedro: Accepting a donation does not oblige the university to give the donor any special privileges. If it did, then it wouldn' t really be a charitable contribution. We should award the contract to whatever company makes the most competitive bid.", "question": "The dialogue provides the most support for the claim that Nick and Pedro disagree over whether", "answers": "['the Pincus family and their construction company donated money for the purpose of acquiring special privileges from the university', 'the acceptance of donations places a university under a special obligation to the donor', 'the university should be more grateful to donors with a long history of financial support than to new donors', 'loyalty should sometimes be a consideration in making business decisions']", "label": 1 }, { "id": "train_1986", "context": "Godinez: In the past, land was sometimes measured by the amount of time required to plow it. Thus, two plots of equal physical dimensions were considered unequal if one was more difficult to plow than the other. However, knowing how long an area takes to plow reveals little about how many apartment complexes it can hold. Ttherefore, it became necessary to adopt new measures of land, such as acreage, when land uses diversified.", "question": "Which one of the following most accurately expresses the main conclusion drawn in Godinez's argument?", "answers": "['Some plots of land that would have been considered unequal by plowing-time measurements are of equal physical dimensions.', 'The advent of diversified land uses made new measures of land necessary.', 'Modem measures of land were adopted when people realized that plowing time was an inadequate measure for some land uses.', 'It is now easier to measure a piece of land in terms of acres than in terms of plowing time.']", "label": 1 }, { "id": "train_1987", "context": "Wild cheetahs live in the African grasslands. Previous estimates of the size that the wild cheetah population must be in order for these animals to survive a natural disaster in the African grasslands region were too small, and the current population barely meets the previous estimates. At present, however, there is not enough African grassland to support a wild cheetah population larger than the current population.", "question": "The statements above, if true, most strongly support which one of the following conclusions?", "answers": "['The principal threat to the endangered wild cheetah population is neither pollution nor hunting, but a natural disaster.', 'Previous estimates of the size of the existing wild cheetah population were inaccurate.', \"The cheetah's natural habitat is decreasing in size at a faster rate than is the size of the wild cheetah population.\", 'In the short term, the wild cheetah population will be incapable of surviving a natural disaster in the African grasslands.']", "label": 3 }, { "id": "train_1988", "context": "A survey of historians shows that most believe written texts to be the best source for historical understanding. None of the historians regarded painting, architecture, music, dance, or culinary arts as the best source for historical understanding. So these historians neglect many important repositories of historical knowledge.", "question": "The reasoning in the argument is flawed because the argument takes for granted that", "answers": "['something other than written texts is the best source for historical understanding', 'there are no sources for historical understanding that are neither considered best by historians nor neglected by them', 'the other sources for historical understanding mentioned by the historians surveyed are not important repositories of historical knowledge', 'painting, architecture, music, dance, and culinary arts are important only as sources for historical understanding']", "label": 1 }, { "id": "train_1989", "context": "The male sage grouse has air sacs that, when not inflated, lie hidden beneath the grouse' s neck feathers. During its spring courtship ritual, the male sage grouse inflates these air sacs and displays them to the female sage grouse. Some scientists hypothesize that this courtship ritual serves as a means for female sage grouse to select healthy mates.", "question": "Which one of the following, if true, most strongly supports the scientists' hypothesis?", "answers": "['When diseased male sage grouse were treated with antibiotics, they were not selected by female sage grouse during the courtship ritual.', 'The sage grouse is commonly afflicted with a strain of malaria that tends to change as the organism that causes it undergoes mutation.', 'Some healthy male sage grouse do not inflate their air sacs as part of the courtship ritual.', \"Male sage grouse are prone to parasitic infections that exhibit symptoms visible on the birds' air sacs.\"]", "label": 3 }, { "id": "train_1990", "context": "The local agricultural official gave the fruit growers of the District 10 Farmers' Cooperative a new pesticide that they applied for a period of three years to their pear orchards in place of the pesticide they had formerly applied. During those three years, the proportion of pears lost to insects was significantly less than it had been during the previous three-year period. On the basis of these results, the official concluded that the new pesticide was more effective than the old pesticide, at least in the short term, in limiting the loss of certain fruit to insects.", "question": "Each of the following, if true, weakens the official's argument EXCEPT:", "answers": "['Some of the species of insects in District 10 that infest pear trees are water breeders, and the reservoirs and marshlands in this district have been shrinking rapidly over the past three years.', 'The amount of fruit that an orchard can potentially produce depends in part on how many mature trees it contains, and the number of mature pear trees in District 10 has declined steadily over the past eight years.', 'Over the past five years, one of the several species of birds that typically prey on the insects that feed on pears has gradually shifted its migratory patterns, spending more and more months each year in the region that contains District 10.', \"During the past five years, the farmers of the District 10 Farmers' Cooperative have been gradually implementing a variety of insect-abatement programs, and some of these programs have proven successful.\"]", "label": 1 }, { "id": "train_1991", "context": "If newly hatched tobacco hornworms in nature first feed on plants from the nightshade family, they will not eat leaves from any other plants thereafter. However, tobacco hornworms will feed on other sorts of plants if they feed on plants other than nightshades just after hatching. To explain this behavior, scientists hypothesize that when a hornworm' s first meal is from a nightshade, its taste receptors become habituated to the chemical indioside D, which is found only in nightshades, and after this habituation nothing without indioside D tastes good.", "question": "Which one of the following, if true, adds the most support for the hypothesis?", "answers": "['Indioside D is not the only chemical that occurs only in nightshade plants.', 'Tobacco hornworm eggs are most commonly laid on nightshade plants.', 'Tobacco hornworms that first fed on nightshade leaves show no preference for any one variety of nightshade plant over any other.', 'If taste receptors are removed from tobacco hornworms that first fed on nightshade leaves, those hornworms will subsequently feed on other leaves.']", "label": 3 }, { "id": "train_1992", "context": "Notice to subscribers: In order for us to provide you with efficient and reliable newspaper service, please note the following policies. You will be billed for home delivery every four weeks, in advance. If you do not receive delivery, call us promptly to receive a replacement copy. Credit can be given only if the missed copy is reported to us within twenty-four hours and only if a replacement copy is unavailable. Request for temporary nondelivery must be made at least three days prior to the first day on which delivery is to stop. No subscription will be canceled unless the subscriber explicitly requests the cancellation beforehand and in writing.", "question": "The Daily Gazette If The Daily Gazette denies each of the following subscriber's requests, each of the denials could be justified solely on the basis of the policy stated above EXCEPT:", "answers": "['Mr. Rathanan did not send in his advance payment two weeks ago; he states that his inaction was intended as cancellation and requests that he not be charged for the past two weeks of delivery of The Daily Gazette.', 'Ms. Silverman was out of town on Sunday and Monday and when she returned on Tuesday she found that her Sunday edition had not been delivered; she called The Daily Gazette on Tuesday afternoon requesting credit for the undelivered copy.', 'Dr. Broder called The Daily Gazette Monday morning to report that her Sunday edition had not been delivered; she requests credit instead of the offered replacement copy.', \"The Daily Gazette was delivered to Ms. Herrera during her one-week vacation even though she called on a Wednesday to stop delivery the following Monday for the entire week; she requests credit for the full week's delivery.\"]", "label": 3 }, { "id": "train_1993", "context": "Sanderson intentionally did not tell his cousin about overhearing someone say that the factory would close, knowing that if he withheld this information, his cousin would assume it would remain open. Clearly this was morally wrong. After all, lying is morally wrong. And making a statement with the intention of misleading someone is lying. True, it was Sanderson' s failing to state something that misled his cousin. Yet there is no moral difference between stating and failing to state if they are done with the same intention.", "question": "Which one of the following is an assumption required by the argument?", "answers": "['Sanderson believed that the factory would in fact be closing.', \"Sanderson had something to gain by his cousin's continuing to believe that the factory would remain open.\", 'Sanderson would have lied to his cousin if his cousin had asked him whether the factory would be closing.', \"No one ever told Sanderson's cousin about the factory closing.\"]", "label": 0 }, { "id": "train_1994", "context": "Sviatovin is a medieval Moringian text whose author and exact date of composition are unknown. However, the events in the life of Prince Sviatov that the text describes occurred in 1165, and in the diagram of Sviatov' s family that accompanies the text his father, who died in 1167, is identified as still living. Thus Sviatov must have been written between 1165 and 1167, assuming that __.", "question": "Which of the following most logically completes the argument?", "answers": "[\"Sviatovin was not written by Sviatov's father himself\", 'the life of Prince Sviatov is not the subject of any other medieval Moringian texts', \"the author of Sviatovin intended it to provide as accurate a report about Prince Sviatov's exploits as possible\", \"the diagram accurately represents the composition of Sviatov's family at the time Sviatovin was written\"]", "label": 3 }, { "id": "train_1995", "context": "One method of dating the emergence of species is to compare the genetic material of related species. Scientists theorize that the more genetically similar two species are to each other, the more recently they diverged from a common ancestor. After comparing genetic material from giant pandas, red pandas, raccoons, coatis, and all seven bear species, scientists concluded that bears and raccoons diverged 30 to 50 million years ago. They further concluded that red pandas separated from the ancestor of today' s raccoons and coatis a few million years later, some 10 million years before giant pandas diverged from the other bears.", "question": "Which one of the following can be properly inferred from the passage?", "answers": "['Giant pandas and red pandas are more closely related than scientists originally thought they were.', 'Scientists now count the giant panda as the eighth species of bear.', 'It is possible to determine, within a margin of just a few years, the timing of divergence of various species.', 'Scientists have found that giant pandas are more similar genetically to bears than to raccoons.']", "label": 3 }, { "id": "train_1996", "context": "Researcher: Overhearing only one side of a cell-phone conversation diverts listeners' attention from whatever they are doing. Hearing only part of a conversation leaves listeners constantly trying to guess what the unheard talker has just said. Listeners' attention is also diverted because cell-phone talkers speak abnormally loudly.", "question": "The researcher's statements, if true, most strongly support which one of the following?", "answers": "['The risk that a driver will cause an accident is increased when the driver is talking on a cell phone.', 'People who overhear one side of a cell-phone conversation inevitably lose track of their thoughts.', \"When a driver hears a passenger in the driver 's vehicle talking on a cell phone, that detracts from the driver 's performance .\", \"Conversing on a cell phone requires making more guesses about what one's conversational partner means than other forms of conversation do.\"]", "label": 2 }, { "id": "train_1997", "context": "Botanist: In an experiment, scientists raised domesticated radishes in a field with wild radishes, which are considered weeds. Within several generations, the wild radishes began to show the same flower color as the domesticated ones. This suggests that resistance to pesticides, which is often a genetically engineered trait, would also be passed from domesticated crop plants to their relatives that are considered weeds.", "question": "Which one of the following, if true, most strengthens the botanist's argument?", "answers": "['It is more difficult for flower color to be transferred between domesticated and wild radishes than it is for almost any other trait to be passed between any two similarly related plant species.', 'When the ratio of domesticated radishes to wild radishes in the field increased, the speed with which the flower color passed to the wild radishes also increased.', 'Radishes are not representative of crop plants in general with respect to the ease with which various traits are passed among members of closely related species.', 'It is much easier in principle for genetic traits to be passed from wild plants to their domesticated relatives than it is for such traits to be passed from the domesticated plant to the wild relative.']", "label": 0 }, { "id": "train_1998", "context": "A coffee manufacturer wants more restaurant chains to serve its brands of coffee. The manufacturer is considering a plan to offer its coffee to large chains at a significantly lower price, at least for a certain period. This lower price initially will reduce the manufacturer' s profits, but they hope to get into enough nationwide restaurant chains that their volume increases significantly. Once they have a much higher volume, even a small increase in their price would have an enormous effect on their profits.", "question": "In evaluating the plan's chances of success, it would be most helpful to know which of the following?", "answers": "['Whether the prices of some mixes of coffee will be discounted more than the prices of others.', 'Whether an alternate plan would allow the coffee manufacturer to take greater profits from the restaurant chains to which it currently provides coffee.', 'Whether their discounted price is lower than the prices of the coffee manufacturers who currently provide coffee to these nationwide restaurant chains.', 'Whether the manufacturer will use the same shipping system as it has been using to ship coffee to restaurants across the country.']", "label": 2 }, { "id": "train_1999", "context": "A company that produces men' s cologne had been advertising the product in general-circulation magazines for several years. Then one year the company decided to advertise its cologne exclusively in those sports magazines with a predominantly male readership. That year the company sold fewer bottles of cologne than it had in any of the three immediately preceding years.", "question": "Which one of the following, if true, best helps to explain why the sale of the company's cologne dropped that year?", "answers": "['The general-circulation magazines in which the company had placed its advertisements experienced a large rise in circulation recently.', \"Successful advertisements for men's cologne often feature well-known athletes.\", 'Most men do not wear cologne on a regular basis.', 'Women often buy cologne as gifts for male friends or relatives.']", "label": 3 }, { "id": "train_2000", "context": "During the 1980' s Japanese collectors were very active in the market for European art, especially as purchasers of nineteenth-century Impressionist paintings. This striking pattern surely reflects a specific preference on the part of many Japanese collectors for certain aesthetic attributes they found in nineteenth-century Impressionist paintings.", "question": "Which one of the following, if true, most strongly supports the explanation above?", "answers": "['Several nineteenth-century Impressionist painters adopted certain techniques and visual effects found in Japanese prints that are highly esteemed in Japan.', 'Impressionist paintings first became popular among art collectors in Europe at the beginning of the twentieth century.', \"During the 1980s, the Japanese economy underwent a sustained expansion that was unprecedented in the country's recent history.\", 'During the 1960s and 1970s, the prices of nineteenth-century Impressionist paintings often exceeded the prices of paintings by older European masters.']", "label": 0 }, { "id": "train_2001", "context": "Well-intentioned people sometimes attempt to resolve the marital problems of their friends. But these attempts are usually ineffectual and thereby foster resentment among all parties. Thus, even well-intentioned attempts to resolve the marital problems of friends are usually unjustified.", "question": "Which one of the following principles, if valid, most strongly supports the reasoning above?", "answers": "['The intentions of an action are irrelevant to whether or not that action is justified.', \"One should get involved in other people's problems only with the intention of producing the best overall consequences.\", 'No actions based on good intentions are justified unless they also result in success.', 'Interpersonal relations should be conducted in accordance with doing whatever is right, regardless of the consequences.']", "label": 2 }, { "id": "train_2002", "context": "Since 1945 pesticide use in the United States has increased tenfold despite an overall stability in number of acres planted. During the same period, crop loss from insects has approximately doubled, from about seven to thirteen percent.", "question": "Which one of the following, if true, contributes most to explaining the paradoxical findings above?", "answers": "['Extension agents employed by state governments to advise farmers have recently advocated using smaller amounts of pesticide, though in past years they promoted heavy pesticide use.', \"While pesticide-resistant strains of insects were developing, crop rotation, which for insects disrupts a stable food supply, was gradually abandoned because farmers' eligibility to receive government crop subsidies depended on continuing to plant the same crop.\", 'Although some pesticides can be removed from foodstuffs through washing, others are taken up into the edible portion of plants, and consumers have begun to boycott foods containing pesticides that cannot be washed off.', 'Since 1970 the pesticides most lethal to people have generally been replaced by less-lethal chemicals that are equally effective against insects and have a less-damaging effect on the fish in streams fed by water that runs off from treated agricultural fields.']", "label": 1 }, { "id": "train_2003", "context": "Anne: Halley' s Comet, now in a part of its orbit relatively far from the Sun, recently flared brightly enough to be seen by telescope. No comet has ever been observed to flare so far from the Sun before, so such a flare must be highly unusual. Sue: Nonsense. Usually no one bothers to try to observe comets when they are so far from the Sun. This flare was observed only because an observatory was tracking Halley' s Comet very carefully.", "question": "Sue challenges Anne's reasoning by", "answers": "[\"presenting evidence that directly contradicts Anne's evidence\", 'offering an alternative explanation for the evidence Anne cites', \"undermining some of Anne's evidence while agreeing with her conclusion\", 'pointing out that Anne\\'s use of the term \"observed\" is excessively vague']", "label": 1 }, { "id": "train_2004", "context": "Price: A corporation' s primary responsibility is to its shareholders. They are its most important constituency because they take the greatest risks. If the corporation goes bankrupt, they lose their investment. Albrecht: Shareholders typically have diversified investment portfolios. For employees, however, the well-being of the corporation for which they have chosen to work represents their very livelihood. The corporation' s primary responsibility should be to them.", "question": "On the basis of their statements, Price and Albrecht are committed to disagreeing about whether", "answers": "['corporations are responsible for the welfare of their employees', \"the livelihood of some of the shareholders depends on the corporation's success\", \"a corporation's shareholders have more at stake than anyone else does in the corporation's success or failure\", \"means should be provided for a corporation's investors to recoup their losses if the corporation goes bankrupt\"]", "label": 2 }, { "id": "train_2005", "context": "1990 editorial: Local pay phone calls have cost a quarter apiece ever since the 1970s, when a soft drink from a vending machine cost about the same. The price of a soft drink has more than doubled since, so phone companies should be allowed to raise the price of pay phone calls too.", "question": "Which one of the following, if true, most weakens the editorial's argument?", "answers": "['A pay phone typically cost less than a soft- drink machine in the 1970s.', 'Between the 1970s and 1990 the cost of ingredients for soft drinks increased at a greater rate than the cost of telephone equipment.', 'Technological advances made telephone equipment more sophisticated between the 1970s and 1990.', 'Government regulation of phone call prices did not become more stringent between the 1970s and 1990.']", "label": 1 }, { "id": "train_2006", "context": "Consumer: If you buy a watch at a department store and use it only in the way it was intended to be used, but the watch stops working the next day, then the department store will refund your money. So by this very reasonable standard, Bingham' s Jewelry Store should give me a refund even though they themselves are not a department store, since the watch I bought from them stopped working the very next day.", "question": "The consumer's argument relies on the assumption that", "answers": "['a seller should refund the money that was paid for a product if the product does not perform as the purchaser expected it to perform', 'the consumer did not use the watch in a way contrary to the way it was intended to be used', \"a watch bought at a department store and a watch bought at Bingham's Jewelry Store can both be expected to keep working for about the same length of time if each is used only as it was intended to be used\", 'one should not sell something unless one expects that it will function in the way it was originally designed to function']", "label": 1 }, { "id": "train_2007", "context": "Catmull: Although historians consider themselves to be social scientists, different historians never arrive at the same conclusions about specific events of the past. Thus historians never determine what actually happened; like novelists, they merely create interesting fictional stories about the many different problems that people have faced.", "question": "The reasoning in Catmull's argument is flawed because the argument", "answers": "[\"presumes, without providing justification, that unless historians' conclusions are objectively true, they have no value whatsoever\", 'bases its conclusion on premises that contradict each other', 'draws a conclusion that simply restates a claim presented in support of that conclusion', 'concludes, solely on the basis of the claim that different people have reached different conclusions about a topic, that none of these conclusions is true']", "label": 3 }, { "id": "train_2008", "context": "Lawmaker: Raising taxes is not the only means of reducing government debt. The government' s stockpile of helium is worth 25 percent more, at current market prices, than the debt accumulated in acquiring and storing it. Ttherefore, by selling the helium, the government can not only pay off that debt but reduce its overall debt as well.", "question": "Which of the following is an assumption on which the argument depends?", "answers": "[\"Attempts to sell the government's helium will not depress the market price of helium by more than 25 percent.\", \"It is not in the lawmaker's interest to advocate raising taxes as a means of reducing government debt.\", \"Twenty-five percent of the debt the government has accumulated in stockpiling helium is not an insignificant portion of the government's total debt.\", 'The government has no current need for helium.']", "label": 0 }, { "id": "train_2009", "context": "At a gathering at which bankers, athletes, and lawyers are present, all of the bankers are athletes and none of the lawyers are bankers.", "question": "If the statements above are true, which one of the following statements must also be true?", "answers": "['Some of the lawyers are not athletes.', 'Some of the athletes are not lawyers.', 'None of the lawyers are athletes.', 'All of the athletes are bankers.']", "label": 1 }, { "id": "train_2010", "context": "In most industrial waste products that contain the toxic chemical XTX, the concentration of this chemical is approximately 1, 000 parts per million. A federal law intended to reduce the harm that can result from the introduction of XTX into the environment permits a company to dispose of these waste products in a dump for hazardous waste, but only if the concentration of XTX is below 500 parts per million. Waste products with concentrations above that level must be destroyed by incineration. The law further specifies that manufacturers may not dilute XTX-containing waste products to bring their concentration of XTX down to a permissible level for dumping.", "question": "Which one of the following, if true, argues most strongly for the inclusion of the antidilution provision of the law?", "answers": "['Most owners of dumps for hazardous waste are willing to accept XTX for disposal in their facilities only in concentrations below 800 parts per million.', 'If present in the environment in sufficient quantities, the diluted XTX is as harmful as the more concentrated XTX.', 'If improperly incinerated, waste products containing undiluted concentrations of XTX can release into the environment a gaseous form of the chemical that is more than twice as toxic as XTX is in its usual liquid state.', 'To manufacturers, the cost of diluting and disposing of waste products containing XTX is approximately the same as the cost of destroying these products by incineration.']", "label": 1 }, { "id": "train_2011", "context": "If the recording now playing on the jazz program is really \"Louis Armstrong recorded in concert in 1989, \" as the announcer said, then Louis Armstrong was playing some of the best jazz of his career years after his death. Since the trumpeter was definitely Louis Armstrong, somehow the announcer must have gotten the date of the recording wrong.", "question": "The pattern of reasoning in the argument above is most similar to that in which one of the following arguments?", "answers": "[\"This painting titled La Toilette is Berthe Morisot's La Toilette only if a painting can be in two museums at the same time. Since nothing can be in two places at once, this painting must some how have been mistitled.\", 'Only if a twentieth-century Mexican artist painted in Japan during the seventeenth century can this work both be \"by Frida Kahlo\" as labeled and the seventeenth-century Japanese landscape it appears to be. Since it is what it appears to be, the label is wrong.', 'Unless Kathe Kollwitz was both a sculptor and a printmaker, the volunteer museum guide is wrong in his attribution of this sculpture. Since what Kollwitz is known for is her prints, the guide must be wrong.', 'If this painting is a portrait done in acrylic, it cannot be by Elisabeth Vigee-Lebrun, since acrylic paint was developed only after her death. Thus, since it is definitely a portrait, the paint must not be acrylic.']", "label": 1 }, { "id": "train_2012", "context": "Curator: Since ancient times, the fine arts were developed and sustained with the aid of large subsidies from the aristocracies and religious institutions that were the public sectors of their day; it is doubtful that the arts would have survived without these subsidies. Clearly, contemporary societies should fulfill their obligation as stewards of cultural heritage without the assistance of aristocracies or religious institutions, so governments must help finance the maintenance, advancement, and enrichment of the fine arts today.", "question": "The curator's argument depends on assuming which one of the following?", "answers": "['In contemporary societies, aristocracies and religious institutions are not willing to help finance the fine arts.', 'Maintenance, advancement, and enrichment of the fine arts in any era require governmental subsidies.', 'If contemporary governments help to maintain and enrich the fine arts, private support for the arts will become unnecessary.', 'Serving as stewards of cultural heritage requires that contemporary societies help to maintain the fine arts.']", "label": 3 }, { "id": "train_2013", "context": "Standard aluminum soft-drink cans do not vary in the amount of aluminum that they contain. Fifty percent of the aluminum contained in a certain group (M) of standard aluminum soft-drink cans was recycled from another group (L) of used, standard aluminum softdrink cans. Since all the cans in L were recycled into cans in M and since the amount of material other than aluminum in an aluminum can is negligible, it follows that M contains twice as many cans as L.", "question": "The conclusion of the argument follows logically if which one of the following is assumed?", "answers": "['The aluminum in the cans of M cannot be recycled further.', 'None of the soft-drink cans in group L had been made from recycled aluminum.', 'Aluminum soft-drink cans are more easily recycled than are soft-drink cans made from other materials.', 'All of the aluminum in an aluminum can is recovered when the can is recycled.']", "label": 3 }, { "id": "train_2014", "context": "The percentage of households with an annual income of more than $40, 000 is higher in Merton county than in any other county. However, the percentage of households with an annual income of $60, 000 or more is highest in Sommer county.", "question": "If the statements above are true, which of the following can properly be concluded on the basis of them?", "answers": "['Some households in Merton county have an annual income between $40, 000 and $60, 000.', 'No household in Merton county has an annual income of $60, 000 or more.', 'The percentage of households with an annual income of $80, 000 is higher in Sommer than in Merton county.', 'The number of households with an annual income of more than $40, 000 is greater in Merton than in Sommer county.']", "label": 0 }, { "id": "train_2015", "context": "Columnist: Almost anyone can be an expert, for there are no official guidelines determining what an expert must know. Anybody who manages to convince some people of his or her qualifications in an area -- whatever those may be -- is an expert.", "question": "The columnist's conclusion follows logically if which one of the following is assumed?", "answers": "['Every expert has convinced some people of his or her qualifications in some area.', 'Convincing certain people that one is qualified in an area requires that one actually be qualified in that area.', 'Some experts convince everyone of their qualifications in almost every area.', 'Almost anyone can convince some people of his or her qualifications in some area.']", "label": 3 }, { "id": "train_2016", "context": "A new law gives ownership of patents-documents providing exclusive right to make and sell an invention-to universities, not the government, when those patents result from government-sponsored university research. Administrators at Logos University plan to sell any patents they acquire to corporations in order to fund programs to improve undergraduate teaching.", "question": "Which of the following, if true, would cast the most doubt on the viability of the college administrators' plan described above?", "answers": "['Government-sponsored research conducted at Logos University for the most part duplicates research already completed by several profit-making corporations.', 'Profit-making corporations interested in developing products based on patents held by universities are likely to try to serve as exclusive sponsors of ongoing university research projects.', 'Logos University is unlikely to attract corporate sponsorship of its scientific research.', 'Research scientists at Logos University have few or no teaching responsibilities and participate little if at all in the undergraduate programs in their field.']", "label": 0 }, { "id": "train_2017", "context": "In Asia, where palm trees are nonnative, the trees' flowers have traditionally been pollinated by hand, which has kept palm fruit productivity unnaturally low. When weevils known to be efficient pollinators of palm flowers were introduced into Asia in 1980, palm fruit productivity increased-by up to 50 percent in some areas-but then decreased sharply in 1984.", "question": "Which of the following statements, if true, would best explain the 1984 decrease in productivity?", "answers": "['Prices for palm fruit fell between 1980 and 1984 following the rise in production and a concurrent fall in demand.', 'Prior to 1980 another species of insect pollinated the Asian palm trees, but not as efficiently as the species of weevil that was introduced in 1980.', 'Rapid increases in productivity tend to deplete trees of nutrients needed for the development of the fruit-producing female flowers.', 'The weevil population in Asia remained at approximately the same level between 1980 and 1984.']", "label": 2 }, { "id": "train_2018", "context": "Even though trading in ivory has been outlawed by international agreement, some piano makers still use ivory, often obtained illegally, to cover piano keys. Recently, experts have devised a synthetic ivory that, unlike earlier ivory substitutes, has found favor with concert pianists throughout the world. But because piano makers have never been major consumers of ivory, the development of the synthetic ivory will ttherefore probably do little to help curb the killing of elephants, from whose tusks most natural ivory is obtained.", "question": "Which one of the following, if true, most helps to strengthen the argument?", "answers": "['The most common use for natural ivory is in ornamental carvings, which are prized not only for the quality of their workmanship but also for the authenticity of their materials.', 'The new synthetic ivory can be manufactured to resemble in color and surface texture any of the various types of natural ivory that have commercial uses.', 'Most people who play the piano but are not concert pianists can nonetheless easily distinguish between the new synthetic ivory and inferior ivory substitutes.', 'It costs significantly less to produce the new synthetic ivory then it does to produce any of the ivory substitutes that scientists had developed previously.']", "label": 0 }, { "id": "train_2019", "context": "Consumer: The latest Chintan Report suggests that Omkar prepackaged meals are virtually devoid of nutritional value. But the Chintan Report is commissioned by Dilip Foods, Omkar' s largest corporate rival, and early drafts of the report are submitted for approval to Dilip Foods' public relations department. Because of the obvious bias of this report, it is clear that Omkar' s prepackaged meals really are nutritious.", "question": "The reasoning in the consumer's argument is most vulnerable to criticism on the grounds that the argument", "answers": "[\"treats evidence that there is an apparent bias as evidence that the Chintan Report's claims are false\", \"presumes, without providing justification, that Dilip Foods' public relations department would not approve a draft of a report that was hostile to Dilip Foods' products\", 'fails to take into account the possibility that Omkar has just as much motivation to create negative publicity for Dilip as Dilip has to create negative publicity for Omkar', \"draws a conclusion based solely on an unrepresentative sample of Omkar's products\"]", "label": 0 }, { "id": "train_2020", "context": "The number of aircraft collisions on the ground is increasing because of the substantial increase in the number of flights operated by the airlines. Many of the fatalities that occur in such collisions are caused not by the collision itself, but by an inherent flaw in the cabin design of most aircraft, in which seats, by restricting access to emergency exits, impede escape. Ttherefore, to reduce the total number of fatalities that result annually from such collisions, the airlines should be required to remove all seats that restrict access to emergency exits.", "question": "Which one of the following, if true, provides the most support for the proposal?", "answers": "['The number of deaths that occurred in theater fires because theater patrons could not escape was greatly reduced when theaters were required to have aisles leading to each exit.', 'In the event of fire, public buildings equipped with smoke detectors have fewer fatalities than do public buildings not so equipped.', 'Removing the seats that block emergency exits on aircraft will require a costly refitting of aircraft cabins.', 'In the event of collision, passengers on planes with a smaller passenger capacity generally suffer more serious injury than do passengers on planes with a larger passenger capacity.']", "label": 0 }, { "id": "train_2021", "context": "Editorial: A recent survey shows that 77 percent of people feel that crime is increasing and that 87 percent feel the judicial system should be handing out tougher sentences. Ttherefore, the government must firmly address the rising crime rate.", "question": "The reasoning in the editorial's argument is most vulnerable to criticism on the grounds that the argument", "answers": "['presumes, without providing justification, that there is a correlation between criminal offenders being treated leniently and a high crime rate', \"fails to distinguish between the crime rate's actually rising and people's believing that the crime rate is rising\", 'presumes, without providing justification, that tougher sentences are the most effective means of alleviating the crime problem', 'appeals to survey results that are inconsistent because they suggest that more people are concerned about the sentencing of criminals than are concerned about crime itself']", "label": 1 }, { "id": "train_2022", "context": "Few animals brave the midday heat of the Sahara desert. An exception is the silver ant, which chooses this time of day to leave its nest and scout for food, typically the corpses of heat-stricken animals. Even the silver ant, however, must be careful: at such times they can become victims of the heat themselves.", "question": "Which one of the following, if true, LEAST helps to explain the silver ant's choice of scavenging times?", "answers": "['The chief predators of the silver ant must take cover from the sun during midday.', 'The cues that silver ants use to navigate become less reliable as the afternoon progresses.', 'Silver ants cool themselves by climbing onto small pieces of dried vegetation to take advantage of random light breezes.', 'Other scavengers remove any remaining corpses as soon as the temperature begins to drop in the afternoon.']", "label": 2 }, { "id": "train_2023", "context": "Cezanne' s art inspired the next generation of artists, twentieth-century modernist creators of abstract art. While most experts rank Cezanne as an early modernist, a small few reject this idea. Fran^oise Cachin, for example, bluntly states that such an ascription is \"overplayed, \" and says that Cezanne' s work is \"too often observed from a modern point of view. ", "question": "Which one of the following statements is most strongly supported by the information above?", "answers": "['Cezanne was an early creator of abstract art.', 'Modern art owes less to Cezanne than many experts believe.', \"Cezanne's work helped to develop modernism.\", \"Cezanne's work tends to be misinterpreted as modernist.\"]", "label": 2 }, { "id": "train_2024", "context": "Letter to the editor: According to last Thursday' s editorial, someone who commits a burglary runs almost no risk of being caught. This is clearly false. Police reports show that at least 70 percent of people who commit crimes are caught. All burglars are criminals, so although some burglars will undoubtedly escape, a large percentage of them will eventually be caught.", "question": "Which one of the following arguments exhibits a flawed pattern of reasoning parallel to that exhibited in the argument that a large percentage of burglars will be caught?", "answers": "['Since a large percentage of professional persons have changed their careers, and since career changes require new training, all professional persons who have changed their careers required new training.', 'A large percentage of professional persons are self-employed. Thus, since nurses are professional persons, a large percentage of nurses are self-employed.', 'Many engineers are employed in management positions, and since anyone in a management position needs training in management, many engineers need training in management.', 'A large percentage of doctors are specialists. Since anyone who is a specialist must have training beyond the usual medical curriculum, it follows that many doctors have training beyond the usual medical curriculum.']", "label": 1 }, { "id": "train_2025", "context": "North American eastern white cedars grow both on cliff faces and in forests. Cedars growing on exposed cliff faces receive very few nutrients, and rarely grow bigger than one-tenth the height of cedars growing in forests, where they benefit from moisture and good soil. Yet few eastern white cedars found in forests are as old as four hundred years, while many on cliff faces are more than five hundred years old.", "question": "Which one of the following, if true, most helps to explain the difference in the ages of the cedars on cliff faces and those in forests?", "answers": "['Trees that are older than a few hundred years start to lose the protective outer layer of their bark.', 'The conditions on cliff faces are similar to those in most other places where there are few tall trees.', 'In areas where eastern white cedars grow, forest fires are relatively frequent, but fires cannot reach cliff faces.', 'Eastern white cedar wood is too soft to be used for firewood or modern buildings, but it is occasionally used to make furniture.']", "label": 2 }, { "id": "train_2026", "context": "Challenger: The mayor claims she has vindicated those who supported her in the last election by fulfilling her promise to increase employment opportunities in our city, citing the 8 percent increase in the number of jobs in the city since she took office. But during her administration, the national government relocated an office to our city, bringing along nearly the entire staff from the outside. The 8 percent increase merely represents the jobs held by these newcomers. Mayor: Clearly my opponent does not dispute the employment statistics. The unemployed voters in this city want jobs. The 8 percent increase in the number of jobs during my term exceeds that of any of my predecessors.", "question": "As a response to the challenger, the mayor's answer is flawed in that it", "answers": "['does not consider whether the number of unemployed persons within the city represents more than 8 percent of the eligible voters', 'explicitly attributes to the challenger beliefs that the challenger has neither asserted nor implied', \"fails to address the challenger's objection that the 8 percent increase did not result in an increase in job availability for those who lived in the city at the time of the last election\", 'takes for granted that those who supported the mayor in the last election believed job availability to be a significant city issue']", "label": 2 }, { "id": "train_2027", "context": "Elena: The best form of government is one that fosters the belief among its citizens that they have a say in how the government is run. Thus, democracy is the best form of government. Marsha: But there are many forms of government under which citizens can be manipulated into believing they have a say when they don' t.", "question": "Marsha's claim that it is possible for governments to manipulate people into thinking that they have a say when they do not is used to", "answers": "['suggest that the premise Elena uses to support her conclusion could be used to support a conflicting conclusion', \"concur with Elena's claim that democracy is the best form of government\", \"reject Elena's conclusion that the best form of government is democracy\", \"support Marsha's unstated conclusion that the best form of government is one that appears to be democratic but really is not\"]", "label": 0 }, { "id": "train_2028", "context": "The quality of unrefined olive oil is not actually defined in terms of acidity, yet extensive tests have shown that the less free oleic acid an unrefined olive oil contains per liter, the higher its quality. The proportion of free oleic acid that an olive oil contains is an accurate measure of the oil' s acidity.", "question": "If the statements above are all true, which of the following conclusions is best supported by them ?", "answers": "['Free oleic acid is the only acid that unrefined olive oil contains.', 'When an olive oil is refined, the concentration of oleic acid in the oil is reduced.', 'The quality of an unrefined olive oil can be determined only by accurately measuring its acidity.', 'If an unrefined olive oil is intermediate in acidity between two other unrefined olive oils, it will also be intermediate between them in quality.']", "label": 3 }, { "id": "train_2029", "context": "Most of the students who took Spanish 101 at the university last semester attended every class session. However, each student who received a grade lower than B minus missed at least one class session.", "question": "Which one of the following statements about the students who took Spanish 101 at the university last semester can be properly inferred from the information above?", "answers": "['Most, if not all, of the students who missed at least one class session received a grade lower than B minus.', 'At least some of the students who received a grade of A minus or higher attended every class session.', 'More than half of the students received a grade of B minus or higher.', 'Most of the students received a grade higher than B minus.']", "label": 2 }, { "id": "train_2030", "context": "Country X' s recent stock-trading scandal should not diminish investors' confidence in the country's stock market. For one thing, the discovery of the scandal confirms that Country X has a strong regulatory system , as the following considerations show. In any stock market, some fraudulent activity is inevitable. If a stock market is well regulated, any significant stock-trading fraud in it will very likely be discovered. This deters potential perpetrators and facilitates improvement in regulatory processes.", "question": "In the argument, the portion in boldface plays which of the following roles?", "answers": [ "It is the argument's main conclusion and is supported by another explicitly stated conclusion for which further support is provided.", "It is a compound statement containing both the argument's main conclusion and an assumption used to support that conclusion.", "It is a conclusion for which the argument provides support and which itself is used to support the argument's main conclusion.", "It is the argument's only conclusion." ], "label": 2 }, { "id": "train_2031", "context": "Naturalist: For decades we have known that the tuatara, a New Zealand reptile, have been approaching extinction on the South Island. But since South Island tuatara were thought to be of the same species as North Island tuatara there was no need to protect them. But new research indicates that the South Island tuatara are a distinct species, found only in that location. Because it is now known that if the South Island tuatara are lost an entire species will thereby be lost, human beings are now obliged to prevent their extinction, even if it means killing many of their unendangered natural predators.", "question": "Which one of the following principles most helps to justify the naturalists' argumentation?", "answers": "['In order to maximize the number of living things on Earth, steps should be taken to preserve all local populations of animals.', 'Human activities that either intentionally or unintentionally threaten the survival of an animal species ought to be curtailed.', 'The threat of local extinction imposes no obligation to try to prevent that extinction, whereas the threat of global extinction does impose such an obligation.', 'When an animal is in danger of dying, there is an obligation to help save its life, if doing so would not interfere with the health or well-being of other animals or people.']", "label": 2 }, { "id": "train_2032", "context": "Ringtail opossums are an Australian wildlife species that is potentially endangered. A number of ringtail opossums that had been orphaned and subsequently raised in captivity were monitored after being returned to the wild. Seventy-five percent of those opossums were killed by foxes, a species not native to Australia. Conservationists concluded that the native ringtail opossum population was endangered not by a scarcity of food, as had been previously thought, but by non-native predator species against which the opossum had not developed natural defenses.", "question": "Which one of the following, if true, most strongly supports the conservationists' argument?", "answers": "['Ringtail opossums that grow to adulthood in the wild defend themselves against foxes no more successfully than do ringtail opossums raised in captivity.', 'Foxes, which were introduced into Australia over 200 years ago, adapted to the Australian climate less successfully than did some other foreign species.', 'The ringtail opossums that were raised in captivity were fed a diet similar to that which ringtail opossums typically eat in the wild.', 'There are fewer non-native predator species that prey on the ringtail opossum than there are native species that prey on the ringtail opossum.']", "label": 0 }, { "id": "train_2033", "context": "Wind farms, which generate electricity using arrays of thousands of wind-powered turbines, require vast expanses of open land. County X and County Y have similar terrain, but the population density of County X is significantly higher than that of County Y. Ttherefore, a wind farm proposed for one of the two counties should be built in County Y rather than in County X.", "question": "Which of the following, if true, most seriously weakens the planner's argument?", "answers": "['Some of the electricity generated by wind farms in County Y would be purchased by users outside the county.', 'Wind farms require more land per unit of electricity generated than does any other type of electrical-generation facility.', \"Nearly all of County X's population is concentrated in a small part of the county, while County Y's population is spread evenly throughout the country.\", 'County X and County Y are adjacent to each other, and both are located in the windiest area of the state.']", "label": 2 }, { "id": "train_2034", "context": "The goblin fern, which requires a thick layer of leaf litter on the forest floor, is disappearing from North American forests. In spots where it has recently vanished, the leaf litter is unusually thin and, unlike those places where this fern still thrives, is teeming with the European earthworm Lumbricus rubellus, which eats leaf litter. L. rubellus is thus probably responsible for the fern' s disappearance.", "question": "Which one of the following is an assumption on which the argument depends?", "answers": "['None of the earthworms that are native to North America eat leaf litter.', 'Wherever there is a thick layer of leaf litter in North American forests, goblin ferns can be found.', 'L. rubellus does not favor habitats where the leaf litter layer is considerably thinner than what is required by goblin ferns.', 'Dead leaves from goblin ferns make up the greater part of the layer of leaf litter on the forest floors where the goblin fern has recently vanished.']", "label": 2 }, { "id": "train_2035", "context": "Substantial economic growth must be preceded by technological innovations that expanding industries incorporate into their production or distribution procedures. Since a worldwide ban on the use of fossil fuels would surely produce many technological innovations, it is obvious that such a ban would be followed by an economic boom rather than by the economic depression forecast by the critics of such a ban.", "question": "Which one of the following most accurately describes a flaw in the argument's reasoning?", "answers": "['The argument confuses a necessary condition for a phenomenon with a sufficient condition for that phenomenon .', 'The argument assumes the truth of the conclusion for which it purports to be providing evidence.', 'The argument attempts to establish the falsehood of a proposition by criticizing the reasoning of those who assert its truth.', 'The argument presumes, without providing warrant, that because certain conditions only sometimes precede a certain phenomenon, these conditions always bring about the phenomenon .']", "label": 0 }, { "id": "train_2036", "context": "Not all certified loan consultants have college degrees. Ttherefore, although every loan consultant at the local bank is certified, it must be the case that not all the loan consultants at the local bank have college degrees.", "question": "The flawed pattern of reasoning exhibited by the argument above is most similar to that exhibited by which of the following?", "answers": "['Although some paintings created by great artists remain unfinished, all paintings in this museum were done by great artists. Ttherefore, some of the paintings in this museum are unfinished.', 'All government agencies are large, but not all government agencies are well funded. Ttherefore, large government agencies need not be well funded.', \"Although some cheeses do contain goat's milk, all cheese is delicious. Ttherefore, cheese can be delicious without containing goat milk.\", 'Not all teachers have good communications skills, but some teachers have good writing skills. Ttherefore, all teachers have either good communications skills or good writing skills.']", "label": 0 }, { "id": "train_2037", "context": "Dr. Godfrey: Now that high school students are allowed to work more than 15 hours per week at part-time jobs, those who actually do so show less interest in school and get lower grades than those who do not work as many hours at part-time jobs. Obviously, working long hours at part-time jobs during the school year contributes to the academic problems that many of our high school students experience. Dr. Nash: That' s not so. Many of our high school students set out to earn as much money as they can simply to compensate for their lack of academic success.", "question": "The answer to which one of the following would be the most helpful in determining whether the conclusion that Dr. Godfrey draws could be logically defended against Dr. Nash's counterargument?", "answers": "['whether people who have had academic problems in high school are ultimately less successful in their careers than people who have not had such problems', 'whether employers and high school students typically obey all the laws that regulate the conditions under which young people may legally be employed', 'whether students are allowed to spend more than 15 hours per week at school-sponsored nonacademic extracurricular activities such as team sports or clubs', 'whether the students who work more than 15 hours per week and have academic problems had such problems before they began to work that many hours']", "label": 3 }, { "id": "train_2038", "context": "Studies have shown that highway drivers are less likely to drive at speeds greater than the posted speed limit if houses or buildings are in some way visible from the road. Assuming similar car, road and traffic conditions, highway drivers are more likely to drive at speeds in excess of the posted speed limit if the natural surroundings are not interrupted by buildings. Psychologists hypothesize that seeing signs of civilization reminds drivers of their responsibility to the safety of their fellow humans, thus making them more likely to obey the posted speed limit.", "question": "Which of the following, if true, most strongly supports the psychologists' interpretation of the study?", "answers": "['Highway drivers passing a large plowed and cultivated field are more likely to obey the speed limit than those passing a large field of wild flowers.', 'Highway police officers are more densely located close to towns and cities, and ttherefore most citations for speeding are issued in these locations.', 'Drivers are equally likely to obey the speed limit whether driving past a town with or without tall buildings.', 'Traffic tends to be more congested around towns and cities.']", "label": 0 }, { "id": "train_2039", "context": "Because a large disparity in pay between the public and private sectors has developed in recent years, many experienced and extremely capable government administrators have quit their posts and taken positions in private-sector management. Government will be able to recapture these capable administrators by raising salaries to a level comparable to those of the private sector. In that way, the functioning of public agencies will be improved.", "question": "The position taken above presupposes which one of the following?", "answers": "['If the disparity in pay between government administration and private-sector management increases, administrators will move to the public sector in large numbers.', 'Experience gained from private-sector management will be very valuable in government administration.', 'People who moved from jobs in government administration to private-sector management would choose to change careers again.', 'The most important factor determining how well government agencies function is the amount of experience the administrators have.']", "label": 2 }, { "id": "train_2040", "context": "Robyn: A new line of designer imposter cosmetics could be very successful in certain markets. Manufacturers developing a new line of designer imposter cosmetics should focus on matching the packaging of the new line as closely as possible to the name brand cosmetic because the packaging is a significant part of brand recognition. Linda: Packaging is a significant part of brand recognition, but it is not the most important aspect. Generic food brands whose packaging closely resembles the name brand versions are not successful at luring the name brand' s customers unless the quality of the product is the same. Ttherefore, manufacturers that want to develop a successful line of designer imposter cosmetics should focus more on creating a product that is similar in quality to the original rather than focusing their energies on the packaging in an attempt to take advantage of brand recognition.", "question": "The claim that generic food brands whose packaging closely resembles the name brand versions are not successful at luring the name brand products' customers unless the quality of the product is the same plays which one of the following roles in Linda's argument?", "answers": "['It provides background information related to manufacturing generic cosmetic lines.', 'It provides a specific example that illustrates her general claim.', 'It is a secondary conclusion used to support the main conclusion.', 'It is the main conclusion of her argument.']", "label": 1 }, { "id": "train_2041", "context": "Many professional economists describe economics as a science. Sciences, however, are by definition non-normative: they describe but they do not prescribe. Yet economists are often called on to recommend a course of action for governments and financial institutions. Ttherefore, since economists play a prescriptive role in society, economics should not be thought of as a science.", "question": "The reasoning in the argument is flawed because the argument", "answers": "['attacks the proponents of a claim rather than addressing the merits of the claim itself', 'overlooks the necessity of divisions of labor within society', 'fails to recognize the significance of the distinction between a discipline and the people who work within that discipline', 'insists on a change in terminology when that change would have no practical consequences']", "label": 2 }, { "id": "train_2042", "context": "Generally scientists enter their field with the goal of doing important new research and accept as their colleagues those with similar motivation. Ttherefore, when any scientist wins renown as an expounder of science to general audiences, most other scientists conclude that this popularizer should no longer be regarded as a true colleague.", "question": "The explanation offered above for the low esteem in which scientific popularizers are held by research scientists assumes that", "answers": "['serious scientific research is not a solitary activity, but relies on active cooperation among a group of colleagues', 'research scientists believe that those who are well known as popularizers of. science are not motivated to do important new research', 'a scientist can become a famous popularizer without having completed any important research', 'no important new research can be accessible to or accurately assessed by those who are not themselves scientists']", "label": 1 }, { "id": "train_2043", "context": "Various studies have concluded that song overlapping, the phenomenon where one bird begins a song while another of its species is singing, is a signal of aggression. These studies are based solely on receiver-response tests, which seek to derive conclusions about the intent of a signal based on how others respond to it. However, any response -- even no response -- can be interpreted as a reaction to perceived aggression. Ttherefore, __.", "question": "Which one of the following most logically completes the argument?", "answers": "['receiver-response tests can provide no insight into bird behavior', 'the conclusion of these studies is unconvincing', 'song overlapping is likely not a signal of aggression', 'birds do not respond in a predictable manner to signals of aggression']", "label": 1 }, { "id": "train_2044", "context": "A recent survey conducted in one North American city revealed widespread concern about the problems faced by teenagers today. Seventy percent of the adults surveyed said they would pay higher taxes for drug treatment programs, and 60 percent said they were willing to pay higher taxes to improve the city' s schools. Yet in a vote in that same city, a proposition to increase funding for schools by raising taxes failed by a narrow margin to win majority approval.", "question": "Which one of the following factors, if true, would LEAST contribute to an explanation of the discrepancy described above?", "answers": "['Many of the people who were surveyed did not respond truthfully to all of the questions put to them.', 'A proposition for increasing funds for local drug treatment centers also failed to win approval.', 'The proposition to raise taxes for schools was couched in terminology that many of the voters found confusing.', 'The survey sample was not representative of the voters who voted on the proposition.']", "label": 1 }, { "id": "train_2045", "context": "When hypnotized subjects are told that they are deaf and are then asked whether they can hear the hypnotist, they reply, \"No. \" Some theorists try to explain this result by arguing that the selves of hypnotized subjects are dissociated into separate parts, and that the part that is deaf is dissociated from the part that replies.", "question": "Which of the following challenges indicates the most serious weakness in the attempted explanation described above?", "answers": "['Why does the part that replies not answer, \"Yes\"?', \"Why do the subjects appear to accept the hypnotist's suggestion that they are deaf?\", 'Why do hypnotized subjects all respond the same way in the situation described?', 'Why are the observed facts in need of any special explanation?']", "label": 0 }, { "id": "train_2046", "context": "With a record number of new companies starting up in Derderia, and with previously established companies adding many new jobs, a record number of new jobs were created last year in the Derderian economy. This year, previously established companies will not be adding as many new jobs overall as such companies added last year. Ttherefore, unless a record number of companies start up this year, Derderia will not break its record for new jobs created.", "question": "Which of the following is an assumption on which the argument relies?", "answers": "['This year, the new companies starting up will not provide substantially more jobs per company than did new companies last year.', 'The number of people seeking employment is no larger this year than it was last year.', 'In a given year, new companies starting up create more new jobs on average than do previously established companies.', 'The number of jobs created in the Derderian economy last year was substantially larger than the number of jobs lost.']", "label": 0 }, { "id": "train_2047", "context": "Linda: The governor' s proposal to raise taxes to improve carpool lanes is unjust. Why should commuters who never use carpool lanes be forced to subsidize them? Mario: You are mistaken. The state government has always designated more funding to noncarpool lanes. The additional revenue raised from the tax increase will just allow the governor to distribute funds to all lanes more equally.", "question": "Mario uses which of the following argumentative strategies in answering Linda?", "answers": "[\"pointing out that his opponent's arguments are based on self-contradictory premises\", \"expanding the context of the problem in order to make the governor's proposal appear more equitable\", 'disagreeing that arbitrary concerns like fairness are relevant to the discussion', 'arguing that programs that benefit the state as a whole benefit all citizens in the state']", "label": 1 }, { "id": "train_2048", "context": "With seventeen casinos, Moneyland operates the most casinos in a certain state. Although intent on expanding, it was outmaneuvered by Apex Casinos in negotiations to acquire the Eldorado chain. To complete its acquisition of Eldorado, Apex must sell five casinos to comply with a state law forbidding any owner to operate more than one casino per county. Since Apex will still be left operating twenty casinos in the state, it will then have the most casinos in the state.", "question": "Which of the following, if true, most seriously undermines the prediction?", "answers": "[\"Moneyland already operates casinos in the majority of the state's counties.\", 'Apex, Eldorado, and Moneyland are the only organizations licensed to operate casinos in the state.', 'Apex will use funds it obtains from the sale of the five casinos to help fund its acquisition of the Eldorado chain.', \"Some of the state's counties do not permit casinos.\"]", "label": 1 }, { "id": "train_2049", "context": "The more demand there is for something, the easier it is to sell. Any junkyard will buy a used car that is less than ten years old, because the parts can easily be resold. However, because there is little demand for car parts that are ten years old or older, junkyards tend not to buy those cars. Thus, used cars that are less than ten years old are generally easier to sell than are cars that are ten years old or older.", "question": "Which one of the following is an assumption on which the argument depends?", "answers": "['The salability of something is not influenced by any factors other than the level of demand for it.', 'All used cars that are ten years old or older are sold to junkyards.', 'When determining the selling price of cars less than ten years old, the lack of demand would not be offset by a lack of supply.', 'The salability of cars that are ten years old or older is largely a function of the level of demand for their parts.']", "label": 3 }, { "id": "train_2050", "context": "It is easy to see that the board of directors of the construction company is full of corruption and should be replaced. There are many instances of bribery by various persons on the staff of board member Wagston that are a matter of public record. These bribes perniciously influenced the awarding of government contracts.", "question": "The argument's reasoning is most vulnerable to criticism on the grounds that", "answers": "[\"the argument fails to show that corruption is not limited to Wagston's staff\", \"the argument presumes without giving justification that all of Wagston's staff have engaged in corruption\", 'the argument attempts to deflect attention away from substantive issues by attacking the character of the board', 'the argument fails to specify the relation between bribery and corruption']", "label": 0 }, { "id": "train_2051", "context": "The water of Lake Laberge, in Canada, currently contains high levels of the pesticide toxaphene. Authorities are puzzled because toxaphene was banned in North America in the early 1980s and now is used only in a few other parts of the world.", "question": "Which one of the following, if true, does most to explain why the water of Lake Laberge currently contains high levels of toxaphene?", "answers": "['North American manufacturers opposed banning toxaphene.', 'Levels of pesticides in the environment often continue to be high for decades after their use ends.', 'Toxic chemicals usually do not travel large distances in the atmosphere.', 'Toxic chemicals become more readily detectable once they enter organisms the size of fish.']", "label": 1 }, { "id": "train_2052", "context": "Scientist: The FDA is yet to weigh in on the effects of electronic cigarettes on long-term health. Electronic cigarettes heat up a liquid and produce the vapor inhaled by the user. The liquid consists of vegetable glycerin and propylene glycerol at varying ratios. Artificial flavoring is also added to the liquid. Although the FDA has approved vegetable glycerin, propylene glycerol, and artificial flavors for consumption, little is known about the effects of consuming their vapors. However, electronic cigarettes do not produce tar, which is one of the most dangerous chemicals in tobacco cigarettes.", "question": "Which one of the following most accurately expresses the scientist's main point?", "answers": "['The FDA is inefficient and ineffective at protecting public health.', 'Although all of the information is not yet available, electronic cigarettes are promising alternatives to tobacco since the former does not produce tar.', 'Smokers should quit tobacco and start using electronic cigarettes.', 'Tar is the reason why cigarettes are unhealthy.']", "label": 1 }, { "id": "train_2053", "context": "One way to furnish a living room is with modular furniture. Instead of buying a standard sofa, for example, one can buy a left end, a right end, and a middle piece that can be combined to create an L-shaped sofa. Modular furniture, however, is far more expensive than standard furniture. On average, a three-piece modular sofa costs almost twice as much as a standard sofa of comparable size and quality.", "question": "Each of the following, if true, helps to account for the greater cost of modular furniture EXCEPT:", "answers": "['Modular sofas, on average, have a greater area of upholstered surfaces than do standard sofas.', 'Modular furniture, unlike standard furniture, is not mass-produced.', 'The consumer demand for sofas sometimes increases more quickly than the supply.', 'Because modular furniture pieces are custom ordered, they are never put on sale.']", "label": 2 }, { "id": "train_2054", "context": "Economist: Historically, sunflower seed was one of the largest production crops in Kalotopia, and it continues to be a major source of income for several countries. The renewed growing of sunflowers would provide relief to Kalotopia' s farming industry, which is quite unstable. Further, sunflower oil can provide a variety of products, both industrial and consumer, at little cost to Kalotopia' s already fragile environment.", "question": "The economist's statements, if true, most strongly support which one of the following?", "answers": [ "Stabilizing Kalotopia's farming industry would improve the economy without damaging the environment.", "Kalotopia's farming industry would be better off now if it had never ceased to grow any of the crops that historically were large production crops.", "Kalotopia's farming industry will deteriorate if sunflowers are not grown there.", "A crop that was once a large production crop in Kalotopia would, if it were grown there again, benefit that country's farmers and general economy." ], "label": 3 }, { "id": "train_2055", "context": "Since 1995, Congress has exempted oil companies that have leases issued by the federal government allowing them to drill for deep-water oil off the Gulf of Mexico from royalty payments as an incentive to spur development in times of low oil and gas prices. These leases were supposed to have included a provision that reinstates the royalties should the market prices of oil and gas exceed a certain level. Because of an error by the federal government, however, the language that reinstates the royalties is missing from the more than 1, 100 leases issued by the U. S. government in 1998 and 1999. Since the market price of oil and gas has recently risen far above the threshold levels, this error could allow the oil companies to reap a windfall of more than $10 billion through the life of the leases. In response, the government is pressuring the oil companies to renegotiate the leases. The executives of the oil companies strongly oppose renegotiation; all have issued statements stating that they expect the government to honor the terms of the contracts and that renegotiating a duly signed agreement would set a bad precedent.", "question": "Which of the following statements best reflects the position of the oil company executives?", "answers": "['Opportunity seldom knocks twice.', \"One man's loss is another man's gain.\", \"You don't change the rules in the middle of the game.\", 'Revenge is so sweet.']", "label": 2 }, { "id": "train_2056", "context": "The administration at a certain university has explained this year' s tuition increase by citing increased spending on faculty salaries and on needbased aid to students. However, this year' s budget indicated that faculty salaries constitute a small part of the university' s expenditure, and the only significant increases in scholarship aid have gone to academic scholarships awarded regardless of need. The administration' s explanation is not believable.", "question": "Which one of the following, if true, most strengthens the argument that the administration's explanation is not believable?", "answers": "['Faculty salaries at the university have increased in line with the national average, and substantial cuts in government student-loan programs have caused financial difficulties for many students at the university.', 'Of the substantial items in the budget, the greatest increase was in administrative costs, facilities maintenance costs, and costs associated with the provision of athletic facilities.', \"With this year's budget, the university has increased its total spending on scholarship aid by 5 percent.\", \"With this year's budget, the university increased the allotment for faculty salaries by 5 percent while tuition was increased by 6 percent.\"]", "label": 1 }, { "id": "train_2057", "context": "Researcher: We have found that some cases of high blood pressure can be treated effectively with medicine. Since it is generally accepted that any illness caused by stress is treatable only by the reduction of stress, some cases of high blood pressure must not be caused by stress.", "question": "Which one of the following is an assumption required by the researcher's argument?", "answers": "['Some conditions that are treated effectively by medicines are not also treatable through the reduction of stress.', \"The reduction of stress in a person's life can at times lower that person's blood pressure.\", \"Reduced stress does not reduce a person's responsiveness to medicine used to treat high blood pressure.\", 'Medicine used to treat high blood pressure does not itself reduce stress.']", "label": 3 }, { "id": "train_2058", "context": "Critic to economist: In yet another of your bumbling forecasts, last year you predicted that this country' s economy would soon go into recession if current economic policies were not changed. Instead, economic growth is even stronger this year. Economist: There was nothing at all bumbling about my warning. Indeed, it convinced the country' s leaders to change economic policies, which is what prevented a recession.", "question": "The economist responds to the critic by", "answers": "['offering a particular counterexample to a general claim asserted by the critic', \"indicating that the state of affairs on which the economist's prediction was conditioned did not obtain\", 'distinguishing between a prediction that has not yet turned out to be correct and one that has turned out to be incorrect', \"attempting to show that the critic's statements are mutually inconsistent\"]", "label": 1 }, { "id": "train_2059", "context": "Junior biomedical researchers have long assumed that their hirings and promotions depend significantly on the amount of their published work. People responsible for making hiring and promotion decisions in the biomedical research field, however, are influenced much more by the overall impact that a candidate' s scientific publications have on his or her field than by the number of those publications.", "question": "The information above, if accurate, argues most strongly against which of the following claims?", "answers": "['Even biomedical researchers who are just beginning their careers are expected already to have published articles of major significance to the field.', 'Biomedical researchers can substantially increase their chances of promotion by fragmenting their research findings so that they are published in several journals instead of one.', 'Contributions to the field of biomedical research are generally considered to be significant only if the work is published.', 'The potential scientific importance of not-yet-published work is sometimes taken into account in decisions regarding the hiring or promotion of biomedical researchers.']", "label": 1 }, { "id": "train_2060", "context": "Economist: Unemployment will soon decrease. If total government spending significantly increases next year, the economy will be stimulated in the short term and unemployment will decrease. If, on the other hand, total government spending significantly decreases next year, businesses will retain more of their earnings in the short term and employ more workers, thereby decreasing unemployment.", "question": "The conclusion drawn by the economist is properly inferred if which one of the following is assumed?", "answers": "['Government officials are currently implementing policies that are intended to reduce unemployment.', 'If there is a significantly increased demand for workers, then there will be a significant decrease in unemployment.', 'If the economy is not stimulated and businesses do not retain more of their earnings, then unemployment will not decrease.', 'Either total government spending will significantly decrease next year or else total government spending will significantly increase next year.']", "label": 3 }, { "id": "train_2061", "context": "In Europe, many large animal species, such as mammoths, became extinct soon after humans first migrated to the animals' areas of habitation. The spread of such extinctions closely followed the pattern of human migration. However, since humans tended to migrate to areas as the climate in those areas began to warm, the extinctions might have been precipitated by the climatic warming rather than by human migration.", "question": "Which of the following, if true, provides the best evidence that it was human migration and not climatic change that precipitated the extinctions?", "answers": "['Several periods of marked climatic warming have occurred in Europe, during which many large animal species that lived there became extinct.', 'Many animal species, such as deer and bison, continued to thrive in Europe even after humans migrated there.', 'Many animal species that became extinct in Europe survived longer in areas that were isolated from human populations but that experienced intense climatic warming.', 'Some large animals had become extinct in Europe even before humans migrated there.']", "label": 2 }, { "id": "train_2062", "context": "The use of phrases like \"as so-and-so said\" or \"as the saying goes\" suggests that the quote that follows has just been illustrated. Such phrases are inappropriately used when an apparent counterexample has just been given.", "question": "Which one of the following contains an inappropriate usage of a phrase, according to the principle stated above?", "answers": "['The best model of vacuum cleaner was the most expensive on the market, but it would have made Roger unhappy to purchase it. For although he never wanted anything but the best, he was also quite frugal, and would never have forgiven himself for spending the money. As the saying goes, \"A penny saved is a penny earned. \"', 'Sharon loved cats, but her husband was allergic to them. Still, he was occasionally willing to accompany her to cat shows. As the saying goes, \"Shared lives mean shared loves. \"', 'Harold\\'s friends were surprised when he revealed that he had left his wallet at home and asked that someone lend him money. But he had done the same thing many times before. As Halliard said, \"The force of selfishness is as inevitable and as calculable as the force of gravitation. \"', 'Raoul spent a year planning and preparing for a fantastic ski trip. He enjoyed his ski trip greatly until he broke his leg and had to spend two weeks in the hospital. As the saying goes, \"All\\'s well that ends well. \"']", "label": 3 }, { "id": "train_2063", "context": "Dr. Bradshaw: Gastric bypass surgery will provide sustained health and weight-loss benefits in morbidly obese patients, since it allows morbidly obese patients to have the size of their stomachs reduced to allow them to consume less food, and also allows food to bypass part of the small intestine, ensuring that fewer calories are absorbed. Morbidly obese patients will realize more sustained benefit from gastric bypass surgery than any other treatment regimen could provide. Dr. Hobbes: Not so. Gastric bypass surgery might help morbidly obese patients initially. However, patients undergoing this surgery will eventually become extremely dehydrated and deprived of critical nutrients needed to maintain their health due to the vastly decreased amount of food and beverages they can consume. Eventually, most will develop severe complications from dehydration coupled with nutrient loss. Hence, sustained health benefits will not be obtained.", "question": "Which one of the following is the point at issue between Dr. Bradshaw and Dr. Hobbes?", "answers": "['whether the technology of gastric bypass surgery will benefit patients in the long run', 'whether gastric bypass surgery in general is more beneficial to health than other health care plans', 'whether gastric bypass surgery should be used for all morbidly obese patients', 'whether gastric bypass surgery is likely to be widely undergone by morbidly obese patients in future years']", "label": 0 }, { "id": "train_2064", "context": "Archaeologists working in the Andes Mountains recently excavated a buried 4, 000-year-old temple containing structures that align with a stone carving on a distant hill to indicate the direction of the rising sun at the summer solstice. Alignments in the temple were also found to point toward the position, at the summer solstice, of a constellation known in Andean culture as the Fox. Since the local mythology represents the fox as teaching people how to cultivate and irrigate plants, the ancient Andeans may have built the temple as a religious representation of the fox.", "question": "Which of the following is an assumption on which the argument is based?", "answers": "['In the region around the temple, the summer solstice marks the time for planting.', 'The constellation known as the Fox has the same position at the summer solstice as it did 4, 000 years ago.', 'Other structural alignments at the temple point to further constellations with agricultural significance.', 'The temple was protected from looters by dirt and debris built up over thousands of years.']", "label": 1 }, { "id": "train_2065", "context": "Any student who is not required to hand in written homework based on the reading assignments in a course will not complete all of the reading assignments. Even highly motivated students will neglect their reading assignments if they are not required to hand in written homework. Ttherefore, if the students in a course are given several reading assignments and no written assignments, no student in that course will receive a high grade for the course.", "question": "The conclusion of the argument follows logically if which one of the following is assumed?", "answers": "['Any student who completes all of the reading and written assignments for a course will earn a high grade in that course.', 'No student who completes anything less than all of the reading assignments for a course will earn a high grade for that course.', 'All highly motivated students who complete all of the reading assignments for a course will receive high grades for that course.', 'Some highly motivated students will earn high grades in a course if they are required to hand in written homework on their reading assignments.']", "label": 1 }, { "id": "train_2066", "context": "All works of art are beautiful and have something to teach us. Thus, since the natural world as a whole is both beautiful and instructive, it is a work of art.", "question": "The reasoning in the argument is flawed because the argument", "answers": "['uses the inherently vague term \"beautiful\" without providing an explicit definition of that term', 'concludes, simply because an object possesses two qualities that are each common to all works of art, that the object is a work of art', 'attempts to establish an evaluative conclusion solely on the basis of claims about factual matters', 'fails to consider the possibility that there are many things that are both beautiful and instructive but are not part of the natural world']", "label": 1 }, { "id": "train_2067", "context": "The current proposal to give college students a broader choice in planning their own courses of study should be abandoned. The students who are supporting the proposal will never be satisfied, no matter what requirements are established. Some of these students have reached their third year without declaring a major. One first-year student has failed to complete four required courses. Several others have indicated a serious indifference to grades and intellectual achievement.", "question": "A flaw in the argument is that it does which one of the following?", "answers": "['distorts the proposal advocated by opponents', 'avoids the issue by focusing on supporters of the proposal', 'uses the term \"student\" equivocally', 'argues circularly by assuming the conclusion is true in stating the premises']", "label": 1 }, { "id": "train_2068", "context": "Reconstructing ships so that they have a double hull can protect them from serious damage in the roughly 20 percent of shipping accidents that unavoidably result from severe weather conditions. Avoidable human error, however, causes about 80 percent of shipping accidents. It would ttherefore be far more cost-effective to reduce damage to ships by retraining sailors than by reconstructing ships.", "question": "Which one of the following, if true, most strengthens the argument?", "answers": "['Reconstructing ships would provide protection from damage in accidents caused by human error.', 'One should always try to reduce the severity of the damage caused by each individual accident rather than try to minimize the number of accidents.', 'The least expensive course of action in the long term is the one that minimizes the total cost of damage from accidents.', 'Damage from accidents can be reduced more cost-effectively by reducing the likelihood of accidents than by mitigating their effects.']", "label": 3 }, { "id": "train_2069", "context": "Although the earliest surviving Greek inscriptions written in an alphabet date from the eighth century B. C. , the fact that the text of these Greek inscriptions sometimes runs from right to left and sometimes from left to right indicates that the Greeks adopted alphabetic writing at least two centuries before these inscriptions were produced. After all, the Greeks learned alphabetic writing from the Phoenicians, and presumably, along with the alphabet, they also adopted the then-current Phoenician practice with respect to the direction of text. And although Phoenician writing was originally inconsistent in direction, by the eighth century B. C. Phoenician was consistently written from right to left and had been for about two centuries .", "question": "In the argument given, the two portions in boldface play which of the following roles?", "answers": "['The first and the second each describe evidence that has been used to challenge the position that the argument seeks to establish.', 'The first provides evidence in support of the position that the argument seeks to establish; the second is that position.', 'The first is evidence that forms the basis for an objection to the position that the argument seeks to establish; the second is a consideration that is introduced to counter the force of that evidence.', 'The first and the second each provide evidence in support of the position that the argument seeks to establish.']", "label": 3 }, { "id": "train_2070", "context": "The expansion of large-scale farming in Africa and Asia has destroyed much of the natural vegetation on which elephants have historically depended, forcing them to turn to cultivated land to satisfy their enormous appetites. As a result, farmers have lost millions of dollars worth of crops annually. Yet even if elephant sanctuaries were created on a widespread basis to guarantee elephants sufficient natural vegetation, the raiding would likely persist, since", "question": "Which of the following most logically completes the argument below?", "answers": "['when elephants forage for food, they typically travel in herds.', 'Elephants tend to prefer cultivated crops to wild vegetation as a food source.', 'Some of the land where crops have suffered extensive damage from elephants has been allowed to return to its natural state.', 'Elephant sanctuaries are usually created in areas that are rich in the natural vegetation on which elephants have historically depended.']", "label": 1 }, { "id": "train_2071", "context": "Aid workers have worried that residents of the Eastern Gobi Refugee Camp might be at risk for beriberi, a severe deficiency of Thiamine, a B vitamin. Thiamine is found in high concentrations in cereal grains, and many of the residents have grain free diets because of allergies. Doctors and nutritionists assigned to investigate the situation have disagreed, noting that the residents grow sustenance amounts of potatoes, kale, and cauliflower on the surrounding farm lands, and these foods provide substantial sources of Thiamine as well. Because the residents of this refugee camp do not have any allergies to any of these latter foods, and crops have been good because of favorable weather conditions, their risk of beriberi is much lower than originally thought. ", "question": "In the argument given, the two portions in boldface play which of the following roles?", "answers": "['The first is evidence that seems to contradict the conclusion of the argument; the second is that conclusion.', 'The first is evidence in support of the conclusion of the argument; the second is evidence that contradicts the conclusion of the argument.', 'The first is evidence in support of the conclusion of the argument; the second is that conclusion.', 'The first is the conclusion of the argument; the second calls the conclusion of the argument into question.']", "label": 0 }, { "id": "train_2072", "context": "After the first bite, the Mystery Recipe Contest judge has detected cumin, red pepper flakes, and lemon grass in the winning dish. These seasonings have a tendency to be potent, producing distinct flavors. Since the quantity of any seasoning in a recipe depends directly on how potent that seasoning tastes in the dish, the judge has determined that the dish contains cumin, lemon grass, and red pepper flakes, in order of decreasing relative quantity.", "question": "The judge's argument relies on which of the following assumptions?", "answers": "['The winning dish contains at least one other seasoning ingredient besides cumin, lemon grass, and red pepper flakes.', 'The judge was unable to detect any other seasonings in the winning dish.', 'The bite that the judge took was representative of the whole dish.', 'Recipes that contain cumin generally contain other spices that are more potent.']", "label": 2 }, { "id": "train_2073", "context": "A society in which there are many crimes, such as thefts and murders, should not be called \"lawless. \" That is an abuse of the meaning of words. As a suffix, \"-less\" means \"without, \" so \"lawless\" means \"without laws. \" However, a society that has no laws has no crimes, because no laws can be broken. A lawless society would, ttherefore, be a crimeless society. So what some have termed a lawless society should actually be called \"crimeful. ", "question": "If the statements in the passage are true, which one of the following must also be true?", "answers": "['A society that has many laws has many crimes.', 'A society that has some crimes has some laws.', 'A society that has no crimes has no laws.', 'A society that has many crimes has many laws.']", "label": 1 }, { "id": "train_2074", "context": "The \"suicide wave\" that followed the United States stock market crash of October 1929 is more legend than fact. Careful examination of the monthly figures on the causes of death in 1929 shows that the number of suicides in October and in November was comparatively low. In only three other months were the monthly figures lower. During the summer months, when the stock market was flourishing, the number of suicides was substantially higher.", "question": "Which one of the following, if true, would best challenge the conclusion of the passage?", "answers": "['Because of seasonal differences, the number of suicides in October and November of 1929 would not be expected to be the same as those for other months.', 'During the years surrounding the stock market crash, suicide rates were typically lower at the beginning of any calendar year than toward the end of that year.', 'October and November have almost always had relatively high suicide rates, even during the 1920s and 1930s.', 'The suicide rate in October and November of 1929 was considerably higher than the average for those months during several preceding and following years.']", "label": 3 }, { "id": "train_2075", "context": "Naresh: In the near future we will be able to construct machines capable of conversing as humans do. Teaching computers English syntax is not as problematic as once thought, and we are making great strides in discovering what background knowledge these machines will require. Ashok: But being able to converse as humans do is not solely about possessing the correct syntax and background knowledge. It also involves the capacity to communicate the often emotional and confused knowledge one has; plainly, no computer will ever be able to do that.", "question": "Naresh and Ashok most clearly disagree on whether", "answers": "['there will ever be a computer capable of conversing as humans do', 'only humans communicate emotional and confused knowledge', 'computers will become more adept at communicating emotional and confused knowledge', 'in the near future humans and machines will be able to converse with one another']", "label": 0 }, { "id": "train_2076", "context": "Expert witness: Ten times, and in controlled circumstances, a single drop of the defendant' s blood was allowed to fall onto the fabric. And in all ten cases, the stained area was much less than the expected 9. 5 cm2. In fact, the stained area was always between 4. 5 and 4. 8 cm2. I conclude that a single drop of the defendant' s blood stains much less than 9. 5 cm2 of the fabric.", "question": "Which one of the following, if true, most undermines the value of the evidence for the expert witness's conclusion?", "answers": "[\"Expert witnesses have sometimes been known to fudge their data to accord with the prosecution's case.\", 'Not all expert witnesses are the authorities in their fields that they claim to be.', \"If similar results had been found after 100 test drops of the defendant's blood, the evidence would be even stronger.\", \"In an eleventh test drop of the defendant's blood, the area stained was also less than 9. 5 cm2 -- this time staining 9. 3 cm2.\"]", "label": 3 }, { "id": "train_2077", "context": "According to a government official involved in overseeing airplane safety during the last year, over 75 percent of the voice-recorder tapes taken from small airplanes involved in relatively minor accidents record the whistling of the pilot during the fifteen minutes immediately preceding the accident. Even such minor accidents pose some safety risk. Ttherefore, if passengers hear the pilot start to whistle they should take safety precautions, whether instructed by the pilot to do so or not.", "question": "The argument is most vulnerable to criticism on the grounds that it", "answers": "[\"ignores the fact that in nearly one-quarter of these accidents following the recommendation would not have improved passengers' safety\", 'fails to specify the percentage of all small airplane flights that involve relatively minor accidents', 'provides no information about the percentage of all small airplane flights during which the pilot whistles at some time during that flight', 'accepts the reliability of the cited statistics on the authority of an unidentified government of official']", "label": 2 }, { "id": "train_2078", "context": "Computer modeling of reasoning tasks is far easier than computer modeling of other cognitive tasks, such as the processing of sense images. Computers can defeat chess champions, but cannot see. So, it appears that we understand the analytical capabilities of our minds much better than we understand our senses.", "question": "Which one of the following principles, if valid, most helps to justify the reasoning above?", "answers": "['The degree of difficulty of constructing computer models of cognitive tasks is a good index of the degree of difficulty of performing those tasks.', \"A computer's defeat of a chess champion should count as an indication that the computer possesses true intelligence.\", 'The less difficult it is to construct a computer model of a process the better understood is that process.', \"The better we understand a computer's ability to perform a type of task, the better we will understand our own ability to perform it\"]", "label": 2 }, { "id": "train_2079", "context": "The public is aware of the possibility of biases in the mass media and distrusts the media as too powerful. The body of information against which the public evaluates the plausibility of each new media report comes, however, from what the public has heard of through the mass media.", "question": "If the view above is correct, it provides a reason for accepting which one of the following conclusions?", "answers": "['News reporters and their public hold largely the same views about what is most important in society, because news reporters come out of that society.', 'The biases that news media impose on reporting tend not to be conscious distortions but rather part of a sense they share about what is interesting and believable.', 'If there is a pervasive bias in the presentation of news by the mass media, it would be hard for the public to discern that bias.', 'The mass media tailor their reports to conform to a specific political agenda.']", "label": 2 }, { "id": "train_2080", "context": "Certain genetically modified strains of maize produce a powerful natural insecticide. The insecticide occurs throughout the plant, including its pollen. Maize pollen is dispersed by the wind and frequently blows onto milkweed plants that grow near maize fields. Caterpillars of monarch butterflies feed exclusively on milkweed leaves. When these caterpillars are fed milkweed leaves dusted with pollen from modified maize plants, they die. Ttherefore, by using genetically modified maize, farmers put monarch butterflies at risk.", "question": "Which of the following would it be most useful to determine in order to evaluate the argument?", "answers": "['Whether the natural insecticide is as effective against maize-eating insects as commercial insecticides typically used on maize are', 'Whether monarch butterfly caterpillars are actively feeding during the part of the growing season when maize is releasing pollen', \"Whether insects that feed on genetically modified maize plants are likely to be killed by insecticide from the plant's pollen\", 'Whether any maize-eating insects compete with monarch caterpillars for the leaves of milkweed plants growing near maize fields']", "label": 1 }, { "id": "train_2081", "context": "The people most likely to watch a televised debate between political candidates are the most committed members of the electorate and thus the most likely to have already made up their minds about whom to support. Furthermore, following a debate, uncommitted viewers are generally undecided about who won the debate. Hence, winning a televised debate does little to bolster one' s chances of winning an election.", "question": "The reasoning in the argument is most vulnerable to criticism because the argument fails to consider the possibility that", "answers": "[\"people's voting behavior may be influenced in unpredictable ways by comments made by the participants in a televised debate\", 'people who are committed to a particular candidate will vote even if their candidate is perceived as having lost a televised debate', 'the voting behavior of people who do not watch a televised debate is influenced by reports about the debate', 'watching an exciting debate makes people more likely to vote in an election']", "label": 2 }, { "id": "train_2082", "context": "Over the last 25 years, the average price paid for a new car has steadily increased in relation to average individual income. This increase indicates that individuals who buy new cars today spend, on average, a larger amount relative to their incomes buying a car than their counterparts did 25 years ago.", "question": "Which one of the following, if true, most weakens the argument?", "answers": "['Allowing for inflation, average individual income has significantly declined over the last 25 years.', 'There has been a significant increase over the last 25 years in the proportion of individuals in households with more than one wage earner.', 'During the last 25 years, annual new-car sales and the population have both increased, but new-car sales have increased by a greater percentage.', 'Sales to individuals make up a smaller proportion of all new-car sales than they did 25 years ago.']", "label": 3 }, { "id": "train_2083", "context": "Voting records regularly show that people over 65 vote in the highest percentages while young adults are least likely to vote. This indicates that citizens are becoming increasingly disconnected from the political system with each passing generation.", "question": "The argument's reasoning is questionable in that the argument", "answers": "['overlooks the possibility that voting patterns among age groups will change in the future', 'compares an early stage of one generation to a later stage of another', 'fails to take into account the relative sizes of the generations compared', 'confuses the cause of an effect with the effect itself']", "label": 1 }, { "id": "train_2084", "context": "A park' s user fees are employed to maintain the park. When fewer people use the park, it suffers less wear. Thus raising user fees improves park maintenance even if the number of people who stop using the park because of higher fees is great enough to reduce the revenues devoted to maintenance.", "question": "Which one of the following conforms most closely to the principle illustrated by the statements above?", "answers": "['By spending more on zoo maintenance, a city increases the number of zoo patrons. The extra revenue generated by the sale of memorabilia more than makes up for the extra costs of maintenance.', 'Road taxes are raised to encourage more people to use mass transit. But since the fee paid by each commuter does not equal the cost of providing transit for that commuter, a mass transit service will deteriorate even as it takes in more money.', \"To increase its market share, a car company improves the service warranty it provides to those who purchase a new car. Making good on the warranties proves expensive enough that the company's profits decrease even though its market share increases.\", 'Library fees have been increased to raise money for book repair. Since the library now has fewer patrons, the books are in better repair even though the number of library patrons has decreased to such an extent that the money available for book repair has decreased.']", "label": 3 }, { "id": "train_2085", "context": "Alan: Government subsidies have been proposed in Cariana to encourage farmers in Rochelle, the country' s principal agricultural region, to implement certain new farming techniques. Unless these techniques are implemented, erosion of productive topsoil cannot be controlled. Unfortunately, farmers cannot afford to shoulder the entire cost of the new techniques, which are more expensive than those currently used. Ttherefore, without subsidies, agricultural output in Rochelle will inevitably decline. Betty: But erosion in Rochelle is caused by recurring floods, which will end next year once Cariana completes the hydroelectric dam it is building across the region' s major river. Ttherefore, Rochelle' s total agricultural output will stabilize at its present level even without subsidies.", "question": "Which one of the following is an assumption on which Betty's argument depends?", "answers": "[\"Building a dam across Rochelle's major river will not reduce any recurrent flooding that occurs in regions of Cariana other than Rochelle.\", 'The cost to the government of Cariana to operate the hydroelectric dam will not be greater than the projected cost of subsidizing the farmers of Rochelle in the implementation of the new farming techniques.', \"The current yearly output, if any, from Rochelle's land that will be permanently under water once the dam is completed will at least be matched by additional yearly output from Rochelle's remaining land.\", \"The government of Cariana has sufficient financial resources both to subsidize its farmers' implementation of new farming techniques and to operate a hydroelectric dam.\"]", "label": 2 }, { "id": "train_2086", "context": "Residents at a retirement community always play card games for two hours, starting at 3: 00 p. m. every afternoon. Although it is clear that the residents take pleasure in this activity, they never continue their games for longer than two hours. Caregivers assume that since card games require great concentration and strain on the elderly residents' eyes, playing for more than two hours would give the residents headaches that would interfere with their ability to play bingo in the evenings.", "question": "Which of the following, if true, most seriously calls into question the caregivers' explanation?", "answers": "['Experts have proven that playing card games has tremendous health benefits for elderly people.', 'Residents at the retirement community are served dinner at 5:00 p. m. every evening.', 'Playing bingo takes just as much concentration and causes equal strain on the eyes as card games.', 'Most of the residents at the retirement community rank bingo at the top of their list of favorite activities.']", "label": 1 }, { "id": "train_2087", "context": "Any fruit that is infected is also rotten. No fruit that was inspected is infected. Ttherefore, any fruit that was inspected is safe to eat.", "question": "The conclusion of the argument follows logically if which one of the following is assumed?", "answers": "['It is safe to eat any fruit that is uninfected.', 'It is not safe to eat any fruit that is rotten.', 'It is not safe to eat any fruit that is infected.', 'It would have been safe to eat infected fruit if it had been inspected.']", "label": 0 }, { "id": "train_2088", "context": "Television network executive: Some scientists have expressed concern about the numerous highly popular television programs that emphasize paranormal incidents, warning that these programs will encourage superstition and thereby impede the public' s scientific understanding. But these predictions are baseless. Throughout recorded history, dramatists have relied on ghosts and spirits to enliven their stories, and yet the scientific understanding of the populace has steadily advanced.", "question": "The television network executive's argument is most vulnerable to criticism on which one of the following grounds?", "answers": "['It fails to consider that one phenomenon can steadily advance even when it is being impeded by another phenomenon.', 'It fails to consider that the occurrence of one phenomenon can indirectly affect the pervasiveness of another even if the former does not impede the latter.', 'It fails to consider that just because one phenomenon is known to affect another, the latter does not also affect the former.', 'It takes for granted that if a correlation has been observed between two phenomena, they must be causally connected.']", "label": 0 }, { "id": "train_2089", "context": "Most of the employees of the Compujack Corporation are computer programmers. Since most computer programmers receive excellent salaries from their employers, at least one Compujack employee must receive an excellent salary from Compujack.", "question": "Which one of the following arguments exhibits a flawed pattern of reasoning most similar to the flawed pattern of reasoning exhibited by the argument above?", "answers": [ "Most of Molly's classmates are gardeners with a great deal of patience. Since most of Molly's classmates are women, at least one female classmate of Molly's must be a gardener with a great deal of patience.", "Most gardeners are people with a great deal of patience. Since most of Molly's classmates are gardeners, at least one of Molly's classmates must be a person with a great deal of patience.", "Most gardeners are people with a great deal of patience. Since most of Molly's classmates are gardeners, at least one of Molly's classmates who is a gardener must be a person with a great deal of patience.", "Most gardeners are people with a great deal of patience. Since most of Molly's classmates who garden are women, at least one female classmate of Molly's must be a person with a great deal of patience." ], "label": 1 }, { "id": "train_2090", "context": "According to promotional material published by the city of Springfield, more tourists stay in hotels in Springfield than stay in the neighboring city of Harristown. A brochure from the largest hotel in Harristown claims that more tourists stay in that hotel than stay in the Royal Arms Hotel in Springfield. If both of these sources are accurate, however, the county' s \"Report on Tourism\" must be in error in indicating that __.", "question": "Which of the following most logically completes the argument below?", "answers": "['some hotels in Harristown have fewer tourist guests each year than the Royal Arms Hotel has', 'more tourists stay in hotel accommodations in Harristown than stay in the Royal Arms Hotel', 'the Royal Arms Hotel is the only hotel in Springfield', 'some of the tourists who have stayed in hotels in Harristown have also stayed in the Royal Arms Hotel']", "label": 2 }, { "id": "train_2091", "context": "Extinction is inevitable for all biological species. In fact, the vast majority of all species that have ever lived are now extinct. Since all species die out eventually, there is no justification for trying to protect species that are presently endangered, even those that can be saved from extinction now.", "question": "The reasoning in the argument above is most closely paralleled by the argument that there is no reason to", "answers": "['spend money on preventive maintenance of a car because no car can last indefinitely', 'look for a book in the library because it is sometimes checked out', 'take a route that will avoid the normal traffic jams because traffic jams can occur along any route', 'plant a flower garden in soil that is not beneficial to plants because the plants are likely to die in such soil']", "label": 0 }, { "id": "train_2092", "context": "It is theoretically possible that bacteria developed on Mars early in its history and that some were carried to Earth by a meteorite. However, strains of bacteria from different planets would probably have substantial differences in protein structure that would persist over time, and no two bacterial strains on Earth are different enough to have arisen on different planets. So, even if bacteria did arrive on Earth from Mars, they must have died out.", "question": "The argument is most vulnerable to which of the following criticisms?", "answers": "['It fails to consider whether all bacteria now on Earth could have arisen from transported Martian bacteria.', 'It fails to consider whether there could have been strains of bacteria that originated on Earth and later died out.', 'It fails to consider whether there were means other than meteorites by which Martian bacteria could have been carried to Earth.', 'It fails to establish whether bacteria actually developed on Mars.']", "label": 0 }, { "id": "train_2093", "context": "In Brindon County, virtually all of the fasteners-such as nuts, bolts, and screws-used by workshops and manufacturing firms have for several years been supplied by the Brindon Bolt Barn, a specialist wholesaler. In recent months many of Brindon County' s workshops and manufacturing firms have closed down, and no new ones have opened. Ttherefore, the Brindon Bolt Barn will undoubtedly show a sharp decline in sales volume and revenue for this year as compared to last year.", "question": "The argument depends on assuming which of the following?", "answers": "['The workshops and manufacturing firms that have remained open have a smaller volume of work to do this year than they did last year.', \"Last year the Brindon Bolt Barn's sales volume and revenue were significantly higher than they had been the previous year.\", 'The Brindon Bolt Barn is not a company that gets the great majority of its business from customers outside Brindon County.', \"Soon the Brindon Bolt Barn will no longer be the only significant supplier of fasteners to Brindon County's workshops.\"]", "label": 2 }, { "id": "train_2094", "context": "Studies show that the most creative engineers get their best and most useful ideas only after doodling and jotting down what turn out to be outlandish ideas. Now that many engineers do their work with computers instead of on paper, however, doodling is becoming much less common, and some experts fear that the result will be fewer creative and useful engineering ideas. These experts argue that this undesirable consequence would be avoided if computer programs for engineering work included simulated notepads that would allow engineers to suspend their \"serious\"work on the computer, type up outlandish ideas, and then quickly return to their original work.", "question": "Which one of the following is an assumption on which the experts' reasoning depends?", "answers": "['No engineers who work with computers keep paper and pencils near their computers in order to doodle and jot down ideas.', 'The physical act of working on paper is not essential in providing engineers with the benefits that can be gained by doodling.', 'Simulated notepads would not be used by engineers for any purpose other than typing up outlandish ideas.', 'Most of the outlandish ideas engineers jot down while doodling are later incorporated into projects that have practical applications.']", "label": 1 }, { "id": "train_2095", "context": "University budget committee: Athletes experience fewer injuries on artificial-turf athletic fields than on natural-grass fields. Additionally, natural-grass fields are more expensive to maintain than fields made of artificial turf. Nevertheless, this committee recommends replacing the university' s current artificial-turf field with a natural-grass field.", "question": "Which one of the following, if true, most helps to resolve the apparent discrepancy in the committee's position?", "answers": "[\"The university's current artificial-turf athletic field has required extensive maintenance since its original installation.\", 'Maintaining artificial-turf fields involves the occasional replacement of damaged sections of turf, whereas natural-grass fields require daily watering and periodic fertilization.', 'Most injuries sustained on artificial-turf fields take longer to heal and require more expensive physical therapy than do injuries sustained on natural-grass fields.', 'Athletes who have spent most of their playing time on natural-grass fields generally prefer not to play on artificial-turf fields.']", "label": 2 }, { "id": "train_2096", "context": "Copyright laws protect the rights of writers to profits earned from their writings, whereas patent laws protect inventors' rights to profits earned from their inventions. In Jawade, when computer-software writers demanded that their rights to profit be protected, the courts determined that information written for a machine does not fit into either the copyright or the patent category. Clearly, ttherefore, the profit rights of computer-software writers remain unprotected in Jawade.", "question": "Which one of the following is an assumption on which the argument depends?", "answers": "[\"Computer-software writers are not an influential enough group in Jawade for the government to consider modifying existing copyright laws in order to protect this group's profit rights.\", 'No laws exist, other than copyright laws and patent laws, that would protect the profit rights of computer-software writers in Jawade.', 'Copyright laws and patent laws in Jawade have not been modified since their original adoption.', 'Most of the computer software used in Jawade is imported from other countries.']", "label": 1 }, { "id": "train_2097", "context": "Although Pluto has an atmosphere and is much larger than any asteroid, Pluto is not a true planet. Pluto formed in orbit around the planet Neptune and was then ejected from orbit around Neptune when Triton, Neptune' s largest moon, was captured by Neptune' s gravity.", "question": "The conclusion of the argument follows logically if which one of the following is assumed?", "answers": "['If Pluto had not been ejected from its orbit around Neptune, Pluto would not have its current orbit around the sun and would still be a moon.', 'For a celestial body to be a true planet it must have formed in orbit around the sun exclusively.', 'No celestial body can simultaneously be a moon and a planet.', 'Not all celestial bodies that have an atmosphere and orbit the sun are true planets.']", "label": 1 }, { "id": "train_2098", "context": "Polling data reveal that an overwhelming majority of nine-year-olds can correctly identify the logos of major cigarette brands. However, of those nine-year-olds who recognize such logos, less than 1 percent smoke. Ttherefore, there is little or no connection between recognition of cigarette brand logos and smoking.", "question": "Which one of the following uses flawed reasoning most similar to the flawed reasoning above?", "answers": "['A large percentage of men exercise moderately throughout their lives, but the average life span of those who do so is not significantly greater than of those who get little or no exercise. So there is little or no correlation between moderate exercise and good health.', 'The concern about the long-term effect on dolphins of small quantities of mercury in the ocean is unfounded. During a three-month observation period, 1, 000 dolphins were exposed to small quantities of mercury in seawater, with no effect on the animals.', 'Most dentists recommend using fluoride to reduce the incidence of cavities, but few recommend giving up candy entirely; so, using fluoride is probably more effective in preventing cavities than is avoiding sweets.', 'Many ten-year-olds dream of becoming actors. Yet it is not likely they will seriously consider becoming actors, because most parents discourage their children from pursuing such a highly competitive career.']", "label": 1 }, { "id": "train_2099", "context": "Gigantic passenger planes currently being developed will have enough space to hold shops and lounges in addition to passenger seating. However, the additional space will more likely be used for more passenger seating. The number of passengers flying the air-traffic system is expected to triple within 20 years, and it will be impossible for airports to accommodate enough normal-sized jet planes to carry that many passengers.", "question": "Which one of the following most accurately states the conclusion drawn in the argument?", "answers": "['Gigantic planes currently being developed will have enough space in them to hold shops and lounges as well as passenger seating.', 'The additional space in the gigantic planes currently being developed is more likely to be filled with passenger seating than with shops and lounges.', 'The number of passengers flying the air-traffic system is expected to triple within 20 years.', 'In 20 years, it will be impossible for airports to accommodate enough normal-sized planes to carry the number of passengers that are expected to be flying then.']", "label": 1 }, { "id": "train_2100", "context": "Advertisement: Most power hedge trimmers on the market do an adequate job of trimming hedges, but many power hedge trimmers are dangerous to operate and can cause serious injury when used by untrained operators. Bolter Industries' hedge trimmer has been tested by National Laboratories, the most trusted name in safety testing. So you know, if you buy a Bolter' s, you are buying a power hedge trimmer whose safety is assured.", "question": "The answer to which one of the following questions would be most useful in evaluating the truth of the conclusion drawn in the advertisement?", "answers": "[\"What were the results of National Laboratories' tests of Bolter Industries' hedge trimmer?\", 'Has National Laboratories performed safety tests on other machines made by Bolter Industries?', \"Does any other power hedge trimmer on the market do a better job of trimming hedges than does Bolter Industries' hedge trimmer?\", 'How important to the average buyer of a power hedge trimmer is safety of operation?']", "label": 0 }, { "id": "train_2101", "context": "Some anthropologists argue that the human species could not have survived prehistoric times if the species had not evolved the ability to cope with diverse natural environments. However, there is considerable evidence that Australopithecus afarensis, a prehistoric species related to early humans, also thrived in a diverse array of environments, but became extinct. Hence, the anthropologists' claim is false.", "question": "The reasoning in the argument is most vulnerable to criticism on the grounds that the argument", "answers": "[\"confuses a condition's being required for a given result to occur in one case with the condition's being sufficient for such a result to occur in a similar case\", 'fails to consider the possibility that, even if a condition caused a result to occur in one case, it was not necessary to cause the result to occur in a similar case', 'generalizes, from the fact that one species with a certain characteristic survived certain conditions, that all related species with the same characteristic must have survived exactly the same conditions', 'fails to consider the possibility that Australopithecus afarensis had one or more characteristics that lessened its chances of surviving prehistoric times']", "label": 0 }, { "id": "train_2102", "context": "Data from satellite photographs of the tropical rain forest in Melonia show that last year the deforestation rate of this environmentally sensitive zone was significantly lower than in previous years. The Melonian government, which spent millions of dollars last year to enforce laws against burning and cutting of the forest, is claiming that the satellite data indicate that its increased efforts to halt the destruction are proving effective.", "question": "Which one of the following, if true, most seriously undermines the government's claim?", "answers": "['Much of the money that was designated last year for forest preservation has been spent on research and not on enforcement.', 'The inaccessibility of much of the rain forest has made it impossible to confirm the satellite data by direct observation from the field.', 'Rainfall during the usually dry 6-month annual burning season was abnormally heavy last year.', \"Landowner opposition to the government's antideforestation efforts grew more violent last year in response to the increased enforcement.\"]", "label": 2 }, { "id": "train_2103", "context": "Statistician: A financial magazine claimed that its survey of its subscribers showed that North Americans are more concerned about their personal finances than about politics. One question was: \"Which do you think about more: politics or the joy of earning money? \" This question is clearly biased. Also, the readers of the magazine are a self-selecting sample. Thus, there is reason to be skeptical about the conclusion drawn in the magazine' s survey.", "question": "Each of the following, if true, would strengthen the statistician's argument EXCEPT:", "answers": "['Other surveys suggest that North Americans are just as concerned about politics as they are about finances.', 'The conclusions drawn in most magazine surveys have eventually been disproved.', 'Other surveys suggest that North Americans are concerned not only with politics and finances, but also with social issues.', 'The credibility of the magazine has been called into question on a number of occasions.']", "label": 2 }, { "id": "train_2104", "context": "Curator: The decision to restore the cloak of the central figure in Veronese' s painting from its present red to the green found underneath is fully justified. Reliable x-ray and chemical tests show that the red pigment was applied after the painting had been completed, and that the red paint was not mixed in Veronese' s workshop. Hence it appears likely that an artist other than Veronese tampered with Veronese' s painting after its completion. Art critic: But in a copy of Veronese' s painting made shortly after Veronese died, the cloak is red. It is highly unlikely that a copyist would have made so major a change so soon after Veronese' s death.", "question": "The assertion that a later artist tampered with Veronese's painting serves which one of the following functions in the curator's argument?", "answers": "['It is a clarification of a key term of the argument.', 'It is a particular instance of the general position to be defended.', \"It is a subsidiary conclusion that supports the argument's main conclusion.\", 'It is a reiteration of the main point that is made for the sake of emphasis.']", "label": 2 }, { "id": "train_2105", "context": "Essayist: Politicians deserve protection from a prying press. No one wants his or her private life spread across the pages of the newspapers. Furthermore, the press' s continual focus on politicians' private lives dissuades talented people from pursuing a career in politics and turns reporters into character cops who walk their beats looking for minute and inconsequential personality flaws in public servants. It is time to put a halt to this trivial journalism.", "question": "Each of the following, if true, strengthens the essayist's argument EXCEPT:", "answers": "[\"Much writing on politicians'private lives consists of rumors circulated by opposing candidates.\", \"Politicians' personality flaws often ultimately affect their performance on the job.\", 'In recent elections, the best local politicians have refused to run for national office because of the intrusiveness of press coverage.', \"The press is unusually inaccurate when it reports on people's private lives.\"]", "label": 1 }, { "id": "train_2106", "context": "When there is less rainfall than normal, the water level of Australian rivers falls and the rivers flow more slowly. Because algae whose habitat is river water grow best in slow-moving water, the amount of algae per unit of water generally increases when there has been little rain. By contrast, however, following a period of extreme drought, algae levels are low even in very slow-moving river water.", "question": "Which of the following, if true, does most to explain the contrast described above?", "answers": "['Except during periods of extreme drought, algae levels tend to be higher in rivers in which the flow has been controlled by damming than in rivers that flow freely.', 'Australian rivers dry up completely for short intervals in periods of extreme drought.', 'During periods of extreme drought, the populations of some of the species that feed on algae tend to fall.', 'When algae populations reach very high levels, conditions within the river can become toxic for some of the other species that normally live there.']", "label": 1 }, { "id": "train_2107", "context": "Lawyer: This witness acknowledges being present at the restaurant and watching when my client, a famous television personality, was assaulted. Yet the witness claims to recognize the assailant, but not my famous client. Ttherefore, the witness' s testimony should be excluded.", "question": "The lawyer's conclusion follows logically if which one of the following is assumed?", "answers": "[\"There are other witnesses who can identify the lawyer's client as present during the assault.\", 'The testimony of a witness to an assault should be included only if the witness claims to recognize both parties involved in the assault.', 'It is impossible to determine whether the witness actually recognized the assailant.', \"It is unlikely that anyone would fail to recognize the lawyer's client.\"]", "label": 1 }, { "id": "train_2108", "context": "Because it was long thought that few people would watch lengthy televised political messages, most televised political advertisements, like commercial advertisements, took the form of short messages. Last year, however, one candidate produced a half-hour-long advertisement. At the beginning of the half-hour slot a substantial portion of the viewing public had tuned in to that station. Clearly, then, many more people are interested in lengthy televised political messages than was previously thought.", "question": "Which of the following, if true, most seriously weakens the argument?", "answers": "['The candidate who produced the half-hour-long advertisement did not win election at the polls.', \"In general a regular-length television program that features debate about current political issues depends for its appeal on the personal qualities of the program's moderator.\", 'The half-hour-long advertisement was widely publicized before it was broadcast.', 'The half-hour-long advertisement was aired during a time slot normally taken by one of the most popular prime-time shows.']", "label": 3 }, { "id": "train_2109", "context": "Cecile' s association requires public disclosure of an officer' s investments in two cases only: when an officer is authorized to disburse association funds, and when an officer sits on the board of a petrochemical company. Cecile, an officer who is not authorized to disburse funds, sits on the board of just one company, a small timber business. Ttherefore, there is no reason for Cecile to publicly disclose her investments at this time.", "question": "The conclusion of the argument follows logically if which one of the following is assumed?", "answers": "[\"The association's requirements provide the only reasons there might be for Cecile to disclose her investments.\", \"Cecile's office and her position on the timber business's board create no conflicts of interest.\", 'Cecile owns no investments in the petrochemical industry.', 'The timber business on whose board Cecile sits is owned by a petrochemical company.']", "label": 0 }, { "id": "train_2110", "context": "The gray squirrel, introduced into local woodlands ten years ago, threatens the indigenous population of an endangered owl species, because the squirrels' habitual stripping of tree bark destroys the trees in which the owls nest. Some local officials have advocated setting out poison for the gray squirrels. The officials argue that this measure, while eliminating the squirrels, would pose no threat to the owl population, since the poison would be placed in containers accessible only to squirrels and other rodents.", "question": "Which one of the following, if true, most calls into question the officials' argument?", "answers": "['The owls whose nesting sites are currently being destroyed by the gray squirrels feed primarily on rodents.', \"The officials' plan entails adding the poison to food sources that are usually eaten by rodents but not by other animals.\", 'No indigenous population of any other bird species apart from the endangered owls is threatened by the gray squirrels.', 'The owls that are threatened build their nests in the tops of trees, but the gray squirrels strip away bark from the trunks.']", "label": 0 }, { "id": "train_2111", "context": "To be horrific, a monster must be threatening. Whether or not it presents psychological, moral or social dangers, or triggers enduring infantile fears, if a monster is physically dangerous then it is threatening. In fact, even a physically benign monster is horrific if it inspires revulsion.", "question": "Which one of the following logically follows from the statements above?", "answers": "['Any horror-story monster that is threatening is also horrific.', 'If a monster triggers infantile fears but is not physically dangerous, then it is not horrific.', 'All monsters that are not physically dangerous, but that are psychologically dangerous and inspire revulsion, are threatening.', 'If a monster is both horrific and psychologically threatening, then it does not inspire revulsion.']", "label": 2 }, { "id": "train_2112", "context": "The arousal of anger is sometimes a legitimate artistic aim, and every legitimate artwork that has this aim calls intentionally for concrete intervention in the world. Even granting that most art is concerned with beauty in some way, it follows that those critics who maintain that a concern for beauty is a characteristic of all legitimate art are mistaken.", "question": "The conclusion of the argument follows logically if which one of the following is assumed?", "answers": "['No works of art that call for intervention are concerned with beauty.', 'Only works that call for intervention are legitimate works of art.', 'Only those works that are exclusively concerned with beauty are legitimate works of art.', 'There are works that are concerned with beauty but that are not legitimate works of art.']", "label": 0 }, { "id": "train_2113", "context": "We are in a new industrial revolution that requires management trainees to develop \"action learning\" from real experience within business and industry, rather than getting tied up with theory and academia. Business schools seem unable, on their own, to tear themselves away from their largely academic roots and move closer to the realities of today' s business and industry; too often, trainees in business schools find themselves studying hypothetical cases instead of real ones. Furthermore, business schools have been slow to respond to the needs of business. Ttherefore, business schools should allow business executives to set curricula for management trainees that could then be taught by academics.", "question": "The argument relies on which one of the following assumptions?", "answers": "['Academics in business schools deal only with hypothetical situations in their business case studies.', 'Academics are not capable of teaching curricula suitable for relevant management training.', \"Today's business executives have valuable insight into business that academics in business schools do not have.\", 'Academics in business schools have no practical business experience that is valuable.']", "label": 2 }, { "id": "train_2114", "context": "Although there is no record of poet Edmund Spenser's parentage, we do know that as a youth Spenser attended the Merchant Tailors' School in London for a period between 1560 and 1570. Records from this time indicate that the Merchant Tailors' Guild then had only three members named Spenser: Robert Spenser, listed as a gentleman; Nicholas Spenser, elected the Guild's Warden in 1568; and John Spenser, listed as a \"journeyman cloth-maker. \" Of these, the last was likely the least affluent of the three-and most likely Edmund's father, since school accounting records list Edmund as a scholar who attended the school at a reduced fee.", "question": "Which of the following is an assumption on which the argument depends?", "answers": "[\"The fact that Edmund Spenser attended the Merchant Tailors' School did not necessarily mean that he planned to become a tailor.\", 'No member of the Guild could become Guild warden in sixteenth century London unless he was a gentleman.', \"Anybody in sixteenth century London who made clothing professionally would have had to be a member of the Merchant Tailors' Guild.\", \"The Merchant Tailors' School did not reduce its fees for the children of the more affluent Guild members.\"]", "label": 3 }, { "id": "train_2115", "context": "Moderate exercise lowers the risk of blockage of the arteries due to blood clots, since anything that lowers blood cholesterol levels also lowers the risk of hardening of the arteries, which in turn lowers the risk of arterial blockage due to blood clots; and, if the data reported in a recent study are correct, moderate exercise lowers blood cholesterol levels.", "question": "The conclusion drawn above follows logically if which one of the following is assumed?", "answers": "['The data reported in the recent study are correct.', 'The recent study investigated the relationship between exercise and blood cholesterol levels.', 'Hardening of the arteries increases the risk of blockage of the arteries due to blood clots.', 'Lowering blood cholesterol levels lowers the risk of blockage of the arteries.']", "label": 0 }, { "id": "train_2116", "context": "Many scientists believe that bipedal locomotion (walking on two feet) evolved in early hominids in response to the move from life in dense forests to life in open grasslands. Bipedalism would have allowed early hominids to see over tall grasses, helping them to locate food and to detect and avoid predators. However, because bipedalism also would have conferred substantial advantages upon early hominids who never left the forest -- in gathering food found within standing reach of the forest floor, for example -- debate continues concerning its origins. It may even have evolved, like the upright threat displays of many large apes, because it bettered an individual' s odds of finding a mate.", "question": "Which one of the following statements is most supported by the information above?", "answers": "['Bipedal locomotion would have helped early hominids gather food.', 'For early hominids, forest environments were generally more hospitable than grassland environments.', 'For early hominids, gathering food was more relevant to survival than was detecting and avoiding predators.', 'Bipedal locomotion actually would not be advantageous to hominids living in open grassland environments.']", "label": 0 }, { "id": "train_2117", "context": "Over 40, 000 lead seals from the early Byzantine Empire remain today. Apart from the rare cases where the seal authenticated a document of special importance, most seals had served their purpose when the document was opened. Lead was not expensive, but it was not free: most lead seals would have been recast once they had served their purpose. Thus the number of early Byzantine documents sealed in such a fashion must have been many times the number of remaining lead seals.", "question": "Which one of the following statements, if true, most strengthens the argument?", "answers": "['Most of the lead seals produced during the early Byzantine Empire were affixed to documents that have since been destroyed.', 'During the time of the early Byzantine Empire there were fewer than 40, 000 seals affixed to documents at any given time.', 'Most of the lead seals produced during the early Byzantine Empire were affixed to documents that were then opened during that period.', 'During the time of the early Byzantine Empire there were at most 40, 000 documents of enough importance to prevent the removing and recycling of the seal.']", "label": 2 }, { "id": "train_2118", "context": "Critic: Although some people claim it is inconsistent to support freedom of speech and also support legislation limiting the amount of violence in TV programs, it is not. We can limit TV program content because the damage done by violent programs is more harmful than the decrease in freedom of speech that would result from the limitations envisioned by the legislation.", "question": "Which one of the following principles, if valid, most helps to justify the critic's reasoning?", "answers": "['One can support freedom of speech while at the same time recognizing that it can sometimes be overridden by other interests.', 'When facing a choice between restricting freedom of speech or not, we must decide based on what would make the greatest number of people the happiest.', 'If the exercise of a basic freedom leads to some harm, then the exercise of that freedom should be restricted.', 'In evaluating legislation that would impinge on a basic freedom, we should consider the consequences of not passing the legislation.']", "label": 0 }, { "id": "train_2119", "context": "Alcohol consumption has been shown to be a health hazard; ttherefore, governments should ban all alcohol sales.", "question": "Which one of the following principles, if established, most strongly supports the argument?", "answers": "['The government should not permit people to use products that pose health hazards.', 'People will tend to decide not to use harmful products without government assistance.', \"Smoking is more hazardous to a person's health than is alcohol consumption.\", 'People will use harmful products unless the government steps in.']", "label": 0 }, { "id": "train_2120", "context": "Activist: Although the environmental bill before the legislature is popular with voters, it will have mainly negative economic consequences if it is passed, especially when we try to lure new businesses to our country. Great leaders have the courage to look beyond popularity to what is sound policy; legislators ought to do the same by not voting for this bill.", "question": "The activist's argumentation is most vulnerable to criticism on the grounds that it", "answers": "['takes for granted that if a bill is popular, it will not be sound economically', 'presumes, without providing justification, that a bill is less likely to pass if it is deemed to have negative economic consequences', 'fails to consider whether there are noneconomic reasons for supporting the bill that outweigh the reason given for rejecting it', 'presumes, without providing justification, that most of the legislators are great leaders']", "label": 2 }, { "id": "train_2121", "context": "Doctor: There will be more local cases of flu infection this year than there were last year. In addition to the strains of flu that were present in this area last year, a new strain has infected some people this year.", "question": "The conclusion of the doctor's argument can be properly drawn if which one of the following is assumed?", "answers": "['It is rare for new strains of flu to appear.', 'The new strain of flu cannot be addressed with the approaches used to deal with the strains of flu that were present last year.', 'There will be no decline this year in the number of cases of flu infection due to strains that were present last year.', 'The new strain of flu is expected to be more dangerous than the strains of flu that were present last year.']", "label": 2 }, { "id": "train_2122", "context": "Insufficient rain can cause crops to falter and agricultural prices to rise. Records indicate that during a certain nation' s recent crisis, faltering crops and rising agricultural prices prompted the government to take over food distribution in an effort to prevent starvation. Thus, the weather must have played an important role in bringing about the crisis.", "question": "The argument's reasoning is most vulnerable to criticism on the grounds that the argument", "answers": "['infers, merely from the fact that one event could have caused a second event, that the first event in fact caused the second', 'uses the term \"crisis\" equivocally in the reasoning, referring to both a political crisis and an economic crisis', \"concludes, merely from the fact that the period of insufficient rain occurred before the nation's crisis, that insufficient rain caused the nation's crisis\", \"fails to take into account the possibility that the scarcity was not severe enough to justify the government's taking over food distribution\"]", "label": 0 }, { "id": "train_2123", "context": "Rachel: Though contemporary artists are pleased to be free of the constraints that bound their predecessors, this freedom has caused a decline in the quality of art. Great art can be produced only when artists struggle to express themselves within externally imposed boundaries. James: People have always been critical of the art of their own time. They forget all but the greatest art from past eras. Since inferior contemporary artworks have not yet been forgotten, people today mistakenly think that contemporary art is generally inferior to earlier art.", "question": "On the basis of their statements, Rachel and James are committed to disagreeing with each other about whether", "answers": "['great art is produced only when an artist struggles against limitations', 'inferior art from past eras is generally forgotten', 'one can correctly assess the quality of art only if it was produced in past eras', 'contemporary art is of lower quality than earlier art']", "label": 3 }, { "id": "train_2124", "context": "A recent study revealed that the percentage of people treated at large, urban hospitals who recover from their illnesses is lower than the percentage for people treated at smaller, rural hospitals.", "question": "Each of the following, if true, contributes to an explanation of the difference in recovery rates EXCEPT:", "answers": "['Because space is relatively scarce in large hospitals, doctors are encouraged to minimize the length of time that patients are held for observation following a medical procedure.', 'The less friendly, more impersonal atmosphere of large hospitals can be a source of stress for patients at those hospitals.', \"Although large hospitals tend to draw doctors trained at the more prestigious schools, no correlation has been found between the prestige of a doctor's school and patients' recovery rate.\", 'Doctors at large hospitals tend to have a greater number of patients and consequently less time to explain to staff and to patients how medications are to be administered.']", "label": 2 }, { "id": "train_2125", "context": "Millions of homes are now using low-energy lighting, but millions more have still to make the switch, a fact that the government and the home lighting industry are eager to change. Although low-wattage bulbs cost more per bulb than normal bulbs, their advantages to the homeowner are enormous, and ttherefore everyone should use low-wattage bulbs.", "question": "Information about which one of the following would be LEAST useful in evaluating the argument?", "answers": "['the opinion of current users of low-wattage bulbs as to their effectiveness', 'the profits the home lighting industry expects to make from sales of low-wattage bulbs', 'the specific cost of a low-wattage bulb compared with that of a normal bulb', 'the actual cost of burning low-wattage bulbs compared to that of burning normal bulbs']", "label": 1 }, { "id": "train_2126", "context": "Historian: Newton developed mathematical concepts and techniques that are fundamental to modern calculus. Leibniz developed closely analogous concepts and techniques. It has traditionally been thought that these discoveries were independent. Researchers have, however, recently discovered notes of Leibniz' s that discuss one of Newton' s books on mathematics. Several scholars have argued that since the book includes a presentation of Newton' s calculus concepts and techniques, and since the notes were written before Leibniz' s own development of calculus concepts and techniques, it is virtually certain that the traditional view is false. A more cautious conclusion than this is called for, however. Leibniz' s notes are limited to early sections of Newton' s book, sections that precede the ones in which Newton' s calculus concepts and techniques are presented.", "question": "In the historian's reasoning, the two portions in boldface play which of the following roles?", "answers": "['The first has been used in support of a position that the historian rejects; the second is a conclusion that the historian draws from that position.', 'The first provides evidence in support of a position that the historian defends; the second is that position.', 'The first is evidence that has been used to support a conclusion about which the historian expresses reservations; the second is that conclusion.', 'The first and the second each provide evidence in support of a position that the historian defends.']", "label": 2 }, { "id": "train_2127", "context": "If the majority of the residents of the apartment complex complain that their apartments are infested with ants, then the management of the complex will have to engage the services of an exterminator. But the majority of the residents of the complex indicate that their apartments are virtually free of ants. Ttherefore, the management of the complex will not have to engage the services of an exterminator.", "question": "Which one of the following arguments contains a flawed pattern of reasoning parallel to that contained in the argument above?", "answers": "[\"Either the school's principal or two-thirds of the parent council must approve a change in the school dress code in order for the code to be changed. Since the principal will not approve a change in the dress code, the code will not be changed.\", 'Most employees will attend the company picnic if the entertainment committee is successful in getting a certain band to play at the picnic. But that band will be out of the country on the day of the picnic, so it is not true that most employees will attend.', 'The number of flights operated by the airlines cannot be reduced unless the airlines can collect higher airfares. But people will not pay higher airfares, so it is not the case that the number of flights will be reduced.', \"A theater will be constructed in the fall if funds collected are at least sufficient to cover its cost. To date, the funds collected exceed the theater's cost, so the theater will be constructed in the fall.\"]", "label": 1 }, { "id": "train_2128", "context": "Laird: Pure research provides us with new technologies that contribute to saving lives. Even more worthwhile than this, however, is its role in expanding our knowledge and providing new, unexplored ideas. Kim: Your priorities are mistaken. Saving lives is what counts most of all. Without pure research, medicine would not be as advanced as it is.", "question": "Laird and Kim disagree on whether pure research", "answers": "['expands the boundaries of our knowledge of medicine', 'should have the saving of human lives as an important goal', 'has its most valuable achievements in medical applications', 'derives its significance in part from its providing new technologies']", "label": 2 }, { "id": "train_2129", "context": "For years, university administrators, corporations, and government agencies have been predicting an imminent and catastrophic shortage of scientists and engineers. But since there is little noticeable upward pressure on the salaries of scientists and engineers, and unemployment is as high in these fields as any other, these doomsayers are turning out to be wrong.", "question": "Which one of the following would, if true, most strengthen the argument above?", "answers": "['The proportion of all research in science and engineering being carried out by corporations is larger than it was five years ago.', 'The knowledge and skills acquired during university programs in science and engineering need to be kept current through periodic retraining and professional experience.', 'Most students choose fields of study that offer some prospect of financial success.', 'The number of students in university programs in science and engineering has increased significantly in the last five years.']", "label": 3 }, { "id": "train_2130", "context": "The dean of computing must be respected by the academic staff and be competent to oversee the use of computers on campus. The only deans whom academics respect are those who hold doctoral degrees, and only someone who really knows about computers can competently oversee the use of computers on campus. Furthermore, the board of trustees has decided that the dean of computing must be selected from among this university' s staff. Ttherefore, the dean of computing must be a professor from this university' s computer science department.", "question": "Which one of the following is an assumption on which the argument depends?", "answers": "['All academics who hold doctoral degrees are respected by their academic colleagues.', 'Academics respect only people who hold doctoral degrees.', 'At this university, every professor who holds a doctoral degree in computer science really knows about computers.', \"Among this university's staff members with doctoral degrees, only those in the computer science department really know about computers.\"]", "label": 3 }, { "id": "train_2131", "context": "Tea made from camellia leaves is a popular beverage. However, studies show that regular drinkers of camellia tea usually suffer withdrawal symptoms if they discontinue drinking the tea. Furthermore, regular drinkers of camellia tea are more likely than people in general to develop kidney damage. Regular consumption of this tea, ttherefore, can result in a heightened risk of kidney damage.", "question": "Which one of the following, if true, most seriously weakens the argument?", "answers": "['Many people who regularly consume camellia tea also regularly consume other beverages suspected of causing kidney damage.', 'Most people who regularly drink camellia tea do not develop kidney damage.', 'Addictive chemicals are unlikely to cause kidney damage solely by virtue of their addictive qualities.', 'Several other popular beverages contain the same addictive chemical that is found in camellia tea.']", "label": 0 }, { "id": "train_2132", "context": "A new process enables ordinary table salt to be fortified with iron. This advance could help reduce the high incidence of anemia in the world' s population due to a deficiency of iron in the diet. Salt is used as a preservative for food and a flavor enhancer all over the globe, and people consume salt in quantities that would provide iron in significant amounts.", "question": "Which one of the following most accurately describes the role played in the argument by the statement that people consume salt in quantities that would provide iron in significant amounts?", "answers": "['It illustrates a principle that underlies the argument.', 'It is a claim that the argument is directed against.', 'It is the conclusion of the argument.', 'It provides support for the conclusion of the argument.']", "label": 3 }, { "id": "train_2133", "context": "In Western economies, more energy is used to operate buildings than to operate transportation. Much of the decline in energy consumption since the oil crisis of 1973 is due to more efficient use of energy in homes and offices. New building technologies, which make lighting, heating, and ventilation systems more efficient, have cut billions of dollars from energy bills in the West. Since energy savings from these efficiencies save several billion dollars per year today, we can conclude that 50 to 100 years from now they will save more than $200 billion per year (calculated in current dollars).", "question": "On which one of the following assumptions does the argument rely?", "answers": "['Technology used to make buildings energy efficient will not become prohibitively expensive over the next century.', 'Energy-efficient technologies based on new scientific principles will be introduced in the next 50 to 100 years.', 'Energy bills in the West will be $200 billion lower in the next 50 to 100 years.', 'Another oil crisis will occur in the next 50 to 100 years.']", "label": 0 }, { "id": "train_2134", "context": "There are two supposedly conflicting hypotheses as to what makes for great national leaders: one is that such leaders successfully shape public opinion, and the other is that they are adept at reacting to it. However, treating these hypotheses as mutually exclusive is evidently a mistake. All leaders who have had success getting their programs passed by their country' s legislature have been adroit both in shaping and reacting to public opinion.", "question": "Which one of the following is an assumption on which the argument depends?", "answers": "['Having success getting programs passed by the legislature is indicative of being a great national leader.', 'It is impossible to successfully shape public opinion without in some way reacting to it.', 'To lead, one must either successfully shape public opinion or be adept at reacting to it, or both.', 'Having a good rapport with the members of the legislature allows a leader to shape public opinion.']", "label": 0 }, { "id": "train_2135", "context": "Car companies solicit consumer information on such human factors as whether a seat is comfortable or whether a set of controls is easy to use. However, designer interaction with consumers is superior to survey data; the data may tell the designer why a feature on last year' s model was given a low rating, but data will not explain how that feature needs to be changed in order to receive a higher rating.", "question": "The reasoning above conforms most closely to which one of the following propositions?", "answers": "['A car will have unappealing features if consumers are not consulted during its design stage.', 'Car companies traditionally conduct extensive postmarket surveys.', 'Getting consumer input for design modifications can contribute to successful product design.', 'Designers aim to create features that will appeal to specific market niches.']", "label": 2 }, { "id": "train_2136", "context": "Professor: Members of most species are able to communicate with other members of the same species, but it is not true that all communication can be called \"language. \" The human communication system unquestionably qualifies as language. In fact, using language is a trait without which we would not be human.", "question": "Student: I understand that communication by itself is not language, but how do you know that the highly evolved communication systems of songbirds, dolphins, honeybees, and apes, for example, are not languages? The student has interpreted the professor's remarks to mean that", "answers": "['different species can have similar defining traits', 'every human trait except using language is shared by at least one other species', 'using language is a trait humans do not share with any other species', 'humans cannot communicate with members of other species']", "label": 2 }, { "id": "train_2137", "context": "Principle: It is healthy for children to engage in an activity that promotes their intellectual development only if engaging in that activity does not detract from their social development. Application: Although Megan' s frequent reading stimulates her intellectually, it reduces the amount of time she spends interacting with other people. Ttherefore, it is not healthy for her to read as much as she does.", "question": "The application of the principle is most vulnerable to criticism on which one of the following grounds?", "answers": "[\"It takes for granted that any decrease in the amount of time a child spends interacting with others detracts from that child's social development.\", 'It overlooks the possibility that the benefits of a given activity may sometimes be important enough to outweigh the adverse health effects.', \"It takes a necessary condition for an activity's being healthy as a sufficient condition for its being so.\", 'It misinterprets the principle as a universal claim intended to hold in all cases without exception, rather than as a mere generalization.']", "label": 0 }, { "id": "train_2138", "context": "Economist: Some sociologists argue that because capitalism intrinsically involves competition, it weakens the ties between the people of a community. Although this may formerly have been true, modern capitalism requires that there be large corporations. Thus, modern capitalism promotes, rather than weakens, communal ties.", "question": "Which one of the following is an assumption on which the economist's argument depends?", "answers": "['Modern capitalism is designed primarily to distribute goods and services, not to create communal ties between people.', 'Few economic systems are more successful than modern capitalism in fostering communal ties between citizens.', 'Corporations that compete with each other must develop some ties to each other in order to reach agreement on the rules of the competition.', 'Having large corporations in a modern capitalist system promotes the strength of communal ties.']", "label": 3 }, { "id": "train_2139", "context": "Medieval Arabs had manuscripts of many ancient Greek texts, which were translated into Arabic when there was a demand for them. Medieval Arab philosophers were very interested in Aristotle' s Poetics, an interest that evidently was not shared by medieval Arab poets, because a poet interested in the Poetics would certainly have wanted to read Homer, to whose epics Aristotle frequently refers. But Homer was not translated into Arabic until modern times.", "question": "Which one of the following, if true, most strongly supports the argument above?", "answers": "['Medieval Arabic story cycles, such as the Arabian Nights, are in some ways similar to parts of the Homeric epics.', \"Aristotle's Poetics is largely concerned with drama, and dramatic works were written and performed by medieval Arabs.\", 'A number of medieval Arab translators possessed manuscripts of the Homeric epics in their original Greek.', \"Aristotle' s Poetics has frequently been cited and commented on by modern Arab poets.\"]", "label": 2 }, { "id": "train_2140", "context": "It is often said that high rates of inflation tend to diminish people' s incentive to save and invest. This view must be incorrect, however, because people generally saved and invested more of their income in the 1970' s when inflation rates were high than they did in the 1980' s when inflation rates were low.", "question": "Of the following, the best criticism of the argument above is that it overlooks the possibility that", "answers": "[\"the population was larger in the 1980's than it was in the 1970's\", 'the proponents of the view cited would stand to gain if inflation rates become lower', \"a factor that affects people's savings behavior in a certain way could affect people's investment behavior quite differently\", \"certain factors operating in the 1980's but not in the 1970's diminished people's incentive to save and invest\"]", "label": 3 }, { "id": "train_2141", "context": "Most popular historical films are not documentaries; they are dramatic presentations of historical events. Such presentations cannot present the evidence for the accuracy of what they portray. Consequently, uninformed viewers of dramatic historical films should not regard them as accurate portrayals of historical events.", "question": "Which one of the following principles, if valid, most helps to justify the reasoning in the argument?", "answers": "['One should never regard a historical account to be accurate unless one has considered the evidence on which it is based.', 'Historical documentaries should be careful to present all the evidence when attempting to inform their audiences about historical events.', 'Writers of historical dramas should attempt to provide their own distinctive insights into the meaning of the historical events they are portraying.', 'Dramatic presentations of historical events are better suited for educational purposes if evidence supporting the accuracy of the presentation is also presented.']", "label": 0 }, { "id": "train_2142", "context": "Lawyer: Juries are traditionally given their instructions in convoluted, legalistic language. The verbiage is intended to make the instructions more precise, but greater precision is of little use if most jurors have difficulty understanding the instructions. Since it is more important for jurors to have a basic but adequate understanding of their role than it is for the details of that role to be precisely specified, jury instructions should be formulated in simple, easily comprehensible language.", "question": "Each of the following, if true, strengthens the lawyer's argument EXCEPT:", "answers": "['Most jurors do not have an adequate understanding of their role after being given jury instructions in convoluted, legalistic language.', 'The details of the role of the jurors cannot be specified with complete precision in simple, easily comprehensible language.', 'Most jurors are less likely to understand instructions given in convoluted, legalistic language than instructions given in simple, easily comprehensible language.', 'Jury instructions formulated in simple, easily comprehensible language can adequately describe the role of the jurors.']", "label": 1 }, { "id": "train_2143", "context": "Although human economic exchange predates historical records, it is clear that the very first economies were based on barter and that money came later. This can be inferred from occasions in history when, in isolated places, currency largely disappeared from the local economy. At such times, the economy typically reverts to the original barter system, but then quickly abandons this form of exchange when currency becomes available again.", "question": "Which one of the following most accurately describes a flaw in the argument's reasoning?", "answers": "['The argument concludes that something can cause a particular outcome merely because it is necessary for that outcome.', 'The argument relies on a premise that presupposes what the argument attempts to show in the conclusion.', 'The argument contains premises that contradict one another.', 'The argument presumes that something should be done merely because historically it has been done.']", "label": 1 }, { "id": "train_2144", "context": "Archaeologist: The extensive network of ancient tracks on the island of Malta was most likely created through erosion caused by the passage of wheeled vehicles. Some researchers have suggested that the tracks were in fact manually cut to facilitate the passage of carts, citing the uniformity in track depth. However, this uniformity is more likely indicative of wheel diameter: Routes were utilized until tracks eroded to a depth that made vehicle passage impossible.", "question": "Which one of the following is the overall conclusion of the archaeologist's argument?", "answers": "['The extensive network of ancient tracks on the island of Malta was most likely created through erosion caused by the passage of wheeled vehicles.', 'The uniformity of depth of the ancient tracks on the island of Malta is probably indicative of the wheel diameter of the carts that passed over them.', 'Some researchers have suggested that the ancient tracks on the island of Malta were in fact manually cut to facilitate the passage of carts.', 'Some researchers cite the uniformity of the depth of the ancient tracks on the island of Malta to support the suggestion that they were manually cut.']", "label": 0 }, { "id": "train_2145", "context": "But if a public place is uncomfortable it is not well designed, and all comfortable public places have spacious interiors If all of the interior decorator' s statements are true, then which one of the following must be true?", "question": "Interior decorator: All coffeehouses and restaurants are public places. Most well-designed public places feature artwork.", "answers": "['Any well-designed coffeehouse or restaurant has a spacious interior.', 'Any restaurant that has a spacious interior is comfortable.', 'Most coffeehouses that are well designed feature artwork.', 'Most public places that feature artwork are well designed.']", "label": 0 }, { "id": "train_2146", "context": "Until 1985 all commercial airlines completely replenished the cabin air in planes in flight once every 30 minutes. Since then the rate has been once every hour. The less frequently cabin air is replenished in a plane in flight, the higher the level of carbon dioxide in that plane and the easier it is for airborne illnesses to be spread.", "question": "Which one of the following is most strongly supported by the information above?", "answers": "['In 1985 there was a loosening of regulations concerning cabin air in commercial airline flights.', 'In 1980 the level of carbon dioxide in the cabin air on a two-hour commercial airline flight was lower than it is today on a similar flight.', 'People who fly today are more likely to contract airborne illnesses than were people who flew prior to 1985.', 'Low levels of carbon dioxide in cabin air make it impossible for airborne illnesses to spread.']", "label": 1 }, { "id": "train_2147", "context": "Perry: Worker-owned businesses require workers to spend time on management decision-making and investment strategy, tasks that are not directly productive. Also, such businesses have less extensive divisions of labor than do investor-owned businesses. Such inefficiencies can lead to low profitability, and thus increase the risk for lenders. Ttherefore, lenders seeking to reduce their risk should not make loans to worker-owned businesses.", "question": "Which one of the following, if true, most seriously weakens Perry's argument?", "answers": "['Worker-owned businesses have traditionally obtained loans from cooperative lending institutions established by coalitions of worker-owned businesses.', 'Lenders who specialize in high-risk loans are the largest source of loans for worker-owned businesses.', 'In most worker-owned businesses, workers compensate for inefficiencies by working longer hours than do workers in investor-owned businesses.', \"Businesses with the most extensive divisions of labor sometimes fail to make the fullest use of their most versatile employees' potential.\"]", "label": 2 }, { "id": "train_2148", "context": "Manager: Our new computer network, the purpose of which is to increase productivity, can be installed during the day, which would disrupt our employees' work, or else at night, which would entail much higher installation charges. Since saving money is important, we should have the network installed during me day.", "question": "The manager's argument assumes which one of the following?", "answers": "[\"Most of the company's employees would be able to work productively while a daytime installation is in progress.\", 'The monetary value of the network equipment would not exceed the cost of having the equipment installed at night.', \"Once the network has been installed, most of the company's employees will be able to use it immediately to increase their productivity.\", 'The monetary value of any productivity lost during a daytime installation would be less than the difference between daytime and nighttime installation costs.']", "label": 3 }, { "id": "train_2149", "context": "Despite legislation designed to stem the accumulation of plastic waste, the plastics industry continued to grow rapidly last year, as can be seen from the fact that sales of the resin that is the raw material for manufacturing plastics grew by 10 percent to $28 billion.", "question": "In assessing the support provided by the evidence cited above for the statement that the plastics industry continued to grow, in addition to the information above it would be most useful to know", "answers": "['the dollar amount of resin sales the year before last', \"what proportion of the plastics industry's output eventually contributes to the accumulation of plastic waste\", 'whether sales of all goods and services in the economy as a whole were increasing last year', 'whether the resin has other uses besides the manufacture of plastics']", "label": 3 }, { "id": "train_2150", "context": "Body temperature varies over a 24-hour period, with a low point roughly between 4 a. m. and 5 a. m. Speed of reaction varies in line with body temperature, such that whenever body temperature is low, speed of reaction is low. If low body temperature caused slow reaction, the speed of reaction should increase if we artificially raised body temperature during the period 4 a. m. to 5 a. m. But the speed of reaction does not increase.", "question": "Which one of the following conclusions can properly be drawn from the above statements?", "answers": "['Low speeds of reaction do not cause low body temperature.', 'Low body temperatures cause low speeds of reaction.', 'Low body temperatures do not cause low speeds of reaction.', 'Low speeds of reaction cause low body temperature.']", "label": 2 }, { "id": "train_2151", "context": "Amoebas, like human beings, generally withdraw from stimuli that cause them physical damage. Humans do this because such stimuli cause them pain. Thus all microscopic organisms must also be capable of feeling pain.", "question": "Which one of the following exhibits flawed reasoning most similar to that exhibited by the argument above?", "answers": "[\"Like nonprofit organizations, corporations usually provide some free public services. Nonprofit organizations do this solely because of their members' desire to make the world a better place. Thus this is probably also the main motive of most corporations.\", 'Most professional athletes practice regularly for the same reason. Professional boxers spend several hours a day practicing in order to excel in competition. Thus professional skaters probably also practice in order to excel in competition.', 'Poets, like people under hypnosis, frequently use language in odd, incomprehensible ways. People under hypnosis do this because their inhibitions are lower than those of most people. Thus all artists must have lower inhibitions than most people have.', 'Hiking trails in British Columbia, like those in New Mexico, are concentrated in mountainous regions. In New Mexico this is partly because low-lying areas are too hot and arid for comfortable hiking. Thus hikers must also feel less comfortable hiking in low-lying areas of British Columbia.']", "label": 2 }, { "id": "train_2152", "context": "Geneticist: Ethicists have fears, many of them reasonable, about the prospect of cloning human beings, that is, producing exact genetic duplicates. But the horror-movie image of a wealthy person creating an army of exact duplicates is completely unrealistic. Clones must be raised and educated, a long-term process that could never produce adults identical to the original in terms of outlook, personality, or goals. More realistic is the possibility that wealthy individuals might use clones as living \"organ banks. ", "question": "The claim that cloning will not produce adults with identical personalities plays which one of the following roles in the geneticist's argument?", "answers": "['It is evidence that genetic clones will never be produced successfully.', 'It is evidence for the claim that wealthy people might use genetic duplicates of themselves as sources of compatible organs for transplantation.', 'It is a reason for discounting one possible fear concerning the cloning of human beings.', 'It illustrates the claim that only wealthy people would be able to have genetic duplicates made of themselves.']", "label": 2 }, { "id": "train_2153", "context": "Consumers are deeply concerned about the quantity of plastic packaging on the market and have spurred manufacturers to find ways to recycle plastic materials. Despite their efforts, however, only 6. 5 percent of plastic is now being recycled, as compared to 33 percent of container glass.", "question": "Each of the following, if true, helps to explain the relatively low rate of plastic recycling EXCEPT:", "answers": "['Many factories are set up to accept and make economical use of recycled glass, whereas there are few factories that make products out of recycled plastic.', 'Plastic polymers tend to break down during the recycling process and weaken the resulting product, whereas glass does not break down.', 'Many plastic products are incompatible and cannot be recycled together, whereas most containers made of glass are compatible.', 'The manufacture of new plastic depletes oil reserves, whereas the manufacture of new glass uses renewable resources.']", "label": 3 }, { "id": "train_2154", "context": "The city depends on supplemental federal funding for its education budget, and the federal money is dependent on the city raising its cumulative high school literacy rate from the previous year. Over the past ten years, the city has received supplemental federal funding every year, but the city has had the highest illiteracy rate in the country for ten years.", "question": "Which one of the following would resolve the apparent discrepancy in the passage?", "answers": "['The high school literacy test is arbitrary and subject to change on a yearly basis.', \"The city's manufacturing industry makes it a desirable destination for illiterate immigrants.\", \"The city's government is the most corrupt in the country.\", 'The city increases the difficulty of the high school test for literacy every year.']", "label": 1 }, { "id": "train_2155", "context": "Two randomly selected groups of 30 adults each were asked to write short stories on a particular topic. One group was told that the best stories would be awarded cash prizes, while the other group was not told of any prizes. Each story was evaluated by a team of judges who were given no indication of the group from which the story came. The stories submitted by those who thought they were competing for prizes were ranked on average significantly lower than the stories from the other group.", "question": "Which one of the following, if true, most helps to explain the difference in average ranking between the two groups' stories?", "answers": "['People writing to win prizes show a greater than usual tendency to produce stereotypical stories that show little creativity.', 'The cash prizes were too small to motivate an average adult to make a significant effort to produce stories of high quality.', 'Most adults show little originality in writing stories on a topic suggested by someone else.', 'No one explained clearly to either group what standards would be used in judging their stories.']", "label": 0 }, { "id": "train_2156", "context": "Several recent studies establish that most people would want to be informed if they had any serious medical condition. In each study, over 80 percent of the people surveyed indicated that they would want to be told.", "question": "Each of the following, if true, weakens the argument EXCEPT:", "answers": "['Some questions asked in the studies suggested that reasonable people would want to be told if they had a serious medical condition.', 'In another recent study, most of the people surveyed indicated that they would not want to be told if they had a serious medical condition.', 'Some of the researchers conducting the studies had no background in medicine.', 'The people surveyed in the studies were all young students in introductory psychology courses.']", "label": 2 }, { "id": "train_2157", "context": "John: As I was driving to work this morning, I was stopped by a police officer and ticketed for speeding. Since there were many other cars around me that were going as fast as I was, the police officer clearly treated me unfairly. Mary: You were not treated unfairly, since the police officer was obviously unable to stop all the drivers who were speeding. Everyone who was speeding at that time and place had an equal chance of being stopped.", "question": "Which one of the following principles, if established, would most help to justify Mary's position?", "answers": "['If all of those who violate a traffic law on a particular occasion are equally likely to be penalized for violating it, then the law is fairly applied to whoever among them is then penalized.', \"Fairness in the application of a traffic law is ensured not by all violators' having an equal chance of being penalized for their violation of the law, but rather by penalizing all known violators to the same extent.\", 'It is fairer not to enforce a traffic law at all than it is to enforce it in some, but not all, of the cases to which it applies.', 'The penalties attached to traffic laws should be imposed on all people who violate those laws, and only those people.']", "label": 0 }, { "id": "train_2158", "context": "Bank depositors in the United States are all financially protected against bank failure because the government insures all individuals' bank deposits. An economist argues that this insurance is partly responsible for the high rate of bank failures, since it removes from depositors any financial incentive to find out whether the bank that holds their money is secure against failure. If depositors were more selective. then banks would need to be secure in order to compete for depositors' money.", "question": "Which of he following, if true, most seriously weakens the economist s argument?", "answers": "[\"When the government did not insure deposits frequent bank failures occurred as a result of depositors' fears of losing money in bank failures.\", \"There is an upper limit on the amount of an individual's deposit that the government will insure. but very few individuals' deposits exceed this limit.\", 'Before the government started to insure depositors against bank failure, there was a lower rate of bank failure than there is now.', 'Surveys show that a significant proportion of depositors are aware that their deposits are insured by the government.']", "label": 0 }, { "id": "train_2159", "context": "Linda says that, as a scientist, she knows that no scientist appreciates poetry. And, since most scientists are logical, at least some of the people who appreciate poetry are illogical.", "question": "Which one of the following is most parallel in its reasoning to the flawed reasoning above?", "answers": "['Xi says that, as an experienced photographer, she knows that no color film produces images as sharp as the best black-and-white film. And, since most instant film is color film, at least some instant film produces images less sharp than the best black-and-white film.', 'Yuri says that, as a wine connoisseur, he knows that no wine aged in metal containers is equal in quality to the best wine aged in oak. And, since most California wine is aged in metal containers, California wine is inferior to at least the best French wine aged in oak.', 'Franz says that, as a father of four children, he knows that no father wants children to eat candy at bedtime. And, since most fathers are adults, at least some of the people who want children to eat candy at bedtime are children.', 'Betty says that, as a corporate executive, she knows that no corporate executives like to pay taxes. And, since most corporate executives are honest people, at least some people who like to pay taxes are honest people.']", "label": 2 }, { "id": "train_2160", "context": "In response to viral infection, the immune systems of mice typically produce antibodies that destroy the virus by binding to proteins on its surface. Mice infected with the herpesvirus generally develop keratitis, a degenerative disease affecting part of the eye. Since proteins on the surface of cells in this part of the eye closely resemble those on the herpesvirus surface, scientists hypothesize that these cases of keratitis are caused by antibodies to the herpesvirus.", "question": "Which of the following, if true, most helps to support the scientists' reasoning?", "answers": "['Mice that are infected with the herpesvirus but do not develop keratitis produce as many antibodies as infected mice that do develop keratitis', 'There are mice that are unable to form antibodies in response to herpes infections, and these mice contract herpes at roughly the same rate as other mice', 'Other types of virus have surface proteins that closely resemble proteins found in various organs of mice.', 'Mice infected With a new strain of the herpesvirus that has different surface proteins did not develop keratitis']", "label": 3 }, { "id": "train_2161", "context": "Industries waste large amounts of valuable water. Government subsidies allow industries to pay little or nothing for water. Ttherefore, if industries are required by the government to pay full price for the water they use, inefficient use of water by industry would soon cease altogether.", "question": "A flaw in the argument's reasoning is that the argument", "answers": "['bases its conclusion on an ambiguous interpretation of the word \"inefficient\"', 'draws a conclusion that is stronger than what is warranted by the evidence presented', \"offers as evidence considerations that have no relevance to the argument's conclusion\", 'presents one possible solution to a problem as the only solution to that problem']", "label": 1 }, { "id": "train_2162", "context": "Automobile-emission standards are enforced through annual inspection. At those inspections cars are tested while idling; that is, standing still with their engines running. Testing devices measure the levels of various pollutants as exhaust gases leave the tail pipe.", "question": "Which one of the following, if true, most strongly indicates that current enforcement of automobileemission standards might be ineffective in controlling overall pollutant levels?", "answers": "['The testing devices used must be recalibrated frequently to measure pollutant levels with acceptable accuracy.', 'As an emission-control technology approaches its limits, any additional gains in effectiveness become progressively more expensive.', 'When emission standards are set, no allowances are made for older cars.', 'The adjustments needed to make a car idle cleanly make it likely that the car will emit high levels of pollutants when moving at highway speeds.']", "label": 3 }, { "id": "train_2163", "context": "Although most species of nondomestic mammals in Australia are marsupials, over 100 species -- including seals, bats, and mice -- are not marsupials but placentals. It is clear, however, that these placentals are not native to this island continent: all nonhuman placentals except the dingo, a dog introduced by the first humans that settled Australia, are animals whose ancestors could swim long distances, fly, or float on driftwood.", "question": "The conclusion above is properly drawn if which one of the following is assumed?", "answers": "['The only Australian placentals that could be native to Australia would be animals whose ancestors could not have reached Australia from elsewhere.', 'Some marsupials now found in Australia might not be native to that continent, but rather might have been introduced to Australia by some other means.', 'Seals, bats, and mice are typically found only in areas where there are no native marsupials.', 'No marsupials now found in Australia can swim long distances, fly, or float on driftwood.']", "label": 0 }, { "id": "train_2164", "context": "In Teruvia, the quantity of rice produced per year is currently just large enough to satisfy domestic demand. Teruvia' s total rice acreage will not be expanded in the foreseeable future, nor will rice yields per acre increase appreciably. Teruvia' s population, however, will be increasing significantly for years to come. Clearly, ttherefore, Teruvia will soon have to begin importing rice.", "question": "Which of the following is an assumption on which the argument depends?", "answers": "['No pronounced trend of decreasing per capita demand for rice is imminent in Teruvia.', 'There are no populated regions in Teruvia in which the population will not increase.', 'Not all of the acreage in Teruvia currently planted with rice is well suited to the cultivation of rice.', 'There are no major crops other than rice for which domestic production and domestic demand are currently in balance in Teruvia.']", "label": 0 }, { "id": "train_2165", "context": "Inez: In these poor economic times, people want to be sure they are getting good value for their money. I predict people would be more willing to buy antiques at our fair if we first have the object inspected by professional appraisers who would remove any objects of questionable authenticity. Anika: I disagree with your prediction. Our customers already are antiques experts. Furthermore, hiring professional appraisers would push up our costs considerably, thus forcing us to raise the prices on all our antiques.", "question": "Anika's response proceeds by", "answers": "['indicating that a particular plan would have an effect contrary to the anticipated effect', 'claiming that a particular plan should not be adopted because, while effective, it would have at least one undesirable consequence', 'questioning the assumption that authorities are available who have special knowledge of the problem under discussion', 'arguing that an alternative plan could achieve a desired result more easily than the plan originally proposed']", "label": 0 }, { "id": "train_2166", "context": "Coffee and tea contain methylxanthines, which cause temporary increases in the natural production of vasopressin, a hormone produced by the pituitary gland. Vasopressin causes clumping of blood cells, and the clumping is more pronounced in women than in men. This is probably the explanation of the fact that women face as much as a tenfold higher risk than men do of complications following angioplasty, a technique used to clear clogged arteries.", "question": "Which one of the following statements is most strongly supported by the information above?", "answers": "['Men, but not women, should be given methylxanthines prior to undergoing angioplasty.', 'Prior to undergoing angioplasty, women should avoid coffee and tea.', 'Angioplasty should not be used to treat clogged arteries.', 'In spite of the risks, angioplasty is the only effective treatment for clogged arteries.']", "label": 1 }, { "id": "train_2167", "context": "Sociologist: The intended function of news is to give us information on which to act. But in a consumer society, news becomes a product to be manufactured and dispensed to the consumer. An enormous industry for the production and consumption of news has evolved, and we ingest news with an insatiable appetite. Under such circumstances, news is primarily entertaining and cannot, ttherefore, serve its intended function.", "question": "Which one of the following is an assumption on which the sociologist's argument depends?", "answers": "['News that serves its intended function should not be entertaining.', 'News that primarily entertains does not give us information on which to act.', 'A news industry that aims to make a profit inevitably presents news as entertainment.', 'News has only one important function.']", "label": 1 }, { "id": "train_2168", "context": "The general availability of high-quality electronic scanners and color printers for computers has made the counterfeiting of checks much easier. In order to deter such counterfeiting, several banks plan to issue to their corporate customers checks that contain dots too small to be accurately duplicated by any electronic scanner currently available; when such checks are scanned and printed, the dots seem to blend together in such a way that the word \"VOID\" appears on the check.", "question": "A questionable assumption of the plan is that", "answers": "['the size of the smallest dots that generally available electronic scanners are able to reproduce accurately will not decrease significantly in the near future', 'most corporations served by these banks will not have to pay more for the new checks than for traditional checks', 'most counterfeiters who use electronic scanners counterfeit checks only for relatively large amounts of money', 'the smallest dots on the proposed checks cannot be distinguished visually except under strong magnification']", "label": 0 }, { "id": "train_2169", "context": "President of Max Development Company: Our quality control manager has proposed that we redistribute hourly employees so that all hourly workers are both employed in the departments they like best and are engaged in activities at which they excel. Our quality control manager assures us that this will \"increase quarterly profits by fully taking advantage of our existing resources. \" However, Max Development has a firm company policy against taking advantage of its hourly employees. Ttherefore, implementing our quality control manager' s proposal would make us violate our own company policy.", "question": "The president's argument for rejecting the quality control manager's suggestion is most vulnerable to criticism on which one of the following grounds?", "answers": "['being unable to differentiate between two discrete uses of a significant phrase', 'attempting to criticize an action on the ground that it is rarely carried out', 'coming to a conclusion that merely reiter- ates one of the main premises of the argument', 'suggesting that the proposal is more distasteful because of the source of the proposal']", "label": 0 }, { "id": "train_2170", "context": "Obviously, we cannot in any real sense mistreat plants. Plants do not have nervous systems, and having a nervous system is necessary to experience pain.", "question": "The conclusion above follows logically if which one of the following is assumed?", "answers": "['Any organism that has nervous system can experience pain.', 'Only organisms that can experience pain can be mistreated.', 'Only organisms that have nervous systems can experience pain.', 'Any organism that can experience pain can be mistreated.']", "label": 1 }, { "id": "train_2171", "context": "Beta Corporation claims that it no longer has enough work for the 60 computer data-entry workers that it is laying off. These workers have heard, however, that the company is buying 100 new computers. So the workers concluded that the company' s real reason for laying them off is to fill their jobs with lower-paid workers.", "question": "Which of the following, if true, would most undermine the workers' conclusion?", "answers": "['A recent memorandum from the president of Beta Corporation requested that all data generated by the company be stored in computerized form.', \"Beta Corporation's computer data-entry workers are more experienced and hence more highly paid than people doing comparable work in other companies.\", \"Beta Corporation's new computers will allow its current management and sales staff to perform many of the tasks that the data-entry workers were previously doing.\", 'Most of the workers being laid off know how to enter data on a number of different computer systems.']", "label": 2 }, { "id": "train_2172", "context": "Political scientist: One of the most interesting dilemmas in contemporary democratic politics concerns the regulation of political campaign spending. People certainly should be free, within broad limits, to spend their money as they choose. On the other hand, candidates who can vastly outspend all rivals have an unfair advantage in publicizing their platforms. Democratic governments have a strong obligation to ensure that all voices have an equal chance to be heard, but governments should not subsidize expensive campaigns for each candidate. The resolution of the dilemma, ttherefore, is clear: __.", "question": "Which one of the following most logically completes the political scientist's argument?", "answers": "['all wealthy persons should be prohibited from spending their own money on political campaigns', 'only candidates with significant campaign resources should be permitted to run for public office', 'an upper limit on the political campaign spending of each candidate is warranted', 'government subsidization of all political campaigns at a low percentage of their total cost is warranted']", "label": 2 }, { "id": "train_2173", "context": "Of all the Arabic epic poems that have been popular at various times, only Sirat Bani Hilal is still publicly performed. Furthermore, while most other epics were only recited, Sirat Bani Hilal has usually been sung. The musical character of the performance, ttherefore, is the main reason for its longevity.", "question": "The argument is most vulnerable to criticism on the grounds that it", "answers": "['relies on evidence that is in principle impossible to corroborate', 'takes a sufficient condition to be a necessary condition', 'takes for granted that a particular correlation is causal', 'relies on a source of evidence that may be biased']", "label": 2 }, { "id": "train_2174", "context": "Researchers hope to find clues about the A' mk peoples who lived in the Kaumpta region about one thousand years ago but who left few obvious traces. The researchers plan to hire the few remaining shamans of the modern-day indigenous people in Kaumpta, who are believed to be descended from the A' mk, to lead them to ancestral sites that may be the remains of A' mk buildings or ceremonial spaces. The shamans were taught the location of such sites as part of their traditional training as youths, and their knowledge of traditional Kaumpta customs may help determine the nature of any sites the researchers find.", "question": "Which of the following is an assumption on which the success of the plan depends?", "answers": "[\"The shamans have a reasonably accurate memory of A'mk sites they learned about as youths.\", \"Most or all A'mk sites are likely to be found within the Kaumpta region.\", 'Kaumpta shamans are generally held in high esteem for their traditional knowledge.', \"The researchers have reliable evidence that the A'mk of one thousand years ago built important ceremonial spaces.\"]", "label": 0 }, { "id": "train_2175", "context": "Pretzel vendor: The new license fee for operating a pretzel stand outside the art museum is prohibitively expensive. Charging typical prices, a vendor would need to sell an average of 25 pretzels per hour to break even. At my stand outside city hall, I average only 15 per hour. Ttherefore, I could not break even running a pretzel stand outside the art museum, much less turn a profit.", "question": "Which of the following, if true, most strongly supports the pretzel vendor's argument?", "answers": "['Fewer people passing the art museum than passing city hall are to buy pretzels.', 'People who buy pretzels at pretzel stands are most likely to do so during the hours at which the art museum is open to the visitors.', 'The number of pretzel stands outside the art museum is no greater than the number of pretzel stands now outside city hall.', 'Pretzel vendors who operate stands outside the art museum were making a profit before the imposition of the new license fee.']", "label": 0 }, { "id": "train_2176", "context": "Most economists believe that reducing the price of any product generally stimulates demand for it. However, most wine merchants have found that reducing the price of domestic wines to make them more competitive with imported wines with which they were previously comparably priced is frequently followed by an increase in sales of those imported wines.", "question": "Which one of the following, if true, most helps to reconcile the belief of most economists with the consequences observed by most wine merchants?", "answers": "['Consumers are generally willing to forgo purchasing other items they desire in order to purchase a superior wine.', 'An increase in the demand for a consumer product is compatible with an increase in demand for a competing product.', \"Economists' studies of the prices of grocery items and their rates of sales rarely cover alcoholic beverages.\", 'Imported wines in all price ranges are comparable in quality to domestic wines that cost less.']", "label": 1 }, { "id": "train_2177", "context": "Jorge: You won' t be able to write well about the rock music of the 1960s, since you were just an infant then. Rock music of the 1960s was created by and for people who were then in their teens and early twenties. Ruth: Your reasoning is absurd. There are living writers who write well about ancient Roman culture, even though those writers are obviously not a part of ancient Roman culture.", "question": "Why should my youth alone prevent me from writing well about the music of a period as recent as the 1960s? Which one of the following most accurately represents what is at issue between Jorge and Ruth?", "answers": "['whether only people who are past their early twenties can write well about ancient cultures', \"whether Ruth's ideas about the rock music of the 1960s are likely to appeal to people who were in their teens or early twenties during that period\", 'whether only those people who were in their teens or early twenties during the 1960s can be qualified to write about the rock music of that period', 'whether people who are not now in their teens or early twenties can write well about the rock music of the 1960s']", "label": 2 }, { "id": "train_2178", "context": "Recently discovered prehistoric rock paintings on small islands off the northern coast of Norway have archaeologists puzzled. The predominant theory about northern cave paintings was that they were largely a description of the current diets of the painters. This theory cannot be right, because the painters must have needed to eat the sea animals populating the waters north of Norway if they were to make the long journey to and from the islands, and there are no paintings that unambiguously depict such creatures.", "question": "Each of the following, if true, weakens the argument against the predominant theory about northern cave paintings EXCEPT:", "answers": "['Parts of the cave paintings on the islands did not survive the centuries.', 'The cave paintings that were discovered on the islands depicted many land animals.', 'The cave paintings on the islands were done by the original inhabitants of the islands who ate the meat of land animals.', 'Once on these islands, the cave painters hunted and ate land animals.']", "label": 1 }, { "id": "train_2179", "context": "Last year the Lalolah River was ranked by the Sunvale Water Commission as the most polluted of the fifteen rivers in the Sunvale Water District. Measures taken to clean up the river must be working, though, since this year the Lalolah River is ranked as only the third most polluted river in the district.", "question": "The reasoning in the argument is flawed because the argument", "answers": "['equates a decrease relative to the other ranked rivers with an absolute decrease', 'interprets lack of evidence for a claim as support for an opposing claim', 'confuses the state of the individual rivers in the water district with that of the water district as a whole', 'does not disclose the basis for the ranking used']", "label": 0 }, { "id": "train_2180", "context": "John wants to win the annual Mayfield raffle next year because he needs the Mayfield raffle' s prize. If he enters more than one raffle next year, the likelihood of his winning one of them would be greater than if he entered only a single raffle. Hence, to have this greater likelihood of winning the Mayfield prize, John should enter several other raffles next year.", "question": "The argument exhibits which one of the following flaws in reasoning?", "answers": "['confusing the likelihood that at least one event in a set of events will occur with the likelihood that a designated event in that set will occur', 'assuming that an event, if it is highly improbable, cannot possibly occur', \"presupposing that a person's strong desire for a certain outcome increases the likelihood that the actual outcome will be as desired\", 'assuming without warrant that a person will be successful if the person engages only in those activities that are likely to be successful']", "label": 0 }, { "id": "train_2181", "context": "Psychiatrist: We are learning that neurochemical imbalances can cause behavior ranging from extreme mental illness to less serious but irritating behavior such as obsessive fantasizing, petulance, or embarrassment. These findings will promote compassion and tolerance when looking at a mental illness, quirk, or mere difference between two persons, since being mentally healthy can now begin to be seen as simply having the same neurochemical balances as most people.", "question": "Which one of the following most accurately expresses the conclusion of the psychiatrist's argument?", "answers": "['Advances in neurochemistry enhance our theories of mental illness.', 'Understanding the role of the neurochemical in behavior will foster empathy toward others.', 'Neurochemical balances and imbalances are the main determinants of mental behavior.', 'Neurochemical imbalances can cause mental illness and other behaviors.']", "label": 1 }, { "id": "train_2182", "context": "Despite the efforts of a small minority of graduate students at one university to unionize, the majority of graduate students there remain unaware of the attempt. Most of those who are aware believe that a union would not represent their interests or that, if it did, it would not effectively pursue them. Thus, the graduate students at the university should not unionize, since the majority of them obviously disapprove of the attempt.", "question": "The reasoning in the argument is most vulnerable to criticism on the grounds that the argument", "answers": "['blurs the distinction between active disapproval and mere lack of approval', 'tries to establish a conclusion simply on the premise that the conclusion agrees with a long-standing practice', 'presumes that simply because a majority of a population is unaware of something, it must not be a good idea', 'fails to exclude alternative explanations for why some graduate students disapprove of unionizing']", "label": 0 }, { "id": "train_2183", "context": "Antonio: One can live a life of moderation by never deviating from the middle course. But then one loses the joy of spontaneity and misses the opportunities that come to those who are occasionally willing to take great chances, or to go too far. Marla: But one who, in the interests of moderation, never risks going too far is actually failing to live a life of moderation: one must be moderate even in one' s moderation.", "question": "Antonio and Marla disagree over", "answers": "['what a life of moderation requires of a person', 'how often a person ought to deviate from the middle course in life', 'whether it is possible for a person to embrace other virtues along with moderation', 'whether it is desirable for people to be moderately spontaneous']", "label": 0 }, { "id": "train_2184", "context": "Logan: Newspapers have always focused on ephemeral matters while ignoring important societal changes. For this and other reasons, old newspapers are useless to both amateur and professional historians. Mendez: But news stories, along with popular art, provide a wealth of information about what the people of an era thought and felt.", "question": "On the basis of their statements, Logan and Mendez are committed to disagreeing over whether", "answers": "['newspapers ought to focus more on the types of stories they have glossed over in the past', 'newspapers accurately report the most important changes taking place in a society', 'the study of previous eras should include investigations of the conventions of newspaper reporting', 'newspaper reports from former eras are useful sources of material for understanding the past']", "label": 3 }, { "id": "train_2185", "context": "A well-known sports figure found that combining publicity tours with playing tours led to problems, so she stopped combining the two. She no longer allows bookstore appearances and playing in competition to occur in the same city within the same trip. This week she is traveling to London to play in a major competition, so during her stay in London she will not be making any publicity appearances at any bookstore in London.", "question": "Which one of the following most closely parallels the reasoning used in the passage?", "answers": "['Wherever there is an Acme Bugkiller, many wasps are killed. The Z family garden has an Acme Bugkiller, so any wasps remaining in the garden will soon be killed.', \"Butter knives are not sharp. Q was not murdered with a sharp blade, so suspect X's butter knife may have been the murder weapon.\", 'Higher grades lead to better job opportunities, and studying leads to higher grades. Ttherefore, studying will lead to better job opportunities.', \"The only times that the hospital's emergency room staff attends to relatively less serious emergencies are times when there is no critical emergency to attend to. On Monday night the emergency room staff attended to a series of fairly minor emergencies, so there must not have been any critical emergencies to take care of at the time.\"]", "label": 3 }, { "id": "train_2186", "context": "Patti: Most parents are eager for their preschoolers to learn as much as possible. However, instead of providing general opportunities for their children to learn, parents often direct their children' s learning to their own personal concerns. Because children have a natural curiosity and thirst for knowledge, they learn an enormous amount simply through growing and adapting to the world. Ttherefore, this type of directed learning is unlikely to improve a child' s preschool education.", "question": "Which one of the following is an assumption on which Patti's argument depends?", "answers": "['The type of directed learning in question is not a necessary part of the process of growing and adapting to the world.', 'Children will have difficulty adapting to the world without the unique help and guidance of their parents.', 'The type of directed learning in question is likely to enhance the general opportunities for children to learn.', 'General opportunities to learn are not typical of the early years of formal education.']", "label": 0 }, { "id": "train_2187", "context": "For several centuries there have been hairless dogs in western Mexico and in coastal Peru. It is very unlikely that a trait as rare as hairlessness emerged on two separate occasions. Since the dogs have never existed in the wild, and the vast mountainous jungle separating these two regions would have made overland travel between them extremely difficult centuries ago, the dogs must have been transported from one of these regions to the other by boat, probably during trading expeditions.", "question": "Which one of the following is an assumption that the argument requires?", "answers": "['Hairless dogs have never been found anywhere except in the regions of western Mexico and coastal Peru.', 'Centuries ago, it was easier to travel by boat between western Mexico and coastal Peru than to travel by an overland route.', 'If hairless dogs were at one time transported between western Mexico and coastal Peru by boat, they were traded in exchange for other goods.', 'Most of the trade goods that came into western Mexico centuries ago were transported by boat.']", "label": 1 }, { "id": "train_2188", "context": "Educator: Traditional classroom education is ineffective because education in such an environment is not truly a social process and only social processes can develop students' insights. In the traditional classroom, the teacher acts from outside the group and interaction between teachers and students is rigid and artificial.", "question": "The educator's conclusion follows logically if which one of the following is assumed?", "answers": "['The teacher does not act from outside the group in a nontraditional classroom.', 'All social processes involve interaction that is neither rigid nor artificial.', 'Education is not effective unless it leads to the development of insight.', 'Development of insight takes place only if genuine education also occurs.']", "label": 2 }, { "id": "train_2189", "context": "Zack' s Coffeehouse schedules free poetry readings almost every Wednesday. Zack' s offers half-priced coffee all day on every day that a poetry reading is scheduled.", "question": "Which one of the following can be properly inferred from the information above?", "answers": [ "Zack's offers half-priced coffee all day on most if not all Wednesdays.", "On some Wednesdays Zack's does not offer half-priced coffee all day.", "Free poetry readings are scheduled on almost every day that Zack's offers half-priced coffee all day.", "Most free poetry readings given at Zack's are scheduled for Wednesdays." ], "label": 0 }, { "id": "train_2190", "context": "At the company picnic, all of the employees who participated in more than four of the scheduled events, and only those employees, were eligible for the raffle held at the end of the day. Since only a small proportion of the employees were eligible for the raffle, most of the employees must have participated in fewer than four of the scheduled events.", "question": "Which one of the following arguments exhibits a flawed pattern of reasoning most like that exhibited by the argument above?", "answers": "['Only third- and fourth-year students are allowed to keep cars on campus. Since one quarter of the third-year students keep cars on campus and one half of the fourth-year students keep cars on campus, it must be that fewer third-year students than fourth-year students keep cars on campus.', 'All of the members of the service club who volunteered at the hospital last summer were biology majors. Since ten of the club members are biology majors, those ten members must have volunteered at the hospital last summer.', 'All of the swim team members who had decreased their racing times during the season were given awards that no other members were given. Since fewer than half the team members were given such awards, the racing times of more than half the team members must have increased during the season.', \"The only students honored at a special banquet were the band members who made the dean's list last semester. Since most of the band members were honored, most of the band members must have made the dean's list.\"]", "label": 2 }, { "id": "train_2191", "context": "When a person with temporal lobe epilepsy is having an epileptic seizure, part of the brain' s temporal lobe produces abnormal electrical impulses, which can often, but not always, be detected through a test called an electroencephalogram (EEG). Ttherefore, although a positive EEG reading -- that is, evidence of abnormal electrical impulses -- during an apparent seizure is a reasonably reliable indicator of temporal lobe epilepsy, __.", "question": "Of the following, which one logically completes the conclusion above?", "answers": "['a negative reading is just as reliable an indicator of the presence of temporal lobe epilepsy', 'a positive reading is more frequently an erroneous reading than is a negative one', 'a negative reading does not mean that temporal lobe epilepsy can be ruled out', 'a positive reading can also indicate the presence of other forms of epilepsy']", "label": 2 }, { "id": "train_2192", "context": "The continents of South America and Africa were once joined together as one landmass. Evidence for this claim can be found in the fact that fossils of mesosaurs, extinct reptiles, have been unearthed in both western Africa and eastern South America.", "question": "Which one of the following statements, if true, most weakens the argument?", "answers": "['The fossils of mesosaurs are the only ones unearthed in both western Africa and eastern South America.', 'The fossils of mesosaurs found in western Africa are of the same geologic period as those unearthed in eastern South America.', 'Mesosaurs in Africa migrated from Europe and mesosaurs in South America migrated from Asia.', 'The fossils of mesosaurs were found very near the west African and east South American coasts.']", "label": 2 }, { "id": "train_2193", "context": "A gas tax of one cent per gallon would raise one billion dollars per year at current consumption rates. Since a tax of fifty cents per gallon would ttherefore raise fifty billion dollars per year, it seems a perfect way to deal with the federal budget deficit. This tax would have the additional advantage that the resulting drop in the demand for gasoline would be ecologically sound and would keep our country from being too dependent on foreign oil producers.", "question": "Which one of the following most clearly identifies an error in the author's reasoning?", "answers": "['The author makes incompatible assumptions.', 'The author mistakes an effect for a cause.', 'The author appeals to conscience rather than reason.', 'The author cites irrelevant data.']", "label": 0 }, { "id": "train_2194", "context": "A century in certain ways is like a life, and as the end of a century approaches, people behave toward that century much as someone who is nearing the end of life does toward that life. So just as people in their last years spend much time looking back on the events of their life, people at a century' s end __.", "question": "Which one of the following most logically completes the argument?", "answers": "['focus on what the next century will bring', 'reminisce about their own lives', 'fear that their own lives are about to end', 'become very interested in the history of the century just ending']", "label": 3 }, { "id": "train_2195", "context": "The importance of the ozone layer to terrestrial animals is that it entirely filters out some wavelengths of light but lets others through. Holes in the ozone layer and the dangers associated with these holes are well documented. However, one danger that has not been given sufficient attention is that these holes could lead to severe eye damage for animals of many species.", "question": "Which one of the following is most strongly supported by the statements above, if they are true?", "answers": "[\"Few species of animals live on a part of the earth's surface that is not threatened by holes in the ozone layer.\", 'A single wavelength of sunlight can cause severe damage to the eyes of most species of animals.', \"Some wavelengths of sunlight that cause eye damage are more likely to reach the earth's surface where there are holes in the ozone layer than where there are not.\", 'All wavelengths of sunlight that can cause eye damage are filtered out by the ozone layer, where it is intact.']", "label": 2 }, { "id": "train_2196", "context": "Criminologist: Increasing the current prison term for robbery will result in no significant effect in discouraging people from committing robbery.", "question": "Each of the following, if true, supports the criminologist's claim EXCEPT:", "answers": "['Most people committing robbery believe that they will not get caught.', 'Many people who rob are motivated primarily by thrill-seeking and risk-taking.', 'An increase in the prison term for embezzlement did not change the rate at which that crime was committed.', 'Prison terms for robbery have generally decreased in length recently.']", "label": 3 }, { "id": "train_2197", "context": "Pundit: Clearly, the two major political parties in this city have become sharply divided on the issues. In the last four elections, for example, the parties were separated by less than 1 percent of the vote.", "question": "The reasoning in the argument is most vulnerable to criticism on the grounds that the argument", "answers": "['presumes, without argument, that sharp division is a bad thing', 'takes for granted that an almost even division in votes indicates a sharp division on issues', 'fails to indicate how what is happening in one city compares with what is happening in other cities', 'has a conclusion that is merely a restatement of one of its premises']", "label": 1 }, { "id": "train_2198", "context": "The shoe factory in Centerville is the town' s largest firm, and it employs more unskilled workers on a full-time basis than all of the other businesses in town combined. Ttherefore, if the shoe factory closes down and ceases all operations, more than half of Centerville' s residents who are unskilled workers with full-time jobs in Centerville will lose their jobs.", "question": "The conclusion above logically follows from the premises if which one of the following is assumed?", "answers": "['More people who now are residents of Centerville are employed than are unemployed.', 'The shoe factory in Centerville employs no one who is not a resident of Centerville.', 'Centerville has more unskilled workers among its population than it has skilled workers.', 'There are no factories in Centerville other than the shoe factory.']", "label": 1 }, { "id": "train_2199", "context": "Book publishers have traditionally published a few books that they thought were of intrinsic merit even though these books were unlikely to make a profit. Nowadays, however, fewer of these books are being published. It seems, ttherefore, that publishers now, more than ever, are more interested in making money than in publishing books of intrinsic value.", "question": "Which one of the following statements, if true, most seriously weakens the argument?", "answers": "['There has been a notable decline in the quality of books written in recent years.', 'In recent years, profits in the book publishing industry have been declining.', 'There have always been authors unwilling to be published unless a profit is guaranteed.', 'In the past, often books of intrinsic value would unexpectedly make a sizable profit.']", "label": 0 }, { "id": "train_2200", "context": "While the total enrollment of public elementary and secondary schools in Sondland is one percent higher this academic year than last academic year, the number of teachers there increased by three percent. Thus, the Sondland Education Commission' s prediction of a teacher shortage as early as next academic year is unfounded.", "question": "Which of the following, if true, most seriously weakens the claim that the prediction of a teacher shortage as early as next academic year is unfounded?", "answers": "['Because of reductions in funding, the number of students enrolling in teacher-training programs in Sondland is expected to decline beginning in the next academic year.', 'Funding for public elementary schools in Sondland is expected to increase over the next ten years.', \"A new law has mandated that there be ten percent more teachers per pupil in Sondland's public schools next academic year than there were this academic year.\", \"Average salaries for Sondland's teachers increased at the rate of inflation from last academic year to this academic year.\"]", "label": 2 }, { "id": "train_2201", "context": "Some political thinkers hope to devise a form of government in which every citizen' s rights are respected. But such a form of government is impossible. For any government must be defined and controlled by laws that determine its powers and limits; and it is inevitable that some individuals will learn how to interpret these laws to gain a greater share of political power than others have.", "question": "Which one of the following is an assumption required by the argument?", "answers": "[\"In any form of government, if anybody gains a greater share of political power than others have, then somebody's rights will be violated.\", \"A government can ensure that every citizen's rights are respected by keeping the citizens ignorant of the laws.\", 'In any form of government that leads to unequal distribution of political power, the rights of the majority of people will be violated.', 'People who have more political power than others have tend to use it to acquire an even greater share of political power.']", "label": 0 }, { "id": "train_2202", "context": "Researcher: People who participate in opinion surveys often give answers they believe the opinion surveyor expects to hear, and it is for this reason that some opinion surveys do not reflect the actual views of those being surveyed. However, in well-constructed surveys, the questions are worded so as to provide respondents with no indication of which answers the surveyor might expect. So if a survey is well constructed, survey respondents' desire to meet surveyors' expectations has no effect on the survey' s results.", "question": "The reasoning in the researcher's argument is questionable in that the argument overlooks the possibility that", "answers": "[\"an opinion survey that disguises the surveyor's expectations may be flawed in a number of ways, some of which have nothing to do with the surveyor's expectations\", \"some people who know what answers an opinion surveyor expects to hear will purposefully try to thwart the surveyor's expectations\", 'many opinion surveyors have no expectations whatsoever regarding the answers of people who respond to surveys', \"the answers of opinion-survey respondents can be influenced by beliefs about the surveyor's expectations even if those beliefs are unfounded\"]", "label": 3 }, { "id": "train_2203", "context": "John: In 80 percent of car accidents, the driver at fault was within five miles of home, so people evidently drive less safely near home than they do on long trips. Judy: But people do 80 percent of their driving within five miles of home.", "question": "How is Judy's response related to John's argument?", "answers": "[\"It suggests that John's conclusion is merely a restatement of his argument's premise.\", 'It restates the evidence that John presents in different terms.', 'It gives additional evidence that is needed by John to support his conclusion.', 'It shows that the evidence that John presents, by itself, is not enough to prove his claim.']", "label": 3 }, { "id": "train_2204", "context": "Newspaper editors should not allow reporters to write the headlines for their own stories. The reason for this is that, while the headlines that reporters themselves write are often clever, what typically makes them clever is that they allude to little-known information that is familiar to the reporter but that never appears explicitly in the story itself.", "question": "Which of the following, if true, most strengthens the argument?", "answers": "[\"The reporter who writes a story is usually better placed than the reporter's editor is to judge what 'the story's most newsworthy features are.\", 'Most reporters rarely bother to find out how other reporters have written stories and headlines about the same events that they themselves have covered.', \"The kind of headlines that newspaper editors want are those that anyone who has read a reporter's story in its entirety will recognize as clever.\", \"For virtually any story that a reporter writes, there are at least a few people who know more about the story's subject matter than does the reporter.\"]", "label": 2 }, { "id": "train_2205", "context": "Myrna: People should follow diets in which fat represents no more than 30 percent of total calories, not the 37 percent the average diet in this country contains. Roland: If everyone in the country followed your recommendation during his or her entire life, just 0. 2 percent would lengthen their lives at all, and then only by an average of 3 months. Modifying our diet is not worthwhile. A lifetime of sacrifice spent eating an unappealing low-fat diet is too high a price to pay for the chance of extending that sacrifice for 3 months. Myrna: But for everyone who dies early from a high-fat diet, many more people suffer from serious chronic diseases because they followed such diets.", "question": "Myrna responds to Roland by", "answers": "['showing that the factors considered by Roland are not the only ones relevant in evaluating her recommendation', 'demonstrating that the statistics used by Roland to dispute her recommendation are inaccurate', \"pointing out that Roland's argument assumes the very proposition it sets out to prove\", 'disputing the correctness of the facts cited by Roland and offering facts that she considers correct']", "label": 0 }, { "id": "train_2206", "context": "The highest-ranking detectives in the city' s police department are also the most adept at solving crimes. Yet in each of the past ten years, the average success rate for the city' s highest-ranking detectives in solving criminal cases has been no higher than the average success rate for its lowest-ranking detectives.", "question": "Which one of the following, if true, most helps to resolve the apparent paradox?", "answers": "['The police department generally give the criminal cases that it expects to be the easiest to solve to its lowest-ranking detectives.', \"It generally takes at least ten years for a detective to rise from the lowest to the highest ranks of the city's detective force.\", 'None of the lowest-ranking detectives in the police department had experience in solving criminal cases prior to joining the police department.', 'The detectives who have the highest success rate in solving criminal cases are those who have worked as detectives the longest.']", "label": 0 }, { "id": "train_2207", "context": "Engineer: Wide roads free of obstructions have been shown to encourage drivers to take more risks. Likewise, a technical fix to slow or reverse global warming by blocking out a portion of the sun' s rays would encourage more carbon dioxide emissions, which might cause more global warming in the future.", "question": "The engineer's argument can most reasonably be interpreted as invoking which one of the following principles?", "answers": "['Problems created by humans require human-created solutions.', 'Conditions that create a feeling of security also encourage risk taking.', 'Technical fixes cannot discourage risk-taking behavior.', 'Technical fixes are inevitably temporary.']", "label": 1 }, { "id": "train_2208", "context": "Employee: My boss says that my presentation to our accounting team should have included more detail about profit projections. But people' s attention tends to wander when they are presented with too much detail. So, clearly my boss is incorrect.", "question": "The reasoning in the employee's argument is flawed because the argument", "answers": "['infers a generalization based only on a single case', \"fails to consider that an audience's attention might wander for reasons other than being presented with too much detail\", 'confuses two distinct meanings of the key term \"detail\"', 'fails to distinguish between more of something and too much of it']", "label": 3 }, { "id": "train_2209", "context": "Albert: Swenson' s popular book, which argues that sun exposure does not harm skin cells, is a model of poor scholarship. Nonetheless, it is valuable because it has stimulated new research on sun exposure. Yvonne: You' re kidding me! You might as well say that a virus is valuable because it stimulates epidemiologists.", "question": "The dialogue provides the most support for the claim that Albert and Yvonne disagree over whether", "answers": "['sun exposure harms skin cells', 'something that does not stimulate new research can have value', \"Swenson's book should be considered valuable\", \"Swenson's book is a model of poor scholarship\"]", "label": 2 }, { "id": "train_2210", "context": "Political candidates' speeches are loaded with promises and with expressions of good intention, but one must not forget that the politicians' purpose in giving these speeches is to get themselves elected. Clearly, then, these speeches are selfishly motivated and the promises made in them are unreliable.", "question": "Which one of the following most accurately describes a flaw in the argument above?", "answers": "['The argument presumes, without providing justification, that promises made for selfish reasons are never kept.', 'The argument overlooks the fact that a candidate who makes promises for selfish reasons may nonetheless be worthy of the office for which he or she is running.', \"The argument presumes, without providing justification, that if a person's promise is not selfishly motivated then that promise is reliable.\", 'The argument overlooks the fact that a promise need not be unreliable just because the person who made it had an ulterior motive for doing so.']", "label": 3 }, { "id": "train_2211", "context": "High cholesterol levels are highly correlated with cardiovascular disease. In the Italian town of Limone, however, each of the residents has had high cholesterol levels for many years, and yet they have not developed cardiovascular disease.", "question": "Which one of the following, if true, most helps to explain the unusual health condition of the residents of Limone?", "answers": "['The residents of Limone inherited from common ancestors a blood protein that prevents vascular blockage, which is a cause of cardiovascular disease.', 'Olive oil is a staple of the diet in some parts of Italy, but unlike butter, olive oil is a monosaturated fat, and monosaturated fats do not contribute to cardiovascular disease.', 'The stress and pollution found in large cities exacerbate existing cardiovascular disease, but there is little stress or pollution in Limone.', 'The residents of Limone have normal blood sugar levels, and very low blood sugar levels tend to cancel out the cardiovascular effects of a high cholesterol level.']", "label": 0 }, { "id": "train_2212", "context": "Several critics have claimed that any contemporary poet who writes formal poetry -- poetry that is rhymed and metered -- is performing a politically conservative act. This is plainly false. Consider Molly Peacock and Marilyn Hacker, two contemporary poets whose poetry is almost exclusively formal and yet who are themselves politically progressive feminists.", "question": "The conclusion drawn above follows logically if which one of the following is assumed?", "answers": "['No one who is politically progressive is capable of performing a politically conservative act.', 'Anyone who sometimes writes poetry that is not politically conservative never writes poetry that is politically conservative.', \"The content of a poet's work, not the work's form, is the most decisive factor in determining what political consequences, if any, the work will have.\", 'No poet who writes unrhymed or unmetered poetry is politically conservative.']", "label": 0 }, { "id": "train_2213", "context": "It has long been thought that lizards evolved from a group of amphibians called anthracosaurs, no fossils of which have been found in any rocks older than 300 million years. However, a fossil of a lizard was recently found that is estimated to be 340 million years old. Lizards could not have evolved from creatures that did not exist until after the first lizards. Ttherefore, lizards could not have evolved from anthracosaurs.", "question": "An assumption made in the argument is that there are no", "answers": "['unknown anthracosaur fossils older than 340 million years', 'known anthracosaur fossils that predate some lizard fossils', 'known lizard fossils that predate some anthracosaur fossils', 'unknown lizard fossils older than 340 million years']", "label": 0 }, { "id": "train_2214", "context": "The law firm of Sutherlin, Perez, and Associates is one of the most successful law firms whose primary specialization is in criminal defense cases. In fact, the firm has a better than 90 percent acquittal rate in such cases. Dalton is an attorney whose primary specialization is in divorce cases, so Dalton certainly cannot be a member of Sutherlin, Perez, and Associates.", "question": "The reasoning in the argument is flawed because the argument", "answers": "['states a generalization based on a selection that is not representative of the group about which the generalization is supposed to hold true', 'takes a high rate of success among the members of a group to indicate that the successes are evenly spread among the members', 'concludes that someone is not a member of a group on the grounds that that person does not have a characteristic that the group as a whole has', 'offers in support of its conclusion pieces of evidence that are mutually contradictory']", "label": 2 }, { "id": "train_2215", "context": "In recent years, the Holdsville Transportation Authority (HTA) has noted consistent delays on its Holdsville-River Valley bus, which runs from downtown Holdsville to the suburb of River Valley. In order to decrease the commuting time from Holdsville to River Valley, the HTA recently eliminated the James Street stop on the Holdsville-River Valley line. However, data show that the average commuting time from Holdsville to River Valley has actually increased since the elimination of the stop.", "question": "Which of the following provides the best explanation for the increase in commuting time from Holdsville to River Valley?", "answers": "['Almost all of the commuters who previously used the James Street stop now use the adjacent Green Street stop, causing overcrowding and excessive boarding and de-boarding delays at the Green Street stop.', 'The Holdsville-River Valley bus route runs along River Street, which is always congested with heavy automobile traffic.', '90% of Holdsville-River Valley commuters were in favor of eliminating the James Street stop.', 'The Johnstown bus line, another line operated by the HTA, has also experienced an increase in average commuting time since eliminating a stop on its route.']", "label": 0 }, { "id": "train_2216", "context": "Carter: Our hypothesis is that drivers in rural areas are guilty of more frequently texting while driving compared to urban drivers, so we need to make sure our demographics survey includes a question about the subject' s town of residence' s population size. Olga: It' s doubtful that most people know the exact population of their hometown, plus, we don' t need specific population figure s, so it makes more sense to ask subjects to identify the population range that their town of residence falls into.", "question": "Which of the following principles, if valid, best justifies Olga's position?", "answers": "['Subjects are less likely to answer survey questions accurately if they inquire about sensitive personal information.', 'Surveys that collect de mographic information about participants should be as detailed as possible to achieve the necessary information for the hypothesis.', 'Surveys that collect de mographic information about participants should be as generalized as possible to achieve the necessary information for the hypothesis.', 'Surveys that collect de mographic information about participants should be as precise as possible if the information is needed for the research question.']", "label": 2 }, { "id": "train_2217", "context": "Last year, the composite El-Hi education (CEHE) ranking for Silver County was its lowest score on record, since composite were first calculated in 1964. The CEHE ranking is a sophisticated weighed average of standardized testing results from Grade 1 to Grade 12. This year, the testing result for grades 1-6 in Silver County are about the same as they were last year, and the testing results for high school (grades 9-12) in Silver County are lower than last year' s high school results in Silver County. Surely the middle school testing results (grades 7-9) in Silver County will also be as low as, or lower than, last year' s results. Ttherefore, Silver County will undoubtedly establish a new low in its CEHE ranking. ", "question": "In the argument, the two portions in boldface play which of the following roles?", "answers": "['The first is an objection that the main argument rejects; the second present a conclusion that could be drawn if that object were allowed to stand.', 'The first is a claim that has been advanced in support of a position the argument opposes; the second is a claim advanced in support of the main conclusions of the argument.', 'The first is a prediction that, if accurate, would provide support for the main conclusion of the argument; the second is a conclusion drawn in order to support the main conclusion.', 'The first is a prediction that, if accurate, would provide support for the main conclusion of the argument; the second is that main conclusion.']", "label": 3 }, { "id": "train_2218", "context": "Terry: If You want to get a decent job, you should go to college. Mark: That is not true. There are other reasons to go to college than wanting to get a good job.", "question": "Mark's response shows that he interpreted Terry's remarks to mean that", "answers": "['all people who want decent jobs go to college', 'training for decent jobs is available only at colleges', 'college is one of many places to get trained for a job', 'wanting to get a decent job is the only reason for going to college']", "label": 3 }, { "id": "train_2219", "context": "Many office buildings designed to prevent outside air from entering have been shown to have elevated levels of various toxic substances circulating through the air inside, a phenomenon known as sick building syndrome. Yet the air in other office buildings does not have elevated levels of these substances, even though those buildings are the same age as the \"sick\" buildings and have similar designs and ventilation systems.", "question": "Which of the following, if true, most helps to explain why not all office buildings designed to prevent outside air from entering have air that contains elevated levels of toxic substances?", "answers": "['Certain adhesives and drying agents used in particular types of furniture, carpets, and paint contribute the bulk of the toxic substances that circulate in the air of office buildings.', 'The toxic substances that are found in the air of \"sick\" office buildings are substances that are found in at least small quantities in nearly every building.', 'Office buildings with windows that can readily be opened are unlikely to suffer from sick building syndrome.', 'Among buildings designed to prevent outside air from entering, houses are no less likely than office buildings to have air that contains elevated levels of toxic substances.']", "label": 0 }, { "id": "train_2220", "context": "Environmentalist: Pollution from gasoline burned by cars contributes to serious environmental problems. But the cost of these problems is not reflected in gasoline prices, and hence usually does not affect consumers' decisions about how much to drive. Heavier taxes on gasoline, however, would reflect this cost, and as a result consumers would pollute less.", "question": "The environmentalist's statements, if true, most strongly support which one of the following?", "answers": "['The cost of pollution from driving should not be reflected in the price of gasoline unless the amount of pollution produced would be reduced as a result.', 'The only cost considered by most consumers when they are deciding how much to drive is the cost of gasoline.', 'Consumers would purchase less gasoline, on average, if the cost of the environmental problems to which pollution from driving contributes were fully reflected in the price of gasoline.', 'Pollution from gasoline burned by cars will be reduced only if consumers give more consideration to the cost of that pollution when deciding how much to drive.']", "label": 2 }, { "id": "train_2221", "context": "Psychologist: A study of 436 university students found that those who took short naps throughout the day suffered from insomnia more frequently than those who did not. Moreover, people who work on commercial fishing vessels often have irregular sleep patterns that include frequent napping, and they also suffer from insomnia. So it is very likely that napping tends to cause insomnia.", "question": "The reasoning in the psychologist's argument is most vulnerable to criticism on the grounds that the argument", "answers": "['fails to consider the possibility that frequent daytime napping is an effect rather than a cause of insomnia', 'presumes, without providing justification, that university students suffer from insomnia more frequently than do members of the general population', 'presumes, without providing justification, that there is such a thing as a regular sleep pattern for someone working on a commercial fishing vessel', 'presumes that all instances of insomnia have the same cause']", "label": 0 }, { "id": "train_2222", "context": "Essayist: Some researchers criticize British governmental security agencies for not releasing enough information about sightings of unidentified flying objects (UFOs) made by these military and intelligence agencies. Requests for information by civilian researchers are brushed aside. This leads one to suspect that there have been spacecraft sighted near Earth that are extraterrestrial in origin.", "question": "Which one of the following, if true, would most strengthen the essayist's argument?", "answers": "['The British government would deny the requests by civilian researchers to have access to the data only if this government had something to hide.', 'The British government is less trusting of civilian researchers than it is of military researchers.', 'The British government would withhold information pertaining to UFOs only if it were established that they were from other planets.', 'The British government has always attempted to deny the existence of UFOs.']", "label": 2 }, { "id": "train_2223", "context": "Occultist: If there are ghosts, then they are supernatural beings. But science studies only natural phenomena. Ttherefore, there can be no scientific basis for criticizing the evidence that leads people to believe in ghosts.", "question": "The occultist's argument is most vulnerable to criticism on the grounds that it takes for granted that", "answers": "['other supernatural beings are not responsible for the evidence that leads people to believe in ghosts', 'people who believe in ghosts do so on the basis of evidence', 'the evidence that leads people to believe in ghosts cannot consist of natural phenomena', 'there are no methods for ascertaining the truth or falsity of any belief about the supernatural']", "label": 2 }, { "id": "train_2224", "context": "Scientist: There is a lot of concern that human behavior may be responsible for large-scale climate change. But this should be seen as more of an opportunity than a problem. If human behavior is responsible for climate change, then we can control future climate change to make it less extreme than previous climate shifts.", "question": "The scientist's argument requires assuming which one of the following?", "answers": "['At least some previous large-scale climate changes have been caused by human behavior.', 'Human beings can control the aspects of their behavior that have an impact on climate change.', 'It is easier to identify the human behaviors that cause climate change than it is to change those behaviors.', 'The same degree of climate change produces less damage if it is caused by human behavior than if it has a purely natural cause.']", "label": 1 }, { "id": "train_2225", "context": "Typically during thunderstorms most lightning strikes carry a negative electric charge; only a few carry a positive charge. Thunderstorms with unusually high proportions of positive-charge strikes tend to occur in smoky areas near forest fires. The fact that smoke carries positively charged smoke particles into the air above a fire suggests the hypothesis that the extra positive strikes occur because of the presence of such particles in the storm clouds.", "question": "Which of the following, if discovered to be true, most seriously undermines the hypothesis?", "answers": "['The positive-charge strikes that occur near forest fires tend to be no more powerful than positive strikes normally are.', 'Other kinds of rare lightning also occur with unusually high frequency in the vicinity of forest fires.', 'Thunderstorms that occur in drifting clouds of smoke have extra positive-charge strikes weeks after the charge of the smoke particles has dissipated.', 'A positive-charge strike is as likely to start a forest fire as a negative-charge strike is.']", "label": 2 }, { "id": "train_2226", "context": "Viewers surveyed immediately after the televised political debate last year between Lopez and Tanner tended to think that Lopez had made the better arguments, but the survey respondents who reported that Lopez' s arguments were better may have been biased in favor of Lopez. After all, Lopez eventually did win the election.", "question": "Which one of the following, if true, most seriously undermines the argument?", "answers": "['Lopez won the election over Tanner by a very narrow margin.', 'Most of the viewers surveyed immediately prior to the debate said that they would probably vote for Tanner.', 'Most people in the live audience watching the debate who were surveyed immediately afterward said that they thought that Tanner was more persuasive in the debate than was Lopez.', 'The people who watched the televised debate were more likely to vote for Tanner than were the people who did not watch the debate.']", "label": 1 }, { "id": "train_2227", "context": "If Juan went to the party, it is highly unlikely that Maria would have enjoyed the party. But in fact it turned out that Maria did enjoy the party; ttherefore, it is highly unlikely that Juan was at the party.", "question": "The pattern of reasoning in the argument above is most similar to that in which one of the following?", "answers": "[\"This year's election will probably be fair. But Popov probably will not win unless the election is unfair. So, Popov will not win the election.\", 'Kapinski, our new neighbor, is probably friendly, for Kapinski sells insurance and most people who sell insurance are friendly.', 'If Clarissa missed the bus today, it is quite unlikely that she would have gotten to work on time. So, it is quite unlikely that Clarissa missed the bus, since she actually was at work on time today.', 'If the lottery were fair, the person who won the lottery would not have been likely to win it. Thus, since this person would have been likely to win the lottery if it were unfair, the lottery was probably unfair.']", "label": 2 }, { "id": "train_2228", "context": "In countries in which new life-sustaining drugs cannot be patented, such drugs are sold at widely affordable prices; those same drugs, where patented, command premium prices because the patents shield patent-holding manufacturers from competitors. These facts show that future access to new life-sustaining drugs can be improved if the practice of granting patents on newly developed life-sustaining drugs were to be abolished everywhere.", "question": "Which of the following, if true, most seriously weakens the argument?", "answers": "['In some countries specific processes for the manufacture of pharmaceutical drugs can be patented even in cases in which the drugs themselves cannot be patented.', 'Countries that grant patents on life-sustaining drugs almost always ban their importation from countries that do not grant such patents.', 'Countries that do not currently grant patents on life-sustaining drugs are, for the most part, countries with large populations.', 'Pharmaceutical companies can afford the research that goes into the development of new drugs only if patents allow them to earn high profits.']", "label": 3 }, { "id": "train_2229", "context": "The Sumpton town council recently voted to pay a prominent artist to create an abstract sculpture for the town square. Critics of this decision protested that town residents tend to dislike most abstract art, and any art in the town square should reflect their tastes. But a town council spokesperson dismissed this criticism, pointing out that other public abstract sculptures that the same sculptor has installed in other cities have been extremely popular with those cities' local residents.", "question": "The statements above most strongly suggest that the main point of disagreement between the critics and the spokesperson is whether", "answers": "['abstract sculptures by the same sculptor have truly been popular in other cities', 'it would have been reasonable to consult town residents on the decision', 'a more traditional sculpture in the town square would be popular among local residents', 'most Sumpton residents will find the new sculpture to their taste']", "label": 3 }, { "id": "train_2230", "context": "In a study conducted in Pennsylvania, servers in various restaurants wrote \"Thank you\" on randomly selected bills before presenting the bills to their customers. Tips on these bills were an average of three percentage points higher than tips on bills without the message. Ttherefore, if servers in Pennsylvania regularly wrote \"Thank you\" on restaurant bills, their average income from tips would be significantly higher than it otherwise would have been.", "question": "Which of the following is an assumption on which the argument relies?", "answers": "['Virtually all patrons of the Pennsylvania restaurants in the study who were given a bill with \"Thank you\" written on it left a larger tip than they otherwise would have.', 'Regularly seeing \"Thank you\" written on their bills would not lead restaurant patrons to revert to their earlier tipping habits.', 'The \"Thank you\" messages would have the same impact on regular patrons of a restaurant as they would on occasional patrons of the same restaurant.', 'The written \"Thank you\" reminds restaurant patrons that tips constitute a significant part of the income of many food servers.']", "label": 1 }, { "id": "train_2231", "context": "Reza: Language requires the use of verbal signs for objects as well as for feelings. Many animals can vocally express hunger, but only humans can ask for an egg or an apple by naming it. And using verbal signs for objects requires the ability to distinguish these objects from other objects, which in turn requires conceptual thought.", "question": "If all of Reza's statements are true, then which one of the following must also be true?", "answers": "['All humans are capable of conceptual thought.', 'Conceptual thought requires the use of verbal signs for objects.', 'Conceptual thought is required for language.', 'The vocal expressions of animals other than humans do not require conceptual thought.']", "label": 2 }, { "id": "train_2232", "context": "When the Hurricane Research Institute first determined that 2005 would be the most active hurricane season in recorded history, the researchers at the Institute alerted the federal government. The federal government sent out an alert to at risk communities within two weeks of receiving the alert from the Institute. Although the alert was sent out, the hurricane season proved to be the most disastrous in American history. It is obvious that the local governments of at risk communities did not heed the warning.", "question": "The conclusion above would be most undermined if it were true that", "answers": "['because of the power and frequency of the hurricanes, the damage caused could not have been prevented', 'the damage from hurricanes in areas not at risk was negligible', 'hurricanes cannot be prevented', 'the federal government offered all available resources to help protect the communities']", "label": 0 }, { "id": "train_2233", "context": "The government has no right to tax earnings from labor. Taxation of this kind requires the laborer to devote a certain percentage of hours worked to earning money for the government. Thus, such taxation forces the laborer to work, in part, for another' s purpose. Since involuntary servitude can be defined as forced work for another' s purpose, just as involuntary servitude is pernicious, so is taxing earnings from labor.", "question": "The argument uses which one of the following argumentative techniques?", "answers": "['citing the authority of an economic theory in order to justify a moral principle', 'inferring that since two institutions are similar in one respect, they are similar in another respect', 'deriving a general principle about the rights of individuals from a judgment concerning the obligations of governments', 'presupposing the inevitability of a hierarchical class system in order to oppose a given economic practice']", "label": 1 }, { "id": "train_2234", "context": "Reporter: A team of scientists has recently devised a new test that for the first time accurately diagnoses autism in children as young as 18 months old. When used to evaluate 16, 000 children at their 18-month checkup, the test correctly diagnosed all 10 children later confirmed to be autistic, though it also wrongly identified 2 children as autistic. Autistic children can ttherefore now benefit much earlier in life than before from the treatments already available.", "question": "Which one of the following is an assumption on which the reporter's argument depends?", "answers": "['There was no reliable evidence that autism could affect children so young until the advent of the new test.', 'The new test can be used to evaluate all children, regardless of the level of development of their verbal skills.', 'A diagnostic test that sometimes falsely gives a positive diagnosis can still provide a reasonable basis for treatment decisions.', 'No test intended for diagnosing autism at such an early age existed before the new test was devised.']", "label": 2 }, { "id": "train_2235", "context": "Conservationist: The risk to train passengers from collisions between trains using the train station and deer from the wildlife refuge is insignificant. In the 12 years since the refuge was established, only 15 trains have been damaged in collisions with deer, and no passenger has been injured as a result of such a collision. The wildlife refuge, ttherefore, poses no safety risk. Conductor: You neglect to mention that 13 of those 15 collisions occurred within the past year and a half, and that the number of deer in the refuge is rapidly increasing. As the number of collisions between deer and trains increases, so does the likelihood that at least one such collision will result in passenger injuries.", "question": "The conductor counters the conservationist by", "answers": "[\"attempting to show that the conservationist's description of the facts is misleading\", 'asserting that dangerous situations inevitably become more dangerous with the passage of time', \"discrediting the moral principle on which the conservationist's argument is based\", \"questioning the conservationist's motives for reaching a certain conclusion\"]", "label": 0 }, { "id": "train_2236", "context": "The lobbyist accused of offering a large cash bribe to the senator defended himself: \"When I left the house that day, I was carrying no money with me, so I could not possibly have had anything to offer to the senator. Moreover, immediately before I met with the senator, I spent all my cash on lunch with a colleague at an upscale restaurant, which also explains why I was not in a position to offer the senator a bribe. ", "question": "This argument is most vulnerable to what criticism?", "answers": "['It offers a conclusion that is no more than a paraphrase of one piece of the pieces of information provided in its support.', 'It presents as evidence in support of a claim information that is inconsistent with other evidence presented in support of the same claim.', 'It does not preserve the proper time relationship between cause and effect.', 'It presents two pieces of evidence that do not support the same conclusion.']", "label": 1 }, { "id": "train_2237", "context": "While it is true that bees' vision is well suited to the task of identifying flowers by their colors, it is probable that flowers developed in response to the type of vision that bees have, rather than bees' vision developing in response to flower color.", "question": "Which one of the following, if true, most strongly supports the statement above?", "answers": "[\"Many insects that have vision very similar to that of bees do not depend on perceiving an object's color.\", 'Present-day bees rely exclusively on flowers for their food.', 'The number of different species of flowers is greater than the number of different species of bees.', 'Many nonflowering plants rely on bees.']", "label": 0 }, { "id": "train_2238", "context": "Columnist: The advent of television helps to explain why the growth in homicide rates in urban areas began significantly earlier than the growth in homicide rates in rural areas. Television sets became popular in urban households about five years earlier than in rural households. Urban homicide rates began increasing in 1958, about four years earlier than a similar increase in rural homicide rates began.", "question": "Which one of the following, if true, most supports the columnist's argument?", "answers": "['The portrayal of violence on television is a cause, not an effect, of the violence in society.', 'There were no violent television programs during the early years of television.', \"Increasing one's amount of leisure time increases one's inclination to act violently.\", 'The earlier one is exposed to violence on television, the more profound the effect.']", "label": 0 }, { "id": "train_2239", "context": "Economist: Although obviously cuts in personal income tax rates for the upper income brackets disproportionately benefit the wealthy, across-the-board cuts for all brackets tend to have a similar effect. Personal income tax rates are progressive (i. e. , graduated), and if total revenue remains constant, then across-the-board cuts in these taxes require increasing the amount of revenue generated through nonprogressive taxes, thereby favoring the wealthy. Yet if nonprogressive taxes are not increased to compensate for the cuts, then the budget deficit will increase, requiring more government borrowing and driving up interest rates. This favors those who have money to lend, once again benefiting primarily the wealthy.", "question": "Which one of the following statements most accurately expresses the main conclusion of the economist's argument?", "answers": "['It is the wealthy who are favored by generating a high amount of revenue through nonprogressive taxes.', 'Across-the-board personal income tax rate cuts generally benefit the wealthy more than they benefit others.', 'Across-the-board cuts in personal income tax rates do not generate enough additional economic activity to prevent a net loss of revenue.', 'Cuts in personal income tax rates for upper income brackets benefit the wealthy more than they benefit others.']", "label": 1 }, { "id": "train_2240", "context": "Chiu: The belief that a person is always morally blameworthy for feeling certain emotions, such as unjustifiable anger, jealousy, or resentment, is misguided. Individuals are responsible for only what is under their control, and whether one feels such an emotion is not always under one' s control.", "question": "Chiu's conclusion follows logically if which one of the following is assumed?", "answers": "[\"The emotions for which a person is most commonly blamed are those that are under that person's control.\", 'Although a person may sometimes be unjustifiably angry, jealous, or resentful, there are occasions when these emotions are appropriate.', 'Individuals do not have control over their actions when they feel certain emotions.', 'If a person is morally blameworthy for something, then that person is responsible for it.']", "label": 3 }, { "id": "train_2241", "context": "A moderately large city is redesigning its central downtown area and is considering a plan that would reduce the number of lanes for automobiles and trucks and increase those for bicycles and pedestrians. The intent is to attract more workers and shoppers to downtown businesses by making downtown easier to reach and more pleasant to move around in.", "question": "Which of the following would, if true, most strongly support the prediction that the plan would achieve its goal?", "answers": "['Most people who prefer to shop at suburban malls instead of downtown urban areas do so because parking is easier and cheaper at the former.', 'If the proposed lane restrictions on drivers are rigorously enforced, more people will likely be attracted to downtown businesses than would otherwise be.', 'In other moderately sized cities where measures were taken to make downtowns more accessible for walkers and cyclists, downtown businesses began to thrive.', 'People who make a habit of walking or bicycling whenever feasible derive significant health benefits from doing so.']", "label": 2 }, { "id": "train_2242", "context": "The use of money causes a civilization to decline. That this is true is shown by the way the troubles of Western civilization began with the invention of money. While real money (gold and silver) is bad enough, imitation money (paper money) is a horror. The decline of Western civilization exactly parallels the increasing use of money -- both real money and worthless paper money -- as a substitute for things of intrinsic value.", "question": "Which one of the following, if true, could contribute most to a refutation of the argument?", "answers": "['Eastern cultures have used money, and Eastern civilizations have not declined.', \"Some employers exchange goods for their employees' services in order to avoid the exchange of money.\", 'People prefer using money to having a system in which goods are bartered for other goods of equal intrinsic value.', 'The rate of exchange between gold and paper money has fluctuated greatly in Western civilization.']", "label": 0 }, { "id": "train_2243", "context": "If retail stores experience a decrease in revenues during this holiday season, then either attitudes toward extravagant gift-giving have changed or prices have risen beyond the level most people can afford. If attitudes have changed, then we all have something to celebrate this season. If prices have risen beyond the level most people can afford, then it must be that salaries have not kept pace with rising prices during the past year. Assume the premises above to be true.", "question": "If salaries have kept pace with rising prices during the past year, which one of the following must be true?", "answers": "['Attitudes toward extravagant gift-giving have changed.', 'Prices in retail stores have not risen beyond the level that most people can afford during this holiday season.', 'Attitudes toward extravagant gift-giving have not changed, and stores will not experience a decrease in revenues during this holiday season.', 'Either attitudes toward extravagant gift-giving have changed or prices have risen beyond the level that most people can afford during this holiday season.']", "label": 1 }, { "id": "train_2244", "context": "Nearly everyone has complained of a mistaken utility bill that cannot easily be corrected or of computer files that cannot readily be retrieved. Yet few people today would tolerate waiting in long lines while clerks search for information that can now be found in seconds, and almost no one who has used a word processor would return to a typewriter.", "question": "The information above conforms most closely to which one of the following principles?", "answers": "['The fact that people complain about some consequences of technology cannot be taken as a reliable indication that they would choose to live without it.', 'If people do not complain about some technology, then it is probably not a significant factor in their daily lives.', 'The complaints people make about technological innovations are more reliable evidence of the importance of those innovations than the choices people actually make.', 'The degree to which technologies elicit complaints from people is always an accurate measure of the extent to which people have become dependent on them.']", "label": 0 }, { "id": "train_2245", "context": "Raj: Dogs that have been traditionally used for violent tasks should not be kept as pets. Hardly a day goes by when the newspapers do not feature a story where a child has been mauled by a Pit Bull or a Rottweiler. These animals' innate desire to attack and guard make them an unmitigated danger to young children who might accidentally cause the animal to feel threatened. Melanie: That is an unfair assessment of these animals' character. Any innate traits that have been acquired through breeding and bloodlines can be overcome by proper training and discipline. For instance, many Bloodhounds, known for their hunting ability, have been trained to not attack rabbits or squirrels.", "question": "The statement that Bloodhounds have been trained to not attack rabbits or squirrels plays which of the following roles in Melanie's argument?", "answers": "['It offers a counterexample to illustrate the general conclusion.', 'It is offered as supplementary proof that no dogs have an innate desire to attack and guard.', \"It provides support for Melanie's argument that also inadvertently strengthens Raj's argument.\", 'It offers a theory that harmonizes both the stated position and the stated conclusion.']", "label": 0 }, { "id": "train_2246", "context": "In considering the fact that many people believe that promotions are often given to undeserving employees because the employees successfully flatter their supervisors, a psychologist argued that although many people who flatter their supervisors are subsequently promoted, flattery generally is not the reason for their success, because almost all flattery is so blatant that it is obvious even to those toward whom it is directed.", "question": "Which one of the following, if assumed, enables the psychologist's conclusion to be properly drawn?", "answers": "['People in positions of responsibility expect to be flattered.', 'Official guidelines for granting promotion tend to focus on merit.', 'Supervisors are almost never influenced by flattery when they notice it.', 'Flattery that is not noticed by the person being flattered is ineffective.']", "label": 2 }, { "id": "train_2247", "context": "Despite improvements in treatment for asthma, the death rate from this disease has doubled during the past decade from its previous rate. Two possible explanations for this increase have been offered. First, the recording of deaths due to asthma has become more widespread and accurate in the past decade than it had been previously. Second, there has been an increase in urban pollution. However, since the rate of deaths due to asthma has increased dramatically even in cities with long-standing, comprehensive medical records and with little or no urban pollution, one must instead conclude that the cause of increased deaths is the use of bronchial inhalers by asthma sufferers to relieve their symptoms.", "question": "Each of the following, if true, provides support to the argument EXCEPT:", "answers": "['By temporarily relieving the symptoms of asthma, inhalers encourage sufferers to avoid more beneficial measures.', 'Evidence suggests that bronchial inhalers make the lungs more sensitive to irritation by airborne pollen.', 'Ten years ago bronchial inhalers were not available as an asthma treatment.', 'Urban populations have doubled in the past decade.']", "label": 3 }, { "id": "train_2248", "context": "Teachers are effective only when they help their students become independent learners. Yet not until teachers have the power to make decisions in their own classrooms can they enable their students to make their own decisions. Students' capability to make their own decisions is essential to their becoming independent learners. Ttherefore, if teachers are to be effective, they must have the power to make decisions in their own classrooms.", "question": "According to the argument, each of the following could be true of teachers who have enabled their students to make their own decisions EXCEPT:", "answers": "['Their students have not become independent learners.', 'They are not effective teachers.', 'They do not have the power to make decisions in their own classrooms.', 'They are effective teachers.']", "label": 2 }, { "id": "train_2249", "context": "A photograph of the night sky was taken with the camera shutter open for an extended period. The normal motion of stars across the sky caused the images of the stars in the photograph to appear as streaks. However, one bright spot was not streaked. Even if the spot were caused, as astronomers believe, by a celestial object, that object could still have been moving across the sky during the time the shutter was open, since __.", "question": "Which of the following most logically completes the argument?", "answers": "['the spot was not the brightest object in the photograph', 'the photograph contains many streaks that astronomers can identify as caused by noncelestial objects', 'the spot could have been caused by an object that emitted a flash that lasted for only a fraction of the time that the camera shutter was open', 'stars in the night sky do not appear to shift position relative to each other']", "label": 2 }, { "id": "train_2250", "context": "A law that is not consistently enforced does not serve its purpose. Law without enforcement is not law; it is merely statute -- a promise of law. To institute real law is not merely to declare that such and such behavior is forbidden; it is also to punish those who violate that edict. Furthermore, those who enforce law must punish without favor for their friends or malice for their enemies. To punish only those one dislikes while forgiving others is not to enforce law but to engage in the arbitrary and unjust exercise of power.", "question": "The main point of the passage is that instituting real law consists in", "answers": "['the exercise of power', 'understanding the purpose of law', 'clearly defining unacceptable behavior', 'the unbiased punishment of prohibited behavior']", "label": 3 }, { "id": "train_2251", "context": "Neither a rising standard of living nor balanced trade, by itself, establishes a country's ability to compete in the international marketplace. Both are required simultaneously since standards of living can rise because of growing trade deficits and trade can be balanced by means of a decline in a country's standard of living.", "question": "If the facts stated in the passage above are true, a proper test of a country's ability to be competitive is its ability to", "answers": "['balance its trade while its standard of living falls', 'increase trade deficits while its standard of living rises', 'decrease trade deficits while its standard of living falls', 'balance its trade while its standard of living rises']", "label": 3 }, { "id": "train_2252", "context": "Some music theorists argue that music can arouse pity in the listener. But since pity can be felt only when there is someone or something to be pitied, and since nothing but the music is available to be the object of the listener' s pity, if anything is pitiable, then it is the music itself. But it makes no sense to say that a piece of music is pitiable; so music cannot arouse pity in the listener.", "question": "The pattern of reasoning in the argument above is most similar to that in which one of the following arguments?", "answers": "[\"Some people claim that the quality of one's life can be estimated by summing all the positive aspects of one's life and subtracting all the negative aspects. This implies that it is possible to place a numerical value on such things as health and emotional well-being; but trying to put a numerical value on such things is nonsense. Thus, the quality of a life cannot be measured in this way.\", 'Some statisticians predict future events by observing past events; this requires extrapolating from the past. Though it is impossible to make completely accurate predictions through extrapolation, many predictions can be made with at least a reasonable degree of confidence. Ttherefore, these statisticians are justified in making predictions about the future.', 'Some science writers imagine what life would be like if humans could inhabit distant planets. Inhabiting these planets, they claim, is an inevitable consequence of current research. But it is absurd to think that humans will actually live on other planets. Ttherefore, it must be wrong to claim that this follows inevitably from current research.', 'Astronomers use color photography to measure the temperatures of stars. Using color photography implies that the object photographed has color. But it makes no sense to speak of the color of stars because stars are clouds of gas. Ttherefore, color photographs of stars must represent something other than the color of the stars.']", "label": 0 }, { "id": "train_2253", "context": "Mayor: Four years ago, when we reorganized the city police department in order to save money, critics claimed that the reorganization would make the police less responsive to citizens and would thus lead to more crime. The police have compiled theft statistics from the years following the reorganization that show that the critics were wrong. There was an overall decrease in reports of thefts of all kinds, including small thefts.", "question": "Which of the following, if true, most seriously challenges the mayor's argument?", "answers": "['In other cities where police departments have been similarly reorganized, the numbers of reported thefts have generally risen following reorganization.', 'When city police are perceived as unresponsive, victims of theft are less likely to report thefts to the police.', \"The mayor's critics generally agree that police statistics concerning crime reports provide the most reliable available data on crime rates.\", \"The mayor's reorganization of the police department failed to save as much money as it was intended to save.\"]", "label": 1 }, { "id": "train_2254", "context": "Political scientist: It is not uncommon for a politician to criticize his or her political opponents by claiming that their exposition of their ideas is muddled and incomprehensible. Such criticism, however, is never sincere. Political agendas promoted in a manner that cannot be understood by large numbers of people will not be realized for, as every politician knows, political mobilization requires commonality of purpose.", "question": "Which one of the following is the most accurate rendering of the political scientist's main conclusion?", "answers": "['People who promote political agendas in an incomprehensible manner should be regarded as insincere.', 'Sincere critics of the proponents of a political agenda should not focus their criticisms on the manner in which that agenda is promoted.', 'A politician criticizing his or her political opponents for presenting their political agendas in an incomprehensible manner is being insincere.', 'To mobilize large numbers of people in support of a political agenda, that political agenda must be presented in such a way that it cannot be misunderstood.']", "label": 2 }, { "id": "train_2255", "context": "It is crucially important to farmers that the herbicides they use to control weeds do not damage their crops. One very effective herbicide is safe for corn, but soybeans are damaged even by the herbicide' s residue, which remains in the soil more than a year after the herbicide is applied. Soybeans and corn are not sown together in the same field; nevertheless, most farmers are reluctant to use the herbicide on their corn.", "question": "Which of the following, if true, provides the strongest justification for the farmers' reluctance?", "answers": "['The demand for soybeans is growing faster than is the demand for corn.', 'To maintain the nutrients in the soil, corn and soybeans are often planted in a field in alternate years.', 'The residue of the herbicide in the soil a year after application is not enough to control most weeds effectively.', 'The application of herbicides is less critical for soybean than for corn crops.']", "label": 1 }, { "id": "train_2256", "context": "In a poll conducted by interviewing eligible voters in their homes just before the recent election, incumbent candidate Kenner was significantly ahead of candidate Muratori. Nonetheless, Muratori won the recent election.", "question": "Which one of the following, if true, most helps to resolve the apparent discrepancy described by the statements above?", "answers": "['In the year leading up to the election, Kenner was implicated in a series of political scandals.', 'In the poll, supporters of Muratori were more likely than others to describe the election as important.', 'Kenner had held elected office for many years before the recent election.', 'The positions taken by Muratori and Kenner on many election issues were not very similar to each other.']", "label": 1 }, { "id": "train_2257", "context": "Studies have shown that the more high-stress points a bridge has, the more likely it is to fracture eventually. This might lead one to expect fractures to develop at high-stress points. Surprisingly, however, fractures develop not at high-stress points but elsewhere on the bridge.", "question": "Which one of the following, if true, contributes most to an explanation of why bridges fracture elsewhere than at high-stress points?", "answers": "['Structures with no high-stress points can nonetheless have a high probability of fracturing.', 'Fractures do not develop at high-stress points, because bridges are reinforced at those points; however, stress is transferred to other points on the bridge where it causes fractures.', 'In many structures, the process of fracturing often causes high-stress points to develop.', 'In many structures other than bridges, such as ship hulls and airplane bodies, fractures do not develop at high-stress points.']", "label": 1 }, { "id": "train_2258", "context": "Psychologist: It is well known that becoming angry often induces temporary incidents of high blood pressure. A recent study further showed, however, that people who are easily angered are significantly more likely to have permanently high blood pressure than are people who have more tranquil personalities. Coupled with the long-established fact that those with permanently high blood pressure are especially likely to have heart disease, the recent findings indicate that heart disease can result from psychological factors.", "question": "Which one of the following would, if true, most weaken the psychologist's argument?", "answers": "['The physiological factors that cause permanently high blood pressure generally make people quick to anger.', 'Those who are easily angered are less likely to recover fully from episodes of heart disease than are other people.', 'Those who discover that they have heart disease tend to become more easily frustrated by small difficulties.', 'People with permanently high blood pressure who have tranquil personalities virtually never develop heart disease.']", "label": 0 }, { "id": "train_2259", "context": "In 1983 Argonia' s currency, the argon, underwent a reduction in value relative to the world' s strongest currencies. This reduction resulted in a significant increase in Argonia' s exports over 1982 levels. In 1987 a similar reduction in the value of the argon led to another increase in Argonia' s exports. Faced with the need to increase exports yet again, Argonia' s finance minister has proposed another reduction in the value of the argon.", "question": "Which of the following, if true, most strongly supports the prediction that the finance minister's plan will not result in a significant increase in Argonia's exports next year?", "answers": "[\"The value of the argon rose sharply last year against the world's strongest currencies.\", \"All of Argonia's export products are made by factories that were operating at full capacity last year, and new factories would take years to build.\", \"The value of Argonia's exports was lower last year than it was the year before.\", 'Reductions in the value of the argon have almost always led to significant reductions in the amount of goods and services that Argonians purchase from abroad.']", "label": 1 }, { "id": "train_2260", "context": "The seventeenth-century physicist Sir Isaac Newton is remembered chiefly for his treatises on motion and gravity. But Newton also conducted experiments secretly for many years based on the arcane theories of alchemy, trying unsuccessfully to transmute common metals into gold and produce rejuvenating elixirs. If the alchemists of the seventeenth century had published the results of their experiments, chemistry in the eighteenth century would have been more advanced than it actually was.", "question": "Which one of the following assumptions would allow the conclusion concerning eighteenth-century chemistry to be properly drawn?", "answers": "['Scientific progress is retarded by the reluctance of historians to acknowledge the failures of some of the great scientists.', 'Advances in science are hastened when reports of experiments, whether successful or not, are available for review by other scientists.', 'The seventeenth-century alchemists could have achieved their goals only if their experiments had been subjected to public scrutiny.', \"Newton's work on motion and gravity would not have gained wide acceptance if the results of his work in alchemy had also been made public.\"]", "label": 1 }, { "id": "train_2261", "context": "Engineers are investigating the suitability of Wantastiquet Pass as the site of a new bridge. Because one concern is whether erosion could eventually weaken the bridge' s foundations, they contracted for two reports on erosion in the region. Although both reports are accurate, one claims that the region suffers relatively little erosion, while the other claims that regional erosion is heavy and a cause for concern.", "question": "Which one of the following, if true, most helps to explain how both reports could be accurate?", "answers": "['One report cost nearly twice as much to prepare as did the other report.', 'One report focuses on regional topsoil erosion, while the other report focuses on riverbank erosion resulting from seasonal floods.', 'One report was prepared by scientists from a university, while the other report was prepared by scientists from a private consulting firm.', 'Both reports include computer-enhanced satellite photographs.']", "label": 1 }, { "id": "train_2262", "context": "Editorialist: There would seem to be little hazard for consumers associated with chemicals used in treated lumber because the lumber is used outside where fumes cannot accumulate. However, immediate steps should be taken to determine the safety of these chemicals since consumers could ingest them. If the lumber is used for children' s playground equipment, youngsters could put their mouths on the wood, and if it is used to contain soil in a vegetable garden, the chemicals could leach into the soil.", "question": "Which one of the following most accurately expresses the main conclusion of the editorialist's argument?", "answers": "['The chemicals used in treated lumber are apparently not dangerous to the consumer.', 'Treated lumber is more dangerous than was once believed.', 'Parents should not allow children to put their mouths on playground equipment.', 'The effects on humans from the chemicals in treated lumber should be studied.']", "label": 3 }, { "id": "train_2263", "context": "Biologist: Many paleontologists have suggested that the difficulty of adapting to ice ages was responsible for the evolution of the human brain. But this suggestion must be rejected, for most other animal species adapted to ice ages with no evolutionary changes to their brains.", "question": "The biologist's argument is most vulnerable to criticism on which one of the following grounds?", "answers": "['It fails to address adequately the possibility that a condition can produce a change in a species even if it does not produce that change in other species.', 'It presumes without warrant that human beings were presented with greater difficulties during ice ages than were individuals of most other species.', 'It takes for granted that, if a condition coincided with the emergence of a certain phenomenon, that condition must have been causally responsible for the phenomenon.', 'It overlooks the possibility that a condition that is needed to produce a change in one species is not needed to produce a similar change in other species.']", "label": 0 }, { "id": "train_2264", "context": "A product that represents a clear technological advance over competing products can generally command a high price. Surprisingly, perhaps, the strategy to maximize overall profit from a new product is to charge less than the greatest price the market will bear . Many companies charge the maximum possible price for such a product, because they want to make as much profit as they can and technological advances tend to be quickly surpassed. The drawback is that large profits on the new product give competitors a strong incentive to quickly match the new product' s capabilities .", "question": "In the argument above, the two portions in boldface play which of the following roles?", "answers": "['The first is the position the argument advocates; the second presents grounds for rejecting an alternative position.', 'The first presents a strategy for achieving a certain goal; the second presents a drawback to that strategy.', 'The first is the position the argument advocates; the second is an alternative position that the argument rejects.', 'The first presents a strategy for achieving a certain goal; the second presents grounds for preferring a different goal.']", "label": 0 }, { "id": "train_2265", "context": "George: Parents should not give toddlers access to cellphones, tablets, or other screens because it is damaging to their eyes and brain. Parents just do it because they are lazy and don' t want to get on the floor and actually play with their child or teach them something. Increasingly younger kids are navigating and playing with phones and are missing out on books and real learning. Karen: Not all screen time is bad; in fact, it' s helpful for young kids to start tinkering with technology like smartphones because they will be using such devices their whole lives. Starting them young optimizes their skills; toddlers are like sponges! They learn so quickly and they can learn from the right apps on tablets and phones. There are many educational apps for babies and toddlers.", "question": "Which of the following best aligns with Karen's argument?", "answers": "['Toddlers should just be left alone to decide for themselves.', 'Children of all ages should be allowed 1-2 hours per day of screen time.', 'Toddlers should be made to read at least 2 hours per day.', 'Children of all ages should be allowed unlimited screen time.']", "label": 1 }, { "id": "train_2266", "context": "Parents: Wedid not hire the babysitter who was recommended because during the interview he made promises solely to persuade us to hire him. What the babysitter has said he would do if hired are merely the tasks he has learned from other interviews that parents want babysitters to do. Hence, we were not being informed of what tasks the babysitter really meant to do if we hired him.", "question": "Which of the following is a dubious assumption that the argument above relies on?", "answers": "['If the babysitter had been hired, he would have been unable to complete the tasks that parents want babysitters to do.', 'The babysitter did not really intend, if hired, to do the tasks he has learned from other interviews that parents want babysitters to do.', 'Most parents would not hire a babysitter who promised to do the tasks that parents want babysitters to do.', 'The babysitter did not also plan to do other tasks in addition to those that parents want babysitters to do.']", "label": 1 }, { "id": "train_2267", "context": "Inflation rates will not stabilize unless the rate of economic growth decreases. Yet in order to slow the economy, the full cooperation of world leaders will be required. Thus, it would be overly optimistic to expect stable inflation rates in the near future.", "question": "Which one of the following is most closely parallel in its reasoning to the reasoning in the argument above?", "answers": "['If the board is serious about improving management efficiency, it must eliminate organizational redundancy. Unfortunately, it will not be possible to eliminate wasteful redundancy without dismissing a number of senior employees. Thus, no option is available but to dismiss some senior employees.', 'If we are to produce the safest vehicles possible, we must conduct objective structural tests. However, the performance of such objective tests will inevitably result in huge cost overruns. It is ttherefore unavoidable that the level of vehicle safety will not be optimal.', 'Only if we thoroughly examine all options will we be able to arrive at the optimal decision. Such a thorough examination, however, will necessitate a delay in the presentation of our proposal. Ttherefore, we will be able to arrive at an optimal decision only if we delay the presentation of our proposal.', \"If honesty is the best policy, we should report our company's poor performance in the last year. But if we do so, we will put our jobs at risk and our stockholders will complain. Ttherefore, we must not report our poor performance.\"]", "label": 1 }, { "id": "train_2268", "context": "Peter: Unlike in the past, most children' s stories nowadays don' t have clearly immoral characters in them. They should, though. Children need to learn the consequences of being bad. Yoko: Children' s stories still tend to have clearly immoral characters in them, but now these characters tend not to be the sort that frighten children. Surely that' s an improvement.", "question": "Peter and Yoko disagree over whether today's children's stories", "answers": "[\"tend to be less frightening than earlier children's stories were\", \"differ significantly in overall quality from earlier children's stories\", 'tend to have clearly immoral characters in them', 'should be less frightening than they are']", "label": 2 }, { "id": "train_2269", "context": "Pretzels can cause cavities. Interestingly, the longer that a pretzel remains in contact with the teeth when it is being eaten, the greater the likelihood that a cavity will result. What is true of pretzels in this regard is also true of caramels. Ttherefore, since caramels dissolve more quickly in the mouth than pretzels do, eating a caramel is less likely to result in a cavity than eating a pretzel is.", "question": "The reasoning in the argument is vulnerable to criticism on the grounds that the argument", "answers": "['mistakes the cause of a particular phenomenon for the effect of that phenomenon', 'treats a correlation that holds within individual categories as thereby holding across categories as well', 'relies on the ambiguous use of a key term', 'is based on premises that cannot all be true']", "label": 1 }, { "id": "train_2270", "context": "Conservationist: The population of a certain wildflower is so small that the species is headed for extinction. However, this wildflower can cross-pollinate with a closely related domesticated daisy, producing viable seeds. Such cross-pollination could result in a significant population of wildflower-daisy hybrids. The daisy should ttherefore be introduced into the wildflower' s range, since although the hybrid would differ markedly from the wildflower, hybridization is the only means of preventing total loss of the wildflower in its range.", "question": "Which one of the following principles, if valid, most helps to justify the conservationist's reasoning?", "answers": "['It is better to change a type of organism that would otherwise be lost, even if the changes are radical, than to lose it entirely.', 'It is better to preserve a type of organism that is in danger of extinction, even if surviving organisms of that type are not vigorous, than to allow something more vigorous to replace it.', 'It is better to take measures to preserve a valued type of organism, even if those measures are drastic, than to accept a less valuable substitute for the organism.', 'It is better to destroy one of two competing types of organisms, even if both are irreplaceable, than to allow both of them to be lost.']", "label": 0 }, { "id": "train_2271", "context": "Mortgage banker: There is a remarkable correlation between fluctuations in the federal interest rates and the strength of our nation' s currency. Undoubtedly, despite the opinions of the majority of economists, the federal interest rates effectively control the strength of our nation' s currency. Economist: I beg to differ. Any experienced economist will testify that in regards to something as complex as our nation' s economic status, no consequential component is shaped or controlled by any one factor.", "question": "The renunciation by the economist of the banker's conclusion utilizes which one of the following techniques of argumentation?", "answers": "['offering a detailed counterexample that illustrates that a conclusion is incorrect', 'challenging a conclusion regarding an explicit statement by appealing to an applicable principle', 'forcing an opponent to elaborate on his facts in order to reveal a discrepancy', \"drawing an analogy to show the similarities between his opponent's argument and the economist's own conclusion\"]", "label": 1 }, { "id": "train_2272", "context": "Excavation of the house of a third-century Camarnian official revealed that he had served four magistrates-public officials who administer the law-over his thirty-year public career, in four provincial capital cities. However, given the Camarnian administrative system of that era, it is unclear whether he served them simultaneously, as a traveling administrator living for part of the year in each provincial capital, or else did so sequentially, leaving one magistrate after several years to join another.", "question": "Which of the following would, if found in the excavation, most likely help reveal the pattern of the official's administrative service?", "answers": "['Maps and documents describing each of the four provincial capitals', \"A cache of the official's documents related to work from early in his career\", 'A set of cups of a type made only in the city of the first magistrate whom the official is known to have served', 'Several pieces of furniture in the styles of two of the provincial capital cities']", "label": 1 }, { "id": "train_2273", "context": "A large amount of rainfall in April and May typically leads to an increase in the mosquito population and thus to an increased threat of encephalitis. People cannot change the weather. Thus people cannot decrease the threat of encephalitis.", "question": "The reasoning in the argument above is flawed in that the argument", "answers": "['takes for granted that a threat that is aggravated by certain factors could not occur in the absence of those factors', 'takes for granted that because one event precedes another the former must be the cause of the latter', 'ignores the possibility that a certain type of outcome is dependent on more than one factor', 'presumes, without providing justification, that a certain outcome would be desirable']", "label": 2 }, { "id": "train_2274", "context": "Science journalist: Europa, a moon of Jupiter, is covered with ice. Data recently transmitted by a spacecraft strongly suggest that there are oceans of liquid water deep under the ice. Life as we know it could evolve only in the presence of liquid water. Hence, it is likely that at least primitive life has evolved on Europa.", "question": "The science journalist's argument is most vulnerable to criticism on the grounds that it", "answers": "['fails to address adequately the possibility that there are conditions necessary for the evolution of life in addition to the presence of liquid water', 'takes for granted that no conditions on Europa other than the supposed presence of liquid water could have accounted for the data transmitted by the spacecraft', 'takes for granted that if a condition would be necessary for the evolution of life as we know it, then such life could not have evolved anywhere that this condition does not hold', 'takes for granted that life is likely to be present on Europa if, but only if, life evolved on Europa']", "label": 0 }, { "id": "train_2275", "context": "A nationwide poll of students, parents, and teachers showed that over 90 percent believe that an appropriate percentage of their school' s budget is being spent on student counseling programs. It seems, then, that any significant increase in a school' s budget should be spent on something other than student counseling programs.", "question": "Which one of the following describes a flaw in the reasoning of the argument above?", "answers": "['The argument fails to justify its presumption that what is true of a part of the budget is also true of the total budget.', 'The argument confuses a mere coincidence with a causal relationship.', 'The argument confuses the percentage of the budget spent on a program with the overall amount spent on that program.', 'The argument fails to consider that if more money is spent on a program, then more money cannot also be used for other purposes.']", "label": 2 }, { "id": "train_2276", "context": "Economist: ChesChem, a chemical manufacturer located in Chester, uses natural gas for its enormous energy needs. Currently, natural gas costs twice as much in Chester as it does in Tilsen. If the cost of natural gas in Chester becomes more than twice that in Tilsen, ChesChem will move its manufacturing operations to Tilsen. So if the cost of natural gas in Chester increases at all, ChesChem will move its manufacturing operations to Tilsen.", "question": "The economist's argument requires assuming that", "answers": "['the only benefit ChesChem would receive by moving its manufacturing operations to Tilsen is lower energy costs', 'ChesChem spends far more on natural gas than on any other expense', 'the price of natural gas in Tilsen will not increase', 'ChesChem would not be profitable if its energy costs increased']", "label": 2 }, { "id": "train_2277", "context": "The everyday behavior of whales is particularly difficult to study because introducing novel stimuli, such as divers or submarines, into the whales' environment causes whales to behave in unusual ways. Some biologists plan to train sea lions to carry video cameras on their backs and, on command, to swim along with whales. They argue that since whales are accustomed to there being sea lions nearby, using the sea lions to film the whales would allow biologists to study the everyday behavior of the whales.", "question": "Which one of the following is an assumption on which the biologists' reasoning depends?", "answers": "['The trained sea lions will not be aware that they are carrying video cameras on their backs.', 'Whales will often react aggressively in the presence of divers and submarines although aggressive behavior is unusual for whales.', 'The behavior of the sea lions under human command will be within the range of sea lion behavior to which the whales are accustomed.', 'Sea lions carrying video cameras will be able to film whales at a much closer range than divers can.']", "label": 2 }, { "id": "train_2278", "context": "When a lawmaker spoke out against a research grant awarded to a professor in a university' s psychology department as a foolish expenditure of public money, other professors in that department drafted a letter protesting the lawmaker' s interference in a field in which he was not trained. The chair of the psychology department, while privately endorsing the project, refused to sign the protest letter on the ground that she had previously written a letter applauding the same legislator when he publicized a senseless expenditure by the country' s military.", "question": "Which one of the following principles, if established, provides the strongest justification for the department chair's refusal, on the ground she gives, to sign the protest letter?", "answers": "[\"A person who has praised a lawmaker for playing a watchdog role in one case should not criticize the lawmaker for attempting to play a watchdog role in another case that involves the person's professional interests.\", 'Academic institutions have the same responsibility as military institutions have to spend public money wisely.', 'A person should not publicly criticize the actions of a lawmaker in different cases without giving careful consideration to the circumstances of each particular case.', 'The chair of an academic department has an obligation to ensure that public funds allocated to support projects within that department are spent wisely.']", "label": 0 }, { "id": "train_2279", "context": "The solution to any environmental problem that is not the result of government mismanagement can only lie in major changes in consumer habits. But major changes in consumer habits will occur only if such changes are economically enticing. As a result, few serious ecological problems will be solved unless the solutions are made economically enticing.", "question": "The conclusion drawn in the argument above follows logically if which one of the following is assumed?", "answers": "['Major changes in consumer habits can be made economically enticing.', 'Few serious ecological problems are the result of government mismanagement.', 'Few serious ecological problems can be solved by major changes in consumer habits.', 'Most environmental problems that are not the result of government mismanagement are major ecological problems.']", "label": 1 }, { "id": "train_2280", "context": "Agricultural societies cannot exist without staple crops. Several food plants, such as kola and okra, are known to have been domesticated in western Africa, but they are all supplemental, not staple, foods. All the recorded staple crops grown in western Africa were introduced from elsewhere, beginning, at some unknown date, with rice and yams. Ttherefore, discovering when rice and yams were introduced into western Africa would establish the earliest date at which agricultural societies could have arisen there.", "question": "Which of the following is an assumption on which the argument depends?", "answers": "['People in western Africa did not develop staple crops that they stopped cultivating once rice and yams were introduced.', 'Kola and okra were domesticated in western Africa before rice and yams were introduced there.', 'There are no plants native to western Africa that, if domesticated, could serve as staple food crops.', 'Kola and okra are better suited to growing conditions in western Africa than domesticated rice and yams are.']", "label": 0 }, { "id": "train_2281", "context": "To become an expert on a musical instrument, a person must practice. If people practice a musical instrument for three hours each day, they will eventually become experts on that instrument. Ttherefore, if a person is an expert on a musical instrument, that person must have practiced for at least three hours each day.", "question": "Which one of the following most accurately describes a flaw in the reasoning above?", "answers": "['The conclusion fails to take into account that practicing for less than three hours each day may be enough for some people to become experts.', 'The conclusion fails to take into account that people who practice for three hours every day might not yet have reached a degree of proficiency that everyone would consider expert.', 'The conclusion fails to take into account that few people have the spare time necessary to devote three hours daily to practice.', 'The conclusion fails to take into account that three consecutive hours of daily practice is not recommended by all music teachers.']", "label": 0 }, { "id": "train_2282", "context": "Economist: Currently, many countries rely primarily on taxing income to fund government expenditures. But taxing income does nothing to promote savings and investment. Taxing consumption, on the other hand, would encourage savings. The most important challenge facing these countries is improving their economies, and the only way to accomplish this is to increase their savings rates. Hence, __.", "question": "Which one of the following most logically completes the economist's argument?", "answers": "['the tax laws of many countries should be revised to focus on taxing consumption rather than income', 'most governments should stop taxing savings and investment', 'it is detrimental to the economic improvement of any country to continue to tax income', 'the economies of countries will rapidly improve if their governments adopt tax policies that encourage savings and investment']", "label": 0 }, { "id": "train_2283", "context": "The calm, shallow waters of coastal estuaries are easily polluted by nutrient-rich sewage. When estuary waters become overnutrified as a result, algae proliferate. The abundant algae, in turn, sometimes provide a rich food source for microorganisms that are toxic to fish, thereby killing most of the fish in the estuary.", "question": "Which one of the following can be properly inferred from the information above?", "answers": "['Overnutrifying estuary waters by sewage can result in the death of most of the fish in the estuary.', 'In estuary waters that contain abundant algae, microorganisms that are toxic to fish reproduce more quickly than other types of microorganisms.', 'Fish in an estuary that has been polluted by sewage are generally more likely to die from pollution than are fish in an estuary that has been polluted in some other way.', 'Nutrients and other components of sewage do not harm fish in coastal estuaries in any way other than through the resulting proliferation of toxic microorganisms.']", "label": 0 }, { "id": "train_2284", "context": "Finance minister: The World Bank' s \"Doing Business\" report ranks countries in terms of ease of doing business in them. In producing the rankings, the World Bank assesses how difficult it is for a hypothetical business to comply with regulations and pay taxes. Since the last \"Doing Business\" report came out, our government has dramatically simplified tax filing for small and even midsized businesses. So our \"Doing Business\" ranking will probably improve.", "question": "The answer to which one of the following questions would most help in evaluating the finance minister's argument?", "answers": "['Was the finance minister in office when the last \"Doing Business\" report was issued?', 'Has compliance with tax laws by small and midsized businesses increased since tax filing was simplified?', \"For small and midsized businesses in the finance minister's country, is tax preparation and filing more difficult than complying with other regulations?\", 'Is what the finance minister considers to be a midsized business smaller than the hypothetical business used to produce the \"Doing Business\" report?']", "label": 3 }, { "id": "train_2285", "context": "Essayist: Many social critics have claimed that modern society has become so heavily regulated by corporate and government bureaucracies that individuals lead lives over which they feel they have nearly no control. That such a perceived loss of freedom is a complete myth, however, should be obvious from the fact that people who live in modern society voluntarily become members of clubs, political movements, and other small groups that place additional restrictions on their decisions.", "question": "Which one of the following, if true, most strengthens the essayist's argument?", "answers": "['People living in societies dominated by large institutions tend to desire participation in smaller, more intimate groups of people.', 'Only people who think their lives are controlled by large, impersonal bureaucracies are willing to accept additional restrictions on their decisions.', 'Only people who do not feel highly restricted are willing to assume further restrictions on their decisions.', 'Many people who live in societies that are not heavily regulated by corporate and government bureaucracies voluntarily become members of groups that place restrictions on their decisions.']", "label": 2 }, { "id": "train_2286", "context": "In the United States, average fuel efficiency of newly manufactured domestic cars, although remaining worse than that of newly manufactured imported cars, substantially improved between 1983 and 1988. Average fuel efficiency of new domestic cars has not improved since, but the difference in average fuel efficiencies of new domestic cars and new imported cars has steadily decreased.", "question": "If the statements above are true, which of the following must also be true on the basis of them?", "answers": "['Average fuel efficiency of imported cars manufactured in 1983 was better than that of imported cars manufactured in 1988.', 'Average fuel efficiency of newly manufactured imported cars has steadily improved since 1983.', 'Average fuel efficiency of newly manufactured domestic cars has steadily worsened since 1988.', 'Average fuel efficiency of newly manufactured imported cars has steadily worsened since 1988.']", "label": 3 }, { "id": "train_2287", "context": "Premiums for automobile accident insurance are often higher for red cars than for cars of other colors. To justify these higher charges, insurance companies claim that, overall, a greater percentage of red cars are involved in accidents than are cars of any other color. If this claim is true, then lives could undoubtedly be saved by banning red cars from the roads altogether.", "question": "The reasoning in the argument is flawed because the argument", "answers": "['does not specify precisely what percentage of red cars are involved in accidents', 'fails to consider whether red cars cost the same to repair as cars of other colors', 'ignores the possibility that drivers who drive recklessly have a preference for red cars', 'accepts without question that insurance companies have the right to charge higher premiums for higher-risk clients']", "label": 2 }, { "id": "train_2288", "context": "Dr. Lopez: As anthropologists, we are committed to learning about cultures without intervening in the lives of the people in those cultures. For this reason, we should observe people passively by means of on-site video and sound recordings, rather than conducting interviews with those people. Dr. Tseung: Whereas your aim is admirable, your method is completely misguided. After all, the mere presence of video cameras and recording devices invariably constitutes an intervention into the lives of the people being filmed or recorded.", "question": "Which one of the following is a point at issue between Dr. Lopez and Dr. Tseung?", "answers": "['whether it is less troublesome for anthropologists to conduct interviews than it is for them to make on-site video and sound recordings', 'whether interviews conducted by anthropologists with people from a particular culture are an effective means of learning about that culture', 'whether it is possible to observe people passively by means of on-site video and sound recordings without intervening in the lives of those people', 'whether video and sound recording devices would be more intrusive in cultures in which those devices are relatively uncommon than they are in cultures in which they are relatively common']", "label": 2 }, { "id": "train_2289", "context": "After a hepadnavirus inserts itself into a chromosome of an animal, fragments of the virus are passed on to all of that animal' s descendants. A hepadnavirus fragment is present in a chromosome of the zebra finch and in precisely the same location in a corresponding chromosome of the dark-eyed junco. The fact that these two bird species diverged from each other about 25 million years ago ttherefore means that the hepadnavirus is at least 25 million years old.", "question": "Which one of the following, if true, most strengthens the argument?", "answers": "[\"When a virus inserts itself into an animal's chromosome, the insertion occurs at a random spot.\", \"The presence of a hepadnavirus in an animal species does not affect the likelihood of that species' survival.\", 'The chromosomes of the zebra finch and the dark-eyed junco contain fragments of no virus other than the hepadnavirus.', 'Viruses can affect the evolution of an organism and can thereby influence the likelihood of their diverging into two species.']", "label": 0 }, { "id": "train_2290", "context": "Political scientist: The concept of freedom is hopelessly vague. Any definition of freedom will either exclude some acts that intuitively qualify as free, or admit some acts that intuitively fall outside the concept. The notions of justice, fairness, and equality are equally indeterminate. This is why political organization should be disavowed as futile.", "question": "The reasoning in the argument is questionable because the argument", "answers": "['fails to show any specific link between the vagueness of concepts such as freedom and the rejection of political organization', 'is mounted by someone who has a vested interest in the rejection of political organization', 'ignores the fact that some people view freedom as indispensable', 'generalizes from an unrepresentative sample to every political idea']", "label": 0 }, { "id": "train_2291", "context": "One reason why European music has had such a strong influence throughout the world, and why it is a sophisticated achievement, is that over time the original function of the music -- whether ritual, dance, or worship -- gradually became an aspect of its style, not its defining force. Dance music could stand independent of dance, for example, and sacred music independent of religious worship, because each composition has so much internal coherence that the music ultimately depends on nothing but itself.", "question": "The claims made above are compatible with each of the following EXCEPT:", "answers": "['Music that is unintelligible when it is presented independently of its original function tends to be the most sophisticated music.', 'The original functions of many types of Chinese music are no longer their defining forces.', 'Some works of art lose their appeal when they are presented to serve a function other than their original one.', 'African music has had a more powerful impact on the world than European music has had.']", "label": 0 }, { "id": "train_2292", "context": "Today' s farmers plant only a handful of different strains of a given crop. Crops lack the diversity that they had only a few generations ago. Hence, a disease that strikes only a few strains of crops, and that would have had only minor impact on the food supply in the past, would devastate it today.", "question": "Which one of the following, if true, would most weaken the argument?", "answers": "['Affected crops can quickly be replaced from seed banks that store many strains of those crops.', \"Today's crops are much less vulnerable to damage from insects or encroachment by weeds than were crops of a few generations ago.\", 'Some of the less popular seed strains that were used in the past were more resistant to many diseases than are the strains popular today.', 'Humans today have more variety in their diets than in the past, but still rely heavily on cereal crops like rice and wheat.']", "label": 0 }, { "id": "train_2293", "context": "Farmer: My neighbor claims that my pesticides are spreading to her farm in runoff water, but she is wrong. I use only organic pesticides, and there is no evidence that they harm either people or domestic animals. Furthermore, I am careful to avoid spraying on my neighbor' s land.", "question": "Which one of the following most accurately describes a reasoning flaw in the farmer's argument?", "answers": "['It treats lack of evidence that organic pesticides harm people or domestic animals as proof that they cannot do so.', \"It fails to provide an alternative explanation for the presence of pesticides on the neighbor's land.\", \"It does not address the neighbor's claim that pesticides used by the farmer are spreading onto her land.\", 'It presumes, without providing justification, that being careful to avoid something usually results in its avoidance.']", "label": 2 }, { "id": "train_2294", "context": "Researchers have studied the cost-effectiveness of growing halophytes -- salt-tolerant plant species -- for animal forage. Halophytes require more water than conventional crops, but can be irrigated with seawater, and pumping seawater into farms near sea level is much cheaper than pumping freshwater from deep wells. Thus, seawater agriculture near sea level should be cost-effective in desert regions although its yields are smaller than traditional, freshwater agriculture.", "question": "Which one of the following, if true, most strengthens the argument above?", "answers": "['Costs other than the costs of irrigation are different for halophytes grown by means of seawater irrigation than for conventional crops.', 'Large research expenditures are needed to develop the strains of halophytes best suited for agricultural purposes.', 'A given volume of halophytes is significantly different in nutritional value for animal forage from the same volume of conventional forage crops.', 'Pumping water for irrigation is proportionally one of the largest costs involved in growing, harvesting, and distributing any forage crop for animals.']", "label": 3 }, { "id": "train_2295", "context": "Critic: As modern methods of communication and transportation have continued to improve, the pace of life today has become faster than ever before. This speed has created feelings of impermanence and instability, making us feel as if we never have enough time to achieve what we want -- or at least what we think we want.", "question": "The critic's statements most closely conform to which one of the following assessments?", "answers": "[\"Changes in people's feelings fuel the need for technological advancement.\", \"Changes in people's feelings about life can result from technological changes.\", 'The perception of impermanence in contemporary life makes it more difficult for people to know what they want.', 'The fast pace of modern life has made it difficult for people to achieve their goals.']", "label": 1 }, { "id": "train_2296", "context": "Biologists found that off the northeast coast of a certain country the P-plankton population has recently dropped 10 percent. Additionally, fish species X, Y, and Z are beginning to show extraordinarily high death rates in the region. Since these species of fish are known to sometimes eat P-plankton, biologists believe the two phenomena are connected, but the exact nature of the connection is unknown. No other species in the ecosystem appear to be affected.", "question": "Which one of the following, if true, most helps to explain the biologists' findings?", "answers": "['Global warming has changed the climatic conditions of the ocean all along the northeast coast of the country.', 'A powerful toxin in the water is killing off P-plankton by inhibiting their production of a chemical they use in reproduction.', 'Fish species X, Y, and Z are all experiencing widespread starvation within the affected region, and the loss of P-plankton is driving their death rates up even higher.', 'A new strain of bacteria is attacking P-plankton by destroying their cell walls and is attacking the respiratory systems of fish species X, Y, and Z.']", "label": 3 }, { "id": "train_2297", "context": "Columnist: Shortsighted motorists learn the hard way about the wisdom of preventive auto maintenance; such maintenance almost always pays off in the long run. Our usually shortsighted city council should be praised for using similar wisdom when they hired a long-term economic development adviser. In hiring this adviser, the council made an investment that is likely to have a big payoff in several years. Other cities in this region that have devoted resources to economic development planning have earned large returns on such an investment.", "question": "Which one of the following, if true, most weakens the columnist's argument?", "answers": "[\"The columnist's city has a much smaller population and economy than the other cities did when they began devoting resources to economic development planning.\", 'Qualified economic development advisers generally demand higher salaries than many city councils are willing to spend.', \"Cities that have earned large returns due to hiring economic development advisers did not earn any returns at all in the advisers' first few years of employment.\", 'Even some cars that receive regular preventive maintenance break down, requiring costly repairs.']", "label": 0 }, { "id": "train_2298", "context": "In Colorado subalpine meadows, nonnative dandelions co-occur with a native flower, the larkspur. Bumblebees visit both species, creating the potential for interactions between the two species with respect to pollination. In a recent study, researchers selected 16 plots containing both species; all dandelions were removed from eight plots; the remaining eight control plots were left undisturbed. The control plots yielded significantly more larkspur seeds than the dandelion-free plots, leading the researchers to conclude that the presence of dandelions facilitates pollination (and hence seed production) in the native species by attracting more pollinators to the mixed plots.", "question": "Which of the following, if true, most seriously undermines the researchers' reasoning?", "answers": "['Bumblebees preferentially visit dandelions over larkspurs in mixed plots.', 'Soil disturbances can result in fewer blooms, and hence lower seed production.', 'If left unchecked, nonnative species like dandelions quickly crowd out native species.', 'In mixed plots, pollinators can transfer pollen from one species to another to augment seed production.']", "label": 1 }, { "id": "train_2299", "context": "Historian: Political regimes that routinely censor various forms of expression on the grounds that they undermine public morality inevitably attempt to expand the categories of proscribed expression to include criticisms that these regimes perceive to threaten their power. Accordingly, many totalitarian regimes classify as blasphemous or pornographic those writings that would, if widely influential, reduce public passivity.", "question": "Which one of the following is an assumption on which the historian's reasoning depends?", "answers": "['Most writings that totalitarian regimes label blasphemous or pornographic would, if widely influential, reduce public passivity.', 'Not all political regimes that routinely censor forms of expression on the grounds that they erode public morality are totalitarian regimes.', 'A totalitarian regime can perceive loss of public passivity as a threat to its power.', 'Widespread public passivity is usually needed for a regime to retain political power.']", "label": 2 }, { "id": "train_2300", "context": "Advertisement: GreenBank gives all of its customers unlimited free automatic teller machine (ATM) use. TekBank charges 25 cents for each ATM transaction. So, clearly, it costs more to bank at TekBank than at GreenBank.", "question": "The reasoning in the advertisement's argument is misleading in that the argument", "answers": "['draws a conclusion about the overall cost of a service solely on the basis of a claim about the cost of one component of that service', 'bases a recommendation solely on economic factors without considering whether other factors are more important', 'concludes that a component of a service must have a property that the service as a whole possesses', 'concludes that a claim must be false because of the mere absence of evidence in its favor']", "label": 0 }, { "id": "train_2301", "context": "Mary Ann: Our country should, above all, be strong. Strength gains the respect of other countries and makes a country admirable. Inez: There are many examples in history of countries that were strong but used their strength to commit atrocities. We should judge a country by the morality of its actions, not by its strength. If the actions are morally good, the country is admirable.", "question": "Which one of the following is a presupposition that underlies Inez' argument?", "answers": "['Countries cannot be both strong and moral.', 'It is possible to assign moral weight to the actions of countries.', 'The citizens of any country believe that whatever their country does is good.', 'At least one country is admirable.']", "label": 1 }, { "id": "train_2302", "context": "The governmental archives of a country recently freed from dictatorship contain no material incriminating the country' s most notorious dictator in certain crimes that political observers believed him to have instigated. In fact, the archives contain documents implicating others in these crimes. Ttherefore, even though this dictator was probably guilty of many other crimes, the political observers' belief that he was also guilty of these crimes is almost certainly wrong.", "question": "The argument is most vulnerable to criticism on the grounds that it", "answers": "['presumes, without providing justification, that rulers should not be held responsible for unjust actions carried out by their subordinates', 'fails to consider the possibility that the political observers might have wished to make the dictator appear worse than he really was', \"fails to justify its presumption about the reliability of the archives as a source of information regarding the dictator's criminal involvement\", 'fails to take into account that there might be other documents exonerating those persons that the archives did implicate']", "label": 2 }, { "id": "train_2303", "context": "Exposure to certain chemicals commonly used in elementary schools as cleaners or pesticides causes allergic reactions in some children. Elementary school nurses in Renston report that the proportion of schoolchildren sent to them for treatment of allergic reactions to those chemicals has increased significantly over the past ten years. Ttherefore, either Renston' s schoolchildren have been exposed to greater quantities of the chemicals, or they are more sensitive to them than schoolchildren were ten years ago.", "question": "Which of the following is an assumption on which the argument depends?", "answers": "[\"Children attending elementary school do not make up a larger proportion of Renston's population now than they did ten years ago.\", 'The chemicals are not commonly used as cleaners or pesticides in houses and apartment buildings in Renston.', 'Children who are allergic to the chemicals are no more likely than other children to have allergies to other substances.', 'Children who have allergic reactions to the chemicals are not more likely to be sent to a school nurse now than they were ten years ago.']", "label": 3 }, { "id": "train_2304", "context": "It was misleading for James to tell the Core Curriculum Committee that the chair of the Anthropology Department had endorsed his proposal. The chair of the Anthropology Department had told James that his proposal had her endorsement, but only if the draft proposal she saw included all the recommendations James would ultimately make to the Core Curriculum Committee.", "question": "The argument relies on which one of the following assumptions?", "answers": "[\"The draft proposal that the chair of the Anthropology Department had seen did not include all of the recommendations in James's proposal to the Core Curriculum Committee.\", \"If the chair of the Anthropology Department did not endorse James's proposed recommendations, the Core Curriculum Committee would be unlikely to implement them.\", 'James thought that the chair of the Anthropology Department would have endorsed all of the recommendations that he proposed to the Core Curriculum Committee.', 'The chair of the Anthropology Department would have been opposed to any recommendations James proposed to the Core Curriculum Committee other than those she had seen.']", "label": 0 }, { "id": "train_2305", "context": "Plant scientists have been able to genetically engineer vegetable seeds to produce crops that are highly resistant to insect damage. Although these seeds currently cost more than conventional seeds, their cost is likely to decline. Moreover, farmers planting them can use far less pesticide, and most consumers prefer vegetables grown with less pesticide, ttherefore, for crops for which these seeds can be developed, their use is likely to become the norm.", "question": "which of the following would be most useful to know in evaluating the argument above?", "answers": "['Whether seeds genetically engineered to produce insect-resistant crops generate significantly lower per acre crop yields than do currently used seeds.', 'Whether plants grown from the new genetically engineered seeds can be kept completely free of insect damage.', 'Whether farmers typically use agricultural pesticides in larger amounts than is necessary to prevent crop damage.', 'Whether plant scientists have developed insect-resistant seeds for every crop that is currently grown commercially']", "label": 0 }, { "id": "train_2306", "context": "Chronic fatigue syndrome, a condition that afflicts thousands of people, is invariably associated with lower-than-normal concentrations of magnesium in the blood. Further, malabsorption of magnesium from the digestive tract to the blood is also often associated with some types of fatigue. These facts in themselves demonstrate that treatments that raise the concentration of magnesium in the blood would provide an effective cure for the fatigue involved in the syndrome.", "question": "The argument is most vulnerable to which one of the following criticisms?", "answers": "['It fails to establish that lower-than-normal concentrations of magnesium in the blood are invariably due to malabsorption of magnesium.', 'It offers no evidence that fatigue itself does not induce lowered concentrations of magnesium in the blood.', 'It neglects to state the exact concentration of magnesium in the blood which is considered the normal concentration.', 'It ignores the possibility that, even in people who are not afflicted with chronic fatigue syndrome, concentration of magnesium in the blood fluctuates.']", "label": 1 }, { "id": "train_2307", "context": "Interviewer: You have shown that biofeedback, dietary changes, and adoption of proper sleep habits all succeed in curing insomnia. You go so far as to claim that, with rigorous adherence to the proper treatment, any case of insomnia is curable. Yet in fact some patients suffering from insomnia do not respond to treatment. Therapist: If patients do not respond to treatment, this just shows that they are not rigorous in adhering to their treatment.", "question": "The therapist's reply to the interviewer is most vulnerable to which one of the following criticisms?", "answers": "['It does not provide statistical evidence to back up its claim.', 'It precludes the possibility of disconfirming evidence.', 'It overlooks the possibility that some cases of insomnia might improve without any treatment.', 'It depends on the ambiguous use of the term \"treatment. \"']", "label": 1 }, { "id": "train_2308", "context": "Because it was long thought that few people would watch lengthy televised political messages, most televised political advertisements, like commercial advertisements, took the form of short messages. Last year, however, one candidate produced a half-hour-long advertisement. During the half hour the advertisement was aired, a substantial portion of the viewing public tuned into the advertisement. Clearly, then, many more people are interested in watching lengthy televised political messages than was previously thought.", "question": "Which of the following is an assumption on which the argument depends?", "answers": [ "Most of the viewers who tuned in to the candidate's half-hour-long advertisement last year did not change channels after the first few minutes.", "Political advertisements have become increasingly influential in determining voters' decisions at the polls.", "The candidate's ratings improved significantly as a result of the half-hour-long political advertisement.", "Many people would appreciate the opportunity to become better acquainted with political candidates' views on current political issues." ], "label": 0 }, { "id": "train_2309", "context": "Drug manufacturer: Television audiences are sure to realize that the \"physician\" recommending our brand of cough syrup in our advertisement is actually an actor playing a role. Hence they will not place undue trust in the advice given by this actor. Ttherefore, networks should relax their guidelines to permit our company to broadcast this advertisement. Television executive: If the audience can tell that the actor is not a physician, then your advertisement need not have a physician figure recommending your product.", "question": "Which of the following is an argumentative strategy used by the television executive in response to the drug manufacturer?", "answers": "['Invoking subjective opinions concerning audience reaction to television advertisements as if those opinions constituted objective evidence.', \"Indicating that the reason the drug manufacturer offers for relaxing the guidelines conflicts with the manufacturer's presumed motive for presenting the image of a physician in the advertisement.\", \"Asserting that the drug manufacturer's expressed desire to broadcast the advertisement is motivated by self-interest rather than by genuine interest in the good of the audience\", \"Pointing out that the goals of the drug manufacturer's company differ from those of television networks.\"]", "label": 1 }, { "id": "train_2310", "context": "In order to combat Carville's rampant homeless problem, Mayor Bloomfield recently proposed a ban on sleeping outdoors in the city's many parks. He claims that such a measure will force the homeless to either leave Carville or to find means other than sleeping in public parks.", "question": "Which of the following, if true, suggests that Mayor Bloomfield's plan will be successful?", "answers": "['Adjacent cities have even tougher measures on the homeless sleeping outdoors.', \"Many homeless tend to congregate underneath Carville's numerous overpasses.\", \"Until the ban, the city's many homeless shelters were at less than fifty percent occupancy.\", \"The percent of Carville's population that has been homeless has been slowly decreasing in the last five years.\"]", "label": 2 }, { "id": "train_2311", "context": "More than a year ago, the city announced that police would crack down on illegally parked cars and that resources would be diverted from writing speeding tickets to ticketing illegally parked cars. But no crackdown has taken place. The police chief claims that resources have had to be diverted from writing speeding tickets to combating the city' s staggering drug problem. Yet the police are still writing as many speeding tickets as ever. Ttherefore, the excuse about resources being tied up in fighting drug-related crime simply is not true.", "question": "The conclusion in the passage depends on the assumption that", "answers": "['writing speeding tickets should be as important a priority for the city as combating drug-related crime', 'drug-related crime is not as serious a problem for the city as the police chief claims it is', 'the police cannot continue writing as many speeding tickets as ever while diverting resources to combating drug-related crime', \"every member of the police force is qualified to work on combating the city's drug problem\"]", "label": 2 }, { "id": "train_2312", "context": "Outbreaks of Rift Valley fever occur irregularly in East Africa, several years apart. When outbreaks do occur, they kill thousands of cattle. A livestock vaccine against the disease exists but is rarely used. It is too expensive for farmers to use routinely, and since it is not effective until a month after vaccination, administering it after an outbreak begins helps very little. Nevertheless, experts predict that use of the vaccine will increase significantly within the next few years.", "question": "Which of the following, if true, provides the strongest justification for the experts' prediction?", "answers": "['When an outbreak of Rift Valley fever occurs, unaffected countries often refuse to import livestock from the countries affected by the outbreak.', 'Recently published research has shown that certain identifiable climatic conditions are almost invariably followed, within two to five months, by an outbreak of Rift Valley fever.', 'Rift Valley fever is spread by mosquitoes, but each outbreak is so widespread that it is impractical to control it by using insecticides.', 'It would take less than a month for producers of the vaccine to adjust their production operations to cope with a large increase in demand.']", "label": 1 }, { "id": "train_2313", "context": "Mayor of Plainsville: In order to help the economy of Plainsville, I am using some of our tax revenues to help bring a major highway through the town and thereby attract new business to Plainsville. Citizens' group: You must have interests other than our economy in mind. If you were really interested in helping our economy, you would instead allocate the revenues to building a new business park, since it would bring in twice the business that your highway would.", "question": "The argument by the citizens' group relies on which one of the following assumptions?", "answers": "[\"Plainsville's economy will not be helped unless a new business park of the sort envisioned by the citizens' group is built.\", 'The mayor is required to get approval for all tax revenue allocation plans from the city council.', 'Plainsville presently has no major highways running through it.', 'The mayor accepts that a new business park would bring in more new business than would the new highway.']", "label": 3 }, { "id": "train_2314", "context": "Life expectancy\" is the average age at death of the entire live-born population. In the middle of the nineteenth century, life expectancy in North America was 40 years, whereas now it is nearly 80 years. Thus, in those days, people must have been considered old at an age that we now consider the prime of life.", "question": "Which of the following, if true, undermines the argument above?", "answers": "['In the middle of the nineteenth century, the population of North America was significantly smaller than it is today.', 'The proportion of people who die in their seventies is significantly smaller today than is the proportion of people who die in their eighties.', 'Most of the gains in life expectancy in the last 150 years have come from reductions in the number of infants who die in their first year of life.', 'Many of the people who live to an advanced age today do so only because of medical technology that was unknown in the nineteenth century.']", "label": 2 }, { "id": "train_2315", "context": "Copyright was originally the grant of a temporary government-supported monopoly on copying a work. Its sole purpose was to encourage the circulation of ideas by giving authors the opportunity to derive a reasonable financial reward from their works. However, copyright sometimes goes beyond its original purpose since sometimes __.", "question": "The conclusion of the argument is most strongly supported if which one of the following completes the passage?", "answers": "['there is no practical way to enforce copyrights', \"copyrights hold for many years after an author's death\", 'authors are willing to circulate their works even without any financial reward', 'publication of copyrighted works is not the only way to circulate ideas']", "label": 1 }, { "id": "train_2316", "context": "Every time people get what they want they feel pleasure. Pleasure is a natural result of getting what one wants. We can conclude that no one fundamentally desires anything except pleasure.", "question": "Which one of the following uses questionable reasoning most similar to that used in the argument above?", "answers": "['I have never been skiing, but just thinking about it terrifies me, so I guess I must not want to learn how.', 'Every time I eat pizza I get a stomachache, so I suppose the reason I eat pizza in the first place is so that I can have a stomachache.', 'I never enjoy a soccer game without eating hot dogs, so I guess I would not enjoy going to a basketball game if I could not eat hot dogs at the game.', 'I sure am enjoying the party even though I was sure I would not, so I guess I wanted to come after all.']", "label": 1 }, { "id": "train_2317", "context": "The theory of military deterrence was based on a simple psychological truth, that fear of retaliation makes a would-be aggressor nation hesitate before attacking and is often sufficient to deter it altogether from attacking. Clearly, then, to maintain military deterrence, a nation would have to be believed to have retaliatory power so great that a potential aggressor nation would have reason to think that it could not defend itself against such retaliation.", "question": "If the statements above are true, which one of the following can be properly inferred?", "answers": "['It is in the interests of a nation that seeks deterrence and has unsurpassed military power to let potential aggressors against it become aware of its power of retaliatory attack.', 'A nation will not attack another nation if it believes that its own retaliatory power surpasses that of the other nation.', 'Maintaining maximum deterrence from aggression by other nations requires that a nation maintain a retaliatory force greater than that of any other nation.', \"One nation's failing to attack another establishes that the nation that fails to attack believes that it could not withstand a retaliatory attack from the other nation.\"]", "label": 0 }, { "id": "train_2318", "context": "Religious Scholar: Truly moral people are naturally inclined to benefit their fellow man. People who act morally solely as a means to garner some divine benefit or avoid cosmic retribution are not, in fact, moral. They are blind sheep. To be moral requires true selflessness, rather than following any specific religious text.", "question": "Which one of the following individual's actions best adheres to the religious scholar's moral code?", "answers": "['Jorge caught his wife in the midst of an extra-marital affair with a co-worker. Naturally, he was extremely angry. Jorge wanted to physically harm his unfaithful wife and her treacherous coworker. But he remembered the words of the Sixth Commandment -- thou shall not kill -- and walked out of the room before he lost control.', \"Every weekend, Arianna volunteers at a soup kitchen run by her church. She also donates a substantial part of her paycheck to children's cancer charities.\", \"Carlos visits his grandmother every weekend. She's an old bitter woman, and the rest of her family refuses to visit her except on special occasions. However, Carlos started practicing a new religion, which demands its followers respect the elderly.\", \"Elizabeth baby sat her sister's four kids over the weekend. She had recently been studying Eastern religious practices and learned of the karmic principle, which states that an individual will be rewarded for his or her good deeds. Elizabeth hoped babysitting would help her chances at getting an overdue promotion at work.\"]", "label": 1 }, { "id": "train_2319", "context": "Oceanographer: To substantially reduce the amount of carbon dioxide in Earth' s atmosphere, carbon dioxide should be captured and pumped deep into the oceans, where it would dissolve. The cool, dense water in ocean depths takes centuries to mix with the warmer water near the surface, so any carbon dioxide pumped deep into oceans would be trapped there for centuries.", "question": "Which one of the following is an assumption that the oceanographer's argument requires?", "answers": "['Carbon dioxide will dissolve much more thoroughly if it is pumped into cold water than it will if it is pumped into warmer water.', \"Carbon dioxide dissolved in cool, dense water in ocean depths will not escape back into Earth's atmosphere a long time before the water in which that carbon dioxide is dissolved mixes with warmer water near the surface.\", 'Carbon dioxide should be pumped into ocean depths to reduce the amount of carbon dioxide in the atmosphere only if the carbon dioxide pumped into ocean depths would be trapped there for hundreds of years.', 'It is the density of the water in the ocean depths that plays the main role in the trapping of the carbon dioxide.']", "label": 1 }, { "id": "train_2320", "context": "Political theorist: Newly enacted laws need a period of immunity during which they can be repealed only if circumstances are dire. This is because the short-term consequences of any statutory change are likely to be painful, since people are not accustomed to it, while its longterm benefits are initially obscure, because people require time to learn how to take advantage of it.", "question": "Which one of the following principles, if valid, most helps to justify the political theorist's argument?", "answers": "[\"The short-term consequences of a law's repeal should be considered more carefully than the short-term consequences of its passage.\", 'Whether a law should be retained is independent of what the voters think its consequences will be.', 'Whether a law should be retained depends primarily on the long-term consequences of its enactment.', 'The long-term consequences of the enactment of a law should be more beneficial than its short-term consequences.']", "label": 2 }, { "id": "train_2321", "context": "The cities of Oldtown and Spoonville are the same in area and size of population. Since certain health problems that are caused by crowded living conditions are widespread in Oldtown, such problems must be as widespread in Spoonville.", "question": "The reasoning in the argument is most vulnerable to criticism on the grounds that the argument", "answers": "['fails to take into account that having identical overall population density is consistent with great disparity in living conditions', 'fails to distinguish between the size of the total population of a city and the size of the geographic region covered by that city', 'presupposes without warrant that the health problems that are widespread in any particular city cannot be caused by the living conditions in that city', 'fails to indicate whether average life expectancy is lowered as a result of living in crowded conditions']", "label": 0 }, { "id": "train_2322", "context": "The number of tornadoes recorded annually in North America has more than tripled since 1953. Yet meteorologists insist that the climatic factors affecting the creation of tornadoes are unchanged.", "question": "Which one of the following, if true, most helps to resolve the apparent discrepancy described above?", "answers": "['Many more citizens are helping authorities detect tornadoes now than in 1953.', 'The number of tornadoes recorded annually has increased only slightly in the last five years.', 'The factors affecting the creation of tornadoes were not well known to meteorologists before 1953.', 'The amount of property damage done by tornadoes has grown substantially since 1953.']", "label": 0 }, { "id": "train_2323", "context": "John: It was wrong of you to blame me for that traffic accident. You know full well that the accident was due to my poor vision, and I certainly cannot be held responsible for the fact that my vision has deteriorated. Michiko: But I can hold you responsible for your hazardous driving, because you know how poor your vision is. People are responsible for the consequences of actions that they voluntarily undertake, if they know that those actions risk such consequences.", "question": "The principle that Michiko invokes, if established, would justify which one of the following judgments?", "answers": "['Colleen was responsible for missing her flight home from Paris, because she decided to take one more trip to the Eiffel Tower even though she knew she might not have sufficient time to get to the airport if she did so.', 'Colleen was not responsible for losing her job, because, knowing that her position was in danger of being eliminated, she did everything possible to preserve it.', \"Colleen was responsible for her cat's being frightened, because, even though it was her brother who allowed the door to slam shut, she knew that cats are often frightened by loud noises.\", 'Colleen was responsible for having offended her brother when she reported to him an offensive comment made about his colleague, although she did not know her brother would mistakenly understand the comment to be about himself.']", "label": 0 }, { "id": "train_2324", "context": "Critic: It is common to argue that there is a distinction between \"literary\" and \"genre\" fiction. The first should be interpreted, so this argument goes, while the second is merely a source of easy pleasure. But this is a specious distinction -- not because every work should be interpreted, but because no work should be. When we evaluate a work principally for its themes and ideas, we cut ourselves off from the work' s emotional impact.", "question": "Which one of the following most accurately describes the role played in the critic's argument by the claim that when we evaluate a work principally for its themes and ideas, we cut ourselves off from the work's emotional impact?", "answers": "['It states the conclusion.', 'It is offered as support for the conclusion.', 'It attempts to explain the nature of the distinction that the critic considers.', \"It attempts to anticipate an objection to the critic's conclusion.\"]", "label": 1 }, { "id": "train_2325", "context": "Columnist: It has been noted that attending a live musical performance is a richer experience than is listening to recorded music. Some say that this is merely because we do not see the performers when we listen to recorded music. However, there must be some other reason, for there is relatively little difference between listening to someone read a story over the radio and listening to someone in the same room read a story.", "question": "Which one of the following most accurately expresses the role played in the argument by the observation that attending a live musical performance is a richer experience than is listening to recorded music?", "answers": "[\"It is what the columnist's argument purports to show.\", \"It is what the columnist's argument purports to refute.\", 'It is what the position that the columnist tries to undermine is purported to explain.', \"It is what the columnist's argument purports to explain.\"]", "label": 2 }, { "id": "train_2326", "context": "In order to expand its mailing lists for e-mail advertising, the Outdoor Sports Company has been offering its customers financial incentives if they provide the e-mail addresses of their friends. However, offering such incentives is an unethical business practice, because it encourages people to exploit their personal relationships for profit, which risks damaging the integrity of those relationships.", "question": "Which one of the following principles, if valid, most helps to justify the reasoning in the argument?", "answers": "['It is unethical for people to exploit their personal relationships for profit if in doing so they risk damaging the integrity of those relationships.', \"It is an unethical business practice for a company to deliberately damage the integrity of its customers' personal relationships in any way.\", 'If it would be unethical to use information that was gathered in a particular way, then it is unethical to gather that information in the first place.', 'It is unethical to encourage people to engage in behavior that could damage the integrity of their personal relationships.']", "label": 3 }, { "id": "train_2327", "context": "Psychologist: We asked 100 entrepreneurs and 100 business managers to answer various questions and rate how confident they were that their responses were correct. While members of each group were overconfident, in general the entrepreneurs were much more so than the business managers. This indicates that people who are especially overconfident are more likely to attempt to start a business in spite of the enormous odds against success than people who are less confident.", "question": "Which one of the following, if true, lends the most support to the psychologist's conclusion?", "answers": "['Another survey showed that degree of confidence was highly correlated with success in business.', 'At least some of the entrepreneurs surveyed had accurately determined before attempting to start their businesses what the odds were against their attempts being successful.', \"How confident each person surveyed was that his or her answers to the questions asked were correct corresponded closely to that person's confidence in his or her business acumen.\", 'The business managers who were most overconfident were found to have attempted to start businesses in the past.']", "label": 3 }, { "id": "train_2328", "context": "Tanya would refrain from littering if everyone else refrained from littering. None of her friends litter, and ttherefore she does not litter either.", "question": "Which one of the following uses flawed reasoning most similar to the flawed reasoning in the argument above?", "answers": "['All residents of the same neighborhood have some goals in common. One group of neighborhood residents wants improvements made to a local park, so some other residents of that neighborhood must share this goal.', \"If all of a restaurant's customers like its food, it must be an exceptional restaurant. Everyone whom Sherryl consulted liked the food at Chez Louis, so it must be an exceptional restaurant.\", \"If a talented artist is willing to starve for her career, then her friends should take her choice of profession seriously. Donna's friends take her choice of profession seriously, and she is willing to starve for her career, so she must be a talented artist.\", 'Herbert will stop selling office supplies in his store if none of his regular customers complains. Some of his regular customers never knew that Herbert sold office supplies, so those customers will not complain.']", "label": 1 }, { "id": "train_2329", "context": "Although aspirin has been proven to eliminate moderate fever associated with some illnesses, many doctors no longer routinely recommend its use for this purpose. A moderate fever stimulates the activity of the body's disease-fighting white blood cells and also inhibits the growth of many strains of disease-causing bacteria.", "question": "If the statements above are true, which of the following conclusions is most strongly supported by them?", "answers": "[\"The more white blood cells a patient's body produces, the less severe the patient's illness will be.\", 'Aspirin, an effective painkiller, alleviates the pain and discomfort of many illnesses.', 'Aspirin inhibits the growth of white blood cells, which are necessary for fighting some illnesses.', \"Aspirin can prolong a patient's illness by eliminating moderate fever helpful in fighting some diseases.\"]", "label": 3 }, { "id": "train_2330", "context": "Certain minor peculiarities of language are used unconsciously by poets. If such peculiarities appear in the works of more than one poet, they are likely to reflect the language in common use during the poets' time. However, if they appear in the work of only one poet, they are likely to be personal idiosyncrasies. As such, they can provide a kind of \"fingerprint\" that allows scholars, by comparing a poem of previously unknown authorship to the work of a particular known poet, to identify the poem as the work of that poet.", "question": "For which one of the following reasons can the test described above never provide conclusive proof of the authorship of any poem?", "answers": "[\"A poet's use of some peculiarities of language might have been unconscious in some poems and conscious in other poems, and the two uses would be indistinguishable to scholars at a later date.\", 'The labor of analyzing peculiarities of language both in the work of a known poet and in a poem of unknown authorship would not be undertaken unless other evidence already suggested that the poem of unknown authorship was written by the known poet.', 'A peculiarity of language that might be used as an identifying mark is likely to be widely scattered in the work of a poet, so that a single poem not known to have been written by that poet might not include that peculiarity.', 'A peculiarity of language in a poem of unknown authorship could be evidence either that the poem was written by the one author known to use that peculiarity or that the peculiarity was not unique to that author.']", "label": 3 }, { "id": "train_2331", "context": "The percentage of households with an annual income of more than $40, 000 is higher in Merton County than in any other county. However, the percentage of households with an annual income of $60, 000 or more is higher in Sommer County.", "question": "If the statements above are true, which of the following must also be true?", "answers": "['Average annual household income is higher in Sommer County than in Merton County.', 'The percentage of households with an annual income of $80, 000 is higher in Sommer County than in Merton County.', 'The number of households with an annual income of more than $40, 000 is greater in Merton County than in Sommer County.', 'Some households in Merton County have an annual income between $40, 000 and $60, 000.']", "label": 3 }, { "id": "train_2332", "context": "Using clean-coal technologies to \"repower\" existing factories promises ultimately a substantial reduction of polluting emissions, and will affect the full range of pollutants implicated in acid rain. The strategy of using these technologies could cut sulfur dioxide emissions by more than 80 percent and nitrogen oxide emissions by more than 50 percent. The emission of a smaller quantity of nitrogen pollutants would in turn reduce the formation of noxious ozone in the troposphere.", "question": "Which one of the following statements is an inference that can be drawn from the information given in the passage?", "answers": "['Noxious ozone is formed in factories by chemical reactions involving sulfur dioxide.', 'Twenty percent of the present level of sulfur dioxide emissions in the atmosphere is not considered a harmful level.', 'The choice of technologies in factories could reduce the formation of noxious ozone in the troposphere.', 'Sulfur dioxide emissions are the most dangerous pollutants implicated in acid rain.']", "label": 2 }, { "id": "train_2333", "context": "Mayor: Migrating shorebirds stop at our beach just to feed on horseshoe-crab eggs, a phenomenon that attracts tourists. To bring more tourists, the town council plans to undertake a beach reclamation project to double the area available to crabs for nesting. Birdwatcher: Without a high density of crabs on a beach, migrating shorebirds will go hungry because shorebirds only eat eggs that a crab happens to uncover when it is digging its own nest.", "question": "Which of the following, if true, would provide the mayor with the strongest counter to the birdwatcher's objection?", "answers": "['The additional land made available by the reclamation project will give migrating shorebirds more space.', 'On average, tourists who come to the town in order to watch birds spend more money there than tourists who come for other purposes.', 'Horseshoe crabs are so prolific that given favorable circumstances their numbers increase rapidly.', 'Some of the migrating shorebirds make only one stop during their migration form South America to Canada.']", "label": 2 }, { "id": "train_2334", "context": "When people experience throbbing in their teeth or gums, they have serious dental problems, and if a dental problem is serious, it will be a problem either of tooth decay or of gum disease. Ttherefore, since throbbing in the teeth or gums is a sign of serious dental problems, and neither Sabina' s teeth nor her gums are throbbing, Sabina can be suffering from neither tooth decay nor gum disease.", "question": "Which one of the following contains an error of reasoning most similar to that made in the argument above?", "answers": "[\"Someone who is neither an ophthalmologist nor an optometrist lacks specialized training for diagnosing defects of the eye. Ttherefore, Kim must have been trained in ophthalmology or optometry, given that she accurately diagnosed John's eye defect.\", 'A legally practicing psychiatrist must have both a medical degree and psychiatric training. Thus, since Emmett has not undergone psychiatric training, if he is practicing as a psychiatrist, he is not doing so legally.', 'People who drink a lot of coffee are said to have jittery nerves. Ttherefore, medical students who drink a lot of coffee should not become neonatologists or surgeons since neither neonatology nor surgery should be practiced by people with jittery nerves.', 'If a person is interested in either physics or chemistry, then that person would be wise to consider a career in medicine. Yolanda, however, is interested in neither physics nor chemistry, so it would not be wise for her to consider a career in medicine.']", "label": 3 }, { "id": "train_2335", "context": "Nutritionist: Many people claim that simple carbohydrates are a reasonable caloric replacement for the fatty foods forbidden to those on law-fat diets. This is now in doubt. New studies show that, for many people, a high intake of simple carbohydrates stimulates an overproduction of insulin, a hormone that is involved in processing sugars and starches to create energy when the body requires energy, or, when energy is not required, to store the resulting by-products as fat.", "question": "Which one of the following is most strongly supported by the nutritionist's statements?", "answers": "['People who produce enough insulin to process their intake of simple carbohydrates should not feel compelled to adopt low-fat diets.', 'People on low-fat diets should avoid consumption of simple carbohydrates if they wish to maintain the energy that their bodies require.', 'People who consume simple carbohydrates should limit their intake of foods high in fat.', 'People who wish to avoid gaining body fat should limit their intake of foods high in simple carbohydrates.']", "label": 3 }, { "id": "train_2336", "context": "People ought to take into account a discipline' s blemished origins when assessing the scientific value of that discipline. Take, for example, chemistry. It must be considered that many of its landmark results were obtained by alchemists -- a group whose superstitions and appeals to magic dominated the early development of chemical theory.", "question": "The reasoning above is most susceptible to criticism because the author", "answers": "[\"fails to consider how chemistry's current theories and practices differ from those of the alchemists mentioned\", 'uses the word \"discipline\"in two different senses', 'uses an example to contradict the principle under consideration', 'fails to establish that disciplines with unblemished origins are scientifically valuable']", "label": 0 }, { "id": "train_2337", "context": "When deciding where to locate or relocate, businesses look for an educated work force, a high level of services, a low business-tax rate, and close proximity to markets and raw materials. However, although each of these considerations has approximately equal importance, the lack of proximity either to markets or to raw materials often causes municipalities to lose prospective businesses, whereas having a higher-than-average business-tax rate rarely has this effect.", "question": "Which one of the following, if true, most helps to resolve the apparent discrepancy in the statements above?", "answers": "['Businesses have sometimes tried to obtain tax reductions from municipalities by suggesting that without such a reduction the business might be forced to relocate elsewhere.', 'Taxes paid by businesses constitute only a part of the tax revenue collected by most municipalities.', 'Businesses sometimes leave a municipality after that municipality has raised its taxes on businesses.', 'In general, the higher the rate at which municipalities tax businesses, the more those municipalities spend on education and on providing services to businesses.']", "label": 3 }, { "id": "train_2338", "context": "If a President is elected, then he or she won the nomination of a major party and received at least 270 Electoral College votes, even if he or she did not win the popular vote.", "question": "Which one of the following must be true?", "answers": "['No President who won the nomination of a major party received 270 Electoral College votes.', 'Some Presidents have received less than 270 Electoral College votes.', 'No President has received less than 270 Electoral College votes.', 'Some Presidents have won the popular vote.']", "label": 2 }, { "id": "train_2339", "context": "Most children find it very difficult to explain exactly what the words they use mean when those words do not refer to things that can be seen or touched. Yet, since children are able to use these words to convey the feelings and emotions they are obviously experiencing, understanding what a word means clearly does not depend on being able to explain it.", "question": "Which one of the following principles, if accepted, would provide the most justification for the conclusion?", "answers": "['Words can be explained satisfactorily only when they refer to things that can be seen or touched.', 'Anyone who can provide an exact explanation of a word has a clear understanding of what that word means.', 'When someone appropriately uses a word to convey something that he or she is experiencing, that person understands what that word means.', 'The fact that a task is very difficult for most people does not mean that no one can do it.']", "label": 2 }, { "id": "train_2340", "context": "Mike: Tom did not tell me that I could use his computer, but it would not be wrong for me to use it anyway. Last week Tom used Mary' s bicycle even though she had not told him he could use it.", "question": "Which one of the following principles, if valid, would most help to justify Mike's reasoning?", "answers": "['Using the possessions of others without their permission is not always theft.', 'It is permissible to treat people in a way that is similar to the way in which they have treated others.', 'If people have used your property without your permission, it is not wrong for you to use their property without their permission.', 'Generally one should tell the truth, but there are cases in which it is permissible not to.']", "label": 1 }, { "id": "train_2341", "context": "The capture of a wild animal is justified only as a last resort to save that animal' s life. But many wild animals are captured not because their lives are in any danger but so that they can be bred in captivity. Hence, many animals that have been captured should not have been captured.", "question": "Which one of the following arguments is most similar in its pattern of reasoning to the argument above?", "answers": "[\"Parents who never punish a child are not justified in complaining if the child regularly behaves in ways that disturb them. But many parents who prefer not to punish their children complain regularly about their children's behavior. Hence, many parents who complain about their children have no right to complain.\", 'Society has no right to punish children for deeds that would be crimes if the children were adults. But society does have the right to protect itself from children who are known threats. Hence, confinement of such children does not constitute punishment.', \"Punishing a young child is justified only if it is done out of concern for the child's future welfare. But many young children are punished not in order to promote their welfare but to minimize sibling rivalry. Hence, many children who are punished should not have been punished.\", \"A teacher is entitled to punish a child only if the child's parents have explicitly given the teacher the permission to do so. But many parents never give their child's teacher the right to punish their child. Hence, many teachers should not punish their pupils.\"]", "label": 2 }, { "id": "train_2342", "context": "Historian: The central claim of the \"end-of-history\" theory is that history has reached its final stage of development. According to its adherents, democratic ideals have triumphed over their rivals, and history is effectively at an ideological end. But, this view fails to consider that it is impossible to stand outside all of history to judge whether history is really at an end.", "question": "Which one of the following can be most reasonably inferred from the historian's statement?", "answers": "['Ideological developments are the essential elements of history.', 'We can never know whether the end-of-history theory is true.', 'If we were at the end of history, we would automatically know whether the end-of-history theory is true.', 'Advocates of the end-of-history theory have too ideological an understanding of history.']", "label": 1 }, { "id": "train_2343", "context": "A store was vandalized repeatedly over a six-month period . When discussing the problem with a friend, the store owner mentioned having heard that bright lighting around the perimeter of commercial establishments had been known to reduce the incidence of vandalism. Three months later, the store owner reported to the same friend that there had been no incidents of vandalism since their previous conversation. The friend concluded that bright lighting had been installed around the perimeter of the store.", "question": "Each of the following, if true, would call into question the friend's conclusion EXCEPT:", "answers": "['There had been an increase in police patrolling of the area.', 'The store owner reported that all the stores adjacent to the perimeter also experienced a reduction in vandalism, although stores one block away did not.', 'The store owner brought in a watchdog to protect the store from vandals.', 'Bright lights must be specially ordered from a security company, and installation by the company usually takes at least five months.']", "label": 1 }, { "id": "train_2344", "context": "A bodybuilder needs to win a regional contest before competing in the national championship. Arnold just won the West Coast regional contest, so he is eligible to compete in the national championship. The last five winners of the West Coast regional contest have all won the national championship. Arnold' s trainer has worked with the last three national champions, and he believes Arnold will win the national championship this year. Thus, Arnold will win the national championship.", "question": "The argument above is most vulnerable to criticism on the grounds that:", "answers": "['it draws a hasty generalization based on a small sample size.', 'it confuses a probability with a certainty.', 'it confuses correlation with causation.', 'it confuses a necessary and sufficient condition.']", "label": 1 }, { "id": "train_2345", "context": "Laila: Though lying may be unacceptable in most cases, there are exceptions: when lying brings about more good than harm, lying is morally permissible.", "question": "Which one of the following judgments conforms most closely to the principle stated by Laila?", "answers": "['It is morally permissible for Marcus to lie to his parents about where he is going for the evening as long as what he is going to do is not itself a bad thing to do.', 'It is morally permissible for Debra to lie to keep Thomas from being unhappy as long as the lie helps Thomas and does no harm.', 'It is morally permissible for Lane to lie to the police about the whereabouts of a friend even if Lane suspects the friend has committed a crime.', 'It is morally permissible to lie to innocent people if the lie will cause those people to make a choice that will benefit them but may harm others.']", "label": 1 }, { "id": "train_2346", "context": "Scientists propose placing seismic stations on the floor of the Pacific Ocean to warn threatened coastal communities on the northwestern coast of the United States of approaching tidal waves caused by earthquakes. Since forewarned communities could take steps to evacuate, many of the injuries and deaths that would otherwise occur could be avoided if the government would implement this proposal.", "question": "The answer to which of the following questions would be most important in determining whether implementing the proposal would be likely to achieve the desired result?", "answers": "['How soon after a tidal wave hits land is it safe for evacuees to return to their communities?', 'Would there be enough time after receiving warning of an approaching tidal wave for communities to evacuate safely?', 'When was the last time that the coastal communities were threatened by an approaching tidal wave?', 'Can the stations be equipped to collect and relay information about phenomena other than tidal waves caused by earthquakes?']", "label": 1 }, { "id": "train_2347", "context": "It is now a common complaint that the electronic media have corroded the intellectual skills required and fostered by the literary media. But several centuries ago the complaint was that certain intellectual skills, such as the powerful memory and extemporaneous eloquence that were intrinsic to oral culture, were being destroyed by the spread of literacy. So, what awaits us is probably a mere alteration of the human mind rather than its devolution.", "question": "The reference to the complaint of several centuries ago that powerful memory and extemporaneous eloquence were being destroyed plays which one of the following roles in the argument?", "answers": "['an example of a cultural change that did not necessarily have a detrimental effect on the human mind overall', 'possible evidence, mentioned and then dismissed, that might be cited by supporters of the hypothesis being criticized', 'an illustration of the general hypothesis being advanced that intellectual abilities are inseparable from the means by which people communicate', 'evidence supporting the claim that the intellectual skills fostered by the literary media are being destroyed by the electronic media']", "label": 0 }, { "id": "train_2348", "context": "No one with a serious medical problem would rely on the average person to prescribe treatment. Similarly, since a good public servant has the interest of the public at heart, __.", "question": "Which one of the following statements would most reasonably complete the argument?", "answers": "['public servants should not be concerned about the outcomes of public opinion surveys', \"public servants should base decisions on something other than the average person's recommendations\", 'the average public servant knows more about what is best for society than the average person does', 'one is a good public servant if one is more knowledgeable about the public good than is the average person']", "label": 1 }, { "id": "train_2349", "context": "Scientist: Some pundits claim that the public is afraid of scientists. This isn' t true. I have been a scientist for several decades, and I have never met anyone who is afraid of scientists.", "question": "Which one of the following is an assumption required by the scientist's argument?", "answers": "['If the public were afraid of scientists, then over several decades a scientist would encounter at least one person who was afraid of scientists.', 'If a person understood what science is really about, then that person would not be afraid of scientists.', 'Alleged scientific claims may be used to manipulate people, and it is understandable that people would be on their guard against such manipulation.', 'People may be apprehensive about technological developments that result from science even if they are not afraid of scientists themselves.']", "label": 0 }, { "id": "train_2350", "context": "It was once believed that cells grown in laboratory tissue cultures were essentially immortal. That is, as long as all of their needs were met, they would continue dividing forever. However, it has been shown that normal cells have a finite reproductive limit. A human liver cell, for example, divides 60 times and then stops. If such a cell divides 30 times and then is put into a deep freeze for months or even years, it \"remembers\" where it stopped dividing. After thawing, it divides another 30 times -- but no more.", "question": "If the information above is accurate, a liver cell in which more than 60 divisions took place in a tissue culture CANNOT be which one of the following?", "answers": "['a normal human liver cell that had been frozen after its first division and afterward thawed', 'a normal liver cell that came from an individual of a nonhuman species and had been frozen after its first division and afterward thawed', 'a normal cell that came from the liver of an individual of a nonhuman species and had never been frozen', 'an abnormal human liver cell']", "label": 0 }, { "id": "train_2351", "context": "The radiation absorbed by someone during an ordinary commercial airline flight is no more dangerous than that received during an ordinary dental X-ray. Since a dental X-ray does negligible harm to a person, we can conclude that the radiation absorbed by members of commercial airline flight crews will also do them negligible harm.", "question": "A flaw in the argument is its failure to consider that", "answers": "['there may be many forms of dangerous radiation other than X-rays and the kinds of radiation absorbed by members of commercial airline flight crews', 'receiving a dental X-ray may mitigate other health risks, whereas flying does not', 'flying at high altitude involves risks in addition to exposure to minor radiation', 'the longer and the more often one is exposed to radiation, the more radiation one absorbs and the more seriously one is harmed']", "label": 3 }, { "id": "train_2352", "context": "All orchid species that are pollinated solely by insects have features that attract insects. The flower of a recently discovered orchid species contains edible tendrils that invariably attract insects to the inside of the flower. Hence, it follows that this orchid species is one that is pollinated solely by insects.", "question": "The argument is flawed because it", "answers": "['assumes without warrant that a characteristic that distinguishes one class of things from another is the only characteristic that distinguishes that class from the other', 'mistakes a characteristic that is unique to one particular class of things for a characteristic that is unique to an unrelated class of things', 'treats a characteristic known to be true of one class of things as if that characteristic were unique to that class', 'makes broad generalizations about all members of a particular class of things on the basis of what is known about a member of an unrelated class of things']", "label": 2 }, { "id": "train_2353", "context": "Television news coverage gives viewers a sense of direct involvement with current events but does not provide the depth of coverage needed for the significance of those events to be appreciated. Newspapers, on the other hand, provide depth of coverage but no sense of direct involvement. Unfortunately, a full understanding of current events requires both an appreciation of their significance and a sense of direct involvement with them. Ttherefore, since few people seek out news sources other than newspapers and television, few people ever fully understand current events.", "question": "The reasoning in the argument is flawed because the argument", "answers": "['treats two things, neither one of which can plausibly be seen as excluding the other, as though they were mutually exclusive', 'makes crucial use of the term \"depth of coverage\" without defining it', 'fails to consider the possible disadvantages of having a sense of direct involvement with tragic or violent events', 'ignores the possibility that people read newspapers or watch television for reasons other than gaining a full understanding of current events']", "label": 0 }, { "id": "train_2354", "context": "Although the code of the Nutri-Fare Grocery Store states that the company must employ some workers with disabilities, no blind or deaf employees have yet been hired for employment by the grocery store. Ttherefore, the grocery store is currently in violation of its code.", "question": "The conclusion of the argument follows logically if which one of the following is assumed?", "answers": "['The grocery store currently has no employees who are blind or deaf.', 'All employees of Nutri-Fare live within five miles of the store.', 'All employees with disabilities are blind or deaf.', \"The grocery store's code cannot be modified in order to avoid its being violated.\"]", "label": 2 }, { "id": "train_2355", "context": "Twelve healthy volunteers with the Apo-A-IV-1 gene and twelve healthy volunteers who instead have the Apo-A-IV-2 gene each consumed a standard diet supplemented daily by a high-cholesterol food. A high level of cholesterol in the blood is associated with an increased risk of heart disease. After three weeks, the blood cholesterol levels of the subjects in the second group were unchanged, whereas the blood cholesterol levels of those with the Apo-A-IV-1 gene rose 20 percent.", "question": "Which one of the following is most strongly supported by the information above?", "answers": "['The presence of the Apo-A-IV-2 gene may inhibit the elevation of blood cholesterol.', 'Most of those at risk of heart disease may be able to reduce their risk by adopting a low-cholesterol diet.', 'Approximately half the population carries a gene that lowers cholesterol levels.', 'The presence of the Apo-A-IV-1 gene seems to indicate that a person has a lower risk of heart disease.']", "label": 0 }, { "id": "train_2356", "context": "In the last few years, a drastic spike in sea temperatures around Prudhoe Bay has caused the eggs of snow crabs to hatch earlier in the year than they had previously. As a result, baby snow crabs are more likely to be caught by deep sea fishing boats, since deep sea fishing boats are more numerous in Arctic waters during the fall season than they are during the winter season.", "question": "Which of the following pieces of information is necessary to evaluate the argument?", "answers": "['The time of year the deep sea fishing boats arrive to the waters around Prudhoe Bay', 'The total number of snow crabs that hatch versus the total number that had hatched before the increase in temperatures', 'The month in which the boats leave, and the month in which the snow crabs eggs hatch', 'The amount of time the deep sea fishing boats spend in water directly above where the snow crabs are likely to hatch']", "label": 2 }, { "id": "train_2357", "context": "Anthropologist: One of the distinctive traits of humans is the ability to support a large brain with a small gut, which requires getting more calories from less food. It was likely the development of cooking that made this possible. After all, our ancestors developed large brains around the time that they began to control fire. And even today, people who choose to eat only raw food have difficulty getting enough calories.", "question": "Which one of the following, if true, most strengthens the anthropologist's argument?", "answers": "['Raw meat contains more calories than a similar quantity of raw vegetables.', 'The human body uses more calories to process raw food than it uses to process cooked food.', 'Cooked foods contain the same number of calories as raw foods.', 'Domesticated plants and animals are richer in calories than their wild counterparts are.']", "label": 1 }, { "id": "train_2358", "context": "When a planetary system forms, the chances that a planet capable of supporting life will be formed are high. The chances that a large planet the size of Jupiter or Saturn will be formed, however, are low. Without Jupiter and Saturn, whose gravitational forces have prevented Earth from being frequently struck by large comets, intelligent life would never have arisen on Earth. Since planetary systems are unlikely to contain any large planets, the chances that intelligent life will emerge on a planet are, ttherefore, low.", "question": "Knowing which one of the following would be most useful in evaluating the argument?", "answers": "['how likely it is that planetary systems containing large planets will also contain planets the size of Earth', 'whether large comets could be deflected by only one large planet rather than by two', 'how high the chances are that planetary systems will contain many large comets', 'whether all planetary systems are formed from similar amounts of matter']", "label": 2 }, { "id": "train_2359", "context": "Joshi is clearly letting campaign contributions influence his vote in city council. His campaign for re-election has received more financial support from property developers than any other city councilor' s has. And more than any other councilor' s, his voting record favors the interests of property developers.", "question": "The reasoning in the argument is most vulnerable to criticism on the grounds that the argument", "answers": "[\"has a conclusion that is simply a restatement of one of the argument's stated premises\", 'takes for granted that because certain events occurred sequentially, the earlier events caused the later events', \"confuses one thing's being necessary for another to occur with its being sufficient to make it occur\", 'presumes that one thing is the cause of another when it could easily be an effect of it']", "label": 3 }, { "id": "train_2360", "context": "Five years ago, the hair dryer produced by the Wilson Appliance Company accounted for 50 percent of all sales of hair dryers nationwide. Currently, however, Wilson Appliance' s product makes up only 25 percent of such sales. Because of this decline, and because the average net income that Wilson receives per hair dryer sold has not changed over the last 5 years, the company' s net income from sales of the product must be only half of what it was 5 years ago.", "question": "The reasoning in the argument is flawed because the argument", "answers": "[\"mistakes a decline in the market share of Wilson Appliance's hair dryer for a decline in the total sales of that product\", 'does not provide specific information about the profits hair dryers generate for the companies that produce them', \"fails to discuss sales figures for Wilson Appliance's products other than its hair dryers\", \"overlooks the possibility that the retail price of Wilson Appliance's hair dryer may have increased over the past 5 years\"]", "label": 0 }, { "id": "train_2361", "context": "Surgeon General: Smoking causes more deaths than the combined causes of HIV, illegal drug use, alcohol consumption, motor vehicle accidents, and firearms, killing more than 480, 000 Americans each year. As opposed to nonsmokers, smokers are two-to-four times more likely to suffer from heart disease and twenty-five times more likely to develop lung cancer. Nevertheless, the United States is founded on the principle of liberty and free market business. If Americans want to smoke, they should be free to do so, and if a market exists, businesses should be able to meet that demand. The most we can possibly do is educate Americans about the risks of smoking.", "question": "Which of the following is most strongly suggested by the Surgeon General's statement above?", "answers": "['Americans should be able to do whatever they want with their bodies.', 'Without big businesses marketing cigarettes, no Americans would smoke.', 'Protecting some principles is more important than health.', 'The United States should ban smoking.']", "label": 2 }, { "id": "train_2362", "context": "A prominent investor who holds a large stake in the Burton Tool Company has recently claimed that the company is mismanaged, citing as evidence the company's failure to slow down production in response to a recent rise in its inventory of finished products. It is doubtful whether an investors sniping at management can ever be an thing other than counterproductive, but in this case it is clearly not justified. It is true that an increased inventory of finished products often indicates that production is outstripping demand , but in Burton' s case it indicates no such thing. Rather, the increase in inventory is entirely attributable to products that have already been assigned to orders received from customers .", "question": "In the argument given, the two boldfaced portions play which of the following roles?", "answers": "['The first states a generalization that underlies the position that the argument as a whole opposes; the second clarifies the meaning of a specific phrase as it is used in that generalization.', 'The first provides evidence to support the conclusion of the argument as a whole the second states that conclusion', 'The first states a generalization that underlies the position that the argument as a whole opposes; the second provides evidence to show that the generalization does not apply in the case at issue', 'The first provides evidence to support the conclusion of the argument as a whole, the second is evidence that has been used to support the position that the argument as a whole opposes']", "label": 2 }, { "id": "train_2363", "context": "Aroca City currently funds its public schools through taxes on property. In place of this system, the city plans to introduce a sales tax of 3 percent on all retail sales in the city. Critics protest that 3 percent of current retail sales falls short of the amount raised for schools by property taxes. The critics are correct on this point. Nevertheless, implementing the plan will probably not reduce the money going to Aroca' s schools. Several large retailers have selected Aroca City as the site for huge new stores, and these are certain to draw large numbers of shoppers from neighboring municipalities, where sales are taxed at rates of 6 percent and more. In consequence, retail sales in Aroca City are bound to increase substantially.", "question": "In the argument given, the two portions in boldface play which of the following roles?", "answers": "['The first presents a plan that the argument seeks to defend against a certain criticism; the second is that criticism.', 'The first presents a plan that the argument contends is the best available; the second is a conclusion drawn by the argument to justify that contention.', 'The first presents a plan that the argument concludes is unlikely to achieve its goal; the second expresses that conclusion.', \"The first presents a plan one of whose consequences is at issue in the argument; the second is the argument's conclusion about that consequence.\"]", "label": 3 }, { "id": "train_2364", "context": "A recently invented fabric specially designed for tee shirts is much more practical than the cotton fabrics it could replace: The new fabric costs only twice as much per yard as cotton fabrics, yet it holds its shape through up to 50 more laundry cycles than cotton fabrics. Still, regardless of the new fabric' s impressive attributes, clothing manufacturers have envisaged that the new fabric will not be popular with consumers.", "question": "Each of the following, if true, provides support for the clothing manufacturers' expectations EXCEPT:", "answers": "['Tee shirts in general have become less popular recently because of changes in fashion.', 'Another fabric company has recently introduced a fabric that costs the same per yard as cotton fabrics and holds its shape through 60 more laundry cycles.', 'People generally are unconcerned with how long their tee shirts maintain their shape.', 'The new fabric is going to be available in unusual colors and textures, as well as more common ones.']", "label": 3 }, { "id": "train_2365", "context": "In discussing the pros and cons of monetary union among several European nations, some politicians have claimed that living standards in the countries concerned would first have to converge if monetary union is not to lead to economic chaos. This claim is plainly false, as is demonstrated by the fact that living standards diverge widely between regions within countries that nevertheless have stable economies.", "question": "In attempting to refute the politicians' claim, the author does which one of the following?", "answers": "['presents an earlier instance of the action being considered in which the predicted consequences did not occur', 'gives an example of a state of affairs, assumed to be relevantly similar, in which the allegedly incompatible elements coexist', 'argues that the feared consequence would occur regardless of what course of action was followed', 'argues that those making the claim are mistaken about a temporal relationship that has been observed']", "label": 1 }, { "id": "train_2366", "context": "Aroca City currently funds its public schools through taxes on property. In place of this system, the city plans to introduce a sales tax of three percent on all retail sales in the city. Critics protest that three percent of current retail sales falls short of the amount raised for schools by property taxes. The critics are correct on this point. Nevertheless, implementing the plan will probably not reduce the money going to Aroca' s schools. Several large retailers have selected Aroca City as the site for huge new stores, and these are certain to draw large numbers of shoppers from neighboring municipalities, where sales are taxed at rates of six percent and more. In consequence, retail sales in Aroca City are bound to increase substantially.", "question": "In the argument given, the two potions in boldface play which of the following roles?", "answers": "['The first presents a plan that the argument criticizes; the second presents a consideration that has been raised against that criticism.', 'The first presents a plan that the argument seeks to defend against a certain criticism; the second gives part of the basis for that criticism.', 'The first presents a plan that the argument criticizes; the second is a consideration raised by the argument in support of that criticism.', 'The first presents a plan that the argument seeks to defend against a certain criticism; the second gives part of the basis for that defense.']", "label": 3 }, { "id": "train_2367", "context": "Essayist: The way science is conducted and regulated can be changed. But we need to determine whether the changes are warranted, taking into account their price. The use of animals in research could end immediately, but only at the cost of abandoning many kinds of research and making others very expensive. The use of recombinant DNA could be drastically curtailed. Many other restrictions could be imposed, complete with a system of fraud police. But such massive interventions would be costly and would change the character of science.", "question": "Which one of the following most accurately expresses the main conclusion of the essayist's argument?", "answers": "['If we regulate science more closely, we will change the character of science.', 'The regulation of science and the conducting of science can be changed.', 'We should not make changes that will alter the character of science.', 'We need to be aware of the impact of change in science before changes are made.']", "label": 3 }, { "id": "train_2368", "context": "By examining the fossilized leaves of any prehistoric plant it is possible to determine the climate in which that specimen grew because the size and shape of a leaf are unique to a given climate. Since the climate at a given location depends on the altitude at that location, it follows that the size and shape of a fossilized leaf also indicates the altitude at which the plant grew.", "question": "The reasoning in the argument is vulnerable to criticism on the grounds that it", "answers": "['treats the size and shape of a leaf as if they were the only physical characteristics of a leaf that depend on climate', 'relies on a weak analogy between a leaf and the fossil of a leaf as evidence for the claims advanced', 'fails to demonstrate that no species of plant can long survive a violent change in its environment', 'overlooks the possibility that locations at different altitudes can have the same climate']", "label": 3 }, { "id": "train_2369", "context": "Sociologist: A recent study of 5, 000 individuals found, on the basis of a physical exam, that more than 25 percent of people older than 65 were malnourished, though only 12 percent of the people in this age group fell below government poverty standards. In contrast, a greater percentage of the people 65 or younger fell below poverty standards than were found in the study to be malnourished.", "question": "Each of the following, if true, helps to explain the findings of the study cited by the sociologist EXCEPT:", "answers": "['People 65 or younger are less likely to have medical conditions that interfere with their digestion than are people over 65.', 'Doctors are less likely to correctly diagnose and treat malnutrition in their patients who are over 65 than in their younger patients.', 'People 65 or younger are no more likely to fall below government poverty standards than are people over 65.', 'People over 65 are more likely to take medications that increase their need for certain nutrients than are people 65 or younger.']", "label": 2 }, { "id": "train_2370", "context": "Carrillo: Using the number of existing primate species, along with measures of the genetic diversify among these primates and among the extinct primate species, our statistical model strongly supports the conclusion that the first primate developed around 81. 5 million years ago. Olson: Given that the oldest primate fossils discovered so far date back only 55 million years, your estimate of how long primate species' development has gone on is sheer speculation.", "question": "The dialogue provides the most support for the claim that Carrillo and Olson disagree over whether", "answers": "['primates have been around for more than 55 million years', 'fossils of the first primate species that developed have been discovered', \"Carrillo's statistical model is a reliable way of dating the first appearance of primate species\", 'the available sample of primate fossils is representative of the variety of primate species that have existed']", "label": 2 }, { "id": "train_2371", "context": "The reason music with a simple recurring rhythm exerts a strong primordial appeal is that it reminds us of the womb environment. After all, the first sound heard within the womb is the comforting sound of the mother' s regular heartbeat. So in taking away from us the warmth and security of the womb, birth also takes away a primal and constant source of comfort. Thus it is extremely natural that in seeking sensations of warmth and security throughout life, people would be strongly drawn toward simple recurring rhythmic sounds.", "question": "Which one of the following most accurately expresses the main conclusion drawn in the reasoning above?", "answers": "[\"The comforting sound of the mother's regular heartbeat is the first sound that is heard inside the womb.\", 'Birth deprives us of a primal and constant source of comfort when it takes away the warmth and security of the womb.', 'People seek sensations of warmth and security throughout life because birth takes away the warmth and security of the womb.', 'The explanation of the strong primordial appeal of music with a simple recurring rhythm is that it reminds us of the womb environment.']", "label": 3 }, { "id": "train_2372", "context": "A study of 86 patients, all of whom suffered from disease T and received the same standard medical treatment, divided the patients into 2 equal groups. One group' s members all attended weekly support group meetings, but no one from the other group attended support group meetings. After 10 years, 41 patients from each group had died. Clearly, support group meetings do not help patients with disease T live longer.", "question": "Which one of the following statements, if true, most seriously weakens the argument?", "answers": "['The members of the group that attended weekly support group meetings lived 2 years longer, on average, than the members of the other group.', 'Some physicians have argued that attending weekly support group meetings gives patients less faith in the standard treatment for disease T.', 'Everyone in the group whose members attended weekly support group meetings reported after 1 year that those meetings had helped them to cope with the disease.', 'For many diseases, attending weekly support group meetings is part of the standard medical treatment.']", "label": 0 }, { "id": "train_2373", "context": "Experienced gardeners advise against planting snap peas after late April because peas do not develop properly in warm weather. This year, however, the weather was unusually cool into late June, and ttherefore the fact that these snap peas were planted in mid-May is unlikely to result in crop failure despite the experts' warnings.", "question": "The pattern of reasoning displayed above is most closely paralleled in which one of the following?", "answers": "['Since African violets do not thrive in direct sunlight, it is said that in this region these plants should be placed in windows facing north rather than south; however, since these south-facing windows are well shaded by evergreen trees, the African violets placed in them are likely to grow satisfactorily.', 'House plants generally grow best in pots slightly larger than their existing root systems, so the usual advice is to repot when roots first reach the sides of the pot; this rule should not be followed with amaryllis plants, however, because they are likely to do best with tightly compressed roots.', 'Where flowers are to be planted under shade trees, gardening experts often advise using impatiens since impatiens does well in conditions of shade; however, it is unlikely to do well under maple trees since maple tree roots are so near the surface that they absorb all available moisture.', \"According to many gardening authorities, tomatoes should not be planted near dill because doing so is likely to affect their taste adversely; however, since these tomatoes were grown near dill and taste fine, there is clearly no reason to pay much attention to the so-called experts' advice.\"]", "label": 0 }, { "id": "train_2374", "context": "The local radio station will not win the regional ratings race this year. In the past ten years the station has never finished better than fifth place in the ratings. The station' s manager has not responded to its dismal ratings by changing its musical format or any key personnel, while the competition has often sought to respond to changing tastes in music and has aggressively recruited the region' s top radio personalities.", "question": "The reasoning in which one of the following is most similar to that in the argument above?", "answers": "[\"All lions are mammals. Ttherefore Leo, the local zoo's oldest lion, is a mammal too.\", 'Recently stock prices have always been lower on Mondays. Ttherefore they will be lower this coming Monday too.', 'Only trained swimmers are lifeguards, so it follows that the next lifeguard at the local pool will be a trained swimmer.', 'Every swan I have seen was white. Ttherefore all swans are probably white.']", "label": 1 }, { "id": "train_2375", "context": "Megan: People pursue wealth beyond what their basic needs require only if they see it as a way of achieving high status or prestige. Channen: Not everybody thinks that way. After all, money is the universal medium of exchange. So, if you have enough of it, you can exchange it for whatever other material goods you may need or want even if you are indifferent to what others think of you.", "question": "Megan and Channen disagree over whether", "answers": "[\"it is rational to maximize one's ability to purchase whatever one wants only when the motive for doing so is something other than the desire for prestige\", \"the motive for pursuing wealth beyond what one's basic needs require is ever anything other than the desire for prestige or high status\", \"it is irrational to try to achieve high status or prestige in the eyes of one's society\", 'people ever pursue wealth beyond what is required for their basic needs']", "label": 1 }, { "id": "train_2376", "context": "Tony: A new kind of videocassette has just been developed. It lasts for only half as many viewings as the old kind does but costs a third as much. Ttherefore, video rental stores would find it significantly more economical to purchase and stock movies recorded on the new kind of videocassette than on the old kind. Anna: But the videocassette itself only accounts for 5 percent of the price a video rental store pays to buy a copy of a movie on video; most of the price consists of royalties the store pays to the studio that produced the movie. So the price that video rental stores pay per copy would decrease by considerably less than 5 percent, and royalties would have to be paid on additional copies.", "question": "Anna's reply is structured to lead to which one of the following conclusions?", "answers": "['The largest part of the fee a customer pays to rent a movie from a video rental store goes toward the royalties the store paid in purchasing that movie.', 'If the price a video rental store pays to buy a movie on videocassette does not decrease, the rental fee the store charges on the movie will not decrease.', 'The cost savings to video rental stores that buy movies recorded on the cheaper videocassettes rather than movies recorded on the more durable ones will be small or nonexistent.', 'Video rental stores should always stock the highest-quality videocassettes available, because durability is more important than price.']", "label": 2 }, { "id": "train_2377", "context": "David: Forbidding companies from hiring permanent replacements for striking employees would be profoundly unfair. Such companies would have little leverage in their negotiations with strikers. Lin: No, the companies would still have sufficient leverage in negotiations if they hired temporary replacements.", "question": "Which one of the following statements is most strongly supported by the exchange between David and Lin?", "answers": "['David and Lin believe that companies should be allowed as much leverage in negotiations as the striking employees.', 'Lin believes it is unfair to forbid companies from hiring permanent replacements for their striking employees.', 'David and Lin disagree over the amount of leverage companies lose in their negotiations with strikers by not being able to hire permanent replacements.', 'David and Lin disagree over how much leverage should be accorded companies in their negotiations with strikers.']", "label": 2 }, { "id": "train_2378", "context": "Marine biologists had hypothesized that lobsters kept together in lobster traps eat one another in response to hunger. Periodic checking of lobster traps, however, has revealed instances of lobsters sharing traps together for weeks. Eight lobsters even shared one trap together for two months without eating one another. The marine biologists' hypothesis, ttherefore, is clearly wrong.", "question": "The argument against the marine biologists' hypothesis is based on which one of the following assumptions?", "answers": "['Any food that the eight lobsters in the trap might have obtained was not enough to ward off hunger.', 'Lobsters not caught in lobster traps have been observed eating one another.', 'It is unusual to find as many as eight lobsters caught together in one single trap.', 'Two months is the longest known period during which eight or more lobsters have been trapped together.']", "label": 0 }, { "id": "train_2379", "context": "Advertisement: The new Reflex computer represents a conceptual advance. Unlike traditional computers, the Reflex has a built-in monitoring function that continuously checks all other computer operations and lets you know if they are malfunctioning in any way, thus preventing the loss of data. With the Reflex, ttherefore, you' ll never lose data again!", "question": "Which one of the following is an assumption on which the advertisement's argument depends?", "answers": "['The monitoring function provides suggestions on how to prevent future malfunctions.', 'The monitoring function of the Reflex is not subject to frequent unpredictable malfunctioning.', 'The Reflex computer continues to process data while it is warning of a malfunction.', \"The monitoring function of the Reflex does not reduce the computer's speed.\"]", "label": 1 }, { "id": "train_2380", "context": "Approximately 7. 6 million women who earn incomes have preschool-age children, and approximately 6. 4 million women are the sole income earners for their families. These figures indicate that there are comparatively few income-earning women who have preschool-age children but are not the sole income earners for their families.", "question": "A major flaw in the reasoning is that it", "answers": "['overlooks the possibility that there is little or no overlap between the two populations of women cited', 'fails to indicate whether the difference between the two figures cited will tend to remain stable over time', 'provides no information on families in which men are the sole income earners', 'relies on figures that are too imprecise to support the conclusion drawn']", "label": 0 }, { "id": "train_2381", "context": "In general, jobs are harder to get in times of economic recession because many businesses cut back operations. However, any future recessions in Vargonia will probably not reduce the availability of teaching jobs at government-funded schools. This is because Vargonia has just introduced a legal requirement that education in government-funded schools be available, free of charge, to all Vargonian children regardless of the state of the economy, and that current student-teacher ratios not be exceeded.", "question": "Which of the following, if true, most strengthens the argument?", "answers": "[\"Teachers in Vargonia's government-funded schools are well paid relative to teachers in most privately funded schools in Vargonia, many of which rely heavily on part-time teachers\", \"Nearly 20 percent more teachers are currently employed in Vargonia's government-funded schools than had been employed in those schools in the period before the last economic recession\", 'During the last economic recession in Vargonia, the government permanently closed a number of the schools that it had funded', 'During recent periods when the Vargonian economy has been strong, almost 25 percent of Vargonian children have attended privately funded schools, many of which charge substantial fees']", "label": 3 }, { "id": "train_2382", "context": "Soccer is the most popular sport in the world. Unlike other sports, soccer does not require very much equipment. The field can be any size, goal posts can be makeshift, and anything that can be kicked can serve as the ball. Ttherefore, it is entirely unsurprising that the most recent FIFA World Cup -- the largest international soccer tournament -- attracted more than three billion viewers, which was a world record.", "question": "The argument depends on which one of the following assumptions?", "answers": "[\"Soccer is relatively less popular in the United States due to the country's prosperity.\", 'People who play a sport are likely to be fans of that sport.', 'Other sports would be similarly popular if they could reduce their economic cost.', 'Soccer is the most popular in South America and Europe.']", "label": 1 }, { "id": "train_2383", "context": "An overly centralized economy, not the changes in the climate, is responsible for the poor agricultural production in Country X since its new government came to power. Neighboring Country Y has experienced the same climatic conditions, but while agricultural production has been falling in Country X, it has been rising in Country Y.", "question": "Which of the following, if true, would most weaken the argument above?", "answers": "['Industrial production also is declining in Country X.', 'The crops that have always been grown in Country X are different from those that have always been grown in Country Y.', 'Both Country X and Country Y have been experiencing drought conditions.', \"Country X's new government instituted a centralized economy with the intention of ensuring an equitable distribution of goods.\"]", "label": 1 }, { "id": "train_2384", "context": "There are far fewer independent bookstores than there were 20 years ago, largely because chain bookstores prospered and multiplied during that time. Thus, chain bookstores' success has been to the detriment of book consumers, for the shortage of independent bookstores has prevented the variety of readily available books from growing as much as it otherwise would have.", "question": "Which one of the following is an assumption on which the argument relies?", "answers": "['Independent bookstores typically do not sell the kinds of books that are available in chain bookstores.', 'Book consumers would be better off if there were a greater variety of readily available books than there currently is.', 'The average bookstore today is larger than the average bookstore of 20 years ago.', 'Some book consumers value low prices more highly than wide selection.']", "label": 1 }, { "id": "train_2385", "context": "The Green Ensemble, a nonprofit theater group, has always been financially dependent on contributions from corporations and would have been forced to disband this year if any of its corporate sponsors had withdrawn their financial support. But the Green Ensemble has not only been able to continue in operation throughout the year, but has recently announced its schedule for next year.", "question": "Which one of the following is a conclusion that can be properly drawn from the information above?", "answers": "['Earlier this year the Green Ensemble found other sources of funding for next year, making the group less dependent on corporations for financial support.', \"This year corporate funding was the source of more than half of the Green Ensemble's income.\", 'During this year corporate funding for the Green Ensemble has been steadily increasing.', \"None of the Green Ensemble's corporate sponsors withdrew their financial support of the group this year.\"]", "label": 3 }, { "id": "train_2386", "context": "Internet music downloading websites allow consumers to download music onto their home computers. Patrons of these music downloading websites purchase more albums from retail stores than people who do not download music through these websites. Ttherefore, downloading music on these websites actually encourages consumers to purchase more albums than they otherwise would.", "question": "The argument is most vulnerable to criticism because it", "answers": "['uses inherently self-contradicting claims to support its conclusion', 'generalizes about the behavior of a large group based on the actions of a few unrepresentative samples', 'suggests a premise must be false because of the lack of evidence to support it', 'fails to consider that the concurrent existence of two states of affairs does not imply that one caused the other']", "label": 3 }, { "id": "train_2387", "context": "Civil libertarian: The categorical prohibition of any nonviolent means of expression inevitably poisons a society' s intellectual atmosphere. Ttherefore, those advocating censorship of all potentially offensive art are pursuing a course that is harmful to society. Censorship advocate: You' re wrong, because many people are in agreement about what constitutes potentially offensive art.", "question": "The censorship advocate's rebuttal is flawed because it", "answers": "[\"relies on an irrelevant reason for rejecting the civil libertarian's argument\", 'attempts to extract a general rule from a specific case', 'extracts an erroneous principle from a commonly held belief', \"attacks the civil libertarian's character instead of the argument\"]", "label": 0 }, { "id": "train_2388", "context": "People with a certain eye disorder are virtually unable to see in moderately bright light, which seems to them unbearably intense, since the cells of their retinas are overwhelmed by moderately bright light. These people do, however, show normal sensitivity to most components of dim light. Their retinal cells are also not excessively sensitive to red components of moderately bright light.", "question": "The information above best supports which of the following hypotheses about people with the disorder described, if they have no other serious visual problems?", "answers": "['In all moderately dim light in which people without the disorder can read large print, people with the disorder cannot read such print.', 'Eyeglasses that are transparent to red components of light but filter out other components of light help these people see in moderately bright light.', 'These people perceive colors other than red in the same way as do most people who do not have the disorder.', 'These people typically see more acutely at night and in dim light than do most people who do not have the disorder.']", "label": 1 }, { "id": "train_2389", "context": "Historian: Flavius, an ancient Roman governor who believed deeply in the virtues of manual labor and moral temperance, actively sought to discourage the arts by removing state financial support for them. Also, Flavius was widely unpopular among his subjects, as we can conclude from the large number of satirical plays that were written about him during his administration.", "question": "The historian's argumentation is most vulnerable to criticism on the grounds that it", "answers": "[\"fails to consider the percentage of plays written during Flavius's administration that were not explicitly about Flavius\", 'fails to consider whether manual labor and moral temperance were widely regarded as virtues in ancient Rome', \"treats the satirical plays as a reliable indicator of Flavius's popularity despite potential bias on the part of the playwrights\", 'presumes, without providing evidence, that Flavius was unfavorably disposed toward the arts']", "label": 2 }, { "id": "train_2390", "context": "When released into the atmosphere, the refrigerant Freon damages the Earth' s ozone layer. A new kind of refrigerant does not have this effect. The manufacturer claims that replacing Freon with the new refrigerant in both new and existing refrigerators will prevent any further Freon damage to the ozone layer apart from that being done by the Freon already in the atmosphere.", "question": "Which of the following must be true if the manufacturer's prediction is to prove accurate?", "answers": "[\"The new refrigerant can counteract the damaging effects of Freon on the Earth's atmosphere.\", 'The damage already done to the ozone layer is not of environmentally significant proportions.', 'The new refrigerant causes no environmental damage of any kind when it is released into the atmosphere.', 'Freon can be replaced with the new refrigerant without releasing any Freon into the atmosphere.']", "label": 3 }, { "id": "train_2391", "context": "Everyone who is excessively generous is not levelheaded, and no one who is levelheaded is bold.", "question": "Which one of the following is strictly implied by the above?", "answers": "['Everyone who is excessively generous is not bold.', 'If someone is levelheaded, then that person is neither bold nor excessively generous.', 'No one who is not bold lacks excessive generosity.', 'Everyone who is not bold is excessively generous.']", "label": 1 }, { "id": "train_2392", "context": "Marine biologist: Scientists have long wondered why the fish that live around coral reefs exhibit such brilliant colors. One suggestion is that coral reefs are colorful and, ttherefore, that colorful fish are camouflaged by them. Many animal species, after all, use camouflage to avoid predators. However, as regards the populations around reefs, this suggestion is mistaken. A reef stripped of its fish is quite monochromatic. Most corals, it turns out, are relatively dull browns and greens.", "question": "Which one of the following most accurately expresses the main conclusion drawn in the marine biologist's argument?", "answers": "['The fact that many species use camouflage to avoid predators is one reason to believe that brightly colored fish living near reefs do too.', 'It turns out that the corals in a coral reef are mostly dull hues of brown and green.', 'A reef stripped of its fish is relatively monochromatic.', 'The suggestion that the fish living around coral reefs exhibit bright colors because they are camouflaged by the reefs is mistaken.']", "label": 3 }, { "id": "train_2393", "context": "Albert: The government has proposed new automobile emissions regulations designed to decrease the amount of polycyclic aromatic hydrocarbons (PAHs) released into the atmosphere by automobile exhaust. I don' t see the need for such regulations; although PAHs are suspected of causing cancer, a causal link has never been proven. Erin: Scientists also blame PAHs for 10, 000 premature deaths in this country each year from lung and heart disease. So the proposed regulations would save thousands of lives.", "question": "Which one of the following, if true, is the logically strongest counter that Albert can make to Erin's argument?", "answers": "['PAHs are one of several components of automobile exhaust that scientists suspect of causing cancer.', 'It is not known whether PAHs are a causal factor in any diseases other than heart and lung disease and cancer.', 'Most automobile manufacturers are strongly opposed to additional automobile emissions regulations.', 'Most of the PAHs released into the atmosphere are the result of wear and tear on automobile tires.']", "label": 3 }, { "id": "train_2394", "context": "Advertisement: Anyone who exercises knows from firsthand experience that exercise leads to better performance of such physical organs as the heart and the lungs, as well as to improvement in muscle tone. And since your brain is a physical organ, your actions can improve its performance, too. Act now. Subscribe to Stimulus: read the magazine that exercises your brain.", "question": "The advertisement employs which one of the following argumentative strategies?", "answers": "['It ridicules people who do not subscribe to Stimulus by suggesting that they do not believe that exercise will improve brain capacity.', 'It implies that brains and muscle are similar in one respect because they are similar in another respect.', 'It cites experimental evidence that subscribing to the product being advertised has desirable consequences.', 'It supports its recommendation by a careful analysis of the concept of exercise.']", "label": 1 }, { "id": "train_2395", "context": "Household indebtedness, which some theorists regard as causing recession, was high preceding the recent recession, but so was the value of assets owned by households. Admittedly, if most of the assets were owned by quite affluent households, and most of the debt was owed by low-income households, high household debt levels could have been the cause of the recession despite high asset values: low-income households might have decreased spending in order to pay off debts while the quite affluent ones might simply have failed to increase spending. But, in fact, quite affluent people must have owed most of the household debt, since money is not lent to those without assets. Ttherefore, the real cause must lie elsewhere.", "question": "The argument is structured to lead to which one of the following conclusions?", "answers": "['When people borrowed money prior to the recent recession, they did not use it to purchase assets.', 'High levels of household debt did not cause the recent recession.', 'Low-income households succeeded in paying off their debts despite the recent recession.', 'High levels of household debt have little impact on the economy.']", "label": 1 }, { "id": "train_2396", "context": "Researchers have found that some unprotected areas outside of a national park that was designed to protect birds have substantially higher numbers of certain bird species than comparable areas inside the park.", "question": "Which one of the following, if true, most helps to explain the researchers' finding?", "answers": "['The park was designed to protect endangered bird species, but some of the bird species that are present in higher numbers in the unprotected areas are also endangered.', 'The researchers also found that some unprotected areas outside of the park have substantially higher numbers of certain reptile species than comparable areas inside the park.', 'Moose are much more prevalent inside the park, where hunting is prohibited, than outside the park, and moose eat much of the food that the birds need to survive.', 'Both inside the park and just outside of it, there are riverside areas containing willows and other waterside growth that the bird species thrive on.']", "label": 2 }, { "id": "train_2397", "context": "This semester Professor Popkin will commute to Montreal by plane, leaving every Tuesday and returning to Toronto every Friday. She instructs her travel agent to schedule for her one round-trip that departs from Toronto on the first Tuesday and returns to Toronto on the last Friday of the semester and additional round-trips that depart from and return to Montreal for each weekly commute in between.", "question": "Which one of the following, if true, most helps to explain Professor Popkin's instructions to her travel agent regarding the grouping of her flights into round-trips?", "answers": "[\"If all Professor Popkin's tickets are paid for at one time, she will be eligible for a frequent-traveler discount.\", \"Professor Popkin's round-trip tickets are least expensive if each trip is scheduled so that at least one Saturday night intervenes between the departing flight and the returning flight.\", 'In order for Professor Popkin to receive the lowest possible airfare, each of her round trips must be paid for at least seven days in advance of the departing flight.', 'A ticket for a round-trip in which the departing flight and the returning flight are separated by several months will cost Professor Popkin more than a ticket for a round-trip in which they are separated by less than one week.']", "label": 1 }, { "id": "train_2398", "context": "Principle: If one does not criticize a form of behavior in oneself or vow to stop it, then one should not criticize that form of behavior in another. Application: If Shimada does not vow to stop being tardy himself, he should not criticize McFeney for tardiness.", "question": "Which one of the following, if true, justifies the above application of the principle?", "answers": "['Shimada criticizes McFeney for regularly being tardy, but also criticizes himself for occasional tardiness.', 'Neither McFeney nor Shimada is regularly tardy, but Shimada criticizes McFeney for tardiness nonetheless.', 'McFeney often criticizes Shimada for being tardy, but neither Shimada nor McFeney ever vows to cease being tardy.', \"Both McFeney and Shimada are regularly tardy, but Shimada criticizes McFeney's tardiness without criticizing his own.\"]", "label": 3 }, { "id": "train_2399", "context": "Restaurant manager: In response to requests from our customers for a children' s menu, we recently introduced three different options designed for kids: a grilled cheese sandwich with fries, chicken tenders with onion rings, and a hot dog with potato chips. The first two are frequently ordered, but no one orders the hot dog, although it costs less than the other two. Clearly, then, the children who eat here prefer not to eat hot dogs.", "question": "Which one of the following is an error of reasoning in the restaurant manager's argument?", "answers": "['attempting to prove a claim on the basis of evidence that a number of people hold that claim to be true', 'drawing a conclusion that is inconsistent with one premise of the argument', 'ignoring possible differences between what people say they want and what they actually choose', 'treating one of several plausible explanations of a phenomenon as the only possible explanation']", "label": 3 }, { "id": "train_2400", "context": "Political commentator: Voters tend to elect the candidate whose visual image most evokes positive feelings. Thus, laws designed to increase the fairness of elections should not allow one candidate to buy significantly more media exposure than any rival candidates can afford.", "question": "Which one of the following is an assumption on which the political commentator's argument depends?", "answers": "['Elections are unfair only if at least one of the candidates has more resources than any rival candidate has.', \"The tendency of a candidate's visual image to evoke positive feelings in voters at least sometimes increases as media exposure increases.\", 'Candidates invariably buy as much media exposure as they can afford and campaign laws allow.', \"People have positive feelings toward election candidates only when they find the candidates' visual images familiar.\"]", "label": 1 }, { "id": "train_2401", "context": "Dillworth: More and more people are deciding not to have children because of the personal and economic sacrifices children require and because so often children are ungrateful for the considerable sacrifices their parents do make for them. However, such considerations have no bearing on the fact that their children provide the best chance most people have of ensuring that their values live on after them. Ttherefore, for anyone with deeply held values, foregoing parenthood out of reluctance to make sacrifices for which little gratitude can be expected would probably be a mistake. Travers: Your reasoning ignores another fact that deserves consideration: children' s ingratitude for parental sacrifices usually stems from a wholesale rejection of parental values.", "question": "Dillworth employs which one of the following argumentative strategies?", "answers": "['indirectly establishing that a given course of action is obligatory by arguing that the alternative course of action is prohibited', 'showing that considerations cited as drawbacks to a given course of action are not really drawbacks at all', 'distinguishing a category of person for whom the reason presented in favor of a given course of action is more telling than the reasons cited against that course of action', 'using evidence that a certain course of action would be appropriate under one set of conditions to arrive at a general conclusion about what would be appropriate in all cases']", "label": 2 }, { "id": "train_2402", "context": "Monroe, despite his generally poor appetite thoroughly enjoyed the three meals he ate at the Tip-Top Restaurant, but, unfortunately, after each meal he became ill. The first time he ate an extra-large sausage pizza with a side order of hot peppers; the second time he took full advantage of the all-you-can-eat fried shrimp and hot peppers special; and the third time he had two of Tip-Top' s giant meatball sandwiches with hot peppers. Since the only food all three meals had in common was the hot peppers, Monroe concludes that it is solely due to Tip-Top' s hot peppers that he became ill.", "question": "Monroe's reasoning is most vulnerable to which one of the following criticisms?", "answers": "['He allows his desire to continue dining at Tip-Top to bias his conclusion.', 'He overlooks the fact that at all three meals he consumed what was, for him, an unusually large quantity of food.', 'He posits a causal relationship without ascertaining that the presumed cause preceded the presumed effect.', \"He fails to establish that everyone who ate Tip-Top's hot peppers became ill.\"]", "label": 1 }, { "id": "train_2403", "context": "Currently, warm-blooded pets are allowed to enter the country only if they have a certificate showing that they have been vaccinated against rabies. This policy is flawed because vaccinations sometimes fail to take effect. If the vaccination is successful, the animal has a high level of rabies antibodies in its blood. Ttherefore, a more effective means of preventing pets with rabies from entering the country would be to administer blood tests and refuse entry to any pets whose blood did not contain a high level of rabies antibodies.", "question": "Which one of the following, if true, most substantially weakens the argument?", "answers": "['Under existing policies, some species of animals are quarantined and observed for symptoms of rabies before a decision is made to allow them into the country.', 'The proposed policy would be ineffective in preventing wild animals infected with rabies from entering the country.', 'A significantly larger percentage of rabies vaccinations than of vaccinations against other animal diseases fail to produce immunity.', 'There is no way to distinguish between high antibody levels caused by active infection and high levels caused by successful immunization.']", "label": 3 }, { "id": "train_2404", "context": "Principle: People should not feed wild animals because it makes them dependent on humans and less likely to survive on their own. Situation: Bird lovers commonly feed wild birds to attract them to their yards and gardens.", "question": "Which one of the following, if assumed, would most help to justify treating the human feeding of wild birds as an exception to the principle above?", "answers": "['Some species of wild birds benefit humans by consuming large numbers of mosquitoes and other insect pests.', 'Human settlement is so pervasive in the habitat of most wild birds that they must depend in part on human sources of food for survival.', 'Wild birds are much more likely to congregate in yards where they are fed than in yards where they are not fed.', 'Congregating around human bird feeders makes wild birds more vulnerable to predators and diseases.']", "label": 1 }, { "id": "train_2405", "context": "In a study, parents were asked to rate each television program that their children watched. The programs were rated for violent content on a scale of one to five, with \"one\" indicating no violence and \"five\" indicating a great deal. The number of times their children were disciplined in school was also recorded. Children who watched programs with an average violence rating of three or higher were 50 percent more likely to have been disciplined than other children.", "question": "Each of the following, if true, helps to explain the statistical relationship described above EXCEPT:", "answers": "[\"Parents who rated their children's television viewing low on violence had become desensitized to the violence on television by watching too much of it.\", 'Children who are excited by violent action programs on television tend to become bored with schoolwork and to express their boredom in an unacceptable fashion.', \"Children learn from violent programs on television to disrespect society's prohibitions of violence and, as a result, are more likely than other children to disrespect the school disciplinary codes.\", 'When parents watch violent programs on television with their children, those children become more likely to regard antisocial behavior as legitimate.']", "label": 0 }, { "id": "train_2406", "context": "Among North American school-age children, there is a strong positive correlation between obesity and the amount of television watched. Ttherefore, with the arrival of interactive television, obesity among North American school-age children will increase.", "question": "The argument requires the assumption that", "answers": "['North American school-age children will increase their television viewing with the arrival of interactive television', 'a sedentary lifestyle is among the most important causes of childhood obesity', 'genetics makes no significant contribution to obesity', 'obesity among North American school-age children increased as cable television became widely available']", "label": 0 }, { "id": "train_2407", "context": "Fragments of charred antelope bone have recently been found at a million-year-old settlement of Homo erectus, an early human species. This discovery demonstrates that humans were making controlled use of fire to cook meat at a very early stage of their development.", "question": "The argument above makes which of the following assumptions?", "answers": "['The diet of Homo erectus consisted in large part of antelope meat.', 'Homo erectus could not have eaten antelope meat unless the meat had been cooked.', 'The antelope bones were charred in a fire controlled by humans.', 'Early human settlements can be identified by the presence of charred antelope bones.']", "label": 2 }, { "id": "train_2408", "context": "Many breakfast cereals are fortified with vitamin supplements. Some of these cereals provide 100 percent of the recommended daily requirement of vitamins. Nevertheless, a well-balanced breakfast, including a variety of foods, is a better source of those vitamins than are such fortified breakfast cereals alone.", "question": "Which of the following, if true, would most strongly support the position above?", "answers": "['Cereals containing vitamin supplements are no harder to digest than similar cereals without added vitamins.', 'People who regularly eat cereals fortified with vitamin supplements sometimes neglect to eat the foods in which the vitamins occur naturally.', 'Unprocessed cereals are naturally high in several of the vitamins that are usually added to fortified breakfast cereals.', 'In many foods, the natural combination of vitamins with other nutrients makes those vitamins more usable by the body than are vitamins added in vitamin supplements.']", "label": 3 }, { "id": "train_2409", "context": "For years scientists have been scanning the skies in the hope of finding life on other planets. But in spite of the ever-increasing sophistication of the equipment they employ, some of it costing hundreds of millions of dollars, not the first shred of evidence of such life has been forthcoming. And there is no reason to think that these scientists will be anymore successful in the future, no matter how much money is invested in the search. The dream of finding extraterrestrial life is destined to remain a dream, as science' s experience up to this point should indicate.", "question": "Which one of the following most accurately states the main point of the argument?", "answers": "['Only if scientists had already found evidence of life on other planets would continued search be justified.', 'Scientists searching for extraterrestrial life will not find it.', 'There is no reason to believe that life exists on other planets.', 'The equipment that scientists employ is not as sophisticated as it should be.']", "label": 1 }, { "id": "train_2410", "context": "Theater critic: The theater is in a dismal state. Audiences are sparse and revenue is down. Without the audience and the revenue, the talented and creative people who are the lifeblood of the theater are abandoning it. No wonder standards are deteriorating. Producer: It' s not true that the theater is in decline. Don' t you realize that your comments constitute a self-fulfilling prophecy? By publishing these opinions, you yourself are discouraging new audiences from emerging and new talent from joining the theater.", "question": "Which one of the following is a questionable technique employed by the producer in responding to the critic?", "answers": "['accusing the critic of relying solely on opinion unsupported by factual evidence', 'relying on emphasis rather than on argument', \"challenging the motives behind the critic's remarks rather than the remarks themselves\", \"focusing on the effects of the critic's evaluation rather than on its content\"]", "label": 3 }, { "id": "train_2411", "context": "James: In my own house, I do what I want. In banning smoking on passenger airlines during domestic flights, the government has ignored the airlines' right to set smoking policies on their own property. Eileen: Your house is for your own use. Because a passenger airline offers a service to the public, the passengers' health must come first.", "question": "The basic step in Eileen's method of attacking James' argument is to", "answers": "['establish an analogy', 'question its motivation', 'draw a distinction', 'derive a contradiction from it']", "label": 2 }, { "id": "train_2412", "context": "Advertisement: In a recent survey, a sample representative of all new Popelka Auto Insurance policyholders reported savings of $250 a year, on average, as a result of switching their auto insurance coverage to Popelka. Thus, most people who hold auto insurance policies with other companies could save hundreds of dollars by switching to Popelka.", "question": "The argument in the advertisement is most vulnerable to criticism on which one of the following grounds?", "answers": "['It takes for granted that the new Popelka Auto Insurance policyholders pay no less for their auto insurance, on average, than do people who have held Popelka Auto Insurance policies for a longer period of time.', 'It fails to address adequately the possibility that people capable of saving hundreds of dollars by switching their auto insurance coverage to Popelka are disproportionately represented among the new Popelka auto insurance policyholders.', 'It fails to address adequately the possibility that switching to another insurance company would enable many auto insurance policyholders to save even more money than they would save by switching to Popelka.', 'It takes for granted that few if any of the Popelka Auto Insurance policyholders surveyed underestimated how much they saved when they switched their auto insurance coverage to Popelka.']", "label": 1 }, { "id": "train_2413", "context": "Combustion of gasoline in automobile engines produces benzene, a known carcinogen. Environmentalists propose replacing gasoline with methanol, which does not produce significant quantities of benzene when burned. However, combustion of methanol produces formaldehyde, also a known carcinogen. Ttherefore the environmentalists' proposal has little merit.", "question": "Which one of the following, if true, most supports the environmentalists' proposal?", "answers": "['The engines of some automobiles now on the road burn diesel fuel rather than gasoline.', 'Several large research efforts are underway to formulate cleaner-burning types of gasoline.', 'Formaldehyde is a less potent carcinogen than benzene.', 'Since methanol is water soluble, methanol spills are more damaging to the environment than gasoline spills.']", "label": 2 }, { "id": "train_2414", "context": "All savings accounts are interest-bearing accounts. The interest from some interest-bearing accounts is tax-free, so there must be some savings accounts that have tax-free interest.", "question": "Which one of the following arguments is flawed in a way most similar to the way in which the passage is flawed?", "answers": "['All great photographers are artists. Some artists are intellectuals. Ttherefore, some great photographers are intellectuals.', 'All great photographers are artists. No artists are intellectuals. Ttherefore, some great photographers must not be intellectuals.', 'All great photographers are artists. Some great photographers are intellectuals. Ttherefore, some artists must be intellectuals.', 'All great photographers are artists. All artists are intellectuals. Ttherefore, some great photographers must be intellectuals.']", "label": 0 }, { "id": "train_2415", "context": "A business analysis of the Appenian railroad system divided its long-distance passenger routes into two categories: rural routes and interurban routes. The analysis found that, unlike the interurban routes, few rural routes carried a high enough passenger volume to be profitable. Closing unprofitable rural routes, however, will not necessarily enhance the profitability of the whole system, since__.", "question": "Which of the following most logically completes the passage?", "answers": "[\"not all of Appenia's large cities are equally well served by interurban railroad services\", 'the rural routes were all originally constructed at least one hundred years ago, whereas some of the interurban routes were constructed recently for new high-speed express trains', 'a large part of the passenger volume on interurban routes is accounted for by passengers who begin or end their journeys on rural routes', \"the greatest passenger volume, relative to the routes' capacity, is not on either category of long-distance routes but is on suburban commuter routes\"]", "label": 2 }, { "id": "train_2416", "context": "Hendry: Most employee strikes should be legally permitted. But strikes by university faculty are an exception. Faculty strikes harm a university' s students, and I accept the principle that an employee strike shouldn' t be legally permitted if it would harm the employer' s customers. Menkin: If your principle is correct, then, contrary to what you claim, employee strikes should almost never be legally permitted.", "question": "On the basis of their statements, Hendry and Menkin are committed to disagreeing over whether", "answers": "[\"a university's students should be considered customers of the university\", \"faculty strikes harm a university's students\", \"most employee strikes would harm the employer's customers\", 'most employee strikes should be legally permitted']", "label": 2 }, { "id": "train_2417", "context": "Anthropologist: Violence is an extreme form of aggression, and is distinct from the self-expression sufficient for survival under normal conditions. Human beings in certain situations react to unpleasant stimuli with violence -- but only because they are conditioned by their culture to react in this manner.", "question": "Each of the following can be logically inferred from the anthropologist's statements EXCEPT:", "answers": "[\"Violent behavior is a product of one's cultural environment.\", 'The self-expression required for survival is generally nonagressive.', 'Some behaviors are influenced by the cultures in which human beings live.', 'Not all aggression is violent.']", "label": 1 }, { "id": "train_2418", "context": "Recently, highly skilled workers in Eastern Europe have left jobs in record numbers to emigrate to the West. It is ttherefore likely that skilled workers who remain in Eastern Europe are in high demand in their home countries.", "question": "Which one of the following, if true, most seriously weakens the argument?", "answers": "['Eastern European factories prefer to hire workers from their home countries rather than to import workers from abroad.', 'Because of the departure of skilled workers from Eastern European countries, many positions are now unfilled.', 'Major changes in Eastern European economic structures have led to the elimination of many positions previously held by the highly skilled emigrants.', 'Eastern European countries plan to train many new workers to replace the highly skilled workers who have emigrated.']", "label": 2 }, { "id": "train_2419", "context": "During the day in Lake Constance, the zooplankton D. hyalina departs for the depths where food is scarce and the water cold. D. galeata remains near the warm surface where food is abundant. Even though D. galeata grows and reproduces much faster, its population is often outnumbered by D. hyalina.", "question": "Which of the following, if true, would help resolve the apparent paradox presented above?", "answers": "[\"D. galeata clusters under vegetation during the hottest part of the day to avoid the Sun's rays.\", 'The number of species of zooplankton living at the bottom of the lake is twice that of species living at the surface.', 'In order to make the most of scarce food resources, D. hyalina matures more slowly than D. galeata.', 'Predators of zooplankton, such as whitefish and perch, live and feed near the surface of the lake during the day.']", "label": 3 }, { "id": "train_2420", "context": "Until now, only injectable vaccines against influenza have been available. Parents are reluctant to subject children to the pain of injections, but adults, who are at risk of serious complications from influenza, are commonly vaccinated. A new influenza vaccine, administered painlessly in a nasal spray, is effective for children. However, since children seldom develop serious complications from influenza, no significant public health benefit would result from widespread vaccination of children using the nasal spray.", "question": "Which of the following is an assumption on which the argument depends?", "answers": "['The injectable vaccine is affordable for all adults.', 'Adults do not contract influenza primarily from children who have influenza.', 'Any person who has received the injectable vaccine can safely receive the nasal-spray vaccine as well.', 'The new vaccine uses the same mechanism to ward off influenza as injectable vaccines do.']", "label": 1 }, { "id": "train_2421", "context": "To produce seeds, plants must first produce flowers. Two kinds of tarragon plants, Russian tarragon and French tarragon, look very similar except that Russian tarragon produces flowers and French tarragon does not. The leaves of Russian tarragon, however, lack the distinctive flavor that makes French tarragon a desirable culinary herb.", "question": "If the information presented is true, which of the following can most reliably be concluded on the basis of it?", "answers": "['The flowers of Russian tarragon plants are probably not flavorful.', 'There are no other kinds of tarragon besides Russian tarragon and French tarragon', 'Garden plants that have flavorful leaves generally do not produce flowers.', 'Plants that grow from seeds sold in a packet labeled \"tarragon\" are not French tarragon.']", "label": 3 }, { "id": "train_2422", "context": "Voters commonly condemn politicians for being insincere, but politicians often must disguise their true feelings when they make public statements. If they expressed their honest views-about, say, their party's policies-then achieving politically necessary compromises would be much more difficult. Clearly, the very insincerity that people decry shows that our government is functioning well.", "question": "Which of the following, if true, most seriously undermines this reasoning?", "answers": "[\"A political party's policies could turn out to be detrimental to the functioning of a government.\", 'Some political compromises are not in the best long-term interest of the government.', \"Some of the public statements made by politicians about their party's policies could in fact be sincere.\", 'Achieving political compromises is not all that is necessary for the proper functioning of a government.']", "label": 3 }, { "id": "train_2423", "context": "Alcohol consumption has been clearly linked to high blood pressure, which increases the likelihood of developing heart disease. Yet in a study of the effects of alcohol consumption, the incidence of heart disease was lower among participants who drank moderate quantities of alcohol every day than it was among participants identified as nondrinkers.", "question": "Which one of the following, if true, most helps to resolve the apparent discrepancy in the information above?", "answers": "['Some of the participants who drank moderate quantities of alcohol every day had high blood pressure.', 'Many of the participants identified as nondrinkers were people who had been heavy drinkers but had stopped drinking alcohol prior to participating in the study.', \"The two groups of participants were similar to each other with respect to the participants' age, sex, geographical origin, and economic background.\", 'Because many people who do not drink alcohol are conscious of their health habits, they are likely to engage in regular exercise and to eat nutritionally well-balanced meals.']", "label": 1 }, { "id": "train_2424", "context": "To reduce the frequency of ground collisions, Innovair will begin giving its ground crews additional training, at an annual cost of $500, 000. Although this will cut the number of ground collisions by about half at best, the drop in repair costs can be expected to be much greater, since __ .", "question": "Which of the following most logically completes the argument? In a typical year, Innovair's airplanes are involved in 35 collisions while parked or being towed in airports, with a resulting yearly cost of $1, 000, 000 for repairs.", "answers": "['most ground collisions have been caused by the least experienced ground-crew members', 'a ground collision typically occurs when there are no passengers on the airplane', 'the additional training will focus on helping ground crews avoid those kinds of ground collisions that cause the most costly damage', 'the $500, 000 cost figure for the additional training of ground crews includes the wages that those crews will earn during the time spent in actual training']", "label": 2 }, { "id": "train_2425", "context": "People who take what others regard as a ridiculous position should not bother to say, \"I mean every word! \" For either their position truly is ridiculous, in which case insisting that they are serious about it only exposes them to deeper embarrassment, or else their position has merit, in which case they should meet disbelief with rational argument rather than with assurances of their sincerity.", "question": "Which one of the following arguments is most similar in its reasoning to the argument above?", "answers": "['People whose taste in clothes is being criticized should not reply, \"Every penny I spent on these clothes I earned honestly. \" For the issue raised by the critics is not how the money was come by but rather whether it was spent wisely.', 'Scholars who champion unpopular new theories should not assume that the widespread rejection of their ideas shows that they \"must be on the right track. \" The truth is that few theories of any consequence are either wholly right or wholly wrong and thus there is no substitute for patient work in ascertaining which parts are right.', 'A practice that has been denounced as a poor practice should not be defended on the grounds that \"this is how we have always done it. \" If the practice is a poor one, so much the worse that it has been extensively used; if it is not a poor one, there must be a better reason for engaging in it than inertia.', 'People who are asked why they eat some of the unusual foods they eat should not answer, \"because that is what I like. \" This sort of answer will sound either naive or evasive and thus will satisfy no one.']", "label": 2 }, { "id": "train_2426", "context": "Cable-television spokesperson: Subscriptions to cable television are a bargain in comparison to \"free\" television. Remember that \"free\" television is not really free. It is consumers, in the end, who pay for the costly advertising that supports \"free\" television.", "question": "Which of the following, if true, is most damaging to the position of the cable-television spokesperson?", "answers": "['Consumers who do not own television sets are less likely to be influenced in their purchasing decisions by television advertising than are consumers who own television sets.', 'Cable-television subscribers can choose which channels they wish to receive.', 'Subscriptions to cable television include access to some public-television channels, which do not accept advertising.', 'There is as much advertising on many cable television channels as there is on \"free\" television channels.']", "label": 3 }, { "id": "train_2427", "context": "Several recent studies establish that most people would not quit their jobs if they won the lottery. In each study, over 80 percent of the people surveyed indicated that they would not quit their jobs.", "question": "Each of the following, if true, weakens the argument EXCEPT:", "answers": "['The people surveyed were all college students in first-year business courses.', 'In another recent study, the majority of respondents stated that they would quit their jobs as soon as they found out they had won the lottery.', 'People rarely tell the truth when responding to such surveys.', 'Several of the researchers performing the surveys had no background in business.']", "label": 3 }, { "id": "train_2428", "context": "Professor: A person who can select a beverage from among 50 varieties of cola is less free than one who has only these 5 choices: wine, coffee, apple juice, milk, and water. It is clear, then, that meaningful freedom cannot be measured simply by the number of alternatives available; the extent of the differences among the alternatives is also a relevant factor.", "question": "The professor's argument proceeds by", "answers": "['inferring one general principle from another, more general, principle', 'supporting its conclusion by means of an analogy', 'supporting a general principle by means of an example', 'claiming that whatever holds for each member of a group must hold for the whole group']", "label": 2 }, { "id": "train_2429", "context": "Ethicist: The penalties for drunk driving are far more severe when the drunk driver accidentally injures people than when no one is injured. Moral responsibility for an action depends solely on the intentions underlying the action and not on the action' s results. Ttherefore, legal responsibility, depending as it does in at least some cases on factors other than the agent' s intentions, is different than moral responsibility.", "question": "The claim that the penalties for drunk driving are far more severe when the drunk driver accidentally injures people than when no one is injured plays which one of the following roles in the ethicist's argument?", "answers": "[\"It is a premise offered in support of the claim that moral responsibility depends solely on the intentions underlying the action and not on the action's result.\", 'It is a premise offered in support of the claim that legal responsibility for an action is based solely upon features of the action that are generally unintended by the agent.', \"It is a premise offered in support of the claim that legal responsibility depends in at least some cases on factors other than the agent's intentions.\", 'It is offered as an illustration of the claim that people may be held morally responsible for an action for which they are not legally responsible.']", "label": 2 }, { "id": "train_2430", "context": "Scientists hypothesized that the end of a certain geological period came about when a cataclysmic meteor impact near the equator caused debris to fly up into the atmosphere, where it was scattered by winds, blocking out the sunlight and abruptly killing off many species. In European rock layers, the fossil record of the boundary layer that marks the end of the period supports this picture of rapid, mass extinction of life. But the boundary layer in the Antarctic is much thicker, so that it appears that the extinction process there lasted a much longer time, contrary to what the hypothesis would have predicted.", "question": "Which one of the following, if true, resolves the apparent discrepancy noted above?", "answers": "['It was recently discovered that Europe accumulated sediment at a slower rate than did the Antarctic region, so that a layer of sediment of a given thickness represents a longer period of time in Europe than it does in the Antarctic.', 'Although scientists have traditionally used the last appearance of creatures called ammonites to define the boundary layer, they recently found ammonites 20 meters above the previously identified boundary layer in the Antarctic.', 'The findings in Europe agree with data about the boundary layer from all other mid-latitude sites.', 'The rock layers in the Antarctic were examined many years after the rock layers in Europe.']", "label": 0 }, { "id": "train_2431", "context": "All known deposits of the mineral tanzanite are in Tanzania. Ttherefore, because Ashley collects only tanzanite stones, she is unlikely ever to collect a stone not originally from Tanzania.", "question": "Which one of the following is most similar in its reasoning to the argument above?", "answers": "['Frogs are the only animals known to live in the lagoon on Scrag Island. The diet of the owls on Scrag Island consists of nothing but frogs from the island. Ttherefore, the owls are unlikely ever to eat an animal that lives outside the lagoon.', 'Every frog ever seen on Scrag Island lives in the lagoon. The frogs on the island are eaten only by the owls on the island, and hence the owls may never eat an animal that lives outside the lagoon.', 'The lagoon on Scrag Island is home to many frogs. Since the owls on Scrag Island eat nothing but frogs from the island, the owls will probably never eat many frogs that live outside the lagoon.', 'The only frogs yet discovered on Scrag Island live in the lagoon. The diet of all the owls on Scrag Island consists entirely of frogs on the island, so the owls will probably never eat an animal that lives outside the lagoon.']", "label": 3 }, { "id": "train_2432", "context": "A letter submitted to the editor of a national newsmagazine was written and signed by a Dr. Shirley Martin who, in the text of the letter, mentions being a professor at a major North American medical school. Knowing that fewer than 5 percent of the professors at such schools are women, the editor reasons that the chances are better than 19 to 1 that the letter was written by a man.", "question": "Which one of the following involves flawed reasoning most like that used by the editor?", "answers": "['Since more than 19 out of every 20 animals in the wildlife preserve are mammals and fewer than 1 out of 20 are birds, there is a greater than 95 percent chance that the animal Emily saw flying between two trees in the wildlife refuge yesterday morning was a mammal.', \"More than 95 percent of the planes built by UBC last year met government standards for large airliners. Since small planes account for just under 5 percent of UBC's output last year, it is almost certain that all their large planes met government standards.\", 'Since 19 out of 20 home computers are purchased primarily for use with computer games, and the first computer sold today was purchased solely for word processing, the next 19 computers sold will almost certainly be used primarily for computer games.', 'Fewer than 1 in 20 of the manuscripts submitted to Argon Publishing Co. are accepted for publication. Since only 15 manuscripts were submitted last week, there is almost no chance that any of them will be accepted for publication.']", "label": 0 }, { "id": "train_2433", "context": "A person with a type B lipid profile is at much greater risk of heart disease than a person with a type A lipid profile. In an experiment, both type A volunteers and type B volunteers were put on a low-fat diet. The cholesterol levels of the type B volunteers soon dropped substantially, although their lipid profiles were unchanged. The type A volunteers, however, showed no benefit from the diet, and 40 percent of them actually shifted to type B profiles.", "question": "If the information above is true, which one of the following must also be true?", "answers": "['The reduction in cholesterol levels in the volunteers is solely responsible for the change in their lipid profiles.', 'Apart from adopting the low-fat diet, most of the volunteers did not substantially change any aspect of their lifestyle that would have affected their cholesterol levels or lipid profiles.', 'For at least some of the volunteers in the experiment, the risk of heart disease increased after having been put on the low-fat diet.', 'In the experiment, most of the volunteers had their risk of heart disease reduced at least marginally as a result of having been put on the diet.']", "label": 2 }, { "id": "train_2434", "context": "Conventional wisdom suggests vaccinating elderly people first in flu season, because they are at greatest risk of dying if they contract the virus. This year' s flu virus poses particular risk to elderly people and almost none at all to younger people, particularly children. Nevertheless, health professionals are recommending vaccinating children first against the virus rather than elderly people.", "question": "Which of the following, if true, provides the strongest reason for the health professionals' recommendation?", "answers": "['Children who catch one strain of the flu virus and then recover are likely to develop immunity to at least some strains with which they have not yet come in contact.', 'Children are particularly unconcerned with hygiene and ttherefore are the group most responsible for spreading the flu virus to others.', 'Children are no more likely than adults to have immunity to a particular flu virus if they have never lived through a previous epidemic of the same virus.', 'Children are vulnerable to dangerous infections when their immune systems are severely weakened by other diseases.']", "label": 1 }, { "id": "train_2435", "context": "In the past the country of Malvernia has relied heavily on imported oil. Malvernia recently implemented a program to convert heating systems from oil to natural gas. Malvernia currently produces more natural gas each year than it uses, and oil production in Malvernian oil fields is increasing at a steady pace. If these trends in fuel production and usage continue, ttherefore, Malvernian reliance on foreign sources for fuel is likely to decline soon.", "question": "Which of the following would it be most useful to establish in evaluating the argument?", "answers": "['Have any existing oil-burning heating systems in Malvernia already been converted to natural-gas-burning heating systems?', \"What proportion of Malvernia's total energy needs is met by hydroelectric, solar, and nuclear power?\", 'Is the amount of oil used each year in Malvernia for generating electricity and fuel for transportation increasing?', 'Is Malvernia among the countries that rely most on imported oil?']", "label": 2 }, { "id": "train_2436", "context": "The cause of the wreck of the ship Edmund Fitzgeraid in a severe storm on lake Superior is still unknown , when the sunken wreckage of the vessel was found , searchers discovered the hull in two pieces lying close together , The storm' s violent waves would have caused separate pieces floating even briefly on the surface to drift apart. Ttherefore, the breakup of the hull can be ruled out as the cause of the sinking.", "question": "Which of the following is an assumption on which the argument depends?", "answers": "['If the ship broke up before sinking , the pieces of the hull would not have remained on the surface for very long', 'Under water currents at the time of the storm did not move the separated pieces of the hull together again .', 'Ships as large as the Edmund Fitzgerald rarely sink except in the most violent weather.', 'Pieces of the hull would have sunk more quickly than the intact hull would have']", "label": 1 }, { "id": "train_2437", "context": "The photographs that the store developed were quite unsatisfactory. The customer claims to have handled the film correctly. Neither the film nor the camera was defective. If a store does not process pictures properly, the customer is owed a refund, so if the customer' s claim is correct, the store owes her a refund.", "question": "The argument relies on assuming which one of the following?", "answers": "['If neither the film nor the camera was defective, and the customer handled the film correctly, then the store processed it improperly.', 'If pictures are taken with a defective camera, then it is not possible for the store to develop those pictures improperly.', \"If the customer's claim was not correct, then the store does not owe her a refund.\", 'If the customer handled the film incorrectly, that is what caused the photographs that the store developed to be unsatisfactory.']", "label": 0 }, { "id": "train_2438", "context": "Anthropologist: Every human culture has taboos against eating certain animals. Some researchers have argued that such taboos originated solely for practical reasons, pointing out, for example, that in many cultures it is taboo to eat domestic animals that provide labor and that are ttherefore worth more alive than dead. But that conclusion is unwarranted; taboos against eating certain animals might instead have arisen for symbolic, ritualistic reasons, and the presence of the taboos might then have led people to find other uses for those animals.", "question": "In the argument, the anthropologist", "answers": "['calls an explanation of a phenomenon into question by pointing out that observations cited as evidence supporting it are also compatible with an alternative explanation of the phenomenon', 'describes a hypothesis about the sequence of events involved in the origins of a phenomenon, and then argues that those events occurred in a different sequence', 'establishes that an explanation of a phenomenon is false by demonstrating that the evidence that had been cited in support of that explanation was inadequate', 'argues in support of one explanation of a phenomenon by citing evidence incompatible with a rival explanation']", "label": 0 }, { "id": "train_2439", "context": "John works five days each week except when on vacation or during weeks in which national holidays occur. Four days a week he works in an insurance company; on Fridays he works as a blacksmith. Last week there were no holidays, and John was not on vacation. Ttherefore, he must have worked in the insurance company on Monday, Tuesday, Wednesday, and Thursday last week.", "question": "Which one of the following is an assumption on which the argument depends?", "answers": "['Every day last week that John worked, he worked for an entire workday.', 'John does not take vacations in weeks in which national holidays occur.', 'John never takes a vacation of more than one week in length.', 'Last week John worked neither on Saturday nor on Sunday.']", "label": 3 }, { "id": "train_2440", "context": "The only songs Amanda has ever written are blues songs and punk rock songs. Most punk rock songs involve no more than three chords. So if the next song Amanda writes is not a blues song, it probably will not involve more than three chords.", "question": "The reasoning in which one of the following arguments is most similar to that in the argument above?", "answers": "['The only pets the Gupta family has ever owned are fish and parrots. Most parrots are very noisy. So if the next pet the Gupta family owns is a parrot, it will probably be very noisy.', 'All the pets the Gupta family has ever owned have been fish and parrots. Most parrots are very noisy. So any pet the Gupta family ever owns that is not a fish will probably be very noisy.', 'Every pet the Gupta family has ever owned has been a fish or a parrot. Most parrots are very noisy. So if the next pet the Gupta family owns is not a parrot, it will probably not be very noisy.', 'The Gupta family has never owned any pets other than fish and parrots. Most parrots are very noisy. So the next pet the Gupta family owns will probably be very noisy if it is not a fish.']", "label": 3 }, { "id": "train_2441", "context": "Traditionally, the foundations of houses were made of concrete. A builder is experimenting with using a hard plastic polymer for foundations. A foundation made with the polymer is about the same price to construct and install as is concrete foundation. Unlike concrete, the polymer will block all water flow and is not prone to cracking with age. The builder argues that houses built with the polymer foundation will last longer.", "question": "Which of the following must be studied in order to evaluate the argument presented above?", "answers": "['Whether consumers seeking to buy a house will trust a house with an alternate foundation.', 'Whether soil acidity, to which all stone is resistant, can dissolve the polymer.', 'How dry the soil in this region becomes during typical summer droughts.', 'Whether repairs to polymer foundations are more expensive than repairs to concrete foundations.']", "label": 1 }, { "id": "train_2442", "context": "For the first few weeks after birth, the dunnart has such poor control over its respiratory muscles that it cannot use them to breathe. Instead, this tiny marsupial breathes through its thin skin, which gradually thickens as the dunnart matures inside its mother' s pouch. The dunnart is unique among warm-blooded animals, the rest of which need thick skin throughout their lives to maintain body temperature and reduce water loss.", "question": "Which one of the following, if true, most helps explain how newborn dunnarts manage to survive despite their thin skins?", "answers": "['Adult dunnarts experience more heat and water loss through their skin than other adult marsupials do.', \"The dunnart's body temperature is higher than that of many other warm-blooded animals.\", \"Its mother's pouch keeps a newborn dunnart warm and reduces water loss through its skin.\", \"The dunnart's respiratory muscles begin to develop a few days after birth.\"]", "label": 2 }, { "id": "train_2443", "context": "Vitamin XYZ has long been a favorite among health food enthusiasts. In a recent large study, those who took large amounts of vitamin XYZ daily for two years showed on average a 40 percent lower risk of heart disease than did members of a control group. Researchers corrected for differences in relevant health habits such as diet.", "question": "Which one of the following inferences is most supported by the passage?", "answers": "[\"Taking large amounts of vitamin XYZ daily over the course of one's adult life should be recommended to most adults.\", 'Health food enthusiasts are probably correct in believing that large daily doses of multiple vitamins promote good health.', 'Those who take large doses of vitamin XYZ daily for the next two years will exhibit on average an increase in the likelihood of avoiding heart disease.', 'Taking large amounts of vitamins is probably worth risking the side effects.']", "label": 2 }, { "id": "train_2444", "context": "A recent study of several hundred female physicians showed that their tendency to develop coronary disease was inversely proportional to their dietary intake of two vitamins, folate and B6. The researchers concluded that folate and B6 inhibit the development of heart disease in women.", "question": "Which one of the following would, if true, most weaken the researchers'conclusion?", "answers": "['Physicians are more likely than nonphysicians to know a great deal about the link between diet and health.', 'The foods that contain significant amounts of the vitamins folate and B6 also contain significant amounts of nonvitamin nutrients that inhibit heart disease.', 'The physicians in the study had not been screened in advance to ensure that none had preexisting heart conditions.', 'It is very unlikely that a chemical compound would inhibit coronary disease in women but not in men.']", "label": 1 }, { "id": "train_2445", "context": "Psychologist: Some astrologers claim that our horoscopes completely determine our personalities, but this claim is false. I concede that identical twins -- who are, of course, born at practically the same time -- often do have similar personalities. However, birth records were examined to find two individuals who were born 40 years ago on the same day and at exactly the same time -- one in a hospital in Toronto and one in a hospital in New York. Personality tests revealed that the personalities of these two individuals are in fact different.", "question": "Which one of the following is an assumption on which the psychologist's argument depends?", "answers": "['The personality differences between the two individuals cannot be explained by the cultural differences between Toronto and New York.', 'Astrologers have not subjected their claims to rigorous experimentation.', 'Complete birth records for the past 40 years were kept at both hospitals.', 'The geographical difference between Toronto and New York did not result in the two individuals having different horoscopes.']", "label": 3 }, { "id": "train_2446", "context": "In response to mounting public concern, an airplane manufacturer implemented a program with the well-publicized goal of reducing by half the total yearly amount of hazardous waste generated by its passenger-jet division. When the program began in 1994, the division' s hazardous waste output was 90 pounds per production worker; last year it was 40 pounds per production worker. Clearly, ttherefore, charges that the manufacturer' s program has not met its goal are false.", "question": "Which of the following is an assumption on which the argument depends?", "answers": "['The number of production workers assigned to the passenger-jet division was not significantly less in 1994 than it was last year.', 'The average number of weekly hours per production worker in the passenger-jet division was not significantly greater last year than it was in 1994.', 'Since 1994, other divisions in the company have achieved reductions in hazardous waste output that are at least equal to that achieved in the passenger-jet division.', 'The amount of nonhazardous waste generated each year by the passenger-jet division has not increased significantly since 1994.']", "label": 0 }, { "id": "train_2447", "context": "Philosopher: An action is morally right if it would be reasonably expected to increase the aggregate well-being of the people affected by it. An action is morally wrong if and only if it would be reasonably expected to reduce the aggregate wellbeing of the people affected by it. Thus, actions that would be reasonably expected to leave unchanged the aggregate well-being of the people affected by them are also right.", "question": "The philosopher's conclusion follows logically if which one of the following is assumed?", "answers": "['There are actions that would be reasonably expected to leave unchanged the aggregate well-being of the people affected by them.', 'Any action that is not morally wrong is morally right.', 'Only right actions have good consequences.', 'Only wrong actions would be reasonably expected to reduce the aggregate well-being of the people affected by them.']", "label": 1 }, { "id": "train_2448", "context": "Zoologist: Plants preferentially absorb heavy nitrogen from rainwater. Heavy nitrogen consequently becomes concentrated in the tissues of herbivores, and animals that eat meat in turn exhibit even higher concentrations of heavy nitrogen in their bodily tissues. We compared bone samples from European cave bears of the Ice Age with blood samples from present-day bears fed meat-enriched diets, and the levels of heavy nitrogen present in these samples were identical. Thus, the prehistoric European cave bears were not exclusively herbivores.", "question": "Which one of the following, if true, would most strengthen the zoologist's argument?", "answers": "['Bone samples from present-day bears fed meat-enriched diets exhibit the same levels of heavy nitrogen as do their blood samples.', 'Plants can also absorb heavy nitrogen from a variety of sources other than rainwater.', 'The level of heavy nitrogen in the bones of any bear fed a meat-enriched diet is the same as that in the bones of any other meat-eating bear.', 'The same number of samples was taken from present-day bears as was taken from Ice Age cave bears.']", "label": 0 }, { "id": "train_2449", "context": "The total number of book titles published annually in North America has approximately quadrupled since television first became available. Retail sales of new titles, as measured in copies, increased rapidly in the early days of television, though the rate of increase has slowed in recent years. Library circulation has been flat or declining in recent years.", "question": "Which one of the following is most strongly supported by the information above?", "answers": "['Television has, over the years, brought about a reduction in the amount of per capita reading in North America.', 'Book publishers in North America now sell fewer copies per title than they sold in the early days of television.', 'The availability of television does not always cause a decline in the annual number of book titles published or in the number of books sold.', 'The introduction of television usually brings about a decrease in library use.']", "label": 2 }, { "id": "train_2450", "context": "The radio station claims that its new format is popular with listeners because more than three-quarters of the listeners who call in requests to the station say they are pleased with the format. This, however, is hardly conclusive. It would be like trying to determine whether a political candidate is popular with voters by interviewing only those people who have already decided to vote for the candidate.", "question": "The argument proceeds by", "answers": "['claiming that an inference leads to a contradiction in order to show that the inference is unreasonable', 'referring to an inference that is clearly flawed in order to undermine an analogous inference', 'questioning the legitimacy of an inference by proposing a more reasonable inference that could be drawn from the evidence', 'providing a direct counterexample to a conclusion in order to show that the conclusion is false']", "label": 1 }, { "id": "train_2451", "context": "Professor: The most important function of epic poetry is to transmit the values by which a group of people is to live. This transmission is accomplished not by an explicit discussion of those values, but rather by their embodiment in heroic figures, who are presented as role models. Imitating those role models gives meaning and direction to the lives of those who hear the poems.", "question": "If the professor's statements are true, which one of the following must also be true?", "answers": "['An important function of poetry is to give meaning and direction to the lives of those who hear or read it.', 'Only epic poetry presents heroic figures as role models that, if imitated, give meaning and direction to the lives of those who hear it.', 'For many groups of people, heroic figures serve as role models embodying the values by which those people are to live.', 'Epic poems accomplish their most important function by presenting heroic figures as role models.']", "label": 3 }, { "id": "train_2452", "context": "The experts who now assure us that genetically engineered plants are harmless are the same experts who claimed that introducing non-native plants into the wild was a good idea. It is too late to be skeptical now that some non-native plants have become a serious problem. But we should conclude, while we still have time, that genetically engineered plants will also be harmful.", "question": "The flawed reasoning in which one of the following most closely parallels the flawed reasoning in the argument above?", "answers": "['Children who beg and plead with their parents to buy them trendy toys are the same children who begged for trendy toys last year. I conclude that parents ought to ignore such pleadings and simply buy traditional toys.', \"The film critics who called Meisner's last film brilliant are the same ones who are calling her new film confused and boring. But because Meisner's last film was excellent I conclude that this one will be also.\", 'The same people who complain that taxes are too high complain that the government does not provide enough services. We should conclude that high taxes and big government go together.', 'The economists who tell us that the current economic expansion will soon be over are the same economists who failed to predict the end of the last recession. Wise investors will conclude that the expansion will continue for some time.']", "label": 3 }, { "id": "train_2453", "context": "Economist: Prosperity is a driving force behind increases in the release of carbon dioxide, the main cause of global warming. As incomes rise, more people spend money on energy-consuming devices such as cars, thereby producing more carbon dioxide. Also, in countries that experienced deep economic recessions, there were steep drops in carbon dioxide emissions.", "question": "Which one of the following most accurately states the overall conclusion drawn in the economist's argument?", "answers": "['When incomes rise, more people spend money on energy-consuming devices.', 'When people spend money on energy-consuming devices, more carbon dioxide is produced as a result.', 'Prosperity is an important cause of increases in the release of carbon dioxide.', 'Carbon dioxide is the main cause of global warming.']", "label": 2 }, { "id": "train_2454", "context": "Because the recycling of glass is not typically cost-effective, there is a shortage of glass recycling facilities and glass recycling is not an obligation we all bear. So some people ought not to recycle glass.", "question": "Which one of the following arguments contains a flaw in reasoning most similar to that in the argument above?", "answers": "['It is false that you should break your promise, so you do not have an obligation to break your promise.', 'Some convention participants do not have to register on the first day of the convention since their flights were delayed past midnight. So some participants should not register on the first day of the convention.', 'Because operating museums is so expensive, the government can no longer afford to support them. So private foundations that can afford to do so have an obligation to provide the financial support that the government previously provided.', 'Anyone who rides a bicycle at night ought not to ride on a highway. So we ought not to bike on the highway during our trip today.']", "label": 1 }, { "id": "train_2455", "context": "Astrophysicist: Gamma ray bursts (GRBs) -- explosions of powerful radiation from deep space -- have traditionally been classified as either \"short\" or \"long, \" terms that reflect the explosion' s relative duration. However, an unusual GRB has been sighted. Its duration was long, but in every other respect it had the properties of a short GRB. Clearly, the descriptive labels \"short\" and \"long\" have now outlived their usefulness.", "question": "The conclusion of the astrophysicist's argument is most strongly supported if which one of the following is assumed?", "answers": "['No other GRBs with unusual properties have been sighted.', 'GRBs cannot be classified according to the different types of cosmic events that create them.', 'Properties other than duration are more important than duration in the proper classification of the unusual GRB.', 'Descriptive labels are easily replaced with nondescriptive labels such as \"type I\" and \"type II. \"']", "label": 2 }, { "id": "train_2456", "context": "Designer: Any garden and adjoining living room that are separated from one another by sliding glass doors can visually merge into a single space. If the sliding doors are open, as may happen in summer, this effect will be created if it does not already exist and intensified if it does. The effect remains quite strong during colder months if the garden is well coordinated with the room and contributes strong visual interest of its own.", "question": "The designer's statements, if true, most strongly support which one of the following?", "answers": "['A garden separated from an adjoining living room by closed sliding glass doors cannot be well coordinated with the room unless the garden contributes strong visual interest.', 'A garden and an adjoining living room separated by sliding glass doors cannot visually merge in summer unless the doors are open.', 'Except in summer, opening the sliding glass doors that separate a garden from an adjoining living room does not intensify the effect of the garden and room visually merging into a single space.', 'A garden can visually merge with an adjoining living room into a single space even if the garden does not contribute strong visual interest of its own.']", "label": 3 }, { "id": "train_2457", "context": "The local fair held its annual photography contest and accepted entries from both amateurs and professionals. The contest awarded prizes in each of several categories. As it turned out, most of the prizes in the contest were won by amateurs.", "question": "Each of the following, if true, could by itself constitute an explanation of the outcome of the photography contest EXCEPT:", "answers": "['Amateurs tend to enter their best photographs while professionals tend to save their best work for their clients.', 'Three times as many amateurs entered the contest as had entered in any previous year.', 'Many more of the entries in the contest were from amateurs than were from professionals.', 'Each category in the contest was restricted to amateurs only or professionals only, and there were more categories open to amateurs.']", "label": 1 }, { "id": "train_2458", "context": "Nutritionists have recommended that people eat more fiber. Advertisements for a new fiber-supplement pill state only that it contains \"44 percent fiber. ", "question": "The advertising claim is misleading in its selection of information on which to focus if which one of the following is true?", "answers": "['There are other products on the market that are advertised as providing fiber as a dietary supplement.', 'The recommended daily intake of fiber is 20 to 30 grams, and the pill contains one-third gram.', 'Nutritionists base their recommendation on medical findings that dietary fiber protects against some kinds of cancer.', 'The label of the advertised product recommends taking 3 pills every day.']", "label": 1 }, { "id": "train_2459", "context": "From a newspaper editorial: Many people who are addicted to heroin will eventually attempt to overcome their addiction, principally for two reasons: the expense of maintaining a heroin addiction and the fear of arrest. If heroin were legalized and made available cheaply, as some people advocate, neither of these reasons would apply.", "question": "The considerations above can best serve as part of an argument that", "answers": "['making heroin available legally and cheaply would make it less likely that heroin addicts will attempt to overcome their addiction', 'legalizing the sale of heroin would cause the price of this drug to go down', 'decreasing the severity of penalties for individuals who use heroin would not increase the number of new heroin addicts', 'legalizing the sale of heroin would increase the number of crimes committed by heroin addicts to support their addiction']", "label": 0 }, { "id": "train_2460", "context": "In all mammalian species, the period of a young mammal' s life in which it is most frequently playful coincides with the period of most rapid growth of the neural connections in the mammal' s brain that give rise to various complex patterns of movement, posture, and social response. Indeed, the neural connections created by frequent play during this period later become indispensable for the mammal' s survival and well-being as an adult.", "question": "The statements above, if true, serve LEAST well as evidence for which one of the following?", "answers": "['The young of nonmammalian species such as fish, reptiles, and birds do not normally engage in playful behavior that serves the function served by play in the development of young mammals.', 'Young mammals of species that are preyed on by other animals are likely to engage in forms of sudden mock flight, bolting away from locations where no predators are to be found.', 'Young mammals of predatory species tend to practice in their play inoffensive versions of motions and actions that are useful in finding and catching prey when these mammals become adults.', 'Adult mammals are more likely to engage in interactive play with their young if they engaged in similar forms of play when they themselves were young.']", "label": 0 }, { "id": "train_2461", "context": "Musicologist: Many critics complain of the disproportion between text and music in Handel' s da capo arias. These texts are generally quite short and often repeated well beyond what is needed for literal understanding. Yet such criticism is refuted by noting that repetition serves a vital function: it frees the audience to focus on the music itself, which can speak to audiences whatever their language.", "question": "Which one of the following sentences best expresses the main point of the musicologist's reasoning?", "answers": [ "At least one frequent criticism of Handel's da capo arias is undeserved.", "Handel's da capo arias are superior to most in their accessibility to diverse audiences.", "Most criticism of Handel's da capo arias is unwarranted.", "Handel's da capo arias contain a disproportionate amount of music." ], "label": 0 }, { "id": "train_2462", "context": "Most doctors recommend that pregnant women eat a nutritious diet to promote the health of their babies. However, most babies who are born to women who ate nutritious diets while pregnant still develop at least one medical problem in their first year.", "question": "Which one of the following, if true, does most to resolve the apparent discrepancy in the information above?", "answers": "['Women who regularly eat a nutritious diet while pregnant tend to eat a nutritious diet while breast-feeding.', 'Many of the physicians who initially recommended that pregnant women consume nutritious diets have only recently reaffirmed their recommendation.', 'Babies of women who did not eat nutritious diets while pregnant tend to have more medical problems in their first year than do other babies.', 'Medical problems that develop in the first year of life tend to be more serious than those that develop later in childhood.']", "label": 2 }, { "id": "train_2463", "context": "In many families adults speak to babies in simplified language . Yet constantly repeating simple phrases like \"Nice kitty. See the kitty? \" does not provide extra help to children in learning a language. We know this because there are families in which no one speaks to babies this way, yet the children in these families master the grammatical structure of their language just as well and as quickly as other children do.", "question": "Which one of the following is an assumption on which the argument depends?", "answers": "['Any child who has mastered the grammatical structure of a language has learned the language.', 'Many linguists believe that speaking to babies in simplified language helps the babies learn language.', 'Speaking to babies in simplified language could impair their language learning.', 'To learn a language one must acquire its vocabulary as well as its grammatical structure.']", "label": 0 }, { "id": "train_2464", "context": "In contemplating major purchases, businesses often consider only whether there is enough money left from monthly revenues after paying monthly expenses to cover the cost of the purchase. But many expenses do not occur monthly; taking into account only monthly expenses can cause a business to overexpand. So the use of a cash-flow statement is critical for all businesses.", "question": "Which one of the following, if true, most strengthens the argument?", "answers": "['Only a cash-flow statement can accurately document all monthly expenses.', 'A cash-flow statement is the only way to track both monthly expenses and expenses that are not monthly.', 'Any business that has overexpanded can benefit from the use of a cash-flow statement.', 'When a business documents only monthly expenses it also documents only monthly revenue.']", "label": 1 }, { "id": "train_2465", "context": "People who are red / green color-blind cannot distinguish between green and brown. Gerald cannot distinguish between green and brown. Ttherefore Gerald is red / green color-blind.", "question": "Which one of the following most closely parallels the reasoning in the argument presented in the passage?", "answers": "['People who are fair-skinned suffer from sunburn. William is fair-skinned. Ttherefore William suffers from sunburn.', 'People who are suffering from sinusitis lose their sense of smell. Mary has lost her sense of smell. Ttherefore Mary is suffering from sinusitis.', 'People who are diabetic cannot eat large amounts of sugar. Freda is diabetic. Ttherefore Freda is on a special diet.', 'People who are color-blind cannot become airline pilots. Arthur is color-blind. Ttherefore Arthur cannot become an airline pilot.']", "label": 1 }, { "id": "train_2466", "context": "The end of an action is the intended outcome of the action and not a mere by-product of the action, and the end' s value is thus the only reason for the action. So while it is true that not every end' s value will justify any means, and even, perhaps, that there is no end whose value will justify every means, it is clear that nothing will justify a means except an end' s value.", "question": "Which one of the following most accurately expresses the main conclusion of the argument?", "answers": "['One can always justify a given action by appeal to the value of its intended outcome.', 'Only the value of the by-products of an action can justify that action.', 'One can justify an action only by appeal to the value of its intended outcome.', \"Nothing can justify the intended outcome of an action except the value of that action's actual outcomes.\"]", "label": 2 }, { "id": "train_2467", "context": "Manuscripts written by first-time authors generally do not get serious attention by publishers except when these authors happen to be celebrities. My manuscript is unlikely to be taken seriously by publishers for I am a first-time author who is not a celebrity.", "question": "The structure of which one of the following arguments is most similar to the structure of the argument above?", "answers": "['Fruit salad that contains bananas is ordinarily a boring dish unless it contains two or more exotic fruits. This fruit salad has bananas in it, and the only exotic fruit it has is guava. Thus, it will probably be boring.', 'Normally about 40 percent of the deer population will die over the winter unless it is extremely mild. The percentage of the deer population that died over the recent winter was the normal 40 percent. I conclude that the recent winter was not unusually mild.', 'The bulk of an estate generally goes to the spouse, if surviving, and otherwise goes to the surviving children. In this case there is no surviving spouse; hence the bulk of the estate is likely to go to the surviving children.', 'Challengers generally do not win elections unless the incumbent has become very unpopular. The incumbent in this election has become very unpopular. Ttherefore, the challenger may win.']", "label": 0 }, { "id": "train_2468", "context": "Shelton: The recent sharp decline in the number of moose in this region was caused by a large increase in the white-tailed deer population. While the deer do not compete with moose for food, they carry a dangerous parasite that can be transferred to any moose living nearby. Russo: The neighboring region has also experienced a large increase in the white-tailed deer population, but the moose population there has remained stable.", "question": "Which one of the following, if true, most helps to resolve the apparent conflict between Shelton's and Russo's statements?", "answers": "['There is a large overlap in the ranges of moose and white-tailed deer in the region with the declining moose population, but not in the neighboring region.', 'Moose require a habitat with very little human settlement, whereas white-tailed deer often thrive in and around areas with considerable human settlement.', 'The region with the declining moose population is larger than the neighboring region and, even after the decline, has more moose than the neighboring region.', 'The region with the declining moose population consists mainly of high-quality moose habitat, but the quality of moose habitat in the neighboring region is marginal.']", "label": 0 }, { "id": "train_2469", "context": "According to the last pre-election poll in Whippleton, most voters believe that the three problems government needs to address, in order of importance, air pollution, crime, and unemployment. Yet in the election, candidates from parties perceived as strongly against pollution were defeated, while those elected were all from parties with a history of opposing legislation designed to reduce pollution. These results should not be taken to indicate that the poll was inaccurate, however, since __ .", "question": "Which of the following most logically completes the passage?", "answers": "['some voters in Whippleton do not believe that pollution needs to be reduced', 'all the candidates who were elected were perceived as being stronger against both crime and unemployment than the candidates who were defeated', 'many of the people who voted in the election refused to participate in the poll', 'every candidate who was defeated had a strong antipollution record']", "label": 1 }, { "id": "train_2470", "context": "Philosopher: Scientists talk about the pursuit of truth, but, like most people, they are self-interested. Accordingly, the professional activities of most scientists are directed toward personal career enhancement, and only incidentally toward the pursuit of truth. Hence, the activities of the scientific community are largely directed toward enhancing the status of that community as a whole, and only incidentally toward the pursuit of truth.", "question": "The reasoning in the philosopher's argument is flawed because the argument", "answers": "['improperly infers that each and every scientist has a certain characteristic from the premise that most scientists have that characteristic', 'improperly draws an inference about the scientific community as a whole from a premise about individual scientists', 'presumes, without giving justification, that the aim of personal career enhancement never advances the pursuit of truth', 'illicitly takes advantage of an ambiguity in the meaning of\"self-interested\"']", "label": 1 }, { "id": "train_2471", "context": "Scientist: Earth' s average annual temperature has increased by about 0. 5 degrees Celsius over the last century. This warming is primarily the result of the buildup of minor gases in the atmosphere, blocking the outward flow of heat from the planet.", "question": "Which one of the following, if true, would count as evidence against the scientist's explanation of Earth's warming?", "answers": "['Most of the warming occurred before 1940, while most of the buildup of minor gases in the atmosphere occurred after 1940.', 'Only some of the minor gases whose presence in the atmosphere allegedly resulted in the phenomenon described by the scientist were produced by industrial pollution.', \"The accumulation of minor gases in the atmosphere has been greater over the last century than at any other time in Earth's history.\", \"Volcanic dust and other particles in the atmosphere reflect much of the Sun's radiation back into space before it can reach Earth's surface.\"]", "label": 0 }, { "id": "train_2472", "context": "Social critic: The whole debate over the legal right of rock singers to utter violent lyrics misses the point. Legally, there is very little that may not be said. But not everything that may legally be said, ought to be said. Granted, violence predates the rise in popularity of such music. Yet words also have the power to change the way we see and the way we act.", "question": "Which one of the following is most strongly supported by the passage?", "answers": "['Stopping the production of rock music that contains violent lyrics would eliminate much of the violence within society.', 'If rock musicians voluntarily censor their violent lyrics, this may help to reduce violence in society.', 'Violent rock song lyrics do not incite violence, they merely reflect the violence in society.', 'If rock music that contains violent lyrics is morally wrong, then it should be illegal.']", "label": 1 }, { "id": "train_2473", "context": "After several attempts to distract his young parrot from chewing on furniture, George reluctantly took an expert' s advice and gently hit the parrot' s beak whenever the bird started to chew furniture. The bird stopped chewing furniture, but it is now afraid of hands and will sometimes bite. Since chewing on the furniture would not have hurt the bird, George should not have hit it. When Carla' s puppy escaped from her yard, it bounded into a busy street. Although Carla does not generally approve of physical discipline, she hit the puppy sharply with her hand. Now the puppy enters the street only when accompanied by Carla, so Carla was justified in disciplining the puppy.", "question": "Which one of the following principles, if established, would justify the judgments about George's and Carla's actions?", "answers": "['When disciplining an animal physically, a trainer should use an object such as a rolled up newspaper to avoid making the animal frightened of hands.', 'Physical discipline should not be used by an animal trainer except to correct persistent behavior problems.', 'When training an animal, physical discipline should be used only when such discipline is necessary to correct behavior that could result in serious harm to the animal.', 'Using physical discipline to train an animal is justified only when all alternative strategies for correcting undesirable behavior have failed.']", "label": 2 }, { "id": "train_2474", "context": "Music historian: In the past, radio stations would not play rock songs that were more than three minutes in length. Rock musicians claimed that such commercial barriers limited their creativity, and some critics argue that only since those barriers have been lifted has rock music become artistic. In fact, however, when these barriers were lifted, the standards for song structures broke down and the music became aimless, because the styles from which rock derived were not well suited to songs of extended length.", "question": "Which one of the following is most strongly supported by the music historian's claims?", "answers": "['Rock music can sometimes benefit from the existence of commercial barriers rather than being harmed by them.', 'Rock music is not a good outlet for creative musicians who have a great many ideas.', 'Rock music requires more discipline than some other forms of music.', 'Rock music must borrow from styles more conducive to songs of extended length if it is to become artistic.']", "label": 0 }, { "id": "train_2475", "context": "In 1992, a major newspaper circulated throughout North America paid its reporters an average salary that was much lower than the average salary paid by its principal competitors to their reporters. An executive of the newspaper argued that this practice was justified, since any shortfall that might exist in the reporters' salaries is fully compensated by the valuable training they receive through their assignments.", "question": "Which one of the following, if true about the newspaper in 1992, most seriously undermines the justification offered by the executive?", "answers": "['The circulation of the newspaper had recently reached a plateau, after it had increased steadily throughout the 1980s.', \"Most of the newspaper's reporters had worked there for more than ten years.\", \"The union that represented reporters at the newspaper was different from the union that represented reporters at the newspaper's competitors.\", 'The newspaper was widely read throughout continental Europe and Great Britain as well as North America.']", "label": 1 }, { "id": "train_2476", "context": "Literary critic: A folktale is a traditional story told in an entertaining way, which may lead some to think that folktales lack deeper meaning. But this is not the case. A folktale is passed along by a culture for generations, and each storyteller adds something of his or her own to the story, and in this way folktales provide great insight into the wisdom of the culture.", "question": "The main conclusion of the literary critic's argument can be properly inferred if which one of the following is assumed?", "answers": "['A story that is told primarily for entertainment does not necessarily lack deeper meaning.', 'Not every tale that lacks deep meaning or beauty is told solely for entertainment.', 'Any tale that provides insight into the wisdom of a culture is deeply meaningful in some respect.', 'Any tale that is passed along by a culture for generations can provide great insight into the wisdom of that culture.']", "label": 2 }, { "id": "train_2477", "context": "Scientist: Any theory that is to be taken seriously must affect our perception of the world. Of course, this is not, in itself, enough for a theory to be taken seriously. To see this, one need only consider astrology.", "question": "The point of the scientist's mentioning astrology in the argument is to present", "answers": "['an example of a theory that should not be taken seriously because it does not affect our perception of the world', 'an example of a theory that affects our perception of the world, and thus should be taken seriously', 'an example of a theory that should be taken seriously, even though it does not affect our perception of the world', 'an example of a theory that should not be taken seriously despite its affecting our perception of the world']", "label": 3 }, { "id": "train_2478", "context": "The recent decline in the value of the dollar was triggered by a prediction of slower economic growth in the coming year. But that prediction would not have adversely affected the dollar had it not been for the government' s huge budget deficit, which must ttherefore be decreased to prevent future currency declines.", "question": "Which of the following, if true, would most seriously weaken the conclusion about how to prevent future currency declines?", "answers": "['The government has made little attempt to reduce the budget deficit.', \"Before there was a large budget deficit, predictions of slower economic growth frequently caused declines in the dollar's value.\", 'The value of the dollar declined several times in the year prior to the recent prediction of slower economic growth.', 'The budget deficit has not caused a slowdown in economic growth.']", "label": 1 }, { "id": "train_2479", "context": "Economist: Countries with lower tax rates tend to have stronger economies. Although higher taxes raise more revenue, highly taxed consumers have less disposable income. An economy can never grow if consumers aren' t able to purchase goods and services. Ttherefore, the government should lower tax rates across the board.", "question": "The economist's argument depends on assuming that:", "answers": "['Economic growth is more important than supporting social welfare programs.', 'Lower tax rates will be much more popular with consumers.', \"Consumers' disposable income is directly related to their ability to purchase goods and services.\", 'Increasing disposable income is the only way to ensure economic growth.']", "label": 2 }, { "id": "train_2480", "context": "Policy Adviser: Freedom of speech is not only a basic human right; it is also the only rational policy for this government to adopt. When ideas are openly aired, good ideas flourish, silly proposals are easily recognized as such, and dangerous ideas can be responded to by rational argument. Nothing is ever gained by forcing citizens to disseminate their thoughts in secret.", "question": "The policy adviser's method of persuasion, in recommending a policy of free speech to the government, is best described by which one of the following?", "answers": "['a warning about the difficulty of suppressing the truth', 'a circular justification of the idea of free speech as an idea that flourishes when free speech is allowed', 'advocating respect for basic rights of citizens for its own sake', 'a coupling of moral ideals with self-interest']", "label": 3 }, { "id": "train_2481", "context": "Company spokesperson: Household Products magazine claims that our Filterator X water filter does not remove chemical contaminants in significant amounts. This attack on the quality of our product is undermined by the experience of the millions of Filterator X owners who are satisfied with the product' s performance.", "question": "Which one of the following, if true, most seriously undermines the company spokesperson's argument?", "answers": "[\"Household Products' evaluations of Filterator X water filters have been consistently negative.\", 'Household Products did not evaluate whether the Filterator X water filter significantly improved the taste of drinking water.', 'Most Filterator X owners have no way to determine how effectively the product removes chemical contaminants from water.', 'People whose household water contains chemical contaminants are more likely than other people to buy a Filterator X water filter.']", "label": 2 }, { "id": "train_2482", "context": "It is unethical for government officials to use their knowledge of impending policies to financially benefit themselves if that knowledge is not available to the general public.", "question": "Which one of the following actions would be unethical according to the principle stated above?", "answers": "['A company whose former manager is now an official with the Department of Natural Resources was one of several bidders for an extremely lucrative contract with the department; the names of the bidders were not disclosed to the public.', 'A retired high-ranking military officer now owns a company that contracts with the Department of Defense. He uses his contacts with department officials to help his company obtain contracts.', 'A Finance Department official, one of the few people who knew of a plan to tax luxury cars, bought a luxury car just before the plan was announced to the public in order to avoid paying the tax.', 'An official with a government agency that regulates securities sold her stock in Acme just after she announced to the public that her agency was investigating Acme for improper accounting.']", "label": 2 }, { "id": "train_2483", "context": "Political advertisement: Sherwood campaigns as an opponent of higher taxes. But is anybody fooled? For the last 10 years, while Sherwood served on the city council, the council consistently increased taxes year after year. Break the cycle of higher and higher taxes: reject Sherwood' s bid for reelection to city council.", "question": "The argument in the political advertisement is most vulnerable to criticism on the grounds that it", "answers": "['takes for granted that a characteristic of a group as a whole is shared by an individual member of that group', 'makes a personal attack on someone who holds a certain view rather than addressing the reasonableness of that view', 'bases a crucial generalization on a very limited sample', 'mistakes something that is sufficient to bring about a result for something that is necessary to bring about that result']", "label": 0 }, { "id": "train_2484", "context": "Editor Y: This is a good photograph: the composition is attractive, especially in the way the image is blurred by smoke in one corner. Editor Z: It' s very pretty, but it' s a bad photograph. It doesn' t make a statement; there' s no obvious reason for the smoke to be there.", "question": "The editors' dialogue provides the most support for the claim that they disagree with each other about whether", "answers": "['a photograph that is not attractive can still be a good photograph', 'attractiveness by itself can make a photograph a good photograph', \"a photograph's composition should be related to a statement that it makes\", 'a photograph that makes no statement can still be attractive']", "label": 1 }, { "id": "train_2485", "context": "Sociologist: The more technologically advanced a society is, the more marked its members' resistance to technological innovations. This is not surprising, because the more technologically advanced a society is, the more aware its members are of technology' s drawbacks. Specifically, people realize that sophisticated technologies deeply affect the quality of human relations.", "question": "The claim that the more technologically advanced a society is, the more aware its members are of technology's drawbacks plays which one of the following roles in the sociologist's argument?", "answers": "[\"It is offered as an explanation of why people's resistance to technological innovations is more marked the more technologically advanced the society in which they live is.\", 'It is a conclusion supported by the claim that people realize that sophisticated technologies deeply affect the quality of human relations.', 'It is a generalization based on the claim that the more people resist technological innovations, the more difficult it is for them to adjust to those innovations.', 'It is an example presented to illustrate the claim that resistance to technological innovations deeply affects the quality of human relations.']", "label": 0 }, { "id": "train_2486", "context": "For each action we perform, we can know only some of its consequences. Thus the view that in no situation can we know what action is morally right would be true if an action' s being morally right were the same as the action' s having the best consequences.", "question": "The conclusion follows logically if which one of the following is assumed?", "answers": "['Knowing that an action has the best consequences requires knowing all the consequences of that action.', 'On some occasions we can know what action is morally right.', 'Only the immediate consequences of our actions are relevant in determining whether they are morally right.', 'On some occasions we can come to learn that it is morally wrong to perform a certain action.']", "label": 0 }, { "id": "train_2487", "context": "Playwright A is credited with being the first playwright to break the convention of having one character converse with a chorus, by casting two lead actors in a play. Yet, Playwright B, who was born several decades before Playwright A, also incorporates this theatrical device in two of his plays.", "question": "Which of the following, if true, best helps to account for the inconsistency noted above?", "answers": "['Playwright A influenced the work of all of those playwrights born after him.', 'The work of Playwright B is credited with influencing the work of Playwright A.', 'The idea of including two actors on stage did not originate with Playwright A.', 'Playwright B and Playwright A died within one year of each other.']", "label": 3 }, { "id": "train_2488", "context": "If legislators are to enact laws that benefit constituents, they must be sure to consider what the consequences of enacting a proposed law will actually be. Contemporary legislatures fail to enact laws that benefit constituents. Concerned primarily with advancing their own political careers, legislators present legislation in polemical terms; this arouses in their colleagues either repugnance or enthusiasm for the legislation.", "question": "Which one of the following is an assumption on which the argument depends?", "answers": "['Legislatures that enact laws that benefit constituents are successful legislatures.', 'The passage of laws cannot benefit constituents unless constituents generally adhere to those laws.', 'The inability of legislators to consider the actual consequences of enacting a proposed law is due to their strong feelings about that law.', 'Legislators considering a proposed law for which they have repugnance or enthusiasm do not consider the consequences that it will actually have.']", "label": 3 }, { "id": "train_2489", "context": "Although large cities are generally more polluted than the countryside, increasing urbanization may actually reduce the total amount of pollution generated nationwide. Residents of large cities usually rely more on mass transportation and live in smaller, more energy-efficient dwellings than do people in rural areas. Thus, a given number of people will produce less pollution if concentrated in a large city than if dispersed among many small towns.", "question": "Which one of the following most accurately describes the role played in the argument by the claim that increasing urbanization may actually reduce the total amount of pollution generated nationwide?", "answers": "['It is a statement offered to call into question the claim that large cities are generally more polluted than the countryside.', 'It is a claim that the rest of the argument is designed to establish.', 'It is a statement serving merely to introduce the topic to be addressed in the argument and plays no logical role.', 'It is used to support the conclusion that people should live in large cities.']", "label": 1 }, { "id": "train_2490", "context": "One is likely to feel comfortable approaching a stranger if the stranger is of one' s approximate age. Ttherefore, long-term friends are probably of the same approximate age as each other since most long-term friendships begin because someone felt comfortable approaching a stranger.", "question": "The reasoning in the argument is flawed in that it", "answers": "['presumes, without warrant, that one is likely to feel uncomfortable approaching a person only if that person is a stranger', 'infers that a characteristic is present in a situation from the fact that that characteristic is present in most similar situations', \"overlooks the possibility that one is less likely to feel comfortable approaching someone who is one's approximate age if that person is a stranger than if that person is not a stranger\", \"fails to address whether one is likely to feel comfortable approaching a stranger who is not one's approximate age\"]", "label": 3 }, { "id": "train_2491", "context": "Leaf beetles damage willow trees by stripping away their leaves, but a combination of parasites and predators generally keeps populations of these beetles in check. Researchers have found that severe air pollution results in reduced predator populations. The parasites, by contrast, are not adversely affected by pollution; nevertheless, the researchers' discovery probably does explain why leaf beetles cause particularly severe damage to willows in areas with severe air pollution, since __", "question": "Which of the following most logically completes the passage?", "answers": "['where air pollution is not especially severe, predators have much more impact on leaf-beetle populations than parasites do', 'willows often grow in areas where air pollution is especially severe', 'neither the predators nor the parasites of leaf beetles themselves attack willow trees', 'the parasites that attack leaf beetles actually tend to be more prevalent in areas with severe air pollution than they are elsewhere']", "label": 0 }, { "id": "train_2492", "context": "Farah: Health officials know that in East Asia cancer is much less common than it is in North America. And it is widely known that typical East Asian diets include a larger proportion of whole grains than typical North American diets do. So North American health officials are derelict in their duty to protect the public health if they do not encourage North Americans to increase the proportion of whole grains in their diets. Prasad: Tea contains substances that can help prevent several types of cancer, and tea is consumed much more widely in East Asia than in North America.", "question": "Prasad responds to Farah by", "answers": "[\"undermining Farah's argument by providing evidence that supports an alternative explanation\", 'demonstrating that the beneficial effects that Farah attributes to diet can be counteracted by other factors', \"calling into question the accuracy of Farah's claim concerning the proportions of whole grains consumed in East Asia and North America\", \"introducing further evidence that supports Farah's conclusion\"]", "label": 0 }, { "id": "train_2493", "context": "The more modern archaeologists learn about Mayan civilization, the better they understand its intellectual achievements. Not only were numerous scientific observations and predictions made by Mayan astronomers, but the people in general seem to have had a strong grasp of sophisticated mathematical concepts. We know this from the fact that the writings of the Mayan religious scribes exhibit a high degree of mathematical competence.", "question": "The argument's reasoning is most vulnerable to criticism on the grounds that the argument", "answers": "['relies on two different senses of the term \"scientific\"', 'overlooks the impressive achievements of other past civilizations', 'bases a generalization on a sample that is likely to be unrepresentative', 'fails to provide an adequate definition of the term \"intellectual achievement\"']", "label": 2 }, { "id": "train_2494", "context": "A company determines its yearly profits by deducting its operating expenses from its revenue for that year. Company X manufactures only widgets and gadgets. Company X' s profits for this year are significantly higher than those for last year, although the company' s operating expenses remained the same and the company sold the same number of widgets and gadgets as it did last year.", "question": "The statements above, if true, best support which of the following conclusions?", "answers": "['Consumers purchased more widgets and gadgets this year than they did last year.', 'Company X has raised the price of its widgets or of its gadgets since last year.', \"Company X's main competitors sold fewer widgets and gadgets this year than they did Company X.\", 'Company X spent more on advertising this year than it did last year.']", "label": 1 }, { "id": "train_2495", "context": "Lopez: Our university is not committed to liberal arts, as evidenced by its decision to close the classics department. The study of classical antiquity is crucial to the liberal arts, and it has been so since the Renaissance. Warrington; Although the study of classical works is essential to the liberal arts, a classics department isn' t, since other departments often engage in that study.", "question": "Warrington's argument proceeds by", "answers": [ "responding to a possible objection to the reasoning in Lopez's argument", "presenting a consideration in order to undermine the reasoning in Lopez's argument", "offering additional reasons in favor of the conclusion of Lopez's argument", "claiming that the reasoning in Lopez's argument rests on an illicit appeal to tradition" ], "label": 1 }, { "id": "train_2496", "context": "Many corporations have begun decorating their halls with motivational posters in hopes of boosting their employees' motivation to work productively. However, almost all employees at these corporations are already motivated to work productively. So these corporations' use of motivational posters is unlikely to achieve its intended purpose.", "question": "The reasoning in the argument is most vulnerable to criticism on the grounds that the argument", "answers": "['fails to consider that even if motivational posters do not have one particular beneficial effect for corporations, they may have similar effects that are equally beneficial', \"does not adequately address the possibility that employee productivity is strongly affected by factors other than employees' motivation to work productively\", \"fails to consider whether corporations that do not currently use motivational posters would increase their employees' motivation to work productively if they began using the posters\", 'fails to consider that even if employees are already motivated to work productively, motivational posters may increase that motivation']", "label": 3 }, { "id": "train_2497", "context": "Of the various food containers made of recycled Styrofoam, egg cartons are among the easiest to make. Because egg shells keep the actual food to be consumed from touching the Styrofoam, used Styrofoam need not be as thoroughly cleaned when made into egg cartons as when made into other food containers.", "question": "Which one of the following is most strongly supported by the information above?", "answers": "['Not every type of food container made of recycled Styrofoam is effectively prevented from coming into contact with the food it contains.', 'The main reason Styrofoam must be thoroughly cleaned when recycled is to remove any residual food that has come into contact with the Styrofoam.', 'No food containers other than egg cartons can safely be made of recycled Styrofoam that has not been thoroughly cleaned.', 'There are some foods that cannot be packaged in recycled Styrofoam no matter how the Styrofoam is recycled.']", "label": 0 }, { "id": "train_2498", "context": "Frank: The Internet has allowed everyone to obtain a vast amount of information on every topic imaginable. Jerry: That is not true. Much of the information available online is false.", "question": "Jerry's response shows that he interprets Frank's statement to imply that", "answers": "['other sources are more reliable than the Internet', 'everyone has an equal opportunity to obtain information on the Internet', 'the majority of people do not use the Internet', 'all the information available on the Internet is true']", "label": 3 }, { "id": "train_2499", "context": "A recent poll found that over 80 percent of the residents of Nalmed Province favored a massive expansion of the commuter rail system as a means of significantly easing congestion on the province' s highways and were willing to help pay for the expansion through an increase in their taxes. Nevertheless, the poll results indicate that expansion of the rail system, if successfully completed, would be unlikely to achieve its goal of easing congestion, because __.", "question": "Which of the following, if true, most logically completes the passage?", "answers": "['expanding the commuter rail system will require the construction of dozens of miles of new railbed', 'most people in favor of expanding the rail system reported less congestion during their highway commute as the primary benefit they would experience', 'the proposed expansion to the commuter rail system will make it possible for some people who both live and work at suburban locations to commute by rail', 'of the less than 20 percent of residents not counted as favoring the expansion, about half claimed to have no opinion one way or the other']", "label": 1 }, { "id": "train_2500", "context": "Even though apes are the only nonhuman creatures able to learn human language, no ape has ever used its human language skills to ask such philosophical questions as, \"How am I different from all other creatures? \" Ttherefore, philosophical thought is unique to humans.", "question": "The conclusion in the passage above relies on which one of the following assumptions?", "answers": "['Apes are incapable of thinking in human language.', 'Human language is unique to humans.', 'Speaking in human language is easier than thinking in human language.', 'Philosophical thought can be expressed only in human language.']", "label": 3 }, { "id": "train_2501", "context": "Each major earthquake in this region has been preceded by a series of minor tremors. Since the region has recently experienced a series of minor tremors, a major earthquake will strike the region in the near future.", "question": "Which one of the following arguments exhibits a pattern of questionable reasoning most similar to that exhibited by the argument above?", "answers": "['This river has overflowed in every spring thaw following a winter with high snowfall. As this winter the area received a record amount of snow, the river will overflow this coming spring.', 'So far, all local outbreaks of this disease among humans occurred soon after the detection of high infection rates among local wildlife. An unusually high infection rate among these animals was recently detected, so a local outbreak of the disease among humans is imminent.', \"The introduction of non-native species to an island always results in the extinction of some of its native species. Since non-native species are currently being introduced to the Galapagos Islands, there will be extinction among the Galapagos' native species.\", 'On planets other than Earth, the presence of certain rare minerals indicates that these planets experienced a period of heavy meteor bombardment. Since the same minerals are found on Earth, Earth experienced a period of heavy meteor bombardment as well.']", "label": 1 }, { "id": "train_2502", "context": "Clinician: Patients with immune system disorders are usually treated with a class of drugs that, unfortunately, increase the patient' s risk of developing osteoporosis, a bone-loss disease. So these patients take another drug that helps to preserve existing bone. Since a drug that enhances the growth of new bone cells has now become available, these patients should take this new drug in addition to the drug that helps to preserve existing bone.", "question": "Which one of the following would be most useful to know in order to evaluate the clinician's argument?", "answers": "['How large is the class of drugs that increase the risk of developing osteoporosis?', 'Why are immune system disorders treated with drugs that increase the risk of developing osteoporosis?', 'To what extent does the new drug retain its efficacy when used in combination with the other drugs?', 'Is the new drug more expensive than the drug that helps to preserve existing bone?']", "label": 2 }, { "id": "train_2503", "context": "Harunia Province has a relatively dry climate and is attracting a fast-growing population that has put increasing demands on its water supply. The two companies that supply water to the region have struggled to keep up with demand and still remain profitable. Yet now they are asking Harunian authorities to write residential water-use regulations that could reduce their revenues and restrict their future flexibility in supplying water profitably.", "question": "Which of the following would, if true, most logically help explain why the watersupply companies are asking the authorities to regulate residential water use?", "answers": "['The companies believe that the population is not likely to continue to grow.', 'Few, if any, Harunian government officials have investments in the companies or serve on their boards of directors.', 'The companies believe regulation is inevitable and that having it in place now will allow better planning and thus future profitability.', 'The companies are planning large water-transportation and irrigation systems that require the approval of neighboring provinces.']", "label": 2 }, { "id": "train_2504", "context": "Ronald Thump will be the next great President of the United States. His cutthroat business tactics will be quite effective as the nation' s top executive. Mr. Thump' s manipulation of tax and bankruptcy loopholes helped grow his father' s fortune.", "question": "The author would most likely agree that:", "answers": "['Business experience is directly relevant to succeeding as president.', \"Ronald Thump's fortune would not exist without his father.\", 'Manipulating tax and bankruptcy loopholes is always advisable.', 'Businessmen always make the best presidents.']", "label": 0 }, { "id": "train_2505", "context": "The annual Journal for Publication, which often solicits articles, publishes only those articles that are both submitted before March 6 and written by certified psychoanalysts. Stevens, who publishes frequently in psychoanalytic literature, submitted an article to the Journal before March 6. This article was accepted for publication in the Journal.", "question": "Which one of the following conclusions follows logically from the statements above?", "answers": "[\"Stevens' recently accepted article will be interesting to Journal readers.\", 'The Journal asked Stevens to write an article.', 'Stevens is a psychoanalyst.', 'Stevens is an authority on a large number of topics in psychoanalysis.']", "label": 2 }, { "id": "train_2506", "context": "Editorial: The gates at most railroad crossings, while they give clear warning of oncoming trains, are not large enough to prevent automobile drivers from going around them onto the tracks. Some people claim that the ensuing accidents are partly the fault of the railroad company, but this is a mistake. Granted, if one has a small child in the house, then one ought to block access to stairs completely; but a licensed driver is a capable adult who should know better.", "question": "The editorial's conclusion follows logically if which one of the following is assumed?", "answers": "['Small children are not involved in accidents resulting from drivers going around the gates.', 'Capable adults have a responsibility to take some measures to ensure their own safety.', 'The gates could be made larger, yet irresponsible drivers might still be able to go around them onto the tracks.', 'When the warnings of companies are disregarded by capable adults, the adults are fully responsible for any resulting accidents.']", "label": 3 }, { "id": "train_2507", "context": "The chairperson of Acme Corporation has decided to move the company from its current location in Milltown to Ocean View. Most Acme employees cannot afford housing within a 30-minute commute of Ocean View. So once the company has moved, most Acme employees will have a commute of more than 30 minutes.", "question": "The argument requires assuming which one of the following?", "answers": "['All Acme employees can afford housing within a 30-minute commute of Milltown.', 'Currently, most Acme employees have a commute of less than 30 minutes.', 'The chairperson of Acme has good financial reasons for wanting to move the company to Ocean View.', \"Acme's move to Ocean View will not be accompanied by a significant pay raise for Acme employees.\"]", "label": 3 }, { "id": "train_2508", "context": "It is obvious that one ought to have a will stating how one wishes one' s estate to be distributed. This can easily be seen from the fact that, according to current laws, in the absence of a legal will distant relatives whom one has never even met have a greater legal right to one' s estate than one' s beloved friends do.", "question": "Which one of the following is an assumption on which the argument depends?", "answers": "['No one wants his or her estate to go to someone he or she has never met.', 'People are generally not indifferent about how their estates are distributed.', \"One's beloved friends have a greater legal right to one's estate than one's distant relatives do.\", \"One's estate should go only to a person who is deserving.\"]", "label": 1 }, { "id": "train_2509", "context": "Two different dates have been offered as the approximate end point of the last ice age in North America. The first date was established by testing insect fragments found in samples of sediments to determine when warmth-adapted open-ground beetles replaced cold-adapted arctic beetles. The second date was established by testing pollen grains in those same samples to determine when ice masses yielded to spruce forests. The first date is more than 500 years earlier than the second.", "question": "The statements above, if true, most strongly support which one of the following conclusions about the last ice age and its aftermath in North America?", "answers": "['Ice masses continued to advance through North America for several hundred years after the end of the ice age.', 'Toward the end of the ice age, warmth-adapted open-ground beetles ceased to inhabit areas where the predominant tree cover consisted of spruce forests.', 'Among those sediments deposited toward the end of the ice age, those found to contain cold-adapted arctic beetle fragments can also be expected to contain spruce-pollen grains.', 'Toward the end of the ice age, warmth-adapted open-ground beetles colonized the new terrain opened to them faster than soil changes and seed dispersion established new spruce forests.']", "label": 3 }, { "id": "train_2510", "context": "As far as we know, Earth is the only planet on which life has evolved, and all known life forms are carbon-based. Ttherefore, although there might exist noncarbon-based life on planets very unlike Earth, our scientific estimates of the probability of extraterrestrial life should be generated from estimates of the number of planets like Earth and the likelihood of carbon-based life on those planets.", "question": "Which one of the following general principles most strongly supports the recommendation?", "answers": "['Estimations of probability that are more closely tied to what is known are preferable to those that are less closely tied to what is known.', 'It is preferable for scientists to restrict their studies to phenomena that are observable and forego making estimates about unobservable things.', 'There is no good reason to think that unobserved phenomena closely resemble those that have been observed.', 'A scientific theory that explains observed phenomena on the basis of a few principles that are independent of each other is preferable to a theory that explains those same phenomena on the basis of many independent principles.']", "label": 0 }, { "id": "train_2511", "context": "Until recently, the sole electric company in country Y was owned and operated by the government with notoriously poor customer service. Finally, increased pressure by the international community forced country Y to allow private electric companies to operate in country Y. However, because government taxes on outside electric companies have made these companies largely uncompetitive in country Y, the government-owned electric company still has notoriously poor customer service.", "question": "If the statements above are true, which one of the following must also be true?", "answers": "['Excellent customer service is not always a prerequisite for a company to survive in country Y.', 'Without the government taxes put on outside electric companies, these electric companies would provide better customer service.', 'Without the government taxes, the majority of the citizens of country Y would do business with the outside electric companies.', 'The outside electric companies will no longer continue to serve country Y unless the government taxes are abolished.']", "label": 0 }, { "id": "train_2512", "context": "The giant Chicxulub crater in Mexico provides indisputable evidence that a huge asteroid, about six miles across, struck Earth around the time many of the last dinosaur species were becoming extinct. But this catastrophe was probably not responsible for most of these extinctions. Any major asteroid strike kills many organisms in or near the region of the impact, but there is little evidence that such a strike could have a worldwide effect. Indeed, some craters even larger than the Chicxulub crater were made during times in Earth' s history when there were no known extinctions.", "question": "Which one of the following, if true, would most weaken the argument?", "answers": "['There is no evidence that any other asteroid of equal size struck Earth at the same time as the asteroid that produced the Chicxulub crater.', \"During the period immediately before the asteroid that produced the Chicxulub crater struck, most of the world's dinosaurs lived in or near the region of the asteroid's impending impact.\", 'Fossils have been discovered of a number of dinosaurs that clearly died as a result of the asteroid impact that produced the Chicxulub crater.', 'The size of a crater caused by an asteroid striking Earth generally depends on both the size of that asteroid and the force of its impact.']", "label": 1 }, { "id": "train_2513", "context": "Doctor: It would benefit public health if junk food were taxed. Not only in this country but in many other countries as well, the excessive proportion of junk food in people' s diets contributes to many common and serious health problems. If junk food were much more expensive than healthful food, people would be encouraged to make dietary changes that would reduce these problems.", "question": "Which one of the following most accurately expresses the conclusion drawn in the doctor's argument?", "answers": "['Taxing junk food would benefit public health.', 'Junk food should be taxed if doing so would benefit public health.', \"In many countries, the excessive proportion of junk food in people's diets contributes to many common and serious health problems.\", 'Taxing junk food would encourage people to reduce the proportion of junk food in their diets.']", "label": 0 }, { "id": "train_2514", "context": "People perceive color by means of certain photopigments in the retina that are sensitive to certain wavelengths of light. People who are colorblind are unable to distinguish between red and green, for example, due to an absence of certain photopigments. What is difficult to explain, however, is that in a study of people who easily distinguish red from green, 10 to 20 percent failed to report distinctions between many shades of red that the majority of the subjects were able to distinguish.", "question": "Each of the following, if true, helps to explain the result of the study cited above EXCEPT:", "answers": "['Some people are unable to distinguish red from green due to an absence in the retina of the photopigment sensitive to green.', 'Some people are uninterested in fine gradations of color and fail to notice or report differences they do not care about.', 'Some people fail to report distinctions between certain shades of red because they lack the names for those shades.', 'People with abnormally low concentrations of the photopigments for perceiving red can perceive fewer shades of red than people with normal concentrations.']", "label": 0 }, { "id": "train_2515", "context": "When, on a particular shopping trip, a consumer purchases an item which he previously had no intention of purchasing, this sale is called an \"impulse purchase. \" The objects of impulse purchases are occasionally essential items (i. e. items that satisfy basic subsistence needs), but much more frequently are luxury or non-essential items. Researchers have determined that, at the end of a shopping trip, a consumer is much more excited if she has bought a luxury item on an impulse purchase, than if she had made no impulse purchases.", "question": "If the information above is true, and if the researchers' investigation was properly conducted, then which of the following must also be true?", "answers": "['The researchers had a reliable way to determine whether the consumer had planned to buy the luxury or non-essential item he purchased on that trip.', 'The impulse purchase of luxury or non-essential item is more exciting than the impulse purchase of an essential need.', 'Consumers seeking a high level of excitement often make impulse purchases.', 'A consumer who, for whatever reason, is not able to purchase an item she had planned to buy is necessarily disappointed.']", "label": 0 }, { "id": "train_2516", "context": "To succeed in a particular career, a person must have both talent and tenacity. Hans did not succeed in his career. Ttherefore, he must not have had both talent and tenacity.", "question": "Which one of the following contains the same logical flaw as that in the passage above?", "answers": "['Some plants grow fast when given fertilizer or a lot of light. Ttherefore, this fast-growing plant received either fertilizer or a lot of light.', 'It does not always snow when it is cold and cloudy. Ttherefore, it need not be either cold or cloudy for it to snow.', 'If a person does not have both strong muscles and efficient oxygen transfer, that person cannot run far. Ttherefore, if Erica can run far, either she has strong muscles or else she has efficient oxygen transfer.', 'To make good coffee, you need clear water and fresh coffee beans. Whoever made this bad coffee must not have used both clear water and fresh coffee beans.']", "label": 3 }, { "id": "train_2517", "context": "When industries rapidly apply new technology, people who possess the skills and knowledge to master it prosper, while many others lose their jobs. But firms that resist technological innovations will eventually be superseded by those that do not, resulting in the loss of all their employees' jobs. Obviously, then, resisting the application of new technology in industry__.", "question": "Which one of the following most logically completes the argument?", "answers": "['is less likely to dislocate workers than it is to create job security for them', 'will affect only those who possess technical skills', 'cannot prevent job loss in the long run', 'eventually creates more jobs than it destroys']", "label": 2 }, { "id": "train_2518", "context": "Most of the year, the hermit thrush, a North American songbird, eats a diet consisting mainly of insects, but in autumn, as the thrushes migrate to their Central and South American wintering grounds, they feed almost exclusively on wild berries. Wild berries, however, are not as rich in calories as insects, yet thrushes need to consume plenty of calories in order to complete their migration. One possible explanation is that berries contain other nutrients that thrushes need for migration and that insects lack.", "question": "Which of the following, if true, most seriously calls into question the explanation given for the thrush's diet during migration?", "answers": "['Hermit thrushes, if undernourished, are unable to complete their autumn migration before the onset of winter.', \"Along the hermit thrushes' migration routes, insects are abundant throughout the migration season.\", 'For songbirds, catching insects requires the expenditure of significantly more calories than eating wild berries does.', 'There are some species of wild berries that hermit thrushes generally do not eat, even though these berry species are exceptionally rich in calories.']", "label": 2 }, { "id": "train_2519", "context": "Sociologist: Poverty is the number one cause of crime. When basic needs, like food and shelter, are not met, people become more willing to engage in criminal activity. The easiest way to reduce crime is to lessen poverty.", "question": "Which one of the following statements, if true, best supports the sociologist's argument?", "answers": "['The easiest way to lessen poverty is to redistribute wealth.', 'Studies show that most crimes involve food theft and trespassing.', 'The typical criminal is less wealthy than the average person.', 'Drug addiction and substance abuse is the second largest cause of crime, and drug addicts are more impoverished than the average person.']", "label": 1 }, { "id": "train_2520", "context": "Theodore will be able to file his tax return on time only in the event that he has an accountant prepare his tax return and the accountant does not ask Theodore for any additional documentation of his business expenses. If he does have an accountant prepare his return, the accountant will necessarily ask Theodore to provide this additional documentation. Ttherefore, Theodore will not be able to file on time.", "question": "The pattern of reasoning in which one of the following arguments most closely parallels the pattern of reasoning in the argument above?", "answers": "[\"Mark's children will not be content this weekend unless he lets them play video games some of the time. Mark will let them play video games, but only at times when he has no other activities planned. Ttherefore, unless Mark and his children take a break from planned activities, Mark's children will not be content this weekend.\", \"Given the demands of Timothy's job, his next free evening will occur next Friday. Since he spent a lot of money on his last evening out, he will probably decide to spend his next free evening at home. Ttherefore, Timothy will probably be at home next Friday evening.\", \"Susannah will have a relaxing vacation only if her children behave especially well and she does not start to suspect that they are planning some mischief. Since she will certainly start to suspect that they are planning some mischief if they behave especially well, Susannah's vacation cannot possibly be relaxing.\", 'If Teresa is not seated in first class on her airline flight, she will be seated in business class. Ttherefore, since she cannot be seated in first class on that flight, she will necessarily be seated in business class.']", "label": 2 }, { "id": "train_2521", "context": "Company president: Most of our best sales representatives came to the job with a degree in engineering but little or no sales experience. Thus, when we hire sales representatives, we should favor applicants who have engineering degrees but little or no sales experience over applicants with extensive sales experience but no engineering degrees.", "question": "Which one of the following, if true, most seriously weakens the company president's argument?", "answers": "['Some of the people who the company has hired as sales representatives and who were subsequently not very good at the job did not have extensive previous sales experience.', 'Most of the people hired by the company as sales representatives have had a degree in engineering but no sales experience.', \"Some of the company's sales representatives completed a degree in engineering while working for the company.\", 'Most of the people who apply for a sales representative position with the company do not have a degree in engineering.']", "label": 1 }, { "id": "train_2522", "context": "All Labrador retrievers bark a great deal. All Saint Bernards bark infrequently. Each of Rosa' s dogs is a cross between a Labrador retriever and a Saint Bernard. Ttherefore, Rosa' s dogs are moderate barkers.", "question": "Which one of the following uses flawed reasoning that most closely resembles the flawed reasoning used in the argument above?", "answers": "['All students who study diligently make good grades. But some students who do not study diligently also make good grades. Jane studies somewhat diligently. Ttherefore, Jane makes somewhat good grades.', 'All type A chemicals are extremely toxic to human beings. All type B chemicals are nontoxic to human beings. This household cleaner is a mixture of a type A chemical and a type B chemical. Ttherefore, this household cleaner is moderately toxic.', 'All transcriptionists know shorthand. All engineers know calculus. Bob has worked both as a transcriptionist and as an engineer. Ttherefore, Bob knows both shorthand and calculus.', 'All students at Hanson School live in Green County. All students at Edwards School live in Winn County. Members of the Perry family attend both Hanson and Edwards. Ttherefore, some members of the Perry family live in Green County and some live in Winn County.']", "label": 1 }, { "id": "train_2523", "context": "Between 1976 and 1985, chemical wastes were dumped into Cod Bay. Today, 3 percent of the bay' s bluefin cod population have deformed fins, and wary consumers have stopped buying the fish. In seeking financial reparations from companies that dumped the chemicals, representatives of Cod Bay' s fishing industry have claimed that since the chemicals are known to cause genetic mutations, the deformity in the bluefin cod must have been caused by the presence of those chemicals in Cod Bay.", "question": "The answer to each of the following questions would be helpful in evaluating the representatives' claim EXCEPT:", "answers": "['Are there gene-altering pollutants present in Cod Bay other than the chemical wastes that were dumped by the companies?', 'Has the consumption of the bluefin cod from Cod Bay that have deformed fins caused any health problems in the people who ate them?', 'Are bluefin cod prone to any naturally occurring diseases that can cause fin deformities of the same kind as those displayed by the bluefin cod of Cod Bay?', 'What was the incidence of deformed fins in bluefin cod in Cod Bay before the chemical dumping began?']", "label": 1 }, { "id": "train_2524", "context": "After the Second World War, the charter of the newly formed United Nations established an eleven-member Security Council and charged it with taking collective action in response to threats to world peace. The charter further provided that the five nations that were then the major powers would permanently have sole authority to cast vetoes. The reason given for this arrangement was that the burden of maintaining world peace would rest on the world' s major powers, and no nation should be required to assume the burden of enforcing a decision it found repugnant.", "question": "The reason given for the structure of the Security Council assumes that", "answers": "['no nation that was not among the major powers at the end of the Second World War would become a major power', 'minor powers would not ally themselves with major powers to gain the protection of the veto exercised by major powers', 'it does not make sense to provide for democracy among nations when nations themselves are not all democracies', 'decisions reached by a majority of nations in response to threats to world peace would be biased in favor of one or more major powers']", "label": 0 }, { "id": "train_2525", "context": "In criminal proceedings, defense attorneys occasionally attempt to establish that a suspect was not present at the commission of a crime by comparing the suspect' s DNA to the DNA of blood or hair samples taken from the scene of the crime. Although every person' s DNA is unique, DNA tests often fail to distinguish among DNA samples taken from distinct individuals. Hence, it is a mistake to exonerate a suspect simply because that person' s DNA did not match the DNA samples taken from the scene of the crime.", "question": "Which one of the following is an error in the reasoning above?", "answers": "['It confuses a test that incorrectly identifies DNA samples as coming from the same person with a test that incorrectly shows as coming from different persons samples that come from a single person.', 'It relies on experimental data derived from DNA testing that have not been shown to hold under nonexperimental conditions.', 'It generalizes about the reliability of all methods used to identify those involved in the commission of a crime on the basis of results that pertain to only a few such methods.', 'It assumes without warrant that the use of physical evidence in identifying suspects is never mistaken.']", "label": 0 }, { "id": "train_2526", "context": "Shaw: Regulatory limits on pollution emissions from power plants should be set in terms of the long-term average emissions level rather than peak emissions levels. Levin: But short periods of high pollution emissions pose dangers to the environment. Your proposal is akin to enforcing the highway speed limit by measuring vehicles' average speed, including the time spent in traffic or at stoplights, rather than their peak speed.", "question": "Based on the analogy in Levin's argument, time that a vehicle spends at a stoplight is analogous to time that a power plant", "answers": "['emits pollutants at a very low level', 'operates at peak efficiency', 'emits no pollutants at all', 'emits pollutants at its peak level']", "label": 2 }, { "id": "train_2527", "context": "Generally scientists enter their field with the goal of doing important new research and accept as their colleagues those with similar motivation. Ttherefore, when any scientist wins renown as an expounder of science to general audiences, most other scientists conclude that this popularizer should no longer be regarded as a true colleague.", "question": "The explanation offered above for the low esteem in which scientific popularizers are held by research scientists assumes that", "answers": "['serious scientific research is not a solitary activity, but relies on active cooperation among a . group of colleagues', 'a scientist can become a famous popularizer without having completed any important research', 'research scientists tend not to regard as colleagues those scientists whose renown they envy', 'research scientists believe that those who are well known as popularizers of science are not motivated to do important new research']", "label": 3 }, { "id": "train_2528", "context": "Paleomycologists, scientists who study ancient forms of fungi, are invariably acquainted with the scholarly publications of all other paleomycologists. Professor Mansour is acquainted with the scholarly publications of Professor DeAngelis, who is a paleomycologist. Ttherefore, Professor Mansour must also be a paleomycologist.", "question": "The flawed pattern of reasoning in the argument above is most similar to that in which one of the following arguments?", "answers": [ "Any time the price of fuel decreases, Global Airlines' expenses decrease and its income is unaffected. The price of fuel decreased several times last year. Ttherefore, Global Airlines must have made a profit last year.", "All employees of Global Airlines can participate in its retirement plan after they have been with the company a year or more. Gavin has been with Global Airlines for three years. We can ttherefore be sure that he participates in Global's retirement plan.", "When a flight on Global Airlines is delayed, all connecting Global Airlines flights are also delayed so that the passengers can make their connections. Since Frieda's connecting flight on Global was delayed, her first flight must have also been a delayed Global Airlines flight.", "Any time that one of Global Airlines' local ticket agents misses a shift, the other agents on that shift need to work harder than usual. Since none of Global's local ticket agents missed a shift last week, the airline's local ticket agents did not have to work harder than usual last week." ], "label": 2 }, { "id": "train_2529", "context": "A favorable biography of a politician omits certain incriminating facts about the politician that were available to anyone when the book was written. The book' s author claims that, because he was unaware of these facts when he wrote the book, he is not accountable for the fact that readers were misled by this omission. In a biographer, however, ignorance of this kind cannot be used to evade blame for misleading readers.", "question": "Which one of the following principles, if established, does most to justify the position advanced by the passage?", "answers": "['An author of a biography should not be blamed for whether the book is perceived to be favorable or unfavorable by readers of the biography.', \"An author of a biography should not be blamed for omitting facts if those facts would have supported the author's view.\", \"An author of a biography should be blamed for readers' misperceptions caused by omitting facts that were widely available when the biography was written.\", \"An author of a biography should be blamed for readers' misperceptions only when facts are omitted deliberately in order to mislead the readers.\"]", "label": 2 }, { "id": "train_2530", "context": "Last year all refuse collected by Shelbyville city services was incinerated. This incineration generated a large quantity of residual ash. In order to reduce the amount of residual ash Shelbyville generates this year to half of last year's total, the city has revamped its collection program. This year city services will separate for recycling enough refuse to reduce the number of truckloads of refuse to be incinerated to half of last year's number.", "question": "Which of the following is required for the revamped collection program to achieve its aim?", "answers": "['This year, no materials that city services could separate for recycling will be incinerated.', 'Refuse collected by city services will contain a larger proportion of recyclable materials this year than it did last year.', 'Separating recyclable materials from materials to be incinerated will cost Shelbyville less than half what it cost last year to dispose of the residual ash.', 'The refuse incinerated this year will generate no more residual ash per truckload incinerated than did the refuse incinerated last year.']", "label": 3 }, { "id": "train_2531", "context": "Principle: Anyone who has more than one overdue book out on loan from the library at the same time must be fined if some of the overdue books are not children' s books and that person has previously been fined for overdue books. Application: Since three of the books that Kessler currently has out on loan from the library are overdue, Kessler must be fined.", "question": "Which one of the following, if true, justifies the above application of the principle?", "answers": "[\"None of the books that Kessler currently has out on loan from the library is a children's book and in previous years Kessler has returned various books late.\", 'One of the overdue books that Kessler currently has out on loan from the library is a novel for adults, and Kessler was fined last year for returning this book late.', \"Some of the books that Kessler currently has out on loan from the library are not children's books, and Kessler was fined last year for returning a children's book late.\", 'Kessler has never before been fined for overdue books, but the three overdue books that Kessler currently has out on loan from the library are months overdue.']", "label": 1 }, { "id": "train_2532", "context": "Until fairly recently, classroom computers were considered a luxury. Today, educators argue that students who have not had training in computer skills will lack the skills necessary to compete in the global marketplace . However, studies show that schools emphasizing computer technology spend more time teaching computer skills but less time developing students' basic math and reading skills.", "question": "Which one of the following propositions is best illustrated by the statements above?", "answers": "['Schools cannot emphasize the teaching of computer skills without neglecting other skills.', 'A complete rethinking of traditional academic subjects is required in order to keep pace with global developments.', 'Attempting to keep pace with recent educational developments can result in neglecting basic skills in favor of other skills.', 'A knowledge of the latest technologies is no more valuable than a knowledge of the fundamental academic disciplines .']", "label": 2 }, { "id": "train_2533", "context": "A recent study concludes that prehistoric birds, unlike modern birds, were cold-blooded. This challenges a widely held view that modern birds descended from warm-blooded birds. The conclusion is based on the existence of growth rings in prehistoric birds' bodily structures, which are thought to be found only in cold-blooded animals. Another study, however, disputes this view. It concludes that prehistoric birds had dense blood vessels in their bones, which suggests that they were active creatures and ttherefore had to be warm-blooded.", "question": "Which one of the following, if true, would most help to resolve the dispute described above in favor of one party to it?", "answers": "['In some cold-blooded species the gene that is responsible for growth rings is also responsible for dense blood vessels.', 'Dense blood vessels are not found in all warm- blooded species.', 'Having growth rings is not the only physical trait of cold-blooded species.', 'Modern birds did not evolve from prehistoric species of birds.']", "label": 0 }, { "id": "train_2534", "context": "When choosing a dog, families should not only evaluate the dog' s ability to interact with other animals, but should also inquire about the dog' s friendliness to unfamiliar people. Dog experts attempt to justify inquiries about a dog' s friendliness to unfamiliar people by saying that this behavior might also mean the dog could adjust well to new children in the family. But such behavior might also show that the dog is an incompetent watchdog. Ttherefore, families looking to choose a dog should not be concerned with the dog' s friendliness to unfamiliar people.", "question": "The argument is flawed because it overlooks each of the following possibilities EXCEPT:", "answers": "['A dog might be able to distinguish between children and those people who pose a danger to the family.', 'Dogs that are adopted by families often view and treat unfamiliar people like they do other animals.', \"A dog's friendliness to unfamiliar people might be a good indicator of how the dog interacts with other animals.\", 'A dog might appear to be friendly to unfamiliar people at pet stores, but might really be an unfriendly dog.']", "label": 3 }, { "id": "train_2535", "context": "Environmentalist: Snowmobiles in the park north of Milville create unacceptable levels of air pollution and should be banned. Milville Business Spokesperson: Snowmobiling brings many out-of-towners to Milville in the winter months, to the great direct financial benefit of many local residents. In addition, the money the town collects in fees for the recreational use of the park indirectly benefits all Milville residents. So, it is basic economics for us to put up with the pollution.", "question": "Which of the following, if true, could best be used by the environmentalist to counter the business spokesperson's argument?", "answers": "['Not all of the people who go snowmobiling in the vicinity of Milville are from out of town.', 'Many Milville residents object to having to pay fees for recreational use of the park in the winter.', 'A great many cross-country skiers are now kept from visiting Milville by the noise and pollution that snowmobiles generate.', 'Snowmobiles, because they run on two-cycle engines, emit greater amounts of hydrocarbons and particulate matter than cars do.']", "label": 2 }, { "id": "train_2536", "context": "Meyer was found by his employer to have committed scientific fraud by falsifying data. The University of Williamstown, from which Meyer held a PhD, validated this finding and subsequently investigated whether he had falsified data in his doctoral thesis, finding no evidence that he had. But the university decided to revoke Meyer' s PhD anyway.", "question": "Which one of the following university policies most justifies the decision to revoke Meyer's PhD?", "answers": "['Anyone who holds a PhD from the University of Williamstown and is found to have committed scientific fraud will have the PhD revoked.', 'Anyone who holds a PhD from the University of Williamstown and is found to have committed academic fraud in the course of pursuing that PhD will have the PhD revoked.', 'No PhD program at the University of Williamstown will admit any applicant who has been determined to have committed any sort of academic fraud.', 'Any University of Williamstown student who is found to have submitted falsified data as academic work will be dismissed from the university.']", "label": 0 }, { "id": "train_2537", "context": "Meat from chickens contaminated with salmonella bacteria can cause serious food poisoning. Capsaicin, the chemical that gives chili peppers their hot flavor, has antibacterial properties. Chickens do not have taste receptors for capsaicin and will readily eat feed laced with capsaicin. When chickens were fed such feed and then exposed to salmonella bacteria, relatively few of them became contaminated with salmonella.", "question": "In deciding whether the feed would be useful in raising salmonella-free chicken for retail sale, it would be most helpful to determine which of the following?", "answers": "['Whether eating capsaicin reduces the risk of salmonella poisoning for humans', 'Whether appropriate cooking of chicken contaminated with salmonella can always prevent food poisoning', 'Whether feeding capsaicin to chickens affects the taste of their meat', 'Whether capsaicin can be obtained only from chili peppers']", "label": 2 }, { "id": "train_2538", "context": "Foster: Many species of extremely large North American mammals became extinct during the last ice age, which was also the time of the first human migration to North America. These species could not survive the dramatic changes wrought by this human migration. Fisch: Those extinctions were caused by the dramatic shift to a harsher climate. The climate changed so rapidly that the species could not adapt.", "question": "Which one of the following, if true, most strengthens Fisch's argument?", "answers": "['Similar species living in parts of the world where there were dramatic changes in climate did not become extinct.', 'The huge size of the mammals made it difficult for them to migrate the great distances to milder environments.', 'Archaeological evidence reveals that the human migrants had a number of different kinds of large weapons.', 'Human migration to other previously isolated areas has resulted in mammal species becoming extinct.']", "label": 1 }, { "id": "train_2539", "context": "Smith, a basketball player, successfully made 80% of the shots that he attempted this year -- an exceptionally high percentage for a player in Smith' s league. However, this statistic does not mean that Smith is an exceptional shooter, because the best shooters take many shots from risky locations 20 or more feet away from the basket. Instead, Smith' s high shooting percentage simply shows that he takes most of his shots from locations very close to the basket.", "question": "Which of the following statements, if true, provides additional support for the argument?", "answers": "['The position that Smith plays allows him relatively few shooting opportunities from locations close to the basket.', \"Smith's shooting percentage, the highest in the league, was almost 10% higher than that of the player with the next highest percentage.\", 'Smith took most of his shots during games that were closely contested and in which his team could not afford the time and effort to set up easier shots.', \"Smith's shooting percentage for shots taken from more than 20 feet away from the basket is no higher than other players' shooting percentages from that distance.\"]", "label": 3 }, { "id": "train_2540", "context": "Press release: A comprehensive review evaluating the medical studies done up to the present time has found no reason to think that drinking coffee in normal amounts harms the coffee-drinker' s heart. So coffee drinkers can relax and enjoy their beverage -- it is safe to drink coffee.", "question": "Which one of the following points to a weakness in the reasoning in the press release's argument?", "answers": "['The health of the heart is not identical with the general health of the body.', 'Drinking unusually large amount of coffee could be caused by stress that itself directly harms the heart.', 'Other beverages besides coffee might contain stimulants that have some effect on the heart.', 'Coffee drinkers might choose to eat, along with their coffee, foods containing substances that harm the heart.']", "label": 0 }, { "id": "train_2541", "context": "To protect certain fledgling industries, the government of Country Z banned imports of the types of products those industries were starting to make. As a direct result, the cost of those products to the buyers, several export-dependent industries in Z, went up, sharply limiting the ability of those industries to compete effectively in their export markets.", "question": "Which of the following conclusions about Country Z's adversely affected export-dependent industries is best supported by the passage?", "answers": "['Profit margins in those industries were not high enough to absorb the rise in costs mentioned above.', 'Steps to offset rising materials costs by decreasing labor costs were taken in those industries.', 'Those industries started to move into export markets that they had previously judged unprofitable.', 'Those industries succeeded in expanding the domestic market for their products.']", "label": 0 }, { "id": "train_2542", "context": "Psychiatrist: Breaking any habit is difficult, especially when it involves an addictive substance. People who break a habit are more likely to be motivated by immediate concerns than by long-term ones. Ttherefore, people who succeed in breaking their addiction to smoking cigarettes are more likely to be motivated by the social pressure against smoking -- which is an immediate concern -- than by health concerns, since __.", "question": "The conclusion of the psychiatrist's argument is most strongly supported if which one of the following completes the argument?", "answers": "['for most people who successfully quit smoking, smoking does not create an immediate health concern at the time they quit', 'most people who succeed in quitting smoking succeed only after several attempts', 'a habit that involves an addictive substance is likely to pose a greater health threat than a habit that does not involve any addictive substance', 'some courses of action that exacerbate health concerns can also relieve social pressure']", "label": 0 }, { "id": "train_2543", "context": "During 1991 the number of people in the town of Bayburg who received municipal food assistance doubled, even though the number of people in Bayburg whose incomes were low enough to qualify for such assistance remained unchanged.", "question": "Which one of the following, if true, most helps to resolve the apparent discrepancy in the information above?", "answers": "['In 1990 the Bayburg Town Council debated whether or not to alter the eligibility requirements for the food assistance program but ultimately decided not to change them.', 'During 1991 the number of applicants for food assistance in Bayburg who were rejected on the basis that their incomes were above the maximum allowable limit was approximately the same as it had been in 1990.', \"During 1991 Bayburg's program of rent assistance for low-income tenants advertised widely and then informed all applicants about other assistance programs for which they would be qualified.\", 'During 1991 many residents of a nearby city lost their jobs and moved to Bayburg in search of work.']", "label": 2 }, { "id": "train_2544", "context": "If Blankenship Enterprises has to switch suppliers in the middle of a large production run, the company will not show a profit for the year. Ttherefore, if Blankenship Enterprises in fact turns out to show no profit for the year, it will also turn out to be true that the company had to switch suppliers during a large production run.", "question": "The reasoning in the argument is most vulnerable to criticism on which one of the following grounds?", "answers": "['The argument is a circular argument made up of an opening claim followed by a conclusion that merely paraphrases that claim.', 'The argument fails to establish that a condition under which a phenomenon is said to occur is the only condition under which that phenomenon occurs.', 'The argument explains one event as being caused by another event, even though both events must actually have been caused by some third, unidentified event.', 'The argument involves an equivocation, in that the word \"profit\" is allowed to shift its meaning during the course of the argument.']", "label": 1 }, { "id": "train_2545", "context": "All people prefer colors that they can distinguish easily to colors that they have difficulty distinguishing. Infants can easily distinguish bright colors but, unlike adults, have difficulty distinguishing subtle shades. A brightly colored toy for infants sells better than the same toy in subtle shades at the same price.", "question": "Which one of the following conclusions is most strongly supported by the information in the passage?", "answers": "['The sales of toys for infants reflect the preferences of infants in at least one respect.', 'Toy makers study infants to determine what colors the infants can distinguish easily.', 'Infants prefer bright primary colors to bright secondary colors.', 'Color is the most important factor in determining which toys an infant will prefer to play with.']", "label": 0 }, { "id": "train_2546", "context": "Comcorp Shipping Clerk: Last week, no shipments of building supplies were sent out on Friday. The five specially ordered shipments sent out last week were sent out on Thursday, and each of those specially ordered shipments consisted entirely of building supplies. Four shipments were sent to Truax Construction last week, none of which consisted of building supplies.", "question": "If the shipping clerk's statements are true, which of the following must also be true?", "answers": "[\"All of Comcorp's shipments of building supplies last week were specially ordered.\", 'None of the shipments sent by Comcorp to Truax Construction last week was specially ordered.', \"All of Comcorp's shipments of building supplies last week were sent out on Thursday.\", \"None of Comcorp's shipments sent on Thursday of last week was sent to Truax Construction.\"]", "label": 1 }, { "id": "train_2547", "context": "Sarah: Our regulations for staff review are vague and thus difficult to interpret. For instance, the regulations state that a staff member who is performing unsatisfactorily will face dismissal, but they fail to define unsatisfactory performance. Thus, some staff may be dismissed merely because their personal views conflict with those of their supervisors.", "question": "Which one of the following generalizations, if applicable to Sarah's company, most helps to justify her reasoning?", "answers": "['A vague regulation can be used to keep those subject to it in subordinate positions.', 'Interpreting regulations is a prerogative that belongs solely to supervisors.', 'Performance that falls only somewhat below expectations results in disciplinary measures short of dismissal.', 'A vague regulation can be used to make those subject to it answer for their performance.']", "label": 1 }, { "id": "train_2548", "context": "Columnist: Computer voice-recognition technology currently cannot distinguish between homophones such as \"their\" and \"there. \" As a consequence, until voice-recognition technology is improved to recognize and utilize grammatical and semantic relations among words, voice-recognition programs will not accurately translate a computer user ' s spoken words into written text.", "question": "Which one of the following is an assumption required by the columnist's argument?", "answers": "['Computer programs that check the spelling and grammar of written text are currently able to distinguish between homophones.', 'Unless voice-recognition technology can distinguish between words that are homophones, it cannot recognize grammatical and semantic relations among words.', 'Humans can distinguish between homophones without using information about the grammatical and semantic relations among words.', 'In order for computer voice-recognition technology to distinguish between homophones, it must be able to recognize grammatical and semantic relations among words.']", "label": 3 }, { "id": "train_2549", "context": "In North America there has been an explosion of public interest in, and enjoyment of, opera over the last three decades. The evidence of this explosion is that of the 70 or so professional opera companies currently active in North America, 45 were founded over the course of the last 30 years.", "question": "The reasoning above assumes which one of the following?", "answers": "['The size of the average audience at performances by professional opera companies has increased over the past three decades.', 'All of the 70 professional opera companies are commercially viable operations.', 'There were fewer than 45 professional opera companies that had been active 30 years ago and that ceased operations during the last 30 years.', 'The 45 most recently founded opera companies were all established as a result of enthusiasm on the part of a potential audience.']", "label": 2 }, { "id": "train_2550", "context": "Reformer: A survey of police departments keeps track of the national crime rate, which is the annual number of crimes per 100, 000 people. The survey shows no significant reduction in the crime rate in the past 20 years, but the percentage of the population in prison has increased substantially, and public expenditure on prisons has grown at an alarming rate. This demonstrates that putting more people in prison cannot help to reduce crime.", "question": "A flaw in the reformer's argument is that it", "answers": "['overlooks the possibility that the population has increased significantly over the past 20 years', 'ignores the possibility that the crime rate would have significantly increased if it had not been for the greater rate of imprisonment', 'takes for granted that the number of prisoners must be proportional to the number of crimes committed', 'presumes, without providing warrant, that alternative measures for reducing crime would be more effective than imprisonment']", "label": 1 }, { "id": "train_2551", "context": "For a species of large abalone shellfish to develop from a species of smaller ones, they must spend less energy on finding food and avoiding predators, and more on competition in mating. So it is surprising that the fossil record shows that a species of large abalones developed from a smaller one only after otters, which prey on abalones, began to dominate the waters in which the abalones lived.", "question": "Which one of the following, if true, most helps to resolve the apparent discrepancy in the information above?", "answers": "['Otters have a preference for large abalones over small ones and so prefer waters in which large abalones are found.', 'Small abalone species tend to reproduce more rapidly than larger abalone species.', \"Otters also prey on the abalones' competitors for food and so indirectly make it easier for abalones to get food.\", 'Otters and abalones also compete for the same types of food and so are drawn to the same waters.']", "label": 2 }, { "id": "train_2552", "context": "Marketing chief: The aggressive sales campaign of Product J has made it the most popular product in the sector. Every individual move we made in that campaign was wildly successful, and sales of the product continuously rose. If we mount a similar style sales campaign with Product S, we are likely to vault this into popularity in its own sector, with a similarly robust sales trajectory. Consultant: The popularity of Product J preceded the sales campaign and was responsible for it.", "question": "The consultant uses which of the following techniques in responding to the marketing chief?", "answers": "['questioning the posited relationship of cause and effect', 'strengthening the argument with further evidence', 'citing evidence that contradicts the assumption of the argument', 'pointing out that the same premises might support an opposing conclusion']", "label": 0 }, { "id": "train_2553", "context": "Sammy: For my arthritis, I am going to try my aunts diet: large amounts of wheat germ and garlic. She was able to move more easily right after she started that diet Pat: When my brother began that diet, his arthritis got worse. But he has been doing much better since he stopped eating vegetables in the nightshade family, such as tomatoes and peppers", "question": "Which of the following, if true, would provide a basis for explaining the fact that Sammy's aunt and Pats brother had contrasting experiences with the same diet?", "answers": "['The compounds in garlic that can lessen the symptoms of arthritis are also present in tomatoes and peppers', 'Arthritis is a chronic condition whose symptoms improve and worsen from time to time without regard to diet.', 'A change in diet, regardless of the nature of the change, frequently brings temporary relief from arthritis symptoms', 'In general, men are more likely to have their arthritis symptoms alleviated by avoiding vegetables in the nightshade family than are women']", "label": 1 }, { "id": "train_2554", "context": "Appliance dealer: Appliance manufacturers commonly modify existing models without giving the modified versions new model names. Some people have complained that this practice makes it impossible for consumers to be certain that the appliance they are about to purchase is identical to the one they may have seen at a neighbor' s or read about in a consumer magazine. Yet manufacturers' modifications to existing models are invariably improvements that benefit the buyer. Ttherefore, consumers have little reason to object to this practice.", "question": "Which one of the following, if true, most seriously weakens the dealer's arguments?", "answers": "['The high cost of product advertising makes appliance manufacturers generally reluctant to change model names to reflect modifications to their products.', 'Appliances are generally purchased with the expectation that they will continue to be used for several years.', 'Appliances usually carry a model number that provides substantially more detailed information about the product than does the model name.', 'Improved versions of appliances typically become available before vendors have stopped selling the older versions of the appliance with the same model name.']", "label": 3 }, { "id": "train_2555", "context": "Caldwell: The government recently demolished a former naval base. Among the complex' s facilities were a gymnasium, a swimming pool, office buildings, gardens, and housing for hundreds of people. Of course the government was legally permitted to use these facilities as it wished. But clearly, using them for the good of the community would have benefited everyone, and thus the government' s actions were not only inefficient but immoral.", "question": "Caldwell's argument is most vulnerable to criticism on the grounds that it", "answers": "['inappropriately treats two possible courses of action as if they were the only options', \"fails to consider that an action may be morally permissible even if an alternative course of action is to everyone's advantage\", 'presumes, without providing justification, that the government never acts in the most efficient manner', \"presumes, without providing justification, that the actual consequences of an action are irrelevant to the action's moral permissibility\"]", "label": 1 }, { "id": "train_2556", "context": "The principal of School X has proposed a plan that would add an hour-long study period to the end of the school day. She claims that the extension would improve the school's average score on the math section of the state assessment by allowing students more time to complete math homework that they wouldn't otherwise have time to complete.", "question": "Which of the following statements, if true, would most weaken the argument presented above?", "answers": "['Music department faculty members at School X strongly oppose the plan because they feel any time added to the school day should be used to bolster the music curriculum.', \"The core components of School X's math curriculum are not aligned with the topics tested on the state math assessment.\", \"Parents of students from School X have expressed concern that the proposed schedule change would interfere with students' extracurricular activities.\", 'Administrators from neighboring School Y recently implemented a school day extension and have seen no significant improvement in student test scores on the state assessment.']", "label": 1 }, { "id": "train_2557", "context": "Historian: The only evidence we have for claims about the past exists in the present. How things actually occurred is beyond our knowledge. Historians construct coherent stories that explain the available evidence and why the present is as it is. Such stories about the past, however, do not need to be true to be good history; they need only explain the evidence about the past and what we know about the present better than do rival accounts.", "question": "Which one of the following judgments conforms most closely to the historian's position?", "answers": "['It is not necessary to know what actually occurred during the 1857 War of Independence to write a good history of it.', 'It is not possible to write a good history of the 1857 War of Independence without studying rival accounts.', 'It is likely that the 1857 War of Independence did not actually occur in the way we think it did.', 'An account of the 1857 War of Independence that is true is better as history than one that is not true but better explains the evidence.']", "label": 0 }, { "id": "train_2558", "context": "Investigators have not proved that the forest fire was started by campers. Nor have they proved that lightning triggered the fire. So the investigators have not proved that the blaze was caused by campers or lightning.", "question": "The flawed pattern of reasoning in which one of the following arguments most closely resembles the flawed pattern of reasoning in the argument above?", "answers": "['We have no proof either for the theory that the thief escaped through the vent in the ceiling or for the theory that the thief escaped through the window. Ttherefore, one theory is as plausible as the other.', 'Kim has no reason to believe that Sada will win the election. Kim also has no reason to believe that Brown will win the election. So Kim has no reason to believe that either Sada or Brown will win the election.', 'In some parts of the forest camping is permitted. Also, hunting is permitted in some parts of the forest. So there are some parts of the forest in which both hunting and camping are permitted.', 'The evidence shows that the car could have been driven by Jones at the time of the accident; however, it also shows that it could have been driven by Katsarakis at the time of the accident. Ttherefore, the evidence shows that the car could have been driven by both Jones and Katsarakis at the time of the accident.']", "label": 1 }, { "id": "train_2559", "context": "Plan: Concerned about the welfare of its senior citizens, the government of Runagia decided two years ago to increase by 20 percent the government provided pension paid to all Runagians age sixty-five and older. Result: Many Runagian senior citizens are no better off financially now than they were before the increase. Further information: The annual rate of inflation since the pension increase has been below 5 percent, and the increased pension has been duly received by all eligible Runagians.", "question": "In light of the further information, which of the following, if true, does most to explain the result that followed implementation of the plan?", "answers": "['The Runagian banking system is so inefficient that cashing a pension check can take as much as three weeks.', 'The prices of goods and services that meet the special needs of many senior citizens have increased at a rate much higher than the rate of inflation.', 'The majority of senior citizens whose financial position has not improved rely entirely on the government pension for their income.', 'The pension increase occurred at a time when the number of Runagians age sixty-five and older who were living below the poverty level was at an all-time high.']", "label": 1 }, { "id": "train_2560", "context": "Cigarette smoking has been shown to be a health hazard; ttherefore, governments should ban all advertisements that promote smoking.", "question": "Which one of the following principles, if established, most strongly supports the argument?", "answers": "['Advertisers should not make misleading claims about the healthfulness of their products.', 'Advertisements should promote only healthful products.', 'Advertisements should not be allowed to show people doing things that endanger their health.', 'All products should conform to strict government health and safety standards.']", "label": 1 }, { "id": "train_2561", "context": "A recent test of an electric insect control device discovered that, of the more than 300 insects killed during one 24-hour period, only 12 were mosquitoes. Thus this type of device may kill many insects, but will not significantly aid in controlling the potentially dangerous mosquito population.", "question": "Which one of the following, if true, most seriously weakens the argument?", "answers": "['Many of the insects that were killed by the device are mosquito-eating insects.', 'A careful search discovered no live mosquitoes in the vicinity of the device after the test.', 'The device does not succeed in killing all of the insects that it attracts.', 'A very large proportion of the insects that were attracted to the device were not mosquitoes.']", "label": 1 }, { "id": "train_2562", "context": "To treat a person justly is to treat that person with perfect impartiality. But our friends naturally expect us to attach more importance to their interests than to the interests of others. Hence, given that we try to maintain our friendships, we cannot succeed in being just to our friends.", "question": "Which of the following must be assumed in order for the conclusion of the argument above to be properly drawn?", "answers": "['People should not treat their friends less justly than they treat others.', \"One cannot both treat someone impartially and value that person's interests above the interests of others.\", 'The ideal of justice is irrelevant to most interpersonal relationships.', 'It is morally important to maintain close friendships.']", "label": 1 }, { "id": "train_2563", "context": "Technological progress makes economic growth and widespread prosperity possible; it also makes a worker' s particular skills less crucial to production. Yet workers' satisfaction in their work depends on their believing that their work is difficult and requires uncommon skills. Clearly, then, technological progress __.", "question": "Which one of the following most logically completes the argument?", "answers": "['is generally opposed by the workers whose work will be directly affected by it', \"eliminates many workers' jobs\", 'causes workers to feel less satisfaction in their work', 'decreases the quality of most products']", "label": 2 }, { "id": "train_2564", "context": "Book Review: When I read a novel set in a city I know well, I must see that the writer knows the city at least as well as I do if I am to take that writer seriously. If the writer is faking, I know immediately and do not trust that writer. When a novelist demonstrates the required knowledge, I trust the storyteller, so I trust the tale. This trust increases my enjoyment of a good novel. Peter Lee' s second novel is set in San Francisco. In this novel, as in his first, Lee passes my test with flying colors.", "question": "Which one of the following can be properly inferred from the passage?", "answers": "[\"Peter Lee's first novel was set in San Francisco.\", 'The book reviewer enjoys virtually any novel written by a novelist whom she trusts.', 'The book reviewer does not trust any novel set in a city that she does not know well.', 'The book reviewer does not believe that she knows San Francisco better than Peter Lee does.']", "label": 3 }, { "id": "train_2565", "context": "Whether one is buying men' s or women' s clothing, it pays to consider fashion trends. A classic suit may stay in style for as long as five years, so it is worthwhile to pay more to get a well-constructed one. A trendy hat that will go out of style in a year or two should be purchased as cheaply as possible.", "question": "Which one of the following most accurately expresses the principle underlying the reasoning above?", "answers": "['One should not buy a cheaply made garment when a well-constructed garment is available.', 'Formal attire tends to be designed and constructed to last longer than casual attire.', 'The amount of money one spends on a garment should be roughly proportionate to the length of time one plans to keep wearing it.', \"The amount of money one spends on clothing should be roughly the same whether one is purchasing men's or women's attire.\"]", "label": 2 }, { "id": "train_2566", "context": "Environmentalist: Discarding old appliances can be dangerous: refrigerators contain chlorofluorocarbons; electronic circuit boards and cathode-ray tubes often contain heavy metals like lead; and old fluorescent bulbs contain mercury, another heavy metal. When landfills are operated properly, such materials pose no threat. However, when landfills are not operated properly, lead and mercury from them contaminate groundwater, for example. On the other hand, when trash is incinerated, heavy metals poison the ash and escape into the air.", "question": "The environmentalist's statements, if true, most strongly support which one of the following inferences?", "answers": "['Newer appliances are more dangerous to the environment than older ones.', 'Old fluorescent bulbs should be recycled.', 'Appliances should be kept out of landfills.', 'Appliances containing heavy metals should not be incinerated.']", "label": 3 }, { "id": "train_2567", "context": "For every 50 dogs that contract a certain disease, one will die from it. A vaccine exists that is virtually 100 percent effective in preventing this disease. Since the risk of death from complications of vaccination is one death per 5, 000 vaccinations, it is ttherefore safer for a dog to receive the vaccine than not to receive it.", "question": "Which one of the following would it be most helpful to know in order to evaluate the argument?", "answers": "['the number of dogs that die each year from diseases other than the disease in question', 'the total number of dogs that die each year from all causes taken together', 'whether the vaccine is effective against the disease in household pets other than dogs', 'the likelihood that an unvaccinated dog will contract the disease in question']", "label": 3 }, { "id": "train_2568", "context": "Any museum that owns the rare stamp that features an airplane printed upside down should not display it. Ultraviolet light causes red ink to fade, and a substantial portion of the stamp is red. If the stamp is displayed, it will be damaged. It should be kept safely locked away, even though this will deny the public the chance to see it.", "question": "The reasoning above most closely conforms to which one of the following principles?", "answers": "['A museum piece that would be damaged by display should not be displayed.', 'Museum display cases should protect their contents from damage caused by ultraviolet light.', 'The primary purpose of a museum is to educate the public.', 'The public should judge the quality of a museum by the rarity of the objects in its collection.']", "label": 0 }, { "id": "train_2569", "context": "Only poetry cannot be translated well, and ttherefore it is poets who preserve languages, for we would not bother to learn a language if we could get everything written in it from translation. So, since we cannot witness the beauty of poetry except in the language in which it is composed, we have motivation to learn the language.", "question": "The information above provides the LEAST support for which one of the following?", "answers": "['One purpose of writing poetry is to preserve the language in which it is written.', 'All nonpoetic literature can be translated well.', 'The beauty of poetry is not immediately accessible to people who do not understand the language in which the poetry was written.', 'Some translations do not capture all that was expressed in the original language.']", "label": 0 }, { "id": "train_2570", "context": "Because state sanctions against hunting have been eased, Deersdale Preserve has experienced an influx of hunters in the last several months. During this time, the silvertail fox, a popular target for hunters, has seen a marked decrease in population. Ttherefore, to prevent the population of silvertail fox-an important apex predator responsible for keeping the Deersdale County's rabbit population in check-from falling even lower, the state should once again place strict sanctions against hunting.", "question": "Which of the following, if true, most strongly suggests that stricter sanctions against hunting would not have the desired effect?", "answers": "['The silvertail fox had been experiencing a population surge shortly before the state sanctions against hunting were eased.', 'The number of hunters entering the park is expected to drop drastically in the next few months as hunting season comes to an end.', 'Many rabbits in the Deersdale Preserve recently have become carriers of a virus that destroys the livers of carnivorous animals.', 'The silvertail fox population varies greatly throughout the year, especially during winter when prey becomes scarce.']", "label": 2 }, { "id": "train_2571", "context": "Politician: My opponents argue that the future of our city depends on compromise -- that unless the city' s leaders put aside their differences and work together toward common goals, the city will suffer. However, the founders of this city based the city' s charter on definite principles, and anyone who compromises those principles betrays the city founders' goals. What my opponents are advocating, ttherefore, is nothing less than betraying the goals of the city' s founders. Critic: I' m afraid your argument is flawed. Unless you' re assuming that the differences among the city' s leaders are differences of principle, your argument depends on a misleading use of the term __ .", "question": "Which one of the following provides the most logical completion of the critic's statement?", "answers": "['compromise', 'common', 'betray', 'principles']", "label": 0 }, { "id": "train_2572", "context": "Archaeologist: Neanderthals, a human-like species living 60, 000 years ago, probably preserved meat by smoking it. Burnt lichen and grass have been found in many Neanderthal fireplaces. A fire of lichen and grass produces a lot of smoke but does not produce nearly as much heat or light as a wood fire.", "question": "Which one of the following, if true, would most weaken the archaeologist's argument?", "answers": "['In close proximity to the fireplaces with lichen and grass are other fireplaces that, evidence suggests, burned material that produced more heat than smoke.', 'Some of the fireplaces containing burnt lichen are in regions in which lichen is not believed to have been plentiful and so would have had to have been brought in from some distance.', 'In the region containing the Neanderthal fireplaces in which lichen and grass were burnt, no plants that could be burned more effectively to produce heat or light were available 60, 000 years ago.', 'There is clear evidence that at least some groups of Neanderthals living more recently than 60, 000 years ago developed methods of preserving meat other than smoking it.']", "label": 2 }, { "id": "train_2573", "context": "Manufacturers are now required to make all cigarette lighters child-resistant by equipping them with safety levers. But this change is unlikely to result in a significant reduction in the number of fires caused by children playing with lighters, because children given the opportunity can figure out how to work the safety levers and__.", "question": "Which of the following, if true, most logically completes the argument below?", "answers": "['approximately 5, 000 fires per year have been attributed to children playing with lighters before the safety levers were required', 'unlike child-resistant lighters, lighters that are not child-resistant can be operated by children as young as two years old', 'many of the fires started by young children are quickly detected and extinguished by their parents', 'adults are more likely to leave child-resistant lighters than non-child-resistant lighters in places that are accessible to children']", "label": 3 }, { "id": "train_2574", "context": "Plankton generally thrive in areas of the ocean with sufficient concentrations of certain nitrogen compounds near the surface, where plankton live. Nevertheless, some areas, though rich in these nitrogen compounds, have few plankton. These areas have particularly low concentrations of iron, and oceanographers hypothesize that this shortage of iron prevents plankton from thriving. However, an experimental release of iron compounds into one such area failed to produce a thriving plankton population, even though local iron concentrations increased immediately.", "question": "Which of the following, if true, argues most strongly against concluding, on the basis of the information above, that the oceanographers' hypothesis is false?", "answers": "['Certain areas of the ocean support an abundance of plankton despite having particularly low concentrations of iron.', 'A few days after the iron compounds were released, ocean currents displaced the iron-rich water from the surface.', 'The iron compounds released into the area occur naturally in areas of the ocean where plankton thrive.', 'Not all of the nitrogen compounds that are sometimes found in relatively high concentrations in the oceans are nutrients for plankton.']", "label": 1 }, { "id": "train_2575", "context": "Mayor: False alarms from home security systems waste so much valuable police time that in many communities police have stopped responding to alarms from homes whose systems frequently produce false alarms. This policy reduces wastage of police time but results in a loss of protection for some residents. To achieve a comparable reduction in wastage without reducing protection for residents, the council has enacted a measure to fine residents for repeated false alarms.", "question": "Which of the following, if true, casts the most doubt on whether the measure enacted by the council will achieve its goal?", "answers": "['A fine in the amount planned by the council will not cover the expenses police typically incur when they respond to a false alarm.', 'Many home security systems have never produced false alarms.', 'The threat of fines is likely to cause many residents to deactivate their security systems.', 'Homes equipped with security systems are far less likely to be broken into than are homes without security systems.']", "label": 2 }, { "id": "train_2576", "context": "The best way to write a good detective story is to work backward from the crime. The writer should first decide what the crime is and who the perpetrator is, and then come up with the circumstances and clues based on those decisions.", "question": "Which one of the following illustrates a principle most similar to that illustrated by the passage?", "answers": "['Good architects do not extemporaneously construct their plans in the course of an afternoon; an architectural design cannot be divorced from the method of constructing the building.', 'When planning a trip, some people first decide where they want to go and then plan accordingly, but, for most of us, much financial planning must be done before we can choose where we are going.', 'To make a great tennis shot, you should visualize where you want the shot to go. Then you can determine the position you need to be in to execute the shot properly.', 'In solving mathematical problems, the best method is to try out as many strategies as possible in the time allotted. This is particularly effective if the number of possible strategies is fairly small.']", "label": 2 }, { "id": "train_2577", "context": "Among Trinidadian guppies, males with large spots are more attractive to females than are males with small spots, who consequently are presented with less frequent mating opportunities. Yet guppies with small spots are more likely to avoid detection by predators, so in waters where predators are abundant only guppies with small spots live to maturity.", "question": "The situation described above most closely conforms to which one of the following generalizations?", "answers": "['Those organisms that survive the longest have the greatest number of offspring.', 'A trait that is helpful to procreation can also hinder it in certain environments.', 'A trait that helps attract mates is sometimes more dangerous to one sex than to another.', 'Whether a trait is harmful to the organisms of a species can depend on which sex possesses it.']", "label": 1 }, { "id": "train_2578", "context": "This summer, Jennifer, who has worked at KVZ Manufacturing for just over three years, plans to spend with her family the entire four weeks of paid vacation to which she is entitled this year. Anyone who has worked at KVZ Manufacturing for between one and four years is automatically entitled to exactly three weeks paid vacation each year but can apply up to half of any vacation time that remains unused at the end of one year to the next year' s vacation.", "question": "If the statements above are all true, which one of the following must also be true on the basis of them?", "answers": "['If Jennifer continues to work for KVZ Manufacturing, she will only be entitled to three weeks paid vacation next year.', 'Jennifer did not use two weeks of the paid vacation time to which she was entitled last year.', 'Last year Jennifer took only one week of the paid vacation time to which she was entitled.', 'KVZ Manufacturing sometimes allows extra vacation time to employees who need to spend more time with their families.']", "label": 1 }, { "id": "train_2579", "context": "Raisins are made by drying grapes in the sun. Although some of the sugar in the grapes is caramelized in the process, nothing is added. Moreover, the only thing removed from the grapes is the water that evaporates during the drying, and water contains no calories or nutrients. The fact that raisins contain more iron per calorie than grapes do is thus puzzling.", "question": "Which one of the following, if true, most helps to explain why raisins contain more iron per calorie than do grapes?", "answers": "['The body can absorb iron and other nutrients more quickly from grapes than from raisins because of the relatively high water content of grapes.', 'Raisins are often eaten in combination with other iron-containing foods, while grapes are usually eaten by themselves.', 'Since grapes are bigger than raisins, it takes several bunches of grapes to provide the same amount of iron as a handful of raisins does.', 'Caramelized sugar cannot be digested, so its calories do not count toward the calorie content of raisins.']", "label": 3 }, { "id": "train_2580", "context": "Paleontologists recently discovered teeth from several woolly mammoths on an isolated Arctic island where no mammoth fossils had previously been found. The teeth were 25 percent smaller on average than adult mammoth teeth that have been found elsewhere, but they are clearly adult mammoth teeth. Ttherefore, provided that the teeth are representative of their respective populations, woolly mammoths that lived on the island were smaller on average than those that lived elsewhere.", "question": "Which one of the following, if assumed, would allow the conclusion to be properly drawn?", "answers": "['Neither tooth size nor overall body size is completely uniform among adult members of most species, including woolly mammoths.', 'The tooth wear that naturally occurs in many animals over the course of their adult years did not result in a significant change in tooth size among adult woolly mammoths as they aged.', 'Tooth size among adult woolly mammoths was always directly proportional to the overall size of those mammoths.', 'Woolly mammoths of the kind that lived on the island had the same number and variety of teeth as mammoths that lived elsewhere had.']", "label": 2 }, { "id": "train_2581", "context": "Copyright statutes benefit society by providing incentive to produce original works, so some kind of copyright statute is ultimately justified. But these statutes also represent a significant cost to society because they create protected monopolies. In many countries, copyright statutes grant copyright protection for the life of the author plus several decades. This is too long, since the societal benefit from the additional years of copyright is more than offset by the societal cost.", "question": "Which one of the following principles, if valid, most strongly supports the reasoning in the argument?", "answers": "['A statute should not limit rights unless it can be shown that it thereby enhances other rights.', 'A statute should be written in a way that eliminates any appearance of its being inconsistent in its aims.', 'If a statute is to be justified by its benefit to society, it should be designed so that its societal benefit always exceeds its societal cost.', 'A statute should be repealed if the conditions that originally justified enacting the statute no longer hold true.']", "label": 2 }, { "id": "train_2582", "context": "Some companies in fields where skilled employees are hard to find make signing an \"agreement not to compete\" a condition of employment. In such an agreement the employee promises not to go to work for a competing firm for a set period after leaving his or her current employer. Courts are increasingly ruling that these agreements are not binding. Yet paradoxically, for people who signed such agreements when working for competing firms, many firms are unwilling to consider hiring them during the period covered by the agreement.", "question": "Which one of the following, if true, most helps to resolve the paradox?", "answers": "['Many companies will not risk having to become involved in lawsuits, even suits that they expect to have a favorable outcome, because of the associated costs and publicity.', 'Most companies that require their employees to sign agreements not to compete are aware that these documents are not legally binding.', 'Many people who have signed agreements not to compete are unwilling to renege on a promise by going to work for a competing firm.', \"Many companies consider their employees' established relationships with clients and other people outside the company to be valuable company assets.\"]", "label": 0 }, { "id": "train_2583", "context": "The State has proposed a plan requiring all young people to perform community service as a requirement for graduation from high school. The goal is to use the young people to provide services to correct social ills, especially those in housing and crime. Young people, however, should only be required to participate in educational activities to graduate from high school. For that reason, the proposed program should not be implemented.", "question": "Which one of the following is an assumption on which the argument depends?", "answers": "['Community service does not provide any educational benefit to young people.', 'Some of the social ills that could be helped are being ignored by this program.', 'Young people do not cause the social ills mentioned.', 'Community service is not beneficial to young people.']", "label": 0 }, { "id": "train_2584", "context": "The mayor boasts that the average ambulance turnaround time, the time from summons to delivery of the patient, has been reduced this year for top-priority emergencies. This is a serious misrepresentation. This \"reduction\" was produced simply by redefining \"top priority. \" Such emergencies used to include gunshot wounds and electrocutions, the most time-consuming cases. Now they are limited strictly to heart attacks and strokes.", "question": "Which one of the following would strengthen the author's conclusion that it was the redefinition of \"top priority\" that produced the reduction in turnaround time?", "answers": "['Other cities include gunshot wound cases in their category of top-priority emergencies.', \"The mayor redefined the city's financial priorities this year.\", \"One half of all of last year's top-priority emergencies were gunshot wounds and electrocution cases.\", 'Experts disagree with the mayor\\'s definition of \"top-priority emergency. \"']", "label": 2 }, { "id": "train_2585", "context": "Editorial: The threat of harsh punishment for a transgression usually decreases one' s tendency to feel guilt or shame for committing that transgression, and the tendency to feel guilt or shame for committing a transgression reduces a person' s tendency to commit transgressions. Thus, increasing the severity of the legal penalties for transgressions may amplify people' s tendency to ignore the welfare of others.", "question": "Which one of the following is an assumption required by the editorial's argument?", "answers": "['The threat of harsh punishment deters people from committing transgressions only if this threat is at least sometimes carried out.', 'Everyone has at least some tendency to feel guilt or shame for committing extremely severe transgressions.', 'People who are concerned about threats to their own well-being tend to be less concerned about the welfare of others.', 'At least some actions that involve ignoring the welfare of others are transgressions.']", "label": 3 }, { "id": "train_2586", "context": "Red blood cells in which the malarial-fever parasite resides are eliminated from a person's body after 120 days. Because the parasite cannot travel to a new generation of red blood cells, any fever that develops in a person more than 120 days after that person has moved to a malaria-free region is not due to the malarial parasite.", "question": "Which of the following, if true, most seriously weakens the conclusion above?", "answers": "['The fever caused by the malarial parasite may resemble the fever caused by flu viruses.', 'Many malarial symptoms other than the fever, which can be suppressed with antimalarial medication, can reappear within 120 days after the medication is discontinued.', 'The anopheles mosquito, which is the principal insect carrier of the malarial parasite, has been eradicated in many parts of the world.', \"In some cases, the parasite that causes malarial fever travels to cells of the spleen, which are less frequently eliminated from a person's body than are red blood cells.\"]", "label": 3 }, { "id": "train_2587", "context": "Brownlea' s post office must be replaced with a larger one. The present one cannot be expanded. Land near the present location in the center of town is more expensive than land on the outskirts of town. Since the cost of acquiring a site is a significant part of the total construction cost, the post office clearly could be built more cheaply on the outskirts of town.", "question": "Which one of the following, if true, most seriously undermines the argument's stated conclusion?", "answers": "['If the new post office is built near the center of town, disruptions to city traffic would have to be minimized by taking such steps as doing some construction work in stages at night and on weekends.', 'The new post office will have to be built in accordance with a demanding new citywide building code.', 'If the new post office is built on the outskirts of town, residents will make decreased use of post office boxes, with the result that mail carriers will have to deliver more mail to homes.', 'If the new post office is built on the outskirts of town, it will require a parking lot, but if sited near the present post office it will not.']", "label": 3 }, { "id": "train_2588", "context": "In Kravonia, the average salary for jobs requiring a college degree has always been higher than the average salary for jobs that do not require a degree. Current enrollments in Kravonia' s colleges indicate that over the next four years the percentage of the Kravonian workforce with college degrees will increase dramatically. Ttherefore, the average salary for all workers in Kravonia is likely to increase over the next four years.", "question": "Which of the following is an assumption on which the argument depends?", "answers": "['The higher average salary for jobs requiring a college degree is not due largely to a scarcity among the Kravonian workforce of people with a college degree.', 'The average salary in Kravonia for jobs that do not require a college degree will not increase over the next four years.', 'The percentage of Kravonians who attend college in order to earn higher salaries is higher now than it was several years ago.', 'Kravonians with more than one college degree earn more, on average, than do Kravonians with only one college degree.']", "label": 0 }, { "id": "train_2589", "context": "Politician: It is widely accepted that because democratic politics cannot exist without debate about political issues, and because self-governance flourishes when citizens are allowed to express their feelings verbally, a democratically governed society should refrain from interfering in individual citizens' speech. I argue that a democratically governed society should also refrain from exercising strict control over the clothing and grooming of its citizens, for this is clearly a venue of self-expression, and it can also serve to make a variety of political statements, without using words.", "question": "A logical strategy used in the politician's argument is to", "answers": "['reach a conclusion about what democratically governed societies actually do based on premises about what democratically governed societies should do', 'support a conclusion by claiming that it is widely accepted', 'reach a general conclusion based on the absence of clear counterexamples to an empirical thesis', 'reach a conclusion based on evidence that is similar to evidence commonly thought to support an analogous case']", "label": 3 }, { "id": "train_2590", "context": "It is a given that to be an intriguing person, one must be able to inspire the perpetual curiosity of others. Constantly broadening one' s abilities and extending one' s intellectual reach will enable one to inspire that curiosity. For such a perpetual expansion of one' s mind makes it impossible to be fully comprehended, making one a constant mystery to others.", "question": "Which one of the following most accurately expresses the conclusion drawn in the argument above?", "answers": [ "If one's mind becomes impossible to fully comprehend, one will always be a mystery to others.", "If one constantly broadens one's abilities and extends one's intellectual reach, one will always have curiosity.", "If one constantly broadens one's abilities and extends one's intellectual reach, one will be able to inspire the perpetual curiosity of others.", "To inspire the perpetual curiosity of others, one must constantly broaden one's abilities and extend one's intellectual reach." ], "label": 2 }, { "id": "train_2591", "context": "Since Jackie is such a big fan of Moral Vacuum' s music, she will probably like The Cruel Herd' s new album. Like Moral Vacuum, The Cruel Herd on this album plays complex rock music that employs the acoustic instrumentation and harmonic sophistication of early sixties jazz. The Cruel Herd also has very witty lyrics, full of puns and sardonic humor, like some of Moral Vacuum' s best lyrics.", "question": "Which one of the following, if true, most strengthens the argument?", "answers": "[\"Jackie has not previously cared for The Cruel Herd, but on the new album The Cruel Herd's previous musical arranger has been replaced by Moral Vacuum's musical arranger.\", \"Jackie's favorite Moral Vacuum songs have lyrics that are somber and marked by a strong political awareness.\", 'Like Moral Vacuum, The Cruel Herd regularly performs in clubs popular with many students at the university that Jackie attends.', \"Though The Cruel Herd's previous albums' production quality was not great, the new album is produced by one of the most widely employed producers in the music industry.\"]", "label": 0 }, { "id": "train_2592", "context": "Council member P: Alarmists are those who see an instance of pollution and exaggerate its significance into a major character fault of society. Such alarmists fail to distinguish the incident and the behavior that caused it from the disposition of people not to pollute. Council member Q: To think that there is a lot of pollution based on the discovery of a serious single instance of pollution is simply an application of the widely accepted principle that actions tend to follow the path of least resistance, and it is surely easier to pollute than not to pollute.", "question": "Council members P and Q disagree over whether", "answers": "['people are responsible for pollution', 'people are inclined to pollute', 'actions tend to follow the path of least resistance', 'people can change their behavior and not pollute']", "label": 1 }, { "id": "train_2593", "context": "Journalist: The advice of social scientists is frequently overlooked by politicians making social policy. Because it is not unreasonable to discount scientific assertions backed by weak evidence, politicians should not generally be criticized for ignoring social science, for social scientists, unlike physical scientists, seldom agree on the claims made even within their own specialty.", "question": "Which one of the following is an assumption required by the journalist's argument?", "answers": "['The failure of scientists to agree that a claim within their specialty is true can indicate that the evidence for the claim is not strong.', 'Only policy that is based on scientific findings is credible.', 'Politicians should follow the advice of experts on issues about which those experts agree among themselves.', \"When creating policy, politicians' decisions should be determined by relevant scientific findings, except when the evidence for those findings is weak.\"]", "label": 0 }, { "id": "train_2594", "context": "In the past century, North America has shifted its main energy source first from wood to coal, then from coal to oil and natural gas. With each transition, the newly dominant fuel has had less carbon and more hydrogen than its predecessor had. It is logical to conclude that in the future the main energy source will be pure hydrogen.", "question": "Which one of the following expresses a general principle that could underlie the argument?", "answers": "['If each step in a series of changes involves a decrease of one attribute of the thing undergoing the change and an increase of another, the series will terminate with the first attribute eliminated and only the second attribute present.', 'If the second stage of a process has been completed more quickly than the first stage, the third stage of that process will be completed more quickly than the second stage.', 'If each of two desirable attributes belongs to a useful substance, then the most useful form of that substance will have those two attributes in equal amounts.', 'If one substance is better for a certain purpose than another substance is, then the best substance for that purpose is one that includes among its attributes all of the attributes of the first substance and none of the attributes of the second substance.']", "label": 0 }, { "id": "train_2595", "context": "Aviation Analyst: To combat human error involved in air traffic control, Miro Jets Inc. has recently installed each of its DC-10 passenger planes with a special anti-collision device. The device alerts the DC-10, via a red light, when another plane is slightly more than three minutes away from a potential collision. The red light comes on and stays on until the approaching plane is safely out of range. Aviation experts at Miro Jet Inc. have determined that three minutes is ample time for a plane to divert its respective flight path to avoid a collision. Ttherefore, if the red light on the anti-collision device is off, the DC-10 is more than three minutes flying time from any plane.", "question": "Which of the following, if true, most fundamentally calls into question the aviation analyst's argument?", "answers": "['Recently, a near collision in which both planes were less than 90 seconds flying distance from each other was averted only by the prompt actions of air traffic controllers.', \"When two DC-10's approach each other the red lights on each aircraft do not necessarily turn on simultaneously.\", 'Some aviation experts warn that in certain cases three minutes may not provide sufficient time for aircrafts to maneuver without causing on board injuries.', 'The anti-collision device only signals planes of oncoming directions, not those planes that are flying in parallel.']", "label": 3 }, { "id": "train_2596", "context": "Tech start-ups need more than one million dollars in initial investments for product development and marketing to survive. Of course, tech start-ups also need a compelling product that fills a need in the market. In addition, tech start-ups need to hit the market within twelve months of development, or else they will risk becoming obsolete. Jason has raised two hundred thousand dollars in initial investments for a video chat application. There are currently six video chat applications on the market, but Jason believes his recently-developed application offers a superior product. Following a blind test, participants preferred his video chat application over the competition. Jason' s video chat application stands the best chance at surviving if he. . .", "question": "Which one of the following best completes the passage?", "answers": "['runs another blind test trial to confirm the results.', 'seeks a larger initial investment.', 'spends his existing investment on marketing.', 'hires another developer.']", "label": 1 }, { "id": "train_2597", "context": "Psychologists recently conducted a study in which people from widely disparate cultures were asked to examine five photographs. Each photograph depicted the face of a person expressing one of five basic human emotions -- fear, happiness, disgust, anger, and sadness. The people in the study were asked to identify the emotion being expressed in each photograph. For each photograph, everyone identified the same emotion. This shows that people are genetically predisposed to associate certain facial expressions with certain basic emotions.", "question": "Which one of the following is an assumption on which the argument depends?", "answers": "['If there is a behavior common to people of widely disparate cultures, then there is probably a genetic predisposition to that behavior.', \"One's emotional disposition is not influenced by one's culture.\", 'For each photograph, the emotion that the subjects agreed was being expressed was the emotion that the person photographed was, in fact, feeling.', 'The people whose faces were depicted in the photographs were not all from the same culture.']", "label": 0 }, { "id": "train_2598", "context": "Art critic: The aesthetic value of a work of art lies in its ability to impart a stimulating character to the audience' s experience of the work.", "question": "Which one of the following judgments most closely conforms with the principle cited above?", "answers": "['This painting is aesthetically valuable because it was painted by a highly controversial artist.', 'This painting is aesthetically deficient because it is an exact copy of a painting done 30 years ago.', 'This sculpted four-inch cube is beautiful because it is carved from material which, although much like marble, is very rare.', 'This poem is aesthetically deficient because it has little impact on its audience.']", "label": 3 }, { "id": "train_2599", "context": "The fact that people who exercise vigorously are sick less often than average does not prove that vigorous exercise prevents illness, for whether one exercises vigorously or not depends in part on one' s preexisting state of health.", "question": "The reasoning in which one of the following arguments is most similar to that in the argument above?", "answers": "['The fact that two diseases have similar symptoms does not establish that they have the same underlying cause, for dissimilar causes can have similar effects.', 'Musical and mathematical skills are often produced by the same talent for perceiving abstract patterns, so the fact that some mathematicians are not skilled musicians does not prove that they lack the talent that can produce musical skill.', 'Having strong verbal skills encourages people to read more, so the fact that habitual readers tend to be verbally skilled does not prove that reading produces verbal skill.', 'The fact that taller children often outperform other children at basketball does not show that height is a decisive advantage in basketball, for taller children tend to play basketball more frequently than do other children.']", "label": 2 }, { "id": "train_2600", "context": "Moralist: Humans have a natural disposition to altruism -- that is, to behavior that serves the needs of others regardless of one' s own needs -- but that very disposition prevents some acts of altruism from counting as moral. Reason plays an essential role in any moral behavior. Only behavior that is intended to be in accordance with a formal set of rules, or moral code, can be considered moral behavior.", "question": "Which one of the following most accurately states the main conclusion of the moralist's argument?", "answers": "['All moral behavior is motivated by altruism.', 'Altruism develops through the use of reason.', 'Behavior must serve the needs of others in order to be moral behavior.', 'Not all altruistic acts are moral behavior.']", "label": 3 }, { "id": "train_2601", "context": "Party spokesperson: The opposition party' s proposal to stimulate economic activity in the province by refunding $600 million in provincial taxes to taxpayers, who could be expected to spend the money, envisions an illusory benefit. Since the province' s budget is required to be in balance, either new taxes would be needed to make up the shortfall, in which case the purpose of the refund would be defeated, or else workers for the province would be dismissed. So either the province' s taxpayers or its workers, who are also residents of the province, will have the $600 million to spend, but there can be no resulting net increase in spending to stimulate the province' s economy.", "question": "The spokesperson proceeds by", "answers": "['seeking to show that the assumption that taxpayers would spend money that might be refunded to them is dubious', 'reinterpreting a term that is central to an opposing argument', 'arguing that a predicted advantage would be offset by an accompanying disadvantage', 'casting doubt on the motives of opponents']", "label": 2 }, { "id": "train_2602", "context": "A government ought to protect and encourage free speech, because free speech is an activity that is conducive to a healthy nation and thus is in the best interest of its people.", "question": "The main conclusion above follows logically if which one of the following is assumed?", "answers": "['Basic, inalienable rights of the people ought to be protected and encouraged by government.', \"Universal human rights that are in the best interest of the people ought to be protected and encouraged by a nation's government.\", \"An activity that is in the best interest of the people ought to be protected and encouraged by a nation's government.\", 'A government ought to protect and encourage an activity that is conducive to the interests of that government.']", "label": 2 }, { "id": "train_2603", "context": "Film director: It' s inaccurate to say that filmgoers stayed away from my film because it received one or two negative reviews. My film had such a small audience during its opening weekend simply because it was competing with several other films that appeal to the same type of filmgoer that mine does, and the number of such viewers is relatively small.", "question": "Which one of the following, if true, most helps to support the film director's explanation?", "answers": "[\"The film director's film received no positive reviews.\", 'Filmgoers seldom see more than one film in a weekend.', 'Each of the other films that the film director alludes to received one or two positive reviews.', 'Most filmgoers are drawn to a variety of kinds of film.']", "label": 1 }, { "id": "train_2604", "context": "Archaeologists in Michigan have excavated a Native American camp near Dumaw Creek. Radiocarbon dating of animal bones found at the site indicates that the camp dates from some time between 1605 and 1755. However, the camp probably dates to no later than 1630, since no European trade goods were found at the site, and European traders were active in the region from the 1620' s onward.", "question": "Which of the following, if true, most strengthens the argument?", "answers": "['Due to trade among Native Americans, some European trade goods would have reached the area before the European traders themselves did.', \"At all camps in the region that have been reliably dated to the late 1620's, remains of European trade goods have been found.\", 'The site is that of a temporary camp that would have been used seasonally for a few years and then abandoned.', 'The first European trade goods to reach the area would have been considered especially valuable and preserved as much as possible from loss or destruction.']", "label": 1 }, { "id": "train_2605", "context": "An unusually severe winter occurred in Europe after the continent was blanketed by a blue haze resulting from the eruption of the Laki Volcano in the European republic of Iceland in the summer of 1984. Thus, it is evident that major eruptions cause the atmosphere to become cooler than it would be otherwise.", "question": "Which of the following statements, if true, most seriously weakens the argument above?", "answers": "[\"A few months after El Chichon's large eruption in April 1982, air temperatures throughout the region remained higher than expected, given the long-term weather trends.\", 'The climatic effects of major volcanic eruptions can temporarily mask the general warming trend resulting from an excess of carbon dioxide in the atmosphere.', 'Three months after an early springtime eruption in South America during the late nineteenth century, sea surface temperatures near the coast began to fall.', 'There is a strong statistical link between volcanic eruptions and the severity of the rainy season in India.']", "label": 0 }, { "id": "train_2606", "context": "Since Mayor Drabble always repays her political debts as soon as possible, she will almost certainly appoint Lee to be the new head of the arts commission. Lee has wanted that job for a long time, and Drabble owes Lee a lot for his support in the last election.", "question": "Which one of the following is an assumption on which the argument depends?", "answers": "['There is no one to whom Mayor Drabble owes a greater political debt for support in the last election than the political debt she owes to Lee.', \"Whether Lee is qualified to head the arts commission is irrelevant to Mayor Drabble's decision.\", 'Lee is the only person to whom Mayor Drabble owes a political debt who would be willing to accept an appointment from her as the new head of the arts commission.', 'Mayor Drabble has no political debt that is both of longer standing than the one she owes to Lee and could as suitably be repaid by an appointment to be the new head of the arts commission.']", "label": 3 }, { "id": "train_2607", "context": "If a person chooses to walk rather than drive, there is one less vehicle emitting pollution into the air than there would be otherwise. Ttherefore if people would walk whenever it is feasible for them to do so, then pollution will be greatly reduced.", "question": "Which one of the following is an assumption on which the argument depends?", "answers": "['There are people who never drive but who often walk.', 'People sometimes drive when it is feasible to walk instead.', 'Cutting down on pollution can be achieved in a variety of ways.', 'Taking public transportation rather than driving is not always feasible.']", "label": 1 }, { "id": "train_2608", "context": "Like a number of other articles, Ian Raghnall' s article relied on a recent survey in which over half the couples applying for divorces listed \"money\"as a major problem in their marriages. Raghnall' s conclusion from the survey data is that financial problems are the major problem in marriages and an important factor contributing to the high divorce rate. Yet couples often express other types of marital frustrations in financial terms. Despite appearances, the survey data do not establish that financial problems are the major problem in contemporary marriages.", "question": "Which one of the following sentences best expresses the main point of the passage?", "answers": "['Many articles wrongly claim that financial problems are the major factor contributing to the divorce rate.', \"The conclusion drawn in Raghnall's article is inadequately justified.\", 'Over half the couples applying for divorces listed money as a major problem in their marriages.', 'Financial problems are not an important factor contributing to the divorce rate.']", "label": 1 }, { "id": "train_2609", "context": "Rapid population growth can be disastrous for a small city. Ideally there should be at least one municipal employee for every hundred residents; when too many people move in at once, city services responsible for utilities and permits are quickly overloaded. Most city budgets do not allow for the immediate hiring of new staff.", "question": "Which one of the following, if true, most strengthens the argument?", "answers": "['New residents of any city bring with them new ideas about how a city should be run.', 'New residents of most small cities do not start paying city taxes for at least a year.', 'Some large cities can absorb rapid population growth more readily than many small cities can.', 'During budget shortages, small cities tend to place a high priority on basic municipal services while cutting back on less essential services.']", "label": 1 }, { "id": "train_2610", "context": "The advent of chemical fertilizers led the farmers in a certain region to abandon the practice of periodically growing a \"green-manure\" crop, such as alfalfa, in a field to rejuvenate its soil. As a result, the soil structure in a typical farm field in the region is poor. So to significantly improve the soil structure, farmers will need to abandon the use of chemical fertilizers.", "question": "The argument relies on the assumption that", "answers": "['most, if not all, farmers in the region who abandon the use of chemical fertilizers will periodically grow alfalfa', 'many, if not all, farmers in the region will not grow green-manure crops unless they abandon the use of chemical fertilizers', 'the most important factor influencing the soil quality of a farm field is soil structure', 'applying chemical fertilizers to green-manure crops, such as alfalfa, has no positive effect on their growth']", "label": 1 }, { "id": "train_2611", "context": "Two doctrines have been greatly influential in this century. The first holds that the explanation of any historical event must appeal to economic factors. The second attempts to account psychologically for all historical events, especially in terms of early childhood experience. Both doctrines, however, are mistaken. Certainly there have been events that were due both to economic forces and to the nature of the early childhood experiences of the major participants in the event.", "question": "The argument depends on assuming which one of the following?", "answers": "['One is likely to find that both economic and psychological explanations have been proposed for any given historical event.', 'Appeals to both economic and psychological factors are needed to understand any historical event properly.', 'The first doctrine precludes any noneconomic factors in explanations of historical events.', 'The second doctrine places importance only on childhood experiences.']", "label": 2 }, { "id": "train_2612", "context": "Environmentalist: It takes less energy to make molten glass from recycled glass than from raw materials. Once the recycled glass or raw materials have been turned into molten glass, making bottles from recycled glass follows the same process as making bottles from raw materials. Obviously, soft drink bottlers who make a large percentage of their bottles from recycled glass have significant energy savings. Ttherefore, by using recycled glass instead of glass made from raw materials, bottlers can lower their costs and benefit the environment at the same time.", "question": "Which one of the following is an assumption on which the argument relies?", "answers": "['Purchasing and transport costs are not so much greater for recycled glass than for raw materials that they outweigh the savings in energy costs resulting from the use of recycled glass.', 'The amount of glass that is currently recycled each year is enough to supply the major soft drink bottlers with materials for a large percentage of the glass bottles they make that year.', 'Most consumers are not able to distinguish bottles made from recycled glass from glass bottles made from raw materials.', 'The process of making molten glass from recycled glass requires fewer steps than does the process of making molten glass from raw materials.']", "label": 0 }, { "id": "train_2613", "context": "When a community opens a large shopping mall, it often expects a boost to the local economy, and in fact a large amount of economic activity goes on in these malls. Yet the increase in the local economy is typically much smaller than the total amount of economic activity that goes on in the mall.", "question": "Which one of the following, if true, most helps to explain the discrepancy described above?", "answers": "['When large shopping malls are new they attract a lot of shoppers but once the novelty has worn off they usually attract fewer shoppers than does the traditional downtown shopping district.', 'Most of the jobs created by the construction of a large shopping mall are temporary, and most of the permanent jobs created are low paying.', 'Most of the money spent in a large shopping mall is spent by tourists who are drawn specifically by the mall and who would not have visited the community had that mall not been built.', 'Most of the money spent in a large shopping mall is money that would have been spent elsewhere in the same community had that mall not been built.']", "label": 3 }, { "id": "train_2614", "context": "For similar cars and comparable drivers, automobile insurance for collision damage has always cost more in Greatport than in Fairmont. Police studies, however, show that cars owned by Greatport residents are, on average, slightly less likely to be involved in a collision than cars in Fairmont. Clearly, ttherefore, insurance companies are making a greater profit on collision damage insurance in Greatport than in Fairmont.", "question": "In evaluating the argument, it would be most useful to compare", "answers": "[\"the condition of Greatport's roads and streets with the condition of Fairmont's roads and streets\", 'the rates Greatport residents pay for other forms of insurance with the rates paid for similar insurance by residents of Fairmont', 'the cost of repairing collision damage in Greatport with the cost of repairing collision damage in Fairmont', 'the level of traffic congestion in Greatport with the level of traffic congestion in Fairmont']", "label": 2 }, { "id": "train_2615", "context": "A country ' s employment in the manufacturing industry has dramatically decreased over the last thirty years. During that same period, the country has led the world in developing automation. Ttherefore, automation is the reason behind the decreased unemployment in the manufacturing industry.", "question": "The author relies most upon which of the following assumptions?", "answers": "['The manufacturing industry uses more automation than any other economic sector.', 'The manufacturing industry is the only economic sector impacted by automation.', 'The manufacturing industry is driving the automation.', 'The manufacturing industry uses automation.']", "label": 3 }, { "id": "train_2616", "context": "Physician: In order to investigate diseases caused by hormonal imbalances, a certain researcher wants to study, among others, 200 children whose pituitary glands fail to produce typical amounts of Human Growth Hormone (HGH). The study would involve administering a synthetic version of HGH to the children over a two-year period. But medical research should be permitted only if it is likely to reveal important information about a medical condition and is known to pose only a minimal risk to the subjects. The researcher' s proposed study should be prohibited.", "question": "Which one of the following, if true, would most help to justify the physician's argumentation?", "answers": "['About 10, 000 children have already been given synthetic HGH without obvious side effects.', 'Obtaining informed consent from children is impossible, because they are not yet mature enough to understand complex medical issues.', 'Although hormonal imbalances can cause disease, the imbalances themselves do not constitute a medical condition.', 'The long-term effects of synthetic HGH have never been tested and are unknown.']", "label": 3 }, { "id": "train_2617", "context": "Anna: Did you know that rainbows always occur opposite the sun, appearing high in the sky when the sun is low, and low in the sky when the sun is high? The Roman scholar Pliny the Elder claimed that this was so, in the first century A. D. William: His claim cannot be correct. After all, Pliny the Elder wrote that there are tribes of dogheaded people and beings with no heads or necks but with eyes on their shoulders, and said that smearing snails on your forehead cures headaches!", "question": "William's argument against Anna's claims about rainbows is most vulnerable to criticism because it", "answers": [ "inappropriately distorts Anna's conclusion, making it appear more extreme than it really is", "implies that Pliny the Elder's writings are too outdated to be of any value", "illicitly infers that, because Pliny the Elder made some incorrect assertions, Pliny the Elder's assertions about rainbows are also incorrect", "accepts the assertions of an ancient scholar without presenting contemporary verification of that scholar's views" ], "label": 2 }, { "id": "train_2618", "context": "Louise McBride, a homeowner, filed a complaint against a nearby nightclub through the Licensing Bureau, a government agency. Although regulations clearly state that Form 283 is to be used for formal complaints, Bureau staff gave McBride Form 5, which she used with the intention of filing a formal complaint. The nightclub argues that the complaint should be dismissed because the incorrect form was used. But that would be unfair.", "question": "Which one of the following principles, if valid, most helps to justify the judgment that dismissing the complaint would be unfair?", "answers": "[\"A government agency should not make its procedures so complex that even the agency's employees cannot understand the procedures.\", \"It is unfair for someone's complaint to be dismissed because of an incorrect action on the part of a government agency.\", 'People who wish to file complaints through the Licensing Bureau should be informed of all relevant regulations.', 'It is unfair for a business to be subject to a formal complaint unless the complaint is made in a way that provides the business with an opportunity to defend itself.']", "label": 1 }, { "id": "train_2619", "context": "New Age philosopher: Nature evolves organically and nonlinearly. Furthermore, it can best be understood as a whole; its parts are so interconnected that none could exist without support from many others. Ttherefore, attaining the best possible understanding of nature requires an organic, holistic, nonlinear way of reasoning rather than the traditional linear reasoning of science, which proceeds through experiments on deliberately isolated parts of nature.", "question": "The reasoning in the New Age philosopher's argument is most vulnerable to criticism on the grounds that the argument", "answers": "['overlooks the possibility that the overall structure of a phenomenon is not always identical to the overall structure of the reasoning that people do about that phenomenon', 'fails to distinguish adequately between the characteristics of a phenomenon as a whole and those of the deliberately isolated parts of that phenomenon', 'takes for granted that a phenomenon that can best be understood as having certain properties can best be understood only through reasoning that shares those properties', \"takes for granted that if a statement must be true for the argument's conclusion to be true, then that statement's truth is sufficient for the truth of the conclusion\"]", "label": 2 }, { "id": "train_2620", "context": "Pamela: Physicians training for a medical specialty serve as resident staff physicians in hospitals. They work such long hours -- up to 36 consecutive hours -- that fatigue impairs their ability to make the best medical decisions during the final portion of their shifts. Quincy: Thousands of physicians now practicing have been trained according to the same regimen, and records show they generally made good medical decisions during their training periods.", "question": "Why should what has worked in the past be changed now? Which one of the following, if true, is the most effective counter Pamela might make to Quincy's argument?", "answers": "['Because medical reimbursement policies now pay for less recuperation time in hospitals, patients in hospitals are, on the average, more seriously ill during their stays than in the past.', \"The training of physicians should include observation and recognition of the signs indicating a hospitalized patient's progress or decline over a period of at least 36 hours.\", 'It is important that emergency-room patients receive continuity of physician care, insofar as possible, over the critical period after admission, generally 24 hours.', 'The load of work on resident physicians-in-training varies according to the medical specialty for which each is being trained.']", "label": 0 }, { "id": "train_2621", "context": "Whenever she considers voting in an election to select one candidate for a position and there is at least one issue important to her, Kay uses the following principle in choosing which course of action to take: it is acceptable for me to vote for a candidate whose opinions differ from mine on at least one issue important to me whenever I disagree with each of the other candidates on even more such issues; it is otherwise unacceptable to vote for that candidate. In the upcoming mayoral election, the three candidates are Legrand, Medina, and Norton. There is only one issue important to Kay, and only Medina shares her opinion on that issue.", "question": "If the statements in the passage are true, which one of the following must also be true about Kay's course of action in any election to select one candidate for a position?", "answers": "['If she agrees with a particular candidate on only one issue important to her, it is unacceptable for her to vote for that candidate.', 'If she agrees with each of the candidates on most of the issues important to her, it is unacceptable for her to vote for any candidate in the election.', 'If she disagrees with each of the candidates on exactly three issues important to her, it is unacceptable for her to vote for any candidate in the election.', 'If there are more issues important to her on which she disagrees with a particular candidate than there are such issues on which she agrees with that candidate, it is unacceptable for her to vote for that candidate.']", "label": 2 }, { "id": "train_2622", "context": "A survey of clerical workers' attitudes toward their work identified a group of secretaries with very positive attitudes. They responded \"Strongly agree\" to such statements as \"I enjoy word processing\"and \"I like learning new secretarial skills. \" These secretaries had been rated by their supervisors as excellent workers -- far better than secretaries whose attitudes were identified as less positive. Clearly these secretaries' positive attitudes toward their work produced excellent job performance.", "question": "Which one of the following identifies a reasoning error in the argument?", "answers": "[\"It identifies the secretaries' positive attitudes as the cause of their excellent job performance although their attitudes might be an effect of their performance.\", 'It attempts to prove a generalization about job performance by using the single example of clerical workers.', 'It uses the term \"positive attitudes\" to mean two different things.', 'It does not consider the possibility that secretaries with very positive attitudes toward their work might also have had very positive attitudes toward other activities.']", "label": 0 }, { "id": "train_2623", "context": "Urban air contains more sulfur dioxide than does rural air, and plants in cities typically grow more slowly than do plants in rural areas. In an experiment to see how much of the difference in growth is due to sulfur dioxide, classes in an urban and a rural school grew plants in greenhouses at their schools and filtered the greenhouse air to eliminate sulfur dioxide. Plants in the urban greenhouse grew more slowly than those in the rural greenhouse.", "question": "Which of the following, if true, would it be most important to take into account in evaluating the result?", "answers": "['The urban school was located in a part of the city in which levels of sulfur dioxide in the air were usually far lower than is typical for urban areas.', 'The urban class conducting the experiment was larger than the rural class conducting the experiment.', 'Heavy vehicular traffic such as is found in cities constantly deposits grime on greenhouse windows, reducing the amount of light that reaches the plants inside.', \"Because of the higher levels of sulfur dioxide in the air at the urban school, the air filters for the urban school's greenhouse were changed more frequently than were those at the rural school.\"]", "label": 2 }, { "id": "train_2624", "context": "Snowmaking machines work by spraying a mist that freezes immediately on contact with cold air. Because the sudden freezing kills bacteria, QuickFreeze is planning to market a wastewater purification system that works on the same principle. The process works only when temperatures are cold, however, so municipalities using it will still need to maintain a conventional system.", "question": "Which of the following, if true, provides the strongest grounds for a prediction that municipalities will buy QuickFreeze's purification system despite the need to maintain a conventional purification system as well?", "answers": "['Conventional wastewater purification systems have not been fully successful in killing bacteria at cold temperatures.', \"During times of warm weather, when it is not in use, QuickFreeze's purification system requires relatively little maintenance.\", 'Bacteria are not the only impurities that must be removed from wastewater.', 'Many municipalities have old wastewater purification systems that need to be replaced.']", "label": 0 }, { "id": "train_2625", "context": "Federal investigators, called in at the request of the management of Ploutos National Bank, recently apprehended a ring of seven embezzlers from among the bank' s employees. The bank management decided to call in the federal investigators when they were unable to account for millions of dollars missing in their budget for this year. All the funds those seven individuals embezzled have been returned to the bank, and that accounts for about two thirds of the total amount missing. All seven of the accused have plea-bargained to avoid trial and are now serving in prison on reduced sentences.", "question": "Which of following conclusions can most properly be drawn from the information above?", "answers": "['Federal investigators have the means at their disposal to detect any large illegal transfers of money.', 'The Ploutos National Bank still has reason to suspect more embezzlers beyond the seven apprehended by Federal investigators.', 'In initiating a federal investigation of their own company, the managers of Ploutos National Bank were subject to no fees from the federal government.', 'The seven embezzlers would have wound up with longer prison sentences if they had not plea bargained.']", "label": 1 }, { "id": "train_2626", "context": "Advertisement: Campfire Jack' s Breakfast Sausage, made with our secret family recipe, is simply tops for taste. And here is the proof: In a recent market survey, 8 out of every 10 shoppers who expressed a preference said that Campfire Jack' s was the only breakfast sausage for them, no ifs, ands, or buts.", "question": "Of the following, which one is the strongest reason why the advertisement is potentially misleading?", "answers": "['Shoppers who buy breakfast sausage might buy only breakfast sausage.', \"The preference for Campfire Jack's brand might be based on such a factor as a remarkably low price.\", 'Other brands of breakfast sausage might also be made with a secret family recipe.', 'No market survey covers more than a sizable minority of the total population of consumers.']", "label": 1 }, { "id": "train_2627", "context": "Gwin: Apparently the community playhouse is going out of business. This means that there is little to no cultural feeling in the town of Hillview. If you' re planning on opening another playhouse in Hillview, I would think again. Chambal: It' s true that the playhouse is going out of business. However, its closing has nothing to do with the amount of culture in Hillview. All this means is that there is perfect opportunity for another playhouse to come to Hillview.", "question": "The dialogue provides the most support for the claim that Gwin and Chambal disagree over which of the following?", "answers": "['Playhouses can close because of insufficient cultural feeling.', 'The plays in Hillview were of inferior quality.', 'Other cultural spaces have gone out of business in Hillview.', 'It is a good idea for another playhouse to come to Hillview.']", "label": 3 }, { "id": "train_2628", "context": "Advertisement: Most nutritionists recommend eating fish twice a week. Eating tilapia fillets is a perfect choice for those who want the benefits of eating fish but do not care for the taste of fish. Tilapia fillets lack the . strong fishy taste that many people find objectionable.", "question": "Which one of the following, if true, most seriously weakens the advertisement's argument?", "answers": "[\"Eating more than the recommended amount of fish can cause toxins that are present in high concentrations in many varieties of fish to accumulate m a person's body.\", 'Tilapia fillets contain little of the beneficial fish oils that are the main reason nutritionists recommend eating fish frequently.', 'Most people who do not care for the taste of fish eat less fish than is recommended by most nutntiomsts.', 'People who rarely or never eat fish usually dislike any food with a strong fishy taste.']", "label": 1 }, { "id": "train_2629", "context": "Philosopher: Effective tests have recently been developed to predict fatal diseases having a largely genetic basis. Now, for the first time, a person can be warned well in advance of the possibility of such life-threatening conditions. However, medicine is not yet able to prevent most such conditions. Simply being informed that one will get a disease that is both fatal and incurable can itself be quite harmful to some people. This raises the question of whether such \"early warning\" tests should be made available at all.", "question": "Which one of the following statements is best illustrated by the state of affairs described by the philosopher?", "answers": "['The advance of medicine creates new contexts in which ethical dilemmas can arise.', 'The advance of technology is of questionable value.', 'The advance of medicine fails to provide solutions to every problem.', 'Medical technologies continue to advance, increasing our knowledge and understanding of disease.']", "label": 0 }, { "id": "train_2630", "context": "Earthworms, vital to the health of soil, prefer soil that is approximately neutral on the acid-to-alkaline scale. Since decomposition of dead plants makes the top layer of soil highly acidic, application of crushed limestone, which is highly alkaline, to the soil' s surface should make the soil more attractive to earthworms.", "question": "Which one of the following is an assumption on which the argument depends?", "answers": "['As far as soil health is concerned, aiding the decomposition of dead plants is the most important function performed by earthworms.', 'By itself, acidity of soil does nothing to hasten decomposition of dead plants.', 'Crushed limestone contains available calcium and magnesium, both of which are just as vital as earthworms to healthy soil.', \"After its application to the soil's surface, crushed limestone stays in the soil's top layer long enough to neutralize some of the top layer's acidity.\"]", "label": 3 }, { "id": "train_2631", "context": "Psychologist: Some people contend that children should never be reprimanded. Any criticism, let alone punishment, they say, harms children' s self-esteem. This view is laudable in its challenge to the belief that children should be punished whenever they misbehave, yet it gives a dangerous answer to the question of how often punishment should be inflicted. When parents never reprimand their children, they are in effect rewarding them for unacceptable behavior, and rewarded behavior tends to recur.", "question": "The view that children should never be reprimanded functions in the psychologist's argument as a statement of a position that the psychologist's argument", "answers": "['is designed to establish as well intentioned', 'claims has a serious flaw though is not without value', 'is designed to discredit entirely', 'is designed to establish as true']", "label": 1 }, { "id": "train_2632", "context": "In preagricultural societies, social roles were few and were easily predicted for each phase of a person' s life. Accordingly, interpersonal relations, although not always pleasant or fair, were stable and predictable. Modern society, on the other hand, has thousands of different social roles. Technology and consumerism require that each of us fill highly particularized niches, and these niches are differentiating at an astonishing pace. Ttherefore, __ .", "question": "Which one of the following most logically completes the argument?", "answers": "['modern society is characterized by greater injustice and unpleasantness than were preagricultural societies', \"in modern societies, people must rely on technology to effectively predict and interpret other people's actions\", 'interpersonal relations in modern societies are less stable and less predictable than they were in preagricultural societies', 'the most important difference between modern and preagricultural societies is the variety and type of social roles permitted in each']", "label": 2 }, { "id": "train_2633", "context": "Ornithologist: This bird species is widely thought to subsist primarily on vegetation, but my research shows that this belief is erroneous. While concealed in a well-camouflaged blind, I have observed hundreds of these birds every morning over a period of months, and I estimate that over half of what they ate consisted of insects and other animal food sources.", "question": "The reasoning in the ornithologist's argument is most vulnerable to criticism on the grounds that the argument", "answers": "['fails to specify the nature of the animal food sources, other than insects, that were consumed by the birds', 'neglects the possibility that the birds have different patterns of food consumption during different parts of the day and night', \"assumes, without providing justification, that the feeding behavior of the birds observed was not affected by the ornithologist's act of observation\", \"fails to consider the possibility that the birds' diet has changed since the earlier belief about their diet was formed\"]", "label": 1 }, { "id": "train_2634", "context": "Two centuries ago, Tufe Peninsula became separated from the mainland, isolating on the newly formed Tufe Island a population of Turfil sunflowers. This population' s descendants grow to be, on average, 40 centimeters shorter than Turfil sunflowers found on the mainland. Tufe Island is significantly drier than Tufe Peninsula was. So the current average height of Tufe' s Turfil sunflowers is undoubtedly at least partially attributable to changes in Tufe' s environmental conditions.", "question": "Which of the following is an assumption on which the argument depends?", "answers": "[\"The mainland's environment has not changed in ways that have resulted in Turfil sunflowers on the mainland growing to be 40 centimeters taller than they did two centuries ago.\", 'There were about as many Turfil sunflowers on Tufe Peninsula two centuries ago as there are on Tufe Island today.', 'There are no types of vegetation on Tufe Island that are known to benefit from dry conditions.', 'The 40-centimeter height difference between the Turfil sunflowers on Tufe Island and those on the mainland is the only difference between the two populations.']", "label": 0 }, { "id": "train_2635", "context": "Although the concept of free will is essential to that of moral responsibility, its role in determining responsibility is not the same in all situations. We hold criminals morally responsible for the damage they cause, assuming that they freely chose their activities. But we do not hold someone who has a heart attack while driving morally responsible for the damage caused, if any, even when we have good reason to believe that the heart attack could have been prevented by eating different foods and that one' s choice of diet is made freely.", "question": "The claim that a choice of diet can affect whether or not one has a heart attack plays which one of the following roles in the argument?", "answers": "['It is a subsidiary conclusion of the argument.', 'It is used to show that we should hold someone morally responsible for damages caused by having a heart attack while driving.', 'It is cited as evidence that our concept of moral responsibility should be the same in all situations.', 'It is used in support of the conclusion of the argument.']", "label": 3 }, { "id": "train_2636", "context": "Commissioner: I have been incorrectly criticized for having made my decision on the power plant issue prematurely. I based my decision on the report prepared by the neighborhood association and, although I have not studied it thoroughly, I am sure that the information it contains is accurate. Moreover, you may recall that when I received input from the neighborhood association on jail relocation, I agreed with its recommendation.", "question": "The commissioner's argument is LEAST vulnerable to which one of the following criticisms?", "answers": "[\"It takes for granted that agreeing with the association's past recommendation helps to justify agreeing with its current recommendation.\", 'It draws a conclusion about the recommendations of the association from incomplete recollections.', \"It takes for granted that the association's information is not distorted by bias.\", \"It takes for granted that the association's report is the only direct evidence that needed to be considered.\"]", "label": 1 }, { "id": "train_2637", "context": "In West Calverton, most pet stores sell exotic birds, and most of those that sell exotic birds also sell tropical fish. However, any pet store there that sells tropical fish but not exotic birds does sell gerbils; and no independently owned pet stores in West Calverton sell gerbils.", "question": "If the statements above are true, which one of the following must be true?", "answers": "['No pet stores in West Calverton that sell tropical fish and exotic birds sell gerbils.', 'No independently owned pet store in West Calverton sells tropical fish but not exotic birds.', 'Any independently owned pet store in West Calverton that does not sell tropical fish sells exotic birds.', 'Most pet stores in West Calverton that are not independently owned do not sell exotic birds.']", "label": 1 }, { "id": "train_2638", "context": "During 2006, the number of applicants who applied to law school from college X tripled, even though the number of students who were declared pre-law majors stayed the same.", "question": "Which of the following, if true, most helps to resolve the apparent discrepancy above?", "answers": "['During 2006, the local law schools around college X unanimously agreed to toughen admissions standards.', 'During 2006, the career counselors at college X gave several presentations to the engineering and psychology departments about careers in law.', 'Career counselors at college X have always supported students who wish to change their majors or areas of study.', 'During 2006, college X held several fundraisers to make an addition to the engineering building.']", "label": 1 }, { "id": "train_2639", "context": "Columnist: Tagowa' s testimony in the Pemberton trial was not heard outside the courtroom, so we cannot be sure what she said. Afterward, however, she publicly affirmed her belief in Pemberton' s guilt. Hence, since the jury found Pemberton not guilty, we can conclude that not all of the jury members believed Tagowa' s testimony.", "question": "Which one of the following describes a flaw in the columnist's reasoning?", "answers": "[\"It fails to consider that jury members sometimes disagree with each other about the significance of a particular person's testimony.\", 'It confuses facts about what certain people believe with facts about what ought to be the case.', \"It presumes, without providing warrant, that a jury's finding a defendant not guilty is evidence of dishonesty on the part of someone who testified against the defendant.\", \"It overlooks that a witness may think that a defendant is guilty even though that witness's testimony in no way implicates the defendant.\"]", "label": 3 }, { "id": "train_2640", "context": "Muriel: I admire Favilla' s novels, but she does not deserve to be considered great writer. The point is that, no matter how distinctive her style may be, her subject matter is simply not varied enough. John: I think you are wrong to use that criterion. A great writer does not need any diversity in subject matter; however, a great writer must at least have the ability to explore a particular theme deeply.", "question": "Which one of the following is a point at issue between Muriel and John?", "answers": "['whether treating a variety of subjects should be a prerequisite for someone to be considered a great writer', 'whether Favilla should be considered a great writer because her style is distinctive', 'whether the number of novels that a novelist has written should be a factor in judging whether that novelist is great', 'whether there are many novelists who are considered to be great but do not deserve to be so considered']", "label": 0 }, { "id": "train_2641", "context": "Leadership depends as much on making one' s followers aware of their own importance as it does on conveying a vivid image of a collective goal. Only if they are convinced both that their efforts are necessary for the accomplishment of this goal, and that these efforts, if expended, will actually achieve it, will people follow a leader.", "question": "If all of the statements above are true, then which one of the following CANNOT be true?", "answers": "['Some leaders who convince their followers of the necessity of their efforts in achieving a goal fail, nevertheless, to lead them to the attainment of that goal.', 'Sometimes people who remain unsure of whether their efforts are needed for the attainment of a collective goal nevertheless follow a leader.', 'Only if one is a leader must one convince people of the necessity of their efforts for the attainment of a collective goal.', \"One who succeeds in conveying to one's followers the relationship between their efforts and the attainment of a collective goal succeeds in leading these people to this goal.\"]", "label": 1 }, { "id": "train_2642", "context": "Laurel: Modem moral theories must be jettisoned, or at least greatly reworked, because they fail to provide guidance in extreme cases, which are precisely the times when people most need guidance. Miriam: A moral theory, like an overcoat, can be quite useful even if it is not useful in every possible situation. Being useful in a wide variety of common circumstances is all we need from a moral theory.", "question": "Laurel's and Miriam's statements provide the most support for the claim that they disagree about whether", "answers": "[\"a moral theory's adequacy depends on its ability to provide guidance in extreme cases\", 'a moral theory developed in the light of extreme cases is unlikely to provide adequate guidance in more usual cases', 'it is preferable to develop a moral theory that provides solutions to all the moral dilemmas that could arise', 'people abandoned earlier moral theories when they encountered dilemmas that those theories did not adequately address']", "label": 0 }, { "id": "train_2643", "context": "Libel is defined as damaging the reputation of someone by making false statements. Ironically, strong laws against libel can make it impossible for anyone in the public eye to have a good reputation. For the result of strong libel laws is that, for fear of lawsuits, no one will say anything bad about public figures.", "question": "Which one of the following principles, if valid, most helps to justify the reasoning in the argument?", "answers": "['The absence of laws against libel makes it possible for everyone in the public eye to have a good reputation.', 'If one makes statements that one sincerely believes, then those statements should not be considered libelous even if they are in fact false and damaging to the reputation of a public figure.', 'Public figures can have good reputations only if there are other public figures who have bad reputations.', 'In countries with strong libel laws, people make negative statements about public figures only when such statements can be proved.']", "label": 2 }, { "id": "train_2644", "context": "Human intelligence is not possible without human emotions. A computer is something that can never have emotions, so for that reason alone a computer will never be able to display intelligence.", "question": "Which one of the following is an assumption on which the argument depends?", "answers": "['Someone or something is intelligent only if it can identify its emotions.', 'Computer technology will not greatly advance beyond its current state.', 'A computer could have emotions only if it could display intelligence.', 'Being intelligent requires the capacity to have emotions.']", "label": 3 }, { "id": "train_2645", "context": "Politician: Now that we are finally cleaning up the industrial pollution in the bay, we must start making the bay more accessible to the public for recreational purposes. Reporter: But if we increase public access to the bay, it will soon become polluted again. Politician: Not true. The public did not have access to the bay, and it got polluted. Ttherefore, if and when the public is given access to the bay, it will not get polluted.", "question": "Which one of the following most closely parallels the flawed pattern of reasoning in the politician's reply to the reporter?", "answers": "['In order to like abstract art, you have to understand it. Ttherefore, in order to understand abstract art, you have to like it.', 'When prisoners convicted of especially violent crimes were kept in solitary confinement, violence in the prisons increased. Ttherefore, violence in the prisons will not increase if such prisoners are allowed to mix with fellow prisoners.', 'If there had been a full moon last night, the tide would be higher than usual today. Since the tide is no higher than usual, there must not have been a full moon last night.', 'The detective said that whoever stole the money would be spending it conspicuously by now. Jones is spending money conspicuously, so he must be the thief.']", "label": 1 }, { "id": "train_2646", "context": "Due to wider commercial availability of audio recordings of authors reading their own books, sales of printed books have dropped significantly.", "question": "Which one of the following conforms most closely to the principle illustrated above?", "answers": "['Because of the rising cost of farm labor, farmers began to make more extensive use of machines.', 'Because of the wide variety of new computer games on the market, sales of high-quality computer video screens have improved.', 'Because neither of the two leading word processing programs has all of the features consumers want, neither has been able to dominate the market.', 'Because a new brand of soft drink entered the market, consumers reduced their consumption of an established brand of soft drink.']", "label": 3 }, { "id": "train_2647", "context": "Only engineering is capable of analyzing the nature of a machine in terms of the successful working of the whole; physics and chemistry determine the material conditions necessary for this success, but cannot express the notion of purpose. Similarly, only physiology can analyze the nature of an organism in terms of organs' roles in the body' s healthy functioning. Physics and chemistry cannot ascertain by themselves any of these operational principles.", "question": "Which one of the following is an assumption required by the analogy?", "answers": "['The functioning of the human organism is machine-like in nature.', 'Physiology as a science is largely independent of physics and chemistry.', 'Biological processes are irreducible to mechanical or chemical processes.', 'The notion of purpose used by engineers to judge the success of machinery has an analog in organisms.']", "label": 3 }, { "id": "train_2648", "context": "In 1975, a province reduced its personal income tax rate by 2 percent for most taxpayers. In 1976, the personal income tax rate for those taxpayers was again reduced by 2 percent. Despite the decreases in the personal income tax rate, the total amount of money collected from personal income taxes remained constant from 1974 to 1975 and rose substantially in 1976.", "question": "Each of the following, if true, could help to resolve the apparent discrepancy described above EXCEPT:", "answers": "['The definition of \"personal income\" used by the province was widened during 1975 to include income received from personal investments.', 'A large number of people from other provinces moved to the province during 1975 and 1976.', 'The personal income tax rate for the wealthiest individuals in the province rose during 1975 and 1976.', \"The province's total revenue from all taxes increased during both 1975 and 1976.\"]", "label": 3 }, { "id": "train_2649", "context": "Political scientist: While voters have a legal right to know what is being done by those whom they elect, there must be limits placed on public access to the detailed workings of the legislative process. Legislators receive little credit for reaching compromises but much criticism for failing to stick to their principles, and thus become less willing to modify their demands so that needed legislation can be passed.", "question": "Which one of the following principles, if valid, would most help to justify the political scientist's reasoning?", "answers": "['It is acceptable to restrict some legal rights if doing so makes the political process more efficient.', 'Legislators should be given credit for reaching compromises that facilitate the passage of needed legislation.', 'Compromise between factions with equally compelling interests can only be effected by unprincipled legislators.', 'Legislation should be written so as to benefit the welfare of all citizens, rather than only a few.']", "label": 0 }, { "id": "train_2650", "context": "Songwriters get much of the money they earn from their songs from radio airplay. A hit song is played thousands of times, and the songwriter is paid for each play. Only a fraction of songwriters actually achieve a hit, however, and even fewer manage to write several. Writers of hit songs are often asked to write songs for movie sound tracks, but they sometimes decline, because although such songs frequently become hits, their writers receive single up-front payments rather than continued revenues from radio airplay.", "question": "If the statements above are true, which one of the following must be true?", "answers": "['Any songwriter who receives revenue from radio airplay has written a hit song.', 'Some songs written for movie sound tracks are played on the radio thousands of times.', 'All songwriters who write songs for movie sound tracks have had their songs played on the radio thousands of times.', 'Some songwriters earn money solely from the radio airplay of their songs.']", "label": 1 }, { "id": "train_2651", "context": "Some paleontologists believe that certain species of dinosaurs guarded their young in protective nests long after the young hatched. As evidence, they cite the discovery of fossilized hadrosaur babies and adolescents in carefully designed nests. But similar nests for hatchlings and adolescents are constructed by modern crocodiles, even though crocodiles guard their young only for a very brief time after they hatch. Hence, __.", "question": "Which one of the following most logically completes the argument?", "answers": "['paleontologists who believe that hadrosaurs guarded their young long after the young hatched have no evidence to support this belief', 'we will never be able to know the extent to which hadrosaurs guarded their young', \"the construction of nests for hatchlings and adolescents is not strong evidence for the paleontologists' belief\", 'it is unclear whether what we learn about hadrosaurs from their fossilized remains tells us anything about other dinosaurs']", "label": 2 }, { "id": "train_2652", "context": "Commemorative plaques cast from brass are a characteristic art form of the Benin culture of West Africa. Some scholars, noting that the oldest surviving plaques date to the 1400s, hypothesize that brass-casting techniques were introduced by the Portuguese, who came to Benin in 1485 A. D. But Portuguese records of that expedition mention cast-brass jewelry sent to Benin' s king from neighboring Ife. So it is unlikely that Benin' s knowledge of brass casting derived from the Portuguese.", "question": "Which of the following, if true, most strengthens the argument?", "answers": "['The Portuguese had no contact with Ife until the 1500s.', 'As early as 1500 A. D. , Benin artists were making brass plaques incorporating depictions of Europeans.', 'In the 1400s the Portuguese did not use cast brass for commemorative plaques.', 'Copper, which is required for making brass, can be found throughout Benin territory.']", "label": 0 }, { "id": "train_2653", "context": "Mark: To convey an understanding of past events, a historian should try to capture what it was like to experience those events. For instance, a foot soldier in the Battle of Waterloo knew through direct experience what the battle was like, and it is this kind of knowledge that the historian must capture. Carla: But how do you go about choosing whose perspective is the valid one? Is the foot soldier' s perspective more valid than that of a general? Should it be a French or an English soldier? Your approach would generate a biased version of history, and to avoid that, historians must stick to general and objective characterizations of the past.", "question": "Carla does which one of the following in disputing Mark's position?", "answers": "[\"questions Mark's presupposition that one person can understand another's feelings\", \"gives reason to believe that Mark's recommendation is motivated by his professional self-interest\", 'questions whether Mark accurately describes the kind of historical writing he deplores', \"argues that the selection involved in carrying out Mark's proposal would distort the result\"]", "label": 3 }, { "id": "train_2654", "context": "Economist: If minimum wage levels are low, employers have a greater incentive to hire more workers than to buy productivity-enhancing new technology. As a result, productivity growth, which is necessary for higher average living standards, falls off. Conversely, high minimum wage levels result in higher productivity. Thus, raising our currently low minimum wage levels would improve the country' s overall economic health more than any hiring cutbacks triggered by the raise would harm it.", "question": "Which one of the following, if true, most strengthens the economist's argument?", "answers": "['Productivity growth in a country usually leads to an eventual increase in job creation.', 'Productivity-enhancing new technology tends to quickly become outdated.', \"The economist's country has seen a slow but steady increase in its unemployment rate over the last decade.\", \"The economist's country currently lags behind other countries in the development of new technology.\"]", "label": 0 }, { "id": "train_2655", "context": "Helen: Reading a book is the intellectual equivalent of investing money: you' re investing time, thereby foregoing other ways of spending that time, in the hope that what you learn will later afford you more opportunities than you' d get by spending the time doing something other than reading that book. Randi: But that applies only to vocational books. Reading fiction is like watching a sitcom: it' s just wasted time.", "question": "Which one of the following most accurately describes the technique Randi uses in responding to Helen's claims?", "answers": "['questioning how the evidence Helen uses for a claim was gathered', \"disputing the scope of Helen's analogy by presenting another analogy\", 'denying the relevance of an example presented by Helen', \"arguing that Helen's reasoning ultimately leads to an absurd conclusion\"]", "label": 1 }, { "id": "train_2656", "context": "In response to several bacterial infections traced to its apple juice, McElligott now flash pasteurizes its apple juice by quickly heating and immediately rechilling it. Intensive pasteurization, in which juice is heated for an hour, eliminates bacteria more effectively than does any other method, but is likely to destroy the original flavor. However, because McElligott' s citrus juices have not been linked to any bacterial infections, they remain unpasteurized.", "question": "The statements above, if true, provide the most support for which one of the following claims?", "answers": "[\"McElligott's citrus juices contain fewer infectious bacteria than do citrus juices produced by other companies.\", \"Apple juice that undergoes intensive pasteurization is less likely than McElligott's apple juice is to contain bacteria.\", 'The most effective method for eliminating bacteria from juice is also the method most likely to destroy flavor.', \"McElligott's apple juice is less likely to contain infectious bacteria than are McElligott's citrus juices.\"]", "label": 1 }, { "id": "train_2657", "context": "To use the pool at City Gym, one must have a membership there. Sarah has a membership at City Gym. She must ttherefore use the pool there at least occasionally.", "question": "The reasoning in the argument is flawed in that the argument", "answers": "['mistakes a policy that is strictly enforced for a policy to which exceptions are made', 'presumes that one or the other of two alternatives must be the case without establishing that no other alternative is possible', 'treats a statement whose truth is required for the conclusion to be true as though it were a statement whose truth ensures that the conclusion is true', 'concludes that a person has a certain attribute simply because that person belongs to a group most of whose members have that attribute']", "label": 2 }, { "id": "train_2658", "context": "The city' s transportation department has noticed that the city' s extensive public bus system generally runs behind schedule on days when traffic is heavier in the city. The transportation commissioner concluded that the traffic prevents the buses from reaching their stops on time, despite the existence of a number of bus-only lanes throughout the city. He has ttherefore proposed creating additional bus-only lanes on city streets to allow more buses to bypass traffic.", "question": "The success of the commissioner's proposal depends on which of the following assumptions?", "answers": "['Automobile drivers will not object to the proposal.', \"The city's bus-only lanes do not contribute substantially to gridlock by reducing the number of lanes available to other traffic.\", 'The city owns enough buses to cover all established routes.', 'An increase in bus fare is not necessary as a result of the proposal.']", "label": 1 }, { "id": "train_2659", "context": "Dean: The mathematics department at our university has said that it should be given sole responsibility for teaching the course Statistics for the Social Sciences. But this course has no more mathematics in it than high school algebra does. The fact that a course has mathematics in it does not mean that it needs to be taught by a mathematics professor, any more than a course approaching its subject from a historical perspective must be taught by a history professor. Such demands by the mathematics department are ttherefore unjustified.", "question": "The dean's argument is most vulnerable to criticism on the grounds that it", "answers": "['presumes, without providing justification, that any policies that apply to history courses must be justified with respect to mathematics courses', 'purports to refute a view by showing that one possible reason for that view is insufficient', 'presumes, without providing justification, that expertise in a subject does not enable one to teach that subject well', 'presumes, without providing justification, that most students are as knowledgeable about mathematics as they are about history']", "label": 1 }, { "id": "train_2660", "context": "A recent poll of a large number of households found that 47 percent of those with a cat had at least one person with a university degree, while 38 percent of households with a dog had at least one person with a university degree. Clearly, people who hold university degrees are more likely to live in a household with a cat than one with a dog.", "question": "The reasoning in the argument is flawed in that the argument", "answers": "['fails to consider how many of the households have at least one person without a university degree', 'ignores the possibility that a significant number of households might have both a cat and a dog', 'takes for granted that there are not significantly more households with a dog than ones with a cat', 'ignores the possibility that two things can be correlated without being causally connected']", "label": 2 }, { "id": "train_2661", "context": "Parent P: Children will need computer skills to deal with tomorrow' s world. Computers should be introduced in kindergarten, and computer languages should be required in high school. Parent Q: That would be pointless. Technology advances so rapidly that the computers used by today' s kindergartners and the computer languages taught in today' s high schools would become obsolete by the time these children are adults.", "question": "Which one of the following, if true, is the strongest logical counter parent P can make to parent Q's objection?", "answers": "['In the process of learning to work with any computer or computer language, children increase their ability to interact with computer technology.', \"Automotive technology is continually advancing too, but that does not result in one's having to relearn to drive cars as the new advances are incorporated into new automobiles.\", \"Throughout history people have adapted to change, and there is no reason to believe that today's children are not equally capable of adapting to technology as it advances.\", \"When technology is advancing rapidly, regular training is necessary to keep one's skills at a level proficient enough to deal with the society in which one lives.\"]", "label": 0 }, { "id": "train_2662", "context": "Downtown Petropolis boasted over 100 large buildings 5 years ago. Since then, 60 of those buildings have been demolished. Since the number of large buildings in a downtown is an indicator of the economic health of that downtown, it is clear that downtown Petropolis is in a serious state of economic decline.", "question": "Which one of the following is an assumption required by the argument?", "answers": "['Significantly fewer than 60 new large buildings have been built in downtown Petropolis during the past 5 years.', 'There have never been significantly more than 100 large buildings in downtown Petropolis.', 'Most of the buildings demolished during the past 5 years were torn down because they were structurally unsound.', 'The large buildings demolished over the past 5 years have been replaced with small buildings built on the same sites.']", "label": 0 }, { "id": "train_2663", "context": "On average, the number of speeding tickets issued in County X every year is three times greater than the number of speeding tickets issued in County Y during the same period. Ttherefore, the number of people who exceed the speed limit must be higher in County X than in County Y.", "question": "Which of the following describes a flaw in the reasoning above?", "answers": "['The argument fails to take into account that residents of County Y may be more law-abiding overall.', 'The argument fails to take into account that the speed limit may be different in the two counties.', 'The argument fails to take into account that the number of tickets issued in County X may reflect a lower proportion of drivers overall in that county.', 'The argument fails to take into account that a single driver can receive more than one ticket in a given year.']", "label": 3 }, { "id": "train_2664", "context": "Medical research has shown that chronic fatigue syndrome is definitely not the product of mental or emotional disorders, but is instead completely caused either by immunologic or genetic factors. Yet, other medical research has found that a large percentage of patients treated for chronic fatigue syndrome are also treated for depression and anxiety.", "question": "Which one of the following, if true, helps explain the information presented above?", "answers": "['Patients with chronic medical conditions often subsequently develop depression and anxiety.', 'People with depression are often unaware that they have other medical problems that require professional treatment.', 'Most people with chronic fatigue syndrome believe that their condition is caused by immunologic rather than genetic factors.', 'Patients are often unaware that they suffer from depression and anxiety.']", "label": 0 }, { "id": "train_2665", "context": "When Cortez arrived in Mexico in A. D. 1519, he observed the inhabitants playing a ceremonial game with a rubber ball. The pre-Columbian inhabitants of Mexico began to use rubber around A. D. 1000. Thus, we can be sure that the game must have originated sometime between approximately A. D. 1000 and Cortez' arrival.", "question": "The conclusion reached above depends on which one of the following assumptions?", "answers": "['The game had been played since its inception with a rubber ball.', 'The making of rubber balls was one of the earliest uses of rubber by the inhabitants of Mexico.', 'The pre-Columbian inhabitants of Mexico played games on all ceremonial occasions.', 'The ceremonial game referred to was popular throughout Mexico.']", "label": 0 }, { "id": "train_2666", "context": "The city' s center for disease control reports that the rabies epidemic is more serious now than it was two years ago: 2 years ago less than 25 percent of the local raccoon population was infected, whereas today the infection has spread to more than 50 percent of the raccoon population. However, the newspaper reports that whereas two years ago 32 cases of rabid raccoons were confirmed during a 12-month period, in the past 12 months only 18 cases of rabid raccoons were confirmed.", "question": "Which one of the following, if true, most helps to resolve the apparent discrepancy between the two reports?", "answers": "['Since the outbreak of the epidemic, raccoons, which are normally nocturnal, have increasingly been seen during daylight hours.', 'A significant proportion of the raccoon population succumbed to rabies in the year before last.', 'The number of cases of rabies in wild animals other than raccoons has increased in the past 12 months.', 'The symptoms of distemper, another disease to which raccoons are susceptible, are virtually identical to those of rabies.']", "label": 1 }, { "id": "train_2667", "context": "Sometimes when their trainer gives the hand signal for \"Do something creative together, \" two dolphins circle a pool in tandem and then leap through the air simultaneously. On other occasions the same signal elicits synchronized backward swims or tail-waving. These behaviors are not simply learned responses to a given stimulus. Rather, dolphins are capable of higher cognitive functions that may include the use of language and forethought.", "question": "Which one of the following, if true, most strengthens the argument?", "answers": "['The dolphins often exhibit complex new responses to the hand signal.', 'Some of the behaviors mentioned are exhibited by dolphins in their natural habitat.', 'Dolphins do not interact with humans the way they interact with one another.', 'The dolphins are given food incentives as part of their training.']", "label": 0 }, { "id": "train_2668", "context": "Researcher: People with certain personality disorders have more theta brain waves than those without such disorders. But my data show that the amount of one' s theta brain waves increases while watching TV. So watching too much TV increases one' s risk of developing personality disorders.", "question": "A questionable aspect of the reasoning above is that it", "answers": "['draws a conclusion from an unrepresentative sample of data', 'infers that watching TV is a consequence of a personality disorder', 'takes correlation to imply a causal connection', 'fails to define the phrase \"theta brain waves\"']", "label": 2 }, { "id": "train_2669", "context": "Ecologists predict that the incidence of malaria will increase if global warming continues or if the use of pesticides is not expanded. But the use of pesticides is known to contribute to global warming, so it is inevitable that we will see an increase in malaria in the years to come.", "question": "The pattern of reasoning in which one of the following is most similar to that in the argument above?", "answers": "['The crime rate will increase if the economy does not improve or if we do not increase the number of police officers. But we will be able to hire more police officers if the economy does improve. Ttherefore, the crime rate will not increase.', 'If global warming continues or if the rate of ozone depletion is not reduced, there will be an increase in the incidence of skin cancer. But reduced use of aerosols ensures both that global warming will not continue and that ozone depletion will be reduced. Thus, the incidence of skin cancer will not increase.', 'If deforestation continues at the current rate and the use of chemicals is not curtailed, wildlife species will continue to become extinct. But because of increasing population worldwide, it is inevitable that the current rate of deforestation will continue and that the use of chemicals will not be curtailed. Thus, wildlife species will continue to become extinct.', 'If interest rates increase or demand for consumer goods does not decline, inflation will rise. But if there is a decline in the demand for consumer goods, that will lead to higher interest rates. Ttherefore, inflation will rise.']", "label": 3 }, { "id": "train_2670", "context": "Aroca County' s public schools are supported primarily by taxes on property. The county plans to eliminate the property tax and support schools with a new three percent sales tax on all retail items sold in the county. Three percent of current retail sales is less than the amount collected through property taxes, but implementation of the plan would not necessarily reduce the amount of money going to Aroca County public schools, because __.", "question": "Which of the following, if true, most logically completes the argument?", "answers": "['many Aroca County residents have already left the county because of its high property taxes', 'retailers in Aroca County are not likely to absorb the sales tax by reducing the pretax price of their goods', 'a shopping mall likely to draw shoppers from neighboring counties , which have much higher sales-tax rates, is about to open in Aroca county', 'at least some Aroca County parents are likely to use the money they will save on property taxes to send their children to private schools not funded by the county']", "label": 2 }, { "id": "train_2671", "context": "Brooks: I' m unhappy in my job, but I don' t know whether I can accept the risks involved in quitting my job. Morgenstern: The only risk in quitting is that of not finding another job. If you don' t find one, you' re going to be pretty unhappy. But you' re already unhappy, so you might as well just quit.", "question": "Morgenstern's argument is flawed in that it", "answers": "['reaches a generalization on the basis of a single case', 'fails to take into account that unhappiness can vary in intensity or significance', 'conflates two different types of risk', 'relies on an assumption that is tantamount to assuming that the conclusion is true']", "label": 1 }, { "id": "train_2672", "context": "Critics of certain pollution-control regulations have claimed that the money spent over the last decade in order to reduce emissions of carbon monoxide and of volatile organic compounds has been wasted . The evidence they offer in support of this claim might appear compelling: despite the money spent, annual emissions of these pollutants have been increasing steadily. This evidence is far from adequate, however, since over the last decade a substantial number of new industrial facilities that emit these pollutants have been built .", "question": "In the reasoning given, the two portions in boldface play which of the following roles?", "answers": "[\"The first is a position that the reasoning contends is inadequately supported by the evidence; the second is evidence used to support the reasoning's contention.\", 'The first identifies a claim that the reasoning seeks to show is false; the second is evidence that has been cited by others in support of that claim.', 'The first is a position that the reasoning contends is inadequately supported by the evidence; the second is a position for which the reasoning seeks to provide support.', 'The first is a position that the reasoning contends is inadequately supported by the evidence; the second is evidence that has been used to support that position.']", "label": 0 }, { "id": "train_2673", "context": "The dramatic rise in the number of homeless people in the 1980' s cannot be attributed to the discharge of mentally ill persons from hospitals into \"community care, \" even though it is true that provisions for such community care are for the most part nonexistent.", "question": "Which of the following, if true, best supports the claim above?", "answers": "['The policy of discharging mentally ill persons from hospitals was based on the supposition that their illnesses could be controlled by newly developed drugs.', \"The policy of discharging significant numbers of mentally ill persons from hospitals took effect in the 1960's.\", \"Some community-care programs for mentally ill persons did exist in the 1980's, but those programs consisted primarily of outpatient clinics.\", 'Without supervision in a hospital setting those people whose mental illness can be controlled by medication might not actually take prescribed medication.']", "label": 1 }, { "id": "train_2674", "context": "It is a mistake to think, as ecologists once did, that natural selection will eventually result in organisms that will be perfectly adapted to their environments. After all, perfect adaptation of an individual to its environment is impossible, for an individual' s environment can vary tremendously; no single set of attributes could possibly prepare an organism to cope with all the conditions that it could face.", "question": "Which one of the following most accurately expresses the main conclusion of the argument?", "answers": "[\"Because an individual's environment can vary tremendously, no individual can be perfectly adapted to its environment.\", 'Natural selection will never result in individuals that will be perfectly adapted to their environments.', 'It is not possible for an individual to be perfectly adapted to its environment.', 'Ecologists once believed that natural selection would eventually result in individuals that will be perfectly adapted to their environments.']", "label": 1 }, { "id": "train_2675", "context": "Satellite navigation systems (satnavs) for cars, in which computer voices announce directions as you drive, save fuel and promote safety. Studies show that, when assigned to novel destinations, drivers using satnavs took, on average, 7 percent fewer miles per journey than drivers using paper maps. Fewer miles driven means, on average, less fuel consumed. Also, the drivers who used satnavs drove more carefully in that they were not taking their eyes off the road to check paper maps.", "question": "Which one of the following, if true, most strengthens the argument?", "answers": "['Drivers who are given directions as needed are less likely to change course suddenly or make other risky maneuvers.', 'Drivers who do not routinely need to drive to an unfamiliar location are more likely to plan out their route carefully prior to departure.', 'The more fuel a vehicle consumes, the more motivation a driver has to find the shortest route to his or her destination.', 'Drivers who own satnavs usually prefer to drive to their accustomed destinations by using their customary routes rather than by following the directions given by the satnavs.']", "label": 0 }, { "id": "train_2676", "context": "People who have habitually slept less than six hours a night and then begin sleeping eight or more hours a night typically begin to feel much less anxious. Ttherefore, most people who sleep less than six hours a night can probably cause their anxiety levels to fall by beginning to sleep at least eight hours a night.", "question": "The reasoning in which one of the following arguments is most similar to that in the argument above?", "answers": "['Usually, the financial situation of a small company that has never advertised on the Internet will improve only if that company starts to advertise on the Internet. Ttherefore, a typical small company that has never advertised on the Internet can probably improve its financial situation by doing so.', 'When a small company first begins to advertise on the Internet, its financial situation generally improves. This shows that most small companies that have never advertised on the Internet can probably improve their financial situation by doing so.', 'It must be true that any small company that increases its Internet advertising will improve its financial situation, since most small companies that advertise on the Internet improved their financial situations soon after they first began to do so.', \"A small company's financial situation usually improves soon after that company first begins to advertise on the Internet. Thus, most small companies that have never advertised on the Internet could probably become financially strong.\"]", "label": 1 }, { "id": "train_2677", "context": "An eyeglass manufacturer tried to boost sales for the summer quarter by offering its distributors a special discount if their orders for that quarter exceeded those for last year' s summer quarter by at least 20 percent. Many distributors qualified for this discount. Even with much merchandise discounted, sales increased enough to produce a healthy gain in net profits. The manufacturer plans to repeat this success by offering the same sort of discount for the fall quarter.", "question": "Which of the following, if true, most clearly points to a flaw in the manufacturer's plan to repeat the successful performance of the summer quarter?", "answers": [ "The distributors' ordering more goods in the summer quarter left them overstocked for the fall quarter.", "The distributors who qualified for the manufacturer's special discount were free to decide how much of that discount to pass on to their own customers.", "The distributors most likely to qualify for the manufacturer's special discount are those whose orders were unusually low a year earlier.", "Along with offering special discounts to qualifying distributors, the manufacturer increased newspaper and radio advertising in those distributors' sales areas." ], "label": 0 }, { "id": "train_2678", "context": "Beverage company representative: The plastic rings that hold six-packs of beverage cans together pose a threat to wild animals, which often become entangled in the discarded rings and suffocate as a result. Following our lead, all beverage companies will soon use only those rings consisting of a new plastic that disintegrates after only three days' exposure to sunlight. Once we all complete the switchover from the old to the new plastic rings, ttherefore, the threat of suffocation that plastic rings pose to wild animals will be eliminated.", "question": "The argument depends on which one of the following assumptions?", "answers": "['The switchover to the new plastic rings can be completed without causing significant financial hardship to the beverage companies.', 'Wild animals will not become entangled in the new plastic rings before the rings have had sufficient exposure to sunlight to disintegrate.', 'Use of the old plastic rings poses no substantial threat to wild animals other than that of suffocation.', 'Any wild animal that becomes entangled in the old plastic rings will suffocate as a result.']", "label": 1 }, { "id": "train_2679", "context": "Essayist: Lessing contended that an art form' s medium dictates the kind of representation the art form must employ in order to be legitimate; painting, for example, must represent simultaneous arrays of colored shapes, while literature, consisting of words read in succession, must represent events or actions occurring in sequence. The claim about literature must be rejected, however, if one regards as legitimate the imagists' poems, which consist solely of amalgams of disparate images.", "question": "Which one of the following, if assumed, enables the essayist's conclusion to be properly drawn?", "answers": "['All art represents something either as simultaneous or as successive.', 'Poems whose subject matter is not appropriate to their medium are illegitimate.', 'An amalgam of disparate images cannot represent a sequence of events or actions.', \"All art, even the imagists' poetry, depicts or represents some subject matter.\"]", "label": 2 }, { "id": "train_2680", "context": "In Metroville, the KP subway line follows the riverfront from downtown to the neighborhoods surrounding the university. No matter how many trains the subway runs on the KP line, there are always congestion and delays on the KP line. The subway submitted a proposal for an alternate subway line that would travel from downtown to the neighborhoods surrounding the university via the inland neighborhoods, hoping to draw some of the congestion from the KP line. The city hired urban planning consultants who concluded the subway' s proposal of a new line would not reduce commuter congestion on the KP line.", "question": "Which of the following, if true, most helps to explain the urban planning consultants' position?", "answers": "['The most popular after-work destinations, such as riverfront restaurants and bars as well as the ballpark, would only be accessible by the current KP line.', 'Construction of a new subway line would inevitably stimulate the growth of new business and restaurants along that line, especially in the vicinity of the stations of that new line.', 'The city buses that run along the river between downtown and the university would be more attractive to commuters than an inland subway line.', 'Construction of a new subway line would involve constructing a number of new stations throughout the city.']", "label": 0 }, { "id": "train_2681", "context": "Those influenced by modern Western science take it for granted that a genuine belief in astrology is proof of a credulous and unscientific mind. Yet, in the past, people of indisputable intellectual and scientific brilliance accepted astrology as a fact. Ttherefore, there is no scientific basis for rejecting astrology.", "question": "The argument is most vulnerable to criticism on which one of the following grounds?", "answers": "['Although the conclusion is intended to hold in all cultures, the evidence advanced in its support is drawn only from those cultures strongly influenced by modern Western science.', 'The implicit assumption that all practitioners of Western science believe in astrology is false.', 'A belief can be consistent with the available evidence and accepted scientific theories at one time but not with the accepted evidence and theories of a later time.', 'Since it is controversial whether astrology has a scientific basis, any argument that attempts to prove that it has will be specious.']", "label": 2 }, { "id": "train_2682", "context": "Public reports by national commissions, governors' conferences, and leadership groups have stressed the great need for better understanding of international affairs by the citizenry. If the country is to remain a leading nation in an era of international competitiveness, the need is undeniable. If there is such a need for the citizenry to have a better understanding of international affairs, then all of our new teachers must be prepared to teach their subject matter with an international orientation.", "question": "If all of the statements in the passage are true, which one of the following must also be true?", "answers": "['If the country is to remain a leading nation in an era of international competitiveness, then there is no need for the citizenry to have a better understanding of international affairs.', 'Public reports from various groups and commissions have stressed the need for a more international orientation in the education of teachers.', 'If the country is to remain a leading nation in an era of international competitiveness, then new teachers must be prepared to teach their subject matter with an international orientation.', 'If there is better understanding of international affairs by the citizenry, then the country will remain a leading nation in an era of international competitiveness.']", "label": 2 }, { "id": "train_2683", "context": "Many people suffer an allergic reaction to sulfites, including those that are commonly added to wine as preservatives. However, since there are several winemakers producing wine to which no sulfites are added, those who would like to drink wine but are allergic to sulfites can drink these wines without risking an allergic reaction to sulfites.", "question": "Which of the following, if true, most seriously weakens the argument?", "answers": "['Sulfites occur naturally in most wine.', 'The sulfites that can produce an allergic reaction are also commonly found in beverages other than wine.', 'Wine without added sulfites sometimes becomes undrinkable even before the wine is sold to consumers.', 'Apart from sulfites, there are other substances commonly present in wine that can trigger allergic reactions.']", "label": 0 }, { "id": "train_2684", "context": "Scientist: It seems likely that the earliest dinosaurs to fly did so by gliding out of trees rather than, as some scientists think, by lifting off the ground from a running start. Animals gliding from trees are able to fly with very simple wings. Such wings represent evolutionary middle stages toward developing the large wings that we associate with flying dinosaurs.", "question": "Each of the following, if true, strengthens the scientist's argument EXCEPT:", "answers": "['Early flying dinosaurs had sharp claws and long toes suitable for climbing.', 'Early flying dinosaurs built their nests at the base of trees.', 'Early flying dinosaurs had unusual feathers that provided lift while gliding, but little control when taking flight.', 'Early flying dinosaurs lived at a time when their most dangerous predators could not climb trees.']", "label": 1 }, { "id": "train_2685", "context": "In a democratic society, when a political interest group exceeds a certain size, the diverse and sometimes conflicting economic interests that can be found in almost any large group of people tend to surface. Once these conflicting interests have surfaced, they can make it impossible for the political interest group to unite behind a common program. Yet to have the political impact necessary to influence legislation, a group must be united.", "question": "The statements above, if true, most strongly support which one of the following views?", "answers": "['A political interest group can become ineffective by expanding to include as wide a membership as possible.', 'Political interest groups generally start out effectively but lose their effectiveness over time.', 'For a democratic society to function effectively, it is necessary for political groups within that society to make compromises with each other.', 'Political interest groups are generally less influential when their membership is expanding than when it is numerically stable.']", "label": 0 }, { "id": "train_2686", "context": "City councilperson: Many city residents oppose the city art commission' s proposed purchase of an unusual stone edifice, on the grounds that art critics are divided over whether the edifice really qualifies as art. But I argue that the purpose of art is to cause experts to debate ideas, including ideas about what constitutes art itself. Since the edifice has caused experts to debate what constitutes art itself, it does qualify as art.", "question": "Which one of the following, if assumed, enables the conclusion of the city councilperson's argument to be properly inferred?", "answers": "['If an object causes debate among experts, no expert can be certain whether that object qualifies as art.', 'Any object that fulfills the purpose of art qualifies as art.', 'The purchase of an object that fulfills the purpose of art should not be opposed.', 'Nothing qualifies as art unless it causes debate among experts.']", "label": 1 }, { "id": "train_2687", "context": "With the passage of the new tax reform laws, the annual tax burden on low-income taxpayers will be reduced, on average, by anywhere from $100 to $300. Clearly, tax reform is in the interest of low-income taxpayers.", "question": "Which one of the following, if true, most undermines the conclusion above?", "answers": "['Low-income taxpayers have consistently voted for those political candidates who are strong advocates of tax reform.', 'Under the new tax reform laws, many low-income taxpayers who now pay taxes will no longer be required to do so.', 'Tax reform, by eliminating tax incentives to build rental housing, will push up rents an average of about $40 per month for low-income taxpayers.', 'The new tax reform laws will permit low-and middle-income taxpayers to deduct child-care expenses from their taxes.']", "label": 2 }, { "id": "train_2688", "context": "Sometimes it is advisable for a medical patient to seek a second opinion. But this process can be awkward for both the patient and the physicians, since the patient often worries that the first physician will be alienated. In addition, for the first physician there is the issue of pride: a second opinion tacitly highlights a physician' s fallibility. And the second physician is in the position of evaluating not only a patient' s health, but also, inevitably and uncomfortably, a colleague' s work.", "question": "Which one of the following most accurately states the conclusion of the argument as a whole?", "answers": "['Physicians who are called upon to offer second opinions are always uncomfortable about evaluating the work of colleagues.', 'The process of obtaining a second medical opinion can be awkward for those involved.', 'In cases in which second opinions are necessary, the first physician often feels that his or her professional judgment is called into question.', 'Because of the awkwardness involved, it is best for patients not to seek second opinions unless it is absolutely necessary.']", "label": 1 }, { "id": "train_2689", "context": "New technologies that promise to extend life and decrease pain involve innovations that require extensive scientific research. Ttherefore, investment in such technologies is very risky, because innovations requiring extensive scientific research also require large amounts of capital but are unlikely to provide any financial return. Nonetheless, some people are willing to invest in these new technologies.", "question": "Which one of the following, if true, most helps to explain why some people are willing to invest in new technologies that promise to extend life and decrease pain?", "answers": "['When investments in new technologies that promise to extend life and decrease pain do provide financial return, they generally return many times the original investment, which is much more than the return on safer investments.', 'A large variety of new technologies that promise to extend life and decrease pain have been developed in the last decade.', 'The scientific research necessary to develop new technologies that promise to extend life and decrease pain sometimes leads to no greater understanding of the natural world.', 'The development of certain new technologies other than those that promise to extend life and decrease pain is also very risky, because these technologies require large amounts of capital but are unlikely to provide any financial return.']", "label": 0 }, { "id": "train_2690", "context": "During the earliest period of industrialization in Britain, steam engines were more expensive to build and operate than either windmills or water mills, the other practicable sources of power for factories. Yet despite their significant cost disadvantages, steam-powered factories were built in large numbers well before technical improvements brought their cost down. Furthermore, they were built even in regions where geographical conditions permitted the construction of wind-and water-powered factories close to major markets.", "question": "Which of the following, if true, most helps to explain the proliferation of steam-powered factories during the earliest period of industrialization in Britain?", "answers": "['In Britain, the number of sites where a wind-or water-powered factory could be built was insufficient to provide for all of the demand for factory-produced goods at the time.', 'It was both difficult and expensive to convert a factory from wind power or water power to steam power.', 'Unlike wind-or water-powered factories, steam-powered factories were fueled with coal, which sometimes had to be transported significant distances from the mine to the site of the factory.', 'In many areas of Britain, there were fewer steam-powered factories than wind-or water-powered factories in the earliest period of industrialization.']", "label": 0 }, { "id": "train_2691", "context": "When complaints against staff are brought to Human Resources at Sarpedon Inc. , the Human Resources department cannot always determine whether there was just cause for the complaint. This is usually due to insufficient evidence provided by the claimant. Nevertheless, Sarpedon will sometimes dismiss an employee because of such a complaint, even if unsubstantiated.", "question": "Which of the following conclusions can most properly be drawn from the information above?", "answers": "['It is possible that an unsubstantiated complaint could be unfairly held against the employee that it implicates.', \"In a 1-on-1 conflict in which the only two employees involved give conflicting view of each other's words and actions, managers have to make a judgment at their own discretion.\", 'In certain kinds of harassment, victims are reluctant to press charges, for fear of reprisals or unfavorable judgments from other colleagues.', 'Many of the employees dismissed from Sarpedon would have substantial grounds for a lawsuit concerning their dismissal because of this Human Resources policy.']", "label": 0 }, { "id": "train_2692", "context": "Because learned patterns of behavior, such as the association of a green light with \"go\" or the expectation that switches will flip up for \"on, \" become deeply ingrained, designers should make allowances for that fact, in order not to produce machines that are inefficient or dangerous.", "question": "In which one of the following situations is the principle expressed most clearly violated?", "answers": "['Manufacturers have refused to change the standard order of letters on the typewriter keyboard even though some people who have never learned to type find this arrangement of letters bewildering.', 'On some tape recorders the \"start\" button is red and the \"stop\" button is yellow.', 'Automobile manufacturers generally design for all of their automobiles a square or oblong ignition key and a round or oval luggage compartment key.', 'Government regulations require that crucial instruments in airplane cockpits be placed in exactly the same array in all commercial aircraft.']", "label": 1 }, { "id": "train_2693", "context": "In household electric ovens, the heating element has only two settings -- on and off. A thermostat connected to the oven' s temperature knob is the only control on temperature, automatically switching the element off when the indicated temperature is reached, which happens quickly in modern ovens, and subsequently on or off as needed to maintain temperature.", "question": "Which of the following statements is most strongly supported by the information above?", "answers": "[\"If the thermostat of a modern household electric oven is disabled so that the heating element remains switched on, the oven's temperature can eventually become higher than the maximum temperature setting on its temperature knob.\", 'Once the indicated temperature has been reached and is being maintained by the thermostat, the heating element will be switched off for a greater proportion of the time than it is switched on.', 'Because in each case the heating element alternates between on and off, a modern household electric oven uses little more power at its maximum temperature setting than it does at its lowest temperature setting.', 'The accuracy with which household electric ovens maintain their temperature could not be improved on by ovens whose heating elements have more than two settings.']", "label": 0 }, { "id": "train_2694", "context": "Investigators of the fire at the Shelburne factory have determined that the traces of gasoline on the property resulted from a leaking truck and were not, as they previously thought, the work of an arsonist. The investigators had originally maintained that the fire could have resulted from arson, faulty wiring, or an explosion of factory equipment. However, in light of all the evidence collected so far, they are currently exploring only the faulty-wiring theory.", "question": "The statements above provide the most support for which one of the following inferences?", "answers": "['The gasoline leaking from the truck did not help to cause the truck to explode.', 'The investigators believe that the fire was not caused by an explosion of factory equipment.', 'The investigators have until now seriously explored only the possibility that the fire resulted from arson.', 'The investigators have physical evidence that the wiring in the Shelburne factory was faulty.']", "label": 1 }, { "id": "train_2695", "context": "Most of the world' s supply of uranium currently comes from mines. It is possible to extract uranium from seawater, but the cost of doing so is greater than the price that uranium fetches on the world market. Ttherefore, until the cost of extracting uranium from seawater can somehow be reduced, this method of obtaining uranium is unlikely to be commercially viable.", "question": "Which of the following would it be most useful to determine in evaluating the argument?", "answers": "['Whether the total amount of uranium in seawater is significantly greater than the total amount of uranium on land', 'Whether uranium can be extracted from freshwater at a cost similar to the cost of extracting it from seawater', 'Whether the uranium in deposits on land is rapidly being depleted', 'Whether there are any technological advances that show promise of reducing the cost of extracting uranium from seawater']", "label": 2 }, { "id": "train_2696", "context": "Skeletal remains of early humans indicate clearly that our ancestors had fewer dental problems than we have. So, most likely, the diet of early humans was very different from ours.", "question": "Which one of the following, if true, most strengthens the argument?", "answers": "['Skeletal remains indicate that some early humans had a significant number of cavities.', 'A healthy diet leads to healthy teeth.', 'Diet is by far the most significant factor contributing to dental health.', 'Early humans had a shorter average life span than we do, and the most serious dental problems now tend to develop late in life.']", "label": 2 }, { "id": "train_2697", "context": "Gotera: Infants lack the motor ability required to voluntarily produce particular sounds, but produce various babbling sounds randomly. Most children are several years old before they can voluntarily produce most of the vowel and consonant sounds of their language. We can conclude that speech acquisition is entirely a motor control process rather than a process that is abstract or mental.", "question": "Which one of the following is an assumption required by Gotera's argument?", "answers": "['Control of tongue and mouth movements requires a sophisticated level of mental development.', \"Speech acquisition is a function only of one's ability to produce the sounds of spoken language.\", 'The initial babbling stage is completed during infancy.', 'The initial babbling stage is the first stage of the speech acquisition process.']", "label": 1 }, { "id": "train_2698", "context": "Over the last five years, demand for hotel rooms in Cenopolis has increased significantly, as has the average price Cenopolis hotels charge for rooms. These trends are projected to continue for the next several years. In response to this economic forecast, Centennial Commercial, a real estate developer, is considering a plan to convert several unoccupied office buildings it owns in Cenopolis into hotels in order to maximize its revenues from these properties.", "question": "Which of the following would it be most useful for Centennial Commercial to know in evaluating the plan it is considering?", "answers": "['Whether the population of Cenopolis is expected to grow in the next several years', 'Whether demand for office space in Cenopolis is projected to increase in the near future', 'Whether, on average, hotels that have been created by converting office buildings have fewer guest rooms than do hotels that were built as hotels', 'Whether demand for hotel rooms has also increased in other cities where Centennial owns office buildings']", "label": 1 }, { "id": "train_2699", "context": "For a ten-month period, the total monthly sales of new cars within the country of Calistan remained constant. During this period the monthly sales of new cars manufactured by Marvel Automobile Company doubled, and its share of the new car market within Calistan increased correspondingly. At the end of this period, emission standards were imposed on new cars sold within Calistan. During the three months following this imposition, Marvel Automobile Company' s share of the Calistan market declined substantially even though its monthly sales within Calistan remained constant at the level reached in the last month of the ten-month period.", "question": "If the statements above are true, which one of the following CANNOT be true?", "answers": "[\"Since the imposition of the emission standards, Marvel Automobile Company's average profit on each new car sold within Calistan has increased.\", 'The total monthly sales within Calistan of new cars by companies other than Marvel Automobile Company decreased over the three months following the imposition of the emission standards.', 'Over the three months before the imposition of the emission standards, the combined market share of companies other than Marvel Automobile Company selling new cars in Calistan decreased.', 'If the emission standards had not been imposed, Marvel Automobile Company would have lost an even larger share of the number of new cars sold in Calistan than, in fact, it did.']", "label": 1 }, { "id": "train_2700", "context": "Michele: In my professional experience, it' s usually not a good idea for a company to overhaul its databases. The rewards rarely exceed the problems experienced along the way, and I' d suggest that anyone considering a database overhaul think twice before proceeding. Alvaro: But the problems are always caused by a failure to recode the database properly. The best advice for a company considering a database overhaul is to do the job right.", "question": "Michele and Alvaro disagree with each other about which one of the following?", "answers": "['why companies should consider overhauling their databases', 'whether the problems experienced during a database overhaul ever outweigh the. rewards', 'which kinds of database overhauls have more problems than are justified by the rewards', 'what a company should do when considering a database overhaul']", "label": 3 }, { "id": "train_2701", "context": "Inspector: The only fingerprints on the premises are those of the owner, Mr. Tannisch. Ttherefore, whoever now has his guest' s missing diamonds must have worn gloves.", "question": "Which one of the following exhibits a flaw in its reasoning most similar to that in the inspector's reasoning?", "answers": "[\"All of Marjorie's cavities are on the left side of her mouth. Hence, she must chew more on the left side than on the right.\", 'The campers at Big Lake Camp, all of whom became ill this afternoon, have eaten food only from the camp cafeteria. Ttherefore, the cause of the illness must not have been something they ate.', \"All of these tomato plants are twice as big as they were last year. So if we grow peas, they will probably be twice as big as last year's peas.\", 'The second prototype did not perform as well in inclement weather as did the first prototype. Hence, the production of the second prototype might have deviated from the design followed for the first.']", "label": 1 }, { "id": "train_2702", "context": "At present the Hollywood Restaurant has only standard-height tables. However, many customers come to watch the celebrities who frequent the Hollywood, and they would prefer tall tables with stools because such seating would afford a better view of the celebrities. Moreover, diners seated on stools typically do not stay as long as diners seated at standard-height tables. Ttherefore, if the Hollywood replaced some of its seating with high tables and stools, its profits would increase.", "question": "The argument is vulnerable to criticism on the grounds that it gives reason to believe that it is likely that", "answers": "['some celebrities come to the Hollywood to be seen, and so might choose to sit at the tall tables if they were available', \"with enough tall tables to accommodate all the Hollywood's customers interested in such seating, there would be no view except of other tall tables\", 'a customer of the Hollywood who would choose to sit at a tall table would be an exception to the generalization about lingering', \"a restaurant's customers who spend less time at their meals typically order less expensive meals than those who remain at their meals longer\"]", "label": 2 }, { "id": "train_2703", "context": "The distance that animals travel each day and the size of the groups in which they live are highly correlated with their diets. And diet itself depends in large part on the sizes and shapes of animals' teeth and faces.", "question": "The statements above provide the most support for which one of the following?", "answers": "[\"Information about the size and shape of an animal's face is all that is needed to identify the species to which that animal belongs.\", 'When individual herd animals lose their teeth through age or injury, those animals are likely to travel at the rear of their herd.', 'Animals that have varied diets can be expected to be larger and more robust than animals that eat only one or two kinds of food.', \"Information about the size and shape of an extinct animal's teeth and face can establish whether that animal is likely to have been a herd animal.\"]", "label": 3 }, { "id": "train_2704", "context": "Scientist: Some critics of public funding for this research project have maintained that only if it can be indicated how the public will benefit from the project is continued public funding for it justified. If the critics were right about this, then there would not be the tremendous public support for the project that even its critics acknowledge.", "question": "If the scientist's claims are true, which one of the following must also be true?", "answers": "['There is tremendous public support for the research project because it can be indicated how the public will benefit from the project.', 'Continued public funding for the research project is justified.', 'Public support for the research project is the surest indication of whether or not it is justified.', \"That a public benefit can be indicated is not a requirement for the justification of the research project's continued public funding.\"]", "label": 3 }, { "id": "train_2705", "context": "Wood smoke contains dangerous toxins that cause changes in human cells. Because wood smoke presents such a high health risk, legislation is needed to regulate the use of open-air fires and wood-burning stoves.", "question": "Which of the following, if true, provides the most support for the argument above?", "answers": "['The amount of dangerous toxins contained in wood smoke is much less than the amount contained in an equal volume of automobile exhaust.', 'No significant beneficial effect on air quality would result if open-air fires were banned within the jurisdiction covered by the proposed legislation.', 'In valleys where wood is used as the primary heating fuel, the concentration of smoke results in poor air quality.', 'Smoke produced by coal-burning stoves is significantly more toxic than smoke from woodburning stoves.']", "label": 2 }, { "id": "train_2706", "context": "Critic: A novel cannot be of the highest quality unless most readers become emotionally engaged with the imaginary world it describes. Thus shifts of narrative point of view within a novel, either between first and third person or of some other sort, detract from the merit of the work, since such shifts tend to make most readers focus on the author.", "question": "Which one of the following is an assumption necessary for the critic's conclusion to be properly drawn?", "answers": "[\"Most readers cannot become emotionally involved with a novel's imaginary world if they focus on the author.\", \"Most readers regard a novel's narrative point of view as representing the perspective of the novel's author.\", 'Most readers become emotionally engaged with the imaginary world described by a novel only if the novel is of the highest quality.', 'Shifts in narrative point of view serve no literary purpose.']", "label": 0 }, { "id": "train_2707", "context": "Head injury is the most serious type of injury sustained in motorcycle accidents. The average cost to taxpayers for medical care for nonhelmeted motorcycle-accident victims is twice that for their helmeted counterparts. Jurisdictions that have enacted motorcycle-helmet laws have reduced the incidence and severity of accident-related head injuries, thereby reducing the cost to taxpayers. Ttherefore, to achieve similar cost reductions, other jurisdictions should enact motorcycle-helmet laws. For the same reason jurisdictions should also require helmets for horseback riders, since horseback-riding accidents are even more likely to cause serious head injury than motorcycle accidents are.", "question": "Which one of the following is an assumption upon which the author's conclusion concerning helmets for horseback riders depends?", "answers": "[\"When deciding whether to enact helmet laws for motorcyclists and horseback riders, the jurisdiction's primary concern is the safety of its citizens.\", 'The medical costs associated with treating head injuries are higher than those for other types of injury.', 'The higher rate of serious head injury suffered by victims of horseback-riding accidents is due to the difference in size between horses and motorcycles.', 'Medical care for victims of horseback-riding accidents is a financial drain on tax funds.']", "label": 3 }, { "id": "train_2708", "context": "Platinum is a relatively rare metal vital to a wide variety of industries. Xagor Corporation, a major producer of platinum, has its production plant in a country that will soon begin imposing an export tax on platinum sold and shipped to customers abroad. As a consequence, the price of platinum on the world market is bound to rise.", "question": "Which of the following, if true, tends to confirm the conclusion above?", "answers": "['New deposits of platinum have been found in the country in which Xagor has its production plant.', 'The largest of the industries that depend on platinum reported a drop in sales last month.', 'An inexpensive substitute for platinum has been developed and will be available to industry for the first time this month.', 'The producers of platinum in other countries taken together cannot supply enough platinum to meet worldwide demand.']", "label": 3 }, { "id": "train_2709", "context": "Last year, Mayor Stephens established a special law-enforcement task force with the avowed mission of eradicating corruption in city government. The mayor' s handpicked task force has now begun prosecuting a dozen city officials. Since all of these officials were appointed by Mayor Bixby, Mayor Stephens' predecessor and longtime political foe, it is clear that those being prosecuted have been targeted because of their political affiliations.", "question": "Which of the following, if true, most weakens the editorial's argument?", "answers": "['Complaints of official corruption in city government have decreased since the anticorruption task force began operating.', 'Almost all of the officials who have served in city government for any length of time are appointees of Mayor Bixby.', \"Former mayor Bixby did not publicly oppose Mayor Stephens' establishment of the anticorruption task force.\", 'All of the members of the anticorruption task force had other jobs in city government before the task force was formed.']", "label": 1 }, { "id": "train_2710", "context": "Virginia and her brother William disagree over when their father was born: Virginia claims it was in 1935 and William claims it was in 1933. The hospital where their father was born has no records for 1933 but has complete records for 1935 -- records that do not include a birth record for their father. Ttherefore, he must have been born in 1933.", "question": "The argument depends on which of the following assumptions?", "answers": "['The records of the hospital where their father was born date back to 1933.', \"Virginia and William know the day and the month of their father's birth.\", \"There was urgent practical reasons why Virginia and William must know the date of their father's birth.\", \"Either Virginia's claim or William's claim is correct.\"]", "label": 3 }, { "id": "train_2711", "context": "S. R. Evans: A few critics have dismissed my poems as not being poems and have dismissed me as not being a poet. But one principle of criticism has it that only true poets can recognize poetic creativity or function as critics of poetry -- and that the only true poets are those whose work conveys genuine poetic creativity. But I have read the work of these critics; none of it demonstrated poetic creativity. These critics' judgments should be rejected, since these critics are not true poets.", "question": "The argument above is vulnerable to criticism on the grounds that it", "answers": "[\"presupposes what it sets out to conclude, since the principle requires that only true poets can determine whether the critics' work demonstrates poetic creativity\", 'makes an unjustifiable distinction, since it is possible that some critics are also poets', 'uses the distinction between poets and critics as though everyone fell into one category or the other', 'inevitably leads to the conclusion that poets can never learn to improve their poetry, since no poet is in a position to criticize his or her own work']", "label": 0 }, { "id": "train_2712", "context": "Because metallic mirrors absorb some light, they waste energy. When light strikes a metallic mirror, electrons in the mirror move, using energy and dimming the reflected image. As a result, metallic mirrors cannot be used in applications in which minimizing energy loss is important, such as high-powered lasers.", "question": "Which one of the following most accurately expresses the argument's main conclusion?", "answers": "['Part of the light falling on metallic mirrors tends to be absorbed by them.', 'Metallic mirrors are unsuitable for applications where it is crucial to minimize energy loss.', 'High-powered lasers require mirrors that conserve energy.', 'Metallic mirrors reduce the effectiveness of high-powered lasers.']", "label": 1 }, { "id": "train_2713", "context": "If one has recently been overwhelmed by overstimulation, peaceful rest feels pleasant by contrast. Similarly, recent experience of boredom makes most forms of excitement pleasurable, even dangerous ones. No level of stimulation is intrinsically pleasant or unpleasant.", "question": "The statements above, if true, most strongly support which one of the following?", "answers": "['Boredom can be just as overwhelming as overstimulation.', 'One cannot experience pleasure without first experiencing boredom.', 'How pleasant a person finds a situation can depend on previous levels of stimulation.', 'A high level of stimulation is never pleasant, but it often precedes pleasant relaxation.']", "label": 2 }, { "id": "train_2714", "context": "Principle: If the burden of a proposed policy change would fall disproportionately on people with low incomes, that policy change should not be made. Application: The city of Centerburgh plans to reintroduce rock salt as a road de-icing agent, after having stopped its use several years ago on the grounds that it accelerated the corrosion of automobiles. Although the city claims that cars are now better protected from salt' s corrosive properties than they were even as recently as five years ago, the city' s plan should be halted.", "question": "Which one of the following, if true of Centerburgh, most justifies the above application of the principle?", "answers": "['Road maintenance is primarily funded by local sales taxes, which disproportionately burden people with low incomes.', 'People with low incomes are more likely to purchase older vehicles than are people with higher incomes.', 'Individuals with low incomes are more likely to use public transportation and are less likely to drive cars than are individuals with higher incomes.', 'Cars now cost twice what they did when rock salt was last used as a road de-icing agent.']", "label": 1 }, { "id": "train_2715", "context": "It has been said that authors who write in order to give pleasure cannot impart to their readers the truth of their subject matter. That claim cannot be true. If it were, one could determine the truthfulness of a book simply by looking at its sales figures. If the book were very popular, one could reasonably conclude that it gave people pleasure and ttherefore that at least some of what is written in the book is not true.", "question": "Which one of the following is an assumption required by the argument?", "answers": "['A book will not give its readers pleasure unless it was intended by its author to have that effect.', 'A book can be popular for reasons other than its ability to give readers pleasure.', 'When people choose to read a book, they generally do not already know whether reading it will give them pleasure.', \"In many cases, a book's readers are unconcerned about the truth of the book's contents.\"]", "label": 0 }, { "id": "train_2716", "context": "The last members of a now-extinct species of a European wild deer called the giant deer lived in Ireland about 16, 000 years ago. Prehistoric cave paintings in France depict this animal as having a large hump on its back. Fossils of this animal, however, do not show any hump. Nevertheless, there is no reason to conclude that the cave paintings are ttherefore inaccurate in this regard, since __.", "question": "Which of the following most logically completes the argument?", "answers": "['only one currently existing species of deer has any anatomical feature that even remotely resembles a hump', 'animal humps are composed of fatty tissue, which does not fossilize', 'the cave paintings of the giant deer were painted well before 16, 000 years ago', 'some prehistoric cave paintings in France also depict other animals as having a hump']", "label": 1 }, { "id": "train_2717", "context": "In the past year Harborville has had an unusually high rate of new cases of a certain disease. An investigation revealed that over the past five years Harborville' s public water supply has had a high level of pollutants. Medical researchers have hypothesized that Harborville' s water is responsible for the high rate of new cases of the disease.", "question": "Each of the following, if true, provides additional support for the researchers' hypothesis EXCEPT:", "answers": "['The rate of new cases of the disease among residents of Harborville who get their water from private sources is not unusually high.', \"The same type of pollutants that were found in Harborville's water have independently been linked to occurrences in other towns of high rates of the disease.\", \"Most of the people afflicted with new cases of the disease had regularly drunk larger quantities of Harborville's water than had other Harborville residents.\", \"Some of the people with newly diagnosed cases of the disease began to develop the disease before Harborville's water became polluted.\"]", "label": 3 }, { "id": "train_2718", "context": "Consumer health advocate: Your candy company adds caffeine to your chocolate candy bars so that each one delivers a specified amount of caffeine. Since caffeine is highly addictive, this indicates that you intend to keep your customers addicted. Candy manufacturer: Our manufacturing process results in there being less caffeine in each chocolate candy bar than in the unprocessed cacao beans from which the chocolate is made.", "question": "The candy manufacturer's response is flawed as a refutation of the consumer health advocate's argument because it", "answers": "['assumes without warrant that all unprocessed cacao beans contain a uniform amount of caffeine', \"treats the consumer health advocate's argument as though it were about each candy bar rather than about the manufacturer's candy in general\", \"merely contradicts the consumer health advocate's conclusion without giving any reason to believe that the advocate's reasoning is unsound\", 'fails to address the issue of whether the level of caffeine in the candy bars sold by the manufacture is enough to keep people addicted']", "label": 3 }, { "id": "train_2719", "context": "Annette: To persuade the town council to adopt your development plan, you should take them on a trip to visit other towns that have successfully implemented plans like yours. Sefu: But I have a vested interest in their votes. If council members were to accept a trip from me, it would give the appearance of undue influence.", "question": "The dialogue provides the most support for the claim that Annette and Sefu disagree over whether", "answers": "['Sefu should take the council on a trip to visit other towns', 'other towns have successfully implemented similar development plans', \"Sefu has a vested interest in the council's votes\", 'the appearance of undue influence should be avoided']", "label": 0 }, { "id": "train_2720", "context": "People who accuse the postal service of incompetence and inefficiency while complaining of the proposed five-cent increase in postal rates do not know a bargain when they see one. Few experiences are more enjoyable than reading a personal letter from a friend. Viewed in this way, postal service is so underpriced that a five-cent increase is unworthy of serious debate.", "question": "The reasoning in the argument is flawed because the argument", "answers": "['appeals to an outside authority for support of a premise that should be established by argument', 'confuses the value of the object delivered with the value of delivering that object', 'fails to establish whether or not the critics of the postal service are employees of the postal service', 'claims that the proposed increase is insignificant but does not say at what level the increase would be worthy of serious debate']", "label": 1 }, { "id": "train_2721", "context": "Dr. Kim: Electronic fetal monitors, now routinely used in hospital delivery rooms to check fetal heartbeat, are more intrusive than ordinary stethoscopes and do no more to improve the chances that a healthy baby will be born. Ttherefore, the additional cost of electronic monitoring is unjustified and such monitoring should be discontinued. Dr. Anders: I disagree. Although you and I know that both methods are capable of providing the same information, electronic monitoring has been well worth the cost. Doctors now know the warning signs they need to listen for with stethoscopes, but only because of what was learned from using electronic monitors.", "question": "As a reply to Dr. Kim's argument, Dr. Anders' response is inadequate because it", "answers": "['misses the point at issue', 'overlooks the fact that a procedure can be extensively used without being the best procedure available', 'assumes what it sets out to prove', 'confuses high cost with high quality']", "label": 0 }, { "id": "train_2722", "context": "Studies of enoki mushrooms show that the tall, white specimens grow by cultivation in a rich carbon dioxide environment. The wild mushrooms are a darker brown in color and grow on the sides of trees such as ash, persimmon and the Chinese Hackberry tree. Thus, plants that are white in color must be grown in a carbon dioxide-rich environment.", "question": "The argument is most vulnerable to criticism on the grounds that it", "answers": "['commits the ad hominem fallacy which attacks the arguer instead of the argument.', 'commits the slippery slope fallacy, which states one event must inevitably follow from another event without questioning the inevitability of the event in question.', 'commits the post hoc fallacy, which mistakes correlation for causation.', 'commits the wrong direction fallacy where cause and effect are reversed.']", "label": 2 }, { "id": "train_2723", "context": "The purpose of the physical sciences is to predict the order in which events will succeed one another. Human behavior, also, can sometimes be successfully predicted. However, even successful predictions of human behavior do not provide an understanding of it, for understanding a human action requires knowing its goal, even though such knowledge of goals either cannot or need not be obtained in the case of nonhuman behavior.", "question": "Which one of the following most accurately expresses the argument's conclusion?", "answers": "['The methods used to predict human behavior must involve reference to the psychological states of human agents.', 'Successful predictions of human behavior do not constitute an understanding of that behavior.', 'In some cases, but not in others, understanding an event consists in the ability to predict the occurrence of that event.', 'The goal of the physical sciences is to predict the order in which events will occur.']", "label": 1 }, { "id": "train_2724", "context": "Market research traditionally entails surveying consumers about why they buy and use particular products and brands. Observational research -- actually watching consumers shopping and interacting with products -- is now increasingly used by market researchers to supplement surveys. Market researchers claim that observational research yields information about consumer behavior that surveys alone cannot provide.", "question": "Which one of the following, if true, provides the strongest support for the market researchers' claim?", "answers": "['Consumers are becoming increasingly self-conscience about their buying habits, and some consumers have stopped buying some items that they normally used to buy.', 'Consumers are more likely to respond to oral surveys than they are to respond to written questionnaires.', 'Even consumers who are unable to explain their preference for or rejection of particular brands reveal which brands they are considering by picking up and putting down products while they are shopping.', 'Market researchers find that consumers are almost always willing to participate in observational research for which the consumer is paid by the hour.']", "label": 2 }, { "id": "train_2725", "context": "At one sitting, a typical doughnut eater consumes 4 doughnuts containing a total of 680 calories and 40 grams of fat. The typical bagel eater consumes exactly one bagel, at 500 calories and one or two grams of fat per sitting, though the addition of spreads can raise calorie and fat content to the four-doughnut range. Thus, as far as total calorie content is concerned, there is very little difference between what a typical doughnut eater and a typical bagel eater each consumes at one sitting.", "question": "The argument depends on assuming which one of the following?", "answers": "['Most typical doughnut eaters are not also bagel eaters.', 'Eating bagels instead of eating doughnuts provides no real health benefit.', 'Most bagel eaters are not fully aware of the calorie and fat content of a bagel.', 'The typical doughnut eater does not add to doughnuts any substances that increase the total caloric intake.']", "label": 3 }, { "id": "train_2726", "context": "After the rush-hour speed limit on the British M25 motorway was lowered from 70 miles per hour (115 kilometers per hour) to 50 miles per hour (80 kilometers per hour), rush-hour travel times decreased by approximately 15 percent.", "question": "Which one of the following, if true, most helps to explain the decrease in travel times described above?", "answers": "['Before the rush-hour speed limit was lowered, rush-hour accidents that caused lengthy delays were common, and most of these accidents were caused by high-speed driving.', 'Travel times during periods other than rush hours were essentially unchanged after the rush-hour speed limit was lowered.', 'The number of people who drive on the M25 during rush hours did not increase after the rush-hour speed limit was lowered.', 'After the decrease in the rush-hour speed limit, the average speed on the M25 was significantly lower during rush hours than at other times of the day.']", "label": 0 }, { "id": "train_2727", "context": "The incidence of heart disease in China is much lower than the incidence of heart disease in the United States. The rate of heart disease for Chinese people who immigrate to the United States approximates the higher rate of heart disease prevalent in the United States. However, most Chinese people continue to eat the same type of food that they ate in China after they immigrate to the United States.", "question": "If the statements above are true, they provide the most support for which one of the following?", "answers": "['The relatively low rate of heart disease in China is not due to the different food eaten by the Chinese people.', 'People in China have a lower rate of cancer than those in the United States, even Chinese Americans.', 'The stress of life in China is higher than the stress of life in the United States.', 'Chinese Americans are healthier than other Americans.']", "label": 0 }, { "id": "train_2728", "context": "Advice columnist: Several scientific studies have shown that, when participating in competitive sports, those people who have recently been experiencing major stress in their lives are several times more likely to suffer serious injuries than are other participants in competitive sports. Since risking serious injury is unwise, no sports activity should be used as a method for coping with stress.", "question": "Which one of the following principles, if valid, most helps to justify the reasoning in the advice columnist's argument?", "answers": "['When people have been under considerable stress, they should engage in competitive activities in order to relieve the stress.', 'A method for coping with stress should be used only if it has been subjected to scientific study.', 'People with a history of sports injuries should not engage in sports activities if they have recently been under stress.', 'If people recently under stress should avoid a subset of activities of a certain type, they should avoid all activities of that type.']", "label": 3 }, { "id": "train_2729", "context": "Computer manufacturers have sought to make computer chips ever smaller, since decreasing the size of a computer' s central processing unit (CPU) chip -- without making that CPU chip any less sophisticated -- will proportionally increase the speed of the CPU chip and the computer containing it. But since CPU chips cannot be made significantly smaller without decreasing their sophistication, computers cannot currently be made significantly faster.", "question": "Which one of the following is an assumption on which the argument depends?", "answers": "['If both the size and the sophistication of a CPU chip are decreased, the speed of that chip will decrease.', 'Even if CPU chips are made slightly less sophisticated, they cannot currently be made much smaller.', 'Computers cannot currently be made faster unless their CPU chips are made smaller.', 'Few, if any, computer manufacturers believe that computers can be made significantly faster.']", "label": 2 }, { "id": "train_2730", "context": "Commentator: For a free market to function properly, each prospective buyer of an item must be able to contact a large number of independent prospective sellers and compare the prices charged for the item to what the item is worth. Thus, despite advertised prices and written estimates available from many of its individual businesses, the auto repair industry does not constitute a properly functioning free market.", "question": "The conclusion of the commentator's argument follows logically if which one of the following is assumed?", "answers": "['People do not usually shop for auto repairs but instead take their autos to their regular repair shop out of habit.', 'Because it is not regulated, the auto repair industry does not have standardized prices.', 'Many auto repair shops charge more for auto repairs than these repairs are worth.', 'Some persons who are shopping for auto repairs cannot determine what these repairs are worth.']", "label": 3 }, { "id": "train_2731", "context": "Company president: For the management consultant position, we shall interview only those applicants who have worked for management consulting firms generally recognized as in the top 1 percent of firms worldwide. When we finally select somebody, then, we can be sure to have selected one of the best management consultants available.", "question": "The company president's reasoning is most vulnerable to criticism on the grounds that it", "answers": "['takes for granted that if something is true of each member of a collection, then it is also true of the collection as a whole', 'presumes, without providing warrant, that persons who have worked for the top companies will accept a job offer', 'generalizes from too small a sample of management consulting firms worldwide', 'takes for granted that only the best management consultants have worked for the top management consulting firms']", "label": 3 }, { "id": "train_2732", "context": "Advocate: A study of people who had recently recovered from colds found that people who took cold medicine for their colds reported more severe symptoms than those people who did not take cold medicine. Ttherefore, taking cold medicine is clearly counterproductive.", "question": "The reasoning in the advocate's argument is flawed because the argument", "answers": "['treats something as true simply because most people believe it to be true', 'treats some people as experts in an area in which there is no reason to take them to be reliable sources of information', 'rests on a confusion between what is required for a particular outcome and what is sufficient to cause that outcome', 'confuses what is likely the cause of something for an effect of that thing']", "label": 3 }, { "id": "train_2733", "context": "Archaeologist: The mosaics that were removed from Zeugma, the ancient city now flooded by the runoff from Turkey' s Birecik Dam, should have been left there. We had all the information about them that we needed to draw archaeological conclusions, and future archaeologists studying the site, who may not have access to our records, might be misled by their absence.", "question": "Which one of the following, if assumed, most helps to justify the reasoning in the archaeologist's argument?", "answers": "['The only considerations that bear upon the question of whether the mosaics should have been removed are archaeological.', 'The removal of artifacts from archaeological sites rarely has any environmental impact.', 'Archaeological sites from which artifacts have been removed rarely mislead archaeologists who later study the site.', 'The materials used in the construction of a mosaic are readily apparent when the mosaic is examined in its original location.']", "label": 0 }, { "id": "train_2734", "context": "Three major laundry detergent manufacturers have concentrated their powdered detergents by reducing the proportion of inactive ingredients in the detergent formulas. The concentrated detergents will be sold in smaller packages. In explaining the change, the manufacturers cited the desire to reduce cardboard packaging and other production costs. Market analysts predict that the decision of these three manufacturers, who control 80 percent of the laundry detergent market, will eventually bring about the virtual disappearance of old-style bulky detergents.", "question": "Which one of the following, if true, most strongly supports the prediction made by the market analysts?", "answers": "['Most smaller manufacturers of laundry detergents will consider it too expensive to retool factories for the production of the smaller detergent packages.', 'Some analysts believe that consumers will have to pay a greater cost per load of laundry to use the new concentrated detergent than they did to use the old-style detergent.', 'Consumers are increasingly being persuaded by environmental concerns to buy concentrated detergents when available in order to reduce cardboard waste.', 'Major supermarkets have announced that they will not charge the detergent manufacturers less to display their detergents, even though the detergents will take up less shelf space.']", "label": 2 }, { "id": "train_2735", "context": "Lance: If experience teaches us nothing else, it teaches us that every general rule has at least one exception. Frank: What you conclude is itself a general rule. If we assume that it is true, then there is at least one general rule that has no exceptions. Ttherefore, you must withdraw your conclusion.", "question": "Frank's argument is an attempt to counter Lance's conclusion by", "answers": "['demonstrating that Lance assumes the very thing he sets out to prove', 'establishing that experience teaches us the opposite of what Lance concludes', \"showing that Lance's conclusion involves him in a contradiction\", 'showing that it has no implications for any real cases']", "label": 2 }, { "id": "train_2736", "context": "The Volunteers for Literacy Program would benefit if Dolores takes Victor' s place as director, since Dolores is far more skillful than Victor is at securing the kind of financial support the program needs and Dolores does not have Victor' s propensity for alienating the program' s most dedicated volunteers.", "question": "The pattern of reasoning in the argument above is most closely paralleled in which one of the following?", "answers": "[\"Joshua's interest would be better served by taking the bus to get to his parent's house rather than by taking an airplane, since his primary concern is to travel as cheaply as possible and taking the bus is less expensive than going by airplane.\", \"It would be to Fred's advantage to exchange his bus tickets for train tickets, since he needs to arrive at his meeting before any of the other participants and if he goes by bus at least one of the other participants will arrive first.\", 'It would be more convenient for Dominique to take a bus to school than to take the subway, since the bus stops closer to her house than does the subway and, unlike the subway, the bus goes directly to the school.', 'Anita would benefit financially by taking the train to work rather than driving her car, since when she drives she has to pay parking fees and the daily fee for parking a car is higher than a round-trip train ticket.']", "label": 2 }, { "id": "train_2737", "context": "A retrospective study is a scientific study that tries to determine the causes of subjects' present characteristics by looking for significant connections between the present characteristics of subjects and what happened to those subjects in the past, before the study began. Because retrospective studies of human subjects must use the subjects' reports about their own pasts, however, such studies cannot reliably determine the causes of human subjects' present characteristics.", "question": "Which one of the following, if assumed, enables the argument's conclusion to be properly drawn?", "answers": [ "A retrospective study cannot reliably determine the causes of human subjects' present characteristics unless there exist correlations between the present characteristics of the subjects and what happened to those subjects in the past.", "In studies of human subjects that attempt to find connections between subjects' present characteristics and what happened to those subjects in the past, the subjects' reports about their own pasts are highly susceptible to inaccuracy.", "Every scientific study in which researchers look for significant connections between the present characteristics of subjects and what happened to those subjects in the past must use the subjects' reports about their own pasts.", "Whether or not a study of human subjects can reliably determine the causes of those subjects' present characteristics may depend at least in part on the extent to which that study uses inaccurate reports about the subjects' pasts." ], "label": 1 }, { "id": "train_2738", "context": "The price of the SuperPixel high definition television, by Lux Electronics, has typically been out of the range of most consumers, a few of whom nonetheless save up for the television. This past July, the SuperPixel reduced its price by 40%, and sales during that month nearly tripled. TechWare, a popular electronics magazine, claims that the SuperPixel television should continue to see sales grow at this rate till the end of August.", "question": "Which of the following suggests that TechWare's forecast is misguided?", "answers": "['Electronics sales tend to peak in August and December.', 'The SuperPixel is the only model for which Lux Electronics plans a price reduction.', 'The SuperPixel tends to be an unreliable television and Lux Electronics makes a considerable profit from repairs.', 'Most of the customers who had been saving up for the SuperPixel bought the television in July.']", "label": 3 }, { "id": "train_2739", "context": "Columnist: Analysts argue that as baby boomers reach the age of 50, they will begin seriously planning for retirement. This will lead them to switch from being primarily consumers to being savers. Thus, these analysts conclude, more money will flow into the stock market, resulting in continued gains in stock prices. Analysts would stand to gain if this were true, but they are being overly optimistic. As consumption decreases, so will corporate earnings; ttherefore high stock prices will not be justified, and thus boomers' money will more likely flow into investments other than stocks.", "question": "The columnist's argument does which one of the following?", "answers": [ "attempts to undermine the analysts' argument by drawing an alternative conclusion from the analysts' premises", "attempts to undermine the analysts' argument by suggesting that the analysts present it for self-serving reasons", "attempts to undermine the analysts' argument by questioning the truth of its premises", "argues in favor of the analysts' conclusion, but does so on the basis of a different body of evidence" ], "label": 0 }, { "id": "train_2740", "context": "The frequently expressed view that written constitutions are inherently more liberal than unwritten ones is false. No written constitution is more than a paper with words on it until those words are both interpreted and applied. Properly understood, then, a constitution is the sum of those procedures through which the power of the state is legitimately exercised and limited. Ttherefore, even a written constitution becomes a liberal constitution only when it is interpreted and applied in a liberal way.", "question": "The main point of the argument above is that", "answers": "['the idea of a written constitution, properly understood, is inherently self-contradictory', 'there are criteria for evaluating the interpretation and application of a constitution', 'written constitutions are no more inherently liberal than are unwritten constitutions', 'liberal constitutions are extremely difficult to preserve']", "label": 2 }, { "id": "train_2741", "context": "No one who lacks knowledge of a subject is competent to pass judgment on that subject. Since political know-how is a matter, not of adhering to technical rules, but of insight and style learned through apprenticeship and experience, only seasoned politicians are competent to judge whether a particular political policy is fair to all.", "question": "A major weakness of the argument is that it", "answers": "['assumes that when inexperienced politicians set policy they are guided by the advice of more experienced politicians', 'equates political know-how with understanding the social implications of political policies', 'uses the term \"apprenticeship\" to describe what is seldom a formalized relationship', 'fails to give specific examples to illustrate how political know-how can be acquired']", "label": 1 }, { "id": "train_2742", "context": "Rose: The book is either by Deerson or else by Jones; I' m not sure which. However, Deerson' s books are generally published by Quince Press, as are Jones' s. Ttherefore, the book is probably published by Quince.", "question": "The pattern of reasoning in which one of the following is most similar to that in Rose's argument?", "answers": "['Tomas will probably participate in community service, since he will attend either Dunkeld College or Steventon University and at both most students currently enrolled say that they participate in some form of community service.', 'Judging by what he said in the interview, George is either a liar or incredibly naive. Both these attributes are unsuitable in a customs inspector. Ttherefore George should not be hired as a customs inspector.', 'Margarethe the Second was born either in Luppingshavn or else in Kindelberg. Most of the people in each city then were of Mondarian descent, so Margarethe probably had Mondarian ancestors.', 'That tree is either a beech or else an elm, and Mercedes can identify most trees, so she will probably be able to tell which it is.']", "label": 2 }, { "id": "train_2743", "context": "Environmentalist: The use of snowmobiles in the vast park north of Milville creates unacceptable levels of air pollution and should be banned. Milville business spokesperson: Snowmobiling brings many out-of-towners to Milville in winter months, to the great financial benefit of many local residents. So, economics dictate that we put up with the pollution. Environmentalist: I disagree: A great many cross-country skiers are now kept from visiting Milville by the noise and pollution that snowmobiles generate.", "question": "Environmentalist responds to the business spokesperson by doing which of the following?", "answers": "['Challenging an assumption that certain desirable outcome can derive from only one set of circumstances', \"Claiming that the spokesperson is deliberately misrepresenting the environmentalist's position in order to be better able to attack it\", 'Denying that an effect that the spokesperson presents as having benefited a certain group of people actually benefited those people', 'Maintaining that the benefit that the spokesperson desires could be achieved in greater degree by a different means']", "label": 2 }, { "id": "train_2744", "context": "Hospital patients generally have lower infection rates and require shorter hospital stays if they are housed in private rooms rather than semiprivate rooms. Yet in Woodville' s hospital, which has only semiprivate rooms, infection rates and length of stays are typically the same as in several nearby hospitals where most of the rooms are private, even though patients served by these hospitals are very similar to those served by Woodville' s hospital.", "question": "Which one of the following, if true, most helps to resolve the apparent conflict in the information above?", "answers": "[\"Woodville's hospital has a policy of housing one patient per room in semiprivate rooms whenever possible.\", 'Infection is more likely to be spread where people come into close contact with one another than where they do not.', \"Most of the nearby hospitals were built within the last 10 years, whereas Woodville's hospital was built about 50 years ago.\", \"Many of the doctors who routinely treat patients in Woodville's hospital also routinely treat patients in one or more of the nearby hospitals.\"]", "label": 0 }, { "id": "train_2745", "context": "When the ancient fossils of a primitive land mammal were unearthed in New Zealand, they provided the first concrete evidence that the island country had once had indigenous land mammals. Until that discovery, New Zealand had no known native land mammals. The discovery thus falsifies the theory that New Zealand' s rich and varied native bird population owes its existence to the lack of competition from mammals.", "question": "Which one of the following, if true, most seriously weakens the argument?", "answers": "['Some other island countries that are believed to have no native land mammals in fact had indigenous land mammals at one time.', 'The unearthed land mammal is only one of several ancient land mammals that were indigenous to New Zealand.', 'Countries with rich and varied native land mammal populations do not have rich and varied native bird populations.', 'The recently discovered land mammal became extinct long before the native bird population was established.']", "label": 3 }, { "id": "train_2746", "context": "Books about artwork, unless they are intended for a general audience, ought to include discussions of both the meaning behind the piece of art and the background of the artist who created that piece. If they do not, they are flawed. Gardener' s book on paintings of the Renaissance era describes the significance of the paintings, but fails to mention that the artist of several paintings discussed at length in the book is considered to be one of the greatest artists of all time.", "question": "If the statements above are true, it would be necessary to establish which one of the following to conclude that Gardener's book is flawed?", "answers": [ "Gardener's descriptions of the meaning of the artists' paintings are inaccurate.", "Most prominent art historians agree that discussing an artist's critical reputation is essential to a complete portrayal of his or her background.", "The paintings discussed at length have some of the deepest meanings of those treated in Gardener's book.", "Gardener's book is intended for a general audience." ], "label": 1 }, { "id": "train_2747", "context": "In some ill-considered popularizations of interesting current research, it is argued that higher apes have the capacity for language but have never put it to use -- a remarkable biological miracle, given the enormous selectional advantage of even minimal linguistic skills. It is rather like claiming that some animal has wings adequate for flight but has never thought to fly.", "question": "Which one of the following is most similar in its reasoning to the argument above?", "answers": "['Arguing that the human brain has telekinetic powers that no humans have ever exercised is rather like arguing that some insect has legs but never uses them to walk.', 'Arguing that Earth has been visited by aliens from outer space is rather like claiming that early explorers had visited North America but never founded cities.', 'Claiming that some people raised tobacco but did not smoke it is rather like claiming that a society that knew how to brew alcohol never drank it.', 'Arguing that there are some humans who never sleep is rather like discovering a species of lion that does not eat meat.']", "label": 0 }, { "id": "train_2748", "context": "It has recently been found that job prospects for college graduates have never been better. The trend is likely to continue over the next decade. A recent survey found that most employers simply did not know that the number of students graduating would drop by 25 percent over the past ten years, and had not anticipated or planned for this trend. Most employers were not aware that, although the supply of graduates currently meets demand, this situation could change. The same survey revealed that the number of undergraduates choosing to study subjects in high demand, like mathematics and engineering, has dropped substantially. This trend is likely to continue over the next decade.", "question": "Which one of the following can properly be concluded from the passage above?", "answers": "['Soon, fewer graduates are likely to be competing for fewer available jobs.', 'Employers who are well-informed about future trends have anticipated and planned for them.', 'Soon, there is likely to be a shortage of graduates to fill certain vacancies.', 'Employers are aware of changing trends in subjects studied by undergraduates.']", "label": 2 }, { "id": "train_2749", "context": "Installing scrubbers in smokestacks and switching to cleaner-burning fuel are the two methods available to Northern Power for reducing harmful emissions from its plants. Scrubbers will reduce harmful emissions more than cleaner-burning fuels will. Ttherefore, by installing scrubbers, Northern Power will be doing the most that can be done to reduce harmful emissions from its plants.", "question": "Which of the following is an assumption on which the argument depends?", "answers": "['Northern Power can choose from among various kinds of scrubbers, some of which are more effective than others.', \"Harmful emissions from Northern Power's plants cannot be reduced more by using both methods together than by the installation of scrubbers alone.\", 'Aside from harmful emissions from the smokestacks of its plants, the activities of Northern Power do not cause significant air pollution.', 'Switching to cleaner-burning fuel will not be more expensive than installing scrubbers.']", "label": 1 }, { "id": "train_2750", "context": "The enthusiastic acceptance of ascetic lifestyles evidenced in the surviving writings of monastic authors indicates that medieval societies were much less concerned with monetary gain than are contemporary Western cultures.", "question": "The reasoning in the argument is most vulnerable to criticism on the grounds that the argument", "answers": "['inserts personal opinions into what purports to be a factual debate', 'employs the imprecise term \"ascetic\"', 'generalizes from a sample that is likely to be unrepresentative', 'applies contemporary standards inappropriately to medieval societies']", "label": 2 }, { "id": "train_2751", "context": "A tree' s age can be determined by counting the annual growth rings in its trunk. Each ring represents one year, and the ring' s thickness reveals the relative amount of rainfall that year. Archaeologists successfully used annual rings to determine the relative ages of ancient tombs at Pazyryk. Each tomb was constructed from freshly cut logs, and the tombs' builders were constrained by tradition to use only logs from trees growing in the sacred Pazyryk Valley.", "question": "Which one of the following, if true, contributes most to an explanation of the archaeologists' success in using annual rings to establish the relative ages of the tombs at the Pazyryk site?", "answers": "['The Pazyryk Valley, surrounded by extremely high mountains, has a distinctive yearly pattern of rainfall, and so trees growing in the Pazyryk Valley have annual rings that are quite distinct from trees growing in nearby valleys.', 'The archaeologists determined that the youngest tree used in any of the tombs was 90 years old and that the oldest tree was 450 years old.', 'Each log in the Pazyryk tombs has among its rings a distinctive sequence of twelve annual rings representing six drought years followed by three rainy years and three more drought years.', \"The Pazyryk tombs were all robbed during ancient times, but breakage of the tombs' seals allowed the seepage of water, which soon froze permanently, thereby preserving the tombs' remaining artifacts.\"]", "label": 2 }, { "id": "train_2752", "context": "Brian: Reporters and photographers are legally required to automatically distort the features of any bystanders whose faces might be shown on television before they can broadcast the taped segment. Yet when people are interviewed for a story, this practice is not followed. If we were as concerned about protecting the privacy of people as we say we are, then reporters would be required to automatically distort the features of the people they interview too. Jessica: But consider this: A person being interviewed can simply ask to have his or her features distorted if he or she wants privacy, where it might not be possible for the bystanders to know they are being filmed. So you see, automatically distorting the features of people being interviewed is not necessary.", "question": "Jessica attempts to counter Brian's argument by", "answers": "[\"presenting a detailed example to show that Brian's argument is based on premises that cannot be confirmed or disproved\", \"drawing another analogy that uncovers a serious mistake in Brian's reasoning\", 'showing that information on which Brian bases his conclusion is false', 'pointing out a relevant distinction between the two groups to weaken an analogy that Brian bases his argument on']", "label": 3 }, { "id": "train_2753", "context": "Elaine: The purpose of art museums is to preserve artworks and make them available to the public. Museums, ttherefore, should seek to acquire and display the best examples of artworks from each artistic period and genre, even if some of these works are not recognized by experts as masterpieces. Frederick: Art museums ought to devote their limited resources to acquiring the works of recognized masters in order to ensure the preservation of the greatest artworks.", "question": "Elaine's and Frederick's statements provide the most support for the claim that they would disagree about whether", "answers": "['museums should seek to represent all genres of art in their collections', 'many artistic masterpieces are not recognized as such by art experts', 'an art museum ought to acquire an unusual example of a period or genre if more characteristic examples are prohibitively expensive', 'art museums should seek to preserve works of art']", "label": 0 }, { "id": "train_2754", "context": "Male bowerbirds construct elaborately decorated nests, or bowers. Basing their judgment on the fact that different local populations of bowerbirds of the same species build bowers that exhibit different building and decorative styles, researchers have concluded that the bowerbirds' building styles are a culturally acquired, rather than a genetically transmitted, trait.", "question": "Which of the following, if true, would most strengthen the conclusion drawn by the researchers?", "answers": "['The bowers of one species of bowerbird lack the towers and ornamentation characteristic of the bowers of most other species of bowerbird.', 'There are more common characteristics than there are differences among the bower-building styles of the local bowerbird population that has been studied most extensively.', 'Young male bowerbirds are inept at bowerbuilding and apparently spend years watching their elders before becoming accomplished in the local bower style.', 'Bowerbirds are found only in New Guinea and Australia, where local populations of the birds apparently seldom have contact with one another.']", "label": 2 }, { "id": "train_2755", "context": "The foreign minister of Zeria announced today that her country was severing diplomatic relations with Nandalo because of Nandalo' s flagrant violations of human rights. But Zeria continues to maintain diplomatic relations with many countries that the minister knows to have far worse human-rights records than Nandalo does. Ttherefore, despite the foreign minister' s claim, this latest diplomatic move cannot be explained exclusively by Zeria' s commitment to upholding human rights.", "question": "Which one of the following, if true, provides the most support for the argument in the passage?", "answers": "['The opposition party in Zeria has long advocated severing trade relations with countries that systematically violate human rights but has opposed severing diplomatic relations.', \"Two weeks after the Zerian minister's announcement, several other countries cited human-rights violations as a reason for severing diplomatic relations with Nandalo.\", 'More countries have expressed concern over reported human-rights violations in Nandalo than have expressed concern over human-rights violations in Zeria.', \"The country that currently buys most of Zeria's exports recently suggested that it might severely restrict its imports from Zeria unless Zeria broke off diplomatic relations with Nandalo.\"]", "label": 3 }, { "id": "train_2756", "context": "Each of many different human hormones can by itself raise the concentration of glucose in the blood. The reason for this is probably a metabolic quirk of the brain. To see this, consider that although most human cells can produce energy from fats and proteins, brain cells can use only glucose. Thus, if blood glucose levels fall too low, brain cells will rapidly starve, leading to unconsciousness and death.", "question": "Which one of the following most accurately expresses the main conclusion of the argument above?", "answers": "['The reason that many different hormones can each independently raise blood glucose levels is probably a metabolic quirk of the brain.', 'If blood glucose levels fall too low, then brain cells starve, resulting in loss of consciousness and death.', 'Although most human cells can produce energy from fats and proteins, brain cells can produce energy only from glucose.', 'The reason brain cells starve if deprived of glucose is that they can produce energy only from glucose.']", "label": 0 }, { "id": "train_2757", "context": "The number of hospital emergency room visits by heroin users grew by more than 25 percent during the 1980s. Clearly, then, the use of heroin rose in that decade.", "question": "Which one of the following, if true, would account for the statistic above without supporting the author's conclusion?", "answers": "['Many hospital emergency rooms were barely able to accommodate the dramatic increase in the number of medical emergencies related to drug abuse during the 1980s.', 'Viral and bacterial infections, malnourishment, and overdoses account for most hospital emergency room visits linked to heroin.', 'Heroin use increased much more than is reflected in the rate of heroin-linked hospital emergency room visits.', 'Widespread use of automatic weapons in the drug trade during the 1980s raised the incidence of physical injury to heroin users.']", "label": 3 }, { "id": "train_2758", "context": "Neural connections carrying signals from the cortex (the brain region responsible for thought) down to the amygdala (a brain region crucial for emotions) are less well developed than connections carrying signals from the amygdala up to the cortex. Thus, the amygdala exerts a greater influence on the cortex than vice versa.", "question": "The argument's conclusion follows logically if which one of the following is assumed?", "answers": "['The degree of development of a set of neural connections is directly proportional to the influence transmitted across those connections.', 'The region of the brain that has the most influence on the cortex is the one that has the most highly developed neural connections to the cortex.', 'The influence that the amygdala exerts on the rest of the brain is dependent on the influence that the cortex exerts on the rest of the brain.', 'The amygdala is not itself controlled by one or more other regions of the brain.']", "label": 0 }, { "id": "train_2759", "context": "Fishing columnist: When an independent research firm compared the five best-selling baits, it found that Benton baits work best for catching trout. It asked a dozen top anglers to try out the five best-selling baits as they fished for speckled trout in a pristine northern stream, and every angler had the most success with a Benton bait. These results show that Benton is the best bait for anyone who is fishing for trout.", "question": "Each of the following describes a flaw in the reasoning in the fishing columnist's argument EXCEPT:", "answers": "['The argument overlooks the possibility that two best-selling brands of bait may be equally effective.', 'The argument overlooks the possibility that what works best for expert anglers will not work best for ordinary anglers.', 'The argument overlooks the possibility that the relative effectiveness of different baits changes when used in different locations.', 'The argument overlooks the possibility that some other bait is more successful than any of the five best-selling baits.']", "label": 0 }, { "id": "train_2760", "context": "Driver: My friends say I will one day have an accident because I drive my sports car recklessly. But I have done some research, and apparently minivans and larger sedans have very low accident rates compared to sports cars. So trading my sports car in for a minivan would lower my risk of having an accident.", "question": "The reasoning in the driver's argument is most vulnerable to criticism on the grounds that this argument", "answers": "['misinterprets evidence that a result is likely as evidence that the result is certain', 'infers a cause from a mere correlation', 'relies on a source that is probably not well-informed', 'mistakes a condition sufficient for bringing about a result for a condition necessary for doing so']", "label": 1 }, { "id": "train_2761", "context": "If a mechanical aerator is installed in a fish pool, the water in the pool can be properly aerated. So, since John' s fish pool does not have a mechanical aerator, it must be that his pool is not properly aerated. Without properly aerated water, fish cannot thrive. Ttherefore, any fish in John' s fish pool will not thrive.", "question": "Which one of the following arguments contains an error of reasoning that is also contained in the argument above?", "answers": "['If pectin is added to jam, the jam will gel. Without a setting agent such as pectin, jam will not gel. So in order to make his jam gel, Harry should add a setting agent such as pectin to the jam.', 'If stored potatoes are not exposed to ethylene, the potatoes will not sprout. Beets do not release ethylene. Ttherefore, if Sara stores her potatoes together with beets, the potatoes will not sprout.', \"If tomatoes are not stored in a dark place, their seeds sometimes sprout. Sprouted seeds can make tomatoes inedible. Ttherefore, since Maria does not store her tomatoes in a dark place, some of Maria's tomatoes could be inedible.\", \"If alum is added to pickle brine, brine can replace the water in the pickles. Ttherefore, since Paula does not add alum to her pickle brine, the water in the pickles cannot be replaced by brine. Unless their water is replaced with brine, pickles will not stay crisp. Thus, Paula's pickles will not stay crisp.\"]", "label": 3 }, { "id": "train_2762", "context": "Yeasts capable of leavening bread are widespread, and in the many centuries during which the ancient Egyptians made only unleavened bread, such yeasts must frequently have been mixed into bread doughs accidentally. The Egyptians, however, did not discover leavened bread until about 3000 B. C. That discovery roughly coincided with the introduction of a wheat variety that was preferable to previous varieties because its edible kernel could be removed from the husk without first toasting the grain.", "question": "Which of the following, if true, provides the strongest evidence that the two developments were causally related?", "answers": "['Even after the ancient Egyptians discovered leavened bread and the techniques for reliably producing it were well known, unleavened bread continued to be widely consumed.', 'Only when the Egyptians stopped the practice of toasting grain were their stone-lined grain-toasting pits available for baking bread.', 'The new variety of wheat, which had a more delicate flavor because it was not toasted, was reserved for the consumption of high officials when it first began to be grown.', 'Heating a wheat kernel destroys its gluten, a protein that must be present in order for yeast to leaven bread dough.']", "label": 3 }, { "id": "train_2763", "context": "It is repeatedly claimed that the dumping of nuclear waste poses no threat to people living nearby. If this claim could be made with certainty, there would be no reason for not locating sites in areas of dense population. But the policy of dumping nuclear waste only in the more sparsely populated regions indicates, at the very least, some misgiving about safety on the part of those responsible for policy.", "question": "Which one of the following, if true, would most seriously weaken the argument?", "answers": "['Until there is no shred of doubt that nuclear dumps are safe, it makes sense to situate them where they pose the least threat to the public.', 'There are dangers associated with chemical waste, and it, too, is dumped away from areas of dense population.', 'In the event of an accident, it is certain that fewer people would be harmed in a sparsely populated than in a densely populated area.', 'Dumping of nuclear waste poses fewer economic and bureaucratic problems in sparsely populated than in densely populated areas.']", "label": 3 }, { "id": "train_2764", "context": "Using rational argument in advertisements does not persuade people to buy the products being advertised. Ttherefore, advertisers who replace rational argument with nonrational appeals to emotion in advertisements will persuade people to buy the products being advertised.", "question": "Which one of the following contains flawed reasoning most similar to the flawed reasoning in the argument above?", "answers": "['In the past, people who have tried to solve their problems by discussing them have often failed. Ttherefore, in the future, people who try to solve their problems by discussing them will often fail.', 'A person who does to have positive letters of reference cannot get a good job. Ttherefore, the better the letters of reference a person has, the better the job that person will get.', 'People who ask others for favors are refused. Ttherefore, anyone who has not had the experience of being refused has never asked for a favor.', 'People never learn to program a computer by reading poorly written directions. Ttherefore, if people read well-written directions, they will learn to program a computer.']", "label": 3 }, { "id": "train_2765", "context": "Colfax Beta-80 is a rare genetic defect found primarily in people of Scandinavian descent. Over 97% of known carriers of this defect are citizens of, or are direct descendants of immigrants from, Denmark, Norway, and Sweden. People who carry the Colfax Beta-80 defect are at substantially higher risk for contracting Lupus and related autoimmune diseases.", "question": "Assuming the statements above are true, which of the following can be inferred from them?", "answers": "['Genetic engineering that eradicated this genetic defect would constitute a de facto cure for Lupus.', 'A person not of Scandinavian descent born with the Colfax Beta-80 defect is more likely to contract Lupus than is a Scandinavian who is born without this defect.', 'The majority of people who contract Lupus are either Scandinavian or of Scandinavian descent.', 'Find a cure for Lupus would eliminate most of the health threats associated with the Colfax Beta-80 defect.']", "label": 1 }, { "id": "train_2766", "context": "University president: We will be forced to reduce spending next year if we do not increase our enrollment. So, if we are to maintain the quality of the education we provide, we must market our programs more aggressively. Without such marketing we will be unable to increase our enrollment.", "question": "The conclusion of the university president's argument can be properly drawn if which one of the following is assumed?", "answers": "['The university will not maintain the quality of the education it provides if it reduces spending next year.', 'The university will not need to reduce spending next year if it increases its enrollment.', 'The university will not need to reduce spending next year if it markets its programs more aggressively.', 'The university will not maintain the quality of the education it provides if it increases its enrollment.']", "label": 0 }, { "id": "train_2767", "context": "The vast majority of first-year engineering students at Bighorn University are registered in Physics 121. Simone is registered in Physics 121; thus it seems likely that she is a first-year engineering student.", "question": "Which one of the following statements, if assumed, enables the conclusion above to be properly drawn?", "answers": "['The engineering program at Bighorn University is highly structured and leaves its students few opportunities to take nonscience elective courses.', 'Every engineering student at Bighorn University must pass Physics 121 or an equivalent in order to meet degree requirements.', 'The number of first-year engineering students at Bighorn University is slightly larger than the total number of students registered in Physics 121.', 'Some engineering students at Bighorn University take Physics 121 in their second year, but none take it in later years.']", "label": 2 }, { "id": "train_2768", "context": "Cats spend much of their time sleeping; they seem to awaken only to stretch and yawn. Yet they have a strong, agile musculature that most animals would have to exercise strenuously to acquire.", "question": "Which one of the following, if true, most helps to resolve the apparent paradox described above?", "answers": "['Many other animals also spend much of their time sleeping yet have a strong, agile musculature.', 'Cats derive ample exercise from frequent stretching.', 'Cats require strength and agility in order to be effective predators.', 'Cats have a greater physiological need for sleep than other animals.']", "label": 1 }, { "id": "train_2769", "context": "Television commercial: Compared to other television cable services, Acme is clearly superior. Around the nation, thousands of satisfied Acme customers have testified on Acme' s website that they have had uninterrupted cable service for two or more years. Some of these Acme customers even testify that they have had uninterrupted cable service for five years! Obviously, if you sign up for Acme television cable service, you can be assured that you will have uninterrupted cable service for years at a time.", "question": "The television commercial's reasoning is most susceptible to criticism on which of the following grounds?", "answers": "[\"The commercial incorrectly uses a key term to conceal the terms' innate ambiguities.\", 'The commercial makes a broad conclusion based on a few choice circumstances that may, or may not, be representative.', 'The commercial simply reiterates one of its premises and claims that it is a conclusion.', 'The commercial bases its argument on premises that are inherently self-contradictory.']", "label": 1 }, { "id": "train_2770", "context": "In August, the Williamstown main city parking garage was completely full every single day of the month. This phenomenon indicates that the main city parking garage in Williamstown is a popular place for parking. therefore, the Williamstown parking garage will be full every day of the month for September.", "question": "Which of the following, if true, cast the most doubt on the conclusion above?", "answers": "['Capeville, a neighboring city, has only one parking garage, though it was not filled every day of the month.', 'In July, the Williamstown parking garage was full every day of the month.', 'Williamstown is popular beach town and tourists across the nation flock to its beaches from June to August, but the city becomes virtually empty come September.', 'The Grand Hotel, a popular hotel in Williamstown, expects to have a full garage the entire month of September.']", "label": 2 }, { "id": "train_2771", "context": "Industrial scientists have hypothesized that much of the nitrous oxide that contributes to air pollution originates from the burning of organic matter in forest fires. The results of one extensive study in which smoke samples taken from forest fires were bottled and then analyzed in a research laboratory supports this hypothesis, since the analysis showed that all of the samples contained high concentrations of nitrous oxide.", "question": "Which one of the following, if true, most undermines the argument?", "answers": "['The production of synthetic products such as nylon is responsible for up to 10 percent of the nitrous oxide present in the atmosphere.', 'Soil bacteria that feed on ammonium compounds present in organic ash are thought by some scientists to excrete small amounts of nitrous oxide that then enter the atmosphere.', 'By using gas-analyzing devices mounted on helicopters, researchers can perform on-site analyses of smoke instead of sending smoke samples to laboratories for analysis.', 'When in a closed container, the gases in smoke produced by forest fires quickly react with each other thereby producing high concentrations of nitrous oxide.']", "label": 3 }, { "id": "train_2772", "context": "In Gandania, where the government has a monopoly on tobacco sales, the incidence of smoking-related health problems has risen steadily for the last twenty years. The health secretary recently proposed a series of laws aimed at curtailing tobacco use in Gandania. Profits from tobacco sales, however, account for ten percent of Gandania' s annual revenues. Ttherefore, Gandania cannot afford to institute the proposed laws.", "question": "Which of the following, if true, most seriously weakens the argument?", "answers": "['Implementing the proposed laws is not likely to cause a significant increase in the amount of tobacco Gandania exports.', 'No government official in Gandania has ever previously proposed laws aimed at curtailing tobacco use.', 'All health care in Gandania is government-funded.', 'The percentage of revenue Gandania receives from tobacco sales has remained steady in recent years.']", "label": 2 }, { "id": "train_2773", "context": "Ann: All the campers at Camp Winnehatchee go to Tri-Cities High School. Bill: That' s not true. Some Tri-Cities students are campers at Camp Lakemont.", "question": "Bill's answer can be best explained on the assumption that he has interpreted Ann's remark to mean that", "answers": "['some Tri-Cities High School students have withdrawn from Camp Lakemont', 'most of the campers at Camp Lakemont come from high schools other than Tri-Cities', 'only campers at Camp Winnehatchee are students at Tri-Cities High School', 'most Tri-Cities High School students are campers at Camp Winnehatchee']", "label": 2 }, { "id": "train_2774", "context": "Proponent: Irradiation of food by gamma rays would keep it from spoiling before it reaches the consumer in food stores. The process leaves no radiation behind, and vitamin losses are comparable to those that occur in cooking, so there is no reason to reject irradiation on the grounds of nutrition or safety. Indeed, it kills harmful Salmonella bacteria, which in contaminated poultry have caused serious illness to consumers. Opponent: The irradiation process has no effect on the bacteria that cause botulism, a very serious form of food poisoning, while those that cause bad odors that would warn consumers of botulism are killed. Moreover, Salmonella and the bacteria that cause botulism can easily be killed in poultry by using a safe chemical dip.", "question": "The opponent's argument proceeds by", "answers": "[\"showing that claims made in the proponent's argument result in a self-contradiction\", 'shifting perspective from safety with respect to consumers to safety with respect to producers', 'establishing that undesirable consequences result from the adoption of either one of two proposed remedies', 'pointing out an alternative way of obtaining an advantage claimed by the proponent without risking a particular disadvantage']", "label": 3 }, { "id": "train_2775", "context": "Situation: In the island nation of Bezun, the government taxes gasoline heavily in order to induce people not to drive. It uses the revenue from the gasoline tax to subsidize electricity in order to reduce prices charged for electricity. Analysis: The greater the success achieved in meeting the first of these objectives, the less will be the success achieved in meeting the second.", "question": "The analysis provided for the situation above would be most appropriate in which one of the following situations?", "answers": "['A mail-order store imposes a stiff surcharge for overnight delivery in order to limit use of this option. The store uses revenue from the surcharge to pay the extra expenses it incurs for providing the overnight delivery service.', \"The park management charges an admission fee so that a park's users will contribute to the park's upkeep. In order to keep admission fees low, the management does not finance any new projects from them.\", 'The highway administration charges a toll for crossing a bridge in order to get motorists to use other routes. It uses the revenue from that toll to generate a reserve fund in order to be able one day to build a new bridge.', 'A library charges a late fee in order to induce borrowers to return books promptly. The library uses revenue from the late fee to send reminders to tardy borrowers in order to reduce the incidence of overdue books.']", "label": 2 }, { "id": "train_2776", "context": "The hearts of patients who are given vitamin E before undergoing heart surgery are less susceptible to certain postoperative complications than are the hearts of patients who are not given vitamin E before heart surgery. From six hours after surgery onward, however, the survivors from both groups show the same level of heart function, on average, and also recover equally well. Despite this lack of long-term effect, doctors often recommend vitamin E for patients about to undergo heart surgery.", "question": "Which one of the following, if true, most helps to explain the doctors' recommendation?", "answers": "['Many patients who are told that they are being given vitamin E actually receive a medically inert substance.', 'Postoperative complications occur six or more hours after surgery for some patients who have not been given vitamin E.', 'Postoperative complications pose the greatest threat to patients during the first six hours after heart surgery.', 'Certain categories of patients are inherently less likely to develop postoperative complications than other patients are.']", "label": 2 }, { "id": "train_2777", "context": "Antibiotics are standard ingredients in animal feed because they keep animals healthy and increase meat yields. However, scientists have recommended phasing out this practice, believing it may make antibiotics less effective in humans. If meat yields are reduced, however, some farmers will go out of business.", "question": "Which one of the following is most strongly supported by the information above?", "answers": "['If scientists are correct that antibiotic use in animal feed makes antibiotics less effective in humans, then some farmers will go out of business.', 'If antibiotic use in animal feed is phased out, some farmers will go out of business unless they use other means of increasing meat yields.', \"If the scientists' recommendation is not heeded, no farmers will go out of business due to reduced meat yields.\", 'If the health of their animals declines, most farmers will not be able to stay in business.']", "label": 1 }, { "id": "train_2778", "context": "Commentator: The Duke of Acredia argued long ago that only virtuous Acredian rulers concerned with the well-being of the people will be able to rule successfully. Since then, when Acredian governments have fallen, their falls have always been during the rule of one who viciously disregards the people' s needs. The Duke, then, was right about at least one thing: Concern for the welfare of the people is necessary for the successful governance of Acredia.", "question": "The reasoning in the commentator's argument is most vulnerable to criticism on the grounds that the argument", "answers": "['infers that a certain condition is required for success from the fact that the lack of that condition is associated with failure', 'ignores the possibility that the conditions that are necessary for the welfare of the people are likely to change over time', 'appeals to evidence from sources that are likely to be in some way biased or unreliable', 'infers the necessity of a certain condition for success from the fact that its absence has always led to failure']", "label": 0 }, { "id": "train_2779", "context": "Commemorative plaques cast from brass are a characteristic art form of the Benin culture of West Africa. Some scholars, noting that the oldest surviving plaques date to the 1400s, hypothesize that brass-casting techniques were introduced by the Portuguese, who came to Benin in 1485 A. D. But Portuguese records of that expedition mention cast-brass jewelry sent to Benin's king from neighboring Ife. So it is unlikely that Benin's knowledge of brass casting derived from the Portuguese.", "question": "Which of the following, if true, most strengthens the argument?", "answers": "['In the 1400s the Portuguese did not use cast brass for commemorative plaques.', 'Copper, which is required for making brass, can be found throughout Benin territory.', 'As early as 1500 A. D. , Benin artists were making brass plaques incorporating depictions of Europeans.', 'The Portuguese had no contact with Ife until the 1500s.']", "label": 3 }, { "id": "train_2780", "context": "Lydia: Red squirrels are known to make holes in the bark of sugar maple trees and to consume the trees' sap. Since sugar maple sap is essentially water with a small concentration of sugar, the squirrels almost certainly are after either water or sugar. Water is easily available from other sources in places where maple trees grow, so the squirrels would not go to the trouble of chewing holes in trees just to get water. Ttherefore, they are probably after the sugar. Galina: It must be something other than sugar, because the concentration of sugar in the maple sap is so low that a squirrel would need to drink an enormous amount of sap to get any significant amount of sugar.", "question": "Lydia's argument proceeds by", "answers": "['rejecting a possible alternative explanation for an observed phenomenon', 'dismissing potentially disconfirming data', 'drawing an analogy between well-understood phenomena and an unexplained phenomenon', 'presenting an observed action as part of a larger pattern of behavior']", "label": 0 }, { "id": "train_2781", "context": "Labor representative: Social historians have shown conclusively that if workers strike when the working conditions at their jobs are poor, those conditions usually significantly improve after five years. Although workers in this industry are familiar with this fact, they nonetheless refuse to strike even though their working conditions are poor.", "question": "Which one of the following, if true, most helps to resolve the apparent discrepancy described by the labor representative?", "answers": "['Wages in this industry have increased each year.', 'Workers typically plan to work in this industry only three years.', 'Working conditions in many other industries are worse than conditions in this industry.', 'Most factories in this industry change ownership every two years.']", "label": 1 }, { "id": "train_2782", "context": "Joanna: The only way for a company to be successful, after emerging from bankruptcy, is to produce the same goods or services that it did before going bankrupt. It is futile for such a company to try to learn a whole new business. Ruth: Wrong. The Kelton Company was a major mining operation that went into bankruptcy. On emerging from bankruptcy, Kelton turned its mines into landfills and is presently a highly successful waste-management concern.", "question": "Ruth uses which one of the following argumentative techniques in countering Joanna's argument?", "answers": "['She undermines a claim by showing that it rests on an ambiguity.', 'She offers an alternative explanation for a phenomenon.', 'She presents a counterexample to a claim.', 'She establishes a conclusion by excluding the only plausible alternative to that conclusion.']", "label": 2 }, { "id": "train_2783", "context": "A recent study showed that the immune system blood cells of the study' s participants who drank tea but no coffee took half as long to respond to germs as did the blood cells of participants who drank coffee but no tea. Thus, drinking tea boosted the participants' immune system defenses.", "question": "Which one of the following is an assumption on which the argument depends?", "answers": "['Coffee and tea do not have in common any chemicals that fight disease in the human body.', 'In the study, drinking coffee did not cause the blood cell response time to double.', \"Coffee has no health benefits that are as valuable as the boost that tea purportedly gives to the body's immune system.\", 'All of the participants in the study drank either tea or coffee, and none drank both.']", "label": 1 }, { "id": "train_2784", "context": "Byrne: One of our club' s bylaws specifies that any officer who fails to appear on time for any one of the quarterly board meetings, or who misses two of our monthly general meetings, must be suspended. Thibodeaux, an officer, was recently suspended. But Thibodeaux has never missed a monthly general meeting. Ttherefore, Thibodeaux must have failed to appear on time for a quarterly board meeting.", "question": "The reasoning in Byrne's argument is flawed in that the argument", "answers": "['presumes, without providing justification, that if certain events each produce a particular result, then no other event is sufficient to produce that result', \"takes for granted that an assumption required to establish the argument's conclusion is sufficient to establish that conclusion\", 'fails to consider the possibility that Thibodeaux has arrived late for two or more monthly general meetings', 'does not specify how long Thibodeaux has been an officer']", "label": 0 }, { "id": "train_2785", "context": "Linguist: The Sapir-Whorf hypothesis states that a society' s world view is influenced by the language or languages its members speak. But this hypothesis does not have the verifiability of hypotheses of physical science, since it is not clear that the hypothesis could be tested.", "question": "If the linguist's statements are accurate, which one of the following is most supported by them?", "answers": "['Only verifiable hypotheses should be seriously considered.', 'Only the hypotheses of physical science should be taken seriously.', 'Only the hypotheses of physical science are verifiable.', 'We do not know whether the Sapir-Whorf hypothesis is true or false.']", "label": 3 }, { "id": "train_2786", "context": "Politician P: My opponent claims that the government is obligated to raise taxes to increase funding for schools and health care. Because raising taxes to increase funding for schools and health care would make taxpayers upset over their loss of buying power, my opponent is simply mistaken.", "question": "Politician P's reasoning is questionable because it involves", "answers": "['presupposing that a claim is mistaken on the grounds that the person defending it advocates other unpopular views', 'assuming that a claim is false on the grounds that the person defending it is of questionable character', 'concluding that a view is false on the grounds that its implementation would lead to unhappiness', 'appealing to wholly irrelevant issues to deflect attention away from the real issue']", "label": 2 }, { "id": "train_2787", "context": "Journalist: The new mayor is undeniably bold. His assertions are made with utter certainty and confidence. While these kinds of assertions may make him popular with the public, they also demonstrate that he is not an introspective person.", "question": "Which one of the following is an assumption required by the journalist's argument?", "answers": "['Politicians who are not bold are unpopular with the public.', 'People who are bold make public assertions with utter certainty and confidence.', 'Politicians who make assertions with utter certainty and confidence are popular with the public.', 'Introspective people do not make assertions with utter certainty and confidence.']", "label": 3 }, { "id": "train_2788", "context": "Gerrit: While browsing in a record store I noticed that one copy of a recording I wanted had mistakenly been priced at a quarter of the list price. When I finally reached the cashier I was told that the price had been mismarked and I would have to pay the full list price. Since I had wasted an hour standing in line, the retailer was morally obligated to sell me the recording at the lower price. Saskia: I disagree. You knew that a mistake had been made, and you were simply trying to take advantage of that mistake.", "question": "Which one of the following principles, if established, would most help to justify Saskia's position?", "answers": "['Customers of retail stores are morally entitled to profit from any mistakes that the retailers make in marking prices.', \"Retailers are morally entitled to update marked prices periodically in order to reflect changes in manufacturers' suggested prices.\", 'Retailers are morally obligated to sell an item to a customer at a mismarked price only if that customer was genuinely misled about the intended price by the mismarking.', 'The price displayed on an item in a retail store morally constitutes an irrevocable offer to sell the item at that price.']", "label": 2 }, { "id": "train_2789", "context": "Moralist: A statement is wholly truthful only if it is true and made without intended deception. A statement is a lie if it is intended to deceive or if its speaker, upon learning that the statement was misinterpreted, refrains from clarifying it.", "question": "Which one of the following judgments most closely conforms to the principles stated by the moralist?", "answers": "['Tony was not lying when he told his granddaughter that he did not wear dentures, for even though Tony meant to deceive his granddaughter, she made it clear to Tony that she did not believe him.', 'Walter\\'s claim to a potential employer that he had done volunteer work was a lie. Even though Walter had worked without pay in his father\\'s factory, he used the phrase \"volunteer work\" in an attempt to deceive the interviewer into thinking he had worked for a socially beneficial cause.', \"Ted's statement to the investigator that he had been abducted by extraterrestrial beings was wholly truthful even though no one has ever been abducted by extraterrestrial beings. After all, Ted was not trying to deceive the investigator.\", \"The tour guide intended to deceive the tourists when he told them that the cabin they were looking at was centuries old. Still, his statement about the cabin's age was not a lie, for if he thought that this statement had been misinterpreted, he would have tried to clarify it.\"]", "label": 1 }, { "id": "train_2790", "context": "All Red Sox fans hate the Yankees. All Yankees fans hate the Red Sox. Since Frank hates the Red Sox, he must be a Yankees fan.", "question": "The pattern of reasoning displayed above is most closely paralleled in which one of the following?", "answers": "['Dogs generally have four legs. Since Charlie does not have four legs, he must not be dog.', 'Every man is a mammal. Ttherefore, every mammal is a man.', 'Every redbird eats insects. Every bluebird eats plants. Since this particular bird eats plants, it must be a bluebird.', 'Every redbird sings. Most bluebirds sing. Since this particular bird does not sing, it must not be a bluebird or a redbird.']", "label": 2 }, { "id": "train_2791", "context": "Newspaper report: The government' s health department is unable to explain the recent finding that over time a 10 percent increase in the price of cigarettes caused a 5 percent reduction in the number of smokers. This is a surprising finding because cigarettes are known to be highly addictive and numerous studies have demonstrated that an increase in the price of cigarettes almost never causes people to quit smoking.", "question": "Which one of the following, if true, most helps to explain the finding that the increase in the price of cigarettes reduced the number of smokers?", "answers": "['The extent to which cigarettes are addictive depends on the brain physiology unique to the human species.', 'The 10 percent price increase followed a recent 15 percent price increase.', 'Previous price increases have also, over time, caused a reduction in the number of smokers.', 'As cigarette prices rise, the annual number of deaths of smokers surpasses the annual number of new smokers.']", "label": 3 }, { "id": "train_2792", "context": "Gerald: Unless a consumer secures his or her home wireless Internet service, anyone strolling by is able to access that person' s service with certain laptop computers or smartphones. Such use cannot be considered illegal under current laws: it' s no more like trespassing than is enjoying music playing on someone' s radio as you walk down the street. Kendra: But unlike hearing music while walking by, accessing wireless service requires stopping for a considerable length of time. And that could be considered loitering or even harassment.", "question": "Gerald' s and Kendra's positions indicate that they disagree over whether accessing someone's wireless Internet service while walking down the street", "answers": "['can be considered illegal under current law', 'is like trespassing', 'could be done without intending to do so', 'should be prohibited by law']", "label": 0 }, { "id": "train_2793", "context": "Of the citizens who disapprove of the prime minister' s overall job performance, most disapprove because of the prime minister' s support for increasing the income tax. However, Theresa believes that the income tax should be increased. So Theresa probably approves of the prime minister' s overall job performance.", "question": "Which one of the following arguments exhibits flawed reasoning that is most parallel to that in the argument above?", "answers": "['Of the people who oppose funding a study to determine the feasibility of building a light rail line in the Loffoch Valley, most also believe that the Valley Freeway should be built. Donna opposes increasing funding for a study, so she probably supports building the Valley Freeway.', 'Of the people who support allowing limited logging in the Grizzly National Forest, most support it because they think it will reduce the risk of fire in the forest. Andy thinks that limited logging will not reduce the risk of fire in the forest, so he probably opposes allowing limited logging there.', 'Of the people who believe that there will be a blizzard tomorrow, most believe it because of the weather report on the Channel 9 news. Eduardo believes that there will be a blizzard tomorrow, so he probably saw the weather report on the Channel 9 news.', 'Of the people who expect the population in the area to increase over the next ten years, most think that an expected population increase is a good reason to build a new school. Bonita does not expect the population to increase over the next ten years, so she probably does not favor building a new school.']", "label": 1 }, { "id": "train_2794", "context": "Professor Knight has noticed that students who guess on her multiple-choice tests are significantly more likely to choose answer choice C instead of A, B, D, or E. Basing her judgment on the fact that a variety of students from different social groups and backgrounds all do this, she has concluded that this tendency was acquired from her colleague Professor Leigh' s habit of predominantly using C as a correct answer choice, rather than students' natural tendency to choose the middle letter. Professors Knight and Leigh are the only professors who give multiple choice tests in the required classes of the first and second year.", "question": "Which of the following, if true, would most strengthen the conclusion drawn by Professor Knight?", "answers": "[\"Professor Knight's first-year students do not pick C more than any other letter, but her second-year students, who have all taken the same required classes, do.\", 'She has noticed that students tend to choose C only at her current school, rather than having noticed this tendency throughout her career.', 'The students are required to take the same classes during their first year, but, with the exception of one required lecture, can choose their electives during their second year.', 'Professor Leigh has a similar theory and believes that Professor Knight is the cause.']", "label": 0 }, { "id": "train_2795", "context": "Some species are called \"indicator species\"because the loss of a population of such a species serves as an early warning of problems arising from pollution. Environmentalists tracking the effects of pollution have increasingly paid heed to indicator species; yet environmentalists would be misguided if they attributed the loss of a population to pollution in all cases. Though declines in population often do signal environmental degradation, they are just as often a result of the natural evolution of an ecosystem. We must remember that, in nature, change is the status quo.", "question": "Which one of the following most accurately expresses the argument's conclusion?", "answers": "['The loss of a specific population is often the result of natural changes in an ecosystem and in such cases should not be resisted.', \"Environmentalists' use of indicator species in tracking the effects of pollution is often problematic.\", 'The loss of a specific population should not always be interpreted as a sign of environmental degradation.', \"The loss of a specific population as a result of pollution is simply part of nature's status quo.\"]", "label": 2 }, { "id": "train_2796", "context": "Three large companies and seven small companies currently manufacture a product with potential military applications. If the government regulates the industry, it will institute a single set of manufacturing specifications to which all ten companies will have to adhere. In this case, ttherefore, since none of the seven small companies can afford to convert their production lines to a new set of manufacturing specifications, only the three large companies will be able to remain in business.", "question": "Which of the following is an assumption on which the author's argument relies?", "answers": "['Assembly of the product produced according to government manufacturing specifications would be more complex than current assembly procedures.', 'It would cost more to convert the production lines of the small companies to a new set of manufacturing specifications than it would to convert the production lines of the large companies.', 'None of the seven small companies currently manufactures the product to a set of specifications that would match those the government would institute if the industry were to be regulated.', 'Industry lobbyists will be unable to dissuade the government from regulating the industry.']", "label": 2 }, { "id": "train_2797", "context": "Praveen excels at volleyball. Anyone who plays volleyball daily excels at volleyball. Thus, Praveen plays volleyball daily.", "question": "The reasoning error in the above argument is most similar to that in which one of the following?", "answers": [ "Anyone who is a superb gourmet cooks often. D'Souza cooks often. Thus, D'Souza is a superb gourmet.", "D'Souza is a sculptor. Anyone who is not a sculptor is a painter. Thus, D'Souza is not a painter.", "Anyone who hikes exercises. D'Souza does not exercise. Thus, D'Souza does not hike.", "D'Souza is a jogger. Anyone who dances is not a jogger. Thus, D'Souza is not a dancer." ], "label": 0 }, { "id": "train_2798", "context": "Passenger volume in the airline industry has declined dramatically over the past two years, and thus fewer travelers and fewer planes are using airports. Since airport expansion can be warranted only by increases in air traffic volume, and since it will probably be at least five years before passenger volume returns to and then exceeds its previous level, it seems surprising that now is the time that airports are going ahead with plans to expand their passenger terminal facilities.", "question": "Which one of the following, if true, provides the best reason in favor of the airports' timing of their planned expansions?", "answers": "['When passenger volume begins to grow again after a period of decline, most airlines can, initially, absorb the increase without adding new routes or new planes.', 'A sustained decline in passenger travel could lead to the failure of many airlines and the absorption of their routes by those airlines that survive.', 'It is generally more difficult to finance major construction projects when the economy is in a period of decline.', 'Low volume in passenger air travel permits airport expansion with relatively little inconvenience to the public.']", "label": 3 }, { "id": "train_2799", "context": "Michelangelo' s sixteenth-century Sistine Chapel paintings are currently being restored. A goal of the restorers is to uncover Michelangelo' s original work, and so additions made to Michelangelo' s paintings by later artists are being removed. However, the restorers have decided to make one exception: to leave intact additions that were painted by da Volterra.", "question": "Which one of the following, if true, most helps to reconcile the restorers'decision with the goal stated in the passage?", "answers": "['Michelangelo is famous primarily for his sculptures and only secondarily for his paintings, whereas da Volterra is known exclusively for his paintings.', \"Da Volterra's work is considered by certain art historians to be just as valuable as the work of some of the other artists who painted additions to Michelangelo's work.\", 'The restorers believe that da Volterra stripped away all previous layers of paint before he painted his own additions to the Sistine Chapel.', 'Because da Volterra used a type of pigment that is especially sensitive to light, the additions to the Sistine Chapel that da Volterra painted have relatively muted colors.']", "label": 2 }, { "id": "train_2800", "context": "Some people are Montagues and some people are Capulets. No Montague can be crossed in love. All Capulets can be crossed in love. Ttherefore, Capulets are not Montagues. Anyone who is not a Montague is intemperate. Assume that all of the statements in the passage are true.", "question": "If it is also true that no Montague is intemperate, then which one of the following must be true?", "answers": "['All intemperate people can be crossed in love.', 'The only people who can be crossed in love are intemperate Capulets.', 'All intemperate people are Capulets.', 'All Capulets are intemperate.']", "label": 3 }, { "id": "train_2801", "context": "People with high blood pressure are generally more nervous and anxious than are people who do not have high blood pressure. This fact shows that this particular combination of personality traits -- the so-called hypertensive personality -- is likely to cause a person with these traits to develop high blood pressure.", "question": "The reasoning in the argument is most vulnerable to criticism on the ground that the argument", "answers": "['focuses on nervousness and anxiety only, ignoring other personality traits that people with high blood pressure might have', 'takes a correlation between personality traits and high blood pressure as proof that the traits cause high blood pressure', 'simply restates the claim that there is a \"hypertensive personality\" without providing evidence to support that claim', 'presupposes that people have permanent personality traits']", "label": 1 }, { "id": "train_2802", "context": "Solicitor: Loux named Zembaty executor of her will. Her only beneficiary was her grandson, of whom she was very fond. Prior to distributing the remainder to the beneficiary, Zembaty was legally required to choose which properties in the estate should be sold to clear the estate' s heavy debts. Loux never expressed any particular desire about the Stoke Farm, which includes the only farmland in her estate. Thus, it is unlikely that Loux would have had any objection to Zembaty' s having sold it rather than having transferred it to her grandson.", "question": "Which one of the following, if true, most weakens the solicitor's argument?", "answers": "[\"The Stoke Farm was the main cause of the estate's debts.\", \"The estate's debts could not have been cleared without selling the Stoke Farm.\", 'Loux was well aware of the legal requirements the executor of her will would have to satisfy.', \"Loux's grandson had repeatedly expressed his desire to own a farm.\"]", "label": 3 }, { "id": "train_2803", "context": "Whoever murdered Jansen was undoubtedly in Jansen' s office on the day of the murder, and both Samantha and Herbert were in Jansen' s office on that day. If Herbert had committed the murder, the police would have found either his fingerprints or his footprints at the scene of the crime. But if Samantha was the murderer, she would have avoided leaving behind footprints or fingerprints. The police found fingerprints but no footprints at the scene of the crime. Since the fingerprints were not Herbert' s, he is not the murderer. Thus Samantha must be the killer.", "question": "Which one of the following, if assumed, allows the conclusion that Samantha was the killer to be properly inferred?", "answers": "[\"The fingerprints found at the scene of the crime were not Samantha's.\", \"The fingerprints found at the scene of the crime were not Jansen's.\", \"No one but Herbert and Samantha was in Jansen's office on the day of the murder.\", 'If there had been footprints at the scene of the crime, the police would have found them.']", "label": 2 }, { "id": "train_2804", "context": "Studies show that individuals with a high propensity for taking risks tend to have fewer ethical principles to which they consciously adhere in their business interactions than do most people. On the other hand, individuals with a strong desire to be accepted socially tend to have more such principles than do most people. And, in general, the more ethical principles to which someone consciously adheres, the more ethical is that person' s behavior. Ttherefore, business schools can promote more ethical behavior among future businesspeople by promoting among their students the desire to be accepted socially and discouraging the propensity for taking risks.", "question": "The reasoning in the argument is flawed because the argument", "answers": "['draws a conclusion that simply restates a claim presented in support of that conclusion', 'concludes merely from the fact that two things are correlated that one causes the other', 'infers from the fact that something is usually true that it is always true', 'takes for granted that certain actions are morally wrong simply because most people believe that they are morally wrong']", "label": 1 }, { "id": "train_2805", "context": "The current theory about earthquakes holds that they are caused by adjoining plates of rock sliding past each other; the plates are pressed together until powerful forces overcome the resistance. As plausible as this may sound, at least one thing remains mysterious on this theory. The overcoming of such resistance should create enormous amounts of heat. But so far no increases in temperature unrelated to weather have been detected following earthquakes.", "question": "Which one of the following most accurately expresses the main point of the argument?", "answers": "['No one will ever be sure what the true cause of earthquakes is.', 'The current theory does not fully explain earthquake data.', 'Contrary to the current theory, earthquakes are not caused by adjoining plates of rock sliding past one another.', 'No increases in temperature have been detected following earthquakes.']", "label": 1 }, { "id": "train_2806", "context": "Oscar: I have been accused of plagiarizing the work of Ethel Myers in my recent article. But that accusation is unwarranted. Although I admit I used passages from Myers' book without attribution, Myers gave me permission in private correspondence to do so. Millie: Myers cannot give you permission to plagiarize. Plagiarism is wrong, not only because it violates authors' rights to their own words, but also because it misleads readers: it is fundamentally a type of lie. A lie is no less a lie if another person agrees to the deception.", "question": "Which one of the following principles, if established, would justify Oscar's judgment?", "answers": "['An author is entitled to quote freely without attribution the work of a writer if that writer relinquishes his or her exclusive right to the material.', 'Authors are entitled to quote without attribution passages that they themselves have written and published in other books or articles.', 'The writer of an article must cite the source of all passages that were not written by that writer if those passages are more than a few sentences long.', 'Plagiarism is never justified, but writers are justified in occasionally quoting without attribution the work of other writers if the work quoted has not been published.']", "label": 0 }, { "id": "train_2807", "context": "Nutritionists believe that a person' s daily requirement for vitamins can readily be met by eating five servings of fruits and vegetables daily. However, most people eat far less than this. Thus, most people need to take vitamin pills.", "question": "Which one of the following statements, if true, most seriously weakens the argument?", "answers": "['Fruits and vegetables are also important sources of fiber, in forms not found in vitamin pills.', 'Many commonly consumed foods that are neither fruits nor vegetables are fortified by manufacturers with the vitamins found in fruits and vegetables.', 'Certain commonly available fruits and vegetables contain considerably more nutrients than others.', 'Nutritionists sometimes disagree on how much of a fruit or vegetable constitutes a complete serving.']", "label": 1 }, { "id": "train_2808", "context": "Since 1990 the percentage of bacterial sinus infections in Aqadestan that are resistant to the antibiotic perxicillin has increased substantially. Bacteria can quickly develop resistance to an antibiotic when it is prescribed indiscriminately or when patients fail to take it as prescribed. Since perxicillin has not been indiscriminately prescribed, health officials hypothesize that the increase in perxicillin-resistant sinus infections is largely due to patients' failure to take this medication as prescribed.", "question": "Which of the following, if true of Aqadestan, provides most support for the health officials' hypothesis?", "answers": "['Aqadestani health clinics provide antibiotics to their patients at cost.', 'When it first became available, perxicillin was much more effective in treating bacterial sinus infections than any other antibiotic used for such infections at the time.', 'A large number of Aqadestanis never seek medical help when they have a sinus infection.', 'Many patients who take perxicillin experience severe side effects within the first few days of their prescribed regimen.']", "label": 3 }, { "id": "train_2809", "context": "Columnist: It is impossible for there to be real evidence that lax radiation standards that were once in effect at nuclear reactors actually contributed to the increase in cancer rates near such sites. The point is a familiar one: who can say if a particular case of cancer is due to radiation, exposure to environmental toxins, smoking, poor diet, or genetic factors.", "question": "The argument's reasoning is most vulnerable to criticism on which one of the following grounds?", "answers": "['The argument fails to recognize that there may be convincing statistical evidence even if individual causes cannot be known.', 'The argument inappropriately presupposes that what follows a certain phenomenon was caused by that phenomenon.', 'The argument concludes that a claim about a causal connection is false on the basis of a lack of evidence for the claim.', 'The argument ignores other possible causes of the increase in cancer rates near the nuclear reactor complexes.']", "label": 0 }, { "id": "train_2810", "context": "During the nineteenth century, the French academy of art was a major financial sponsor of painting and sculpture in France; sponsorship by private individuals had decreased dramatically by this time. Because the academy discouraged innovation in the arts, there was little innovation in nineteenth century French sculpture. Yet nineteenth century French painting showed a remarkable degree of innovation.", "question": "Which one of the following, if true, most helps to explain the difference between the amount of innovation in French painting and the amount of innovation in French sculpture during the nineteenth century?", "answers": "['Because stone was so much more expensive than paint and canvas, far more unsponsored paintings were produced than were unsponsored sculptures in France during the nineteenth century.', 'The French academy in the nineteenth century financially supported a greater number of sculptors than painters, but individual painters received more support, on average, than individual sculptors.', 'Very few of the artists in France in the nineteenth century who produced sculptures also produced paintings.', 'In France in the nineteenth century, the French academy gave more of its financial support to painting than it did to sculpture.']", "label": 0 }, { "id": "train_2811", "context": "Manager: I have circulated a posting for the position of Social Scientific Researcher. Applicants must have either an earned doctorate and a track record of published research, or else five years' work experience. The relevant fields for these requirements are sociology, psychology, and education.", "question": "Which one of the applicants, as described below, does NOT meet the manager's requirements?", "answers": "['Alvin Johnson is a doctoral candidate at a local university and is currently working on a dissertation. Prior to undertaking doctoral studies, he worked as a psychology researcher for seven years.', 'Alicia Arias holds an earned doctorate in sociology from a prestigious university and has published one book and fifteen research articles in sociology.', \"Michael Roberts has published two highly regarded books on the problems of urban public schools and has a master's degree in special education. He taught special education classes for two years and then for four years served as a research associate with the Mayor's Task Force on Education.\", \"Edward St. John has worked as a business consultant for the past ten years, during which time he has published six novels. He holds an earned doctorate from one of the nation's foremost business schools.\"]", "label": 3 }, { "id": "train_2812", "context": "Essayist: Common sense, which is always progressing, is nothing but a collection of theories that have been tested over time and found useful. When alternative theories that prove even more useful are developed, they gradually take the place of theories already embodied in common sense. This causes common sense to progress, but, because it absorbs new theories slowly, it always contains some obsolete theories.", "question": "If all of the essayist's statements are true, then which one of the following must be true?", "answers": "['Of the useful theories within the body of common sense, the older ones are generally less useful than the newer ones.', 'Each theory within the body of common sense is eventually replaced with a new theory that is more useful.', 'At least some new theories that have not yet been found to be more useful than any theory currently part of common sense will never be absorbed into the body of common sense.', 'At least some theories that have been tested over time and found useful are less useful than some other theories that have not been fully absorbed into the body of common sense.']", "label": 3 }, { "id": "train_2813", "context": "Because the statement \"all gray rabbits are rabbits\" is true, it follows by analogy that the statement \"all suspected criminals are criminals\" is also true.", "question": "The reasoning above is flawed because it fails to recognize that", "answers": "['the relationship between being a gray rabbit and being a rabbit is not of the same kind as that between being a suspected criminal and being a criminal', 'not all criminals are suspected', 'the relationship between being a criminal and being a rabbit is not of the same kind as that between being suspected and being gray', 'not all rabbits are gray']", "label": 0 }, { "id": "train_2814", "context": "A company has developed a new sensing device that, according to the company' s claims, detects weak, ultralow-frequency electromagnetic signals associated with a beating heart. These signals, which pass through almost any physical obstruction, are purportedly detected by the device even at significant distances. Ttherefore, if the company's claims are true, their device will radically improve emergency teams ability to locate quickly people who are trapped within the wreckage of collapsed buildings.", "question": "Which of the following, if true, most strengthens the argument?", "answers": "['Ultralow-frequency signals are not the only electromagnetic signals that can pass through almost any physical obstruction.', \"Most People who have survived after being trapped in collapsed buildings were rescued within two hours of the building's collapse.\", 'Extensive training is required in order to operate the device effectively.', 'The device gives a distinctive reading when the signals it detects come from human beings rather than from any other living beings.']", "label": 3 }, { "id": "train_2815", "context": "Critic: People today place an especially high value on respect for others; yet, in their comedy acts, many of today' s most popular comedians display blatant disrespect for others. But when people fail to live up to the very ideals they hold in highest esteem, exaggeration of such failings often forms the basis of successful comedy. Thus the current popularity of comedians who display disrespect in their acts is hardly surprising.", "question": "The critic's argument depends on which one of the following assumptions?", "answers": "['People who enjoy comedians who display disrespect in their acts do not place a high value on respect for others.', 'People who value an ideal especially highly do not always succeed in living up to this ideal.', 'Many people disapprove of the portrayal of blatant disrespect for others in comedy acts.', 'People today fail to live up to their own ideals more frequently than was the case in the past.']", "label": 1 }, { "id": "train_2816", "context": "Biologist: Some speculate that the unusually high frequency of small goats found in island populations is a response to evolutionary pressure to increase the number of goats so as to ensure a diverse gene pool. However, only the reproductive success of a trait influences its frequency in a population. So, the only kind of evolutionary pressure that can reduce the average size of the members of a goat population is that resulting from small goats achieving greater reproductive than their larger cousins.", "question": "The biologist's view, if true, provides the most support for which one of the following?", "answers": "['Contrary to what some believe, large goats achieve greater reproductive success than small goats even when space is limited.', 'The evolutionary pressure to ensure a diverse gene pool does not have the effect of increasing the frequency of a gene for small size.', 'The evolutionary pressure to ensure a diverse gene pool could have the effect of increasing the frequency of a gene for small size.', 'A diverse gene pool cannot be achieved in a goat population unless the average size of its members is reduced.']", "label": 1 }, { "id": "train_2817", "context": "The effort involved in lying produces measurable physiological reactions such as a speedup of the heartbeat. Since lying is accompanied by physiological reactions, lie-detector tests that can detect these reactions are a sure way of determining when someone is lying.", "question": "Which one of the following statements, if true, most seriously weakens the argument?", "answers": "['Lie-detector tests can measure only some of the physiological reactions that occur when someone is lying.', 'When employers use lie-detector tests as part of their preemployment screening, some candidates tested are highly motivated to lie.', 'People are often unaware that they are having physiological reactions of the sort measured by lie-detector tests.', 'For people who are not lying, the tension of taking a lie-detector test can produce physiological reactions identical to the ones that accompany the act of lying.']", "label": 3 }, { "id": "train_2818", "context": "Essayist: Human history is filled with the longing for eternal youth and immortality. Yet aging and death are normal and inevitable, indeed even desirable. Just imagine the ethical, social, and economic consequences that would result if it ever became possible to prevent them.", "question": "The claim that human history is filled with the longing for eternal youth and immortality figures in the essayist's argument in which one of the following ways?", "answers": "['It is one of the conclusions of the argument, though it is not the main conclusion.', 'It is a claim that the argument as a whole is directed toward discrediting.', 'It introduces the subject of the argument, but plays no logical role in establishing the main conclusion.', 'It sets out a problem to which the argument as a whole maintains there is no satisfactory response.']", "label": 2 }, { "id": "train_2819", "context": "A natural history museum contains several displays of wild animals. These displays are created by drying and mounting animal skins. In some of the older displays, the animals' skins have started to deteriorate because of low humidity and the heat of the lights. The older displays are lit by tungsten lamps but the newer ones are lit by compact fluorescent lamps designed for use in museums. These lamps give off as much light as the tungsten lamps but less heat.", "question": "The statements above, if true, most strongly support which one of the following?", "answers": "['The humidity in the museum is lower today than it was when the older displays were first put in place.', 'The displays that are lit by many compact fluorescent lamps are more prone to deterioration than the displays that are lit by a few tungsten lamps.', 'Some of the older displays will last longer if the tungsten lamps that illuminate them are replaced by compact fluorescent lamps.', 'More of the displays are lit by compact fluorescent lamps than are lit by tungsten lamps.']", "label": 2 }, { "id": "train_2820", "context": "Modern flamingos derive their pink coloration from pigments stored in tiny shrimp that they filter from shallow, salty waters. The shrimp get this pigment from tiny red algae that they filter through their leg bristles. In the Jurassic period (about 200 million years ago), both algae and shrimp were an excellent source of food for any larger animal equipped to sieve them out of the water through an anatomical strainer. The Argentine pterodactyl possessed a row of thin, bristlelike teeth through which it pumped water, straining out any tiny food particles in the process. Thus, because it was able to filter both algae and shrimp, it is reasonable to conclude that the pterodactyl acquired a pink coloration.", "question": "Which one of the following statements, if true, strengthens the argument for the existence of a pink pterodactyl?", "answers": "[\"The Argentine pterodactyl's habitat included shallow seas that were particularly rich in red algae and shrimp.\", 'If the Argentine pterodactyl did not eat a diet containing red algae, its color was determined by factors other than diet.', 'The Argentine pterodactyl inhabited the shores of shallow freshwater seas in Jurassic South America.', 'There is a specific type of shrimp that does not eat the algae immediately but carries them on its bristles and eats them later.']", "label": 0 }, { "id": "train_2821", "context": "The result of attempting to whip cream in a blender is a thick, velvety substance rather than fluffy whipped cream. This is because a blender' s container does not let in enough air to whip cream effectively. Although using a special whipping-cream attachment in a blender can help somewhat, it cannot fully compensate for the container' s poor air intake.", "question": "If all of the statements above are true, which one of the following must be true?", "answers": "['The use of a special whipping-cream attachment in a blender does not suffice to whip cream completely effectively.', 'The use of a special whipping-cream attachment in a blender can reduce the total amount of air required to whip cream effectively in that blender.', 'Cream that has been whipped ineffectively generally becomes a thick, velvety substance rather than fluffy whipped cream.', 'When attempting to whip cream in a blender, using a special whipping-cream attachment always produces a fluffier result than could be obtained without using such an attachment.']", "label": 0 }, { "id": "train_2822", "context": "It is not known whether bovine spongiform encephalopathy (BSE), a disease of cattle invariably deadly to them, can be transmitted directly from one infected animal to another at all stages of the infection. If it can be, there is now a reservoir of infected cattle incubating the disease. There are no diagnostic tests to identify infected animals before the animals show overt symptoms. Ttherefore, if such direct transmission occurs, the disease cannot be eradicated by __.", "question": "Which one of the following best completes the argument?", "answers": "['destroying all cattle in areas where BSE occurs and raising cattle only in areas to which BSE is known not to have spread', 'developing a drug that kills the agent that causes BSE, and then treating with that drug all cattle that might have the disease', 'removing from the herd and destroying any diseased animal as soon as it shows the typical symptoms of advanced BSE', 'developing a vaccine that confers lifelong immunity against BSE and giving it to all cattle, destroying in due course all those animals for which the vaccine protection came too late']", "label": 2 }, { "id": "train_2823", "context": "Juries in criminal trials do not base verdicts on uncorroborated testimony given by any one witness. Rightly so, because it is usually prudent to be highly skeptical of unsubstantiated claims made by any one person. But then, to be consistent, juries should end an all-too-common practice: convicting defendants on the basis of an uncorroborated full confession.", "question": "Which of the following, if true, most strengthens the argument above?", "answers": "[\"The process of jury selection is designed to screen out people who have a firm opinion about the defendant's guilt in advance of the trial.\", 'Highly suggestible people who are accused of wrongdoing sometimes become so unsure of their own recollection of the past that they can come to accept the accusations made against them.', 'Defendants sometimes make full confessions when they did in fact do what they are accused of doing and have come to believe that the prosecutor has compelling proof of this.', 'Many people believe that juries should not convict defendants who have not made a full confession.']", "label": 1 }, { "id": "train_2824", "context": "To improve the long-term savings rate of the citizens of Levaska, the country' s legislature decided to implement a plan that allows investors to save up to $1, 000 per year in special accounts without paying taxes on the interest earned unless withdrawals are made before the investor reaches age sixty-five. Withdrawals from these accounts prior to age sixty-five would result in the investor' s having to pay taxes on all the accumulated interest at the time of withdrawal.", "question": "Which of the following, if true, most strongly supports the prediction that the legislature's plan will have its intended effect?", "answers": "['A significant number of the citizens of Levaska will invest in the tax-free savings accounts well before they reach the age of sixty-five.', 'The majority of people choosing to take advantage of the tax-free savings accounts will withdraw their money prior to age sixty-five.', 'People who are not citizens of Levaska are not eligible to invest in the tax-free savings accounts, even if their income is taxable in Levaska.', 'The money saved in the tax-free savings accounts will be deposited primarily in those banks and financial institutions that supported the legislation instituting the plan.']", "label": 0 }, { "id": "train_2825", "context": "Bureaucrat: The primary, constant goal of an ideal bureaucracy is to define and classify all possible problems and set out regulations regarding each eventuality. Also, an ideal bureaucracy provides an appeal procedure for any complaint. If a complaint reveals an unanticipated problem, the regulations are expanded to cover the new issue, and for this reason an ideal bureaucracy will have an everexpanding system of regulations.", "question": "Which one of the following is an assumption the bureaucrat's argument requires?", "answers": "['Any complaint that an ideal bureaucracy receives will reveal an unanticipated problem that the bureaucracy is capable of defining and classifying.', 'An ideal bureaucracy will provide an appeal procedure for complaints even after it has defined and classified all possible problems and set out regulations regarding each eventuality.', 'For each problem that an ideal bureaucracy has defined and classified, the bureaucracy has received at least one complaint revealing that problem.', \"An ideal bureaucracy will never be permanently without complaints about problems that are not covered by that bureaucracy's regulations.\"]", "label": 3 }, { "id": "train_2826", "context": "Sharita: Anyone who owns a cat should have it spayed or neutered unless they are willing to take care of the cat' s offspring. It is because people fail to do this that there are so many stray cats around. Chad: Stray cats are not only a nuisance, they spread diseases and cause injuries to other cats and to humans. People feed these animals out of kindness, but doing so only exacerbates the problem unless the cats are then captured and adopted.", "question": "Sharita's and Chad's statements provide the most support for the claim that they agree about which one of the following?", "answers": "['It is mainly out of kindness that people feed stray cats.', 'Stray cats spread diseases to other cats.', 'Stray cats are a problem because of the risk they pose to humans.', 'There are more stray cats than there should be.']", "label": 3 }, { "id": "train_2827", "context": "Scientists have determined that an effective way to lower cholesterol is to eat three servings of whole grains every day. Studies have shown that the cholesterol levels of people who did so were significantly lower after six months than were those of people who did not, even though the cholesterol levels of the two groups had been the same before the studies began. Clearly, eating whole grains can have an appreciable effect on cholesterol levels.", "question": "The answer to which of the following questions, if true, would be most useful in evaluating the claim about whole grains above?", "answers": "['Were the two groups of people in the study involved in the same exercise program?', 'Can the same drop in cholesterol be achieved through medication?', 'Are most consumers aware of the different between whole grains and processed grains?', 'Did the study continue to track the subjects beyond six months?']", "label": 0 }, { "id": "train_2828", "context": "In many scientific disciplines, scientists generally do not do highly creative work beyond the age of forty, a tendency that has normally been taken to show that aging carries with it a loss of creative capacity. However, by the age of forty most scientists have been working in their chosen field for at least fifteen years, so an alternative explanation is that spending too long in a single field reduces the opportunity for creative thought.", "question": "Investigating which of the following would be most useful in choosing between the competing explanations described above?", "answers": "['Whether funding agencies are more inclined to award research grants to scientists who are veterans in their field than to scientists who are relative newcomers', \"Whether scientists' choice of research projects tends to be influenced by their own belief that their most creative work will be done relatively early in their career\", 'Whether among those scientists who do highly creative work beyond age forty a large proportion entered their field at a considerably later age than is common', 'Whether there is significant variation among scientific fields in the average age at which scientists working in those fields are at their most productive']", "label": 2 }, { "id": "train_2829", "context": "Agricultural economist: Over the past several years, increases in worldwide grain production have virtually ceased. Further increases will be extremely difficult; most usable farmland is already being farmed with near-maximal efficiency. But worldwide demand for grain has been increasing steadily, due largely to continuing population growth. Hence, a severe worldwide grain shortage is likely.", "question": "Which one of the following most accurately describes the role played in the agricultural economist's argument by the claim that further increases in worldwide grain production will be extremely difficult?", "answers": "['It is a description of a phenomenon, a causal explanation of which is the main conclusion of the argument.', 'It is one of the two conclusions drawn by the agricultural economist, neither of which is used to provide support for the other.', \"It is a prediction for which the agricultural economist's first claim is offered as the primary justification.\", \"It is an intermediate conclusion that is presented as evidence for the argument's main conclusion.\"]", "label": 3 }, { "id": "train_2830", "context": "Some of the most prosperous nations in the world have experienced a pronounced drop in national savings rates -- the percentage of after-tax income an average household saves. This trend will undoubtedly continue if the average age of these nations' populations continues to rise, since older people have fewer reasons to save than do younger people.", "question": "Which one of the following indicates an error in the reasoning leading to the prediction above?", "answers": "['It only takes into account the comparative number of reasons older and younger people, respectively, have for saving, and not the comparative strength of those reasons.', 'It fails to cite statistics showing that the average age of the population of certain nations is rising.', 'It uses after-tax income as the base for computing the national savings rate without establishing by argument that after-tax income is a more appropriate base than before-tax income.', \"It assumes that a negative savings rate -- the result of the average household's spending all of its after-tax income as well as some of its existing savings -- cannot ever come about in any nation.\"]", "label": 0 }, { "id": "train_2831", "context": "If the city council maintains spending at the same level as this year' s, it can be expected to levy a sales tax of 2 percent next year. Thus, if the council levies a higher tax, it will be because the council is increasing its expenditures.", "question": "Which one of the following exhibits a pattern of reasoning most closely similar to that of the argument above?", "answers": "[\"If the companies in the state do not increase their workers'wages this year, the prices they charge for their goods can be expected to be much the same as they were last year. Thus, if the companies do increase prices, it will be because they have increased wages.\", 'If newspaper publishers wish to publish good papers, they should employ good journalists. Thus, if they employ poor journalists, it will not be surprising if their circulation falls as a result.', 'If shops wish to reduce shoplifting, they should employ more store detectives. Thus, if shops do not, they will suffer reduced profits because of their losses from stolen goods.', 'If house-building costs are not now rising, builders cannot be expected to increase the prices of houses. Thus, if they decrease the prices of houses, it will be because that action will enable them to sell a greater number of houses.']", "label": 0 }, { "id": "train_2832", "context": "NowNews, although still the most popular magazine covering cultural events in Kalopolis, has recently suffered a significant drop in advertising revenue because of falling circulation. Many readers have begun buying a competing magazine that, at 50 cents per copy, costs less than NowNews at $1. 50 per copy. In order to boost circulation and thus increase advertising revenue, NowNews's publisher has proposed making it available at no charge, but this proposal has a serious drawback, since __.", "question": "Which of the following most logically completes the argument below?", "answers": "[\"NowNews's competitor would begin to lose large amounts of money if it were forced to lower its cover price.\", 'Those Kalopolis residents with the greatest interest in cultural events are regular readers of both magazines.', \"One reason NowNews's circulation fell was that its competitor's reporting on cultural events was superior.\", 'The newsstands and stores that currently sell NowNews will no longer carry it if it is being given away for free.']", "label": 3 }, { "id": "train_2833", "context": "Politician: Over the next decade, our city will be replacing all of its street signs with signs that are designed for improved readability. But since no one is complaining about the current signs, installing the new ones is a colossal waste of time and money.", "question": "Which one of the following would be most useful to know in evaluating the politician's argument?", "answers": "['What percentage of its street signs does the city replace annually in the course of ordinary maintenance?', 'What features of the new street signs improve the readability of the signs?', 'Do any other cities plan to replace their street signs with signs designed for improved readability?', 'Are the new street signs considerably more expensive to manufacture than the current street signs were?']", "label": 0 }, { "id": "train_2834", "context": "Northern Air has dozens of flights daily into and out of Belleville Airport, which is highly congested. NorthernAir depends for its success on economy and quick turnaround and consequently is planning to replace its large planes with Skybuses, whose novel aerodynamic design is extremely fuel efficient. The Skybus' fuel efficiency results in both lower fuel costs and reduced time spent refueling.", "question": "Which of the following, if true, could present the most serious disadvantage for Northern Air in replacing their large planes with Skybuses?", "answers": "['Aviation fuel is projected to decline in price over the next several years.', 'The Skybus would enable Northern Air to schedule direct flights to destinations that currently require stops for refueling.', 'The aerodynamic design of the Skybus causes turbulence behind it when taking off that forces other planes on the runway to delay their takeoffs.', \"None of Northern Air's competitors that use Belleville Airport are considering buying Skybuses.\"]", "label": 2 }, { "id": "train_2835", "context": "Leslie: I enjoy P' s movies, but he should not be deemed a fabulous actor. Despite how memorable his performances have been, the plots in his movies have not been diverse enough to show a real range of talent. Molly: You are incorrect in your assessment of what makes a fabulous actor. A fabulous actor does not have to appear in movies with greatly varied plots. But, a fabulous actor must, at a minimum, be able to master a specific genre of film.", "question": "Which of the following is the disagreement between Leslie and Molly?", "answers": "['whether appearing in movies with diverse plots is necessary to be a fabulous actor', 'whether there are other prerequisites to becoming a fabulous actor besides giving memorable performances and appearing in movies with greatly varied plots', 'whether all actors who give memorable performances are fabulous actors', 'whether P has appeared in movies with greatly varied plots']", "label": 0 }, { "id": "train_2836", "context": "Tiger beetles are such fast runners that they can capture virtually any nonflying insect. However, when running toward an insect, the beetles intermittently stop, and then, a moment later, resume their attack. Perhaps they cannot maintain their pace and must pause for a moment' s rest; but an alternative hypothesis is that while running tiger beetles are unable to process the resulting rapidly changing visual information, and so quickly go blind and stop.", "question": "Which of the following, if discovered in experiments using artificially moved prey insects, would support one of the two hypotheses and undermine the other?", "answers": "[\"In pursuing a moving insect, the beetles usually respond immediately to changes in the insect's direction, and pause equally frequently whether the chase is up or down an incline.\", 'The beetles maintain a fixed time interval between pauses, although when an insect that had been stationary begins to flee, the beetle increases its speed after its next pause.', 'When an obstacle is suddenly introduced just in front of running beetles, the beetles sometimes stop immediately, but they never respond by running around the barrier.', 'If, when a beetle pauses, it has not gained on the insect it is pursuing, the beetle generally ends its pursuit.']", "label": 1 }, { "id": "train_2837", "context": "If a meat thermometer is poked into a steak, the steak' s temperature can be correctly assessed. Ttherefore, since Will does not have access to a meat thermometer, his steak' s temperature cannot be correctly assessed. If the steak' s temperature cannot be correctly assessed, that steak will be over- or undercooked. Ttherefore, Will' s steak will be over- or undercooked.", "question": "Which of the following arguments contains an error of reasoning like the one in the argument above?", "answers": "['If dogs are not walked frequently, the dog will not be healthy. Ttherefore, if Lee does not walk his dog frequently, his dog will not be healthy.', 'If dogs are not walked frequently, sometimes they will be unhealthy. Unhealthiness can make a dog unhappy. Ttherefore, since Nina walks her dog frequently, her dog will be happy.', \"If dogs are walked frequently, they will get plenty of exercise. Ttherefore, since Jackie does not walk her dog frequently, her dog will not get plenty of exercise. Dogs that do not get plenty of exercise will be unhealthy. Ttherefore, Jackie's dog will be unhealthy.\", 'If a dog is walked frequently, the dog will be healthy. Without health, a dog cannot be happy. Since Roger walks his dog frequently, his dog will be both healthy and happy.']", "label": 2 }, { "id": "train_2838", "context": "All cattle ranchers dislike long winters. All ski resort owners like long winters because long winters mean increased profits. Some lawyers are cattle ranchers.", "question": "Which one of the following statements, if true and added to those above, most supports the conclusion that no ski resort owners are lawyers?", "answers": "['All people with increasing profits own ski resorts.', 'Some people who dislike long winters are not cattle ranchers.', 'All lawyers are cattle ranchers.', 'All people who dislike long winters are cattle ranchers.']", "label": 2 }, { "id": "train_2839", "context": "All bridges built from 1950 to 1960 are in serious need of rehabilitation. Some bridges constructed in this period, however, were built according to faulty engineering design. That is the bad news. The good news is that at least some bridges in serious need of rehabilitation are not suspension bridges, since no suspension bridges are among the bridges that were built according to faulty engineering design.", "question": "If the statements above are true, then, on the basis of those statements, which one of the following must also be true?", "answers": "['Some bridges that were built according to faulty engineering design are not bridges other than suspension bridges.', 'Some bridges that were built according to faulty engineering design are in serious need of rehabilitation.', 'Some suspension bridges are in serious need of rehabilitation.', 'Some bridges built from 1950 to 1960 are not in serious need of rehabilitation.']", "label": 1 }, { "id": "train_2840", "context": "Criminals often have an unusual self-image. Embezzlers often think of their actions as \"only borrowing money. \"Many people convicted of violent crimes rationalize their actions by some sort of denial; either the victim \"deserved it\"and so the action was justified, or \"it simply wasn't my fault. \" Thus, in many cases, by criminals' characterization of their situations, __.", "question": "Which one of the following most logically completes the passage?", "answers": "['their actions are not truly criminal', 'they are perceived to be the victim of some other criminal', 'they deserve only a light sentence for their crimes', 'they ought to be rewarded for their actions']", "label": 0 }, { "id": "train_2841", "context": "If this parking policy is unpopular with the faculty, then we should modify it. If it is unpopular among students, we should adopt a new policy. And, it is bound to be unpopular either with the faculty or among students.", "question": "If the statements above are true, which one of the following must also be true?", "answers": "['We should attempt to popularize this parking policy among either the faculty or students.', 'If the parking policy is popular among students, then we should adopt a new policy.', 'We should modify this parking policy if modification will not reduce its popularity with the faculty.', 'If this parking policy is popular with the faculty, then we should adopt a new policy.']", "label": 3 }, { "id": "train_2842", "context": "The cafeteria of a certain elementary school provides a free lunch every day to the entire student body - a significant expense in the school' s annual budget. A vice principal was exploring cost-reducing measures that would not compromise nourishment. A local Italian restaurant offered the school a large number of pizzas at a vastly reduced bulk rate. The pizzas were served one day, and afterward, the vice principal calculated that the cost of the pizza and delivery was slightly more than the average cost of typical cafeteria lunch day. Ttherefore, he concluded that the school could not save money by replacing a day of cafeteria lunch with a day of pizza from this restaurant.", "question": "Which of the following would be most useful for evaluating the conclusion drawn by the vice principal?", "answers": "['Whether other, more expensive dishes from that Italian restaurant, would provide a more balanced and varied set of nutrients.', 'Whether another Italian restaurant in this town would provide a similar deal and offer free delivery.', 'Whether pizza provides the same level of nutrition as a typical cafeteria lunch.', \"How many pizzas were uneaten at the end of that day's lunch.\"]", "label": 3 }, { "id": "train_2843", "context": "From 1996 to 2004, the average family income in a certain country decreased by 10 percent, after adjustments for inflation. Opponents of the political party that ruled during this time claim that this was due to mismanagement of the economy by that party.", "question": "Each of the following rejoinders, if true, directly counters the opponents' explanation of the decrease in average family income EXCEPT:", "answers": "['There had been a rise in family income in 1996, after adjustments for inflation.', 'Younger wage earners usually earn less than older ones, and the average age of household wage earners fell during most years in the past several decades.', \"During the period, international events beyond the control of the country's government had a negative effect on family incomes in the country.\", 'The biggest decreases in family income resulted from policies enacted before the ruling party came to power in 1996.']", "label": 0 }, { "id": "train_2844", "context": "Archaeologists are currently analyzing plant remains found at a site that was last occupied more than 10, 000 years ago. If the plants were cultivated, then the people who occupied the site discovered agriculture thousands of years before any other people are known to have done so. On the other hand, if the plants were wild -- that is, uncultivated -- then the people who occupied the site ate a wider variety of wild plants than did any other people at the time.", "question": "The statements above, if true, most strongly support which one of the following?", "answers": "['The people who occupied the site used some plants in ways that no other people did at that time.', 'It is more likely that the people who occupied the site discovered agriculture thousands of years before people anywhere else did than it is that they ate a wider variety of wild plants than any other people at the time.', 'If the people who occupied the site had reached a more advanced stage in the use of wild plants than any other people at the time, then the plants found at the site were uncultivated.', 'The archaeologists analyzing the plant remains at the site will be able to determine whether the plants were cultivated or were wild.']", "label": 0 }, { "id": "train_2845", "context": "Thomas: The club president had no right to disallow Jeffrey' s vote. Club rules say that only members in good standing may vote. You' ve admitted that club rules also say that all members whose dues are fully paid are members in good standing. And since, as the records indicate, Jeffrey has always paid his dues on time, clearly the president acted in violation of club rules. Althea: By that reasoning my two-year-old niece can legally vote in next month' s national election since she is a citizen of this country, and only citizens can legally vote in national elections.", "question": "The reasoning in Thomas' argument is flawed because his argument", "answers": "[\"overlooks the possibility that Althea is not an authority on the club's rules\", 'offers evidence that casts doubt on the character of the club president and thereby ignores the question of voting eligibility', 'wrongly assumes that if a statement is not actually denied by someone, that statement must be regarded as true', 'fails to take into account the distinction between something not being prohibited and is being authorized']", "label": 3 }, { "id": "train_2846", "context": "It is widely assumed that people need to engage in intellectual activities such as solving crossword puzzles or mathematics problems in order to maintain mental sharpness as they age. In fact, however, simply talking to other people--that is, participating in social interaction, which engages many mental and perceptual skills--suffices. Evidence to this effect comes from a study showing that the more social contact people report, the better their mental skills.", "question": "Which of the following, if true, most seriously weakens the force of the evidence cited?", "answers": "[\"Many medical conditions and treatments that adversely affect a person's mental sharpness also tend to increase that person's social isolation.\", 'The study did not itself collect data but analyzed data bearing on the issue from prior studies.', 'As people grow older, they are often advised to keep exercising their physical and mental capacities in order to maintain or improve them.', 'The tasks evaluating mental sharpness for which data were compiled by the study were more akin to mathematics problems than to conversation.']", "label": 0 }, { "id": "train_2847", "context": "A scholar discovered an unlabeled nineteenth-century recording of someone reciting a poem written by Walt Whitman. During the nineteenth century, recordings of poetry were not made for wide commercial sale. Rather, they were made either as rare private souvenirs of the voices of famous poets or else as publicity stunts, in which actors recorded poems that were familiar to the public. Since the Whitman poem in the recording was never even published, it is likely that the voice in the recording is actually Whitman' s.", "question": "The argument proceeds by", "answers": "['explaining the historical context of an incident in order to demonstrate that each of the two possible scenarios involving that incident is as likely as the other', 'distinguishing a phenomenon into two subtypes and then for a particular case eliminating one of those subtypes', 'showing that two apparently mutually exclusive alternatives are actually compatible with one another', 'offering a general principle and then demonstrating that the general principle is violated in a particular case']", "label": 1 }, { "id": "train_2848", "context": "Electric bug zappers, which work by attracting insects to light, are a very effective means of ridding an area of flying insects. Despite this, most pest control experts now advise against their use, recommending instead such remedies as insect-eating birds or insecticide sprays.", "question": "Which one of the following, if true, most helps to account for the pest control experts' recommendation?", "answers": "['Bug zappers use more electricity but provide less light than do most standard outdoor light sources.', 'Insect-eating birds will take up residence in any insect-rich area if they are provided with nesting boxes, food, and water.', 'Bug zappers kill many more beneficial insects and fewer harmful insects than do insect-eating birds and insecticide sprays.', 'Developers of certain new insecticide sprays claim that their products contain no chemicals that are harmful to humans, birds, or pets.']", "label": 2 }, { "id": "train_2849", "context": "Ethicist: A person who treats others well is more worthy of praise if this treatment is at least partially motivated by feelings of compassion than if it is entirely motivated by cold and dispassionate concern for moral obligation. This is so despite the fact that a person can choose to do what is morally right but cannot choose to have feelings.", "question": "If the ethicist's statements are true, then each of the following could be true EXCEPT:", "answers": "['If someone wants to have compassion toward others but does not, that person is worthy of praise.', 'Someone who acts without feelings of compassion toward those affected by the actions is worthy of praise if those actions enhance the welfare of the people affected.', \"Only what is subject to a person's choice should be used in measuring the praiseworthiness of that person.\", \"If a person feels compassion toward the people affected by that person's actions, yet these actions diminish the welfare of those people, that person does not deserve praise.\"]", "label": 2 }, { "id": "train_2850", "context": "Everyone likes repertory theater. Actors like it because playing different roles each night decreases their level of boredom. Stagehands like it because changing sets every night means more overtime and, thus, higher pay. Theater managers like it because, if plays that reflect audience demand are chosen for production, most performances generate large revenues. It is evident, ttherefore, that more theaters should change to repertory.", "question": "The argument above would be strengthened if which one of the following were true?", "answers": "['In a repertory theater, storage space for sets for more than one production must be available.', 'In a repertory theater, a large capital outlay is required at the beginning of each season.', 'In a repertory theater, plays can be rescheduled to meet audience demand.', 'In a repertory theater, some actors who change roles from night to night find it difficult to master all of the roles they play.']", "label": 2 }, { "id": "train_2851", "context": "Recently discovered bird fossils are about 20 million years older than the fossils of the birdlike dinosaurs from which the birds are generally claimed to have descended. So these newly discovered fossils show, contrary to the account espoused by most paleontologists, that no bird descended from any dinosaur.", "question": "The reasoning in the argument is flawed in that the argument", "answers": "['draws a generalization that is broader than is warranted by the findings cited', 'ignores the possibility that dinosaurs and birds descended from a common ancestor', 'fails to consider the possibility that dinosaurs descended from birds', 'attacks the adherents of the opposing view personally instead of addressing any reason for their view']", "label": 0 }, { "id": "train_2852", "context": "An art critic, by ridiculing an artwork, can undermine the pleasure one takes in it; conversely, by lavishing praise upon an artwork, an art critic can render the experience of viewing the artwork more pleasurable. So an artwork' s artistic merit can depend not only on the person who creates it but also on those who critically evaluate it.", "question": "The conclusion can be properly drawn if which one of the following is assumed?", "answers": "['Most people seek out critical reviews of particular artworks before viewing those works.', 'Most people lack the confidence necessary for making their own evaluations of art.', 'The merit of an artistic work is determined by the amount of pleasure it elicits.', 'The pleasure people take in something is typically influenced by what they think others feel about it.']", "label": 2 }, { "id": "train_2853", "context": "Traces of cultivated emmer wheat have been found among the earliest agricultural remains of many archaeological sites in Europe and Asia. The only place where the wild form of emmer wheat has been found growing is a relatively narrow strip of southwest Asia. Since the oldest remains of cultivated emmer wheat yet found are from village sites in the same narrow strip, it is clear that emmer wheat was first domesticated somewhere in that strip.", "question": "Which of the following, if true, most strengthens the argument?", "answers": "['It is very difficult, without genetic testing, to differentiate the wild form of emmer wheat from a closely related wild wheat that also grows in southwest Asia.', 'Modern experiments show that wild emmer wheat can easily be domesticated so as to yield nearly as well as traditionally domestic strains.', 'In the region containing the strip where wild emmer wheat has been found, climatic conditions have changed very little since before the development of agriculture.', 'The present-day distribution of another wild wheat, einkorn, which was also domesticated early in the development of agriculture, covers a much larger area of southwest Asia.']", "label": 2 }, { "id": "train_2854", "context": "We should do what will make others more virtuous and not do what will make others less virtuous. It is an irony of human existence that praise makes those who are less virtuous more virtuous, while it makes those who are more virtuous less virtuous. And, of course, none except the more virtuous deserve praise.", "question": "From the statements above, if true, which one of the following can be properly inferred?", "answers": "['We should withhold praise from those who deserve it least.', 'We should withhold praise from everyone, regardless of whether or not they deserve it.', 'We should praise those who do not deserve it and withhold praise from those who deserve it.', 'We should praise everyone, regardless of whether or not they deserve it.']", "label": 2 }, { "id": "train_2855", "context": "We should accept the proposal to demolish the old train station, because the local historical society, which vehemently opposes this, is dominated by people who have no commitment to long-term economic well-being. Preserving old buildings creates an impediment to new development, which is critical to economic health.", "question": "The flawed reasoning exhibited by the argument above is most similar to that exhibited by which one of the following arguments?", "answers": "['You should have your hair cut no more than once a month. After all, beauticians suggest that their customers have their hair cut twice a month, and they do this as a way of generating more business for themselves.', 'One should not borrow even small amounts of money unless it is absolutely necessary. Once one borrows a few dollars, the interest starts to accumulate. The longer one takes to repay, the more one ends up owing, and eventually a small debt has become a large one.', 'The committee should endorse the plan to postpone construction of the new expressway. Many residents of the neighborhoods that would be affected are fervently opposed to that construction, and the committee is obligated to avoid alienating those residents.', 'Documents of importance to local heritage should be properly preserved and archived for the sake of future generations. For, if even one of these documents is damaged or lost, the integrity of the historical record as a whole will be damaged.']", "label": 0 }, { "id": "train_2856", "context": "The government' s proposed 8 percent cut in all subsidies to arts groups will be difficult for those groups to absorb. As can be seen, however, from their response to last year' s cut, it will not put them out of existence. Last year there was also an 8 percent cut, and though private fund-raising was very difficult for the arts groups in the current recessionary economy, they did survive.", "question": "The reasoning in the argument is flawed because the argument", "answers": "['overlooks the possibility that the cumulative effect of the cuts will be more than the arts groups can withstand', 'equates the mere survival of the arts groups with their flourishing', 'relies without warrant on the probability that the economy will improve', \"does not take into account that the dollar amount of the proposed cut is lower than the dollar amount of last year's cut\"]", "label": 0 }, { "id": "train_2857", "context": "Suncorp, a new corporation with limited funds, has been clearing large sections of the tropical Amazon forest for cattle ranching. This practice continues even though greater profits can be made from rubber tapping, which does not destroy the forest, than from cattle ranching, which does destroy the forest.", "question": "Which of the following, if true, most helps to explain why Suncorp has been pursuing the less profitable of the two economic activities mentioned above?", "answers": "['In certain districts, profits made from cattle ranching are more heavily taxed than profits made from any other industry.', 'Some of the cattle that are raised on land cleared in the Amazon are killed by wildcats.', 'The amount of money required to begin a rubber-tapping operation is twice as high as the amount needed to begin a cattle ranch.', 'Cattle-ranching operations that are located in tropical climates are more profitable than cattle-ranching operations that are located in cold-weather climates.']", "label": 2 }, { "id": "train_2858", "context": "Durham: The mayor will agree to a tax increase because that is the only way the city council will agree to her road repair proposal, and that proposal is her top priority. Espinoza: The mayor will not get her road repair proposal passed because it is more important to her that taxes not increase.", "question": "The dialogue provides the most support for the claim that Durham and Espinoza agree about which one of the following?", "answers": "['The mayor will agree to a tax increase.', \"The only way that the city council will agree to pass the mayor's road repair proposal is if she agrees to a tax increase.\", 'It is more important to the mayor that taxes not increase than it is that her road repair proposal passes.', 'The mayor will not get her road repair proposal passed.']", "label": 1 }, { "id": "train_2859", "context": "Most apartments on the upper floors of The Vista Arms apartment building have scenic views. So there is in the building at least one studio apartment with scenic views.", "question": "The conclusion of the argument follows logically if which one of the following is assumed?", "answers": "['Most of the apartments in the building are studio apartments.', 'Most of the apartments with scenic views are on the upper floors of the building.', 'All of the apartments on the lower floors of the building have scenic views.', 'Most of the apartments on the upper floors of the building are studio apartments.']", "label": 3 }, { "id": "train_2860", "context": "Dr. Libokov: Certain islands near New Zealand are home to the tuatara, reptiles that are the sole surviving members of the sphenodontidans. Sphenodontidans were plentiful throughout the world during the age of the dinosaurs. But the survival of sphenodontidans near New Zealand, and their total disappearance elsewhere, is no mystery. New Zealand and nearby islands have no native land mammals. Land mammals, plentiful elsewhere, undoubtedly became major predators of sphenodontidans and their eggs, leading to their extinction. Dr. Santos: In fact, the tuatara thrive only on a few islands near New Zealand. On all those where land mammals, such as rats, dogs, or cats, have been introduced in recent years, the tuatara are now extinct or nearly so.", "question": "Which one of the following most accurately characterizes Dr. Santos' response to the hypothesis advanced by Dr. Libokov?", "answers": [ "It contradicts one of Dr. Libokov's assertions.", "It identifies a flaw in Dr. Libokov's reasoning.", "It restates Dr. Libokov's major hypothesis and thus adds nothing to it.", "It provides additional evidence in support of Dr. Libokov's hypothesis." ], "label": 3 }, { "id": "train_2861", "context": "In an experiment, some volunteers were assigned to take aerobics classes and others to take weight-training classes. After three months, each performed an arduous mathematical calculation. Just after that challenge, the measurable stress symptoms of the volunteers in the aerobics classes were less than those of the volunteers in the weight-training classes. This provides good evidence that aerobic exercise helps the body handle psychological stress.", "question": "Which one of the following is an assumption the argument requires?", "answers": "['The volunteers who were assigned to the aerobics classes did not also lift weights outside the classes.', 'Three months is enough time for the body to fully benefit from aerobic exercise.', 'On average, the volunteers who were assigned to the aerobics classes got more exercise in the months in which they took those classes than they had been getting before beginning the experiment.', 'On average, the volunteers assigned to the aerobics classes got a greater amount of aerobic exercise overall during the experiment, including any exercise outside the classes, than did the volunteers assigned to the weight-training classes.']", "label": 3 }, { "id": "train_2862", "context": "Political Advertisement: Mayor Delmont' s critics complain about the jobs that were lost in the city under Delmont' s leadership. Yet the fact is that not only were more jobs created than were eliminated, but the average pay for these new jobs has been higher than the average pay for jobs citywide every year since Delmont took office. So there can be no question that throughout Delmont' s tenure the average paycheck in this city has been getting steadily bigger.", "question": "Which of the following, if true, most strengthens the argument in the advertisement?", "answers": "[\"The average pay for jobs created in the city during the past three years was higher than the average pay for jobs created in the city earlier in Mayor Delmont's tenure.\", 'The average pay for jobs in the city is currently higher than it is for jobs in the suburbs surrounding the city.', \"The average pay for jobs eliminated in the city during Mayor Delmont's tenure has been roughly equal every year to the average pay for jobs citywide.\", 'Average pay in the city was at a ten-year low when Mayor Delmont took office.']", "label": 2 }, { "id": "train_2863", "context": "Armstrong: For the treatment of a particular disease, Dr. Sullivan argues for using nutritional supplements rather than the pharmaceuticals that most doctors prescribe. But this is in his self-interest since he is paid to endorse a line of nutritional supplements. Thus, we should not use nutritional supplements in treating the disease.", "question": "Armstrong's argument is flawed in that it", "answers": "['relies solely on an appeal to an authority whose trustworthiness should not necessarily be taken for granted', \"appeals to people's emotions regarding the treatment of disease rather than to the efficacy of the two approaches to treatment\", \"criticizes Dr. Sullivan's motives for holding a position rather than addressing the position itself\", 'fails to justify its presumption that nutritional supplements cannot be used in conjunction with other treatments']", "label": 2 }, { "id": "train_2864", "context": "Angela' s Hair Salon no longer takes reservations. Foot traffic already draws enough customers into the business. The Hair Salon' s success is solely due to their terrific location.", "question": "Which one of the following, if true, most seriously weakens the argument?", "answers": "['Foot traffic is greater across the street.', \"Angela's Hair Salon has an enormous sign that is visible for miles down the road.\", 'All of the nearby businesses are not profitable.', \"Angela's Hair Salon employs the town's best hairdresser whose skills are well known in the community.\"]", "label": 3 }, { "id": "train_2865", "context": "Futurist: Artists in the next century will be supported largely by private patrons. Because these patrons will almost invariably be supporters of the social order -- whatever it happens to be at the time -- art in the next century will rarely express social and political doctrines that are perceived to be subversive of that social order.", "question": "Which one of the following principles, if valid, provides the most support for the futurist's inference?", "answers": "['Artists tend to become more critical of contemporary social and political arrangements after they are freed of their dependency on private patrons.', 'Art patrons tend not to support artists whose art expresses social and political views that are in opposition to their own.', 'Art patrons tend to oppose all social change except that initiated by artists.', 'Artists are as prone to attack the contemporary social and political order in their work as they are to defend it.']", "label": 1 }, { "id": "train_2866", "context": "People who browse the web for medical information often cannot discriminate between scientifically valid information and quackery. Much of the quackery is particularly appealing to readers with no medical background because it is usually written more clearly than scientific papers. Thus, people who rely on the web when attempting to diagnose their medical conditions are likely to do themselves more harm than good.", "question": "Which one of the following is an assumption the argument requires?", "answers": "['People who browse the web for medical information typically do so in an attempt to diagnose their medical conditions.', 'Many people who browse the web assume that information is not scientifically valid unless it is clearly written.', 'People who attempt to diagnose their medical conditions are likely to do themselves more harm than good unless they rely exclusively on scientifically valid information.', 'People attempting to diagnose their medical conditions will do themselves more harm than good only if they rely on quackery instead of scientifically valid information.']", "label": 2 }, { "id": "train_2867", "context": "Each of the many people who participated in the town' s annual spring cleanup received a community recognition certificate. Because the spring cleanup took place at the same time as the downtown arts fair, we know that there are at least some spring cleanup participants who are not active in the town' s artistic circles.", "question": "If the statements above are true, which one of the following must be true?", "answers": "[\"Some of the persons who are active in the town's artistic circles received community recognition certificates.\", 'No person who received a community recognition certificate has not participated in the spring cleanup.', \"Not all of those who received community recognition certificates are active in the town's artistic circles.\", 'No participants in the downtown arts fair received community recognition certificates.']", "label": 2 }, { "id": "train_2868", "context": "Some of the politicians who strongly supported free trade among Canada, the United States, and Mexico are now refusing to support publicly the idea that free trade should be extended to other Latin American countries.", "question": "If the statement above is true, which one of the following must also be true?", "answers": "['Not all politicians who now publicly support extending free trade to other Latin American countries strongly supported free trade among Canada, the United States, and Mexico.', 'Some of the politicians who strongly supported free trade among Canada, the United States, and Mexico now publicly oppose extending free trade to other Latin American countries.', 'Some of the politicians who now publicly support extending free trade to other Latin American countries did not support free trade among Canada, the United States, and Mexico.', 'Not all politicians who strongly supported free trade among Canada, the United States, and Mexico now publicly support extending free trade to other Latin American countries.']", "label": 3 }, { "id": "train_2869", "context": "Jean: Our navigational equipment sells for $1, 100 and dominates the high end of the market, but more units are sold by our competitors in the $700 to $800 range. We should add a low-cost model, which would allow us to increase our overall sales while continuing to dominate the high end. Tracy: I disagree. Our equipment sells to consumers who associate our company with quality. Moving into the low-cost market would put our competitors in the high-cost market on an equal footing with us, which could hurt our overall sales.", "question": "Jean's and Tracy's statements most strongly suggest that they disagree over which one of the following propositions?", "answers": "['There is a greater potential for profits in the low-cost market than there is in the high-cost market.', 'Decreased sales of the high-cost model would result in poor sales for the proposed low-cost model.', 'The company would no longer dominate the high-cost market if it began selling a low-cost model.', \"The proposed cheaper model, if it were made available, would sell to customers who would otherwise be buying the company's present model.\"]", "label": 2 }, { "id": "train_2870", "context": "A psychiatrist argued that there is no such thing as a multiple personality disorder on the grounds that in all her years of clinical practice, she had never encountered one case of this type.", "question": "Which one of the following most closely parallels the questionable reasoning cited above?", "answers": "['Lyla said that no one in the area has seen a groundhog and so there are probably no groundhogs in the area.', \"Sauda argued that because therapy rarely had an effect on her patient's type of disorder, therapy was not warranted.\", 'Jerod had never spotted a deer in his area and concluded from this that there are no deer in the area.', 'Thomas argued that because Natasha has driven her car to work every day since she bought it, she would probably continue to drive her car to work.']", "label": 2 }, { "id": "train_2871", "context": "Letter to the editor: Middle-class families in wealthy nations are often criticized for the ecological damage resulting from their lifestyles. This criticism should not be taken too seriously, however, since its source is often a movie star or celebrity whose own lifestyle would, if widely adopted, destroy the environment and deplete our resources in a short time.", "question": "The reasoning in the letter to the editor is vulnerable to criticism in that it", "answers": "['criticizes a characteristic of the people giving an argument rather than criticizing the argument itself', 'takes failure to act consistently with a belief as an indication of the sincerity with which that belief is held', 'fails to recognize that evidence advanced in support of a conclusion actually undermines that conclusion', 'presumes that a viewpoint must be unreasonable to accept simply because some of the grounds advanced to support it do not adequately do so']", "label": 0 }, { "id": "train_2872", "context": "To discover what percentage of teenagers believe in telekinesis -- the psychic ability to move objects without physically touching them -- a recent survey asked a representative sample of teenagers whether they agreed with the following statement: \"A person' s thoughts can influence the movement of physical objects. \" But because this statement is particularly ambiguous and is amenable to a naturalistic, uncontroversial interpretation, the survey' s responses are also ambiguous.", "question": "The reasoning above conforms most closely to which one of the following general propositions?", "answers": "['Every statement is amenable to several interpretations.', 'Responses people give to poorly phrased questions are likely to be ambiguous.', \"Responses to surveys are always unambiguous if the survey's questions are well phrased.\", 'Uncontroversial statements are useless in surveys.']", "label": 1 }, { "id": "train_2873", "context": "When a new restaurant, Martin' s Cafe, opened in Riverville last year, many people predicted that business at the Wildflower Inn, Riverville' s only other restaurant, would suffer from the competition. Surprisingly, however, in the year since Martin' s Cafe opened, the average number of meals per night served at the Wildflower Inn has increased significantly.", "question": "Which of the following, if true, most helps to explain the increase?", "answers": [ "The profit per meal is higher, on average, for meals served at Martin's Cafe than for those served at the Wildflower Inn.", "Most of the customers of Martin's Cafe had never dined in Riverville before this restaurant opened, and on most days Martin's Cafe attracts more customers than it can seat.", "A significant proportion of the staff at Martin's Cafe are people who formerly worked at the Wildflower Inn and were hired away by the owner of Martin's Cafe.", "The Wildflower Inn is not open on Sundays, and ttherefore Riverville residents who choose to dine out on that day must either eat at Martin's Cafe or go to neighboring towns to eat." ], "label": 1 }, { "id": "train_2874", "context": "Literary critic: Often the heirs of a successful writer decide to publish the manuscripts and the letters the dead writer left behind, regardless of the merit of the work. However, many writers have manuscripts that they judge to be unworthy of publication and with which they would not like to be publicly associated even after they die. Hence a successful writer who decides not to publish a recently completed manuscript should destroy it immediately.", "question": "Which one of the following statements, if true, most calls into question the soundness of the literary critic's advice?", "answers": "[\"Most successful writers' judgments of their recently completed work is unnecessarily harsh and is often later revised.\", 'Writers who achieve a certain degree of fame can expect that some of their personal correspondence will become publicly available after they die.', 'Many posthumously published books would have been published by the author had the author lived.', 'Some writers whose work becomes both popular and respected after they die received no literary recognition during their lifetimes.']", "label": 0 }, { "id": "train_2875", "context": "Pundit: For many high school graduates, attending a university would be of no help in getting a corporate job. The attributes corporations value most in potential employees are initiative, flexibility, and the ability to solve practical problems. Many new high school graduates have these attributes already.", "question": "The pundit's argument is most vulnerable to criticism on the grounds that it", "answers": "['overlooks the possibility that corporations may require an attribute that potential employees can obtain only by attending a university', 'takes for granted that initiative, flexibility, and the ability to solve practical problems are attributes that can be acquired through study', 'provides no justification for the presumption that corporations only hire employees who have initiative, flexibility, and the ability to solve practical problems', 'fails to establish that university graduates do not have initiative, flexibility, and the ability to solve practical problems']", "label": 0 }, { "id": "train_2876", "context": "A group of unusual meteorites was found in Shergotty, India. Their structure indicates that they originated on one of the geologically active planets, Mercury, Venus, or Mars. Because of Mercury' s proximity to the Sun, any material dislodged from that planet' s surface would have been captured by the Sun, rather than falling to Earth as meteorites. Nor could Venus be the source of the meteorites, because its gravity would have prevented dislodged material from escaping into space. The meteorites, ttherefore, probably fell to Earth after being dislodged from Mars, perhaps as the result of a collision with a large object.", "question": "The argument derives its conclusion by", "answers": "['eliminating competing alternative explanations', 'contrasting present circumstances with past circumstances', 'abstracting a general principle from specific data', 'offering a counterexample to a theory']", "label": 0 }, { "id": "train_2877", "context": "During the 1980s the homicide rate in Britain rose by 50 percent. The weapon used usually was a knife. Potentially lethal knives are sold openly and legally in many shops. Most homicide deaths occur as a result of unpremeditated assaults within the family. Even if these are increasing, they would probably not result in deaths if it were not for the prevalence of such knives. Thus the blame lies with the permissiveness of the government that allows such lethal weapons to be sold.", "question": "Which one of the following is the strongest criticism of the argument above?", "answers": "['The argument assumes without justification that the knives used to commit homicide are generally purchased as part of a deliberate plan to commit murder or to inflict grievous harm on a family member.', 'If the potentially lethal knives referred to are ordinary household knives, such knives were common before the rise in the homicide rate; but if they are weaponry, such knives are not generally available in households.', 'Knives are used in other homicides besides those that result from unpremeditated assaults within the family.', 'There are other means besides knives, such as guns or poison, that can be used to accomplish homicide by a person who intends to cause the death of another.']", "label": 1 }, { "id": "train_2878", "context": "While conditions on Mars are no longer favorable for the evolution of life, scientists point out that in many ways Mars resembles portions of Antarctica, which do manage to support colonies of microbes. But these organisms require liquid water, as do all forms of life. And if there is liquid water on Mars at all, it is only seasonal and in small amounts.", "question": "The statements above, if true, most strongly support which one of the following?", "answers": "['If there is life on Mars, then there is liquid water on Mars.', 'If there are colonies of microbes in Antarctica, then there are colonies of microbes on Mars.', 'It is unreasonable to suppose that Mars has ever supported any forms of life.', 'If there is liquid water on Mars, then there are colonies of microbes on Mars.']", "label": 0 }, { "id": "train_2879", "context": "A recent study proves that at least some people possess an independent \"sixth sense\"that allows them to detect whether someone is watching them. In the study, subjects were seated one at a time in the center of a room facing away from a large window. On average, subjects decided correctly 60 percent of the time whether or not they were being watched through the window.", "question": "Which one of the following, if true, most supports the conclusion drawn from the study mentioned above?", "answers": "['The room in which the subjects were seated was not soundproof.', 'A similar result was found when the subjects were watched from another room on a video monitor.', 'The person recording the experimental results was careful not to interact with the subjects after the experiment ended.', 'Most of the time, subjects said they were being watched.']", "label": 1 }, { "id": "train_2880", "context": "In Brazil, side-by-side comparisons of Africanized honeybees and the native honeybees have shown that the Africanized bees are far superior honey producers. Ttherefore, there is no reason to fear that domestic commercial honey production will decline in the United States if local honeybees are displaced by Africanized honeybees.", "question": "Each of the following, if true, would weaken the argument EXCEPT:", "answers": "['If Africanized honeybees replace local honeybees, certain types of ornamental trees will be less effectively pollinated.', 'Commercial honey production is far more complicated and expensive with Africanized honeybees than it is with the more docile honeybees common in the United States.', 'The area of Brazil where the comparative study was done is far better suited to the foraging habits of the Africanized honeybees than are most areas of the United States.', 'In the United States a significant proportion of the commercial honey supply comes from hobby beekeepers, many of whom are likely to abandon beekeeping with the influx of Africanized bees.']", "label": 0 }, { "id": "train_2881", "context": "Each of the elements of Girelli' s recently completed design for a university library is copied from a different one of several historic libraries. The design includes various features from Classical Greek, Islamic, Mogul, and Romanesque structures. Since no one element in the design is original, it follows that the design of the library cannot be considered original.", "question": "Which one of the following is a reasoning error made in the argument?", "answers": "['concluding that an unknown instance of a phenomenon must have all the properties of the known instances', 'deriving a factual conclusion from evidence derived from reports of aesthetic preferences', 'generalizing illegitimately from a few instances of a certain kind to all instances of that kind', 'assuming that because something is true of each of the parts of a whole it is true of the whole itself']", "label": 3 }, { "id": "train_2882", "context": "Shortly after the Persian Gulf War, investigators reported that the area, which had been subjected to hundreds of smoky oil fires and deliberate oil spills when regular oil production slowed down during the war, displayed less oil contamination than they had witnessed in prewar surveys of the same area. They also reported that the levels of polycyclic aromatic hydrocarbons (PAHs) -- used as a marker of combustion products spewed from oil wells ignited during the war -- were also relatively low, comparable to those recorded in the temperate oil-producing areas of the Baltic Sea.", "question": "Which one of the following, if true, does most to resolve the apparent discrepancy in the information above?", "answers": "['Oil contamination and PAH pollution dissipate more rapidly in desert regions than in temperate regions.', 'The Persian Gulf War ended before the oil fires and spills caused as much damage as originally expected.', 'Oil contaminants have greater environmental effects in temperate regions than in desert regions.', 'Peacetime oil production and transport in the Persian Gulf result in high levels of PAHs and massive oil dumping.']", "label": 3 }, { "id": "train_2883", "context": "It would be erroneous to assume that an employee is embezzling funds if there is no other evidence than the fact that the employee has a high balance on his company credit card. It is important to remember that embezzling only happens if the employee is using his company credit card for purposes other than to entertain clients.", "question": "The reasoning of which of the following is most parallel to that in the paragraph above?", "answers": "['A child cannot be diagnosed with asthma just because he or she becomes out of breath quickly, because most children are simply out of shape.', 'Most people who appear to be physically fit are not actually physically fit because being slender is not the only sign of physical fitness.', 'Water purifiers should be used in areas where there are large quantities of iron in the water, but bottled water should never be drunk because it does not normally contain fluoride.', 'Even if a small child is cranky, this does not necessarily mean that he or she needs discipline, because discipline is needed when a child is cranky and will not take a nap.']", "label": 3 }, { "id": "train_2884", "context": "The senator has long held to the general principle that no true work of art is obscene, and thus that there is no conflict between the need to encourage free artistic expression and the need to protect the sensibilities of the public from obscenity. When well-known works generally viewed as obscene are cited as possible counterexamples, the senator justifies accepting the principle by saying that if these works really are obscene then they cannot be works of art.", "question": "The senator's reasoning contains which one of the following errors?", "answers": "['It assumes what it seeks to establish.', 'It seeks to persuade by emotional rather than intellectual means.', 'It contains an implicit contradiction.', \"It relies on an assertion of the senator's authority.\"]", "label": 0 }, { "id": "train_2885", "context": "Agricultural scientist: Wild apples are considerably smaller than cultivated apples found in supermarkets. In one particular region, archaeologists have looked for remains of cultivated apples dating from 5, 000 years ago, around the time people first started cultivating fruit. But the only remains of apples that archaeologists have found from this period are from fruits the same size as the wild apples native to the region. So apples were probably not cultivated in this region 5, 000 years ago.", "question": "The agricultural scientist's argument is most vulnerable to criticism on the grounds that the argument", "answers": "['uses a claim that presupposes the truth of its main conclusion as part of the justification for that conclusion', 'takes for granted that all apples are either the size of wild apples or the size of the cultivated apples now found in supermarkets', 'fails to consider that even if a plant was not cultivated in a given region at a specific time, it may have been cultivated in nearby regions at that time', 'fails to consider that plants that have been cultivated for only a short time may tend to resemble their wild counterparts much more closely than plants that have been cultivated for a long time']", "label": 3 }, { "id": "train_2886", "context": "Governments have only one response to public criticism of socially necessary services: regulation of the activity of providing those services. But governments inevitably make the activity more expensive by regulating it, and that is particularly troublesome in these times of strained financial resources. However, since public criticism of childcare services has undermined all confidence in such services, and since such services are socially necessary, the government is certain to respond.", "question": "Which one of the following statements can be inferred from the passage?", "answers": "['The cost of providing child-care services will increase.', 'The quality of child care will improve.', 'If child-care services are not regulated, the cost of providing child care will not increase.', 'The government will use funding to foster advances in child care.']", "label": 0 }, { "id": "train_2887", "context": "The initial causes of serious accidents in space shuttle missions so far have not been flaws in the advanced technology used in the space shuttles. Rather, the causes have been attributed to human error. Unfortunately, human error is bound to occur at some point.", "question": "Which one of the following is most strongly supported by the statements above?", "answers": "['Space shuttle missions are not worth the risk.', 'As the technology used in space shuttles improves, it will eventually overcome human error.', 'If space shuttle missions continue, a serious accident is not improbable.', 'No serious accident will be caused in the future by some flaw in the advanced technology used in space shuttles.']", "label": 2 }, { "id": "train_2888", "context": "It is probably not true that colic in infants is caused by the inability of those infants to tolerate certain antibodies found in cow' s milk, since it is often the case that symptoms of colic are shown by infants that are fed breast milk exclusively.", "question": "Which one of the following, if true, most seriously weakens the argument?", "answers": "[\"Symptoms of colic generally disappear as infants grow older, whether the infants have been fed breast milk exclusively or have been fed infant formula containing cow's milk.\", \"When mothers of infants that are fed only breast milk eliminate cow's milk and all products made from cow's milk from their own diets, any colic symptoms that their infants have manifested quickly disappear.\", \"Infants that are fed breast milk develop mature digestive systems at an earlier age than do those that are fed infant formulas, and infants with mature digestive systems are better able to tolerate certain proteins and antibodies found in cow's milk.\", 'A study involving 500 sets of twins has found that if one infant has colic, its twin will probably also have colic.']", "label": 1 }, { "id": "train_2889", "context": "Throughout European history famines have generally been followed by periods of rising wages, because when a labor force is diminished, workers are more valuable in accordance with the law of supply and demand. The Irish potato famine of the 1840s is an exception; it resulted in the death or emigration of half of Ireland' s population, but there was no significant rise in the average wages in Ireland in the following decade.", "question": "Which one of the following, if true, would LEAST contribute to an explanation of the exception to the generalization?", "answers": "['The birth rate increased during the decade following the famine, and this compensated for much of the loss of population that was due to the famine.', 'Eviction policies of the landowners in Ireland were designed to force emigration of the elderly and infirm, who could not work, and to retain a high percentage of able-bodied workers.', 'Improved medical care reduced the mortality rate among able-bodied adults in the decade following the famine to below prefamine levels.', 'England, which had political control of Ireland, legislated artificially low wages to provide English-owned industry and agriculture in Ireland with cheap labor.']", "label": 0 }, { "id": "train_2890", "context": "There are only two possible reasons that it would be wrong to engage in an activity that causes pollution: because pollution harms ecosystems, which are valuable in themselves; or, ecosystems aside, because pollution harms human populations. Either way, it would not be wrong to perform mining operations on Mars. Although doing so would pollute Mars, the small human presence needed to run the mining operation would be completely protected from the Martian environment and would suffer no harm.", "question": "The conclusion drawn above follows logically if which one of the following is assumed?", "answers": "['There are no ecosystems on Mars.', 'The more complex an ecosystem is, the more valuable it is.', 'Mining creates less pollution than many other human activities.', 'It is technologically feasible to perform mining operations on Mars.']", "label": 0 }, { "id": "train_2891", "context": "The increase in the price of housing in city Y has been caused by a decline in the number of housing units available compared to the influx of new residents to city Y searching for housing. Nevertheless, the total number of available housing units is presently larger than it was last month.", "question": "If the statements above are true, which one of the following can be properly inferred?", "answers": "['The number of residents seeking housing has increased over the last month.', 'The number of landlords leaving the housing market has increased this month.', 'A lack of available housing will deter new people from living in city Y.', 'The cost of housing will continue to rise in proportion to demand.']", "label": 0 }, { "id": "train_2892", "context": "Generic drugs contain exactly the same active ingredients as their brand-name counterparts, but usually cost much less to develop, produce, and market. So, generic drugs are just as effective as their brand-name counterparts, but cost considerably less.", "question": "Which one of the following, if true, most strengthens the argument?", "answers": "['Generic drugs are no more likely than brand name drugs to suffer from defects in composition.', 'Generic drugs are just as likely as brand-name drugs to be readily available in pharmacies.', \"The higher costs of brand-name drugs underwrite drug companies' heavy investment in research.\", 'Because of advertising, doctors frequently prescribe brand-name drugs by their brand name, rather than by their chemical name.']", "label": 0 }, { "id": "train_2893", "context": "Economist: In 2015, the average per-person amount paid for goods and services purchased by consumer in Country X was the equivalent of $17, 570 in United States dollars, just 30 percent of the corresponding figure of $58, 566 for Country Y. Yet in 2015, there was already a substantial middle class in Country X that had discretionary income for middle-class consumer goods such as new vehicles, computers, or major household appliances, while a significant portion of the middle class in Country Y did not have sufficient income to purchase such items.", "question": "Which of the following, if true, most helps explain the discrepancy in the relationships described by the economist?", "answers": "['The economy of Country Y is recovering from a downturn that affected both Country Y and Country X.', 'There are many consumer goods, such as household appliances, that are produced in Country X to be sold in the Country Y market.', 'The volume of trade between Country X and Country Y is increasing rapidly in both directions.', 'Country X residents pay much less than their Country Y counterparts for housing, transportation, and child care.']", "label": 3 }, { "id": "train_2894", "context": "The average level of fat in the blood of people suffering from acute cases of disease W is lower than the average level for the population as a whole. Nevertheless, most doctors believe that reducing blood-fat levels is an effective way of preventing acute W.", "question": "Which one of the following, if true, does most to justify this apparently paradoxical belief?", "answers": "[\"The progression from latent to acute W can occur only when the agent that causes acute W absorbs large quantities of fat from the patient's blood.\", 'Several of the symptoms characteristic of acute W have been produced in laboratory animals fed large doses of a synthetic fat substitute, though acute W itself has not been produced in this way.', 'High levels of fat in the blood are indicative of several diseases that are just as serious as W.', 'The blood level of fat for patients who have been cured of W is on average the same as that for the population at large.']", "label": 0 }, { "id": "train_2895", "context": "Because most hospitals suffer a chronic undersupply of physicians, patients must sometimes wait hours in the emergency room to see a doctor. Nurses should ttherefore perform initial examinations in hospital emergency rooms to determine which patients merit immediate treatment and which can wait until the emergency physicians have more time to see them.", "question": "Which of the following is an assumption on which the argument above is based?", "answers": "[\"Nurses are competent to judge the severity of patients' conditions.\", 'Hospitals should expand their medical staffs.', 'Emergency rooms will run more smoothly if initial examinations are performed.', 'Physicians cannot be trained to perform initial examinations themselves.']", "label": 0 }, { "id": "train_2896", "context": "Critic: Works of modern literature cannot be tragedies as those of ancient playwrights and storytellers were unless their protagonists are seen as possessing nobility, which endures through the calamities that befall one. In an age that no longer takes seriously the belief that human endeavors are governed by fate, it is ttherefore impossible for a contemporary work of literature to be a tragedy.", "question": "Which one of the following is an assumption required by the critic's argument?", "answers": "['Most plays that were once classified as tragedies were misclassified.', 'If an ignoble character in a work of literature endures through a series of misfortunes, that work of literature is not a tragedy.', 'The belief that human endeavors are governed by fate is false.', 'Those whose endeavors are not regarded as governed by fate will not be seen as possessing nobility.']", "label": 3 }, { "id": "train_2897", "context": "Professor: The number of new university students who enter as chemistry majors has not changed in the last ten years, and job prospects for graduates with chemistry degrees are better than ever. Despite this, there has been a significant decline over the past decade in the number of people earning chemistry degrees.", "question": "Which one of the following, if true, most helps to explain the decline?", "answers": "['Over the years, first-year chemistry has come to be taught in a more routinely methodical fashion, which dampens its intellectual appeal.', 'There has been a significant decline in the number of undergraduate degrees earned in the natural sciences as a whole.', 'Many students enter universities without the academic background that is necessary for majoring in chemistry.', 'Job prospects for graduates with chemistry degrees are no better than prospects for graduates with certain other science degrees.']", "label": 0 }, { "id": "train_2898", "context": "Hospital executive: At a recent conference on nonprofit management, several computer experts maintained that the most significant threat faced by large institutions such as universities and hospitals is unauthorized access to confidential data. In light of this testimony, we should make the protection of our clients' confidentiality our highest priority.", "question": "The hospital executive's argument is most vulnerable to which one of the following objections?", "answers": "['The argument draws a general conclusion about a group based on data about an unrepresentative sample of that group.', 'The argument infers that a property belonging to large institutions belongs to all institutions.', 'The argument confuses the causes of a problem with the appropriate solutions to that problem.', 'The argument relies on the testimony of experts whose expertise is not shown to be sufficiently broad to support their general claim.']", "label": 3 }, { "id": "train_2899", "context": "Government-subsidized insurance available to homeowners makes it feasible for anyone to build a house on a section of coastline regularly struck by hurricanes. Each major storm causes billions of dollars worth of damage in such coastal areas, after which owners who have insurance are able to collect an amount of money sufficient to recoup a high percentage of their losses.", "question": "The passage provides the most support for an argument against a government bill proposing", "answers": "['that power companies be required to bury power lines in areas of the coastline regularly struck by hurricanes', 'an increase in funding of weather service programs that provide a hurricane watch and warning system for coastal areas', 'establishment of an agency committed to managing coastal lands in ecologically responsible ways', 'establishment of a contingency fund protecting owners of uninsured houses in the coastal areas from catastrophic losses due to the hurricane damage']", "label": 3 }, { "id": "train_2900", "context": "Crowding on Mooreville' s subway frequently leads to delays, because it is difficult for passengers to exit from the trains. Subway ridership is projected to increase by 20 percent over the next 10 years. The Mooreville Transit Authority plans to increase the number of daily train trips by only 5 percent over the same period. Officials predict that this increase is sufficient to ensure that the incidence of delays due to crowding does not increase.", "question": "Which of the following, if true, provides the strongest grounds for the officials' prediction?", "answers": "['Most of the projected increase in ridership is expected to occur in off-peak hours when trains are now sparsely used.', 'The 5 percent increase in the number of train trips can be achieved without an equal increase in Transit Authority operational costs.', 'By changing maintenance schedules, the Transit Authority can achieve the 5 percent increase in train trips without purchasing any new subway cars.', 'The Transit Authority also plans a 5 percent increase in the number of bus trips on routes that connect to subways.']", "label": 0 }, { "id": "train_2901", "context": "Campaigning for election to provincial or state office frequently requires that a candidate spend much time and energy catering to the interests of national party officials who can help the candidate to win office. The elected officials who campaign for reelection while they are in office thus often fail to serve the interests of their local consistencies.", "question": "Which one of the following is an assumption made by the argument?", "answers": "['Officials elected to provincial or state office are obligated to serve only the interests of constituents who belong to the same party as do the officials.', 'The interests of local constituencies are well served only by elected officials who do not cater to the interests of national party officials.', \"Catering to the interests of national party officials sometimes conflicts with serving the interests of a provincial or state official's local constituencies.\", 'All elected officials are likely to seek reelection to those offices that are not limited to one term.']", "label": 2 }, { "id": "train_2902", "context": "The desire for praise is the desire to obtain, as a sign that one is good, the favorable opinions of others. But because people merit praise only for those actions motivated by a desire to help others, it follows that one who aids others primarily out of a desire for praise does not deserve praise for that aid.", "question": "Which one of the following, if assumed, enables the conclusion of the argument to be properly drawn?", "answers": "[\"One deserves praise for advancing one's own interests only if one also advances the interests of others.\", 'People who are indifferent to the welfare of others do not deserve praise.', 'An action that is motivated by a desire for the favorable opinion of others cannot also be motivated by a desire to help others.', 'No action is worthy of praise if it is motivated solely by a desire for praise.']", "label": 2 }, { "id": "train_2903", "context": "Last year a global disturbance of weather patterns disrupted harvests in many of the world's important agricultural areas. Worldwide production of soybeans, an important source of protein for people and livestock alike, was not adversely affected, however. Indeed, last year's soybean crop was actually slightly larger than average. Nevertheless, the weather phenomenon is probably responsible for a recent increase in the world price of soybeans.", "question": "Which of the following, if true, provides the strongest justification for the attribution of the increase in soybean prices to the weather phenomenon?", "answers": "[\"Last year's harvest of anchovies, which provide an important protein source for livestock, was disrupted by the effects of the weather phenomenon.\", 'The world price of soybeans also rose several years ago, immediately after an earlier occurrence of a similar global weather disturbance.', 'Most countries that produce soybeans for export had above-average harvests of a number of food crops other than soybeans last year.', 'Heavy rains attributable to the weather phenomenon improved grazing pastures last year, allowing farmers in many parts of the world to reduce their dependence on supplemental feed.']", "label": 0 }, { "id": "train_2904", "context": "At the end of 1997 several nations stated that their oil reserves had not changed since the end of 1996. But oil reserves gradually drop as old oil fields are drained and rise suddenly as new oil fields are discovered. Ttherefore, oil reserves are unlikely to remain unchanged from one year to the next. So most of the nations stating that their oil reserves were unchanged are probably incorrect.", "question": "Which one of the following is an assumption the argument requires?", "answers": "[\"For any nation with oil reserves, it is more likely that the nation was mistaken in its statements about changes in its oil reserves than that the nation's oil reserves remained unchanged.\", 'It is likely that in 1997, in most of the nations that stated that their oil reserves were unchanged, old oil fields were drained or new oil fields were discovered, or both.', 'If a nation incorrectly stated at the end of 1997 that its oil reserves had not changed since the end of 1996, then during 1997 that nation drained its old oil fields and discovered new ones.', \"If a nation's oil reserves change from one year to the next, then that nation is obligated to report the change correctly.\"]", "label": 1 }, { "id": "train_2905", "context": "Advertisement: My five-year-old identical twins wear matching clothing. I washed two of their soiled matching shirts, one in my regular laundry detergent, and one in new WAVE laundry detergent. The shirt washed in WAVE came out cleaner than the other shirt, so WAVE cleans better than my regular laundry detergent.", "question": "Which of the following is an assumption on which the argument in the advertisement above depends?", "answers": "['Most pairs of young identical twins tend to wear matching clothing.', 'The shirt laundered in the regular laundry detergent was not significantly less clean than the other shirt before being laundered.', 'Clothes washed in WAVE laundry detergent will come out cleaner than clothes washed in any other brand of laundry detergent', 'A soiled shirt laundered on a regular basis in the same laundry detergent will come out equally clean each time it is laundered.']", "label": 1 }, { "id": "train_2906", "context": "Increased use of incineration is sometimes advocated as a safe way to dispose of chemical waste. But opponents of incineration point to the 40 incidents involving unexpected releases of dangerous chemical agents that were reported just last year at two existing incinerators commissioned to destroy a quantity of chemical waste material. Since designs for proposed new incinerators include no additional means of preventing such releases, leaks will only become more prevalent if use of incineration increases.", "question": "Which of the following, if true, most seriously weakens the argument?", "answers": "['The capacity of existing incinerators is sufficient to allow for increased incineration of chemical waste without any need for new incinerators.', 'Other means of disposing of chemical waste, such as chemical neutralization processes, have not been proven safer than incineration.', 'The frequency of reports of unexpected releases of chemical agents at newly built incinerators is about the same as the frequency at older incinerators.', 'At the two incinerators at which leaks were reported, staff had had only cursory training on the proper procedures for incinerating chemical waste.']", "label": 3 }, { "id": "train_2907", "context": "Political scientist: As a political system, democracy does not promote political freedom. There are historical examples of democracies that ultimately resulted in some of the most oppressive societies. Likewise, there have been enlightened despotisms and oligarchies that have provided a remarkable level of political freedom to their subjects.", "question": "The reasoning in the political scientist's argument is flawed because it", "answers": "['fails to consider that a substantial increase in the level of political freedom might cause a society to become more democratic', 'confuses the conditions necessary for political freedom with the conditions sufficient to bring it about', 'overlooks the possibility that democracy promotes political freedom without being necessary or sufficient by itself to produce it', 'appeals to historical examples that are irrelevant to the causal claim being made']", "label": 2 }, { "id": "train_2908", "context": "Prehistoric chimpanzee species used tools similar to those used by prehistoric humans; prehistoric tools recently found in East Africa are of a type used by both species. The area where the tools were found, however, is a savanna, and whereas there were prehistoric humans who lived in savanna habitats, prehistoric chimpanzees lived only in forests. Ttherefore, the tools must have been used by humans rather than by chimpanzees.", "question": "Which one of the following is an assumption on which the argument depends?", "answers": "['The prehistoric ancestors of modern chimpanzees were not capable of using tools more sophisticated than those found recently in East Africa.', 'Prehistoric humans did not carry their tools with them when they traveled from one place to another.', 'Prehistoric humans never ventured into areas of the forest that were inhabited by prehistoric chimpanzees.', 'The area where the tools were found was not a forest at the time the tools were in use.']", "label": 3 }, { "id": "train_2909", "context": "Vanessa: All computer code must be written by a pair of programmers working at a single workstation. This is needed to prevent programmers from writing idiosyncratic code that can be understood only by the original programmer. Jo: Most programming projects are kept afloat by the best programmers on the team, who are typically at least 100 times more productive than the worst. Since they generally work best when they work alone, the most productive programmers must be allowed to work by themselves.", "question": "Each of the following assignments of computer programmers is consistent both with the principle expressed by Vanessa and with the principle expressed by Jo EXCEPT:", "answers": "['Olga and Kensuke are both programmers of roughly average productivity who feel that they are more productive when working alone. They have been assigned to work together at a single workstation.', 'John is experienced but is not among the most productive programmers on the team. He has been assigned to mentor Tyrone, a new programmer who is not yet very productive. They are to work together at a single workstation.', 'Although not among the most productive programmers on the team, Chris is more productive than Jennifer. They have been assigned to work together at a single workstation.', 'Yolanda is the most productive programmer on the team. She has been assigned to work with Mike, who is also very productive. They are to work together at the same workstation.']", "label": 3 }, { "id": "train_2910", "context": "The symptoms that United States President Zachary Taylor began showing five days before his death are consistent with arsenic poisoning. Recent examination of Taylor' s bones, however, revealed levels of arsenic comparable to those found in the bones of Taylor' s contemporaries. These levels are much lower than the levels of arsenic that remain in the bones of arsenic poisoning victims who live for more than a day after ingesting a lethal dose of the poison.", "question": "Which of the following is most strongly supported by the statements given?", "answers": "[\"Taylor's death was not the result of any kind of poisoning.\", 'The symptoms that Taylor began showing five days before his death are consistent with poisoning other than arsenic poisoning.', 'It is unusual for a person who has ingested a lethal dose of arsenic to survive for more than a day.', 'The symptoms that Taylor began showing five days before his death were not caused by a lethal dose of arsenic.']", "label": 3 }, { "id": "train_2911", "context": "Doris: I' ve noticed that everyone involved in student government is outspoken. So if we want students to be more outspoken, we should encourage them to become involved in student government. Zack: Those who are in student government became involved precisely because they are outspoken in the first place. Encouraging others to become involved will do nothing to make them more outspoken.", "question": "Doris and Zack disagree over whether", "answers": "['students will not become more outspoken unless they become involved in student government', 'students should be more outspoken', 'becoming involved in student government makes students more outspoken', 'students should be encouraged to become involved in student government']", "label": 2 }, { "id": "train_2912", "context": "Motor oil serves to lubricate engines and thus retard engine wear. A study was conducted to assess the effectiveness of various brands of motor oil by using them in taxicabs over a 6, 000-mile test period. All the oils did equally well in retarding wear on pistons and cylinders, the relevant parts of the engine. Hence, cheaper brands of oil are the best buys.", "question": "Which one of the following, if true, most weakens the argument?", "answers": "['The lubricating properties of all motor oils deteriorate over time, and the rate of deterioration is accelerated by heat.', 'Cheaper brands of motor oil are often used by knowledgeable automobile mechanics for their own cars.', 'Ability to retard engine wear is not the only property of motor oil important to the running of an engine.', 'Tests other than of the ability to reduce engine wear also can reliably gauge the quality of motor oil.']", "label": 2 }, { "id": "train_2913", "context": "Jacqueline: I work one full-time job. With this job, I can make my own hours and work more if I needed to. However, I want to get a second job to change things up a bit. The second job would be part-time and I could work it on the weekends. Admittedly, I would be earning half the hourly wage with this part-time job and would do better to simply add more hours to my first job. Nonetheless, I still think I should get a second part-time job.", "question": "Which one of the following principles, if valid, most helps to justify Jacqueline's reasoning?", "answers": "['It is better to get a second part-time job because working on weekends does not bother Jacqueline.', 'It is better to stay with the full-time job and add more hours because it pays more.', 'It is better to get a second part-time job, even if it pays less, for a mental change.', 'It is better not to worry about making so much money; having freedom is the most important thing in life.']", "label": 2 }, { "id": "train_2914", "context": "Cooking teacher: Lima beans generally need about an hour of boiling to reach the proper degree of doneness. The precise amount of time it takes depends on size: larger beans require a longer cooking time than smaller beans do. It is important that lima beans not be overcooked since overcooking robs beans of many of their nutrients. Undercooking should also be avoided, since undercooked beans cannot be completely digested.", "question": "If the statements above are true, they most strongly support which one of the following?", "answers": "['Large lima beans, even when fully cooked, are more difficult to digest than small lima beans.', 'Lima beans that are added to the pot together should be as close to the same size as possible if they are to yield their full nutritional value.', 'The nutrients that are lost when lima beans are overcooked are the same as those that the body fails to assimilate when lima beans are not completely digested.', 'Lima beans that are completely digestible have lost many of their nutrients in cooking.']", "label": 1 }, { "id": "train_2915", "context": "In a mature tourist market such as Bellaria there are only two ways hotel owners can increase profits: by building more rooms or by improving what is already there. Rigid land-use laws in Bellaria rule out construction of new hotels or, indeed, any expansion of hotel capacity.", "question": "It follows that hotel owners cannot increase their profits in Bellaria since Bellarian hotels Which one of the following logically completes the argument?", "answers": "['are already operating at an occupancy rate approaching 100 percent year-round', 'have shifted from serving mainly Bellarian tourists to serving foreign tourists traveling in organized tour groups', 'could not have been sited any more attractively than they are even in the absence of land-use laws', 'already provide a level of luxury that is at the limits of what even wealthy patrons are prepared to pay for']", "label": 3 }, { "id": "train_2916", "context": "Joshua Smith' s new novel was criticized by the book editor for The Daily Standard as implausible. That criticism, like so many other criticisms from the same source in the past, is completely unwarranted. As anyone who has actually read the novel would agree, each one of the incidents in which Smith' s hero gets involved is the kind of incident that could very well have happened to someone or other.", "question": "Which one of the following is the most serious error of reasoning in the argument?", "answers": "['It relies on the assumption that a criticism can legitimately be dismissed as unwarranted if it is offered by someone who had previously displayed questionable judgment.', 'It calls into question the intellectual integrity of the critic in order to avoid having to address the grounds on which the criticism is based.', 'It ignores the fact that people can agree about something even though what they agree about is not the case.', 'It takes for granted that a whole story will have a given characteristic if each of its parts has that characteristic.']", "label": 3 }, { "id": "train_2917", "context": "Meteorologist: Heavy downpours are likely to become more frequent if Earth' s atmosphere becomes significantly warmer. A warm atmosphere heats the oceans, leading to faster evaporation, and the resulting water vapor forms rain clouds more quickly. A warmer atmosphere also holds more moisture, resulting in larger clouds. In general, as water vapor in larger clouds condenses, heavier downpours are more likely to result.", "question": "Which one of the following most accurately describes the role played in the meteorologist's argument by the claim that, in general, as water vapor in larger clouds condenses, heavier downpours are more likely to result?", "answers": "['It provides a causal explanation of the phenomenon described by the conclusion of the argument as a whole, but it is not intended to provide support for that conclusion.', 'It is a statement that the argument is intended to support but is not the conclusion of the argument as a whole.', 'It is used to support the only conclusion in the argument.', 'It is the only conclusion in the argument.']", "label": 2 }, { "id": "train_2918", "context": "The proposed coal-burning electric plant should be approved, since no good arguments have been offered against it. After all, all the arguments against it have been presented by competing electricity producers.", "question": "Which one of the following is an assumption on which the reasoning above depends?", "answers": "['If good arguments are presented for a proposal, then that proposal should be approved.', 'The competing electricity producers would stand to lose large amounts of revenue from the building of the coal-burning electric plant.', 'Arguments made by those who have a vested interest in the outcome of a proposal are not good arguments.', 'Approval of the coal-burning electric plant would please coal suppliers more than disapproval would please suppliers of fuel to the competing electricity producers.']", "label": 2 }, { "id": "train_2919", "context": "Based on studies by the state health department, the Surgeon General has formulated tables that list standard exercise requirements for various age groups. People whose activity level falls within the range given for their age experience fewer cardiovascular problems than those whose activity level falls below the given range. Ttherefore, if people whose activity level falls below their given range increased their time exercising to fall within that range, their cardiovascular health would improve.", "question": "Which one of the following is an assumption on which the argument relies?", "answers": "['The Surgeon General intended her table to help motivate people to increase their activity level and improve their cardiovascular health.', \"People who follow the Surgeon General's table guidelines have better overall health than those who do not.\", 'Increasing the activity level would not itself cause cardiovascular health problems for people.', 'Some individuals would be willing to increase their activity level solely to improve their cardiovascular health.']", "label": 2 }, { "id": "train_2920", "context": "Workers may complain about many things at work, but stress is not high on the list. In fact, in a recent survey a majority placed boredom at the top of their list of complaints. The assumption that job-related stress is the most serious problem for workers in the corporate world is thus simply not warranted.", "question": "Which one of the following, if true, most seriously weakens the argument?", "answers": "['Those workers who are responsible for the planning and supervision of long-term projects are less likely to complain of either boredom or stress.', 'Workers responding to opinion surveys tend to emphasize those experiences that have happened most recently.', 'Workers are less likely to complain about work if they feel that their jobs are secure.', 'Workers who complain of boredom exhibit more stress-related symptoms than do those who claim their work is interesting.']", "label": 3 }, { "id": "train_2921", "context": "If the ivory trade continues, experts believe, the elephant will soon become extinct in Africa, because poaching is rife in many areas. A total ban on ivory trading would probably prevent the extinction. However, the country of Zimbabwe -- which has virtually eliminated poaching within its borders and which relies on income from carefully culling elephant herds that threaten to become too big -- objects to such a ban. Zimbabwe holds that the problem lies not with the ivory trade but with the conservation policies of other countries.", "question": "Which one of the following principles forms a logical basis for Zimbabwe's objection to a ban?", "answers": "['Effective conservation cannot be achieved without eliminating poaching.', 'International measures to correct a problem should not adversely affect countries that are not responsible for the problem.', 'Freedom of trade is not a right but a consequence of agreements among nations.', 'Prohibitions affecting several countries should be enforced by a supranational agency.']", "label": 1 }, { "id": "train_2922", "context": "Albinism is a rare genetic condition that inhibits the production of melanin, or pigmentation, in the skin and hair. People born with albinism are unusually susceptible to sunburn, melanoma, and a range of other health issues that are generally connected to excessive exposure to the sun.", "question": "The statements above, if true, provide the most support for which of the following conclusions?", "answers": "['In humans, melanin plays a role in protecting the skin from developing sunburn and other sun-related ailments.', 'People born with albinism develop other biological protections against melanoma and other sun-related health issues.', 'It is not possible for a person born with albinism to adopt other artificial protective measures against excessive exposure to the sun.', 'When a non-albino person gets sunburn, the amount of melanin produced by that person decreases.']", "label": 0 }, { "id": "train_2923", "context": "Statistician: Changes in the Sun' s luminosity correlate exceedingly well with average land temperatures on Earth. Clearly -- and contrary to accepted opinion among meteorologists -- the Sun' s luminosity essentially controls land temperatures on Earth. Meteorologist: I disagree. Any professional meteorologist will tell you that in a system as complicated as that giving rise to the climate, no significant aspect can be controlled by a single variable.", "question": "The rejection by the meteorologist of the statistician's conclusion employs which one of the following techniques of argumentation?", "answers": "['rejecting a conclusion because it is a proposition that cannot be experimentally tested', 'supporting a conclusion about a specific case by invoking a relevant generalization', 'pointing out that potentially unfavorable evidence has been systematically neglected', 'reanalyzing a correlation as reflecting the multiple effects of a single cause']", "label": 1 }, { "id": "train_2924", "context": "The case of the French Revolution is typically regarded as the best evidence for the claim that societies can reap more benefit than harm from a revolution. But even the French Revolution serves this role poorly, since France at the time of the Revolution had a unique advantage. Despite the Revolution, the same civil servants and functionaries remained in office, carrying on the day-to-day work of government, and thus many of the disruptions that revolutions normally bring were avoided.", "question": "Which one of the following most accurately characterizes the argumentative strategy used in the passage?", "answers": "['opposing a claim by undermining evidence offered in support of that claim', 'demonstrating that the claim argued against is internally inconsistent', 'comparing two positions in order to illustrate their relative strengths and weaknesses', 'justifying a view through the use of a series of persuasive examples']", "label": 0 }, { "id": "train_2925", "context": "Ecologist: Smallpox, one of the worst diseases ever to afflict humans, has in some outbreaks killed as much as 50 percent of local populations. The last known surviving cultures of variola -- the smallpox virus -- are confined to two high-security laboratories. Some scientists are anxious to destroy the remaining variola cultures, to which humans are susceptible. Research on the cultures, however, may someday lead to important benefits for humans, and so the cultures should not be destroyed.", "question": "Which one of the following, if true, would do most to strengthen the ecologist's argument?", "answers": "['Smallpox has killed millions of humans over the centuries, and when it reaches the stage of an epidemic it is extremely difficult to eradicate.', 'It is becoming increasingly important to prevent any nation from acquiring the means to wage biological warfare.', 'It is more likely that the virus, if left available to researchers, will lead to an important medical breakthrough than that the virus will be accidentally released from the laboratory.', 'It is likely that the virus, if released, will develop a resistance to vaccines previously used to control it.']", "label": 2 }, { "id": "train_2926", "context": "In Australia, in years with below-average rainfall, less water goes into rivers and more water is extracted from rivers for drinking and irrigation. Consequently, in such years, water levels drop considerably and the rivers flow more slowly. Because algae grow better the more slowly the water in which they are growing moves, such years are generally beneficial to populations of algae. But, by contrast, populations of algae drop in periods of extreme drought.", "question": "Which of the following, if true, does most to explain the contrast?", "answers": "['Algae cannot survive in the absence of water.', 'The more slowly water moves, the more conducive its temperature is to the growth of algae.', 'The larger the population of algae in a body of water, the less sunlight reaches below the surface of the water.', 'Algae grow better in ponds and lakes than in rivers.']", "label": 0 }, { "id": "train_2927", "context": "Many gardeners believe that the variety of clematis vine that is most popular among gardeners in North America is jackmanii. This belief is apparently correct since, of the one million clematis plants sold per year by the largest clematis nursery in North America, ten percent are jackmanii.", "question": "Which of the following is an assumption on which the argument depends?", "answers": "['Some of the jackmanii sold by the nursery are sold to gardeners outside North America.', 'Most North American gardeners grow clematis in their gardens.', 'The nursery sells more than ten different varieties of clematis.', 'For all nurseries in North America that specialize in clematis, at least ten percent of the clematis plants they sell are jackmanii.']", "label": 2 }, { "id": "train_2928", "context": "Edgar: Some of the pumps supplying water to our region have been ordered shut down in order to protect a species of small fish. But it is absurd to inconvenience thousands of people for the sake of something so inconsequential. Rafaela: You' re missing the point. The threat to that fish species is a sign of a very serious threat to our water supply.", "question": "The dialogue provides the most support for the claim that Edgar and Rafaela disagree over whether", "answers": "['the order to shut down the pumps was legal', 'shutting down the pumps will actually inconvenience a large number of people', 'shutting down the pumps will be sufficient to protect the fish species', 'the survival of the fish species is the only reason for shutting down the pumps']", "label": 3 }, { "id": "train_2929", "context": "Flagpole Army Base in Oregon and Klondike Army Base in Alaska are similar bases with similar duty loads. Despite the similarities, Klondike always has had more men than Flagpole has. Clearly, each soldier at Klondike must have to perform a smaller share of the duties, and ttherefore soldiers at Klondike must have more free time than do soldiers at Flagpole.", "question": "In evaluating the argument, it would be most useful to know which of the following?", "answers": "['Whether those recruits who are in a position to choose, choose Flagpole over Klondike.', 'Whether the officers administering the Flagpole and Klondike bases have had similar training.', 'Whether regular furloughs from Klondike are longer than furloughs from Flagpole, to account for added travel time.', 'Whether the morale level at Klondike is higher than the morale at Flagpole.']", "label": 2 }, { "id": "train_2930", "context": "Tania: A good art critic is not fair in the ordinary sense; it is only about things that do not interest one that one can give a truly unbiased opinion. Since art is a passion, good criticism of art cannot be separated from emotion. Monique: Art is not simply a passion. The best art critics passionately engage with the artwork, but render their criticism only after shedding all of their biases and consulting general principles of aesthetics.", "question": "The dialogue most strongly supports the claim that Tania and Monique disagree about whether", "answers": "['art critics should not feel emotion toward artworks', 'fairness generally requires minimizing the influence of bias', 'good art criticism is sometimes unbiased', 'art is not simply a passion']", "label": 2 }, { "id": "train_2931", "context": "A politician can neither be reelected nor avoid censure by his or her colleagues if that politician is known to be involved in any serious scandals. Several prominent politicians have just now been shown to be involved in a conspiracy that turned into a serious scandal. These politicians will ttherefore not be reelected.", "question": "If the statements above are all true, which one of the following statements must also be true?", "answers": "['No politician is censured unless he or she is known to be involved in a serious scandal.', 'The prominent politicians cannot escape censure by their colleagues.', 'The prominent politicians initially benefited from the conspiracy that caused the scandal.', 'Some politicians who are involved in scandalous conspiracies avoid detection and censure.']", "label": 1 }, { "id": "train_2932", "context": "Kim: In northern Europe during the eighteenth century a change of attitude occurred that found expression both in the adoption of less solemn and elaborate death rites by the population at large and in a more optimistic view of the human condition as articulated by philosophers. This change can be explained as the result of a dramatic increase in life expectancy that occurred in northern Europe early in the eighteenth century. Lee: Your explanation seems unlikely, because it could not be correct unless the people of the time were aware that their life expectancy had increased.", "question": "Which one of the following, if true, provides the strongest defense of Kim's explanation against Lee's criticism?", "answers": "['The concept of life expectancy is based on statistical theories that had not been developed in the eighteenth century.', \"An increase in life expectancy in a population often gives rise to economic changes that, in turn, directly influence people's attitudes.\", 'Philosophers in northern Europe during the eighteenth century made many conjectures that did not affect the ideas of the population at large.', \"Present-day psychologists have noted that people's attitudes toward life can change in response to information about their life expectancy.\"]", "label": 1 }, { "id": "train_2933", "context": "John of Worcester, an English monk, recorded the sighting, on December 8, 1128, of two unusually large sunspots. Five days later a brilliant aurora borealis (northern lights) was observed in southern Korea. Sunspot activity is typically followed by the appearance of an aurora borealis, after a span of time that averages five days. Thus, the Korean sighting helps to confirm John of Worcester' s sighting.", "question": "Which one of the following, if true, most strengthens the argument?", "answers": "['Chinese sources recorded the sighting of sunspots more than 1000 years before John of Worcester did.', 'An aurora borealis can sometimes occur even when there has been no significant sunspot activity in the previous week.', 'Only heavy sunspot activity could have resulted in an aurora borealis viewable at a latitude as low as that of Korea.', 'Because it is impossible to view sunspots with the naked eye under typical daylight conditions, the sighting recorded by John of Worcester would have taken place under unusual weather conditions such as fog or thin clouds.']", "label": 2 }, { "id": "train_2934", "context": "Most land-dwelling vertebrates have rotating limbs terminating in digits, a characteristic useful for land movement. Biologists who assume that this characteristic evolved only after animals abandoned aquatic environments must consider the Acanthostega, a newly discovered ancestor of all land vertebrates. It possessed rotating limbs terminating in digits, but its skeleton was too feeble for land movement. It also breathed using only internal gills, indicating that it and its predecessors were exclusively aquatic.", "question": "The statements above, if true, most strongly support which one of the following?", "answers": "['Certain anatomical characteristics common to some aquatic animals represent an advantage for survival on land.', 'None of the anatomical characteristics common to most aquatic animals represent an advantage for survival on land.', 'Many anatomical characteristics common to most land animals represent a disadvantage for survival underwater.', 'Acanthostega originated as a land-dwelling species, but evolved gills only after moving to an underwater environment.']", "label": 0 }, { "id": "train_2935", "context": "Manufacturers issue cents-off coupons to get consumers to try their brand of product with the hope that the consumers who try their brand will switch their brand loyalty. So in the initial marketing of their new brand X, Hartman Industries should issue cents-off coupons, thereby attracting a large segment of potential consumers as loyal customers.", "question": "Which of the following, if true, casts the most serious doubt on the likelihood that the marketing strategy recommended above will have the result that is claimed?", "answers": "['A marketing campaign that uses cents-off coupons is most effective when combined with a television advertising campaign.', 'Typically less than one-third of the coupons issued by a manufacturer are redeemed by consumers.', \"Many grocery stores attract customers by doubling the face value of manufacturer's coupons.\", 'The consumers whose purchases are strongly influenced by cents-off coupons tend not to become loyal customers of any particular brand.']", "label": 3 }, { "id": "train_2936", "context": "Physics professor: Some scientists claim that superheated plasma in which electrical resistance fails is a factor in causing so-called \"ball lightning. \" If this were so, then such lightning would emit intense light and, since plasma has gaslike properties, would rise in the air. However, the instances of ball lightning that I observed were of low intensity and floated horizontally before vanishing. Thus, superheated plasma with failed electrical resistance is never a factor in causing ball lightning.", "question": "The physics professor's conclusion follows logically if which one of the following is assumed?", "answers": "['All types of ball lightning have the same cause.', 'The phenomena observed by the physics professor were each observed by at least one other person.', 'Ball lightning can occur as the result of several different factors.', 'Superheated plasma in which electrical resistance fails does not cause types of lightning other than ball lightning.']", "label": 0 }, { "id": "train_2937", "context": "Economist: Markets work most efficiently without any government interference, since competition increases in free markets. Government regulation will never achieve its intended goal, since the most sophisticated corporations will always be able to game the system at the expense of the start-ups that are necessary to spur growth. Competition between corporations also forces those entities to self-regulate and it protects the interests of consumers. Politician: Unregulated markets are ripe for abuse. Under the current regulatory scheme, a handful of corporations dominate the marketplace. Vertical integration, under the umbrella of a larger corporation, expands a single corporation' s power across multiple economic sectors. We need to increase regulations to disrupt this integration and allow start-ups to compete on a level playing field.", "question": "What is the main issue in dispute between the economist and politician?", "answers": "['Increasing regulations will increase competitiveness.', \"Competition is important for a nation's economic health.\", \"Corporations cannot be trusted to act in consumers' best interest.\", 'Consolidating economic power in a handful of corporations is healthy.']", "label": 0 }, { "id": "train_2938", "context": "Many nurseries sell fruit trees that they label \"miniature. \" Not all nurseries, however, use this term in the same way. While some nurseries label any nectarine trees of the Stark Sweet Melody variety as \"miniature, \" for example, others do not. One thing that is clear is that if a variety of fruit tree is not suitable for growing in a tub or a pot, no tree of that variety can be correctly labeled \"miniature. ", "question": "Which one of the following can be properly inferred from the information above?", "answers": "['Any nectarine tree of the Stark Sweet Melody variety that a nursery labels \"miniature\" is labeled incorrectly.', 'Unless the Stark Sweet Melody variety of nectarine tree is suitable for growing in a tub or a pot, some nurseries mislabel this variety of tree.', 'Some nectarine trees that are not labeled \"miniature\" are labeled incorrectly.', 'Some of the nurseries have correctly labeled nectarine trees of the Stark Sweet Melody variety only if the variety is unsuitable for growing in a tub or a pot.']", "label": 1 }, { "id": "train_2939", "context": "Studies have shown that, contrary to popular belief, middle-aged people have more fear of dying than do elderly people.", "question": "Each of the following, if true, contributes to an explanation of the phenomenon shown by the studies EXCEPT:", "answers": "['The longer one lives, the more imperturbable one becomes.', 'Middle-aged people have more people dependent upon them than people of any other age group.', 'Many people who suffer from depression first become depressed in middle age.', 'The longer one lives, the more likely it is that one has come to terms with dying.']", "label": 2 }, { "id": "train_2940", "context": "Insect infestations in certain cotton-growing regions of the world have caused dramatic increases in the price of cotton on the world market. By contrast, the price of soybeans has long remained stable. Knowing that cotton plants mature quickly, many soybean growers in Ortovia plan to cease growing soybeans and begin raising cotton instead, thereby taking advantage of the high price of cotton to increase their income significantly, at least over the next several years.", "question": "Which of the following, if true, most seriously weakens the plan's chances for success?", "answers": "['Tests of a newly developed, inexpensive pesticide have shown it to be both environmentally safe and effective against the insects that have infested cotton crops.', 'The species of insect that has infested cotton plants has never been known to attack soybean plants.', 'The cost of raising soybeans has increased significantly over the past several years and is expected to continue to climb.', 'Few consumers would be willing to pay significantly higher prices for cotton goods than they are now paying.']", "label": 0 }, { "id": "train_2941", "context": "Music critic: Fewer and fewer musicians are studying classical music, decreasing the likelihood that those with real aptitude for such music will be performing it. Audiences who hear these performances will not appreciate classical music' s greatness and will thus decamp to other genres. So to maintain classical music' s current meager popularity, we must encourage more young musicians to enter the field.", "question": "Which of the following, if true, most weakens the music critic's reasoning?", "answers": "['It is possible to enjoy the music in a particular genre even when it is performed by musicians who are not ideally suited for that genre.', 'Musicians who choose to study classical music do so because they believe they have an aptitude for the. music.', 'The continued popularity of a given genre of music depends in part on the audiences being able to understand why that genre attained its original popularity.', 'Most people who appreciate classical music come to do so through old recordings rather than live performances.']", "label": 3 }, { "id": "train_2942", "context": "Incumbent politicians tend to win elections in times of economic prosperity and lose during recessions. Knowing this, incumbents have an incentive to boost the economy temporarily by cutting taxes and raising government spending several months before an election. Thus, in democratic nations, upcoming elections may be a major cause of short-term economic expansions.", "question": "Each of the following, if true, strengthens the argument EXCEPT:", "answers": "['In democratic nations, incumbent politicians have more control than anyone else over decisions about when taxes will be cut and government spending raised.', 'Cutting taxes several months before an election is an effective means for incumbent politicians to boost the economy temporarily.', 'In democratic nations, short-term economic expansions more often start within the six months preceding a national election than within the six months following one.', 'Politicians in democratic nations sometimes cut taxes and raise government spending in the months following an election.']", "label": 3 }, { "id": "train_2943", "context": "Large deposits of the rare mineral nahcolite formed in salty lakes 50 million to 52 million years ago during the Eocene epoch. Laboratory tests found that, in salty water, nahcolite can form only when the atmosphere contains at least 1, 125 parts per million of carbon dioxide.", "question": "The statements above, if true, most strongly support which one of the following?", "answers": "['For most of the time since the Eocene epoch, the level of carbon dioxide in the atmosphere has been lower than it was during most of the Eocene epoch.', 'The atmosphere contained at least 1, 125 parts per million of carbon dioxide during at least some part of the Eocene epoch.', 'No significant deposits of nahcolite have formed at any time since the Eocene epoch.', 'Levels of carbon dioxide in the atmosphere fluctuated greatly during the Eocene epoch.']", "label": 1 }, { "id": "train_2944", "context": "Some people take their moral cues from governmental codes of law; for them, it is inconceivable that something that is legally permissible could be immoral.", "question": "Those whose view is described above hold inconsistent beliefs if they also believe that", "answers": "['a legally impermissible action is never morally excusable', 'some governmental regulations are so detailed that they are burdensome to the economy', 'governmental officials sometimes behave illegally', 'law does not cover all circumstances in which one person morally wrongs another']", "label": 3 }, { "id": "train_2945", "context": "Mayor: The law prohibiting pedestrians from crossing against red lights serves no useful purpose. After all, in order to serve a useful purpose, a law must deter the kind of behavior it prohibits. But pedestrians who invariably violate this law are clearly not dissuaded by it; and those who comply with the law do not need it, since they would never cross against red lights even if there were no law prohibiting pedestrians from crossing against red lights.", "question": "The mayor's argument is flawed because it", "answers": "['fails to consider whether the law ever dissuades people who sometimes but not always cross against red lights', 'takes for granted that most automobile drivers will obey the law that prohibits them from driving through red lights', 'uses the word \"law\" in one sense in the premises and in another sense in the conclusion', 'provides no evidence that crossing against red lights is more dangerous than crossing on green lights']", "label": 0 }, { "id": "train_2946", "context": "Expert: What criteria distinguish addictive substances from nonaddictive ones? Some have suggested that any substance that at least some habitual users can cease to use is nonaddictive. However, if this is taken to be the sole criterion of nonaddictiveness, some substances that most medical experts classify as prime examples of addictive substances would be properly deemed nonaddictive. Any adequate set of criteria for determining a substance' s addictiveness must embody the view, held by these medical experts, that a substance is addictive only if withdrawal from its habitual use causes most users extreme psychological and physiological difficulty.", "question": "Which one of the following can be properly inferred from the expert's statements?", "answers": "['A substance that some habitual users can cease to use with little or no psychological or physiological difficulty is addictive only if that is not true for most habitual users.', 'Fewer substances would be deemed addictive than are deemed so at present if an adequate definition of \"addictive\" were employed.', 'If a person experiences extreme psychological and physiological difficulty in ceasing to use a substance habitually, that substance is addictive.', '\"Addiction\" is a term that is impossible to define with precision.']", "label": 0 }, { "id": "train_2947", "context": "Industrial adviser: If two new processes under consideration are not substantially different in cost, then the less environmentally damaging process should be chosen. If, however, a company already employs an environmentally damaging process and retooling for a less damaging process would involve substantial cost, then that company should retool only if retooling is either legally required or likely to bring long-term savings substantially greater than the cost.", "question": "Which one of the following judgments conforms most closely to the principles described by the industrial adviser?", "answers": "['In manufacturing pincushions, a company uses a process that, though legal, has come under heavy public criticism for the environmental damage it causes. The company should change its process to preserve its public image, despite some expected long-term losses from doing so.', 'A company is considering two new processes for the manufacture of staples. Process A is more expensive than process B but not substantially so. However, process A is substantially less environmentally damaging than process B. The company should implement process A.', 'Two new processes are being considered for the manufacture of ball bearings. The processes are similar, except that the chemicals used in process A will pollute a nearby river slightly more than will the chemicals for process B. Process A is also slightly cheaper than process B. The company should use process A.', 'A company is considering changing its process for manufacturing shoelaces. The new process is cheaper and less environmentally damaging than the old. Both are legal. Changing processes would be costly, but the cost would be almost entirely recovered in long-term savings. The company should switch processes.']", "label": 1 }, { "id": "train_2948", "context": "The Earth's rivers constantly carry dissolved salts into its oceans. Clearly, ttherefore, by taking the resulting increase in salt levels in the oceans over the past hundred years and then determining how many centuries of such increases it would have taken the oceans to reach current salt levels from a hypothetical initial salt-free state, the maximum age of the Earth's oceans can be accurately estimated.", "question": "Which of the following is an assumption on which the argument depends?", "answers": [ "There are salts that leach into the Earth's oceans directly from the ocean floor", "There is no method superior to that based on salt levels for estimating the maximum age of the Earth's oceans", "None of the salts carried into the Earth's oceans by rivers are used up by biological activity in the oceans", "The quantities of dissolved salts deposited by rivers in the Earth's oceans have not been unusually large during the past hundred years" ], "label": 3 }, { "id": "train_2949", "context": "Climatologist: Over the coming century, winter temperatures are likely to increase in the Rocky Mountains due to global warming. This will cause a greater proportion of precipitation to fall as rain instead of snow. Ttherefore, the mountain snowpack will probably melt more rapidly and earlier in the season, leading to greater spring flooding and less storable water to meet summer demands.", "question": "Which one of the following, if true, most strengthens the climatologist's argument?", "answers": "['Global warming will probably cause a substantial increase in the average amount of annual precipitation in the Rocky Mountains over the coming century.', 'On average, in the regions of the world with the mildest winters, there is more spring flooding and less storable water than in regions of the world with much colder winters.', 'The larger a mountain snowpack is, the greater the amount of spring flooding it is likely to be responsible for producing.', 'In other mountainous regions after relatively mild winters, the melting of snowpacks has led to greater spring flooding and less storable water, on average, than in those mountainous regions after colder winters.']", "label": 3 }, { "id": "train_2950", "context": "Although fullerenes--spherical molecules made entirely of carbon--were first found in the laboratory, they have since been found in nature, formed in fissures of the rare mineral shungite. Since laboratory synthesis of fullerenes requires distinctive conditions of temperature and pressure, this discovery should give geologists a test case for evaluating hypotheses about the state of the Earth' s crust at the time these naturally occurring fullerenes were formed.", "question": "Which of the following, if true, most seriously undermines the argument?", "answers": "['Some fullerenes have also been found on the remains of a small meteorite that collided with a spacecraft.', 'The naturally occurring fullerenes are arranged in a previously unknown crystalline structure.', 'Confirming that the shungite genuinely contained fullerenes took careful experimentation.', 'Shungite itself is formed only under distinctive conditions.']", "label": 1 }, { "id": "train_2951", "context": "Beads were used as currency for centuries. But why beads? The striking fact about many objects that have been used as currency-gold, silver, and feathers, to name a few-is that they were first used mainly, and often solely, as decorative objects. Thus, it is natural that beads, which were initially valued as objects of adornment, also came to be used as currency.", "question": "Which one of the following principles, if valid, would most help to justify the argument above?", "answers": "['The similarity between the primary uses of two different objects can cause the secondary use of one to be transferred to the other.', 'An object cannot take on a derivative use unless it ceases to have its original use.', 'An object having a certain original use is likely to have the same derivative use as do other objects having that original use.', 'The similarity between the secondary uses of two different objects can cause the primary use of one to be transferred to the other.']", "label": 2 }, { "id": "train_2952", "context": "Animals generally avoid foods that are harmful to them and favor those that benefit them. Thus it is surprising that gypsy moth caterpillars prefer to eat leaves that contain high concentrations of certain naturally occurring toxins called alkaloids, even though caterpillars grow faster on alkaloid-free diets.", "question": "Which one of the following, if true, most helps to resolve the apparent discrepancy?", "answers": "['In the alkaloid-containing plants on which gypsy moth caterpillars feed, the toxins are found only in the leaves.', 'The alkaloid-containing plants on which gypsy moth caterpillars feed increase their production of toxins when their leaves are injured.', 'Only gypsy moth caterpillars, not adult gypsy moths, feed on the leaves of alkaloid-containing plants.', \"Alkaloids that have been eaten by caterpillars are poisonous to parasitic wasp larvae that feed on caterpillars' internal tissues.\"]", "label": 3 }, { "id": "train_2953", "context": "Commentator: Human behavior cannot be fully understood without inquiring into nonphysical aspects of persons. As evidence of this, I submit the following: suppose that we had a complete scientific account of the physical aspects of some particular human action -- every neurological, physiological, and environmental event involved. Even with all that we would obviously still not truly comprehend the action or know why it occurred.", "question": "Which one of the following most accurately describes a flaw in the argument's reasoning?", "answers": "['It fails to indicate whether the speaker is aware of any evidence that could undermine the conclusion.', 'No support is offered for its conclusion other than an analogy that relates only superficially to the issue at hand.', 'The purported evidence that it cites in support of its conclusion presumes that the conclusion is true.', 'It concludes that a proposition must be true merely on the grounds that it has not been proven false.']", "label": 2 }, { "id": "train_2954", "context": "For the next year, the Chefs' Union has requested a 10 percent salary increase for each of its members, whereas the Hotel Managers' Union has requested only an 8 percent salary increase for each of its members. These facts demonstrate that the average dollar amount of the raises that the Chefs' Union has requested for next year is greater than that of the raises requested by the Hotel Managers' Union.", "question": "Which one of the following, if true, most strengthens the argument?", "answers": [ "The average dollar amount of the raises that the members of the Chefs' Union received last year was equal to the average dollar amount of the raises that the members of the Hotel Managers' Union received.", "The Chefs' Union has many more members than does the Hotel Managers' Union.", "The Chefs' Union is a more powerful union than is the Hotel Managers' Union and is ttherefore more likely to obtain the salary increases it requests.", "The current salaries of the members of the Chefs' Union are, on average, higher than the current salaries of the members of the Hotel Managers' Union." ], "label": 3 }, { "id": "train_2955", "context": "Highway Official: When resurfacing our concrete bridges, we should use electrically conductive concrete (ECC) rather than standard concrete. In the winter, ECC can be heated by passing an electric current through it, thereby preventing ice buildup. The cost of the electricity needed is substantially lower than the cost of the de-icing salt we currently use. Taxpayer: But construction costs for ECC are much higher than for standard concrete, so your proposal is probably not justifiable on economic grounds.", "question": "Which of the following, if true, could best be used to support the highway official's proposal in the face of the taxpayer's objection?", "answers": "['The use of de-icing salt causes corrosion of the reinforcing steel in concrete bridge decks and damage to the concrete itself, thereby considerably shortening the useful life of concrete bridges.', 'Aside from its potential use for de-icing bridges, ECC might also be an effective means of keeping other concrete structures such as parking garages and airport runways ice free.', 'In weather conditions conducive to icing, ice generally forms on the concrete surfaces of bridges well before it forms on parts of the roadway that go over solid ground.', 'Severe icing conditions can cause power outages and slow down the work of emergency crews trying to get power restored.']", "label": 0 }, { "id": "train_2956", "context": "The Venetian Renaissance painter Vittore Carpaccio used sumptuous reds in most of his paintings. Since the recently discovered Venetian Renaissance painting Erato Declaiming contains notable sumptuous reds, it is probably by Carpaccio.", "question": "Which one of the following contains a pattern of flawed reasoning most similar to that in the argument above?", "answers": "['Since late-Renaissance paintings were mostly in oil, the Venetian late-Renaissance painter Arnoldi, whose works are now lost, probably painted in oil.', \"Most Renaissance painters worked in a single medium, either tempera or oil. Since the Renaissance painting Calypso's Bower is in oil, its painter probably always used oil.\", 'The anonymous painting St. Sebastian is probably an early Florentine painting since it is in tempera, and most early Florentine paintings were in tempera.', 'Works of art in the Renaissance were mostly commissioned by patrons, so the Renaissance work The Dances of Terpsichore was probably commissioned by a patron.']", "label": 2 }, { "id": "train_2957", "context": "An energy company proposes placing 250 giant turbines into the Gulf Stream to generate electricity for North America. Some Europeans worry, however, about the potential threat to their climate. Without the warming effects of the Gulf Stream, much of Europe would be as cold as Labrador and the Yukon, areas at the same latitude that are unaffected by warming currents. However, their concern is unwarranted: the company claims that 250 turbines would slow the Gulf Stream about 1 percent, which is not enough to affect the European climate.", "question": "Which one of the following most accurately describes a flaw in the reasoning of the argument?", "answers": "['The argument ignores the potential threat to marine life posed by placing turbines in the ocean.', 'The argument relies on an authority that may be biased.', 'The conclusion of the argument contradicts at least one of its premises.', 'The argument presumes, without providing justification, that latitude and temperature are linked.']", "label": 1 }, { "id": "train_2958", "context": "Only a very small percentage of people from the service professions ever become board members of the 600 largest North American corporations. This shows that people from the service professions are underrepresented in the most important corporate boardrooms in North America.", "question": "Which one of the following points out a flaw committed in the argument?", "answers": "['Six hundred is too small a sample on which to base so sweeping a conclusion about the representation of people from the service professions.', 'It is a mistake to take the 600 largest North American corporations to be typical of corporate boardrooms generally.', 'The percentage of people from the service professions who serve on the boards of the 600 largest North American corporations reveals little about the percentage of the members of these boards who are from the service professions.', 'It is irrelevant to smaller corporations whether the largest corporations in North America would agree to have significant numbers of workers from the service professions on the boards of the largest corporations.']", "label": 2 }, { "id": "train_2959", "context": "The kinds of hand and wrist injuries that result from extended use of a computer while maintaining an incorrect posture are common among schoolchildren in Harnville. Computers are important to the school curriculum there, so instead of reducing the amount their students use computers, teachers plan to bring about a sharp reduction in the number of these injuries by carefully monitoring their students' posture when using computers in the classroom.", "question": "Which of the following would it be most useful to know in order to assess the likelihood that the teachers' plan will be successful?", "answers": "['What proportion of schoolchildren in Harnville with hand and wrist injuries use computers extensively outside the classroom', \"Whether changes in the curriculum could reduce the schools' dependence on computers\", 'What proportion of schoolchildren in Harnville already use correct posture while using a computer', 'Whether hand and wrist injuries not caused by computer use are common among schoolchildren in Harnville']", "label": 0 }, { "id": "train_2960", "context": "Last August the XT chain of gasoline stations had a temporary sales promotion in effect. In the promotion , any customer who made a purchase of ten or more gallons of gasoline was entitled to a free car wash . For the month of August , XT experienced a ten percent increase in gasoline sales as compared to sales in August the previous year so evidently the promotion was successful as a means of boosting sales.", "question": "In evaluating the argument , it would be most helpful to answer which of the following ?", "answers": "['Did XT or any of its gasoline stations have to pay other businesses to provide the car washes that customers were offered in the promotion?', \"Are XT's gasoline sales in August usually significantly higher than one twelfth of XT's annual gasoline sales ?\", \"In the areas in which XT's gasoline stations operate , how did total combined gasoline sales for all gasoline stations last August compare with sales for the previous August?\", 'Was the money that XT earned from the increase in gasoline sales enough to offset the cost of providing free car washes during the promotion?']", "label": 2 }, { "id": "train_2961", "context": "Because some student demonstrations protesting his scheduled appearance have resulted in violence, the president of the Imperialist Society has been prevented from speaking about politics on campus by the dean of student affairs. Yet to deny anyone the unrestricted freedom to speak is to threaten everyone' s right to free expression. Hence, the dean' s decision has threatened everyone' s right to free expression.", "question": "The pattern of reasoning displayed above is most closely paralleled in which one of the following?", "answers": "[\"In order to rescue a drowning child, Isabel jumped into a freezing river. Such acts of heroism performed to save the life of one enrich the lives of all. Hence, Isabel's action enriched the lives of all.\", 'Fire fighters are often expected to perform heroically under harsh conditions. But no one is ever required to act heroically. Hence, fire fighters are often expected to perform actions they are not required to perform.', 'Because anyone who performs an act of heroism acts altruistically rather than selfishly, a society that rewards heroism encourages altruism rather than pure self-interest.', \"Dr. Pacheco saved a child's life by performing emergency surgery. But surgery rarely involves any risk to the surgeon. Ttherefore, if an act is not heroic unless it requires the actor to take some risk, Dr. Pacheco's surgery was not heroic.\"]", "label": 0 }, { "id": "train_2962", "context": "A study followed a group of teenagers who had never smoked and tracked whether they took up smoking and how their mental health changed. After one year, the incidence of depression among those who had taken up smoking was four times as high as it was among those who had not. Since nicotine in cigarettes changes brain chemistry, perhaps thereby affecting mood, it is likely that smoking contributes to depression in teenagers.", "question": "Which of the following, if true, most strengthens the argument?", "answers": "['Few, if any, of the participants in the study were friends or relatives of other participants.', 'Participants who were depressed at the start of the study were no more likely to be smokers after one year than those who were not depressed.', 'Some participants entered and emerged from a period of depression within the year of the study.', 'The study did not distinguish between participants who smoked only occasionally and those who were heavy smokers.']", "label": 1 }, { "id": "train_2963", "context": "In trying to reduce the amount of fat in their diet, on average people have decreased their consumption of red meat by one-half in the last two decades. However, on average those who have reduced their consumption of red meat actually consume substantially more fat than those who have not.", "question": "Which one of the following, if true, most helps to resolve the apparent discrepancy described above?", "answers": "['Higher prices over the last two decades have done as much to decrease the consumption of red meat as health concerns have.', 'People who reduce their consumption of red meat tend to replace it with cheese and baked goods, which are richer in fat than red meat.', 'People who reduce their consumption of red meat tend to consume as much of other foods that are high in fat as do those who have not reduced their consumption of red meat.', 'Many more people have reduced their consumption of red meat over the last two decades than have not.']", "label": 1 }, { "id": "train_2964", "context": "City dog licensing records show that more cocker spaniels are registered to addresses in the Flynn Heights neighborhood than to addresses in all other neighborhoods combined. So if an animal control officer finds a stray cocker spaniel anywhere near Flynn Heights, it is likely that the dog belongs to someone in Flynn Heights.", "question": "Which one of the following would be most useful to know in order to evaluate the argument?", "answers": "['whether the number of pets owned, per capita, is greater for residents of Flynn Heights than for residents of any other neighborhood', 'whether there are more cocker spaniels registered to addresses in Flynn Heights than any other breed of dog', \"whether the city's animal control officers find more stray dogs in and around Flynn Heights than in any other part of the city\", 'whether residents of Flynn Heights are more likely to license their dogs than residents of other neighborhoods are']", "label": 3 }, { "id": "train_2965", "context": "Melinda: Hazard insurance decreases an individual' s risk by judiciously spreading the risk among many policyholders. Jack: I disagree. It makes sense for me to buy fire insurance for my house, but I don' t see how doing so lessens the chances that my house will burn down.", "question": "Jack's response most clearly trades on an ambiguity in which one of the following expressions used by Melinda?", "answers": "['judiciously spreading', 'risk', 'hazard insurance', 'many policyholders']", "label": 1 }, { "id": "train_2966", "context": "Museum curator: Our ancient Egyptian collection includes an earthenware hippopotamus that resembles a child' s toy. It was discovered in a tomb, upside down, with its legs broken off. We know that the ancient Egyptians believed the dead had to wage eternal war with beasts. Breaking the legs off a representation of an animal was thought to help a deceased person in this war. We conclude that, far from being a toy, this hippopotamus was a religious object.", "question": "Which one of the following is an assumption required by the curator's argument?", "answers": "[\"Earthenware figures were never used as children's toys in ancient Egypt.\", \"The hippopotamus' legs were not broken through some natural occurrence after it was placed in the tomb.\", 'The tomb in which the hippopotamus was found was not reentered from the time of burial until archaeologists opened it.', 'The hippopotamus was originally placed upside down in the tomb.']", "label": 1 }, { "id": "train_2967", "context": "Megatrash Co. , the country' s largest waste-disposal company, has been sued by environmental groups who have accused the firm of negligent handling of hazardous waste. The fines and legal fees that have resulted from the legal attacks against Megatrash have cost the company substantial amounts of money. Surprisingly, as successful lawsuits against the company have increased in number, the company has grown stronger and more profitable.", "question": "Which one of the following, if true, does the most to resolve the apparent paradox?", "answers": "['Some of the judgments against Megatrash have legally barred it from entering the more profitable areas of the waste-management business.', 'In cases in which Megatrash has been acquitted of charges of negligence, the company has paid more in legal fees than it would have been likely to pay in fines.', 'Although waste-disposal firms merely handle but do not generate toxic waste, these firms have been held legally responsible for environmental damage caused by this waste.', \"The example of Megatrash's legal entanglements has driven most of the company's competitors from the field and deterred potential rivals from entering it.\"]", "label": 3 }, { "id": "train_2968", "context": "Politician: A government that taxes incomes at a rate of 100 percent will generate no revenue because all economic activity will cease. So it follows that the lower the rate of income tax, the more revenue the government will generate by that tax. Economist: Your conclusion cannot be correct, since it would mean that an income tax of 0 percent would generate the maximum revenue.", "question": "Which one of the following argumentative strategies is used by the economist in responding to the politician?", "answers": "[\"undermining the credibility of the politician by openly questioning the politician's understanding of economics\", \"attacking the politician's argument by giving reason to doubt the truth of a premise\", \"providing evidence that where the politician's advice has been adopted, the results have been disappointing\", 'arguing that the principle derived by the politician, if applied in the limiting case, leads to an absurdly false conclusion']", "label": 3 }, { "id": "train_2969", "context": "Terrorism aims to instill fear in the target population, disrupting all daily activities and forcing an irrational backlash. More people die in car accidents in a day than terrorists have killed in two decades. Our country spends more money on fighting terrorism than any other single initiative, including healthcare. As such, our country should. . .", "question": "Which one of the following most logically completes the argument?", "answers": "[\"spend money in proportion to terrorism's threat.\", 'spend money in proportion to the fear terrorism inspires.', 'spend more money on healthcare.', 'spend more money on reducing car accidents.']", "label": 0 }, { "id": "train_2970", "context": "The Z-300 is a particularly fast new sports car, popular among young drivers. It is an expensive model, and relatively few are on the roads. Two months ago, seven different young people in Catoville were arrested for operating the Z-300 at excessive speeds on local roads while having a blood alcohol level well above the legal limit. Since then, a police watch group has reported that Catoville traffic cops frequently have been pulling over drivers of the Z-300 and testing them for alcohol: the total number of Z-300 drivers stopped and tested is five times greater than the total number of drivers of all other makes or models stopped and tested.", "question": "Which of following conclusions can most properly be drawn from the information above?", "answers": "['The Catoville traffic cops have provided an effective deterrent to drunk driving within Catoville.', 'Some drivers of the Z-300 are unfairly subjected to delays.', 'Drivers of the Z-300 exceed local speed limits by much higher margins than that by which they exceed highway speed limits.', 'Drivers of fast sports cars are more likely to drive under the influence of alcohol than drivers of any other make or model of car.']", "label": 1 }, { "id": "train_2971", "context": "Recently, a report commissioned by a confectioners trade association noted that chocolate, formerly considered a health scourge, is an effective antioxidant and so has health benefits. Another earlier claim was that oily foods clog arteries, leading to heart disease, yet reports now state that olive oil has a positive influence on the circulatory system. From these examples, it is clear that if you wait long enough, almost any food will be reported to be healthful.", "question": "The reasoning in the argument is flawed in that the argument", "answers": "['bases an overly broad generalization on just a few instances', 'fails to consider that there are many foods that are reported to be unhealthful', 'relies on the truth of a claim by a source that is likely to be biased', 'applies a general rule to specific cases to which it does not pertain']", "label": 0 }, { "id": "train_2972", "context": "Any ornamental garden has both plants and structural elements such as rocks and fences. Because the plants constantly change with growth and decay, it is difficult to keep a proper balance in the garden between the plants and the structures. Balance can be achieved only with careful planning, and even when established, balance has to be constantly maintained by pruning and thinning.", "question": "Which one of the following most accurately expresses the main conclusion of the argument?", "answers": "['The reason why it is difficult to keep a proper balance in a garden between structures and plants is that the plants constantly change with growth and decay.', 'It is difficult to keep a proper balance in a garden between structures and plants.', 'Without careful planning and maintenance, the balance in a garden between structures and plants constantly changes with growth and decay.', 'It is difficult to constantly maintain the balance in a garden between structures and plants by pruning and thinning.']", "label": 1 }, { "id": "train_2973", "context": "A new computer system will not significantly increase an organization' s efficiency unless the computer system requires the organization' s employees to adopt new, more productive ways of working. The Ministry of Transportation is having a new computer system custom built to fit the ministry' s existing ways of working, so__", "question": "Which one of the following most logically completes the argument?", "answers": "['the new computer system will be worthwhile if it automates many processes that are currently performed manually', 'it will be easy for employees of the Ministry of Transportation to learn to use the new computer system', 'the new computer system will not increase the efficiency of the Ministry of Transportation to any appreciable degree', \"the leaders of the Ministry of Transportation must not be concerned with the productivity of the ministry's employees\"]", "label": 2 }, { "id": "train_2974", "context": "The cotton farms of Country Q became so productive that the market could not absorb all that they produced. Consequently, cotton prices fell. The government tried to boost cotton prices by offering farmers who took 25 percent of their cotton acreage out of production direct support payments up to a specified maximum per farm.", "question": "The government's program, if successful, will not be a net burden on the budget. Which of the following, if true, is the best basis for an explanation of how this could be so?", "answers": "['Depressed cotton prices meant operating losses for cotton farms, and the government lost revenue from taxes on farm profits.', 'The first year that the support-payment program was in effect, cotton acreage in Q was 5% below its level in the base year for the program.', 'Cotton production in several countries other than Q declined slightly the year that the support-payment program went into effect in Q.', 'Farmers who wished to qualify for support payments could not use the cotton acreage that was withdrawn from production to grow any other crop.']", "label": 0 }, { "id": "train_2975", "context": "Why should the disappearance of beetles, plants, or birds concern us? First, a species, the irreplaceable product of millions of years of development, is of intrinsic value. Another, perhaps more compelling, reason to conserve biological diversity is pure self-interest. Like every species, ours is intimately dependent on others for its well-being. Time after time, creatures, even those once thought useless or harmful, are found to play crucial roles in natural systems. Predators driven to extinction no longer keep populations of potential pests in check; earthworms or termites killed by pesticides no longer aerate soils; mangroves cut for firewood no longer protect coastlines from the erosive force of the sea.", "question": "The above argument as a whole seeks to establish that", "answers": "['there are no species in nature that are, properly considered, harmful to people', 'it is only selfish people who wish to conserve biological diversity', 'whether species to which people pay little attention become extinct is not of great importance except to scientists', 'species of plant and animal life should be preserved because they are valuable both in themselves and for the well-being of people']", "label": 3 }, { "id": "train_2976", "context": "In opposing the 1970 Clean Air Act, the United States automobile industry argued that meeting the act' s standards for automobile emissions was neither economically feasible nor environmentally necessary. However, the catalytic converter, invented in 1967, enabled automakers to meet the 1970 standards efficiently. Currently, automakers are lobbying against the government' s attempt to pass legislation that would tighten restrictions on automobile emissions. The automakers contend that these new restrictions would be overly expensive and unnecessary to efforts to curb air pollution. Clearly, the automobile industry' s position should not be heeded.", "question": "Which one of the following most accurately expresses the method used to counter the automakers' current position?", "answers": "['Evidence is provided that the new emissions legislation is both economically feasible and environmentally necessary.', 'A flaw is pointed out in the reasoning used by the automakers to reach their conclusion.', \"A comparison is drawn between the automakers' current position and a position they held in the past.\", \"The automakers' premises are shown to lead to a contradiction.\"]", "label": 2 }, { "id": "train_2977", "context": "Exporters in Country X are facing lower revenues due to a shortage of the large metal shipping containers in which they send their goods by sea to other countries. Fewer containers arrive in Country X due to reductions in imports. This has meant lost orders, costly delays, and a scramble for alternatives, such as air freight, all of which are costlier. Moreover, the revenues of exporters in Country X will probably continue to decline in the near future. This is because other countries are likely to find it increasingly unprofitable to export their goods to Country X, and because __.", "question": "Which of the following would most logically complete the passage?", "answers": "['consumers in Country X are purchasing more products than ever before', 'the cost of shipping alternatives such as air freight is likely to stabilize in the near future', 'shipping companies are willing to move containers from country to country only when the containers are full', 'production of shipping containers in Country X is growing rapidly as a response to the shortage']", "label": 2 }, { "id": "train_2978", "context": "Most small children are flat-footed. This failure of the foot to assume its natural arch, if it persists past early childhood can sometimes result in discomfort and even pain later in life. Traditionally, flat-footedness in children has been treated by having the children wear special shoes that give extra support to the foot, in order to foster the development of the arch.", "question": "Which one of the following, if true, most calls into question the efficacy of the traditional treatment described above?", "answers": "[\"Although most children's flat-footedness is corrected by the time the children reach puberty, some people remain flat-footed for life.\", 'Many small children who have normal feet wear the same special shoes as those worn by flat-footed children.', 'Flat-footed children who do not wear the special shoes are as likely to develop natural arches as are flat-footed children who wear the special shoes.', 'Some children who are not flat-footed have hip and lower leg bones that are rotated excessively either inward or outward.']", "label": 2 }, { "id": "train_2979", "context": "Psychologist: Doctors should never prescribe sedatives for people with insomnia. Most cases of insomnia that psychologists treat are known to be caused by psychological stress. This suggests that insomniacs do not need drugs that alter their biochemistry, but rather need psychotherapy to help them alleviate the stress causing their insomnia.", "question": "Each of the following describes a flaw in the psychologist's reasoning EXCEPT:", "answers": "['It presumes, without providing warrant, that insomnia contributes to an inability to cope with stress.', 'It overlooks the possibility that sedatives could help insomniacs cope with stress.', 'It neglects the possibility that for some people psychotherapy is a completely ineffective treatment for stress.', 'It fails to consider the possibility that sedatives are the only treatment known to be effective for cases of insomnia not caused by stress.']", "label": 0 }, { "id": "train_2980", "context": "Biotechnology companies say that voluntary guidelines for their industry are sufficient to ensure that no harm will result when a genetically altered organism is released into the environment. It is foolish, however, to rely on assurances from producers of genetically altered organisms that their products will not be harmful. Ttherefore, a biotechnology company should be required to apply to an independent regulatory board composed of scientists outside the biotechnology industry for the right to sell newly created organisms.", "question": "Which one of the following principles, if accepted, most strongly justifies drawing the conclusion above?", "answers": "['People who engage in an activity and have a financial stake in that activity should not be the sole regulators of that activity.', 'Voluntary guidelines are sufficient to regulate activities that pose little danger to the environment.', 'Issues of environmental protection are so important that they should not be left to scientific experts.', 'Methods that result in harm to the environment must sometimes be used in order to avoid even greater harm.']", "label": 0 }, { "id": "train_2981", "context": "All art criticism is political because all art has political implications. Clearly, the critic who chooses to address overtly an artwork' s political implications is engaged in political discourse. But those critics who attempt a purely aesthetic evaluation of a work, and ttherefore ignore the work' s political implications, necessarily, though perhaps inadvertently, end up endorsing the politics of the artist.", "question": "If the statements above are all true, which one of the following must also be true?", "answers": "['A political critique of a work of art has more value than any other sort of critique of that work.', 'Art that makes an overt social or political statement is the sort of art with which critics are typically concerned.', 'Art criticism that is intended to evaluate a work on purely aesthetic grounds never ends up rejecting the politics of the artist whose work is under review.', 'Critics who are politically naive always attempt purely apolitical critiques of art.']", "label": 2 }, { "id": "train_2982", "context": "Claude: When I' m having lunch with job candidates, I watch to see if they salt their food without first tasting it. If they do, I count that against them, because they' re making decisions based on inadequate information. Larissa: That' s silly. It' s perfectly reasonable for me to wear a sweater whenever I go into a supermarket, because I already know supermarkets are always too cool inside to suit me. And I never open a credit card offer that comes in the mail, because I already know that no matter how low its interest rate may be, it will never be worthwhile for me.", "question": "The two analogies that Larissa offers can most reasonably be interpreted as invoking which one of the following principles to criticize Claude's policy?", "answers": "['Individuals whose behavior in specific circumstances does not conform to generally expected norms should not automatically be considered unconcerned with meeting social expectations, because such individuals may be acting in accordance with reasoned policies that they believe should be generally adopted by people in similar circumstances.', 'In matters involving personal preference, performing an action without first ascertaining whether it is appropriate in the specific circumstances should not be taken as good evidence of faulty decision making, because the action may be based on a reasoned policy relating to knowledge of a general fact about the circumstances.', 'Evidence that a particular individual uses bad decision-making strategies in matters of personal taste should not be considered sufficient to warrant a negative assessment of his or her suitability for a job, because any good decision maker can have occasional lapses of rationality with regard to such matters.', \"In professional decision-making contexts, those who have the responsibility of judging other people's suitability for a job should not use observations of job-related behavior as a basis for inferring general conclusions about those people's character.\"]", "label": 1 }, { "id": "train_2983", "context": "The last outdoor drive-in movie theater in Nova Scotia closed recently. The owners claimed that it could not regularly attract large enough audiences to remain viable. However, for an entire week -- the final week of the theater' s operation, after the announcement of the impending closure -- the place was sold out every night and made a healthy profit. Ttherefore, the owners' claim was clearly false.", "question": "Which one of the following contains an error of reasoning most similar to that made in the argument above?", "answers": "['The proposed cuts in library funding would require reducing the hours of the periodicals room. But that is a room in which many students like to study, so the proposed cuts are bound to stir up considerable opposition.', \"The administration claims that the library's funding must be cut because of last year's poor library fund drive. However the athletic department's budget is being increased. Ttherefore, the administration's claim must be false.\", \"The overall reduction in the university's budget is 10 percent. The library's budget is being cut by 10 percent. Ttherefore the library's budget cut is fair.\", 'All of the students who came to the meeting about proposed cuts in library funding were strongly opposed to the cuts, so all of the students currently enrolled must be strongly opposed.']", "label": 3 }, { "id": "train_2984", "context": "Journalist: Some critics argue that as the entertainment value of news reporting increases, the caliber of that reporting decreases. Yet the greatest journalists have been the most entertaining. So these critics are mistaken.", "question": "The journalist's conclusion is properly drawn if which one of the following is assumed?", "answers": "['The greatest journalists have been entertainers who report the news.', 'The news reporting of the greatest journalists has been of the highest caliber.', 'Journalistic greatness involves producing news that is very valuable in some sense.', 'Entertainment and news are not mutually exclusive categories.']", "label": 1 }, { "id": "train_2985", "context": "Advertisement: Ten years ago, the Cormond Hotel' s lobby was carpeted with Duratex carpet while the lobby of a nearby hotel was being carpeted with our competitor' s most durable carpet. Today, after a decade in which the two hotels have had similar amounts of foot traffic through their lobbies, that other hotel is having to replace the worn-out carpeting near its lobby entrances, whereas the Cormond' s Duratex carpeting has years of wear left in it.", "question": "Which of the following, if true, most seriously weakens the force of the advertisement's evidence for concluding that Duratex carpet is more durable than the carpet of its competitor?", "answers": "[\"The carpet that is being used to replace carpeting near the other hotel's lobby entrances is not Duratex carpet.\", 'The other hotel has a popular restaurant that can be reached from outside without walking through the hotel lobby.', 'The lobby of the Cormond Hotel has five different entrances, but the lobby of the other hotel has only two.', 'There is a third hotel near the other two that has not replaced the Duratex carpet in its lobby for more than 15 years.']", "label": 2 }, { "id": "train_2986", "context": "Economist: A tax is effective if it raises revenue and burdens all and only those persons targeted by the tax. A tax is ineffective, however, if it does not raise revenue and it costs a significant amount of money to enforce.", "question": "Which one of the following inferences is most strongly supported by the principles stated by the economist?", "answers": "['The tax on steel does not cost a significant amount of money to enforce, but it does not raise revenue either. Thus, the tax is ineffective.', 'The tax on coal burdens all of the people targeted by it, and this tax does not burden anyone who is not targeted by it. Thus, the tax is effective.', 'The tax on alcohol raises a modest amount of revenue, but it costs a significant amount of money to enforce. Thus, the tax is ineffective.', 'The tax on gasoline costs a significant amount of money to enforce. Thus, if it does not raise revenue, the tax is ineffective.']", "label": 3 }, { "id": "train_2987", "context": "In most corporations the salaries of executives are set by a group from the corporation' s board of directors. Since the board' s primary mission is to safeguard the economic health of the corporation rather than to make its executives rich, this way of setting executives' salaries is expected to prevent excessively large salaries. But, clearly, this expectation is based on poor reasoning. After all, most members of a corporation' s board are themselves executives of some corporation and can expect to benefit from setting generous benchmarks for executives' salaries.", "question": "The point made by the author is that the most common way of setting executives' salaries might not keep those salaries in bounds because", "answers": "[\"many board members might let their self- interest as executives interfere with properly discharging their role, as board members, in setting executives' salaries\", 'most corporate executives, thanks to their generous salaries, are not financially dependent on money earned as board members', \"many board members who set executives' salaries unreasonably high do so because they happen to be on the board of a corporation of which they expect later to become executives\", 'many board members are remunerated generously and wish to protect this source of income by pleasing the executives to whom they owe their appointments on the board']", "label": 0 }, { "id": "train_2988", "context": "At Law Practice, Inc. , last year the average annual salary for attorneys was $75, 000, while the average salary for paralegals was $50, 000. The average annual salary for all Law Practice employees was $45, 000.", "question": "If the information above is correct, which one of the following conclusions can properly be drawn on the basis of it?", "answers": "['There were two attorneys and three paralegals at Law Practice last year.', 'At least one paralegal made less than $50, 000 last year.', 'There were more paralegals than attorneys at Law Practice last year.', 'There was at least one Law Practice employee who earned less than the average paralegal earned last year.']", "label": 3 }, { "id": "train_2989", "context": "Editorial: It has been suggested that private, for-profit companies should be hired to supply clean drinking water to areas of the world where it is unavailable now. But water should not be supplied by private companies. After all, clean water is essential for human health, and the purpose of a private company is to produce profit, not to promote health.", "question": "Which one of the following principles, if valid, would most help to justify the reasoning in the editorial?", "answers": "['The mere fact that something actually promotes human health is not sufficient to show that its purpose is to promote health.', 'Drinking water should never be supplied by an organization that is not able to consistently supply clean, safe water.', 'If something is necessary for human health, then it should be provided by an organization whose primary purpose is the promotion of health.', 'A private company should not be allowed to supply a commodity that is essential to human health unless that commodity is also supplied by a government agency.']", "label": 2 }, { "id": "train_2990", "context": "All Labrador retrievers love playing fetch. Only German shepherds love protecting their home. Some dogs are easy to train. Brittany' s dog loves playing fetch and loves protecting her home.", "question": "Which one of the following statements must be true?", "answers": [ "Brittany 's dog is easy to train.", "Brittany's dog is half Labrador retriever and half German shepherd, and her dog is also easy to train.", "Brittany 's dog is a Labrador retriever.", "Brittany 's dog is a German shepherd." ], "label": 3 }, { "id": "train_2991", "context": "In a recent experiment, researchers concluded that the phenomenon known as runner' s high was caused by a biochemical reaction in the brain. Long-distance runners who ran long enough produced chemicals that have an opioid-like effect on mood. These runners not only felt euphoric after running, but they had a higher pain tolerance than the average person as well.", "question": "Which one of the following is most strongly supported by the information above?", "answers": "[\"The average person will not experience feelings of euphoria unless they have runner's high.\", 'The feeling of euphoria is almost always caused by the act of running.', \"Those who have runner's high will experience euphoria as well as a higher pain tolerance.\", 'Long-distance runners have a higher pain tolerance than the average person.']", "label": 2 }, { "id": "train_2992", "context": "All too many weaklings are also cowards, and few cowards fail to be fools. Thus there must be at least one person who is both a weakling and a fool.", "question": "The flawed pattern of reasoning in the argument above is most similar to that in which one of the following?", "answers": "['A majority of the voting population favors a total ban, but no one who favors a total ban is opposed to stiffer tariffs, so at least one voter is not opposed to stiffer tariffs.', 'If an act is virtuous, then it is autonomous, for acts are not virtuous unless they are free, and acts are not free unless they are autonomous.', 'All weasels are carnivores and no carnivores fail to be nonherbivores, so some weasels are nonherbivores.', 'Some painters are dancers, since some painters are musicians, and some musicians are dancers.']", "label": 3 }, { "id": "train_2993", "context": "A mechanized factory with over 3000 separate conveyor belts wants to minimize the cost associated with maintenance, repair, and replacement of these belts. The conveyer belts currently are composed vulcanized rubber around a cotton web base, but the factory owners are investigating new plastic polymer design. For any given size belt, the costs for belts of the two compositions are comparable. Compared to the current rubber/cotton belts, the plastic polymer belts are lighter and easier to move, so operating the conveyor belts made of plastic polymer would consume less electric energy. A change from rubber/cotton belts to plastic polymer belts would require minor modification to some of the drive cylinders of the conveyor belts, but the costs of these changes are negligible.", "question": "Which of the following would it be most useful to know in determining whether switching to the plastic polymer conveyor belts would be likely to help minimize the factory's cost related to its conveyor belt system?", "answers": "['Whether other mechanized factories with comparable number of conveyor belts in their systems have switched from rubber/cotton to plastic polymer conveyor belts.', 'Whether the crew that currently maintains the conveyor belts knows how to perform the modification to the drive cylinders necessary to switch to plastic polymer belts.', 'Whether a typical plastic polymer conveyor belt has an approximate serviceable life as least as long as the typical rubber/cotton belt of the same size and shape.', 'Whether, in remodeling, the factory plans to increase the total number of conveyor belts in its mechanized system.']", "label": 2 }, { "id": "train_2994", "context": "Either food scarcity or excessive hunting can threaten a population of animals. If the group faces food scarcity, individuals in the group will reach reproductive maturity later than otherwise. If the group faces excessive hunting, individuals that reach reproductive maturity earlier will come to predominate. Ttherefore, it should be possible to determine whether prehistoric mastodons became extinct because of food scarcity or human hunting, since there are fossilized mastodon remains from both before and after mastodon populations declined, and __.", "question": "Which of the following most logically completes the reasoning?", "answers": "['it is not known when humans first began hunting mastodons', 'there are more fossilized mastodon remains from the period before mastodon populations began to decline than from after that period', 'the average age at which mastodons from a given period reached reproductive maturity can be established from their fossilized remains', 'it can be accurately estimated from fossilized remains when mastodons became extinct']", "label": 2 }, { "id": "train_2995", "context": "Aroca City currently funds its public schools through taxes on property. In place of this system, the city plans to introduce a sales tax of three percent on all retail sales in the city. Critics protest that three percent of current retail sales falls short of the amount raised for schools by property taxes . The critics are correct on this point. Nevertheless, implementing the plan will probably not reduce the money going to Aroca' s schools. Several large retailers have selected Aroca City as the site for huge new stores, and these are certain to draw large numbers of shoppers from neighboring municipalities, where sales are taxed at rates of six percent and more. In consequence, retail sales in Aroca City are bound to increase substantially .", "question": "In the argument given, the two potions in boldface play which of the following roles?", "answers": "['The first is a criticism, endorsed by the argument, of a funding plan; the second is a point the argument makes in favor of adopting an alternative plan.', 'The first is an objection that has been raised against a certain plan; the second is a prediction that, if accurate, undermines the force of that objection.', 'The first is a criticism, endorsed by the argument, of a funding plan; the second is the main reason cited by the argument for its endorsement of the criticism.', 'The first is a claim that the argument accepts with certain reservations; the second presents that claim in a rewarding that is not subject to those reservations.']", "label": 1 }, { "id": "train_2996", "context": "Advertisement: At most jewelry stores, the person assessing the diamond is the person selling it, so you can see why an assessor might say that a diamond is of higher quality than it really is. But because all diamonds sold at Gem World are certified in writing, you' re assured of a fair price when purchasing a diamond from Gem World.", "question": "The reasoning in the advertisement would be most strengthened if which one of the following were true?", "answers": "['The diamonds sold at Gem World are generally of higher quality than those sold at other jewelry stores.', 'The certifications of diamonds at Gem World are written by people with years of experience in appraising gems.', 'The diamond market is so volatile that prices of the most expensive diamonds can change by hundreds of dollars from one day to the next.', 'The written certifications of diamonds at Gem World are provided by an independent company of gem specialists.']", "label": 3 }, { "id": "train_2997", "context": "When the Pinecrest Animal Shelter, a charitable organization, was in danger of closing because it could not pay for important repairs, its directors appealed to the townspeople to donate money that would be earmarked to pay for those repairs. Since more funds were ultimately donated than were used for the repairs, the directors plan to donate the surplus funds to other animal shelters. But before doing so, the directors should obtain permission from those who made the donations.", "question": "Which one of the following principles, if valid, most helps to justify the position advocated above and yet places the least restriction on the allocation of funds by directors of charitable organizations?", "answers": "['People who contribute money to charitable organizations should be considered to be placing their trust in the directors of those organizations to use the money wisely according to whatever circumstance might arise.', 'People who solicit charitable donations from the public for a specific cause should spend the funds only on that cause, or, if that becomes impossible, should dispose of the funds according to the express wishes of the donors.', 'The directors of charitable organizations cannot allocate publicly solicited funds to any purposes for which the directors had not specifically earmarked the funds in advance.', 'Donors of money to charitable organizations cannot delegate to the directors of those organizations the responsibility of allocating the funds received to various purposes consonant with the purposes of the organization as the directors of the organization see fit.']", "label": 1 }, { "id": "train_2998", "context": "Science teacher: In any nation, a flourishing national scientific community is essential to a successful economy. For such a community to flourish requires that many young people become excited enough about science that they resolve to become professional scientists. Good communication between scientists and the public is necessary to spark that excitement.", "question": "The science teacher's statements provide the most support for which one of the following?", "answers": "['An essential component of success in any scientific endeavor is good communication between the scientists involved in that endeavor and the public.', 'If scientists communicate with the public, many young people will become excited enough about science to resolve to become professional scientists.', 'The extent to which a national scientific community flourishes depends principally on the number of young people who become excited enough about science to resolve to become professional scientists.', 'No nation can have a successful economy unless at some point scientists have communicated well with the public.']", "label": 3 }, { "id": "train_2999", "context": "Feathers recently taken from seabirds stuffed and preserved in the 1880s have been found to contain only half as much mercury as feathers recently taken from living birds of the same species. Since mercury that accumulates in a seabird' s feathers as the feathers grow is derived from fish eaten by the bird, these results indicate that mercury levels in saltwater fish are higher now than they were 100 years ago.", "question": "The argument depends on assuming that", "answers": "[\"mercury derived from fish is essential for the normal growth of a seabird's feathers\", \"the proportion of a seabird's diet consisting of fish was not as high, on average, in the 1880s as it is today\", \"the process used to preserve birds in the 1880s did not substantially decrease the amount of mercury in the birds' feathers\", 'the amount of mercury in a saltwater fish depends on the amount of pollution in the ocean habitat of the fish']", "label": 2 }, { "id": "train_3000", "context": "Scientist: To study the comparative effectiveness of two experimental medications for athlete' s foot, a representative sample of people with athlete' s foot were randomly assigned to one of two groups. One group received only medication M, and the other received only medication N. The only people whose athlete' s foot was cured had been given medication M. Reporter: This means, then, that if anyone in the study had athlete' s foot that was not cured, that person did not receive medication M.", "question": "Which one of the following most accurately describes the reporter's error in reasoning?", "answers": "[\"The reporter fails to allow for the possibility that athlete's foot may be cured even if neither of the two medications studied is taken.\", \"The reporter presumes, without providing justification, that there is no sizeable subgroup of people whose athlete's foot will be cured only if they do not take medication M.\", 'The reporter illicitly draws a conclusion about the population as a whole on the basis of a study conducted only on a sample of the population.', \"The reporter concludes from evidence showing only that M can cure athlete's foot that M always cures athlete's foot.\"]", "label": 3 }, { "id": "train_3001", "context": "Many nursing homes have prohibitions against having pets, and these should be lifted. The presence of an animal companion can yield health benefits by reducing a person' s stress. A pet can also make one' s time at a home more rewarding, which will be important to more people as the average life span of our population increases.", "question": "Which one of the following most accurately expresses the conclusion drawn in the argument above?", "answers": "['Residents of nursing homes should enjoy the same rewarding aspects of life as anyone else.', 'The benefits older people derive from having pets need to be recognized, especially as the average life span increases.', 'As the average life span increases, it will be important to more people that life in nursing homes be rewarding.', 'The policy that many nursing homes have should be changed so that residents are allowed to have pets.']", "label": 3 }, { "id": "train_3002", "context": "Alissa: If, as the mayor says, the city can no longer continue to fund both the children' s museum and local children' s television programming, then it should cease funding the television programming. The interactive character of the exhibits at the museum makes for a richer educational experience than watching television, which is largely passive. Greta: We should stop funding the museum, not the television programming, because, as the mayor has also pointed out, the museum reaches a much smaller audience.", "question": "On the basis of their statements, it can be inferred that Alissa and Greta disagree on which one of the following?", "answers": [ "whether local children's television programming provides a beneficial educational experience to a greater number of children in the city than does the children's museum", "whether the city will need to cease funding local children's television programming if it continues funding the children's museum", "whether the city should cease funding local children's television programming if continuing to fund it would mean that the city would have to cease funding the children's museum", "whether the mayor has spoken truthfully about what will need to happen if the city does not cease funding local children's television programming" ], "label": 2 }, { "id": "train_3003", "context": "Commissioner: Budget forecasters project a revenue shortfall of a billion dollars in the coming fiscal year. Since there is no feasible way to increase the available funds, our only choice is to decrease expenditures. The plan before you outlines feasible cuts that would yield savings of a billion dollars over the coming fiscal year. We will be able to solve the problem we face, ttherefore, only if we adopt this plan.", "question": "The reasoning in the commissioner's argument is flawed because this argument", "answers": "['confuses being an adequate solution with being a required solution', 'inappropriately relies on the opinions of experts', 'takes for granted that there is no way to increase available funds', 'inappropriately employs language that is vague']", "label": 0 }, { "id": "train_3004", "context": "Historian: Leibniz, the seventeenth-century philosopher, published his version of calculus before Newton did. But then Newton revealed his private notebooks, which showed he had been using these ideas for at least a decade before Leibniz' s publication. Newton also claimed that he had disclosed these ideas to Leibniz in a letter shortly before Leibniz' s publication. Yet close examination of the letter shows that Newton' s few cryptic remarks did not reveal anything important about calculus. Thus, Leibniz and Newton each independently discovered calculus.", "question": "Which one of the following is an assumption required by the historian's argument?", "answers": "['Leibniz did not tell anyone about calculus prior to publishing his version of it.', 'Neither Newton nor Leibniz learned crucial details about calculus from some third source.', 'No third person independently discovered calculus prior to Newton and Leibniz.', \"Neither Newton or Leibniz knew that the other had developed a version of calculus prior to Leibniz's publication.\"]", "label": 1 }, { "id": "train_3005", "context": "A product that represents a clear technological advance over competing products can generally command a high price. Because technological advances tend to be quickly surpassed and companies want to make large profits while they still can, many companies charge the greatest price the market will bear when they have such a product. But large profits on the new product will give competitors a strong incentive to quickly match the new product' s capabilities . Consequently, the strategy to maximize overall profit from a new product is to charge less than the greatest possible price.", "question": "In the argument above, the two portions in boldface play which of the following roles?", "answers": "['The first is an assumption that forms the basis for a course of action that the argument criticizes; the second presents the course of action endorsed by the argument.', 'The first is a consideration raised to explain the appeal of a certain strategy; the second is a consideration raised to call into question the wisdom of adopting that strategy.', 'The first is a consideration raised to show that adopting a certain strategy is unlikely to achieve the intended effect; the second is presented to explain the appeal of that strategy.', 'The first is a consideration raised in support of a strategy the argument endorses; the second presents grounds in support of that consideration.']", "label": 1 }, { "id": "train_3006", "context": "New evidence indicates that recent property development bordering a national park has not adversely affected the park' s wildlife. On the contrary, a comparison of the most recent survey of the park' s wildlife with one conducted just prior to the development shows that the amount of wildlife has in fact increased over the intervening decade. Moreover, the park' s resources can support its current wildlife populations without strain.", "question": "Which one of the following, if true, most strengthens the argument?", "answers": "['The more recent survey not only involved counting the animals found in the park but, unlike the earlier survey, also provided an inventory of the plant life found within the park.', 'While both surveys found the same species of animals in the park, the more recent survey found greater numbers of animals belonging to each species.', 'The most recent techniques for surveying wildlife are better at locating difficult-to-find animals than were older techniques.', 'Migration of wildlife into the park from the adjacent developing areas has increased animal populations to levels beyond those that the resources of the park could have supported a decade ago.']", "label": 1 }, { "id": "train_3007", "context": "In parts of South America, vitamin-A deficiency is a serious health problem, especially among children. In one region, agriculturists are attempting to improve nutrition by encouraging farmers to plant a new variety of sweet potato called SPK004 that is rich in beta-carotene, which the body converts into vitamin A. The plan has good chances of success, since sweet potato is a staple of the region' s diet and agriculture, and the varieties currently grown contain little beta-carotene.", "question": "Which of the following, if true, most strongly supports the prediction that the plan will succeed?", "answers": "['The flesh of SPK004 differs from that of the currently cultivated sweet potatoes in color and texture, so traditional foods would look somewhat different when prepared from SPK004.', 'There are no other varieties of sweet potato that are significantly richer in beta-carotene than SPK004 is.', 'The varieties of sweet potato currently cultivated in the region contain some important nutrients that are lacking in SPK004.', 'The growing conditions required by the varieties of sweet potato currently cultivated in the region are conditions in which SPK004 can flourish.']", "label": 3 }, { "id": "train_3008", "context": "Economist: Ordinarily, when energy efficiency improves, less energy is used to satisfy the same needs. So presumably, if a country improves its energy efficiency, then ordinarily its energy consumption should decrease. Yet empirical data show that as a country' s energy efficiency increases, energy consumption there either rises or stays the same.", "question": "Which one of the following, if true, most helps to explain the conflict between the economist's presumption and the empirical data?", "answers": "['When countries increase their energy efficiency, they usually sell their surplus of energy to other countries.', 'When countries increase their energy efficiency, more energy-efficient appliances are bought than appliances that are not energy efficient.', 'Causes other than increased energy efficiency can account for decreased energy consumption.', 'Increases in energy efficiency in a country are typically accompanied by the introduction of new energy-consuming products.']", "label": 3 }, { "id": "train_3009", "context": "College is increasingly unaffordable for everyone that is not independently wealthy, outpacing inflation every year since the early 1970s. This year the average cost of tuition at a private, four-year university is more than $31, 000 per year. In 1971, tuition cost less than $2, 000, even after adjusting for inflation. The trend is similar at public, four-year institutions. Fortunately, with the advent of the Internet, independent learning is easier than ever before. Students can learn hard skills online for relatively minimal costs, like coding and web design. Online courses can also replicate a traditional college education, with options such as math, science, and liberal arts courses. As a result, high school students would be wise to weigh their options before choosing to attend a traditional four-y ear college.", "question": "Which one of the following strengthens the argument?", "answers": "['The independently wealthy should still attend traditional college programs.', 'Students who do not attend traditional college miss out on important life experiences.', 'Employers increasingly value work experience and self-starters more than formal education.', 'The government should have capped tuition at traditional four-year colleges.']", "label": 2 }, { "id": "train_3010", "context": "One of the 10 computer manuals published by company T in 2005 was made for home computers. Exactly five of the advertisements released by company T featured those manuals written for home computers. Since 20 advertisements were released by company T in 2006, no more than one-quarter of them featured the computer manuals published by company T in 2005.", "question": "Which one of the following, if true, allows the conclusion above to be properly drawn?", "answers": "['None of the advertisements released by company T in 2006 featured computer manuals other than the ones written for home computers.', 'None of the graphics used in the advertisements released by company T in 2006 were done by professional artists.', 'Some of the advertisements released by company T in 2006 were actually redesigned versions of old advertisements.', 'None of the computer manuals published by company T in 2006 were based on technology gained from the home computer manuals published in 200']", "label": 0 }, { "id": "train_3011", "context": "Newspaper editorial: In an attempt to reduce the crime rate, the governor is getting tough on criminals and making prison conditions harsher. Part of this effort has been to deny inmates the access they formerly had to college-level courses. However, this action is clearly counter to the governor' s ultimate goal, since after being released from prison, inmates who had taken such courses committed far fewer crimes overall than other inmates.", "question": "Which of the following is an assumption on which the argument depends?", "answers": "['Former inmates are no more likely to commit crimes than are members of the general population.', 'Not being able to take college-level courses while in prison is unlikely to deter anyone from a crime that he or she might otherwise have committed.', 'The group of inmates who chose to take college-level courses were not already less likely than other inmates to commit crimes after being released.', \"Taking high school level courses in prison has less effect on an inmate's subsequent behavior than taking college-level courses does.\"]", "label": 2 }, { "id": "train_3012", "context": "One year ago, a municipality banned dishwasher detergents containing phosphates. Anecdotal evidence indicates that many residents continued to use detergents containing phosphates; they just purchased them from out-of-town stores. However, it is clear that some residents did switch to phosphate-free detergents, since phosphate pollution from the municipal wastewater treatment plant decreased significantly in the past year.", "question": "The answer to which one of the following questions would most help in evaluating the argument above?", "answers": "['Did municipal officials try to stop people from bringing detergents containing phosphates into the municipality?', \"Were any changes made in the past year to the way the municipality's wastewater treatment plant treats phosphates?\", 'What pollutants, if any, are present in phosphate-free dishwashing detergents?', 'Why did many residents continue to use detergents containing phosphates?']", "label": 1 }, { "id": "train_3013", "context": "A primate jawbone found in Namibia in southern Africa has been identified by anthropologists as that of an ape that lived between 10 million and 15 million years ago. Researchers generally agree that such ancient primates lived only in dense forests. Consequently, the dry, treeless expanses now dominating the landscape in and around Namibia must have replaced an earlier, heavily forested terrain.", "question": "The argument assumes which one of the following?", "answers": "['There were no apes living in the area that is now Namibia prior to 15 million years ago.', 'The ape whose jawbone was found lived in or near the area that is now Namibia.', 'The ancient primates were numerous enough to have caused severe damage to the ecology of the forests in which they lived.', 'Modern apes also tend to live only in heavily forested terrain.']", "label": 1 }, { "id": "train_3014", "context": "Two years ago, the government of Runagia increased by 20 percent the government-provided pensions paid to Runagians over 65. The aim of the increase was to stimulate the economy in the rural regions of the country, where most pension recipients live. Statistics, however, show that there has been no increase in economic activity in those regions since then, but that there has been noticeably more spending in the urban areas.", "question": "Which of the following, if true, most helps to explain why the increase resulted in the unintended state of affairs described?", "answers": "['The pensions were increased when the number of people below the poverty level in rural areas of Runagia reached an all-time high.', 'Until the pensions were increased, many Runagians over 65 in rural regions had been receiving support from their children who live in urban areas.', 'The Runagian postal system is so inefficient that it can take up to three weeks for pension checks to reach recipients in rural areas.', 'City-dwellers in Runagia rarely travel to rural regions of the country.']", "label": 1 }, { "id": "train_3015", "context": "In Stenland, many workers have been complaining that they cannot survive on minimum wage, the lowest wage an employer is permitted to pay. The government is proposing to raise the minimum wage. Many employers who pay their workers the current minimum wage argue that if it is raised, unemployment will increase because they will no longer be able to afford to employ as many workers.", "question": "Which of the following, if true in Stenland, most strongly supports the claim that raising the minimum wage there will not have the effects that the employers predict?", "answers": "['Raising the minimum wage does not also increase the amount employers have to contribute in employee benefits.', 'For any position with wages below a living wage, the difficulty of finding and retaining employees adds as much to employment costs as would raising wages.', 'Many employers who pay some workers only the minimum wage also pay other workers wages that are much higher than the minimum.', 'When inflation is taken into account, the proposed new minimum wage is not high as the current one was when it was introduced.']", "label": 1 }, { "id": "train_3016", "context": "A translation invariably reflects the writing style of the translator. Sometimes when a long document needs to be translated quickly, several translators are put to work on the job, each assigned to translate part of the document. In these cases, the result is usually a translation marked by different and often incompatible writing styles. Certain computer programs for language translation that work without the intervention of human translators can finish the job faster than human translators and produce a stylistically uniform translation with an 80 percent accuracy rate. Ttherefore, when a long document needs to be translated quickly, it is better to use a computer translation program than human translators.", "question": "Which one of the following issues would be LEAST important to resolve in evaluating the argument?", "answers": "['whether the computer translation contains errors of grammar and usage that drastically alter the meaning of the text', 'whether the problem of stylistic variety in human translation could be solved by giving stylistic guidelines to human translators', 'whether computer translation programs, like human translators, each have their own distinct writing style', \"how the accuracy rate of computer translation programs compares with that of human translators in relation to the users' needs\"]", "label": 2 }, { "id": "train_3017", "context": "Studies have found that human tears contain many of the same hormones that the human body produces in times of emotional stress. Hence, shedding tears removes significant quantities of these hormones from the body. Ttherefore, crying must have the effect of reducing emotional stress.", "question": "The reasoning in the argument is most vulnerable to criticism on the grounds that the argument", "answers": "['overlooks the possibility that if crying has a tendency to reduce emotional stress, this tendency might arise because of something other than the shedding of tears', 'takes for granted that because certain substances are present whenever a condition occurs, those substances are a cause of that condition', 'fails to adequately distinguish between two distinct factors that are jointly responsible for causing a given phenomenon', 'confuses a condition that is required for the production of a given phenomenon with a condition that in itself would be sufficient to cause the production of that phenomenon']", "label": 1 }, { "id": "train_3018", "context": "Art Historian: Robbins cannot pass judgment on Stuart' s art. While Robbins understands the art of Stuart too well to dismiss it, she does not understand it well enough to praise it.", "question": "The art historian's argument depends on the assumption that", "answers": "['if art can be understood well, it should be either dismissed or praised', \"in order to understand Stuart's art, Robbins must be able to pass judgment on it\", \"in order to pass judgment on Stuart's art, Robbins must be able either to dismiss it or to praise it\", 'if Robbins understands art well, she will praise it']", "label": 2 }, { "id": "train_3019", "context": "Either food scarcity or excessive hunting can threaten a population of animals. If the group faces food scarcity, individuals in the group will reach reproductive maturity later than otherwise. If the group faces excessive hunting, individuals that reach reproductive maturity earlier will come to predominate. Ttherefore, it should be possible to determine whether prehistoric mastodons became extinct because of food scarcity or human hunting, since there are fossilized mastodon remains from both before and after mastodon populations declined, and __.", "question": "Which of the following most logically completes the reasoning?", "answers": "['it is not known when humans first began hunting mastodons', 'it can be accurately estimated from fossilized remains when mastodons became extinct', 'the average age at which mastodons from a given period reached reproductive maturity can be established from their fossilized remains', 'climate changes may have gradually reduced the food available to mastodons']", "label": 2 }, { "id": "train_3020", "context": "When adults toss balls to very young children they generally try to toss them as slowly as possible to compensate for the children' s developing coordination. But recent studies show that despite their developing coordination, children actually have an easier time catching balls that are thrown at a faster speed.", "question": "Which one of the following, if true, most helps to explain why very young children find it easier to catch balls that are thrown at a faster speed?", "answers": "['Children are able to toss balls back to the adults with more accuracy when they throw fast than when they throw the ball back more slowly.', 'Adults generally find it easier to catch balls that are thrown slowly than balls that are thrown at a faster speed.', 'Balls thrown at a faster speed, unlike balls thrown at a slower speed, trigger regions in the brain that control the tracking of objects for self-defense.', 'There is a limit to how fast the balls can be tossed to the children before the children start to have more difficulty in catching them.']", "label": 2 }, { "id": "train_3021", "context": "Advertisers have learned that people are more easily encouraged to develop positive attitudes about things toward which they originally have neutral or even negative attitudes if those things are linked, with pictorial help rather than exclusively through prose, to things about which they already have positive attitudes. Ttherefore, advertisers are likely to __.", "question": "Which one of the following most logically completes the argument?", "answers": "['use little if any written prose in their advertisements', 'try to encourage people to develop positive attitudes about products that can be better represented pictorially than in prose', 'highlight the desirable features of the advertised product by contrasting them pictorially with undesirable features of a competing product', 'create advertisements containing pictures of things most members of the target audience like']", "label": 3 }, { "id": "train_3022", "context": "Only some strains of the tobacco plant are naturally resistant to tobacco mosaic virus, never becoming diseased even when infected. When resistant strains were experimentally infected with the virus, levels of naturally occurring salicylic acid in these plants increased fivefold; no such increase occurred in the nonresistant plants. In a second experiment, 50 nonresistant tobacco plants were exposed to tobacco mosaic virus, and 25 of them were injected with salicylic acid. None of these 25 plants showed signs of infection; however, the other 25 plants succumbed to the disease.", "question": "Which one of the following conclusions is most strongly supported by the results of the experiments?", "answers": "['Salicylic acid is not produced in strains of tobacco plants that are not resistant to tobacco mosaic virus.', 'It is possible to test an uninfected tobacco plant for resistance to tobacco mosaic virus by measuring the level of salicylic acid it contains.', 'Producing salicylic acid is at least part of the mechanism by which some tobacco plants naturally resist the disease caused by tobacco mosaic virus.', 'Tobacco plants that have become diseased by infection with tobacco mosaic virus can be cured by injecting them with salicylic acid.']", "label": 2 }, { "id": "train_3023", "context": "Scientists, puzzled about the development of penicillin-resistant bacteria in patients who had not been taking penicillin, believe they have found an explanation. The relevant group of patients have dental fillings made of mercury-containing amalgam, and the bacteria the patients develop are immune to mercury poisoning. Scientists have concluded that the genes causing resistance to penicillin are closely bundled on the chromosomes of bacteria with the gene that produces immunity to mercury poisoning. Exposure to the mercury kills off bacteria that lack the relevant immunity gene, and leaves room for those that possess both the mercury-immunity gene and the penicillin-resistance gene to flourish.", "question": "Which one of the following most accurately characterizes the role played in the passage by the unstated assumption that some patients who take penicillin develop bacteria with an immunity to penicillin?", "answers": "[\"It is the tentative conclusion of previous research that appears to be falsified by the scientists' discovery of the mechanism by which bacteria become resistant to mercury poisoning.\", 'It is a generalization that, if true, rules out the possibility that some people who do not take penicillin develop bacteria resistant to it.', 'It is a generalization assumed by the scientists to conclusively prove that the explanation of their problem case must involve reference to the genetic makeup of the penicillin-resistant bacteria.', 'It is a point that, in conjunction with the fact that some patients who do not take penicillin develop penicillin-resistant bacteria, generates the problem that prompted the research described in the passage.']", "label": 3 }, { "id": "train_3024", "context": "Zeida: Dr. Ladlow, a research psychologist, has convincingly demonstrated that his theory about the determinants of rat behavior generates consistently accurate predictions about how rats will perform in a maze. On the basis of this evidence, Dr. Ladlow has claimed that his theory is irrefutably correct. Anson: Then Dr. Ladlow is not a responsible psychologist. Dr. Ladlow' s evidence does not conclusively prove that his theory is correct. Responsible psychologists always accept the possibility that new evidence will show that their theories are incorrect.", "question": "Anson bases his conclusion about Dr. Ladlow on which one of the following?", "answers": "['the application of a general principle', 'the rejection of a theoretical explanation', 'the discrediting of facts', 'the use of an ambiguous term']", "label": 0 }, { "id": "train_3025", "context": "Lawyer: The defendant wanted to clear the snow off his car and in doing so knocked snow on the sidewalk. This same snow melted and refroze, forming ice on which the plaintiff fell, breaking her hip. We argue that the defendant maliciously harmed the plaintiff, because malice is intention to cause harm and the defendant intentionally removed the snow from his car and put it on the sidewalk, which, unbeknownst to the defendant at the time, would subsequently cause the injury suffered by the plaintiff.", "question": "The flawed reasoning in which one of the following is most similar to that in the lawyer's argument?", "answers": "['Edwina bought a car from Mr. Yancy, then resold it. Unbeknownst to Edwina, Mr. Yancy had stolen the car. So Edwina sold a stolen car.', \"Alice asked her sister to lie in court. Unbeknownst to Alice's sister, lying in court is against the law. So what Alice asked her sister to do was illegal.\", 'Bruce wanted to eat the mincemeat pie. Unbeknownst to Bruce, the mincemeat pie was poisonous. So Bruce wanted to eat poison.', 'Deon had lunch with Ms. Osgood. Unbeknownst to Deon, Ms. Osgood is generally thought to be an industrial spy. So Deon had lunch with an industrial spy.']", "label": 2 }, { "id": "train_3026", "context": "Mr. Nance: Ms. Chan said that she retired from Quad Cities Corporation, and had received a watch and a wonderful party as thanks for her 40 years of loyal service. But I overheard a colleague of hers say that Ms. Chan will be gone for much of the next year on business trips and is now working harder than she ever did before; that does not sound like retirement to me. At least one of them is not telling the truth.", "question": "Mr. Nance's reasoning is flawed because it", "answers": "['is based in part on hearsay', 'draws a conclusion based on equivocal language', 'criticizes Ms. Chan rather than the claims she made', 'fails to infer that Ms. Chan must be a person of superior character, given her long loyal service']", "label": 1 }, { "id": "train_3027", "context": "During the recent spate of brushfires in the Southwest, homeowners who lived near affected areas were advised to douse their roofs with water to prevent their houses from catching fire before evacuating the area. After the fires were brought under control and the homeowners were allowed to return to the area, many who doused their roofs discovered significant fire damage to their houses. Clearly, then, dousing their roofs was a wasted effort.", "question": "Which of the following, if true, would most weaken the conclusion above?", "answers": "['Not all homeowners who doused their roofs did so to the same extent.', 'The houses that suffered the least damage were those in which the owners remained and continuously doused the roofs.', 'The houses of owners who did not douse the roofs with water suffered appreciably more fire damage than did those of owners who did douse the roofs with water.', 'The fire insurance rates for those who doused their roofs did not increase after the fire.']", "label": 2 }, { "id": "train_3028", "context": "Editorialist: Some people argue that we have an obligation not to cut down trees. However, there can be no obligation to an entity unless that entity has a corresponding right. So if we have an obligation toward trees, then trees have rights. But trees are not the sort of things that can have rights. Ttherefore, we have no obligation not to cut down trees.", "question": "The editorialist's argument depends on assuming which one of the following?", "answers": "['Avoiding cutting down trees is not an obligation owed to some entity other than trees.', \"One does not always have the right to cut down the trees on one's own property.\", 'Only conscious entities are the sort of things that can have rights.', 'Any entity that has rights also has obligations.']", "label": 0 }, { "id": "train_3029", "context": "Mayor: Periodically an ice cream company will hold a free ice cream day as a promotion. Showing up may not cost you any money, but it sure does cost you time. We learn from this that when something valuable costs no money you get overconsumption and long lines. Currently, those who drive to work complain about the congestion they face in their rush-hour commutes. What is needed is a system for charging people for the use of roads during rush hour. Then rush hour congestion will abate.", "question": "The claim that when something valuable costs no money you get overconsumption and long lines plays which one of the following roles in the mayor's argument?", "answers": "[\"It is a general claim used in support of the argument's overall conclusion.\", \"It is a hypothesis that is rejected in favor of the hypothesis stated in the argument's overall conclusion.\", 'It is a concession made to those who dispute an analogy drawn in the argument.', \"It helps establish the importance of the argument's overall conclusion, but is not offered as evidence for that conclusion.\"]", "label": 0 }, { "id": "train_3030", "context": "Museums that house Renaissance oil paintings typically store them in environment that are carefully kept within narrow margins of temperature and humidity to inhibit any deterioration. Laboratory tests have shown that the kind of oil paint used in these paintings actually adjusts to climatic changes quite well. If, as some museum directors believe, paint is the most sensitive substance in these works , then by relaxing the standards for temperature and humidity control, museums can reduce energy costs without risking damage to these paintings . Museums would be rash to relax those standards, however, since results of preliminary tests indicate that gesso, a compound routinely used by Renaissance artists to help paint adhere to the canvas, is unable to withstand significant variations in humidity.", "question": "In the argument above, the two portions in boldface play which of the following roles?", "answers": "['The first is a judgment that has been offered in support of the position that the argument calls into question; the second is a circumstance on which that judgment is, in part based', 'The first is the position taken by the argument; the second is the position that the argument calls into question', 'The first is a claim that the argument calls into question; the second is the position taken by the argument', 'The first is a judgment that has been offered in support of the position that the argument calls into question; the second is that position']", "label": 3 }, { "id": "train_3031", "context": "Over the past ten years, the population of Dismaston has grown five times as large as it was. During this time, the average income in the city has risen substantially, and a tremendous amount of capital has flowed into city. An independent audit found that, somewhat surprisingly, the number of violent felonies reported per year is now lower than it was ten years ago.", "question": "Each of the following statements below, if true, would explain the somewhat surprising finding EXCEPT:", "answers": "[\"During this time, the state considerably lengthened felony convicts' waiting period for parole.\", 'The police force has expanded in number and is equipped with the latest crime detection technology.', 'The police now have a computerized filing system, so that it is almost impossible for a violent crime to be unrecorded.', 'The city is now much better lit at night, and security cameras protect a large number of public venues.']", "label": 2 }, { "id": "train_3032", "context": "Although the earliest surviving Greek inscriptions written in an alphabet date from the eighth century B. C. , a strong case can be made that the Greeks actually adopted alphabetic writing at least two centuries earlier . Significantly, the text of these earliest surviving Greek inscriptions sometimes runs from right to left and sometimes from left to right. Now, the Greeks learned alphabetic writing from the Phoenicians, and in the process they would surely have adopted whatever convention the Phoenicians were then using with respect to the direction of writing . Originally, Phoenician writing ran in either direction, but by the eighth century B. C. it had been consistently written from right to left for about two centuries.", "question": "In the argument given, the two portions in boldface play which of the following roles?", "answers": "['The first is the position that the argument seeks to establish; the second presents an assumption on which the argument relies.', 'The first presents evidence that is used in support of the position that the argument seeks to establish; the second presents an assumption on which the argument relies.', 'The first is an objection raised against a position that the argument opposes; the second is evidence that has been used to support that position.', 'The first is the position that the argument seeks to establish; the second reports a discovery that has been used to support a position that the argument opposes.']", "label": 0 }, { "id": "train_3033", "context": "Concerned about the financial well-being of its elderly citizens, the government of Runagia decided two years ago to increase by 20 percent the government-provided pension paid to all Runagians over 65. Inflation in the intervening period has been negligible, and the increase has been duly received by all eligible Runagians. Nevertheless, many of them are no better off financially than they were before the increase, in large part because __.", "question": "Which of the following most logically completes the passage?", "answers": "['they buy goods whose prices tend to rise especially fast in times of inflation', 'in Runagia children typically supplement the income of elderly parents, but only by enough to provide them with a comfortable living', 'Runagian banks are so inefficient that it can take up to three weeks to cash a pension check', 'the pension was increased when the number of elderly Runagians below the poverty level reached an all-time high']", "label": 1 }, { "id": "train_3034", "context": "It has been a staple of drama to feature an innocent young protagonist, eager to make a mark on the world, who is stymied by an indifferent or hostile society. Since the playwrights of such works wished the audience to empathize with the protagonist, historians do not regard these plays as serious revelations of what the societies presented in the plays were really like.", "question": "Which one of the following, if true, most helps to explain the viewpoint of the historians described above?", "answers": "['The historians believe that plays often contain serious revelations of what the societies presented in those plays were like.', 'The historians believe that only the most popular plays within a society accurately portray that society.', 'The historians believe that playwrights tend to exaggerate the weaknesses of a society for the sake of dramatic effect.', 'The historians believe that plays tend to provide useful information about the time and society in which they were written.']", "label": 2 }, { "id": "train_3035", "context": "In ancient Greece, court witnesses were not crossexamined and the jury, selected from the citizenry, received no guidance on points of law; thus, it was extremely important for litigants to make a good impression on the jurors. For this reason, courtroom oratory by litigants is a good source of data on the common conceptions of morality held by the citizens of ancient Greece.", "question": "Which one of the following, if true, would most strengthen the argument?", "answers": "['Litigants believed jurors were likely to be impressed by litigants whose professed moral code most resembled their own.', 'Litigants believed jurors were likely to render their decisions based on a good understanding of the law.', 'Litigants believed jurors were more likely to be impressed by litigants whose personality they preferred.', \"Litigants believed jurors were more likely to subject the litigants' personal moral codes to close critical scrutiny than were people who did not sit on juries.\"]", "label": 0 }, { "id": "train_3036", "context": "Claude: To introduce greater public accountability into French foreign-policy decisions, France should hold referenda on major foreign-policy issues. Election results are too imprecise to count as a mandate, since elections are decided on multiple issues. Lorraine: The general public, unlike people in government, is unwilling or unable to become informed about foreign-policy issues. Ttherefore, the introduction of such referenda would lead to foreign-policy disaster.", "question": "Which one of the following responses by Claude would, if true, most strongly counter Lorraine's objection?", "answers": "['Some of the information on which foreign- policy decisions are based comes from intelligence sources that must not be compromised by publicity.', 'Foreign proponents and opponents of a particular policy would attempt to sway French public opinion before a referendum.', 'Foreign governments friendly to France would be reluctant to share information with France if it might become public in a referendum debate.', 'The general public has little desire to become informed about foreign-policy issues precisely because it has little or no power to influence foreign-policy decisions.']", "label": 3 }, { "id": "train_3037", "context": "Fremont: Simpson is not a viable candidate for chief executive of Pod Oil because he has no background in the oil industry. Galindo: I disagree. An oil industry background is no guarantee of success. Look no further than Pod Oil' s last chief executive, who had decades of oil industry experience but steered the company to the brink of bankruptcy.", "question": "Galindo's argument is flawed in that it", "answers": "['bases a conclusion that an attribute is always irrelevant to success on evidence that it is sometimes irrelevant to success', 'rests on a confusion between whether an attribute is necessary for success and whether that attribute is sufficient for success', 'fails to distinguish between relevant experience and irrelevant experience', \"fails to justify its presumption that Fremont's objection is based on personal bias\"]", "label": 1 }, { "id": "train_3038", "context": "Sales of telephones have increased dramatically over the last year. In order to take advantage of this increase, Mammoth Industries plans to expand production of its own model of telephone, while continuing its already very extensive advertising of this product.", "question": "Which of the following, if true, provides most support for the view that Mammoth Industries cannot increase its sales of telephones by adopting the plan outlined above?", "answers": [ "Although it sells all of the telephones that it produces, Mammoth Industries' share of all telephone sales has declined over the last year.", "Mammoth Industries' average inventory of telephones awaiting shipment to retailers has declined slightly over the last year.", "Despite a slight decline in the retail price, sales of Mammoth Industries' telephones have fallen in the last year.", "Advertising has made the brand name of Mammoth Industries' telephones widely known, but fewconsumers know that Mammoth Industries owns this brand." ], "label": 2 }, { "id": "train_3039", "context": "Historian: A democracy' s citizens must know some history if the democracy is to meet its challenges. However, popular historical awareness is inevitably distorted, for most people learn history through popular narratives that sustain readers' interest by implying that a few famous heroes and notorious villains have shaped all of history.", "question": "The historian's argument depends on assuming which one of the following?", "answers": "['Most historical narratives sustain interest by implying that a few famous heroes and notorious villains have shaped all of history.', \"Only narratives written for a purpose other than sustaining readers' interest can convey an undistorted awareness of history.\", 'History cast in the narrative format inevitably distorts historical reality.', 'The implication that a few famous heroes and notorious villains have shaped all of history distorts history.']", "label": 3 }, { "id": "train_3040", "context": "The greater the number of people who regularly use a product, the greater the number whose health is potentially at risk due to that product. More people regularly use household maintenance products such as cleaning agents and lawn chemicals than regularly use prescription medicines. Ttherefore, it is even more important for such household products to be carefully tested to ensure their safety than it is for prescription medicines to be so tested.", "question": "Which one of the following principles, if valid, most helps to justify drawing the conclusion in the argument above?", "answers": "['The more people whose health might be at risk from the regular use of a particular product, the more important it is for that product to be carefully tested to ensure its safety.', 'It is very important for any product that is regularly used by a large number of people to be carefully tested to ensure its safety.', 'If one type of medicine must be taken in more frequent doses than another type of medicine, it is more important for the former to be carefully tested than for the latter.', \"It is generally more important for a medicine than it is for a nonmedical product to be carefully tested to ensure its safety unless more people's health would be at risk from the nonmedical product than from the medicine.\"]", "label": 0 }, { "id": "train_3041", "context": "Waller: If there were really such a thing as extrasensory perception, it would generally be accepted by the public since anyone with extrasensory powers would be able to convince the general public of its existence by clearly demonstrating those powers. Indeed, anyone who was recognized to have such powers would achieve wealth and renown. Chin: It' s impossible to demonstrate anything to the satisfaction of all skeptics. So long as the cultural elite remains closed-minded to the possibility of extrasensory perception, the popular media reports, and thus public opinion, will always be biased in favor of such skeptics.", "question": "Waller's and Chin's statements commit them to disagreeing on whether", "answers": "['skeptics about extrasensory perception have a weak case', 'the failure of the general public to believe in extrasensory perception is good evidence against its existence', 'extrasensory perception, if it were a real phenomenon, could be demonstrated to the satisfaction of all skeptics', 'extrasensory perception is a real phenomenon']", "label": 1 }, { "id": "train_3042", "context": "Critic: The criticism of the popular film comedy Quirks for not being realistic is misguided. It is certainly true that the characters are too stylized to be real people. That could be problematic, but in this case the resulting film is funny. And that is the important thing for a comedy.", "question": "Which one of the following principles, if valid, most helps to justify the reasoning in the critic's argument?", "answers": "['Films are successful if they succeed within their genre.', 'Films should try to stay entirely within a single genre.', 'Films should be judged on how well they accurately capture the world.', 'Films are successful as long as they are popular.']", "label": 0 }, { "id": "train_3043", "context": "If Max were guilty, he would not ask the police to investigate. Ttherefore, his asking the police to investigate shows that he is not guilty.", "question": "The logical structure of the argument above is most similar to which one of the following?", "answers": "['If Lucille were in the next room, I would not be able to see her. Ttherefore, the fact that I can see her shows that she is not in the next room.', 'If Joe were over 40 he would not want to learn to ski. Ttherefore, the fact that he does not want to learn to ski shows that he is over 40.', 'If Sam were rich, he would not spend his vacation in Alaska. Ttherefore, his spending his vacation in the Bahamas shows that he is rich.', 'If Sally were sociable, she would not avoid her friends. Ttherefore, the fact that she is sociable shows that she does not avoid her friends. GO ON TO THE NEXT PAGE.']", "label": 0 }, { "id": "train_3044", "context": "Although some nutritional facts about soft drinks are listed on their labels, exact caffeine content is not. Listing exact caffeine content would make it easier to limit, but not eliminate, one' s caffeine intake. If it became easier for people to limit, but not eliminate, their caffeine intake, many people would do so, which would improve their health.", "question": "If all the statements above are true, which one of the following must be true?", "answers": "['The health of at least some people would worsen if everyone knew exactly how much caffeine was in their soft drinks.', 'The health of at least some people would improve if exact caffeine content were listed on soft-drink labels.', 'Many people will find it difficult to eliminate their caffeine intake if they have to guess exactly how much caffeine is in their soft drinks.', 'Many people will be unable to limit their caffeine intake if exact caffeine content is not listed on soft-drink labels.']", "label": 1 }, { "id": "train_3045", "context": "Taxpayer: For the last ten years, Metro City' s bridge-maintenance budget of $1 million annually has been a prime example of fiscal irresponsibility. In a well-run bridge program, the city would spend $15 million a year on maintenance, which would prevent severe deterioration, thus limiting capital expenses for needed bridge reconstruction to $10 million. However, as a result of its attempt to economize, the city is now faced with spending $400 million over two years on emergency reconstruction of its bridges.", "question": "The main point of the taxpayer's argument is that Metro City", "answers": "['should have budgeted substantially more money for maintenance of its bridges', 'is economizing on its bridge program to save money in case of emergencies', 'is spending more than it needs to on maintenance of its bridges', 'would have had a well-run bridge program if it had spent more money for reconstruction of its bridges']", "label": 0 }, { "id": "train_3046", "context": "Principle: Only if a professor believes a student knowingly presented someone else' s ideas without attribution should the professor make an official determination that the student has committed plagiarism. Application: It is not the case that Professor Serfin should make an official determination that Walters committed plagiarism in the term paper about Willa Cather that Walters wrote for Serfin' s class.", "question": "Which one of the following, if true, justifies the above application of the principle?", "answers": "[\"Although the main thesis of Walters's term paper is identical to that of a book that he did not cite, Professor Serfin is convinced that Walters did not knowingly try to pass anyone else's ideas off as his own.\", \"Professor Serfin does not have completely compelling evidence to conclude that Walters presented someone else's ideas as if they were his own in the term paper about Willa Cather.\", 'Walters does not believe that Professor Serfin should make an official determination that he plagiarized.', 'If Walters had realized that the main thesis of his term paper is identical to the main thesis of a book he had read, Walters would have attributed the idea to the book.']", "label": 0 }, { "id": "train_3047", "context": "It is the mark of a superior conductor that he or she has the authority to insist, even with a top orchestra, that rehearsal work must be intensified. This authority cannot simply be claimed; the conductor must earn it by winning the orchestra' s respect for the artistic interpretations he or she is currently pursuing.", "question": "In taking the position outlined, the author presupposes which one of the following?", "answers": "['Superior conductors are perfectionists who are never satisfied with any performance even by a top orchestra.', 'Superior conductors devise different interpretations of a composition for each orchestra with which they perform it.', 'Top orchestras are always ready to put in additional work on rehearsals if the conductor considers additional rehearsing necessary.', 'Top orchestras can appreciate the merits of an interpretation even before they have brought it to full realization.']", "label": 3 }, { "id": "train_3048", "context": "Among small- to medium-sized marine mammals such as seals and dolphins, the longer an animal can stay submerged during a dive, the greater the depth the animal can reach. Dolphins can dive to greater depths than northern fur seals can, and elephant seals can stay submerged longer than Weddell seals can.", "question": "If the information above is accurate, then each of the following statements could be true EXCEPT:", "answers": "['Northern fur seals can stay submerged longer than Weddell seals can, but elephant seals can dive to greater depths than northern fur seals can.', 'Northern fur seals can stay submerged longer than elephant seals can, but Weddell seals can dive to greater depths than dolphins can.', 'Weddell seals can dive to greater depths than dolphins can and can stay submerged longer than northern fur seals can.', 'Weddell seals can stay submerged longer than northern fur seals can, but dolphins can dive to greater depths than Weddell seals can.']", "label": 1 }, { "id": "train_3049", "context": "In 1578 Moroccan troops defeated a Portuguese army on Moroccan soil. Accounts written by Portuguese contemporaries report the defeat but omit mention of the fact that King Sebastian of Portugal was killed in the battle. Such omissions cannot simply be the result of ignorance of Sebastian' s death. Sebastian' s death is not even mentioned in the battle accounts written by two captured Portuguese officers while waiting to be ransomed from a Moroccan prison. These two officers actually shared their cells with the Portuguese soldiers who identified the king' s body to Moroccan officials. The omissions ttherefore had to have had a psychological cause: the Portuguese evidently found Sebastian' s death so humiliating that they could not bring themselves to write about it.", "question": "The discussion about the two Portuguese officers advances the argument by", "answers": "['providing grounds for eliminating an alternative explanation', 'supplying indirect evidence that a disputed death actually occurred', 'giving evidence supporting a general psychological principle on which the main conclusion is explicitly based', \"offering grounds for doubting the reliability of historical reports that conflict with the argument's main conclusion\"]", "label": 0 }, { "id": "train_3050", "context": "Spreading iron particles over the surface of the earth' s oceans would lead to an increase in phytoplankton, decreasing the amount of carbon dioxide in the atmosphere and thereby counteracting the greenhouse effect. But while counteracting the greenhouse effect is important, the side effects of an iron-seeding strategy have yet to be studied. Since the oceans represent such an important resource, this response to the greenhouse effect should not be implemented immediately.", "question": "The reasoning above most closely conforms to which one of the following principles?", "answers": "['As long as there is a possibility that a strategy for solving a problem may instead exacerbate that problem, such a solution should not be adopted.', 'We should not implement a problem-solving strategy if the consequences of doing so are more serious than the problem itself.', 'A problem-solving strategy should be implemented if the side effects of the strategy are known.', 'Implementing a problem-solving strategy that alters an important resource is impermissible if the consequences are not adequately understood.']", "label": 3 }, { "id": "train_3051", "context": "Tony: A short story is little more than a novelist' s sketch pad. Only novels have narrative structures that allow writers to depict human lives accurately by portraying characters whose personalities gradually develop through life experience. Raoul: Life consists not of a linear process of personality development, but rather of a series of completely disjointed vignettes, from many of which the discerning observer may catch glimpses of character. Thus, the short story depicts human lives more faithfully than does the novel.", "question": "The dialogue most supports the claim that Tony and Raoul disagree about whether", "answers": "['short stories provide glimpses of facets of character that are usually kept hidden', \"only short stories are used as novelists' sketch pads\", 'novels and short stories employ the same strategies to depict human lives', 'human lives are best understood as series of completely disjointed vignettes']", "label": 3 }, { "id": "train_3052", "context": "Sedimentary rock hardens within the earth' s crust as layers of matter accumulate and the pressure of the layers above converts the layers below into rock. One particular layer of sedimentary rock that contains an unusual amount of the element iridium has been presented as support for a theory that a meteorite collided with the earth some sixty million years ago. Meteorites are rich in iridium compared to the earth' s crust, and geologists theorize that a meteorite' s collision with the earth raised a huge cloud of iridium-laden dust. The dust, they say, eventually settled to earth where it combined with other matter, and as new layers accumulated above it, it formed a layer of iridium-rich rock.", "question": "Which one of the following, if true, would counter the claim that the iridium-rich layer described in the passage is evidence for the meteorite collision theory?", "answers": "[\"The huge dust cloud described in the passage would have blocked the transmission of sunlight and lowered the earth's temperature.\", 'Layers of sedimentary rock are used to determine the dates of prehistoric events whether or not they contain iridium.', 'A layer of sedimentary rock takes millions of years to harden.', 'Sixty million years ago there was a surge in volcanic activity in which the matter spewed from the volcanoes formed huge iridium-rich dust clouds.']", "label": 3 }, { "id": "train_3053", "context": "Sleep research has demonstrated that sleep is characterized by periods of different levels of brain activity. People experience dreams during only one of these periods, known as REM (rapid eye movement) sleep. Test subjects who are chronically deprived of REM sleep become irritable during waking life. This shows that REM sleep relieves the stresses of waking life.", "question": "Which one of the following, if true, most strengthens the argument?", "answers": "['Other factors being equal, people who normally have shorter periods of REM sleep tend to experience more stress.', 'Test subjects who are chronically deprived of non-REM sleep also become irritable during waking life.', \"During times of increased stress, one's REM sleep is disturbed in a way that prevents one from dreaming.\", 'Only some people awakened during REM sleep can report the dreams they were having just before being awakened.']", "label": 0 }, { "id": "train_3054", "context": "One of the effects of lead poisoning is an inflammation of the optic nerve, which causes those who have it to see bright haloes around light sources. In order to produce the striking yellow effects in his \"Sunflowers\" paintings, Van Gogh used Naples yellow, a pigment containing lead. Since in his later paintings, Van Gogh painted bright haloes around the stars and sun, it is likely that he was suffering from lead poisoning caused by ingesting the pigments he used.", "question": "Which one of the following is an assumption on which the argument relies?", "answers": "['Van Gogh continued to use paints containing lead after having painted the \"Sunflowers\" paintings.', \"In Van Gogh's later paintings he painted some things as he saw them.\", 'The effects of Naples yellow could not have been achieved using other pigments.', 'Van Gogh did not have symptoms of lead poisoning aside from seeing bright haloes around light sources.']", "label": 1 }, { "id": "train_3055", "context": "Lucinda will soon be attending National University as an engineering major. At National University, most residents of Western Hall are engineering majors. Ttherefore, Lucinda will probably live in Western Hall.", "question": "Which one of the following arguments exhibits a flawed pattern of reasoning most similar to that exhibited by the argument above?", "answers": "['Cities that are regional economic hubs generally experience tremendous economic growth at some point. Our city is a regional economic hub that has never experienced tremendous economic growth. Thus it will probably experience tremendous economic growth in the future.', 'Most cities that are regional economic hubs contain major shopping malls. A major shopping mall is now being constructed in our city. Ttherefore, our city will probably become a regional economic hub.', \"Cities that are regional economic hubs always have excellent transportation systems. It is widely agreed that our city's transportation system is inadequate. Ttherefore, our city will probably never become a regional economic hub.\", 'A major shopping mall was built in our city ten years ago, and our city has experienced tremendous economic growth since then. Ttherefore, most cities in which major shopping malls are built will experience tremendous economic growth shortly afterward.']", "label": 1 }, { "id": "train_3056", "context": "Politician: The bill that makes using car phones while driving illegal should be adopted. My support of this bill is motivated by a concern for public safety. Using a car phone seriously distracts the driver, which in turn poses a threat to safe driving. People would be deterred from using their car phones while driving if it were illegal to do so.", "question": "The argument's main conclusion follows logically if which one of the following is assumed?", "answers": "['Any proposed law that would reduce a threat to public safety should be adopted.', \"Some distractions interfere with one's ability to safely operate an automobile.\", 'The only way to reduce the threat to public safety posed by car phones is through legislation.', 'The more attention one pays to driving, the safer a driver one is.']", "label": 0 }, { "id": "train_3057", "context": "Most serious students are happy students, and most serious students go to graduate school. Furthermore, all students who go to graduate school are overworked.", "question": "Which one of the following can be properly inferred from the statements above?", "answers": "['Some unhappy students go to graduate school.', 'Some happy students are overworked.', 'All overworked students are serious students.', 'All serious students are overworked.']", "label": 1 }, { "id": "train_3058", "context": "Advertisement: Seventy-five percent of dermatologists surveyed prefer Dermactin to all other brands of skin cream. Why? We consulted dermatologists during the development of Dermactin to ensure that you have the best skin cream on the market. So if you need a skin cream, use Dermactin.", "question": "The reasoning in the advertisement is questionable because the advertisement", "answers": "['presumes, without providing justification, that some dermatologists are less qualified than others to evaluate skin cream', 'fails to state the number of dermatologists surveyed, which leaves open the possibility that the sample of doctors is too small to be reliable', 'overlooks the possibility that for a few people, using no skin cream is preferable to using even the best skin cream', 'overlooks the possibility that other types of doctors have cause to use Dermactin, which would render the sample unrepresentative']", "label": 1 }, { "id": "train_3059", "context": "Poor writers often express mundane ideas with elaborate syntax and esoteric vocabulary. Inattentive readers may be impressed but may well misunderstand the writing, while alert readers will easily see through the pretentiousness. Thus, a good principle for writers is: __.", "question": "Which one of the following completes the passage most logically?", "answers": "['the simpler the style, the better the writing', 'a writing style should not be more complex than the ideas expressed', 'only the most talented writers can successfully adopt a complex style', 'alert readers are the only readers who are sensitive to writing style']", "label": 1 }, { "id": "train_3060", "context": "All people residing in the country of Gradara approve of legislation requiring that certain hazardous wastes be disposed of by being burned in modern high-temperature incinerators. However, waste disposal companies planning to build such incinerators encounter fierce resistance to their applications for building permits from the residents of every Gradaran community that those companies propose as an incinerator site.", "question": "Which one of the following, if true, most helps to explain the residents' simultaneously holding both of the positions ascribed to them?", "answers": "['The substantial cost of high-temperature incineration can be partially offset by revenue from sales of electric energy generated as a by-product of incineration.', 'High-temperature incineration is more expensive than any of the available alternatives would be, and the higher costs would be recovered through higher product prices.', 'High-temperature incineration minimizes the overall risk to the human population of the country from the wastes being disposed of, but it concentrates the remaining risk in a small number of incineration sites.', 'The toxic fumes generated within a high-temperature incinerator can be further treated so that all toxic residues from a properly operating incinerator are solids.']", "label": 2 }, { "id": "train_3061", "context": "Spectroscopic analysis has revealed the existence of frozen nitrogen, methane, and carbon monoxide on the surface of Pluto. Such ices have a tendency to vaporize, producing an atmosphere. Since the proportion of any gas in such an atmosphere depends directly on how readily the corresponding ice vaporizes, astronomers have concluded that the components of Pluto' s atmosphere are nitrogen, carbon monoxide, and methane, in order of decreasing abundance.", "question": "The astronomers' argument relies on which one of the following assumptions?", "answers": "['There is no more frozen nitrogen on the surface of Pluto than there is either frozen carbon monoxide or methane.', 'Until space probes reach Pluto, direct analysis of the atmosphere is impossible.', 'A mixture of nitrogen, carbon monoxide, and methane is characteristic of the substances from which the Solar System formed.', 'There is no frozen substance on the surface of Pluto that vaporizes more readily than methane but less readily than carbon monoxide.']", "label": 3 }, { "id": "train_3062", "context": "Counselor: Constantly comparing oneself to those one sees as more able or more successful almost invariably leads to self-disparagement. Conversely, constantly comparing oneself to those one sees as less able or less successful almost invariably leads to being dismissive of others. So, those who for the most part refrain from comparing themselves to others will most likely be, on the whole, self-accepting and accepting of others.", "question": "The counselor's reasoning is most vulnerable to criticism because it", "answers": "['takes for granted that if one is both dismissive of others and self-disparaging, one will not be self-accepting and accepting of others', 'overlooks the possibility that constantly comparing oneself to others may have beneficial effects that those who refrain from making such comparisons are deprived of', 'overlooks the possibility that self-disparagement and being dismissive of others can result from something other than comparing oneself to others', 'overlooks the possibility that one can compare oneself both to those one perceives to be more able and more successful than oneself and to those one perceives to be less able and less successful than oneself']", "label": 2 }, { "id": "train_3063", "context": "The government-owned gas company has begun selling stoves and other gas appliances to create a larger market for its gas. Merchants who sell such products complain that the competition will hurt their businesses. That may well be; however, the government-owned gas company is within its rights. After all, the owner of a private gas company might well decide to sell such appliances and surely there would be nothing wrong with that.", "question": "Which one of the following principles, if valid, most helps to justify the reasoning above?", "answers": "['There is nothing wrong with a government- owned company selling products so long as owners of private companies do not complain.', 'A government should always take seriously the complaints of merchants.', 'Government-owned companies have the right to do whatever private businesses have the right to do.', 'Private businesses have no right to compete with government monopolies.']", "label": 2 }, { "id": "train_3064", "context": "Enterprise Bank currently requires customers with checking accounts to maintain a minimum balance or pay a monthly fee. Enterprise plans to offer accounts with no monthly fee and no minimum-balance requirement; to cover their projected administrative costs of $3 per account per month they plan to charge $30 for overdrawing an account. Since each month on average slightly more than 10 percent of Enterprises customers overdraw their accounts, bank officials predict the new accounts will generate a profit.", "question": "Which of the following, if true, most strongly supports the bank officials prediction?", "answers": "[\"Some of Enterprise Bank's current checking account customers are expected to switch to the new accounts once they are offered.\", 'Customers whose checking accounts do not have a minimum-balance requirement are more likely than others to overdraw their checking accounts .', 'Customers whose checking accounts do not have a minimum-balance requirement are more likely than others to write checks for small amounts .', 'Many checking account customers who occasionally pay a fee for not maintaining a minimum balance in their account generally maintain a balance well above the minimum.']", "label": 1 }, { "id": "train_3065", "context": "Brain scans of people exposed to certain neurotoxins reveal brain damage identical to that found in people suffering from Parkinson' s disease. This fact shows not only that these neurotoxins cause this type of brain damage, but also that the brain damage itself causes Parkinson' s disease. Thus brain scans can be used to determine who is likely to develop Parkinson' s disease.", "question": "The argument contains which one of the following reasoning errors?", "answers": [ "It overestimates the importance of early diagnosis in determining appropriate treatments for people suffering from Parkinson's disease.", "It neglects to specify how the information provided by brain scans could be used either in treating Parkinson's disease or in monitoring the progression of the disease.", "It assumes that people would want to know as early as possible whether they were likely to develop Parkinson's disease.", "It mistakes a correlation between the type of brain damage described and Parkinson's disease for a causal relation between the two." ], "label": 0 }, { "id": "train_3066", "context": "Art historian: Great works of art have often elicited outrage when first presented; in Europe, Stravinsky' s Rite of Spring prompted a riot, and Manet' s Dejeuner sur 1' herbe elicited outrage and derision. So, since it is clear that art is often shocking, we should not hesitate to use public funds to support works of art that many people find shocking.", "question": "Which one of the following is an assumption that the art historian's argument requires in order for its conclusion to be properly drawn?", "answers": "['Art used to be more shocking than it currently is.', 'Most art is shocking.', 'Public funds should support art.', 'Anything that shocks is art.']", "label": 2 }, { "id": "train_3067", "context": "Radio producer: Our failure to attract new listeners over the past several years has forced us to choose between devoting some airtime to other, more popular genres of music, and sticking with classical music that appeals only to our small but loyal audience. This audience, however loyal, did not generate enough advertising revenue for us to pay our bills, so if we appeal to them alone, our station risks going out of business. We should not take that risk. We should, ttherefore, devote some airtime to other, more popular genres of music.", "question": "Which one of the following arguments is most similar in its pattern of reasoning to that used by the radio producer?", "answers": "['We should either buy blinds for the windows or make full-length curtains. Blinds would be very expensive to purchase. Thus, if cost is our greatest concern, we should make curtains.', 'We should either make curtains for the windows or buy blinds. Since the windows are not standard sizes, if we buy blinds we will have to special order them. Since we do not have time to wait for special orders, we should make the curtains.', 'For the living room windows, we can make curtains or valances or both. We want to have privacy; and while curtains provide privacy, valances do not. So we should make curtains but not valances.', 'We should either buy blinds or make curtains for the windows. If we buy blinds but do not make valances, the windows will look bare. We should not have bare windows. So if we do not make the curtains, we should make the valances.']", "label": 1 }, { "id": "train_3068", "context": "Mayor McKinney' s policies have often been criticized on the grounds that they benefit only wealthy city residents, but that is not a fair evaluation. Some of McKinney' s policies have clearly benefited the city' s less affluent residents. McKinney actively supported last year' s proposal to lower the city' s high property taxes. Because of this tax decrease, more development is taking place in the city, helping to end the housing shortage and stabilize the rents in the city.", "question": "Which one of the following most accurately expresses the main conclusion of the argument?", "answers": "['The decrease in property taxes that McKinney supported caused more development to take place in the city.', \"The criticism that McKinney's policies benefit only the wealthy is unjustified.\", \"McKinney's efforts helped end the housing shortage and stabilize the rents in the city.\", \"McKinney's policies have often been criticized for benefiting only wealthy city residents.\"]", "label": 1 }, { "id": "train_3069", "context": "Poetry journal patron: Everybody who publishes in The Brick Wall Review has to agree in advance that if a poem is printed in one of its regular issues, the magazine also has the right to reprint it, without monetary compensation, in its annual anthology. The Brick Wall Review makes enough money from sales of its anthologies to cover most operating expenses. So, if your magazine also published an anthology of poems first printed in your magazine, you could depend less on donations. After all, most poems published in your magazine are very similar to those published in The Brick Wall Review.", "question": "Which one of the following, if true, most weakens the patron's argument?", "answers": "[\"The Brick Wall Review's annual poetry anthology always contains a number of poems by famous poets not published in the regular issues of the magazine.\", 'Many of the poets whose work appears in The Brick Wall Review have had several poems rejected for publication by the other magazine under discussion.', 'Neither The Brick Wall Review nor the other magazine under discussion depends on donations to cover most operating expenses.', 'The only compensation poets receive for publishing in the regular issues of the magazines under discussion are free copies of the issues in which their poems appear.']", "label": 0 }, { "id": "train_3070", "context": "With employer-paid training, workers have the potential to become more productive not only in their present employment but also in any number of jobs with different employers. To increase the productivity of their workforce, many firms are planning to maintain or even increase their investments in worker training. But some training experts object that if a trained worker is hired away by another firm, the employer that paid for the training has merely subsidized a competitor. They note that such hiring has been on the rise in recent years.", "question": "Which of the following would, if true, contribute most to defeating the training experts' objection to the firms' strategy?", "answers": "['Research shows that workers whose training is wholly or partially subsidized by their employer tend to get at least as much training as do workers who pay for all their own training.', 'In many industries, employees who take continuing-education courses are more competitive in the job market.', 'More and more educational and training institutions are offering reduced tuition fees to firms that subsidize worker training.', \"For most firms that invest in training their employees, the value added by that investment in employees who stay exceeds the value lost through other employees' leaving to work for other companies.\"]", "label": 3 }, { "id": "train_3071", "context": "Broadcaster: Our radio station has a responsibility to serve the public interest. Hence, when our critics contend that our recent expose of events in the private lives of local celebrities was excessively intrusive, we can only reply that the overwhelming public interest in these matters makes it our responsibility to publicize them.", "question": "Which one of the following is a flaw in the broadcaster's defense of the radio station's practice?", "answers": "['confusing legal responsibility with moral obligation', 'improperly exploiting an ambiguity in the phrase \"public interest\"', 'intentionally failing to specify what is meant by \"excessively intrusive\"', 'assuming without argument that there is a right to privacy']", "label": 1 }, { "id": "train_3072", "context": "Clarissa: The natural sciences would not have made such progress but for the power of mathematics. No observation is worth serious attention unless it is stated precisely in quantitative terms. Myungsook: I disagree. Converting observations into numbers is the hardest and last task; it can be done only when you have thoroughly explored the observations themselves.", "question": "Clarissa and Myungsook's statements provide the most support for claiming that they disagree about whether", "answers": "['successfully doing natural science demands careful consideration of observations not stated precisely in quantitative terms', 'not all observations can be stated precisely in quantitative terms', 'useful scientific theories require the application of mathematics', 'mathematics has been a highly significant factor in the advance of the natural sciences']", "label": 0 }, { "id": "train_3073", "context": "Researcher: The vast majority of a person' s dreams bear no resemblance whatsoever to real events that follow the dreams. Thus, it is unreasonable to believe that one has extrasensory perception solely on the basis of having had several vivid dreams about events that happen after the dreams.", "question": "Which one of the following arguments is most similar in its reasoning to the argument above?", "answers": "['A number of people who die prematurely take aspirin. But it is unreasonable to conclude that aspirin is dangerous. Most people who take aspirin do not die prematurely.', 'Even though many cancer patients experience remissions without drinking herbal tea, it is unreasonable to believe that not drinking herbal tea causes such remissions. Several factors are known to be relevant to cancer remission.', 'Many people who undergo surgery for ulcers show no long-term improvement. So it is unreasonable to believe that surgery for ulcers is effective, even though ulcer surgery benefits many people as well.', 'A significant number of children raised near power lines develop cancer. So it is unreasonable to deny a connection between living near power lines and developing cancer, even though many people living near power lines never develop cancer.']", "label": 0 }, { "id": "train_3074", "context": "Expert: Some people claim that, since food production has thus far increased faster than population has, there is no need to be concerned about widespread food shortages. These people fail to recognize that the planet' s resources allow for food to be produced at only a few times the current amount, beyond which no increase in production will be possible. Thus, widespread food shortages are inevitable.", "question": "Which one of the following, if true, most strengthens the expert's reasoning?", "answers": "['Periodic regional food shortages have occurred at least briefly throughout history.', 'Population will continue to grow at least briefly when food production has reached its maximum level.', \"Food resources from the world's oceans will eventually be fully utilized.\", \"The world's population has recently remained fairly stable because of falling birth rates.\"]", "label": 1 }, { "id": "train_3075", "context": "Excavation of the ancient city of Kourion on the island of Cyprus revealed a pattern of debris and collapsed buildings typical of towns devastated by earthquakes. Archaeologists have hypothesized that the destruction was due to a major earthquake known to have occurred near the island in AD 365.", "question": "Which of the following, if true, most strongly supports the archaeologists' hypothesis?", "answers": "['Bronze ceremonial drinking vessels that are often found in graves dating from years preceding and following AD 365 were also found in several graves near Kourion.', 'Most modern histories of Cyprus mention that an earthquake occurred near the island in AD 365.', 'Stone inscriptions in a form of the Greek alphabet that was definitely used in Cyprus after AD 365 were found in Kourion.', 'No coins minted after AD 365 were found in Kourion, but coins minted before that year were found in abundance.']", "label": 3 }, { "id": "train_3076", "context": "Mark: Advances in technology have caused the decline of newspaper sales. News can be reported in articles on the Internet as it happens and at any time of day. No print newspaper can possibly keep up with that. Fatuma: Newspaper writing is hampered by conventions that do not add to readers' understanding of the news. It is not the fault of technology that newspapers are losing their audience to the Internet. Newspaper articles are just too long. On the Internet, news articles get to the point .", "question": "Mark and Fatuma disagree over the truth of which one of the following?", "answers": "['News articles on the Internet get to the point.', \"Newspaper writing is hampered by conventions that do not add to the reader's understanding of the news.\", 'Readers are abandoning newspapers in favor of news articles on the Internet because of the speed with which they appear on the Internet.', 'Newspaper articles are too long.']", "label": 2 }, { "id": "train_3077", "context": "When scientific journals began to offer full online access to their articles in addition to the traditional printed volumes, scientists gained access to more journals and easier access to back issues. Surprisingly, this did not lead to a broader variety of articles being cited in new scientific articles. Instead, it led to a greater tendency among scientists to cite the same articles that their fellow scientists cited.", "question": "Which one of the following, if true, most helps to explain the surprising outcome described above?", "answers": "['Several new scientific journals appeared at roughly the same time that full online access to scientific articles became commonplace.', 'Online searching made it easier for scientists to identify the articles that present the most highly regarded views on an issue, which they prefer to cite.', 'Scientists who wrote a lot of articles were the most enthusiastic about accessing journal articles online.', 'Scientists are more likely to cite articles by scientists that they know than they are to cite articles by scientists they have never met, even if the latter are more prominent.']", "label": 1 }, { "id": "train_3078", "context": "Nearly all mail that is correctly addressed arrives at its destination within two business days of being sent. In fact, correctly addressed mail takes longer than this only when it is damaged in transit. Overall, however, most mail arrives three business days or more after being sent.", "question": "If the statements above are true, which one of the following must be true?", "answers": "['More mail arrives within two business days of being sent than arrives between two and three business days after being sent.', 'A large proportion of the mail that is correctly addressed is damaged in transit.', 'A large proportion of mail is incorrectly addressed.', 'Most mail that arrives within two business days of being sent is correctly addressed.']", "label": 2 }, { "id": "train_3079", "context": "Whoever is kind is loved by somebody or other, and whoever loves anyone is happy. It follows that whoever is kind is happy.", "question": "The conclusion follows logically if which one of the following is assumed?", "answers": "['Whoever loves no one is loved by no one.', 'Whoever is happy loves everyone.', 'Whoever loves everyone loves someone.', 'Whoever loves everyone is kind.']", "label": 0 }, { "id": "train_3080", "context": "Geneticist: Genes, like viruses, have a strong tendency to self-replicate; this has led some biologists to call genes \"selfish. \" This term is, in this instance, intended to be defined behaviorally: it describes what genes do without ascribing intentions to them. But even given that genes are ascribed no intentions, the label \"selfish\" as applied to genes is a misnomer. Selfishness only concerns bringing about the best conditions for oneself; creating replicas of oneself is not selfish.", "question": "Which one of the following, if assumed, allows the geneticist's conclusion to be properly drawn?", "answers": "['Creating replicas of oneself does not help bring about the best conditions for oneself.', 'Bringing about the best conditions for oneself is less important than doing this for others.', 'The behavioral definition of \"selfish\" is incompatible with its everyday definition.', 'Biologists have insufficient evidence about genetic behavior to determine whether it is best described as selfish.']", "label": 0 }, { "id": "train_3081", "context": "When a society undergoes slow change, its younger members find great value in the advice of its older members. But when a society undergoes rapid change, young people think that little in the experience of their elders is relevant to them, and so do not value their advice. Thus, we may measure the rate at which a society is changing by measuring the amount of deference its younger members show to their elders.", "question": "Which one of the following is an assumption on which the argument depends?", "answers": "[\"Young people value their elders' advice just insofar as the elders' experience is practically useful to them.\", \"The deference young people show to their elders varies according to how much the young value their elders' advice.\", 'The faster a society changes, the less relevant the experience of older members of the society is to younger members.', \"How much deference young people show to their elders depends on how much of the elders' experience is practically useful to them.\"]", "label": 1 }, { "id": "train_3082", "context": "Professor: It has been argued that freedom of thought is a precondition for intellectual progress, because freedom of thought allows thinkers to pursue their ideas, regardless of whom these ideas offend, in whatever direction they lead. However, it is clear that one must mine the full implications of interrelated ideas to make intellectual progress, and for this, thinkers need intellectual discipline. Ttherefore, this argument for freedom of thought fails.", "question": "The conclusion drawn by the professor follows logically if which one of the following is assumed?", "answers": "['Without intellectual discipline, thinkers can have no freedom of thought.', 'Thinkers who limit their line of thought to a particular orthodoxy are hindered in their intellectual progress.', 'Freedom of thought engenders creativity, which aids the discovery of truth.', 'In societies that protect freedom of thought, thinkers invariably lack intellectual discipline.']", "label": 3 }, { "id": "train_3083", "context": "City council member: Demand for electricity has been increasing by 1. 5 percent a year, and there simply is no more space to build additional power plants to meet future demand increases. We must ttherefore begin to curtail usage, which is why I propose passing ordinances requiring energy-conservation measures in all city departments.", "question": "The city council member's proposal assumes which of the following?", "answers": "['Residential consumers are not responsible for the recent increases in demand for electricity.', 'Passing ordinances designed to curtail electricity usage will not have negative economic consequences for the city.', 'City departments that successfully conserve energy will set a good example for residential and industrial consumers of electricity.', 'Existing power plants do not have the capacity to handle all of the projected increase in demand for electricity.']", "label": 3 }, { "id": "train_3084", "context": "A recent study of major motion pictures revealed that the vast majority of their plots were simply variations on plots that had been used many times before. Despite this fact, many people enjoy seeing several new movies each year.", "question": "Each of the following, if true, would contribute to an explanation of the apparent discrepancy in the information above EXCEPT:", "answers": "['If the details of their stories are sufficiently different, two movies with the same basic plot will be perceived by moviegoers as having different plots.', 'Because of the large number of movies produced each year, the odds of a person seeing two movies with the same general plot structure in a five-year period are fairly low.', 'Although most modern movie plots have been used before, most of those previous uses occurred during the 1940s and 1950s.', 'Movies based on standard plots are more likely to be financially successful than are ones based on original plots.']", "label": 3 }, { "id": "train_3085", "context": "Columnist: Some people argue that the government should not take over failing private-sector banks because the government does not know how to manage financial institutions. However, rather than managing a bank' s day-to-day operations, the government would just need to select the bank' s senior management. Most politicians have never been military professionals, yet they appoint the top military officials entrusted with defending the country -- at least as great a responsibility as managing a bank.", "question": "The columnist's statements, if true, provide reason for rejecting which one of the following?", "answers": "['Politicians do an adequate job of appointing the top military officials entrusted with defending the country.', 'Banks that are owned by the government cannot be well managed.', 'The government should not take over private-sector banks that are financially sound.', \"Politicians are not capable of managing a bank's day-to-day operations.\"]", "label": 1 }, { "id": "train_3086", "context": "Outsourcing is the practice of obtaining from an independent supplier a product or service that a company has previously provided for itself. Since a company's chief objective is to realize the highest possible year-end profits, any product or service that can be obtained from an independent supplier for less than it would cost the company to provide the product or service on its own should be outsourced.", "question": "Which of the following, if true, most seriously weakens the argument?", "answers": "['If a company decides to use independent suppliers for a product, it can generally exploit the vigorous competition arising among several firms that are interested in supplying that product.', \"When a company decides to use an independent supplier for a product or service, the independent supplier sometimes hires members of the company's staff who formerly made the product or provided the service that the independent supplier now supplies.\", \"Certain tasks, such as processing a company's payroll, are commonly outsourced, whereas others, such as handling the company's core business, are not.\", 'Successful outsourcing requires a company to provide its suppliers with information about its products and plans that can fall into the hands of its competitors and give them a business advantage.']", "label": 3 }, { "id": "train_3087", "context": "Robot satellites relay important communications and identify weather patterns. Because the satellites can be repaired only in orbit, astronauts are needed to repair them. Without repairs, the satellites would eventually malfunction. Ttherefore, space flights carrying astronauts must continue.", "question": "Which of the following, if true, would most seriously weaken the argument above?", "answers": "['Although satellites are indispensable in the identification of weather patterns, weather forecasters also make some use of computer projections to identify weather patterns.', 'The government, responding to public pressure, has decided to cut the budget for space flights and put more money into social welfare programs.', 'Satellites falling from orbit because of malfunctions burn up in the atmosphere.', 'Technical obsolescence of robot satellites makes repairing them more costly and less practical than sending new, improved satellites into orbit.']", "label": 3 }, { "id": "train_3088", "context": "The country of Baurisia has, until now, been self-sufficient in both grain and meat. However, with growing prosperity in Baurisia has come a steadily increasing per capita consumption of meat, and it takes several pounds of grain to produce one pound of meat. Ttherefore, since per capita income in Baurisia is almost certain to rise further but increases in domestic grain production are highly unlikely, Baurisia is soon likely to become an importer of grain.", "question": "Which of the following, if true, most seriously weakens the argument?", "answers": "['The per capita consumption of meat in Baurisia is roughly the same across all income levels.', 'It is more economical for Baurisians to import meat than grain.', 'When people increase their consumption of meat, they also tend to increase their consumption of grain.', \"During Baurisia's years of growing prosperity, the country's population has remained relatively stable.\"]", "label": 1 }, { "id": "train_3089", "context": "Twenty percent of the stores in Morganville' s downtown shopping district will fail within five years because they will be competing directly with the SaveMart discount department store newly opened in East Morganville. The downtown shopping district has lost business at this rate before and has always completely rebounded. Confidence that it will rebound again from the losses it is now about to suffer is ill founded, however, because __.", "question": "Which of the following most logically completes the argument below?", "answers": "['it is conceivable that the downtown shopping district could shrink substantially without collapsing altogether', \"the stores likely to be put out of business by direct competition from SaveMart are the downtown shopping district's anchor stores, on whose ability to draw shoppers many of the other downtown stores depend\", 'the bus line that has long connected the downtown area of Morganville with East Morganville has a tradition of carrying shoppers who reside in East Morganville into downtown Morganville to shop', 'when the downtown shopping district has rebounded before, the business premises of a failed business were typically taken over by a business of the same kind as had been there before']", "label": 1 }, { "id": "train_3090", "context": "Gardener: Researchers encourage us to allow certain kinds of weeds to grow among garden vegetables because they can repel caterpillars from the garden. While it is wise to avoid unnecessary use of insecticides, the researchers' advice is premature. For all we know, those kinds of weeds can deplete the soil of nutrients and moisture that garden crops depend on, and might even attract other kinds of damaging pests.", "question": "Which one of the following most accurately expresses the main conclusion of the gardener's argument?", "answers": "['Allowing certain kinds of weeds to grow in vegetable gardens may contribute to a net increase in unwanted garden pests.', 'Allowing the right kinds of weeds to grow in vegetable gardens can help toward controlling caterpillars without the use of insecticides.', 'To the extent that it is possible to do so, we should eliminate the use of insecticides in gardening.', 'We should be cautious about the practice of allowing certain kinds of weeds to grow among garden vegetables.']", "label": 3 }, { "id": "train_3091", "context": "Ecologist: Before finding a mate, male starlings decorate their nests with fragments of aromatic plants rich in compounds known to kill parasitic insects. Since these parasites are potentially harmful to nestlings, some researchers have hypothesized that the function of these decorations is nestling protection. However, males cease to incorporate such greenery once egg laying starts, which suggests instead that the function of the decorations is to attract females.", "question": "Which one of the following, if true, most strengthens the support for the ecologist's conclusion?", "answers": "['Nestlings grow faster in nests that incorporate aromatic plants than in nests that do not.', 'Male starlings tend to decorate their nests with a greater number of aromatic plants when a caged female is positioned adjacent to the nest.', 'The compounds in the aromatic plants used by the male starlings to decorate their nests are harmless to nestlings.', 'Male starlings do not decorate their nests in areas with unusually small populations of parasitic insects.']", "label": 1 }, { "id": "train_3092", "context": "The Gulches is an area of volcanic rock that is gashed by many channels that lead downhill from the site of a prehistoric glacier to a river. The channels clearly were cut by running water. It was once accepted as fact that the cutting occurred gradually, as the glacier melted. But one geologist theorized that the channels were cut in a short time by an enormous flood. The channels do show physical evidence of having been formed quickly, but the flood theory was originally rejected because scientists knew of no natural process that could melt so much ice so quickly. Paradoxically, today the scientific community accepts the flood theory even though scientists still do not know of a process that can melt so much ice so quickly.", "question": "Which one of the following is supported by the information in the passage?", "answers": "['The physical effects of water on rock vary with the speed with which those effects are produced.', 'Only running water can cause deep channels in volcanic rock.', 'The river did not exist before the channels were cut.', 'Geologists cannot determine the amount of heat required to melt a glacier quickly.']", "label": 0 }, { "id": "train_3093", "context": "When people evade income taxes by not declaring taxable income, a vicious cycle results. Tax evasion forces lawmakers to raise income tax rates, which causes the tax burden on nonevading taxpayers to become heavier. This, in turn, encourages even more taxpayers to evade income taxes by hiding taxable income.", "question": "The vicious cycle described above could not result unless which of the following were true?", "answers": "['An increase in tax rates tends to function as an incentive for taxpayers to try to increase their pretax incomes.', 'No one who routinely hides some taxable income can be induced by a lowering of tax rates to stop hiding such income unless fines for evaders are raised at the same time.', 'When lawmakers establish income tax rates in order to generate a certain level of revenue, they do not allow adequately for revenue that will be lost through evasion.', 'Some methods for detecting tax evaders, and thus recovering some tax revenue lost through evasion, bring in more than they cost, but their success rate varies from year to year.']", "label": 2 }, { "id": "train_3094", "context": "Curator: Our museum displays only twentieth-century works, which are either on loan from private collectors or in the museum' s permanent collection. Prints of all of the latter works are available in the museum store. The museum store also sells prints of some works that are not part of the museum' s permanent collection, such as Hopper' s Nighthawks.", "question": "If the curator's statements are true, which one of the following must be true?", "answers": "['Every print that is sold in the museum store is a copy of a twentieth-century work.', \"Hopper's Nighthawks is not displayed in the museum.\", 'There are prints in the museum store of every work that is displayed in the museum and not on loan from a private collector.', \"Every print in the museum store is of a work that is either on loan to the museum from a private collector or part of the museum's permanent collection.\"]", "label": 2 }, { "id": "train_3095", "context": "All smart people read more than six books per year, and the smartest people read more than twelve books per year. John is not a smart person.", "question": "If the statements above are correct, which one of the following must be true?", "answers": "['John reads twelve books per year.', 'John reads more than six books per year.', 'John reads fewer than seven books per year.', 'John reads six books per year.']", "label": 2 }, { "id": "train_3096", "context": "Lawyer: In addition to any other penalties, convicted criminals must now pay a \"victim surcharge\" of $30. The surcharge is used to fund services for victims of violent crimes, but this penalty is unfair to nonviolent criminals since the surcharge applies to all crimes, even nonviolent ones like petty theft.", "question": "Which one of the following principles, if valid, would most help to justify the reasoning in the lawyer's argument?", "answers": "['Convicted thieves should be fined an amount at least as great as the value of the property stolen.', 'A criminal should not be required to pay for services provided to victims of crimes that are more serious than the type of crime the criminal has been convicted of.', 'The penalties for a crime should be severe enough to deter most people who would commit the crime if there were no penalties.', 'A surcharge intended to provide services to victims is justified only if all proceeds of the surcharge are used to provide services.']", "label": 1 }, { "id": "train_3097", "context": "Insurers and doctors are well aware that the incidence of lower-back injuries among office workers who spend long hours sitting is higher than that among people who regularly do physical work of a type known to place heavy stresses on the lower back. This shows that office equipment and furniture are not properly designed to promote workers' health.", "question": "Which one of the following, if true, most undermines the reasoning above?", "answers": "['Consistent physical exercise is one of the most effective ways to prevent or recover from lower-back injuries.', 'Most of the lower-back injuries that office workers suffer occur while they are on the job.', 'When they are at home, laborers and office workers tend to spend similar amounts of time sitting.', 'Insurance companies tend to dislike selling policies to companies whose workers often claim to have back pain.']", "label": 0 }, { "id": "train_3098", "context": "S: People who are old enough to fight for their country are old enough to vote for the people who make decisions about war and peace. This government clearly regards 17 year olds as old enough to fight, so it should acknowledge their right to vote. T: Your argument is a good one only to the extent that fighting and voting are the same kind of activity. Fighting well requires strength, muscular coordination, and in a modern army instant and automatic response to orders. Performed responsibly, voting, unlike fighting, is essentially a deliberative activity requiring reasoning power and knowledge of both history and human nature.", "question": "T responds to S's argument by", "answers": "[\"calling into question S's understanding of the concept of rights\", 'arguing for a conclusion opposite to the one drawn by S', \"challenging the truth of a claim on which S's conclusion is based\", 'showing that S has ignored the distinction between having a right to do something and having an obligation to do that thing']", "label": 2 }, { "id": "train_3099", "context": "The Malbec grape, originally grown in France, has become the main varietal in Argentina. This is surprising because most Malbec grown in Argentina is grown at high altitudes, whereas the Malbec grape once was grown at low altitudes. Ttherefore, Argentinian winegrowers should grow the Malbec grape at low elevations.", "question": "Which of the following, if true, would most weaken the conclusion of the argument?", "answers": "['The Malbec vine is susceptible to phylloxera, a plant louse that only grows at low altitudes.', 'Some varietals are unable to grow at high altitudes.', 'Malbec has recently enjoyed a surge in popularity, and can be found in many different countries.', \"The soil at high altitudes is filled with nutrients that help many of Argentina's varietals flourish.\"]", "label": 0 }, { "id": "train_3100", "context": "Pecan growers get a high price for their crop when pecans are comparatively scarce, but the price drops sharply when pecans are abundant. Thus, in high-yield years, growers often hold back part of their crop in refrigerated warehouses for one or two years, hoping for higher prices in the future. This year' s pecan crop was the smallest in five years. It is nonetheless quite possible that a portion of this year' s crop will be held back, since __.", "question": "Which of the following most logically completes the passage?", "answers": "[\"the practice of holding back part of one year's crop had not yet become widespread the last time the pecan crop was as small as it was this year\", \"the quality of this year's pecan crop is no worse than the quality of the pecan crops of the previous five years\", 'each of the last two years produced record- breaking pecan yields', \"for some pecan growers, this year's crop was no smaller than last year's\"]", "label": 2 }, { "id": "train_3101", "context": "Graham: The defeat of the world' s chess champion by a computer shows that any type of human intellectual activity governed by fixed principles can be mastered by machines and thus that a truly intelligent machine will inevitably be devised. Adelaide: But you are overlooking the fact that the computer in the case you cite was simply an extension of the people who programmed it. It was their successful distillation of the principles of chess that enabled them to defeat a chess champion using a computer.", "question": "The statements above provide the most support for holding that Graham and Adelaide disagree about whether", "answers": "['intelligence can be demonstrated by the performance of an activity in accord with fixed principles', \"a computer's defeat of a human chess player is an accomplishment that should be attributed to the computer\", 'tools can be designed to aid in any human activity that is governed by fixed principles', 'chess is the best example of a human intellectual activity that is governed by fixed principles']", "label": 1 }, { "id": "train_3102", "context": "Arnold: I was recently denied a seat on an airline flight for which I had a confirmed reservation, because the airline had overbooked the flight. Since I was forced to fly on the next available flight, which did not depart until two hours later, I missed an important business meeting. Even though the flight on which I had a reservation was canceled at the last minute due to bad weather, the airline should still pay me compensation for denying me a seat on the flight. Jamie: The airline is not morally obligated to pay you any compensation. Even if you had not been denied a seat on the earlier flight, you would have missed your business meeting anyway.", "question": "A principle that, if established, justifies Jamie's response to Arnold is that an airline is morally obligated to compensate a passenger who has been denied a seat on a flight for which the passenger has confirmed reservations", "answers": "['only if the passenger would not have been forced to take a later flight had the airline not overbooked the original flight', 'even if the passenger would still have been forced to take a later flight had the airline not overbooked the original flight', 'if the only reason the passenger is forced to take a later flight is that the airline overbooked the original flight', 'even if the only reason the passenger is forced to take a later flight were that the original flight is canceled due to bad weather']", "label": 0 }, { "id": "train_3103", "context": "The hepatitis B virus is common in China relative to other countries in the region. The disease can easily be prevented through vaccination, regulation of barbershop and manicure practices, and health education. Yet, the incidence of this disease continues to rise in China, while it remains stable or is falling in China's neighboring countries.", "question": "Which of the following, if true, would best explain the situation described above?", "answers": "['Twenty years ago, a complete and accurate study by an international health agency revealed that the incidence of hepatitis B was much higher in China than in neighboring countries.', 'Medicine is socialized in China but not in neighboring countries, meaning that more people in China have access to health care than in neighboring countries.', 'To speed its industrial development and the education level of its citizenry, China does not burden small businesses with many regulations nor does it require its school population to have vaccinations, as neighboring countries do.', 'Illegal drug use has risen faster in China than in neighboring countries.']", "label": 2 }, { "id": "train_3104", "context": "Until now, only injectable vaccines against influenza have been available. They have been used primarily by older adults, who are at risk for complications from influenza. A new vaccine administered in a nasal spray has proven effective in preventing influenza in children. Since children are significantly more likely than adults to contract and spread influenza, making the new vaccine widely available for children will greatly reduce the spread of influenza across the population.", "question": "Which of the following, if true, most strengthens the argument?", "answers": "['The new vaccine uses the same mechanism to ward off influenza as injectable vaccines do.', 'Of the older adults who contract influenza, relatively few contract it from children with influenza.', 'Many parents would be more inclined to have their children vaccinated against influenza if the vaccination did not require an injection.', 'Government subsidies have kept the injectable vaccine affordable for all older adults.']", "label": 2 }, { "id": "train_3105", "context": "The existing works of ancient literature, science, and philosophy do not represent the most original work produced in ancient times. The most compelling evidence for this view is that the existing works frequently refer to other works that we no longer have.", "question": "Which one of the following statements, if added to the argument, most helps to justify its conclusion?", "answers": "['Subversive ideas tend to be suppressed whenever they are proposed.', 'Works that contain frequent references to other works tend to be derivative.', 'Many extant works have laudable things to say about the missing works.', 'A surprisingly large number of ancient texts have been irretrievably lost.']", "label": 1 }, { "id": "train_3106", "context": "The Green Peas Grocery Store in the remote wealthy enclave of Luxville charges more than the Green Peas Grocery Store in Oak City charges for the same items. Clearly, on any given item, the Green Peas grocery franchise is taking advantage of its location in Luxville to reap higher profits on that item.", "question": "In evaluating the argument, it would be most useful to compare", "answers": "['The percent of average household income spent on groceries in Oak city with the comparable percentage in Luxville', 'The cost of these items in Oak City and in Luxville with the cost at other Green Peas stores throughout the state.', 'the average cost of the same or comparable items at other grocery stores in Oak City with the average cost at other stores in Luxville', 'the cost of transporting merchandise to the Oak City location with the comparable cost to the Luxville location']", "label": 3 }, { "id": "train_3107", "context": "Ethicist: Every moral action is the keeping of an agreement, and keeping an agreement is nothing more than an act of securing mutual benefit. Clearly, however, not all instances of agreement-keeping are moral actions. Ttherefore, some acts of securing mutual benefit are not moral actions.", "question": "The pattern of reasoning in which one of the following arguments is most similar to that in the ethicist's argument?", "answers": "['All metaphors are comparisons, and not all comparisons are surprising. However, all metaphors are surprising. Ttherefore, some comparisons are not metaphors.', 'All calculators are kinds of computers, and all computers are devices for automated reasoning. However, not all devices for automated reasoning are calculators. Ttherefore, some devices for automated reasoning are not computers.', 'All books are texts, and all texts are documents. However, not all texts are books. Ttherefore, some documents are not books.', 'All architecture is design and all design is art. However, not all design is architecture. Ttherefore, some art is not design.']", "label": 2 }, { "id": "train_3108", "context": "The supernova event of 1987 is interesting in that there is still no evidence of the neutron star that current theory says should have remained after a supernova of that size. This is in spite of the fact that many of the most sensitive instruments ever developed have searched for the tell-tale pulse of radiation that neutron stars emit. Thus, current theory is wrong in claiming that supernovas of a certain size always produce neutron stars.", "question": "Which one of the following, if true, most strengthens the argument?", "answers": "['Most supernova remnants that astronomers have detected have a neutron star nearby.', 'The supernova of 1987 was the first that scientists were able to observe in progress.', 'Several important features of the 1987 supernova are correctly predicted by the current theory.', 'Sensitive astronomical instruments have detected neutron stars much farther away than the location of the 1987 supernova.']", "label": 3 }, { "id": "train_3109", "context": "Commercially available chemical fertilizers contain the same nutrients and micronutrients as organic fertilizers such as manure and composted garden clippings. Plants have tiny root hairs that absorb nutrients, and whether the source of the nutrients is organic or chemical is irrelevant to the plant's ability to take in the nutrients. Nevertheless, organic fertilizers are better for the health of the plant than are chemical fertilizers alone.", "question": "Which of the following, if true, would most strongly support the position above?", "answers": "[\"The typical gardener can't easily know the exact amount and type of nutrients in an organic fertilizer such as manure.\", 'The nutrients in organic fertilizers are slowly released to the plant over time, rather than being absorbed by the plant immediately.', 'In organic fertilizers, the natural combination of nutrients with other natural materials makes the nutrients more usable by the plant than nutrients found in chemical fertilizers.', \"Chemical fertilizers produced by the leading manufacturer contain nutrients that are no harder for the plants to absorb than those provided by the competitor's chemical fertilizer.\"]", "label": 2 }, { "id": "train_3110", "context": "Attorney for Ziegler: My client continued to do consulting work between the time of his arrest for attempted murder and the start of this trial. But I contend that Ziegler was insane at the time that he fired the shot. This is the only reasonable conclusion to draw from the fact that the accusers have submitted no evidence that he was sane at the time he pulled the trigger, only that he was sane some time after he did so.", "question": "Which one of the following most accurately describes a flaw in the reasoning of Ziegler's attorney?", "answers": "['It presumes that being a well-educated professional is relevant to being guilty or innocent.', 'It fails to consider that Ziegler might have been insane when he worked as a consultant.', \"It fails to consider the possibility that Ziegler's being sane after the shooting is an indication that he was sane at the time of the shooting.\", \"It concludes on the basis of evidence against Ziegler's being sane that there is a lack of evidence for Ziegler's being sane.\"]", "label": 2 }, { "id": "train_3111", "context": "The Jacksons regularly receive wrong-number calls for Sara, whose phone number was misprinted in a directory. Sara contacted the Jacksons, informing them of the misprint and her correct number. The Jacksons did not lead Sara to believe that they would pass along the correct number, but it would be helpful to Sara and of no difficulty for them to do so. Thus, although it would not be wrong for the Jacksons to tell callers trying to reach Sara merely that they have dialed the wrong number, it would be laudable if the Jacksons passed along Sara' s correct number.", "question": "Which one of the following principles, if valid, most helps to justify the reasoning in the argument?", "answers": "['Being helpful to someone is laudable whenever it is not wrong to do so.', 'It is always laudable to do something helpful to someone, but not doing so would be wrong only if one has led that person to believe one would do it.', 'If one can do something that would be helpful to someone else and it would be easy to do, then it is laudable and not wrong to do so.', 'The only actions that are laudable are those that it would not be wrong to refrain from doing, whether or not it is difficult to do so.']", "label": 1 }, { "id": "train_3112", "context": "Legislator: Your agency is responsible for regulating an industry shaken by severe scandals. You were given funds to hire 500 investigators to examine the scandals, but you hired no more than 400. I am forced to conclude that you purposely limited hiring in an attempt to prevent the full extent of the scandals from being revealed. Regulator: We tried to hire the 500 investigators but the starting salaries for these positions had been frozen so low by the legislature that it was impossible to attract enough qualified applicants.", "question": "The regulator responds to the legislator's criticism by", "answers": "[\"showing that the legislator's statements are self-contradictory\", \"claiming that compliance with the legislature's mandate would have been an insufficient response\", \"rephrasing the legislator's conclusion in terms more favorable to the regulator\", 'providing information that challenges the conclusion drawn by the legislator']", "label": 3 }, { "id": "train_3113", "context": "Doctor: Being overweight has long been linked with a variety of health problems, such as high blood pressure and heart disease. But recent research conclusively shows that people who are slightly overweight are healthier than those who are considerably underweight. Ttherefore, to be healthy, it suffices to be slightly overweight.", "question": "The argument's reasoning is flawed because the argument", "answers": "['does not take into account the fact that appropriate weight varies greatly from person to person', 'ignores medical opinions that tend to lead to a conclusion contrary to the one drawn', 'never adequately defines what is meant by \"healthy\"', 'mistakes a merely relative property for one that is absolute']", "label": 3 }, { "id": "train_3114", "context": "Raymond Burr played the role of Perry Mason on television. Burr' s death in 1993 prompted a prominent lawyer to say, \"Although not a lawyer, Mr. Burr strove for such authenticity that we feel as if we lost one of our own. \" This comment from a prestigious attorney provides appalling evidence that, in the face of television, even some legal professionals are losing their ability to distinguish fiction from reality.", "question": "The reasoning in the argument is flawed because the argument", "answers": "['presumes that the lawyer is qualified to evaluate the performance of an actor', 'takes the views of one lawyer to represent the views of all lawyers', \"criticizes the lawyer rather than the lawyer's statement\", \"ignores the part of the lawyer's remark that indicates an awareness of the difference between reality and fiction\"]", "label": 3 }, { "id": "train_3115", "context": "Ten years ago the number of taxpayers in Green County was slightly greater than the number of registered voters. The number of taxpayers has doubled over the last ten years, while the number of registered voters has increased, but at a lower rate than has the number of taxpayers.", "question": "Which of the following must be true in Greenspace County if the statements above are true?", "answers": "['The proportion of registered voters to taxpayers has remained unchanged over the last ten years.', 'Everyone who is a registered voter is also a taxpayer.', 'The number of taxpayers is now smaller than the number of registered voters.', 'The proportion of registered voters to taxpayers has decreased over the last ten years.']", "label": 3 }, { "id": "train_3116", "context": "However, other research consistently shows that, in Naota, the average annual earnings of all employed women is 65 percent of the average annual earnings of all employed men, Which one of the following, if also established by research, most helps explain the apparent discrepancy between the research results described above?", "question": "A study of the difference in earnings between men and women in the country of Naota found that the average annual earnings of women who are employed full time is 80 percent of the average annual earnings of men who are employed full time.", "answers": "['In Naota, a larger proportion of female workers than male workers are part-time workers, and part-time workers typically earn less than full-time workers earn.', 'In Naota, the difference between the average annual earnings of all female workers and the average annual earnings of all male workers has been gradually increasing over the past 30 years.', 'In ten other countries where the proportion of women in the work force is similar to that of Naota, the average annual earnings of women who work full time ranges from a low of 50 percent to a high of 90 percent of the average annual earnings of men who work full time.', \"In Naota, the average annual earnings of women who work full time in exactly the same occupations and under exactly the same conditions as men is almost the same as the men's average annual earnings.\"]", "label": 0 }, { "id": "train_3117", "context": "A lack of trust in one' s neighbors leads to their lack of respect for the law. A new study provides compelling evidence for this. Neighborhoods in which people routinely lock their doors have higher burglary rates than neighborhoods in which people do not routinely lock their doors.", "question": "The reasoning in the argument is flawed in that the argument", "answers": "['bases its conclusion on data that are contradictory', 'asserts in a premise what it is trying to establish in its conclusion', 'treats what could be the effect of something as if it were the cause of that thing', 'treats something that is merely sufficient to produce a result as if it were necessary to produce that result']", "label": 2 }, { "id": "train_3118", "context": "Researchers have developed a membrane that quickly removes the oxygen from a refrigerator, thereby killing bacteria and enabling food to be stored almost indefinitely. Since food need not be kept as cold to prevent spoilage, energy costs will be greatly reduced. Thus, over the long run, oxygen-removing refrigerators will prove far less expensive than current models.", "question": "The reasoning in the argument is most vulnerable to criticism on the grounds that it does not", "answers": "['take into account the inconvenience caused by food spoilage in current refrigerators', 'take into account the effectiveness of current refrigerator technology in preventing food spoilage', 'explain the technology that enabled the oxygen-removing membrane to be developed', 'address the expense of building or refitting a refrigerator with the new technology']", "label": 3 }, { "id": "train_3119", "context": "Robert: The school board is considering adopting a year-round academic schedule that eliminates the traditional three-month summer vacation. This schedule should be adopted, since teachers need to cover more new material during the school year than they do now. Samantha: The proposed schedule will not permit teachers to cover more new material. Even though the schedule eliminates summer vacation, it adds six new two-week breaks, so the total number of school days will be about the same as before.", "question": "Which one of the following, if true, is a response Robert could make that would counter Samantha's argument?", "answers": "['Teachers spend no more than a day of class time reviewing old material when students have been away from school for only a few weeks, but have to spend up to a month of class time reviewing after a three-month summer vacation.', 'Most parents who work outside the home find it difficult to arrange adequate supervision for their school-age children over the traditional three-month summer vacation.', 'In school districts that have adopted a year-round schedule that increases the number of school days per year, students show a deeper understanding and better retention of new material.', 'Students prefer taking a long vacation from school during the summer to taking more frequent but shorter vacations spread throughout the year.']", "label": 0 }, { "id": "train_3120", "context": "The law of the city of Weston regarding contributions to mayoral campaigns is as follows: all contributions to these campaigns in excess of $100 made by nonresidents of Weston who are not former residents of Weston must be registered with the city council. Brimley' s mayoral campaign clearly complied with this law since it accepted contributions only from residents and former residents of Weston.", "question": "If all the statements above are true, which one of the following statements must be true?", "answers": [ "Some contributions to Brimley's campaign in excess of $100 were registered with the city council.", "No nonresident of Weston contributed in excess of $100 to Brimley's campaign.", "All contributions to Brimley's campaign that were registered with the city council were in excess of $100.", "No contributions to Brimley's campaign needed to be registered with the city council." ], "label": 3 }, { "id": "train_3121", "context": "For many centuries it was believed that only classical Euclidean geometry could provide a correct way of mathematically representing the universe. Nevertheless, scientists have come to believe that a representation of the universe employing non-Euclidean geometry is much more useful in developing certain areas of scientific theory. In fact, such a representation underlies the cosmological theory that is now most widely accepted by scientists as accurate.", "question": "Which one of the following is most strongly supported by the statements above?", "answers": "['Scientists generally do not now believe that classical Euclidean geometry is uniquely capable of giving a correct mathematical representation of the universe.', 'Non-Euclidean geometry is a more complete way of representing the universe than is Euclidean geometry.', 'Scientists who use Euclidean geometry are likely to believe that progress in mathematical theory results in progress in natural science.', 'The usefulness of a mathematical theory is now considered by scientists to be more important than its mathematical correctness.']", "label": 0 }, { "id": "train_3122", "context": "All police officers carry guns, and all soldiers carry guns; ttherefore, police officers and soldiers are similar.", "question": "Which one of the following most closely parallels the argument?", "answers": "['All dogs have fur, and all mammals have fur; ttherefore, all dogs are mammals.', 'Farms and fisheries are similar. All farms produce food, and all fisheries produce food.', 'Some professional football players lift weights every day, and all bodybuilders lift weights every day; ttherefore, football players and bodybuilders are similar.', 'All apples have seeds and all grapefruits have seeds.']", "label": 1 }, { "id": "train_3123", "context": "Several cosmetics firms are committed to the active development, validation, and adoption of new product-safety tests that use cultures of human cells. They argue that the new tests serve to reduce the need for tests on live animals.", "question": "The statements above most strongly support which one which one of the following conclusions?", "answers": "['The pressure on cosmetics firms to cease conducting experiments that use live animals was initiated by groups of social activists.', 'Researchers in the cosmetics firms believe that fewer tests of products will be needed if cell cultures rather than live animals are used.', 'Consumers are no more likely to buy products whose safety was tested on cultures of human cells than they are to buy products whose safety was tested on animals.', 'Managers of the cosmetics firms believe that it is better for their firms not to perform tests on live animals if there is an acceptable alternative way of determining product safety.']", "label": 3 }, { "id": "train_3124", "context": "It is very difficult to prove today that a painting done two or three hundred years ago, especially one without a signature or with a questionably authentic signature, is indubitably the work of this or that particular artist. This fact gives the traditional attribution of a disputed painting special weight, since that attribution carries the presumption of historical continuity. Consequently, an art historian arguing for a deattribution will generally convince other art historians only if he or she can persuasively argue for a specific reattribution.", "question": "Which one of the following, if true, most strongly supports the position that the traditional attribution of a disputed painting should not have special weight?", "answers": "['There are not always clearly discernible differences between the occasional inferior work produced by a master and the very best work produced by a lesser talent.', 'Art dealers have always been led by economic self-interest to attribute any unsigned paintings of merit to recognized masters rather than to obscure artists.', 'When a painting is originally created, there are invariably at least some eyewitnesses who see the artist at work, and thus questions of correct attribution cannot arise at that time.', 'Even though some masters had specialists assist them with certain detail work, such as depicting lace, the resulting works are properly attributed to the masters alone.']", "label": 1 }, { "id": "train_3125", "context": "Philosopher: People are not intellectually well suited to live in large bureaucratic societies. Ttherefore, people can find happiness, if at all, only in smaller political units such as villages.", "question": "The reasoning in the philosopher's argument is flawed because the argument takes for granted that", "answers": "['no one can ever be happy living in a society in which she or he is not intellectually well suited to live', 'all societies that are plagued by excessive bureaucracy are large', 'the primary purpose of small political units such as villages is to make people happy', 'anyone who lives in a village or other small political unit that is not excessively bureaucratic can find happiness']", "label": 0 }, { "id": "train_3126", "context": "A table made entirely from the trunk of a tree said to have lived a thousand years was recently claimed to be that of a much younger tree. In order to rebut this charge, the craftsman summoned a team of dendrochronologists to prove that the tree lived to be at least to 1, 000 years old. Dendrochronology, or the technique of using tree rings to date wood, is based on the fact that for each passing year a tree develops exactly one ring, as seen in a horizontal cross-section of the trunk. Given that dendrochronology is accurate for trees that lived less than 2, 000 total years, the dendrochronologists will be able to determine whether the work comes from a tree that lived to be at least 1, 000 years old.", "question": "Which of the following is an assumption that the argument makes?", "answers": "['The tree was not less than 1, 000 years old when it was cut down.', 'The wood used in the table is large enough to contain a span of one thousand tree rings.', 'The artist has not used the trunk of the same tree in other works of art he has produced.', 'Dendrochronology has shown to be inaccurate for the oldest trees in the world, since parts of the trunks are so worn down that traces of tree rings are difficult to discern.']", "label": 1 }, { "id": "train_3127", "context": "Professor: If both parents have type O blood then their children can only have type O blood. This is a genetic law. Student: But that' s not true. My father has type B blood and I have type O blood.", "question": "The student has most likely misinterpreted the professor's remark to imply that", "answers": "['only people with type O blood can have children with type O blood', 'if both parents have type B blood, then their child will have type B blood', 'people with type O blood cannot have children with type B blood', 'what is true of one child in the family must also be true of all children in that family']", "label": 0 }, { "id": "train_3128", "context": "Donations of imported food will be distributed to children in famine-stricken countries in the form of free school meals. The process is efficient because the children are easy to reach at the schools and cooking facilities are often available on site.", "question": "Which of the following, if true, casts the most serious doubt on the efficiency of the proposed process?", "answers": "['The donation of food tends to strengthen the standing of the political party that happens to be in power when the donation is made.', 'A massive influx of donated food will tend to lower the price of food in the areas near the schools.', 'The reduction in farm surpluses in donor countries benefits the donor countries to a greater extent than the recipient countries are benefited by the donations.', 'Supplies of fuel needed for cooking at the schools arrive there only intermittently and in inadequate quantities.']", "label": 3 }, { "id": "train_3129", "context": "When students do not find their assignments challenging, they become bored and so achieve less than their abilities would allow. On the other hand, when students find their assignments too difficult, they give up and so again achieve less than what they are capable of achieving. It is, ttherefore, clear that no student' s full potential will ever be realized.", "question": "Which one of the following is an error of reasoning contained in the argument?", "answers": "['Assuming without warrant that a situation allows only two possibilities.', 'Mistakenly equating what is actual and what is merely possible.', 'Confusing the coincidence of two events with a causal relation between the two.', 'Depending on the ambiguous use of a key term.']", "label": 0 }, { "id": "train_3130", "context": "Some dairy farmers in the province of Takandia want to give their cows a synthetic hormone that increases milk production. Many Takandians, however, do not want to buy milk from cows given the synthetic hormone. For this reason Takandia's legislature is considering a measure requiring milk from cows given the hormone to be labeled as such. Even if the measure is defeated, dairy farmers who use the hormone will probably lose customers, since __.", "question": "Which of the following, if true, most logically completes the argument?", "answers": "['it has not been proven that any trace of the synthetic hormone exists in the milk of cows given the hormone', 'some farmers in Takandia who plan to use the synthetic hormone will probably not do so if the measure were passed', \"the legislature's consideration of the bill has been widely publicized\", 'milk from cows that have not been given the synthetic hormone can be labeled as such without any legislative action']", "label": 3 }, { "id": "train_3131", "context": "To improve productivity, manufacturing companies have recently begun restructuring work to produce more goods with fewer assembly-line workers, and the companies have laid off many workers as a consequence. The workers laid off have been those with the least seniority (time on the job), generally the younger workers.", "question": "The statements above, if true, most strongly support which of the following as a conclusion?", "answers": "['Some of the innovations in assembly-line processes and procedures that were made to increase productivity have instead proved to be counterproductive.', 'The products manufactured by the companies are not undergoing design changes while the manufacturing jobs are being restructured.', 'When assembly-line workers have made suggestions for improvements in manufacturing processes, some suggestions have been implemented, but many have not.', 'The manufacturing companies are increasing the average age of their assembly- line workforce while still seeking to increase production.']", "label": 0 }, { "id": "train_3132", "context": "Damien: Stores that have generous return policies have the most satisfied customers. Customer loyalty increases significantly in a company when the consumer is confident that he or she can return a product no matter what, especially if the product isn' t defective but the customer still isn' t satisfied. Erin: Even if a company has to incur frequent losses from returned products, it behoove s them to have a lenient return policy, because doing so positions them to take command of a greater share of the market because of their customer loyalty.", "question": "Which of the following best describes Erin's response to Damien's position?", "answers": [ "She provides an alternative explanation for the same situation described in Damien's stance.", "She points out potential undesirable consequences of Damien's stance.", "She further elaborates on Damien's stance with additional supportive evidence.", "She provides a sensible rebuttal but demonstrating inconsistencies in Damien's stance." ], "label": 2 }, { "id": "train_3133", "context": "Private industry is trying to attract skilled research scientists by offering them high salaries. As a result, most research scientists employed in private industry now earn 50 percent more than do comparably skilled research scientists employed by the government. So, unless government-employed research scientists are motivated more by a sense of public duty than by their own interests, the government is likely to lose its most skilled research scientists to private industry, since none of these scientists would have problems finding private-sector jobs.", "question": "Which one of the following is an assumption on which the argument depends?", "answers": "['The government does not employ as many research scientists who are highly skilled as does any large company in the private sector which employs research scientists.', 'Research scientists employed in the private sector generally work longer hours than do researchers employed by the government.', 'Government research scientists are less likely to receive acknowledgment for their research contributions than are research scientists in the private sector.', 'The government does not provide its research scientists with unusually good working conditions or fringe benefits that more than compensate for the lower salaries they receive.']", "label": 3 }, { "id": "train_3134", "context": "A certain gene can be stimulated by chemicals in cigarette smoke, causing lung cells to metabolize the chemicals in a way that makes the cells cancerous. Yet smokers in whom this gene is not stimulated have as high a risk of developing lung cancer from smoking as other smokers do.", "question": "If the statements above are true, it can be concluded on the basis of them that", "answers": "['the gene is more likely to be stimulated by chemicals in cigarette smoke than by other chemicals', 'stimulation of the gene by chemicals in cigarette smoke is not the only factor affecting the risk for smokers of developing lung cancer', 'nonsmokers have as high a risk of developing lung cancer as do smokers in whom the gene has not been stimulated', 'smokers are less likely to develop lung cancer if they do not have the gene']", "label": 1 }, { "id": "train_3135", "context": "Editorial: Painting involves a sequential application of layers, each of which adheres satisfactorily only if the underlying layer has been properly applied. Education is, in this respect, like the craft of painting. Since the most important steps in painting are preparation of the surface to be painted and application of the primer coat, it makes sense to suppose that__.", "question": "Which one of the following most logically completes the editorial's argument?", "answers": "['teaching new students is rewarding but much more difficult than teaching more advanced students', \"the success of a student's overall educational experience depends above all upon that student's initial educational experience\", 'students who have a secure grasp of the fundamentals of a subject are likely to make progress in that subject', 'educators who are not achieving the goals they intended should revise their teaching methods']", "label": 1 }, { "id": "train_3136", "context": "Sigatoka disease drastically reduces the yield of banana trees and is epidemic throughout the areas of the world where bananas are grown. The fungus that causes the disease can be controlled with fungicides, but the fungicides can pose a health hazard to people living nearby. The fungicides are thus unsuitable for small banana groves in populated areas. Fortunately, most large banana plantations are in locations so isolated that fungicides can be used safely there. Ttherefore, most of the world' s banana crop is not seriously threatened by Sigatoka disease.", "question": "Which one of the following is an assumption on which the argument depends?", "answers": "['Sigatoka disease is the only disease that threatens bananas on a worldwide scale.', 'Most of the banana trees that have not been exposed to the Sigatoka fungus grow in small banana groves.', \"Large plantations produce most or all of the world's bananas.\", 'Sigatoka disease spreads more slowly on large plantations than in small banana groves.']", "label": 2 }, { "id": "train_3137", "context": "Although exposure to asbestos is the primary cause of mesothelioma, a slow-developing cancer, researchers believe that infection by the SV40 virus is a contributing cause, since in the United States 60 percent of tissue samples from mesotheliomas, but none from healthy tissue, contain SV40. SV40 is a monkey virus; however, in 1960 some polio vaccine was contaminated with the virus. Researchers hypothesize that this vaccine was the source of the virus found in mesotheliomas decades later.", "question": "Which of the following, if true, most strongly supports the researchers' hypothesis?", "answers": "['In Finland, where the polio vaccine was never contaminated, samples from mesotheliomas do not contain SV40.', 'Changes in the technique of manufacturing the vaccine now prevent contamination with SV40.', 'Recently discovered samples of the vaccine dating from 1960 still show traces of the virus.', 'SV40 is widely used as a research tool in cancer laboratories.']", "label": 0 }, { "id": "train_3138", "context": "A certain automaker aims to increase its market share by deeply discounting its vehicles' prices for the next several months. The discounts will cut into profits, but because they will be heavily advertised the manufacturer hopes that they will attract buyers away from rival manufacturers' cars. In the longer term, the automaker envisions that customers initially attracted by the discounts may become loyal customers.", "question": "In assessing the plan's chances of achieving its aim, it would be most useful to know which of the following?", "answers": "[\"Whether some of the automaker's models will be more deeply discounted than others\", 'Whether an alternative strategy might enable the automaker to enhance its profitability while holding a constant or diminishing share of the market', \"Whether the automaker's competitors are likely to respond by offering deep discounts on their own products\", 'Whether the automaker will be able to cut costs sufficiently to maintain profit margins even when the discounts are in effect']", "label": 2 }, { "id": "train_3139", "context": "Ecologist: Landfills are generally designed to hold ten years' worth of waste. Some people maintain that as the number of active landfills consequently dwindles over the coming decade, there will inevitably be a crisis in landfill availability. However, their prediction obviously relies on the unlikely assumption that no new landfills will open as currently active ones close and is ttherefore unsound.", "question": "The claim that there will be a crisis in landfill availability plays which one of the following roles in the ecologist's argument?", "answers": "[\"It follows from the claim stated in the argument's first sentence.\", 'It is the main conclusion of the argument.', 'It is a claim on which the argument as a whole is designed to cast doubt.', \"It establishes the truth of the argument's conclusion.\"]", "label": 2 }, { "id": "train_3140", "context": "Davis: The only relevant factor in determining appropriate compensation for property damage or theft is the value the property loses due to damage or the value of the property stolen; the harm to the victim is directly proportional to the pertinent value. Higuchi: I disagree. More than one factor must be considered: A victim who recovers the use of personal property after two years is owed more than a victim who recovers its use after only one year.", "question": "Davis's and Higuchi's statements most strongly support the view that they would disagree with each other about which one of the following?", "answers": "['Victims who are deprived of their property are owed compensation in proportion to the harm they have suffered.', 'Some victims are owed increased compensation because of the greater amount of time they are deprived of the use of their property.', 'Some victims are owed increased compensation because of the greater dollar value of the damage done to their property.', 'It is possible to consistently and reliably determine the amount of compensation owed to someone whose property was damaged or stolen.']", "label": 1 }, { "id": "train_3141", "context": "Everything that is commonplace and ordinary fails to catch our attention, so there are things that fail to catch our attention but that are miracles of nature.", "question": "The conclusion of the argument follows logically if which one of the following is assumed?", "answers": "['Everything that fails to catch our attention is commonplace and ordinary.', 'Only extraordinary or unusual things catch our attention.', 'Some things that are commonplace and ordinary fail to catch our attention.', 'Some things that are ordinary and commonplace are miracles of nature.']", "label": 3 }, { "id": "train_3142", "context": "In the earliest video games, the player typically controlled the movements of a simple icon on the screen. But in newer video games, players often control the movements of detailed human figures -- a feature possible because of the greater technical sophistication of these games. It is hard for players to identify with these figures, since the players can see that the figures represent other people. Thus, in this respect the technical sophistication of the newer video games often makes them less compelling to players.", "question": "The conclusion of the argument can be properly drawn if which one of the following is assumed?", "answers": "['Most early video games in which the player controlled a simple icon on the screen were in other respects less compelling to players than newer video games.', 'When players cannot easily identify with the figure or icon whose movements they control in a video game, they usually find that game less compelling than it otherwise would be.', 'There are no newer, more technically sophisticated video games in which the player controls the movements of a simple icon on the screen.', 'The technical sophistication necessary for creating detailed human figures in video games cannot in itself make those video games fully compelling even to players who identify with those figures.']", "label": 1 }, { "id": "train_3143", "context": "Critic: Many popular psychological theories are poor theories in that they are inelegant and do not help to dispel the mystery that surrounds our psyche. However, this is not really important. The theories produce the right results: therapeutically, they tend to have greater success than their more scientific rivals.", "question": "The statement about the relative therapeutic success of many popular psychological theories plays which one of the following roles in the critic's argument?", "answers": "['It is used to suggest that popular psychological theories are actually better scientific explanations than are their rivals.', 'It is used to illustrate what the critic takes to be the most important aspect of scientific theories.', 'It is used to override some considerations against these theories.', 'It is used to disprove evidence against these theories.']", "label": 2 }, { "id": "train_3144", "context": "On the weekends, Eliza only reads romance novels. During the week, Eliza paints three days a week unless she has pressing family obligations or is substitute teaching at the local art school. Four days a week, Eliza practices her cello, and on Wednesdays, she gives cello lessons. Last week, Eliza had no pressing family obligations, nor did she substitute teach at the local art school. Ttherefore, she must have practiced her cello on Monday, Tuesday, Thursday, and Friday.", "question": "Which one of the following is an assumption on which the argument relies?", "answers": "['There was at least one day last week when Eliza read romance novels and painted.', 'Eliza does not practice her cello on the day that she gives cello lessons.', 'Eliza does not give cello lessons on days that she substitute teaches at the local art school.', 'Eliza never has pressing family obligations on the day she gives cello lessons.']", "label": 1 }, { "id": "train_3145", "context": "Musician: Some fans enjoy dancing in front of the stage, while others like to listen from a distance. The best music venues offer a mix of standing room and seats. If a suitable mix is impossible, then it' s always best for the venue to be standing room only. Our band will never play at a concert hall without at least some standing room.", "question": "Which of the following arguments most closely parallels the musician's argument?", "answers": "['Writer: Some writers prefer to type their stories, while others prefer handwriting them. The best writers always type since they can easily identify errors and edit more effectively. If a writer only writes by hand, then their work is always worse than those who type. I will never write my stories by hand.', \"Lawyer: Some clients enjoy written updates, while others prefer to be updated on the phone. The best lawyers adapt to their clients' needs. If a client's preference can't be discerned, then it's always best to first call and then follow up with written notice. I highly prefer talking on the phone.\", 'Businessman: Some suits have two buttons, while others have three. The best suits always have two buttons. I will never wear a three-button suit.', \"Coach: Some players respond to yelling, while others thrive in a more nurturing environment. The best coaches alternate between yelling and nurturing, depending on the player and situation. If the coach doesn't know how a player will respond, then it's always best to only be nurturing. I will never yell at my players without first offering a nurturing approach.\"]", "label": 3 }, { "id": "train_3146", "context": "Company X and Company Y have each been building a rocket ship able to travel to Mars. After five years of design and construction, the rocket ships have been unveiled to the public, which instantly notices the rocket ships are so similar in design as to be indistinguishable. Ttherefore, one of the companies must have had information pertaining to the design of the other rocket ship.", "question": "Which of the following, if true, would most call in to question the validity of the argument?", "answers": "['Both companies have been highly profitable and have been heavily funded throughout the five years in which the rocket ships have been constructed.', 'The engineers responsible for the design and construction of the rocket ship are subjected to a rigorous vetting process.', \"Each space ship uses a different color scheme, which matches that of each company's logo.\", 'There is only one possible design given the current materials that confers the aerodynamic requirements to reach Mars. .']", "label": 3 }, { "id": "train_3147", "context": "Politician: Governments should tax any harmful substance that is available to the general public at a level that the tax would discourage continued use of the substance.", "question": "Which one of the following is an application of the politician's principle of taxation?", "answers": "[\"The tax on products that emit fluorocarbons, substances that have proven to be harmful to the earth's ozone layer, is lowered to stimulate the development of new, less environmentally harmful ways of using these substances.\", 'The tax on certain pain relievers that, even though harmful, are available over the counter is raised, since studies have shown that the demand for these products will not be affected.', 'The tax on domestically produced alcoholic beverages is not raised, since recent studies show that the tax would have a negative impact on the tourist industry.', 'The tax on a pesticide that contains an organic compound harmful to human beings is raised to give people an incentive to purchase pesticides not containing the compound.']", "label": 3 }, { "id": "train_3148", "context": "Bevex, an artificial sweetener used only in soft drinks, is carcinogenic for mice, but only when it is consumed in very large quantities. To ingest an amount of Bevex equivalent to the amount fed to the mice in the relevant studies, a person would have, to drink 25 cans of Bevex-sweetened soft drinks per day. For that reason, Bevex is in fact safe for people.", "question": "In order for the conclusion that Bevex is safe for people to he properly drawn, which one of the following must be true?", "answers": "['Cancer from carcinogenic substances develops more slowly in mice than it does in people.', 'People drink fewer than 25 cans of Bevex- sweetened soda per day.', 'If all food additives that are currently used in foods were tested, some would be found to be carcinogenic for mice.', 'People can obtain important health benefits by controlling their weight through the use of artificially sweetened soft drinks.']", "label": 1 }, { "id": "train_3149", "context": "Amphibian populations are declining in numbers worldwide. Not coincidentally, the earth' s ozone layer has been continuously depleted throughout the last 50 years. Atmospheric ozone blocks UV-B, a type of ultraviolet radiation that is continuously produced by the sun, and which can damage genes. Because amphibians lack hair, hide, or feathers to shield them, they are particularly vulnerable to UV-B radiation. In addition, their gelatinous eggs lack the protection of leathery or hard shells. Thus, the primary cause of the declining amphibian population is the depletion of the ozone layer.", "question": "Each of the following, if true, would strengthen the argument EXCEPT:", "answers": "['Of the various types of radiation blocked by atmospheric ozone, UV-B is the only type that can damage genes.', 'Amphibian populations are declining far more rapidly than are the populations of nonamphibian species whose tissues and eggs have more natural protection from UV-B.', 'Amphibian populations have declined continuously for the last 50 years.', 'Atmospheric ozone has been significantly depleted above all the areas of the world in which amphibian populations are declining.']", "label": 0 }, { "id": "train_3150", "context": "In defending the Hyperion School of Journalism from charges that its program is of little or no value to its students, the dean of the school pointed to its recent success in placing students: 65 percent of its graduates went on to internships or jobs in print or broadcast journalism.", "question": "Which one of the following, if true, most seriously undermines the defense offered by the dean?", "answers": "['The program offered by the Hyperion School of Journalism is similar in quality and content to those offered by its peer institutions.', \"More than half of the school's students came from jobs in journalism to improve their skills.\", 'The number of cities with more than one major newspaper has declined sharply over the last 25 years.', 'Some newspaper editors do not regard journalism schools as a necessary part of the training of a journalist.']", "label": 1 }, { "id": "train_3151", "context": "Principle: A law whose purpose is to protect wild animal populations should not be enforced against those whose actions do not threaten wild animal populations. Application: Even though there is a law against capturing wild snakes, which was enacted to protect wild snake populations, snake charmers who violate this law should not be prosecuted.", "question": "Which one of the following, if true, most justifies the above application of the principle?", "answers": "['Snake populations are much less threatened than the populations of several other species for which capture is legal.', 'Snake charmers capture wild snakes only because they believe they would be unable to earn a living otherwise.', 'Many attempts to prosecute snake charmers under this law have failed because prosecutors lacked adequate knowledge of the procedures used to capture snakes.', 'Since there are relatively few snake charmers and they each capture relatively few snakes per year, snake charmers have a minimal effect on wild populations.']", "label": 3 }, { "id": "train_3152", "context": "A recent epidemiological study found that businesspeople who travel internationally on business are much more likely to suffer from chronic insomnia than are businesspeople who do not travel on business. International travelers experience the stresses of dramatic changes in climate, frequent disruption of daily routines, and immersion in cultures other than their own, stresses not commonly felt by those who do not travel. Thus, it is likely that these stresses cause the insomnia.", "question": "Which one of the following would, if true, most strengthen the reasoning above?", "answers": "['Businesspeople who already suffer from chronic insomnia are no more likely than businesspeople who do not to accept assignments from their employers that require international travel.', 'Experiencing dramatic changes in climate and disruption of daily routines through international travel can be beneficial to some people who suffer from chronic insomnia.', 'Most international travel for the sake of business occurs between countries with contiguous borders.', 'Some businesspeople who once traveled internationally but no longer do so complain of various sleep-related ailments.']", "label": 0 }, { "id": "train_3153", "context": "A study of ticket sales at a summer theater festival found that people who bought tickets to individual plays had a no-show rate of less than 1 percent, while those who paid in advance for all ten plays being performed that summer had a no-show rate of nearly 30 percent. This may be at least in part because the greater the awareness customers retain about the cost of an item, the more likely they are to use it.", "question": "Which of the following would, if true, best serve as an alternative explanation of the results of the study?", "answers": "['The price per ticket was slightly cheaper for those who bought all ten tickets in advance.', 'Those who attended all ten plays became eligible for a partial refund.', 'People who arrived just before the performance began could not be assured of obtaining seats in a preferred location.', 'Usually, people who bought tickets to individual plays did so immediately prior to each performance that they attended.']", "label": 3 }, { "id": "train_3154", "context": "Klein: The fact that the amount of matter that we have found in our galaxy is only one-tenth of what Einstein' s theory predicts gives us good reason for abandoning his view. Brown: Given the great successes of Einstein' s theory, it would be better to conclude that most of the matter in our galaxy has not yet been found.", "question": "On the basis of their statements, Klein and Brown are committed to disagreeing over the truth of which one of the following statements?", "answers": "[\"It is possible to determine the amount of matter in our galaxy without relying on Einstein's theory.\", \"The failure to find all of the matter predicted by Einstein's theory should lead us to abandon it.\", \"Einstein's theory has achieved many successes.\", 'Scientists are able to accurately judge the amount of matter that has been found in our galaxy.']", "label": 1 }, { "id": "train_3155", "context": "Among the various models of Delta vacuum cleaners, one cannot accurately predict how effectively a particular model cleans simply by determining how powerful its motor is. The efficiency of dust filtration systems varies significantly, even between models of Delta vacuum cleaners equipped with identically powerful motors.", "question": "The argument's conclusion is properly drawn if which one of the following is assumed?", "answers": "['All Delta vacuum cleaners that clean equally effectively have identically powerful motors.', \"One cannot accurately assess how effectively any Delta vacuum cleaner cleans without knowing how powerful that vacuum cleaner's motor is.\", 'For each Delta vacuum cleaner, the efficiency of its dust filtration system has a significant impact on how effectively it cleans.', 'For any two Delta vacuum cleaners with equally efficient dust filtration systems, the one with the more powerful motor cleans more effectively.']", "label": 2 }, { "id": "train_3156", "context": "A first-term board member should not be on the finance committee unless he or she is an accountant or his or her membership on the committee is supported by all the members of the board.", "question": "Which one of the following arguments most closely conforms to the principle stated above?", "answers": "['Simkins is a first-term board member and not an accountant; thus, Simkins should not be on the finance committee.', \"Ruiz is on the finance committee but is not an accountant; thus, Ruiz's membership must have been supported by all the members of the board.\", 'Timmons is a third-term board member but not an accountant; thus, if all other board members think that Timmons should be on the finance committee, then Timmons should be on that committee.', \"Klein is a first-term board member who is not an accountant; thus, Klein should not be allowed on the finance committee if any board member opposes Klein's appointment to that committee.\"]", "label": 3 }, { "id": "train_3157", "context": "According to a study, after a week of high-altitude living, twenty men had slimmed down. The men, middle-aged residents of low-altitude areas, had been taken to a research station at 2, 650 meters (8, 694 feet) above sea level. They had unrestricted access to food and were forbidden vigorous exercise, yet they lost an average of 1. 5 kilograms (3. 3 pounds) during their one-week stay. Clearly, the lower availability of oxygen at higher altitudes, or hypobaric hypoxia, can be said to have caused the weight loss, since __.", "question": "Which of the following would, if true, most logically complete the argument?", "answers": "['exercise at higher altitudes is more difficult than exercise at lower altitudes is', 'a decrease in oxygen intake has been shown to depress appetite', 'the men all participated in the same kinds of exercise during their stay', 'several weeks after returning home, the men still weighed less than they had before the study']", "label": 1 }, { "id": "train_3158", "context": "If the needle on an industrial sewing machine becomes badly worn, the article being sewn can be ruined. In traditional apparel factories, the people who operate the sewing machines monitor the needles and replace those that begin to wear out. Industrial sewing operations are becoming increasingly automated, however, and it would be inefficient for a factory to hire people for the sole purpose of monitoring needles. Ttherefore a sophisticated new acoustic device that detects wear in sewing machine needles is expected to become standard equipment in the automated apparel factories of the future.", "question": "Which one of the following is most strongly supported by the information above?", "answers": "['The needles of industrial sewing machines wear out at unpredictable rates.', 'Traditional apparel factories do not use any automated equipment.', 'In the automated apparel factories of the future, each employee will perform only one type of task.', 'In automated apparel factories, items will be ruined by faulty needles less frequently than happens in traditional apparel factories.']", "label": 0 }, { "id": "train_3159", "context": "Climate change is caused by an increase of carbon dioxide in the Earth' s atmosphere. Carbon dioxide traps heat and remains in the atmosphere longer than other heat-trapping gases. Between 2000 and 2012, 890, 000 square miles of trees around the world were cut down -- more than one-eighth of the original forest covering Earth. Overall, deforestation has led to the loss of half the world' s trees. Unless the rapid pace of deforestation is halted, the Earth' s climate will change drastically in the near future.", "question": "The argument relies on which one of the following assumptions being true?", "answers": "['Deforestation is a manmade phenomenon.', 'Trees produce carbon dioxide, releasing the gas into the atmosphere.', 'Climate change negatively impacts the Earth.', 'Trees lower the amount of carbon dioxide in the atmosphere.']", "label": 3 }, { "id": "train_3160", "context": "Newspaper editorial: Many pharmaceutical companies develop \"me too\" drugs, drugs designed to duplicate, more or less, the effect of another company' s product that is already on the market. Some critics object that such development constitutes an unnecessary, redundant use of resources that provides no new benefits to consumers. However, the entry of \"me too\" drugs into the market can result in a price reduction for the drugs they resemble. Ttherefore, \"me too\" drugs can indeed benefit consumers.", "question": "Which one of the following, if true, most strengthens the editorial's argument?", "answers": "['Pharmaceutical companies often make more money on a \"me too\" drug than on an original drug.', 'Some pharmaceutical companies lose money on the development of innovative drugs because of the development by other companies of \"me too\" drugs.', 'Some \"me too\" drugs turn out to be more effective than the drugs they were designed to imitate.', 'If all pharmaceutical companies developed \"me too\" drugs, fewer innovative drugs would be developed.']", "label": 2 }, { "id": "train_3161", "context": "The United States deploys two types of submarines -- attack submarines and ballistic submarines. Attack submarines carry cruise missiles to attack specific locations on land, and they are also used to spy on foreign countries. Ballistic submarines carry intercontinental ballistic missiles that deliver nuclear missiles at a minimum range of 3, 400 miles. Consequently, there is little advantage to placing a ballistic submarine near the coast of any country. Both submarines are nuclear-powered, but only the ballistic submarines carry nuclear weapons. The United States believes that a foreign country is plotting to attack her homeland, but more intelligence must be collected. The United States plans to deploy a submarine off the coast of the foreign country.", "question": "Which one of the following would be a reasonable conclusion based on the passage?", "answers": "['The United States should develop a new type of submarine to gather the evidence.', 'The United States should pre-emptively attack the foreign country to best defend itself.', 'The United States should deploy an attack submarine off the coast of the foreign country.', \"The United States should plant an agent in the foreign country's intelligence service.\"]", "label": 2 }, { "id": "train_3162", "context": "There should be a greater use of gasohol. Gasohol is a mixture of alcohol and gasoline, and has a higher octane rating and fewer carbon monoxide emissions than straight gasoline. Burning gasohol adds no more carbon dioxide to the atmosphere than plants remove by photosynthesis.", "question": "Each of the following, if true, strengthens the argument above EXCEPT:", "answers": "['Cars burn on the average slightly more gasohol per kilometer than they do gasoline.', 'Cars run less well on gasoline than they do on gasohol.', 'Since less gasoline is needed with the use of gasohol, an energy shortage is less likely.', 'Gasohol is cheaper to produce and hence costs less at the pump than gasoline.']", "label": 0 }, { "id": "train_3163", "context": "In the first decade following the founding of the British Labour party, the number of people regularly voting for Labour increased fivefold. The number of committed Labour voters increased a further fivefold during the party' s second decade. Since the increase was thus the same in the first as in the second decade, the often-made claim that the Labour party gained more voters in the party' s second decade than in its first is clearly false.", "question": "The reasoning in the argument is flawed because the argument", "answers": "['relies on statistical evidence that, strictly speaking, is irrelevant to establishing the conclusion drawn', 'fails to allow for the possibility that the policy positions advocated by the Labour party changed during the period in question', 'fails to specify dates necessary to evaluate the truth of the conclusion, even though the argument depends on distinguishing between two time periods', 'draws a conclusion that cannot be true if all the data advanced in its support are true']", "label": 3 }, { "id": "train_3164", "context": "A university professor researching sleep disorders occasionally taught class after spending whole nights working in a laboratory. She found lecturing after such nights difficult: she reported that she felt worn out and humorless, and she had difficulty concentrating and finding the appropriate words. After several weeks of lectures, she asked her students to guess which lectures had been given after nights without sleep. Interestingly, very few students were able to correctly identify them.", "question": "Which one of the following statements is most strongly supported by the information above?", "answers": "['No one can assess the overall effects of sleep deprivation on a particular person as well as that sleep-deprived person can.', 'University students in a lecture audience tend to be astute observers of human behavior.', 'Occasional sleep deprivation is not as debilitating as extended sleep deprivation.', 'The subjective effects of occasional sleep deprivation are more pronounced than are its effects on overt behavior.']", "label": 3 }, { "id": "train_3165", "context": "The blue-tipped puffer is a migratory bird that is found along the coast of this state in summer months. The puffers enjoy the seeds of Thomson' s Mulberry trees; because these trees are plentiful in the state' s Coastal Park, traditionally many blue-tipped puffers would be found in the park during summer months. Ten years ago, the grey fox, a primary predator of the puffer, was re-introduced into the park, and its numbers have been thriving. Over that time, we have seen many fewer puffers in the park during the summer months. Clearly, the puffers are choosing to spend their summers elsewhere.", "question": "Which of the following would be the most important to determine in order to evaluate the argument?", "answers": "['Whether the range of the grey fox within the park extends all the way to the coast.', 'Whether other coastal areas in the state have seen an increase in the number of blue-tipped puffers over the last decade.', 'How the speed of a blue-tipped puffer during take-off from the ground compares to the speed of a grey fox running', 'What other birds and small mammals the grey fox typically hunts besides the blue-tipped puffers.']", "label": 1 }, { "id": "train_3166", "context": "Each year, an official estimate of the stock of cod in the Grand Banks is announced. This estimate is obtained by averaging two separate estimates of how many cod are available, one based on the number of cod caught by research vessels during a once-yearly sampling of the area and the other on the average number of tons of cod caught by various commercial vessels per unit of fishing effort expended there in the past year -- a unit of fishing effort being one kilometer of net set out in the water for one hour. In previous decades, the two estimates usually agreed closely. However, for the last decade the estimate based on commercial tonnage has been increasing markedly, by about the same amount as the samplingbased estimate has been decreasing.", "question": "If the statements in the passage are true, which one of the following is most strongly supported by them?", "answers": "['The once-yearly sampling by research vessels should be used as the sole basis for arriving at the official estimate of the stock of cod.', 'The number of commercial vessels fishing for cod in the Grand Banks has increased substantially over the past decade.', \"Last year's official estimate was probably not much different from the official estimate ten years ago.\", 'Twenty years ago, the overall stock of cod in the Grand Banks was officially estimated to be much larger than it is estimated to be today.']", "label": 2 }, { "id": "train_3167", "context": "Bureaucratic mechanisms are engineered to resist change. Thus, despite growing dissatisfaction with complex bureaucratic systems, it is unlikely that bureaucracies will be simplified.", "question": "The claim that bureaucratic mechanisms are engineered to resist change plays which one of the following roles in the argument?", "answers": "['It is cited as evidence that bureaucratic systems are becoming more and more complex.', 'It is used to weaken the claim that bureaucracies should be simplified.', 'It is a premise offered in support of the claim that it is unlikely that bureaucracies will be simplified.', 'It is a conclusion for which the claim that bureaucracies are unlikely to be simplified is offered as support.']", "label": 2 }, { "id": "train_3168", "context": "Goodbody, Inc. , is in the process of finding tenants for its newly completed Parrot Quay commercial development, which will make available hundreds of thousands of square feet of new office space on what was formerly derelict property outside the financial center of the city. Surprisingly enough, the coming recession, though it will hurt most of the city' s businesses, should help Goodbody to find tenants.", "question": "Which one of the following, if true, does most to help resolve the apparent paradox?", "answers": "['Public transportation links the financial center with the area around Parrot Quay.', 'Businesses forced to economize by the recession will want to take advantage of the lower rents available outside the financial center.', \"Many of Goodbody's other properties are in the financial center and will become vacant if the recession is severe enough to force Goodbody's tenants out of business.\", 'The area in which the Parrot Quay development is located became derelict after the heavy industry that used to be there closed down in a previous recession.']", "label": 1 }, { "id": "train_3169", "context": "Patterson: Bone flutes dating to the Upper Paleolithic are the earliest evidence for music. Thus it is likely that music first arose during this period. Garza: But the Upper Paleolithic is exceptional for the intensive use of bone, which typically survives well in archaeological contexts, unlike other materials commonly used for musical instruments, such as wood.", "question": "Garza responds to Patterson by doing which one of the following?", "answers": [ "arguing that the body of evidence to which Patterson appeals is insufficient for Patterson's purposes", "using Patterson's evidence to draw a conclusion inconsistent with the conclusion drawn in Patterson's argument", "presenting a counterexample to the general conclusion drawn in Patterson's argument", "presenting an argument analogous to Patterson's argument to reveal a potential flaw in Patterson's reasoning" ], "label": 0 }, { "id": "train_3170", "context": "Researcher: The role of chemicals called pheromones in determining the sexual behavior of some animals is well documented. But, even though humans also produce these chemicals, it is clear that psychological factors have taken over this role in us. Whereas for animals these behaviors are involuntary, which is a clear sign of chemical control, humans, by virtue of their free will, choose how they behave, and thus psychological factors take over. So pheromones are merely a vestige of our evolutionary past.", "question": "The researcher's argument requires the assumption that", "answers": "['free will can be found only in humans', 'whatever does not have a chemical explanation must have a purely psychological one', 'voluntary action cannot have a chemical explanation', 'voluntary action cannot have an evolutionary explanation']", "label": 2 }, { "id": "train_3171", "context": "Mayor: There has been a long debate in city council about how to accommodate projected increases in automobile traffic. Today, our choice is clear: either we adopt my plan to build a new expressway, or we do nothing. Doing nothing is not a viable option because our existing system of roads would be in gridlock within ten years given even a conservative estimate of future traffic levels. City council should ttherefore adopt my plan.", "question": "The reasoning in the mayor's argument is most vulnerable to which one of the following criticisms?", "answers": "['It bases a projection only on conservative estimates rather than considering a wider range of estimates.', 'It takes for granted that the options it considers are mutually exclusive.', 'It presents a choice that is limited to two options, without giving reasons for not considering any other options.', \"It fails to address the issue of the cost of traffic gridlock to the city's economy.\"]", "label": 2 }, { "id": "train_3172", "context": "It is clear that none of the volleyball players at yesterday' s office beach party came to work today since everyone who played volleyball at that party got badly sunburned and no one at work today is even slightly sunburned.", "question": "Which one of the following exhibits a pattern of reasoning that most closely parallels that in the argument above?", "answers": "['Since everyone employed by TRF who was given the opportunity to purchase dental insurance did so and everyone who purchased dental insurance saw a dentist, it is clear that no one who failed to see a dentist is employed by TRF.', 'Since no one who was promoted during the past year failed to attend the awards banquet, evidently none of the office managers attended the banquet this year since they were all denied promotion.', \"Since the Donnely report was not finished on time, no one in John's group could have been assigned to contribute to that report since everyone in John's group has a reputation for getting assignments in on time.\", 'Everyone with an office on the second floor works directly for the president and, as a result, no one with a second floor office will take a July vacation because no one who works for the president will be able to take time off during July.']", "label": 3 }, { "id": "train_3173", "context": "Healthy lungs produce a natural antibiotic that protects them from infection by routinely killing harmful bacteria on airway surfaces. People with cystic fibroses, however, are unable to fight off such bacteria, even though their lungs produce normal amounts of the antibiotic. Since the fluid on airway surfaces in the lungs of people with cystic fibrosis has an abnormally high salt concentration, scientists hypothesize that in high salt environments the antibiotic becomes ineffective at killing harmful bacteria.", "question": "Which of the following, if it were obtained as an experimental result, would most decisively undermine the scientists' hypothesis?", "answers": "['The salt concentration of the airway-surface fluid in the lungs of people who suffer from cystic fibrosis tends to return to its former high levels after having been reduced to levels typical of healthy lungs.', 'Healthy lungs in which the salt concentration of the airway-surface fluid has been substantially increased are able to reestablish their normal salt concentration within a relatively short period of time.', 'The lungs of people who suffer from cystic fibrosis are unable to fight off harmful bacteria even when the salt concentration is reduced to levels typical of healthy lungs.', 'The antibiotic produced by the lungs is effective at killing harmful bacteria even when salt concentrations are below levels typical of healthy lungs.']", "label": 2 }, { "id": "train_3174", "context": "Vincent: No scientific discipline can study something that cannot be measured, and since happiness is an entirely subjective experience, it cannot be measured. Yolanda: Just as optometry relies on patients' reports of what they see, happiness research relies on subjects' reports of how they feel. Surely optometry is a scientific discipline.", "question": "Vincent's and Yolanda's statements provide the most support for concluding that they disagree over which one of the following?", "answers": "['A scientific discipline can rely on subjective reports.', 'Happiness is an entirely subjective experience.', 'Happiness research is as much a scientific discipline as optometry is.', 'Optometry is a scientific discipline.']", "label": 0 }, { "id": "train_3175", "context": "After the recent court rulings, commercial shark fishing and shark hunting will resume in Diamond Bay . Many citizens of the communities around the bay have hailed this move, believing that a decrease in sharks will lead to an increase in all smaller fish, including the endangered Green-Gilled Silverfish, whose only ecosystem is within Diamond Bay. Ecologists, though, disagree, pointing out that a decrease in sharks will lead to a surge in Sea Lions, which are the principal predator of the Green-Gilled Silverfish. ", "question": "In the argument given, the two portions in boldface play which of the following roles?", "answers": "['The first supports an action as a beneficial; the second calls that evaluation into question.', 'The first describes evidence that has been taken as supporting a conclusion; the second states a contrary conclusion that is the main conclusion of the argument.', 'The first is an opinion that the argument opposes; the second is evidence inconsistent with the conclusion of the argument.', 'The first is a conclusion that the argument as a whole seeks to justify; the second is an explanation that supports this conclusion.']", "label": 1 }, { "id": "train_3176", "context": "The temperature in Taychester is always at least 10 degrees lower than the temperature in Chariesville. However, the average resident of Charlesville spends 10 to 20 percent more on winter heating expenses than does the average resident of Taychester.", "question": "Each of the following, if true, helps to resolve the apparent paradox described above EXCEPT:", "answers": "['Heat loss due to wind is less in Taychester than in Charlesville.', 'Houses in colder climates are usually better insulated than houses in warmer climates.', 'Although Chariesville is always fairly warm during the daytime, temperatures in Charlesville drop steeply at night.', 'Utility rates in Taychester are lower than utility rates in Charlesville.']", "label": 2 }, { "id": "train_3177", "context": "Congenial guests and a plentiful supply of good things to eat and drink will ensure a successful dinner party. Since Sylvia has prepared more than enough to eat and drink and her guests are all congenial people, her dinner party is certain to be a success.", "question": "The pattern of flawed reasoning exhibited by the argument above is most similar to that exhibited by which one of the following?", "answers": "['Fresh greens, carefully washed and served with a light dressing, always produce a refreshing salad. Since Tisha has developed an exceptionally light dressing but never washes her fresh greens, no salad she serves will be a refreshing one.', 'If soup is made with a well-seasoned meat stock and fresh ingredients, it will always be welcome at dinner. Since to his meat stock Arnold added only very fresh ingredients, the resulting soup is certain to be welcome at dinner.', 'Making pie dough using ice water and thoroughly chilling the dough before rolling it out will ensure a flaky crust. Andrew thoroughly chilled his pie dough before rolling it out, so, since he used ice water in making it, his pie is certain to have a flaky crust.', 'If corn is baked with its husks on, the resulting dish will always be moist and sweet. Since George wishes to ensure that the corn he plans to serve is moist, he will be certain both to bake it and to leave its husks on.']", "label": 1 }, { "id": "train_3178", "context": "Most people who have taken a seminar for building organizational skills in the workplace have indeed become more organized as a result; however, despite having become more organized, few have become any more efficient.", "question": "Which one of the following, if true, would most help to resolve the apparent discrepancy described above?", "answers": "['Some of the people who are most efficient in the workplace are not among the most organized.', 'Most seminars for building organizational skills in the workplace are designed for people who have been selected for management training.', 'Most people who have taken a seminar for building organizational skills in the workplace have below-average organizational skills before they enroll in the seminar.', 'Most people who have taken a seminar for building organizational skills in the workplace consequently expend a great amount of time organizing their activities.']", "label": 3 }, { "id": "train_3179", "context": "When politicians resort to personal attacks, many editorialists criticize these attacks but most voters pay them scant attention. Everyone knows such attacks will end after election day, and politicians can be excused for mudslinging. Political commentators, however, cannot be. Political commentators should be engaged in sustained and serious debate about ideas and policies. In such a context, personal attacks on opponents serve not to beat those opponents but to cut off the debate.", "question": "Which one of the following most accurately states the main point of the argument?", "answers": "['Political commentators should not resort to personal attacks on their opponents.', 'Editorialists are right to criticize politicians who resort to personal attacks on their opponents.', 'The purpose of serious debate about ideas and policies is to counteract the effect of personal attacks by politicians.', 'Personal attacks on opponents serve a useful purpose for politicians.']", "label": 0 }, { "id": "train_3180", "context": "Some species of tarantula make good pets. However, no creature with poison fangs makes a good pet. Ttherefore, not all tarantula species have poison fangs.", "question": "Which one of the following arguments is most similar in its pattern of reasoning to the argument above?", "answers": "[\"Some of the poems in this collection were written by Strawn. However, none of the poems in this collection has a regular meter. Ttherefore, only Strawn's unpublished poetry has a regular meter.\", 'Some of the poetry written by Strawn has a regular meter. However, this collection contains no poetry written by Strawn. Ttherefore, not all of the poetry in this collection has a regular meter.', 'None of the poetry written by Strawn has a regular meter. But some of the poems in this collection have a regular meter. Ttherefore, at least some of the poems in this collection were not written by Strawn.', 'No poems with a regular meter are found in this collection and none of the poetry in this collection was written by Strawn. Ttherefore, not all of the poetry written by Strawn has a regular meter.']", "label": 2 }, { "id": "train_3181", "context": "The folk medicine of many different cultures has extolled the use of garlic both to prevent and to cure many diseases. Modern medical research is absorbing some of these beliefs. Recent studies indicate that many of the more than 50 sulphur compounds that can be produced from garlic (depending upon whether it is boiled, fried in oil, or dried) have medicinal utility. Some of these compounds lower blood cholesterol, others reduce platelet clumping, while still others shrink some kinds of tumors. Other plants and herbs recommended by traditional medicine may also prove valuable for medicine generally.", "question": "Each of the following statements is supported by the information above EXCEPT:", "answers": "['Research on some traditional medications has yet to be conducted by modern medical researchers.', 'It can be medically beneficial to reduce platelet clumping or to lower blood cholesterol.', 'The belief that garlic is an effective way to prevent or cure some diseases is not limited to a small number of closely related cultures.', 'Garlic that is neither boiled, fried in oil, nor dried has no medicinal value.']", "label": 3 }, { "id": "train_3182", "context": "Astorga' s campaign promises are apparently just an attempt to please voters. What she says she will do if elected mayor is simply what she has learned from opinion polls that voters want the new mayor to do. Ttherefore, voters are not being told what Astorga actually intends to do if she becomes mayor.", "question": "Which one of the following is a questionable assumption on which the argument relies?", "answers": "['Astorga does not actually intend, if elected, to do what she has learned from the public opinion polls that voters want the new mayor to do.', 'If she is elected mayor, Astorga will not be capable of carrying out the campaign promises she has made.', \"The opinion polls on which Astorga's promises are based do not accurately reflect what voters want the new mayor to do.\", \"Most voters are unlikely to be persuaded by Astorga's campaign promises to vote for her in the mayoral election.\"]", "label": 0 }, { "id": "train_3183", "context": "McBride: The proposed new fuel-efficiency standards, if implemented, will discourage the manufacture of full-size cars. This prospect is troubling because when a subcompact and a full-size car collide, the people in the subcompact are more likely to be seriously injured than if theirs had also been a full-size car. The new fuel-efficiency standards should ttherefore be opposed. Leggett: But whenever any two cars collide, it is more likely that someone will be seriously injured if one of the cars is a full-size car than if neither car is full-size. So the new fuel-efficiency standards should be supported precisely because they discourage the manufacture of full-size cars.", "question": "McBride's and Leggett's statements commit them to disagreeing about the truth of which one of the following?", "answers": "['Fuel conservation is less important than safety in case of a collision.', 'The manufacture of full-size cars should be discouraged.', 'The new fuel-efficiency standards will encourage automobile manufacturers to build more subcompact cars.', 'Reducing the number of full-size cars on the highway will reduce the frequency of collisions between automobiles.']", "label": 1 }, { "id": "train_3184", "context": "The higher a dam, the more exposed it is to forces that can cause it to collapse. Of the world' s hundreds of arch dams, more than half are over 100 meters high. Yet all six of the arch dam collapses that have occurred during the past 40 years have occurred in arch dams under 100 meters high.", "question": "Which one of the following, if true, most helps to resolve the apparent paradox?", "answers": "['The larger the structure, the more careful the attention it received in design and construction.', 'The higher a dam must be, the greater the likelihood that it will be built as an arch dam.', 'Arch dams are generally more complicated and more costly to construct than other types of dams of comparable size.', 'Arch dams under 100 meters high are less subject to collapse than are dams of other types also under 100 meters high.']", "label": 0 }, { "id": "train_3185", "context": "Wong: Although all countries are better off as democracies, a transitional autocratic stage is sometimes required before a country can become democratic. Tate: The freedom and autonomy that democracy provides are of genuine value, but the simple material needs of people are more important. Some countries can better meet these needs as autocracies than as democracies.", "question": "Wong's and Tate's statements provide the most support for the claim that they disagree over the truth of which one of the following?", "answers": "['The freedom and autonomy that democracy provides are of genuine value.', 'All democracies succeed in meeting the simple material needs of people.', 'Nothing is more important to a country than the freedom and autonomy of the individuals who live in that country.', 'There are some countries that are better off as autocracies than as democracies.']", "label": 3 }, { "id": "train_3186", "context": "Construction contractors working on the cutting edge of technology nearly always work on a \"cost-plus\" basis only. One kind of cost-plus contract stipulates the contractor' s profit as a fixed percentage of the contractor' s costs; the other kind stipulates a fixed amount of profit over and above costs. Under the first kind of contract, higher costs yield higher profits for the contractor, so this is where one might expect final costs in excess of original cost estimates to be more common. Paradoxically, such cost overruns are actually more common if the contract is of the fixed-profit kind.", "question": "Which one of the following, if true, most helps to resolve the apparent paradox in the situation described above?", "answers": "['On long-term contracts, cost projections take future inflation into account, but since the figures used are provided by the government, they are usually underestimates.', \"Clients billed under a cost-plus contract are free to review individual billings in order to uncover wasteful expenditures, but they do so only when the contractor's profit varies with cost.\", 'On any sizable construction project, the contractor bills the client monthly or quarterly, so any tendency for original cost estimates to be exceeded can be detected early.', 'The practice of submitting deliberately exaggerated cost estimates is most common in the case of fixed-profit contracts, because it makes the profit, as a percentage of estimated cost, appear modest.']", "label": 1 }, { "id": "train_3187", "context": "Obviously, entrepreneurial ability is needed to start a successful company. Yet many entrepreneurs who succeed in starting a company fail later for lack of managerial skills. For instance, they do not adequately analyze market trends and, consequently, they fail in managing company growth. Hence, the lack of managerial skills and the lack of entrepreneurial ability can each inhibit the development of successful companies.", "question": "The proposition that certain entrepreneurs fail in managing company growth plays which one of the following roles in the argument above?", "answers": "[\"It is an example that is offered in support of a premise that is intended to support the argument's main conclusion directly.\", 'It is presented as an example of the phenomenon the argument seeks to explain.', 'It is the main conclusion drawn in the argument.', \"It is a premise that is intended to support the argument's main conclusion directly.\"]", "label": 0 }, { "id": "train_3188", "context": "Mature white pines intercept almost all the sunlight that shines on them. They leave a deep litter that dries readily, and they grow to prodigious height so that, even when there are large gaps in a stand of such trees, little light reaches the forest floor. For this reason white pines cannot regenerate in their own shade. Thus, when in a dense forest a stand of trees consists of nothing but mature white pines, it is a fair bet that __.", "question": "Which one of the following most logically concludes the argument?", "answers": "['other species of trees will soon begin to colonize the stand, eventually replacing all of the white pines', \"competition among the trees in the stand for sunlight will soon result in some trees' dying and the stand thus becoming thinner\", 'the ages of the trees in the stand do not differ from each other by much more than the length of time it takes a white pine to grow to maturity', 'the land on which the stand is now growing had been cleared of all trees at the time when the first of the white pines started growing']", "label": 2 }, { "id": "train_3189", "context": "Economist: The price of tap water in our region should be raised drastically. Supplies in local freshwater reservoirs have been declining for years because water is being used faster than it can be replenished. Since the price of tap water has been low, few users have bothered to adopt even easy conservation measures. ", "question": "The two sections in boldface play which of the following roles in the economist's argument?", "answers": "['The first is an observation for which the second provides an explanation; the second is the main conclusion but not the only conclusion.', 'The first is the only conclusion; the second provides an explanation for the first.', 'The first is a conclusion for which support is provided, and which in turn supports the main conclusion; the second is the main conclusion.', \"The first is a premise supporting the argument's main conclusion; so is the second.\"]", "label": 3 }, { "id": "train_3190", "context": "Lucy, the skeleton of a female hominin species, was discovered in 1974 in Africa. Scientists think the skeleton is about 3. 2 million years old. For six years, Lucy' s remains were part of an exhibition called Lucy' s Legacy: The Hidden Treasures of Ethiopia. However, there was much opposition to the exhibition for fear that Lucy' s fossils would be damaged. It' s a good thing that they started using casts instead; the preservation of these fossils should be the number one concern of excavators and scientists.", "question": "Which of the following would be used in opposition to the passage?", "answers": "[\"Lucy's fossils, if they are kept in exhibition for too long, are certainly at risk for damage. Even if you can't see it with the naked eye, the constant handling of fossils will cause wear and tear that time could never recover.\", 'Some scientists think that Lucy died from falling out of a tree, although they are not sure. There is also a tooth mark on top of a left pubic bone. What\\'s even more curious is Lucy\\'s age. Although she was considered \"fully mature, \" Lucy only lived for about twelve years.', 'The exhibition of Lucy is an important step in raising an awareness of where humans originated and the studies thereof. Keeping the fossils isolated from the public will certainly preserve the remains; however, we might risk the deterrence of public knowledge.', \"The controversy over the exhibition of Lucy's bones is irrelevant. Even if the bones are damaged, we have preserved the memory with scans, photographs, as well as accurate castings of the fossils. Scientists are making a huge deal over something that doesn't matter in the long run.\"]", "label": 2 }, { "id": "train_3191", "context": "Forest fires destroy thousand of acres of land in this country every year. Careless campers who do not properly extinguish their campfires cause many of the forest fires. Unless something is done to cut down on the number of campers in this nation' s forests, we will face a day when there will be no forests remaining in this country.", "question": "The argument depends on the assumption that", "answers": "['that forest will not regenerate after forest fires', \"forests are vital to this nation's environmental survival\", 'that national parks are not beneficial to this nation', 'there are no other causes of forest fires']", "label": 0 }, { "id": "train_3192", "context": "A new silencing device for domestic appliances operates by producing sound waves that cancel out the sound waves produced by the appliance. The device, unlike conventional silencers, actively eliminates the noise the appliance makes, and for that reason vacuum cleaners designed to incorporate the new device will operate with much lower electricity consumption than conventional vacuum cleaners.", "question": "Which one of the following, if true, most helps to explain why the new silencing device will make lower electricity consumption possible?", "answers": "['Governmental standards for such domestic appliances as vacuum cleaners allow higher electricity consumption when vacuum cleaners are quieter.', 'The need to incorporate silencers in conventional vacuum cleaners makes them heavier and less mobile than they might otherwise be.', \"Designers of vacuum cleaner motors typically have to compromise the motors' efficiency in order to reduce noise production.\", 'Conventional vacuum cleaners often use spinning brushes to loosen dirt in addition to using suction to remove dirt.']", "label": 2 }, { "id": "train_3193", "context": "Marta: There have been complaints about the lack of recreational areas in our city. Some people favor turning the old railway land into walking trails, but there may be more productive ways of using that land. Arthur: But the old railway land is ideal for walking trails. Our citizens have gone too long with inadequate recreational areas; we should not dismiss this proposal for walking trails without further consideration.", "question": "Arthur's criticism suggests that he interpreted Marta to be", "answers": "['asserting that it may not be possible to convert the old railway land into adequate walking trails', 'assuming that the complaints concerning a shortage of recreational areas are ill founded', 'recommending that the proposal for converting the old railway land into walking trails should be promptly dismissed', 'maintaining that converting the old railway land into walking trails would be an entirely unproductive use of that land']", "label": 2 }, { "id": "train_3194", "context": "As a result of modern medicine, more people have been able to enjoy long and pain-free lives. But the resulting increase in life expectancy has contributed to a steady increase in the proportion of the population that is of advanced age. This population shift is creating potentially devastating financial problems for some social welfare programs.", "question": "Which one of the following propositions is most precisely exemplified by the situation presented above?", "answers": "['All social institutions are affected by a preoccupation with prolonging life.', 'Implementing technological innovations should be delayed until the resulting social changes can be managed.', 'Solving one set of problems can create a different set of problems.', 'Technical or scientific innovation cannot be the solution to all problems.']", "label": 2 }, { "id": "train_3195", "context": "Extinction is the way of nature. Scientists estimate that over half of the species that have ever come into existence on this planet were already extinct before humans developed even the most primitive of tools. This constant natural process of species emergence and extinction, however, is ignored by those who wish to trace the blame for more recent extinctions to humanity' s use of technology, with its consequent effects on the environment. These people must be made to understand that the species that have become extinct in modern times would have become extinct by now even if humans had never acquired technology.", "question": "Which one of the following identifies a reasoning error in the passage?", "answers": "['The author provides no specific evidence that the species that have become extinct in modern times are the same species that would have become extinct in the absence of human technology.', 'The author ignores the fact that some species that are not yet extinct are in danger of extinction.', 'The author fails to consider that there are probably species in existence that have not yet been identified and studied by scientists.', 'The author mistakenly assumes that technology has not caused any harm to the environment.']", "label": 0 }, { "id": "train_3196", "context": "In 1960' s studies of rats, scientists found that crowding increases the number of attacks among the animals significantly. But in recent experiments in which rhesus monkeys were placed in crowded conditions, although there was an increase in instances of \"coping\" behavior-such as submissive gestures and avoidance of dominant individuals-attacks did not become any more frequent. Ttherefore it is not likely that, for any species of monkey, crowding increases aggression as significantly as was seen in rats.", "question": "Which of the following, if true, most strengthens the argument?", "answers": "['Rhesus monkeys respond with aggression to a wider range of stimuli than any other monkeys do.', 'Some of the coping behavior displayed by rhesus monkeys is similar to behavior rhesus monkeys use to bring to an end an attack that has begun.', 'All the observed forms of coping behavior can be found among rhesus monkeys living in uncrowded conditions.', 'In the studies of rats, nondominant individuals were found to increasingly avoid dominant individuals when the animals were in crowded conditions.']", "label": 0 }, { "id": "train_3197", "context": "Humanitarian considerations aside, sheer economics dictates that country X should institute, as country Y has done, a nationwide system of air and ground transportation for conveying seriously injured persons to specialized trauma centers. Timely access to the kind of medical care that only specialized centers can provide could save the lives of many people. The earnings of these people would result in a substantial increase in country X' s gross national product, and the taxes paid on those earnings would substantially augment government revenues.", "question": "The argument depends on the assumption that", "answers": "['there would be a net increase in employment in country X if more persons survived serious injury', 'the treatment of seriously injured persons in trauma centers is not more costly than treatment elsewhere', 'there are no specialized trauma centers in country X at present', 'most people seriously injured in automobile accidents in country X do not now receive treatment in specialized trauma centers']", "label": 0 }, { "id": "train_3198", "context": "If I borrow a generator from a neighbor, then I am morally obligated to return it when my immediate need for it is over. But suppose, instead, I borrow a car, and when I go to return it, the neighbor from whom I borrowed it is very drunk and wants to drive the car; then the obligation to return the car immediately is much less clear. So not all cases of borrowing are equivalent.", "question": "Which one of the following most accurately expresses the main point of the argument?", "answers": "['There is an obligation to refrain from acting in a manner that could result in harm to people, and this obligation overrides lesser obligations when they conflict with it.', 'Though some cases may seem to be exceptions, the rule that one ought to return to others what one borrowed from them is an exceptionless rule.', 'When someone that we know is likely to injure someone else, it is not altogether clear whether we have an obligation to intervene.', 'Although having borrowed something gives rise, in an ordinary case, to an obligation to return the item promptly, it does not so obviously give rise to such an obligation in every case.']", "label": 3 }, { "id": "train_3199", "context": "Oscar: Clearly, student evaluations provide the best assessment of teacher performance. Who is in a better position to judge a teacher than that teacher ' s students? Bettina: Student evaluations of teachers are usually conducted at the end of a class. However, students generally fail to appreciate the impact of a teacher until many years later. So, peer evaluations of teachers might be a good supplement or a better alternative.", "question": "The dialogue provides the most support for the claim that Oscar and Bettina disagree over whether", "answers": "['student evaluations furnish the optimal assessment of teacher performance', \"students are ever capable of adequately judging a teacher's performance\", 'student evaluations are usually conducted at the end of a class', 'teacher performance should be assessed without conducting any student evaluations']", "label": 0 }, { "id": "train_3200", "context": "Guidebook writer: I have visited hotels throughout the country and have noticed that in those built before 1930 the quality of the original carpentry work is generally superior to that in hotels built afterward. Clearly carpenters working on hotels before 1930 typically worked with more skill, care, and effort than carpenters who have worked on hotels built subsequently.", "question": "Which of the following, if true, most seriously weakens the guidebook writer's argument?", "answers": "['The better the quality of original carpentry in a building, the less likely that building is to fall into disuse and be demolished.', 'The quality of original carpentry in hotels is generally far superior to the quality of original carpentry in other structures, such as houses and stores.', 'The average length of apprenticeship for carpenters has declined significantly since 1930.', 'The materials available to carpenters working before 1930 were not significantly different in quality from the materials available to carpenters working after 1930.']", "label": 0 }, { "id": "train_3201", "context": "When a group of children who have been watching television programs that include acts of violence is sent to play with a group of children who have been watching programs that do not include acts of violence, the children who have been watching violent programs commit a much greater number of violent acts in their play than do the children who have been watching nonviolent programs. Ttherefore, children at play can be prevented from committing violent acts by not being allowed to watch violence on television.", "question": "The argument in the passage assumes which one of the following?", "answers": "['Children who are treated violently will respond with violence.', 'There are no other differences between the two groups of children that might account for the difference in violent behavior.', 'Violent actions and passive observation of violent actions are not related.', 'Television has a harmful effect on society.']", "label": 1 }, { "id": "train_3202", "context": "The laboratory experiment, the most effective method for teaching science, is disappearing from most secondary school curricula, and students are now simulating experiments with computers. This trend should be stopped. It results in many students' completing secondary school and going on to a university without knowing how to work with laboratory equipment.", "question": "Which one of the following, if true, most weakens the argument?", "answers": "['Computers have proven to be a valuable tool for teaching secondary school students scientific terminology.', 'In some secondary schools, teachers conduct laboratory experiments while students observe.', 'Secondary schools and universities across the nation have put a great deal of money into purchasing computers.', 'University students can learn science effectively without having had experience in working with laboratory equipment.']", "label": 3 }, { "id": "train_3203", "context": "Exposure to certain chemicals commonly used in elementary schools as cleaners or pesticides causes allergic reactions in some children. Elementary school nurses in Renston report that the proportion of schoolchildren sent to them for treatment of allergic reactions to those chemicals has increased significantly over the past ten years. Ttherefore, either Renston' s schoolchildren have been exposed to greater quantities of the chemicals, or they are more sensitive to them than schoolchildren were ten years ago.", "question": "Which of the following is an assumption on which the argument depends?", "answers": "[\"The number of school nurses employed by Renston's elementary schools has not decreased over the past ten years.\", 'The chemicals are not commonly used as cleaners or pesticides in houses and apartment buildings in Renston.', 'Children who are allergic to the chemicals are no more likely than other children to have allergies to other substances.', 'Children who have allergic reactions to the chemicals are not more likely to be sent to a school nurse now than they were ten years ago.']", "label": 3 }, { "id": "train_3204", "context": "Some people think that in every barrel of politicians there are only a few rotten ones. But if deceit is a quality of rottenness, I believe all effective politicians are rotten. They must be deceitful in order to do the job properly. Someone who is scrupulously honest about obeying the rules of society will never be an effective politician.", "question": "Assuming that the author's statements are accurate, which one of the following statements CANNOT be true?", "answers": "['Some scrupulously honest politicians are effective.', \"Some people define a politician's job as obeying the rules of society.\", 'Some politicians are scrupulously honest.', 'Some people think all politicians are rotten.']", "label": 0 }, { "id": "train_3205", "context": "Observations of the Arctic reveal that the Arctic Ocean is covered by less ice each summer than the previous summer. If this warming trend continues, within 50 years the Arctic Ocean will be ice free during the summer months. This occurrence would in itself have little or no effect on global sea levels, since the melting of ice floating in water does not affect the water level. However, serious consequences to sea levels would eventually result, because __.", "question": "Which of the following most logically completes the passage?", "answers": "['in the spring, melting sea ice would cause more icebergs to be created and to drift south into shipping routes', 'significant changes in Arctic sea temperatures would be accompanied by changes in sea temperatures in more temperate parts of the world', 'such a warm Arctic Ocean would trigger the melting of massive landbased glaciers in the Arctic', 'large masses of floating sea ice would continue to form in the wintertime']", "label": 2 }, { "id": "train_3206", "context": "It is difficult to keep deep wounds free of bacteria. Even strong antibiotics fail to kill the bacteria that live in such wounds. However, many physicians have succeeded in eliminating bacteria from deep wounds by packing the wound with a sweet substance like sugar.", "question": "Which one of the following, if true, most helps to explain why treating deep wounds with sugar as described above is successful?", "answers": "['Sugar that is nearly pure is readily available for use in medical treatments.', 'Many kinds of bacteria can use sugar as a nutrient and will reproduce rapidly when sugar is available to them.', 'Bacteria that live in deep wounds thrive in a moist environment, and sugar has a dehydrating effect.', 'Some foods that contain sugar can weaken the effects of certain antibiotics.']", "label": 2 }, { "id": "train_3207", "context": "In a study, shoppers who shopped in a grocery store without a shopping list and bought only items that were on sale for half price or less spent far more money on a comparable number of items than did shoppers in the same store who used a list and bought no sale items.", "question": "Which one of the following, if true, most helps to explain the apparent paradox in the study's results?", "answers": "['The grocery store in the study carries many expensive items that few other grocery stores carry.', 'The shoppers who did not use lists bought many unnecessary items.', 'Usually, only the most expensive items go on sale in grocery stores.', 'Only the shoppers who used a list used a shopping cart.']", "label": 2 }, { "id": "train_3208", "context": "Most business ethics courses and textbooks confine themselves to considering specific cases and principles. For example, students are often given lists of ethical rules for in-class discussion and role-playing. This approach fails to provide a framework for understanding specific principles and should thus be changed to include abstract ethical theory.", "question": "Which one of the following, if valid, most helps to justify the reasoning above?", "answers": "['Abstract ethical theory is the most appropriate of any context for understanding specific principles.', 'An ethics course should acquaint students with a wide range of specific principles and appropriate applications.', 'People have no obligation to always behave ethically unless they are acquainted with abstract ethical theory.', 'Courses that concentrate mainly on role- playing are undesirable because students must adopt alien personae.']", "label": 0 }, { "id": "train_3209", "context": "Bobo the clown books more shows and makes more money than Gob the magician. Despite rampant coulrophobia -- an irrational fear of clowns -- Bobo still books more parties and receives higher rates of compensation per show. Gob' s magic shows are no worse than Bobo' s clown performances.", "question": "Which of the following statements, if true, best explains the apparent paradox?", "answers": "['Gob is a below average magician.', \"Despite rampant coulrophobia, statistical data shows that people generally prefer clowns to magicians for children's birthday parties.\", 'Bobo is an experienced clown.', 'Bobo works in a densely populated city, while Gob works in a rural town.']", "label": 3 }, { "id": "train_3210", "context": "Gas station owner: Increased fuel efficiency reduces air pollution and dependence on imported oil, which has led some people to suggest that automobile manufacturers should make cars smaller to increase their fuel efficiency. But smaller cars are more likely to be seriously damaged in collisions and provide less protection for their occupants. Greater fuel efficiency is not worth the added risk to human lives; ttherefore, manufacturers should not seek to increase fuel efficiency.", "question": "The reasoning in the gas station owner's argument is flawed because the argument", "answers": "['presumes, without providing justification, that it would be impossible to reduce the likelihood of dangerous accidents for small cars', 'concludes, on the basis of the claim that one means to an end is unacceptable, that the end should not be pursued', 'presumes, without providing justification, that increasing fuel efficiency is the only way to reduce air pollution', 'presupposes the truth of what it sets out to prove']", "label": 1 }, { "id": "train_3211", "context": "Capuchin monkeys often rub their bodies with a certain type of millipede. Laboratory tests show that secretions from the bodies of these millipedes are rich in two chemicals that are potent mosquito repellents, and mosquitoes carry parasites that debilitate capuchins. Some scientists hypothesize that the monkeys rub their bodies with the millipedes because doing so helps protect them from mosquitoes.", "question": "Which of the following, if true, provides the most support for the scientists' hypothesis?", "answers": "['The two insect-repelling chemicals in the secretions of the millipedes are carcinogenic for humans but do not appear to be carcinogenic for capuchins.', 'The capuchins rarely rub their bodies with the millipedes except during the rainy season, when mosquito populations are at their peak.', 'The two chemicals that repel mosquitoes also repel several other varieties of insects.', 'Although the capuchins eat several species of insects, they do not eat the type of millipede they use to rub their bodies.']", "label": 1 }, { "id": "train_3212", "context": "The recent concert was probably not properly promoted. Wells, who is quite knowledgeable about the concert business, was certain that it would sell out unless it was poorly promoted. But the concert did not sell out.", "question": "The pattern of reasoning in which one of the following is most similar to that in the argument above?", "answers": "['Professor Willis, who is quite knowledgeable about organic chemistry, said that the sample probably did not contain any organic compounds. So, the sample probably is not labeled correctly, for if it were, it would contain organic compounds.', \"The builder said that the school's roof would not require repairs for years, unless it is damaged in a storm. The roof is already leaking. Thus, since there have been no major storms, the builder was probably wrong.\", 'My neighbor, who is an experienced home renovator, said the damage to the wall would not be noticeable if it were properly repaired. Thus, the repair to the wall probably was not properly done, since one can still notice the damage.', 'Dr. Smith, a well-trained cardiologist, said the patient would probably survive the heart transplant if it were performed by a highly skilled surgeon. Thus, since the patient did not survive the surgery, it probably was not properly performed.']", "label": 2 }, { "id": "train_3213", "context": "Sea turtle hatchlings leaving their hatching grounds on Florida beaches reach ocean currents by swimming to the northeast, as defined by the north of the Earth' s magnetic field. Florida hatchlings placed in a large indoor tank also swim toward the northeast. But when the tank is surrounded by an artificial magnetic field twice as strong as the Earth' s field and opposite in direction, the hatchlings swim in the direction opposite to that in which they swim without the artificial magnetic field.", "question": "The information in the statements above most strongly supports which one of the following?", "answers": "['If a sea turtle hatches on the coast of Africa, it will swim toward the southwest.', 'Once baby sea turtles reach the open sea, they join groups of adults in the North Atlantic.', 'Baby sea turtles are able to sense the magnetic field of the Earth.', 'The direction in which ocean currents flow is determined by the magnetic field of the Earth.']", "label": 2 }, { "id": "train_3214", "context": "Legal theorist: It is unreasonable to incarcerate anyone for any other reason than that he or she is a serious threat to the property or lives of other people. The breaking of a law does not justify incarceration, for lawbreaking proceeds either from ignorance of the law or of the effects of one' s actions, or from the free choice on the part of the lawbreaker. Obviously mere ignorance cannot justify incarcerating a lawbreaker, and even free choice on the part of the lawbreaker fails to justify incarceration, for free choice proceeds from the desires of an agent, and the desires of an agent are products of genetics and environmental conditioning, neither of which is controlled by the agent.", "question": "The claim in the first sentence of the passage plays which one of the following roles in the argument?", "answers": "['It is offered as evidence for the stated claim that protection of life and property is more important than retribution for past illegal acts.', \"It is offered as evidence for the stated claim that lawbreaking proceeds from either ignorance of the law, or ignorance of the effects of one's actions, or free choice.\", 'It is offered as the main conclusion that the argument is designed to establish.', 'It is offered as background information necessary to understand the argument.']", "label": 2 }, { "id": "train_3215", "context": "The ability of mammals to control their internal body temperatures is a factor in the development of their brains and intelligence. This can be seen from the following facts: the brain is a chemical machine, all chemical reactions are temperature dependent, and any organism that can control its body temperature can assure that these reactions occur at the proper temperatures.", "question": "Which one of the following is an assumption on which the argument depends?", "answers": "['Organisms incapable of controlling their internal body temperatures are subject to unpredictable chemical processes.', 'Organisms unable to control their body temperatures do not have the capacity to generate internal body heat without relying on external factors.', 'The development of intelligence in mammals is not independent of the chemical reactions in their brains taking place at the proper temperatures.', 'The brain cannot support intelligence if the chemical reactions within it are subject to uncontrolled temperatures.']", "label": 2 }, { "id": "train_3216", "context": "Physician: The patient is suffering either from disease X or else from disease Y, but there is no available test for distinguishing X from Y. Ttherefore, since there is an effective treatment for Y but no treatment for X, we must act on the assumption that the patient has a case of Y.", "question": "The physician's reasoning could be based on which one of the following principles?", "answers": "['When only one strategy carries the possibility of success, circumstances must as much as possible be changed to fit this strategy.', 'When the soundness of a strategy depends on the truth of a certain assumption, the first step in putting the strategy into effect must be to test the truth of this assumption.', 'In treating a patient who has one or the other of two diseases, it is more important to treat the diseases than to determine which of the two diseases the patient has.', \"When success is possible only if a circumstance beyond one's control is favorable, then one's strategy must be based on the assumption that this circumstance is in fact favorable.\"]", "label": 3 }, { "id": "train_3217", "context": "Many small roads do not have painted markings along their edges. Clear edge markings would make it easier for drivers to see upcoming curves and to judge the car' s position on the road, particularly when visibility is poor, and would ttherefore seem to be a useful contribution to road safety. However, after Greatwater County painted edge markings on all its narrow, winding roads, the annual accident rate along those roads actually increased slightly.", "question": "Which of the following, if true, most helps to explain the increase in accident rate?", "answers": "[\"Prior to the painting of the edge markings, Greatwater County's narrow, winding roads already had a somewhat higher accident rate than other Greatwater County roads.\", 'After the markings were painted on the roads, many drivers who had gone out of their way to avoid driving on those roads at night no longer did so.', 'In bad weather it can be nearly as difficult for drivers to see the road as it is at night.', 'Many of the accidents on narrow, winding roads involve a single vehicle veering off the road, rather than the collision of two vehicles.']", "label": 1 }, { "id": "train_3218", "context": "Rats injected with morphine exhibit decreased activity of the immune system, the bodily system that fights off infections. These same rats exhibited heightened blood levels of corticosteroids, chemicals secreted by the adrenal glands. Since corticosteroids can interfere with immune-system activity, scientists hypothesized that the way morphine reduces immune responses in rats is by stimulating the adrenal glands to secrete additional corticosteroids into the bloodstream.", "question": "Which of the following experiments would yield the most useful results for evaluating the scientists' hypothesis?", "answers": "[\"Testing the level of immune-system activity of rats, removing their adrenal glands, and then testing the rats' immune-system activity levels again\", 'Injecting rats with corticosteroids and then observing how many of the rats contracted infections', 'Injecting morphine into rats that already have heightened blood levels of corticosteroids and then observing their new blood levels of corticosteroids', \"Removing the adrenal glands of rats, injecting the rats with morphine, and then testing the level of the rats' immune-system responses\"]", "label": 3 }, { "id": "train_3219", "context": "In their native habitat, amaryllis plants go dormant when the soil in which they are growing dries out during the dry season. Ttherefore, if amaryllis plants kept as houseplants are to thrive, water should be withheld from them during part of the year so that the plants go dormant.", "question": "Which one of the following is an assumption on which the argument depends?", "answers": "['Most kinds of plants go dormant at some time or other during the year.', 'Amaryllis are more difficult to keep as houseplants than other kinds of plants are.', 'Any amaryllis plant that fails to thrive is likely to have been dormant for too short a time.', 'Going dormant benefits amaryllis plants in their native habitat in some way other than simply preventing death during overly dry periods.']", "label": 3 }, { "id": "train_3220", "context": "Classical Roman architecture is beautiful, primarily because of its use of rounded arches and its symmetry. Postmodern architecture is dramatic, primarily because of its creative use both of materials and of the surrounding environment. An architectural style that combines elements of both classical Roman and postmodern architecture would ttherefore be both beautiful and dramatic.", "question": "The reasoning in the argument is flawed in that it", "answers": "['neglects to consider that an architectural style combining elements of two other architectural styles may lack certain qualities of one or both of those styles', 'fails to justify its presumption that because postmodern architecture is dramatic, that is its most salient feature', 'presumes, without providing justification, that for an architectural style to have certain qualities, its components must have those qualities', 'ignores the possibility that there are other architectural styles whose defining qualities include both drama and beauty']", "label": 0 }, { "id": "train_3221", "context": "When a city experiences a sharp decline in population, the city' s tax revenues, which pay for such city services as police protection and maintenance of water lines, also decrease. The area to be policed and the number and length of the water lines to be maintained, however, do not decrease. Attempting to make up the tax revenue lost by raising tax rates is not feasible, since higher tax rates would cause even more residents to leave.", "question": "The information given most strongly supports which of the following general claims?", "answers": "[\"If a city's tax rates are held stable over a period of time, neither the population nor the levels of city services provided will tend to decline over that period.\", 'A city that is losing residents because tax rates are perceived as too high by those residents can reverse this population trend by bringing its tax rates down to a more moderate level.', 'If, in a city with sharply declining population, police protection and water line maintenance do not deteriorate, some other service previously provided by the city will deteriorate or be eliminated.', \"A city that suffers revenue losses because of a sharp decline in population can make up some of the lost tax revenue by raising tax rates, provided the city's tax rates are low in relation to those of other cities.\"]", "label": 2 }, { "id": "train_3222", "context": "The traditional way to define the difference between rural and urban lifestyles is geographically. But with the impact of communications technology it makes more sense to draw the distinction in informational terms. People who rarely communicate electronically with anyone are living rural lifestyles, irrespective of where they live, while people who communicate daily with dozens of people via fax or modem are living urban lifestyles, even if they live in the country.", "question": "The situation described above most closely illustrates which one of the following propositions?", "answers": "['Many people who use electronic technology find urban lifestyles more satisfying than they find rural lifestyles.', 'We are unable to foresee the magnitude of the changes that the information revolution may have in defining our lives.', 'People are choosing to live in different regions of the nation than previously because of the impact of electronic communications technology.', 'Frequency of electronic communication with others is superseding geographical considerations in defining our lifestyles.']", "label": 3 }, { "id": "train_3223", "context": "In Rubaria, excellent health care is available to virtually the entire population, whereas very few people in Terland receive adequate medical care. Yet, although the death rate for most diseases is higher in Terland than in Rubaria, the percentage of the male population that dies from prostate cancer is significantly higher in Rubaria than in Terland.", "question": "Which of the following, if true, most helps to explain the disparity between the prostate cancer death rate in Rubaria and Terland?", "answers": "[\"Among men in Rubaria, the death rate from prostate cancer is significantly higher for those who do not take full advantage of Rubaria's health care system than for those who do.\", 'Most men who have prostate cancer are older than the average life expectancy for male inhabitants of Terland.', 'Effective treatment of prostate cancer in its early stages generally requires medical techniques available in Rubaria but not in Terland.', \"It is possible to decrease one's risk of getting prostate cancer by eating certain kinds of foods, and such foods are more readily available in Rubaria than in Terland.\"]", "label": 1 }, { "id": "train_3224", "context": "Dietitian: It is true that nutrients are most effective when provided by natural foods rather than artificial supplements. While it is also true that fat in one' s diet is generally unhealthy, eating raw carrots (which are rich in beta carotene) by themselves is nonetheless not an effective means of obtaining vitamin A, since the body cannot transform beta carotene into vitamin A unless it is consumed with at least some fat.", "question": "The statement that fat in one's diet is generally unhealthy plays which one of the following roles in the dietitian's argument?", "answers": "['It is mentioned as a generally accepted hypothesis that the dietitian attempts to undermine completely.', 'It is mentioned as a reason for adopting a dietary practice that the dietitian provides a reason for not carrying to the extreme.', 'It is cited as a bad reason for adopting a dietary habit that the dietitian recommends.', 'It is attacked as inadequate evidence for the claim that nutrients are most effective when provided by natural foods rather than artificial supplements.']", "label": 1 }, { "id": "train_3225", "context": "Anthropologist: It has been claimed that religious prohibitions against eating certain types of food are evidence against the belief that all cultural phenomena have a purely economic explanation. After all, the reasoning goes, only a moral or spiritual motive could persuade people to forgo readily available sources of nutrients. But the species whose consumption is prohibited are usually essential elements of ecosystems containing other species that are used as food. The preservation of the prohibited species thus tends to help preserve the other species. Ttherefore, such prohibitions do indeed have an economic motivation.", "question": "The reasoning in the anthropologist's argument is flawed because the argument", "answers": "['takes for granted that people could have a reason for adopting a certain policy and nevertheless not realize what that reason is', 'uses the key term \"prohibition\" in two different senses', 'infers, merely from the claim that a practice has a certain desirable consequence, that this consequence must provide a motivation for the practice', 'infers, merely from the claim that a certain cultural phenomenon has no economic explanation, that the explanation of this phenomenon must be spiritual or moral']", "label": 2 }, { "id": "train_3226", "context": "Any health care plan that does not cover comprehensive annual checkups is risking the lives of its members, who are unlikely to obtain care that is not covered by their health care plans, and who may have conditions that, if not detected early, are more likely to become fatal.", "question": "The argument above logically depends on which of the following assumptions?", "answers": "[\"People are less likely to purchase a health care plan that does not cover annual checkups, thus negatively affecting the insurers' profitability.\", 'Doctors who perform comprehensive annual checkups charge high fees, such that health care plan members cannot afford checkups that are not covered.', 'Health care plans cannot save lives unless they also cover medical procedures that treat any potentially fatal conditions that are detected.', 'Many potentially lethal conditions are likely to be detected during an annual checkup.']", "label": 3 }, { "id": "train_3227", "context": "Florist: Some people like to have green carnations on St. Patrick' s Day. But flowers that are naturally green are extremely rare. Thus, it is very difficult for plant breeders to produce green carnations. Before St. Patrick' s Day, then, it is wise for florists to stock up on white carnations, which are fairly inexpensive and quite easy to dye green.", "question": "Which one of the following most accurately expresses the overall conclusion of the florist's argument?", "answers": "['White carnations are fairly inexpensive and can easily be dyed green.', \"There are some people who like to have green carnations on St. Patrick's Day.\", \"It is a good idea for florists to stock up on white carnations before St. Patrick's Day.\", 'It is very difficult to breed green carnations.']", "label": 2 }, { "id": "train_3228", "context": "Sunflowers growing in pots were placed, with their roots submerged, in the pond contaminated with radioactive elements. The sunflowers kept growing; in the process, they absorbed radioactive elements. Within twelve days, 85 percent of the radioactive elements were removed from the water, which is no less than can be accomplished with the much more expensive conventional filtration techniques. Scientists ttherefore propose using sunflowers for decontamination wherever there are radioactively contaminated ponds.", "question": "Which of the following, if true, points to a limitation on the applicability of the proposed method of decontamination?", "answers": "['The water in many ponds contaminated with radioactive elements is so cold that it would kill sunflowers whose roots were submerged in it.', 'Sunflowers that grow with their roots submerged in water grow less well than sunflowers growing under optimal conditions on dry land.', 'Some plants other than sunflowers can also remove radioactive elements from water.', 'In ponds in which the circulation of the water is artificially increased, sunflowers absorb radioactive elements far faster than they do in other ponds.']", "label": 0 }, { "id": "train_3229", "context": "In the Hartshorn Building, most but not all of the third-floor offices are larger than any office on the second floor. The fourth-floor offices are all larger than any office on the second floor. However, all the second-floor offices are larger than any office on the first floor.", "question": "If the statements above are true, which one of the following must also be true?", "answers": "['Some first-floor offices are as large as the smallest fourth-floor offices.', 'Some fourth-floor offices are not as large as the largest third-floor offices.', 'Some first-floor offices are as large as the smallest third-floor offices.', 'Some third-floor offices are not as large as the smallest fourth-floor offices.']", "label": 3 }, { "id": "train_3230", "context": "Trade official: Country X deserves economic retribution for its protectionism. However, it is crucial that we recognize that there are overriding considerations in this case. We should still sell to X the agricultural equipment it ordered; there is high demand in our country for agricultural imports from X.", "question": "The argument depends on assuming which one of the following principles?", "answers": "['In most cases, punishing a projectionist country should have priority over the interests of our people.', 'We should balance the justice of an action with the consequences for our interests of undertaking that action.', 'We should never jeopardize the interests of our people to punish a projectionist country.', 'The ability to keep popular products available domestically is less important than our being able to enter international markets.']", "label": 1 }, { "id": "train_3231", "context": "From a magazine article: Self-confidence is a dangerous virtue: it often degenerates into the vice of arrogance. The danger of arrogance is evident to all who care to look. How much more humane the twentieth century would have been without the arrogant self-confidence of a Hitler or a Stalin!", "question": "The author attempts to persuade by doing all of the following EXCEPT", "answers": "['appealing to authority to substantiate an assertion', 'introducing value-laden terms, such as \"vice\"', \"implying that Hitler's arrogance arose from self-confidence\", 'using extreme cases to evoke an emotional response']", "label": 0 }, { "id": "train_3232", "context": "Would it be right for the government to abandon efforts to determine at what levels to allow toxic substances in our food supply? Only if it can reasonably be argued that the only acceptable level of toxic substances in food is zero. However, virtually all foods contain perfectly natural substances that are toxic but cause no harm because they do not occur in food in toxic concentrations. Furthermore, we can never be certain of having reduced the concentration of any substance to zero; all we can ever know is that it has been reduced to below the threshold of detection of current analytical methods.", "question": "The main conclusion of the argument is that", "answers": "['the only acceptable level of toxic substances in food is zero', 'the government should continue trying to determine acceptable levels for toxic substances in our food supply', 'the government needs to refine its methods of detecting toxic substances in our food supply', 'naturally occurring toxic substances in food present little danger because they rarely occur in toxic concentrations']", "label": 1 }, { "id": "train_3233", "context": "Economist: Every business strives to increase its productivity, for this increases profits for the owners and the likelihood that the business will survive. But not all efforts to increase productivity are beneficial to the business as a whole. Often, attempts to increase productivity decrease the number of employees, which clearly harms the dismissed employees as well as the sense of security of the retained employees.", "question": "Which one of the following most accurately expresses the main conclusion of the economist's argument?", "answers": "['Decreasing the number of employees in a business undermines the sense of security of retained employees.', \"If an action taken to secure the survival of a business fails to enhance the welfare of the business's employees, that action cannot be good for the business as a whole.\", 'Only if the employees of a business are also its owners will the interests of the employees and owners coincide, enabling measures that will be beneficial to the business as a whole.', 'Some measures taken by a business to increase productivity fail to be beneficial to the business as a whole.']", "label": 3 }, { "id": "train_3234", "context": "Advertisement: The pride the people at Austin Stables take in their work accounts for their success in producing more winning racehorses than any other stable. Such a tradition of pride is not only found in the business of horse racing. For generations we at Barr Motor Company have demonstrated similar pride. You can rely on Barr Motor Company to produce more winning automobiles than our competitors.", "question": "The advertisement proceeds by", "answers": "['using an analogy to reach the conclusion that Barr Motor Company is superior to its competitors', 'demonstrating that Barr Motor Company has more repeat customers than its competitors', 'asserting that Barr Motor Company has an older tradition of pride than does Austin Stables', 'proving that Barr Motor Company has a long- standing tradition of pride']", "label": 0 }, { "id": "train_3235", "context": "Principle: The government should not prevent someone from expressing a true belief unless expressing it would be harmful to people generally. Application: The government was wrong to prevent Calista from publicly expressing her belief that there is evidence that cancer rates have increased slightly over the last two decades and that this increase was due partly to excessive use of cell phones.", "question": "Which one of the following, if true, would most help to justify the above application of the principle?", "answers": "['Unless there is strong evidence of a link between use of a product and disease, the suggestion that use of the product causes disease is usually harmful to people.', 'The government has conducted extensive research to determine whether there is any causal link between use of cell phones and cancer.', 'Most people would reduce their use of cell phones if they were convinced that they were using them enough to increase their risk of developing cancer.', 'Several studies have found evidence that use of cell phones has been partially responsible for the increase in cancer rates over the last two decades, and it would benefit people to know this.']", "label": 3 }, { "id": "train_3236", "context": "Novelists cannot become great as long as they remain in academia. Powers of observation and analysis, which schools successfully hone, are useful to the novelist, but an intuitive grasp of the emotions of everyday life can be obtained only by the kind of immersion in everyday life that is precluded by being an academic.", "question": "Which one of the following is an assumption on which the argument depends?", "answers": "['Participation in life, interspersed with impartial observation of life, makes novelists great.', 'Novelists require some impartiality to get an intuitive grasp of the emotions of everyday life.', 'Novelists cannot be great without an intuitive grasp of the emotions of everyday life.', 'No great novelist lacks powers of observation and analysis.']", "label": 2 }, { "id": "train_3237", "context": "On the Caribbean island of Guadeloupe, a researcher examined 35 patients with atypical Parkinson' s disease and compared their eating habits to those of 65 healthy adults. She found that all of the patients with atypical Parkinson' s regularly ate the tropical fruits soursop, custard apple, and pomme cannelle, whereas only 10 of the healthy adults regularly ate these fruits. From this, she concluded that eating these fruits causes atypical Parkinson' s.", "question": "Which one of the following, if true, most strengthens the researcher's reasoning?", "answers": "[\"The 10 healthy adults who regularly ate soursop, custard apple, and pomme cannelle ate significantly greater quantities of these fruits, on average, than did the 35 atypical Parkinson's patients.\", 'Of the healthy adults who did not regularly eat soursop, custard apple, and pomme cannelle, most had eaten each of these fruits on at least one occasion.', \"In areas other than Guadeloupe, many people who have never eaten soursop, custard apple, and pomme cannelle have contracted atypical Parkinson's.\", \"For many of the atypical Parkinson's patients, their symptoms stopped getting worse, and in some cases actually abated, when they stopped eating soursop, custard apple, and pomme cannelle.\"]", "label": 3 }, { "id": "train_3238", "context": "The proportion of fat calories in the diets of people who read the nutrition labels on food products is significantly lower than it is in the diets of people who do not read nutrition labels. This shows that reading these labels promotes healthful dietary behavior.", "question": "The reasoning in the argument above is flawed in that the argument", "answers": "['illicitly infers a cause from a correlation', 'relies on a sample that is unlikely to be representative of the group as a whole', 'confuses a condition that is necessary for a phenomenon to occur with a condition that is sufficient for that phenomenon to occur', 'draws a conclusion about the intentions of a group of people based solely on data about the consequences of their behavior']", "label": 0 }, { "id": "train_3239", "context": "The police department has two suspects for the burglary that occurred last night, Schaeffer and Forster. Schaeffer has an ironclad alibi, so Forster must be the burglar.", "question": "Which one of the following arguments exhibits a flawed pattern of reasoning that is most similar to that exhibited by the argument above?", "answers": "['If Iano Industries does not borrow money so that it can upgrade its factories, it will be unable to compete. While it is undesirable for Iano to take on more debt, being unable to compete would be even worse. So Iano should borrow the money needed to upgrade its factories.', 'It has been known for some time that the Wrightsburg Zoo might build a new primate house and that it might refurbish its polar bear exhibit. There is now good reason to believe the zoo will build a new primate house. Ttherefore, the zoo will not refurbish its polar bear exhibit.', 'Baxim Corporation announced last year that it was considering moving its headquarters to Evansville and that it was also considering moving to Rivertown. But Baxim has now decided not to move to Evansville. Thus, we can be sure that Baxim will move to Rivertown.', 'If Watson, a robbery suspect, had been picked out of a police lineup by the victim, then charging Watson with robbery would have been reasonable. But the victim did not pick Watson out of the lineup. So Watson should not be charged.']", "label": 2 }, { "id": "train_3240", "context": "Theater Critic: The play La Finestrina, now at Central Theater, was written in Italy in the eighteenth century. The director claims that this production is as similar to the original production as is possible in a modern theater. Although the actor who plays Harlequin the clown gives a performance very reminiscent of the twentieth-century American comedian Groucho Marx, Marx's comic style was very much within the comic acting tradition that had begun in sixteenth-century Italy.", "question": "The considerations given best serve as part of an argument that", "answers": "['Groucho Marx once performed the part of the character Harlequin in La Finestrina', \"the performance of the actor who plays Harlequin in La Finestrina does not serve as evidence against the director's claim\", 'in the United States the training of actors in the twentieth century is based on principles that do not differ radically from those that underlay the training of actors in eighteenth-century Italy', 'modern audiences would find it hard to tolerate certain characteristics of a historically accurate performance of an eighteenth-century play']", "label": 1 }, { "id": "train_3241", "context": "In experiments in which certain kinds of bacteria were placed in a generous supply of nutrients, the populations of bacteria grew rapidly, and genetic mutations occurred at random in the populations. These experiments show that all genetic mutation is random.", "question": "Which one of the following, if true, enables the conclusion to be properly drawn?", "answers": "['The nutrients used were the same as those that nourish the bacteria in nature.', 'Either all genetic mutations are random or none are random.', 'The kind of environment in which genetic mutation takes place has no effect on the way genetic mutation occurs.', 'The bacteria tested in the experiments were of extremely common forms.']", "label": 1 }, { "id": "train_3242", "context": "Human beings have cognitive faculties that are superior to those of other animals, and once humans become aware of these, they cannot be made happy by anything that does not involve gratification of these faculties.", "question": "Which one of the following statements, if true, most calls into question the view above?", "answers": "['Someone who never experienced classical music as a child will usually prefer popular music as an adult.', 'Many people familiar both with intellectual stimulation and with physical pleasures enjoy the latter more.', 'Many people who are serious athletes consider themselves to be happy.', 'Many people who are serious athletes love gourmet food.']", "label": 1 }, { "id": "train_3243", "context": "Essayist: Every contract negotiator has been lied to by someone or other, and whoever lies to anyone is practicing deception. But, of course, anyone who has been lied to has also lied to someone or other.", "question": "If the essayist' s statements are true, which one of the following must also be true?", "answers": "['Not everyone who lies to someone is practicing deception.', 'Every contract negotiator has practiced deception.', 'Not everyone who practices deception is lying to someone.', 'Whoever lies to anyone is lied to by someone. ST']", "label": 1 }, { "id": "train_3244", "context": "Last year, a woman was able to demonstrate that she contracted a bad case of food poisoning from a meal at one Chinese restaurant in Bairenville, and she successfully sued the restaurant for a large sum of money. The story was popular in the town, and the size of the financial settlement made the national news. This year, we have seen a number of \"copycat\" suits filed against each one of the eleven Chinese restaurants in Bairenville, forcing these restaurants to hire lawyers and take steps to defend themselves in court.", "question": "Which of following conclusions can most properly be drawn from the information above?", "answers": "['The cooking methods used for Chinese food are less likely to kill germs than are the cooking methods used in other cuisines.', 'This year, a number of citizens of Bairenville have won substantial settlements as the result of lawsuits.', 'This year, some restaurants in Bairenville have been subjected to legal expenses, irrespective of whether any of their patrons have suffered from food poisoning.', 'For each of the eleven Chinese restaurants in Bairenville, at least one person has contracted food poisoning from a meal on some occasion.']", "label": 2 }, { "id": "train_3245", "context": "Since the deregulation of airlines, delays at the nation' s increasingly busy airports have increased by 25 percent. To combat this problem, more of the takeoff and landing slots at the busiest airports must be allocated to commercial airlines.", "question": "Which of the following, if true, casts the most doubt on the effectiveness of the solution proposed above?", "answers": "[\"The major causes of delays at the nation's busiest airports are bad weather and overtaxed air traffic control equipment.\", 'After a small Midwestern airport doubled its allocation of takeoff and landing slots, the number of delays that were reported decreased by 50 percent.', \"Since deregulation the average length of delay at the nation's busiest airports has doubled.\", \"Over 60 percent of the takeoff and landing slots at the nation's busiest airports are reserved for commercial airlines.\"]", "label": 0 }, { "id": "train_3246", "context": "Manager: This company' s supply chain will develop significant weaknesses unless we make changes to our vendor contracts now. Some will argue that this problem is so far in the future that there is no need to address it today. But that is an irresponsible approach. Just imagine if a financial planner offered the same counsel to a 30-year-old client: \"Don' t worry, Jane, retirement is 35 years away; you don' t need to save anything now. \" That planner would be guilty of gross malpractice.", "question": "Which one of the following most accurately expresses the overall conclusion drawn in the manager's argument?", "answers": "['Some people argue that the supply-chain problem is so far in the future that there is no need to address it now.', 'In planning to meet its future obligations, a company should follow the same practices that are appropriate for an individual who is planning for retirement.', 'Financial planners should advise their clients to save money for retirement only if retirement is many years away.', 'It would be irresponsible to postpone changes to the vendor contracts just because the supply chain will not develop weaknesses for a long time.']", "label": 3 }, { "id": "train_3247", "context": "People often praise poems for their truth. But to argue that expressing true propositions contributes to the aesthetic merit of a poem is misguided. Most of the commonplace beliefs of most people are true. Whatever the basis of poetic excellence is, it must certainly be rare rather than common.", "question": "Which one of the following most accurately describes the role played in the argument by the claim that whatever the basis of poetic excellence is, it must certainly be rare rather than common?", "answers": "[\"It is a premise that, in conjunction with another premise, is intended to support the argument's conclusion.\", 'It is the overall conclusion drawn by the argument.', \"It is a premise offered as the sole support for the argument's conclusion.\", 'It is a proposition for which the argument seeks to advance an explanation.']", "label": 0 }, { "id": "train_3248", "context": "Most household appliances use electricity only when in use. Many microwave ovens, however, have built-in clocks and so use some electricity even when they are not in use. The clocks each consume about 45 kilowatt-hours per year. Ttherefore, households whose microwave oven has no built-in clock use 45 kilowatt-hours per year less, on average, than do comparable households whose microwave oven is otherwise similar but has a built-in clock.", "question": "Which of the following is an assumption on which the argument depends?", "answers": "['Households whose microwave oven does not have a built-in clock are no more likely to have a separate electric clock plugged in than households whose microwave oven has one.', 'All households that have a microwave oven also have either a gas oven or a conventional electric oven.', 'There are more households that have a microwave oven with a built-in clock than there are households that have a microwave oven without a built-in clock.', 'Households that do not have a microwave oven use less energy per year, on average, than do households that have a microwave oven.']", "label": 0 }, { "id": "train_3249", "context": "Museum visitor: The national government has mandated a 5 percent increase in the minimum wage paid to all workers. This mandate will adversely affect the museum-going public. The museum' s revenue does not currently exceed its expenses, and since the mandate will significantly increase the museum' s operating expenses, the museum will be forced either to raise admission fees or to decrease services.", "question": "Which one of the following is an assumption required by the museum visitor's argument?", "answers": "[\"Some of the museum's employees are paid more than the current minimum wage.\", \"Some of the museum's employees are not paid significantly more than the minimum wage.\", 'The annual number of visitors to the museum has increased steadily.', 'Not all visitors to the museum are required to pay an admission fee.']", "label": 1 }, { "id": "train_3250", "context": "People with serious financial problems are so worried about money that they cannot be happy. Their misery makes everyone close to them -- family, friends, colleagues -- unhappy as well. Only if their financial problems are solved can they and those around them be happy.", "question": "Which one of the following statements can be properly inferred from the passage?", "answers": "['People who do not have serious financial problems will be happy.', 'Only serious problems make people unhappy.', 'People who solve their serious financial problems will be happy.', 'If people are happy, they do not have serious financial problems.']", "label": 3 }, { "id": "train_3251", "context": "The percentage of people between the ages of 18 and 24 living with their parents increased from 48 percent in 1980 to 53 percent in 1986. It can be concluded that in 1986 it was harder for people in this age group to afford to live by themselves.", "question": "The conclusion drawn in the passage above depends on which of the following assumptions?", "answers": "['People in this age group who lived with their parents did not make any financial contribution toward housing expenses.', 'People in this age group who could not afford to live by themselves preferred living with their parents.', 'There are people in this age group who although they lied with their parents at the time of survey, had previously lived alone.', 'The number of rental housing units suitable for single people dropped between 1980 and 1986.']", "label": 1 }, { "id": "train_3252", "context": "Anyone who insists that music videos are an art form should also agree that television gave rise to an art form, since television gave rise to music videos.", "question": "The pattern of reasoning displayed in the argument above most closely parallels that displayed in which one of the following?", "answers": "['Anyone who holds that avocados are a fruit should also hold that pound cake is lower in fat than some fruit, since pound cake is lower in fat than avocados.', 'A person who claims to prefer fruit to vegetables should also prefer cake to bread, since fruit is sweeter than vegetables and cake is sweeter than bread.', 'Anyone who claims that all vegetables are nutritious should also agree that some vegetables are harmful if eaten in large quantities.', 'A person who eats a variety of vegetables is probably well nourished, since most people who eat a variety of vegetables generally eat well-balanced meals.']", "label": 0 }, { "id": "train_3253", "context": "Parent: The city education department is unable to distinguish between annoyances and important problems. For instance, prohibiting students from having cell phones is an overreaction. If a student uses one and thus interferes with instruction, confiscate it. All in all, we need educational leadership that can solve problems, not create them.", "question": "Which of the following is an assumption made by the parent?", "answers": "['Faculty and staff should be allowed to possess cell phones.', 'Students need to have cell phones because some of them have no stay-at-home parent.', 'Students having cell phones does not constitute an important problem for the city schools.', 'Students have no need for cell phones in school.']", "label": 2 }, { "id": "train_3254", "context": "Ethicist: Robert Gillette has argued that because a thorough knowledge of genetics would enable us to cure the over 3, 000 inherited disorders that affect humanity, deciphering the human genetic code will certainly benefit humanity despite its enormous cost. Gillette' s argument is not persuasive, however, because he fails to consider that such knowledge might ultimately harm human beings more than it would benefit them.", "question": "Which one of the following most accurately expresses the conclusion of the ethicist's argument?", "answers": "[\"Gillette's argument wrongly assumes that deciphering the genetic code will lead to cures for genetic disorders.\", \"Gillette's claim that a thorough knowledge of genetics would enable us to cure over 3, 000 disorders is overstated.\", 'Deciphering the genetic code might ultimately harm human beings more than benefit them.', \"Gillette's argument is unconvincing because it ignores certain possible consequences of genetic research.\"]", "label": 3 }, { "id": "train_3255", "context": "Child development specialists have observed that adolescents who receive large weekly allowances tend to spend money on items considered frivolous by their parents whereas adolescents who receive small weekly allowances do not. Thus, in order to ensure that their children do not spend money on frivolous items, parents should not give their children large weekly allowances.", "question": "Which of the following pieces of information would be most useful in evaluating the validity of the conclusion above?", "answers": "['The percentage of adolescents who receive no weekly allowance', 'Any differences among parents in the standard used to judge an item as frivolous', 'The educational background of the child development specialists who made this observation', 'The average amount of money received by adolescents who receive large weekly allowances']", "label": 1 }, { "id": "train_3256", "context": "Letter to the editor: You say that if the government were to confiscate a portion of the wages of convicted burglars when they reenter the workforce, it would be a form of stealing, hence an abuse of power. Yet under the proposal now being considered, the government would confiscate such wages in order to fund an account to compensate burglary victims. So even if confiscating a portion of burglars' wages were a form of stealing, it would still be justified.", "question": "Which one of the following principles, if valid, most helps to support the argument in the letter to the editor?", "answers": "['Burglars are obligated to provide compensation to the same individuals they victimized.', 'The motive prompting an action determines whether or not that action is justified.', 'A crime is justified only if it is a means of compensating people who deserve compensation.', \"Money stolen from a burglar should be given to that burglar's victims.\"]", "label": 1 }, { "id": "train_3257", "context": "Modern navigation systems, which are found in most of today' s commercial aircraft, are made with low-power circuitry, which is more susceptible to interference than the vacuum-tube circuitry found in older planes. During landing, navigation systems receive radio signals from the airport to guide the plane to the runway. Recently, one plane with low-power circuitry veered off course during landing, its dials dimming, when a passenger turned on a laptop computer. Clearly, modern aircraft navigation systems are being put at risk by the electronic devices that passengers carry on board, such as cassette players and laptop computers.", "question": "Which one of the following, if true, LEAST strengthens the argument above?", "answers": "['No problems with navigational equipment or instrument dials have been reported on flights with no passenger-owned electronic devices on board.', 'After the laptop computer was turned off, the plane regained course and its navigation instruments and dials returned to normal.', 'Planes were first equipped with low-power circuitry at about the same time portable electronic devices became popular.', 'When in use all electronic devices emit electromagnetic radiation, which is known to interfere with circuitry.']", "label": 2 }, { "id": "train_3258", "context": "Economist: Paying extra for fair-trade coffee-coffee labeled with the Fairtrade logo-is intended to help poor farmers, because they receive a higher price for the fair-trade coffee they grow. But this practice may hurt more farmers in developing nations than it helps. By raising average prices for coffee, it encourages more coffee to be produced than consumers want to buy. This lowers prices for non-fair-trade coffee and thus lowers profits for non-fair-trade coffee farmers.", "question": "To evaluate the strength of the economist's argument, it would be most helpful to know which of the following?", "answers": "['Whether many coffee farmers in developing nations also derive income from other kinds of farming', 'What proportion of coffee farmers in developing nations produce fair-trade coffee', 'Whether consumers should pay extra for fair-trade coffee if doing so lowers profits for non-fair-trade coffee farmers', \"Whether there is a way of alleviating the impact of the increased average prices for coffee on non-fair-trade coffee farmers' profits\"]", "label": 1 }, { "id": "train_3259", "context": "Toddlers are not being malicious when they bite people. For example, a child may want a toy, and feel that the person he or she bites is preventing him or her from having it.", "question": "The situation as described above most closely conforms to which one of the following generalizations?", "answers": "['Toddlers do not recognize that by biting people they often thwart their own ends.', 'Biting people is sometimes a way for toddlers to try to solve problems.', 'Toddlers sometimes engage in biting people in order to get attention from adults.', 'Resorting to biting people is in some cases an effective way for toddlers to get what they want.']", "label": 1 }, { "id": "train_3260", "context": "Large discount chains can make a profit even while offering low prices, because they buy goods in large quantities at favorable cost. This creates a problem for small retailers. If they try to retain their customers by lowering prices to match those of large discount chains, the result is a lower profit margin. But small retailers can retain their customer base without lowering prices if they offer exceptional service. Hence, small retailers that are forced to compete with large discount chains must offer exceptional service in order to retain their level of profitability.", "question": "The reasoning is flawed because it fails to take into account the possibility that", "answers": "['not all small retailers are forced to compete with large discount chains', 'exceptional service is not the only reason customers prefer small retail stores', 'not all large discount chains do in fact make a profit', 'small retailers are often motivated by things other than the desire for profit']", "label": 1 }, { "id": "train_3261", "context": "It is virtually certain that the government contract for building the new highway will be awarded to either Phoenix Contracting or Cartwright Company. I have just learned that the government has decided not to award the contract to Cartwright Company. It is ttherefore almost inevitable that Phoenix Contracting will be awarded the contract.", "question": "The argument proceeds by", "answers": "['concluding that it is extremely likely that an event will occur by ruling out the only probable alternative', 'refuting a claim that a particular event is inevitable by establishing the possibility of an alternative event', 'predicting a future event on the basis of an established pattern of past events', 'inferring, from a claim that one of two possible events will occur, that the other event will not occur']", "label": 0 }, { "id": "train_3262", "context": "Allowing more steel imports would depress domestic steel prices and harm domestic steel manufacturers. Since the present government will not do anything that would harm the domestic steel industry, it will not lift restrictions on steel imports.", "question": "The pattern of reasoning in the argument above is most similar to that in which one of the following?", "answers": "['Building construction increases only when people are confident that the economy is doing well. Ttherefore, since people are now confident in the economy we can expect building construction to increase.', \"The chief executive officer of Silicon, Inc. , will probably not accept stock in the company as a bonus next year, since next year's tax laws will require companies to pay a new tax on stock given to executives.\", \"The installation of bright floodlights on campus would render the astronomy department's telescope useless. The astronomy department will not support any proposal that would render its telescope useless; it will ttherefore not support proposals to install bright floodlights on campus.\", 'Since workers are already guaranteed the right to a safe and healthful workplace by law, there is no need for the government to establish further costly health regulations for people who work all day at computer terminals.']", "label": 2 }, { "id": "train_3263", "context": "Advertisement: RediMed is the cold treatment to choose. People who treat their colds with RediMed have symptoms half as severe as those suffered by people who leave their colds untreated.", "question": "The flawed pattern of reasoning in which one of the following is most similar to that in the advertisement above?", "answers": "['When cooking spaghetti, you should add the salt to the water when the water is already boiling, since the spaghetti takes on a bitter flavor otherwise.', 'Tuff-Grip tires are the ones to choose, since in trials by a consumer magazine they were found to be the most durable and the safest.', 'If you are not going to use butter for frying, you should use olive oil, since European chefs use olive oil.', 'The lubricant to select for locks is graphite, since unlubricated locks are stiffer than locks lubricated with graphite and sometimes freeze completely.']", "label": 3 }, { "id": "train_3264", "context": "There are only two plausible views about where the aesthetic value of a painting lies: either in its purely formal qualities or in what the painting means. But there exists no compelling general account of how a painting could derive its value from its purely formal characteristics. Ttherefore, the aesthetic value of a painting lies in what it means.", "question": "The pattern of questionable reasoning in the argument above is most similar to that in which one of the following?", "answers": "['This cardiac patient could be treated with surgery or angioplasty, among other methods. But his weak condition would make recovery from surgery a very long process. Ttherefore, the doctors ought to perform angioplasty.', 'Some analysts are forecasting that if the economy expands, the inflation rate will rise or the unemployment rate will fall. But the unemployment rate promises to remain stable. Ttherefore, the inflation rate will not change either.', 'History is driven primarily by economic forces or primarily by political forces. But no historian has shown convincingly that history is driven mainly by economic forces. Ttherefore, it is driven primarily by political forces.', 'Should the company be outbid on the new project, it will either have to lay off workers or find new business. But it does not expect to find new business in the foreseeable future. Ttherefore, it must be expecting to win the bid on the new project.']", "label": 2 }, { "id": "train_3265", "context": "Changes in Britain' s National Health Service have led many British hospitals to end on-site laundry services for their staff. Although the water in a typical residential washing machine, unlike that in the industrial washing machines used by hospitals, does not reach temperatures high enough to kill the dangerous bacterium Acinetobacter, hospital officials believe that the discontinuation of these services will not put patients at risk.", "question": "Which one of the following, if true, most helps to justify the hospital officials' belief?", "answers": "['Hospital patients infected with Acinetobacter can be isolated from other patients.', 'Water in residential washing machines reaches temperatures high enough to kill all dangerous bacteria other than Acinetobacter.', 'Most hospital staff made use of the on-site laundry services before they were discontinued.', 'Hospital staff are instructed to use clothes dryers at temperatures high enough to kill Acinetobacter.']", "label": 3 }, { "id": "train_3266", "context": "Close friends and family in attendance and a good caterer will ensure a successful wedding reception. Since Liz and John have all of their close friends and family in attendance and a caterer, their wedding reception is a success.", "question": "The pattern of flawed reasoning exhibited by the argument above is most similar to that exhibited by which one of the following?", "answers": "[\"If a shirt is made of quality material and is unwrinkled, it will always be in style. Since Larry's shirt is made of quality material, his shirt will always be in style.\", 'The right mixture of red and blue paint, properly mixed and poured will always produce the perfect shade of purple. Since the artist has properly mixed and poured the right mixture of red and blue paint, she will have the perfect shade of purple.', 'Putting stain remover on a stain and care- fully soaking the garment in cold water before washing it will ensure that the stain does not set. Marie carefully soaked the garment in cold water before washing it, so since she put stain remover on the stain before washing it, the stain is certain not to have set.', 'Onions, carefully browned and served on hamburgers, always produces a delicious entree. Since Renee has made a hamburger, but has not carefully browned onions, her hamburger will not be a delicious entree.']", "label": 0 }, { "id": "train_3267", "context": "People who have spent a lot of time in contact with animals often develop animal-induced allergies, some of them quite serious. In a survey of current employees in major zoos, about 30 percent had animal-induced allergies. Based on this sample, experts conclude that among members of the general population who have spent a similarly large amount of time in close contact with animals, the percentage with animal-induced allergies is not 30 percent but substantially more.", "question": "Which of the following, if true, provides the strongest grounds for the experts' conclusion?", "answers": "['A zoo employee who develops a serious animal-induced allergy is very likely to switch to some other occupation.', 'The percentage of the general population whose level of exposure to animals matches that of a zoo employee is quite small.', 'Zoo employees seldom wear protective gear when they handle animals in their care.', 'A zoo employee is more likely than a person in the general population to keep one or more animal pets at home']", "label": 0 }, { "id": "train_3268", "context": "The town of Greenfield recently instituted a substantial supplementary tax on all households, whereby each household is taxed in proportion to the volume of the trash that it puts out for trash collectors to pick up, as measured by the number of standard-sized garbage bags put out. In order to reduce the volume of the trash on which their tax bill is based, Greenfield households can deliver their recyclable trash to a conveniently located local commercial recycling center, where such trash is accepted free of charge.", "question": "The supplementary tax provides some financial incentive to Greenfield households to do each of the following EXCEPT", "answers": "['compress and nest items of nonrecyclable trash before putting them out for pickup', 'dump nonrecyclable trash illegally at parks and roadsides', 'deliver recyclable materials to the recycling center instead of passing them on to neighbors who want to reuse them', 'sort out recyclable trash thoroughly from their other trash']", "label": 2 }, { "id": "train_3269", "context": "Editorial: When legislators discover that some public service is not being adequately provided, their most common response is to boost the funding for that public service. Because of this, the least efficiently run government bureaucracies are the ones that most commonly receive an increase in funds.", "question": "The statements in the editorial, if true, most strongly support which one of the following?", "answers": "['When legislators discover that a public service is not being adequately provided, they never respond to the problem by reducing the funding of the government bureaucracy providing that service.', 'The least efficiently run government bureaucracies are the bureaucracies that legislators most commonly discover to be failing to provide some public service adequately.', 'The most inefficiently run government bureaucracy receives the most funding of any government bureaucracy.', 'Throughout the time a government bureaucracy is run inefficiently, legislators repeatedly boost the funding for the public service that this bureaucracy provides.']", "label": 1 }, { "id": "train_3270", "context": "Telomerase is an enzyme that is produced only in cells that are actively dividing. For this reason it is generally absent from body tissues in adults. Bone marrow is an exception to this rule, however, since even in adults, bone marrow cells continually divide to replace old blood cells. Cancers are another exception, because their cells are rapidly dividing.", "question": "The information provided most strongly supports which of the following?", "answers": "['The presence of telomerase in bone marrow is no indication of bone marrow cancer.', 'In children, the only body tissues from which telomerase is absent are those in which cells are not rapidly dividing.', 'Cancer of the bone marrow develops more rapidly than cancer growing in any other kind of adult tissue.', 'The level of telomerase production is always higher in cancerous tissue than in noncancerous tissue.']", "label": 0 }, { "id": "train_3271", "context": "Psychiatrist: While the first appearance of a phobia is usually preceded by a traumatizing event, not everyone who is traumatized by an event develops a phobia. Furthermore, many people with phobias have never been traumatized. These two considerations show that traumatizing events do not contribute to the occurrence of phobias.", "question": "The reasoning in the psychiatrist's argument is most vulnerable to criticism on the grounds that the argument", "answers": "['derives a causal connection from mere association when there is no independent evidence of causal connection', 'presumes, without providing justification, that some psychological events have no causes that can be established by scientific investigation', 'takes for granted that a type of phenomenon contributes to the occurrence of another type of phenomenon only if phenomena of these two types are invariably associated', 'builds the conclusion drawn into the support cited for that conclusion']", "label": 2 }, { "id": "train_3272", "context": "Journal editor: Our treasurer advises that because of our precarious financial situation, we should change from a paper version to an online version only if doing so will not increase the cost of publication. The cost of converting from a paper version to an online version is high; however, once the conversion is made, the cost per issue is much lower for an online version. Since a benefactor has agreed to cover the costs of conversion, and since we can safely assume that our treasurer is right, we should change to an online version.", "question": "The journal editor's argument is flawed in that it", "answers": "[\"bases its conclusion on the argument of an authority speaking outside the authority's field of expertise\", 'fails to rule out the possibility that the journal will remain in a precarious financial position whether it changes to an online version or not', 'overlooks the possibility that an online version would have other advantages over a paper version than cost', 'treats meeting a necessary condition for changing from a paper to an online version as a sufficient reason for changing']", "label": 3 }, { "id": "train_3273", "context": "Problems caused by the leaching of pollutants from dumps and landfills are worst in countries with an annual per capita economic output of $4, 000 to $5, 000, and less severe for considerably poorer and considerably richer countries. This is so because pollution problems increase during the early stages of a country' s industrial development but then diminish as increasing industrial development generates adequate resources to tackle such problems. Ttherefore, problems caused by such leaching in Country X, where the annual per capita economic output is now $5, 000, should begin to diminish in the next few years.", "question": "Which one of the following is an assumption on which the argument depends?", "answers": "['Industrial development in Country X will increase in the next few years.', 'Countries surrounding Country X will reduce the amount of pollution that their factories release into the air and water.', 'No other country with a similar amount of industrial development has pollution problems that are as severe as those in Country X.', 'Within the next few years, Country X will impose a system of fines for illegal waste disposal by its industrial companies.']", "label": 0 }, { "id": "train_3274", "context": "In marketing their products, drug companies often send gifts to physicians. According to a recent survey, most physicians believe that their own choices when prescribing drugs are not influenced by drug companies' gifts. The same survey indicates that the majority of physicians believe that most other physicians' prescription choices are influenced by such gifts.", "question": "If the survey results are accurate, which one of the following must be true?", "answers": "['Physicians who do not accept gifts from drug companies are less likely to prescribe unnecessary drugs than those who do accept such gifts.', \"All physicians who admit that their own choices when prescribing drugs are influenced by drug companies' gifts believe that most other physicians' prescription choices are also influenced by such gifts.\", 'Most physicians believe that drug companies should adopt new guidelines that regulate their practices in sending gifts to physicians.', 'Some physicians are mistaken either about the degree to which they are influenced by gifts from drug companies or about the degree to which such gifts influence other physicians.']", "label": 3 }, { "id": "train_3275", "context": "Physician: There were approximately 83, 400 trampoline-related injuries last year. This suggests that trampolines are quite dangerous and should ttherefore be used only under professional supervision. Trampoline enthusiast: I disagree. In the past ten years sales of home trampolines have increased much more than trampoline-related injuries have: 260 percent in sales compared with 154 percent in injuries. Every exercise activity carries risks, even when carried out under professional supervision.", "question": "The dialogue provides the most support for the claim that the physician and the trampoline enthusiast disagree over whether", "answers": "['trampoline use is an activity that warrants mandatory professional supervision', 'trampolines cause injuries to a significant number of people using them', 'home trampolines are the main source of trampoline-related injuries', 'the rate of trampoline-related injuries, in terms of the number of injuries per trampoline user, is declining']", "label": 0 }, { "id": "train_3276", "context": "A recent study shows that those highways that carry the most traffic, and thus tend to be the most congested, have the lowest rate of fatal traffic accidents.", "question": "Which one of the following, if true, most helps to explain the phenomenon described above?", "answers": "['Most of the motorists on very heavily traveled highways tend to be commuting to or from work.', 'Most serious accidents occur when vehicles are moving at a high rate of speed.', 'Drivers have more accidents when they become distracted.', 'The highways that have the highest rate of fatal accidents have moderate volumes of traffic.']", "label": 1 }, { "id": "train_3277", "context": "Researchers took a group of teenagers who had never smoked and for one year tracked whether they took up smoking and how their mental health changed. Those who began smoking within a month of the study' s start were four times as likely to be depressed at the study' s end as those who did not begin smoking. Since nicotine in cigarettes changes brain chemistry, perhaps thereby affecting mood, it is likely that smoking contributes to depression in teenagers.", "question": "Which of the following, if true, most strengthens the argument?", "answers": "[\"Participants who were depressed at the study's start were no more likely to be smokers at the study's end than those who were not depressed.\", \"Participants who began smoking within a month of the study's start were no more likely than those who began midway through to have quit smoking by the study's end.\", 'Some participants entered and emerged from a period of depression within the year of the study.', 'Few, if any, of the participants in the study were friends or relatives of other participants.']", "label": 0 }, { "id": "train_3278", "context": "Pauline: Some environmentalists claim that for the salmon to be saved, the hydroelectric dams on the river must be breached. But if the dams are breached, given the region' s growing population and booming industry, electrical costs will skyrocket. Roger: The dams are already producing electricity at optimal capacity. So regardless of whether they are breached, we will have to find additional energy sources for the region.", "question": "The dialogue provides the most support for the claim that Pauline and Roger agree that", "answers": "['there will be no significant decrease in demand for electricity in the region in the near future', 'if the dams remain in service but do not operate at optimal capacity, electrical costs in the region will rise', 'finding additional energy sources will not decrease the electrical costs in the region', 'some environmentalists who advocate saving the salmon believe that that goal overrides concerns about electrical costs']", "label": 0 }, { "id": "train_3279", "context": "Brain-scanning technology provides information about processes occurring in the brain. For this information to help researchers understand how the brain enables us to think, however, researchers must be able to rely on the accuracy of the verbal reports given by subjects while their brains are being scanned. Otherwise brain-scan data gathered at a given moment might not contain information about what the subject reports thinking about at that moment, but instead about some different set of thoughts.", "question": "Which one of the following most accurately expresses the main conclusion of the argument?", "answers": "['It is unlikely that brain-scanning technology will ever enable researchers to understand how the brain enables us to think.', 'Information from brain scans can help researchers understand how the brain enables us to think only if the verbal reports of those whose brains are being scanned are accurate.', 'Because subjects whose brains are being scanned may not accurately report what they are thinking, the results of brain-scanning research should be regarded with great skepticism.', 'There is no way that researchers can know for certain that subjects whose brains are being scanned are accurately reporting what they are thinking.']", "label": 1 }, { "id": "train_3280", "context": "There is no genuinely altruistic behavior. Everyone needs to have sufficient amount of self-esteem, which crucially depends on believing oneself to be useful and needed. Behavior that appears to be altruistic can be understood as being motivated by the desire to reinforce that belief, a clearly self-interested motivation.", "question": "A flaw in the argument is that it", "answers": "['presumes, without providing justification, that if one does not hold oneself in sufficient self-esteem one cannot be useful or needed', \"fails to consider that self-esteem also depends on maintaining an awareness of one's own value\", 'takes for granted that any behavior that can be interpreted as self-interested is in fact self-interested', 'illicitly infers that behavior is altruistic merely because it seems altruistic']", "label": 2 }, { "id": "train_3281", "context": "Many leadership theories have provided evidence that leaders affect group success rather than the success of particular individuals. So it is irrelevant to analyze the effects of supervisor traits on the attitudes of individuals whom they supervise. Instead, assessment of leadership effectiveness should occur only at the group level.", "question": "Which of the following would it be most useful to establish in order to evaluate the argument?", "answers": "['Whether some types of groups do not need supervision in order to be successful in their endeavors', \"Whether individuals' attitudes toward supervisors affect group success\", \"Whether any of the leadership theories in question hold that leaders should assess other leaders' attitudes\", \"Whether it is possible to assess individual supervisees' attitudes toward their supervisors without thereby changing those attitudes\"]", "label": 1 }, { "id": "train_3282", "context": "Maize contains the vitamin niacin, but not in a form the body can absorb. Pellagra is a disease that results from niacin deficiency. When maize was introduced into southern Europe from the Americas in the eighteenth century, it quickly became a dietary staple, and many Europeans who came to subsist primarily on maize developed pellagra. Pellagra was virtually unknown at that time in the Americas, however, even among people who subsisted primarily on maize.", "question": "Which of the following, if true, most helps to explain the contrasting incidence of pellagra described above?", "answers": "['In southern Europe many of the people who consumed maize also ate niacin-rich foods.', 'Once introduced into southern Europe, maize became popular with landowners because of its high yields relative to other cereal crops.', \"Before the discovery of pellagra's link with niacin, it was widely believed that the disease was an infection that could be transmitted from person to person.\", \"Traditional ways of preparing maize in the Americas convert maize's niacin into a nutritionally useful form.\"]", "label": 3 }, { "id": "train_3283", "context": "Advertisement: Northwoods Maple Syrup, made the old-fashioned way, is simply tops for taste. And here is the proof: in a recent market survey, 7 out of every 10 shoppers who expressed a preference said that Northwoods was the only maple syrup for them, no ifs, ands, or buts.", "question": "Of the following, which one is the strongest reason why the advertisement is potentially misleading?", "answers": "['The proportion of shoppers expressing no preference might have been very small.', 'The preference for the Northwoods brand might be based on such a factor as an exceptionally low price.', 'Other brands of maple syrup might also be made the old-fashioned way.', 'No market survey covers more than a sizable minority of the total population of consumers.']", "label": 1 }, { "id": "train_3284", "context": "Medical ethicist: Assuming there is a reasonable chance for a cure, it is acceptable to offer experimental treatments for a disease to patients who suffer from extreme symptoms of that disease. Such patients are best able to weigh a treatment' s risks against the benefits of a cure. Ttherefore, it is never acceptable to offer experimental treatments to patients who experience no extreme symptoms of the relevant disease.", "question": "The flawed reasoning in which one of the following is most similar to the flawed reasoning in the medical ethicist's argument?", "answers": "['One is always in a better position to judge whether an automobile would be worth its cost if one has test-driven that automobile. Ttherefore, if an automobile proves to be not worth its cost, it is likely that it was not test-driven.', 'Someone born and raised in a country, who has lived abroad and then returned, is exceptionally qualified to judge the merits of living in that country. That is why someone who has not lived in that country should not form judgments about the merits of living there.', 'Almost any industrial development will have unwelcome environmental side effects. Ttherefore, it is not worthwhile to weigh the costs of potential environmental side effects since such side effects are unavoidable.', 'Even a geological engineer with a background in economics can lose money investing in mineral extraction. So, those who are less knowledgeable about geology or economics should not expect to make money in every investment in mineral extraction.']", "label": 1 }, { "id": "train_3285", "context": "Critics of a new law that will allow extra-long passenger trains to travel on the state' s railroads are mistaken in claiming that these extra-long passenger trains are more dangerous than other types of passenger trains. In our neighboring state P, where extra-long passenger trains have been permitted, these extra-long passenger trains have resulted in fewer fatalities per mile than other types of passenger trains used in our state. Clearly, extra-long passenger trains are safer than other types of passenger trains.", "question": "Which one of the following, if true, most seriously weakens the argument above?", "answers": "['Fatalities involving all trains in state P are fewer due to lower speed limits and stricter training requirements for train conductors.', 'The new law would require the extra-long passenger trains to be equipped with special brakes that would allow them to stop more quickly than other trains.', 'Extra-long passenger trains reduce ticket prices by allowing the train to carry more passengers each trip.', \"Critics of the new law are unaware that state P has allowed extra-long passenger trains to travel on state P's railroads.\"]", "label": 0 }, { "id": "train_3286", "context": "Wolves generally avoid human settlements. For this reason, domestic sheep, though essentially easy prey for wolves, are not usually attacked by them. In Hylantia prior to 1910, farmers nevertheless lost considerable numbers of sheep to wolves each year. Attributing this to the large number for wolves, in 1910 the government began offering rewards to hunters for killing wolves. From 1910 to 1915, large numbers of wolves were killed. Yet wolf attacks on sheep increased significantly.", "question": "Which of the following, if true, most helps to explain the increase in wolf attacks on sheep?", "answers": "['Populations of deer and other wild animals that wolves typically prey on increased significantly in numbers from 1910 to 1915.', 'The systematic hunting of wolves encouraged by the program drove many wolves in Hylantia to migrate to remote mountain areas uninhabited by humans.', 'Prior to 1910, there were no legal restrictions in Hylantia on the hunting of wolves.', \"After 1910 hunters shot and wounded a substantial number of wolves, thereby greatly diminishing these wolves' ability to prey on wild animals.\"]", "label": 3 }, { "id": "train_3287", "context": "Lawyer: A body of circumstantial evidence is like a rope, and each item of evidence is like a strand of that rope. Just as additional pieces of circumstantial evidence strengthen the body of evidence, adding strands to the rope strengthens the rope. And if one strand breaks, the rope is not broken nor is its strength much diminished. Thus, even if a few items of a body of circumstantial evidence are discredited, the overall body of evidence retains its basic strength.", "question": "The reasoning in the lawyer's argument is most vulnerable to criticism on the grounds that the argument", "answers": "['takes for granted that no items in a body of circumstantial evidence are significantly more critical to the strength of the evidence than other items in that body', 'fails to consider the possibility that if many items in a body of circumstantial evidence were discredited, the overall body of evidence would be discredited', 'presumes, without providing justification, that the strength of a body of evidence is less than the sum of the strengths of the parts of that body', 'offers an analogy in support of a conclusion without indicating whether the two types of things compared share any similarities']", "label": 0 }, { "id": "train_3288", "context": "Judge: The defendant admits noncompliance with national building codes but asks that penalties not be imposed because he was confused as to whether national or local building codes applied to the area in which he was building. This excuse might be acceptable had he been charged with noncompliance with local codes, but since he is charged with noncompliance with national codes, his excuse is unacceptable.", "question": "Which one of the following principles, if valid, most helps to justify the judge's reasoning?", "answers": "['Local codes may be less strict, but not more strict, than national codes.', 'Local codes and national codes must not overlap with each other.', 'Any behavior required by national codes is also required by local codes.', 'A behavior that is in compliance with one law is not necessarily in compliance with another.']", "label": 2 }, { "id": "train_3289", "context": "Loggerhead turtles are an endangered species. Aquarium officials presumably know and are concerned about the declining number of wild loggerheads. Nevertheless, aquariums keep loggerheads and display them. These turtles are being kept in captivity and are thus prevented from adding to the population of wild turtles.", "question": "Each of the following, if true, contributes to an explanation of why aquarium officials keep loggerheads in captivity EXCEPT:", "answers": "['The population of loggerheads in captivity has declined slightly over the last 20 years.', 'The captive loggerheads are bred, and their offspring are released into the wild, which helps increase the number of wild turtles.', 'The captive loggerheads are used to help educate the public about the needs of wild loggerheads so that the public will take greater care not to harm them.', 'The baby loggerheads in captivity are hatchlings too weak to survive in the wild.']", "label": 0 }, { "id": "train_3290", "context": "A purse containing 32 ancient gold coins that had been minted in Morocco was discovered in the ruins of an ancient Jordanian city some 4, 000 kilometers to the east of Morocco. In its time the Jordanian city was an important trading center along the trade route linking China and Europe, and it was also a popular stopover for pilgrims on the route between Morocco and Mecca. The purse of a trader in the city would probably have contained a more diverse set of coins.", "question": "The statements above, if true, most strongly support which one of the following hypotheses?", "answers": "['Most gold coins available during the time when the ancient city thrived were minted in Morocco.', 'Pilgrims and traders in the ancient city were unlikely to have interacted with one another.', 'Moroccan coins were more valuable in the ancient city than were Jordanian coins.', 'The purse with the gold coins had been brought to the ancient city by a pilgrim on the route between Morocco and Mecca.']", "label": 3 }, { "id": "train_3291", "context": "Public policy dictates the health risks the public routinely takes. Statistical arguments about health risks are used primarily to deflect public fears, while contributing little to policy debate. For example, statistics are cited to imply that wearing a seat belt reduces one' s risk of death in an automobile accident, deflecting attention from the fact that a transportation policy that promotes increasing use of automobiles inherently increases any individual' s risk of death in an automobile accident.", "question": "The way the example functions above is most closely paralleled in which one of the following?", "answers": "['Statistics indicate that the number of people dependent on alcohol far exceeds the number dependent on illegal addictive drugs; thus, any policy for the control and treatment of substance abuse should provide for treatment of alcoholism.', \"Statistics indicate that an individual's risk of contracting cancer from radiation emitted by a nuclear power plant is less than that of contracting cancer from sunshine. These statistics draw attention away from the fact that a policy of energy conservation is safer for human health than a policy based on nuclear energy.\", 'Statistics indicate that the average life expectancy of males is shorter than that of females. When one accounts for the fact that females smoke less and are less likely to work in jobs in the chemical and manufacturing industries, the difference in life expectancy is narrowed.', \"Statistics indicate that an urban resident's risk of accidental death from any cause is no greater than that of an individual who lives in a suburban or rural area. These statistics counter the widely held public belief that urban areas are more dangerous than suburban or rural areas.\"]", "label": 1 }, { "id": "train_3292", "context": "Animals with a certain behavioral disorder have unusually high levels of aluminum in their brain tissue. Since a silicon-based compound binds to aluminum and prevents it from affecting the brain tissue, animals can be cured of the disorder by being treated with the compound.", "question": "The argument is based on which one of the following assumptions?", "answers": "['Aluminum is the cause of the disorder rather than merely an effect of it.', 'Aluminum is never present in normal brain tissue.', 'Introducing the compound into the brain tissue has no side effects.', 'Animals with the disorder have unusually high but invariable levels of aluminum in their brain tissue.']", "label": 0 }, { "id": "train_3293", "context": "The number of codfish in the North Atlantic has declined substantially as the population of harp seals has increased from two million to more than three million. Some blame the seal for the shrinking cod population, but cod plays a negligible role in the seal' s diet. It is ttherefore unlikely that the increase in the seal population has contributed significantly to the decline in the cod population.", "question": "Which one of the following, if true, most seriously weakens the argument?", "answers": "['The harp seal thrives in water that is too cold to support a dense population of cod.', \"Cod feed almost exclusively on capelin, a fish that is a staple of the harp seal's diet.\", 'Water pollution poses a more serious threat to cod than to the harp seal.', 'People who fish for cod commercially are inconvenienced by the presence of large numbers of seals near traditional fishing grounds.']", "label": 1 }, { "id": "train_3294", "context": "A study compared a sample of Swedish people older than 75 who needed in-home assistance with a similar sample of Israel people. The people in the two samples received both informal assistance, provided by family and friends, and formal assistance, professionally provided. Although Sweden and Israel have equally well-funded and comprehensive systems for providing formal assistance, the study found that the people in the Swedish sample received more formal assistance, on average, than those in the Israeli sample.", "question": "Which of the following, if true, does most to explain the difference that the study?", "answers": "['More Swedish than Israeli people older than 75 live in rural areas where formal assistance services are sparse or nonexistent.', 'In recent decades, the increase in life expectancy of someone who is 75 years old has been greater in Israel than in Sweden.', 'In Israel, people older than 75 tend to live with their children, whereas in Sweden people of that age tend to live alone.', 'A companion study found that among children needing special in-home care , the amount of formal assistance they received was roughly the same in Sweden as in Israel.']", "label": 2 }, { "id": "train_3295", "context": "Health association: In certain studies, most volunteers who used to eat meat have permanently excluded meat from their diets and now eat only vegetables and fruits. These volunteers suffered no ill effects and many even prefer the new regimen. So people who eat meat can change their diets to meatless ones and not suffer as a result. Critic: Participants in these studies were all favorably disposed to the substitution before they started, and even so, many of them failed to make a complete transition to a vegetarian diet.", "question": "The critic's response would most seriously call into question which one of the following conclusions, if that conclusion were drawn on the basis of the evidence given by the health association?", "answers": "['Participants in the study who failed to make the transition to a vegetarian diet do not regret the attempt.', 'The diets of most people who eat meat would be improved if those people ate only vegetables and fruits instead.', 'The number of people who have adopted a strictly vegetarian diet has increased in the last few years.', 'Most people, if told by their physicians to exclude meat from their diets and eat only vegetables and fruits, would succeed in doing so.']", "label": 3 }, { "id": "train_3296", "context": "Wart-Gone cream is a well-known cure for reducing the spread of painful warts. A dime size amount rubbed on the infected area daily is the prescribed dosage, and applying the cream more often does not enhance the cream' s healing effects. Hence, distributing coupons giving $3. 00 off the price of Wart-Gone will not boost sales and, ttherefore, will not increase profits either.", "question": "Which one of the following statements, if true, would most strengthen the argument?", "answers": "['The $3. 00 coupons, if distributed, would not induce new customers, who might not have purchased the product otherwise, to try Wart-Gone.', 'Wart-Gone has proven to be more effective on certain types of warts than others.', 'Warts, though all caused by a virus, differ greatly in shape and appearance.', 'Wart-Gone can be produced in small or large batches without appreciably lowering the pre-tube production costs.']", "label": 0 }, { "id": "train_3297", "context": "Urban planner: When a city loses population due to migration, property taxes in that city tend to rise. This is because there are then fewer residents paying to maintain an infrastructure that was designed to support more people. Rising property taxes, in turn, drive more residents away, compounding the problem. Since the city of Stonebridge is starting to lose population, the city government should ttherefore refrain from raising property taxes.", "question": "Which of the following, if true, would most weaken the urban planner's argument?", "answers": "['Demographers project that the population of a region that includes Stonebridge will start to increase substantially within the next several years.', 'Stonebridge at present benefits from grants provided by the national government to help maintain certain parts of its infrastructure.', 'If Stonebridge does not raise taxes on its residents to maintain its infrastructure, the city will become much less attractive to live in as that infrastructure decays.', 'If there is a small increase in property taxes in Stonebridge and a slightly larger proportion of total revenue than at present is allocated to infrastructure maintenance, the funding will be adequate for that purpose.']", "label": 2 }, { "id": "train_3298", "context": "One test to determine whether a person has been infected with tuberculosis consists of injecting the person with proteins extracted from the tuberculosis bacterium. Once a person has been infected by a bacterium, the person' s immune system subsequently recognizes certain proteins present in that bacterium and attacks the bacterium. This recognition also takes place in the test and results in a skin irritation at the injection site. Hence the physicians who designed the test reasoned that anyone who reacts in this manner to an injection with the tuberculosis proteins has been infected with tuberculosis.", "question": "Which one of the following is an assumption on which the physicians' reasoning depends?", "answers": "['The ability of the proteins present in the tuberculosis bacterium to trigger the skin irritation is exclusive to that bacterium.', \"All of the proteins present in disease-causing bacteria can be recognized by the body's immune system.\", 'Localized skin irritations are a characteristic symptom of tuberculosis in most people.', 'Some people who have been injected with proteins extracted from the tuberculosis bacterium will contract tuberculosis as a result of the injection.']", "label": 0 }, { "id": "train_3299", "context": "A report on the likely effects of current levels of air pollution on forest growth in North America concluded that, since nitrogen is a necessary nutrient for optimal plant growth, the nitrogen deposited on forest soil as a result of air pollution probably benefits eastern forests. However, European soil scientists have found that in forests saturated with sulfate and nitrate, trees begin to die when the nitrogen deposited exceeds the amount of nitrogen absorbed by the forest system. Since this finding is likely to apply to forests everywhere, large areas of eastern forests of North America are, undoubtedly, already being affected adversely.", "question": "Which one of the following most accurately expresses the main point of the passage?", "answers": "['Contrary to the report cited, the nitrogen pollution now in the air is more likely to cause trees to die in eastern forests than to benefit them.', 'The implication of the report cited is that the amount of nitrogen reaching eastern forests by way of polluted air is approximately what those forests need for optimal growth.', 'The eastern forests are the only forests of North America currently affected by polluted air.', 'The type of analysis used by European soil scientists does not necessarily apply to eastern forests of North America.']", "label": 0 }, { "id": "train_3300", "context": "More and more computer programs that provide solutions to mathematical problems in engineering are being produced, and it is thus increasingly unnecessary for practicing engineers to have a thorough understanding of fundamental mathematical principles. Consequently, in training engineers who will work in industry, less emphasis should be placed on mathematical principles, so that space in the engineering curriculum will be available for other important subjects.", "question": "Which one of the following, if true, most seriously weakens the argument given for the recommendation above?", "answers": "['The engineering curriculum already requires that engineering students be familiar with and able to use a variety of computer programs.', 'Development of composites and other such new materials has meant that the curriculum for engineers who will work in industry must allow time for teaching the properties of these materials.', 'Most of the computer programs that provide solutions to mathematical problems in engineering can be run on the types of computers available to most engineering firms.', 'The effective use of computer programs that provide solutions to mathematical problems in engineering requires an understanding of mathematical principles.']", "label": 3 }, { "id": "train_3301", "context": "Farmer: A report has just been released that criticizes our corn, alleging that the genetically-modified seed we use is harmful to consumers. However, the report was commissioned by our main competition -- a large-scale corporate farm. The conflict of interest is so obvious that consumers can continue to eat our corn without worry.", "question": "Which one of the following best explains why the farmer's argument is vulnerable to criticism?", "answers": "[\"The farmer fails to elaborate on the corporate farm's conflict of interest.\", 'The farmer draws a conclusion without considering alternative motivations for the commissioning the report by the corporate farm.', \"The farmer wrongly assumes that a conflict of interest automatically negates the validity of the report's conclusion.\", 'The farmer does not provide any evidence as to why their corn is safe for consumption.']", "label": 2 }, { "id": "train_3302", "context": "In the decade from the mid-1980s to the mid-1990s, large corporations were rocked by mergers, re-engineering, and downsizing. These events significantly undermined employees' job security. Surprisingly, however, employees' perception of their own job security hardly changed over that period. Fifty-eight percent of employees surveyed in 1984 and 55 percent surveyed in 1994 stated that their own jobs were very secure.", "question": "Each of the following contributes to an explanation of the surprising survey results described above EXCEPT:", "answers": "['The corporate downsizing that took place during this period had been widely anticipated for several years before the mid-1980s.', 'A large number of the people in both surveys work in small companies that were not affected by mergers, re-engineering, and downsizing.', 'In the mid-1 990s, people were generally more optimistic about their lives, even in the face of hardship, than they were a decade before.', 'Employees who feel secure in their jobs tend to think that the jobs of others are secure.']", "label": 3 }, { "id": "train_3303", "context": "Board member: The J Foundation, a philanthropic organization, gave you this grant on the condition that your resulting work not contain any material detrimental to the J Foundation' s reputation. But your resulting work never mentions any of the laudable achievements of our foundation. Hence your work fails to meet the conditions under which the grant was made.", "question": "The reasoning in the board member's argument is vulnerable to criticism on the grounds that the argument", "answers": "[\"presumes, without providing justification, that a work that does not mention a foundation's laudable achievements is harmful to that foundation's reputation\", \"fails to consider the possibility that the work that was produced with the aid of the grant may have met all conditions other than avoiding detriment to the J Foundation's reputation\", 'takes for granted that a work that never mentions any laudable achievements cannot be of high intellectual value', \"fails to consider that recipients of a grant usually strive to meet a foundation's conditions\"]", "label": 0 }, { "id": "train_3304", "context": "Hospital Administrator: The majority of HealthWize Hospital' s income currently comes from three main insurance companies. In order to diversify its sources of income, the hospital will develop its outpatient services to provide clients with the opportunity to visit a nutritionist, a gym, and/or a masseuse on the hospital premises. Critics have protested that, at other hospitals, such plans have not worked because the funds raised through insurance payments far outweigh the money that will be brought in through these additional services. Nevertheless, this initiative will not result in a reduction of the hospital' s income. Major service providers, including BeWell Nutrition and Fitness, have committed to moving their main branches to HealthWize Hospital, which will definitely prove to be an attraction for people from neighboring areas, making this a more popular venture than other hospitals' .", "question": "In the argument given, the two portions in boldface play which of the following roles?", "answers": "[\"The first presents a plan, one of whose consequences is debated in the argument; the second is the speaker's conclusion about this consequence.\", 'The first presents a plan that has significantly more disadvantages than advantages; the second describes a benefit of this plan.', 'The first presents a plan that the argument attempts to defend against criticism; the second is the criticism of the plan itself.', \"The first presents a plan to reorganize the hospital's finances; the second is evidence in support of this plan.\"]", "label": 0 }, { "id": "train_3305", "context": "The great medieval universities had no administrators, yet they endured for centuries. Our university has a huge administrative staff, and we are in serious financial difficulties. Ttherefore, we should abolish the positions and salaries of the administrators to ensure the longevity of the university.", "question": "Which one of the following arguments contains flawed reasoning that most closely parallels the flawed reasoning in the argument above?", "answers": "[\"A bicycle should not be ridden without a helmet. Since a good helmet can save the rider's life, a helmet should be considered the most important piece of bicycling equipment.\", 'No airplane had jet engines before 1940, yet airplanes had been flying since 1903. Ttherefore, jet engines are not necessary for the operation of airplanes.', 'The great cities of the ancient world were mostly built along waterways. Archaeologists searching for the remains of suchs cities should ttherefore try to determine where major rivers used to run.', \"The novelist's stories began to be accepted for publication soon after she started using a computer to write them. You have been having trouble getting your stories accepted for publication, and you do not use a computer. To make sure your stories are accepted for publication, then, you should write them with the aid of a computer.\"]", "label": 3 }, { "id": "train_3306", "context": "Springfield Fire Commissioner: the vast majority of false fire alarms are prank calls made anonymously from fire alarm boxes on street corners. Since virtually everyone has access to a private telephone, these alarm boxes have outlived their usefulness. Ttherefore, we propose to remove the boxes. Removing the boxes will reduce the number of prank calls without hampering people' s ability to report a fire.", "question": "Which of the following, if true, most strongly supports the claim that the proposal, if carried out, will have the announced effect?", "answers": "['The fire department traces all alarm calls made from private telephones and records where they came from.', 'A telephone call can provide the fire department with more information about the nature and size of a fire than can an alarm placed from an alarm box.', 'Maintaining the fire alarm boxes costs Springfield approximately $5 million annually.', 'On any given day, a significant percentage of the public telephones in Springfield are out of service.']", "label": 0 }, { "id": "train_3307", "context": "The hospital at university X has one of the top-rated cancer treatment centers in the nation. In fact, the hospital at university X has earned a top five rating from every cancer association in the country. Dr. Gould is a physician who exclusively specializes in psychiatry, so Dr. Gould obviously cannot work at the hospital at university X.", "question": "The reasoning in the argument is flawed because the argument", "answers": "['fails to see that physicians can practice medicine outside a hospital setting', 'finds one unrepresentative sample of a class and concludes that this unrepresentative sample is, in fact, representative', 'assumes that members of a class have each individually earned the success of the group', 'maintains that an entity is not part of some class on the basis that that entity does not share a certain attribute of the class as a whole']", "label": 3 }, { "id": "train_3308", "context": "Auditor: XYZ, a construction company, purchased 20 new trucks 3 years ago, and there is no record of any of those trucks being sold last year. Records indicate, however, that XYZ sold off all of its diesel-powered trucks last year. We can thus conclude that none of the 20 trucks purchased 3 years ago were diesel powered.", "question": "Which one of the following is an assumption required by the auditor's reasoning?", "answers": "['All of the trucks that XYZ sold last year were diesel powered.', 'None of the 20 trucks was sold before last year.', 'XYZ did not purchase any used trucks 3 years ago.', 'XYZ no longer owns any trucks that it purchased more than 3 years ago.']", "label": 1 }, { "id": "train_3309", "context": "A certain retailer promotes merchandise by using the following policy: At all times there is either a \"manager' s sale\" or a \"holiday sale\" or both going on. All sales are run for exactly one calendar month. In any given month, if a manager wishes to clear out a particular line of merchandise, then a manager' s sale is declared. If a holiday falls within the calendar month and there is excess merchandise in the warehouse, then a holiday sale is declared. However, there is no holiday that falls within the month of August and, in that month, the warehouse never contains excess merchandise.", "question": "Which one of the following can be concluded from the passage?", "answers": "[\"If there is not a manager's sale being run in some month, then there is a holiday sale being run in that month.\", 'If a holiday sale is not being run, then it is the month of August.', 'If there is no excess merchandise in the warehouse, then it is the month of August.', \"If a manager's sale is being run in some month, then there is no excess merchandise in the warehouse in that month.\"]", "label": 0 }, { "id": "train_3310", "context": "Editorialist: To ensure justice in the legal system, citizens must be capable of criticizing anyone involved in determining the punishment of criminals. But when the legal system' s purpose is seen as deterrence, the system falls into the hands of experts whose specialty is to assess how potential lawbreakers are affected by the system' s punishments. Because most citizens lack knowledge about such matters, justice is not then ensured in the legal system.", "question": "The editorialist's argument requires assuming which one of the following?", "answers": "['In order to be just, a legal system must consider the effect that punishment will have on individual criminals.', \"Citizens without knowledge about how the legal system's punishments affect potential lawbreakers are incapable of criticizing experts in that area.\", 'The primary concern in a legal system is to administer punishments that are just.', 'Most citizens view justice as primarily concerned with the assignment of punishment to those who deserve it.']", "label": 1 }, { "id": "train_3311", "context": "The CFO attributes our increased sales last year to the new markets in Eastern Europe. Nevertheless, a careful study of our revenue reveals a boom of sales in Southeast Asia, while the markets in Eastern Europe, after an early surge, settled at levels much lower than expected. After all, in Eastern Europe, our firm has fierce competition from both German and Russian exports, some of which offer exceptionally high quality at low cost. By contrast, no producers in this sector operate to the East of Turkey, and our firm is the only one in our sector with significant inroads into India, Thailand, Vietnam, and Taiwan. Current plans call for expansion into Eastern Europe, but the Southeast Asian markets hold much more promise. ", "question": "In the argument given, the two portions in boldface play which of the following roles?", "answers": "['The first provides evidence to support the conclusion; the second is that conclusion.', 'The first is evidence that has been used to support an alternate conclusion that the author challenges; the second is this alternate conclusion', 'The first is evidence that has been used to support a conclusion the argument challenges; the second is a competing conclusion that the argument favors.', 'The first is a claim, the accuracy of which is at issue in the argument; the second is a conclusion drawn on the basis of that claim.']", "label": 0 }, { "id": "train_3312", "context": "Scientific research will be properly channeled whenever those who decide which research to fund give due weight to the scientific merits of all proposed research. But when government agencies control these funding decisions, political considerations play a major role in determining which research will be funded, and whenever political considerations play such a role, the inevitable result is that scientific research is not properly channeled.", "question": "Which one of the following can be properly inferred from the statements above?", "answers": "['When scientific research is not properly channeled, governments tend to step in and take control of the process of choosing which research to fund.', 'There is no proper role for political considerations to play in determining who will decide which scientific research to fund.', 'Giving political considerations a major role in determining which scientific research to fund is incompatible with giving proper weight to the scientific merits of proposed research.', 'It is inevitable that considerations of scientific merit will be neglected in decisions regarding the funding of scientific research.']", "label": 2 }, { "id": "train_3313", "context": "Early in the development of a new product line, the critical resource is talent. New marketing ventures require a degree of managerial skill disproportionate to their short-term revenue prospects. Usually, however, talented managers are assigned only to established high-revenue product lines and, as a result, most new marketing ventures fail. Contrary to current practice, the best managers in a company should be assigned to development projects.", "question": "Which one of the following, if true, most strengthens the author's argument?", "answers": "[\"For most established companies, the development of new product lines is a relatively small part of the company's total expenditure.\", 'Early short-term revenue prospects of a new product line are usually a good predictor of how successful a product line will ultimately be.', \"The current revenue and profitability of an established product line can be maintained even if the company's best managers are assigned elsewhere.\", 'The more talented a manager is, the less likely he or she is to be interested in undertaking the development of a new product line.']", "label": 2 }, { "id": "train_3314", "context": "The new heart scans offer patients significant benefits. They can be completed in a fraction of the time required for an angiogram, with no recovery time necessary. Furthermore, the scans are more sensitive and can identify problem areas that an angiogram might not perceive. However, heart scans use more radiation than most diagnostic procedures, and can cause undue concern over and treatment for the harmless abnormalities often picked up by such sensitive technology.", "question": "Which of the following conclusions is best supported by the statements above?", "answers": "['Angiograms are more widely used than heart scans.', 'Patients should not be concerned about heart abnormalities that appear in a heart scan.', 'A heart scan could result in indirect harm by causing a patient to undergo unnecessary procedures.', 'A heart scan is the more appropriate of the two procedures for most patients.']", "label": 2 }, { "id": "train_3315", "context": "Tires may be either underinflated, overinflated, or neither. We are pretty safe in assuming that underinflation or overinflation of tires harms their tread. After all, no one has been able to show that these do not harm tire tread.", "question": "Which one of the following most accurately describes a flaw in the argument's reasoning?", "answers": "['The argument assumes what it is attempting to demonstrate.', 'The argument fails to precisely define the terms \"underinflation\"and \"overinflation. \"', 'The argument rejects the possibility that what has not been proven is nevertheless true.', 'The argument overlooks that what is not in principle susceptible to proof might be false.']", "label": 2 }, { "id": "train_3316", "context": "Politician: The level of violence in the United States is higher than that in every other industrialized country. Some people attribute this entirely to a lenient judicial system, but this cannot be right, for the judicial systems in many other industrialized countries are just as lenient.", "question": "The politician's argument attempts to undermine the explanation by", "answers": "['showing that the explanation relies on a misguided notion of the purpose of judicial systems', 'presenting evidence that contradicts an assumption that the explanation relies on', 'providing an alternative explanation', 'arguing that the purported fact it is intended to explain is actually untrue']", "label": 1 }, { "id": "train_3317", "context": "Lawyer: Did Congleton assign the best available graphic artist to the project? Witness: Yes. Lawyer: And the best writer? Witness: Yes. Lawyer: In fact everyone she assigned to work on the project was top notch? Witness: That' s true.", "question": "Lawyer: So, you lied to the court when you said, earlier, that Congleton wanted the project to fail? Each of the following accurately describes a flaw in the lawyer's reasoning displayed above EXCEPT:", "answers": "['It takes for granted that Congleton was not forced to assign the people she did to the project.', 'It ignores the possibility that the witness failed to infer from known facts what should have been inferred and ttherefore was not lying.', 'It takes for granted that the project could fail only if Congleton wanted it to fail.', 'It ignores the possibility that Congleton knew that the people assigned to the project would not work well together.']", "label": 2 }, { "id": "train_3318", "context": "Art Historian: Recently, computer analysis has revealed that a few of a famous Flemish artist's works are forgeries, and are actually the work of noted forger Elmyr de Hory. While such a development may emit violent reverberations through the art world, even those museums that have a wealth of the Flemish artists in their collections should not be overly concerned. Hundreds of this Flemish artist's works were tested to determine whether they were forgeries, yet only a slim few turned out to be actual forgeries. Thus, the master's reputation as one of the greatest artists humanity has ever produced will surely remain undiminished.", "question": "Which of the following, if true, casts the most doubt on the art historian's conclusion?", "answers": "[\"Some of the works upon which the Flemish artist's standing rests were identified by the computer analysis to be the work of de Hory.\", \"Though few in the art world dispute the outcome of the computer analysis of the Flemish artist's work, many contend that the identified forgeries are not the work of Elmyr de Hory but some other highly skilled forger.\", 'The computer analysis involved is more likely to mistake an actual work as a forgery than to mistake a forgery as an actual work.', \"Many of the Flemish artist's well-known portraits are in the collection of private owners and were ttherefore not subjected to computer analysis.\"]", "label": 0 }, { "id": "train_3319", "context": "The news exclusively covers important current events. Reality television stars are never covered on the news, except when they become pregnant.", "question": "Which one of the following must be true?", "answers": "['Some pregnancies are important current events.', 'The news always covers pregnancies.', 'Reality television stars never qualify as an important current event.', 'All pregnancies are important current events.']", "label": 0 }, { "id": "train_3320", "context": "Studies conducted by consumer safety advocates have shown that all-wheel drive reduces the risk of automobile accidents. But a study by the National Automobile Insurers Associations claims that vehicles equipped with all-wheel drive are more likely to be involved in accidents than vehicles that do not have all-wheel drive.", "question": "Which one of the following, if true, does the most to resolve the apparent paradox?", "answers": "['Owners of vehicles with all-wheel drive usually drive in more congested areas where more accidents occur.', 'All-wheel drive is a feature only available on newer model vehicles.', 'All-wheel drive vehicles are stolen at a higher rate than non-all-wheel drive vehicles.', 'Owners of all-wheel drive vehicles tend to take better care of their vehicles than do people who do not own all-wheel drive vehicles.']", "label": 0 }, { "id": "train_3321", "context": "A worldwide ban on the production of certain ozone-destroying chemicals would provide only an illusion of protection. Quantities of such chemicals, already produced, exist as coolants in millions of refrigerators. When they reach the ozone layer in the atmosphere, their action cannot be halted. So there is no way to prevent these chemicals from damaging the ozone layer further.", "question": "Which of the following, if true, most seriously weakens the argument above?", "answers": "['The coolants in refrigerators can be fully recovered at the end of the useful life of the refrigerators and reused.', 'In modern societies, refrigeration of food is necessary to prevent unhealthy and potentially life-threatening conditions.', 'Even if people should give up the use of refrigeration, the coolants already in existing refrigerators are a threat to atmospheric ozone.', 'It is impossible to measure with accuracy the quantity of ozone-destroying chemicals that exist as coolants in refrigerators.']", "label": 0 }, { "id": "train_3322", "context": "Activities that pose risks to life are acceptable if and only if each person who bears the risks either gains some net benefit that cannot be had without such risks, or bears the risks voluntarily.", "question": "Which one of the following judgments most closely conforms to the principle above?", "answers": "['Motor vehicles are allowed to emit certain low levels of pollution; the resulting health risks are acceptable because all users of motor vehicles share the resulting benefit of inexpensive, convenient travel.', 'A motorcyclist rides without a helmet; the risk of fatal injury to the motorcyclist thus incurred is acceptable because the motorcyclist incurs this risk willingly.', 'A smoker subjects people to secondhand smoke at an outdoor public meeting; the resulting risks are acceptable because the danger from secondhand smoke is minimal outdoors, where smoke dissipates quickly.', 'A nation requires all citizens to spend two years in national service; since such service involves no risk to life, the policy is acceptable.']", "label": 1 }, { "id": "train_3323", "context": "Because no employee wants to be associated with bad news in the eyes of a superior, information about serious problems at lower levels is progressively softened and distorted as it goes up each step in the management hierarchy. The chief executive is, ttherefore, less well informed about problems at lower levels than are his or her subordinates at those levels.", "question": "The conclusion drawn above is based on the assumption that", "answers": "['problems should be solved at the level in the management hierarchy at which they occur', 'problem-solving ability is more important at higher levels than it is at lower levels of the management hierarchy', 'chief executives obtain information about problems at lower levels from no source other than their subordinates', 'employees should be rewarded for accurately reporting problems to their superiors']", "label": 2 }, { "id": "train_3324", "context": "Principle: One should criticize the works or actions of another person only if the criticism will not seriously harm the person criticized and one does so in the hope or expectation of benefiting someone other than oneself. Application: Jarrett should not have criticized Ostertag' s essay in front of the class, since the defects in it were so obvious that pointing them out benefited no one.", "question": "Which one of the following, if true, justifies the above application of the principle?", "answers": "['Jarrett knew that the criticism might antagonize Ostertag.', 'Jarrett hoped to gain prestige by criticizing Ostertag.', \"Jarrett did not expect the criticism to be to Ostertag's benefit.\", 'Jarrett knew that the defects in the essay were so obvious that pointing them out would benefit no one.']", "label": 3 }, { "id": "train_3325", "context": "Commentator: Many people argue that the release of chlorofluorocarbons into the atmosphere is harming humans by damaging the ozone layer, thus allowing increased amounts of ultraviolet radiation to reach Earth. But 300, 000 years ago a supernova greatly damaged the ozone layer, with no significant effect on our earliest ancestors. Because the supernova' s disruption was much greater than the estimated effect of chlorofluorocarbons today, there is no reason to think that these chemicals in the atmosphere harm humans in this way.", "question": "Which one of the following, if true, would most seriously weaken the commentator's argument?", "answers": "['The ozone layer regenerates at a slow rate, barring counteractive processes.', 'Our earliest ancestors possessed genetic characteristics making them more resistant than we are to the harmful effects of ultraviolet radiation.', 'Scientists have discovered that genetic changes occurred in our ancestors during the period in which the supernova affected Earth.', 'Extraterrestrial influences on the ozone layer tend to occur less often than terrestrial ones.']", "label": 1 }, { "id": "train_3326", "context": "For democracy to survive, it is imperative that the average citizen be able to develop informed opinions about important policy issues. In today' s society, this means that citizens must be able to develop informed opinions on many scientific subjects, from ecosystems to defense systems. Yet, as scientific knowledge advances, the average citizen is increasingly unable to absorb enough information to develop informed opinions on many important issues.", "question": "Of the following, which one follows logically from the passage?", "answers": "['The survival of democracy is threatened by the advance of scientific knowledge.', 'Every citizen has a duty to and can become scientifically literate.', 'The most effective democracy is one that is the most scientifically unsophisticated.', 'Democracy will survive if there are at least some citizens who are capable of developing informed opinions on important scientific issues.']", "label": 0 }, { "id": "train_3327", "context": "Wu: Jurgens is dishonest and so should not be elected mayor. Miller: I disagree with your conclusion. Jurgens should be elected mayor. Honest people usually are not tough enough to run a city.", "question": "Miller's questionable reasoning in which one of the following dialogues is most closely parallel to Miller's questionable reasoning in the dialogue above?", "answers": "[\"Wu: We should not paint the office this shade of yellow. It's too bright. Miller: Yes, we should. Bright colors keep people feeling wide awake.\", 'Wu: Bailey should not be the company spokesperson. He is too aggressive. Miller: Yes, he should. Aggressive people generally do not get pushed around.', 'Wu: We should not go back to that restaurant. The service is too slow. Miller: Yes, we should. Food that is served quickly is often of poor quality.', \"Wu: We should not upgrade the software. It's too expensive. Miller: Yes, we should. The best normally costs more.\"]", "label": 2 }, { "id": "train_3328", "context": "Herbalist: Many of my customers find that their physical coordination improves after drinking juice containing certain herbs. A few doctors assert that the herbs are potentially harmful, but doctors are always trying to maintain a monopoly over medical therapies. So there is no reason not to try my herb juice.", "question": "The reasoning in the herbalist's argument is flawed because the argument", "answers": "['rejects a claim by attacking the proponents of the claim rather than addressing the claim itself', 'relies on evidence presented in terms that presuppose the truth of the claim for which the evidence is offered', 'bases a conclusion on claims that are inconsistent with each other', 'attempts to force acceptance of a claim by inducing fear of the consequences of rejecting that claim']", "label": 0 }, { "id": "train_3329", "context": "Susan: Many rare plants have provided cures for various diseases. Since many plants have been lost to extinction, it is likely that there could have been cures for many more diseases. That is why it is important for us to manage our resources responsibly and prevent plant extinction. Robert: But plants are not our resources, they are part of the same environment we inhabit, and we are not superior to them. We must treat plants as the co-equal inhabitants of this planet that they are.", "question": "Which one of the following is an issue about which Susan and Robert disagree?", "answers": "['whether humans should prevent plant extinction', 'whether animals should be prevented from extinction', 'whether plants should be treated humanely', 'whether humans should exploit plants for human benefit']", "label": 3 }, { "id": "train_3330", "context": "Plant Manager: We could greatly reduce the amount of sulfur dioxide our copper-smelting plant releases into the atmosphere by using a new process. The new process requires replacing our open furnaces with closed ones and moving the copper from one furnace to the next in solid, not molten, form. However, not only is the new equipment expensive to buy and install, but the new process also costs more to run than the current process, because the copper must be reheated after it has cooled. So overall, adopting the new process will cost much but bring the company no profit. Supervisor: I agree with your overall conclusion, but disagree about one point you make, since the latest closed furnaces are extremely fuel-efficient.", "question": "The point about which the supervisor expresses disagreement with the plant manager is", "answers": "['whether the new copper-smelting process releases less sulfur dioxide gas into the atmosphere than the current process', 'whether the new process should be adopted in the copper-smelting plant', 'whether the new copper-smelting process is more expensive to run than the current process', 'whether closed copper-smelting furnaces are more fuel-efficient than open furnaces']", "label": 2 }, { "id": "train_3331", "context": "People who have political power tend to see new technologies as a means of extending or protecting their power, whereas they generally see new ethical arguments and ideas as a threat to it. Ttherefore, technical ingenuity usually brings benefits to those who have this ingenuity, whereas ethical inventiveness brings only pain to those who have this inventiveness.", "question": "Which one of the following statements, if true, most strengthens the argument?", "answers": "['Ethical inventiveness and technical ingenuity are never possessed by the same individuals.', 'Many people who possess ethical inventiveness conceal their novel ethical arguments for fear of retribution by the politically powerful.', 'New technologies are often used by people who strive to defeat those who currently have political power.', 'Politically powerful people tend to reward those who they believe are useful to them and to punish those who they believe are a threat.']", "label": 3 }, { "id": "train_3332", "context": "Columnist: Donating items to charity may be a sign of generosity, but any generosity it may demonstrate is rarely a permanent virtue, since most donors make donations only intermittently.", "question": "Which one of the following most accurately describes a flaw in the columnist's argument?", "answers": "['The argument takes for granted that a character trait is present only when manifested.', 'The argument attacks the character of those whose values are different from those of the columnist.', 'The argument takes for granted that most people donate out of generosity.', 'The argument generalizes from too small a sample of cases.']", "label": 0 }, { "id": "train_3333", "context": "Mario: The field of cognitive science is not a genuinely autonomous discipline since it addresses issues also addressed by the disciplines of computer science, linguistics, and psychology. A genuinely autonomous discipline has a domain of inquiry all its own. Lucy: Nonsense. You' ve always acknowledged that philosophy is a genuinely autonomous discipline and that, like most people, you think of philosophy as addressing issues also addressed by the disciplines of linguistics, mathematics, and psychology. A field of study is a genuinely autonomous discipline by virtue of its having a unique methodology rather than by virtue of its addressing issues that no other field of study addresses.", "question": "Lucy responds to Mario by", "answers": "['establishing that Mario is not a philosopher', \"questioning Mario's expertise in cognitive science\", \"showing that some of Mario's beliefs are not compatible with the principle on which he bases his conclusion\", 'demonstrating that Mario confuses the notion of a field of study with that of a genuinely autonomous discipline']", "label": 2 }, { "id": "train_3334", "context": "Because of the threat of medical malpractice suits, many physicians are subjecting their patients to unnecessary medical testing. Physicians want to be able to explain their actions if anything goes wrong, and they want to be able to show that they tried everything to properly diagnose and treat an ailment. It is difficult for physicians to judge exactly how thorough they need to be in order to avoid potential liability. It is generally wise, ttherefore, for patients to avoid going to a physician unless the patients are very sick.", "question": "Which one of the following, if true, would most seriously weaken the argument in the passage?", "answers": "['Many diseases, if they are not caught early, can cause death.', 'Under pressure from the medical malpractice insurers, many doctors have raised their fees.', 'Most insurance companies subsidize the additional expense associated with unnecessary medical tests.', 'Patients are unable to determine what is the appropriate level of testing.']", "label": 0 }, { "id": "train_3335", "context": "If future improvements to computer simulations of automobile crashes enable computers to provide as much reliable information about the effectiveness of automobile safety features as is provided by actual test crashes, then manufacturers will use far fewer actual test crashes. For the costs of designing and running computer simulations are much lower than those of actual test crashes.", "question": "Which one of the following, if true, most strongly supports the argument?", "answers": "['The cost per automobile of testing and designing safety features is decreasing and will continue to decrease for the foreseeable future.', 'If computer simulations will soon be able to provide more information about the effectiveness of automobile safety features, automobile manufacturers will soon be able to produce safer cars.', 'For years, the aviation industry has been successfully using computer simulations of airplane crashes to test the effectiveness of safety features of airplane designs.', 'Apart from information about safety features, actual test crashes provide very little information of importance to automobile manufacturers.']", "label": 3 }, { "id": "train_3336", "context": "Jones fell unconscious on the job and it was suspected that he had swallowed a certain chemical, so he was rushed to the local hospital' s emergency room. In making her diagnosis, the emergency-room physician knew that if Jones had swallowed the chemical, a deficiency in the content of a mineral in his blood would result. She also knew that deficiency in the mineral causes inflammation of the skin. Since Jones' s skin was not inflamed when he was admitted to the emergency room, the physician concluded that Jones had not swallowed the chemical.", "question": "Which one of the following, if true, would undermine the physician's conclusion?", "answers": "['It takes 48 hours for the chemical to bring about deficiency of the mineral in the blood.', 'Deficiency in minerals other than the mineral in question can cause inflammation of the skin.', 'Jones did not know that the chemical was dangerous.', 'Jones had suffered inflammation of the skin in the past.']", "label": 0 }, { "id": "train_3337", "context": "Sarah: Some schools seek to foster a habit of volunteering in their students by requiring them to perform community service. But since a person who has been forced to do something has not really volunteered and since the habit of volunteering cannot be said to have been fostered in a person who has not yet volunteered for anything, there is no way this policy can succeed by itself. Paul: I disagree. Some students forced to perform community service have enjoyed it so much that they subsequently actually volunteer to do something similar. In such cases, the policy can clearly be said to have fostered a habit of volunteering.", "question": "Paul responds to Sarah's argument using which one of the following argumentative techniques?", "answers": "[\"He introduces considerations that call into question one of Sarah's assumptions.\", \"He argues that Sarah's conception of what it means to volunteer excludes certain activities that ought to be considered instances of volunteering.\", 'He argues that a policy Sarah fails to consider could accomplish the same aim as the policy that Sarah considers.', 'He argues that Sarah is assuming just what she sets out to prove.']", "label": 0 }, { "id": "train_3338", "context": "According to a study concerning environmental economics, any manufacturer that offers an environmentally sound lawnmower, such as a manual push mower, will almost certainly reap a huge profit because consumers say that they want a mower that requires neither gasoline nor electricity. A spokesman for the trade association of gasoline lawnmower manufacturers asserted that this claim had little factual support.", "question": "Which of the following, if true, would best support the claim made by the spokesperson?", "answers": "['Most people mow their lawns less often then professionals recommend.', 'More people refuse to purchase electric lawnmowers than gasoline lawnmowers.', 'The last manufacturer of manual push lawnmowers just declared bankruptcy because of declining profits.', 'A majority of the public believes that the price of gasoline is too high.']", "label": 2 }, { "id": "train_3339", "context": "Healthy lungs produce a natural antibiotic that protects them from infection by routinely killing harmful bacteria on airway surfaces. People with cystic fibrosis, however, are unable to fight off such bacteria, even though their lungs produce normal amounts of the antibiotic. The fluid on airway surfaces in the lungs of people with cystic fibrosis has an abnormally high salt concentration; accordingly, scientists hypothesize that the high salt concentration is what makes the antibiotic ineffective.", "question": "Which of the following, if true, most strongly supports the scientists' hypothesis?", "answers": "['When the salt concentration of the fluid on the airway surfaces of healthy people is raised artificially, the salt concentration soon returns to normal.', 'A sample of the antibiotic was capable of killing bacteria in an environment with an unusually low concentration of salt.', 'When lung tissue from people with cystic fibrosis is maintained in a solution with a normal salt concentration, the tissue can resist bacteria.', 'Many lung infections can be treated by applying synthetic antibiotics to the airway surfaces.']", "label": 2 }, { "id": "train_3340", "context": "The Japanese haiku is defined as a poem of three lines with five syllables in the first line, seven syllables in the second line, and five syllables in the third line. English poets tend to ignore this fact. Disregarding syllable count, they generally call any three-line English poem with a \"haiku feel\" a haiku. This demonstrates that English poets have little respect for foreign traditions, even those from which some of their own poetry derives.", "question": "The reasoning is flawed because it", "answers": "['draws a conclusion that is broader in scope than is warranted by the evidence advanced', 'overlooks the possibility that the case it cites is not unique', 'relies on stereotypes instead of presenting evidence', 'fails to acknowledge that ignoring something implies a negative judgment about that thing']", "label": 0 }, { "id": "train_3341", "context": "Joe: All vampire stories are based on an absurd set of premises. Since, according to such stories, every victim of a vampire becomes a vampire, and vampires have existed since ancient times and are immortal, vampires would by now have almost completely eliminated their prey. Maria: In most of the vampire stories I am familiar with, vampires tum only a few of their victims into vampires. The rest are permanently dead.", "question": "Joe and Maria disagree over the truth of which one of the following?", "answers": "['In all vampire stories, every victim of a vampire becomes a vampire.', 'Vampires are always depicted in vampire stories as having existed since ancient times.', 'No vampire stories are incoherent.', 'Vampires are always depicted in vampire stories as immortal.']", "label": 0 }, { "id": "train_3342", "context": "Astronomer: Proponents of the hypothesis that life evolved extraterrestrially and drifted here in spores point out that, 3. 8 billion years ago, Earth was bombarded by meteorites that would have destroyed any life already here. Yet 3. 5 billion years ago, Earth had life forms complex enough to leave fossil remains. Such life could not have evolved here in the 0. 3 billion years following the meteorite bombardments, they claim. There is good reason to regard their hypothesis as false, however, for they merely offer empirical arguments against the view that life evolved on Earth; neither they nor anyone else has provided positive support for the extraterrestrial-spore theory of the origin of terrestrial life.", "question": "The reasoning in the astronomer's argument is flawed because the argument", "answers": "['attempts to derive a conclusion from premises that contradict it', 'concludes, simply because there is no evidence in favor of a hypothesis, that there is evidence against that hypothesis', \"grants the truth of claims that are made by the advocates of the hypothesis but that do nothing to strengthen the astronomer's own argument\", 'reasons that a hypothesis is false simply because there is another hypothesis that is equally likely to be true']", "label": 1 }, { "id": "train_3343", "context": "City leader: If our city adopts the new tourism plan, the amount of money that tourists spend here annually will increase by at least $2 billion, creating as many jobs as a new automobile manufacturing plant would. It would be reasonable for the city to spend the amount of money necessary to convince an automobile manufacturer to build a plant here, but adopting the tourism plan would cost less.", "question": "The city leader's statements, if true, provide the most support for which one of the following?", "answers": "['It would be reasonable for the city to adopt the new tourism plan.', 'In general, it is reasonable for the city to spend money to try to convince manufacturing companies to build plants in the city.', 'The city should implement the least expensive job creation measures available.', 'The only way the city can create jobs is by increasing tourism.']", "label": 0 }, { "id": "train_3344", "context": "Numismatist: In medieval Spain, most gold coins were minted from gold mined in West Africa, in the area that is now Senegal. The gold mined in this region was the purest known. Its gold content of 92 percent allowed coins to be minted without refining the gold, and indeed coins minted from this source of gold can be recognized because they have that gold content. The mints could refine gold and produced other kinds of coins that had much purer gold content, but the Senegalese gold was never refined.", "question": "Which one of the following inferences about gold coins minted in medieval Spain is most strongly supported by the information the numismatist gives?", "answers": "['No gold coins were minted that had a gold content of less than 92 percent.', 'Two coins could have the same monetary value even though they differed from each other in the percentage of gold they contained.', 'The only unrefined gold from which coins could be minted was Senegalese gold.', 'The source of some refined gold from which coins were minted was unrefined gold with a gold content of less than 92 percent.']", "label": 3 }, { "id": "train_3345", "context": "If one has evidence that an act will benefit other people and performs that act to benefit them, then one will generally succeed in benefiting them.", "question": "Which one of the following best illustrates the proposition above?", "answers": "['Zachary hoped that psychotherapy could help his parents overcome their marital difficulties. He persuaded his parents to call a psychotherapist, and eventually their problems were resolved.', \"Betsy overheard a heating contractor say that regularly changing the filter in a furnace helps to keep the furnace efficient. So Betsy has regularly changed the furnace filter in her daughter's house. As a result, the furnace has never required maintenance due to becoming clogged with dust or dirt.\", 'Sejal learned in a psychology class that the best way to help someone overcome an addiction is to confront that person. So she confronted her friend Bob, who was struggling with a chemical dependency.', \"A country's leaders realized that fostering diplomatic ties with antagonistic nations reduces the chances of war with those nations. Because those leaders worried that war would harm their chances of being reelected, they engaged in diplomatic discussions with a hostile country, and the two countries avoided a confrontation.\"]", "label": 1 }, { "id": "train_3346", "context": "Questions have arisen regarding the accuracy of the reports the university' s archaeological museum issues on its sales and acquisitions for the year. To forestall controversy, this year' s report is being reviewed by three archaeologists from other universities. Since these archaeologists will be given full access to all documents on which the report is based, they will be able to determine whether it is indeed accurate.", "question": "The reasoning in the argument is flawed because the argument", "answers": "['does not describe what will occur if the reviewers discover discrepancies between the report and the documents on which it was based', \"omits any mention of whether the museum's collection is on display or is available only to researchers\", 'ignores the possibility that there might have been some sales or acquisitions during the past year that were not mentioned in the documents on which the report was based', \"provides no information regarding the size or quality of the archaeological museum's collection\"]", "label": 2 }, { "id": "train_3347", "context": "James: Chemists have recently invented a new technique for extracting rhodium, an element necessary for manufacturing catalytic converters for automobiles, from nuclear waste. Catalytic converters function to remove noxious gases from automobile exhaust. The use of nuclear power is ttherefore contributing in at least one way to creating a cleaner environment. Marta: The technique you mention, though effective, is still at an experimental stage, so there has been no shift in the sources of the rhodium currently used in manufacturing catalytic converters.", "question": "Marta responds to James's argument by", "answers": "['pointing out a fact that James, in drawing his conclusion, did not take into account', 'indicating that James is assuming the truth of the conclusion that he intends to establish', 'casting doubt on the accuracy of the claims made by James in support of his conclusion', 'questioning the credibility of advocates of nuclear power']", "label": 0 }, { "id": "train_3348", "context": "Some species of Arctic birds are threatened by recent sharp increases in the population of snow geese, which breed in the Arctic and are displacing birds of less vigorous species. Although snow geese are a popular quarry for hunters in the southern regions where they winter, the hunting season ends if and when hunting has reduced the population by five percent, according to official estimates. Clearly, dropping this restriction would allow the other species to recover.", "question": "Which of the following, if true, most seriously undermines the argument?", "answers": "['It has been many years since the restriction led to the hunting season for snow geese being closed earlier than the scheduled date.', 'As their population has increased, snow geese have recolonized wintering grounds that they had not used for several seasons.', 'The number of snow geese taken by hunters each year has grown every year for several years.', \"In the snow goose's winter habitats, the goose faces no significant natural predation.\"]", "label": 0 }, { "id": "train_3349", "context": "Previously, Autoco designed all of its cars itself and then contracted with specialized parts suppliers to build parts according to its specifications. Now it plans to include its suppliers in designing the parts they are to build. Since many parts suppliers have more designers with specialized experience than Autoco has, Autoco expects this shift to reduce the overall time and cost of the design of its next new car.", "question": "Which of the following, if true, most strongly supports Autoco's expectation?", "answers": "['When suppliers provide their own designs, Autoco often needs to modify its overall design.', 'When suppliers built parts according to specifications provided by Autoco, the suppliers competed to win contracts.', 'In order to provide designs for Autoco, several of the parts suppliers will have to add to their existing staffs of designers.', \"Most of Autoco's suppliers have on hand a wide range of previously prepared parts designs that can readily be modified for a new car.\"]", "label": 3 }, { "id": "train_3350", "context": "Problem: The Thimble Corporation recently distributed rebate coupons for one of its products. Some of the coupons bore an expiration date that was too early. This created an unfair situation in which some customers believed, incorrectly, that the rebate offer had already expired. Principle: Anyone who creates an unfair situation has an obligation to rectify any unfair result of that situation.", "question": "The principle, if valid, most helps to justify which one of the following judgments concerning the problem?", "answers": "['Because it cannot identify all of the customers who were adversely affected by the incorrect expiration date, the Thimble Corporation should deny the rebate to all of the customers who applied for it.', 'If a customer did not rely on an incorrect expiration date when applying for the rebate but was denied the rebate for any other reason, the Thimble Corporation is not obligated to offer that customer the rebate.', 'If there is a chance that any customers did not apply for the rebate because of an incorrect expiration date on their rebate coupon, the Thimble Corporation is obligated to try to identify them and offer them the rebate.', 'If a customer believed that the expiration date had passed but applied for the rebate anyway, the Thimble Corporation is not obligated to give a rebate to that customer.']", "label": 2 }, { "id": "train_3351", "context": "Theories generated by scientific research were used to develop several products that, although useful, damage the environment severely. The scientists who conducted the research, however, should not be held responsible for that damage, since they merely generated the theories and could neither foresee nor restrict the kinds of products that might be designed using those theories.", "question": "Which one of the following principles, if established, justifies the conclusion above?", "answers": "['Individuals should be held responsible for the foreseeable undesirable consequences of the activities that they perform and receive credit for the foreseeable desirable consequences of those activities.', 'Individuals who perform an activity should not be held responsible for any unforeseen undesirable consequences that arise from the use to which others put the results of that activity.', 'Individuals are justified in performing an activity that has both desirable and undesirable foreseeable consequences only if they alone bear its undesirable consequences.', 'Individuals who develop something that has desirable characteristics should not be held responsible for any undesirable characteristics that the thing has if improperly used.']", "label": 1 }, { "id": "train_3352", "context": "Robust crops not only withstand insect attacks more successfully than other crops, they are also less likely to be attacked in the first place, since insects tend to feed on weaker plants. Killing insects with pesticides does not address the underlying problem of inherent vulnerability to damage caused by insect attacks. Thus, a better way to reduce the vulnerability of agricultural crops to insect pest damage is to grow those crops in good soil -- soil with adequate nutrients, organic matter, and microbial activity.", "question": "Which one of the following is an assumption on which the argument depends?", "answers": "['The application of pesticides to weak crops fails to reduce the extent to which they are damaged by insect pests.', 'Growing crops without the use of pesticides generally produces less robust plants than when pesticides are used.', 'Crops that are grown in good soil tend to be more robust than other crops.', \"The application of nutrients and organic matter to farmland improves the soil's microbial activity.\"]", "label": 2 }, { "id": "train_3353", "context": "Asa general rule, the larger a social group of primates, the more time its members spend grooming one another. The main purpose of this social grooming is the maintenance of social cohesion. Furthermore, group size among primates tends to increase proportionally with the size of the neocortex, the seat of higher thought in the brain. Extrapolating upon the relationship between group size and neocortex size, we can infer that early human groups were quite large. But unexpectedly, there is strong evidence that, apart from parents grooming their children, these humans spent virtually no time grooming one another.", "question": "Which one of the following, if true, would do most to resolve the apparent discrepancy described above?", "answers": "['Many types of primates other than humans have fairly large neocortex regions and display frequent social grooming.', 'Early humans were not as extensively covered with hair as are other primates, and consequently they had less need for social grooming.', 'While early humans probably lived in large groups, there is strong evidence that they hunted in small groups.', 'Early humans developed languages, which provided a more effective way of maintaining social cohesion than social grooming.']", "label": 3 }, { "id": "train_3354", "context": "It is difficult to grow cacti in a humid climate. It is difficult to raise orange trees in a cold climate. In most parts of a certain country, it is either easy to grow cacti or easy to raise orange trees.", "question": "If the statements above are true, which one of the following must be false?", "answers": "['Most of the country is hot.', 'It is not possible to raise cacti in the country.', 'Half of the country is both humid and cold.', 'Most parts of the country are humid.']", "label": 2 }, { "id": "train_3355", "context": "A new device uses the global positioning system to determine a cow' s location and, when a cow strays outside of its pasture, makes noises in the cow' s ears to steer it back to its home range. Outfitting all of the cattle in a herd with this device is far more expensive than other means of keeping cattle in their pastures, such as fences. The device' s maker nevertheless predicts that ranchers will purchase the device at its current price.", "question": "Which one of the following, if true, does the most to support the prediction made by the device's maker?", "answers": "['The device has been shown to be as effective as fences at keeping cattle in their pastures.', \"The price of the device will come down appreciably if the device's maker is able to produce it in large quantities.\", 'The device has been shown not to cause significant stress to cattle.', 'As they graze, cattle in a herd follow the lead of the same few members of the herd.']", "label": 3 }, { "id": "train_3356", "context": "Scientists have made genetic modifications to cotton to increase its resistance to insect pests. According to farmers' report, the amount of insecticide needed per acre to control insect pests was only slightly lower for those who tried the modified seed than for those who did not. Ttherefore, since the modified seed costs more than ordinary seed without producing yields of higher market value, switching to the modified seed would be unlikely to benefit most cotton farmers economically.", "question": "Which of the following would it be most useful to know in order to evaluate the argument?", "answers": "['Whether the farmers who have tried the modified seed planted as many acres of cotton, on average, as farmers using the ordinary seed did', 'Whether farmers who tried the modified cotton seed had ever tried growing other crops from genetically modified seed', 'Whether the insecticides typically used on ordinary cotton tend to be more expensive than insecticides typically used on other crops', 'Whether most of the farmers who tried the modified seed did so because they had previously had to use exceptionally large quantities of insecticide']", "label": 0 }, { "id": "train_3357", "context": "One child pushed another child from behind, injuring the second child. The first child clearly understands the difference between right and wrong, so what was done was wrong if it was intended to injure the second child.", "question": "Which one of the following principles, if valid, most helps to justify the reasoning in the argument?", "answers": "['It is wrong for a person who understands the difference between right and wrong to intentionally harm another person.', 'An act that harms another person is wrong if the person who did it understands the difference between right and wrong and did not think about whether the act would injure the other person.', 'A person who does not understand the difference between right and wrong does not bear any responsibility for harming another person.', 'An action that is intended to harm another person is wrong only if the person who performed the action understands the difference between right and wrong.']", "label": 0 }, { "id": "train_3358", "context": "Economist: On average, the emergency treatment for an elderly person for injuries resulting from a fall costs $11, 000. A new therapeutic program can significantly reduce an elderly person' s chances of falling. Though obviously desirable for many reasons, this treatment program will cost $12, 500 and thus cannot be justified.", "question": "Which of the following, if true, most seriously undermines the conclusion of the argument?", "answers": "['Falls resulting in serious injuries are less common among elderly people living in nursing homes than they are among elderly people who live alone at home.', 'The new therapeutic program focuses on therapies other than medication, since overmedication can cause disorientation and hence increase the likelihood that an elderly person will have a serious fall.', 'Among elderly people who had followed the program for only a few months, the number of serious falls reported was higher than it was for people who had followed the program for its recommended minimum length of one year.', \"A frequent result of injuries sustained in falls is long-term pain, medication for which is not counted among the average per-person costs of emergency treatment for elderly people's injuries from such falls.\"]", "label": 3 }, { "id": "train_3359", "context": "In response to viral infection, the immune systems of mice typically produce antibodies that destroy the virus by binding to proteins on its surface. Mice infected with a herpesvirus generally develop keratitis, a degenerative disease affecting part of the eye. Since proteins on the surface of cells in this part of the eye closely resemble those on the herpesvirus surface, scientists hypothesize that these cases of keratitis are caused by antibodies to herpesvirus.", "question": "Which of the following, if true, gives the greatest additional support to the scientists' hypothesis?", "answers": "['Mice that have never been infected with a herpesvirus can sometimes develop keratitis.', 'Mice that are infected with a herpesvirus but do not develop keratitis produce as many antibodies as infected mice that do develop keratitis.', 'Other types of virus have surface proteins that closely resemble proteins found in various organs of mice.', 'There are mice that are unable to form antibodies in response to herpes infections, and these mice survive these infections without ever developing keratitis.']", "label": 3 }, { "id": "train_3360", "context": "Children fall into three groups -- nontasters, regular tasters, and supertasters -- depending on how strongly they experience tastes. Supertasters strongly prefer mild cheddar cheese to sharp, regular tasters weakly prefer mild to sharp, and nontasters show no preference. Also, the more bitter a food tastes, the less children like it. Thus, supertasters experience sharp cheddar as tasting more bitter than mild cheddar, but nontasters experience sharp cheddar as tasting no more bitter than mild cheddar.", "question": "Which one of the following, if assumed, enables the conclusion above to be properly inferred?", "answers": "['The age of the child is the most important factor in determining whether that child is a nontaster, a regular taster, or a supertaster.', 'Supertasters like mild cheddar cheese more than do regular tasters.', 'Bitterness is the only factor relevant to how strongly children prefer sharp cheddar cheese to mild cheddar cheese.', 'Nontasters tend to like a wider variety of foods than do regular tasters, who in turn like a wider variety of foods than do supertasters.']", "label": 2 }, { "id": "train_3361", "context": "Gortland has long been narrowly self-sufficient in both grain and meat. However, as per capita income in Gortland has risen toward the world average, per capita consumption of meat has also risen toward the world average, and it takes several pounds of grain to produce one pound of meat. Ttherefore, since per capita income continues to rise, whereas domestic grain production will not increase, Gortland will soon have to import either grain or meat or both.", "question": "Which of the following is an assumption on which the argument depends?", "answers": "['The per capita consumption of meat in Gortland is increasing at roughly the same rate across all income levels.', 'The total acreage devoted to grain production in Gortland will soon decrease.', \"Importing either grain or meat will not result in a significantly higher percentage of Gortlanders' incomes being spent on food than is currently the case.\", 'People in Gortland who increase their consumption of meat will not radically decrease their consumption of grain.']", "label": 3 }, { "id": "train_3362", "context": "Oil company representative: We spent more money on cleaning the otters affected by our recent oil spill than has been spent on any previous marine mammal rescue project. This shows our concern for the environment. Environmentalist: You have no such concern. Your real concern is evident in your admission to the press that news photographs of oil-covered otters would be particularly damaging to your public image, which plays an important role in your level of sales.", "question": "The environmentalist's conclusion would be properly drawn if it were true that the", "answers": "[\"oil company's efforts toward cleaning the affected otters have been more successful than have such efforts in previous projects to clean up oil spills\", 'oil company cannot have more than one motive for cleaning the otters affected by the oil spill', 'otter population in the area of the oil spill could not have survived without the cleaning project', 'government would have spent the money to clean the otters if the oil company had not agreed to do it']", "label": 1 }, { "id": "train_3363", "context": "Mariah: Joanna has argued that Adam should not judge the essay contest because several of his classmates have entered the contest. However, the essays are not identified by author to the judge and, moreover, none of Adam' s friends are classmates of his. Still, Adam has no experience in critiquing essays. Ttherefore, I agree with Joanna that Adam should not judge the contest.", "question": "Which one of the following principles, if valid, most helps to justify Mariah's argument?", "answers": "['A suspicion of bias is insufficient grounds on which to disqualify someone from judging a contest.', 'In selecting a contest judge, fairness concerns should override concern for the appropriate expertise.', 'Expertise should be the primary prerequisite for serving as a contest judge.', \"The ability of a judge to make objective decisions is more important than that judge's content expertise.\"]", "label": 2 }, { "id": "train_3364", "context": "Although fiber-optic telephone cable is more expensive to manufacture than copper telephone cable, a telephone network using fiber-optic cable is less expensive overall than a telephone network using copper cable. This is because copper cable requires frequent amplification of complex electrical signals to carry them for long distances, whereas the pulses of light that are transmitted along fiber-optic cable can travel much farther before amplification is needed.", "question": "The above statements, if true, most strongly support which one of the following?", "answers": "['Signals transmitted through fiber-optic cable travel at the same speed as signals transmitted through copper cable.', 'The cost associated with frequent amplification of signals traveling through copper cable exceeds the extra manufacturing cost of fiber-optic cable.', 'The material from which fiber-optic cable is manufactured is more expensive than the copper from which copper cable is made.', 'The increase in the number of transmissions of complex signals through telephone cables is straining those telephone networks that still use copper cable.']", "label": 1 }, { "id": "train_3365", "context": "People who do regular volunteer work tend to live longer, on average, than people who do not. It has been found that \"doing good, \" a category that certainly includes volunteer work, releases endorphins, the brain's natural opiates, which induce in people a feeling of well-being. Clearly, there is a connection: Regular releases of endorphins must in some way help to extend people's lives.", "question": "Which of the following, if true, most seriously undermines the force of the evidence given as support for the hypothesis that endorphins promote longevity?", "answers": "['There are many people who have done some volunteer work but who do not do such work regularly.', 'Although extremely high levels of endorphins could be harmful to health, such levels are never reached as a result of the natural release of endorphins.', 'People tend not to become involved in regular volunteer work unless they are healthy and energetic to begin with.', 'People who do regular volunteer work are only somewhat more likely than others to characterize the work they do for a living as \"doing good. \"']", "label": 2 }, { "id": "train_3366", "context": "Oxygen-18 is a heavier-than-normal isotope of oxygen. In a rain cloud, water molecules containing oxygen-18 are rarer than water molecules containing normal oxygen. But in rainfall, a higher proportion of all water molecules containing oxygen-18 than of all water molecules containing ordinary oxygen descends to earth. Consequently, scientists were surprised when measurements along the entire route of rain clouds' passage from above the Atlantic Ocean, the site of their original formation, across the Amazon forests, where it rains almost daily, showed that the oxygen-18 content of each of the clouds remained fairly constant.", "question": "Which one of the following statements, if true, best helps to resolve the conflict between scientists' expectations, based on the known behavior of oxygen-18, and the result of their measurements of the rain clouds' oxygen-18 content?", "answers": "['The amount of rainfall over the Amazon rain forests is exactly the same as the amount of rain originally collected in the clouds formed above the Atlantic Ocean.', 'The amount of rain recycled back into the atmosphere from the leaves of forest vegetation is exactly the same as the amount of rain in river runoffs that is not recycled into the atmosphere.', 'Like the oceans, tropical rain forests can create or replenish rain clouds in the atmosphere above them.', 'Rain clouds above tropical forests are poorer in oxygen-18 than rain clouds above unforested regions.']", "label": 2 }, { "id": "train_3367", "context": "Swimming pools should be fenced to protect children from drowning, but teaching children to swim is even more important. And there is a principle involved here that applies to childrearing generally. Thus, while we should restrict children' s access to the soft drinks and candies advertised on television shows directed towards children, it is even more important to teach them __", "question": "Which one of the following most logically completes the passage?", "answers": "['how to creatively entertain themselves without watching television', 'how to make nutritional choices that are conducive to their well-being', 'that television can be a good source of accurate information about many things', 'the importance of physical activity to health and well-being']", "label": 1 }, { "id": "train_3368", "context": "Proofs relying crucially on computers provide less certainty than do proofs not requiring computers. Human cognition alone cannot verify computer-dependent proofs; such proofs can never provide the degree of certainty that attends our judgments concerning, for instance, simple arithmetical facts, which can be verified by human calculation. Of course, in these cases one often uses electronic calculators, but here the computer is a convenience rather than a supplement to human cognition.", "question": "The statements above, if true, most strongly support which one of the following?", "answers": "['We can never be completely sure that proofs relying crucially on computers do not contain errors that humans do not detect.', 'It is impossible to supplement the cognitive abilities of humans by means of artificial devices such as computers.', 'If one can corroborate something by human calculation, one can be completely certain of it.', \"Only if a proof's result is arrived at without the help of a computer can one judge with any degree of certainty that the proof is correct.\"]", "label": 0 }, { "id": "train_3369", "context": "The purpose of a general theory of art is to explain every aesthetic feature that is found in any of the arts. Premodern general theories of art, however, focused primarily on painting and sculpture. Every premodern general theory of art, even those that succeed as theories of painting and sculpture, fails to explain some aesthetic feature of music.", "question": "The statements above, if true, most strongly support which one of the following?", "answers": "['A general theory of art that explains every aesthetic feature of music will achieve its purpose.', 'No premodern general theory of art achieves its purpose unless music is not art.', 'No premodern general theory of art explains any aesthetic features of music that are not shared with painting and sculpture.', 'Any general theory of art that explains the aesthetic features of painting also explains those of sculpture.']", "label": 1 }, { "id": "train_3370", "context": "Some people see no harm in promoting a folk remedy that in fact has no effect. But there is indeed harm: many people who are convinced to use an ineffective remedy continue with it for years rather than pursuing conventional treatments that would almost certainly help them.", "question": "Which one of the following principles, if valid, most helps to justify the reasoning in the argument?", "answers": "['It is harmful to interfere with someone doing something that is likely to benefit that person.', 'To convince people of something for which one knows there is no evidence is to be dishonest.', 'A person is responsible for harm he or she does to someone even if the harm was done unintentionally.', 'One should not promote a remedy if one believes that using that remedy will cause harm.']", "label": 0 }, { "id": "train_3371", "context": "Mark: The decongestant drug Zokaz was discontinued by its manufacturer because long-term studies revealed that it increased the risk of heart attack. Qualzan, another decongestant, works by essentially the same physiological mechanism as Zokaz. So Qualzan probably also increases the risk of heart attack. Kathy: The decongestive effects of the two drugs do stem from the same physiological mechanism. But since they are different chemically, the two drugs probably have different side effects.", "question": "Which one of the following is a technique of reasoning used in Kathy's response to Mark?", "answers": "['attempting to discredit an argument by comparing it to another obviously flawed argument that is logically parallel', 'arguing against a conclusion by raising questions about the validity of scientific studies cited in support of that conclusion', 'challenging an argument from analogy by focusing on a dissimilarity between the things being compared 9', 'attempting to undermine an argument by showing that it is incompatible with a fundamental principle of medicine']", "label": 2 }, { "id": "train_3372", "context": "Giordano Bruno, the Renaissance philosopher, must have been a spy for England during the time he served in the French embassy there. English state records show that a spy who foiled at least two assassination plots against Queen Elizabeth of England was in place in the French embassy in London at that time. Since the spy is identified in confidential English state documents only as \"the only clergyman working at the French embassy\" at that time, Bruno must have been the spy: Bruno had been ordained a member of the clergy long before he started work at the French embassy.", "question": "Which one of the following, if true, most weakens the force of the evidence advanced in the argument?", "answers": "[\"During Bruno's tenure in the French embassy, a high degree of tension and an atmosphere of mutual suspicion prevailed between France and England.\", 'Long before his employment at the embassy, Bruno had been excommunicated from his church and thereafter neither dressed nor functioned as a member of the clergy.', 'The French ambassador at the time was a man who trusted Bruno implicitly, often defending him from the criticisms of others on the embassy staff.', 'During the Renaissance, well-educated members of the clergy often served in nonclerical roles as tutors, secretaries, and personal assistants to wealthy aristocrats.']", "label": 1 }, { "id": "train_3373", "context": "Large national budget deficits do not cause large trade deficits. If they did, countries with the largest budget deficits would also have the largest trade deficits. In fact, when deficit figures are adjusted so that different countries are reliably comparable to each other, there is no such correlation.", "question": "If the statements above are all true, which of the following can properly be inferred on the basis of them?", "answers": "['Countries with large national budget deficits tend to restrict foreign trade.', 'Reliable comparisons of the deficit figures of one country with those of another are impossible.', \"Reducing a country's national budget deficit will not necessarily result in a lowering of any trade deficit that country may have.\", 'Countries with the largest trade deficits never have similarly large national budget deficits.']", "label": 2 }, { "id": "train_3374", "context": "The school principal insisted that student failures are caused by bad teaching. In a relatively short time failing grades disappeared from the school. The principal happily recognized this as evidence that the teaching had improved at the school.", "question": "The flawed pattern of reasoning in the above is most similar to that in which one of the following?", "answers": "['The nutritionist insisted that the weight gain that team members complained of was caused by overeating. In a brief time all the members stopped overeating. The nutritionist was pleased to conclude that they had stopped gaining weight.', 'The nutritionist insisted that the weight gain that team members complained of was caused by their thinking of food too often. The nutritionist was happy to conclude that the weight gain had stopped once the team members reported that they had stopped thinking of food so often.', 'The manager insisted that the workers who filed complaints did not have enough to do. Soon there were no more complaints filed. The manager was pleased to conclude that the workers were now productively filling their time.', 'The nutritionist insisted that the weight gain that team members complained of was merely in their imagination. Members were given weight charts for the last three months. The nutritionist was pleased to conclude that the complaints of weight gain had stopped.']", "label": 2 }, { "id": "train_3375", "context": "Consumer advocate: Tropical oils are high in saturated fats, which increase the risk of heart disease. Fortunately, in most prepared food tropical oils can be replaced by healthier alternatives without noticeably affecting taste. Ttherefore, intensive publicity about the disadvantage of tropical oils will be likely to result in dietary changes that will diminish many people' s risk of developing heart disease. Nutritionist: The major sources of saturated fat in the average North American diet are meat, poultry, and dairy products, not tropical oils. Thus, focusing attention on the health hazards of tropical oils would be counterproductive, because it would encourage people to believe that more substantial dietary changes are unnecessary.", "question": "Which one of the following is a point at issue between the nutritionist and the consumer advocate?", "answers": "['whether a diet that regularly includes large quantities of tropical oil can increase the risk of heart disease', \"whether intensive publicity campaigns can be effective as a means of changing people's eating habits\", 'whether conducting a publicity campaign that, by focusing on the health hazards of tropical oils, persuades people to replace such oils with healthier alternatives is a good public-health strategy', 'whether more people in North America would benefit from reducing the amount of meat they consume than would benefit from eliminating tropical oils from their diets']", "label": 2 }, { "id": "train_3376", "context": "Students in a college ethics class were asked to judge whether two magazines had been morally delinquent in publishing a particular classified advertisement that was highly offensive in its demeaning portrayal of some people. They were told only that the first magazine had undertaken to screen all classified advertisements and reject for publication those it found offensive, whereas the second magazine' s policy was to publish any advertisement received from its subscribers. Most students judged the first magazine, but not the second, to have been morally delinquent in publishing the advertisement.", "question": "Which one of the following principles, if established, provides the strongest justification for the judgment that the first magazine and not the second was morally delinquent?", "answers": "['If two similar agents commit two similar actions, those agents should be held to the same standard of accountability.', 'Failure to uphold a moral standard is not necessarily a moral failing except for those who have specifically committed themselves to upholding that standard.', \"A magazine should not be considered at fault for publishing a classified advertisement if that advertisement would not be offensive to any of the magazine's subscribers.\", 'Anyone regularly transmitting messages to the public has a moral responsibility to monitor the content of those messages.']", "label": 1 }, { "id": "train_3377", "context": "Principle: A police officer is eligible for a Mayor' s Commendation if the officer has an exemplary record, but not otherwise; an officer eligible for the award who did something this year that exceeded what could be reasonably expected of a police officer should receive the award if the act saved someone' s life. Conclusion: Officer Franklin should receive a Mayor' s Commendation but Officer Penn should not.", "question": "From which one of the following sets of facts can the conclusion be properly drawn using the principle?", "answers": "['In saving a child from drowning this year, Franklin and Penn both risked their lives beyond what could be reasonably expected of a police officer. Franklin has an exemplary record but Penn does not.', \"Both Franklin and Penn have exemplary records. On several occasions this year Franklin has saved people's lives, and on many occasions this year Franklin has exceeded what could be reasonably expected of a police officer. On no occasions this year has Penn saved a person's life or exceeded what could be reasonably expected of an officer.\", \"Neither Franklin nor Penn has an exemplary record. But, in saving the life of an accident victim, Franklin went beyond what could be reasonably expected of a police officer. In the only case in which Penn saved someone's life this year, Penn was merely doing what could be reasonably expected of an officer under the circumstances.\", \"At least once this year, Franklin has saved a person's life in such a way as to exceed what could be reasonably expected of a police officer. Penn has not saved anyone's life this year.\"]", "label": 0 }, { "id": "train_3378", "context": "Since there is no survival value in an animal' s having an organ that is able to function when all its other organs have broken down to such a degree that the animal dies, it is a result of the efficiency of natural selection that no organ is likely to evolve in such a way that it greatly outlasts the body' s other organs.", "question": "Of the following, which one illustrates a principle that is most similar to the principle illustrated by the passage?", "answers": "['One car model produced by an automobile manufacturer has a life expectancy that is so much longer than its other models that its great popularity requires the manufacturer to stop producing some of the other models.', 'A store in a lower-income neighborhood finds that it is unable to sell its higher-priced goods and so stocks them only when ordered by a customer.', 'Automotive engineers find that it is not cost- effective to manufacture a given automobile part of such high quality that it outlasts all other parts of the automobile, as doing so would not raise the overall quality of the automobile.', 'Athletes occasionally overdevelop some parts of their bodies to such a great extent that other parts of their bodies are more prone to injury as a result.']", "label": 2 }, { "id": "train_3379", "context": "Efforts to get the public to exercise regularly, which have emphasized the positive health effects of exercise rather than the dangers of a sedentary lifestyle, have met with little success. In contrast, efforts to curb cigarette smoking, which have emphasized the dangers of smoking rather than the positive health effects of quitting, have been highly successful. Thus, efforts to get the public to exercise regularly would be more successful if they emphasized the dangers of a sedentary lifestyle rather than the positive health effects of exercise.", "question": "Which one of the following is an assumption on which the argument relies?", "answers": "['The health risks associated with a sedentary lifestyle are as great as those associated with smoking.', 'Efforts to curb cigarette smoking would not be more successful if they emphasized the positive health effects of quitting rather than the dangers of smoking.', 'The majority of people who successfully quit smoking cite health concerns as the primary motivation for their success.', 'Although most smokers are aware of the dangers of smoking, few are aware of the positive health effects of quitting.']", "label": 1 }, { "id": "train_3380", "context": "Studies have shown that photosynthesis, the process by which plants manufacture life-sustaining proteins from sunlight and carbon, is actually intensified if the level of carbon dioxide in the atmosphere is increased. Since carbon dioxide levels are increased by the burning of fossil fuels and by other human industrial activities, it is obvious that these industrial activities are purely beneficial to agriculture and those of us who depend upon it.", "question": "The flawed reasoning in the argument above is most similar to that in which one of the following?", "answers": "['Consistently consuming more calories than one expends inevitably leads to excessive weight gain, so if one wishes to avoid the health problems associated with this condition, one ought to fast periodically.', 'Because a high fiber diet has been shown to be more healthful than a low fiber diet, a diet in which foods with a low fiber content have been entirely replaced by foods with a high fiber content is bound to be even more healthful.', 'It has been shown that one can obtain more vitamins and minerals from fresh fruits and vegetables than from processed fruits and vegetables. One ought, ttherefore, to completely abandon consumption of the latter in favor of the former.', 'Because exercise has been shown to prevent a number of diseases, injuries, and other human ills, clearly no harm, and a lot of good, can come from exercise.']", "label": 3 }, { "id": "train_3381", "context": "Ethicist: A society is just when, and only when, first, each person has an equal right to basic liberties, and second, inequalities in the distribution of income and wealth are not tolerated unless these inequalities are to everyone' s advantage and are attached to jobs open to everyone.", "question": "Which one of the following judgments most closely conforms to the principle described above?", "answers": "['Society S allows inequalities in the distribution of income and wealth, although everyone benefits, and these inequalities are attached to jobs that are open to everyone. Thus, society S is just.', 'Society S guarantees everyone an equal right to basic liberties, while allowing inequalities in the distribution of income and wealth that are to the advantage of everyone. Further, the jobs to which these inequalities are attached are open to most people. Thus, society S is just.', 'Society S distributes income and wealth to everyone equally, but at the expense of creating inequalities in the right to basic liberties. Thus, society S is not just.', 'Society S gives everyone an equal right to basic liberties, but at the expense of creating inequalities in the distribution of income and wealth. Thus, society S is not just.']", "label": 2 }, { "id": "train_3382", "context": "Moralist: TV talk shows are contributing to the moral decline in our country. By constantly being shown the least moral people in our society, viewers begin to think that such people are the norm, and that there is something wrong with being morally upright. TV talk show host: Well, if there is such a decline, it' s not because of TV talk shows: we simply show people what they want to see. What can be wrong with letting the viewers decide? Furthermore, if restrictions were put on my show, that would amount to censorship, which is wrong.", "question": "The moralist's and the TV talk show host's statements provide the most support for holding that they disagree about whether", "answers": "['TV talk shows should be censored', \"people's moral standards have changed\", 'it is wrong not to let the viewers decide what they want to see', 'TV talk shows, by presenting immoral guests, are causing a moral decline']", "label": 3 }, { "id": "train_3383", "context": "Historian: The early Egyptian pharaohs spent as much wealth on largely ceremonial and hugely impressive architecture as they did on roads and irrigation systems. This was not mere frivolousness, however, for if people under a pharaoh' s rule could be made to realize the extent of their ruler' s mastery of the physical world, their loyalty could be maintained without military coercion.", "question": "The claim that early Egyptian expenditure on largely ceremonial architecture was not frivolous plays which one of the following roles in the historian's argument?", "answers": "[\"It is a conclusion purportedly justified by the argument's appeal to the psychological effects of these structures on the Egyptian population.\", \"It is a premise given in support of the claim that the loyalty of people under a pharaoh's rule was maintained over time without reliance on military force.\", 'It is offered as an illustration of the principle that social and political stability do not depend ultimately on force.', 'It is offered in support of the claim that Egyptian pharaohs spent as much on ceremonial architecture as they did on roads and irrigation systems.']", "label": 0 }, { "id": "train_3384", "context": "Most successful entrepreneurs work at least 18 hours a day, and no one who works at least 18 hours a day has time for leisure activities. But all happy entrepreneurs have time for leisure activities.", "question": "If the statements above are true, each of the following could be true EXCEPT:", "answers": "['Some entrepreneurs who work at least 18 hours a day are happy.', 'Some entrepreneurs who work at least 18 hours a day are successful.', 'Anyone who has no time for leisure activities works at least 18 hours a day.', 'Some happy entrepreneurs are successful.']", "label": 0 }, { "id": "train_3385", "context": "A person' s intake of refined sugar and of foods that break down into sugars during digestion is the dietary source of virtually all of the glucose (a type of sugar) that reaches that person' s bloodstream. Coffee, however, which itself does not break down into sugars during digestion, sometimes causes a person' s blood-glucose level to rise dramatically, even if the coffee is consumed without cream or any sweeteners.", "question": "Which of the following, if true, best helps to explain coffee's effect on blood-glucose levels as it is described above?", "answers": "['People with sedentary office jobs are more likely to drink large amounts of coffee and to have higher blood-glucose levels than are people with jobs requiring constant physical activity.', 'People often drink coffee after a meal consisting of several types of foods that themselves rapidly break down into sugars during digestion.', 'For many people, the consumption of one piece of chocolate cake has the same stimulating effect as one cup of plain coffee.', \"Drinking more than two cups of coffee in an hour increases a person's level of stress, and the body reacts to stress by releasing stored glucose into the blood.\"]", "label": 3 }, { "id": "train_3386", "context": "Critic: To be a literary classic a book must reveal something significant about the human condition. Furthermore, nothing that is unworthy of serious study reveals anything significant about the human condition.", "question": "If the critic's statements are true, which one of the following must also be true?", "answers": "['A book is a literary classic only if it is worthy of serious study.', 'Some books worthy of serious study do not reveal anything significant about the human condition.', 'Any book worthy of serious study is a literary classic.', 'Some books that reveal something significant about the human condition are not literary classics.']", "label": 0 }, { "id": "train_3387", "context": "Editorialist: Many professional musicians claim that unauthorized music-sharing services, which allow listeners to obtain music for free, rob musicians of royalties. While it is true that musicians are deprived of royalties they deserve, music-sharing services are not to blame since record companies, publishers, managers, and other intermediaries take an inequitably large cut of the revenues from music sales.", "question": "The reasoning in the editorialist's argument is most vulnerable to criticism on the grounds that the argument", "answers": "['attacks a position based solely on the character of the people who hold that position', 'attempts to promote a particular behavior simply by showing that many people engage in that behavior', 'treats a necessary condition for blameworthiness as though it were a sufficient condition for blameworthiness', 'concludes that one party is not blameworthy merely because another party is blameworthy']", "label": 3 }, { "id": "train_3388", "context": "Sambar deer are physically incapable of digesting meat. Yet sambar deer have been reported feeding on box turtles after killing them.", "question": "Which one of the following, if true, best resolves the discrepancy above?", "answers": "['Sambar deer kill box turtles only occasionally.', 'Box turtles are much slower and clumsier than are sambar deer.', 'Sambar deer eat only the bony shells of box turtles.', 'Box turtles sometimes compete with sambar deer for food.']", "label": 2 }, { "id": "train_3389", "context": "Calories consumed in excess of those with which the body needs to be provided to maintain its weight are normally stored as fat and the body gains weight. Alcoholic beverages are laden with calories. However, those people who regularly drink two or three alcoholic beverages a day and thereby exceed the caloric intake necessary to maintain their weight do not in general gain weight.", "question": "Which one of the following, if true, most helps to resolve the apparent discrepancy?", "answers": "['Some people who take in fewer calories than are normally necessary to maintain their weight do not lose weight.', 'Some people who regularly drink two or three alcoholic beverages a day avoid exceeding the caloric intake necessary to maintain their weight by decreasing caloric intake from other sources.', 'Many people who regularly drink more than three alcoholic beverages a day do not gain weight.', 'Excess calories consumed by people who regularly drink two or three alcoholic beverages a day tend to be dissipated as heat.']", "label": 3 }, { "id": "train_3390", "context": "Although the discount stores in Goreville' s central shopping district are expected to close within five years as a result of competition from a SpendLess discount department store that just opened, those locations will not stay vacant for long. In the five years since the opening of Colson' s, a nondiscount department store, a new store has opened at the location of every store in the shopping district that closed because it could not compete with Colson' s.", "question": "Which of the following, if true, most seriously weakens the argument?", "answers": "['At present, the central shopping district has as many stores operating in it as it ever had.', \"Over the course of the next five years, it is expected that Goreville's population will grow at a faster rate than it has for the past several decades.\", \"Increasingly, the stores that have opened in the central shopping district since Colson's opened have been discount stores.\", \"Many stores in the central shopping district sell types of merchandise that are not available at either SpendLess or Colson's.\"]", "label": 2 }, { "id": "train_3391", "context": "A recent magazine article argued that most companies that do not already own videoconferencing equipment would be wasting their money if they purchased it. However, this is clearly not true. In a recent survey of businesses that have purchased such equipment, most of the respondents stated that the videoconferencing equipment was well worth its cost.", "question": "The reasoning in the argument is flawed in that the argument", "answers": "['concludes that something is worth its cost merely on the grounds that many businesses have purchased it', 'takes a condition sufficient to justify purchasing costly equipment to be necessary in order for the cost of the purchase to be justified', 'confuses the cost of an item with its value to the purchaser', 'relies on a sample that it is reasonable to suppose is unrepresentative of the group about which it draws its conclusion']", "label": 3 }, { "id": "train_3392", "context": "Airport official: Local residents have been complaining that night flights into Plainsville airport disturb their sleep and should be sharply reduced in number. This complaint is completely unreasonable-there have been night flights coming into the airport from the very beginning, twenty years ago, and these residents should have taken that fact into account when buying their homes.", "question": "Which of the following is an assumption on which the airport argument depends?", "answers": "['The residents who are complaining have been in their current homes fewer than twenty years.', 'There are fewer night flights now than there were originally.', 'The economic success of the airport depends entirely on the existence of the night flights.', 'The residents who are complaining are ignoring the benefits they gain from the presence of the airport.']", "label": 0 }, { "id": "train_3393", "context": "A computer game publisher has recently released its latest adventure game. The game' s inventive puzzles and compelling plot induce even casual players to become preoccupied with completing it. The game can be purchased from retail outlets or rented for two-day intervals. The publisher offers a rebate equal to the cost of one rental for renters who go on to purchase the game, saving them a significant portion of the purchase price. Since the rate of sales now meets expectations and rentals are exceeding expectations, the publisher predicts that soon sales of the game will also exceed expectations.", "question": "Which one of the following, if true, most helps to justify the publisher's prediction?", "answers": "[\"The publisher's games are among the most popular computer games on the market.\", 'It takes several weeks for most players to complete the game.', 'The game can be purchased directly from the publisher as well as from retailers.', 'Most people who complete the game do not play it extensively afterward.']", "label": 1 }, { "id": "train_3394", "context": "Dana: It is wrong to think that the same educational methods should be used with all children. Many children have been raised in more communal environments than others and would ttherefore learn better through group, rather than individual, activities. A child' s accustomed style of learning should always dictate what method is used. Pat: No, not always. The flexibility of being able to work either on one' s own or in a group is invaluable in a world where both skills are in demand.", "question": "The conversation lends the most support to the claim that Dana and Pat disagree on which one of the following?", "answers": "['Many children would learn better through group, rather than individual, activities.', 'It is sometimes desirable to tailor educational methods to the way a child learns best.', 'All children can learn valuable skills from individual activities.', 'All children should learn to adapt to various educational methods.']", "label": 3 }, { "id": "train_3395", "context": "Less than 10 percent of the world's people, and of the participants in combat sports worldwide, are left-handed. However, a majority of top-level competitors in combat sports worldwide are left-handed.", "question": "Which of the following, if true, most helps to resolve the discrepancy presented in the passage?", "answers": "['A significant number of left-handed competitors in combat sports have trained under coaches who have forced them to fight from right-handed positions and angles.', 'Left-handedness is more common in primitive cultures with high rates of death by violence than in other primitive cultures.', 'Because left-handedness is so rare, opponents of left-handed fighters are ill equipped to handle the positions and angles from which those fighters attack.', 'Among top-level competitors in many non-combat sports, such as tennis and baseball, left-handedness is also much more common than in the general population.']", "label": 2 }, { "id": "train_3396", "context": "Premature babies who receive regular massages are more active than premature babies who do not. Even when all the babies drink the same amount of milk, the massaged babies gain more weight than do the unmassaged babies. This is puzzling because a more active person generally requires a greater food intake to maintain or gain weight.", "question": "Which of the following, if true, best reconciles the apparent discrepancy described above?", "answers": "['Massage does not increase the growth rate of babies over one year old, if the babies had not been previously massaged.', 'Increased activity causes the intestines of premature babies to mature more quickly, enabling the babies to digest and absorb more of the nutrients in the milk they drink.', 'Increased activity leads to increased levels of hunger, especially when food intake is not also increased.', \"Massage increases premature babies' curiosity about their environment, and curiosity leads to increased activity.\"]", "label": 1 }, { "id": "train_3397", "context": "Public-sector (government-owned) companies are often unprofitable and a drain on the taxpayer. Such enterprises should be sold to the private sector, where competition will force them either to be efficient and profitable or else to close.", "question": "Which of the following, if true, identifies a flaw in the policy proposed above?", "answers": "['By buying a public-sector company and then closing the company and selling its assets, a buyer can often make a profit.', 'The costs of environmental protection, contributions to social programs, and job-safety measures are the same in the public and private sectors.', 'The services provided by many public-sector companies must be made available to citizens, even when a price that covers costs cannot be charged.', 'Some unprofitable private-sector companies have become profitable after being taken over by the government to prevent their closing.']", "label": 2 }, { "id": "train_3398", "context": "Prosecutor: Dr. Yuge has testified that, had the robbery occurred after 1: 50 A. M. , then, the moon having set at 1: 45 A. M. , it would have been too dark for Klein to recognize the perpetrator. But Yuge acknowledged that the moon was full enough to provide considerable light before it set. And we have conclusively shown that the robbery occurred between 1: 15 and 1: 30 A. M. So there was enough light for Klein to make a reliable identification.", "question": "The prosecutor's reasoning is most vulnerable to criticism because it overlooks which one of the following possibilities?", "answers": "['Without having been there, Dr. Yuge has no way of knowing whether the light was sufficient.', \"During the robbery the moon's light may have been interfered with by conditions such as cloud cover.\", 'Klein may be mistaken about the time of the robbery and so it may have taken place after the moon had set.', 'Klein may have been too upset to make a reliable identification even in good light.']", "label": 1 }, { "id": "train_3399", "context": "Psychologists have found that the implementation of policies allowing work schedules to be tailored to individuals' needs does not typically increase managers' job satisfaction or their efficiency -- although this may be because most managers already have the autonomy to adjust their own schedules. But these flexible-schedule policies do increase job satisfaction, productivity, and attendance among nonmanagerial employees. The benefits dissipate somewhat over time, however, and they are reduced even further if schedules are too elastic.", "question": "Which one of the following statements is most supported by the information above?", "answers": "[\"Flexible-schedule policies should be expected to improve the morale of some individual employees but not the overall morale of a company's workforce.\", 'Implementing flexible schedules would be an effective means of increasing the job satisfaction and efficiency of managers who do not already have scheduling autonomy.', 'The typical benefits of flexible-schedule policies cannot be reliably inferred from observations of the effects of such policies on managers.', \"Flexible schedules should be expected to substantially improve a company's productivity and employee satisfaction in the long run.\"]", "label": 2 }, { "id": "train_3400", "context": "When chimpanzees become angry at other chimpanzees, they often engage in what primatologists call \"threat gestures\": grunting, spitting, or making abrupt, upsweeping arm movements. Chimpanzees also sometimes attack other chimpanzees out of anger. However, when they do attack, they almost never take time to make threat gestures first. And, conversely, threat gestures are rarely followed by physical attacks.", "question": "Which one of the following, if true, most helps to explain the information about how often threat gestures are accompanied by physical attacks?", "answers": "[\"Chimpanzees often respond to other chimpanzees' threat gestures with threat gestures of their own.\", 'The chimpanzees that most often make threat gestures are the ones that least often initiate physical attacks.', 'Making threat gestures helps chimpanzees vent aggressive feelings and thereby avoid physical aggression.', 'Threat gestures and physical attacks are not the only means by which chimpanzees display aggression.']", "label": 2 }, { "id": "train_3401", "context": "Editorial in Krenlandian Newspaper: Krenland' s steelmakers are losing domestic sales because of lower-priced imports, in many cases because foreign governments subsidize their steel industries in ways that are banned by international treaties. But whatever the cause, the cost is ultimately going to be jobs in Krenland' s steel industry. Ttherefore, it would protect not only steel companies but also industrial employment in Krenland if our government took measures to reduce cheap steel imports.", "question": "Which of the following, if true, most seriously weakens the editorial's argument?", "answers": "['Because of advances in order-taking, shipping, and inventory systems, the cost of shipping steel from foreign producers to Krenland has fallen considerably in recent years.', \"Because steel from Krenland is rarely competitive in international markets, only a very small portion of Krenlandian steelmakers' revenue comes from exports.\", 'For many Krenlandian manufacturers who face severe international competition in both domestic and export markets, steel constitutes a significant part of their raw material costs.', 'The international treaties that some governments are violating by giving subsidies to steelmakers do not specify any penalties for such violations.']", "label": 2 }, { "id": "train_3402", "context": "Recent unexpectedly heavy rainfalls in the metropolitan area have filled the reservoirs and streams; water rationing, ttherefore, will not be necessary this summer.", "question": "Which one of the following, if true, most undermines the author's prediction?", "answers": "['Water rationing was imposed in the city in three of the last five years.', 'In most years the city receives less total precipitation in the summer than it receives in any other season.', \"The water company's capacity to pump water to customers has not kept up with the increased demand created by population growth in the metropolitan area.\", 'The long-range weather forecast predicts lower-than-average temperatures for this summer.']", "label": 2 }, { "id": "train_3403", "context": "Zoos provide a space for the public to view wild animals, allowing people to forge a stronger relationship with nature. In addition, zoos save endangered species through breeding programs. A combination of public funds and private donations funds most zoos. More tigers now live in Texas than in the wild, as state law allows tigers to be kept as pets.", "question": "The author would most likely agree with which one of the following?", "answers": "['The fact that more tigers live in Texas than the in the wild is a positive development.', 'The government should provide more funding for zoos.', 'All wild animals should be held in captivity.', 'Wild animals should sometimes be held in captivity.']", "label": 3 }, { "id": "train_3404", "context": "The symptoms of mental disorders are behavioral, cognitive, or emotional problems. Some patients with mental disorders can be effectively treated with psychotherapy. But it is now known that in some patients mental disorders result from chemical imbalances affecting the brain. Thus these patients can be effectively treated only with medication that will reduce or correct the imbalance.", "question": "The argument depends on assuming which one of the following?", "answers": "['Treatment with medication always shows faster results for patients with mental disorders than does treatment with psychotherapy.', 'Treatment with psychotherapy has no effect on mental disorders other than a reduction of the symptoms.', 'Treatment by psychotherapy can produce no effective reduction in or correction of chemical imbalances that cause mental disorders.', 'Most mental disorders are not the result of chemical imbalances affecting the brain.']", "label": 2 }, { "id": "train_3405", "context": "Linguist: Three of the four subfamilies of the so-called \"Austronesian\" languages are found only among indigenous peoples in Taiwan, whereas the fourth is found on islands over a huge area stretching from Madagascar to the eastern Pacific Ocean. Since these subfamilies all originated in the same language, which must have been originally spoken in a single geographic location, these facts suggest that Taiwan is the homeland where Austronesian languages have been spoken longest and, hence, that Austronesian-speaking peoples originated in Taiwan and later migrated to other islands.", "question": "Which one of the following most accurately expresses the overall conclusion drawn in the linguist's argument?", "answers": "['The Austronesian family of languages has four subfamilies, three of which are found only among indigenous peoples in Taiwan.', 'Austronesian-speaking peoples probably originated in Taiwan and later migrated to other islands.', 'Austronesian-speaking peoples originated in the homeland where Austronesian languages have been spoken longest.', 'Taiwan is probably the homeland where Austronesian languages have been spoken longest.']", "label": 1 }, { "id": "train_3406", "context": "Medical researcher: A survey of more than 1 million adults found that there was a greater frequency of illness among people who regularly slept at least 8 hours a night than among people who slept significantly less. This shows that mild sleep deprivation is not unhealthy and, in fact, probably bolsters the body' s defenses against illness.", "question": "The reasoning in the medical researcher's argument is most vulnerable to criticism on the grounds that the argument", "answers": "['concludes, from the claim that a certain phenomenon occurs and the claim that a certain condition is sufficient for that phenomenon to occur, that the condition also exists', 'takes for granted that there will be an observable correlation between two phenomena if either of those phenomena causally contributes to the other', 'fails to consider that even if a specific negative consequence is not associated with a given phenomenon, that phenomenon may have other negative consequences', 'fails to address the possibility that an observed correlation between two phenomena is due to another factor that causally contributes to both phenomena']", "label": 3 }, { "id": "train_3407", "context": "All Labrador retrievers bark a great deal. All Saint Bernards bark infrequently. Each of Rani' s dogs is a cross between a Labrador retriever and a Saint Bernard. Ttherefore, Rani' s dogs are moderate barkers.", "question": "Which one of the following uses flawed reasoning that most closely resembles the flawed reasoning used in the argument above?", "answers": "['All transcriptionists know shorthand. All engineers know calculus. Bala has worked both as a transcriptionist and as an engineer. Ttherefore, Bala knows both shorthand and calculus.', \"All of Karuna's dresses are very well made. All of Chitra's dresses are very badly made. Half of the dresses in this closet are very well made, and half of them are very badly made. Ttherefore, half of the dresses in this closet are Karuna's and half of them are Chitra's.\", 'All students who study diligently make good grades. But some students who do not study diligently also make good grades. Jyoti studies somewhat diligently. Ttherefore, Jyoti makes somewhat good grades.', 'All type A chemicals are extremely toxic to human beings. All type B chemicals are nontoxic to human beings. This household cleaner is a mixture of a type A chemical and a type B chemical. Ttherefore, this household cleaner is moderately toxic.']", "label": 3 }, { "id": "train_3408", "context": "Legislator: My colleague says we should reject this act because it would deter investment. But because in the past she voted for legislation that inhibited investment, this surely is not the real reason she opposes the act. Since she has not revealed her real reason, it must not be very persuasive. So we should vote to approve the act.", "question": "The reasoning in the legislator 's argument is most vulnerable to the criticism that the argument", "answers": "[\"presumes, without providing justification, that the colleague's opposition to the act is the minority position in the legislature\", \"fails to consider that the colleague's opposition to the act may be a response to constituents' wishes\", 'treats a personal character trait as if it were evidence of the professional viewpoint of the person having that trait', 'fails to address the grounds on which the colleague claims the act should be rejected']", "label": 3 }, { "id": "train_3409", "context": "Mall manager: By congregating in large groups near the stores in our mall, teenagers create an atmosphere in which many adult shoppers feel uncomfortable. As a result, the adults have begun to spend less time shopping than they have in the past. The mall' s goal in this situation is to prevent a significant loss in overall sales, so merchants should do their utmost to discourage teenagers from congregating near stores. Merchant: But the amount spent by teenagers who congregate near mall stores constitutes a significant percentage of the total amount spent in those stores.", "question": "The merchant's response to the manager's argument is most accurately described as", "answers": "[\"contending that the manager's recommendation is sound but for reasons other than those given by the manager\", \"suggesting that the mall's goal is an undesirable one\", \"giving information that pertains to the relation between the manager's recommendation and the mall's goal\", 'using the information cited by the manager to make an additional recommendation that would help achieve the goal']", "label": 2 }, { "id": "train_3410", "context": "P: Because an elected official needs the support of a political party to be effective, the independent candidate for the legislature cannot possibly be an effective legislator is she wins. Q: I disagree. By your reasoning, our current legislator, who has the support of a political party, ought to have been effective, but he has not been.", "question": "Which one of the following is the best criticism of Q's statement?", "answers": "[\"It mistakenly interprets P to be claiming that a factor assures, rather than is necessary for, a legislator's effectiveness.\", \"It simply contradicts P's claim without offering evidence against it.\", 'It does not consider the possibility that a political party might decide to support an elected legislator even though he or she ran as an independent.', 'It fails to prove a precise definition for a key term -- the word \"effective. \"']", "label": 0 }, { "id": "train_3411", "context": "Limited research indicates that therapeutic intervention before the onset of mental disorders can mitigate factors identified as major contributors to them. But a much more comprehensive research program is needed to verify these results and allow for the design of specific health care measures. Thus, in order to explore a potential means of cost-effectively helping people prone to mental disorders, we should increase funding for intervention research.", "question": "Which one of the following, if true, most strengthens the argument?", "answers": "['Once a mental disorder disappears, there is a fair chance that it will recur, given that complete cures are rare.', 'Most minor mental disorders are more expensive to treat than other minor health problems.', 'Prevention research can be coordinated by drawing together geneticists, neurologists, and behavioral scientists.', 'Reducing known risk factors for mental disorders is relatively inexpensive compared to the long-term treatment required.']", "label": 3 }, { "id": "train_3412", "context": "Anthropologist: It was formerly believed that prehistoric Homo sapiens ancestors of contemporary humans interbred with Neanderthals, but DNA testing of a Neanderthal' s remains indicates that this is not the case. The DNA of contemporary humans is significantly different from that of the Neanderthal.", "question": "Which one of the following is an assumption required by the anthropologist's argument?", "answers": "['Any similarity in the DNA of two species must be the result of interbreeding.', 'The DNA of prehistoric Homo sapiens ancestors of contemporary humans was not significantly more similar to that of Neanderthals than is the DNA of contemporary humans.', 'At least some Neanderthals lived at the same time and in the same places as prehistoric Homo sapiens ancestors of contemporary humans.', 'DNA testing of remains is significantly less reliable than DNA testing of samples from living species.']", "label": 1 }, { "id": "train_3413", "context": "A car' s antitheft alarm that sounds in the middle of the night in a crowded city neighborhood may stop an attempted car theft. On the other hand, the alarm might signal only a fault in the device, or a response to some harmless contact, such as a tree branch brushing the car. But whatever the cause, the sleep of many people in the neighborhood is disturbed. Out of consideration for others, people who have these antitheft alarms on their cars should deactivate them when they park in crowded city neighborhoods at night.", "question": "Which one of the following, if assumed by the author of the passage, would allow her properly to draw her conclusion that the owners of alarm-equipped cars should deactivate the alarms when parking in crowded city neighborhoods at night?", "answers": "['The inconvenience of false alarms is a small price to pay for the security of a neighborhood.', 'In most cases when a car alarm sounds at night, it is a false alarm.', 'Allowing the residents of a crowded city neighborhood to sleep undisturbed is more important than preventing car theft.', 'People who equip their cars with antitheft alarms are generally inconsiderate of others.']", "label": 2 }, { "id": "train_3414", "context": "Many industrialized nations are trying to reduce atmospheric concentrations of carbon dioxide, a gas released by the burning of fossil fuels. One proposal is to replace conventional cement, which is made with calcium carbonate, by a new \"eco-cement. \" This new cement, made with magnesium carbonate, absorbs large amount of carbon dioxide when exposed to the atmosphere. Ttherefore, using eco-cement for new concrete building projects will significantly help reduce atmospheric concentrations of carbon dioxide.", "question": "Which of the following, if true, most strengthens the argument?", "answers": "['Eco-cement is strengthened when absorbed carbon dioxide reacts with the cement.', 'The cost of magnesium carbonate, currently greater than the cost of calcium carbonate, probably will fall as more magnesium carbonate is used in cement manufacture.', 'The manufacture of eco-cement uses considerably less fossil fuel per unit of cement than the manufacture of conventional cement does.', 'Most building-industry groups are unaware of the development or vailability of eco-cement.']", "label": 2 }, { "id": "train_3415", "context": "In practice the government will have the last word on what an individual' s rights are, because its police will do what its officials and courts say. But that does not mean that the government' s view is necessarily the correct view; anyone who thinks it is must believe that persons have only such moral rights as the government chooses to grant, which means that they have no moral rights at all.", "question": "Which one of the following most accurately expresses the conclusion of the argument?", "answers": "[\"One should always try to uphold one's individual rights against the government's view of what those rights are.\", \"The police always agree with government officials and the courts about what an individual's rights are.\", 'Individuals have no rights at all unless the government says that they do.', \"What government officials and courts say an individual's rights are may not be correct.\"]", "label": 3 }, { "id": "train_3416", "context": "A recent study suggests that Alzheimer' s disease, which attacks the human brain, may be caused by a virus. In the study, blood from 11 volunteers, each of whom had the disease, was injected into rats. The rats eventually exhibited symptoms of another degenerative neurological disorder, Creutzfeldt-Jakob disease, which is caused by a virus. This led the scientist who conducted the study to conclude that Alzheimer' s disease might be caused by a virus.", "question": "Which one of the following statements, if true, would most strengthen the scientist's hypothesis that Alzheimer's disease is caused by a virus?", "answers": "[\"Creutzfeldt-Jakob disease affects only motor nerves in rats' limbs, not their brains.\", 'The virus that causes Creutzfeldt-Jakob disease in rats has no effect on humans.', \"The symptoms known, respectively, as Creutzfeldt-Jakob disease and Alzheimer's disease are different manifestations of the same disease.\", \"Alzheimer's disease in rats is not caused by a virus.\"]", "label": 2 }, { "id": "train_3417", "context": "Political Advertisement: Mayor Delmont' s critics complain about the jobs that were lost in the city under Delmont' s leadership. Yet the fact is that not only were more jobs created than were eliminated, but each year since Delmont took office the average pay for the new jobs created has been higher than that year' s average pay for jobs citywide. So it stands to reason that throughout Delmont' s tenure the average paycheck in this city has been getting steadily bigger.", "question": "Which of the following, if true, most seriously weakens the argument in the advertisement?", "answers": "['The average pay for jobs in the city was at a ten-year low when Mayor Delmont took office.', \"Each year during Mayor Delmont's tenure, the average pay for jobs that were eliminated has been higher than the average pay for jobs citywide.\", 'The unemployment rate in the city is higher today than it was when Mayor Delmont took office.', \"Most of the jobs eliminated during Mayor Delmont's tenure were in declining industries.\"]", "label": 1 }, { "id": "train_3418", "context": "A year ago several regional hospitals attempted to reduce the number of patient injuries resulting from staff errors by implementing a plan to systematically record all such errors. The incidence of these injuries has substantially decreased at these hospitals since then. Clearly, the knowledge that their errors were being carefully monitored made the hospitals' staffs much more meticulous in carrying out their patient-care duties.", "question": "Which one of the following, if true, most strengthens the argument?", "answers": "['The incidence of patient injuries at a regional hospital that did not participate in the plan also decreased over the year in question.', 'The decrease in the incidence of the injuries did not begin at any hospital until the staff there became aware that the records were being closely analyzed.', 'The plan did not call for the recording of staff errors that could have caused patient injuries but did not.', \"Under the plan, the hospitals' staff members who were found to have made errors that caused injuries to patients received only reprimands for their first errors.\"]", "label": 1 }, { "id": "train_3419", "context": "Theorist: Hatred and anger, grief and despair, love and joy are pairs of emotions that consist of the same core feeling and are distinguishable from each other only in terms of the social conditions that cause them and the behavior they in turn cause. So even if the meaning of a given piece of music is the emotion it elicits, this can mean only that music produces the core of a given emotion, for music is merely sound and, ttherefore, by itself creates neither social conditions nor human behavior.", "question": "The claim that music is merely sound plays which one of the following roles in the theorist's argument?", "answers": "['It is a generalization a particular instance of which is cited by the argument in order to undermine the viewpoint that the argument is attacking.', \"It is a claim that is offered as partial support for the argument's conclusion.\", 'It is a portion of the conclusion drawn in the argument.', 'It is a hypothesis that must be rejected, according to the argument, because it is inconsistent with certain evidence.']", "label": 1 }, { "id": "train_3420", "context": "Some people believe that advertising is socially pernicious -- it changes consumers' preferences, thereby manipulating people into wanting things they would not otherwise want. However, classes in music and art appreciation change people' s preferences for various forms of art and music, and there is nothing wrong with these classes. Ttherefore, __.", "question": "Which one of the following most logically completes the argument?", "answers": "[\"the fact that advertising changes consumers' preferences does not establish that it is bad\", \"if advertising changes consumers' preferences, it generally does so in a positive way\", \"the social perniciousness of advertising is not limited to its effect on people's preferences\", 'consumers would still want most of the things they want even if they were not advertised']", "label": 0 }, { "id": "train_3421", "context": "After receiving numerous complaints from residents about loud, highly amplified music played at local clubs, Middletown is considering a law that would prohibit clubs located in residential areas from employing musical groups that consist of more than three people.", "question": "The likelihood that the law would be effective in reducing noise would be most seriously diminished if which of the following were true?", "answers": "['Groups that consist of more than three musicians are usually more expensive for clubs to hire than are groups that consist of fewer than three musicians.', 'In towns that have passed similar laws, many clubs in residential areas have relocated to nonresidential areas.', 'Much of the music popular at the local clubs can be played only by groups of at least four musicians.', 'Amplified music played by fewer than three musicians generally is as loud as amplified music played by more than three musicians.']", "label": 3 }, { "id": "train_3422", "context": "False chicory' s taproot is always one half as long as the plant is tall. Furthermore, the more rain false chicory receives, the taller it tends to grow. In fact, false chicory plants that receive greater than twice the average rainfall of the species' usual habitat always reach above-average heights for false chicory.", "question": "If the statements above are true, then which one of the following must also be true?", "answers": [ "If a false chicory plant has a longer-than- average taproot, then it is likely to have received more than twice the average rainfall of the species'usual habitat.", "If two false chicory plants differ in height, then it is likely that the one with the shorter taproot has received less than twice the average rainfall of the species'usual habitat.", "It is not possible for a false chicory plant to receive only the average amount of rainfall of the species'usual habitat and be of above average height.", "If a false chicory plant receives greater than twice the average rainfall of the species'usual habitat, then it will have a longer taproot than that of an average-sized false chicory plant." ], "label": 3 }, { "id": "train_3423", "context": "Mysterious ancient tracks cut into limestone have recently been found on the island of Malta. The tracks wander, sometimes disappearing under modem structures. Their origin and purpose are unknown, but evidence indicates that they could have connected settlements or water sources. One archaeologist hypothesizes, based on the tracks' physical appearance and surroundings, that they were made in about 1000 B. C. by animal-drawn carts.", "question": "Which one of the following, if true, most helps to support the archaeologist's hypothesis mentioned above?", "answers": "['Some of the tracks connect areas that are sources of fresh water on Malta today.', 'Some terrain on the island of Malta is more easily traversed on foot than are certain other types of terrain there.', 'Historically, inhabitants of the island of Malta have not been innovative users of transportation technology.', 'Areas near the tracks have yielded relatively large amounts of fossilized animal excrement dating from approximately 1000 B. C.']", "label": 3 }, { "id": "train_3424", "context": "Ditalgame Corporation' s computer video games are subject to widespread illegal copying. To combat this piracy, Ditalgame will begin using a new copy protection feature on its games. Ditalgame' s president predicts a substantial increase in sales of the company' s games once the new copy protection feature is implemented.", "question": "Which one of the following, if true, provides the most support for the president's prediction?", "answers": "[\"Over the last several years, the market for computer games has grown steadily, but Ditalgame's share of that market has shrunk considerably.\", 'Game Review Monthly, the most commonly read magazine among people who frequently copy computer games, generally gives favorable reviews to Ditalgame games.', 'The copy protection feature causes a copied game to be playable just long enough for most people to come to enjoy it so much that they decide they have to have it.', 'Ditalgame has spent millions of dollars developing the new copy protection feature, and the company can recoup these costs only if its sales increase substantially.']", "label": 2 }, { "id": "train_3425", "context": "A survey of entrepreneurs who started companies last year shows that while virtually all did substantial preparatory research and planning, only half used that work to produce a formal business plan. Since, on average, the entrepreneurs without formal plans secured the capital they needed in half the time of those with plans, these survey results indicate that, in general, formal plans did not help the entrepreneurs who produced them to secure the capital they needed.", "question": "Which of the following, if true, most seriously weakens the argument?", "answers": "['Among the entrepreneurs surveyed, those who did not produce formal business plans sought and received a much larger proportion of their capital from investors with whom they had a long-standing business relationship.', 'The entrepreneurs surveyed who did not produce a formal business plan spent nearly as much time doing preparatory research and planning as the entrepreneurs who produced plans.', 'Companies started by entrepreneurs who had used formal business plans to attract investment were on the whole as profitable in their first year as were companies started by entrepreneurs who had not produced such plans.', 'In surveys of entrepreneurs who have attempted without success to raise sufficient capital, more than half of the respondents indicate that they have produced a formal business plan.']", "label": 0 }, { "id": "train_3426", "context": "Zebra mussels, a nuisance when they clog the intake pipes of nuclear power plants and water plants along the Great Lakes, have some redeeming qualities. Since the mussels feed voraciously on algae that they filter from the water that passes by them, bags of zebra mussels suspended in the discharge streams of chemical plants significantly improve water quality, even removing some hazardous wastes.", "question": "Which one of the following is most strongly supported on the basis of the statements above, if they are true?", "answers": "['There is no mechanical means available for clearing intake pipes by scraping the mussels from them.', 'Zebra mussels arrived in the Great Lakes on transatlantic freighters and, since they have no natural enemies there, are rapidly displacing the native species of clams.', 'If the mussels spread to areas of the Mississippi River where native clams provide the basis for a cultured-pearl industry, that industry will collapse, since the mussels are unsuitable for such use and would displace the clams.', 'Any hazardous waste the mussels remove from chemical-plant discharge will remain in the mussels, if they do not transform it, and they then must be regarded as hazardous waste.']", "label": 3 }, { "id": "train_3427", "context": "It is primarily by raising interest rates that central bankers curb inflation, but an increase in interest rates takes up to two years to affect inflation. Accordingly, central bankers usually try to raise interest rates before inflation becomes excessive, at which time inflation is not yet readily apparent either. But unless inflation is readily apparent, interest rate hikes generally will be perceived as needlessly restraining a growing economy. Thus, central bankers' success in temporarily restraining inflation may make it harder for them to ward off future inflation without incurring the public' s wrath.", "question": "Which one of the following most accurately describes the role played in the argument by the claim that it is primarily by raising interest rates that central bankers curb inflation?", "answers": "[\"It is a premise offered in support of the conclusion that central bankers' success in temporarily restraining inflation may make it harder for them to ward off future inflation without incurring the public's wrath.\", \"It is presented as a complete explanation of the fact that central bankers' success in temporarily restraining inflation may make it harder for them to ward off future inflation without incurring the public's wrath.\", 'It is a premise offered in support of the conclusion that unless inflation is readily apparent, interest rate hikes generally will be perceived as needlessly restraining a growing economy.', 'It is a description of a phenomenon for which the claim that an increase in interest rates takes up to two years to affect inflation is offered as an explanation.']", "label": 0 }, { "id": "train_3428", "context": "Knowledge in all fields is expanding and Ph. D. students take longer than ever before -- sometimes eight years -- to complete degree requirements. Yet, instead of agreeing that the longer period is needed, some noted scholars are recommending that Ph. D. programs reduce their requirements and have students finish within three years.", "question": "Which one of the following, if true, would most contribute to a justification of the noted scholars' recommendation?", "answers": "['Quality of research matters more than quantity, even though, on average, the more time a Ph. D. student spends on research, the greater the quantity of research produced.', 'For at least the last 50 years, no researcher has been able to be conversant with any more than a small fraction of the existing knowledge within any given field.', 'The most important objectives of Ph. D. programs can be adequately fulfilled with the reduced requirements recommended.', 'Some unusually talented Ph. D. students already complete all Ph. D. requirements within three years.']", "label": 2 }, { "id": "train_3429", "context": "If the proposed tax reduction package is adopted this year, the library will be forced to discontinue its daily story hours for children. But if the daily story hours are discontinued, many parents will be greatly inconvenienced. So the proposed tax reduction package will not be adopted this year.", "question": "Which one of the following, if assumed, allows the argument's conclusion to be properly drawn?", "answers": "['Every tax reduction package that would force the library to discontinue daily story hours would greatly inconvenience parents.', 'Any tax reduction package that will not force the library to discontinue daily story hours will be adopted this year.', 'No tax reduction package that would greatly inconvenience parents would fail to force the library to discontinue daily story hours.', 'No tax reduction package that would greatly inconvenience parents will be adopted this year.']", "label": 3 }, { "id": "train_3430", "context": "Police published a \"wanted\" poster for a criminal fugitive in a medical journal, because the fugitive was known to have a certain acute noninfectious skin problem that would eventually require a visit to a doctor. The poster asked for information about the whereabouts of the fugitive. A physician' s responding to the poster' s request for information would not violate medical ethics, since physicians are already subject to requirements to report gunshot wounds to police and certain infectious diseases to health authorities. These exceptions to confidentiality are clearly ethical.", "question": "Which one of the following principles, while remaining compatible with the requirements cited above, supports the view that a physician's responding to the request would violate medical ethics?", "answers": "[\"Since a patient comes to a physician with the expectation that the patient's visit and medical condition will remain confidential, it is not ethical for a physician to share this information with anyone except personnel within the physician's office.\", \"Since the primary concern of medicine is individual and public health, it is not ethical for a physician, except in the case of gunshot wounds, to reduce patients' willingness to come for treatment by a policy of disclosing their identities to law-enforcement agencies.\", 'Except to other medical personnel working to preserve or restore the health of a patient or of other persons, physicians cannot ethically disclose information about the identity of patients or their medical condition.', \"Except as required by the medical treatment of the patient, physicians cannot ethically disclose to others information about a patient's identity or medical condition without the patient's consent.\"]", "label": 1 }, { "id": "train_3431", "context": "More women than men suffer from Alzheimer' s disease -- a disease that is most commonly contracted by elderly persons. This discrepancy has often been attributed to women' s longer life span, but this theory may be wrong. A recent study has shown that prescribing estrogen to women after menopause, when estrogen production in the body decreases, may prevent them from developing the disease. Men' s supply of testosterone may help safeguard them against Alzheimer' s disease because much of it is converted by the body to estrogen, and testosterone levels stay relatively stable into old age.", "question": "Which one of the following most accurately expresses the main conclusion of the argument?", "answers": [ "A decrease in estrogen, rather than longer life span, may explain the higher occurrence of Alzheimer's disease in women relative to men.", "As one gets older, one's chances of developing Alzheimer's disease increase.", "The conversion of testosterone into estrogen may help safeguard men from Alzheimer's disease.", "Testosterone is necessary for preventing Alzheimer's disease in older men." ], "label": 0 }, { "id": "train_3432", "context": "Researchers have discovered that caffeine can be as physically addictive as other psychoactive substances. Some people find that they become unusually depressed, drowsy, or even irritable if they do not have their customary dose of caffeine. This is significant because as many people consume caffeine as consume any one of the other addictive psychoactive substances.", "question": "Which one of the following can be logically concluded from the information above?", "answers": "['Not all substances to which people can become physically addicted are psychoactive.', 'If one is physically addicted to a psychoactive substance, one will become unusually depressed when one is no longer ingesting that substance.', 'If alcohol is a physically addictive psychoactive substance, there are not more people who consume alcohol than consume caffeine.', 'There is no psychoactive substance to which more people are physically addicted than are addicted to caffeine.']", "label": 2 }, { "id": "train_3433", "context": "Rhonda: As long as the cost is not too great, you should use your time, energy, or money to help others. People who are active participants in charitable causes have richer lives than miserly hermits, however prosperous the hermits may be. Brad: You should ignore the problems of complete strangers and focus your generosity on your immediate relatives and close friends, since these are the people who will remember your sacrifices and return the kindness when you yourself need help.", "question": "Which one of the following principles, if valid, would most help to justify both Rhonda's and Brad's arguments?", "answers": "['One should act in ways that will benefit oneself.', 'One should treat others as one expects to be treated by them.', 'One should make sacrifices for others only if they will eventually return the favor.', 'One should always act in a manner that one can reflect on with pride.']", "label": 0 }, { "id": "train_3434", "context": "Paleontologists had long supposed that the dinosaur Diplodocus browsed for high-growing vegetation such as treetop leaves by raising its very long neck. But now computer models have shown that the structure of Diplodocus\"s neck bones would have prevented such movement. The neck could, however, bend downward and even extend below ground level, allowing Diplodocus to access underwater vegetation from dry land. Thus, Diplodocus must have fed on plants on or near the ground, or underwater.", "question": "Which one of the following is an assumption required by the argument?", "answers": "['Diplodocus was not able to browse for underwater vegetation by kneeling beside bodies of water or by walking into them.', 'The same type of neck structure is found in modern ground-feeding animals.', 'Diplodocus was not able to see in front of itself unless its head was angled steeply downward.', 'Diplodocus had no other way of accessing high-growing vegetation, such as by rising up on its hind legs.']", "label": 3 }, { "id": "train_3435", "context": "Linguist: Only if a sentence can be diagrammed is it grammatical. Any grammatical sentence is recognized as grammatical by speakers of its language. Speaker X' s sentence can be diagrammed. So, speaker X' s sentence will be recognized as grammatical by speakers of its language.", "question": "The linguist's reasoning is flawed because it fails to consider the possibility that", "answers": "['some ungrammatical sentences are diagrammable', 'all grammatical sentences can be diagrammed', 'some ungrammatical sentences are recognized as ungrammatical', 'most people are unable to diagram sentences correctly']", "label": 0 }, { "id": "train_3436", "context": "Any government action that intrudes on the right of privacy of an individual is unconstitutional. Ttherefore, the requirement that students in public school submit to random locker searches is unconstitutional.", "question": "Which of the following, if true, most weakens the conclusion that random locker searches in public schools are unconstitutional?", "answers": "['The search of lockers helps ensure the safety of the students.', 'The requirement is clearly posted throughout each school.', 'The Supreme Court, which is the final arbiter on issues of civil rights, has ruled that students in public school have no right to privacy on public school grounds.', \"Many of the lockers don't have locks.\"]", "label": 2 }, { "id": "train_3437", "context": "If Slater wins the election, McGuinness will be appointed head of the planning commission. But Yerxes is more qualified to head it since she is an 16 architect who has been on the planning commission for fifteen years. Unless the polls are grossly inaccurate, Slater will win.", "question": "Which one of the following can be properly inferred from the information above?", "answers": "['Either Slater will win the election or Yerxes will be appointed head of the planning commission.', 'If the polls are grossly inaccurate, someone more qualified than McGuinness will be appointed head of the planning commission.', 'If the polls are a good indication of how the election will turn out, someone less qualified than Yerxes will be appointed head of the planning commission.', 'McGuinness is not an architect and has not been on the planning commission for fifteen years or more.']", "label": 2 }, { "id": "train_3438", "context": "Asked by researchers to sort objects by shape, most toddlers in a large study had no trouble doing so. When subsequently told to sort by color, the toddlers seemed to have difficulty following the new rule and almost invariably persisted with their first approach. The researchers suggest such failures to adapt to new rules often result from insufficient development of the prefrontal cortex in toddlers. The cortex is essential for functions like adapting to new rules, yet is slow to mature, continuing to develop right into adolescence.", "question": "Which one of the following is most supported by the information above?", "answers": "['Skill at adapting to new situations is roughly proportional to the level of development of the prefrontal cortex.', 'The maturing of the prefrontal cortex is more important than upbringing in causing the development of adaptive behavior.', 'Toddlers unable to sort objects by color tend to have a less developed prefrontal cortex than other children of the same age.', 'Certain kinds of behavior on the part of toddlers may not be willfully disobedient.']", "label": 3 }, { "id": "train_3439", "context": "Marianne is a professional chess player who hums audibly while playing her matches, thereby distracting her opponents. When ordered by chess officials to cease humming or else be disqualified from professional chess, Marianne protested the order. She argued that since she was unaware of her humming, her humming was involuntary and that ttherefore she should not be held responsible for it.", "question": "Which one of the following principles, if valid, most helps to support Marianne's argument against the order?", "answers": "['Types of behavior that are not considered voluntary in everyday circumstances should be considered voluntary if they occur in the context of a professional chess match.', 'Chess players who hum audibly while playing their matches should not protest if their opponents also hum.', \"A person should be held responsible for those involuntary actions that serve that person's interests.\", \"Of a player's actions, only those that are voluntary should be used as justification for disqualifying that player from professional chess.\"]", "label": 3 }, { "id": "train_3440", "context": "Film critic: There has been a recent spate of so-called \"documentary\" films purporting to give the \"true story\" of one historical event or another. But most of these films have been inaccurate and filled with wild speculations, usually about conspiracies. The filmmakers defend their works by claiming that freedom of speech entitles them to express their views. Although that claim is true, it does not support the conclusion that anyone ought to pay attention to the absurd views expressed in the films.", "question": "To which one of the following principles does the film critic's commentary most closely conform?", "answers": "['Although filmmakers are entitled to express absurd views, they are not justified in doing so.', 'Freedom of speech does not entitle filmmakers to present inaccurate speculations as truth.', 'Views that people are entitled to express need not be views to which anyone is obliged to pay attention.', 'Freedom of speech sometimes makes the expression of absurd views necessary.']", "label": 2 }, { "id": "train_3441", "context": "From 1978 to 1988, beverage containers accounted for a steadily decreasing percentage of the total weight of household garbage in the United States. The increasingly widespread practice of recycling aluminum and glass was responsible for most of this decline. However, although aluminum recycling was more widely practiced in this period than glass recycling, it was found that the weight of glass bottles in household garbage declined by a greater percentage than the weight of aluminum cans.", "question": "Which of the following, if true of the United States in the period 1978 to 1988, most helps to account for the finding?", "answers": "['Manufacturers replaced many glass bottles, but few aluminum cans, with plastic containers.', 'Glass bottles are significantly heavier than aluminum cans of comparable size.', 'Recycled aluminum cans were almost all beverage containers, but a significant fraction of the recycled glass bottles had contained products other than beverages.', 'In many areas, glass bottles had to be sorted by color of the glass before being recycled, whereas aluminum cans required no sorting.']", "label": 0 }, { "id": "train_3442", "context": "Many of the presidents and prime ministers who have had the most successful foreign policies had no prior experience in foreign affairs when they assumed office. Although scholars and diplomats in the sacrosanct inner circle of international affairs would have us think otherwise, anyone with an acute political sense, a disciplined temperament, and a highly developed ability to absorb and retain information can quickly learn to conduct a successful foreign policy. In fact, prior experience alone will be of little value to a foreign policymaker who lacks all three of these traits.", "question": "If all of the statements above are true, which one of the following must be true?", "answers": "['An acute political sense, a disciplined temperament, and a highly developed ability to absorb and retain information are each necessary conditions for a president or prime minister to have a successful foreign policy.', 'A president or prime minister with years of experience in foreign affairs will have a more successful foreign policy than one who does not have experience in foreign affairs.', 'Prior experience in foreign affairs is neither a sufficient nor a necessary condition for a president or prime minister to have a successful foreign policy.', 'Prior experience in foreign affairs is a necessary but not sufficient condition for a president or prime minister to have a successful foreign policy.']", "label": 2 }, { "id": "train_3443", "context": "Samples from the floor of a rock shelter in Pennsylvania were dated by analyzing the carbon they contained. The dates assigned to samples associated with human activities formed a consistent series, beginning with the present and going back in time, a series that was correlated with the depth from which the samples came. The oldest and deepest sample was dated at 19, 650 years before the present, plus or minus 2, 400 years. Skeptics, viewing that date as to early and inconsistent with the accepted date of human migration into North America, suggested that the samples could have been contaminated by dissolved \"old carbon\" carried by percolating groundwater from nearby coal deposits.", "question": "Which of the following considerations, if true, argues most strongly against the suggestion of the skeptics?", "answers": "['No sample in the series, when retested by the carbon-dating procedure, was assigned an earlier date than that assigned to a sample from a layer above it.', 'Not every application of the carbon-dating procedure has led to results that have been generally acceptable to scientists.', 'No likely mechanism of contamination involving percolating groundwater would have affected the deeper samples from the site without affecting the uppermost sample.', 'There is no evidence that people were using coal for fuel at any time when the deepest layer might have been laid down.']", "label": 2 }, { "id": "train_3444", "context": "Conservationists have established land reserves to preserve the last remaining habitat for certain species whose survival depends on the existence of such habitat. A grove of trees in Mexico that provide habitat for North American monarch butterflies in winter is a typical example of such a land reserve. If global warming occurs as predicted, however, the temperature bands within which various types of vegetation can grow will shift into regions that are currently cooler.", "question": "If the statements above are true, they provide the most support for which one of the following?", "answers": "['Monarch butterflies will succeed in adapting to climatic change by shortening their migration.', 'If global warming occurs as predicted, the conservation land reserves will cease to serve their purpose.', 'If global warming occurs rapidly, species of plants and animals now protected in conservation land reserves will move to inhabit areas that are currently used for agriculture.', 'The natural world has adapted many times in the past to drastic global warming and cooling.']", "label": 1 }, { "id": "train_3445", "context": "Scientists examined diamonds that were formed on Earth about 2. 9 billion years ago. These diamonds had a higher-than-normal concentration of sulfur-33. This concentration can be explained only by certain chemical reactions that are stimulated by ultraviolet light. If there had been more than a trace of oxygen in Earth' s atmosphere 2. 9 billion years ago, then not enough ultraviolet light would have reached Earth' s surface to stimulate the chemical reactions.", "question": "The information above most strongly supports which one of the following?", "answers": "[\"Earth's atmosphere contained very little, if any, oxygen 2. 9 billion years ago.\", 'Most diamonds with higher-than-normal concentrations of sulfur-33 were formed at least 2. 9 billion years ago.', \"Ultraviolet light causes the oxygen in Earth's atmosphere to react chemically with sulfur-33.\", 'The formation of diamonds occurs only in the presence of ultraviolet light.']", "label": 0 }, { "id": "train_3446", "context": "The Breton Symphony Orchestra board of directors and the musicians' union are currently in contract negotiations. The musicians' union is considering proposing a new contract that would demand the following: a 10% increase in musician salaries, the creation of a new financial bonus system for musicians who have been with the orchestra for at least six years, and a limit on the number of rehearsals per week.", "question": "The answer to which of the following questions is LEAST important in helping the union to determine whether the board will be inclined to accept the new contract proposal?", "answers": "['Is the proposed limit on the number of rehearsals per week reasonable when compared with industry standards?', \"What percentage of the orchestra's revenue is generated from ticket sales?\", 'Does the orchestra have the means to pay for a 10% increase in musician salaries?', 'Are there comparable musicians not currently with the orchestra who would be willing to work under a less favorable contract?']", "label": 1 }, { "id": "train_3447", "context": "Journalist: Newspapers generally report on only those scientific studies whose findings sound dramatic. Furthermore, newspaper stories about small observational studies, which are somewhat unreliable, are more frequent than newspaper stories about large randomized trials, which generate stronger scientific evidence. Ttherefore, a small observational study must be more likely to have dramatic findings than a large randomized trial.", "question": "Which one of the following most accurately expresses a flaw in the journalist's reasoning?", "answers": "['It casts doubt on the reliability of a study by questioning the motives of those reporting it.', 'It confuses a claim about scientific studies whose findings sound dramatic with a similar claim about small observational studies.', 'It overlooks the possibility that small observational studies are far more common than large randomized trials.', \"It fails to rule out the possibility that a study's having findings that sound dramatic is an effect rather than a cause of the study's being reported on.\"]", "label": 2 }, { "id": "train_3448", "context": "Denoma, a major consumer-electronics maker, had a sizeable decline in sales revenue for its most recent fiscal year. This result appears surprising, because electronics retailers report that although their overall sales were considerably lower than in the previous year, their sales revenue from Denoma models actually grew, largely thanks to some innovative and popular models that Denoma introduced.", "question": "Which of the following, if true, does most to explain the apparently surprising result?", "answers": "[\"A significant proportion of Denoma's revenue comes from making components for other consumer-electronics manufacturers.\", 'Unlike some of its major competitors, Denoma has no lines of business outside consumer electronics to provide revenue when retail sales of consumer electronics are weak.', \"Because of the need to educate the public about its new models' capabilities, Denoma's advertising spending was higher than normal over the period.\", \"During the period, consumer-electronics retailers sold remaining units of Denoma's superseded models at prices that were deeply discounted from those models' original prices.\"]", "label": 0 }, { "id": "train_3449", "context": "Logging industry official: Harvesting trees from old-growth forests for use in manufacture can reduce the amount of carbon dioxide in the atmosphere, since when large old trees die in the forest they decompose, releasing their stored carbon dioxide. Harvesting old-growth forests would, moreover, make room for rapidly growing young trees, which absorb more carbon dioxide from the atmosphere than do trees in old-growth forests.", "question": "Which one of the following, if true, most seriously weakens the official's argument?", "answers": "['It can take many years for the trees of a newly planted forest to reach the size of those found in existing old-growth forests.', 'Much of the organic matter from old-growth trees, unusable as lumber, is made into products that decompose rapidly.', 'Much of the carbon dioxide present in forests is eventually released when wood and other organic debris found on the forest floor decompose.', 'A young tree contains less than half the amount of carbon dioxide that is stored in an old tree of the same species.']", "label": 1 }, { "id": "train_3450", "context": "Music professor: Because rap musicians can work alone in a recording studio, they need not accommodate supporting musicians' wishes. Further, learning to rap is not as formal a process as learning an instrument. Thus, rap is an extremely individualistic and nontraditional musical form. Music critic: But rap appeals to tradition by using bits of older songs. Besides, the themes and styles of rap have developed into a tradition. And successful rap musicians do not perform purely idiosyncratically but conform their work to the preferences of the public.", "question": "The music critic's response to the music professor's argument", "answers": "['challenges the grounds on which the music professor generalizes from the particular context of rap music to the broader context of musical tradition and individuality', 'challenges each of a group of claims about tradition and individuality in music that the music professor gives as evidence in his argument', 'challenges its conclusion concerning rap music by offering certain additional observations that the music professor does not take into account in his argument', 'challenges it by offering evidence against one of the stated premises on which its conclusion concerning rap music is based']", "label": 2 }, { "id": "train_3451", "context": "To face danger solely because doing so affords one a certain pleasure does not constitute courage. Real courage is manifested only when a person, in acting to attain a goal, perseveres in the face of fear prompted by one or more dangers involved.", "question": "Which one of the following statements can be properly inferred from the statements above?", "answers": "['A person who faces danger in order to benefit others is acting courageously only if the person is afraid of the danger.', 'A person who must face danger in order to avoid future pain cannot properly be called courageous for doing so.', 'A person who has no fear of the situations that everyone else would fear cannot be said to be courageous in any situation.', 'A person who experiences fear of some aspects of a dangerous situation cannot be said to act courageously in that situation.']", "label": 0 }, { "id": "train_3452", "context": "A survey of address changes filed with post offices and driver' s license bureaus over the last ten years has established that households moving out of the city of Weston outnumbered households moving into the city two to one. Ttherefore, we can expect that next year' s census, which counts all residents regardless of age, will show that the population of Weston has declined since the last census ten years ago.", "question": "Which one of the following, if true, most helps strengthen the argument?", "answers": "[\"Many people moving into Weston failed to notify either the post office or the driver's license bureau that they had moved to the city.\", 'Most people moving out of Weston were young adults who were hoping to begin a career elsewhere, whereas most adults remaining in or moving into the city had long-standing jobs in the city.', 'Over the past century any census of Weston showing a population loss was followed ten years later by a census showing a population gain.', 'Most adults moving out of Weston were parents who had children living with them, whereas most adults remaining in or moving into the city were older people who lived alone.']", "label": 3 }, { "id": "train_3453", "context": "In order to control the deer population, a biologist has proposed injecting female deer during breeding season with 10 milligrams of a hormone that would suppress fertility. Critics have charged that the proposal poses health risks to people who might eat the meat of treated deer and thereby ingest unsafe quantities of the hormone. The biologist has responded to these critics by pointing out that humans can ingest up to 10 milligrams of the hormone a day without any adverse effects, and since no one would eat even one entire deer a day, the treatment would be safe.", "question": "The biologist's response to critics of the proposal is based on which one of the following assumptions?", "answers": "['Most people do not consider deer meat to be part of their daily diet and eat it only on rare occasions.', 'The hormone in question does not occur naturally in the female deer that would be injected.', 'Hunting season for deer could be scheduled so that it would not coincide with breeding season.', 'The hormone that would be injected into the deer is chemically similar to hormones used in human contraceptives.']", "label": 1 }, { "id": "train_3454", "context": "The 1988 drought in North America was probably caused by shifts in the temperature patterns of large equatorial stretches of the Pacific Ocean. The drought, ttherefore, is not evidence for the hypothesis that a long-term global warming trend, allegedly caused by atmospheric pollutants such as carbon dioxide, is occurring.", "question": "Which of the following, if true, constitutes the best criticism of the argument above?", "answers": "['There has been no warming trend in the United States over the last 100 years.', 'The consequences of global warming occur long after the actual emission of pollutants into the atmosphere.', 'A global warming trend could cause increases in the frequency and severity of shifts in temperature patterns in the Pacific Ocean.', 'Most pre-1988 droughts for which we have records were preceded by shifts in temperature patterns in the Pacific Ocean.']", "label": 2 }, { "id": "train_3455", "context": "The commissioner has announced that Judge Khalid, who was on the seven-member panel appointed to resolve the Amlec labor dispute, will have sole responsibility for resolving the Simdon labor dispute. Since in its decision the Amlec panel showed itself both reasonable and fair, the two sides in the Simdon dispute are undoubtedly justified in the confidence they have expressed in the reasonableness and fairness of the arbitrator assigned to their case.", "question": "Which one of the following contains flawed reasoning most parallel to that contained in the passage?", "answers": "['Ula Borg, who has sold real estate for Arcande Realty for many years, undoubtedly sold fewer houses last year than she had the year before since the number of houses sold last year by Arcande Realty is far lower than the number sold the previous year.', \"Representing the school board, Marcia Barthes presented to the school's principal a list of recently elected school board members. Since only an elected member of the school board can act as its representative, Ms. Barthes's name undoubtedly appears on that list.\", \"Jorge Diaz is a teacher at a music school nationally known for the excellence of its conducting faculty. Since Mr. Diaz has recently been commended for the excellence of his teaching, he is undoubtedly a member of the school's conducting faculty.\", 'The members of the local historical society unanimously support designating the First National Bank building a historical landmark. Since Evelyn George is a member of that society, she undoubtedly favors according landmark status to the city hall as well.']", "label": 0 }, { "id": "train_3456", "context": "Analyst: A recent survey showed that although professors of biology who teach but do not pursue research made up one twentieth of all science professors, they were appointed to fewer than one twentieth of all the scientific administrative positions in universities. We can conclude from this survey that failing to pursue research tends to bias university administrators against appointing these professors to scientific administrative positions.", "question": "Which one of the following, if true, most seriously weakens the support for the analyst's conclusion?", "answers": "['Biologists who hold scientific administrative positions in the university tend to hold those positions for a shorter time than do other science professors.', 'Biology professors get more than one twentieth of all the science grant money available.', 'Conducting biological research tends to take significantly more time than does teaching biology.', 'Biologists who do research fill a disproportionately low number of scientific administrative positions in universities.']", "label": 3 }, { "id": "train_3457", "context": "Science writer: Scientists' astounding success rate with research problems they have been called upon to solve causes the public to believe falsely that science can solve any problem. In fact, the problems scientists are called upon to solve are typically selected by scientists themselves. When the problems are instead selected by politicians or business leaders, their formulation is nevertheless guided by scientists in such a way as to make scientific solutions feasible. Scientists are almost never asked to solve problems that are not subject to such formulation.", "question": "The science writer's statements, if true, most strongly support which one of the following?", "answers": "['Scientists would probably have a lower success rate with research problems if their grounds for selecting such problems were less narrow.', 'The only reason for the astounding success rate of science is that the problems scientists are called upon to solve are usually selected by the scientists themselves.', 'If a problem can be formulated in such a way as to make a scientific solution feasible, scientists will usually be called upon to solve that problem.', 'Most of the problems scientists are called upon to solve are problems that politicians and business leaders want solved, but whose formulation the scientists have helped to guide.']", "label": 0 }, { "id": "train_3458", "context": "Political theorist: Many people believe that the punishment of those who commit even the most heinous crimes should be mitigated to some extent if the crime was motivated by a sincere desire to achieve some larger good. Granted, some criminals with admirable motives deserve mitigated punishments. Nonetheless, judges should never mitigate punishment on the basis of motives, since motives are essentially a matter of conjecture and even vicious motives can easily be presented as altruistic.", "question": "Which one of the following principles, if valid, most helps to justify the political theorist's reasoning?", "answers": "['No law that cannot be enforced should be enacted.', 'It is better to err on the side of overly severe punishment than to err on the side of overly lenient punishment.', 'A legal system that, if adopted, would have disastrous consequences ought not be adopted.', 'Laws that prohibit or permit actions solely on the basis of psychological states should not be part of a legal system.']", "label": 1 }, { "id": "train_3459", "context": "If deep-sea divers ascend too rapidly from ocean depths, the decreasing pressure causes nitrogen to be released as gas bubbles in the blood. This condition is known as \"the bends. \" Sea snakes, who, like humans, breathe air that contributes nitrogen to the blood, are nevertheless able to make extremely rapid ascents and descents in ocean waters without suffering from the bends.", "question": "Which one of the following, if true, contributes most to an explanation of the difference described between humans and sea snakes?", "answers": "['Human deep-sea divers are trained to make several stops on ascending from deep water in order to adjust to decreasing pressure gradually, whereas sea snakes ascend from ocean depths without stopping.', 'Sea snakes, unlike humans, can excrete nitrogen from their blood into the sea by means of extremely small blood vessels beneath their skin.', 'A rapid release of bubbles of nitrogen gas into the blood inhibits circulation and can deprive organs of the blood they need to function.', \"The rapidity with which sea snakes are able to descend or ascend in ocean water is partially determined by the degree of buoyancy permitted by the inflation of the sea snake's lung.\"]", "label": 1 }, { "id": "train_3460", "context": "Naturalist: To be dependable, the accounting framework used by national economists to advise the government must take into account all of our nation' s assets; but the current accounting framework used by our national economists assigns no value to government-owned natural resources, which are clearly assets.", "question": "The naturalist's statements, if true, most strongly support which one of the following?", "answers": "[\"Economists' indifference toward the destruction of natural resources will lead policymakers to make poor decisions.\", 'The accounting framework used by national economists is not reliable.', 'Naturalists and economists disagree about whether natural resources have value.', 'Changes in the environment have a value that is not represented in any accounting framework.']", "label": 1 }, { "id": "train_3461", "context": "Wood that is waterlogged or desiccated can be preserved for a significant period, but, under normal conditions, wood usually disintegrates within a century or two. For this reason, archaeologists have been unable to find many remains of early wheeled vehicles to examine. However, archaeologists have found small ceramic models of wheeled vehicles made at approximately the same time as those early vehicles. Since these models have been much less susceptible to disintegration than the vehicles themselves, the main evidence regarding early wheeled vehicles has come from these models.", "question": "Which one of the following is most strongly supported by the information above?", "answers": "['Most of the small ceramic models of early wheeled vehicles were made by the very individuals who made the vehicles upon which the ceramic vehicles were modeled.', 'The individuals who made the early wheeled vehicles were not always aware that wood can be preserved through waterlogging or desiccation.', 'Of the early wheeled vehicles not preserved, more were made of wood than were made of materials no more susceptible to disintegration than are ceramic items.', 'Few, if any, small models of early wheeled vehicles were made of wood or other materials equally susceptible to disintegration under normal conditions.']", "label": 2 }, { "id": "train_3462", "context": "The excessive paperwork used to process claims at Brakeland Insurance Company, not the increase in accident claims involving privately owned cars, is responsible for the declining profits at Brakeland since the appointment of its new CEO. Sterling Auto Insurance Company has seen a similar increase in accidents among its insured vehicles, but while Brakeland' s profits have decreased, Sterling' s have increased in the same time period.", "question": "Which of the following, if true, would most weaken the argument above?", "answers": "[\"Sterling's customer-service approval ratings have always been higher than Brakeland's.\", 'Brakeland is experiencing a decrease in corporate insurance claims that have to be paid.', \"Individual policies traditionally account for the majority of Brakeland's profits, while corporate policies have accounted for the majority of Sterling's.\", 'Both Brakeland and Sterling have experienced an overall rise in claims.']", "label": 2 }, { "id": "train_3463", "context": "Community organizations wanting to enhance support for higher education programs need to convince the public that such programs benefit society as a whole. Taking this approach makes the public more receptive. It is much easier, for example, to get the public to support road building, which is seen as benefiting everyone, than it is to get them to support programs that are seen as benefiting only a relatively small segment of society.", "question": "Which one of the following most accurately expresses the overall conclusion drawn in the argument?", "answers": "['It is easy to get the public to support road building, because road building is seen as benefiting society as a whole.', 'Community organizations seeking to encourage higher education programs must persuade the public that these programs benefit society as a whole.', 'It is easier to get the public to support programs that are seen as benefiting everyone than it is to get them to support programs that are seen as benefiting only a small segment of society.', 'Convincing the public that higher education programs will benefit society as a whole makes the public more receptive to those programs.']", "label": 1 }, { "id": "train_3464", "context": "The painter Peter Brandon never dated his works, and their chronology is only now beginning to take shape in the critical literature. A recent dating of a Brandon self- portrait to 1930 is surely wrong. Brandon was 63 years old in 1930, yet the painting shows a young, dark-haired man- obviously Brandon, but clearly not a man of 63.", "question": "Which of the following, if justifiably assumed, allows the conclusion to be properly drawn?", "answers": "['Until recently, there was very little critical literature on the works of Brandon', 'Brandon at age 63 would not have portrayed himself in a painting as he had looked when he was a young man.', 'There is no securely dated self-portrait of Brandon that he painted when he was significantly younger than 63', 'In refraining from dating his works, Brandon intended to steer critical discussion of them away from considerations of chronology']", "label": 1 }, { "id": "train_3465", "context": "Those who claim that governments should not continue to devote resources to space exploration are wrong. Although most people' s lives are relatively unaffected by the direct consequences of space exploration, many modern technologies that have a tremendous impact on daily life -- e. g. , fiber optics, computers, and lasers -- are unexpected consequences of it. Society might have missed the benefits of these technologies if governments had not devoted resources to space exploration.", "question": "Which one of the following most accurately expresses the principle underlying the argument above?", "answers": "['Governments should continue to support those projects that have, in the past, produced unintended benefits.', 'Governments should not be prevented from allocating resources to projects whose intended consequences do not directly benefit most people.', 'One can never underestimate the beneficial consequences of government support of ambitious technological undertakings.', 'The less practical the goal of a government- supported project, the more unexpected the consequences of that project.']", "label": 0 }, { "id": "train_3466", "context": "Trust, which cannot be sustained in the absence of mutual respect, is essential to any long-lasting relationship, personal or professional. However, personal relationships, such as marriage or friendship, additionally require natural affinity. If a personal relationship is to endure, it must be supported by the twin pillars of mutual respect and affinity.", "question": "If the statements above are true, then which one of the following must also be true?", "answers": "['In the context of any professional relationship, mutual respect presupposes trust.', 'Basing a marriage on a natural affinity will ensure that it will endure.', 'Personal relationships, such as marriage or friendship, are longer-lasting than professional relationships.', 'A friendship supported solely by trust and mutual respect will not be long-lasting.']", "label": 3 }, { "id": "train_3467", "context": "Photovoltaic power plants produce electricity from sunlight. As a result of astonishing recent technological advances, the cost of producing electric power at photovoltaic power plants, allowing for both construction and operating costs, is one-tenth of what it was 20 years ago, whereas the corresponding cost for traditional plants, which burn fossil fuels, has increased. Thus, photovoltaic power plants offer a less expensive approach to meeting demand for electricity than do traditional power plants.", "question": "The conclusion of the argument is properly drawn if which one of the following is assumed?", "answers": "['The cost of producing electric power at traditional plants has increased over the past 20 years.', 'None of the recent technological advances in producing electric power at photovoltaic plants can be applied to producing power at traditional plants.', 'The cost of producing electric power at photovoltaic plants is expected to decrease further, while the cost of producing power at traditional plants is not expected to decrease.', 'Twenty years ago, the cost of producing electric power at photovoltaic plants was less than 10 times the cost of producing power at traditional plants.']", "label": 3 }, { "id": "train_3468", "context": "Helen: It was wrong of my brother Mark to tell our mother that the reason he had missed her birthday party the evening before was that he had been in a traffic accident and that by the time he was released from the hospital emergency room the party was long over. Saying something that is false can never be other than morally wrong, and there had been no such accident -- Mark had simply forgotten all about the party.", "question": "The main conclusion drawn in Helen's argument is that", "answers": "[\"the real reason Mark missed his mother's birthday party was that he had forgotten all about it\", 'it is always wrong not to tell the truth', 'it was wrong of Mark to tell his mother that he had missed her birthday party as a result of having been in a traffic accident', 'Mark did not tell his mother the truth']", "label": 2 }, { "id": "train_3469", "context": "Politician: There should be a mandatory prison sentence for everyone convicted of a violent crime. Some people object to such a policy on the grounds that it overlooks differences among individual cases that, if taken into consideration by judges and juries, could result in less prison overcrowding. But we can dismiss this objection since these critics would take a different view if they had themselves been victims of violent crime.", "question": "The politician's argument is most vulnerable to criticism on the grounds that this argument", "answers": "['takes for granted that prison overcrowding is caused mainly by a policy of mandatory prison sentences', 'takes for granted that, without mandatory prison sentences, judges and juries would always give sentences that are too lenient', \"takes for granted that one's view is unjustified if one would, under different circumstances, take a different view\", 'attempts to establish a factual generalization using anecdotal evidence']", "label": 2 }, { "id": "train_3470", "context": "Manufacturers sometimes discount the price of a product to retailers for a promotion period when the product is advertised to consumers. Such promotions often result in a dramatic increase in amount of product sold by the manufacturers to retailers. Nevertheless. the manufacturers could often make more profit by not holding the promotions.", "question": "Which of the following, if true, most strongly supports the claim above about the manufacturers' profit?", "answers": "[\"The amount of discount generally offered by manufacturers to retailers is carefully calculated to represent the minimum needed to draw consumers' attention to the product.\", 'During such a promotion retailers tend to accumulate in their warehouses inventory bought at discount; they then sell much of it later at their regular price.', 'For many consumer products the period of advertising discounted prices to consumers is about a week. not sufficiently long for consumers to become used to the sale price.', 'For products that are not newly introduced. the purpose of such promotions is to keep the products in the minds of consumers and to attract consumers who are currently using competing products.']", "label": 1 }, { "id": "train_3471", "context": "In a certain democratic country the legislature passed a new tax law over the principled objections of the parliamentary opposition. Some opposition leaders broke the new law by refusing to pay the new tax. They defended their lawbreaking by citing the historical precedent in the country of activists' having to break laws in winning for women the right to vote.", "question": "Which one of the following, if true, most weakens the opposition leaders' argument in defense of their actions?", "answers": "['The activists fought for equality of the sexes, a principle easier to define than the goal pursued by the opposition leaders.', 'Although they had principled objections to the new law, the opposition leaders derived a personal monetary benefit from breaking the law.', 'The opposition leaders, unlike the activists, fought for a return to an earlier state of affairs.', 'The opposition leaders, unlike the activists, were part of the democratic process that they are defying.']", "label": 3 }, { "id": "train_3472", "context": "Some managers think that the best way to maximize employee performance is to institute stiff competition among employees. However, in situations where one competitor is perceived to be clearly superior, other competitors become anxious and doubt their own ability to perform. Thus, stiff competition can undermine the result it was intended to achieve.", "question": "The conclusion of the argument can be properly drawn if which one of the following is assumed?", "answers": "['When competitors perceive the competition as winnable, their overall performance generally improves.', 'Competitors who work to undermine the confidence of other participants often do better in competitions.', 'Those who are perceived to be clearly superior almost always win.', \"Doubting one's own ability to perform can decrease one's overall performance.\"]", "label": 3 }, { "id": "train_3473", "context": "A corporation created a new division. To staff it, applicants were rigorously screened and interviewed. Those selected were among the most effective, efficient, and creative workers that the corporation had ever hired. Thus, the new division must have been among the most effective, efficient, and creative divisions the corporation had ever created.", "question": "The flawed pattern of reasoning in which one of the following is most similar to that in the argument above?", "answers": "['A law firm decided to add a department devoted to family law. To obtain the best employees it could, the firm studied the credentials and composition of several other firms well known to have successful staffs working in family law. Eventually, the firm hired a staff of new lawyers and support personnel having training and aptitudes as much like those of the studied firms as possible. Thus the law firm must have created one of the best family-law departments.', 'Several salespeople were given incentives to recruit the largest number of new customers in one month. To monitor the incentive program, the boss interviewed one of the salespeople and found that the salesperson had already exceeded the minimum goals of the program. Thus the incentive program was indeed effective.', \"Various schools chose teams of students to compete in a debate tournament. Each school's team presented a position and rebutted the others' positions. After the initial scores were in, the ten top teams competed against each other. Since one team eventually emerged with the highest average score, it was clearly the best team.\", \"To put together this year's two All-Star Teams, the best players in the league were selected. Half of them were put on Team One, and half were put on Team Two. Since each player on the two teams was one of the best players in the league this year, it follows that the two AllStar Teams are the two best teams this year.\"]", "label": 3 }, { "id": "train_3474", "context": "Columnist: On average, about 70 percent of the profit from tourism in developing countries goes to foreign owners of tourist businesses. In general, as a country becomes a more established tourist destination, the proportion of revenues exported in this way increases. However, tourists can counteract this effect by obtaining accommodations and other services directly from local people.", "question": "Which one of the following is most strongly supported by the statements made by the columnist?", "answers": "['In at least some of the developing countries that are most established as tourist destinations, most of the profits from tourism go to foreign owners of tourist businesses.', 'In general, as a developing country becomes a more established tourist destination, local people become progressively poorer.', 'In at least some developing countries, tourists obtain most of their accommodations and other services directly from local people.', 'Tourists who obtain accommodations and other services directly from local people do not contribute in any way to the profits of foreign owners of tourist businesses.']", "label": 0 }, { "id": "train_3475", "context": "Over the past 20 years, skiing has become a relatively safe sport due to improvements in ski equipment. There has been a 50 percent drop in the number of ski injuries over the last 20 years. Clearly, however, there have not been decreases in the number of injuries in all categories, as statistical data readily show, for although broken legs and ankle injuries have decreased by an astounding 90 percent, knee injuries now represent 16 percent of all ski injuries, up significantly from the 11 percent of 20 years ago.", "question": "The reasoning in the argument is flawed because the argument does which one of the following?", "answers": "['It fails to allow for there being ski injuries other than broken legs, ankle injuries, and knee injuries.', 'It proceeds as though there could be a greater decrease in injuries in each category on injury than there is in injuries overall.', 'It assumes that an increase in the proportion of knee injuries rules out a decrease in the number of knee injuries.', 'It ignores the possibility that the number of skiers has increased over the past 20 years.']", "label": 2 }, { "id": "train_3476", "context": "Human Resources Director: Some people dislike their jobs but still go to work every day because they feel that it is ethically wrong to miss work. Others enjoy their jobs but sometimes miss work because they genuinely believe that they are too sick to work or that they might infect others if they go to work. This makes it difficult to__.", "question": "Which one of the following most reasonably completes the argument?", "answers": "[\"make any reliable predictions about an employee's future behavior from his or her absenteeism record\", \"draw any conclusions about an employee's job satisfaction from his or her absenteeism record\", 'maintain accurate absenteeism records for all the different employees of a company', 'determine whether employees absent from work more often than others are any less fearful of losing their jobs']", "label": 1 }, { "id": "train_3477", "context": "Economist: The economy seems to be heading out of recession. Recent figures show that consumers are buying more durable goods than before, indicating that they expect economic growth in the near future.", "question": "That consumers are buying more durable goods than before figures in the economist's argument in which one of the following ways?", "answers": "[\"It is the primary evidence from which the argument's conclusion is drawn.\", \"Its truth is required in order for the argument's conclusion to be true.\", 'It is an inference drawn from the premise that consumers expect economic growth in the near future.', 'It is an inference drawn from the premise that the recession seems to be ending.']", "label": 0 }, { "id": "train_3478", "context": "Powell: Private waste-removal companies spend 60 percent of what public waste-removal companies spend per customer, yet give their customers at least as good service. Private waste-removal companies, ttherefore, work more efficiently. Freeman: Your conclusion is unwarranted. Different customers have different wasteremoval needs. Since private companies, unlike their public counterparts, can select which customers to serve, they choose to exclude the potential customers whom they judge to be the most costly to serve.", "question": "The issue in dispute between Powell and Freeman is the", "answers": "['accuracy of the figure of 60 percent with regard to the difference in service costs between private and public waste-removal companies', 'ability of private versus public waste-removal companies to select which customers to serve', 'reason private waste-removal companies are able to offer service comparable to that offered by public ones while spending less money per customer', 'relationship between the needs of a waste- removal customer and the amount of money it takes to serve that customer']", "label": 2 }, { "id": "train_3479", "context": "These days, drug companies and health professionals alike are focusing their attention on cholesterol in the blood. The more cholesterol we have in our blood, the higher the risk that we shall die of a heart attack. The issue is pertinent since heart disease kills more North Americans every year than any other single cause. At least three factors -- smoking, drinking, and exercise -- can each influence levels of cholesterol in the blood.", "question": "Which one of the following can be properly concluded from the passage?", "answers": "['Smoking in moderation can entail as great a risk of fatal heart disease as does heavy smoking.', \"The only way that smoking increases one's risk of fatal heart disease is by influencing the levels of cholesterol in the blood.\", 'The risk of fatal heart disease can be altered by certain changes in lifestyle.', 'A high-cholesterol diet is the principal cause of death in North America.']", "label": 2 }, { "id": "train_3480", "context": "People who listen to certain recordings of music are in danger of being unduly influenced by spoken messages that have been recorded backwards on the records or tapes.", "question": "A consequence of the view above is that", "answers": "['backwards messages can be added to a recording while still preserving all the musical qualities of the recorded performance', 'the recordings on which such messages appear are chosen for this purpose either because they are especially popular or because they induce a trancelike state', 'when people listen to recorded music, they pay full attention to the music as it plays', 'if such messages must be comprehended to exert influence, then people must be able to comprehend spoken messages recorded backwards']", "label": 3 }, { "id": "train_3481", "context": "Chef: This mussel recipe' s first step is to sprinkle the live mussels with cornmeal. The cornmeal is used to clean them out: they take the cornmeal in and eject the sand that they contain. But I can skip this step, because the mussels available at seafood markets are farm raised and ttherefore do not contain sand.", "question": "Which one of the following is an assumption required by the chef's argument?", "answers": "[\"The chef's mussel recipe was written before farm-raised mussels became available.\", 'Mussels contain no contaminants other than sand.', 'The mussels the chef is using for the mussel recipe came from a seafood market.', 'Sprinkling the mussels with cornmeal does not affect their taste.']", "label": 2 }, { "id": "train_3482", "context": "Fossil-fuel emissions, considered a key factor in the phenomenon known as global warming, contain two gases, carbon dioxide and sulfur dioxide, that have opposite effects on atmospheric temperatures. Carbon dioxide traps heat, tending to warm the atmosphere, whereas sulfur dioxide turns into sulfate aerosols that reflect sunlight back toward space, thereby tending to cool the atmosphere. Given that the heat-trapping effect is stronger than the cooling effect, cutting fossil-fuel emissions might be expected to slow the rise in global temperatures. Yet, surprisingly, if fossil-fuel emissions were cut today, global warming would actually be enhanced for more than three decades before the temperature rise began to slow.", "question": "Which one of the following, if true, most helps to explain the claim made in the last sentence above?", "answers": "['Carbon dioxide is produced not only by automobiles but also by power plants that burn fossil fuels.', 'Sulfur pollution is not spread evenly around the globe but is concentrated in the Northern Hemisphere, where there is a relatively high concentration of industry.', 'Carbon dioxide stays in the atmosphere for many decades, while the sulfate aerosols fall out within days.', 'Because fossil-fuel emissions contain sulfur dioxide, they contribute not only to global warming but also to acid rain.']", "label": 2 }, { "id": "train_3483", "context": "A transit company' s bus drivers are evaluated by supervisors riding with each driver. Drivers complain that this affects their performance, but because the supervisor' s presence affects every driver' s performance, those drivers performing best with a supervisor aboard will likely also be the best drivers under normal conditions.", "question": "Which one of the following is an assumption on which the argument depends?", "answers": "['The bus drivers are each affected in roughly the same way and to the same extent by the presence of the supervisor.', \"The supervisors are excellent judges of a bus driver's performance.\", 'For most bus drivers, the presence of a supervisor makes their performance slightly worse than it otherwise would be.', 'The bus drivers themselves are able to deliver accurate assessments of their driving performance.']", "label": 0 }, { "id": "train_3484", "context": "In the course of his reading, George Orwell probably encountered certain storytelling conventions over and over again, and these are the devices he would have most likely used in his work. That is why it does not follow that, even though his 1984 resembles other books of its futuristic genre, Orwell read those books; it is possible that he and the other authors were simply drawing on the same body of literary conventions.", "question": "Which one of the following most closely illustrates the principle that the passage illustrates?", "answers": "['To direct an effective movie within the cowboy genre, a director must study previously successful cowboy movies.', 'A mystery novel may not resemble novels from other genres, even though it was directly influenced by such novels.', 'A historical romance novel does not fit into its literary genre unless it employs certain kinds of conventions.', 'A recent film that involves car chases, explosions, and clever villains is not necessarily directly influenced by other films of the action genre.']", "label": 3 }, { "id": "train_3485", "context": "Debater: The average amount of overtime per month worked by an employee in the manufacturing division of the Haglut Corporation is 14 hours. Most employees of the Haglut Corporation work in the manufacturing division. Furthermore, the average amount of overtime per month worked by any employee in the company generally does not fluctuate much from month to month. Ttherefore, each month, most employees of the Haglut Corporation almost certainly work at least some overtime.", "question": "The debater's argument is most vulnerable to criticism on which of these grounds?", "answers": "['It overlooks the possibility that even if, on average, a certain amount of overtime is worked by the members of some group, many members of that group may work no overtime at all.', 'It takes for granted that the manufacturing division is a typical division of the corporation with regard to the average amount of overtime its employees work each month.', \"It confuses a claim from which the argument's conclusion about the Haglut Corporation would necessarily follow with a claim that would follow from the argument's conclusion only with a high degree of probability.\", 'It takes for granted that if a certain average amount of overtime is worked each month by each employee of the Haglut Corporation, then approximately the same amount of overtime must be worked each month by each employee of the manufacturing division.']", "label": 0 }, { "id": "train_3486", "context": "James: Many people claim that the voting public is unable to evaluate complex campaign issues. The television commercials for Reade in the national campaign, however, discuss complex campaign issues, and Reade is, at present, more popular than any other candidate. Maria: Yes, Reade is the most popular. However, you are incorrect in claiming that this is because of Reade' s discussion of complex campaign issues. Reade simply strikes the voters as the most competent and trustworthy candidate.", "question": "Which one of the following, if true, most supports Maria's counter to James?", "answers": "[\"Polling data show that most voters cannot identify Reade's positions on campaign issues.\", \"Reade's opponents charge that Reade oversimplifies complex campaign issues.\", 'Polling data show that some voters consider Reade competent and trustworthy.', \"Polling data show that Reade's present popularity will probably diminish over time.\"]", "label": 0 }, { "id": "train_3487", "context": "In a party game, one person leaves the room with the understanding that someone else will relate a recent dream to the remaining group. The person then returns and tries to reconstruct the dream by asking only yes-or-no questions. In fact, no dream has been related: the group simply answers the questions according to some arbitrary rule. Surprisingly, the person usually constructs a dream narrative that is both coherent and ingenious.", "question": "The example presented above most closely conforms to which one of the following propositions?", "answers": "['Dreams are often just collections of images and ideas without coherent structures.', 'The presumption that something has order and coherence can lead one to imbue it with order and coherence.', 'People often invent clever and coherent stories to explain their behavior to other people.', \"Interpreting another person's dream requires that one understand the dream as a coherent narrative.\"]", "label": 1 }, { "id": "train_3488", "context": "Faced with a financial crisis, Upland University' s board of trustees reduced the budget for the university' s computer center from last year' s $4 million to $1. 5 million for the coming year. However, the center cannot operate on less than $2. 5 million. Since the board cannot divert funds from other programs to the computer center, there is no way that the center can be kept operating for the coming year.", "question": "The conclusion of the argument is properly drawn if which one of the following is assumed?", "answers": "[\"The computer center has no source of funds other than those budgeted to it for the coming year by the university's board of trustees.\", 'The budgets of other programs at the university were also reduced.', 'The computer center did not use all of the $4 million that was budgeted to it last year.', 'The board of trustees at the university value other programs at the university more highly than they do the computer center.']", "label": 0 }, { "id": "train_3489", "context": "The relaxation of regulations governing the manufacture and sale of new medicines to increase their availability should not be accompanied by a lifting of all regulations that restrict industrial activity generally. Unless strict environmental regulations are maintained, endangered species of plants and animals will become extinct. And since a large majority of new medicines are derived from plants and animals, a general deregulatory approach could actually undermine the original intent of the relaxation of regulations governing the manufacture and sale of new medicines.", "question": "The statement that a large majority of new medicines are derived from plants and animals plays which one of the following roles in the argument?", "answers": "['support for the hypothesis that only very narrowly focused efforts at deregulation of industrial activity actually have beneficial results', 'evidence for a point of view that the argument is designed to undermine', 'a link between the extinction of species and the potentially decreased availability of new medicines', 'an illustration of the potential disaster that could result from continued overregulation of industrial activity']", "label": 2 }, { "id": "train_3490", "context": "A chimp who displays feelings of affection toward the other members of its social group is more likely to be defended by these group members from raiders outside of the group -- even at the risk of harm to these defenders -- than are those chimps who rarely or never display feelings of affection toward their associates. This shows that, from a sociological perspective, affection plays the same role in chimp communities as in human communities, since humans are more willing to face risks to protect those toward whom they have feelings of affection.", "question": "Which one of the following is an assumption on which the argument depends?", "answers": "['Expression of affection in chimps is limited to members of the social group to which they belong.', 'Feelings of affection are the only reason humans protect each other.', 'Chimps express their emotions behaviorally whenever they feel them.', 'Feelings of affection in chimp communities are at least sometimes reciprocated.']", "label": 3 }, { "id": "train_3491", "context": "When an airplane is taken out of service for maintenance, it is often repainted as well, and during the repainting no other maintenance work can be done on the plane. In order to reduce maintenance time, airline officials are considering using a new nontoxic plastic film instead of paint. The film takes just as long to apply as paint does, but many other maintenance tasks can be carried out at the same time.", "question": "Which of the following, if true, is further evidence that using the film will help the airline officials achieve their goal?", "answers": "['Even at speeds considerably higher than the normal speed of a passenger jet, the film remains securely attached.', 'Unlike paint, the film gives a milky tone to certain colors.', 'The film can be applied only by technicians who have received special training.', 'At the end of its useful life, the film can be removed much more quickly than paint can.']", "label": 3 }, { "id": "train_3492", "context": "According to sources who can be expected to know, Dr. Maria Esposito is going to run in the mayoral election. But if Dr. Esposito runs, Jerome Krasman will certainly not run against her. Ttherefore Dr. Esposito will be the only candidate in the election.", "question": "The flawed reasoning in the argument above most closely parallels that in which one of the following?", "answers": "[\"The press release says that the rock group Rollercoaster is playing a concert on Saturday. It won't be playing on Friday if it plays on Saturday. So Saturday will be the only day this week on which Rollercoaster will perform.\", \"According to its management, Brown's Stores will move next year. Without Brown's being present, no new large store can be attracted to the downtown area. Ttherefore the downtown area will no longer be viable as a shopping district.\", 'An informant says that Rustimann was involved in the bank robbery. If Rustimann was involved, Jones was certainly not involved. Since these two are the only people who could have been involved, Rustimann is the only person the police need to arrest.', 'Joshua says the interviewing panel was impressed by Marilyn. But if they were impressed by Marilyn, they probably thought less of Sven. Joshua is probably right, and so Sven will probably not get the job.']", "label": 0 }, { "id": "train_3493", "context": "High schools should only teach abstinence. Students who practice abstinence will never experience an unplanned pregnancy or contract a sexually-transmitted disease.", "question": "Each of the following weakens the argument EXCEPT:", "answers": "['Religious organizations support the teaching of abstinence.', 'Students are less likely to follow teaching about abstinence than safe sex.', 'Failing to teach about contraceptives increases the spread of sexually-transmitted diseases.', 'The percentage of students engaging in abstinence is lowest in school districts that only teach abstinence.']", "label": 0 }, { "id": "train_3494", "context": "Sociologist: Climate and geology determine where human industry can be established. Drastic shifts in climate always result in migrations, and migrations bring about the intermingling of ideas necessary for rapid advances in civilization.", "question": "The sociologist's statements, if true, most strongly support which one of the following?", "answers": "['Populations settle in every place where human industry can be established.', 'All shifts in climate produce a net gain in human progress.', 'Climate is the primary cause of migration.', 'A population remains settled only where the climate is fairly stable.']", "label": 3 }, { "id": "train_3495", "context": "The availability of television reduces the amount of reading children do. When television is made unavailable, a nearly universal increase in reading, both by parents and by children, is reported. When television is available again, the level of reading by both parents and children relapses to its previous level.", "question": "The reasoning in which one of the following is most similar to the reasoning above?", "answers": "['Global warming is caused by increased carbon dioxide in the atmosphere. Furthermore, industrial pollution causes increased carbon dioxide in the atmosphere. So industrial pollution causes global warming.', \"Voting behavior is affected by factors other than political candidates' records of political achievement. For example, a candidate who projects confidence will gain votes as a result, whereas a candidate with a supercilious facial expression will lose votes.\", 'Adults read less than they once did because there are so many other activities to divert them. This can be seen from the fact that the more time they spend on such other activities, the less they read. Conversely, the less they read, the more time they spend on such other activities.', \"Whenever the money supply in an economy fluctuates, interest rates tend to fluctuate. When the money supply remains constant, interest rates tend to remain stable. Thus, the money supply's remaining constant stabilizes interest rates.\"]", "label": 3 }, { "id": "train_3496", "context": "Actor: Bertolt Brecht' s plays are not genuinely successful dramas. The roles in Brecht' s plays express such incongruous motives and beliefs that audiences, as well as the actors playing the roles, invariably find it difficult, at best, to discern any of the characters' personalities. But, for a play to succeed as a drama, audiences must care what happens to at least some of its characters.", "question": "The conclusion of the actor's argument can be properly drawn if which one of the following is assumed?", "answers": [ "An audience that cannot readily discern a character's personality will not take any interest in that character.", "All plays that, unlike Brecht's plays, have characters with whom audiences empathize succeed as dramas.", "The extent to which a play succeeds as a drama is directly proportional to the extent to which the play's audiences care about its characters.", "A character's personality is determined primarily by the motives and beliefs of that character." ], "label": 0 }, { "id": "train_3497", "context": "A seriously maladaptive trait is unlikely to persist in a given animal population for long, since there is enough genetic variation in populations that some members will lack the trait. Those lacking the trait will compete more successfully for the available resources. Hence these members of the population survive and reproduce at a higher rate, crowding out those with the maladaptive trait.", "question": "The proposition that those lacking a maladaptive trait will compete more successfully for the available resources figures in the argument in which one of the following ways?", "answers": "['It is an intermediate conclusion of the argument.', 'It expresses a view that the argument as a whole is designed to discredit.', 'It is a premise of the argument.', 'It presents evidence that the argument attempts to undermine.']", "label": 2 }, { "id": "train_3498", "context": "Videocassette recorders (VCRs) enable people to watch movies at home on videotape. People who own VCRs go to movie theaters more often than do people who do not own VCRs. Contrary to popular belief, ttherefore, owning a VCR actually stimulates people to go to movie theaters more often than they otherwise would.", "question": "The argument is most vulnerable to criticism on the grounds that it", "answers": "['cites, in support of the conclusion, evidence that is inconsistent with other information that is provided', 'bases a broad claim about the behavior of people in general on a comparison between two groups of people that together include only a small proportion of people overall', 'fails to establish that the phenomena interpreted as cause and effect are not both direct effects of some other factor', 'takes a condition that by itself guarantees the occurrence of a certain phenomenon to be a condition that ttherefore must be met for that phenomenon to occur']", "label": 2 }, { "id": "train_3499", "context": "Science journalist: Brown dwarfs are celestial objects with more mass than planets but less mass than stars. They are identified by their mass and whether or not lithium is present in their atmospheres. Stars at least as massive as the Sun have lithium remaining in their atmospheres because the mixing of elements in their internal nuclear furnaces is incomplete. Stars with less mass than the Sun have no lithium because the element has been fully mixed into their nuclear furnaces and consumed. A brown dwarf does not have a fully functional nuclear furnace and so its lithium cannot be consumed.", "question": "Which one of the following is most strongly supported by the science journalist's statements?", "answers": "[\"Any celestial object with lithium in its atmosphere has a nuclear furnace that has incompletely mixed the object's elements.\", 'No celestial object less massive than a brown dwarf has lithium in its atmosphere.', 'No celestial object with lithium in its atmosphere has less mass than the Sun.', 'No celestial object that has no lithium in its atmosphere is a brown dwarf.']", "label": 3 }, { "id": "train_3500", "context": "Box office receipts at movie theaters increased 40 percent last year over the previous year. Thus, the film industry overall evidently is prospering. Yet twice as many movie theaters went bankrupt last year as in the preceding two years combined.", "question": "Which one of the following, if true, most helps to resolve the apparent discrepancy in the information above?", "answers": "['Films cost, on average, twice as much to produce today as they did ten years ago.', \"Those of last year's films that were successful were very profitable films that were shown by exclusive engagement at only a selection of the largest theaters.\", 'Ticket prices at some theaters fell last year.', 'The amount of money spent on film advertising increased greatly last year over the year before, and the majority of this expense was absorbed by the producers and the distributors of films, not by the theater owners.']", "label": 1 }, { "id": "train_3501", "context": "Martha' s friend, who is very knowledgeable about edible flowers, told Martha that there are no edible daisies, at least not any that are palatable. Martha, however, reasons that since there are daisies that are a kind of chrysanthemum and since there are edible chrysanthemums that are quite palatable, what her friend told her must be incorrect.", "question": "Which one of the following has a flawed pattern of reasoning most like that in Martha's reasoning?", "answers": "[\"Many of Teresa's colleagues have written books. Most of the books they have written are on good writing. So some of Teresa's colleagues are good writers.\", \"Some of Noriko's sisters are on the debate team, and some members of the debate team are poor students. So at least one of Noriko's sisters must be a poor student.\", 'Rolfe belongs to the library reading group, and all members of that group are avid readers. So Rolfe is an avid reader.', \"Most of Leon's friends are good swimmers, and good swimmers are quite strong. So it is likely that at least some of Leon's friends are quite strong.\"]", "label": 1 }, { "id": "train_3502", "context": "Twenty percent of the population of Springhill has been to Italy at least once in the last five years, and thirty percent of the population of Springhill has been to France at least once in the last five years. Ttherefore, half of the population of Springhill has been to Europe at least once in the last five years.", "question": "The argument is faulty because it ignores the possibility that", "answers": "['some of the population of Springhill has been neither to Italy nor to France in the last five years', 'none of the population of Springhill has been either to Italy or to France more than once in the last five years', 'some of the population of Springhill has been both to Italy and to France in the last five years', 'some of the population of Springhill has been either to Italy or to France in the last five years, but not to both']", "label": 2 }, { "id": "train_3503", "context": "A television manufacturing plant has a total of 1, 000 workers, though an average of 10 are absent on any given day for various reasons. On days when exactly 10 workers are absent, the plant produces televisions at its normal rate. Thus, it is reasonable to assume that the plant could fire 10 workers without any loss in production.", "question": "The argument is most vulnerable to criticism on the grounds that it", "answers": "['fails to show that the absentee rate would drop if 10 workers were fired', 'overlooks the possibility that certain workers are crucial to the production of televisions', 'takes for granted that the normal rate of production can be attained only when no more than the average number of workers are absent', 'takes for granted that the rate of production is not affected by the number of workers employed at the plant']", "label": 0 }, { "id": "train_3504", "context": "In parts of South America, vitamin-A deficiency is a serious health problem, especially among children. In one region, agriculturists hope to improve nutrition by encouraging farmers to plant a new variety of sweet potato called SPK004 that is rich in betacarotene, which the body converts into vitamin A. The plan has good chances of success, since sweet potato is a staple of the region' s diet and agriculture, and the varieties currently grown contain little beta-carotene.", "question": "Which of the following, if true, most strongly supports the prediction that the plan will succeed?", "answers": "['There are no other varieties of sweet potato that are significantly richer in beta-carotene than SPK004 is.', 'The currently cultivated varieties of sweet potato contain no important nutrients that SPK004 lacks.', 'The flesh of SPK004 differs from that of the currently cultivated sweet potatoes in color and texture, so traditional foods would look somewhat different when prepared from SPK004.', 'There are other vegetables currently grown in the region that contain more beta-carotene than the currently cultivated varieties of sweet potato do.']", "label": 1 }, { "id": "train_3505", "context": "The Acme Corporation has found that improvements in its information technology infrastructure allow its employees to make more decisions that are both sound and well-informed than was previously feasible. Consequently, the corporation plans to improve employee productivity by introducing new managerial techniques that delegate much of the decision-making to lower levels of the organizational hierarchy. Managers will simply set clear standards and guidelines and then allow employee teams to undertake tasks without centralized control.", "question": "Which of the following, if true, would most strongly support a prediction that the Acme Corporation's plan will achieve its goal?", "answers": "[\"Most of the tasks currently performed by the corporation's employees require few if any significant decisions to be made.\", \"The corporation will be able to cut its managerial staff only if the managers' sole function is to set clear standards and guidelines for employees.\", 'When employees can make decisions themselves, rather than submitting the same matters for decision to managers, there is more time available for directly productive activities.', \"After Acme's information technology infrastructure was improved, very few of Acme's managers continued to believe that only managers should make most decisions.\"]", "label": 2 }, { "id": "train_3506", "context": "It is even more important that we criticize democracies that have committed human rights violations than that we criticize dictatorships that have committed more violent human rights offenses. Human rights violations are always inexcusable, but those committed by governments that represent the will of the people are even more reprehensible than those committed by dictators. Further, our criticism is more likely to have an effect on the former than on the latter.", "question": "Which one of the following is a proper inference from the passage?", "answers": "['All governments commit some inexcusable and reprehensible acts.', 'Some human rights violations are more reprehensible than other, more violent human rights violations.', 'Human rights violations are more likely to occur in democracies than in dictatorships.', 'Those who do represent the will of the people are less likely to be moved by criticism than are those who merely claim to represent the will of the people.']", "label": 1 }, { "id": "train_3507", "context": "The government is being urged to prevent organizations devoted to certain views on human nutrition from advocating a diet that includes large portions of uncooked meat, because eating uncooked meat can be very dangerous. However, this purported fact does not justify the government' s silencing the groups, for surely the government would not be justified in silencing a purely political group merely on the grounds that the policies the group advocates could be harmful to some members of society. The same should be true for silencing groups with certain views on human nutrition.", "question": "Which one of the following principles most helps to justify the reasoning in the argument?", "answers": "['The government ought to do whatever is in the best interest of society.', 'The government ought not to silence an opinion merely on the grounds that it could be harmful to disseminate the opinion.', 'The government should not silence any group for advocating a position that a significant proportion of society believes to be beneficial.', 'One ought to advocate a position only if one believes that it is true or would be beneficial.']", "label": 1 }, { "id": "train_3508", "context": "Inez: Space-exploration programs pay for themselves many times over, since such programs result in technological advances with everyday, practical applications. Space exploration is more than the search for knowledge for its own sake; investment in space exploration is such a productive investment in developing widely useful technology that we can' t afford not to invest in space exploration. Winona: It is absurd to try to justify funding for space exploration merely by pointing out that such programs will lead to technological advances. If technology with practical applications is all that is desired, then it should be funded directly.", "question": "Winona responds to Inez by", "answers": "['showing that there is no evidence that the outcome Inez anticipates will in fact be realized', 'suggesting that Inez has overlooked evidence that directly argues against the programs Inez supports', 'claiming that a goal that Inez mentions could be pursued without the programs Inez endorses', 'demonstrating that the pieces of evidence that Inez cites contradict each other']", "label": 2 }, { "id": "train_3509", "context": "A student has taken twelve courses and received a B in a majority of them. The student is now taking another course and will probably, given her record, receive a B in it.", "question": "Each of the following, if true, strengthens the argument EXCEPT:", "answers": "['The student previously studied alone but is receiving help from several outstanding students during the present course.', 'The current course is a continuation of one of the twelve courses in which the student received a B.', 'The student previously studied in the library and continues to do so.', 'The student received a B in all but one of the twelve courses.']", "label": 0 }, { "id": "train_3510", "context": "Agricultural economist: We can increase agricultural production without reducing biodiversity, but only if we abandon conventional agriculture. Thus, if we choose to sustain economic growth, which requires increasing agricultural production, we should radically modify agricultural techniques.", "question": "Which one of the following principles, if valid, most helps to justify the agricultural economist's reasoning?", "answers": "['Economic growth should not be pursued at the expense of a loss of biodiversity.', 'Agricultural production should be reduced if doing so would increase biodiversity.', 'Preserving biodiversity is no more important than increasing agricultural production.', 'Economic growth should be sustained only as long as agricultural production continues to increase.']", "label": 0 }, { "id": "train_3511", "context": "Advertisement: Fabric-Soft leaves clothes soft and fluffy, and its fresh scent is a delight. We conducted a test using over 100 consumers to prove Fabric-Soft is best. Each consumer was given one towel washed with Fabric-Soft and one towel washed without it. Ninety-nine percent of the consumers preferred the Fabric-Soft towel. So Fabric-Soft is the most effective fabric softener available.", "question": "The advertisement's reasoning is most vulnerable to criticism on the grounds that it fails to consider whether", "answers": "['Fabric-Soft is more or less harmful to the environment than other fabric softeners', 'the consumers tested had the opportunity to evaluate fabric softeners other than Fabric-Soft', 'any of the consumers tested are allergic to fabric softeners', 'the consumers tested find the benefits of using fabric softeners worth the expense']", "label": 1 }, { "id": "train_3512", "context": "Hea Sook: One should not readily believe urban legends. Most legends are propagated because the moral lesson underlying them supports a political agenda. People will repeat a tale if it fits their purpose. They may not deliberately spread untruths, but neither are they particularly motivated to investigate deeply to determine if the tale they are telling is true. Kayla: But people would not repeat stories that they did not believe were true. Ttherefore, one can safely assume that if a story has been repeated by enough people then it is more likely to be true.", "question": "Kayla's reply is most vulnerable to the criticism that it", "answers": "['fails to address the claim that people will not verify the truth of a story that fits their purpose', 'overstates the significance of political agendas in the retelling of stories', 'does not specify how many people need to repeat a story before someone is justified believing it', 'implicitly supports the claim that the people repeating legends are not deliberately spreading untruths']", "label": 0 }, { "id": "train_3513", "context": "Mice that have been given morphine are very likely to develop blood poisoning because bacteria that normally reside in the intestine typically respond to morphine by migrating into the bloodstream. However, when mice are given both morphine and the new drug naltrexone, blood poisoning is much less frequent, although it does still occur. These results provide support for researchers' prediction that naltrexone will turn out to be toxic to certain types of bacteria.", "question": "Which of the following, if discovered to be true, would most seriously weaken the support for the researchers' prediction?", "answers": "['After being administered to mice, naltrexone does not pass from the bloodstream into the intestine.', 'Naltrexone inhibits morphine from triggering the migration of intestinal bacteria into the bloodstream.', 'Mice that have been given naltrexone but not morphine have no greater risk of developing blood poisoning than do mice that have not been given either substance.', 'The increased risk of blood poisoning is not the only harmful effect on mice of being given morphine.']", "label": 1 }, { "id": "train_3514", "context": "Public health experts have waged a long-standing educational campaign to get people to eat more vegetables, which are known to help prevent cancer. Unfortunately, the campaign has had little impact on people' s diets. The reason is probably that many people simply dislike the taste of most vegetables. Thus, the campaign would probably be more effective if it included information on ways to make vegetables more appetizing.", "question": "Which one of the following, if true, most strengthens the argument?", "answers": "['The campaign to get people to eat more vegetables has had little impact on the diets of most people who love the taste of vegetables.', 'People who dislike the taste of most vegetables would eat many more vegetables if they knew how to make them more appetizing.', 'People who find a few vegetables appetizing typically do not eat substantially more vegetables than do people who dislike the taste of most vegetables.', \"Some ways of making vegetables more appetizing diminish vegetables' ability to help prevent cancer.\"]", "label": 1 }, { "id": "train_3515", "context": "Certain methods of creating high-quality counterfeit banknotes involve making accurate measurements of the images printed on genuine banknotes. Hence, if the production of high-quality counterfeit banknotes is to be prevented, some of the images on banknotes must be made very difficult or impossible to measure accurately.", "question": "The argument's conclusion can be properly drawn if which one of the following is assumed?", "answers": "[\"Once the images printed on a banknote have been measured accurately, there is no further impediment to the banknote's being exactly replicated.\", \"Today's copying technology is sophisticated enough to replicate almost any paper product with great precision.\", 'Governments have better printing technology available to them than counterfeiters do.', 'Few countries produce banknotes with images that are difficult for counterfeiters to measure accurately.']", "label": 0 }, { "id": "train_3516", "context": "Antinuclear activist: The closing of the nuclear power plant is a victory for the antinuclear cause. It also represents a belated acknowledgment by the power industry that they cannot operate such plants safely. Nuclear power plant manager: It represents no such thing. The availability of cheap power from nonnuclear sources, together with the cost of mandated safety inspections and safety repairs, made continued operation uneconomic. Thus it was not safety considerations but economic considerations that dictated the plant' s closing.", "question": "The reasoning in the manager's argument is flawed because the argument", "answers": "['overlooks the possibility that the sources from which cheap power is available might themselves be subject to safety concerns', \"takes as one of its premises a view about the power industry's attitude toward nuclear safety that contradicts the activist's view\", 'fails to acknowledge that the power industry might now believe nuclear power plants to be unsafe even though this plant was not closed for safety reasons', 'counts as purely economic considerations some expenses that arise as a result of the need to take safety precautions']", "label": 3 }, { "id": "train_3517", "context": "A member of the British Parliament is reputed to have said, \"The first purpose of good social reform is to increase the sum total of human happiness. So, any reform which makes somebody happy is achieving its purpose. Since the reform I propose would make my constituents happy, it is a good social reform. ", "question": "Which one of the following, if true, most seriously weakens the argument attributed to the member of Parliament?", "answers": "['Different things make different people happy.', \"Increasing some people's happiness might not increase the sum total of human happiness if others are made unhappy.\", 'Good social reforms usually have widespread support.', \"The proposed reform would affect only the member of Parliament's constituents and would make them happy.\"]", "label": 1 }, { "id": "train_3518", "context": "For a contest at the upcoming county fair, Marie advocates for a ring toss because the ring toss is consistently the favorite activity of the children who attend the fair. Anne, however, points out that adults win most of the prizes when playing ring toss. To make the chances of winning fair for everyone, Anne proposes a raffle. Marie rebuts that those who can afford to spend more on raffle tickets will have better chances of winning. The two agree to hold a raffle in which each person can buy only one ticket.", "question": "The two boldface portions in the argument above are best described by which of the following statements?", "answers": "['The first is an argument promoting one activity; the second invalidates the reason for promoting the other activity.', 'The first is the conclusion of one of the participants; the second is an argument that supports that conclusion.', \"The first is the conclusion of one of the participants; the second invalidates the other participant's reasoning.\", 'The first is an argument promoting one activity; the second is an argument against promoting that activity.']", "label": 0 }, { "id": "train_3519", "context": "The television star Markus Hermann refuses to grant interviews with newspapers unless he is given the right to approve the article before publication. The Greyburg Messenger newspaper refuses to do anything that its editors believe will compromise their editorial integrity.", "question": "So the Messenger will not interview Hermann, since The conclusion of the argument is properly drawn if which one of the following completes the passage?", "answers": "['the editors of the Messenger believe that giving an interviewee the right to approve an article before publication would compromise their editorial integrity', 'Hermann believes that the Messenger frequently edits interviews in ways that result in unflattering portrayals of the interviewees', 'the Messenger has never before given an interviewee the right to approve an article before publication', 'Hermann usually requests substantial changes to interview articles before approving them']", "label": 0 }, { "id": "train_3520", "context": "When interest rates are high, insurance companies reduce the premiums they charge for many kinds of insurance policies. The reason is that insurance companies want to take in as much money as possible in premiums so that they can invest the money at high rates of interest. And premium reductions help achieve this objective, since __.", "question": "Which of the following most logically completes the argument below?", "answers": "['an increase in the number of policies sold eventually leads to an increase in the number of claims that an insurance company has to pay', 'smaller insurance companies are not able to amass enough money to take advantage of investing at high interest rates', 'insurance companies can sell many more insurance policies if they charge lower premiums than they would if they left premiums unchanged', 'interest rates are likely to decrease when large amounts of money are available for loans']", "label": 2 }, { "id": "train_3521", "context": "A certain species of bird has two basic varieties crested and noncrested. The birds, which generally live in flocks that contain only crested or only noncrested birds, tend to select mates of the same variety as themselves. However, if a bird that is raising a flock in which all other members are crested is later moved to a mixed flock, then that bird -- whether crested or noncrested -- is likely to select a crested mate. This fact indicates that the birds' preference for crested or noncrested mates is learned rather than genetically determined.", "question": "Which one of the following, if true, provides the most support for the argument?", "answers": "['In general there are few behavioral differences between the crested and noncrested birds of the species.', 'Birds of other species also tend to show preferences for mates that have one or another specific physical feature.', 'Both the crested and noncrested birds of the species tend to select mates that are similar to themselves in size and age.', 'If a bird of the species is raised in a flock that contains both crested and noncrested birds, that bird shows no preference for one variety or the other in its selection of a mate.']", "label": 3 }, { "id": "train_3522", "context": "Nations that have little interaction with one another have little knowledge of one another' s needs and problems. Because both sympathy and justice depend largely on understanding the needs and problems of others, it follows that__.", "question": "Which one of the following most logically completes the argument?", "answers": "[\"only nations that have some interaction with one another have knowledge of one another's needs and problems\", 'without some interaction, nations are bound to find it difficult to extend sympathy and justice to one another', 'there is no way to eliminate conflict among nations', 'almost all problems between nations stem from lack of sympathy and justice']", "label": 1 }, { "id": "train_3523", "context": "Mate is a beverage found in much of South America. While it is uncertain where mate was first made, there are more varieties of it found in Paraguay than anywhere else. Also, mate is used more widely there than anywhere else. Ttherefore, Paraguay is likely the place where mate originated.", "question": "Which one of the following, if true, would most strengthen the argument?", "answers": "['Many Paraguayans believe that the best mate is found in Paraguay.', 'It is rare for there to be a great variety of types of a beverage in a place where the beverage has not been in use for a very long time.', 'Typically, the longer a beverage has been in use in a particular place, the more widely that beverage is used there.', 'There are few places outside of South America where mate is regularly consumed.']", "label": 2 }, { "id": "train_3524", "context": "Some people believe that saying that an organization is hierarchical says everything there is to say about how that organization operates. All bureaucratically controlled organizations are hierarchical. Yet the Public Works Department, although bureaucratically controlled, operates quite differently than most other bureaucratically controlled organizations operate.", "question": "If the statements above are true, which one of the following must also be true on the basis of them?", "answers": "['Not all hierarchical organizations operate in the same way.', 'The Public Works Department operates more like a nonbureaucratically controlled organization than like a bureaucratically controlled organization.', 'Whether or not an organization is bureaucratically controlled has nothing to do with how that organization operates.', 'From the fact that a given organization is hierarchical nothing can reliably be concluded about how that organization operates.']", "label": 0 }, { "id": "train_3525", "context": "In a recent experiment, half the subjects were expert chess players; the other half had never played. First, the subjects were shown chessboards at various stages of actual games for ten seconds. Each expert was able to recall the positions of the pieces at least twice as well as any nonplayer. Second, the subjects were shown chessboards with the pieces randomly arranged, again for ten seconds. In this case the experts recalled the positions only marginally better, on average, than did the nonplayers.", "question": "Which one of the following is most strongly supported by the results of the experiment?", "answers": "['Ten seconds is not long enough to memorize random information.', 'People who play chess have better memories than people who do not play chess.', 'An expert is more likely than a nonexpert to attempt to assimilate new information according to known rules and principles.', 'People remember information better when they can organize it according to rules that they know.']", "label": 3 }, { "id": "train_3526", "context": "If the law punishes littering, then the city has an obligation to provide trash cans. But the law does not punish littering, so the city has no such obligation.", "question": "Which one of the following exhibits a flawed pattern of reasoning most similar to that in the argument above?", "answers": "['If today is a holiday, then the bakery will not be open. The bakery is not open for business. Thus today is a holiday.', 'In the event that my flight had been late, I would have missed the committee meeting. Fortunately, my flight is on time. Ttherefore, I will make it to the meeting.', 'The new regulations will be successful only if most of the students adhere to them. Since most of the students will adhere to those regulations, the new regulations will be successful.', 'When the law is enforced, some people are jailed. But no one is in jail, so clearly the law is not enforced.']", "label": 1 }, { "id": "train_3527", "context": "Science fiction creates an appetite for interstellar space exploration among certain people. Unfortunately, this appetite cannot be satisfied with any technology humanity will soon possess. Since gaps between expectations and reality spur discontent, no doubt one effect of science fiction has been to create an unproductive dissatisfaction with the way the world actually is.", "question": "Which one of the following is an assumption the argument requires?", "answers": "[\"The fact that the appetite for interstellar space exploration cannot be satisfied with any technology humanity will soon possess has created a gap between reality and some people's expectations.\", 'If science fiction has created an unproductive dissatisfaction with the way the world actually is, it has done so only by creating an appetite for interstellar space exploration among certain people.', 'If the appetites science fiction has created in people could all be satisfied with technologies that humanity will soon possess, then science fiction could not create an unproductive dissatisfaction with the way the world is.', 'Most people unrealistically expect that technology that humanity will soon possess could satisfy the appetite for interstellar space exploration.']", "label": 0 }, { "id": "train_3528", "context": "In an experiment testing whether hyperactivity is due to a brain abnormality, the brain activity of 25 hyperactive adults was compared to the brain activity of 25 adults who were not hyperactive. The tests revealed that the hyperactive adults had much less brain activity in the premotor cortex, a region of the brain believed to control action, than did the nonhyperactive adults. The experimenters concluded that diminished activity in the premotor cortex is one cause of hyperactivity.", "question": "Which one of the following, if true, most undermines the conclusion drawn by the experimenters?", "answers": "['All of the hyperactive adults in the study had been treated for hyperactivity with a medication that is known to depress activity in some regions of the brain, while none of the nonhyperactive adults had been so treated.', 'The test was performed only on adults because even though the method by which the test measured brain activity is harmless to adults, it does require the use of radiation, which could be harmful to children.', 'Some of the nonhyperactive adults in the study had children who suffer from hyperactivity.', 'The neuropsychologists who designed the experiment were not present when the tests were performed.']", "label": 0 }, { "id": "train_3529", "context": "In a study, one group of volunteers was fed a high-protein, low-carbohydrate diet; another group was fed a low-protein, high-carbohydrate diet. Both diets contained the same number of calories, and each volunteer' s diet prior to the experiment had contained moderate levels of proteins and carbohydrates. After ten days, those on the low-carbohydrate diet had lost more weight than those on the high-carbohydrate diet. Thus, the most effective way to lose body fat is to eat much protein and shun carbohydrates.", "question": "Which one of the following, if true, most weakens the argument above?", "answers": "['A low-protein, high-carbohydrate diet causes the human body to retain water, the added weight of which largely compensates for the weight of any body fat lost, whereas a high-protein, low-carbohydrate diet does not.', 'In the experiment, the volunteers on the high- carbohydrate diet engaged in regular exercise of a kind known to produce weight loss, and those on the low-carbohydrate diet did not.', 'Many of the volunteers who had been on the low-carbohydrate diet eventually regained much of the weight they had lost on the diet after returning to their normal diets. GO ON TO THE NEXT PAGE.', 'A high-protein, low-carbohydrate diet will often enable the human body to convert some body fat into muscle, without causing any significant overall weight loss.']", "label": 0 }, { "id": "train_3530", "context": "Ethicist: In a recent judicial decision, a contractor was ordered to make restitution to a company because of a bungled construction job, even though the company had signed a written agreement prior to entering into the contract that the contractor would not be financially liable should the task not be adequately performed. Thus, it was morally wrong for the company to change its mind and seek restitution.", "question": "Which one of the following principles, if valid, most helps to justify the ethicist's reasoning?", "answers": "['It is morally wrong to seek a penalty for an action for which the agent is unable to make restitution.', 'It is morally wrong for one party not to abide by its part of an agreement only if the other party abides by its part of the agreement.', 'It is morally wrong for one person to seek to penalize another person for an action that the first person induced the other person to perform.', 'It is morally wrong to seek compensation for an action performed in the context of a promise to forgo such compensation.']", "label": 3 }, { "id": "train_3531", "context": "The fact that politicians in a certain country are trying to reduce government spending does not by itself explain why they have voted to eliminate all government-supported scholarship programs. Government spending could have been reduced even more if instead they had cut back on military spending.", "question": "Which one of the following arguments is most similar in its reasoning to the argument above?", "answers": "[\"The fact that Thelma's goal is to become famous does not by itself explain why she took up theatrical acting. It is easier to become famous through writing or directing plays than through theatrical acting.\", 'The fact that Sallie and Jim have different work styles does not by itself explain why they could not work together. Sallie and Jim could have resolved their differences if they had communicated more with one another when they began to work together.', 'The fact that Brooks has a part-time job does not by itself explain why he is doing poorly in school. Many students with part-time jobs are able to set aside enough time for study and thus maintain high grades.', 'The fact that Roger wanted more companionship does not by itself explain why he adopted ten cats last year. He would not have adopted them all if anyone else had been willing to adopt some of them.']", "label": 0 }, { "id": "train_3532", "context": "Albumin, one element of blood plasma, is an effective substitute for plasma in treating or preventing shock, the collapse of blood vessels due to a drop in the liquid volume of the bloodstream. Injected into the bloodstream, albumin absorbs enough liquid from surrounding tissues to keep the blood vessels open. It has an advantage over plasma because it can be made available for emergencies while taking up a fraction of the shipping and storage space of plasma.", "question": "Of the following, which one best illustrates the principle illustrated by the stated advantage of using albumin as a substitute for plasma?", "answers": "['A newspaper can increase advertising revenue without increasing its bulk or weight if it is printed in a smaller format.', 'Hardbound books can be a better choice than paperbacks despite their extra bulk if the books need to be more durable than paperbacks.', 'An advantage that compact discs have over vinyl records is that they are also used for storing information for computers, so advances in computer disc technology will probably apply to compact discs used for music.', 'Dehydrated foods have much the same food value as ordinary foods and are more convenient for hikers because they are lighter and take up less space in backpacks.']", "label": 3 }, { "id": "train_3533", "context": "Fishery officials are still considering options for eliminating Lake Davis' s population of razor-toothed northern pike, a fierce game fish that could threaten salmon and trout populations if it slips into the adjoining river system. Introducing pike-specific diseases and draining the lake have been ruled out. Four years ago, poison was added to the lake in order to eliminate the pike. This outraged local residents, because the water remained tainted for months and the region' s tourism economy suffered.", "question": "Which one of the following is most strongly supported by the information above?", "answers": "[\"Salmon and trout populations in the Lake Davis area are essential to the region's economy.\", 'The poison added to the lake four years ago was not successful in ridding the lake of the pike.', 'Four years ago was the only time that poison was used against the pike in the lake.', \"Draining the lake would not cause the region's tourism economy to suffer.\"]", "label": 1 }, { "id": "train_3534", "context": "Columnist: Many car manufacturers trumpet their cars' fuel economy under normal driving conditions. For all three of the cars I have owned, I have been unable to get even close to the fuel economy that manufacturers advertise for cars of those makes. So manufacturers probably inflate those numbers.", "question": "The reasoning in the columnist's argument is most vulnerable to criticism on the grounds that the argument", "answers": "['uses the term \"fuel economy\" in two different senses', 'draws a conclusion on the basis of a sample that is too small', 'presumes, without providing justification, that driving conditions are the same in every geographical region', 'presumes, without providing justification, that car manufacturers knowingly market cars that fail to meet minimum fuel efficiency standards']", "label": 1 }, { "id": "train_3535", "context": "Research indicates that college professors generally were raised in economically advantaged households. For it was discovered that, overall, college professors grew up in communities with average household incomes that were higher than the average household income for the nation as a whole.", "question": "The reasoning in the argument is flawed because the argument", "answers": "['presumes without justification that college professors generally were raised in households with incomes that are average or above average for their communities', 'fails to note there are some communities with high average household incomes in which no college professors grew up', 'fails to take into account the fact that many college professors live in rural communities, which generally have low average household incomes', 'does not take into account the fact that college professors generally have lower salaries than their counterparts in the private sector']", "label": 0 }, { "id": "train_3536", "context": "The kind of thoughts that keep a person from falling asleep can arise in either half of the brain. Ttherefore, a person being prevented from sleeping solely by such thoughts would be able to fall asleep by closing the eyes and counting sheep, because this activity fully occupies the left half of the brain with counting and the right half of the brain with imagining sheep, thereby excluding the sleep-preventing thoughts.", "question": "According to the hypothesis, for a person to use counting imaginary sheep as an effective method of inducing sleep, which one of the following must be true?", "answers": "['The person is able to imagine a wide variety of things.', \"Thoughts of sheep rarely, if ever, arise in the person's dreams.\", 'Thoughts of sheep would not keep the person awake at that time.', 'Thoughts of sheep would induce sleep in the person whenever those thoughts arose.']", "label": 2 }, { "id": "train_3537", "context": "When officials in Tannersburg released their plan to widen the city's main roads, environmentalists protested that widened roads would attract more traffic and lead to increased air pollution. In response, city officials pointed out that today's pollution-control devices are at their most effective in vehicles traveling at higher speeds and that widening roads would increase the average speed of traffic. However, this effect can hardly be expected to offset the effect pointed out by environmentalists, since __.", "question": "Which of the following most logically completes the argument?", "answers": "[\"a vehicle traveling on a route that goes through Tannersburg will spend less time on Tannersburg's roads once the roads are widened\", 'most of the air pollution generated by urban traffic comes from vehicles that do not have functioning pollution-control devices', 'several of the roads that are slated for widening will have to be closed temporarily while construction is underway', 'increases in traffic volume generally produce decreases in the average speed of traffic unless roads are widened']", "label": 1 }, { "id": "train_3538", "context": "The population of desert tortoises in Targland' s Red Desert has declined, partly because they are captured for sale as pets and partly because people riding all-terrain vehicles have damaged their habitat. Targland plans to halt this population decline by blocking the current access routes into the desert and announcing new regulations to allow access only on foot. Targland' s officials predict that these measures will be adequate, since it is difficult to collect the tortoises without a vehicle.", "question": "Which of the following would it be most important to establish in order to evaluate the officials' prediction?", "answers": "['Whether people who travel on foot in the Red Desert often encounter the tortoises', 'Whether Targland is able to enforce the regulations with respect to all-terrain vehicle entry at points other than the current access routes', 'Whether the Targland authorities held public hearings before restricting entry by vehicle into the Red Desert', 'Whether possessing the tortoises as pets remains legally permissible in Targland']", "label": 1 }, { "id": "train_3539", "context": "A recent study of 10, 000 people who were involved in automobile accidents found that a low percentage of those driving large automobiles at the time of their accidents were injured, but a high percentage of those who were driving small automobiles at the time of their accidents were injured. Thus, one is less likely to be injured in an automobile accident if one drives a large car rather than a small car.", "question": "Which one of the following, if true, most seriously weakens the argument?", "answers": "['Most people who own small cars also drive large cars on occasion.', 'Only a small percentage of those people involved in an automobile accident are injured as a result.', 'Half of the study participants drove mediumsized cars at the time of their accidents.', 'A large automobile is far more likely to be involved in an accident than is a small automobile.']", "label": 3 }, { "id": "train_3540", "context": "Spokesperson: Contrary to what some have claimed, our group' s \"Clean City\" campaign has been a rousing success. After all, the amount of trash on the city' s streets today is significantly lower than when the campaign began.", "question": "Which one of the following is an assumption required by the spokesperson's argument?", "answers": "['Those who claim that the campaign has not been a rousing success are unaware of the degree of the decline in the amount of trash since the campaign began.', \"The amount of trash on the city's streets was not declining at the same rate or faster before the campaign began than it did during the campaign.\", \"The amount of trash on the city's streets has declined steadily throughout the course of the campaign.\", \"The campaign has been more successful in reducing the amount of trash on the city's streets than has any other campaign in the past.\"]", "label": 1 }, { "id": "train_3541", "context": "A significant amount of the acquisition budget of a typical university library is spent on subscriptions to scholarly journals. Over the last several years, the average subscription rate a library pays for such a journal has increased dramatically, even though the costs of publishing a scholarly journal have remained fairly constant. Obviously, then, in most cases publishing a scholarly journal must be much more profitable now than it was several years ago.", "question": "Which one of the following, if true, most seriously weakens the argument?", "answers": "['Many university libraries have begun to charge higher and higher fines for overdue books and periodicals as a way of passing on increased journal subscription costs to library users.', 'Publishing a scholarly journal is an expensive enterprise, and publishers of such journals cannot survive financially if they consistently lose money.', 'The majority of scholarly journals are published no more than four times a year.', 'Most subscribers to scholarly journals are individuals, not libraries, and the subscription rates for individuals have generally remained unchanged for the past several years.']", "label": 3 }, { "id": "train_3542", "context": "Scientist: While studying centuries-old Antarctic ice deposits, I found that several years of relatively severe atmospheric pollution in the 1500s coincided with a period of relatively high global temperatures. So it is clear in this case that atmospheric pollution did cause global temperatures to rise.", "question": "The reasoning in the scientist's argument is most vulnerable to criticism on the grounds that the argument", "answers": "['draws a general conclusion based on a sample that is likely to be unrepresentative', 'takes for granted that the method used for gathering data was reliable', 'infers, merely from a claim that two phenomena are associated, that one phenomenon causes the other', 'inappropriately generalizes from facts about a specific period of time to a universal claim']", "label": 2 }, { "id": "train_3543", "context": "Recent research shows that sound change (pronunciation shift) in a language is not gradual. New sounds often emerge suddenly. This confounds the classical account of sound change, whose central tenet is gradualness. Since this classical account must be discarded, sound-change theory in general must also be.", "question": "Which one of the following, if assumed, does most to justify the argument's conclusion?", "answers": "['The emergence of new sounds appears to be random.', 'All theories of sound change rely heavily on the classical theory.', 'For most languages, historical records of their earlier stages are scarce or nonexistent.', 'The data on which the classical account of sound-change theory was based are now known to be inaccurate.']", "label": 1 }, { "id": "train_3544", "context": "The average temperature of the lobster-rich waters off the coast of Foerkland has been increasing for some years. In warmer water, lobsters grow faster. In particular, lobster larvae take less time to reach the size at which they are no longer vulnerable to predation by young cod, the chief threat to their survival. Consequently, the survival rate of lobster larvae must be going up, and the lobster population in Foerkland' s coastal waters is bound to increase.", "question": "Which of the following, if true, most seriously weakens the argument?", "answers": "[\"Cod are a cold-water species, and the increasing water temperatures have caused a northward shift in Foerkland's cod population.\", 'There are indications that in recent years the fishing fleet operating off the coast of Foerkland has been taking cod at an unsustainably high rate.', 'The increase in water temperatures off Foerkland has not been as pronounced as the increase in average soil temperatures in Foerkland.', 'Because of their speeded-up growth, lobsters now get large enough to be legal catch before they reach reproductive maturity.']", "label": 3 }, { "id": "train_3545", "context": "In scientific journals, authors and reviewers have praised companies in which they have substantial investments. These scientists, with their potential conflict of interest, call into question the integrity of scientific inquiry, so there should be full public disclosure of scientific authors' commercial holdings.", "question": "Which one of the following conforms most closely to the principle illustrated by the argument above?", "answers": "['People with access to otherwise private information regarding the value of stocks should not be allowed to sell or purchase those stocks.', 'Financial advisers should inform their clients about any incentives the advisers receive for promoting investments in particular companies.', 'Magazine publishers should not be allowed to invest in the companies that advertise in their magazines.', 'Claims about the effectiveness of pharmaceuticals should be based on scientific studies.']", "label": 1 }, { "id": "train_3546", "context": "Prof. Hernandez' s monumental work The History of Central America covers everything about the region from the origin of the Mesoamerican period to the end of the Cold War. While the book has several informative maps and charts, many of the chapters spend less time describing facts and more time explaining Prof. Hernandez' s theories. Indeed, the last two chapters consist exclusively of his exposition of theory of the role of Central America in post WWII world politics. Ttherefore, properly speaking, this book is not a history book.", "question": "Which of the following is an assumption that supports drawing the conclusion above from the reasons given for that conclusion?", "answers": "['History as a discipline is concerned only with historical facts, not with the theoretical explanations of those facts.', 'Historians should be more explicit than most are now about the theoretical framework with which they write.', 'Different historians have different understanding of the relative importance between facts and theories within the study of history.', 'Most books that present a wealth of historical facts include maps and charts as well.']", "label": 0 }, { "id": "train_3547", "context": "Some people have questioned why the Homeowners Association is supporting Cooper' s candidacy for mayor. But if the Association wants a mayor who will attract more businesses to the town, Cooper is the only candidate it could support. So, since the Association is supporting Cooper, it must have a goal of attracting more businesses to the town.", "question": "The reasoning in the argument is in error because", "answers": "['the Homeowners Association might discover that attracting more businesses to the town would not be in the best interest of its members', 'Cooper might not have all of the skills that are needed by a mayor who wants to attract businesses to a town', 'the reasons the Homeowners Association should want to attract more businesses to the town are not given', \"the Homeowners Association could be supporting Cooper's candidacy for reasons unrelated to attracting businesses to the town\"]", "label": 3 }, { "id": "train_3548", "context": "Technological improvements and reduced equipment costs have made converting solar energy directly into electricity far more cost-efficient in the last decade. However, the threshold of economic viability for solar power (that is, the price per barrel to which oil would have to rise in order for new solar power plants to be more economical than new oil-fired power plants) is unchanged at thirty-five dollars.", "question": "Which of the following, if true, does most to help explain why the increased cost-efficiency of solar power has not decreased its threshold of economic viability?", "answers": "['Technological changes have increased the efficiency of oil-fired power plants.', 'Most electricity is generated by coal-fired or nuclear, rather than oil-fired, power plants.', 'The cost of oil has fallen dramatically.', 'When the price of oil increases, reserves of oil not previously worth exploiting become economically viable.']", "label": 0 }, { "id": "train_3549", "context": "Conservation officers justified their decision to remove a pack of ten coyotes from a small island by claiming that the coyotes, which preyed on wild cats and plover, were decimating the plover population and would soon wipe it out. After the coyotes were removed, however, the plover population plummeted dramatically, and within two years plover could no longer be found on the island.", "question": "Which one of the following would, if true, most help explain the phenomenon described above?", "answers": "['The coyotes preyed mainly on wild cats, and wild cats prey on plover.', 'Plover are ground-nesting birds, which makes them easy prey for coyotes.', 'Coyotes are not susceptible to any of the diseases that commonly infect plover or wild cats.', 'The wild cat population on the island was once significantly larger than it is currently.']", "label": 0 }, { "id": "train_3550", "context": "Historian: Roman nobles, known as patricians, often owned dozens of slaves. It was extremely common in ancient Rome. Roman society did not consider the practice to be immoral or illegal in any way. Rome would simply enslave the many people conquered by the Empire. A fresh supply of slaves was integral to sustaining the Roman Empire.", "question": "Which one of the following is most strongly supported by the historian's argument?", "answers": "['Conquered people welcomed their new lives as slaves.', 'Slavery was a necessary evil for the Romans.', 'The Roman Empire would have collapsed earlier without enslaving new peoples.', 'Romans treated their slaves with more humanity, compassion, and respect than any other contemporary civilization.']", "label": 2 }, { "id": "train_3551", "context": "Politician: A major social problem is children hurting other children. The results of a recent experiment by psychologists establish that watching violent films is at least partly responsible for this aggressive behavior. The psychologists conducted an experiment in which one group of children watched a film of people punching Bobo the Clown dolls. A second group of children was not shown the film. Afterward, both groups of children played together in a room containing a Bobo doll. Most of the children who had seen the film punched the Bobo doll, while most of the other children did not.", "question": "Which one of the following, if true, most weakens the politician's argument?", "answers": "['Some children who had not been shown the film imitated the behavior of those who had been shown the film and who punched the doll.', 'The child who punched the Bobo doll the hardest and the most frequently had not been shown the film.', 'Some of the children who did not punch the Bobo doll, including some who had been shown the film, chastised those who did punch the doll.', 'The children who had been shown the film were found to be no more likely than the children who had not been shown the film to punch other children.']", "label": 3 }, { "id": "train_3552", "context": "Constance: The traditional definition of full employment as a 5 percent unemployment rate is correct, because at levels below 5 percent, inflation rises. Brigita: That traditional definition of full employment was developed before the rise of temporary and part-time work and the fall in benefit levels. When people are juggling several part-time jobs with no benefits, or working in a series of temporary assignments, as is now the case, 5 percent unemployment is not full employment.", "question": "The dialogue most strongly supports the claim that Constance and Brigita disagree with each other about which one of the following?", "answers": "['whether a person with a part-time job should count as fully employed', 'what definition of full employment is applicable under contemporary economic conditions', 'whether it is a good idea, all things considered, to allow the unemployment level to drop below 5 percent', 'whether unemployment levels above 5 percent can cause inflation levels to rise']", "label": 1 }, { "id": "train_3553", "context": "Valitania' s long-standing practice of paying high salaries to its elected politicians has had a disastrous effect on the level of integrity among politicians in that country. This is because the prospect of earning a high salary is always attractive to anyone whose primary aim in life is to make money, so that inevitably the wrong people must have been attracted into Valitanian politics: people who are more interested in making money than in serving the needs of the nation.", "question": "Which one of the following, if true, would weaken the argument?", "answers": "['Most of those who are currently Valitanian politicians could have obtained better-paid work outside politics.', 'Many Valitanian candidates for elected office spend some of their own money to finance their campaigns.', 'Most Valitanian elective offices have four-year terms.', 'No more people compete for elected office when officeholders are paid well than when they are paid poorly.']", "label": 0 }, { "id": "train_3554", "context": "One approach to the question of which objects discussed by a science are real is to designate as real all and only those entities posited by the most explanatorily powerful theory of the science. But since most scientific theories contain entities posited solely on theoretical grounds, this approach is flawed.", "question": "Which one of the following principles, if valid, most helps to justify the reasoning above?", "answers": "['A scientific theory should not posit any entity that does not enhance the explanatory power of the theory.', 'Any object that is posited by a scientific theory and that enhances the explanatory power of that theory should be designated as real.', 'Only objects posited by explanatorily powerful theories should be designated as real.', 'Objects posited for theoretical reasons only should never be designated as real.']", "label": 3 }, { "id": "train_3555", "context": "Hank is a professional writer. He submits regular columns at two blogs and self-publishes romance novels. Hank recently signed with an agent based in New York. To date, Hank has never made any money off his writing.", "question": "The strength of the argument depends on which of the following?", "answers": "['Being a professional writer does not require earning money.', 'Being a professional writer requires representation by an agent.', \"Hank's self-published novels and blogs have received generally positive reviews.\", \"Hank's agent works at the biggest firm in New York.\"]", "label": 0 }, { "id": "train_3556", "context": "X: In order to reduce the amount of plastic in landfills, legislatures should impose a ban on the use of plastics for packaging goods. Y: Impossible! Plastic packaging is necessary for public safety. Consumers will lose all of the safety features that plastic offers, chiefly tamper-resistant closures and shatterproof bottles.", "question": "Which of the following best describes the weak point in Y's response to X's proposal?", "answers": "['Y attempts to shift the blame for the large amount of plastic in landfills from the users of plastic packaging to the legislators.', 'Y wrongly assumes that X defends the interests of the manufacturers rather than the interests of the consumers.', 'Y does not consider the concern of some manufacturers that safety features spoil package appearances.', 'Y ignores the possibility that packaging goods in materials other than plastic might provide the same safety features that packaging in plastic offers.']", "label": 3 }, { "id": "train_3557", "context": "Several years ago, as a measure to reduce the population of gypsy moths, which depend on oak leaves for food, entomologists introduced into many oak forests a species of fungus that is poisonous to gypsy moth caterpillars. Since then, the population of both caterpillars and adult moths has significantly declined in those areas. Entomologists have concluded that the decline is attributable to the presence of the poisonous fungus.", "question": "Which one of the following, if true, most strongly supports the conclusion drawn by the entomologists?", "answers": "['The current decline in the gypsy moth population in forests where the fungus was introduced is no greater than a decline that occurred concurrently in other forests.', 'In the past several years, air pollution and acid rain have been responsible for a substantial decline in oak tree populations.', 'A strain of gypsy moth whose caterpillars are unaffected by the fungus has increased its share of the total gypsy moth population.', 'An increase in numbers of both gypsy moth caterpillars and gypsy moth adults followed a drop in the number of some of the species that prey on the moths.']", "label": 2 }, { "id": "train_3558", "context": "Secret passwords are often used to control access to computers. When employees are allowed to make up their own passwords, they are likely to setup as passwords their initials or birth dates. To improve security, employers should assign randomly generated passwords to employees rather than allowing employees to make up their own", "question": "Which of the following, if true, most seriously undermines the conclusion drawn above?", "answers": "['Randomly generated passwords are so difficult for employees to recall that they often record the passwords in places where the passwords could be easily seen by others.', 'Control of physical access to computers by the use of locked doors and guards should be used in addition to passwords in order to maintain security', 'If passwords are generated randomly, it is theoretically possible that employees will be assigned passwords that they might have selected on their own.', 'Computer systems protected by passwords are designed to ignore commands that are entered by employees or others who use invalid passwords.']", "label": 0 }, { "id": "train_3559", "context": "The obesity invariably associated with some high-fat diets is caused by an absence in these diets of certain nutrients that are necessary for an active metabolism, not by excessive caloric intake. Hence, people on these high-fat diets do not consume too many calories.", "question": "The questionable pattern of reasoning in the argument above is most similar to that in which one of the following?", "answers": "['The disorientation observed in airline pilots after transoceanic flights is caused not by sleep deprivation but by disruption in their exposure to daylight. Hence, transoceanic pilots do not suffer from sleep deprivation.', 'Stock market crashes are due, not to panic in the face of predicted economic downturns, but to mere rumormongering without any basis in fact. Hence, economic downturns cannot be accurately predicted.', 'The impression that most viewers of sports programming are beer drinkers is due not to mere stereotyping but to the vast number of beer commercials broadcast during televised sports. Hence, most beer drinkers are avid fans of sports programs.', 'Electrical storms are strongly correlated with precipitous drops in barometric pressure. So, electrical storms are caused by such drops in pressure, rather than by air turbulence.']", "label": 0 }, { "id": "train_3560", "context": "Physicist: Determinism is the view that every event has a preceding cause sufficient for its occurrence. That is, if determinism is true, then the events that are presently occurring could not have failed to occur given the state of the universe a moment ago. Determinism, however, is false because it is impossible to know the complete state of the universe at any given time since it is impossible to measure accurately both the position and velocity of any given subatomic particle at a particular time.", "question": "The physicist's reasoning is most vulnerable to criticism on which one of the following grounds?", "answers": "['That it is impossible to know the complete state of the universe at any given time does not imply that there is no complete state of the universe at that time.', 'That it is impossible to measure accurately both the position and velocity of any given subatomic particle does not imply that it is impossible to know either the position or velocity of all subatomic particles.', 'That the position and velocity of any given subatomic particle cannot be jointly measured with accuracy does not imply that this is the case for the position and velocity of all subatomic particles.', 'That it is impossible to measure accurately both the position and velocity of any given subatomic particle at a particular time does not imply that its position or velocity cannot be accurately measured separately.']", "label": 0 }, { "id": "train_3561", "context": "Statistics indicating a sudden increase in the incidence of a problem often merely reflect a heightened awareness of the problem or a greater ability to record its occurrence. Hence we should be wary of proposals for radical solutions to problems when those proposals are a reaction to new statistical data.", "question": "The argumentation conforms most closely to which one of the following principles?", "answers": "['A better cognizance of a problem does not warrant the undertaking of a radical solution to the problem.', 'Radical solutions to problems can cause other problems and end up doing more harm than good.', 'Statistical data should not be manipulated to make a radical solution to a problem seem more justified than it actually is.', 'Proposals for radical solutions to problems should be based on statistical data alone.']", "label": 0 }, { "id": "train_3562", "context": "An economist has argued that consumers often benefit when government permits a corporation to obtain a monopoly. Without competition, a corporation can raise prices without spending nearly as much on advertising. The corporation can then invest the extra money in expensive research or industrial infrastructure that it could not otherwise afford, passing the fruits of these investments on to consumers.", "question": "Which one of the following, if true, most strengthens the economist's argument?", "answers": "['If a corporation obtains a monopoly, the disadvantage to consumers of any higher prices will be outweighed by the advantages from extra investments in expensive research or industrial infrastructure made by that corporation.', \"The government's permitting a corporation to obtain a monopoly is advantageous for consumers only if that corporation passes the fruits of at least some of its investments on to consumers.\", 'The benefits to consumers are typically greater if a corporation invests in expensive research or industrial infrastructure than if that corporation spends the same amount of money in any other way.', 'Even if a corporation is not permitted to obtain a monopoly, it typically invests some money in expensive research or industrial infrastructure.']", "label": 0 }, { "id": "train_3563", "context": "Davisville' s mayor: The Davisville Airport is actually located in the city of Millersburg. Millersburg officials have argued that it is illegal for restaurants in the Davisville Airport to serve alcoholic beverages because Millersburg has outlawed alcoholic beverages. But they are mistaken. Since the City of Davisville owns the Davisville Airport, and it is legal for restaurants to serve alcoholic beverages in Davisville, it is legal for them to do so in the Davisville Airport.", "question": "Which one of the following principles, if valid, most helps to justify the reasoning in the Davisville mayor's argument?", "answers": "[\"If one city owns a facility located in another city, government officials of both cities should settle through negotiation which city's laws apply in that facility.\", 'A facility located outside of a city is not subject to the laws of that city even if the facility is owned by the city.', \"Ownership takes precedence over any other factor in determining which city's laws apply in a facility owned by one city but located in another.\", 'If one city owns a facility located in another city and a dispute about legal jurisdiction arises, then the city that owns the facility should offer to sell it to the other city.']", "label": 2 }, { "id": "train_3564", "context": "Statistics reveal that more collisions between bicycles and motor vehicles occur on roads having specifically designated bicycle lanes than on roads having no such lanes. Hence, adding such lanes to existing roads is unlikely to enhance the safety of bicyclists.", "question": "The argument is most vulnerable to criticism on the grounds that it", "answers": "['overlooks the possibility that injuries sustained by bicyclists in accidents on roads with bicycle lanes are as serious, on average, as those sustained by bicyclists in accidents on roads without such lanes', 'takes for granted that any road alteration that enhances the safety of bicyclists also enhances the safety of motorists', 'concludes that adding bicycle lanes to roads will fail to enhance the safety of bicyclists on the grounds that only some roads that currently have such lanes are safe', 'fails to address the possibility that there are more bicyclists riding on roads with bicycle lanes than there are riding on roads without such lanes']", "label": 3 }, { "id": "train_3565", "context": "This region' s swimmers generally swim during the day because they are too afraid of sharks to swim after dark but feel safe swimming during daylight hours. Yet all recent shark attacks on swimmers in the area have occurred during the day, indicating that, contrary to popular opinion, it is not more dangerous to swim here at night than during the day.", "question": "The reasoning in the argument is most vulnerable to criticism on the grounds that it", "answers": "['presumes, without providing justification, that swimmers cannot be the most knowledgeable about which times of day are safest for swimming', 'overlooks the possibility that some sharks are primarily nocturnal hunters', \"overlooks the possibility that swimmers might feel anxiety caused by not being able to see one's surroundings in the dark\", 'fails to take into account the possibility that the number of shark attacks at night would increase dramatically if more people swam at night']", "label": 3 }, { "id": "train_3566", "context": "Almost all advances in genetic research give rise to ethical dilemmas. Government is the exclusive source of funding for most genetic research; those projects not funded by government are funded solely by corporations. One or the other of these sources of funding is necessary for any genetic research.", "question": "If all the statements above are true, then which one of the following must be true?", "answers": "['At least some advances in genetic research occur in projects funded by corporations.', 'Most genetic research funded by government results in advances that give rise to ethical dilemmas.', 'As long as government continues to fund genetic research, that research will give rise to ethical dilemmas.', 'No ethical dilemmas resulting from advances in genetic research arise without government or corporate funding.']", "label": 3 }, { "id": "train_3567", "context": "Many people suffer an allergic reaction to certain sulfites, including those that are commonly added to wine as preservatives. However, since there are several wine makers who add sulfites to none of the wines they produce, people who would like to drink wine but are allergic to sulfites can drink wines produced by these wine makers without risking an allergic reaction to sulfites.", "question": "Which of the following is an assumption on which the argument depends?", "answers": "['Wine is the only beverage to which sulfites are commonly added.', 'Sulfites are not naturally present in the wines produced by these wine makers in amounts large enough to produce an allergic reaction in someone who drinks these wines.', 'Apart from sulfites, there are no substances commonly present in wine that give rise to an allergic reaction.', 'Not all forms of sulfite are equally likely to produce the allergic reactions.']", "label": 1 }, { "id": "train_3568", "context": "Executives of Company G receive high salaries and excellent benefits, in addition to stock options and numerous other perks. However, a majority of shareholders of Company G have objected to paying for its executives to travel on Company G' s corporate jets. These shareholders believe that these jets should only be used for purposes related to increasing the profits of Company G.", "question": "Which of the following must be true to support the reasoning behind the shareholders' objection?", "answers": "['Company G executives primarily use the corporate jets for personal travel.', 'Company G has not been profitable for the past 3 years.', 'Most companies pay significant amounts of money to lease corporate jets.', 'Other perks provided to the executives of Company G are not directly related to the profitability of Company G.']", "label": 0 }, { "id": "train_3569", "context": "Company president: Whenever you subcontract the manufacturing of a product, you lose some control over the quality of that product. We do subcontract some manufacturing, but only with companies that maintain complete control over the quality of the products they supply.", "question": "Which one of the following can be properly inferred from the company president's statements?", "answers": "[\"The company president insists on having as much control as possible over the quality of the company's products.\", \"When consumers know that a product has been manufactured by a subcontractor, they are generally dubious about the product's quality.\", 'Companies that subcontract the manufacturing of products are often disappointed in the quality of those products.', \"When the president's company subcontracts manufacturing of a product, it does not allow the subcontractor to further subcontract manufacturing of that product.\"]", "label": 3 }, { "id": "train_3570", "context": "Bill is capable of reading two pages per minute, typing one hundred words per minute, and speaking twenty words per minute. All lawyers can read two pages per minute, and some philosophers can read two pages per minute. Only secretaries can type one hundred words per minute. Many chief executive officers can speak twenty words per minute, and few doctors can speak more than twenty words per minute.", "question": "Which one of the following statements can be deduced from the argument?", "answers": "['Bill is a doctor.', 'Bill is a secretary.', 'Bill is a philosopher.', 'Bill is a chief executive officer.']", "label": 1 }, { "id": "train_3571", "context": "Scientist: Venus contains a hot molten core, like that of Earth. Also like Earth, Venus must expel the excess heat the core generates. On Earth, this occurs entirely through active volcanos and fissures created when tectonic plates separate. Yet Venus has neither active volcanos nor fissures caused by the movement of tectonic plates.", "question": "Which one of the following, if true, does the most to resolve the apparent discrepancy described by the scientist?", "answers": "['The atmosphere of Venus is significantly hotter than that of Earth.', 'Rock on the surface of Venus remains solid at much higher temperatures than does rock on Earth.', 'Though Venus lacks active volcanoes and heat- diffusing fissures, it has surface movement somewhat like that of Earth.', 'The surface of Venus is relatively thin, allowing internally produced heat to radiate into space.']", "label": 3 }, { "id": "train_3572", "context": "Bicycle safety expert: Bicycling on the left half of the road is much more likely to lead to collisions with automobiles than is bicycling on the right. After all, in three different studies of bicycle-automobile collisions, the bicyclist was riding on the left in 15, 17, and 25 percent of the cases, respectively. Skeptic: But in places where a comparatively high percentage of bicyclists used to ride on the left, there was surprisingly little decrease in collisions between bicyclists and automobiles after bicycling on the left was made illegal.", "question": "One reason the strength of the bicycle safety expert's argument cannot be evaluated is that", "answers": "['the statistics it cites do not include the percentage of bicycling that took place on the left', 'bicycling on the left is singled out for criticism without consideration of other bicycling practices that are unsafe', 'no statistics are provided on the proportion of bicycle accidents that are due to bicycle-automobile collisions', 'it does not distinguish between places in which bicycling on the left is legal and places in which it is illegal']", "label": 0 }, { "id": "train_3573", "context": "Geneticist: Billions of dollars are spent each year on high-profile experiments that attempt to link particular human genes with particular personality traits. Though such experiments seem to promise a new understanding of human nature, they have few practical consequences. Meanwhile, more mundane and practical genetic projects -- for example, those that look for natural ways to make edible plants hardier or more nutritious -- are grossly underfunded. Thus, funding for human gene research should be reduced while funding for other genetic research should be increased.", "question": "Which one of the following principles, if valid, most helps to justify the geneticist's reasoning?", "answers": "['Experiments that help prevent malnutrition are more worthwhile than those that help prevent merely undesirable personality traits.', 'Experiments that focus on the genetics of plants are more practical than those that focus on the genetics of human nature.', 'Experiments that get little media attention and are not widely supported by the public are more valuable than are those that get much media coverage and have wide public support.', 'Experiments that have modest but practical goals are more worthwhile than those that have impressive goals but few practical consequences.']", "label": 3 }, { "id": "train_3574", "context": "One should never sacrifice one' s health in order to acquire money, for without health, happiness is not obtainable.", "question": "The conclusion of the argument follows logically if which one of the following is assumed?", "answers": "['Money should be acquired only if its acquisition will not make happiness unobtainable.', 'In order to be happy one must have either money or health.', 'Being wealthy is, under certain conditions, conducive to unhappiness.', 'Health is more conducive to happiness than wealth is.']", "label": 0 }, { "id": "train_3575", "context": "Typically, people who have diets high in saturated fat have an increased risk of heart disease. Those who replace saturated fat in their diets with unsaturated fat decrease their risk of heart disease. Ttherefore, people who eat a lot of saturated fat can lower their risk of heart disease by increasing their intake of unsaturated fat.", "question": "Which one of the following, if assumed, most helps to justify the reasoning above?", "answers": "['People who add unsaturated fat to their diets will eat less food that is high in saturated fat.', 'It is difficult to move from a diet that is high in saturated fat to a diet that includes very little fat.', 'Adding unsaturated fat to a diet brings health benefits other than a reduced risk of heart disease.', 'Taking steps to prevent heart disease is one of the most effective ways of increasing life expectancy.']", "label": 0 }, { "id": "train_3576", "context": "Ditrama is a federation made up of three autonomous regions: Korva, Mitro, and Guadar. Under the federal revenue-sharing plan, each region receives a share of federal revenues equal to the share of the total population of Ditrama residing in that region, as shown by a yearly population survey. Last year, the percentage of federal revenues Korva received for its share decreased somewhat even though the population survey on which the revenue-sharing was based showed that Korva' s population had increased.", "question": "If the statements above are true, which one of the following must also have been shown by the population survey on which last year's revenue-sharing in Ditrama was based?", "answers": "['Of the three regions, Korva had the smallest number of residents.', \"Korva's population grew by a smaller percentage than did the population of at least one of the other two autonomous regions.\", 'The populations of Mitro and Guadar each increased by a percentage that exceeded the percentage by which the population of Korva increased.', 'The population of Korva grew by a smaller percentage than it did in previous years.']", "label": 1 }, { "id": "train_3577", "context": "Homeowners who commute more than 60 minutes to work each day spend, on average, 200 minutes less time each month doing outdoor chores than homeowners who commute less than 60 minutes each day. Despite what one would expect, studies show that the yards of the two groups of homeowners do not differ extensively with respect to the number of planted trees, the length of the grass, or the number of weeds.", "question": "Which one of the following, if true, most helps to resolve the apparent discrepancy in the information above?", "answers": "['Homeowners who commute more than 60 minutes to work each day spend 300 fewer minutes on all household chores than homeowners who commute less than 60 minutes to work each day.', 'Homeowners who commute more than 60 minutes to work each day have 40 percent smaller yards than homeowners who commute less than 60 minutes to work each day.', 'Homeowners who commute more than 60 minutes to work each day are less likely to invest in the more advanced gardening tools.', 'Homeowners who commute more than 60 minutes to work each day tend to do more inside chores than homeowners who commute less than 60 minutes to work each day.']", "label": 1 }, { "id": "train_3578", "context": "The incidence in Japan of most types of cancer is remarkably low compared to that in North America, especially considering that Japan has a modern life-style, industrial pollution included. The cancer rates, however, for Japanese people who immigrate to North America and adopt the diet of North Americans approximate the higher cancer rates prevalent in North America.", "question": "If the statements above are true, they provide the most support for which one of the following?", "answers": "['The staple foods of the Japanese diet contain elements that cure cancer.', 'The stress of life in North America is greater than that of life in Japan and predisposes to cancer.', 'The higher cancer rates of Japanese immigrants to North America are caused by fats in the North American diet.', 'The relatively low rate of cancer among people in Japan does not result from a high frequency of a protective genetic trait among Japanese people.']", "label": 3 }, { "id": "train_3579", "context": "Although the geological record contains some hints of major meteor impacts preceding mass extinctions, there were many extinctions that did not follow any known major meteor impacts. Likewise, there are many records of major meteor impacts that do not seem to have been followed by mass extinctions. Thus the geological record suggests that there is no consistent causal link between major meteor impacts and mass extinctions.", "question": "Which one of the following assumptions enables the argument's conclusion to be properly inferred?", "answers": "['If there were a consistent causal link between major meteor impacts and mass extinctions, then all major meteor impacts would be followed by mass extinctions.', 'Of the mass extinctions that did not follow any known major meteor impacts, few if any followed major meteor impacts of which the geological record contains no hints.', 'There could be a consistent causal link between major meteor impacts and mass extinctions even if not every major meteor impact has been followed by a mass extinction.', 'Major meteor impacts and mass extinctions cannot be consistently causally linked unless many mass extinctions have followed major meteor impacts.']", "label": 0 }, { "id": "train_3580", "context": "A construction project has been tasked with building a four-lane road from a remote mountain fortress to a major expressway lying at the base of the mountain. The slope leading to the mountain fortress is so precipitous that the only conceivable way to carry out the project is by building the road downhill, since construction equipment is able to move down, but not up, the steep terrain. The foreman has devised the following solution: dismantle each construction vehicle and tie a part upon the back of a donkey, an animal capable of making the steep ascent. The foreman has at his disposal more than 100 donkeys, each of which is capable of making multiple trips up the mountain.", "question": "In order for the foreman's plan to be successful, which of the following pieces of information is most important?", "answers": "['Whether any donkey is capable of transporting multiple parts', 'Whether the second strongest donkey is able to transport the heaviest piece of equipment', 'Whether the weakest donkey is able to transport the heaviest piece of equipment', 'Whether the strongest donkey is able to transport the heaviest piece of equipment']", "label": 3 }, { "id": "train_3581", "context": "Nursing schools cannot attract a greater number of able applicants than they currently do unless the problems of low wages and high-stress working conditions in the nursing profession are solved. If the pool of able applicants to nursing school does not increase beyond the current level, either the profession will have to lower its entrance standards, or there will soon be an acute shortage of nurses. It is not certain, however, that lowering entrance standards will avert a shortage. It is clear that with either a shortage of nurses or lowered entrance standards for the profession, the current high quality of health care cannot be maintained.", "question": "Which one of the following can be properly inferred from the passage?", "answers": "['The current high quality of health care will not be maintained if the problems of low wages and high-stress working conditions in the nursing profession are not solved.', 'If the nursing profession fails to solve the problems of low wages and high-stress working conditions, there will soon be an acute shortage of nurses.', 'The nursing profession will have to lower its entrance standards if the pool of able applicants to nursing school does not increase beyond the current level.', 'If the nursing profession solves the problems of low wages and high-stress working conditions, it will attract able applicants in greater numbers than it currently does.']", "label": 0 }, { "id": "train_3582", "context": "Newspaper: Increases in produce prices apparently have led to an increase in the planting of personal gardens. The two largest retail seed companies each reported sales increases of around 19 percent in the last year alone, a year in which the price of produce spiked.", "question": "Which one of the following, if true, most seriously weakens the newspaper's argument?", "answers": "['A large retail seed company went out of business early last year.', 'Community gardens report that waiting lists for renting garden plots have gotten longer over the last year.', 'Personal gardens are usually popular in economic downturns.', 'Increases in produce prices are largely driven by increases in the fuel costs of bringing crops to market.']", "label": 0 }, { "id": "train_3583", "context": "Alice: In democracies, politicians garner support by emphasizing the differences between their opponents and themselves. Because they must rule in accord with their rhetoric, policies in democracies fluctuate wildly as one party succeeds another. Elwell: But despite election rhetoric, to put together majority coalitions in democracies, politicians usually end up softening their stands on individual issues once they are elected.", "question": "The statements above provide the most support for the claim that Alice and Elwell disagree about whether", "answers": "['politicians heighten the differences between themselves and their opponents during elections', 'in a democracy the best way of ensuring continuity in policies is to form a coalition government', 'basic policies change drastically when one party succeeds another in a democracy', 'most voters stay loyal to a particular political party even as it changes its stand on particular issues']", "label": 2 }, { "id": "train_3584", "context": "Much of today' s literature is inferior: most of our authors are intellectually and emotionally inexperienced, and their works lack both the intricacy and the focus on the significant that characterize good literature. However, Hypatia' s latest novel is promising; it shows a maturity, complexity, and grace that far exceeds that of her earlier works.", "question": "Which one of the following statements is most strongly supported by the information in the passage?", "answers": [ "Hypatia's latest novel has at least one property of good literature to a greater degree than her earlier works.", "Hypatia's latest novel is clearly better than the majority of today's literature.", "Much of today's literature at least lacks the property of grace.", "Hypatia's latest novel is good literature when judged by today's standards." ], "label": 0 }, { "id": "train_3585", "context": "In the country of Boldavia at present, only 1 percent of 80-year-olds are left-handed, whereas 10 percent of 50-year-olds and 15 percent of 20-year-olds are lefthanded. Yet over the past 80 years, the proportion of Boldavians who are born left-handed has not increased, nor have Boldavian attitudes toward lefthandedness undergone any important changes.", "question": "Which one of the following, if true, most helps to explain the variation in incidence of left-handedness among Boldavians?", "answers": "['In Boldavia, left-handed people are no more likely than right-handed people to be involved in accidents', 'Ambidexterity is highly valued in Boldavia.', 'Left-handed people have never accounted for more than 18 percent of the population of Boldavia.', 'In Boldavia, where men have a shorter average life expectancy than do women, lefthandedness is less common among women than among men.']", "label": 3 }, { "id": "train_3586", "context": "Why should the government, rather than industry or universities, provide the money to put a network of supercomputers in place? Because there is a range of problems that can be attacked only with the massive data-managing capacity of a supercomputer network. No business or university has the resources to purchase by itself enough machines for a whole network, and no business or university wants to invest in a part of a network if no mechanism exists for coordinating establishment of the network as a whole.", "question": "Which one of the following indicates a weakness in the argument?", "answers": "['It overlooks the possibility that businesses or universities, or both, could cooperate to build the network.', 'It does not establish the impossibility of creating a supercomputer network as an international network.', 'It does not furnish a way in which the dilemma concerning the establishment of the network can be resolved.', 'It fails to address the question of who would maintain the network if the government, rather than industry or universities, provides the money for establishing it.']", "label": 0 }, { "id": "train_3587", "context": "In a poll of eligible voters conducted on the eve of a mayoral election, more of those polled stated that they favored Panitch than stated that they favored any other candidate. Despite this result, another candidate, Yeung, defeated Panitch by a comfortable margin.", "question": "Each of the following, if true, contributes to a resolution of the discrepancy described above EXCEPT:", "answers": "['A third candidate, Mulhern, conducted a press conference on the morning of the election and withdrew from the race.', \"The poll, conducted on a Monday, surveyed persons in the downtown area, and the percentage of Yeung's supporters who work downtown is lower than that of Panitch's supporters.\", 'Of the poll respondents supporting Yeung, 70 percent described the election as \"important\" or \"very important, \" while 30 percent of respondents supporting Panitch did the same.', \"Of Yeung's supporters, a smaller percentage were eligible to vote than the percentage of Panitch's supporters who were eligible to vote.\"]", "label": 3 }, { "id": "train_3588", "context": "Editorial: Teenagers tend to wake up around 8: 00 A. M. , the time when they stop releasing melatonin, and are sleepy if made to wake up earlier. Since sleepiness can impair driving ability, car accidents involving teenagers driving to school could be reduced if the school day began later than 8: 00 A. M. Indeed, when the schedule for Granville' s high school was changed so that school began at 8: 30 A. M. rather than earlier, the overall number of car accidents involving teenage drivers in Granville declined.", "question": "Which one of the following, if true, provides the most support for the argument in the editorial?", "answers": "['Teenagers start releasing melatonin later at night and stop releasing it later in the morning than do young children.', 'Car accidents involving teenage drivers rose in the region surrounding Granville during the time they declined in Granville.', 'Many of the car accidents involving teenage drivers in Granville occurred in the evening rather than in the morning.', 'Teenagers who work at jobs during the day spend more time driving than do teenagers who attend high school during the day.']", "label": 1 }, { "id": "train_3589", "context": "It is a principle of economics that a nation can experience economic growth only when consumer confidence is balanced with a small amount of consumer skepticism.", "question": "Which one of the following is an application of the economic principle above?", "answers": "['Any nation in which consumer confidence is balanced with a small amount of consumer skepticism will experience economic growth.', 'Any nation in which the prevailing attitude of consumers is not skepticism will experience economic growth.', 'Any nation in which the prevailing attitude of consumers is exclusively confidence will not experience economic growth.', 'Any nation in which the prevailing attitude of consumers is either exclusively confidence or exclusively skepticism will experience economic growth.']", "label": 2 }, { "id": "train_3590", "context": "Storytelling appears to be a universal aspect of both past and present cultures. Comparative study of traditional narratives from widely separated epochs and diverse cultures reveals common themes such as creation, tribal origin, mystical beings and quasi-historical figures, and common story types such as fables and tales in which animals assume human personalities.", "question": "The evidence cited above from the study of traditional narratives most supports which one of the following statements?", "answers": "[\"Certain human concerns and interests arise in all of the world's cultures.\", 'Storytelling was no less important in ancient cultures than it is in modern cultures.', 'Storytellers have long understood that the narrative is a universal aspect of human culture.', 'The best way to understand a culture is to understand what motivates its storytellers.']", "label": 0 }, { "id": "train_3591", "context": "The Eurasian ruffe, a fish species inadvertently introduced into North America' s Great Lakes in recent years, feeds on the eggs of lake whitefish, a native species, thus threatening the lakes' natural ecosystem. To help track the ruffe' s spread, government agencies have produced wallet-sized cards about the ruffe. The cards contain pictures of the ruffe and explain the danger they pose; the cards also request anglers to report any ruffe they catch.", "question": "Which of the following, if true, would provide most support for the prediction that the agencies' action will have its intended effect?", "answers": "['Ruffe generally feed at night, but most recreational fishing on the Great Lakes is done during daytime hours.', 'The ruffe is one of several nonnative species in the Great Lakes whose existence threatens the survival of lake whitefish populations there.', 'The bait that most people use when fishing for whitefish on the Great Lakes is not attractive to ruffe.', 'Most people who fish recreationally on the Great Lakes are interested in the preservation of the lake whitefish because it is a highly prized game fish.']", "label": 3 }, { "id": "train_3592", "context": "Over the last 10 years, there has been a dramatic increase in the number of people over the age of 65 living in this region. This is evident from the fact that during this time the average age of people living in this region has increased from approximately 52 to 57 years.", "question": "Which one of the following, if true, would most strengthen the argument?", "answers": "['The number of people in the region under the age of 18 has increased over the last 10 years.', 'The birth rate for the region decreased significantly over the last 10 years.', 'The total number of people living in the region has decreased over the last 10 years.', 'The average age for people in the region is higher than that for people in surrounding regions.']", "label": 0 }, { "id": "train_3593", "context": "Australia has considerably fewer species of carnivorous mammals than any other continent does but about as many carnivorous reptile species as other continents do. This is probably a consequence of the unusual sparseness of Australia' s ecosystems. To survive, carnivorous mammals must eat much more than carnivorous reptiles need to; thus carnivorous mammals are at a disadvantage in ecosystems in which there is relatively little food.", "question": "Which one of the following most accurately expresses the main conclusion of the argument?", "answers": "['The reason that carnivorous mammals are at a disadvantage in ecosystems in which there is relatively little food is that they must eat much more in order to survive than carnivorous reptiles need to.', \"The unusual sparseness of Australia's ecosystems is probably the reason Australia has considerably fewer carnivorous mammal species than other continents do but about as many carnivorous reptile species.\", \"Because Australia's ecosystems are unusually sparse, carnivorous mammals there are at a disadvantage relative to carnivorous reptiles.\", 'In ecosystems in which there is relatively little food carnivorous mammals are at a disadvantage relative to carnivorous reptiles.']", "label": 1 }, { "id": "train_3594", "context": "Counselor: Many people assume that personal conflicts are inevitable, but that assumption is just not so. Personal conflicts arise primarily because people are being irrational. For instance, people often find it easier to ascribe bad qualities to a person than good ones -- even when there is more evidence of the latter. If someone suspects that a friend is unreliable, for example, a single instance may turn this suspicion into a feeling of certainty, whereas a belief that someone is reliable is normally built up only after many years of personal interaction.", "question": "Which one of the following most accurately expresses the main conclusion drawn in the argument?", "answers": "['Personal conflicts are not inevitable.', \"It is irrational to allow a single instance to turn one's suspicion that a friend is unreliable into a feeling of certainty.\", 'Even when there is more evidence of good qualities than of bad ones, people find it easier to ascribe bad qualities than good ones.', 'Unlike a suspicion that a friend is unreliable, a belief that someone is reliable is normally built up only after many years of personal interaction.']", "label": 0 }, { "id": "train_3595", "context": "Until recently, endosulfan was one of the most widely used pesticides. Some others -- toxaphene, dieldrin, and chlordane -- were banned or restricted in many countries in the 1980s but linger in the environment. All four have a weak effect individually in increasing estrogen levels in women. Scientists have found, however, that their potential to cause health hazards increases dramatically when the chemicals are combined. For example, a mixture of endosulfan and dieldrin had 160 to 1, 600 times more estrogen-boosting potency than either chemical did individually. Increased estrogen is associated with an increased cancer risk in women.", "question": "Which one of the following is most strongly supported by the information above?", "answers": "['Any two pesticides in combination pose greater health risks than do the same pesticides uncombined.', 'If certain pesticides combine in the environment, they may constitute greatly increased human health hazards.', 'Pesticides that boost estrogen levels are more dangerous than others.', 'Because of its hazards when combined with other chemicals, endosulfan is more dangerous than most other pesticides.']", "label": 1 }, { "id": "train_3596", "context": "Journalist: In physics journals, the number of articles reporting the results of experiments involving particle accelerators was lower last year than it had been in previous years. Several of the particle accelerators at major research institutions were out of service the year before last for repairs, so it is likely that the low number of articles was due to the decline in availability of particle accelerators.", "question": "Which of the following, if true, most seriously undermines the journalist's argument?", "answers": "['The number of physics journals was the same last year as in previous years.', 'Every article based on experiments with particle accelerators that was submitted for publication last year actually was published.', 'Recent changes in the editorial policies of several physics journals have decreased the likelihood that articles concerning particle accelerator research will be accepted for publication.', 'The average time scientists must wait for access to a particle accelerator has declined over the last several years.']", "label": 2 }, { "id": "train_3597", "context": "The young orchestral conductor Sabado Domingo is a true prodigy, able to recognize when any single instrument in his orchestra is even a bit out of tune. In a recent test, an orchestra played a hundred selections from different well-known classical pieces; in approximately half of the selections, exactly one instrument would be played slightly out of tune. In every instance in which an instrument was played out of tune, Domingo pointed out that the orchestra was out of tune, and correctly identified the instrument at fault.", "question": "Which of the following, if true, most strongly supports the conclusion of the argument above?", "answers": "['During the test, the orchestra was arranged in a traditional arrangement, similar to the arrangement in which they would be seated in a classical concert.', 'Because the test was performed in an empty concert hall, the acoustics of the concert hall differed somewhat from those of a concert hall populated by an audience.', 'The instruments played out of tune were all played at a pitch exactly one half-step lower than the true pitch.', \"Domingo did not mistakenly label any of the orchestra's in-tune performances as out of tune.\"]", "label": 3 }, { "id": "train_3598", "context": "Budget constraints have made police officials consider reassigning a considerable number of officers from traffic enforcement to work on higher-priority, serious crimes. Reducing traffic enforcement for this reason would be counterproductive, however, in light of the tendency of criminals to use cars when engaged in the commission of serious crimes. An officer stopping a car for a traffic violation can make a search that turns up evidence of serious crime.", "question": "Which of the following, if true, most strengthens the argument given?", "answers": "['When the public becomes aware that traffic enforcement has lessened, it typically becomes lax in obeying traffic rules.', 'The great majority of persons who are stopped by officers for traffic violations are not guilty of any serious crimes.', 'Those willing to break the law to commit serious crimes are often in committing such crimes unwilling to observe what they regard as the lesser constraints of traffic law.', 'The offenders committing serious crimes who would be caught because of traffic violations are not the same group of individuals as those who would be caught if the arresting officers were reassigned from traffic enforcement.']", "label": 2 }, { "id": "train_3599", "context": "Hotco oil burners, designed to be used in asphalt plants, are so efficient that Hotco will sell one to the Clifton Asphalt plant for no payment other than the cost savings between the total amount the asphalt plant actually paid for oil using its former burner during the last two years and the total amount it will pay for oil using the Hotco burner during the next two years. On installation, the plant will make an estimated payment, which will be adjusted after two years to equal the actual cost savings.", "question": "Which of the following, if it occurred, would constitute a disadvantage for Hotco of the plan described above?", "answers": "[\"The Clifton Asphalt plant's need for more than one new burner\", 'A steady increase in the price of oil beginning soon after the new burner is installed', \"Another manufacturer's introduction to the market of a similarly efficient burner\", 'A decrease in the demand for asphalt']", "label": 1 }, { "id": "train_3600", "context": "Insurance agent: Cars are no safer than motorcycles; that is, when driven at high speeds, cars are not involved in more fatal crashes than are motorcycles. We conducted a survey about car Y and found that highspeed crashes involving car Y resulted in 40% of the fatalities in 2005 while motorcycles were involved in only 35% of the fatalities in 2005.", "question": "Each of the following describes a potential flaw in the insurance agent's reasoning EXCEPT:", "answers": "['A conclusion is drawn about cars in general on the basis of a study focusing on car Y.', 'The survey being used to support the conclusion failed to take into account the significance of the total number of motorcycles that are driven compared to the numbers of car Y that are driven.', 'The safety of a vehicle might depend on other factors besides how often that vehicle is involved in high-speed crashes resulting in fatalities.', 'The insurance agent based his conclusion on the total number of car crashes, not just on those resulting in fatalities.']", "label": 3 }, { "id": "train_3601", "context": "Because quitting smoking is very stressful and leads to weight gain, it is difficult to do. The key to quitting, however, may be as simple as replacing an unhealthy activity with a healthy one. In one study, half of those attempting to quit were assigned to a smoking-cessation program alone, and the other half were assigned to the same program plus fifteen weeks of aerobic exercise. At the one-month mark, none in the first group had quit, but 40 percent of those in the second group had not smoked.", "question": "Each of the following, if true, provides some support for the argument EXCEPT:", "answers": "['Regular exercise prevents weight gain.', 'Each group in the study included four hundred randomly selected participants.', 'Nonsmokers accustomed to regular exercise do not gain weight when they stop exercising.', \"Aerobic exercise can stimulate the brain's production of endorphins, which reduce tension.\"]", "label": 2 }, { "id": "train_3602", "context": "Jack' s aunt gave him her will, asking him to make it public when she died; he promised to do so. After her death, Jack looked at the will; it stipulated that all her money go to her friend George. Jack knew that if he made the will public, George would squander the money, benefiting neither George nor anyone else. Jack also knew that if he did not make the will public, the money would go to his own mother, who would use it to benefit herself and others, harming no one. After reflection, he decided not to make the will public.", "question": "Which one of the following principles, if valid, would require Jack to act as he did in the situation described?", "answers": "['Violating a promise is impermissible whenever doing so would become known by others.', 'Duties to family members take priority over duties to people who are not family members.', 'When faced with alternatives it is obligatory to choose whichever one will benefit the greatest number of people.', 'A promise becomes nonbinding when the person to whom the promise was made is no longer living.']", "label": 2 }, { "id": "train_3603", "context": "Anthony: It has been established that over 80 percent of those who use heroin have a history of having used marijuana. Such evidence would seem to prove that smoking marijuana definitely leads to heroin use. Judith: Maybe smoking marijuana does lead to heroin use, but it is absurd to think that citing those statistics proves that it does. After all, 100 percent of the people who take up heroin had a previous history of drinking water.", "question": "Judith's reply to Anthony's argument relies on which one of the following argumentative strategies?", "answers": "['offering evidence suggesting that the statistics Anthony cites in support of his conclusion are inaccurate', 'providing an example to show that not everything that promotes heroin use is unsafe', 'undermining the credibility of his conclusion by showing that it is a statement from which absurd consequences can be derived', \"demonstrating that Anthony's line of reasoning is flawed by showing that such reasoning can lead to clearly false conclusions\"]", "label": 3 }, { "id": "train_3604", "context": "Ann will either take a leave of absence from Technocomp and return in a year or else she will quit her job there; but she would not do either one unless she were offered a one-year teaching fellowship at a prestigious university. Technocomp will allow her to take a leave of absence if it does not find out that she has been offered the fellowship, but not otherwise. Ttherefore, Ann will quit her job at Technocomp only if Technocomp finds out she has been offered the fellowship.", "question": "Which one of the following, if assumed, allows the conclusion above to be properly drawn?", "answers": "['Ann would be offered the fellowship only if she quit her job at Technocomp.', 'Technocomp will find out about Ann being offered the fellowship only if someone informs on her.', 'Ann will take a leave of absence if Technocomp allows her to take a leave of absence.', 'The reason Ann wants the fellowship is so she can quit her job at Technocomp.']", "label": 2 }, { "id": "train_3605", "context": "Sam: In a recent survey, over 95 percent of people who purchased a Starlight automobile last year said they were highly satisfied with their purchase. Since people who have purchased a new car in the last year are not highly satisfied if that car has a manufacturing defect, Starlight automobiles are remarkably free from such defects. Tiya: But some manufacturing defects in automobiles become apparent only after several years of use.", "question": "Which one of the following most accurately describes how Tiya's response is related to Sam's argument?", "answers": "[\"It argues that Sam's conclusion is correct, though not for the reasons Sam gives.\", 'It offers a consideration that undermines the support Sam offers for his conclusion.', \"It presents new information that implies that Sam's conclusion is false.\", \"It points out that Sam's argument presupposes the truth of the conclusion Sam is defending.\"]", "label": 1 }, { "id": "train_3606", "context": "A prominent investor who holds a large stake in the Burton Tool Company has recently claimed that the company is mismanaged. As evidence for this claim, the investor cited the company' s failure to slow production in response to a recent rise in its inventory of finished products. It is doubtful whether an investor' s sniping at management can ever be anything other than counterproductive, but in this case it is clearly not justified . It is true that an increased inventory of finished products often indicates that production is outstripping demand. In Burton' s case it indicates no such thing , however: the increase in inventory is entirely attributable to products that have already been assigned to orders received from customers.", "question": "In the argument given, the two boldfaced portions play which of the following roles?", "answers": "['The first states the conclusion of the argument as a whole; the second states an intermediate conclusion that is drawn in order to support that conclusion.', 'The first provides evidence to support the conclusion of the argument as a whole; the second states that conclusion.', 'The first is the position that the argument as a whole opposes; the second provides evidence against the position being opposed.', 'The first and the second both state intermediate conclusions that are drawn in order to support jointly the conclusion of the argument as a whole.']", "label": 0 }, { "id": "train_3607", "context": "Parland' s alligator population has been declining in recent years, primarily because of hunting. Alligators prey heavily on a species of freshwater fish that is highly valued as food by Parlanders, who had hoped that the decline in the alligator population would lead to an increase in the numbers of these fish available for human consumption. Yet the population of this fish species has also declined, even though the annual number caught for human consumption has not increased.", "question": "Which of the following, if true, most helps to explain the decline in the population of the fish species?", "answers": "[\"In several neighboring countries through which Parland's rivers also flow, alligators are at risk of extinction as a result of extensive hunting.\", \"Over the last few years, Parland's commercial fishing enterprises have increased the number of fishing boats they use.\", 'The decline in the alligator population has meant that fishers can work in some parts of lakes and rivers that were formerly too dangerous.', \"During Parland's dry season, holes dug by alligators remain filled with water long enough to provide a safe place for the eggs of this fish species to hatch.\"]", "label": 3 }, { "id": "train_3608", "context": "Watanabe: To protect the native kokanee salmon in the lake, we must allow fishing of native trout. Each mature trout eats about 250 mature kokanee annually. Lopez: The real problem is mysis shrimp, which were originally introduced into the lake as food for mature kokanee; but mysis eat plankton -- young kokanees' food. The young kokanee are starving to death. So eradicating the shrimp is preferable to allowing trout fishing.", "question": "Which one of the following principles, if valid, most strongly supports Lopez's conclusion?", "answers": "['When seeking to increase the population of a given species, it is most important that one preserve the members of the species who are in the prime reproductive stage of their lives.', 'When trying to protect the food supply of a particular species, it is best to encourage the method that will have the quickest results, all else being equal.', 'No non-native species should be introduced into a habitat unless all the potential effects of that introduction have been considered.', 'Eliminating a non-native species from a habitat in which it threatens a native species is preferable to any other method of protecting the threatened native species.']", "label": 3 }, { "id": "train_3609", "context": "A study claims that the average temperature on Earth has permanently increased, because the average temperature each year for the last five years has been higher than any previous yearly average on record. However, periods of up to ten years of average temperatures that have consistently been record highs are often merely part of the random fluctuations in temperature that are always occurring.", "question": "Which one of the following is most strongly supported by the information above?", "answers": "['Five successive years of increasing annual average temperature does not always signify a permanent increase in temperature.', 'Record high temperatures can be expected on Earth for another five years.', 'The average temperature on Earth never increases except in cases of random temperature fluctuation.', 'All large increases in average temperature on record have occurred in ten-year periods.']", "label": 0 }, { "id": "train_3610", "context": "Recent studies have demonstrated that smokers are more likely than nonsmokers to develop heart disease. Other studies have established that smokers are more likely than others to drink caffeinated beverages. Ttherefore, even though drinking caffeinated beverages is not thought to be a cause of heart disease, there is a positive correlation between drinking caffeinated beverages and the development of heart disease.", "question": "The argument's reasoning is most vulnerable to criticism on the grounds that the argument fails to take into account the possibility that", "answers": "['smokers who drink caffeinated beverages are less likely to develop heart disease than are smokers who do not drink caffeinated beverages', 'it is only among people who have a hereditary predisposition to heart disease that caffeine consumption is positively correlated with the development of heart disease', 'there is a common cause of both the development of heart disease and behaviors such as drinking caffeinated beverages and smoking', 'drinking caffeinated beverages is more strongly correlated with the development of heart disease than is smoking']", "label": 0 }, { "id": "train_3611", "context": "Censor: All anarchist novels have two objectionable characteristics: a subversive outlook and the depiction of wholesale violence. Ttherefore, it is permissible to ban any anarchist novel that would do more harm than good to society.", "question": "Which one of the following principles, if valid, most helps to justify the censor's reasoning?", "answers": "['If a novel depicts wholesale violence, then it is permissible to ban it if doing so would do more good than harm to society.', 'It is permissible to ban a novel that depicts wholesale violence only if that novel has at least one other objectionable characteristic.', 'If a novel has a subversive outlook but does not depict wholesale violence, it is impermissible to ban it.', 'It is permissible to ban a novel that would cause society more harm than good if the novel has two or more objectionable characteristics.']", "label": 3 }, { "id": "train_3612", "context": "Because of increases in the price of oil and because of government policies promoting energy conservation, the use of oil to heat homes fell by 40 percent from 1970 to the present, and many homeowners switched to natural gas for heating. Because switching to natural gas involved investing in equipment, a significant switch back to oil in the near future is unlikely.", "question": "The prediction that ends the passage would be most seriously called into question if it were true that in the last few years", "answers": "['the price of natural gas to heat homes has remained constant, while the cost of equipment to heat homes with natural gas has fallen sharply', 'the cost of equipment to heat homes with oil has fallen sharply, while the price of heating with oil has fallen below the price of heating with natural gas', 'the cost of equipment to heat homes with natural gas has fallen sharply, while the price of home heating oil has fallen to 1970 levels', 'the use of oil to heat homes has continued to decline, while the price of heating oil has fallen to 1970 levels']", "label": 1 }, { "id": "train_3613", "context": "Maria: Popular music is bad art because it greatly exaggerates the role love plays in everyday life and thereby fails to represent reality accurately. Theo: Popular music is not supposed to reflect reality; it performs other artistic functions, such as providing consoling fantasies and helping people create some romance in their often difficult lives. You should understand popular music before you condemn it.", "question": "The dialogue provides the most support for the claim that Maria and Theo disagree over whether", "answers": "['art need not represent reality accurately to be good art', 'most good art creates consoling illusions', 'art should always represent reality as it could be, not as it is', 'popular music should not be considered to be an art form']", "label": 0 }, { "id": "train_3614", "context": "Newspaper editor: Our newspaper has a responsibility to serve the public interest. When our critics contend that our recent story about the affair of Governor Byce was excessively intrusive, we must reply that the overwhelming public interest in this matter made it our responsibility to publicize facts about the affair.", "question": "Which one of the following is a flaw in the newspaper editor's defense of the newspaper story?", "answers": "['Improperly taking advantage of an ambiguity in the phrase \"public interest\"', 'Confusing his responsibility as a newspaper editor with the responsibility of the governor', 'Forgetting his legal responsibility for reporting such stories', 'Ignoring his moral obligation to listen to critics']", "label": 0 }, { "id": "train_3615", "context": "Council member: The profits of downtown businesses will increase if more consumers live in the downtown area, and a decrease in the cost of living in the downtown area will guarantee that the number of consumers living there will increase. However, the profits of downtown businesses will not increase unless downtown traffic congestion decreases.", "question": "If all the council member's statements are true, which one of the following must be true?", "answers": "['If the cost of living in the downtown area decreases, the profits of downtown businesses will increase.', 'If downtown traffic congestion decreases, the cost of living in the downtown area will decrease.', 'If downtown traffic congestion decreases, the number of consumers living in the downtown area will increase.', 'If downtown traffic congestion decreases, the cost of living in the downtown area will increase.']", "label": 0 }, { "id": "train_3616", "context": "For many types of crops, hybrid strains have been developed that have been found in test plantings to produce significantly higher yields than were produced by traditional nonhybrid strains of those crops planted alongside them. However, in many parts of the world where farmers have abandoned traditional nonhybrid strains in favor of the hybrid strains, crop yields have not increased.", "question": "Which one of the following, if true, most helps to resolve the apparent discrepancy?", "answers": "['Most farmers who plant the hybrid strains of their crops have larger farms than do farmers who continue to plant traditional nonhybrid strains of the same crops.', 'Hybrid strains of crops produced higher yields in some areas than did nonhybrid strains in those areas.', 'Many governments subsidize farmers who plant only hybrid strains of staple crops.', 'The hybrid strains were tested under significantly better farming conditions than are found in most areas where farmers grow those strains.']", "label": 3 }, { "id": "train_3617", "context": "Editorial: Consumers in North America think that by drinking frozen concentrated orange juice, they are saving energy, because it takes fewer truckloads to transport it than it takes to transport an equivalent amount of not-from-concentrate juice. But they are mistaken, for the amount of energy required to concentrate the juice is far greater than the energy difference in the juices' transport.", "question": "Which of the following, if true, would provide the greatest additional support for the editorial's conclusion?", "answers": "['Frozen juice can be stored for several years, while not-from-concentrate juice lasts a much shorter time.', 'Freezer trucks use substantially more energy per mile driven than do any other types of trucks.', 'A serving of not-from-concentrate juice takes up to six times more space than a serving of frozen concentrated juice.', 'Oranges grown in Brazil make up an increasing percentage of the fruit used in not-from-concentrate juice production.']", "label": 1 }, { "id": "train_3618", "context": "Politician: It has been proposed that the national parks in our country be managed by private companies rather than the government. A similar privatization of the telecommunications industry has benefited consumers by allowing competition among a variety of telephone companies to improve service and force down prices. Ttherefore, the privatization of the national parks would probably benefit park visitors as well.", "question": "Which one of the following, if true, most weakens the politician's argument?", "answers": "['It would not be politically expedient to privatize the national parks even if doing so would, in the long run, improve service and reduce the fees charged to visitors.', 'The privatization of the national parks would produce much less competition between different companies than did the privatization of the telecommunications industry.', 'Privatizing the national parks would benefit a much smaller number of consumers to a much smaller extent than did the privatization of the telecommunications industry.', 'The privatization of the telecommunications industry has been problematic in that it has led to significantly increased unemployment and economic instability in that industry.']", "label": 1 }, { "id": "train_3619", "context": "Newspaper subscriber: Arnot' s editorial argues that by making certain fundamental changes in government we would virtually eliminate our most vexing social ills. But clearly this conclusion is false. After all, the argument Arnot makes for this claim depends on the dubious assumption that government can be trusted to act in the interest of the public.", "question": "Which one of the following most accurately expresses a flaw in the argument's reasoning?", "answers": "[\"it treats a change that is required for virtual elimination of society's most vexing social ills as a change that will guarantee the virtual elimination of those ills\", 'it repudiates a claim merely on the grounds that an inadequate argument has been given for it', \"it distorts the opponent's argument and then attacks this distorted argument\", 'it uses the key term \"government\" in one sense in a premise and in another sense in the conclusion']", "label": 1 }, { "id": "train_3620", "context": "Market forecaster: The price of pecans is high when pecans are comparatively scarce but drops sharply when pecans are abundant. Thus, in high-yield years, growers often store part of their crop in refrigerated warehouses until after the next year' s harvest, hoping for higher prices then. Because of bad weather, this year' s pecan crop will be very small. Nevertheless, pecan prices this year will not be significantly higher than last year, since __.", "question": "Which of the following most logically completes the market forecaster's argument?", "answers": "[\"the last time the pecan crop was as small as it was this year, the practice of holding back part of one year's crop had not yet become widely established\", 'pecan yields for some farmers were as high this year as they had been last year', 'pecan prices have remained relatively stable in recent years', \"last year's pecan harvest was the largest in the last 40 years\"]", "label": 3 }, { "id": "train_3621", "context": "Councillor Miller opposes all proposals to raise taxes. Councillor Philopoulos supports increased funding for schools, which in this area are funded entirely by property taxes. It follows that Miller will oppose and Philopoulos will support Councillor Callari' s proposal to increase school funding by raising property taxes.", "question": "Which one of the following exhibits flawed reasoning most similar to the flawed reasoning exhibited by the argument above?", "answers": "['Ed dislikes any food that is extremely sweet, but Bill likes most extremely sweet food. It follows that Ed will dislike these extremely sweet brownies but Bill will probably like them.', 'In the past, the citizens of Lake County have voted down every proposal to increase property taxes. It follows that citizens of Lake County will probably vote down the new proposed increase in property taxes.', 'Jane refuses to live downtown. Denise wants to rent a penthouse apartment. It follows that Jane will not rent one of the penthouse apartments in the Joliet Towers complex downtown but Denise will rent one of those apartments.', \"Tara finds Ms. Burke's English class, which has paper assignments but no exams, easier than Mr. Kent's English class, which has exams but no paper assignments. It follows that Tara finds it easier to write a paper than to take an exam.\"]", "label": 2 }, { "id": "train_3622", "context": "The United States ranks far behind countries such as Sweden and Canada when it comes to workplace safety. In all three countries, joint labor-management committees that oversee workplace safety conditions have been very successful in reducing occupational injuries. In the United States, such committees are found only in the few companies that have voluntarily established them. However, in Sweden and several Canadian provinces, joint safety committees are required by law and exist in all medium-sized and large workplaces.", "question": "Which one of the following is supported by the information above?", "answers": "['The United States would surpass Sweden and Canada in workplace safety if joint safety committees were required in all medium-sized and large workplaces in the United States.', 'A joint safety committee that is required by law is more effective at reducing occupational injuries than is a joint safety committee that is voluntarily established.', 'Workplace safety in Sweden and Canada was superior to that in the United States even prior to the passage of laws requiring joint safety committees in all medium-sized and large workplaces.', 'The establishment of joint safety committees in all medium-sized and large workplaces in the United States would result in a reduction of occupational injuries.']", "label": 3 }, { "id": "train_3623", "context": "Tiger sharks are common in the waters surrounding Tenare Island. Usually tiger sharks feed on smaller sharks, but sometimes they have attacked tourists swimming and surfing at Tenare' s beaches. This has hurt Tenare' s tourism industry, which is second only to its fishing industry in annual revenues. In order to help the economy, ttherefore, the mayor of the island has proposed an ongoing program to kill any tiger sharks within a mile of the beaches.", "question": "Which of the following, if true, most strongly calls into question the likelihood that implementation of the mayor's proposal will have the desired consequence?", "answers": "['Even if not all the tiger sharks that come close to the beaches are killed, the existence of the program would reassure tourists.', 'Not all tourists who come to Tenare Island enjoy swimming or surfing.', 'Business owners who depend on tourism are willing to pay most of the cost of implementing the program.', \"The small sharks on which tiger sharks prey feed on fish that are commercially important to the island's fisheries.\"]", "label": 3 }, { "id": "train_3624", "context": "Early in the twentieth century, Lake Konfa became very polluted. Recently fish populations have recovered as release of industrial pollutants has declined and the lake' s waters have become cleaner. Fears are now being voiced that the planned construction of an oil pipeline across the lake' s bottom might revive pollution and cause the fish population to decline again. However, a technology for preventing leaks is being installed. Ttherefore, provided this technology is effective, the fears are groundless.", "question": "Which of the following, if true, most seriously weakens the argument?", "answers": "[\"Changes in land use and drainage patterns around the lake mean that the lake's waters are increasingly likely to be affected by agricultural runoff.\", \"The pipeline's construction will disturb sediments at the bottom of the lake, dispersing into the water toxic pollutants that have settled there.\", \"A major leak of oil from the pipeline would harm not only the lake's fish but also populations of other aquatic animals and plants.\", 'The leak-preventing technology has been in use for several years without any pipeline in which it is installed developing serious leaks.']", "label": 1 }, { "id": "train_3625", "context": "Often a type of organ or body structure is the only physically feasible means of accomplishing a given task, so it should be unsurprising if, like eyes or wings, that type of organ or body structure evolves at different times in a number of completely unrelated species. After all, whatever the difference of heritage and habitat, as organisms animals have fundamentally similar needs and so __.", "question": "Which one of the following most logically completes the last sentence of the passage?", "answers": "['will resemble other species having different biological needs', 'will develop adaptations allowing them to satisfy these needs', 'will in many instances evolve similar adaptations enabling them to satisfy these needs', 'will all develop eyes or wings as adaptations']", "label": 2 }, { "id": "train_3626", "context": "Utrania was formerly a major petroleum exporter, but in recent decades economic stagnation and restrictive regulations inhibited investment in new oil fields. In consequence, Utranian oil exports dropped steadily as old fields became depleted. Utrania' s currently improving economic situation, together with less restrictive regulations, will undoubtedly result in the rapid development of new fields. However, it would be premature to conclude that the rapid development of new fields will result in higher oil exports, because__.", "question": "Which of the following most logically completes the argument?", "answers": "['the price of oil is expected to remain relatively stable over the next several years', 'the improvement in the economic situation in Utrania is expected to result in a dramatic increase in the proportion of Utranians who own automobiles', 'most of the investment in new oil fields in Utrania is expected to come from foreign sources', 'new technology is available to recover oil from old oil fields formerly regarded as depleted']", "label": 1 }, { "id": "train_3627", "context": "Because of steep increases in the average price per box of cereal over the last 10 years, overall sales of cereal have recently begun to drop. In an attempt to improve sales, one major cereal manufacturer reduced the wholesale prices of its cereals by 20 percent. Since most other cereal manufacturers have announced that they will follow suit, it is likely that the level of overall sales of cereal will rise significantly.", "question": "Which of the following would it be most useful to establish in evaluating the argument?", "answers": "['Whether the variety of cereals available on the market has significantly increased over the last 10 years', 'Whether the high marketing expenses of the highly competitive cereal market led to the increase in cereal prices', 'Whether the prices that supermarkets charge for these cereals will reflect the lower prices the supermarkets will be paying the manufacturers', 'Whether the sales of certain types of cereal have declined disproportionately over the last 10 years']", "label": 2 }, { "id": "train_3628", "context": "Journalist: Although a recent poll found that more than half of all eligible voters support the idea of a political party whose primary concern is education, only 26 percent would like to join it, and only 16 percent would be prepared to donate money to it. Furthermore, there is overwhelming historical evidence that only a party that has at least 30 percent of eligible voters prepared to support it by either joining it or donating money to it is viable in the long run. Ttherefore, it is unlikely that an education party is viable in the long run.", "question": "The reasoning in the journalist's argument is most vulnerable to criticism on the grounds that the argument fails to consider that", "answers": "['the 16 percent of eligible voters prepared to donate money to an education party might donate almost as much money as a party would ordinarily expect to get if 30 percent of eligible voters contributed', 'an education party could possibly be viable with a smaller base than is customarily needed', 'some of the eligible voters who would donate money to an education party might not be prepared to join such a party', 'some of those who said they were willing to donate money to an education party might not actually do so if such a party were formed']", "label": 2 }, { "id": "train_3629", "context": "In Wareland last year, 16 percent of licensed drivers under 21 and 11 percent of drivers ages 21-24 were in serious accidents. By contrast, only 3 percent of licensed drivers 65 and older were involved in serious accidents. These figures clearly show that the greater experience and developed habits of caution possessed by drivers in the 65-and-older group make them far safer behind the wheel than the younger drivers are.", "question": "Which of the following is an assumption on which the argument depends?", "answers": "['There is no age bracket for which the accident rate is lower than it is for licensed drivers 65 and older.', 'The difference between the accident rate of drivers under 21 and of those ages 21-24 is attributable to the greater driving experience of those in the older group.', 'Drivers 65 and older are less likely than are drivers 24 and younger to drive during weather conditions that greatly increase the risk of accidents.', 'Drivers 65 and older do not, on average, drive very many fewer miles per year than drivers 24 and younger.']", "label": 3 }, { "id": "train_3630", "context": "Some scientists have expressed reservations about quantum theory because of its counterintuitive consequences. But despite rigorous attempts to show that quantum theory' s predictions were inaccurate, they were shown to be accurate within the generally accepted statistical margin of error. These results, which have not been equaled by quantum theory' s competitors, warrant acceptance of quantum theory.", "question": "Which one of the following principles most helps to justify the reasoning above?", "answers": "['A scientific theory should be accepted if it has been subjected to serious attempts to disprove it and has withstood all of them.', \"The consequences of a scientific theory should not be considered counterintuitive if the theory's predictions have been found to be accurate.\", 'A scientific theory should be accepted if it has fewer counterintuitive consequences than do its competitors.', 'A theory should be accepted only if its predictions have not been disproved by experiment.']", "label": 0 }, { "id": "train_3631", "context": "Marijuana advocate: If marijuana were legalized in this state, the state could start assessing tax on the drug, increasing state revenues. Since sales would be legal, the criminal culture supporting the drug would vanish; as crimes ceased, the state would save money on fighting crime. Overall, the state has a tremendous amount to gain by making the drug legal. Attorney General: Studies of legalizing previously illegal drugs in other countries suggests that criminals controlling the business will not be eager either to sacrifice their profits or to play by the rules. Moreover, diverting money from crime-fighting after such legalization gives those criminals more free rein.", "question": "The Attorney General uses which of the following techniques in responding to the marijuana advocate?", "answers": "[\"pointing out that the conclusion doesn't follow properly from the premises\", 'calling into question the validity of the evidence cited', 'suggesting, by analogy, potential drawbacks that might outweigh the predicted advantages', 'arguing that the same evidence could be used to prove an opposing conclusion']", "label": 2 }, { "id": "train_3632", "context": "Some psychologists claim that empathic responses are forms of moral behavior. Having observed that young children who witness another' s distress respond by expressing sadness and offering help, these psychologists believe that moral behavior begins early in life. A second group of psychologists claims that empathic response is not, by itself, moral behavior and that in order to count as moral, behavior must be based on a clear understanding of moral principles and a certain degree of moral reasoning skill. On the basis of children' s unsophisticated verbal responses to hypothetical moral dilemmas, these psychologists conclude that children lack the degree of moral reasoning skill necessary for their behavior, however compassionate, to be considered moral.", "question": "Which one of the following is consistent with the claims of the first group of psychologists and inconsistent with the claims of the second group of psychologists?", "answers": "['The morality of an action should be judged on the basis of the intention behind it rather than on the basis of its consequences.', 'People with strong empathic responses are less likely to be skilled in moral reasoning than people with weaker empathic responses.', 'Cats and dogs, which are incapable of moral reasoning but capable of empathic responses, are capable of moral behavior.', 'People with strong empathic responses are more likely to be skilled in moral reasoning than are people with weaker empathic responses.']", "label": 2 }, { "id": "train_3633", "context": "Early in the twentieth century, Lake Konfa became very polluted. Recently fish populations have recovered as release of industrial pollutants has declined and the lake' s waters have become cleaner. Fears are now being voiced that the planned construction of an oil pipeline across the lake' s bottom might revive pollution and cause the fish population to decline again. However, a technology for preventing leaks is being installed. Ttherefore, provided this technology is effective, those fears are groundless.", "question": "The argument depends on assuming which of the following?", "answers": "['There is no reason to believe that the leak-preventing technology would be ineffective when installed in the pipeline in Lake Konfa.', \"Damage to the lake's fish populations would be the only harm that a leak of oil from the pipeline would cause.\", \"Other than the possibility of a leak, there is no realistic pollution threat posed to the lake by the pipeline's construction.\", 'Apart from development related to the pipeline, there will be no new industrial development around the lake that will create renewed pollution in its waters.']", "label": 2 }, { "id": "train_3634", "context": "Consumer advocate: Manufacturers of children' s toys often place warnings on their products that overstate the dangers their products pose. Product-warning labels should overstate dangers only if doing so reduces injuries. In fact, however, manufacturers overstate their products' dangers merely for the purpose of protecting themselves from lawsuits brought by parents of injured children. Ttherefore, manufacturers of children' s toys should not overstate the dangers their products pose.", "question": "Which one of the following most accurately describes a reasoning flaw in the consumer advocate's argument?", "answers": "['The argument confuses a necessary condition for reducing the number of injuries caused by a product with a sufficient condition.', 'The argument relies on the unjustified assumption that an action has an effect only if it was performed in order to bring about that effect.', 'The argument presumes, without providing justification, that if a warning overstates a danger, then the warning will fail to prevent injuries.', 'The argument overlooks the possibility that warnings that do not overstate the dangers that their products pose do not always reduce injuries.']", "label": 1 }, { "id": "train_3635", "context": "Of the many works in a collection from Japan' s Tokugawa period that the museum will soon put on display, those that are most sensitive to light, as well as the most valuable pieces, will be on display for two weeks only. Sakai Hoitsu' s \"Spring and Autumn Maples\" will be on display for two weeks only, so it is clearly among the most valuable pieces in the collection.", "question": "The flawed pattern of reasoning in the argument above most closely parallels that in which one of the following?", "answers": "['The city council will soon commission surveyors to update the city map. The new map will reflect existing structures as well as planned housing developments. But the housing development I live in was built several decades ago, so it will not be on the updated map.', 'The legend on this city map indicates that historical monuments are designated by purple dots. Hospitals are also designated by purple dots. There is a purple dot on Wilson Street. So there must be a hospital on Wilson Street.', 'On this city map, a solid line designates the city limits. Solid lines also designate major thoroughfares. So there is no way of determining whether a particular solid line on the map is a thoroughfare or an indicator of the city limits.', 'I have just purchased the new city map, whose legend indicates that thoroughfares are marked by solid lines and that dotted lines designate one-way streets. Shearing Street is marked by a dotted line, even though it is a thoroughfare. So the mapmakers must have made a mistake.']", "label": 1 }, { "id": "train_3636", "context": "Editorial: In Ledland, unemployed adults receive government assistance. To reduce unemployment, the government proposes to supplement the income of those who accept jobs that pay less than government assistance, thus enabling employers to hire workers cheaply. However, the supplement will not raise any worker' s income above what government assistance would provide if he or she were not gainfully employed. Ttherefore, unemployed people will have no financial incentive to accept jobs that would entitle them to the supplement.", "question": "Which of the following, if true about Ledland, most seriously weakens the argument of the editorial?", "answers": "['People who are employed and look for a new job tend to get higher-paying jobs than job seekers who are unemployed.', 'The yearly amount unemployed people receive from government assistance is less than the yearly income that the government defines as the poverty level.', 'Neighboring countries with laws that mandate the minimum wage an employer must pay an employee have higher unemployment rates than Ledland currently has.', 'People sometimes accept jobs that pay relatively little simply because they enjoy the work.']", "label": 0 }, { "id": "train_3637", "context": "Sarah: Reporters, by allotting time to some events rather than others, are exercising their judgment as to what is newsworthy and what is not. In other words, they always interpret the news. Ramon: Reporters should never interpret the news. Once they deem a story to be newsworthy, they are obliged to relay the facts to me untainted.", "question": "Sarah and Ramon's remarks provide the most support for holding that they disagree about the truth of which one of the following statements?", "answers": "['Reporters actually do interpret the news every time they report it.', 'Reporters should not allot time to reporting some events rather than others.', 'Reporters should exercise their own judgment as to which events are newsworthy.', 'Reporting on certain events rather than others qualifies as interpreting the news.']", "label": 3 }, { "id": "train_3638", "context": "A number of seriously interested amateur astronomers have tested the new Exodus refractor telescope. With it, they were able to observe in crisp detail planetary features that were seen only as fuzzy images in their 8-inch (approximately 20-centimeter) Newtonian telescopes, even though the 8-inch telescopes, with their wider apertures, gather more light than the 4-inch (approximately 10-centimeter) Exodus. Given these amateur astronomers' observational findings, any serious amateur astronomer ought to choose the Exodus if she or he is buying a telescope for planetary observation.", "question": "The argument proceeds by", "answers": "['supporting a recommendation to a group on the basis of the experience of a subset of that group', 'distinguishing between the actual reasons why a certain group did a particular thing and the best reasons for doing that thing', 'detailing the ways in which a testing situation approximates the conditions of ordinary use', 'placing a phenomenon in a wider context in order to explain it']", "label": 0 }, { "id": "train_3639", "context": "In the United States, of the people who moved from one state to another when they retired, the percentage who retired to Florida has decreased by three percentage points over the past ten years. Since many local businesses in Florida cater to retirees, this decline is likely to have a noticeably negative economic effect on these businesses.", "question": "Which of the following, if true, most seriously weakens the argument?", "answers": "['The number of people who move out of Florida to accept employment in other states has increased over the past ten years.', 'The number of people who left Florida when they retired to live in another state was greater last year than it was ten years ago.', 'The total number of people who retired and moved to another state for their retirement has increased significantly over the past ten years.', 'Florida attracts more people who move from one state to another when they retire than does any other state.']", "label": 2 }, { "id": "train_3640", "context": "Yuriko: Our city' s campaign to persuade parents to have their children vaccinated ought to be imitated by your city. In the 16 months since the enactment of legislation authorizing the campaign, vaccinations in our city have increased by 30 percent. Susan: But the major part of that increase occurred in the first 6 months after that legislation was enacted, right after your city' s free neighborhood health clinics opened, and before the vaccination campaign really got going.", "question": "In responding to Yuriko, Susan does which one of the following?", "answers": "['She cites facts that tend to weaken the force of the evidence with which Yuriko supports her recommendation.', \"She denies Yuriko's assumption that Susan's city wants to increase the vaccination rate for children.\", 'She advances the claim that a campaign such as Yuriko recommends is not necessary because most parents already choose to have their children vaccinated.', 'She presents evidence to suggest that vaccination campaigns are usually ineffective.']", "label": 0 }, { "id": "train_3641", "context": "After being subjected to clinical tests like those used to evaluate the effectiveness of prescription drugs, a popular nonprescription herbal remedy was found to be as effective in treating painful joints as is a certain prescription drug that has been used successfully to treat this condition. The manufacturer of the herbal remedy cited the test results as proof that chemical agents are unnecessary for the successful treatment of painful joints.", "question": "The test results would provide the proof that the manufacturer claims they do if which one of the following is assumed?", "answers": "['The prescription drug treats the discomfort associated with painful joints without eliminating the cause of that condition.', 'People are likely to switch from using prescription drugs to using herbal remedies if the herbal remedies are found to be as effective as the prescription drugs.', 'The researchers who analyzed the results of the clinical testing of the herbal remedy had also analyzed the results of the clinical testing of the prescription drug.', 'The herbal remedy contains no chemical agents that are effective in treating painful joints.']", "label": 3 }, { "id": "train_3642", "context": "The government provides insurance for individuals' bank deposits, but requires the banks to pay the premiums for this insurance. Since it is depositors who primarily benefit from the security this insurance provides, the government should take steps to ensure that depositors who want this security bear the cost of it and thus should make depositors pay the premiums for insuring their own accounts.", "question": "Which one of the following principles, if established, would do most to justify drawing the conclusion of the argument on the basis of the reasons offered in its support?", "answers": "['Any rational system of insurance must base the size of premiums on the degree of risk involved.', 'The people who stand to benefit from an economic service should always be made to bear the costs of that service.', 'The choice of not accepting an offered service should always be available, even if there is no charge for the service.', 'Government-backed security for investors, such as bank depositors, should be provided only when it does not reduce incentives for investors to make responsible investments.']", "label": 1 }, { "id": "train_3643", "context": "Physiological research has uncovered disturbing evidence linking a number of structural disorders to jogging. Among the ailments seemingly connected with this now-popular sport are spinal disk displacements, stress fractures of the feet and ankles, knee and hip joint deterioration, and tendonitis. Furthermore, these injuries do not occur exclusively among beginning runners -- veteran joggers suffer an equal percentage of injuries. What the accumulating data suggest is that the human anatomy is not able to withstand the stresses of jogging.", "question": "Which one of the following is an assumption of the argument?", "answers": "['Some sports are safer for the human body than jogging.', 'Jogging causes more serious disorders than other sports.', 'The link between jogging and certain structural disorders appears to be a causal one.', \"The jogger's level of experience is a factor determining the likelihood of a jogging injury.\"]", "label": 2 }, { "id": "train_3644", "context": "For several years, per capita expenditure on prescription drugs in Voronia rose by fifteen percent or more annually. In order to curb these dramatic increases, the ministry of health prohibited drug manufacturers from raising any of their products' prices. Even though use of prescription drugs did not expand after this price freeze, per capita expenditure for prescription drugs continued to increase by a substantial percentage each year.", "question": "Which of the following, if true, most helps to explain why the ministry's action did not achieve its goal?", "answers": "['In addition to imposing a price freeze, the government encouraged doctors to prescribe generic versions of common drugs instead of the more expensive brand-name versions.', 'After price increases were prohibited, drug manufacturers concentrated on producing new medications to replace existing products.', 'After price increases were prohibited, some foreign manufacturers of expensive drugs ceased marketing them in Voronia.', 'Improvements in manufacturing processes enabled drug manufacturers to maintain high profit levels on drugs despite the price freeze.']", "label": 1 }, { "id": "train_3645", "context": "A small car offers less protection in an accident than a large car does, but since a smaller car is more maneuverable, it is better to drive a small car because then accidents will be less likely.", "question": "Which one of the following arguments employs reasoning most similar to that employed by the argument above?", "answers": "[\"Although it is important to limit the amount of sugar and fat in one's diet, it would be a mistake to try to follow a diet totally lacking in sugar and fat. It is better to consume sugar and fat in moderation, for then the cravings that lead to uncontrolled binges will be prevented.\", 'For this work, vehicles built of lightweight materials are more practical than vehicles built of heavy materials. This is so because while lighter vehicles do not last as long as heavier vehicles, they are cheaper to replace.', 'A person who exercises vigorously every day has less body fat than an average person to draw upon in the event of a wasting illness. But one should still endeavor to exercise vigorously every day, because doing so significantly decreases the chances of contracting a wasting illness.', \"An artist's best work is generally that done in the time before the artist becomes very well known. When artists grow famous and are diverted from artistic creation by demands for public appearances, their artistic work suffers. So artists' achieving great fame can diminish their artistic reputations.\"]", "label": 2 }, { "id": "train_3646", "context": "Many new tech companies follow the Orange Company' s model. Orange Company often utilizes lavish release parties to market their products. New tech companies are now doing the same. However, almost all new tech companies cannot afford to throw parties as lavish as Orange' s parties. Ttherefore, these companies are foolish to throw parties they can' t afford to market their products.", "question": "The reasoning in the argument is most vulnerable to criticism on the grounds that the argument:", "answers": "[\"Fails to consider that new tech companies could benefit from applying Orange Company's marketing strategy on a smaller and more affordable scale.\", \"Fails to consider that new tech companies would throw lavish release parties without Orange Company's example.\", \"Fails to consider that even if new tech companies threw lavish release parties, they would still not match Orange Company's profitability.\", 'Does not adequately address the possibility that lavish release parties are still cheaper than national marketing campaigns.']", "label": 0 }, { "id": "train_3647", "context": "Each new car in the lot at Rollway Motors costs more than $18, 000. Any car in their lot that is ten or more years old costs less than $5, 000. Thus, if a car in Rollway' s lot costs between $5, 000 and $18, 000, it is a used car that is less than ten years old.", "question": "The pattern of reasoning in which one of the following arguments is most similar to that in the argument above?", "answers": "['Each apartment above the fourth floor of the building has more than two bedrooms. But all apartments below the fourth floor have fewer than two bedrooms. Thus, any apartment on the fourth floor of the building has exactly two bedrooms.', 'No apartment above the fourth floor of the building has more than two bedrooms. But only three-bedroom apartments have balconies. Thus, if any apartment in the building has a balcony, it is on the fourth floor or lower.', 'Each apartment above the fourth floor of the building has more than two bedrooms. The building has no vacant apartments on or below the fourth floor. Thus, if there is any vacant apartment in the building, it will have more than two bedrooms.', 'No apartment above the fourth floor of the building has fewer than three bedrooms. But all apartments below the fourth floor have fewer than two bedrooms. Thus, if there are apartments in the building with exactly two bedrooms, they are on the fourth floor.']", "label": 3 }, { "id": "train_3648", "context": "Even the earliest known species of land animals, known from fossils dating from the late Silurian period, 400 million years ago, show highly evolved adaptations to life on land. Since neither aquatic nor amphibious animals exhibit these adaptations, early species of land animals must have evolved very rapidly after leaving an aquatic environment.", "question": "Which one of the following is an assumption on which the argument depends?", "answers": "['Known fossils of early land animals include fossils of animals that lived relatively soon after the first emergence of land animals.', 'No plants were established on land before the late Silurian period.', 'No present-day species of aquatic animal is descended from a species of animal that once lived on land.', 'Fossils from the late Silurian period represent only a small number of the animal species that were alive at that time.']", "label": 0 }, { "id": "train_3649", "context": "The cattle egret is a bird that lives around herds of cattle. The only available explanation of the fact that the cattle egret follows cattle herds is that the egrets consume the insects stirred up from the grasses as the cattle herds graze.", "question": "Which one of the following, if true, would most seriously undermine the claim that the explanation given above is the only available one?", "answers": "['Cattle egrets are not generally known to live outside the range of large, slow-moving animals.', 'The presence of cattle dissuades many would- be predators of the cattle egret.', 'Forests are generally inhospitable to cattle egrets because of a lack of insects of the kind egrets can consume.', 'Cattle egrets are known to follow other slow- moving animals, such as rhinoceroses and buffalo.']", "label": 1 }, { "id": "train_3650", "context": "Fearing competition from stores that rent video games for home use, owners of stores that sell video games lobbied for protective legislation. Citing as a precedent legislation that postpones home film rentals until one year after general release to theaters, the video sellers proposed as an equitable solution a plan that would postpone rental of any video game until it had been available for sale for one year.", "question": "Which of the following, if true, would support an objection by owners of video rental stores that the fairness of the proposed legislation is not supported by the precedent cited?", "answers": "['Most people are interested in playing only the latest video games and tend to find video games that have been available for over a year unappealing whether they have played them before or not, whereas films maintain their appeal far longer after their release.', 'Although the daily rental fee for home use of films is generally the same as that for video games, the average rental period for a video game is longer than that for a film.', 'People who rent video games tend to play them by themselves, whereas people who rent films invite friends and neighbors to watch.', 'A slight decline in revenues from films that have been recently released in theaters has been attributed to the growing market for rentals of films for home use.']", "label": 0 }, { "id": "train_3651", "context": "In grasslands near the Namib Desert there are \"fairy circles\" -- large, circular patches that are entirely devoid of vegetation. Since sand termite colonies were found in every fairy circle they investigated, scientists hypothesize that it is the burrowing activities of these termites that cause the circles to form.", "question": "Which one of the following, if true, most supports the scientists' hypothesis?", "answers": "['The grasses that grow around fairy circles are able to survive even the harshest and most prolonged droughts in the region.', 'Dying grass plants within newly forming fairy circles are damaged only at the roots.', 'The soil in fairy circles typically has higher water content than the soil in areas immediately outside the circles.', 'Species of animals that feed on sand termites are often found living near fairy circles.']", "label": 1 }, { "id": "train_3652", "context": "Pandas are rapidly disappearing from the wild. Ttherefore, in order to preserve the species, existing pandas should be captured and placed in zoos around the world.", "question": "Which of the following statements, if true, casts most doubt on the conclusion drawn above?", "answers": "['Newborn pandas in zoos are not likely to die from infectious diseases, whereas newborn pandas in the wild are likely to die from these diseases.', 'Pandas in zoos have as many offspring that survive to adulthood as do pandas in the wild.', \"Sufficient quantities of bamboo, the panda's only food, cannot be made available outside the panda's native habitat.\", 'Many zoos are eager to increase their stock of rare and exotic animals, but there are not enough pandas for all the zoos that want one.']", "label": 2 }, { "id": "train_3653", "context": "In the city of Glasgow, Scotland, trade doubled between 1 750, when the first bank opened there, and 1 765, when government regulations on banking were first implemented in Scotland.", "question": "Each of the following, if true, could contribute to an explanation of the doubling described above EXCEPT:", "answers": "['The technological revolution that started in the early eighteenth century in England resulted in increased trade between England and Scotland.', 'Reductions in tariffs on foreign goods in 1752 led to an increase in imports to Glasgow.', \"The initial government regulation of Scottish banks stimulated Glasgow's economy.\", 'Improvements in Scottish roads between 1750 and 1758 facilitated trade between Glasgow and the rest of Scotland.']", "label": 2 }, { "id": "train_3654", "context": "Advertisement: Cigarettes are deadly. Hundreds of thousands of people die every year from smoking-related causes, such as lung cancer or heart disease. The science is clear -- smoking a pack per day for years will shorten one' s life. Sitting in a room where someone is smoking might as well be a gas chamber in terms of damage to long-term health.", "question": "Which one of the following best describes the flaw in the author's reasoning?", "answers": "['The advertisement uses overly broad generalization.', 'The advertisement relies on shoddy science.', 'The advertisement draws an unjustified analogy.', 'The advertisement confuses cause and effect.']", "label": 2 }, { "id": "train_3655", "context": "When 100 people who have not used cocaine are tested for cocaine use, on average only 5 will test positive. By contrast, of every 100 people who have used cocaine 99 will test positive. Thus, when a randomly chosen group of people is tested for cocaine use, the vast majority of those who test positive will be people who have used cocaine.", "question": "A reasoning error in the argument is that the argument", "answers": "['ignores the fact that some cocaine users do not test positive', 'attempts to infer a value judgment from purely factual premises', 'advocates testing people for cocaine use when there is no reason to suspect that they have used cocaine', 'fails to take into account what proportion of the population have used cocaine']", "label": 3 }, { "id": "train_3656", "context": "Because ethylene dibromide, a chemical used to fumigate grain, was blamed for the high rate of nerve damage suffered by people who work in grain-processing plants, many such plants switched to other chemical fumigants two years ago. Since then, however, the percentage of workers at these plants who were newly diagnosed with nerve damage has not dropped significantly. Ttherefore, either ethylene dibromide was wrongly blamed or else the new chemicals also cause nerve damage.", "question": "Which of the following is an assumption on which the argument depends?", "answers": "['Workers at grain-processing plants that still use ethylene dibromide continue to have a high rate of nerve damage.', 'There are no chemical fumigants that are completely safe for workers in grain-processing plants.', 'If the new chemicals cause nerve damage, the nerve damage caused would be different from any nerve damage that ethylene dibromide may cause.', 'If ethylene dibromide causes nerve damage, it does not take two years or longer for that damage to become detectable.']", "label": 3 }, { "id": "train_3657", "context": "Politician: Members of the national legislature have received a very large number of phone calls and letters from people wanting to express their opinions about the new bill before the legislature, which would increase the current tax on bidis and cigarettes by one rupee per pack to pay for new antismoking advertisements. Since a great majority of these people expressed opposition to the bill, it is clear that the public overwhelmingly opposes this tax increase.", "question": "Which one of the following, if true, most seriously weakens the argument?", "answers": "['People who oppose a tax bill are far more likely to contact a legislator to express their opinions than are those who do not oppose the bill.', 'People who do not smoke bidis or cigarettes but work in tobacco-related industries are just as likely as smokers to oppose a bidi and cigarette tax.', 'Most antismoking advertisements are currently paid for by private organizations rather than being funded by taxes paid to the government.', 'Increasing the tax on bidis and cigarettes by one rupee per pack would reduce bidi and cigarette consumption so that total revenue from such taxes would not increase.']", "label": 0 }, { "id": "train_3658", "context": "Many conceptual categories are parts of dichotomous (distinct and mutually exclusive) pairs: good or bad, right or wrong, rational or irrational, etc. However, advances in scientific understanding have shown some long-held dichotomies to be untenable. Some life forms have characteristics of both animals and plants; also, matter can be converted into energy and vice versa. Ttherefore, dichotomous classifications into mutually exclusive categories should generally be abandoned.", "question": "Which one of the following exhibits flawed reasoning most similar to that in the argument above?", "answers": "['This budget is based on the assumption that revenue will increase for the next two years. However, revenue figures for past years show that assumption to be untenable. Ttherefore, this budget should be replaced by a more realistic one.', 'Recent clinical trials have shown that some antianxiety drugs are addictive and can have life-threatening side effects. Ttherefore, the use of drugs for the treatment of anxiety should be discontinued.', \"Review by outside consultants has shown that this company should replace all of its computers with more powerful models. Ttherefore, not all of this company's computers are powerful enough.\", 'The longer fruit is kept, the more likely it is to become rotten. While these peaches seem to be fine, they have been kept for a rather long time. So it is best to throw them away now before they begin to rot.']", "label": 1 }, { "id": "train_3659", "context": "When three Everett-owned Lightning-built airplanes crashed in the same month, the Everett company ordered three new Lightning-built airplanes as replacements. This decision surprised many in the airline industry because, ordinarily when a product is involved in accidents, users become reluctant to buy that product.", "question": "Which of the following, if true, provides the best indication that the Everett company's decision was logically well supported?", "answers": "['Consumer advocates pressured two major airlines into purchasing safer airplanes so that the public would be safer while flying.', 'The Federal Aviation Association issued new guidelines for airlines in order to standardize safety requirements governing preflight inspections.', 'The Lightning-built airplanes crashed due to pilot error, but because of the excellent quality of the planes there were many survivors.', 'Although during the previous year only one Lightning-built airplane crashed, competing manufacturers had a perfect safety record.']", "label": 2 }, { "id": "train_3660", "context": "Sheila: It has been argued that using computer technology to add color to a movie originally filmed in black and white damages the integrity of the original film. But no one argues that we should not base a movie on a novel or a short story because doing so would erode the value of the book or story. The film adaptation of the written work is a new work that stands on its own. Judgments of it do not reflect on the original. Similarly, the colorized film is a new work distinct from the original and should be judged on its own merit. It does not damage the integrity of the original black-and-white film.", "question": "Sheila's argument uses which one of the following techniques of argumentation?", "answers": "['It offers a counterexample to a general principle.', 'It draws an inference from a general principle and a set of facts.', 'It appeals to an analogy between similar cases.', 'It distinguishes facts from value judgments.']", "label": 2 }, { "id": "train_3661", "context": "Normally, the pineal gland governs a person' s sleep-wake cycle by secreting melatonin in response to the daily cycle of light and darkness as detected by the eye. Nonetheless, many people who are totally blind due to lesions in the visual cortex of the brain easily maintain a 24-hour sleep-wake cycle. So the neural pathway by which the pineal gland receives information from the eye probably does not pass through the visual cortex.", "question": "For purposes of evaluating the argument it would be useful to establish which of the following?", "answers": "['Whether there are any people with normal vision whose melatonin levels respond abnormally to periods of light and darkness', 'Whether the melatonin levels of most totally blind people who successfully maintain a 24-hour sleep-wake cycle change in response to changes in exposure to light and darkness', 'Whether melatonin is the only substance secreted by the pineal gland', 'Whether melatonin supplements help people who have difficulty maintaining a 24-hour sleep cycle to establish such a pattern']", "label": 1 }, { "id": "train_3662", "context": "Rifka: We do not need to stop and ask for directions. We would not need to do that unless, of course, we were lost. Craig: The fact that we are lost is precisely why we need to stop.", "question": "In the exchange above, the function of Craig's comment is to", "answers": [ "provide a counterexample to Rifka's generalization", "affirm the truth of the stated premise of Rifka's argument while remaining noncommittal about its conclusion", "contradict the conclusion of Rifka's argument without offering any reason to reject any of Rifka's implicit premises", "deny one of Rifka's implicit premises and thereby arrive at a different conclusion" ], "label": 3 }, { "id": "train_3663", "context": "Most students are bored by history courses as they are usually taught, primarily because a large amount of time is spent teaching dates and statistics. The best way to teach history, ttherefore, is to spend most class time recounting the lives of historical figures and very little on dates and statistics.", "question": "Each of the following is an assumption on which the argument depends EXCEPT:", "answers": "['It is possible to recount the lives of historical figures without referring to dates and statistics.', 'Students are more bored by history courses as they are usually taught than they would be by courses that spend most class time recounting the lives of historical figures.', \"One should avoid boring one's students when teaching a history course.\", 'It is compatible with the attainable goals of teaching history to spend most class time recounting the lives of historical figures.']", "label": 0 }, { "id": "train_3664", "context": "Critics have argued that because Freudianism holds that people have unconscious desires that can defeat their attempts to follow rational life plans, it is incompatible with the predominantly rationalistic spirit of Western philosophical and psychological thought. But it is a central tenet of Freudianism that through psychoanalysis one can become conscious of one' s previously unconscious desires, enabling one to avoid being defeated by them. Ttherefore, __.", "question": "Which one of the following most logically completes the argument?", "answers": "['Freudianism does not run counter to the rationalistic mainstream of Western philosophical and psychological thought', 'psychoanalysis provides one with a rational life plan', 'Freudianism holds that people can always achieve happiness through psychoanalysis', 'Freudianism reflects the predominantly rationalistic spirit of Western philosophical and psychological thought more than any other psychological theory']", "label": 0 }, { "id": "train_3665", "context": "Sigerson argues that the city should adopt ethical guidelines that preclude its politicians from accepting campaign contributions from companies that do business with the city. Sigerson' s proposal is dishonest, however, because he has taken contributions from such companies throughout his career in city politics.", "question": "The reasoning in the argument is most vulnerable to criticism on the grounds that the argument", "answers": "['rejects a proposal on the grounds that an inadequate argument has been given for it', 'confuses a sufficient condition for adopting ethical guidelines for politicians with a necessary condition for adopting such guidelines', 'rejects a proposal on the grounds that the person offering it is unfamiliar with the issues it raises', \"overlooks the fact that Sigerson's proposal would apply only to the future conduct of city politicians\"]", "label": 3 }, { "id": "train_3666", "context": "Teresa: If their goal is to maximize profits, film studios should concentrate on producing bigbudget films rather than small-budget ones. For, unlike big-budget films, small-budget films never attract mass audiences. While small-budget films are less expensive to produce and, hence, involve less risk of unprofitability than big-budget films, low production costs do not guarantee the highest possible profits.", "question": "Which one of the following is an assumption required by Teresa's argument?", "answers": "['A film studio will not maximize its profits unless at least some of its films attract mass audiences.', \"A film studio's primary goal should be to maximize profits.\", 'It is impossible to produce a big-budget film in a financially efficient manner.', 'Each big-budget film is guaranteed to attract a mass audience.']", "label": 0 }, { "id": "train_3667", "context": "Town councillor: The only reason for the town to have ordinances restricting where skateboarding can be done would be to protect children from danger. Skateboarding in the town' s River Park is undoubtedly dangerous, but we should not pass an ordinance prohibiting it. If children cannot skateboard in the park, they will most certainly skateboard in the streets. And skateboarding in the streets is more dangerous than skateboarding in the park.", "question": "The pattern of reasoning in which one of the following is most similar to that in the town councillor's argument?", "answers": "[\"The purpose of compulsory vaccination for schoolchildren was to protect both the children themselves and others in the community against smallpox. Smallpox was indeed a dreadful disease, but it has now been eliminated from the world's population. So children should not be vaccinated against it.\", 'Traffic patterns that involve one-way streets are meant to accelerate the flow of traffic in otherwise congested areas. However, it would be detrimental to the South Main Street area to have traffic move faster. So traffic patterns involving one-way streets should not be implemented there.', 'The reason for requiring environmental reviews is to ensure that projected developments do not harm the natural environment. Currently, environmental concerns are less compelling than economic concerns, but in the long run, the environment must be protected. Ttherefore, the requirement for environmental reviews should not be waived.', 'Insecticides are designed to protect crops against insect damage. Aphids damage tomato crops, but using insecticides against aphids kills wasps that prey on insecticide-resistant pests. Since aphids damage tomato crops less than the insecticide-resistant pests do, insecticides should not be used against aphids on tomato crops.']", "label": 3 }, { "id": "train_3668", "context": "TV meteorologist: Our station' s weather forecasts are more useful and reliable than those of the most popular news station in the area. After all, the most important question for viewers in this area is whether it will rain, and on most of the occasions when we have forecast rain for the next day, we have been right. The same cannot be said for either of our competitors.", "question": "Which one of the following, if true, most strengthens the meteorologist's argument?", "answers": "[\"On most of the occasions when the meteorologist's station forecast that it would not rain, at least one of its competitors also forecast that it would not rain.\", \"The meteorologist's station forecast rain more often than did the most popular news station in the area.\", \"The meteorologist's station has a policy of not making weather forecasts more than three days in advance.\", 'The most popular news station in the area is popular because of its investigative news reports.']", "label": 1 }, { "id": "train_3669", "context": "The assassination of Archduke Franz Ferdinand of Austria is often ascribed as the cause of World War I. However, the assassination merely lit the fuse in a combustible situation since many of the world powers were in complicated and convoluted military alliances. For example, England, France, and Russia entered into a mutual defense treaty seven years prior to World War I. Even without Franz Ferdinand' s assassination __.", "question": "Which of the following most logically completes the passage?", "answers": "['Austria would have started the war.', 'World War I never would have happened.', 'A war between the world powers was extremely likely.', 'England, France, and Russia would have started the war.']", "label": 2 }, { "id": "train_3670", "context": "Many Seychelles warblers of breeding age forgo breeding, remaining instead with their parents and helping to raise their own siblings. This behavior, called cooperative breeding, results from the scarcity of nesting territory for the birds on the tiny island that, until recently, was home to the world' s population of Seychelles warblers. Yet when healthy warblers were transplanted to a much larger neighboring island as part of an experiment, most of those warblers maintained a pattern of cooperative breeding.", "question": "Which one of the following, if true, most helps to explain the result of the experiment?", "answers": "['Most of the terrain on the neighboring island was not of the type in which Seychelles warblers generally build their nests.', 'Cooperative breeding in species other than the Seychelles warbler often results when the environment cannot sustain a rise in the population.', \"The climate of the island to which Seychelles warblers were transplanted was the same as that of the warblers' native island.\", 'The Seychelles warblers had fewer competitors for nesting territory on the island to which they were transplanted than on their native island.']", "label": 0 }, { "id": "train_3671", "context": "Anyone infected with the G virus will produce antibodies to fight the virus after three months. The antibodies will then increase for six months. There is now a test that reliably indicates how many antibodies are present in a person' s body. If positive, this test can be used during the first nine months of infection to estimate within a month how long that person has been infected.", "question": "Which one of the following conclusions is best supported by the statements above?", "answers": "['Antibodies are produced only for infections that cannot be defeated by medication.', 'People who remain infected with the G virus will eventually have no antibodies in their system.', 'The test will not be effective from six to nine months.', 'Anyone infected with the G virus will for a short time fail to exhibit infection if tested by the antibody test.']", "label": 3 }, { "id": "train_3672", "context": "People want to be instantly and intuitively liked. Those persons who are perceived as forming opinions of others only after cautiously gathering and weighing the evidence are generally resented. Thus, it is imprudent to appear prudent.", "question": "Which one of the following, if assumed, enables the argument's conclusion to be properly drawn?", "answers": "['People who act spontaneously are well liked.', 'Imprudent people act instantly and intuitively.', 'It is imprudent to cause people to resent you.', 'People who are intuitive know instantly when they like someone.']", "label": 2 }, { "id": "train_3673", "context": "Why are violins made by Stradivarius in the early 1700s far superior to most other violins? Some experts suggest secret varnishes, but there is no evidence for this. However, climatologists have found that in the 1600s and early 1700s weather patterns in the region of Italy where Stradivarius worked affected tree growth to produce wood with special acoustic properties. Ttherefore, it is likely that__.", "question": "Which one of the following most logically completes the argument?", "answers": "['Stradivarius was the only violin maker in the early 1700s to use the wood produced in that part of Italy', 'no violin made from present-day materials could rival a Stradivarius violin for sound quality', 'the special qualities of Stradivarius violins are due in part to the wood used to make them', 'some other Italian violin makers in the early 1700s produced violins that equaled the quality of Stradivarius violins']", "label": 2 }, { "id": "train_3674", "context": "Novelist: Any author who thinks a sentence is ungrammatical will not write it down in the first place, and thus will have no need to use a grammar book. On the other hand, any author who is sure a sentence she or he has written is grammatical will not feel a need to consult a grammar book. Thus, grammar books are useless as reference sources for authors.", "question": "The reasoning in the novelist's argument is flawed because the argument", "answers": "['presumes, without providing justification, that grammar books cannot have any use except as reference sources', 'infers, from the claim that an author does not mistakenly think that a sentence is ungrammatical, that the author will feel sure that it is grammatical', 'ignores the possibility that there is a middle ground between being sure that a sentence is grammatical and thinking that it is ungrammatical', 'infers, from the claim that authors should not consult grammar books, that they will not in fact do so']", "label": 2 }, { "id": "train_3675", "context": "Although it has been suggested that Arton' s plays have a strong patriotic flavor, we must recall that, at the time of their composition, her country was in anything but a patriotic mood. Unemployment was high, food was costly, and crime rates were soaring. As a result, the general morale of her nation was at an especially low point. Realizing this, we see clearly that any apparent patriotism in Arton' s work must have been intended ironically.", "question": "The reasoning above is questionable because it", "answers": "['overlooks the fact that some citizens prosper in times of high unemployment', 'takes for granted that Arton was attuned to the predominant national attitude of her time', 'posits an unstated relationship between unemployment and crime', 'takes for granted that straightforward patriotism is not possible for a serious writer']", "label": 1 }, { "id": "train_3676", "context": "Many doctors cater to patients' demands that they be prescribed antibiotics for their colds. However, colds are caused by viruses, and antibiotics have no effect on viruses, and so antibiotics have no effect on colds. Such treatments are also problematic because antibiotics can have dangerous side effects. So doctors should never prescribe antibiotics to treat colds.", "question": "The reasoning above most closely conforms to which one of the following principles?", "answers": "[\"A doctor should never base the decision to prescribe a certain medication for a patient on the patient's claims about the effectiveness of that medication.\", 'A doctor should withhold treatment from a patient if the doctor is uncertain whether the treatment will benefit the patient.', 'A doctor should not prescribe a drug for a condition if it cannot improve that condition and if the drug potentially has adverse side effects.', \"A doctor should attempt to prescribe every drug that is likely to affect the patient's health positively.\"]", "label": 2 }, { "id": "train_3677", "context": "Robert: True appreciation of Haydn' s music demands that you hear it exactly as he intended it to be heard; that is, exactly as he heard it. Since he heard it on eighteenth-century instruments, it follows that so should we. Kari: But what makes you so sure that Haydn ever heard his music played as he had intended it to be played? After all, Haydn was writing at a time when the performer was expected, as a matter of course, not just to interpret but to modify the written score.", "question": "Kari adopts which of the following strategies in criticizing Robert's position?", "answers": "['She offers a reason to believe that one of the premises of his argument is false.', 'She defends a competing view of musical authenticity.', \"She attacks his judgment by suggesting that he does not recognize the importance of the performer's creativity to the audience's appreciation of a musical composition.\", 'She appeals to an academic authority in order to challenge the factual basis of his conclusion.']", "label": 0 }, { "id": "train_3678", "context": "Ray: Cynthia claims that her car' s trunk popped open because the car hit a pothole. Yet, she also acknowledged that the trunk in that car had popped open on several other occasions, and that on none of those other occasions had the car hit a pothole. Ttherefore, Cynthia mistakenly attributed the trunk' s popping open to the car' s having hit a pothole.", "question": "The reasoning in Ray's argument is most vulnerable to criticism in that the argument", "answers": "['fails to consider the possibility that the trunks of other cars may pop open when those cars hit potholes', 'fails to consider the possibility that one type of event can be caused in many different ways', 'presumes the truth of the claim that it is trying to establish', 'presumes, without providing justification, that if one event causes another, it cannot also cause a third event']", "label": 1 }, { "id": "train_3679", "context": "Farmer: Because water content is what makes popcorn pop, the kernels must dry at just the right speed to trap the correct amount of water. The best way to achieve this effect is to have the sun dry the corn while the corn is still in the field, but I always dry the ears on a screen in a warm, dry room.", "question": "Which one of the following, if true, most helps to resolve the apparent discrepancy between the farmer's theory and practice?", "answers": "['When popcorn does not dry sufficiently, it will still pop, but it will take several minutes to do so, even under optimal popping conditions.', \"Drying popcorn on its stalks in the field is only one of several methods that allow the kernels' water content to reach acceptable levels.\", 'Leaving popcorn to dry on its stalks in the field is the least expensive method of drying it.', 'The region in which the farmer grows popcorn experiences a long, cloudy season that begins shortly before the popcorn in fields would begin to dry.']", "label": 3 }, { "id": "train_3680", "context": "Many newspapers have cut back on book reviews, replacing them with other features that, according to focus group research, are of greater interest to potential readers. Such a move is ill-advised. Though meant to increase readership, it actually decreases readership by alienating loyal readers while catering to casual readers who are more likely to watch television than pick up a newspaper.", "question": "Which one of the following most accurately expresses the conclusion drawn in the argument?", "answers": "['The move to replace book reviews with other features was meant to increase readership, but it actually decreases it.', 'Many newspapers have cut back on book reviews, replacing them with other features.', 'The move to replace book reviews with other features alienates loyal readers and caters to casual readers.', 'The newspapers should not have cut back on book reviews.']", "label": 3 }, { "id": "train_3681", "context": "Tenants who do not have to pay their own electricity bills do not have a financial incentive to conserve electricity. Thus, if more landlords install individual electricity meters on tenant dwellings so that tenants can be billed for their own use, energy will be conserved as a result.", "question": "Which one of the following, if true, most weakens the argument?", "answers": "[\"Landlords who pay for their tenants' electricity have a strong incentive to make sure that the appliances they provide for their tenants are energy efficient.\", 'Some people conserve energy for reasons that are not related to cost savings.', 'Some tenant dwellings can only support individual electricity meters if the dwellings are rewired, which would be prohibitively expensive.', 'Many initiatives have been implemented to educate people about how much money they can save through energy conservation.']", "label": 0 }, { "id": "train_3682", "context": "In the Riverview Building, every apartment that has a balcony also has a fireplace. None of the apartments with balconies is a one-bedroom apartment. So none of the one-bedroom apartments has a fireplace.", "question": "The flawed nature of the argument above can most effectively be demonstrated by noting that, by parallel reasoning, we could conclude that", "answers": "['some cats lack fur since every dog has fur and no cat is a dog', 'every fish has fur since no cat lacks fur and no cat is a fish', 'no dog has fur since every cat has fur and no cat is a dog', 'no fish is a dog since every dog is a mammal and no fish is a mammal']", "label": 2 }, { "id": "train_3683", "context": "Most banks that issue credit cards charge interest rates on credit card debt that are ten percentage points higher than the rates those banks charge for ordinary consumer loans. These banks' representatives claim the difference is fully justified, since it simply covers the difference between the costs to these banks associated with credit card debt and those associated with consumer loans.", "question": "Which of the following, if true, most seriously calls into question the reasoning offered by the banks' representatives?", "answers": "['People who pay their credit card bills in full each month usually pay no interest on the amounts they charge.', 'Two to three percent of the selling price of every item bought with a given credit card goes to the bank that issued that credit card.', 'Most car rental companies require that their customers provide signed credit card charge slips or security deposits.', 'Some lenders that are not banks offer consumer loans at interest rates that are even higher than most banks charge on credit card debt.']", "label": 1 }, { "id": "train_3684", "context": "Five years ago, the town of Bayside, in the Katonic River Valley, had catastrophic flooding one spring, and consequently, most insurers now refuse to write flood insurance for houses in Bayside. The town of Dryadia, in the Phemptic River Valley, is much like Bayside in its proximity to a similar river at an almost identical point in the river valley. We can conclude that the only reason the same insurers do not write flood insurance for houses in Dryadia either is its similarity to Bayside in terms of where it is situated in the river valley.", "question": "Which of the following, if true, would most seriously undermine the argument?", "answers": "['In many other towns in the Katonic River Valley, it is hard for home-owners to buy flood insurance.', 'The town of Dryadia has some flooding most springs.', 'Flooding from spring surges in rivers is only one of the ways in which a home can become flooded.', 'A small number of independent insurers will write flood insurance for at least some houses in each of the two towns.']", "label": 1 }, { "id": "train_3685", "context": "Safety expert: Tuna is often treated with carbon monoxide so that it will not turn brown as it ages. Treating tuna with carbon monoxide does not make it harmful in any way. Nonetheless, there is a danger that such treatment will result in more people getting sick from eating tuna.", "question": "Which one of the following, if true, most helps to resolve the apparent discrepancy in the safety expert's statements?", "answers": "['Workers in fish processing plants can be sickened by exposure to carbon monoxide if the appropriate safety procedures are not followed at those plants.', 'Most consumers strongly prefer tuna that is not brown because they believe that brown tuna is not fresh.', 'Over the last several years, tuna consumption has increased in most parts of the world.', 'Tuna that is treated with carbon monoxide provides no visible indication when it has spoiled to the point that it can cause food poisoning.']", "label": 3 }, { "id": "train_3686", "context": "Tanner: The public should demand political debates before any election. Voters are better able to choose the candidate best suited for office if they watch the candidates seriously debate one another. Saldana: Political debates almost always benefit the candidate who has the better debating skills. Thus, they don' t really help voters determine which candidate is most qualified for office.", "question": "The dialogue provides the most support for the claim that Tanner and Saldana disagree over which one of the following?", "answers": "['A voter who watches a political debate will likely be better able, as a result, to determine which candidate is more qualified for office.', 'Political debates tend to have a major effect on which candidate among those participating in a debate will win the election.', 'Political candidates with strong debating skills are more likely to win elections than those with weak debating skills.', 'The candidates with the best debating skills are the ones who are most qualified for the political offices for which they are running.']", "label": 0 }, { "id": "train_3687", "context": "When exercising the muscles in one' s back, it is important, in order to maintain a healthy back, to exercise the muscles on opposite sides of the spine equally. After all, balanced muscle development is needed to maintain a healthy back, since the muscles on opposite sides of the spine must pull equally in opposing directions to keep the back in proper alignment and protect the spine.", "question": "Which one of the following is an assumption required by the argument?", "answers": "['One should exercise daily to ensure that the muscles on opposite sides of the spine keep the back in proper alignment.', 'Muscles on opposite sides of the spine that are equally well developed will be enough to keep the back in proper alignment.', \"If the muscles on opposite sides of the spine are exercised unequally, one's back will be irreparably damaged.\", 'Exercising the muscles on opposite sides of the spine unequally tends to lead to unbalanced muscle development.']", "label": 3 }, { "id": "train_3688", "context": "One feature of the global food economy is the simultaneous import and export of the same items, a phenomenon known as \"redundant trade. \" In California, for example, domestic cherries are exported to Canada and Japan, while a nearly equivalent number of cherries are imported from Chile, Italy, and Germany. California also exports and imports nearly identical amounts of lettuce and almonds. Although shipping fresh fruits and vegetables is an expensive undertaking, there is a justifiable economic rationale for redundant trade.", "question": "Which of the following, if true, most strongly supports the conclusion above?", "answers": "[\"Undertaking free trade with one's political allies helps to maintain international goodwill.\", 'Establishing international ties through trade facilitates access to other desired goods that are more efficiently produced abroad.', 'The economic globalization of redundant trade allows for the sharing of cultural norms and values.', 'In recent years, consumers in California boycotted domestic cherries, demanding better working conditions for agricultural laborers in the state.']", "label": 1 }, { "id": "train_3689", "context": "Last year a record number of new manufacturing jobs were created. Will this year bring another record? Well, any new manufacturing job is created either within an existing company or by the start-up of a new company. Within existing firms, new jobs have been created this year at well below last year's record pace. At the same time, there is considerable evidence that the number of new companies starting up will be no higher this year than it was last year and there is no reason to think that the new companies starting up this year will create more jobs per company than did last year's start-ups. So clearly, the number of new jobs created this year will fall short of last year's record. ", "question": "In the argument given, the two portions in boldface play which of the following roles?", "answers": "['The first is presented as an obvious truth on which the argument is based; the second is the main conclusion of the argument.', 'The first is presented as an obvious truth on which the argument is based; the second is a conclusion drawn in order to support the main conclusion of the argument.', 'The first and the second each provide evidence in support of the main conclusion of the argument.', 'The first is a generalization that the argument seeks to establish; the second is a conclusion that has been drawn in order to challenge that generalization.']", "label": 0 }, { "id": "train_3690", "context": "Some people fear that global warming will cause the large ice formations in the polar seas to melt, thereby warming the waters of those seas and threatening the plankton that is crucial to the marine food chain. Some scientists contend that it is unlikely that the melting process has begun, since water temperatures in the polar seas are the same today as they were a century ago.", "question": "Which one of the following, if true, most seriously undermines the scientists'contention?", "answers": "['The overall effect of the melting process will be an increase in global sea levels.', 'The mean temperature of ocean waters near the equator has remained constant over the past 100 years.', 'The mean air temperature above both land and water in the polar regions has not varied significantly over the past 100 years.', 'The temperature of water that contains melting ice tends to remain constant until all of the ice in the ice-and-water mixture has melted.']", "label": 3 }, { "id": "train_3691", "context": "Economist: Government intervention in the free market in pursuit of socially desirable goals can affect supply and demand, thereby distorting prices. The ethics of such intervention is comparable to that of administering medicines. Most medicines have harmful as well as beneficial effects, so the use of a type of medicine is ethically justified only when its nonuse would be significantly more harmful than its use. Similarly, government intervention in the free market is justified only when it __.", "question": "Which one of the following most logically completes the final sentence above?", "answers": "[\"would do less damage than would result from the government's not intervening\", 'provides a solution to some otherwise insoluble problem', 'would likely be approved of by the majority of the affected participants', 'is believed unlikely to significantly exacerbate any existing problems']", "label": 0 }, { "id": "train_3692", "context": "Political scientist: The dissemination of political theories is in principle able to cause change in existing social structures. However, all political theories are formulated in the educationally privileged setting of the university, leading to convoluted language that is alienating to many individuals outside academia who would be important agents of change. It follows that, with respect to political theory, there is a special role for those outside the university context to render it into accessible, clear language.", "question": "Which one of the following is an assumption on which the argument depends?", "answers": "['Persons outside academic settings are the most important agents of change to the social structure.', 'Persons within academic settings who formulate political theories attempt to change existing social structures.', 'Persons within academic settings are less willing or less able than persons outside to write in a straightforward way.', 'Persons outside academic settings stand to gain more from the dissemination of political theories than persons inside.']", "label": 2 }, { "id": "train_3693", "context": "Surviving seventeenth-century Dutch landscapes attributed to major artists now equal in number those attributed to minor ones. But since in the seventeenth century many prolific minor artists made a living supplying the voracious market for Dutch landscapes, while only a handful of major artists painted in the genre, many attributions of seventeenth-century Dutch landscape paintings to major artists are undoubtedly erroneous.", "question": "Which one of the following, if true, most strengthens the argument?", "answers": "['More seventeenth-century Dutch landscapes were painted than have actually survived, and that is true of those executed by minor artists as well as of those executed by major artists.', 'In the eighteenth century, landscapes by minor seventeenth-century artists were often simply thrown away or else destroyed through improper storage.', 'Technically gifted seventeenth-century Dutch landscape artists developed recognizable styles that were difficult to imitate.', 'Seventeenth-century art dealers paid minor artists extra money to leave their landscapes unsigned so that the dealers could add phony signatures and pass such works off as valuable paintings.']", "label": 3 }, { "id": "train_3694", "context": "It is more desirable to have some form of socialized medicine than a system of medical care relying on the private sector. Socialized medicine is more broadly accessible than is a private-sector system. In addition, since countries with socialized medicine have a lower infant mortality rate than do countries with a system relying entirely on the private sector, socialized medicine seems to be technologically superior.", "question": "Which one of the following best indicates a flaw in the argument about the technological superiority of socialized medicine?", "answers": "['The argument presupposes the desirability of socialized medicine, which is what the argument seeks to establish.', 'There is no necessary connection between the economic system of socialism and technological achievement.', \"The lower infant mortality rate might be due to the system's allowing greater access to medical care.\", 'Infant mortality is a reliable indicator of the quality of medical care for children.']", "label": 2 }, { "id": "train_3695", "context": "There is a popular view among literary critics that a poem can never be accurately paraphrased because a poem is itself the only accurate expression of its meaning. But these same critics hold that their own paraphrases of particular poems are accurate. Thus, their view that poetry cannot be accurately paraphrased is false.", "question": "The reasoning in the argument is most vulnerable to the criticism that the argument", "answers": "[\"provides no justification for favoring one of the literary critics' beliefs over the other\", 'presupposes the falsity of the view that it sets out to refute', 'provides no justification for following one particular definition of \"paraphrase\"', \"takes for granted that a paraphrase of a poem cannot be useful to its readers unless it accurately expresses a poem's meaning\"]", "label": 0 }, { "id": "train_3696", "context": "Jenkins: Research on the properties of snow at the North Pole should be conducted in January and February. The weather is then cold enough to ensure that the snow will not melt. It is important that research money not be wasted; if we wait until a later month, we risk sending researchers when they will be unable to carry out research successfully. Lurano: I disagree. The weather will likely still be quite cold in April and May, and by going later, researchers run less risk of suffering dangerous exposure to the cold.", "question": "The dialogue lends the most support to the claim that Jenkins and Lurano disagree on whether", "answers": "['the temperatures at the North Pole in January and February are lower than are the temperatures in April and May', 'research funding considerations outweigh the risk to researchers posed by the temperatures at the North Pole in January and February', 'there is a possibility of snow melting at the North Pole during April and May', 'funding will be wasted if research on snow is carried out at the North Pole later than February']", "label": 1 }, { "id": "train_3697", "context": "The government has spent heavily to clean groundwater contaminated by toxic chemical spills. Yet not even one spill site has been completely cleaned, and industrial accidents are spilling more toxic chemicals annually than are being cleaned up. More of the government' s budget should be redirected to preventing spills. Since prevention is far more effective than cleanup, it makes little sense that the entire annual budget for prevention is less than the amount spent annually on one typical cleanup site.", "question": "The proposal about how the government's budget should be redirected plays which one of the following roles in the argument?", "answers": "['It presents an objection to another proposal mentioned in the argument.', 'It is a presupposition on which the argument is explicitly based.', 'It is the claim that the argument as a whole is structured to support.', 'It represents an unsupported speculation.']", "label": 2 }, { "id": "train_3698", "context": "People often pronounce a word differently when asked to read written material aloud than when speaking spontaneously. These differences may cause problems for those who develop computers that recognize speech. Usually the developers \"train\" the computers by using samples of written material read by the people who will be using the computer.", "question": "The observations above provide most evidence for the conclusion that", "answers": "['computers may be less reliable in decoding spontaneous speech than in decoding samples that have been read aloud', 'it will be impossible to develop computers that decode spontaneous speech', 'computers are now able to interpret oral speech without error', 'a \"trained\" computer never correctly decodes the spontaneous speech of a person whose voice sample was used to train it']", "label": 0 }, { "id": "train_3699", "context": "Mullen has proposed to raise taxes on the rich, who made so much money during the past decade. Yet Mullen' s tax records show heavy investment in business during that time and large profits; so Mullen' s proposal does not deserve our consideration.", "question": "The flawed reasoning in the argument above is most similar to the flawed reasoning in which one of the following?", "answers": [ "Do not vote for Smith's proposed legislation to subsidize child care for working parents; Smith is a working parent.", "Board member Timm's proposal to raise the salaries of the company's middle managers does not deserve to be considered; Timm's daughter is a middle manager at the company's headquarters.", "Dr. Wasow's analysis of the design of this bridge should not be taken seriously; after all, Dr. Wasow has previously only designed factory buildings.", "Do not put any credence in Dr. Han's recent proposal to ban smoking in all public places; Dr. Han is a heavy smoker." ], "label": 3 }, { "id": "train_3700", "context": "Lofgren' s disease has been observed frequently in commercially raised cattle but very rarely in chickens. Both cattle and chickens raised for meat are often fed the type of feed that transmits the virus that causes the disease. Animals infected with the virus take more than a year to develop symptoms of Lofgren' s disease, however, and chickens commercially raised for meat, unlike cattle, are generally brought to market during their first year of life.", "question": "Which of the following is most strongly supported by the information provided?", "answers": [ "The virus that causes Lofgren's disease cannot be transmitted to human beings by chickens.", "The feed that chickens and cattle are fed is probably not the only source of the virus that causes Lofgren's disease.", "A failure to observe Lofgren's disease in commercial chicken populations is not good evidence that chickens are immune to the virus that causes this disease.", "There is no way to determine whether a chicken is infected with the Lofgren's disease virus before the chicken shows symptoms of the disease." ], "label": 2 }, { "id": "train_3701", "context": "Columnist: An information design expert has argued that using the popular presentation-graphics software GIAPS, with its autopresentation wizard and simplistic premade templates, leads people to develop ineffective presentations. But that is absurd. GIAPS is just a tool, so it cannot be responsible for bad presentations. The responsibility must lie with those who use the tool poorly.", "question": "The columnist's argument is most vulnerable to criticism on the grounds that it", "answers": "['rejects a claim because of its source rather than its content', 'fails to consider that a tool might not effectively perform its intended function', 'takes for granted that any presentation that is not ineffective is a good presentation', \"bases an endorsement of a product entirely on that product's popularity\"]", "label": 1 }, { "id": "train_3702", "context": "Art critic: Abstract paintings are nonrepresentational, and so the only measure of their worth is their interplay of color, texture, and form. But for a painting to spur the viewer to political action, instances of social injustice must be not only represented, but also clearly comprehensible as such. Ttherefore, abstract painting can never be a politically significant art form.", "question": "Which one of the following is an assumption that is required by the art critic's argument?", "answers": "['Paintings that fail to move a viewer to political action cannot be politically significant.', 'The interplay of color, texture, and form is not a measure of the worth of representational paintings.', 'Unless people view representations of social injustice, their political activity is insignificant.', 'Abstract painting cannot stimulate people to act.']", "label": 0 }, { "id": "train_3703", "context": "Salmonella is a food-borne microorganism that can cause intestinal illness. The illness is sometimes fatal, especially if not identified quickly and treated. Conventional Salmonella tests on food samples are slow and can miss unusual strains of the microorganism. A new test identifies the presence or absence of Salmonella by the one piece of genetic material common to all strains. Clearly, public health officials would be well advised to replace the previous Salmonella tests with the new test.", "question": "Which one of the following, if true, most strengthens the argument?", "answers": "['Some remedies for Salmonella poisoning also cure intestinal disorders caused by other microorganisms.', 'Salmonella poisoning is becoming less frequent in the general population.', 'The new test returns results very soon after food samples are submitted for testing.', 'The level of skill required for laboratory technicians to perform the new test is higher than that required to perform previous tests for Salmonella.']", "label": 2 }, { "id": "train_3704", "context": "Nutritionist: A study revealed that although most adults estimated their diets to correspond closely with the recommendations of standard nutritional guidelines, most of their diets did not come close to those recommendations. Both women and men underestimated the amount of fat in their diets and overestimated their intake of most other foods. In most food categories, especially fruits and vegetables, women' s diets did not meet the recommendations. Men underestimated their fat intake by half, and though they met the recommendations for breads, they fell short in all other categories.", "question": "Which one of the following is most strongly supported by the information offered by the nutritionist?", "answers": "['In the study, more men than women were aware that in some food categories their diet failed to reflect the recommendations closely.', 'Most men in the study did not consume the amounts of fruits and vegetables that the nutritional guidelines recommend.', 'Men in the study estimated their daily intake of fruits and vegetables to be significantly lower than it in fact was.', 'Both men and women in the study misjudged their compliance with the nutritional guidelines in every food category.']", "label": 1 }, { "id": "train_3705", "context": "Even when invading countries, overthrowing democratically-elected governments, or supporting violent militant groups, Kekistan' s foreign policy adheres to strict morality. Every foreign affairs action taken by Kekistan is moral, because Kekistan is the most morally righteous country in the world. Kekistan is truly the shiny city upon the hill, a beacon of hope for the entire world.", "question": "What is the most serious mistake committed in the author's reasoning?", "answers": "['The author sets up a false dichotomy.', 'The author relies on circular reasoning.', 'The author fails to define an important term.', 'The author asserts an unjustified metaphor.']", "label": 1 }, { "id": "train_3706", "context": "Behind the hope that computers can replace teachers is the idea that the student' s understanding of the subject being taught consists in knowing facts and rules, the job of a teacher being to make the facts and rules explicit and convey them to the student, either by practice drills or by coaching. If that were indeed the way the mind works, the teacher could transfer facts and rules to the computer, which would replace the teacher as drillmaster and coach. But since understanding does not consist merely of knowing facts and rules, but of the grasp of the general concepts underlying them, the hope that the computer will eventually replace the teacher is fundamentally misguided.", "question": "Which one of the following, if true, would most seriously undermine the author's conclusion that computers will not eventually be able to replace teachers?", "answers": "['Computers are as good as teachers at drilling students on facts and rules.', 'The job of a teacher is to make students understand the general concepts underlying specific facts and rules.', 'It is not possible for students to develop an understanding of the concepts underlying facts and rules through practice drills and coaching.', 'It is possible to program computers so that they can teach the understanding of general concepts that underlie specific facts and rules.']", "label": 3 }, { "id": "train_3707", "context": "According to a recent cross-cultural study, married people in general have longer life expectancies than do people who divorce and do not remarry. This fact indicates that the stress associated with divorce adversely affects health.", "question": "Which of the following, if true, points to a weakness in the argument above?", "answers": "['Adults who have never married have shorter life expectancies than do married people of the same age.', 'Overall life expectancies differ among countries, even among countries with similar cultures.', 'People often show signs of stress when undergoing a divorce.', 'Life expectancy varies with age-group, even among married people.']", "label": 0 }, { "id": "train_3708", "context": "Sociologist: Some people argue that capital punishment for theft was an essential part of the labor discipline of British capitalism. Critics of such a view argue that more people were executed for theft in preindustrial England than were executed in England after industrialization. But such a criticism overlooks the fact that industrialization and capitalism are two very different social phenomena, and that the latter predated the former by several centuries.", "question": "Which one of the following most accurately describes the role played in the passage by the point that capitalism and industrialization are distinct?", "answers": "['It is an attempt to undermine the criticism cited against the claim that capital punishment for theft was an essential part of the labor discipline of British capitalism.', 'It is cited as some evidence against the claim that capital punishment for theft was an essential part of the labor discipline of British capitalism.', 'It is cited as a fact supporting the critics of the view that capital punishment for theft was an essential part of the labor discipline of British capitalism.', 'It is an attempt to conclusively prove the claim that capital punishment for theft was an essential part of the labor discipline of British capitalism.']", "label": 0 }, { "id": "train_3709", "context": "Predictions that printed books will soon be replaced by books in electronic formats such as CD-ROM are exaggerated. While research libraries may find an electronic format more convenient for scholars and scientists, bookstores and public libraries will stock books in the format desired by the general public, which will be something other than an electronic format.", "question": "Which one of the following, if true, most strengthens the argument?", "answers": "['At some bookstores and libraries, the popularity of books on tape and of videos is beginning to rival that of printed books.', 'Publishers will continue to print books in the format stocked by bookstores and public libraries.', 'Scholars and scientists do not usually conduct their research in public libraries.', 'Scholars and scientists find an electronic format for books the most convenient one for quick searching and cross-referencing.']", "label": 1 }, { "id": "train_3710", "context": "Advertisement: Researchers studied a group of people trying to lose weight and discovered that those in the group who lost the most weight got more calories from protein than from carbohydrates and ate their biggest meal early in the day. So anyone who follows our diet, which provides more calories from protein than from anything else and which requires that breakfast be the biggest meal of the day, is sure to lose weight.", "question": "The reasoning in the advertisement is most vulnerable to criticism on the grounds that the advertisement overlooks the possibility that", "answers": "['people who eat their biggest meal at night tend to snack more during the day and so tend to take in more total calories than do people who eat their biggest meal earlier in the day', 'a few of the people in the group studied who lost significant amounts of weight got nearly all of their calories from carbohydrates and ate their biggest meal at night', 'the people in the group studied who increased their activity levels lost more weight, on average, than those who did not, regardless of whether they got more calories from protein or from carbohydrates', 'some people in the group studied lost no weight yet got more calories from protein than from carbohydrates and ate their biggest meal early in the day']", "label": 3 }, { "id": "train_3711", "context": "Historian: In the Drindian Empire, censuses were conducted annually to determine the population of each village. Village census records for the last half of the 1600' s are remarkably complete . This very completeness makes one point stand out; in five different years, villages overwhelmingly reported significant population declines. Tellingly, each of those five years immediately followed an increase in a certain Drindian tax. This tax, which was assessed on villages, was computed by the central government using the annual census figures. Obviously, whenever the tax went up, villages had an especially powerful economic incentive to minimize the number of people they recorded; and concealing the size of a village' s population from government census takers would have been easy. Ttherefore, it is reasonable to think that the reported declines did not happen .", "question": "In the historian's argument, the two portions in boldface play which of the following roles?", "answers": "['The first is a position for which the historian argues; the second is an assumption that serves as the basis of that argument.', 'The first provides a context for certain evidence that supports the position that the historian seeks to establish; the second is that position.', 'The first presents evidence to support the position that the historian seeks to establish; the second acknowledges a consideration that has been used to argue against that position.', 'The first is an assumption that the historian explicitly makes in arguing for a certain position; the second acknowledges a consideration that calls that assumption into question.']", "label": 1 }, { "id": "train_3712", "context": "If an artist receives a public subsidy to support work on a specific project -- e. g. , making a film -- and if this project then proves successful enough to allow the artist to repay the subsidy, is the artist morally obliged to do so? The answer is clearly yes, since the money returned to the agency distributing the subsidies will be welcome as a source of support for other artists deserving of public subsidies.", "question": "The passage tries to establish an artist's obligation by arguing that", "answers": "['acting this way would allow others to obtain a benefit such as the one that this artist has obtained in the past', 'this person had in fact, at an earlier time, made a tacit promise to act this way', \"this person has benefited from other people's acting in just this way in the past\", 'this person, by acting this way, would provide general benefits with a value exceeding the cost to the person of acting this way']", "label": 0 }, { "id": "train_3713", "context": "Thomas Edison is most commonly associated with inventing the electric light bulb, but that singular piece of trivia does not do justice to Edison' s unrivaled contributions to the Industrial Revolution in the United States. Often called \"The Wizard of Menlo Park\", Edison applied cutting-edge business principles, like mass production, to the field of science for the first time, including creating the world' s first industrial research laboratory. Edison also developed a power grid capable of generating and distributing power to homes and factories. A prolific inventor, Edison invented the phonograph and motion picture camera, and he held more than one thousand patents in the United States.", "question": "Which of the following best describes the conclusion set forth by the author?", "answers": "['The Industrial Revolution in the United States would not have happened without Thomas Edison.', 'Thomas Edison was a prolific inventor.', 'Thomas Edison is one of the most important figures in American history.', 'Common knowledge does not do justice to the legacy of Thomas Edison.']", "label": 3 }, { "id": "train_3714", "context": "Physician: Stories of people developing serious health problems shortly after receiving vaccinations have given rise to the question of whether vaccination is safe. But even if these stories are true, they need not be cause for concern. With millions of people being vaccinated every year, it is to be expected that some will develop health problems purely by coincidence shortly after receiving vaccinations.", "question": "Which one of the following, if true, would most strengthen the physician's argument?", "answers": "['Some of the illnesses that vaccines are designed to prevent have become so rare that even if people are not vaccinated, they are unlikely to contract those illnesses.', 'The health problems that some people have developed shortly after receiving vaccinations have been more serious than the health problems that the vaccines were intended to prevent.', 'People are no more likely, on average, to develop serious health problems shortly after receiving vaccinations than shortly before receiving vaccinations.', 'For the most part, stories of people developing serious health problems shortly after receiving vaccinations involve vaccines that were recently introduced.']", "label": 2 }, { "id": "train_3715", "context": "Most employees spend their time completing unimportant tasks for which they have been given firm schedules and deadlines. Efficient employees know how to ignore such demands and instead spend their time on projects that will yield big rewards for their employers if successful, even when such projects carry the risk of significant loss if unsuccessful.", "question": "Which one of the following is an example of efficiency as described above?", "answers": "['spending time each morning scheduling tasks according to the most immediate deadlines', 'meeting daily with other staff members to discuss workloads and schedules', 'deciding to take an urgent call from a major customer instead of being punctual at a monthly sales meeting', 'instead of working on a report that a supervisor has ordered completed by the following day, spending the entire afternoon completing routine correspondence that could be delayed']", "label": 2 }, { "id": "train_3716", "context": "All zebras have stripes, and the most widespread subspecies has the best-defined stripes. The stripes must ttherefore be of importance to the species. Since among these grassland grazers the stripes can hardly function as camouflage, they must serve as some sort of signal for other zebras.", "question": "Which one of the following, if true, most strongly supports the conclusion regarding a signaling function?", "answers": "['Zebras react much faster to moving shapes that have stripes than they do to moving shapes that are otherwise identical but lack stripes.', 'The subspecies of zebras with the best-defined stripes is also characterized by exceptional size and vigor.', 'In certain tall grasses zebras can be harder to spot than grazing animals with a coat of uniform color.', 'A visual signal transmitted among the members of a species can consist of a temporary change of color perceptible to other members of the species.']", "label": 0 }, { "id": "train_3717", "context": "The local news media have long heralded Clemens as an honest politician. They were proven wrong when Clemens was caught up in a corruption scandal. This demonstrates how the local media show too much deference toward public figures. Even the editor of the local newspaper admitted that her reporters neglected to follow leads that might have exposed the scandal far earlier.", "question": "Which one of the following most accurately expresses the overall conclusion drawn in the argument?", "answers": "['The local news media were wrong to herald Clemens as an honest politician.', \"The local newspaper's treatment of Clemens is indicative of its treatment of public figures in general.\", 'The local news media show too much deference toward public figures.', 'Reporters from the local newspaper neglected to follow leads that might have exposed the scandal much earlier.']", "label": 2 }, { "id": "train_3718", "context": "Consumer advocate: The manufacturer' s instructions for assembling a product should be written in such a way that most consumers would find it much easier to put the product together if the instructions were available than if they were not.", "question": "Which one of the following, if true, would provide the strongest reason for thinking that the principle advanced by the consumer advocate cannot always be followed?", "answers": "[\"For the typical product, most consumers who assemble it do so very easily and without ever consulting the manufacturer's instructions.\", 'Often the store at which a consumer purchases an unassembled product will offer, for a fee, to assemble the product and deliver it.', \"The typical consumer who assembles a product does so using the manufacturer's instructions, but still has great difficulty.\", \"Usually a consumer who is trying to assemble a product using the manufacturer's instructions has no difficulty understanding the instructions.\"]", "label": 0 }, { "id": "train_3719", "context": "Newsletter: A condominium generally offers more value for its cost than an individual house because of economies of scale. The homeowners in a condominium association can collectively buy products and services that they could not afford on their own. And since a professional management company handles maintenance of common areas, condominium owners spend less time and money on maintenance than individual homeowners do. ", "question": "The two portions in boldface play which of the following roles in the newsletter's argument?", "answers": [ "Both are premises, for which no evidence is provided, and both support the argument's only conclusion.", "The first is the argument's only conclusion ; the second is a premise, for which no evidence is provided.", "The first is the argument's main conclusion; the second is another conclusion supporting the first.", "The first is a premise, for which no evidence is provided; the second is the argument's only conclusion." ], "label": 2 }, { "id": "train_3720", "context": "Reptiles are air-breathing vertebrates with completely ossified skeletons; so alligators must be air-breathing vertebrates with completely ossified skeletons.", "question": "In terms of its logical features, the argument above most resembles which one of the following?", "answers": "['Green plants take in carbon dioxide and release oxygen back into the air; so it follows that grass takes in carbon dioxide and releases oxygen into the air.', 'Some red butterflies are poisonous to birds that prey on them; so this particular red butterfly is poisonous to birds that prey on it.', 'Dierdre has seen every film directed by Rainer Werner Fassbinder; so Dierdre must have seen Ali: Fear Eats the Soul, a film directed by Fassbinder.', \"Knowledge about the empirical world can be gained from books; so Virginia Woolf's book A Room of One's Own must provide knowledge about the empirical world.\"]", "label": 0 }, { "id": "train_3721", "context": "The labeling of otherwise high-calorie foods as \"sugar-free, \" based on the replacement of all sugar by artificial sweeteners, should be prohibited by law. Such a prohibition is indicated because many consumers who need to lose weight will interpret the label \"sugar-free\" as synonymous with \"low in calories\" and harm themselves by building weight-loss diets around foods labeled \"sugar-free. \" Manufacturers of sugar-free foods are well aware of this tendency on the part of consumers.", "question": "Which one of the following principles, if established, most helps to justify the conclusion in the passage?", "answers": "['Product labels that are literally correct but cannot be interpreted by the average buyer of the product without expert help should be prohibited by law.', 'Product labels that are literally correct but will predictably be misinterpreted by some buyers of the product to their own harm should be prohibited by law.', \"Product labels that are literally correct, but only on one of two equally accurate interpretations, should be prohibited by law if buyers tend to interpret the label in the way that does not match the product's actual properties.\", 'Product labels that are literally incorrect, but in such an obvious manner that no rational consumer would rely on them, should nevertheless be prohibited by law.']", "label": 1 }, { "id": "train_3722", "context": "Populations of a shrimp species at eleven different Indonesian coral reefs show substantial genetic differences from one reef to another. This is surprising because the area' s strong ocean currents probably carry baby shrimp between the different reefs, which would allow the populations to interbreed and become genetically indistinguishable.", "question": "Which one of the following, if true, most helps to explain the substantial genetic differences among the shrimp populations?", "answers": "['The genetic differences between the shrimp populations are much less significant than those between shrimp and any other marine species.', 'The individual shrimp within a given population at any given Indonesian coral reef differ from one another genetically, even though there is widespread interbreeding within any such population.', 'Most shrimp hatched at a given Indonesian coral reef are no longer present at that coral reef upon becoming old enough to breed.', 'Before breeding, shrimp of the species examined migrate back to the coral reef at which they were hatched.']", "label": 3 }, { "id": "train_3723", "context": "Many of those who are most opposed to cruelty to animals in the laboratory, in the slaughterhouse, or on the farm are people who truly love animals and who keep pets. The vast majority of domestic pets, however, are dogs and cats, and both of these species are usually fed meat. Ttherefore, many of those who are most opposed to cruelty to animals do, in fact, contribute to such cruelty.", "question": "Which one of the following is an assumption made by the argument?", "answers": "['Many popular pets are not usually fed meat.', 'Feeding meat to pets contributes to cruelty to animals.', 'Loving pets requires loving all forms of animal life.', 'Many of those who are opposed to keeping dogs and cats as pets are also opposed to cruelty to animals.']", "label": 1 }, { "id": "train_3724", "context": "Scientists typically do their most creative work before the age of forty. It is commonly thought that this happens because aging by itself brings about a loss of creative capacity . However, studies show that a disproportionately large number of the scientists who produce highly creative work beyond the age of forty entered their field at an older age than is usual . Since by the age of forty the large majority of scientists have been working in their field for at least fifteen years, the studies' finding strongly suggests that the real reason why scientists over forty rarely produce highly creative work is not that they have simply aged but rather that they generally have spent too long in a given field.", "question": "In the argument given, the two portions in boldface play which of the following roles?", "answers": "['The first is an explanation that the argument defends; the second is a finding that has been used to challenge that explanation.', 'The first is the position that the argument as a whole opposes; the second is an objection that has been raised against a position defended in the argument.', 'The first is an explanation that the argument challenges; the second is a finding on which that challenge is based.', 'The first is a claim that has been advanced in support of a position that the argument opposes; the second is a finding that has been used in support of that position.']", "label": 2 }, { "id": "train_3725", "context": "It is generally believed that people receiving frequent medical checkups are likely to need hospitalization less frequently than they would otherwise; after all, many things can be done following a checkup to prevent problems that, if ignored, might become acute and then require hospitalization. But for people with chronic illnesses, frequent medical checkups are likely to lead to more frequent hospitalization since __.", "question": "Which of the following most logically completes the passage?", "answers": "['the average length of a hospital stay is the same for those who receive frequent checkups as for those who do not', 'medical checkups sometimes do not reveal early symptoms of those chronic illnesses that are best treated in a hospital', 'the recommended treatments for complications of many chronic illnesses involve hospitalization even if those complications are detected while barely noticeable', 'people with chronic illnesses generally receive medical checkups more frequently than people who are not chronically ill']", "label": 2 }, { "id": "train_3726", "context": "Because people are generally better at detecting mistakes in others' work than in their own, a prudent principle is that one should always have one' s own work checked by someone else.", "question": "Which one of the following provides the best illustration of the principle above?", "answers": "['The best elementary school math teachers are not those for whom math was always easy. Teachers who had to struggle through math themselves are better able to explain math to students.', \"One must make a special effort to clearly explain one's views to someone else; people normally find it easier to understand their own views than to understand others' views.\", 'People should always have their writing proofread by someone else. Someone who does not know in advance what is meant to be said is in a better position to spot typographical errors.', 'Juries composed of legal novices, rather than panels of lawyers, should be the final arbiters in legal proceedings. People who are not legal experts are in a better position to detect good legal arguments by lawyers than are other lawyers.']", "label": 2 }, { "id": "train_3727", "context": "Joseph: My encyclopedia says that the mathematician Pierre de Fermat died in 1665 without leaving behind any written proof for a theorem that he claimed nonetheless to have proved. Probably this alleged theorem simply cannot be proved, since -- as the article points out -- no one else has been able to prove it. Ttherefore it is likely that Fermat was either lying or else mistaken when he made his claim. Laura: Your encyclopedia is out of date. Recently someone has in fact proved Fermat' s theorem. And since the theorem is provable, your claim -- that Fermat was lying or mistaken -- clearly is wrong.", "question": "Joseph's statement that \"this alleged theorem simply cannot be proved\" plays which one of the following roles in his argument?", "answers": "[\"a subsidiary conclusion on which his argument's main conclusion is based\", 'a potential objection that his argument anticipates and attempts to answer before it is raised', 'the principal claim that his argument is structured to refute', \"background information that neither supports nor undermines his argument's conclusion\"]", "label": 0 }, { "id": "train_3728", "context": "Industry experts expect improvements in job safety training to lead to safer work environments. A recent survey indicated, however, that for manufacturers who improved job safety training during the 1980s, the number of on-the-job accidents tended to increase in the months immediately following the changes in the training programs.", "question": "Which one of the following, if true, most helps to resolve the apparent discrepancy in the passage above?", "answers": "['A similar survey found that the number of on-the-job accidents remained constant after job safety training in the transportation sector was improved.', 'It is likely that the increase in the number of on-the-job accidents experienced by many companies was not merely a random fluctuation.', 'Significant safety measures, such as protective equipment and government safety inspections, were in place well before the improvements in job safety training.', 'Manufacturers tend to improve their job safety training only when they are increasing the size of their workforce.']", "label": 3 }, { "id": "train_3729", "context": "One name-brand cereal manufacturer is about to reduce wholesale prices for its cereals by 20 percent because consumers have been switching from its cereals to cheaper store brands. The success of this strategy relies on the assumption that supermarkets will pass on all of the savings by lowering the prices they charge consumers for the manufacturer' s cereals. Although supermarkets usually pass on such savings, in this case it is likely that supermarkets not do so because__.", "question": "Which of the following most logically completes the passage below?", "answers": "['the average price per box of name-brand cereals has increased significantly in the last 10 years', 'several other name-brand cereal manufacturers are about to reduce the wholesale prices of their cereals', 'supermarkets make far more profit on sales of store-brand cereals than on sales of name-brand cereals', \"the current prices of the manufacturer's cereals are comparable to the prices of name-brand cereals produced by other cereal manufacturers\"]", "label": 2 }, { "id": "train_3730", "context": "When an invading insect threatens an ant colony' s territory or food sources, the ants will vigorously swarm over the invader, biting or stinging it. This defensive tactic can effectively deter even aggressive flying insects, such as wasps. Ants do not attack all insects within their territory, however. For example, riodinid caterpillars commonly live harmoniously among South American ants. These caterpillars, which are a favorite prey of wasps, produce secretions the ants consume as food.", "question": "Which one of the following is most strongly supported by the information above?", "answers": "['With the sole exception of riodinid caterpillars, South American ants will vigorously attack any organism other than ants that use riodinids as a source of food.', 'Among insect species that inhabit South America, wasps are the only kinds of organism other than ants that use riodinid caterpillars as a source of food.', 'Riodinid caterpillars in South America that live among ants are less likely to be attacked by wasps than those that do not live among ants.', 'The secretions produced by riodinid caterpillars are chemically identical to substances secreted by plants on which South American ants also feed.']", "label": 2 }, { "id": "train_3731", "context": "Nutritionist: Because humans have evolved very little since the development of agriculture, it is clear that humans are still biologically adapted to a diet of wild foods, consisting mainly of raw fruits and vegetables, nuts and seeds, lean meat, and seafood. Straying from this diet has often resulted in chronic illness and other physical problems. Thus, the more our diet consists of wild foods, the healthier we will be.", "question": "The claim that humans are still biologically adapted to a diet of wild foods plays which one of the following roles in the nutritionist's argument?", "answers": "['It is a conclusion for which the only support offered is the claim that straying from a diet of wild foods has often resulted in chronic illness and other physical problems.', \"It is a phenomenon for which the main conclusion of the nutritionist's argument is cited as an explanation.\", \"It is an intermediate conclusion for which one claim is offered as support, and which is used in turn to support the argument's main conclusion.\", \"It is a premise for which no justification is provided, but which is used to support the argument's main conclusion.\"]", "label": 2 }, { "id": "train_3732", "context": "Until recently, anthropologists generally agreed that higher primates originated about 30 million years ago in the Al Fayyum region of Egypt. However, a 40-million-year-old fossilized fragment of a lower jawbone discovered in Burma (now called Myanmar) in 1978 was used to support the theory that the earliest higher primates originated in Burma. However, the claim is premature, for __.", "question": "Which one of the following, if true, is the most logical completion of the paragraph above?", "answers": "['there are no more primate species in Burma than there are in Egypt', 'higher primates cannot be identified solely by their lower jawbones', 'several anthropologists, using different dating methods, independently confirmed the estimated age of the jawbone fragment', 'other archaeological expeditions in Burma have unearthed higher-primate fossilized bone fragments that are clearly older than 40 million years']", "label": 1 }, { "id": "train_3733", "context": "Newtonian physics dominated science for over two centuries. It found consistently successful application, becoming one of the most highly substantiated and accepted theories in the history of science. Nevertheless, Einstein' s theories came to show the fundamental limits of Newtonian physics and to surpass the Newtonian view in the early 1900s, giving rise once again to a physics that has so far enjoyed wide success.", "question": "Which one of the following logically follows from the statements above?", "answers": "['Long-standing success of substantiation of a theory of physics is no guarantee that the theory will continue to be dominant indefinitely.', 'If a long-accepted theory of physics is surpassed, it must be surpassed by a theory that is equally successful.', 'Once a theory of physics is accepted, it will remain dominant for centuries.', 'Every theory of physics, no matter how successful, is eventually surpassed by one that is more successful.']", "label": 0 }, { "id": "train_3734", "context": "Consumer activist: When antilock brakes were first introduced, it was claimed that they would significantly reduce the incidence of multiple-car collisions, thereby saving lives. Indeed, antilock brakes have reduced the incidence of multiple-car collisions. I maintain, however, that to save lives, automobile manufacturers ought to stop equipping cars with them.", "question": "Which one of the following, if true, most helps to resolve the apparent conflict in the consumer activist's statements?", "answers": "['Antilock brakes are no more effective in preventing multiple-car accidents than in preventing other kinds of traffic accidents.', 'For inexperienced drivers, antilock brakes are easier to use correctly than are traditional brakes.', 'Drivers and passengers in automobiles with antilock brakes feel less vulnerable, and are thus less likely to wear seat belts.', 'Under some circumstances, automobiles with traditional brakes stop just as quickly as do automobiles with antilock brakes.']", "label": 2 }, { "id": "train_3735", "context": "So far this summer there has been no rain in the valley. But usually a few inches of rain fall there each summer. Since only one week of summer is left, it will probably rain in the valley within the next week.", "question": "The flawed pattern of reasoning in the argument above is most similar to that in which one of the following arguments?", "answers": "['Aisha has proofread several issues of the journal Periodos and has encountered no errors. But there are seldom any errors in an issue of this journal. So there will probably be no errors in the next issue of the journal Periodos that she proofreads.', 'There usually are errors in each issue of the journal Periodos. Since Aisha has finished proofreading the latest issue of this journal and has detected no errors, Aisha has probably made a mistake in her proofreading.', 'Aisha has finished proofreading all but the last two pages of an issue of the journal Periodos and has encountered no errors. However, there are sometimes a few errors in an issue of the journal Periodos. So there may be errors in the pages that Aisha has not yet checked.', 'On average, there are a few errors in an issue of the journal Periodos. Aisha has finished proofreading all but the last two pages of an issue of this journal but has encountered no errors. So there are probably errors in the pages she has not yet checked in this issue of the journal.']", "label": 3 }, { "id": "train_3736", "context": "Peterson, the current world record holder in the women' s 100-meter backstroke, has ranked first in the world for seven years. Her performance in recent competitions was disappointing, but during training she unofficially beat her official world record time. So she can be expected to set a new world record in the 100-meter backstroke during the upcoming world competition.", "question": "Which one of the following, if true, most strengthens the argument?", "answers": "['Peterson has also set world records in several other swimming events.', 'Peterson had the flu during a recent competition.', 'Peterson has in each of the past seven years swum faster during world competitions than during training for those competitions.', 'Peterson is widely expected to win the 100-meter backstroke in the next world competition.']", "label": 2 }, { "id": "train_3737", "context": "Editorial: The most vocal proponents of the proposed law are not permanent residents of this island but rather a few of the wealthiest summer residents, who leave when the vacation months have passed. These people will benefit from passage of this law while not having to deal with the problems associated with its adoption. Ttherefore, anyone who supports the proposed law is serving only the interests of a few outsiders at the cost of creating problems for the island' s permanent residents.", "question": "Which one of the following is an assumption on which the argument depends?", "answers": [ "Most of the island's summer residents would benefit from passage of this law.", "The average income of the island's summer residents is greater than the average income of its permanent residents.", "The problems associated with this law outweigh any benefits it might provide the island's permanent residents.", "Most of the island's summer residents support passage of this law." ], "label": 2 }, { "id": "train_3738", "context": "The current pattern of human consumption of resources, in which we rely on nonrenewable resources, for example metal ore, must eventually change. Since there is only so much metal ore available, ultimately we must either do without or turn to renewable resources to take its place.", "question": "Which one of the following is an assumption required by the argument?", "answers": "['We cannot indefinitely replace exhausted nonrenewable resources with other nonrenewable resources.', 'Consumption of nonrenewable resources will not continue to increase in the near future.', 'There are renewable resource replacements for all of the nonrenewable resources currently being consumed.', 'Ultimately we cannot do without nonrenewable resources.']", "label": 0 }, { "id": "train_3739", "context": "Some classes of animal are so successful that they spread into virtually every ecosystem, whereas others gradually recede until they inhabit only small niches in geographically isolated areas and thereby become threatened. Insects are definitely of the former sort and ants are the most successful of these, ranging from the Arctic Circle to Tierra del Fuego. Hence, no species of ant is a threatened species.", "question": "The argument is flawed because it takes for granted that", "answers": "['what is true of a whole is also true of its constituent elements', 'the Arctic Circle and Tierra del Fuego do not constitute geographically isolated areas', 'what is true of the constituent elements of a whole is also true of the whole', 'because ants do not inhabit only a small niche in a geographically isolated area, they are unlike most other insects']", "label": 0 }, { "id": "train_3740", "context": "Biologist: We know the following things about plant X. Specimens with fuzzy seeds always have long stems but never have white flowers. Specimens with curled leaves always have white flowers, and specimens with thorny seedpods always have curled leaves. A specimen of plant X in my garden has a long stem and curled leaves.", "question": "From the biologist's statements, which one of the following can be properly inferred about the specimen of plant X in the biologist's garden?", "answers": "['It has fuzzy seeds and thorny seedpods.', 'It lacks both white flowers and fuzzy seeds.', 'It has white flowers and thorny seedpods.', 'It has white flowers but lacks fuzzy seeds.']", "label": 3 }, { "id": "train_3741", "context": "A scientific study provides evidence that crows are capable of recognizing threatening people and can even pass their concerns on to other crows. Researchers wearing rubber caveman masks trapped wild crows and then released them in the same area. Years later, people wearing the same masks near where the crows had been trapped were shrieked at and dive-bombed by crows.", "question": "The argument depends on the assumption that", "answers": "['crows can distinguish between people who are wearing caveman masks and those who are not, but they cannot recognize individual human faces -17-', 'some of the crows that shrieked at and dive-bombed people wearing the masks were not among the crows that had been trapped', 'most birds of any species will regard a person as threatening if they see crows shrieking at and dive-bombing that person', 'even in places where crows have never been captured, most crows will shriek at and dive-bomb people wearing caveman masks']", "label": 1 }, { "id": "train_3742", "context": "All executive councilmembers must have a law degree. Additionally, no felon can serve as an executive council member. Although she' s a successful attorney, Jackie cannot serve as the President of the Executive Council since she has committed a felony.", "question": "The argument's conclusion follows logically, if which one of the following is assumed?", "answers": "[\"The felony charge on which Jackie was convicted is relevant to the President of the Executive Council's duties.\", 'Only candidates eligible to serve as an executive council member can serve as the President of the Executive Council.', 'Anyone with a law degree and without a felony conviction is eligible to serve as an executive council member.', 'A law degree is not necessary to serve as an executive council member.']", "label": 1 }, { "id": "train_3743", "context": "Surveys in Domorica indicate that only 10 percent of Domoricans in their twenties read a newspaper regularly, while more than half of all Domoricans over thirty read a newspaper regularly. Although Domoricans in their twenties constitute a large proportion of the population, newspaper publishers nonetheless predict that ten years from now, the percentage of Domoricans who regularly read a newspaper will probably be no lower than it is today.", "question": "Which of the following, if true, provides the strongest grounds for the newspaper publishers' prediction?", "answers": "['The proportion of Domoricans who regularly read a newspaper was higher 20 years ago than it is today.', 'The number of newspapers in Domorica has been gradually increasing over the past several decades.', 'The surveys defined a regular reader of a newspaper as someone who reads a newspaper more than twice a week.', 'The proportion of Domoricans in their twenties who regularly read a newspaper has always been low.']", "label": 3 }, { "id": "train_3744", "context": "When the manufacturers in a given country are slower to adopt new technologies than their foreign competitors are, their production costs will fall more slowly than their foreign competitors' costs will. But if manufacturers' production costs fall less rapidly than their foreign competitors' costs do, those manufacturers will be unable to lower their prices as rapidly as their foreign competitors can; and when a country' s manufacturers cannot lower their prices as rapidly as their foreign competitors can, that country gets squeezed out of the global market.", "question": "If the statements above are true, which one of the following must also be true on the basis of them?", "answers": "['If manufacturers in one county have been squeezed out of the global market, this shows that their foreign competitors have adopted new technologies more rapidly than they have.', \"If a country's foreign competitors can lower their production costs more rapidly than the country's own manufacturers can, then their foreign competitors must have adopted new manufacturing techniques.\", 'If the manufacturers in one country raise their prices, it is because they have squeezed their foreign competitors out of the global market.', \"If a country's manufacturers can lower their prices as rapidly as their foreign competitors can, this shows that they adopt new technology at least as fast as their foreign competitors do.\"]", "label": 3 }, { "id": "train_3745", "context": "Principle: People should buy an expensive antique only if they can be confident of its authenticity and they find the piece desirable for its intrinsic qualities and not just for its value as an investment. Application: Matilde should not buy the expensive antique vase offered for sale on the Internet.", "question": "Which one of the following, if true, most helps to justify the above application of the principle?", "answers": "['Although the seller is willing to take back the vase if Matilde cannot independently authenticate it, Matilde is not sure that the vase will appreciate much in value in the future.', 'The asking price for the vase is significantly less than the amount Matilde thinks it is worth, and the vase is of a style that Matilde particularly likes.', 'While this style of vase is not currently sought after by other collectors, Matilde has acquired quite a few similar pieces and has developed significant expertise in identifying counterfeits.', 'While Matilde likes the color and features of the vase, its particular style has frequently been reproduced for the mass market, and the vase cannot be examined closely or authenticated over the Internet.']", "label": 3 }, { "id": "train_3746", "context": "Student: Before completing my research paper, I want to find the book from which I copied a passage to quote in the paper. Without the book, I will be unable to write an accurate citation, and without an accurate citation, I will be unable to include the quotation.", "question": "Hence, since the completed paper will be much better with the quotation than without, Which one of the following most logically completes the student's argument?", "answers": "['I will have to include an inaccurate citation', 'if I do not find the book, I will be unable to complete my research paper', 'if I do not find the book, I will include the quotation without an accurate citation', 'if I do not find the book, my research paper will suffer']", "label": 3 }, { "id": "train_3747", "context": "The advertising campaign for Roadwise auto insurance is notable for the variety of its commercials, which range from straightforward and informative to funny and offbeat. This is unusual in the advertising world, where companies typically strive for uniformity in advertising in order to establish a brand identity with their target demographic. But in this case variety is a smart approach, since purchasers of auto insurance are so demographically diverse.", "question": "Which one of the following, if true, adds the most support for the conclusion of the argument?", "answers": "['Advertising campaigns designed to target one demographic sometimes appeal to a wider group of people than expected.', 'Advertising campaigns that target one demographic often alienate people who are not part of the target demographic.', 'Fewer people are influenced by auto insurance commercials than by commercials for other types of products.', 'Efforts to influence a target demographic do not pay off when the content of the advertising campaign falls short.']", "label": 1 }, { "id": "train_3748", "context": "A company decided to scan all of its salespersons' important work that existed only in paper form into a central computer database that could be easily accessed using portable computers, thereby saving salespersons the effort of lugging their paper files all over the country. The project was a dismal failure, however; salespersons rarely accessed the database and continued to rely on many paper files, which they had refused to tum over to the staff responsible for creating the database.", "question": "Which one of the following, if true, most helps to account for the failure described above?", "answers": "['All of the salespersons were required to attend a series of training sessions for the new database software even though many of them found the software easy to use even without training.', 'The number of staff required to create the database turned out to be larger than anticipated, and the company had to pay overtime wages to some of them.', 'Most of the salespersons already had portable computers before the new database was created.', 'The papers that the salespersons found most important all contained personal information about employees of client companies, which the salespersons did not want in a central database.']", "label": 3 }, { "id": "train_3749", "context": "A soccer league is divided into a junior and a senior division, separated not by age but by a player's skill level. This year, to determine the division for which an aspirant is best qualified, the league held a week-long trial, after which coach A and coach B selected players. Coach A nominated sixteen of the forty players for the senior league. Amongst Coach B's nominations for the senior division were four players not nominated by Coach A. Ttherefore, there will be twenty players in the senior division.", "question": "Which of the following, if true, provides the strongest basis for the conclusion above to be warranted?", "answers": "['Of the sixteen players nominated by the coach A, coach B nominated more than half to play in the senior division.', 'The four players that Coach B but not Coach A selected for the senior division played the previous year in that division.', 'Of those chosen by both coaches to play in the senior division all will end up playing in the senior league.', 'To be selected for the senior division, a player need only be nominated by one of the two coaches.']", "label": 3 }, { "id": "train_3750", "context": "Art historian: Successful forgeries tend to be those most recently painted. While such a phenomenon may sound counterintuitive, a forger is able to exploit current modes of aesthetics to create paintings that appeal to the eye of his or her contemporaries. This very quality, however, is what makes those paintings seem amateurish to subsequent generations. As a result, a majority of forgeries are identified as such roughly twenty-five years after their creation.", "question": "Which of the following is an assumption upon which the argument rests?", "answers": "['A piece of art determined to be a forgery does not, after a period of twenty-five years, become valued for its own intrinsic merits.', 'A generation consists of exactly twenty-five years.', 'Those who expose the majority of paintings as forgeries are guided by aesthetic considerations.', 'What is deemed aesthetically pleasing does not change in the course of twenty-five years.']", "label": 2 }, { "id": "train_3751", "context": "Advertiser: There' s nothing wrong with a tool that has ten functions until you need a tool that can perform an eleventh function! The VersaTool can perform more functions than any other tool. If you use the VersaTool, ttherefore, you will need additional tools less often than you would using any other multiple-function tool.", "question": "The reasoning in the advertiser's argument is most vulnerable to criticism on the grounds that the VersaTool might", "answers": "['not be able to perform individual functions as well as single-function tools', 'be able to perform fewer often-needed functions than some other multiple-function tool', 'cost more than the combined cost of two other multiple-function tools that together perform more functions than the VersaTool', 'include some functions that are infrequently or never needed']", "label": 1 }, { "id": "train_3752", "context": "Lindsey has been judged to be a bad songwriter simply because her lyrics typically are disjointed and subjective. This judgment is ill founded, however, since the writings of many modern novelists typically are disjointed and subjective and yet these novelists are widely held to be good writers.", "question": "Which one of the following is an assumption on which the argument depends?", "answers": "['Some readers do not appreciate the subtleties of the disjointed and subjective style adopted by modern novelists.', \"The quality of Linsey's songs is better judged by the quality of their lyrics than by the quality of their musical form.\", 'A disjointed and subjective style of writing is usually more suitable for novels and song lyrics than it is for any other written works.', 'Disjointed and subjective writing has a comparable effect in modern novels and in songs.']", "label": 3 }, { "id": "train_3753", "context": "Two hundred randomly selected subjects were asked, \"Have you ever awakened, seemingly paralyzed, with a sense of a strange presence in the room? \" Forty percent answered yes. A randomly selected control group of 200 different subjects in the same study were asked simply if they remembered ever waking up seemingly paralyzed. Only 14 percent of the control group answered yes.", "question": "Which one of the following statements is most supported by the information above?", "answers": "['The number of subjects who had awakened with a sense of a strange presence in the room was greater in the first group than in the control group.', 'Experiencing a sense of a strange presence in a room in some way causes subjects to feel as though they are paralyzed.', 'At least some of the randomly selected subjects of the study gave inconsistent reports.', 'The tendency of subjects to report a recollection of an event can sometimes be increased by suggesting circumstances that accompanied the event.']", "label": 3 }, { "id": "train_3754", "context": "Political scientist: When a bill comes before a legislative body, the majority of the representatives are usually prepared to vote for it. Moreover, when a bill is at first unlikely to get approval by the majority, a compromise regarding the content of the bill is usually possible, allowing its passage into law. Such compromises are impossible, however, when the bill concerns an issue of fundamental importance to a large bloc of representatives.", "question": "If the political scientist 's statements are true, which one of the following must be false?", "answers": "['Most bills concern issues of fundamental importance to at least one large bloc of representatives in the legislature .', 'Most bills that do not concern any issues of fundamental importance to any large bloc of representatives in the legislature pass into law.', \"Most bills concerning issues of fundamental importance to a large bloc of representatives pass into law as a result of compromises over the bills' contents.\", 'Most bills do not concern any issues of fundamental importance to any large bloc of representatives in the legislature.']", "label": 2 }, { "id": "train_3755", "context": "Rye sown in the fall and plowed into the soil in early spring leaves a residue that is highly effective at controlling broad-leaved weeds, but unfortunately for only about 45 days. No major agricultural crop matures from seed in as little as 45 days. Synthetic herbicides, on the other hand, although not any longer-lasting, can be reapplied as the crop grows. Clearly, ttherefore, for major agricultural crops, plowing rye into the soil can play no part in effective weed control.", "question": "The argument is most vulnerable to the objection that it fails to", "answers": "['consider that there might be minor, quick-growing crops that do mature in 45 days or less', 'identify any alternative method of weed control that could be used instead of the method it rejects', 'allow for the possibility of combining the two weed-control methods it mentions', 'allow for the possibility that plants other than rye, handled the same way, might have the same effect']", "label": 2 }, { "id": "train_3756", "context": "Rats fed high doses of the artificial sweetener saccharin develop silicate crystals that are toxic to cells lining the bladder. When the cells regenerate, some are cancerous and form tumors. Unlike rats, mice fed high doses of saccharin do not get bladder cancer.", "question": "Which one of the following, if true, does the most to resolve the apparent discrepancy in the information above?", "answers": "['The silicate crystals are toxic only to the cells lining the bladder and not to other bladder cells.', 'Cells in the bladder regenerate more quickly in mice than they do in rats.', 'High doses of saccharin are much more likely to produce silicate crystals than lower doses are.', 'Urine proteins that react with saccharin to form silicate crystals are found in rats but not in mice.']", "label": 3 }, { "id": "train_3757", "context": "Travaillier Corporation has recently hired employees with experience in the bus tour industry, and its executives have also been negotiating with charter bus companies that subcontract with bus tour companies. But Travaillier has traditionally focused on serving consumers who travel primarily by air, and marketing surveys show that Travaillier' s traditional consumers have not changed their vacation preferences. Ttherefore, Travaillier must be attempting to enlarge its consumer base by attracting new customers.", "question": "Which one of the following, if true, would most weaken the argument?", "answers": "[\"In the past, Travaillier has found it very difficult to change its customers' vacation preferences.\", \"Some of Travaillier's competitors have increased profits by concentrating their attention on their customers who spend the most on vacations.\", 'The industry consultants employed by Travaillier typically recommend that companies expand by introducing their current customers to new products and services.', \"At least one of Travaillier's new employees not only has experience in the bus tour industry but has also designed air travel vacation packages.\"]", "label": 2 }, { "id": "train_3758", "context": "While antibiotics have done inestimable good to humankind over the last seventy years, there are several drawbacks to using antibiotics that, until recently, have been overlooked. The human microbiome, which consists of the trillions of bacteria that reside in each person's body, is essential to good health. Specifically, the body contains and requires both \"good\" and \"bad\" bacteria. It is when the proper equilibrium between the \"good\" bacteria and the \"bad\" bacteria is disrupted that a number of health issues can emerge. Nonetheless, antibiotics indiscriminately kill both the \"good\" and the \"bad\" bacteria, so each course of antibiotics should be followed by a treatment that __.", "question": "Which of the following most logically completes the argument above?", "answers": "['completely reverses any effects from the antibiotics', 'restores the correct balance between the \"good\" and \"bad\" bacteria', 'targets and eliminates only the \"bad\" bacteria', 'bypasses the microbiome altogether']", "label": 1 }, { "id": "train_3759", "context": "The profitability of a business is reduced by anything that undermines employee morale. This is why paying senior staff with stock options, which allows them to earn more when the enterprise prospers, is not a wise policy because it increases dramatically the difference in income between senior staff and employees who are paid only a fixed salary.", "question": "Which one of the following is an assumption on which the argument depends?", "answers": "['Large income differences between fixed-salary employees and senior staff tend to undermine employee morale.', 'Business firms that pay senior staff with stock options are less profitable than other firms.', 'Employees whose incomes rise as the profits of their employers rise are more productive than those paid only a fixed salary.', 'Reductions in the profitability of a company are usually due to low employee morale.']", "label": 0 }, { "id": "train_3760", "context": "Rain-soaked soil contains less oxygen than does drier soil. The roots of melon plants perform less efficiently under the low-oxygen conditions present in rain-soaked soil. When the efficiency of melon roots is impaired, the roots do not supply sufficient amounts of the proper nutrients for the plants to perform photosynthesis at their usual levels. It follows that melon plants have a lower-than-usual rate of photosynthesis when their roots are in rain-soaked soil. When the photosynthesis of the plants slows, sugar stored in the fruits is drawn off to supply the plants with energy. Ttherefore, ripe melons harvested after a prolonged period of heavy rain should be less sweet than other ripe melons.", "question": "In the argument given, the two portions in boldface play which of the following roles?", "answers": "['The first provides support for the conclusion of the argument as a whole; the second provides evidence that supports an objection to that conclusion.', 'The first provides support for an intermediate conclusion that supports a further conclusion stated in the argument; the second states that intermediate conclusion.', 'The first serves as an intermediate conclusion that supports a further conclusion stated in the argument; the second states the position that the argument as a whole opposes.', 'The first states the position that the argument as a whole opposes; the second supports the conclusion of the argument.']", "label": 1 }, { "id": "train_3761", "context": "People who are allergic to cats are actually allergic to certain proteins found in the animals' skin secretions and saliva; which particular proteins are responsible, however, varies from allergy sufferer to allergy sufferer. Since all cats shed skin and spread saliva around their environment, there is no such thing as a cat incapable of provoking allergic reactions, although it is common for a given cat to cause an allergic reaction in some -- but not all -- people who are allergic to cats.", "question": "Which one of the following statements is most strongly supported by the information above?", "answers": "['Not all cats are identical with respect to the proteins contained in their skin secretions and saliva.', 'The allergic reactions of some people who are allergic to cats are more intense than the allergic reactions of other allergy sufferers.', 'Any particular individual will be allergic to some breeds of cat but not to others.', 'No cat is capable of causing an allergic reaction in all types of allergy sufferers.']", "label": 0 }, { "id": "train_3762", "context": "A society can achieve a fair distribution of resources only under conditions of economic growth. There can be no economic growth unless the society guarantees equality of economic opportunity to all of its citizens. Equality of economic opportunity cannot be guaranteed unless a society' s government actively works to bring it about.", "question": "If the statements given are true, it can be properly concluded from them that", "answers": "['no government can achieve a fair distribution of resources under conditions of economic growth', 'a society can achieve a fair distribution of resources only if its government actively works to bring about equality of economic opportunity', 'all societies that guarantee equality of economic opportunity to all of their members are societies that distribute resources fairly', 'some societies that experience economic growth fail to guarantee equality of opportunity to all of their citizens']", "label": 1 }, { "id": "train_3763", "context": "Essentially all polar ice forms from precipitation that falls as snow. Extremely cold air cannot hold much moisture and consequently cannot produce much snowfall. In recent years, air masses in both polar regions have been, without exception, extremely cold.", "question": "The information above most strongly supports which of the following conclusions?", "answers": "['If air temperatures in the polar regions were considerably warmer, much polar ice would melt off.', 'The thicker the polar ice is, the colder the air masses that are in contact with it.', 'For snow to turn into ice in the polar regions, the air has to be extremely cold.', 'If polar ice is currently growing and expanding at all, it is doing so only slowly.']", "label": 3 }, { "id": "train_3764", "context": "In the last decade there has been a significant decrease in coffee consumption. During this same time, there has been increasing publicity about the adverse long-term effects on health of the caffeine in coffee. Ttherefore, the decrease in coffee consumption must have been caused by consumers' awareness of the harmful effects of caffeine.", "question": "Which of the following, if true, most seriously calls into question the explanation above?", "answers": "['On average, people consume 30 percent less coffee today than they did 10 years ago.', 'The consumption of fruit juices and caffeine-free herbal teas has increased over the past decade.', 'Heavy coffee drinkers may have mild withdrawal symptoms, such as headaches, for a day or so after significantly decreasing their coffee consumption.', 'Coffee prices increased steadily in the past decade because of unusually severe frosts in coffee-growing nations.']", "label": 3 }, { "id": "train_3765", "context": "Waste management companies, which collect waste for disposal in landfills and incineration plants, report that disposable plastics make up an ever-increasing percentage of the waste they handle. It is clear that attempts to decrease the amount of plastic that people throw away in the garbage are failing.", "question": "Which one of the following, if true, most seriously weakens the argument?", "answers": "['Because plastics create harmful pollutants when burned, an increasing percentage of the plastics handled by waste management companies are being disposed of in landfills.', 'Although many plastics are recyclable, most of the plastics disposed of by waste management companies are not.', 'An increasing proportion of the paper, glass, and metal cans that waste management companies used to handle is now being recycled.', 'People are more likely to save and reuse plastic containers than containers made of heavier materials like glass or metal.']", "label": 2 }, { "id": "train_3766", "context": "A government agency publishes ratings of airlines, ranking highest the airlines that have the smallest proportion of late flights. The agency' s purpose is to establish an objective measure of the relative efficiency of different airlines' personnel in meeting published flight schedules.", "question": "Which one of the following, if true, would tend to invalidate use of the ratings for the agency's purpose?", "answers": "['Flights are defined as \"late\" only if they arrive more than fifteen minutes past their scheduled arrival time, and a record is made of how much later than fifteen minutes they are.', 'Flights are often made late by bad weather conditions that affect some airlines more than others.', 'Airline personnel are aware that the government agency is monitoring all airline flights for lateness.', 'Travelers sometimes have no choice of airlines for a given trip at a given time.']", "label": 1 }, { "id": "train_3767", "context": "Prolonged exposure to sulfur fumes permanently damages one' s sense of smell. In one important study, 100 workers from sulfur-emitting factories and a control group of 100 workers from other occupations were asked to identify a variety of chemically reproduced scents, including those of foods, spices, and flowers. On average, the factory workers successfully identified 10 percent of the scents compared to 50 percent for the control group.", "question": "Each of the following, if true, weakens the argument EXCEPT:", "answers": "['The chemicals used in the study closely but not perfectly reproduced the corresponding natural scents.', 'The subjects in the study were tested in the environments where they usually work.', 'Most members of the control group had participated in several earlier studies that involved the identification of scents.', 'Every sulfur-emitting factory with workers participating in the study also emits other noxious fumes.']", "label": 0 }, { "id": "train_3768", "context": "Downtown Villieu was once a flourishing business district, but most Villieu-area businesses are now located only in the suburbs. The office buildings downtown lack the modern amenities most business operators demand today. To lure more businesses downtown, Villieu officials plan to have several modern office buildings constructed and to offer reduced local tax rates to any business that leases space in a new downtown building.", "question": "Which of the following, if true, most threatens the plan's likelihood of success?", "answers": "['With the new office buildings, downtown Villieu would have a greater amount of modern office space than any other downtown business district in the region.', 'Most of the businesses currently located in downtown Villieu have long-term leases on the space they occupy there.', \"The existing office buildings in downtown Villieu have, on average, a much higher vacancy rate than do office buildings in Villieu's suburbs.\", \"The local tax rates in Villieu's suburbs are significantly lower than downtown Villieu's proposed rate for businesses that lease space in the new office buildings.\"]", "label": 3 }, { "id": "train_3769", "context": "Cotrell is, at best, able to write magazine articles of average quality. The most compelling pieces of evidence for this are those few of the numerous articles submitted by Cotrell that are superior, since Cotrell, who is incapable of writing an article that is better than average, must obviously have plagiarized superior ones.", "question": "The argument is most vulnerable to criticism on which one of the following grounds?", "answers": "['It generalizes from atypical occurrences.', 'It infers limits on ability from a few isolated lapses in performance.', 'It presupposes what it seeks to establish.', 'It simply ignores the existence of potential counterevidence.']", "label": 2 }, { "id": "train_3770", "context": "Each of two drugs, S and T, greatly reduces the effects of potentially fatal heart attacks if given as soon as possible after the attack begins, but a trial has shown that use of drug T instead of drug S would prevent death in one additional case out of 120. Drug T, however, costs $2, 000 more per treatment than drug S. Ttherefore society is presented with a stark policy decision: whether or not to pay the $240, 000 it would cost to use drug T in order to save one additional patient.", "question": "Which one of the following is an assumption on which the argument relies?", "answers": "['Drug S has certain side effects not shared by drug T.', 'After a heart attack, drug T remains relatively effective if given at a time at which drug S is no longer effective.', 'Drug T works significantly faster than drug S.', 'There is no quick, practical, and relatively inexpensive way of telling for any individual case whether drug S will be as effective as drug T.']", "label": 3 }, { "id": "train_3771", "context": "Editor: Articles in Gardening Magazine often spur sales of the plants they describe, particularly among people new to gardening. Accordingly, we will no longer publish articles or accept advertisements praising the beauty of rare wildflowers. Most such plants sold to gardeners have been difficult to propagate under cultivation, so plant sellers often collect them in the wild. Our new policy is part of our efforts to halt this yearly plundering of our native plant populations.", "question": "Which of the following, if true, casts the most doubt on the wisdom of the magazine's new policy as a way of pursuing the intended effect?", "answers": "['Revenues from sales of plants collected in the wild are supporting the discovery of new low-cost techniques enabling rare wildflowers to be readily propagated in nurseries.', 'When people new to gardening buy plants, they often fail to take adequate care of the plants that they buy and become discouraged from buying those varieties again.', 'The demand for rare wildflowers rarely exceeds the number of such plants that can be collected in the wild by plant sellers.', 'Plant sellers who sell rare wildflowers have no reasonably inexpensive alternative way to offer their wares directly to new gardeners.']", "label": 0 }, { "id": "train_3772", "context": "Marketing executive for Magu Corporation: Whenever Magu opens a manufacturing facility in a new city, the company should sponsor, or make donations to, a number of nonprofit organizations in that city. Doing so would improve Magu' s image in the community, and thus the money spent on such charitable ventures would lead to increased sales.", "question": "Which statement would, if true, point to the most serious weakness in the marketing executive's advice?", "answers": "['If market conditions change, Magu may have to close any such facility or relocate it.', 'Some nonprofit organizations are poorly organized, so money donated to them would be of little benefit to the community.', 'Spending on charitable ventures would require Magu to decrease direct advertisements, which are the most effective means of reaching its target customers.', 'Magu sells its products internationally, so sales in any one city represent only a small portion of total revenue.']", "label": 2 }, { "id": "train_3773", "context": "The symptoms of hepatitis A appear no earlier than 60 days after a person has been infected. In a test of a hepatitis A vaccine, 50 people received the vaccine and 50 people received a harmless placebo. Although some people from each group eventually exhibited symptoms of hepatitis A, the vaccine as used in the test is completely effective in preventing infection with the hepatitis A virus.", "question": "Which one of the following, if true, most helps resolve the apparent discrepancy in the information above?", "answers": "['The people who received the placebo were in better overall physical condition than were the people who received the vaccine.', 'Of the people who developed symptoms of hepatitis A, those who received the vaccine recovered more quickly, on average, than those who did not.', 'The vaccinated people who exhibited symptoms of hepatitis A were infected with the hepatitis A virus before being vaccinated.', 'The placebo did not produce any side effects that resembled any of the symptoms of hepatitis A.']", "label": 2 }, { "id": "train_3774", "context": "Analyst: The increasing complexity of computers may lead those who pursue a career in computer programming to think that job security and higher wages can be attained by becoming more specialized as the field becomes more complex. Even though specialists earn higher wages than generalists within computer programming, this move is ill-advised because one risks specializing in a technology that will become obsolete. Consider the plight of people who used to repair eight-track tape players.", "question": "Which one of the following, if true, most weakens the analyst's argument?", "answers": "['The technological complexity of computers may eventually grow less rapidly than at present.', 'The current technological knowledge of most specialists within computer programming could also be applied to many technologies that will replace present ones.', 'Many other careers will soon offer greater job security and higher wages than computer programming.', 'The average wages earned by generalists within computer programming will never be as high as the average wages earned by specialists within computer programming.']", "label": 1 }, { "id": "train_3775", "context": "A drug that is highly effective in treating many types of infection can, at present, be obtained only from the bark of the ibora, a tree that is quite rare in the wild. It takes the bark of 5, 000 trees to make one kilogram of the drug. It follows, ttherefore, that continued production of the drug must inevitably lead to the ibora's extinction.", "question": "Which of the following, if true, most seriously weakens the argument above?", "answers": "['The leaves of the ibora are used in a number of medical products.', 'The ibora generally grows in largely inaccessible places.', 'The ibora can be propagated from cuttings and grown under cultivation.', 'The drug made from ibora bark is expensive to produce.']", "label": 2 }, { "id": "train_3776", "context": "No small countries and no countries in the southern hemisphere have permanent seats on the United Nations Security Council. Each of the five countries with a permanent seat on the Security Council is in favor of increased international peacekeeping efforts and a greater role for the United Nations in moderating regional disputes. However, some countries that are in favor of increased international peacekeeping efforts are firmly against increased spending on refugees by the United Nations.", "question": "If the statements above are true, which one of the following must also be true?", "answers": "['Some small countries are in favor of a greater role for the United Nations in moderating regional disputes.', 'Some small countries do not want the United Nations to increase its spending on refugees.', 'Some countries that have permanent seats on the United Nations Security Council are against increased spending on refugees by the United Nations.', 'Some countries that are in favor of a greater role for the United Nations in moderating regional disputes are not located in the southern hemisphere.']", "label": 3 }, { "id": "train_3777", "context": "A medieval manuscript called L contains all eighteen extant tragedies by the Greek playwright Euripides. Of these, ten called the \"select plays, \" are accompanied in L by ancient commentaries and also appear in other medieval manuscripts; this group includes some of Euripides' best-known works, including the Medea. The other eight, which appear only in L, are called the \"alphabeticals\" because they appear in alphabetical order, without commentary. The Electra is one of the \"alphabeticals\".", "question": "Which of the following can be reliably concluded on the basis of the Statements given?", "answers": [ "Only Euripides' best-known works are accompanied by ancient commentaries in extant medieval manuscripts", "Euripides' Electra does not appear accompanied by a commentary in any extant medieval manuscript", "No commentaries were written about Euripides' Electra in ancient times", "Euripides' Medea never appears in medieval manuscripts unaccompanied by ancient commentary" ], "label": 1 }, { "id": "train_3778", "context": "Backyard gardeners who want to increase the yields of their potato plants should try growing stinging nettles alongside the plants, since stinging nettles attract insects that kill a wide array of insect pests that damage potato plants. It is true that stinging nettles also attract aphids, and that many species of aphids are harmful to potato plants, but that fact in no way contradicts this recommendation, because __.", "question": "Which one of the following most logically completes the argument?", "answers": "['the types of aphids that stinging nettles attract do not damage potato plants', 'most aphid species that are harmful to potato plants cause greater harm to other edible food plants', 'insect pests typically cause less damage to potato plants than other harmful organisms do', 'stinging nettles require little care and thus are easy to cultivate']", "label": 0 }, { "id": "train_3779", "context": "Leaders of a miners' union on strike against Coalco are contemplating additional measures to pressure the company to accept the union' s contract proposal. The union leaders are considering as their principal new tactic a consumer boycott against Gasco gas stations, which are owned by Energy Incorporated, the same corporation that owns Coalco.", "question": "The answer to which of the following questions is LEAST directly relevant to the union leaders' consideration of whether attempting a boycott of Gasco will lead to acceptance of their contract proposal?", "answers": "['Would revenue losses by Gasco seriously affect Energy Incorporated?', 'Have other unions that have employed a similar tactic achieved their goals with it?', 'Can current Gasco customers easily obtain gasoline elsewhere?', 'Do other corporations that own coal companies also own gas stations?']", "label": 3 }, { "id": "train_3780", "context": "Many famous painters employ preliminary sketches before embarking on the final version of their work. Yet frequently these preliminary sketches are beautiful and accomplished works of art in their own right. Museums with small budgets will display these preliminary works instead of what the artists consider to be their finished works of art.", "question": "Which one of the following propositions is best illustrated by the situation described above?", "answers": "[\"Artists' preliminary sketches are as beautiful as the final versions of their work.\", 'Artifacts may have uses different from those intended by their creators.', 'Artists are not the best judges of the value of their own work.', 'A finished work of art cannot be produced without the execution of a high-quality preliminary sketch.']", "label": 1 }, { "id": "train_3781", "context": "In the troposphere, the lowest level of the earth' s atmosphere, the temperature decreases as one progresses straight upward. At the top, the air temperature ranges from -50 degrees Celsius over the poles to -85 degrees Celsius over the equator. At that point the stratosphere begins, and the temperature stops decreasing and instead increases as one progresses straight upward through the stratosphere. The stratosphere is warmed by ozone. When an ozone particle absorbs a dose of ultraviolet sunlight, heat is generated.", "question": "If the statements above are true, which one of the following must also be true?", "answers": "['The troposphere over the poles is thicker than the troposphere over the equator.', 'The temperature at any point at the top of the stratosphere is at least as great as the temperature at the top of the troposphere directly beneath that point.', 'The temperature in the middle part of the stratosphere over the North Pole is at least as great as the temperature in the middle part of the stratosphere over the equator.', 'It is warmer at the top of the stratosphere over the poles than it is at the top of the stratosphere over the equator.']", "label": 1 }, { "id": "train_3782", "context": "Manufacturers of mechanical pencils make most of their profit on pencil leads rather than on the pencils themselves. The Write Company, which cannot sell its leads as cheaply as other manufacturers can, plans to alter the design of its mechanical pencil so that it will accept only a newly designed Write Company lead, which will be sold at the same price as the Write Company' s current lead.", "question": "Which of the following, if true, most strongly supports the Write Company's projection that its plan will lead to an increase in its sales of pencil lead?", "answers": "['A rival manufacture recently announced similar plans to introduce a mechanical pencil that would accept only the leads produced by that manufacturer.', 'First-time buyers of the mechanical pencils tend to buy the least expensive mechanical pencils available.', 'A Write Company executive is studying ways to reduce the cost of manufacturing pencil leads.', 'In extensive text marketing, mechanical-pencil users found the new Write Company pencil markedly superior to other mechanical pencils they had used.']", "label": 3 }, { "id": "train_3783", "context": "Unless tiger hunting decreases, tigers will soon be extinct in the wild. The countries in which the tigers' habitats are located are currently debating joint legislation that would ban tiger hunting. Thus, if these countries can successfully enforce this legislation, the survival of tigers in the wild will be ensured.", "question": "The reasoning in the argument is most vulnerable to criticism on the grounds that the argument", "answers": "['considers the effects of hunting on tigers without also considering the effects of hunting on other endangered animal species', 'neglects to consider the results of governmental attempts in the past to limit tiger hunting', \"takes the removal of an impediment to the tigers' survival as a guarantee of their survival\", 'assumes without sufficient warrant that a ban on tiger hunting could be successfully enforced']", "label": 2 }, { "id": "train_3784", "context": "Historian: Those who claim that Shakespeare did not write the plays commonly attributed to him are motivated purely by snobbery. Shakespeare was the son of a glove maker, whereas every other person proposed as the true author of the plays was an aristocrat, and many of those who argue that one or another of these aristocrats wrote the plays are the aristocrats' descendants.", "question": "The reasoning in the historian's argument is most vulnerable to criticism on the grounds that the argument", "answers": "['makes use of an assumption that one would accept only if one has already accepted the truth of the conclusion', 'takes for granted that anyone who is motivated purely by snobbery cannot also be motivated by legitimate historical evidence', \"fails to exclude the possibility that there might be legitimate evidence motivating those who reject Shakespeare's authorship\", 'presumes, without providing justification, that a claim cannot be true if those who advance it are motivated by snobbery']", "label": 2 }, { "id": "train_3785", "context": "Politician: The huge amounts of money earned by oil companies elicit the suspicion that the regulations designed to prevent collusion need to be tightened. But just the opposite is true. If the regulations designed to prevent collusion are not excessively burdensome, then oil companies will make profits sufficient to motivate the very risky investments associated with exploration that must be made if society is to have adequate oil supplies. But recent data show that the oil industry' s profits are not the highest among all industries. Clearly, the regulatory burden on oil companies has become excessive.", "question": "The reasoning in the politician's argument is most vulnerable to criticism on the grounds that the argument", "answers": "['fails to justify its presumption that profits sufficient to motivate very risky investments must be the highest among all industries', 'illicitly draws a general conclusion from a specific example that there is reason to think is atypical', 'fails to justify its presumption that two events that are correlated must also be causally related', 'attacks the character of the oil companies rather than the substance of their conduct']", "label": 0 }, { "id": "train_3786", "context": "Big-budget movies often gross two or three times the cost of their production and marketing. However, most of the movie industry' s total revenue comes from low-budget movies.", "question": "Which one of the following, if true, most helps to resolve the apparent discrepancy above?", "answers": "[\"The movie industry's revenues, when adjusted for inflation, have declined sharply in the last 30 years.\", 'Big-budget movies need to sell many more tickets than do low-budget movies, just to recoup their production costs.', 'There are many more low-budget movies produced than there are big- and mediumbudget movies.', 'Big-budget movies, because of their elaborate special effects, cost more in insurance premiums than low-budget movies do.']", "label": 2 }, { "id": "train_3787", "context": "Essayist: Some linguists claim that competent speakers of a language have explicit knowledge of the rules of grammar for that language. However, linguistic ability is not the possession and utilization of a body of knowledge, or rules of grammar, but is more similar to a skill like riding a bicycle. Just as the typical cyclist does not need to know physics, neither does the language user need to know grammar rules.", "question": "Which one of the following most accurately expresses a principle underlying the essayist's argument?", "answers": "['There is a difference between knowing a set of rules and behaving in accordance with a set of rules.', 'Not everyone follows the same set of rules in acquiring a skill.', 'Studying a description of a particular skill is of no help in acquiring that skill.', 'No set of rules can exhaustively describe the behavior of someone who is engaged in a complex activity.']", "label": 0 }, { "id": "train_3788", "context": "Based on data collected from policyholders, life insurance companies have developed tables that list standard weight ranges for various heights. Policyholders whose weight fell within the range given for their height lived longer than those whose weight fell outside their given range. Ttherefore, if people whose weight falls outside their given range modified their weight to fall within that range, their overall life expectancies would improve.", "question": "Which one of the following is an assumption on which the argument relies?", "answers": "['Holders of life insurance policies do not have longer overall life expectancies than the general population.', \"Some people would be unwilling to modify their weights solely to increase the general population's overall life expectancies.\", 'The tables include data gathered from policyholders whose deaths resulted from accidents in addition to those whose deaths resulted from natural causes.', \"People's efforts to modify their weight to conform to a given range would not damage their health enough to decrease their overall life expectancies.\"]", "label": 3 }, { "id": "train_3789", "context": "Yang: Yeast has long been known to be a leaven, that is, a substance used in baking to make breads rise. Since biblical evidence ties the use of leavens to events dating back to 1200 B. C. , we can infer that yeast was already known to be a leaven at that time. Campisi: I find your inference unconvincing; several leavens other than yeast could have been known in 1200 B. C.", "question": "Campisi counters Yang's argument by", "answers": [ "questioning the truth of a presumption underlying Yang's argument", "suggesting that an alternative set of evidence better supports Yang's conclusion", "calling into question the truth of the evidence presented in Yang's argument", "pointing out that the premises of Yang's argument more strongly support a contrary conclusion" ], "label": 0 }, { "id": "train_3790", "context": "Liang: Watching movies in which violence is portrayed as an appropriate way to resolve problems increases levels of aggression in viewers. Ttherefore, children' s access to these movies should be restricted. Sarah: Watching a drama whose characters are violent allows the audience to vicariously experience the emotions associated with aggression and thus be purged of them. Hence, the access by mature audiences to such forms of entertainment should not be restricted.", "question": "The dialogue provides the most support for inferring that Liang and Sarah agree with each other that", "answers": "['if violence in certain movies causes violence in viewers, access to those movies should be restricted', 'people who experience an emotion vicariously are likely to purge themselves of that emotion', 'the members of a mature audience are unlikely to believe that violence is sometimes an appropriate way to resolve problems', 'the effects of dramatic depictions of violence on audiences are at least partially understood']", "label": 3 }, { "id": "train_3791", "context": "Philosopher: It is absurd to argue that people are morally obligated to act in a certain way simply because not acting in that way would be unnatural. An unnatural action is either a violation of the laws of nature or a statistical anomaly. There is no possibility of acting as one cannot, nor does the mere fact that something is not usually done provide any good reason not to do it.", "question": "Which one of the following most accurately describes a technique used in the philosopher's argument?", "answers": "['stating the definition of a key term of the argument', 'using statistical findings to dispute a claim', 'using empirical evidence to support one definition of a key term of the argument over another', 'undermining a claim by showing that the claim is self-contradictory']", "label": 0 }, { "id": "train_3792", "context": "Most veterinarians, and especially those at university veterinary research centers, have a devoted interest in the biological sciences. But most veterinarians choose their profession primarily because they love animals. Among persons who are seriously interested in biological science but lack any special love for animals, one does not find any prominent veterinarians.", "question": "If all of the statements above are true, which one of the following CANNOT be true?", "answers": "['Few veterinarians at university research centers chose their profession primarily because they love animals.', 'Prominent veterinarians at some veterinary research centers are intensely devoted to the biological sciences but do not feel any pronounced affection for animals.', 'Most veterinarians who are not prominent regard an understanding of the biological sciences as the most important quality for success in their profession.', 'Some veterinarians have a greater love for biological science than for individual animals.']", "label": 1 }, { "id": "train_3793", "context": "In 1563, in Florence' s Palazzo Vecchio, Giorgio Vasari built in front of an existing wall a new wall on which he painted a mural. Investigators recently discovered a gap between Vasari' s wall and the original, large enough to have preserved anything painted on the original. Historians believe that Leonardo da Vinci had painted, but left unfinished, a mural on the original wall; some historians had also believed that by 1563 the mural had been destroyed. However, it is known that in the late 1560s, when renovating another building, Santa MariaNovella, Vasari built a facade over its frescoes, and the frescoes were thereby preserved. Thus, Leonardo' s Palazzo Vecchio mural probably still exists behind Vasari' s wall.", "question": "Which of the following is an assumption on which the argument depends?", "answers": "['When Vasari preserved the frescoes of Santa Maria Novella he did so secretly.', 'Vasari probably would not have built the Palazzo Vecchio wall with a gap behind it except to preserve something behind the new wall.', 'Leonardo would probably have completed the Palazzo Vecchio mural if he had had the opportunity to do so.', 'Leonardo rarely if ever destroyed artworks that he left unfinished.']", "label": 1 }, { "id": "train_3794", "context": "The public interest comprises many interests and the broadcast media must serve all of them. Perhaps most television viewers would prefer an action show to an opera. But a constant stream of action shows on all channels is not in the public interest. Thus, __.", "question": "Which one of the following most logically completes the argument?", "answers": "[\"broadcasters' obligations are not satisfied if they look only to popularity to decide their programming schedules\", 'the popularity of a television program is a poor indicator of its artistic quality', 'broadcast media could be rightly accused of neglecting the public interest only if all channels carried mostly action shows', 'television networks should broadcast more artistic and cultural shows and fewer action shows']", "label": 0 }, { "id": "train_3795", "context": "Radon-210 is more radioactive than Technetium-99. Since Promethium-61 is more radioactive than Protactinium-234, it follows that Radon-210 is more radioactive than Protactinium-234.", "question": "Any of the following, if introduced into the argument as an additional premise, makes the argument above logically correct EXCEPT:", "answers": "['Promethium-61 and Radon-210 have equal levels of radioactivity.', 'Promethium-61 and Technetium-99 have equal levels of radioactivity.', 'Technetium-99 is more radioactive than Protactinium-234.', 'Promethium-61 is more radioactive than Radon-210.']", "label": 3 }, { "id": "train_3796", "context": "Advertisement: A leading economist has determined that among people who used computers at their place of employment last year, those who also owned portable (\"laptop\") computers earned 25 percent more on average than those who did not. It is obvious from this that owning a laptop computer led to a higher-paying job.", "question": "Which one of the following identifies a reasoning error in the argument?", "answers": "['It offers information as support for a conclusion when that information actually shows that the conclusion is false.', 'Its conclusion merely restates a claim made earlier in the argument.', \"It concludes that one thing was caused by another although the evidence given is consistent with the first thing's having caused the second.\", 'It attempts to support a sweeping generalization on the basis of information about only a small number of individuals.']", "label": 2 }, { "id": "train_3797", "context": "Dyson: The proposal paper to add solar panels to the new outdoor university seating in order to charge electronic devices includes a section at the end encouraging student participation by logging opinions online in relation to the proposal. This is a vital step in the proposal' s content. Myers: The proposal to add solar panels to the new outdoor seating does not necessarily need student participation at the end. Our audience is not the students, but the administration committee that approves space and funding for the project. We should instead appeal directly to the administrators to act in the name of progress and technology.", "question": "Which one of the following principles, if valid, most justifies Myers stance?", "answers": "[\"Appealing to students will in turn appeal to the proposal's committee. Students are the lifeblood of the university and it's critical that the committee determine whether or not students are on board.\", \"Proposals should appeal to the project's recipients, or those who the project will directly affect. This way, the project will gain support from its followers.\", 'Proposal audience should not be called upon to respond to an ethical or moral obligation; this is considered inappropriate and unprofessional in the art of proposal writing.', 'Proposals should appeal to the decision makers of the proposal, or those who will be reading the proposal in order to make a decision in favor or in opposition to the proposed project.']", "label": 3 }, { "id": "train_3798", "context": "Antarctic seals dive to great depths and stay submerged for hours. They do not rely solely on oxygen held in their lungs, but also store extra oxygen in their blood. Indeed, some researchers hypothesize that for long dives these seals also store oxygenated blood in their spleens.", "question": "Each of the following, if true, provides some support for the researchers' hypothesis EXCEPT:", "answers": "['The spleens of Antarctic seals contain greater concentrations of blood vessels than are contained in most of their other organs.', \"The oxygen contained in the seals' lungs and bloodstream alone would be inadequate to support the seals during their dives.\", 'The spleen is much larger in the Antarctic seal than in aquatic mammals that do not make long dives.', 'Many species of seal can store oxygen directly in their muscle tissue.']", "label": 3 }, { "id": "train_3799", "context": "The writers of the television show Ambitions could make their characters more realistic than they currently are, but they know their viewership would shrink if they did. The writers will choose to maximize their audience, so the characters will not be developed in a more realistic manner.", "question": "Which one of the following arguments is most similar in its reasoning to the argument above?", "answers": [ "If a company's failure is due to a broader economic collapse, then it is not fair to blame the company's executives for the failure. There was a broader economic collapse when ViqCo went bankrupt. So it is probably not fair to blame ViqCo's executives for the failure.", "Since ViqCo's failure was due to a broader economic collapse, it is not fair to blame the company's executives for the failure. But that means that when ViqCo was succeeding because the broader economy was growing, the executives did not deserve the credit.", "If ViqCo's executives were responsible for the company's losses, then ViqCo's losses would have been greater than those of its competitors. But ViqCo's losses were less than those of its competitors. So ViqCo's executives were not responsible for the company's losses.", "If ViqCo's executives were responsible for the company's failure, then it must be possible to say what they should have done differently. Ttherefore, if you cannot say what ViqCo's executives should have done differently, then you should not blame them for the failure." ], "label": 2 }, { "id": "train_3800", "context": "The soaring prices of scholarly and scientific journals have forced academic libraries used only by academic researchers to drastically reduce their list of subscriptions. Some have suggested that in each academic discipline subscription decisions should be determined solely by a journal' s usefulness in that discipline, measured by the frequency with which it is cited in published writings by researchers in the discipline.", "question": "Which one of the following, if true, most seriously calls into question the suggestion described above?", "answers": "['The nonacademic readership of a scholarly or scientific journal can be accurately gauged by the number of times articles appearing in it are cited in daily newspapers and popular magazines.', 'Researchers often will not cite a journal article that has influenced their work if they think that the journal in which it appears is not highly regarded by the leading researchers in the mainstream of the discipline.', 'The increasingly expensive scholarly journals are less and less likely to be available to the general public from nonacademic public libraries.', 'In some academic disciplines, controversies which begin in the pages of one journal spill over into articles in other journals that are widely read by researchers in the discipline.']", "label": 1 }, { "id": "train_3801", "context": "Botanist: It has long been believed that people with children or pets should keep poinsettia plants out of their homes. Although this belief has been encouraged by child-rearing books, which commonly list poinsettias as poisonous and ttherefore dangerous, it is mistaken. Our research has shown, conclusively, that poinsettias pose no risk to children or pets.", "question": "Which one of the following most accurately expresses the conclusion drawn in the botanist's argument?", "answers": "['According to many child-rearing books, poinsettias are dangerous.', 'Poinsettias are not dangerously poisonous.', 'The belief that households with children or pets should not have poinsettias is mistaken.', 'Poinsettias pose no risk to children or pets.']", "label": 2 }, { "id": "train_3802", "context": "Legislator: My staff conducted a poll in which my constituents were asked whether they favor high taxes. More than 97 percent answered \"no. \" Clearly, then, my constituents would support the bill I recently introduced, which reduces the corporate income tax.", "question": "The reasoning in the legislator's argument is most vulnerable to criticism on the grounds that the argument", "answers": "[\"confuses an absence of evidence that the legislator's constituents oppose a bill with the existence of evidence that the legislator's constituents support that bill\", 'treats a result that proves that the public supports a bill as a result that is merely consistent with public support for that bill', \"fails to establish that the opinions of the legislator's constituents are representative of the opinions of the country's population as a whole\", \"fails to consider whether the legislator's constituents consider the current corporate income tax a high tax\"]", "label": 3 }, { "id": "train_3803", "context": "Most bicycle helmets provide good protection for the top and back of the head, but little or no protection for the temple regions on the sides of the head. A study of head injuries resulting from bicycle accidents showed that a large proportion were caused by blows to the temple area. Ttherefore, if bicycle helmets protected this area, the risk of serious head injury in bicycle accidents would be greatly reduced especially since __.", "question": "Which of the following most logically completes the passage?", "answers": "['even those bicyclists who regularly wear helmets have a poor understanding of the degree and kind of protection that helmets afford', \"among the bicyclists included in the study's sample of head injuries, only a very small proportion had been wearing a helmet at the time of their accident\", 'the bone in the temple area is relatively thin and impacts in that area are thus very likely to cause brain injury', 'bicyclists generally land on their arm or shoulder when they fall to the side, which reduces the likelihood of severe impacts on the side of the head']", "label": 2 }, { "id": "train_3804", "context": "People who do not believe that others distrust them are confident in their own abilities, so people who tend to trust others think of a difficult task as a challenge rather than a threat, since this is precisely how people who are confident in their own abilities regard such tasks.", "question": "The conclusion above follows logically if which one of the following is assumed?", "answers": "[\"Confidence in one's own abilities gives one confidence in the trustworthiness of others.\", 'People who are not threatened by difficult tasks tend to find such tasks challenging.', 'People who tend to trust others do not believe that others distrust them.', 'People tend to distrust those who they believe lack self-confidence.']", "label": 2 }, { "id": "train_3805", "context": "Graphologists claim that it is possible to detect permanent character traits by examining people' s handwriting. For example, a strong cross on the \"t\"is supposed to denote enthusiasm. Obviously, however, with practice and perseverance people can alter their handwriting to include this feature. So it seems that graphologists must hold that permanent character traits can be changed.", "question": "The argument against graphology proceeds by", "answers": "['citing apparently incontestable evidence that leads to absurd consequences when conjoined with the view in question', 'arguing that a particular technique of analysis can never be effective when the people analyzed know that it is being used', 'demonstrating that an apparently controversial and interesting claim is really just a platitude', 'showing that proponents of the view have no theoretical justification for the view']", "label": 0 }, { "id": "train_3806", "context": "Rumored declines in automobile-industry revenues are exaggerated. It is true that automobile manufacturers' share of the industry' s revenues fell from 65 percent two years ago to 50 percent today, but over the same period suppliers of automobile parts had their share increase from 15 percent to 20 percent and service companies (for example, distributors, dealers, and repairers) had their share increase from 20 percent to 30 percent.", "question": "Which one of the following best indicates why the statistics given above provide by themselves no evidence for the conclusion they are intended to support?", "answers": "['No explanation is given for why the revenue shares of different sectors of the industry changed.', \"No matter what changes the automobile industry's overall revenues undergo, the total of all shares of these revenues must be 100 percent.\", \"The possibility is left open that the statistics for manufacturers' share of revenues come from a different source than the other statistics.\", \"Manufacturers and parts companies depend for their revenue on dealers' success in selling cars.\"]", "label": 1 }, { "id": "train_3807", "context": "Nutritionists are advising people to eat more fish, since the omega-3 fatty acids in fish help combat many diseases. If everyone took this advice, however, there would not be enough fish in oceans, rivers, and lakes to supply the demand; the oceans are already being overfished. The obvious method to ease the pressure on wild fish populations is for people to increase their consumption of farmed fish.", "question": "Which of the following, if true, raises the most serious doubt concerning the prospects for success of the solution proposed above?", "answers": "['The feed for farmed ocean fish is largely made from small wild-caught fish, including the young of many popular food species.', 'There are large expanses of ocean waters that do not contain enough nutrients to support substantial fish populations.', 'Some of the species that are now farmed extensively were not commonly eaten when they were only available in the wild.', 'Some fish farming, particularly of shrimp and other shellfish, takes place in enclosures in the Ocean.']", "label": 0 }, { "id": "train_3808", "context": "Trancorp currently transports all its goods to Burland Island by truck. The only bridge over the channel separating Burland from the mainland is congested, and trucks typically spend hours in traffic. Trains can reach the channel more quickly than trucks, and freight cars can be transported to Burland by barges that typically cross the channel in an hour. Ttherefore, to reduce shipping time, Trancorp plans to switch to trains and barges to transport goods to Burland.", "question": "Which of the following would be most important to know in determining whether Trancorp's plan, if implemented, is likely to achieve its goal?", "answers": "['Whether loading the freight cars onto barges is very time consuming', 'Whether the average number of vehicles traveling over the bridge into Burland has been relatively constant in recent years', 'Whether most trucks transporting goods into Burland return to the mainland empty', 'Whether there are boats that can make the trip between the mainland and Burland faster than barges can']", "label": 0 }, { "id": "train_3809", "context": "Politician: The funding for the new nationwide health-awareness campaign should come from an increase in taxes on cigarettes. It is well established that cigarette smoking causes many serious health problems, and it is only reasonable that people whose unhealthful habits cause so many health problems should bear the costs of that campaign. Smoker: But it is equally well established that regularly eating high-fat, high-cholesterol foods causes as many serious health problems as does smoking, yet is would be manifestly unreasonable to force those who purchase such foods to bear the burden of financing this campaign.", "question": "Which one of the following is the point at issue between the politician and the smoker?", "answers": "['whether it is unreasonable to require people who do not benefit from certain governmental programs to share the costs of those programs', \"whether the politician's proposal for financing the health-awareness campaign is an unreasonable one\", 'whether the proposed increase on cigarette taxes is an efficient means of financing the health-awareness campaign', 'whether smokers are more aware of the harmful effects of their habit than are people who regularly eat high-fat, high-cholesterol foods']", "label": 1 }, { "id": "train_3810", "context": "Adult female rats who have never before encountered rat pups will start to show maternal behaviors after being confined with a pup for about seven days. This period can be considerably shortened by disabling the female' s sense of smell or by removing the scent-producing glands of the pup.", "question": "Which of the following hypotheses best explains the contrast described above?", "answers": "['Female rats that have given birth are more affected by olfactory cues than are female rats that have never given birth.', 'A female rat that has given birth shows maternal behavior toward rat pups that she did not bear more quickly than does a female rat that has never given birth.', \"The development of a female rat's maternal interest in a rat pup that she did not bear is inhibited by the odor of the pup.\", 'The amount of scent produced by rat pups increases when they are in the presence of a female rat that did not bear them.']", "label": 2 }, { "id": "train_3811", "context": "No computer will ever be able to do everything that some human minds can do, for there are some problems that cannot be solved by following any set of mechanically applicable rules. Yet computers can only solve problems by following some set of mechanically applicable rules.", "question": "Which one of the following is an assumption on which the argument depends?", "answers": "['Every problem that is solvable by following more than one set of mechanically applicable rules is solvable by almost every human mind.', 'At least one problem not solvable by following any set of mechanically applicable rules is solvable]e by at least one human mind.', 'At least one problem solvable by following some set of mechanically applicable rules is not solvable by any human mind.', 'Every problem that is solvable by following at least one set of mechanically applicable rules is solvable by at least one human mind.']", "label": 1 }, { "id": "train_3812", "context": "Numerous studies have demonstrated a pronounced negative correlation between high-fiber diets and the incidence of colon cancer. For example, the colon cancer rate in Western countries is much higher than in many non-Western countries where people eat more fiber-rich foods, such as fruits and vegetables. Furthermore, in Scandinavia it has been conclusively shown that the higher the colon cancer rate in a given area, the lower the consumption in that area of cereals, which, like fruits and vegetables, are high in fiber. All of this shows that insufficient consumption of fiber causes colon cancer, and sufficient consumption of fiber prevents it.", "question": "The argument's reasoning is vulnerable to criticism because the argument overlooks the possibility that", "answers": "['foods containing fiber also contain other substances that, when consumed, tend to prevent colon cancer', 'the fiber in fruits and vegetables and the fiber in cereals have cancer-fighting properties to different degrees', 'the consumption of fiber in many countries is rising appreciably', 'the risk of many types of cancer is reduced by high-fiber diets']", "label": 0 }, { "id": "train_3813", "context": "At mock trials in which jury instructions were given in technical legal jargon, jury verdicts tended to mirror the judge' s own opinions. Jurors had become aware of the judge' s nonverbal behavior: facial expressions, body movements, tone of voice. Jurors who viewed the same case but were given instruction in clear, nontechnical language, however, were comparatively more likely to return verdicts at odds with the judge' s opinion.", "question": "Which one of the following is best illustrated by the example described above?", "answers": "['Technical language tends to be more precise than nontechnical language.', \"The way in which a judge instructs a jury can influence the jury's verdict.\", \"A person's influence is proportional to that person's perceived status.\", 'Real trials are better suited for experimentation than are mock trials.']", "label": 1 }, { "id": "train_3814", "context": "Generic medicines contain the same active ingredients, in the same quantity, as the original brand-name medicines for which they are designed to be substituted. Nonetheless, there are sometimes important differences in the effects produced in a patient by generic medicines as compared to the brand-name equivalents.", "question": "Which of the following, if true, most helps to resolve the seeming discrepancy described above?", "answers": "['Because producers of generic medicines are not involved in research and development their products can be sold at lower prices.', 'The inactive ingredients and fillers, which differ in different versions of a medicine, can affect the rate at which the medicine is absorbed and its concentration in the bloodstream.', 'When the patent on an original brand-name medicine expires, federal law permits the production of a generic version without further studies on the effectiveness of the active ingredients.', 'Older persons, who are more likely to require prescription medicines, are more sensitive than younger persons are to slight variations in dosage levels.']", "label": 1 }, { "id": "train_3815", "context": "A certain city has had many vacant homes in the downtown neighborhoods. The city loses significant tax revenue each year because those houses stand unoccupied. The city felt that the fear of crime was keeping professionals from moving into these houses. Over 80% of the city' s professional work force live outside of the city, in nearby suburbs, and commute daily. Plan: Five years ago, the city tripled the size of the municipal police force, increasing patrols throughout the city. All studies have confirmed that crime has dropped substantially. Result: the vacancy rate in these single-family houses has not changed substantially in five years. Further information: Almost all of the vacant houses are in excellent condition, ready for occupancy. The city supports a number of high-paying professional jobs, and the state economy has been booming over the past five years.", "question": "In light of the further information, which of the following, if true, does most to explain the result that followed the implementation of the plan?", "answers": "['Homes in this city, close to downtown, cost on average 40% more than a home of comparable size in the surrounding suburbs.', 'In a recent survey, almost three-quarters of the professionals who live in the suburbs and work in the city say that they would prefer to live in the city.', 'Recent studies confirm that the crime-rate in the city is no higher than the average crime-rate in the surrounding suburbs.', 'The commuting costs in this metropolitan region, in dollar per mile, are among the highest in the entire country.']", "label": 0 }, { "id": "train_3816", "context": "Walter: For the economically privileged in a society to tolerate an injustice perpetrated against one of society' s disadvantaged is not just morally wrong but also shortsighted: a system that inflicts an injustice on a disadvantaged person today can equally well inflict that same injustice on a well-to-do person tomorrow. Larissa: In our society, the wealthy as well as the well-educated can protect themselves against all sorts of injustices suffered by the less well-off. Allowing such injustices to persist is bad policy not because it places everyone at equal risk of injustice but because it is a potent source of social unrest.", "question": "Larissa responds to Walter by doing which one of the following?", "answers": "['charging Walter with stopping short of recognizing the full implications of his position', \"questioning Walter's authority to address matters of social policy\", \"drawing implausible consequences from Walter's assumptions\", \"providing an alternative reason for accepting the truth of Walter's conclusion\"]", "label": 3 }, { "id": "train_3817", "context": "At one time, many astronomers assumed that Earth remains motionless while the stars revolve around it. They concluded from this that the stars were not more than a few million miles from Earth. They reasoned that if the stars were farther away, they would have to move at tremendously great speeds in order to circle Earth during the day and reappear in roughly the same positions each night.", "question": "Which one of the following is an assumption required by the reasoning described above?", "answers": "['All stars move at exactly the same speed when they are revolving around Earth.', 'Earth does not remain motionless while the stars revolve around it.', 'Stars do not move at tremendously great speeds.', 'If the stars do not revolve around Earth, it is possible for at least some stars to be more than a few million miles from Earth.']", "label": 2 }, { "id": "train_3818", "context": "A bacterial species will inevitably develop greater resistance within a few years to any antibiotics used against it, unless those antibiotics eliminate that species completely. However, no single antibiotic now on the market is powerful enough to eliminate bacterial species X completely.", "question": "Which one of the following is most strongly supported by the statements above?", "answers": "['Bacterial species X is more resistant to at least some antibiotics that have been used against it than it was before those antibiotics were used against it.', 'Bacterial species X will inevitably become more virulent in the course of time.', 'If any antibiotic now on the market is used against bacterial species X, that species will develop greater resistance to it within a few years.', 'It is unlikely that any antibiotic can be developed that will completely eliminate bacterial species X.']", "label": 2 }, { "id": "train_3819", "context": "A contract between two parties is valid only if one party accepts a legitimate offer from the other; an offer is not legitimate if someone in the position of the party to whom it was made would reasonably believe the offer to be made in jest.", "question": "The principle stated above, if valid, most helps to justify the reasoning in which one of the following arguments?", "answers": "[\"Kenta accepted Gus's offer to buy a shipment of goods, but Gus, unknown to Kenta, made the offer in jest. Thus, the contract was not valid.\", \"Frank's offer to buy Mindy's business from her was legitimate. Thus, if Mindy is a reasonable person, she will accept the offer.\", 'The only offer that Sal made to Veronica was not a legitimate one. Thus, regardless of whether Sal made the offer in jest, there is no valid contract between them.', \"Joe made a legitimate offer to buy Sandy's car and Sandy has not rejected the offer. Thus, there was a valid contract.\"]", "label": 2 }, { "id": "train_3820", "context": "Educational Theorist: Recent editorials have called for limits on the amount of homework assigned to children. They point out that free-time activities play an important role in childhood development and that large amounts of homework reduce children' s free time, hindering their development. But the average homework time for a ten year old, for example, is little more than 30 minutes per night. Clearly, ttherefore, there is no need to impose the limits these editorials are calling for.", "question": "Which of the following is an assumption on which the educational theorist's argument relies?", "answers": "['In most schools, if not all, the homework assignments given are of a length that does not diverge widely from the average.', 'The free-time activities that ten year olds engage in most are all approximately equally effective at fostering development', 'Individual teachers are not the best judges of how much homework to assign the children they teach', 'Free-time activities rarely teach children skills or information that they can use in their academic work.']", "label": 0 }, { "id": "train_3821", "context": "When astronomers observed the comet Schwassman - Wachmann 3 becoming 1, 000 times brighter in September 1995, they correctly hypothesized that its increased brightness was a result of the comet' s breaking up -- when comets break up, they emit large amounts of gas and dust, becoming visibly brighter as a result. However, their observations did not reveal comet Schwassman-Wachmann 3 actually breaking into pieces until November 1995, even though telescopes were trained on it throughout the entire period.", "question": "Which one of the following, if true, most helps to resolve the apparent conflict in the statements above?", "answers": "['The amount of gas and dust emitted steadily increased during the period from September through November.', 'Gas and dust can be released by cracks in a comet even if the comet is not broken all the way through.', 'Comets often do not emit gas and dust until several weeks after they have begun to break up.', 'The reason comets become brighter when they break up is that the gas and dust that they emit refract light.']", "label": 1 }, { "id": "train_3822", "context": "Scientists have created double-blind studies so that neither the subjects of the experiment nor scientists know whether the subject is receiving an actual drug or a placebo, which is nothing more than a sugar pill. Essentially, if one knows whether one is receiving actual medicine or a placebo, such knowledge can affect the outcome of a study. A recent double-blind study on the effectiveness of the Selective Serotonin Reuptake Inhibitor (SSRI) Freloxamine on depression found that those subjects administered the drug were 15% more likely to have a decrease in symptoms than the control group, which was comprised of those who received a placebo. Since neither group knew which they were receiving, the placebo or the SSRI, the observed drop in depression can only be attributed to Freloxamine.", "question": "Which of the following, if true, best calls into question the conclusion of the argument?", "answers": "['Neither the subjects nor the doctors in either group (the control group or the Freloxamine group) knew which drug they were receiving.', 'At dosages two-fold of those employed in the trial, Freloxamine has been shown to cause brief episodes of psychosis.', 'Since subjects in both groups were debriefed on the potential side effects of SSRI, which can often be pronounced, many in the Freloxamine group, upon exhibiting side effects, concluded that they were being administered the SSRI.', 'Freloxamine does not exhibit a uniform effect in all subjects, with many reporting little improvement in symptoms of depression, even after several months of taking the drug.']", "label": 2 }, { "id": "train_3823", "context": "A new medication for migraine seems effective, but there is concern that the medication might exacerbate heart disease. If patients with heart disease take the medication under careful medical supervision, however, harmful side effects can definitely be averted. The concern about those side effects is thus unfounded.", "question": "The argument depends on which one of the following assumptions?", "answers": "['The new medication has various other side effects, but none as serious as that of exacerbating heart disease.', 'No migraine sufferers with heart disease will take the new medication except under careful medical supervision.', 'The new medication actually is effective when taken by patients with heart disease.', 'The new medication will displace all migraine medications currently being used.']", "label": 1 }, { "id": "train_3824", "context": "Editorial: The government claims that the country' s nuclear power plants are entirely safe and hence that the public' s fear of nuclear accidents at these plants is groundless. The government also contends that its recent action to limit the nuclear industry' s financial liability in the case of nuclear accidents at power plants is justified by the need to protect the nuclear industry from the threat of bankruptcy. But even the government says that unlimited liability poses such a threat only if injury claims can be sustained against the industry; and the government admits that for such claims to be sustained, injury must result from a nuclear accident. The public' s fear, ttherefore, is well founded.", "question": "If all of the statements offered in support of the editorial's conclusion correctly describe the government's position, which one of the following must also be true on the basis of those statements?", "answers": [ "The government's claim about the safety of the country's nuclear power plants is false.", "Unlimited financial liability in the case of nuclear accidents poses no threat to the financial security of the country's nuclear industry.", "The government's position on nuclear power plants is inconsistent.", "The government misrepresented its reasons for acting to limit the nuclear industry's liability." ], "label": 2 }, { "id": "train_3825", "context": "Editorial: The premier' s economic advisor assures her that with the elimination of wasteful spending the goal of reducing taxes while not significantly decreasing government services can be met. But the premier should not listen to this advisor, who in his youth was convicted of embezzlement. Surely his economic advice is as untrustworthy as he is himself, and so the premier should discard any hope of reducing taxes without a significant decrease in government services.", "question": "Which one of the following is a questionable argumentative strategy employed in the editorial's argument?", "answers": "['rejecting a proposal on the grounds that a particular implementation of the proposal is likely to fail', \"trying to win support for a proposal by playing on people's fears of what could happen otherwise\", 'criticizing the source of a claim rather than examining the claim itself', 'presupposing what it sets out to establish']", "label": 2 }, { "id": "train_3826", "context": "Principle: Employees of telemarketing agencies should never do anything that predisposes people to dislike the agencies' clients. Application: If an employee of a telemarketing agency has been told by a person the employee has called that he or she does not want to buy the product of a client of the agency, the employee should not try to talk that person into doing so.", "question": "Which one of the following, if true, justifies the given application of the principle above?", "answers": "['Some people that an employee of a telemarketing agency calls to ask them to buy the product of a client of the agency will refuse to do so even though they are not predisposed to dislike the client.', 'Any employee of a telemarketing agency who tries to get someone to buy the product of a client of the agency after the person has said that he or she does not want to will engender animosity toward the client.', 'Some employees of telemarketing agencies are unlikely to be certain about whether trying to talk someone into buying the product of a client of the agency after the person has said that he or she does not want to will likely engender animosity toward the client.', 'People who are already predisposed to dislike the client of a telemarketing agency are more likely to refuse to buy the product of that client than are people who are predisposed to like the client.']", "label": 1 }, { "id": "train_3827", "context": "No occupation should be subject to a licensing requirement unless incompetence in the performance of tasks normally carried out within that occupation poses a plausible threat to human health or safety.", "question": "The principle stated above, if valid, most helps to justify the reasoning in which one of the following arguments?", "answers": "['Because some of the duties that police officers carry out have no connection to human health or safety, police officers should not be subject to a licensing requirement.', 'Because there are no realistic circumstances in which poor work by an interior designer poses a danger to human beings, interior designers should not be subject to a licensing requirement.', 'Because hospital administrators routinely make decisions that affect the health of hundreds of people, hospital administrators should be subject to a licensing requirement.', 'Because hair stylists regularly use substances that can pose a threat to human health if handled improperly, hair stylists should be subject to a licensing requirement.']", "label": 1 }, { "id": "train_3828", "context": "By competing with rodents for seeds, black ants help control rodent populations that pose a public health risk. However, a very aggressive species of black ant, the Loma ant, which has recently invaded a certain region, has a venomous sting that is often fatal to humans. Ttherefore, the planned introduction into that region of ant flies, which prey on Loma ants, would benefit public health.", "question": "Which of the following, if true, most strengthens the argument?", "answers": "['Certain other species of black ants are more effective than Loma ants in competing with rodents for seeds.', 'The sting of Loma ants can also be fatal to rodents.', 'Ant flies do not attack black ants other than Loma ants.', 'The use of pesticides to control Loma ants could have harmful effects on the environment.']", "label": 2 }, { "id": "train_3829", "context": "Once consumers recognize that a period of inflation has begun, there is generally an increase in consumer spending. This increase can be readily explained by consumers' desire not to postpone purchases that will surely increase in price. But during protracted periods of inflation, consumers eventually begin to put off making even routine purchases, despite the fact that consumers continue to expect prices to rise and despite the fact that salaries also rise during inflationary periods.", "question": "Which one of the following, if true, most helps to explain the apparent inconsistency in consumer behavior described above?", "answers": "[\"Consumers' purchasing power decreases during periods of protracted inflation since salaries do not keep pace with prices.\", \"There is usually a lag between the leading economic indicators' first signaling the onset of an inflationary period and consumers' recognition of its onset.\", 'If significant numbers of consumers are unable to make purchases, prices will eventually fall but salaries will not be directly affected.', 'No generalization that describes human behavior will be true of every type of human behavior.']", "label": 0 }, { "id": "train_3830", "context": "Owners of deeply indebted and chronically unprofitable small businesses sometimes try to convince others to invest money in their companies. Since the money thus acquired will inevitably be used to pay off debts, rather than to expand operations, this money will not stimulate sales growth in such companies. Thus, most people are reluctant to make these investments. Surprisingly, however, such investments often earn handsome returns in the very first year they are made.", "question": "Which one of the following, if true, most helps to explain the surprising results of suchs investments?", "answers": "['Expanding production in such companies would usually require more funds than would paying off debts.', 'Banks are reluctant to lend money to any company that is already heavily in debt and chronically unprofitable.', \"Paying off debts, by saving a company the money it would otherwise owe in interest, decreases the company's overall expenses and thereby increases its profits.\", 'Investors usually choose to reinvest their returns on such investments.']", "label": 2 }, { "id": "train_3831", "context": "The media now devote more coverage to crime than they did ten years ago. Yet this is not because the crime rate has increased, but rather because the public is now more interested in reading and hearing about crime. After all, a crucial factor in the media' s decisions about what issues to cover and to what extent to cover them is the interests of their audiences.", "question": "The proposition that the public is now more interested in reading and hearing about crime plays which one of the following roles in the argument?", "answers": "['It is a generalization based on the claim that the crime rate has increased over the past ten years.', 'It is presented as evidence that the media decide what to cover and to what extent to cover it depending on the interests of the public.', 'It is offered as an alternative explanation of why the media devote more coverage to crime now than they did ten years ago.', 'It supports the conclusion that the media now devote more coverage to crime than the crime rate alone justifies.']", "label": 2 }, { "id": "train_3832", "context": "To cut costs, a high school modified its airconditioning system to increase its efficiency. The modified system, however, caused the humidity in the school air to decrease by 18 percent. Twenty-four hours after the decrease in air humidity, a 25 percent increase in the number of visits to the school nurse was reported. This shows that a decrease in humidity can make people ill.", "question": "The argument depends on assuming which one of the following?", "answers": "['Most of the students at the high school suffered from the decrease in air humidity.', 'Modifying the air-conditioning system proved to be an ineffective way to cut costs.', 'It takes 24 hours after a person is infected with a virus for that person to exhibit symptoms.', 'At least some of the visits to the school nurse after the system was modified were due to illness.']", "label": 3 }, { "id": "train_3833", "context": "Some members have criticized the club' s president for inviting Dr. Hines to speak at the annual awards banquet without consulting other club members beforehand. But a few years ago the previous club president hired a tax accountant even though he had not discussed it with club members beforehand. So the current president acted appropriately in the way in which she invited Dr. Hines.", "question": "Which one of the following is an assumption on which the argument relies?", "answers": "[\"The club's previous president acted appropriately in hiring the tax accountant without first consulting other club members.\", 'At the time the previous club president hired the tax accountant, most club members did not expect to be consulted about such matters.', 'The previous club president had also invited speakers without first consulting other club members.', \"Dr. Hines accepted the president's invitation to speak at the club's annual awards banquet.\"]", "label": 0 }, { "id": "train_3834", "context": "State spokesperson: Many businesspeople who have not been to our state believe that we have an inadequate road system. Those people are mistaken, as is obvious from the fact that in each of the past six years, our state has spent more money per mile on road improvements than any other state.", "question": "Which of the following, if true, most seriously undermines the reasoning in the spokesperson's argument?", "answers": "[\"Over the past six years, numerous businesses have left the spokesperson's state, but about as many businesses have moved into the state.\", \"In general, the number of miles of road in a state's road system depends on both the area and the population of the state.\", 'Only states with seriously inadequate road systems need to spend large amounts of money on road improvements.', \"Adequacy of a state's road system is generally less important to a businessperson considering doing business there than is the availability of qualified employees.\"]", "label": 2 }, { "id": "train_3835", "context": "Although the school would receive financial benefits if it had soft drink vending machines in the cafeteria, we should not allow them. Allowing soft drink machines there would not be in our students' interest. If our students start drinking more soft drinks, they will be less healthy.", "question": "The argument depends on which of the following?", "answers": "['If the soft drink vending machines were placed in the cafeteria, students would consume more soft drinks as a result.', \"The school's primary concern should be to promote good health among its students.\", 'The amount of soft drinks that most students at the school currently drink is not detrimental to their health.', 'Students will not simply bring soft drinks from home if the soft drink vending machines are not placed in the cafeteria.']", "label": 0 }, { "id": "train_3836", "context": "Economist: The wages of many of the lowest-paid corporate employees in this country would be protected from cuts by enacting a maximum wage law that prohibits executives at any corporation from earning more than, say, 50 times what the corporation' s lowest-paid employees in this country earn. Currently, some executives try to increase corporate profits -- and their own salaries -- by cutting the pay and benefits of their corporations' employees. A maximum wage law would remove this incentive for these executives to cut the wages of their lowest-paid employees.", "question": "Which one of the following is an assumption the economist's argument requires?", "answers": [ "All of the lowest-paid corporate employees in the economist's country are employed at corporations at which the executives earn more than 50 times what the corporations' lowest-paid employees in the economist's country earn.", "If such a maximum wage law were enacted in the economist's country, one or more corporate executives would not cut the pay and benefits of their corporations' lowest-paid employees in the economist's country.", "If corporate executives could not increase their own salaries by cutting the pay and benefits of their corporations' lowest-paid employees in the economist's country, they would never change the wages of those employees.", "No corporate executives in the economist's country would raise the wages of their corporations' lowest-paid employees in the economist's country unless such a maximum wage law linked executive wages to those of their corporations' lowest-paid employees in the economist's country." ], "label": 1 }, { "id": "train_3837", "context": "Nutritionist: Recently a craze has developed for home juicers, $300 machines that separate the pulp of fruits and vegetables from the juice they contain. Outrageous claims are being made about the benefits of these devices: drinking the juice they produce is said to help one lose weight or acquire a clear complexion, to aid digestion, and even to prevent cancer. But there is no indication that juice separated from the pulp of the fruit or vegetable has any properties that it does not have when unseparated. Save your money. If you want carrot juice, eat a carrot.", "question": "Which one of the following, if true, most calls into question the nutritionist's argument?", "answers": "['Drinking juice from home juicers is less healthy than is eating fruits and vegetables because such juice does not contain the fiber that is eaten if one consumes the entire fruit or vegetable.', 'Most people find it much easier to consume a given quantity of nutrients in liquid form than to eat solid foods containing the same quantity of the same nutrients.', 'To most people who would be tempted to buy a home juicer, $300 would not be a major expense.', 'The nutritionist was a member of a panel that extensively evaluated early prototypes of home juicers.']", "label": 1 }, { "id": "train_3838", "context": "Two alternative drugs are available to prevent blood clots from developing after a heart attack. According to two major studies, drug Y does this no more effectively than the more expensive drug Z, but drug Z is either no more or only slightly more effective than drug Y. Drug Z' s manufacturer, which has engaged in questionable marketing practices such as offering stock options to doctors who participate in clinical trials of drug Z, does not contest the results of the studies but claims that they do not reveal drug Z' s advantages. However, since drug Z does not clearly treat the problem more effectively than drug Y, there is no established medical reason for doctors to use drug Z rather than drug Y on their heart-attack victims.", "question": "A major flaw in the argument is that the argument", "answers": "['fails to recognize that there may be medical criteria relevant to the choice between the two drugs other than their effectiveness as a treatment', \"neglects to compare the marketing practices of drug Y's manufacturer with those of drug Z's manufacturer\", 'confuses economic reasons for selecting a treatment with medical reasons', 'does not consider drugs or treatments other than drug Y and drug Z that may be used to prevent blood clotting in heart-attack patients']", "label": 0 }, { "id": "train_3839", "context": "Commentator: Unfortunately, Roehmer' s opinion column has a polarizing effect on national politics. She has always taken a partisan stance, and lately she has taken the further step of impugning the motives of her adversaries. That style of argumentation is guaranteed not to change the minds of people with opposing viewpoints; it only alienates them. But that is likely not a problem for Roehmer, since her column is just an attempt to please her loyal readers.", "question": "The reasoning in the commentator's argument is most vulnerable to criticism on the grounds that the argument", "answers": "['concludes that one event caused another merely because that event occurred immediately prior to the other', 'criticizes a column merely by invoking the personal characteristics of its author', 'employs a tactic at one point that it elsewhere objects to', 'fails to rule out the possibility that a purported cause of a phenomenon is actually an effect of that phenomenon']", "label": 2 }, { "id": "train_3840", "context": "A good way to get over one' s fear of an activity one finds terrifying is to do it repeatedly. For instance, over half of people who have parachuted only once report being extremely frightened by the experience, while less than 1 percent of those who have parachuted ten times or more report being frightened by it.", "question": "The reasoning in the argument is most vulnerable to criticism on the grounds that the argument", "answers": "['neglects to consider those people who have parachuted more than once but fewer than ten times', 'takes for granted that the greater the number of dangerous activities one engages in the less one is frightened by any one of them', 'takes for granted that people do not know how frightening something is unless they have tried it', 'overlooks the possibility that most people who have parachuted many times did not find it frightening initially']", "label": 3 }, { "id": "train_3841", "context": "Grammarians have for years condemned as ungrammatical the English phrase \"between you and I\", insisting that the correct phrasing is \"between you and me, \" with the objective case after a preposition. Such condemnations, however, are obviously unfounded, because Shakespeare himself, in The Merchant of Venice, wrote, \"All debts are cleared between you and I. ", "question": "Which of the following, if true, most seriously weakens the argument above?", "answers": "['The more modern an English word or phrase, the less likely that modern grammarians will consider it acceptable for formal usage.', 'In his plays, Shakespeare intentionally had some of his characters use phrases he considered ungrammatical.', 'The phrase \"between you and I\" appears infrequently in Shakespeare\\'s writings.', 'Most native speakers of English who choose to say \"between you and I\" do so because they know that Shakespeare used that phrase.']", "label": 1 }, { "id": "train_3842", "context": "Party X has recently been accused by its opposition, Party Y, of accepting international campaign contributions, which is illegal. Such accusations are, however, ill founded. Three years ago, Party Y itself was involved in a scandal in which it was discovered that its national committee seriously violated campaign laws.", "question": "Which one of the following contains flawed reasoning most similar to the flawed reasoning in the argument above?", "answers": "[\"The plaintiff accuses the defendant of violating campaign laws, but the accusations are ill founded. While the defendant's actions may violate certain laws, they are not immoral, because the laws in question are unjust.\", \"The plaintiff accuses the defendant of voting only for campaign laws that would favor the defendant's party. This accusation is ill founded, however, because it attacks the defendant's motivations instead of addressing the arguments the defendant has put forth justifying these votes.\", 'The plaintiff accuses the defendant of violating campaign laws, but these accusations show the plaintiff to be hypocritical, because the plaintiff has engaged in similar conduct.', \"The plaintiff accuses the defendant of violating campaign laws, and, in the past, courts have declared such violations illegal. Nevertheless, because the plaintiff recently engaged in actions that were similar to those of the defendant, the plaintiff's accusations are ill founded.\"]", "label": 3 }, { "id": "train_3843", "context": "The waters surrounding Shooter' s Island have long been a dumping ground for ruined ships and boats, and the wreckage there has caused these waters to be exceptionally still. An ornithologist found that the overall abundance of waterbirds around Shooter' s Island is similar to that around each of the neighboring islands, but that juvenile waterbirds are much more abundant around Shooter' s Island than around those other islands. This suggests that the still waters around Shooter' s Island serve as a nursery for the juveniles.", "question": "Which one of the following, if true, provides the most support for the argument's conclusion?", "answers": "['Waterbirds are typically much more abundant in areas that serve as nurseries for juvenile waterbirds than in areas that do not.', 'Waterbirds use still waters as nurseries for juveniles whenever possible.', \"The waters around the islands neighboring Shooter's Island are much rougher than the waters around Shooter's Island.\", \"The number of juvenile waterbirds around Shooter's Island, as well as the number around each neighboring island, does not fluctuate dramatically throughout the year.\"]", "label": 1 }, { "id": "train_3844", "context": "At a certain period in Earth' s history, its atmosphere contained almost no oxygen, although plants were producing vast quantities of oxygen. As a way of reconciling these two facts, scientists have hypothesized that nearly all of the oxygen being produced was taken up by iron on Earth' s surface. Clearly, however, this explanation is inadequate. New studies show that the amount of iron on Earth' s surface was not sufficient to absorb anywhere near as much oxygen as was being produced. Ttherefore, something in addition to the iron on Earth' s surface must have absorbed much of the oxygen produced by plant life.", "question": "In the argument given, the two portions in boldface play which of the following roles?", "answers": "['The first is a judgment made by the argument about a certain explanation; the second is that explanation.', \"The first sums up the argument's position with regard to a certain hypothesis; the second provides grounds for that position.\", 'The first is a concession by the argument that its initial formulation of the position it seeks to establish requires modification; the second presents that position in a modified form.', \"The first expresses the argument's dismissal of an objection to the position it seeks to establish; the second is that position.\"]", "label": 1 }, { "id": "train_3845", "context": "The tulu, a popular ornamental plant, does not reproduce naturally, and is only bred and sold by specialized horticultural companies. Unfortunately, the tulu is easily devastated by a contagious fungal rot. The government ministry plans to reassure worried gardeners by requiring all tulu plants to be tested for fungal rot before being sold. However, infected plants less than 30 weeks old have generally not built up enough fungal rot in their systems to be detected reliably. And many tulu plants are sold before they are 24 weeks old.", "question": "Which of the following, if performed by the ministry, could logically be expected to overcome the problem with their plan to test for the fungal rot?", "answers": "['Releasing a general announcement that tulu plants less than 30 weeks old cannot be effectively tested for fungal rot', 'Ensuring that tulu plants not be sold before they are 30 weeks old', 'Researching possible ways to test tulu plants less than 24 weeks old for fungal rot', 'Quarantining all tulu plants from horticultural companies at which any case of fungal rot has been detected until those tulu plants can be tested for fungal rot']", "label": 1 }, { "id": "train_3846", "context": "Scientific research at a certain university was supported in part by an annual grant from a major foundation. When the university' s physics department embarked on weapons-related research, the foundation, which has a purely humanitarian mission, threatened to cancel its grant. The university then promised that none of the foundation' s money would be used for the weapons research, whereupon the foundation withdrew its threat, concluding that the weapons research would not benefit from the foundation' s grant.", "question": "Which one of the following describes a flaw in the reasoning underlying the foundation's conclusion?", "answers": "['It assumes that if the means to achieve an objective are humanitarian in character, then the objective is also humanitarian in character.', 'It overlooks the possibility that the university made its promise specifically in order to induce the foundation to withdraw its threat.', \"It overlooks the possibility that the physics department's weapons research is not the only one of the university's research activities with other than purely humanitarian purposes.\", \"It overlooks the possibility that the availability of the foundation's money for humanitarian uses will allow the university to redirect other funds from humanitarian uses to weapons research.\"]", "label": 3 }, { "id": "train_3847", "context": "The reforms to improve the quality of public education that have been initiated on the part of suppliers of public education have been insufficient. Ttherefore, reforms must be demanded by consumers. Parents should be given government vouchers with which to pay for their children' s education and should be allowed to choose the schools at which the vouchers will be spent. To attract students, academically underachieving schools will be forced to improve their academic offerings.", "question": "The argument assumes that", "answers": "['there is a single best way to educate students', 'in selecting schools parents would tend to prefer a reasonable level of academic quality to greater sports opportunities or more convenient location', \"children are able to recognize which schools are better and would influence their parents' decisions\", 'schools would each improve all of their academic offerings and would not tend to specialize in one particular field to the exclusion of others']", "label": 1 }, { "id": "train_3848", "context": "Statistics teachers often debate whether to teach the mathematical theories underlying statistics or to focus on the use of statistical methods to solve real problems. This is like the difference between learning to build a car and learning to drive one. Almost all statistics courses teach \"building, \" and if there is extra time, they may cover a tip or two on \"driving. \" But, just as ordinary drivers do not need to know how to build a car in order to drive one, __", "question": "Which one of the following most logically completes the argument?", "answers": "['students should not be presumed by their teachers to have a particular goal or purpose in taking a class', 'users of statistics do not need to understand the underlying theory in order to solve real problems', 'statistics should be considered as a discipline that is distinct from mathematics', 'statistics teachers should focus on providing their students with the skills that they are most likely to use']", "label": 1 }, { "id": "train_3849", "context": "Psychologist: The best way to recall a certain word or name that one is having trouble remembering is to occupy one' s mind with other things, since often the more we strive to remember a certain word or name that we can' t think of, the less likely it becomes that the word will come to mind.", "question": "The principle that underlies the psychologist's argument underlies which one of the following arguments?", "answers": "[\"The best way to overcome a serious mistake is to continue on confidently as though all is well. After all, one can overcome a serious mistake by succeeding in new challenges, and dwelling on one's errors usually distracts one's mind from new challenges.\", 'Often, the best way to achieve happiness is to pursue other things besides wealth and fame, for there are wealthy and famous people who are not particularly happy, which suggests that true happiness does not consist in wealth and fame.', \"The best way to cope with sorrow or grief is to turn one's attention to those who are experiencing even greater hardship, for in many circumstances this will make our own troubles seem bearable by comparison.\", 'The best way to fall asleep quickly is to engage in some mental diversion like counting sheep, because frequently the more one concentrates on falling asleep the lower the chance of falling asleep quickly.']", "label": 3 }, { "id": "train_3850", "context": "There are many structural and thematic similarities between Piers Plowman by Langland (1330-1400) and House of Fame by Chaucer (1342-1400), two Middle English poems relating dream visions. Some critics have argued that because a number of the shared elements are uncommon in Middle English poetry, and because Langland' s poem probably predates Chaucer' s by a few years, Chaucer was most likely influenced by Piers Plowman when writing House of Fame.", "question": "Which of the following, if true, most seriously weakens the critics' argument?", "answers": "[\"Piers Plowman is one of Langland's major works, whereas House of Fame is a minor work of Chaucer's\", 'Many of the themes and structures of Piers Plowman are also found in Latin, Italian, French works with which Chaucer could well have been familiar.', 'House of Fame survives in only three manuscript copies, substantially fewer than the number of manuscript copies that exist of Piers Plowman.', 'Because Piers Plowman became a well-known work in its day, it is likely that the similarities between it and House of Fame were detected by many of the people who read House of Fame soon after Chaucer wrote it.']", "label": 1 }, { "id": "train_3851", "context": "That Shakespeare' s The Winter s Tale ( 1610-1611) is modeled after Euripides' Alcestis (fifth century B. C. ) seems undeniable. However, it is generally accepted that Shakespeare knew little or no Greek, so Euripides' original play would be an unlikely source. Thus, it seems most likely that Shakespeare came to know Euripides' play through a Latin translation.", "question": "Which one of the following, if true, LEAST strengthens the argument?", "answers": "['There is strong evidence to suggest that Shakespeare relied on Latin translations of Greek plays as sources for some of his other works.', \"The only English language version of Alcestis available in Shakespeare's time differed drastically from the original in ways The Winter s Tale does not.\", \"Latin phrases that were widely used in England during Shakespeare's time appear in a number of his plays.\", \"Paul Buchanan's 1539 Latin translation of Alcestis was faithful to the original and widely available during the 1600s.\"]", "label": 2 }, { "id": "train_3852", "context": "A poor farmer was fond of telling his children: \"In this world, you are either rich or poor, and you are either honest or dishonest. All poor farmers are honest. Ttherefore, all rich farmers are dishonest. ", "question": "The farmer's conclusion is properly drawn if the argument assumes that", "answers": "['everyone who is poor is honest', 'every honest farmer is poor', 'everyone who is dishonest is a rich farmer', 'every honest person is a farmer']", "label": 1 }, { "id": "train_3853", "context": "One way kidney stones can form is when urine produced in the kidneys is overly concentrated with calcium or oxalate. Reducing dietary calcium has been thought, ttherefore, to decrease the likelihood that calcium will concentrate and form additional stones. Oddly enough, for many people the chances of recurrence are decreased by increasing calcium intake.", "question": "Which one of the following, if true, most helps to resolve the apparent discrepancy described above?", "answers": "['Increasing dietary oxalate while reducing dietary calcium does not reduce the chances of kidney stone recurrence.', 'Kidney stone development is sometimes the result of an inherited disorder that can result in excessive production of calcium and oxalate.', 'Some kidney stones are composed of uric acid rather than a combination of calcium and oxalate.', 'Increasing calcium intake increases the amount of calcium eliminated through the intestines, which decreases the amount to be filtered by the kidneys.']", "label": 3 }, { "id": "train_3854", "context": "Many homeowners regularly add commercial fertilizers to their lawns and gardens to maintain a healthy balance of nutrients in soil. The widely available commercial fertilizers contain only macronutrients -- namely, nitrogen, phosphorus, and potassium. To remain healthy in the long term, soil for lawns requires the presence of these macronutrients and also trace amounts of micronutrients such as zinc, iron, and copper, which are depleted when grass clippings are raked up rather than allowed to decay and return to the soil.", "question": "Which one of the following can be properly inferred from the statements above?", "answers": "['Widely available commercial fertilizers are not alone sufficient to maintain a healthy balance of nutrients in soil for lawns where grass clippings are not allowed to decay and return to the soil.', 'Homeowners who rake up their grass clippings are unable to maintain the long-term health of the soil in their lawns and gardens.', \"There is no single fertilizer that provides both the macronutrients and micronutrients necessary for maintaining soil's long-term health.\", 'For soil to remain healthy in the long term, it requires the regular addition of both commercial fertilizers and a source of micronutrients such as grass clippings that are allowed to decay and return to the soil.']", "label": 0 }, { "id": "train_3855", "context": "Astronomer: In most cases in which a planet has been detected orbiting a distant star, the planet' s orbit is distinctly oval, whereas the orbits of Earth and several other planets around our sun are approximately circular. However, many comets orbiting our sun have been thrown into oval orbits by close encounters with planets orbiting our sun. So some of the planets in oval orbits around distant stars were probably thrown into those orbits by close encounters with other planets orbiting the same stars.", "question": "Which one of the following, if true, would most strengthen the astronomer's argument?", "answers": "['There is no indication that the orbit of any planet orbiting our sun has been affected by a close encounter with another planet orbiting our sun.', 'In most cases in which planets have been discovered orbiting a distant star, more than one planet has been found orbiting the star.', 'When two planets or other large objects in orbit have a close encounter, usually the smaller of the two is the more greatly affected.', \"For each distant star that has been found to have a planet, no other object large enough to affect the planet's orbit has been found orbiting the star.\"]", "label": 1 }, { "id": "train_3856", "context": "Editorial: The structure of the present school calendar was established to satisfy the requirements of early-twentieth-century agricultural life. In those days, farmers needed their children to have long breaks during which they could remain at home and help with the harvest. The contemporary school year is thus made up of periods of study interspersed with long breaks. But agricultural life no longer occupies most of our citizens, so we can now make changes that serve the interests of children. Ttherefore, long breaks should be removed from the school calendar.", "question": "Which one of the following is an assumption on which the editorial's argument depends?", "answers": "['During long breaks children have a tendency to forget what they have learned.', 'A change in the present school calendar that shortened breaks would serve the interests of agricultural life.', 'A school calendar made up of periods of study without long breaks would serve the interests of children more than a school calendar with long breaks.', \"Long breaks in the school calendar should be replaced with breaks that are no longer than workers' average vacations.\"]", "label": 2 }, { "id": "train_3857", "context": "Researchers asked volunteers to imagine they were running a five-kilometer race against 50 people and then against 500 people, races in each of which the top 10 percent would receive a $1, 000 prize. Asked about the effort they would apply in the respective cases, the volunteers indicated, on average, that they would run slower in the race against the greater number of people. A likely explanation of this result is that those of the volunteers who were most comparatively inclined-those who most tended to compare themselves with others in the social environment-determined (perhaps unconsciously) that extreme effort would not be worthwhile in the 500-competitor race.", "question": "Which of the following would, if known to be true, most help justify the explanation offered above?", "answers": "['The volunteers who were the least comparatively inclined had no greater desire to win the $1, 000 than those who were the most comparatively inclined.', 'The most comparatively inclined volunteers believed that they were significantly less likely to finish in the top 10 percent in the race against 500 than in the race against 50.', 'The volunteers who were most comparatively inclined were also those that had the greatest desire to win a $1, 000 prize.', 'The volunteers who were most comparatively inclined were likely to indicate that they would run the two races at the same speed.']", "label": 1 }, { "id": "train_3858", "context": "A contract, whether expressed or unexpressed, exists when two parties engage with each other for the reciprocal transfer of benefits. Thus, in accepting support from public funds, an artist creates at least an unexpressed contract between himself or herself and the public, and the public can rightly expect to benefit from the artist' s work.", "question": "Which one of the following most accurately describes an error in reasoning in the passage?", "answers": "['treating an issue that requires resolution through political processes as if it were merely a matter of opinion', 'speaking only in abstract terms about matters that involve contingencies and that must be judged on a case-by-case basis', 'concluding that a definition is fully applicable to a situation when it is known only that the situation conforms partially to that definition', 'attempting to justify a rule of conduct on the grounds that it confers benefits on all of the parties involved']", "label": 2 }, { "id": "train_3859", "context": "Researcher: The use of the newest drug in treating this disease should be discontinued. The treatment usually wreaks havoc with the normal functioning of the human body, causing severe side effects such as total loss of hair, debilitating nausea, and intense pain in the joints.", "question": "The argument's reasoning is flawed because the argument", "answers": "['does not acknowledge that the effects of the treatment may not be of the same severity in all cases', 'fails to consider the consequences of not administering the treatment', 'does not consider the length of time needed for the treatment to begin taking effect', 'fails to specify what is meant by \"normal functioning of the human body\"']", "label": 1 }, { "id": "train_3860", "context": "Researcher: A number of studies have suggested that, on average, clients in short-term psychotherapy show similar levels of improvement regardless of the kind of psychotherapy they receive. So any client improvement in short-term psychotherapy must be the result of some aspect or aspects of therapy that are common to all psychotherapies -- for example, the presence of someone who listens and gives attention to the client.", "question": "Which one of the following, if true, would most weaken the researcher's argument?", "answers": "['The studies found that psychotherapy by a trained counselor does not result in any greater improvement, on average, among clients than does simple counseling by an untrained layperson.', 'On average, clients improve more dramatically when they receive long-term psychotherapy, a year or longer in duration, than when clients receive short-term psychotherapy.', 'More-experienced therapists tend to use a wider range of techniques and interventions in psychotherapy than do inexperienced therapists.', 'The methods by which the studies measured whether clients improved primarily concerned immediate symptom relief and failed to address other important kinds of improvement.']", "label": 3 }, { "id": "train_3861", "context": "Some anthropologists study modern-day societies of foragers in an effort to learn about our ancient ancestors who were also foragers. A flaw in this strategy is that forager societies are extremely varied. Indeed, any forager society with which anthropologists are familiar has had considerable contact with modern, non-forager societies.", "question": "Which of the following, if true, would most weaken the criticism made above of the anthropologists' strategy?", "answers": "['All anthropologists study one kind or another of modern-day society.', 'Most ancient forager societies either dissolved or made a transition to another way of life.', 'Even those modern-day forager societies that have not had significant contact with modern societies are importantly different from ancient forager societies.', 'All forager societies throughout history have had a number of important features in common that are absent from other types of societies.']", "label": 3 }, { "id": "train_3862", "context": "Although high cholesterol levels have been associated with the development of heart disease, many people with high cholesterol never develop heart disease, while many without high cholesterol do. Recently, above average concentrations of the blood particle lipoprotein(a) were found in the blood of many people whose heart disease was not attributable to other causes. Dietary changes that affect cholesterol levels have no effect on lipoprotein(a) levels. Hence, there is no reason for anyone to make dietary changes for the sake of preventing heart disease.", "question": "Which one of the following most accurately describes a flaw in the argument?", "answers": "['It presents but ignores evidence that, for some people, high cholesterol contributes to heart disease.', 'It provides no evidence for a link between lipoprotein(a) and heart disease.', 'It fails to consider the possibility that lipoprotein(a) raises cholesterol levels.', 'It fails to consider the possibility that poor diets cause some people to develop health problems other than heart disease.']", "label": 0 }, { "id": "train_3863", "context": "Jones: Prehistoric wooden tools found in South America have been dated to 13, 000 years ago. Although scientists attribute these tools to peoples whose ancestors first crossed into the Americas from Siberia to Alaska, this cannot be correct. In order to have reached a site so far south, these peoples must have been migrating southward well before 13, 000 years ago. However, no such tools dating to before 13, 000 years ago have been found anywhere between Alaska and South America. Smith: Your evidence is inconclusive. Those tools were found in peat bogs, which are rare in the Americas. Wooden tools in soils other than peat bogs usually decompose within only a few years.", "question": "The point at issue between Jones and Smith is", "answers": "['whether the dating of the wooden tools by the scientists could be correct', \"whether the scientists' attribution of tools could be correct in light of Jones's evidence\", 'whether all prehistoric tools that are 13, 000 years or older were made of wood', 'how long ago the peoples who crossed into the Americas from Siberia to Alaska first did so']", "label": 1 }, { "id": "train_3864", "context": "Surrealist: Many artists mistakenly think that models need be taken only from outside the psyche. Although human sensibility can confer beauty upon even the most vulgar external objects, using the power of artistic representation solely to preserve and reinforce objects that would exist even without artists is an ironic waste.", "question": "Which one of the following most accurately expresses the conclusion of the surrealist's argument?", "answers": "['Artistic representation is used solely to preserve and reinforce objects.', \"An artist's work should not merely represent objects from outside the psyche.\", 'Artists should not base all their work on mere representation.', 'Great art can confer beauty even upon very vulgar external objects.']", "label": 1 }, { "id": "train_3865", "context": "Homeowners aged 40 to 50 are more likely to purchase ice cream and are more likely to purchase it in larger amounts than are members of any other demographic group. The popular belief that teenagers eat more ice cream than adults must, ttherefore, be false.", "question": "The argument is flawed primarily because the author", "answers": "['does not supply information about homeowners in age groups other than 40 to 50', 'does not specify the precise amount of ice cream purchased by any demographic group', 'fails to distinguish between purchasing and consuming', 'discusses ice cream rather than more nutritious and healthful foods']", "label": 2 }, { "id": "train_3866", "context": "Economist: In free market systems, the primary responsibility of corporate executives is to determine a nation' s industrial technology, the pattern of work organization, location of industry, and resource allocation. They also are the decision makers, though subject to significant consumer control, on what is to be produced and in what quantities. In short, a large category of major decisions is turned over to business executives. Thus, business executives have become public officials.", "question": "Which one of the following, if true, most weakens the economist's argument?", "answers": "['What a country produces and in what quantities is not always completely controlled by corporate executives.', \"Making decisions about patterns of work organization, resource allocation, and location of industry is not the core of a public official's job.\", 'Public officials and business executives often cooperate in making decisions of national importance.', 'The salaries of business executives are commensurate with the salaries of high-ranking public officials.']", "label": 1 }, { "id": "train_3867", "context": "New types of washing machines designed to consume less energy also extract less water from laundry during their final spin cycles than do washing machines that consume somewhat more energy. The wetter the laundry, the more energy required to dry it in an automatic dryer. Thus using these new types of washing machines could result in an overall increase in the energy needed to wash and dry a load of laundry.", "question": "In which one of the following is the pattern of reasoning most parallel to that in the argument above?", "answers": "['The more weights added to an exercise machine, the greater the muscle strength needed to work out on the machine. Up to a point, using more muscle strength can make a person stronger. Thus an exercise machine with more weights can, but does not necessarily, make a person stronger.', 'Pine is a less expensive wood than cedar but is more susceptible to rot. Outdoor furniture made from wood susceptible to rot must be painted with more expensive paint. Ttherefore, building outdoor furniture from pine rather than cedar could increase the total cost of building and painting the furniture.', 'The more skill required to operate a machine, the harder it is to find people able to do it, and thus the more those people must be paid. Ttherefore, if a factory installs machines that require highly skilled operators, it must be prepared to pay higher wages.', \"The more people who work in the library's reading room, the noisier the room becomes; and the noisier the working environment, the less efficiently people work. Ttherefore, when many people are working in the reading room, those people are working less efficiently.\"]", "label": 1 }, { "id": "train_3868", "context": "For Juanita to get to the zoo she must take either the number 12 bus or else the subway. Everyone knows that the number 12 bus is not running this week; so although Juanita generally avoids using the subway, she must have used it today, since she was seen at the zoo this afternoon.", "question": "The method of the argument is to", "answers": "['show that something is the case by ruling out the only alternative', 'substitute a claim about what invariably occurs for a claim about what typically occurs', 'demonstrate that certain possibilities are not exclusive', 'assert that if something is true, it will be known to be true']", "label": 0 }, { "id": "train_3869", "context": "Regular weightlifting is necessary for good health. Weightlifting with heavy resistance, especially with compound movements, helps break down and rebuild stronger muscle fibers, resulting in strength and size gains.", "question": "Which one of the following is an assumption required by the argument?", "answers": "['Compound movements are the only way to increase strength and size.', 'Regular weightlifting is necessary for good health.', 'Performing compound movements is the only way to break down and rebuild stronger muscle fibers.', 'Strength and size gains are indicators of good health.']", "label": 3 }, { "id": "train_3870", "context": "Automobile ownership was rare in Sabresia as recently as 30 years ago, but with continuing growth of personal income there, automobile ownership has become steadily more common. Consequently, there are now far more automobiles on Sabresia' s roads than there were 30 years ago, and the annual number of automobile accidents has increased significantly. Yet the annual number of deaths and injuries resulting from automobile accidents has not increased significantly.", "question": "Which of the following, if true, most helps to explain why deaths and injuries resulting from automobile accidents have not increased significantly?", "answers": "['Because of a vigorous market in used cars, the average age of cars on the road has actually increased throughout the years of steady growth in automobile ownership.', \"The increases in traffic volume have been most dramatic on Sabresia's highways, where speeds are well above those of other roads.\", 'Automobile ownership is still much less common in Sabresia than it is in other countries.', 'With more and more people owning cars, the average number of passengers in a car on the road has dropped dramatically.']", "label": 3 }, { "id": "train_3871", "context": "Psychologist: Psychotherapists who attempt to provide psychotherapy on radio or television talk shows are expected to do so in ways that entertain a broad audience. However, satisfying this demand is nearly always incompatible with providing high-quality psychological help. For this reason, psychotherapists should never provide psychotherapy on talk shows.", "question": "Which one of the following principles must be assumed in order for the psychologist's conclusion to be properly drawn?", "answers": "['Psychotherapists should never attempt to provide psychological help in a manner that makes it unlikely to be of high quality.', 'Most members of radio and television talk show audiences are seeking entertainment rather than high-quality psychological help.', 'Psychotherapy should never be provided in a context in which there is any chance that the therapy might be of less than high quality.', 'It is never appropriate for psychotherapists to attempt to entertain a broad audience.']", "label": 0 }, { "id": "train_3872", "context": "The sustained massive use of pesticides in farming has two effects that are especially pernicious. First, it often kills off the pests' natural enemies in the area. Second, it often unintentionally gives rise to insecticide-resistant pests, since those insects that survive a particular insecticide will be the ones most resistant to it, and they are the ones left to breed.", "question": "From the passage above, it can be properly inferred that the effectiveness of the sustained massive use of pesticides can be extended by doing which of the following, assuming that each is a realistic possibility?", "answers": "['Periodically switching the type of insecticide used', 'Leaving a few fields fallow every year', 'Breeding higher-yielding varieties of crop plants', 'Gradually increasing the quantities of pesticides used']", "label": 0 }, { "id": "train_3873", "context": "There are just two ways a moon could have been formed from the planet around which it travels: either part of the planet' s outer shell spun off into orbit around the planet or else a large object such as a comet or meteoroid, struck the planet so violently that it dislodged a mass of material from inside the planet. Earth' s moon consists primarily of materials different from those of the Earth' s outer shell.", "question": "If the statements above are true, which one of the following, if also true, would most help to justify drawing the conclusion that Earth's moon was not formed from a piece of the Earth?", "answers": "[\"Earth's moon consists primarily of elements that differ from those inside the Earth.\", 'Comets and large meteoroids normally move at very high speeds.', \"The craters on the surface of Earth's moon show that it has been struck by many thousands of large meteoroids.\", \"The moons of some planets in Earth's solar system were not formed primarily from the planets' outer shells.\"]", "label": 0 }, { "id": "train_3874", "context": "A survey taken ten years ago of residents of area L showed that although living conditions were slightly below their country' s average, most residents of L reported general satisfaction with their living conditions. However, this year the same survey found that while living conditions are now about the same as the national average, most residents of L report general dissatisfaction with their living conditions.", "question": "Which one of the following, if true, would most help to resolve the apparent conflict between the results of the surveys described above?", "answers": "['Ten years ago the residents of area L were not aware that their living conditions were below the national average.', 'Living conditions in an area generally improve only if residents perceive their situation as somehow in need of improvement.', 'Optimal living conditions were established in the survey by taking into account governmental policies and public demands on three continents.', \"Between the times that the two surveys were conducted, the average living conditions in L's country had substantially declined.\"]", "label": 3 }, { "id": "train_3875", "context": "Political leader: In this political dispute, our side will benefit from showing a desire to compromise with the opposition. If the opposition responds positively, then a compromise will be reached. If they do not, then they will be held responsible for the failure to reach a compromise and our side will benefit.", "question": "The conclusion of the political leader's argument follows logically if which one of the following is assumed?", "answers": [ "The political leader's side has a desire to compromise with the opposition.", "The opposition is rarely willing to compromise with the political leader's side.", "The opposition will compromise if the political leader's side shows a desire to compromise.", "The political leader's side will benefit if a compromise is reached." ], "label": 3 }, { "id": "train_3876", "context": "Most of Western music since the Renaissance has been based on a seven-note scale known as the diatonic scale, but when did the scale originate? A fragment of a bone flute excavated at a Neanderthal campsite has four holes, which are spaced in exactly the right way for playing the third through sixth notes of a diatonic scale. The entire flute must surely have had more holes, and the flute was made from a bone that was long enough for these additional holes to have allowed a complete diatonic scale to be played. Ttherefore, the Neanderthals who made the flute probably used a diatonic musical scale. ", "question": "In the argument given, the two portions in boldface play which of the following roles?", "answers": "['The first introduces evidence to support the main conclusion of the argument; the second is the main conclusion stated in the argument.', 'The first introduces the phenomenon that the argument as a whole seeks to explain; the second gives a reason to rule out one possible explanation.', 'The first introduces the phenomenon that the argument as a whole seeks to explain; the second presents a position to which the argument is opposed.', 'The first introduces evidence to support the main conclusion of the argument; the second presents a position to which the argument is opposed.']", "label": 0 }, { "id": "train_3877", "context": "Critic: Fillmore, an influential television executive, argues that watching television regularly is not detrimental to very young children. Fillmore bases this on the claim, which I grant, that children can learn much that is beneficial from television. But we should reject Fillmore' s argument, because clearly it is to Fillmore' s benefit to convince parents that television is not harmful to their children.", "question": "Which one of the following most accurately describes a flaw in the critic's reasoning?", "answers": "['It is based on an appeal to the views of someone with questionable authority on the subject matter.', 'It rejects an argument solely on the grounds that the argument could serve the interests of the person making that argument.', 'It bases its conclusion on claims that are inconsistent with one another.', \"It takes a necessary condition for something's being harmful to be a sufficient condition for being harmful.\"]", "label": 1 }, { "id": "train_3878", "context": "Enthusiasm for the use of calculators in the learning of mathematics is misplaced. Teachers rightly observe that in some cases calculators enable students to focus on general principles rather than the tedious, largely rote calculations that constitute the application of these principles. But principles are more likely to be remembered when knowledge of them is grounded in habits ingrained by painstaking applications of those principles. The very fact that calculators make calculation easier, ttherefore, makes it reasonable to restrict their use.", "question": "Which one of the following, if true, most strengthens the argument?", "answers": "['Some students who know how to use calculators also thoroughly understand the mathematical principles that calculators obey.', 'Slide rules, which are less technologically sophisticated analogues of calculators, were widely used in the learning of mathematics several decades ago.', 'Habits that are acquired by laborious and sometimes tedious practice are not as valuable as those that are painlessly mastered.', 'It is much more important that students retain the knowledge of general principles than that this knowledge be easily acquired.']", "label": 3 }, { "id": "train_3879", "context": "Doctor: Medical researchers recently examined a large group of individuals who said that they had never experienced serious back pain. Half of the members of the group turned out to have bulging or slipped disks in their spines, conditions often blamed for serious back pain. Since these individuals with bulging or slipped disks evidently felt no pain from them, these conditions could not lead to serious back pain in people who do experience such pain.", "question": "The reasoning in the doctor's argument is most vulnerable to the criticism that it fails to consider which one of the following possibilities?", "answers": "['A characteristic found in half of a given sample of the population might not occur in half of the entire population.', 'A factor that is not in itself sufficient to produce a certain effect may nonetheless be partly responsible for that effect in some instances.', 'A factor that need not be present in order for a certain effect to arise may nonetheless be sufficient to produce that effect.', 'An effect that occurs in the absence of a particular phenomenon might not occur when that phenomenon is present.']", "label": 1 }, { "id": "train_3880", "context": "The most common amphibian in the Tropicalico jungle is a unique variety of tree frog, but this variety is almost never seen, even by trained researchers studying the amphibian. Surprisingly, this frog is seen even less than tinier and less populous amphibian varieties, possibly less than any other amphibian type within the jungle.", "question": "Each of the following, if true, helps to reconcile the statements above EXCEPT:", "answers": "[\"The tree frog's croak is high-pitched and clearly identifiable while the croaks of most other frogs are low-pitched and indistinguishable.\", 'While many varieties of jungle amphibian will frequently move from plant to plant, the tree frog will often stay on the same tree for hours at a time without moving.', 'Most amphibians in the Tropicalico jungle exhibit some sort of communal behavior and often gather around common water sources, but the tree frog takes in water through its skin so it has no need to congregate near a water source.', 'Almost all of the trees within the Tropicalico jungle are covered in vines that bear a remarkable similarity to the patterns on the skin of the tree frog.']", "label": 0 }, { "id": "train_3881", "context": "All etching tools are either pin-tipped or bladed. While some bladed etching tools are used for engraving, some are not. On the other hand, all pin-tipped etching tools are used for engraving. Thus, there are more etching tools that are used for engraving than there are etching tools that are not used for engraving.", "question": "The conclusion of the argument follows logically if which one of the following is assumed?", "answers": "['All etching tools that are not used for engraving are bladed.', 'No etching tool is both pin-tipped and bladed.', 'All tools used for engraving are etching tools as well.', 'There are as many pin-tipped etching tools as there are bladed etching tools.']", "label": 3 }, { "id": "train_3882", "context": "Elephants are often observed licking clay from mud holes. There is ample evidence that other animals ingest clay because it neutralizes toxins in their diets. Since elephants eat many plants that contain toxic compounds, their licking of clay is probably triggered by toxins too.", "question": "Which one of the following, if true, most strengthens the argument?", "answers": "['Elephants do not lick clay at times when their diet consists almost entirely of toxin-free foods.', 'Some of the clay that elephants lick contains minerals that are nutritionally essential for any animal.', 'The clay that elephants ingest does not contain any compounds that are toxic to them.', 'Elephants typically drink water immediately before or after they lick clay.']", "label": 0 }, { "id": "train_3883", "context": "A local chemical plant produces pesticides that can cause sterility in small mammals such as otters. Soon after the plant began operating, the incidence of sterility among the otters that swim in a nearby river increased dramatically. Ttherefore, pesticides are definitely contaminating the river.", "question": "Which one of the following arguments contains a flaw in reasoning that is similar to one in the argument above?", "answers": "['A diet low in calcium can cause a drop in egg production in poultry. When chickens on a local farm were let out in the spring to forage for food, their egg production dropped noticeably. So the food found and eaten by the chickens is undeniably low in calcium.', 'The bacteria that cause tetanus live in the digestive tract of horses. Tetanus is a highly infectious disease. Consequently it must be that horses contract tetanus more frequently than do most other animals.', 'Animals that are undernourished are very susceptible to infection. Animals in the largest metropolitan zoos are not undernourished, so they surely must not be very susceptible to disease.', 'The only animal that could have produced a track similar to this one is a bear. But there are no bears in this area of the country, so this animal track is a fake.']", "label": 0 }, { "id": "train_3884", "context": "Any announcement authorized by the head of the department is important. However, announcements are sometimes issued, without authorization, by people other than the head of the department, so some announcements will inevitably turn out not to be important.", "question": "The reasoning is flawed because the argument", "answers": "['leaves open the possibility that the head of the department never, in fact, authorizes any announcements', 'overlooks the possibility that people other than the head of the department have the authority to authorize announcements', \"assumes without warrant that just because satisfying a given condition is enough to ensure an announcement's importance, satisfying that condition is necessary for its importance\", 'fails to distinguish between the importance of the position someone holds and the importance of what that person may actually be announcing on a particular occasion']", "label": 2 }, { "id": "train_3885", "context": "Editorial: It is common to find essays offering arguments that seem to show that our nation is in decline. There is no cause for alarm, however. The anxious tone of these essays shows that the problem is with the psychological state of their writers rather than with the actual condition of our nation.", "question": "Which one of the following most accurately describes a flaw in the editorial' s reasoning?", "answers": "['The editorial dismisses a particular view while offering evidence feat actually supports that view.', 'The editorial overlooks the possibility that the nation is neither thriving nor in decline.', 'The editorial compares two situations without considering the obvious differences between them.', 'The editorial dismisses a claim without considering any reasons presented in arguments for that claim.']", "label": 3 }, { "id": "train_3886", "context": "United Lumber will use trees from its forests for two products. The tree trunks will be used for lumber and the branches converted into wood chips to make fiberboard. The cost of this conversion would be the same whether done at the logging site, where the trees are debranched, or at United' s factory. However, wood chips occupy less than half the volume of the branches from which they are made.", "question": "The information given, if accurate, most strongly supports which of the following?", "answers": "['Converting the branches into wood chips at the logging site would require transporting a fully assembled wood-chipping machine to and from the site.', \"The debranching of trees and the conversion of the branches into chips are the only stages in the processing of branches that it would be in United's economic advantage to perform at the logging site.\", 'It would be more economical to debranch the trees at the factory where the fiberboard is manufactured.', 'Transportation costs from the logging site to the factory that are determined by volume of cargo would be lower if the conversion into chips is done at the logging site rather than at the factory.']", "label": 3 }, { "id": "train_3887", "context": "The common ancestors of Australian land- and treedwelling kangaroos had prehensile (grasping) tails and long opposable thumbs, attributes that are well-adapted to tree-dwelling but offer kangaroos few advantages on land. It is hardly surprising, ttherefore, that land-dwelling kangaroos eventually lost these attributes; what is puzzling is the fact that all modern tree-dwelling kangaroos now lack them as well.", "question": "Which one of the following, if true, most helps explain the puzzling fact cited above?", "answers": "['Modern tree-dwelling kangaroos must back down tree trunks slowly and carefully, but the common ancestors of modern tree-and land-dwelling kangaroos used their opposable thumbs to descend trees quickly headfirst.', \"Modern tree-dwelling kangaroos' tails cannot grasp branches, but they are somewhat longer and more flexible than those of modern land-dwelling kangaroos.\", 'Modern tree-dwelling kangaroos are descended from species of land-dwelling kangaroos that had been land-dwellers for many generations before modern treedwelling kangaroos started to develop.', 'Modern tree-dwelling kangaroos are smaller than most modern land-dwelling kangaroos but larger than their common ancestors.']", "label": 2 }, { "id": "train_3888", "context": "The local board of education found that, because the current physics curriculum has little direct relevance to today's world, physics classes attracted few high school students. So to attract students to physics classes, the board proposed a curriculum that emphasizes principles of physics involved in producing and analyzing visual images.", "question": "Which of the following, if true, provides the strongest reason to expect that the proposed curriculum will be successful in attracting students?", "answers": "[\"In today's world the production and analysis of visual images is of major importance in communications, business, and recreation.\", 'Several of the fundamental principles of physics are involved in producing and analyzing visual images.', 'Equipment that a large producer of photographic equipment has donated to the high school could be used in the proposed curriculum.', 'The number of students interested in physics today is much lower than the number of students interested in physics 50 years ago.']", "label": 0 }, { "id": "train_3889", "context": "Traditionally, students at Kelly University have evaluated professors on the last day of class. But some professors at Kelly either do not distribute the paper evaluation forms or do so selectively, and many students cannot attend the last day of class. Soon, students will be able to use school computers to evaluate their professors at any time during the semester. Ttherefore, evaluations under the new system will accurately reflect the distribution of student opinion about teaching performance.", "question": "Which one of the following is an assumption required by the argument?", "answers": "['Professors who distribute the paper evaluation forms selectively distribute them only to students they personally like.', 'Dissatisfied students are in general not more likely than satisfied students to submit a computerized evaluation.', \"Students can wisely and insightfully assess a professor's performance before the end of the semester.\", 'Nearly all professors who fail to distribute the paper evaluation forms do so because they believe the students will evaluate them unfavorably.']", "label": 1 }, { "id": "train_3890", "context": "In polluted environments, dolphins gradually accumulated toxins in their body fat, and the larger the dolphin the more accumulated toxin it can tolerate. Nearly 80 percent of the toxins a female dolphin has accumulated pass into the fat-rich milk her nursing calf ingests. Ttherefore, the unusually high mortality rate among dolphin calves in the industrially contaminated waters along Florida' s Gulf Coast is probably the result of their being poisoned by their mother' s milk.", "question": "Which of the following, if true, most strengthens the argument?", "answers": "['Dolphins, like other marine mammals, have a higher proportion of body fat than do most land mammals.', 'As dolphins age, they accumulate toxins from the environment more slowly than when they were young.', \"The rate at which adult dolphins living in the waters along Florida's Gulf Coast accumulate toxins is no higher than that of adult dolphins in comparably polluted waters elsewhere.\", \"The survival rate of firstborn dolphin calves in the area along Florida's Gulf Coast is highest for those whose mothers were killed before they were weaned.\"]", "label": 3 }, { "id": "train_3891", "context": "Physical education should teach people to pursue healthy, active lifestyles as they grow older. But the focus on competitive sports in most schools causes most of the less competitive students to turn away from sports. Having learned to think of themselves as unathletic, they do not exercise enough to stay healthy.", "question": "Which one of the following is most strongly supported by the statements above, if they are true?", "answers": "['Physical education should include noncompetitive activities.', 'Children should be taught the dangers of a sedentary lifestyle.', 'The mental aspects of exercise are as important as the physical ones.', 'Competition causes most students to turn away from sports.']", "label": 0 }, { "id": "train_3892", "context": "Bolivia ranks far behind Western European nations such as Great Britain and France when it comes to infant mortality rates. In all three countries, government agencies that oversee postnatal medical programs have been very successful in reducing the number of infants who are unvaccinated. In Bolivia, these government agencies are only active in urban areas. However, in Great Britain and France, government agencies are required by law to work in rural and suburban areas as well.", "question": "Which one of the following is supported by the information above?", "answers": "['Bolivia would have a lower infant mortality rate than Great Britain and France if its government agencies overseeing postnatal medical programs worked in rural and suburban areas.', 'Infant mortality rates have always been better in Western European nations than in Bolivia.', 'The requirement that government agencies overseeing post-natal medical program work in rural and suburban areas may help decrease infant mortality rates in Bolivia.', 'Government agencies that oversee post- natal medical programs in Bolivia would voluntarily begin working in rural and suburban areas.']", "label": 2 }, { "id": "train_3893", "context": "Economist: Global recessions can never be prevented, for they could be prevented only if they were predictable. Yet economists, using the best techniques at their disposal, consistently fail to accurately predict global recessions.", "question": "The economist's argument is most vulnerable to the criticism that it", "answers": "['presupposes in a premise the conclusion that it purports to establish', 'fails to establish that economists claim to be able to accurately predict global recessions', 'fails to address the possibility that the techniques available to economists for the prediction of global recessions will significantly improve', 'implicitly bases an inference that something will not occur solely on the information that its occurrence is not predictable']", "label": 2 }, { "id": "train_3894", "context": "All of the cargo ships of the Blue Star Line are over 100 meters long, and all of its passenger ships are under 100 meters long. Most of the ships of the Blue Star Line were built before 1980. All of the passenger and cargo ships of the Gold Star line were built after 1980, and all are under 100 meters long. The dockside facilities of Port Tropica, which is open only to ships of these two lines, can accommodate only those ships that are less than 100 meters long. The S. S. Coral is a cargo ship that is currently docked at Port Tropica.", "question": "If the statements above are true, which one of the following must be true on the basis of them?", "answers": "['All of the ships of the Blue Star Line are older than any of the ships of the Gold Star Line.', 'The S. S. Coral belongs to the Blue Star Line.', 'The S. S. Coral was built after 1980.', 'Port Tropica is served only by cargo ships.']", "label": 2 }, { "id": "train_3895", "context": "In Gilavia, the number of reported workplace injuries has declined 16 percent in the last five years. However, perhaps part of the decline results from injuries going unreported: many employers have introduced safety-incentive programs, such as prize drawings for which only employees who have a perfect work-safety record are eligible. Since a workplace injury would disqualify an employee from such programs, some employees might be concealing injury, when it is feasible to do so.", "question": "Which of the following, if true in Gilavia, most strongly supports the proposed explanation?", "answers": "['Employers generally have to pay financial compensation to employees who suffer work-related injuries.', 'In the last five years, there has been no decline in the number of workplace injuries leading to immediate admission to a hospital emergency room.', 'Employers who have instituted safety-incentive programs do not in general have a lower proportion of reported workplace injuries among their employees than do employers without such programs.', \"Many injuries that happen on the job are injuries that would be impossible to conceal and yet would not be severe enough to require any change to either the employee's work schedule or the employee's job responsibilities.\"]", "label": 1 }, { "id": "train_3896", "context": "No member of the Richardson Theater Group is both a performer and an administrator. Since Leon and Marta are both members of the Richardson Theater Group but neither is an administrator, it follows that both are performers.", "question": "Which one of the following arguments displays a flawed pattern of reasoning most similar to that in the argument above?", "answers": "['Not all of the employees of the Tedenco Company are salaried employees of that company. Since Mr. Lopez and Ms. Allen are both salaried employees of the Tedenco Company, it follows that they are not the only employees of the Tedenco Company.', 'No corporate attorney represents both the Dumone Company and the Tedenco Company. Since Ms. Tseung is a corporate attorney who represents the Dumone Company, it follows that she does not also represent the Tedenco Company.', 'No company can have its headquarters in both Canada and Mexico. Since neither the Dumone Company nor the Tedenco Company has its headquarters in Mexico, it follows that both have their headquarters in Canada.', 'No member of the board of directors of the Dumone Company is also a member of the board of directors of the Tedenco Company. Since neither company has fewer than five board members, it follows that both boards together include at least ten members.']", "label": 2 }, { "id": "train_3897", "context": "Cool weather typically weakens muscle power in cold-blooded creatures. In the veiled chameleon, a cold-blooded animal, the speed at which the animal can retract its tongue declines dramatically as the temperature falls. However, the speed at which this chameleon can extend its tongue does not decline much as the temperature falls.", "question": "Which one of the following, if true, most helps to resolve the apparent discrepancy in the information above?", "answers": "[\"Compared with the muscles in the tongues of most cold-blooded animals, the retraction muscles in the veiled chameleon's tongue are considerably stronger.\", 'Most cold-blooded animals are much more active in warmer weather than in cooler weather.', 'In the veiled chameleon, tongue retraction is powered by muscles, whereas tongue extension is driven by energy stored in a rubber band-like sheath.', 'Veiled chameleons are found in a wide range of habitats, including ones with wide variations in temperature and ones with moderate climates.']", "label": 2 }, { "id": "train_3898", "context": "Stage performances are judged to be realistic to the degree that actors reproduce on stage the behaviors generally associated by audiences with the emotional states of the characters portrayed. Traditional actors imitate those behaviors, whereas Method actors, through recollection of personal experience, actually experience the same emotions that their characters are meant to be experiencing. Audiences will ttherefore judge the performances of Method actors to be more realistic than the performances of traditional actors.", "question": "Which one of the following is an assumption on which the argument depends?", "answers": "['Realism is an essential criterion for evaluating the performances of both traditional actors and Method actors.', 'The behavior that results when a Method actor feels a certain emotion will conform to the behavior that is generally associated by audiences with that emotion.', \"Traditional actors do not aim to produce performances that are realistic representations of a character's emotional states.\", \"Performances based on an actor's own experience of emotional states are more likely to affect an audience's emotions than are performances based on imitations of the behaviors generally associated with those emotional states.\"]", "label": 1 }, { "id": "train_3899", "context": "While sales of other highly fuel-efficient automobiles are in decline, sales of the Hydro are rising. The Hydro' s manufacturers attribute its success to the Hydro' s price and very low fuel consumption. However, the Hydro is comparable in price and fuel efficiency to its competitors, so it is more likely that its success is due to the fact that people want to appear environmentally conscious to their neighbors.", "question": "Which one of the following is an assumption required by the argument?", "answers": "['Hydro buyers are more likely to have neighbors who also drive Hydros.', 'Hydro buyers have less interest in environmental causes than buyers of other highly fuel-efficient automobiles.', 'The Hydro is recognizable as environmentally friendly in a way that its competitors are not.', 'The Hydro is the most popular highly fuel-efficient automobile available.']", "label": 2 }, { "id": "train_3900", "context": "Insurance industry statistics demonstrate that cars with alarms or other antitheft devices are more likely to be stolen or broken into than cars without such devices or alarms. Ttherefore antitheft devices do not protect cars against thieves.", "question": "The pattern of flawed reasoning in the argument above is most similar to that in which one of the following?", "answers": "['Societies that support free public libraries are more likely to support free public universities than are societies without free public libraries. Hence a society that wishes to establish a free public university should first establish a free public library.', 'Studies reveal that people who are regular users of libraries purchase more books per year than do people who do not use libraries regularly. Hence using libraries regularly does not reduce the number of books that library patrons purchase.', 'Since youngsters who read voraciously are more likely to have defective vision than youngsters who do not read very much, it follows that children who do not like to read usually have perfect vision.', 'Since surveys reveal that communities with flourishing public libraries have, on average, better-educated citizens, it follows that good schools are typically found in communities with public libraries.']", "label": 1 }, { "id": "train_3901", "context": "It is proposed to introduce mosquitoes into the wild with genetic alterations that destroy their diseasecarrying capacity. In this way the dangerous wild population could eventually be replaced by a harmless one without leaving room for another disease-transmitting strain to flourish. One candidate gene would interfere with the mosquitoes' finding mates; another would cause the destruction of a disease parasite before the stage at which it could be transmitted; another would disable the mechanism of the mosquito' s own resistance to disease, so that it would die before transmitting the disease.", "question": "Which one of the following identifies a discrepancy in the proposal above?", "answers": "['Two of the proposed ways of destroying disease-carrying capacity in the wild mosquito population would jeopardize the goal of the proposal.', 'Evidence is not presented to show that each alternative method has been successfully tested on a limited scale.', 'None of the three proposed alternatives would assure that there would be fewer mosquitoes in any given area.', 'It does not take into account positive roles that mosquitoes play in the environment, such as serving, in the larval stage, as food for fish.']", "label": 0 }, { "id": "train_3902", "context": "Investment banks often have conflicting roles. They sometimes act for a client company by raising capital from other investment institutions as advantageously as possible, but their analysts also sometimes send unfavorable reports on the financial health of companies for whom they are raising capital to other clients who wish to make investments. Analyses of companies' financial health need to be unbiased if an investment bank is to achieve long-term success.", "question": "If the statements above are true, which of the following practices, if adopted by an investment bank, would hinder its long-term success?", "answers": "['Sharing the task of raising capital for a client with other investment banks', \"Monitoring the success or failure of analysts' current predictions about how companies will perform financially, in order to determine the value of future predictions\", 'Ensuring that conflicts between analysts and those who raise capital for clients are carefully mediated and resolved by impartial arbitrators', \"Evaluating and rewarding the bank's analysts on the basis of recommendations made by managers who are solely engaged in raising capital for clients\"]", "label": 3 }, { "id": "train_3903", "context": "Melchior: Some studies have linked infants' consumption of formula made from cow' s milk to subsequent diabetes. Nonetheless, parents should feed cow' s milk formula to their infants. After all, cow' s milk is an excellent source of several nutrients important to infants' development.", "question": "The reasoning in Melchior's argument is most vulnerable to criticism on the grounds that it", "answers": "['defends a certain practice on the basis that it has a certain benefit without considering whether an alternative practice has the same benefit', 'inappropriately introduces normative claims in support of a conclusion that is entirely factual', 'confuses an absence of evidence in support of a claim with the existence of evidence against a claim', 'draws a conclusion that simply restates a claim that is presented in support of that conclusion']", "label": 0 }, { "id": "train_3904", "context": "When girls are educated in single-sex secondary schools, they tend to do better academically than girls who attend mixed-sex schools. Since Alice achieved higher grades than any other woman in her first year at the university, she was probably educated at a single-sex school.", "question": "Which one of the following most closely parallels the flawed reasoning used in the argument above?", "answers": "['When children have parents who help them with their homework, they usually do well in school. Ttherefore, having help with homework is probably the cause of high academic achievement.', 'When students have individual tutoring in math, they usually get good grades on their final exams. Celia had individual tutoring in math so she will probably get a good grade.', 'When children practice their piano scales for half an hour each day, they usually pass their piano exams. Sally practices scales for less than half an hour each day, so she will probably fail her piano exam.', 'When babies are taught to swim, they have more than the average number of ear infections as they grow up. Janice has more ear infections than any other person at the local swimming club, so she probably was taught to swim when she was a baby.']", "label": 3 }, { "id": "train_3905", "context": "The number of Tundra swan in an arctic refuge has been steadily dwindling. It has been determined that the majority of the birds have died from lead poisoning, which is contained in the buckshot that hunters use to kill the swan. Local authorities have responded by banning hunting, and the number of hunters cited has since dropped dramatically, yet the number of birds that continue to die each month from lead poisoning has not changed.", "question": "Which of the following, if true, best resolves the discrepancy above?", "answers": "['Those hunters who continue to defy the hunting ban do not, on average, shoot more birds than did those who hunted in the park before the ban.', 'Most swans that have perished in the refuge have died as a result of ingesting pellets scattered about the ground, which introduce lead into their bloodstreams.', 'Park rangers have been unable to cite all of the the hunters that enter the park despite the ban.', 'Eventually, there will be so few swan left in the refuge that the overall number of swan dying each month will decrease.']", "label": 1 }, { "id": "train_3906", "context": "Home ownership is a sign of economic prosperity. This makes it somewhat surprising that across the various regions of Europe and North America, high levels of home ownership correspond with high levels of unemployment.", "question": "Which one of the following, if true, helps to resolve the apparent conflict described above?", "answers": "['People who own homes are more likely than those who rent to form support networks that help them to learn of local jobs.', 'People are more likely to buy homes when they are feeling economically secure.', 'Over the last few decades jobs have been moving from centralized areas to locations that are closer to homeowners.', 'Home ownership makes it more difficult to move to a place where jobs are more plentiful.']", "label": 3 }, { "id": "train_3907", "context": "Each species of moth has an optimal body temperature for effective flight, and when air temperatures fall much below that temperature, the moths typically have to remain inactive on vegetation for extended periods, leaving them highly vulnerable to predators. In general, larger moths can fly faster than smaller ones and hence have a better chance of evading flying predators, but they also have higher optimal body temperatures, which explains why __.", "question": "Which of the following most logically completes the passage?", "answers": "['large moths are proportionally much more common in warm climates than in cool climates', 'most predators of moths prey not only on several different species of moth but also on various species of other insects', 'large moths typically have wings that are larger in proportion to their body size than smaller moths do', 'large moths are generally able to maneuver better in flight than smaller moths']", "label": 0 }, { "id": "train_3908", "context": "Critic: The perennial image of the \"city on a hill\" associates elevated locations with elevated purposes. The city' s concert hall -- its newest civic building -- is located on a spectacular hilltop site. But because it is far from the center of the city, it cannot fulfill the purpose of a civic building. An example of a successful civic building is the art museum, which is situated in a densely populated downtown area. It encourages social cohesion and makes the city more alive.", "question": "The critic's reasoning most closely conforms to which one of the following principles?", "answers": "['A civic building that is located in a downtown area should, if possible, be located on an elevated site.', 'A city needs to have civic buildings if it is to have social cohesion.', 'The purpose of a civic building is to encourage social cohesion and to make a city more alive. 8', \"The downtown area of a city should be designed in a way that complements the area's civic buildings.\"]", "label": 2 }, { "id": "train_3909", "context": "Prolonged exposure to nonionizing radiation -- electromagnetic radiation at or below the frequency of visible light -- increases a person' s chances of developing soft-tissue cancer. Electric power lines as well as such electrical appliances as electric blankets and video-display terminals are sources of nonionizing radiation.", "question": "Which one of the following conclusions is best supported by the statements above?", "answers": "[\"Soft-tissue cancers are frequently cured spontaneously when sources of nonionizing radiation are removed from the patient's home.\", 'People will short-term exposure to nonionizing radiation are not at risk of developing soft-tissue cancers.', 'Soft-tissue cancers are more common than other cancers.', 'Certain electrical devices can pose health risks for their users.']", "label": 3 }, { "id": "train_3910", "context": "On the basis of research with young children, a developmental psychologist hypothesized that the skills involved in copying curves must be developed before the skills involved in copying angles can be developed.", "question": "Which one of the following, if true, supports the developmental psychologist's hypothesis?", "answers": "['All of the children who can copy angles can also copy curves.', 'All of the children who can copy curves can also copy straight lines.', 'The ability to discriminate angles must be developed before angles can be copied.', 'Some of the children who cannot copy curves can copy angles.']", "label": 0 }, { "id": "train_3911", "context": "A recent report determined that although only 3 percent of drivers on Maryland highways equipped their vehicles with radar detectors, 33 percent of all vehicles ticketed for exceeding the speed limit were equipped with them. Clearly, drivers who equip their vehicles with radar detectors are more likely to exceed the speed limit regularly than are drivers who do not.", "question": "The conclusion drawn above depends on which of the following assumptions?", "answers": "['Drivers who are ticketed for exceeding the speed limit are more likely to exceed the speed limit regularly than are drivers who are not ticketed.', 'Drivers on Maryland highways exceeded the speed limit more often than did drivers on other state highways not covered in the report.', 'Many of the vehicles that were ticketed for exceeding the speed limit were ticketed more than once in the time period covered by the report.', 'Drivers who equip their vehicles with radar detectors are less likely to be ticketed for exceeding the speed limit than are drivers who do not.']", "label": 0 }, { "id": "train_3912", "context": "Local resident: An overabundance of algae must be harmful to the smaller fish in this pond. During the fifteen or so years that I have lived here, the few times that I have seen large numbers of dead small fish wash ashore in late summer coincide exactly with the times that I have noticed abnormally large amounts of algae in the water.", "question": "The local resident's argument is most vulnerable to criticism on the grounds that it", "answers": "['presumes, without providing justification, that smaller fish are somehow more susceptible to harm as a result of overabundant algae than are larger fish', 'ignores the possibility that the overabundance of algae and the deaths of smaller fish are independent effects of a common cause', 'ignores the possibility that the same cause might have different effects on fish of different sizes', 'fails to consider that the effects on smaller fish of overabundant algae may be less severe in larger bodies of water with more diverse ecosystems']", "label": 1 }, { "id": "train_3913", "context": "The most common bird in Stillwater Marsh is a species of marsh hen, yet this species is rarely seen, even by experienced bird-watchers who seek it. In fact, this bird is seen far less frequently than any other bird inhabiting the marsh, including those that are much smaller and much less abundant.", "question": "Each of the following, if true, helps to reconcile the statements above EXCEPT:", "answers": "['Although many small marsh birds fly in groups to several feeding areas each day, the marsh hen tends to be solitary and flies only when it is in danger.', \"The marsh hen's call is harsh and repetitive, whereas the calls of many other marsh birds are pleasant and melodious.\", 'Unlike many small marsh birds, which dash along the banks of the marsh, the marsh hen remains completely still for long periods of time.', 'Many marsh birds are most active during daylight hours, but the marsh hen is usually most active at night.']", "label": 1 }, { "id": "train_3914", "context": "The miscarriage of justice in the Barker case was due to the mistaken views held by some of the forensic scientists involved in the case, who believed that they owed allegiance only to the prosecuting lawyers. Justice was thwarted because these forensic scientists failed to provide evidence impartially to both the defense and the prosecution. Hence it is not forensic evidence in general that should be condemned for this injustice.", "question": "Which one of the following, if true, most strengthens the argument?", "answers": "['Many forensic scientists do not believe that any miscarriage of justice occurred in the Barker case.', 'Most forensic scientists acknowledge a professional obligation to provide evidence impartially to both the defense and the prosecution.', 'Most prosecuting lawyers believe that forensic scientists owe a special allegiance to the prosecution.', 'Many instances of injustice in court cases are not of the same type as that which occurred in the Barker case.']", "label": 1 }, { "id": "train_3915", "context": "Law student: Law students cannot have a social life if they have any hope of succeeding academically. The daily reading and never-ending exam preparation frustrate all aspects of friendships. My friends sometimes invite me to watch a movie or go to a baseball game, but I can' t go. Our professors warned us of the workload and its affect on free time at the start of the semester. It' s completely impossible to budget fun into my busy schedule. I don' t know any law students who have any fun whatsoever.", "question": "The reasoning in the law student's argument is most vulnerable to criticism on the grounds that the argument:", "answers": "['Improperly draws a hasty generalization.', \"Dismisses all evidence that contradicts the law student's argument.\", 'Improperly uses extreme language.', 'Improperly relies on an inappropriate authority.']", "label": 2 }, { "id": "train_3916", "context": "The existence of polluting industries near schools does not lead to a decrease in the air quality at those schools. Proof of this is shown at middle school X in southern California, where an oil well was built on middle school X' s campus. The air quality at middle school X is no worse than the air quality at other middle schools in the state, and the students at middle school X do not suffer more frequently from air pollution-related allergies than do students in other middle schools in the state.", "question": "Each of the following statements, if true, weakens the argument EXCEPT:", "answers": "['More animal and plant life thrive near middle school X than at other middle schools in the state.', 'More students at middle school X suffer from allergies, commonly known to be caused by air pollution, than students at middle schools not located near pollution sources.', 'Middle school X, located near the oil well, is also farther from major highways and other sources of air pollution than the other middle schools in the state.', 'Prior to the oil well being built near middle school X, the air quality at middle school X was marginally better than the air quality of other middle schools in the state.']", "label": 0 }, { "id": "train_3917", "context": "Anthropological studies indicate that distinct cultures differ in their moral codes. Thus, as long as there are distinct cultures, there are no values shared across cultures.", "question": "Each of the following, if true, would weaken the argument EXCEPT:", "answers": "['As a result of advancing technology and global communication, we will someday all share the same culture and the same values.', \"Anthropologists rely on inadequate translation techniques to investigate the values of cultures that use languages different from the anthropologists' languages.\", 'The anthropologists who have studied various cultures have been biased in favor of finding differences rather than similarities between distinct cultures.', 'Although specific moral values differ across cultures, more general moral principles, such as \"Friendship is good, \" are common to all cultures.']", "label": 0 }, { "id": "train_3918", "context": "The size of the spleen is a good indicator of how healthy a bird is: sickly birds generally have significantly smaller spleens than healthy birds. Researchers found that, in general, birds that had been killed by predators had substantially smaller spleens than birds killed accidentally.", "question": "Which one of the following is most strongly supported by the information above?", "answers": "['Sickly birds are more likely than healthy birds to be killed by predators.', 'Small spleen size is one of the main causes of sickness in birds.', 'Most birds with smaller than average spleens are killed by predators.', 'Predators can sense whether a bird is sick.']", "label": 0 }, { "id": "train_3919", "context": "People who live in large houses tend to make more money than people who live in small houses. If you desire to make more money, you should move into a larger house.", "question": "The reasoning in this argument most closely parallels which one of the following?", "answers": "['People who like to sing tend to sing Karaoke. If you like to sing, you should sing Karaoke.', 'People who eat cake tend to gain weight. If you want to lose weight, you should eat less cake.', 'People who work hard tend to make more money. If you work hard, you will make more money.', 'People who wear smaller size clothes tend to be thinner. If you want to be thinner, you should wear smaller size clothing.']", "label": 3 }, { "id": "train_3920", "context": "A prominent investor who holds a large stake in the Burton Tool Company has recently claimed that the company is mismanaged , citing as evidence the company' s failure to slow production in response to a recent rise in its inventory of finished products. It is doubtful whether an investor' s sniping at management can ever be anything other than counterproductive, but in this case it is clearly not justified . It is true that an increased inventory of finished products often indicates that production is outstripping demand, but in Burton' s case it indicates no such thing. Rather, the increase in inventory is entirely attributable to products that have already been assigned to orders received from customers.", "question": "In the argument given, the two boldfaced portions play which of the following roles?", "answers": "['The first is evidence that has been used to support a position that the argument as a whole opposes; the second states the conclusion of the argument as a whole.', 'The first states the position that the argument as a whole opposes; the second is evidence that has been used to support the position being opposed.', 'The first states the position that the argument as a whole opposes; the second states the conclusion of the argument as a whole.', 'The first is evidence that has been used to support a position that the argument as a whole opposes; the second provides information to undermine the force of that evidence.']", "label": 2 }, { "id": "train_3921", "context": "In his new book on his complex scientific research, R frequently imputes bad faith to researchers disagreeing with him. A troubling aspect of R' s book is his stated conviction that other investigators' funding sources often determine what \"findings\" those investigators report. Add to this that R has often shown himself to be arrogant, overly ambitious, and sometimes plain nasty, and it becomes clear that R' s book does not merit attention from serious professionals.", "question": "The author of the book review commits which one of the following reasoning errors?", "answers": "['taking it for granted that an investigator is unlikely to report findings that are contrary to the interests of those funding the investigation', 'presenting as facts several assertions about the book under review that are based only on strong conviction and would be impossible for others to verify', 'using an attack on the character of the writer of the book as evidence that his person is not competent on matters of scientific substance', 'dismissing a scientific theory by giving a biased account of it']", "label": 2 }, { "id": "train_3922", "context": "It is often said that high rates of inflation tend to diminish people's incentive to save and invest. This view must be incorrect, however, because people generally saved and invested more of their income in the 1970's when inflation rates were high than they did in the 1980's when inflation rates were low.", "question": "Of the following, the best criticism of the argument above is that it overlooks the possibility that", "answers": "['the proponents of the view cited would stand to gain if inflation rates become lower', \"a factor that affects people's savings behavior in a certain way could affect people's investment behavior quite differently\", \"certain factors operating in the 1980's but not in the 1970's diminished people's incentive to save and invest\", \"the population was larger in the 1980's than it was in the 1970's\"]", "label": 2 }, { "id": "train_3923", "context": "Politician: Hybrid cars use significantly less fuel per kilometer than nonhybrids. And fuel produces air pollution, which contributes to a number of environmental problems. Motorists can save money by driving cars that are more fuel efficient, and they will be encouraged to drive hybrid cars if we make them aware of that fact. Ttherefore, we can help reduce the total amount of pollution emitted by cars in this country by highlighting this advantage of hybrid cars.", "question": "Which of the following, if true, would most indicate a vulnerability of the politician's argument?", "answers": "['People with more fuel-efficient cars typically drive more than do those with less fuel-efficient cars.', 'Hybrid cars have already begun to gain popularity.', 'Not all air pollution originates from automobiles.', 'The future cost of gasoline and other fuel cannot be predicted with absolute precision or certainty.']", "label": 0 }, { "id": "train_3924", "context": "The familiar slogan \"survival of the fittest\" is popularly used to express the claim, often mistakenly attributed to evolutionary biologists, that the fittest are most likely to survive. However, biologists use the term \"fittest\" to mean \"most likely to survive, \" so the slogan is merely claiming that the most likely to survive are the most likely to survive. While this claim is clearly true, it is a tautology and so is neither informative nor of scientific interest.", "question": "The argument above depends on assuming which one of the following?", "answers": "['Only claims that are true are of scientific interest.', 'Informative scientific claims cannot use terms in the way they are popularly used.', 'Popular slogans are seldom informative or of scientific interest.', 'The truth of a purported scientific claim is not sufficient for it to be of scientific interest.']", "label": 3 }, { "id": "train_3925", "context": "Manager: Although our corporation lists rules in an orientation booklet, few employees read the booklet carefully enough to familiarize themselves with all the rules. Lecturing employees for inadvertent rule violations often makes them resentful and less cooperative. Thus, to improve employee adherence to rules, we plan to issue gentle reminders about various rules in each issue of our weekly newsletter.", "question": "Which of the following would it be most helpful to discover about the employees in the corporation in order to evaluate the likelihood that the plan will succeed?", "answers": "['Whether most of them violate at least some rules with which they are familiar', 'Whether most of them who inadvertently violate rules already feel resentful and uncooperative', 'Whether most of them would usually read with sufficient care the portions of the weekly newsletter that are reminders of rules', 'Whether most of them who regularly read the weekly newsletter are familiar with at least some rules']", "label": 2 }, { "id": "train_3926", "context": "An advertising agency employs ten times the number of employees relative to its competitors. Thus, any company should hire that advertising agency.", "question": "Which one of the following arguments contains the most similar reasoning to the argument above?", "answers": "['A blog produces ten times the amount of content relative to its competitors. Thus, that blog produces more content than its competitors.', 'A tree produces more apples than any other tree. Thus, that tree is the best tree for growing apples.', 'A widget company produces more products per year than its competitors and sells each widget for the highest price in the industry. Thus, that widget company earns more revenue than any other widget company.', \"A building is twice as tall as any other building on the city's skyline. Thus, that building is the tallest building on the city's skyline.\"]", "label": 1 }, { "id": "train_3927", "context": "The blues is a modern musical form whose lyrics usually address such topics as frustration, anger, oppression, and restlessness. Yet blues musicians claim to find joy in performing, and the musicians and fans alike say that the blues' overall effect is an affirmation of life, love, and hope.", "question": "Each of the following, if true, helps to resolve the apparent conflict in the passage EXCEPT:", "answers": "['Blues musicians who do not draw on their personal tragedies are no more successful than blues musicians who do.', \"The irony and wit found in the blues provide a sense of perspective on life's troubles.\", 'The conversion of personal sorrow into an artistic work can have a cathartic effect on artists and their audiences.', 'The sharing of blues music serves to create a cohesive, sympathetic social network.']", "label": 0 }, { "id": "train_3928", "context": "Paleontologist: About 2. 8 million years ago, many species that lived near the ocean floor suffered substantial population declines. These declines coincided with the onset of an ice age. The notion that cold killed those bottom-dwelling creatures outright is misguided, however; temperatures near the ocean floor would have changed very little. Nevertheless, the cold probably did cause the population declines, though indirectly. Many bottom-dwellers depended for food on plankton, small organisms that lived close to the surface and sank to the bottom when they died. Most probably, the plankton suffered a severe population decline as a result of sharply lower temperatures at the surface, depriving many bottom-dwellers of food. ", "question": "In the paleontologist's reasoning, the two portions in boldface play which of the following roles?", "answers": "['The first is a generalization put forward by the paleontologist; the second presents certain exceptional cases in which that generalization does not hold.', 'The first is an explanation challenged by the paleontologist; the second is an explanation proposed by the paleontologist.', 'The first is a judgment advanced in support of a conclusion reached by the paleontologist; the second is that conclusion.', 'The first introduces the hypothesis proposed by the paleontologist; the second is a judgment offered in spelling out that hypothesis.']", "label": 3 }, { "id": "train_3929", "context": "Advertisement: Hypnosis videos work to alter behavior by subliminally directing the subconscious to act in certain ways. Directions to the subconscious must, however, be repeated many times in order to be effective. Hypnosis videos from Mesmosis, Inc. induce a hypnotic stale and then issue an initial command to the subject' s subconscious to experience each subsequent instruction as if it had been repeated 1, 000 times. Because of the initial instruction, the subsequent instructions on Mesmosis videos are extremely effective -- it is as if they had actually been repeated 1, 000 times!", "question": "The advertisement's reasoning is most vulnerable to criticism on the grounds that the advertisement", "answers": "['overlooks a requirement that it states for the effectiveness of directions to the subconscious', 'takes for granted that the effectiveness of a direction to the subconscious is always directly proportional to the number of times the direction is repeated', 'concludes that hypnosis videos will be effective simply because they have never been proven to be ineffective', 'concludes that hypnosis is the most effective technique for altering behavior without considering evidence supporting other techniques']", "label": 0 }, { "id": "train_3930", "context": "Archaeologist: A large corporation has recently offered to provide funding to restore an archaeological site and to construct facilities to make the site readily accessible to the general public. The restoration will conform to the best current theories about how the site appeared at the height of the ancient civilization that occupied it. This offer should be rejected, however, because many parts of the site contain unexamined evidence.", "question": "Which one of the following principles, if valid, justifies the archaeologist's argument?", "answers": "[\"Any restoration of an archaeological site should represent only the most ancient period of that site's history.\", 'The risk of losing evidence relevant to possible future theories should outweigh any advantages of displaying the results of theories already developed.', \"Only those with a true concern for an archaeological site's history should be involved in the restoration of that site.\", 'The ownership of archaeological sites should not be under the control of business interests.']", "label": 1 }, { "id": "train_3931", "context": "Council chair: The traditional code of parliamentary procedure contains a large number of obscure, unnecessary rules, which cause us to quibble interminably over procedural details and so to appear unworthy of public confidence. Admittedly, the code is entrenched and widely accepted. But success in our endeavors depends on the public' s having confidence in our effectiveness. Ttherefore, it is imperative that we adopt the alternate code, which has been in successful use elsewhere for several years.", "question": "Which one of the following, if true, most seriously undermines the chair's conclusion?", "answers": "['Revision of the traditional code is underway that will eliminate the problematic rules.', 'The alternate code contains few provisions that have thus far been criticized as obscure or unnecessary.', \"Those who have adopted the alternate code sometimes attempt to use it to obscure their opponents' understanding of procedures.\", \"It is not always reasonable to adopt a different code in order to maintain the public's confidence.\"]", "label": 0 }, { "id": "train_3932", "context": "Deep tillage is even more deleterious to the world' s topsoil supply than previously believed. For example, farmers who till deeply are ten times more likely to lose topsoil to erosion than are farmers who use no-till methods. Results like these make it clear that farmers who now till deeply should strive, by using other topsoil aeration techniques, to incorporate no-till methods instead.", "question": "The argument depends on assuming which one of the following?", "answers": "['In deep-tillage farming, the deeper one tills, the greater the susceptibility to topsoil erosion.', 'Topsoil erosion does not make farmers want to till more deeply.', 'The most expensive farming methods employ topsoil aeration techniques other than deep tillage.', 'Tilling by any method other than deep tillage is not a viable option.']", "label": 3 }, { "id": "train_3933", "context": "A convenience store manager noticed that a cooler which had been stocked with only a cola product and an iced-tea product had 15 colas left at the end of the day but only 3 iced-tea beverages. As a result, the manager reasoned that he should increase the amount of iced tea and decrease the amount of cola he ordered from the distributor.", "question": "Which of the following, if true, would most strengthen the manager's rationale for ordering more iced tea and less cola?", "answers": "['On the subsequent day, the remaining three iced tea beverages all sold within the first hour after the store opened.', 'The cooler in question is the only place in the store where the cola and iced tea beverages are stocked.', 'At the beginning of the day, the cooler was stocked with at least as many of the iced tea beverages as of the cola beverages.', 'During that week, a special \"buy one, get one free\" sale was in effect for the cola beverage.']", "label": 2 }, { "id": "train_3934", "context": "Currently, the city of Grimchester is liable for any injury incurred because of a city sidewalk in need of repair or maintenance. However, Grimchester' s sidewalks are so extensive that it is impossible to hire enough employees to locate and eliminate every potential danger in its sidewalks. Governments should be liable for injuries incurred on public property only if they knew about the danger beforehand and negligently failed to eliminate it.", "question": "Which one of the following describes an injury for which the city of Grimchester is now liable, but should not be according to the principle cited above?", "answers": "['A person is injured after tripping over a shopping bag that someone had left lying in the middle of the sidewalk.', 'A person is injured after stepping in a large hole in a city sidewalk, and the city administration had first learned of the need to repair that sidewalk minutes before.', 'A person who is heavily intoxicated is injured after falling on a perfectly even city sidewalk with no visible defects.', 'A person is injured after tripping on a badly uneven city sidewalk, and the city administration had been repeatedly informed of the need to repair the sidewalk for several years.']", "label": 1 }, { "id": "train_3935", "context": "Drivers in the country of Holston want highway tolls eliminated. The cost of maintaining the highways is paid entirely out of revenue from the highway tolls. Holston must maintain its highways. Thus, it follows that if the tolls are eliminated, then the entire cost of maintaining the highways will have to be paid for by an increase in general taxes.", "question": "Which one of the following is an assumption made by the argument?", "answers": "['The amount of money saved by eliminating the costs associated with toll collection would not be available to pay part of the total cost of maintaining the highways.', 'The total cost of maintaining the highways will be less if the highway tolls rather than the general taxes pay for the maintenance.', 'If the highway tolls are not eliminated, there will be no need to increase the general taxes.', 'If the highway tolls are eliminated and the general taxes are increased to pay for maintaining the highways, there will be less emphasis on preventive maintenance.']", "label": 0 }, { "id": "train_3936", "context": "After years of good health, Jacob finally visited the doctor for the first time in a decade. Immediately after his doctor' s appointment, Jacob fell ill with the flu.", "question": "Each of the following, if true, explains the apparent paradox, EXCEPT:", "answers": [ "It was below freezing during the week before his doctor's appointment, and Jacob did not wear his winter coat.", "Jacob's flu was dormant and didn't show symptoms until after the doctor's visit.", "Jacob's immune system did not worsen after the doctor's visit.", "Jacob's doctor administered him several vaccines that temporarily weakened his immune system." ], "label": 2 }, { "id": "train_3937", "context": "Critic: The idealized world portrayed in romance literature is diametrically opposed to the debased world portrayed in satirical literature. Nevertheless, the major characters in both types of works have moral qualities that reflect the worlds in which they are presented. Comedy and tragedy, meanwhile, require that the moral qualities of major characters change during the course of the action. Ttherefore, neither tragedy nor comedy can be classified as satirical literature or romance literature.", "question": "The critic's conclusion follows logically if which one of the following is assumed?", "answers": "['If a character in a tragedy is idealized at the beginning of the action depicted in the tragedy, he or she must be debased at the end.', 'Some characters in comedies and tragedies are neither debased nor idealized.', \"In romance literature and satirical literature, characters' moral qualities do not change during the course of the action.\", 'The visions of the world portrayed in works of tragedy and works of comedy change during the course of the action.']", "label": 2 }, { "id": "train_3938", "context": "Farmer: Agricultural techniques such as crop rotation that do not use commercial products may solve agricultural problems at least as well as any technique, such as pesticide application, that does use such products. Nonetheless, no private for-profit corporation will sponsor research that is unlikely to lead to marketable products. Thus, for the most part, only government-sponsored research investigates agricultural techniques that do not use commercial products.", "question": "Which one of the following, if true, most strengthens the farmer's argument?", "answers": "['For almost any agricultural problem, there is at least one agricultural technique that does not use commercial products but that would solve that agricultural problem.', 'Most if not all government-sponsored agricultural research investigates agricultural techniques that do not use commercial products.', 'Most if not all investigations of agricultural techniques that use commercial products are sponsored by private for-profit corporations.', 'Investigations of agricultural techniques are rarely sponsored by individuals or by any entity other than private for-profit corporations or the government.']", "label": 3 }, { "id": "train_3939", "context": "Human skin gives off an array of gaseous substances, including carbon dioxide and lactic acid, both of which attract mosquitoes. However, neither of these two substances, whether alone or combined with one another, will attract mosquitoes as much as a bare human arm will, even in complete darkness, where a mosquito has no visual cues. Ttherefore, some other gaseous substance given off by human skin also attracts mosquitoes.", "question": "The reasoning in the argument requires which one of the following assumptions?", "answers": "['Mosquitoes are no more successful in finding a bare human arm in darkness than in light.', 'Human skin gives off gaseous substances in greater amounts during the day than during the night.', 'Mosquitoes are not attracted to humans by body heat.', 'Human skin never gives off any gaseous substances that repel mosquitoes.']", "label": 2 }, { "id": "train_3940", "context": "Unless the residents of Glen Hills band together, the proposal to rezone that city will be approved. If it is, the city will be able to build the water and sewer systems that developers need in order to construct apartment houses there. These buildings would attract new residents, and the increased population would probably result in overcrowded schools and would certainly result in roads so congested that new roads would be built. Neither new roads nor additional schools could be built without substantial tax increases for the residents of Glen Hills. Ultimately, this growth might even destroy the rural atmosphere that makes Glen Hills so attractive.", "question": "Which one of the following can be properly concluded from the passage?", "answers": "['If developers build apartment houses in Glen Hills, there will be substantial tax increases for the residents of Glen Hills.', 'If the rezoning proposal does not pass, the rural atmosphere in Glen Hills will not be lost.', 'If developers do not build apartment houses in Glen Hills, the schools of Glen Hills will not be overcrowded and roads will not be congested.', 'If developers do not build apartment houses in Glen Hills, the taxes of the residents of Glen Hills will not increase substantially.']", "label": 0 }, { "id": "train_3941", "context": "Taxi drivers, whose income is based on the fares they receive, usually decide when to finish work each day by setting a daily income target; they stop when they reach that target. This means that they typically work fewer hours on a busy day than on a slow day.", "question": "The facts described above provide the strongest evidence against which one of the following?", "answers": "[\"The number of hours per day that a person is willing to work depends on that person's financial needs.\", 'People who are paid based on their production work more efficiently than those who are paid a fixed hourly wage.', 'People work longer when their effective hourly wage is high than when it is low.', 'Workers will accept a lower hourly wage in exchange for the freedom to set their own schedules.']", "label": 2 }, { "id": "train_3942", "context": "Food that is very high in fat tends to be unhealthy. These brownies are fat-free, while those cookies contain a high percentage of fat. Ttherefore, these fat-free brownies are healthier than those cookies are.", "question": "Which one of the following exhibits flawed reasoning most similar to the flawed reasoning exhibited by the argument above?", "answers": "['Vegetables that are overcooked generally have few vitamins. Ttherefore, these carrots, which are overcooked, contain fewer vitamins than those peas, which are uncooked.', 'Canned foods always contain more salt than frozen foods do. Ttherefore, these canned peas contain more salt than those frozen peas do.', \"Some types of nuts make Roy's throat itch. These cookies contain a greater percentage of nuts than that pie contains. Ttherefore, these cookies are more likely to make Roy's throat itch.\", 'The human body needs certain amounts of many minerals to remain healthy. Ttherefore, this distilled water, which has no minerals, is unhealthy.']", "label": 0 }, { "id": "train_3943", "context": "A survey of a group of people between the ages of 75 and 80 found that those who regularly played the card game bridge tended to have better short-term memory than those who did not play bridge. It was originally concluded from this that playing bridge can help older people to retain and develop their memory. However, it may well be that bridge is simply a more enjoyable game for people who already have good short-term memory and who are thus more inclined to play.", "question": "In countering the original conclusion the reasoning above uses which one of the following techniques?", "answers": "['conceding the suggested relationship between playing bridge and short-term memory, but questioning whether any conclusion about appropriate therapy can be drawn', 'arguing that the original conclusion relied on an inaccurate understanding of the motives that the people surveyed have for playing bridge', 'providing an alternative hypothesis to explain the data on which the original conclusion was based', 'challenging the representativeness of the sample surveyed']", "label": 2 }, { "id": "train_3944", "context": "Editorial: In order to encourage personal responsibility in adults, society should not restrict the performance of any of the actions of adults or interfere with the likely results except to prevent negative effects on others.", "question": "Which one of the following expresses a view that is inconsistent with the principle stated in the editorial?", "answers": "['Highway speed limits are a justified restriction of freedom. For drivers who speed do not risk only their own lives; such drivers often injure or kill other people. Moreover, speed limits have been shown to significantly reduce highway accident and fatality rates.', 'Even though public smoking may lead to indirect harm to others, it should not be banned. There are several other ways to eliminate this harm that do not restrict the conduct of smokers and hence are preferable to a complete ban on public smoking.', \"The scientist who invented this technology is not the only one who should be allowed to profit from it. After all, there is no evidence that allowing others to profit from this technology will reduce the scientist's own profits.\", 'It is not enough that consumable products containing harmful substances have warning labels. Many adults simply ignore such warnings and continue to consume these substances in spite of the harm it may cause them. This is why consuming such substances should be illegal.']", "label": 3 }, { "id": "train_3945", "context": "Pro-Tect Insurance Company has recently been paying out more on car-theft claims than it expected. Cars with special antitheft devices or alarm systems are much less likely to be stolen than are other cars. Consequently Pro-Tect, as part of an effort to reduce its annual payouts, will offer a discount to holders of car-theft policies if their cars have antitheft devices or alarm systems.", "question": "Which of the following, if true, provides the strongest indication that the plan is likely to achieve its goal?", "answers": "['The number of policyholders who have filed a claim in the past year is higher for Pro-Tect than for other insurance companies.', 'Currently, Pro-Tect cannot legally raise the premiums it charges for a given amount of insurance against car theft.', 'In one or two years, the discount that Pro-Tect is offering will amount to more than the cost of buying certain highly effective antitheft devices.', 'The amount Pro-Tect has been paying out on car-theft claims has been greater for some models of car than for others.']", "label": 2 }, { "id": "train_3946", "context": "A fundamental illusion in robotics is the belief that improvements in robots will liberate humanity from \"hazardous and demeaning work. \" Engineers are designing only those types of robots that can be properly maintained with the least expensive, least skilled human labor possible. Ttherefore, robots will not eliminate demeaning work -- only substitute one type of demeaning work for another.", "question": "The reasoning in the argument is most vulnerable to the criticism that it", "answers": "['fails to address the possibility that the amount of demeaning work eliminated by robots might be significantly greater than the amount they create', 'attempts to support its conclusion by an appeal to the emotion of fear, which is often experienced by people faced with the prospect of losing their jobs to robots', 'does not specify whether or not the engineers who design robots consider their work demeaning', 'assumes what it sets out to prove, that robots create demeaning work']", "label": 0 }, { "id": "train_3947", "context": "The prairie vole, a small North American grassland rodent, breeds year-round, and a group of voles living together consists primarily of an extended family, often including two or more litters. Voles commonly live in large groups from late autumn through winter; from spring through early autumn, however, most voles live in far smaller groups. The seasonal variation in group size can probably be explained by a seasonal variation in mortality among young voles.", "question": "Which of the following, if true, provides the strongest support for the explanation offered?", "answers": "['Prairie vole populations vary dramatically in size from year to year.', 'It is in the spring and early summer that prairie vole communities generally contain the highest proportion of young voles.', 'The prairie vole subsists primarily on broad-leaved plants that are abundant only in spring.', 'Snakes, a major predator of young prairie voles, are active only from spring through early autumn.']", "label": 3 }, { "id": "train_3948", "context": "Hunter: Hunters alone are blamed for the decline in Greenrock National Forest' s deer population over the past ten years. Yet clearly, black bears have also played an important role in this decline . In the past ten years, the forest' s protected black bear population has risen sharply, and examination of black bears found dead in the forest during the deer hunting season showed that a number of them had recently fed on deer.", "question": "In the hunter's argument, the boldface portion plays which of the following roles?", "answers": "['It is a finding that the argument seeks to explain.', 'It is a judgment that the argument opposes.', 'It is an objection that has been raised against the main conclusion of the argument.', 'It is the main conclusion of the argument.']", "label": 3 }, { "id": "train_3949", "context": "In the nation of Partoria, large trucks currently account for 6 percent of miles driven on Partoria' s roads but are involved in 12 percent of all highway fatalities. The very largest trucks-those with three trailers-had less than a third of the accident rate of single- and double-trailer trucks. Clearly, ttherefore, one way for Partoria to reduce highway deaths would be to require shippers to increase their use of triple-trailer trucks.", "question": "Which of the following, if true, most seriously weakens the argument?", "answers": "['So far only the best, most experienced drivers for Partorian trucking companies have been driving triple-trailer trucks.', 'Very few fatal collisions involving trucks in Partoria are collisions between two trucks.', 'In Partoria, the maximum legal payload of a triple-trailer truck is less than three times the maximum legal payload of the largest of the single-trailer trucks.', 'In Partoria, the safety record of the trucking industry as a whole has improved slightly over the past ten years.']", "label": 0 }, { "id": "train_3950", "context": "A person with low self-esteem will be treated disrespectfully more often than will a person with high self-esteem. Moreover, a recent experiment found that, when people with low self-esteem and those with high self-esteem are both confronted with the same treatment by others, people with low self-esteem are much more likely to feel that they have been treated disrespectfully. Thus, __.", "question": "Which one of the following most logically completes the argument?", "answers": "['people with low self-esteem more frequently think that they are being treated disrespectfully than do people with high self-esteem', 'people with low self-esteem are usually right when they think they have been treated disrespectfully', 'a person with low self-esteem will be more inclined to treat others disrespectfully than will a person with high self-esteem', 'if an individual has been treated disrespectfully, it is probably because the individual was perceived to have low self-esteem']", "label": 0 }, { "id": "train_3951", "context": "Flavonoids are a common component of almost all plants, but a specific variety of flavonoid in apples has been found to be an antioxidant. Antioxidants are known to be a factor in the prevention of heart disease.", "question": "Which one of the following can be properly inferred from the passage?", "answers": "['At least one type of flavonoid helps to prevent heart disease.', 'Eating at least one apple each day will prevent heart disease.', 'Flavonoids are essential to preventing heart disease.', 'A diet composed largely of fruits and vegetables will help to prevent heart disease.']", "label": 0 }, { "id": "train_3952", "context": "Fifty chronic insomniacs participated in a one-month study conducted at an institute for sleep disorders. Half were given a dose of a new drug and the other half were given a placebo every night before going to bed at the institute. Approximately 80 percent of the participants in each group reported significant relief from insomnia during the first two weeks of the study. But in each group, approximately 90 percent of those who had reported relief claimed that their insomnia had returned during the third week of the study.", "question": "Which one of the following, if true, most helps to explain all the data from the study?", "answers": "[\"Most insomniacs sleep better in a new environment, and the new drug has no effect on an insomniac's ability to sleep.\", 'Some insomniacs cannot reliably determine how much sleep they have had or how well they have slept.', 'The psychological comfort afforded by the belief that one has taken a sleep-promoting drug is enough to prevent most episodes of insomnia.', 'The new drug is very similar in chemical composition to another drug, large doses of which have turned out to be less effective than expected.']", "label": 0 }, { "id": "train_3953", "context": "A survey published in a leading medical journal in the earty 1970s found that the more frequently people engaged in aerobic exercise, the lower their risk of lung disease tended to be. Since other surveys have confirmed these results, it must be the case that aerobic exercise has a significant beneficial effect on people' s health.", "question": "The reasoning above is questionable because the argument", "answers": "[\"fails to consider that even infrequent aerobic exercise may have some beneficial effect on people's health\", 'concludes merely from the fact that two things are correlated that one causes the other', 'ignores anecdotal evidenee and bases its conclusion entirely on scientific research', 'presumes, without providing justffication, that anyone who does not have lung disease is in good health']", "label": 1 }, { "id": "train_3954", "context": "Inez: The book we are reading, The Nature of Matter, is mistitled. A title should summarize the content of the whole book, but nearly half of this book is devoted to discussing a different, albeit closely related subject: energy. Antonio: I do not think that the author erred; according to modern physics, matter and energy are two facets of the same phenomenon.", "question": "Which one of the following is most strongly supported by the conversation above?", "answers": "[\"Inez believes that the book's title should not mention matter without mentioning energy.\", 'Inez and Antonio disagree on whether matter and energy are related.', 'Inez and Antonio disagree about the overall value of the book.', 'Antonio believes that there are no differences between matter and energy.']", "label": 0 }, { "id": "train_3955", "context": "Galanin is a protein found in the brain. In an experiment, rats that consistently chose to eat fatty foods when offered a choice between lean and fatty foods were found to have significantly higher concentrations of galanin in their brains than did rats that consistently chose lean over fatty foods. These facts strongly support the conclusion that galanin causes rats to crave fatty foods.", "question": "Which one of the following, if true, most supports the argument?", "answers": "['The chemical components of galanin are present in both fatty foods and lean foods.', 'The rats that preferred fatty foods had the higher concentrations of galanin in their brains before they were offered fatty foods.', 'Rats that metabolize fat less efficiently than do other rats develop high concentrations of galanin in their brains.', 'The brains of the rats that consistently chose to eat fatty foods did not contain significantly more fat than did the brains of rats that consistently chose lean foods.']", "label": 1 }, { "id": "train_3956", "context": "According to a theory embraced by some contemporary musicians, music is simply a series of sounds, bereft of meaning. But these musicians, because they understand that their theory is radically nonconformist, encourage audience acceptance by prefacing their performances with explanations of their intentions. Thus, even their own music fails to conform to their theory.", "question": "Which one of the following, if assumed, enables the argument's conclusion to be properly drawn?", "answers": "['Music that opposes current popular conceptions of music is less likely to be enjoyed by audiences than is music that accords with such conceptions.', 'Musicians whose music has no meaning do not preface their performances with explanations of their intentions.', 'The fact that music is distinguishable from a random series of sounds only when it has meaning makes music with meaning more appealing to audiences than music without meaning.', 'The human ability to think symbolically and to invest anything with meaning makes it very difficult to create music that has no meaning.']", "label": 1 }, { "id": "train_3957", "context": "Our cabbage soup provides good nutrition; a warm bowl of it contains more units of vitamin C than does a serving of mango or fresh broccoli!", "question": "The advertisement is misleading if which one of the following is true?", "answers": "['Cabbage soup contains important nutrients other than vitamin C.', 'The amount of vitamin C provided by a serving of the advertised soup is less than the amount furnished by a serving of fresh apples.', 'Few people depend exclusively on mango and broccoli to supply vitamin C to their diets.', 'Mango and fresh broccoli are widely known to be nutritious, but their contribution consists primarily in providing a large amount of vitamin A, not a large amount of vitamin C.']", "label": 3 }, { "id": "train_3958", "context": "Enrique: The city' s transit authority does not have enough money to operate for the next twelve months without cutting service or increasing fares, and the federal government has so far failed to provide additional fending. Nonetheless, the transit authority should continue operating without service cuts or fare increases until it has exhausted its funds. At that point, the federal government will be willing to provide funding to save the authority. Cynthia: If the transit authority tries that maneuver, the federal government will probably just Jet the authority go out of business. The transit authority cannot risk allowing that to happen.", "question": "The dialogue most strongly supports the claim that Enrique and Cynthia disagree over whether", "answers": "['the federal government is willing to provide additional funding to the transit authority now', 'the transit authority should continue operating without cutting service or increasing fares until it has exhausted its funds', 'the transit authority can afford to operate for the next twelve months without cutting service even if it does not receive additional funding', 'the federal government should provide additional funding to the transit authority']", "label": 1 }, { "id": "train_3959", "context": "Economist: Tropicorp, which constantly seeks profitable investment opportunities, has been buying and clearing sections of tropical forest for cattle ranching, although pastures newly created there become useless for grazing after just a few years. The company has not gone into rubber tapping, even though greater profits can be made from rubber tapping, which leaves the forest intact. Thus, some environmentalists argue that Tropicorp's actions do not serve even its own economic interest. However, the initial investment required for a successful rubber-tapping operation is larger than that needed for a cattle ranch; there is a shortage of workers employable in rubber-tapping operations; and taxes are higher on profits from rubber tapping than on profits from cattle ranching. Consequently, the environmentalists' conclusion is probably wrong. ", "question": "In the economist's argument, the two boldface portions play which of the following roles?", "answers": [ "The first states the conclusion of the economists' argument: the second supports that conclusion.", "The first supports the conclusion of the economists' argument, the second calls that conclusion into question.", "Each supports the conclusion of the economist's argument.", "The first states the conclusion of the environmentalists' argument; the second states the conclusion of the economist's argument" ], "label": 3 }, { "id": "train_3960", "context": "Speaker: Contemporary business firms need to recognize that avoiding social responsibility leads to the gradual erosion of power. This is Davis and Blomstrom' s Iron Law of Responsibility: \"In the long run, those who do not use power in a manner which society considers responsible will tend to lose it. \" The law' s application to human institutions certainly stands confirmed by history. Though the \"long run\" may require decades or even centuries in some instances, society ultimately acts to reduce power when society thinks it is not being used responsibly. Ttherefore, a business that wishes to retain its power as long as it can must act responsibly.", "question": "Which one of the following statements, if true, most weakens the speaker's argument?", "answers": "['Government institutions are as subject to the Iron Law of Responsibility as business institutions.', 'Since no institution is eternal, every business will eventually fail.', 'Public relations programs can cause society to consider an institution socially responsible even when it is not.', 'The power of some institutions erodes more slowly than the power of others, whether they are socially responsible or not.']", "label": 2 }, { "id": "train_3961", "context": "Marketing consultant: Last year I predicted that LRG' s latest advertising campaign would be unpopular with customers and ineffective in promoting new products. But LRG ignored my predictions and took the advice of a competing consultant. This season' s sales figures show that sales are down and LRG' s new products are selling especially poorly. Thus, the advertising campaign was ill conceived.", "question": "The marketing consultant's reasoning is most vulnerable to criticism on the grounds that", "answers": "['it confuses a condition necessary for increasing product sales with a condition that will ensure increased sales', \"it takes for granted that LRG's sales would not have been lower still in the absence of the competitor's advertising campaign\", \"it fails to consider that economic factors unrelated to the advertising campaign may have caused LRG's low sales figures\", \"it takes for granted that in LRG's industry, new products should outsell established products\"]", "label": 2 }, { "id": "train_3962", "context": "Prominent business executives often play active roles in United States presidential campaigns as fund-raisers or backroom strategists, but few actually seek to become president themselves. Throughout history the great majority of those who have sought to become president have been lawyers, military leaders, or full-time politicians. This is understandable, for the personality and skills that make for success in business do not make for success in politics. Business is largely hierarchical, whereas politics is coordinative. As a result, business executives tend to be uncomfortable with compromises and power-sharing, which are inherent in politics.", "question": "Which one of the following, if true, most seriously weakens the proposed explanation of why business executives do not run for president?", "answers": "['Many of the most active presidential fund-raisers and backroom strategists are themselves politicians.', 'Some of the skills needed to become a successful lawyer are different from some of those needed to become a successful military leader.', 'Some hierarchically structured companies have been major financial supporters of candidates for president.', 'Military leaders are generally no more comfortable with compromises and power-sharing than are business executives.']", "label": 3 }, { "id": "train_3963", "context": "It has been claimed that television networks should provide equal time for the presentation of opposing views whenever a television program concerns scientific issues -- such as those raised by the claims of environmentalists -- about which people disagree. However, although an obligation to provide equal time does arise in the case of any program concerning social issues, it does so because social issues almost always have important political implications and seldom can definitely be settled on the basis of available evidence. If a program concerns scientific issues, that program gives rise to no such equal time obligation.", "question": "Which one of the following, if true, most seriously weakens the argument?", "answers": "['Some social issues could be definitely settled on the basis of evidence if the opposing sides would give all the available evidence a fair hearing.', 'Many scientific issues have important political implications and cannot be definitely settled on the basis of the available evidence.', 'No scientific issues raised by the claims of environmentalists have important political implications.', 'There are often more than two opposing views on an issue that cannot be definitely settled on the basis of available evidence.']", "label": 1 }, { "id": "train_3964", "context": "The prehistoric fish Tiktaalik is the earliest known animal with fingers. Since variations were so great among prehistoric fish species, Tiktaalik would not have stood out as unusual at the time. However, Tiktaalik' s fingers were an important development in animal evolution because it is likely that Tiktaalik is an ancestor to the many land animals with fingers.", "question": "The statements above, if true, most strongly support which one of the following?", "answers": "['Tiktaalik is not the ancestor of any currently surviving fish species.', \"Tiktaalik's fingers were its only feature to play a significant role in the development of modern land animals.\", 'The evolutionary significance of Tiktaalik could not be determined just through comparison to fish species of its time.', 'No fish without fingers would ever be able to move on land.']", "label": 2 }, { "id": "train_3965", "context": "Exposure to a large dose of something that causes bodily damage -- such as excessive heat, poison, or nuclear radiation -- is of course harmful to an organism. But, surprisingly, exposure to small doses of such stressors has been shown to extend life span in various species, including fruit flies, protozoans, worms, and rodents.", "question": "Which one of the following, if true, most helps to explain the surprising phenomenon described above?", "answers": "['In most of the species in which exposure to small doses of stressors increases longevity, the increase is so small that it is barely measurable.', 'Repeated exposure to a stressor is much more likely than a single exposure to cause permanent damage to an organism.', 'Exposure to a given dose of a poison or other stressor may cause more serious damage to some members of a species than to others.', \"Exposure to small doses of stressors stimulates an organism's natural repair mechanisms to fix any damage caused by the stressors as well as some unrelated damage.\"]", "label": 3 }, { "id": "train_3966", "context": "Editorial: An arrest made by a Midville police officer is provisional until the officer has taken the suspect to the police station and the watch commander has officially approved the arrest. Such approval is denied if the commander judges that the evidence on which the provisional arrest is based is insufficient. A government efficiency expert has observed that almost all provisional arrests meet the standards for adequacy of evidence that the watch commanders enforce . The expert has ttherefore recommended that because the officers' time spent obtaining approval is largely wasted , the watch commander' s approval no longer be required. This recommendation should be rejected as dangerous, however, since there is no assurance that the watch commanders' standards will continue to be observed once approval is no longer required.", "question": "In the editorial, the two portions in boldface play which of the following roles?", "answers": "['The first is an observation that the editorial disputes; the second is a conclusion that was drawn from that observation.', 'The first is a finding that was used in support of a proposal that the editorial opposes; the second is a judgment that was based on that finding and in turn was used to support the proposal.', 'The first is a conclusion, the evidence for which the editorial evaluates; the second is part of the evidence cited in favor of that conclusion.', 'The first is a finding introduced to support the main conclusion of the editorial; the second is that main conclusion.']", "label": 1 }, { "id": "train_3967", "context": "Suppose I have promised to keep a confidence and someone asks me a question that I cannot answer truthfully without thereby breaking the promise. Obviously, I cannot both keep and break the same promise. Ttherefore, one cannot be obliged both to answer all questions truthfully and to keep all promises.", "question": "Which one of the following arguments is most similar in its reasoning to the argument above?", "answers": "['It is claimed that we have the unencumbered right to say whatever we want. It is also claimed that we have the obligation to be civil to others. But civility requires that we not always say what we want. So, it cannot be true both that we have the unencumbered right to say whatever we want and that we have the duty to be civil.', 'Some politicians could attain popularity with voters only by making extravagant promises; this, however, would deceive the people. So, since the only way for some politicians to be popular is to deceive, and any politician needs to be popular, it follows that some politicians must deceive.', 'If we extend our business hours, we will either have to hire new employees or have existing employees work overtime. But both new employees and additional overtime would dramatically increase our labor costs. We cannot afford to increase labor costs, so we will have to keep our business hours as they stand.', 'If we put a lot of effort into making this report look good, the client might think we did so because we believed our proposal would not stand on its own merits. On the other hand, if we do not try to make the report look good, the client might think we are not serious about her business. So, whatever we do, we risk her criticism.']", "label": 0 }, { "id": "train_3968", "context": "Consumer advocate: There is no doubt that the government is responsible for the increased cost of gasoline, because the government' s policies have significantly increased consumer demand for fuel, and as a result of increasing demand, the price of gasoline has risen steadily.", "question": "Which one of the following is an assumption required by the consumer advocate's argument?", "answers": "['If the government pursues policies that do not increase the demand for fuel, gasoline prices tend to remain stable.', 'The government has an obligation to ensure that demand for fuel does not increase excessively.', 'The government can bear responsibility for that which it indirectly causes.', 'Consumer demand for gasoline cannot increase without causing gasoline prices to increase.']", "label": 2 }, { "id": "train_3969", "context": "Willett: Lopez and Simmons, a married couple, have both been offered jobs at Evritech Corporation. Because Evritech has a rule against hiring more than one member of the same family, Lopez and Simmons have decided to reveal their marriage to Evritech. Their decision is foolish, however, since it will mean that one of them will have a job offer withdrawn. After all, they could easily keep their marriage secret initially and, if they want, later claim to have married after they were hired: Evritech has no policy of terminating one of two employees who marry each other.", "question": "The main conclusion of Willett's argument is that", "answers": "['Lopez and Simmons should not both have applied for jobs at Evritech Corporation', 'Lopez and Simmons would be unwise to reveal their marriage to Evritech Corporation without already having started to work there', \"Evritech Corporation's rule against hiring more than one member of the same family is often not enforced\", 'Evritech Corporation is not likely to discover the marital status of Lopez and Simmons if they do not volunteer the information']", "label": 1 }, { "id": "train_3970", "context": "Over the past few decades dozens of people have claimed to have sighted the Yeti in the Himalayas. This provides strong evidence that the creature exists.", "question": "The reasoning in the argument is questionable because the argument fails to", "answers": "['account for why most people still do not believe in the Yeti', 'consider the absence of photographs of the Yeti', 'evaluate historical evidence for the existence of the Yeti', 'consider alternative explanations for the reported sightings']", "label": 3 }, { "id": "train_3971", "context": "Neuroscientists subjected volunteers with amusia -- difficulty telling different melodies apart and remembering simple tunes -- to shifts in pitch comparable to those that occur when someone plays one piano key and then another. The volunteers were unable to discern a difference between the tones. But the volunteers were able to track timed sequences of musical tones and perceive slight changes in timing.", "question": "The statements above, if true, most strongly support which one of the following hypotheses?", "answers": "['Amusia results more from an inability to discern pitch than from an inability to discern timing.', 'The ability to tell melodies apart depends on the discernment of pitch alone and not at all on the perception of timing.', 'People who are unable to tell pitches apart in isolation are able to do so in the context of a melody by relying upon timing.', 'Whereas perception of timing can apparently be learned, discernment of pitch is most likely innate.']", "label": 0 }, { "id": "train_3972", "context": "The most reliable way to detect the presence of life on a planet would be by determining whether or not its atmosphere contains methane. This is because methane completely disappears from a planet' s atmosphere through various chemical reactions unless it is constantly replenished by the biological processes of living beings.", "question": "Which one of the following statements, if true, most seriously weakens the argument?", "answers": "['Not all living beings have the ability to biologically produce methane.', 'Earth is the only planet whose atmosphere is known to contain methane.', 'There are other ways of detecting the presence of life on a planet.', 'Some living beings biologically produce only very small amounts of methane.']", "label": 0 }, { "id": "train_3973", "context": "When mercury-vapor streetlights are used in areas inhabited by insect-eating bats, the bats feed almost exclusively around the lights, because the lights attract flying insects. In Greenville, the mercury-vapor streetlights are about to be replaced with energy-saving sodium streetlights, which do not attract insects. This change is likely to result in a drop in the population of insect-eating bats in Greenville, since __", "question": "Which of the following most logically completes the argument below?", "answers": "['bats use echolocation to catch insects and ttherefore gain no advantage from the fact that insects flying in the vicinity of streetlights are visible at night', 'in the absence of local concentrations of the flying insects on which bats feed, the bats expend much more energy on hunting for food, requiring much larger quantities of insects to sustain each bat', 'the bats do not begin to hunt until after sundown', 'the bats are unlikely to feed on insects that do not fly']", "label": 1 }, { "id": "train_3974", "context": "Nicotine has long been known to cause heart attacks and high blood pressure. Yet a recent study has shown that the incidence of heart attacks and high blood pressure is significantly higher among cigarette smokers who do not chew tobacco than among nonsmokers exposed to an equal amount of nicotine through tobacco chewing.", "question": "Which one of the following, if true, helps LEAST to resolve the apparent discrepancy described above?", "answers": "['Chemicals other than nicotine present in chewing tobacco but not present in cigarette smoke can cause cancer.', 'People who smoke but do not chew tobacco tend to exercise less than those who chew tobacco but do not smoke.', 'Chemicals other than nicotine present in cigarette smoke but not present in chewing tobacco raise blood pressure.', 'Chemicals other than nicotine present in chewing tobacco but not present in cigarette smoke mitigate the effects that nicotine has on the cardiovascular system.']", "label": 0 }, { "id": "train_3975", "context": "X: Since many chemicals useful for agriculture and medicine derive from rare or endangered plant species, it is likely that many plant species that are now extinct could have provided us with substances that would have been a boon to humanity. Ttherefore, if we want to ensure that chemicals from plants are available for use in the future, we must make more serious efforts to preserve for all time our natural resources. Y: But living things are not our \"resources. \" Yours is a selfish approach to conservation. We should rather strive to preserve living species because they deserve to survive, not because of the good they can do us.", "question": "Which one of the following is an issue about which X and Y disagree?", "answers": "['whether the cost of preserving plant species outweighs the cost of artificially synthesizing chemicals that could otherwise be derived from those species', 'whether it is prudent to conserve natural resources', 'whether the benefits humans derive from exploiting nonhuman species provide a good reason for preserving nonhuman species', 'whether humans should make efforts to prevent the extinction of living species']", "label": 2 }, { "id": "train_3976", "context": "Speaker: Like many contemporary critics, Smith argues that the true meaning of an author' s statements can be understood only through insight into the author' s social circumstances. But this same line of analysis can be applied to Smith' s own words. Thus, if she is right we should be able, at least in part, to discern from Smith' s social circumstances the \"true meaning\" of Smith' s statements. This, in turn, suggests that Smith herself is not aware of the true meaning of her own words.", "question": "The speaker's main conclusion logically follows if which one of the following is assumed?", "answers": "['There is just one meaning that Smith intends her work to have.', \"Smith's theory about the relation of social circumstances to the understanding of meaning lacks insight.\", 'Smith lacks insight into her own social circumstances.', \"The intended meaning of an author's work is not always good evidence of its true meaning.\"]", "label": 2 }, { "id": "train_3977", "context": "A local television station is considering a plan to create a panel of child psychologists to review programs in advance of their airing and rate the level of violence. A program that portrays a high level of violence would be listed in newspapers with four guns after the title. On the other hand, if a show has little violence, one gun would appear after its listing. The station believes that this remedy would forewarn parents about the level of violence in any given program.", "question": "Which one of the following must the television station assume in order to conclude that the plan will meet its stated purpose?", "answers": "['There would be fewer shows rated with one gun than with four guns.', 'Parents would read and pay attention to the ratings listed in the newspapers.', 'The local television station has an obligation to forewarn parents of the level of violence in television shows.', 'The rating system described in the passage is the most effective system available.']", "label": 1 }, { "id": "train_3978", "context": "Science cannot adequately explain emotional phenomena such as feeling frustrated, falling in love, or being moved by a painting. Since they cannot be explained by physics, chemistry, or neurophysiology, human emotions must not be physical phenomena.", "question": "The conclusion follows logically if which one of the following is assumed?", "answers": "['Nothing that can be felt by only one subject can be studied scientifically.', 'Every physical phenomenon can be explained by physics, chemistry, or neurophysiology.', 'Whatever is not a physical phenomenon is an emotional one.', 'Whatever is not a physical phenomenon cannot be explained by science.']", "label": 1 }, { "id": "train_3979", "context": "More and more law firms specializing in corporate taxes are paid on a contingency-fee basis. Under this arrangement, if a case is won, the firm usually receives more than it would have received if it had been paid on the alternate hourly rate basis. If the case is lost, the firm receives nothing. Most firms are likely to make more under the contingency-fee arrangement.", "question": "Which of the following, if true, would most strengthen the prediction above?", "answers": "['Since the majority of firms specialize in certain kinds of cases, they are able to assess accurately their chances of winning each potential case', 'Some litigation can last for years before any decision is reached, and, even then the decision may be appealed.', 'Firms working under the contingency-fee arrangement take in fewer cases per year than do firms working under the hourly rate arrangement.', 'Firms that work exclusively under the hourly rate arrangement spend, on average, fewer hours on cases that are won than on cases that are lost.']", "label": 0 }, { "id": "train_3980", "context": "The Great Sphinx is a huge statue in Egypt that has a lion' s body with a man' s head. The face of the Sphinx has long been claimed to be that of pharaoh Khafre, who lived around 2600 B. C. , but it cannot be: erosion patterns recently discovered on the lion' s legs can only have been caused by heavy rains, and the Sahara has not had heavy rains in over 10, 000 years.", "question": "Which of the following, if true, most seriously weakens the argument?", "answers": "['Other erosion patterns that appear on the body of the Sphinx are of a sort that could be caused by wind and sand alone', 'The face of the Sphinx bears a resemblance to the faces on certain stylized statues dating from both before and after the reign of Khafre.', 'Other than the Sphinx, there are no surviving sculptures that have been claimed to portray the face of Khafre.', 'The face of the Sphinx is small relative to the rest of the head, indicating that the face may have been recarved long after the Sphinx was built.']", "label": 3 }, { "id": "train_3981", "context": "Physician: The rise in blood pressure that commonly accompanies aging often results from a calcium deficiency. This deficiency is frequently caused by a deficiency in the active form of vitamin D needed in order for the body to absorb calcium. Since the calcium in one glass of milk per day can easily make up for any underlying calcium deficiency, some older people can lower their blood pressure by drinking milk.", "question": "The physician's conclusion is properly drawn if which one of the following is assumed?", "answers": "['Anyone who has a deficiency in the active form of vitamin D also has a calcium deficiency.', \"Older people's drinking one glass of milk per day does not contribute to a deficiency in the active form of vitamin D needed in order for the body to absorb the calcium in that milk.\", 'People who consume high quantities of calcium together with the active form of vitamin D and any other substances needed in order for the body to absorb calcium have normal blood pressure.', 'There is in milk, in a form that older people can generally utilize, enough of the active form of vitamin D and any other substances needed in order for the body to absorb the calcium in that milk.']", "label": 3 }, { "id": "train_3982", "context": "The reason J. S. Bach is remembered is not that he had a high ratio of outstanding compositions to mediocre compositions. It is rather because he was such a prolific composer. He wrote more than a thousand full-fledged compositions, so it was inevitable that some of them would be outstanding and, being outstanding, survive the ages.", "question": "Which one of the following, if true, most seriously weakens the argument?", "answers": "['Bach wrote many compositions that were considered mediocre in his lifetime, and a large proportion of these compositions have been forgotten.', \"Several of Bach's contemporaries who produced more works than he did have been largely forgotten.\", \"The exact number of Bach's compositions is not known, since many of them have been lost to posterity.\", 'There are a few highly regarded composers who wrote a comparatively small number of compositions.']", "label": 1 }, { "id": "train_3983", "context": "A company' s two divisions performed with remarkable consistency over the past three years: in each of those years, the pharmaceuticals division has accounted for roughly 20 percent of dollar sales and 40 percent of profits, and the chemicals division for the balance.", "question": "Regarding the past three years, which of the following can properly be inferred from the statement above?", "answers": "[\"The product mix offered by each of the company's divisions has remained unchanged.\", 'The chemicals division has realized lower profits per dollar of sales than has the pharmaceuticals division.', \"Total dollar sales for each of the company's divisions have remained roughly constant.\", 'The pharmaceuticals division has faced stiffer competition in its markets than has the chemicals division.']", "label": 1 }, { "id": "train_3984", "context": "Donna claims to have found the museum' s current exhibition of bronzes from Benin less interesting than many of the other exhibitions recently put on by the museum. According to all the critical reviews, however, the Benin exhibition is more interesting than any other exhibition that the museum has put on in years. Ttherefore, Donna' s claim must be false.", "question": "Which one of the following arguments contains flawed reasoning that is most similar to the flawed reasoning in the argument above?", "answers": [ "Loren claims to like the taste of the fish at the Diggin's Diner. However, since according to everyone who knows anything about food, the fish at the Diggin's Diner tastes dreadful. Loren's claim must be false.", "Alice claims to have completed her assignment. According to her supervisor, however, the task that Alice was assigned to do has not yet been completed. Alice's claim, ttherefore, must be false.", "Morris claims to have seen the famous fire at the Charles Theater. According to all the news reports, however, that fire took place in 1982 and Morris was out of the country during all of 1982. Morris's claim, ttherefore, must be false.", "Douglas claims to have climbed the tallest mountain in the country. According to the atlas, however, the county contains two other mountains that are taller than the one Douglas climbed. Douglas's claim, ttherefore, must be false." ], "label": 0 }, { "id": "train_3985", "context": "Politician: The legal right to free speech does not protect all speech. For example, it is illegal to shout \"Fire! \" in a crowded mall if the only intent is to play a practical joke; the government may ban publication of information about military operations and the identity of undercover agents; and extortion threats and conspiratorial agreements are also criminal acts. The criminalization of these forms of speech is justified, since, although they are very different from each other, they are all likely to lead directly to serious harm.", "question": "In the statements above, the politician argues that", "answers": "['a form of speech can be restricted only if it is certain that it would lead directly to serious harm', 'all but one of several possible reasons for restricting freedom of speech are unjustified', 'it is legitimate to prohibit some forms of speech on the grounds that they are likely to lead directly to serious harm', 'in all but a few cases, restricting speech eventually leads directly to serious harm']", "label": 2 }, { "id": "train_3986", "context": "Frieda: Lightning causes fires and damages electronic equipment. Since lightning rods can prevent any major damage, every building should have one. Erik: Your recommendation is pointless. It is true that lightning occasionally causes fires, but faulty wiring and overloaded circuits cause far more fires and damage to equipment than lightning does.", "question": "Erik's response fails to establish that Frieda's recommendation should not be acted on because his response", "answers": "['introduces an irrelevant comparison between overloaded circuits and faulty wiring', \"does not show that the benefits that would follow from Frieda's recommendation would be offset by any disadvantages\", 'does not offer any additional way of lessening the risk associated with lightning', \"appeals to Frieda's emotions rather than to her reason\"]", "label": 1 }, { "id": "train_3987", "context": "Editorialist: Drivers with a large number of demerit points who additionally have been convicted of a serious driving-related offense should either be sentenced to jail or be forced to receive driver re-education, since to do otherwise would be to allow a crime to go unpunished. Only if such drivers are likely to be made more responsible drivers should driver re-education be recommended for them. Unfortunately, it is always almost impossible to make drivers with a large number of demerit points more responsible drivers.", "question": "If the editorialist's statements are true, they provide the most support for which one of the following?", "answers": "['Drivers with a large number of demerit points but no conviction for a serious driving-related offense should receive driver reeducation rather than jail.', 'Drivers with a large number of demerit points who have been convicted of a serious driving-related offense should be sent to jail.', 'Driver re-education is not harsh enough punishment for anyone convicted of a serious driving-related offense who has also accumulated a large number of demerit points.', 'Driver re-education should not be recommended for those who have committed no serious driving-related offenses.']", "label": 1 }, { "id": "train_3988", "context": "The Plexis Corporation, a leading computer chip manufacturer, is currently developing a new chip, which is faster and more efficient than any computer chip currently in use. The new chip will be released for sale in twelve months. Plexis' market research has shown that initial sales of the new chip would be maximized by starting to advertise it now, but the company has decided to wait another six months before doing so.", "question": "Which of the following, if true, provides the Plexis Corporation with the best reason for postponing advertising its new chip?", "answers": "['The cost of an advertising campaign capable of maximizing initial sales of the new chip would be no greater than campaigns previously undertaken by Plexis.', \"Advertising the new chip now will significantly decrease sales of Plexis' current line of computer chips.\", \"Plexis' major rivals in the computer chip manufacturing business are developing a chip with capabilities that are comparable to those of Plexis' new chip.\", 'Some computer users are reluctant to purchase new computer products when they are first released.']", "label": 1 }, { "id": "train_3989", "context": "In the bodies of reptiles, some industrial by-products cause elevated hormonal activity. Hormones govern the development of certain body parts, and in reptiles abnormal development of these parts occurs only with elevated hormonal activity. Recently, several alligators with the telltale developmental abnormalities were discovered in a swamp. So, apparently, industrial by-products have entered the swamp' s ecosystem.", "question": "The reasoning in the argument is most vulnerable to criticism on the grounds that the argument", "answers": "['fails to say whether reptiles other than alligators were examined for the same developmental abnormalities that were discovered in the alligators', 'fails to address the possibility that industrial by-products were contained in food the alligators ate', 'provides no explanation for developmental abnormalities that do not result from elevated hormonal activity', 'fails to consider whether elevated hormonal activity can result from factors other than the presence of industrial by-products']", "label": 3 }, { "id": "train_3990", "context": "Football is unsafe regardless of the precautions undertaken or rule changes implemented. Repeated head trauma always leads to long-term head injury. Parents are already refusing to allow their children to play youth football. Eventually, nobody will want to play football due to safety concerns. Ttherefore, the NFL will dramatically decline in popularity.", "question": "Which of the following, if true, most weakens the argument?", "answers": "['Removing helmets would reduce head collisions by fifty percent.', \"The NFL's popularity will naturally decline as tastes change, like the decline in popularity of boxing and horse racing.\", 'Scientific studies of brain trauma are inconclusive as to long-term effects.', 'No matter the danger, there will always be players willing to sign waivers and play.']", "label": 3 }, { "id": "train_3991", "context": "Professor: One cannot frame an accurate conception of one' s physical environment on the basis of a single momentary perception, since each such glimpse occurs from only one particular perspective. Similarly, any history book gives only a distorted view of the past, since it reflects the biases and prejudices of its author.", "question": "The professor's argument proceeds by", "answers": "[\"making a case for the conclusion of one argument by showing that argument's resemblance to another, presumably cogent, argument\", 'attempting to show that one piece of reasoning is incorrect by comparing it with another, presumably flawed, piece of reasoning', 'arguing that because something has a certain group of characteristics, it must also have another, closely related, characteristic', 'arguing that a type of human cognition is unreliable in one instance because it has been shown to be unreliable under similar circumstances']", "label": 0 }, { "id": "train_3992", "context": "Although many customers do not make a sufficient effort to conserve water, water companies must also be held responsible for wasteful consumption. Their own policies, in fact, encourage excessive water use, and attempts at conservation will succeed only if the water companies change their practices.", "question": "Which of the following, if true, would most strongly support the view above?", "answers": "['Most water companies keep detailed records of the quantity of water used by different customers.', 'Federal authorities limit the range of policies that can be enforced by the water companies.', 'Most water companies reduce the cost per unit of water as the amount of water used by a customer increases.', 'The price per unit of water charged by the water companies has risen steadily in the last 10 years.']", "label": 2 }, { "id": "train_3993", "context": "Editorial: The city has chosen a contractor to upgrade the heating systems in public buildings. Only 40 percent of the technicians employed by this contractor are certified by the Heating Technicians Association. So the city selected a contractor 60 percent of whose technicians are unqualified, which is an outrage.", "question": "Which one of the following is an assumption required by the argument in the editorial?", "answers": "['Qualified technicians installed the heating systems to be upgraded.', 'Technicians who lack certification are not qualified technicians.', 'The contractor hired by the city has personal ties to city officials.', 'There are no contractors with fewer than 40 percent of their technicians certified.']", "label": 1 }, { "id": "train_3994", "context": "Our computer experts are asked from time to time to allocate funds for new hardware and software for our company. Unfortunately, these experts favor cutting-edge technologies, because that is what excites them, despite the fact that such experimental technologies are highly expensive, full of undiscovered \"bugs, \"and thus are not the most profitable investments.", "question": "Of the following, which one conforms most closely to the principle illustrated by the situation described above?", "answers": "['When librarians choose books for the library, they choose the kinds that they enjoy reading, not the kinds of books that serve the interests of the community.', 'When senior executives choose to promote junior executives, they tend to favor those who share their professional interests, not those who have had the most education.', 'When television executives choose programs to air, they choose the ones with the most sex and violence because that is what viewers want, not the shows with the highest artistic merit.', 'When supermarkets choose foods, they choose the kinds that can be sold for the most profit, not the kinds of foods that are the most healthful for consumers.']", "label": 0 }, { "id": "train_3995", "context": "All of the athletes lined up in the park this afternoon were waiting to participate in a charity run. Blake, Tina, and Sara had all signed up to participate in the charity run. No professional runners would sign up to participate in a charity run.", "question": "If the statements above are true, which of the following must also be true?", "answers": "['Blake, Tina, and Sara were the only athletes lined up in the park this afternoon.', 'Neither Tina nor Sara is a professional runner.', 'None of the athletes lined up in the park this afternoon had ever participated in a charity run.', 'Everyone in the park this afternoon was an amateur athlete.']", "label": 1 }, { "id": "train_3996", "context": "Farmer: Crops genetically engineered to produce toxins that enable them to resist insect pests do not need to be sprayed with insecticides. Since excessive spraying of insecticides has harmed wildlife populations near croplands, using such genetically engineered crops more widely is likely to help wildlife populations to recover.", "question": "Which one of the following is an assumption the farmer's argument requires?", "answers": "['Wildlife populations that have been harmed by the excessive spraying of insecticides on croplands are likely to recover if the amount of insecticides sprayed on those croplands is reduced even slightly.', 'Crops that have been genetically engineered to resist insect pests are never sprayed with insecticides that harm wildlife populations.', 'Use of the crops that have been genetically engineered to resist insect pests in place of crops that have been sprayed with insecticides will cause less harm to wildlife populations.', 'If a wider use of certain crops that have been genetically engineered to resist insect pests is likely to help at least some wildlife populations to recover, it is likely to have that effect only because its use will prevent excessive and ineffective spraying of insecticides on croplands.']", "label": 2 }, { "id": "train_3997", "context": "Psychologist: In a study, researchers gave 100 volunteers a psychological questionnaire designed to measure their self-esteem. The researchers then asked each volunteer to rate the strength of his or her own social skills. The volunteers with the highest levels of self-esteem consistently rated themselves as having much better social skills than did the volunteers with moderate levels. This suggests that attaining an exceptionally high level of self-esteem greatly improves one's social skills.", "question": "The psychologist's argument is most vulnerable to criticism on which of the following grounds?", "answers": "['It relies on evidence from a group of volunteers that is too small to provide any support for any inferences regarding people in general.', 'It fails to adequately address the possibility that many of the volunteers may not have understood what the psychological questionnaire was designed to measure.', 'It overlooks the possibility that people with very high levels of self-esteem may tend to have a less accurate perception of the strength of their own social skills than do people with moderate levels of self-esteem.', \"It overlooks the possibility that factors other than level of self-esteem may be of much greater importance in determining the strength of one's social skills.\"]", "label": 2 }, { "id": "train_3998", "context": "Tina: For centuries oceans and human eccentricity have been linked in the literary and artistic imagination. Such linkage is probably due to the European Renaissance practice of using ships as asylums for the socially undesirable. Sergio: No. Oceans have always been viewed as mysterious and unpredictable -- qualities that people have invariably associated with eccentricity.", "question": "Tina's and Sergio's statements lend the most support to the claim that they disagree about which one of the following statements?", "answers": "['The linkage between oceans and eccentricity predates the European Renaissance.', 'The linkage between oceans and eccentricity explains the European Renaissance custom of using ships as asylums.', 'People have never attributed the same qualities to oceans and eccentrics.', 'Oceans have always been viewed as mysterious and unpredictable.']", "label": 0 }, { "id": "train_3999", "context": "Large quantities of lead dust can be released during renovations in houses with walls painted with lead-based paint. Because the dust puts occupants at high risk of lead poisoning, such renovations should be done only in unoccupied houses by contractors who are experienced in removing all traces of lead from houses and who have the equipment to protect themselves from lead dust. Even when warned, however, many people will not pay to have someone else do renovations they believe they could do less expensively themselves. Ttherefore, Homeowners' Journal should run an article giving information to homeowners on how to reduce the risk of lead poisoning associated with do-it-yourself renovation.", "question": "Which one of the following, if true, argues most strongly against the passage's recommendation about an article?", "answers": "['Most people who undertake do-it-yourself renovation projects do so for the satisfaction of doing the work themselves and so are unlikely to hire a professional to do that sort of work.', 'Most homeowners know whether or not the walls of their houses are painted with lead-based paint, even if the walls were painted by previous owners.', 'Whenever information on do-it-yourself home renovation is published, many people who would otherwise hire professionals decide to perform the renovations themselves, even when there are risks involved.', 'When professionally done home renovations are no more expensive than do-it-yourself renovations, most people choose to have their homes renovated by professionals.']", "label": 2 }, { "id": "train_4000", "context": "Some people have been promoting a new herbal mixture as a remedy for the common cold. The mixture contains, among other things, extracts of the plants purple coneflower and goldenseal. A cold sufferer, skeptical of the claim that the mixture is an effective cold remedy, argued, \"Suppose that the mixture were an effective cold remedy. Since most people with colds wish to recover quickly, it follows that almost everybody with a cold would be using it. Ttherefore, since there are many people who have colds but do not use the mixture, it is obviously not effective. ", "question": "Each of the following is an assumption required by the skeptical cold sufferer's argument EXCEPT:", "answers": "['The mixture does not have side effects severe enough to make many people who have colds avoid using it.', 'Enough of the mixture is produced to provide the required doses to almost everybody with a cold.', 'There are no effective cold remedies available that many people who have colds prefer to the mixture.', 'The mixture is powerful enough to prevent almost everybody who uses it from contracting any further colds.']", "label": 3 }, { "id": "train_4001", "context": "Economist: In our country, the price of cola is regulated by the Cola Commission. Decades ago, when the commission was formed, such regulation was reasonable because there was a monopoly on cola production. Now, however, fierce competition in this market keeps cola prices low. So the Cola Commission should be abolished.", "question": "The economist's reasoning is most vulnerable to criticism on the grounds that it fails to consider the possibility that", "answers": "['many dedicated and talented people serve on the Cola Commission', \"ending regulation of cola prices in the economist's country will have no effect on these prices, which are already below the ceilings set by the Cola Commission\", \"the Cola Commission regulates aspects of the cola industry other than cola's price\", 'the Cola Commission was originally set up by economists']", "label": 2 }, { "id": "train_4002", "context": "Psychology professor: Applied statistics should be taught only by the various social science departments. These departments can best teach their respective students which statistical methodologies are most useful for their discipline, and how best to interpret collected data and the results of experiments. Mathematics professor: I disagree. My applied statistics course covers much of the same material taught in the applied statistics courses in social science departments. In fact, my course uses exactly the same textbook as those courses!", "question": "Which one of the following most accurately describes a questionable aspect of the reasoning in the mathematics professor's response to the psychology professor?", "answers": "['The response gives no evidence for its presumption that students willing to take a course in one department would choose a similar course in another.', 'The response gives no evidence for its presumption that social science students should have the same competence in statistics as mathematics students.', \"The response does not effectively address a key reason given in support of the psychology professor's position.\", 'The response takes for granted that unless the course textbook is the same the course content will not be the same.']", "label": 2 }, { "id": "train_4003", "context": "Each of the candidates in this year' s mayoral election is a small-business owner. Most small-business owners are competent managers. Moreover, no competent manager lacks the skills necessary to be a good mayor. So, most of the candidates in this year' s mayoral election have the skills necessary to be a good mayor.", "question": "The pattern of flawed reasoning in which one of the following is most similar to that in the argument above?", "answers": "[\"Everything on the menu at Maddy's Shake Shop is fat-free. Most fat-free foods and drinks are sugar-free. And all sugar-free foods and drinks are low in calories. Hence, most items on the menu at Maddy's are low in calories.\", \"Anyone who has worked in sales at this company has done so for at least a year. Most of this company's management has worked in its sales department. So, since no one who has worked in the sales department for more than a year fails to understand marketing, most of this company's upper management understands marketing.\", 'All of the bicycle helmets sold in this store have some plastic in them. Most of the bicycle helmets sold in this store have some rubber in them. So, since no helmets that have rubber in them do not also have plastic in them, it follows that most of the helmets in this store that have plastic in them have rubber in them.', \"Each of the avant-garde films at this year's film festival is less than an hour long. Most films less than an hour long do not become commercially successful. So, since no movie less than an hour long has an intermission, it follows that most of the movies at this year's film festival do not have an intermission.\"]", "label": 0 }, { "id": "train_4004", "context": "Lyme disease is caused by a bacterium transmitted to humans by deer ticks. Generally, deer ticks pick up the bacterium while in the larval stage by feeding on infected white-footed mice. However, certain other species on which the larvae feed do not harbor the bacterium. If the population of these other species were increased, more of the larvae would be feeding on uninfected hosts, so the number of ticks acquiring the bacterium would likely decline.", "question": "Which of the following would it be most important to ascertain in evaluating the argument?", "answers": "['Whether populations of the other species on which deer tick larvae feed are found only in areas also inhabited by white-footed mice', \"Whether the size of the deer tick population is currently limited by the availability of animals for the tick's larval stage to feed on\", 'Whether the infected deer tick population could be controlled by increasing the number of animals that prey on white-footed mice', 'Whether deer ticks that were not infected as larvae can become infected as adults by feeding on deer on which infected deer ticks have fed']", "label": 1 }, { "id": "train_4005", "context": "Although instinct enables organisms to make complex responses to stimuli, instinctual behavior involves no reasoning and requires far fewer nerve cells than does noninstinctual (also called flexible) behavior. A brain mechanism capable of flexible behavior must have a large number of neurons, and no insect brain has yet reached a size capable of providing a sufficiently large number of neurons.", "question": "Which one of the following can be properly inferred from the statements above?", "answers": "['All organisms with large brains are biologically equipped for flexible behavior.', 'Only organisms with brains of insect size or smaller engage in purely instinctual behavior.', 'Insect behavior is exclusively instinctual.', \"All organisms with brains larger than insects' brains are capable of some measure of flexible behavior.\"]", "label": 2 }, { "id": "train_4006", "context": "Years ago, consumers in Frieland began paying an energy tax in the form of two Frieland pennies for each unit of energy consumed that came from nonrenewable sources. Following the introduction of this energy tax, there was a steady reduction in the total yearly consumption of energy from nonrenewable sources.", "question": "If the statements in the passage are true, then which of the following must on the basis of them be true?", "answers": "['There was a steady decline in the total amount of energy consumed each year in Frieland.', 'There was a steady decline in the yearly revenues generated by the energy tax in Frieland.', 'There was a steady increase in the use of renewable energy source in Frieland.', 'The use of renewable energy sources in Frieland greatly increased relative to the use of nonrenewable energy sources.']", "label": 1 }, { "id": "train_4007", "context": "Most industrialized nations face the prospect of losing most of their manufacturing jobs to less-developed nations within the next few decades. Many of these nations will have a number of idle manufacturing facilities that will be unusable for other purposes. To combat this future job loss and waste of resources, industrialized nations have to train their workers to be more productive.", "question": "Which one of the following would most strengthen the above argument?", "answers": "['If workers were more productive in industrialized nations, manufacturers would not move their jobs to less-developed nations.', 'If workers were trained in other fields, there would be no need to try to maintain manufacturing jobs in industrialized nations.', 'Manufacturing jobs are important for industrialized nations.', 'Many manufacturing workers will be unable to make as much money in other fields.']", "label": 0 }, { "id": "train_4008", "context": "P: Complying with the new safety regulations is useless. Even if the new regulations had been in effect before last year' s laboratory fire, they would not have prevented the fire or the injuries resulting from it because they do not address its underlying causes. Q: But any regulations that can potentially prevent money from being wasted are useful. If obeyed, the new safety regulations will prevent some accidents, and whenever there is an accident here at the laboratory, money is wasted even if no one is injured.", "question": "A point at issue between P and Q is whether", "answers": "['accidents at the laboratory inevitably result in personal injuries', 'the new safety regulations are likely to be obeyed in the laboratory', \"the new safety regulations address the underlying cause of last year's fire\", 'it is useful to comply with the new safety regulations']", "label": 3 }, { "id": "train_4009", "context": "The master plan for the new park calls for the planting of trees of any species native to this area, except for those native trees that grow to be very large, such as the cottonwood. The trees that the community group donated were purchased at Three Rivers Nursery, which sells mostly native trees and shrubs. Thus, the donated trees are probably consistent with the master plan.", "question": "Which one of the following, if true, most strengthens the argument?", "answers": "['Three Rivers Nursery sells cottonwood trees.', 'Three Rivers Nursery does not sell any tree species that grow to be very large.', 'Some tree species that grow to be very large are consistent with the master plan.', 'Tree species that are not native to this area and that are consistent with the master plan are rare and hard to find.']", "label": 1 }, { "id": "train_4010", "context": "SafeZone mosquito repellant has been shown in laboratory settings to be effective for twelve hours against the female anopheles mosquito, the only mosquito that transmits malaria. SafeZone, however, is not effective against mosquitoes that do not transmit malaria. The only instance in which SafeZone does not repel the female anopheles mosquito is when this mosquito can detect any ruptured blood vessels, a state that inevitably results whenever any kind of mosquito bites a person. Ttherefore, assuming one does not have any ruptured blood vessels on their skin before applying SafeZone, that person will not be able to catch malaria for up to twelve hours.", "question": "Which of the following, if true, would argue most against the use of SafeZone in areas in which malaria is endemic?", "answers": "['SafeZone mosquito repellant has been shown to cause irritation that may result in a mild rash.', 'The male Anopheles mosquito also feeds on humans and is active at the same time as the female.', 'Once a person has contracted malaria, he or she may experience ruptured blood vessels in the form of bleeding gums and a bloody nose.', 'The culex mosquito, a different species of mosquito, is found in some areas in which the anopheles mosquito is found.']", "label": 1 }, { "id": "train_4011", "context": "Pediatrician: Child psychologists recommend that mothers of one-year-olds not only name objects in the immediate environment, but that they include the color of the object, when applicable. For instance, if a mother hands her child a red ball, she should say, \"This is a red ball. \" Nonetheless, even if mothers began to name the color, instead of simply saying, \"This is a ball, \" no real benefit will be conferred to the child, because studies have shown that children under 18-months old do not grasp the notion of color.", "question": "The pediatrician's argument is most vulnerable to which of the following?", "answers": "['Psychologists have conducted studies showing that most children under the age of 30 months often utter the names of colors even when that color is not present in their environments.', 'Infants who hear words embedded in sentences that have each part of speech present tend to speak sooner than those infants who do not.', 'Children usually benefit from having toys in their environments.', 'The range at which children utter their first sentence typically ranges from 15-months to 30-months.']", "label": 1 }, { "id": "train_4012", "context": "If a corporation obtains funds fraudulently, then the penalty should take into account the corporation' s use of those funds during the time it held them. In such cases, the penalty should completely offset any profit the corporation made in using the funds.", "question": "Which one of the following conforms most closely to the principle illustrated above?", "answers": "['If someone is sentenced to perform community service, the court has a responsibility to ensure that the community at large rather than a private group benefits from that service.', \"If a convicted criminal writes a memoir describing the details of that criminal's crime, any proceeds of the book should be donated to a charity chosen by a third party.\", 'If a driver causes an accident because the automobile being driven was not properly maintained, that driver should be required from then on to regularly demonstrate that his or her automobile is being properly maintained.', 'If a factory is found to have been recklessly violating pollution laws, that factory should be required to make the expenditures necessary to bring it into compliance with those laws to the satisfaction of the regulators.']", "label": 1 }, { "id": "train_4013", "context": "Two crucial claims of relativity theory can be directly confirmed. Utilizing elementary particles in high-energy accelerators, we can demonstrate that at high velocities objects are subject to time dilation and an increase in mass. However, a third claim of the theory, the Lorentz contraction thesis, which is connected to the other two, is not directly confirmable. But the fact that the theory in general is supported by experimental results indirectly confirms the contraction thesis.", "question": "Which one of the following most accurately expresses the principle underlying the reasoning above?", "answers": "['Any theory that is not inconsistent with experimental results is acceptable.', 'One must confirm all the theses of a theory before accepting the theory generally.', 'The theses of a physical theory that cannot be confirmed by observable phenomena can only be confirmed indirectly.', 'A thesis that is related to other theses can be indirectly confirmed by the direct verification of the others.']", "label": 3 }, { "id": "train_4014", "context": "Zoologists warn of an imminent surge in the number of bird species that will become extinct within this century. Nevertheless, these zoologists are wrong. One need only consider the information gathered on native North American raptors, such as bald eagles and peregrine falcons. Several of these species came close to vanishing between 1900 and 1970, but since 1970, the local populations of these raptors have rebounded.", "question": "The answer to which of the following questions provides information that would be most helpful in evaluating the argument above?", "answers": "['What special efforts, if any, have been made to rescue native North American raptors since 1970?', 'To what degree have native North American raptors migrated to other parts of the world?', 'How many species of non-native raptors have been introduced into North America since 1970?', \"How many years' experience do the zoologists have in evaluating patterns of extinction among animals?\"]", "label": 0 }, { "id": "train_4015", "context": "Consumer advocate: One advertisement that is deceptive, and thus morally wrong, states that \"gram for gram, the refined sugar used in our chocolate pies is no more fattening than the sugars found in fruits and vegetables. \" This is like trying to persuade someone that chocolate pies are not fattening by saying that, calorie for calorie, they are no more fattening than celery. True, but it would take a whole shopping cart full of celery to equal a chocolate pie' s worth of calories. Advertiser: This advertisement cannot be called deceptive. It is, after all, true.", "question": "Which one of the following principles, if established, would do most to support the consumer advocate's position against the advertiser's response?", "answers": "['A true statement should be regarded as deceptive if it is made with the expectation that people hearing or reading the statement will draw a false conclusion from it.', 'A true statement should be regarded as deceptive only if the person making the statement believes it to be false, and thus intends the people reading or hearing it to acquire a false belief.', \"To make statements that impart only a small proportion of the information in one's possession should not necessarily be regarded as deceptive.\", 'It is morally wrong to make a true statement in a manner that will deceive hearers or readers of the statement into believing that it is false.']", "label": 0 }, { "id": "train_4016", "context": "Political opinion and analysis outside the mainstream rarely are found on television talk shows, and it might be thought that this state of affairs is a product of the political agenda of the television stations themselves. In fact, television stations are driven by the same economic forces as sellers of more tangible goods. Because they must attempt to capture the largest possible share of the television audience for their shows, they air only those shows that will appeal to large numbers of people. As a result, political opinions and analyses aired on television talk shows are typically bland and innocuous.", "question": "An assumption made in the explanation offered by the author of the passage is that", "answers": "['there are television shows on which economic forces have an even greater impact than they do on television talk shows', 'the television talk shows of different stations resemble one another in most respects', 'each television viewer holds some opinion that is outside the political mainstream, but those opinions are not the same for everyone', 'there are television viewers who might refuse to watch television talk shows that they knew would be controversial and disturbing']", "label": 3 }, { "id": "train_4017", "context": "In an experiment designed to show how life may have begun on Earth, scientists demonstrated that an electrical spark -- or lightning -- could produce amino acids, the building blocks of Earth' s life. However, unless the spark occurs in a \"reducing\" atmosphere, that is, one rich in hydrogen and lean in oxygen, amino acids do not form readily and tend to break apart when they do form. Scientists now believe that Earth' s atmosphere was actually rich in oxygen and lean in nitrogen at the time life began.", "question": "Assuming that the scientists' current belief about Earth's atmosphere at the time life began is correct, which one of the following, if true, would most help to explain how lightning could have produced the first amino acids on Earth?", "answers": "['Meteorite impacts at the time life began on Earth temporarily created a reducing atmosphere around the impact site.', 'A single amino acid could have been sufficient to begin the formation of life on Earth.', 'Lightning was less common on Earth at the time life began than it is now.', \"Earth's atmosphere has changed significantly since fife first began.\"]", "label": 0 }, { "id": "train_4018", "context": "Gartland has long been narrowly self-sufficient in both grain and meat. However, as per capita income in Gartland has risen toward the world average, per capita consumption of meat has also risen toward the world average, and it takes several pounds of grain to produce one pound of meat. Ttherefore, since per capita income continues to rise, whereas domestic grain production will not increase, Gartland will soon have to import either grain or meat or both.", "question": "Which of the following is an assumption on which the argument depends?", "answers": "[\"The population of Gartland has remained relatively constant during the country's years of growing prosperity.\", 'The per capita consumption of meat in Gartland is roughly the same across all income levels.', 'In Gartland, neither meat nor grain is subject to government price controls.', 'People in Gartland who increase their consumption of meat will not radically decrease their consumption of grain.']", "label": 3 }, { "id": "train_4019", "context": "After a major toll highway introduced a system of electronic toll paying, delays at all of its interchanges declined significantly. Travel time per car trip decreased by an average of 10 percent. Tailpipe pollution for each trip decreased commensurately. Despite this, the total air pollution from vehicles on that highway did not decrease measurably.", "question": "Which one of the following, if true, most helps to resolve the apparent discrepancy in the information above?", "answers": "[\"Even after the switch to electronic toll paying, there were sometimes long delays at the highway's interchanges.\", 'The prospect of faster, more convenient travel induced more drivers to use the highway.', 'Travel time on the highway for car trips under 30 kilometers (18. 6 miles) did not decrease appreciably.', 'The highway began charging higher tolls when it switched to electronic toll paying.']", "label": 1 }, { "id": "train_4020", "context": "When amphibians first appeared on Earth millions of years ago, the amount of ultraviolet radiation penetrating Earth' s atmosphere was much greater than it is today. Ttherefore, current dramatic decreases in amphibian populations cannot be the result of recent increases in ultraviolet radiation penetrating Earth' s atmosphere.", "question": "Which of the following is an assumption on which the argument depends?", "answers": "['The skin of amphibians is generally more sensitive to ultraviolet radiation than the skin of other animals is.', 'The eggs of modern amphibians are not significantly more vulnerable to ultraviolet radiation than the eggs of the first amphibians were.', 'Modern amphibians are not as likely as the first amphibians were to live in habitats that shield them from ultraviolet radiation.', 'Populations of modern amphibians are not able to adapt to changing levels of radiation as readily as populations of early amphibians were.']", "label": 1 }, { "id": "train_4021", "context": "A provincial government plans to raise the gasoline tax to give people an incentive to drive less, reducing traffic congestion in the long term. However, skeptics point out that most people in the province live in areas where cars are the only viable transportation to jobs and stores and ttherefore cannot greatly change their driving habits in response to higher gasoline prices.", "question": "In light of the skeptics' objection, which of the following, if true, would most logically support the prediction that the government's plan will achieve its goal of reducing traffic congestion?", "answers": "['The tax will encourage many residents to switch to more fuel-efficient cars, reducing air pollution and other problems.', 'Most residents who cannot greatly change their driving habits could compensate for high gasoline prices by reducing other expenses.', 'The revenue from the tax will be used to make public transportation a viable means of transportation to jobs and stores for far more people.', 'Traffic congestion is an especially serious problem for people for whom cars are the only viable means of transportation.']", "label": 2 }, { "id": "train_4022", "context": "Super Express Shipping Company has implemented a new distribution system that can get almost every package to its destination the day after it is sent. The company worries that this more efficient system will result in lower sales of its premium next-day delivery service, because its two day service will usually arrive the following day anyway. The company plans to encourage sales of its next-day service by intentionally delaying delivery of its two-day packages so that they will not be delivered the following day, even if the package arrives at its destination city in time for next-day delivery.", "question": "The company's plan assumes that", "answers": "[\"deliberate delay of packages will not affect the company's image in a way that significantly reduces its ability to attract and retain customers\", 'if the plan is not implemented, the company would lose more money in lost sales of overnight deliveries than it would save with its new efficient distribution system', 'the overnight service is too expensive to be attractive to most customers currently', \"competing companies' delivery services rarely deliver packages to their destination earlier than their promised time\"]", "label": 0 }, { "id": "train_4023", "context": "Economist: Money, no matter what its form and in almost every culture in which it has been used, derives its value from its scarcity, whether real or perceived. Anthropologist: But cowrie shells formed the major currency in the Solomon Island economy of the Kwara' ae, and unlimited numbers of these shells washed up daily on the beaches to which the Kwara' ae had access.", "question": "Which one of the following, if true about the Kwara'ae, best serves to resolve the apparently conflicting positions cited above?", "answers": "['They accepted as money only cowrie shells that were polished and carved by a neighboring people, and such shell preparation required both time and skilled labor.', 'After Western traders brought money in the form of precious-metal coins to the Solomon Islands, cowrie-shell money continued to be used as one of the major media of exchange for both goods and services.', 'They considered porpoise teeth valuable, and these were generally threaded on strings to be worn as jewelry.', 'During festivals they exchanged strings of cowrie-shell money with each other as part of a traditional ritual that honored their elders.']", "label": 0 }, { "id": "train_4024", "context": "Scientists have long thought that omega-3 fatty acids in fish oil tend to lower blood cholesterol and strongly suspected that a diet that includes a modest amount of fish would provide substantial health benefits. Now these views have acquired strong support from a recent study showing that middleaged people who eat fish twice a week are nearly 30 percent less likely to develop heart disease than are those who do not eat fish.", "question": "Which one of the following is an assumption required by the argument?", "answers": "['The test subjects in the recent study who ate fish twice a week did not have a diet that was otherwise conducive to the development of heart disease.', 'The test subjects in the recent study who ate fish twice a week were no more likely than those who did not to have sedentary occupations.', 'The test subjects in the recent study who did not eat fish were significantly more likely to eat red meat several times per week than were those who did eat fish.', 'The test subjects in the recent study who ate fish twice a week were not significantly more likely than those who did not to engage regularly in activities known to augment cardiorespiratory health.']", "label": 3 }, { "id": "train_4025", "context": "The constitution of Country F requires that whenever the government sells a state-owned entity, it must sell that entity for the highest price it can command on the open market. The constitution also requires that whenever the government sells a state-owned entity, it must ensure that citizens of Country F will have majority ownership of the resulting company for at least one year after the sale.", "question": "The government of Country F must violate at least one of the constitutional requirements described above if it is faced with which one of the following situations?", "answers": "[\"The government has agreed to sell National Silver, a state-owned mine, to a corporation. Although citizens of Country F have majority ownership of the corporation, most of the corporation's operations and sales take place in other countries.\", 'The government will sell StateRail, a state-owned railway. The government must place significant restrictions on who can purchase StateRail to ensure that citizens of Country F will gain majority ownership. However, any such restrictions will reduce the price the government receives for StateRail.', 'The government will sell StateAir, a state-owned airline. The highest bid received was from a corporation that was owned entirely by citizens of Country F when the bid was received. Shortly after the bid was received, however, noncitizens purchased a minority share in the corporation.', \"The government will sell PetroNat, a state-owned oil company. World Oil Company has made one of the highest offers for PetroNat, but World Oil's ownership structure is so complex that the government cannot determine whether citizens of Country F have majority ownership.\"]", "label": 1 }, { "id": "train_4026", "context": "A museum director, in order to finance expensive new acquisitions, discreetly sold some paintings by major artists. All of them were paintings that the director privately considered inferior. Critics roundly condemned the sale, charging that the museum had lost first-rate pieces, thereby violating its duty as a trustee of art for future generations. A few months after being sold by the museum, those paintings were resold, in an otherwise stagnant art market, at two to three times the price paid to the museum. Clearly, these prices settle the issue, since they demonstrate the correctness of the critics' evaluation.", "question": "The reasoning in the argument is vulnerable to the criticism that the argument does which one of the following?", "answers": "['It concludes that a certain opinion is correct on the grounds that it is held by more people than hold the opposing view.', 'It bases its conclusion on facts that could, in the given situation, have resulted from causes other than those presupposed by the argument.', 'It rejects the judgment of the experts in an area in which there is no better guide to the truth than expert judgment.', 'It bases a firm conclusion about a state of affairs in the present on somewhat speculative claims about a future state of affairs.']", "label": 1 }, { "id": "train_4027", "context": "Compact discs (CDs) offer an improvement in artistic freedom over vinyl records. As the record needle moves in toward a vinyl record' s center, it must fight centrifugal force. Wide, shallow, or jagged grooves will cause the needle to jump; consequently, the song nearest the center -- the last song on the side -- cannot have especially loud, high-pitched, or low-pitched passages. The CD suffers no such limitations, leaving artists free to end recordings with any song.", "question": "Which one of the following most accurately expresses the main conclusion of the argument?", "answers": "['On vinyl records, the song farthest from the center can have loud, high-pitched, or low-pitched passages.', \"As the record needle moves in toward the vinyl record's center, the centrifugal force on the needle becomes stronger.\", 'CDs can have louder passages, as well as both higher- and lower-pitched passages, than can vinyl records.', 'CDs provide greater artistic latitude than do vinyl records.']", "label": 3 }, { "id": "train_4028", "context": "Advertisement: Auto accidents are the most common cause of whiplash injury, a kind of injury that is caused by a sudden sharp motion of the neck. However, many other types of accidents can produce a sudden sharp motion of the neck and thereby result in whiplash injury. A sudden sharp motion of the neck can be caused by a fall, a bump on the head, or even by being shoved from behind. That is why you should insist on receiving Lakeside Injury Clinic' s complete course of treatment for whiplash after any accident that involves a fall or a bump on the head.", "question": "Which one of the following, if true, provides the strongest basis for criticizing the reasoning in the advertisement?", "answers": "['Being shoved from behind rarely causes whiplash.', 'It is very uncommon for falling or being bumped on the head to result in a sudden sharp motion of the neck.', 'Auto accidents often involve falling or being bumped on the head.', 'The appropriate treatment for whiplash caused by a fall or a bump on the head is no different from that for whiplash caused by an auto accident.']", "label": 1 }, { "id": "train_4029", "context": "Computer operating system software has become increasingly standardized. But when a large business with multiple, linked computer systems uses identical operating system software on all of its computers, a computer vandal who gains access to one computer automatically has access to the data on all the computers. Using a program known as a \"virus, \" the vandal can then destroy much of the data on all the computers. If such a business introduced minor variations into its operating system software, unauthorized access to all the computers at the same time could be virtually eliminated. Furthermore, variations in operating system software can be created without any loss of computer compatibility to the business. Ttherefore, it is advisable for businesses to implement such variations.", "question": "Which one of the following, if true, supports the conclusion in the passage?", "answers": "['It is not costly for a business to maintain incompatible computer operating systems.', 'Standardization of computer operating system software has increased computer compatibility among different businesses.', 'Correcting any damage resulting from an invasion by a computer virus program is more expensive than preventing the damage.', 'Not all businesses need to share data among their internal computer systems.']", "label": 2 }, { "id": "train_4030", "context": "In some countries, there is a free flow of information about infrastructure, agriculture, and industry, whereas in other countries, this information is controlled by a small elite. In the latter countries, the vast majority of the population is denied vital information about factors that determine their welfare. Thus, these countries are likely to experience more frequent economic crises than other countries do.", "question": "The conclusion follows logically if which one of the following is assumed?", "answers": "['In nations in which the government controls access to information about infrastructure, agriculture, and industry, economic crises are common.', 'A small elite that controls information about infrastructure, agriculture, and industry is likely to manipulate that information for its own benefit.', 'It is more likely that people without political power will suffer from economic crises than it is that people in power will.', 'Economic crises become more frequent as the amount of information available to the population about factors determining its welfare decreases.']", "label": 3 }, { "id": "train_4031", "context": "Studies have shown that treating certain illnesses with treatment X produces the same beneficial changes in patients' conditions as treating the same illnesses with treatment Y. Furthermore, treatment X is quicker and less expensive than treatment Y. Thus, in treating these illnesses, treatment X should be preferred to treatment Y.", "question": "Which one of the following, if true, would most weaken the argument above?", "answers": "['Until recently, treatment X was more expensive than treatment Y.', 'Unlike treatment Y, treatment X has produced harmful side effects in laboratory animals.', 'Treatment Y is prescribed more often by physicians than treatment X.', 'A third treatment, treatment Z, is even quicker and less expensive than treatment X.']", "label": 1 }, { "id": "train_4032", "context": "If all works of art evoke intense feelings, and this sculpture is a work of art, then it follows that this sculpture evokes intense feelings. But this sculpture does not evoke intense feelings at all. So either this sculpture is not a work of art, or not all works of art evoke intense feelings.", "question": "Which one of the following arguments is most similar in its pattern of reasoning to the argument above?", "answers": "['If all vitamins are safe in large doses, and betacarotene is a vitamin, then it follows that beta-carotene is safe in large doses. But betacarotene is not safe in large doses. So not all vitamins are safe in large doses, or betacarotene is not a vitamin.', 'If all sciences rely heavily on mathematics, and clinical psychology is a science, then it follows that clinical psychology relies heavily on mathematics. But clinical psychology does not rely heavily on mathematics. So clinical psychology is not a science.', 'If all classes are canceled today, then it follows that today is a holiday and the library is closed. But today is not a holiday. So some classes are not canceled, or the library is open.', 'If all medical research is significant, and this research is medical, then it follows that this research is significant. But this research is actually of no significance. So not all medical research is significant, and this research is not medical.']", "label": 0 }, { "id": "train_4033", "context": "Clothes made from natural fibers such as cotton, unlike clothes made from artificial fibers such as polyester often shrink when washed at high temperatures. The reason for this shrinkage is that natural fibers are tightly curled in their original state. Since the manufacturer of cloth requires straight fibers, natural fibers are artificially straightened prior to being made into cloth. High temperatures cause all fibers in cloth to return to their original states.", "question": "Which one of the following is most strongly supported by the information above?", "answers": "['High temperatures have no effect on the straightness of fibers in clothes made from a blend of natural and artificial fibers.', 'Artificial fibers are straight in their original state.', 'Clothes made from natural fibers stretch more easily than do clothes made from artificial fibers.', 'If natural fibers that have been straightened and used for cloth are curled up again by high temperatures, they cannot be straightened again.']", "label": 1 }, { "id": "train_4034", "context": "Students from outside the province of Markland, who in any given academic year pay twice as much tuition each as do students from Markland, had traditionally accounted for at least two-thirds of the enrollment at Central Markland College. Over the past 10 years academic standards at the college have risen, and the proportion of students who are not Marklanders has dropped to around 40 percent.", "question": "Which one of the following can be properly inferred from the statements above?", "answers": "[\"If the college's per capita revenue from tuition has remained the same, tuition fees have increased over the past 10 years.\", 'Over the past 10 years, academic standards at Central Markland College have risen by more than academic standards at any other college in Markland.', 'Over the past 10 years, the number of students from Markland increased and the number of students from outside Markland decreased.', 'If it had not been for the high tuition paid by students from outside Markland, the college could not have improved its academic standards over the past 10 years.']", "label": 0 }, { "id": "train_4035", "context": "Country G had long had a problem with congested roads. To cut down on congestion, Country G developed a high-speed train system connecting most midsize and large cities within the country. The train system was intended to cut down on the number of people utilizing private vehicles on the country' s roads. Within one year after the train system was complete, there were approximately 100, 000 riders using the high-speed train system. The citizens of Country G were ecstatic about the success of the train system.", "question": "It can properly be inferred from the passage that Country G believes that the train system is a success", "answers": "['because trains generally arrive at their destinations on time', 'because trains generally are not successful in modern times', \"because the large number of people riding the train means that fewer people are using the country's roads\", 'because the increase in public transportation will benefit tourism throughout the country']", "label": 2 }, { "id": "train_4036", "context": "Roberta is irritable only when she is tired, and loses things only when she is tired. Since she has been yawning all day, and has just lost her keys, she is almost certainly irritable.", "question": "The reasoning above is flawed in that it", "answers": "[\"takes a necessary condition for Roberta's losing things to be a sufficient condition\", 'generalizes on the basis of a single instance', 'assumes the conclusion that it sets out to prove', \"takes a necessary condition for Roberta's being irritable to be a sufficient condition\"]", "label": 3 }, { "id": "train_4037", "context": "Although many seventeenth-century broadsides, popular ballads printed on a single sheet of paper and widely sold by street peddlers, were moralizing in nature, this is not evidence that most seventeenth-century people were serious about moral values. While over half of surviving broadsides contain moralizing statements, and it is known that many people purchased such compositions, it is not known why they did so, nor is it known how their own beliefs related to what they read.", "question": "Which one of the following, if true, most strengthens the argument?", "answers": "['In many moralizing ballads, the moral content was confined to a single stanza expressing a pious sentiment tacked onto a sensationalized account of crime and adultery.', 'Well-educated people of the seventeenth century held broadsides in contempt and considered broadside peddlers to be disreputable vagrants.', 'The clergy occasionally stuck broadsides warning about the danger of strong drink on the doors of seventeenth-century alehouses.', 'Some seventeenth-century ballad sellers also sold sermons printed in pamphlet form.']", "label": 0 }, { "id": "train_4038", "context": "Many scholars claim that Shakespeare' s portrayal of Richard III was extremely inaccurate, arguing that he derived that portrayal from propagandists opposed to Richard III. But these claims are irrelevant for appreciating Shakespeare' s work. The character of Richard III as portrayed in Shakespeare' s drama is fascinating and illuminating both aesthetically and morally, regardless of its relation to historical fact.", "question": "Which one of the following principles, if valid, most helps to justify the reasoning in the argument above?", "answers": "['History is always told by propagandists from the winning side.', \"Shakespeare's historical importance puts him beyond the scope of all literary criticism.\", 'In dealing with real people, dramatists should reflect their lives accurately.', 'In historical drama, the aesthetic value of the work is not necessarily undermined by historical inaccuracies.']", "label": 3 }, { "id": "train_4039", "context": "The mind and the immune system have been shown to be intimately linked, and scientists are consistently finding that doing good deeds benefits one' s immune system. The bone marrow and spleen, which produce the white blood cells needed to fight infection, are both connected by neural pathways to the brain. Recent research has shown that the activity of these white blood cells is stimulated by beneficial chemicals produced by the brain as a result of magnanimous behavior.", "question": "The statements above, if true, support the view that", "answers": "[\"magnanimity is beneficial to one's own interests\", 'the number of white blood cells will increase', 'magnanimous behavior can be regulated by the presence or absence of certain chemicals in the brain', 'lack of magnanimity is the cause of most serious illnesses']", "label": 0 }, { "id": "train_4040", "context": "Many winemakers use cork stoppers; but cork stoppers can leak, crumble, or become moldy, so that those winemakers must often discard a significant proportion of their inventory of bottled wine. Bottlemaster plastic stoppers, which cannot leak, crumble, or mold, have long been available to winemakers, at a price slightly higher than that of traditional cork stoppers. Cork prices, however, are expected to rise dramatically in the near future. Clearly, ttherefore, winemakers who still use cork but wish to keep production costs from rising will be forced to reconsider plastic stoppers . And since the wine-buying public' s association of plastic stoppers with poor-quality wine is weakening, there is an excellent chance that the Bottlemaster plastic stopper will gain an increased share of the marked for wine-bottle stoppers .", "question": "In the argument given, the two portions in boldface play which of the following roles?", "answers": "['The first is a judgment that has been advanced in support of a position that the argument opposes; the second is a conclusion drawn in order to support the main conclusion of the argument.', 'The first is the main conclusion of the argument; the second provides evidence in support of that main conclusion.', 'The first is a conclusion drawn in order to support the main conclusion of the argument; the second is that main conclusion.', 'The first is a judgment that has been advanced in support of a position that the argument opposes; the second is the main conclusion of the argument.']", "label": 2 }, { "id": "train_4041", "context": "Cholesterol, which is a known factor in coronary heart disease and stroke, needs a carrier, known as a lipoprotein, to transport it through the bloodstream. Low-density lipoproteins (LDLs) increase the risk of coronary heart disease and stroke, but we can tentatively conclude that high-density lipoproteins (HDLs) help prevent coronary heart disease and stroke. First, aerobic exercise increases one' s level of HDLs. Second, HDL levels are higher in women than in men. And both aerobic exercise and being female are positively correlated with lower risk of coronary heart disease and stroke.", "question": "Each of the following, if true, strengthens the argument EXCEPT:", "answers": "['A high level of HDLs mitigates the increased health risks associated with LDLs.', 'Persons who are overweight tend to have a higher risk of early death due to coronary heart disease and stroke, and tend to have low levels of HDLs.', 'HDLs, unlike LDLs, help the body excrete cholesterol.', 'HDLs are less easily removed from the bloodstream than are LDLs.']", "label": 3 }, { "id": "train_4042", "context": "Currently, the U. S. Highway Transportation Division is liable for any car accident that occurs because of a highway in need of physical upkeep or repairs. However, the U. S. highway system is so vast that it is impossible to hire a large enough road crew to locate and eliminate every potential danger in its highways. It is proposed that the Transportation Division should be liable for accidents on the highway only if they knew about the hazard beforehand and carelessly failed to manage it. Which one of the following describes any accidents for which the U. S.", "question": "Highway Transportation Department is now liable, but should not be according to the proposed change cited above?", "answers": "['A person merging onto the highway is in an accident when he hits a car that was in his blind spot.', 'A truck skids into the ditch after hitting a patch of black ice on a snowy winter day.', 'An SUV is in a rollover accident on an extremely rutted stretch of highway, and the Transportation Department had been repeatedly informed of the need to repair this stretch for several years.', 'A person who was not paying attention to road signs runs into the ditch when the highway curves sharply.']", "label": 1 }, { "id": "train_4043", "context": "At Southgate Mall, mattresses are sold only at Mattress Madness. Every mattress at Mattress Madness is on sale at a 20 percent discount. So every mattress for sale at Southgate Mall is on sale at a 20 percent discount.", "question": "Which one of the following arguments is most similar in its reasoning to the argument above?", "answers": [ "The only food in Diane's apartment is in her refrigerator. Diane purchased all the food in her refrigerator within the past week. Ttherefore, all the food she purchased within the past week is in her apartment.", "All the food in Diane's apartment is in her refrigerator. Diane purchased all the food in her refrigerator within the past week. Ttherefore, she purchased all the food in her apartment within the past week.", "The only food in Diane's apartment is in her refrigerator. All the food she purchased within the past week is in her refrigerator. Ttherefore, she purchased all the food in her apartment within the past week.", "Diane's refrigerator, and all the food in it, is in her apartment. Diane purchased all the food in her refrigerator within the past week. Ttherefore, she purchased all the food in her apartment within the past week." ], "label": 1 }, { "id": "train_4044", "context": "Certain governments subsidize certain basic agricultural products in order to guarantee an adequate domestic production of them. But subsidies encourage more intensive farming, which eventually leads to soil exhaustion and drastically reduced yields.", "question": "The situation above is most nearly similar to which one of the following situations with respect to the relationship between the declared intent of a governmental practice and a circumstance relevant to it?", "answers": "['Certain governments build strong armed forces in order to forestall armed conflict. But in order to maintain the sort of discipline and morale that keeps armed forces strong, those forces must be used in actual combat periodically.', 'Certain governments pass traffic laws in order to make travel safer. But the population-driven growth in volumes of traffic often has the effect of making travel less safe despite the passage of new traffic laws.', 'Certain governments restrict imports in order to keep domestic producers in business. But, since domestic producers do not have to face the full force of foreign competition, some domestic producers are able to earn inordinately high profits.', 'Certain governments subsidize theaters in order to attract foreign tourists. But tourists rarely choose a destination for the theatrical performances it has to offer.']", "label": 0 }, { "id": "train_4045", "context": "President of Central Supply Company: Profits are at an all-time low this fiscal year because of decreased demand for our products. If this situation continues, the company may have to declare bankruptcy. So it is important to prevent any further decrease in profits. Consequently, the only options are to reduce planned expansion or to eliminate some less profitable existing operations.", "question": "Which one of the following most accurately describes a flaw, in the company president's reasoning?", "answers": "['It presumes without giving justification that survival of the company has been a good thing.', 'It does not allow for the possibility that profits will decrease only slightly during the next fiscal year.', 'It does not take into account that there are alternatives to declaring bankruptcy.', 'It does not take into account that there may be other ways to stop the decrease in profits.']", "label": 3 }, { "id": "train_4046", "context": "A stingray without parasites is healthier than it would be if it had parasites. Nevertheless, the lack of parasites in stingrays is an indicator that the ecosystem in which the stingrays live is under environmental stress such as pollution.", "question": "Which one of the following, if true, most helps to reconcile the discrepancy indicated above?", "answers": "['During part of their life cycles, the parasites of stingrays require as hosts shrimp or oysters, which are environmentally vulnerable organisms.', 'An ecosystem can be considered stressed if only a few species of very simple organisms can live there.', 'Since the life of parasites depends on that of their host, they need to live without killing their host or else not reproduce and infect other individuals before their own host dies.', 'A parasite drains part of the vitality of its host by drawing nourishment from the host.']", "label": 0 }, { "id": "train_4047", "context": "Six years ago, the police force of Harrisville was reduced, in an effort to make a smaller but more efficient force. In the first two years, crimes in Harrisville, especially by career criminals, were at a higher rate than they had been in the previous decades. Then, for the most recent four years, the crime rate has been the lowest in the seventy years on record, with the significant drop in crimes by career criminals.", "question": "Which of the following, if true, most helps to explain the change in the crime rate between the first two years after the change and the last four years?", "answers": "['The population of Harrisville increased slightly each year over the last six years, with a large number of senior citizens moving to a substantial retirement community within the city.', 'By the end of the second year of the newly organized police force, a large proportion of career criminals operating in Harrisville had been captured and incarcerated.', 'During each of the last six year period, retention and graduation rates at the local high schools have increased.', 'Of the seventy similar sized cities in this and neighboring states, 40% reported some drop in the crime rate over the last four years.']", "label": 1 }, { "id": "train_4048", "context": "Meade: People who are injured as a result of their risky behaviors not only cause harm to themselves but, because we all have important ties to other people, inevitably impose emotional and financial costs on others. To protect the interests of others, ttherefore, governments are justified in outlawing behavior that puts one' s own health at risk.", "question": "Which one of the following principles, if valid, most undermines the reasoning in Meade's argument?", "answers": "[\"People's obligation to avoid harming others outweighs their obligation to avoid harming themselves.\", 'Preventing harm to others is not by itself a sufficient justification for laws that limit personal freedom.', \"Governments are not justified in limiting an individual's behavior unless that behavior imposes emotional or financial costs on others.\", 'People who have important ties to others have a personal obligation not to put their own health at risk.']", "label": 1 }, { "id": "train_4049", "context": "Tom: Critics of recent high court decisions claim that judges' willingness to abide by earlier decisions is necessary to avoid legal chaos. Since high courts of the past often repudiated legal precedents and no harm to the legal system ensued, these critics' objections must be politically motivated and ought to be ignored. Mary: High courts have repudiated precedents in the past, but they were careful to do so only when the previous rulings were old and had clearly become outdated. The recently overturned rulings were themselves recent. Overturning any recent legal ruling diminishes the law, which comes to be viewed as unstable and capricious.", "question": "Which one of the following most accurately expresses the point at issue between Tom and Mary?", "answers": "['whether the overturning of recent high court precedents was politically motivated', 'whether judicial decisions that seem progressive at first can quickly become outdated', 'whether the overturning of recent high court precedents will harm the legal system', 'whether critics of recent high court decisions in fact advanced the claim Tom cites']", "label": 2 }, { "id": "train_4050", "context": "Overexposure to certain wavelengths of strong sunlight is the main cause of melanoma, a virulent form of skin cancer. For this reason, doctors now urge everyone to put adequate sunblock on skin exposed to strong sunlight. Adequate sunblock, according to doctors, is any preparation that prevents sunburn even if the person is exposed to strong sunlight for a significant length of time.", "question": "Which one of the following, if true, most weakens the recommendation that people wear adequate sunblock?", "answers": "['Sunburns appear immediately after exposure to the sun but melanoma appears years after repeated exposures.', 'There is no evidence that there are wavelengths of sunlight that lead to both sunburn and melanoma.', 'Toxins contained in certain chemical compounds also cause melanoma.', 'There are people who have allergic reactions to certain chemicals found in many sunblocks.']", "label": 1 }, { "id": "train_4051", "context": "Dolphins can swim at high speeds and achieve high acceleration in the water. In 1936, Sir James Gray calculated the force dolphins should be able to exert based on their physiology. He concluded that the propulsive force they were able to exert was not enough to explain how fast they swim and accelerate. In the 2000s, experimenters used special computer-enhanced measurements of the water in which dolphins were swimming. Through mathematical modeling, they were able to measure the force dolphins exert with their tails. As it turns out, dolphins exert considerably more force with their tails than Sir James Gray or anybody else ever expected. Ttherefore, the force exerted by their tails easily explains how fast they swim and accelerate. ", "question": "In the argument, the two portions in boldface play which of the following roles?", "answers": "['The first is a generally held assumption; the second is a conclusion that violates that assumption.', 'This first is a piece of evidence supporting the main conclusion; the second is the main conclusion.', 'The first is an opinion the author seeks to refute; the second is the opinion the author supports.', 'The first is a false conclusion based on an incorrect premise; the second is the revised conclusion drawn from the corrected premise.']", "label": 3 }, { "id": "train_4052", "context": "In Kantovia, physicians' income comes from insurance companies, which require physicians to document their decisions in treating patients and to justify deviations from the companies' treatment guidelines. Ten years ago physicians were allowed more discretion. Most physicians believe that the companies' requirements now prevent them from spending enough time with patients. Yet the average amount of time a patient spends with a physician during an office visit has actually increased somewhat over the last ten years.", "question": "Which of the following, if true, most helps to resolve the apparent discrepancy between physicians' perceptions and the change in the actual time spent?", "answers": "[\"Since the insurance companies pay physicians a set amount for each office visit, it is to physicians' financial advantage to see as many patients as possible.\", 'Physicians today typically have a wider range of options in diagnosis and treatment to consider with the patient before prescribing.', 'Physicians are increasingly likely to work in group practices, sharing the responsibility of night and weekend work.', 'Most patients would rather trust their physicians than their insurance companies to make decisions about their treatment.']", "label": 1 }, { "id": "train_4053", "context": "Winds, the movement of gases in the atmosphere of a planet, are ultimately due to differences in atmospheric temperature. Winds on Earth are the result of heat from the Sun, but the Sun is much too far away from Jupiter to have any significant impact on the temperature of Jupiter' s atmosphere. Nevertheless, on Jupiter winds reach speeds many times those of winds found on Earth.", "question": "Which one of the following, if true, most helps to explain the facts cited above about Jupiter and its winds?", "answers": "['Planets even farther from the Sun than Jupiter are known to have atmospheric winds.', 'Gaseous planets such as Jupiter sometimes have stronger winds than do rocky planets such as Earth.', \"Jupiter's atmosphere is composed of several gases that are found in Earth's atmosphere only in trace amounts.\", \"Unlike Earth, Jupiter's atmosphere is warmed by the planet's internal heat source.\"]", "label": 3 }, { "id": "train_4054", "context": "Accidents involving drivers who do not wear seatbelts have been shown to cause higher rates of serious injury than accidents involving drivers who do wear seatbelts. Because drivers who do not wear seatbelts can end up costing taxpayers money in medical bills, Thoracic County should make it illegal to drive without a seatbelt.", "question": "Which of the following, if true, provides the most support for the argument above?", "answers": "['Motorcycle accidents are statistically more significant in terms of the money they cost taxpayers in Thoracic County.', 'In Thoracic County, the number of severe injuries and deaths caused by driving without a seatbelt is less than the number caused by drunk driving.', 'Within Thoracic County, the majority of citizens already wear their seatbelts.', 'On particularly dangerous roads in Thoracic County, vehicular accidents of all kinds are common.']", "label": 3 }, { "id": "train_4055", "context": "Taylor: Researchers at a local university claim that 61 percent of the information transferred during a conversation is communicated through nonverbal signals. But this claim, like all such mathematically precise claims, is suspect, because claims of such exactitude could never be established by science. Sandra: While precision is unobtainable in many areas of life, it is commonplace in others. Many scientific disciplines obtain extremely precise results, which should not be doubted merely because of their precision.", "question": "The statements above provide the most support for holding that Sandra would disagree with Taylor about which one of the following statements?", "answers": "['Research might reveal that 61 percent of the information taken in during a conversation is communicated through nonverbal signals.', 'It is possible to determine whether 61 percent of the information taken in during a conversation is communicated through nonverbal signals.', 'Some sciences can yield mathematically precise results that are not inherently suspect.', \"The study of verbal and nonverbal communication is an area where one cannot expect great precision in one's research results.\"]", "label": 2 }, { "id": "train_4056", "context": "Jake works for Bank Conglomerate of America (BCA), the largest investment bank in the United States. Jake has worked at Bank Conglomerate of America for a decade. Every American investment bank employs dozens of lawyers to defend against insider-trading allegations. Some Bank Conglomerate of America employees must pass a certification course. However, all employees must complete a mandatory class on insider trading.", "question": "If the statements above are correct, which of the following must not be true?", "answers": "['Jake took a class on insider trading.', 'Jake passed a certification course.', 'Jake never took a class on insider trading.', 'Jake has worked at Bank Conglomerate of America for a decade.']", "label": 2 }, { "id": "train_4057", "context": "If Skill' s book is published this year, Professor Nguyen vows she will urge the dean to promote Skiff. Thus, if Skiff' s book is as important and as well written as Skiff claims, he will be promoted, for Nguyen will certainly keep her promise, and the dean will surely promote Skiff if Nguyen recommends it.", "question": "The argument's conclusion can be properly inferred if which one of the following is assumed?", "answers": "['Skiff needs to publish a book before he can be promoted.', \"Skiff's book will not be published unless it is as important and as well written as he claims it is.\", 'Skiff will not be promoted unless Professor Nguyen urges the dean to do so.', \"Skiff's book will be published this year if it is as important as he claims it is.\"]", "label": 3 }, { "id": "train_4058", "context": "Scientist: Isaac Newton' s Principia, the seventeenth-century work that served as the cornerstone of physics for over two centuries, could at first be understood by only a handful of people, but a basic understanding of Newton' s ideas eventually spread throughout the world. This shows that the barriers to communication between scientists and the public are not impermeable. Thus recent scientific research, most of which also can be described only in language that seems esoteric to most contemporary readers, may also become part of everyone' s intellectual heritage.", "question": "Which one of the following most accurately describes the role played in the scientist's argument by the claim that recent scientific research can often be described only in language that seems esoteric to most contemporary readers?", "answers": "[\"It is a premise that supports the argument's main conclusion by suggesting that the results of recent scientific research are only superficially different from claims made in Newton's Principia.\", \"It is a claim that serves mainly to help establish the relevance of the preceding statements to the argument's final conclusion.\", \"It serves to cast doubt on an alleged similarity between Newton's Principia and recent scientific research.\", 'It is cited as further evidence for the conclusion that the barriers to communication between scientists and the public are not impermeable.']", "label": 1 }, { "id": "train_4059", "context": "Decentralization enables divisions of a large institution to function autonomously. This always permits more realistic planning and strongly encourages innovation, since the people responsible for decision making are directly involved in implementing the policies they design. Decentralization also permits the central administration to focus on institution-wide issues without being overwhelmed by the details of daily operations.", "question": "The statements above most strongly support which one of the following?", "answers": "['For large institutions the advantages of decentralization outweigh its disadvantages.', 'In large institutions whose divisions do not function autonomously, planning is not maximally realistic.', 'Innovation is not always encouraged in large centralized institutions.', 'The central administrations of large institutions are usually partially responsible for most of the details of daily operations.']", "label": 1 }, { "id": "train_4060", "context": "In a yearlong study, half of the participants were given a simple kit to use at home for measuring the cholesterol level of their blood. They reduced their cholesterol levels on average 15 percent more than did participants without the kit. Participants were selected at random from among people with dangerously high cholesterol levels.", "question": "Which one of the following, if true, most helps to explain the study's finding?", "answers": "['The lower a blood-cholesterol level is, the less accurate are measurements made by the kit.', 'Participants using the kit reported that each reading reinforced their efforts to reduce their cholesterol levels.', 'All the participants in the study showed some lowering of cholesterol levels, the most striking decreases having been achieved in the first three months.', 'Participants with the kit were more likely to avoid foods that lower cholesterol level.']", "label": 1 }, { "id": "train_4061", "context": "Psychologist: Some have argued that Freudian psychotherapy is the most effective kind because it is so difficult and time consuming. But surely this does not follow. Similar reasoning -- e. g. , concluding that a car-repair chain has the most effective technique for repairing cars because the cars it services receive so much work and spend so much time in the shop -- would never be accepted.", "question": "The reasoning technique employed by the psychologist is that of attempting to undermine an argument by", "answers": "['suggesting that a supposed cause of a phenomenon is actually an effect of that phenomenon', 'questioning the truth of its premises', 'presenting an analogous argument whose conclusion is thought to be obviously false', 'introducing a principle that contradicts the one on which the argument is based']", "label": 2 }, { "id": "train_4062", "context": "Some teachers claim that students would not learn curricular content without the incentive of grades. But students with intense interest in the material would learn it without this incentive, while the behavior of students lacking all interest in the material is unaffected by such an incentive. The incentive of grades, ttherefore, serves no essential academic purpose.", "question": "The reasoning in the argument is flawed because the argument", "answers": "['takes for granted that the only purpose of school is to convey a fixed body of information to students', 'fails to consider that the incentive of grades may serve some useful nonacademic purpose', 'fails to consider that some students may be neither fascinated by nor completely indifferent to the subject being taught', 'takes for granted that students who are indifferent to the grades they receive are genuinely interested in the curricular material']", "label": 2 }, { "id": "train_4063", "context": "United States politician: Although the amount of United States goods shipped to Mexico doubled in the year after tariffs on trade between the two countries were reduced, it does not follow that the reduction in tariffs caused the sales of United States goods to companies and consumers in Mexico to double that year, because __", "question": "Which of the following, if true, most logically completes the politician's argument?", "answers": "['many of the United States companies that produced goods that year had competitors based in Mexico that had long produced the same kind of goods', 'there was no significant change in the employment rate in either of the countries that year', \"most of the increase in goods shipped by United States companies to Mexico was in parts shipped to the companies' newly relocated subsidiaries for assembly and subsequent shipment back to the United States\", 'the amount of Mexican goods shipped to the United States remained the same as it had been before the tariff reductions']", "label": 2 }, { "id": "train_4064", "context": "Laura: Harold is obviously lonely. He should sell his cabin in the woods and move into town. In town he will be near other people all the time, so he will not be lonely anymore. Ralph: Many very lonely people live in towns. What is needed to avoid loneliness is not only the proximity of other people but also genuine interaction with them.", "question": "Ralph responds to Laura by pointing out that", "answers": "['what is appropriate in one case is not necessarily appropriate in all cases', 'what is logically certain is not always intuitively obvious', 'something needed for a certain result does not necessarily guarantee that result', 'various alternative solutions are possible for a single problem']", "label": 2 }, { "id": "train_4065", "context": "The important parts of any university professor' s job include not only classroom teaching but also research, publication, and lectures to colleagues and the public. This is why university professors who are good classroom teachers but have not engaged in any other scholarly activities are usually denied tenure.", "question": "The statements above, if true, most strongly support which one of the following?", "answers": "['Some university professors have been denied tenure even though they have done an important part of their jobs well.', 'University professors who do not engage in other scholarly activities are usually good classroom teachers.', \"Classroom teaching is the least important part of a university professor's job.\", 'University professors who have not failed to perform important parts of their jobs are not usually denied tenure.']", "label": 0 }, { "id": "train_4066", "context": "Space programs have recently suffered several setbacks with respect to their large projects, and much money has been lost. Consequently, these grand projects should be abandoned in favor of several small ones.", "question": "Which one of the following, if true, provides the most support for the reasoning above?", "answers": "['Large projects are intrinsically more likely to fail and so are more financially risky than small projects.', 'It is just as easy to revise, and even scrap, small projects as it is large ones.', 'Large space projects can explore a few places thoroughly, while small projects can investigate more regions, though less thoroughly.', 'The cost of starting a space project increases every year.']", "label": 0 }, { "id": "train_4067", "context": "Mayor Tyler: In 1982 the courthouse that Roseville still needs would have cost $26 million. Now in 1992 the same building is costing the city close to $30 million to build. If the courthouse had been built in 1982 when I first showed how the building would relieve the overcrowding we were experiencing, Roseville would have saved at least $4 million by now. Councillor Simon: Your own financial reports inform us that $26 million in 1982 dollars is equivalent to $37 million in 1992 dollars. Adding that difference to the money Roseville has saved by not having to maintain an underutilized courthouse for ten years, we can only view the delay as a financial boon for Roseville.", "question": "A point at issue between Mayor Tyler and Councillor Simon is whether", "answers": "['Roseville will build a courthouse in 1992', 'Roseville would have expended $4 million to maintain a courthouse from 1982 to 1992', 'Roseville actually needed a new courthouse between 1982 and 1992', '$37 million in 1992 dollars is equivalent to $26 million in 1982 dollars']", "label": 2 }, { "id": "train_4068", "context": "Researchers had three groups of professional cyclists cycle for one hour at different levels of intensity. Members of groups A, B, and C cycled at rates that sustained, for an hour, pulses of about 60 percent, 70 percent, and 85 percent, respectively, of the recommended maximum pulse rate for recreational cyclists. Most members of Group A reported being less depressed and angry afterward. Most members of Group B did not report these benefits. Most members of Group C reported feeling worse in these respects than before the exercise.", "question": "Which one of the following is most strongly supported by the information above?", "answers": "['Moderate cycling tends to benefit professional cyclists physically as much or more than intense cycling.', 'Physical factors, including pulse rate, contribute as much to depression as do psychological factors.', 'For professional cyclists, the best exercise from the point of view of improving mood is cycling that pushes the pulse no higher than 60 percent of the maximum pulse rate.', 'The effect that a period of cycling has on the mood of professional cyclists tends to depend at least in part on how intense the cycling is.']", "label": 3 }, { "id": "train_4069", "context": "Nylon industry spokesperson: Even though cotton and nylon are used for similar purposes, some people have the mistaken notion that cotton is natural but nylon is not. However, nylon' s main components come from petroleum and from the nitrogen in the atmosphere. Clearly the atmosphere is natural. And petroleum comes from oil, which in turn comes from ancient plants -- a natural source.", "question": "Which one of the following principles, if valid, most helps to justify the nylon industry spokesperson's reasoning?", "answers": "['A substance is natural if the origins of its main components are natural.', 'A substance is unnatural only if the function it serves is unnatural.', 'A substance is no more natural than its least natural component.', 'A substance is no less natural than the processes used in its production.']", "label": 0 }, { "id": "train_4070", "context": "Parent: Pushing very young children into rigorous study in an effort to make our nation more competitive does more harm than good. Curricula for these young students must address their special developmental needs, and while rigorous work in secondary school makes sense, the same approach in the early years of primary school produces only short-term gains and may cause young children to burn out on schoolwork. Using very young students as pawns in the race to make the nation economically competitive is unfair and may ultimately work against us.", "question": "Which one of the following can be inferred from the parent's statements?", "answers": "['A curriculum of rigorous study does not adequately address the developmental needs of primary school students.', 'Our country can be competitive only if the developmental needs of all our students can be met.', 'Unless our nation encourages more rigorous study in the early years of primary school, we cannot be economically competitive.', 'For our nation to be competitive, our secondary school curriculum must include more rigorous study than it now does.']", "label": 0 }, { "id": "train_4071", "context": "Therapist: Cognitive psychotherapy focuses on changing a patient' s conscious beliefs. Thus, cognitive psychotherapy is likely to be more effective at helping patients overcome psychological problems than are forms of psychotherapy that focus on changing unconscious beliefs and desires, since only conscious beliefs are under the patient' s direct conscious control.", "question": "Which one of the following, if true, would most strengthen the therapist's argument?", "answers": "['Psychological problems are frequently caused by unconscious beliefs that could be changed with the aid of psychotherapy.', \"It is difficult for any form of psychotherapy to be effective without focusing on mental states that are under the patient's direct conscious control.\", \"Cognitive psychotherapy is the only form of psychotherapy that focuses primarily on changing the patient's conscious beliefs.\", \"No form of psychotherapy that focuses on changing the patient's unconscious beliefs and desires can be effective unless it also helps change beliefs that are under the patient's direct conscious control.\"]", "label": 1 }, { "id": "train_4072", "context": "A survey of a city' s concertgoers found that almost all of them were dissatisfied with the local concert hall. A large majority of them expressed a strong preference for wider seats and better acoustics. And, even though the survey respondents were told that the existing concert hall cannot feasibly be modified to provide these features, most of them opposed the idea of tearing down the existing structure and replacing it with a concert hall with wider seats and better acoustics.", "question": "Which one of the following, if true, most helps to explain the apparent conflict in the concertgoers' views, as revealed by the survey?", "answers": "['Many popular singers and musicians who currently do not hold concerts in the city would begin to hold concerts there if a new concert hall were built.', \"The city's construction industry will receive more economic benefit from the construction of a new concert hall than from renovations to the existing concert hall.\", 'A well-publicized plan is being considered by the city government that would convert the existing concert hall into a public auditorium and build a new concert hall nearby.', 'Before any of the survey questions were asked, the respondents were informed that the survey was sponsored by a group that advocates replacing the existing concert hall.']", "label": 2 }, { "id": "train_4073", "context": "Jurist: A nation' s laws must be viewed as expressions of a moral code that transcends those laws and serves as a measure of their adequacy. Otherwise, a society can have no sound basis for preferring any given set of laws to all others. Thus, any moral prohibition against the violation of statutes must leave room for exceptions.", "question": "Which one of the following can be properly inferred from the jurist's statements?", "answers": "[\"There is no statute that a nation's citizens have a moral obligation to obey.\", \"A nation's laws can sometimes come into conflict with the moral code they express.\", 'Unless it is legally forbidden ever to violate some moral rules, moral behavior and compliance with laws are indistinguishable.', 'Those who formulate statutes are not primarily concerned with morality when they do so.']", "label": 1 }, { "id": "train_4074", "context": "Under the legal doctrine of jury nullification, a jury may legitimately acquit a defendant it believes violated a law if the jury believes that law to be unjust. Proponents argue that this practice is legitimate because it helps shield against injustice. But the doctrine relies excessively on jurors' objectivity. When juries are empowered to acquit on grounds of their perceptions of unfairness, they too often make serious mistakes.", "question": "The argument uses which one of the following techniques in its attempt to undermine the position that it attributes to the proponents of jury nullification?", "answers": "['attacking the motives of the proponents of the doctrine', 'arguing that the application of the doctrine has undesirable consequences', 'attempting to show that a premise put forward in support of the position is false', 'identifying an inconsistency within the reasoning used to support the position']", "label": 1 }, { "id": "train_4075", "context": "An artificial hormone has recently been developed that increases milk production in cows. Its development has prompted some lawmakers to propose that milk labels should be required to provide information to consumers about what artificial substances were used in milk production. This proposal should not be implemented: just imagine trying to list every synthetic fertilizer used to grow the grass and grain the cows ate, or every fungicide used to keep the grain from spoiling!", "question": "The argument proceeds by", "answers": "['questioning the motivation of those who made the proposal being argued against', 'raising considerations in order to show that the proposal being argued against, if strictly implemented, would lead to absurd consequences', 'introducing a case analogous to the one under consideration to show that a general implementation of the proposal being argued against would be impossible', 'using specific examples in order to show that an alternative to the proposal being argued against would better achieve the ends to which the original proposal was directed']", "label": 1 }, { "id": "train_4076", "context": "The owners of Uptown Apartments are leaning toward not improving the apartment complex; they believe that the increased rents they could charge for improved apartments would not cover the costs of the improvements. But the improvements would make the surrounding housing, which they also own, more valuable and rentable for higher rents. So the owners should make the improvements.", "question": "The reasoning in which one of the following is most similar to the reasoning in the argument above?", "answers": "[\"John's injured knee does not cause him a lot of pain, so he does not want to undergo the pain of surgery to heal it. But the surgery would enable him to exercise regularly again. Thus John should have the surgery.\", 'Since its fishing season lasts only six months, Laketown Fishing Company prefers renting boats to buying its own. But since boats can be used for other purposes during the fishing season, it has made the wrong decision.', \"Max's mechanic thinks there is a crack in the left cylinder head of Max's car and wants to remove the engine to check. Such a diagnostic engine removal would cost about $175, even if the cylinder head does not need replacement. But if the cylinder head is cracked and is not replaced, the engine will be ruined. So Max should have the mechanic check for the crack.\", 'Bulk Fruit Company is deciding whether to market a new hybrid fruit. It is enthusiastic about the idea, since research suggests that people will come to like this fruit. Ttherefore, it is in the long-term interest of the company to market the hybrid fruit.']", "label": 0 }, { "id": "train_4077", "context": "As often now as in the past, newspaper journalists use direct or indirect quotation to report unsupported or false claims made by newsmakers. However, journalists are becoming less likely to openly challenge the veracity of such claims within their articles.", "question": "Each of the following, if true, helps to explain the trend in journalism described above EXCEPT:", "answers": "['Journalists who challenge the veracity of claims are often criticized for failing their professional obligation to be objective.', 'Newspaper publishers have found that many readers will cancel a subscription simply because a view they take for granted has been disputed by the publication.', 'A basic principle of journalism holds that debate over controversial issues draws the attention of the public.', 'The areas of knowledge on which journalists report are growing in specialization and diversity, while journalists themselves are not becoming more broadly knowledgeable.']", "label": 2 }, { "id": "train_4078", "context": "Being articulate has been equated with having a large vocabulary. Actually, however, people with large vocabularies have no incentive for, and tend not to engage in, the kind of creative linguistic self-expression that is required when no available words seem adequate. Thus a large vocabulary is a hindrance to using language in a truly articulate way.", "question": "Which one of the following is an assumption made in the argument?", "answers": "['In unfamiliar situations, even people with large vocabularies often do not have specifically suitable words available.', 'In educating people to be more articulate, it would be futile to try to increase the size of their vocabularies.', 'When people are truly articulate, they have the capacity to express themselves in situations in which their vocabularies seem inadequate.', 'The most articulate people are people who have large vocabularies but also are able to express themselves creatively when the situation demands it.']", "label": 2 }, { "id": "train_4079", "context": "University administrator: Any proposal for a new department will not be funded if there are fewer than 50 people per year available for hire in that field and the proposed department would duplicate more than 25 percent of the material covered in one of our existing departments. The proposed Area Studies Department will duplicate more than 25 percent of the material covered in our existing Anthropology Department. However, we will fund the new department.", "question": "Which one of the following statements follows logically from the university administrator's statements?", "answers": "['If the proposed Area Studies Department did not duplicate more than 25 percent of the material covered in Anthropology, then the new department would not be funded.', 'The proposed Area Studies Department would not duplicate more than 25 percent of the material covered in any existing department other than Anthropology.', 'The field of Area Studies has fewer than 50 people per year available for hire.', 'The field of Area Studies has at least 50 people per year available for hire.']", "label": 3 }, { "id": "train_4080", "context": "The vision test for obtaining a driver' s license should not be limited to measuring the adequacy of vision in daylight conditions, as is the current practice. Many people whose daylight vision is adequate have night vision that is inadequate for safe night driving. Most car accidents occur at night, and inadequate vision plays a role in 80 percent of these accidents.", "question": "The main point of the argument is that", "answers": "['inadequate vision is the primary factor in the majority of car accidents that occur at night', \"the vision test for obtaining a driver's license should measure the adequacy of vision in night conditions\", 'inadequate vision does not play a role in most of the accidents that occur in daylight', \"the current vision test for obtaining a driver's license ensures that most licensed drivers have adequate vision for night driving\"]", "label": 1 }, { "id": "train_4081", "context": "Brown dwarfs -- dim red stars that are too cool to burn hydrogen -- are very similar in appearance to red dwarf stars, which are just hot enough to burn hydrogen. Stars, when first formed, contain substantial amounts of the element lithium. All stars but the coolest of the brown dwarfs are hot enough to destroy lithium completely by converting it to helium. Accordingly, any star found that contains no lithium is not one of these coolest brown dwarfs.", "question": "The argument depends on assuming which one of the following?", "answers": "['Most stars that are too cool to burn hydrogen are too cool to destroy lithium completely.', 'No stars are more similar in appearance to red dwarfs than are brown dwarfs.', 'Most stars, when first formed, contain roughly the same percentage of lithium.', 'None of the coolest brown dwarfs has ever been hot enough to destroy lithium.']", "label": 3 }, { "id": "train_4082", "context": "Three-year-old Sara and her playmate Michael are both ill and have the same symptoms. Since they play together every afternoon, Sara probably has the same illness as Michael does. Since Michael definitely does not have a streptococcal infection, despite his having some symptoms of one, the illness that Sara has is definitely not a streptococcal infection either.", "question": "The reasoning in the argument is flawed because the argument", "answers": "['fails to distinguish between acute streptococcal infections on the one hand, and less severe streptococcal infections on the other', 'presupposes what it sets out to prove', 'mistakes the cause of a particular phenomenon for the effect of that phenomenon', 'treats evidence that the conclusion is probably true as if that evidence establishes the certainty of the conclusion']", "label": 3 }, { "id": "train_4083", "context": "Some credit card companies allow cardholders to skip payments for up to six months under certain circumstances, but it is almost never in a cardholder' s interest to do so. Finance charges accumulate during the skipped-payment period, and the cost to the cardholder is much greater in the long run.", "question": "Which one of the following arguments illustrates a principle most similar to the principle underlying the argument above?", "answers": "['It is better to invest in a used piece of equipment than to purchase a new one. Although used equipment requires more repairs and is sometimes more costly in the long run, buying a new machine requires a far greater initial outlay of capital.', 'Sports cars are impractical for most drivers. While there is undoubtedly a certain thrill associated with driving these cars, their small size makes them incapable of transporting any but the smallest amounts of cargo.', 'It is unwise to use highway maintenance funds for construction of new roads. There is some immediate benefit from new roads, but if these funds are not used for maintenance, the total maintenance cost will be greater in the long run.', 'Although insecticides are effective in ridding the environment of insect pests, they often kill beneficial insects at the same time. Since these beneficial insects are so important, we must find other ways to combat insect pests.']", "label": 2 }, { "id": "train_4084", "context": "Adam: Marking road edges with reflecting posts gives drivers a clear view of the edges, thereby enabling them to drive more safely. Ttherefore, marking road edges with reflecting posts will decrease the annual number of road accidents. Aiesha: You seem to forget that drivers exceed the speed limit more frequently and drive close to the road edge more frequently on roads that are marked with reflecting posts than on similar roads without posts, and those are driving behaviors that cause road accidents.", "question": "Aiesha responds to Adam's argument by", "answers": "[\"raising a consideration that challenges the argument's assumption that facilitating safe driving will result in safer driving\", 'providing additional evidence to undermine the claim that safer driving does not necessarily reduce the number of road accidents', \"questioning Adam's assertion that reflecting posts give drivers a clear view of road edges\", 'presenting a possible alternative method for decreasing road accidents']", "label": 0 }, { "id": "train_4085", "context": "Ana: On libertarian principles, I oppose the proposed smoking ban. It is not the government' s business to prevent people from doing things that harm only themselves. Pankaj: But keep in mind that the ban would apply only to smoking in public places. People could still smoke all they want in private.", "question": "The dialogue provides the most support for the claim that Ana and Pankaj disagree over whether", "answers": "['government should be restrained by libertarian principles', 'the proposed smoking ban is intended to prevent harm only to smokers themselves', 'the proposed ban would prohibit smoking in public places', 'there are cases in which government should attempt to regulate private behavior']", "label": 1 }, { "id": "train_4086", "context": "Zookeeper: Big cats are undoubtedly among the smartest land mammals. Lions, tigers, and jaguars immediately adjust to their new surroundings. Other animals refuse to eat or drink in captivity, but the big cats relish their timely prepared meals. Big cats never attempt to escape their enclosures.", "question": "Which one of the following, if true, most weakens the zookeeper's argument?", "answers": "[\"Big cats don't attempt to escape because they can't figure out their enclosures' weak spots.\", 'No qualified expert believes that adjusting to captivity is a measure of intelligence.', 'Bears also do not have any trouble adjusting to captivity.', 'A recent study comparing the brain scans of large mammals revealed that big cats exhibit the most brain activity when stimulated.']", "label": 1 }, { "id": "train_4087", "context": "The average life expectancy of people who drink alcohol heavily is lower than that of people who do not drink heavily, and people who smoke tobacco have lower life expectancies on average than those who refrain from smoking. Yet the average life expectancy of people who both drink heavily and smoke tends to decrease when such people give up drinking.", "question": "Which one of the following most helps to reconcile the discrepancy described above?", "answers": "['Most heavy drinkers who are also smokers tend to smoke more heavily if they stop drinking.', 'The life expectancy of smokers who stop smoking increases, even if they have smoked for many years.', 'Most heavy smokers who are not heavy drinkers tend to gain weight if they stop smoking.', 'The average life expectancy of relatively light smokers is lower than that of people who have never smoked at all.']", "label": 0 }, { "id": "train_4088", "context": "Normal full-term babies are all born with certain instinctive reflexes that disappear by the age of two months. Because this three-month-old baby exhibits these reflexes, this baby is not a normal full-term baby.", "question": "Which one of the following has a logical structure most like that of the argument above?", "answers": "['Because opossums have abdominal pouches and this animal lacks any such pouch, this animal is not an opossum.', 'Because some types of trees shed their leaves annually and this tree has not shed its leaves, it is not normal.', 'Because no ape can talk and Suzy is an ape, Suzy cannot talk.', 'Because carbon dioxide turns limewater milky and this gas is oxygen, it will not turn limewater milky.']", "label": 0 }, { "id": "train_4089", "context": "Police captain: The chief of police has indicated that gifts of cash or objects valued at more than $100 count as graft. However, I know with certainty that no officer in my precinct has ever taken such gifts, so the recent accusations of graft in my precinct are unfounded.", "question": "The reasoning in the police captain's argument is most vulnerable to criticism on the grounds that the argument", "answers": "['fails to consider that there may be other instances of graft besides those indicated by the chief of police', 'relies on a premise that contradicts the conclusion drawn in the argument', 'bases a claim about the actions of individuals on an appeal to the character of those individuals', 'takes for granted that if the accusations of graft are unfounded, so is any accusation of corruption']", "label": 0 }, { "id": "train_4090", "context": "Cardiologist: Coronary bypass surgery is commonly performed on patients suffering from coronary artery disease when certain other therapies would be as effective. Besides being relatively inexpensive, these other therapies pose less risk to the patient since they are less intrusive. Bypass surgery is especially debatable for single-vessel disease.", "question": "The cardiologist's statements, if true, most strongly support which one of the following?", "answers": "['Needless bypass surgery is more common today than previously.', 'Sometimes there are equally effective alternatives to bypass surgery that involve less risk.', 'Bypass surgery should be performed when more than one vessel is diseased.', 'Bypass surgery is riskier than all alternative therapies.']", "label": 1 }, { "id": "train_4091", "context": "Learning how to build a nest plays an important part in the breeding success of birds. For example, Dr. Snow has recorded the success of a number of blackbirds in several successive years. He finds that birds nesting for the first time are less successful in breeding than are older birds, and also less successful than they themselves are a year later. This cannot be a mere matter of size and strength, since blackbirds, like the great majority of birds, are fully grown when they leave the nest. It is difficult to avoid the conclusion that they benefit by their nesting experience.", "question": "Which one of the following, if true, would most weaken the argument?", "answers": "['Up to 25 percent of all birds are killed by predators before they start to nest.', 'The breeding success of birds nesting for the second time is greater than that of birds nesting for the first time.', 'Blackbirds build better nests than other birds.', 'The capacity of blackbirds to lay viable eggs increases with each successive trial during the first few years of reproduction.']", "label": 3 }, { "id": "train_4092", "context": "In Borania many people who want to quit smoking wear nicotine skin patches, which deliver small doses of nicotine through the skin. Beginning next month, these patches can be purchased without a doctor' s prescription. Although nonprescription patches will be no more effective than those obtained by prescription and will be equally expensive, patch manufacturers are nevertheless predicting that the patches' new nonprescription status will boost sales, which have slowed considerably in recent years.", "question": "Which of the following, if true in Borania, most strongly supports the manufacturers' prediction?", "answers": "['Several nonprescription aids aimed at helping people to quit smoking have been widely available for a number of years.', 'Many smokers who want to quit smoking feel that they cannot afford to visit a doctor for a prescription.', 'Most people who wanted to quit smoking and who found the nicotine skin patch helpful in quitting have quit.', 'People who use nicotine skin patches have approximately the same rate of success in quitting smoking as do people who use other aids aimed at helping people to quit smoking.']", "label": 1 }, { "id": "train_4093", "context": "Last summer, after a number of people got sick from eating locally caught anchovies, the coastal city of San Martin advised against eating such anchovies. The anchovies were apparently tainted with domoic acid, a harmful neurotoxin. However, a dramatic drop in the population of P. australis plankton to numbers more normal for local coastal waters indicates that it is once again safe to eat locally caught anchovies.", "question": "Which one of the following, if true, would most help to explain why it is now safe to lift the advisory?", "answers": "['Scientists have used P. australis plankton to obtain domoic acid in the laboratory.', 'A sharp decline in the population of P. australis is typically mirrored by a corresponding drop in the local anchovy population.', 'P. australis naturally produces domoic acid, though anchovies consume enough to become toxic only when the population of P. australis is extraordinarily large.', 'P. australis cannot survive in large numbers in seawater that does not contain significant quantities of domoic acid along with numerous other compounds.']", "label": 2 }, { "id": "train_4094", "context": "Recently ApexCo has begun drilling off the coast of Mycondia. In this time, the number of animals that end up on the beach, coated in oil and other industrial by-products, has increased dramatically. Nonetheless, no Peterson's loggerhead turtles, a tortoise common in the waters off the coast of Mycondia, have washed upon shore. ApexCo's public relations claim that while some of the by-products are clearly harmful to aquatic life, the lack of Peterson's loggerhead turtles that are washed ashore suggest that not all the aquatic life there is adversely affected.", "question": "Which of the following, if true, casts the most doubt on the argument employed by ApexCo's public relations firm?", "answers": "['ApexCo has also drilled off the coast of Sylvania, which witnessed many of its local aquatic life washing ashore.', \"The Peterson's loggerhead turtle has never been seen washed upon the shores of Mycondia.\", 'Sea otters are the most common beached animals, followed by dolphins.', 'There have been no reports of beached cephalopods, which are common in the waters of the coast of Mycondia.']", "label": 1 }, { "id": "train_4095", "context": "There is no reason why the work of scientists has to be officially confirmed before being published. There is a system in place for the confirmation or disconfirmation of scientific findings, namely, the replication of results by other scientists. Poor scientific work on the part of any one scientist, which can include anything from careless reporting practices to fraud, is not harmful. It will be exposed and rendered harmless when other scientists conduct the experiments and obtain disconfirmatory results.", "question": "Which one of the following, if true, would weaken the argument?", "answers": "['Most scientists work as part of a team rather than alone.', 'In scientific experiments, careless reporting is more common than fraud.', 'Most scientists are under pressure to make their work accessible to the scrutiny of replication.', 'Scientific experiments can go unchallenged for many years before they are replicated.']", "label": 3 }, { "id": "train_4096", "context": "The proper way to plan a scientific project is first to decide its goal and then to plan the best way to accomplish that goal. The United States space station project does not conform to this ideal. When the Cold War ended, the project lost its original purpose, so another purpose was quickly grafted onto the project, that of conducting limited-gravity experiments, even though such experiments can be done in an alternative way. It is, ttherefore, abundantly clear that the space station should not be built.", "question": "The reasoning in the argument is flawed because the argument", "answers": "['attacks the proponents of a claim rather than arguing against the claim itself', 'faults planners for not foreseeing a certain event, when in fact that event was not foreseeable', 'concludes that a shortcoming is fatal, having produced evidence only of the existence of that shortcoming', 'contains statements that lead to a self-contradiction']", "label": 2 }, { "id": "train_4097", "context": "In a recession, a decrease in consumer spending causes many businesses to lay off workers or even to close. Workers who lose their jobs in a recession usually cannot find new jobs. The result is an increase in the number of people who are jobless. Recovery from a recession is defined by an increase in consumer spending and an expansion of business activity that creates a need for additional workers. But businesspeople generally have little confidence in the economy after a recession and ttherefore delay hiring additional workers as long as possible.", "question": "The statements above, if true, provide most support for which one of the following conclusions?", "answers": "['Sometimes recovery from a recession does not promptly result in a decrease in the number of people who are jobless.', 'Governmental intervention is required in order for an economy to recover from a recession.', 'Employees of businesses that close during a recession make up the majority of the workers who lose their jobs during that recession.', 'Workers who lose their jobs during a recession are likely to get equally good jobs when the economy recovers.']", "label": 0 }, { "id": "train_4098", "context": "Often, a product popularly believed to be the best of its type is no better than any other; rather, the product' s reputation, which may be independent of its quality, provides its owner with status. Thus, although there is no harm in paying for status if that is what one wants, one should know that one is paying for prestige, not quality.", "question": "Which one of the following arguments is most similar in its reasoning to the argument above?", "answers": "['Many theatrical actors cannot enjoy watching a play because when they watch others, they yearn to be on stage themselves. Thus, although there is no harm in yearning to perform, such performers should, for their own sakes, learn to suppress that yearning.', 'Those beginning a new hobby sometimes quit it because of the frustrations involved in learning a new skill. Thus, although it is fine to try to learn a skill quickly, one is more likely to learn a skill if one first learns to enjoy the process of acquiring it.', 'Personal charm is often confused with virtue. Thus, while there is nothing wrong with befriending a charming person, anyone who does so should realize that a charming friend is not necessarily a good and loyal friend.', \"Most people have little tolerance for boastfulness. Thus, although one's friends may react positively when hearing the details of one's accomplishments, it is unlikely that their reactions are entirely honest.\"]", "label": 2 }, { "id": "train_4099", "context": "Deer mice normally do not travel far from their nests, and deer mice that are moved more than half a kilometer from their nests generally never find their way back. Yet in one case, when researchers camped near a deer mouse nest and observed a young deer mouse for several weeks before moving it to an area over two kilometers away, the deer mouse found its way back to its nest near their camp in less than two days.", "question": "Which one of the following, if true, most helps to explain how the deer mouse might have found its way back to its nest?", "answers": "['The area to which the deer mouse was moved was dryer and more rocky than the area in which its nest was located.', 'Animals that prey on deer mice were common in the area to which the deer mouse was moved.', 'The researchers had moved the deer mouse in a small dark box, keeping the mouse calm before it was released.', 'The researchers released the deer mouse in a flat area across which their campfire smoke drifted.']", "label": 3 }, { "id": "train_4100", "context": "Books about architectural works, unless they are not intended for a general audience, ought to include discussions of both the utility and the aesthetic appeal of each of the buildings they consider. If they do not, they are flawed. Morton' s book on Italian Baroque palaces describes these palaces' functional aspects, but fails to mention that the main hall of a palace he discusses at length has a ceiling that is one of the truly breathtaking masterpieces of Western art.", "question": "If the statements above are true, it would be necessary to establish which one of the following in order to conclude that Morton's book is flawed?", "answers": [ "Morton's description of the palaces' utility is inaccurate.", "The passage discussing the palace plays a very important role in helping to establish the overall argument of Morton's book.", "Morton's book is intended for a general audience.", "The palace discussed at length is one of the most aesthetically important of those treated in Morton's book." ], "label": 2 }, { "id": "train_4101", "context": "Scientists once believed that the oversized head, long hind legs, and tiny forelimbs that characterized Tyrannosaurus rex developed in order to accommodate the great size and weight of this prehistoric predator. However, this belief must now be abandoned. The nearly complete skeleton of an earlier dinosaur has recently been discovered. This specimen had the characteristic T. rex features but was one-fifth the size and one-hundredth the weight.", "question": "The answer to which one of the following questions would most help in evaluating the argument?", "answers": "['Was the ratio of the head size of the recently discovered dinosaur to its body size the same as that for T. rex?', 'At what stage in its life did the recently discovered dinosaur die?', 'Did the recently discovered dinosaur prey on species as large as those that T. rex preyed on?', 'Was T. rex the largest and heaviest prehistoric predator?']", "label": 1 }, { "id": "train_4102", "context": "Water vapor evaporated from the ocean contains a greater proportion of oxygen-16 and a smaller proportion of the heavier oxygen-18 than does seawater. Normally, this phenomenon has no effect on the overall composition of the ocean, because evaporated seawater returns to the ocean through precipitation. During an ice age, however, a large amount of precipitation falls on ice caps, where it is trapped as ice.", "question": "Which one of the following conclusions about a typical ice age is most strongly supported by the statements above?", "answers": "['The proportions of oxygen-16 and oxygen-18 are the same in vapor from seawater as in the seawater itself.', 'The concentration of oxygen-18 in seawater is increased.', \"During the ice age, more of the Earth's precipitation falls over land than falls over the ocean.\", 'The composition of seawater changes more slowly than it does in interglacial periods.']", "label": 1 }, { "id": "train_4103", "context": "The Blackridge Company, a software retailer, recently implemented a service offering free telephone support for its customers. After 3 months of service, the average length of a technical support call was 15 minutes, 3 minutes higher than the company's target length of 12 minutes. After studying the operations of the new telephone support division, the company discovered that customer support agents were putting customers on hold for an average of 4 minutes per call while these agents researched possible solutions to the technical issues at hand.", "question": "From the passage above, it can be inferred that the Blackridge Company could reduce the average length of its customer support calls by doing which of the following, assuming that each is a realistic possibility?", "answers": "['Providing an online support service in addition to the telephone support service', 'Expanding the hours of operation of its telephone support service', 'Improving the quality of its software products', 'Better training customer support agents in the technical aspects of the software']", "label": 3 }, { "id": "train_4104", "context": "It is popularly believed that a poem has whatever meaning is assigned to it by the reader. But objective evaluation of poetry is possible only if this popular belief is false; for the aesthetic value of a poem cannot be discussed unless it is possible for at least two readers to agree on the correct interpretation of the poem.", "question": "Which one of the following is an assumption required by the argument?", "answers": "['Aesthetic evaluation of literature is best accomplished through discussion by more than two readers.', 'If two readers agree about the meaning of a given poem, that ensures that an objective evaluation of the poem can be made.', 'Discussion of a poem is possible only if it is false that a poem has whatever meaning is assigned to it by the reader.', \"A given poem can be objectively evaluated only if the poem's aesthetic value can be discussed.\"]", "label": 3 }, { "id": "train_4105", "context": "Contrary to recent speculations, no hardware store will be opening in the shopping plaza. If somebody were going to open a store there, they would already have started publicizing it. But there has been no such publicity.", "question": "Which one of the following most accurately expresses the conclusion drawn in the argument?", "answers": "['A hardware store will not be opening in the shopping plaza.', 'Some people have surmised that a hardware store will be opening in the shopping plaza.', 'If somebody were going to open a hardware store in the shopping plaza, that person would already have started publicizing it.', 'It would be unwise to open a hardware store in the shopping plaza.']", "label": 0 }, { "id": "train_4106", "context": "Among a sample of diverse coins from an unfamiliar country, each face of any coin portrays one of four things: a judge' s head, an explorer' s head, a building, or a tree. By examining the coins, a collector determines that none of them have heads on both sides and that all coins in the sample with a judge' s head on one side have a tree on the other.", "question": "If the statements above are true, which one of the following must be true of the coins in the sample?", "answers": [ "All those with an explorer's head on one side have a building on the other.", "None of those with a tree on one side have an explorer's head on the other.", "All those with a tree on one side have a judge's head on the other.", "None of those with a building on one side have a judge's head on the other." ], "label": 3 }, { "id": "train_4107", "context": "Randy: After Mega Cable Television Company refused to carry the competing Azco News Service alongside its own news channels, the mayor used her influence to get Azco time on a community channel, demonstrating her concern for keeping a diversity of news programming in the city. Marion: The mayor' s action is fully explained by cruder motives: she' s rewarding Azco' s owner, a political supporter of hers.", "question": "Of the following, which one, if true, is the logically strongest counter Randy can make to Marion's objection?", "answers": "[\"The mayor also used her influence to get time on a community channel for another news service, whose owner supported the mayor's opponent in the last election.\", \"The many people whose jobs depend on Azco's continued presence on a community channel are a potential source of political support for the mayor.\", \"Azco's news coverage of the mayor has never been judged to be biased by an impartial, independent organization.\", 'The owner of Azco supported the mayor simply because he liked her political agenda, and not for any expected reward.']", "label": 0 }, { "id": "train_4108", "context": "Investors: Your company is responsible for discovering fraudulent bookkeeping practices of your accountants. You were given access to investment funds to hire six auditors to examine the books, but you hired only four auditors. We must conclude that your company purposely hired only the limited number of auditors in an effort to avoid the full extent of the fraudulent bookkeeping practices from being exposed. Company president: Our company tried to hire six auditors, but the stringent hiring requirements set by the investors were so strict that it was impossible to find two more qualified auditors.", "question": "The company president responds to the investors' accusation by", "answers": "[\"proving that the investors' comments are self-contradictory\", 'rewording the investors conclusion to make it more favorable to the company president', 'highlighting a relevant fact that challenges the conclusion drawn by the investors', 'accusing the investors of attempting to conceal the fraudulent bookkeeping practices']", "label": 2 }, { "id": "train_4109", "context": "Advertising by mail has become much less effective, with fewer consumers responding. Because consumers are increasingly overwhelmed by the sheer amount of junk mail they receive, most discard almost all offers without considering them. Thus, an effective way for corporations to improve response rates would be to more carefully target the individuals to whom they mail advertising, thereby cutting down on the amount of junk mail each consumer receives.", "question": "Which of the following, if true, would most support this recommendation?", "answers": "['Response rates to carefully targeted advertisements by mail are considerably higher, on average, than response rates to most other forms of advertising.', 'Improvements in the quality of the advertising materials used in mail that is carefully targeted to individuals can improve the response rate for such mail.', 'There are cost-effective means by which corporations that currently advertise by mail could improve response rates.', 'Many successful corporations are already carefully targeting the individuals to whom they mail advertising.']", "label": 0 }, { "id": "train_4110", "context": "Even though most universities retain the royalties from faculty members' inventions, the faculty members retain the royalties from books and articles they write. Ttherefore, faculty members should retain the royalties from the educational computer software they develop.", "question": "The conclusion above would be more reasonably drawn if which of the following were inserted into the argument as an additional premise?", "answers": "['In terms of the criteria used to award royalties, educational software programs are more nearly comparable to books and articles than to inventions.', 'Faculty members are more likely to produce educational software programs than inventions.', 'Royalties from inventions are higher than royalties from educational software programs.', 'In the experience of most universities, educational software programs are more marketable than are books and articles.']", "label": 0 }, { "id": "train_4111", "context": "On the surface, Melville' s Billy Budd is a simple story with a simple theme. However, if one views the novel as a religious allegory, then it assumes a richness and profundity that place it among the great novels of the nineteenth century. However, the central question remains: Did Melville intend an allegorical reading? Since there is no textual or historical evidence that he did, we should be content with reading Billy Budd as a simple tragedy.", "question": "Which one of the following most accurately expresses the principle underlying the argument?", "answers": "[\"Without relevant evidence as to a novel's intended reading, one should avoid viewing the work allegorically.\", 'Given a choice between an allegorical and a nonallegorical reading of a novel, one should choose the latter.', \"The only relevant evidence in deciding in which genre to place a novel is the author's stated intention.\", 'In deciding between rival readings of a novel, one should choose the one that is most favorable to the work.']", "label": 0 }, { "id": "train_4112", "context": "Older United States automobiles have been identified as contributing disproportionately to global air pollution. The requirement in many jurisdictions that automobiles pass emission-control inspections has had the effect of taking many such automobiles out of service in the United States, as they fail inspection and their owners opt to buy newer automobiles. Thus the burden of pollution such older United States automobiles contribute to the global atmosphere will be gradually reduced over the next decade.", "question": "Which one of the following, if true, most seriously weakens the argument?", "answers": "['There is a thriving market for used older United States automobiles that are exported to regions that have no emission-control regulations.', 'When automobiles that are now new become older, they will, because of a design change, cause less air pollution than older automobiles do now.', 'Even if all the older automobiles in the United States were retired from service, air pollution from United States automobiles could still increase if the total number of automobiles in use should increase significantly.', 'It is impossible to separate the air of one country or jurisdiction from that of others, since air currents circle the globe.']", "label": 0 }, { "id": "train_4113", "context": "Farmer: Worldwide, just three grain crops-rice, wheat, and corn-account for most human caloric intake. To maintain this level of caloric intake and also keep pace with global population growth, yields per acre from each of these crops will have to increase at least 1. 5 percent every year, given that the supply of cultivated land is diminishing. Ttherefore, the government should increase funding for research into new ways to improve yields", "question": "Which of the following is an assumption on which the farmers argument depends?", "answers": "['Increasing the yields per acre of rice, wheat, and corn is more important than increasing the yields per acre of other crops', 'Increasing government funding for research into new ways to improve the yields per acre of rice, wheat, and corn crops would help to increase total worldwide annual production of food from these crops', 'Current levels of funding for research into ways of improving grain crop yields per acre have enabled grain crop yields per acre to increase by more than 1. 5 percent per year worldwide', 'It is solely the governments responsibility to ensure that the amount of rice wheat, and corn produced worldwide keeps pace with global population growth']", "label": 1 }, { "id": "train_4114", "context": "After a long period of decreased sales, many companies lay off employees or even shut down completely. Out of a sense of camaraderie, employees who are laid off are less likely to buy products from other companies that have laid off their workers. The result is a continuing decrease in sales for struggling companies. Recovery from a long period of decreased sales is defined by an increase in sales and profits and an expansion of the company that creates a need for more employees. But companies who have had to lay off employees due to a long period of decreased sales are not confident in their ability to expand successfully and, ttherefore, are hesitant to hire new employees.", "question": "The statements above, if true, provide the most support for which one of the following conclusions?", "answers": "['Sometimes recovery from a long period of decreased sales does not immediately result in an increased number of employees at a company.', 'Periods of decreased sales are usually caused by a lack of camaraderie between workers.', 'Employees of businesses that close after a long period of decreased sales make up the majority of the unemployed population.', 'Investor intervention is required in order for a company to recover from a long period of decreased sales.']", "label": 0 }, { "id": "train_4115", "context": "Birds have been said to be descended from certain birdlike dinosaur species with which they share distinctive structural features. The fossil record, however, shows that this cannot be so, since there are bird fossils much older than the earliest birdlike dinosaur fossils that have been found.", "question": "Which of the following is an assumption on which the argument relies?", "answers": "['The birdlike dinosaurs have no descendants.', 'Birds cannot have been descended from dinosaur species with which the birds do not share the distinctive structural features.', 'There are no birdlike dinosaur fossils that are older than the bird fossils but have not yet been unearthed.', 'It could not have been the case that some birds were descendant from one of the bird like dinosaur species and other birds from another.']", "label": 2 }, { "id": "train_4116", "context": "Although inflated government spending for weapons research encourages waste at weapons research laboratories, weapons production plants must be viewed as equally wasteful of taxpayer dollars. After all, by the government' s own admission, the weapons plant it plans to reopen will violate at least 69 environmental, health, and safety laws. The government has decided to reopen the plant and exempt it from compliance, even though the weapons to be produced there could be produced at the same cost at a safer facility.", "question": "The reasoning in the argument is most vulnerable to criticism on which one of the following grounds?", "answers": "['It concedes a point regarding weapons research laboratories that undermines its conclusion about weapons production plants.', 'It fails to establish that research laboratories and weapons production plants are similar enough to be meaningfully compared.', 'It offers no evidence that the \"safer\" alternative production site actually complies with any of the laws mentioned.', 'It relies on evidence that does not directly address the issue of wasteful spending.']", "label": 3 }, { "id": "train_4117", "context": "Utrania was formerly a major petroleum exporter, but in recent decades economic stagnation and restrictive regulations inhibited investment in new oil fields. In consequence, Utranian oil exports dropped steadily as old fields became depleted. Utrania' s currently improving economic situation, together with less-restrictive regulations, will undoubtedly result in the rapid development of new fields. However, it would be premature to conclude that the rapid development of new fields will result in higher oil exports, because __.", "question": "Which of the following most logically completes the argument?", "answers": "['new technology is available to recover oil from old oil fields formerly regarded as depleted', 'the improvement in the economic situation in Utrania is expected to result in a dramatic increase in the proportion of Utranians who own automobiles', 'the price of oil is expected to remain relatively stable over the next several years', 'most of the investment in new oil fields in Utrania is expected to come from foreign sources']", "label": 1 }, { "id": "train_4118", "context": "Travel industry consultant: Several airlines are increasing elbow room and leg room in business class, because surveys show that business travelers value additional space more than, say, better meals. But airlines are overconcerned about the comfort of passengers flying on business; they should instead focus on the comfort of leisure travelers, because those travelers purchase 80 percent of all airline tickets.", "question": "Which one of the following, if true, most weakens the reasoning in the travel industry consultant's argument?", "answers": "['Some airlines have indicated that they will undertake alterations in seating space throughout the entire passenger area of their planes in the near future.', 'Business travelers often make travel decisions based on whether they feel a given airline values their business.', 'Sleeping in comfort during long flights is not the primary concern of leisure travelers.', \"A far greater proportion of an airline's revenues is derived from business travelers than from leisure travelers.\"]", "label": 3 }, { "id": "train_4119", "context": "Jay: The development of a plain writing style in seventeenth-century England was mainly due to an increase in the literacy rate. To reach moderately educated readers, writers simplified their style. Chandra: No, the pivotal factor was the increasing interest in science among the most highly educated people; a complex, artificial style, however beautiful, interfered with the presentation of scientific facts.", "question": "Jay's and Chandra's comments indicate that they disagree about", "answers": "['what was the primary cause of the development of a plain writing style in seventeenth-century England', 'whether there was an increase in the percentage of people who were able to read in England during the seventeenth century', 'the extent of the changes in writing style that took place in seventeenth-century England', 'whether the quality of written works in seventeenth-century England increased or decreased as a result of the development of a plain writing style']", "label": 0 }, { "id": "train_4120", "context": "The Common Loon is a migratory bird that winters in warmer regions and returns to its breeding lakes in the spring. Typically, only one pair of loons occupies a single lake. Breeding pairs in search of breeding territory either occupy a vacant lake or take over an already occupied one. Surprisingly, almost half the time, returning loons choose to intrude on a territory already occupied by another pair of loons and attempt to oust its residents. This happens even when there are vacant lakes nearby that are perfectly suitable breeding territories.", "question": "Which one of the following, if true, most helps to explain the surprising behavior described above?", "answers": "['Loons that intrude on an occupied breeding territory are successful in ousting its residents about half the time.', 'Loons frequently determine that a lake is a suitable breeding territory by observing the presence of a breeding pair there.', 'Contests for occupied breeding territory may be initiated either by male loons or by female loons.', 'Lakes that are perfectly suitable for loon breeding have fish for food, a site for a nest, and a sheltered area to rear chicks.']", "label": 1 }, { "id": "train_4121", "context": "Dietician: \"The French Paradox\" refers to the unusual concurrence in the population of France of a low incidence of heart disease and a diet high in fat. The most likely explanation is that the French consume a high quantity of red wine, which mitigates the ill effects of the fat they eat. So North Americans, with nearly the highest rate of heart disease in the world, should take a cue from the French: if you want to be healthier without cutting fat intake, drink more red wine.", "question": "Which one of the following statements, if true, most seriously undermines the conclusion of the dietician's argument?", "answers": "['A greater intake of red wine among North Americans would likely lead to a higher incidence of liver problems and other illnesses.', 'Many other regions have much lower rates of heart disease than France, though their populations consume even less red wine than do North Americans.', 'French men consume as much red wine as French women do, yet French men have a higher rate of heart disease than do French women.', 'All evidence suggests that the healthiest way to decrease the chance of heart disease is to exercise and keep a diet low in fat.']", "label": 0 }, { "id": "train_4122", "context": "Red algae contains the pigment phycoerythrin. Phycoerythrin reflects red light and absorbs blue light. As blue light has the ability to penetrate water to great depths, phycoerythrin allows red algae to absorb the light needed to produce the fuel essential to their survival. Because of their need for light, red algae cannot survive in waters where the surface is covered with other plant life or debris. 300 years ago, Japanese cultures began cultivating red algae for food.", "question": "The statements above, if true, most strongly support which of the following claims?", "answers": "['Red algae was not cultivated in Japan until 300 years ago.', '300 years ago, the Japanese had access to water with at least partially clear surfaces.', 'No light has the ability to penetrate to greater depths than blue light.', 'Surface water has increasingly become cluttered with debris and plant life since the Japanese began cultivating red algae 300 years ago.']", "label": 1 }, { "id": "train_4123", "context": "Can any research be found to validate the contention that those who spend time plucking out their gray hairs have more negative attitudes toward the elderly than those who shrug their shoulders about their gray hairs? Unless a person' s psychopathology leads him or her to overgeneralize, there is no necessary connection. Certainly it is reasonable to like the elderly yet dislike the idea of impaired eyesight and hearing. Furthermore, holding negative attitudes toward older people merely because they are old is immoral, according to nearly universally accepted ethical standards. But there is nothing immoral about disliking some concomitants of the aging process.", "question": "Which one of the following best expresses the main point of the passage?", "answers": "['Being elderly is fine, but the process of becoming elderly is not; and people need to understand the distinction between the two.', 'To dislike the elderly is immoral, and to do so just because one dislikes some of the physical concomitants of growing old is unreasonable.', 'It cannot be assumed that people who dislike some of the physical concomitants of growing old necessarily have negative feelings toward the elderly.', 'Since no one likes the physical concomitants of growing old, it is wrong to dislike the elderly merely because of their physical characteristics.']", "label": 2 }, { "id": "train_4124", "context": "Certain groups of Asian snails include both \"left-handed\" and \"right-handed\" species, with shells coiling to the left and right, respectively. Some left-handed species have evolved from right-handed ones. Also, researchers found that snaileating snakes in the same habitat have asymmetrical jaws, allowing them to grasp right-handed snail shells more easily. If these snakes ate more right-handed snails over time, this would have given left-handed snails an evolutionary advantage over right-handed snails, with the left handed snails eventually becoming a new species. Thus, the snakes' asymmetrical jaws probably helped drive the emergence of the left-handed snail species.", "question": "Which of the following would, if true, most strengthen the argument that asymmetrical snake jaws helped drive left-handed snail evolution?", "answers": "['Anatomical differences prevent left-handed snails from mating easily with right-handed snails.', 'Some right-handed snails in this habitat have shells with a very narrow opening that helps prevent snakes from extracting the snails from inside their shells.', 'In one snake species, the snakes with asymmetrical jaws eat snails, while the snakes with symmetrical jaws do not eat snails.', 'Experiments show that the snail-eating snakes in this habitat fail more often in trying to eat left-handed snails than in trying to eat right-handed snails.']", "label": 3 }, { "id": "train_4125", "context": "Dumping\" is defined as selling a product in another country for less than production cost. Shrimp producers from Country F are selling shrimp in Country G below the cost of producing shrimp in Country G. So Country F' s producers are dumping shrimp.", "question": "In order to evaluate the argument above, it is necessary to determine whether", "answers": "['\"production cost\" in the definition of dumping refers to the cost of producing the product in the country where it originates or in the country where it is sold', 'shrimp producers from Country F charge more for shrimp that they sell within their own country than for shrimp that they sell in Country G', 'shrimp producers from Country F are selling shrimp in Country G for considerably less than production cost or just slightly less', 'shrimp producers from Country F will eventually go out of business if they continue to sell shrimp in Country G for less than production cost']", "label": 0 }, { "id": "train_4126", "context": "Traditionally, the most highly sought cars have been the \"sports cars\" and similar two-door models. Nevertheless, Zincstone Motors has chosen to eliminate the last two-door models and produce only four-door models.", "question": "Which of the following would, if true, most help to explain Zincstone Motors' strategy?", "answers": "['In almost every instance, Zincstone Motors models lead all comparable models of competitors in fuel efficiency and have lower average maintenance costs as well.', \"After a spate of recent additional safety requirements, the cost of frame and doors of Zincstone Motors' standard two-door models are now three times as expensive as standard four-door frame and doors.\", 'Many of Zincstone Motors models are exported and sold overseas, including in some countries like Japan, which import a significant number of cars into the United States.', \"As American consumers lose access to car manufacturers who produce two-door cars, and as two-door cars occupy smaller and smaller shares of the United States car market, American consumers' tastes tend to shift from two-door cars.\"]", "label": 1 }, { "id": "train_4127", "context": "Lea: Contemporary art has become big business. Nowadays art has less to do with self-expression than with making money. The work of contemporary artists is utterly bereft of spontaneity and creativity, as a visit to any art gallery demonstrates. Susan: I disagree. One can still find spontaneous, innovative new artwork in most of the smaller, independent galleries.", "question": "Lea's and Susan's remarks provide the most support for holding that they disagree about whether", "answers": "['contemporary art has become big business', 'most galleries contain some artwork that lacks spontaneity and creativity', 'some smaller art galleries still exhibit creative new artwork', 'large galleries contain creative artwork']", "label": 2 }, { "id": "train_4128", "context": "We already knew from thorough investigation that immediately prior to the accident, either the driver of the first vehicle changed lanes without signaling or the driver of the second vehicle was driving with excessive speed. Either of these actions would make a driver liable for the resulting accident. But further evidence has proved that the first vehicle' s turn signal was not on, though the driver of that vehicle admits to having changed lanes. So the driver of the second vehicle is not liable for the accident.", "question": "Which one of the following would be most important to know in evaluating the conclusion drawn above?", "answers": "['whether the driver of the first vehicle was a reliable witness', 'whether the driver of the second vehicle would have seen the turn signal flashing had it been on', 'whether the second vehicle was being driven at excessive speed', 'whether any other vehicles were involved in the accident']", "label": 2 }, { "id": "train_4129", "context": " Plant scientists have used genetic engineering on seeds to produce crop plants that are highly resistant to insect damage. Unfortunately, the seeds themselves are quite expensive, and the plants require more fertilizer and water to grow well than normal ones. Thus, for most farmers the savings on pesticides would not compensate for the higher seed costs and the cost of additional fertilizer. However, since consumer demand for grains, fruits, and vegetables grown without the use of pesticides continues to rise, the use of genetically engineered seeds of this kind is likely to become widespread.", "question": "In the argument given, the two portions in boldface play which of the following roles?", "answers": "[\"The first and the second each provide evidence to support the argument's main conclusion.\", \"The first supplies a context for the argument; the second is the argument's main conclusion.\", 'The first provides evidence to support a prediction that the argument seeks to defend; the second is that prediction.', 'The first presents a development that the argument predicts will have a certain outcome; the second acknowledges a consideration that weighs against that prediction.']", "label": 3 }, { "id": "train_4130", "context": "The city of Carina is considering implementing an Alternative Minimum Tax (AMT) for taxpayers earning more than 90% of the population, those individuals who earn more than $350, 000 annually. Under this tax plan, Carina citizens in the AMT bracket would be exempt from paying taxes on the first $80, 000 of their income and be taxed at a flat rate on the income thereafter. People paying the AMT would not be given allowances regular taxpayers are, such as deductions for children, healthcare expenses, or college tuition.", "question": "Which of the following would be most important to determine in order to evaluate whether implementing the AMT plan would be profitable for the city of Carina?", "answers": "['The dollar value that processing the AMT would require, compared to the dollar value that running the current tax system in Carina requires.', 'The dollar value of the tax exemptions the top 10% of wage earners in Carina currently take.', \"The average dollar value of Carina citizens' tax payments for the previous year.\", 'The average revenue generated by cities in that country that charge AMT when such a system is implemented.']", "label": 1 }, { "id": "train_4131", "context": "To prevent a newly built dam on the Chiff River from blocking the route of fish migrating to breeding grounds upstream, the dam includes a fish pass, a mechanism designed to allow fish through the dam. Before the construction of the dam and fish pass, several thousand fish a day swam upriver during spawning season. But in the first season after the project's completion, only 300 per day made the journey. Clearly, the fish pass is defective.", "question": "Which of the following, if true, most seriously weakens the argument?", "answers": "['On other rivers in the region, the construction of dams with fish passes has led to only small decreases in the number of fish migrating upstream.', 'Populations of migratory fish in the Chiff River have been declining slightly over the last 20 years.', 'During spawning season, the dam releases sufficient water for migratory fish below the dam to swim upstream', 'The construction of the dam stirred up potentially toxic river sediments that were carried downstream.']", "label": 3 }, { "id": "train_4132", "context": "Moore: Sunscreen lotions, which are designed to block skin-cancer-causing ultraviolet radiation, do not do so effectively. Many scientific studies have shown that people who have consistently used these lotions develop, on average, as many skin cancers as those who have rarely, if ever, used them.", "question": "The reasoning in Moore's argument is most vulnerable to criticism on the grounds that the argument", "answers": "['takes for granted that there are no other possible health benefits of using sunscreen lotions other than blocking skin-cancer-causing ultraviolet radiation', 'fails to consider the effectiveness of sunscreen lotions that are not specifically designed to block skin-cancer-causing ultraviolet radiation', 'overlooks the possibility that people who consistently use sunscreen lotions spend more time in the sun, on average, than people who do not', 'fails to distinguish between the relative number of cases of skin cancer and the severity of those cases in measuring effectiveness at skin cancer prevention']", "label": 2 }, { "id": "train_4133", "context": "The drinking age should remain at 21 years of age. Prior to reaching the age of 21, the brain is not fully developed. As a result, people cannot make informed decisions about alcohol, and the abuse of alcohol can lead to severe brain damage.", "question": "Which one of the following, if true, would most weaken the argument's conclusion?", "answers": "['Making informed decisions about alcohol reduces the risk of severe brain damage.', 'The brain fully develops around 24 years of age.', 'People over the age of 21 spend more money on alcohol than people under the age of 21.', 'Alcohol can cause severe brain damage after the age of 21.']", "label": 1 }, { "id": "train_4134", "context": "In 1987 Fabrico, a clothing manufacturer, sold to its customers a total of 2. 3 million dollars worth of clothing, a 17 percent increase over 1986 sales. In January 1988, however, Fabrico closed one of its clothing factories, citing reduced demand for its products as the major reason.", "question": "Which one of the following, if true about 1987, contributes most to a resolution of the apparent discrepancy noted between the sales figures and the reason offered for closing a factory?", "answers": "['Fabrico spent less on capital improvements than it did in 1986, when it added new plant capacity and new equipment.', 'Because of escalating costs for raw materials, prices charged by Fabrico for its products were on average 42 percent higher than they were in 1986.', 'The total worldwide dollar value of clothing sales by manufacturers to customers increased by 5 percent over sales in 1986.', \"Fabrico's employees dropped earlier demands for a 25 percent increase in company-paid benefits and a 15 percent increase in hourly wages.\"]", "label": 1 }, { "id": "train_4135", "context": "Although China considers him a threat against the state, the Dalai Lama is one of the most popular world leaders according to recent polls. People across the globe respect the Dalai Lama' s commitment to religious harmony, self-determination, and humanistic values. The Dalai Lama believes that gun violence in America can only be solved by stricter background check legislation. With any luck, Congress will pass the bill during its next session.", "question": "Which one of the following best describes the flaw in the author's reasoning?", "answers": "['The argument cites an inappropriate expert.', \"The argument's extreme language detracts from its logical coherence.\", 'The argument confuses correlation with causation.', 'The argument makes a hasty generalization.']", "label": 0 }, { "id": "train_4136", "context": "Jeff: Proposed regulations concerning the use of animals in scientific experimentation would prohibit experimentation on those species that humans empathize with: dogs and horses, for example. But extensive neurological research on mammals shows that they are all capable of feeling pain, just as dogs and horses are. Hence, this proposal should be extended to all experimentation on all mammals. Miranda: Yet the issue of pain is not the crux of the matter. Experimentation on any nonhuman animal undermines respect for life itself because only humans are capable of consenting to an experiment. Since any activity that undermines respect for life diminishes the quality of all of our lives, the new regulations should ban all such experimentation.", "question": "Which one of the following is a principle that, if established, would best support Jeff's conclusion?", "answers": "['Scientific experimentation should be prohibited on any creature that is capable of feeling pain.', 'Only those experiments on animals that are known to cause those animals pain should be prohibited.', 'Regulations on the use of animals in scientific experimentation should be primarily concerned with respecting the feelings of the humans who will perform those experiments.', 'Whatever means are used to determine whether dogs and horses feel pain should also be used to determine whether other animals feel pain.']", "label": 0 }, { "id": "train_4137", "context": "Locust swarms have ravaged much of Suwathia, resulting in the loss of vital agriculture. A multinational company has recently proposed using metarhizium, a fungus that invades the exoskeleton of the locust, causing a swift death. The government of Suwathia argues that the one means of application of metarhizia-from an airplane-will require a great amount of the fungus, which is difficult to acquire. Given that the locusts propagate rapidly, the government maintains that only a complete annihilation of a locust swarm will have the desired effect.", "question": "Which of the following, if true, best argues in favor of the multinational company's plan to drop metarhizium over locust swarms?", "answers": "['Some swarms of locust can cover over 100, 000 square miles.', 'If all of the locusts are not destroyed, Suwathia will lose millions of acres of crops.', 'Only one metarhizia-infected locust is necessary to lead to the demise of an entire swarm, since metarhizia can easily spread within a swarm.', 'An airplane must fly several kilometers ahead of a swarm of locusts to ensure that the metarhizia is correctly deployed upon the rapidly moving swarm.']", "label": 2 }, { "id": "train_4138", "context": "A certain type of shrimp habitually hovers around superheated deep-sea geysers, near which the bacteria that form the shrimps' diet can be found, Because the geysers emit a faint light, scientists have concluded that the shrimps' light sensitive dorsal patches were developed to locate the geysers and thereby find food.", "question": "Which of the following, if true, casts the most doubt on the scientists' conclusion?", "answers": "['The light given off by the geysers too faint to be detected by the human eye', 'In other types of shrimp, a heat-seasing organ has developed that could serve the same geyser-detecting purposes as the light sensitive patches are said to serve.', 'The light to which the shrimps are sensitive is not the sort of light that the geysers emit', \"The heat inside a geyser's stream is sufficient to kill instantly any bacteria that move into it\"]", "label": 2 }, { "id": "train_4139", "context": "Between 1977 and 1987, the country of Ravonia lost about 12, 000 jobs in logging and wood processing, representing a 15 percent decrease in employment in the country' s timber industry. Paradoxically, this loss of jobs occurred even as the amount of wood taken from the forests of Ravoinia increased by 10 percent.", "question": "Which one of the following, if true, most helps to resolve the apparent paradox?", "answers": "[\"In 1977, overall unemployment in Ravonia was approximately 10 percent; in 1987, Ravonia's unemployment rate was 15 percent.\", 'Between 1977 and 1987, the total number of acres of timberland in Ravonia fell, while the demand for wood products increased.', 'Since 1 977, a growing proportion of the timber that has been cut in Ravonia has been exported as raw, unprocessed wood.', 'Since 1977, domestic sales of wood and wood products have increased by more than export sales have increased.']", "label": 2 }, { "id": "train_4140", "context": "Statistician: Two major studies found no causal link between medical procedure X and disorder Y, but these studies are flawed. One study looked at 1, 000 people who had undergone procedure X and the other study looked at 1, 100 people who had undergone procedure X. But because disorder Y occurs in only . 02 percent of the population, researchers would need to include many more than 1, 100 people in a study to detect even a doubling of the rate of disorder Y.", "question": "Which one of the following most accurately expresses the main conclusion of the statistician's argument?", "answers": "['Researchers should conduct more-extensive studies of procedure X to determine whether the procedure is causally linked with disorder Y.', 'Contrary to the findings of two major studies, there is reason to think that procedure X causes disorder Y.', 'The two studies cited did not reach a conclusion as to whether disorder Y results from procedure X.', 'Two studies that discovered no causal link between procedure X and disorder Y are unsound.']", "label": 3 }, { "id": "train_4141", "context": "Nuclear reactors are sometimes built in \"geologically quiet\" regions, so called by geologists because such regions are distant from plate boundaries and contain only minor faults. Since no minor fault in a geologically quiet region produces an earthquake more often than once in any given 100, 000-year period, it follows that of all potential nuclear reactor sites in such a region, those that are least likely to be struck by an earthquake are ones located near a fault that has produced an earthquake within living memory.", "question": "Which one of the following is an assumption on which the argument depends?", "answers": "['For any potential nuclear reactor site, the likelihood of being struck by an earthquake is the primary determinant of site safety.', 'Earthquake faults in geologically quiet regions produce earthquakes at least once in 100, 000 years.', 'In a geologically quiet region, every potential nuclear reactor site is near at least one minor fault.', 'Geologically quiet regions are the least dangerous regions in which to build nuclear reactors.']", "label": 2 }, { "id": "train_4142", "context": "A popular book argues that people who are successful in business have, without exception, benefited from a lot of luck on their way to success. But this is ridiculous. Anyone who has studied successful people knows that success requires a lot of hard work.", "question": "The argument commits which one of the following errors of reasoning?", "answers": "['It treats an effect of something as the cause of that thing.', 'It mistakes the claim that something is required for a purpose for the claim that it is sufficient for that purpose.', 'It takes for granted in a premise what it is trying to prove in its conclusion.', 'It accepts a view as authoritative without establishing the authority of the source of the view.']", "label": 1 }, { "id": "train_4143", "context": "Smithtown University' s fund-raisers succeeded in getting donations from 80 percent of the potential donors they contacted. This success rate, exceptionally high for university fund-raisers, does not indicate that they were doing a good job. On the contrary, since the people most likely to donate are those who have donated in the past, good fund-raisers constantly try less-likely prospects in an effort to expand the donor base. The high success rate shows insufficient canvassing effort.", "question": "Which of the following, if true, provides more support for the argument?", "answers": [ "This year the average size of the donations to Smithtown University from new donors when the university's fund-raisers had contacted was larger than the average size of donations from donors who had given to the university before.", "This year most of the donations that came to Smithtown University from people who had previously donated to it were made without the university's fund-raisers having made any contact with the donors.", "More than half of the money raised by Smithtown University's fund-raisers came from donors who had never previously donated to the university.", "Smithtown University's fund-raisers were successful in their contacts with potential donors who had never given before about as frequently as were fund-raisers for other universities in their contacts with such people." ], "label": 3 }, { "id": "train_4144", "context": "If the prosecutor wanted to charge Frank with embezzlement, then Frank would already have been indicted. But Frank has not been indicted. So clearly Frank is not an embezzler.", "question": "The flawed pattern of reasoning exhibited by which one of the following is most similar to that exhibited by the argument above?", "answers": "['If Barry had won the lottery, he would stay home to celebrate. But Barry did not win the lottery, so he will be in to work today.', 'If Makoto believed that he left the oven on, he would rush home. But Makoto is still at work. So obviously he did not leave the oven on.', 'If Tamara believed she was getting a promotion, she would come in to work early. She did come in early. So apparently she is getting a promotion.', 'If Lucy believed she was going to be fired, she would not come in to work today. She is going to be fired, so clearly she will not be coming in today.']", "label": 1 }, { "id": "train_4145", "context": "In a learning experiment a researcher ran rats through a maze. Some of the rats were blind, others deaf, others lacked a sense of smell, and others had no sensory deficiencies; yet all the rats learned the task in much the same amount of time. Of the senses other than sight, hearing, and smell, only kinesthesia had not previously been shown to be irrelevant to maze-learning. The researcher concluded on the basis of these facts that kinesthesia, the sensation of bodily movement, is sufficient for maze-learning.", "question": "The researcher's reasoning is most vulnerable to which one of the following criticisms?", "answers": "['The possibility that the interaction of kinesthesia with at least one other sense is required for maze-learning cannot be ruled out on the basis of the data above.', 'It can be determined from the data that maze-learning in rats depends on at least two sources of sensory stimulation, one of which is kinesthesia, but which of the remaining sources must also be employed is not determinable.', 'The small differences in proficiency found by the researcher did not appear to fall into a systematic pattern by group.', 'It can be determined from the data that rats can learn to run mazes by depending on kinesthesia alone, but the possibility that rats respond to nonkinesthetic stimulation is not ruled out.']", "label": 0 }, { "id": "train_4146", "context": "Zoo director: The city is in a financial crisis and must reduce its spending. Nevertheless, at least one reduction measure in next year' s budget, cutting City Zoo' s funding in half, is false economy. The zoo' s current budget equals less than 1 percent of the city' s deficit, so withdrawing support from the zoo does little to help the city' s financial situation. Furthermore, the zoo, which must close if its budget is cut, attracts tourists and tax dollars to the city. Finally, the zoo adds immeasurably to the city' s cultural climate and thus makes the city an attractive place for business to locate.", "question": "Which one of the following is the main conclusion of the zoo director's argument?", "answers": "['Reducing spending is the only means the city has of responding to the current financial crisis.', \"The city's educational and cultural climate will be irreparably damaged if the zoo is forced to close.\", \"It would be false economy for the city to cut the zoo's budget in half.\", \"City Zoo's budget is only a very small portion of the city's entire budget.\"]", "label": 2 }, { "id": "train_4147", "context": "Essayist writing in 2012: At its onset, a new medium is limited to carrying content from the old medium it replaces. We are in that phase with e-books -- today' s e-books take their content from print books. Thus it is too early to understand the e-book as a medium, since it has not yet taken its ultimate form.", "question": "Which one of the following principles, if valid, most helps to justify the essayist's reasoning?", "answers": "['One medium can replace another only if it can represent richer and more varied content.', 'The ultimate form that a medium will take depends on available technology.', 'A medium cannot be understood without observing the evolution of its content.', 'No electronic medium can resemble a print medium more than it resembles other electronic media.']", "label": 2 }, { "id": "train_4148", "context": "Marisa: Existing zoning regulations must be loosened; in some places the restrictions on development are now so prohibitive as to reduce the property values of undeveloped areas significantly. Tyne: I disagree. Though it is true that the recent increase in the stringency of zoning regulations could be seen by developers as merely an activists' ploy to restrict development further, the value of natural, undisturbed areas can only be better preserved by such regulatory protection.", "question": "Tyne's response to Marisa suggests that Tyne has misinterpreted which one of the following words in Marisa's remarks?", "answers": "['prohibitive', 'regulations', 'values', 'significantly']", "label": 2 }, { "id": "train_4149", "context": "Ramirez: The film industry claims that pirated DVDs, which are usually cheaper than legitimate DVDs and become available well before a film' s official DVD release date, adversely affect its bottom line. But the industry should note what the spread of piracy indicates: consumers want lower prices and faster DVD releases. Lowering prices of DVDs and releasing them sooner would mitigate piracy' s negative effect on film industry profits.", "question": "The argument above relies on which of the following assumptions?", "answers": "['Some people who would otherwise purchase pirated DVDs would be willing to purchase legitimate DVDs if they were less expensive and released earlier than they are now.', 'Current purchasers of pirated DVDs are aware that those DVDs are not authorized by the film industry.', \"Releasing legitimate DVDs earlier would not cause any reduction in the revenue the film industry receives from the films' theatrical release.\", 'The film industry will in the future be able to produce DVDs more cheaply than is currently the case.']", "label": 0 }, { "id": "train_4150", "context": "The layouts of supermarkets are not accidental: they are part of a plan designed to make customers walk all the way to the back of the store just to pick up a loaf of bread, passing tempting displays the whole way. But supermarkets can alienate customers by placing popular items in the rear; surveys list inconvenience as shoppers' top reason for disliking supermarkets.", "question": "Which one of the following propositions does the passage most precisely illustrate?", "answers": "['Even well-thought-out plans can fail.', 'Distracting customers is not good for business.', 'Manipulation of people can have unwelcome consequences.', 'Alienation of customers is not good for business.']", "label": 2 }, { "id": "train_4151", "context": "University administrator: Saying that writing cannot be taught is as silly as saying that playing the violin cannot be taught. Of course writing can be taught. Writing classes have been taught at this university ever since it was founded.", "question": "The reasoning in the university administrator's argument is flawed in that the argument", "answers": "['rests entirely on a weak analogy', 'treats a view with contempt instead of offering evidence against it', 'relies on using a key term in two different senses', 'generalizes on the basis of mere speculation']", "label": 2 }, { "id": "train_4152", "context": "If understanding a word always involves knowing its dictionary definition, then understanding a word requires understanding the words that occur in that definition. But clearly there are people -- for example, all babies -- who do not know the dictionary definitions of some of the words they utter.", "question": "Which one of the following statements follows logically from the statements above?", "answers": "['If it is possible to understand a word without knowing its dictionary definition, then it is possible to understand a word without having to understand any other word.', 'If some words can be understood without knowing their dictionary definitions, then babies understand some words.', 'Any number of people can understand some words without knowing their dictionary definitions.', 'If some babies understand all the words they utter, then understanding a word does not always involve knowing its dictionary definition.']", "label": 3 }, { "id": "train_4153", "context": "A large group of hyperactive children whose regular diets included food containing large amounts of additives was observed by researchers trained to assess the presence or absence of behavior problems. The children were then placed on a low-additive diet for several weeks, after which they were observed again. Originally nearly 60 percent of the children exhibited behavior problems; after the change in diet, only 30 percent did so. On the basis of these data, it can be concluded that food additives can contribute to behavior problems in hyperactive children.", "question": "The evidence cited fails to establish the conclusion because", "answers": "['there is no evidence that the behavior of some of the children was unaffected by additives', 'the evidence is consistent with the claim that some children exhibit more frequent behavior problems after being on the low-additive diet than they had exhibited when first observed', 'exactly how many children exhibited behavior problems after the change in diet cannot be determined, since the size of the group studied is not precisely given', 'there is no way to know what changes would have occurred without the change of diet, since only children who changed to a low-additive diet were studied']", "label": 3 }, { "id": "train_4154", "context": "Columnist: The relief from the drudgery of physical labor that much modern technology affords its users renders them dependent on this technology, and, more importantly, on the elaborate energy systems required to run it. This leads to a loss of self-sufficiency. Clearly, then, in addition to undermining life' s charm, much modern technology diminishes the overall well-being of its users.", "question": "Which one of the following is an assumption required by the columnist's argument?", "answers": "[\"Anything causing a loss in life's charm is unjustifiable unless this loss is compensated by some gain.\", 'Physical labor is essential to a fulfilling life.', 'People are not free if they must depend on anything other than their own capacities.', \"Self-sufficiency contributes to a person's wellbeing.\"]", "label": 3 }, { "id": "train_4155", "context": "Critic: An art historian argues that because fifteenth- century European paintings were generally more planimetric (that is, two-dimensional with no attempt at suggesting depth) than were sixteenth-century paintings, fifteenth-century painters had a greater mastery of painting than did sixteenth-century painters. However, this conclusion is wrong. Fifteenth-century European painters did not have a greater mastery of painting, for the degree to which a painting is planimetric is irrelevant to the painter' s mastery.", "question": "The argument is flawed in that it", "answers": "[\"takes a necessary condition for an argument's being inadequate to be a sufficient condition for an argument's being inadequate\", 'rejects a position merely because the proponent of the position has other objectionable views', 'bases its conclusion on two claims that contradict each other', 'rejects a position on the grounds that an inadequate argument has been made for it']", "label": 3 }, { "id": "train_4156", "context": "The skulls and pelvic bones of some species of dinosaur share characteristics with the skulls and pelvic bones of all modern birds. Even though not all dinosaurs have these characteristics, there are scientists who claim that all animals that do have these characteristics are dinosaurs.", "question": "If the statements above and the claim of the scientists are true, which of the following must also be true?", "answers": "['All dinosaurs are birds.', 'Birds share more characteristics with dinosaurs than they do with other animals.', 'All animals whose skulls share the characteristics of those of modem birds also have pelvic bones that are similar to those of modern birds.', 'Modern birds are dinosaurs.']", "label": 3 }, { "id": "train_4157", "context": "Julian plays the lottery every day. Julian always manages to buy his daily ticket, but he often struggles to pay his rent. Despite the billion-to-one odds, Julian is always close to hitting the power ball. The Local Lottery Commission' s commercials state that the more lottery tickets an individual purchases, the better his or her chances are at winning. Thus, Julian wisely spends his money and will likely win the lottery in the near future.", "question": "What is the flaw in the argument's reasoning?", "answers": "['The argument justifies its conclusion based on a hasty generalization.', \"The argument's extreme language detracts from its logical coherence.\", 'The argument confuses correlation with causation.', 'The argument makes several logical leaps and assumptions.']", "label": 0 }, { "id": "train_4158", "context": "Several excellent candidates have been proposed for the presidency of United Wire, and each candidate would bring to the job different talents and experience. If the others are compared with Jones, however, it will be apparent that none of them has her unique set of qualifications. Jones, ttherefore, is best qualified to be the new president of United Wire.", "question": "The argument is vulnerable to criticism on the ground that it", "answers": "['refutes a distorted version of an opposing position', 'seeks to distinguish one member of a group on the basis of something that applies to all', 'supports a universal claim on the basis of a single example', 'uses flattery to win over those who hold an opposing position']", "label": 1 }, { "id": "train_4159", "context": "Critics worry that pessimistic news reports about the economy harm it by causing people to lose confidence in the economy, of which everyone has direct experience every day. Journalists respond that to do their jobs well they cannot worry about the effects of their work. Also, studies show that people do not defer to journalists except on matters of which they have no direct experience.", "question": "The statements above, if true, most strongly support which one of the following?", "answers": "[\"Critics who think that the economy is affected by the extent of people's confidence in it are wrong.\", \"News reports about the economy are unlikely to have a significant effect on people's opinions about the state of the economy.\", 'Pessimistic news reports about such matters as foreign policy, of which people do not have experience every day, are likely to have a negative impact.', \"Journalists need not be deeply concerned about their reporting's effects on the well-being of the average citizen.\"]", "label": 1 }, { "id": "train_4160", "context": "Dried grass clippings mixed into garden soil gradually decompose, providing nutrients for beneficial soil bacteria. This results in better-than-average plant growth. Yet mixing fresh grass clippings into garden soil usually causes poorer-than-average plant growth.", "question": "Which one of the following, if true, most helps to explain the difference in plant growth described above?", "answers": "['When a mix of fresh and dried grass clippings is mixed into garden soil, plant growth often decreases.', 'The number of beneficial soil bacteria increases whenever any kind of plant material is mixed into garden soil.', 'Some dried grass clippings retain nutrients originally derived from commercial lawn fertilizers, and thus provide additional enrichment to the soil.', 'Fresh grass clippings mixed into soil decompose rapidly, generating high levels of heat that kill beneficial soil bacteria.']", "label": 3 }, { "id": "train_4161", "context": "Moralist: Immoral actions are those that harm other people. But since such actions eventually harm those who perform them, those who act immorally do so only through ignorance of some of their actions' consequences rather than through a character defect.", "question": "Which one of the following is an assumption required by the moralist's argument?", "answers": "['None of those who knowingly harm themselves lack character defects.', 'An action harms those who perform it only if it also eventually harms others.', \"People ignorant of their actions' consequences cannot be held morally responsible for those consequences.\", 'Those who, in acting immorally, eventually harm themselves do not intend that harm.']", "label": 3 }, { "id": "train_4162", "context": "The average life expectancy of the population of Japan has risen steadily since 1960 and is now the highest national average in the world, even though heart disease among the Japanese has increased since they began to eat more of the fatty foods typical of the diets of people in Western countries.", "question": "Which of the following, if true, best helps to explain the steady rise in life expectancy that is cited above?", "answers": "['Since 1960 the decline in illnesses that kill more Japanese people than does heart disease has been greater than the increase in heart disease.', 'The life-style of some Japanese people includes regular exercise, which is thought to help the heart resist the loss of strength that accompanies aging.', 'The average Westerner is still five times more likely to develop heart disease than is the average Japanese person.', 'The Japanese diet has traditionally included many nonfatty foods that are thought to reduce the risk of developing heart disease.']", "label": 0 }, { "id": "train_4163", "context": "Pizzerias are the only restaurants that routinely record the names, addresses, and menu selections of their customers. Simply by organizing these data, they can easily identify regular, average, and infrequent customers. Ttherefore, pizzerias utilize direct-mail marketing more effectively than do other restaurants.", "question": "Which one of the following, if assumed, enables the argument's conclusion to be properly inferred?", "answers": "['Restaurants other than pizzerias cannot easily identify regular, average, and infrequent customers.', 'Restaurants that routinely record names, addresses, and menu selections of their customers always utilize direct-mail marketing more effectively than do any other restaurants.', 'For restaurants, utilizing direct-mail marketing requires the names, addresses, and menu selections of at least some customers.', 'Utilizing direct-mail marketing is rarely beneficial for restaurants that cannot identify regular, average, and infrequent customers.']", "label": 1 }, { "id": "train_4164", "context": "Philosopher: An event is intentional if it is a human action performed on the basis of a specific motivation. An event is random if it is not performed on the basis of a specific motivation and it is not explainable by normal physical processes.", "question": "Which one of the following inferences conforms most closely to the philosopher's position?", "answers": "['Yasuko continued to breathe regularly throughout the time that she was asleep. This was a human action, but it was not performed on the basis of a specific motivation. Ttherefore, her breathing was a random event.', \"Tarik left the keys untouched on the kitchen counter, but he did not do so on the basis of a specific motivation. Ttherefore, the keys' remaining on the kitchen counter was a random event.\", 'Ellis tore the envelope open in order to read its contents, but the envelope was empty. Nevertheless, because Ellis acted on the basis of a specific motivation, tearing the envelope open was an intentional event.', 'Henry lost his hold on the wrench and dropped it because the handle was slippery. This was a human action and is explainable by normal physical processes, so it was an intentional event.']", "label": 2 }, { "id": "train_4165", "context": "Competitive figure skaters are judged by panels of up to nine judges, who use a numerical scale with the highest mark being 6. To arrive at a total score, all judges' marks are summed. Competitive divers are judged by panels of five or seven judges using a scale with 10 as the highest mark. Before all judges' marks are summed to a final score, however, the highest and lowest marks are discarded in order to eliminate the possibility of bias either in favor of or against a particular diver. Competitive figure skating should adopt the approach taken in diving because it is a fairer system.", "question": "Which one of the following can be inferred from the passage above?", "answers": "['Currently, there is a greater possibility of bias in the scoring process for competitive figure skating than in that for diving.', 'There is wider disagreement among figure skating judges than among diving judges.', 'It is more likely that a diver will receive a biased total score than that a skater will.', 'It is fairer to judge a competitor on a 10-point scale than a 6-point scale.']", "label": 0 }, { "id": "train_4166", "context": "Insurgent political parties that are profoundly dissatisfied with the dominant party' s reign and justificatory ideology always produce factions whose views and aims differ as greatly from each other' s as they do from the dominant party' s. Although these factions ignore their own disagreements for the sake of defeating the dominant party, their disagreements inevitably come forward upon victory. Ttherefore, __.", "question": "Which one of the following is the most logical completion of the argument?", "answers": "['it is impossible for the different factions of a victorious insurgent party to effect the compromises necessary to keep the new party in power', 'no victorious insurgent party ever manages to stay in power for as long as the party it displaces did', 'a victorious insurgent party always faces opposition from the party it recently ousted', 'a victorious insurgent party must address the disagreements between its factions if it is to stay in power']", "label": 3 }, { "id": "train_4167", "context": "The initial causes of serious accidents at nuclear power plants have not so far been flaws in the advanced-technology portion of the plants. Rather, the initial causes have been attributed to human error, as when a worker at the Browns Mills reactor in the United States dropped a candle and started a fire, or to flaws in the plumbing, exemplified in a recent incident in Japan. Such everyday events cannot be thought unlikely to occur over the long run.", "question": "Which one of the following is most strongly supported by the statements above?", "answers": "['No serious accident will be caused in the future by some flaw in the advanced-technology portion of a nuclear power plant.', 'Now that nuclear power generation has become a part of everyday life, an ever-increasing yearly incidence of serious accidents at the plants can be expected.', 'The design of nuclear power plants attempts to compensate for possible failures of the materials used in their construction.', 'If nuclear power plants continue in operation, a serious accident at such a plant is not improbable.']", "label": 3 }, { "id": "train_4168", "context": "Scientists agree that ingesting lead harms young children. More lead paint remains in older apartment buildings than newer ones because the use of lead paint was common until only two decades ago. Yet these same scientists also agree that laws requiring the removal of lead paint from older apartment buildings will actually increase the amount of lead that children living in older apartment buildings ingest.", "question": "Which one of the following, if true, most helps to resolve the apparent discrepancy in the scientists' beliefs?", "answers": "['Other sources of lead in older apartment buildings are responsible for most of the lead that children living in these buildings ingest.', 'The money required to finance the removal of lead paint from apartment walls could be spent in ways more likely to improve the health of children.', 'Removing lead paint from walls disperses a great deal of lead dust, which is more easily ingested by children than is paint on walls.', 'Lead-free paints contain substances that make them as harmful to children as lead paint is.']", "label": 2 }, { "id": "train_4169", "context": "Unusually large and intense forest fires swept the tropics in 1997. The tropics were quite susceptible to fire at that time because of the widespread drought caused by an unusually strong El Nino, an occasional global weather phenomenon. Many scientists believe the strength of the El Nino was enhanced by the global warming caused by air pollution.", "question": "Which one of the following can be properly inferred from the information above?", "answers": "['Forest fires in the tropics are generally larger and more intense than usual during a strong El Nino.', 'If the El Nino in 1997 had not been unusually strong, few if any large and intense forest fires would have swept the tropics in that year.', 'Air pollution was largely responsible for the size and intensity of the forest fires that swept the tropics in 1997.', 'If air pollution enhanced the strength of the El Nino in 1997, then it also contributed to the widespread drought in that year.']", "label": 3 }, { "id": "train_4170", "context": "In Arumville, among those young children with elevated levels of lead in their blood, few live in houses containing lead-based paint, but many live in areas where vehicular traffic is heavy. These children' s elevated blood-lead levels probably result from lead added to gasoline, since, although gasoline sold in and around Arumville has been lead-free since 1990, lead from vehicle exhaust remains in the soil for many years.", "question": "Which of the following, if true, most strengthens the argument?", "answers": "[\"In Arumville, children's blood-lead levels typically rise during the warm summer months when children frequently play outdoors but not in the winter when children spend more time indoors.\", 'Sales of lead-based paint for use in homes was banned in Arumville in 1977.', 'Vehicle traffic in most residential areas of Arumville is heavier today than it was twenty years ago.', 'If improperly carried out, the removal of lead-based paint from older houses can significantly increase the risk that children living in such houses will ingest lead dust.']", "label": 0 }, { "id": "train_4171", "context": "Researcher: People are able to tell whether a person is extroverted just by looking at pictures in which the person has a neutral expression. Since people are also able to tell whether a chimpanzee behaves dominantly just by looking at a picture of the chimpanzee' s expressionless face, and since both humans and chimpanzees are primates, we conclude that this ability is probably not acquired solely through culture but rather as a result of primate biology.", "question": "Which one of the following, if true, most strengthens the researcher's argument?", "answers": "['Some of the pictures of people used in the experiments were composites of several different people.', 'People are generally unable to judge the dominance of bonobos, which are also primates, by looking at pictures of them.', 'Any common ancestor of humans and chimpanzees would have to have lived over 7 million years ago.', 'Extroversion in people and dominant behavior in chimpanzees are both indicators of a genetic predisposition to assertiveness.']", "label": 3 }, { "id": "train_4172", "context": "Medications with an unpleasant taste are generally produced only in tablet, capsule, or soft-gel form. The active ingredient in medication M is a waxy substance that cannot tolerate the heat used to manufacture tablets because it has a low melting point. So, since the company developing M does not have soft-gel manufacturing technology and manufactures all its medications itself, M will most likely be produced in capsule form.", "question": "The conclusion is most strongly supported by the reasoning in the argument if which one of the following is assumed?", "answers": "['Medication M can be produced in liquid form.', 'Most medications with a low melting point are produced in soft-gel form.', 'No medication is produced in both capsule and soft-gel form.', 'Medication M has an unpleasant taste.']", "label": 3 }, { "id": "train_4173", "context": "Police chief: During my tenure as chief, crime in this city has fallen by 20 percent. This is clearly the result of my policing strategy, which uses real-time crime data and focuses police resources on the areas with the most crime.", "question": "Which one of the following, if true, most calls into question the police chief's explanation for the drop in crime?", "answers": [ "The crime rate in the country as a whole fell by about 30 percent during the police chief's tenure.", "The crime rate in the police chief's city is higher now than it was several decades before the chief's tenure began.", "The crime rate in the police chief's city is still significantly higher than in many other cities.", "The crime rate in the police chief's city fell significantly during the first few years of the chief's tenure, then it leveled off." ], "label": 0 }, { "id": "train_4174", "context": "Scientist: A small group of islands near Australia is inhabited by several species of iguana; closely related species also exist in the Americas, but nowhere else. The islands in question formed long after the fragmentation of Gondwana, the ancient supercontinent that included present-day South America and Australia. Thus, these species' progenitors must have rafted on floating debris across the Pacific Ocean from the Americas.", "question": "Which one of the following, if true, most weakens the scientist's argument?", "answers": "['A number of animal species that inhabit the islands are not found in the Americas.', 'Genetic analysis indicates that the iguana species on the islands are different in several respects from those found in the Americas.', 'The lineages of numerous plant and animal species found in Australia or in South America date back to a period prior to the fragmentation of Gondwana.', 'Fossils of iguana species closely related to those that inhabit the islands have been found in Australia.']", "label": 3 }, { "id": "train_4175", "context": "The government of Penglai, an isolated island, proposed eliminating outdoor advertising except for small signs of standard shape that identify places of business. Some island merchants protested that the law would reduce the overall volume of business in Penglai, pointing to a report done by the government indicating that in every industry the Penglai businesses that used outdoor advertising had a larger market share than those that did not.", "question": "Which one of the following describes an error of reasoning in the merchants' argument?", "answers": "['presupposing that there are no good reasons for restricting the use of outdoor advertising in Penglai', \"ignoring the question of whether the government's survey of the island could be objective\", \"disregarding the possibility that the government's proposed restrictions are unconstitutional\", 'assuming without giving justification that the outdoor advertising increased market share by some means other than by diverting trade from competing businesses']", "label": 3 }, { "id": "train_4176", "context": "Reducing speed limits neither saves lives nor protects the environment. This is because the more slowly a car is driven, the more time it spends on the road spewing exhaust into the air and running the risk of colliding with other vehicles.", "question": "The argument's reasoning is flawed because the argument", "answers": "['fails to consider that if speed limits are reduced, increased driving times will increase the number of cars on the road at any given time', 'neglects the fact that some motorists completely ignore speed limits', 'presumes, without providing justification, that drivers run a significant risk of collision only if they spend a lot of time on the road', 'presumes, without providing justification, that total emissions for a given automobile trip are determined primarily by the amount of time the trip takes']", "label": 3 }, { "id": "train_4177", "context": "When the economy is in a recession, overall demand for goods and services is low. If overall demand for goods and services is low, bank interest rates are also low. Ttherefore, if bank interest rates are not low, the economy is not in a recession.", "question": "The reasoning in which one of the following most closely parallels the reasoning in the argument above?", "answers": "['If pterodactyls flew by flapping their wings, they must have been warm-blooded, so if they were cold-blooded, they must have flown only by gliding, if they flew at all.', 'If you want to put in pleats, you will have to double the amount of material for the skirt, and that means you will have none left for the top, so if you put in pleats you will not be able to make the top.', 'If the fish is ready, it is cooked all the way through, and if it is cooked through it will be white, so if the fish is not white, it is not ready.', 'If economic forecasters are right, there will be inflation, and if there is inflation, the governing party will lose the election, so if it does lose the election, the economic forecasters were right.']", "label": 2 }, { "id": "train_4178", "context": "This stamp is probably highly valuable, since it exhibits a printing error. The most important factors in determining a stamp' s value, assuming it is in good condition, are its rarity and age. This is clearly a fine specimen, and it is quite old as well.", "question": "The conclusion is properly inferred if which one of the following is assumed?", "answers": "['Most stamps with printing errors are already in the hands of collectors.', \"Rarity and age are of equal importance to a stamp's value.\", 'Printing errors are always confined to a few individual stamps.', 'Even old and rare stamps are usually not valuable if they are in poor condition.']", "label": 2 }, { "id": "train_4179", "context": "Cotton grass, which grows only in arctic regions, has been the only summertime source of protein available to caribou. Caribou that do not get adequate amounts of protein in the summer are unable to reproduce the following year. Rising average temperatures in arctic regions, however, are causing cotton grass to disappear. Ttherefore, if the warming trend continues, caribou are likely to become extinct.", "question": "Which of the following is an assumption on which the argument depends?", "answers": "['Caribou that do not eat enough protein to reproduce do not live as long as caribou that do', 'The warming trend in arctic regions will not enable other plants capable of providing protein to caribou to grow there.', 'The caribou is the only animal that depends on cotton grass as a major source of food.', \"Cotton grass is the only one of the caribou's food source that is becoming scarce as temperatures rise in arctic regions.\"]", "label": 1 }, { "id": "train_4180", "context": "Driving the steep road to the mountaintop Inca ruins of Machu Picchu is potentially dangerous and hiking there is difficult. Now the Peruvian government is installing a cable car that will make access much easier, and hence result in a large increase in tourism. However, since the presence of large numbers of tourists tends to accelerate the deterioration of a site, installation of the cable car is certain to result in harm to the ruins.", "question": "Which of the following, if true, most seriously calls into question the argument?", "answers": "['The daily number of tourists that are expected to take the cable car to Machu Piccu is smaller than the original resident population of Incas.', 'The construction of the cable car terminal at Machu Picchu will require the use of potentially damaging heavy machinery at the site.', 'Machu Picchu is already one of the most popular tourist sites in Peru.', 'The cable car will replace the tour buses whose large wheels and corrosive exhaust at present do significant damage to the site.']", "label": 3 }, { "id": "train_4181", "context": "Bram Stoker' s 1897 novel Dracula portrayed vampires -- the \"undead\" who roam at night to suck the blood of living people -- as able to turn into bats. As a result of the pervasive influence of this novel, many people now assume that a vampire' s being able to turn into a bat is an essential part of vampire myths. However, this assumption is false, for vampire myths existed in Europe long before Stoker' s book.", "question": "Which one of the following is an assumption on which the argument depends?", "answers": "['Vampire myths in Central and South America, where real vampire bats are found, portray vampires as able to turn into bats.', 'At the time he wrote Dracula, Stoker was familiar with earlier European vampire myths.', \"At least one of the European vampire myths that predated Stoker's book did not portray vampires as strictly nocturnal.\", \"At least one of the European vampire myths that predated Stoker's book did not portray vampires as able to turn into bats.\"]", "label": 3 }, { "id": "train_4182", "context": "A pharmaceutical company wanted to test a new antistress medication -- Stress-Ex. To do so, they placed an advertisement in the local business magazine asking for highly stressed business executives to take part in a clinical study. Forty such individuals signed up for the study. The participants were asked to rate their level of stress, and then they were placed in a room over a three-day period and given Stress-Ex at regular intervals. At the conclusion of the study, participants were once again asked to rate their level of stress. Thirty of the participants rated their stress level lower at the end of the study. On the basis of this study, Stress-Ex was deemed a success.", "question": "The evidence cited fails to establish the conclusion because", "answers": "['there is no evidence that using greater quantities of Stress-Ex would reduce stress', 'the study did not clearly determine the long-term effects of Stress-Ex', \"some of the participants' stress levels did not decrease during the study\", 'there is no way to know whether stress levels would have gone down without Stress-Ex']", "label": 3 }, { "id": "train_4183", "context": "In the past five years, the percentage of money spent on discovering renewable yet expensive sources of energy has decreased. An increasing amount of money is being spent on developing nonrenewable resources, which are quite inexpensive to obtain. Regrettably, the nonrenewable resources have proven to be impracticable for daily use. Clearly, less money is being spent on practicable energy sources than was being spent five years ago.", "question": "Which one of the following, if true, allows the conclusion above to be properly drawn?", "answers": "['Most of the money spent on developing renewable resources came from private donors.', 'The total investment in all energy resources is at an all time high.', 'The cost of nonrenewable resources is expected to rise in the next five years.', 'The total amount of money spent on developing energy resources has slowly declined over the past five years.']", "label": 3 }, { "id": "train_4184", "context": "Zoologist: Every domesticated large mammal species now in existence was domesticated thousands of years ago. Since those days, people undoubtedly tried innumerable times to domesticate each of the wild large mammal species that seemed worth domesticating. Clearly, ttherefore, most wild large mammal species in existence today either would be difficult to domesticate or would not be worth domesticating.", "question": "The zoologist's argument requires the assumption that", "answers": "['not all of the large mammal species that were domesticated in the past are still in existence', 'in spite of the difficulties encountered, at one time or another people have tried to domesticate each wild large mammal species', 'of all the domesticated large mammal species in existence today, the very first to be domesticated were the easiest to domesticate', 'it is not much easier today to domesticate wild large mammal species than it was in the past']", "label": 3 }, { "id": "train_4185", "context": "All oceangoing ships carry seawater ballast tanks whose weight improves stability. To maintain the ship' s proper stability, water must be pumped out of these tanks when cargo is loaded and into them when cargo is unloaded. As a result, sea creatures often get into the tanks and are then inadvertently deposited into new habitats, where they can wreak ecological havoc. One viable way of addressing this problem would be to empty and then immediately refill the tanks in midocean, since midocean creatures and coastal sea creatures usually cannot survive in one another' s habitats.", "question": "Which one of the following is an assumption the argument requires?", "answers": "['Sea creatures have rarely, if ever, wreaked ecological havoc in a new habitat, unless they have been able to survive in that habitat after having been deposited there by oceangoing ships.', 'There are at least some oceangoing ships whose stability could be adequately maintained while emptying and refilling their ballast tanks in midocean.', 'Currently, seawater is pumped into or out of the ballast tanks of oceangoing ships to maintain proper stability only when unloading or loading cargo.', \"An oceangoing ship's ballast tanks could be emptied and refilled in midocean only in conditions of calm air and flat seas.\"]", "label": 1 }, { "id": "train_4186", "context": "Boreal owls range over a much larger area than do other owls of similar size. Scientists have hypothesized that it is scarcity of prey that leads the owls to range so widely. This hypothesis would be hard to confirm directly, since it is not possible to produce a sufficiently accurate count of the populations of small mammals inhabiting the forests where boreal owls live. Careful study of owl behavior has, however, shown that boreal owls do range over larger areas when they live in regions where food of the sort eaten by small mammals is comparatively sparse. This indicates that the scientists' hypothesis is not sheer speculation.", "question": "In the argument given, the two boldfaced portions play which of the following roles?", "answers": "['The first describes a position that the argument opposes; the second states the main conclusion of the argument.', 'The first presents an explanatory hypothesis; the second presents evidence tending to support this hypothesis.', 'The first presents an explanatory hypothesis; the second presents evidence to support an alternative explanation.', 'The first describes a position that the argument opposes; the second presents evidence to undermine the support for the position being opposed.']", "label": 1 }, { "id": "train_4187", "context": " In countries where automobile insurance includes compensation for whiplash injuries sustained in automobile accidents, reports of having suffered such injuries are twice as frequent as they are in countries where whiplash is not covered. Presently, no objective test for whiplash exists, so it is true that spurious reports of whiplash injuries cannot be readily identified. Nevertheless, these facts do not warrant the conclusion drawn by some commentators, that in the countries with the higher rates of reported whiplash injuries, half of the reported cases are spurious. Clearly, in countries where automobile insurance does not include compensation for whiplash, people often have little incentive to report whiplash injuries that they actually have suffered .", "question": "In the argument given, the two boldfaced portions play which of the following roles?", "answers": "['The first is a claim that the argument disputes; the second is a conclusion that has been based on that claim.', 'The first is a claim that has been used to support a conclusion that the argument accepts; the second is that conclusion.', 'The first is evidence that has been used to support a conclusion for which the argument provides further evidence; the second is the main conclusion of the argument.', 'The first is a finding whose implications are at issue in the argument; the second is a claim presented in order to argue against deriving certain implications from that finding.']", "label": 3 }, { "id": "train_4188", "context": "The growing popularity of computer-based activities was widely expected to result in a decline in television viewing, since it had been assumed that people lack sufficient free time to maintain current television-viewing levels while spending increasing amounts of free time on the computer. That assumption, however, is evidently false: in a recent mail survey concerning media use, a very large majority of respondents who report increasing time spent per week using computers report no change in time spent watching television.", "question": "In order to evaluate the argument, it would be most useful to determine which of the following?", "answers": "['Whether a large majority of the computer owners in the survey reported spending increasing amounts of time per week using computers', 'Whether the survey collected information about the amount of money respondents spent on free-time media use', 'Whether the amount of time spent watching television is declining among people who report that they rarely or never use computers', \"Whether the survey respondents' reports of time spent using computers included time spent using computers at work\"]", "label": 3 }, { "id": "train_4189", "context": "If the country' s income taxes are decreased, the country' s economy will not be strengthened, because many of the country' s public employees would become unemployed, thereby further weakening the country' s economy.", "question": "The pattern of reasoning in the argument above is most similar to that in which one of the following arguments?", "answers": "['A decrease in taxes on businesses will strengthen the economy because it will encourage businesses to expand employment opportunities, even though many workers will not qualify for the new opportunities.', 'An increase in interest rates will not increase the number of jobs, because increased interest rates will put many companies out of business, and this result will decrease the number of jobs.', 'If both government spending and income taxes are decreased, growth in private businesses might occur, because people will have more money to invest, but decreases in both spending and taxes are unlikely.', \"Tax incentives will lure new businesses to the region, thereby increasing the region's employment, but even if no tax incentives are offered, employment in the region is likely to increase.\"]", "label": 1 }, { "id": "train_4190", "context": "Professor Chan: The literature department' s undergraduate courses should cover only true literary works, and not such frivolous material as advertisements. Professor Wigmore: Advertisements might or might not be true literary works but they do have a powerfully detrimental effect on society -- largely because people cannot discern their real messages. The literature department' s courses give students the critical skills to analyze and understand texts. Ttherefore, it is the literature department' s responsibility to include the study of advertisements in its undergraduate courses.", "question": "Which one of the following principles most strongly supports Professor Wigmore's argument?", "answers": "[\"The literature department's courses ought to enable students to analyze and understand any text that could have a harmful effect on society.\", 'Advertisements ought to be framed in such a way that their real messages are immediately clear.', 'All undergraduate students ought to take at least one course that focuses on the development of critical skills.', \"Any text that is subtly constructed and capable of affecting people's thought and action ought to be considered a form of literature.\"]", "label": 0 }, { "id": "train_4191", "context": "Every year, new reports appear concerning the health risks posed by certain substances, such as coffee and sugar. One year an article claimed that coffee is dangerous to one' s health. The next year, another article argued that coffee has some benefits for one' s health. From these contradictory opinions, we see that experts are useless for guiding one' s decisions about one' s health.", "question": "Which one of the following most accurately describes a flaw in the argument above?", "answers": "['The argument presumes, without providing justification, that because expert opinion is trustworthy in one case, it must ttherefore be trustworthy in all cases.', \"The argument fails to consider that coffee may be harmful to one's health in some respects and beneficial in others.\", \"The argument takes for granted that coffee is dangerous to one's health.\", \"The argument presumes, without providing warrant, that one always wants expert guidance in making decisions about one's health.\"]", "label": 1 }, { "id": "train_4192", "context": "Though many insects die soon after reproducing for the first time, some may live for years after the survival of the next generation has been secured. Among the latter are some insects that work for the benefit of the ecosystem -- for example, bees.", "question": "Which one of the following can be properly inferred from the information above?", "answers": "['Insects that do not play a vital role in the ecosystem are more likely to die after reproducing for the first time.', 'Most bees live well beyond the onset of the generation that follows them.', 'Survival of the species, rather than of the individual, is the goal of most insect populations.', 'Those bees that reproduce do not always die soon after reproducing for the first time.']", "label": 3 }, { "id": "train_4193", "context": "From the observation that each member of a group could possess a characteristic, it is fallacious to conclude immediately that it is possible for all the group' s members to possess the characteristic. An example in which the fallacy is obvious: arguing that because each of the players entering a tennis tournament has a possibility of winning it, there is ttherefore a possibility that all will win the tournament.", "question": "Which one of the following commits the fallacy described above?", "answers": "['Each of the many nominees could be appointed to any one of the three openings on the committee. Ttherefore it is possible for all of the nominees to be appointed to the openings on the committee.', 'Each of the candidates for mayor appears at first glance to possess the necessary qualifications. It would ttherefore be a mistake to rule out any of them without more careful examination.', 'You can fool some of the people all of the time and all of the people some of the time, but you cannot fool all of the people all of the time.', 'It is estimated that ten million planets capable of supporting life exist in our galaxy. Thus to rule out the possibility of life on worlds other than Earth, ten million planetary explorations would be needed.']", "label": 0 }, { "id": "train_4194", "context": "Editorial: An arrest made by a Midville police officer is provisional until the officer has taken the suspect to the police station and the watch commander has officially approved the arrest. Such approval is denied if the commander judges that the evidence on which the provisional arrest is based is insufficient. A government efficiency expert has found that almost all provisional arrests meet standards for adequacy of evidence that watch commanders enforce. The expert ttherefore recommends that the watch commander' s approval should no longer be required since the officers' time spent obtaining approval is largely wasted. This recommendation should be rejected as dangerous , however, since there is no assurance that the watch commanders' standards will continue to be observed once approval is no longer required.", "question": "In the editorial, the two portions in boldface play which of the following roles?", "answers": "['The first is a proposal against which the editorial is directed; the second is a judgment reached by the editorial concerning that proposal.', 'The first is a recommendation that the editorial questions; the second provides evidence against that recommendation.', 'The first is a position that the editorial challenges; the second is a judgment that was made in support of that challenged position.', 'The first is a recommendation made by the editorial; the second acknowledges a potential objection against that recommendation.']", "label": 0 }, { "id": "train_4195", "context": "The chemical adenosine is released by brain cells when those cells are active. Adenosine then binds to more and more sites on cells in certain areas of the brain, as the total amount released gradually increases during wakefulness. During sleep, the number of sites to which adenosine is bound decreases. Some researchers have hypothesized that it is the cumulative binding of adenosine to a large number of sites that causes the onset of sleep.", "question": "Which of the following, if true, provides the most support for the researchers' hypothesis?", "answers": "['Stress resulting from a dangerous situation can preserve wakefulness even when brain levels of bound adenosine are high.', 'Some areas of the brain that are relatively inactive nonetheless release some adenosine.', 'Even after long periods of sleep when adenosine is at its lowest concentration in the brain, the number of brain cells bound with adenosine remains very large.', 'Caffeine, which has the effect of making people remain wakeful, is known to interfere with the binding of adenosine to sites on brain cells.']", "label": 3 }, { "id": "train_4196", "context": "Automotive Company President: We spend more money on developing vehicles that produce low emissions than on any other area of research. This shows our concern for the environment. Environmentalist: You have no such concern. Your real concern was revealed in your admission to the press that the consumer demand for vehicles that produce low emissions has grown by 400% in the last two years.", "question": "The environmentalist's conclusion would be properly drawn if it were true that", "answers": "['Automotive Companies cannot have more than one motive for developing vehicles that produce low emissions.', 'The Automotive Company had fewer sales in the last two years.', 'The Automotive Company is concerned about making more money.', 'The Automotive Company has lost market share in the last two years.']", "label": 0 }, { "id": "train_4197", "context": "Sociologist: The welfare state cannot be successfully implemented because it rests on the assumption that human beings are unselfish -- a seemingly false assumption. The welfare state is feasible only if wage earners are prepared to have their hard-earned funds used to help others in greater need, and that requires an unselfish attitude. But people innately seek their own well-being, especially when the interest of others threaten it.", "question": "Which one of the following most accurately expresses the main conclusion of the sociologist's argument?", "answers": "['The welfare state will not work.', 'The interests of the less fortunate impinge on the interests of others.', 'The assumption that human beings are unselfish is false.', 'The welfare state unfairly asks those who work hard to help those in greater need.']", "label": 0 }, { "id": "train_4198", "context": "The population of peregrine falcons declined rapidly during the 1950' s and 1960' s and reached an all-time low in the early 1970' s. The decline was attributed by scientists to the widespread use of the pesticide DDT in rural areas.", "question": "Which of the following, if true, gives the strongest support to the scientists' claim?", "answers": "['Peregrine falcons, like other birds of prey, abandon eggs that have fallen out of the nest, even if the eggs remain intact.', 'In the time since the use of DDT was banned in 1972, the population of peregrine falcons has been steadily increasing.', 'DDT was not generally in use in areas devoted to heavy industry.', 'Other birds of prey, such as the osprey, the bald eagle, and the brown pelican, are found in the same areas as is the peregrine falcon.']", "label": 1 }, { "id": "train_4199", "context": "Government official: Currently many business travelers rely on a one-hour flight between the cities, but complain about congestion and delays at the airport. The new high-speed railway between Hamport and Dudley, Aplandia's two major financial hubs, will be able to make the same trip in slightly less than three hours. Given this option, many who fly between the two cities during the week will choose the railway, since it will allow them to travel between the city centers in less time than a flight with the expected delays plus the commute to and from airports would take.", "question": "Which of the following, if true, would most cast doubt on his proposal above?", "answers": "['Several trains will be scheduled throughout the day to accommodate a large demand should the route become popular.', 'The majority of those who commute between Hamportand Dudley say that they will continue doing so once the proposed railway is fully constructed.', 'Twice within the last two decades there have been significant recessions in Aplandia.', \"The proposed speed at which trains are to travel will produce static electricity that will interfere with other trains' equipment, thereby causing significant delays.\"]", "label": 3 }, { "id": "train_4200", "context": "Despite the fact that the health-inspection procedure for catering establishments are more stringent than those for ordinary restaurant, more of the cases of food poisoning reported to the city health department were brought on by banquets served by catering services than were brought on by restaurant meals.", "question": "Which of the following, if true, helps explain the apparent paradox in the statement above?", "answers": "['People are unlikely to make a connection between a meal they have eaten and a subsequent illness unless the illness strikes a group who are in communication with one another.', 'Many restaurant provide catering services for banquets in addition to serving individual meals.', 'A significantly large number of people eat in restaurants than attend catered banquets in any given time period.', 'Catering establishments know how many people they expect to serve, and ttherefore are less likely than restaurants to have, and serve, leftover foods, a major source of food poisoning.']", "label": 0 }, { "id": "train_4201", "context": "Naturalist: Different nonhuman primate species exhibit many contrasts in behavior. If a zookeeper leaves a screwdriver within reach of a chimpanzee, the animal is likely to examine and play with it for a time, and then move on to something else. In the same circumstances, an orangutan is likely to pretend to ignore the tool at first; later, in the zookeeper ' s absence, the orangutan may use the screwdriver to try to dismantle its cage.", "question": "Which one of the following is most strongly supported by the naturalist's statements?", "answers": "['Orangutans are the most intelligent of nonhuman primates.', 'Orangutans dislike being caged more than chimpanzees do.', 'Some nonhuman primates are capable of deception.', 'Not all nonhuman primates understand tool use.']", "label": 2 }, { "id": "train_4202", "context": "To find out how barn owls learn how to determine the direction from which sounds originate, scientists put distorting lenses over the eyes of young barn owls before the owls first opened their eyes. The owls with these lenses behaved as if objects making sounds were farther to the right than they actually were. Once the owls matured, the lenses were removed, yet the owls continued to act as if they misjudged the location of the source of sounds. The scientists consequently hypothesized that once a barn owl has developed an auditory scheme for estimating the point from which sounds originate, it ceases to use vision to locate sounds.", "question": "The scientists' reasoning is vulnerable to which one of the following criticisms?", "answers": "[\"It fails to consider whether the owls' vision was permanently impaired by their having worn the lenses while immature.\", 'It uses as evidence experimental results that were irrelevant to the conclusion.', 'It neglects to consider how similar distorting lenses might affect the behavior of other bird species.', 'It attributes human reasoning processes to a nonhuman organism.']", "label": 0 }, { "id": "train_4203", "context": "The public is well aware that high blood cholesterol levels raise the risk of stroke caused by blood clots. But a recent report concludes that people with low blood cholesterol levels are at increased risk of the other lethal type of stroke -- cerebral hemorrhage, caused when a brain artery bursts. The report suggests that because blood cholesterol plays a vital role in maintaining cell membranes, low blood cholesterol weakens artery walls, making them prone to rupture. The conclusion thus supports a long-standing contention by Japanese researchers that Western diets better protect against cerebral hemorrhage than do non-Western diets.", "question": "The argument is based on which one of the following assumptions?", "answers": "['High blood cholesterol levels preclude the weakening of artery walls.', 'Cerebral hemorrhages are more dangerous than strokes caused by blood clots.', 'Western diets result in higher blood cholesterol levels than do non-Western diets.', 'People who have low blood pressure are at increased risk of cerebral hemorrhage.']", "label": 2 }, { "id": "train_4204", "context": "Domesticated animals, such as dogs, have come into existence by the breeding of only the individuals of a wild species that are sufficiently tame. For example, if when breeding wolves one breeds only those that display tameness when young, then after a number of generations the offspring will be a species of dog. Ttherefore, all animals can, in principle, be bred for domesticity.", "question": "Which one of the following, if true, most weakens the argument?", "answers": "['In some animal species, no members ever display tameness.', 'Domesticated animals cannot be turned into wild species by breeding only those animals that display some wild characteristics.', 'In some animal species, wild members mate more frequently than tame members.', 'In some domesticated animal species, some members are much more tame than other members.']", "label": 0 }, { "id": "train_4205", "context": "For the past 13 years, high school guidance counselors nationwide have implemented an aggressive program to convince high school students to select careers requiring college degrees. The government reported that the percentage of last year' s high school graduates who went on to college was 15 percent greater than the percentage of those who graduated 10 years ago and did so. The counselors concluded from this report that the program had been successful.", "question": "The guidance counselors'reasoning depends on which one of the following assumptions about high school graduates?", "answers": "['The number of graduates who went on to college remained constant each year during the 10-year period.', \"Many of last year's graduates who went on to college did so in order to prepare for careers requiring college degrees.\", 'Some of the graduates who went on to college never received guidance from a high school counselor.', 'Any college courses that the graduates take will improve their career prospects.']", "label": 1 }, { "id": "train_4206", "context": "Children clearly have a reasonably sophisticated understanding of what is real and what is pretend. Once they have acquired a command of language, we can ask them which is which, and they generally get it right. Even a much younger child who runs away when she sees her father roaring and prowling like a lion does not act as though she thinks her father is actually a lion. If she believed that, she would be terrified. The pleasure children get from make-believe would be impossible to explain if they could not distinguish the real from the pretend.", "question": "Which one of the following most accurately expresses the overall conclusion drawn in the argument?", "answers": "['Children would be terrified if they believed they were in the presence of a real lion.', 'Children apparently have a reasonably sophisticated understanding of what is real and what is pretend.', 'Children who have acquired a command of language generally answer correctly when asked about whether a thing is real or pretend.', 'The pleasure children get from make-believe would be impossible to explain if they could not distinguish between what is real and what is pretend.']", "label": 1 }, { "id": "train_4207", "context": "Madden: Industrialists address problems by simplifying them, but in farming that strategy usually leads to oversimplification. For example, industrialists see water retention and drainage as different and opposite functions -- that good topsoil both drains and retains water is a fact alien to industrial logic. To facilitate water retention, they use a terrace or a dam; to facilitate drainage, they use drain tile, a ditch, or a subsoiler. More farming problems are created than solved when agriculture is the domain of the industrialist, not of the farmer.", "question": "The situation as Madden describes it best illustrates which one of the following propositions?", "answers": "['The handling of water drainage and retention is the most important part of good farming.', 'Industrial solutions for problems in farming should never be sought.', 'Farmers are better than anyone else at solving farming problems.', 'The problems of farming should be viewed in all their complexity.']", "label": 3 }, { "id": "train_4208", "context": "Not all works of art represent something, but some do, and their doing so is relevant to our aesthetic experience of them; representation is ttherefore an aesthetically relevant property. Whether a work of art possesses this property is dependent upon context. Yet there are no clear criteria for determining whether context-dependent properties are present in an object, so there cannot be any clear criteria for determining whether an object qualifies as art.", "question": "The reasoning above is questionable because it fails to exclude the possibility that", "answers": "['aesthetically relevant properties other than representation can determine whether an object is a work of art', 'some objects that represent things other than themselves are not works of art', 'because some works of art are nonrepresentational, there is no way of judging our aesthetic experience of them', 'an object may have some aesthetic properties and not be a work of art']", "label": 0 }, { "id": "train_4209", "context": "Social critic: One of the most important ways in which a society socializes children is by making them feel ashamed of their immoral behavior. But in many people this shame results in deep feelings of guilt and selfloathing that can be a severe hardship. Thus, moral socialization has had a net effect of increasing the total amount of suffering.", "question": "The social critic's argument is most vulnerable to criticism on the grounds that it", "answers": "['presumes, without providing justification, that a phenomenon that supposedly increases the total amount of suffering in a society should ttherefore be changed or eliminated, regardless of its beneficial consequences', 'presumes, without providing justification, that if many people have a negative psychological reaction to a phenomenon, then no one can have a positive reaction to that phenomenon', 'fails to address adequately the possibility that one phenomenon may causally contribute to the occurrence of another, even though the two phenomena do not always occur together', 'takes for granted that a behavior that sometimes leads to a certain phenomenon cannot also significantly reduce the overall occurrence of that phenomenon']", "label": 3 }, { "id": "train_4210", "context": "Oxygenated petrol, although it reduces pollution, causes frequent stalling in poorly maintained automobiles. However, in laboratory tests of automobiles that had been driven 100, 000 kilometers, those that had regularly used oxygenated petrol stalled less than those that had regularly used nonoxygenated petrol.", "question": "Which one of the following, if true, most helps to explain the results of the laboratory tests described above?", "answers": "['In conducting the tests, the laboratory used each type of petrol on each type of engine.', 'Drivers whose automobiles regularly exhibit adverse effects from oxygenated petrol generally cease to notice the adverse effects by the time their automobiles have been driven 100, 000 kilometers.', 'The adverse effects from oxygenated petrol can be distinguished from mechanical engine problems in the laboratory tests.', 'Automobile owners who regularly use oxygenated petrol get more frequent engine maintenance because of the adverse effects from the petrol.']", "label": 3 }, { "id": "train_4211", "context": "A manufacturer of workstations for computer-aided design seeks to increase sales to its most important corporate customers. Its strategy is to publish very low list prices for workstations in order to generate interest among the buyers for those corporations.", "question": "Which of the following, if characteristic of the marketplace, would tend to cause the manufacturer's strategy to fail?", "answers": "['Customers differ significantly in the percentage of resources they can devote to computer workstations.', 'The capabilities of workstations suitable for given jobs are not significantly different among various manufacturers.', 'Buyers for corporations that purchase workstations for computer-aided design receive bonuses for negotiating large discounts from the list price.', \"The proposed list prices would seem low to a typical buyer for the manufacturer's most important corporate customers.\"]", "label": 2 }, { "id": "train_4212", "context": "Lucien: Public-housing advocates claim that the many homeless people in this city are proof that there is insufficient housing available to them and ttherefore that more low-income apartments are needed. But that conclusion is absurd. Many apartments in my own building remain unrented and my professional colleagues report similar vacancies where they live. Since apartments clearly are available, homelessness is not a housing problem. Homelessness can, ttherefore, only be caused by people' s inability or unwillingness to work to pay the rent. Maria: On the contrary, all recent studies show that a significant percentage of this city' s homeless people hold regular jobs. These are people who lack neither will nor ability.", "question": "Lucien's argument against the public-housing advocates' position is most vulnerable to which one of the following criticisms?", "answers": "[\"It fails to address the issue, raised by the public-housing advocates' argument, of who would pay for the construction of more low-income housing.\", 'It overlooks the possibility that not all apartment buildings have vacant apartments for rent.', \"It offers no justification for dismissing as absurd the housing advocates' claim that there are many homeless people in the city.\", 'It responds to a claim in which \"available\" is used in the sense of\"affordable\" by using \"available\" in the sense of \"not occupied. \"']", "label": 3 }, { "id": "train_4213", "context": "United States advertising agencies are increasingly using interviews at shopping malls, called \"mall intercepts\", to test for advertising effectiveness, product concept viability, and consumer buying habits. Critics of mall intercepts maintain that the shopping habits of mall shoppers are not representative of those of the larger population.", "question": "Which of the following, if true, would provide evidence that most supports the critics' claim about mall intercepts?", "answers": "['Some mall shoppers patronize more than one store in any given shopping trip.', 'Mall shoppers, on average, spend 50 percent more time shopping than shoppers at other locations do.', 'Indoor malls often attract the customary numbers of shoppers even during inclement weather when outdoor malls are likely to lose business.', 'In the course of any year, 95 percent of all households in the United States have at least one member who does some shopping at a mall.']", "label": 1 }, { "id": "train_4214", "context": "Whenever Joe' s car is vacuumed, the employees of K& L Auto vacuum it; they are the only people who ever vacuum Joe' s car. If the employees of K & L Auto vacuumed Joe' s car, then Joe took his car to K & L Auto to be fixed. Joe' s car was recently vacuumed. Ttherefore, Joe took his car to K & L Auto to be fixed.", "question": "The pattern of reasoning exhibited by the argument above is most similar to that exhibited by which one of the following?", "answers": "['Jeff had to choose either a grapefruit or cereal for breakfast this morning. Given that Jeff is allergic to grapefruit, Jeff must have had cereal for breakfast this morning.', \"Emily's water glass is wet and it would be wet only if she drank water from it this morning. Since the only time she drinks water in the morning is when she takes her medication, Emily took her medication this morning.\", 'Lisa went to the hair salon today since either she went to the hair salon today or she went to the bank this morning, but Lisa did not go to the bank this morning.', 'Linda is grumpy only if she does not have her coffee in the morning, and Linda does not have her coffee in the morning only if she runs out of coffee. Ttherefore, Linda runs out of coffee only on days that she is grumpy.']", "label": 1 }, { "id": "train_4215", "context": "When presented with the evidence against him, Ellison freely admitted to engaging in illegal transactions using company facilities. However, the company obtained the evidence by illegally recording Ellison' s conversations. Ttherefore, although the company may demand that he immediately cease, it cannot justifiably take any punitive measures against him.", "question": "Which one of the following judgments best illustrates the principle illustrated by the argument above?", "answers": "[\"Ramirez was forced by the discovery of new evidence to admit that she lied about her role in managing the chief of staff 's financial affairs. Nevertheless, the board of directors cannot justifiably take action against Ramirez, because in past instances it has pardoned others guilty of similar improprieties.\", 'After a conservation officer discovered them, Kuttner admitted that he had set the illegal animal traps on his land. But, because she was trespassing at the time, the conservation officer cannot justifiably punish Kuttner in this case.', \"After Takashi told Sarah's parents that he had seen her at the movies on Tuesday, Sarah confessed to sneaking out that day. On Monday, however, Takashi had violated the local curfew for minors. Hence Sarah's parents cannot justifiably punish her in this case.\", \"Shakila's secretary has admitted that he is illegally receiving cable television without paying for it. Shakila would not be justified in reporting him, though, since she once did the same thing.\"]", "label": 1 }, { "id": "train_4216", "context": "Journalist: A free marketplace of ideas ensures that all ideas get a fair hearing. Even ideas tainted with prejudice and malice can prompt beneficial outcomes. In most countries, however, the government is responsible for over half the information released to the public through all media. For this reason, the power of governments over information needs to be curtailed. Everyone grants that governments should not suppress free expression, yet governments continue to construct near monopolies on the publication and dissemination of enormous amounts of information.", "question": "Which one of the following most accurately expresses the conclusion of the journalist's argument?", "answers": "['Governments have near monopolies on the dissemination of many kinds of information.', 'Preserving a free marketplace of ideas is important.', 'The control that governments have over information needs to be reduced.', 'The freedom of the marketplace of ideas is in jeopardy.']", "label": 2 }, { "id": "train_4217", "context": "In fifth-century B. C. Athenian courts, prosecutors scolded juries far more often for lenience than for harshness. We may conclude that Athenians considered themselves overly inclined to allow people to escape the punishment they deserved in the name of misguided mercy.", "question": "The reasoning in the argument above is flawed because it fails to consider the possibility that", "answers": "['the opinions of the Athenian prosecutors did not represent popular opinion', 'the Athenian prosecutors considered themselves too harsh', 'although Athenians considered themselves too lenient, they might not actually have been too lenient', 'the mercy the Athenians showed was not always misguided']", "label": 0 }, { "id": "train_4218", "context": "The manager of a nuclear power plant defended the claim that the plant was safe by revealing its rate of injury for current workers: only 3. 2 injuries per 200, 000 hours of work, a rate less than half the national average for all industrial plants. The manager claimed that, ttherefore, by the standard of how many injuries occur, the plant was safer than most other plants where the employees could work.", "question": "Which one of the following, if true, most calls into question the manager's claim?", "answers": "['Workers at nuclear power plants have filed only a few lawsuits against the management concerning unsafe working conditions.', 'Medical problems arising from work at a nuclear power plant are unusual in that they are not likely to appear until after an employee has left employment at the plant.', 'The exposure of the workers to radiation at nuclear power plants was within levels the government considers safe.', 'Workers at nuclear power plants are required to report to the manager any cases of accidental exposure to radiation.']", "label": 1 }, { "id": "train_4219", "context": "One can never tell whether another person is acting from an ulterior motive; ttherefore, it is impossible to tell whether someone' s action is moral, and so one should evaluate the consequences of an action rather than its morality.", "question": "Which one of the following principles, if valid, most helps to justify the reasoning above?", "answers": "['The assigning of praise and blame is what is most important in the assessment of the value of human actions.', 'There can be good actions that are not performed by a good person.', 'The intention of an action is indispensable for an evaluation of its morality.', \"One can sometimes know one's own motives for a particular action.\"]", "label": 2 }, { "id": "train_4220", "context": "In casual conversation, people experience little psychological discomfort in admitting that they have some particular character flaw, but only if they consider trivial the flaw to which they admit. Ttherefore, if in a casual conversation an individual readily admits that he or she has some particular character flaw, the individual must not consider that flaw to be serious.", "question": "Which one of the following is an assumption necessary to the argument?", "answers": "['People admit to having only those character flaws that most other people consider trivial.', 'In casual conversation, people readily admit to having a character flaw only when that admission causes them little psychological discomfort.', 'In casual conversation, people admit to having character flaws only when they must.', 'Most character flaws are considered trivial by those who have them.']", "label": 1 }, { "id": "train_4221", "context": "Ethicist: An action is wrong if it violates a rule of the society in which the action is performed and that rule promotes the general welfare of people in the society. An action is right if it is required by a rule of the society in which the action is performed and the rule promotes the general welfare of the people in that society.", "question": "Which one of the following judgments most closely conforms to the principle cited by the ethicist?", "answers": [ "Amelia's society has a rule against lying. However, she lies anyway in order to protect an innocent person from being harmed. While the rule against lying promotes the general welfare of people in the society, Amelia's lie is not wrong because she is preventing harm.", "Jordan lives in a society that requires its members to eat certain ceremonial foods during festivals. Jordan disobeys this rule. Because the rule is not detrimental to the general welfare of people in her society, Jordan's disobedience is wrong.", "Edward's society requires children to take care of their aged parents. Edward's taking care of his aged parents is the right thing for him to do because the rule requiring this action promotes the general welfare of people in the society.", "Dahlia always has a cup of coffee before getting dressed in the morning. Dahlia's action is right because it does not violate any rule of the society in which she lives." ], "label": 2 }, { "id": "train_4222", "context": "In an experiment, tennis players who were told that their performance would be used to assess only the quality of their rackets performed much better than an equally skilled group of tennis players who were told that their tennis-playing talent would be measured.", "question": "The situation described above most closely conforms to which one of the following propositions?", "answers": "['People do less well on a task if they have been told that they will be closely watched while doing it.', 'People execute a task more proficiently when they do not believe their abilities are being judged.', 'People who assess their talents accurately generally perform near their actual level of proficiency.', 'People who think that a superior performance will please those who are testing them generally try harder.']", "label": 1 }, { "id": "train_4223", "context": "Azadeh: The recent increase in the amount of organically produced food indicates that consumers are taking a greater interest in the environment. Thus, there is new hope for a healthier planet. Ben: No, Azadeh, if you interviewed people who buy organic produce, you' d see that they' re actually as selfish as everyone else, since they' re motivated only by worries about their own health.", "question": "Azadeh's and Ben's statements provide the most support for holding that they disagree about whether", "answers": "['people can be persuaded to have a greater concern for the environment than they now have', 'the rise in organic food production shows people to have a greater concern for the environment than they had before', 'people ought to be more concerned about the environment than they currently are', 'people can become healthier by increasing their consumption of organic foods']", "label": 1 }, { "id": "train_4224", "context": "Some claim that migratory birds have an innate homing sense that allows them to return to the same areas year after year. However, there is little evidence to support this belief, since the studies testing whether the accuracy of birds' migratory patterns is due to such an innate ability are inconclusive. After all, birds may simply navigate using landmarks, just as humans do, and we do not say that humans have an innate sense of direction simply because they find their way home time after time.", "question": "Which one of the following statements most accurately expresses the main conclusion drawn in the argument?", "answers": "[\"The ability to use landmarks to find one's way home is probably not an innate ability in birds.\", \"Studies testing whether the accuracy of birds' migratory patterns is due to an innate homing sense are inconclusive.\", 'There is as yet little reason to accept that birds have an innate homing sense.', 'It is as false to claim that humans have an innate sense of direction as it is to claim that birds have an innate homing sense.']", "label": 2 }, { "id": "train_4225", "context": "An ancient Pavonian text describes how an army of one million enemies of Pavonia stopped to drink at a certain lake and drank the lake dry. Recently, archaeologists discovered that water-based life was suddenly absent just after the event was alleged by the text to have occurred. On the basis of reading the text and an account of the archaeological evidence, some students concluded that the events described really took place.", "question": "Which one of the following is a questionable technique used by the students to reach their conclusion?", "answers": "['rejecting a hypothesis because it is seemingly self-contradictory', 'considering people and locations whose existence cannot be substantiated by modern historians', 'ignoring available, potentially useful counterevidence', 'taking evidence that a text has correctly described an effect to show that the text has correctly described the cause']", "label": 3 }, { "id": "train_4226", "context": "It is probably within the reach of human technology to make the climate of Mars inhabitable. It might be several centuries before people could live there, even with breathing apparatuses, but some of the world' s great temples and cathedrals took centuries to build. Research efforts now are justified if there is even a chance of making another planet inhabitable. Besides, the intellectual exercise of understanding how the Martian atmosphere might be changed could help in understanding atmospheric changes inadvertently triggered by human activity on Earth.", "question": "The main point of the argument is that", "answers": "['research efforts aimed at discovering how to change the climate of Mars are justified', 'it is probably technologically possible for humankind to alter the climate of Mars', \"efforts to change the climate of Mars could facilitate understanding of the Earth's climate\", 'it would take several centuries to make Mars even marginally inhabitable']", "label": 0 }, { "id": "train_4227", "context": "Recently, many traffic lights and street markings were temporarily removed from a heavily traveled street in a major metropolitan area. Given that this street experiences significant volumes of automobile traffic, the number of accidents on the street was expected to increase. However, even though the street experienced no reduction in traffic, the number of accidents was greatly reduced.", "question": "Which one of the following, if true, most helps to resolve the apparent conflict described above?", "answers": "['Traffic lights and street markings are intended to have benefits in addition to those related to safety.', 'Most drivers were not aware that traffic lights and street markings had been removed.', 'Drivers were given advance notice that the traffic lights and street markings would be removed.', 'The lack of traffic lights and street markings caused drivers to drive more cautiously.']", "label": 3 }, { "id": "train_4228", "context": "Psychologist: Because of a perceived social stigma against psychotherapy, and because of age discrimination on the part of some professionals, some elderly people feel discouraged about trying psychotherapy. They should not be, however, for many younger people have greatly benefited from it, and people in later life have certain advantages over the young -- such as breadth of knowledge, emotional maturity, and interpersonal skills -- that contribute to the likelihood of a positive outcome.", "question": "Which one of the following most accurately expresses the main conclusion of the psychologist's argument?", "answers": "[\"Characteristics associated with maturity are important factors in psychotherapy's success.\", 'Elderly people should not be reluctant to undergo psychotherapy.', 'Elderly people are better able to benefit from psychotherapy than are younger people.', 'Certain psychotherapists practice age discrimination.']", "label": 1 }, { "id": "train_4229", "context": "Because our club recruited the best volleyball players in the city, we will have the best team in the city. Moreover, since the best team in the city will be the team most likely to win the city championship, our club will almost certainly be city champions this year.", "question": "The reasoning in the argument is flawed because the argument", "answers": "['presumes, without presenting relevant evidence, that an entity can be distinguished as the best only on the basis of competition', 'presumes, without providing warrant, that if an entity is the best among its competitors, then each individual part of that entity must also be the best', 'concludes that because an event is the most likely of a set of possible events, that event is more likely to occur than not', 'predicts the success of an entity on the basis of features that are not relevant to the quality of that entity']", "label": 2 }, { "id": "train_4230", "context": "Psychologist: Birth-order effects, the alleged effects of when one was born relative to the births of siblings, have not been detected in studies of adult personality that use standard personality tests. However, they have been detected in birth-order studies that are based on parents' and siblings' reports of the subjects' personalities. All of these birth-order studies, taken together, show that birth order has no lasting effect on personality; instead, birth order affects merely how a sibling' s behavior is perceived.", "question": "Which one of the following is an assumption required by the psychologist's argument?", "answers": "[\"Parents' and siblings' perceptions of a person's personality tend not to change between that person's early childhood and adulthood.\", 'Parents and siblings have accurate perceptions of the behavior patterns of other family members.', \"Standard personality tests have detected significant birth-order effects in some studies of young children's personalities.\", 'Standard personality tests will detect at least some birth-order effects on personality, if those effects exist.']", "label": 3 }, { "id": "train_4231", "context": "A scientific theory is a good theory if it satisfies two requirements: It must accurately describe a large class of observations in terms of a model that is simple enough to contain only a few elements, and it must make definite predictions about the results of future observations. For example, Aristotle' s cosmological theory, which claimed that everything was made out of four elements -- earth, air, fire, and water -- satisfied the first requirement, but it did not make any definite predictions. Thus, Aristotle' s cosmological theory was not a good theory.", "question": "If all the statements in the passage are true, each of the following must also be true EXCEPT:", "answers": "['Prediction about the results of future observations must be made by any good scientific theory.', \"Observation of physical phenomena was not a major concern in Aristotle's cosmological theory.\", \"Aristotle's cosmological theory described a large class of observations in terms of only four elements.\", 'A scientific model that contains many elements is not a good theory.']", "label": 1 }, { "id": "train_4232", "context": "Carrots are known to be one of the best sources of naturally occurring vitamin A. However, although farmers in Canada and the United States report increasing demand for carrots over the last decade, the number of people diagnosed with vitamin A deficiency in these countries has also increased in that time.", "question": "Each of the following, if true of Canada and the United States over the last decade, helps to resolve the apparent discrepancy described above EXCEPT:", "answers": "['Certain cuisines that have become popular use many more vegetable ingredients, including carrots, than most cuisines that were previously popular.', 'Weather conditions have caused a decrease in the availability of carrots.', 'The purchase of peeled and chopped carrots has become very popular, though carrots are known to lose their vitamins quickly once peeled.', 'Carrot consumption has increased only among those demographic groups that have historically had low vitamin A deficiency rates.']", "label": 0 }, { "id": "train_4233", "context": "According to futuristic writings in the 1960s, robots would soon drastically reduce crime. With night vision and ability to detect the chemicals involved in ballistics, such robots could be programed to paralyze anyone roaming the street at night with a gun: virtually all criminals fit that description. These criminals would be incapacitated and thus unable to resist an easy arrest.", "question": "Which of the following, if true, most strongly indicates that the logic of the prediction is flawed?", "answers": "[\"It's not obvious that reducing the number of criminals will always be beneficial.\", 'Because these robots could pose a hazard to cars at night, special barriers would have to be constructed between the paths of the robots and the lanes of traffic.', 'Since police officers carry guns, the robots would incapacitate them just as efficiently as they incapacitate criminals.', 'Such robots would need to be charged during the daytime.']", "label": 2 }, { "id": "train_4234", "context": "All unemployed artists are sympathetic to social justice. And no employed artists are interested in the prospect of great personal fame.", "question": "If the claims made above are true, then which one of the following must be true?", "answers": "['If an artist is sympathetic to social justice, that artist is unemployed.', 'If there are artists interested in the prospect of great personal fame, they are sympathetic to social justice.', 'All artists are either sympathetic to social justice or are interested in the prospect of great personal fame.', 'All artists uninterested in the prospect of great personal fame are sympathetic to social justice.']", "label": 1 }, { "id": "train_4235", "context": "Columnist: In a recent article an economist argues that corporations have no responsibility to society beyond obeying the law and maximizing profit for shareholders. But in a different article the same economist endorses the view that corporations ought to make financial contributions to social programs in the communities in which they are located. Thus the economist is caught in a contradiction.", "question": "The columnist's argument is questionable because it fails to rule out the possibility that", "answers": "['corporations make nonfinancial contributions to social programs within their local communities', 'many shareholders of corporations are in favor of their corporations making contributions to community social programs', \"financial contributions to community social programs improve a corporation's image in a way that improves its profitability\", \"a corporation's making financial contributions to community social programs violates no laws\"]", "label": 2 }, { "id": "train_4236", "context": "Young people believe efforts to reduce pollution, poverty, and war are doomed to failure. This pessimism is probably harmful to humanity' s future, because people lose motivation to work for goals they think are unrealizable. We must do what we can to prevent this loss of motivation and ttherefore must enable our children to believe that better futures are possible.", "question": "Which one of the following is an assumption on which the argument depends?", "answers": "['Enabling people to believe that better futures are possible will help prevent the loss of motivation that results from pessimistic beliefs about the future.', 'If future generations believe that the future can be better, then pollution, poverty, and war will be eliminated.', \"Motivating people to work to solve humanity's problems will enable them to believe that the future can be better and will cause them to be less pessimistic.\", \"The current prevalence of such problems as pollution and poverty stems from previous generations' inability to believe that futures can be better.\"]", "label": 0 }, { "id": "train_4237", "context": "It is undeniable that regular consumption of fast food can lead to serious health problems. However, most people agree that fast food companies should not be held liable for the effects their product has on the health of consumers. Few people, however, disagree with holding tobacco companies liable for the serious health problems caused by cigarette usage. Ttherefore, fast food must not be as unhealthy as cigarettes.", "question": "Which one of the following is assumed in the passage?", "answers": "['That fewer people use cigarettes than eat fast food', 'That health concerns are the only reason people believe tobacco companies should be held liable', 'That the long-term health impact is the same for cigarettes as fast food', 'That tobacco companies tend to use the same type of marketing for their products as fast food companies']", "label": 1 }, { "id": "train_4238", "context": "Political scientist: Efforts to create a more egalitarian society are often wrongly criticized on the grounds that total equality would necessarily force everyone into a common mold. Equality is presumed by such critics to require unacceptably bland uniformity. But this is not so. By promoting complementary human interests, a society can achieve a greater and more prosperous equality while enhancing rather than minimizing diversity.", "question": "The political scientist's argument proceeds by", "answers": "['undermining a view by showing that its general acceptance would lead to undesirable consequences', 'claiming that whatever is true of a group must be true of each of the members of the group', 'rebutting an objection by attacking the assumption on which it is said to be based', 'attacking a view by claiming that those who propose it are motivated only by self-interest']", "label": 2 }, { "id": "train_4239", "context": "Justine: Pellman, Inc. settled the lawsuit out of court by paying $1 million. That Pellman settled instead of going to trial indicates their corporate leaders expected to lose in court. Simon: It' s unclear whether Pellman' s leaders expected to lose in court. But I think they expected that, whether they won or lost the case, the legal fees involved in going to trial would have been more costly than the settlement. So settling the lawsuit seemed the most cost-effective solution.", "question": "The dialogue provides the most support for the claim that Justine and Simon disagree with each other about which one of the following?", "answers": [ "If Pellman's corporate leaders had expected that the legal fees for going to trial would have been less costly than the settlement, they would have taken the lawsuit to trial.", "If Pellman's corporate leaders had expected to win in court, then they would not have settled the lawsuit out of court for $1 million.", "If Pellman's legal fees for going to trial would have been more costly than the settlement, then settling the lawsuit was the most cost-effective solution for the corporation.", "Pellman's corporate leaders were able to accurately estimate their chances of winning in court." ], "label": 1 }, { "id": "train_4240", "context": "Davison River farmers are currently deciding between planting winter wheat this fall or spring wheat next spring. Winter wheat and spring wheat are usually about equally profitable. Because of new government restrictions on the use of Davison River water for irrigation, per acre yields for winter wheat, though not for spring wheat, would be much lower than average. Ttherefore, planting spring wheat will be more profitable than planting winter wheat, since__.", "question": "Which of the following most logically completes the argument below?", "answers": "['new crops of spring wheat must be planted earlier than the time at which standing crops of winter wheat are ready to be harvested', 'spring wheat has uses that are different from those of winter wheat', 'the smaller-than-average size of a winter wheat harvest this year would not be compensated for by higher winter wheat prices', 'planting spring wheat is more profitable than planting certain other crops, such as rye']", "label": 2 }, { "id": "train_4241", "context": "Since the zoo has more animals than enclosures, and every animal lives in an enclosure, it must be true that at least one of the enclosures contains more than one animal.", "question": "The argument above exhibits a pattern of reasoning that is most closely paralleled by which one of the following?", "answers": "['Since every year there are more marriages than divorces, there must be some marriages that will not end in divorce.', 'At least one of the families in Herndon has more than one child, since in Herndon, there are fewer families than children and every child is a member of a family.', 'Since boys under ten slightly outnumber girls under ten and since some families have more than one child under ten, it follows that at least one girl under ten has more than one brother under ten.', 'There must be fewer families that include teenagers than there are teenagers belonging to such families, since there is at least one family that includes more than one teenager.']", "label": 1 }, { "id": "train_4242", "context": "A careful review of hospital fatalities due to anesthesia during the last 20 years indicates that the most significant safety improvements resulted from better training of anesthetists. Equipment that monitors a patient' s oxygen and carbon dioxide levels was not available in most operating rooms during the period under review. Ttherefore, the increased use of such monitoring equipment in operating rooms will not significantly cut fatalities due to anesthesia.", "question": "A flaw in the argument is that", "answers": "['the evidence cited to show that one factor led to a certain result is not sufficient to show that a second factor will not also lead to that result', 'the reason indicated for the claim that one event caused a second more strongly supports the claim that both events were independent effects of a third event', 'the evidence cited to show that a certain factor was absent when a certain result occurred does not show that the absence of that factor caused that result', 'the reasons given in support of the conclusion presuppose the truth of that conclusion']", "label": 0 }, { "id": "train_4243", "context": "Advances in photocopying technology allow criminals with no printing expertise to counterfeit paper currency. One standard anticounterfeiting technique, microprinting, prints paper currency with tiny designs that cannot be photocopied distinctly. Although counterfeits of microprinted currency can be detected easily by experts, such counterfeits often circulate widely before being detected. An alternative, though more costly, printing technique would print currency with a special ink. Currency printed with the ink would change color depending on how ordinary light strikes it, whereas photocopied counterfeits of such currency would not. Because this technique would allow anyone to detect photocopied counterfeit currency easily, it should be adopted instead of microprinting, despite the expense.", "question": "Which one of the following, if true, provides the most support for the recommendation made by the argument?", "answers": "['Many criminals do not have access to the advanced photocopiers that are needed to produce counterfeits of microprinted paper currency that cashiers will accept as real.', 'Before photocopying technology existed, most counterfeits of paper currency were accomplished by master engravers.', 'When an anticounterfeiting technique depends on the detection of counterfeits by experts, the cost of inspection by experts adds significantly to the cost to society of that technique.', 'The process of microprinting paper currency involves fewer steps than does the printing of paper currency with the special ink.']", "label": 2 }, { "id": "train_4244", "context": "Sociologist: The claim that there is a large number of violent crimes in our society is false, for this claim is based upon the large number of stories in newspapers about violent crimes. But since violent crimes are very rare occurrences, newspapers are likely to print stories about them.", "question": "The sociologist's argument is flawed because it", "answers": "['mistakes a property of each member of a group taken as an individual for a property of the group taken as a whole', 'assumes without warrant that the newspaper stories in question are not biased', 'presupposes that most newspaper stories are about violent crime', 'presupposes the truth of the conclusion it is attempting to establish']", "label": 3 }, { "id": "train_4245", "context": "Paleontologists recently excavated two corresponding sets of dinosaur tracks, one left by a large grazing dinosaur and the other by a smaller predatory dinosaur. The two sets of tracks make abrupt turns repeatedly in tandem, suggesting that the predator was following the grazing dinosaur and had matched its stride. Modern predatory mammals, such as lions, usually match the stride of prey they are chasing immediately before they strike those prey. This suggests that the predatory dinosaur was chasing the grazing dinosaur and attacked immediately afterwards.", "question": "Which one of the following most accurately describes the role played in the argument by the statement that the predatory dinosaur was following the grazing dinosaur and had matched its stride?", "answers": "[\"It is presented to counteract a possible objection to the argument's overall conclusion.\", \"It provides the basis for an analogy used in support of the argument's overall conclusion.\", 'It is the overall conclusion of the argument.', \"It helps establish the scientific importance of the argument's overall conclusion, but is not offered as evidence for that conclusion.\"]", "label": 1 }, { "id": "train_4246", "context": "Every adult male tiger is larger than even the largest female tiger. In groups of tigers, any adult male tiger will dominate any female tiger.", "question": "If the statements above are true, which one of the following must, on the basis of them, be true?", "answers": "['Size is the most important factor in deter- mining dominance among tigers.', 'Some large female tigers dominate young male tigers.', 'Some lions are more ferocious than tigers.', 'If a female tiger dominates a male of the species, the male is not an adult.']", "label": 3 }, { "id": "train_4247", "context": "Anderson: Taking the long view, history shows that word usage and grammar rules are constantly changing and evolving-sometimes resulting in entirely new languages. Since they will change regardless of our efforts, we shouldn' t worry about violations of grammar rules. Lipton: That' s like arguing that we shouldn' t worry about enforcing laws since, in the larger scheme of things, laws change and nations come and go. But of course it is good that laws are enforced.", "question": "The dialogue provides the most support for the claim that Anderson and Lipton disagree over whether", "answers": "['users of a language can easily adapt to changes in that language', 'grammar violations should be resisted', 'languages evolve through an accumulation of changes in usage and rules', 'people only rarely violate grammar rules']", "label": 1 }, { "id": "train_4248", "context": "The government has recently adopted a policy of publishing airline statistics, including statistics about each airline' s number of near collisions and its fines for safety violations. However, such disclosure actually undermines the government' s goal of making the public more informed about airline safety, because airlines will be much less likely to give complete reports if such information will be made available to the public.", "question": "The reasoning in the argument is most vulnerable to criticism on the grounds that it", "answers": "['presumes, without providing justification, that the public has a right to all information about matters of public safety', 'presumes, without providing justification, that information about airline safety is impossible to find in the absence of government disclosures', 'fails to consider whether the publication of airline safety statistics will have an effect on the revenues of airlines', 'fails to consider that, even if the reports are incomplete, they may nevertheless provide the public with important information about airline safety']", "label": 3 }, { "id": "train_4249", "context": "Fossilized bones from the hominids Australopithecus robustus and Homo erectus were chemically analyzed. The robustus bones contained a lower ratio of strontium to calcium than did the erectus bones. The lower the ratio of strontium to calcium in fossilized hominid bones, the more meat the hominid had in its diet. H. erectus is known to have eaten meat.", "question": "The statements above, if true, most strongly support which one of the following?", "answers": "['The diet of H. erectus was richer in calcium than was the diet of A. robustus.', 'The diet of A. robustus included at least some meat.', 'The diets of A. robustus and H. erectus both contained less strontium than calcium.', 'The meat in the diet of H. erectus was higher in strontium than was the meat in the diets of other hominids.']", "label": 1 }, { "id": "train_4250", "context": "Bethany: Psychologists have discovered a technique for replacing one' s nightmares with pleasant dreams, and have successfully taught it to adults suffering from chronic nightmares. Studies have found that nightmare-prone children are especially likely to suffer from nightmares as adults. Thus, psychologists should direct efforts toward identifying nightmare-prone children so that these children can be taught the technique for replacing their nightmares with pleasant dreams.", "question": "Which one of the following principles, if valid, most helps to justify drawing the conclusion in Bethany's argument?", "answers": "['Any psychological technique that can be successfully taught to a child can also be successfully taught to an adult.', 'Psychologists should do everything they can to minimize the number of adults troubled by chronic nightmares.', 'Psychologists should not teach the technique for replacing nightmares with pleasant dreams to children who are unlikely to suffer from nightmares as adults.', 'Identifying nightmare-prone children is generally more difficult than teaching adults the technique for replacing nightmares with pleasant dreams.']", "label": 1 }, { "id": "train_4251", "context": "Only 1, 000 to 2, 000 species of fruit flies exist worldwide. Nowhere in the world are fruit flies more taxonomically diverse than in the Hawaiian islands, which host some 500 species. A subset of fruit flies called the picture-winged drosophilids is represented in Hawaii by 106 species. All of the fruit fly species now present in the Hawaiian archipelago are thought to be the descendants of the same one or two ancestral females.", "question": "Which one of the following can be inferred from the passage?", "answers": "['All of the 1, 000 to 2, 000 species of fruit flies worldwide are believed to be the descendants of one or two females.', 'All of the picture-winged drosophilids in Hawaii are believed to be the descendants of the same one or two ancestral female fruit flies.', 'Some fruit flies originated in Hawaii and spread from there to other parts of the world.', 'Picture-winged drosophilids are found only in the Hawaiian islands.']", "label": 1 }, { "id": "train_4252", "context": "A manager is hoping to reach a certain target for camera sales in his store, which sells between 10 and 20 cameras a week. Typically, most cameras sold in any week are the less expensive economy models, and his store has sold relatively fewer of the more expensive, high-end cameras. The manager realizes that if, on average, three more cameras sold each week were high-end instead of economy models, the store would reach its target in sales. The manager prepares a detailed information sheet for the sales associates, outlining the numerous advantages of the high-end cameras over the economy cameras, and provides each sales associate with a portfolio of contrasting photos of the same images, showing the clearly superior image quality of the high-end cameras.", "question": "Which of the following, if true, would provide most support for the prediction that the detailed information sheet and photo portfolio given to sales associates will have its intended effect of allowing the store to reach its target in sales?", "answers": "['Camera stores that are part of the same national franchise in major metropolitan locations, like New York or Los Angeles, sell comparatively large numbers of the high end cameras.', 'The sales associates are already well informed about the capabilities of all the cameras, and often know detailed technical information about their circuitry.', 'The high end cameras can generate photographs of profession quality, such as those a portrait photographer might produce', 'The only reason many customers buy the economy cameras is that they do not appreciate that the high-end cameras are significantly better.']", "label": 3 }, { "id": "train_4253", "context": "Theories in certain scientific fields may be in flux today, but this unsettled state must be attributed to scientific progress, not to a lack of theoretical rigor. Several decades of scientific research have recently culminated in a wealth of new discoveries in these fields, and whenever many new facts come to light in a field, the ways in which that field organizes its knowledge inevitably need adjustment.", "question": "The argument proceeds by", "answers": "['showing that two alternate explanations for a situation are equally probable', 'citing a law of nature to explain a particular kind of change', 'explaining why a situation came about by referring to the intended outcome of a course of action', 'presenting the situation to be explained as part of a general pattern']", "label": 3 }, { "id": "train_4254", "context": "Anju: The Adkjos corporation does not fund social programs. Ttherefore, although Adkjos does make fine products, it is not socially responsible. Sanjeev: That doesn' t mean that Adkjos is not socially responsible. If a business offers good pay and benefits to its employees, and fine service and value to customers, it is socially responsible. Adkjos does those things.", "question": "On the basis of their statements, Anju and Sanjeev are committed to disagreeing about which one of the following?", "answers": "['Socially responsible companies offer better pay than companies that are not socially responsible.', 'Funding social programs is required for a business to be socially responsible.', 'To be socially responsible, it is not enough for a company to make fine products.', 'Adkjos treats its employees and customers well.']", "label": 1 }, { "id": "train_4255", "context": "To acquire a better understanding of the structure and development of the human personality, some psychologists study the personalities of animals.", "question": "Each of the following, if true, contributes to an explanation of the practice mentioned above EXCEPT:", "answers": "['Field observations of the behavior of young animals often inspire insightful hypotheses about human personality development.', 'It is generally less expensive to perform experiments on animals than it is to perform them on humans.', 'Proper understanding of human personality is thought to provide a model for better understanding the personality of animals.', 'The actions of humans and animals are believed to be motivated by similar instincts, but these instincts are easier to discern in animals.']", "label": 2 }, { "id": "train_4256", "context": "Ethicist: Some would ban cloning on the grounds that clones would be subpeople, existing to indulge the vanity of their \"originals. \" It is not illegal, however, to use one person as a vehicle for the ambitions of another. Some people push their children to achieve in academics or athletics. You do not have to have been born in a test tube to be an extension of someone else' s ego.", "question": "The assertion that it is not illegal to use one person as a vehicle for another's ambitions is used in the ethicist's argument in which one of the following ways?", "answers": "['It supports the conclusion that forcing children to pursue academic success is not objectionable.', \"It supports the ethicist's view that vanity's being the motivation for cloning is not enough of a reason to ban cloning.\", 'It describes a legal position that the ethicist argues should be changed.', \"It is implied by the ethicist's conviction that clones are not subpeople.\"]", "label": 1 }, { "id": "train_4257", "context": "The town council of North Tarrytown favored changing the name of the town to Sleepy Hollow. Council members argued that making the town' s association with Washington Irving and his famous \"legend\" more obvious would increase tourism and result immediately in financial benefits for the town' s inhabitants.", "question": "The council members' argument requires the assumption that", "answers": "['the immediate per capita cost to inhabitants of changing the name of the town would be less than the immediate per capita revenue they would receive from the change', 'other towns in the region have changed their names to reflect historical associations and have, as a result, experienced a rise in tourism', 'the town can accomplish, at a very low cost per capita, the improvements in tourist facilities that an increase in tourism would require', 'many inhabitants would be ready to supply tourists with information about Washington Irving and his \"legend\"']", "label": 0 }, { "id": "train_4258", "context": "For house painting, acrylic paints are an excellent choice. They provide everything that a good paint should provide: smooth and even coverage, quick drying time, durability, and easy cleanup. Even acrylics, however, cannot correct such surface defects as badly cracked paint. Such conditions indicate some underlying problem, such as water damage, that needs repair.", "question": "Which one of the following is most strongly supported by the statements above?", "answers": "['It is not a requirement of house paints that they correct surface defects such as badly cracked paint.', 'Acrylic paints come in as wide a range of colors as do any other paints.', 'Acrylics should not be used to paint over other types of house paint.', 'Badly cracked paint is not a result of harsh weather conditions.']", "label": 0 }, { "id": "train_4259", "context": "Scientists typically do their most creative work before the age of forty. It is commonly thought that this happens because aging by itself brings about a loss of creative capacity . However, a study has found that almost all scientists who produce highly creative work beyond the age of forty entered their fields late and less than a dozen years before their creative breakthroughs. Since creative breakthroughs by scientists under forty also generally occur within a dozen years of the scientist' s entry into the field , the study' s finding strongly suggests that the real reason why scientists over forty rarely produce highly creative work is not due to age but rather because most have spent too long in their fields.", "question": "In the argument given, the two portions in boldface play which of the following roles?", "answers": "['The first is an explanation that the argument challenges; the second provides evidence in support of a competing explanation that the argument defends.', 'The first and second are both claims that have been advanced in support of a position that the argument as a whole opposes.', 'The first is an explanation that the argument defends; the second is evidence that has been used to challenge that explanation.', 'The first is an explanation that the argument challenges; the second is evidence that has been used against an alternative explanation that the argument defends.']", "label": 0 }, { "id": "train_4260", "context": "Recent research indicates that increased consumption of fruits and vegetables by middle-aged people reduces their susceptibility to stroke in later years. The researchers speculate that this may be because fruits and vegetables are rich in folic acid. Low levels of folic acid are associated with high levels of homocysteine, an amino acid that contributes to blocked arteries.", "question": "Which one of the following statements is most strongly supported by the information above?", "answers": "['A decreased risk of stroke is correlated with increased levels of folic acid.', 'An increased propensity for blocked arteries is correlated with decreased levels of homocysteine.', 'Stroke is prevented by ingestion of folic acid in quantities sufficient to prevent a decline in the levels of homocysteine.', 'A decreased propensity for blocked arteries is correlated with low levels of folic acid.']", "label": 0 }, { "id": "train_4261", "context": "Carpal tunnel syndrome, a nerve disorder that affects the hands and wrists, is often caused by repetitive motions such as typing on a keyboard. A recent study of office workers found that, among those who do similar amounts of typing, workers reporting the least control over their own work had almost three times the risk of developing carpal tunnel syndrome as did those who reported the most control.", "question": "Which one of the following, if true, most helps to explain the study's findings?", "answers": "['Among office workers who rarely use keyboards, the rate of carpal tunnel syndrome is much higher for those who feel that they lack control over their own work.', 'Office workers who have the most control over their own work tend to perform repetitive motions other than typing more often than do office workers with the least control over their own work.', \"The keyboards on which office workers type tend to put typists' arms and hands in positions that promote the development of carpal tunnel syndrome.\", \"Feeling a lack of control over one's own work tends to put one under emotional stress that makes one more susceptible to nerve disorders.\"]", "label": 3 }, { "id": "train_4262", "context": "Studies in restaurants show that the tips left by customers who pay their bill in cash tend to be larger when the bill is presented on a tray that bears a credit card logo. Consumer psychologists hypothesize that simply seeing a credit-card logo makes many credit card holders willing to spend more because it reminds them that their spending power exceeds the cash they have immediately available.", "question": "Which of the following, if true, most strongly supports the psychologists' interpretation of the studies?", "answers": "[\"The percentage of restaurant bills paid with a given brand of credit card increases when that credit card's logo is displayed on the tray with which the bill is presented.\", 'In general, restaurant patrons who pay their bills in cash leave larger tips than do those who pay by credit card.', 'Patrons who are under financial pressure from their credit-card obligations tend to tip less when presented with a restaurant bill on a tray with a credit-card logo than when the tray has no logo.', 'The effect noted in the studies is not limited to patrons who have credit cards.']", "label": 2 }, { "id": "train_4263", "context": "In a sample containing 1, 000 peanuts from lot A and 1, 000 peanuts from lot B, 50 of the peanuts from lot A were found to be infected with Aspergillus. Two hundred of the peanuts from lot B were found to be infected with Aspergillus. Ttherefore, infection with Aspergillus is more widespread in lot B than in lot A.", "question": "The reasoning in which one of the following is most similar to the reasoning in the argument above?", "answers": "['In the past 1, 000 experiments, whenever an experimental fungicide was applied to coffee plants infected with coffee rust, the infection disappeared. The coffee rust never disappeared before the fungicide was applied. Ttherefore, in these experiments, application of the fungicide caused the disappearance of coffee rust.', 'Three thousand registered voters -- 1, 500 members of the Liberal party and 1, 500 members of the Conservative party -- were asked which mayoral candidate they favored. Four hundred of the Liberals and 300 of the Conservatives favored Pollack. Ttherefore, Pollack has more support among Liberals than among Conservatives.', 'Every one of these varied machine parts is of uniformly high quality. Ttherefore, the machine that we assemble from them will be of equally high quality.', 'All of my livestock are registered with the regional authority. None of the livestock registered with the regional authority are free-range livestock. Ttherefore, none of my livestock are free-range livestock.']", "label": 1 }, { "id": "train_4264", "context": "Two paleontologists, Dr. Tyson and Dr. Rees, disagree over the interpretation of certain footprints that were left among other footprints in hardened volcanic ash at site G. Dr. Tyson claims they are clearly early hominid footprints since they show human characteristics: a squarish heel and a big toe immediately adjacent to the next toe. However, since the footprints indicate that if hominids made those prints they would have had to walk in an unexpected cross-stepping manner, by placing the left foot to the right of the right foot, Dr. Rees rejects Dr. Tyson' s conclusion.", "question": "Which one of the following, if true, most seriously undermines Dr. Tyson's conclusion?", "answers": "['The footprints showing human characteristics were clearly those of at least two distinct individuals.', \"Footprints shaped like a human's that do not show a cross-stepping pattern exist at site M, which is a mile away from site G, and the two sets of footprints are contemporaneous.\", 'When the moist volcanic ash became sealed under additional layers of ash before hardening, some details of some of the footprints were erased.', 'Certain species of bears had feet very like human feet, except that the outside toe on each foot was the biggest toe and the innermost toe was the smallest toe.']", "label": 3 }, { "id": "train_4265", "context": "Psychologist: Research has shown that a weakened immune system increases vulnerability to cancer. So, cancer-patient support groups, though derided by those who believe that disease is a purely biochemical phenomenon, may indeed have genuine therapeutic value, as it is clear that participation in such groups reduces participants' stress levels.", "question": "Which one of the following is an assumption required by the psychologist's argument?", "answers": "['Disease is not a biochemical phenomenon at all.', 'Stress can weaken the immune system.', \"Discussing one's condition eliminates the stress of being in that condition.\", 'Cancer patients can learn to function well under extreme stress.']", "label": 1 }, { "id": "train_4266", "context": "Taken together, some 2, 000 stocks recommended on a popular television show over the course of the past 12 years by the show' s guests, most of whom are successful consultants for multibillion-dollar stock portfolios, performed less successfully than the market as a whole for this 12-year period. So clearly, no one should ever follow any recommendations by these so-called experts.", "question": "Each of the following, if true, weakens the argument EXCEPT:", "answers": "['The stock portfolios for which the guests were consultants performed better for the past 12 year period than the market as a whole.', 'Performance of the stocks recommended on the television show was measured independently by a number of analysts, and the results of all the measurements concurred.', 'Taken together, the stocks recommended on the television show performed better than the market as a whole for the past year.', 'Performance of the stocks recommended on the telephone show was measured by stock dividends, whereas the performance of the market as a whole was measured by change in share value.']", "label": 1 }, { "id": "train_4267", "context": "Sales manager: The highest priority should be given to the needs of the sales department, because without successful sales the company as a whole would fail. Shipping manager: There are several departments other than sales that also must function successfully for the company to succeed. It is impossible to give the highest priority to all of them.", "question": "The shipping manager criticizes the sales manager's argument by pointing out", "answers": "[\"that departments other than sales are more vital to the company's success\", \"that the sales department taken by itself is not critical to the company's success as a whole\", \"an absurd consequence of its apparent assumption that a department's necessity earns it the highest priority\", 'that the sales manager makes a generalization from an atypical case']", "label": 2 }, { "id": "train_4268", "context": "Forestry official: Many people think that if forest fires are not extinguished as quickly as possible, the Forestry Department is not doing its job properly. But relatively frequent, small fires clear out small trees and forest debris, which, if allowed to accumulate, would create the conditions for large, devastating fires.", "question": "Ttherefore, it's best to let small fires burn The statement that relatively frequent, small fires clear out small trees and forest debris plays which one of the following roles in the official' s argument?", "answers": "['It is a conclusion based on the premise in the argument that it is best to let small forest foes bum.', 'It is used as evidence against the contention that the Forestry Department is not doing its job properly if it does not extinguish forest fires as quickly as possible.', 'It is used to show what the consequences would be if the Forestry Department based its policies on the ideas most people have about how it should do its job.', 'It is offered as support for the contention that the Forestry Department is not doing its job properly if it does not extinguish forest fires as quickly as possible.']", "label": 1 }, { "id": "train_4269", "context": "Scientist: Given the human tendency to explore and colonize new areas, some people believe that the galaxy will eventually be colonized by trillions of humans. If so, the vast majority of humans ever to live would be alive during this period of colonization. Since all of us are humans and we have no reason to think we are unrepresentative, the odds are overwhelming that we would be alive during this period, too. But, because we are not alive during this period, the odds are slim that such colonization will ever happen.", "question": "The scientist's argument proceeds by", "answers": "['taking for granted that dependable predictions about the future cannot ever be made simply on the basis of the present facts', 'inferring that since an event that is taken to be likely on a given hypothesis has not occurred, the hypothesis is probably false', 'reasoning that because an event has not occurred, that event has a low probability of occurring', 'making a prediction far into the future based on established human tendencies']", "label": 1 }, { "id": "train_4270", "context": "Panelist: Medical research articles cited in popular newspapers or magazines are more likely than other medical research articles to be cited in subsequent medical research. Thus, it appears that medical researchers' judgments of the importance of prior research are strongly influenced by the publicity received by that research and do not strongly correspond to the research' s true importance.", "question": "The panelist's argument is most vulnerable to criticism on the grounds that it", "answers": "['presents counterarguments to a view that is not actually held by any medical researcher', 'fails to consider the possibility that popular newspapers and magazines do a good job of identifying the most important medical research articles', 'fails to consider the possibility that popular newspapers and magazines are able to review only a minuscule percentage of medical research articles', 'draws a conclusion that is logically equivalent to its premise']", "label": 1 }, { "id": "train_4271", "context": "The university' s purchasing department is highly efficient overall. We must conclude that each of its twelve staff members is highly efficient.", "question": "Which one of the following arguments exhibits flawed reasoning most similar to that exhibited by the argument above?", "answers": "['The outside audit of our public relations department has exposed serious deficiencies in the competence of each member of that department. We must conclude that the department is inadequate for our needs.', 'The employees at this fast-food restaurant are the youngest and most inexperienced of any fast-food workers in the city. Given this, it seems obvious that customers will have to wait longer for their food at this restaurant than at others.', 'This supercomputer is the most sophisticated -- and the most expensive -- ever built. It must be that each of its components is the most sophisticated and expensive available.', 'Literature critics have lavished praise on every chapter of this book. In light of their reviews, one must conclude that the book is excellent.']", "label": 2 }, { "id": "train_4272", "context": "There is relatively little room for growth in the overall carpet market, which is tied to the size of the population. Most who purchase carpet do so only once or twice, first in their twenties or thirties, and then perhaps again in their fifties or sixties. Thus as the population ages, companies producing carpet will be able to gain market share in the carpet market only through purchasing competitors, and not through more aggressive marketing.", "question": "Which one of the following, if true, casts the most doubt on the conclusion above?", "answers": "['Most of the major carpet producers market other floor coverings as well.', 'The carpet market is unlike most markets in that consumers are becoming increasingly resistant to new patterns and styles.', 'Price reductions, achieved by cost-cutting in production, by some of the dominant firms in the carpet market are causing other producers to leave the market altogether.', \"Two of the three mergers in the industry's last ten years led to a decline in profits and revenues for the newly merged companies.\"]", "label": 2 }, { "id": "train_4273", "context": "Corruption is the cost of doing business in the context of the modern governmental climate. In some countries, corruption takes the form of campaign donations to politicians for passing favorable legislation, while in other countries, politicians receive kickbacks for the successful completion of a government initiative. As long as humans are involved in government, corruption will never be eliminated.", "question": "Which one of the following most accurately describes how the argument proceeds?", "answers": "['It begins with a generalization, offers some examples, and concludes with a definition.', 'It begins with a definition, offers some examples, and concludes with speculation.', 'It begins with a generalization, defines some terms, and concludes with an example.', 'It begins with a bright-line test, offers some examples, and concludes with a definition.']", "label": 1 }, { "id": "train_4274", "context": "If temperatures had dropped below freezing when I was gone last week, the impatiens in my garden would have died. If the impatiens had died, they obviously could not continue to bloom. However, since the impatiens in my garden are still in bloom today, temperatures did not drop below freezing last week.", "question": "The pattern of reasoning in which one of the following arguments most closely parallels that in the argument above?", "answers": "['If a species is highly adaptable, it will thrive when introduced into a new environment. If a species thrives in its new environment, it will have an adverse effect on species already existing in that environment. But, since this species has not had an adverse effect on any species already existing in its new environment, it is not highly adaptable.', 'If the introduction of a new species would adversely affect some species already existing in an environment, that species should not be introduced into it. Ttherefore, since the introduction of species into new environments will result in some species in those environments being adversely affected, species should probably not be introduced into new environments.', 'If a species thrives in a new environment, that species is adaptable. Species that adapt to new environments adversely affect some species already existing in those environments. So, if a species does not adversely affect any species already existing in its new environment, it has not adapted to it.', 'If a species is introduced into a new environment, it adversely affects some species already existing in that environment, but only if it adapts well to it. Ttherefore, if a species does not adapt well to a new environment, it will not adversely affect any species already existing in it.']", "label": 0 }, { "id": "train_4275", "context": "Philosopher: A person is morally responsible for an action only if that action is performed freely. And an action is free only if there is an alternative action that is genuinely open to the person. But an alternative action is genuinely open only if performing that alternative action is not morally wrong.", "question": "If the philosopher 's statements are true, which one of the following must also be true?", "answers": "['People are not morally responsible for most of the actions that they perform .', 'A person is morally responsible for an action if there is an alternative action that is genuinely open to the person.', 'An alternative action is not genuinely open to a person unless that person would be morally responsible for performing the alternative action.', 'An action is not free unless there is an alternative action that is not morally wrong.']", "label": 3 }, { "id": "train_4276", "context": "Asthma, a chronic breathing disorder, is significantly more common today among adult competitive swimmers than it is among competitive athletes who specialize in other sports. Although chlorine is now known to be a lung irritant and swimming pool water is generally chlorinated, it would be rash to assume that frequent exposure to chlorine is the explanation of the high incidence of asthma among these swimmers, since __ .", "question": "Which of the following most logically completes the argument given?", "answers": "['young people who have asthma are no more likely to become competitive athletes than are young people who do not have asthma', 'many people have asthma without knowing they have it and thus are not diagnosed with the condition until they begin engaging in very strenuous activities, such as competitive athletics', 'competitive athletes who specialize in sports other than swimming are rarely exposed to chlorine', 'until a few years ago, physicians routinely recommended competitive swimming to children with asthma, in the belief that this form of exercise could alleviate asthma symptoms']", "label": 3 }, { "id": "train_4277", "context": "Researchers have found that people who drink five or more cups of coffee a day have a risk of heart disease 2. 5 times the average after corrections are made for age and smoking habits. Members of the research team say that, on the basis of their findings, they now limit their own daily coffee intake to two cups.", "question": "Which one of the following, if true, indicates that the researchers' precaution might NOT have the result of decreasing their risk of heart disease?", "answers": "['The study found that for people who drank three or more cups of coffee daily, the additional risk of heart disease increased with each extra daily cup.', 'The study did not collect information that would show whether variations in level of coffee consumption are directly related to variations in level of stress, a major causal factor in heart disease.', 'Subsequent studies have shown that heavy coffee consumption tends to cause an elevated blood-cholesterol level, an immediate indicator of increased risk of heart disease.', 'Per capita coffee consumption has been declining over the past 20 years because of the increasing popularity of soft drinks and also because of health worries.']", "label": 1 }, { "id": "train_4278", "context": "Mayor: Local antitobacco activists are calling for expanded antismoking education programs paid for by revenue from heavily increased taxes on cigarettes sold in the city. Although the effectiveness of such education programs is debatable, there is strong evidence that the taxes themselves would produce the sought-after reduction in smoking. Surveys show that cigarette sales drop substantially in cities that impose stiff tax increases on cigarettes.", "question": "Which one of the following, if true, most undermines the reasoning in the argument above?", "answers": "['Antismoking education programs that are funded by taxes on cigarettes will tend to lose their funding if they are successful.', 'People who are well informed about the effects of long-term tobacco use are significantly less likely to smoke than are people who are not informed.', 'Usually, cigarette sales will increase substantially in the areas surrounding a city after that city imposes stiff taxes on cigarettes.', 'A city-imposed tax on cigarettes will substantially reduce the amount of smoking in the city if the tax is burdensome to the average cigarette consumer.']", "label": 2 }, { "id": "train_4279", "context": "In addition to the labor and materials used to make wine, the reputation of the vineyard where the grapes originate plays a role in determining the price of the finished wine. Ttherefore, an expensive wine is not always a good wine.", "question": "Which one of the following is an assumption on which the argument depends?", "answers": "[\"The reputation of a vineyard generally plays a greater role than the quality of its grapes in determining its wines' prices.\", 'The reputation of a vineyard does not always indicate the quality of its wines.', \"Wines produced by lesser-known vineyards generally are priced to reflect accurately the wines' quality.\", 'Price is never an accurate indication of the quality of a bottle of wine.']", "label": 1 }, { "id": "train_4280", "context": "In a new police program, automobile owners in some neighborhoods whose cars are not normally driven between 1 A. M. and 5 A. M. can display a special decal in the cars' windows and authorize police to stop the cars during those hours to check the drivers' licenses. The theft rate for cars bearing such decals is much lower than had been usual for cars in those neighborhoods.", "question": "If it is concluded from the statements above that automobile theft has been reduced by the program, which one of the following would it be most important to answer in evaluating that conclusion?", "answers": "[\"Will owners who have placed decals on their cars'windows but who find it necessary to drive between 1 A. M. and 5 A. M. be harassed by police?\", 'Are the neighborhoods in which the program has been put into effect a representative cross section of neighborhoods with respect to the types of automobiles owned by residents?', 'In how many neighborhoods is the police program operating?', 'Are owners who are cautious enough to join the program taking other special measures to protect their cars against theft?']", "label": 3 }, { "id": "train_4281", "context": "Scientists hypothesize that a particular type of fat known as \"P-fat\" is required for the development of eyesight. Researchers were led to this hypothesis by observing that babies who are fed formulas low in P-fat tend to have worse eyesight than babies fed mother' s milk, which is high in P-fat. It has also been shown that babies that are five to six weeks premature tend to have worse eyesight than babies carried to term.", "question": "Which one of the following, if true, most supports the scientists' hypothesis?", "answers": "['The eyesight of a fetus develops during the last trimester of pregnancy.', \"Babies generally prefer mother's milk to formulas low in P-fat.\", 'Adults whose diets lack P-fat tend to have worse eyesight than those whose diets are high in P-fat.', 'A fetus typically receives high levels of P-fat from the mother during only the last four weeks of pregnancy.']", "label": 3 }, { "id": "train_4282", "context": "Lutsina: Because futuristic science fiction does not need to represent current social realities, its writers can envisage radically new social arrangements. Thus it has the potential to be a richer source of social criticism than is conventional fiction. Priscilla: That futuristic science fiction writers more skillfully envisage radically new technologies than new social arrangements shows how writers' imaginations are constrained by current realities. Because of this limitation, the most effective social criticism results from faithfully presenting the current social realities for critical examination, as happens in conventional fiction.", "question": "Lutsina and Priscilla disagree with each other about whether", "answers": "['envisaging radically new technologies rather than radically new social arrangements is a shortcoming of futuristic science fiction', 'writers of conventional fiction are more skillful than are writers of futuristic science fiction', 'some science fiction writers have succeeded in envisaging convincing, radically new social arrangements', 'futuristic science fiction has more promise as a source of social criticism than does conventional fiction']", "label": 3 }, { "id": "train_4283", "context": "The average normal infant born in the United States weighs between 12 and 14 pounds at the age of three months. Ttherefore, if a three-month-old child weighs only 10 pounds, its weight gain has been below the United States average.", "question": "Which of the following indicates a flaw in the reasoning above?", "answers": "['Some three-month-old children weigh as much as 17 pounds.', 'The phrase \"below average\" does not necessarily mean insufficient.', 'It is possible for a normal child to weigh 10 pounds at birth.', 'Average weight gain is not the same as average weight.']", "label": 3 }, { "id": "train_4284", "context": "Airport administrator: According to the latest figures, less than 1 commercial flight in 2 million strays off course while landing, a number low enough to allow runways to be built closer together without a significant increase in risk. Opponents of closer runways claim that the number is closer to 1 in 20, 000, but this figure is based on a partial review of air traffic control tapes and so is relatively unreliable compared to the other figure, which is based on a thorough study of the flight reports required of pilots for all commercial flights.", "question": "Which one of the following most accurately describes a flaw in the airport administrator's argument?", "answers": "['The argument questions the integrity of those who are opposed to allowing runways to be built closer together.', 'The argument presumes, without providing justification, that the air traffic control tapes studied do not provide accurate information concerning specific flights.', 'The argument overlooks the fact that those who make mistakes are often unreliable sources of information about those mistakes.', 'The argument presumes, without providing justification, that building runways closer together will encourage pilots to be more cautious while landing.']", "label": 2 }, { "id": "train_4285", "context": "Some judges complain about statutes that specify mandatory minimum sentences for criminal offenses. These legal restrictions, they complain, are too mechanical and prevent judges from deciding when a given individual can or cannot be rehabilitated. But that is precisely why mandatory minimum sentences are necessary. History amply demonstrates that when people are free to use their own judgment they invariably believe themselves to act wisely when in fact they are often arbitrary and irrational. There is no reason to think that judges are an exception to this rule.", "question": "Which one of the following sentences most accurately expresses the main point of the passage?", "answers": "['Mandatory minimum sentences are too mechanical and reduce judicial discretion.', 'Mandatory minimum sentences are needed to help prevent judicial arbitrariness.', 'People believe that they have good judgment but never do.', 'Judges should be free to exercise their own judgment.']", "label": 1 }, { "id": "train_4286", "context": "Gilbert: This food label is mistaken. It says that these cookies contain only natural ingredients, but they contain alphahydroxy acids that are chemically synthesized by the cookie company at their plant. Sabina: The label is not mistaken. After all, alphahydroxy acids also are found occurring naturally in sugarcane.", "question": "Which one of the following, if true, would most strengthen Sabina's argument?", "answers": "['All substances except those that do not occur naturally in any source are considered natural.', 'Many other foods advertising all natural ingredients also contain some ingredients that are chemically synthesized.', 'The label was printed before the cookie company decided to switch from sugarcane alphahydroxy acids to synthesized ones.', 'Not all chemicals that are part of the manufacturing process are ingredients of the cookies.']", "label": 0 }, { "id": "train_4287", "context": "All actions are motivated by self-interest, since any action that is apparently altruistic can be described in terms of self-interest. For example, helping someone can be described in terms of self-interest: the motivation is hope for a reward or other personal benefit to be bestowed as a result of the helping action.", "question": "Which one of the following most accurately describes an error in the argument's reasoning?", "answers": "['The argument takes evidence showing merely that its conclusion could be true to constitute evidence showing that the conclusion is in fact true.', 'The argument depends for its appeal only on the emotional content of the example cited.', 'The argument does not explain what is meant by \"reward\" and \"personal benefit. \"', 'The term \"self-interest\" is allowed to shift in meaning over the course of the argument.']", "label": 0 }, { "id": "train_4288", "context": "Rhonda will see the movie tomorrow afternoon only if Paul goes to the concert in the afternoon. Paul will not go to the concert unless Ted agrees to go to the concert. However, Ted refuses to go to the concert. So Rhonda will not see the movie tomorrow afternoon.", "question": "The pattern of reasoning displayed above is most closely paralleled in which one of the following?", "answers": "['Gary will do his laundry tomorrow only if Peter has to go to work. Unless Cathy is ill, Peter will not have to go to work. Since Cathy is not ill, Gary will not do his laundry tomorrow.', 'Lisa will attend the family reunion next week only if one of her brothers, Jared or Karl, also attends. Karl will not attend the reunion, but Jared will. So Lisa will attend the reunion.', 'If Janice comes to visit, Mary will not pay the bills tomorrow. Janice will not come to visit unless she locates a babysitter. However, Janice has located a babysitter, so she will visit Mary.', 'George will not go to the museum tomorrow unless Mark agrees to go. Mark will go to the museum only if he can postpone most of his appointments. Mark has postponed some of his appointments, so he will go to the museum.']", "label": 0 }, { "id": "train_4289", "context": "Some works of art that today are recognized masterpieces were considered obscene when first created. It ttherefore follows that what is considered obscene or not obscene has changed over time.", "question": "Which one of the following is an assumption on which the argument depends?", "answers": "['Displays of works of art that are considered obscene change the way in which obscenity is defined.', 'Public opinion does not determine the artistic value of a work of art.', 'The number of things that are considered obscene has decreased with the passage of time.', 'Not all currently recognized masterpieces that were once considered obscene are still considered obscene.']", "label": 3 }, { "id": "train_4290", "context": "Chaco Canyon, a settlement of the ancient Anasazi culture in North America, had massive buildings. It must have been a major Anasazi center. Analysis of wood samples shows that some of the timber for the buildings came from the Chuska and San Mateo mountains, 50 miles from Chaco Canyon. Only a major cultural center would have the organizational power to import timber from 50 miles away. ", "question": "In the argument given, the two portions in boldface play which of the following roles?", "answers": "['The first is inferred from another statement in the argument; the second is inferred from the first.', \"The first is one of two premises used to support the argument's main conclusion; the second is the other of those two premises.\", \"The first is the argument's main conclusion; the second is a premise used to support that conclusion.\", \"The first is a premise used to support the argument's main conclusion; the second is a premise used to support another conclusion drawn in the argument.\"]", "label": 2 }, { "id": "train_4291", "context": "When permits for the discharge of chemicals into a waterway are issued, they are issued in terms of the number of pounds of each chemical that can be discharged into the waterway per day. These figures, calculated separately for each chemical for which a permit is issued, are based on an estimate of the effect of the dilution of the chemical by the amount of water flowing through the waterway. The waterway is ttherefore protected against being adversely affected by chemicals discharged under the permits.", "question": "The argument depends on the assumption that", "answers": "['the danger of chemical pollution of waterways is to be evaluated in terms of human health only and not in terms of the health of both human beings and wildlife', 'there is a swift flow of water in the waterway that ensures rapid dispersion of chemicals discharged', 'relatively harmless chemicals do not interact with each other in the water to form harmful compounds', 'there are no chemicals for which discharge into waterways is entirely prohibited']", "label": 2 }, { "id": "train_4292", "context": "Council member: The preservation of individual property rights is of the utmost importance to the city council. Yet, in this city, property owners are restricted to little more than cutting grass and weeding. Anything more extensive, such as remodeling, is prohibited by our zoning laws.", "question": "Which one of the following provides a resolution to the apparent inconsistency described by the council member?", "answers": "['Property owners are sometimes allowed exemptions from restrictive zoning laws.', \"An individual's property rights may be infringed upon by other people altering their own property .\", 'Zoning laws ensure that property rights are not overly extensive.', 'It is in the best interest of property owners to maintain current laws in order to prevent an increase in their property taxes.']", "label": 1 }, { "id": "train_4293", "context": "Mayor: The city council has proposed additional funding for the town library. However, the library's own studies show that fewer than 10% of town residents use the library. The city council claims that the library is underutilized because the books, electronic media, and computer equipment are outdated, and that if these were updated, more residents would use the library. However, the last time the town allocated additional funding to the library, usage of the library did not increase. Ttherefore the town should not allocate additional funding for the library.", "question": "Which of the following most seriously weakens the mayor's argument?", "answers": "[\"Many of the patrons who do use the library are dissatisfied with the library's resources.\", 'The last time the town allocated additional funding for the library, the funding was used to repair the crumbling staircase and ramp leading to the library entrance.', 'A large percentage of library users are children and teens, who do not vote or pay taxes.', 'Usage of the library did increase in 1994, when the town voted to allocate funds for an advertising campaign promoting reading.']", "label": 1 }, { "id": "train_4294", "context": "When drivers are deprived of sleep there are definite behavioral changes, such as slower responses to stimuli and a reduced ability to concentrate, but people' s self-awareness of these changes is poor. Most drivers think they can tell when they are about to fall asleep, but they cannot.", "question": "Each of the following illustrates the principle that the passage illustrates EXCEPT:", "answers": "['Industrial workers who have just been exposed to noxious fumes are not good judges of whether they should keep working.', 'People who are being treated for schizophrenia are not good judges of whether they should continue their medical treatments.', \"Elementary school students who dislike arithmetic are not good judges of whether multiplication tables should be included in the school's curriculum.\", 'People who have been drinking alcohol are not good judges of whether they are too drunk to drive.']", "label": 2 }, { "id": "train_4295", "context": "For similar cars and drivers, automobile insurance for collision damage has always cost more in Greatport than in Fairmont. Police studies, however, show that cars owned by Greatport residents are, on average, slightly less likely to be involved in a collision than cars in Fairmont. Clearly, ttherefore, insurance companies are making a greater profit on collision-damage insurance in Greatport than in Fairmont.", "question": "Which of the following is an assumption on which the argument depends?", "answers": "['The insurance companies were already aware of the difference in the likelihood of collisions before the publication of the police reports.', 'Repairing typical collision damage does not cost more in Greatport than in Fairmont.', 'Fairmont and Greatport are the cities with the highest collision-damage insurance rates.', 'There are no more motorists in Greatport than in Fairmont.']", "label": 1 }, { "id": "train_4296", "context": "Manufacturers of writing paper need to add mineral \"filler\" to paper pulp if the paper made from the pulp is to look white. Without such filler, paper products look grayish. To make writing paper that looks white from recycled paper requires more filler than is required to make such paper from other sources. Ttherefore, barring the more efficient use of fillers in paper manufacturing or the development of paper-whitening technologies that do not require mineral fillers, if writing paper made from recycled paper comes to replace other types of writing paper, paper manufacturers will have to use more filler than they now use.", "question": "Which one of the following is an assumption on which the argument depends?", "answers": "['The total amount of writing paper manufactured worldwide will increase significantly in the future.', 'Certain kinds of paper cannot be manufactured from recycled paper.', 'Beyond a certain limit, increasing the amount of filler added to paper pulp does not increase the whiteness of the paper made from the pulp.', 'Grayish writing paper will not be a universally acceptable alternative to white writing paper.']", "label": 3 }, { "id": "train_4297", "context": "The First Amendment prevents the government from violating a person' s right to free speech. But it does not protect government employees from disciplinary action for all statements they make in the course of their official duties. However, laws should protect them in cases in which employees bring to light information that advances the public interest in honest government and the rule of law.", "question": "Which of the following would provide the best example for the argument above?", "answers": "['A prominent plastic surgeon loses her license after an employee sues her for malpractice after she operates on him.', 'A Congressional aide reveals cases of bribery in the House of Representatives.', \"An energy company's stock plunges after an employee publicizes its accounting records.\", \"A movie star loses popularity because a studio employee tells a magazine about the star's troubled personal life.\"]", "label": 1 }, { "id": "train_4298", "context": "When species are extensively hunted, individuals that reach reproductive maturity early make up a larger proportion of the population, because they have a better chance of reproducing. When species face diminished food resources, on the other hand, individuals tend to take longer to reach reproductive maturity. These considerations may help settle whether the primary cause of the gradual disappearance of North America' s mastodons, prehistoric animals related to elephants, was diminished food resources or human hunting, since __.", "question": "Which of the following most logically completes the reasoning?", "answers": "['from the location of certain hunting weapons in juxtaposition with mastodon remains, it is possible to determine whether those weapons were used to hunt mastodons', 'it is possible to determine whether the earliest human dwellings constructed from mastodon tusks and bones come from an earlier period than the earliest hunting weapons unambiguously associated with mastodon remains', 'from the remains of mastodons already discovered, it can be accurately estimated when mastodons became extinct', 'the average age at which mastodons reached reproductive maturity can be established from mastodon tusks, of which there are many fossils dating from different periods']", "label": 3 }, { "id": "train_4299", "context": "Only a reduction of 10 percent in the number of scheduled flights using Greentown' s airport will allow the delays that are so common there to be avoided. Hevelia airstrip, 40 miles away, would, if upgraded and expanded, be an attractive alternative for fully 20 percent of the passengers using Greentown airport. Nevertheless, experts reject the claim that turning Hevelia into a full-service airport would end the chronic delays at Greentown.", "question": "Which of the following, if true, most helps to justify experts' position?", "answers": "['A second largely undeveloped airstrip close to Greentown airport would be a more attractive alternative than Hevelia for many passengers who now use Greentown.', \"If an airplane has to wait to land, the extra jet fuel required adds significantly to the airline's costs.\", 'Turning Hevelia into a full-service airport would require not only substantial construction at the airport itself, but also the construction of new access highways.', 'Several airlines use Greentown as a regional hub, so that most flights landing at Greentown have many passengers who then take different flights to reach their final destinations.']", "label": 3 }, { "id": "train_4300", "context": "Political scientist: People become unenthusiastic about voting if they believe that important problems can be addressed only by large numbers of people drastically changing their attitudes and that such attitudinal changes generally do not result from government action. The decreasing voter turnout is thus entirely due to a growing conviction that politicians cannot solve the most important problems.", "question": "The reasoning in the political scientist's argument is most vulnerable to criticism on the grounds that the argument", "answers": "['undermines its claim that people no longer believe there are political solutions to important problems by suggesting that people are dissatisfied with politicians', 'presumes, without providing justification, that there is no cause of decreasing voter turnout other than the belief that few important problems can be solved by government action', 'presumes, without providing justification, that there are no political solutions to the most important problems', \"infers that important problems can be seriously addressed if people's attitudes do change from the premise that these problems cannot be addressed if people's attitudes do not change\"]", "label": 1 }, { "id": "train_4301", "context": "Detective: Bill has been accused of committing the burglary at the warehouse last night. But no one saw Bill in the vicinity of the warehouse. So we must conclude that Bill did not commit the burglary.", "question": "The reasoning in the detective's argument is most vulnerable to criticism on the grounds that the argument", "answers": "['treats a lack of evidence against Bill as if it exonerated Bill', 'fails to establish the true identity of the burglar', \"treats evidence that is irrelevant to the burglar's identity as if it were relevant\", \"merely attacks the character of Bill's accusers\"]", "label": 0 }, { "id": "train_4302", "context": "A commission has been formed to report on the nation' s preparedness for a major natural disaster. The commission' s report will not be effective unless the commission speaks with a unified voice. Since individual members of the commission have repeatedly expressed their own opinions about disaster preparedness in the news media well in advance of completion of the report, it will not be effective.", "question": "The conclusion of the argument follows logically if which one of the following is assumed?", "answers": "[\"The commission's report will not be effective if some of the commission members already had opinions about the nation's disaster preparedness even before the commission was formed.\", 'Commission members who have expressed their opinions about disaster preparedness in the news media have also emphasized their commitment to producing an effective report.', \"The commission will be able to speak with a uniform voice only if individual members' opinions about disaster preparedness are not made public before the report is completed.\", \"News organizations should not provide a platform for members of the commission to express their opinions about disaster preparedness if doing so will undermine the effectiveness of the commission's report.\"]", "label": 2 }, { "id": "train_4303", "context": "If one of the effects of a genetic mutation makes a substantial contribution to the survival of the species, then, and only then, will that mutation be favored in natural selection. This process is subject to one proviso, namely that the traits that were not favored, yet were carried along by a trait that was favored, must not be so negative as to annul the benefits of having the new, favored trait.", "question": "If the statements above are true, each of the following could be true EXCEPT:", "answers": "['A species possesses a trait whose effects are all neutral for the survival of that species.', \"A species possesses a trait that reduces the species' survival potential.\", 'All the effects of some genetic mutations contribute substantially to the survival of a species.', 'A genetic mutation whose effects are all neutral to a species is favored in natural selection.']", "label": 3 }, { "id": "train_4304", "context": "All coffee drinkers in an office ought to contribute equally to the fund that pays for the office' s coffee, because, although some coffee drinkers would prefer to pay for their coffee by the cup, or in some other manner, it is better if everyone who drinks the office' s coffee provides the same amount of support to the fund.", "question": "The reasoning in the argument is most vulnerable to criticism on which one of the following grounds?", "answers": "['It offers two alternatives that do not exhaust the possibilities available and then treats those alternatives as the only possible ones.', 'It applies a double standard whereby one group of people is judged wrong and another group judged right for engaging in similar behavior.', 'It offers, in place of support for its conclusion, a mere restatement of that conclusion.', 'It uses emotionally charged terms to characterize unfairly the position it attempts to refute.']", "label": 2 }, { "id": "train_4305", "context": "Garbage in this neighborhood probably will not be collected until Thursday this week. Garbage is usually collected here on Wednesdays, and the garbage collectors in this city are extremely reliable. However, Monday was a public holiday, and after a public holiday that falls on a Monday, garbage throughout the city is supposed to be collected one day later than usual.", "question": "The argument proceeds by", "answers": "['treating something that is probable as though it were inevitable', 'generalizing about all actions of a certain kind on the basis of a description of one such action', 'treating several pieces of irrelevant evidence as though they provide support for the conclusion', 'providing information that allows application of a general rule to a specific case']", "label": 3 }, { "id": "train_4306", "context": "In 2006, hospitals in state W treated over 200, 000 people for secondhand smoking-related diseases. These 200, 000 patients cost healthcare insurance companies over $20 million. Over 120, 000 people with these secondhand smoking-related diseases died in 2006 in state W. Many new smokers take up the habit yearly. Politicians in the legislature have suggested a bill that would require all smokers to pay small premiums to all the major healthcare insurance companies to cover the cost of treating people with secondhand-smoking diseases.", "question": "Which of the following statements, if true, support the politicians' proposed bill?", "answers": "['The 200, 000 people treated in 2006 for secondhand smoking-related diseases were each insured by one of the major healthcare insurance companies.', 'Treating people for secondhand smoking-related diseases is not more costly than treating diseases caused by smoking.', 'People most commonly are exposed to secondhand smoke at bars and restaurants.', 'New smokers are more likely to expose others to secondhand smoke.']", "label": 0 }, { "id": "train_4307", "context": "As a large corporation in a small country, Hachnut wants its managers to have international experience, so each year it sponsors management education abroad for its management trainees . Hachnut has found, however, that the attrition rate of graduates from this program is very high, with many of them leaving Hachnut to join competing firms soon after completing the program. Hachnut does use performance during the program as a criterion in deciding among candidates for management positions, but both this function and the goal of providing international experience could be achieved in other ways . Ttherefore, if the attrition problem cannot be successfully addressed, Hachnut should discontinue the sponsorship program.", "question": "In the argument given, the two boldfaced portions play which of the following roles?", "answers": "['The first introduces a consideration supporting a policy that the argument seeks to evaluate; the second provides evidence for concluding that the policy should be abandoned.', 'The first describes a practice that the argument seeks to justify; the second states a judgment that is used in support of a justification for that practice.', 'The first introduces a practice that the argument seeks to evaluate; the second provides grounds for holding that the practice cannot achieve its objective.', 'The first introduces a policy that the argument seeks to evaluate; the second provides grounds for holding that the policy is not needed.']", "label": 3 }, { "id": "train_4308", "context": "Standard archaeological techniques make it possible to determine the age of anything containing vegetable matter, but only if the object is free of minerals containing carbon. Prehistoric artists painted on limestone with pigments composed of vegetable matter, but it is impossible to collect samples of this prehistoric paint without removing limestone, a mineral containing carbon, with the paint. Ttherefore, it is not possible to determine the age of prehistoric paintings on limestone using standard archaeological techniques.", "question": "Which one of the following, if true, most seriously weakens the argument?", "answers": "['The age of the limestone itself can be determined from samples that contain no vegetable-based paint.', 'The proportion of carbon to other elements in limestone is the same in all samples of limestone.', 'Prehistoric artists did not use anything other than vegetable matter to make their paints.', 'Laboratory procedures exist that can remove all the limestone from a sample of prehistoric paint on limestone.']", "label": 3 }, { "id": "train_4309", "context": "Ampicillin and other modern antibiotics kill a much wider variety of bacteria than penicillin does. They also carry higher profit margins, so drug companies now have an incentive to stop manufacturing the older, less profitable antibiotics. This could cause a penicillin shortage, forcing doctors to use the much more powerful new antibiotics in cases where they might otherwise be unnecessary. Thus, these newer antibiotics are likely to result in an outbreak of diseases caused by drug-resistant bacteria, since __.", "question": "The conclusion of the argument is most strongly supported if which one of the following completes the passage?", "answers": "['drug-resistant bacteria flourish in the absence of competition from a wide variety of other bacteria', 'most bacteria that are resistant to penicillin are not resistant to ampicillin and other modern antibiotics', 'a shortage of penicillin would drive up its price and profit margin', 'treatment of diseases with the powerful new antibiotics is much more expensive than treatment with the older ones']", "label": 0 }, { "id": "train_4310", "context": "In order to maintain a high standard of living, a nation must maintain a functioning infrastructure. Major investment in the improvement of its infrastructure will, over time, reward a nation with a corresponding rise in its standard of living. Hence a nation whose standard of living is on the rise can be safely assumed to be a nation that has invested heavily in improving its infrastructure.", "question": "The reasoning in the argument is flawed because the argument fails to take into account that", "answers": "['nations often experience short-term crises that require that resources be diverted to purposes other than the maintenance and improvement of infrastructure', \"a rise in a nation's standard of living need not be the result of major investments in its infrastructure\", 'many nations are unable to make the needed investments in infrastructure', \"the rise in a nation's standard of living that is prompted by investment in its infrastructure may take a long time to occur\"]", "label": 1 }, { "id": "train_4311", "context": "A hospital should not be entitled to patent the research breakthroughs of its resident physicians. Hospitals, as sources of medical advancement, should encourage the free flow of ideas and the general dissemination of medical knowledge. Yet a hospital that retains the right to patent the breakthroughs of its resident physicians has a motive to suppress information about the discovery of a potentially valuable medicine until the patent for it has been secured. Clearly, suppressing information concerning such discoveries is incompatible with the hospital' s obligation to promote the general dissemination of medical knowledge.", "question": "Which one of the following is an assumption that the argument makes?", "answers": "['If the discoveries of a resident physician are not patented by that hospital, then they will be patented by the physician instead.', 'Hospitals are the only institutions that have an obligation to serve as a source of medical advancement.', 'Most breakthroughs by resident physicians would be profitable if patented.', 'Hospitals that have a motive to suppress information concerning research breakthroughs by their resident physicians will occasionally act on that motive.']", "label": 3 }, { "id": "train_4312", "context": "Political theorist: Even with the best spies, area experts, and satellite surveillance, foreign policy assessments can still lack important information. In such circumstances intuitive judgment is vital. A national leader with such judgment can make good decisions about foreign policy even when current information is incomplete, since__ .", "question": "Which of the following, if true, most logically completes the argument?", "answers": "['good foreign policy decisions often lead to improved methods of gathering information', 'the central reason for failure in foreign policy decision making is the absence of critical information', 'intuitive judgment can produce good decisions based on past experience, even when there are important gaps in current information', 'those leaders whose foreign policy decisions have been highly ranked have also been found to have good intuitive judgment']", "label": 2 }, { "id": "train_4313", "context": "Several legislators claim that the public finds many current movies so sexually explicit as to be morally offensive. However, these legislators have misrepresented public opinion. In a survey conducted by a movie industry guild, only 15 percent of respondents think that movies are too sexually explicit, and only 4 percent found any recent movie to be morally offensive. These low percentages are telling because the respondents see far more current movies than does the average moviegoer.", "question": "The reasoning in the argument is flawed in that the argument", "answers": "['fails to consider the possibility that sexually explicit movies increase the prevalence of antisocial behavior', \"attempts to undermine the legislators' credibility instead of addressing their argument\", 'generalizes from a sample that is unlikely to be representative of public sentiment', 'bases its conclusion on subjective judgments rather than on an objective criterion of moral offensiveness']", "label": 2 }, { "id": "train_4314", "context": "For years, a rare variety of camel was endangered because much of its habitat was used as a weapons testing range. After the testing range closed, however, the population of these camels began falling even more quickly.", "question": "Which one of the following, if true, most helps to explain the increased rate of population loss?", "answers": "['The weapons tests had kept wildlife poachers out of the testing range.', 'Because of unexploded bombs, the land within the testing range was still somewhat dangerous after the range closed down.', 'The weapons tests were most harmful to the camels in years when food was scarce.', 'The camels had to overcome two different outbreaks of disease during the time the testing range was in operation.']", "label": 0 }, { "id": "train_4315", "context": "Professor: The United States faces several threats, and of those threats, terrorism is by far the most dangerous. Instability across the Middle East breeds extremism. The United States must be proactive in protecting herself and her allies.", "question": "The professor would most likely agree that:", "answers": "['Extremism is even more dangerous than terrorism.', 'The United States should invade the Middle East.', 'If the United States becomes aware of an imminent threat to her security, the country should respond militarily.', 'The United States should spread democracy in the Middle East.']", "label": 2 }, { "id": "train_4316", "context": "In Mernia commercial fossil hunters often sell important fossils they have found, not to universities or museums, but to individual collectors, who pay much better but generally do not allow researchers access to their collections. To increase the number of fossils available for research, some legislators propose requiring all fossils that are found in Mernia to be sold only to universities or museums.", "question": "Which of the following, if true, most strongly indicates that the legislators' proposal will fail to achieve its goal?", "answers": "['Some fossil hunters in Mernia are not commercial fossil hunters, but rather are amateurs who keep the fossils that they find.', 'Commercial fossil hunters in Mernia currently sell some of the fossils they find to universities and museums.', 'Many universities in Mernia do not engage in fossil research.', 'Most fossils are found by commercial fossil hunters, and they would give up looking for fossils if they were no longer allowed to sell to individual collectors.']", "label": 3 }, { "id": "train_4317", "context": "Most large nurseries sell raspberry plants primarily to commercial raspberry growers and sell only plants that are guaranteed to be disease-free. However, the shipment of raspberry plants that Johnson received from Wally' s Plants carried a virus that commonly afflicts raspberries.", "question": "Which one of the following is most strongly supported by the information above?", "answers": [ "Wally's Plants is probably not a large, well-run nursery if it sells its raspberry plants primarily to commercial raspberry growers.", "If Wally's Plants is a large nursery, then the raspberry plants that Johnson received in the shipment were probably not entirely as they were guaranteed to be.", "If Johnson is a commercial raspberry grower and Wally's Plants is not a large nursery, then the shipment of raspberry plants that Johnson received was probably guaranteed to be disease-free.", "If Johnson is not a commercial raspberry grower, then Wally's Plants is probably not a large nursery." ], "label": 1 }, { "id": "train_4318", "context": "The demand for used cars has risen dramatically in Germany in recent years. Most of this demand is generated by former East Germans who cannot yet afford new cars and for whom cars were generally unavailable prior to unification. This demand has outstripped supply and thus has exerted an upward pressure on the prices of used cars. Consequently, an increasing number of former West Germans, in order to take advantage of the improved market, will be selling the cars they have owned for several years. Hence, the German new-car market will most likely improve soon as well.", "question": "Which one of the following, if true, would most help to support the conclusion about the German new-car market?", "answers": "['Most Germans own very few cars in the course of their lives.', 'The demand for old cars in former West Germany is greater than the demand for new cars in former East Germany.', 'Most former West Germans purchase new cars once they sell their used cars.', 'In most European countries, the sale of a used car is subject to less tax than is the sale of a new car.']", "label": 2 }, { "id": "train_4319", "context": "Marianna: The problem of drunk driving has been somewhat ameliorated by public education and stricter laws. Additional measures are nevertheless needed. People still drive after drinking, and when they do the probability is greatly increased that they will cause an accident involving death or serious injury. David: I think you exaggerate the dangers of driving while drunk. Actually, a driver who is in an automobile accident is slightly less likely to be seriously injured if drunk than if sober.", "question": "In responding to Marianna's argument, David makes which one of the following errors of reasoning?", "answers": "['He argues against a point that is not one that Marianna was making.', \"He contradicts Marianna's conclusion without giving any evidence for his point of view.\", 'He directs his criticism against the person making the argument rather than directing it against the argument itself.', 'He contradicts himself.']", "label": 0 }, { "id": "train_4320", "context": "Any course that teaches students how to write is one that will serve them well in later life. Ttherefore, since some philosophy courses teach students how to write, any student, whatever his or her major, will be served well in later life by taking any philosophy course.", "question": "A flaw in the reasoning of the argument is that the argument", "answers": "['draws a weaker conclusion than is warranted by the strength of its premises', 'draws a conclusion about all cases of a certain kind on the basis of evidence that justifies such a conclusion only about some cases of that kind', 'presumes, without providing justification, that what is true of a whole must also be true of each of its constituent parts', 'fails to consider the possibility that some students in certain majors may be required to take a philosophy course']", "label": 1 }, { "id": "train_4321", "context": "A newly discovered painting seems to be the work of one of two 17th-century artists, either the northern German Johannes Drechen or the Frenchman Louis Birelle, who sometimes painted in the same style as Drechen. Analysis of the carved picture frame, which has been identified as the painting' s original 17th century frame, showed that it is made of wood found widely in northern Germany at the time, but rare in the part of France where Birelle lived. This shows that the painting is most likely the work of Drechen.", "question": "Which of the following is an assumption that the argument requires?", "answers": "['Sometimes a painting so closely resembles others of its era that no expert is able to confidently decide who painted it.', 'The carving style of the picture frame is not typical of any specific region of Europe.', 'Drechen is unlikely to have ever visited the home region of Birelle in France.', 'The frame was made from wood local to the region where the picture was painted.']", "label": 3 }, { "id": "train_4322", "context": "The Central City Church building was recently damaged by a fire that occurred overnight. The insurance company will not pay on a claim if it is determined that a fire was started deliberately. Since nobody was in the Central City Church building at the time the fire swept through the structure, the insurance company will most certainly reimburse the church for the damage caused by the fire.", "question": "Which of the following is an assumption that is required to reach the conclusion above?", "answers": "['No individual would fraudulently profit by setting a fire in the church building.', 'The insurance company typically pays claims for similar situations in other non-residential structures, such as office buildings and restaurants.', 'There was no one in the building at the time the fire swept through the structure.', 'The fire was not deliberately started by someone who then left the building before the fire grew.']", "label": 3 }, { "id": "train_4323", "context": "Psychologists today recognize childhood as a separate stage of life which can only be understood in its own terms, and they wonder why the Western world took so long to see the folly of regarding children simply as small, inadequately socialized adults. Most psychologists, however, persist in regarding people 70 to 90 years old as though they were 35 year olds who just happen to have white hair and extra leisure time. But old age is as fundamentally different from young adulthood and middle age as childhood is -- a fact attested to by the organization of modern social and economic life. Surely it is time, ttherefore, to acknowledge that serious research into the unique psychology of advanced age has become indispensable.", "question": "Which one of the following principles, if established, would provide the strongest backing for the argument?", "answers": "['Whenever most practitioners of a given discipline approach a particular problem in the same way, that uniformity is good evidence that all similar problems should also be approached in that way.', 'Whenever psychologists agree that a single psychology is inadequate for two distinct age groups, they should be prepared to show that there are greater differences between the two age groups than there are between individuals in the same age group.', 'Whenever two groups of people are so related to each other that any member of the second group must previously have been a member of the first, people in the first group should not be regarded simply as deviant members of the second group.', \"Whenever a society's economic life is so organized that two distinct times of life are treated as being fundamentally different from one another, each time of life can be understood only in terms of its own distinct psychology.\"]", "label": 3 }, { "id": "train_4324", "context": "In many languages other than English there is a word for \"mother' s brother\"which is different from the word for \"father' s brother, \" whereas English uses the word \"uncle\" for both. Thus, speakers of these languages evidence a more finely discriminated kinship system than English speakers do. The number of basic words for colors also varies widely from language to language. Ttherefore, speakers of languages that have fewer basic words for colors than English has must be perceptually unable to distinguish as many colors as speakers of English can distinguish.", "question": "Which one of the following, if true, undermines the conclusion concerning words for colors?", "answers": "['Khmer uses a basic word corresponding to English \"blue\"for most leaves, but uses its basic word corresponding to English \"green\" for unripe bananas', 'Speakers of English are able to distinguish between lighter and darker shades of the color they call \"blue, \"for which Russian has two different basic words.', 'The word \"orange\"in English has the same origin as the equivalent word in Spanish.', 'Almost every language distinguishes red from the other colors']", "label": 1 }, { "id": "train_4325", "context": "The high cost of production is severely limiting which operas are available to the public. These costs necessitate reliance on large corporate sponsors, who in return demand that only the most famous operas be produced. Determining which operas will be produced should rest only with ticket purchasers at the box office, not with large corporate sponsors. If we reduce production budgets so that operas can be supported exclusively by box-office receipts and donations from individuals, then the public will be able to see less famous operas.", "question": "Which one of the following, if true, would weaken the argument?", "answers": "['A few opera ticket purchasers go to the opera for the sake of going to the opera, not to see specific operatic productions.', 'The reduction of opera production budgets would not reduce the desire of large corporate sponsors to support operas.', 'Large corporate sponsors will stop supporting opera productions if they are denied control over which operas will be produced.', 'Without the support of large corporate sponsors, opera companies could not afford to produce any but the most famous of operas.']", "label": 3 }, { "id": "train_4326", "context": "Columnist: It may soon be possible for an economy to function without paper money. Instead, the government would electronically record all transactions as they take place. However, while this may be technologically feasible it would never be willingly accepted by a society, for it gives the government too much power. People are rightly distrustful of governments with too much power.", "question": "Which one of the following most accurately expresses the overall conclusion of the columnist's argument?", "answers": "['It is reasonable for people to distrust a government that has too much power.', 'Even though it may be technologically feasible, no government will be able to operate an economy without the use of paper money.', 'New technology may soon make it possible for an economy to operate without paper money.', 'A society would never willingly accept a system in which, in lieu of paper money, the government keeps track of every transaction electronically.']", "label": 3 }, { "id": "train_4327", "context": "One of the limiting factors in human physical performance is the amount of oxygen that is absorbed by the muscles from the bloodstream. Accordingly, entrepreneurs have begun selling at gymnasiums and health clubs bottles of drinking water, labeled \"SuperOXY, \" that has extra oxygen dissolved in the water. Such water would be useless in improving physical performance, however, since the amount of oxygen in the blood of someone who is exercising is already more than the muscle cells can absorb .", "question": "Which of the following, if true, would serve the same function in the argument as the statement in boldface?", "answers": "['the only way to get oxygen into the bloodstream so that it can be absorbed by the muscles is through the lungs', \"frequent physical exercise increases the body's ability to take in and use oxygen\", 'lack of oxygen is not the only factor limiting human physical performance', 'the water lost in exercising can be replaced with ordinary tap water']", "label": 0 }, { "id": "train_4328", "context": "A patient complained of feeling constantly fatigued. It was determined that the patient averaged only four to six hours of sleep per night, and this was determined to contribute to the patient' s condition. However, the patient was not advised to sleep more.", "question": "Which one of the following, if true, most helps to resolve the apparent discrepancy in the information above?", "answers": "['Most people who suffer from nightmares experience them in the last hour of sleep before waking.', 'Worry about satisfying the need for sufficient sleep can make it more difficult to sleep.', 'The first two hours of sleep do the most to alleviate fatigue.', \"The shorter one's sleep time, the easier it is to awaken from sleeping.\"]", "label": 1 }, { "id": "train_4329", "context": "Ullman: Plato argued that because of the harmful ways in which music can manipulate the emotions, societies need to put restrictions on the music their citizens hear. However, because musicians seek not to manipulate the emotions but to create beauty, this argument is misguided.", "question": "Ullman's argument is most vulnerable to criticism on the grounds that it fails to consider the possibility that", "answers": "[\"artists who are trying to manipulate people's emotions to control them are not likely to admit it\", 'those with the power to censor music would not censor other forms of expression', 'there are other, more convincing arguments for allowing the censorship of music', 'what musicians intend their music to do and what it actually does are different']", "label": 3 }, { "id": "train_4330", "context": "Popular science publications that explain new developments in science face a dilemma. In order to reach a wide audience, these publications must rely heavily on metaphorical writing, which usually fails to convey the science accurately. If the writing is more rigorous, they get the science right but fail to reach a wide audience. These publications should ttherefore give up trying to explain new developments in science to a wide audience.", "question": "Which one of the following principles, if valid, most helps to justify the reasoning in the argument?", "answers": "['In reporting scientific developments, it is better to fail to reach a wide audience than to be inaccurate.', 'In reporting scientific developments, it is better to reach a wide audience than to be accurate.', 'Science publications should balance the use of metaphors with more rigorous writing.', 'Even the most rigorous explanations of some scientific concepts must still contain metaphors.']", "label": 0 }, { "id": "train_4331", "context": "The top 50 centimeters of soil on Tiliga Island contain bones from the native birds eaten by the islanders since the first human immigration to the island 3, 000 years ago. A comparison of this top layer with the underlying 150 centimeters of soil -- accumulated over 80, 000 years -- reveals that before humans arrived on Tiliga, a much larger and more diverse population of birds lived there. Thus, the arrival of humans dramatically decreased the population and diversity of birds on Tiliga.", "question": "Which one of the following statements, if true, most seriously weakens the argument?", "answers": "['Many of the bird species that disappeared from Tiliga did not disappear from other, similar, uninhabited islands until much later.', 'Bones from bird species known to have been eaten by the islanders were found in the underlying 150 centimeters of soil.', 'The bird species known to have been eaten by the islanders had few natural predators on Tiliga.', 'The arrival of a species of microbe, carried by some birds but deadly to many others, immediately preceded the first human immigration to Tiliga.']", "label": 3 }, { "id": "train_4332", "context": "Student representative: Our university, in expelling a student who verbally harassed his roommate, has erred by penalizing the student for doing what he surely has a right to do: speak his mind! Dean of students: But what you' re saying is that our university should endorse verbal harassment. Yet surely if we did that, we would threaten the free flow of ideas that is the essence of university life.", "question": "Which one of the following is a questionable technique that the dean of students uses in attempting to refute the student representative?", "answers": "['relying on a position of power to silence the opposing viewpoint with a threat', 'questioning the motives of the student representative rather than offering reasons for the conclusion defended', \"misdescribing the student representative's position, thereby making it easier to challenge\", 'invoking a fallacious distinction between speech and other sorts of behavior']", "label": 2 }, { "id": "train_4333", "context": "To reduce waste of raw materials, the government of Sperland is considering requiring household appliances to be broken down for salvage when discarded. To cover the cost of salvage, the government is planning to charge a fee, which would be imposed when the appliance is first sold. Imposing the fee at the time of salvage would reduce waste more effectively, however, because consumers tend to keep old appliances longer if they are faced with a fee for discarding them.", "question": "Which of the following, if true, most seriously weakens the argument?", "answers": "['Many nonfunctioning appliances that are currently discarded could be repaired at relatively little expense.', 'The fee provides manufacturers with no incentive to produce appliances that are more durable.', 'Increasing the cost of disposing of an appliance properly increases the incentive to dispose of it improperly.', 'For people who have bought new appliances recently, the salvage fee would not need to be paid for a number of years.']", "label": 2 }, { "id": "train_4334", "context": "Editorial: It is clear that what is called \"health education\" is usually propaganda rather than education. Propaganda and education are never the same thing. The former is nothing but an attempt to influence behavior through the repetition of simplistic slogans, whereas the latter never involves such a method. Though education does attempt to influence behavior, it does so by offering information in all its complexity, leaving it up to the individual to decide how to act on that information. Sadly, however, propaganda is much more successful than education.", "question": "The conclusion drawn by the editorial follows logically if it is assumed that what is called \"health education\" usually", "answers": "['attempts to influence behavior solely by repeating simplistic slogans', 'does not leave it up to the individual to decide how to act on information', 'does not offer information in all its complexity', \"is very successful in influencing people's behavior\"]", "label": 0 }, { "id": "train_4335", "context": "The closest distance from which an asteroid has been photographed using ground-based radar is 2. 2 million miles, the distance from which the asteroid Toutatis was recently photographed. The closest photograph of an asteroid is of Gaspra, which was photographed from a distance of only 10, 000 miles.", "question": "Which of the following can be properly concluded from the statements above?", "answers": "['Ground-based radar photography cannot take photographs of objects much beyond 2. 2 million miles from Earth.', 'The photograph of Gaspra was not taken using ground-based radar.', 'Toutatis, unlike Gaspra, has only recently been discovered.', 'Toutatis is more likely to collide with the Earth than Gaspra is.']", "label": 1 }, { "id": "train_4336", "context": "Mark: Plastic-foam cups, which contain environmentally harmful chlorofluorocarbons, should no longer be used; paper cups are preferable. Styrene, a carcinogenic by-product, is generated in foam production, and foam cups, once used, persist indefinitely in the environment. Tina: You overlook the environmental effects of paper cups. A study done 5 years ago showed that making paper for their production burned more petroleum than was used for foam cups and used 12 times as much steam, 36 times as much electricity, and twice as much cooling water. Because paper cups weigh more, their transportation takes more energy. Paper mills produce water pollution, and when the cups decay they produce methane, a gas that contributes to harmful global warming. So they are a worse choice.", "question": "Which one of the following, if true, could Mark cite to counter evidence offered by Tina?", "answers": "['The production and transportation of petroleum occasions serious environmental pollution, but the energy that runs paper mills now comes from burning waste wood rather than petroleum.', 'The use of energy for chain saws that cut down trees and for trucks that haul logs is part of the environmental cost of manufacturing paper.', 'Foam cups are somewhat more acceptable to consumers than paper cups because of their better insulating qualities.', 'Acre for acre, tree farms for the production of wood for paper have fewer beneficial effects on the environment than do natural forests that remain uncut.']", "label": 0 }, { "id": "train_4337", "context": "Many people say that the press should not pry into the personal lives of private individuals. But the press has the right to publish any story of interest to the public unless that story is libelous. So, if a story about a private individual is not libelous, the press has an obligation to publish it, for such information is clearly of interest to the public.", "question": "The argument's reasoning is vulnerable to criticism on the grounds that the argument presumes, without giving warrant, that", "answers": "[\"the press's right to publish always outweighs the individual's right not to be libeled\", \"one's having a right to do something entails one's having an obligation to do it\", 'the press can publish nonlibelous stories about private individuals without prying into their personal lives', 'if one has an obligation to do something then one has a right to do it']", "label": 1 }, { "id": "train_4338", "context": "X: Medical research on animals should not be reduced in response to a concern for animals, because results of such research serve to avert human suffering. In such research a trade-off between human and animal welfare is always inevitable, but we should give greater weight to human welfare. Y: With technology that is currently available, much of the research presently performed on animals could instead be done with computer modeling or human subjects without causing any suffering.", "question": "The relationship of Y's response to X's argument is that Y's response", "answers": "[\"supplies a premise to X's argument that was not explicitly stated\", 'disagrees with X about the weight to be given to animal suffering as opposed to human suffering', \"presents a logical consequence of the premises of X's argument\", \"contradicts a premise on which X's argument relies\"]", "label": 3 }, { "id": "train_4339", "context": "Baumgartner' s comparison of the environmental hazards of gasoline-powered cars with those of electric cars is misleading. He examines only production of the cars, whereas it is the product' s total life cycle -- production, use, and recycling -- that matters in determining its environmental impact. A typical gasoline-powered car consumes 3 times more resources and produces 15 to 20 times more air pollution than a typical electric car.", "question": "Which one of the following most accurately expresses the conclusion of the argument?", "answers": "['The total life cycle of a product is what matters in assessing its environmental impact.', 'Baumgartner uses inaccurate data in his comparison of the environmental hazards of gasoline-powered and electric cars.', 'The production of gasoline-powered cars creates more environmental hazards than does that of electric cars.', 'Baumgartner makes a deceptive comparison between the environmental hazards of gasoline-powered and electric cars.']", "label": 3 }, { "id": "train_4340", "context": "Worldwide, more books were sold last year than in any previous year. In particular, there were more cookbooks sold. For the first time ever, most of the cookbooks sold were not intended for beginners. Indeed, more cookbooks than ever were purchased by professional cooks. However, one of the few books available on every continent is a cookbook written for beginners, entitled Problem-Free Cooking.", "question": "Which one of the following is most strongly supported by the information above?", "answers": "['Last year there were more cookbooks sold that were not intended for beginners than in any previous year.', 'Sales of cookbooks intended for beginners were lower last year than in previous years.', 'Problem-Free Cooking sold more copies last year than did any cookbook written for professional cooks.', 'The best-selling cookbook last year was a cookbook that was intended for beginners.']", "label": 0 }, { "id": "train_4341", "context": "Measurements of the motion of the planet Uranus seem to show Uranus being tugged by a force pulling it away from the Sun and the inner planets. Neptune and Pluto, the two known planets whose orbits are farther from the Sun than is the orbit of Uranus, do not have enough mass to exert the force that the measurements indicate. Ttherefore, in addition to the known planets, there must be at least one planet in our solar system that we have yet to discover.", "question": "Which one of the following, if true, most seriously weakens the argument?", "answers": "['There is a belt of comets beyond the orbit of Pluto with powerful gravitational pull.', 'The force the Sun exerts on Uranus is weaker than the force it exerts on the inner planets.', 'Neither Neptune nor Pluto is as massive as Uranus.', 'Pluto was not discovered until 1930.']", "label": 0 }, { "id": "train_4342", "context": "The Fieldpark nuclear power plant, cited three years ago by a governmentally led safety commission for exceeding safe levels of radiation, has recently allowed the media access to its facilities. A spokesperson for the plant has claimed that the media has full access to the plant and is not prohibited from taking pictures on the premises. The spokesperson promises that, as a result, the citizens of Fieldpark can rest assured that, until next year's governmental inspection, the facility will not exceed the federal regulations on the amount of sieverts, or radiation, to which a person can be exposed.", "question": "Which of the following is an assumption upon which the spokesperson's conclusion depends?", "answers": "['There were verifiable cases of radiation sickness in the year before the plant was cited by the safety commission.', 'The number of sieverts the plant releases has not increased since the last governmental inspection.', 'Photos can establish with the same reliability what a government safety commission can.', 'The media will publish all of the photos it deems incriminating.']", "label": 2 }, { "id": "train_4343", "context": "Municipal officials originally estimated that it would be six months before municipal road crews could complete repaving a stretch of road. The officials presumed that private contractors could not finish any sooner. However, when the job was assigned to a private contractor, it was completed in just 28 days.", "question": "Which one of the following, if true, does most to resolve the discrepancy between the time estimated for completion of the repaving job, and the actual time taken by the private contractor?", "answers": "['The labor union contract for road crews employed by both municipal agencies and private contractors stipulates that employees can work only eight hours a day, five days a week, before being paid overtime.', 'Many road-crew workers for private contractors have previously worked for municipal road crews, and vice versa.', 'Municipal agencies must conduct a lengthy bidding process to procure supplies after repaving work is ordered and before they can actually start work, whereas private contractors can obtain supplies readily as needed.', 'Road repaving work can only be done in the summer months of June, July, and August.']", "label": 2 }, { "id": "train_4344", "context": "The years 1917, 1937, 1956, 1968, 1979, and 1990 are all notable for the occurrence of both popular uprisings and near-maximum sunspot activity. During heavy sunspot activity, there is a sharp rise in positively charged ions in the air people breathe, and positively charged ions are known to make people anxious and irritable. Ttherefore, it is likely that sunspot activity has actually been a factor in triggering popular uprisings.", "question": "Which one of the following exhibits a pattern of reasoning most similar to that in the passage?", "answers": "['The ancient Greeks sometimes attempted to predict the outcome of future events by watching the flight patterns of birds. Since the events themselves often matched the predictions, the birds were probably responding to some factor that also influenced the events.', \"The thirty healthiest people in a long-term medical study turned out to be the same thirty whose regular diets included the most vegetables. Since specific substances in vegetables are known to help the body fight disease, vegetables should be part of everyone's diet.\", \"Martha, Sidney, and Hilary are the city's three most powerful politicians, and all three graduated from Ridgeview High School. Although Ridgeview never had a reputation for excellence, it must have been a good school to have produced three such successful graduates.\", \"Acme's most productive managers are consistently those who occupy the corner offices, which have more windows than other offices at Acme. Since people are more alert when they are exposed to abundant natural light, the greater productivity of these managers is probably at least in part a result of their working in the corner offices.\"]", "label": 3 }, { "id": "train_4345", "context": "More and more academic institutions are using citation analysis as the main technique for measuring the quality of scientific research. This technique involves a yearly scanning of scientific journals to count the number of references to a researcher' s work. Although academic institutions want to encourage good research, use of citation analysis actually works against this goal since scientists seeking to maximize citation counts will avoid multiyear projects in favor of short-term projects in faddish areas.", "question": "Which one of the following, if true, provides the strongest support for the argument?", "answers": "[\"Scientists often cite their colleagues' work when they think it is unfairly neglected by the scientific establishment.\", 'In general scientific research is not referred to in journals until the research is completed.', 'Research that is initially criticized in scientific journals sometimes turns out to be groundbreaking work.', 'Scientists are sometimes hostile to interim assessments of ongoing research, since such assessments might threaten continuity of funding.']", "label": 1 }, { "id": "train_4346", "context": "Most parents who are generous are good parents, but some self-centered parents are also good parents. Yet all good parents share one characteristic: they are good listeners.", "question": "If all of the statements in the passage are true, which one of the following must also be true?", "answers": "['All parents who are good listeners are good parents.', 'Some parents who are good listeners are self-centered.', 'Some parents who are good listeners are not good parents.', 'Most parents who are good listeners are generous.']", "label": 1 }, { "id": "train_4347", "context": "In Patton City, days are categorized as having heavy rainfall (more than two inches), moderate rainfall (more than one inch, but no more than two inches), light rainfall (at least a trace, but no more than one inch), or no rainfall. In 1990, there were fewer days with light rainfall than in 1910 and fewer with moderate rainfall, yet total rainfall for the year was 20 percent higher in 1990 than in 1910.", "question": "If the statements above are true, then it is also possible that in Patton City", "answers": "['the number of days with heavy rainfall was lower in 1990 than in 1910', 'the average amount of rainfall per month was lower in 1990 than in 1910', 'the number of days with some rainfall, but no more than two inches, was the same in 1990 as in 1910', 'the total number of inches of rain that fell on days with moderate rainfall in 1990 was more than twice what it had been in 1910']", "label": 0 }, { "id": "train_4348", "context": "Physician: Hatha yoga is a powerful tool for helping people quit smoking. In a clinical trial, those who practiced hatha yoga for 75 minutes once a week and received individual counseling reduced their smoking and cravings for tobacco as much as did those who went to traditional self-help groups once a week and had individual counseling.", "question": "Which one of the following is an assumption on which the physician's argument relies?", "answers": "['People who practice hatha yoga for 75 minutes once a week are not damaging themselves physically.', 'Most smokers are able to practice hatha yoga more than once a week.', 'The individual counseling received by the smokers in the clinical trial who practiced hatha yoga did not help them quit smoking.', 'Traditional self-help groups are powerful tools for helping people quit smoking.']", "label": 3 }, { "id": "train_4349", "context": "Politician: Every regulation currently being proposed by the Committee for Overseas Trade will reduce the trade deficit. Our country' s trade deficit is so large that it weakens the economy. Ttherefore, each of the proposed regulations would help the economy.", "question": "The reasoning in the politician's argument is flawed in that the argument", "answers": "['takes for granted that the trade deficit will increase in size if no action is taken to reduce it', 'merely appeals to the authority of the committee without evaluating any reasons for the proposed regulations', 'concludes that every regulation in a set will have the same effects as a set of regulations as a whole', 'fails to consider the possibility that one effect of a regulation will be offset by other effects']", "label": 3 }, { "id": "train_4350", "context": "Psychologists observing a shopping mall parking lot found that, on average, drivers spent 39 seconds leaving a parking space when another car was quietly waiting to enter it, 51 seconds if the driver of the waiting car honked impatiently, but only 32 seconds leaving a space when no one was waiting. This suggests that drivers feel possessive of their parking spaces even when leaving them, and that this possessiveness increases in reaction to indications that another driver wants the space.", "question": "Which one of the following, if true, most weakens the reasoning?", "answers": "['The amount of time drivers spend entering a parking space is not noticeably affected by whether other drivers are waiting for them to do so, nor by whether those other drivers are honking impatiently.', 'Parking spaces in shopping mall parking lots are unrepresentative of parking spaces in general with respect to the likelihood that other cars will be waiting to enter them.', 'Almost any driver leaving a parking space will feel angry at another driver who honks impatiently, and this anger will influence the amount of time spent leaving the space.', 'The more pressure most drivers feel because others are waiting for them to perform maneuvers with their cars, the less quickly they are able to perform them.']", "label": 3 }, { "id": "train_4351", "context": "Educational television is a contradiction in terms. While a classroom encourages social interaction, television encourages solitude. School is centered on the development of language, but television depends upon constantly changing visual images. And in a classroom, fun is merely a means to an end, but on television it is the end in itself.", "question": "Upon which one of the following assumptions does the author rely in the passage?", "answers": "['The classroom should not be a place where anyone has fun.', 'Television programs reinforce some of the values of the school environment.', 'Only experiences that closely resemble what takes place in the school environment can be educational.', 'The potential of television as a powerful learning tool has not yet been realized.']", "label": 2 }, { "id": "train_4352", "context": "Coach: An acceptable way of building a more successful team is not readily attainable. Paying large salaries to star athletes for short contract periods limits our ability to plan for long-term team restructuring. If the team' s budget is focused on talented, yet unskilled, rookies, our coaching staff is forced to expend large amounts of time and energy getting these players ready for the season. In addition, trading older players who are no longer as reliable can cause discontent among nostalgic fans.", "question": "The claim that trading older players who are no longer reliable could cause discontent among nostalgic fans plays which one of the following roles in the coach's argument?", "answers": "['It supports the claim that an acceptable way of building a more successful team is not readily attainable.', \"It argues for an alternative solution to the problem of focusing the team's budget on talented, yet unskilled, rookies.\", 'It presents another consideration for why the coaching staff should not expend large amounts of time and energy on rookie athletes.', \"It presents an objection to the claim that short contract periods limit a team's ability to plan for long-term team restructuring.\"]", "label": 0 }, { "id": "train_4353", "context": "The air quality board recently informed Coffee Roast, a small coffee roasting firm, of a complaint regarding the smoke from its roaster. Recently enacted air quality regulations requires machine roasting more than 10 pounds of coffee to be equipped with expensive smoke-dissipating afterburners. The firm, however, roasts only 8 pounds of coffee at a time. Nevertheless, the company has decided to purchase and install an afterburner.", "question": "Which of the following, if true, most strongly supports the firm's decision?", "answers": "['Coffee roasted in a machine equipped with an afterburner has its flavor subtly altered.', 'Until setting on the new air quality regulations, the board had debated whether to require afterburners for machines roasting more than 5 pounds of coffee at a time.', 'The firm has reason to fear that negative publicity regarding the complaints could result in lost sales.', 'The cost to the firm of an afterburner is less than the cost of replacing its roaster with a smaller one.']", "label": 2 }, { "id": "train_4354", "context": "People who consume a lot of honey tend to have fewer cavities than others have. Yet, honey is high in sugar, and sugar is one of the leading causes of tooth decay.", "question": "Which one of the following, if true, most helps to resolve the apparent paradox described above?", "answers": "['People who eat a lot of honey tend to consume very little sugar from other sources.', 'Refined sugars have been linked to more health problems than have unrefined sugars.', 'Honey contains bacteria that inhibit the growth of the bacteria that cause tooth decay.', 'Many people who consume a lot of honey consume much of it dissolved in drinks.']", "label": 2 }, { "id": "train_4355", "context": "Advertisement: Each of the Economic Merit Prize winners from the past 25 years is covered by the Acme retirement plan. Since the winners of the nation' s most prestigious award for economists have thus clearly recognized that the Acme plan offers them a financially secure future, it is probably a good plan for anyone with retirement needs similar to theirs.", "question": "The advertisement's argumentation is most vulnerable to criticism on which one of the following grounds?", "answers": "['It fails to address adequately the possibility that any of several retirement plans would be good enough for, and offer a financially secure future to, Economic Merit Prize winners.', \"It presumes, without providing justification, that each of the Economic Merit Prize winners has retirement plan needs that are identical to the advertisement's intended audience's retirement plan needs.\", 'It takes for granted that some winners of the Economic Merit Prize have deliberately selected the Acme retirement plan, rather than having had it chosen for them by their employers.', \"It appeals to the fact that supposed experts have endorsed the argument's main conclusion, rather than appealing to direct evidence for that conclusion.\"]", "label": 2 }, { "id": "train_4356", "context": "At a recent conference on environmental threats to the North Sea, most participating countries favored uniform controls on the quality of effluents, whether or not specific environmental damage could be attributed to a particular source of effluent. What must, of course, be shown, in order to avoid excessively restrictive controls, is that __ .", "question": "Which of the following best completes the passage below?", "answers": "['any uniform controls that are adopted are likely to be implemented without delay', 'all of any given pollutant that is to be controlled actually reaches the North Sea at present', 'any substance to be made subject to controls can actually cause environmental damage', 'the countries favoring uniform controls are those generating the largest quantities of effluents']", "label": 2 }, { "id": "train_4357", "context": "A six-month public health campaign sought to limit the spread of influenza by encouraging people to take precautions such as washing their hands frequently and avoiding public places when they experience influenza symptoms. Since the incidence of influenza was much lower during those months than experts had predicted, the public evidently heeded the campaign.", "question": "Which one of the following, if true, most strengthens the argument?", "answers": "['The incidence of food-borne illnesses, which can be effectively controlled by frequent hand washing, was markedly lower than usual during the six-month period.', \"Independently of the public health campaign, the news media spread the message that one's risk of contracting influenza can be lessened by frequent hand washing.\", 'During the six-month period, the incidence of the common cold, which has many of the same symptoms as influenza, was about the same as usual.', 'There were fewer large public gatherings than usual during the six-month period.']", "label": 0 }, { "id": "train_4358", "context": "In her new book on ancient Egyptian historical research, M frequently attributes bad faith to researchers disagreeing with her. A troubling aspect of M' s book is her stated conviction that other archaeologists' funding sources often determine what \"findings\" archaeologists report. Add to this that M has often shown herself to be pompous, pushy, and sometimes just plain cruel, and it becomes clear that M' s book does not merit attention from serious professionals.", "question": "The author of the book review commits which one of the following reasoning errors?", "answers": "['using an attack on the character of the writer of the book as evidence that this person is not competent on matters of scientific substance', 'presenting as facts several assertions about the book under review that are based only on strong conviction and would be impossible for others to verify', 'taking it for granted that an archaeologist is unlikely to report findings that are contrary to the interests of those funding the investigation', 'failing to distinguish between the criteria of being true and of being sufficiently interesting to merit attention']", "label": 0 }, { "id": "train_4359", "context": "Jane: Professor Harper' s ideas for modifying the design of guitars are of no value because there is no general agreement among musicians as to what a guitar should sound like and, consequently, no widely accepted basis for evaluating the merits of a guitar' s sound. Mark: What' s more, Harper' s ideas have had enough time to be adopted if they really resulted in superior sound. It took only ten years for the Torres design for guitars to be almost universally adopted because of the improvement it makes in tonal quality.", "question": "Which one of the following most accurately describes the relationship between Jane's argument and Mark's argument?", "answers": "['Mark and Jane use similar techniques to argue for different conclusions.', \"Mark's argument and Jane's argument are based on conflicting suppositions.\", \"Mark's argument shows how a weakness in Jane's argument can be overcome\", \"Mark's argument has a premise in common with Jane's argument.\"]", "label": 1 }, { "id": "train_4360", "context": "In the next decade, a large percentage of municipalities throughout the country will experience power failures if they do not modernize their electrical substations. The vast majority of substations in the country already handle currents between 60% and 80% of their maximum capacity, and currents are expected to rise, perhaps by as much as a factor of 3 in the next decade, because of population increase and the increased demand from both industry and electronics in individual homes.", "question": "Which of the following is an assumption of the above argument?", "answers": "['An electrical substation fails when the current it handles rises above its maximum capacity.', 'Ten years from now, the average household will own a greater number of electronic devices than does the average household today.', 'A modernized electric substation would be able to handle at least three times the amount of current as does a current substation.', \"Many electrical substations in operation today are old, and have several aged components that could break down, leading to power failures, even if current levels don't increase substantially.\"]", "label": 0 }, { "id": "train_4361", "context": "Current maps showing the North American regions where different types of garden plants will flourish are based on weather data gathered 60 years ago from a few hundred primitive weather stations. New maps are now being compiled using computerized data from several thousand modern weather stations and input from home gardeners across North America. These maps will be far more useful.", "question": "Each of the following, if true, helps to support the claim that the new maps will be more useful EXCEPT:", "answers": "['Home gardeners can provide information on plant flourishing not available from weather stations.', 'Weather patterns have changed in the past 60 years.', 'Weather conditions are the most important factor in determining where plants will grow.', 'Some of the weather stations currently in use are more than 60 years old.']", "label": 3 }, { "id": "train_4362", "context": "Certain items -- those with that hard-to-define quality called exclusivity -- have the odd property, when they become available for sale, of selling rapidly even though they are extremely expensive. In fact, trying to sell such an item fast by asking too low a price is a serious error, since it calls into question the very thing -- exclusivity -- that is supposed to be the item' s chief appeal. Ttherefore, given that a price that will prove to be right is virtually impossible for the seller to gauge in advance, the seller should make sure that any error in the initial asking price is in the direction of setting the price too high.", "question": "The argument recommends a certain pricing strategy on the grounds that", "answers": "['the error associated with this strategy, unlike the error associated with the rejected alternative, is likely to go unnoticed', 'this strategy has all of the advantages of the rejected alternative, but fewer of its disadvantages', 'experience has proven this strategy to be superior, even though the reasons for this superiority elude analysis', 'this strategy lacks a counterproductive feature of the rejected alternative']", "label": 3 }, { "id": "train_4363", "context": "Historian: There is no direct evidence that timber was traded between the ancient nations of Poran and Nayal, but the fact that a law setting tariffs on timber imports from Poran was enacted during the third Nayalese dynasty does suggest that during that period a timber trade was conducted. Critic: Your reasoning is flawed. During its third dynasty, Nayal may well have imported timber from Poran, but certainly on today' s statute books there remain many laws regulating activities that were once common but in which people no longer engage.", "question": "The critic's response to the historian's reasoning does which one of the following?", "answers": "['It distinguishes between what has been established as a certainty and what has been established as a possibility.', \"It identifies a general principle that the historian's reasoning violates.\", 'It establishes explicit criteria that must be used in evaluating indirect evidence.', 'It implies an analogy between the present and the past.']", "label": 3 }, { "id": "train_4364", "context": "Sarah: When commercial fishing boats with permits to fish for certain species accidentally catch a type of fish for which they have no permit, the latter must be thrown back. This is a very wasteful practice because many, if not most, of the rejected fish do not survive. Fishing permits should ttherefore be altered so that fishers can keep fish caught accidentally. Amar: Making it legal to keep those fish would probably lead to a lot more \"accidents. ", "question": "The technique Amar uses in responding to Sarah's argument is to", "answers": "[\"contend that Sarah's recommendation has an important negative consequence\", 'maintain that Sarah overlooks important lessons from past policies', \"allude to a factor that supposedly strengthens the case for Sarah's recommendation\", 'point out that Sarah used a crucial term in two distinct senses']", "label": 0 }, { "id": "train_4365", "context": "Several industries have recently switched at least partly from older technologies powered by fossil fuels to new technologies powered by electricity. It is thus evident that less fossil fuel is being used as a result of the operations of these industries than would have been used if these industries had retained their older technologies.", "question": "Which of the following, if true, most strengthens the argument above?", "answers": "['Less fossil fuel was used to manufacture the machinery employed in the new technologies than was originally used to manufacture the machinery employed in the older technologies.', 'Some of the industries that have switched at least partly to the new technologies still use primarily technologies that are powered by fossil fuels.', 'The amount of fossil fuel used to generate the electricity needed to power the new technologies is less than the amount that would have been used to power the older technologies.', 'Many of the industries that have switched at least partly to the new technologies have increased their output.']", "label": 2 }, { "id": "train_4366", "context": "There were several early attempts to forge a reconciliation between Shintoism and Buddhism based on mutual respect among their adherents. The evidence for this includes extant sculptures depicting Shinto gods wearing Buddhist vestments.", "question": "Which one of the following is an assumption on which the argument depends?", "answers": "['No sculptures that have not survived depicted Shinto gods wearing Buddhist vestments.', 'Shintoism did not originate as a sect of Buddhism.', 'Most sculptures contemporaneous with the sculptures mentioned were intended to have religious significance.', 'The depiction of Shinto gods wearing Buddhist vestments was not intended to represent the triumph of Shintoism over Buddhism.']", "label": 3 }, { "id": "train_4367", "context": "Neuroscientist: Memory evolved to help animals react appropriately to situations they encounter by drawing on the past experience of similar situations. But this does not require that animals perfectly recall every detail of all their experiences. Instead, to function well, memory should generalize from past experiences that are similar to the current one.", "question": "The neuroscientist's statements, if true, most strongly support which of the following conclusions?", "answers": "['Generalizing from past experiences requires clear memories of most if not all the details of those experiences.', 'Recalling every detail of all past experiences would be incompatible with any ability to generalize from those experiences.', 'Animals can often react more appropriately than they otherwise would to situations they encounter if they draw on generalizations from past experiences of similar situations.', 'At least some animals perfectly recall every detail of at least some past experiences.']", "label": 2 }, { "id": "train_4368", "context": "Reviewer: Although finalism -- the view that there are purposes in nature -- has been universally rejected, this book launches another attack on that view. Its arguments are based on a complete misunderstanding of the operation of pure chance in nature and so it fails as a critique of finalism. Finalism, ttherefore, is clearly more plausible than people have thought.", "question": "Which one of the following is most closely parallel in its flawed reasoning to the flawed reasoning in the reviewer's argument?", "answers": "['Few botanists deny that a recently discovered fern is the same species represented in ancient fossils. The botanists who first discovered this specimen deny that it is the same species as the ancient one, but they have spent little time studying the specimen. Ttherefore, the specimen is likely to be the same species represented in the ancient fossils.', 'No literary historian still believes the claim that a single author wrote every word of this collection of works. Evidence on which that claim is based can be shown to be false by manuscript dating techniques, but these dating results have not been cited before. Ttherefore, it is quite likely that a single author did write every word of this collection of works.', 'Bicycle engineers no longer believe that aluminum is as good a frame material as titanium. An engineer at Ace Bicycles has argued that aluminum is not as good as titanium because it breaks under pressure. But he is confused about the kind of pressure exerted on bicycles. Ttherefore, the claim that aluminum is as good a frame material as titanium makes more sense than bicycle engineers believe.', \"Experts agree that red wines from France's Bordeaux region are the best in the world, and five are on this year's list of the world's top ten red wines. However, the best currently available Bordeaux wines are more than one year old, whereas other countries' best are not. Ttherefore, it is false that Bordeaux red wines are the best in the world this year.\"]", "label": 2 }, { "id": "train_4369", "context": "Teacher: Journalists who conceal the identity of the sources they quote stake their professional reputations on what may be called the logic of anecdotes. This is so because the statements reported by such journalists are dissociated from the precise circumstances in which they were made and thus will be accepted for publication only if the statements are high in plausibility or originality or interest to a given audience -- precisely the properties of a good anecdote. Student: But what you are saying, then, is that the journalist need not bother with sources in the first place. Surely, any reasonably resourceful journalist can invent plausible, original, or interesting stories faster than they can be obtained from unidentified sources.", "question": "The student's response contains which one of the following reasoning flaws?", "answers": "['confusing a marginal journalistic practice with the primary work done by journalists', 'ignoring the possibility that the teacher regards as a prerequisite for the publication of an unattributed statement that the statement have actually been made', \"judging the merits of the teacher's position solely by the most extreme case to which the position applies\", 'confusing the characteristics of reported statements with the characteristics of the situations in which the statements were made']", "label": 1 }, { "id": "train_4370", "context": "Criminologist: Those who propose a rule mandating a life sentence for any criminal who has multiple convictions for serious crimes argue that it would be a welcome crackdown on career criminals. In reality, however, few repeat offenders are convicted of anything other than minor violations.", "question": "The criminologist's statements, if true, most strongly support which one of the following?", "answers": "['The sentencing of most repeat offenders would be unaffected by the proposed rule if it became law.', 'People who have never been convicted of minor violations are unlikely to become career criminals.', 'Many first-time offenders are convicted of serious crimes as well as minor violations.', 'Most people who have committed serious crimes are not convicted of anything other than minor violations.']", "label": 0 }, { "id": "train_4371", "context": "Even if a crime that has been committed by computer is discovered and reported, the odds of being both arrested and convicted greatly favor the criminal.", "question": "Each of the following, if true, supports the claim above EXCEPT:", "answers": "['The priorities of local police departments, under whose jurisdiction most computer crime falls, are weighted toward visible street crime that communities perceive as threatening.', 'Computer criminals have rarely been sentenced to serve time in prison, because prisons are overcrowded with violent criminals and drug offenders.', 'The preparation of computer-fraud cases takes much more time than is required for average fraud cases, and the productivity of prosecutors is evaluated by the number of good cases made.', 'The many police officers who are untrained in computers often inadvertently destroy the physical evidence of computer crime.']", "label": 1 }, { "id": "train_4372", "context": "Jane: Television programs and movies that depict violence among teenagers are extremely popular. Given how influential these media are, we have good reason to believe that these depictions cause young people to engage in violent behavior. Hence, depictions of violence among teenagers should be prohibited from movies and television programs, if only in those programs and movies promoted to young audiences. Maurice: But you are recommending nothing short of censorship! Besides which, your claim that television and movie depictions of violence cause violence is mistaken: violence among young people predates movies and television by centuries.", "question": "Which one of the following, if true, most strengthens Jane's argument?", "answers": "['Many adolescents who engage in violent behavior had already displayed such behavior before they were exposed violence in movies.', \"The movies that have been shown to have the most influence on young people's behavior are those that are promoted to young audiences.\", 'The people who make the most profits in the movie and television industry are those who can successfully promote their work to both young and old audiences.', 'The most violent characters depicted in movies and on television programs are adult characters which are portrayed by adult actors.']", "label": 1 }, { "id": "train_4373", "context": "Etiquette firmly opposes both obscene and malicious talk, but this does not imply that speech needs to be restricted by law. Etiquette does not necessarily even oppose the expression of offensive ideas. Rather, it dictates that there are situations in which the expression of potentially offensive, disturbing, or controversial ideas is inappropriate and that, where appropriate, the expression and discussion of such ideas is to be done in a civil manner.", "question": "Which one of the following judgments most closely corresponds to the principles of etiquette stated above?", "answers": "['The journalists at a news conference should not ask a politician potentially embarrassing questions about a controversial policy issue.', 'The moderator of a panel discussion of a divisive moral issue should not allow participants to engage in name-calling.', 'Neighbors should not be gruff or unfriendly to one another when they meet on the street.', 'When prosecutors elicit testimony from a cooperative witness they should do so without intensive questioning.']", "label": 1 }, { "id": "train_4374", "context": "Purebred dogs are prone to genetically determined abnormalities. Although such abnormalities often can be corrected by surgery, the cost can reach several thousand dollars. Since nonpurebred dogs rarely suffer from genetically determined abnormalities, potential dog owners who want to reduce the risk of incurring costly medical bills for their pets would be well advised to choose nonpurebred dogs.", "question": "Which one of the following if true, most seriously weakens the argument?", "answers": "['All dogs, whether purebred or nonpurebred, are subject to the same common nongenetically determined diseases.', 'A dog that does not have genetically determined abnormalities may nevertheless have offspring with such abnormalities.', 'The purchase price of nonpurebred dogs tends to be lower than the purchase price of purebred dogs.', \"Most genetically determined abnormalities in dogs do not seriously affect a dog's general well-being.\"]", "label": 3 }, { "id": "train_4375", "context": "Because migrant workers are typically not hired by any one employer for longer than a single season, migrant workers can legally be paid less than the minimum hourly wage that the government requires employers to pay all their permanent employees. Yet most migrant workers work long hours each day for eleven or twelve months a year and thus are as much full-time workers as are people hired on a year-round basis. Ttherefore, the law should require that migrant workers be paid the same minimum hourly wage that other full-time workers must be paid.", "question": "The pattern of reasoning displayed above most closely parallels that displayed in which one of the following arguments?", "answers": "['Because food additives are not considered drugs, they have not had to meet the safety standards the government applies to drugs. But food additives can be as dangerous as drugs. Ttherefore, food additives should also be subject to safety regulations as stringent as those covering drugs.', 'Because some countries regulate gun sales more strictly than do other countries, some people can readily purchase a gun, whereas others cannot. Ttherefore, all countries should cooperate in developing a uniform international policy regarding gun sales.', 'Because it is a democratic principle that laws should have the consent of those affected by them, liquor laws should be formulated not by politicians but by club and restaurant owners, since such laws directly affect the profitability of their businesses.', 'Because many rural areas have few restrictions on development, housing estates in such areas have been built where no adequate supply of safe drinking water could be ensured. Thus, rural areas should adopt building codes more like those large cities have.']", "label": 0 }, { "id": "train_4376", "context": "Columnist: The managers of some companies routinely donate a certain percentage of their companies' profits each year to charity. Although this practice may seem totally justified and even admirable, it is not. After all, corporate profits are not the property of the managers, but of the companies' owners. The legendary Robin Hood may have stolen from the rich to give to the poor, but he was nevertheless stealing.", "question": "Which one of the following, if true, most weakens the analogy used in the argument?", "answers": "['The profits that a company makes in a given year are, in part, returned to the owners of the company.', \"Managers who routinely donate a certain percentage of corporate profits to charity do so with the owners' tacit consent.\", 'Any charity that accepts corporate donations needs to be able to account for how that money is spent.', 'Company managers often donate part of their own income to charities or other philanthropic organizations.']", "label": 1 }, { "id": "train_4377", "context": "The Aquipos toucan is able to use its long beak to reach inside apertures in a tree's bark and feed on the insects that dwell within and that are the toucan's main food source. Several noted ornithologists have speculated that because the toucan's beak is curved in a manner similar to the openings in the bark, the beak evolved so that the toucan can best take advantage of the insects lying deep inside.", "question": "Which of the following, if true, argues most strongly against the ornithologists' speculation?", "answers": "['The Aquipos toucan is also able to use its beak to break bark, thereby accessing insects that dwell in the trunk.', 'Many insects crawl on the bark of the tree and a toucan can sometimes become satiated without having to access insects living beneath the bark.', 'The tree holes in which the Aquipos toucan is able to access insects have a characteristic shape that differs only very slightly from tree to tree.', \"Not all trees in the Aquipos toucan's environment house sufficient quantities of insects to satisfy the caloric needs of the toucan.\"]", "label": 0 }, { "id": "train_4378", "context": "Historically, monetary systems have developed only in population centers with marketplaces. Through the fourth century B. C. , Mesopotamian cities engaged in trade, but had never had marketplaces. By that period, however, Greek cities all had marketplaces, or agorae. The Greek cities' agorae were centrally located and goods were traded there either for money or for commodities.", "question": "If all of the statements in the passage are true, then which one of the following must also be true?", "answers": "['In the fourth century B. C. , the Greeks and the Mesopotamians traded with each other.', 'The Mesopotamian cities of the fourth century B. C. did not have monetary systems.', 'The development of monetary systems has historically led to the development of marketplaces.', 'In the fourth century B. C. , Greek cities were the only population centers with monetary systems.']", "label": 1 }, { "id": "train_4379", "context": "The journalistic practice of fabricating remarks after an interview and printing them within quotation marks, as if they were the interviewee' s own words, has been decried as a form of unfair misrepresentation. However, people' s actual spoken remarks rarely convey their ideas as clearly as does a distillation of those ideas crafted, after an interview, by a skilled writer. Ttherefore, since this practice avoids the more serious misrepresentation that would occur if people' s exact words were quoted but their ideas only partially expressed, it is entirely defensible.", "question": "Which one of the following is a questionable technique used in the argument?", "answers": "['claiming that the prestige of a profession provides ample grounds for dismissing criticisms of that profession', 'answering an exaggerated charge by undermining the personal authority of those who made that charge', 'concluding that a practice is right on the grounds that it is necessary', 'offering as an adequate defense of a practice an observation that discredits only one of several possible alternatives to that practice']", "label": 3 }, { "id": "train_4380", "context": "Linguist: In English, the past is described as \"behind\" and the future \"ahead, \" whereas in Aymara the past is \"ahead\" and the future \"behind. \" Research indicates that English speakers sway backward when discussing the past and forward when discussing the future. Conversely, Aymara speakers gesture forward with their hands when discussing the past and backward when discussing the future. These bodily movements, ttherefore, suggest that the language one speaks affects how one mentally visualizes time.", "question": "The linguist's reasoning depends on assuming which of the following?", "answers": "['The researchers also examined the movements of at least some speakers of languages other than English and Aymara discussing the past and the future.', 'At least some Aymara speakers sway forward when discussing the past and backward when discussing the future.', 'Most people mentally visualize time as running either forward or backward.', 'How people move when discussing the future correlates to some extent with how they mentally visualize time.']", "label": 3 }, { "id": "train_4381", "context": "Observatory director: Some say that funding the megatelescope will benefit only the astronomers who will work with it. This dangerous point of view, applied to the work of Maxwell, Newton, or Einstein, would have stifled their research and deprived the world of beneficial applications, such as the development of radio, that followed from that research.", "question": "If the statements above are put forward as an argument in favor of development of the megatelescope, which one of the following is the strongest criticism of that argument?", "answers": "['It does not identify those opposed to development of the megatelescope.', 'It launches a personal attack on opponents of the megatelescope by accusing them of having a dangerous point of view.', 'It does not distinguish between the economic and the intellectual senses of \"benefit. \"', 'It does not show that the proposed megatelescope research is worthy of comparison with that of eminent scientists in its potential for applications.']", "label": 3 }, { "id": "train_4382", "context": "Editorial: Regulations recently imposed by the government of Risemia call for unprecedented reductions in the amounts of pollutants manufacturers are allowed to discharge into the environment. It will take costly new pollution control equipment requiring expensive maintenance to comply with these regulations. Resultant price increases for Risemian manufactured goods will lead to the loss of some export markets. Clearly, ttherefore, annual exports of Risemian manufactured goods will in the future occur at diminished levels.", "question": "Which of the following, if true, most seriously weakens the argument in the editorial?", "answers": "['Savings from utilizing the chemicals captured by the pollution control equipment will remain far below the cost of maintaining the equipment.', 'The need to comply with the new regulations will stimulate the development within Risemia of new pollution control equipment for which a strong worldwide demand is likely to emerge.', \"The stockholders of most of Risemia's manufacturing corporations exert substantial pressure on the corporations to comply with environmental laws.\", 'The proposed regulations include a schedule of fines for noncompliance that escalate steeply in cases of repeated noncompliance.']", "label": 1 }, { "id": "train_4383", "context": "Once children begin to read they acquire new vocabulary most naturally as the indirect result of reading difficult, challenging material. The major alternative -- direct studying of new vocabulary items -- is less natural; most people are not so inclined. From age six to age thirteen, children learn thousands of new words a year, mostly through reading; direct studying accounts for less than one-tenth of all new words learned. So it is bound to be inefficient for young students to learn new vocabulary by direct studying.", "question": "Which one of the following, if true, most seriously weakens the argument?", "answers": "['Reading difficult material sometimes leaves one confused as to the meaning of certain words.', 'Adults who habitually read difficult material tend not to learn many new words from doing so.', 'Experts disagree on the estimates of how many words children typically learn from age six to age thirteen.', 'Children from age six to age thirteen spend vastly much more time reading than they spend directly studying new vocabulary.']", "label": 3 }, { "id": "train_4384", "context": "In a survey of consumers in an Eastern European nation, respondents were asked two questions about each of 400 famous Western brands: whether or not they recognized the brand name and whether or not they thought the products bearing that name were of high quality. The results of the survey were a rating and corresponding rank order for each brand based on recognition, and a second rating-plus-ranking based on approval. The brands ranked in the top 27 for recognition were those actually available in that nation. The approval ratings of these 27 brands often differed sharply from their recognition ratings. By contrast, most of the other brands had ratings, and thus rankings, that were essentially the same for recognition as for approval.", "question": "Which one of the following, if each is a principle about consumer surveys, is violated by the survey described ?", "answers": "['Never ask a question that is likely to generate a large variety of responses that are difficult to group into a manageable number of categories.', 'It is best to ask questions that a respondent can answer without fear of having gotten the answer wrong.', 'Never ask all respondents a question that respondents cannot answer without giving up their anonymity.', 'Never ask all respondents a question if it cannot reasonably be answered by respondents who make a particular response to another question in the same survey.']", "label": 3 }, { "id": "train_4385", "context": "The tiny hummingbird weighs little, but its egg is 15 percent of the adult hummingbird' s weight. The volume and weight of an adult goose are much greater than those of a hummingbird, but a goose' s egg is only about 4 percent of its own weight. An adult ostrich, much larger and heavier than a goose, lays an egg that is only 1. 6 percent of its own weight.", "question": "Which one of the following propositions is best illustrated by the statements above?", "answers": "['The eggs of different bird species vary widely in their ratio of volume to weight.', 'The smaller and lighter the average adult members of a bird species are, the larger and heavier the eggs of that species are.', 'The ratio of egg weight of a species to body weight of an adult member of that species is smaller for larger birds than for smaller ones.', \"The size of birds' eggs varies greatly from species to species but has little effect on the volume and weight of the adult bird.\"]", "label": 2 }, { "id": "train_4386", "context": "Editorial in Golbindian Newspaper: For almost three months, opposition parties have been mounting daily street demonstrations in the capital in an effort to pressure the ruling party into calling an election. Though the demonstrations were well attended at first, attendance has declined steadily in recent weeks. However, the decline in attendance does not indicate that popular support for the opposition' s demands is dropping, since", "question": "Which of the following most logically completes the editorial below?", "answers": "['A recent sharp decrease in unemployment has led to increased popular support for the government.', 'The state-controlled media have ceased any mention of the demonstrations, leaving many citizens outside the capital with no way of knowing that demonstrations continue.', 'There have not recently been any antigovernment demonstrations in cities other than the capital.', \"No foreign governments have expressed any support for the opposition's demands.\"]", "label": 1 }, { "id": "train_4387", "context": "Dr. Khan: Professor Burns recognizes that recent observations fail to confirm earlier ones that apparently showed a comet reservoir far out in our solar system. She claims this nonconfirmation is enough to show that the earlier observations are incorrect. But the recent observations occurred under poor conditions.", "question": "Which one of the following is most supported by Dr. Khan's statements?", "answers": "[\"Professor Burns's claim about the implications of the recent observations is incorrect.\", 'The poor conditions present during recent observations render them worthless.', 'The recent observations, even if they had been made under good conditions, would not have been enough to suggest that the earlier ones are incorrect.', \"Contrary to Professor Burns's view, the recent observations confirm the earlier ones.\"]", "label": 0 }, { "id": "train_4388", "context": "YXK is currently the television network with the highest overall number of viewers. Among YXK' s programs, Bliss has the highest numbers of viewers. So Bliss currently has more viewers than any other program on television.", "question": "The flawed reasoning exhibited by the argument above is most similar to that exhibited by which one of the following?", "answers": "['Soccer players suffer more leg injuries, on average, than any other athletes at this university. Linda Wilson has suffered more leg injuries than any other soccer player at this university. Thus, Linda Wilson is the athlete at this university who has suffered the most leg injuries.', \"The Olson Motor Company manufactures the three best-selling automobile models in the country. The Decade is the Olson Motor Company's best-selling model. Thus, the Decade is the best-selling model in the country.\", 'Falling Fast is the film that is currently earning the most at the box office in the country. The most successful film in the country is typically the one that is showing in the most theaters. So Falling Fast is probably the film that is currently showing in the most theaters.', 'In this city the highest-paid police officer earns more than the highest-paid firefighter, and the lowest-paid police officer earns more than the lowest-paid firefighter. So in this city police officers earn more, on average, than firefighters do.']", "label": 0 }, { "id": "train_4389", "context": "Mary: Computers will make more information available to ordinary people than was ever available before, thus making it easier for them to acquire knowledge without consulting experts. Joyce: As more knowledge became available in previous centuries, the need for specialists to synthesize and explain it to nonspecialists increased. So computers will probably create a greater dependency on experts.", "question": "The dialogue most strongly supports the claim that Mary and Joyce disagree with each other about whether", "answers": "['dependency on computers will increase with the increase of knowledge', 'computers will make more information available to ordinary people', 'computers will increase the need for ordinary people seeking knowledge to turn to experts', 'synthesizing knowledge and explaining it to ordinary people can be accomplished only by computer experts']", "label": 2 }, { "id": "train_4390", "context": "Expert: A group of researchers claims to have shown that for an antenna to work equally well at all frequencies, it must be symmetrical in shape and have what is known as a fractal structure. Yet the new antenna developed by these researchers, which satisfies both of these criteria, in fact works better at frequencies below 250 megahertz than at frequencies above 250 megahertz. Hence, their claim is incorrect.", "question": "The reasoning in the expert's argument is flawed because the argument", "answers": "['takes for granted that there are only two possible alternatives, either below or above 250 megahertz', 'fails to provide a definition of the technical term \"fractal\"', 'contradicts itself by denying in its conclusion the claim of scientific authorities that it relies on in its premises', 'interprets an assertion that certain conditions are necessary as asserting that those conditions are sufficient']", "label": 3 }, { "id": "train_4391", "context": "Curator: A magazine recently ran a very misleading story on the reaction of local residents to our controversial art exhibit. They quoted the responses of three residents, all of whom expressed a sense of moral outrage. These quotations were intended to suggest that most local residents oppose the exhibit; the story failed to mention, however, the fact that the three residents are all close friends.", "question": "Which one of the following principles most helps to justify the curator's argumentation?", "answers": "['It is misleading to present the opinions of people with no special expertise on a subject as though they were experts.', 'It is misleading to present the opinions of a few people as evidence of what the majority thinks unless the opinions they express are widely held.', 'It is misleading to present the opinions of a potentially nonrepresentative sample of people as if they represent public opinion.', 'It is misleading to present the opinions of people on only one side of an issue when the population is likely to be evenly divided on that issue.']", "label": 2 }, { "id": "train_4392", "context": "The use of growth-promoting antibiotics in hog farming can weaken their effectiveness in treating humans because such use can spread resistance to those antibiotics among microorganisms. But now the Smee Company, one of the largest pork marketers, may stop buying pork raised on feed containing these antibiotics. Smee has 60 percent of the pork market, and farmers who sell to Smee would certainly stop using antibiotics in order to avoid jeopardizing their sales. So if Smee makes this change, it will probably significantly slow the decline in antibiotics' effectiveness for humans.", "question": "Which of the following, if true, would most strengthen the argument above?", "answers": "['A phaseout of use of antibiotics for hogs in one country reduced usage by over 50 percent over five years.', \"If Smee stops buying pork raised with antibiotics, the firm's costs will probably increase.\", 'Other major pork marketers will probably stop buying pork raised on feed containing growth-promoting antibiotics if Smee no longer buys such pork.', 'The decline in hog growth due to discontinuation of antibiotics can be offset by improved hygiene.']", "label": 2 }, { "id": "train_4393", "context": "Dietary researcher: A recent study reports that laboratory animals that were fed reduced-calorie diets lived longer than laboratory animals whose caloric intake was not reduced. In response, some doctors are advocating reduced-calorie diets, in the belief that North Americans' life spans can thereby be extended. However, this conclusion is not supported. Laboratory animals tend to eat much more than animals in their natural habitats, which leads to their having a shorter life expectancy. Restricting their diets merely brings their caloric intake back to natural, optimal levels and reinstates their normal life spans.", "question": "Which one of the following, if true, would most weaken the dietary researcher's argument?", "answers": "['North Americans with high-fat, low-calorie diets generally have a shorter life expectancy than North Americans with low-fat, low-calorie diets.', 'Not all scientific results that have important implications for human health are based on studies of laboratory animals.', 'North Americans, on average, consume a higher number of calories than the optimal number of calories for a human diet.', 'Some North Americans who follow reduced- calorie diets are long-lived.']", "label": 2 }, { "id": "train_4394", "context": "Food co-ops are a type of consumer cooperative. Consumer cooperatives offer the same products as other stores but usually more cheaply. It is ttherefore more economical to shop at a food co-op than at a supermarket.", "question": "Which one of the following is most appropriate as an analogy demonstrating that the reasoning in the argument above is flawed?", "answers": "['By that line of reasoning, we could conclude that it is better to buy frozen vegetables than fresh vegetables, since fresh vegetables are more expensive than frozen vegetables and spoil more quickly.', 'By that line of reasoning, we could conclude that a person who rides a bicycle causes more pollution per mile traveled than one who rides a public bus, since bicycling is a private means of transportation and private means of transportation tend to generate more pollution per mile traveled than do public means.', \"By that line of reasoning, we could conclude that the best way to lose weight is to increase one's consumption of artificially sweetened foods, since artificially sweetened foods have fewer calories than foods sweetened with sugar, and excessive calorie intake contributes to weight gain.\", 'By that line of reasoning, we could conclude that more people must be shopping at health food stores than ever before, since people tend to choose healthful food over unhealthful food as long as the healthful food tastes at least as good, and healthful food today is better tasting than ever.']", "label": 1 }, { "id": "train_4395", "context": "Editorialist: In all cultures, it is almost universally accepted that one has a moral duty to prevent members of one' s family from being harmed. Thus, few would deny that if a person is known by the person' s parents to be falsely accused of a crime, it would be morally right for the parents to hide the accused from the police. Hence, it is also likely to be widely accepted that it is sometimes morally right to obstruct the police in their work.", "question": "The reasoning in the editorialist's argument is most vulnerable to criticism on the grounds that this argument", "answers": "['presumes, without providing justification, that allowing the police to arrest an innocent person assists rather than obstructs justice', 'takes for granted that there is no moral obligation to obey the law', \"takes for granted that the parents mentioned in the example are not mistaken about their child's innocence\", 'fails to consider the possibility that other moral principles would be widely recognized as overriding any obligation to protect a family member from harm']", "label": 3 }, { "id": "train_4396", "context": "At any given time, approximately fifteen percent of all homes in Florida are on the market. In Texas, however, only seven percent of all homes are on the market at any given time. Ttherefore, one will have a wider selection of homes to choose from if one looks for a home in Florida rather than in Texas.", "question": "Which of the following, if true, would most strongly strengthen the argument above?", "answers": "['The total number of homes in Florida is three times greater than the total number in Texas.', 'Homes in Florida tend to be less expensive than those in Texas.', 'The cost of constructing new homes in Texas is higher than in Florida.', 'Mortgages are easier to obtain for homes in Florida than for homes in Texas.']", "label": 0 }, { "id": "train_4397", "context": "Climatologists believe they know why Earth has undergone a regular sequence of ice ages beginning around 800, 000 years ago. Calculations show that Earth' s orbit around the Sun has fluctuations that coincide with the ice-age cycles. The climatologists hypothesize that when the fluctuations occur, Earth passes through clouds of cosmic dust that enters the atmosphere; the cosmic dust thereby dims the Sun, resulting in an ice age. They concede, however, that though cosmic dust clouds are common, the clouds would have to be particularly dense in order to have this effect.", "question": "Each of the following, if true, would lend support to the climatologists' hypothesis EXCEPT:", "answers": "[\"Large bits of cosmic rock periodically enter Earth's atmosphere, raising large amounts of dust from Earth's surface.\", \"Earth's average temperature drops slightly shortly after volcanic eruptions spew large amounts of dust into Earth's atmosphere.\", 'Earth did not pass through clouds of cosmic dust earlier than 800, 000 years ago.', 'Two large asteroids collided 800, 000 years ago, producing a tremendous amount of dense cosmic dust that continues to orbit the Sun.']", "label": 0 }, { "id": "train_4398", "context": "The current move to patent computer programs is a move in the wrong direction and should be stopped. The patent system was originally designed solely to protect small-time inventors from exploitation, not to give large corporations control over a methodology. Any computer program is merely the implementation of a methodology.", "question": "Which one of the following is an assumption on which the argument depends?", "answers": "['The issue of whether or not to patent computer programs presents the patent system with problems that have never before arisen.', 'Computer programs should be developed not only by large corporations but by small-time inventors as well.', 'Large corporations should not hold patents for implementations of methodologies.', 'Implementing a methodology always requires less creative effort than does true invention.']", "label": 2 }, { "id": "train_4399", "context": "Consumer magazine: Because front-loading washers use less water than top-loading washers, ordinary powder detergent does not dissolve readily in front-loading washers. So, to get clothes really clean in a front-loading machine you need to use a detergent formulated especially for front-loading washers, instead of ordinary powder detergent.", "question": "Which one of the following is an assumption required by the argument in the consumer magazine?", "answers": "['A laundry detergent does not get clothes really clean in a washer unless it dissolves readily in it.', 'A washing machine gets clothes really clean only with a laundry detergent specially formulated for that machine.', 'A laundry detergent formulated especially for front-loading washers dissolves more readily in them than it does in top-loading washers.', 'Washers that use more water get clothes cleaner than those that use less.']", "label": 0 }, { "id": "train_4400", "context": "Egidio' s Gym has been in operation for seven years, and offers regular weekly weight training and aerobic classes, as well as occasional programs in dance and martial arts. The gym has seen a surge of new members in the past twelve months, despite keeping the same membership fee over the past three years. Of the 450 current members, more than 250 have joined in the past year. Clearly, the membership of Egidio' s Gym has risen a significant amount because of its superior facilities.", "question": "Which of the following is an assumption on which the argument depends?", "answers": "['None of the other gyms in this town saw any increase in membership during the past year.', 'Many of the new members joined because of personal recommendations of those who were already members.', 'Dance and martial arts have become particularly popular among the general public over the past year.', \"Fewer than 250 members ended their membership at Egidio's Gym over the last year.\"]", "label": 3 }, { "id": "train_4401", "context": "Anyone believing that no individual can have an effect on society' s future will as a result feel too helpless to act to change society for the better. Thus, anyone who wants to improve society should reject the belief that its future will be determined entirely by vast historical forces that individuals are powerless to change.", "question": "Which one of the following principles, if valid, most helps to justify the argument?", "answers": "['Anyone who feels too helpless to act to change society for the better should reject the belief that its future will be determined by vast historical forces that individuals are powerless to change.', 'Each individual should act to improve society if individuals in general feel powerless in the face of vast historical forces.', \"Anyone who believes that individuals can have an effect on society's future should act to change society for the better.\", 'No one who wants to improve society should accept any belief that makes him or her feel too helpless to act to change society for the better.']", "label": 3 }, { "id": "train_4402", "context": "Even in ancient times, specialized farms (farms that grow a single type of crop or livestock) existed only where there were large commercial markets for farm products, and such markets presuppose urban populations. Ttherefore the extensive ruins in the archaeological site at Kadshim are probably the remains of a largely uninhabited ceremonial structure rather than of a densely populated city, since the land in the region of Kadshim could never have supported any farms except mixed farms, which grow a variety of crops and livestock.", "question": "Which one of the following is an error of reasoning in the argument?", "answers": "['drawing a conclusion that is simply a restatement of one of the premises on which the argument is based', 'taking the nonexistence of something as evidence that a necessary precondition for that thing also did not exist', 'taking the fact that something is true of one sample of a class of things as evidence that the same is true of the entire class of things', 'interpreting an ambiguous claim in one way in one part of the argument and in another way in another part of the argument']", "label": 1 }, { "id": "train_4403", "context": "Letter to the editor: I have never seen such flawed reasoning and distorted evidence as that which you tried to pass off as a balanced study in the article \"Speed Limits, Fatalities, and Public Policy. \" The article states that areas with lower speed limits had lower vehicle-related fatality rates than other areas. However, that will not be true for long, since vehicle-related fatality rates are rising in the areas with lower speed limits. So the evidence actually supports the view that speed limits should be increased.", "question": "The reasoning in the letter writer's argument is flawed because the argument", "answers": "['does not present any claims as evidence against the opposing viewpoint', 'bases its conclusion on findings from the same article that it is criticizing', 'fails to consider the possibility that the vehicle-related fatality rates in other areas are also rising', 'fails to consider the possibility that automobile accidents that occur at high speeds often result in fatalities']", "label": 2 }, { "id": "train_4404", "context": "It is widely believed that eating chocolate can cause acne. Indeed, many people who are susceptible to acne report that, in their own experience, eating large amounts of chocolate is invariably followed by an outbreak of that skin condition. However, it is likely that common wisdom has mistaken an effect for a cause. Several recent scientific studies indicate that hormonal changes associated with stress can cause acne and there is good evidence that people who are fond of chocolate tend to eat more chocolate when they are under stress.", "question": "Of the following, which one most accurately expresses the main point of the argument?", "answers": "['People are mistaken who insist that whenever they eat large amounts of chocolate they invariably suffer from an outbreak of acne,', 'Eating large amounts of chocolate is more likely to cause stress than it is to cause outbreaks of acne.', 'The more chocolate a person eats, the more likely that person is to experience the hormonal changes associated with stress.', 'It is less likely that eating large amounts of chocolate causes acne than that both the chocolate eating and the acne are caused by stress.']", "label": 3 }, { "id": "train_4405", "context": "From 1973 to 1976, total United States consumption of cigarettes increased 3. 4 percent, and total sales of chewing tobacco rose 18. 0 percent. During the same period, total United States population increased 5. 0 percent.", "question": "If the statements above are true, which of the following conclusions can be properly drawn?", "answers": "['A large percentage of United States smokers switched from cigarettes to chewing tobacco between 1973 and 1976.', 'Per capita consumption of cigarettes in the United States was lower in 1976 than in 1973.', 'The proportion of nonsmokers in the United States population dropped slightly between 1973 and 1976.', 'United States manufacturers of tobacco products realize a lower profit on cigarettes than on chewing tobacco.']", "label": 1 }, { "id": "train_4406", "context": " Environmental organizations want to preserve the land surrounding the Wilgrinn Wilderness Area from residential development. They plan to do this by purchasing that land from the farmers who own it. That plan is ill-conceived: if the farmers did sell their land, they would sell it to the highest bidder, and developers would outbid any other bidders. On the other hand, these farmers will never actually sell any of the land, provided that farming it remains viable . But farming will not remain viable if the farms are left unmodernized, and most of the farmers lack the financial resources modernization requires. And that is exactly why a more sensible preservation strategy would be to assist the farmers to modernize their farms to the extent needed to maintain viability.", "question": "In the argument as a whole, the two boldface proportions play which of the following roles?", "answers": "[\"The first presents a goal, strategies for achieving which are being evaluated in the argument; the second is a judgment providing a basis for the argument's advocacy of a particular strategy.\", 'The first presents a goal that the argument concludes cannot be attained; the second is a reason offered in support of that conclusion.', 'The first presents a goal that the argument concludes can be attained; the second is a judgment disputing that conclusion.', 'The first presents a goal that the argument endorses; the second presents a situation that the argument contends must be changed if that goal is to be met in the foreseeable future.']", "label": 0 }, { "id": "train_4407", "context": "Because of the lucrative but illegal trade in rhinoceros horns, a certain rhinoceros species has been hunted nearly to extinction. Ttherefore an effective way to ensure the survival of that species would be to periodically trim off the horns of all rhinoceroses, thereby eliminating the motivation for poaching.", "question": "Which one of the following is an assumption required by the argument?", "answers": "['Poachers hunt at least some immature rhinoceroses whose horns have not yet started to develop.', 'The demand for rhinoceros horns will remain constant even if the supply decreases after the periodic trimming-off of the rhinoceros horns has begun.', 'Rhinoceroses whose horns have been trimmed off are unable to defend themselves against predators .', 'At least some rhinoceroses whose horns are periodically trimmed off will be able to attract mates.']", "label": 3 }, { "id": "train_4408", "context": "Historian: Concern with achievement and power surged dramatically in the latter part of the eighteenth century, which is around the time that the Industrial Revolution began in Europe. So, it is clear that this surge in concern with achievement and power was a result of the Industrial Revolution.", "question": "The reasoning in the argument is flawed because", "answers": "['it fails to adequately take into account that the beginning of the Industrial Revolution in Europe cannot be located with any great precision', 'it ignores the fact that the Industrial Revolution did not reach its full maturity until the nineteenth century', 'increasing concern with achievement and power may very well have been a cause of the Industrial Revolution', 'it fails to consider that there was some concern with achievement and power before the Industrial Revolution']", "label": 2 }, { "id": "train_4409", "context": "In the past, the railroads in Ostronia were run as regional monopolies and operated with little regard for what customers wanted. In recent years, with improvements to the Ostronian national highway network, the railroad companies have faced heavy competition from long-distance trucking companies. But because of government subsidies that have permitted Ostronain railroad companies to operate even while incurring substantial losses, the companies continue to disregard customers' needs and desires.", "question": "If the statements above are true, which one of the following must also be true on the basis of them?", "answers": "['Without government subsidies, railroad companies in Ostronia would have to increase the prices they charge their customers.', 'In recent years, some companies in Ostronia that have had little regard for the desires of their customers have nonetheless survived.', 'If the government of Ostronia ceases to subsidize railroad companies, few of those companies will continue to operate.', 'The transportation system in Ostronia is no more efficient today than in was in the past.']", "label": 1 }, { "id": "train_4410", "context": "Advertisement: Today' s customers expect high quality. Every advance in the quality of manufactured products raises customer expectations. The company that is satisfied with the current quality of its products will soon find that its customers are not. At MegaCorp, meeting or exceeding customer expectations is our goal.", "question": "Which of the following must be true on the basis of the statements in the advertisement above?", "answers": "['If a company becomes satisfied with the quality of its products, then the quality of its products is sure to decline.', 'A company that does not correctly anticipate the expectations of its customers is certain to fail in advancing the quality of its products.', \"MegaCorp's goal is possible to meet only if continuing advances in product quality are possible.\", \"MegaCorp's competitors will succeed in attracting customers only if those competitors adopt MegaCorp's goal as their own.\"]", "label": 2 }, { "id": "train_4411", "context": "In a certain state, over 80% of the land is comprised of farms, but historically, large farm machinery has not sold well in this state. The percentage of land devoted to farms is not expected to increase. In fact, the number of farms in the state has been slowly declining over the past decade. A new manufacturer of large farm equipment is building a factory in the middle of this state, and the manufacturer' s plans for success depend on strong in-state sales of their product. Both the manufacturer and the industry analysts are expecting this manufacturer to be quite successful over the next few years.", "question": "Which of the following, if true, most helps to provide a justification for the manufacturer's and the industry analysts' optimistic expectations?", "answers": "['The percentage of food imported into the state, from other states and from other countries, has increased significantly over the past 20 years.', 'The number of models of large farm machinery this manufacturer will offer is larger than the number of models offered by most other comparable manufacturers of large farm machinery.', \"The manufacturer's new factory will be on or close to five different railroad lines, and is expected to have a beneficial impact on the struggling rail transport industry in the state.\", 'Land previously owned by small, independent farmers has been bought up and consolidated into large industrial farms, which have a much greater need of large farm equipment.']", "label": 3 }, { "id": "train_4412", "context": "One thousand people in Denmark were questioned about their views on banning cigarette advertising. The sample comprised adults who are representative of the general population, and who, ten years previously, had been questioned on the same issue. Interestingly, their opinions changed little. Results show that 31 percent are in favor of such a ban, 24 percent are against it, 38 percent are in favor, but only for certain media, and 7 percent have no opinion.", "question": "The survey results in the passage best support which one of the following conclusions?", "answers": [ "Most of Denmark's population favors some sort of ban on cigarette advertising.", "Most of Denmark's population does not smoke cigarettes.", "A minority of Denmark's population feels that banning cigarette advertising would set a bad precedent.", "People's opinions never change very much." ], "label": 0 }, { "id": "train_4413", "context": "Bank deposits are credited on the date of the transaction only when they are made before 3 P. M. Alicia knows that the bank deposit was made before 3 P. M. So, Alicia knows that the bank deposit was credited on the date of the transaction.", "question": "Which one of the following exhibits both of the logical flaws exhibited by the argument above?", "answers": "['It is clear that George knows he will be promoted to shift supervisor, because George will be promoted to shift supervisor only if Helen resigns, and George knows Helen will resign.', 'Journalists are the only ones who will be permitted to ask questions at the press conference. Since Marjorie is a journalist, she will be permitted to ask questions.', 'John believes that 4 is a prime number and that 4 is divisible by 2. Hence John believes that there is a prime number divisible by 2.', 'We know that Patrice works only on Thursday. Today is Thursday, so it follows that Patrice is working today.']", "label": 0 }, { "id": "train_4414", "context": "If the city starts requiring residents to sort the materials that they put out for recycling, then many residents will put more recyclables in with their regular garbage. This will result in more recyclables being buried in the city' s landfill. However, because of the cost of having city workers do the sorting, the sanitation department will not stay within its budget unless the sorting requirement for residents is implemented.", "question": "Which one of the following statements logically follows from the information above?", "answers": "['If the city implements the sorting requirement, the sanitation department will stay within its budget.', \"The amount of recyclables going to the city's landfill will increase if the sanitation department stays within its budget.\", \"Most of the city's residents will continue to recycle even if a sorting requirement is implemented.\", \"Implementing the sorting requirement would not cause the city's annual cost of sending garbage to its landfill to exceed its current annual cost of sorting recyclables.\"]", "label": 1 }, { "id": "train_4415", "context": "Citizen: Our legislators need to act quickly to counter the effects of the recession, especially the present level of unemployment, which is the highest ever. We urgently need a major tax cut for our upper-income citizens. There would then be a correspondingly large increase in investment that would create new jobs. If this measure is not taken, investment will not grow.", "question": "The citizen's argument depends on the assumption that", "answers": "[\"the recession in the citizen's country is the worst one in its history\", 'upper-income citizens could use the money gained from the tax cut in ways that increase investment', 'in the past tax cuts for certain groups of people have tended to create new jobs', 'the greater the tax cut given to a group of people, the more likely it is that members of that group will invest the money']", "label": 1 }, { "id": "train_4416", "context": "In older commercial airplanes, the design of the control panel allows any changes in flight controls made by one member of the flight crew to be immediately viewed by the other crew members. In recently manufactured aircraft, however, a crew member' s flight control changes are harder to observe, thereby eliminating a routine means for performing valuable cross-checks. As a result, the flight crews operating recently manufactured airplanes must inform each other verbally about flight control changes much more frequently.", "question": "The statements above, if true, most strongly support which one of the following?", "answers": "[\"How frequently an airplane's flight crew members will inform each other verbally about flight control changes depends in large part on how long it takes to perform those changes.\", 'How often flight crew members must share information verbally about flight control changes depends in part on what other means for performing cross-checks are available to the crew.', 'The flight crew members operating a recently manufactured airplane cannot observe the flight control changes made by other crew members by viewing the control panel.', 'In recently manufactured aircraft, the most valuable means available for performing cross-checks involves frequent verbal exchanges of information among the flight crew members.']", "label": 1 }, { "id": "train_4417", "context": "A county airport, designed to serve the needs of private aircraft owners, planned to cover its operating expenses in part by charging user fees to private aircraft using the airport. The airport was unable to pay its operating expenses because the revenue from user fees was lower than expected.", "question": "If the statements above are true, which one of the following must also be true?", "answers": "['Private aircraft owners were unwilling to pay the user fees charged at the airport.', \"The number of owners of private aircraft who use the county's airport facilities will not change appreciably in the future.\", \"Most of the county's citizens live a convenient distance from one or another airport now offering commercial airline services.\", \"The airport's operating expenses were greater than the revenue raised from sources other than the airport user fees for private planes.\"]", "label": 3 }, { "id": "train_4418", "context": "Some psychologists claim that, in theory, the best way to understand another person would be through deep empathy, whereby one would gain a direct and complete grasp of that person' s motivations. But suppose they are right; then there would be no way at all to achieve understanding, since it is psychologically impossible to gain a direct and complete grasp of another person' s motivations. But obviously one can understand other people; thus these psychologists are wrong.", "question": "The argument is most vulnerable to the criticism that it", "answers": "['fails to adequately define the key phrase \"deep empathy\"', 'assumes something that it later denies, resulting in a contradiction', 'accepts a claim on mere authority, without requiring sufficient justification', 'confuses a theoretically best way of accomplishing something with the only way of accomplishing it']", "label": 3 }, { "id": "train_4419", "context": "Keith: Compliance with new government regulations requiring the installation of smoke alarms and sprinkler systems in all theaters and arenas will cost the entertainment industry $25 billion annually. Consequently, jobs will be lost and profits diminished. Ttherefore, these regulations will harm the country's economy. Laura: The $25 billion spent by some businesses will be revenue for others. Jobs and profits will be gained as well as lost.", "question": "Laura responds to Keith by", "answers": [ "suggesting that Keith's argument overlooks a mitigating consequence", "demonstrating that Keith's conclusion is based on evidence that is not relevant to the issue at hand", "agreeing with the main conclusion of Keith's argument but construing that conclusion as grounds for optimism rather than for pessimism", "challenging the plausibility of the evidence that serves as the basis for Keith's argument" ], "label": 0 }, { "id": "train_4420", "context": "Scientist: Laboratory animals have access to ample food, and they get relatively little exercise. These factors can skew the results of research using animals, since such studies often rely on the assumption that the animal subjects are healthy. For instance, animal studies that purport to show that extreme caloric restriction can extend lifespans take for granted that their subjects were not overfed to begin with.", "question": "The scientist's argument requires assuming which one of the following?", "answers": "['Laboratory animals are healthy if they are fed a carefully restricted diet and get plenty of exercise.', 'Laboratory conditions that provide animals with ample food but relatively little exercise can be unhealthy for the animals.', 'Some animal studies take into consideration the differences between the living conditions of laboratory animals and those of other animals.', 'When provided with unlimited food over a long period of time, animals show little day-to-day variation in their eating habits.']", "label": 1 }, { "id": "train_4421", "context": "A certain experimental fungicide causes no harm to garden plants, though only if it is diluted at least to ten parts water to one part fungicide. Moreover, this fungicide is known to be so effective against powdery mildew that it has the capacity to eliminate it completely from rose plants. Thus this fungicide, as long as it is sufficiently diluted, provides a means of eliminating powdery mildew from rose plants that involves no risk of harming the plants.", "question": "Which one of the following is an assumption on which the argument depends?", "answers": "['The effectiveness of the fungicide does not depend on its being more concentrated than one part in ten parts of water.', 'When the fungicide is sufficiently diluted, it does not present any risk of harm to people, animals, or beneficial insects.', 'If a fungicide is to be effective against powdery mildew on rose plants, it must eliminate the powdery mildew completely.', 'Powdery mildew is the only fungal infection that affects rose plants.']", "label": 0 }, { "id": "train_4422", "context": "Lakshmi: I know that wildflowers are threatened because of the reduced number of suitable spaces where they can grow and that people should generally leave them where they are growing. This large field, however, is full of plants of one wildflower species, and it would be all right for me to take just one plant, since there are so many. Malini: There will not be many there in the future if many people act on your principle.", "question": "Malini's criticism proceeds by", "answers": "['contending that Lakshmi is presupposing that an exception can properly be made for her but not for anyone else', 'relying on the principle that a selfish act is wrong even when it has no harmful effect', 'pointing out that apparently insignificant individual acts of a certain kind can have a large cumulative effect', \"accusing Lakshmi of improper motives instead of responding to Lakshmi's argument\"]", "label": 2 }, { "id": "train_4423", "context": "Biographer: Arnold' s belief that every offer of assistance on the part of his colleagues was a disguised attempt to make him look inadequate and that no expression of congratulations on his promotion should be taken at face value may seem irrational. In fact, this belief was a consequence of his early experiences with an admired older sister who always made fun of his ambitions and achievements. In light of this explanation, ttherefore, Arnold' s stubborn belief that his colleagues were duplicitous emerges as clearly justified.", "question": "The flawed reasoning in the biographer's argument is most similar to that in which one of the following?", "answers": "[\"As a child, Joan was severely punished whenever she played with her father's prize Siamese cat. Ttherefore, since this information makes her present belief that cats are not good pets completely understandable, that belief is justified.\", \"Emily suspected that apples are unhealthy ever since she almost choked to death while eating an apple when she was a child. Now, evidence that apples treated with certain pesticides can be health hazards shows that Emily's long held belief is fully justified.\", \"The fact that top executives generally have much larger vocabularies than do their subordinates explains why Sheldon's belief, instilled in him during his childhood, that developing a large vocabulary is the way to get to the top in the world of business is completely justified.\", 'Studies show that when usually well-behaved children become irritable, they often exhibit symptoms of viral infections the next day. The suspicion, still held by many adults, that misbehavior must always be paid for is thus both explained and justified.']", "label": 0 }, { "id": "train_4424", "context": "A recently discovered fossil, which is believed by some to come from Archaeoraptor liaoningensis, a species of dinosaur, can serve as evidence that birds evolved from dinosaurs only if the entire fossil is from a single animal. However, the fossil is a composite of bones collected from various parts of the discovery site, so it does not provide evidence that birds evolved from dinosaurs.", "question": "The conclusion drawn in the argument follows logically if which one of the following is assumed?", "answers": "['If the entire fossil is from a single animal, then it is a well-preserved specimen.', 'The only paleontologists who believe that the entire fossil is from a single animal are those who were already convinced that birds evolved from dinosaurs.', 'The fossil was stolen from the discovery site and sold by someone who cared much more about personal profit than about the accuracy of the fossil record.', 'If the fossil is a composite, then it has pieces of more than one animal.']", "label": 3 }, { "id": "train_4425", "context": "If the city builds the proposed convention center, several national professional organizations will hold conventions there. And if several large conventions are held in the city, the total number of visitors will of course increase. Tax revenues will certainly increase if the number of visitors increases. Thus, building the convention center will increase the city' s tax revenues.", "question": "The conclusion of the argument follows logically if which one of the following is assumed?", "answers": "['People who are now regular visitors to the city will continue to visit the city if the new convention center is built.', \"The city's tax revenues will not increase unless the convention center is built.\", 'If several national professional organizations hold their conventions in the convention center, those conventions will be large.', 'If the number of visitors to the city increases, then the amount of money spent by visitors will increase.']", "label": 2 }, { "id": "train_4426", "context": "A leading critic of space exploration contends that it would be wrong, given current technology, to send a group of explorers to Mars, since the explorers would be unlikely to survive the trip. But that exaggerates the risk. There would be a well-engineered backup system at every stage of the long and complicated journey. A fatal catastrophe is quite unlikely at any given stage if such a backup system is in place.", "question": "The reasoning in the argument is flawed in that the argument", "answers": "['infers that something cannot occur merely from the fact that it is unlikely to occur', 'rejects a view merely on the grounds that an inadequate argument has been made for it', 'draws a conclusion about what must be the case based on evidence about what is probably the case', 'infers that something is true of a whole merely from the fact that it is true of each of the parts']", "label": 3 }, { "id": "train_4427", "context": "Fact: Asthma, a bronchial condition, is much less common ailment than hay fever, an allergic inflammation of the nasal passages. Fact: Over 95 percent of people who have asthma also suffer from hay fever.", "question": "If the information given as facts above is true, which of the following must also be true?", "answers": "['The percentage of people suffering from hay fever who also have asthma is lower than 95 percent.', 'The number of people who have both of these ailments is greater than the number of people who have only one of them.', 'Hay fever is a prerequisite for the development of asthma.', 'Those who have neither hay fever nor asthma comprise less than 5 percent of the total population.']", "label": 0 }, { "id": "train_4428", "context": "A professor of business placed a case-study assignment for her class on her university' s computer network. She later found out that instead of reading the assignment on the computer screen, 50 out of the 70 students printed it out on paper. Thus, it is not the case that books delivered via computer will make printed books obsolete.", "question": "Which one of the following, if true, most strengthens the argument?", "answers": "['Some people get impaired vision from long periods of reading printed matter on computer screens, even if they use high quality computer screens.', 'Studies consistently show that most computer users will print reading material that is more than a few pages in length rather than read it on the computer screen.', 'Books on cassette tape have only a small fraction of the sales of printed versions of the same books, though sales of videos of books that have been turned into movies remain strong.', 'Scanning technology is very poor, causing books delivered via computer to be full of errors unless editors carefully read the scanned versions.']", "label": 1 }, { "id": "train_4429", "context": "An article claims that many medical patients have an instinctual ability to predict sudden changes in their medical status. But the evidence given is anecdotal and should not be trusted. The case is analogous to empirically disproven reports that babies are born in disproportionately high numbers during full moons. Once that rumor became popular, maternity room staff were more likely to remember busy nights with full moons than busy nights without them.", "question": "The argument requires the assumption that", "answers": "['the idea that medical patients have an instinctual ability to predict sudden changes in their medical status is not a widely held belief', 'the patients in the article were not being serious when they predicted sudden changes in their medical status', \"patients' predictions of sudden changes in their medical status are less likely to be remembered by medical staff if no such change actually occurs\", 'babies are less likely to be born during a night with a full moon than during a night without a full moon']", "label": 2 }, { "id": "train_4430", "context": "The Kiffer Forest Preserve, in the northernmost part of the Abbimac Valley, is where most of the bears in the valley reside. During the eight years that the main road through the preserve has been closed the preserve' s bear population has nearly doubled. Thus, the valley' s bear population will increase if the road is kept closed.", "question": "Which one of the following, if true, most undermines the argument?", "answers": "[\"Only some of the increase in the preserve's bear population over the past eight years is due to migration of bears from outside the Abbimac Valley.\", 'The bear population in areas of the Abbimac Valley outside the Kiffer Forest Preserve has decreased over the past eight years.', 'The bear population in the Abbimac Valley has remained about the same over the past eight years.', \"Only some of the increase in the preserve's bear population over the past eight years is due to migration of bears from other parts of the Abbimac Valley.\"]", "label": 2 }, { "id": "train_4431", "context": "If rational-choice theory is correct, then people act only in ways that they expect will benefit themselves. But this means that rational-choice theory cannot be correct, because plenty of examples exist of people acting in ways that result in no personal benefit whatsoever.", "question": "The argument above is most vulnerable to criticism on the grounds that it", "answers": "['takes for granted that people who are acting in ways that are personally beneficial expected that their actions would be personally beneficial', 'presumes, without justification, that examples of people acting in ways that are not personally beneficial greatly outnumber examples of people acting in ways that are personally beneficial', 'concludes that a theory is false merely on the grounds that the evidence for it is hypothetical', 'fails to consider that people acting in ways that result in no personal benefit may nonetheless have expected that acting in those ways would produce personal benefit']", "label": 3 }, { "id": "train_4432", "context": "Advertisement: VIVVY, a video-based foreign language course for children, was introduced seventeen years ago. Amy, Matt, and Evelyn were among the first children to use VIVVY. Now they are successful university students. So if your child uses VIVVY, you can expect him or her to become a successful university student.", "question": "Which one of the following demonstrates most effectively by parallel reasoning that the argument in the advertisement is flawed?", "answers": "['Similarly, you could conclude that Eric, Diane, and Martin are the only employees who will be laid off. After all, any employee hired within the last year can expect to be laid off, and these three employees are the only ones who were hired within the last year.', 'Similarly, you could conclude that Jesse should not expect to get food poisoning. After all, Jesse, Doris, and Christine all attended the company picnic, and only Christine has gotten food poisoning as a result.', 'Similarly, you could conclude that Ken, Norma, and Mary routinely drive faster than the speed limit. After all, if you routinely exceed the speed limit, you can expect to get a speeding ticket eventually, and these three people have gotten speeding tickets.', 'Similarly, you could conclude that you can expect to win the lottery if you carry a good-luck charm. After all, Annie, Francisco, and Sean carry good-luck charms, and these three people are lottery winners.']", "label": 3 }, { "id": "train_4433", "context": "Most hospitals have treatment options for cancer patients. All hospitals are able to treat all kinds of burn victims as well as all kinds of Neurology disorders. Ttherefore, there must be some hospitals that treat both cancer and burn victims and some hospitals that treat both cancer and Neurology disorders.", "question": "The pattern of flawed reasoning in which of the following arguments is most parallel to that in the argument above?", "answers": "['Most insects are helpful to humans. Nearly all insects have an exoskeleton and a body made up of three parts: a head, thorax, and abdomen. Ttherefore, unless there are some insects that are both helpful to humans and have an exoskeleton, there must be some insects that are both helpful to humans and are made up of three parts.', \"All insects have an exoskeleton as well as a body that's composed of three parts: a head, thorax, and abdomen. Most insects are helpful to humans. Ttherefore, there must be some insects that are both helpful to humans and have an exoskeleton and some insects that are helpful to humans and have a head, thorax, and abdomen.\", 'Most insects are helpful to humans. Virtually every insect has an exoskeleton and is made up of three parts: a head, thorax, and abdomen. Ttherefore, if there are insects that are helpful to humans and have an exoskeleton, there must also be insects that are helpful to humans and have three parts.', 'Most certainly, most insects are helpful to humans, for almost all insects have an exoskeleton or three parts (a head, thorax, and abdomen), or both, and there are some insects that are helpful to humans and have exoskeletons and some that are helpful to humans and have three parts.']", "label": 1 }, { "id": "train_4434", "context": "The imposition of quotas limiting imported steel will not help the big American steel mills. In fact, the quotas will help \"mini-mills\" flourish in the United States. Those small domestic mills will take more business from the big Americal steel mills than would have been taken by the foreign steel mills in the absence of quotas.", "question": "Which of the following, if true, would cast the most serious doubt on the claim made in the last sentence above?", "answers": "['Domestic \"mini-mills\" produce low-volume, specialized types of steels that are not produced by the big American steel mills.', 'American quotas on imported goods have often induced other countries to impose similar quotas on American goods.', 'Domestic \"mini-mills\" consistently produce better grades of steel than do the big American mills.', 'Foreign steel mills have long produced grades of steel comparable in quality to the steel produced by the big American mills.']", "label": 0 }, { "id": "train_4435", "context": "In the last decade, the country of Alumba has undergone a massive construction boom. In one city, Semanta, hundreds of new buildings have been erected and, as a result, the ambient light at night has increased from 0. 10 lux to 10 lux, causing migratory birds flying to alter their traditional path and fly into Semanta, where they often crash into buildings. The mayor believes that to ensure the birds keep to their traditional flight path while flying through Alumba, the city should turn off lights throughout the city at night during the bird's migratory season, so that ambient light is around 0. 10 lux.", "question": "Which of the following, is necessary, to evaluate the mayor's proposal for limiting the amount of night light that Semanta emits?", "answers": "['Whether there are any other cities in Alumba that might also emit enough ambient light to confuse migrating birds.', 'Whether the period in which birds migrate is consistent each year.', 'Whether there are any other migratory birds flying through Alumba.', \"The percent of birds flying through Alumba that are drawn to Semanta's night light.\"]", "label": 0 }, { "id": "train_4436", "context": "If the concrete is poured while the ground is wet, it will not form a solid foundation. If the concrete does not form a solid foundation, it will either settle unevenly or crack. So if the concrete settles evenly, either it was poured while the ground was dry or it will crack.", "question": "Which one of the following arguments is most closely parallel in its reasoning to the reasoning in the argument above?", "answers": "['The film will not be properly exposed if the camera is not working properly. If the film is not properly exposed, then the photograph will be either blurred or dark. So if the photograph is not blurred, either the camera is working properly or the photograph will be dark.', 'If the camera is working properly, the photograph will not be blurred. The photograph will be blurred if the film is either not properly exposed or not properly developed. So if the camera is working properly, the film will be both properly exposed and properly developed.', 'If the camera is working properly, the film will be properly exposed. If either the film is properly exposed or corrections are made during the developing process, the photograph will not be dark. So if the camera is working properly, the photograph will not be dark.', 'The camera will work properly only if the film is properly exposed. But the film cannot be properly exposed if there is either not enough or too much light. So the camera will not work properly if there is either too much or not enough light.']", "label": 0 }, { "id": "train_4437", "context": "Anderson maintains that travel writing has diminished in quality over the last few decades. Although travel writing has changed in this time, Anderson is too harsh on contemporary travel writers. Today, when the general public is better traveled than in the past, travel writers face a challenge far greater than that of their predecessors: they must not only show their readers a place but also make them see it anew. That the genre has not only survived but also flourished shows the talent of today' s practitioners.", "question": "Which one of the following most accurately describes the role played in the argument by the statement that the general public is better traveled today than in the past?", "answers": "['It is cited as evidence that contemporary travel writing is intended for a wider readership.', 'It is claimed to be a result of good travel writing.', 'It is cited as a reason that travel writing flourishes more today than it has in the past.', 'It is cited as a condition that has transformed the task of the travel writer.']", "label": 3 }, { "id": "train_4438", "context": "This region must find new ways to help business grow. After all, shoe manufacturing used to be a major local industry, but recently has experienced severe setbacks due to overseas competition, so there is a need for expansion into new manufacturing areas. Moreover, our outdated public policy generally inhibits business growth.", "question": "Which one of the following most accurately expresses the main conclusion drawn in the argument?", "answers": "['Outdated public policy inhibits business growth in the region.', 'The region needs to find new ways to enhance business growth.', 'Shoe manufacturing is no longer a major source of income in the region.', 'Business in the region must expand into new areas of manufacturing.']", "label": 1 }, { "id": "train_4439", "context": "In some jurisdictions, lawmakers have instituted sentencing guidelines that mandate a penalty for theft that is identical to the one they have mandated for bribery. Hence, lawmakers in those jurisdictions evidently consider the harm resulting from theft to be equal to the harm resulting from bribery.", "question": "Which one of the following, if true, would most strengthen the argument?", "answers": "[\"If lawmakers mandate penalties for crimes that are proportional to the harm resulting from those crimes, crime in those lawmakers' jurisdictions will be effectively deterred.\", 'Often, in response to the unusually great harm resulting from a particular instance of a crime, lawmakers will mandate an increased penalty for that crime.', 'In general, lawmakers mandate penalties for crimes that are proportional to the harm they believe to result from those crimes.', 'In most cases, a victim of theft is harmed no more than a victim of bribery is harmed.']", "label": 2 }, { "id": "train_4440", "context": "Editorial: In rejecting the plan proposed by parliament to reform the electoral process, the president clearly acted in the best interests of the nation. Anyone who thinks otherwise should remember that the president made this decision knowing it would be met with fierce opposition at home and widespread disapproval abroad. All citizens who place the nation' s well-being above narrow partisan interests will applaud this courageous action.", "question": "The reasoning in the editorial is in error because", "answers": "['it fails to distinguish between evidence concerning the courage required to make a certain decision and evidence concerning the wisdom of making that decision', 'it confuses a quality that is merely desirable in a political leader with a quality that is essential to effective political decision-making', 'it ignores the likelihood that many citizens have no narrow partisan interest in the proposed election reform plan', \"it overlooks the possibility that there was strong opposition to the parliament's plan among members of the president's own party\"]", "label": 0 }, { "id": "train_4441", "context": "The National Farm Administration (NFA) has been concerned over the last decade with the struggles of barley growers. Plan: In an effort to support these barley growers, two years ago, the NFA began a program of sending them, each autumn, a free special mix of fertilizer and enzymes designed to multiply barley yield, to be applied the following spring during first growth. This mix had been stunningly successful in multiplying the yield of barley in laboratory conditions. Results: Most barley growers reported little change in their economic status over this two year period.", "question": " Further information: All barley growers received the shipments, and all used them. Weather conditions have been fair to optimal for barley growth over the past two years. In light of the further information, which of the following, if true, does most to explain the result that followed the implementation of the plan?", "answers": "['This was the second such NFA program to aid barley growers; the first one, 14 years ago, was started with high hopes, but did little to change their situation.', 'This program was implemented at a time when more than half of barley growers nationwide were reported barely breaking even in their yearly expenses.', 'Some of the enzymes in the special mix multiply the growth of a bacteria that feeds on the young barley plants.', 'The trucks that drove the special mix from the depot in Wisconsin to the individual farms sometime took as much as 4 or 5 days.']", "label": 2 }, { "id": "train_4442", "context": "Although custom prosthetic bone replacements produced through a new computer-aided design process will cost more than twice as much as ordinary replacements, custom replacements should still be cost-effective. Not only will surgery and recovery time be reduced, but custom replacements should last longer, thereby reducing the need for further hospital stays.", "question": "Which of the following must be studied in order to evaluate the argument presented above?", "answers": "['The amount by which the cost of producing custom replacements has declined with the introduction of the new technique for producing them', 'The degree to which the use of custom replacements is likely to reduce the need for repeat surgery when compared with the use of ordinary replacements', 'The amount by which custom replacements produced with the new technique will drop in cost as the production procedures become standardized and applicable on a larger scale', 'The amount of time a patient spends in surgery versus the amount of time spent recovering from surgery']", "label": 1 }, { "id": "train_4443", "context": "Cereal advertisement: Fitness experts say that regular exercise is the most effective way to become physically fit, and studies have shown that adults who eat cereal every day exercise more regularly than adults who do not eat cereal. So by eating Fantastic Flakes every morning, you too will be on the most effective path to physical fitness.", "question": "The argumentation in the advertisement is flawed in that it", "answers": "['presumes, without providing justification, that Fantastic Flakes are more nutritious than other cereals', 'infers a cause from a mere correlation', 'draws a conclusion about all adults from a sample that is too small to be representative', 'infers that a given factor is the sole predictor of a result merely on the grounds that the factor has been shown to contribute to that result']", "label": 1 }, { "id": "train_4444", "context": "People are usually interested in, and often even moved by, anecdotes about individuals, whereas they rarely even pay attention to statistical information, much less change their beliefs in response to it. However, although anecdotes are generally misleading in that they are about unrepresentative cases, people tend to have fairly accurate beliefs about society.", "question": "Which one of the following, if true, would most help to explain why people tend to have accurate beliefs about society despite the facts described above?", "answers": "[\"The more emotionally compelling an anecdote is, the more likely it is to change a person's beliefs.\", 'Statistical information tends to obscure the characteristics of individuals.', 'Most people recognize that anecdotes tend to be about unrepresentative cases.', 'People tend to base their beliefs about other people on their emotional response to those people.']", "label": 2 }, { "id": "train_4445", "context": "Medical researcher: Scientists compared a large group of joggers who habitually stretch before jogging to an equal number of joggers who do not stretch before jogging. Both groups of joggers incurred roughly the same number of injuries. This indicates that stretching before jogging does not help to prevent injuries.", "question": "Which one of the following, if true, would most weaken the medical researcher's argument?", "answers": "['Most jogging injuries result from falls, collisions, and other mishaps on which the flexibility resulting from stretching would have little if any effect.', 'For both groups of joggers compared by the scientists, the rate of jogging injuries during the study was lower than the overall rate of jogging injuries.', 'Studies have found that, for certain forms of exercise, stretching beforehand can reduce the severity of injuries resulting from that exercise.', 'The more prone a jogger is to jogging injuries, the more likely he or she is to develop the habit of performing stretches before jogging.']", "label": 3 }, { "id": "train_4446", "context": "Most respondents to a magazine survey who had recently listened to a taped reading of a certain best-selling novel said that they had enjoyed the novel, while most respondents who had recently read the novel themselves said they had not enjoyed it. These survey results support the contention that a person who listens to a taped reading of a novel is more likely to enjoy the novel than a person who reads it is.", "question": "Which one of the following, if true, would most weaken the argument?", "answers": "['Most people can read a novel in considerably less time than it would take them to listen to a taped reading of it.', 'Most of the respondents who had listened to a taped reading of the novel had never read it, and most of the respondents who had read the novel had never listened to a taped reading of it.', 'The novel in question, unlike most novels, included dialogue in many different dialects that are more understandable when heard than when read.', 'When people are asked their opinion of a best- selling novel that they have read or listened to on tape, they are more likely to say that they enjoyed the novel than that they did not enjoy it.']", "label": 2 }, { "id": "train_4447", "context": "Genuine happiness consists not in pleasurable feelings but instead in one' s sense of approval of one' s character and projects. Thus the happy life, in fact, tends to be the good life, where the good life is understood not -- as it usually is these days -- as a life of material well-being but rather as a morally virtuous life.", "question": "Which one of the following is an assumption required by the argument?", "answers": "['A morally virtuous life requires the rejection of material well-being.', 'People who approve of their own character and projects tend to lead morally virtuous lives.', 'Attaining happiness is the real goal of people who strive for material well-being.', \"Approval of one's own character and projects tends not to result in pleasurable feelings.\"]", "label": 1 }, { "id": "train_4448", "context": "The heavy traffic in Masana is a growing drain on the city' s economy--the clogging of the streets of the central business district alone cost the economy more than $1. 2 billion over the past year. In order to address this problem, officials plan to introduce congestion pricing, by which drivers would pay to enter the city's most heavily trafficked areas during the busiest times of the day.", "question": "Which of the following, if true, would most strongly indicate that the plan will be a success?", "answers": "['Planners expect that, without congestion pricing, traffic in Masana is likely to grow by 6 percent in the next five years.', 'In other urban areas, congestion pricing has strongly encouraged carpooling (sharing of rides by private commuters).', \"Over 30 percent of the vehicles in the city's center are occupied by more than one person.\", 'Approximately one-fifth of the vehicles in the central business district are in transit from one side of the city to the other.']", "label": 1 }, { "id": "train_4449", "context": "Spiders of many species change color to match the pigmentation of the flowers they sit on. The insects preyed on by those spiders, unlike human beings, possess color discrimination so acute that they can readily see the spiders despite the seeming camouflage. Clearly, then, it must be in evading their own predators that the spiders' color changes are useful to them.", "question": "Which of the following, if true, most strengthens the argument?", "answers": "['Certain animals that feed on color-changing spiders do so only sparingly in order to keep from ingesting harmful amounts of spider venom.', 'The color discrimination of certain birds that feed on color-changing spiders is no more acute than that of human beings.', 'Color-changing spiders possess color discrimination that is more acute than that of spiders that lack the ability to change color.', 'Among the animals that feed on color- changing spiders are a few species of bat, which find their prey through sound echoes.']", "label": 1 }, { "id": "train_4450", "context": "Columnist: People should avoid using a certain artificial fat that has been touted as a resource for those whose medical advisers have advised them to reduce their fat intake. Although the artificial fat, which can be used in place of fat in food preparation, has none of the negative health effects of fat, it does have a serious drawback: it absorbs certain essential vitamins, thereby preventing them from being used by the body.", "question": "In evaluating the columnist's position, it would be most useful to determine which of the following?", "answers": "['Whether the vitamins that the artificial fat absorbs are present in foods that contain the fat', \"Whether increasing one's intake of the vitamins can compensate for the effects of the artificial fat\", 'Whether there are any foods that cannot be prepared using the artificial fat as a substitute for other fats', 'Whether having an extremely low fat intake for an extended period can endanger the health']", "label": 1 }, { "id": "train_4451", "context": "Smokers of pipes or cigars run a distinctly lower risk to their health than do cigarette smokers. However, whereas cigarette smokers who quit smoking altogether sharply reduce their risk of smoking related health problems, those who give up cigarettes and take up pipes or cigars remain in as much danger as before.", "question": "Which one of the following, if true, offers the best prospects for an explanation of why the two changes in smoking habits do not both result in reduced health risks?", "answers": "['Cigarette smokers who quit smoking for a time and who then resume cigarette smoking do not necessarily reduce their risk of smoking-related health problems.', 'The kinds of illnesses that smokers run an increased risk of contracting develop no earlier in cigarette smokers than they do in smokers of pipes or cigars.', 'At any given period in their lives, virtually all smokers smoke either cigarettes exclusively or cigars exclusively or pipes exclusively, rather than alternating freely among various ways of smoking.', 'People who switch from cigarette smoking to smoking pipes or cigars inhale smoke in a way that those who have never smoked cigarettes do not.']", "label": 3 }, { "id": "train_4452", "context": "Millions of irreplaceable exhibits in natural history museums are currently allowed to decay. Yet without analyses of eggs from museums, the studies linking pesticides with the decline of birds of prey would have been impossible. Ttherefore, funds must be raised to preserve at least those exhibits that will be most valuable to science in the future.", "question": "The argument presupposes that", "answers": "['the decay of organic material in natural history exhibits is natural and cannot be prevented', 'the scientific analysis of museum exhibits can be performed in a nondestructive way', 'if a museum exhibit is irreplaceable, its preservation is of an importance that overrides economic considerations', 'it can be known at this time what data will be of most use to scientific investigators in the future']", "label": 3 }, { "id": "train_4453", "context": "Most of the mines that Moradco operates in the province of Velyena have never violated environmental regulations. Every one of the gold mines that Moradco operates throughout the world has at some time or another violated environmental regulations.", "question": "Which one of the following statements follows logically from the statements above?", "answers": "['Most of the gold mines that Moradco operates are not located in Velyena.', 'Moradco operates more mines in Velyena than any other company operates there.', 'Most of the mines that Moradco operates throughout the world are not gold mines.', 'Most of the mines that Moradco operates in Velyena are not gold mines.']", "label": 3 }, { "id": "train_4454", "context": "The tiny country of Minlandia does not produce its own television programming. Instead, the citizens of Minlandia, who generally are fluent not only in their native Minlandian, but also in Boltese, watch Boltese-language television programs from neighboring Bolta. Surveys show that the Minlandians spend on average more hours per week reading for pleasure and fewer hours per week watching television than people anywhere else in the world. A prominent psychologist accounts for the survey results by explaining that people generally prefer to be entertained in their native language even if they are perfectly fluent in other languages.", "question": "The explanation offered by the psychologist accounts for the Minlandians' behavior only if which one of the following is assumed?", "answers": "['At least some of what the Minlandians read for pleasure is in the Minlandian language.', 'The study of Boltese is required of Minlandian children as part of their schooling.', 'The proportion of bilingual residents to total population is greater in Minlandia than anywhere else in the world.', 'When Minlandians watch Boltese television programs, they tend to ignore the fact that they are hearing a foreign language spoken.']", "label": 0 }, { "id": "train_4455", "context": "The irradiation of food kills bacteria and thus retards spoilage. However, it also lowers the nutritional value of many foods. For example, irradiation destroys a significant percentage of whatever vitamin B1 a food may contain. Proponents of irradiation point out that irradiation is no worse in this respect than cooking. However, this fact is either beside the point, since much irradiated food is eaten raw, or else misleading, since __.", "question": "Which of the following most logically completes the argument?", "answers": "[\"many of the proponents of irradiation are food distributors who gain from food's having a longer shelf life\", 'for food that is both irradiated and cooked, the reduction of vitamin B1 associated with either process individually is compounded', 'it is clear that killing bacteria that may be present on food is not the only effect that irradiation has', 'cooking is usually the final step in preparing food for consumption, whereas irradiation serves to ensure a longer shelf life for perishable foods']", "label": 1 }, { "id": "train_4456", "context": "The number of North American children who are obese -- that is, who have more body fat than do 85 percent of North American children their age -- is steadily increasing, according to four major studies conducted over the past 15 years.", "question": "If the finding reported above is correct, it can be properly concluded that", "answers": "['the incidence of obesity in North American children tends to increase as the children grow older', 'when four major studies all produce similar results, those studies must be accurate', 'the number of North American children who are not obese increased over the past 15 years', 'over the past 15 years, the number of North American children who are underweight has declined']", "label": 2 }, { "id": "train_4457", "context": "Artist: Avant-garde artists intend their work to challenge a society' s mainstream beliefs and initiate change. And some art collectors claim that an avant-garde work that becomes popular in its own time is successful. However, a society' s mainstream beliefs do not generally show any significant changes over a short period of time. Ttherefore, when an avant-garde work becomes popular it is a sign that the work is not successful, since it does not fulfil the intentions of its creator.", "question": "The reference to the claim of certain art collectors plays which one of the following roles in the artist's argument?", "answers": "['It identifies a view that is ultimately disputed by the argument.', 'It provides support for the initial premise in the argument.', 'It provides support for a counterargument to the initial premise.', 'It identifies a position supported by the initial premise in the argument.']", "label": 0 }, { "id": "train_4458", "context": "Nutritionist: The healthiest soups all contain spinach and various other vegetables. Few vegetable soups are thought to be rich in nutrients. The lower the nutritional benefit of a food, the less healthy it is. No vegetable soup that does not contain spinach then should be considered healthy.", "question": "Which one of the following would it be most relevant to investigate in evaluating the conclusion of the nutritionist's argument?", "answers": "['Is spinach the only high-nutrient vegetable that is used in soup?', 'Which vegetables contain the least nutrients?', 'Do substances added during the canning process make soup healthier?', 'Are cream soups healthier than vegetable soups?']", "label": 0 }, { "id": "train_4459", "context": "Politician: Of the candidates running, Thompson is the best person to lead this nation. For one thing, Thompson opposes higher taxes whereas the other candidates support them. Many would agree that anyone who opposes higher taxes will make a better leader than someone who supports them.", "question": "Which one of the following, if true, casts the most doubt on the politician's argument?", "answers": "['All of the past leaders who supported higher taxes were hardworking.', 'All of the past leaders who supported higher taxes performed their jobs adequately.', 'Being opposed to higher taxes is not a sufficient condition for good leadership.', 'Opposing higher taxes is not a factor contributing to good leadership.']", "label": 3 }, { "id": "train_4460", "context": "According to the rules of the university' s housing lottery, the only students guaranteed dormitory rooms are fourth-year students. In addition, any fourth-year student on the dean' s list can choose a dormitory room before anyone who is not a fourth-year student.", "question": "Which one of the following inferences is most strongly supported by the rules described above?", "answers": [ "Anissa is a fourth-year student who is on the dean's list. Thus, since Jehan is a second-year student who is also on the dean's list, he can choose a dormitory room before Anissa.", "Ivan and Naomi are both fourth-year students but only Naomi is on the dean's list. Ttherefore, Ivan can choose a dormitory room before Naomi.", "Halle, a third-year student, is on the dean's list. Thus, she is guaranteed a dormitory room.", "Gerald and Katrina are both on the dean's list but only Gerald is a fourth-year student. Thus, Gerald can choose a dormitory room before Katrina." ], "label": 3 }, { "id": "train_4461", "context": "Musicologist: Classification of a musical instrument depends on the mechanical action through which it produces music. So the piano is properly called a percussion instrument, not a stringed instrument. Even though the vibration of the piano' s strings is what makes its sound, the strings are caused to vibrate by the impact of hammers.", "question": "Which one of the following most accurately expresses the main conclusion of the musicologist's argument?", "answers": "['Musical instruments should not be classified based on the way musicians interact with them.', 'It is correct to classify the piano as a percussion instrument rather than as a stringed instrument.', 'The piano should be classified as a stringed instrument rather than as a percussion instrument.', 'Some people classify the piano as a stringed instrument because of the way the piano produces sound.']", "label": 1 }, { "id": "train_4462", "context": "Commentator: If a political administration is both economically successful and successful at protecting individual liberties, then it is an overall success. Even an administration that fails to care for the environment may succeed overall if it protects individual liberties. So far, the present administration has not cared for the environment but has successfully protected individual liberties.", "question": "If all of the statements above are true, then which one of the following must be true?", "answers": "['If the present administration is economically successful, then it is an overall success.', 'If the present administration had been economically successful, it would have cared for the environment.', 'The present administration is not an overall success.', 'The present administration is economically successful.']", "label": 0 }, { "id": "train_4463", "context": "Every brick house on River Street has a front yard. Most of the houses on River Street that have front yards also have two stories. So most of the brick houses on River Street have two stories.", "question": "Which one of the following is most appropriate as an analogy demonstrating that the reasoning in the argument above is flawed?", "answers": "['By that line of reasoning, we could conclude that most politicians have run for office, since all legislators are politicians and most legislators have run for office.', 'By that line of reasoning, we could conclude that not every public servant has run for office, since every legislator is a public servant but some public servants are not legislators.', 'By that line of reasoning, we could conclude that most public servants are legislators, since most legislators have run for office and most politicians who have run for office are public servants.', 'By that line of reasoning, we could conclude that most legislators have never run for office, since most public servants have never run for office and all legislators are public servants.']", "label": 3 }, { "id": "train_4464", "context": "Advertisement: HomeGlo Paints, Inc. , has won the prestigious Golden Paintbrush Award given to the one paint manufacturer in the country that has increased the environmental safety of its product most over the past three years for HomeGlo Exterior Enamel. The Golden Paintbrush is awarded only on the basis of thorough tests by independent testing laboratories. So when you choose HomeGlo Exterior Enamel, you will know that you have chosen the most environmentally safe brand of paint manufactured in this country today.", "question": "The flawed reasoning in the advertisement most closely parallels that in which one of the following?", "answers": "[\"The ZXC audio system received the overall top ranking for looks, performance, durability, and value in Listeners' Report magazine's ratings of currently produced systems. Ttherefore, the ZXC must have better sound quality than any other currently produced sound system.\", \"Jerrold's teachers recognize him as the student who has shown more academic improvement than any other student in the junior class this year. Ttherefore, if Jerrold and his classmates are ranked according to their current academic performance, Jerrold must hold the highest ranking.\", \"The number of consumer visits increased more at Countryside Market last year than at any other market in the region. Ttherefore, Countryside's profits must also have increased more last year than those of any other market in the region.\", 'Margaret Durring\\'s short story \"The Power Lunch\" won three separate awards for best short fiction of the year. Ttherefore, any of Margaret Durring\\'s earlier stories certainly has enough literary merit to be included in an anthology of the best recent short fiction.']", "label": 1 }, { "id": "train_4465", "context": "Aesthetician: Zahib' s rejection of contemporary literature' s aesthetic value depends on his claim that today' s writing generally fails to grapple seriously enough with life' s deepest ethical questions -- whereas great books, he maintains, present profound moral lessons and \"the stuff of conscience. \" But what resounding moral lesson does Vikram Seth' s A Suitable Boy or Devaki Nandan Khatri' s Chandrakanta impart? People read these two great novels because they are engaging, even thrilling, stories. The absence of a profound moral lesson in no way detracts from the aesthetic value of a novel.", "question": "The aesthetician criticizes Zahib's position by arguing that it depends on the questionable premise that", "answers": "['only novels that have aesthetic value fail to present profound moral lessons', \"today's writing generally fails to confront deep ethical questions\", 'for a literary work to have aesthetic value it must present a profound ethical message', 'there is no distinction between engaging stories and profound moral lessons']", "label": 2 }, { "id": "train_4466", "context": "Many mountain climbers regard climbing Mount Everest as the ultimate achievement. But climbers should not attempt this climb since the risk of death or serious injury in an Everest expedition is very high. Moreover, the romantic notion of gaining \"spiritual discovery\" atop Everest is dispelled by climbers' reports that the only profound experiences they had at the top were of exhaustion and fear.", "question": "Which one of the following principles, if valid, most helps to justify the reasoning above?", "answers": "['Profound spiritual experiences can be achieved without undergoing the serious danger involved in mountain climbing.', 'Activities that are extremely dangerous ought to be legally prohibited unless they are necessary to produce spiritual enlightenment.', 'Dangerous activities that are unlikely to result in significant spiritual benefits for those undertaking them should be avoided.', 'Mountain climbers and other athletes should carefully examine the underlying reasons they have for participating in their sports.']", "label": 2 }, { "id": "train_4467", "context": "Naturalist: The recent claims that the Tasmanian tiger is not extinct are false. The Tasmanian tiger' s natural habitat was taken over by sheep farming decades ago, resulting in the animal' s systematic elimination from the area. Since then naturalists working in the region have discovered no hard evidence of its survival, such as carcasses or tracks. In spite of alleged sightings of the animal, the Tasmanian tiger no longer exists.", "question": "Which one of the following is an assumption on which the naturalist's argument depends?", "answers": "['Those who have reported sightings of the Tasmanian tiger are not experienced naturalists.', \"Every naturalist working in the Tasmanian tiger's natural habitat has looked systematically for evidence of the tiger's survival.\", 'Some scavengers in Tasmania are capable of destroying tiger carcasses without a trace.', 'The Tasmanian tiger did not move and adapt to a different region in response to the loss of habitat.']", "label": 3 }, { "id": "train_4468", "context": "How the pigment known as Han purple was synthesized by the ancient Chinese of the Qin and Han dynasties has puzzled scientists. The Chinese chemists employed the same chemical ingredients used for Han purple in the production of a common type of white glass during that period. Both were produced in processes that involved subjecting the mixtures to high heat and mixing in lead to decrease the melting temperature. Thus, Han purple was probably discovered by fortuitous accident during glass production.", "question": "Which one of the following, if true, would most strengthen the argument?", "answers": "['Chemical analysis shows that most of the known fragments of both Han purple and the white glass were produced within a small geographical radius.', 'The technique used for producing Han purple was known to very few people during the Qin and Han dynasties.', 'The ingredients used in producing both Han purple and the white glass were easily obtainable during the Qin and Han dynasties.', 'The white glass is found in more surviving artifacts from the Qin and Han dynasties than Han purple is.']", "label": 0 }, { "id": "train_4469", "context": "Journalist: It is unethical for journalists to lie -- to say something untrue with the purpose of deceiving the listener -- to get a story. However, journalists commonly withhold relevant information in interviews in order to elicit new information. Some argue that this, like lying, is intentional deception and ttherefore unethical. However, this argument fails to recognize the distinction between failing to prevent a false belief and actively encouraging one. Lying is unethical because it actively encourages a false belief.", "question": "The journalist argues by", "answers": "['defining a concept and then showing that under this definition the concept applies to all of the cases under discussion', 'pointing out a difference between the two cases being compared in order to show that a conclusion based on their similarities should not be drawn', 'clarifying and defending a moral principle by comparing a case in which it applies to one in which it does not apply', 'appealing to a counterexample to undermine an ethical principle that supports an argument the journalist is trying to refute']", "label": 1 }, { "id": "train_4470", "context": "Colorless diamonds can command high prices as gemstones. A type of less valuable diamonds can be treated to remove all color. Only sophisticated tests can distinguish such treated diamonds from naturally colorless ones. However, only 2 percent of diamonds mined are of the colored type that can be successfully treated, and many of those are of insufficient quality to make the treatment worthwhile. Surely, ttherefore, the vast majority of colorless diamonds sold by jewelers are naturally colorless.", "question": "A serious flaw in the reasoning of the argument is that", "answers": "['information about the rarity of treated diamonds is not combined with information about the rarity of naturally colorless, gemstone diamonds', 'the currently available method for making colorless diamonds from colored ones is treated as though it were the only possible method for doing so', 'comparisons between the price diamonds command as gemstones and their value for other uses are omitted', 'the difficulty that a customer of a jeweler would have in distinguishing a naturally colorless diamond from a treated one is not taken into account']", "label": 0 }, { "id": "train_4471", "context": "Once people habitually engaged in conversation; now the television competes for their attention. When the television is on, communication between family members stops. Where there is no communication, family ties become frayed and eventually snap. Ttherefore, the only solution is to get rid of the television.", "question": "Which one of the following is most closely parallel in its reasoning to the flawed reasoning in the argument above?", "answers": "['Once sports enthusiasts regularly engaged in sports, but now they watch spectator sports when they could be getting physical exercise. Without physical exercise, health deteriorates. Ttherefore, the only remedy is to eliminate spectator sports.', 'Once people listened to the radio while pursuing other activities. Now they passively watch television. Ttherefore, radio was less distracting for most people than television is.', 'Once people were willing to tailor their day to the constraints of a bus or train schedule; now they are spoiled by the private car. The only solution is for government to offer financial incentives to encourage the use of public transportation.', 'Once friendships thrived on shared leisure time. But contemporary economic pressures minimize the amount of free time people have and thus jeopardize many friendships.']", "label": 0 }, { "id": "train_4472", "context": "Educator: If there is a crisis in education today, it is one of maintaining quality. People love to reduce serious learning to degrees and certificates. But one also can obtain these credentials by plodding through courses without ever learning much of value. When that happens, the credentials one receives are almost meaningless.", "question": "If the educator's statements are true, then which one of the following must be true?", "answers": "['A person benefits from an education only to the extent that he or she invests effort in it.', 'Degrees and certificates do not guarantee that a person has acquired much worthwhile knowledge.', 'It has become easier for students to complete their coursework without learning anything of importance.', 'Educational institutions should cease to grant degrees and certificates.']", "label": 1 }, { "id": "train_4473", "context": "Farmer: In the long run, it is counterproductive for farmers to use insecticides. Because insects' resistance to insecticides increases with insecticide use, farmers have to use greater and greater amounts of costly insecticides to control insect pests.", "question": "Which one of the following most accurately describes the role played in the farmer's argument by the proposition that farmers have to use greater and greater amounts of costly insecticides to control insect pests?", "answers": [ "It identifies a phenomenon for which the argument's main conclusion offers a causal explanation.", "It is the argument's only conclusion.", "It is a claim for which a causal explanation is provided and which itself is used as direct support for the argument's only conclusion.", "It is the argument's main conclusion, but not its only conclusion." ], "label": 2 }, { "id": "train_4474", "context": "The only motives that influence all human actions arise from self-interest. It is clear, ttherefore, that self-interest is the chief influence on human action.", "question": "The reasoning in the argument is fallacious because the argument", "answers": "['denies that an observation that a trait is common to all the events in a pattern can contribute to a causal explanation of the pattern', 'undermines its own premise that a particular attribute is present in all instances of a certain pattern or class of events', 'concludes that a characteristic of a pattern or class of events at one time is characteristic of similar patterns or classes of events at all times', 'takes the occurrence of one particular influence on a pattern or class of events as showing that its influence outweighs any other influence on those events']", "label": 3 }, { "id": "train_4475", "context": "Essayist: Winners of a Nobel prize for science, who are typically professional scientists, have all made significant contributions to science. But amateur scientists have also provided many significant contributions. And unlike professional scientists, who are often motivated by economic necessity or a desire for fame, amateur scientists are motivated by the love of discovery alone.", "question": "If the essayist's statements are true, then which one of the following must also be true?", "answers": "['The love of discovery is the motive behind many significant contributions to science.', 'Typically, winners of a Nobel prize for science are not motivated at all by the love of discovery.', 'A professional scientist is more likely to make a significant contribution to science if he or she is motivated by the love of discovery.', 'Some amateur scientists who did not win a Nobel prize for science nevertheless made significant contributions to science.']", "label": 0 }, { "id": "train_4476", "context": "Researchers gave 100 first-graders after-school lessons in handwriting. They found that those whose composition skills had improved the most had learned to write letters the most automatically. This suggests that producing characters more automatically frees up mental resources for other activities.", "question": "Which one of the following, if true, most strengthens the argument?", "answers": "['Over the course of the lessons, the first-graders who showed greater improvement in their ability to write letters automatically also generally showed greater improvement in their composition skills.', 'The first-graders who wrote letters the most automatically before receiving the after-school lessons in handwriting showed the greatest improvement in their composition skills over the course of the lessons.', 'Among the first-graders who received the lessons in handwriting, those who started out with strong composition skills showed substantial improvement in how automatically they could write letters.', 'Before receiving the after-school lessons in handwriting, the 100 first-graders who received the lessons were representative of first-graders more generally, with respect to their skills in both handwriting and composition.']", "label": 0 }, { "id": "train_4477", "context": "You should not praise an act of apparent generosity unless you believe it is actually performed out of selfless motives, and you should not condemn an act of apparent selfishness unless you believe it is actually performed out of self-centered motives.", "question": "Which one of the following judgments conforms to the principle stated above?", "answers": "['Caroline rightly blamed her coworker Monica for failing to assist her in doing a time-consuming project, even though she knew that Monica had offered to help in the project earlier but that her offer had been vetoed by their supervisor.', 'Albert correctly criticized Louise for adopting an abandoned dog because he believed that, although she felt sorry for the dog, she did not have sufficient time or space to care for it adequately.', \"Daniel was right not to praise Margaret for offering to share her house with a visiting French family, since he believed that she made the offer only because she hoped it would be reciprocated by an invitation to use the family's apartment in Paris.\", 'It was correct for Sarah not to praise Michael for being charitable when he told her that he donates a tenth of his income to charity, since she guessed that he only told that fact in order to impress her.']", "label": 2 }, { "id": "train_4478", "context": "Infotek, a computer manufacturer in Katrovia, has just introduced a new personal computer model that sells for significantly less than any other model. Market research shows, however, that very few Katrovian households without personal computers would buy a computer, regardless of its price. Ttherefore, introducing the new model is unlikely to increase the number of computers in Katrovian homes.", "question": "Which of the following is an assumption on which the argument depends?", "answers": "['Infotek achieved the lower price of the new model by using components of lower quality than those used by other manufacturers.', 'Most personal computers purchased in Katrovia are intended for home use.', 'Katrovians in households that already have computers are unlikely to purchase the new Infotek model as an additional computer for home use.', 'The main reason cited by consumers in Katrovia for replacing a personal computer is the desire to have an improved model.']", "label": 2 }, { "id": "train_4479", "context": "Although the number of large artificial satellites orbiting the Earth is small compared to the number of small pieces of debris in orbit, the large satellites interfere more seriously with telescope observations because of the strong reflections they produce. Because many of those large satellites have ceased to function, the proposal has recently been made to eliminate interference from nonfunctioning satellites by exploding them in space. This proposal, however, is ill conceived, since __.", "question": "Which of the following most logically completes the argument below?", "answers": "['the only way to make telescope observations without any interference from debris in orbit is to use telescopes launched into extremely high orbits around the Earth', \"a greatly increased number of small particles in Earth's orbit would result in a blanket of reflections that would make certain valuable telescope observations impossible\", \"there are no known previous instances of satellites' having been exploded on purpose\", 'for satellites that have ceased to function, repairing them while they are in orbit would be prohibitively expensive']", "label": 1 }, { "id": "train_4480", "context": "The town of Stavanger, Norway, was quiet and peaceful until the early 1960' s, when Stavanger became Norway' s center for offshore oil exploration. Between then and now, violent crime and vandalism in Stavanger have greatly increased. Clearly, these social problems are among the results of Stavanger' s oil boom.", "question": "Which of the following, if it occurred between the early 1960's and now, gives the strongest support to the argument above?", "answers": "['Violent crime and vandalism have remained low in Norwegian towns that had no oil boom.', 'Norwegian sociologists expressed grave concern about the increase in violent crime and vandalism in Stavanger.', \"The people of Stavanger rarely regret that their town was chosen to be Norway's center for offshore oil exploration\", 'The oil boom necessitated the building of wider roads for the increased traffic in Stavanger.']", "label": 0 }, { "id": "train_4481", "context": "Following several years of declining advertising sales, the Greenville Times reorganized its advertising sales force two years ago. Before the reorganization, the sales force was organized geographically, with some sales representatives concentrating on city-center businesses and others concentrating on different outlying regions. The reorganization attempted to increase the sales representatives' knowledge of clients' businesses by having each sales representative deal with only one type of industry or of retailing. After the reorganization, advertising sales increased.", "question": "In assessing whether the improvement in advertising sales can properly be attributed to the reorganization, it would be helpful to find out each of the following EXCEPT:", "answers": "[\"Two years ago, what proportion of the Greenville Times's total revenue was generated by advertising sales?\", 'Has the circulation of the Greenville Times increased substantially in the last two years?', 'Has there been a substantial turnover in personnel in the advertising sales force over the last two years?', 'Has the economy in Greenville and the surrounding regions been growing rapidly over the last two years?']", "label": 0 }, { "id": "train_4482", "context": "If the regulation of computer networks is to be modeled on past legislation, then its model must be either legislation regulating a telephone system or else legislation regulating a public broadcasting service. If the telephone model is used, computer networks will be held responsible only for ensuring that messages get transmitted. If the public broadcast model is used, computer networks will additionally be responsible for the content of those messages. Yet a computer network serves both these sorts of functions: it can serve as a private message service or as a publicly accessible information service. Thus neither of these models can be appropriate for computer networks.", "question": "The passage is structured to lead to which one of the following conclusions?", "answers": "['Regulation of computer networks is required in order to ensure the privacy of the messages transmitted through such networks.', 'Legislators who do not have extensive experience with computers should not attempt to write legislation regulating computer networks.', 'The regulation of computer networks should not be modeled on any single piece of past legislation.', 'A computer network merely duplicates the functions of a telephone system and a television network.']", "label": 2 }, { "id": "train_4483", "context": "The average size of marine life that washes up on the shore of the Japanese island Ryukyu is smaller than the average size that washes up on the Western coast of Australia. Giant squid have recently been found washed up on the shores of Ryukyu as well as the Western coast of Australia. It can be concluded that the average size of the giant squids on the shore Ryukyu must be less than that of giant squids washed up on the shores of Western Australia.", "question": "The argument above can be attacked on the grounds that it does which of the following?", "answers": "['It assumes that a general pattern is likely to hold true in a specific case.', 'It fails to distinguish between giant squids and more diminutive variants.', 'It mistakenly asserts that one instance holds true for all cases.', 'It discounts the possibility that the largest giant squid was found on the shores of Ryukyu.']", "label": 0 }, { "id": "train_4484", "context": "In our solar system only one of the nine planets -- Earth -- qualifies as fit to sustain life. Nonetheless, using this ratio, and considering the astonishingly large number of planetary systems in the universe, we must conclude that the number of planets fit to sustain some form of life is extremely large.", "question": "The argument is questionable because it presumes which one of the following without providing justification?", "answers": "[\"Life similar to Earth's could evolve under conditions very different from those on Earth.\", 'If a planet is Earthlike, then life will arise on it.', 'Our solar system is similar to many other planetary systems in the universe.', 'Most other planetary systems in the universe have nine planets.']", "label": 2 }, { "id": "train_4485", "context": "Electric stovetop burners would cause fewer fires if their highest temperature were limited to 350 degree C (662 degree F), which provides more than enough heat for efficient and effective cooking. The lowest temperature at which cooking oil and most common fibers ignite is 387 degree C, and electric burners on high go well above 700 degree C.", "question": "Which one of the following most accurately expresses the conclusion drawn in the argument?", "answers": "['A maximum temperature of 350 degree C provides more than enough heat for efficient and effective cooking.', 'Electric stovetop burners cause fires because they go well above 700 degree C when set on high.', 'Electric stovetop burners would cause fewer fires if their highest temperature were limited to 350 degree C.', 'The lowest ignition temperature for cooking oil and most common fibers is 387 degree C.']", "label": 2 }, { "id": "train_4486", "context": "Eurasian watermilfoil, a weed not native to Frida Lake, has reproduced prolifically since being accidentally introduced there. In order to eliminate the weed, biologists proposed treating infested parts of the lake with a certain herbicide that is nontoxic for humans and aquatic animals. However, the herbicide might damage populations of certain rare plant species that the lake contains. For this reason local officials rejected the proposal.", "question": "Which of the following, if true, points out the most serious weakness in the officials' grounds for rejecting the biologists' proposal?", "answers": "[\"The continuing spread of Eurasian watermilfoil in Frida Lake threatens to choke out the lake's rare plant species.\", 'Because of ecological conditions prevailing in its native habitat, Eurasian watermilfoil is not as dominant there as it is in Frida Lake.', 'Unless Eurasian watermilfoil is completely eliminated from Frida Lake, it will quickly spread again once herbicide treatments or other control measures cease.', 'The proliferation of Eurasian watermilfoil in Frida Lake has led to reductions in the populations of some species of aquatic animals.']", "label": 0 }, { "id": "train_4487", "context": "Researcher: Each subject in this experiment owns one car, and was asked to estimate what proportion of all automobiles registered in the nation are the same make as the subject' s car. The estimate of nearly every subject has been significantly higher than the actual national statistic for the make of that subject' s car. I hypothesize that certain makes of car are more common in some regions of the nation than in other regions; obviously, that would lead many people to overestimate how common their make of car is nationally. That is precisely the result found in this experiment, so certain makes of car must indeed be more common in some areas of the nation than in others.", "question": "Which one of the following most accurately expresses a reasoning flaw in the researcher's argument?", "answers": "['The argument attempts to draw its main conclusion from a set of premises that are mutually contradictory.', 'The argument treats a result that supports a hypothesis as a result that proves a hypothesis.', 'The argument applies a statistical generalization to a particular case to which it was not intended to apply.', 'The argument fails to estimate the likelihood that most subjects in the experiment did not know the actual statistics about how common their make of car is nationwide.']", "label": 1 }, { "id": "train_4488", "context": "To hold criminals responsible for their crimes involves a failure to recognize that criminal actions, like all actions, are ultimately products of the environment that forged the agent' s character. It is not criminals but people in the law-abiding majority who by their actions do most to create and maintain this environment. Ttherefore, it is law-abiding people whose actions, and nothing else, make them alone truly responsible for crime.", "question": "The reasoning in the argument is most vulnerable to criticism on the grounds that", "answers": "['the way it distinguishes criminal from crimes implicitly denies that someone becomes a criminal solely in virtue of having committed a crime', 'it exploits an ambiguity in the term \"environment\"by treating two different meanings of the word as though they were equivalent', 'it fails to distinguish between actions that are socially acceptable and actions that are socially unacceptable', 'its conclusion contradicts an implicit principle on which an earlier part of the argument is based']", "label": 3 }, { "id": "train_4489", "context": "When surveyed about which party they would like to see in the legislature, 40 percent of respondents said Conservative, 20 percent said Moderate, and 40 percent said Liberal. If the survey results are reliable, we can conclude that most citizens would like to see a legislature that is roughly 40 percent Conservative, 20 percent Moderate, and 40 percent Liberal.", "question": "Which one of the following most accurately describes a flaw in the reasoning of the argument?", "answers": "['The argument uses evidence that supports only rough estimates to draw a precisely quantified conclusion.', 'The argument draws a conclusion that merely restates a premise presented in favor of it.', 'The argument takes for granted that the preferences of a group as a whole are the preferences of most individual members of the group.', 'The argument fails to consider that the survey results might have been influenced by the political biases of the researchers who conducted the survey.']", "label": 2 }, { "id": "train_4490", "context": "Hoping to increase his company' s market share, the CEO of QuikFashion has decided to hold a month-long sale in which all of the clothing in QuikFashion' s stores will be discounted by at least 50 percent. Although this discount will mean that the company is selling clothing at cost, the CEO hopes that this broadly advertised sale will attract repeat buyers to QuikFashion, who will return to buy clothing at full price after the month of sales.", "question": "In assessing the CEO's ability to achieve his goals with this plan, it would be most useful to know which of the following?", "answers": "[\"Whether QuikFashion's competitors would respond by discounting their own clothing.\", 'Whether some items of clothing will be discounted more than others.', 'Whether QuikFashion can cut the cost of production sufficiently to still make profits from its discounted clothing.', 'Whether following a different plan might allow QuikFashion to maintain its current market share but increase profits.']", "label": 0 }, { "id": "train_4491", "context": "For one academic year all the students at a high school were observed. The aim was to test the hypothesis that studying more increased a student' s chances of earning a higher grade. It turned out that the students who spent the most time studying did not earn grades as high as did many students who studied less. Nonetheless, the researchers concluded that the results of the observation supported the initial hypothesis.", "question": "Which one of the following, if true, most helps to explain why the researchers drew the conclusion described above?", "answers": "['In each course, the more a student studied, the better his or her grade was in that course.', 'The students who spent the least time studying tended to be students with no more than average involvement in extracurricular activities.', 'Students who spent more time studying understood the course material better than other students did.', 'The students who spent the most time studying earned higher grades than did some students who studied for less time than the average.']", "label": 0 }, { "id": "train_4492", "context": "Activist: All drugs should be legalized. Despite the current crackdown, the black market has filled the supply, ensuring that people can obtain any drug at any time, if desired. The government currently incarcerates millions of nonviolent drug users at the cost of $50, 000 per year per prisoner. This figure doesn' t even take into account the financial burden placed on the police and judicial system. Legalizing drugs would not only reduce the cost of law enforcement, but the drugs could also be taxed to generate revenue.", "question": "Which one of the following, if true, most weakens the activist's argument?", "answers": "['Legalized drugs would be more potent than what currently exists on the black market.', 'Legalizing drugs would increase the number of drug users.', 'Legalizing drugs would raise government healthcare expenditures.', 'Some drugs are more dangerous than others.']", "label": 2 }, { "id": "train_4493", "context": "Whole wheat bread is more nutritious than white bread, which has a taste that many people prefer. Bread made from whole-grain white wheat flour, newly available, provides nutrition comparable to whole wheat bread but tastes just like white bread. Once consumers try whole-grain white wheat bread, they will prefer eating it to eating white bread.", "question": "The conclusion of the argument above is properly drawn if which of the following is assumed?", "answers": "['Some people prefer the taste of whole wheat bread to that of white bread.', 'Consumers will prefer eating foods that are more nutritious to eating foods that are less nutritious when the foods taste the same.', 'Whole-grain white wheat flour costs consumers more than does white flour but less than does whole wheat flour.', 'Whole-grain white wheat flour is not significantly more difficult to produce than is white flour.']", "label": 1 }, { "id": "train_4494", "context": "Gallery owner: Because this painting appears in no catalog of van Gogh' s work, we cannot guarantee that he painted it. But consider: the subject is one he painted often, and experts agree that in his later paintings van Gogh invariably used just such broad brushstrokes and distinctive combinations of colors as we find here. Internal evidence, ttherefore, makes it virtually certain that this is a previously uncataloged, late van Gogh, and as such, a bargain at its price.", "question": "The reasoning used by the gallery owner is flawed because it", "answers": "['neglects to cite expert authority to substantiate the claim about the subject matter of the painting', 'ignores the fact that there can be general agreement that something is the case without its being the case', \"attempts to establish a particular conclusion because doing so is in the reasoner's self-interest rather than because of any genuine concern for the truth of the matter\", \"provides no evidence that the painting is more likely to be an uncataloged van Gogh than to be a painting by someone else who painted that particular subject in van Gogh's style\"]", "label": 3 }, { "id": "train_4495", "context": "Parent: I had tried without success to get my young child to brush her teeth. I had hoped that she would imitate me, or that she would be persuaded by reason to brush her teeth. Then, I made a point of brushing her teeth for her immediately before reading her a story before her naps and at night. After several weeks, when I would pick up a storybook at these times, she began automatically to retrieve her toothbrush and brush her teeth herself.", "question": "The parent's experience with the child most closely conforms to which one of the following generalizations?", "answers": "['Children more readily adopt a behavior through habit and repetition than through other means.', 'Children often imitate the behavior of others rather than listening to reason.', \"Children ordinarily act contrary to their parents' expectations in order to get more attention.\", 'Children are too young to understand rational arguments for adopting a behavior.']", "label": 0 }, { "id": "train_4496", "context": "NorthAir charges low fares for its economy class seats, but it provides very cramped seating and few amenities. Market research shows that economy passengers would willingly pay more for wider seating and better service, and additional revenue provided by these higher ticket prices would more than cover the additional cost of providing these amenities. Even though NorthAir is searching for ways to improve its profitability, it has decided not to make these improvements.", "question": "Which of the following, if true, would most help to explain NorthAir ' s decision in light of its objectives?", "answers": "['The number of people who would be willing to pay the high fares NorthAir charges for its business-class seats would decrease if its economy-class seating were more acceptable.', \"A few of NorthAir' s economy-class passengers are satisfied with the service they receive, given the low price they pay.\", 'Very few people avoid flying on NorthAir because of the cramped seating and poor service offered in economy class.', \"None of NorthAir' s competitors offers significantly better seating and service to economy-class passengers than NorthAir does.\"]", "label": 0 }, { "id": "train_4497", "context": "A study found that patients referred by their doctors to psychotherapists practicing a new experimental form of therapy made more progress with respect to their problems than those referred to psychotherapists practicing traditional forms of therapy. Therapists practicing the new form of therapy, ttherefore, are more effective than therapists practicing traditional forms.", "question": "Which one of the following most accurately describes a flaw in the argument?", "answers": "['It ignores the possibility that the patients referred to therapists practicing the new form of therapy had problems more amenable to treatment than did those referred to therapists practicing traditional forms.', 'It ignores the possibility that therapists practicing the new form of therapy systematically differ from therapists practicing traditional forms of therapy with regard to some personality attribute relevant to effective treatment.', 'It presumes, without providing justification, that the personal rapport between therapist and patient has no influence on the effectiveness of the treatment the patient receives.', 'It ignores the possibility that therapists trained in traditional forms of therapy use the same techniques in treating their patients as therapists trained in the new form of therapy do.']", "label": 0 }, { "id": "train_4498", "context": "In comparing different methods by which a teacher' s performance can be evaluated and educational outcomes improved, researchers found that a critique of teacher performance leads to enhanced educational outcomes if the critique is accompanied by the information that teacher performance is merely one of several factors that, in concert with other factors, determines the educational outcomes.", "question": "Which one of the following best illustrates the principle illustrated by the finding of the researchers?", "answers": "['People are generally more willing to accept a negative characterization of a small group of people if they do not see themselves as members of the group being so characterized.', 'It is easier to correct the mistakes of an athlete if it is made clear to him that the criticism is part of an overarching effort to rectify the shortcomings of the entire team on which he plays.', 'An actor can more effectively evaluate the merits of her own performance if she can successfully convince herself that she is really evaluating the performance of another actor.', 'Children can usually be taught to master subject matter in which they have no interest if they believe that successfully mastering it will earn the respect of their peers.']", "label": 1 }, { "id": "train_4499", "context": "Editorialist: Some people argue that ramps and other accommodations for people using wheelchairs are unnecessary in certain business areas because those areas are not frequented by wheelchair users. What happens, however, is that once ramps and other accommodations are installed in these business areas, people who use wheelchairs come there to shop and work.", "question": "Which one of the following is most strongly supported by the editorialist's statements?", "answers": "['Businesses that install proper accommodations for wheelchair users have greater profits than those that do not.', 'Some business areas are not frequented by wheelchair users because the areas lack proper accommodations.', 'Most businesses are not modified to accommodate wheelchair users.', 'Owners of business areas not frequented by wheelchair users generally are reluctant to make modifications.']", "label": 1 }, { "id": "train_4500", "context": "When soil is plowed in the spring, pigweed seeds that have been buried in the soil all winter are churned up to the surface and redeposited just under the surface. The brief exposure of the seeds to sunlight stimulates receptors, which have become highly sensitive to sunlight during the months the seeds were buried in the soil, and the stimulated receptors trigger germination. Without the prolonged darkness, followed by exposure to sunlight, the seeds do not germinate.", "question": "The statements above, if true, most strongly support which one of the following statements about a field that will be plowed in the spring and in which pigweed seeds have been buried in the soil all winter?", "answers": "['Fewer pigweed plants will grow in the field if it is plowed at all than if it is plowed only at night.', 'Fewer pigweed plants will grow in the field if it is plowed only at night than if it is plowed during the day.', 'Fewer pigweed plants will grow in the field if it is plowed just before sunrise than if it is plowed just after sunset.', 'The pigweed seeds that are churned up to the surface of the soil during the plowing will not geminate unless they are redeposited under the surface of the soil.']", "label": 1 }, { "id": "train_4501", "context": "Most disposable plastic containers are now labeled with a code number (from 1 to 9) indicating the type or quality of the plastic. Plastics with the lowest code numbers are the easiest for recycling plants to recycle and are thus the most likely to be recycled after use rather than dumped in landfills. Plastics labeled with the highest numbers are only rarely recycled. Consumers can make a significant long-term reduction in the amount of waste that goes unrecycled, ttherefore, by refusing to purchase those products packaged in plastic containers labeled with the highest code numbers.", "question": "Which one of the following, if true, most seriously undermines the conclusion above?", "answers": "['Communities that collect all discarded plastic containers for potential recycling later dump in landfills plastics with higher-numbered codes only when it is clear that no recycler will take them.', 'A plastic container almost always has a higher code number after it is recycled than it had before recycling because the recycling process causes a degradation of the quality of the plastic.', 'The cost of collecting, sorting, and recycling discarded plastics is currently higher than the cost of manufacturing new plastics from virgin materials.', 'Many consumers are unaware of the codes that are stamped on the plastic containers.']", "label": 1 }, { "id": "train_4502", "context": "At HappyDay farms, seasonal staff is employed to pick apples during the harvest season each year. This staff is trained to identify and select only high quality apples, table-worthy apples. As part of this process, apples that have been visibly damaged by insects or disease are left to rot in the fields. The farmer who owns the property and sells the fruit is, ttherefore, less well informed about potential problems with his crops, such as insect infestations, than are the pickers themselves.", "question": "The conclusion drawn above is based on the assumption that", "answers": "['the farmer obtains all of his information about problems with fruit quality from no source other than the pickers.', 'the farmer, concerned with maximizing profits from sales, would want fewer apples left to rot in the field.', 'the pickers are more qualified to assess the quality of the fruit sold by the farmer than is the farmer.', 'during a complete year, the farmer spends more time in the field than any single seasonal picker.']", "label": 0 }, { "id": "train_4503", "context": "In the last year, biologists have learned that there are many more species of amphibians in existence than had previously been known. This definitely undermines environmentalists' claim that pollution is eliminating many of these species every year.", "question": "The reasoning in the argument above is most vulnerable to criticism on the grounds that it involves a confusion between", "answers": "['a correlation between two phenomena and a causal relationship between them', 'changes in our knowledge of objects and changes in the objects themselves', 'a cause and an effect', 'a condition necessary for a phenomenon and one that is sufficient for it']", "label": 1 }, { "id": "train_4504", "context": "Using fossil energy more efficiently is in the interest of the nation and the global environment, but major improvements are unlikely unless proposed government standards are implemented to eliminate products or practices that are among the least efficient in their class. Objection: Decisions on energy use are best left to the operation of the market.", "question": "Which one of the following, if true, most directly undermines the objection above?", "answers": "['Often the purchaser of a product, such as a landlord buying an appliance, chooses on the basis of purchase price because the purchaser is not the person who will pay for energy used by the product.', 'When energy prices rise, consumers of energy tend to look for new ways to increase energy efficiency, such as by adding insulation to their houses.', 'Whenever a new mode of generating energy, such as a new fuel, is introduced, a number of support systems, such as a fuel-distribution system, must be created or adapted.', 'It would be unrealistic to expect society to make the changes necessary to achieve maximum energy efficiency all at once.']", "label": 0 }, { "id": "train_4505", "context": "Automobile manufacturers who began two decades ago to design passenger vehicles that were more fuel-efficient faced a dilemma in the fact that the lighter, more efficient vehicles were less safe on high-speed highways. However, the manufacturers avoided this dilemma by producing two types of passenger vehicles: a lighter vehicle for medium-speed, local transportation, and a heavier, safer vehicle for long-distance travel. Since most automobile traffic is local, a net savings in fuel use was achieved with no loss in safety.", "question": "Which one of the following, if true, most strengthens the argument?", "answers": "['Most high-speed highways are used by both commercial vehicles and passenger vehicles.', 'Most households whose members do any long- distance driving own at least two passenger vehicles.', 'Some automobile manufacturers designed prototypes for fuel-efficient passenger vehicles more than two decades ago.', 'There are more cars using high-speed highways today than there were two decades ago.']", "label": 1 }, { "id": "train_4506", "context": "Treatment for hypertension forestalls certain medical expenses by preventing strokes and heart disease. Yet any money so saved amounts to only one-fourth of the expenditures required to treat the hypertensive population. Ttherefore, there is no economic justification for preventive treatment for hypertension.", "question": "Which of the following, if true, is most damaging to the conclusion above?", "answers": "['In matters of health care. Economic considerations should ideally not be dominant.', 'The many fatal strokes and heart attacks resulting from untreated hypertension cause insignificant medical expenditures but large economic losses of other sorts.', 'The net savings in medical resources achieved by some preventive health measures are smaller than the net losses attributable to certain other measures of this kind.', 'The cost, per patient, of preventive treatment for hypertension would remain constant even if such treatment were instituted on a large scale.']", "label": 1 }, { "id": "train_4507", "context": "When demand for a factory's products is high, more money is spent at the factory for safety precautions and machinery maintenance than when demand is low. Thus the average number of on-the-job accidents per employee each month should be lower during periods when demand is high than when demand is low and less money is available for safety precautions and machinery maintenance.", "question": "Which of the following, if true about a factory when demand for its products is high, casts the most serious doubt on the conclusion drawn above?", "answers": "['Its management hires new workers but lacks the time to train them properly.', 'Its management sponsors a monthly safety award for each division in the factory.', 'Its employees ask for higher wages than they do at other times.', 'Its old machinery is replaced with modern, automated models.']", "label": 0 }, { "id": "train_4508", "context": "Birds startled by potential predators generally try to take cover in nearby vegetation. Yet many birds that feed at bird feeders placed in suburban gardens are killed when, thus startled, they fly away from the vegetation in the gardens and into the windowpanes of nearby houses.", "question": "Which one of the following, if true, most helps to explain the anomalous behavior of the birds that fly into windowpanes?", "answers": "['Most of the birds startled while feeding at bird feeders placed in suburban gardens are startled by loud noises rather than by predators.', 'The windowpanes of many houses clearly reflect surrounding vegetation.', 'The bird feeders in some suburban gardens are placed at a considerable distance from the houses.', 'Predator attacks are as likely to occur at bird feeders surrounded by dense vegetation as they are at feeders surrounded by little or no vegetation.']", "label": 1 }, { "id": "train_4509", "context": "There can be no individual freedom without the rule of law, for there is no individual freedom without social integrity, and pursuing the good life is not possible without social integrity.", "question": "The conclusion drawn above follows logically if which one of the following is assumed?", "answers": "['One cannot pursue the good life without the rule of law.', 'There can be no social integrity without the rule of law.', 'Social integrity is possible only if individual freedom prevails.', 'There can be no rule of law without social integrity.']", "label": 1 }, { "id": "train_4510", "context": "Outsiders in any field often believe that they can bring in fresh, useful solutions that have been overlooked by insiders. But in fact, attempts at creativity that are not grounded in relevant experience are futile. Problems can be solved only by people who really understand them, and no one gains such understanding without experience.", "question": "Which one of the following is most strongly supported by the information above?", "answers": "['Those people who are experienced in a field rarely overlook creative solutions.', \"The experience required for effective problem-solving in a field does not vary depending on the field's complexity.\", 'The more experience a person has in a field, the more creatively that person can solve problems in the field.', 'Creative solutions in a field always come from people with experience in that field.']", "label": 3 }, { "id": "train_4511", "context": "Educator: Reducing class sizes in our school district would require hiring more teachers. However, there is already a shortage of qualified teachers in the region. Although students receive more individualized instruction when classes are smaller, education suffers when teachers are underqualified. Ttherefore, reducing class sizes in our district would probably not improve overall student achievement.", "question": "Which one of the following is an assumption required by the educator's argument?", "answers": "[\"Qualified teachers could not be persuaded to relocate in significant numbers to the educator's region to take teaching jobs.\", 'Students place a greater value on having qualified teachers than on having smaller classes.', 'At least some qualified teachers in the school district would be able to improve the overall achievement of students in their classes if class sizes were reduced.', 'Class sizes in the school district should be reduced only if doing so would improve overall student achievement.']", "label": 0 }, { "id": "train_4512", "context": "Politician: From the time our party took office almost four years ago the number of people unemployed city-wide increased by less than 20 percent. The opposition party controlled city government during the four preceding years, and the number of unemployed city residents rose by over 20 percent. Thus, due to our leadership, fewer people now find themselves among the ranks of the unemployed, whatever the opposition may claim.", "question": "The reasoning in the politician's argument is most vulnerable to the criticism that", "answers": "['no evidence has been offered to show that any decline in unemployment over the past four years was uniform throughout all areas of the city', 'the evidence cited in support of the conclusion actually provides more support for the denial of the conclusion', 'the possibility has not been addressed that any increase in the number of people employed is due to programs supported by the opposition party', 'the issue of how much unemployment in the city is affected by seasonal fluctuations is ignored']", "label": 1 }, { "id": "train_4513", "context": "University spokesperson: Most of the students surveyed at the university said they would prefer that the current food vendor be replaced with a different food vendor next year. Several vendors have publicly expressed interest in working for the university. For a variety of reasons, however, the only alternative to the current vendor is Hall Dining Services, which served as the university' s food vendor up until this past year. Since, other things being equal, the preferences of the majority of students should be adhered to, we should rehire Hall Dining next year.", "question": "The spokesperson's argument is most vulnerable to criticism on the grounds that it", "answers": "['argues that a certain action ought to be undertaken merely on the grounds that it would be popular', 'overlooks the possibility that student preference is not the only factor to be considered when it comes to deciding which food vendor the university should hire', 'overlooks the possibility that there is disagreement among students concerning the issue of food vendors', 'overlooks the possibility that the students surveyed were unaware that only Hall Dining Services could be hired if the current vendor were not hired']", "label": 3 }, { "id": "train_4514", "context": "Researchers have found that children in large families -- particularly the younger siblings -- generally have fewer allergies than children in small families do. They hypothesize that exposure to germs during infancy makes people less likely to develop allergies.", "question": "Which one of the following, if true, most supports the researchers' hypothesis?", "answers": "['Children whose parents have allergies have an above-average likelihood of developing allergies themselves.', 'Children from small families who entered day care before age one were less likely to develop allergies than children from small families who entered day care later.', 'Children in small families generally eat more kinds of very allergenic foods than children in large families do.', 'Some allergies are life threatening, while many diseases caused by germs produce only temporary discomfort.']", "label": 1 }, { "id": "train_4515", "context": "Paternalism is the practice by some governments of imposing regulations meant to promote their citizens' own good regardless of whether the citizens consent to those regulations. However, many members of free societies disapprove when individual freedom is curtailed for the sake of what the government deems to be the citizens' own good. They generally believe that they, not the government, know what activities are in their best interest.", "question": "If the statements above are true, which one of the following must also be true?", "answers": "[\"In free societies, many of a government's citizens disapprove of their government's acts of paternalism.\", 'The good of citizens is usually not advanced by the practice of paternalism.', 'The goals of free societies and the goals of their citizens always conflict.', \"In free societies, many of a government's citizens know what activities are in their own best interests better than their government does.\"]", "label": 0 }, { "id": "train_4516", "context": "Last year all refuse collected by Shelbyville city services was incinerated. This incineration generated a large quantity of residual ash. In order to reduce the amount of residual ash Shelbyville generates this year to half of last year's total, the city has revamped its collection program. This year city services will separate for recycling enough refuse to reduce the number of truckloads of refuse to be incinerated to half of last year's number.", "question": "Which of the following is required for the revamped collection program to achieve its aim?", "answers": "['The refuse incinerated this year will generate no more residual ash per truckload incinerated than did the refuse incinerated last year.', 'The total quantity of refuse collected by Shelbyville city services this year will be no greater than that collected last year.', 'This year, no materials that city services could separate for recycling will be incinerated.', 'Refuse collected by city services will contain a larger proportion of recyclable materials this year than it did last year.']", "label": 0 }, { "id": "train_4517", "context": "Jones is selling a house to Smith. The contract between the two specifies that for up to a year after ownership is transferred, Jones will be responsible for repairing any \"major structural defects, \" defined as defects in the roof or roof-supporting components of the house, that might be found. Jones is not responsible for any other repairs. The house has a truss roof, which means that the only walls that support the roof are the exterior walls.", "question": "It can be properly concluded from the information above that", "answers": "[\"the contract does not oblige Jones to repair any defects in the house's nonexterior walls after ownership of the house has been transferred\", 'Jones did not know of any defects in the roof or roof-supporting components of the house at the time the contract was written', 'Smith will be obligated to repair all structural defects in the house within a year after ownership is transferred, except those for which Jones is responsible', 'although other components of the house may contain defects, the roof and roof-supporting components of the house are currently free from such defects']", "label": 0 }, { "id": "train_4518", "context": "In economics, a \"Veblen good\" is an item which people buy in greater quantity when the price goes up. According to the Law of Supply and Demand, when the price of an ordinary item goes up, demand drops, i. e. people buy fewer of them. A Veblen good is a luxury item to which status is attached, such as a designer outfit or luxury car. As the price of such an item increases, its prestige increases, which makes the item that much more desirable. Ttherefore, as the price increases, the demand also increases, and more of these items are sold. ", "question": "In the argument, the two portions in boldface play which of the following roles?", "answers": "['The first is a view that the author opposes; the second is the main conclusion.', 'The first is an ironic description of what could happen in the marketplace; the second is a more realistic description of what actually happens.', 'This first is a piece of evidence supporting the main conclusion; the second is the main conclusion.', 'The first is a general rule, the violation of which seems to indicate a paradox; the second is the resolution of that apparent paradox.']", "label": 3 }, { "id": "train_4519", "context": "Columnist: If you received an unsigned letter, you would likely have some doubts about the truth of its contents. But news stories often include statements from anonymous sources, and these are usually quoted with the utmost respect. It makes sense to be skeptical of these sources, for, as in the case of the writer of an unsigned letter, their anonymity makes it possible for them to plant inaccurate or slanted statements without ever having to answer for them.", "question": "The columnist's argument proceeds by", "answers": "['calling into question the motives of those presenting certain information, and concluding for this reason that the information is likely to be false', 'calling into question a certain type of evidence by drawing an analogy between that evidence and other evidence that the argument shows is usually false', 'inferring that an attitude would be justified in all situations of a given type on the grounds that this attitude is justified in a hypothetical situation of that type', 'pointing out that a certain attitude would presumably be adopted in one situation, in order to support the claim that a similar attitude would be justified in an analogous situation']", "label": 3 }, { "id": "train_4520", "context": "Crimes in which handguns are used are more likely than other crimes to result in fatalities. However, the majority of crimes in which handguns are used do not result in fatalities. Ttherefore, there is no need to enact laws that address crimes involving handguns as distinct from other crimes.", "question": "The pattern of flawed reasoning displayed in the argument above most closely resembles that in which one of the following?", "answers": "['Most physicians recommend a balanced diet for those who want to remain in good health. Yet many people find that nontraditional dietary regimens such as extended fasting do their health no serious harm. Ttherefore, there is no need for everyone to avoid nontraditional dietary regimens.', 'Overweight people are at higher risk of developing heart disease than other people. However, more than half of all overweight people never develop heart disease. Hence it is unnecessary for physicians to be more careful to emphasize the danger of heart disease to their overweight patients than to their other patients.', 'Many people swim daily in order to stay physically fit. Yet people who swim daily increase their risk of developing ear infections. Hence people who want to remain in good health are better off not following fitness programs that include swimming daily.', 'Foods rich in cholesterol and fat pose a serious health threat to most people. However, many people are reluctant to give up eating foods that they greatly enjoy. Ttherefore, people who refuse to give up rich foods need to spend more time exercising than do other people.']", "label": 1 }, { "id": "train_4521", "context": "Ethicist: Both ASA and TPA are clot-dissolving agents. Recent studies show that the more expensive agent, TPA, would save at most two more lives than would ASA out of every 50 cardiac patients to whom they are postoperatively administered. However, since the relatives of the patients who die simply because they were given the less expensive medicine would be particularly grieved, the financial saving involved in using ASA over TPA must also be weighed against such considerations.", "question": "Which one of the following most accurately expresses the conclusion of the ethicist's argument?", "answers": "['TPA is a slightly more effective clot-dissolving agent than ASA.', 'Relatives of a patient who has died grieve more if the patient received ASA rather than TPA.', 'The extra expense of TPA cannot be weighed simply against the few additional lives saved.', 'ASA should never be given to postoperative cardiac patients in place of TPA.']", "label": 2 }, { "id": "train_4522", "context": "Landscape architect: If the screen between these two areas is to be a hedge, that hedge must be of either hemlocks or Leyland cypress trees. However, Leyland cypress trees cannot be grown this far north. So if the screen is to be a hedge, it will be a hemlock hedge.", "question": "In which one of the following is the pattern of reasoning most similar to that in the landscape architect's argument?", "answers": "['A space the size of this meadow would be suitable for a playground or a picnic area. However, a playground would be noisy and a picnic area would create litter. So it will be best for the area to remain a meadow.', 'Any path along this embankment must be either concrete or stone. But a concrete path cannot be built in this location. So if there is to be a path on the embankment, it will be a stone path.', 'If there is to be an entrance on the north side of the building, it will have to be approached by a ramp. However, a ramp would become impossibly slippery in winter, so there will be no entrance on the north side.', 'If visitors are to travel to this part of the site by automobile, there will be a need for parking spaces. However, no parking spaces are allowed for in the design. So if visitors are likely to come by automobile, the design will be changed.']", "label": 1 }, { "id": "train_4523", "context": "Economist: No economic system that is centrally planned can efficiently allocate resources, and efficient allocation of resources is a necessary condition for achieving a national debt of less than 5 percent of Gross Domestic Product (GDP). It follows that any nation with a centrally planned economy has a national debt that is at least 5 percent of GDP.", "question": "The pattern of reasoning exhibited by the economist's argument is most similar to that exhibited by which one of the following?", "answers": "['Every mutual fund manager knows someone who trades on inside information, and no one who trades on inside information is unknown to every mutual fund manager. One must conclude that no mutual fund manager is unknown to everyone who trades on inside information.', 'All of the rural districts are free of major air pollution problems because such problems occur only where there is a large concentration of automobiles, and there are no such places in the rural districts.', 'All rock stars who are famous have their own record companies, and all rock stars with their own record companies receive company profits over and above their regular royalties. This implies that receiving large regular royalties is a necessary condition of being a famous rock star.', 'All of the ungulates are herbivores, and most herbivores would not attack a human being. It follows that any animal that would attack a human being is unlikely to be an ungulate.']", "label": 1 }, { "id": "train_4524", "context": "Magazine article: The Environmental Commissioner' s new proposals are called \"Fresh Thinking on the Environment, \"and a nationwide debate on them has been announced. Well, \"fresh thinking\"from such an unlikely source as the commissioner does deserve closer inspection. Unfortunately we discovered that these proposals are virtually identical to those issued three months ago by Tsarque Inc. under the heading \"New Environmentalism\" (Tsarque Inc. ' s chief is a close friend of the commissioner). Since Tsarque Inc. ' s polluting has marked it as an environmental nightmare, in our opinion the \"nationwide debate\" can end here.", "question": "A flaw in the magazine article's reasoning is that it", "answers": "['dismisses the proposals because of their source rather than because of their substance', 'uses emotive language in labeling the proposals', 'assumes without any justification that since two texts are similar one of them must be influenced by the other', \"appeals to the authority of Tsarque Inc. 's chief without giving evidence that this person's opinion should carry special weight\"]", "label": 0 }, { "id": "train_4525", "context": "In two months, the legal minimum wage in the country of Kirlandia will increase from five Kirlandic dollars(KD5. 00) Per hour to KD5. 50 per hour. Opponents of this increase have argued that the resulting rise in wages will drive the inflation rate up. In fact its impact on wages will probably be negligible, since only a very small proportion of all Kirfandic workers are currently receiving less than KD5. 50 per hour.", "question": "Which of the following, if true, most seriously weakens the argument?", "answers": "['Most people in kirlandia who are currently earning the minimum wage have been employed at their current jobs for less than a year.', 'Some firms in Kirlandia have paid workers considerably less than KD5. 00 per hour, in violation of kirlandic employment regulations.', \"The greatest growth in Kirlandia's economy in recent years has been in those sectors where workers earn wages that tend to be much higher than the minimum wage.\", 'Many businesses hire trainees at or near the minimum wage but must reward trained workers by keeping their paylevels above the pay level of trainees.']", "label": 3 }, { "id": "train_4526", "context": "Asbestos, an almost indestructible mineral once installed as building insulation, poses no health risk unless the asbestos is disturbed and asbestos fibers are released into the environment. Since removing asbestos from buildings disturbs it, thereby releasing asbestos fibers, the government should not require removal of all asbestos insulation.", "question": "Which one of the following, if true, most strengthens the argument?", "answers": "['Some kinds of asbestos, when disturbed, pose greater health risks than do other kinds.', 'Much of the time, removed asbestos is buried in landfills and forgotten, with no guarantee that it will not be disturbed again.', 'Asbestos can pose a health threat to workers who remove it without wearing required protective gear.', 'Asbestos poses far less risk to health than does smoking, drug and alcohol abuse, improper diet, or lack of exercise.']", "label": 1 }, { "id": "train_4527", "context": "Caterpillars of all species produce an identical hormone called \"juvenile hormone\" that maintains feeding behavior. Only when a caterpillar has grown to the right size for pupation to take place does a special enzyme halt the production of juvenile hormone. This enzyme can be synthesized and will, on being ingested by immature caterpillars, kill them by stopping them from feeding.", "question": "Which of the following, if true, most strongly supports the view that it would NOT be advisable to try to eradicate agricultural pests that go through a caterpillar stage by spraying croplands with the enzyme mentioned above?", "answers": "['Many agriculturally beneficial insects go through a caterpillar stage.', 'Since caterpillars of different species emerge at different times, several sprayings would be necessary.', 'Many agricultural pests do not go through a caterpillar stage.', 'Although the enzyme has been synthesized in the laboratory, no large-scale production facilities exist as yet.']", "label": 0 }, { "id": "train_4528", "context": "Ordinary mountain sickness, a common condition among mountain climbers, and one from which most people can recover, is caused by the characteristic shortage of oxygen in the atmosphere at high altitudes. Cerebral edema, a rarer disruption of blood circulation in the brain that quickly becomes life-threatening if not correctly treated from its onset, can also be caused by a shortage of oxygen. Since the symptoms of cerebral edema resemble those of ordinary mountain sickness, cerebral edema is especially dangerous at high altitudes.", "question": "Which one of the following is an assumption on which the argument depends?", "answers": "['Unlike cerebral edema, ordinary mountain sickness involves no disruption of blood circulation in the brain.', 'The treatment for ordinary mountain sickness differs from the treatment for cerebral edema.', 'Most people who suffer from ordinary mountain sickness recover without any special treatment.', 'Cerebral edema can cause those who suffer from it to slip into a coma within a few hours.']", "label": 1 }, { "id": "train_4529", "context": "When a certain gland becomes cancerous in humans, it produces high levels of a particular protein. A blood test can determine the level of this protein well before a cancer of the gland could be detected by other means. Some doctors recommend that aggressive anticancer treatment should be begun as early as possible for anyone who is tested and is found to have high levels of the protein.", "question": "Which one of the following, if true, most seriously weakens the doctors' recommendation?", "answers": "['The blood test for the protein has been in use for some time to monitor the condition of patients who have been diagnosed as having cancer of the gland.', 'Before the blood test became available, about one third of all cases of cancer of the gland were detected in early stages.', 'So far, no patients whose protein levels were found to be normal have subsequently developed cancer of the gland.', 'Enlargement of the gland, a common condition infrequently associated with cancer, results in high levels of the protein.']", "label": 3 }, { "id": "train_4530", "context": "Editorial: One of our local television stations has been criticized for its recent coverage of the personal problems of a local politician' s nephew, but the coverage was in fact good journalism. The information was accurate. Furthermore, the newscast had significantly more viewers than it normally does, because many people are curious about the politician' s nephew' s problems.", "question": "Which one of the following principles, if valid, would most help to justify the reasoning in the editorial?", "answers": "['Any journalism that intentionally misrepresents the facts of a case deserves to be criticized.', \"Journalism that neither satisfies the public's curiosity nor provides accurate information can never be considered good journalism.\", 'Good journalism will always provide people with information that they desire or need.', 'Any journalism that provides accurate information on a subject about which there is considerable interest is good journalism.']", "label": 3 }, { "id": "train_4531", "context": "Braille is a method of producing text by means of raised dots that can be read by touch. A recent development in technology will allow flat computer screens to be made of a material that can be heated in patterns that replicate the patterns used in braille. Since the thermal device will utilize the same symbol system as braille, it follows that anyone who is accustomed to reading braille can easily adapt to the use of this electronic system.", "question": "Which one of the following is an assumption on which the conclusion depends?", "answers": "['People with the tactile ability to discriminate symbols in braille have an ability to discriminate similar patterns on a flat heated surface.', 'Only people who currently use braille as their sole medium for reading text will have the capacity to adapt to the use of the thermal screen.', 'Eventually it will be possible to train people to read braille by first training them in the use of the thermal screen.', 'Some symbol systems encode a piece of text by using dots that replicate the shape of letters of the alphabet.']", "label": 0 }, { "id": "train_4532", "context": "Balance is particularly important when reporting the 5. background of civil wars and conflicts. Facts must not be deliberately manipulated to show one party in a favorable light, and the views of each side should be fairly represented. This concept of balance, however, does not justify concealing or glossing over basic injustices in an effort to be even-handed. If all the media were to adopt such a perverse interpretation of balanced reporting, the public would be given a picture of a world where each party in every conflict had an equal measure of justice on its side, contrary to our experience of life and, indeed, our common sense.", "question": "Which one of the following best expresses the main point of the argument?", "answers": "['Balanced reporting presents the public with a picture of the world in which all sides to a conflict have equal justification.', 'Balanced reporting is an ideal that cannot be realized, because judgments of balance are necessarily subjective.', 'Common sense tells us that balance is especially needed when reporting the background of civil wars and conflicts.', 'Balanced reporting requires impartially revealing injustices where they occur no less than fairly presenting the views of each party in a conflict.']", "label": 3 }, { "id": "train_4533", "context": "Zachary: The term \"fresco\" refers to paint that has been applied to wet plaster. Once dried, a fresco indelibly preserves the paint that a painter has applied in this way. Unfortunately, additions known to have been made by later painters have obscured the original fresco work done by Michelangelo in the Sistine Chapel. Ttherefore, in order to restore Michaelangelo' s Sistine Chapel paintings to the appearance that Michelangelo intended them to have, everything except the original fresco work must be stripped away. Stephen: But it was extremely common for painters of Michelangelo' s era to add painted details to their own fresco work after the frescos had dried.", "question": "Stephen's response to Zachary proceeds by", "answers": [ "denying the truth of one of the stated premises of Zachary's argument", "demonstrating the Zachary's conclusion is not consistent with the premises he uses to support it", "challenging the definition of a key term in Zachary's argument", "calling into question an assumption on which Zachary's conclusion depends" ], "label": 3 }, { "id": "train_4534", "context": "A city plans to attract new citizens with new housing and new facilities such as parks, recreation centers, and libraries. One component of the city' s plan is to require that developers seeking permission to build this new housing provide these additional facilities at no cost to the city.", "question": "Which of the following, if true, would point to a possible flaw in the city's plan?", "answers": "['Other towns and cities nearby have yet to embark on any comparable plans to attract new citizens.', 'Most developers see the extra expense of providing municipal facilities as simply one of the many costs of doing business.', 'Studies show that purchasers of new houses, especially first-time buyers, rank recreational resources as an important factor in deciding to buy a particular house.', 'Developers would pass along their costs to the buyer, thereby raising the cost of housing units beyond the ability of likely purchasers to afford them.']", "label": 3 }, { "id": "train_4535", "context": "A recently completed study of several hundred subjects, all of approximately the same age, showed that those who exercised regularly during the study were much less likely to die during the study. This indicates that exercise can actually increase one' s life span. .", "question": "Which one of the following, if true, most strengthens the argument?", "answers": "['The subjects who did not exercise regularly during the study tended to have diets that were more unhealthy.', 'A person who exercises regularly is probably doing so out of concern for his or her own health.', 'A large number of the deaths recorded were attributable to preexisting conditions or illnesses.', 'Whether or not a given subject was to exercise during the study was determined by the researchers on a random basis.']", "label": 3 }, { "id": "train_4536", "context": "John looks like a professional bodybuilder. He weighs 210 pounds and stands six feet tall, which is the size of an NFL linebacker. John looks huge when he enters the room. Years of gym time have clearly paid off in spades.", "question": "Which of the following, if true, weakens the argument?", "answers": "[\"John's father, brothers, and male cousins all look like professional bodybuilders, and none of them have ever worked out.\", 'John works out five times every week.', 'John weighed considerably less before he started working out.', 'The average professional bodybuilder is considerably heavier and taller than the average NFL linebacker.']", "label": 0 }, { "id": "train_4537", "context": "If you have no keyboarding skills at all, you will not be able to use a computer. And if you are not able to use a computer, you will not be able to write your essays using a word processing program.", "question": "If the statements above are true, which one of the following must be true?", "answers": "['If you are not able to write your essays using a word processing program, you have no keyboarding skills.', 'If you are able to write your essays using a word processing program, you have at least some keyboarding skills.', 'If you are not able to write your essays using a word processing program, you are not able to use a computer.', 'If you have some keyboarding skills, you will be able to write your essays using a word processing program.']", "label": 1 }, { "id": "train_4538", "context": "Columnist: The country is presently debating legislation that, if passed, would force manufacturers to increase the number of paid vacation days for employees, to pay higher overtime wages, and to pay all day-care expenses for children of each employee. This legislation is being supported by members of groups that have resorted to violent tactics in the past, and by individuals who are facing indictment on tax-evasion charges. We must defeat this legislation and what it stands for.", "question": "The columnist's argument is flawed because it", "answers": "['assails legislation on the basis of the questionable character of supporters of the legislation', 'attacks legislation by calling into question the integrity of the originators of the legislation', 'attempts to discredit legislation by appealing to public sentiment for those who would be adversely affected', \"presupposes that legislation is bad legislation whenever it has only a small number of supporters outside the country's national legislative body\"]", "label": 0 }, { "id": "train_4539", "context": "Professor: Different countries have different economic systems, but all economic systems have prosperity as their primary goal. Because people all want the same thing, there cannot be radical disagreement among people from different economic systems about practical economic matters. Thus all apparent disagreement in practical economic issues is illusory.", "question": "Which one of the following most accurately describes a reasoning error in the professor's argument?", "answers": "['The argument contradicts itself about whether there are in fact differences between economic systems in different countries.', 'The argument ignores the possibility that groups of people may share the same goal but not agree about how best to achieve that goal.', 'The argument bases a general conclusion about economic systems on one example of such a system, which there is reason to think is atypical.', 'The argument depends on using the key term \"economic system\" in two incompatible senses.']", "label": 1 }, { "id": "train_4540", "context": "Essayist: When the first prehistoric migrations of humans from Asia to North America took place, the small bands of new arrivals encountered many species of animals that would be extinct only 2, 000 years later. Since it is implausible that hunting by these small bands of humans could have had such an effect, and since disease-causing microorganisms not native to North America were undoubtedly borne by the new arrivals as well as by the animals that followed them, these microorganisms were probably the crucial factor that accounts for the extinctions.", "question": "Which one of the following, if true, most weakens the essayist's argument?", "answers": "['Very few species of North American animals not hunted by the new arrivals from Asia were extinct 2, 000 years after the first migrations.', 'Individual humans and animals can carry a disease-causing microorganism without themselves suffering from the disease.', 'Animals weakened by disease are not only less able to avoid hunters but are also less able to avoid their other predators.', 'Human beings generally have a substantial degree of biological immunity to the diseases carried by other species.']", "label": 0 }, { "id": "train_4541", "context": "The prevailing view among historians is that medieval European peasants were deeply dedicated to religion . But the record keepers in medieval Europe were members of the clergy. It should not surprise us that chroniclers who were professionally occupied with religion would exaggerate people' s religious devotion. Thus, there is reason to doubt the prevailing view of medieval peasants' piety.", "question": "Which one of the following, if true, would most weaken the argument above?", "answers": "['Many members of the clergy in medieval Europe spent more time among peasants than among people of any other social class.', 'Documents produced by clergy in medieval Europe contain detailed descriptions of participation by peasants and others in religious activities .', 'Historians cannot develop a reliable account of the religious attitudes held during any given historical period unless they have consulted all of the relevant surviving records from that period .', 'Written records produced by clergy in medieval Europe very rarely portray merchants or nobles as being dedicated to religion.']", "label": 3 }, { "id": "train_4542", "context": "Scientists have shown that older bees, which usually forage outside the hive for food, tend to have larger brains than do younger bees, which usually do not forage but instead remain in the hive to tend to newly hatched bees. Since foraging requires greater cognitive ability than does tending to newly hatched bees, it appears that foraging leads to the increased brain size of older bees.", "question": "Which one of the following, if true, most seriously weakens the argument above?", "answers": "['The brains of older bees that stop foraging to take on other responsibilities do not become smaller after they stop foraging.', 'In some species of bees, the brains of older bees are only marginally larger than those of younger bees.', 'Those bees that travel a long distance to find food do not have significantly larger brains than do bees that locate food nearer the hive.', 'The brains of older bees that never learn to forage are the same size as those of their foraging counterparts of the same age.']", "label": 3 }, { "id": "train_4543", "context": "Marion knows that the bridge on her usual route is closed and that, unless she takes the train instead of driving, she can get to work on time only by leaving at least 45 minutes early. She must go to her neighborhood bank before leaving for work, however, and the bank will not be open in time for her to do so if she leaves 45 minutes early. Ttherefore, since she hates taking the train, Marion cannot avoid being late for work.", "question": "The reasoning in the argument is flawed because the argument", "answers": "['mistakes a situation that almost certainly affects many people for one that affects a particular person alone', 'overlooks the possibility that someone might occasionally adopt a given course of action without having a good reason for doing so', 'ignores the fact that people often know that something is the case without considering all the consequences that follow from its being the case', 'treats evidence that someone will adopt a particular course of action as though that evidence excluded the possibility of an alternative course of action']", "label": 3 }, { "id": "train_4544", "context": "In setting environmental standards for industry and others to meet, it is inadvisable to require the best results that state-of-the-art technology can achieve. Current technology is able to detect and eliminate even extremely minute amounts of contaminants, but at a cost that is exorbitant relative to the improvement achieved. So it would be reasonable instead to set standards by taking into account all of the current and future risks involved.", "question": "The argument given concerning the reasonable way to set standards presupposes that", "answers": "['minute amounts of some contaminants can be poisonous', 'the only contaminants worth measuring are generated by industry', 'industry currently meets the standards that have been set by environmental authorities', 'there are effective ways to take into account all of the relevant risks posed by allowing different levels of contaminants']", "label": 3 }, { "id": "train_4545", "context": "Fire ants from Brazil now infest the southern United States. Unlike queen fire ants in Brazil, two queens in the United States share a nest. Ants from these nests are more aggressive than those from single-queen nests. By destroying virtually all insects in the nest area, these aggressive ants gain sole access to food sources, and the ant population skyrockets. Since certain predator insects in Brazil limit the fire-ant population there, importing such predator insects into the United States would be of overall benefit to the environment by stopping the increase of the fire-ant population in the United States.", "question": "Each of the following is an assumption made in the argument EXCEPT:", "answers": "['The predator insects from Brazil could survive in the ecological environment found in the United States.', 'The imported insects would not prove more damaging to the environment in the United States than are the fire ants themselves.', 'The rate of increase of the fire-ant population would not exceed the rate at which the predator insects could kill the ants.', 'The predator insects would stop the increase of the ant population before the ants spread to states that are farther north.']", "label": 3 }, { "id": "train_4546", "context": "Anthropologist: After mapping the complete dominance hierarchy for a troupe of vervet monkeys by examining their pairwise interaction, we successfully predicted more complex forms of their group behavior by assuming that each monkey had knowledge of the complete hierarchy. Since our prediction was so accurate, it follows that the assumption we used to reach it was in fact true. Primatologist: Although I agree that your assumption helped you make those predictions, your conclusion does not follow. You might as well argue that since we can predict the output of some bank cash machines by assuming that these machines actually want to satisfy the customers' requests, these cash machines must really have desires.", "question": "The primatologist uses which one of the following argumentative techniques in countering the anthropologist's argument?", "answers": "[\"citing various facts that could not obtain if the anthropologist's conclusion were correct\", \"applying one of the anthropologist's reasoning steps in another argument in an attempt to show that it leads to an absurd conclusion\", 'offering another argument that has as its premise the denial of the thesis that the anthropologist defends', \"suggesting that the anthropologist's argument relies on a misinterpretation of a key scientific term\"]", "label": 1 }, { "id": "train_4547", "context": "Although free international trade allows countries to specialize, which in turn increases productivity, such specialization carries risks. After all, small countries often rely on one or two products for the bulk of their exports. If those products are raw materials, the supply is finite and can be used up. If they are foodstuffs, a natural disaster can wipe out a season' s production overnight.", "question": "Which one of the following most accurately expresses the conclusion of the argument as a whole?", "answers": "['Specialization within international trade comes with risks.', 'Some countries rely on a small number of products for the export-based sectors of their economies.', 'When international trade is free, countries can specialize in what they export.', \"A small country's supply of raw materials can be used up in a short period.\"]", "label": 0 }, { "id": "train_4548", "context": "Some studies show that students are more creative after being given monetary rewards for their academic performance, but other studies show that students are less creative after receiving monetary rewards. Ttherefore, to avoid the risk of discouraging students' creativity, monetary rewards should not be given to them for academic performance.", "question": "The reasoning in which one of the following is most similar to that in the argument above?", "answers": "['If the current model of the most popular low-priced car is redesigned to make it more powerful, then its price will be raised. A higher price for this model may lead to decreased sales. Ttherefore, this model should not be redesigned.', 'Studies are producing contrary results about the level of exercise that is optimal for maintaining fitness. Ttherefore, it is better to concentrate on diet as a means of maintaining fitness.', 'Life will be much more convenient for Javier if he buys a new car, but he will save a lot of money if he buys no car. Ttherefore, to have more convenience and save some money, Javier should buy a used car.', \"If Donna's supervisor learns that Donna is applying for another job, then the supervisor might offer Donna incentives to stay in her current position. However, the supervisor might react negatively to the news. Because of this possibility, Donna should keep her plans from her supervisor.\"]", "label": 3 }, { "id": "train_4549", "context": "The head baker at Barry's Bagels can either purchase flour in-person from the local flour mill, Larry's Local Mill, or order a shipment of flour from an out-of-state mill, Isadore's Interstate Mill. The cost of the flour from Isadore's Interstate Mill is 10 percent less than the cost of the flour from Larry's Local Mill. Even after shipping and handling fees are added, it is still cheaper to order flour that has to be shipped from Isadore's than to buy flour locally from Larry's.", "question": "The statements above, if true, best support which of the following assertions?", "answers": [ "The shipping and handling fees for a batch of flour purchased from Isadore's Interstate Mill are less than 10 percent of the cost of an identical batch of flour purchased from Larry's Local Mill.", "Isadore's Interstate Mill produces flour 10% more efficiently than Larry's Local Mill does.", "Production costs at Isadore's Interstate Mill are 10 percent below those at Larry's Local Mill.", "The shipping and handling fees for a batch of flour purchased from Isadore's Interstate Mill are more than 10 percent of the cost of Isadore's flour." ], "label": 0 }, { "id": "train_4550", "context": "President of the Smalltown Chamber of Commerce: In the past decade, the businesses in Smalltown have reported average profits of 15% above inflation. The mayor of Smalltown has explained this by citing decreased crime and a growing population, but he is incorrect. In reality, this increase in profitability is not due to these factors, but rather to the sharing of business acumen and market research that occurs at the monthly Chamber of Commerce meetings.", "question": "Which of the following, if true, best supports the President of the Smalltown Chamber of Commerce's attribution of the increased profits to her organization's meetings?", "answers": "[\"The growth of Smalltown's population has primarily been due to a higher birth rate, which has traditionally been associated with a drop in crime.\", 'Businesses in Smalltown whose members did not attend these monthly meetings regularly saw their profits increase only 2% above inflation.', 'The effects of taxes on local businesses may not be immediately apparent in all cases, particularly when populations are fluctuating.', 'Crime has decreased throughout the state, and the businesses in other towns with similar populations have seen similar profits in the past ten years.']", "label": 1 }, { "id": "train_4551", "context": "Most people who ride bicycles for pleasure do not ride until the warm weather of spring and summery arrives. Yet it is probably more effective to advertise bicycles earlier in the year. Most bicycles are purchased in the spring, but once shoppers are ready to shop for a bicycle, they usually have already decided which brand and model of bicycle they will purchase. By then it is generally too late to induce them to change their minds.", "question": "The main point of the argument is that", "answers": "['bicycle advertisements are probably more effective if they appear before the arrival of warm spring weather', 'most bicycle purchasers decide on the brand and model of bicycle that they will buy before beginning to shop for a bicycle', 'in general, once a bicycle purchaser has decided which bicycle he or she intends to purchase, it is difficult to bring about a change in that decision', 'spring and summer are the time of year in which bicycle riding as a leisure activity is most popular']", "label": 0 }, { "id": "train_4552", "context": "Journalist: A manufacturers' trade group that has long kept its membership list secret inadvertently sent me a document listing hundreds of manufacturing companies. A representative of the trade group later confirmed that every company listed in the document does indeed belong to the trade group. Because Bruch Industries is not listed on the document, it is evidently not a member of the trade group.", "question": "The journalist's reasoning in the argument is flawed in that the journalist", "answers": "['does not explain how it is that the trade group could have inadvertently sent out a secret document', 'does not present any evidence that the document names every member of the trade group', 'gives no reason to think that Bruch Industries would want to belong to the trade group', 'takes for granted the accuracy of a statement by a representative who had a reason to withhold information']", "label": 1 }, { "id": "train_4553", "context": "The legislature of the Philippines voted recently to abolish the death penalty. In contrast, the death penalty remains legal in the United States. The difference in the legality of capital punishment demonstrates that the majority of American citizens believe in the death penalty, while the majority of Filipino citizens do not.", "question": "Which of the following is an assumption upon which the above argument depends?", "answers": "['There are not strong voices opposing the death penalty in the United States.', 'The legal status of capital punishment in the United States and the Philippines aligns with how the majority of citizens in those respective countries view the death penalty.', 'There are more murders per year in the United States than in the Philippines.', 'The legal standard used to determine whether a criminal should be sentenced to the death penalty in the United States is similar to the legal standard used in the Philippines before capital punishment was abolished there.']", "label": 1 }, { "id": "train_4554", "context": "The Indonesian mimic octopus is able to camouflage itself from predators by blending into the background. Sometimes, though, even when a predator is nearby, the mimic octopus takes on a color pattern that is not present in the immediate environment. A marine biologist of cephalopods claims that this behavior is a sign of an octopus engaging in play, meaning that the behavior does not confer any specific advantage.", "question": "Which of the following, if true, would most call in to doubt the marine biologist's claim?", "answers": "[\"Marine biologists are not familiar with all of the environments that the Indonesian mimic octopuses inhabit, since it is known to swim into crevices deep in the ocean's floor.\", 'An observational study that tracked a mimic octopus over the course of a week found that it changed the color of its skin even when there were no predators in the area.', 'The Indonesian mimic octopus engages in other forms of behavior that seem to serve no purpose, yet several marine biologists are reluctant to describe such behavior as play.', \"The color pattern an Indonesian mimic octopus assumes when a predator is nearby, but that does not match its immediate environment, has been found to parallel that of various jellyfish venomous to the octopus's predators.\"]", "label": 3 }, { "id": "train_4555", "context": "When weeding a vegetable garden, one should not try to remove all the weeds. It is true that the more weeds, the less productive the garden. Nevertheless, avoiding the painstaking effort of finding and pulling every single weed more than compensates for the slight productivity loss resulting from leaving a few.", "question": "The principle underlying which one of the following arguments is most similar to the principle underlying the argument above?", "answers": [ "If one is trying to improve one's personality by removing imperfections, one should not try to remove them all. For while each imperfection makes one's personality worse, it is no longer worth one's time to remove imperfections if there are only a few left.", "It is a mistake to try to remove every imperfection from one's personality. Personality imperfections make life difficult sometimes, but people cannot be truly happy if their personalities lack defects.", "One who is trying to improve one's personality by removing imperfections should not try to remove them all. Granted, the fewer imperfections one's personality has, the happier one will be. However, it is never possible to remove all of the imperfections from one's personality.", "When one is trying to improve one's personality, one should not try to remove imperfections that do not cause one serious difficulties. Often, removing such an imperfection will only lead to greater imperfections." ], "label": 0 }, { "id": "train_4556", "context": "Each December 31 in Country Q, a tally is made of the country' s total available coal supplies -- that is, the total amount of coal that has been mined throughout the country but not consumed. In 1991 that amount was considerably lower than it had been in 1990. Furthermore, Country Q has not imported or exported coal since 1970.", "question": "If the statements above are true, which one of the following must also be true on the basis of them?", "answers": "['In Country Q, the amount of coal consumed in 1990 was greater than the amount of coal consumed in 1991.', 'In Country Q, more coal was consumed during the first half of 1991 than was consumed during the first half of 1990.', 'In Country Q, the amount of coal consumed in 1991 was greater than the amount of coal mined in 1991.', 'In Country Q, more coal was mined in 1990 than was mined in 1991.']", "label": 2 }, { "id": "train_4557", "context": "Gaby: In school, children should be allowed fully to follow their own interests, supported by experienced teachers who offer minimal guidance. This enables them to be most successful in their adult lives. Logan: I disagree. Schoolchildren should acquire the fundamental knowledge necessary for future success, and they learn such fundamentals only through disciplined, systematic instruction from accredited teachers.", "question": "Gaby's and Logan's comments provide most support for the claim that they disagree about", "answers": "[\"the sort of school environment that most fosters children's creativity\", \"the extent to which teachers should direct schoolchildren's education\", 'the extent to which schoolchildren are interested in fundamental academic subjects', \"the importance of having qualified teachers involved in schoolchildren's education\"]", "label": 1 }, { "id": "train_4558", "context": "An air traveler in Beijing cannot fly to Lhasa without first flying to Chengdu. Unfortunately, an air traveler in Beijing must fly to Xian before flying to Chengdu. Any air traveler who flies from Beijing to Lhasa, ttherefore, cannot avoid flying to Xian.", "question": "The pattern of reasoning exhibited by the argument above is most similar to that exhibited by which one of the following?", "answers": "['It is impossible to be fluent in a language without knowing its grammatical rules. A person who knows the grammatical rules of a language has learned them by means of exhaustive and difficult study or else by growing up in an environment in which the language is spoken. There are two major ways, ttherefore, for a person to become fluent in a language.', 'In the City Ballet Company any dancer who has danced in Giselle has also danced in Sleeping Beauty, and some dancers who have danced in Sleeping Beauty have also danced in Swan Lake. Ttherefore, some dancers in the City Ballet Company who have danced in Giselle have also danced in Swan Lake.', 'An ice-sculpture artist cannot reach the yellow level of achievement without first achieving the green level. The green level is impossible to achieve unless the white level has already been achieved. Ttherefore, an ice-sculpture artist who has reached the yellow level must have previously achieved the white level.', 'A doctor cannot prescribe porozine for a patient without first prescribing anthroxine for that patient. Unfortunately, anthroxine makes most patients who take it feel either extremely drowsy or else extremely nervous. It is likely, ttherefore, that a patient who has taken porozine has felt extremely nervous.']", "label": 2 }, { "id": "train_4559", "context": "Each child in a group of young children read aloud both a short paragraph and a list of randomly ordered words from the paragraph. The more experienced readers among them made fewer pronunciation errors in whichever task they performed second, whether it was the list or the paragraph. The order in which the two tasks were performed, however, had no effect on the performance of beginning readers, who always made fewer pronunciation errors when reading the paragraph than when reading the list.", "question": "Which one of the following, if true, most helps to explain why the order in which the tasks were performed was not significant for the beginning readers?", "answers": "['The beginning readers made more pronunciation errors than the more experienced readers did in reading both the paragraph and the list.', 'Because several words were used more than once in the paragraph but only once in the list, the list was shorter than the paragraph.', 'The more experienced readers sounded out difficult words, while the beginning readers relied solely on context to guess at difficult words.', 'In reading the paragraph, the more experienced readers were better at using context to guess at difficult words than were the beginning readers.']", "label": 2 }, { "id": "train_4560", "context": "Historian: Fifteenth-century advances in mapmaking contributed to the rise of modern nation-states. In medieval Europe (from the fifth to the fifteenth century), sovereignty centered in cities and towns and radiated outward, with boundaries often ambiguously defined. The conceptual shift toward the modern state began in the late fifteenth century, when mapmakers learned to reflect geography accurately by basing maps on latitude-longitude grids. By the midseventeenth century, nearly all maps showed boundary lines.", "question": "Which of the following would, if true, most strengthen the historian's reasoning?", "answers": "['During the fifteenth century in Europe, mapmakers simplified the borders of sovereignty by drawing clear lines of demarcation between political powers.', 'Seventeenth-century treatises and other sources of political authority describe areas of sovereignty rather than illustrate them using maps.', 'Borders did not become codified in Europe until certain treaties were signed in the early nineteenth century.', 'During the medieval period, various authorities in Europe claimed power over collections of cities and towns, not contiguous territories.']", "label": 0 }, { "id": "train_4561", "context": "Biologists often announce that a certain kind of animal has been found capable of using tools; this usually refers to something like using a stick to hunt for ants in a log, or a stone to crack nuts. But such announcements are completely unsurprising, since all animals use tools. Birds build nests, fish hide in the mud to escape predators, and squirrels use buildings as shortcuts between trees. If an animal executes its purpose by means of an external physical object, then that object can reasonably be regarded as a tool.", "question": "Which one of the following most accurately describes the role played in the argument by the claim that the biologists' announcements that a certain animal has been found capable of using tools are unsurprising?", "answers": "['It addresses a weakness in the biologists\\' announcements that stems from their ambiguous use of the word \"external. \"', \"It provides evidence that the animals' activities given as examples are purposeful.\", 'It is the conclusion of the argument.', \"It calls into question the basis of the biologists' conception of a tool.\"]", "label": 2 }, { "id": "train_4562", "context": "The 1980s have been characterized as a period of selfish individualism that threatens the cohesion of society. But this characterization is true of any time. Throughout history all human actions have been motivated by selfishness. When the deeper implications are considered, even the simplest \"unselfish\" acts prove to be instances of selfish concern for the human species.", "question": "Which one of the following is a flaw in the argument?", "answers": "['The argument relies on two different uses of the term \"selfish. \"', 'The claim that selfishness has been present throughout history is not actually relevant to the argument.', 'The argument mentions only humans and does not consider the behavior of other species.', 'No statistical evidence is provided to show that humans act selfishly more often than they act unselfishly.']", "label": 0 }, { "id": "train_4563", "context": "Political advocate: Campaigns for elective office should be subsidized with public funds. One reason is that this would allow politicians to devote less time to fund-raising, thus giving campaigning incumbents more time to serve the public. A second reason is that such subsidies would make it possible to set caps on individual campaign contributions, thereby reducing the likelihood that elected officials will be working for the benefit not of the public but of individual large contributors. Critic: This argument is problematic: the more the caps constrain contributions, the more time candidates have to spend finding more small contributors.", "question": "The critic objects that the advocate's argument is flawed because", "answers": "['one of the projected results cited in support of the proposal made is entailed by the other and ttherefore does not constitute independent support of the proposal', 'of the two projected results cited in support of the proposal made, one works against the other', 'any resourceful large contributor can circumvent caps on individual contributions by sending in smaller amounts under various names', 'it overlooks the possibility that large contributors will stop contributing if they cannot contribute at will']", "label": 1 }, { "id": "train_4564", "context": "Those who participate in risky sports often do so to confront their fears. For example, rock climbers are more likely than others to have once suffered from a fear of heights. Those who participate in such risk-taking activities also have more self-confidence than others, so it is probably true that confronting one' s fears increases one' s self-confidence.", "question": "Which one of the following, if true, most weakens the reasoning above?", "answers": "[\"Participating in risky sports is not the only way to confront one's fears.\", 'In general, people who currently participate in risky sports had above-average self-confidence even before participating in any risky sport.', 'Most of those who do not participate in risky sports believe that they lack the capacity to excel in such activities.', 'Most people who refrain from engaging in risky sports refrain from doing so for reasons other than a fear of death or injury.']", "label": 1 }, { "id": "train_4565", "context": "Toxicologist: A survey of oil-refinery workers who work with MBTE, an ingredient currently used in some smog-reducing gasolines, found an alarming incidence of complaints about headaches, fatigue, and shortness of breath. Since gasoline containing MBTE will soon be widely used, we can expect an increased incidence of headaches, fatigue, and shortness of breath.", "question": "Each of the following, if true, strengthens the toxicologist's argument EXCEPT:", "answers": "['Most oil-refinery workers who do not work with MBTE do not have serious health problems involving headaches, fatigue, and shortness of breath.', 'Since the time when gasoline containing MBTE was first introduced in a few metropolitan areas, those areas reported an increase in the number of complaints about headaches, fatigue, and shortness of breath.', 'Headaches, fatigue, and shortness of breath are among the symptoms of several medical conditions that are potentially serious threats to public health.', 'The oil-refinery workers surveyed were carefully selected to be representative of the broader population in their medical histories prior to exposure to MBTE, as well as in other relevant respects.']", "label": 2 }, { "id": "train_4566", "context": "A person is more likely to become visually impaired as that person ages. Among adults in the state of Illinois, however, the proportion receiving benefit payments from their vision insurance shrinks from 56 percent among 50-64 year olds to 24 percent for those aged 65-79, and 13 percent for those aged 79 and older. The explanation of this discrepancy is that the proportion of jobs offering such an insurance benefit has greatly increased in recent years.", "question": "Which one of the following, if true about Illinois, shows that the explanation above is, at best, incomplete?", "answers": "[\"Under most employers' plans, vision insurance payments stop when a visually impaired employee reaches the usual retirement age of 65.\", \"Medical advances have improved the average person's vision beyond what it was 20 years ago.\", 'The treatment of newly incurred visual impairments is more successful now than in the past in restoring vision.', 'For persons receiving vision insurance benefit payments, those payments on average represent a smaller share of their income now than was the case 20 years ago.']", "label": 0 }, { "id": "train_4567", "context": "In 1974 the speed limit on highways in the United States was reduced to 55 miles per hour in order to save fuel. In the first 12 months after the change, the rate of highway fatalities dropped 15 percent, the sharpest one-year drop in history. Over the next 10 years, the fatality rate declined by another 25 percent. It follows that the 1974 reduction in the speed limit saved many lives.", "question": "Which one of the following, if true, most strengthens the argument?", "answers": "['The fatality rate in highway accidents involving motorists driving faster than 55 miles per hour is much higher than in highway accidents that do not involve motorists driving at such speeds.', 'There was no decline in the rate of highway fatalities during the twelfth year following the reduction in the speed limit.', 'Motorists are more likely to avoid accidents by matching their speed to that of the surrounding highway traffic than by driving at faster or slower speeds.', 'The 1974 fuel shortage cut driving sharply for more than a year.']", "label": 0 }, { "id": "train_4568", "context": "The National Association of Fire Fighters says that 45 percent of homes now have smoke detectors, whereas only 30 percent of homes had them 10 years ago. This makes early detection of house fires no more likely, however, because over half of the domestic smoke detectors are either without batteries or else inoperative for some other reason.", "question": "In order for the conclusion above to be properly drawn, which one of the following assumptions would have to be made?", "answers": "['The proportion of domestic smoke detectors that are inoperative has increased in the past ten years.', 'Fifteen percent of domestic smoke detectors were installed less than 10 years ago.', 'Not all of the smoke detectors in homes are battery operated.', 'Unlike automatic water sprinklers, a properly functioning smoke detector cannot by itself increase fire safety in a home.']", "label": 0 }, { "id": "train_4569", "context": "The mayor was not telling the truth when he said that the bridge renovation did not waste taxpayers' money. The very commission he set up to look into government waste reported that the Southern Tier Project, of which the bridge renovation was a part, was egregiously wasteful.", "question": "The reasoning in the argument is flawed in that the argument", "answers": "['infers that a part has a certain quality merely on the grounds that the whole to which it belongs has that quality', 'draws a general conclusion about government waste on the basis of a single instance of such waste', \"attacks the mayor's character rather than assessing the strength of the evidence supporting the mayor's claim\", 'rejects a position on the grounds that the motives of the person who has advanced the position were not disinterested']", "label": 0 }, { "id": "train_4570", "context": "Publisher: The new year is approaching, and with it the seasonal demand for books on exercise and fitness. We must do whatever it takes to ship books in that category on time; our competitors have demonstrated a high level of organization, and we cannot afford to be outsold.", "question": "Which one of the following most accurately expresses the main conclusion drawn in the publisher's argument?", "answers": "['It is imperative that the company ship fitness and exercise books on time.', \"The company should do whatever is required in order to adopt its competitors' shipping practices.\", 'The company should make shipping books its highest priority.', \"The company will be outsold if it does not maintain its competitors' high level of organization.\"]", "label": 0 }, { "id": "train_4571", "context": "Many economists claim that financial rewards provide the strongest incentive for people to choose one job over another. But in many surveys, most people do not name high salary as the most desirable feature of a job. This shows that these economists overestimate the degree to which people are motivated by money in their job choices.", "question": "Which one of the following, if true, most weakens the argument?", "answers": "['In many surveys, people say that they would prefer a high-wage job to an otherwise identical job with lower wages.', 'Jobs that pay the same salary often vary considerably in their other financial benefits.', 'Even high wages do not enable people to obtain all the goods they desire.', 'Some people are not aware that jobs with high salaries typically leave very little time for recreation.']", "label": 1 }, { "id": "train_4572", "context": "People buy prestige when they buy a premium product. They want to be associated with something special. Mass-marketing techniques and price-reduction strategies should not be used because__.", "question": "Which of the following best completes the passage below?", "answers": "['purchasers of premium products are concerned with the quality as well as with the price of the products', 'continued sales depend directly on the maintenance of an aura of exclusivity', 'affluent purchasers currently represent a shrinking portion of the population of all purchasers', 'manufacturing a premium brand is not necessarily more costly than manufacturing a standard brand of the same product']", "label": 1 }, { "id": "train_4573", "context": "Farmer: Several people in the past few years have claimed to have seen a mountain lion in the suburban outskirts-the latest just last month-and, while mountain lions were thought to have been driven from this entire region about twenty years ago, there is no reason for the people who reported seeing a mountain lion to have deliberately concocted a false report. Ttherefore, local wildlife managers should begin to urgently address the mountain lion' s presence.", "question": "Which of the following would, if true, most seriously weaken the farmer's argument?", "answers": "['Mountain lions are dissimilar in size and color to other wild animals found near the suburban outskirts.', 'No person who claimed to have seen a mountain lion had anyone else with them at the purported sighting.', 'Farmers in the suburban outskirts mostly raise cattle and hogs, which when fully grown are generally not attacked by mountain lions.', 'There have been no regional reports in the past year of mountain lions migrating to the area.']", "label": 1 }, { "id": "train_4574", "context": "Psychologists have found that candidates for top political offices who blink excessively during televised debates are judged by viewers to have done less well than competing candidates who exhibit average blink rates. Any impact this phenomenon has on election results is surely deleterious: Many features -- knowledgeableness, confidence, and so forth -- contribute to a political official' s ability to perform well in office, but having an average blink rate is certainly not such a feature.", "question": "Which one of the following, if true, most weakens the argument?", "answers": "['Excessive blinking has been shown to be a mostly reliable indicator of a lack of confidence.', \"Blinking too infrequently during televised debates has the same effect on viewers' judgments of candidates as blinking excessively.\", \"Viewers' judgments about candidates' debate performances are generally not affected by how knowledgeable the candidates appear to be.\", \"Voters' judgments about candidates' debate performances rarely affect the results of national elections.\"]", "label": 0 }, { "id": "train_4575", "context": "As symbols of the freedom of the wilderness, bald eagles have the unique capacity to inspire people and foster in them a sympathetic attitude toward the needs of other threatened species. Clearly, without that sympathy and the political will it engenders, the needs of more obscure species will go unmet. The conservation needs of many obscure species can only be met by beginning with the conservation of this symbolic species, the bald eagle.", "question": "Which one of the following is the main point of the passage as a whole?", "answers": "[\"People's love of the wilderness can be used to engender political support for conservation efforts.\", 'Without increased public sympathy for conservation, the needs of many symbolic species will go unmet.', 'The conservation of bald eagles is the first necessary step in conserving other endangered species.', 'Because bald eagles symbolize freedom, conservation efforts should be concentrated on them rather than on other, more obscure species.']", "label": 2 }, { "id": "train_4576", "context": "At Happywell, Inc. , last year the average annual salary for dieticians was $50, 000, while the average annual salary for physical therapists was $42, 000. The average annual salary for all Happywell employees last year was $40, 000.", "question": "If the information above is correct, which one of the following conclusions can properly be drawn on the basis of it?", "answers": "['There was no dietician at Happywell last year who earned less than the average for a physical therapist.', 'At least one Happywell employee earned less than the average for a physical therapist last year.', 'At least one dietician earned more than the highest-paid Happywell physical therapist last year.', 'There were more physical therapists than dieticians at Happywell last year.']", "label": 1 }, { "id": "train_4577", "context": "Paint on a new airliner is usually applied in two stages: first, a coat of primer, and then a top coat. A new process requires no primer, but instead uses two layers of the same newly developed coating, with each layer of the new coating having the same thickness and weight as a traditional top coat. Using the new process instead of the old process increases the price of a new aircraft considerably.", "question": "Which of the following, if true, most strongly indicates that it is in an airline's long-term economic interest to purchase new airliners painted using the new process rather than the old process?", "answers": "['Because the new coating has a viscosity similar to that of a traditional top coat, aircraft manufacturers can apply it using the same equipment as is used for a traditional top coat.', \"A layer of primer on an airliner weighs more than a layer of the new coating would by an amount large enough to make a difference to that airliner's load-bearing capacity.\", 'A single layer of the new coating provides the aluminum skin of the airliner with less protection against corrosion than does a layer of primer of the usual thickness.', 'Unlike the old process, the new process was originally invented for use on spacecraft, which are subject to extremes of temperature to which airliners are never exposed.']", "label": 1 }, { "id": "train_4578", "context": "People with higher-than-average blood levels of a normal dietary by-product called homocysteine are twice as likely to be diagnosed with Alzheimer' s disease as are those with average or below-average homocysteine levels. Thus, it is likely that the risk of developing Alzheimer' s disease could be reduced by including in one' s diet large amounts of B vitamins and folic acid, which convert homocysteine into substances known to have no relation to Alzheimer' s disease.", "question": "Which one of the following, if true, most seriously weakens the argument?", "answers": "[\"Alzheimer's disease tends to increase the levels of homocysteine in the blood.\", \"Many Alzheimer's patients have normal homocysteine levels.\", 'B vitamins and folic acid are not metabolized by the body very efficiently when taken in the form of vitamin-mineral supplements.', \"People whose relatives contracted Alzheimer's disease are much more likely to develop Alzheimer's than those whose relatives did not.\"]", "label": 0 }, { "id": "train_4579", "context": "Most antidepressant drugs cause weight gain. While dieting can help reduce the amount of weight gained while taking such antidepressants, some weight gain is unlikely to be preventable.", "question": "The information above most strongly supports which one of the following?", "answers": "['All patients taking antidepressant drugs should diet to maintain their weight.', 'A physician should not prescribe any antidepressant drug for a patient if that patient is overweight.', 'At least some patients taking antidepressant drugs gain weight as a result of taking them.', 'The weight gain experienced by patients taking antidepressant drugs should be attributed to lack of dieting.']", "label": 2 }, { "id": "train_4580", "context": "Critic: Historians purport to discover the patterns inherent in the course of events. But historians actually impose, rather than find, such patterns by choosing what to include in and exclude from their historical narratives. Thus, properly understood, histories reveal more about the presuppositions underlying different historians' attempts to understand what happened than about what actually happened.", "question": "The critic's argument depends on which one of the following assumptions?", "answers": "[\"Which pattern a historian imposes upon events is affected by that historian's presuppositions.\", 'Historians have many presuppositions in common with one another.', 'Most historians cannot become aware of the presuppositions that they bring to their narratives.', 'There is no way to determine with certainty whether a pattern described by a historian is actually present in and not merely imposed upon the events.']", "label": 0 }, { "id": "train_4581", "context": "Some people mistakenly believe that since we do not have direct access to the distant past we cannot learn much about it. Contemporary historians and archaeologists find current geography, geology, and climate to be rich in clues about a given region' s distant history. However, the more distant the period we are studying is, the less useful the study of the present becomes.", "question": "Of the following, which one most closely conforms to the principle that the passage illustrates?", "answers": "['Astronomers often draw inferences about the earlier years of our solar system on the basis of recently collected data. Unfortunately, they have been able to infer comparatively little about the origin of our solar system.', \"One crucial clue to the extent of the ancient Egyptians' mathematical knowledge came from studying the pyramids. The more we studied such structures, the more impressed we were by how much the Egyptians knew.\", \"To understand a literary text one needs to understand the author's world view. However, the farther that world view gets from one's own the less one will be able to appreciate the text.\", 'We often extrapolate from ordinary sensory experience to things beyond such experience and form a rash judgment, such as the claim that the earth is the center of the universe because it appears that way to us.']", "label": 0 }, { "id": "train_4582", "context": "A few people who are bad writers simply cannot improve their writing, whether or not they receive instruction. Still, most bad writers can at least be taught to improve their writing enough so that they are no longer bad writers. However, no one can become a great writer simply by being taught how to be a better writer, since great writers must have not just skill, but also talent.", "question": "Which one of the following can be properly inferred from the passage above?", "answers": "['All great writers had to be taught to become better writers.', 'Some bad writers can never become great writers.', 'All bad writers can become better writers.', 'Some great writers can be taught to be even better writers.']", "label": 1 }, { "id": "train_4583", "context": "Carl' s Coffee Emporium stocks only two decaffeinated coffees: French Roast and Mocha Java. Yusef only serves decaffeinated coffee, and the coffee he served after dinner last night was far too smooth and mellow to have been French Roast. So, if Yusef still gets all his coffee from Carl' s, what he served last night was Mocha Java.", "question": "The argument above is most similar in its logical structure to which one of the following?", "answers": "['Werdix, Inc. , has offered Arno a choice between a job in sales and a job in research. Arno would like to work at Werdix but he would never take a job in sales when another job is available, so if he accepts one of these jobs, it will be the one in research.', 'Samuel wants to take three friends to the beach. His mother owns both a sedan and a convertible. The convertible holds four people so, although the sedan has a more powerful engine, if Samuel borrows a vehicle from his mother, he will borrow the convertible.', 'Rose can either take a two-week vacation in July or wait until October and take a three-week vacation. The trail she had planned to hike requires three weeks to complete but is closed by October, so if Rose takes a vacation, it will not be the one she had planned.', \"If Teresa does not fire her assistant, her staff will rebel and her department's efficiency will decline. Losing her assistant would also reduce its efficiency, so, if no alternative solution can be found, Teresa's department will become less efficient.\"]", "label": 0 }, { "id": "train_4584", "context": "City council member: The Senior Guild has asked for a temporary exception to the ordinance prohibiting automobiles in municipal parks. Their case does appear to deserve the exception. However, if we grant this exception, we will find ourselves granting many other exceptions to this ordinance, some of which will be undeserved. Before long, we will be granting exceptions to all manner of other city ordinances. If we are to prevent anarchy in our city, we must deny the Senior Guild' s request.", "question": "The city council member's argument is most vulnerable to criticism on the grounds that it", "answers": "['presumes, without sufficient warrant, that one event will lead to a particular causal sequence of events', 'dismisses a claim because of its source rather than because of its content', 'fails to make a needed distinction between deserved exceptions and undeserved ones', 'contains premises that contradict one another']", "label": 0 }, { "id": "train_4585", "context": "Some thinkers hold that whatever brings pleasure to people is good, and vice versa, and that whatever brings pain is evil, and vice versa. This, however, is obviously untenable. The concepts of good and evil are contradictory, whereas those of pleasure and pain are not. Clearly it is possible to be in a state of pleasure and pain at once.", "question": "The argument challenges a claim that two pairs of concepts are equivalent by claiming that", "answers": "['\"good\" does not apply to any of the same things that \"pleasure\" does', '\"good\" and \"evil\" do not bear the same relationship to one another that \"pleasure\" and \"pain\" do to one another', '\"pleasure\" and \"pain\" possess a property not possessed by any other possible pair of concepts', 'it is merely accidental that \"pleasure\" and \"pain\" are applied in the same way as \"good\" and \"evil\" are applied']", "label": 1 }, { "id": "train_4586", "context": "Professor Clemens: Many people die in traffic accidents each year. Since airbags can prevent deaths in traffic accidents, motor vehicles should be equipped with airbags. Professor Sanford: Your recommendation is useless. It is true that airbags may save some lives in traffic accidents, but seatbelts can prevent far more deaths each year than can airbags.", "question": "Professor Sanford's response fails to establish that Professor Clemens's recommendation should not be implemented because her response", "answers": "[\"does not show that the benefits that would follow from Professor Clemens's recommendation would be offset by any disadvantages\", 'confuses the notion of saving lives with the expense associated with the remedy', 'is not supported by any evidence that seatbelts do, in fact, save more lives', 'does not offer any additional way of lessening the risk associated with traffic accidents']", "label": 0 }, { "id": "train_4587", "context": "Eva: A \"smart highway\" system should be installed, one that would monitor areawide traffic patterns and communicate with computers in vehicles or with programmable highway signs to give drivers information about traffic congestion and alternate routes. Such a system, we can infer, would result in improved traffic flow in and around cities that would do more than improve drivers' tempers; it would decrease the considerable loss of money and productivity that now results from traffic congestion. Louis: There are already traffic reports on the radio. Why would a \"smart highway\" system be any better? .", "question": "Eva's argument depends on the assumption that", "answers": "['the type of equipment used in \"smart highway\" systems would vary from one city to another', 'on \"smart highways\" there would not be the breakdowns of vehicles that currently cause traffic congestion', 'traffic flow in and around cities is not now so congested that significant improvement is impossible', 'older vehicles could not be fitted with equipment to receive signals sent by a \"smart highway\" system']", "label": 2 }, { "id": "train_4588", "context": "Tommy: Many people claim that the voting public is unable to evaluate complex campaign issues. The radio advertisements for Peterson in the national campaign, however, discuss complex campaign issues, and Peterson is currently more popular than the other candidates. Jamie: Yes, Peterson is the most popular. However, you are incorrect in claiming that this is a result of Peterson' s discussion of complex campaign issues. Peterson simply strikes the voters as the most competent and trustworthy candidate.", "question": "Which one of the following, if true, most supports Jamie's counter to Tommy?", "answers": "[\"Polling data shows that most voters cannot identify Peterson's positions on campaign issues.\", \"Polling data shows that Peterson's present popularity will probably diminish over time.\", \"Peterson's opponents are discussing some of the same issues as Peterson is discussing.\", 'Polling data shows that some voters consider Peterson competent and trustworthy.']", "label": 0 }, { "id": "train_4589", "context": "Human resources director: While only some recent university graduates consider work environment an important factor in choosing a job, they all consider salary an important factor. Further, whereas the only workers who consider stress level an important factor in choosing a job are a few veteran employees, every recent university graduate considers vacation policy an important factor.", "question": "If all of the statements of the human resources director are true, then which one of the following must be true?", "answers": "['At least some veteran employees do not consider work environment an important factor in choosing a job.', 'At least some people who consider work environment an important factor in choosing a job consider vacation policy an important factor as well.', 'No one for whom salary is an important factor in choosing a job also considers stress level an important factor.', 'All people who consider work environment an important factor in choosing a job also consider salary an important factor.']", "label": 1 }, { "id": "train_4590", "context": "Harrold Foods is attempting to dominate the softdrink market by promoting \"Hero, \" its most popular carbonated drink product, with a costly new advertising campaign. But survey results show that, in the opinion of 72 percent of all consumers, \"Hero\" already dominates the market. Since any product with more than 50 percent of the sales in a market is, by definition, dominant in that market, Harrold Foods dominates the market now and need only maintain its current market share in order to continue to do so.", "question": "The argument commits which one of the following errors in reasoning?", "answers": "['failing to exclude the possibility that what appears to be the result of a given market condition may in fact be the cause of that condition', 'mistaking a condition required if a certain result is to obtain for a condition that by itself is sufficient to guarantee that result', 'taking evidence that a claim is believed to be true to constitute evidence that the claim is in fact true', 'treating the failure to establish that a certain claim is false as equivalent to a demonstration that that claim is true']", "label": 2 }, { "id": "train_4591", "context": "Kendrick: Governments that try to prevent cigarettes from being advertised are justified in doing so, since such advertisements encourage people to engage in an unhealthy practice. But cigarette advertisements should remain legal since advertisements for fatty foods are legal, even though those advertisements also encourage people to engage in unhealthy practices.", "question": "Which one of the following, if true, most helps to resolve the apparent conflict between Kendrick's statements?", "answers": "['Governments should try to prevent the advertisement of cigarettes by means of financial disincentives rather than by legal prohibition.', 'The advertisement of fattening foods, unlike that of cigarettes, should not be prevented, because fattening foods, unlike cigarettes, are not addictive.', 'Any advertisement that encourages people to engage in an unhealthy practice should be made illegal, even though the legality of some such advertisements is currently uncontroversial.', 'Governments should place restrictions on cigarette advertisements so as to keep them from encouraging people to engage in unhealthy practices, but should not try to prevent such advertisements.']", "label": 0 }, { "id": "train_4592", "context": "Hazel: Faster and more accurate order processing would help our business. To increase profits, we should process orders electronically rather than manually, because customers' orders will then go directly to all relevant parties. Max: We would lose money if we started processing orders electronically. Most people prefer to interact with human beings when placing orders. If we switch to electronic order processing, our business will appear cold and inhuman, and we will attract fewer customers.", "question": "Hazel and Max disagree over whether", "answers": "['faster and more accurate order processing would be financially beneficial to their business', 'electronic order processing is faster and more accurate than is manual order processing', 'their business has an obligation to be as profitable as possible', 'switching to electronic order processing would be financially beneficial to their business']", "label": 3 }, { "id": "train_4593", "context": "Historian: Alexander the Great should not be judged by appeal to current notions of justice. Alexander, an ancient figure of heroic stature, should be judged by the standards of his own culture. That is, did he live up to his culture' s ideals of leadership? Did Alexander elevate the contemporary standards of justice? Was he, in his day, judged to be a just and wise ruler? Student: But you cannot tell whether or not Alexander raised the contemporary standards of justice without invoking standards other than those of his own culture.", "question": "Which one of the following argumentative strategies does the student use in responding to the historian?", "answers": "[\"questioning the historian's motivation for determining whether a standard of behavior has been raised or lowered\", \"arguing that applying the historian's principle would require a knowledge of the past that is necessarily inaccessible to current scholarship\", 'showing that the principle the historian invokes, when applied to Alexander, does not justify the assertion that he was heroic', \"claiming that one of the historian's criteria for judging Alexander is inconsistent with the principle that the historian has advanced\"]", "label": 3 }, { "id": "train_4594", "context": "Biologist: Researchers believe that dogs are the descendants of domesticated wolves that were bred to be better companions for humans. It has recently been found that some breeds of dog are much more closely related genetically to wolves than to most other breeds of dog. This shows that some dogs are descended from wolves that were domesticated much more recently than others.", "question": "Which one of the following principles underlies the biologist's argument?", "answers": "['If one breed of dog is more closely related to wolves than to another breed of dog, then the former breed of dog has more recent undomesticated wolf ancestors than the latter breed has.', 'Any breed of dog descended from wolves that were domesticated is more closely related genetically to at least some other breeds of dog than to wolves.', 'Any two breeds of dog that are more closely related to each other than to wolves are both descended from wolves that were domesticated long ago.', 'If one breed of dog is more closely related to wolves than another breed of dog is, then the former breed of dog is more closely related to wolves than to the latter breed of dog.']", "label": 0 }, { "id": "train_4595", "context": "S: It would be premature to act to halt the threatened \"global warming trend, \" since that alleged trend might not be real. After all, scientists disagree about it, some predicting over twice as much warming as others, so clearly their predictions cannot be based on firm evidence. W: Most scientists consider discussions of accepted ideas boring, and prefer to argue about what is not known. According to the International Science Council, there is a consensus among reputable investigators that average global warming in the next century will be from 1. 5 degree to 4. 5 degree C.", "question": "W's rejoinder proceeds by", "answers": "['putting disagreements cited by S in perspective by emphasizing similarities', 'reasoning in a circle by accepting evidence only if it agrees with a desired conclusion', \"accepting S's conclusion while disputing the reasons offered for it\", 'relying on authorities whose views conflict with the views of the authorities cited by S']", "label": 0 }, { "id": "train_4596", "context": "Although parapsychology is often considered a pseudoscience, it is in fact a genuine scientific enterprise, for it uses scientific methods such as controlled experiments and statistical tests of clearly stated hypotheses to examine the questions it raises.", "question": "The conclusion above is properly drawn if which of the following is assumed?", "answers": "['Since parapsychology raises clearly statable questions, they can be tested in controlled experiments.', 'Any field of study that employs scientific methods is a genuine scientific enterprise.', 'Any enterprise that does not use controlled experiments and statistical tests is not genuine science.', 'If a field of study can conclusively answer the questions it raises, then it is a genuine science.']", "label": 1 }, { "id": "train_4597", "context": "Any writer whose purpose is personal expression sometimes uses words ambiguously. Every poet' s purpose is personal expression. Thus no poetry reader' s enjoyment depends on attaining a precise understanding of what the poet means.", "question": "The conclusion can be properly inferred if which one of the following is assumed?", "answers": "[\"Most writers whose readers' enjoyment does not depend on attaining a precise understanding of the writers' words are poets.\", 'Readers who have a precise understanding of what a writer has written derive their enjoyment from that understanding.', 'No writer who ever uses words ambiguously has any reader whose enjoyment depends on attaining a precise understanding of what the writer means.', 'Writers whose purpose is personal expression are unconcerned with whether anyone enjoys reading their works.']", "label": 2 }, { "id": "train_4598", "context": "Microbiologist: A lethal strain of salmonella recently showed up in a European country, causing an outbreak of illness that killed two people and infected twenty-seven others. Investigators blame the severity of the outbreak on the overuse of antibiotics, since the salmonella bacteria tested were shown to be drug-resistant But this is unlikely because patients in the country where the outbreak occurred cannot obtain antibiotics to treat illness without a prescription, and the country' s doctors prescribe antibiotics less readily than do doctors in any other European country.", "question": "Which of the following, if true, would most weaken the microbiologist's reasoning?", "answers": "['People in the country where the outbreak occurred often consume foods produced from animals that eat antibiotics-laden livestock feed.', 'Physicians in the country where the outbreak occurred have become hesitant to prescribe antibiotics since they are frequently in short supply.', 'Drug-resistant strains of salmonella have not been found in countries in which antibiotics are not generally available.', 'Use of antibiotics in two countries that neighbor the country where the outbreak occurred has risen over the past decade.']", "label": 0 }, { "id": "train_4599", "context": "Cynthia: Corporations amply fund research that generates marketable new technologies. But the fundamental goal of science is to achieve a comprehensive knowledge of the workings of the universe. The government should help fund those basic scientific research projects that seek to further our theoretical knowledge of nature. Luis: The basic goal of government support of scientific research is to generate technological advances that will benefit society as a whole. So only research that is expected to yield practical applications in fields such as agriculture and medicine ought to be funded.", "question": "Cynthia's and Luis's statements provide the most support for the contention that they would disagree with each other about the truth of which one of the following statements?", "answers": "['A proposed study of the effects of chemical fertilizers on crops, for the purpose of developing more-resistant and higher-yielding breeds, should not receive government funding.', 'Knowledge gained through basic scientific research need not be expected to lead to new and useful technologies in order for the research to merit government funding.', 'The government should help fund pure theoretical research because such research might have unforeseen practical applications in fields such as agriculture and medicine.', 'Although some research projects in theoretical science yield practical benefits, most do not, and so no research projects in theoretical science should be funded by the government.']", "label": 1 }, { "id": "train_4600", "context": "Environmentalists who seek stricter governmental regulations controlling water pollution should be certain to have their facts straight. For if it turns out, for example, that water pollution is a lesser threat than they proclaimed, then there will be a backlash and the public will not listen to them even when dire threats exist.", "question": "Which one of the following best illustrates the principle illustrated by the argument above?", "answers": "['Whoever wants to advance a political agenda ought to take the time to convince legislators that their own political careers are at stake in the matter at hand; otherwise, the agenda will simply be ignored.', 'Politicians who defend the rights of unpopular constituencies ought to see to it that they use cool, dispassionate rhetoric in their appeals. Even if they have their facts straight, inflammatory rhetoric can cause a backlash that results in more negative reactions to these constituencies, whether or not they are deserving of more rights.', 'People who are trying to convince others to take some sort of action should make every effort to present evidence that is emotionally compelling. Such evidence is invariably more persuasive than dry, technical data, even when the data strongly support their claims.', 'Middle-level managers who ask their companies to hire additional employees should have strong evidence that doing so will benefit the company; otherwise, higher-level managers will refuse to follow their suggestions to hire additional employees even when doing so really would benefit the company.']", "label": 3 }, { "id": "train_4601", "context": "Letter to the editor: Our city plans to allow major commercial development along the south waterfront and, in so doing, to increase its tax revenue. But, to succeed commercially, the development would inevitably create far more road traffic than the existing roads to the waterfront can handle, causing serious congestion. Providing enough roads or public transportation to the area would cost far more than the city could gain from the increased tax revenue.", "question": "Which of the following, if added to the city's plan, would be most likely to help solve the problem the letter describes?", "answers": "[\"Evaluating the net benefits that the commercial development would bring to the city in terms of improved quality of life for the city's residents rather than in financial terms\", 'Giving tax breaks to developers of businesses along the waterfront to offset any tax levied on them for construction of roads or public transportation', 'Allowing residential development along the waterfront so that there will be waterfront residents who can walk to the commercial development', 'Funding construction of new roads to the waterfront with a system of tolls on the new roads to the waterfront']", "label": 3 }, { "id": "train_4602", "context": "Editorialist: Research reveals that patients of hospital cardiac units in which doctors perform many aggressive, high-tech procedures are less likely to be alive a month after leaving the hospital than are patients of hospital cardiac units that rely almost exclusively on standard treatments. This indicates that aggressive, high-tech treatments of cardiac disease are less effective than standard treatments.", "question": "Which one of the following, if true, most weakens the editorialist's argument?", "answers": "['Doctors who do not perform aggressive, high-tech procedures tend to place much emphasis on the prevention of cardiac problems through healthy lifestyle choices.', 'Patients with the most-severe cardiac problems tend to receive treatment at hospitals where many aggressive, high-tech treatments are performed.', 'All cardiac patients have access to hospitals in which relatively large numbers of aggressive, high-tech procedures are performed.', 'Some cardiac patients have lived for many years after receiving aggressive, high-tech treatments.']", "label": 1 }, { "id": "train_4603", "context": "At some point in any discussion of societal justice, the only possible doctrinal defense seems to be \"That is the way we do things here. \" Different communities that each recognize the dignity and equality of all citizens will, for example, nevertheless settle on somewhat different provisions for the elderly. So we can see that general principles of justice are never sufficient to determine the details of social policies fixed within a particular state.", "question": "Which one of the following statements, if true, most strengthens the argument concerning the general principles of justice?", "answers": "['Two societies based on different principles of justice, each adhering to its own principles, had the same unemployment benefits.', 'Although each adhered to its own principles, a democracy and a monarchy each had the same distribution of wealth in its population.', 'Although two socialist states each adhered to the same electoral principles, one had a different type of machine for counting ballots in public elections than the other did.', 'Two states founded on and adhering to similar principles of justice had different requirements that had to be met in order to be eligible for government-subsidized day care.']", "label": 3 }, { "id": "train_4604", "context": "Human beings can live happily only in a society where love and friendship are the primary motives for actions. Yet economic needs can be satisfied in the absence of this condition, as, for example, in a merchant society where only economic utility motivates action. It is obvious then that human beings __.", "question": "Which one of the following most logically completes the argument?", "answers": "['can satisfy their basic economic needs without obtaining happiness', 'cannot satisfy economic needs by means of interactions with family members and close friends', 'can live happily only when economic utility is not a motivator in their society', 'cannot really be said to have satisfied their economic needs unless they are happy']", "label": 0 }, { "id": "train_4605", "context": "The total market value of real estate in Altonville has steadily declined over the past four years. This decline has meant that the overall figure on which the city' s property tax is based - the assessed value of that real estate - has also declined. Moreover, the percentage of assessed value that was paid as property taxes to the city did not change during this period.", "question": "The information above most strongly supports which of the following?", "answers": "['During the past four years, Altonville officials also did not increase tax rates on other sources of revenue such as retail sales or business profits.', \"The percentage of Altonville's overall revenue that was collected from property taxes did not change over the past four years.\", 'The total amount of revenue that Altonville has collected from property taxes has declined over the past four years.', 'Altonville officials had proposed raising property tax rates during the past four years but never did so because of strong opposition from property owners.']", "label": 2 }, { "id": "train_4606", "context": "Sociologist: Some economists hold that unregulated markets should accompany democratic sovereignty because they let people vote with their money. But this view ignores the crucial distinction between the private consumer and the public citizen. In the marketplace the question is, \"What do I want? \" At the voting booth the question is always, \"What do we want? \" Hence, supporters of political democracy can also support marketplace regulation.", "question": "Which one of the following most accurately expresses the conclusion drawn by the sociologist?", "answers": "['Voters think of themselves as members of a community, rather than as isolated individuals.', 'Opposition to unregulated markets is consistent with support for democracy.', 'Where there is democratic sovereignty there should be unregulated markets.', 'Unregulated markets are incompatible with democratic sovereignty.']", "label": 1 }, { "id": "train_4607", "context": "In a recent study of stroke patients, those who exhibited continuing deterioration of the nerve cells in the brain after the stroke also exhibited the highest levels of the protein glutamate in their blood. Glutamate, which functions within nerve cells as a neurotransmitter, can kill surrounding nerve cells if it leaks from damaged or oxygen-starved nerve cells. Thus glutamate leaking from damaged or oxygen-starved nerve cells is a cause of longterm brain damage resulting from strokes.", "question": "Which one of the following, if true, most strengthens the argument?", "answers": "['Stroke patients exhibit a wide variety of abnormal chemical levels in their blood.', 'Glutamate is the only neurotransmitter that leaks from oxygen-starved or physically damaged nerve cells.', 'Leakage from damaged or oxygen-starved nerve cells is the only possible source of glutamate in the blood.', 'Any neurotransmitter that leaks from a damaged or oxygen-starved nerve cell will damage surrounding nerve cells.']", "label": 2 }, { "id": "train_4608", "context": "An overwhelming number of industry' s chief executive officers who earn over $250, 000 annually attended prestigious business schools. Ttherefore Greta Harris, who attended a prestigious business school, must be a chief executive officer who earns over $250, 000 annually.", "question": "Which one of the following exhibits flawed reasoning most nearly parallel to that exhibited in the argument above?", "answers": "['A great many successful opera singers studied more than one language. Hence Eileen must be a successful opera singer, since she studied more than one language.', 'Quite a few opera singers carry a bent nail on stage for good luck. Ttherefore George, an opera singer, must owe his good luck to the bent nail that he always carries.', 'The most famous opera singers practiced constantly in order to improve their voices. Ttherefore Franz will be more famous than will his rival Otto, who rarely practices.', 'Many opera singers are high-strung. Consequently it must be true that Fred, a high-strung opera singer, will develop the health problems associated with being high-strung.']", "label": 0 }, { "id": "train_4609", "context": "Columnist: A recent research report suggests that by exercising vigorously, one significantly lowers one' s chances of developing certain cardiorespiratory illnesses. But exercise has this effect, the report concludes, only if the exercise is vigorous. Thus, one should not heed older studies purporting to show that nonstrenuous walking yields the same benefits.", "question": "The reasoning in the columnist's argument is most vulnerable to criticism on the grounds that this argument", "answers": "['fails to show that a certain conclusion of the recent report is better justified than an opposing conclusion reached in older studies', 'fails to consider that those who exercise vigorously are at increased risk of physical injury caused by exercise', 'fails to consider the possibility that the risk of developing certain cardio-respiratory illnesses can be reduced by means other than exercise', 'fails to consider the possibility that those who engage in vigorous physical exercise are more likely than others to perceive themselves as healthy']", "label": 0 }, { "id": "train_4610", "context": "When a chain of service stations began applying a surcharge of $0. 25 per purchase on fuel paid for by credit card, the chain' s owners found that this policy made their customers angry. So they decided instead to simply raise the price of fuel a compensatory amount and give a $0. 25 discount to customers paying with cash. Customers were much happier with this policy.", "question": "Which one of the following generalizations does the situation described above most clearly illustrate?", "answers": "[\"Peopie's evaluations of a situation sometimes depend less on the situation itself than on how it is presented to them.\", 'People often change their minds about issues that do not make significant differences to their lives.', \"People's perceptions of the fairness of a policy sometimes depend on whether that policy benefits them personally.\", 'People usually adopt beliefs without carefully assessing the evidence for and against those beliefs.']", "label": 0 }, { "id": "train_4611", "context": "An advertising firm creates campaigns for both television and Internet platforms, and both campaigns are comparable in size. The audience for television advertisements is one thousand times the size of the Internet audiences, but the firm generates two-thirds of its revenue from Internet advertisements.", "question": "Which one of the following statements, if true, would resolve the apparent paradox?", "answers": "['The Internet and television advertisements are identical.', 'The firm has considerably more experience with television advertising.', 'The firm pays more for Internet advertising than for television advertising.', 'Internet advertisements allow the firm to more accurately target audiences.']", "label": 3 }, { "id": "train_4612", "context": "Researcher: All defects in short-term memory are caused by a malfunction of a part of the brain called the hippocampus. In short-term memory, the mind holds a piece of information for only a few moments, after which it is either stored in long-term memory or forgotten. Learning is the accumulation of new information in long-term memory. Thus, whenever a child exhibits a learning deficit, the hippocampus is malfunctioning.", "question": "The reasoning in the researcher's argument is most vulnerable to criticism on the grounds that this argument", "answers": "['draws a general conclusion based on too small a sample of learning deficits', 'presumes, without giving justification, that all learning deficits in children involve shortterm memory', 'presumes, without giving justification, that short-term memory is disabled whenever the hippocampus is disabled', 'takes for granted that learning deficits in adults have a cause unrelated to the cause of learning deficits in children']", "label": 1 }, { "id": "train_4613", "context": "Banking analyst: Banks often offer various services to new customers at no charge. But this is not an ideal business practice, since regular, long-term customers, who make up the bulk of the business for most banks, are excluded from these special offers.", "question": "Which one of the following, if true, most strengthens the banking analyst's argument'?", "answers": "['Most banks have similar charges for most services and pay similar interest rates on deposits.', 'Banks do best when offering special privileges only to their most loyal customers.', 'Offering services at no charge to all of its current customers would be prohibitively expensive for a bank.', 'Once they have chosen a bank, people tend to remain loyal to that bank.']", "label": 1 }, { "id": "train_4614", "context": "It has been claimed that an action is morally good only if it benefits another person and was performed with that intention; whereas an action that harms another person is morally bad either if such harm was intended or if reasonable forethought would have shown that the action was likely to cause harm.", "question": "Which one of the following judgments most closely conforms to the principle cited above?", "answers": "['Intending to help her elderly neighbor by clearing his walkway after a snowstorm, Teresa inadvertently left ice on his steps; because of this exposed ice, her neighbor had a bad fall, thus showing that morally good actions can have bad consequences.', \"Jonathan agreed to watch his three-year-old niece while she played but, becoming engrossed in conversation, did not see her run into the street where she was hit by a bicycle; even though he intended no harm, Jonathan's action was morally bad.\", 'Marilees, asked by a homeless man for food, gave the man her own sandwich; however, because the man tried to talk while he was eating the sandwich, it caused him to choke, and thus Marilees unintentionally performed a morally bad action.', \"In order to secure a promotion, Jeffrey devoted his own time to resolving a backlog of medical benefits claims; Jeffrey's action was morally good since it alone enabled Sara's claim to be processed in time for her to receive much-needed treatment.\"]", "label": 1 }, { "id": "train_4615", "context": "A chemical present in all grapes helps reduce blood cholesterol in humans. The chemical is also present in red wine and grape juice, bECACAut not in white wine. Both red wine and grape juice are produced using whole grapes; white wine is produced without using the grape skins.", "question": "The information above, if true, most strongly supports which of the following conclusions?", "answers": "['The skins of red grapes but not of other grapes contain the chemical that reduces blood cholesterol.', 'The chemical that reduces blood cholesterol is found in the skins but not in other parts of grapes.', 'The chemical that reduces blood cholesterol is present only in beverages, rather than solid foods, that are made from grapes.', 'Regular consumption of alcoholic substances such as red or white wine can substantially reduce blood cholesterol.']", "label": 1 }, { "id": "train_4616", "context": "Economist: Some policymakers believe that our country' s continued economic growth requires a higher level of personal savings than we currently have. A recent legislative proposal would allow individuals to set up savings accounts in which interest earned would be exempt from taxes until money is withdrawn from the account. Backers of this proposal claim that its implementation would increase the amount of money available for banks to loan at a relatively small cost to the government in lost tax revenues. Yet, when similar tax-incentive programs were tried in the past, virtually all of the money invested through them was diverted from other personal savings, and the overall level of personal savings was unchanged.", "question": "The passage as a whole provides the most support for which one of the following conclusions?", "answers": "['The proposed tax incentive is unlikely to attract enough additional money into personal savings accounts to make up for the attendant loss in tax revenues.', 'Backers of the tax-incentive proposal undoubtedly have some motive other than their expressed aim of increasing the amount of money available for banks to loan.', 'The economy will be in danger unless some alternative to increased personal savings can be found to stimulate growth.', 'A tax-incentive program that resulted in substantial loss of tax revenues would be likely to generate a large increase in personal savings.']", "label": 0 }, { "id": "train_4617", "context": "Mayville Airport and Newcomb Airport have the same number of flight departures each day. Mayville Airport experiences 26 departure delays per 100 flights, while Newcomb Airport experiences 20 departure delays per 100 flights. When delays caused by bad weather are disregarded, Mayville Airport has 5 fewer departure delays per 100 flights than Newcomb Airport does.", "question": "Which of the following conclusions is best supported by the information given above?", "answers": "['The fleet of airplanes leaving from Newcomb Airport is better equipped to handle inclement weather than the fleet of airplanes leaving from Mayville Airport.', 'Mechanical problems cause a greater number of delays at Newcomb Airport than at Mayville Airport.', 'Newcomb airport employees are better able to manage weather problems than are Mayville airport employees.', 'Bad weather causes a greater number of departure delays at Mayville Airport than at Newcomb Airport.']", "label": 3 }, { "id": "train_4618", "context": "Children in the first six standards of school who are publicly singled out for enthusiastic praise by their teachers show marked improvement in their schoolwork. But students in higher standards tend to react negatively to such praise, and their academic performance tends to decline as a result.", "question": "Which one of the following most helps to explain the differing reactions described above?", "answers": "[\"Older students believe that their teachers' praise will inspire the disapproval of their peers, to which they have a greater aversion than do younger students.\", 'Younger children respond more to the tone of voice with which criticism is offered than to its explicit content.', 'Older students are more likely than younger students to believe that there are better indicators of their academic performance than the grades they earn in class.', 'Younger students are more concerned with public appearance than are older students.']", "label": 0 }, { "id": "train_4619", "context": "A 24-year study of 1, 500 adults showed that those subjects with a high intake of foods rich in betacarotene were much less likely to die from cancer or heart disease than were those with a low intake of such foods. On the other hand, taking beta-carotene supplements for 12 years had no positive or negative effect on the health of subjects in a separate study of 20, 000 adults.", "question": "Each of the following, if true, would help to resolve the apparent discrepancy between the results of the two studies EXCEPT:", "answers": "['The human body processes the beta-carotene present in foods much more efficiently than it does beta-carotene supplements.', 'In the 24-year study, the percentage of the subjects who had a high intake of betacarotene-rich foods who smoked cigarettes was much smaller than the percentage of the subjects with a low intake of beta-carotene-rich foods who smoked.', 'Beta-carotene must be taken for longer than 12 years to have any cancer-preventive effects.', 'In the 12-year study, half of the subjects were given beta-carotene supplements and half were given a placebo.']", "label": 3 }, { "id": "train_4620", "context": "A major health insurance company in Lagolia pays for special procedures prescribed by physicians only if the procedure is first approved as \"medically necessary\" by a company-appointed review panel. The rule is intended to save the company the money it might otherwise spend on medically unnecessary procedures. The company has recently announced that in order to reduce its costs, it will abandon this rule.", "question": "Which of the following, if true, provides the strongest justification for the company's decision?", "answers": "['Patients often register dissatisfaction with physicians who prescribe nothing for their ailments.', 'The review process is expensive and practically always results in approval of the prescribed procedure.', 'Physicians often prescribe special procedures that are helpful but not altogether necessary for the health of the patient.', 'The number of members of the company-appointed review panel who review a given procedure depends on the cost of the procedure.']", "label": 1 }, { "id": "train_4621", "context": "The caffeine in coffee stimulates the production of irritating acid in the stomach. But darker roasts of coffee, produced by roasting the coffee beans longer, contain more N-methylpyridinium (NMP) than lighter roasts, and NMP tends to suppress production of acid in the stomach. Ttherefore if you drink caffeinated coffee, darker roasts will irritate your stomach less than lighter roasts.", "question": "The answer to which one of the following questions most helps in evaluating the argument?", "answers": "['Does a reduction in acid production in the stomach have an adverse effect on stomach function?', 'Does extending the roasting time of coffee beans increase the amount of caffeine present in the brewed coffee?', 'Would coffee drinkers who drink caffeinated coffee increase their coffee consumption if the coffee they drank contained less caffeine?', 'Do lighter roasts of coffee have any important health benefits that darker roasts of coffee lack?']", "label": 1 }, { "id": "train_4622", "context": "Products sold under a brand name used to command premium prices because, in general, they were superior to nonbrand rival products. Technical expertise in product development has become so widespread, however, that special quality advantages are very hard to obtain these days and even harder to maintain. As a consequence, brand-name products generally neither offer higher quality nor sell at higher prices. Paradoxically, brand names are a bigger marketing advantage than ever.", "question": "Which of the following, if true, most helps to resolve the paradox outlined above?", "answers": "['In the days when special quality advantages were easier to obtain than they are now, it was also easier to get new brand names established.', 'Consumers recognize that the quality of products sold under invariant brand names can drift over time.', \"The advertising of a company's brand-name products is at times transferred to a new advertising agency, especially when sales are declining.\", 'Brand names are taken by consumers as a guarantee of getting a product as good as the best rival products.']", "label": 3 }, { "id": "train_4623", "context": "The number of sports cars per square mile is 40% greater in downtown Terrera than in downtown Goldsville. If the entire metropolitan area of each city is compared-which includes downtown as well as suburbs and exurbs-then the number of sports cars per square mile is 20% greater in Goldsville.", "question": "Based on the information above, which of the following can be inferred?", "answers": "['Metropolitan Goldsville covers a larger area than metropolitan Terrera.', 'Per square mile, there are fewer sports cars in the exurbs and suburbs of Terrera than in those of Goldsville.', 'In total, there is a greater number of sports cars in metropolitan Goldsville than in metropolitan Terrera.', 'There is a greater number of sports cars in the suburbs and exurbs of Goldsville than in those of Terrera.']", "label": 1 }, { "id": "train_4624", "context": "The song of the yellow warbler signals to other yellow warblers that a particular area has been appropriated by the singer as its own feeding territory. Although the singing deters other yellow warblers from taking over the feeding territory of the singer, other yellow warblers may range for food within a portion of the singer' s territory. However, a warbler sings a special song when it molts (sheds its feathers). Other yellow warblers will not enter the smaller core territory of a yellow warbler singing its molting song. Ttherefore yellow warblers, which can only fly short distances during molting, have no competition for the food supply within the range of their restricted flying.", "question": "The argument makes which one of the following assumptions?", "answers": "['The core areas contain just enough food to sustain one yellow warbler while it molts.', 'The core areas of each feeding territory are the same size for each molting warbler.', 'Warblers often share their feeding areas with other kinds of birds, which often do not eat the same insects or seeds as warblers do.', 'There are no birds other than yellow warblers that compete with yellow warblers for food.']", "label": 3 }, { "id": "train_4625", "context": "Real Estate Agent: Next year, the occupancy rate for residential rental properties in Riverton will increase despite projections of decreasing population in the city. Generally, when a city's population decreases, so does the occupancy rate for residential rental properties. However, in the continuing climate of increasing interest rates on home mortgages, an unusually large number of Riverton residents who would typically buy homes will opt to rent instead. ", "question": "The boldfaced statements in the argument above play which of the following roles?", "answers": "['The first describes a pattern of cause and effect; the second describes a situation for which this pattern will not hold.', \"The first is a conclusion drawn by the agent; the second is evidence that weighs against the agent's conclusion.\", \"The first is a consideration that weighs against the agent's prediction; the second is that prediction.\", \"The first is the agent's prediction; the second is reasoning offered in support of this prediction.\"]", "label": 3 }, { "id": "train_4626", "context": "Cultural historian: Universal acceptance of scientific theories that regard human beings only as natural objects subject to natural forces outside the individual' s control will inevitably lead to a general decline in morality. After all, if people do not believe that they are responsible for their actions, they will feel unashamed when they act immorally, and a widespread failure of individuals to feel ashamed of their immoral actions is bound to lead to a general moral decline.", "question": "The conclusion drawn by the cultural historian follows logically if which one of the following is assumed?", "answers": "['People who have a sense of shame for their moral transgressions will feel responsible for their actions.', 'Human beings who regard themselves only as natural objects will as a result lose their sense of responsibility for their actions.', 'Some scientific theories hold that human beings are not responsible for their actions.', 'Science does not enable human beings to control natural forces.']", "label": 1 }, { "id": "train_4627", "context": "Pedigreed dogs, including those officially classified as working dogs, must conform to standards set by organizations that issue pedigrees. Those standards generally specify the physical appearance necessary for a dog to be recognized as belonging to a breed but stipulate nothing about other genetic traits, such as those that enable breeds originally developed as working dogs to perform the work for which they were developed. Since dog breeders try to maintain only those traits specified by pedigree organizations, and traits that breeders do not try to maintain risk being lost, certain traits like herding ability risk being lost among pedigreed dogs. Ttherefore, pedigree organizations should set standards requiring working ability in pedigreed dogs classified as working dogs.", "question": "Which one of the following principles, if valid, justifies the argument's conclusion that pedigree organizations should set standards for working ability in dogs?", "answers": "['Any standard currently in effect for a product or an activity should be rigorously enforced regardless of when the standard was first set.', 'Organizations that set standards for products or activities should not set standards calling for a particular characteristic if such standards increase the risk of some other characteristic being lost.', 'Any standard that is set for a product or an activity should reflect the uses to which that product or activity will eventually be put.', 'Organizations that set standards for products or activities should attempt to ensure that those products or activities can serve the purposes for which they were originally developed.']", "label": 3 }, { "id": "train_4628", "context": "If the play were successful, it would be adapted as a movie or revived at the Decade Festival. But it is not successful. We must, regrettably, conclude that it will neither become a movie nor be revived at the Decade Festival.", "question": "The argument's reasoning is flawed because the argument", "answers": "['fails to draw the conclusion that the play will not both be adapted as a movie and be revived at the Decade Festival, rather than that it will do neither', 'fails to explain in exactly what way the play is unsuccessful', \"fails to recognize that the play's not satisfying one sufficient condition does not preclude its satisfying a different sufficient condition for adaptation as a movie or revival at the Decade Festival\", \"equates the play's aesthetic worth with its commercial success\"]", "label": 2 }, { "id": "train_4629", "context": "The only preexisting recordings that are transferred onto compact disc are those that record companies believe will sell well enough on compact disc to be profitable. So, most classic jazz recordings will not be transferred onto compact disc, because few classic jazz recordings are played on the radio.", "question": "The conclusion above follows logically if which one of the following is assumed?", "answers": "['The only recordings that are played on the radio are ones that record companies believe can be profitably sold as compact discs.', 'Few compact discs featuring classic jazz recordings are played on the radio.', 'No recording that is not played on the radio is one that record companies believe would be profitable if transferred to compact disc.', 'Few of the preexisting recordings that record companies believe can be profitably transferred to compact disc are classic jazz recordings.']", "label": 2 }, { "id": "train_4630", "context": "Editorial: This political party has repeatedly expressed the view that increasing spending on education is a worthy goal. On other occasions, however, the same party has claimed that the government should not increase spending on education. So this party' s policy is clearly inconsistent.", "question": "The argument in the editorial depends on assuming which one of the following?", "answers": "['A consistent political policy does not hold that an action that comprises a worthy goal should not be performed.', 'It is inconsistent for a legislator both to claim that increasing spending on education is a worthy goal and to vote against increasing spending on education.', 'Even if a goal is a morally good one, one should not necessarily try to achieve it.', 'A consistent course of action in educational policy is usually the course of action that will reduce spending on education in the long run.']", "label": 0 }, { "id": "train_4631", "context": "The rate at which a road wears depends on various factors, including climate, amount of traffic, and the size and weight of the vehicles using it. The only land transportation to Rittland's seaport is via a divided highway, one side carrying traffic to the seaport and one carrying traffic away from it. The side leading to the seaport has worn faster, even though each side has carried virtually the same amount of traffic, consisting mainly of large trucks.", "question": "Which of the following, if true, most helps to explain the difference in the rate of wear?", "answers": [ "Wear on the side of the highway that leads to Rittland's seaport has encouraged people to take buses to the seaport rather than driving there in their own automobiles.", "A greater tonnage of goods is exported from Rittland's seaport than is imported through it.", "The volume of traffic to and from Rittland's seaport has increased beyond the intended capacity of the highway that serves it.", "All of Rittland's automobiles are imported by ship." ], "label": 1 }, { "id": "train_4632", "context": "Occultist: The issue of whether astrology is a science is easily settled: it is both an art and a science. The scientific components are the complicated mathematics and the astronomical knowledge needed to create an astrological chart. The art is in the synthesis of a multitude of factors and symbols into a coherent statement of their relevance to an individual.", "question": "The reasoning in the occultist's argument is most vulnerable to criticism on the grounds that the argument", "answers": "['denies the possibility that astrology involves components that are neither artistic nor scientific', 'presumes, without providing justification, that any science must involve complicated mathematics', 'incorrectly infers that a practice is a science merely from the fact that the practice has some scientific components', 'presumes, without providing justification, that any art must involve the synthesis of a multitude of factors and symbols']", "label": 2 }, { "id": "train_4633", "context": "A survey of alumni of the class of 1960 at Aurora University yielded puzzling results. When asked to indicate their academic rank, half of the respondents reported that they were in the top quarter of the graduating class in 1960.", "question": "Which one of the following most helps account for the apparent contradiction above?", "answers": "['Academic rank at Aurora University was based on a number of considerations in addition to average grades.', 'Almost all of the alumni who graduated in 1960 responded to the survey.', 'Not all the alumni who were actually in the top quarter responded to the survey.', 'A disproportionately large number of high-ranking alumni responded to the survey.']", "label": 3 }, { "id": "train_4634", "context": "The manufacturers of NoSmoke claim that their product reduces smokers' cravings for cigarettes. However, in a recent study, smokers given the main ingredient in NoSmoke reported no decrease in cravings for cigarettes. Thus, since NoSmoke has only two ingredients, if similar results are found for the second ingredient, we can conclude that NoSmoke does not reduce smokers' cravings.", "question": "The argument above is flawed in that it", "answers": "['illicitly presumes that a claim must be false because the people making the claim are biased', \"overlooks the possibility that NoSmoke helps people to quit smoking in ways other than by reducing smokers' cravings for cigarettes\", 'illicitly presumes that a whole must lack a certain quality if all of its parts lack that quality', 'relies on a sample that is likely to be unrepresentative']", "label": 2 }, { "id": "train_4635", "context": "Editorial: Given the law of supply and demand, maximum total utility is assured only in a pure free market economy, although other types of economies might be able to achieve it. Obviously, then, a country that has a highly controlled economy, and is not trying to bring about a pure free market economy, is not acting in the way that is most likely to bring about maximum total utility.", "question": "The editorial's argument is most vulnerable to criticism on the grounds that it", "answers": "['presumes, without providing justification, that any country that does not have a pure free market economy has a highly controlled economy', 'presumes, without providing justification, that the way in which utility is distributed is less important than the total amount of utility', 'fails to consider that the way most likely to achieve a particular end may not be the only way to achieve that end', 'presumes, without providing justification, that trying to bring about a condition that will ensure the achievement of an end must always be the way most likely to achieve that end']", "label": 3 }, { "id": "train_4636", "context": "Music critic: Fewer and fewer musicians are studying classical music, decreasing the likelihood that those with real aptitude for such music will be performing it. Audiences who hear these performances will not appreciate classical music's greatness and will thus decamp to other genres. So to maintain classical music's current meager popularity, we must encourage more young musicians to enter the field.", "question": "Which of the following, if true, most weakens the music critic's reasoning?", "answers": "['Musicians who choose to study classical music do so because they believe they have an aptitude for the music.', 'It is possible to enjoy the music in a particular genre even when it is performed by musicians who are not ideally suited for that genre.', 'The continued popularity of a given genre of music depends in part on the audiences being able to understand why that genre attained its original popularity.', 'Most people who appreciate classical music come to do so through old recordings rather than live performances.']", "label": 3 }, { "id": "train_4637", "context": "A certain moral system holds that performing good actions is praiseworthy only when one overcomes a powerful temptation in order to perform them. Yet this same moral system also holds that performing good actions out of habit is sometimes praiseworthy.", "question": "Which one of the following, if true, does the most to reconcile the apparent conflict in the moral system described above?", "answers": "['Most people face strong moral temptation from time to time but few people have to endure it regularly.', 'It is far more common for people to perform good actions out of habit than for them to do so against strong temptation.', 'People who perform good actions out of habit have often acquired this habit after years of having resisted temptation.', 'People virtually always perform actions they think are good, regardless of what other people may think.']", "label": 2 } ]